You are on page 1of 801

Graber andWilbur’s

Fa mil y meDiCiNe
exa miNat io N
& Bo a r D r eview
Graber andWilbur’s
Fa mil y meDiCiNe
ex a miNa t io N
& Bo a r D r ev iew
Fo u r t h Ed it io n
Editors

Jas K. W lb , Md
Clinical Associate Professor
Department of Family Medicine
Roy J. and Lucille A. Carver College of Medicine
University of Iowa
Iowa City, Iowa

Ma k A. G abe , Md , MSh CE, FACEP


Clinical Professor
Departments of Family Medicine and Emergency Medicine
Roy J. and Lucille A. Carver College of Medicine
University of Iowa
Iowa City, Iowa

B g E. r ay, Md , MME
Faculty Associate Physician
Roy J. and Lucille A. Carver College of Medicine
University of Iowa
Iowa City, Iowa

New Yor /Chicago/San Francisco/Athens/London/Madrid/Me ico City


Milan/New Delhi/Singapore/Sydney/Toronto
Graber and Wilbur’s Family Medicine Examination & Board Review, Fourth Edition

Copyright © 2017 by McGraw-Hill Education. All rights reserved. Printed in China. Except as permitted under the United States Copyright Act
o 1976, no part o this publication may be reproduced or distributed in any orm or by any means, or stored in a data base or retrieval system,
without the prior written permission o the publisher.

Previous editions copyright © 2013, 2009, 2005 by he McGraw-Hill Companies, Inc.

1 2 3 4 5 6 7 8 9 DSS 21 20 19 18 17 16

ISBN 978-1-259-58533-3
MHID 1-259-58533-6

Notice
Medicine is an ever-changing science. As new research and clinical experience broaden our knowledge, changes in treatment
and drug therapy are required. T e authors and the publisher o this work have checked with sources believed to be reliable
in their e orts to provide in ormation that is complete and generally in accord with the standards accepted at the time o
publication. However, in view o the possibility o human error or changes in medical sciences, neither the authors nor the
publisher nor any other party who has been involved in the preparation or publication o this work warrants that the in ormation
contained herein is in every respect accurate or complete, and they disclaim all responsibility or any errors or omissions or or
the results obtained rom use o the in ormation contained in this work. Readers are encouraged to con rm the in ormation
contained herein with other sources. For example and in particular, readers are advised to check the product in ormation
sheet included in the package o each drug they plan to administer to be certain that the in ormation contained in this work is
accurate and that changes have not been made in the recommended dose or in the contraindications or administration. T is
recommendation is o particular importance in connection with new or in requently used drugs.

his book was set in Minion Pro by Aptara, Inc.


he editors were Amanda Fielding and Kim J. Davis.
he production supervisor was Catherine H. Saggese.
Project management was provided by Amit Kashyap, Aptara, Inc.
RR Donnelley was printer and binder.

his book is printed on acid- ree paper.

Library o Congress Cataloging-in-Publication Data

Names: Wilbur, Jason K., editor. | Graber, Mark A. (Mark Alan), editor. |
Ray, Brigit E., editor.
itle: Graber and Wilbur’s amily medicine examination & board review /
editors, Jason K. Wilbur, Mark A. Graber, Brigit E. Ray.
Other titles: Family practice examination and board review. | Family medicine
examination and board review
Description: Fourth edition. | New York : McGraw-Hill Education Medical,
[2017] | Preceded by Family practice examination and board review /
editors, Jason K. Wilbur, Mark A. Graber. 3rd ed. 2013. | Includes
bibliographical re erences and index.
Identi iers: LCCN 2016011464| ISBN 9781259585333 (pbk. : alk. paper) |
ISBN 1259585336 (pbk. : alk. paper) | ISBN 9781259589775 (ebook) |
ISBN 1259589773 (ebook)
Subjects: | MESH: Family Practice | Examination Questions
Classi ication: LCC R834.5 | NLM WB 18.2 | DDC 610.76–dc23 LC record available at
https://lccn.loc.gov/2016011464

McGraw-Hill books are available at special quantity discounts to use as premiums and sales promotions, or or use in corporate training
programs. o contact a representative, please visit the Contact Us pages at www.mhpro essional.com.
To Russell and Aurelia Weil, who treated me with loving kindness rom
the moment they met me and have accepted me into their amily with
open arms.
—JKW

To Eastern Virginia Medical School which, in a strange warp o


reality, actually admitted me despite my grades. And to the staf o the
Prescriber’s Letter, who have put up with me or all these years.
—MAG

To my late Grandma Trudy and Grandpa Leon Rees who supported


me in my endeavors to become a physician and who made all things
possible by encouraging a well-rounded education.
—BER
Contents
Contributors........................................................................................... ix 15 Obstetrics and Women’s Health ......................................... 409
Preface ...................................................................................................xiii 16 Men’s Health .............................................................................. 454
AFew Words on Studying and Taking the Board Examination .....xv
17 Dermatology ............................................................................. 475
1 Emergency Medicine................................................................... 1 18 Neurology................................................................................... 498
2 Cardiology .....................................................................................40 19 Ophthalmology ........................................................................ 526
3 Pulmonology................................................................................97 20 Otolaryngology ........................................................................ 548
4 Allergy and Immunology ...................................................... 131 21 Care of the Older Patient ....................................................... 569
5 Nephrology ................................................................................ 140 22 Care of the Surgical Patient .................................................. 598
6 Hematology and Oncology.................................................. 173 23 Psychiatry ................................................................................... 626
7 Gastroenterology..................................................................... 200 24 Nutrition and Herbal Medicine ........................................... 660
8 Infectious Diseases .................................................................. 237 25 Substance Use Disorders ...................................................... 672
9 HIV/AIDS ..................................................................................... 258 26 Ethics ............................................................................................ 688
10 Endocrinology .......................................................................... 271 27 End-of-Life Care ........................................................................ 698
11 Rheumatology .......................................................................... 303 28 Evidence-Based Medicine .................................................... 712
12 Orthopedics and Sports Medicine .................................... 334 29 Patient-Centered Care ............................................................ 724
13 Pediatrics .................................................................................... 362 30 Final E amination .................................................................... 736
14 Adolescent Medicine .............................................................. 396
Index .....................................................................................................761

v
Contributors
Vijay Aluri, MD David A. Bedell, MD
Rheumatology Fellow Clinical Associate Pro essor
Division o Immunology Department o Family Medicine
Department o Internal Medicine Roy J. and Lucille A. Carver College o Medicine
University o Iowa Hospital and Clinics University o Iowa
Iowa City, Iowa Iowa City, Iowa
11. Rheumatology 25. Substance Use Disorders

A. Ben Appenheimer, MD Julia Buchkina, MD, MPH


Fellow Physician Upstream Functional Medicine
Department o In ectious Diseases Iowa City, Iowa
Roy J. and Lucille A. Carver College o Medicine 24. Nutrition and Herbal Medicine
University o Iowa
Iowa City, Iowa Chris Buresh, MD, MPH, FAAP, FACEP
9. HIV/AIDS Associate Pro essor
Department o Emergency Medicine
Stacey Appenheimer, MD University o Iowa
Clinical Assistant Pro essor Iowa City, Iowa
Department o Family Medicine 20. Otolaryngology
Roy J. and Lucille A. Carver College o Medicine
University o Iowa Nicholas R. Butler, MD, MBA
Iowa City, Iowa Clinical Assistant Pro essor
5. Nephrology Department o Family Medicine
Roy J. and Lucille A. Carver College
Olivia E. Bailey, MD, FACEP
o Medicine
Clinical Associate Pro essor
University o Iowa
Clerkship Director
Iowa City, Iowa
Department o Emergency Medicine
21. Care o the Older Patient
Roy J. and Lucille A. Carver College o Medicine
University o Iowa
Iowa City, Iowa Rachel R. Butler, MD
1. Emergency Medicine Pulmonary Critical Care Fellow
Department o Internal Medicine
David Baumgartner, MD, MBA Roy J. and Lucille A. Carver College
Resident Physician o Medicine
Department o Emergency Medicine University o Iowa
University o Iowa Healthcare Iowa City, Iowa
Iowa City, Iowa 3. Pulmonology
22. Care o the Surgical Patient
Meghan Connett, MD
Martin O. Bazelak, MD Clinical Assistant Pro essor
Fellow Physician Department o Family Medicine
Department o Internal Medicine and Hospice Roy J. and Lucille A. Carver College
and Palliative Medicine o Medicine
University o Iowa University o Iowa
Iowa City, Iowa Iowa City, Iowa
27. End-o -Li e Care 14. Adolescent Medicine
x
x CONTRIBUTORS

Elizabeth S. Cramer, MD Bharat Kumar, MD


Clinical Assistant Pro essor Rheumatology Fellow
Department o Family Medicine Allergy/Immunology Fellow
Roy J. and Lucille A. Carver College o Medicine Division o Immunology
University o Iowa Department o Internal Medicine
Iowa City, Iowa Roy J. and Lucille A. Carver College o Medicine
15. Obstetrics and Women’s Health University o Iowa
Iowa City, Iowa
Kimberly S. Delcour, DO 11. Rheumatology
Clinical Assistant Pro essor, Department
o Internal Medicine Ye-Jin Lee, MD
Division o Cardiovascular Disease, Fellow Physician
Interventional Cardiology Division o Gastroenterology/Hepatology
University o Iowa Heart & Vascular Clinic Department o Internal Medicine
University o Iowa Hospitals & Clinics Roy J. and Lucille A. Carver College o Medicine
Iowa City, Iowa University o Iowa
2. Cardiology Iowa City, Iowa
7. Gastroenterology
Nicholas Edwards, MD
Resident Physician Britt L. Marcussen, MD
Department o Emergency Medicine Clinical Associate Pro essor
University o Iowa Department o Family Medicine and Sports Medicine
Iowa City, Iowa Roy J. and Lucille A. Carver College o Medicine
12. Orthopedics and Sports Medicine University o Iowa
Iowa City, Iowa
Mark A. Graber, MD, MSHCE, FACEP 12. Orthopedics and Sports Medicine
Clinical Pro essor
Departments o Family Medicine and Emergency Medicine Denise A. Martinez, MD
Roy J. and Lucille A. Carver College o Medicine Clinical Assistant Pro essor
University o Iowa Department o Family Medicine
Iowa City, Iowa Assistant Dean
1. Emergency Medicine; 2. Cardiology; 7. Gastroenterology; O ce o Cultural A airs and Diversity Initiatives
26. Ethics; 28. Evidence-Based Medicine; 30. Final Examination Roy J. and Lucille A. Carver College o Medicine
University o Iowa
Erin R. Howe, MD Iowa City, Iowa
Clinical Assistant Pro essor 29. Patient-Centered Care
Stead Family Department o Pediatrics
Roy J. and Lucille A. Carver College o Medicine Ross E. Mathiasen, MD
University o Iowa Assistant Pro essor
Iowa City, Iowa Department o Emergency Medicine
13. Pediatrics Department o Orthopaedic Surgery
University o Nebraska Medical Center
Monika Jindal, MD Omaha, Nebraska
Chie Resident 12. Orthopedics and Sports Medicine
Departments o Family Medicine and Psychiatry
Roy J. and Lucille A. Carver College o Medicine Megan H. Noe, MD, MPH
University o Iowa Clinical Instructor
Iowa City, Iowa Department o Dermatology
23. Psychiatry University o Pennsylvania
Philadelphia, Pennsylvania
Brian R. Kirschling, OD, FAAO 17. Dermatology
Clinical Assistant Pro essor
Department o Ophthalmology and Visual Sciences Spriha Pavuluri, MD
Roy J. and Lucille A. Carver College o Medicine Neurology Resident
University o Iowa Hospitals and Clinics Department o Neurology
Iowa City VA Medical Center Roy J. and Lucille A. Carver College o Medicine
Iowa City, Iowa University o Iowa
19. Ophthalmology Iowa City, Iowa
18. Neurology
CONTRIBUTORS x

Brigit E. Ray, MD, MME Jack . Stapleton, MD


Faculty Associate Physician Pro essor
Department o Family Medicine Departments o Internal Medicine and Microbiology
Roy J. and Lucille A. Carver College Director, Helen C. Levitt Center or Viral Pathogenesis
o Medicine Roy J. and Lucille A. Carver College o Medicine
University o Iowa University o Iowa
Iowa City, IA Iowa City, Iowa
15. Obstetrics and Women’s Health; 22. Care o the 9. HIV/AIDS
Surgical Patient; 30. Final Examination
Rebecca uetken, MD, PhD
Sandra R. Rosen eld-O’ ool, MD Clinical Associate Pro essor
Clinical Assistant Pro essor Department o Internal Medicine, Division o Immunology
Department o Family Medicine Roy J. and Lucille A. Carver College o Medicine
Roy J. and Lucille A. Carver College o Medicine University o Iowa
University o Iowa Iowa City, Iowa
Iowa City, Iowa 11. Rheumatology
15. Obstetrics and Women’s Health
Jon N. Van Heukelom, MD
Margo Schilling, MD Clinical Associate Pro essor
Clinical Pro essor Department o Emergency Medicine
Department o Internal Medicine Roy J. and Lucille A. Carver College o Medicine
Roy J. and Lucille A. Carver College o Medicine University o Iowa
University o Iowa Iowa City, Iowa
Iowa City, Iowa 12. Orthopedics and Sports Medicine
8. In ectious Diseases
Karolyn A. Wanat, MD
Adam B. Schlichting, MD, MPH Assistant Pro essor
Clinical Assistant Pro essor Associate Program Director
Department o Emergency Medicine Department o Dermatology, Pathology, In ectious Diseases
Department o Internal Medicine University o Iowa
Division o Pulmonary, Critical Care and Iowa City, Iowa
Occupational Medicine 17. Dermatology
Roy J. and Lucille A. Carver College o Medicine
University o Iowa Michelle . Weckmann, MD, MS
Iowa City, Iowa Assistant Pro essor
1. Emergency Medicine Departments o Family Medicine, Psychiatry, and
Palliative Medicine
Victoria J. A. Sharp, MD, MBA Roy J. and Lucille A. Carver College o Medicine
Clinical Pro essor o Urology and Family Medicine University o Iowa
Roy J. and Lucille A. Carver College o Medicine Iowa City, Iowa
University o Iowa 27. End-o -Li e Care
Iowa City, Iowa
16. Men’s Health Natasha B. Wheaton, MD
Clinical Assistant Pro essor
Wendy Shen, MD, PhD Department o Emergency Medicine
Clinical Associate Pro essor Roy J. and Lucille A. Carver College o Medicine
Department o Family Medicine University o Iowa
Roy J. and Lucille A. Carver College o Medicine Iowa City, Iowa
University o Iowa 22. Care o the Surgical Patient
Iowa City, Iowa
4. Allergy and Immunology; 6. Hematology and Oncology Jason K. Wilbur, MD
Clinical Associate Pro essor
Brian X. Shian, MD Department o Family Medicine
Clinical Assistant Pro essor Roy J. and Lucille A. Carver College o Medicine
Department o Family Medicine University o Iowa
Roy J. and Lucille A. Carver College o Medicine Iowa City, Iowa
University o Iowa 4. Allergy and Immunology;
Iowa City, Iowa 5. Nephrology; 6. Hematology and Oncology; 16. Men’s Health;
10. Endocrinology 21. Care o the Older Patient; 30. Final Examination
Preface
Welcome to the ourth edition! We hope that you are as excited o er help ul suggestions. Likewise, the comments posted online
to be here as we are. First, let’s start with a quick tour o the (not usually the place one goes or a rmation) have been mostly
book. You will notice some new things as well as amiliar aspects approving, constructive, and enthusiastic.
rom the previous editions. What has changed? In order to keep T e rst edition was published amid a less-than- riendly
up with the times, we have altered the title slightly to Graber & environment, with declining interest in print media and several
Wilbur’s Family Medicine Examination & Board Review. Okay, well-known board review books already on the market. Because
big deal. But wait; there’s more. In addition, you will see a new our book carried a di erent tone, readers slowly gravitated to it,
and attractive layout. T e whole book is in color, so you do not and its market share grew by word-o -mouth. As o 2015, the
have to ip to the glossy pages in the middle o the book to see third edition was the best-selling board review book in print
the color versions o photos and gures. For those o you who and made Amazon’s top 10 list o general medicine books. o
love your e-readers and tablets, there is now an online version what do we attribute this success? Engaged and supportive read-
o the book complete with explanations or the Final Examina- ers play a huge role!
tion questions. We added 50 more questions to the last chapter, In preparation or work on the ourth edition, we saved all o
“Final Examination.” Each chapter now ends with a list o quick your emails and scoured the Internet or reader comments and
clinical pearls, which amount to several evidence-based one-liners reviews. We read and considered all that we could nd—which
per chapter to help consolidate high-yield in ormation. O course, amounted to several hundred readers’ ideas. So, you, the reader,
we have updated everything, so that you will have the most recent have helped shape this book. Keep those emails coming!
medical in ormation. Last but not least, we have a new editor, Brigit With all o the board review books out there, why should you
Ray, who has brought a resh perspective to the book. choose our text? T ere are two crucial dif erences between this
What has not changed? Our essential style remains the same. book and other board review books on the market. First, we
T e book is divided into 29 chapters based on body system and have written this book not only to help you pass the boards but
elements o patient care, ollowed by the “Final Examination” also to broaden your knowledge o amily medicine. T e major-
(chapter 30). T e thousands o questions in the book are woven ity o questions contain a detailed explanation not only o why
into cases, which we hope you will nd interesting, practical, an answer is right but also why the other answers are wrong.
and relevant. o test your acquisition o knowledge, each case In the rapidly changing world o medical knowledge, we have
ends with the learning objectives. o break the monotony o endeavored to provide you with the most relevant and up-to-
slogging through a study guide, you will nd “Quick Quizzes” date evidence. When the current evidence is controversial and
and “Help ul ips” peppered throughout each chapter. we are not certain what the American Board o Family Medicine
A dozen years ago when we wrote the rst edition, we made (ABFM) will do with it, we acknowledge the uncertainty and try
the decision to use the second person voice in order to engage to help you navigate the current evidence.
the reader better and to give the book a conversational appeal. We have tried to make this book as broad and as compre-
We have tried to keep the book rom being boring. Yes, we are hensive as possible. In addition to its use as a board review
aware that this is a study aid. But why must studying be an exer- book or amily medicine, it can be employed as a general
cise in tedium and endurance? It should be enjoyable, appli- review or primary care physicians, physician assistants, and
cable to real li e and provide a surprise every now and again. nurse practitioners. Students and residents studying or Step
You will nd (sometimes eeble) attempts at humor throughout 3 o the licensing examination should ind the book help ul
the book. We have noticed that an occasional reader does not as well. However, no board review book can possibly cover
appreciate our sense o humor. Well, we’re doctors, not writers the entire scope o amily medicine. Use these questions as a
or Saturday Night Live (but i Lorne Michaels is reading this— guide: what areas are your strengths and what do you need
give Jason a call). to study urther? Each answer o the “Final Examination” is
We have been impressed with the level o engagement our re erenced in the book so you can go back and review any
readers possess. Over the years, we have received scores o emails topic that you might have missed. For the online version, we
rom readers who have thanked us, corrected us, and sometimes realize it may be hard to switch back and orth through the
chided us. No matter the intent o the message, the tone is almost book so we have included the answer explanations in a sec-
universally positive—readers are invested in the book and want to tion or you.
x
xv PREFACE

In this book, the use o eponymous medical terms such as must acknowledge that the book would never get nished with-
Crohn disease and Wilson disease, re ects the current Ameri- out large amounts o co ee; so, he thanks everyone involved
can Medical Association recommendations or these and simi- in the worldwide production o co ee, rom the pickers on the
lar terms where the possessive orm is dropped. In addition, Central American ncas to the local baristas.
there is a general trend toward using ewer eponyms, such as Mark would like to thank Jason and Brigit; this has been a
Wegener, which has been dropped completely. We have made collaborative e ort, which has made it easier or all o us. T anks
note o both new and old terms when we have deemed the old to you or buying this book. T anks also to his amily: Hetty,
term more recognizable. Rachel, and Abe (as always). But not to the dogs, Nietzsche and
We enjoyed writing this book and we hope that you enjoy using Dante. T ey need to learn to stay either in or out o the house.
it. I you have suggestions or complaints (okay, maybe all o our No more o this back and orth. Music that has kept him awake:
jokes aren’t politically correct or even unny), do not hesitate to Faith No More, the Beach Boys (actually amazingly good . . .
write us at jason-wilbur@uiowa.edu or mark-graber@uiowa.edu. check out “T e Sounds o Summer: T e Very Best o the Beach
We take your comments seriously as we endeavor to make studying Boys”), and the Kinks. Finally, thanks to his bicycle or keeping
or the board examination more e ective and more un. him sane . . . although some would argue this point.
We acknowledge and thank all our chapter authors who have Brigit has been using the Family Medicine Board Review text
brought their expertise to bear on this project. We also want to study or years and thanks Jason and Mark or including her
to thank the good people at McGraw-Hill who have edited the as a new editor in this project! T is was an honor or her and
book to keep errors to a minimum and created a handsome and she enjoyed working on their team. Foremost, she would like
readable layout. to thank her ance, Austin, and her amily or being so patient
Jason thanks his loving and supportive amily. Af er some when she has been cranky and sleep deprived juggling every-
initial threats, Deb has granted her patience and understanding thing. It’s not easy juggling a master’s degree program with clini-
to the project, and Jason simply owes her dinner every night . . . cal duties, two busy schedules in the medical pro ession, and
or a year. Jason thanks his boys, Ken and ed, who o er a great long distance relationships! In act, she eels she is getting the
distraction rom work and nd it entertaining that their dad is hang o juggling so much that she may start to study how to
some orm o an author. Finally, as with every edition, Jason actually do it!
AFewWords on Studying and
Taking the Board Examination
T roughout the book, you will nd that we give advice on what Next, know what is on the examination. T e percentage o
we think is likely to be on the examination. T at’s what you’re examination content devoted to various systems is posted on
paying or, right? However, in previous editions, we have not the ABFM website, and we recommend you review it. T e top
advised readers on study habits or test-taking strategies. Here, systems tested are usually cardiovascular, respiratory, and mus-
we o er our humble suggestions on how to prepare or the culoskeletal systems. I you are weak in any o these areas, be
examination and enhance your overall per ormance. We rec- certain to ocus your studying on them.
ommend you read this section prior to diving into the meat o Now, what material should you use when studying? Some o
the book. our readers have been overly kind, suggesting in their reviews
While we acknowledge that some people are simply better test that this book is the only study tool needed or the board exami-
takers than others, there is good evidence to show that anyone can nation. While we would like to believe it, we cannot endorse this
improve his or her scores. In act, examination scores are directly point o view.
proportional to time spent studying or the examination (although o get a avor or the questions on the examination, the best
this association grows weaker or those who have high scores strategy is to go to the source. T e ABFM posts its In- raining
already). T e point is, you don’t have to be a genius who got a 36 on Exams (I E) or the last 3 years on its website (www.theab m.
the AC in order to rock the ABFM Certi cation/Recerti cation org). A login is required, which board-certi ed amily physi-
Examination. But you may need to put in the work. cians should all have. T e I Es are perhaps the best source
Your rst step in studying or the examination—af er pur- or assessing your knowledge—we strongly recommend you
chasing this book, o course—should be to develop a study use them. Although we do not recommend relying on the I Es
plan. Plotting out time and dedicating that time to uninter- as your only study aid (obviously; we’re trying to sell books
rupted study is important. How much time do you have be ore here!), you can use these as a way to measure your progress as
the examination? How many hours per week can you devote to you study. T e critiques are available as well, so you can learn
studying? When are you most productive in your studying— what the ABFM thinks you should know. T e ABFM also has
morning or night? What are the chances o a worldwide ailure extensive in ormation on what you should expect when you sit
o co ee crops? Will a new Star Wars movie open be ore the or the certi ying/recerti ying examination, including a tutorial
examination? T inking through these questions, get a calendar, that simulates the examination. I you are an anxious test-taker,
mark the examination date, and plot out days and times that be certain to check out the tutorial. While the ABFM has sev-
you will devote to studying. I you have taken the examination eral use ul tools, be aware that the Sel -Assessment Modules are
be ore and it didn’t turn out so well, you may need to change not representative o the types o questions you will nd on the
your daily work schedule or 2 to 3 months be ore the exami- certi ying/recerti ying examination.
nation to accommodate studying 10 to 15 hours per week. We Another great source or questions is the American Academy
endorse neither “cramming” or the examination nor “adding o Family Physicians (AAFP) website. I you are a member o
on” studying to an already ull schedule. o get the most out o AAFP, you can access questions or ree. T ey are categorized by
studying, you need to approach it like a daily devotion. body system and can be done in chunks o ten at a time worth
In order to maximize your return on your studying and to 0.25 CME credits. T is question bank o ers another opportu-
ocus on de ciencies, try taking a pre-test. T e best pre-test is nity to test your knowledge and determine where you need to
the ABFM I E (keep reading or more on this). You can use ocus your studying.
your results on the pre-test to see what areas are your weakest. Also, the AAFP markets a comprehensive board review sel -
Studying weak areas is less un but will net higher yield results study course, which will set you back about $1,000 i you are a
than studying areas o relative strength. I your practice is nar- member and more i you are not. Indeed, it covers everything
row in scope (e.g., a hospitalist), you probably already have a you need to know or the examination. But so does this book!
sense o areas o strength and weakness. Make sure you address So, the choice is yours, but we doubt that you will need both our
your weak areas with relatively more time on them. book and the AAFP board examination sel -study package.
xv
xv A FEW WORDS ON STUDYING AND TAkING THE BOARD ExAMINATION

What about texts and primary sources? Well, while we would In the past, the ABFM recommended relying on evidence in
admire your perseverance in slogging through whole texts prepar- place up to 2 years be ore the examination rather than the most
ing or the examination, we do not recommend attempting to read recent medical evidence. Now, the ABFM recommends exam-
cover-to-cover texts like Robert Rakel’s Textbook o Family Medi- inees rely on the most up-to-date evidence available. So, when
cine or re erence material like UpToDate. Don’t get us wrong. We you are looking at a question and thinking, “Well, the answer
like these sources and recommend them to you as re erences as you last year might have been ‘A’ but now the evidence points to ‘B’.”
are studying, but you should not rely on them as your sole study Choose “B.”
material. Likewise, using primary sources, like medical journals, is Success ul test-takers do not use grand strategies to outsmart
impractical as a study oundation but use ul to expand your knowl- the question writers; instead they tend to employ a ew simple
edge when you don’t understand something. rules when answering multiple-choice questions. T ese simple
As ar as board review courses: to each his own. I you are rules that ollow amount to guidelines that cannot be blindly
considering attending a course, the AAFP o ers comprehensive applied to the entire test but are of en true. No secret to many o
courses multiple times per year in locations all across the coun- you, perhaps, but here they are:
try. For-pro t entities provide additional options. I you learn • Go with your rst thought ul choice unless you have a solid
best in a live lecture setting, these courses may be a good option
reason to change it (e.g., you misread the question).
or you, but you need not attend a course to get all that in orma- • Look or catch words in the answers, such as “always” and
tion (c’mon—you’ve got this book!).
“never.” T ese will of en be incorrect.
T ere are some important basic things you need to know • Avoid answers with un amiliar terms (e.g., obscure disease
about the examination. First, the examination is composed o
names or rarely per ormed procedures). T ese are of en
our sections, two o which are general in nature and two o
incorrect.
which are modules that you will select on the day o the exami- • T e most detailed answer is of en the correct answer.
nation. It is best to choose modules with which you are more • I two answers are similar, they are probably both wrong.
amiliar. For example, i you practice primarily in an emer- • Stick with amily medicine principles (e.g., answers with
gency department, you may want to choose Emergent/Urgent
“more history” or “shared decision making” are more likely
Care and Hospital Medicine rather than Maternity Care (unless
to be correct).
you’re looking or the additional challenge). • I you don’t know, guess and move on. Do not waste time
T e examination consists entirely o our-item multiple-
deliberating on a single question.
choice questions. You are not penalized or guessing. An unan-
swered question will always be wrong; whereas, a guessed Finally, we part o ering advice that we know busy doctors
question has a 25% chance o being right. I you have no idea, seldom ollow: get plenty o rest. Seriously! Be prepared or
go ahead and guess. As a corollary to that rule, never exit the the examination day by getting a good night’s sleep. Don’t stop
examination without rst completing all the items. You cannot taking care o your health prior to the examination, and that
return to answer unmarked items. includes rest. Eat a good break ast, bring a snack or your breaks
Read every stem and option care ully. Although we doubt and plan to take yoursel out or a nice lunch (but skip the mar-
that the ABFM writes “trick questions,” they do use catch tini—you’ve got an examination to nish). Just like a mountain
words/phrases, such as “except,” “most likely,” “ rst step,” and climber, wear layers. Some o those test-taking centers are reez-
“least likely.” I you are not attending to the catch phrase, you are ing; some are boiling. Stay positive, take a deep breath and keep
likely to answer the question wrong. moving through it. You will pass this thing! Good luck.
EmergencyMedicine
Adam B. Schlichting, Olivia E. Bailey, and Mark A. Graber
1
Question 1.1.2 Which o the ollowing statements is true
CASE 1.1 about gastric lavage?
You get a call rom a panicked mother because her 4-year-old A) Exce t in extr ordin ry circu st nces it should only be
took some o her theophylline. She thinks it may have been as done in the rst hour er n overdose.
many as 10 pills but is not clear on the actual number. She is B) P tients who h ve h d g stric l v ge h ve higher incidence
about 35 minutes rom the hospital. o ul on ry s ir tion th n tients who h ve not.
C) P tients who undergo g stric l v ge h ve higher incidence
Question 1.1.1 Your advice to her is: o eso h ge l er or tion.
A) Give i ec c to ro ote sto ch e tying nd reduce D) It c n ush ill r g ents beyond the ylorus.
theo hylline bsor tion. E) All o the bove re true.
B) Do not give i ec c nd roceed directly to the hos it l.
C) C ll oison control nd then roceed to the hos it l. Answer 1.1.2 The correct answer is “E.” All o the o tions re
D) Bre the dee ly nd c l down; theo hylline is h r less. true. Gener lly, the e c cy o g stric l v ge is li ited. Out-
co e d t do not su ort the use o g stric l v ge er the rst
Answer 1.1.1 The correct answer is “B.” Do not give i ec c hour. In rticul rly severe overdose or in n overdose th t is
but roceed to the hos it l. “A” is incorrect or cou le o likely to del y g stric e tying (e.g., tricyclic ntide ress nts),
re sons. First, i ec c is not rticul rly e ective ethod o you ight w nt to consider l v ge, but such circu st nces re
e tying g stric contents. More i ort ntly, i the tient unusu l. G stric l v ge incre ses the risk o s ir tion, eso h-
should st rt to seize while vo iting s result o the i ec c, ge l er or tion, nd c n ush ill r g ents beyond the
she could s ir te the vo itus c using n s ir tion neu o- ylorus.
nitis. “C” is incorrect bec use you do not w nt to del y de ni-
tive tre t ent. You c n c ll oison control while the tient is Question 1.1.3 A ter care ul consideration, you decided not
on the w y. to lavage. She is now 55 minutes a ter the ingestion. The next
best step to take in this patient is to:
A) Check blood theo hylline levels nd re er or he odi lysis
HELPFUL TIP: i rkedly elev ted.
Ipecac is ine ective and possibly harm ul. It causes my- B) Ad inister 1 g/kg o ch rco l with sorbitol.
opathy and cardiac problems when used chronically C) Pro hyl ctic lly tre t this tient or seizures using lor z-
(such as in those with anorexia nervosa). The Food and e .
Drug Administration (FDA) has considered making ip- D) Pro hyl ctic lly tre t this tient or seizures using henyt-
ecac available by prescription only, and many pharma- oin.
cies currently do not stock it. E) C ll the Child Li e Activities T er ist to distr ct the child
while you settle in or 4-hour observ tion eriod.

T e patient arrives in your emergency department (ED). Answer 1.1.3 The correct answer is “B.” Giving ch rco l is
She is alert but with a tachycardia o 160 beats per minute likely hel ul only within the rst hour af er ingestion. “A” is
and a stable blood pressure (BP). T e ingestion occurred incorrect bec use the tient’s situ tion could deterior te by
about 50 minutes ago. You contemplate gastrointestinal (GI) the ti e blood levels return. “C” nd “D” re incorrect bec use
decontamination. seizure ro hyl xis is not indic ted in this tient. Although
1
2 FAMILY MEDICINE EXAMINATION & BOARD REVIEW

seizures re jor ni est tion o theo hylline toxicity,


they re ore likely to occur in tients who t ke theo hylline QUICK QUIZ: BIOTERRORISM
chronic lly nd h ve toxic blood levels. Acute ingestions re less
worriso e. Observ tion lone (“E”) is not the best choice here. Oh no. Godzill is tt cking okyo. And this ti e it is with
we ons o ss destruction. Which o the ollowing ro erly
HELPFUL TIP:
describes the isol tion require ents o tient with ul o-
Although requently given, single dose activated char-
n ry nthr x?
coal has limited or no e ect on outcomes. It reduces
A) No isol tion necess ry. T e tient y be in the s e roo
absorption by about 30% i given within 1 hour o in-
with n unin ected tient.
gestion and likely has no bene it a ter 1 hour. It can also
B) Res ir tory isol tion only.
cause vomiting with aspiration. For this reason, it has
C) Res ir tory nd cont ct isol tion.
allen out o avor (I don’t remember the last time I used
D) Neg tive ressure roo (such s with tuberculosis) + cont ct
it in our ED [MG]). We are not sure what the correct an-
isol tion.
swer on the test will be . . .
The correct answer is “A.” Pul on ry nthr x is not tr ns it-
ted erson to erson. Cont ct isol tion is indic ted in those
with cut neous nthr x nd GI nthr x (where di rrhe y
HELPFUL TIP:
be in ectious).
Do NOT give activated charcoal to patients with an
altered mental status or who are otherwise unable to
Godzill is not done yet . . . Which o the ollowing drugs should
protect their airway. To prevent aspiration, do not give
be used s ro hyl xis g inst inh led nthr x should ex osure
charcoal to a patient likely to have a seizure (such as
to erosolized s ores be docu ented?
with tricyclic overdose).
A) A rst-gener tion ce h los orin.
B) ri etho ri /sul ethox zole.
C) Ci ro ox cin.
Question 1.1.4 For which o these overdoses is charcoal D) A third-gener tion ce h los orin.
NOT indicated?
A) Acet ino hen. The correct answer is “C.” Fluoroquinolones re the drugs o
B) As irin. choice when tre ting those ex osed to nthr x. Doxycycline
C) Iron. y lso be used. Ce h los orins nd MP/SMX re not ctive
D) Digoxin. g inst nthr x.
E) O i tes.
Godzill , rustr ted by his iled nthr x tt ck, is now s re d-
Answer 1.1.4 The correct answer is “C.” Ch rco l will not bind ing s ll ox. Which o the ollowing is NO true bout s ll-
iron. Ch rco l will lso not bind C ustics/corrosives, He vy ox?
et ls, Alcohols, R id-onset cy nide, Chlorine (or iodine), A) Isol tion is best done t ho e i ossible.
Other insoluble t blets, Ali h tics (hydroc rbons), or L x tives B) T e tient is in ectious until he or she beco es ebrile.
(mnemonic: CHARCOAL). So e o you y h ve nswered C) All lesions re gener lly in the s e st ge o evolution,
“A.” T eoretic lly, ch rco l could inter ere with the ction o unlike wh t is seen in v ricell .
N- cetylcysteine, the ntidote or cet ino hen ingestion by D) S ll ox i uniz tion c uses n ence h litis in 1:300,000
bsorbing it. However, this is ore o theoretic l concern th n o which 25% o c ses re t l.
n ctu l one. First, the drugs should be used t di erent ti es.
Ch rco l should be given i edi tely, while N- cetylcysteine is The correct answer is “B.” T e tient is in ectious until ll le-
given only er 4-hour levels re v il ble. Second, the doses o sions crust over nd in ectivity h s nothing to do with the res-
N- cetylcysteine reco ended re quite high, nd you c n give ence or bsence o ever. “A” is true. Isol tion is best done t
higher dose i you will be using it with ch rco l. Fin lly, intr - ho e since this will li it s re d (those in the household h ve
venous (IV) N- cetylcysteine is v il ble nd is obviously not likely lre dy been ex osed). “C” is lso true; ll lesions re in
ected by ch rco l. “B,” “D,” nd “E” re ll incorrect. While we si il r st te o evolution. Fin lly, “D” is true nd is the re son
do h ve ntidotes or digoxin nd o i tes (Digibind, n loxone), we do not currently i unize g inst s ll ox—well, th t nd
ch rco l is still indic ted to reduce bsor tion within the rst the ct we er dic ted it (W y to go, hu ns!).
hour.
Objectives: Did you learn to . . .
• Manage a patient with an acute ingestion?
CASE 1.2
• Describe the appropriate use o gastric lavage and charcoal A 22-year-old emale presents to the ED with an overdose.
administration? She has a history o depression, and there were empty bottles
• Identi y situations where charcoal may not be indicated? ound at her bedside. T e bottles had contained clonazepam
CHAPTER 1 • EMERGENCY MEDICINE 3

and nortriptyline. T e patient is unconscious with dimin-


ished breathing and is unable to protect her airway. You notice that the patient begins to have an abnormal trac-
ing on the cardiac monitor, so you order an ECG.
Question 1.2.1 The BEST next step is to:
A) Intub te the tient. Question 1.2.2 Which o the ollowing ndings would you
B) Begin g stric l v ge nd d inister ch rco l. expect to nd in a tricyclic overdose?
C) Ad inister u zenil, benzodi ze ine nt gonist, to w ken A) Nor l QRS co lex.
her nd i rove her res ir tions. B) Second- nd third-degree he rt block.
D) Ad inister bic rbon te. C) Widened QRS co lex.
E) Ad inister li id e ulsion. D) Sinus t chyc rdi .
E) Any o the bove.
Answer 1.2.1 The correct answer is “A.” T is tient should
be intub ted. Re e ber in ny e ergency situ tion th t the Answer 1.2.2 The correct answer is “E.” All o the bove nd-
ABCs ( irw y, bre thing, nd circul tion) re the riority. “B” ings c n be seen with tricyclic overdose. In ct, the ost co -
is incorrect bec use, s noted e rlier, tients who undergo g s- on resenting rhyth is n rrow-co lex sinus t chyc rdi .
tric l v ge h ve higher incidence o ul on ry s ir tion— n As toxicity rogresses, you c n see rolonged PR interv l,
even gre ter concern in the obtunded tient. In ct, irw y widened QRS co lex, nd rolonged Q interv l. A QRS
rotection is MANDA ORY be ore undert king l v ge. “C” is >100 s is redictive o seizures nd QRS >160 s is highly
incorrect. Flu zenil will reverse the benzodi ze ine. However, redictive o ventricul r rrhyth i in tients with tricyclic
we know ro ex erience th t seizures in tients who h ve ntide ress nt overdose. He rt blocks (second nd third degree)
h d u zenil re rticul rly di cult to control. T is would her ld oor outco e nd y be seen l te in the course. Asys-
be rticul rly roble tic in tient with ixed overdose, tole is not ri ry rhyth in tricyclic overdose nd tends to
such s with tricyclic, where seizures re co on. T us, it is re ect the end st ge o nother rrhyth i .
reco ended th t u zenil be used only s revers l gent
er rocedur l sed tion in tients who re not on chronic YIKES!! T e patient becomes unresponsive and you look at the
benzodi ze ines. “E” is incorrect. Li id e ulsion re ers to the monitor. You obtain an ECG which shows the ollowing: (Fig. 1-1).
liquid tty cids given s rt o tot l renter l nutrition nd
theoretic lly c n be used to bind t-soluble drugs in the blood. Question 1.2.3 What is the patient’s rhythm?
C se series su ort consider tion o li id e ulsion or c lciu A) Mono or hic ventricul r t chyc rdi .
ch nnel blocker, bet -blocker, tricyclic ntide ress nt overdose, B) Sinus t chyc rdi with bundle br nch block.
nd other t-soluble drugs but only in c ses o re r ctory c rdi c C) P roxys l su r ventricul r t chyc rdi .
rrest or c rdiov scul r coll se— nd cert inly not be ore the D) ors des de ointes.
irw y h s been secured. E) T ird degree he rt block.

FIGURE 1-1. ECG or patient in question 1.2.3.


4 FAMILY MEDICINE EXAMINATION & BOARD REVIEW

Answer 1.2.3 The correct answer is “D.” T is is tors des de cl ss Ib nti rrhyth ic, which y urther rolong the QRS nd
ointes (in French it liter lly e ns “twisting o the oints,” but worsen the c rdi c toxicity o the tricyclic. “D” is incorrect or
in every l ngu ge it e ns “ hysici n’s stress test”), which is two re sons. First, since os henytoin is et bolized to henyt-
subty e o oly or hic ventricul r t chyc rdi . ors des de oin, the concern bout e c cy lies. Second, os henytoin is
ointes c n be recognized by the v rying litude o the co - rodrug nd requires dequ te circul tion nd ren l nd he tic
lex in so ewh t regul r ttern. “A” is incorrect bec use the unction to be converted into ctive drug. I our tient beco es
co lexes re not ono or hic. “B” is incorrect or two re - hy otensive with oor liver nd ren l er usion, dequ te drug
sons. First, there re no P w ves visible. Second, sinus t chyc r- levels ight not be chieved. Fin lly, both henytoin nd os-
di should not h ve v ried litude. “C” is incorrect bec use, henytoin c n c use hy otension—not wh t you need in this
g in, there re no P w ves nd the co lexes re oly or hic. unst ble tient.
“E” is incorrect bec use there re no P w ves.
You correct the arrhythmia and stop the seizures, and she is
Question 1.2.4 This patient needs treatment post haste. admitted to the intensive care unit.
A ter taking care o the ABCs, what is the ONE BEST drug
or the treatment o this arrhythmia in a patient with a tri HELPFUL TIP:
cyclic overdose? A patient who is entirely asymptomatic 6 hours a ter a
A) Es olol. tricyclic overdose is unlikely to have any serious con-
B) Lidoc ine. sequences rom the ingestion. They can be “medically
C) Sodiu bic rbon te. cleared” at that point or admission to psychiatric unit.
D) Proc in ide. Note that “symptomatic” may just be tachycardia or
E) A iod rone. mild con usion. We mean the entirely asymptomatic
patient.
Answer 1.2.4 The correct answer is “C.” T e tre t ent o
choice or rrhyth i s in tients with tricyclic overdose is
sodiu bic rbon te. R ising the H nd d inistering sodiu Objectives: Did you learn to . . .
see to “ ri e” the sodiu ch nnels in the he rt, reversing • Understand the importance o the A–B–Cs in an unstable
the toxicity o the tricyclic. Proc in ide (“D”) nd quinidine patient?
should not be used bec use they ct in si il r shion to tricy- • Describe the role o umazenil in toxicologic emergencies?
clics nd y worsen the roble . Lidoc ine (“B”) c n be used • Manage a tricyclic overdose?
s c n iod rone (“E”), but they re not the best choices. Bet - • Recognize ECG ndings in a tricyclic overdose?
blockers such s es olol (“A”) c n worsen hy otension nd • Recognize torsades de pointes and its treatment in the
should be voided. context o a tricyclic overdose?

T is is not your patient’s lucky day. She begins to seize a er


the administration o the bicarbonate. QUICK QUIZ: DESIGNER AND CLUB DRUGS

Question 1.2.5 The treatment o choice or this seizing An 18-ye r-old le resents er rty. He is h ving ltern t-
patient is: ing e isodes o co b tive beh vior inters ersed with e isodes o
A) Lor ze . co . He beco es l ost neic during the e isodes o co .
B) Re e t the bolus o sodiu bic rbon te nd st rt bic r- He h s ltern ting br dyc rdi (while in co ) nd t chyc rdi
bon te dri . when w ke. T e tient is lso h ving yoclonic seizures. His
C) Phenytoin (Dil ntin). seru lcohol level is zero, nd his u ils re iotic.
D) Fos henytoin (Cerebryx).
E) None o the bove. T e ost likely drug c using this is:
A) Ecst sy (MDMA).
Answer 1.2.5 The correct answer is “A.” Benzodi ze ines re B) GHB (g -hydroxybutyr te k “liquid ecst sy”).
the tre t ents o choice in tricyclic-induced seizures. While C) Meth het ine.
ost seizures re sel -li ited, it is i ort nt to control sei- D) LSD (lysergic cid diethyl ine k “ cid”).
zures bec use the result nt cidosis c n worsen tricyclic tox- E) O i te overdose.
icity (beyond the ct th t rolonged seizures c n c use CNS
injury). “B” is incorrect. T is tient is lre dy lk linized, nd The correct answer is “B.” T e e isodic co nd br dy-
lthough sodiu bic rbon te is the re erred ther y or tricy- c rdi inters ersed with e isodes o extre e git tion re
clic-induced c rdiov scul r toxicity, sodiu bic rbon te is not l ost thogno onic o GHB overdose. GHB intoxic tion
rticul rly e ective in tricyclic-induced seizures. “C,” henyt- lso c uses in oint u ils. “A” is incorrect bec use MDMA
oin, c n be used, but benzodi ze ines nd henob rbit l should c uses n het ine-like re ction with git tion, hy er-
be d inistered rst i ossible. In ddition to not working well tension, hy erther i , t chyc rdi , etc. “C” is incorrect or
s n ntie ile tic drug in tricyclic overdose, henytoin is lso the s e re son. “D” is incorrect bec use LSD r rely (i ever)
CHAPTER 1 • EMERGENCY MEDICINE 5

TABLE 1-1 TOXIDROMES


Drug Class Examples Signs and Symptoms
Anticholinergic Tricyclics, diphenhydramine, scopolamine, Tachycardia, ushing, dilated pupils, low-grade temperature, and
loco weed (jimson weed), some con usion. Mnemonic: Dry as a bone, red as a beet, mad as a hatter, blind
mushrooms, etc. as a bat

Opiates Morphine, heroin, codeine, oxycodone, etc. Pinpoint pupils, hypotension, hypopnea, coma, hypothermia

Cholinergic Organophosphate or carbamate Lacrimation, salivation, muscle weakness, diarrhea, vomiting, miosis.
pesticides, some mushrooms Mnemonic: SLUDGE BBB (salivation, lacrimation, urination, diarrhea, GI
upset, emesis . . . Bradycardia, bronchorrhea, bronchospasm)

Sympathomimetic Cocaine, ecstasy, methamphetamine Tachycardia, hypertension, elevated temperature, dilated pupils (mydriasis)

Gamma-hydroxybutyrate GHB, liquid ecstasy, etc. Alternating coma with agitation, hypopnea while comatose, bradycardia
(GHB) while comatose, and myoclonus

c uses co . “E” is incorrect bec use tients with o i te Question 1.3.1 Which o the ollowing IS NOT a criterion or
overdoses re gener lly so nolent or co tose without in- clearing the cervical spine clinically?
ters ersed e isodes o git tion, lthough o i te use may lso A) Absence o ll neck in.
c uses iosis (be w re th t not ll n rcotic overdoses re s- B) Nor l ent l st tus including no drugs or lcohol.
soci ted with in oint u ils). GHB is odorless nd h s slight C) Absence o distr cting injury (such s n nkle r cture).
s lty t ste. It h s beco e drug o choice or “d te r e.” T e D) Absence o r lysis or nother “h rd” sign th t could be
toxicity tends to be sel -li ited nd c n be tre ted with intu- c used by neck injury
b tion i needed long with tincture o ti e. T e h l -li e is E) Absence o retrogr de nesi .
only 27 inutes.
Answer 1.3.1 The correct answer is “A.” P tients c n h ve l t-
er l neck in nd still h ve their cervic l s ines cle red clini-
QUICK QUIZ: TOXIDROMES c lly. However, no one will ult you or obt ining r diogr hs
in tients with l ter l uscul r (e.g., tr ezius) neck in.
A tient resents to the hos it l with di henhydr ine P tients with centr l neck in (e.g., over the s inous rocesses)
overdose. DO need i ging (r diogr hs ± C ) to cle r their cervic l
s ine. All o the other criteri re required in order to clinic lly
Which o the ollowing signs nd sy to s re you likely to cle r the cervic l s ine ( ble 1-2). T ese criteri h ve been
nd in this tient? v lid ted in both dult nd edi tric tients.
A) Br dyc rdi , dil ted u ils, ushing, nd incre sed bowel
sounds.
B) Br dyc rdi , in oint u ils, ushing, nd decre sed bowel HELPFUL TIP:
sounds. The most common cause o missed ractures is an in-
C) chyc rdi , dil ted u ils, di horesis, nd incre sed adequate series o radiographs. An adequate series o
bowel sounds. radiographs or the cervical spine includes an AP ilm,
D) chyc rdi , dil ted u ils, ushing, nd decre sed bowel a lateral ilm including the top o T-1, and an odontoid
sounds. ilm. CT should be done i radiographs are negative
E) chyc rdi , in oint u ils, ushing, nd incre sed bowel and there is still clinical suspicion o a racture. Flexion–
sounds. extension views add little and should be avoided.

The correct answer is “D.” T is tient h s n nticholiner-


gic toxidro e. oxidro es re sy to co lexes ssoci-
TABLE 1-2 CLEARING THE CERVICAL
ted with rticul r overdose th t should be i edi tely
SPINE CLINICALLY
recognized by the clinici n. Co on toxidro es re listed
in ble 1-1. No central neck pain on questioning or palpation

No distracting, pain ul injury (e.g., bone racture)


CASE 1.3
No symptoms or signs re erable to the neck (paralysis, stinger-type
A patient presents to your o ce with neck pain a er a motor injury, etc.)
vehicle accident. He was restrained and the airbag deployed.
He notes that he had some lateral neck pain at the scene. He Normal mental status including no drugs or alcohol. This includes any
retrograde amnesia, etc.
continues to have lateral neck pain.
6 FAMILY MEDICINE EXAMINATION & BOARD REVIEW

T e patient’s daughter, aged 4 years, was in the same motor HELPFUL TIP:
vehicle accident and also had her cervical spine cleared by There is currently controversy regarding treatment o
radiograph. However, you get a call rom the ED 48 hours acute spinal cord injuries with IVmethylprednisolone 30
a er the initial accident that she is paralyzed rom just mg/kg bolus (3 g in an adult) ollowed by a 5.4 mg/kg
above the nipple line down (never a good thing—you drip or 24 hours. The e icacy o this therapy in spinal
quickly make a mental note to make sure your malpractice cord injury is limited (i it exists at all), and its e icacy in
insurance premiums are paid up). You review the initial SCIWORA is unknown. There are also secondary compli-
radiographs with the radiologist, which are negative as is cations rom the steroids including hyperglycemia, my-
a C o the cervical spine bones done a er the onset o the opathy, and in ections (e.g., pneumonia). We don’t use it.
paralysis.
Question 1.3.4 The ather is, understandably, irate that his child
Question 1.3.2 The most likely cause o this patient’s paral is now paralyzed. You can tell him that the natural history o
ysis is: SCIWORA syndrome in THIS CHILD is likely to be the ollowing:
A) Missed tr nsection o the thor cic cord. A) Continued r lysis with the necessity o long-ter , er -
B) Conversion re ction ro the sychologic l tr u o the nent d t tion to the injury.
ccident. B) Progression o the injury over the next week to include ur-
C) Sub r chnoid he orrh ge. ther r lysis in n scending shion.
D) SCIWORA syndro e. C) Resolution o r lysis nd sensory sy to s over the next
E) Guill in–B rre syndro e. sever l onths.
D) Resolution o ll sy to s exce t sensory sy to s o the
Answer 1.3.2 The correct answer is “D.” T is likely re resents next sever l onths.
SCIWORA syndro e (s in l cord injury without r diologic E) L rge l wsuit yout on the w y. Do not ss go; do not col-
abnor lity). T is occurs ro stretching o the cord second- lect $200; go directly to l r ctice ttorney.
ry to exion/extension-ty e ove ent in n ccident. P tients
with SCIWORA syndro e y be r lyzed t the ti e o ini- Answer 1.3.4 The correct answer is “C.” Gener lly, tients
ti l resent tion (in the event o cord tr nsection) or y h ve with SCIWORA syndro e reg in their strength nd sensory
del yed resent tion u to 72 hours er the injury. “A” is incor- bilities over ti e. However, this depends on when they pres-
rect bec use cord tr nsection would resent with r lysis ent with symptoms! P tients who resent with r lysis right
i edi tely t the ti e o injury. “B” is incorrect bec use this er the ccident y h ve co lete cord tr nsection nd
child is 4 ye rs old, nd conversion re ction is unlikely in chil- thus will not reg in unction. For this re son, it is i ort nt to
dren. In ddition, conversion re ction IS ALWAYS di gnosis obt in n MRI on ll tients with SCIWORA syndro e ( nd
o exclusion. “C” nd “E” re incorrect bec use this is neither ny tr u -induced r lysis or th t tter).
the resent tion o sub r chnoid he orrh ge (he d che, sti
neck, erh s oc l neurologic sy to s) nor o Guill in– Objectives: Did you learn to . . .
• Clinically “clear” the cervical spine and decide when to order
B rre syndro e ( rogressive nu bness nd we kness ro
cervical spine radiographs?
utoi une yelitis).
• Understand the physiology, natural history, and management
o SCIWORA syndrome?
Question 1.3.3 The next step in the management o this
patient is:
A) Avoid hy otension nd hy oxi to revent second ry insult CASE 1.4
to the cord A patient with an extensive history o alcohol use presents to
B) Fluid restriction nd diuretics to reduce cord ede . the ED a er drinking a bottle o automobile winter gas treat-
C) M nnitol to reduce cord ede . ment (Rothschild Vintage, 1954). He is intoxicated, has a
D) Neurosurgic l intervention to deco ress the cord. headache, and describes a “misty” vision, “like a snowstorm”
E) Lolli o nd gi c rd or “service recovery.” (i you live in southern Florida or Cali ornia, call one o us
in Iowa or a description). He is tachycardic and tachypneic.
Answer 1.3.3 The correct answer is “A.” P tients with cord You start an IV and administer saline. You obtain a blood
injury should be onitored closely to void hy otension nd gas, which shows a mild metabolic acidosis.
hy oxi , both o which will urther d ge the lre dy co ro-
ised s in l cord. Neither diuretics (“B”) nor nnitol (“C”) Question 1.4.1 A metabolic acidosis is consistent with all o
will be use ul in this situ tion. “D” is incorrect bec use the ro- the ollowing ingestions EXCEPT:
cess o SCIWORA involves stretching o the cord ( nd subse- A) Ethylene glycol.
quent dys unction) r ther th n cord co ression such s would B) Meth nol.
be seen with bony injury. “E” ight be the right choice i you C) Eth nol (e.g., vodk , gin).
re t king this test s “ tient ex erience ex ert” inste d o D) Petroleu distill tes (e.g., non- lcohol cont ining g soline
doctor; but doctors should choose “A.” roducts).
CHAPTER 1 • EMERGENCY MEDICINE 7

Answer 1.4.1 The correct answer is “D.” Ethylene glycol, eth- TABLE 1-3 CAUSES OF ACIDOSIS
nol, nd eth nol c n ll c use et bolic cidosis. Hydroc r-
Causes o an Lactic acidosis
bons (e.g., g soline roducts) do not c use et bolic cidosis.
elevated anion gap Diabetic ketoacidosis
T e in ni est tion o hydroc rbon toxicity is second ry to acidosis Ingestions such as ethanol, methanol, etc.
the inh l tion nd s ir tion o the hydroc rbon nd the result-
Uremia
ing neu onitis.
Alcoholic ketoacidosis

T is patient’s electrolytes are as ollows: sodium 135 mEq/L, Causes o a normal GI bicarbonate loss (e.g., chronic diarrhea)
bicarbonate 12 mEq/L, chloride 108 mEq/L, BUN 12 mg/dL, anion gap acidosis Renal tubular acidosis (types I, II, and IV)
Cr. 1.0 mg/dL. Interstitial renal disease
Ureterosigmoid loop
Question 1.4.2 This patient’s anion gap is: Acetazolamide and other ingestions
A) 13. Small bowel drainage
B) 15.
C) 23.
D) Un ble to c lcul te the nion g with the in or tion A) Hy o ne .
rovided. B) O tic disk bnor lities.
C) Abdo in l in nd vo iting.
Answer 1.4.2 The correct answer is “B.” By convention, the D) B s l g ngli he orrh ge.
nion g is c lcul ted without using jor c tion, ot ssiu . E) Meninge l signs, such s nuch l rigidity.
T us, the nion g is c lcul ted s ollows:
Answer 1.4.4 The correct answer is “A.” Hy o ne is not
sodiu − (chloride + bic rbon te). co only seen in eth nol oisoning until the tient is close
to de th. In ct, the reverse is true. chy ne is requent
In this tient, the nion g = 135 − (108 + 12) = 15. nding in eth nol overdose. T is kes sense. T e tient
is trying to co ens te or et bolic cidosis by blowing o
T e nor l nion g is ty ic lly considered to be 12 or less. CO2. O tic disk bnor lities, bdo in l in nd vo iting,
However, since lbu in is the jor un e sured nion in the b s l g ngli he orrh ge, nd eninge l signs re ll seen s
seru , the nion g should be djusted or hy o lbu in- rt o eth nol toxicity. It is thought th t ny o these signs
e i . Every 1 g decre se in lbu in will incre se the nion nd sy to s re second ry to centr l nervous syste (CNS)
g by bout 3. T ere ore, you should subtr ct (3 × [nor l he orrh ge.
lbu in- ctu l lbu in]) to get the “re l” nion g . T e
nor l lbu in is considered to be 4. So, let us s y we c l- You can test or ethanol at your hospital but do not have a
cul te n nion g o 16 but the lbu in is 2. In this c se test or methanol on a stat basis and want to be sure that this
the corrected nion g will be (16 − [3 × (4 − 2)] = 16 − 6), patient is not just saying he has a methanol ingestion in order
or 10. to obtain alcohol (a treatment or methanol ingestion—break
out the single malt scotch!).

HELPFUL TIP: Question 1.4.5 What test is most likely to help you deter
In methanol ingestions, the severity o acid–base distur- mine i the patient has methanol ingestion?
bance is generally a better predictor o outcome than A) Co lete blood cell count (CBC).
serum methanol levels. B) BUN/cre tinine.
C) Liver enzy es.
D) Seru os ol lity.
Question 1.4.3 All o the ollowing are causes o an anion E) A yl se nd li se.
gap acidosis EXCEPT:
A) L ctic cidosis. Answer 1.4.5 The correct answer is “D.” With e sured
B) Di betic keto cidosis. seru os ol lity, you c n c lcul te the os ol r g . o do
C) Ren l tubul r cidosis. so, subtr ct the tot l measured seru os oles ro the
D) Ure i . os oles known to be due to eth nol (e ch 100 g/dL o eth -
E) Ingestions such s eth nol. nol ccounts or roxi tely 22 os oles). I there is n ele-
v ted os ol r g , it is evidence o circul ting, un e sured
Answer 1.4.3 The correct answer is “C.” See ble 1-3 or ore os ole. A nor l os ole g is so ewhere between −10 nd
on c uses o nion g cidosis. 20 Os /kg.
In this c se, or ex le:
Question 1.4.4 Which o the ollowing ndings IS NOT
requently seen in patients with methanol ingestion? Me sured seru os ol lity = 368.
8 FAMILY MEDICINE EXAMINATION & BOARD REVIEW

Blood lcohol = 200 g/dL or bout 44 os oles. A) C lciu chloride.


B) Gluc gon.
C lcul ted os ol lity = 2(N ) + BUN/2.8 + glucose/18 = C) Milrinone.
280 + 6 + 8 = 294. D) High-dose insulin.
E) All o the bove.
So, os ol r g = 368 − (294 + 44) = 30.
The correct answer is “E.” In bet -blocker overdoses, the ol-
T is e ns th t there re 30 un e sured os oles th t could, in lowing ndings y be observed: br dyc rdi , AV block, hy o-
the clinic l context o the c se, re resent eth nol. T us, we know tension, bronchos s , n use , e esis, nd hypoglycemia. T is
th t the tient did not si ly overindulge on eth nol ( eth nol, is very si il r to the resent tion o c lciu ch nnel blocker
such s “g s dry,” will ke one intensely drunk . . . but h s obvi- overdose, but c lciu ch nnel blocker overdose o en l cks
ous downsides). bronchos s nd tients re hyperglycemic s insulin rele se
ro the islet cells is c lciu de endent. When bet -blocker
You decide that there is su cient evidence that this patient overdose h s been identi ed, the usu l tre t ents re e loyed
has ingested methanol to institute treatment. (e.g., IV uids, v so ressors, irw y rotection, ch rco l). I
convention l v so ressors h ve iled, gluc gon in dose o 3
Question 1.4.6 Appropriate treatment(s) or this patient to 5 g IV bolus nd dri t 1 to 5 g/hr y be e ective
include: in tre ting bet -blocker overdose. It is gener lly re erred over
A) Fo e izole (4-MP). tro ine in this situ tion. Milrinone nd other hos hodiester-
B) Acetyls licylic cid. se inhibitors y lso be used but re considered third-line
C) Eth nol. gent. Like-wise, c lciu is considered third-line gent in
D) A nd C. bet -blocker overdose. C lciu chloride y otenti te the c-
E) All o the bove. tion o gluc gon. T ere is lso growing body o liter ture su -
orting use o high dose insulin-euglycemia or bet -blocker,
Answer 1.4.6 The correct answer is “D.” Both Fo e izole c lciu ch nnel blocker, or co bin tion overdoses. T e insulin
(4-MP) nd eth nol re used or eth nol ingestion. T e ide is st rted t 0.5 units/kg/hr nd titr ted to s high s 10 units/
is to slow down the et bolis o the eth nol. T e toxicity kg/hr, titr ted by he odyn ic i rove ent (see Clin Toxicol
o eth nol is c used by or ic cid, which is by- roduct (Phila). 2011;49(4):277–283, or ex le). Does it see coun-
o eth nol et bolis . Eth nol is et bolized by lcohol terintuitive to give gluc gon nd insulin? Both e r to iti-
dehydrogen se, the s e enzy e th t bre ks down eth nol. g te the deleterious e ects o bet -blockers on c rdi c yocyte
T us, eth nol et bolis is co etitively inhibited by eth - et bolis , lthough the ech nis is not co letely under-
nol. T e s e holds true or o e izole, which is co etitive stood. Both c n be given (usu lly in sequence with the gluc gon
inhibitor o lcohol dehydrogen se. Fo e izole nd eth nol rst) to the s e tient or bet -blocker overdose. Remember
c n both be used or ethylene glycol ingestion s well. “B” is to maintain euglycemia i using insulin or beta-blocker or
incorrect. Acetyls licylic cid (ASA), or s irin, h s no role in calcium channel blocker overdose.
eth nol ingestion, nd would likely worsen ny g stritis or
he orrh ging.
QUICK QUIZ: TOXICOLOGY 1

HELPFUL TIP: Which o the ollowing c n be used to incre se the et bolis


Hemodialysis should be available or any patient who o lcohol in n intoxic ted tient?
has ingested methanol. Indications or hemodialysis in- A) IV uids.
clude methanol level >50 mg/dL, severe and resistant B) Ch rco l.
acidosis, and renal ailure. C) Forced diuresis.
D) GABA nt gonists such s u zenil.
Objectives: Did you learn to . . . E) None o the bove.
• Recognize mani estations o alcohol ingestion?
• Identi y causes o metabolic acidosis with elevated and The correct answer is “E.” Drunk tients, no tter how
normal anion gaps? uch they nnoy you, will just h ve to slee it o . T e r te o
• Use the osmolar gap to narrow down the dif erential diagno- lcohol et bolis is xed with zero-order kinetics t lower
sis o metabolic acidosis? doses ( xed et bolic r te) nd rst-order kinetics t higher
doses (r te ro ortion l to levels). In gener l, this r te is in
the r nge o 9 to 36 g/dL/hr with 20 g/dL/hr being the
QUICK QUIZ: BETA-BLOCKER OVERDOSE cce ted nor . At this oint, there re no v il ble gents to
incre se the et bolis o eth nol. “B” is incorrect bec use
Which o the ollowing h s been shown to be use ul in bet -blocker eth nol is too r idly bsorbed or ch rco l to be o ny ben-
overdose when convention l, drenergic ressors re ine ective? e t.
CHAPTER 1 • EMERGENCY MEDICINE 9

c rboxyhe oglobin level in these tients. I the tre t ent deci-


QUICK QUIZ: TOXICOLOGY 2 sion is de b sed on the c rboxyhe oglobin level, the level
rojected to ti e zero gives the ost ccur te in or tion bout
T e best ther y or seizures second ry to isoni zid ingestion is: the degree o ex osure. T e rest o the nswers re incorrect.
A) Lor ze .
B) Phenytoin. T e ather has a headache and a time zero carboxyhemoglo-
C) Pyridoxine. bin level o 12%. T e mother, who is pregnant, is asymptom-
D) T i ine. atic and has a time zero carboxyhemoglobin level o 18%.
E) Phenob rbit l. Both o the children are asymptomatic. T e 6-year-old has
a time zero carboxyhemoglobin o level o 18% while the
The correct answer is “C.” Isoni zid is vit in B6 nt gonist. 8-year-old has a level o 23%.
T us, yridoxine (in ssive doses!) is the drug o choice in
isoni zid-induced seizures. T ese seizures re o en resist nt to Question 1.5.3 The rst step in the treatment o these
convention l ther y. Look or this ty e o overdose in tients patients is:
who re being tre ted or tuberculosis (either ctive or l tent A) St rt n IV nd d inister s line.
dise se). B) St rt N- cetylcysteine, which is ree r dic l sc venger.
C) St rt continuous ositive irw y ressure (CPAP) to xi-
CASE 1.5 ize ir ow by kee ing the irw ys ro coll sing.
D) Ad inister 100% oxygen.
A amily o our comes into your ED a er being exposed to E) Intub te the ost severe tient, 100% oxygen or the others.
carbon monoxide (CO). T ey were in an idling car and were
running the engine and heater to stay warm. You want to get Answer 1.5.3 The correct answer is “D.” Bec use CO co eti-
a carboxyhemoglobin level on the whole amily but cannot tively binds to he oglobin in l ce o oxygen nd in ct h s
get an arterial blood gas rom the youngest child. gre ter nity or he oglobin th n oxygen, high- ow 100%
oxygen is the cornerstone o tre ting CO oisoning. T e h l -
Question 1.5.1 What is your response? li e o CO in tient bre thing roo ir is roxi tely 300
A) Check ulse oxi etry, nd i the oxygen s tur tion is nor l, inutes; this is reduced to 90 inutes when bre thing high ow
be re ssured. oxygen nd reduced to 30 inutes when bre thing 100% hy er-
B) Check end-tid l c rbon dioxide. b ric oxygen. T us, the rst step in CO poisoning is to administer
C) Check venous c rboxyhe oglobin level. 100% oxygen. T e rest o the nswers re incorrect. I the tient
D) Check venous c rboxyhe oglobin nd correct or the di - is not ventil ting well nd requires intub tion, this would be
erence between venous nd rteri l s les. ro ri te, nd the FiO2 should be set to 100%, reg rdless o
the tient’s ulse oxi etry or rteri l oxygen re dings. How-
Answer 1.5.1 The correct answer is “C.” A venous c rboxyhe- ever, in our tients who re bre thing without di culty, there
oglobin is just s ccur te s n rteri l c rboxyhe oglobin— will be no dv nt ge ( nd uch higher risk) to intub tion.
in ct, no correction is needed, which is why “D” is wrong— nd
it is uch less in ul to dr w. “A” is incorrect bec use the ulse Question 1.5.4 Which o the ollowing can be seen with
oxi eter does not re ect hy oxi in c rbon onoxide oison- carbon monoxide poisoning?
ing. T us, st nd rd ulse oxi etry is useless in deter ining A) Rh bdo yolysis.
the c rboxyhe oglobin level. “B” is incorrect bec use end-tid l B) C rdi c ische i .
c rbon dioxide is e suring CO2 nd not CO. C) Long-ter neurologic sequel e, including de enti .
D) Pul on ry ede .
Question 1.5.2 When determining which patients need E) All o the bove.
hyperbaric oxygen on the basis o a carboxyhemoglobin
level, the level to rely upon is: Answer 1.5.4 The correct answer is “E.” All o the bove c n
A) T e c rboxyhe oglobin level on rriv l to the ED. be seen with c rbon onoxide oisoning. Addition l ndings
B) T e c rboxyhe oglobin level t 4 hours er ex osure. include l ctic cidosis, seizures, synco e, nd he d che. “C”
C) T e c rboxyhe oglobin level rojected to “ti e zero” (e.g., deserves bit ore discussion. Long-ter neurologic sequel e
t the ti e o ex osure). c n develo ro d ys to onths er the ex osure nd include
D) None o the bove. cognitive de cits, oc l neurologic de cits, ove ent disor-
ders, nd erson lity ch nges. It e rs th t using hy erb ric
Answer 1.5.2 The correct answer is “C.” A jor consider- oxygen in the ro ri te tient c n reduce long-ter neuro-
tion reg rding the initi tion o hy erb ric oxygen ther y is logic sequel e.
the tient’s clinic l situ tion. More severely ill tients with
CO oisoning (e.g., severe cidosis, unconscious, unres onsive) Your closest diving chamber is about 90 minutes away and
should be considered c ndid tes or hy erb ric oxygen, nd will hold only one patient at a time. You need to make a deci-
so e hy erb ric oxygen centers will tre t reg rdless o e sured sion about who to send or hyperbaric oxygen.
10 FAMILY MEDICINE EXAMINATION & BOARD REVIEW

Question 1.5.5 Which patient will bene t most rom hyper C) A co on org nis in in ected dog bites is Staphylococcus
baric oxygen therapy? aureus.
A) Asy to tic regn nt other, ti e zero c rboxyhe o- D) Pri ry closure o dog bite wounds is n cce t ble o tion
globin o 18%. (exce t erh s on the h nds nd eet).
B) Asy to tic 6-ye r-old, ti e zero c rboxyhe oglobin o E) T ey lw ys require ntibiotics.
18%.
C) Asy to tic 8-ye r-old, ti e zero c rboxyhe oglobin o Answer 1.6.1 The correct answer is “E.” All o the rest re
23%. true st te ents. Dog bites (exce t, erh s, or those ro
D) Adult le with ild he d che only, ti e zero c rboxyhe- te cu oodles n ed Fi ) tend to be crush injuries ( s con-
oglobin level o 12%. tr sted with c t bites, which re ri rily uncture wounds).
T e in ection r te is bout the s e s other l cer tions. Bites
Answer 1.5.5 The correct answer is “A.” Gener lly cce ted on the h nds nd eet tend to h ve higher r te o in ection.
criteri or hy erb ric oxygen include: ent l st tus ch nges, Most dog bite in ections re oly icrobi l with ixed erobic
sy to tic c rboxyhe oglobin levels >25%, cidosis, c rdio- nd n erobic b cteri . S. aureus is o en resent, long with
v scul r dise se, nd ge >60. Obviously, these re rel tive criteri . other org nis s including Pasteurella nd Capnocytophaga (s y
An otherwise nor l 61-ye r-old with ild ex osure need not th t one ten ti es st). Other org nis s include Streptococcal
h ve HBO. Pregnancy is an indication or HBO therapy bec use s ecies nd Gr -neg tive s ecies. Dog bites do not gener lly
et l he oglobin h s high nity or c rbon onoxide, with require ntibiotic ro hyl xis, exce t under cert in circu -
the etus cting s “sink” or CO. T e high erobic et bolic st nces (e.g., resent >9 hours er bite, i unoco ro ised,
ctivity o et l develo ent is i cted gre tly by ex osure to l rge, or co lic ted wound).
the n erobic environ ent cre ted by c rbon onoxide.
Question 1.6.2 You are concerned about rabies prophylaxis.
Question 1.5.6 All o the ollowing are well established Which o the ollowing is the best next step?
consequences o hyperbaric oxygen EXCEPT? A) Isol te the sus ect ni l or 3 d ys.
A) Seizures. B) S cri ce the sus ect ni l nd ex ine the liver.
B) Psychosis. C) Ad inister r bies i une globulin IM.
C) Myo i . D) Ad inister r bies i une globulin IV ollowed by r bies
D) E r nd ul on ry b rotr u s. v ccin tion series.
E) Direct ul on ry oxygen toxicity. E) Ad inister r bies i une globulin by in ltr ting it round
the wound ollowed by r bies v ccin tion series.
Answer 1.5.6 The correct answer is “B.” All o the rest re
ound s result o hy erb ric oxygen. “C,” yo i , is ctu lly Answer 1.6.2 The correct answer is “E.” You should in ltr te
ound in u to 20% o tients being tre ted with hy erb ric r bies i une globulin round the wound nd then begin the
oxygen. It is due to direct toxicity o oxygen on the lens nd usu- r bies v ccin tion series. In ltr te s uch o the i une glob-
lly recovers within weeks to onths. ulin s ossible round the wound nd d inister the re inder
Objectives: Did you learn to . . . IM t di erent site. Do not give more than the recommended
• Diagnose and manage patients with carbon monoxide dose o immunoglobulin. T is can reduce the immunogenic-
poisoning? ity o the vaccine. “A” is incorrect bec use ni ls need to be
• Describe complications o carbon monoxide poisoning? isol ted or 10 days, not 3. “B” is incorrect. I c tured, the ni-
• Identi y patients who may bene t rom hyperbaric oxygen l c n be s cri ced but the br in should be ex ined—not
therapy? the liver. “C” nd “D” re both incorrect ethods o d inis-
• Describe the complications o hyperbaric oxygen therapy? tering the v ccine nd i une globulin. Note th t the ver ge
incub tion eriod o r bies is 85 d ys. So, i unizing u to
3 onths (90 d ys) er the bite-event is indic ted.
CASE 1.6
A 50-year-old man comes to your ED a er being bitten by a Question 1.6.3 Which o the ollowing requires rabies pro
stray dog outside your hospital. Apparently, there is a prob- phylaxis in all cases?
lem with roving packs o eral dogs in your part o town. T e A) Str y r bbit bites.
bite was unprovoked and is on the abdomen. T e patient has B) Str y r t bites.
no other health history o note and has not taken antibiotics C) Str y b t bites.
or over a year. T ere is a 3-cm laceration on the abdomen. D) Str y squirrel bites.
E) Str y sn ke bites.
Question 1.6.1 All o the ollowing are true about dog bites
EXCEPT: Answer 1.6.3 The correct answer is “C.” All b ts should be
A) T ey tend to be ri rily crush-ty e injuries. considered r bid unless v il ble or observ tion nd testing.
B) In gener l, the in ection r te is si il r to l cer tion ro ny See ble 1-4 or det iled reco end tions. Also, see the CDC
other ech nis (e.g., kni e cut), exce t on the h nds nd eet. website or in or tion bout r tes o in ection in wild ni ls
CHAPTER 1 • EMERGENCY MEDICINE 11

TABLE 1-4 GUIDELINES FOR RABIES PROPHYLAXIS solutions, etc.) c n be used but re ore ex ensive nd do not
o er ny bene t in reduction in in ection r tes. “D” nd “E” re
General Rule Animals
both incorrect. As with ovidone-iodine, lcohol y be used
Always assume rabid unless Foxes, bats, raccoons, skunks, dogs, or cle ning skin but should be ke t out o the wound. It is toxic
available or testing cats, errets, other carnivores to tissue nd cts x tive. Hydrogen eroxide is lso toxic to
Judge on an individual basis Rodents (rats, mice, etc.), tissue nd should not be used in o en wounds—no tter wh t
lagomorphs (rabbits, etc.), squirrels gr nd s ys!
Never require rabies Nonmammals (snakes, lizards, etc.)
Question 1.6.5 How long a ter a laceration occurs can the
prophylaxis
wound be closed primarily?
A) 6 hours.
B) 12 hours.
in your re . O note, nd true, there re r bid squirrels in Iow C) 18 hours.
City, our illustrious ho e: Google it. D) 24 hours.
E) Any o the bove c n be correct de ending on the wound.
HELPFUL TIP:
Patients should receive a tetanus booster every Answer 1.6.5 The correct answer is “E.” T ere is no rbitr ry
10 years. For a contaminated wound, the tetanus boost- ti e li it to when wound c n be closed. F ci l wounds y
er should be within the last 5 years. Patients should re- be closed u to 24 hours er injury or cos etic re sons, while
ceive at least one dose o Tdap (tetanus, diphtheria, and you y not w nt to close other, cont in ted wounds ore
acellular pertussis) between ages 11 and 18 and a single th n 12 hours er injury. So e wounds you y not w nt
dose between ages 18 and 64. In addition, health-care to close t ll (e.g., bites to the h nd, wounds cont in ted
workers and those > 65 years o age who will be around with gre se, wounds cont in ted with nure, hu n bite
in ants should receive a single dose o Tdap. wounds), r ther llowing the to close by second ry intention.
Objectives: Did you learn to . . .
• Describe the indications or rabies prophylaxis?
HELPFUL TIP:
• Recognize the issues that arise with animal bites and indica-
I a patient at risk or tetanus has not had a primary se-
tions or closure and/or prophylactic antibiotics?
ries o tetanus immunizations, administer tetanus im-
• List recommendations or tetanus prophylaxis?
mune globulin, and start the primary tetanus series.
• Use various wound irrigation solutions or cleansing
wounds?
• Decide upon the time rame or wound closure?
You decide to irrigate this patient’s wound.

Question 1.6.4 Which o the ollowing statements is true


CASE 1.7
about irrigating a wound and subsequent risk o wound A 52-year-old male presents to your ED via ambulance
in ections? complaining o a headache a er a all. He was working and
A) Povidone-iodine s 50% irrig tion solution (e.g., Bet dine) ell approximately 10 . He notes no injury except or head
in the wound will decre se the in ection r te. and neck pain. A quick survey reveals that he has a BP o
B) Irrig tion with nor l s line is the only reco ended 128/86 mm Hg, pulse 100 bpm, and respirations o 12. T ere
ethod o cle ning wound. was no loss o consciousness at the scene. He “saw stars” and
C) Irrig tion with nor l s line nd irrig tion with t w ter was clumsy, dazed, and slow at the scene without any ocal
re equ lly e ective in reducing wound in ection r tes. neurologic de cit. He is now back to his baseline.
D) Use o e ine hrine with lidoc ine in wound incre ses the
r te o in ection. Question 1.7.1 A concussion is de ned as:
E) Irrig tion o wound with either lcohol or hydrogen er- A) Any neurologic sy to s (e.g., clu sy, d zed, or slow) er
oxide will reduce the r te o wound in ection. he d injury.
B) Loss o consciousness ollowed by return to b seline.
Answer 1.6.4 The correct answer is “C.” In ection r tes (in the C) Loss o consciousness with continued neurologic sy to s.
United St tes) re the s e whether the wound is irrig ted with D) Con usion er he d tr u reg rdless o whether the
nor l s line or t w ter. “A” is incorrect. Povidone-iodine is tient lost consciousness or not.
toxic to tissue nd oly or honucle r leukocytes nd ctu lly E) Any tr u tic injury to the he d.
y increase in ection r tes unless solution o 1% or less is
used. Full strength ovidone-iodine can be used on int ct skin Answer 1.7.1 The correct answer is “A.” A concussion is
s cle nser but should not be used in wound. “B” is incor- de ned s any neurologic sy to er he d tr u . Note
rect bec use other solutions ( olox er 188, b l nced s lt th t concussion does not require loss o consciousness. For
12 FAMILY MEDICINE EXAMINATION & BOARD REVIEW

this re son, “B” nd “C” re incorrect. “D” is incorrect bec use D) T ere is u to 4% ch nce this tient will need neurosurgi-
ni est tions o concussion re not li ited to con usion but c l intervention.
lso include rotr cted vo iting, tr nsient nesi , slowed E) None o the bove.
ent tion, “dizziness,” nd other neurologic sy to s. “E” is
incorrect bec use by de nition, concussion requires neuro- Answer 1.7.3 The correct answer is “D.” In ro ri tely
logic sy to s. selected tients (e.g., those with signi c nt ech nis o
injury), bout 18% with GCS o 15 will h ve so e intr cr -
Your patient opens his eyes spontaneously, ollows com- ni l lesion, nd u to 4% will eventu lly require neurosurgic l
mands, answers all orientation questions correctly, but intervention. T ese re gener lly tients who h ve de ressed
appears unsteady when ambulating. skull r cture but nor l GCS. “A” is incorrect since nor l
GCS in nd o itsel does not llow one to orgo he d C in
Question 1.7.2 His Glasgow Coma Scale (GCS) is: tients with signi c nt ech nis o injury. “B” nd “C” re
A) 5. lso incorrect or the re sons noted e rlier. Re e ber th t the
B) 10. GCS is not line r; GCS o 14 is b d. P tients with GCS o 14
C) 14. must h ve C sc n—unless nother ctor in the clinic l deci-
D) 15. sion king dict tes otherwise (e.g., the nding is re-existing
E) 20. ro de enti or otherwise).

Answer 1.7.2 The correct answer is “D.” T e GCS is sc le Question 1.7.4 In an adult patient with a signi cant head
used to indic te the severity o neurologic dys unction nd is injury, which o the ollowing is NOT an indication or a
o en lied to victi s o he d tr u . Re e ber, however, head CT scan?
th t it does not redict ort lity or orbidity but is only used A) Intoxic tion with drugs or lcohol.
s descri tive sc le o the tient’s current st te. Only the x- B) Persistent vo iting.
i u score o 15 is considered nor l GCS. T ere re three C) A nesi or e ory de cit.
co onents to the GCS, listed in ble 1-5. D) Age gre ter th n 40.
E) Seizure.
HELPFUL TIP:
Your chair and re rigerator each have a GCS o 3. Remember Answer 1.7.4 The correct answer is “D.” Older tients re t
that nothing can have a GCS less than 3. gre ter risk o develo ing serious intr cr ni l injuries, nd the
ge o 60 is usu lly considered n inde endent indic tion or
he d C with signi cant injury. While there is no “u er li it
Question 1.7.3 In patients with head injury, and indepen o nor l” or vo iting er he d tr u , the best d t v il-
dent o other actors, a GCS score o 15 indicates that: ble suggest th t ny vo iting er he d tr u in n adult
A) T e tient does not require he d C sc n. indic tes the need or he d C . T e currently reco ended
B) T ere is essenti lly no ossibility th t this tient h s n criteri or C o the he d in v rious ge grou s re listed in
intr cr ni l injury requiring surgic l intervention. ble 1-6.
C) T ere is little or no ossibility th t this tient h s ny oc l
intr cr ni l bleed. You obtain the head C and nd a subdural hematoma. You
arrange to trans er this patient or neurosurgical interven-
TABLE 1-5 GLASGOW COMA SCALE tion in order to drain the subdural hematoma. It is about a
Eye opening Spontaneous = 4
4-hour drive by ambulance to the nearest acility that has a
Mnemonic: “4 eyes” To speech = 3
neurosurgeon.
To pain = 2
No response = 1
Question 1.7.5 Which o the ollowing is indicated as pro
phylaxis against increased intracranial pressure in this
Verbal response Alert and oriented = 5 patient?
Mnemonic: “Jackson 5” Disoriented conversation = 4 A) Hy erventil tion er intub tion.
Nonsensical speech = 3 B) IV nnitol.
Moaning = 2 C) rendelenburg osition.
No response = 1 D) IV dex eth sone.
Motor response Follows commands = 6
E) None o the bove.
Mnemonic: “Six Cylinders” Localizes pain = 5
Answer 1.7.5 The correct answer is “E.” None o the bove is
Withdraws rom pain = 4
indic ted s ro hyl xis or incre sed intr cr ni l ressure. “A”
Decorticate exion = 3
is incorrect or two re sons. First, this tient does not need to
Decerebrate extension = 2
be intub ted. Second, routine hy erventil tion s ro hyl xis
No response = 1
or incre sed intr cr ni l ressure is o no bene t. T is h s
CHAPTER 1 • EMERGENCY MEDICINE 13

TABLE 1-6 INDICATIONS FOR HEAD CT BY AGE HELPFUL TIP:


Indications or Head CT A ter Trauma About two-thirds o patients with a mild head injury (not
Patient Age with a Signi cant Mechanism deemed severe enough to obtain a CT scan) will have
Adult • Intoxication some measurable decrement in unction at 1 month
• Age >60 secondary to post-concussion syndrome (J Emerg Med.
• Any memory de cit 2011;40:262).
• Vomiting (number o times unde ned)
• Seizure
• Headache

PECARN Rules for Pediatric Head Trauma and Need for CT Scan
Objectives: Did you learn to . . .
(The La ncet. 2009;374(9696):1160–1170.) • Use the GCS?
CT needed if: • Recognize which patients with head trauma are appropriate
Children >2 years • GCS <15 to obtain a head CT?
• Signs o a basilar skull racture • Manage patients presenting with potential intracranial injuries?
• Agitation, somnolence, slow responses, or
perseveration
• Vomiting
• Loss o consciousness CASE 1.8
• Severe headache
• Severe mechanism (Fall >5 ft, MVC with A 23-year-old male is in a bar ght. He only had “two beers”
ejection, roll over, atality) and was just standing there “minding my own business” when
• Bike/pedestrian vs. car without helmet
he was jumped by those in amous “two dudes” (how can
• High Impact Object
• I there are changes during observation those two dudes be in so many places at once!). He presents
(MS change, worsening headache, new or to you about 1 hour a er the event with acial trauma. His
persistent vomiting) consider CT based on vitals are normal and he is mentating well (with the exception
clinical judgment. o some impaired judgment secondary to the alcohol). His
Children <2 years • GCS <15 blood alcohol level is 150 mg/dL, showing that he is legally
• Palpable skull racture intoxicated. On examination, you notice that the patient has
• Agitation, somnolence, slow responses, some epistaxis and a quite swollen nose. In addition, there is
perseveration one avulsed tooth and one tooth that is displaced.
• Scalp hematoma (except or rontal)
• LOC >5 seconds
• Severe mechanism ( all >3 ft, MVC with Question 1.8.1 The best way to transport an avulsed tooth
ejection, roll over, atality) is:
• Bike/pedestrian vs. car without helmet A) In sterile w ter.
• High impact object B) In the bucc l ucos er thorough w shing with so .
• I there are changes during observation
C) In gl ss o ilk.
(MS change, worsening headache, new or
persistent vomiting) consider CT based on D) Wr ed in s line-so ked g uze.
clinical judgment. I child <3 months o age E) Under illow.
consider parenteral pre erence and consider
scanning Answer 1.8.1 The correct answer is “C.” T e best w y to tr ns-
ort n vulsed tooth is (1) in gl ss o ilk, (2) in H nks’ b l-
nced s lt solution (good luck nding this when you need it!),
been well studied. Wh t h ens is th t hy erventil tion does or (3) in the bucc l ucos or under the tongue in tient in
c use v soconstriction reducing intr cr ni l blood ow nd who the risk o s ir tion is not concern. “A” is incorrect
there ore intr cr ni l ressure. However, hy erventil tion lso bec use sterile w ter is hy otonic nd yd ge the tooth
c uses ische i round the re o the injury nd y worsen root decre sing the success r te o re-i l nt tion. “B” is incor-
outco es. “B” is incorrect bec use ro hyl ctic nnitol, like rect bec use washing the tooth with soap is not ro ri te.
ro hyl ctic hy erventil tion, con ers no bene t. “C” is incor- Ag in, you w nt to int in the vi bility o the root i ossible.
rect. rendelenburg ositioning, or elev ting the legs bove the “D” is incorrect s well. I this is the only o tion v il ble to you,
he rt, would result in incre sed intr cr ni l ressure. T ere is it is better th n nothing, but gl ss o ilk or under the bucc l
very li ited d t to su ort or re ute ro hyl ctic elev tion o ucos is re erred. “E” is cce t ble only i you re tooth iry.
the he d o the bed to prevent incre sed intr cr ni l ressure;
while this will reduce intr cr ni l ressure, cerebr l er usion You call the dentist who is (o course) out o town. A dentist
ressure will lso be ildly reduced. For treatment o incre sed will not be available or at least 12 hours.
intr cr ni l ressure, there is slightly ore evidence or bene t
o elev ting the he d o the bed “D” is incorrect since steroids Question 1.8.2 Your best course o action at this point is:
re not use ul cutely in he d tr u . However, steroids re A) Continue to kee the tooth vi ble in gl ss o ilk.
use ul in cerebr l ede second ry to tu or. B) Continue to kee the tooth vi ble in the bucc l ucos .
14 FAMILY MEDICINE EXAMINATION & BOARD REVIEW

C) Cle n the tooth nd kee it sterile nd dry or re-i l nt tion Answer 1.8.4 The correct answer is “B.” A se t l he to
in 12 hours re lizing th t bridge will rob bly be needed to is considered n e ergency. T e roble is th t the erichon-
hold the tooth in osition. driu , which su lies nutrition to the se tu , is no longer in
D) Reinsert the tooth into the socket yoursel . cont ct with the se tu bec use o the intervening he to .
T us, the se t l c rtil ge c n necrose le ding to er or ted
Answer 1.8.2 The correct answer is “D.” I there is going to se tu . Se t l he to s should be dr ined cutely nd the
be ny del y in rei l nt tion by dentist, the best course o nose cked to kee the erichondriu in cont ct with the
ction is to reinsert the tooth into the socket yoursel . “A,” “B,” se t l c rtil ge. “A” is incorrect (see revious question). “C”
nd “C” re ll incorrect bec use they will reduce the r te o suc- is incorrect. Kiesselb ch lexus is in the nterior nose nd is
cess ul rei l nt tion. venous lexus. Bleeding is e sily controlled nd gener lly is
sel -li ited. “D,” devi ted se tu , y indic te n underlying
HELPFUL TIP: r cture but in nd o itsel is not n e ergency.
Primary (“baby”) teeth should not be reinserted into
the socket! They ankylose to the bone preventing the You continue to evaluate this patient and note that he has the
eruption o the permanent tooth and cause a cosmetic loss o upward gaze in the right eye, the side on which he was
de ormity. hit. All o the other extraocular motions are intact.

Question 1.8.5 The most likely diagnosis in this patient is:


HELPFUL TIP: A) Blowout r cture with entr ent o the in erior rectus.
Any patient who is in the ED, says he only had three B) Blowout r cture with dys unction o the su erior rectus.
beers, and was “minding his own business” is probably C) Injury to cr ni l nerve III, which controls the su erior AND
not telling the truth on either account. in erior rectus uscles.
D) Volition l re us l to er or u w rd g ze on the right side
in this intoxic ted tient.
You now turn your attention to this patient’s bloody nose and
are trying to decide whether or not get an x-ray. Answer 1.8.5 The correct answer is “A.” T e ost likely di g-
nosis is blowout r cture with entr ent o the in erior rectus.
Question 1.8.3 The BEST timing or a radiograph o the T e orce o blow to the globe is tr ns itted to the in erior
nose is: orbit l w ll, which is the we kest oint in the orbit. T is c n
A) As soon s ossible er the tr u once other injuries re c use entr ent o the contents o the in erior orbit, includ-
st bilized nd ore i ort nt roble s re ddressed. ing the in erior rectus, c using n in bility to er or u w rd
B) As soon s ossible to ssure th t there re no bone r g- g ze. Due to disconjug te g ze, tients with entr ent o the
ents thre tening the br in. in erior rectus uscle ro blowout r cture y co l in o
C) T ere is no need or r diogr h cutely. You c n w it or 3 di lo i . “B” is incorrect bec use blowout r cture gener lly
or 4 d ys. re ers to the in erior orbit l w ll, which would not entr the
D) T ere is never ny indic tion or n s l r diogr hs. su erior rectus. In ddition, tients with n entr ed su e-
rior rectus would h ve di culty with downw rd g ze. “C” is
Answer 1.8.3 The correct answer is “C.” T ere is no need or incorrect bec use it is unlikely th t being hit in the ce would
r diogr hs cutely exce t in extr ordin ry circu st nces. T e c use n injury to CN III. In ddition, CN III lesion would
re sons or r diogr h re to docu ent r cture and to ssist in ect ll extr ocul r uscles exce t or the l ter l rectus (CN
reduction. Bec use o swelling, it is di cult to get good cos etic VI) nd the su erior oblique (CN IV). “D” is incorrect bec use
result reducing n s l r cture cutely. T us, r diogr h is indi- it is i ossible to ove the eyes inde endently o one nother
c ted in 3 to 4 d ys only i there is evidence o n s l de or ity once unless you re ch eleon or rticul rly t lented.
swelling h s resolved. I there is good cos esis nd the tient c n
bre the through his (they re l ost lw ys le) nose, r dio- HELPFUL TIP:
gr h is unnecess ry just to docu ent r cture. “A” nd “B” re Note that a blowout racture may be a good thing. Hav-
incorrect bec use, s noted e rlier, there is no re son to do r dio- ing the racture allows pressures to equilibrate and pre-
gr h t ll unless there is evidence o de or ity once the swelling vents orbital compartment syndrome (proptosis, visual
is resolved. “D” is incorrect or the re sons noted e rlier. loss, etc.). Proptosis with visual loss is a surgical emer-
gency mandating an immediate lateral canthotomy
Question 1.8.4 You get the epistaxis stopped and exam (easy to do . . . . check YouTube).
ine the nasal mucosa. Which one o these is considered an
emergency?
A) Closed n s l r cture.
B) Se t l he to . T e patient has had a long night o partying, and it is 3:00
C) r u to Kiesselb ch lexus. AM Saturday morning when you call your consultant about
D) A devi ted se tu . the blowout racture. T e consultant is not happy and re uses
CHAPTER 1 • EMERGENCY MEDICINE 15

to see the patient acutely. He wants you to send him to the C) C ll the olice to re ove this tient ro your ED.
o ce in 3 days ( uesday morning). D) Use restr ints on the tient nd observe hi until sober, s
sed tive drugs y rolong ti e in the ED.
Question 1.8.6 Your response is: E) Let the tient le ve the ED with co etent dult.
A) o c ll nother consult nt; blowout r cture should be
ttended to i edi tely. Answer 1.8.7 The correct answer is “E.” T e tient w s ini-
B) Do nothing; ev lu tion in 2 to 3 d ys or blowout r cture, ti lly coo er tive nd co etent. Co etence is not b sed on
even with in erior rectus entr ent, is ro ri te. blood lcohol level but r ther on your judg ent o the tient’s
C) St rt steroids to reduce uscle ede to cilit te the s on- bility to ke r tion l decisions. We llow tients on n r-
t neous rele se o the entr ed uscle. cotics to ke decisions bout their own c re ll o the ti e
D) St rt ntibiotics nd hos it lize the tient so th t he c n be des ite h ving n rcotics on bo rd. T ere re tients who will
seen in the orning when the consult nt kes rounds. h ve c city nd re s e t blood lcohol o 200 g/dL nd
E) Stick ins in voodoo doll o your consult nt. others who y be i ired t 80 g/dL. So, judge c city
individu lly.
Answer 1.8.6 The correct answer is “B.” While blowout r c-
Objectives: Did you learn to . . .
tures with uscle entr ent require close ollow-u , there is • Treat acute dental trauma?
no need to intervene cutely. In ct, decision to o er te y
• Diagnose and manage nasal and periorbital trauma?
be del yed or u to 14 d ys. I the entr ent s ont neously
• Care or the intoxicated patient with minor trauma?
resolves when the swelling goes down (not unco on) nd
there is no di lo i or other co lic ting sy to s, surgery
is not needed. T e other nswers re ll incorrect bec use cute
intervention is not required in this tient. “E,” however, y be CASE 1.9
o so e bene t . . . de ending on your voodoo skills.
A 17-year-old emale ell asleep with her contact lenses in her
eyes last evening. T is morning she notes quite a bit o eye
HELPFUL TIP: pain and photophobia. You evert the eyelids (something that
Caveat to the above: In the pediatric population, im- should be done in all cases o possible oreign body) and nd
mediate surgical repair should be undertaken in trap- no evidence o a oreign body. When you stain her eye, you
door fractures. A trapdoor racture is one in which there nd a corneal ulcer.
is signi icant entrapment o the in erior rectus muscle. I
the muscle is le t entrapped in the pediatric population, Question 1.9.1 The treatment or this patient is:
restriction and ibrosis may occur, so immediate evalua- A) Debride ent with burr nd syste ic ntibiotics.
tion by a surgeon is warranted. Oral steroids at a dose o B) Debride ent with cotton sw b nd syste ic ntibiotics.
1 mg/kg may decrease edema in the irst 7 days limiting C) o ic l ntibiotics, cyclo legi , nd re err l to o hth l ol-
ultimate ibrosis. In patients with signi icant sinus dis- ogy.
ease, antibiotics may be considered, usually a penicillin D) Co ious irrig tion, syste ic ntibiotics, nd cyclo legi .
or cephalosporin.
Answer 1.9.1 The correct answer is “C.” T is is n o hth l o-
logic e ergency th t requires to ic l ntibiotics, cyclo legi
( or in control), nd re err l to n o hth l ologist. T ese
T e patient mentioned above has a “ riend” who was also in ulcers c n beco e quite dee nd result in ru tured globe.
the altercation. He, too, was just “minding his business”—
like everyone in the bar—until there was a gentleman’s dis- You consult with your ophthalmologist who would like you
agreement that could only be resolved with a broken bottle. start a cycloplegic agent in this patient prior to trans er.
He has a simple laceration o the chin, which you repair. T is
patient has a blood alcohol level o 150 mg/dL (the legal limit Question 1.9.2 The drug you would choose or a cycloplegic
in most states is 80 mg/dL). Since he is intoxicated, the nurses agent is:
are reluctant to allow the patient to leave because o liability A) Piloc r ine eye dro s.
issues. He seems initially very cooperative and competent. B) i olol eye dro s (e.g., i o tic).
However, the nurse manager reminds you o the legal issues. C) etr c ine eye dro s.
T e patient is getting more agitated; he wants to go home. D) Cyclo entol te eye dro s.

Question 1.8.7 Your response is: Answer 1.9.2 The correct answer is “D.”Cyclo entol te is the only
A) Sed te the tient with h lo eridol nd observe hi until cyclo legic gent listed bove. Cyclo legic gents r lyze the cili-
sober. ry uscle so the eye c nnot cco od te. Piloc r ine is iotic
B) Sed te the tient with benzodi ze ine nd observe hi gent. i olol is bet -blocker used in the tre t ent o gl uco .
until sober. etr c ine eye dro s re to ic l nesthetic. T us, “D” is the only
16 FAMILY MEDICINE EXAMINATION & BOARD REVIEW

correct nswer. Other cyclo legic gents include ho o tro ine The correct answer is “B.” T is tient likely h s UV ker titis.
nd tro ine. However, these h ve rolonged e ect. T e others re not likely bec use they gener lly resent unil t-
er lly. In ddition, in the c ses o “A” nd “C,” they should res-
Question 1.9.3 I your patient just had a simple corneal ent directly er the event r ther th n 9 hours l ter, s in our
abrasion, you would not have had to think so hard! Regard tient.
ing corneal abrasions, you realize that: UV ker titis is ound in tients who re welders or h ve
A) P tching n eye er corne l br sion reduces in nd been out in the sun or n extended eriod o ti e ( t the be ch,
ro otes he ling. snow skiing [“snow blindness”], t nning bed, etc.). UV ker ti-
B) I to ic l ntibiotic is needed er l rge corne l br sion, tis gener lly resents s severe, bil ter l, eye in bout 6 to
gent icin o hth l ic oint ent is the drug o choice. 10 hours ollowing the ctivity. It is tre ted with cyclo legic
C) etr c ine is good to ic l nesthetic nd should be consid- gents nd in edic tion, o en requiring n rcotics.
ered or ho e use in tients with in ul corne l br sion.
D) P tients should void we ring cont ct lenses until the eye
h s been he led or t le st week. HELPFUL TIP:
Patients who have a oreign body in the eye ollowing
Answer 1.9.3 The correct answer is “D.” “A” is incorrect a high-speed injury (e.g., grinding wheel) should be as-
bec use tching n eye y ctu lly incre se in nd decre se sumed to have a globe per oration until proven other-
he ling. Whether or not to use tch should be tter o wise.
tient co ort only. “B” is incorrect bec use gent icin o h-
th l ic oint ent ( s well s other to ic l inoglycosides)
ctu lly reduces he ling o the corne , nd ntibiotics re not QUICK QUIZ: ORTHOPEDIC EMERGENCIES
necess ry unless there re signs o in ection. “C” is incorrect
bec use tients should never be sent ho e with to ic l nes- Which o the ollowing is ost co only ssoci ted with sig-
thetic. T ey reduce he ling nd c n le d to urther injury i ni c nt v scul r injury?
the tient, whose eyes re now insens te, continues h r ul A) Pubic r us r cture.
ctivity, rubs his/her eyes, etc. B) Knee disloc tion.
C) Shoulder disloc tion.
HELPFUL TIP: D) Elbow disloc tion.
To di erentiate a topical ophthalmologic problem rom E) Ankle disloc tion.
iritis, put in some tetracaine. I the pain resolves, it is
likely, but does not prove, that the problem is su- The correct answer is “B.” In u to 33% o knee disloc tions
per icial (e.g., corneal abrasion). Posttraumatic iritis is (not patellar dislocations), o lite l rtery injury c n be iden-
mani ested by ciliary lare, anterior chamber cells, and ti ed. It is deb ted s to whether ll tients with knee dislo-
marked photophobia. These patients really need a slit c tions require ngiogr hy or C ngiogr hy or i hysic l
lamp examination. ex in tion nd nkle-br chi l indices re su cient to rule out
o lite l rtery injury, but v scul r injury is jor c use o
Objectives: Did you learn to . . . li b loss nd orbidity. “A” is incorrect bec use ubic r us
• Recognize a corneal ulceration and treat it appropriately?
r ctures re rel tively inor injuries without v scul r involve-

ent, requiring only in control. Shoulder disloc tions (“C”)
Treat corneal abrasions?
re co only ssoci ted with injury to the xill ry nerve.
• Understand the proper use o cycloplegic agents?
Elbow disloc tions (“D”) c n be ssoci ted with injury to the
edi n nerve nd br chi l rtery. However, rteri l injuries re
QUICK QUIZ: EYE TRAUMA uch less co on th n with knee disloc tions. Ankle disloc -
tions (“E”) re r rely ssoci ted with v scul r injury.
You re on c ll or your grou nd welder who w s welding
nd grinding resents t 2:00 a m with severe bil ter l eye in.
When he le work t 5:00 pm the d y be ore, he did not notice
CASE 1.10
ny roble . He notes th t he w s we ring his d rk goggles A 55-year-old male armer is injured by a grass- ed cow
so e o the ti e while he w s welding but did quite bit o that pins him against a ence. His leg was trapped against
work without goggles s well. the ence or a several minutes. Being a typical Midwestern
armer, he ignores the injury until his wi e convinces him
T e ost likely di gnosis in this tient is: later that a ernoon to have it evaluated. He presents to your
A) Foreign body. o ce complaining o severe pain in the cal area. A radio-
B) Ultr violet (UV) ker titis. graph is normal, and the patient has normal distal pulses.
C) Globe enetr tion second ry to the welding nd oreign body. T e cal (his leg, not the cow) is tender with increased pain
D) Iritis. on passive stretch. His pain seems to be out o proportion to
CHAPTER 1 • EMERGENCY MEDICINE 17

his injury. T e cal (the cow) may also be tender and USDA co gul tion studies y be ro ri te de ending on the clini-
grade prime. c l situ tion but re not use ul in est blishing the resence o
yoglobinuri .
Question 1.10.1 Which o the ollowing is true?
A) Since the tient h s excellent ulses, co rt ent syn- HELPFUL TIP:
dro e is not likely. Myoglobin can be measured in the urine. However,
B) Co rt ent syndro e is de ned s co rt ent res- many laboratories have stopped doing this test avor-
sure >30 Hg. ing the positive dipstick/negative microscopic exami-
C) Co rt ent syndro e is only ssoci ted with signi c nt nation approach. There can be other causes of a heme
crush injuries or r ctures. positive dipstick. Thus, always check a CPK as well i
D) P in out o ro ortion to the injury is red g or co rt- rhabdomyolysis is a consideration.
ent syndro e.

Answer 1.10.1 The correct answer is “D.” P in out o ro or- T e patient has a positive dipstick or blood with no red blood
tion to the injury is red g or co rt ent syndro e. “A” cells on microscopic examination (presumptive myoglobin-
is incorrect bec use ulses c n be int ined until there is uria). His serum CPK is 32,000 U/L, which is well above ve
signi c nt incre se in co rt ent ressures nd signi c nt times the upper limit o normal (the cuto or consideration
injury to uscle nd nerves. “B” is incorrect bec use it is di - o rhabdomyolysis, although levels o > 15,000 U/L are com-
cult to de ne s eci c cut o or co rt ent syndro e. mon when one has symptomatic rhabdomyolysis), so you
So e tients toler te higher ressures nd others c nnot make the diagnosis o rhabdomyolysis.
toler te 30 Hg (nor l co rt ent ressure is zero).
However, when the ressure gets bove 20 to 30 Hg, strong Question 1.10.3 The most common adverse consequence
consider tion should be given to the resence o co rt ent and greatest danger o rhabdomyolysis is:
syndro e. “C” is incorrect. Co rt ent syndro e c n be A) Disse in ted intr v scul r co gul tion.
due to nu ber o ctors including electric l injury, exces- B) Acute kidney injury.
sive uscle use, tet ny, re er usion er ische i , ggressive C) Seizure ro hy oc lce i .
volu e resuscit tion, etc. D) Acute gout ro hy erurice i .
E) C rdi c rrhyth i ro hy erk le i .
HELPFUL TIP:
The classic indings o arterial insu iciency (the “5 Ps” be- Answer 1.10.3 The correct answer is “B.” Myoglobin reci i-
ing pulselessness, paresthesia, pallor, pain, and paralysis) t tes in the ren l tubules c using cute kidney injury. “A,” DIC,
are o ten considered necessary or compartment syn- c n occur but is r re. “C,” seizures ro hy oc lce i , h ve not
drome to be diagnosed. This is incorrect. O these, pain is been re orted in this condition, nor h s “D,” gout. T e ot s-
o ten the only symptom; the second most requent would siu elev tion ro rh bdo yolysis y re ch level su cient
be paresthesia. I your patient has compartment syndrome to c use rrhyth i s; this is ex cerb ted by ossible coexistent
with the 5 Ps present, there is likely extensive injury. hy oc lce i .

Question 1.10.4 The primary treatment or rhabdomyolysis


is:
You decide that it is likely that this patient has a compart- A) M nnitol in usion.
ment syndrome. B) S line in usion.
C) Furose ide.
Question 1.10.2 Which o the ollowing labs will be the D) Di lysis.
most help ul in guiding treatment or this patient?
A) CBC. Answer 1.10.4 The correct answer is “B.” T e ost i or-
B) Urin lysis. t nt tre t ent or rh bdo yolysis is s line in usion. L ct ted
C) Glucose. Ringers should be voided s it cont ins ot ssiu , which y
D) Sodiu . urther contribute to hy erk le i s entioned bove. T ere
E) P /P . h s reviously been deb te i lk liniz tion o the urine (using
IV sodiu bic rbon te) h s ny dditive bene t or revent-
Answer 1.10.2 The correct answer is “B.” One o the jor ing cute kidney injury over s line lone, but there is no evi-
co lic tions o co rt ent syndro e is rh bdo yolysis. dence th t su orts this r ctice. “A,” nnitol, c n be used to
T is will ni est itsel s urine which is di stick ositive incre se urine ow, but this is re lly tre t ent th t is second-
or blood but with neg tive icrosco ic ex in tion or red ry to good hydr tion nd y c use hy ovole i co ound-
blood cells. T e ositive di stick is icking u yoglobin in ing the roble “C,” urose ide, is controversi l nd h s no
the urine. Rh bdo yolysis c n be con r ed by seru level outco e bene t but c n be used or uid overlo d. “D,” di lysis,
o cre tine hos hokin se (CPK). CBC, glucose, sodiu , nd is wh t we re trying to void using s line.
18 FAMILY MEDICINE EXAMINATION & BOARD REVIEW

chills, dys ne , r les, nd rhonchi. T e etiology is unknown, but


HELPFUL TIP:
it y be second ry to in rction o the lung nd/or t e boli.
In adult patients with rhabdomyolysis, the goal is to
All o the other di gnoses should lso be entert ined t this
maintain urine output o 200 to 300 cc/hr.
oint, but cute chest syndro e is ost likely. Sickle cell-rel ted
eric rditis (“D”) is r re co lic tion o the dise se.

T e patient is able to maintain urine output a er you insti- Question 1.11.2 All o the ollowing are recommended in
tute saline. the initial treatment o acute chest syndrome EXCEPT:
A) Hydroxyure .
Question 1.10.5 What treatment are you going to suggest B) Oxygen.
or the underlying compartment syndrome? C) IV nor l s line.
A) F scioto y. D) Mor hine.
B) I obiliz tion nd tr ction.
C) Hot cks nd elev tion o the ected li b. Answer 1.11.2 The correct answer is “A.” Hydroxyure , while
D) Ice nd elev tion o the ected li b. use ul or the chronic tre t ent o sickle cell ne i , is not
indic ted or the tre t ent o cute chest syndro e. However,
Answer 1.10.5 The correct answer is “A.” T e tre t ent o it c n reduce the incidence o cute chest syndro e by 50%
co rt ent syndro e is scioto y. A r id surgic l or when used chronic lly. M n ge ent o cute chest syndro e
ortho edic consult tion is critic l in the tre t ent o co rt- includes llevi ting hy oxi (“B”), IV uid resuscit tion (“C”),
ent syndro e. nd ro ri te n lgesi (“D”). Other tre t ents include
IV ntibiotics to cover or co unity- cquired neu oni
T e patient does well and everyone is happy . . . except or the ( lthough cute chest syndro e is not bacterial). It is rudent
cow, who nds his way onto the table as the centerpiece o to cover these tients with ntibiotics bec use dult tients
Christmas dinner. with sickle cell re de cto s lenecto ized nd the initi l re-
Objectives: Did you learn to . . . sent tion o cute chest syndro e c n be e sily con used with
• Recognize mani estations o compartment syndrome and neu oni .
understand that compartment syndrome can be present
with pain alone? T e patient continues to be hypoxic despite your therapy. His
• Identi y patients at risk or compartment syndrome and CBC shows a slight elevation in the WBC count and a hemo-
rhabdomyolysis? globin o 9 g/dL. A chest radiograph indicates progression o
• Manage compartment syndrome? in ltrates.
• Diagnose and treat rhabdomyolysis?
Question 1.11.3 The next step in treating this patient is:
A) Fresh rozen l s .
CASE 1.11 B) Pentoxi ylline.
C) P cked red blood cells.
A 24-year-old A rican-American male presents to the ED D) Exch nge tr ns usion.
complaining o ever, chills, and dyspnea. He has chest pain E) Any o the bove.
that is respirophasic (“pleuritic”) in nature. He is noted to be
tachypneic with a respiratory rate o 36 and an oxygen satu- Answer 1.11.3 The correct answer is “D.” P tients with cute
ration o 90%. He has a history o sickle cell disease and has chest syndro e who re in hy oxic with rogressing in l-
had a number o sickle cell crises in the past. He is up to date tr tes re c ndid tes or exch nge tr ns usion to bring the level
on immunizations, including Streptococcus pneumoniae and o HbS to <30% o the tot l. Si ly d inistering blood (“C”)
Haemophilus inf uenzae vaccines. will not resolve the roble bec use HbS will still be resent in
signi c nt ounts. I this tient h d ore signi c nt ne-
Question 1.11.1 The patient’s current symptoms are MOST i , cked red cell tr ns usion would be ore vi ble o tion.
concerning or and suggestive o : But tr ns usion t level o 9 g/dL o he oglobin is not indi-
A) Pneu othor x. c ted (gener lly the threshold is 7 g/dL in the he odyn i-
B) Pul on ry e bolis . c lly st ble tient). “A” nd “B” re lso incorrect. Fresh rozen
C) Acute chest syndro e. l s h s no role in the tre t ent o cute chest syndro e,
D) Sickle cell-rel ted eric rditis. nor does entoxi ylline.
E) T or cic ortic neurys dissection.
Your patient recovers rom this episode. He has had numer-
Answer 1.11.1 The correct answer is “C.” T is tient likely h s ous pain crises in the past, as well as hospitalizations or other
cute chest syndro e, which is ssoci ted with sickle cell ne- reasons. You have an opportunity to provide some patient
i and may be indistinguishable rom pneumonia. Acute chest education. You answer a ew o your patient’s questions and
syndro e is ch r cterized by leuritic chest in, ever, cough, then review potential mani estations o sickle cell disease.
CHAPTER 1 • EMERGENCY MEDICINE 19

Question 1.11.4 Which o the ollowing may be a mani es


tation o sickle cell disease? CASE 1.12
A) Joint nd bone in. A 52-year-old truck driver presents to your ED a er being out
B) Acute bdo in l in. in subzero temperatures or several hours trying to repair his
C) Acute sequestr tion syndro e. truck. He is hypothermic when you use a low-reading rectal
D) A l stic crisis. thermometer with appropriate calibration (“T anks or
E) All o the bove. getting the most accurate temperature, doc!”). His initial core
temperature is noted to be 28◦ C. He has a pulse o 24 bpm, a
Answer 1.11.4 The correct answer is “E.” All o the bove c n BP o 70/30 mm Hg, and slow mentation. However, he is awake,
be ssoci ted with sickle cell ne i (kee re ding or ddi- and thus able to joke about a thermometer in his rectum.
tion l in or tion).
Question 1.12.1 The appropriate rst line treatment or
Question 1.11.5 Which o the ollowing in ections is a this patient’s pro ound bradycardia is:
common cause o aplastic crisis in sickle cell anemia? A) Atro ine.
A) P rvovirus B-19. B) E ine hrine.
B) In uenz virus. C) Do ine.
C) CMV virus. D) Lidoc ine.
D) P r in uenz virus. E) Re-w r ing.
E) None o the bove.
Answer 1.12.1 The correct answer is “E.” T e hy other ic
Answer 1.11.5 The correct answer is “A.” P tients with sickle he rt is gener lly resist nt to drugs. T us, the best tre t ent or
cell ne i c n develo l stic ne i in res onse to rvo- this tient is re-w r ing. Br dyc rdi ro other c uses c n
virus B-19 in ection. E stein–B rr virus nd so e b cteri h ve be tre ted with tro ine 0.5 g IV ush or in usion o e ine h-
lso been re orted to c use l stic crisis in tients with sickle rine or do ine.
cell ne i .
Question 1.12.2 All o the ollowing are acceptable meth
Question 1.11.6 Acute sequestration syndrome is a mani ods o re warming THIS patient EXCEPT:
estation o sickle cell anemia. In which group does acute A) Active extern l re-w r ing (e.g., hot cks).
sequestration syndrome occur? B) I ersion in 40°C w ter.
A) Younger th n 5 ye rs. C) P ssive extern l re-w r ing (e.g., bl nkets).
B) 5 to 12 ye rs old. D) He ted, hu idi ed oxygen.
C) 12 to 25 ye rs old. E) T or cic l v ge with w r uids.
D) Older th n 25 ye rs.
E) Older th n 65 ye rs. Answer 1.12.2 The correct answer is “C.” P tients with te -
er ture o below 30°C gener lly do not h ve enough endog-
Answer 1.11.6 The correct answer is “A.” Acute sequestr tion enous he t roduction to e ectively re-w r the selves.
syndro e occurs when the s leen sequesters red blood cells, le d- T us, extern l or intern l active re-w r ing is indic ted. All
ing to dro in he oglobin. T e resent tion c n be quite dr - o the other o tions re cce t ble ethods o re-w r ing this
tic with severe le u er qu dr nt in, s leno eg ly, nd tient. Extr cor ore l blood w r ing vi ECMO or di lysis
ro ound ne i , so eti es resulting in hy ovole ic shock nd long with thor cic c vity l v ge vi chest tubes nd w r crys-
de th. Bec use it requires unction l s leen, it is ost co on in t lloid re lso e ective. G stric, rect l, nd bl dder l v ge with
children younger th n 5 ye rs. P tient with sickle cell ne i who wr uids re gener lly not very e ective bec use o the li -
re older th n 5 ye rs gener lly do not h ve unctioning s leen; ited sur ce re involved nd c n c use l rge electrolyte shi s.
ost o en it h s in rcted so th t cute sequestr tion syndro e
no longer occurs. T e ort lity is 15% er e isode nd 50% recur. Question 1.12.3 Rapid re warming o the extremities is
associated with:
HELPFUL TIP: A) Alk losis, hy ok le i .
Exchange trans usions to reduce the percent o HbS to B) Acidosis, hy ok le i .
<30% is also indicated in stroke. Also keep the hemoglo- C) Acidosis, hy erk le i .
bin >9.0 g/dL in those with a sickle cell–related stroke. D) Alk losis, hy erk le i .
E) Mixed cid–b se disorder.
Objectives: Did you learn to . . . Answer 1.12.3 The correct answer is “C.” Re-w r ing o the
• Recognize acute chest syndrome? extre ities c n le d to return o cold blood to the core le ding
• Manage a patient with acute chest syndrome? to r doxic l dro in body te er ture. In ddition, hy o-
• Use exchange trans usion in a patient with sickle cell anemia? ther i c uses l ctic cidosis with hy erk le i in the extre -
• Recognize various other mani estations o sickle cell anemia? ities. As the eri her l blood is re-w r ed nd eri her l
20 FAMILY MEDICINE EXAMINATION & BOARD REVIEW

v sodil tion occurs, the hy erk le ic, cidotic blood is obi- Answer 1.12.6 The correct answer is “E.” T e ro ri te
lized to the tient’s centr l circul tion, with result nt syste ic dosing o or hine in cute in is never-ending source o
et bolic cidosis nd hy erk le i . ze ent to our resident hysici ns who re er to st rt with
1 to 2 g IV. Nonetheless, the correct dose o IV or hine is
Question 1.12.4 Which o the ollowing is NOT associated 0.1 g/kg or 10 g in this 100 kg le. Si il rly, the correct
with an increased risk o hypothermia? dose o ent nyl is 1 µg/kg (100 µg in 100-kg dult) nd the
A) Di betes ellitus. dose o hydro or hone is 0.015 g/kg (1 to 2 g in 100-kg
B) Obesity. dult). However, there re lly is no “ xed” dose o n rcotic in
C) Alcohol use. edic tion in the ED. itr te the dose until you obt in in
D) Old ge. relie —with the tient still bre thing, we ho e.
E) Chronic illness.
It is 2 days later. T e patient is noted to have black eschar on
Answer 1.12.4 The correct answer is “B.”In Iow , we st rt to work multiple ngers and toes. T ere is no obvious per usion to
on our winter t l yer in October or just this re son. Why do you these areas.
think we e t ll th t c ndy corn? Obese tients h ve s ller
body ss to sur ce re r tio nd do not h ve n incre sed risk Question 1.12.7 The best course at this point is:
o hy other i . “C,” lcohol use, c uses tients to be rel tively A) Debride ent o the nonvi ble tissue.
insens te to cold (thus the ter “liquid j cket”), c uses eri h- B) Skin gr ing over o en re s er debride ent.
er l v sodil t tion, incre sing he t loss, nd c uses oor choices C) Observ tion or nu ber o weeks des ite the bl ck esch r.
(like not we ring re l j cket. Just t ke look t those young col- D) A ut tion o the nonvi ble dist l digits.
lege ootb ll ns in Nove ber without shirts!). T er oregul -
tion is i ired s we ge. T us, “D,” old ge, is ssoci ted with Answer 1.12.7 The correct answer is “C.” It c n t ke weeks or
gre ter ro ensity tow rd hy other i . Di betes (“A”) nd ny the ro er de rc tion line or debride ent nd gr ing to
chronic illness (“E”) c n lso redis ose to hy other i . beco e rent. T us, ggressive intervention t this oint is
counter roductive nd y le d to ddition l tissue loss. For
T e patient’s mental status clears and he complains that his n- this re son, “A” nd “D” re incorrect. Skin gr ing is lso not
gers and toes, which were numb and cold, are now quite pain- ro ri te t this ti e bec use debride ent o the esch r is
ul. You note that there is probably reezing o tissue ( rostbite). not ro ri te.
Objectives: Did you learn to . . .
Question 1.12.5 The best method o re warming the rost
• Identi y severe bradycardia in hypothermia and treat it
bite is:
appropriately?
A) Slowly in te id w ter.
• Manage a patient with hypothermia?
B) R idly in the hottest w ter he c n st nd (tested by you, o
• Use methods o re-warming and identi y complications o
course, to ensure th t there will be no burns).
re-warming?
C) Using hot ir source such s h ir dryer.
• Recognize risk actors or hypothermia?
D) Using oist he t vi he ting d.
• Diagnose and manage rostbite?
E) We ring ittens.

Answer 1.12.5 The correct answer is “B.” Frostbitten rts


should be re-w r ed s quickly s ossible in hot w ter between QUICK QUIZ: DANGER IN THE LAUNDRY ROOM
37°C nd 40°C. W ter te er ture cooler nd hotter th n this
c n le d to incre sed tissue loss. T e other ethods “A,” “C,” Which o the ollowing is true bout the ingestion o household
“D,” nd “E” re not reco ended. Do not re-warm parts that ble ch?
may become rozen again (e.g., i you re in the eld). Re- reez- A) P tients who drink household ble ch re t high risk o
ing will c use ddition l d ge. eso h ge l nd g stric burns.
B) Or l burns re good redictor o eso h ge l burns.
T e patient has a lot o pain a er thawing and reper usion. C) All tients who ingest household ble ch should be re erred
You control the pain with morphine. or u er endosco y to rule out burns.
D) Household ble ch ingestions re gener lly benign nd re-
Question 1.12.6 Which o the ollowing is the most appro quire no tre t ent i the tient is not sy to tic.
priate dose o morphine in this hemodynamically stable
100 kg male? The correct answer is “D.” Most household ble ch ingestions
A) 2 g IV. re benign nd need no ther y i the tient is sy to tic.
B) 4 g IV. However, this does not extend to industrial bleach or drain
C) 6 g IV. cleaner. T ere is high risk o eso h ge l nd g stric burns
D) 8 g IV. with industri l ble ch. “B” is incorrect. T e or l ucos y
E) 10 g IV. be nor l in industri l ble ch or dr in cle ner ingestion nd
CHAPTER 1 • EMERGENCY MEDICINE 21

there y still be signi c nt eso h ge l nd g stric burns. For is less ev or tive sur ce re er kilogr o weight. All o
this re son, ll tients with dr in cle ner or industri l ble ch the others redis ose to he t-rel ted illness. “A” nd “C” reduce
ingestion should undergo u er endosco y. “C” is incorrect; swe ting nd, in the c se o “A,” incre se et bolic r te. Both
tients with household ble ch ingestions do not require u er o these redis ose to he t-rel ted dise se. O rticul r note is
endosco y. “E.” S ll children swe t less re dily th n do dults. T is redis-
oses the to he t-rel ted dise se, but kes the less stinky
CASE 1.13 th n dults. T e elderly do not h ve the s e co ens tory
bility s younger eo le.
An 18-year-old male was working outside in the heat and
humidity. T e outside temperature reached 105°F with 90% HELPFUL TIP:
humidity. He usually lives in northern Canada and works or Up to 80% o patients with heat stroke will not have a
the government tracking the migration o caribou—but he is prodrome o nausea, lightheadedness, con usion, head-
here in Iowa on a job detasseling corn. (Don’t believe it? Look ache, etc., which is seen in heat exhaustion. Make sure
at www.teamcorn.com—seasonal hard work that pays well.) you check hepatic enzymes in patients in whom you
His riends noticed that he became con used, complained o suspect heat stroke. They are almost uni ormly elevated
a headache and muscle cramps, and became light-headed. On (beware they may take several hours to rise) and nor-
arrival to your ED, he is not sweating and is lethargic. His mal liver enzymes should cause you to question your
rectal temperature is 41.5°C. He says with a smile, “Guess I diagnosis.
just can’t handle the heat, eh?”

Question 1.13.1 All o the ollowing are indicated in the Objectives: Did you learn to . . .
treatment o this patient EXCEPT: • Recognize and manage heat exhaustion/heat stroke?
A) P ck the tient in ice to reduce core te er ture. • Recognize risk actors or heat exhaustion/heat stroke?
B) IV uids.
C) Use n nd s r y w ter on the tient to ro ote ev o-
r tive cooling.
D) Ad inister glucose i the tient is hy oglyce ic.
CASE 1.14
A 19-year-old emale presents to the ED with complaints o
Answer 1.13.1 The correct answer is “A.” P cking tients in wheezing. She has a history o asthma and you have been
ice is contr indic ted. ot l body i ersion in ice water is use- ollowing her since her eighth birthday, when her mother
ul but cking the erson in ice ctu lly reduces cooling or noticed that she couldn’t blow out her candles. In general,
two re sons. First, it c uses cut neous v soconstriction. Second, she has mild asthma not requiring an inhaled steroid. How-
it does not llow conductive cooling such s would be seen in ice ever, over the past several months, things have accelerated so
w ter sub ersion: solid ice does not h ve s uch skin cont ct that she now uses her rescue inhaler daily. On examination,
s w ter or the circul tion to conduct w y the he t. Re e ber she is tachypneic, using accessory muscles o respiration with
th t sub ersion in ice is lso ssoci ted with c using hypother- a respiratory rate o 30 and wheezing in all elds. Her oxy-
i .T e ro ri te tre t ent o he t exh ustion/he t stroke gen saturation is 95% and pulse is 110 bpm with a normal
(he t stroke being de ned s CNS dys unction with ch nge BP. Her blood gas is as ollows: pH 7.40, CO2 40 mm Hg, O2
in the level o consciousness) is cool w ter-so ked bl nkets 80 mm Hg, and HCO3 24 mEq/L.
nd towels with ns i ed t the tient. T is llows ev or -
tive cooling nd lso conductive he t loss (to the w ter in the Question 1.14.1 A normal blood gas in this patient sug
towels). Anti yretics re gener lly not e ective bec use by this gests that:
oint, the tient’s endogenous ther oregul tion is k ut. A) T is is ild ex cerb tion th t should res ond well to ther-
y.
Question 1.13.2 Which o the ollowing IS NOT a contribut B) She h s res ir tory cidosis.
ing actor to heat exhaustion/heat stroke? C) She h s res ir tory lk losis.
A) Use o sti ul nts such s e hedr or het ines. D) T is is severe ex cerb tion th t will require ggressive
B) Dehydr tion. ther y.
C) Anticholinergic drugs. E) T is is nor l blood g s; she c n be disch rged.
D) T in body h bitus.
E) Extre es o ge. Answer 1.14.1 The correct answer is “D.” A H o 7.4 with
CO2 o 40 Hg in tient who is sth tic nd t chy neic
Answer 1.13.2 The correct answer is “D.” T is is why Iow ns is b d sign. T e CO2 should be low in t chy neic tient
st rt to work on their swi suit gures in A ril—it’s t- bec use they will be blowing o CO2. T us, nor l CO2 nd
ter o li e-or-de th, not n rcissis . A thin body h bitus is not nor l H indic te th t the tient is ret ining CO2. T is is
risk ctor or he t stroke/exh ustion; the o osite is true. just nother c se where looking t the tient is ore i or-
Obesity redis oses to he t stroke/exh ustion bec use there t nt th n looking t the l bs. Even though the blood g s itsel is
22 FAMILY MEDICINE EXAMINATION & BOARD REVIEW

technic lly within nor l li its, this tient clinic lly e rs Answer 1.14.4 The correct answer is “E.” Albuterol c n be
sick. “B” nd “C” re both incorrect, since the blood g s indi- d inistered vi nebulizer continuously i needed, even in the
c tes neither n cidosis nor lk losis. edi tric ge grou . chyc rdi , one o the in side e ects
o lbuterol tre t ent, will o en improve with continuous
Question 1.14.2 Which o the ollowing tests are indicated in lbuterol. T is occurs bec use the tient’s t chyc rdi is o en
routine evaluation o a patient with an asthma exacerbation? driven by hy oxi . Once the sth is dequ tely tre ted, oxy-
A) Chest x-r y. gen tion i roves, nd the ulse co es down.
B) CBC.
C) Arteri l blood g s. T e patient does not respond well to albuterol alone, so you
D) All o the bove. request the addition o ipratropium. At this point, you also
E) None o the bove. want to order steroids.

Answer 1.14.2 The correct answer is “E.” None o the bove Question 1.14.5 Which o the ollowing statements about
tests re indic ted in the routine ev lu tion o n sth ex c- steroid use in asthma exacerbation is true?
erb tion. A chest x-r y should be reserved or those tients in A) IV steroids re su erior to PO steroids in the tre t ent o
who neu oni or other ul on ry rocess is sus ected. A sth .
CBC is not going to ch nge your ther y in the routine sth B) All tients who re steroid de endent should h ve ddi-
ex cerb tion nd is not indic ted. Likewise, n ABG is unnec- tion l syste ic steroids even i they h ve lre dy t ken their
ess ry in ost sth ex cerb tions. It c n be used to ssist in dose or the d y.
your clinic l ev lu tion to deter ine whether or not the tient C) T e e ective dose r nge or steroids in sth is well est b-
is ret ining CO2; however, even in the “cr shing tient,” n lished.
ABG is not necess ry bec use intubation is a clinical decision D) Only tients requiring d ission should h ve or l or r-
and should not be based on the blood gas. enter l steroids.

Question 1.14.3 You decide to initiate therapy or this Answer 1.14.5 The correct answer is “B.” All tients who re
patient. O the ollowing options, the initial treatment o steroid de endent should get steroids i they resent to the ED
this patient is: with n cute ex cerb tion o sth . “A” is incorrect. IV ste-
A) Subcut neous e ine hrine. roids nd or l steroids h ve the s e e c cy in cute sth
B) Albuterol MDI ( etered-dose inh ler) with s cer. ex cerb tions. T us, the choice o route de ends ostly on
C) Nebulized i r tro iu . convenience nd cost. “C” is incorrect. Multi le steroid dosing
D) Or l steroids. regi ens nd r nges o doses h ve been used in sth with
E) IV steroids. success. “D” is incorrect. Disch rged tients who h ve ny-
thing ore th n inor sth ex cerb tion should receive
Answer 1.14.3 The correct answer is “B.” T e initi l tre t ent steroids.
or this tient— nd ny tient resenting with n sth
ex cerb tion—is bronchodil tor. A bet - gonist is re erred, HELPFUL TIP:
in this c se lbuterol. It kes little di erence whether this is When compared with oral steroids, IV steroids may in-
vi nebulizer or MDI, s long s one uses dequ te doses. One crease hospitalizations, cost, and treatment ailure in
lbuterol nebuliz tion is equ l to bout 8 to 10 u s o n lb- those with chronic obstructive pulmonary disease. Use
uterol MDI with s cer. “A” is incorrect bec use subcut neous oral steroids whenever possible; the bioavailability is
e ine hrine is second or third line in the tre t ent o sth . high (JAMA. 2010;303(23):2359–2367).
“C” is incorrect. While i r tro iu is e ective in sth , it is
second ry to lbuterol in the tre t ent o sth . “D” nd “E”
re incorrect. Steroids re indic ted, but bronchodil tor ther y Question 1.14.6 Which o the ollowing is true about the
is the ri ry tre t ent in cute sth ex cerb tions. role o theophylline in the treatment o acute asthma
exacerbation?
T ere is no albuterol MDI available to you in your ED, so the A) heo hylline/ ino hylline should be used in c ses
patient receives nebulized albuterol. However, she continues unres onsive to two to three doses o nebulized lbuterol
to wheeze. since it h s dded bene its when used with n inh led
bet - gonist.
Question 1.14.4 How many albuterol treatments can this B) P tients who get theo hylline/ ino hylline h ve ore
patient sa ely receive? side e ects th n do tients who get continuously nebulized
A) One every other hour. lbuterol nd get no bene t ro the drug.
B) One er hour. C) I you choose to use theo hylline/ ino hylline, the ther -
C) wo er hour. eutic go l is seru level o 150 µg/dL.
D) T ree er hour. D) T eo hylline/ ino hylline h s side e ect ro le su e-
E) Continuous nebuliz tion o lbuterol is s e. rior to bet - gonists like lbuterol.
CHAPTER 1 • EMERGENCY MEDICINE 23

Answer 1.14.6 The correct answer is “B.” P tients who re Objectives: Did you learn to . . .
tre ted with theo hylline h ve ore side e ects, including • Recognize clinical and blood gas mani estations o a severe
t chyc rdi , n use , nd rrhyth i s, th n do tients who get asthma exacerbation?
continuously nebulized lbuterol (“D” is incorrect). T eo hyl- • Evaluate a patient presenting with an asthma exacerbation?
line/ ino hylline h s essenti lly no role in the tre t ent o • Initiate treatment or asthma in the ED?
cute sth ex cerb tions. T ere is no bene t to theo hyl- • Recognize the pit alls in using theophylline/aminophylline
line or ino hylline over optimal bet - gonist ther y (e.g., or asthma?
continuous nebulized lbuterol i required). “C” is incorrect • Formulate a plan or discharging an asthma patient rom the
bec use i used t ll, the ther eutic go l or theo hylline is ED?
seru level o 15 µg/dL.
CASE 1.15
HELPFUL TIP:
A 7-year-old presents to the ED with wheezing and hives a er
Magnesium sul ate (2 g over 10 minutes in adults,
being stung by a “bee.” He was evidently throwing rocks at
25 mg/kg in children) can be used in patients with sta-
a yellow-jacket nest when he was stung, so at least he hope-
tus asthmaticus. Magnesium is a direct smooth muscle
ully learned something. On examination, the patient has
relaxant. Not all patients will respond, and in patients
hives and wheezing with a normal BP or his age. He is mildly
who will respond, you can expect a 60- to 90-minute
tachycardic.
e ect. Avoid using magnesium in patients with renal
ailure since they may become toxic.
Question 1.15.1 Potentially use ul treatments or this patient
include all o the ollowing EXCEPT:
A) IV di henhydr ine.
T e patient responds to nebulizers and steroids. You decide B) Intr uscul r (IM) e ine hrine.
to send her home. C) Subcut neous (SC) di henhydr ine.
D) IV ci etidine.
Question 1.14.7 Which o the ollowing is true?
A) You should disch rge the tient on 2 u s o n lbuterol Answer 1.15.1 The correct answer is “C.” Subcut neous
MDI vi s cer to be used PRN. di henhydr ine c n c use skin necrosis nd is contr indi-
B) You should l ce the tient on steroid, t ering the dose c ted. Either IV or IM di henhydr ine c n be used. O the
over 3 weeks. others, intramuscular (not SC) e ine hrine should be used in
C) You should disch rge the tient on 8 to 10 u s o n lb- the tient with n hyl xis; it is the drug o choice or n hy-
uterol MDI vi s cer to be used every 6 hours round the l xis. Re e ber th t di henhydr ine will not reverse bron-
clock. chos s or hy otension. Subcut neous e ine hrine is err ti-
D) You should st rt the tient on steroid inh ler. c lly bsorbed. I hy otension ersists des ite IM e ine hrine
E) You should st rt the tient on long- cting bronchodil tor nd IV uids, IV e ine hrine should be d inistered. Intr ve-
inh ler. nous H2 blockers (e.g., ci etidine, r nitidine) re rticul rly
e ective in the tre t ent o n hyl xis nd should be used
Answer 1.14.7 The correct answer is “D.” T e tient should be routinely in these tients, lso.
st rted on steroid inh ler to revent recurrent ex cerb tions.
She h s been using her lbuterol d ily, indic ting oor control.
Overl ing this with or l steroids will give the inh led steroid HELPFUL TIP:
ch nce to work while the tient is being covered with the or l Only honeybees generally leave a stinger. Remove it by
steroids. “A” is incorrect. One nebuliz tion is equ l to 8 to 10 any means possible. The amount o envenomation is
u s o n MDI. I you si ly go b ck to low-dose lbuterol, the directly proportional to the amount o time the stinger
tient is ore likely to do oorly. We tend to underdose lb- is in the skin and not to how you remove it (credit card,
uterol inh lers; tients c n s ely t ke ore th n 2 u s. “B” is orceps, etc.).
incorrect bec use tients do not need a steroid taper i they re
not on chronic steroids nd will not be t king steroids or ore
th n 10 d ys. You c n si ly tre t the tient (e.g., with red- T e patient responds well to the therapy as noted earlier. You
nisone 40 g PO QD or 5–10 d ys) nd then sto . No t er is are going to discharge him and want to write his prescriptions.
needed. Note that this is not true or patients on chronic steroids
who clearly do need a taper. “C” is incorrect bec use scheduled Question 1.15.2 The patient should be discharged with
lbuterol is not s e ective s PRN use. In ddition, lbuterol which o the ollowing?
c n cert inly be used ore th n every 6 hours. “E” is incorrect A) Di henhydr ine Q 6 hours or the next 48 hours.
bec use long- cting inh led bronchodil tor ther y should not B) Ci etidine Q 12 hours or the next 48 hours.
be used lone in sth due to the otenti l to incre se ort l- C) An n hyl xis (“bee sting”) kit.
ity; it should only be used with n inh led steroid. D) All o the bove edic tions.
24 FAMILY MEDICINE EXAMINATION & BOARD REVIEW

Answer 1.15.2 The correct answer is “D.” P tients c n h ve Vital signs: temperature 37.0°C, pulse 110 bpm, respira-
bi h sic re ction edi ted by “slow re cting subst nce o n - tions 18, and BP 120/85 mm Hg. He is in distress secondary
hyl xis” which is now believed to be neutro hil che ot ctic to pain. Abdomen: normal bowel sounds, nontender, so , no
ctor. T is recurrence y occur u to 48 hours er the initi l masses. Genitourinary: circumcised male, no penile lesions,
event. T us, rescribing edic tions to revent the recurrence no discharge. T e le testicle is tender to palpation and has a
is rudent. Also, the tient should h ve “bee sting” kit v il- normal lay in the scrotum. T e cremasteric ref ex is normal
ble, which should include re- lled syringe or e ine hrine bilaterally.
injection (e.g., E i-Pen or E i-Pen Junior). It is reco ended
th t tients h ve two syringes v il ble t ho e s the ilure Question 1.16.1 What is the signi cance o the normal lay
r te with one injection is irly high. and cremasteric ref ex?
A) T e cre steric re ex should be bnor l in e ididy itis.
T e parents are concerned about this child who likes to play B) T e resence o cre steric re ex e ectively rules out tes-
outside. T ey worry that he will get stung again. ticul r torsion.
C) T e nor l l y o the testicle in the scrotu e ectively rules
Question 1.15.3 You let them know that: out testicul r torsion.
A) Any sting should be tre ted s n e ergency. D) T e resence or bsence o cre steric re ex is not hel ul
B) He will continue to be llergic to “bee stings” in the uture. in ruling out testicul r torsion.
C) He should t ke ro hyl ctic edic tion be ore going out to
l y in the woods or other re s where he ight get stung. Answer 1.16.1 The correct answer is “D.” T e resence or
D) None o the bove. bsence o cre steric re ex (cre steric contr ction with
elev tion o the testis in res onse to stroking o the s e side
Answer 1.15.3 The correct answer is “D.” Here is why. P tients u er thigh) is neither sensitive nor s eci c enough to con r
who re llergic to one s ecies o hy eno ter n re not neces- or rule out the resence o testicul r torsion. Likewise, the l y o
s rily llergic to others. In gener l, the llergy is s ecies s eci c. the testicle c n be nor l in tients with testicul r torsion. An
T us, ost stings will be benign in n llergic tient unless it is bnor l testicul r l y nd the bsence o the cre steric re ex
sting ro the o ending s ecies. “B” is incorrect. M ny chil- y oint tow rd testicul r torsion. However, you c nnot rely
dren tend to “outgrow” “bee sting” llergies. T is is in contr st on these ndings to rule out testicul r torsion.
to dults in who re ctions tend to get worse over ti e. “C” is
incorrect. Obviously the child should be c re ul not to irrit te Question 1.16.2 The LEAST likely diagnosis in this patient
yellow j ckets (did ny o you hurl rocks t w s nests s kid?), is:
but ro hyl ctic tre t ent is not routinely indic ted. A) orsion o testis.
B) E ididy itis.
HELPFUL TIP: C) orsion o endix testis.
Adults with a systemic allergic reaction to an insect D) orsion o endix e ididy is.
sting have a 30% to 60% risk o experiencing another E) esticul r tu or.
systemic reaction upon being stung again. There ore,
adults are more likely to bene it rom venom testing Answer 1.16.2 The correct answer is “E.” esticul r torsion is
and prophylaxis (which can reduce the risk to 5%). All ch r cterized by cute onset o unil ter l testicul r in, o en
patients with a history o anaphylaxis should be pro- during ctivity such s running. It h s bi od l ge distribution,
vided with an anaphylaxis kit. during the rst ye r o li e nd g in during uberty. T e di er-
enti l di gnosis is de endent on the tient’s ge. I the tient is
younger th n 15 ye rs, the di erenti l consists o testicul r tor-
Objectives: Did you learn to . . . sion, e ididy itis, torsion o endix testis or endix e idid-
• Describe the physiology and natural course o bee sting y is, orchitis, hydrocele, nd v ricocele. In tients older th n
reactions? 15 ye rs, the di erenti l includes ll o these di gnoses lus tes-
• Treat a patient with an anaphylactic reaction to a bee sting? ticul r tu or. However, testicul r tu ors re gener lly inless.

Question 1.16.3 What is the most reliable method or diag


CASE 1.16 nosing testicular torsion?
A) Do ler (Du lex color).
A 14-year-old otherwise healthy male presents to the ED B) R dionuclide sc n.
with acute onset le testicular pain when running 1 hour C) Surgic l ex lor tion.
prior to presentation. He denies any trauma to the region. He D) X-r y.
states that his pain is severe and only on the le . T e pain is E) MRI.
increased with ambulation and movement. He has had nau-
sea and vomiting. He denies diarrhea, ever, chills, dysuria, Answer 1.16.3 The correct answer is “C.” Every patient with
hematuria, or penile discharge. suspected testicular torsion should have surgical exploration o
CHAPTER 1 • EMERGENCY MEDICINE 25

the scrotum. All o the other studies re djunctive. For ex - hematuria, ever, chills, vaginal discharge, nausea, vomiting,
le, r dionuclide sc n y result in lse-neg tive nd t kes or diarrhea. Her last menses was 2 weeks ago, and she states
sever l hours to er or . Ultr sound h s sensitivity s low s she is not sexually active. She is on oral contraceptives to treat
82%. Surgic l ex lor tion is the only de nitive di gnostic tool. menstrual cramps and denies any allergies.
T e window o o ortunity or surgery is bout 6 hours, er
which the testicle y not be s lv ged. Orchio exy should be Question 1.17.1 A urine beta HCG is NOT indicated or
er or ed on the involved nd uninvolved sides to revent tor- which o the ollowing patients who presents with abdomi
sion. M nu l detorsion c n be tte ted nd requires sed tion nal pain?
nd in edic tions. wist testis like you re o ening book A) A 32-ye r-old e le who h s h d tub l lig tion.
(the right testis counterclockwise nd the le clockwise). I in B) A 16-ye r-old e le who by history h s never been sexu lly
resolves, you h ve detorsed the testis. ctive.
C) A 25-ye r-old e le who h s h d nor l eriod 1 week
Objectives: Did you learn to . . .

go nd swe rs on st ck o Bibles th t she couldn’t ossibly
Examine a patient presenting with acute scrotal pain?
be regn nt.
• Generate a dif erential diagnosis or scrotal pain based on
D) A 24-ye r-old, rried, ro ession l e le who is t king
the patient’s age?
or l contr ce tives nd h d nor l l st enses.
• Evaluate a patient with suspected testicular torsion?
E) A 25-ye r-old le.

QUICK QUIZ: UROLOGIC INFECTION 1 Answer 1.17.1 The correct answer is “E.” O course les do
not need regn ncy test ( lthough the HCG y be elev ted
in testicul r c ncer). All e le tients o re roductive ge,
Wh t is the ost co on gent c using e ididy itis in
exce t or those who h ve h d hysterecto y, ust h ve reg-
21-ye r-old le?
n ncy test s rt o the ev lu tion o bdo in l in. T ere re
A) Escherichia coli.
sever l re sons or this osition. First, ny tients y not be
B) Neisseria gonorrhoeae.
c ndid bout their sexu l ctivity. In ct, in one study, l ost
C) Chlamydia trachomatis.
one-third o tients who s id “they could not ossibly be reg-
D) Pseudomonas s ecies.
n nt,” including one who denied ever h ving intercourse, were
E) Ureaplasma urealyticum.
regn nt. Second, the ilure r te o tub l lig tion is u to 5%
over 10 ye rs de ending on the technique used (l rosco ic
The correct answer is “C.” In young les, e ididy itis is
tub l lig tion is the le st reli ble). o r ise your concern little
usu lly the result o sexu lly tr ns itted dise ses. O these,
higher, l ost ll o the regn ncies in tients who h ve h d
C. trachomatis is currently the ost co on etiologic gent.
tub l lig tion re ecto ic.
N. gonorrhoeae is second ost co on in this ge grou . It is
there ore essenti l to tre t or both gents when the di gnosis o
e ididy itis is sus ected.
HELPFUL TIP:
When examining a patient whose history is consistent
QUICK QUIZ: UROLOGIC INFECTION 2 with vulvovaginitis, remember that a KOH preparation
is only 65% to 80% sensitive or Candida and treatment
Wh t is the ost co on gent c using e ididy itis in based on symptoms and physical indings is certainly
55-ye r-old le? reasonable.
A) E. coli.
B) N. gonorrhoeae.
C) C. trachomatis.
You get a urinalysis (UA) on this patient, mostly out o habit.
D) Pseudomonas s ecies.
T e UA shows 5 to 10 WBCs/HPF, 2 + bacteria, 2 + leukocyte
E) U. urealyticum.
esterase, and 1 + nitrite.
The correct answer is “A.” Gr -neg tive rods re the ost
Question 1.17.2 Which o the ollowing antibiotic regimens
co on c use o e ididy itis in older en. O these, E. coli
IS NOT indicated or the treatment o simple cystitis?
is the ost co on etiologic gent, ollowed by Klebsiella nd
A) 3-d y course o tri etho ri –sul ethox zole ( MP–SMX).
Pseudomonas s ecies.
B) 3-d y course o uoroquinolone.
C) 5-d y course o nitro ur ntoin.
D) Single dose o os o ycin.
CASE 1.17 E) Single dose o ce h lexin.
A 22-year-old otherwise healthy emale college student pres-
ents to the ED with dysuria and urinary requency o 2 days Answer 1.17.2 The correct answer is “E.” T e usu l c us tive
duration. She denies any abdominal/pelvic pain, f ank pain, gents or unco lic ted cystitis re Gr -neg tive org nis s
26 FAMILY MEDICINE EXAMINATION & BOARD REVIEW

such s E. coli. In re s th t h ve high r tes o resist nce to Question 1.18.1 What is the most likely diagnosis in this
MP–SMX (>30% or ore o isol ted E. coli b cteri resist nt), patient?
it is wise to use quinolone s the rst-line gent; however, qui- A) Pyelone hritis.
nolones re ore costly nd quinolone resist nce is rising. All B) Perirect l bscess.
o the bove regi ens re usu lly e ective or tre ting cystitis C) E ididy itis.
exce t single dose o ce h lexin. Ce h lexin is e ectively used D) Acute rost titis.
in regn nt e les, lthough 7-d y course is reco ended. E) Cystitis.
Fos o ycin h s lower cure r te th n the other regi ens nd
is ore ex ensive. Answer 1.18.1 The correct answer is “D.” T is tient’s sy -
to s ost closely t those o so eone with cute rost titis.
Although his UA is lso consistent with yelone hritis or cysti-
HELPFUL TIP: tis, his ex in tion ndings re ore suggestive o cute ros-
A alse-negative urinalysis is common in women with t titis. He l cks costovertebr l ngle tenderness ( yelone hri-
uncomplicated cystitis. Empiric treatment o urinary tis), nd he h s signi c nt te er ture th t rgues g inst
tract in ection (UTI) is reasonable in a emale o child- si le cystitis. In the st, rost tic ss ge w s reco ended
bearing years presenting with one or more typical when obt ining urine s eci en; but this r ctice is to be
symptoms (urgency, requency, dysuria) and no vaginal voided since it is quite in ul nd b cteri l seeding into the
discharge. bloodstre y occur. In the bsence o scrot l tenderness,
e ididy itis is lso quite unlikely.

Question 1.17.3 All o the ollowing patients with pyelone Question 1.18.2 What should be included in the treatment
phritis should be admitted EXCEPT: regimen or this patient?
A) A 22-ye r-old G1 P0 e le <24 weeks o gest tion, he o- A) Or l uoroquinolone or MP–SMX or t le st 3 weeks.
dyn ic lly st ble. B) Instructions or hydr tion, sitz b ths, stool so eners, nd
B) A 22-ye r-old e le un ble to toler te PO uids or edic tions. nonsteroid l nti-in tory drugs (NSAIDs).
C) A 22-ye r-old e le with unreli ble soci l situ tion nd/or C) Ad ission or IV ntibiotics i he e rs toxic or he ody-
co li nce. n ic lly unst ble.
D) A 22-ye r-old e le with n uncle r di gnosis or extre e D) Foley or su r ubic c theter i urin ry retention is rob-
in. le .
E) All o the bove.
Answer 1.17.3 The correct answer is “A.” T e old d ge th t ll
regn nt tients with yelone hritis ust be d itted h s gone Answer 1.18.2 The correct answer is “E.” P tients with cute
out o vor. It is s e to send tients ho e who re <24 weeks rost titis should be tre ted or t le st 3 weeks with or l ntibi-
o gest tion, co li nt, h ve st ble vit l signs, nd re cces- otics to revent chronic rost titis; so e suggest u to 42 d ys.
sible by tele hone. P tients should be given cle r instructions to re t ent should be initi ted with quinolone while urine cul-
return or ny co lic tions. All o the other situ tions require tures re ending, since sul resist nce is high in so e re s o
in-hos it l c re. the country.
Objectives: Did you learn to . . .
• Decide which patients should have a urine beta-HCG in the ED? While this patient is still in the ED, he develops acute uri-
• Provide appropriate antibiotic treatment to a patient with an nary retention. A Foley catheter is placed without di culty
uncomplicated UTI? and 300 cc o slightly cloudy urine is obtained. Your patient
• Identi y patients with pyelonephritis who require hospital eels much better and thanks you or alleviating his pain.
admission? You decide to discharge him home with the Foley catheter
and a leg bag a er discussion with a urologist and ollow-up
arrangement.
CASE 1.18
Question 1.18.3 Which o the ollowing cause urinary reten
A 63-year-old male presents to the ED with a 2-day history o tion in men?
ever, urinary requency, dysuria, and di culty initiating the A) Phi osis, urethr l stricture, benign rost tic hy er l si
urinary stream. He also relates having some perineal pain. (BPH), c lculi.
On examination, his vitals are stable except or a temperature B) Anticholinergics, sy tho i etics, n rcotics, nti sychotics.
o 38.5°C. His rectal examination is remarkable or a tender, C) Psychogenic.
warm, edematous prostate. T ere are no perirectal masses D) C ud equin syndro e, di betes, s in l cord injuries.
and the stool is heme negative. He has no penile lesions, dis- E) All o the bove.
charge, scrotal masses, or tenderness. He does not exhibit
any costovertebral angle tenderness. His UA is positive or Answer 1.18.3 The correct answer is “E.” All o the bove c n
10 WBCs/HPF, 1+ nitrite, 1+ leukocyte esterase. c use urin ry retention in en. By r, the ost co on c use
CHAPTER 1 • EMERGENCY MEDICINE 27

o cute urin ry retention is BPH. T e c tegories o cute urin ry uncircu cised les by the ge o 3 to 5 ye rs. Be ore this, no
retention y be divided into neurogenic (s in l cord injuries, ction need be t ken.
c ud equin syndro e, di betes, syringo yeli , etc.), obstruc-
tive (BPH, hi osis, r hi osis, c lculi, urethr l stricture, CASE 1.19
etc.), h r cologic ( nticholinergics, ntihist ines, n rcot-
ics, nti sychotics, tricyclics, etc.), nd sychogenic, which is A 20-year-old emale presents to your ED complaining o
di gnosis o exclusion. lower quadrant abdominal pain. She is on “the ring” or con-
traception and has been aith ully using it. She has had regu-
HELPFUL TIP: lar menses and has not noticed any change in her pattern o
Sending patients home on an alpha-blocker (e.g., doxa- menses. Her pain had a sudden onset but is not associated
zosin, tamsulosin) may reduce the need or re-catheter- with any vaginal bleeding. On vaginal examination, you nd
ization a ter the catheter is removed. marked cervical motion tenderness but no palpable adnexal
mass.

Objectives: Did you learn to . . . Question 1.19.1 Based on this in ormation you decide that:
• Recognize the clinical presentation o prostatitis? A) T e bsence o n dnex l ss e ectively rules out ecto ic
• Treat a patient with prostatitis? regn ncy.
• Identi y causes o urinary retention in a male? B) I tient beco es regn nt, ll or s o contr ce tion
reduce the risk o ecto ic regn ncy.
C) T e ct the tient h s h d nor l eriods e ectively rules
QUICK QUIZ: FORBIDDEN FORESKIN out n ecto ic regn ncy.
D) Cervic l otion tenderness e ectively clinches the di gno-
Which o the ollowing is ch r cterized by swollen, in ul sis o elvic in tory dise se.
oreskin th t c nnot be reduced b ck to its nor l osition? E) None o the bove is true.
A) Phi osis.
B) P r hi osis. Answer 1.19.1 The answer is “E,” none o the above. “A” is
C) B l no osthitis. incorrect bec use only 10% o tients with n ecto ic reg-
D) Me t l stenosis. n ncy will h ve l ble ss in the dnex . “B” is incorrect
bec use both intr uterine devices nd tub l lig tion increase
The correct answer is “B.” P r hi osis is condition in which the risk o ecto ic regn ncy i the tient beco es regn nt.
the oreskin is retr cted, swollen, nd un ble to reduce into its “C” is incorrect bec use 15% to 20% o tients with ecto ic
nor l osition. Ice nd ste dy nu l co ression o en er- regn ncy h ve no history o issed enses. “D” is incorrect
its reduction. Surgery is indic ted i nu l reduction ils. “A,” bec use cervic l otion tenderness c n be resent not only in
hi osis, is condition in which the dist l oreskin is too tight elvic in tory dise se but lso in other illnesses such s
to be retr cted to llow ex osure o the gl ns. It is o en con used ov ri n torsion, ecto ic regn ncy, etc.
with enile dhesions in those younger th n 2 ye rs. “C,” b l no-
osthitis, is or o cellulitis involving the oreskin nd gl ns in Question 1.19.2 Risk actors or ectopic pregnancy include
the uncircu cised le ssoci ted with oor hygiene. re t ent all o the ollowing EXCEPT:
is with w r so ks, ntibiotics, nd ossible circu cision. “D,” A) Prior ecto ic regn ncy.
e t l stenosis, is co on in circumcised les, ssoci ted with B) Or l contr ce tive use.
n in tory re ction involving the e tus. Sy to s th t C) History o elvic in tory dise se.
indic te the need or surgic l tre t ent include s r ying o the D) re t ent or in ertility.
urine stre or dors l de ection o the stre . E) Current intr uterine device use.

Answer 1.19.2 The correct answer is “B.” All o the others


QUICK QUIZ: FORESAKEN FORESKIN incre se the risk o n ecto ic regn ncy. Other risk ctors
include cig rette s oking, recent elective bortion, revious
Until wh t ge is it nor l to h ve dhesions between the gl ns tub l surgery, nd tub l lig tion.
nd oreskin in uncircu cised les?
A) Adhesions re lw ys bnor l. You decide that this patient may have an ectopic pregnancy.
B) Age 6 onths. A urine HCG test is positive or pregnancy.
C) Age 1 ye r.
D) Age 2 ye rs. Question 1.19.3 The signi cance o a positive pregnancy
E) Age 3 ye rs. test is that:
A) An ultr sound will be ble to detect n ecto ic regn ncy i
The correct answer is “E.” So e dhesions re nor l in young one is resent.
children. However, the oreskin should be ully retr ct ble in B) T e seru level o HCG is at least 1,000 IU/ L.
28 FAMILY MEDICINE EXAMINATION & BOARD REVIEW

C) Co bined with the tient’s bdo in l in nd cervic l Objectives: Did you learn to . . .
otion tenderness, it e ectively rules in n ecto ic regn ncy. • Evaluate a ertile emale with pelvic pain?
D) T e urine HCG is 98% sensitive or regn ncy 7 d ys er • Diagnose an ectopic pregnancy?
i l nt tion.

Answer 1.19.3 The correct answer is “D.” “A” is incorrect. T e QUICK QUIZ: TWISTED SISTER
regn ncy test is ositive very e rly nd ultr sound y not be
ositive by n ex erienced o er tor until 6 weeks o regn ncy. Which o the ollowing is ty ic l o ov ri n torsion?
“B” is incorrect. T e urine y be ositive t seru HCG lev- A) Periu bilic l in gr du lly igr ting to both the right nd
els o 25 to 50 IU/L. P tients y not h ve n HCG level o le qu dr nts.
1,000 IU/L until 6 weeks o regn ncy. “C” is incorrect bec use B) Sudden onset o colicky bdo in l in with v gin l bleed-
tients with nor l regn ncy y lso h ve bdo in l ing.
in nd cervic l otion tenderness. C) Sudden onset o colicky bdo in l in in one o the lower
qu dr nts.
T e patient’s serum HCG is 440 IU/L. You order an ultra- D) Sudden low b ck in with r di tion to the erineu .
sound and nd no evidence o an intrauterine or ectopic
pregnancy. The correct answer is “C.” P tients with ov ri n torsion resent
with sudden onset o severe lower bdo in l in. T e in is
Question 1.19.4 Your next step is to: requently colicky. Since only one ov ry is involved, the in
A) Re ssure the tient th t she does not h ve n ecto ic reg- is loc ted in one side or the other. S ont neous torsion/detor-
n ncy. sion y lso occur so th t the in y re it s ont neously.
B) Recheck the HCG in 48 hours. Ov ri n torsion c n be di gnosed by Do ler ultr sound th t
C) Re er or l rosco y to rule out ecto ic regn ncy. ex ines ow to the ov ries. M ny gynecologists consider this
D) Recheck n HCG in 1 to 2 weeks. clinic l di gnosis, so obt in consult tion e rly or sus ected
E) Follow the tient clinic lly. ov ri n torsion c ses.

Answer 1.19.4 The correct answer is “B.” T e HCG should CASE 1.20
double in nor l regn ncy every 1.8 to 3 d ys. I the HCG is
not doubling in this ti e r e, it is likely n ecto ic regn ncy. A middle-aged unresponsive, disheveled patient is brought
Re e ber, the ct th t you did not see n ecto ic regn ncy by emergency medical services (EMS) to your ED. T ey had
on ultr sound is irrelev nt. T e seru HCG is gener lly at least been called by his girl riend who had seen him lying in the
1,500 IU/L be ore nything is seen on ultr sound. By n HCG o grass outside his home this morning. He has spontaneous
6,500 IU/L, n ex erienced ultr sonogr her should cert inly respirations and has shallow respirations o 20 per minute
be ble to see regn ncy on ultr sound; i not seen, n ecto ic and a weak but palpable pulse at 110 beats per minute.
should be reconsidered. “A” is incorrect bec use o the bove.
“C” is incorrect. T is is inv sive nd not needed. “D” is incorrect Question 1.20.1 What should be your rst steps in assess
bec use o the ti e r e; the HCG should be rechecked in 24 ment and treatment?
to 48 hours. An ecto ic y well ru ture within 1 to 2 weeks. A) Oxygen by nonrebre ther sk (NRB), st t seru glucose,
“E” is incorrect. I you ollow the tient clinic lly, you re b si- n loxone, nd thi ine.
c lly s ying th t you will w it until the ecto ic ru tures be ore B) Oxygen by NRB, ECG, he d C .
ddressing the roble . C) Oxygen by NRB, intub te, ECG, he d C .
D) Intub te, ECG, he d C .
Question 1.19.5 You re check the HCG in 48 hours and it is
now 1,000 IU/L (prior level 440 IU/L). Your interpretation is Answer 1.20.1 The correct answer is “A.” T ere re sever l
that: c uses o unres onsiveness th t c n be i edi tely corrected.
A) T is tient does not likely h ve n ecto ic regn ncy. A hel ul lgorith to rec ll in the initi l tre t ent or n unre-
B) T is tient h s ol r regn ncy. s onsive tient is “DON’ ”: Dextrose, Oxygen, N loxone,
C) T is tient h s blighted ovu . hi ine (the so-c lled “co cockt il”). Answer “A” is correct
D) T e tient h s et l de ise o n intr uterine regn ncy. bec use n loxone nd oxygen re d inistered nd glucose is
E) All o the bove re ossible. checked. I r id blood sug r is un v il ble, e iric l d in-
istr tion o dextrose would be ro ri te. R id tre t ent
Answer 1.19.5 The correct answer is “A.” Since the HCG dou- o hy oxi , hy oglyce i , nd n rcotic overdose c n i rove
bled s ex ected in nor l regn ncy, it is not likely th t this ent l st tus nd thus void intub tion. ECG nd he d C y
is n ecto ic regn ncy, blighted ovu (“C”) or intr uterine be indic ted l ter in the ev lu tion. You could rgue or intub -
et l de ise (“D”). In ll o these conditions, the HCG would tion in this tient since he is unres onsive (“C” nd “D”) but
not double. “B” is lso not likely bec use in ol r regn ncy, the next ste s would not be ECG nd he d C . So “A” is the ost
the HCG would rise dr tic lly correct nswer.
CHAPTER 1 • EMERGENCY MEDICINE 29

incorrect; the tient still h s GCS o less th n 8 nd is un ble


T e patient is ound to be hypothermic, hypoglycemic, and to rotect his irw y so he should not be extub ted.
hypoxic. He is placed on oxygen and given warm normal saline,
an amp o D50W, and naloxone. T e patient now is saturating Objectives: Did you learn to . . .
at 98%on NRB. He is responding to pain ul stimuli by moan- • Rapidly assess and treat an unresponsive patient?
ing and withdrawing his extremities but is not opening his • Use the GCS to determine need or intubation?
eyes. He has gurgling respirations and still has no gag ref ex.

Question 1.20.2 What is your next step? CASE 1.21


A) Intub te. You are working in a rural ED and get a call that the volun-
B) Obt in he d C . teer ambulance service is bringing an unresponsive, adult
C) Obt in ECG. male patient status post motor vehicle collision. T ey bring
D) Continue on NRB. the patient on a backboard with a c-collar.
E) Obt in n ABG.
Question 1.21.1 The primary survey o a trauma patient
Answer 1.20.2 The correct answer is “A.” Although the tient includes all o the ollowing EXCEPT:
h s i roved nd h s nor l oxygen s tur tion, his level o A) Check or ulses.
consciousness is still too low to rotect his irw y. T us, he B) I obilize the c-s ine, ev lu te the irw y, nd listen or
should be intub ted be ore urther di gnostic studies re er- bre th sounds.
or ed. A si le ethod to deter ine the need or intub tion C) GCS.
is the GCS ( ble 1-5). P tients with GCS o 8 or less should be D) Abdo in l ex in tion.
intub ted, s they c nnot rotect their irw y ro s ir tion o E) Unclothe the tient.
or l secretions nd/or e esis. T e rhy e “GCS o 8, intub te”
ssists in recollection o this rule. T is tient h s GCS o 7 Answer 1.21.1 The correct answer (and what you do
(eyes 1, verb l 2, ove ent 4). He h s gurgling res ir tions, not want to do in the primary survey) is “D.” T e ri ry
nd you c nnot ensure he is ble to rotect his irw y. T ere- survey is the initi l ev lu tion er or ed on every tr u
ore he should be intub ted be ore other studies or interven- tient by the lgorith “ABCDE.” A: Airw y ssess ent
tions. “D” is incorrect bec use he c nnot rotect his irw y. “E” includes c-s ine i obiliz tion; o ening the irw y by j w
is incorrect because the decision to intubate is a clinical one thrust/chin li ; nd, when indic ted, b g-v lve sk, intub -
and not tied directly to the blood gas! tion, or cricothyroto y. B: Bre thing includes listening or
bre th sounds, d inistering oxygen, nd tre ting neu o-
T e girl riend arrives and gives urther history that the thor ces. C: Circul tion requires ssess ent o BP, checking
patient is an alcoholic and had told her he had quit drinking ulses nd tre t ent o hy otension nd t chyc rdi with
2 days ago. She states he has had a seizure in the past when he cryst lloids nd blood. D: Dis bility is the r id neurologic
stops drinking. He starts to seize be ore your eyes. ex in tion or otenti l cord injury nd GCS. E: Ex osure
involves disrobing the tient nd rolling the to ssess ny
Question 1.20.3 What should you do now? injury to the b ck.
A) Give lor ze nd d it or rob ble deliriu tre ens
(D s). On the primary survey, the patient was not protecting his
B) Give lor ze , obt in he d C , blood cultures, nd ECG. airway and was intubated with an 8-mm endotracheal tube
C) Give lor ze , extub te, nd d it or rob ble D s. (E ) with rapid sequence intubation (RSI). T e patient
D) Give henytoin nd d it or rob ble D s. is noted to have breath sounds on the right but no breath
sounds on the le .
Answer 1.20.3 The correct answer is “B,” give lorazepam to
abort the seizure. Even though it is e sy to ssu e th t the Question 1.21.2 What is the next best step in evaluation
tient h d seizure ro D s, which y h ve resulted in and treatment o this patient?
hy oxi , hy other i , nd hy oglyce i , this kind o think- A) Re ove the E ; you ust be in the eso h gus.
ing c n le d to errors. It is still ossible th t the tient h s B) Get chest x-r y to con r tube l ce ent.
s ont neous or tr u tic br in he orrh ge, thus the need or C) Do needle deco ression o le chest.
he d C . It is lso ossible th t the tient is se tic; re e - D) Insert le chest tube.
ber th t hy other i c n be seen with se sis. T us, blood cul- E) Check E or de th t the teeth nd osition.
tures should be obt ined nd ossibly n LP er or ed. Fin lly,
n ECG c n show yoc rdi l in rction or rrhyth i th t Answer 1.21.2 The correct answer is “E.” T is tient h s
y lso result in seizure. “D” is incorrect bec use henytoin bre th sounds on the right; there ore, eso h ge l intub tion is
is not the drug o choice or n ctively seizing tient; ben- unlikely, king “A” incorrect. T e ost likely nd e sily rec-
zodi ze ine such s lor ze should be d inistered. And, ogniz ble source o bsent bre th sounds on the le is right
henytoin does not work in lcohol withdr w l seizures. “C” is in ste bronchus intub tion. T us, looking t the de th o
30 FAMILY MEDICINE EXAMINATION & BOARD REVIEW

l ce ent o the E t the teeth (“E”) is the initi l ev lu tion


indic ted. T e E should be l ced t bout three ti es the T e patient now has a chest tube in place but remains hypo-
size o the E (i.e., 3 × 8 = 24 c ) ssu ing th t the size o tensive. wo large bore (18 gauge or larger) IVs were estab-
E w s correctly chosen. T is is n i ort nt c lcul tion to lished. No external source o bleeding is identi ed.
re e ber, s it lso lies to edi tric tients. A chest x-r y
c n lso ev lu te or right in ste intub tion but should not Question 1.21.5 When should blood be administered?
be the rst ste . A neu othor x y be the c use o unil ter l A) I edi tely.
bre th sounds, but right, or less co only, le , in ste intu- B) I tient is hy otensive.
b tion should be considered rst. C) I ersistent hy otension er 1 L o nor l s line.
D) I ersistent hy otension er 4 L o nor l s line.
Question 1.21.3 You now note that there is an open chest E) I FAS ex in tion shows intr bdo in l ree uid.
wound to the le t lateral rib cage. Funny . . . you didn’t
notice that be ore. What is the initial treatment o this new Answer 1.21.5 The correct answer is “C.” I tient rrives
nding? hy otensive with no signs o extern l bleeding, 1 L o cryst lloid
A) Needle thor costo y. (nor l s line) should be given i edi tely. T e tr dition l
B) Chest tube l ce ent. te ching h s been to st rt blood er hy otension unres onsive
C) Occlusive dressing. to 2 liters o s line. T is h s ch nged in the ost recent A LS
D) Chest x-r y. guidelines; we don’t know which will be the correct nswer
on the test. I the tient continues to be hy otensive, cked
Answer 1.21.3 The correct answer is “C.” T is tient h s n red blood cells should be st rted long with ddition l nor l
o en “sucking” chest wound. E ch ti e the tient ins ires, ir s line. T e FAS ex in tion is r id bedside ultr sound to
c n be sucked into the chest c vity cting s one-w y v lve. identi y ree uid in the tr u tient’s bdo en nd eric r-
T is c n result in tension neu othor x. T us, the initi l di l s c. Persistent hy otension with ositive FAS ex in tion
tre t ent is to ly n occlusive dressing to the wound (e.g., is n indic tion or e ergent ex lor tory l roto y.
such s etrol tu g uze). Objectives: Did you learn to . . .
• Employ the primary assessment o a trauma patient?
Following the placement o the occlusive dressing the patient • Treat an open chest wound?
continues to have absence o breath sounds on the le , is now • Resuscitate an unstable trauma patient?
hypotensive, and has distended neck veins. T e presumed
diagnosis is a tension pneumothorax.
HELPFUL TIP(S):
Question 1.21.4 What should you do now? Tranexamic acid can be used as a hemostatic agent in
A) Chest x-r y. hypotensive trauma patients. I you are going to trans-
B) Chest tube l ce ent through wound. use a large amount o blood the best evidence sug-
C) Chest tube l ce ent through se r te site. gests blood:FFP:platelets in a 1:1:1 ratio.
D) Re ove occlusive dressing.

Answer 1.21.4 The correct answer is “D.” T e occlusive dress- QUICK QUIZ: CHEST PAIN
ing itsel y c use tension neu othor x, so its l ce ent
should be i edi tely ollowed by chest tube l ce ent. I A 54-ye r-old e le resents to your ED with chie co l int o
tension neu othor x develo s be ore the tube is l ced, chest in. She st tes it c e on suddenly while she w s trying to rid
re oving the dressing (“D”) c n usu lly llevi te the tension her g rden o l gue o thistles (or g rlic ust rd–your choice).
co onent. E ergency edic l technici ns will o en l ce She describes it s “sh r ” nd it r di tes through to her b ck. She
dressing th t is closed only on three sides to serve s rele se re orts di culty bre thing. Her st edic l history is ertinent or
v lve nd void this ossibility. T e di gnosis o tension neu- hy ertension, bre st c ncer, nd obesity. She is s oker.
othor x is clinic l. T e ti e required to obt in chest x-r y B sed on this history, wh t di gnosis c n be excluded ro
y result in the de th o the tient. When l cing chest tube your di erenti l?
in tient with n o en wound, never pass the tube through A) Acute yoc rdi l in rction.
the wound, s it is likely to ollow the th o the initi l enetr - B) Aortic dissection.
tion into lung renchy . C) Pul on ry e bolis .
D) Pneu othor x.
E) None o the bove.
HELPFUL TIP:
Needle thoracostomy is also use ul in a tension The correct answer is “E.” T e tient’s history is ost sugges-
pneumothorax and should be the initial step in most tive o ul on ry e bolis with her co l int o sh r chest
cases. in, trouble bre thing, c ncer history, nd s oking. However,
t this oint in ti e, ll o the etiologies listed— nd ore—
CHAPTER 1 • EMERGENCY MEDICINE 31

ust be considered. Wo en o en h ve ty ic l resent tions Answer 1.22.1 The correct answer is “C.” M ny children who
o c rdi c chest in. In ddition, tients o en use the ter re vo iting nd h ve di rrhe will be ble to toler te s ll
“sh r ” to describe “intense” or “strong” in. (5 cc) si s o uid d inistered every ew inutes. Or l u-
ids should be tte ted rior to IV ther y. One ight lso
d inister ond nsetron to hel reduce vo iting (2 g in those
QUICK QUIZ: AORTIC DISSECTION <15 kg, 4 g in those >15 kg). In children who re severely
dehydr ted, s evidenced by ltered ent l st tus or ch nge in
In ortic dissection, the BP is di erent between the extre ities skin turgor, IV uid resuscit tion should begin i edi tely. I
in less th n 30% o c ses. I you diagnose thoracic aortic dis- IV rehydr tion is considered ro ri te, use nor l s line in
section, and the patient’s BP is di erent in the upper extremi- 20 cc/kg liquots. T ere ore, nswers “A” nd “B” re not cor-
ties, what limb should you use to guide BP management? rect. Pro eth zine (“D”) is n ntie etic th t h s been used
A) Right r . in children in the st. However, ro eth zine h s received
B) Le r . “bl ck box” w rning ro the FDA or children younger th n
C) Either lower extre ity. 2 ye rs, s there is risk o res ir tory de ression.
D) T e li b with the highest BP.
E) T e li b with the lowest BP. You obtain some lab work and notice that the child has nor-
mal renal unction but low serum bicarbonate, indicating a
The correct answer is “D.” An ortic dissection yi ir the possible metabolic acidosis. In speaking with his mother, you
blood ow to cert in extre ities due to the lse lu en. T e discover that earlier in the day, he was playing unsupervised
BP should be int ined t systolic BP o 100 to 120 Hg in the bathroom, where she keeps her prenatal vitamin. Upon
(or bit lower) in the extre ity with the highest BP. T is will questioning the child, he states that he ate a bunch o “candy”
decre se the orces ro g ting the dissection. in the bathroom about 3 hours ago (all the more reason to
keep things up high and away rom children!).
HELPFUL TIP:
Patients with an aortic dissection should be started Question 1.22.2 What component o prenatal vitamins is
on a beta-blocker (e.g., esmolol) and vasodilator drip most concerning or toxicity?
(nitroglycerin, nitroprusside) to control BP and mini- A) Folic cid.
mize stress to the aortic wall. The goal is a pulse o B) Iron.
approximately 60 bpm and a blood pressure o 100 C) C lciu .
to 120 mm Hg systolic. Remember that lowering the D) Vit in D.
pulse rate is just as important as reducing the blood
pressure. Answer 1.22.2 The correct answer is “B.” Folic cid, c lciu ,
nd vit in D re ll toler ted well in high doses, s their
bsor tion ro the GI tr ct is li ited. Iron, however, c n con-
tinue to be bsorbed while it re ins in the GI tr ct. Iron is
CASE 1.22 direct irrit nt to the GI ucos (there ore, the bloody e esis
nd di rrhe ) nd inter eres with the electron tr ns ort ch in
A mother brings her 3-year-old child into the ED. She states nd erobic et bolis .
that the child has been vomiting and complaining o abdom-
inal pain all a ernoon. He has had between 8 and 10 epi- Question 1.22.3 The nurse asks i you should add on an iron
sodes o emesis; the last two have contained small amounts level to the blood that was drawn 3 hours a ter the inges
o bright red blood. He has had a little nonbloody diarrhea. tion. You respond:
He has not been tolerating f uids. On examination, you A) “No th nks. Iron levels re not hel ul.”
nd the child to be moderately ill appearing with normal B) “No th nks. It’s too e rly. We need to w it until t le st
color, but he seems less interactive than you would expect. 12 hours h ve el sed.”
His vitals reveal a temperature o 36.5 degrees C, a pulse C) “Yes, le se. I it’s nor l, we don’t need ny urther tre t ent.”
o 170 bpm, a respiratory rate o 28, and a BP o 98/58 mm D) “Yes, le se. It y hel us deter ine the severity o toxicity.”
Hg. His abdomen is slightly and di usely tender. He has dry
mucous membranes. Answer 1.22.3 The correct answer is “D.” T e iron level
between 2 nd 4 hours er ingestion is the ost ccur te;
Question 1.22.1 In general (not speci cally in this patient) beyond this eriod, the jority o the iron is oving intr cel-
what is the initial treatment o a moderately dehydrated lul rly nd c nnot be e sured. For slow-rele se iron, seru
child? concentr tions should be e sured t 6 to 8 hours er inges-
A) 20 cc/kg bolus o D5 1/2 NS. tion. T ese e sures will give you e k seru iron concentr -
B) 10 cc/kg bolus o isotonic cryst lloid uid. tion th t correl tes well with the severity o toxicity. However, a
C) Or l ch llenge o s ll ount o electrolyte solution. low serum level o iron does not mean the symptomatic patient is
D) 12.5 g ro eth zine su ository. OK. Treatment is based on clinical ndings and NOT on serum
32 FAMILY MEDICINE EXAMINATION & BOARD REVIEW

iron levels. Once the iron oves into the eri hery, the seru TABLE 1-7 MANIFESTATIONS OF IRON TOXICITY
levels c n be low des ite signi c nt toxicity.
First (or early) Hours 0–6 Vomiting and diarrhea, o ten
phase (rarely > 6 bloody
Question 1.22.4 How do patients with an iron overdose hours) Metabolic acidosis
present?
Shock
A) Abdo in l in, vo iting, nd di rrhe .
B) He te esis, shock, nd co . Second (or 3–48 hours Resolving acidosis
C) Rel tively sy to tic. quiescent) (time variable) Resolving hypovolemia
phase
D) All o the bove. Frequently, asymptomatic

Third phase 12–48 hours GI hemorrhage


Answer 1.22.4 The correct answer is “D.” P tients who h ve (time variable) Lethargy, coma, shock
h d n iron overdose cl ssic lly ss through ive di erent
Cardiovascular collapse
h ses. he irst h se is ch r cterized by n use , vo iting,
Metabolic acidosis
di rrhe , nd bdo in l in. here y be he te esis
Renal ailure (variable)
nd he tochezi s the GI ucos beco es irrit ted. he
second h se is rel tively sy to tic eriod s the GI Fourth phase 2 days or Hepatotoxicity
sy to s resolve. During this quiet h se, iron is bsorbed more Hepatic necrosis
nd tr ns orted to the eri hery where it c uses the inter- Coma
ru tion o erobic et bolis . In the next (third) h se,
Fi th phase 2–4 weeks GI obstruction due to strictures
tients beco e hy otensive, cidotic, nd c n develo and scarring
ultisyste org n ilure nd co . It is this shock th t is
the usu l c use o de th in iron toxicity. he ourth h se
is her lded by he tic necrosis. Liver ilure, which does
not occur in ll tients, is the second ost requent c use HELPFUL TIP:
o de th in c ses o iron toxicity. Fin lly, the tient y Patients who are entirely asymptomatic 6 hours a ter
develo bowel obstructions 2 to 4 weeks or longer ter the iron ingestion and do not have any radiographic evi-
ingestion due to stricture or tion t the site o ucos l dence o iron in the GI tract are not at risk or toxicity.
irrit tion. See ble 1-7. They can be sa ely discharged with close ollow-up.
The caveat is that chewable multivitamins are not radi-
Question 1.22.5 Abdominal lms reveal radiopaque pills opaque and will not show up on x-ray.
in the stomach. What is the best next step in treatment or
this patient?
A) Whole bowel irrig tion with olyethylene glycol solution.
B) G stric l v ge. Question 1.22.6 The patient is symptomatic (vomiting and
C) Activ ted ch rco l. diarrhea), acidotic, and also has an iron level 650 µg/dL,
D) Syru o i ec c. which puts him at signi cant risk or toxicity. What is your
next step?
Answer 1.22.5 The correct answer is “A.” G stric l v ge nd A) Correction o cid–b se disturb nce nd ggressive uid
vo iting induced by syru o i ec c both ent il ir ount resuscit tion.
o risk nd neither h s been shown to be bene ci l. In ddi- B) ED A.
tion, there is the risk o s ir tion nd subsequent neu oni- C) De erox ine.
tis. T ere ore, “B” nd “D” re not correct. Iron, lithiu , nd D) A nd B.
le d will not dsorb to ctiv ted ch rco l; there ore, it is o E) A nd C.
no bene t in such c ses, nd “C” is incorrect. re t ent or
iron toxicity involves whole bowel irrig tion with olyethyl- Answer 1.22.6 The correct answer is “E.” In ddition to sy -
ene glycol solution to ush the iron out o the GI tr ct. T ere to s nd cidosis, n iron level >500 µg/dL or ingestion o
re v rious doses nd r tes o d inistr tion ublished, but ore th n 60 g/kg o ele ent l iron re considered high-risk
10 to 15 L/kg/hr, u to 2,000 cc/hr, see s to be re son ble situ tions, nd chel tion with de erox ine is w rr nted. De -
l ce to st rt. T is requires the l ce ent o n sog stric erox ine is used to chel te iron, while ED A is chel tion
tube. I tient does not toler te the volu e o the in usion, tre t ent or le d oisoning. F stidious su ortive c re, with
the r te should be decre sed by 50%. T e irrig tion should correction o the tient’s volu e nd cid–b se disturb nces,
continue until the rect l ef uent is cle r nd there re no is i er tive. Ensuring th t the tient is euvole ic is es e-
visible ill r g ents. I ollow-u r diogr hs de onstr te ci lly i ort nt when using chel tion ther y, given th t the
ersistent iron t blets in the sto ch, consider the ossibility jor side e ect o de erox ine is hy otension. T e dose o
o bezo r h ving or ed, which y require endosco ic or de erox ine is 15 g/kg/hr or 24 hours, but y be slowed
surgic l intervention or re ov l. down i the tient beco es hy otensive.
CHAPTER 1 • EMERGENCY MEDICINE 33

colonizes the genit l tr ct o nor l he lthy wo en. GBS y


HELPFUL TIP:
be the ost co on c use o b cteri l in ection in the newborn.
Dialysis does not remove iron rom the blood stream
T e e k incidence o GBS dise se is in the rst 7 d ys o li e, but
nor the intracellular space, where the majority o it will
there y be del yed resent tion out to 30 d ys. Pseudomonas
be ound. Dialysis may be indicated to treat renal ailure
in ections re not co only seen in the neon t l eriod.
or persistent pro ound acidosis.

HELPFUL TIP:
HELPFUL TIP: Look care ully or cold sores or other vesicular lesions on
The much touted “de eroxamine challenge” to see i children with rashes. Also try to get a history o any close
there is ree iron in the blood is not an accurate predic- contacts between the patient and people with cold sores.
tor o toxicity. The test is done by giving an individual a Herpes virus in ection can be devastating to the newborn,
single challenge dose o de eroxamine and seeing i the and they may require treatment with antiviral medication.
urine changes to a “vin rose” color re lecting circulat-
ing ree iron. However, this does not predict who needs
therapy, since the iron may already be working its evil T e child is seen in her ather’s arms. She appears to have
in the periphery. normal color and tone. She is sleeping, but arouses a er some
stimulation. She seems ussy, but can be consoled by her par-
ents. Vitals: temperature 38.7°C rectally, pulse 165 bpm, and
Objectives: Did you learn to . . . respiratory rate 32. She appears to be well hydrated, and oth-
• Rehydrate a child with GI symptoms? erwise has a completely normal physical examination.
• Recognize the mani estations o iron poisoning?
• Manage a child with an iron ingestion? Question 1.23.2 What urther evaluation is indicated now?
A) CBC, blood cultures, c theterized urine or n lysis nd culture.
B) CBC, blood cultures, b g urine or n lysis nd culture.
C) CBC, blood cultures, chest r diogr h, c theterized urine
CASE 1.23 or n lysis nd culture.
A 25-day-old emale newborn is brought to the ED by her D) CBC, blood cultures, c theterized urine or n lysis nd cul-
parents. T ey state that she has not been breast- eeding well ture, lu b r uncture, nd chest r diogr h.
this morning and has elt warm. T ey measured her axillary
temperature as 100.6°F with an axillary digital thermom- Answer 1.23.2 The correct answer is “D.” It is i ort nt th t
eter at home. T ey have not noticed any rhinorrhea, cough, co lete ev lu tion nd se tic work-u be er or ed on ll
or rashes. T e baby is having ve to six wet diapers per day children younger th n 28 d ys old without de nite source o
and ve to six yellow seedy stools per day. T e child has not in ection. T is includes CBC, blood cultures, c theterized urine
had any sick contacts. She slept normally last night, but was a s eci ens or n lysis nd culture, nd lu b r uncture. A
little hard to wake up rom her morning nap today. T e baby chest x-r y need not be done in the tient without res ir tory
was the 7 lb 8 oz product o an uncomplicated term gestation, sy to s but is highly reco ended. LP is nd tory. Even i
born via normal spontaneous vaginal delivery to a group B you sus ect neu oni or U I, n LP should still be consid-
streptococcus (GBS)-negative mother. T ere were no compli- ered, s it is i ossible to tell i the b cteri h ve s re d he -
cations in the early neonatal period, and the baby was dis- togenously to the eninge l s ce.
charged with the mother at 2 days o li e a er receiving rou-
tine neonatal care. At her 2-week weight check, she seemed to
HELPFUL TIP:
be gaining weight well and her doctor had no concerns.
Do not delay antibiotic therapy to obtain a lumbar punc-
ture. Lumbar punctures per ormed within 2 to 4 hours
Question 1.23.1 Which o these is NOT a common cause o
o receiving antibiotics should still yield valid results.
serious in ections in children younger than 1 month?
A) Listeria monocytogenes.
B) Neisseria meningitides. HELPFUL AND CONTROVERSIAL TIP:
C) GBS. Some would argue that a bag urine should be done as
D) Pseudomonas aeruginosa. the initial urine examination. While not as speci ic as
a catheterized urine, it is more sensitive or UTI. I the
Answer 1.23.1 The correct answer is “D.” “A,” L. monocyto- bag UA comes back positive, a catheterized specimen
genes, is n oblig te intr cellul r n erobe th t is tr ns itted should be sent or culture.
tr ns l cent lly ro other to child. “B,” N. meningitides, is
Gr -neg tive di lococcus th t colonizes the res ir tory
tr ct o u to 15% o he lthy individu ls. It is usu lly s re d Question 1.23.3 You are awaiting lab results. Should you
through close cont ct. “C,” GBS, is Gr - ositive org nis th t play World o Warcra t, Minecra t or start antibiotics? You
34 FAMILY MEDICINE EXAMINATION & BOARD REVIEW

decide to start antibiotics. Which antibiotics are most on gener l edi trics oor inste d o n intensive c re unit.
appropriate or empiric therapy in this patient? T is child should be d itted, reg rdless o wh t the l bor -
A) A icillin nd gent icin. tory results de onstr te. So e ex erienced r ctitioners will
B) Ce ri xone. disch rge nontoxic ebrile child ro the ED i he or she is
C) V l cyclovir. older th n 2 onths nd h s ollow-u within 24 hours. How-
D) A oxicillin with or without cl vul n te. ever, there is so e risk inherent in this r ctice—n ely, th t
E) Any o the bove re equ lly v lid choices. deterior tion in the tient’s condition y go unrecognized t
ho e nd th t the ily will il to ollow-u . T e st nd rd o
Answer 1.23.3 The correct answer is “A.” A icillin nd gen- c re is to d it ll children who h ve ever when they re less
t icin cover ll o the co on c uses o serious b cteri l th n 30 d ys old.
in ection in the newborn, nd both ntibiotics enetr te into
the cerebros in l uid (CSF) well. Ce ri xone lso enetr tes HELPFUL TIP:
the CSF well, but is highly bound to lbu in nd y dis l ce With the advent o the polyvalent pneumococcal vac-
bilirubin. T ere h ve been c se re orts o kernicterus ollow- cine and the implementation o universal screening or
ing the d inistr tion o ce ri xone in newborns, so its use is GBS, the incidence o occult serious bacterial in ection is
not reco ended in children younger th n 1 onth. I there alling. It may be that in the near uture, the way in which
is concern or her es virus in ection, cyclovir IV would be ebrile in ants younger than 3 months are evaluated and
re erred over v l cyclovir, but neither o these is used on n treated will change. However, the practice outlined in
e iric b sis routinely. A oxicillin, with or without cl vul - Table 1-8 represents the current standard o care.
n te, does not enetr te the CSF s well s icillin nd is
thus not re erred when eningitis is ossibility. HELPFUL TIP:
Even when present, otitis media is not considered
Question 1.23.4 What is the appropriate disposition or this a source o ever when evaluating the neonate. You
child? should continue with your clinical and laboratory evalu-
A) Ad ission to the gener l edi trics oor. ation as i you did not even see the ears!
B) Ad ission to the edi tric intensive c re unit.
C) Monitor or 3 hours in the ED, nd decide b sed on l bor -
tory results. Objectives: Did you learn to . . .
D) Disch rge with 24-hour ollow-u . • Describe common bacterial agents causing in ection in the
early neonatal period?
Answer 1.23.4 The correct answer is “A.” T is child does • Evaluate the ebrile newborn?
not look toxic nd c n rob bly be n ged ro ri tely • Manage the ebrile newborn?

TABLE 1-8 CURRENT RECOMMENDATIONS FOR EVALUATING THE FEBRILE CHILD


Age ≤ 28 days
• These neonates are assumed to have bacteremia and potential seeding o the CSF, even i a source is discovered.
• Work-up should include cultures o blood, urine, CSF, and stool (i GI symptoms present) and CXR (i respiratory symptoms present).
• CBC and/or CRP can be obtained, but the decision about whether or not to proceed with evaluation should not be based on these results!
• The child should be admitted or IV antibiotics until cultures are negative.

Age 1–3 months


• It is sa est to assume they are still unable to contain bacterial in ections at this age.
• Patients at low risk o having a serious bacterial in ection have the ollowing labs:
• WBC >5,000, <15,000/mm 3 with band count <1,500/mm 3
• Normal urinalysis
• Normal CSF
• Stool microscopy <5 WBC/HPF i diarrhea present
• I no source is ound on examination, it is reasonable or patients meeting these low-risk criteria to be managed with intramuscular ce triaxone in the
ED/clinic—i ollow-up can be arranged to receive a second dose in 24 hours.
• It should be emphasized that these in ants are still vulnerable to dissemination o bacterial in ections. There ore, those with an obvious source, those
who appear clinically ill, or those who do not meet the low-risk criteria should be cultured and admitted or IV antibiotics until cultures are negative.

Age 3–36 months


• Management o ever in this group is somewhat controversial, as the advent o Prevnar (pneumococcal vaccine) and continued use o HIB vaccine will
presumably reduce the risk o invasive bacterial disease.
• It is generally accepted that well-appearing children with evers less than 39°C do not require urther evaluation or antibiotics.
• Up to 5% o children with temperature >39°C who appear clinically well will have positive blood cultures (occult bacteremia), putting them at risk or
serious in ections. One approach is to obtain screening WBC on those with evers >39°C. I WBC <5,000 or >15,000/mm 3 or bands >l,500/mm 3, then
urther evaluation o blood, urine, and CSF should be considered.
CHAPTER 1 • EMERGENCY MEDICINE 35

however, children y require control o the irw y due to


CASE 1.24 hy oxi . “A,” neu oni , is n in ection o the lower irw ys th t
A 6-month-old male is brought to the ED by his ather. He c n be either b cteri l or vir l in n ture. T ese children gener lly
has had a little bit o rhinorrhea or the last 36 hours but no h ve ever nd roductive cough. T ey y be t chy neic nd
evers. A ew hours ago, he began coughing and seemed to h ve n incre sed work o bre thing, but they usu lly do not h ve
be having some di culty breathing. He has been taking his ins ir tory stridor. “C,” l ryngo l ci , is congenit l disorder.
bottle and rice cereal well. His ather states that there have T ese children usu lly develo sy to s t ew weeks o ge
been no changes in his stools. He is ully vaccinated, has no nd resent with ins ir tory stridor th t gets worse with crying. It
signi cant past medical history, and has had no known sick tends to be little better when the child is c l nd in the su ine
contacts. osition. L ryngo l ci resolves s ont neously in the jor-
ity o children s the l rynx beco es ore r nd the irw y
Question 1.24.1 What is the most common cause o respi di eter incre ses, but so e children will require surgic l inter-
ratory distress in a 6 month old (and not necessarily the vention to cilit te eeding nd growth. T e child in this vignette
diagnosis in this child)? is resenting with new roble , s o osed to chronic one, so
A) Pneu oni . this is not l ryngo l ci . “D,” sth , is nother dise se o the
B) Foreign body s ir tion. lower irw ys, nd wheezing is ex ir tory in n ture.
C) Bronchiolitis.
D) Second-h nd s oke ex osure. You decide to do a radiograph o this child’s neck to aid in the
diagnosis (although this is certainly not necessary nor advo-
Answer 1.24.1 The correct answer is “C.” Bronchiolitis is very cated in most cases—but this is a board review book, not real
co on, es eci lly in the winter onths. It is usu lly c used li e).
by the res ir tory syncyti l virus, but c n lso be c used by
r in uenz , in uenz , nd hu n et neu ovirus. Bron- Question 1.24.3 You are most likely to see which o the
chiolitis is usu lly ssoci ted with ro use rhinorrhe , broncho- ollowing on cervical radiograph?
s s , nd ucus lugging o the bronchiole tree. Although, A) T u b sign.
“A,” neu oni is serious c use o res ir tory roble s, it is B) Quincke sign
not terribly co on in in nts. “B,” oreign body s ir tion, is C) S ine sign.
so ething th t ust be lw ys considered in n in nt, es e- D) Retro h rynge l s ce swelling.
ci lly 6- onth-old who is beco ing ore obile ( nd to E) Stee le sign.
who everything looks like ood). Second-h nd s oke ex o-
sure c n c use chronic irrit tion to the res ir tory tr ct nd c n Answer 1.24.3 The correct answer is “E.” R diogr hs in
ex cerb te bronchos s , but it is in requently the sole c use o crou show the “stee le sign,” which is subglottic n rrowing
res ir tory distress. o the tr che ro ede , giving it stee le-like e r nce.
“A,” the thu b sign, is seen in e iglottitis. “B,” Quincke sign is
As you examine the child, you note that he is mildly tachy- incorrect, s it h s nothing to do with r diology. Quincke sign
pneic with some suprasternal, subcostal, and intercos- is the ungu l c ill ry uls tion ssoci ted with ortic insu -
tal retractions. He makes a whistling, wheezing sound on ciency—it’s r re, not seen in children, nd ne rly useless knowl-
inspiration (stridor) that seems to get worse the harder he edge . . . but th t is wh t you re ying us or! “C,” the s ine
breathes. He also has a brassy-sounding cough that does not sign, is loss o rogressive r diolucency o the s ine on l ter l
seem to be productive. chest r diogr h. T is is seen when so ething—cl ssic lly
n in ltr te indic tive o neu oni —is overl ying the lower
Question 1.24.2 What is the most likely diagnosis at this thor cic s ine king the vertebr l bodies e r ore dense.
point? Fin lly, “D,” retro h rynge l s ce swelling, is seen in retro h -
A) Pneu oni . rynge l bscess.
B) Crou .
C) L ryngo l ci . Question 1.24.4 What is the most appropriate de nitive
D) Asth . therapy or this patient at this time?
A) E ine hrine 0.01 g SQ.
Answer 1.24.2 The correct answer is “B.” Crou , or l ryngotr - B) Nebulized lbuterol.
cheobronchitis, is co on in ection o the u er nd lower C) Dex eth sone 0.6 g/kg PO/IM/IV.
res ir tory tr ct. It is ost co only c used by r in uenz D) High ow oxygen nd re re to intub te.
virus, but y lso be c used by in uenz nd res ir tory syn-
cyti l virus. Cl ssic lly, this ects children younger th n 5 Answer 1.24.4 The correct answer is “C.” Corticosteroids hel
ye rs, lthough it is occ sion lly seen in older children. As the to decre se the glottic ede . One dose o dex eth sone 0.3
glottis swells, children develo wheeze/whistle on ins ir tion to 0.6 g/kg ( xi u o 10 g) c n be given vi ulti le
(ins ir tory stridor) nd ch r cteristic br ssy “se l-like” b rk- routes (PO/IM/IV) nd is usu lly su cient to i rove the ir-
ing cough. T e v st jority o c ses re ild. Occ sion lly, w y swelling enough to llow the child to bre the co ort bly.
36 FAMILY MEDICINE EXAMINATION & BOARD REVIEW

T e dv nt ge o dex eth sone over rednisone or nother


corticosteroid is th t its long h l -li e obvi tes the need or ur- QUICK QUIZ: FOREIGN BODY
ther dosing t ho e. While w iting or the dex eth sone to
work, e ine hrine y be d inistered vi nebulizer or severe A 3-ye r-old boy nd his 5-ye r-old sister were being silly,
c ses. T is usu lly le ds to signi c nt clinic l i rove ent nd voiding bedti e, nd ju ing on their rent’s bed. N tur lly,
gives ti e or the steroid to begin to t ke e ect. Subcut neous he h d nickel in his outh th t he’d ound on the oor. When
e ine hrine is usu lly unnecess ry. Albuterol, while hel ul or their other w lked in to wr ngle the into their j s, they
bronchos s , does not do nything to tre t the glottic ede redict bly collided in id- ir nd both ell o the bed nd onto
th t is c using the jority o the res ir tory distress. Intub - the oor. She w s un ble to nd the nickel er the incident.
tion is not indic ted t this oint i the child is not in i ending Coincident lly, the 3-ye r-old eels like he h s so ething stuck
res ir tory ilure, hy oxic, or ini lly res onsive. in his thro t—so ething with T o s Je erson’s he d on it.
When the ily rrives in your o ce, he t kes liquids
HELPFUL TIP: without uch trouble, but won’t t ke nything solid. He s ys
While classically we have used racemic epinephrine, the his “thro t hurts.” He’s not drooling or h ving ny trouble
“d” isomer is inactive. In addition, racemic epinephrine is bre thing.
more expensive and must be kept re rigerated i a multi-
dose vial is used. L-epinephrine, 5 cc o 1:1,000, delivered How do you con r th t the coin is not in the irw y?
by nebulizer is as—i not more—e ective, than racemic A) C o the l rynx nd chest without IV contr st.
epinephrine, is cheaper, and (our avorite since we can’t B) C o the l rynx nd chest with IV contr st.
do simple math!) is the same dose or everyone. C) AP nd l ter l l in l s o the neck.
D) Direct l ryngosco y.
E) C ll ul onologist or e ergent bero tic bronchosco y.
You administer the appropriate dose o dexamethasone to
the child along with a treatment o nebulized epinephrine. The correct answer is “C.” Fortun tely coins re r dio que—
He improves markedly. You watch him or 2 hours. He is able t le st s o the rinting o this book ( ybe we’ll h ve wood-
to tolerate oral f uids well, and is active and play ul. He still en coins g in so ed y!). So e l stic toys leg lly cont in
has a brassy cough, but no inspiratory stridor. b riu nd re r dio que while others re in dvertently
r dio que (like those de in Chin cont in ted with
Question 1.24.5 What should his disposition be? le d . . . ixed blessing t best!). T e eso h gus tends to col-
A) Ad it or 23 hours o observ tion. l se ro nterior to osterior when there is nothing in the
B) Ad inister second dose o r ce ic e ine hrine nd re- lu en. T ere ore, coin in the eso h gus should look round
ev lu te. on n AP x-r y. By contr st, the tr che is su orted by c rti-
C) Disch rge to ho e with close out tient ollow-u . l ginous rings round ost o its circu erence. T e osterior
D) Ad inister lbuterol nd re-ev lu te. rt o the tr che , however, buts the eso h gus nd h s no
c rtil ge. T ere ore, coins th t ll into the tr che gener lly
Answer 1.24.5 The correct answer is “C.” A er d inister- h ve n end-on e r nce on AP r diogr hs nd look like
ing dex eth sone nd e ine hrine, it is i er tive to observe disc on l ter l l .
children or t le st 2 hours. I child redevelo s stridor t rest,
he/she should receive second dose o e ine hrine. Any child T e coin ell into the stomach on the way to x-ray and the
who needs second tre t ent h s ore severe crou , is t little boy eels better.
higher risk o h ving co lic tions, nd should be considered
or hos it l d ission. I the child is ree o stridor 2 hours er Wh t oreign bodies in the stomach need to be re oved e er-
nebulized e ine hrine, she c n be s ely disch rged with close gently?
out tient ollow-u (within 24 hours) s long s the rents re A) A button b ttery.
reli ble, ble to onitor the child, co ort ble with the l n, B) A ercli th t is olded in its origin l or .
nd ble to return i the child’s condition should deterior te. C) wo s ll gnets.
D) A doll’s shoe.

HELPFUL TIP:
Remember that an oxygen saturation o less than 95% is The correct answer is “C.” wo gnets c n ttr ct e ch other
singularly abnormal in a child. through o osing loo s o bowel, c using bowel necrosis nd
er or tion. One gnet should not c use ny trouble, but
two should be t ken seriously. Button batteries lodged in the
Objectives: Did you learn to . . . esophagus need to be removed emergently. However, once a
• Describe common causes o respiratory distress in children? button battery transitions to the stomach, it will likely pass
• Manage pediatric airway problems? without causing any di culty. However, it should be re-
• Treat children with croup? moved i it remains in the stomach or more than 48 hours
CHAPTER 1 • EMERGENCY MEDICINE 37

or is ≥ 15 mm. Other s ooth or rounded objects re unlikely to c lciu st bilizes c rdi c cell e br nes within 1 inute.
c use ny trouble. Even s ll sh r objects ( ush ins) gener lly C lciu glucon te 1 g or c lciu chloride 1 g y be given.
ss without c using er or tion or other signi c nt d ge. C lciu chloride is irrit ting to veins nd y c use necrosis
i it extr v s tes: centr l line d inistr tion is re erred. All o
CASE 1.25 the other nswers do lower ot ssiu levels (including continu-
ously nebulized lbuterol), but when there is evidence o ECG
A 67-year-old emale with a history o dialysis presents ch nges, giving c lciu is your to riority. T e other gents
slumped over and complaining o generalized weakness. y t ke s long s 30 to 60 inutes to ct. As note, bic rbon-
By the time she is in a room, her eyes are closed, she is non- te y not be rticul rly e ective in tients with end-st ge
verbal, withdraws rom pain ul stimuli and does not ollow ren l dise se, nother re son to use c lciu rst in this tient.
commands but has a palpable pulse at 80 beats per minute.
Her husband notes that she has been using “light salt,” which A er two doses o calcium gluconate, the QRS narrows and
contains potassium. You remember the “coma cocktail” you want to get the excess potassium out o your patient’s
rom earlier in this chapter: glucose, naloxone, thiamine and body.
oxygen. You also remember that not all narcotic overdoses
are associated with pinpoint pupils. You treat her appropri- Question 1.25.3 Which o the ollowing removes potassium
ately. T e patient’s blood sugar is 211 mg/dL. T e patient’s rom the body?
husband states that dialysis was not per ormed today. T e A) He odi lysis.
patient also missed her last dialysis appointment 2 days ago B) Insulin/glucose.
because she elt ill at home. Her mental status is unchanged C) K yex l te (sodiu olystyrene)
a er the “coma cocktail”; she is unable to protect her airway. D) Albuterol.
Husband con rms patient is ull code. You decide to proceed E) A nd C.
with intubation.
Answer 1.25.3 The correct answer is “E.” Both he odi lysis
Question 1.25.1 The best medications to use in this patient nd sodiu olystyrene will re ove ot ssiu ro the body.
are (induction agent/paralytic agent): Insulin/glucose will drive it intr cellul rly in 30 to 60 inutes
A) Eto id te/succinylcholine. but does not rid the body o ot ssiu . Be c re ul: sodiu oly-
B) Eto id te/ket ine. styrene exch nges sodiu or ot ssiu nd thus y worsen
C) Eto id te/rocuroniu . congestive he rt ilure. T e use o sodium polystyrene with
D) An induction gent nd r lytic re unnecess ry in this sorbitol is discouraged. It is ssoci ted with bowel ische i
tient since she is only res onsive to dee sti uli. nd necrosis. In addition, be care ul using sodium polysty-
rene in those with GI problems (ileus, stricture, etc.) or the
Answer 1.25.1 The correct answer is “C.” Rocuroniu would be same reason. Note th t tiro er, new ot ssiu binder, h s
the r lytic o choice in this tient. In tient with end-st ge recently been roved or use in the United St tes. However, it
ren l dise se on he odi lysis, ssu e the resence o hy erk - y not lower ot ssiu or u to 7 hours er d inistr tion
le i until roven otherwise. Succinylcholine y c use hy er- nd is NO roved or the cute lowering o ot ssiu .
k le i ( nd c rdi c rrest) so it is contr indic ted in tients
with high likelihood o hy erk le i . Succinylcholine should T e patient does well and, in the uture, knows to avoid
lso be voided in crush injuries, neurologic injuries/ yo - “light” salt (KCl) (o course, “light” is now spelled “lite” or
thies, nd burn tients where it y c use lign nt hy er- some unknown reason).
ther i . A sed tive should be used or induction to revent in
nd either eto id te or ket ine y be used—not both. Objectives: Did you learn to . . .
• Describe contraindications o succinylcholine in intubation?
You secure the irw y. A S A ot ssiu returns t 7.9.
D rn! You look t the onitor nd notice the QRS is looking • Treat hyperkalemia in the acute situation?
little wide. Double d rn!

Question 1.25.2 While you are waiting or an ECG to be


obtained, you administer:
CASE 1.26
A) K yex l te. A 24-year-old male presents to your ED complaining o
B) Sodiu bic rbon te. “dental pain.” He reports a long history o poor dental hygiene
C) C lciu glucon te. and has not seen a dentist in several years. He smokes two
D) Insulin/glucose. packs o cigarettes per day (and proudly wears a “Marlboro”
E) Albuterol nebulizer. hat) and admits drinking a 2 liter bottle o “Hillbilly Holler”
every day at work (it includes a ree coupon or dental service
Answer 1.25.2 The correct answer is “C.” In hy erk le i a er 100 bottles, but he has never availed himsel o this). T e
with evidence o ECG ch nges ( e ked w ves, wide QRS, pain is described as constant and throbbing in nature located
sine w ve), c lciu needs to be d inistered i edi tely. T e in the right lower jaw area.
38 FAMILY MEDICINE EXAMINATION & BOARD REVIEW

Question 1.26.1 What ndings during your examination or a mild sore throat and given a “Z-Pak” (azithromycin—
would raise your concern regarding the patient’s clinical mostly on a whim). He has gotten progressively worse. Chest
condition? x-ray shows in ltrates and small abscesses.
A) L rge ounts o secretions.
B) Decre sed bility to o en his outh or the ex in tion. Question 1.26.3 Your presumptive diagnosis is:
C) Swollen nd elev ted tongue. A) Mononucleosis.
D) P in nd decre sed r nge o otion o the neck. B) Fusobacterium.
E) All o the bove. C) Arcanobacterium hemolyticum.
D) Vir l URI
Answer 1.26.1 The correct answer is “E.” I tient is h v-
ing trouble sw llowing secretions (“A”), it should r ise concern Answer 1.26.3 The correct answer is “B.” Fusobacterium is
or swelling in the osterior h rynx which c n co ro ise n n erobic in ection th t h s been ound with incre sing
not only the bility to sw llow one’s s liv but c n lso sign l requency in dolescent/college ge tients. It c n initi lly
high risk or u er irw y occlusion second ry to swelling. resent si il rly to stre thro t but y go on to “Le ierre
ris us (“B”) is lso concerning bec use it y indic te dee er syndro e,” which is se tic thro bo hlebitis o the intern l
in ection involving the uscles o stic tion. “C,” swollen jugul r vein. T is c n then le d to se tic e boli to the lungs,
nd elev ted tongue, y indic te the develo ent o Ludwig se sis, nd ultiorg n ilure. T e oint here is th t enicillin
ngin , li e-thre tening in ection o the oor o the outh, is still the drug o choice or stre thro t, nd it covers Fusobac-
which c n s re d to the dee er tissues o the sub ndibu- terium, which the crolides do not. As or the other nswers,
l r nd sub xill ry s ces. “D,” in nd decre sed r nge o this is not likely ononucleosis (“A”), since the neck swelling is
otion o the neck, could indic te the ossibility o retro h - unil ter l nd tients with ono usu lly re not this toxic nor
rynge l bscess. h ve in ltr te on chest x-r y. “C,” A. hemolyticum, is lso co -
on in college ge students nd is initi lly clinic lly indistin-
HELPFUL TIP: guish ble ro stre thro t. However, 50% o tients will h ve
Adult epiglottitis o ten presents with sore throat and culo ul r or sc rl tini or r sh st rting on the extre i-
neck tenderness. So, i the patient has neck tenderness ties nd involving the trunk nd b ck but s ring the he d; s
out o proportion to what you would expect, consider o osed to the r sh o sc rlet ever the r sh does not eel. It
epiglottitis or a retropharyngeal abscess. r rely c uses inv sive dise se such s neu oni or eningitis.
It will res ond to erythro ycin or clind ycin nd less so to
enicillin. “D” is obviously incorrect nd i you chose this one,
Luckily, the patient is otherwise clinically stable aside rom b ck to edic l school or you!
the ocal dental pain. Your examination shows the tissues
around tooth number 29 (we can never remember that pesky T e patient spends 3 weeks in the ICU and eventually suc-
numbering scheme either) to be swollen and inf amed with cumbs to his disease. Not good (way or us to end on a
signs o f uctuance and severe tenderness on palpation. An “downer” you say, but at least you get the message that
orthopantogram shows a periodontal abscess. “Z-paks” aren’t always the answer!).

Question 1.26.2 Which antibiotic is NOT an appropriate Objectives: Did you learn to . . .
• Identi y Fusobacterium and Arcanobacterium in ections?
choice or this clinical condition?
A) Penicillin VK. • Initial treatment o dental abscesses?
B) Clind ycin.
C) A oxicillin/cl vul n te.
D) ri etho ri –sul ethox zole. Clinical Pearls
Avoid CT scans in patients with known urolithiasis who have
Answer 1.26.2 The correct answer is “D.” Penicillin VK, the same symptoms again unless you suspect a postobstruc-
clind ycin, nd oxicillin/cl vul n te ll rovide good cov- tive in ection or have reason to believe it could be an aortic
er ge or n erobic b cteri including n erobic Gr -neg tive problem (e.g., at-risk patients over 50 years old).
cocci, which re the in cul rits in dent l in ections. MP– Avoid the use o narcotics or migraine headaches i possible.
SMX h s so e Gr -neg tive cover ge but l cks the n erobic Prochlorperazine or metoclopramide work well with pro-
cover ge needed to tre t in ections o the or l c vity. chlorperazine IV being the pre erred agent.
Do not do a chest CT i a patient is PERC negative or is low
T e same gentleman returns to you 3 weeks later complaining risk or a PE and has a negative d-dimer. Besides the expense,
o a sore throat. On examination, he looks toxic and has ever, the cancer risk or a 20-year-old emale rom a single 64 slice
chills, dyspnea/cough, and unilateral neck swelling. His throat chest CT is approximately 1 in 250.
looks similar to a streptococcal tonsillitis. However, the neck Do not do a head CT or patients with simple syncope: syn-
is tender and this guy really looks sick. Pulse is 130 bpm and cope is not a brain disease. It is generally a problem o CNS
BP is 80/50 mm Hg. He was seen last week by someone else
CHAPTER 1 • EMERGENCY MEDICINE 39

Frithsen IL, Si son WM Jr. Recognition nd n ge-


per usion; you have to knock out both hemispheres simulta-
ent o cute edic tion oisoning. Am Fam Physician.
neously to lose consciousness.
2010;81:316–323.
Do not do a head CT in low risk patients with head trauma. Green NE, Allen L. V scul r injuries ssoci ted with disloc -
Use a prediction rule. However, it is reasonable to CT those tion o the knee. J Bone Joint Surg Am. 1977;59:236–239.
on anticoagulants.
Herz AM, et l. Ch nging e ide iology o out tient b ctere-
Do not do a head CT looking or the cause o vertigo; it is i in 3- to 36- onth-old children er the introduction
almost always useless. I you need imaging, do an MRI but o the he t v lent-conjug ted neu ococc l v ccine.
recognize that and MRI will be negative or posterior circula- Pediatr In ect Dis J. 2006;25:293–300.
tion stroke in 30% o cases within the irst 24 hours. Hu JS, et l; ACEP Clinic l Policies Co ittee. Clinic l
Do not do a head CT o children with a irst uncomplicated Policy: critic l issues in the ev lu tion nd n ge ent
seizure. o dult tients resenting to the e ergency de rt ent
Do not get back imaging or acute low back pain unless there with seizures. Ann Emerg Med. 2014;63:437–447.
are red lags, such as: ever, cancer history, signi icant trauma Isbister GK, Ku r VV. Indic tions or single-dose ctiv ted
(not just twisting or picking something up), neurologic symp- ch rco l d inistr tion in cute overdose. Curr Opin Crit
toms or signs. Care. 2011;17:351–357.
Only give as much blood as is necessary. Giving one unit is Leung AK, Sig let DL. Acute bdo in l in in children. Am
ine. As a general rule, don’t trans use a stable patient with a Fam Physician. 2003;67:2321–2326.
hemoglobin o 7.0 g/dL or greater. M rx J, et l. Rosen’s Emergency Medicine Concepts and Clinical
Practice. 8th ed. St. Louis, MO: Mosby Elsevier He lth
Phenytoin and osphenytoin do not work or alcohol with-
Sciences; 2010.
drawal seizures, seizures caused by drug overdose or with-
drawal (in general). Use a benzodiazepine. Mokhlesi B, et l. Adult toxicology in critic l c re: P rt II:
S eci c oisonings. Chest. 2003;123:897–922.
Mosc ti RM, et l. A ulticenter co rison o t w ter ver-
sus sterile s line or wound irrig tion. Acad Emerg Med.
2007;14(5):404–409.
BIBLIOGRAPHY Nikk nen H, Skolnik A. Di gnosis nd n ge ent o c rbon
Benson BE, et l; A eric n Ac de y o Clinic l oxicology; onoxide oisoning in the e ergency de rt ent. Emerg
Euro e n Associ tion o Poisons Centres nd Clinic l Med Pract. 2011;13:1–14.
oxicologists. Position er u d te: g stric l v ge or P nju AA, et l. T e r tion l clinic l ex in tion. Is this tient
g strointestin l decont in tion. Clin Toxicol (Phila). h ving yoc rdi l in rction? JAMA. 1998;280:1256–1263.
2013;51(3):140–146. Rogers RL, McCor ck R. Aortic dis sters. Emerg Med Clin
Brown CV, et l. Preventing ren l ilure in tients with North Am. 2004;22:887–908.
rh bdo yolysis: do bic rbon te nd nnitol ke Steiner RW. re ting cute bronchiolitis ssoci ted with RSV.
di erence? J Trauma. 2004;56(6):1191–1196. Am Fam Physician. 2004;69:325–330.
C rr ro S, et l. Bronchiolitis: ro e iricis to scienti c ouger M, et l. Rel tionshi between venous nd rteri l
evidence. Minerva Pediatr. 2009;61:217–225. c rboxyhe oglobin levels in tients with sus ected c rbon
Diercks DB, et l; ACEP Clinic l Polices Co ittee. Clinic l onoxide oisoning. Ann Emerg Med. 1995;25(4):481–483.
Policy: critic l issues in the ev lu tion nd n ge ent o Wol SJ, et l; ACEP Clinic l Policies Co ittee. Clinic l Poli-
dult tients with sus ected cute nontr u tic thor cic cy: critic l issues in the n ge ent o dult tients re-
ortic dissection. Ann Emerg Med. 2015;65:32–42. senting to the e ergency de rt ent with cute c rbon
Engebretsen KM, et l. High-dose insulin ther y in bet - onoxide oisoning. Ann Emerg Med. 2008;51:138–152.
blocker nd c lciu ch nnel-blocker oisoning. Clin Yucesoy K, Yuksel KZ. SCIWORA in MRI er . Clin Neurol
Toxicol (Phila). 2011;49(4):277–283. Neurosurg. 2008;110:429–433.
Cardiology
K mbe S. De c u and Ma k A. G abe
2
risk actors is only one consideration in the evaluation o this
CASE 2.1 patient. Smoking, hypertension, amily history, etc., do not
A 35-year-old emale presents with a 1-hour history o chest change the prior probability o cardiac disease enough to allow
pain, which resolved spontaneously. T e pain is described them to be used to rule out or rule in cardiac disease. O note,
as a chest pressure radiating to both arms. T e patient is a male gender and diabetes DO increase the pretest probabil-
smoker but has no other risk actors (no amily history o car- ity o coronary artery disease (CAD) (luckily our patient is a
diac disease, hypertension, diabetes, hyperlipidemia, etc.). woman!). Evaluation o pretest probability is important in the
T e patient is diaphoretic and has a normal blood pressure. diagnostic algorithm, but should be used in addition to, not in
Physical examination reveals that the patient has tenderness exclusion o , clinical judgment and ndings. “D” is incorrect. It
to palpation o the anterior chest wall that reproduces the is true that chest pain reproduced by palpation o the chest wall
chest pressure. She is now otherwise ree o chest pain and all makes cardiac disease less likely. However, 15% o patients with
her lab assays, including cardiac enzymes, are normal. cardiac disease and 17% o patients with a pulmonary embolism
(PE) will have their pain reproduced by chest wall pressure. T is
Question 2.1.1 Which o the ollowing is true about this does not mean that you are making their cardiac pain worse. It is
patient’s physical ndings and history? likely because o the patient’s inability to discriminate between
A) Pain radiating to both arms makes it unlikely that this the types o pain (cardiac vs. chest wall).
patient’s pain is cardiac.
B) T e physical ndings that are most highly associated with You decide that urther testing is warranted, including an
an acute myocardial in arction (AMI) include hypotension, ECG and cardiac enzymes.
diaphoresis, and a new S3 gallop.
C) T e absence o risk actors makes it unlikely that this patient Question 2.1.2 Which o the ollowing statements is TRUE?
has cardiac disease. A) A normal initial ECG in the emergency department (ED)
D) T e act that the pain is reproducible on palpation o the e ectively rules out cardiac disease.
chest wall e ectively rules out cardiac disease. B) Creatine phosphokinase MB raction (CPK-MB) is more
E) Based on the in ormation available, urther cardiac evalua- sensitive but less speci c than troponin.
tion is unnecessary. C) Serum troponin is an unreliable marker o cardiac ischemia
in patients with renal ailure.
Answer 2.1.1 The correct answer is “B.” T e ndings that D) T e serum troponin is 100% speci c or myocardial in arc-
are most likely to be associated with an AMI are hypoten- tion.
sion, diaphoresis, and a new S3 gallop. “A” is not true because E) A normal troponin and CPK in the ED cannot be used to
pain radiating to both arms can still be associated with car- make decisions about who to admit.
diac disease. In act, compared with le arm radiation, right
arm radiation or bilateral arm radiation doubles the likeli- Answer 2.1.2 The correct answer is “E.” Except in cases where
hood o the pain being cardiac (LR 2.3 or radiation to the le the chest pain has been continuous or over 12 hours, normal
arm vs. LR 4.1–4.7 or radiation to the right or bilateral arms). cardiac enzymes (troponin, CPK-MB) do not rule out cardiac
(JAMA. 2005;294(20):2623–2629.) Women with AMI o en pres- disease. I they did, we would not admit patients or a “rule out”
ent atypically and may experience more chest pain radiating to the but would rather rely on the single level drawn in the ED. “A” is
right arm/shoulder and the anterior neck or with abdominal incorrect since 9% o patients with AMI will have a normal ini-
pain as compared with men. “C” is incorrect. T e absence o tial ECG in the ED. In act, only about 50% o those with AMI
40
CHAPTER 2 • CAr Dio l o Gy 41

have a diagnostic ECG in the ED. Even a normal ECG obtained Question 2.1.4 Which o the ollowing drug(s) is/are indi
during chest pain does not reliably rule out AMI (Acad Emerg cated in the initial management o this patient?
Med. 2009;16:495). “B” is incorrect since the CPK-MB is over- A) Aspirin.
all less sensitive than a high sensitivity troponin. CPK-MB and B) T rombolytic such as tPA or streptokinase.
myoglobin may become positive earlier than the troponin. C) Heparin.
However, CPK-MB and myoglobin add little, i anything, to the D) Glycoprotein IIb/IIIa inhibitor (e.g., apciximab [ReoPro]).
troponin; many labs no longer per orm these assays in house. E) All o the above.
Eighty percent o AMIs will have one positive marker within the
rst 3 hours o ED arrival (but 20% will not). “C” is incorrect. Answer 2.1.4 The correct answer is “A.” Immediate therapy
Patients with renal disease may have a mildly elevated troponin in the ED requires ASA 325 mg orally (chewed). Since we are
at baseline due to poor clearance, but troponin can still be use- not sure that this patient has AMI or unstable angina, there is
ul in these patients i it continues to rise. It may be use ul to no indication or thrombolytic therapy (“B”), heparin (“C”) or
have knowledge o the patient’s baseline troponin in renal ail- glycoprotein IIb/IIIa inhibitor (“D”). Since she is currently pain
ure, but this is NO an indication to start drawing a baseline ree, heparin carries more o a risk than a bene t at this junc-
troponin on all o your patients with renal ailure. “D” is incor- ture and is not recommended. However, all patients with pos-
rect because we now know that other processes, such as PE, can sible angina or an AMI should have aspirin unless they are truly
elevate the serum troponin. allergic (hives, anaphylaxis). “B” is incorrect because thrombo-
lytics are indicated or acute S elevation myocardial in arctions
HELPFUL TIP: (S EMI), not or a simple chest pain evaluation.
E evated t p n n eve s ma be due t c nd t ns the
than AMi, nc ud ng hea t a u e, PE, bu ns, seps s HELPFUL TIP:
the c t ca ness, st ke, and m e. Sh u d “Mo NA (m ph ne, x gen, n t g ce n, as-
p n) g eet a pat ents,” as the ed t s ea ned n med
sch ? Pe m e ecent 2010 C ch ane Database S s-
HELPFUL TIP: temat c r ev ew, the e cac x gen n AMi has been
The new u t a-sens t ve t p n n ma be p s t ve w th n quest ned and ma even be det menta n th se w th
3 h u s. y u need t kn w what test u h sp ta s a n ma x gen satu at n (94% g eate , C ch ane
d ng. T p n n eve s peak n ab ut 36 h u s a te an Database S st r ev. 2010;6:CD007160). S , w thh d x-
n a ct and ma sta e evated 7 t 10 da s, s the gen un ess the x gen satu at n s <94%. M ph ne
n a ct ma have ccu ed an whe e w th n th s t me sh u d be used n a te a pat ent a s spec c the a-
ame. F w the t end the t p n n t he p dete - p , such as asp n and n t g ce n. And n t g ce n
m ne when the n a ct ccu ed. d es n t change n a ct s ze.

Question 2.1.3 All o the ollowing statements are true HELPFUL TIP:
EXCEPT: Cu ent use wa a n asp n sh u d not p ec ude
A) All myocardial in arctions present with chest pain. the adm n st at n asp n n the ED a pat ent
B) Dyspnea may be the only presenting symptom o myocar- w th chest pa n that ma be ca d ac n g n. y u neve
dial in arction. kn w whethe the pat ent s actua tak ng t n t. S ,
C) Patients with myocardial in arction can present with syn- un ess the e s a ea a e g t asp n, t must be g ven
cope. t chest pa n pat ents n the ED.
D) Females, the elderly, and diabetic patients are more likely to
present with atypical symptoms o myocardial in arction.
T e patient tells you that she is allergic to aspirin, which
Answer 2.1.3 The correct answer is “A.” As the saying goes, causes hives and bronchospasm. She can, however, take other
“Never say never, and never say always.” Many elderly and nonsteroidal anti-in ammatory drugs (NSAIDs) without
diabetic patients (“D”) will present with atypical symptoms or di culty. Oh, great. Now you need to go to plan B (no, not
painless, “silent” myocardial in arctions. In act, up to 30% o the “morning a er pill”).
myocardial in arctions are pain ree. “B” is a correct statement
because, especially in the elderly, dyspnea may be the only pre- Question 2.1.5 Which o the ollowing is an acceptable
senting symptom due to le ventricular ailure secondary to substitute or aspirin in this situation?
ischemia. “C” is a correct statement because syncope (as well as A) Dipyridamole.
lightheadedness and atigue) can be presenting symptoms o a B) Clopidogrel (Plavix).
myocardial in arction. C) Ibupro en or naproxen.
D) Celecoxib (Celebrex).
Her ECG shows nonspeci c S - changes. E) Salsalate.
42 FAMil y MEDiCiNE EXAMiNATio N & Bo Ar D r EViEW

Answer 2.1.5 The correct answer is “B.” Clopidogrel in a Answer 2.1.6 The correct answer is “E.” Only a small majority
loading dose o 600 mg can be used as a substitute or aspi- (59%) o patients with pulmonary emboli have pain as a eature.
rin in the setting o unstable angina or AMI. “A” is incorrect “A” is incorrect because about 20% o patients with cardiac pain
because dipyridamole (in combination with aspirin) is indi- will have their pain relieved by a GI cocktail. Conversely, “D” is
cated only or stroke prevention. Dipyridamole itsel is a rela- incorrect because nitroglycerin can relieve pain rom esopha-
tively weak platelet inhibitor. “C” is incorrect because ibupro- geal spasm as it is a nonselective smooth muscle relaxer. “B” is
en and naproxen are reversible platelet inhibitors that do not incorrect because only 50% o patients with an aortic dissec-
give adequate platelet inhibition and have NO been shown to tion will have unequal pulses and blood pressures, and only
be o bene t in angina/AMI. In addition, both ibupro en and 75% will have an abnormal chest x-ray. T e consideration o an
naproxen can block the e ect o aspirin by making its binding aortic dissection mandates a chest C scan with contrast, trans-
sites on platelets unavailable. In act, stopping NSAIDs in any esophageal echo, or angiogram. Remember that about 20% o
patient being admitted or possible CAD is considered good the population will have unequal blood pressures in the upper
practice; they increase the risk o a cardiac event. “D” and “E” extremities at baseline. “C” is incorrect because most patients
are both incorrect because celecoxib and salsalate have not been with spontaneous pneumothorax can be treated with a “pigtail”
shown to inhibit platelets to a signi cant degree and thus would catheter with a Heimlich valve. T is type o treatment reduces
be o no use in this situation. the morbidity associated with a chest tube.

HELPFUL TIP:
HELPFUL TIP:
Chest x- a nd ngs n pat ents w th th ac c a t c d s-
N te that the ad ng d se c p d g e has been n-
sect n ma nc ude w dened med ast num, b te ated
c eased m 300 t 600 mg. But e the s ke an accept-
a t c kn b, p eu a “capp ng,” t achea dev at n, de-
ab e answe n the b a d exam nat n (and n p act ce).
p ess n e t ma n stem b nchus, es phagea dev a-
Newe ant p ate et agents, such as p asug e (E ent) and
t n, and ss the pa at achea st pe.
t cag e (B nta) a e bec m ng m e w de used as
ad ng agents. H weve , b eed ng sks a e h ghe w th
the newe agents.
T e patient’s pain recurs in the ED. You suspect that she is
having a myocardial in arction, but do not yet have unequiv-
ocal proo , such as ECG changes or elevated cardiac enzymes.
HELPFUL TIP: T e patient becomes markedly hypotensive in response to
o the NSAiDs (except asp n), nap xen appea s t another dose o sublingual nitroglycerin.
ca the west ca d ac sk. H weve , t has m e Gi
s de e ects. C x-2 nh b t s, such as ce ec x b, and Question 2.1.7 Which o the ollowing is TRUE?
d c enac a e Gi end but have m e adve se ca - A) Intravenous nitroglycerin is contraindicated in this patient.
d ac events. ibup en acts m e ke nap xen (l ancet. B) Hypotension caused by nitroglycerin is usually unrespon-
2013;382(9894):769–779). Guidelines suggest stop- sive to IV saline.
ping all NSAIDs except aspirin upon admitting a C) Hypotension caused by nitroglycerin may be indicative o a
patient for ACS or a question of ACS. Please do not right ventricular in arction, which is most commonly asso-
use ketorolac to treat chest pain as this is also an ciated with an in erior wall myocardial in arction (IAMI).
NSAID. D) Hypotension caused by nitroglycerin is diagnostic o car-
diogenic shock, suggesting that this patient will have a poor
outcome.
Well, not all chest pain is cardiac, and this patient may have
another cause or hers. Answer 2.1.7 The correct answer is “C.” Hypotension in
response to nitroglycerin may be indicative o a right ventricu-
Question 2.1.6 Which o the ollowing is TRUE? lar in arct, which is most commonly associated with an in erior
A) Giving a “GI cocktail” (e.g., combination o Maalox and wall MI (IAMI). Since the right ventricle is dependent on lling
viscous lidocaine) can reliably di erentiate cardiac rom pressure (preload), nitroglycerin, which drops the preload, will
esophageal/GI causes o chest pain. requently result in hypotension in those with a right ventricu-
B) A normal chest radiograph and symmetrical pulses in the lar in arct. “A” is incorrect because hypotension rom sublingual
upper extremities reliably rules out a thoracic aortic dissec- nitroglycerin is not a contraindication to additional nitrates
tion. once the patient’s blood pressure is stable. A typical sublingual
C) Most patients with a spontaneous pneumothorax should be dose is 400 µg (0.4 mg). A typical IV dose starts at 20 µg/min.
treated with a chest tube. T us, the sublingual dose is quite a bit larger than the IV dose.
D) I nitroglycerin relieves the chest pain, then the pain is cer- In such a situation, you could consider starting IV nitroglycerin
tainly cardiac. at 10 to 20 µg/min and titrating up as the blood pressure allows.
E) Pain is a nding in only about 60% o patients with a PE. “B” is incorrect because hypotension rom nitroglycerin will
CHAPTER 2 • CAr Dio l o Gy 43

generally respond to a saline bolus. “D” is incorrect. Certainly, saline does acutely) once right ventricular unction returns to
patients with cardiogenic shock will be hypotensive, but hypo- normal.
tension with nitroglycerin is a common result o the drug itsel
and does not de ne cardiogenic shock. T e patient’s pain continues despite treatment with nitro-
glycerin, and you obtain another ECG (Fig. 2-1).

HELPFUL TIP: Question 2.1.9 Which o the ollowing is TRUE regarding


C ns de h d ng beta-b cke s n n e wa acute this ECG?
Mi (iAMi), as these pat ents ten have b ad ca d a and A) T is injury pattern on ECG is most consistent with an ante-
hea t b ck. A s , bewa e at p ca p esentat ns rior wall MI.
iAMi such as nausea, v m t ng, and the Gi s mpt ms. B) In this situation, intervention in the cath lab with percuta-
H d beta-b cke s n th se w th h p tens n, hea t neous transluminal coronary angioplasty (P CA) and stent
b ck hea t a u e. placement is superior to tPA or other thrombolytic.
C) T is injury pattern on ECG is most consistent with pericar-
ditis.
Question 2.1.8 Which o the ollowing is TRUE o patients D) T is injury pattern on ECG proves that this patient does not
with an IAMI? have an aortic dissection.
A) T ey will likely continue to have problems with right ven- E) T is pattern on ECG is totally ne. What, me worry?
tricular unctioning in the uture.
B) T ey will need to increase their salt intake in order to Answer 2.1.9 The correct answer is “B.” Intervention in the cath
increase preload and right ventricular lling pressure. lab with P CA and/or stent placement is superior to thrombo-
C) T eir right ventricular unction should return to normal or lytic therapy in the treatment o AMI, provided that the “door to
close to normal ollowing their in arction. balloon” time is 90 minutes or less. In cases where the patient is
D) A and B. located in a acility without a cardiac catheterization laboratory,
the patient may receive thrombolytic therapy. “A” is incorrect
Answer 2.1.8 The correct answer is “C.” Most patients will because this pattern is indicative o an in erior wall, not an ante-
have return o right ventricular unctioning ollowing a myo- rior wall, MI. You will note that this ECG shows S elevations
cardial in arction. “B” is incorrect because there will be no need in leads II, III, and aVF (in erior leads) along with reciprocal S
to increase right ventricular lling pressure (which is what IV segment depression in leads V1 and V2. An anterior wall MI is

FIGURE 2-1. ECG pat ent n quest n 2.1.9.


44 FAMil y MEDiCiNE EXAMiNATio N & Bo Ar D r EViEW

de ned by S elevations in leads V3, V4, and V5, and an antero- TABLE 2-1A ACC/AHA GUIDELINES FOR THE
septal MI shows S elevations in leads V1, V2, and V3. For IAMI MANAGEMENT OF PATIENTS WITH ST-ELEVATION
concerning or RV in arction, consider “right-sided chest leads.” MYOCARDIAL INFARCTION
“D” is incorrect because patients with pericarditis should have
C ass 1 ec mmendat ns the use th mb t cs n
S elevations in all leads (although an ECG is only 80% sensitive
m ca d a n a ct n nc ude an o NE the w ng th ee
or pericarditis). “D” is incorrect because patients with an aortic ECG nd ngs:
dissection can present with an abnormal ECG that looks simi- • ≥1 mm ST-segment e evat n n at east 2 c nt gu us mb
lar to an in arct pattern. So, ECG changes do not prove that the eads
patient does not have an aortic dissection. “E” is just plain wrong • 1–2 mm ST-segment e evat n n at east 2 c nt gu us p ec d a
eads
and you should be worried i you see this pattern!
• New c mp ete bund e b anch b ck that bscu es the ST segment
ana s s p us a h st suggest ve Mi
You now have all the evidence that you need to show that this And
patient is indeed having an ongoing myocardial in arction. • P esent ng 12 h u s s nce the nset pa n, age <75 ea s (a th ugh
Since your rural hospital is “just around the corner rom t eat ng th se >75 ea s age s st a c ass 2 ec mmendat n
be w)
nowhere,” stenting is not going to happen within 90 minutes.
You decide to initiate thrombolytic therapy. C ass 2 ec mmendat ns the use th mb t cs n m ca d a
n a ct n nc ude an o NE the w ng:
• ≥1 mm ST-segment e evat n n at east 2 c nt gu us mb
Question 2.1.10 All o the ollowing are true statements eads and age >75 ea s o r p esent ng 12–24 h u s a te nset
EXCEPT: n a ct n
A) Patients who are candidates or thrombolytics must have at • 1–2 mm ST-segment e evat n n at east 2 c nt gu us p ec d a
least 1 mm o S -segment elevation in at least 2 contiguous eads and age >75 ea s o r p esent ng 12–24 h u s a te the nset
n a ct n
limb leads or at least 1 to 2 mm o S -segment elevation in • B d p essu e >180 mm Hg s st c and >100 mm Hg d ast c
at least 2 contiguous precordial leads. n a pat ent w th a “h gh- sk” m ca d a n a ct n (e.g., the h gh
B) Patients who are candidates or thrombolytics must have an sk the Mi m t gates the wa n ng ab ut th mb t c use n
absence o prior history o hemorrhagic stroke within the unc nt ed h pe tens n)
past year.
r ep duced m Antman EM, et a . ACC/AHA gu de nes the
C) Patients who are candidates or thrombolytics should have
management pat ents w th ST-e evat n m ca d a n a ct n—
no active bleeding, including menstrual bleeding. execut ve summa . J Am C Ca d . 2004;44(3): 671–719. C p ght 2004,
D) Patients who are candidates or thrombolytics should have w th pe m ss n m E sev e .
no history o recent head trauma.
E) Patients who are candidates or thrombolytics should not be
pregnant.

Answer 2.1.10 The correct answer is “C.” While active inter-


nal bleeding is a contraindication to the use o thrombolytics,
TABLE 2-1B ACC/AHA GUIDELINES FOR THE
menstrual bleeding is not. While there are no controlled trials,
MANAGEMENT OF PATIENTS WITH ST-ELEVATION
anecdotal evidence suggests that thrombolytics are sa e with
MYOCARDIAL INFARCTION
menstrual bleeding. “A” is correct. In addition to these ECG
criteria, the presence o a new complete le bundle branch Class 3 “Absolute” contraindications to the use of thrombolytic
therapy in MI
block (LBBB) with characteristic MI pain also indicates that the
• P ev us hem hag c st ke at an t me st ke w th n the ast
patient may bene t rom thrombolysis. Patients with only S - 12 m nths
segment depression or a normal ECG, even with symptoms, do • Kn wn nt ac an a ne p asm
not bene t. “B,” “D,” and “E” are all true statements. Patients • Act ve nte na b eed ng (but n t menst ua b eed ng)
are not candidates or thrombolytics i they have recent head • Suspected a t c d ssect n
trauma, are pregnant, or have had a hemorrhagic stroke in the Relative contraindications to the use of thrombolytic therapy in MI
last year. T ere are additional criteria or and contraindications • Unc nt ed h pe tens n (>180/110 mm Hg) at the t me
to the use o thrombolytics. See ables 2-1A and 2-1B. p esentat n.
• H st b eed ng d athes s, ng ng ant c agu at n (iNr >2–3)
• T auma, nc ud ng t aumat c CPr w th n 2–4 weeks, maj su ge
HELPFUL TIP: w th n 3 weeks
“Fac tated PCi,” that s, adm n st at n th mb t cs • N nc mp ess b e vascu a punctu es (e.g., subc av an ne)
w th the ntent t pe m PCi w th n 2 m e h u s • inte na b eed ng w th n the ast 2–4 weeks
• P egnanc
g v ng th mb t cs has ve m xed (and m st nega-
• Pept c u ce d sease (b eed ng n t)
t ve) data. o utc mes a e w se (and n t just because PCi • Seve e, ch n c h pe tens n
s a “ escue” techn que at th s p nt). it ce ta n s NOT
the standa d ca e and, n act, s n nge c ns de ed r ep duced m Antman EM, et a . ACC/AHA gu de nes the management
as pa t the t eatment a g thm. i th mb t cs pat ents w th ST-e evat n m ca d a n a ct n—execut ve summa . J Am C
Ca d . 2004;44(3): 671–719. C p ght 2004, w th pe m ss n m E sev e .
CHAPTER 2 • CAr Dio l o Gy 45

ve sus p ma PCi s be ng c ns de ed, t s va uab e t TABLE 2-3 CLASS I INDICATIONS FOR PACEMAKER IN
c ntact the ca d g st at the cath cente t dete m ne PATIENTS WITH AN ACUTE MYOCARDIALINFARCTION
whethe he/she w u d ke th mb t cs be e t ans- New e t bund e b anch b ck + st-deg ee AV b ck
e —t m ng (b th m s mpt m nset and t ans e ) New ght bund e b anch b ck + e t ante p ste asc cu a
s cent a t the dec s n mak ng p cess. “r escue PCi” b ck + st-deg ee AV b ck
a ed epe us n a te th mb t c the ap sh u d M b tz t pe ii hea t b ck
be pe med as s n as g st ca p ss b e, dea Th d-deg ee hea t b ck
w th n the st 24 h u s, but No T w th n the st 2 t S mpt mat c b ad ca d a un esp ns ve t at p ne.
3 h u s p stth mb t c the ap .

importantly, the placement o a transvenous pacemaker should


not delay trans er or catheterization since a pacemaker may be
HELPFUL TIP:
placed in the cath lab. However, apply an external pacemaker
r emembe t epeat the ECG after th mb t cs t
as required.
p ve that ST e evat ns have es ved. Ev dence suc-
cess u epe us n a te th mb t cs s suggested b :
T e patient requires heparin with the thrombolytic that you
nea sudden and c mp ete e e chest pa n, >70%
choose (and is indicated, by guidelines, or a minimum o
ST e evat n es ut n n the ndex ead sh w ng the
48 hours and pre erably or the duration o the index hospital-
g eatest deg ee e evat n, p us m nus the p es-
ization, up to 8 days or until revascularization is per ormed).
ence epe us n a h thm a.
Question 2.1.12 Which o the ollowing dosing regimens is
the best accepted or use in AMIs?
A er con erring with your closest cath center, you give a A) Enoxaparin 30 mg subcutaneously (SC) every 12 hours.
thrombolytic—and cross your ngers. Un ortunately, the B) Enoxaparin 1 mg/kg SC every 12 hours.
patient develops a new LBBB. In addition, the ECG shows C) Heparin 5,000 units bolus and a drip at 1,000 units per hour.
evidence o a rst-degree heart block (a prolonged PR inter- D) Heparin 100 unit/kg bolus with a drip at 25 units/kg/hr.
val), although the heart rate remains normal at 80 bpm. E) None o the above represents the best dosing option in this
situation.
Question 2.1.11 The proper response to this is to:
A) Insert a Swan–Ganz catheter to monitor central pressures. Answer 2.1.12 The correct answer is “B.” For anticoagulation
B) Insert a temporary pacemaker regardless o the heart rate. in AMI, the dose o enoxaparin is 1 mg/kg SC every 12 hours.
C) Administer atropine to this patient. “A” is incorrect since 30 mg SC every 12 hours is the dose or
D) Administer isoproterenol to this patient. DV prophylaxis, not or anticoagulation. “C” is incorrect. T is
E) Do nothing, other than observe this patient. is the classic way that heparin has been dosed but it is not the
best option listed. “D” is incorrect as well. T e correct dose or
Answer 2.1.11 The correct answer is “B.” For patients with heparin when given with a thrombolytic is 60 units/kg bolus
an AMI, a transvenous pacemaker should be inserted i she (maximum o 4,000 units) with a drip o 15 units/kg/hr (maxi-
develops (1) complete heart block, (2) second-degree heart mum dose o 1,000 units/hr), with rate adjusted to achieve an
block type II (Mobitz II), or (3) new LBBB with rst-degree AV activated partial thromboplastin time (aP ) o 1.5 to two
block. See ables 2-2 and 2-3 or more on arrhythmia and pace- times control ( or 48 hours or until revascularization). T e bot-
makers in the setting o AMI. “A” is incorrect because a Swan- tom line here is that either enoxaparin or heparin can be used in
Ganz catheter will be o no help in arrhythmias. “C” is incor- this setting, and they are more or less equivalent. I you choose
rect because atropine is indicated or symptomatic bradycardia to use heparin, do not use f xed dose heparin but rather weight-
and not or a bundle branch block. “D” is incorrect or the same based dosing. Also, keep in mind that in the United States, it is
reason as “C.” In addition, isoproterenol is arrhythmogenic and likely that most interventional cardiologists would pre er hepa-
is no longer recommended. “E” is incorrect because the patient rin over enoxaparin, due to both amiliarity with dosing as well
may rapidly progress into a complete heart block. O note and as drug pharmacology in the cath lab.

TABLE 2-2 TYPE OF HEART BLOCK ASSOCIATED HELPFUL TIP:


WITH INFARCTION D d u kn w that ST-e evat n Mi, an n t a d se
Ante m ca d a Bund e b anch b cks 30 mg IV enoxaparin (that’s ght— nt aven us)
n a ct n M b tz t pe ii sec nd-deg ee hea t b ck sh u d be g ven w th the first and only the first d se
th se age <75 ea s. The iV d se sh u d be g ven at
in e m ca d a B ad ca d a m:
n a ct n • M b tz t pe i sec nd-deg ee hea t b ck the same t me as the st 1 mg/kg subcutane us d se.
• Th d-deg ee hea t b ck D t!
46 FAMil y MEDiCiNE EXAMiNATio N & Bo Ar D r EViEW

Question 2.1.15 The patient passes her stress test with y


T e patient receives her thrombolytic, enoxaparin, and trans- ing colors (and you pass your Board Examination). Patients
venous pacing, and she is admitted to the hospital to a moni- a ter a myocardial in arction should be routinely discharged
tored bed. You get a call rom the nursing staf 5 hours later. on all o the ollowing medications EXCEPT:
T e rhythm strip shows 3 PVCs per minute. Your patient A) Aspirin.
remains pain ree and is hemodynamically stable. B) Beta-blocker.
C) Continuous nitroglycerin (e.g., patch or isosorbide).
Question 2.1.13 The nurse (who has more than a ew gray D) HMG-CoA reductase inhibitor (“statin”).
hairs) would like an order or lidocaine. Your response is: E) Sublingual nitroglycerin or PRN use.
A) “Do it. Give the lidocaine.”
B) “Give amiodarone—it works better than lidocaine.” Answer 2.1.15 The correct answer is “C.” T ere is no ben-
C) “Give no antiarrhythmic at this point in time.” e t to scheduled nitrates unless needed or a speci c indica-
D) “Check labs including potassium and magnesium.” tion (e.g., recurrent angina). All postmyocardial in arction
E) C and D. patients should be discharged on aspirin, beta-blocker, statin,
nitroglycerin PRN, and an angiotensin-converting enzyme
Answer 2.1.13 The correct answer is “E.” T e use o lidocaine (ACE) inhibitor (i tolerated, o course). Also, per ACC NS E-
in this setting incurs no bene t and is proarrhythmic. T e same ACS guidelines, individuals treated with an ischemia-guided
is true or prophylactic amiodarone, which can cause torsades strategy (vs. early invasive strategy), a P2Y12 inhibitor should
de pointes, albeit at a lower requency versus other antiar- be continued or up to 12 months i there are no contrain-
rhythmics (such as quinidine, procainamide, sotalol, and newer dications (options include clopidogrel 75 mg daily, ticagrelor
Class III antiarrhythmic agents). In the setting o AMI, antiar- 90 mg BID). Finally, per ACC guidelines, all patients with
rhythmics may be indicated only or complex arrhythmias (PVC S -elevation MI or non-S EMI/unstable angina should be
couplets, triplets, nonsustained ventricular tachycardia [<30 discharged on an intensive lipid-lowering regimen with an
seconds], or >10 PVCs per minute). More than 90% o patients HMG-CoA reductase inhibitor (“statin”), such as atorvastatin
will have isolated PVCs in the peri-in arct period, and there is 40 to 80 mg per day.
no association with increased mortality. Correcting hypokale-
mia and hypomagnesemia can help to reduce arrhythmias, and
checking these labs is prudent.
HELPFUL TIP:
in pat ents w th a h st acute c na s nd me
T e patient remains pain ree while in the hospital. She is
(ACS), stat n the ap we s the sk ecu ent Mi,
ready to be discharged 4 days later.
st ke, CAD- e ated death, and the need c na
evascu a zat n. Stat n the ap a te ACS s bene c a
Question 2.1.14 Which o the ollowing tests is the most
n Al l pat ents, nc ud ng th se w th a base ne l Dl
appropriate or this patient prior to discharge?
<70 mg/dl . D scha ge ACS pat ents n a stat n.
A) Coronary angiography.
B) Submaximal stress test.
C) Full Bruce protocol, symptom limited, stress test.
D) Spiral C to assess or coronary artery calci cation. Question 2.1.16 I this patient had a STEMI (or “Q wave” MI),
E) Family Medicine Board Examination. which o the ollowing statements would be TRUE?
A) Patients with a non-S EMI have the same, or perhaps a
Answer 2.1.14 The correct answer is “B.” Submaximal stress bit worse, outcomes long term than do patients with a
testing is considered the standard o care. Patients with a posi- S EMI.
tive submaximal stress test should be re erred or catheter- B) Patients with a non-S EMI have worse in-hospital outcomes
ization. Patients with a borderline stress test can be sent or a when compared with patients with a S EMI.
radionuclide study. Coronary angiography is not routinely rec- C) Unstable angina and non-S EMI can be readily di erenti-
ommended or all patients who have had a myocardial in arc- ated rom one another on presentation.
tion unless they are considered to be at high risk (continued D) None o the above is true.
symptoms, positive screening test such as submaximal stress
test, heart ailure, cardiogenic shock, etc.). “C” is incorrect Answer 2.1.16 The correct answer is “A.” Patients with a
because a symptom-limited, ull-protocol stress test should be non-S EMI actually have the same, or perhaps even slightly
done only 14 to 21 days a er an in raction. Finally, spiral C worse, outcomes long term as do patients with a S EMI. T is
(“D”) to assess or coronary artery calci cation has no role in makes sense; there is still myocardium le to in arct a er a
risk strati cation a er a myocardial in arction . . . their risk is non-S EMI. As to the other answers, patients with a S EMI
100%! Also, keep in mind that the weight o the clinical evidence do have worse in-hospital outcomes, and unstable angina and
avors trans er or early catheterization, especially or higher non-S EMI look similar on ECG with -wave inversion, etc.,
risk patients. “E” is the test YOU are studying or . . . right? Dis- but without the S elevations that are classically seen in a trans-
oriented already? It’s only Chapter 2! mural in arction.
CHAPTER 2 • CAr Dio l o Gy 47

Objectives: Did you learn to . . . medications, but the patient continues to have pain. You con-
• De ne the accu ac the n t a h st , ECG, and abs n the sult a cardiologist who suggests the use o a glycoprotein IIb/IIIa
d agn s s ca d ac d sease n the ED ce? inhibitor.
• r ec gn ze the e and s gn cance ( ack the e ) sk
act s, such as d abetes, am h st , sm k ng, and h pe - Which o the ollowing is true about the glycoprotein IIb/IIIa
tens n, n the dec s n whethe n t t adm t a pat ent inhibitors?
t the h sp ta chest pa n? A) T ey are best used in patients who are not candidates or
• Gene ate a d f e ent a d agn s s chest pa n? P CA and stenting.
• ident the es va us d agn st c tests n the eva uat n B) T ey cause no increase in the rate o intracranial bleeding.
chest pa n? C) T ey are use ul in all groups o patients with ACS.
• T eat a pat ent w th an AMi? D) T ey are most e ective in patients going to P CA and/or
stenting.

QUICK QUIZ: Co r o NAr y CAl CiUM The correct answer is “D.” T e glycoprotein IIb/IIIa inhibi-
tors are most e ective in patients who are undergoing P CA or
stenting. T e GUS O V trial showed NO di erence in 30-day
You are seeing a 47-year-old male patient. His presenting com-
mortality in patients who were not scheduled or catheteriza-
plaint is chest pain. T e chest pain is right sided and is not
tion. T us, “A” is incorrect. “B” is incorrect because glycoprotein
associated with exertion. T e patient looks well conditioned
IIb/IIIa inhibitors do increase rates o intracranial and other
and admits (well, beams . . . the jerk) he exercises daily without
bleeding. “C” is incorrect because patients who have an ACS
any chest pain. On clinical examination, you nd that the chest
that is well-controlled with other drugs (e.g., heparin, metopro-
pain is reproducible on palpation o the anterior chest wall. He
lol, and ASA) are not likely to bene t rom glycoprotein IIb/IIIa
smokes one pack o cigarettes per day and has a blood pressure
inhibitors.
o 135/87 mm Hg, a total cholesterol o 179 mg/dL, and HDL o
30 mg/dL. He takes no medication.
CASE 2.2
Which o the ollowing is RUE regarding the role o a coronary A 53-year-old male with a history o hypertension and smok-
calcium score in this patient? ing, but no amily history o cardiac disease, presents to your
A) Coronary calcium score may be a help ul tool in the evalu- o ce complaining o a chest pain. T e pain is substernal,
ation o this patient in combination with the Framingham radiates to his le arm, and is associated with exertion. T e
risk score. patient notes that this same pain has been going on or the
B) Coronary calcium score is a help ul tool in the evaluation o last 6 months and has not changed at all in duration, inten-
this patient independent o the Framingham risk score. sity, or characteristic. It generally lasts 5 minutes or so and
C) Coronary calcium score is only use ul in evaluating this resolves with rest.
patient once he has had an echocardiogram.
D) Coronary calcium score cannot provide use ul in ormation Question 2.2.1 You tell the patient that:
regarding cardiac unction. A) Without doing any test, you know that the probability that
this pain is cardiac is greater than 85%.
The correct answer is “A.” T e patient presented with a Fram- B) I his ECG in the o ce is normal, his pain is unlikely to rep-
ingham risk score o 13%, which places him in the intermedi- resent cardiac disease.
ate-risk category or 10-year cardiac risk. Coronary calcium C) Even with risk actors, his probability o having CAD with
scores may be use ul in those with an intermediate risk to help “typical angina” is still only 50% or so.
strati y risk (e.g., between 10% and 20% risk by Framingham D) T e only intervention indicated at this point are li estyle
per JAMA. 2010 28;303:1610 and JACC Cardiovasc Imaging. modi cations (e.g., stop smoking) and addressing his cho-
2015;8(5):579–596). T is remains controversial, however. Won- lesterol and hypertension.
der why we are talking about Framingham rather than the more E) It is likely that he has unstable angina.
recent and more popular ACC/AHA 2013 cholesterol guideline
(which we discuss later)? Coronary calcium scores have not (as Answer 2.2.1 The correct answer is “A.” A 50-year-old male
o the time o this writing) been studied using the new ACC/ with “classic” angina symptoms has greater than a 90% prob-
AHA guideline. ability o having CAD. “B” is incorrect because patients with
angina who are pain ree may have a normal electrocardiogram
(as will many patients with active angina or even a myocardial
QUICK QUIZ: Giib/iiia iNHiBiTo r S in arction). T us, his pain could still be cardiac in origin. “C” is
incorrect because, based on demographic data, his risk o CAD
You are seeing a patient in the ED with chest pain. T e ECG is much higher than 50%. “D” is incorrect because he needs a
shows elevated S segments in leads V1, V2, and V3 with recip- urther evaluation and treatment o his chest pain. “E” is incor-
rocal changes in eriorly. You have run through the “standard” rect since this pain represents “stable angina.” T ere has been
48 FAMil y MEDiCiNE EXAMiNATio N & Bo Ar D r EViEW

no change in quality, duration, amount o exertion required to TABLE 2-4 CONTRAINDICATIONS TO EXERCISE
bring on symptoms, etc., eliminating unstable angina as a diag- STRESS TESTING
nosis. Use o tools such as the IMI Risk Score and/or Grace
Abs ute • Acute m ca d a n a ct n w th n 2 da s
Risk Model can assist in estimation o the level o risk and to
c nt a nd cat ns • D ssect ng aneu sm
help guide management decisions (see 2014 AHA/ACC NS E- • r ecent pu m na emb sm
ACS Guideline). • Act ve th mb ph eb t s
• BP > 200/120 mm Hg
• Hem d nam ca s gn cant a h thm as
HELPFUL TIP: • Seve e CHF
Kn w u p etest p bab t ca d ac d sease be e • Seve e a t c sten s s
emba k ng up n test ng chest pa n. Th s va es b • Act ve m ca d t s, pe ca d t s, end -
ca d t s
age and t pe chest pa n. An app x mat n the
• inab t t c mp ete test
p bab t ca d ac d sease s as ws: • Unstab e ang na
Ma e, at p ca ang na: age 30 t 39 = 34%, age 40 t 49 =
r e at ve • l e t bund e b anch b ck
51%, age 50 t 59 = 65%, age 60 t 69 = 72%.
c nt a nd cat ns • M de ate a t c sten s s
Ma e, typica l a ngina : age 30 t 39 = 76%, age 40 t 49 = • H pe t ph c ca d m path
87%, age 50 t 59 = 93%, age 60 t 69 = 94%. • E ect te d stu bance
• H gh g ade AV b ck
Fema e, at p ca ang na: age 30 t 39 = 12%, age 40 t
• Tach a h thm as b ad a h thm as
49 = 22%, age 50 t 59 = 31%, age 60 t 69 = 52%. nc ud ng unc nt ed at a b at n
Fema e, typica l a ngina : age 30 t 39 = 26%, age 40 t
49 = 55%, age 50 t 59 = 73%, age 60 t 69 = 86%.

Answer 2.2.3 The correct answer is “A.” Stress testing is best


You send the patient home on aspirin with a prescription or suited to patients with an intermediate pretest probability o
sublingual nitroglycerin or PRN use and arrange or a stress cardiac disease (between 25% and 75%). “B” and “C” are incor-
test. rect since patients with a high risk o cardiac disease should go
directly to another study, such as thallium testing and stress
Question 2.2.2 All o the ollowing are considered absolute echocardiography. “D” and “E” are incorrect since these are
contraindications to exercise stress testing EXCEPT: not the best groups in whom to use exercise stress testing;
A) Le bundle branch block (LBBB). there will be a greater proportion o alse-positive results in
B) Presence o severe heart ailure. these low-risk patients. Exercise stress testing in these groups
C) Critical aortic stenosis. is best used to allay patient ears that they do not have cardiac
D) Myocarditis. disease, not to prove they do have cardiac disease. However, a
E) Unstable angina. alse-positive stress test may lead to other unnecessary inva-
sive testing!
Answer 2.2.2 The correct answer is “A.” An LBBB is a rela-
tive—not absolute—contraindication to stress testing. In the You decide to do an exercise stress test on this patient. It
setting o LBBB, there are already repolarization abnormalities turns out to be negative.
that limit the use ulness o the stress test. One should add an
imaging modality, such as myocardial per usion scanning, in Question 2.2.4 Your next step is to:
cases o LBBB. T e same holds or any baseline ECG pattern A) Reassure the patient that he does not have cardiac disease.
that would inter ere with S segment interpretation (baseline B) Suggest a chest C scan to rule out possible aortic aneu-
S changes, LBBB, LVH with repolarization changes involv- rysm.
ing the S - wave, intraventricular conduction delay, paced C) Schedule the patient or another cardiac test such as stress
rhythm, preexcitation, or S - changes due to digoxin therapy); echocardiogram, exercise myocardial per usion test, or
the stress test SHOULD include an imaging modality. T e rest angiography.
are all “absolute” contraindications to exercise stress testing. See D) Schedule the patient or endoscopy to rule out gastroesoph-
able 2-4 or a list o contraindications to stress testing. ageal disease as a cause o these symptoms.
E) Start an anxiolytic to treat the panic disorder, which is the
Question 2.2.3 Exercise stress testing is best suited to which underlying cause o his chest pain.
group o individuals?
A) Men with an intermediate probability o cardiac disease. Answer 2.2.4 The correct answer is “C.” T is patient who is
B) Women with a high risk o cardiac disease. in his 50s and who has a “classic” history or angina has greater
C) Men at a high risk o cardiac disease. than a 90% pretest probability o cardiac disease. T us, it is
D) Men at a low risk o cardiac disease. likely that the negative stress test is a alse negative. T us, a
E) Women with a low risk o cardiac disease. stress test probably should not have been done in this patient
CHAPTER 2 • CAr Dio l o Gy 49

in the rst place, since a negative test just leads to urther stressing this patient. Dobutamine is an acceptable method
testing (as would have a positive test, probably resulting in o chemically stressing those on theophylline or ca eine
angiography). For this reason, “A” is incorrect. “B,” “D,” and “E” (like us).
are incorrect. Initiating evaluation and management or another
cause o chest pain is premature, since we still have not proven Question 2.2.7 The patient’s thallium stress test shows an
that this patient does not have cardiac disease. anteroapical nonreversible (“ xed”) de ect. The best inter
pretation o this is that it indicates:
You are considering whether to do a thallium stress test or a A) Attenuation arti act rom breast tissue.
stress echocardiogram. B) Prior myocardial in arction.
C) Angina.
Question 2.2.5 Which o the ollowing is true? D) Anomalous cardiac circulation.
A) Stress echocardiography is more sensitive or cardiac disease E) It is not signi cant and there ore adds no value to this test.
than is a thallium test.
B) Stress echocardiography is more speci c than is stress thal- Answer 2.2.7 The correct answer is “B.” A nonreversible
lium. (“ xed”) de ect suggests prior myocardial in arction. A revers-
C) T allium testing is more speci c or cardiac disease than is ible de ect suggests inducible ischemia. “A” is incorrect since
stress echocardiography. breast attenuation occurs mostly in women (but can also be
D) None o the above is true. seen in obese males, which is part o the rationale or per orm-
ing PE in patients with BMI > 35 kg/m 2 since PE uses a
Answer 2.2.5 The correct answer is “B.” Stress echocardiogra- higher-energy isotope to mitigate so tissue attenuation). “C”
phy is more speci c or cardiac disease than is thallium test- is incorrect since angina would likely be mani ested by a revers-
ing. Alternatively, thallium testing is more sensitive. able 2-5 ible de cit.
summarizes this data. Remember that positive and negative
predictive values o these tests will vary depending on the pre- Since a reversible de ect was not ound on the thallium
test probability o disease in the patient and severity o disease. stress test, you conclude that there is no myocardium cur-
Numbers given above are overall. rently at risk. However, the patient continues to have chest
pain and now at an increasing requency with less exer-
You decide to send the patient or a thallium stress test. tion. He is asymptomatic when he presents to your o ice.
However, since his exercise capacity is limited, you choose to He was noted at the last visit to have an elevated glucose at
stress him chemically. T e patient is taking theophylline or 350 mg/dL.
chronic obstructive pulmonary disease (a pox on the doctors
still prescribing that drug!). Question 2.2.8 What is the next step in the evaluation or
treatment o this patient?
Question 2.2.6 The LEAST desirable method o stressing A) Stress echocardiogram to document what segments are
this patient is: involved.
A) Adenosine. B) Start the patient on insulin to control his blood sugars.
B) Dobutamine. C) Proceed directly to cardiac catheterization.
C) Dipyridamole. D) Since there were no reversible de cits on thallium stress,
D) All o the above are equally acceptable methods o chemi- schedule the patient to see a gastroenterologist.
cally stressing this patient. E) Give a trial o NSAIDs to help di erentiate chest wall pain
E) Neither A nor C is desirable. rom other causes.

Answer 2.2.6 The correct answer is “E.” heophylline (and Answer 2.2.8 The correct answe r is “C.” “A” is incorrect
ca eine) interacts with both adenosine and dipyridamole, since we already have done a noninvasive test. We already know
attenuating their e ect; thus, neither is a good choice or what segment was previously in arcted, as noted on the thal-
lium stress test. “B” is incorrect or two reasons. First, address-
ing his diabetes will not address the immediate problem o what
TABLE 2-5 OVERALL SENSITIVITY AND SPECIFICITY you must presume is unstable angina. Second, insulin is not
OF NONINVASIVE CARDIAC TESTING necessarily the rst drug to use in this patient who presumably
Sensitivity Speci city has type 2 diabetes. Certainly, the blood glucose needs to be
(%) (%) addressed and so does the chest pain. Which is going to kill him
rst? “D” is incorrect. T e sensitivity o thallium testing is in the
Exe c se st ess test ng 45–68 77
88% range (see able 2-5), so it will miss 12% o disease. T us,
Tha um st ess test ng (SPECT) 88 77 we still have not proven in this high-risk patient that he does
not have treatable cardiac disease causing his chest pain. “E” is
St ess ech ca d g aph 76 88
incorrect or the same reason.
50 FAMil y MEDiCiNE EXAMiNATio N & Bo Ar D r EViEW

T e patient has a catheterization done that shows three-vessel HELPFUL TIP:


disease including le main CAD. T e cardiologist calls you A p ea m u ed t s. D n’t c nt nue c p d g e ,
with the report the next day and suggests P CA with stent- etc., uts de the t me ame n wh ch the have been
ing, since, in his opinion, “this is the best modality or dia- sh wn t be use u (3–6 m nths a bea meta stent,
betics and diabetics are high-risk candidates when it comes 1 ea a d ug e ut ng stent) un ess the e s ng ng
to surgery.” ACS. The b eed ng sk s nc eased n m n ma ben-
e t. C nt nue asp n nde n te , c u se (N Eng J Med.
Question 2.2.9 Your opinion is that: 2014;371:2225–2226). F pat ents n wa a n an the
A) Patients generally have better outcomes in terms o con- a ant c agu ant, c ns de at n sh u d be g ven t e-
trol o angina with stenting when compared with coronary duc ng the du at n t p e the ap ( a ant c agu ant +
artery bypass gra ing (CABG). ASA + c p d g e , examp e) due t b eed ng sk w th-
B) Diabetic patients do particularly well with stenting when ut s gn cant bene t. D p ne the ant p ate et d ugs.
compared with CABG. H weve , the pt mum du at n the ap has n t been
C) Medical control o symptoms is indicated as the best man- estab shed. it ma be sa e t st p t p e the ap as ea as
agement in this diabetic patient with three-vessel disease. 4 t 6 weeks a te a DES (J Am C Ca d . 2015;65:1619).
D) You would like to send this patient or CABG.
Your patient has a CABG and comes into your o ce com-
Answer 2.2.9 The correct answer is “D.” T is patient should
plaining o chest pain and ever 3 weeks a er the surgery. He
probably have surgery or his three-vessel disease because dia-
has had the pain and ever or 4 days and does not seem to
betic patients generally have worse outcomes with stenting than
be getting any better. He has no cough, no sputum produc-
do nondiabetic patients in terms o angina relie and need or
tion, and the pain seems to be worse when he breathes or lies
repeat revascularization. “A” is incorrect because a proportion
down. He reports no dyspnea and has 97%oxygen saturation
o patients with stents have to go on to have an open CABG due
on room air. T e wound rom the surgery is well healed, and a
to in stent restenosis or incomplete revascularization with per-
chest radiograph shows no evidence o abnormalities.
cutaneous revascularization. “B” is incorrect. Diabetic patients
have a much higher rate o in stent restenosis or secondary
Question 2.2.10 Which o these studies is LEAST likely to be
occlusion (meaning a narrowing/stenosis somewhere else in
abnormal in this patient?
the diseased vessel). “C” is incorrect. T e indications or CABG
A) ECG.
are signi cant le main CAD (>50%), three-vessel disease with
B) V/Q scan.
evidence o LV dys unction (ejection raction <50%), or ori-
C) Echocardiogram.
gin/proximal LAD/LCX disease (le main equivalent). T is
D) Sedimentation rate.
patient has le main vessel disease and thus medical control
is NO the best option or this patient. He is also young and, Answer 2.2.10 The correct answer is “B.” A V/Q scan is not
rom the available data, appears to be a good surgical candi- likely to be positive in this patient. T is patient is unlikely to have
date. T ere ore, CABG is the guideline-recommended option a PE given the duration o symptoms, the act that the patient has
is this patient. chest pain that worsens with inspiration ( ound in only 59% o
those with PE), ever, absence o dyspnea, and has normal oxy-
gen saturation. Certainly, this could still be a PE, but it would
HELPFUL TIP: be less likely than other, more plausible, explanations. T e most
D ug-e ut ng stents (DES) dec ease ve a e cc us n likely diagnosis in this patient, given the lack o other symptoms,
ates n c mpa s n t ba e meta stents (BMS). W men is postpericardotomy syndrome. T is is similar to Dressler syn-
wh have mu t p e stents and mu t -vesse d sease a e drome, which occurs a er a myocardial in arction and presents
at a h ghe sk esten s s, as a e pat ents w th a sma with ever and chest pain several days to weeks a er the inciting
p ststent ng umen s ze. Ea e cc us n sec nda event. T e white blood count is o en elevated, as is the sedimen-
t th mb s s s h ghe w th DES ve sus BMS, and the tation rate. T e ECG can be help ul as can an echocardiogram.
m ta t sk due t stent th mb s s s h gh ( ange 20–
45%). Th s s because t takes the b d nge t c ve You obtain an ECG on this patient that shows a pattern con-
these stents w th nt ma ( .e., end the a zat n). Thus, sistent with pericarditis.
c p d g e , t cag e p asug e (n t u av te)
1 ea PLUS asp n e sh u d be used n pat ents Question 2.2.11 Which o the ollowing patterns can be
wh have a d ug-e ut ng stent nse ted. F ba e meta seen in a patient with pericarditis?
stents, the ec mmendat n s t use ne the ant - A) Di use S segment elevation.
p ate et agents 3 to 6 months PLUS asp n e. B) Normal ECG.
(N te: the sk stent th mb s s w th a ba e meta C) LBBB.
stent s h ghest n the st 14–30 da s.) D) A and B.
E) All o the above.
CHAPTER 2 • CAr Dio l o Gy 51

Answer 2.2.11 The answer is “D.” Both di use S segment


elevations and a normal ECG can be seen with pericarditis. He gets admitted to a cardiac inpatient bed and you give a
T e initial ECG is only 80% sensitive or pericarditis. Small bolus o IV saline. Despite this, he remains dyspneic with
(low voltage) QRS complexes or electrical alternans can also elevated neck veins and has a pulsus paradoxus o 14 mm Hg
be seen i there is a pericardial e usion. “C” is incorrect since (normal <10 mm Hg).
bundle branch blocks have nothing to do with pericarditis. You
will have a chance to look at an ECG o pericarditis later in the Question 2.2.14 The next step or this patient is:
chapter. Note: PR depression accompanying di use S eleva- A) Change the patient to steroids rom indomethacin.
tion +/– sinus tachycardia may also be seen in pericarditis. B) Per orm a pericardiocentesis.
C) Start a positive inotrope (e.g., dopamine) to improve right
A er a complete history, physical examination, ECG and heart unction.
echocardiogram, you determine that he has pericarditis. D) Start an a erload reducer to reduce cardiac demand.

Question 2.2.12 Which o the ollowing drugs might be Answer 2.2.14 The correct answer is “B.” T e patient is clearly
help ul in this patient? not doing well i he is getting more short o breath and not
A) Heparin. responding to your treatment. T e pulsus paradoxus is 14 mm Hg.
B) War arin. T is is indicative o possible cardiac tamponade, but it may be
C) Furosemide. seen in constrictive pericarditis, severe asthma, or anything else
D) Indomethacin. that reduces right heart lling (e.g., tension pneumothorax).
T is patient’s clinical picture is consistent with decompensated
Answer 2.2.12 The correct answer is “D.” Didn’t we just tell cardiac tamponade, and drastic action is indicated to relieve the
you not to use NSAIDs in patients who have CAD? Well, yes. symptoms o right heart ailure. T e de nitive treatment is peri-
However, in this particular case, an NSAID is indicated. Vari- cardiocentesis. “A” is incorrect because more drastic action is
ous doctors weighing risks and bene ts o NSAIDs in this required. You would be correct to change the patient to predni-
case might come to di erent conclusions, but o the available sone or to add colchicine. Colchicine is being increasingly used
options, “D” is the only one which is a treatment or pericarditis. or pericarditis. “C” is incorrect since an inotrope will do little
o treat pericarditis, you can use aspirin, another NSAID, ste- to help this problem. “D” is incorrect or two reasons: the rst
roid, or colchicine. Generally, indomethacin or aspirin are con- is that this is a right heart problem and reducing a erload (sys-
sidered rst-line drugs with steroids being reserved or those temic vascular resistance) will not help the right heart, which
who ail NSAID therapy. Colchicine is becoming increasingly pumps against pulmonary resistance; second, most drugs that
popular as a second-line treatment a er an NSAID. Do not use reduce systemic vascular resistance will also decrease preload to
anticoagulation, either heparin or war arin, in patients with some degree, worsening the symptoms o tamponade.
pericarditis. T is can cause bleeding into the pericardial space
and tamponade. T us, “A” and “B” are incorrect. “C” is incorrect
HELPFUL TIP:
because urosemide will likely make this patient worse. Patients
Be awa e that up t 25% pat ents w th ca d ac tam-
with increased pericardial pressures are dependent on circulat-
p nade w NOT dem nst ate an e evated pu sus pa a-
ing preload volume in order to ll the right heart. Decreasing
d xus. S , a n ma pu sus pa ad xus d es n t u e ut
the preload may cause dyspnea in this patient. Steroids are not
ca d ac tamp nade.
rst line and are typically reserved or recurrence a er treat-
ment with NSAIDs and colchicine.
HELPFUL TIP:
T e patient returns the next day and is now eeling short Measu ng pu sus pa ad xus: Pump up the BP cu
o breath. On examination, you notice JVD and peripheral t g eate than the s st c BP. De ate de ate BP cu
edema. s w and sten the st K tk s und hea d
only du ng exp at n; c nt nue s w de at ng unt
Question 2.2.13 The best initial treatment o this patient is: u hea the K tk s unds w th nsp at n as we .
A) Furosemide. The d e ence s the pu sus pa ad xus.
B) Nitroglycerin.
C) IV saline.
D) Morphine. You per orm a pericardiocentesis and the patient gets better.
O course, a good outcome never protected anyone rom a
Answer 2.2.13 The correct answer is “C.” T is patient is in
lawsuit . . .
“pure” right heart ailure secondary to possible cardiac tampon-
ade (eek! Let’s get him a cardiac inpatient bed!). He is preload Objectives: Did you learn to . . .
dependent. T e treatment is to increase his preload by using IV • Eva uate a pat ent w th t p ca ang na chest pa n?
saline. All the other options reduce the preload and will worsen • Desc be the test cha acte st cs va us t pes n n nva-
this patient’s symptoms. s ve ca d ac test ng?
52 FAMil y MEDiCiNE EXAMiNATio N & Bo Ar D r EViEW

• Bec me am a w th the nte p etat n n n nvas ve The correct answer is “D.” According to JNC 8, the goal or a dia-
ca d ac test ng? betic patient (or any patient age 30–59 years, including those with
• r ec gn ze va us nd cat ns PTCA w th stent p acement chronic kidney disease) is <140/90. ighter control does not seem
ve sus CABG? to con er any bene t. T e BP goal in those over age 60 is <150/90.
• Unde stand the ph s g , p esentat n, and t eatment T ese goals are consistent with the best current evidence at the
p stpe ca d t m s nd me? time JNC 8 was published. In act, tight blood pressure control in
• T eat pe ca d t s and ca d ac tamp nade? diabetics increases mortality (JAMA. 2010;304(1):61–68).

HELPFUL TIP:
QUICK QUIZ: CAr DiAC CT SCAN r ec mmendat ns t eat ng h pe tens n n the
e de va a tt e based n the gan zat n pub sh-
When planning or treatment or diagnosing (i.e., “ruling out” ng the g a s. F pat ents ve age 80 ea s, the e s
MI/acute ACS) based on a cardiac C angiogram (CC A), un ve sa ag eement (ACC/AHA, JNC 8, ASH, ESH/ESC)
which o the ollowing is true? that <150/90 sh u d be the g a . F pat ents age
A) CC A is associated with better long-term clinical outcomes 60 t 79 ea s, JNC 8 ec mmends a g a <150/90
than is cardiac per usion (thallium) scanning. wh e the the gan zat ns ec mmend a g a
B) T e radiation exposure or C angiogram is less than that o <140/ 90—but w th a s st c p essu e ema n ng g eat-
thallium scanning. e than 115 mm Hg.
C) T e downstream radiation exposure a er thallium scanning
is less than that o C angiogram.
D) T e C angiogram is best done in those with a pulse o 70 to CASE 2.3
80 bpm.
E) A creatinine o 1.6 mg/dl is a contraindication to the use o A 24-year-old male presents to your clinic with a 50-hour
C angiogram because o the contrast load. history o an irregular heart rate. He is generally well but
has a history o hypertension (too many super-jumbo burg-
ers . . . with bacon . . . he’s been “supersized”), which he has
The correct answer is “B.” T e radiation exposure o CC A been trying to control with exercise and diet (he switched
is less than that o thallium scanning, both initially and down- to to u burgers yesterday). T ere is no prior history o car-
stream (thus, “C” is wrong). More patients who have thallium diac disease or palpitations. He did “have a bit to drink” cel-
scanning go on to angiogram and additional radiation ex- ebrating . . . well, whatever, just celebrating . . . who needs
posure. “D” is incorrect. T e slower the heart rate, the better a reason! He was embarrassed about his drinking and thus
the images (within reasonable limits, o course . . . . as pulse o waited 2 days to seek care. T ere is no amily history o heart
12 BPM isn’t so good). So, patients are o en pretreated with disease and the patient does not smoke. Vital signs reveal an
metoprolol to slow the heart rate be ore CC A. “E” is also in- irregular pulse o 130 bpm and a blood pressure o 160/100
correct. It turns out that IV contrast is less o a actor in renal mm Hg. T e patient is a ebrile and has normal respirations.
ailure than we have thought (Radiology. 2014;273(3):714). I He has no heart murmur. T e ECG is shown below (Fig. 2-2).
you eel the patient really needs the C and has a creatinine o
1.7 mg/dl or less, go ahead with the C . Question 2.3.1 The most appropriate diagnosis is:
What about answer “A”? It turns out that even though CCTA A) Multi ocal atrial tachycardia (MA ).
is better at identi ying cardiac disease (CCTA is more sensi- B) Wandering atrial pacemaker.
tive) and leads to more interventions than is thallium scanning C) Atrial brillation.
and echo, its use is not associated with better outcomes. T is is D) Ventricular tachycardia.
true even though more patients who have had a CC A go on to E) Accelerated junctional rhythm.
have revascularization (N Engl J Med. 2015 Mar 14). Feel ree to
do that thallium test . . . . Answer 2.3.1 The correct answer is “C,” atrial brillation.
T is is characterized by the lack o P waves and an irregularly
irregular rhythm. “A” and “B” are incorrect. While both MA
QUICK QUIZ: HyPEr TENSio N and a wandering atrial pacemaker are irregularly irregular, both
have P waves. “D” is incorrect. Ventricular tachycardia is a wide
According to the members o the Eighth Joint National complex tachycardia and is regular. “E” is incorrect. While there
Committee (JNC 8), the blood pressure goal in a patient with are no P waves in an accelerated junctional rhythm, it should be
diabetes is: a regular, organized rhythm.
A) <100/50 mm Hg.
B) <110/70 mm Hg. Question 2.3.2 What is the most likely cause o this patient’s
C) <130/80 mm Hg. dysrhythmia?
D) <140/90 mm Hg. A) Congenital prolonged Q syndrome.
E) None o the above. B) Hypertrophic cardiomyopathy.
CHAPTER 2 • CAr Dio l o Gy 53

FIGURE 2-2. ECG pat ent n quest n 2.3.1.

C) Alcohol. brillation. Certainly, stroke and other intracranial injuries can


D) Marijuana use. be associated with arrhythmias. However, these are generally
E) Ischemic cardiac disease. isolated PVCs. Stroke may also be associated with heart ailure
and ischemic changes on the ECG, but it is rarely an isolated
Answer 2.3.2 The correct answer is “C.” T e most likely cause cause o atrial brillation. Valvular heart disease, hyperthyroid-
o atrial brillation in this 24-year-old is alcohol. T is is also ism, heart ailure, and PE are all causes o atrial brillation.
known by the moniker “holiday heart.” It occurs a er episodes Valvular heart disease, heart ailure, and PE all have a similar
o signi cant alcohol intake. T e underlying mechanism is not mechanism: stretching o the atrium leading to atrial irritability.
known, but alcohol is known to be cardiotoxic at higher vol- Atrial brillation is ound in 10% to 20% o those with hyper-
umes which varies per individual. “A” is incorrect because pro- thyroidism, especially in the elderly.
longed Q typically causes polymorphic ventricular tachycar-
dia (torsades de pointes). “B,” hypertrophic cardiomyopathy, is T e patient con des that he was indeed at a bachelor party
unlikely since the patient has never had a murmur, and hyper- several days ago (so that was what he was celebrating) and
trophic cardiomyopathy generally presents with signs o aortic had a bit too much to drink. T is is quite unusual or the
outlet obstruction (syncope or angina with exercise) although patient. He generally drinks 2 to 3 beers per week, but on
a subset o patients do not have obstructive physiology. “D” is this particular night had 12 or more (hmm . . . we’re wonder-
incorrect because marijuana is not implicated in causing atrial ing). T e patient’s pulse increases to 160 bpm, but he remains
brillation, and “E” is incorrect because a patient who is 24 asymptomatic.
years old is unlikely to have ischemic cardiac disease.
Question 2.3.4 The INITIAL goal or this patient with
Question 2.3.3 Other states that can cause atrial brillation 50 hours o atrial brillation is:
include all o the ollowing EXCEPT: A) Anticoagulation.
A) Valvular disease, especially mitral disease. B) Immediate cardioversion (DCCV).
B) Hyperthyroidism. C) ransesophageal echocardiogram to rule out vegetations.
C) Stroke. D) Rate control.
D) Heart ailure. E) Blood pressure lowering
E) Acute PE.
Answer 2.3.4 The correct answer is “D.” Since this patient has
Answer 2.3.3 The correct answer is “C.” Stroke does not had >48 hours o atrial brillation, rate control is the goal. I
generally cause atrial brillation but can be a result o atrial the onset o atrial brillation is indeterminate or >48 hours,
54 FAMil y MEDiCiNE EXAMiNATio N & Bo Ar D r EViEW

one should withhold cardioversion. For those with <48 hours o provided. Some physicians will start heparin or low-molecular-
atrial brillation, cardioversion is a viable option. Anticoagula- weight heparin at the same time as initiation o war arin; how-
tion is also important to initiate while attempting to achieve rate ever, the combination is not strictly necessary in patients with
control. See more on whom to anticoagulate below. non-valvular atrial brillation. T is scenario is much di erent
than when patients have a PE/DV . For non-valvular atrial
T e patient’s heart rate remains elevated at 160 bpm with brillation, starting war arin alone is su cient. “B” is incor-
occasional orays into the 170 bpm range. rect. T ere is no need to bridge patients starting rivaroxaban
with heparin; rivaroxaban’s anticoagulant e ect is “immediate”
Question 2.3.5 Which o the ollowing is the best drug to and there is no transient hypercoagulable state. However, note
administer to this patient? that since he gave a history o an “irregular heart beat or 50
A) Digoxin. hours” on presentation, he has now presumably been in atrial
B) Lidocaine. brillation or >48 hours (approximately 74 hours at this point),
C) Amiodarone. so immediate cardioversion is contraindicated (see next ques-
D) Adenosine. tion). Heparin (“C”) or aspirin (“D”) alone would not be the
E) Diltiazem. best choices in this case.

Answer 2.3.5 The correct answer is “E.” “A” is incorrect. Question 2.3.7 How long must you wait a ter the patient
Digoxin will be o limited use since it takes at least 30 minutes has been anticoagulated be ore cardioverting him?
to have an e ect. It can be used in those with atrial brillation A) 1 week.
secondary to heart ailure but will still not signi cantly help B) 3 weeks.
with rate control—especially in younger patients with robust C) 3 months.
sympathetic tone. “B” is incorrect because lidocaine is indicated D) Only until therapeutic on war arin.
or a wide complex tachycardia. “C” is incorrect. Amiodarone E) T ere is no need to wait be ore cardioversion in this patient.
will work as a treatment o atrial brillation but is a second-line
drug because it can cause torsades de pointes. It can be used Answer 2.3.7 The correct answer is “B.” I the patient has been
in patients with atrial brillation and congestive ailure, where in atrial brillation or more than 48 hours, one should wait
verapamil or diltiazem might be contraindicated. Amiodarone or cardioversion until the patient has been anticoagulated or
is considered to be advantageous in maintaining sinus rhythm, 3 weeks with war arin (target INR 2–3) or standard dose riva-
especially when given as pretreatment prior to cardioversion, roxaban (Xarelto), apixaban (Eliquis), or dabigatran (Pradaxa).
but is not as e cacious in prompt heart rate control due to I atrial brillation has been present or less than 48 hours, you
longer onset o action. “D” is incorrect. Adenosine is ultra- can proceed directly to cardioversion. Some physicians may
short-acting, blocks the AV node, and can be used to convert a order a transesophageal echocardiogram to assess or throm-
paroxysmal supraventricular tachycardia (PSV ) or slow down bus ormation be ore cardioversion; this approach is acceptable.
the rate o the arrhythmia temporarily i you are not sure what T e patient will also need anticoagulation or 4 weeks a er
the diagnosis is (e.g., a rapid atrial f utter vs. PSV ). However, success ul cardioversion.
adenosine will not reduce the ventricular rate in atrial bril-
lation since atrial brillation does not require the AV node to
propagate. A beta-blocker could also be used in this situation. HELPFUL TIP:
r va xaban, ap xaban ( act Xa nh b t s) and dab -
Being the astute clinician that you are, you realize that 50%o gat an (a d ect b n nh b t ) a e a app ved use
atrial brillation will spontaneously convert to normal sinus n n n-va vu a at a b at n. The ch ce d ugs
rhythm, especially i present <72 hours. A transthoracic s up t u. The e s tt e c n ca d e ence between
echocardiogram was per ormed and did not demonstrate any utc mes w th wa a n, ap xaban and dab gat an (a -
structural heart disease. T us, you choose to give verapamil th ugh wa a n s ess expens ve, even when check ng
and watch the patient. At 24 hours, he still is in atrial brilla- iNr s). The e ma s me s ght c n ca advantage t va-
tion, although the rate is controlled well with verapamil and xaban. But, the e s n c mpe ng ev dence t av
he is now normotensive. ne ve the the . We n w have a eve sa agent
dab gat an but n t the the s (as Feb ua , 2016).
Question 2.3.6 I the patient desires cardioversion, the next The sh u d be ava ab e s n, th ugh. y u sh u d
step in the management o this patient is: av d ed xaban (Sava sa). The utc me data sn’t as
A) Start war arin. g d as w th the act Xa nh b t s.
B) Start heparin and rivaroxaban at the same time.
C) Start heparin.
D) Start aspirin. Question 2.3.8 All o the ollowing can be used to cardiovert
atrial brillation EXCEPT:
Answer 2.3.6 The correct answer is “A.”We may want to cardio- A) Ibutilide.
vert this patient in the uture. “A” is the most reasonable option B) Electrical cardioversion.
CHAPTER 2 • CAr Dio l o Gy 55

C) Quinidine. His atrial brillation has not been addressed since it was
D) Digoxin. picked up by the surgeon at a pre-op visit. His heart rate is
E) Procainamide. 80 bpm when you see him, his rhythm is irregularly irregular,
and he has no signs o heart ailure.
Answer 2.3.8 The correct answer is “D.” Digoxin does not
work to cardiovert atrial brillation. Digoxin may acilitate Question 2.4.1 Which o the ollowing options would be
cardioversion in patients with heart ailure by reducing atrial appropriate or this patient?
stretching. However, it does not convert atrial brillation. All A) Anticoagulate the patient with war arin and allow him to
o the other answers are correct. Because o potential induction stay in atrial brillation.
o arrhythmias with the other agents, electrical cardioversion B) Place the patient on aspirin and allow him to stay in atrial
is becoming the pre erred method o restoring normal sinus brillation.
rhythm. C) Give digoxin to cardiovert the patient.
D) Strongly suggest cardioversion to this patient since sus-
Objectives: Did you learn to . . .
tained normal sinus rhythm yields the best long-term
• r ec gn ze the c n ca and ECG p esentat n at a
outcomes.
b at n?
E) Add urosemide to prevent the development o heart ailure
• Use ate-c nt ng d ugs t t eat a pat ent w th at a
and edema.
b at n?
• App p ate emp ant c agu at n n at a b at n
Answer 2.4.1 The correct answer is “B.” T is patient’s CHA2-
a pat ent unde g ng ca d ve s n?
DS2-VASC (see able 2-6A) score is “1” allowing him to take
• ident app p ate s tuat ns ca d ve s n at a
aspirin rather than being ully anticoagulated with oral antico-
b at n?
agulants in the presurgical setting. A CHA2-DS2-VASC score o
“1” corresponds to what we used to call “lone atrial brillation.”
It is reasonable to allow the patient to remain in atrial bril-
QUIZ: ANTiHyPEr TENSiVE AGENTS
lation, as long as they are rate-controlled with the knowledge
that antiplatelet agents alone or in combination are in erior to
A 70-year-old male complains o erectile dys unction and oral anticoagulants or stroke prophylaxis. Outcomes o patients
requests sildena l (Viagra) or erectile dys unction, which you who stay in atrial brillation and are given appropriate therapy
believe is secondary to vascular disease. are the same (or a bit better) than in patients in whom one tries
to maintain sinus rhythm with drugs such as amiodarone, etc.
Which o the ollowing antihypertensive drugs can cause T us, “D” is wrong.
prolonged hypotension when used with sildena l?
A) Peripheral alpha-blockers.
B) Calcium channel blockers (CCBs).
C) An ACE inhibitor. TABLE 2-6A CALCULATING THE
D) Diuretics. CHA2DS2-VASc SCORE
E) Beta-blockers.
Criteria Points

The correct answer is “A.” T e peripheral alpha-blockers C ngest ve Hea t Fa u e 1


(doxazosin, prazosin, and tamsulosin) can cause symptomatic H pe tens n (t eated 1
hypotension when combined with sildena l or other drugs o ab ve 140/90 mm Hg)
this class (Cialis [tadala l], Levitra [vardena l]). T is hypoten-
Age >75 2
sive e ect is more severe when these drugs are combined with
a nitrate. Nitrates should not be administered within 24 hours D abetes 1
(or longer in patients with renal or hepatic dys unction) o these
St ke, TiA 2 Score and Risk of CVA
drugs, as the combination has reportedly resulted in strokes.
th mb emb c d sease ( n %/ )
None o the other drugs (“B”–“E”) cause this hypotensive e ect
2 = 2.2%/
when combined with sildena l.
3 = 3.2%/
4 = 4%/
CASE 2.4 5 = 7%/
6 = 10%/
A 55-year-old male with a history o newly identi ed atrial
brillation is re erred to you or “medical clearance” or sur- Vascu a d sease (CAD, 1
PAD, a t c p aque)
gery. He has a history o hypertension and hypercholesterol-
emia (calculate his CHA2-DS2-VASC score, see able 2-6A). Age 65–74 1
He has normal cardiac unction otherwise with a normal
Sex Categ Fema e 1
ejection raction and no valvular disease on echocardiogram.
56 FAMil y MEDiCiNE EXAMiNATio N & Bo Ar D r EViEW

TABLE 2-6B TREATMENT OF ATRIAL FIBRILLATION T e 75-year-old patient has his surgery and returns to your
BASED ON THE CHA2DS2-VASc SCORE clinic or a postoperative check-up 1 month a er his surgery.
CHA2DS2 VASc Score Recommended Treatment You check his INR and it is noted to be 5.2. T ere is no active
0–1 (ma e) Asp n 81–300 mg/da (0.5–0.7% bleeding.
st ke sk/ )
Question 2.4.3 The most appropriate action at this point
1 ( ema es) Asp n 81–300 mg/da (0.1–0.2%
is to:
st ke sk/ )
A) Hospitalize the patient or observation since he is at a high
(N te that w men aut mat ca have a
sc e 1 based n gende ) risk o bleeding.
B) Give the patient 5 mg o vitamin K orally.
2 g eate , ma e Ant c agu at n w th wa a n ( the C) Give the patient 2 units o resh rozen plasma to reverse his
ema e ant c agu ant)a
anticoagulation.
a
Xa e t ( va xaban), E qu s (ap xaban) and P adaxa (dab gat an).
D) Hold the next war arin dose and reduce the maintenance
dose.
E) A, B, and C.

HELPFUL TIP: Answer 2.4.3 The correct answer is “D.” T e risk o bleeding
H w sh u d u app the CHA2-DS2-VASC sk sc e t in a relatively healthy patient with an INR o 5.2 is very low.
the ap ? Check ut Tab e 2-6B. T us, simply holding the next one to two doses o war arin and
reducing the maintenance dose o war arin is appropriate. “A”
is incorrect because the patient does not need hospitalization.
T e patient returns at age 75. He is older. You, however, have “B” is incorrect because it will be di cult to reanticoagulate the
not aged a day because doctors are immortal, right? He is now patient a er vitamin K is administered. “C” is incorrect because
hypertensive, has diabetes, and he needs surgery to remove there is no active bleeding.
his gallbladder. His CHA2-DS2-VASC score is now 4 and he is
on anticoagulation war arin. T e patient misunderstands your instructions and takes an
extra dose o war arin that evening and or the next 2 days.
Question 2.4.2 Which o the ollowing approaches is the He returns to your clinic and his INR is now 13.
best or controlling his anticoagulation given that he needs
surgery? Question 2.4.4 What is the best option or therapy at this
A) Stop the war arin several days be ore surgery to allow his point?
INR to normalize. Restart the war arin a er surgery. A) Vitamin K 5 mg IV × 1.
B) Hospitalize the patient a couple o days ahead o time and B) Fresh rozen plasma.
start heparin. T en stop his war arin. Restart the war arin C) Vitamin K 5 mg PO × 1.
a er surgery. D) Vitamin K 10 mg IV × 1.
C) Use low-molecular weight heparin at home and stop the
war arin once this is started. Restart the war arin a er sur- Answer 2.4.4 The correct answer is “C.” Giving this patient
gery. 5 mg o PO vitamin K is the best solution. T is has been ound
D) Stop the war arin several days be ore surgery to allow his to reduce the INR while still allowing the patient to be anticoag-
INR to normalize. Start heparin a er surgery and simulta- ulated relatively easily a er treatment. “B” is incorrect because
neously restart war arin. there is no call or FFP in this asymptomatic patient. T e other
answers are incorrect because there is no advantage to higher
Answer 2.4.2 The correct answer is “A.” For patients with doses or IV doses o vitamin K in this patient, and the higher
nonvalvular atrial brillation who are undergoing surgery doses will make continued anticoagulation more di cult.
or invasive diagnostic procedures, it is reasonable to inter-
rupt anticoagulation or up to 1 week without substituting HELPFUL TIP: r EVEr SiNG WAr FAr iN
heparin (assuming they haven’t had a recent stroke or other in pat ents wh a e n t b eed ng:
thromboembolic event). “Bridge” therapy with IV heparin or • i the iNr s <4.5 s mp educe the d se wa a n
low-molecular weight heparin con ers no bene t (Circulation. h d the next d se wa a n.
2015;131(5):488). T e risk o perioperative bleeding with hepa- • i the iNr s 4.5 t 10 u have seve a pt ns. i the e
rin is actually greater than the risk o thromboembolism rom s n b eed ng: h d ne tw d ses wa a n.
atrial brillation. “B,” “C,” and “D” are incorrect because the r ut ne v tam n K s n t ec mmended. i the pat ent
patient does not need heparin. Bridging therapy is typically w equ e su ge s at h gh b eed ng sk, adm n-
indicated or patients at higher risk or thromboembolic events ste v tam n K ≤ 5 mg w th an the 1 t 1.5 mg n
such as those with mechanical heart valves, prior stroke, or 24 h u s.
CHA2DS2-VASc score >5.
CHAPTER 2 • CAr Dio l o Gy 57

B) Re er the patient to a cardiologist or an EP study to deter-


• i the iNr s >10 adm n ste 2.5 t 5 mg v tam n K mine the best drug to control this rhythm.
even the e s n b eed ng. C) Implant an automatic de brillator to prevent sudden death.
in pat ents wh a e b eed ng: D) Start a beta-blocker.
• Adm n ste p th mb n c mp ex c ncent ate (e.g. E) Order transthoracic echocardiogram to rule out structural
Kcent a . . . sh u d the bu a v we ? The can a d t!). heart disease.
Th s s p e e ed ve esh zen p asma, wh ch s sec-
nd ne. in add t n, adm n ste 5 t 10 mg v tam n K. Answer 2.5.2 The correct answer is “E.” Nonsustained ventric-
See A g thm m P esc be ’s l ette ava ab e at: http:// ular tachycardia may have an adverse prognosis in the presence
pha mac sts ette .the apeut c esea ch.c m/p /A t c eDD. o structural heart disease such as hypertrophic cardiomyopathy
aspx?n dchk=1&cs=&s=Pl &pt=2& pt=2&dd=280509 or ischemic heart disease. An echocardiogram as well as stress
test may be help ul in ruling them out. T ere is no evidence that
nonsustained, asymptomatic ventricular tachycardia worsens
Objectives: Did you learn to . . . outcomes as long as the patient has no underlying cardiac
• We gh the advantages and d sadvantages ate c nt disease. In an otherwise healthy, asymptomatic patient, the risk
ve sus h thm c nt st ateg es at a b at n? o trying to use antiarrhythmic drugs to suppress ventricular
• Manage ant c agu at n and at a b at n v s-à-v s ectopy leads to worse outcomes than doing nothing. Quinidine,
su ge ? mexiletine, amiodarone, and other antiarrhythmics all have
• Manage the ve -ant c agu ated pat ent? proarrhythmic e ects. In general, there is more sudden death in
these patients i they are treated with antiarrhythmic drugs than
CASE 2.5 i they are watched. T ere ore, “A” is incorrect because these
drugs will actually increase mortality. “B” is incorrect since the
A 62-year-old emale presents to your o ce with a history o patient has asymptomatic, sel -limited episodes. T e reason to
occasional palpitations that are o great concern to her. She do an EP study is to see i there is an inducible arrhythmia and
notes that she eels a racing heart that lasts or a matter o sec- to determine treatment. T is patient does not need treatment.
onds and occurs every 7 days or so. However, when she has the “C” is incorrect because this patient has asymptomatic nonsus-
symptoms, she will generally get our to ve episodes during tained ventricular tachycardia. T us, an implantable de bril-
that day. She denies any chest pain, dyspnea, lightheadedness, lator is not indicated. A er your evaluation is complete, you
or other associated symptoms. You order an event monitor may prescribe a beta-blocker (“D”) or symptomatic relie . T is
and it shows that the patient is having nonsustained episodes patient is overall asymptomatic so this would not be the best
o monomorphic ventricular tachycardia lasting 4 beats or less option at this time.
each.
T e echocardiogram is normal, and the patient does well or
Question 2.5.1 The best approach at this point is to: the next 3 months but then becomes symptomatic with pro-
A) Start an antiarrhythmic such as quinidine or mexiletine to longed episodes o ventricular tachycardia. While all o the
control the heart rhythm. episodes are sel -limited, the patient has had two episodes o
B) Re er the patient to a cardiologist or an EP study to deter- syncope.
mine the best drug to control this rhythm.
C) Implant an automatic de brillator to prevent sudden death. Question 2.5.3 Which o the ollowing is the next best step
D) Implant a pacemaker. in treating this patient?
E) Check serum potassium, magnesium, SH. A) Sotalol.
B) Implantable de brillator.
Answer 2.5.1 The correct answer is “E.” T e rst step in deter- C) Amiodarone.
mining the treatment o this patient is to make sure that there is D) Electrophysiologic study.
not an underlying metabolic abnormality that could predispose E) ocainide (an oral lidocaine equivalent).
to this rhythm abnormality.
Answer 2.5.3 The correct answer is “D.” An electrophysiologic
You check a panel o laboratory studies including thyroid study is indicated to induce and characterize the ventricular
unction tests, electrolytes, magnesium, glucose, and CBC. tachycardia. Certain types o ventricular tachycardia respond
T ey are all within normal limits. You suggest that the very well to radio requency ablation. Some o you may have
patient avoid potential triggers such as caf eine and sympa- answered “B.” T is is true in patients with ischemic heart
thomimetics. “Darn,” she sighs. “I have to quit my crystal disease, le ventricular dys unction and symptomatic ven-
meth?” tricular tachycardia. T ese patients should get an implant-
able de brillator as should “all” heart ailure patients with an
Question 2.5.2 The next step or this patient is to: ejection raction o <30% to 35% (there are literally over 200
A) Start an antiarrhythmic such as quinidine or mexiletine to variations o this based on heart ailure class, QRS duration, etc.,
control the heart rhythm. but this is the basic idea).
58 FAMil y MEDiCiNE EXAMiNATio N & Bo Ar D r EViEW

Objectives: Did you learn to . . . the Framingham study showed that the symptoms blamed on
• Eva uate a pat ent w th pa p tat ns? mitral valve prolapse (anxiety, PACs, tachycardia, etc.) are no
• Manage n nsusta ned, as mpt mat c, vent cu a more prevalent in those with mitral valve prolapse than in those
tach ca d a? without it. “C” is incorrect. A bicuspid aortic valve may cause
PACs as a result o heart ailure when the patient decompensates
CASE 2.6 and has increased le -sided heart pressures. However, a bicus-
pid aortic valve itsel is not a source o PACs. Similarly, hyper-
A 22-year-old emale presents to your o ce with a history o trophic cardiomyopathy, other causes o heart ailure, drugs
palpitations. You are able to capture the arrhythmia on the (e.g., theophylline and digoxin), and neurologic diseases can be
monitor in your o ce: the rhythm strip shows evidence o associated with PACs.
isolated premature atrial contractions (PACs). She is other-
wise healthy and taking no medications, and there is no am- T is patient is bothered by her PACs. She is rather aware o
ily history o heart disease. them and nds them disconcerting.

Question 2.6.1 All o the ollowing are salient points o the Question 2.6.3 What is the best pharmacologic therapy to
history with regard to PACs EXCEPT: consider at this point?
A) Aged cheese consumption. A) Sotalol.
B) Ca eine use. B) Metoprolol.
C) obacco use. C) rasylol.
D) Alcohol use. D) Amiodarone.
E) COPD. E) Mountain Dew—lots o it.

Answer 2.6.1 The correct answer is “A.” Aged cheese can cause Answer 2.6.3 The correct answer is “B.” A beta-blocker may
problems in combination with monoamine oxidase inhibitors help to reduce this patient’s PACs. “A” is incorrect because,
(MAOIs). In combination with an MAOI, aged cheese and while sotalol can be used or both atrial and ventricular
other sources o tyramine can cause a hypertensive emergency. arrhythmias, it is proarrhythmic and can cause torsades de
However, this patient is not taking any medications. All o the pointes. T us, it should be initiated in the hospital with moni-
other conditions and drugs listed can cause PACs. While there toring and reserved or those with severe arrhythmias. “C” is
are conf icting data about the strength o the association ca - incorrect because rasylol is the trade name or aprotinin, an
eine, it is clear that COPD, tobacco, and alcohol can all cause enzyme that is used to reduce bleeding during surgical pro-
an increase in sympathetic tone, leading to PACs. Neurologic cedures. “D” is incorrect because, like sotalol, amiodarone is
abnormalities (e.g., stroke) can also be associated with PACs, as proarrhythmic, and its use should be limited to those with sig-
can some drugs (e.g., theophylline). ni cant arrhythmias.
Objectives: Did you learn to . . .
HELPFUL TIP: • r ec gn ze causes PACs?
y u have t eat 2 p unds chedda cheese n ha an • T eat a pat ent w th b the s me PACs?
h u n de t deve p a h pe tens ve c s s w th an
MAo i. in stud es whe e the e was ee access t cheese,
the max mum an ne was ab e t eat was 1.9 p unds n QUICK QUIZ: VAl VUl Ar DiSEASE
2 h u s. Be eve t n t, s me ne stud ed th s (p b-
ab s mewhe e n W sc ns n).
Surgery is indicated in which o these patients with valvular
disease?
Question 2.6.2 Which o the ollowing statements about A) An asymptomatic patient with severe mitral regurgitation
PACs is true? and a le ventricular ejection raction (LVEF) o less than
A) Mitral valve prolapse is associated with PACs. 60%.
B) Mitral valve stenosis is associated with PACs. B) An asymptomatic patient with a bicuspid aortic valve.
C) Bicuspid aortic valve is associated with PACs. C) Asymptomatic aortic regurgitation with an LVEF o less
D) None o the above is true. than 50% on echocardiogram.
D) Only symptomatic valvular lesions should be approached
Answer 2.6.2 The correct answer is “B.” Anything that can surgically.
cause an increase in le atrial pressures (and there ore atrial E) A and C.
wall stretching) is associated with an increase in the number
o PACs. Mitral stenosis causes increased pressures in the le The correct answer is “E.” Once patients with mitral regur-
atrium, wall stretching, and enlargement and thus predisposes gitation and aortic regurgitation become symptomatic,
to PACs. “A” is incorrect. Even though multiple problems have the morbidity and mortality increases signi cantly. T us,
been blamed on mitral valve prolapse, a study done as part o these patients should be operated on be ore they become
CHAPTER 2 • CAr Dio l o Gy 59

symptomatic. Patients should have routine echocardiogra-


phy yearly i they have severe valvular disease. In addition to You decide that this patient may have heart ailure. An ECG
evaluating the valve, echocardiography allows you to evalu- shows no evidence o prior or ongoing ischemia. T ere are no
ate ventricular size and unction. Importantly, proper man- signs o atrial enlargement or ventricular hypertrophy.
agement o patients with valvular heart disease depends on
accurate diagnosis o the cause as well as proper staging o the Question 2.7.2 The proper conclusion rom this is:
disease process, which is based on valvular anatomy, valvular A) he patient does not have cardiac chamber enlargement
hemodynamics, severity o le ventricular dilation and sys- or hypertrophy and there ore is unlikely to have heart
tolic unction, and patient symptoms. (NO E: le ventricular ailure.
systolic unction and patient symptoms are only part o the B) T e absence o evidence or prior in arct makes heart ailure
decision making process regarding management and timing unlikely.
o re erral to surgery). C) Regardless o the ECG results, clinical judgment alone is su -
cient to make the diagnosis o heart ailure, being correct
85% o the time.
CASE 2.7 D) T e patient’s edema is likely rom venous insu ciency.
A 74-year-old male presents to your o ce with a chie E) Despite a normal ECG, urther testing is needed in this
complaint o a “long cold” with an intermittent cough or patient to evaluate or heart ailure.
5 months. He has also noticed that he gets up to urinate twice
a night although he has no trouble with his urine stream, Answer 2.7.2 The correct answer is “E.” “A” is not correct
starting urination, or dribbling a erward. He has been a bit because only 30% to 60% o moderate-to-severe le ventricu-
more tired lately and notices that his exercise tolerance has lar hypertrophy (LVH) is detectible on ECG. “B” is incorrect
decreased to several blocks, limited mainly by shortness o because patients with diastolic dys unction (discussed later)
breath. He has not had any chest pain. He has no history o may not have any evidence o prior ischemia or MI. “C” is
asthma or COPD and has not had any exposures to drugs or incorrect. T e clinical diagnosis o heart ailure is incorrect up
chemicals. He has a history o hypertension and noncompli- to 50% o the time. For this reason, con rmation is required
ance with medical recommendations. In act, he is taking no be ore embarking upon a therapeutic adventure or heart ail-
medications except or an aspirin a day. His pulse is 100 bpm ure. “D” is unlikely, since the patient has other symptoms o
with a blood pressure o 160/95 mm Hg. He looks pretty well. heart ailure (exertional dyspnea, cough, etc.) that make simple
On examination, you nd only trace pitting edema o the venous insu ciency unlikely.
lower extremities.
Question 2.7.3 You decide on urther testing. Assuming
Question 2.7.1 Which o the ollowing is NOT a possible every test is easily available to you (which might not be the
cause o cough in this patient? case depending on the setting in which you work), what is
A) Heart ailure. the one best test that you would use to determine i this
B) Asthma. patient has heart ailure?
C) Deconditioning. A) Echocardiography.
D) COPD. B) Brain natriuretic peptide (BNP) level.
E) GERD. C) Chest radiograph looking or evidence o pulmonary edema
(Kerley B lines, etc.).
Answer 2.7.1 The correct answer is “C.” Deconditioning D) SPEC thallium test.
may cause dyspnea on exertion but should not cause a cough. E) Positron emission tomography (PE ) testing.
T e purpose o this question is to point out the act that a
“chronic cold” or “chronic cough” in an elderly person can Answer 2.7.3 The correct answer is “A.” Echocardiography is
be due to a myriad o causes, including “occult” heart ailure the procedure o choice or the diagnosis o heart ailure. T is
(systolic or diastolic HF). Do not make the assumption that is or two reasons. First, you can assess le ventricular systolic
the patient’s diagnosis (e.g., a “chronic cold”) is necessarily unction as well as look or diastolic dys unction to determine
the correct diagnosis. i this is systolic or diastolic heart ailure (i.e. heart ailure with
reduced ejection raction, HFrEF or heart ailure with preserved
ejection raction, HFpEF respectively). Second, you can evalu-
HELPFUL TIP:
ate the potential causes o heart ailure including valvular heart
S st c hea t a u e (sec nda t n a ct n, etc.)
disease, ischemic heart disease, pericardial disease, deposition
s n w te med “Hea t Fa u e w th a educed Ejec-
disease (amyloidosis, hemochromatosis) etc. “B” is incorrect
t n F act n” (HF EV). D ast c hea t a u e s n w
because the BNP will give you less concrete in ormation about
te med “Hea t Fa u e w th a p ese ved Eject n F ac-
the patient versus echocardiography. In this patient, with a high
t n (HFpEF). in th s b k we use b th the “ d” and
pretest probability o heart ailure, BNP will most likely be ele-
new n tat ns.
vated. “D” and “E” are incorrect because SPEC thallium and
PE testing are best used to diagnose ischemia due CAD.
60 FAMil y MEDiCiNE EXAMiNATio N & Bo Ar D r EViEW

HELPFUL TIP: HELPFUL TIP:


in a w- sk pat ent, a BNP <100 pg / ml e ect ve T x ns such as methamphetam ne, c ca ne, a c h ,
u es ut hea t a u e. A BNP 100 t 500 pg/ml s n- and ant ne p ast c chem the apeut c d ugs can cause
dete m nate and ma n t be e ated t ca d ac d sease. a ca d m path and sh u d be c ns de ed as a p s-
A BNP >500 pg/ml s e at ve spec c hea t a - s b e et g CHF n the app p ate pat ent.
u e ( the ca d ac and n nca d ac causes a e exc ud-
ed). A te nate ca d ac causes nc ude ACSs, hea t musc e Question 2.7.5 Which o the ollowing is the most appro
d sease, va vu a hea t d sease, pe ca d a d sease, at a priate next strategy to work up this patient’s systolic heart
b at n, m ca d t s, ca d ac su ge , and status-p st ailure (HFrEF)?
DC ca d ve s n. A) Cardiac MRI to assess myocardial viability.
B) Coronary angiogram.
C) Measure serial troponins to rule out ACS.
(LESS) HELPFUL TIP: D) Electrophysiologic study to assess or inducible ventricular
N nca d ac causes an e evated BNP nc ude ena a - arrhythmia.
u e, anem a, advanced age, seps s/ Sir S, pu m na ( b- E) C to assess calcium scores.
st uct ve s eep apnea, pu m na h pe tens n, seve e
pneum n a), c t ca ness, seve e bu ns, t x cmetab - Answer 2.7.5 The correct answer is “B.” Ischemic heart disease
c events (cance , chem the ap , enven mat n). o be- is the most common etiology or heart ailure associated with
s t can cause a a se w BNP. BNP-d ected the ap systolic dys unction. I you can reverse the ischemia, mortal-
may be use u n s me pat ents. H weve , m st data at ity is decreased rom 16% annually to 3.2% annually. O all the
th s p nt suggests that t eat ng based n s mpt ms options listed above, coronary angiography remains the gold
and t eat ng based n BNP have s m a utc mes (Eu standard to evaluate or CAD in this setting. Coronary angio-
Hea t J. 2014;35(1):16–24). grams provide in ormation about anatomy and easibility o
revascularization but do not predict recovery o unction. T is
patient does not have chest pain or ECG changes to suggest
acute cardiac syndrome (ACS); there ore, “C” is incorrect. “D”
T e patient has an echocardiogram that shows a le ven- is incorrect, as there is no indication or an electrophysiologic
tricular ejection racture (LVEF) o 35% and a regional wall study in the absence o any arrhythmia. “E” is incorrect as the
motion abnormality (RWMA). presence or absence o coronary calci cation would not change
the overall management plan or this patient. C calcium scor-
Question 2.7.4 This is the most consistent with a diagnosis ing may be used as an additional risk strati cation tool in inter-
o : mediate-risk patients (similar to CRP, in addition to traditional
A) Systolic heart ailure secondary to myocarditis. risk actors such as hypertension, dyslipidemia, vascular or
B) Systolic heart ailure secondary to CAD. renal disease, etc.) but not in high-risk, symptomatic patients.
C) Diastolic heart ailure.
D) Systolic heart ailure secondary to constrictive pericarditis. T e coronary angiogram shows dif use CAD; no coronary
E) Age-related changes; there ore, a normal variant. lesions are considered to be amenable to angioplasty and
there are no vessels considered to be viable targets or bypass
Answer 2.7.4 The correct answer is “B.” A regional wall surgery. You decide to initiate medical therapy in this patient.
motion abnormality (RWMA) suggests that this patient has In addition, you advise the patient regarding the nonphar-
ischemic or in arcted myocardium. “A” is not the best choice macologic therapies or heart ailure treatment.
since those with myocarditis ( ulminant or acute) typically
have global hypokinesis (although RWMA has been reported) Question 2.7.6 Nonpharmacologic therapies or systolic
and the patient would typically appear more clinically ill. “C” heart ailure (HFrEF) include all o the ollowing EXCEPT:
is incorrect by de nition. Diastolic heart ailure (HFpEF) A) Fluid restriction o <2 L/day.
requires a LVEF o at least 50%, recognizing that this group B) Sodium restriction o <2 g/day.
may not have an entirely normal LVEF, but the major abnor- C) Dietary consultation.
mality is not a reduction in LV systolic unction. Systolic heart D) Cardiac risk actor modi cation.
ailure (HFrEF) is de ned by an LVEF o <40%. Diastolic heart E) Monthly weight monitoring.
ailure (HFpEF) is associated with a hypertrophied le ven-
tricle and a preserved LVEF. T e echocardiogram in constric- Answer 2.7.6 The correct answer is “E.” Prior to initiation o
tive pericarditis (“D”) generally shows normal le ventricular therapy, it is important to determine the patients NYHA classi-
systolic unction. It may reveal pericardial thickening, dilated cation and ACC/AHA stage o HF, which in orms the clinician
in erior vena cava or hepatic veins, and abnormal mitral and regarding guideline-directed medical therapy. T e keystone o
tricuspid in-f ow Doppler. an e ective heart ailure treatment regimen is sodium and f uid
CHAPTER 2 • CAr Dio l o Gy 61

balance as well as management o comorbidities. Frequently over- All o the others options, including the combination o iso-
looked, these are the most common causes o heart ailure exac- sorbide dinitrate and hydralazine, have been shown to reduce
erbation. It is imperative to get a dietary consultation or every mortality. However, hydralazine and isosorbide dinitrate are
patient with newly diagnosed heart ailure. Cardiac risk actors, generally reserved or those patients who are unable to toler-
including: hypertension, diabetes, hyperlipidemia, sleep apnea, ate ACE inhibitors or angiotensin receptor blockers (ARBs) or
obesity, sedentary li estyle, smoking/alcohol/drug use, need to be remain symptomatic despite maximal medical therapy with the
treated with the same aggressiveness as in a patient with an ACS. other medications, especially in A rican Americans. Enalapril
Patient should be advised about daily weight monitoring rather reduces mortality by 28% when compared with hydralazine and
than monthly monitoring. A weight gain o more than 3 to 5 lb nitrates. T us, hydralazine and nitrates are second line. None o
may necessitate additional doses o a diuretic. NO E: signi cant the “traditional” loop diuretics such as urosemide, bumetanide,
f uid restriction to 1.5 to 2.0 L/day is typically, though not exclu- etc., have been shown to positively a ect mortality.
sively, reserved or Stage D advanced HF patients who are hypo-
natremic or diuretic resistant. Both sodium and f uid balance You start this patient on urosemide or diuresis and lisinopril
recommendations are best implemented in the context o a struc- or HFrEF. You also decide to initiate metoprolol succinate or
tured daily weight and symptom sel -monitoring home program. its survival bene ts. However, the patient’s symptoms worsen.

You wish to start an appropriate drug regimen or this patient’s Question 2.7.8 Which o these is true about the use o
heart ailure. metoprolol in HFrEF?
A) It is the only beta-blocker indicated or use in HFrEF.
Question 2.7.7 All o the drugs below have been shown B) Its best use is in those patients who are still symptomatic
to reduce mortality in patients with systolic heart ailure since it will help to control symptoms.
(HFrEF) EXCEPT: C) It should only be initiated in patients with well-controlled
A) Digoxin. HFrEF who are not currently having signi cant symptoms
B) Metoprolol succinate. (i.e., decompensated).
C) ACE inhibitors. D) Beta-blockers can lead to signi cant hypokalemia when
D) Hydralazine and long-acting nitrates used in combination in combined with diuretics, so potassium levels should be
patients intolerant o ACE inhibitors. monitored closely.
E) Spironolactone. E) Beta-blockers are contraindicated in patients who have a
combination o COPD and HFrEF.
Answer 2.7.7 The correct answer is “A.” Digoxin is an inotro-
pic agent, and as such, would intuitively make sense as an e ec- Answer 2.7.8 The correct answer is “C.” Beta-blockers should
tive drug or HFrEF. However, digoxin has not been shown to not be initiated in patients who are signi cantly symptomatic
increase survival and in act may worsen outcomes, especially or decompensated. While beta-blockers do reduce mortality,
in women. Its use is primarily or symptomatic relie . I using they can increase symptoms and urther exacerbate heart ail-
digoxin in HFrEF, the therapeutic target is a dose that achieves ure. T ere ore, they are best initiated in the stable patient (as
a plasma concentration o drug in the range o 0.5 to 0.9 ng/mL. an outpatient or 24 to 48 hours prior to hospital discharge and
T e common daily dosage to achieve that target is typically on a stable drug regimen). Even then some patients cannot tol-
0.125 to 0.25 mg/day in patients <70 years o age with normal erate the introduction o beta-blockers without worsening o
renal unction and body mass. For patients with abnormal symptoms, which may require additional diuresis, discontinua-
renal unction, low body mass or age >70, low doses (0.125 mg tion o the beta-blocker, a reduction in dose, or other measures.
daily or every other day) are recommended initially, and the “A” is incorrect. Other beta-blockers have been used in HFrEF,
dose then titrated to the target therapeutic range. Digoxin including carvedilol. “B” is incorrect because beta-blockers may
does reduce hospitalizations and improve symptoms in those actually worsen heart ailure symptoms. “D” is incorrect since
with HFrEF who are symptomatic despite use o other maxi- beta-blockers do not cause hypokalemia. “E” is incorrect. Beta-
mal guideline directed therapy. Digoxin may also be utilized blockers can be used in patients with COPD with the same cave-
in addition to beta-blockers or controlling the ventricular ats that apply to any other patient: i the patient is becoming more
response in HFrEF patients with atrial brillation (though symptomatic on the beta-blocker, reduce the dose or discontinue
it is generally not very e ective in ambulatory patients). It is the drug. In act, beta-blockers may improve survival in COPD
ine ective at treating atrial brillation in those with a nor- (BMJ. 2011;342:d2549; BMC Pulmonary Medicine. 2012;12:48).
mal EF. Avoid digoxin in patients with signi cant sinus or AV
nodal block unless they have a pacemaker. NO E: Other car- HELPFUL TIP:
diac medications such as amiodarone, dronedarone, verapamil, When sta t ng a beta-b cke HF EF, sta t w and
propa enone and quinidine as well as noncardiac meds such as g s w! When sta t ng d g x n HF EF, m n t
clarithromycin, erythromycin, itraconazole and cyclosporine h p ka em a, h p magnesem a, h p th d sm t
can increase the serum digoxin concentrations and may pre- av d d g x n t x c t , even w th w d ses.
cipitate digoxin toxicity.
62 FAMil y MEDiCiNE EXAMiNATio N & Bo Ar D r EViEW

consumption and f uid retention), increased metabolic demand


You reduce the dose o metoprolol succinate and consider (e.g., rom in ection), anemia, medication noncompliance,
starting this patient on another medication. arrhythmia, and ischemia. T e inappropriate use o medica-
tions, such as some calcium channel blocker and the institution
Question 2.7.9 Which o the ollowing patients is/are good o beta-blockers when CHF is decompensated, are also com-
candidates or spironolactone? mon causes o exacerbations o HF.
A) A patient with NYHA Class I and Class II heart ailure.
B) A patient with NYHA Class III and Class IV heart ailure. T e patient notes that he did have some chest pain earlier in
C) Both A and B. the day. You want to initiate therapy. You take his vitals, and
D) Neither A nor B. his pulse is 100 bpm, blood pressure 140/95 mm Hg, oxygen
saturation 89%on room air, and respiratory rate 32.
Answer 2.7.9 The correct answer is “B.” Spironolactone has
been shown to reduce mortality in patients with New York Heart Question 2.7.11 Besides oxygen, the one best drug to initi
Association (NYHA) Class III and Class IV heart ailure. It has ate rst in the ED to treat this patient with an acute exacer
not been studied in Class I (thus “A” is wrong), However, it may bation o his chronic HFrEF is:
be use ul in symptomatic patients with Class II with an EF o A) Furosemide.
<30%. Serum potassium needs to be monitored closely a er ini- B) Digoxin.
tiation o spironolactone, especially since it will generally be used C) A positive inotrope, such as dobutamine.
with an ACE inhibitor or ARB, both o which can increase the D) Nitroglycerin.
serum potassium. T is drug should be avoided in patients with E) An ACE inhibitor
renal insu ciency or patients with serum potassium >5 mEq/L.
Spironolactone is indicated or patients with NYHA Classes II– Answer 2.7.11 The correct answer is “D.” T is patient will ben-
IV and who have LVEF ≤ 35% and a creatinine o <2.5 mg/dL e t rom nitroglycerin or several reasons. First, the patient has
in males or <2.0 mg/dL in emales (or estimated GFR >30 mL/ told you that he had chest pain earlier today. T us, it is possible
min/1.73 m 2). Eplerenone is another aldosterone inhibitor but is that this patient’s HF exacerbation is due to ischemic disease.
much more expensive with little, i any, advantage. Nitroglycerin will help this via vasodilation. T e second rea-
son is that the goal here is to restore normal cardiac unction
HELPFUL TIP: by causing vasodilation and decreasing preload and a erload.
A sk en (Tektu na), a en n nh b t , is contraindicat- Nitroglycerin will do both o these. “A,” urosemide, is also a
ed w th an Ar B ACE nh b t . it w sens utc mes good choice but not the one best choice. By inducing diuresis,
and causes h pe ka em a. it a s w sens utc mes urosemide will also signi cantly decrease preload and provide
n pat ents w th d abetes. it d esn’t ea have a we - symptomatic relie . But remember not all HF patients are f uid-
de ned p ace n the ap . overloaded (such as with f ash edema rom ischemia). “B” is
incorrect because it will take some time or digoxin to have a
signi cant impact on this patient’s symptoms. “C” is incorrect
You treat this patient with metoprolol succinate, lisinopril, because dobutamine is a second-line drug reserved or those
urosemide, and aspirin. T is regimen seems to help, and the not responding to more conservative therapy and it would not
patient’s symptoms improve. However, a ew weeks later, he be indicated in active ischemia due to increasing myocardial
presents to the ED with increased dyspnea. T ere have been oxygen demand. “E” is technically not incorrect, but it is not
no changes in his medications, and he assures you that he is the best answer. T ere is ample evidence that ACE inhibitors,
taking his medications as directed. His examination reveals which work as a erload reducers, can be used in acute HF exac-
that he has elevated JVD, rales over the lower hal o his lung erbations either IV (e.g., enalaprilat) or sublingual (e.g., capto-
elds bilaterally, and pedal edema. pril). However, these drugs should be reserved as second-line
therapy or patients who do not respond to more appropriate
Question 2.7.10 Common causes o decompensation in initial measures, and they would not be the rst line to address
patients with otherwise stable heart ailure include all o ischemia among choices listed.
the ollowing EXCEPT:
A) Inactivity. You treat the patient with nitroglycerin, he improves, and
B) Fever. you admit him to the oor. While in the hospital, the patient
C) Arrhythmia. develops some additional chest pain that lasts or 10 min-
D) Dietary indiscretion. utes and responds to additional sublingual nitroglycerin. His
E) Ischemia. BNP is noted to be elevated. His hemoglobin (Hb) is 7.2 g/dL
and hematocrit (HC ) is 22%. He is still in congestive ail-
Answer 2.7.10 The correct answer is “A.” Inactivity will ure. T e pathologist tells you that there is blood available in
not generally cause an exacerbation o HF. T e major causes the blood bank to trans use this gentleman i you so choose.
o increased HF include dietary indiscretion (increased salt T ere is a problem, o course: he is in HF and now is some-
intake—“Say, can you pass the potato chips?”—increased f uid what tachycardic at 110.
CHAPTER 2 • CAr Dio l o Gy 63

Question 2.7.12 You tell the pathologist that: nh b t s, Ar Bs, d g x n, and sp n act ne n the t eat-
A) T e Hb o 7.2 g/dL is not an indication or trans usion. ment HF EF?
B) rans using this gentleman is inappropriate since he is • Desc be the e BNP measu ement n the eva uat n
already in HF and may become more f uid-overloaded with hea t a u e w th educed eject n act n and nes t de
a blood trans usion. n the t eatment hea t a u e?
C) You would like to go ahead with trans using this patient.
D) Making this patient’s blood more viscous with a trans usion
will increase the stress on his heart. CASE 2.8
E) Erythropoiesis-stimulating agents (ESAs), such as darbe-
Your patient with heart ailure does well and is discharged
poetin (Aranesp), are sa er and more e ective than blood
rom the hospital a er a couple o days. You are just beginning
trans usions or patients with HF.
to think that the authors are tired o writing questions about
heart ailure . . . but you are wrong. T e patient’s 70-year-old
Answer 2.7.12 The correct answer is “C.” T is patient should wi e shows up with shortness o breath. Her physical exami-
be trans used. Guidelines suggest triggering trans usion in heart nation is consistent with heart ailure. Since you have learned
disease when the Hb is <8 g/dL and i the patient is symptomatic so much rom this case already, you send her to get an echo-
(i.e., tachycardic, chest pain). T ere is no bene t to trans using cardiogram. You also order the recommended tests: CBC,
patient with a higher Hb (Ann Intern Med. 2013;159:770). Use electrolytes, ECG, thyroid unctions, etc.
clinical judgment o course, as there are no guidelines or trans-
usion in an individual with ACS. ESAs (“E”) seems to increase Question 2.8.1 The results o the echocardiogram show a
thromboembolic events without providing any bene t in those concentric thickening o the le t ventricle with an ejection
with stable HF and mild to moderate anemia. T ey should gen- raction o 75%. This is most consistent with:
erally be avoided in this population. A) Ischemic cardiomyopathy.
T ere are no o cial ACC/AHA recommendations regarding B) Diastolic dys unction (HFpEF).
trans usion. However, the mortality at 30 days is increased i the C) Viral cardiomyopathy.
Hb is <11 mg/dL in a patient with a non-S EMI. Whether or D) Hypertrophic cardiomyopathy.
not trans usion will help this is not known: it may just be that E) None o the above.
patients with anemia are sicker at baseline. Patients with an HC
o <20% to 24% likely bene t rom a trans usion while those
Answer 2.8.1 The correct answer is “B.” “A” is incorrect since
with an HC > 27% to 30% do not. For 25% to 26% use your
there would likely be evidence o RWMA i there had been an
judgment. HF is di erent. Blood trans usion should be reserved
old myocardial in arction. Also, this patient has a preserved
or patients with heart ailure that are severely anemic and the
ejection raction, which is consistent with diastolic dys unc-
trans usion be undertaken slowly and with the concurrent use
tion (HFpEF) rather than the decreased ejection raction
o diuretics to avoid volume overload (Am Heart J. 2009;158:
associated with ischemic cardiomyopathy. “C” is incorrect.
653–658). “A” and “B” are incorrect because this patient should
Viral cardiomyopathy is associated with a dilated ventricle
be trans used care ully as noted above. “D” is incorrect since
rather than a hypertrophic one, and there would be global
trans using this patient to a normal Hb and HC will not cause
dyskinesia with decreased ejection raction. “D” is incor-
excess blood viscosity.
rect; hypertrophic cardiomyopathy is usually associated with
asymmetric hypertrophy, o en septal, rather than concentric
hypertrophy o the le ventricle. Hypertrophic cardiomyopa-
HELPFUL TIP: thy may lead to HFpEF in addition to le ventricular outf ow
Nes t de (Nat ec ), a BNP ana g, can be used tract obstruction.
HF EF but s expens ve, c nt butes t ena a u e, and
ke nc eases m ta t . Nes t de can p duce p -
Question 2.8.2 Diastolic dys unction (HFpEF) is associated
nged h p tens n, wh ch m ts the d se that can be
with which o the ollowing?
used. Th s s a the ap ast es t . . . and that ma be
A) A prolonged history o untreated hypertension.
t cha tab e. Just d n’t use t.
B) Poor relaxation o the ventricular wall.
C) T yroid disease.
D) A and B.
Objectives: Did you learn to . . . E) B and C.
• r ec gn ze at p ca p esentat ns hea t a u e w th e-
duced eject n act n n the e de ? Answer 2.8.2 The correct answer is “D.” HFpEF is o en associ-
• Desc be the sens t v t and spec c t an ECG l VH? ated with long-standing hypertension as well as a sti ventricu-
• Eva uate a pat ent w th hea t a u e w th educed eject n lar wall that does not relax to allow good lling during diastole
act n? (there ore “diastolic dys unction”). “C” is not correct because
• Manage a pat ent w th hea t a u e w th educed eject n hyper- and hypothyroidism are usually associated with a dilated
act n and unde stand the e beta-b cke s, ACE cardiomyopathy.
64 FAMil y MEDiCiNE EXAMiNATio N & Bo Ar D r EViEW

Question 2.8.3 HFpEF represents approximately what


HELPFUL TIP:
percentage o HF?
Hea t a u e s a te m na ness w th a 5- ea su v va
A) <5%.
n 50%. Th s s w se than man cance s.
B) Approximately 10%.
C) Approximately 25%.
D) Approximately 50%. Objectives: Did you learn to . . .
E) >75%. • Unde stand the path ph s g HFpEF d ast c?
• T eat a pat ent w th HFpEF?
Answer 2.8.3 The correct answer is “D.” HFpEF represents
between 40% and 60% o cases o HF when looking at the
population as a whole. T e other answers are incorrect. T e CASE 2.9
point here is that, as discussed earlier, patients with HF need
an echocardiogram to determine what type o heart ailure Your “congestive heart ailure couple,” as they now call them-
they have. selves, are doing so well that the wi e re ers her cousin to
you (we still aren’t tired o writing heart ailure questions).
Her cousin, a 65-year-old male, arrives at your o ce and
HELPFUL TIP: you immediately notice the smell o tobacco leaching rom
HFpEF ccu s m e c mm n n e de p pu at ns. his clothing. T e small burns in his clothing con rm to you
that he smokes, and he in orms you that he has smoked
three packs per day “since birth.” He recently has noticed
some swelling in his eet and increased shortness o breath.
Question 2.8.4 Which o the ollowing drugs is the LEAST He denies a history o cardiac disease. An ECG per ormed
desirable in patients with HFpEF? in the o ce shows right axis deviation and a right bundle
A) Diuretics. branch block (RBBB). An echocardiogram shows that he has
B) ACE inhibitors. normal le ventricular unction but a hypertrophied right
C) Nitrates. heart with paradoxical bulging o the ventricular septum
D) Digoxin. into the le ventricle was noted.
E) Negative inotropes such as beta-blockers and CCBs.
Question 2.9.1 This clinical picture is most consistent with
Answer 2.8.4 The correct answer is “D.” Digoxin and other which o the ollowing?
positive inotropes (e.g., milrinone) are not very use ul in dia- A) Constrictive pericarditis.
stolic dys unction. T is makes sense. T e problem here is not a B) Chronic mitral valve prolapse.
lack o contractility but alternatively a lack o muscle relaxation. C) Cor pulmonale.
While there has not been a superior therapeutic regimen identi- D) Old right ventricular in arction with subsequent dys unction.
ed by randomized control trials, the goals o therapy are blood E) Chiari network.
pressure control, the use o diuretics to relieve congestion and
edema, treatment o ischemia i present, and control o the heart Answer 2.9.1 The correct answer is “C.” A typical picture o
rate to avoid tachycardia. cor pulmonale is right ventricular hypertrophy (RVH) with
paradoxical bulging o the septum into the le ventricle, right
Question 2.8.5 Which o the ollowing drugs or drug classes axis deviation on ECG, and partial or complete RBBB. “A” is
is theoretically the best choice or the treatment o HFpEF? incorrect. Constrictive pericarditis is associated with pericar-
A) ACE inhibitors. dial thickening, dilated in erior vena cava or hepatic veins, and
B) Beta-blockers. abnormal mitral and tricuspid f ow. “B” is incorrect because
C) Diuretics. mitral valve prolapse in the absence o severe mitral regurgi-
D) Hydralazine. tation is not likely to be hemodynamically signi cant. “D” is
E) ARBs. incorrect because with a right ventricular in arct, you would
expect to see a poorly unctioning right ventricle. “E” is incor-
Answer 2.8.5 The correct answer is “B.” Beta-blockers, espe- rect because a Chiari network is normal vestigial variant in the
cially metoprolol succinate, are use ul as initial therapy in right atrium and would not cause RVH.
HFpEF. Beta-blockers (1) slow down the heart to permit lon-
ger LV lling duration during diastole and (2) help to relax Question 2.9.2 Cor pulmonale (not right ventricular ailure)
the myocardium to promote a less restrictive lling pattern. I may result rom all o these disease processes EXCEPT?
a patient ails beta-blockers, try a CCB (e.g., verapamil, diltia- A) Sickle cell anemia.
zem). Unlike systolic dys unction, the treatments o diastolic B) Le ventricular ailure.
dys unction are not well established, and there is no convinc- C) PE.
ing evidence that beta-blockers or ACE inhibitors reduce D) Chronic obstructive lung disease.
mortality. E) Interstitial lung disease.
CHAPTER 2 • CAr Dio l o Gy 65

Answer 2.9.2 The correct answer is “B.” Cor pulmonale is the Objectives: Did you learn to . . .
term used or right heart ailure caused by diseases primarily • D agn se c pu m na e?
a ecting the lungs and pulmonary vasculature. T e chronic pres- • Desc be causes c pu m na e?
sure overload o the right ventricle as it ejects in to the high resis- • T eat a pat ent w th c pu m na e?
tance pulmonary vasculature results initially in RVH with normal
RV systolic unction but over time, the RV contractility declines
leading to RV dilation and right-sided heart ailure with associ- CASE 2.10
ated signi cant tricuspid regurgitation and right atrial dilation.
A 65-year-old male presents to your clinic or a complete his-
Question 2.9.3 A possible nding on the ECG o this patient tory and physical examination. You notice that his abdomi-
would include: nal examination reveals a pulsatile mass, which you sus-
A) P-mitrale (an “m” shaped, notched P wave in lead II). pect may represent an aortic aneurysm (now we are tired o
B) P-pulmonale (an enlarged, peaked, P wave in lead II). writing heart ailure questions). T is nding is con rmed
C) Absent P waves. by ultrasound. T e radiologist reports that the patient has a
D) Inverted P waves. 3.5-cm abdominal aortic aneurysm without evidence o leak
or thrombus ormation.
Answer 2.9.3 The correct answer is “B.” Patients with cor pul-
monale o en have an enlarged and peaked P wave in lead II Question 2.10.1 The best advice to this patient is:
ref ecting right atrial enlargement. “P-mitrale” is ound in le A) Have the aortic aneurysm xed now while he is still
atrial enlargement. healthy.
Question 2.9.4 Besides stopping smoking, the best treat B) Have a ollow-up ultrasound every 3 months.
ment o this patient’s cor pulmonale and pulmonary hyper C) Have a stent placed to prevent urther aortic dilatation.
tension (PHTN) is: D) Have an angiogram in the next several days to rule out vas-
A) Continuous prostacyclin in usion. cular disease below the aorta ( emoral arteries, iliac arteries,
B) Continuous, low f ow oxygen. etc.).
C) CCBs. E) Have a repeat ultrasound at 1 year.
D) Nitroglycerin.
E) Antibiotics to reduce pulmonary inf ammation secondary Answer 2.10.1 The correct answer is “E.” Patients with an
to in ection. abdominal aortic aneurysm less than 4 cm in diameter should
have an ultrasound yearly to check progression. T ose with an
Answer 2.9.4 The correct answer is “B.” In this patient who is a aneurysm 4 to 5 cm in diameter should have an ultrasound
smoker with cor pulmonale, the best drug is continuous, low-f ow every 6 months. An ultrasound on an every 3 to 6 months basis
oxygen. T is will help to reverse the pulmonary vasoconstriction is also indicated or aneurysms that are growing >0.5 cm per
caused by chronic hypoxia. It should go without saying that you year. Bottom line: the larger the aneurysm, the more requently
must do everything you can to get him to stop smoking. His dis- should do an ultrasound. “A” is incorrect (see next question or
ease process will progress much aster i he continues to smoke. an explanation). “C” is incorrect since a stent is not indicated at
“A” is incorrect because prostacyclin in usion is use ul in primary this point. “D” is incorrect. T e only reason to do an angiogram
pulmonary hypertension (PH N), not this type o cor pulmonale. at this point is i the patient is symptomatic or i you are plan-
“C” is incorrect. In some cases o primary PH N, CCBs, PDE5 ning surgical intervention.
inhibitors (e.g., sildena l), and several other medications, which
serve as direct vasodilators to dilate the pulmonary vascular bed, Question 2.10.2 The patient is really worried that this aneu
can be use ul. However, this is not the best choice or this patient rysm will rupture and kill him. You educate him that the
with COPD. “D” is incorrect because patients with cor pulmonale bene t o having the aneurysm repaired is greater than the
are dependent on high right heart lling pressures to get blood risk o the surgery when the aneurysm reaches:
through the pulmonary vasculature. Nitroglycerin will reduce pre- A) ≥4.5 cm.
load, thereby lowering right ventricular pressure and resulting in B) 5.0 to 5.5 cm.
worsening o his symptoms. “E” is also incorrect. Antibiotics might C) 5.5 to 6.0 cm.
be needed in this patient or pneumonia, bronchiectasis, etc., but D) >6.0 cm.
they are not going to help with the treatment o cor pulmonale. E) No repair is indicated until the patient becomes symptom-
atic.
HELPFUL TIP:
r emembe s eep apnea as a cause c pu m na e. N c- Answer 2.10.2 The correct answer is “B.” T e risk o the sur-
tu na x gen desatu at n causes nc eased pu m na gery outweighs the bene ts until the aneurysm reaches some-
vascu a es stance caus ng e evated ght vent cu a where between 5.0 and 5.5 cm. T e rest are incorrect. It would
p essu e and p ss b ght-s ded a u e seve e and be an especially bad idea to wait until an aneurysm is symptom-
unt eated. atic, as a ruptured aortic aneurysm can be lethal in a matter o
minutes.
66 FAMil y MEDiCiNE EXAMiNATio N & Bo Ar D r EViEW

Esmolol is pre erred because o the short hal -li e; you easily can
HELPFUL TIP:
turn it o i there is hypotension. Nitroprusside (or our avor-
Pe cutane us end vascu a stent g a t epa s bec m-
ite, IV nitroglycerin) can be added i the blood pressure control
ng the p cedu e ch ce as c mpa ed t pen epa .
remains suboptimal even a er beta-blockade. In this scenario,
But the e a e t ade s: 30 da m ta t m an end -
nitroprusside should never be given without beta-blockade, as
vascu a g a t s 1.6% ve sus 3.2% pen epa , but at
it may cause ref ex tachycardia induced by vasodilation and
8 ea s uptu e ate was 5.4% ve sus 1.4%. o ve a 8- ea
thus urther aortic shear stress. T e same rationale is true or
m ta t s the same (N Eng J Med. 2015;373:328).
not using intravenous hydralazine without beta-blockers in
this scenario. “A” is incorrect or two very good reasons. First,
ni edipine should never be used sublingually. Syncope, heart
T e patient goes to exas (or Arizona or Florida—somewhere block, MI, stroke, and other serious adverse consequences have
warmer than Iowa) or the winter as part o the re-establish- been reported. Second, ni edipine increases heart rate causing
ment o human annual migration. When he returns, he calls an increase in shear orces on the aorta. “B” is incorrect since
you complaining o back pain that is somewhat sharp and amlodipine does nothing to reduce heart rate, is not titratable to
radiating into his legs. You meet him in the ED and suspect any use ul degree, and the onset o action is too slow to be used
that he is having a dissection o his aneurysm. when prompt blood pressure lowering is desired. “E” is incor-
rect because nitroglycerin alone causes ref ex tachycardia.
Question 2.10.3 All o the ollowing are true regarding aor
tic dissection EXCEPT:
HELPFUL TIP:
A) A substantial number o patients will have palpable pulses
Sc een an a t c aneu sm once n men age 65 t
below the level o the dissection.
75 wh have eve sm ked (pe USPSTF and ACC/AHA
B) Patients may have an elevated LDH and microangiopathic
gu de nes).
ndings on RBC examination.
C) Blood pressure should be kept on the high side to ensure
per usion below the area o the aneurysm.
D) T e pain may migrate down rom the chest to the lower Objectives: Did you learn to . . .
abdominal area over time. • ident the t eatment pt ns and t m ng t eatment an
E) T e pain may be episodic. abd m na a t c aneu sm?
• Manage a pat ent w th a d ssect ng aneu sm?
Answer 2.10.3 The correct answer is “C.” One does not want
to keep the blood pressure on the high side. In act, reducing
the blood pressure is the initial treatment o choice or a dissect-
CASE 2.11
ing aneurysm. “A,” “B,” “D,” and “E” are all correct statements. A 60-year-old male presents with dizziness and palpitations.
Patients may have an elevated LDH and microangiopathic nd- T e patient has a blood pressure o 100/60 mm Hg and a
ings on RBC smear as a result o trauma and cell lysis. “D” is a pulse o 160. His ECG is shown in Figure 2-3.
correct statement, but patients o en do not have this “classic”
migrating pattern o pain. “E” is o en true o pain in aortic dis- Question 2.11.1 Which o the ollowing interventions are
section—it may be episodic. appropriate options in the treatment o this patient?
A) Amiodarone, lidocaine, de brillation, metoprolol.
T e patient has a blood pressure o 160/105 mm Hg. Clearly, B) Amiodarone, lidocaine, de brillation, diltiazem.
this is too high in a patient who has an ongoing dissection. C) Amiodarone, lidocaine, cardioversion, diltiazem.
You decide to treat this patient be ore trans erring him to a D) Procainamide, lidocaine, adenosine, de brillation.
tertiary care center where he can be surgically managed. E) Amiodarone, procainamide, lidocaine, cardioversion.

Question 2.10.4 The best medication(s) to use in this Answer 2.11.1 The correct answer is “E.” T e rhythm is stable
patient to control his blood pressure is/are: ventricular tachycardia. Procainamide, lidocaine, amiodarone,
A) Sublingual ni edipine plus metoprolol. and synchronized cardioversion can all be used or ventricular
B) Amlodipine. tachycardia. “A” is incorrect or two reasons. T e rhythm is ven-
C) Intravenous hydralazine. tricular tachycardia and is stable, and neither metoprolol nor
D) Intravenous esmolol plus nitroprusside. de brillation is appropriate. De brillation could be appropriate
E) Intravenous nitroglycerin. i the patient was unstable, pulseless (including pulseless ven-
tricular tachycardia) or had ventricular brillation. “B” is incor-
Answer 2.10.4 The correct answer is “D.” T e goal o ther- rect because o the inclusion o diltiazem and de brillation. “C”
apy here is not only blood pressure reduction but also control is incorrect because o the inclusion o diltiazem. “D” is incor-
o shear orces on the aorta, which requires the prevention rect because adenosine, which is used or atrial arrhythmias, is
o tachycardia. Intravenous beta-blockers such as labetalol, useless in ventricular arrhythmias and because, again, de bril-
propranolol, metoprolol, or esmolol are the rst-line agents. lation is inappropriate.
CHAPTER 2 • CAr Dio l o Gy 67

FIGURE 2-3. ECG pat ent n quest n 2.11.1.

HELPFUL TIP: HELPFUL TIP:


P ca nam de s n nge n the ACl S p t c s. it s F defibrillation, esca at ng d ses e ect c t a e
st a ea g d d ug u have the t me t ad t. ut the new p t c s. Sta t w th a s ng e sh ck at
360 j u es w th a m n phas c de b at 150 t 200
Question 2.11.2 The patient does not respond to IV amio j u es us ng a b phas c de b at .
darone and you choose to cardiovert him. Which o the ol
lowing is the recommended energy (in joules) or an initial
attempt at synchronized cardioversion?
A) 200 joules, monophasic. You cardiovert the patient, and the rhythm in Figure 2-4 is on
B) 360 joules, monophasic. the monitor. Is it getting hot here, or is it just you?
C) 200 joules, biphasic.
D) 360 joules, biphasic. Question 2.11.3 O the ollowing, what is the rst step you
E) None o the above. will take (while maintaining good compressions and venti
lations, o course)?
Answer 2.11.2 The correct answer is “A.” For cardioversion o A) Reshock the patient at the same energy level.
stable ventricular tachycardia, start with 100 to 200 joules or B) Check another lead to assure the readout is accurate.
monophasic wave orms and 100 joules or biphasic wave orms. C) Give epinephrine, 1 mg IV.
T e rest are incorrect. D) Give atropine, 1 mg.

FIGURE 2-4. ECG pat ent n quest n 2.11.3.


68 FAMil y MEDiCiNE EXAMiNATio N & Bo Ar D r EViEW

Answer 2.11.3 The correct answer is “B.” T is rhythm is asys- Question 2.11.4 The new lead placement continues to show
tole. It is important to quickly check another lead and make asystole. Which o the ollowing drugs and doses are con
sure that all o the leads are connected properly. “A” is incorrect sidered appropriate in asystole?
because cardioversion/de brillation is not routinely indicated A) Epinephrine 1 mg.
in the treatment o asystole. “C” and “D” are incorrect because B) Atropine 0.5 mg.
it is important to ensure that the patient actually is in asystole. C) Atropine 1 mg.
D) Epinephrine 10 mg.
HELPFUL TIP: E) A and C.
W th ega d t ACl S, d ng c mp ess ns and vent a-
t ns a e pa t cu a mp tant. The e s n need t n- Answer 2.11.4 The correct answer is “A.” Atropine is no
tubate the pat ent he/she can be eas bagged. The longer in the ACLS guidelines or asystole. Older ACLS rec-
c ect numbe vent at ns (10–12/m n) and c m- ommendations or asystole included both epinephrine and
p ess ns (100 bpm w th 2 n depth) n a at 15 atropine.
c mp ess ns: 2 b eaths tw p v de CPr 30:1 Objectives: Did you learn to . . .
s ng e p v de s mp tant. The Bee-Gees “Sta - • r ec gn ze and manage vent cu a tach ca d a and as st e?
ng A ve” has the c ect ate c mp ess ns. Queen
• App the cu ent Ame can Hea t Ass c at ns ACl S gu de-
“An the o ne B tes the Dust” a s has the c ect ate nes t a pat ent n as st e?
but s c ns de ed ess dec us n the c de s tuat n (
c u se, u d n’t have t s ng t ut ud!). i u kn w
ne the th se s ngs, u a e n t qu te as d as we a e. CASE 2.12
A 75-year-old emale presents to your o ce complaining o
HELPFUL (AND VERY IMPORTANT) TIP: episodic palpitations with episodes o lightheadedness that
The 2010 and 2015 AHA gu de nes ACl S emphas ze the are not concurrent with the palpitations. You per orm an
mp tance sta t ng w th chest c mp ess ns (C–A–B) and electrocardiogram in your o ce, and the rhythm is shown
NEVEr nte upt ng chest c mp ess ns du ng esusc tat n in Figure 2-5.
( ea that means m n m z ng nte upt ns—the ee that
t ten chest c mp ess ns a e ha ted ess mp tant Question 2.12.1 What rhythm does this represent?
nte vent ns such as ntubat n, ven us access, etc.). A) First-degree heart block.
B) Second-degree heart block type I (Wenckebach).

FIGURE 2-5. ECG pat ent n quest n 2.12.1.


CHAPTER 2 • CAr Dio l o Gy 69

C) Second-degree heart block type II. bradycardia episodes. T us, pacing is necessary. (Note: A pace-
D) T ird-degree heart block. maker would also be indicated in patients with Mobitz type II
E) Atrial f utter with variable block. second-degree heart block without tachy-brady syndrome).
“A” and “C” are incorrect because these two drugs are aimed
Answer 2.12.1 The correct answer is “C.” Your patient’s ECG primarily at ventricular arrhythmias; sick sinus syndrome is a
shows a second-degree heart block, type II (Mobitz II). T is is problem with the SA node. “B” is incorrect because hydralazine
characterized by a xed PR interval with an intermittently non- is an a erload reducer with no direct e ect on cardiac rhythm.
conducting P wave and resultant dropped beats. “A” is incor- “E” is incorrect because patients with sick sinus syndrome do
rect. First-degree heart block is characterized by a prolonged not have ventricular brillation or ventricular tachycardia, and
PR interval without any blocked beats (meaning every QRS is thus there is no need or a de brillator.
preceded by a P wave conducted with a long PR interval). T e
upper limit o normal o the PR interval is 0.2 seconds (and we
admit that this one is darn close, but Mobitz II is the issue here). HELPFUL TIP:
A second-degree heart block, Mobitz type I (Wenckebach), is in s ck s nus s nd me, n add t n t the pacemake , t
de ned by a progressively prolonged PR interval ending with a s ten necessa t add a beta-b cke , d g x n, a
nonconducted P wave and a dropped beat. A third-degree heart CCB (d t azem ve apam ) t add ess the tach ca d a
block is characterized by no consistent pattern between the P (e.g., PSVT at a b at n).
waves and the QRS complex. “E” is incorrect because, by de -
nition, atrial f utter is represented by a rapid atrial rate. In this
patient, the rate is slow. Objectives: Did you learn to . . .
• ident and d f e ent ate sec nd-deg ee hea t b cks?
By the time the patient arrives at the hospital, she is having a • D agn se and t eat s ck s nus s nd me?
rapid, chaotic rhythm, which appears to be atrial brillation
on the monitor. It seems as though there are also episodes o CASE 2.13
atrial utter with 2:1 block.
A 58-year-old male smoker with a history o type 2 diabetes
Question 2.12.2 The most likely diagnosis in this patient mellitus presents with complaints o easy atigability and
with varying rate is: pain in his thighs when exerting himsel . T e le leg is worse
A) Sick sinus syndrome (bradycardia–tachycardia syndrome). than the right. T e pain resolves a er resting and is no worse
B) Hypothyroidism. going downhill than uphill. He works as a carpenter, and the
C) Hyperthyroidism. leg pain is now limiting his ability to work. He will not quit
D) Hyperkalemia. smoking (“It’s the only thing I truly love, Doc”). T e patient
states that his symptoms are better when he hangs his leg over
Answer 2.12.2 The correct answer is “A.”T e most likely diagno- the side o the bed at night.
sis in this patient is “sick sinus syndrome,” also known as “tachy–
brady” syndrome and “bradycardia–tachycardia” syndrome. T is Question 2.13.1 The etiology o this patient’s leg pain is
syndrome is most common in elderly individuals and ref ects the most likely:
replacement o the SA node with brous tissue. “B” is incorrect A) Peripheral venous disease (e.g., venous insu ciency, vari-
because hypothyroidism should cause bradycardia without inter- cose veins).
mittent tachycardia. “C” is incorrect because hyperthyroidism B) Spinal stenosis.
should cause tachycardia without bradycardia. “D” is incorrect C) Diabetic neuropathy.
because hyperkalemia generally causes a widened QRS complex D) Peripheral arterial disease (e.g., arterial stenosis).
on ECG and eventually ventricular tachycardia. Note: With Mob- E) None o the above.
itz type II second degree heart block, the problem is in ra-nodal,
below the AV node and can result in complete heart block. Sick Answer 2.13.1 The correct answer is “D.” Intermittent claudi-
sinus syndrome is due to disease o the SA node. cation is the classic presenting symptom o peripheral arterial
disease. When rest pain is present, relie o symptoms occurs by
Question 2.12.3 De nitive treatment o this syndrome making the a ected area dependent (e.g., hanging the legs over the
generally includes which o the ollowing? side o the bed), letting gravity help increase blood f ow. T e pain
A) Mexiletine. associated with diabetic neuropathy begins distally, has a burning
B) Hydralazine. quality, and is not typically relieved with rest. In act, patients o en
C) Quinidine. notice it more at rest (e.g., during the night). Patients with periph-
D) Pacemaker. eral venous disease will o en have worsening o their symptoms
E) Implantable de brillator. when their legs in a dependent position. Spinal stenosis is o en
made worse by walking downhill and better when walking uphill
Answer 2.12.3 The correct answer is “D.” In general, patients or leaning orward (a kyphotic/ orward f exed position opens up
with sick sinus syndrome become symptomatic because o the the oramen thereby decreasing nerve root compression).
70 FAMil y MEDiCiNE EXAMiNATio N & Bo Ar D r EViEW

0.94 represents mild arterial disease that may or may not be


His examination shows decreased pulses in the lower extrem- associated with claudication. An ABI o 0.41 to 0.80 is classi-
ities bilaterally. You would like to con rm your suspicion ed as moderate arterial disease and is usually associated with
that this patient has peripheral vascular disease. some degree o claudication. An ABI o ≤0.4 represents severe
disease and may be associated with rest pain. Paradoxically, an
Question 2.13.2 What is the rst study you would order in ABI > 1.30 represents noncompressible arteries and may be a
this patient? marker or arterial calci cation. In these cases, a toe-brachial
A) Spiral C to con rm vascular calci cation. index should be measured (really). (Don’t ask us . . . we don’t
B) Ankle-brachial index (ABI). know where to nd a toe BP cu either. Maybe you can use a
C) Color Doppler to assess f ow. ull-sized gnome or pixie cu . We’d love to sell you one).
D) Catheter based arteriography.
E) None o the above is the recommended rst test. You decide to start the patient on a medication to help control
his claudication.
Answer 2.13.2 The correct answer is “B.” T e ABI is sensitive and
speci c or peripheral arterial disease in the lower extremity. T e Question 2.13.5 Which o the ollowing statements is
pressure in the ankle should be higher than that in the brachial correct?
artery in a normal person. T e highest sensitivity is achieved by A) Pentoxi ylline is relatively contraindicated in heart ailure.
measuring pressures in both brachial arteries, both dorsalis pedis, B) Cilostazol is the best choice or claudication in patients with
and both posterior tibial arteries. Neither spiral C or color Dop- heart ailure.
pler are recommended as the initial screening test or the presence C) Beta-blockers are good arterial dilators and are thus use ul
o peripheral vascular disease (although C angiogram may be in claudication.
use ul in the uture to de ne the degree and location o narrowing). D) T e main mechanism o action o pentoxi ylline and cilo-
Catheter based arteriography is an option but should be reserved stazol is selective vasodilation.
or patients with known peripheral artery disease in whom inter- E) obacco smoking paradoxically alleviates claudication.
vention (percutaneous or surgical) is being considered.
Answer 2.13.5 The correct answer is “A.” Cilostazol (Pletal)
T e ABI results are normal. However, you strongly suspect and pentoxi ylline ( rental) are phosphodiesterase inhibitors.
claudication. T eir mechanism o action in improving walking distance is
poorly understood. Other phosphodiesterase inhibitors (such
Question 2.13.3 The next step should be: as milrinone) increase mortality in patients with heart ailure.
A) Catheter based arteriography. T us, pentoxi ylline and cilostazol should be used with extreme
B) Repeat ABI a er an exercise stress test. caution, i at all, in patients with heart ailure. Selective beta-
C) Magnetic resonance arteriography. blockers actually cause peripheral arterial constriction, not
D) C arteriography. arterial dilation. “C” is also incorrect. T e purported bene t o
E) None o the above. pentoxi ylline is to increase RBC malleability and thus reduce
the viscosity o blood in the microcirculation. It has no vasodi-
Answer 2.13.3 The correct answer is “B.” In patients in whom lative e ects. However, cilostazol does have some vasodilative
you strongly suspect peripheral vascular disease, ankle-brachial e ects. T erapeutic bene t with these drugs may take several
indices a er exercise can be positive when a resting test is nega- weeks. A dedicated and supervised walking program is o para-
tive. T is would be the least invasive and most cost-e ective test mount importance and underutilized. Given the strong associa-
o the options given. tion between smoking and peripheral artery disease, smokers
should be encouraged to quit, so “E” is incorrect. Pharmaco-
T e postexercise ankle-brachial indices are as ollows: 0.9 in logic therapy o PAD should include antiplatelet therapy and
the right leg, 0.4 in the le leg. cardiovascular risk actor modi cation.

Question 2.13.4 The proper interpretation o this in orma HELPFUL TIP:


tion is: r emembe that pe phe a a te d sease s c ns de ed
A) 95% probability o some degree o occlusive disease in the a CAD equ va ent.
right leg, severe occlusive disease in the le .
B) No occlusive disease in the right leg, mild disease in the le .
C) Moderate occlusive disease on both sides. Question 2.13.6 In and o themselves, indications or ur
D) No occlusive disease in either leg. ther intervention or peripheral artery disease (e.g., bypass,
stenting) include all o the ollowing EXCEPT:
Answer 2.13.4 The correct answer is “A.” A normal ABI A) Rest pain.
should be 0.95 to 1.29. An ABI o 0.9 is 95% sensitive or nding B) Persistent pain that inter eres with day-to-day unctioning.
some degree o occlusive disease on arteriography (although it C) issue loss.
may not be hemodynamically signi cant). An ABI o 0.81 to D) 80% occlusion o the emoral artery.
CHAPTER 2 • CAr Dio l o Gy 71

Answer 2.13.6 The correct answer is “D.” Classic indications Answer 2.14.1 The correct answer is “D.” As a general rule, aor-
or invasive treatment o lower extremity PAD are (1) salvage tic stenosis is repaired when it becomes symptomatic. Repair o
o a threatened limb (rest pain, nonhealing ulceration, or gan- asymptomatic severe aortic stenosis is indicated in the ollowing
grene) and (2) improvement in unctional capacity. An 80% scenarios: undergoing CABG or other valve or aorta surgery, LVEF
occlusion o the emoral artery, in and o itsel , is not an indica- < 50%, hypotension in response to exercise, or high likelihood o
tion or percutaneous or surgical revascularization in a patient rapid progression. “A” is incorrect because the loudness o the
who is asymptomatic. murmur does not always correlate with its unctional signi cance.
“B” is incorrect as well. As long as the lesion is not hemodynami-
T e patient sees the light, but does not go into it, and quits cally signi cant, the patient should tolerate prostate surgery. “C” is
smoking. T e case ends happily . . . incorrect because surgery is not usually necessary even in severe
valvular disease without symptoms as long as the le ventricular
HELPFUL TIP: unction is normal. (Note: it is not uncommon that patients with
N ant c agu at n eg men seems t p event e cc u- severe aortic stenosis report that they are asymptomatic, but they
s n we ext em t a te es a te stent ng. Pat ents have modi ed their activity to avoid symptoms which may have
sh u d, h weve , c nt nue n the cu ent ant th m- occurred gradually so that they don’t recognize the decline.)
b t c eg men as nd cated ca d vascu a d sease
(ASA, wa a n, etc.). Question 2.14.2 The patient would like to know how o ten
he should have a repeat echocardiogram given that he has
Objectives: Did you learn to . . . mild disease. Your answer is:
• r ec gn ze s mpt ms and s gns pe phe a vascu a d sease?
A) Every 3 to 5 years.
B) Every year.
• o de app p ate d agn st c tests a pat ent suspected
hav ng pe phe a vascu a d sease?
C) Every 6 months.
D) When he develops symptoms.
• Deve p an unde stand ng the agents used t t eat pe-
phe a vascu a d sease?
E) None o the above.

Answer 2.14.2 The correct answer is “A.” Patients with mild


CASE 2.14 aortic stenosis who are asymptomatic can be ollowed by echo-
cardiogram every 3 to 5 years. Patients with severe disease
A 75-year-old male presents to your o ce or a complete should have yearly echocardiography to evaluate or le ven-
physical examination be ore prostate surgery. On examina- tricular dys unction. See able 2-7.
tion, you notice a 3/6 harsh, mid-systolic ejection murmur
hard best at the upper right sternal border and radiating to wo years later, the patient returns or a checkup and states
the neck. S1 and S2 are normal. An echocardiogram notes that he believes he has been having symptoms rom his aortic
mild aortic stenosis. Currently he is asymptomatic. stenosis.

Question 2.14.1 The indications or valve replacement Question 2.14.3 All o the ollowing can occur with symp
surgery include: tomatic aortic stenosis EXCEPT:
A) Grade 4/6 murmur. A) Le -to-right intracardiac shunt.
B) Requirement or major, semi-elective surgery such as pros- B) Exertional dyspnea.
tatectomy. C) Syncope.
C) Severe aortic stenosis without symptoms and normal LV D) Angina.
unction. E) Lightheadedness.
D) Severe aortic stenosis in a patient undergoing coronary
bypass gra ing. Answer 2.14.3 The correct answer is “A.” Intracardiac shunts
E) All o the above. don’t occur with aortic stenosis. I you got this one wrong, back

TABLE 2-7 RECOMMENDED INTERVALS FOR ECHOCARDIOGRAPHIC EVALUATION


FORVALVULAR DISEASEa
Lesion Mild Disease Moderate Disease Severe Disease
A t c sten s s Eve 3–5 ea s Eve 2 ea s Eve 6–12 m nths

M t a sten s s Eve 3–5 ea s Eve 1–2 ea s Eve 1–2 ea s

A t c egu g tat n Eve 3–5 ea s Eve 1–2 ea s Eve 6–12 m nths

M t a egu g tat n Eve 3–5 ea s Eve 1–2 ea s Eve 6–12 m nths

a
r ec mmendat ns va .
72 FAMil y MEDiCiNE EXAMiNATio N & Bo Ar D r EViEW

to anatomy or you! An isolated, xed valvular lesion as an adult Question 2.15.1 What is your initial approach to this
cannot cause intracardiac shunting. Exertional dyspnea, light- patient?
headedness (presyncope), syncope, and chest pain are common A) Start a chronic antihypertensive since he is at risk or a stroke
symptoms in severe aortic stenosis. within the next couple o days with a blood pressure at this
level.
Question 2.14.4 Which o the ollowing statements about B) Administer clonidine in the o ce to reduce the blood pres-
aortic valve disease is INCORRECT? sure to a sa e level o about o 150/100 mm Hg.
A) Aortic stenosis can be treated quite e ectively with valvu- C) Watch the patient over the next 2 weeks and get additional
lotomy (balloon aortic valvuloplasty). blood pressure readings be ore deciding what to do and
B) T ere are no known medical treatments that reduce the instruct him to discontinue pseudoephedrine.
need or aortic valve replacement. D) Schedule the patient or outpatient labs and electrocardio-
C) Risk actors or the development o aortic stenosis are simi- gram.
lar to CAD. E) Fire the patient rom your practice. He’s messing up your
D) Valve replacement surgery is the pre erred treatment o quality measures.
symptomatic aortic stenosis.
Answer 2.15.1 The correct answer is “C.” T e diagnosis o
Answer 2.14.4 The correct answer (and what doesn’t work) hypertension requires two elevated blood pressures on two di -
is “A.” Valvulotomy (balloon aortic valvuloplasty) is not a long- erent occasions. T is patient’s elevated blood pressure could be
term solution or the management o severe symptomatic aortic situational, related to decongestants and current illness. Neither
stenosis. While it may be indicated as a “bridge” to de nitive “A” nor “B” is correct because a blood pressure o 175/103 mm
treatment (surgery or transcatheter aortic valve replacement Hg does not pose a risk o acute stroke, and the pressure need
[ AVR]), durability o the valvulotomy results are poor, only not be lowered acutely unless there is evidence o end-organ
lasting 3 to 6 months. T e procedure carries the attendant risk injury (e.g., angina, heart ailure, hypertensive encephalopathy).
o cerebral embolism causing stroke, aortic rupture, or acute “D” is incorrect because you cannot de nitively establish that
severe aortic insu ciency. Valve replacement surgery is pre- this patient has hypertension based on only one in o ce blood
erred although AVR (see below) can be considered in the pressure measurement. As or “E” . . . really? Is this why we went
nonsurgical candidate. T e epidemiological risk actors or aor- into medicine?
tic stenosis and CAD are similar (as is the pathophysiology).
Un ortunately, there are no drugs that are e ective at reducing
T e patient returns to your o ce with blood pressures mea-
the need or valve replacement. You can provide symptomatic
sured six times over a period o 2 weeks at a local pharmacy.
relie but that is all.
Only three o the six readings suggest that the patient is
hypertensive. T e patient states that the elevated blood pres-
HELPFUL TIP: sures were while he was under stress at work.
in pat ents wh a e n t cand dates pen su g ca
a t c va ve ep acement, a cathete -based app ach Question 2.15.2 Your best response at this point is to:
ep acement the a t c va ve (t anscathete a t c A) Start an antihypertensive.
va ve ep acement “TAVr ”) s ava ab e. TAVr appea s B) Send the patient or a 24-hour ambulatory blood pressure
t educe ve a m ta t c mpa ed w th standa d measurement.
n nsu g ca ca e but nc eases the sk st ke. C) Don’t worry about the blood pressure since the majority o
the readings were within a normal range.
D) Get a nephrology consult to help in decision making.
E) Make another o ce visit so you can buy that Porsche.
Objectives: Did you learn to . . .
• r ec gn ze s mpt ms a t c sten s s?
• Manage a pat ent w th a t c sten s s? Answer 2.15.2 The correct answer is “B.” One way to deter-
• Eva uate a t c va ve d sease and dete m ne ng-te m
mine i a patient with contradictory readings is hypertensive
w-up v s-à-v s pe d c ech ca d g ams? is to per orm 24-hour ambulatory blood pressure monitoring.
T is can be use ul in patients who have elevated blood pres-
sures in the o ce but not at home or vice versa. It can also be
used i you do not trust the blood pressure readings taken out-
CASE 2.15 side o your o ce. “A” is incorrect since we have not yet estab-
A 35-year-old male presents to the o ce with upper respi- lished that this patient is hypertensive. “C” is incorrect since we
ratory symptoms. He is taking no medications except or a have not yet established that this patient is not hypertensive.
bit o pseudoephedrine or his cold. You notice when look- “D” is incorrect because you are smarter than that and should
ing at his vital signs that his blood pressure is 180/106 mm be able to work through this kind o case yoursel ! As to “E,”
Hg. Repeat measurement con rms that the blood pressure is whoops, we orgot. We’re amily physicians, not radiologists.
elevated at 175/103 mm Hg. No Porsche or us!
CHAPTER 2 • CAr Dio l o Gy 73

Question 2.15.3 The ollowing are all well accepted indica Question 2.15.6 The initial evaluation o the hypertension
tions or 24 hour ambulatory blood pressure monitoring includes the ollowing:
EXCEPT: A) History, physical, CBC, urinalysis, glucose, BUN, creatinine,
A) Suspected white coat hypertension. electrolytes, ECG, and lipids.
B) Patients with di cult-to-control hypertension. B) History, physical, CBC, uric acid, glucose, BUN, creatinine,
C) Patients having hypotensive symptoms on their antihyper- electrolytes, and lipids.
tensive treatment. C) History, physical, CBC, urinalysis, glucose, BUN, creatinine,
D) Follow-up a er initiating antihypertensive treatment. electrolytes, ECG, lipids, and echocardiography.
E) Evaluation o patient or autonomic dys unction. D) History, physical, and labs only as indicated by history and
physical.
Answer 2.15.3 The correct answer is “D.” One need not do
24-hour ambulatory blood pressure monitoring to document Answer 2.15.6 The correct answer is “A.” History, physical,
response to antihypertensive therapy in patients in whom CBC, urinalysis, glucose, BUN, creatinine, electrolytes, ECG,
most or all measurements posttreatment are normal. All o the and lipids are the generally agreed-upon initial work-up o the
other answer choices are considered reasonable indications or hypertensive patient. Do we really need all o these? T at is
24-hour ambulatory blood pressure monitoring. an interesting question, but we’re just stating what is recom-
mended by the experts. “C” includes echocardiography, which
Elevated blood pressure in response to stress (especially in is not recommended as part o the routine evaluation but may
the doctor’s o ce) is called “white coat hypertension.” be indicated i signs o cardiac disease are present.

Question 2.15.4 Which o the ollowing statements is true T e patient’s ECG comes back showing evidence o LVH.
about white coat hypertension?
A) As long as the majority o blood pressure readings are nor- Question 2.15.7 This nding suggests that:
mal, the patient does not require treatment because there is A) You should initiate this patient’s therapy with an ACE inhib-
no increased risk o adverse cardiac outcomes. itor since ACE inhibitors prevent disadvantageous cardiac
B) Patients with white coat hypertension have an intermedi- remodeling.
ate risk or adverse outcomes when compared with patients B) T e patient has heart ailure.
with normal blood pressure and those with chronically ele- C) You should recommend an echocardiogram or this patient.
vated blood pressure. D) You should order a BNP level to screen or LVH and early
C) White coat hypertension is more common in young patients. heart ailure.
D) Patients with white coat hypertension have an elevated le
ventricular mass when compared to patients with normal
blood pressures. Answer 2.15.7 The correct answer is “C.” T e sensitivity o
E) B and D. ECG or LVH is only in the 30% to 60% range with a speci city
o 80%. T us, a “positive” ECG is not a strong enough indication
Answer 2.15.4 The correct answer is “E.” Patients with white to initiate therapy or LVH. For this reason, an echocardiogram
coat hypertension have outcomes that are intermediate between should be done to con rm the diagnosis o LVH. “A” is incor-
normotensive and hypertensive patients. In addition, they have rect since an ACE inhibitor is not necessarily the rst drug one
an elevated le ventricular mass. Surprisingly, white coat hyper- would start. In addition, we really don’t know i this patient has
tension is more common in the elderly. LVH yet, though ACE inhibitors do prevent harm ul cardiac
remodeling. “B” is incorrect. Certainly, long-standing hyperten-
Question 2.15.5 Hypertension is de ned as an ambulatory sion and signi cant LVH can cause heart ailure. However, we
24 hour monitor average blood pressure o : cannot conclude that this patient has heart ailure on the basis
A) 135/85 mm Hg during the day and 125/75 mm Hg at night. o an ECG, especially in the absence o symptoms. “D” is incor-
B) 140/90 mm Hg during the day and 130/85 mm Hg at night. rect since the sensitivity o the BNP as a screening tool in an
C) 130/85 mm Hg over 24 hours. asymptomatic population is poor.
D) 140/90 mm Hg over 24 hours.
T e echocardiogram is normal. You have decided to start this
Answer 2.15.5 The correct answer is “A.” Patients with an patient on treatment or his hypertension.
average blood pressure o >135/85 mm Hg during the day and
>125/75 mm Hg at night are de ned by JNC 7 as being hyperten- Question 2.15.8 Based on outcome data, the LEAST ef ec
sive; JNC 8 has not changed this. Another published criterion is tive drug to start on this patient?
a blood pressure o >140/90 mm Hg more than 40% o the time. A) An ACE inhibitor, such as lisinopril.
B) An ARB, such as losartan.
T e ambulatory blood pressure monitor reveals that the C) An alpha-blocker, such as doxazosin.
patient’s blood pressure is >140/90 mm Hg more than 40%o D) A thiazide diuretic, such as chlorthalidone.
the time, indicating that he is indeed hypertensive. E) A CCB, such as amlodipine
74 FAMil y MEDiCiNE EXAMiNATio N & Bo Ar D r EViEW

Answer 2.15.8 The correct answer is “C.” Alpha-blockers have Answer 2.15.10 The correct answer is “B.” Doxazosin is an
worse outcomes in hypertension when compared to other anti- alpha-blocker that is use ul in the treatment o symptomatic
hypertensives. T e JNC 8 recommendations suggest that the BPH. None o the other choices can be used or this indication.
rst line agent, in the general nonblack population, including O course, alpha-blockers are also antihypertensives, and thus
those with diabetes, could appropriately be a thiazide-type serve a use ul purpose by killing two birds with one stone (Why
diuretic, CCB, ACEI or ARB. Notice that beta-blockers do not would you want to kill two birds? And why with stones? Isn’t
make the list o rst-line antihypertensives either. For the gen- there a better way?). Note: T ere is some evidence that alpha-
eral black population, including those with diabetes, the ini- blockers do not con er as much bene t or the hypertensive
tial antihypertensive treatment should include a thiazide-type patient as other classes o drugs. T us, alpha-blockers are not
diuretic or a CCB. the best choice in general but could be used as the initial agent
i you have a compelling reason.
Question 2.15.9 Time to digress a bit. Which o the ollow T e point o these digressions is that you should look at the
ing drugs is the best choice as your initial agent or the patient’s other underlying illnesses when deciding what to rec-
treatment o hypertension in a patient with diabetes and ommend at initial therapy. Another example would be a patient
known microalbuminuria? with CAD and angina starting on a beta-blocker as initial treat-
A) Lisinopril. ment rather than a thiazide (since the beta-blocker may improve
B) Metoprolol. angina symptoms and is indicated or CAD). Recall that not all
C) Losartan. beta-blockers are created equal. Atenolol is least pre erred and
D) Verapamil. metoprolol succinate is among the best. I the patient has renal
E) Amlodipine. disease, consider an ACE inhibitor as rst-line treatment or
hypertension.
Answer 2.15.9 The correct answer is “A.” In a diabetic patient
who has proteinuria, an ACE inhibitor is indicated to slow down
the progression o renal disease. An ARB or nondihydropyridine HELPFUL (AND IMPORTANT) TIP:
CCB (verapamil, diltiazem) are viable alternatives or those who Beta-b cke s have a en ut av as a st- ne
cannot tolerate an ACE inhibitor. (Note: Some elderly patients ch ce h pe tens n. The d n’t seem t c n e as
may not tolerate verapamil or diltiazem due to bradycardia or much su v va bene t as the ant h pe tens ve c asses.
may not be candidates due to LV systolic dys unction). However, The ne except n s n pat ents w th CAD CHF.
ACE inhibitors are still rst line. T ese recommendations stem
rom the renal and cardiac bene ts o ACE inhibitors.

HELPFUL TIP: Remember the 35-year-old guy? You start him on chlortha-
A th ugh th az de d u et cs dec ease the e t vent cu a lidone, but his blood pressure does not respond at a dose o
d amete (due t d u es s), beta-b cke s, CCBs, and ACE 12.5 mg/day (have your patients cut the 25-mg tabs in hal ).
nh b t s all eve se l VH. His blood pressure on ollow up is 148/96 mm Hg.

Question 2.15.11 The best approach or such a patient is to:


HELPFUL TIP: A) Push his chlorthalidone to 25 mg daily be ore starting
Expe ts (and the te atu e) g back and th between another medication.
ch s ng the d standb , h d ch th az de (HCTZ), B) Stop the chlorthalidone and start another medication.
and the even de d ug, ch tha d ne. H p ka em a C) Rely on exercise and diet to normalize the blood pressure.
s m e equent seen w th ch tha d ne than w th D) Start a second drug be ore you have maximized the dose o
HCTZ. P tass um sh u d be m n t ed pe d ca w th the rst drug.
d u et cs an wa , and h p ka em a ten c mp cates E) Start a work-up or secondary causes o hypertension.
h pe tens n. F) A or D.

Answer 2.15.11 The correct answer is “F.” Per the JNC 8


Question 2.15.10 Digressing a bit urther . . . Which o the guidelines, both “A” and “D” are acceptable strategies; you
ollowing drugs might you want to use6 as your initial could push up the dose o a rst drug or add a second drug.
agent or the treatment o hypertension in a 72 year old T ere is a lack o randomized controlled trials to guide these
male who you also diagnosed with symptomatic benign recommendations. JNC 8 urges us to tailor therapy based on
prostatic hypertrophy? individual circumstances, clinician and patient pre erence, and
A) Amlodipine. drug tolerability. Low-dose chlorthalidone (12.5 mg) provides
B) Doxazosin. the greatest blood pressure reduction per mg o drug, and
C) Captopril. there is little clinical bene t o utilizing >25 mg daily o HC Z
D) Losartan. or chlorthalidone. Higher doses are associated with increased
E) Verapamil. adverse e ects with minimal clinical gain in hypertension
CHAPTER 2 • CAr Dio l o Gy 75

management. “B” is incorrect because a patient with this level TABLE 2-8 CAUSES OF SECONDARY HYPERTENSION
o blood pressure elevation will generally require more than
D ugs, nc ud ng ve -the-c unte med cat ns
one drug to achieve a normalized blood pressure. “C” is incor-
S eep apnea
rect because the majority o patients are unable to maintain
an adequate diet or exercise regimen to e ectively treat blood End c ne:
• H pe a d ste n sm
pressure. Exercise and dietary change are certainly laudable • Phe ch m c t ma
goals and should be encouraged in all patients. However, they • Th d d sease
are not likely to normalize blood pressure in most hyperten- • Cush ng s nd me ( nnate at gen c)
sive patients. “D” is also correct as it represents one o the Vascu a :
acceptable JNC 8 guideline strategies to dose antihypertensive • r ena a te sten s s
drugs. “E” is incorrect since this patient has not yet proven to • C a ctat n the a ta
be resistant to treatment. int ns c ena d sease

HELPFUL TIP: to control hypertension. Secondary causes o hypertension


An the st ateg the n t a t eatment h pe ten- include: hyperaldosteronism, RAS, pheochromocytoma, Cush-
s n, depend ng n the pat ent’s b d p essu e, s t ing disease, untreated obstructive sleep apnea, primary hyper-
sta t the ap w th tw d ugs at submax ma d ses (e.g., parathyroidism, medications (OCP), and others. “A” is the thing
s n p 10 mg and HCTZ 12.5 mg), ec gn z ng that to avoid because when checking the serum potassium level or
m st pat ents w eventua need tw d ugs. hyperaldosteronism, the patient must be o all diuretic medi-
cations and have an unrestricted salt intake. All o the others
choices can be a part o a work-up or secondary hypertension
You decide to start this patient on diltiazem as a second agent. caused by RAS (“B”), Cushing disease (“C”), and pheochromo-
cytoma (“D”), respectively (see able 2-8).
Question 2.15.12 Which o the ollowing side ef ects is most
characteristic o diltiazem and other CCBs? Question 2.15.14 You decide to check this patient or RAS.
A) Dehydration. The best choice or a screening test or RAS is:
B) Cough. A) Doppler ultrasound.
C) Dependent edema. B) Captopril renal scan.
D) Hypokalemia. C) Serum renin level.
E) Elevated cholesterol. D) MR or C angiography.
E) Both A and D.
Answer 2.15.12 The correct answer is “C.” As a class, CCBs
tend to cause peripheral edema. Dehydration and hypoka- Answer 2.15.14 The correct answer is “E.” Per the ACC/AHA
lemia can be caused by diuretics. Cough and hyperkalemia guideline or diagnosing RAS, duplex ultrasonography, C A
are characteristic o ACE inhibitors and ARBs. Diuretics in patients with normal renal unction, and MRA are the rec-
can increase cholesterol, while beta-blockers can increase ommended screening tools or diagnosis o RAS (Class I Level
triglycerides. o Evidence B). T e captopril renal scintigraphy, selective renal
vein renin measurements, plasma renin activity and the cap-
Despite the act that the patient is on two medications, he topril test (which includes measurement o plasma renin lev-
remains hypertensive. In act, the blood pressure has barely els at baseline and a er captopril administration) are consider
moved. With your thorough history taking, you have ruled Class III or screening tests or establishing the diagnosis o
out excess alcohol intake (o en an “occult” cause o hyper- RAS.
tension). T e patient is compliant with his medications.

Question 2.15.13 Further investigations that might be HELPFUL TIP:


help ul in determining the cause o hypertension in this C ns de an eva uat n r AS n a pat ent wh has a
patient include all o the ollowing EXCEPT: “p s t ve” c n ca ( nadve tent) “capt p cha enge.” i
A) Checking the potassium level while the patient is taking his u sta t an ACE nh b t and see a d amat c dec ne
current medications to rule out hyperaldosteronism. n ena unct n n a ew da s, r AS, espec a b ate a
B) Assessing or renal artery stenosis (RAS). r AS, ma be the cu p t.
C) Checking a 24-hour urine or glucocorticoids.
D) Checking a 24-hour urine or catecholamines.
Your patient does not have any identi able cause or second-
Answer 2.15.13 The correct answer is “A.” Hypertension is ary hypertension. You add a third agent, and his blood pres-
secondary to another cause in about 1% o patients with mild sure comes under control. Sometimes, you just have to be
hypertension but in 10% to 45% o those with severe, di cult persistent!
76 FAMil y MEDiCiNE EXAMiNATio N & Bo Ar D r EViEW

Question 2.16.2 Which o the ollowing is true?


HELPFUL TIP:
A) Many patients with hyperaldosteronism have normal serum
Add ng sp n act ne t the eg men a pat ent
potassium.
w th d cu t t c nt b d p essu e can ten
B) In hyperaldosteronism, the plasma aldosterone-to-renin
be he p u even n the p esence an the d u et c.
ratio is usually high.
Watch h pe ka em a when sp n act ne s used
C) All antihypertensives should be stopped be ore checking a
w th ACE nh b t s. Ep e en ne s an acceptab e
plasma renin level.
a te nat ve.
D) I a con rmatory 24-hour urine is done, the urine potassium
should be low to con rm the diagnosis o hyperaldosteronism.
Objectives: Did you learn to . . . E) A and B.
• Eva uate a pat ent w th n t a h gh b d p essu e ead ngs?
• Se ect n t a ant h pe tens ve the ap ?
Answer 2.16.2 The correct answer is “E.” Many patients with
hyperaldosteronism will have normal serum potassium levels.
• App p ate ta the t eatment h pe tens n, based n
pat ent-spec c cha acte st cs?
In addition, the plasma aldosterone-to-renin level is usually
high. “C” is incorrect because, although ACE inhibitors and
• Use and nte p et 24-h u ambu at b d p essu e m n -
t ng?
spironolactone (and perhaps all diuretics) should be stopped
be ore renin and aldosterone levels are drawn, other antihy-
• Unde stand the c ncept wh te c at h pe tens n?
pertensives (e.g., CCBs) will have little e ect on plasma renin
• Gene ate a d f e ent a d agn s s and an app p ate eva ua-
levels. “D” is incorrect because hyperaldosteronism causes
t n sec nda h pe tens n?
potassium wasting, so the urine potassium should be elevated.

You diagnose this patient with hyperaldosteronism.


CASE 2.16
You have a patient who is mildly hypertensive and decide Question 2.16.3 The most common cause o hyperaldoste
to check baseline labs. On no medications whatsoever, the ronism is:
patient’s potassium is low at 3.0 mEq/L. You re-check the A) Adrenal adenoma.
potassium be ore getting too excited. It is 2.9 mEq/L. B) Idiopathic.
C) Pituitary adenoma.
Question 2.16.1 O the ollowing, the MOST LIKELY cause D) Aldosterone-secreting tumor such as small-cell carcinoma.
o low potassium in this patient is: E) RAS.
A) Hyperaldosteronism.
Answer 2.16.3 The correct answer is “A.” Adrenal adenomas
B) Hypoaldosteronism.
are the most common cause o hyperaldosteronism. T e second
C) Spuriously low potassium because o an elevated glucose.
leading cause is idiopathic.
D) Metabolic acidosis.
Question 2.16.4 Accepted approaches to the treatment o
Answer 2.16.1 The correct answer is “A.” Hyperaldosteron- hypertension caused by hyperaldosteronism include all o
ism can cause hypokalemia and hypertension. Aldosterone the ollowing EXCEPT:
increases the secretion o potassium, which leads to hypokale- A) Unilateral adrenalectomy in the case o adrenal adenoma.
mia. “B” is incorrect because hypoaldosteronism, such as that B) Liberalized sodium intake.
seen with adrenal ailure secondary to adrenal destruction, C) Use o a potassium-sparing diuretic.
causes hyperkalemia and hypotension. “C” is incorrect because D) Use o a combination o amiloride and H CZ.
elevated glucose does not result in a spuriously low potassium;
i you answered “C,” maybe you were thinking o sodium (the Answer 2.16.4 The correct answer is “B.” Liberalizing sodium
sodium goes down by approximately 1.6 to 2 mEq/L or every intake will actually cause volume expansion, which is counter-
100 mg/dL increase in the glucose). Finally, “D” is incorrect productive and can lead to urther hypokalemia. Once the patient
because a metabolic acidosis should cause an elevated potas- is hypervolemic, there will be a spontaneous diuresis (the so-
sium rather than a low one. called “aldosterone escape”) leading to increased hypokalemia.
T e exact mechanism o aldosterone escape is not known, but it
occurs a er a weight gain o approximately 3 kg rom f uid reten-
HELPFUL TIP:
tion. I you want to sound smart, just say it is “neurohumeral.”
The se um p tass um g es up b app x mate
You will probably be right and it makes you sound cool.
1 mEq/l eve 0.1 dec ease n the pH m 7.4. Thus,
the p tass um w u d g m 4 t 6 mEq/l the pH Objectives: Did you learn to . . .
changes m 7.4 t 7.2. • ident ab at abn ma t es that ccu n h pe a d ste-
n sm?
• Eva uate a pat ent suspected hav ng h pe a d ste n sm?
You suspect that the patient has hyperaldosteronism. • in t ate t eatment h pe a d ste n sm?
CHAPTER 2 • CAr Dio l o Gy 77

FIGURE 2-6. r h thm st p pat ent n quest n 2.17.1.

observation as long as any underlying cardiac disease is


QUICK QUIZ: ACE iNHiBiTo r S treated. You should also stop any medications that might be
contributing to this rhythm disturbance, such as digoxin,
Which o the ollowing side e ects is/are associated with the use beta-blockers and other AV node blocking agents. Pacemaker
o ACE inhibitors? is appropriate or a ew select patients, usually those with
A) Cough. symptoms. Atropine is used in the emergent setting or treat-
B) Dependent edema. ment o bradycardia.
C) Hypokalemia.
D) Angioedema. Question 2.17.3 What is the proper diagnosis o the ECG
E) A and D. shown in Figure 2 8?
A) Anterior wall myocardial in arction.
The correct answer is “E.” Both chronic dry cough and angio- B) Posterior wall myocardial in arction.
edema (more common in blacks or in patients with hereditary C) Pericarditis.
angioedema) are side e ects o ACE inhibitors. Hyperkalemia D) Hyperkalemia.
is another potential concern. T ese side e ects may not occur E) In erior wall myocardial in arction (IMI).
immediately. Hence, you should be wary o these symptoms
in any patient on an ACE inhibitor or any period o time. Answer 2.17.3 The correct answer is “E.” T is is an IAMI. Note
the S elevations in leads II and III, and aVF with reciprocal
changes in leads V2–V5 (see indicator arrows in Fig. 2-9).
CASE 2.17: Rhythm Strips
Question 2.17.1 What is the rhythm on the rhythm strip A patient presents with a history o lightheadedness when he
shown in Figure 2 6? stands and has the ECG shown in Figure 2-10.
A) Second-degree heart block, type I.
B) Second-degree heart block, type II. Question 2.17.4 What is the rhythm?
C) T ird-degree heart block with junctional escape rhythm. A) Atrial f utter with 4:1 block.
D) Sinus rhythm with nonconducted PACs. B) Atrial brillation with slow ventricular response.
C) Atrial tachycardia with third-degree heart block.
Answer 2.17.1 The correct answer is “A.” T is is a Wenckebach D) Mobitz type I (Wenckebach).
block, also known as second-degree heart block type I or Mob-
itz type I AV block. Note the progressive prolongation o the PR Answer 2.17.4 The correct answer is “C.” T is is an atrial
interval be ore a nonconducted P wave on the rhythm strip in tachycardia with a third-degree heart block. T e P wave pre-
Figure 2-7 (arrows indicate P waves). ceding each QRS complex is indicated with an arrow on the
ECG shown in Figure 2-11. Note that there is no consistent
Question 2.17.2 The proper treatment o an asymptomatic relationship between the P waves and the QRS complexes (i.e.,
patient with this rhythm is: the PR interval varies and there is no predictability), giving the
A) reat any underlying causes identi ed and observe. diagnosis third-degree heart block.
B) Place temporary pacemaker ollowed by permanent pace-
maker. Question 2.17.5 The appropriate treatment o this patient
C) Give atropine ollowed by permanent pacemaker. with atrial tachycardia and third degree block is:
D) Re er or an electrophysiologic study. A) Pacemaker.
B) Isoproterenol.
Answer 2.17.2 The correct answer is “A.” Wenckebach/ C) Lidocaine.
second-degree heart block type I can be treated with D) Atropine.

FIGURE 2-7. A ws sh w P waves w th p g ess ve nge Pr nte va .


78 FAMil y MEDiCiNE EXAMiNATio N & Bo Ar D r EViEW

FIGURE 2-8. ECG pat ent n quest n 2.17.3.

FIGURE 2-9. A ws sh w ST segment dep ess ns n ii, iii and aVF w th ec p ca dep ess ns n V2, V3 and V4.
CHAPTER 2 • CAr Dio l o Gy 79

FIGURE 2-10. ECG pat ent n quest n 2.17.4.

FIGURE 2-11. A ws sh w P waves p t each Qr S. N te the unp ed ctab e va at n n Pr nte va s.


80 FAMil y MEDiCiNE EXAMiNATio N & Bo Ar D r EViEW

Answer 2.17.5 The correct answer is “A.” T e treatment o a treat the ischemia rst. I the rhythm is not causing any problem,
third-degree heart block is a pacemaker. Atropine will increase observation is good or now.
the atrial rate, but that is not the problem here. T e problem is
AV conduction. Isoproterenol will increase the ventricular rate Question 2.17.7 The rhythm shown in Figure 2 14 is best
but is arrhythmogenic and may cause hypotension. Lidocaine is described as:
not indicated in this patient. A) Atrial f utter with 2:1 block.
B) 2:1 second-degree heart block, Mobitz type II.
C) Sinus bradycardia.
HELPFUL TIP FOR THE RHYTHM CONNOISSEUR: D) T ird-degree heart block.
At a tach ca d a w th b ck s “c ass c” d g ta s nt x-
cat n. i th s pat ent we e n d g x n, u w u d t eat Answer 2.17.7 The correct answer is “B.” T is rhythm strip
w th D g b nd. represents second-degree heart block, Mobitz type II. Notice
that the PR interval is constant and there are dropped beats (see
Question 2.17.6 The drug o choice or the rhythm in Fig. 2-15, with indicator arrows showing P waves with no asso-
Figure 2 12 is: ciated QRS complexes). T is patient needs a pacemaker.
A) Atropine.
B) Procainamide. Question 2.17.8 The electrocardiogram shown in Figure 2 16
C) Quinidine. is consistent with which o the ollowing?
D) Metoprolol. A) Pericardial e usion.
E) Lidocaine. B) Pneumothorax.
C) PE.
Answer 2.17.6 The correct answer is “D.” T is is an acceler- D) Cardiac contusion.
ated junctional rhythm that generally occurs only in the set- E) None o the above.
ting o cardiac ischemia. Note the absence o P waves. Using a
Class I antiarrhythmic can extinguish this rhythm, causing asys- Answer 2.17.8 The correct answer is “A.” T is is an example
tole (usually considered a bad outcome . . . ). T e patient also o electrical alternans. Note the low QRS voltages that alternate
has in erior wall ischemia (see Fig. 2-13, with indicator arrows in height rom beat to beat. T is type o pattern is seen with
showing depressed S segments in the in erior leads). Slowing pericardial e usion. It is a late nding and one should be very
down this rhythm with metoprolol is acceptable, but you should concerned about tamponade.

FIGURE 2-12. ECG pat ent n quest n 2.17.6.


CHAPTER 2 • CAr Dio l o Gy 81

FIGURE 2-13. Acce e ated junct na h thm; a ws sh w n e ST segment dep ess n.

Question 2.17.9 The ECG in Figure 2 17 is consistent with Objectives: Did you learn to . . .
which o the ollowing? • ident seve a t pes ca d ac a h thm as, nc ud ng hea t
A) Anterior myocardial in arction. b ck and junct na h thm?
B) Anterolateral myocardial in arction. • D agn se iMi b ECG?
C) Pericarditis. • Desc be ECG eatu es pe ca d a ef us n and pe ca d t s?
D) Early repolarization.
E) Everywhere in arction
CASE 2.18: More Rhythm Strips
Answer 2.17.9 The correct answer is “C.” his ECG is A 24-year-old emale presents to the ED with a history o
consistent with pericarditis. his ECG demonstrates several tachycardia and the rhythm strip shown in Figure 2-19. Her
indings that indicate pericarditis, including sinus tachy- blood pressure is 115/70 mm Hg with an oxygen saturation o
cardia, di use S elevations, and PR depression (see also 98%on room air. T ere are no associated symptoms o chest
Fig. 2-18). pain, dyspnea, etc.

FIGURE 2-14. r h thm st p pat ent n quest n 2.17.7.

FIGURE 2-15. Sec nd deg ee hea t b ck, M b tz t pe ii.


82 FAMil y MEDiCiNE EXAMiNATio N & Bo Ar D r EViEW

FIGURE 2-16. ECG pat ent n quest n 2.17.8.

FIGURE 2-17. ECG pat ent n quest n 2.17.9.


CHAPTER 2 • CAr Dio l o Gy 83

FIGURE 2-18. ECG c ns stent w th pe ca d t s.

Question 2.18.1 The appropriate treatment o this patient is: Question 2.18.2 This ECG represents:
A) Adenosine 6 mg IV ollowed by 12 mg IV. A) Normal ECG.
B) Diltiazem 5 mg/kg IV. B) Wol –Parkinson–White (WPW) syndrome.
C) Verapamil 25 mg IV. C) RBBB.
D) Digoxin 0.5 mg IV. D) Right axis deviation.
E) De brillation. E) LVH.

Answer 2.18.1 The correct answer is “A.” T is rhythm is Answer 2.18.2 The correct answer is “B.” T is is an ECG dem-
PSV . T ere are several treatment options or PSV , which onstrating WPW pattern. When combined with documented
include adenosine, diltiazem, and verapamil. However, “B” tachyarrhythmia, it is re erred to as WPW syndrome. Note the
and “D” are incorrect because the dose or diltiazem is 0.25 short PR interval as well as the delta wave (Fig. 2-21).
mg/kg IV, not 5 mg/kg, and the dose or verapamil is 2.5 to 5
mg IV, not 25 mg IV. While cardioversion is also an option Let’s say this patient comes back to the ED with atrial brilla-
in a hemodynamically stable patient, medication should be tion in the upcoming week. T is time you suspect WPW and
tried rst. De brillation is never recommended or a per us- want to choose a rate-controlling medication.
ing rhythm.
Question 2.18.3 Which is the BEST medication choice or
You treat the patient with adenosine but there is no this patient in light o her diagnosis o WPW?
response. T us, you choose to try a CCB. Un ortunately, A) Procainamide.
the patient rapidly deteriorates with the CCB, and her B) Sotalol.
heart rate actually increases, so you success ully cardiovert C) Diltiazem.
the patient. T e ECG done a er cardioversion is shown in D) Verapamil.
Figure 2-20. E) Metoprolol.

FIGURE 2-19. r h thm st p pat ent n quest n 2.18.1.


84 FAMil y MEDiCiNE EXAMiNATio N & Bo Ar D r EViEW

FIGURE 2-20. ECG pat ent n quest n 2.18.2.

FIGURE 2-21. ECG c ns stent w th W -Pa k ns n-Wh te s nd me.


CHAPTER 2 • CAr Dio l o Gy 85

FIGURE 2-22. ECG pat ent n quest n 2.18.4.

Answer 2.18.3 The correct answer choice is “A.” Patients with and aVL. Because the QRS is narrow, neither LBBB nor RBBB
WPW deteriorate with beta-blockers and CCBs (“B”–“E”). T e can be correct. Le posterior block is quite uncommon due to
drug o choice is procainamide in patients with WPW who the size o the posterior ascicle.
presents with PSV , including atrial brillation/f utter. T e
other alternative is ibutilide. T e reason is that the AV node Question 2.18.5 The ECG shown in Figure 2 23 represents
is protective since it helps block most re-entrant conductions which o the ollowing?
( or re-entrant arrhythmias that rely on the AV node). I you A) LBBB.
block the AV node with beta-blockers or CCBs, the re-entrant B) RBBB.
loop is allowed to go “wild” (just like you’ve probably seen that C) LAFB.
reality show, “Rhythms Gone Wild”). T e clues to look or to D) Le posterior ascicular block.
help identi ying patients with WPW are a young patient with E) None o the above.
previous episodes o palpitations, rapid heart rate, or syncope.
Answer 2.18.5 The correct answer is “A.” T is ECG represents
Question 2.18.4 The ECG shown in Figure 2 22 represents an LBBB. Criteria include QRS width ≥ 0.12 ms, upright (mono-
which o the ollowing? phasic) QRS in leads I and V6, and a mostly negative QRS in V1.
A) LBBB.
B) RBBB. HELPFUL TIP:
C) Le anterior ascicular block (LAFB). We d n’t suggest that u e n th s but . . . The r –r
D) Le posterior ascicular block. p me s n the right s de the ECG n an r BBB (V1, V2,
E) None o the above. V3). The r –r p me s n the left s de the ECG n an
l BBB ( ead i).
Answer 2.18.4 The correct answer is “C.” For those o us who
are visually challenged, any patient with a net negative orce in Question 2.18.6 The ECG shown in Figure 2 24 represents
lead II will have le axis deviation and likely le anterior as- which o the ollowing?
cicular block (LAFB) (provided proper lead placement). Also, A) First-degree block.
look or net negative def ection in leads III and aVF. For those B) RBBB.
who like the numbers, LAFB is present when the QRS axis is C) LAFB.
−45 to −90 degrees, there is an rS pattern (with small r waves) in D) All o the above.
leads II, III, and aVF and a qR pattern (with small q waves) in I E) None o the above.
86 FAMil y MEDiCiNE EXAMiNATio N & Bo Ar D r EViEW

FIGURE 2-23. ECG pat ent n quest n 2.18.5.

FIGURE 2-24. ECG pat ent n quest n 2.18.6.


CHAPTER 2 • CAr Dio l o Gy 87

Answer 2.18.6 The correct answer is “D.” T is ECG represents opposite ECG ndings occur with hypokalemia. ECG ndings o
a rst-degree AV block, an RBBB, and an LAFB. T e RBBB is hypokalemia include decreased amplitude o -waves, progressing
de ned by a QRS width o ≥0.12 ms (>3 small blocks) and an to -wave inversions associated with S segment depression, ol-
rsR (“rabbit ears”) in chest leads V1–V3. T is patient also has an lowed by increased PR interval and increased P-wave amplitude.
LAFB (see the ECG in Fig. 2-22 or criteria). T e U-wave is a late and inconsistent nding o hypokalemia.

Objectives: Did you learn to . . . Question 2.19.2 All o the ollowing are potential causes o
• ident PSVT b ECG and p esc be n t a t eatment? this patient’s hyperkalemia EXCEPT:
• Desc be ECG nd ngs WPW? A) Metabolic acidosis.
• D f e ent ate between d f e ent t pes hea t b ck based n B) ACE inhibitors.
ECG nd ngs? C) ARBs.
D) Renal ailure.
CASE 2.19 E) Furosemide.

A 75-year-old patient with chronic kidney disease and diabe- Answer 2.19.2 The correct answer is “E.” Furosemide will
tes presents to your ED or chest tightness, atigue, and palpi- cause hypokalemia rather than hyperkalemia. All the other
tations, and has the ECG shown in Figure 2-25. answer choices are potential causes o hyperkalemia. Other
causes o hyperkalemia include a potassium load rom muscle
Question 2.19.1 What is the most likely electrolyte abnor breakdown (e.g., rhabdomyolysis, burns, trans usion o old
mality in this patient? blood), tumor lysis syndrome, and other exogenous sources o
A) Hypokalemia. potassium such as penicillin, potassium supplements, “lite” salt,
B) Hyperkalemia. and water so eners. Consider also Addison disease and hypoal-
C) Hyponatremia. dosteronism. Digoxin toxicity is also a possibility.
D) Hypermagnesemia.
E) Hypercalcemia. Question 2.19.3 What is the rhythm shown in Figure 2 26?
A) Atrial brillation.
Answer 2.19.1 The correct answer is “B,” hyperkalemia. Note B) Normal sinus rhythm with multiple PACs.
the peaked -waves across the precordium. Note also that the C) T ird-degree heart block with rapid rate.
patient has early repolarization. “A” is incorrect, and many o the D) Multi ocal atrial tachycardia (MA ).

FIGURE 2-25. ECG pat ent n quest n 2.19.1.


88 FAMil y MEDiCiNE EXAMiNATio N & Bo Ar D r EViEW

FIGURE 2-26. ECG pat ent n quest n 2.19.3.

Answer 2.19.3 The correct answer is “D.” T is is a MA . Note Resting transcutaneous oximetry is 92% at rest. BMI is
the multiple morphologies o the P waves indicated by arrows 24 kg/m2. She has JVD but clear lungs. A grade 2/6 mid-
in Figure 2-27 as well as the irregularity o the rhythm. Here are systolic murmur is heard over the le upper sternal border.
help ul tips in diagnosing and treating MA : three or more di - Electrocardiogram is shown (Fig. 2-28).
erent P-wave morphologies with varying PR intervals. Causes
include theophylline, pulmonary disease, and abnormal electro- Question 2.20.1 What is the most likely diagnosis?
lytes (K+ or Mg). Digoxin may worsen MA ! Rarely, AV nodal A) CAD.
ablation with permanent pacing can be considered in re ractory B) Pulmonary hypertension.
cases. C) Asthma.
D) Congenital aortic stenosis.
Question 2.19.4 All o the ollowing are treatments o MAT E) Mitral valve prolapse.
EXCEPT:
A) Nondihydropyridine CCB (verapamil, diltiazem). Answer 2.20.1 The correct answer is “B.” T e physical exami-
B) Beta-blocker. nation is consistent with right ventricular pressure overload.
C) Magnesium. T is is supported by the electrocardiogram demonstrating
D) Improving pulmonary unction and reducing hypoxia. right atrial enlargement, right axis deviation, and RVH. CAD
E) Adenosine. is almost unheard o in a woman younger than 30 years with-
out any risk actors. Asthma may cause her symptom com-
Answer 2.19.4 The correct answer is “E.” All o the others are plex but is not supported by her examination. Aortic stenosis
indicated in the treatment o MA . Adenosine may slow down causes neither resting hypoxemia nor RVH. Because they have
the rhythm temporarily but is not considered a treatment o this a xed cardiac output (limited by lung vascular pressures),
rhythm. patients with pulmonary hypertension o en get presyncopal
Objectives: Did you learn to . . . with exertion.
• r ec gn ze ECG nd ngs ass c ated w th h pe ka em a and Note: Findings that suggest RVH on the ECG: right axis devi-
h p ka em a? ation, right atrial abnormality (P-wave >2.5 boxes tall in lead
• Desc be ECG nd ngs and t eatment pt ns MAT? II), RVH (tall R in V1), and strain pattern in leads II and III.
O en patients with pulmonary hypertension will have an intra-
ventricular conduction delay with R–R′ in V1 (not shown on
this electrocardiogram, Fig. 2-29).
CASE 2.20
A 28-year-old woman with no signi cant past medical his- Question 2.20.2 The ollowing tests may be help ul in eluci
tory presents to clinic with complaints o progressive short- dating the cause o pulmonary hypertension EXCEPT:
ness o breath; she becomes dyspneic with less activity than A) Chest x-ray and pulmonary unction studies.
1 year ago. I she exerts hersel beyond a brisk walk, she B) C scan.
becomes lightheaded, presyncopal, and eels tightness in her C) ANA, HIV antibody/antigen testing and liver unction studies.
chest. She also notes generalized atigue. Your examination D) Nasopharyngoscopy.
discloses a heart rate o 105 bpm and normal blood pressure. E) Polysomnogram.

FIGURE 2-27. A ws sh w P waves va ng m ph g , c ns stent w th MAT.


CHAPTER 2 • CAr Dio l o Gy 89

FIGURE 2-28. ECG pat ent n quest n 2.20.1.

Answer 2.20.2 The correct answer is “D.” An important part history and physical examination. Chest radiography and PF s
o the work-up or pulmonary hypertension is de ning the eti- can identi y chronic lung disease causing (or contributing to)
ology and potentially reversible causes. T ere is no cookbook pulmonary hypertension. A C scan is done to exclude chronic
approach, and diagnostic work-up should be tailored by the thromboembolic pulmonary disease and to evaluate or brosis,

FIGURE 2-29. ECG c ns stent w th r VH. N te a ge r wave n V1 and ght ax s dev at n.


90 FAMil y MEDiCiNE EXAMiNATio N & Bo Ar D r EViEW

sarcoidosis, etc. Connective tissue disease, HIV, and cirrhosis examination, you notice that he had nodules on his ngertips
are known to cause pulmonary hypertension. Sleep apnea is an and notice a slight reddish discoloration under his ngernails.
important, treatable cause o PH N. Nasopharyngoscopy has On urther questioning, he tells you that he recently moved
no role in this work-up. here rom a major city and had used IV drugs in the very dis-
tant past, by which he means as recently as last month.
An echocardiogram con rms ndings suggestive o severe
pulmonary hypertension and changes consistent with right Question 2.21.1 Your next step in management is:
ventricular pressure overload (a right heart catheterization is A) Order a chest x-ray and start empiric levof oxacin.
required or diagnosis). No intracardiac shunt is identi ed by B) Order a chest x-ray and start empiric ce riaxone and
echo. T e remainder o her diagnostic work-up ails to identi y azithromycin.
a secondary cause o pulmonary hypertension. A right heart C) Order a chest x-ray, echocardiogram, draw blood cultures.
catheterization con rms severe pulmonary hypertension but D) Order a chest x-ray and start empiric piperacillin and vanco-
also ails to identi y a shunt. A vasodilator challenge (with ade- mycin.
nosine) is per ormed and no change in pulmonary pressure is E) Order a chest x-ray and do nothing, it is probably viral.
elicited. She is given a diagnosis o idiopathic pulmonary arte-
rial hypertension. Answer 2.21.1 The correct answer is “C.” Although the
T e treatment o pulmonary hypertension is, in general, chest x-ray is overkill or a case o endocarditis, it may iden-
very specialized and a cardiologist should be involved. Excep- ti y other causes o ever and help evaluate or heart ailure.
tions include pulmonary hypertension rom chronic hypoxia he patient is presenting with IV drug use history, Osler
(smoking, sleep apnea) that are amenable to primary care nodes, and Janeway lesions (nail-bed hemorrhages), as well
management. Chronic PE can also be managed by the primary as ever. hese easily meet three minor Duke criteria (see
care practitioner, although the patient should still ollow with able 2-9), quali ying or a “possible diagnosis o endocardi-
a cardiologist due to the potential or right heart ailure. tis.” Now what you need is one major criterion to de initively
diagnose endocarditis. A positive echocardiogram or blood
culture would quali y. Once the blood cultures are drawn, i
HELPFUL TIP:
your suspicion is high, you may start empiric antibiotics or
Add t na the ap pu m na h pe tens n ma
endocarditis.
nc ude p stac c n and ph sph d este ase nh b t s
(e.g., s dena , tada a , va dena ). S me pat ents e-
Question 2.21.2 Which valve(s) are most commonly af ected
qu e ant c agu at n. Management sh u d be d ected
by endocarditis?
b a pu m na h pe tens n spec a st.
A) Aortic valve.
B) Mitral valve.
C) Pulmonic valve.
D) Demonic valve.
E) A and B.
CASE 2.21
A 54-year-old male presents to your clinic complaining o dys- Answer 2.21.2 The correct answer is “E.” T e aortic and mitral
pnea and evers. His temperature in the o ce is 38.5°C and valves are most commonly a ected. T ere is no demonic valve,
heart rate is 113 bpm with pulse oximetry o 93%. On physical we hope, but there is the tricuspid, o course.

TABLE 2-9 REVISED DUKE CRITERIA FOR ENDOCARDITIS DEFINITIVE DIAGNOSIS: TWO MAJOR
CRITERIA OR ONE MAJOR + THREE MINOR CRITERIA. POSSIBLE DIAGNOSIS: ONE MAJOR + ONE
MINOR CRITERION OR THREE MINOR CRITERIA
Major Criteria Minor Criteria
• Tw p s t ve b d cu tu es gan sms t p ca • P ed sp s ng hea t d s de
end ca d t s • iV d ug abuse
• Th ee p s t ve b d cu tu es gan sms c ns stent w th • Feve ≥38°C
end ca d t s • Vascu a phen mena: a te a emb sm, sept c pu m na emb sm,
• Se g c ev dence Coxiella burnetii m c t c aneu sm, nt ac an a hem hage, c njunct va petech ae,
• Ech ca d g aph c ev dence end ca d a nv vement: Janewa es ns
o sc at ng nt aca d ac mass n a hea t va ve, n supp t ng • immun g c phen mena: g me u neph t s, o s e n des, r th sp ts,
st uctu es, n the path egu g tant jets, n mp anted heumat d act
mate a w th ut an the anat m c exp anat n; ca d ac • Mc b g c ev dence n ect n c ns stent w th but n t meet ng maj
abscess; new deh scence p sthet c va ve; new va vu a c te a
egu g tat n • Se g c ev dence n ect n w th gan sms c ns stent w th end ca d t s
CHAPTER 2 • CAr Dio l o Gy 91

Question 2.21.3 Which o the ollowing organisms is most


QUICK QUIZ: WHAT Ar E yo U . . . A HEAr T
common in acute endocarditis?
A) Staphylococcus aureus and group B streptococci. FAil Ur E SPECiAl iST?
B) Alpha-hemolytic streptococci or enterococci.
C) Enterovirus. Which o the ollowing has been shown to improve mortality in
D) Fungi. patients with systolic heart ailure (HFrEF)?
E) Elmo muppetl (can you tell we are getting tired . . . the jokes A) Valsartan + Sacubitril (Entresto).
are getting weaker). B) Ivabradine (Corlanor).
C) Azumumumab (Coregulator).
Answer 2.21.3 The correct answer is “A.” Staph and group D) A and B.
B strep are generally responsible or acute endocarditis while E) B and C.
alpha-hemolytic streptococci and enterococci are more com-
mon with subacute endocarditis. Fungi may be present in IV The correct answer is “A.” First you need to know what these
drug abusers. As to E. muppetl, it is a new species recently things are. Entresto combines an ARB plus a neprilysin inhibi-
described by us. We groveled it in Big Bird cultures. tor. Inhibiting neprilysin increases vasodilatation and sodium
loss. It has been shown to be e ective at reducing mortality rom
Objectives: Did you learn to . . . HFrEF. T is can replace an ACE inhibitor or ARB though how
• in t ate eva uat n and management bacte a it per orms against an ACE inhibitor is not known. It cannot be
end ca d t s? given with an ACE inhibitor because o the risk o angioedema.
O course you already know that an ACE inhibitor and an
ARB should not be used together . . . right?
QUICK QUIZ: yET ANo THEr HEAr T FAil Ur E Corlanor reduces heart rate without lowering BP and is only
QUESTio N (y u th ught we we e d ne) indicated in heart ailure patients who do not reach their target
heart rate with a beta-blocker. Corlanor would be added to an
Cardiac resynchronization or heart ailure is indicated in which ACE inhibitor or ARB, a beta-blocker, and aldosterone antago-
o the ollowing patients? nist. It does not replace the beta-blocker and does not improve
A) A patient who has an ejection raction o 30% and is in atrial mortality. It supplements the beta-blocker in patients with heart
brillation that is rate-controlled. rate >70 bpm a er maximizing other medications. Corlanor can
B) A patient who has an ejection raction o 30% and is in atrial also be used in patients who do not tolerate a beta-blocker. We
brillation that is not rate-controlled (say a rate o 110–115 made up “C,” though it wouldn’t surprise us to see this drug
bpm). name at some point in the uture.
C) A patient who has an ejection raction o 30% and in normal
sinus rhythm who has an LBBB.
D) A patient with an underlying arrhythmia such as PSV who
CASE 2.22
is symptomatic. A new patient presents or his annual examination and has
E) A or B. some questions about cardiovascular disease risk. He also
wants to chat about his avorite sports team (Iowa Hawk-
The correct answer is “C.” Bet we got you on this one, huh? eyes, we hope), but you gently steer him back to the matter
T e idea behind cardiac resynchronization is to put blood at hand.
into the ventricle at the optimum time or unction. T is re-
quires the atrium to be working properly. T us, “A” and “B” are Question 2.22.1 The ollowing are all considered cardiac
wrong. T e concept is to pace the atrium and then also pace risk actors when calculating target cholesterol EXCEPT:
the ventricle so that the ventricle depolarizes at the optimum A) Male > 45 years old.
moment or blood ejection a er the blood is sent to the LV B) First-degree emale relative with CAD >65 years old.
by the atrium. T is optimal blood ejection is disordered by a C) Smoking.
LBBB where depolarization o the ventricle is delayed caus- D) Hypertension.
ing a “dyssynchrony” (as it were) between the timing o atrial E) HDL cholesterol o <40 mg/dL.
unctioning and the timing o the ventricular depolarization.
Cardiac resynchronization is “only” or patients with heart ail- Answer 2.22.1 The correct answer is “B.” It should be emale
ure who are in normal sinus rhythm. So, the basic criteria or relative with CAD at <65 years o age, not >65 years o age. See
cardiac resynchronization include a QRS o >150 ms, with a able 2-10 or a listing o cardiac risk actors. Just to be com-
LBBB, a LV ejection raction o 30% or less, and sinus rhythm plete, i the patient has an HDL > 60 mg/dL, this counts as a
who have class III or class IV heart ailure (J Am Coll Cardiol. protective actor and “cancels out” one o the risk actors. Recall
2013;61(3):e6–75). It seems to improve the overall absolute that we are talking about primary prevention only here. None
survival by about 2%; it also reduces readmissions (Ann Intern o this applies to the patient who has already had a myocardial
Med. 2014;160:603). in arction.
92 FAMil y MEDiCiNE EXAMiNATio N & Bo Ar D r EViEW

TABLE 2-10 RISK FACTORS FOR CORONARY TABLE 2-11 2013 AMERICAN HEART ASSOCIATION
ARTERY DISEASE (AHA)/AMERICAN COLLEGE OF CARDIOLOGY (ACC)
• F st-deg ee ma e e at ve w th CAD at age <55 st-deg ee LIPID GUIDELINES
ema e e at ve w th CAD at age <65 Qua a h gh- 1. l Dl >190 mg/dl or
• Sm k ng d se stat n. 2. HX CAD ( equ va ent) or
• HDl <40 mg/dl 3. DM age 40–79
• D abetes
• H pe tens n Qua a P ma p event n 10- ea sk s >7.5%
• Age: ma es >45, ema es >55 m de ate-d se stat n and the pat ent has an l Dl 70–189 mg/dl
• E evated l Dl (use sk ca cu at )a and age 40–75.

Qua a w- Th se wh qua a stat n but d n’t


d se stat n. t e ate h gh- m de ate-d se stat n.
T is 55-year-old gentleman says, “All my riends take a medi- Goals of Therapy
cine or cholesterol. Shouldn’t I take one too?” You are wor- 1. The primary goal is to reach the appropriate dose of statin.
ried that he is susceptible to peer pressure, but you are also 2. r educt n l Dl b 50% m base ne.
thinking about protecting him rom atherosclerotic cardio- 3. The e a e n xed l Dl g a s (such as past g a s l Dl <100 mg/dl ).
vascular disease (ASCVD). Drug Doses
1. H gh-d se stat n = at vastat n 40–80 mg suvastat n 20–40 mg.
Question 2.22.2 According to the 2013 ACC/AHA Blood 2. M de ate-d se stat n = at vastat n 10–20 mg QD, p avastat n
Cholesterol Guideline, all o the ollowing automatically 40–80 mg BiD, and suvastat n 5–10 mg QD, am ng the s.
quali y as patients who would bene t rom statin therapy a
F nd a ca d vascu a sk ca cu at at http://t s.ca d s u ce. g/
EXCEPT: ASCVD-r sk-Est mat /
A) A patient with known cardiovascular disease.
B) A 40-year-old patient with an LDL o 200 mg/dL.
C) A 55-year-old diabetic patient with an LDL o 130 mg/dL. disease (e.g., amily history, obesity) are not taken into account.
D) A 55-year-old nondiabetic patient without known cardio- T e risk calculator is a tool; you must continue to apply clinical
vascular disease whose 10-year ASCVD risk is calculated as judgment. Also, the calculator, as well as the entire guideline, is
10%. not meant to be used to manage all orms o dyslipidemias.
E) A 35-year-old smoker with hypertension.
T e patient then asks you about something he read about
Answer 2.22.2 The correct answer is “E.” T e 35-year-old “crap.” A light bulb goes of and you realize he wants to know
patient might be a candidate or a statin (HMG-CoA reductase about C-reactive protein (CRP).
inhibitor) but is not an automatic candidate based on the in or-
mation given. All other options describe patients who would Question 2.22.4 Which o the ollowing best represents the
bene t rom statin therapy according to the 2013 ACC/AHA role o CRP in cardiac disease in 2015?
guideline. T e 2013 ACC/AHA guideline relies on a risk cal- A) CRP should be measured in all patients in whom cardiac
culator or primary prevention (located online at: http://tools. disease is suspected.
cardiosource.org/ASCVD-Risk-Estimator/). See able 2-11 or B) CRP should be measured only in patients with intermediate
a summary o the 2013 cholesterol guidelines. cardiac risk actors (e.g., those with a 10-year risk o CAD o
10–20%).
In order to use the ASCVD risk calculator or this patient, C) CRP should be measured in patients with known heart dis-
you need to input some in ormation. ease in order to monitor inf ammation and risk.
D) CRP should be measured in low-risk (<10% risk o CAD
Question 2.22.3 Which o the ollowing actors is included in next 10 years) patients who have no known cardiac dis-
in the ACC/AHA ASCVD risk calculator? ease. An elevated CRP suggests that these patients should be
A) LDL cholesterol level. treated with a lipid-lowering therapy.
B) Serum triglyceride level. E) CRP has not been shown to be use ul and does not contrib-
C) Hb A1c. ute signi cantly to cardiac risk strati cation.
D) Systolic blood pressure.
E) Diastolic blood pressure. Answer 2.22.4 The correct answer is “E.” Although, high-
sensitivity CRP (hsCRP) was initially thought to be a possible
Answer 2.22.3 The correct answer is “D.” T e risk actors biomarker or cardiac risk assessment, it has been shown to be
included in the ASCVD risk calculator include age, sex, race, sys- o marginal bene t. T e use o hsCRP led to minimal reclassi -
tolic blood pressure, total cholesterol, HDL cholesterol, smok- cation o patients (a maximum o 11% o intermediate patients
ing status, presence or absence o diabetes, and treatment or were reclassi ed in one study). T e Class IIa recommendation
hypertension. Unlike many Caribbean resort destinations, the to use hsCRP was published by the AHA in 2003, prior to ur-
calculator is not all-inclusive. Many known risk actors or heart ther studies that have questioned its use ulness.
CHAPTER 2 • CAr Dio l o Gy 93

2013 ACC/AHA guideline has changed the way we determine


HELPFUL TIP:
risk, expanding the use o statins. Patients whose 10-year
We d n t use hsCr P. H weve , t we e use u , t w u d
ASCVD risk is 7.5% or greater should be on moderate-to-high-
be n the nte med ate- sk pat ents.
intensity statin therapy.

Question 2.22.7 Your astute patient appears to be goal


Per the ACC/AHA guideline, you recommend assessing his oriented and asks you what his target LDL should be. You
lipid pro le. reply:
A) “ argets are so 2012. T e goal is to get you on a moderate-
Question 2.22.5 Well, what does the USPSTF recommend to-high-intensity statin. T ere is no speci c LDL goal.”
or screening or lipid disorders? B) “Your target LDL is <100 mg/dL.”
A) Screen all adults annually starting at age 25. C) “Your target LDL is about 70 mg/dL.”
B) Screen men age 35 years and older. D) “Your target LDL is simply as low as we can get without
C) Screen women age 35 years and older. causing rhabdomyolysis . . . . Well, maybe just a little rhabdo
D) Screen all patients age 25 years and older whose BMI is will be OK.”
>25 kg/m 2.
Answer 2.22.7 The correct answer is “A.” T e 2013 ACC/AHA
Answer 2.22.5 The correct answer is “B.” As o the writing o guideline does not recommend treating to speci c LDL targets.
this book, the USPS F is in the process o updating the screen- Instead, appropriate statin therapy should be selected based
ing recommendation or lipid disorders. Published in 2008, the on risk category as previously indicated. Your goal is to get the
USPS F gave a “grade A” recommendation or screening or patient to a maximal dose o statin and not a speci c LDL. T at
lipid disorders in two groups: (1) men 35 years o age and older said, the LDL should be reduced by 50% even i that is a starting
and (2) women 45 years o age and older at increased risk or point o 400 mg/dL and the patient ends up at 200 mg/dL.
ASCVD.
T e USPS F gave a “grade B” recommendation or (1) men
age 20 to 35 years at increased risk or ASCVD and (2) women HELPFUL TIP:
age 20 to 45 years at increased risk or ASCVD. BMI > 25 kg/m 2. The e s n ev dence that “t t at ng up” the d se a
For the purposes o the recommendation, the USPS F con- stat n s necessa he p u n p event ng s de e ects.
siders the ollowing to be signi cant risk actors: personal his- Just sta t the na app p ate d se a stat n n da 1
tory o ASCVD, diabetes, amily history o CAD be ore age 50 t eatment.
in male relative and be ore age 60 in emale relatives, tobacco
use, hypertension and obesity (BMI > 30 kg/m 2). “A” is wrong
because the USPS F does not speci y the optimal screening You advise your patient to start atorvastatin 20 mg daily.
interval. You check baseline transaminases, which are normal. When
your patient returns 3 months later and sees your partner
T e next day, you obtain asting labs. T e patient has normal (because you took a “vacation” to study or the board exami-
electrolytes and the ollowing cholesterol panel: total choles- nation), she checks his lipids and transaminases out o habit.
terol 175 mg/dL, LDL 110 mg/dL, HDL 35 mg/dL, triglycer- I only you had been there! You know that the FDA no longer
ides 150 mg/dL. He is a 55-year-old A rican-American male, recommends periodic liver enzyme testing while on a statin.
nonsmoker, and nondiabetic who takes no medications. His Statin-related hepatotoxicity is an idiosyncratic reaction
systolic blood pressure is 132 mm Hg. You plug all these that is extremely rare and completely unpredictable, so there
data into the ASCVD risk calculator and generate a 10-year is no point in routinely checking transaminases. Well, your
ASCVD risk o 7.5%. partner didn’t get the memo, and you return to nd your
patient’s AL and AS have both doubled while on atorv-
Question 2.22.6 Given the ASCVD risk you calculated, and astatin and are now almost twice the upper limit o normal
being consistent with the 2013 ACC/AHA guideline, you or your lab.
recommend:
A) No changes. Keep calm and carry on. Question 2.22.8 When you nd transaminases are twice
B) A treadmill stress test. the upper limit o normal while on a statin, the proper
C) A high ber diet. response is to:
D) Low-intensity statin therapy (e.g., simvastatin 10 mg daily). A) Stop the statin because o the elevated liver enzymes.
E) Moderate-to-high-intensity statin therapy (e.g., atorvastatin B) Start a di erent statin since this is not a “class e ect.”
20–40 mg daily). C) Continue the statin and consider other causes or the ele-
vated liver enzymes.
Answer 2.22.6 The correct answer is “E.” Did you think this D) Add cholestyramine to help ease the burden on the liver.
patient was a candidate or moderate-to-high-intensity statin E) Re er or liver biopsy to rule out other causes o elevated
therapy prior to this guideline? I not, you are not alone. T e liver enzymes.
94 FAMil y MEDiCiNE EXAMiNATio N & Bo Ar D r EViEW

Answer 2.22.8 The correct answer is “C.” Statins can be contin- B) His atigue and aches are a mani estation o depression and
ued as long as the elevation o liver enzymes is less than three sleep disturbance.
times the upper limit o normal. Never assume that this is a C) You can sa ely increase the dose o his statin.
drug e ect i there is a reason to believe that the patient could D) He still may have statin-induced myopathy despite a normal
have another disease, such as hepatitis C. CPK.
You are compelled to per orm the requisite history and E) Your lab has a serious quality control issue.
physical examination to assess or other causes o liver dis-
ease. In the course o your investigations, other labs and Answer 2.22.10 The correct answer is “D.” He still may
imaging may be in order. However, liver biopsy (“E”) would have statin-induced myopathy despite a normal CPK, so “A”
be taking your duciary duty to the extreme. “A” is incorrect is incorrect. T ere are several crossover trials that demon-
since the levels are only two times the upper limit o nor- strate myopathy in patients with normal muscle enzymes. T e
mal. “B” is incorrect or two reasons. First, there is no need mechanism is thought to be a mitochondrial dys unction. “B”
to act to change the drug at this point. Second, elevated liver is incorrect, and the reverse may actually be true (myopathy
enzymes are a class ef ect. “D” is incorrect because you do not causing poor sleep and anhedonia). “C” is incorrect because
need to add another drug at this time, and cholestyramine he may indeed have a myopathy, and a trial o one or both
will do nothing to “ease the burden on the liver,” whatever drugs is warranted. Remember that gem brozil (and less so
that means. eno brate), which you might prescribe or hypertriglyceride-
mia, interact with statins to increase the risk o statin-induced
A er a thorough history, examination, appropriate lab and myopathy/rhabdomyolysis. Remember also that there is no evi-
imaging tests, you determine that the liver enzyme elevation dence treating with gem brozil or eno brate improves cardio-
was due to atty in ltration. A month later the enzymes have vascular outcomes.
come down to the upper limits o normal. When you see your
patient back, he complains o pain and tenderness in his legs, You stop his atorvastatin. A ew weeks later, he returns and
shoulders and back. He also reports generalized atigue and eels great! You consider starting a dif erent agent to treat his
weakness. lipids.

Question 2.22.9 What is the most appropriate next step in Question 2.22.11 Which o the ollowing is classi ed as a
the evaluation and management o this complaint? bile acid sequestrant?
A) Stop his atorvastatin and replace it with rosuvastatin. A) Ezetimibe.
B) Double the dose o atorvastatin. B) Colestipol.
C) Check liver enzymes. C) Colesevelam.
D) Check CPK. D) B and C.
E) Start coenzyme-Q10 (ubiquinone). E) All o the above.

Answer 2.22.9 The correct answer is “D.” T is patient is pre- Answer 2.22.11 The correct answer is “D.” Ezetimibe (Zetia)
senting with symptoms that may be attributable to statin-induced is not a bile acid sequestrant but rather reduces cholesterol
myopathy. Statins have been associated with myalgia and rhab- absorption by blocking at the brush border o the small intes-
domyolysis. Prior to making any decisions about therapy, the tine. T is is a mechanism that is di erent rom any o the other
next step would be to con rm your suspicions by checking the lipid-lowering agents. It is relatively sa e but expensive and less
CPK level. I this patient has statin-induced muscle symptoms, potent than the statins.
he is unlikely to improve by switching to rosuvastatin (“A”) and
increasing the stating dose (“B”) is the wrong thing to do. While
it may be reasonable to check liver enzymes (“C”) due to his HELPFUL RANT:
complaint o atigue, myopathy is more likely given his constel- D n’t ee b gated t “just d s meth ng.” There is no
lation o symptoms. O course, in your real medical practice, you data suggesting any anti-lipid drugs other than the
could order liver enzymes and CPK, but an examination is not statins reduce cardiovascular outcomes. Some, such
real li e! Finally, “E” is incorrect. Coenzyme Q10 has not been as niacin, may actually worsen outcomes.
shown to prevent or treat statin related myopathy (Mayo Clin
Proc. 2015;90:24).
You nd yoursel staring out the examination room window,
You check a CPK level, and it is . . . normal. Darn. You thought ruminating on cholesterol medications.
you had this mystery wrapped up.
Question 2.22.12 Side ef ects o ezetimibe include which
Question 2.22.10 From this normal CPK, you can conclude o the ollowing?
that: A) Diarrhea.
A) He does not have statin-induced myopathy since the CPK is B) Arthralgia.
within normal limits. C) Angioedema.
CHAPTER 2 • CAr Dio l o Gy 95

D) Liver enzyme elevation.


E) All o the above. Clinical Pearls
A a se-p s t ve t p n n can be caused b a PE, hea t a u e,
Answer 2.22.12 The correct answer is “E.” “C” deserves spe- bu ns, seps s, st ke, and m e.
cial mention. As with ACE inhibitors, angioedema has been
A n ma ECG du ng chest pa n d es n t u e ut ACS. in
reported with the use o ezetimibe during postmarketing
act, 10% pat ents w th an Mi w have an n t a n ma
research. T e rate o occurrence is not known. However, it can ECG w th pa n.
be li e-threatening although no deaths have been reported to
Agg ess ve b d p essu e we ng n pat ents w th d abetes
date. All o the other side e ects are known to occur at a rate
ma be ha m u . Use the same g a as a m st eve ne
greater than with placebo.
e se, <140/90. The JNC 8 ep t suggests a g a <150/90 n
th se 60 ea s age and de .
Av d the c mb nat n a stat n and gem b z en -
HELPFUL TIP: b ate; t can cause habd m s s. The sk s we w th
Ev dence ezet m be s weak, nc ud ng a much- en b ate, h weve .
t uted 2015 stud (N Eng J Med 2015;372:2387). F st,
D n t c nt nue n nasp n ant p ate et agents (e.g., c p d -
the stud s awed and ks at a ve na w g up g e ) nde n te a te stent p acement. Dete m ne a ta get
pat ents (th se h sp ta zed w th kn wn CAD, the e e end date (e.g., 1 ea dua asp n-c p d g e a te p ace-
sec nda p event n). it sa s n th ng ab ut p ma ment a DES).
p event n. Sec nd, man the pat ents we e n t
D n td ut ne st ess test ng as mpt mat c pat ents
even n a stat n, the standa d ca e. F na , u have p e pe at ve . in act, d n’t pe m st ess-test ng n an
t t eat 50 pat ents 7 ea s t p event ne ca d - as mpt mat c pat ents.
vascu a event—n t a g w ng end sement, espec a
D n t use beta-b cke s a ne as ant h pe tens ve the ap
s nce the we en’t n a stat n.
un ess the e s an the nd cat n (e.g., p st-Mi, hea t a u e).
D n t use hsCr P t assess ca d ac sk.
D n t use n nstat n d ugs t we ca d ac sk n pat ents.
HELPFUL TIP:
Metamuc the ps um p ducts a e use u n e- i us ng a beta-b cke p e pe at ve t educe ca d ac sk,
sta t t we n advance the su ge and get the pat ent n
duc ng se um ch este and p v de a “n nd ug” a -
a stab e d se be e su ge .
te nat ve. H weve , at east 7 g s ub e be da s
equ ed. Pe m an ech ca d g am when c ns de ng the d agn s s
hea t a u e. it w d e ent ate s st c hea t a u e m
d ast c hea t a u e.
Pe m ECG and adm n ste asp n 325 mg mmed ate
HELPFUL TIP: t a pat ents p esent ng w th s mpt ms m ca d a
Can’t u gu s eave ch este beh nd a ead ? N . The n a ct n.
new k ds n the b ck a e the PCSK9 (p p te n c n-
P esc be ACE nh b t s ve Ar Bs n pat ents w th h pe ten-
ve tase subt s n kex n t pe 9 nh b t s), wh ch p event
s n, ca d vascu a d sease and/ ch n c k dne d sease.
the b nd ng and the deg ad ng l Dl ecept s. S ,
the e a e m e ecept s ava ab e t em ve l Dl m P esc be stat ns a pat ents w th a h st CAD. The
2013 ACC/AHA gu de ne e m nated an l Dl g a . The g a
the b d. The st tw a e Praluent (a cumab)
s t use h gh-d se stat ns n th se wh a e h gh sk (e.g.,
and Repatha (ev cumab). The a e wa expens ve
h st m ca d a n a ct n).
($10,000/ ea ), a e njectab e, and have No utc me
data (l Dl s a su gate ma ke , c u se . . . we ca e St p a NSAiDs (except asp n) when adm tt ng a pat ent
p ss b e ACS. NSAiDs nc ease ca d ac sk.
h w the pat ent s d ng and n t the eve l Dl . Th s
has been made abundant c ea ). The a e a s n Use the CHA2-DS2-VASC sc e when dete m n ng the need
used n pat ents wh have ess than a 50% l Dl educ- ant c agu at n n at a b at n. W men w th a sc e 2
t n w th stat ns and the d ugs. and men w th a sc e 1 a e cand dates ant c agu at n.

BIBLIOGRAPHY
Objectives: Did you learn to . . .
Adan V, Crown LA. Diagnosis and treatment o sick sinus syn-
• Dete m ne sk act s ASCVD? drome. Am Fam Physician. 2003;67(8):1725–1732.
• l st the sc een ng ec mmendat ns p d d s de s? Amsterdam EA, et al. 2014 AHA/ACC Guideline or the man-
• Desc be the e p d- we ng the ap n the p event n agement o patients with non–S -elevation acute coronary
ca d ac d sease? syndromes: A report o the American College o Cardiol-
• ident s me the p tent a s de ef ects p d- we ng ogy/American Heart Association ask Force on Practice
med cat ns? Guidelines. J Am Coll Cardiol. 2014;64(24):e139–e228.
96 FAMil y MEDiCiNE EXAMiNATio N & Bo Ar D r EViEW

Body R, et al. Do risk actors or chronic coronary heart the Panel Members Appointed to the Eighth Joint National
disease help diagnose acute myocardial in arction in the Committee (JNC 8). JAMA. 2014;311(5):507–520.
emergency department. Ann Emerg Med. 2007;49:145. January C , et al. 2014 AHA/ACC/HRS Guideline or the
Catella-Lawson F, et al. Cyclooxygenase inhibitors and the Management o Patients With Atrial Fibrillation: Execu-
antiplatelet e ects o aspirin. N Engl J Med. 2001;345(25): tive Summary: A Report o the American College o
1809–1817. Cardiology/American Heart Association ask Force on
Dahlo B, et al. Reversal o le ventricular hypertrophy in Practice Guidelines and the Heart Rhythm Society. J Am
hypertensive patients. A meta-analysis o 109 treatment Coll Cardiol. 2014;64(21):2246–2280.
studies. Am J Hypertens. 1992;5(2):95–110. Laussen PC. Neonates with congenital heart disease. Curr Opin
Eagle KA, et al. Perioperative cardiac evaluation or noncardiac Pediatr. 2001;13(3):220–226.
surgery update. 2002. Available at: www.acc.org/clinical/ Moodie DS. Diagnosis and management o congenital heart
guidelines/perio/update/periupdate index.htm. disease in the adult. Cardiol Rev. 2001;9(5):276–281.
Elkayam U. Pregnancy and cardiovascular disease. Braunwald’s Nishimura RA, et al. 2014 AHA/ACC Guideline or the
Heart Disease: A extbook o Cardiovascular Medicine. Management o Patients With Valvular Heart Disease: A
Philadelphia, PA: Saunders, 2005. Report o the American College o Cardiology/American
Fihn SD, et al. 2012 ACCF/AHA/ACP/AA S/PCNA/SCAI/ Heart Association ask Force on Practice Guidelines.
S S Guideline or the Diagnosis and Management o J Am Coll Cardiol. 2014;63(22):e57–e185.
Patients with Stable Ischemic Heart Disease: a report o O’Gara P et al. 2013 ACCF/AHA Guideline or the Manage-
the American College o Cardiology Foundation/Ameri- ment o S -Elevation Myocardial In arction: A Report o
can Heart Association ask Force on Practice Guide- the American College o Cardiology Foundation/Ameri-
lines, and the American College o Physicians, American can Heart Association ask Force on Practice Guidelines.
Association or T oracic Surgery, Preventive Cardio- J Am Coll Cardiol. 2013;61(4):e78–e140.
vascular Nurses Association, Society. J Am Coll Cardiol. Patel AA, Budo MJ. Screening or heart disease: C-reactive
2012;60(24):e44–e164. protein versus coronary artery calcium. Expert Rev
Garty M, et al. Blood trans usion or acute decompensated Cardiovasc T er. 2010;8(1):125–131.
heart ailure– riend or oe? Am Heart J. 2009;158: 653–658. Rector S, et al. Evaluation by patients with heart ailure o the
Gibbons RJ, et al. Management o patients with chronic stable e ects o enalapril compared with hydralazine plus isosorbide
angina update. Available at: http://www.acc.org/clinical/ dinitrate on quality o li e. V-HeF II. T e V-HeF VA Co-
guidelines/stable/update_index.htm. operative Studies Group. Circulation. 1993:87(6, Suppl):
Goyle KK, Walling AD. Diagnosing pericarditis. Am Fam VI71–VI77.
Physician. 2002;66(9):1695–1702. Stone NJ, et al. 2013 ACC/AHA guideline on the treatment o
Gregoratos G, et al. ACC/AHA/NASPE 2002 Guideline update blood cholesterol to reduce atherosclerotic cardiovascular
or implantation o cardiac pacemakers and antiarrhythmia risk in adults. J Am Coll Cardiol. 2014;63(25 Pt B):
devices. Available at: www.acc.org/clinical/guidelines/ 2889–2934.
pacemaker/incorporated/index.htm. aylor AL, et al. Combination o isosorbide dinitrate and
Harris GD. Heart disease in children. Prim Care. 2000;27(3): hydralazine in blacks with heart ailure. N Engl J Med.
767–784. 2004;351(20):2049–2057.
Hebbar AK, Hueston WJ. Management o common arrhyth- Wilson PW, et al. C-reactive protein and risk o cardiovascular
mias: Part I. Supraventricular arrhythmias. Am Fam disease in men and women rom the Framingham Heart
Physician. 2002;65(12):2479–2486. Study. Arch Intern Med. 2005;165(21):2473–2478.
Hebbar AK, Hueston WJ. Management o common arrhyth- Yancy CW, et al. 2013 ACCF/AHA Guideline or the Manage-
mias: Part II. Ventricular arrhythmias in special popula- ment o Heart Failure: A Report o the American College
tions. Am Fam Physician. 2002;65(12):2491–2496. o Cardiology Foundation/American Heart Association
James PA, et al. 2014 Evidence-Based Guideline or the Man- ask Force on Practice Guidelines. J Am Coll Cardiol. 2013;
agement o High Blood Pressure in Adults. Report rom 62(16):e147–e239.
Pulmonology
R h R. B r
3
note, there is a strong association between OD S and devel-
CASE 3.1 opment o chronic bronchitis.
A 42-year-old male who works in a hog con nement area
presents to your o ce complaining o cough, ever, wheeze, Question 3.1.2 Appropriate treatment or this patient
and dyspnea. He and some other workers were cleaning the includes:
con nement area with high-pressure hoses (which aerosol- A) Antibiotics.
ized hog waste), and they all developed the same symptoms, B) Intubation and mechanical ventilation.
which started between 4 and 8 hours a er work. On examina- C) Supportive care.
tion, he is ebrile with a respiratory rate o 28. He is able to D) A and B.
talk in complete sentences. T ere are slight crackles when you E) A and C.
auscultate the lungs. His chest x-ray is normal.
Answer 3.1.2 The correct answer is “C.” Supportive care is the
usual treatment o OD S. Antibiotics are not needed because
Question 3.1.1 The most likely diagnosis is:
the syndrome is mediated by endotoxins rather than direct
A) “Farmer’s lung” (hypersensitivity pneumonitis).
in ection. “B” is incorrect because this patient is not in signi -
B) Organic dust toxicity syndrome.
cant respiratory distress.
C) Reactive airway disease.
D) Hydrogen sul de poisoning.
E) Bronchiolitis obliterans. HELPFUL TIP:
R b r h h r rk p s r s s r,
Answer 3.1.1 The correct answer is “B.” Organic dust tox- “ r” ( br b h h
icity syndrome (OD S) occurs when moldy or decomposed k, r s s h k r r h
hay and other organic material (such as hog manure) is h rk k) s b z .
moved. Endotoxins are aerosolized and inhaled, leading to
the symptoms. T e tip o here is that everyone on the job site
was a ected. Since hypersensitivity pneumonitis (“A”) is spe-
Keep reading to get to the objectives or this case.
ci c to the individual, generally only one worker at the site
will have symptoms. Compared with OD S, hypersensitivity
pneumonitis may present with an abnormal chest x-ray with
micronodular or reticular opacities. “C” is incorrect because
CASE 3.2
everyone is involved and ebrile, which is not consistent with T is patient’s brother, who also works on a arm, notes that
reactive airway disease. “D” is not correct. Hydrogen sul de every time he unloads hay he has ever, cough, dyspnea, and
poisoning presents as a toxic pneumonitis with pulmonary sputum production. It tends to resolve in 2 to 5 days but reoc-
edema, dyspnea, hypoxia, and loss o consciousness. Hydro- curs when he is re-exposed to hay. Does he wear a mask? Well,
gen sul de also acts as a direct cellular toxin that binds to no. He would look silly and the guys would poke un at him.
cytochrome oxidase system, similar to cyanide. In addition, Besides, none o the other workers on the arm are a ected, and
hydrogen sul de exposure comes rom cleaning manure pits they are beginning to wonder i he is malingering. His examina-
(as anyone in Iowa would know). Finally, “E,” bronchiolitis tion reveals tachypnea and ne rales. T ere is no wheezing pres-
obliterans, is a chronic illness rather than an acute one. O ent. A chest radiograph shows bilateral interstitial opacities.

97
98 Fa mil y med ic in e exa min a t io n & Bo a Rd Review

Question 3.2.1 The most likely cause o this patient’s symp Question 3.2.4 Further evaluation will most likely reveal:
toms is: A) Bronchogenic carcinoma.
A) T ermoactinomyces candidus (an actinomyces species). B) Air space disease (e.g., a pneumonia-like picture).
B) . sacchari. C) Decreased carbon monoxide di using capacity (decreased
C) Botrytis cinerea. DLCO).
D) Cryptostroma corticale. D) Markedly abnormal bronchoalveolar lavage (BAL) with
E) None o the above. lymphocytosis.
E) Obstructive changes on pulmonary unction testing.
Answer 3.2.1 The correct answer is “A.” T is patient pres-
ents with classic symptoms o hypersensitivity pneumonitis Answer 3.2.4 The correct answer is “C.” Hypersensitivity pneu-
or, in this case, “Farmer’s lung.” T is is caused by exposure to monitis can become chronic i exposure is not limited. In these
the Actinomyces species. Acute ndings include ever, chills, cases, patients will generally have systemic complaints such as
cough, dyspnea, and chest tightness. Occasionally, the radio- atigue and possibly weight loss; ever will be absent. Dyspnea
graph is normal. High-resolution chest computed tomography and clubbing o the ngers are also generally noted, re ecting
(C ) should then be obtained, which commonly shows cen- chronic pulmonary disease. Along with this nding, pulmonary
trilobular micronodules and ground-glass opaci cation. “B,” . brosis can occur and the DLCO may be decreased. “A” is incor-
sacchari, is involved in hypersensitivity pneumonitis rom sug- rect. Hypersensitivity pneumonitis does not lead to lung cancer.
arcane (so-called Bagassosis). “C,” Botrytis cinerea, is involved “B” is incorrect. While acute hypersensitivity pneumonitis causes
in hypersensitivity pneumonitis rom grapes (so-called Spat- an alveolitis, chronic hypersensitivity pneumonitis causes pul-
lese lung). Finally, “D,” Cryptostroma corticale, is involved in monary brosis with an occasional micronodular pattern. “D”
“Maple bark stripper’s lung,” another type o hypersensitivity is incorrect. BAL in chronic hypersensitivity pneumonitis does
pneumonitis. not contain the markedly elevated lymphocyte count that is seen
with acute hypersensitivity pneumonitis. Finally, “E” is incorrect.
Question 3.2.2 The correct treatment or this patient with One would see a restrictive pattern on pulmonary unction test-
Farmer’s lung includes: ing re ecting the brosis and not an obstructive pattern.
A) Antibiotics.
B) Inhaled steroids. HELPFUL TIP:
C) Oral steroids. i h p hr rr “p ,” -
D) Leukotriene inhibitors. s r h p rs s p s. i h s s s
E) Bronchoalveolar lavage (BAL). r , s p h p 25%
s s.
Answer 3.2.2 The correct answer is “C.” Oral steroids are e ec-
tive in the treatment o hypersensitivity pneumonitis. Neither
Objectives: Did you learn to . . .
antibiotics (“A”) nor inhaled steroids (“B”) are o any bene t.
• R z h pr s s o dt S h p rs s-
“E,” bronchoalveolar lavage, is not a treatment. However, it can
p s?
be used as a diagnostic tool. One would expect to see lympho-
• m p s h s s r r r
cytes on BAL.
p s r s?

Question 3.2.3 You would advise this patient to:


A) Get a new job. QUICK QUIZ: a St Hma
B) Apply or disability.
C) Use a respirator at work and avoid exposure to this toxin i All o the ollowing populations are at increased risk or devel-
possible. oping asthma EXCEP :
D) Sue the employer. A) Obese children.
E) ake up worm arming or monoculture in rhubarb. B) Female children.
C) Children exposed to tobacco.
Answer 3.2.3 The correct answer is “C.” Wearing an appropri- D) Children with atopy.
ate respirator at work can be bene cial. Avoiding exposure is E) City children.
even better. As or the other answers, you are a doctor not a
lawyer or career counselor. Stick with what you know! The correct answer is “B.” Male children have a greater prev-
alence o asthma. Interestingly, adult women “catch up” so
T e patient is unable to change jobs or wear a respirator that there is gender equity in young adulthood. A er age 40,
because it itches and he “keeps orgetting it.” But he’s per- the prevalence is higher in emales. O note, requent respira-
sistent (or brave or thickheaded or unable to learn a new tory in ections seem to be protective. T ere is an inverse asso-
skill), and keeps arming. T ree years later, he returns with ciation between children living on arms and asthma incidence
a chronic cough, weight loss, dyspnea, atigue, and clubbing (another eather in the cap o Iowa). Presumably, this is related
o the ngers. to the greater variety o antigen exposure.
Ch Apt Er 3 • Pu l mo n o l o g y 99

Answer 3.3.2 The correct answer is “C.” Patients with asthma


CASE 3.3 will have a decreased FEV1. T e FVC may all as well, but FEV1
A 20-year-old woman with no signi cant past medical his- alls rst and to a greater degree as the lung becomes obstructed.
tory presents with a 2-month history o episodic shortness T e ratio o FEV1/FVC is very sensitive to air ow limitations, and
o breath. T ese symptoms began with an upper respiratory FEV1/FVC < 0.7 (not predicted, just the ratio o the two numbers)
tract in ection. She has ts o coughing and trouble catching is generally considered diagnostic o obstructive airway disease.
her breath with exertion. She states that her breath “sounds T e rest are incorrect. LC is not measured by spirometry (which
like whistles” at times. She tried a riend’s albuterol inhaler is why “E” is incorrect); but i it were, LC may be increased in
with some improvement and wonders i she has asthma. On patients with obstructive airway disease due to air trapping.
examination, she is breathing com ortably at 16 times per
minute and her oxygen saturation is 96% on room air. Her Your patient’s o ce spirometry shows the ollowing:
lungs are clear to auscultation, and the remainder o her Normal FVC.
examination is unremarkable. You want to better categorize FEV1 82% predicted.
this patient’s disease. FEV1/FVC 0.68.
Question 3.3.3 These ndings are most consistent with
Question 3.3.1 Which o the ollowing tests is most appro which o the ollowing?
priate to order now? A) Normal spirometry.
A) Spirometry. B) Obstructive lung disease.
B) Chest x-ray. C) End-stage emphysema.
C) Arterial blood gas (ABG). D) Interstitial brosis.
D) Methacholine challenge. E) Obesity-hypoventilation syndrome.
E) Chest C .
Answer 3.3.3 The correct answer is “B.” Always go rst to the
Answer 3.3.1 The correct answer is “A.” Since this patient has FEV1/FVC ratio. In this case, it is < 0.70, which is suggestive
symptoms o bronchospasm, spirometry will be essential in o airway obstruction. T e in ormation provided here lacks
determining i there is objective evidence o obstructive lung data regarding reversibility, so you could not really di erenti-
disease. However, spirometry results are o en normal in mild ate between chronic obstructive pulmonary disease (COPD)
cases o asthma, especially when the patient is asymptomatic. and asthma. But this is clearly not end-stage emphysema, so
Bronchoprovocation testing, with methacholine or histamine, “C” is incorrect. “D” is incorrect. Interstitial brosis is generally
may be use ul in such cases, but should ollow basic spirometry. marked by a restrictive pattern on spirometry and decreased
Although chest radiography (x-ray or C ) may reveal an unsus- LC. Both ow rate (e.g., FEV1) and FVC are decreased in inter-
pected process, it is not indicated in otherwise healthy patients stitial lung diseases but in proportion to each other. T us, the
with symptoms o bronchospasm. Bacterial pneumonia is a FEV1/FVC is o en normal or elevated. See able 3-1 or more
potential precipitant o bronchospasm that may be diagnosed on interpreting spirometry results.
on chest x-ray, but this patient has no constitutional symptoms
(like ever) associated with serious bacterial in ection. Obtain- Six months a er you discuss her ndings and prescribe
ing an ABG may be help ul when a patient presents with respi- inhaled beta-agonist therapy, she returns with complaints o
ratory distress but certainly not in the of ce setting. continued wheezing and di culty breathing. Her symptoms
are brought on by cold weather and exercise and she uses her
inhaler two times per week or less. She woke up two nights
HELPFUL TIP:
over the last 6 months with shortness o breath and coughing.
a r b s p h sh -
Her albuterol still works or these symptoms, but she nds
rb h p s s s h s -
them bothersome and asks, “Why haven’t I gotten over this?”
s p r sp r r r .th rb
(P c o 2) sh b p h h p . Question 3.3.4 How would you categorize this patient’s
t h s, r pp r a Bg h r rb - respiratory state?
s r sp r r s A) Intermittent asthma.
r r sp r r r . B) Mild persistent asthma.
C) Moderate persistent asthma.
D) Severe persistent asthma.
Question 3.3.2 I this patient has mild asthma, which o
E) Recurrent lower respiratory tract in ections.
the ollowing pulmonary unction test results would you
expect to nd? Answer 3.3.4 The correct answer is “A.” According to the
A) Forced vital capacity (FVC) 50% o predicted. National Asthma Education and Prevention Program (2007
B) Forced expiratory volume in 1 second (FEV1) 100% o predicted. NHLBI/NAEPP guidelines; able 3-2), your patient meets the
C) FEV1/FVC ratio < 0.7. criteria or intermittent asthma. In such patients, mild symp-
D) otal lung capacity ( LC) 50% o predicted. toms correspond to an FEV1 (not an FEV1/FVC ratio) that is
E) FEV1/ LC < 0.7. greater than 80% predicted.
100 Fa mil y med ic in e exa min a t io n & Bo a Rd Review

TABLE 3-1 GENEr AL INt Er pr Et At ION OF pULMONAr Y FUNCt ION t ESt


r ESULt S COMpAr ING OBSt r UCt IVE AND r ESt r ICt IVE DISEASE (MAY NOt BE
AppLICABLE FOr ALL FOr MS OF LUNG DISEASE)
PFT Result Obstructive Pattern Restrictive Pattern
Fev 1 < 80% pr d r s pr p r ss

Fvc d r s < 80% pr

Fev 1/Fvc < 0.7 > 0.7

FeF25–75% < 60% pr d r s pr p r ss

t lc n r r d r s

d lco n r r sh ; d r s r s r sr s s ;
r r r s c o Pd r r s r r s sk
r sr s s

Fev 1, r pr r 1s ; Fvc , r p ; FeF25–75, r pr r


25–75% p ;t l c , p ;d l co , s p h r rb ;
c o Pd , hr bs r p r s s .

TABLE 3-2 CAt EGOr IZAt ION OF SEVEr It Y OF ASt h MA AND St EpWISE Appr OACh t O t h Er ApY

Determine Severity When Initiating Therapy


Classification of Asthma Severity (≥12 years of age)
Components of Severity Persistent
Intermittent
Mild Moderate Severe
Symptoms ≤2 days/week >2 days/week but not daily Daily Throughout the day
)
C
Nighttime awakenings ≤2×/month 3–4×/month >1×/week but not nightly Often 7×/week
V
F
SABAa use for symptom >2 days/week but not daily
/
1
≤2 days/week Daily Several times per day
V
control (not prevention of EIBb) and more than 1× on any day
E
F
l
Interference with normal activity None Minor limitation Some limitation Extremely limited
a
m
r
o
• Normal EFV1
N
(
between
t
n
Lung function exacerbations
e
m
• EFV1 > 80% • EFV1 > 80% predicted • EFV1 > 60% but • EFV1 < 60% predicted
r
i
pa
predicted <80% predicted
m
I
• EFV1/FVC normal • EFV1/FVC normal • EFV1/FVC reduced 5% • EFV1/FVC reduced > 5%

0–1/year ≥2/year
Exacerbations requiring
k
oral systemic corticosteroids Consider severity and interval since last exacerbation
s
i
R
Frequency and severity may fluctuate over time for patients in any severity category
Relative annual risk of exacerbations may be related to FEV1
Recommended step for Step 1 Step 2 Step 3 Step 4 or 5
initiating therapy and consider short course of
See bar chart in Figure 3–1 oral systemic corticosteroids
for treatment steps In 2–6 weeks, evaluate level of asthma control that is achieved and adjust therapy accordingly.
a
Sa Ba , sh r - h b -2 s ; b eiB, r s - br h sp s .
R pr r g s r h d ss m as h .n as h e Pr Pr r ,n i s s
H h. e p r P R p r 3, p s 305–310, 343–345. h p:// . h b . h. / s/ s h .
Ch Apt Er 3 • Pu l mo n o l o g y 101

Inte rmitte nt Persistent Asthma: Daily Medication


As thma Consult with asthma specialist if Step 4 care or higher is required. Consider consultation at Step 3.

Step 6
Step 5 Step up
Step 4 Preferred
Preferred
if needed
Step 3 High-dose
Preferred High-dose (first, check
Step 2 Preferred Medium-dose ICS + LABA
ICS + LABA +
adherence,
Step 1 oral
Preferred Low-dose ICS + LABA environmental
AND corticosteroid
Preferred Low-dose ICS + LABA§ Alternative control, and
Consider
SABA* PRN ICS† OR Medium-dose AND comorbid
Alternative omalizumab for
Medium-dose ICS ICS + either Consider conditions)
Cromolyn, patients who
Alternative LTRA, omalizumab for Assess
LTRA,‡ have allergies
Low-dose ICS + theophylline, patients who control
nedocromil, or either LTRA, or zileuton have allergies Step down
theophylline theophylline, or if possible
zileuton (and asthma is
Patient Education, Environmental Control, and Management of Comorbidities at Each Step well controlled
Cons ide r s ubcuta ne ous a lle rge n immunothe ra py for pa tie nts who have a lle rgic a s thma a t S te ps 2 through 4 at least
3 months)
Quick-Relief Medication for All Patients
• SABA as needed for symptoms. Intensity of treatment depends on severity of symptoms: up to 3 treatments at
20-minute intervals as needed. Short course of oral systemic corticosteroids may be needed
• Use of SABA >2 days a week for symptom relief (not prevention of EIBµ) generally Indicates inadequate control and
the need to step up treatment
† ‡ ‡ µ
* Short-acting inhaled beta2-agonist. Inhaled corticosteroid. Leukotriene receptor antagonist. Long-acting inhaled beta2-agonist. Exercise-incluced bronchospasm.

FIGURE 3-1. t k s p s r ppr h sh .

Question 3.3.5 Which o the ollowing is most appropriate Answer 3.3.6 The correct answer is “A.” Your patient now has
or this patient given that she has intermittent asthma? mild persistent asthma and should be started on an inhaled
A) Add theophylline. steroid. When asthma symptoms become more persistent (i.e.,
B) Add montelukast. when they occur > 2 days per week or the patient awakens rom
C) Continue albuterol as needed. sleep > 2 times per month), the in ammatory component o the
D) Schedule albuterol every 4 hours. disease should be addressed while simultaneously treating the
E) Prednisone 5 mg daily. bronchospastic component with short-acting beta-2 agonists.
Anti-in ammatory drugs are the mainstay o chronic asthma
Answer 3.3.5 The correct answer is “C.” As already discussed, therapy, and inhaled corticosteroids are the most ef cacious
this patient appears to have intermittent asthma. She is in with the ewest side e ects. Although ipratropium, cromolyn
no respiratory distress, is oxygenating normally, and is still sodium, and montelukast have a place in asthma treatment,
responding well to albuterol by her report. Although there is none o these medications is a rst-line agent. Ipratropium works
some debate about the role o inhaled steroids in intermittent through its bronchodilatory e ects, while cromolyn sodium is
asthma, the NAEPP and most experts do not recommend their a mast cell stabilizer. Montelukast is a leukotriene inhibitor. T e
use. Oral prednisone is certainly not indicated in this case. She long-acting inhaled beta-2 agonists, such as salmeterol, are only
should be continued on a short-acting inhaled beta-2 agonist, recommended at Steps 3 and higher o persistent asthma con-
such as albuterol, without the addition o another medication. trol ( able 3-2, Fig. 3-1; NHLBI recommendations, 2007).
“D” is incorrect. Scheduled albuterol actually yields less e ective
symptom control than does PRN use.
HELPFUL TIP:
Your patient goes on to develop more requent recurrent symp- R b r h “r s”: p h h s>2
toms, such that she is using her albuterol inhaler more than three sh rb sp r kr q r r s -
times per week, although her nighttime symptoms are rare. r h k s h r s p s> 2 s
p r h sh b - r r , pr -
Question 3.3.6 Which medication is the most appropriate r b h r s r . as h ss
next step in treating this patient’s asthma? r h s r s p —
A) Inhaled triamcinolone. 74 p s h( hss h s r rs !). y
B) Inhaled salmeterol. r h s, h ps://
C) Inhaled cromolyn sodium. . h b . h. /h h-p r / s/ rr /
D) Inhaled ipratropium. sh - s/s r -r p r -2007
E) Oral montelukast.
102 Fa mil y med ic in e exa min a t io n & Bo a Rd Review

A) Inhaled triamcinolone and inhaled albuterol as a “rescue.”


HELPFUL TIP:
B) Inhaled triamcinolone, oral montelukast, and inhaled
th k r h b rs ( . ., k s, z r -
albuterol as a “rescue.”
k s) r h z h s r
C) Oral montelukast and inhaled albuterol as a “rescue.”
h r p .i , h r r s s -
D) Inhaled albuterol as a “rescue.”
h s r s. t h sh b s p
E) Inhaled salmeterol and inhaled triamcinolone.
h s h s r s sh b h
r ; h r s bs r h s r s.
Answer 3.3.8 The correct answer is “B.” Leukotriene inhibitors
(e.g., montelukast, za rlukast) have demonstrated e ectiveness
Your patient does quite well over the next year, having very ew in reducing symptoms and improving peak ow in patients with
exacerbations. During one o her visits, you note slightly edem- aspirin sensitive asthma. Leukotriene inhibitors should be
atous nasal mucosa and nasal polyps. You prescribe intranasal used only in asthma patients who are already using a corticoste-
steroids. roid inhaler—or those who cannot tolerate inhaled corticoste-
roid therapy. T ere ore, “C” is not an appropriate choice. “D” is
One night when you are on call, she comes in severely dys- incorrect because there is no anti-in ammatory. Although “E”
pneic with audible wheezing. She talks in two- or three-word o ers an anti-in ammatory agent, there is no rescue inhaler,
phrases. She reports a headache today, which she treated with and patients with asthma must always have access to a short-
aspirin (something she never takes but a riend gave it to her acting inhaled bronchodilator.
thinking it was acetaminophen). Her asthma attack started
about an hour a er the aspirin dose. She has been otherwise
HELPFUL TIP:
well. She denies ever, rhinorrhea, nasal congestion, and sore
o h r n Sa id s h b p “ sp r - r-
throat. Her respiratory rate is 40, heart rate 120 bpm, and
b ” s h .th h r s h h r s b
oxygen saturation 88% on room air. She has poor air move-
b pr - - r rs h s
ment on auscultation o her lung elds.
rb b c o x-1 h b .

Question 3.3.7 Which o the ollowing is the most likely


reason or this patient’s acute exacerbation o asthma? Question 3.3.9 Which o the ollowing medications, when
A) Viral upper respiratory in ection (URI). used alone as maintenance therapy in persistent asthma,
B) Sinusitis. is associated with an increased risk o asthma related
C) Noncompliance with inhaled albuterol. mortality?
D) Sensitivity to aspirin. A) Inhaled uticasone.
E) Noncompliance with nasal steroids. B) Inhaled salmeterol.
C) Oral za rlukast.
Answer 3.3.7 The correct answer is “D.” It is likely that this D) Oral prednisone.
patient has aspirin sensitivity. Up to 10% o adults with asthma
have the clinical triad o asthma, aspirin sensitivity, and nasal Answer 3.3.9 The correct answer is “B.” Inhaled salmeterol,
polyposis. Patients with asthma should be warned about the when used alone as a controller agent or asthma, has been asso-
potential or exacerbations resulting rom consumption o aspi- ciated with a two- to our- old increase in the risk o death related
rin and nonsteroidal anti-in ammatory drugs (NSAIDs). T e to asthma or other respiratory conditions. T us, the Food and
drug-induced bronchial constriction caused by these medica- Drug Administration (FDA) has mandated a “black box” warn-
tions can have an abrupt onset with severe symptoms. Patients ing be applied to salmeterol-containing products. It is not known
with aspirin sensitivity can be desensitized with daily admin- whether inhaled steroid therapy is protective, but NHLBI/
istration o small amounts o aspirin, but this should be done NAEPP guidelines recommend adding long-acting inhaled beta-
care ully with close supervision. Although viral URIs requently agonists only a er inhaled steroids are already in use.
cause exacerbations o asthma, your patient did not report ante-
cedent symptoms o such an in ection. Further discussion o HELPFUL TIP:
the treatment o an acute asthma exacerbation can be ound in th p r p sh -
Chapter 1, “Emergency Medicine.” b rs . P s s h sh
sh r r p , h
A er a brie hospitalization, your patient recovers nicely. r s b -2 s s h s r r
Prior to this incident involving aspirin, she had been ree o s r . wh h r s pr b h p k
exacerbations or about a month. r , hs s r p s r
h r ss. a h p k
Question 3.3.8 In addition to a short course o oral ste r b s s p r h
roids, which o the ollowing medication regimens do you pr ss h b h
prescribe or this patient with aspirin sensitive asthma at h h r pr r h p .
discharge?
Ch Apt Er 3 • Pu l mo n o l o g y 103

Me as ure Ac tual % Pre dic te d How do you interpret these ndings?


12
FVC 3.66 L 103 A) Chronic obstructive lung disease.
10 B) Restrictive lung disease.
FEV1 2.30 L 83
8 C) Fixed upper airway obstruction.
6
FEV1 /FVC 0.63 D) Poor patient e ort.
FEF(25–75) 2.15 L 51 E) Within normal range.
4
2 The correct answer is “C.” T e attened ow/volume loop is
S
/
L
0 consistent with a xed upper airway obstruction. In this case,
w
1 2 3 4 5 6
FEV1 and the FEV1/FVC ratio may look like other obstruc-
o
l
–2
F
tive diseases (i.e., asthma, COPD), so you have to look at the
–4
ow/volume loop (always a good idea). Some examples o
–6 ow/volume loops are given below (Fig. 3-3).
–8
–10 CASE 3.4
–12
Volume (L)
You are seeing a 65-year-old male in the emergency depart-
ment (ED) where he presented with complaint o increasing
FIGURE 3-2. Sp r r r s s. shortness o breath. He has obvious di culty breathing and
cannot speak in ull sentences. However, you are able to elicit
that he has been having increasing respiratory problems over
Objectives: Did you learn to . . . the last 3 to 4 days. He has known COPD with FEV1 o “less
• i r rs br h sp s ? than one” (normal FEV1 is about 4 L or a 50-year-old male
• e s p s h z sp ? and 3 L or a 50-year-old emale— or the calculations based
• c ss sh ? on age, etc., see www.hankconsulting.com/Re Cal.html). He
• Pr s r b ppr pr s r r has been using his inhalers much more than usual but with
p rs s sh ? minimal improvement. He has smoked one pack per day
• d s rb h r s h , sp r s s , s since age 18 (but proudly points out he quit 2 days ago) and
p p s s? has a past medical history o high cholesterol, obesity, and
hypertension.
On examination, he has a respiratory rate o 26 to 28,
QUICK QUIZ: SPiRo met Ry heart rate o 100 bpm, blood pressure o 130/90 mm Hg,
and temperature o 37.7°C. His O2 saturation is 84% on
A 60-year-old emale who smokes two packs o cigarettes per room air. On auscultation, you do not appreciate much
day complains o shortness o breath and ullness in her throat. due to his body habitus but you still manage to hear some
You obtain spirometry in the of ce and the results are given wheezing. He has a normal cardiac examination and no
above along with the ow/volume loop (Fig. 3-2). lower extremity edema.

Flow Flow
Flow 8 8
8
6 6
6
4 4
4
2 2
2
0 0
0

–2 –2 –2

–4 –4 –4

–6 –6 –6
–1 0 1 2 3 4 –1 0 1 2 3 4 –1 0 1 2 3 4
Volume Volume Volume
Norma l pa tte rn Obs tructive pa tte rn Re s trictive pa tte rn
FIGURE 3-3. Sp r r r s s.
104 Fa mil y med ic in e exa min a t io n & Bo a Rd Review

Question 3.4.1 What is the next best step to help this patient? now you cannot tell easily whether his O2 needs are going up
A) Per orm emergent endotracheal intubation. or down. “D” is o special note. Patients with COPD who look
B) Administer supplemental O2 via nasal cannula. com ortable may be becoming hypercapnic and developing CO2
C) Administer Solu-Medrol, 1 g IV. narcosis. T us, while com ort is a goal, it may not be the best
D) Start antibiotics IV. judge o clinical status in patients with COPD exacerbations. o
E) Obtain a chest x-ray. assess CO2 levels, you will need an ABG or VBG.

Answer 3.4.1 The correct answer is “B.” T is patient is hypoxic, Your patient is now on 5 liters per minute (LPM) o oxygen via
and your rst priority should be to improve his oxygenation. nasal cannula. You obtain an ABG: pH 7.29, PCO2 74 mm Hg,
T ere exists a theory that oxygenating patients with COPD will PO2 58 mm Hg. T e patient is awake and alert but says that he
suppress their respiratory drive. T e classical teaching (mostly still eels “like dirt.” He remains tachypneic, in obvious respi-
incorrect) is that COPD causes a switch rom carbon dioxide ratory distress, with a respiratory rate o 28. Albuterol and
levels driving respiration to oxygen levels driving respiration. ipratropium are given via nebulizer.
While this may be partly true, urther study has suggested that
the main reason COPD patients are at risk o worsening hyper- Question 3.4.3 What is the best next step?
capnia is due to loss o hypoxic pulmonary vasoconstriction A) Clearly, he is ailing therapy—emergently intubate.
and worsening ventilation–per usion mismatch that occurs B) Increase his O2 to 100% via ace mask.
with excess oxygen delivery. Regardless o that, you need to rst C) Initiate noninvasive positive pressure ventilation
worry about this patient’s oxygenation. (e.g., BiPAP).
“A” is incorrect. T e patient is protecting his airway and you D) Start IV antibiotics.
have not attempted improving his oxygenation with less inva- E) Obtain a chest C with PE protocol (you are barking up the
sive methods yet. Administering steroids, IV or PO, will have wrong bronchial tree).
no immediate e ect on his respiratory status. In act, IV steroids
might be worse than PO steroids (there is question o whether Answer 3.4.3 The correct answer is “C.” T is patient is retain-
higher IV doses cause immunosuppression and subclinical ing CO2 despite tachypnea and is in impending respiratory
myopathy; see Am J Respir Crit Care Med. 2014;189:1052). Anti- ailure. He is also not oxygenating well despite low- ow oxy-
biotics and a chest x-ray may be reasonable, but with low O2 gen. Noninvasive positive pressure support (BIPAP) can relieve
saturations, your priority is to quickly improve your patient’s hypercapnia and improve oxygenation by decreasing work o
oxygen status. breathing without requiring intubation and its associated mor-
bidity. O en IV antibiotics (“D”) are used or empiric therapy in
severe exacerbations o COPD, but again, improving the respi-
HELPFUL TIP:
ratory status comes rst. He is hypoxic, but his main problem
P s h sp r c o Pd rb
is CO2 retention—increasing his O2 delivery will not alleviate
(h r rz b r s sp , sp pr -
that (thus, “B” is incorrect). Although his respiratory status is
sp p r ) sh b r h r-
tenuous, he is not in imminent respiratory ailure, and intu-
s r s ( r pr s s q iv s r s),
bation (“A”) is not warranted at this time. Chest C (“E”) can
b h rs r sp r r r p h s
rule out a PE in this hypoxic patient and may con rm COPD
( r s b s r p b , z hr -
changes including emphysema but will greatly delay treatment
, , , h rs) -
and would not change your immediate course o action.
h br h rs. S pp r h r p h s pp -
s r q r s .
A er several hours o noninvasive positive pressure ventila-
tion, your patient is doing well and he is trans erred out o
Question 3.4.2 When initiating supplemental oxygen by the intensive care unit (ICU). When you see him the next day,
nasal cannula, you instruct the nurse to keep the patient’s his medications include inhaled bronchodilators, ce riax-
oxygen saturation: one, azithromycin, and prednisone. A chest x-ray shows no
A) Between 96% and 100%. evidence o in ltrate. He is weaned rom the positive pres-
B) Between 90% and 95%. sure support and is now on 2 LPM o oxygen by nasal can-
C) Between 85% and 89%. nula. He appears com ortable, with a respiratory rate o 14
D) At whatever saturation he looks most com ortable. and an oxygen saturation o 95%.

Answer 3.4.2 The correct answer is “B.” T e primary goal o Question 3.4.4 Now you know that he has COPD, what
supplemental oxygen is to reduce the risk o tissue hypoxia. would be the best treatment or him?
Maintaining oxygen saturations above 90% (or PaO2 60–65 mm) A) Inhaled long-acting beta-agonist (LABA) + albuterol.
will ensure tissue oxygenation. Higher oxygen saturations may B) Inhaled steroid + albuterol.
result in CO2 retention and hypercapnia, as noted earlier. Also, C) Inhaled steroid + LABA + albuterol as needed.
aiming at 100% with excessive levels o O2 supplementation D) Albuterol.
takes away an important patient assessment parameter because E) Albuterol and theophylline.
Ch Apt Er 3 • Pu l mo n o l o g y 105

Answer 3.4.4 The correct answer is “C.”T is is a bit tricky, but due Answer 3.4.5 The correct answer is “E.” Aside rom oxygen, no
to the severity o his exacerbation, “C” is the best choice. Compared medical therapy has clearly demonstrated a mortality bene t or
with patients with asthma, where long acting bronchodilators are stable COPD (see Fig. 3-4 or stage o COPD and appropriate
not indicated as rst-line therapy, COPD patients have a clear symp- treatment). Another therapy to consider in advanced COPD is
tomatic bene t rom long-acting bronchodilators. T e two available pulmonary rehabilitation (PR). While having no de nite e ect
choices are anticholinergics (e.g., tiotropium) or LABAs (e.g., sal- on survival, PR improves dyspnea and quality o li e scores
meterol or ormoterol). Either choice is ne. An inhaled steroid while reducing the number o hospitalizations and days in hos-
does not modi y the long-term decline in FEV1 in patients with pital. Finally, there is a new oral phosphodiesterase inhibitor,
COPD. However, it will reduce the requency o exacerbations ro umilast (Dalisrep), or severe COPD. As you probably
and thus improve health status. Current guidelines recommend already know, your authors are curmudgeons who don’t like
addition o inhaled steroids with FEV1< 50% predicted and re- anything new. But ro umilast is particularly on our bad list. It
quent exacerbations (e.g., three in the last 3 years), see Figure 3-4. increases FEV1 by 45 mL over placebo. It has no e ect on qual-
ity o li e and exacerbations are 1.3/year with placebo and 1.2/
year with rol umilast. It also causes diarrhea and depression/
HELPFUL TIP:
a r th r S (a t S) s rz
suicidality. Nonetheless, you should know about it.
bs r r s s b s h p r pr -
Fev 1
Note: Long-acting bronchodilators can be either a beta agonist
h p . t h r sp s br h -
r s s rz b a t S. t s h h r s
or an anticholinergic.
s r sp s br h r, h r h s b
r s b h 200 r s s HELPFUL TIP:
12% h Fev 1. n h h s r s sh b s h s h
S 3 S 4( r s r ) c o Pd b s
h g b r o bs r l ds s
Holding his inhalers and smiling toothlessly, your patient (g o l d ) s. t h r s b p s h
asks, “Which o these is going to keep me alive—just in case I S 1 rS 2( ) c o Pd h r s s
can’t a ord both?” ( r s p ).

Question 3.4.5 Which o the ollowing medication regimens


has demonstrated decreased mortality in the treatment o Later that year, the same patient gets admitted to the hospital
stable COPD? or community-acquired pneumonia. During his stay in the
A) Inhaled tiotropium (a long-acting anticholinergic). hospital, the hospitalist orders a C chest “to rule out other
B) Inhaled salmeterol (LABA). things.” T e patient recovers rom his in ection and returns to
C) Inhaled ipratropium (a short-acting anticholinergic). you with the CD o his C chest images. T e reading o the C
D) Inhaled corticosteroid. chest describes a 2-cm pulmonary nodule in the right upper
E) None o the above. lobe along with extensive subcarinal lymphadenopathy.

S ta ge 1 S ta ge 2 S ta ge 3 S ta ge 4
FEV1/FVC<70% FEV1/ FEV1/ FEV1/FVC <70%
FEV1>80% FVC <70% FVC <70% FEV1 <30% or
pre dicte d FEV1 <80% FEV1 <50% <50% with
but >50% but >30% chronic re s p
fa ilure

Mild COP D Mode ra te S e ve re COP D Ve ry S e ve re


COP D COP D

Influe nza va ccine , s moking ce s s a tion, s hort a cting bronchodila tor

Long a cting bronchodila tor, P ulmona ry re ha bilita tion

Inha le d Glucocorticoids if fre que nt


e xa ce rba tions
Long te rm
Oxyge n, S urge ry

FIGURE 3-4. g b bs r s s c o Pd s r h r p .
106 Fa mil y med ic in e exa min a t io n & Bo a Rd Review

Question 3.4.6 What is the next best step in management spontaneously combusting right and le (like Spinal ap
o this patient? drummers).
A) Repeat C chest in 3 months. D) Low- ow O2 used at least 15 hours a day signi cantly enhances
B) Repeat C chest in 6 months. survival.
C) Re er or bronchoscopy with endobronchial ultrasound- E) Low- ow O2 is a well-tolerated and e ective treatment or
guided biopsy. obstructive sleep apnea.
D) Re er to an oncologist.
E) Re er or a mediastinoscopy. Answer 3.4.8 The correct answer is “D.” Patients who use low-
ow O2 at home or 24 hours a day have an improved rate o sur-
Answer 3.4.6 The correct answer is “C.” T is patient probably vival. Patients should be encouraged to use O2 at least 15 hours
has a malignant disease and tissue is needed to either diagnose a day i possible. “B” is incorrect because patients need at least
it or rule it out. Bronchoscopy with endobronchial ultrasound 15 hours o O2 per day to have any signi cant bene t with regard
(EBUS)-guided ne-needle biopsies is a minimally invasive pro- to pulmonary hypertension. “C” is incorrect. Clearly, smoking
cedure that can relatively sa ely obtain a tissue sample or the while on O2 is not a good idea, but patients can turn o their O2
pathologist. T ere is good evidence that EBUS has high sensitiv- supply while smoking.
ity and speci city compared with PE scanning. “A” and “B” are
incorrect as this patient has a nodule that is larger than 1 cm along
with mediastinal lymphadenopathy. T is needs to be worked up HELPFUL (AND IMPORTANT) TIP:
and cannot wait 3 to 6 months. “D” is incorrect. Your patient will d s ss h p 20% p s h sp z
likely need to see an oncologist, but you need to rst provide a h c o Pd rb k r -
tissue diagnosis. “E,” mediastinoscopy, would likely provide you h Pe. d ’ rr r r s !
with the diagnosis, but it is a ar more invasive procedure than
bronchoscopy and carries higher mortality and morbidity.
Objectives: Did you learn to . . .
• R z p pr s hs s s p s
HELPFUL TIP: c o Pd ?
i r s h p alf p s treated r c o Pd • d s rb s s r r r c o Pd ?
’ h .S b r . c h k PFt s (c h s .
• d p p h p h p r p ?
2015;147:369).
• m s r rb s c o Pd ?
• dr h r p r hr c o Pd ?
Your patient returns with increasing dyspnea now at rest. His • i s s sp h r h c o Pd pr s
p hk c o Pd ?
biopsy was negative. Despite both o you being blue in the
ace, he has not quit smoking.
QUICK QUIZ: c o Pd
Question 3.4.7 Criteria or the use o continuous low ow oxy
gen in those with COPD include all o the ollowing EXCEPT:
A) PO2 < 55 mm Hg. In a patient with COPD, a lung transplantation re erral can be
B) Oxygen saturation < 88%. considered:
C) PO2 o < 59 mm Hg with evidence o cor pulmonale. A) Once the patient requires oxygen.
D) Episodic sleep apnea-related desaturations at night. B) When you eel you have run out o interventions.
C) Once insurance accepts your re erral.
Answer 3.4.7 The correct answer is “D.” Episodic sleep apnea– D) A er the patient meets strict criteria or the re erral.
related oxygen desaturations, while a cause or concern and ame- E) I the patient’s amily keeps asking or the re erral.
nable to treatment (e.g., CPAP), is not one o the criteria or the
use o continuous low- ow oxygen. T e other choices are cor- The correct answer is “D.” T e International Society or Heart
rect. “C” deserves some special attention. Evidence o cor pul- and Lung ransplantation created a set o criteria or when to
monale can include “p-pulmonale” on ECG, peripheral edema, consider transplantation and, more importantly, when should
or a hematocrit > 55%. you consider re erral or a lung transplant consultation. In COPD
patients, the indication or re erral is easy to remember and con-
Question 3.4.8 Concerning hypoxemic patients with COPD, sists o BODE index > 5 (BODE stands or Body-mass-index,
which o the ollowing is true? air ow Obstruction, Dyspnea, and Exercise, which are the char-
A) Patients on continuous, low- ow O2 become oxygen depen- acteristics used in the index). T e BODE index is an alternative
dent and cannot unction without it. to the GOLD criteria. T e BODE index incorporates unctional
B) Continuous low- ow O2 used or at least 8 hours a day helps criteria such as breathlessness and 6-minute walking distance.
to reverse pulmonary hypertension. Find a BODE calculator here: http://www.qxmd.com/calculate-
C) Concurrent smoking is a contraindication to the prescrib- online/respirology/bode-index. Criteria or transplantation in
ing continuous low- ow home O2 because patients are COPD are more complex and are beyond the scope o this book.
Ch Apt Er 3 • Pu l mo n o l o g y 107

ound in patients with a PE is sinus tachycardia. But alas, Sarah


CASE 3.5 had a normal heart rate. T e other choices can certainly be ound
Ms. Sarah Bellum (i you’re not smiling, try saying the name with this condition but are ar less requent. O note, the “text-
out loud or the joke is just lame . . .) is a 32-year-old Caucasian book” S1Q3 3 ECG rarely occurs and historically traces back to
emale who presents to your ED with shortness o breath. She a hand ul o patients in the 1930s that had massive pulmonary
just returned home a er the International Con erence on emboli. Even i you do spot this pattern on an ECG, it is not
Coordination in London. Immediately a er walking through speci c enough to con rm the diagnosis. In the end, the clini-
her ront door, she became acutely short o breath (not attrib- cal signs attributed to pulmonary emboli (such as the shortness
utable to the Justin Bieber poster in her living room). T is o breath and chest pain) are requently more valuable than any
is associated with some moderately sharp chest pain located abnormal ECG nding.
along the le side o her chest. T e pain seems worse when
she attempts to breathe deeply. As you attempt to rule out other potential etiologies or the
patient’s symptoms (e.g., pneumonia, atelectasis, and pneu-
Question 3.5.1 Which important question(s) do you next mothorax), you order a trusty chest radiograph.
ask Ms. Bellum?
A) Do you smoke cigarettes? Question 3.5.3 What is the most common radiographic nd
B) When was your last menstrual period? ing in a patient with a PE?
C) Have you had surgery recently? A) Pleural e usion.
D) Do you have a history o kidney disease? B) No acute cardiopulmonary processes.
E) All o the above. C) Westermark sign.
D) Hilar/mediastinal enlargement.
Answer 3.5.1 The correct answer is “E.” Each o these questions E) Hampton hump.
addresses risk actors associated with pulmonary embolism (PE)
and/or deep vein thrombosis (DV ). Smoking cigarettes and Answer 3.5.3 The correct answer is “B.” Admittedly, this one is
recent surgery are strong risk actors, as is an active pregnancy. a bit tricky. Approximately 75% o the chest radiographs in the
Addressing a patient’s menstrual cycle serves as a natural segue setting o PE are abnormal. However, there are numerous causes
to a discussion about the use o oral contraceptives, which, too, is or these abnormalities and none o them individually surpass
a prominent risk actor. As or renal disease, nephrotic syndrome the requency o the normal chest radiographs. Speci cally, the
has been associated with an increased risk o PE. “textbook” ndings o Westermark sign (loss o peripheral vas-
cular markings) and Hampton hump (a wedge-shaped opacity
A er urther verbal probing, you discover that Ms. Bel- due to pulmonary in arction) are in requent, and both have a
lum recently completed her menstrual cycle and the other low sensitivity and low speci city. In short, all the other options
presented questions turned up no risk actors. However, can be seen as the result o a PE but none are more requent than
your smooth segue did reveal that she takes low-dose estro- a normal chest radiograph.
gen or birth control. During this discourse, you also learn
that her aunt had a blood clot in her leg once. She has no ECG and chest radiograph in hand, you turn your attention
urther details but does not think that her aunt had any toward ordering the appropriate laboratory tests to solidi y
urther complications rom this condition. Regardless, PE your presumptive diagnosis. You are working with a medical
just took a violent leap to the top o your di erential. You student who suggests a number o lab tests. You agree with
glance at her vitals (temperature 37.1°C, heart rate 92 bpm, most o them, but shoot him down on one.
blood pressure 129/68 mm Hg, respiratory rate 21, SpO2 95%
on room air) and notice that she appears mildly uncom ort- Question 3.5.4 Which test should you AVOID ordering?
able but not in any acute distress. Her physical examination A) CBC.
is entirely unremarkable. You order an ECG on the patient B) D-dimer.
to evaluate or potential cardiac etiologies or her symptoms. C) P /P .
D) Basic metabolic panel (Na+ , K+ , Cl− , CO2− , BUN, Cr − , and
Question 3.5.2 Assuming Ms. Bellum does have a PE, what glucose).
is her ECG most likely to show? E) Urine pregnancy test.
A) S1Q3 3.
B) Nonspeci c S - wave changes. Answer 3.5.4 The correct answer is “B.” T e D-dimer can be a
C) Sinus tachycardia. blessing or some but is the bane o existence or others. In this
D) Normal sinus rhythm. patient, a D-dimer is not use ul. T is test has great sensitivity
E) Multi ocal atrial tachycardia. but poor speci city. It is positive in ar more conditions than
PE. Used as a “rule-out” test or PE, it only applies in low-risk
Answer 3.5.2 The correct answer is “D.” T e most common patients. Ms. Bellum is not a low-risk patient as suggested by the
ECG nding associated with the diagnosis o PE remains nor- pulmonary embolism rule-out criteria (PERC) rules (see Help-
mal sinus rhythm. With that said, the most common arrhythmia ul ip) due to her use o exogenous estrogen. T e Wells criteria
108 Fa mil y med ic in e exa min a t io n & Bo a Rd Review

or PE place her in the moderate-risk group (16.2% risk o PE). TABLE 3-3 WELL’S SCOr E FOr pEa
As such, even a negative D-dimer is insuf cient or ruling out
c s sp d vt 3p s
the diagnosis. As or the other tests, they all serve a valuable
role in her evaluation. For instance, the CBC could provide evi- n r ss r k h Pe 3p s
dence o anemia, while the P /P may reveal a coagulopathy.
t h r > 100 1.5 p s
Assessing her renal unction may be needed or her evaluation
moving orward, and the same can be said or veri ying her i b z r≥3 s rs r r h 1.5 p s
nongestational status. Plus, a urine pregnancy test is per ormed pr s4 ks
on almost every woman in an ED. It might as well be part o the
Hs r d vt r Pe 1.5 p s
triage process.
m h 6 hs 1p
T at pesky med student seemed to know a lot about the PERC Pr s h p ss 1p
rules and Wells criteria. But when he listed the Wells criteria,
he got one wrong. S r >6: h h pr b b Pe (78%)
S r 2–6: r pr b b Pe (27.8%)
Question 3.5.5 The Wells criteria or PE include all o the S r <2: pr b b Pe (<3.4%).
a
- r s only s r h pr b b p .o h r s ,
ollowing EXCEPT: h p s - s pr b b h r - r s h h
A) Estrogen use. h r sk r Pe.
B) Pulse > 100.
C) Previous history o venous thromboembolism.
D) Clinical symptoms and signs consistent with PE.
E) Hemoptysis. T e CBC, coagulation studies, and basic metabolic panel all
return within normal limits. In addition, Ms. Bellum is not
Answer 3.5.5 The correct answer is “A.” While an important pregnant. T us, you wish to ( nally) solidi y that diagnosis
risk actor or PE, estrogen use is not included in the Wells crite- you have suspected or quite some time.
ria. All the others count in the Wells criteria ( able 3-3). Using a
medical calculator website, such as www.medcalc.com, is most Question 3.5.6 Since you do not put her in the low risk cat
help ul. egory by your clinical judgment, what diagnostic study
should you order?
A) VQ (ventilation–per usion) scan.
B) C scan o the chest without contrast.
HELPFUL TIP:
C) C scan o the chest with contrast.
t h PeRc r s r s r s h -
D) Pulmonary angiogram.
rz p -r sk r p r
E) Compressive Dopplers o the lower extremities.
Pe .i h p s all h , Pe s
r ( ss b h p s r sk):
Answer 3.5.6 The correct answer is “C.” T e American College
1) a < 50.
o Radiology (ACR) lists the C scan o the chest with contrast
2) H r r < 100.
(i.e., C angiography or C A) as the modality o choice in
3) S o 2 > 94%.
stable patients with a suspected PE. T e C A is considered
4) n r s .
to be the standard o care. Its bene ts include the act that it
5) n h p s s.
is noninvasive, cheaper than pulmonary angiography, and ar
6) n r hs r r rs r r .
more available than VQ scans. It should be noted that pulmo-
7) n pr r d vt r Pe.
nary angiography still remains the “gold standard” or diag-
8) n h r s .
nosing pulmonary emboli, but that is more o an academic
point. As or VQ scans, they are not available in many locales
and o en return nondiagnostic. However, they can be used in
HELPFUL TIP: a patient with a normal chest x-ray. A chest C without con-
R b r d- r -r sk p s. t h s trast (“B”) will not enhance the pulmonary arteries, making
s p r s s h ct r p s r s- the diagnosis o a PE ar more dif cult, i not impossible. Since
s r r . th r r 20- r- most pulmonary emboli are believed to arise rom the lower
r 64-s h s c t s 1:142 (r ) extremity venous system, “E,” Dopplers o the legs, could be
(Ja ma . 2007;298:317–323)! S s l . 2009;374: considered i the patient was a poor candidate or both C A
1160 JP r. 2009;154:912 h rs. P s, and VQ scan (e.g., COPD and Stage 4 chronic kidney disease).
25% “p s ”c t s -r sk p s r s p s- But this approach is obviously not diagnostic o PE; it would
, h h h (Am just help you determine i the patient has an active thrombo-
J Roentgenol. 2015;205:271). sis, and you would manage the same whether she has a PE or
DV or both.
Ch Apt Er 3 • Pu l mo n o l o g y 109

the American College o Chest Physicians (ACCP), published


HELPFUL TIP:
online in January 2016, recommends novel oral anticoagulants
S pr rs s Pe b s ct a
(e.g., dabigatran, apixaban, and others) over war arin or treat-
h b d pp rs h s. w h b h? i
ment o PE in patients without cancer; however, the ACCP
d vt Pe, r h s s
grades this recommendation as “weak with moderate quality
r h h r. a r h s
evidence.” Continue to look at the whole patient when making
Pe, b h .S h d vt .
anticoagulation decisions.

HELPFUL TIP:
Question 3.5.8 How long are you going to maintain this
i pr , b h h s ct v/Q s r -
patient on anticoagulation?
p b p s p b r -
A) 3 months.
s (o bs g . 2011;118:718).
B) 6 months.
C) 9 months.
D) Li etime.
As keenly suspected, Ms. Bellum’s C A o the chest reveals a
Answer 3.5.8 The correct answer is “A.” For a PE that has a
moderate-sized pulmonary embolus in the le pulmonary
reversible cause (oral contraceptive pills in this patient with a
artery. Her vital signs are still stable and her pain is well-
long airplane trip), 3 months o anticoagulation is adequate. For
controlled with intravenous morphine. She is surprised by the
those with a second PE, li etime anticoagulation is warranted.
diagnosis you give her but appears to be taking it in stride.
For those with a cryptogenic PE or PE rom an acquired or
Question 3.5.7 What is the optimal management plan or inherited thrombophilia (e.g., Factor V Leiden), recommenda-
the patient moving orward? tions are all over the place rom 3 months to li e. Nine months
A) Bolus her with un ractionated heparin (UFH), start her on is likely adequate or a patient with a PE rom an irreversible
oral war arin, and discharge her to home. cause, although patients go back to their pre-treatment risk as
B) Administer a dose o enoxaparin in the ED, provide educa- soon as you stop anticoagulation. And, a rst PE trumps every-
tion, and discharge the patient to home with primary care thing else in terms or risk actors or a second PE—including
ollow-up in the next 2 to 3 days. all those ancy thrombophilia tests! So, nding that thrombo-
C) Start the patient on low–molecular-weight heparin (LMWH), philia does not necessarily help your decision-making process.
initiate oral war arin therapy, and admit the patient to the Note that some guidelines suggest anticoagulation or li e a er
amily medicine service. a rst cryptogenic/unprovoked PE i bene t seems to outweigh
D) Bolus her with UFH, initiate a UFH drip, and admit the risks. T is should be decided a er 3 months o anticoagulation.
patient to the amily medicine service. (Chest. 2012;141(2)(Suppl):7S–47S.)
E) Start her on oral war arin and discharge her to home with
primary care ollow-up in the next 2 to 3 days.
HELPFUL TIP:
Answer 3.5.7 The correct answer is “C.” Full anticoagulation a a– r s ss /
is considered mandatory or all patients with a PE. While dis- p r s s h rs b r -
charging a patient with only a DV is considered standard o s, s s, r h - - sh ,
care, this is not (yet) true or PE. T us, any plan that centers r sp r r s r ss s r (a Rd S), r b s ,r -
on a discharge to home is incorrect (a 2014 Cochrane Data- s s r s h , c o Pd h r ,
base review concluded that the available evidence is insuf cient br h ss h p r s h . t h s,
to recommend outpatient therapy and that more studies are a– r s sp r Pe. l k s ,
needed). With regard to selecting an anticoagulant, current r a– r r s r
evidence does not support the use o one agent over another; r Pe! i , p s h r
UFH, LMWH, and ondaparinux are all appropriate. Regard- s s , h Pio Ped s r
less o the selected anticoagulant, the current recommenda- h s r a– r p s
tions stipulate the initiation o war arin at the time o diagnosis h h Pe.
as well. T e UFH, LMWH, or ondaparinux should be contin-
ued until the patient’s INR has been therapeutic (2–3) or at
least 24 hours and at least 5 days. Note that apixaban, dabi- HELPFUL TIP:
gatran, and rivaroxaban can all be used to treat and prevent o 88% p s h Pe r h p , 70% h
PE. Like war arin, dabigatran requires 5 days o overlap with sp r h p , 65% h p r p , s
LMWH when used as treatment. Avoid edoxaban (Savaysa) i s 30% r h r .th p h r s h
possible. It can only be used in those with a CrCl o less than h h s sp h r h s sp
95 mL/min. And, it requires dose adjustment or those with a h k h ss r . t k h s rs , h
CrCl o between 50 and 15 mL/min (do not use under CrCl r p Bn P b p s h Pe.
15 mL/min). Note that the most recent available guideline rom
110 Fa mil y med ic in e exa min a t io n & Bo a Rd Review

• a ppr h PeRc r s w s r r h h
HELPFUL TIP: b s r Pe?
c pr ss s k s pr p s hr b
• i p r pr r Pe?
s r , h r rr s
• u rs h a– r ?
r d vt — r !e s -
. c s r pr s r b pr ss s k s r -
h s p s. t hr b s s pr d vt s QUICK QUIZ: ven a c a va l Fil t eRS
s pr p s hr b s r h -
b r r (n n t ) 7 r pr d vt s. t h s
Which statement best describes the use o vena caval lters in
s h b r h r (n n H) 1 22
preventing PE?
p sr q r b r s s , n n H 1 11
A) T ey reduce the risk o PE but only in patients who are
p Pe, nnH 1 5r q r
maintained on anticoagulation.
r. (Ja ma i r m . 2014;174(9):1494–1501.)
B) Retrievable vena caval lters have been shown to have a
short-term advantage while the patient is in the hospital;
they should be removed as soon as possible.
Your patient does well, completes her course o war arin, and C) T ey unequivocally reduce PE risk in both anticoagulated
has no urther episodes over the next 2 years. She develops and nonanticoagulated patients.
gallstones and plans to have an elective laparoscopic chole- D) One need not work-up a patient or a PE in the presence o a
cystectomy. A surgeon colleague sends her back to see you or vena caval lter since they are close to 100% e ective.
a preoperative evaluation. You nd no evidence o cardiac,
pulmonary, or hematologic disease. She is no longer on war- The correct answer is “B.” All o the rest are wrong. All evi-
arin and is doing well. dence suggests that vena caval lters are essentially useless in
preventing PE (e.g., JAMA. 2015:28;313(16):1627–1635). In
Question 3.5.9 Which o the ollowing postoperative man addition, they can break with strut migration to the heart or
agement strategies do you recommend? through the vena cava. 2016 ACCP guidelines recommend
A) Aspirin 81 mg PO daily. against vena caval lters in patients who can be anticoagulant
B) War arin 5 mg PO daily. even aster a recurrent PE. T ey may have some short-term ben-
C) UFH 5,000 units subcutaneously daily. e t or inpatients such as with trauma.
D) Enoxaparin 40 mg subcutaneously daily.
E) No antiplatelet or anticoagulant drugs. CASE 3.6
A 50-year-old male who is a heavy drinker with a history
Answer 3.5.9 The correct answer is “D.” Even or a relatively o squamous cell carcinoma o the neck presents to your
minor surgical procedure where anesthesia is used or 30 o ce complaining o abdominal pain. He has been cough-
minutes or less and the postoperative recovery is usually quick, ing and expectorating bloody sputum and notes a low grade
your patient is at moderate risk or venous thromboembolism. ever, chills, and mild dyspnea starting about 1 week ago.
Her history o PE puts her in a higher risk category, and she He denies nausea, emesis, and chest pain. His squamous cell
requires prophylaxis. O the choices available, enoxaparin would carcinoma was treated with external beam radiation several
be the most appropriate. LMWH and UFH are both acceptable years ago. Examination reveals an a ebrile male in mild dis-
or prevention o DV /PE in the postoperative period, but “C” tress. His vital signs are normal, and his lungs sound clear.
is wrong because UFH must be dosed every 8 to 12 hours rather T e abdominal examination reveals only mild epigastric
than daily. “A” is incorrect. Aspirin is sometimes used postoper- tenderness.
atively, but the dose should be 160 mg/day or greater. Also, com- T e chest x-ray is available or your review (see Fig. 3-5).
pared with heparin and its derivatives, aspirin is less ef cacious Your colleague, who is on call today, walks by and asks i you
in the prevention o thrombus. “B” is incorrect. War arin alone have any admissions or her.
is not appropriate in this setting due to its slow onset o action.
Question 3.6.1 You consider this 50 year old with a cough
HELPFUL TIP: and reply:
th p h h p s r q r pr - A) “Yes. T is gentleman will need the ICU.”
ph s r s hr b b s rs r r s B) “Yes. T is gentleman will need a respiratory isolation
k .ar s b r pr ph s room.”
h p s b s r h r p r . C) “No. I’m sending this gentleman home with metronidazole.”
D) “No. I’ll work-up this gentleman as an outpatient.”

Objectives: Did you learn to . . . Answer 3.6.1 The correct answer is “B.” Because he is expec-
• R z r sk rs r Pe? torating bloody sputum and has a cavitary lesion on chest x-ray
• u rs h r b s p s s s Pe? (right upper lobe), this patient should be admitted to a respiratory
Ch Apt Er 3 • Pu l mo n o l o g y 111

Chest C urther con rms a parenchymal abscess in the right


upper lobe with cavitation and air within the cavity. Bron-
choscopy reveals pus in the airway and extrinsic compression
o the bronchi. A lavage sample is obtained, but biopsies are
not taken due to the clinical impression that this is a lung
abscess.

Question 3.6.3 What organisms are most commonly


isolated in lung abscesses?
A) Anaerobic bacteria.
B) Aerobic bacteria.
C) uberculosis.
D) Mixed aerobic/anaerobic bacteria.

Answer 3.6.3 The correct answer is “A.” Anaerobes are isolated


most o en, ollowed by mixed anaerobic/aerobic bacteria, ol-
lowed by aerobic bacteria alone (especially staphylococci).

Gram stain o sputum demonstrates Gram-positive cocci and


Gram-negative rods. Cultures are pending. uberculin skin
FIGURE 3-5. P ’s h s -r . test is negative.

Question 3.6.4 What is the most appropriate therapy or


isolation room until tuberculosis is ruled out. He will need this patient?
urther evaluation and possibly intravenous antibiotic therapy, A) Re er or surgical drainage.
both o which may be accomplished during his hospitalization. B) Oral levo oxacin.
“A” is incorrect. T ere is no need to send this patient to the ICU C) Intravenous clindamycin.
based on his current picture. “C” is also incorrect. Metronida- D) Intravenous metronidazole.
zole alone is not an appropriate therapy or this patient even i E) Intravenous ce riaxone.
this is bacterial (see below).
Answer 3.6.4 The correct answer is “C.” Most lung abscesses
Question 3.6.2 What is the best next step in the diagnosis are polymicrobial, but the most important aspect in treatment
o this process? appears to be the use o an antibiotic active against anaerobes.
A) Bronchoscopy. Intravenous clindamycin is the usual choice or lung abscess
B) Sputum cultures. due to its coverage o anaerobes and Streptococcus pneumoniae.
C) Blood cultures. Metronidazole is less e ective, ailing in up to 50% o cases o
D) Chest C . putrid lung abscess. A beta-lactam with beta-lactamase inhibi-
E) Open-lung biopsy. tor (e.g., piperacillin/tazobactam) is another good choice.
Ce riaxone and levo oxacin o er poor coverage o anaerobes.
Answer 3.6.2 The correct answer is “D.” T e chest x-ray dem- Surgical drainage o lung abscesses is needed in only 5% to 10%
onstrates a cavitary lesion in the right upper lobe. Chest C is o cases. Most resolve with just antibiotics.
warranted or urther characterization o the lesion. From his-
tory, examination, and chest x-ray, it is not possible to deter- Objectives: Did you learn to…
mine whether the lesion is an abscess or a malignant process. • R z h pr s r s h s -r ?
An indolent course with low-grade ever is characteristic o • i h s s r s s?
lung abscess. However, the pre-existing squamous cell carci- • d h p p r sp r r s ?
noma has potential to have spread to the lungs, and squamous • m p h bs ss?
cell carcinoma is known to cause cavitations. Culture o sputum
and blood, including evaluation o rst morning sputum or
acid- ast bacilli (AFB), will be an essential part o the assess-
CASE 3.7
ment but may not yield as much in ormation as chest C , and A 53-year-old male is accompanied by his wi e to your o ce
sputum culture should be done in conjunction with cytology and complains o a cough or 6 weeks. It is worse at night and
and Gram stain. Bronchoscopy should be postponed until C any time he lies down. He denies sputum production, short-
results are available. Bronchoscopic biopsy is potentially detri- ness o breath, chest pain, and wheezing. He takes an antacid
mental i the lesion is an abscess since the airway could ood once or twice per day to settle his stomach and notes very bad
with pus i the entire cavity wall is penetrated. heartburn. He smoked three packs o cigarettes per day until
112 Fa mil y med ic in e exa min a t io n & Bo a Rd Review

1 year ago, when he quit “cold turkey.” He takes only hydro- Question 3.7.2 Which o the ollowing is your next step in
chlorothiazide or hypertension and a daily aspirin. He has managing this patient’s cough?
no cardiac disorders. His wi e reports that he snores at night, A) Start a proton pump inhibitor.
and she adds, “He’s always hacking and clearing his throat— B) Start an inhaled steroid.
all night.” T e review o systems is negative. In order to sleep C) Order 24-hour esophageal pH monitoring.
better, he has recently started having a shot (or 2 . . . or 3 . . .) o D) Obtain spirometry.
whiskey be ore going to bed. E) Obtain a chest C .

Question 3.7.1 What is the most likely cause or the cough? Answer 3.7.2 The correct answer is “A.” An empiric trial o an
A) Gastroesophageal re ux. e ective gastric acid-suppressing medication in this symptom-
B) Lung cancer. atic patient is likely to relieve the cough i the diagnosis is accu-
C) Antihypertensive medication. rate. T e ACCP recommends starting therapy with a proton
D) Alcohol abuse. pump inhibitor rather than an H 2-blocker. T e usual anti-re ux
E) Congestive heart ailure (CHF). measures, such as avoiding atty oods, alcohol, and ood be ore
bedtime, should be instituted as well. Prescribers must be aware
Answer 3.7.1 The correct answer is “A.” T is patient appears that sometimes a complete resolution o cough takes months. A
to have a chronic cough that is most likely due to gastroesoph- 24-hour pH monitor (“C”) is invasive and o en not necessary
ageal re ux disease (GERD). He takes antacids and exhibits i an empiric trial o gastric acid suppression resolves the prob-
throat clearing, which can be a subtle sign and is not typically lem. Starting the evaluation o chronic cough with a chest x-ray
identi ed by patients as re ux. In addition, he drinks alcohol is part o the ACCP recommendations, but C scan (“E”) is
at bedtime, urther predisposing to re ux. He has a history o not indicated with a negative chest x-ray. I the cough does not
smoking, which does place him at increased risk or develop- resolve with empiric therapy, spirometry should be considered.
ing a bronchogenic carcinoma, but a lung mass would not be a
common cause or cough. Aspirin is associated with broncho- He does not respond a er 2 months o empiric treatment, and
spasm in some people, but it would not usually present as cough he is becoming more concerned. T e examination is unchanged.
in a patient with no history o asthma. Hydrochlorothiazide is Spirometry is normal with a normal f ow volume loop.
not known to cause cough (although angiotensin-converting
enzyme [ACE] inhibitors are). Also, it is unlikely that symptoms Question 3.7.3 Which o the ollowing management options
would be isolated to nighttime i his cough were medication- is LEAST likely to bene t this patient?
related. A) Combination antihistamine and decongestant.
B) Inhaled corticosteroid.
C) Inhaled beta-2 agonist.
HELPFUL TIP:
D) Antibiotics.
R b r h a c e h b rs s h 5%
20% p s k h .F rp s h -
Answer 3.7.3 The correct answer is “D.” T is patient has no signs
p h r a c e h b r, br r
or symptoms o sinusitis or bacterial pulmonary in ection, so
h s s rk- p r hr
treating with an antibiotic is inappropriate and unlikely to help.
h. u s ,s p sr s h 1 kb
However, some orm o empiric therapy might be tried. He could
p rs s r 1 h. c h ace h b r
have postnasal drainage without signs on physical examination,
rs r p 6 hs rs r h ace
and empiric therapy with combination antihistamine and decon-
h b r.
gestant may improve the cough. Inhaled corticosteroids and beta-2
agonists are the mainstay o chronic asthma therapy and may help
relieve this patient’s chronic cough. T is patient could yet have
HELPFUL TIP: “cough-variant asthma” despite normal spirometry results.
as p r s s s rb s h-
.S , ’b sh s p r Question 3.7.4 The three most common causes o chronic
r . (a J R sp r c r c r m . 2009;180:809 n cough (cough lasting longer than 8 weeks) are:
e J m . 2009;360:1487.) A) Postnasal drip, asthma, GERD.
B) GERD, COPD, congenital lung disease.
C) Lung cancer, postnasal drip, COPD.
On physical examination, you note a mildly overweight male D) Obstructive sleep apnea, respiratory in ections, asthma.
in no distress. His vital signs are normal. His lungs are clear
to auscultation. T e nasal and oropharyngeal mucosae are Answer 3.7.4 The correct answer is “A.” Epidemiologic stud-
intact, moist, and not inf amed. T e remainder o the exami- ies have demonstrated that most cases o chronic cough are
nation is unremarkable. Chest x-ray shows f attened dia- due to postnasal drainage (o en termed “upper airway cough
phragms but is otherwise negative. You suspect GERD, but syndrome”), asthma, or symptomatic GERD. Most cases o
also entertain other diagnoses. chronic cough seem to have only a single cause, although some
Ch Apt Er 3 • Pu l mo n o l o g y 113

TABLE 3-4 SELECt ED COMMON CAUSES OF ACUt E CASE 3.8


AND Ch r ONIC COUGh
You see a 38-year-old emale in ollow-up or a recent epi-
a u Ri, p r ss s, r rh s, c o Pd rb ,
sh , s s s sode o sinusitis. T e illness has been present or about
6 weeks and has not responded to 2 weeks o appropriate
c hr g eRd , p s s r p, s h , hr s s s, antibiotics. She continues to have intermittent nosebleeds,
r / s r rh s, a c e h b rs, s ph
atigue, arthralgias, low grade evers, and night sweats.
br h s, hr br h s, p s s, s h
wo new complaints have sur aced: she has a cough pro-
u Ri, pp r r sp r r ; c o Pd , hr bs r p r ductive o white sputum and she occasionally expectorates
s s ; g eRd , s r s ph r s s ; a c e, s - quarter-sized clots o blood. She has pleuritic chest pain,
r z .
but denies dyspnea, tobacco use, and cardiac or pulmonary
disease.
will have more than one cause. Empiric therapy should be aimed She is a ebrile with a respiratory rate o 16, blood pressure
at these top three causes. O course, in ection (pertussis in par- 120/74 mm Hg, and pulse rate 92 bpm. Her oxygen saturation
ticular), malignancy, and other causes o cough are important to is 98%on room air. T ere is dried blood in the nares, but the
consider—and potentially rule out—as well. oropharynx is clear. Cardiac and pulmonary examinations
T e evaluation o chronic cough should proceed in a logi- are unremarkable.
cal manner. Usually, history and physical examination will nd
the cause. I this is unrevealing, consider a stepwise evaluation
addressing each o the chronic etiologies in able 3-4 in order. I Question 3.8.1 Which initial test is most appropriate?
this still does not give you an answer, consider a methacholine A) Chest x-ray.
challenge test to see i you can reproduce the symptoms that B) Sputum cytologic analysis.
would lead you to a presumptive diagnosis o asthma with nor- C) Bronchoscopy.
mal spirometry. D) Chest C .
E) CBC.

HELPFUL TIP:
l k h rs— r p s hr Answer 3.8.1 The correct answer is “A.” Hemoptysis is alarm-
h s r k .R b r s, ing to the patient and the physician—we hope. A stepwise
sp a c e h b rs ( s b ); h r r s h approach is warranted with chest x-ray as the rst step. Sputum
s h r , z p , s or cytology might help i the suspicion or lung cancer was sub-
sh s ( . ., b r , k s ). stantial, but the yield is likely to be low here. She may eventually
require bronchoscopy i suggested by initial studies. Chest C
is likely to be part o the evaluation, but a chest x-ray should be
Objectives: Did you learn to . . . per ormed rst. Obtaining blood or a CBC is also important,
• R z h s s s hr h? although likely to be normal in the setting o minor hemoptysis.
• e p h hr h?
• d p p r hr h? You obtain the chest x-ray pictured in Figure 3-6.

FIGURE 3-6. P ’s h s -r .
114 Fa mil y med ic in e exa min a t io n & Bo a Rd Review

You get the ollowing laboratory results back: Question 3.8.3 Which o the ollowing is NOT a radiographic
CBC: Leukocytosis, thrombocytosis and normochromic, nding o granulomatosis with polyangiitis?
normocytic anemia. A) Nodules that may be cavitary.
ESR: 70 mm/hr. B) Alveolar opaci cation.
Urine dipstick: Positive or protein, heme, and red C) Pleural opacities.
cells. D) Widened mediastinum.

Question 3.8.2 Which o the ollowing tests will best assist Answer 3.8.3 The correct answer is “D.” A widened medias-
you in the diagnosis o this patient? tinum is not one o the classic ndings in GPA. However, one
A) Antineutrophil cytoplasmic antibody (ANCA). may, on occasion, see hilar adenopathy. All o the other choices
B) Antiglomerular basement membrane antibody. can be ound in GPA. In the patient’s x-ray (Fig. 3-6), a right
C) Antinuclear antibody (ANA). upper lobe mass is easily distinguished. In a young, nonsmok-
D) A and B. ing emale presenting with these symptoms, such a lung mass
E) A and C. should lead to the consideration o GPA or possibly an in ec-
tious process. She is less likely to have a malignant process.
Answer 3.8.2 The correct answer is “D.” T is patient is pre-
senting with the classic triad o granulomatosis with polyangi- HELPFUL TIP:
itis (GPA), a disease o the upper respiratory tract, lower respi- a j r, pr b b h s s , h -
ratory tract, and kidneys. (T is disease was ormerly known as p s s s br h s. t h s s sp r s k rs
Wegener’s granulomatosis. As a response to Wegener’s associa- h r .
tion with the Nazi Party, pro essional bodies and journals have
replaced his name with a descriptive name.) She has some o
the additional signs and symptoms associated with GPA, as T e diagnostic evaluation is in progress. Laboratory tests are
well. Common ndings include: pleuritic chest pain, myalgias, pending, and a chest C is scheduled. You have arranged or
arthralgias, ptosis, ever, weight loss, and purpuric skin lesions, a pulmonologist to see her. When you are on call, the phy-
among others. sician covering the ED calls you to admit her or “massive
ANCA, and especially c-ANCA that is more speci c or hemoptysis.” When you arrive, the patient looks com ort-
GPA, is present in up to 90% o patients with GPA. An ANA able and has normal vital signs. She begins a t o coughing,
is not help ul in diagnosing GPA. An antiglomerular basement expectorating several ounces o bright red blood. Her systolic
membrane antibody (anti-GBM) may be help ul in diagnos- blood pressure alls to 80 mm Hg. Her respiratory rate is 40.
ing Goodpasture syndrome (named or an American patholo- Her work o breathing has increased considerably. T e situa-
gist—not a Nazi—so he gets to keep the eponym). Goodpasture tion does not improve a er 5 minutes o observation, and her
syndrome can be clinically easily con used with GPA; they both O2 saturation is now 83%.
present with respiratory and renal involvement. T us, anti-
GBM antibody will be help ul in di erentiating these two (but Question 3.8.4 Remembering the movie Moulin Rouge
about 10% o patients with Goodpasture syndrome will also (which has nothing to do with this case except or hemop
have GPA—just to add to the con usion). For a partial list o tysis), what is your rst action in this situation?
causes o hemoptysis, see able 3-5. A) Arrange emergent bronchoscopy.
B) rans use 2 units o blood.
C) Per orm endotracheal intubation.
D) Provide bolus IV normal saline.
TABLE 3-5 CAUSES OF h EMOpt YSIS E) ell her, “T e show must go on!”
v s r Pe, s s (g p s r s r ,
Answer 3.8.4 The correct answer is “C.” Massive hemopty-
r ss hp s),
r r s r sis is variably de ned as 100 to 600 mL o blood expectorated
per day, and it can result in hemodynamic compromise and
n p s Br h r , s s s asphyxiation. Quanti cation o the blood loss by the patient is
c ss l p s, rh r hr s usually unreliable. T e main cause o mortality with hemopty-
sis is not hypovolemia but rather asphyxiation rom blood in
c r c HF, r s ss the lungs. As with any patient in acute respiratory distress, the
i s t B, br h s, p , bs ss airway must be controlled rst. T e best choice here is to per-
orm intubation. Since this patient is known to have a poten-
dr s a s, ,s s tial source or bleeding in the right lung, intubation o the le
ms s tr , r b , ps s, h ss mainstem bronchus may protect the le lung rom the blood.
Also, placing this patient on her right side (so that the bleed-
Pe, p r b s ; c HF, s h r r . ing source is dependent) may protect the le lung. I available,
Ch Apt Er 3 • Pu l mo n o l o g y 115

emergent bronchoscopy may allow identi cation o the bleed- C) Bronchogenic carcinoma.
ing site and selective lung intubation. However, bronchoscopy D) Pneumonia.
is not well suited or stopping the hemorrhage. T e most that E) Microscopic polyangiitis.
a bronchoscopist can do is place an endobronchial blocker and
seal o the bleeding lobe. Interventional radiology should usu-
ally be the rst treatment once the bleeding site has been local- Answer 3.9.1 The correct answer is “B.” T e ndings o hilar
ized. Emergent surgery is indicated i the bleeding remains brisk lymphadenopathy and a restrictive pattern on spirometry are
and not responsive to other interventions. Fluid resuscitation most consistent with sarcoidosis. T e chest x-ray ndings do
is important. However, be ore any o these other measures is not support the diagnosis o granulomatosis with polyangiitis.
undertaken, the airway must be protected. Besides, it can’t be “A”—we just did that case (and when would
you see two GPA cases in a row on a test—let alone in your
career?). “E” is incorrect. Microscopic polyangiitis is a systemic
HELPFUL TIP: vasculitis related to GPA, presenting with similar eatures as
i s p r rs h p s hh p- GPA but without granulomatous disease. Bronchogenic carci-
ss s b r h r r noma is unlikely in this relatively young nonsmoker who does
h r b — h h ss h r p s . not demonstrate ndings o carcinoma on x-ray. T e clinical
th b s sph s history is not typical o pneumonia, and chest x-ray shows no
s rpr s h r r h s h gi in ltrate. uberculosis ( B), although not an answer option,
b rs s . a s , s r r should also be considered, and the appropriate history and test-
b r h h h b . ing should be completed. In act, B and sarcoidosis o en pres-
th r r , s r h b s k s s ent in a similar ashion.
h p ssp s.
Question 3.9.2 Which o the ollowing is NOT commonly
associated with sarcoidosis?
A) Hypercalcemia.
T e patient stabilizes in the ICU. You plan to start treatment B) Elevated ACE levels.
or her granulomatosis with polyangiitis. She does better and C) Reduced di usion capacity.
is discharged in 2 days. D) Hypothyroidism.
E) Facial or peripheral nerve palsy.
HELPFUL TIP:
t h 5- r r r r ss h Answer 3.9.2 The correct answer is “D.” Sarcoidosis is marked
p s s 90%. t h s p s r ss by the presence o noncaseating granulomas. While sarcoid can
r .c ph sph + s r ss b in ltrate the thyroid, it rarely, i ever, causes hypothyroidism.
h b s b . Pulmonary sarcoidosis includes a decreased di usion capacity
and decreased vital capacity. Other laboratory ndings include
hypercalcemia, hypercalciuria, elevated liver and pancreatic
Objectives: Did you learn to . . . enzymes, and elevated ACE levels. Neurologic involvement
• P r r ppr pr p h occurs in up to 5% o patients and requently presents as acial
h p s s? paralysis but may present as any central nervous system lesion.
• R z h j r s s h p s s? Peripheral nerves may also be involved.
• d s r ss hp s?
• i r ss h p s s? Question 3.9.3 Which o the ollowing is NOT ound as a
part o sarcoidosis?
A) Erythema nodosum.
CASE 3.9 B) Myocardial in arction.
A 35-year-old A rican-American emale presents with dys- C) Cardiac arrhythmias.
pnea worsening over the last 2 months. She also complains D) Elevated liver enzymes.
o cough, generalized atigue, and intermittent low-grade E) Vision loss.
evers. She does not smoke. Chest x-ray shows hilar adenopa-
thy and small bilateral pleural e usions. Spirometry is con- Answer 3.9.3 The correct answer is “B.” Sarcoidosis does not
sistent with a restrictive pattern. cause myocardial in arctions. While there is cardiac involve-
ment with sarcoidosis, the mani estations are bundle branch
Question 3.9.1 O the ollowing, which is the most likely block, cardiac arrhythmias, and sudden death. Many organs can
diagnosis? be a ected by sarcoidosis, including the skin, eye (iritis), heart,
A) Granulomatosis with polyangiitis. lung, liver, nervous system—essentially anywhere granulomas
B) Sarcoidosis. orm.
116 Fa mil y med ic in e exa min a t io n & Bo a Rd Review

Question 3.9.4 Which o the ollowing is true about ACE is dullness to percussion and decreased tactile remitus over
levels in sarcoidosis? the right lower lung eld.
A) An elevated ACE level is speci c or sarcoidosis.
B) ACE levels o en correlate with disease severity in sarcoidosis. Question 3.10.1 Based on this patient’s history and physi
C) ACE inhibitors are e ective in the treatment o sarcoidosis. cal examination, what do you expect to nd on chest x ray?
D) All o the above. A) Normal chest x-ray.
B) Cavitary lung lesion.
Answer 3.9.4 The correct answer is “B.” One can ollow ACE C) Pleural e usion.
levels to track the progress o the disease. However, since treat- D) Expanded lung elds.
ment is based on symptoms, ollowing ACE levels is not recom- E) Pneumothorax.
mended. “A” is incorrect. ACE levels may be elevated in silicosis,
miliary B, and asbestosis, among others. “C” is incorrect. ACE Answer 3.10.1 The correct answer is “C.” T is patient’s nd-
inhibitors are not used to treat sarcoidosis. ings suggest pleural e usion. Everything is diminished in pleu-
ral e usion: there is dullness to percussion, decreased breath
T is patient is ound to have only pulmonary sarcoidosis sounds, decreased tactile remitus, and decreased voice trans-
with some mild systemic symptoms. mission. A cavitary lung lesion presents with either a normal
examination or ndings similar to an in ltrate (e.g., crackles,
Question 3.9.5 Which o the ollowing is the best initial increased remitus, and dullness to percussion). Expanded lung
choice or management? elds on chest x-ray are o en seen in patients with COPD or
A) Observation. asthma, and examination ndings include prolonged expiratory
B) Oral corticosteroids. phase, wheezing, and resonance to percussion. Pneumothorax
C) Oral antibiotics. presents with hyperresonance to percussion, decreased breath
D) Inhaled corticosteroids. sounds, and decreased remitus.
E) Methotrexate.
HELPFUL TIP:
Answer 3.9.5 The correct answer is “A.” T is patient has ch s r r phs h s s r rb r -
apparent pulmonary-limited disease and has minimal sys- r s. H r, h s s r pr b . t h pr s-
temic symptoms. Nearly 50% o patients with sarcoidosis may r bs rb r r s r r .
have spontaneous resolution o their symptoms without treat- wh r r s s h h r r h r s -
ment. In act, treatment may actually prolong the disease pro- h r h rb r r ,s h s p r
cess. I her pulmonary or systemic symptoms worsen or are s rh h r .
causing major li e problems, she should be started on oral
steroids. Systemic corticosteroid therapy is the mainstay o
treatment or sarcoidosis. Methotrexate and other immune- Your suspicions are con rmed. T e chest x-ray shows oblit-
modulating drugs may be employed as well and o er a steroid- eration o the right hemidiaphragm, and the posterior costo-
sparing e ect, but these are not rst-line agents. Evidence or phrenic angle is obscured on the lateral view, consistent with
the use o inhaled corticosteroids is lacking. Antibiotics are pleural e usion. T ere is also a right upper lobe lung mass.
not e ective.
Objectives: Did you learn to . . . Question 3.10.2 Which o the ollowing will provide the
• R z h s s s r s s? most in ormation and guidance or your thoracentesis?
• m p h s r s s? A) Supine chest x-ray.
B) Chest C .
C) Lateral decubitus chest x-ray.
CASE 3.10 D) Chest ultrasound.
A 57-year-old male with no prior medical history comes in E) Apical view chest radiograph.
to clinic with a 1-week history o right rib pain and low-back
pain. T e rib pain is worse with deep breaths and especially Answer 3.10.2 The correct answer is “D.” Prior to per orming
bothers him at night. T ere has been no trauma. He has a thoracentesis, you must know whether the e usion is locu-
lost 20 lb in the last 3 months. He has a cough productive o lated or reely owing. Portable ultrasound has become a vali-
white sputum. He denies any other symptoms. He smokes dated and widely accepted modality to diagnose and access a
one to two packs o cigarettes per day but does not drink pleural e usion. Also, ultrasonography has been ound to be
alcohol. more sensitive or detection o pleural uid than a chest radio-
Vital signs: temperature 36.5°C, pulse rate 95 bpm, blood graph. Chest C is somewhat more sensitive but ar more cum-
pressure 110/70 mm Hg, respiratory rate 16. On room air, his bersome and does not allow a bedside diagnosis and treatment.
oxygen saturation is 96%. T ere is no adenopathy. His lung A decubitus lm, with the a ected side down, would allow you
sounds are clear on the le and decreased on the right. T ere see the e usion “layer out” unless it is loculated but again is less
Ch Apt Er 3 • Pu l mo n o l o g y 117

sensitive. A supine chest x-ray may cause the e usion to “layer TABLE 3-6 CAt EGOr IZAt ION OF pLEUr AL FLUID AS
out” too, but you will not be able to see it as well, which is why AN EXUDAt E Or t r ANSUDAt E
e usions may be missed when a patient is unable to stand or sit
Exuda e h r rz b
upright or his x-ray.
• P r s r pr r > 0.5
Question 3.10.3 Relative and absolute contraindications to • P r s r l dHr > 0.6
• P r ldH r r h 150 / l ( -hr s h pp r
thoracentesis include all o the ollowing EXCEPT: r s r l d H)
A) Herpes zoster in the area o needle placement.
B) Coagulopathy.
C) Diaphragmatic rupture.
D) Positive pressure ventilation. Answer 3.10.5 The correct answer is “D.” Pleural e usions
E) History o recurrent laryngeal nerve injury or compromise. are broadly categorized as exudates and transudates (see
ables 3-6 and 3-7). Such a categorization helps to narrow the
di erential diagnosis. In this case, several elements o the pleu-
Answer 3.10.3 The correct answer is “E.” Absolute contrain-
ral uid are consistent with an exudate. LDH and protein can
dications include chest wall compromise (e.g., burn, cellulitis,
be used to determine whether the pleural uid is transudative
herpes zoster, ruptured diaphragm) and cases where chest tube
or exudative. Per Light’s criteria, a pleural e usion is suggestive
thoracostomy would be more appropriate. Relative contraindi-
o an exudative process i two o any o the ollowing criteria are
cations are poor patient cooperation, coagulopathy, anticoagu-
met: pleural uid LDH > 2/3 the upper limit o normal serum
lation therapy, very small e usions (< 10 mm on decubitus lm
LDH, a pleural LDH:serum LDH ratio o > 0.6, and a pleural
view), positive pressure ventilation, and pleural adhesions.
protein:serum protein ratio o > 0.5. All three o these indicators
point to an exudate in this case. Also, exudative e usions tend to
Question 3.10.4 On ultrasound, the ef usion appears ree have a higher degree o cellularity than transudative e usions.
owing and not loculated. What is the most appropriate With the in ormation given, it is dif cult to determine i the e u-
next step? sion is related to in ection, cancer, or some other process.
A) Re erral or surgical drainage.
B) Place a chest tube to drain the e usion. T e pleural f uid cytology comes back negative. T e patient’s
C) Per orm an ultrasound-guided diagnostic thoracentesis at symptoms and examination have not changed. Repeat radio-
the bedside. graph still shows an upper lobe mass.
D) Order two pizzas, one or you and one or the patient (you
have both had a long day and are hungry). Question 3.10.6 What is the most appropriate next step in
approaching this pleural ef usion?
Answer 3.10.4 The correct answer is “C.” T e patient has a A) Await pleural uid culture results.
relatively large pleural e usion. Ultrasound-guided thoracen- B) Per orm bedside chest tube drainage o the e usion.
tesis is a good rst step in evaluating this e usion. Ultrasound C) Re er or surgical evacuation o the e usion.
guidance is quickly becoming the standard o care as it has D) Re er or bronchoscopy.
been shown to decrease complications o pneumothorax as well E) Place a chest tube or chemical pleurodesis.
as the number o unsuccess ul clinical attempts or “dry taps.”
Re erral to a thoracic surgeon may eventually be necessary, but
this would not be the rst step. Placing a chest tube into an e u- TABLE 3-7 CAt EGOr IZAt ION OF pLEUr AL EFFUSIONS
sion is not recommended at this point, and the diagnostic study BY CLASS (t r ANSUDAt IVEVEr SUS EXUDAt IVE)a
should be obtained rst.
Type o Ef usion Potential Causes
Ultrasound-guided thoracentesis is success ul in obtaining tr s s s H r r , rrh s s, phr
f uid. T e f uid is amber and cloudy, with a pH 7.3, lactate s r , s s, ,
p r b s , r h r
dehydrogenase (LDH) 800 IU/L, glucose 65 mg/dL, total pro-
tein 5.5 g/dL, WBC 1,300/mm3, RBC 50,000/mm3. Serum stud- e s s Br h r , s
ies done the same day include LDH 155 IU/L, glucose 99 mg/dL, p s , s h ,p ,
and total protein 7.0 g/dL. Cytology, Gram stain, and culture t B, h h r ,p r s, s ph
r p r , s r s s s
o the pleural f uid are pending.
(rh r hr s, Sj r s r ),
r , r s( r r ,
Question 3.10.5 Which o the ollowing is the most accurate r , h r ), h r
statement regarding the pleural uid analysis? r h r h r p ,p r
b s
A) T e uid is due to in ection.
B) T e uid is due to cancer. Note: H r r p r b s s
C) T e uid is a transudate. r s s s.
D) T e uid is an exudate. a
S exudate r r t b 3-5. e r h s s r s .
118 Fa mil y med ic in e exa min a t io n & Bo a Rd Review

Answer 3.10.6 The correct answer is “D.” T e e usion is clearly


exudative, and the patient appears to have a lung mass. Biopsy CASE 3.11
o the lung mass via bronchoscopy is indicated. A negative pleu- A 60-year-old male presents to the ED or a cough. His symp-
ral uid cytology does not rule out lung cancer. Positive cytol- toms began with a cold 2 weeks ago, and the other symp-
ogy indicates advanced stage lung cancer. Chest tube drainage toms have improved, but the cough has persisted. He has
o a pleural e usion is not recommended except under extraor- mild production o white sputum with no hemoptysis. T e
dinary circumstances. Intermittent thoracentesis is pre erred patient denies evers, night sweats, chills, and weight loss.
and has lower morbidity. Surgical evacuation o the uid would He’s had no chest pain or dyspnea. He smokes one pack o
be indicated i the patient were symptomatic, the e usion was cigarettes per day, works in construction, and does not have
loculated and/or related to in ection. I the e usion grows, or is a regular doctor. In act, with some pride, he says, “I haven’t
drained and recurs, it may respond to pleurodesis. Otherwise, seen a doctor in over 30 years.” On physical examination, you
pleurodesis is not indicated at this time. nd a t-appearing male in no acute distress. His vital signs
are normal. His lung sounds are diminished bilaterally, but
HELPFUL TIP: the remainder o the examination is unremarkable. While
P s h p r s s r k breathing ambient air, the patient’s oxygen saturation is 94%.
b r s r r s h r s z You obtain a chest x-ray, which is shown in Figure 3-7.
s r s b b h sp .l - r s
h p r s r b k. Question 3.11.1 Your next step is to:
A) Prescribe a 5-day course o azithromycin.
B) Re er the patient to a pulmonologist.
Now that you have gained expertise with ultrasound o a C) Order a high-resolution C scan o the chest.
pleural e usion, your colleague sends you a patient that has D) Have the patient return to you in 3 months to repeat a chest
a pleural e usion on chest x-ray. Your colleague asks you x-ray.
whether you could “tap” the f uid or him as he does not eel E) Reassure the patient and have him return as needed.
com ortable with the portable ultrasound. Also, he has a tee
time in 30 minutes. Answer 3.11.1 The correct answer is “C.” T e chest x-ray in
Figure 3-7 shows a single nodule in the right lower lobe. T e
Question 3.10.7 On ultrasound, you quickly visualize the nodule is round, less dense than bone, and appears to be > 1 cm
chest and see a septated and loculated pleural ef usion. in diameter. T ese are sometimes called “coin lesions.” T ere are
What is the best next step? no other abnormalities. T e most appropriate next step in the
A) Return the patient back to his doctor—the e usion is too evaluation is to order a high-resolution C scan o the chest.
small to access. reatment with azithromycin is inappropriate in this setting, as
B) Per orm ultrasound-guided needle thoracentesis. this patient has no signs o pulmonary in ection on examination
C) Re er to a thoracic surgeon. or chest x-ray. Re erral to a pulmonologist is premature with-
D) Place a small-bore chest tube. out rst investigating the nodule by C scan. Delaying urther
E) Head to the gol course with your colleague. Nothing you
can do here.

Answer 3.10.7 The correct answer is “C.” A loculated and sep-


tated pleural e usion can very o en be seen in empyema and
evacuation usually requires surgical intervention. T oracentesis
would unlikely be success ul and would expose the patient to
an unnecessary procedure a er which he would still need to see
a surgeon. Placing a chest tube blindly into a loculated pleural
e usion is unsa e. T at procedure should be done under visual-
ization; most commonly, video-assisted thoracic surgery would
be utilized.
Objectives: Did you learn to . . .
• R z h hs r ph s f s
p r s ?
• l s p s p r s ?
• n rr h r ssb s p r
f s?
• d h p r r s h r p
h r s s?
• d h p r r h s b r ? FIGURE 3-7. P ’s h s -r .
Ch Apt Er 3 • Pu l mo n o l o g y 119

TABLE 3-8 r ECOMMENDAt IONS FOr FOLLOW-Up AND MANAGEMENt


OF NODULES SMALLEr t h AN 8 mm DEt ECt ED INCIDENt ALLY At
NONSCr EENING Ct
Nodule
Size (mm) Low Risk Patients High Risk Patients
4 n - p F - p 12 hs. i h ,
rh r

> 4–6 F - p 12 hs. i h , i - p c t 6–12 hs


rh r h 18–24 hs h

> 6–8 i - p c t 6–12 hs i - p c t 3–6 hs


h 18–24 hs h h 9–12 24 hs h

>8 F - p cts r 3, 9, 24 S s r -r sk p s
hs. d r s h
c t , Pet , / r b ps

Notes: t h s s pp r s p rs s 35 rs r r.
l -r sk p sh r bs hs r s k h rk r sk rs. H h-r sk
p sh hs r s k r h rk r sk rs.

imaging and evaluation is also inappropriate since 15% to 75% I the diameter is < 8 mm, the risk o cancer is low. When the
o solitary pulmonary nodules (SPNs) ≥ 8 mm are ultimately diameter is ≥ 3 cm, the SPN is now re erred to as a “pulmonary
diagnosed as cancer. mass” and is highly likely to be cancerous. SPN > 3 cm in diam-
eter should be considered cancer until proven otherwise.
Question 3.11.2 Which o the ollowing is NOT considered a T e Fleischner Society has published a set o widely accepted
benign pattern o calci cation on CT scan? recommendations regarding ollow-up o single pulmonary
A) Di use, homogeneous calci cation. nodules noted on C chest that are < 8 mm in diameter (see
B) Central calci cation. able 3-8).
C) Laminar calci cation.
D) Spiculated, irregular calci cation.
E) “Popcorn” calci cation. HELPFUL TIP:
th r s h s (b s ) s r -
Answer 3.11.2 The correct answer is “D.” We are accustomed s r SPn s. S c h s . 2013 m r; 143:840
to thinking o calci ed nodules as being benign, but that is not h p r . P r s b-s s
always the case. Irregular, spiculated calci cation is not reas- <5 - p s h r k b
suring. T e other options are considered indicative o a benign (< 2% r sk s k r). t h r s -
lesion. wo patterns on C are relatively speci c or cancer: a r h p r h r sk h r :
scalloped border and the corona radiata sign, which is com- h p://r r . s p . / r/s r -p -
posed o ne linear strands extending out rom the nodule. r - -r sk. H s ss h rp .
Question 3.11.3 All o the ollowing are use ul to help assess
the risk o cancer in a patient with an SPN EXCEPT:
A) Smoking status. Later that week, your patient returns with his C scan in
B) Age. hand. His cough is somewhat better (therapeutic C scan
C) Diameter o the nodule. radiation therapy . . . you know, like those C scans in Cali-
D) Gender. ornia that were cooking people’s brains accidentally?). You
review the C scan with him. It shows a round, smooth nod-
Answer 3.11.3 The correct answer is “D.” Determining the ule measuring 2 cm in diameter and located in the periphery
probability o cancer in patients with an SPN is an inexact sci- o the right lower lobe. T ere are no calci cations in the nod-
ence. T e risk o cancer is generally assessed as low, interme- ule and no other abnormalities.
diate, or high based on patient and radiograph characteristics.
Although men are slightly overrepresented in lung cancer diag- Question 3.11.4 Which o the ollowing is the most appro
noses, this is generally thought to be due to greater smoking priate next step?
rates in men and to occupational hazards. Gender itsel does not A) Re erral or bronchoscopy.
help to risk-strati y patients with an SPN. Smoking increases the B) High-resolution C scan every 3 months.
risk o an SPN being cancer, with greater use increasing the risk C) Chest x-ray every 3 months.
o cancer. As with most cancers, increasing age is associated D) Bone scan.
with a higher risk. T e diameter o the nodule is also important. E) Re erral to a thoracic surgeon.
120 Fa mil y med ic in e exa min a t io n & Bo a Rd Review

Answer 3.11.4 The correct answer is “E.” T is patient needs a C) “Here have him spit in this cup, and I’ll send his sputum or
biopsy. T ere are several actors that put your patient at higher cytology.”
risk o having a malignant cause or the SPN, including his D) “Nah, we don’t recommend lung cancer screening at this
age and tobacco use. T ese put him into an intermediate- to time.”
high-risk category or cancer. Although the nodule is smooth
on C , its size is > 8 mm and there are no calci cations. T is Answer 3.11.6 The correct answer is “B.” In 2013, the USP F
patient should be re erred or transthoracic ne-needle biopsy issued a grade B recommendation or annual lung cancer
or open biopsy. “A” is tempting but incorrect. Bronchoscopy is screening with low-dose C scan in adults aged 55 to 80 years
insensitive in the peripheral lung, especially when the lesion who have a 30 pack-year smoking history and currently smoke
is relatively small. “B” and “C” are also wrong here but are or have quit within the past 15 years. Screening should be dis-
appropriate in other settings. In this case, repeat imaging over continued once a person has not smoked or 15 years or devel-
time may delay a diagnosis o malignancy. Without symptoms ops a health problem that substantially limits li e expectancy or
o bone pain or con rmation that the SPN is a cancer that the ability or willingness to have curative lung surgery. T ese
might metastasize to bone, a bone scan will have a very low recommendations are based on the National Lung Screening
yield. rial that showed that or every 1,000 people who got screening
or lung cancer, 3 ewer died o lung cancer because o screen-
Your patient returns rom the surgeon much relieved. Fine- ing. About 300 people need to be screened to save one li e rom
needle biopsy proved the SPN to be a hamartoma. Now your lung cancer. Patients should be given the risks/bene ts o lung
patient wants to quit smoking or good, and he thinks that he cancer screening since approximately 365/1,000 patients will
will need some assistance. You recommend nicotine replace- have a alse-positive result (usually because o a benign pul-
ment products and bupropion, but your patient claims to monary nodule) that leads to more C scans (extra radiation),
have had an allergic reaction to bupropion. Fortunately, you more invasive procedures (biopsies and surgeries), and patient
know o an e ective alternative (unless he is a homicidal stress and anxiety. Con used yet? Don’t worry, the Ameri-
maniac . . . it happens you know). can T oracic Society has developed a patient decision guide
ound at http://www.thoracic.org/patients/patient-resources/
Question 3.11.5 To assist with tobacco cessation, you pre resources/decision-aid-lcs.pd . Your patient’s brother can read
scribe which o the ollowing? the 20 pages while on his next smoke break! Just kidding. It
A) Varenicline. is important to stress that this lung cancer screening should
B) Fluoxetine. not take the place o e orts to stop smoking! O note, “A” and
C) Olanzapine. “C” are incorrect because neither plain chest radiographs nor
D) Metoprolol. sputum cytology have been shown to improve lung cancer
E) Clonidine. mortality.
Answer 3.11.5 The correct answer is “A.” Randomized trials Objectives: Did you learn to . . .
have demonstrated the e ectiveness o the nicotine partial ago- • w h r sk rs h SPn ?
nist, varenicline (Chantix M). T is FDA-approved medication • d p p r SPn ?
appears to be at least as e ective as bupropion as an aid to smok- • a ss s p hs k ss ?
ing cessation. Fluoxetine and other selective serotonin reuptake • l r h r sks/b f s rs r h
inhibitors have not demonstrated a bene t. In schizophrenic ppr pr p p p ?
patients, the use o atypical antipsychotic medications may aid
in smoking cessation when compared with typical antipsychot-
ics. Clonidine is sometimes used to help patients who are with-
CASE 3.12
drawing rom narcotics, and it may have some limited role in A 74-year-old male presents to your ED or weakness, cough,
smoking cessation but is not very e ective. Depression, very and atigue. His wi e relates an incomplete recovery since
vivid dreams or nightmares, aggressive behavior, etc., are a well- his myocardial in arction last year. He continues to have
described side e ect o varenicline, and your patients should be poor appetite and listlessness, and she thinks that he may be
in ormed o that prior to prescribing the drug. depressed. He is short o breath and con used. His wi e says
that yesterday he developed a ever, chills, and a new cough
As you are walking out o your patient’s room, your patient productive o white sputum. His past medical history is oth-
says, “Hey Doc, my brother smokes too, is there a test you can erwise remarkable or a cholecystectomy. He is taking aspi-
do to make sure he doesn’t have lung cancer?” rin, metoprolol, and atorvastatin.
Vitals: temperature 39°C, respiratory rate 30, pulse 90
Question 3.11.6 You respond: bpm, blood pressure 140/80 mm Hg. Oxygen saturation on
A) “Sure, I’ll put in an order or him to get an outpatient chest room air is 90%. He is thin, pale, and oriented to person only.
x-ray.” T e lung examination is remarkable or rales in the le lower
B) “ ell him to make an appointment and we will discuss the eld, with dullness to percussion and increased tactile remi-
risks/bene ts o low-dose C scan.” tus. T e remainder o the examination is normal.
Ch Apt Er 3 • Pu l mo n o l o g y 121

T e chest x-ray shows a le lower lobe in ltrate. Other Question 3.12.2 Based on patient speci c characteristics
laboratory data currently available: hemoglobin 12.4 g/dL, and your knowledge o causative actors involved in pneu
WBC 14,100/mm3, platelets 340,000/mm3, creatinine 1.9 mg/ monia, which o the ollowing is LEAST likely to be the
dL, BUN 50 mg/dL, and normal electrolytes, troponin, and agent causing this patient’s in ection?
CK. An ECG shows normal sinus rhythm. A) Mycoplasma pneumoniae.
B) S. pneumoniae.
Question 3.12.1 What is your next step in managing this C) Haemophilus in uenzae.
patient’s medical condition? D) Pseudomonas aeruginosa.
A) Place a chest tube on the le .
B) Per orm chest C . Answer 3.12.2 The correct answer is “D.” When a pathogen is
C) Administer inhaled bronchodilators. identi ed in adult community-acquired pneumonia, it is usually S.
D) Administer parenteral antibiotics. pneumoniae. In act, S. pneumoniae makes up 40% to 60%o all cases
E) Per orm intubation and mechanical ventilation. o community-acquired pneumonia in the elderly. Nontypeable
H. in uenzae composes about 5% to 10% o cases. Mycoplasma is
Answer 3.12.1 The correct answer is “D.” Given the clinical implicated in 5% o all cases o pneumonia in adults, and it is more
picture and chest x-ray ndings, the patient most likely has common in young adults. P. aeruginosa pneumonia is uncommon
community-acquired pneumonia. T ere ore, the administra- in healthy elders and more likely to occur in patients with serious
tion o parenteral antibiotics is the best choice. “A” is incorrect. underlying lung disease or immunode ciency. Approximately 5%
Since there is no e usion, a chest tube would be useless. “B” o patients or more are in ected with multiple agents.
is incorrect. C is not required in this straight orward case o
pneumonia. “C” is incorrect. T e patient is not wheezing and Question 3.12.3 On the basis o your assessment o his risk,
there is no indication or bronchodilators at this time. As or you decide to admit this patient to the hospital. An IV is in
“E,” since your patient’s respiratory status is stable, he does not place. Which o the ollowing IV antibiotic regimens do you
require intubation. choose?
A) Penicillin.
HELPFUL TIP: B) Azithromycin.
R b r h s p ss b “ p ” rs s C) Penicillin and gentamicin.
“ p ”p b r r ph. d b s D) Azithromycin and ce riaxone.
r h r p r - q r p E) Piperacillin/tazobactam and cipro oxacin.
h r r ph pp r .a p r s s
s b r s p p s Answer 3.12.3 The correct answer is “D.” T e 2007 In ec-
pp r s .n h r s k tious Disease Society o America / American T oracic Society
r r ph pp r . (IDSA/A S) guideline or pneumonia recommends that or
community-acquired pneumonia treated in the hospital set-
ting the optimal antibiotic regimen must o er good coverage
HELPFUL TIP: o S. pneumoniae, H. in uenzae, and atypical organisms such
th p s r s p p r as Mycoplasma and Chlamydia species. Most S. pneumoniae
s r h h r r p h bacteria are resistant to penicillin and about 20% to 30% are
p .H r, s s b rs h b resistant to macrolides such as azithromycin. T ere ore, these
r b r .l k p (h p:// r . agents should not be used alone in the treatment o pneumo-
s . /p r / p/10675. ). nia in hospitalized patients. Gentamicin has no activity against
S. pneumoniae but has a role in P. aeruginosa in ections. Ce ri-
axone o ers good Gram-negative coverage and activity against
HELPFUL TIP: S. pneumoniae. Azithromycin covers atypical organisms. For
a s p rb ss - ss r r- these reasons, “D” is the best choice. An alternative regimen
sp s p hp —h would be monotherapy with a respiratory uoroquinolone, such
“c u RB-65.” c h , , h? H r r h r b s: as moxi oxacin or levo oxacin. T e combination o piperacil-
C s
(b s sp s r s r - lin/tazobactam with cipro oxacin is reserved or patients with
p rs , p , r ). more severe pneumonia, requiring ventilation and ICU care.
Ur (Bu n ) > 20 / l.
R sp r r r > 30 br hs/ . Initial blood cultures grow S. pneumoniae. Sputum Gram
B pr ss r < 90/60. stain and culture are negative. T e patient initially does well
a > 65 rs. and de ervesces a er 2 days o IV antibiotics. However, on
P s h s r 0 r1h r sk day 3, he again spikes a ever. He looks moderately ill. Your
h r r s r p - examination reveals increased dullness to percussion on the
. le . T ere is no jugular venous distention (JVD) or periph-
eral edema. T e radiograph is shown in Figure 3-8.
122 Fa mil y med ic in e exa min a t io n & Bo a Rd Review

FIGURE 3-8. P ’s h s -r .

Question 3.12.4 The most likely diagnosis at this point is: D) Prescribe amoxicillin/clavulanate.
A) Anaerobic abscess. E) Re er or bronchoscopy.
B) Development o resistant S. pneumoniae.
C) Parapneumonic e usion. Answer 3.12.5 The correct answer is “C.” T ere are no clear
D) ransudate secondary to heart ailure. guidelines regarding ollow-up chest x-ray in patients who had
E) Drug-induced transudate. pneumonia. T e British T oracic Society published recommen-
dations in 2007 where it advised repeating chest x-ray in 6 weeks
Answer 3.12.4 The correct answer is “C.” T e most likely prob- in patients with a smoking history. T e reason or that recom-
lem in this patient is a parapneumonic e usion. “A,” an anaero- mendation is a chance that the in ltrate would obscure an under-
bic abscess, is unlikely given that there are no air/ uid levels lying malignancy. T ose over age 50 should also have ollow-up
and the act that the uid appears to be in the pleural space. “B” x-ray. Bacteremic pneumococcal pneumonia has been associated
is unlikely. Development o resistance should take more than with very slowly clearing x-rays, up to 3 to 5 months in some
3 days, especially since this patient is on two drugs. “D” is cases. T us, repeating the chest x-ray in 2 weeks is unlikely to
unlikely given that this patient does not have a history o heart show resolution, so “B” is incorrect. In elderly patients, the chest
ailure, is ebrile, and has no JVD, etc. “E” is unlikely. None o x-ray takes longer to normalize than in younger patients.
the drugs that he is on is known to cause pleural e usions. T e patient is clinically doing well, and does not require treat-
ment or a persistent pulmonary in ection, so “D” is incorrect.
You place a chest tube to drain the pleural e usion ( ree f ow- Chest C (“A”) and bronchoscopy (“E”) would give more in or-
ing on ultrasound) and continue the current antibiotic regi- mation, but in the absence o systemic symptoms, such as weight
men. T e patient does well and is discharged 1 week later on loss, persistent cough, hemoptysis, or ever, they are not indicated.
clarithromycin a er sensitivities conclude that his organism It is important to consider the act that this “in ltrate” may rep-
is susceptible to that drug. resent a neoplastic process i it does not resolve within several
Six weeks a er the onset o illness, he returns or ollow- months.
up to ensure clearing o the chest x-ray. He is eeling well. He
is alert and oriented, and his lung examination is now nor- HELPFUL TIP:
mal. T ere is no lymphadenopathy in the neck or supracla- e h s (b r , sp
vicular areas. T e x-ray still shows le lower lobe in ltrate, r , .), s s
unchanged in size rom the initial x-ray. T e pleural e usion 50% s s p . t h s, r s s
has resolved. p r . Sp r s r b h r p
r s s s p s hp ,
Question 3.12.5 Which o the ollowing is the most appro 30% p s h p r b
priate next step in the evaluation and management o this pr sp .B r s r r
patient? p b r s r s -
A) Chest C . s r h sp z p s. i p ss b , b -
B) Chest x-ray in 2 weeks. r sb r h r p .
C) Chest x-ray in 6 weeks.
Ch Apt Er 3 • Pu l mo n o l o g y 123

Your patient returns a week later looking ill. He has dyspnea, HELPFUL TIP:
ever, and worsening cough. His temperature in the o ce is S r s b h p h sp z p h
39°C, and he has a new in ltrate on chest x-ray. - q r p h r s r
s ppr ss , r hr s r s
Question 3.12.6 This situation is best described as: ’ h r b s( r r -
A) Community-acquired pneumonia. s r s). S l . 2015;385(9977):1511–
B) Double pneumonia. 1518 r hs h ( , h h sp ).
C) Hospital-acquired pneumonia.
D) Healthcare-associated pneumonia.
E) riple pneumonia.
In one o li e’s unny little coincidences, the next day you
Answer 3.12.6 The correct answer is “D.” T e 2007 IDSA/ diagnose this patient’s 36-year-old healthy son with a com-
A S guidelines de ne high-risk pneumonia based on setting. munity-acquired pneumonia. He has a ever, cough, and le
Healthcare-associated pneumonia is de ned as pneumonia lower lobe in ltrate on chest x-ray, but he is hemodynami-
occurring in a nonhospitalized patient who has had exten- cally stable. You determine that he’s t or outpatient man-
sive healthcare contact (meaning IV therapy or chemotherapy agement.
within the last 30 days, residence in a nursing home, attendance
at a hemodialysis center within the last 30 days, or 2 or more Question 3.12.8 Which o the ollowing drug regimens is
days in an acute care hospital in the last 90 days). Your patient appropriate or the treatment o this patient in the outpa
meets this de nition due to his recent hospitalization. “C,” tient setting?
hospital-acquired pneumonia, is one that develops 48 hours A) Cephalexin 250 to 500 mg PO QID or 10 days.
or more a er admission to a hospital and did not appear to B) Penicillin V 250 mg ID or 10 days.
be brewing at the time o admission. T e guidelines also dis- C) Clarithromycin 500 mg BID or 10 days.
tinguish ventilator-associated pneumonia, which occurs in D) Doxycycline 100 mg BID or 10 days.
patients 48 hours or more a er endotracheal intubation. “B” E) C or D.
and “E” are ake oils. Sorry!
Answer 3.12.8 The correct answer is “E.” T e treatment o
Question 3.12.7 Now that he has healthcare associated community-acquired pneumonia requires coverage o “typical”
pneumonia, you order the appropriate antibiotic regimen and “atypical” organisms. Neither “A” nor “B” covers atypical
consisting o : organisms. Guideline-recommended choices or the outpatient
A) Linezolid and amphotericin B. treatment o community-acquired pneumonia include doxycy-
B) Ce epime. cline and macrolides such as clarithromycin. Additional options
C) Gentamicin and vancomycin. include the respiratory uoroquinolones such as moxi oxacin,
D) Cipro oxacin. gemi oxacin, or levo oxacin. However, respiratory uoroqui-
nolones should not be used in all cases. T e IDSA/A S consen-
Answer 3.12.7 The correct answer is “B.” Since he has sus guidelines recommend that respiratory uoroquinolones
received antibiotics in the last 90 days, an anti-pseudomonal be reserved or patients with serious underlying disease (e.g.,
cephalosporin is indicated. Depending on circumstances, COPD, diabetes, immunocompromised states). O the appro-
other antibiotics may be added to broaden empiric coverage. priate regimens, doxycycline and erythromycin are the least
For example, i there are high levels o resistance to Gram- expensive, but erythromycin is associated with a high rate o
negative bacilli in the area, add an aminoglycoside. I MRSA gastrointestinal intolerance.
is suspected, add linezolid or vancomycin. I legionella is
likely, add a uoroquinolone. In this scenario, “A” is incorrect
HELPFUL TIP:
because it does not cover pseudomonas and a ungal in ec-
2007 id Sa /a t S s r r p -
tion is not likely in this immunocompetent patient. T e same
s h - q r p r h
is true o “C.” “D” may be arguable but is not the pre erred
r 5 s. H r, pr r s -
empiric choice according to the 2007 IDSA/A S guidelines.
r , h sh s b br r
Most important: know your local resistance patterns and your
48 72 h rs sh h r h h
patient’s speci c characteristics.
r r s b :h r r > 100 bp ,
r sp r r r > 24, s s b pr ss r < 90
H ,o2 s r < 90% r r, P o 2 < 60 H
HELPFUL TIP: r r, b r k , s .
Pr p p h b rs H2-b k rs r ss Sh h p h r h h s r r ,
h r s r sk p .S p h h r r b h r p s ( h p
. s h h h !).
124 Fa mil y med ic in e exa min a t io n & Bo a Rd Review

Objectives: Did you learn to . . .


sr sh b p r h p s. t h r s
• R z h pr s - q r
h sh s h s h j r s r
p r p p p s?
pr vil i s sr s b r
• d r h ppr pr sp s r p h
6 8 l /k pr b h p s.
- q r p ?
i ,p s h sh r c o Pd q k
• i p p r r -
p h p r h h h r -
q r p ?
s. v p s h bs r s s -
r r r h .
CASE 3.13
While you are covering the ED, a 60-year-old emale comes
in by ambulance. She is unresponsive, and her husband states Your patient is on assist-control mode o ventilation. A
that he ound her 30 minutes ago surrounded by bottles nasogastric tube, 2 IVs, and a bladder catheter are in place.
o pills and an empty bottle o vodka. She has a history o She was given IV N-acetylcysteine. Her blood pressure has
COPD, hypertension, osteoarthritis, and depression. T e improved to 112/67 mm Hg, and her oxygen saturation
EM s brought in her pill bottles, which include lorazepam, is 99%. Her chest x-ray shows an endotracheal tube ter-
acetaminophen/hydrocodone, hydrochlorothiazide, aspirin, minating 3 cm above the carina and no in ltrates. T irty
and sertraline. Only a ew tablets are le in the bottle o minutes a er you intubated her, with the ventilator rate at
hydrochlorothiazide. She is wearing a nonrebreather ace- 14 breaths/min, FiO2 100%, and tidal volume at 400 mL,
mask with 50% oxygen. Her respirations are shallow with you obtain another ABG: pH 7.35, PaCO2 45 mm Hg, PaO2
a rate o 8. T e remainder o her vitals: temperature 36°C, 130 mm Hg. She takes 6 to 8 spontaneous, assisted breaths,
blood pressure 90/50 mm Hg, and pulse rate 90 bpm. Oxygen while the ventilator provides the remaining breaths. She
saturation is 88% and increases to 94% with some assisted appears to be per using her periphery well.
breaths. One nurse is obtaining a blood gas while another
gives naloxone. You decide that this patient cannot protect Question 3.13.2 Your next action is to:
her airway and choose to intubate her. A) Decrease the tidal volume to allow or permissive hypercapnia.
T e blood gas drawn just be ore intubation shows pH 7.16, B) Increase the tidal volume to achieve a pH o 7.45 to 7.50.
PaCO2 60 mm Hg, PaO2 40 mm Hg. C) Reduce FiO2 while maintaining oxygen saturations at or
above 90%.
Question 3.13.1 These ndings imply which o the ollow D) Change to pressure support ventilation.
ing processes?
A) Metabolic acidosis. Answer 3.13.2 The correct answer is “C.” Your patient is per-
B) Metabolic alkalosis. using well, and her PaO2 and measured oxygen saturation are
C) Respiratory alkalosis. much improved. You should now decrease the FiO2, with the
D) Mixed metabolic/respiratory acidosis. goal being to achieve an FiO2 o less than 60% while maintain-
E) Mixed metabolic/respiratory alkalosis. ing adequate per usion and oxygen saturation. An FiO2 o 100%
is somewhat toxic and can lead to airway injury. “A” is incorrect.
Answer 3.13.1 The correct answer is “D.” T e pH is acidotic However, permissive hypercapnia (the CO2 may be allowed to
(< 7.4). In a patient whose baseline PaCO2 is not known to you, rise to > 80 mm Hg as long as the patient tolerates it) may be
you might assume her PaCO2 is usually 40 mm Hg, which is the use ul in ventilated patients with COPD, ARDS, or asthma.
accepted normal or most patients. But remember that you still need to maintain oxygenation.
I the acidosis is purely due to acute respiratory changes and “B” is incorrect. Your patient is doing reasonably well with her
CO2 retention, a rise in PaCO2 o 10 should be accompanied slightly acidotic pH, which has corrected very quickly. It may be
by a all in pH equal to 0.08. In this case, the change in PaCO2 inadvisable to attempt to increase her pH beyond 7.40, as she
is 20. So, 20/10 × 0.08 = 2 × 0.08 = 0.16 resulting in a pH o may develop respiratory alkalosis that can then lead to cardiac
7.4 − 0.16 = 7.24. arrhythmias. Because o her low respiratory rate, she should
However, this patient’s pH is measured at 7.16, lower than remain on some type o assisted volume-cycled ventilation.
expected or a pure respiratory acidosis presenting acutely. T us, Pressure support ventilation, as its name implies, only aug-
you can determine that the acidosis is both metabolic (perhaps ments patient-triggered breaths with increased airway pressure.
rom lactic acidosis rom hypoper usion) and respiratory.
HELPFUL TIP:
HELPFUL TIP: P r ss h p r p s ppr h h
i s r h s r pr p - p h r h pr r s co 2 b p -
s h r sp r r s r ss s r / r s pH ( s 7.1) r r pr -
j r (a Rd S/a l i) pr vil i ( r- pr r s
j r ). i s r h h r h s pr h p r r bs r .
Ch Apt Er 3 • Pu l mo n o l o g y 125

Question 3.13.3 In this patient, which o the ollowing venti


HELPFUL TIP:
lator management techniques will unequivocally decrease
RSBi s h pr rs s ss b .i
her FiO2 requirement?
s b r sp r r r b
A) Increase the respiratory rate.
rs h p s pr ss r s pp r
B) Increase the positive end-expiratory pressure (PEEP).
h s s (pr ss r 5–10, PeeP 5–8). F r
C) Decrease the tidal volume.
p ,p br h 20 sp r h
D) Addition o inhaled nitric oxide (NO).
500 l h s RSBi 20/0.5 = 40. P
br h 34 sp r h 210
Answer 3.13.3 The correct answer is “B.” wo standard tech-
l h s RSBi 34/0.21 = 161 s b s r
niques are usually employed to improve a patient’s oxygenation:
b b .
increasing FiO2 and PEEP. PEEP maintains positive pressure
in the airways at the end o expiration. Its use increases lung
compliance and decreases ventilation/per usion mismatching, HELPFUL TIP:
resulting in better oxygenation. Since FiO2 > 60% over peri- ic u s s h r h o R; b s
ods longer than 48 hours may result in oxygen toxicity, PEEP h ic u rr h h r r sk r .d b r
may be employed to reduce the need or high levels o FiO2. “A” p s b r - b . th r s p r
and “C” are incorrect. Increasing respiratory rate or tidal vol- pr r s ss ic u b sh
ume will cause increases in minute ventilation, which reduces h p 15% ic u p s r q r r - b
PaCO2, but has little e ect on PaO2. Decreasing minute ventila- sp pr rs s ss b .
tion, through decreased respiratory rate or tidal volume, causes
CO2 retention and increased PaCO2. “D” is incorrect. Nitric
oxide has been shown to improve oxygenation in select patients HELPFUL TIP:
with severe pulmonary hypertension and ARDS, but its use is m ( r h p )s b -
not appropriate in this patient. p r sp r r r b . t h s, p rs
20 br hs/ r h r 400 l /
You ollow the patient during her hospitalization. T e next br h r s = 20 × 400 =
day she is more alert and is able to ollow commands. Her 8l/ .
ventilator requirements have decreased. You consider extu-
bation.

Question 3.13.4 All o the ollowing parameters predict a Objectives: Did you learn to . . .
poor outcome or attempted weaning rom ventilation • R z r sp r r / b s s?
EXCEPT: • c p pH h s r sp r r s s?
A) Minute ventilation supplied by ventilator is < 10 L/min. • i s h ppr pr rs -
B) PaO2 < 55 mm Hg while on FiO2 > 35%. s?
C) Rapid shallow breathing index (RSBI) o 140. • i p p s ?
D) Physical examination ndings o increased respiratory • w p r h r?
e ort.

Answer 3.13.4 The correct answer is “A.” Preparing to with-


CASE 3.14
draw a patient rom mechanical ventilation—typically called A 52-year-old male smoker presents or a 3-month history o
weaning or liberation—relies considerably on judgment, but productive cough. He reports multiple episodes o pneumo-
a ew objective parameters can be help ul. In general, the nia, but he appears healthy now. Chest x-ray is unremarkable.
patient to be liberated must be awake, alert, and coopera- Chest C shows enlarged peripheral airways with thickened
tive. She should have reasonably good oxygenation on a lower airway walls in the lower lobes bilaterally. Sputum culture
FiO2, have PEEP < 8 cm H 2O, and be able to generate adequate grows several types o bacteria, including P. aeruginosa.
inspiratory pressures. Minute ventilation rom the ventilator
o less than 10 L/min is associated with greater success with Question 3.14.1 Which o the ollowing do you recommend
weaning. as initial therapy?
Poor prognostic indicators include a minute ventilation A) Corticosteroids.
rom the ventilator > 10 L/min, PaO 2 < 55 with FiO 2 > 35%, B) Antibiotics.
and RSBI (see Help ul ip) > 105. Patients with poor cardio- C) Chemotherapy.
pulmonary reserve or who have signi cant underlying disease D) Supplemental oxygen.
may also have dif culty weaning. Allow patients a period o E) Wedge resection o the a ected lung tissue.
breathing on their own (e.g., a -piece) be ore extubating. T is
way, i the patient ails, you can simply hook her back up to the Answer 3.14.1 The correct answer is “B.” T is patient’s nd-
ventilator. ings are consistent with the diagnosis o bronchiectasis, a
126 Fa mil y med ic in e exa min a t io n & Bo a Rd Review

chronic in ammatory disease o the medium-sized bronchi. O the ollowing tests, which is the most likely to con rm or
Appropriate initial therapy consists o prolonged courses alter the diagnosis?
o antibiotics, usually 2 weeks o a narrow-spectrum anti- A) Spirometry, di using capacity, lung volumes.
biotic ollowed by reassessment. Doxycycline, amoxicillin, B) Spirometry, ABG, di using capacity.
clarithromycin, amoxicillin/clavulanate, and trimethoprim/ C) ABG, di using capacity, lung volumes.
sul amethoxazole are o en used. Respiratory quinolones D) ABG, lung volumes, chest C .
demonstrate some limited use in patients with Pseudomonas.
Patients should be directed to discontinue tobacco use and The correct answer is “A.” T is case demonstrates a commonly
take inhaled bronchodilators. Resection o the a ected lung seen phenomenon: patients who smoke and have dyspnea are
tissue may be necessary, but should not be the initial therapy. assumed to have obstructive lung disease, particularly emphy-
Supplemental oxygen therapy is used i oxygenation is poor. sema. However, pulmonary unction tests are required to make
Chemotherapy and prolonged oral corticosteroids are not the diagnosis o obstructive disease. Furthermore, this patient’s
used to treat bronchiectasis. chest x-ray shows increased interstitial markings in the lower
lung elds suggesting interstitial lung disease. T is disease pro-
cess is associated with a restrictive pattern on pulmonary spi-
HELPFUL TIP: rometry. However, lung volumes and di using capacity will
P s h br h ss r r h provide a more complete picture. T us, spirometry, di using
r r b s pr - capacity, and lung volumes would allow you to make the diag-
rb s. o r s r b r- nosis. In interstitial lung disease, spirometry shows a FEV1/FVC
h pr /s h z , k ratio > 0.7, a decreased di using capacity, and decreased total
h s b s r h rs 10 14 s h lung capacity (the hallmark o restrictive lung disease). ABG is
h. t h ss hs r s unlikely to help di erentiate restrictive rom obstructive lung
s . disease, so options “B” through “D” are not correct.

Question 3.14.2 In most adults with bronchiectasis, its HELPFUL TIP:


cause is: th s p , s s r b dl co , s
A) Genetic. h s s s s: ph s , rs
B) Pseudomonas in ection. s s ( . ., s r , s, p r r -
C) obacco smoking. ,p r r r ss h s r ,
D) Allergic bronchopulmonary aspergillosis (ABPA). p r br s s), P s sp , p -
E) Unknown. r s r s s .a s s
d l c o —s h b rh r sh s
Answer 3.14.2 The correct answer is “E.” T ere are limited b r r h dl co .
data regarding the etiology o bronchiectasis, but many condi-
tions and environmental exposures seem to have an association.
In most patients, no cause is identif ed. Children are more likely HELPFUL TIP:
than adults to have an identi ed etiology o their bronchiecta- dl co b r s b p r h rrh ,
sis, and the most common causes in kids are oreign body aspi- p h , ss b s , - -r h r r
ration, cystic brosis, and gastroesophageal re ux. Identi ed sh , sh , h r r ( r s p -
etiologies in adults include those mentioned or children and r h s).
pulmonary in ections, ABPA, COPD, rheumatic diseases, and
cigarette smoking.
Objectives: Did you learn to . . . QUICK QUIZ: Pu l mo n a Ry in Fec t io n S
• i f s ss h br h s s?
• tr p h br h s s? A 72-year-old woman you admitted to the hospital or pneu-
monia is having worsening dyspnea and hypoxemia. She is
decompensating despite 3 days o antibiotic therapy with intra-
QUICK QUIZ: d ySPn ea venous levo oxacin. According to her husband, the couple
had been working on their Iowa arm, and made a trip to an
A 75-year-old gentleman presents to your of ce with emphy- old barn to collect manure a week be ore the patient developed
sema diagnosed elsewhere. He reports dyspnea on exertion a er a cough and ever (so this is not hypersensitivity pneumonitis
1 to 2 blocks. He smokes 10 cigarettes per day and does not have nor organic toxic dust syndrome). T e barn was noted to be
underlying cardiac disease. Physical examination is remarkable the home o numerous birds. Her respiratory rate is 32, and her
or ne crackles in both lung bases. Chest x-ray shows increased oxygen saturation is 89% on 5 L/min o oxygen by nasal can-
interstitial markings in the lower lobes. He has no pulmonary nula. Chest x-ray reveals a di use interstitial in ltrate, enlarged
unction testing on record. mediastinal nodes, and normal heart size.
Ch Apt Er 3 • Pu l mo n o l o g y 127

Which o the ollowing is the most likely culprit or the cause o orm o hypersensitivity pneumonitis. However, they need not
her current illness? be present in the subacute and chronic orms o the disease.
A) S. pneumoniae. So, based on symptoms, this could be hypersensitivity pneu-
B) H. in uenzae. monitis. However, the radiologic ndings o hypersensitivity
C) Coxiella burnetii. pneumonitis would include interstitial lung disease, rather than
D) Histoplasma capsulatum. central bronchiectasis. T us, this is not likely hypersensitivity
E) Blastomyces dermatitidis. pneumonitis.
“B” and “D” are incorrect. Note that she has no signi cant
The correct answer is “D.” T e case described here is classic or constitutional symptoms that might be more typical o acute
an environmental exposure to a large dose o Histoplasma organ- eosinophilic pneumonia or bacterial in ection. You would also
isms. Histoplasmosis is o en transmitted by bird or bat drop- expect an in ltrate on the chest C . “E” is incorrect. Churg–
pings. Histoplasma occurs most commonly in the Mississippi Strauss vasculitis is characterized by transient patchy intersti-
and Ohio River valleys, causing a sel -limited disease in most tial in ltrates, ever, weight loss, elevated sedimentation rate,
persons. Patients who have Histoplasma in ection requently de- abnormal liver enzymes, and a peripheral blood eosinophilia
velop calci ed mediastinal lymph nodes a er resolution o the > 1,000 cells/µL. T is is o en related to using an oral steroid
in ection. Diagnosis can be made by urinary antigen or broncho- and a leukotriene inhibitor simultaneously. It seems, but isn’t,
scopic biopsy. T e bacterial causes are unlikely to be important related to inhaled steroids because once the patient is on a ste-
actors here, as she was on a broad-spectrum antibiotic or 3 days roid inhaler, the oral steroid is usually tapered unmasking the
with no improvement. C. burnetii, the agent causing Q ever, is vasculitis. Extrapulmonary mani estations distinguish Churg–
rare and tends to a ect workers exposed to resh animal material, Strauss rom other eosinophilic pulmonary conditions.
such as placentas. Blastomyces is ound in the same regions as
Histoplasma, but the site o exposure tends to be more moist, un- Question 3.15.2 Which o the ollowing would be the next
like the dry environment inside a barn. We all have granulomas best step in con rming the diagnosis?
rom histoplasmosis on CXR in Iowa. A) Sputum cultures.
B) ransbronchial biopsy.
CASE 3.15 C) Methacholine challenge.
D) Allergy skin testing or Aspergillus species.
A 42-year-old emale comes to your o ce with a history o E) p-ANCA.
asthma that has been di cult to control. She relates symp-
toms that have been worsening over the last 4 to 6 weeks. Answer 3.15.2 The correct answer is “D.” Most but not all o
She received two courses o oral corticosteroids during that the ollowing criteria ( able 3-9) need to be present in order to
time. Her symptoms improved with this therapy but quickly make the diagnosis o ABPA. ransbronchial biopsy is unneces-
returned a er completing the steroids. She denies ever, sarily invasive, and the other tests will not help to con rm the
chills, and night sweats, but complains o a chronic cough diagnosis.
productive o brownish-colored sputum. She is a homemaker
in a suburban area and has no pets. Physical examination Question 3.15.3 The most appropriate treatment or this
reveals wheezing throughout all lung elds but is otherwise patient with ABPA would include which o the ollowing?
normal. Laboratory evaluation includes CBC with increased A) Antibiotics.
eosinophils, normal C-reactive protein, and an elevated IgE B) Oral corticosteroids.
level o 1250 ng/mL. A high-resolution C scan o the chest C) Leukotriene receptor antagonist.
reveals central bronchiectasis. D) Itraconazole.
E) Inhaled ipratropium bromide.
Question 3.15.1 What is the most likely diagnosis?
A) Hypersensitivity pneumonitis. Answer 3.15.3 The correct answer is “B.” Oral corticosteroids
B) Acute eosinophilic pneumonia. are the treatment o choice or ABPA. Patients are typically
C) Allergic bronchopulmonary aspergillosis (ABPA). treated or several months with tapering doses rather than short
D) Bacterial pneumonia.
E) Churg–Strauss vasculitis.
TABLE 3-9 Cr It Er IA FOr t h E DIAGNOSIS OF ALLEr GIC
Answer 3.15.1 The correct answer is “C.” T is patient’s history Br ONCh OpULMONAr Y ASpEr GILLOSIS (ABpA)
points to the diagnosis o ABPA, which is characterized by the
• as h
presence o severe asthma, brownish mucus plugs, peripheral • c r br h ss
eosinophilia above 10%, elevated serum IgE, and central bronchi- • e s r i e > 1,000 / l
ectasis. IgE elevation is required to be greater than 1,000 ng/mL. • i sk s r Aspergillus
“A” is unlikely but a bit tricky. First, there is no history o • e s r -sp f i e / r i g Aspergillus fumigatus
• P r ph r b s ph > 10%
exposure to a causative agent. Second, let’s ocus on symptoms.
• P r f r s
Constitutional symptoms—o en ever—are present in the acute
128 Fa mil y med ic in e exa min a t io n & Bo a Rd Review

courses o steroids. Serum IgE levels and chest x-rays are used to C) Change classes o antihistamines.
monitor response to treatment. Please note that “D” is incorrect, D) Re er or allergy evaluation, including percutaneous
and azoles are not the mainstay therapy in ABPA. T e goal o aeroallergen skin testing.
treatment is suppression o the immune system responding to E) Recommend a high ef ciency particulate air lter or the
the ungal antigen. T ere are studies that show bene t o adding home.
an anti ungal agent as a steroid-sparing agent. However, there
are risks associated with concomitant use o steroids and azoles, Answer 3.16.2 The correct answer is “C.” Although all o the
namely, marked adrenal suppression. Hence, oral corticoste- above choices may provide relie or allergic symptoms, the best
roids should be your rst choice. rst step would be to try changing the class o antihistamines.
Antihistamines (and NSAIDs or that matter) are grouped into
Objectives: Did you learn to . . .
classes based on chemical structure. One class may be help ul
• i h pr s a BPa ?
or a patient when another class does not work.
• d s r p h a BPa ? T e other choices are suboptimal. Without skin testing,
avoidance measures may be needless, costly, and ine ective.
CASE 3.16 Skin testing alone (“D”) would not immediately improve her
A 16-year-old emale comes to your o ce with complaints o symptoms, and you would still need to do something with
sneezing spells, itchy watery eyes, and nasal congestion or therapy (we hope) while waiting 3 months to see an allergist.
the past 2 years. T ese symptoms are worse during the spring Allergen-impermeable encasements (“A”) are currently recom-
and all and when she plays with her cat. She denies any other mended or patients with dust mite allergy. Although lters
constitutional symptoms and has no other past medical his- (“E”) are o en recommended or pet allergies, the data regard-
tory. She has tried over-the-counter loratadine without relie . ing their e ectiveness in reducing allergic symptoms is contra-
She has lived in the same residence or the past 6 years and dictory. opical decongestant sprays (“B”) are an inappropriate
denies any other environmental exposures. Her examination choice secondary to the addictive nature o these medications
reveals pale nasal mucosa and swollen nasal turbinates bilat- and the risk o causing rebound symptoms. Removing a pet
erally. Her lungs and skin are clear. rom the bedroom may reduce—but not eliminate—allergen
exposure. Further evaluation should be per ormed prior to rec-
ommending any such li estyle modi cations. Another reason-
Question 3.16.1 You are not clear i this is allergic rhinitis able option would be a trial o intranasal corticosteroid. Finally,
or nonallergic rhinopathy (previously known as vasomo you could also put an allergen impermeable encasement around
tor rhinitis). The most ef ective way to determine i this is the cat . . .
allergic is to:
A) Do a methacholine challenge test. Your patient’s 17-year-old brother is in the next examination
B) Do a Hansel stain o nasal mucus. room. He is a Boy Scout who just returned rom a backpack-
C) Check overall IgE levels. ing trip in the our corners area o New Mexico, Arizona, Col-
D) Per orm a nasal mucus electrophoresis to visualize allergic orado, and Nevada (. . . did you catch that? I you know your
bands. geography, you know one o those is wrong). He has noted
E) Wa hot peppers under her nose . . . see what happens. myalgias, ever, and chills. He knows that you are reviewing
pulmonary medicine or your upcoming Board Examina-
Answer 3.16.1 The correct answer is “B.” T e best way to tell tion, so he presents to your o ce complaining o dyspnea
i this is allergic is to do a Hansel stain o the nasal mucous. that has been getting markedly worse over the past several
T is will show eosinophils i it is allergic. I this is nonallergic days. He has no URI symptoms such as coryza, rhinorrhea,
rhinopathy, eosinophils will be absent. I it is in ectious, there ear pain, etc. He has noticed nausea, vomiting, and diarrhea
will likely be a predominance o neutrophils. “A” is incorrect. A with severe abdominal pain. His respiratory rate is 40 with
methacholine challenge test is help ul in diagnosing asthma, not an oxygen saturation o 88%. (“See,” he says, “I told you I’m
allergic rhinitis. Neither “C,” IgE levels, nor nasal mucus electro- sick.”) You place him on nasal oxygen and order a chest x-ray,
phoresis have any use here (and nasal mucus electrophoresis has which shows bilateral pulmonary edema.
no use anywhere that we know o . . . and it is disgusting). IgE
is only elevated in 40% o patients with allergic rhinitis. Does Question 3.16.3 Based on the epidemiology and chest x ray
anyone ever actually do a Hansel stain? Probably not, but once appearance, your best guess at this point in the disease is:
again, this is or the Board Examination, not real li e. A) Plague.
B) Coccidiodomycosis.
Question 3.16.2 The Hansel stain shows eosinophils. “Eureka!” C) Hantavirus rom Sin Nombre (No Name) strain.
you shout. Which o the ollowing would be the most appro D) Noncardiogenic pulmonary edema rom smoking para-
priate next step in managing her symptoms? quat (you never know what is going on at those Boy Scout
A) Recommend allergen-impermeable encasements or camps).
mattress and pillow. E) Ischemic cardiomyopathy rom cocaine use (you never
B) Use topical decongestant sprays. know what is going on at those Boy Scout camps).
Ch Apt Er 3 • Pu l mo n o l o g y 129

Answer 3.16.3 The correct answer is “C.” T is is a typical his-


B j s h r s r hs r s; s h
tory and physical examination or hantavirus. T e absence o h s p s r s r r r b b h p .
URI symptoms, the presence o GI symptoms, and the noncar- th pr h rs h s s .
diogenic pulmonary edema are all symptoms/signs o hantavi-
B r s r r h p p (b
rus. In act, it may present as an acute abdomen. It is spread
s h ).
by aerosolization o mouse excrement or urine. “B” is incor-
rect. Although coccidiodomycosis, “Valley Fever,” is ound in d s - b - s sh h
h s r .i rs s s.
the same geographical region, it presents with lower respira-
tory symptoms, a thin walled cavitary lesion, erythema nodo- d s h s r s r c o Pd . t h
sum (10%), and eosinophilia. Coccidiomycosis is generally a r s p r b .
low grade, subacute process that lasts weeks to months. It does F r c o Pd p s, r s p
not cause noncardiogenic pulmonary edema. “A,” pneumonic p s h sh r - h br h r ( . .,
plague, while also ound in the same area, causes high ever, b r ) h - h br h r
bloody sputum, pleuritic chest pain, and develops over hours to ( . ., r p ) s p s r r .
days. It can be rapidly atal i not recognized and treated within i s p ss b r b r “ p ”r
the rst day. “D” and “E” are wrong or so many reasons that we “ p ”p b -r .
won’t bother to enumerate them here, but you never do know wh s h r p , k s r h r s
what goes on at camp. s r q r s h
ss h br h h p h h sp .
Question 3.16.4 Laboratory ndings suggestive o hantavi
rus include all o the ollowing EXCEPT:
A) T rombocytopenia. BIBLIOGRAPHY
B) Leukocytosis with a le shi .
American College o Chest Physicians. . Pulmonary Medicine
C) A lymphocytic predominance. Board Review. Northbrook, IL: ACCP; 2009;167–173.
D) An immunoblast count < 10%.
Annema J , et al. Endoscopic ultrasound added to medias-
tinoscopy or preoperative staging o patients with lung
Answer 3.16.4 The correct answer is “D.” In act, thrombocy- cancer. JAMA. 2005;294:931–936.
topenia, a 10% or greater immunoblast count, and a le shi Cahill K, et al. Nicotine receptor partial agonists or smok-
constitute the so-called diagnostic triad in a patient with appro- ing cessation. Cochrane Database Syst Rev. 2007;24(1):
priate clinical ndings. Immunoblasts are the most immature CD006103.
cell in the lymphocyte line (they still enjoy scribbling on walls Celli BR, et al. T e body-mass index, air ow obstruction, dys-
and drink irresponsibly). Overall, case atality rate o hantavi- pnea, and exercise capacity index in chronic obstructive
rus is up to 50%. Care is supportive. Extra-corporeal membrane pulmonary disease. N Engl J Med. 2004;350:1005–1012.
oxygenation may be used in seriously ill patients. T ere is o en Daniels JM, et al. Antibiotics in addition to corticosteroids or
an oliguric phase that needs care ul management to prevent acute exacerbations o chronic obstructive pulmonary
uid overload. T is can be problematic because patients are disease. Am J Respir Crit Care Med. 2010;181:150–157.
o en hypotensive and there is a proclivity toward giving them Decramer M, et al or Global Initiative or Chronic Obstructive
uids. Lung Diseases. Global strategies or the diagnosis, man-
agement, and prevention o chronic obstructive pulmo-
Objectives: Did you learn to . . . nary disease: GOLD executive summary. 2015. Available
• a ss ss r r rh s? at http://www.goldcopd.com; accessed August 31, 2015.
• d s h r s ? Ege MJ, et al. Exposure to environmental microorganisms and
childhood asthma. N Engl J Med. 2011;364:701–709.
Gibson PG, et al. Allergic bronchopulmonary aspergillosis.
Semin Respir Crit Care Med. 2006;27(2):185–191.
Clinical Pearls Irwin RS, Madison MJ. T e diagnosis and treatment o cough.
N Engl J Med. 2000;343(23):1715–1721.
a r co 2 b s sp p s ss
Zodang W, et al. Outpatient patient treatment in patients with
co 2 r p r s r sp r r s s.
acute pulmonary embolism. J T romb Haemost. 2011;9:
a sb s h s ss h s s s 1500–1507.
r r b s p r h o2 h h r Lacasse Y, et al. Clinical diagnosis o hypersensitivity pneumo-
r s s r. v s s sh r ss. nitis. Am J Respir Crit Care Med. 2003;168:952–958.
a s s sp r r s c o Pd . o 55% h s Laszlo G. Standardisation o lung unction testing: help ul
s s s s s p sh c o Pd ; h guidance rom the A S/ERS ask Force. T orax. 2006;61:
r s h r r s r h r sp . 744–746.
as p g eRd s s sh rb s. Mandell LA, et al. In ectious Diseases Society o America/
American T oracic Society consensus guidelines on the
130 Fa mil y med ic in e exa min a t io n & Bo a Rd Review

management o community-acquired pneumonia in acute respiratory ailure? Respir Care. 2007;52:556–


adults. Clin In ect Dis. 2007;1(44, Suppl 2):S27–S72. 564.
Maurelus K, et al. Focus on: Ultrasound or T oracentesis. illie-LeblondI, et al. Pulmonary embolism in patients with
2013. Available at http://www.acep.org/Education/Con- unexplained exacerbation o chronic obstructive pulmo-
tinuing-Medical-Education-(CME)/Focus-On/Focus-On– nary disease: prevalence and risk actors. Ann Int Med.
Ultrasound- or-T oracentesis./; accessed June 21, 2015. 2006;144:390–396.
National Asthma Education and Prevention Program. Expert U.S. Preventive Services ask Force. Lung Cancer Screening
Panel Report 3 (EPR-3): Guidelines or the diagnosis and Guidelines. 2013. Available at http://www.uspreventi-
management o asthma-Summary Report 2007. J Allergy veservicestask orce.org/Page/ opic/recommendation-
Clin Immunol. 120;2007:S94–S138. Available at http:// summary/lung-cancer-screening; accessed June 23,
www.nhlbi.nih.gov/ les/docs/guidelines/asthsumm.pd ; 2015.
accessed June 21, 2015. Von Essen SG, et al. A er exposure to the hog barn environ-
Ost D, et al. Clinical practice. T e solitary pulmonary nodule. ment. J Swine Health Prod. 2005;13:273–276.
N Eng J Med. 2003;348(25):2535–2542. Von Essen SG, et al. Organic dust toxic syndrome: a nonin ec-
Palareti G, et al. D-Dimer testing to determine the duration tious ebrile illness. J oxicol Clin oxicol. 1990;28(4):
o anticoagulation therapy. N Engl J Med. 2006;355:1780– 398–420.
1789. Wilbur J, Shian B. Diagnosis o deep venous thrombosis and
Pasteur MC, et al. British T oracic Society guideline or non- pulmonary embolism. Am Fam Physician. 2012;86(1):
CF bronchiectasis. T orax. 2010;65:i1–i58. 913–919.
Paramothayan S, Jones PW. Corticosteroid therapy in pul- Yasu uku K, et al. Comparison o endobronchial ultrasound,
monary sarcoidosis: a systematic review. JAMA. 2002; positron emission tomography, and C or lymph node
287(10):1301–1307. staging o lung cancer. Chest. 2006;130:710–718.
SteinbergKP, KacmarekRM. Respiratory controversies in Yu CJ, et al. Ultrasound study in unilateral hemithorax opaci-
the critical care setting. Should tidal volume be 6 mL/ cation. Image comparison with computed tomography.
kg predicted body weight in virtually all patients with Am Rev Respir Dis. 1993;147:430–434.
Allergyand Immunology
Wendy Shen and Jason K. Wilb r
4
Question 4.1.2 With regard to radioallergosorbent testing
CASE 4.1 (RAST) or allergic rhinitis, which o the ollowing is true?
A 15-year-old girl has a history o acute di culty breathing A) RAS is less expensive than traditional skin testing.
when playing basketball. Her symptoms include inspiratory B) RAS is more sensitive than traditional skin testing.
wheezing/stridor, increased respiratory rate, throat tight- C) RAS has a limited role in testing those with allergic rhinitis.
ness, and chest discom ort. Premedication with adequate D) Antihistamine use is a contraindication to the use o RAS .
doses o albuterol has no e ect.
Answer 4.1.2 The correct answer is “C.” RAS will be nega-
Question 4.1.1 What is the most likely diagnosis? tive in up to 25% o those with a positive skin test, has poorly
A) Exercise-induced asthma. reproducible results, and is more expensive. T us, skin testing
B) Gastroesophageal re ux disease. remains the procedure o choice or identi ying allergens. RAS
C) Musculoskeletal chest pain. can be used i skin testing is unavailable.
D) Hyperventilation.
E) Vocal cord dys unction. Question 4.1.3 Which o the ollowing medications does
NOT need to be discontinued prior to aeroallergen skin
Answer 4.1.1 The correct answer is “E.” Vocal cord dys- testing?
unction (VCD) is one o the most common asthma mim- A) Intranasal steroid spray.
ics. Patients with VCD present with hoarseness, coughing, B) Atenolol.
dyspnea, and loud inspiratory wheezing/stridor, along C) Amitriptyline.
with other symptoms mentioned above. Pulmonary unc- D) Cyproheptadine.
tion testing indicates airway obstruction due to an extra- E) Azelastine nasal spray.
thoracic component. It appears that paradoxical inspiratory
vocal cord adduction causes air ow restriction at the level Answer 4.1.3 The correct answer is “A.” Intranasal steroid
o the larynx, resulting in a attened inspiratory loop on sprays do not need to be discontinued prior to skin testing as
ow–volume diagram. VCD presents a diagnostic challenge, they do not inter ere with immediate-type hypersensitivity
and o en leads to unnecessary treatment o asthma. In this reactions. T ey are not antihistaminic in nature, as opposed
patient, a β 2-agonist was ine ective, even though she displays to amitriptyline, cyproheptadine, or azelastine, which may all
symptoms with exertion. T is argues against answer “A.” T e blunt dermal reactivity. Although azelastine is administered
distinction between VCD and asthma may be less clear in as a nasal spray, its administration may inter ere with skin test
other patients, since the two disorders sometimes coexist. reactivity within 2 days o usage. Beta-blockers, such as ateno-
T e clinical history does not support the diagnoses o gas- lol, have been shown to a ect skin test reactivity and should be
troesophageal re ux disease, musculoskeletal chest pain, or avoided in patients undergoing skin testing.
hyperventilation.
Aeroallergen skin prick and intradermal testing reveals posi-
You make the diagnosis o VCD. However, the patient also tive reactions to dust mites, cat, ragweed, and tree pollens.
complains o rhinorrhea, itchy eyes, sneezing, and itchy nose. T e patient relates that she gets considerable nasal conges-
Because you realize that we need to discuss it in this book, tion and has tried over-the-counter decongestants with no
you kindly re er her or allergy testing (thank you!). relie .

131
132 FAMILY MEDICINE EXAMINATION & BOARD REVIEW

Question 4.1.4 Which o the ollowing interventions would more severe systemic reactions to immunotherapy. Patients
provide the most relie or her nasal symptoms? should be treated with an alternative antihypertensive during
A) Diphenhydramine 25 mg PO BID. immunotherapy. T e current practice parameters or aller-
B) Montelukast 10 mg PO QHS. gen immunotherapy recommend that emergency epinephrine
C) Intranasal steroid spray daily. should be readily available or the treatment o systemic aller-
D) Ipratropium bromide nasal spray BID. gic reactions associated with immunotherapy. o monitor or
E) Getting rid o the cat. No one should have a cat. T ey are these immediate reactions, patients should be observed in the
evil. of ce setting or at least 30 minutes a er immunotherapy shots
are administered. Based on studies o seasonal symptom scores,
Answer 4.1.4 The correct answer is “C.” Intranasal corticoste- it is generally recommended that allergen immunotherapy be
roids would provide the most relie in this patient by addressing continued or 3 to 5 years.
nasal congestion in addition to the other nasal symptoms men-
tioned. Although antihistamines are very help ul in relieving Your next patient has a history o ragweed and grass allergy.
nasal symptoms such as rhinorrhea, nasal itching, and sneezing, He states that he is “allergic” to needles. Accordingly, you
they are generally not as e ective or nasal congestion. Likewise, decide to use sublingual (SL) desensitization instead o sub-
montelukast is a leukotriene modi er, which is approved or the cutaneous (SQ) routes. Luckily, there are FDA-approved sub-
treatment o allergic rhinitis, but studies suggest that intranasal lingual preparations or desensitization to ragweed, imothy
steroids are superior. In addition, nasal steroids are not systemic grass and a 5-grass combination.
(although a bit may be absorbed). Ipratropium is mainly e ec-
tive or rhinorrhea only, while nasal saline irrigations promote Question 4.1.6 Which o the ollowing is true regarding SL
thinning o nasal secretions and drainage, but neither predict- desensitization?
ably improves nasal congestion. As to “E,” yes, cats are evil and A) It is more e ective than SQ desensitization.
shed. But as amily doctors we are supposed to be accepting o B) It is overall sa er than SQ desensitization; there are ewer
our patient’s quirks. adverse systemic reactions than with SQ desensitization.
C) It can cause airway obstruction by causing tongue and phar-
ynx edema.
HELPFUL (AND REALLY COOL) TIP: D) It is well studied as a combination therapy with SQ desensi-
Nasal steroids also seem to improve oc lar symptoms. tization.
Whether some o the dr ets p the nasolacrimal E) B and C.
d cts or is otherwise aerosolized into the eye is nclear.
Obvio sly, do not p t nasal steroids directly in the eye. Answer 4.1.6 The correct answer is “E.” SL desensitization
For moderate-to-severe aller ic rhinitis, combination o can causes ewer anaphylactic reactions but can cause airway
intranasal steroid and intranasal antihistamine is more obstruction rom local edema. “A” is incorrect. T e great major-
e ective than either alone. ity o studies show that SQ desensitization is more e ective than
SL desensitization. “D” is also incorrect. T e use o SL therapy
in combination with SQ desensitization is contraindicated and
may increase the rate o anaphylactic reactions. As a nal note,
She returns to you a year later, having tried intranasal ste- most patients who require desensitization need desensitization
roid sprays and high-dose antihistamines, without gaining to more than just imothy grass or ragweed or the 5-grass com-
signi cant relie . She has instituted appropriate avoidance bination limiting the use ulness o SL therapy. Patients are o en
measures during the interim (there goes the cat), also with- also allergic to tree pollen, animal dander, and other allergens
out improvement in symptoms. You recommend allergen that cannot be treated with SL therapy.
immunotherapy.

Question 4.1.5 All o the ollowing statements are true


regarding allergen immunotherapy EXCEPT: HELPFUL TIP:
A) Patients should carry emergency epinephrine to all immu- Beta-blockers sho ld be stopped in patients with a his-
notherapy shot appointments. tory o anaphylaxis i possible. Beta-blockers ampli y
B) It is unnecessary to stop beta-blocker therapy prior to start- anaphylaxis and make it more di ic lt to treat.
ing immunotherapy.
C) Patients should be observed in the of ce or at least 30 min-
utes a er immunotherapy injections.
D) At least 3 years o immunotherapy should be given to avoid Objectives: Did you learn to . . .
recurrence o symptoms. • Reco nize symptoms o vocal cord dys nction?
• Reco nize symptoms o aller ic rhinitis?
Answer 4.1.5 The correct answer is “B.” It has been shown that • Provide appropriate mana ement or aller ic rhinitis?
patients taking beta-blockers may be at increased risk o having • Describe the role o imm notherapy in treatin aller ic rhinitis?
CHAPTER 4 • ALLERg Y AND IMMu NOLOg Y 133

TABLE 4-1 CLASSIFICATION OF IMMUNOLOGIC REACTIONS


Reaction Type Mechanism Clinical Features Timing
Type I—Immediate Anti en expos re ca ses cross-linka e o I E antibodies Anaphylaxis Less than an ho r
that are bo nd to s r aces o mast cells and basophils, An ioedema a ter expos re
with s bseq ent release o mediators s ch as histamine. Bronchospasm
u rticaria

Type II—Cytotoxic I g or I M antibodies are directed a ainst anti ens on g ra t rejection At least 5 days b t
the individ al’s own tiss es, and s bseq ent complement Hemolytic anemia sometimes many
activation leads to cell destr ction. Ne tropenia weeks a ter expos re
Thrombocytopenia

Type III—Imm ne complex I g or I M anti en–antibody complexes orm and deposit Localized arth s reaction 1 week or more a ter
within blood vessels and tiss es, ca sin complement Ser m sickness expos re
activation and ne trophil recr itment, ltimately res ltin
in tiss e dama e.

Type IV— Delayed Anti en expos re to sensitized T cells ca ses a reaction. Contact dermatitis 24–72 ho rs a ter
Stevens–Johnson syndrome expos re

The correct answer is “A.” Celery is so boring; it can’t even


QUICK QUIZ: ALLERg IC REACTIONS cause an allergic reaction in those with latex allergy! Symptoms
o oral allergy syndrome can include oral pruritus with or with-
A 52-year-old man with common variable immunode ciency out angioedema o the lips, tongue, palate, and posterior oro-
(CVID) receives his rst in usion o intravenous immunoglobu- pharynx. Cross-reactivity has been reported between:
lin (IVIG) therapy. en minutes into the in usion, he complains • Ragweed antigens and the gourd amily and banana.
o dif culty breathing and generalized pruritus. • Birch pollen allergy may result in sensitivity to apple, carrots,
parsnips, celery, hazelnuts, and potatoes.
Based on the traditional classi cation o hypersensitivity
• Latex- ruit cross-reactivity may occur with banana, avocado,
reactions, which o the ollowing best categorizes this patient’s
passion ruit, kiwi, and chestnut, but not celery. Your patient
reaction?
should be warned about these potential reactions.
A) ype I—Immediate hypersensitivity reaction.
B) ype II—Cytotoxic reaction.
C) ype III—Immune complex reaction.
D) ype IV—Delayed-type reaction. HELPFUL TIP:
The vast majority o patients who report penicillin (and
The correct answer is “A.” Based on the clinical history and other) aller ies are not tr ly aller ic. In the case o peni-
timing o the above event, you should suspect an immediate cillin, 0.5% o those with reported penicillin aller y had
hypersensitivity reaction ( ype I). ype I reactions typically skin reactions and less than 10% had any reaction to a
occur within seconds to minutes a er exposure to the o end- ll dose o penicillin. Many patients misinterpret an ad-
ing agent and are due to cross-linkage o IgE antibodies that are verse reaction (s ch as na sea) as an aller y.
bound to sur aces o mast cells or basophils, with subsequent
release o mediators such as histamine. Pruritus, urticaria,
angioedema, laryngeal edema, and possible generalized ana-
HELPFUL TIP:
phylaxis can occur. o see why ypes II to IV are incorrect, re er
There is essentially no cross-reactivity between penicil-
to able 4-1 or de nitions o the immunologic reaction types.
lin and third- eneration cephalosporins. As lon as the
patient did not have tr e anaphylaxis, eel com ortable
QUICK QUIZ: CAN I HAVE THAT BANANA OR NOT? sin these dr s in penicillin-aller ic patients.

A slightly deranged patient o yours has a serious latex allergy


and wants to play “ ruit roulette.”
QUICK QUIZ: CONTRAST ALLERg Y
You tell her that all o the ollowing may be associated with
cross-reactivity in latex allergic patients EXCEP : A 57-year-old man with chest pain is scheduled or an elective
A) Celery. cardiac catheterization. You remember that the patient has a
B) Banana. history o generalized urticaria with lip and tongue angioedema
C) Avocado. shortly a er receiving contrast dye or a C scan several years
D) Kiwi. back.
134 FAMILY MEDICINE EXAMINATION & BOARD REVIEW

Which o the ollowing interventions should be recommended Answer 4.2.1 The correct answer is “B.” Corn is not o en
or this patient prior to undergoing the planned procedure? implicated in ood allergies (a good thing too, since we are writ-
A) Use o higher-osmolality radiocontrast media i possible. ing this in Iowa, surrounded by corn). Although many oods
B) Give a test dose o radiocontrast media and proceed as usual are potentially antigenic, the great majority o ood allergies
i testing is negative. involve only a ew oods. Studies have shown that eight oods
C) Administration o prednisone and diphenhydramine as pre- account or 93% o reactions, and these oods are, in order o
medications. requency: egg, peanuts, milk, soy, tree nuts, sh, crustacean,
D) Percutaneous and intradermal skin testing with radiocon- and wheat. Although these ood allergies may be outgrown,
trast media. sensitivity to peanuts, tree nuts, sh, and crustaceans tend to
E) Desensitization procedure or radiocontrast media. be li e-long.

The correct answer is “C.” Anaphylactoid reactions to radio-


HELPFUL TIP:
contrast material are typically non–IgE-mediated; however,
The peak incidence o ood aller ies occ rs aro nd
they look very similar to a ype I immunologic reaction and
a e 1 year, with most aller ies identi ied by the a e o
can be severe and li e threatening. T is patient will require
2 years. Cow’s milk and e aller ies tend to resolve by
the premedication or this elective procedure. T e appropri-
ad lthood. The 2013 ideline rom American Academy
ate premedication regimen or contrast dye allergy includes
o Aller y, Asthma and Imm nolo y recommended in-
(1) diphenhydramine 50 mg IM/PO 1 hour be ore the pro-
trod ction o hi hly aller enic ood as early as 4 months
cedure and (2) prednisone 50 mg PO 13 hours, 7 hours, and
o a e, which has shown to red ce ood aller y later.
1 hour be ore the procedure. “A” is incorrect. T e use o lower-
osmolality radiocontrast media is associated with ewer adverse
reactions and is appropriate or contrast-allergic patients. “B”
is incorrect. est dosing or radiocontrast media should not wo months later, this patient required an emergency room
be done; patients with no reaction to test doses have had severe visit af er developing increased work o breathing, wheezing,
reactions to ull doses. In addition, test doses themselves have and an urticarial rash af er eating. You re er him or testing.
been reported to result in atalities. Skin testing (“D”) can He is tested by both percutaneous skin testing and RAS and
be done, especially within a window o a ew months a er the is ound to have egg allergy.
reaction. In the past, skin testing was not recommended, but its
role is evolving, and may become available in the near uture. Question 4.2.2 Which o the ollowing vaccinations should
T ere is no desensitization procedure available. NOT be given to this patient in light o his egg allergy?
A) MMR vaccine.
B) Inactivated polio vaccine.
HELPFUL TIP: C) Haemophilus inf uenzae B vaccine.
There is no cross-reaction between shellfish and D) Conjugated pneumococcal vaccine (Prevnar).
iodine-based contrast material. The aller y to sea- E) raditional in uenza vaccine.
ood is an aller y to the protein (tropomyosins) in the
sea ood, not the iodine. Think abo t it or a second. Answer 4.2.2 The correct answer is “E.” raditional in uenza
How many people have yo seen droppin dead rom vaccine should not be given to individuals with egg allergy. O
an anaphylactic reaction to iodized salt? particular note is “A.” “A” is incorrect. It had been common prac-
tice to withhold measles vaccination rom children with a his-
tory o anaphylactic reaction to egg; the measles vaccine is pre-
pared in chick-embryo broblast cultures. However, the MMR
is sa e to use in children with an egg allergy. Options “B,” “C,”
CASE 4.2 and “D” do not contain egg-related products and can there ore
An 18-month-old boy comes to clinic with a history o eczem- be given sa ely to this patient.
atous rash covering his extremities and ace. His parents state
that it worsens af er the ingestion o certain oods. He has HELPFUL TIP:
had increased ussiness over the last several months, as well There are two preparations o e - ree in l enza
as some di culty gaining weight. Food allergy is suspected. vaccine licensed in the u nited States. These are
Fl celvax®and Fl blok®. These are pre erred in e -
Question 4.2.1 Which o the ollowing oods is NOT com- aller ic patients.
monly implicated in ood allergy?
A) Milk.
B) Corn. Objectives: Did you learn to . . .
C) Wheat. • Identi y common ood aller ens?
D) Soy. • Reco nize important associations between ood aller y and
E) Egg. selected vaccines?
CHAPTER 4 • ALLERg Y AND IMMu NOLOg Y 135

use o medications, especially oral contraceptives and ACE


CASE 4.3 inhibitors. Interestingly, H. pylori in ection has been associated
A 34-year-old emale establishes care in your clinic. Her with HAE, and H. pylori eradication reduces the requency o
medical history consists o intermittent colicky, abdominal angioedema attacks.
pain, and episodes o angioedema in the past with unclear
etiology. She has not noticed urticaria with those episodes. Your patient is ultimately diagnosed with HAE.
She has received multiple laparoscopic procedures that were
unrevealing. During the initial interview, she relates several Question 4.3.3 In this particular patient, which medication
past episodes o lip and tongue swelling, or which she has would be most help ul in preventing uture episodes?
not sought medical assistance. She has tried diphenhydr- A) Emergency epinephrine.
amine without signi cant improvement. T e swelling epi- B) Oral diphenhydramine.
sodes resolve without intervention af er 3 to 4 days. C) Fresh rozen plasma (FFP).
D) Androgens (e.g., danazol).
Question 4.3.1 Which o the ollowing laboratory tests E) No prophylactic treatment is available.
would be most help ul in establishing the diagnosis in this
patient? Answer 4.3.3 The correct answer is “D.” Emergency epineph-
A) ANA. rine and antihistamines have not generally been e ective, as
B) C3 complement level. HAE is not allergic in nature. Prophylactic treatment in the
C) C4 complement level. orm o attenuated androgens (danazol) is available. Attenu-
D) Complete blood count (CBC). ated androgens appear to work by up-regulating the synthetic
E) SS-A and SS-B. capability o hepatic cells that make C1-esterase inhibitor,
thus decreasing the e ect o C1 esterase and raising the C4
Answer 4.3.1 The correct answer is “C.” T is patient’s presen- level. T is, in turn, reduces the number and severity o acute
tation should raise concern or hereditary angioedema (HAE), exacerbations.
or C1-esterase inhibitor de ciency. T is entity is clinically char- wo puri ed C1-esterase inhibitor replacement protein
acterized by recurrent episodes o angioedema involving any products (Cinryze® and Berinert®) were approved by the FDA
part o the body. T us, it can present with laryngeal angioedema in 2010 or the treatment and prevention o HAE exacerba-
(the major cause o death) or recurrent abdominal pain (gener- tions. However, they are costly and lack strong ef cacy data.
ally with a normal white count and “always” without peritoneal reatment o acute episodes o HAE includes FFP that contains
signs). Urticaria is not a eature o this disorder. HAE is caused C1-esterase inhibitor, the C1-esterase inhibitor replacement
by a C1-esterase inhibitor de ciency; C1-esterase is a protease protein products, and bradykinin/kallikrein inhibitors (ecal-
that normally acts on C4. I there is an absence o C1-esterase lantide and icatibant). In addition, these three options could be
inhibitor, C4 levels will be low as C1-esterase will continue to used as prophylactic treatment in short-term situations, such as
degradate C4. It is possible that a patient with HAE will have a prior to surgery in a high-risk patient. However, FFP would not
normal level o C1-esterase inhibitor but will have a non unc- be a long-term preventive measure, so “C” is incorrect.
tional allele. So always check a C-1 esterase inhibitor level and
C-1 esterase activity as well as a C4 level. I there is a non unc-
tional allele, the C-1 esterase inhibitor level will be normal but HELPFUL TIP:
C-1 esterase inhibitor unction will be decreased. Make s re patients with HAE receive the hepatitis B vac-
cination series as they may req ire blood prod cts or
While awaiting the test results, you discuss HAE with your ac te treatment o an ioedema.
patient.
Objectives: Did you learn to . . .
Question 4.3.2 In order to minimize uture attacks o angio- • Identi y a patient presentin with hereditary an ioedema?
edema, you recommend which o the ollowing? • Mana e a patient with hereditary an ioedema?
A) Avoidance o estrogen containing medications.
B) Avoidance o ACE inhibitors.
C) esting and treatment o Helicobacter pylori. CASE 4.4
D) All o the above. Your patient’s sister (her biologic sister, not her step-sister or
BFF) presents to clinic the next day with a similar history o
Answer 4.3.2 The correct answer is “D.” Education is the cor- angioedema symptoms, only less requent and less severe.
nerstone o treatment o HAE. Patients need to know what
triggers to avoid and how to identi y symptoms early. Most Question 4.4.1 What is the likelihood that she has HAE as
attacks are precipitated by trauma (o en head and upper air- well?
way trauma), medical procedures (dental and oral surgery in A) In nitesimally small because HAE is a rare disorder.
particular), emotional stress, in ections, menstruation, or the B) About 25% because HAE is autosomal recessive.
136 FAMILY MEDICINE EXAMINATION & BOARD REVIEW

C) About 50% because HAE is autosomal dominant. Warning: T e next section is about immunode ciency syn-
D) Almost 100% because you can tell that she’s got bad luck. dromes. As Dante would say: “Abandon hope all ye who enter
E) Unknown because multiple gene involvement renders a here.” (One o the editors has a dog named Dante and she [the
simple calculation impossible. dog] says this all the time . . . to squirrels). See able 4-2 or a
quick review.
Answer 4.4.1 The correct answer is “C.” HAE is transmit- T e main points and the most important cases are men-
ted in an autosomal dominant ashion with incomplete pen- tioned below. For those o you who are gluttons or punishment
etrance, which may be why the patient’s sister is less symp- (gluttony is only Dante’s 3rd circle), look at the table.
tomatic. New mutations do arise and cause a minority o T e main points:
cases. • Rule out the obvious: Does the patient have HIV, cancer, dia-
betes, lupus, or other chronic disease or is he/she on immu-
Later that same night, you are called to the ED to see a nosuppressing drugs that predispose to recurrent in ections?
patient with acial swelling. “Really?” you ask. “Yeah, • 1% to 3% o patients are heterozygous or an IgG subtype
really. Get in here,” replies Happy Nurse. T is patient is a de ciency, the most common type o immunoglobulin de -
35-year-old emale riend o your patient with HAE (so, ciency. T ese patients may be asymptomatic or may present
her BFF not her biologic sister). She is worried that she too with recurrent sinusitis, otitis, skin in ections, etc. IgG de -
has HAE. T is is the rst ever episode o lip and ace swell- ciency can be diagnosed by immunoglobulin electrophore-
ing, associated with itching in her mouth and a ew hives sis. T ese patients o en do not have an appropriate response
on her arm and torso. She had shrimp gumbo or dinner, to vaccinations.
but has never had a reaction to shrimp be ore. She denies • 1 in 700 patients are IgA de cient. Most are hereditary, but
any recent trauma, surgery, or medication use. She is in no
it can also be caused by several drugs (including captopril
respiratory distress. Her O2 sat is 99% on room air and BP
and thyroxine). Usually this resolves upon stopping the
is 118/76 mm Hg. On lung examination, there is no stridor
drug. Most patients with IgA de ciency are asymptomatic,
or wheezing.
although some have recurrent sinus and GI in ections. Also,
these patients requently develop autoimmune disease and
Question 4.4.2 The most appropriate next step in the treat- may have anaphylaxis to blood products.
ment o this patient is:
• Symptomatic immunoglobulin de ciencies can be treated
A) Supportive care and monitoring overnight in the intensive
with IVIG. T e most common side e ects o IVIG admin-
care unit (ICU).
istration include renal ailure, anaphylaxis, and thromboem-
B) Administration o FFP.
bolic disease.
C) Administration o epinephrine IM × 1.
D) Administration o diphenhydramine, cimetidine, and pred-
nisone PO. CASE 4.5
E) Prophylactic intubation.
A 35-year-old male is seen by you in the hospital setting, af er
being admitted or pneumonia. Blood cultures reveal Strep-
Answer 4.4.2 The correct answer is “D.” T is patient is di - tococcus pneumoniae. T e patient was well until age 25 when
erent rom your HAE patient. It is unlikely that she has HAE he began having recurrent in ections and developed an auto-
or several reasons: she’s never had this reaction be ore, she immune hemolytic anemia. He also relates requent sinus
has urticaria associated with the angioedema, and she did not in ections (real ones . . . not the kind we so of en see) requir-
experience an inciting event. T is episode o angioedema and ing antibiotics 8 to 10 times a year. His last bout o pneumo-
urticaria is more likely to have been caused by a typical aller- nia required a stay in the ICU.
gic reaction and mast-cell release. For these types o reactions,
antihistamines and steroids are the mainstay o therapy. How-
Question 4.5.1 You suspect an immunodef ciency in this
ever, the ef cacy o steroids, antihistamines, and epinephrine is
individual. What is the most use ul test in making the diag-
limited in angioedema (but it is worth a try). “A” is incorrect
nosis in this patient?
because it is overkill to admit her to the ICU, and you should
A) Complement levels.
be treating her rather than just monitoring her. “B” is incorrect
B) Immunoglobulin levels.
because FFP treats HAE not allergic angioedema. “C” is incor-
C) CBC and di erential.
rect because this patient is not having a ull anaphylactic reac-
D) Bone marrow biopsy.
tion. Epinephrine would be appropriate i she had wheezing,
E) Nitroblue tetrazolium test.
airway compromise, or hypotension. I you chose “E” or a walk-
ing, talking patient with no airway compromise, well . . . that just
Answer 4.5.1 The correct answer is “B.” T e clinical picture or
makes us sad.
this patient is most consistent with common variable immune
Objectives: Did you learn to . . . de ciency (CVID). CVID, or acquired agammaglobulinemia,
• Di erentiate HAE rom aller ic an ioedema? is similar to X-linked agammaglobulinemia, but generally has
• Mana e an anaphylactoid reaction? a later age o onset. In addition, it is associated with various
CHAPTER 4 • ALLERg Y AND IMMu NOLOg Y 137

TABLE 4-2 IMMUNODEFICIENCY SYNDROMES


De ect/Laboratory Organisms Likely to
Syndrome Age o Onset Findings Mani estations Cause In ection
Humora l
X-linked Late rst year o li e b t Low I g , almost M ltiple, rec rrent in ections Streptococcus,
a amma lob linemia p to a e 50 (in very mild ndetectable I M, ( enerally start at 6–18 months) Haemophilus, Giardia
disease) I D, I E, and I A. No especially l n , sin ses, ears,
B cells CSF. Event ally bronchiectasis
and p lmonary ins ciency

Common variable Variable b t >2 years o Low levels o ser m Sin s and respiratory in ections. Pneumococcus,
imm node ciency a e and mostly by a e 30. imm no lob lins g I in ections especially Giardia. Haemophilus,
May be older Enhanced chance o lymphoma Mycoplasma, Giardia
as ad lt. A toimm ne disorders

Hyper I M syndrome First 2 years o li e Elevated I M, no I g , Rec rrent severe sinop lmonary Encapsulated bacteria,
I A, I E (altho h ew in ections. A toimm ne Giardia, Pneumocystis,
may have very low disorders Cryptosporidium,
level I A, I E) Histoplasmosis

I A de ciency Not be ore a e o 6 Low I A levels, normal Most asymptomatic. May have Haemophilus,
months; more severe imm no lob lins respiratory tract in ections. Pneumococcus
disease presents by a e 5. otherwise Watch or anaphylaxis with
blood prod cts

Cellular
Myeloperoxidase Variable Poor pha ocytic None except in the presence o Candida
de ciency killin other de ects (e. ., diabetes),
systemic candidiasis

Chronic In ant, toddler b t Ne trophil Rec rrent li e-threatenin Staphylococcus,


ran lomato s occasionally in late li e dys nction meas red illnesses especially p lmonary, Aspergillus
disease by nitrobl e hepatic, skin, and lymphatic
tetrazoli m testin abscesses
(and other tests)

Le kocyte adhesion Early (poor separation o Poor adhesion Periodontal and dental Pseudomonas and other
de ciency mbilical st mp) or later o le kocytes to in ections, rec rrent in ections g ram-ne ative rods,
i mild disease. Those with endotheli m, etc. o skin, pper and lower airways, Staphylococcus
mild disease rarely have bowel, perirectal area
li e-threatenin illnesses

Hyper I E (Job First weeks to month Elevated I E levels, Facial abnormalities M ltiple or anisms, b t
syndrome) o li e poor le kocyte (hypertelorism; prominent, especially Staphylococcus,
chemotaxis protr din trian lar mandible; Haemophilus, Candida
broad, somewhat b lbo s
nose). Eczema, m coc taneo s
candidiasis, sin s, p lmonary,
and skin in ections.
Rec rrent “cold abscesses”
in skin secondary to lack o
inf ammation

Wiskott–Aldrich Early. Fatal by a e 10 Low I M, dimin tion Eczema, thrombocytopenia with Encaps lated or anisms
witho t bone marrow o cell lar imm nity p rp ra, rec rrent in ections
transplant ability to respond
to polysaccharide
caps les

Severe combined Early, by 6 months. Death Le kopenia, no mat re Colon and l n in ections M ltiple or anisms,
imm node ciency by 2 years T cells, low ser m incl din diarrhea, abscesses incl din viral
syndrome imm no lob lin levels

(continued)
138 FAMILY MEDICINE EXAMINATION & BOARD REVIEW

TABLE 4-2 IMMUNODEFICIENCY SYNDROMES (Continued)


De ect/Laboratory Organisms Likely to
Syndrome Age o Onset Findings Mani estations Cause In ection
Dig eor e syndrome Early—in in ancy Absent thym s, Rec rrent in ections b t Pneumococcus,
Red ced T3 + cells variable in penetrance. Some Haemophilus
have normal or near-normal
imm ne nction. Cranio acial
abnormalities, con enital
heart disease, chromosomal
abnormalities, hypocalcemia

gastrointestinal disorders, autoimmune disorders, and malig-


nancy. Immunoglobulin levels are decreased secondary to QUICK QUIZ: I A DEFICIENCY
inadequate B-cell di erentiation. T ere ore, the most use-
ul laboratory test or the diagnosis o CVID would be serum A 37-year-old emale is incidentally ound to have an IgA level
immunoglobulin levels. With regard to the other options and o 3 mg/dL (below the normal range).
other tests you might do to evaluate otherwise unexplained
immunode ciency: What is the most likely clinical picture in this patient?
• Immunode ciency can be secondary to complement disor- A) Pyogenic in ections.
ders and complement levels will be low in these patients. B) T rush.
• Nitroblue tetrazolium test will be abnormal in patients with C) Cold abscesses.
phagocytic disorders. D) Aphthous ulcers.
• Response to vaccines will be muted or absent in humoral im- E) No clinical abnormalities.
munode ciency (e.g., immunoglobulin de ciency).
The correct answer is “E.” IgA de ciency is present in approxi-
• Delayed hypersensitivity skin testing (e.g., candida and
mately 1 in 700 Caucasians in the United States and is the sec-
mumps) may indicate a -cell de ect. ond most common immunode ciency described. Most o these
patients are asymptomatic, but some IgA-de cient patients may
Question 4.5.2 Which o the ollowing is the most appropri- present with an increased rate o respiratory tract in ections.
ate treatment plan or this patient?
A) Prophylactic antibiotics.
HELPFUL TIP:
B) Bone marrow transplantation.
The eval ation o imm node iciency sho ld NOT stop
C) Gene therapy.
when I A de iciency is discovered since I A de iciency
D) IVIG replacement.
is o ten asymptomatic and may be accompanied by a
E) No treatment needed.
more serio s orm o imm node iciency.
Answer 4.5.2 The correct answer is “D.” T e treatment o
choice or CVID includes replacement IVIG, especially in this
patient who has a history o li e-threatening in ections. Prophy- QUICK QUIZ: IVIg
lactic antibiotics may be required in addition to IVIG in some
patients. Gene therapy is not currently possible because the Which o the ollowing patients is most likely to develop an ana-
genetic de ect has not been identi ed. Although bone marrow phylactic reaction in response to the administration o IVIG?
transplantation is use ul in other immunode ciency states, it is A) A patient with IgA de ciency.
not indicated in CVID. B) A patient with IgG de ciency.
C) A patient with sickle cell anemia.
D) A patient with gra -versus-host disease.
HELPFUL TIP: E) A pregnant patient.
Patients with CVID may develop lymphoproli erative
disorders (e. ., non-Hod kin lymphoma). New adenop- The correct answer is “A.” Patients with IgA de ciency have
athy sho ld be taken serio sly. pre- ormed IgG or IgE antibodies against IgA, and thus may
develop an anaphylactic reaction in response to IVIG (and
blood and FFP, or that matter). Patients with IgG de ciency
Objectives: Did you learn to . . . may have a similar problem but it is much less common. Not all
• Describe presentin symptoms o common variable imm - patients with IgA de ciency will develop anaphylaxis, but the
node ciency? risk is high enough to be prepared and to reduce the risk by
• Reco nize treatment options or CVID? using IgA-depleted IVIG and giving the IVIG slowly.
CHAPTER 4 • ALLERg Y AND IMMu NOLOg Y 139

BIBLIOGRAPHY
Clinical Pearls Azar AE. Evaluation o the adult with suspected immunode -
An extensive eval ation or an nderlyin ca se o chronic ciency. Am J Med. 2007;120(9):764.
rticaria is very low yield and sho ld not be done. Tar et the Bowen , et al. 2010 International consensus algorithm or the
eval ation to the patient. diagnosis, therapy and management o hereditary angio-
Beta-blockers sho ld be sed with ca tion in a patient who edema. Allergy Asthma Clin Immunol. 2010;6(1):24.
has had a severe anaphylactic reaction (e. ., req irin epi- Calderon MA, et al. Allergen injection immunotherapy or sea-
nephrine). Beta-blockers increase the incidence and severity sonal allergic rhinitis. Cochrane Database Syst Rev. 2007,
o anaphylaxis and bl nt the response to therape tic doses Issue 1. Art. No.: CD001936.
o epinephrine. Cooper MA. Primary immunode ciencies. Am Fam Physician.
Do not se nontar eted I E testin in the eval ation o 2003;68(10):2001–2008.
asthma. Do testin or speci ic anti ens as indicated based Fleischer DM, et al. Primary prevention o allergic disease
on the patient’s history. through nutritional interventions. J Allergy Clin Immunol
I A imm node iciency is o ten asymptomatic and sho ld
In Pract. 2013;1(1):29–36.
not be considered the ca se o imm node iciency nless no Quillen DM. Diagnosing rhinitis: Allergic versus non-allergic.
other ca se is o nd. Am Fam Physician. 2006;73(9):1583–1590.
Intranasal steroids are irst line or aller ic rhinitis and will
Scurlock AM. Food allergy in children. Immunol Allergy Clin
also improve aller ic eye symptoms (when iven intrana-
North Am. 2005;25(2):369–388, vii–viii.
sally).
It is enerally sa e to se third- eneration cephalosporins in
a patient with penicillin aller y—as lon as the reaction was
not anaphylaxis.
RAST testin or aller y has a 25% alse-ne ative rate. Skin
testin remains the old standard.
The reat majority o patients who are reportedly “aller ic” to
penicillin do not have a tr e penicillin aller y.
There is no cross-reactivity between shell ish and iodinated
radiocontrast.
Vocal cord dys nction is an asthma mimic. Patients s ally
have inspiratory stridor.
Nephrology
Stac A n im and Jas n K. Wi bu
5
CASE 5.1 HELPFUL TIP:
N t n t min . T t m “mic a buminu ia” as
A 49-year-old, emale with a 5-year history o diabetes melli-
b n ac d wit “m d at inc as d a bumin-
tus type 2, presents or an initial visit. She has no known com-
u ia,” but t d initi n mains t sam (30–300 m
plications o diabetes. She takes met ormin, glyburide, and
t in in t u in 24 u s). C m a t is t “s -
aspirin. On examination, you nd a pleasant, obese emale
v inc as d a buminu ia,” d in d as at t an
in no distress. Her blood pressure is 136/86 mm Hg. As you
300 m t in in t u in 24 u s.
discuss monitoring her diabetes, you recommend screening
or early kidney disease.

Question 5.1.1 Which o the ollowing ap p roaches is Question 5.1.2 Her albumin/creatinine ratio is 42 mg/g. The
the recommended way to screen or diab etic kidney next step to con rm moderately increased albuminuria is:
disease? A) Repeat urine albumin/creatinine ratio.
A) Obtain a 24-hour urine collection or albumin now and B) Urine dipstick or protein.
again in 3 years. C) 24-hour urine collection or total protein excretion.
B) Obtain a spot urine albumin every year. D) Serum creatinine.
C) Obtain a spot urine albumin/creatinine ratio every year. E) Re erral to a nephrologist.
D) Obtain a urinalysis every year.
E) Obtain a serum creatinine every year. Answer 5.1.2 The correct answer is “A.” Veri cation by repeat
urine albumin/creatinine ratio is su cient or a diagnosis o
Answer 5.1.1 The correct answer is “C.” Moderately increased moderately increased albuminuria, so 24-hour urine collections
albuminuria (previously known by the misnomer “microalbu- need not be per ormed or con rmation. O note, the diagnosis
minuria”) is a marker or increased risk o uture kidney dis- o moderately increased albuminuria requires 2 o 3 urine speci-
ease in diabetic patients. T e best test to evaluate or moderately mens showing > 30 mg/g albumin/creatinine over a 6-month
increased albuminuria is the urine albumin/creatinine ratio. period. Since protein excretion must exceed 300 to 500 mg/
Its advantages include ease o use, relatively low cost, and good day or a urine dipstick to detect proteinuria, urinalysis (“B”) is
correlation with 24-hour urine collections. Some o you may not sensitive enough to be diagnostic. Serum creatinine eleva-
have chosen “B.” A “spot microalbumin” (now a spot albumin) tion may be a marker or diabetic kidney disease, but it would
is a common but less accurate way to provide screening and may develop late in the process. Nephrology re erral is premature—
still be in use in some areas. As a practical matter, many physi- you went to medical school; you can do this!
cians use urine albumin alone as a method o screening, but this
method does not allow or corrections or variations in urine Question 5.1.3 Which o the ollowing can cause a alse -
volume and dilution. A random spot urine albumin/creatinine negative albumin/creatinine ratio?
ratio is normally less than 30 mg/g. Values above 30 mg/g are A) Vigorous exercise.
consistent with 24-hour measures showing abnormal amounts B) Fever.
o albumin. Answers “D” and “E” o er measures o kidney unc- C) Cachexia.
tion that simply are not sensitive enough to use or screening D) Poor glycemic control.
purposes. E) Large muscle mass.

140
CHAPTER 5 • Neph r o l o g y 141

Answer 5.1.3 The correct answer is “E.” Patients with a large


HELPFUL (AND VERY IMPORTANT) TIP:
muscle mass have a high rate o creatinine excretion, which
Ar Bs a n t just an ACe in ibit wit ut t c u .
may result in a alsely negative albumin/creatinine ratio (as
A a m ta-ana sis s w d t at Ar Bs, w i w -
the urine creatinine goes up, the ratio obviously goes down).
in b d ssu , av n a ciab ct n MI
Cachectic patients have the opposite problem, with low
ca di vascu a m ta it versus pla cebo (BMJ. 2011;342:
amounts o creatinine excretion, resulting in alse-positive
d2234; JAMA Int n M d. 2014;174:773). But t at’s a t
albumin/creatinine ratio. Fever, vigorous exercise, heart ail-
dis as in diab tics (w ic is kind im tant). Back t
ure, and poor glycemic control can cause transient increases
na dis as . . . T n stud (t dat ) t di ct c m-
in albuminuria, potentially resulting in alse-positive albumin/
a an ACe in ibit ( na a i ) t an Ar B (t misa tan)
creatinine ratios.
na t cti n in diab t s did n t d m nst at
a statistica si ni icant di nc b tw n t tw ,
Your patient’s other laboratory studies reveal the ollow-
at u t was a t nd in av t ACe in ibit
ing: hemoglobin A1c 6.4%, serum creatinine 1.4 mg/dL, and
(N en J M d. 2004;351(19):1952). h w v , t is was a
normal electrolytes. A month later, your patient returns.
sma stud (250 individua s) wit a i d ut at .
Her blood pressure is 138/84 mm Hg. Her urine albumin/
An t stud 4,000 subj cts s w d t at w i Ar Bs
creatinine remains elevated on a second measurement.
d duc t inu ia, t av n b n it n m ta -
According to an eye examination yesterday, she has nonpro-
it (N en J M d. 2011;364:907), w ic is w at w a
li erative diabetic retinopathy.
ca ab ut.

Question 5.1.4 Because your patient has type 2 diabetes


mellitus and moderately increased albuminuria, you real-
ize that her likelihood o progressing to overt nephropathy T e patient has a ull urinalysis to rule out renal in amma-
is: tion (e.g., nephritis) and overt proteinuria (nephrotic syn-
A) Almost zero. drome). T e urinalysis is entirely negative.
B) About hal that o a similar patient with type 1 diabetes.
C) Nearly equal to that o a similar patient with type 1 diabetes. Question 5.1.6 What urther investigations must your
D) More than twice that o a similar patient with type 1 diabetes. patient undergo to eliminate other potential causes o
E) Absolutely certain (100% chance). proteinuria?
A) Renal biopsy.
Answer 5.1.4 The correct answer is “C.” Although earlier stud- B) Renal ultrasound with Doppler o the renal arteries.
ies showed a greater progression to overt nephropathy in type C) ANA, ESR, CRP.
1 diabetics, more recent studies demonstrate a nearly equal rate D) All o the above.
o progression in types 1 and 2. About 20% to 40% o Caucasian E) None o the above.
patients with diabetes type 2 and moderately increased albu-
minuria will progress to diabetic nephropathy. T e rate o pro- Answer 5.1.6 The correct answer is “E.” No urther evaluation
gression to nephropathy in non-Caucasian populations is even is necessary in this patient with moderately increased albumin-
higher. uria. T e combination o diabetic retinopathy (a marker or
diabetic renal disease), hypertension (BP > 140/90 mm Hg in
Question 5.1.5 What is the most appropriate next step in a diabetic patient), and abnormal protein in the urine as mea-
the evaluation and management o this patient’s moder- sured by the urine albumin/creatinine ratio is su cient to make
ately increased albuminuria? the diagnosis o early diabetic nephropathy. Renal biopsy is
A) Start an angiotensin-converting enzyme (ACE) inhibitor. quite invasive and unlikely to change management. ANA, ESR,
B) Start an angiotensin receptor blocker (ARB). CRP, and ultrasound are unlikely to o er new in ormation. I
C) Order renal ultrasound with Doppler o the renal arteries. things change (e.g., nephritic urine and gross proteinuria), ur-
D) Start insulin. ther evaluation may be indicated.
E) Order a 24-hour urine collection or total protein.
You continue to ollow this patient or several years. She ulti-
Answer 5.1.5 The correct answer is “A.” ACE inhibitors should mately is admitted or chest pain, rules out or myocardial
be the rst-choice drugs unless there is a contraindication to in arction, but has a positive stress test. She will need to have
their use. “B,” an ARB, should be second-line choice in the event a cardiac catheterization. Her creatinine is now 1.5 mg/dL,
that the patient cannot tolerate an ACE inhibitor. “C,” a renal and her glomerular ltration rate is in the range o stage 3
ultrasound, is not indicated at this point in time. “D” is also chronic kidney disease.
incorrect. Your patient already has good glucose control (HbA1c
o 6.4%). As previously stated, a 24-hour urine collection is not Question 5.1.7 In addition to holding her met ormin,
necessary or diagnosis. which o the ollowing interventions would be most
142 FAMIl y MeDICINe eXAMINATIo N & Bo Ar D r eVIeW

likely to reduce her risk o developing contrast-induced


HELPFUL (AND VERY IMPORTANT) TIP:
nephropathy?
T us Mr I c nt ast ( ad inium) in ati nts wit -
A) N-acetylcysteine and IV saline.
na dis as as b n ass ciat d wit a sc d ma- ik
B) N-acetylcysteine and mannitol.
s nd m (n nic s st mic ib sis). g ad inium
C) IV saline.
s u d b av id d in t s wit a C C < 30 ml /min.
D) Sodium bicarbonate and mannitol.
T data a ss c m in in t s wit a C C 30
E) Mannitol and IV saline.
t 60 ml /min. Sinc n nic s st mic ib sis can
a ct m t an just t kidn s, a dia sis ma
Answer 5.1.7 The correct answer is “C.” For patients at risk
v nt t is dis as b ducin t a -i t
o contrast-induced nephropathy, contrast studies should be
ad inium.
avoided i possible (see help ul tip below). I a contrast study
Fu t , it as c nt b n disc v d t at at-
must be done, stop aggravating medications like nonsteroidal
d us Mr I c nt ast ma main in t CNS and m
anti-in ammatory drugs (NSAIDs-–this patient shouldn’t be
d sits. W at a ct t is as n c niti n, tc. is un-
on these anyway since she has CKD!). Hydration, usually with
kn wn, but is b in inv sti at d b t FDA. FDA M d-
IV saline, should be given i there are no contraindications (e.g.,
Watc su sts imitin t us Mr I c nt ast ( tt ://
heart ailure). Nonionic lower osmolality (or even iso-osmolar)
www. da. v/Sa t /M dWatc /Sa t In mati n/
contrast agents should be used, and we recommend discussing
Sa t A ts h umanM dica p ducts/ucm456012.
these cases individually with your radiologist, cardiologist or
tm).
whomever is per orming the procedure.
Sodium bicarbonate and N-acetylcysteine do not have good
evidence or e ectiveness, but they also do not appear to do
harm. Sodium bicarbonate (isotonic sodium bicarbonate solu- Upon cardiac catheterization, she is ound to have several
tion at 3 mL/kg or 1 hour be ore the procedure and continue lesions. She undergoes coronary artery bypass gra ing and is
at 1 mL/kg or 6 hours af er the procedure) can be used as cur- discharged. Her creatinine remains stable at 1.5 mg/dL.
rent evidence indicates that it has similar e cacy to IV saline. A ew months later, she presents with gradually increasing
N-acetylcysteine needs to be started the day be ore the proce- dyspnea and cough. Her urine output is reported to be normal.
dure and the evidence is con icting. T e diuretics (mannitol, Her vitals show temperature 37°C, pulse 76 bpm, respiratory
urosemide, etc.) may be associated with an increased risk o rate 24, and blood pressure 92/46 mm Hg. You note crackles
nephropathy, and mannitol in particular has an undesirable at both lung bases. Her heart rhythm is regular and an S4 is
side e ect pro le. audible. She has JVD o 9 cm and 2+ pitting pretibial edema.
T e ECG shows sinus tachycardia but no evidence o
potassium toxicity (peaked -waves). Laboratory results:
HELPFUL TIP: troponin- and CK normal, BUN 70 mg/dL, Cr 2.0 mg/dL
T d t ac in ab ut na t xicit and i dinat d c n- (her baseline was 1.5 mg/dL), Na 128 mEq/L, K 5.5 mEq/L
t ast as c nt b n ca d int qu sti n (r adi . (re erence range 3.5–5.0), HCO3 19 mEq/L, WBC 14,500 per
2014;273(3):714–725; r adi . 2014;271(1):65–73). mm3, remainder o the CBC is normal. Urinalysis shows pro-
T s w d n a tic s av b n am c an s. D tein and glucose and a speci c gravity > 1.030, but there are
n t sta t usin c nt ast wit wi d aband n in na dis- ew cells and no casts. Cultures and a chest x-ray are pending.
as . h w v , i it is a n d d, it s ms t at t isk
na inju is w. o n cav at; w ati nts in t s Question 5.1.8 You suspect that her elevated creatinine is
studi s ad a c atinin > 3 m /dl . primarily due to which o the ollowing processes?
A) Adverse toxic e ects o drugs on the kidney.
B) Heart ailure or other prerenal cause o renal ailure such as
HELPFUL TIP: dehydration.
h m i t ati n and m dia sis av b n studi d C) Sudden progression o diabetic nephropathy.
in t v nti n c nt ast-induc d n at D) Urinary obstruction.
ati nts at isk. T d sn ta a t b a b n- E) Urinary tract in ection.
it v m c ns vativ a ac s. A s , t is
a m v t dia z ati nts w a a ad n dia sis Answer 5.1.8 The correct answer is “B.” Your patient has clini-
wit in 24 u s a c nt ast stud . T is n d cal evidence o heart ailure (HF), including edema and rales.
data, w v , and studi s s w t ntia a m. T i With a BUN/Cr ratio greater than 20 and an elevated urine-
kidn s a a ad s t . . . t a n dia sis. W at speci c gravity, she appears to have a prerenal azotemia, likely
additi na a m can u d t t i kidn s? (T is secondary to insu cient cardiac output. Drugs (“A”) may play
s m a um nt ati nts w c ntinu t duc a role in her dehydration, but the BUN/Cr ratio o > 20 argues
u in d s it avin nd-sta na dis as and b in against a renal cause o her elevated creatinine. T ere ore, the
n m dia sis). most likely culprit is HF. Diabetes should not cause a sudden
worsening o renal disease. Although it has not been eliminated
CHAPTER 5 • Neph r o l o g y 143

as a cause, urinary obstruction (“D”) is not likely since she has


HELPFUL TIP:
good urine output. Answer “E” is incorrect since the urinalysis
In a ati nt wit ka mia and eCg c an s ( a
does not support diagnosis o in ection, and CHF explains the
tassium 7.5 m /dl ), it w u d b tim a u c ut
overall clinical picture much better.
ss: ca cium, insu in, uc s , s dium st n su -
nat (Ka xa at ), n bu iz d a but , tc. N t t
HELPFUL TIP: abs nc bica b nat . It ik d sn’t w k and its us
r m mb t at a BUN/C ati > 20 n a (but n t as a n ut av . Fina , d n’t us Ka xa at wit
a wa s!) indicat s a na caus az t mia. T s s bit and b ca u usin v n ain Ka xa at in
caus s inc ud d d ati n and na usi n individua s wit g I d s uncti n. Ka xa at as b n as-
(s ck suc as s sis, h F, and t nsi n). A BUN/ s ciat d wit , and is ik t caus ,b w a-
C ati < 20 is su stiv int insic na dis as ti n (J Am S c N . 2010;21:733).
u ina ut t bst ucti n. T s b ad n a izati ns
a t adu ts n —us c inica jud m nt. Question 5.1.11 Because her creatinine clearance is low
(and she is in HF) you should also discontinue:
A) Insulin.
Question 5.1.9 Given her diabetes and renal disease, which B) Metoprolol.
o the ollowing is the most likely cause o her hyperkale - C) Met ormin.
mia? D) Aspirin.
A) Renal tubular acidosis (R A) type 1. E) Simvastatin.
B) R A type 2.
C) R A type 3. Answer 5.1.11 The correct answer is “C.” Your patient’s cre-
D) R A type 4. atinine increased rom 1.5 to 2.0 mg/dL, corresponding to a
E) R A type 5. 33% reduction in creatinine clearance—a rough measurement
o glomerular ltration rate (GFR). When creatinine clearance
Answer 5.1.9 The correct answer is “D.” R A type 4 is due to decreases, patients on met ormin are at higher risk o devel-
aldosterone de ciency or resistance to the activity o aldoste- oping the rare adverse e ect o lactic acidosis. T ere is some
rone. T e most common cause o R A type 4 is hyporeninemic evidence that patients with mild HF or slight elevations in creat-
hypoaldosteronism, which is of en seen in diabetic nephropa- inine can sa ely take the drug, but stopping met ormin in renal
thy. T e disorder is recognized by hyperkalemia and mild aci- ailure continues to be the standard o care.
dosis. R A types 1 and 2 usually are hypokalemic and these
orms o R A are not associated with diabetes. R A type 3 is HELPFUL TIP:
a rare autosomal-recessive disorder. R A type 5 does not exist. M t min is t n disc ntinu d in diab tic ati nts
wit mi d-t -m d at na dis as . As t manu-
Question 5.1.10 In an ef ort to reduce her serum potassium actu , m t min is c nt aindicat d in m n wit s -
level (not temporize), you should do which o the ollowing? um c atinin ≥ 1.5 m /dl and in w m n wit s um
A) emporarily hold her urosemide and ACE inhibitor. c atinin t ≥ 1.4 m /dl . T is is du t t isk actic
B) Increase her urosemide and temporarily hold her ACE inhi- acid sis. h w v , t isk actic acid sis is xt m
bitor. sma and d s n t v n s m t b at d di ct
C) Administer IV calcium gluconate. t t d na d s uncti n. Unit d Kin d m
D) Bolus IV normal saline 1 to 2 L. uid in s (2008) and Am ican Diab t s Ass ciati n
E) Give albuterol by nebulizer. uid in s (2011) c mm nd c ntinuin m t min in
ati nts wit mi d-t -m d at c nic kidn dis as
Answer 5.1.10 The correct answer is “B.” Ah, test-taking— and m nit in na uncti n v 3 t 6 m nt s.
clearly not real li e! T e best answer here is to increase renal
clearance o potassium with urosemide while holding the ACEI.
Because ACE inhibitors can increase serum potassium, you HELPFUL TIP:
should stop hers. “But what about her heart ailure?” you ask. A u stimat t c atinin c a anc can b
Well, good question. Given that her potassium is only slightly ca cu at d usin t C ckc t–g au t mu a:
elevated, the ACEI could be continued while monitoring her estimat d c atinin c a anc = (140 −
serum potassium. “A” is incorrect; stopping the urosemide may a [ a ])(b d w i t [k ])/(72 × (s um
lead to worsening HF symptoms and worsening hyperkalemia. c atinin [m /dl ]))
In hyperkalemia, calcium gluconate (“C”) is used to protect F w m n, mu ti t is i u b 0.85. o n cav at is
cardiac conduction, but it is not e ective or reducing potas- t at t is mu a ma n t ct a na inju b -
sium concentrations. A bolus o normal saline (“D”) is contra- caus c m nsat t t mainin
indicated in a patient with an acute HF exacerbation. Albuterol m u i. Normal a t adu t is 94 t 140 ml /min
(“E”) is a temporizing measure, reducing serum potassium by m n and 72 t 110 ml /min w m n.
orcing potassium into the intracellular space.
144 FAMIl y MeDICINe eXAMINATIo N & Bo Ar D r eVIeW

A) Add isosorbide dinitrate.


HELPFUL TIP:
B) Add a nondihydropyridine calcium channel blocker (e.g.,
T is n ct quati n ca cu atin t g Fr .
diltiazem, verapamil).
T abb viat d m di i d di t in na dis as (MDr D)
C) Add a dihydropyridine calcium channel blocker (e.g., amlo-
ca cu ati n is inc asin us d b caus it s ms t b
dipine, ni edipine).
an accu at s ntati n kidn uncti n in adu ts
D) Restrict protein in the diet.
wit na dis as . Un ik t C ckc t–g au t quati n,
E) Aim or a higher A1C target, around 8%.
t MDr D mak s an adjustm nt A ican-Am ican
ac but d s n t tak w i t int acc unt. B t qua-
Answer 5.1.13 The correct answer is “B.” Nondihydropyridine
ti ns adjust nd . A t u t quati n is a itt
calcium channel blockers reduce protein excretion in diabetic
c m icat d, man abs n w ca cu at t MDr D g Fr .
patients with nephropathy and slow down the progression o
I u want t ca cu at it u s , us t Nati na
renal disease—but not to the degree o ACE inhibitors. “A,” a
Kidn F undati n w bsit at tt ://www.kidn . /
long-acting nitrate, would be e ective or angina and will lower
ssi na s/KDo QI/ _ca cu at .
blood pressure but does not demonstrate an e ect on diabetic
F m t aut s’ s ctiv , an t s a o K.
nephropathy. T e dihydropyridine calcium channel blockers
W d n’t n d 2 ml /min accu ac in t C C . Just mak
(“C”) do not slow down progression to nephropathy to the same
su u us n t m.
degree as some other antihypertensive drugs. T e e ect o pro-
tein restriction (“D”) on nephropathy is controversial and not
clearly bene cial; there ore, more e ective therapies should be
You administer intravenous urosemide. T e night a er initiated rst. Finally, “E” is incorrect since better glycemic con-
admission she starts vomiting, and your partner inserts a trol is associated with renal protection.
nasogastric tube, which is inadvertently le to continuous
suction overnight (don’t do this . . . there is almost no indica-
tion or an NG tube—especially on continuous suction). T e HELPFUL TIP:
next morning, your patient’s laboratory results are as ollows: F diab tic n at , a wa s sta t wit an ACe
BUN 49 mg/dL, Cr 2.0 mg/dL, Na 132 mEq/L, K 3.5 mEq/L, in ibit and t t maximiz t d s . Fu d s ACe
and HCO3 35 mEq/L (normal 23–29 mEq/L). in ibit t atm nt is ass ciat d wit im v d su -
viva in diab tic n at c m a d wit w-d s
Question 5.1.12 On the basis o history and laboratory data ACe in ibit . T d s ACe in ibit ma b imit d
provided, you strongly suspect: b s um tassium v t ati nt’s b d s-
A) Metabolic acidosis. su . Sodium restriction added to an ACE inhibi-
B) Respiratory acidosis. tor is actually more e ective than adding an ARB
C) Metabolic alkalosis. or proteinuria and blood pressure control. (BMJ.
D) Respiratory alkalosis. 2011;343:d4366). In t is stud , t actua s dium in t
di t was 2,500 m (a t u t a was 1,200 m ).
Answer 5.1.12 The correct answer is “C.” Loss o gastric acid, D s it initia nt usiasm ( d u c m an ma k t-
through emesis or gastric suction, can result in a metabolic alka- in ), an Ar B us an ACe is n t us u in ss ntia an
losis. Although no pH is available to con rm alkalosis, you are ati nt and su ts in inc as d sid cts; t is c mbi-
able to in er the diagnosis based on the elevation in serum HCO3. nati n s u d b av id d.

HELPFUL TIP:
Diu tics wi t n caus a c mic, “c nt ac-
ti n,” a ka sis m v um c nt acti n. Watc t is Over the next year, your patient experiences increasing di -
in u ati nts n a diu tic. culties with glycemic control. Despite your e orts, her pro-
teinuria and serum creatinine increase. As you discuss re er-
ral to a nephrologist, she asks about dialysis.
Your patient recovers surprisingly well rom her heart ail-
ure. Her creatinine returns to baseline (1.5 mg/dL). You try Question 5.1.14 Which o the ollowing is NOT an indication
to re-challenge the patient with an ACE inhibitor. However, or dialysis?
her hyperkalemia recurs and she is unable to tolerate either A) Severe hyperkalemia due to renal ailure.
ACEI or ARB. With beta-blocker and loop diuretic therapy, B) Accelerated hypertension.
her average blood pressure is 130/80 mm Hg. At a ollow-up C) Malnutrition.
visit, you nd no signs or symptoms o HF. D) Persistent nausea and vomiting despite treatment.
E) Bleeding secondary to uremia.
Question 5.1.13 You want to lower your patient’s risk o
progressing to end-stage renal disease. To reduce protein- Answer 5.1.14 The correct answer is “C.” In chronic renal
uria, which o the ollowing strategies is best? ailure due to any cause, absolute clinical indications or
CHAPTER 5 • Neph r o l o g y 145

initiating dialysis include: persistent nausea and vomiting, albuminuria and diabetic retinopathy. You point out that
pericarditis, uid overload, uremic encephalopathy, accelerated her renal unction has progressively declined, which alarms
hypertension, bleeding due to uremia, serum creatinine greater her since her mother is on hemodialysis. Her creatinine was
than 12 mg/dL, and severe electrolyte abnormalities that can- 1.2 mg/dL 3 years ago, and now it is 1.9 mg/dL. Her GFR is
not be otherwise handled. T ese are potentially li e-threatening 36 mL/min/1.73 m2. She weighs 100 kg, and her blood pres-
situations that must be dealt with acutely. Malnutrition is a rela- sure is 120/76 mm Hg.
tive indication that occurs more indolently.
Question 5.2.1 Which o the ollowing is the most appropri-
ate description o her renal unction?
HELPFUL (AND VERY IMPORTANT) TIP: A) Stage 1 chronic kidney disease (CKD).
Counter to what you might expect, early dialy- B) Stage 2 CKD.
sis increases mortality (or at least is o no bene it). C) Stage 3 CKD.
D n t initiat dia sis unti t g Fr is 5 t 7 ml /min/ D) Stage 4 CKD.
1.73 m 2 or t ati nt is avin c inica b ms t at can- E) Stage 5 CKD.
n t b t wis mana d (N en J M d . 2010;363(7):
609–619). pati nts n a dia sis di m in cti n, Answer 5.2.1 The correct answer is “C.” She has stage 3 CKD.
c m icati ns m dia sis ( t nsi n), b ms wit able 5-1 de nes the various stages o CKD. Not all patients will
t i a ts (a ain adin t in cti n), tc. progress through all o the stages o CKD. On the basis o age
alone, many elderly patients may be classi ed as having stage
1 or 2 CKD, and they may never progress. Knowledge o a
Objectives: Did you learn to . . . patient’s CKD stage is use ul in determining how to treat, what
• Sc n m d at inc as d a buminu ia in diab tic drugs to avoid, etc.
ati nts?
• eva uat and t at m d at inc as d a buminu ia and
v t t inu ia in diab tic ati nts? HELPFUL TIP:
• Id nti na acut kidn inju ? pati nts wit CKD, wit wit ut diab t s, a at in-
• D sc ib t atu s r TA t 4? c as d isk c na a t dis as , s t isk ac-
• Id nti ast ic sucti nin as a caus m tab ic a ka sis t s a t dis as s u d b t at d a ssiv . In
in t s ita ? act, man x ts b i v t at CKD s u d b t at d
• Discuss w n t initiat dia sis in a ati nt wit c nic as a “c na a t dis as quiva nt.” At t ast,
na ai u ? CKD is an ind nd nt isk act CAD.

CASE 5.2 You increase the patient’s insulin and lisinopril doses, con-
A 49-year-old A rican-American emale with diabetes mel- rm that she is taking aspirin, and convince her to quit
litus type 2, hypertension and hyperlipidemia returns or a smoking (you are very persuasive—not to mention good
ollow-up visit. She’s had diabetes or 15 years, and her con- looking and strong . . . and humble!). One month later, your
trol has been variable over that time. Now, her glycosylated patient returns with typical symptoms o a urinary tract
hemoglobin is 8%. She has developed moderately increased in ection (U I) without systemic symptoms (e.g., ever,

TABLE 5-1 STAGES OF CHRONIC KIDNEY DISEASE


Stage GFR (mL/min/1.73 m 2) Comments
1 ≥90 Sta 1 d n d b vid nc kidn dama
( . ., st uctu a dama , a buminu ia) wit n ma g Fr

2 60–89 Mi d d c in in na uncti n; ab ut 5.4% US


u ati n

3 30–59 M d at d c in in na uncti n; ab ut 5.4% US


u ati n ( s, sam c nt as sta 2)

4 15–29 S v d c in in na uncti n; a ac in n d
dia sis

5 <15 end-sta dis as

g Fr , m ua t ati n at .
146 FAMIl y MeDICINe eXAMINATIo N & Bo Ar D r eVIeW

nausea). Her urine culture grows Escherichia coli resistant to


HELPFUL TIP:
trimethoprim/sul amethoxazole but susceptible to all other
C m a d wit diu tics, t iazid diu tics a
antibiotics tested.
n a b tt at w in b d ssu . h w v-
, at a g Fr < 30 ml /min/1.73 m 2, t iazid s a ss
Question 5.2.2 The most appropriate antibiotic regimen
ctiv as anti t nsiv a nts. F ati nts
or this patient’s UTI is:
wit CKD sta s 4 sta 5, s cia t s wit
A) Cipro oxacin 250 mg PO every other day or two doses.
v mia, c nsid usin a diu tic m t -
B) Cipro oxacin 250 mg PO once.
az n (w ic acts at mu ti na sit s) inst ad a
C) Cipro oxacin 250 mg PO BID or 3 days.
t iazid .
D) Levo oxacin 750 mg PO daily or 7 days.

Answer 5.2.2 The correct answer is “C.” T is patient’s GFR


is 36 mL/min/1.73 m 2 and her creatinine clearance is 56 mL/
min (using the Cockcrof –Gault equation given earlier). You get the patient through her episode o hypercalcemia
T ere ore, the usual cipro oxacin dose o 250 mg PO BID and advise her to lay o the calcium-based antacids, avor-
or U I is sa e and appropriate. Less requent dosing may ing an H2-antagonist instead. As your patient’s renal disease
be required or a lesser GFR or creatinine clearance. T e progresses over the years, you nd that she has become ane-
important thing here is to recall that CKD may require dos- mic with a hemoglobin o 9.9 g/dL. She takes a multivitamin.
age adjustments or many medications. Numerous drugs are Her iron studies are consistent with anemia o chronic dis-
renally cleared, so it’s always prudent to know the GFR or ease and her screening colonoscopy last year was normal. T e
creatinine clearance and check or dosage adjustments be ore anemia is normocytic and normochromic.
prescribing.
Question 5.2.4 I you were to treat her with an erythropoi-
You assume the U I cleared because you do not hear rom etic agent (e.g., erythropoietin or darbepoetin), her target
her or a ew months. T en her sister brings her in or an hemoglobin would be:
acute illness including nausea, emesis, con usion and gen- A) 9 to 10 g/dL.
eralized weakness. T ese symptoms began yesterday. T e B) 10 to 11 g/dL.
only other thing new with her health is heartburn which she C) 13 to 14 g/dL.
has been sel -treating with increasingly larger amounts o D) > 15 g/dL.
ums® (calcium carbonate), taking “hand uls” over the last
ew days. Her laboratory results now include creatinine Answer 5.2.4 The correct answer is “B.” For patients with
2.5 mg/dL, calcium 14 mg/dL (normal range 8.9–10.5 mg/dL), CKD and anemia o chronic disease (when other causes o
and HCO3 37 mEq/L (normal 22–28 mEq/L). anemia have been ruled out and/or treated), erythropoietic
agents should NO be used to achieve “near normal” hemo-
Question 5.2.3 The most appropriate treatment or her now globin levels. arget levels are determined on an individual
includes: basis but in general will be about 10 to 11 g/dL. Higher
A) Hydrochlorothiazide (HC Z). hemoglobin levels—in particular, attempts to “normalize”
B) Furosemide. the hemoglobin concentration—are associated with a greater
C) Normal saline IV. risk o adverse events, including increased risk o mortality,
D) A and C. need or dialysis, heart ailure, graf thrombosis, uncontrolled
E) B and C. hypertension, etc. T ese ndings are consistent with a simi-
lar nding in cancer patients (JAMA. 2010;303(9):857–864;
Answer 5.2.3 The correct answer is “C.” Your patient is now N Engl J Med. 2009;361:2019–2032). I the patient is an appro-
presenting with a classic case o milk-alkali syndrome. T e diag- priate candidate, the initial erythropoietin dose is approxi-
nosis should be recognized by the triad o hypercalcemia, meta- mately 50 to 100 units/kg per week. Because o the need to
bolic alkalosis, and renal insu ciency in combination with the preserve veins or uture hemodialysis access, we recommend
history o typical symptoms (described in the case) and exces- that erythropoietin be administered subcutaneously. T e
sive calcium carbonate ingestion. reatment includes removal FDA suggests withholding erythropoietin or hemoglobin
o the o ending agent and treatment o the hypercalcemia with > 12 g/dL or i the hemoglobin increases by > 1 g/dL in any
IV saline (to improve renal per usion and metabolic alkalosis). 2-week period.
“A,” thiazide diuretics, results in reabsorption o calcium—the
opposite o what you want to achieve in a patient with hyper- Objectives: Did you learn to . . .
calcemia. “B,” urosemide, has allen out o avor and requires • Sta CKD?
the establishment o euvolemia rst. Even then you need 10 L • Adjust m dicati n d s s bas d n na uncti n?
o saline per day with 100 mg o IV urosemide every 1 to • r c niz and t at mi k-a ka i s nd m ?
2 hours. T is is usually impractical and hasn’t been shown to • r c niz a iat ta ts t t atm nt wit t -
add anything. i tic a nts?
CHAPTER 5 • Neph r o l o g y 147

Question 5.3.2 In your evaluation o this patient, you


CASE 5.3 include all o the ollowing tests EXCEPT:
A 55-year-old male presents to your of ce or evaluation o A) Urine cytology.
blood in his urine. It turns out that he had a li e insurance B) CBC.
physical and the urinalysis showed 2+ blood on urine dip- C) Serum creatinine.
stick and 2 RBC/hp . T e remainder o the urinalysis and D) C scan o the abdomen and pelvis with particular note o
microscopic examination was normal. the kidneys.
E) Renal biopsy.
Question 5.3.1 A ter an appropriate history and physical
examination, your rst step in the evaluation o this urine Answer 5.3.2 The correct answer is “E.” In most cases o micro-
abnormality is to: scopic hematuria, renal biopsy is not indicated. However, i an
A) Repeat the urinalysis and microscopic examination. intrinsic renal cause o hematuria is suspected, renal biopsy may
B) Obtain urine or culture. prove necessary. Intrinsic renal disease is more likely i there
C) Order a renal ultrasound. is proteinuria, hypertension, elevated serum creatinine, or an
D) Order a C scan o the abdomen. active urinary sediment (e.g., nephritic, dysmorphic red cells,
E) Order an intravenous pyelogram (IVP). red cell casts).
T ere is no completely standardized evaluation o micro-
Answer 5.3.1 The correct answer is “A.” According to the uri- scopic hematuria, and recommendations vary depending on
nalysis, there is a small amount o blood in your patient’s urine, the author. However, the recommendations always include
but the number o RBCs is actually normal (< 3 RBC/hp ). Your serum creatinine and usually include CBC, coagulation stud-
rst step should be to repeat the urinalysis and urine micro- ies, and serum chemistries. Depending on the patient’s age, ur-
scopic examination to determine i this patient actually meets ther studies may be indicated. For patients older than 40 years,
the criteria or microscopic hematuria (≥ 3 RBC/hp on two o you should consider studies to evaluate or urinary tract can-
the three properly collected urine specimens, according to the cers. Urine cytology has low sensitivity but high speci city or
American Urological Association). A urine culture may prove bladder cancer and may be quite use ul in conjunction with
use ul later in the evaluation process but is not necessary now. cystoscopy. Imaging o the urinary system is an absolute require-
Likewise, ordering imaging studies is premature because the ment in the work-up o microscopic hematuria in older patients
diagnosis o microscopic hematuria has not been made. (generally, those over age 40 years). C scan appears to have the
greatest sensitivity or detecting masses, but ultrasound, IVP, or
HELPFUL TIP: the combination o the two may also be employed. Cystoscopy
Kidn st n s a a v c mm n caus ss and should be considered i the C scan is normal since C is poor
mic sc ic matu ia. h w v , ca t at 20% at visualizing bladder abnormalities. See able 5-2 or a sug-
ati nts wit kidn st n s wi av n b d in t i gested work-up or microscopic hematuria.
u in and a n ma u ina sis. I A n at is n
t m st c mm n int insic na dis as s t caus mi- Question 5.3.3 The US Preventive Services Task Force rec-
c sc ic matu ia; kidn c sts, inc udin c stic ommends which o the ollowing screening strategies or
kidn dis as , a c mm n caus s as w . N astic detecting microscopic hematuria?
dis as s, in a ticu a b add canc and na c ca - A) Annual urinalysis af er age 50.
cin ma, ma a s b disc v d du in t inv sti a- B) Urinalysis every 2 years af er age 50.
ti n mic sc ic matu ia.

HELPFUL TIP: TABLE 5-2 EVALUATION OF MICROSCOPIC


C m a d wit mic sc ic matu ia, gross hematu- HEMATURIA
ria is m c mm n ass ciat d wit ma i nanc and, A t mic sc ic matu ia as b n id nti d (2 3 u in sam s
nc c n i m d, s u d m tat u va uati n. wit 3 m r BC/ ), t Am ican U ica Ass ciati n (AUA)
B ni n caus s microscopic hematuria in adu ts in- c mm nds t win va uati n:
c ud : vi us x cis , m nst uati n, s xua activit , • In cti n id nti d → t at wit antibi tics and at u ina sis
vi a i n ss, and t auma. • r BC casts, t inu ia, vat d c atinin → b in va uati n
m u n itis and c nsid a t an ist
• N in cti n ima na dis as id nti d in st 2 st s →
u in c t , b add c st sc (i at isk b add canc
Further history reveals that he smokes one to two packs o bas d n nvi nm nta x su s and/ a > 40), and CT scan
cigarettes per day. He has a normal blood pressure and the ( ica CT i st n s sus ct d, c nt ast- n anc d CT i st n s n t
remainder o the physical examination is unrevealing. On sus ct d)
• I nti t u dia n stic va uati n n ativ → w-u
two urine samples, you nd microscopic hematuria, with a u ina sis, u in c t ,b d ssu , and s um c atinin
positive dipstick and 5 RBC/hp . T e rest o the urinalysis is v 6–12 m nt s.
normal, and there are no red cell casts.
148 FAMIl y MeDICINe eXAMINATIo N & Bo Ar D r eVIeW

C) Annual urinalysis af er age 65.


D) Annual urinalysis in all high-risk patients older than 65 years. Your patient does well or a year. When you see him next, he
E) No screening at any age. presents to your of ce as a late a ernoon add-on and com-
plains o severe abdominal pain that woke him rom sleep at
Answer 5.3.3 The correct answer is “E.” he USPS F recom- 3 AM. He describes the pain as “sharp” or “crampy,” occur-
mends against routine screening or microscopic hematuria ring in his le lower quadrant and radiating to the le tes-
to detect urinary tract cancers. In one-time urine specimens ticle. Although the pain has waxed and waned, it has never
in healthy adults, the presence o abnormal numbers o RBCs resolved completely. He is also nauseated and has been vom-
(≥ 3 RBCs/hp ) can be as high as 39%. In up to 70% o iting. On examination, he is a ebrile and tachycardic and has
patients, even a ter imaging o the upper and lower urinary a blood pressure o 110/56 mm Hg. He appears uncom ort-
tract, the source o microscopic hematuria cannot be ound. able and is writhing in pain. T ere is le lower quadrant,
In a low-risk population, the alse-positive rate o micro- ank, and costovertebral angle tenderness.
scopic hematuria ound on urinalysis would be unaccept-
ably high. Also, there is no evidence that early detection o Question 5.3.5 Your next action is to:
urinary tract cancers through screening urinalysis improves A) Prescribe oral ibupro en and morphine and arrange ollow-
prognosis. up tomorrow.
B) Give IM ketorolac and arrange ollow-up tomorrow.
Your patient returns to discuss his laboratory and radiol- C) Bolus 1 L IV saline, administer IV cef riaxone, and arrange
ogy results. His serum creatinine is 1.1 mg/dL, and his CBC, ollow-up tomorrow.
chemistries, and coagulation studies are normal. His urine D) Send him to the emergency department (ED) or pain man-
cytology was negative, as was cystoscopy per ormed by your agement, uids, and possible admission.
riendly neighborhood urologist. C scan o the abdomen
and pelvis reveals normal size kidneys and no masses, but Answer 5.3.5 The correct answer is “D.” In a patient with
three stones, measuring 2 to 3 mm in diameter, are noted in known kidney stones presenting with classic ndings o uro-
the le renal pelvis. T ere does not appear to be any obstruc- lithiasis, the most likely diagnosis is renal colic due to stones.
tion. Your patient denies any history o renal colic. T is patient is nauseated and vomiting requently and may not
do well at home overnight. He is unlikely to tolerate oral medi-
Question 5.3.4 Regarding the nding o stones in the le t cations and could become dehydrated. In addition, we don’t
renal pelvis, which o the ollowing interventions is war- have a urinalysis, and a postobstructive U I is an indication or
ranted at this time? admission and possibly stenting. For these reasons, the most
A) Observation. appropriate action is aggressive pain management, which can
B) Lithotripsy. be accomplished in the ED. I the patient is unable to keep down
C) Ureteral stent placement. oral pain medications af er treatment in the ED, admission to
D) Ketorolac and uids by IV. the hospital is warranted. Narcotic analgesics, IV NSAIDs (e.g.,
E) Prophylactic nephrectomy. ketorolac), and IV uids to maintain euvolemia are all appropri-
ate. T e role o antibiotics will depend on urinalysis ndings,
Answer 5.3.4 The correct answer is “A.” he inciden- but most cases o acute renal colic do not require antibiotics.
tal inding o stones during the evaluation o microscopic
hematuria is common. he stones may be the reason or HELPFUL (AND VERY IMPORTANT) TIP:
your patient’s hematuria. Since the stones are small, they U it iasis and abd mina a tic an u sm can av
may pass without intervention. Most stones less than 5 mm t sam s ntin s m t ms and si ns (inc udin
in diameter will pass spontaneously. No urther intervention matu ia). F t at as n, ima in is mandat in
is warranted in asymptomatic patients. “D” is o particular t d individua in w m an abd mina a tic an-
note. Ketorolac and other NSAIDs are e ective in treating u sm is a c nsid ati n. A s , k t sticu a t si n
the pain o urolithiasis. However, luid is not help ul in treat- n t di ntia un ma ati nts s ntin
ing acute ureteral colic (unless the patient is dehydrated). wit na c ic s m t ms—t s ati nts n d a ni-
Fluid does nothing to “push” the stone out: luid increases ta xaminati n (but u kn w t at, i t?).
pain and the body simply shi ts the excess luid to the non-
obstructed kidney.
You admit the patient to the ED; start IV saline to maintain
hydration; and administer narcotics, NSAIDs (ketorolac)
HELPFUL TIP: and antiemetics. C scan shows a 5-mm stone in the proxi-
D sm ssin (DDAVp), w ic d c as s u in v - mal ureter. T ere is no hydronephrosis. Serum electrolytes,
um s, can b us d t c nt ain in t s wit u i- BUN, creatinine, and CBC are all normal. Urinalysis reveals
t iasis (a t u w d n’t c mm nd it utin 2+ blood, 1+ leukocyte esterase, trace protein, pH 6, speci c
us —t a nt d ana sics ut t ). gravity 1.025, 20 RBC/hp , ew calcium oxalate crystals, and
otherwise normal.
CHAPTER 5 • Neph r o l o g y 149

Question 5.3.6 Which o the ollowing is the most appropri- hypercalciuria). Patients with uric acid stones (“D”) should be
ate management at this point in time? evaluated or symptoms o gout and undergo serum uric acid
A) Add antibiotics to the current therapy. measurements.
B) Continue the current therapy and observe.
C) Re er or extracorporeal shock wave lithotripsy. Your patient passes the stone, his pain completely resolves,
D) Re er or endoscopic lithotripsy. and he is discharged rom the hospital within 24 hours o his
admission. T e stone is pure calcium oxalate. Studies o his
Answer 5.3.6 The correct answer is “B.” T ere is no reason to 24-hour urine collection are pending.
change management at this point in time. Many 5-mm stones
will pass spontaneously. Although leukocyte esterase is detected Question 5.3.8 In order to reduce his risk o orming more
on the urine dipstick, there is no compelling evidence o in ec- stones, you tell him to incorporate all o the ollowing li e -
tion (e.g., ever, elevated WBC count, and WBCs on microscopic style changes EXCEPT:
examination), so antibiotics are not necessary. But culturing the A) Restrict calcium intake.
urine would be prudent. B) Restrict oxalate intake (e.g., lea y green vegetables, chocolate).
C) Increase daily water intake.
D) Add a glass o orange juice daily.
HELPFUL TIP:
E) Reduce meat and sh consumption.
St n s 6 mm at wi ass s ntan us n
10% t tim and t s 4 t 6 mm 50% t tim .
Answer 5.3.8 The correct answer is “A.” o prevent recur-
T s ss t an 4 mm ass t at maj it t
rent urolithiasis, restricted calcium intake was encouraged in
tim . I ain sists t st n d s n t ass wit in
the past. Now, moderate calcium intake (1 g/day) is recognized
72 u s, c nsid u ic int v nti n suc as n -
as bene cial and should be encouraged. Patients should take
st m , st nt ac m nt, and/ it t i s . r na in-
calcium with meals, which will bind oxalate and prevent its
ju m bst ucti n n a d s n t ccu at
absorption. A low-calcium diet actually leads to an increased
ast 72 u s (and, amazin n u ,t is n a
risk o urolithiasis. Increase in urine oxalate greatly increases
20% c anc c m icati ns i a n n bst uctin st n
the risk o stone ormation. Restriction o oxalate (“B”) in the
mains 4 w ks).
diet may help to reduce the risk o recurrent urolithiasis. Foods
high in oxalate include spinach, rhubarb, nuts, and legumes.
Un ortunately, oxalate is also the end product o numerous
HELPFUL TIP: metabolic pathways, and signi cant reduction in urinary oxa-
pati nts disc a d m t eD wit u it iasis s u d late levels of en proves di cult (good news or you rhubarb
b ac d n an NSAID in additi n t a na c tic. T is ans! We like rhubarb pie and chocolate . . . say, I eel a stone
duc s ain and “b unc back” visits. coming on).
Increased uid intake (“C”) to achieve a urine volume
> 2 L/day reduces stone ormation. Water and citrus juices
Question 5.3.7 Your patient—a man possessing incred- (“D”) are traditionally recommended (hypocitraturia is associ-
ible oresight—asks how he can avoid kidney stones in the ated with stone ormation), but most uids consumed are asso-
uture. Which o the ollowing tests will be most use ul in ciated with a positive e ect, including drinks with ca eine.
determining the treatments that may prevent uture stone Red meat, sh, and poultry (“E”) are sources o purine, which
ormation? is metabolized to uric acid. A uric acid crystal can orm a pure
A) Urine culture. uric acid stone or serve as a nidus or calcium stone ormation.
B) Stone recovery and analysis. T ere ore, a general recommendation to avoid recurrent uroli-
C) Urinary calcium excretion. thiasis is to reduce purines (e.g., meat) in the diet.
D) Urinary oxalate excretion.
E) Serum uric acid. Question 5.3.9 Your patient is wondering i he should have
his urolithiasis worked up to determine an etiology. All o
Answer 5.3.7 The correct answer is “B.” T e prevention o ur- the ollowing are reasons to pursue a urther evaluation in
ther stone ormation is aided by knowledge o the stone type. this patient EXCEPT:
You should always attempt to recover the stone and send it or A) A strong amily history o stones.
analysis—unless the patient is a well-known stone ormer and B) A rican ancestry.
the composition o his or her stones is already known. T e other C) Chronic diarrhea.
studies listed may have value. Struvite stones orm during bac- D) Hypertension.
terial in ections o the urinary tract and a urine culture (“A”) E) Bariatric surgery.
will help direct therapy when these stones are identi ed. Cal-
cium oxalate stones are the most common and 24-hour urine Answer 5.3.9 The correct answer is “D.” Simply having hyper-
collection to determine calcium and oxalate excretion (“C”) can tension is not a reason to work-up a patient or renal stones.
lead to diagnoses o metabolic disturbances (hyperoxaluria and All o the other choices are correct. O note is “B.” Patients o
150 FAMIl y MeDICINe eXAMINATIo N & Bo Ar D r eVIeW

A rican ancestry are less likely to have stones. T ere ore, the Answer 5.3.11 The correct answer is “D.” While dehydration
work-up is more likely to reveal a pathologic process in these predisposes patients to a second stone (and thus “E”), maintain-
patients. “C” and “E” are also o note. Anything that can cause ing optimal hydration lowers the risk but is not a oolproo rem-
malabsorption including bowel surgery, history o in amma- edy. Kidney stones are more common in men and have a peak
tory bowel disease, etc., can increase the risk o stones. T us, incidence at age 30. About 60% to 80% o patients with a kidney
evaluation is indicated in these patients. stone will su er a recurrence.

Because o his strong amily history, you decide to proceed Question 5.3.12 Which o these patients with urolithiasis
with evaluation. Results o your patient’s 24-hour urine have requires hospitalization?
returned and are as ollows (normal values): A) A patient with a high-grade obstruction.
B) A patient with intractable pain or vomiting.
Volume 1.6 L.
C) A patient with an associated urinary tract in ection.
pH 6.5.
D) A patient with a solitary kidney or transplanted kidney.
Creatinine clearance normal.
E) All o the above.
Calcium 410 mg (re erence range 100–300 mg).
Uric acid 410 mEq (re erence range 250–750 mEq).
Answer 5.3.12 The correct answer is “E.” All o the patients
Oxalate 42 mg (re erence range 7–44 mg).
listed above should be admitted. Obviously, patients having
Citrate 560 mg (re erence range 100–800 mg).
uncontrollable pain or vomiting will do poorly as outpatients.
Magnesium 3.1 mEq (re erence range 3–5 mEq).
Urolithiasis with a coexistent U I is a signi cant problem due
Question 5.3.10 Which o the ollowing medications is most to the risk o abscess ormation, bacteremia, and renal paren-
likely to reduce his risk o developing kidney stones in the chymal destruction. T ese patients require IV antibiotics and
uture? immediate urologic consultation, particularly in the presence o
A) Allopurinol. comorbidity. Patients with solitary or transplanted kidneys or in
B) Potassium citrate. whom the diagnosis o renal colic is unclear should be admitted
C) Hydrochlorothiazide. or monitoring o renal unction and urther evaluation.
D) Sodium bicarbonate.
E) Furosemide.
HELPFUL TIP:
T m st c nt, i qua it , data su sts t at n it
Answer 5.3.10 The correct answer is “C.” According to the
a a-b ck s ( . ., tamsu sin, d xaz sin) n ca cium
24-hour urine studies, your patient has hypercalciuria, sub-
c ann b ck s ( . ., ni di in ) a us in -
optimal urine volumes (the goal or urine output in urolithi-
in st n x u si n (l anc t. 2015;386(9991):341–349).
asis should be 2 L/day or more), and no other abnormalities.
p vi us m ta-ana s s t at s w d a b n it w
Patients with urolithiasis and hypercalciuria bene t rom the
sma , ativ qua it , studi s. T is is sti c nt -
long-term treatment with thiazide diuretics, such as hydro-
v sia and w sus ct t answ n t t st wi b t
chlorothiazide, which decrease calcium excretion and there ore
us x u siv t a wit an a a-b ck ca cium
stone ormation. Furosemide increases calcium excretion and
c ann b ck .
has the potential to increase stone ormation.
Even though it seems counterintuitive, patients with calcium
stones and hyperuricosuria may bene t rom allopurinol. It is HELPFUL TIP:
thought that hyperuricosuria predisposes to calcium stones. r m mb t at mu ti d u s can m adi uc nt
Also, allopurinol is use ul in the treatment o patients with uric st n s t at wi n t s w u n a CT scan. Am n st
acid stones. Patients with uric acid stones and hyperuricosuria t s a indinavi , ac c vi , and t iamt n .
may bene t rom alkalinization o the urine with sodium bicar-
bonate or potassium citrate. In general, dietary citrate should be
maximized. Oral potassium citrate is indicated or patients with
hypocitraturia. Objectives: Did you learn to . . .
• D n mic sc ic matu ia?
T e patient is wondering whether he can expect to get another • eva uat a ati nt wit mic sc ic matu ia bas d n isk
stone. and di ntia dia n sis?
• D sc ib t di cu ti s in nt in m in u in studi s
Question 5.3.11 All o the ollowing are true EXCEPT: t sc n u ina t act canc s?
A) T e peak incidence o kidney stones occurs at age 30. • eva uat a ati nt wit u it iasis?
B) Kidney stones are more common in men than in women. • Id nti caus s u it iasis bas d n t c a act istics
C) T e rate o a second kidney stone is 60% to 80%. t st n ?
D) Only people who become dehydrated can expect to have a • Mana a ati nt wit s m t matic u it iasis?
second stone. • Us st at i s t v nt st n mati n bas d n t c a -
E) Living in a hot climate can predispose to a second stone. act istics t st n ?
CHAPTER 5 • Neph r o l o g y 151

Question 5.4.3 Which o the ollowing is the best next step


CASE 5.4 in your evaluation and management?
A 19-year-old emale presents to your of ce concerned about A) Start her on a low-protein diet.
protein in her urine. As a college student, she has a part-time B) Order C scan o abdomen and pelvis.
job in a medical laboratory. She “repurposed” a ew urine C) Measure recumbent urine protein level.
dipsticks rom the laboratory—you know, “just checking D) Re er or renal biopsy.
her urine”—and ound that she had 2+ protein on her dip-
stick. She has no urinary symptoms and denies ever, weight Answer 5.4.3 The correct answer is “C.” Proteinuria may be
changes and edema. She is a ebrile with a blood pressure o either transient or persistent. ransient proteinuria is of en due
118/68 mm Hg. Examination is otherwise unremarkable. to ever, exercise, or other causes and is not associated with sig-
Repeat urinalysis in the of ce con rms 2+ protein, speci c ni cant kidney disease. ransient proteinuria is ound in 7%
gravity 1.020, pH 6.5, and no blood. T e urine microscopic o women and 4% o men. It of en resolves spontaneously, and
examination is normal. subsequent urine tests will probably be negative. However, at
this point you don’t know i this case will be transient or persis-
Question 5.4.1 Which o the ollowing is the most appropri- tent proteinuria.
ate next action in evaluating this patient? Orthostatic proteinuria is a common type o transient pro-
A) Repeat urinalysis and urine culture. teinuria seen in young, healthy persons. Up to 5% o adolescents
B) Ultrasound o the kidneys. have orthostatic proteinuria, and young adults may present
C) 24-hour urine collection or protein and creatinine. with it as well. Protein is spilled in the urine when the patient is
D) Random urine protein/creatinine ratio. upright, but not when recumbent. T ere are two ways to deter-
mine recumbent urine protein: an easy way and a hard way.
Answer 5.4.1 The correct answer is “D.” Did you go or “C”? T e easy way is to have the patient void be ore going to bed,
A random urine is much easier than collecting a 24-hour urine stay supine all night (8 hours), and collect urine immediately
and correlates well with 24-hour collections. You could ask the upon waking. T is urine is checked or protein/creatinine ratio.
patient to collect urine or 24 hours or protein and creatinine Another urine sample must be checked or protein/creatinine
measurements, but why? Who wants to lug around a jug o their ratio af er the patient has been upright. I the upright is abnor-
own urine all day? What will she tell her riends and co-workers? mal and the recumbent is normal, you have diagnosed ortho-
Your patient already has protein on urinalysis twice. Another static proteinuria (and impressed your colleagues). T e “hard
urinalysis (“A”) will not help to determine i this is truly a wor- way” involves splitting a 24-hour urine collection—two jugs
risome nding or not. T ere is no evidence o a U I (no leu- o urine and the required arithmetic. Orthostatic proteinuria
kocyte esterase, nitrites, etc., and no symptoms); thus, culture is a benign condition that usually resolves as the patient ages,
would be very low yield. T ere is no indication or renal ultra- requiring no additional evaluation.
sound (“B”) at this point. A nding o 1 g o protein in a 24-hour urine collection is
abnormal (normal < 0.15 g/day), but it does not yet reach the
nephrotic range (> 3 g/day). Regardless, a low-protein diet
HELPFUL TIP: (“A”) is not help ul. C scan o the abdomen and pelvis (“B”) is
C nc nt at d a ka in u in can su t in v stima- unlikely to add any new in ormation. Re erring or renal biopsy
ti n u in t in n di stick. N t t at t is ati nt’s (“D”) is premature.
u in s ci ic avit was n ma .
T e patient collects urine in upright and recumbent positions,
Question 5.4.2 Which type(s) o protein are detected on a and both have abnormal amounts o protein. You re er the
urine dipstick? patient to a nephrologist who recommends ollow-up rather
A) Albumin. than biopsy. Annual ollow-up will include blood pressure,
B) Amino acids. serum creatinine, and spot urine protein/creatinine ratio.
C) Immunoglobulin light chains.
D) A and B. Question 5.4.4 In order to prevent worsening proteinuria,
E) A and C. the nephrologist also recommends which o the ollowing
medications?
Answer 5.4.2 The correct answer is “A.” T e urine dipstick only A) Amlodipine.
detects large–molecular-weight proteins—generally this means B) Diltiazem.
albumin. Amino acids are not detected. Immunoglobulin light C) Furosemide.
chains, such as Bence Jones proteins, are also not detected on D) Benazepril.
urine dipstick. E) Aspirin.

Her urine protein/creatinine ratio is equivalent to 1 g o urine Answer 5.4.4 The correct answer is “D.” Patients with protein-
protein per day. Creatinine clearance is normal. Serum cre- uria tend to respond well to ACE inhibitors. ACE inhibitors have
atinine, BUN, albumin, glucose, and electrolytes are normal. been shown to reduce proteinuria by 35% to 40%. T is e ect is
152 FAMIl y MeDICINe eXAMINATIo N & Bo Ar D r eVIeW

true in nondiabetic patients with proteinuria as well as in dia- TABLE 5-3 CRITERIA FOR NEPHROTIC SYNDROME
betic patients. ACE inhibitors appear to be superior to other
Required or the diagnosis o nephrotic syndrome
antihypertensives, including calcium channel blockers (“A”
• A buminu ia > 3 /da
and “B”). Furosemide (“C”) would be indicated i your patient • h a bumin mia (s um a bumin < 3 /dl )
develops edema, but loop diuretics should not be used primarily • p i a d ma
or the treatment o hypertension or proteinuria. Aspirin (“E”) Other (nondiagnostic but supportive) f ndings:
may be indicated or protection against coronary artery disease • h i id mia
in patients with risk actors, including CKD, but it is not a pri- • T mb tic v nts
mary treatment o proteinuria or hypertension. Remember that
NSAIDs can adversely a ect renal unction.
Answer 5.4.6 The correct answer is “C.” Urine sediment in
HELPFUL (AND IMPORTANT) TIP:
nephrotic syndrome is typically bland. T ere is little cellular
W i ACe in ibit s a i st- in t atm nts -
matter. Oval at bodies and atty casts occur in the urine o
t inu ia, t a s can caus cts n t tus du in
patients with heavy proteinuria and hyperlipidemia. Fat bod-
nanc . Us is n t c nt aindicat d wit in w m n
ies re ect the increased permeability o the glomeruli and sug-
c i d-b a in a , but s u d b und tak n wit
gest some type o glomerular disease (including nephrotic syn-
cauti n. ensu s is n a iab s u c c nt ac -
drome) albeit not necessarily active disease. However, at can
ti n ( a s n -actin v sib c nt ac ti n suc
also be seen in patients with polycystic kidney disease and at
as IUD N x an n) and t at s is c uns d n t
embolism syndrome.
isks t t tus i s w t t nant. I s d s
Red cell casts (“A”), the hallmark o glomerulonephritis,
b c m nant, mak su s disc ntinu s t ACe
are absent in nephrotic urine. White cell casts (“B”) are asso-
in ibit i t awa .
ciated with interstitial nephritis and pyelonephritis. Uric acid
crystals (“D”) are sometimes seen in gout, hyperuricemia and
urate stone disease. Granular casts (“E”) are not speci c to any
Un ortunately, over the next ew years, your patient develops particular pathologic process and may be ound in acute tubu-
hypertension. You maximize the dose o benazepril and need lar necrosis, glomerulonephritis, and other renal diseases. See
to add another drug to control her pressure. Her proteinuria able 5-4 or more details on urine sediment.
increases to 3.5 g/day, and her plasma creatinine increases
to 2 mg/dL. She develops edema, and her serum albumin is Question 5.4.7 Nephrotic syndrome is not a speci c disease
2.8 g/dL. T e urine shows only protein and no in ammatory entity but can be the end result o a number o processes.
components. Which o the ollowing cause(s) nephrotic syndrome?
A) Diabetes.
Question 5.4.5 This patient’s current clinical condition is B) Minimal change disease.
most appropriately described as: C) Amyloidosis.
A) Hypertensive nephropathy. D) Systemic lupus erythematosus.
B) Acute renal ailure. E) All o the above.
C) Focal nephritic glomerulonephritis.
D) Nephrotic syndrome. TABLE 5-4 URINE SEDIMENT FINDINGS AND
E) Floating kidney. ASSOCIATED CONDITIONS
Sediment Finding Associated Condition
Answer 5.4.5 The correct answer is “D.” While identi ying no e it ia c casts ATN, acut m u n itis
speci c disease state, the term nephrotic syndrome re ers to a
constellation o signs and laboratory abnormalities. A num- Fat b di s, att casts Massiv t inu ia (n tic s nd m ),
at mb i, c stic kidn dis as ,
ber o diseases may lead to nephrotic syndrome (keep read-
m u a dis as (ma n t b activ as in
ing or more on this). When urine protein exceeds 3 g/day, it n tic s nd m )
is of en re erred to as “nephrotic range” proteinuria. Complete
nephrotic syndrome is characterized by nephrotic range pro- g anu a casts N ns ci c—man na dis d s
teinuria, edema, hypertension, hypoalbuminemia, and hyper- h a in casts C nc nt at d u in , diu tic us , n ma
lipidemia ( able 5-3). ndin

r dc casts g m u n itis, vascu itis (v s ci c


Question 5.4.6 Which o the ollowing ndings in urine sed-
t t s )
iment is associated with nephrotic syndrome?
A) Red cell casts. Wax casts Advanc d na ai u
B) White cell casts.
W it c casts p n itis, acut int stitia n itis,
C) Oval at bodies. va i us m u a dis as s
D) Uric acid crystals.
E) Granular casts. ATN, acut tubu a n c sis.
CHAPTER 5 • Neph r o l o g y 153

TABLE 5-5 CAUSES OF NEPHROTIC SYNDROME Question 5.4.9 Absolute contraindications to renal biopsy
include which o the ollowing?
• Diab t s
A) Hypertension.
• Am id sis
• S st mic u us t mat sus B) Use o war arin or other anticoagulant or atrial brillation.
• Minima c an dis as (“Ni ” dis as ) C) Pyelonephritis.
• Di us m u n itis D) Solitary renal cyst.
• M mb an us n at E) All o the above.
• I An at
• p stin cti us m u n itis ( . ., st-st t c cca g N)
• M mb an i ativ m u n itis Answer 5.4.9 The correct answer is “C.” T e presence o renal
• Va i us n astic dis as s ( m ma, mu ti m ma, un or perirenal in ection is a contraindication to renal biopsy.
canc , tc.) None o the others are absolute contraindications. However,
• p c am sia uncontrollable hypertension, irreversible coagulopathy, mul-
• Fami ia kidn dis as (A t dis as , Fab dis as )
tiple bilateral cysts, hydronephrosis, small kidneys (indicative
• F ca and s m nta m u sc sis
• M dicati ns (NSAIDs) o chronic, irreversible disease), known renal tumor, and lack o
• Misc an us consent are all contraindications to renal biopsy. Simply being
on war arin or another anticoagulant, which can be stopped or
reversed, is not a contraindication to biopsy.
Answer 5.4.7 The correct answer is “E.” All o the above can
be a cause o nephrotic syndrome. Remember that many causes Question 5.4.10 Which o the ollowing is an indication or
o nephrotic syndrome can present initially with nephritic renal biopsy?
urine. Causes o nephrotic syndrome are summarized in A) Persistent hematuria in a patient with normal renal unction
able 5-5. and an otherwise negative work-up.
B) Persistent low-grade proteinuria (1–2 g/day range) with
Question 5.4.8 Which o the ollowing tests is NOT indi- normal blood pressure and creatinine.
cated in most patients with nephrotic syndrome? C) Suspected case o IgA nephropathy.
A) Hepatitis B and C serology. D) Nephrotic syndrome likely rom diabetes mellitus.
B) ANA. E) Persistent low-grade proteinuria with elevated blood pres-
C) Serum and urine protein electrophoresis. sure and/or elevated creatinine.
D) Antistreptococcal antibodies (e.g., ASO titer).
E) CA-125 antigen. Answer 5.4.10 The correct answer is “E.” Patients with low-
grade proteinuria and elevated serum creatinine and/or hyper-
Answer 5.4.8 The correct answer is “E.” Evaluation or cancer tension should have a renal biopsy in order to determine the
should be done i there is a speci c reason to believe the patient etiology o their disease. Indications or biopsy are listed in
has a malignancy (weight loss, mass on examination, etc.). able 5-6.
CA-125 is a marker or ovarian cancer (among others . . . colon, Objectives: Did you learn to . . .
pancreatic, etc.) and need not be done routinely. Plus, it’s not a • eva uat and w a a t -a a in ati nt wit -
use ul screening test or ovarian cancer. In addition to a thor- t inu ia?
ough history and physical routine evaluation o patients with • D n and dia n s t static t inu ia and und stand
nephrotic syndrome (and nephritis and glomerulonephritis) its si ni canc ?
include the ollowing: • Mana a ati nt wit ssiv t inu ia?
• Hepatitis B and C serology. • D n and va uat n tic s nd m ?
• ANA. • Id nti caus s n tic s nd m ?
• Serum and urine protein electrophoresis.
• ASO titer.
• Cryoglobulins. TABLE 5-6 INDICATIONS FOR RENAL BIOPSY
• Serum complement levels. • N tic s nd m without a s st mic ti und n t
t stin .
• ANCA. • h matu ia m a m u a s u c with t nsi n inc as-
• Serum calcium (to rule out sarcoid). in c atinin .
• Antiglomerular basement membrane antibodies. • p sist nt w ad t inu ia with t nsi n inc asin
c atinin .
Use clinical judgment to guide your work-up. However, • N itis without a s st mic x anati n ( . ., n u us, d u
this evaluation will nd the cause o most cases o nephritis/ x su ).
nephrotic syndrome. • Sus ici n anu mat sis wit an iitis ( m W n
dis as ) a t itis n d sum w t tissu is n t avai ab .
• Acut subacut na ai u wit ut an t x anati n
T e diagnosis in your patient is unclear despite the work-up (a t u ist wi usua su t in a sum tiv dia n sis).
noted above. It is time to consider a renal biopsy.
154 FAMIl y MeDICINe eXAMINATIo N & Bo Ar D r eVIeW

CASE 5.5 HELPFUL TIP:


T su ts a v n us and a t ia b d as a ss n-
While covering the ED, a 62-year-old emale you have known tia t sam xc t t x n, w ic u can ad
or several years presents with her husband. She appears t t anscutan us x n satu ati n m nit . And,
lethargic and is unable to give a coherent history. Her hus- a v n us as u ts ss. W a d a t ia b d as s
band tells you that she began having stomach pain, nausea, and t n a m st xc usiv in ati nts n a v nti at .
and diarrhea 2 days ago. Although she has not been vomit-
ing, she has been unable to drink or eat much due to nausea.
She takes urosemide or edema and albuterol/ipratropium While you are providing supportive care, the patient’s labora-
(Combivent) or chronic obstructive pulmonary disease tory results are completed: Na 134 mEq/L, K 2.1 mEq/L, Cl
(COPD). She smokes a pack o cigarettes per day. 112 mEq/L, HCO3 10 mEq/L, BUN 29 mg/dL, Cr 1.1 mg/dL,
On physical examination, her respiratory rate is 30, pulse Ca 9.1 mg/dL; CBC: WBC 16,100 cells/mm3, Hgb 13.9 g/dL,
104 bpm, blood pressure 112/64 mm Hg, and temperature platelets 167,000 cells/mm3; urinalysis: speci c gravity 1.030,
37.9°C. She is somnolent and disoriented (maybe she’s post- remainder normal. roponin- , CK, and liver enzymes are
call?). Oral mucosa is dry. Lung examination demonstrates normal.
diminished air movement bilaterally. Her abdomen is di -
usely tender, but there is no rebound. Rectal examination is Question 5.5.2 All o the ollowing may be contributing to
negative or occult blood. hypokalemia in this patient EXCEPT:
T e rst laboratory test you have available is a room air A) Hypomagnesemia.
arterial blood gas (although venous would have been ne, B) Furosemide.
right?): pH 7.12, PaCO2 33 mm Hg, PaO2 80 mm Hg, HCO3 C) Acidosis.
10 mEq/L, and oxygen saturation 92%. D) Beta-agonists such as albuterol.
E) Diarrhea.
Question 5.5.1 This blood gas is most consistent with which
o the ollowing processes? Answer 5.5.2 The correct answer is “C.” Acidosis should cause a
A) Compensated metabolic acidosis. spurious elevation in potassium—not hypokalemia. Magnesium
B) Compensated respiratory acidosis. (“A”) depletion promotes potassium loss. T us, hypomagnesemia
C) Poorly compensated metabolic acidosis. can contribute to hypokalemia. “B,” urosemide and other loop
D) Poorly compensated respiratory acidosis. diuretics, causes renal potassium wasting. “D,” beta-agonists
E) Pure respiratory alkalosis. shif potassium into cells thereby lowering the serum potassium.
Diarrhea (“E”) causes direct gastrointestinal losses o potassium.
Other causes o hypokalemia include thiazide diuretics,
Answer 5.5.1 The correct answer is “C.” T is patient is clearly metabolic alkalosis (of en rom protracted emesis—although
acidotic, as her pH is well below the normal range o 7.35 to 7.45. this represents a shif o potassium intracellularly and not a true
So, obviously, whatever she has, it will be poorly compensated, hypokalemia), hyperaldosteronism, and R A types 1 and 2.
ruling out “A” and “B.” Based on the bicarbonate (HCO3) level
and the history o gastrointestinal losses due to diarrhea, you
would suspect a metabolic acidosis. In order to have appropriate INTERESTING (BUT USELESS) TIP:
respiratory compensation, the PaCO2 should all 12 points or Unti 1970, “ i t sa t” c ntain d it ium c id . y u
every 10-point drop in the HCO3 below the normal level (nor- c ud andma’s t nsi n and mania at t
mal HCO3 is around 24 mEq/L). sam tim ! A as in Ja an wit i it ium c nt nt
In this case, the HCO3 is 10 mEq/L (14 points below nor- in t s i av w at s d ssi n and suicid .
mal); there ore, the PaCO2 is expected to drop by about (1.2 ×
14 = 16.8) or approximately 17. However, the PaCO2 is not
23 mm Hg; it is 33 mm Hg (close to the normal range o 35–45). HELPFUL TIP:
T e patient’s PaCO2 is too high to appropriately compensate or Acid sis wi s u i us vat a ati nt’s tassium.
her metabolic acidosis, and she thus has a poorly compensated T s um tassium s u b ab ut 1 meq/l v-
metabolic acidosis. d c as in h 0.1. A ka sis wi caus an quiv-
a nt ka mia. S , i t h is 7.3, 1 meq/l t
vat d tassium is du t t acid sis.
T ere is another way to do this:
T e pH should change by 0.08 or every 10 change in CO2. So, i
a patient’s CO2 is 50, the pH should be 7.32 i it is an uncompen- Question 5.5.3 Since your administration likes to keep
sated respiratory acidosis. I they are more acidotic (e.g., 7.24), the hospital at 99% capacity, there are currently no
they have a mixed respiratory and metabolic acidosis. I they cardiac-monitored beds available or this patient. The most
are less acidotic (e.g., 7.39), they have a compensated respira- appropriate initial therapy to correct her hypokalemia
tory acidosis. (2.1 mEq/L) is to give your patient:
CHAPTER 5 • Neph r o l o g y 155

A) KCl 40 mEq orally. • Discuss caus s ka mia?


B) KCl 80 mEq orally. • Mana a ati nt wit m tab ic acid sis and ka mia?
C) KCL 20 mEq per hour IV.
D) KCl 20 mEq push IV.
E) KCl 60 mEq per hour IV. QUICK QUIZ: r h ABDo Myo l ySIS

Answer 5.5.3 The correct answer is “C.” Remember that this T e primary mechanism by which renal ailure occurs in rhab-
patient is nauseated and lethargic and has been unable to eat. domyolysis is:
T us, oral potassium replacement is not going to work. Couple A) Glomerular destruction.
this with the act that she is pro oundly hypokalemic, and IV B) Acute tubular necrosis (A N).
replacement becomes the treatment o choice. One should not C) Interstitial nephritis.
give more than 20 mEq KCl IV per hour without a cardiac mon- D) None o the above.
itor. It should be given through a large bore peripheral IV or a
central line due to venous irritation. I you chose “D,” you just The correct answer is “B.” Myoglobin deposits in the renal
ailed your test. IV push KCl is atal. tubules, causing local damage and ischemia, which results in
Most commonly, the chloride salt o potassium (KCl) is admin- acute tubular necrosis. Findings in the urinary sediment that
istered to replete potassium stores. In the conscious patient with support acute tubular necrosis include renal tubular epithelial cells
a unctional gastrointestinal tract, oral KCl should be adminis- and dark brown casts o granular material (“muddy” brown casts).
tered. Oral and intravenous bioavailability is very similar, but oral
doses greater than 40 to 60 mEq may not be well tolerated.
HELPFUL TIP:
pati nts wit abd m sis wit c atin kinas (CK)
HELPFUL TIP: < 5,000 U/l and c a u in a un ik t d v acut
T a n iab ducib wa s t au tas- tubu a n c sis but s u d b m nit d t assu
sium d ti n and am unt n d d t mak a ati nt d in CKand ad quat na uncti n.
uka mic. T b st t in t d is sta t acin wit KC
and m nit s um tassium v s, adjustin t KC
d s as u . B ca u w n d in tassium -
ac m nt: ka mia is m st c mm n iat nic. QUICK QUIZ: l UMpy KIDNeyS

Which o the ollowing is/are associated with autosomal domi-


Question 5.5.4 In addition to KCl repletion, which o the ol- nant polycystic kidney disease (ADPKD)?
lowing interventions do you initiate now in an attempt to A) Liver cysts.
correct the acidosis? B) Cerebral aneurysms.
A) Bolus normal saline. C) Colonic diverticula.
B) Sodium bicarbonate. D) Cardiac valvular disease.
C) Bolus 5% dextrose. E) All o the above.
D) Intubation and mechanical ventilation.

Answer 5.5.4 The correct answer is “A.” On the basis o his- The correct answer is “E.” All o the above are associated with
tory, physical examination, and BUN/creatinine ratio, your polycystic kidney disease. O particular importance is the pos-
patient is dehydrated. T e rst step is to correct the dehydra- sibility o cerebral aneurysms (5–20%) leading to subarachnoid
tion with intravenous uids. Normal saline is pre erred over hemorrhage. However, aneurysm rupture remains airly rare
dextrose (“C”), which might lead to urther hyponatremia and (but does seem to run in amilies—and seems to result in a
hypokalemia. In addition, dextrose will not stay intravascular really bad day or your patient). Currently, screening or sub-
and will actually precipitate a diuresis by making the serum arachnoid aneurysms in asymptomatic patients with ADPKD is
hypotonic. “B” is incorrect. T ere is no evidence that bicar- not recommended.
bonate improves outcomes in metabolic acidosis. T e best ADPKD occurs in 1 in every 400 to 2,000 live births and is,
initial approach is to correct the underlying problem. In most as the name implies, autosomal dominant. Common extrarenal
cases, volume replacement will lead to improvement in acidosis mani estations o ADPKD include cerebral aneurysms, hepatic
without needing to resort to bicarbonate. Although she is oxy- cysts, cardiac valve disease, colonic diverticula, and abdominal
genating well now, i your patient’s respiratory condition deteri- and inguinal hernias.
orates, she may require intubation (“D”) and ventilator settings
could be adjusted to aid in correcting the acid/base disorder. QUICK QUIZ: To o SAl Ty
Objectives: Did you learn to . . .
• D n acid sis and distin uis c m nsat d m A 79-year-old emale nursing home resident with moderate
c m nsat d acid sis? dementia presents or worsening con usion over the last 2 days.
156 FAMIl y MeDICINe eXAMINATIo N & Bo Ar D r eVIeW

She just nished antibiotics or a urinary tract in ection. Her tem-


HELPFUL TIP:
perature is 38.1°C, pulse 110 bpm, respiratory rate 18 breaths/
Ca cu at s a avai ab t stimat wat d icit in
minute, and blood pressure 118/60 mm Hg. She is disoriented
v mic nat mic ati nts. T a s v a
and lethargic. Examination o the heart, lungs, and abdomen is
quati ns and it’s n t c a t at n is su i t an-
unremarkable.
t . h w v , t s must b vi w d as u uid s.
Laboratory studies are as ollows: Na 165 mEq/L, K
As uids a ac d, ass ss t ati nt qu nt
4.6 mEq/L, Cl 118 mEq/L, HCO3 28 mEq/L, BUN 31 mg/dL, Cr
usin c inica and ab at m asu s.
1.1 mg/dL. Urine speci c gravity is > 1.030 and urine osmolality
is 700 mmol/kg (elevated, re ecting reabsorption o ree water).
Her CBC is normal.
CASE 5.6
Initial treatment or this patient should be: A surgical colleague asks you to consult on a patient because
A) Normal saline IV bolus. o increasing creatinine (yes, we know this will never happen
B) Dextrose 5% IV bolus. with a surgeon—just consider this a thought experiment). A
C) Sterile water IV bolus. 63-year-old woman was admitted or an elective cholecystec-
D) Loop diuretics. tomy. She is on postoperative day 3 and has ever and delir-
E) DDAVP. ium. Her current medications are morphine, ce otetan, and
acetaminophen as needed. She takes nothing by mouth, but
The correct answer is “A.” Demented or delirious patients with
has intravenous uids (5% dextrose/0.45% saline) running
an acute ebrile illness may not be able to consume enough ree
at 100 cc/hr . . . continuously . . . since surgery. Plasma studies
water to avoid hypernatremia. Several mechanisms may be at
rom the day o surgery and this morning are available:
work: ree water loss due to illness, impaired thirst, and inability
to respond to thirst due to cognitive or physical impairments.
Just as with hyponatremia, the hypernatremic patient should Laboratory Test Be ore Surgery Day o Consultation
not be corrected too quickly. Sudden changes in plasma sodium
S dium (meq/l ) 138 130
may result in cerebral edema. Although there are no standard-
ized guidelines to direct the correction o hypernatremia, most p tassium (meq/l ) 4.5 5.8
authorities recommend a maximal correction o 0.5 to 1 mEq/L/
C id (meq/l ) 103 105
hr. In this patient who appears hypovolemic, the primary con-
cern is to give volume. T e administration o normal saline h Co 3 (meq/l ) 24 18
will allow you to administer volume while lowering her plasma
BUN (m /dl ) 15 30
sodium. Dextrose 5% solution will likely lower the sodium too
quickly. Sterile water should never be administered IV because C atinin (m /dl ) 1.1 2.0
it will cause massive local hemolysis.
“E” is incorrect. T is patient’s kidney unction is appropriate
or her hypernatremia. She is concentrating her urine, as evi- Question 5.6.1 The BUN/Cr ratio on the day o consultation
denced by her high urine speci c gravity and urine osmolality. suggests that:
T ere ore, she does not have diabetes insipidus, which is char- A) She is dehydrated.
acterized by large volumes o dilute urine. DDAVP is an appro- B) She has a prerenal cause o her increased creatinine, such as
priate treatment or central diabetes insipidus. See able 5-7 or hypoper usion.
more on causes o hypernatremia. C) Intrinsic kidney disease is more likely than a prerenal cause
o her increased creatinine.
D) None o the above.
TABLE 5-7 CAUSES OF HYPERNATREMIA
Answer 5.6.1 The correct answer is “C.” A BUN/Cr ratio
• g I ss wat
• o sm tic diu sis < 20 generally indicates an intrinsic renal cause o renal ailure
• exc ss x cis and sw atin (increasing Cr) while a BUN/Cr ratio > 20 indicates a prerenal
• Diab t s insi idus cause such as hypoper usion (e.g., CHF, dehydration). In this
• D c as d acc ss t wat patient, the BUN/Cr ratio is 15, suggesting intrinsic renal disease.

Question 5.6.2 Which o the ollowing is the most appro-


HELPFUL TIP: priate rst step in determining the nature o this patient’s
g n a , u at m c anisms wi maintain a n - elevated creatinine?
ma s um s dium. h w v , t is qui s acc ss t A) Give a trial bolus o normal saline.
wat . Imm bi ati nts ( s cia t d ) a B) C scan o the abdomen.
a ticu a n t b ms b caus t i imit d C) Determine volume o urine output.
m bi it and inabi it t ind nd nt acc ss wat . D) Obtain urine or culture.
E) Give a trial dose o urosemide.
CHAPTER 5 • Neph r o l o g y 157

Answer 5.6.2 The correct answer is “C.” Currently, all you Question 5.6.3 You have now determined that your patient
know about this patient’s kidney unction is that it has declined has become oliguric. Which o the ollowing is most likely
since her admission and that it is not likely rom a prerenal to help you narrow the dif erential diagnosis o renal
cause. It is important to know i this patient is oliguric/anuric ailure?
or has adequate urine output. T is is important because it can A) Calculation o creatinine clearance.
a ect the treatment. For example, oliguric or anuric patients B) Arterial blood gases.
may become volume overloaded easily in response to IV uids. C) C scan o the abdomen.
Although hospital measurements o intake and output are of en D) Fractional excretion o sodium (FENa).
plagued by errors, you should start this evaluation by analyzing E) Furosemide challenge.
the patient’s urine output, as well as uid intake. Urine culture
and C scan may eventually play a role in your evaluation. T e Answer 5.6.3 The correct answer is “D.” In this patient, you
decision to give uids or diuretics cannot be made until more know that her BUN/Cr ratio is < 20, pointing you toward intrin-
in ormation is gathered. sic renal disease. But this ratio is not speci c enough to rely
upon. In oliguric renal ailure, the FENa is a use ul tool to help
di erentiate prerenal causes o renal ailure rom intrinsic renal
HELPFUL TIP:
causes. I the FENa is less than 1%, the kidney is unctioning
r ativ sma c an s in s um c atinin ma -
appropriately to conserve sodium and water to better per use
ct acut kidn inju ( vi us t m d “acut na
the kidney, and a prerenal cause o ailure is more likely such
ai u ”). A ati nt wit a bas in c atinin 0.7 m /
as hypoper usion rom shock, dehydration, heart ailure, etc. I
dl as st hal na uncti n w n c atinin
the FENa is greater than 1%, salt and water losses are excessive,
inc as s t 1.4 m /dl , v n t u t is numb ma
suggesting intrinsic kidney dys unction. T is calculation is most
sti b in t n ma an a a ticu a ab—t us,
meaning ul in oliguric renal ailure (urine output < 400 cc/day),
t n d t ca cu at a c atinin c a anc and/
but can also be used with any acute kidney injury in order to
MDr D g Fr . T a a numb d initi ns acut
help determine the possible underlying cause. Also, FENa may
kidn inju ( . ., r IFl e, AKIN, and KDIg o c it ia), but
be inaccurate in the elderly, patients receiving diuretics, and in
t basic mis is t at b s m bj ctiv m asu
chronic renal ailure. T e equation used to calculate FENa is:
( . ., c atinin , u in ut ut), kidn uncti n as a -
id d c in d. FENa (%) = [(urine Na/plasma Na)/
(urine Cr/plasma Cr)] × 100.

HELPFUL TIP: HELPFUL TIP:


l ik acut kidn inju , t is n sin numb t at W at i t ati nt is n a diu tic and u cann t mak
d in s i u ia. h w v , n c mm n us d d ini- us t FeNa? Us t FeU a, as u a xc ti n is n t
ti n i u ia is: ss t an 1 ml /k / in in ants, ss a ct d b diu tics. h ’s t quati n:
t an 0.5 ml /k / in c i d n, and ss t an 400 ml / FeU a (%) = [(u in u a/ asma u a)/
da adu ts. o c u s , anu ia is d in d as n u in (u in C / asma C )] × 100.
ut ut.
I t FeU a is <35%, a na caus is m ik . I
FeU a is > 50%, acut tubu a n c sis is m ik .
Cav at: c mic diu sis inc as s xc ti n
Vital signs, intake, and output have been recorded by the u a v n in v mic stat s ( . ., 3 + uc s in t
nurses or each shi since admission and are provided in u in can iv ua as i FeU a).
able 5-8.

TABLE 5-8 VITAL SIGNS, INTAKE, AND OUTPUT


Day 1 Day 1 Day 1 Day 2 Day 2 Day 2 Day 3 Day 3 Day 3
Shi t 1 Shi t 2 Shi t 3 Shi t 1 Shi t 2 Shi t 3 Shi t 1 Shi t 2 Shi t 3
Bp 118/60 117/63 102/57 80/42 92/48 102/67 105/60 102/61 102/79

pu s 84 82 92 124 117 106 100 102 93

Tm 37.3 37.0 37.7 40.0 39.1 37.1 38.0 36.9 36.8

rr 12 14 13 15 20 18 20 12 12

IV in (cc) 1,800 800 800 800 800 800 800 800 800

U in ut (cc) 1,650 1,000 750 850 600 400 180 100 80


158 FAMIl y MeDICINe eXAMINATIo N & Bo Ar D r eVIeW

On examination, you nd a mildly disoriented emale in no When you go to check on the patient later in the a ernoon, she
acute distress. She has lower extremity and sacral edema. has become more tachypneic and has rales on examination.
You obtain urine studies, showing a speci c gravity o 1.020,
pH 6, 3 RBCs/hp , 2 WBCs/hp , and muddy brown granular Question 5.6.6 Which o the ollowing recommendations
casts. T e urine creatinine is 6.5 mg/dL and the urine sodium do you make or this patient’s continuing care?
is 45 mEq/L. You calculate the FENa at > 1%. A) Administer dopamine.
B) Increase IV uids to 200 cc/hr, using 5% dextrose/0.45%
Question 5.6.4 Given the clinical course and urine ndings, saline.
which o the ollowing is the best diagnosis? C) Hemodialysis.
A) Acute tubular necrosis (A N). D) Administer hydrochlorothiazide.
B) Acute interstitial nephritis (AIN). E) Administer urosemide.
C) Vasculitis.
D) Heart ailure (HF). Answer 5.6.6 The correct answer is “E.” T e treatment o acute
E) Lactic acidosis. renal ailure due to A N is largely supportive. In this oliguric
Answer 5.6.4 The correct answer is “A.” A N is a major cause patient with signs o volume overload, a trial o a loop diuretic
o acute kidney injury in hospitalized patients. A N is the result is appropriate. Intravenous urosemide dosed at 20 to 100 mg
o toxic and/or ischemic e ects on the kidney tubules. Meta- every 6 to 12 hours is a reasonable way to start. O note, uro-
bolic derangements in A N include progressive hyponatre- semide will of en improve the symptoms o volume overload
mia, hyperkalemia, and metabolic acidosis with a high anion even i there is no diuresis; it has some direct vasodilating a ect
gap, all o which are present in this case. ypically patients with which decreases pulmonary artery pressure. “A,” dopamine, is
A N have a FENa > 1% (or FEUrea > 50%) and a urine sodium not e ective in the treatment o A N (it is not “renal sparing”
> 40 mEq/L (Remember, the unctioning kidney should be and does not increase GFR). “B,” more uid, is clearly not indi-
retaining sodium to increase its per usion with extrinsic AKI). cated in this patient who is already uid overloaded. Why not
T is is not happening here. So it is an intrinsic renal problem. dialysis you ask (“C”)? Early dialysis in A N is associated with
In this case, FENa is 9.9%. In addition, “muddy” brown casts greater mortality and increased kidney damage rom hypoten-
(renal tubular cell casts) are of en ound in the urinary sediment sion, in ection, and complement activation in the kidney. “D,”
o patients with A N. All o these ndings point to A N as the HC Z, is less likely than urosemide to achieve a good diuretic
cause o renal ailure in this patient. e ect in the setting o a low GFR.

Question 5.6.5 Based on the available data (including the


HELPFUL TIP:
vital signs in Table 5-8 and urine values), you suspect that
W i t aditi na tau t, t in t “c nv t” i u ic t
ATN in this patient is the result o :
n n i u ic na ai u (b in t kidn s wit
A) Acetaminophen.
diu tics) is n t u and is ssib amu.T
B) Hypotension.
is a t nd t wa d at m ta it in ati nts w a
C) Intrinsic renal in ection.
t at d t is wa . T ’s m : it can caus d a n ss
D) Ce otetan.
( ?), v ti , and tinnitus. r m mb , it d sn’t
E) Any o the above is equally likely.
t a d ad s ai in kidn s. W did w t ink
Answer 5.6.5 The correct answer is “B.” As you review the vital it w k d in t ast? p bab t w w
signs, intake, and output, you will notice that the patient had a ab t duc u in in s ns t diu tics w ss
hypotensive period associated with tachycardia and ever (day sick t b in wit .
2, shif s 1 and 2). She then developed progressively lower urine
output despite stable IV intake. It is likely that she has had an
Your patient has very little response to urosemide, but you
ischemic insult to her kidneys as a result o hypotension and
match her input and output and she has enough insensible
decreased per usion. While many medications can cause A N,
loss to resolve her rales. She begins to eat, and you monitor
ce otetan and acetaminophen are relatively sa e. More of en,
her uid intake care ully. You match her uid intake with
cephalosporins cause acute interstitial nephritis ( ever, + /−
output, giving normal saline to match her urine output and
rash, white cell casts, and eosinophils in the urine and perhaps
suspected insensible losses. You treat her hyperkalemia with
in the peripheral blood).
sodium polystyrene sul onate without sorbitol (Kayexalate),
HELPFUL TIP: which is e ective. Her BUN and creatinine continue to rise
In additi n t xic insu t (s ck, usi n, over the next two days and reach 60 and 4 mg/dL, respec-
h F, tc.), c mm n caus s ATN inc ud m dicati ns tively. T e plasma HCO3 is 18 mEq/L.
suc as tac imus, va i us ACe in ibit s, s m t
nici in d ivativ s, ntamicin, t b am cin, c c - Question 5.6.7 You now recommend:
s amid , s v a anti- j cti n d u s, and m . A) Hemodialysis.
B) Sodium bicarbonate.
CHAPTER 5 • Neph r o l o g y 159

C) Strict protein restriction.


D) Continuing to match intake and output. CASE 5.7
E) All o the above. While on call, you admit a 75-year-old emale or con usion.
One month ago, she started HC Z or hypertension. Her
Answer 5.6.7 The correct answer is “D.” At this point, there medications include HC Z 25 mg daily, levothyroxine 125
is no reason to change your therapeutic approach. In particu- mcg daily, aspirin 81 mg daily, sertraline (Zolo ) 50 mg daily,
lar, hemodialysis is not necessary. Indications or hemodialysis and atorvastatin (Lipitor) 40 mg daily. She has a 50-pack-year
in this patient might include symptomatic uremia (e.g., coma, history o tobacco use and continues to smoke.
pericarditis), severe hyperkalemia or acidosis unresponsive to On examination, you nd an irritable, con used emale in
other therapies, and complications o volume overload (e.g., no acute distress. Her admission vitals: BP 100/60 mm Hg,
pulmonary edema). Dialysis is actually associated with worse P 120 bpm, RR 12 breaths/minute, 36.1°C, weight 50 kg.
outcomes and prolonged renal disease (complement xation Her oral mucosa is dry and she has poor skin turgor. She has
in the kidney produces additional injury, episodic hypoten- clear lungs, an S4 on heart examination, and no edema. Her
sion rom dialysis increases kidney injury). Sodium bicarbon- neurological examination is non ocal.
ate is used orally in patients with signi cant chronic acidosis in Laboratory results: Na 110 mEq/L (low), K 3.0 mEq/L
end-stage renal disease, but many patients can tolerate the tem- (low), Cl 70 mEq/L (low), HCO3 33 mEq/L (high), BUN
porary mild acidosis associated with acute kidney injury. Dra- 30 mg/dL, Cr 1.0 mg/dL, glucose 110 mg/dL. Plasma osmo-
matic protein restriction will probably help to control uremia, lality 220 mOsm/L (normal 270–299 mOsm/L).
but lower BUN should not be your primary goal. Your patient is
recovering rom surgery and was probably septic. She requires Question 5.7.1 Which o the ollowing statements is true
good nutrition to continue to heal. regarding the etiology o her hyponatremia?
A) She has pseudohyponatremia.
B) She has isovolemic hyponatremia.
HELPFUL TIP:
C) She has hypovolemic hyponatremia.
Acut kidn inju du t ATN t ica asts 7 t
D) She has hypervolemic hyponatremia.
21 da s, wit na uncti n tu nin as t tubu a
c s n at . h w v , t c u s is i va iab
Answer 5.7.1 The correct answer is “C.” T e de nition o hypo-
and d nds n t ati nt’s n a a t and t
natremia is a plasma sodium concentration below 135 mEq/L.
n t and d t initia inju .
T e evaluation o hyponatremia begins with the determination
o the validity o the plasma sodium measurement. First ques-
Question 5.6.8 In patients with acute tubular necrosis, the tion: is this pseudohyponatremia (“A”)? Look at the measured
most common cause o death is: serum osmolality. I it is low (as it is here), the patient does
A) Hemorrhage. not have pseudohyponatremia. Pseudohyponatremia is caused
B) Adverse event o hemodialysis. by either hyperlipidemia or hyperproteinemia decreasing the
C) In ection. water content o plasma. T is distinction was more important
D) rans usion reaction. in the past; newer laboratory techniques compensate or pseu-
E) Heart ailure secondary to uid overload. dohyponatremia.
Hyperglycemia and severe uremia can cause hyponatremia
Answer 5.6.8 The correct answer is “C.” In patients with A N (not called “pseudohyponatremia” or reasons that the editors
who die, in ection is the usual culprit. It is critical to avoid sepsis don’t ully understand). In an instance where a patient’s mea-
in these patients. Also, serious in ection resulting in sepsis is a sured sodium concentration is hyponatremic and his glucose
common cause o A N. It’s the circle o li e . . . or death, in this is 800 mg/dL, the measured serum osmolality may be normal
instance. or elevated (due to the elevated glucose, which “counts” as
osmoles) and should be compared to the calculated osmolality
en days a er her surgery, your patient’s urine output using the equation:
increases markedly. Her BUN and creatinine return to their
premorbid levels. T e National Kidney Foundation makes Osmolality = 2 (sodium) + glucose/18 + (BUN/2.8)
you an honorary nephrologist and awards you the Bronze
I the calculated osmolality approximates the measured osmo-
Nephron.
lality, then much o the “hyponatremia” will autocorrect when
Objectives: Did you learn to . . . the hyperglycemia is corrected. T ere ore, you would not want
• eva uat a s ita iz d ati nt acut kidn inju ? to aggressively correct the serum sodium concentration without
• Us u in studi s, inc udin u in s dium and FeNa, t assist consideration o the patient’s hyperglycemia. When in doubt,
in t dia n sis acut kidn inju ? requent sodium monitoring is advisable.
• Id nti acut tubu a n c sis? T e next step in the evaluation o hyponatremia is to deter-
• D sc ib c mm n caus s acut tubu a n c sis, its t at- mine the patient’s volume status. T is patient has no signs o vol-
m nt, and its n sis? ume overload, such as edema or crackles, making hypervolemia
160 FAMIl y MeDICINe eXAMINATIo N & Bo Ar D r eVIeW

unlikely. She is tachycardic with a low blood pressure, dry hypoosmolality). T us, the urine should have a speci c gravity
mucous membranes, and a BUN/Cr ratio greater than 20, sug- less than 1.005 and osmolality less than 100 mmol/kg.
gesting she is hypovolemic. Most likely, she has a hypovolemic Inappropriately concentrated urine (osmolality > 100 mmol/
hyponatremia. kg) occurs when there is limited excretion o uid and may be
observed in SIADH, HF, cirrhosis, and renal ailure. T is is also
re ected in the urine sodium. I the kidneys respond to hypona-
HELPFUL TIP:
tremia as expected, urine sodium concentration should be low,
A wa s c nsid “b t mania” ( n u av it
typically less than 20 mmol/L and of en less than 10 mmol/L (the
dis as nam s . . . n t t m nti n n u av it
kidneys are attempting to retain sodium to increase the serum
d u s). h is w at a ns: t muc b (
osmolarity). In the setting o hypovolemia, i the urine sodium
an a c ) us t itt s ut qua wat
concentration is inappropriately high, something is inappropri-
xc ti n and nat mia.
ately spurring the kidney to excrete sodium (e.g., diuretic use,
hypoaldosteronism). In this patient, the elevated urine sodium
Question 5.7.2 What urther in ormation will help you is likely due to the diuretic. Diuretic use is the most common
narrow the dif erential diagnosis o hyponatremia in this cause o hypovolemic hypoosmolality, and thiazides are more
patient? commonly associated with hyponatremia than are loop diuret-
A) FENa. ics ( urosemide, etc.). Your patient is hypovolemic and hypona-
B) Urine osmolality and urine sodium concentration. tremic, with a high urine sodium concentration and the history
C) Urine creatinine concentration. o diuretic use: most likely, her hyponatremia is diuretic induced.
D) Urine potassium and calcium concentration. “D,” a reset osmostat, requires special mention. A reset
osmostat, which is responsible or 20% to 30% o hyponatremia,
Answer 5.7.2 The correct answer is “B.” Urine osmolality occurs when the patient’s body “adapts” to hyponatremia and
can be used to distinguish between impaired water excretion gives up trying to correct the problem: the kidneys just throw in
caused by SIADH (syndrome o inappropriate secretion o anti- the towel. Patients with a reset osmostat will present like SIADH
diuretic hormone), and pathologic water intake (polydipsia). (hyponatremia, euvolemia or slight hypervolemia, and inappro-
In SIADH, the urine will be inappropriately concentrated with priately concentrated urine) but will be resistant to treatment.
urine osmoles o > 100 mosmol/kg (the patient is unable to I a patient with apparent SIADH does not respond to the usual
retain sodium). FENa is discussed earlier; although the word treatment, consider a reset osmostat. Since patients with a reset
“sodium” is in it, FENa is not help ul or discovering the etiol- osmostat generally have mild hyponatremia (125–130 mg/dL)
ogy o hyponatremia. Concentration o potassium and calcium and are generally asymptomatic, trying to correct the sodium by
is not likely to add any use ul in ormation. limiting water intake is unnecessary and utile.

You order urine studies. Urinalysis: speci c gravity 1.025, pH HELPFUL TIP:
5.0, trace protein, 0 to 1 RBC/hp , 0 to 1 WBC/hp , otherwise W n u nc unt a ati nt wit nat mic
negative. Spot urine Na 70 mEq/L (elevated) and urine osmo- sm a it , a s k at t asma tassium
lality = 700 mmol/kg (elevated). c nc nt ati n. Diu tic us is a c mm n caus -
nat mia, ka mia, c mia, and s-
Question 5.7.3 Given the patient’s history, urine studies,
m a it ccu in simu tan us .
and hypovolemic status, what is the most likely diagnosis?
A) Syndrome o inappropriate secretion o anti-diuretic hor-
mone (SIADH). Question 5.7.4 In addition to discontinuing her diuretic,
B) Heart ailure (HF). which o the ollowing approaches is the best initial ther-
C) Diuretic-induced hyponatremia. apy or her hyponatremia?
D) Hyponatremia due to reset osmostat. A) Saline 0.9% 1 L bolus ollowed by 150 cc/hr.
E) Potato chip de ciency (hypopringlism). B) Fluid restriction to 1,500 cc/day.
C) Saline 3% 100 cc/hr or 24 hours.
Answer 5.7.3 The correct answer is “C.” In order to be use ul D) Saline 0.45% 100 cc/hr or 24 hours.
in the diagnosis o hyponatremia, urine studies must be viewed E) Saline 3% at 60 cc/hr and urosemide 40 mg IV.
in the context o volume status, plasma osmolality, and electro-
lyte levels. T is patient’s laboratory results are consistent with Answer 5.7.4 The correct answer is “A.” T is patient is hypo-
SIADH. However, patients with SIADH should be euvolemic or tensive and tachycardic and thus needs volume somewhat
slightly hypervolemic. T us, her laboratory results and clinical quickly to address her abnormal vital signs. I this patient was
picture are most consistent with diuretic-induced hyponatre- not hemodynamically compromised, you could orgo the uid
mia. Note that she also has a contraction alkalosis (low chloride, bolus. Excessively rapid correction o hyponatremia may lead to
elevated bicarbonate). pontine myelinolysis.
T e appropriate response o the kidney to hypoosmolality is Sodium concentrations less than 120 mEq/L are considered
to make maximally dilute urine (retain sodium to correct the severe hyponatremia. Patients with acute, severe hyponatremia
CHAPTER 5 • Neph r o l o g y 161

are almost always symptomatic as a result o the low sodium. By Question 5.7.5 In addition to HCTZ, which medication
de nition, acute hyponatremia has been present or 48 hours should you discontinue because o its potential role in
or less and chronic hyponatremia or more than 48 hours. As hyponatremia in this patient?
previously discussed, your patient’s hyponatremia is due to a A) Aspirin.
diuretic, which she started 1 month ago. T ere ore, her hypona- B) Atorvastatin.
tremia is more likely to be chronic. C) Levothyroxine.
As a general rule, hypovolemic hyponatremia should be cor- D) Sertraline.
rected by volume in usion, usually with 0.9% (normal) saline.
Fluid restriction (“C”), while good treatment or SIADH, is Answer 5.7.5 The correct answer is “D.” Serotonin reuptake
not appropriate in this patient who is already volume depleted. inhibitors (SSRIs) and tricyclic antidepressants, among other
Likewise, a loop diuretic is not a good approach (“E”). Both medications, can stimulate the release o ADH rom the pitu-
“C” and “E” will exacerbate the problem. Hypo-osmolar solu- itary gland, ultimately causing hyponatremia with an SIADH-
tions, like 0.45% saline (“D”), will lead to more severe hypo- type presentation. T e association between SSRIs and hypona-
natremia. tremia is strong enough to consider measuring plasma sodium
Your patient has chronic hyponatremia, which needs to be levels in elderly patients be ore and af er starting an SSRI. T e
corrected more slowly than acute hyponatremia. Chronic hypo- other medications are not associated with hyponatremia to any
natremia should be corrected no aster than 0.5 mEq/hr or 12 signi cant degree.
mEq/day. For signi cantly symptomatic patients with acute
hyponatremia, sodium can be corrected more quickly with Your patient does well, recovers rom her hyponatremia, and
maximum increases in plasma sodium concentration o 1 mEq/ is discharged. In place o HC Z, you start an ACE inhibitor
hr or 20 mEq/day. or hypertension. You next see her a ew months later when
In order to determine how quickly sodium concentrations she starts to have problems with mild con usion. Her am-
will rise, you must know the concentrations o the solution ily reports that she is de nitely not taking any diuretics. Her
being in used and the patient’s plasma sodium. A liter o saline vitals are normal. Clinically, she appears euvolemic, and her
will a ect the serum sodium by the ollowing calculation: examination is non ocal. Her plasma sodium is 120 mEq/L,
creatinine 1.1 mg/dL, plasma osmolality 240 mmol/kg, and
Na increase = (solution Na − plasma Na)/ urine osmolality 320 mmol/kg. Urinalysis shows speci c
(total body water + 1). gravity o 1.030 but is normal otherwise.
Patient’s Na = 110 mEq/L.
Na in 3% saline = 513 mEq/L. Question 5.7.6 In addition to the laboratory tests already
Na in 0.9% saline = 154 mEq/L. available to you, which o the ollowing laboratory tests
otal body water = weight (kg) × 0.5 ( emales) or 0.6 (males). should be done in patients with hyponatremia?
A) CBC.
When using 3% saline solution, 1 L will increase the plasma B) ESR.
sodium as ollows: C) SH.
D) Liver enzymes.
Na increase = (513 – 110)/[(50 × 0.5) + 1] E) ADH.
= 15.5 mEq/L.
Answer 5.7.6 The correct answer is “C.” Hyponatremia with
At 100 cc/hr, the plasma sodium will increase at a rate o hypoosmolality and a normal volume status is of en the result
1.55 mEq/hr. o SIADH. However, SIADH is a diagnosis o exclusion. Both
T is correction is too rapid or chronic hyponatremia. hypothyroidism and glucocorticoid de ciency present with
When using normal saline (0.9%), 1 L will increase the similar eatures and should be ruled out. Hypothyroidism is
plasma sodium as ollows: more common, and at a minimum you must check SH. O
special note is option “E.” Most o the time, ADH levels are not
Na increase = (154 – 110)/[(50 × 0.5) + 1] = 1.7 mEq/L.
help ul in the diagnosis o SIADH because up to 20% o patients
T ere ore, 1 L o normal saline will increase the plasma sodium with diagnosable SIADH do not have elevated plasma ADH lev-
by 1.7 mEq initially, and 150 cc/hr will increase the plasma els. SIADH is diagnosed on clinical grounds, with characteristic
sodium by 0.25 mEq/hr, well within the sa e range. laboratory data, and lack o a better explanation.

HELPFUL TIP: HELPFUL TIP:


W n v c ctin s dium, w t wit na- In an iv n ati nt wit nat mia, w at mak s SI-
t mia nat mia, c ck s um s dium c nc n- ADh ik ? A t win data a us d t su t
t ati ns qu nt — v 6 u s is as nab in t t dia n sis SIADh : d c as d asma sm a it ; in-
initia t atm nt i d. a iat c nc nt at d u in ( . ., a u in sm a it
162 FAMIl y MeDICINe eXAMINATIo N & Bo Ar D r eVIeW

> 100 m sm /k ); c inica uv mia mi d - HELPFUL TIP: C niva tan (Va is ®) and t va tan
v mia; vat d u in s dium xc ti n (u in s dium (Samsca) b t a ADh anta nists t at can b us d
> 20 meq/l ); and t abs nc diu tic us , t - SIADh . T a t i d- in a nts. p t ntia b ms
idism, and ad na d s uncti n. inc ud xac bati ns a t ai u , t static -
t nsi n, and ka mia. T must b sta t d in t
s ita b caus t t ntia sid cts and a id
s um s dium c an s.
You order a number o laboratory tests, including SH, CBC,
and electrolytes. In addition to the laboratory tests men-
tioned above, abnormal tests include plasma Cl 90 mEq/L Objectives: Did you learn to . . .
(low), urine Na 50 mEq/L (high) (and urine osmoles, as noted • g n at a di ntia dia n sis nat mia?
above, are 320 mmol/kg). T yroid unction tests are normal. • Id nti m dicati ns t at a c mm n ass ciat d wit
You diagnose SIADH. nat mia?
• Us u in sm a it t id nti im ai d na wat and in ?
• Ca cu at s dium c cti n usin di nt c nc nt ati ns
Question 5.7.7 As initial treatment, you prescribe:
sa in ?
A) Demeclocycline.
• Dia n s , va uat , and t at a ati nt wit SIADh ?
B) Lithium.
C) Saline 0.9% bolus.
D) Furosemide. CASE 5.8
E) Water restriction. A 65-year-old male with a new diagnosis o heart ailure
returns to your of ce a er starting several new medications
Answer 5.7.7 The correct answer is “E.” T e labora- within the last month. A cardiologist at Ivory ower Aca-
tory results and clinical picture are consistent with SIADH demic Medical Center 100 miles away started these medica-
(euvolemia, elevated urine sodium excretion, elevated urine tions but never sent a note, and neither you nor your patient
osmolarity). Water restriction is the mainstay o therapy in knows what drugs he’s taking (“Well, there’s a little white one
SIADH. Free water should be restricted to 1 to 2 L/day. Dem- and an oval one and . . .”). Over that same time period, he
eclocycline and lithium inter ere with the activity o ADH at states that he has elt “worse than I did a er my heart attack.”
the collecting tubules, but these drugs are reserved or SIADH At rst he was just atigued, but in the last ew days, he has
patients with severe hyponatremia unresponsive to water developed nausea, vomiting, and body aches.
restriction. Saline in usion corrects hypovolemic hyponatre- On examination, his vitals are 37.1°C, P 70 bpm, RR
mia, and urosemide is used in hypervolemic hyponatremia 8 breaths/minute, BP 100/58 mm Hg. He has trace pedal
(e.g., CHF, renal insu ciency). Neither o these conditions is edema. His lungs are clear, and his abdomen is di usely ten-
present in this patient. der. When you stand him up to check his blood pressure, he
loses consciousness but quickly recovers when placed supine
( ortunately, you are strong and possess quick re exes and
Question 5.7.8 The patient ails water restriction. The next
caught him on the way down).
step in the treatment o this patient is:
Your nurse draws blood, starts an IV, and obtains a venous
A) Demeclocycline.
blood gas on room air. VBG: pH 7.52, PaCO2 49 mm Hg,
B) Lithium.
PaO2 40 mm Hg, and HCO3 39 mEq/L. His oxygen saturation
C) Sodium chloride tablets.
on monitor is 95%.
D) IV urea.
E) None o the above.
Question 5.8.1 The blood gas is consistent with the diag-
nosis o :
Answer 5.7.8 The correct answer is “C.” Increasing salt intake A) Metabolic alkalosis with respiratory compensation.
and adding a loop diuretic are other ways to treat SIADH in B) Respiratory alkalosis with metabolic compensation.
patients who cannot or will not maintain uid restriction. T e C) Mixed metabolic/respiratory alkalosis.
aggressiveness o the intervention will depend on patient- D) Mixed metabolic alkalosis and respiratory acidosis.
speci c actors, including symptoms and comorbidities. In an
outpatient who is minimally symptomatic or asymptomatic, Answer 5.8.1 The correct answer is “A.” Your patient appears
oral salt supplementation is most reasonable. IV urea (“D”), to have a pure metabolic alkalosis with respiratory compensa-
which causes an osmotic diuresis, could be considered as the tion. His pH is above the upper limit o normal (7.45), and his
next step af er sodium chloride tablets. Demeclocycline and plasma bicarbonate is elevated as well. In metabolic alkalosis,
lithium should be reserved or patients who ail the other treat- you can expect the PaCO2 to rise in proportion to the rise in
ments. In more severe hyponatremia and those with signi - HCO3. PaCO2 should increase by 0.5 to 0.75 times the increase
cant symptoms, admission or more aggressive therapy may be in HCO3 rom baseline (about 24 mEq/L). In this case, the
warranted. HCO3 has increased by 15 (39 – 24 = 15), and the PaCO2 has
CHAPTER 5 • Neph r o l o g y 163

appropriately increased by 9 (9/15 = 0.6). And remember, you TABLE 5-9 CAUSES OF METABOLIC ALKALOSIS
cannot overcorrect your pH (although it is quite possible simul- DIVIDED BY URINE CHLORIDE LEVELS
taneously to have two processes—an acidosis and an alkalosis
Low Urine Chloride
resulting in a normal pH). T us, the primary process has to be
(<25 mEq/L) High Urine Chloride (>40 mEq/L)
an alkalosis.
V mitin h ka mia (s v )
Question 5.8.2 Which o the ollowing urine tests will aid in
Nas ast ic sucti nin Diu tics ( t iazid ; a ct)
determining the cause o this patient’s metabolic alkalosis?
A) Urine sodium. Factiti us dia a A ka i ad
B) Urine potassium. ( axativ abus )
C) Urine chloride. C stic b sis Ba tt /g it man s nd m s
D) Urine bicarbonate.
E) Urine creatinine. l wc id intak p ima min a c tic id xc ss
( a d st nism, c tic st ids)
Answer 5.8.2 The correct answer is “C.” You can approach the
p st ca nia
di erential diagnosis o metabolic alkalosis by measuring the
urine chloride (see next question). T e other urine studies Diu tics (
are less use ul in metabolic alkalosis. O course, an appropri- t iazid ; at ct)
ate history and physical as well as plasma tests are required to
determine the correct diagnosis and treatment.

You have admitted the patient and are on your way to the hos- C) Administer hydrochloric acid through an NG tube.
pital to see him when the laboratory tests return (you aren’t D) In use normal (0.9%) saline.
driving and talking on the phone at the same time are you?). E) Administer ranitidine.
Plasmastudies:Na140 mEq/L, K2.5 mEq/L(low), Cl 102 mEq/L
(normal), BUN 18 mg/dL, creatinine 0.9 mg/dL, Ca 8.4 mg/dL, Answer 5.8.4 The correct answer is “C.” Chloride-responsive
Mg 1.7 mg/dL (marginally low). metabolic alkalosis is usually secondary to volume contraction.
Urine studies: speci c gravity 1.025, chloride 45 mEq/L Your patient has renal salt and water losses due to a diuretic
(elevated). and has gastrointestinal volume losses due to emesis. Physical
Question 5.8.3 O the ollowing choices, which is NOT likely examination (except trace edema—but remember that he has
to contribute to metabolic alkalosis in this patient? heart ailure) and laboratory ndings support hypovolemia. T e
A) Corticosteroids. diuretic should be discontinued and volume should be replaced.
B) Diuretics. An isotonic solution, such as normal saline, is an appropriate
C) Hypokalemia. choice. However, you must monitor your patient’s volume status
D) Vomiting. closely due to his heart ailure. Because his potassium is low, he
requires KCl. An H 2-blocker, like ranitidine, is a good choice or
Answer 5.8.3 The correct answer is “D.” T is makes intuitive a patient with alkalosis and gastrointestinal illness: it will reduce
sense. I there is vomiting (or NG suctioning), the patient will acid secretion in the stomach (and thus urther acid loss) and
have low plasma chloride and thus will have little in the urine, may o er some symptomatic relie as well.
where it is being re-absorbed. All o the rest o the answers will Hydrochloric acid is an option in severe metabolic alkalosis
cause an elevated urine chloride and thus are consistent with unresponsive to other therapies. However, it is not given by NG
this patient’s picture. With mineralocorticoid excess (aldoste- tube. HCl is very corrosive and should only be in used centrally.
rone or extrinsic corticosteroids, “A,” one excretes potassium,
causing hypokalemia. Hypokalemia prevents the kidneys rom Question 5.8.5 Your patient is not responding to the KCl you
optimally re-absorbing chloride (“C”). See able 5-9 or more are administering. You give which o the ollowing electro-
on causes o metabolic alkalosis. lytes to aid in repleting his potassium stores?
A) Sodium.
It only took your nurse 40 minutes, but now you have the car- B) Bicarbonate.
diologist’s notes and the medication list. Your patient started C) Calcium.
taking atorvastatin, captopril, urosemide, isosorbide dini- D) Phosphate.
trate, aspirin, and metoprolol in the last month—and you are E) Magnesium.
not surprised he has orthostatic hypotension.
Answer 5.8.5 The correct answer is “E.” Give magnesium
Question 5.8.4 All o the ollowing are appropriate inter- be ore giving more potassium. Recall that his plasma mag-
ventions in this patient EXCEPT: nesium level was slightly low at 1.7 mg/dL. Even i a patient’s
A) Discontinue urosemide. magnesium is only slightly low or even low-normal, contin-
B) Administer potassium chloride. ued hypokalemia in the ace o KCl repletion may indicate
164 FAMIl y MeDICINe eXAMINATIo N & Bo Ar D r eVIeW

whole-body magnesium depletion. Serum magnesium levels sleeps in a chair due to orthopnea. He has severe atigue and
may not accurately re ect whole-body magnesium stores, espe- a mild, di use headache. Four days ago, he was seen in an
cially in patients with heart ailure. Furosemide causes urther urgent care clinic and diagnosed with bronchitis. He reports
magnesium wasting. None o the other electrolytes listed earlier no medical problems or surgeries. He quit smoking 1 year
is as important to potassium repletion. ago and denies alcohol and drug use. He has a strong amily
history o hypertension. T e review o systems is otherwise
With correction o uid and electrolyte status, your patient negative.
recovers quickly. You try to convey to the cardiologist the On physical examination, his vitals are 36.8°C,
importance o good communication. However, as noted by P 104 bpm, RR 16 breaths/minute, and BP 200/118 mm Hg.
George Bernard Shaw, “T e single biggest problem in com- T ere are bibasilar crackles with dullness to percussion at
munication is the illusion that it has taken place.” the lung bases. T e heart, abdomen, and extremities are
unremarkable.
Objectives: Did you learn to . . .
His chest x-ray shows cardiomegaly, cephalization o lung
• Id nti m tab ic a ka sis?
markings, and bilateral small pleural e usions. An ECG
• Uti iz u in c id in t va uati n m tab ic a ka -
shows sinus tachycardia with le atrial enlargement and le
sis?
ventricular hypertrophy. Laboratory results: troponin- neg-
• r c niz va i us caus s m tab ic a ka sis?
ative, hemoglobin 9.1 g/dL (low), WBC count and platelets
• r c niz t ma n s mia in t atin ka- normal, Na 136 mEq/L, K 4.4 mEq/L, Cl 96 mEq/L, HCO3
mia?
19 mEq/L (low), BUN 108 mg/dL (high), Cr 11.9 mg/dL
(high), glucose 104 mg/dL, calcium 7.8 mg/dL (low), albumin
4 g/dL (normal).
QUICK QUIZ: h eMATUr IA
Question 5.9.1 Which o the ollowing is the most appropri-
A 25-year-old male medical student presents with blood in his ate next step?
urine. When asked urther about this nding, he admits that he A) Prescribe levo oxacin and discharge patient with ollow-up
was using urine dipsticks at home to check his urine. (Did you the next day.
know that 20% o medical students meet the criteria or somati- B) Prescribe urosemide and discharge patient with ollow-up
zation disorder—now known as “somatic symptom disorder” in the next day.
DSM V—at some time during medical school? It’s true.) He has C) Administer a bolus o normal saline intravenously and
no gross hematuria, just 1+ blood on his urine sel -test. He has admit the patient.
no symptoms otherwise. His major concern is that his ather D) Administer urosemide intravenously and admit the patient.
had a renal transplant at age 30 or Alport syndrome. E) Per orm thoracentesis or diagnostic purposes.

Given that his ather had Alport syndrome, what is the prob- Answer 5.9.1 The correct answer is “D.” T e proper disposi-
ability that this student has it? tion o this patient is the hospital. His uremia is quite severe,
A) Certain (100% chance). and he is symptomatic rom his renal ailure. He needs urther
B) One in two. diagnostic tests and requires urther monitoring. He has signs
C) One in our. and symptoms o volume overload, and he may yet respond to
D) Less than 1 in 20. loop diuretics. A trial o IV urosemide is reasonable. Even i
it doesn’t cause a diuresis, as noted previously urosemide can
The correct answer is “D.” Alport syndrome, also known as decrease pulmonary capillary wedge pressure. T e initial diag-
hereditary nephritis, typically presents with microscopic hema- nosis o bronchitis was most likely erroneous and switching to
turia and progresses to complete renal ailure. Alport syndrome another antibiotic will only perpetuate that error. As he is vol-
has a heterogeneous inheritance pattern: 80% o cases are due to ume overloaded, you certainly do not want to administer more
X-linked disease, about 15% are autosomal recessive, and about volume. I you are willing to attribute his pleural e usions to
5% are autosomal dominant. Since most cases are X-linked, volume overload due to kidney ailure, a thoracentesis is not
males are a ected more severely and earlier than emales. I necessary.
your medical student’s ather had Alport syndrome, it was most
likely X-linked, and the patient could not possibly have inher- Question 5.9.2 The patient’s ECG is normal. How should the
ited it. However, there is a 5% chance that the ather had autoso- low calcium be approached in the ED?
mal-dominant disease and then a 50% chance that the trait was A) Administer calcium gluconate intravenously.
passed on to the patient; there ore, “D” is correct. B) Obtain a serum parathyroid hormone level.
C) Monitor or signs and symptoms o hypocalcemia.
D) Obtain an ionized calcium level.
CASE 5.9
A 33-year-old male presents to the ED complaining o Answer 5.9.2 The correct answer is “C.” Currently, your
shortness o breath and cough o 10 days duration. He now patient does not have signs and symptoms o hypocalcemia;
CHAPTER 5 • Neph r o l o g y 165

there ore, he does not need IV calcium replacement in the ED platelet aggregation and will reduce bleeding time or about
(“A”). He should be observed or the development o signs and 24 hours af er administration. Estrogen is another option or
symptoms. Symptoms are mostly neurological—generalized renal ailure-related bleeding and has been used in men or short
seizures, perioral paresthesias, and carpopedal spasms. T e two durations and seems to be well tolerated.
best-known signs o hypocalcemia are Chvostek and rousseau
signs. Chvostek sign is present i a grimace occurs in response Question 5.9.4 In order to reduce the risk o renal osteo-
to tapping the acial nerve. rousseau sign is evoked by in ating dystrophy (elevated serum parathyroid hormone with
a blood pressure cu above the systolic pressure or 3 minutes mobilization o calcium rom bone), which o the ollowing
and observing or hand spasms. medications will you prescribe initially?
Since you expect that the patient’s hypocalcemia is secondary A) Calcium carbonate.
to renal disease and hyperphosphatemia and you will treat him B) Aluminum hydroxide.
symptomatically, serum P H level (“B”) is not required. It is not C) Magnesium hydroxide.
necessary to measure ionized calcium (“D”). You have access to D) Sevelamer (Renagel®).
both the total serum calcium level and the serum albumin, you E) Vitamin D.
can calculate the corrected calcium. In addition, no correction
is required since the albumin level is normal. Answer 5.9.4 The correct answer is “A.” Renal osteodystrophy
occurs when parathyroid hormone levels are elevated and bone
is mobilized. T is occurs because patients with renal ailure can-
HELPFUL TIP: not clear phosphate. T e body tries to compensate by increasing
I t ati nt d s n t s nd t u s mid , IV parathyroid hormone secretion, which reduces phosphate and
nit c in wi ik b succ ss u at ducin t is increases serum calcium. However, it also is detrimental to bone
ati nt’s u m na d ma b d c asin ad and and leads to demineralization.
at ad. Gastrointestinal binding o phosphate requires large doses o
a cation such as calcium (2 g/day). Calcium carbonate is associ-
ated with the least potential toxicity; there ore, it is the initial
choice or treating hyperphosphatemia to reduce the risk o
Your patient is admitted. Initially, his urine output increases renal osteodystrophy. Also, you may consider calcium acetate,
slightly with loop diuretics, but then he becomes oliguric. which is as sa e as calcium carbonate and is a more potent phos-
You ask or a nephrology consult to assist in management phate binder but more expensive. Avoid aluminum and magne-
o this case. T e nephrologist plans to place an IV catheter sium products in renal ailure as these ions accumulate and can
or dialysis and is considering a renal biopsy. I the patient cause toxicity.
develops bleeding with these procedures, hemostasis may be Sevelamer is a cationic polymer that binds phosphate in the
impaired because o his uremia. gastrointestinal tract and avoids the problems that can occur
with the calcium, magnesium, and aluminum compounds. It is
Question 5.9.3 Which o the ollowing treatments is LEAST expensive and does not seem to be superior to calcium, so cal-
likely to reduce the risk o bleeding in this patient? cium products should be tried rst.
A) Hemodialysis. Patients with chronic renal ailure or end-stage renal disease
B) DDAVP. on dialysis should also receive vitamin D. However, vitamin D
C) Cryoprecipitate. causes increased gastrointestinal absorption o phosphate, so
D) Platelet trans usion. it should be given only af er hyperphosphatemia has been
E) Conjugated estrogens. controlled.

Answer 5.9.3 The correct answer is “D.” T e cause o bleed-


ing dys unction in uremia appears to be the e ect o uremic HELPFUL TIP:
toxins on platelet unction. Giving a uremic patient more plate- W n v sc ibin ca cium na st d st -
lets—especially when there is no thrombocytopenia—will not , av ati nts tak it wit m a s— t wis it wi
improve the situation, as the platelet dys unction due to ure- n t bind s at . A s , ca cium is abs b d b tt
mia will occur with the new platelets as well. “A,” hemodialysis, w n tak n wit d.
will remove toxins related to uremia, improving platelet unc-
tion. “B,” DDAVP, is of en the rst-line treatment in a bleeding
patient with uremia. DDAVP is not generally toxic and quickly
reduces bleeding time. It acts by causing the release o actor HELPFUL TIP:
VIII:vWF multimers rom endothelial storage sites. Un or- Cinaca c t (S nsi a ) is an t ti n t atin -
tunately, patients rapidly develop tachyphylaxis to DDAVP; na st d st . B mimickin ca cium in t a a-
once the multimers are depleted rom the endothelial cells, t id, it st a at id int t inkin t at t
DDAVP will not work until these multimers are replaced (usu- s um ca cium is n ma and t us duc s t ut ut
ally a process o several days). “C,” cryoprecipitate, enhances
166 FAMIl y MeDICINe eXAMINATIo N & Bo Ar D r eVIeW

C) Post-streptococcal glomerulonephritis.
a at id m n . S unds c , but t ’s a ic
D) IgA nephropathy.
b in c —in t is cas s v a t usand d a s
E) Membranoproli erative glomerulonephritis.
m nt (in 2015). T a s v a t n w s at
bind s inc udin ic cit at (Au xia), w ic a s in-
Answer 5.10.2 The correct answer is “A.” Minimal change
c as s s um i n v s, and ant anum ca b nat
disease usually presents with clinical signs and symptoms o
(F s n ), w ic qui s w i s t an t ti ns
nephrotic syndrome and not gross hematuria. All o the other
but must b c w d c m t (makin it a ss att ac-
diseases are associated with hematuria, either microscopic or
tiv ti n u ati nts wit ut t t ). B t t s
gross. Henoch–Schönlein purpura, post-streptococcal glo-
a ativ x nsiv (a ximat $800/m nt ).
merulonephritis, IgA nephropathy, and membranoproli era-
tive glomerulonephritis all have more “nephritic” eatures with
“active” urinary sediments (dysmorphic red cells, red cell casts,
Objective: Did you learn to . . .
granular casts, white cells, and protein in the urine). T ese dis-
• Antici at c m icati ns na ai u , inc udin -
v mia, ca c mia, at t d s uncti n, and na eases also have a similar pathologic process in which immune
st d st ? complexes deposit in the glomeruli, resulting in glomerulone-
phritis.
CASE 5.10 CBC, coagulation studies, and electrolytes are all normal.
A 10-year-old male presents with his mother, who appears You are suspicious that he may have had streptococcal phar-
very anxious. She reports several episodes o red-brown yngitis that was unrecognized.
urine this morning. T e patient reports eeling a bit tired,
but otherwise has no complaints. His past medical history Question 5.10.3 Which o the ollowing statements best
is unremarkable and he takes no medications. On review o describes the usual course o post-streptococcal glomeru-
systems, he reports about 10 days o a sore throat that com- lonephritis?
pletely resolved a ew days ago. A) Most patients progress to renal ailure.
On examination, you nd a pleasant young male in B) Af er resolution o the initial episode, recurrent episodes o
no acute distress. He is a ebrile. His blood pressure is gross hematuria are common.
140/94 mm Hg, and he has trace pretibial edema. T e remain- C) In most cases, hypertension and uremia subside within 1 to
der o the examination is unrevealing. 2 weeks.
D) In most cases, hypertension is persistent and requires treat-
Question 5.10.1 All o the ollowing tests are likely to be ment.
help ul in the work-up o this patient EXCEPT: E) Adults tend to recover more quickly than children.
A) Urinalysis.
B) Abdominal X-ray. Answer 5.10.3 The correct answer is “C.” Post-streptococcal
C) CBC. glomerulonephritis, characterized by immune complex deposi-
D) Plasma electrolytes. tion in the glomeruli, is a sel -limited disease in most patients.
E) BUN and creatinine. T ere is a latent period, averaging 10 days, between pharyngitis
and the development o hematuria. Recovery is expected in 1 to
Answer 5.10.1 The correct answer is “B.” Abdominal plain 2 weeks. Unlike IgA nephropathy, recurrent episodes o gross
lms are not use ul in almost any situation unless looking or hematuria are rare in post-streptococcal glomerulonephritis.
constipation or bowel obstruction. (What did you say? Free air? Post-streptococcal glomerulonephritis is more common in chil-
An upright chest is the most sensitive lm or ree air—save dren and tends to be more severe when it a ects adults. Hyper-
or C .) I this patient had a presentation consistent with uro- tension and uremia resolve relatively quickly, but microscopic
lithiasis, an abdominal C scan may be indicated, although an hematuria may persist or 6 months.
ultrasound would be the pre erred rst study or urolithiasis in
a pediatric patient. All o the other laboratory tests should be
ordered.
HELPFUL TIP:
Urinalysis shows 2+ blood, 2+ protein, speci c gravity r na bi s is a indicat d in c i d n wit n -
1.015, and numerous red blood cells with red cell casts. BUN itic u in and mi d na ai u b caus t di -
is 35 mg/dL and creatinine is 1.8 mg/dL. CBC, coagulation ntia dia n sis can b na w d b t c inica
studies, and electrolytes are pending. s ntati n and b caus man t dis as s a
s - imit d. In m s v cas s, na bi s ma
Question 5.10.2 At this point, all o the ollowing should be b n c ssa t dia n s and t at a iat . S
considered in the dif erential diagnosis EXCEPT: Tab 5-10 a a tia ist caus s matu ia in
A) Minimal change disease. c i d n.
B) Henoch–Schönlein purpura.
CHAPTER 5 • Neph r o l o g y 167

TABLE 5-10 CAUSES OF GROSS HEMATURIA blood pressure stabilizes. T is patient may also require an RBC
IN CHILDREN trans usion given her anemia. Although she has had diarrhea,
her potassium is currently in the normal range, probably due to
Idi at ic (usua b ni n wit s uti n v tim )
U ina t act in cti n
decreased glomerular ltration. Given her renal ailure, potas-
T auma sium should not be in her IV uids.
C n nita an ma Consideration o dialysis (“B”) is premature. A platelet count
U t a i itati n t auma o 50,000 is adequate or hemostasis, so answer “C” is incorrect.
N it iasis T e schistocytes suggest a microangiopathic hemolytic anemia.
Sick c dis as /t ait
C a u at
Steroids are not generally help ul or this, so “D” is incorrect.
g m u a dis as ( . ., st-st t c cca m u n itis) Finally, antibiotics (“E”) may (or may not) be bene cial i the
Ma i nanc ( . ., Wi m tum ) patient has bacterial enteritis, but IV uids should be adminis-
M dicati ns ( . ., m a ic c stitis m c c s amid ) tered rst.

Question 5.11.2 Based on the available in ormation, which


Objectives: Did you learn to . . . o the ollowing is the most likely diagnosis?
• eva uat a c i d wit ss matu ia? A) T rombotic thrombocytopenic purpura.
• g n at a di ntia dia n sis matu ia and t in- B) Hemolytic uremic syndrome.
u ia in a c i d? C) Postin ectious glomerulonephritis.
• D sc ib t usua c u s st-st t c cca m u - D) Henoch–Schönlein purpura.
n itis? E) Autosomal-recessive polycystic kidney disease.

CASE 5.11 Answer 5.11.2 The correct answer is “B.” Hemolytic uremic
T e parents o a 2-year-old emale bring her in to your of ce syndrome (HUS) is the most likely diagnosis. T is patient pres-
or a week-long history o diarrhea. Initially, her stools were ents with a classic history o uremia, hemolysis and thrombo-
loose and watery, but over the last 2 days, they have become cytopenia preceded by 5 to 7 days o diarrhea. Patients tend to
bloody. T e patient has appeared to have abdominal pain on become oliguric and sometimes anuric.
occasions, and her appetite is depressed. Despite bloody diar- “A,” thrombotic thrombocytopenic purpura ( P), is related
rhea, her parents attempted to care or her at home until she to HUS but is rare in children. T e average age o a patient with
became more lethargic (well, that didn’t work . . . probably the P is 41 years with women making up about 70% o all cases.
same parents who re used vaccines). T ey are also worried Being Black is also a risk actor or P. In contrast to HUS,
about some bruising on her extremities. patients with P may present with the classic pentad: throm-
Vital signs: 37.2°C, P 145 bpm, BP 88/47 mm Hg, RR 40 bocytopenia, ever, mental status changes, renal insu ciency,
breaths/minute. T e patient appears pale, with slight scleral and hemolytic anemia. Usually, there is a prodrome viral illness
icterus. You note petechiae and purpura on the extremities. with P but diarrhea occurs only rarely. See Chapter 6 or
Her abdomen is di usely tender. She responds to commands more details.
but appears very lethargic. “C,” postin ectious glomerulonephritis, usually occurs af er
While you are arranging her admission to the hospital, pharyngitis or skin in ection with group A beta-hemolytic strep-
some laboratory tests return: Hgb 8 g/dL, Hct 24%, WBC tococci. Common symptoms include edema and hematuria but
14,000/mm3, platelets 50,000/mm3, Na 128 mEq/L, K 3.9 not thrombocytopenia or diarrhea. “D,” Henoch–Schönlein
mEq/L, HCO3 14 mEq/L, BUN 38 mg/dL, creatinine 2.1 mg/ purpura, is a (usually) transient IgA vasculitis ollowing upper
dL. T e peripheral blood smear shows schistocytes, “Burr” respiratory in ections in children and adolescents. Autosomal-
cells, and grossly reduced number o platelets. recessive polycystic kidney disease is a rare disorder that pres-
ents early in childhood with abdominal masses, hypertension,
urinary tract in ections, and renal ailure but not hemolytic
Question 5.11.1 Which o the ollowing is the most appro- anemia.
priate initial management o this patient?
A) Intravenous uids.
B) Dialysis. HELPFUL TIP:
C) Platelet trans usion. h US ma ccu wit ut dia a. T is subt h US
D) Corticosteroids. ccu s ss qu nt , is ass ciat d wit St t c ccus
E) Antibiotics. in cti ns, and ca i s a w s n sis.

Answer 5.11.1 The correct answer is “A.” T e proper initial


management consists o supportive therapy. T is patient has Question 5.11.3 From the blood culture, you expect to nd:
signs o dehydration, which would be expected rom the history A) Shigella species.
o prolonged diarrhea. She is hyponatremic, and isotonic (0.9%) B) E. coli.
saline is the IV uid o choice. Use 20 cc/kg boluses until her C) Streptococcus pneumoniae.
168 FAMIl y MeDICINe eXAMINATIo N & Bo Ar D r eVIeW

D) Haemophilus inf uenzae. rom the bladder pain. A urinalysis and pelvic examination
E) None o the above. are unremarkable except or tenderness over the bladder area.

Answer 5.11.3 The correct answer is “E.” Although HUS is the Question 5.12.1 The MOST IMPORTANT next step is:
result o bacterial enteritis, patients are not bacteremic. Instead, A) Urine culture.
the endothelial damage and hemolysis are caused by Shiga B) rial o 4 weeks o antichlamydial therapy.
toxin, released rom E. coli or Shigella dysenteriae. C) Pelvic ultrasound.
D) Psychiatry consult and/or SSRI therapy or somatization dis-
Question 5.11.4 All o the ollowing are true regarding order (now called somatic symptom disorder).
Shiga toxin HUS in children EXCEPT:
A) I dialysis is needed, renal unction rarely returns. Answer 5.12.1 The correct answer is “C.” T is patient is
B) Hal or more o the cases occur in the summer months. describing typical symptoms o “pain ul bladder syndrome,” the
C) Ingestion o contaminated meat is a common source o disease ormerly known as “interstitial cystitis.” Pain ul bladder
E. coli O157:H7 in ection. syndrome does not involve in ammation. T ere ore, “intersti-
D) Cattle are the main vectors o E. coli O157:H7. tial cystitis” is, and always has been, a misnomer. Pain ul bladder
E) Antibiotics do not reduce the risk o HUS in patients with syndrome consists, appropriately enough, o pain re erable to the
con rmed E. coli O157:H7 in ections. bladder that cannot be attributed to another cause. Patients gen-
erally note pain with lling o the bladder and relie o symptoms
Answer 5.11.4 The correct answer is “A.” All the other state- af er urination. T e pain may be described as urethral, as a supra-
ments are true. T e prognosis o Shiga toxin HUS in children pubic pressure, as a pressure or a burning pain. Pain ul bladder
is generally quite avorable, even i renal ailure requires dialy- syndrome is a diagnosis o exclusion and now treated as a chronic
sis. While 50% will have some residual renal damage, only 4% pain syndrome. It is critical to rule out other pelvic pathology that
go on to require long-term dialysis. HUS is more common in might cause the same symptoms such as an enlarged uterus sitting
rural areas and in the summer. Cows are the culprits (T ey on the bladder, broids, or ovarian cancer. Other causes o similar
get the blame or everything). “E” is o special note. T ere is symptoms could be bladder irritants (ca eine, alcohol), a urethral
controversy over whether or not antibiotics may increase the diverticulum, an sexually transmitted in ection, etc. While less
risk o HUS developing in patients with E. coli O157:H7 in ec- common, pain ul bladder syndrome can be seen in men. In this
tions; however, it is airly clear that antibiotics do not reduce the case prostate pathology must be ruled out (you can pretty much
risk. T e idea is that bacterial death rom antibiotics releases rule out prostate pathology in the emale patients a priori).
more Shiga toxin leading to HUS. Remember that most cases o
bacterial gastroenteritis—E. coli O157:H7 included—will clear Question 5.12.2 All o the ollowing are mandatory at this
without antibiotic therapy. point EXCEPT:
A) Urine cytology in a high-risk patient (smoker, etc.).
HELPFUL TIP:
B) Postvoid residual.
ot subt s e. c i as w as S i a can b -
C) Cystoscopy and hyperdistention o the bladder.
s nsib h US. T abs nc t o 157:h 7 subt
D) Ruling out a bladder stone.
e. c i d s n t u ut h US.
Answer 5.12.2 The correct answer is “C.” Cystoscopy and
hyperdistention are NO required in order to make the diag-
Objectives: Did you learn to . . . nosis o pain ul bladder syndrome. As noted above, pain ul
• eva uat and mana a c i d wit v mia and na bladder syndrome is a clinical diagnosis o exclusion: Is there
ai u ? anything else causing the symptoms? I not, as Sherlock Holmes
• r c niz a c inica ist and ab at ndin s su s- said, “Once you eliminate the impossible, whatever remains, no
tiv m tic u mic s nd m ? matter how improbable, must be the truth.” It is important to
• Id nti caus s m tic u mic s nd m ? rule out tumor (i indicated), stones, and neurogenic bladder
(thus the postvoid residual).
CASE 5.12
HELPFUL TIP:
A 45-year-old emale presents to your clinic complaining o T atm nt ain u b add s nd m ma inc ud -
urinary requency, “bladder” pain, and urinary urgency. a t a ain c inic, su t u s, t atm nt an
T ere is no dysuria, however. She has had hematuria on dip- t und in b ms (in ammat b w dis-
stick several times with UA showing microscopic hematuria as , tc.), t ic c ics, aba ntin, int av sicu a DMSo ,
on more than three occasions. T is has been going on or sev- cu nt d dist nti n t b add , tc. W at
eral months and other practitioners (less skilled than yoursel ) ab ut nt san su at s dium (e mi n)? It is v
have treated with a number o antibiotics without any relie . x nsiv , ma tak u t 6 m nt s t w k, and t
On questioning, she also notes bladder pain during inter- b n it is m d st at b st.
course and some chronic, vague, lower pelvic pain distinct
CHAPTER 5 • Neph r o l o g y 169

nearly normal despite the presence o a disturbance in mea-


HELPFUL TIP:
sured HCO3, which only occurs when a mixed disorder is
W n t c nic naz idin ( . ., p idium)
present.
ain u b add s nd m ? W , u can c nsid it. But
t is a dan m t m bin mia.
HELPFUL TIP:
It is im ssib t v c ct a m tab ic abn ma it .
T us, a ati nt wit a m tab ic acid sis wi n t b -
Objectives: Did you learn to . . . c m a ka tic n ma un ss t is an t ima-
• eva uat and mana a ati nt wit ain u b add c ss s nt ( . ., s i at a ka sis). l ik wis ,
s nd m ? a ati nt wit a s i at acid sis m t t nti n
Co 2 wi n t b c m a ka tic n ma un ss t
QUICK QUIZ: ACID–BASe DISo r Der is a s c nda ima c ss in n ( . ., m tab ic
a ka sis).
While covering your local ED, a 15-year-old emale presents
with her ather. He reports that he came home rom work,
ound her asleep on the couch, and had di culty in waking Proper treatment o salicylate overdoses includes supportive
her up. She is lethargic and complains o nausea, dizziness, and therapy, urine alkalinization with sodium bicarbonate, and pos-
abdominal pain. Apparently, she had muscle aches af er gym- sibly hemodialysis.
nastics practice and then took “hand uls” o aspirin to relieve
her pain. She was taking three to our tablets every hour today
but is not sure about her total ingestion. She denies other inges- QUICK QUIZ: DySUr IA
tions.
On physical examination, her vitals are: 39°C, P 110 bpm, Your nurse comes to you with a patient request. A 25-year-old
RR 18 breaths/minute, BP 104/68 mm Hg. She is diaphoretic. emale called in complaining o 2 days o burning with urina-
T e neurological examination is non ocal, but she becomes tion, urgency to urinate, and increased requency. She has no
progressively more lethargic during the examination. A ever, nausea, abdominal pain, ank pain, or vaginal discharge.
venous blood gas on room air shows pH 7.38 (normal), PaCO 2 T e patient wants to know, “Can’t I just get some antibiotics?”
23 mm Hg (low), PaO 2 40 mm Hg (her oxygen saturation is She’s sure this is a bladder in ection, just like the one she had
98% on transcutaneous monitoring), and HCO 3 15 mEq/L last year. Oh, by the way, she’s leaving or Europe tomorrow or
(low). Other laboratory data: Na 140 mEq/L, K 3.1 mEq/L, Cl her honeymoon.
101 mEq/L, HCO3 15 mEq/L (low), BUN 19 mg/dL, creatinine
1.1 mg/dL. Your response is to:
A) Prescribe cipro oxacin 250 mg BID or 3 days.
T e venous blood gas results are best described as: B) Prescribe trimethoprim/sul amethoxazole BID or 14 days.
A) Metabolic acidosis. C) Ask her to come in or a urinalysis.
B) Metabolic acidosis and metabolic alkalosis. D) Ask her to come in or a urine culture.
C) Metabolic acidosis and respiratory alkalosis. E) Recommend cranberry juice.
D) Metabolic alkalosis and respiratory acidosis.
E) Just dandy! Look at the pH, pro essor. The correct answer is “A.” Although you could argue that
the diagnosis o U I should be con rmed by urinalysis, there
The correct answer is “C.” Although the pH is in the normal is plenty o evidence that history alone is su ciently accurate
range, there is an acid–base disorder present. First, there appears in the right population (in this case, women o child-bearing
to be a metabolic acidosis with an elevated anion gap. T e mea- years—but not children, pregnant women, men, or the elderly).
sured HCO3 is 15 mEq/L, consistent with an acidosis, and the I a woman complains o dysuria and increased requency with-
anion gap is 24 (based on the calculation o Na − (Cl + HCO3), out vaginal discharge, the likelihood ratio o U I is about 25 and
in this case: 140 − (101 + 15) = 24). T is patient has taken an “pretest” probability is greater than 90% that she has a urinary
inadvertent overdose o aspirin. In salicylate overdoses, a high tract in ection (JAMA. 2002;287(20):2701–2710). I her urinaly-
anion gap metabolic acidosis is of en observed. Since salicylates sis were completely normal, she may still have an in ection and
directly stimulate the CNS respiratory center, there is usually a likely has a alse-negative dipstick urine. T us, certain symp-
concurrent respiratory alkalosis. toms and historical elements are more use ul than urinalysis.
In a compensated metabolic acidosis, the PaCO2 should For this patient, empiric antibiotic therapy with an appropriate
drop by 1.25 mm Hg or every 1 mEq/L drop in HCO3. In this antibiotic (nonrespiratory uoroquinolone, trimethoprim/sul-
case, the serum HCO3 is 9 mEq/L below normal (i the normal amethoxazole or other, depending on regional susceptibilities
is counted as 24), so PaCO2 should be about 29 mm Hg (40 − or enteric organisms) or 3 days is reasonable. Fourteen days
(9 × 1.25)). However, the measured PaCO2 is 23 mm Hg, indi- o trimethoprim/sul amethoxazole is overkill. O course, urine
cating the presence o a respiratory alkalosis. Also, the pH is culture is the gold standard or diagnosing U I, but she will
170 FAMIl y MeDICINe eXAMINATIo N & Bo Ar D r eVIeW

be in Europe by the time you get the results—and we’re pretty TABLE 5-11 COMMON DRUGS ASSOCIATED WITH
sure your medical license does not extend to Paris (though ACUTE INTERSTITIAL NEPHRITIS
maybe she needs someone to carry her bags).
• p nici ins
T ere is some evidence that cranberry products may reduce
• As i in
the requency o recurrent U I (maybe), but cranberry juice • Ci f xacin (and ik t fu quin n s)
does not seem to work or treatment. rials o cranberry prod- • A u in
ucts or U I su er rom high dropout rates and variability in • NSAIDs
products and measurements. • S m ACe in ibit s
• e t m cin

ACe, an i t nsin-c nv tin nz m ; NSAIDs, n nst ida anti-


HELPFUL TIP: inf ammat d u s.
Wit t ica UTI s m t ms ( . ., d su ia, qu nc ) and
a n ativ u in cu tu n s ns t antibi tics, D) Glomerulonephritis.
c nsid t caus s: ain u b add s nd m , c a- E) Nephrotic syndrome.
m dia u t itis, statitis (in m n), vic in amma-
t dis as (in w m n), vic mass, s nita is, Answer 5.13.2 The correct answer is “B.” T e combination o
and d u s ( . ., diu tics, ca in , and t in ). ever, rash, mild eosinophilia, exposure to a new drug (na cillin)
and white cell casts in the urine essentially makes the diagnosis o
acute interstitial nephritis. “A” is incorrect. T e patient with A N
may have the same FENa and urine sodium as this patient but
CASE 5.13 should have renal tubular cells and/or granular casts in the urine.
Also, A N does not cause ever, eosinophilia, or rash. “C,” renal
You are asked to consult on a patient who is hospitalized in arction, is unlikely in a young patient and the rest o the clini-
by an orthopedic surgeon (consulted by a surgeon twice in cal picture is more consistent with acute interstitial nephritis. “D,”
a chapter!!). T e patient is a 25-year-old emale who has a glomerulonephritis, is a possibility (e.g., lupus could cause a rash
history o osteomyelitis rom an open racture sustained in and ever). However, glomerulonephritis is associated with red
a skiing accident. She has recently begun to spike a ever to cell casts and not white cell casts. Finally, as you already learned
38.5°C and have a rapid increase in her creatinine. above, nephrotic syndrome presents with a bland urinary sedi-
Medications: na cillin, ibupro en, morphine, lactated Ringer ment. See ables 5-11 and 5-12 or more details.
solution IV 100 cc/hr.
Labs: Cr 3.5 mg/dL, BUN 25 mg/dL. Question 5.13.3 How long a ter drug exposure does acute
CBC shows mild WBC count o 12,500/mm3 and di eren- interstitial nephritis generally begin?
tial shows eosinophilia. A) 2 to 3 days.
B) 10 to 14 days.
Question 5.13.1 All things being equal, what would you C) Several months.
expect to nd? D) A and B.
A) FENa > 2%, urine sodium < 20 mg/dL. E) Any o the above.
B) FENa < 1%, urine sodium < 20 mg/dL.
C) FENa > 2%, urine sodium > 40 mg/dL. Answer 5.13.3 The correct answer is “E.” Patients can
D) FENa < 1%, urine sodium > 40 mg/dL. develop acute interstitial nephritis anywhere rom 1 day to
several months af er beginning a drug. Ri ampin can cause
Answer 5.13.1 The correct answer is “C.” Remember . . . her acute interstitial nephritis on day 1. Interstitial nephritis can
BUN/Cr < 20; there ore, it is likely not prerenal disease (and begin within 2 to 5 days i there has been a prior exposure
she’s receiving volume and has no history o heart ailure). to the drug, will typically begin within 10 to 14 days on rst
T us, the patient likely has intrinsic kidney disease. T is means exposure to a drug, and may delayed or months in the case o
that the FENa should be > 2% and the urine sodium > 40 mg/ NSAID exposure.
dL. In this scenario, the kidney is not trying to hold on to
sodium in an attempt to correct a prerenal cause o increasing TABLE 5-12 SYMPTOMS/SIGNS/LABORATORY
creatinine. FINDINGS IN ACUTE INTERSTITIAL NEPHRITIS
• F v
T e patient’s examination shows a di use rash and the urine • r as (va iab , ma n t b s n in a )
contains white cell casts. T ere are no red cells in the urine. • Acut is in asma c atinin
• Activ u in s dim nt t at inc ud s w it c casts
Question 5.13.2 The most likely diagnosis is: • p i a sin i ia and u in sin i s (in m st cas s)
• r na tubu a acid sis
A) Acute tubular necrosis (A N).
B) Acute interstitial nephritis. Note: Int stitia n itis s c nda t NSAIDs ma ccu wit ut v ,
C) Renal in arction. as , sin i ia.
CHAPTER 5 • Neph r o l o g y 171

HELPFUL TIP: BUN/C ati > 20 n a indicat s t at an acut kidn


pati nts wit acut int stitia n itis wi t n av inju is s c nda t xt a na caus s ( usi n m
sin i s in t u in and i a sm a , but d d ati n Ch F, tc.). T is u d s n t a t c i d n,
sin i s ma b abs nt s cia in t s wit t s wit g I b ds, and t xc ti ns.
NSAID-induc d acut int stitia n itis. T atm nt is Ca cium (di ta su m nta ) tak n wit m a s duc s
t st t ndin d u . I t is d sn’t w k, st ids t isk ca cium xa at st n s b bindin xa at in t
c t t xic d u s ma b n d d. g I t ack.
g ad inium can caus a sc d ma- ik dis as in t s
wit a c atinin c a anc < 30 m /ml /min.
Objectives: Did you learn to . . .
• r c niz t s ntin s m t ms acut int stitia In an t wis at ma wit ut a va ina disc a ,
n itis? t s m t ms d su ia, qu nc and u nc av a 90%
sitiv dictiv va u UTI, b tt t an t at a UA.
• D sc ib caus s and n sis acut int stitia n itis?
pati nts wit u it iasis s u d av an NSAID as a t
t i ain c nt im n un ss it is c nt aindicat d.
QUICK QUIZ: SAVe Th o Se Neph r o NS! r ac ma n sium in t ka mic ati nt w is n t
s ndin as x ct d t t atm nt. T sam is t u in t
A 45-year-old male comes to your o ce to establish care. He reports ati nt wit ca c mia.
a history o hypertension treated with verapamil and chlorthali- S dium bica b nat and N-ac t c st in d n t t ct t
done. His blood pressure today is 130/78 mm Hg. You order some kidn s m i dinat d c nt ast. h w v , t isk na
laboratory tests and nd that his creatinine is 2.5 mg/dL. inju wit c nt ast is sma ; d at t ati nt wit n ma
sa in .
You recommend which strategy to preserve his renal unction? T a initiati n dia sis in ati nts wit c nic kidn
A) Start a low-dose ACE inhibitor. dis as ads t w s utc m s. Sta t dia sis n w n
B) Start a high-dose ACE inhibitor. ati nt m t acc t d indicati ns.
C) Start urosemide.
D) Start amlodipine.
BIBLIOGRAPHY
The correct answer is “A.” Even in patients with renal dys-
unction as evidenced by elevated creatinine, ACE inhibi- Azadi R, et al. Hemolytic uremic syndrome caused by Shiga
tors can reduce the rate o progression o renal disease. More toxin-producing Escherichia coli 0111. J Am Osteopath
importantly, ACE inhibitors can be used sa ely in patients with Assoc. 2010;110(9):538–544.
stage 4 chronic kidney disease. Due to the risk o rising cre- Bangalore S, et al. Angiotensin receptor blockers and risk o
atinine and potassium, ACE inhibitors should be started at a myocardial in arction: meta-analyses and trial sequential
low dose and increased slowly, checking creatinine and potas- analyses o 147020 patients rom randomised trials. BMJ.
2011;342:d2234.
sium levels within 1 to 2 weeks o making a dose adjustment.
Furosemide and amlodipine have not been shown to preserve Barnett AH, et al. Angiotensin-receptor blockade versus
converting-enzyme inhibition in type 2 diabetes and
renal unction.
nephropathy. N Engl J Med. 2004;351(19):1952–1961.
Bent S, et al. Does this woman have an acute uncomplicated
HELPFUL TIP: urinary tract in ection? JAMA. 2002;287(20):2701–2710.
ACe in ibit s av b n s wn t b us u in duc- Brookhart MA, et al. Comparative mortality risk o anemia
in ssi n na dis as v n w n t a- management practices in incident hemodialysis patients.
ti nt’s c atinin is 5.0 m /dl . But b v ca u in JAMA. 2010;303(9):857–886.
t is u . Sta t w, s w, and c ck tassium and Cheng J, et al. E ect o angiotensin-converting enzyme inhibi-
c atinin . In ati nts wit CKD sta 3 4 sta t d n tors and angiotensin II receptor blockers on all-cause mor-
and ACe in ibit , a mi d inc as in c atinin is t ica tality, cardiovascular deaths, and cardiovascular events in
and x ct d and s u d n t su t in disc ntinuati n patients with diabetes mellitus: A meta-analysis. JAMA
t ACe in ibit . Intern Med. 2014;174(5):773–785.
Cooper BA, et al. A randomized, controlled trial o early versus
late initiation o dialysis. N Engl J Med. 2010;363(7):609–
Clinical Pearls 619.
Gill N, et al. Renal ailure secondary to acute tubular necrosis:
ACe in ibit s a t d u s c ic na t cti n. epidemiology, diagnosis, and management. Chest. 2005;
Ar Bs and v a ami di tiaz m a t ti ns. 128(4):2847–2863.
Av id NSAIDs in ati nts wit c nic na dis as m an Haller H, et al. Olmesartan or the delay or prevention o
caus , Ch F t nsi n. microalbuminuria in type 2 diabetes. N Engl J Med.
2011;364:907–917.
172 FAMIl y MeDICINe eXAMINATIo N & Bo Ar D r eVIeW

Hou FF, et al. E cacy and sa ety o benazepril or advanced disease treated with erythropoietin: a meta-analysis.
chronic renal insu ciency. N Engl J Med. 2006;354: Lancet. 2007;369:381–388.
131–140. Pickar R, et al. Medical expulsive therapy in adults with ure-
Kellum J, et al. Acute renal ailure. Am Fam Physician. 2007; teric colic: a multicentre, randomised, placebo-controlled
76(3):418–422. trial. Lancet. 2015;386(9991):341–349.
Kohlstadt I, Frassetto L. reatment and prevention o kidney Schena FP. Management o patients with chronic kidney
stones: An update. Am Fam Physician. 2011;84(11): disease. Intern Emerg Med. 2011;6(Suppl 1):77–83.
1234–1242. Simerville JA, et al. Urinalysis: a comprehensive review. Am
Levey AS, et al. K/DOQI clinical practice guidelines or Fam Physician. 2005;71(6):1153–1162.
chronic kidney disease: evaluation, classi cation, and Slagman MC, et al. Moderate dietary sodium restriction
strati cation. Am J Kidney Dis. 2002;39(Suppl 1):S1–S266. added to angiotensin converting enzyme inhibition
Available at https://www.kidney.org/sites/de ault/ les/ compared with dual blockade in lowering proteinuria
docs/ckd_evaluation_classi cation_strati cation.pd ; and blood pressure: randomised controlled trial. BMJ.
accessed on October 25, 2015. 2011;343:d4366.
Martinez-Bianchi V, Halstater BH. Urologic chronic pelvic Sterns RH, et al. Ion-exchange resins or the treatment o
pain syndrome. Prim Care. 2010;37:527–546. hyperkalemia: are they sa e and e ective? J Am Soc
McDonald JS, et al. Risk o intravenous contrast material- Nephrol. 2010; 21(5):733–735.
mediated acute kidney injury: a propensity score- matched Strippoli GF, et al. Angiotensin converting enzyme inhibitors
study strati ed by baseline-estimated glomerular ltration and angiotensin II receptor antagonists or preventing the
rate. Radiology. 2014;271(1):65–73. progression o diabetic kidney disease. Cochrane Database
McDonald RJ, et al. Intravenous contrast material exposure is Syst Rev. 2006;(4):CD006257.
not an independent risk actor or dialysis or mortality. arr PI, et al. Shiga-toxin-producing Escherichia coli and hae-
Radiology. 2014;273(3):714–725. molytic uraemic syndrome. Lancet. 2005;365:1073–
Miller NL, Lingeman JE. Management o kidney stones. BMJ. 1086.
2007;334(7591):468–472. Vincendeau S, et al. amsulosin hydrochloride vs placebo
P e er MA, et al. A trial o darbepoietin al a in type 2 diabetes or management o distal ureteral stones: a multicen-
and chronic kidney disease. N Engl J Med. 2009;361:2019– tric, randomized, double-blind trial. Arch Intern Med.
2032. 2010;170(22):2021–2027.
Phrommintikul A, et al. Mortality and target haemoglobin Whitmore KE, T eoharides C. When to suspect interstitial
concentrations in anaemic patients with chronic kidney cystitis. J Fam Pract. 2011;60(6):340–348.
Hematologyand Oncology
Wendy Shen and Jason K. Wilbur
6
X-linked but is much less common. “D” deserves special men-
CASE 6.1 tion. Hemophilia rarely occurs in emales but can occur in two
A 6-month-old boy presents to your o ce a er his mother situations: (1) the emale patient is a heterozygote who has early
notices swelling and ecchymosis over his anterior right thigh. inactivation o the second X chromosome during embryogen-
She does not recall any trauma to the area. T e mother denies esis or (2) i both parents are carriers—in which case the ather
any history o bleeding problems, including during his cir- would have the disease overtly. “E” is true. Factor VIIa can be
cumcision. On physical examination the child has a large used in most cases o hemophilia A and B to stop bleeding. It is
hematoma over his thigh. T ere are no obvious bony de or- mostly used in patients who have actor inhibitors in the blood
mities and the child otherwise looks well. You suspect the (developed as a response to repeated exposure to actor VIII or
child may have an inherited coagulation disorder. actor IX [Christmas disease]).

Question 6.1.1 Which o the ollowing statements is NOT HELPFUL TIP:


correct? Generally speaking, petechiae and mucosal bleeding re-
A) Child abuse should be included in the di erential diagnosis. sult rom platelet problems (e.g., mild von Willebrand dis-
B) T e child did not bleed during circumcision, so the possibil- ease [VWD] and thrombocytopenia); while hemarthrosis
ity o hemophilia need not be considered. and hematomas result rom actor de iciency. Severe
C) A care ul amily history is important in the work-up. VWD may present with hemarthrosis and hematomas.
D) Coagulation studies (P , P ) and CBC should be obtained.

Answer 6.1.1 The correct answer is “B.” T e absence o bleed- HELPFUL TIP:
ing during circumcision in no way rules out hemophilia. Up to Von Willebrand disease may be autosomal domi-
hal o hemophiliac patients do not bleed a er circumcision. nant (Type I), variable (Type II), or autosomal recessive
Depending on the severity o actor de ciency, the diagnosis (Type III). There is o ten a amily history o Von Will-
may not be made until the child is very active or even in adult- ebrand actor (vWF) de iciency or bleeding, but a per-
hood, a er surgery, etc. centage o cases occur spontaneously.

Question 6.1.2 Which o the ollowing is NOT TRUE about Question 6.1.3 Which o the ollowing is indicated when
hemophilia A? evaluating or a suspected inherited coagulopathy?
A) It is an X-linked disorder. A) Prothrombin time (P ).
B) It is the result o actor VIII de ciency. B) Partial thromboplastin time (P ).
C) It generally leads to mucosal bleeding. C) Platelet count.
D) It occurs most o en in males. D) PFA-100.
E) It can be treated with actor VIIa E) All o the above.

Answer 6.1.2 The correct (and wrong) answer is “C.” Hemo- Answer 6.1.3 The correct answer is “E.” T e P is actually
philia A is an X-linked de ciency o actor VIII, which presents the most sensitive test or hemophilia. “D” deserves special
with hematomas, bleeding, and hemarthrosis (and not gener- mention. T e PFA-100 (Platelet Function Analyzer) tests or
ally mucosal bleeding or petechiae). De ciency o actor IX, (appropriately enough) platelet unctioning and will be abnor-
or hemophilia B, also known as “Christmas Disease,” is also mal in diseases o platelet dys unction, such as VWD.
173
174 FAMILY MEDICINE EXAMINATION & BOARD REVIEW

Question 6.1.6 In 2015 (as opposed to 1992) which o the


HELPFUL TIP:
ollowing likely scenarios may be seen in a patient with
The PFA-100 can help you make a diagnosis o VWD but
hemophilia?
it doesn’t necessarily correlate with the risk o clinical
A) Hemarthrosis with signi cant arthritis and joint dys unc-
bleeding.
tion.
B) An extremely low risk o HIV/AIDS rom multiple trans u-
sions o FFP.
You diagnose this patient with hemophilia A, and he grows up C) Maturational delay i anti actor antibodies are present.
under your excellent tutelage. As a junior high school student, D) Reduced risk o coronary artery disease (CAD).
he attends summer camp or hemophiliacs in ransylvania. For E) All o the above are ound with hemophilia.
some bizarre reason, the councilors decide to have the kids play
tackle ootball (just in case you think we made this up, this sce- Answer 6.1.6 The correct answer is “E.” “A” is true. Patients
nario was seen by one o your authors while working in the emer- still get recurrent hemarthrosis leading to arthritis and joint
gency department (ED)—but not at the University o Iowa). T e dys unction. “B” is true. Screening and the use o recombinant
patient presents to the ED with bleeding into two o his joints. actor VIII has led to a marked reduction in HIV in hemophili-
acs rom a high o 50% in the 1990s. Patients with hemophilia
Question 6.1.4 What is the initial step in the management
do have delayed maturation, especially i anti actor antibodies
o this patient’s hemarthroses?
are present. And nally, they do have a reduced risk o CAD.
A) Joint aspiration.
T is is also seen in carriers o hemophilia genes.
B) Desmopressin.
C) Factor VIII in usion.
Note: In this chapter, objectives are listed a er several short cases,
D) Heparin in usion.
so keep reading.
E) Leeches.

Answer 6.1.4 The correct answer is “C.” While joint aspiration CASE 6.2
and desmopressin may be help ul, this patient needs to be treated A 4-year-old girl is brought to you a er she is noted to have
with actor VIII in order to stop the bleeding. Joint aspiration may small pink spots on her lower extremities and bleeding rom
play a role in preventing blood-induced arthritis, a risk with recur- her gums. She had a URI a couple o weeks ago. She also
rent joint bleeds. Desmopressin is use ul in some cases o VWD recently got her second dose o her MMR vaccine (mom par-
and in some mild cases o hemophilia. Desmopressin releases ticularly wants to make sure she is protected rom the recent
vWF rom platelets transiently increasing circulating vWF. Anti - resurgence in measles and mumps). You note petechiae on her
brinolytic therapy may also be o use: tranexamic acid or amino- lower extremities and purpura in her oropharynx. T e mother
caproic acid is commonly used to prevent clot breakdown. and patient deny any other bleeding. You obtain a CBC,
which is normal except or a platelet count o 15,000/mm3.
Question 6.1.5 What actor level target is appropriate when You suspect immune thrombocytopenic purpura (I P, also
treating this patient? known as idiopathic thrombocytopenic purpura).
A) 5% to 10%
B) 30% to 40%.
Question 6.2.1 Which o the ollowing may have led to this
C) 75% to 80%.
patient’s ITP?
D) 100%.
A) MMR vaccine.
Answer 6.1.5 The correct answer is “B.” For a hemarthrosis or B) In ection.
other “minor” bleeding, you should maintain a actor level o C) Food allergy.
30% to 40% or 72 hours. For more serious bleeding (intracra- D) Autoimmune hemolytic anemia.
nial, or example), maintain a level o 80% to 100% or 10 days. E) A and B.

HELPFUL TIP: Answer 6.2.1 The correct answer is “E.” Both in ections and
Von Willebrand disease Type I will respond to desmo- the MMR vaccine have been linked to I P (not autism . . . got
pressin (as noted above). Type II has variable response that? . . . not autism). Food allergy does not cause I P and i you
to desmopressin. A trial at the time o diagnosis is rec- chose “D,” time to study some more. An autoimmune hemolytic
ommended. Type III VWD requires in usion o actor VIII anemia is just that; an autoimmune hemolytic anemia. It is a di -
and vWF (such as Humate-P). Remember that actor erent disease process entirely and typically presents with signs
VIII is not enough, you need a preparation that also in- and symptoms o an anemia (pallor, atigue, dyspnea, ailure to
cludes vWF. I you do not have a actor VIII + vWF actor thrive). Peripheral smear with a hemolytic process will typically
available, or i the bleeding continues a ter replacement show schistocytes, “bite cells” or even spherocytes; it depends
therapy, you can always trans use normal platelets as a on the etiology o the hemolysis (more later) T is patient’s CBC
last option. These will have adequate vWF. was normal aside rom a low platelet count so answer “D” is not
correct.
CHAPTER 6 • HEMATOLOGY AND ONCOLOGY 175

Question 6.2.2 Which o the ollowing is NOT likely to lead


HELPFUL TIP:
to a sustained (let’s say at least 2 days) increase in the
Failure o ITP to respond to splenectomy may be due to
patient’s platelet count?
the presence o an accessory spleen, which can o ten be
A) IVIG.
identi ied by a liver–spleen radionuclide study.
B) Steroids (e.g., prednisone).
C) Platelet trans usion.
D) Splenectomy.
Keep reading or the objectives . . . We swear there is a common
Answer 6.2.2 The correct answer (and what will not yield a theme and that we didn’t orget them!
sustained increase in platelet count) is “C.” rans used plate-
lets will just be chomped up by “Mr. Spleen.” However, platelets
can be used to temporize i a patient must go to the OR, etc. CASE 6.3
Most children can simply be observed. T e only indication or A 24-year-old emale G1 P0 at 39 weeks o gestation pres-
treatment is bleeding. All o the treatments have a downside. ents to your o ce with a bruise on her anterior tibia, which
Splenectomy is associated with a risk o sepsis. I possible, delay she noticed a er bumping into a co ee table. She has been
splenectomy until a er the child is over 5 years old. Steroids healthy be ore and during her pregnancy and takes only pre-
may cause behavioral problems and long-term problems such as natal vitamins. Her physical examination is unremarkable
avascular necrosis. IVIG leads to a temporary increase in plate- with the exception o an 8-cm bruise over her right anterior
lets that may last several weeks but can be associated with renal tibia. Her vital signs are normal. Her physical examination
injury and anaphylaxis among other possible adverse e ects. is remarkable or a gravid abdomen consistent with 39 weeks
o gestation, etal heart tones auscultated at 140 bpm,
absence o right upper quadrant pain, and absence o periph-
HELPFUL TIP: eral edema. You obtain the ollowing laboratory tests: CBC,
I a child suspected o having ITP looks sick, you need to which demonstrates white blood cell (WBC) 9,000/mm3,
expand your di erential diagnosis. Sepsis with dissemi- hemoglobin (Hgb) 11.8 g/dL, and platelet count 95,000/mm3;
nated intravascular coagulopathy (DIC) and thrombotic normal P and P /INR; negative urinalysis; and normal
thrombocytopenic purpura—hemolytic uremic syn- liver enzymes.
drome (TTP–HUS) are also in the di erential. ITP is most
common in children between the ages o 2 and 4 years, Question 6.3.1 What is your next step?
with a small peak incidence in adolescence. ITP o ten A) Recommend immediate delivery by cesarean section as the
resolves without speci ic therapy; 80% to 90% o pedi- in ant likely has thrombocytopenia as well and is at high risk
atric patients are back to normal within a ew months, or intracranial hemorrhage.
and ewer than 20% o children will remain thrombocy- B) Recommend immediate delivery by cesarean section as this
topenic or greater than 12 months. Only 0.1% to 0.5% disorder will likely progress to eclampsia.
develop intracranial bleeding. C) Recommend close observation and reassure the patient that
this is typically a sel -limited condition.
D) Start prednisone, 1 mg/kg daily, and taper slowly over the
HELPFUL TIP: next 6 weeks.
Do not bother checking or antiplatelet antibodies in E) Recommend splenectomy as soon as possible a er delivery.
those with ITP. Patients without ITP may have antiplate-
let antibodies and the presence o these antibodies is Answer 6.3.1 The correct answer is “C.” T is patient likely
not predictive o outcome. has gestational thrombocytopenia, a condition that occurs
in up to 5% o pregnant women. It is characterized by mild
thrombocytopenia occurring in late gestation; the plate-
let count is usually > 70,000/mm 3 (two-thirds are between
HELPFUL TIP: 130,000/mm 3 and 150,000/mm 3). T e condition resolves a er
For adults with ITP, treatment is generally indicated. delivery and is not associated with severe neonatal throm-
Treatment includes (in order) steroids, Rho(D) immuno- bocytopenia. No speci c change in routine obstetrical care
globulin for Rh-positive patients, IVIG (which causes is warranted, although the anesthesiologist placing an epi-
a transient rise in platelet numbers), and splenectomy. dural may want a ollow-up platelet count closer to the time
Other treatment options or re ractory cases include o delivery.
rituximab or various immunosuppressive agents. The
thrombopoietin-receptor agonists, romiplostim and
eltrombopag (real names and not Icelandic vocabulary HELPFUL TIP:
words) stimulate platelet production and can also be A platelet count o > 20,000 to 50,000/mm 3 is generally
used in ITP. considered adequate or appropriate clotting.
176 FAMILY MEDICINE EXAMINATION & BOARD REVIEW

Objectives (see, we told you we didn’t orget them!): Did TABLE 6-1 BLOOD TRANSFUSION INDICATIONS
you learn to . . .
Indication Trans usion Threshold
• Obtain a thorough bleeding history?
• Use laboratory testing to assist in the diagnosis o a bleeding Otherwise stable adult Hgb ≤ 7 g/dL
disorder? and pediatric intensive
care patients
• Identi y and treat common inherited bleeding disorders?
• Identi y and treat common acquired bleeding disorders? Hemodynamically stable Hgb ≤ 8 g/dL OR or symptoms o
postoperative patients chest pain, orthostatic hypotension,
tachycardia unresponsive to IVF
QUICK QUIZ: COAGULATION STUDIES boluses, or CHF

Hemodynamically stable Hgb ≤ 8 g/dL OR or symptoms o


Which o the ollowing conditions should be considered i both patients with pre-existing chest pain, orthostatic hypotension,
the P and P are prolonged in a patient noted to be oozing cardiovascular disease tachycardia unresponsive to IVF
boluses, or CHF
rom a surgical incision?
A) Severe liver disease, DIC, actor X de ciency. Hemodynamically stable Unable to make a recommendation
B) Heparin e ect, VWD, actor XII de ciency. patients with acute against liberal or restrictive trans usion.
C) War arin e ect, actor VII de ciency, vitamin K de ciency. coronary syndrome (ACS)
D) All o the above.
Adapted rom Carson JL, Grossman BJ, Kleinman S, et al. Red blood cell
trans usion: a clinical practice guideline rom the AABB. Ann Intern Med.
The correct answer is “A.” T e three actor de ciencies that 2012;157(1):49–58.
may prolong both P and P are II, V, and X. Both P and
P may be prolonged due to severe liver disease and DIC as
well. Mild vitamin K de ciency or mild liver disease generally D) A stable ICU patient with an Hgb o 8.7g/dL.
a ects the P only. Generally, heparin a ects P , and war arin E) All o the above.
a ects P . Remember that the newer agents (rivaroxaban, dabi-
gatran, etc.) cause bleeding without prolonging the P or P . Answer 6.4.1 The correct answer is “B.” In general, a restric-
tive policy or blood trans usion has been shown to improve
outcomes. “A” is incorrect because blood should be given to the
HELPFUL TIP: hemodynamically unstable trauma patient a er at most 2 L o
Why does vitamin K work to improve the PT in liver saline (and per new A LS recommendations a er 1 L o saline).
disease i the problem is not related to war arin, which “B” is correct. A restrictive preoperative trans usion policy lim-
reduces vitamin K dependent actors, or to the liver, ited trans usions to those with an Hb o < 8 g/dL has shown sur-
which o ten responds to vitamin K even in those with vival bene t. “C” is incorrect. rans usion should be considered
cirrhosis? Because most alcoholics and others with liver in the patient with CAD only when the hemoglobin reaches 7
ailure are vitamin K de icient (poor absorption, poor to 8 g/dL. Finally, “D” is incorrect. rans usions should be with-
nutritional intake, antibiotic use—recall that vitamin held rom a nonbleeding, hemodynamically stable ICU patient
K is made in the gut by bacteria). Paradoxically, those until the Hb reaches 7 g/dL. A more liberal trans usion policy
with liver ailure are o ten hypercoagulable leading to is associated with higher mortality. As an aside, some would
thrombotic complications such as hepatic vein throm- trans use postoperative patients with an Hgb o less than 10 g/
bosis. dL i they are at high risk or ischemic disease (ischemic bowel,
CAD, etc.). rans usion or indications other than these are
not bene cial and have been proven harm ul. See able 6-1
or a list o trans usion indications.
CASE 6.4
A 42-year-old gentleman presents to the ED with a gastro-
HELPFUL TIP:
intestinal (GI) bleeding due to ibupro en use. His Hgb is
Erythropoietin may worsen outcomes in anemic patients
6.8 g/dL and he is hemodynamically unstable at this point.
with CHF. It seems to have no bene it and increases
You remember that blood actually increases mortality i used
thrombotic events (Ann Intern Med . 2013;159:746).
incorrectly.

Question 6.4.1 Which o the ollowing is an indication or


trans usion? You decide to trans use one unit o packed red blood cells
A) Hemodynamic instability in a trauma patient due to bleed- (PRBCs). A er 30 minutes, the patient complains o dyspnea
ing unresponsive to no less than 5 L o saline. and back pain. Repeat examination o this patient reveals
B) Preoperative Hgb o 7 to 8 g/dL with expected intraoperative a diaphoretic man with a pulse o 130 and BP o 88/50. His
blood loss. lung elds are clear (initial vitals be ore the trans usion were
C) Hemoglobin o 9.5 g/dL in a patient with angina. a pulse o 110 and BP o 94/52.
CHAPTER 6 • HEMATOLOGY AND ONCOLOGY 177

Question 6.4.2 What is your next step? C) Vitamin K administration orally will produce the most
A) Stop the blood trans usion and begin normal saline through immediate reduction in risk o bleeding rom war arin.
the IV. D) Vitamin K administration subcutaneously will produce the
B) Increase the rate o trans usion. most immediate reduction in risk o bleeding rom war arin.
C) Administer acetaminophen 650 mg PO. E) Cryoprecipitate trans usion will produce the most immedi-
D) Administer urosemide 40 mg IV. ate reduction in risk o bleeding rom war arin.
E) Place a nasogastric tube or lavage.
Answer 6.5.1 The correct answer is “B.” T e patient above
Answer 6.4.2 The correct answer is “A.” T e trans usion must would bene t most immediately rom the administration o
be stopped. T e patient is exhibiting signs and symptoms o a FFP. FFP contains all the soluble plasma proteins ound in
hemolytic trans usion reaction, which is generally the result o whole blood, including the vitamin K-dependent actors that
an ABO incompatibility. Patients may exhibit nausea, ushing, are depleted by war arin. I more sustained reversal is desired,
dyspnea, oliguria, back pain, and hypotension. Other ndings the simultaneous administration o vitamin K is e ective. T e
include markers o hemolysis: hemoglobinuria, elevated serum- pre erred route o administration o vitamin K is oral. Giving
ree Hgb, reduced haptoglobin, and elevated bilirubin. Patients vitamin K IV is second best—lowers INR the same degree as
are positive or direct antiglobulin (Coombs) test. T erapy oral vitamin K at 24 hours, but the IV route may be pre erred in
includes IV saline at a high enough rate to initiate a brisk diure- instances where GI absorption is questionable. Avoid vitamin K
sis and prevent Hgb rom precipitating in the kidneys causing SQ or IM, which are less e ective than PO and IV routes. T e
acute tubular necrosis. e ects o FFP rarely last 24 hours; the e ects o vitamin K are
usually not apparent or 12 to 24 hours. T e use o cryoprecipi-
tate will not provide the appropriate actors depleted by war a-
HELPFUL TIP: rin. Cryoprecipitate contains vWF, actor VIII, actor XIII, and
Additional trans usion reactions include brinogen. Platelet trans usion would not bene t this patient
• Anaphylaxis (especially in those with IgA de iciency) since war arin does not a ect platelet unction.
• Febrile, nonhemolytic reactions (which respond to
meperidine and acetaminophen—“Finally, a use or
HELPFUL TIP:
meperidine!” you exclaim)
REVERSING WARFARIN—The most recent recommen-
• Trans usion related lung injury, which presents as
dations suggest using a 4- actor prothrombin complex
ARDS with dyspnea and hypoxia.
concentrate (e.g., Kcentra) or reversing li e-threatening
• Don’t orget that luid overload rom trans usions
bleeding secondary to war arin. A 3- actor prothrombin
may cause dyspnea.
complex concentrate plus FFP is an alternative; 3- actor
PCCs lack actor VII. I these are not available, FFP is still
an option.
HELPFUL TIP:
Why did we always order “2 units o packed red cells?”
In a nonbleeding patient, the trans usion o one unit o
PRBCs can be expected to raise the hematocrit by 3% HELPFUL TIP:
to 4% (1 g/dL or Hb). Current guidelines recommend The INR o FFP is 1.5. No matter how hard you try, you
giving one unit and reassessing the patient. Every unit cannot reduce the INR to less than 1.5 with FFP.
o blood has its own risk o mismatch, contagion, etc.
Objectives: Did you learn to . . .
• Decide when blood trans usion is appropriate?
CASE 6.5 • Recognize hemolytic trans usion reactions?
• Treat war arin-induced hypocoagulability?
A 56-year-old male presents to the ED with an acute abdo-
men, likely rom a per orated diverticulum. He is taking war-
arin or a DV that occurred a er a total knee arthroplasty.
CASE 6.6
He weighs 65 kg. You are evaluating him or surgery and A 20-year-old emale with acute myeloid leukemia completed
nd the ollowing laboratory results: Hgb 14.3 g/dL, platelet her second cycle o consolidation chemotherapy 5 days ago.
count 478,000/mm3, INR 3.5, and P 28 seconds. She presents to the ED complaining o atigue and ever.
She denies cough, dysuria, abdominal pain, sinus drain-
Question 6.5.1 Which o the ollowing statements about his age, or redness around her Hickman catheter. Her physical
preoperative management is correct? examination reveals a temperature o 38.4°C, pulse 100 bpm,
A) Platelet trans usion perioperatively will produce the most BP 120/58 mm Hg, and respirations 14 breaths per min-
immediate reduction in risk o bleeding rom war arin. ute. Her examination is otherwise unremarkable, including
B) Fresh rozen plasma (FFP) trans usion will produce the no redness or tenderness at the Hickman site. Your magic
most immediate reduction in risk o bleeding rom war arin. crystal ball tells you she does not have a line in ection. Her
178 FAMILY MEDICINE EXAMINATION & BOARD REVIEW

laboratory results reveal the ollowing: WBC 200/mm3, Hgb TABLE 6-2 SUGGESTED ANTIBIOTIC REGIMENS
9 g/dL, hematocrit 27%, and platelet count 47,000/mm3. FOR NEUTROPENIC FEVER
Blood cultures have been drawn.
High-Risk Patients (ANC ≤ 100/mm 3 or expected duration o
neutropenia > 7 days or signi cant comorbidity)
Question 6.6.1 What is your next step, and what is your
Comorbidities = hypotension, pneumonia, new onset abdominal pain or
rationale?
neurologic changes
A) Administer IV amphotericin B; a Candida urinary tract
in ection is most likely. Single Agents
B) Administer IV ce epime; she requires empiric coverage or Antipseudomonal β -lactam
both Gram-negative and Gram-positive organisms. • Ce epime
C) Administer IV na cillin; a Gram-positive bacterial in ection • Ce tazidime
is most likely and broader antibiotic coverage will encourage • Piperacillin-tazobactam
growth o resistant bacteria. Carbapenem
• Imipenem-cilastatin
D) Administer IV vancomycin; she most likely has an MRSA
• Meropenem
sinus in ection.
E) Close observation; there is no ocus o in ection and she Combination Agents
looks well. Add one of the following to the above regimen for management
of complications or suspected/proven resistance:
Answer 6.6.1 The correct answer is “B.” T is patient has a • Aminoglycoside
neutropenic ever, which is a medical emergency. Prompt treat- • Fluoroquinolone (cipro oxacin or levo oxacin)
ment with broad-spectrum antibiotics has drastically improved • Vancomycin
the survival o patients with neutropenic ever. Neutropenia is Additional considerations:
usually de ned as neutrophils plus bands (absolute neutrophil
• Add vancomycin i a line in ection is suspected, MRSA is likely, skin
count, ANC) < 500/mm 3 or < 1,000/mm 3 when the nadir has or so t tissue in ection, patient is clinically more ill (hypotension,
not been reached. Most myelosuppressive chemotherapeutic hemodynamically instable, etc.), or i no improvement a ter
agents produce a reduction in WBCs 4 to 10 days a er com- 3–5 days o empiric therapy.
pletion and nadir at 10 to 14 days. Fever is de ned as a single • Add linezolid or daptomycin or VRE, or patient intolerant to
vancomycin.
oral temperature > 101°F (38.3°C) or a temperature > 100.4◦ F
• Add metronidazole or abdominal symptoms or suspected
(38°C) persisting or 1 hour or more. T is patient does not need Clostridium dif cile in ection.
vancomycin since the absence o a line in ection was stipulated • Add anti ungal ( uconazole, voriconazole, amphotericin B) i still
in the history (via crystal ball that always works in our experi- ebrile 4–7 days a ter a broad-spectrum antibacterial regimen and
ence). Broad-spectrum coverage o Gram-positive and Gram- no identi ed source o ever. The risk o ungal in ection is increased
by this point.
negative organisms, including Pseudomonas, is the cornerstone
• Add acyclovir and/or uconazole i there is oral ulceration.
o therapy with speci c therapy or any localizing symptoms or
risk signs. See able 6-2 or common antibiotic regimens. Low-Risk Patients (anticipated neutropenia < 7 days, able to take PO,
hemodynamically stable, no comorbid conditions, able to comply with
daily ollow-up):
HELPFUL TIP: • PO cipro oxacin and amoxicillin-clavulanate
Patients with a neutropenic ever can have any in ec- OR
tion seen in normal hosts, but you should also con- • Hospital admission and any o the above regimens or persistent
sider IV catheter in ections, perirectal in ections and ever or worsening symptoms.
abscesses, and necrotizing enteritis (aka “typhlitis”).
Data rom In ectious Diseases Society o America. Frei eld AG, et al. Clinical
Remember that they may not have an in lammatory practice guideline or the use o antimicrobial agents in neutropenic
reaction around a catheter site, abscess ormation, in- patients with cancer: 2010 update by the In ectious Diseases Society o
iltrate on chest x-ray, or WBCs in the urine because o America. Clin In ect Dis. 2011;52(4):e56–e93.
the neutropenia. The absence o signs or symptoms
should not dissuade the physician rom starting empiric
antibiotic therapy. Similarly, the absence of fever in a o dull, nonradiating back pain in the lower thoracic/upper
neutropenic patient with focal infection should be lumbar area. He denies trauma or any new activities. He has
approached as a high-risk situation. no associated weakness or paresthesias. He denies di culties
with bowel or bladder unction.

Question 6.7.1 What is your initial diagnostic and/or thera-


CASE 6.7 peutic approach to this patient?
A 63-year-old male with a diagnosis o non–small-cell lung A) MRI o thoracic and lumbar spine.
cancer, undergoing weekly chemotherapy and radiation to a B) Plain lms o the thoracic and lumbar spine and a COX-2
le upper lobe mass, presents to your o ce. He complains inhibitor with a 2-week ollow-up.
CHAPTER 6 • HEMATOLOGY AND ONCOLOGY 179

C) Plain lms o the thoracic spine and NSAIDs. D) Relapse o the Hodgkin lymphoma.
D) Prescription or physical therapy and NSAIDs. E) None o the above.
E) Urinalysis with culture and antibiotics.
Answer 6.8.1 The correct answer is “E.” T e point here is that
Answer 6.7.1 The correct answer is “A.” T is is not your aver- patients with a history o Hodgkin lymphoma and chest radia-
age back pain. Any patient with active malignancy complain- tion are at risk or a wide range o complications—even years
ing o back pain should be investigated or metastasis. While a a er the disease has been success ully treated. Even though 80%
plain lm o the spine may be use ul, the gold standard is MRI. o Hodgkin lymphoma patients have long-term disease- ree sur-
T e real emergency is spinal cord impingement. Spinal cord vival, one in six patients can be expected to die rom late e ects
compression may occur rom direct extension o metastatic o therapy. T e key late e ects o radiotherapy or Hodgkin
disease rom the vertebrae or extension rom retroperitoneal or lymphoma include secondary malignancy and accelerated car-
paravertebral disease. Frequently, the pain predates neurologi- diac disease at a younger age. Other delayed complications that
cal symptoms, and because o the potential or severe, adverse have been associated with radiation therapy include thyroid
outcomes, you want to catch the disease early to prevent chronic dys unction, pulmonary brosis, noncoronary atherosclerotic
impairment. disease, and muscle atrophy.
Although coronary artery disease (“A”) would be extremely
T e patient undergoes MRI o the spine, which demonstrates unusual in a normal 32-year-old, a patient with a history o
a lesion compressing the cord at L1. chest radiation has a relative risk o coronary disease o 5 to
10 times that o age-matched controls. “B,” thyroid disease, is
Question 6.7.2 Which o the ollowing is NOT an appropri- highly likely, with over 50% o patients treated with chest radia-
ate therapeutic modality? tion requiring thyroid hormone replacement. A SH would be
A) Decompression surgery. adequate screening. T ere is a high risk or secondary malig-
B) Dexamethasone 10 mg bolus IV, ollowed by 6 mg every nancy, including breast cancer, lung cancer (“C”), leukemia, sar-
6 hours. coma, and non-Hodgkin lymphoma (NHL). Finally, you should
C) Observation with pain control (e.g., with morphine PCA). always be concerned about recurrence (“D”).
D) Radiation therapy to the a ected area.
T e patient has a normal chest radiograph and ECG.
Answer 6.7.2 The correct answer is “C.” Patients with spinal However, her SH is markedly elevated, and you start her
cord compression who have aggressive interventions are more on levothyroxine. You plan to see her back in 8 weeks or
likely to retain unction, including ambulation and bowel and re-evaluation and will provide additional counseling at that
bladder control. Steroids will help reduce edema surround- time.
ing the tumor and hope ully will relieve pressure on the cord.
Radiation can o en provide symptomatic relie and reduce the Question 6.8.2 Which o the ollowing preventive health
likelihood that the tumor will spread locally to impinge on the issues is/are necessary to address at ollow-up?
cord. Surgical decompression is another option. O course, i A) Early intervention by a ertility specialist i the patient
this patient opted or a palliative care approach and had decided desires pregnancy.
upon hospice, “C” might be appropriate, but he is still getting B) Smoking cessation.
active therapy, and his remaining time could be spent with less C) Yearly mammograms.
pain and better unction i one o the other options were chosen. D) All o the above.
E) None o the above.
CASE 6.8 Answer 6.8.2 The correct answer is “D.” Female patients who
A 32-year-old woman presents to your o ce with com- were treated or Hodgkin lymphoma with chemotherapy and
plaints o dyspnea, constipation, menorrhagia, and atigue radiation prior to age 20 have up to a 35% incidence o breast
that are new over the last ew weeks. She has a distant his- cancer by the age o 40. T e typical latency period is 15 years.
tory o Hodgkin lymphoma treated with chemotherapy and National guidelines recommend that annual breast cancer
radiation to the chest. Her physical examination reveals a screening o Hodgkin lymphoma survivors treated with chest
well-developed woman, who appears com ortable at rest with irradiation should begin 5 to 8 years post-treatment, or age 40,
normal vital signs. She has no adenopathy and the remainder whichever comes rst. Breast cancer screening or these high-
o her examination is unremarkable. Her CBC with WBC di - risk patients includes breast MRI in addition to mammography.
erential is normal. Smokers with a history o Hodgkin lymphoma have a 20- old
increased chance o developing lung cancer when compared to
Question 6.8.1 Which o these diagnoses can be absolutely nonsmokers with a history o Hodgkin lymphoma. Also, emale
ruled out based on this patient’s history? patients with Hodgkin lymphoma have a 69% incidence o pre-
A) Coronary ischemia. mature ovarian ailure i treated or their cancer be ore the age
B) Hypothyroidism. o 29 and up to 96% i treated a er the age o 30. While these
C) Lung cancer. statistics are improving with newer chemotherapy regimens, a
180 FAMILY MEDICINE EXAMINATION & BOARD REVIEW

patient should seek early re erral to a ertility specialist i she the kidney, and it can be prevented by starting allopurinol prior
desires pregnancy but is unable to conceive. to the administration o chemotherapy. Although dehydration
(“A”) is not the cause o renal ailure, it certainly will exacerbate
CASE 6.9 the situation.
A 40-year-old male who has just received his rst course o
Question 6.9.3 Which o the ollowing is MOST LIKELY to
chemotherapy or non-Hodgkin lymphoma (NHL) presents
help this patient’s current condition?
to your ED complaining o weakness, cramps, and decreased
A) Allopurinol 300 mg orally.
urine output. He has no other medical problems and takes
B) Calcium carbonate 500 mg orally.
no medications except or prochlorperazine as needed or
C) Emergent hemodialysis.
nausea. He has been eating and drinking well. His physical
D) IV D5W with 2 amps bicarbonate.
examination reveals a tired-appearing male, with normal
E) IV normal saline 200 mL/hr.
vital signs. His head and neck examination reveals bilateral
palpable cervical lymph nodes (2 cm). His lungs are clear in
Answer 6.9.3 The correct answer is “C.” T e horses are already
the upper lung elds with crackles bilaterally at the bases.
out o the barn, and in act they’re probably over the hills and
Muscle strength is normal, and ref exes are 3+ and symmet-
through the woods. Allopurinol and oral calcium will not help
ric. He exhibits 6 beats o clonus at the ankles. Chest radio-
this situation. T e patient already has renal ailure rom his
graph shows evidence o early pulmonary edema.
tumor lysis syndrome. While he may bene t rom preventive
measures, including aggressive hydration and allopurinol prior
Question 6.9.1 What is the most likely set o laboratory val- to undergoing chemotherapy, none o these measures are
ues you will f nd or this patient (re erence ranges: potas- going to help his current renal ailure. In addition, the patient
sium 3.5 to 5 mg/dL, phosphate 2.4 to 4.1 mg/dL, uric acid may develop worsening pulmonary edema i given too much
3.5 to 7.2 mg/dL, calcium 8.5 to 10.2 mg/dL). volume with IV uids.
A) Potassium 2.3 mEq/L, phosphorus 3.1 mg/dL, uric acid
4 mg/dL, calcium 9 mg/dL. While awaiting hemodialysis, you should treat the patient’s
B) Potassium 2.6 mEq/L, phosphorus 6 mg/dL, uric acid 5 mg/ hyperkalemia. reatment may include IV calcium gluconate,
dL, calcium 12 mg/dL. insulin and dextrose, and oral sodium polystyrene sul onate
C) Potassium 6.5 mEq/L, phosphorus 7 mg/dL, uric acid (Kayexalate). Note that sodium bicarbonate or hyperkale-
18 mg/dL, calcium 6 mg/dL. mia has allen out o avor. See Chapter 1 or more in orma-
D) Potassium 6 mEq/L, phosphorus 6.8 mg/dL, uric acid 5 mg/ tion on hyperkalemia.
dL, calcium 6.7 mg/dL.
E) Potassium 4.5 mEq/L, phosphorus 3.2 mg/dL, uric acid
9 mg/dL, calcium 8 mg/dL. HELPFUL TIP:
Rasburicase has also been approved or the preven-
Answer 6.9.1 The correct answer is “C.” T is patient most tion o tumor lysis syndrome. It converts uric acid into
likely has tumor lysis syndrome, which can occur in a patient a nontoxic, excretable metabolite allantoin. It can also
with a highly responsive leukemia or a bulky lymphoma being be used a ter tumor lysis syndrome has started. It has
treated with chemotherapy (it rarely occurs without treatment). several black box warnings, including anaphylaxis, he-
umor lysis syndrome occurs when there is rapid release o molysis, and methemoglobinemia. Its use is considered
intracellular contents into the bloodstream. It is characterized high risk in pregnancy. Except or the side e ects noted
by high potassium, high phosphorus, high uric acid, and low above, it is well-tolerated, acts rapidly, and is e ective.
calcium. Patients may experience renal ailure, arrhythmias, The need o dialysis secondary to tumor lysis syndrome
atigue, muscle cramps, and tetany. has substantially declined since the introduction o
Rasburicase.
T e patient is ound to have a creatinine o 8 mg/dL.
Objectives: Did you learn to . . .
Question 6.9.2 What is the most likely cause o this patient’s • Recognize and initiate treatment o some oncologic emer-
renal ailure? gencies, including spinal cord compression, tumor lysis
A) Dehydration. syndrome, and neutropenic ever?
B) Heart ailure. • Recognize late complications o cancer and cancer therapy,
C) Hemoglobinuria. including distant radiation and chemotherapy?
D) Rhabdomyolysis related to chemotherapy.
E) Uric acid nephropathy.
QUICK QUIZ: TOO MUCH OF A GOOD THING
Answer 6.9.2 The correct answer is “E.” Patients with tumor
lysis syndrome have renal ailure secondary to uric acid A 60-year-old gentleman presents to your of ce with complaints
nephropathy. T is is caused by the precipitation o uric acid in o atigue. He has a history o alcoholic cirrhosis, diet-controlled
CHAPTER 6 • HEMATOLOGY AND ONCOLOGY 181

diabetes, and hypertension and currently takes hydrochlorothi- > venous) and bleeding events. In order to diagnose E , other
azide, enalapril, and monthly testosterone injections. He smokes causes o thrombocytosis (e.g., in ammation, iron de ciency,
two packs o cigarettes daily and consumes 6 to 8 beers nightly. recent surgery, in ection, bleeding, and malignancy) must be
His physical examination reveals an obese, ruddy- aced man excluded. A bone marrow biopsy may be help ul in establishing
with a temperature o 37°C, pulse 90 bpm, BP 164/80 mm Hg, the diagnosis by demonstrating adequate iron stores and ruling
and respirations 14 bpm. He is ound to have a hematocrit o out chronic myelogenous leukemia (CML) or myelodysplasia.
54%. “A” is incorrect. War arin would not be appropriate prophy-
lactic therapy and would not be used unless the patient had a
Which o the ollowing items in his history is LEAS likely to thromboembolic event. Aspirin can be used especially or the
explain the elevated hematocrit? vasomotor symptoms associated with essential thrombocythe-
A) Alcoholic cirrhosis. mia (erythromeralgia [a burning pain in the hands and eet],
B) Diabetes. acral paresthesias, headache, lightheadedness, etc.). Aspirin
C) Antihypertensive medications. should also be considered i the platelet count is greater than
D) estosterone injections. 1.5 million. “B” is incorrect. Although patients with E have
E) Smoking. a high rate o spontaneous abortion, many can have normal,
healthy pregnancies. T e patient should be counseled regarding
The correct answer is “B.” Diabetes should not cause an pregnancy risks. “C” is also incorrect. T e patient is at low risk
elevated hematocrit and o en causes anemia secondary to or thromboembolic events (platelet count < 1.5 million/mm 3,
renal disease and reduced responsiveness to erythropoietin. young age, no comorbid illness, or prior events). Hydroxyurea
Approach an elevated hematocrit with two questions: (1) Is it could be considered i the patient was at high risk or thrombo-
due to increased RBC mass or decreased plasma volume? (2) Is embolic events or symptomatic.
it primary erythrocytosis or secondary?
T is patient has many potential secondary causes o an
elevated hematocrit. “A,” alcoholic cirrhosis can lead to hepa- HELPFUL TIP:
tocellular carcinoma which, along with other malignancies, can The most common cause o an abnormally high platelet
result in overproduction o erythropoietin, causing an elevated count is reactive thrombocytosis, which can result rom
hematocrit. Diuretics (“C”) decrease plasma volume, causing an iron de iciency, in ection, in lammation, or malignancy.
apparent elevation o hematocrit. estosterone injections (“D”) There is no increase in bleeding or clotting risk in pa-
may cause polycythemia. Finally, he has a signi cant smoking tients with reactive thrombocytosis. The diagnosis o ET
history (“E”) that may produce a secondary polycythemia due can only be made once reactive thrombocytosis and the
to hypoxia and cor pulmonale. presence other chronic myeloproli erative disorders are
ruled out.
CASE 6.10
A 30-year-old emale presents to your o ce or a routine
visit. She was hospitalized or an appendectomy and at the
CASE 6.11
time o surgery, her platelet count was ound to be 1,400,000/ A 15-year-old emale presents to your o ce complaining o
mm3, which her surgeon elt was most likely reactive. She has atigue. She reports menarche at age 13 and complains of heavy
no other past medical history, is asymptomatic, and exercises menses. Her physical examination reveals a well-developed,
three times per week. You repeat the CBC, showing WBC well-nourished, pale emale. You nd no hepatosplenomeg-
5000/mm3, Hgb 13 g/dL, and platelet count 800,000/mm3. aly. Her laboratory results reveal a WBC 6,000/mm3, Hgb
8.9 g/dL, hematocrit 27%, platelet count 400,000/mm3, MCV
Question 6.10.1 What is your next step in managing this 72 L, red blood cell distribution width (RDW) 16. You order
patient? more laboratory tests.
A) Anticoagulation with war arin to a goal INR o 2 to 3.
B) Counseling against becoming pregnant. Question 6.11.1 What are the expected f ndings in this
C) Initiation o hydroxyurea 500 mg BID. patient?
D) No urther evaluation or ollow-up necessary. A) Increased iron, decreased erritin, increased total iron bind-
E) Observation and periodic evaluation o her CBC. ing capacity.
B) Decreased iron, decreased erritin, decreased total iron
Answer 6.10.1 The correct answer is “E.” T is patient likely binding capacity.
has essential thrombocythemia (E ), which is the most com- C) Increased iron, increased erritin, increased total iron
mon myeloproli erative disorder in the United States. E is binding capacity.
more common in emales. Patients are typically older when D) Decreased iron, increased erritin, decreased total iron bind-
diagnosed with median age o 60. It is rare in children younger ing capacity.
than 14 years o age. Patients with E have a higher rate o mor- E) Decreased iron, decreased erritin, increased total iron bind-
tality than matched controls due to risk o thrombosis (arterial ing capacity.
182 FAMILY MEDICINE EXAMINATION & BOARD REVIEW

TABLE 6-3 CAUSES OF ANEMIA BY RED CELLVOLUME


Low MCV (usually < 80 L) Normal MCV (usually 80–100 L) High MCV (usually > 100 L)
Anemia o chronic disease Acute blood loss Alcohol ef ects
Copper de ciency Anemia o chronic disease B12 de ciency
Iron de ciency anemia Chronic renal insu ciency Drug ef ect (e.g., Hydroxyurea, AZT)
Lead poisoning Early iron de ciency Folate de ciency
Sideroblastic anemias Endocrine (e.g., hypothyroidism) Hemolytic anemia
Thalassemias Primary bone marrow disorders Hypothyroidism (less commonly macrocytic,
usually normocytic)
Liver disease
Primary bone marrow disease
Reticulocytosis (hemolytic anemia, response to
blood loss)

Answer 6.11.1 The correct answer is “E.” T is patient likely hemolysis) will there be ree hemoglobin in the blood. “D” is
has iron de ciency anemia related to her heavy menses. Iron incorrect because the erritin is low in iron de ciency anemia
de ciency anemia is characterized by anemia along with a and should increase with therapy. Finally, trans errin saturation
decreased serum iron, decreased erritin, increased total iron (“E”) should increase in patients once you start to treat their
binding capacity ( IBC), and decreased trans errin saturation. anemia, but the reticulocyte count increases rst.
T e decrease in serum erritin is proportional to the decrease in
total body iron stores. Hypochromic microcytic RBCs are ound HELPFUL TIP:
on peripheral smear. See able 6-3 or a general guide to the Ferritin is not a use ul test or iron de iciency in hospital-
causes o anemia based on red cell indices. ized patients or in those who are chronically ill. Ferritin
is an acute-phase reactant and thus may be elevated in
HELPFUL TIP: these patients even when the patient has iron de icien-
The prevalence o VWD in women with menorrhagia cy anemia (where the erritin should be low). However,
ranges rom 5% to 20%. Always make sure to take a you can check a soluble trans errin receptor. See below
good personal and amily bleeding history. or more.

HELPFUL TIP: HELPFUL TIP:


Even i it looks like iron de iciency anemia, always con- There may be no reticulocytosis with treatment o iron
sider other causes o anemia such as B12 de iciency, de iciency i the patient is simply iron de icient without
olate de iciency, and thalassemia. O ten patients will anemia.
have more than one cause or their anemia. Mixed vita-
min B12 and iron de iciencies may present with normo- Question 6.11.3 How long should you continue iron sup -
cytic anemia with an elevated RDW. plementation once the patient’s labs have normalized?
A) Stop immediately once anemia has resolved.
B) Continue 3 to 6 months a er the anemia has resolved.
You start iron supplementation therapy in this patient. C) Continue or 1 year a er the anemia has resolved.
D) Inde nite iron supplementation is indicated.
Question 6.11.2 Which o the ollowing tests will be the f rst
to indicate that you have instituted appropriate therapy Answer 6.11.3 The correct answer is “B.” Continue iron or
and that the patient is responding? 3 to 6 months once the anemia has resolved. Also address the
A) Increase in hematocrit. underlying problem, in this patient her heavy periods, which
B) Increase in reticulocyte count. may respond to hormonal contraception, tranexamic acid, etc.
C) Increase in serum- ree Hgb.
D) Decrease in erritin. T e patient returns in 2 months but her labs, i anything, are
E) Decrease in trans errin saturation. worse than at rst presentation. T e patient swears that she
has been taking the iron aith ully.
Answer 6.11.2 The correct answer is “B.” T e patient’s retic-
ulocyte count will increase rst—be ore the hematocrit (“A”). Question 6.11.4 Which o the ollowing can lead to a ailure
T is should start soon a er treatment and maximize at 7 to o iron therapy or iron def ciency anemia?
10 days. Pica (i present) should also resolve airly early. “C” A) Proton pump inhibitors (PPIs).
is incorrect. Only in exceptional circumstances (intravascular B) Incorrect diagnosis.
CHAPTER 6 • HEMATOLOGY AND ONCOLOGY 183

C) Oral antacids (e.g., calcium carbonate). is incorrect because calcium will inter ere with iron absorption.
D) Atrophic gastritis, celiac disease, or Helicobacter pylori in ec- Not only that but also she has already said she would not take
tion. additional oral iron. “B” is incorrect because she needs more
E) All o the above. iron than can be provided through prenatal vitamins and her
diet. Finally, “D” is incorrect as trans usion carries potential
Answer 6.11.4 The correct answer is “E.” Anything that neu- risks that could be avoided i she responds to IV iron replace-
tralizes the stomach pH will inter ere with absorption including ment.
PPIs, antacids, and loss o acid producing cells (e.g., pernicious
anemia). Other GI diseases (celiac disease, H. pylori) can also
inter ere with iron absorption. ea and some green lea y veg- HELPFUL TIP:
etables can also reduce iron absorption. Any adult patient with microcytic anemia should be
evaluated urther to clari y the etiology. In adults, GI
blood loss is a common cause o microcytic anemia.
HELPFUL TIP: Colitis, malignancy, or malabsorption rom in lamma-
Vitamin C (supplements or orange juice) enhances iron tory disease should be considered in the di erential di-
absorption and should be considered i a patient is not agnosis. Tailor your work-up to the patient’s symptoms.
responding to iron therapy. Meat can also increase iron I he has symptoms re erable to the upper GI tract (e.g.,
absorption (which we hate to say because one o us is a dyspepsia), consider an upper GI endoscopy in addition
vegetarian . . . however, the truth hurts). to colonoscopy.

CASE 6.12
HELPFUL TIP:
A widened RDW and an elevated platelet count are typi- A 52-year-old woman with a history o rheumatoid arthritis
cal o iron de iciency anemia. Conversely, the RDW will is in your clinic or a 1-month ollow-up a er having a knee
be normal in thalassemias. prosthesis removed secondary to a joint in ection and osteo-
myelitis (Staphylococcal aureus). You obtain a CBC, revealing
a WBC 8,000/mm3, Hgb 9.5 g/dL, hematocrit 28%, platelet
count 450,000/mm3, and MCV 83 L (normal). Serum iron
T e patient returns to your o ce and nally admits she levels are low with a normal serum trans errin receptor and
has not been able to take the iron because o side e ects. increased erritin.
Her hemoglobin is now down to 7.2 g/dL. She still eels
atigued. T e patient will not agree to take any urther iron Question 6.12.1 What is the most likely diagnosis?
orally. However, she is willing to consider other sugges- A) Iron de ciency anemia due to rheumatoid arthritis.
tions. B) Anemia o chronic disease due to rheumatoid arthritis and
osteomyelitis.
Question 6.11.5 What is your next step? C) Hemolytic anemia induced by antibiotics.
A) Encourage the patient to take the iron preparation along D) Acute blood loss during surgery.
with calcium carbonate ( ums) to reduce the GI side e ects. E) Myelodysplastic syndrome (MDS) associated with rheuma-
B) Continue her prenatal vitamin only and encourage her to eat toid arthritis.
more red meat.
C) Give iron sucrose 200 mg IV weekly or 4 weeks. Answer 6.12.1 The correct answer is “B.” Anemia o chronic
D) rans use 2 units o PRBCs immediately. disease is a hypoproli erative anemia that occurs in the setting
o chronic in ection, in ammation, malignancy, heart ailure,
Answer 6.11.5 The correct answer is “C.” I oral iron prepara- diabetes, and other serious health conditions. T e anemia is
tions are not tolerated, IV iron preparations are available. Intra- usually mild and characterized by low serum iron, increased
muscular preparations are best avoided due to pain at the injec- erritin (remember that erritin is an acute-phase reactant and
tion site, skin discoloration, and risk or in ection. T e most these patients o en have in ammation), decreased serum trans-
commonly used options or IV replacement include iron dex- errin, normal (or low) serum soluble trans errin receptor level,
tran and iron sucrose. Iron dextran carries a risk o anaphylaxis and decreased trans errin saturation (see below or more on the
in 0.6% to 2.3% o patients and other side e ects in up to 25% soluble trans errin receptor). In addition, the reticulocyte count
o patients, including bronchospasm, ushing, headache, ever, is typically low, the erythropoietin may be mildly elevated, and
urticaria, nausea, vomiting, hypotension, seizures, myalgias, the peripheral smear may show hypochromic, microcytic RBCs
arthralgias, and increased thromboembolic events. Iron sucrose or normochromic, normocytic RBCs. I di erentiation between
has a lower incidence o side e ects—typically nausea, constipa- iron de ciency anemia and anemia o chronic disease is not
tion, diarrhea, or a transient minty taste—and may be given to apparent, a bone marrow biopsy can be obtained to assess iron
patients who have had a previous reaction to iron dextran. “A” stores.
184 FAMILY MEDICINE EXAMINATION & BOARD REVIEW

urine. We could have tipped you o to the answer with a “Flint,


HELPFUL TIP:
Michigan” re erence but that would have been no un.
What is that trans errin receptor anyhow? The trans er-
rin receptor can help to di erentiate between iron de-
iciency anemia and anemia o chronic disease. The se- HELPFUL TIP:
rum trans errin receptor level is inversely correlated to Lead levels > 10 mcg/dL may cause developmental
iron storage levels. When the iron is low, the soluble delay, loss o milestones (especially language), encepha-
transferrin receptor level is high. Thus, high serum lopathy, seizures, cerebral edema, and cognitive impair-
transferrin receptor levels are associated with iron ment. CNS e ects are especially problematic in children
deficiency but not with anemia of chronic disease in < 6 years old who have an incomplete blood–brain bar-
which iron stores are adequate. One caveat, the serum rier. Lead paint in houses built be ore the 1970s and use
trans errin receptor level will also be elevated in states o imported products such as pottery, solder, cosmetics,
in which there is rapid cell turnover (hemolytic anemia, and crayons (and some toys made in China) still provide
or example). However, it should not be checked in this sources o lead ingestion. Children born in a oreign
situation so you shouldn’t get con used. country and/or with recent oreign residence may also
be at risk.

CASE 6.13 HELPFUL TIP:


Other symptoms o lead intoxication include: anorexia,
A previously healthy 3-year-old emale with a history o decreased activity, irritability, insomnia, hearing loss,
anorexia and irritability or 3 days is brought to your ED by peripheral neuropathy, SIADH, decreased renal unc-
her mother. On the day o admission, the child is di cult to tion, and anemia. Laboratory indings may include: ane-
arouse. You learn that her 6-year-old brother has had some mia, signs o hemolysis, coarse basophilic stippling on
di culty reaching appropriate developmental milestones, RBCs, glycosuria, hypophosphatemia, positive qualita-
and is a “ ussy eater.” tive urine coproporphyrin, and moderate increases in
Physical examination reveals a slightly pale-appearing ree erythrocyte protoporphyrin.
child, who responds to tactile stimuli but not to voice. Vitals
are normal and her examination is unremarkable except
or her decreased level o consciousness. Laboratory stud-
ies include a WBC 8,000/mm3, Hgb 10 g/dL, platelet count T e patient’s lead level is 70 mcg/dL. Because o your concern
300,000/mm3, and MCV 75 L (microcytic). Her urine dipstick about acute lead intoxication and resulting encephalopathy,
is normal, except or 1 + glucose; there are no ketones in the you decide to admit the child and treat her with dimercaprol.
urine. Her blood chemistries are within normal limits, with
the exception o a phosphorous o 2.0 mg/dL. Her peripheral Question 6.13.2 All o the ollowing are true EXCEPT:
blood smear shows red cells with coarse basophilic stippling. A) Dimercaprol should not be given i the child has a peanut
allergy.
Question 6.13.1 What is the best working diagnosis at this B) T e child’s diet should be monitored or adequate intake o
time? calories, iron, and calcium.
A) Acute lead poisoning. C) T e child’s sibling should be tested, and arrangements
B) Anemia o chronic disease. should be made or close ollow-up or both children.
C) Early diabetic ketoacidosis (DKA). D) T e dose o dimercaprol should be tripled or patients with
D) Severe iron de ciency anemia. glucose-6-phosphate dehydrogenase (G6PD) de ciency.
E) Unrecognized bacteremia secondary to pyelonephritis.
Answer 6.13.2 The correct answer is “D.” Dimercaprol is a
Answer 6.13.1 The correct answer is “A.” Lead poisoning lead chelator used to treat elevated lead levels. It is adminis-
should be considered in a child presenting with symptoms tered intramuscularly, is water insoluble, and must be given in a
o encephalopathy and anemia with basophilic stippling on peanut oil vehicle. It may cause hemolysis in individuals with
RBCs. Basophilic stippling occurs when ribosome precipitates G6PD de ciency, so these patients should be monitored closely
litter the RBCs. Basophilic stippling can be seen in alcohol and not receive larger doses o dimercaprol. It is important to
abuse, thalassemias, and heavy metal poisoning. “B,” anemia test siblings o an a ected child, because the source o lead is
o chronic disease, is unlikely because this illness is acute. “C,” o en in the house. Close ollow-up o a ected children is essen-
DKA, is unlikely because urine ketones are 99% sensitive or tial to monitor or increasing blood lead levels a er treatment,
DKA and her urine ketones are normal. “D” is unlikely because which may indicate the source o lead has not been removed.
there should be no neurologic symptoms associated with iron T e absorption o lead can be made worse by malnutrition,
de ciency anemia. “E,” pyelonephritis, is unlikely because the iron de ciency, and poor intake o calcium. Iron de ciency
patient is a ebrile and has a normal white count and negative requently occurs in a ected patients and may worsen anemia.
CHAPTER 6 • HEMATOLOGY AND ONCOLOGY 185

Another mainstream chelating agent is calcium disodium C) T e condition will improve with age.
ED A. It is recommended to use a combination o dimercap- D) T e child is likely to have growth retardation and delayed
rol and ED A or children with severe lead intoxication. Other sexual maturation.
chelating agents include d-penicillamine and succimer. E) Without iron chelation therapy, there is a high incidence o
mortality with this disease.

HELPFUL TIP: Answer 6.14.2 The correct answer is “C.” Beta thalassemia
A large percentage o lead is absorbed into bone with re ers to the disease state in which there are mutations in
a hal -li e o greater than 25 years. In periods o physi- both genes that code or beta globin. Without treatment,
ologic stress, the inert pool o lead can be mobilized mortality rom thalassemia major approaches 80% by 5 years
and released into the bloodstream, producing signs o age. Patients with beta thalassemia will eventually develop
and symptoms o lead intoxication years a ter the initial iron overload, even without blood trans usions. he use o
exposure. trans usions and concurrent chelation therapy with de er-
oxamine has reduced long-term complications related to
the disease. De erasirox (Exjade), an oral chelating agent,
CASE 6.14 is available as an alternative to de eroxamine. Bone marrow
transplant can be curative. Beta thalassemia minor (or beta
A 6-month-old male in ant is brought to you or a well-baby trait) is much less severe and occurs when only one o the
check-up. His mother reports that the child has had di culty genes is de ective.
with eeding, has requent colds, and is o en irritable. She
has noted recurrent episodes o jaundice, especially when he
is sick. His physical examination reveals a small child, aintly HELPFUL TIP:
jaundiced. His temperature is 37.5°C, pulse 160 bpm, BP The FDA has approved de eriprone as an alternative
85/50 mm Hg, and respirations 30 bpm. His abdomen is so chelating agent in trans usion-acquired iron overload.
with palpable splenomegaly. T e remainder o the examina- It is second line but can be administered orally. It can
tion is unremarkable. Laboratories reveal a WBC 13,000/mm3, be considered when de erasirox is not e ective and the
Hgb 9 g/dL, platelet count 260,000/mm3, and MCV 60 L (low). patient is unable to use de eroxamine, the parenteral
He is noted to have target cells on peripheral blood smear. option.

Question 6.14.1 Which one o the ollowing is the most


appropriate next step? HELPFUL TIP:
A) Order P /P . Alpha thalassemia has a more variable course. There are
B) Order Hgb electrophoresis. our alpha globin genes. I all our genes are de ective,
C) rans use with 1 unit o PRBCs. intrauterine etal demise is the rule. When one gene is
D) est or G6PD de ciency. de ective, there is a silent carrier state. When two genes
E) est or sickle-cell anemia. are de ective, there is a mild microcytic anemia. Here is
a memory aid: there are our alpha globin genes and
Answer 6.14.1 The correct answer is “B.” T e clinical and “4” looks kind o like the Greek letter alpha; beta is the
laboratory presentation is consistent with a hereditary hemo- second letter in the Greek alphabet and beta globin has
globinopathy, most likely a thalassemia. T e splenomegaly, sig- two genes.
ni cant anemia, target cells, and the pro oundly low MCV are
all suggestive o thalassemia; patients with thalassemia o en
have recurrent episodes o jaundice re ecting hemolysis. He has
no history o abnormal bleeding, so “A” would not be indicated. HELPFUL TIP:
Also, as the child does not appear to be in distress, “C” would Because o the persistence o etal hemoglobin (HbF) in
be an incorrect answer. G6PD-de cient patients experience epi- the circulation up to 6 months o age, hemoglobinopa-
sodes o hemolytic anemia. Peripheral smear review at that time thies might not be apparent until that time.
may reveal bite or blister cells, rather than target cells, so “D” is
not the right answer. “E” is incorrect since the peripheral smear
would reveal sickle cells i the patient had sickle disease. HELPFUL TIP:
Thalassemias may be erroneously diagnosed as iron de-
Beta thalassemia major is con rmed by Hgb electrophoresis. iciency anemia due to a low MCV. Consider testing or
thalassemia in a patient with microcytic anemia, normal
Question 6.14.2 Which o the ollowing regarding his man- or increased iron levels, a normal RDW (less reliable),
agement is NOT true? and appropriate ethnic background (e.g., Mediterra-
A) Untreated, mortality may approach 80% by 5 years o age. nean or A rican descent).
B) Frequent blood trans usions may be required.
186 FAMILY MEDICINE EXAMINATION & BOARD REVIEW

HELPFUL TIP: T e child ollows regularly with you or years, and she begins
In other Hgb synthesis related news . . . when you see to develop more requent pain crises in college. She is ound
that middle-aged emale with abdominal pain and psy- to have an elevated C-reactive protein, increased LDH, and
chiatric symptoms or the tenth (or twentieth or thirti- decreasing Hgb. You explain to her that some measures can
eth) time, think o acute intermittent porphyria be ore be taken to reduce the requency o pain crises.
you make the diagnosis o somatization disorder.
Question 6.15.2 These measures include all o the ollowing
EXCEPT:
A) Initiation o hydroxyurea therapy.
CASE 6.15 B) Avoidance o hot showers.
C) Prompt treatment o in ections.
A woman brings her 10-week-old in ant or evaluation.
D) Adequate hydration and nutrition.
T e child is colicky and irritable a er eeding. She was
E) Avoidance o emotional stress.
born ull term but has not been gaining weight appropri-
ately. T e patient’s sister has sickle-cell trait, so you obtain
Answer 6.15.2 The correct answer is “B.” wo main eatures
an Hgb electrophoresis, which ultimately demonstrates SS
o sickle-cell disease are shortened RBC survival and vasooc-
genotype.
clusion due to poorly de ormable RBC membranes. Pain epi-
sodes occur due to acute episodes o tissue hypoxemia and
Question 6.15.1 Which o the ollowing is INCORRECT
microin arction. Crises can be precipitated by stress, atigue,
regarding the child’s care?
cold weather (not hot showers, which cause problems in mul-
A) T e patient should remain on prophylactic antibiotics until
tiple sclerosis patients), in ection, dehydration, acidosis, and
the age o 5 years due to a high risk o pneumococcal sepsis.
poor nutrition. T e use o hydroxyurea promotes increased
B) T e child may develop splenomegaly and lymphadenopathy
levels o HbF, and HbF is associated with decreased requency
related to her disease.
and severity o pain crises.
C) T e disease provides protection against in ection with par-
vovirus.
D) T e child may not develop pain crisis until she is older due HELPFUL TIP:
to protection rom HbF. Acute chest syndrome (ACS) should be suspected when
E) T e child is likely to have a delay in puberty. a sickle-cell patient presents with ever, cough, chest
pain, and pulmonary in iltrates on chest x-ray. ACS may
Answer 6.15.1 The correct answer is “C.” T e patient has be due to in ection or pulmonary in arction. Treatment
homozygous SS sickle-cell disease. T is child’s mother should involves aggressive pain management, antibiotics or
be counseled regarding the importance o continuing antibiotic respiratory pathogens, supplemental oxygen, cautious
prophylaxis until the age o 5 years because her child is at high hydration, and blood trans usion i the patient is sig-
risk or pneumococcal sepsis. Sickle-cell disease is protective ni icantly anemic. Exchange trans usion may also be
against malaria, not parvovirus (“C”). In act, parvovirus in ec- required in more severe cases.
tion can be li e-threatening in these patients due the develop-
ment o aplastic anemia.
During in ancy, children may be noted to have reticulocy-
HELPFUL TIP:
tosis, hemolytic anemia, and sickling by 10 to 12 weeks. At 5 to
For patients with sickle-cell disease, simply replacing
6 months, splenomegaly may be noted, and lymphadenopathy
volume is usually appropriate. However or primary and
may be prominent between 6 months and 5 years. T e earliest
secondary stroke prevention, trans use/exchange trans-
pain crisis o en involves hands and eet (dactylitis), and occurs
use so that the HbS is < 30%. For presurgical patients,
a er the HbF decreases to adult values; this usually does not
simply getting the Hbg to 10 g/dL seems adequate.
occur until a er the age o 4 years. T e spleen involutes by 5 to
8 years. Puberty is delayed by an average o 2.5 years.

Since you rst met that little baby girl, 22 years have elapsed,
and she is getting married—and you are getting old (the alter-
HELPFUL TIP:
native is worse, o course). She is concerned about signi cant
Sickle trait, the condition in which a patient is hetero-
illness related to her disease. You counsel her regarding the
zygous or Hb S, is associated with a normal li e expec-
most common causes o morbidity and mortality.
tancy and no symptoms other than occasional hema-
turia and inability to concentrate urine. Pain crises are
Question 6.15.3 All o the ollowing statements are true
extremely rare, and occur only in the settings o a low
EXCEPT:
oxygen atmosphere (e.g., very high altitude) or extreme
A) ACS and pain crises are the most common causes or hospi-
physical activity (e.g., running a marathon).
tal admission or patients with sickle-cell disease.
CHAPTER 6 • HEMATOLOGY AND ONCOLOGY 187

B) Eighty percent o patients develop cholelithiasis by the age o D) T e patient’s condition will likely require chronic blood
35 years. trans usions.
C) Strokes may occur in 8% o patients by the age o 14 years, E) T e patient may have a normal serum olate level.
may be either symptomatic or clinically silent, and tend to
be recurrent. The correct answer is “D.” T is patient likely has olate de -
D) Heart ailure, pulmonary brosis, pulmonary hypertension, ciency, given a normal B12, macrocytic anemia, and risk ac-
and renal ailure complicate uid management, trans u- tors or olate de ciency—old age, poor diet, avoidance o glu-
sions, and maintenance o adequate oxygenation. ten ( our is normally orti ed with olate), use o phenytoin,
E) Pregnancy should be avoided under any circumstance. and possible malabsorption due to GI disease (celiac disease in
this case, but also any other in ltrating or in ammatory process
Answer 6.15.3 The correct answer is “E.” T e average li e o the bowel). Dietary de ciency is uncommon in the United
expectancy in patients with Hb SS is 42 years or men and States due to supplementation o grain products with olate.
48 years or women. Maternal mortality occurs in about 1% o Foods naturally high in olate include melons, bananas, lea
pregnancies. All o the rest are true including cerebrovascular vegetables, asparagus, and broccoli. T e recommended intake is
events causing seizures and possible cognitive dif culty early in 400 µg/day, and the body stores approximately a 4-month sup-
li e. O particular note, renal ailure may be the cause o death in ply. I he has had recent adequate olate intake, the serum olate
up to 10% o cases. Avascular necrosis and osteomyelitis (clas- level may be normal, but the RBC olate will still be low, re ect-
sically as a result o Salmonella in ection), chronic skin ulcers, ing the de ciency (think o it as the HbA1C o olate).
and priapism are also common. o di erentiate olate rom B12 de ciency, check serum
homocysteine and methylmalonate levels. Both will be elevated
Objectives: Did you learn to . . .
with B12 de ciency, while only homocysteine will be elevated in
• Recognize the presentation and implication o essential
olate de ciency. Remember, you must exclude B12 de ciency
thrombocytosis?
be ore replacing olate because olate replacement can reverse
• Evaluate a patient with an elevated Hgb or platelet count?
the anemia but will permit progression o neurologic e ects o
• Obtain a thorough history in an anemic patient? B12 de ciency, so “B” is true.
• Use laboratory parameters to identi y the etiology o
anemia?
• Initiate treatment o various causes o anemia? HELPFUL TIP:
• Identi y certain anemias, especially iron de ciency, thalas- Folate replacement is dosed as 1 to 5 mg daily or
semias, and sickle-cell disease, based on clinical and labora- 1 to 4 months or until complete hematologic recovery
tory mani estations? occurs. I macrocytic anemia persists, other causes must
be considered.

QUICK QUIZ: THE CURIOUS CASE OF THE


MISSING VITAMIN QUICK QUIZ: REMEMBER THAT TAPEWORM
FROM MEDICAL SCHOOL? HERE IT IS AGAIN
A 72-year-old gentleman is brought to your of ce because
a home health nurse noted he was becoming progressively A 47-year-old woman presents to your of ce complaining o
weaker and more atigued. He has a history o celiac disease tongue soreness, atigue, and dyspnea with exertion. She denies
or which he ollows a strict gluten- ree diet, but he has not unusual bleeding, weight loss, evers, and night sweats. Her past
been eating well. He also has a history o a seizure disor- medical history includes hypothyroidism, or which she takes
der and takes phenytoin. His review o systems is positive levothyroxine. She does not drink alcohol or smoke. Physical
or dyspnea on exertion and mild anorexia. He has no other examination reveals a tired-appearing woman with lemon-
symptoms. yellow colored skin, temperature 37°C, pulse 110 bpm, BP 120/
Physical examination reveals a thin, pale-appearing man with 74 mm Hg, and respirations 12 bpm. T e examination is otherwise
normal vital signs. His conjunctivae are pale and his tongue is unremarkable. A CBC demonstrates a WBC count 4000/mm 3,
smooth, with moist mucosa and no oropharyngeal lesions. T e Hgb 9 g/dL, platelet count 140,000/mm 3, and an MCV o 105 L.
remainder o the examination is unremarkable. A CBC reveals Her B12 level is 100 pg/mL (normal > 300 pg/mL) and olate
a WBC 4,500/mm 3, Hgb 9 g/dL, hematocrit 28%, platelet count 40 ng/mL.
140,000/mm 3, MCV 102 L, RDW 14, and B12 level 900 ng/L
(normal). Which o the ollowing historical elements is LEAS likely to
contribute to her condition?
Which o the ollowing statements is most likely FALSE? A) History o working as a park ranger in Canada.
A) T e patient may have a high homocysteine level. B) History o Zollinger–Ellison syndrome.
B) T e patient’s condition could be made better with a trial o C) History o hypothyroidism.
olate replacement. D) History o new vegetarian diet started last month.
C) T e patient may have a low RBC olate level. E) History o GI surgery.
188 FAMILY MEDICINE EXAMINATION & BOARD REVIEW

The correct answer is “D.” Most likely, this woman has perni- illness. reatment o the disease includes li elong therapy with
cious anemia caused by B12 de ciency. Pernicious anemia is penicillamine or trientine (chelating agents), and should be
caused by immune destruction o parietal cells in the stomach, considered even or asymptomatic individuals known to have
resulting in decreased absorption o B12. It typically develops the disease. By the way, pennies are primarily zinc and contain
in people over the age o 40 and is more common in people o little copper.
northern European descent or in A rican Americans, as well as
people with type A blood. Laboratory ndings include antipa-
rietal cell antibodies in 90% o a ected patients (5% o normal QUICK QUIZ: “HOLE-Y BONES”
individuals also have the antibody) and antibodies to intrinsic
actor in 70% o patients. Individuals commonly have other A 50-year-old man presents to your of ce with low-back pain o
autoimmune diseases such as thyroid disease, diabetes, and vit- 8 weeks duration. He denies any history o trauma or overexer-
iligo. Individuals may complain o paresthesias, GI symptoms, tion. He notes the pain is constant and does not improve with
sore tongue, or weight loss. T e lemon-yellow appearance o the positioning. Also, he has noticed some atigue. He has no other
skin is due to anemia and mild jaundice. complaints.
Other causes o cobalamin (B12) de ciency include gastrec- Physical examination reveals a well-nourished male with nor-
tomy, Zollinger–Ellison syndrome (inability to alkalinize the mal vital signs. His neurologic examination is normal. On mus-
small intestine), blind loop syndrome, bacterial overgrowth culoskeletal examination, he has midline point tenderness over
rom previous surgery, and ingestion o undercooked sh 12. Plain lms reveal a compression racture at 12. A serum
in ested with the tapeworm Diphyllobothrium latum ( ound in total protein level is 9 mg/dL (elevated), but the remainder o an
Canada, Alaska, and the Baltics—hence, the history o working evaluation or endocrine causes o osteoporosis is normal.
as a park ranger). Although a strict vegetarian diet can cause
B12 de ciency, there are suf cient stores to last 3 to 5 years (in All o the ollowing are necessary or the diagnosis o this
other words, more than 1 month). patient EXCEP :
A) Bone scan.
B) Serum and urine protein electrophoresis with immuno xa-
QUICK QUIZ: SAY, DID HE SWALLOW A PENNY? tion.
C) Bone marrow aspirate and biopsy.
A 21-year-old college student is brought to the ED by his D) Quantitative immunoglobulins.
riends because o bizarre behavior. His riends state that he E) Skeletal survey.
has been acting “a little odd” lately, but they are unaware o
any drug abuse. He does drink alcohol but is not known as The correct answer is “A.” A spontaneous vertebral racture in
a binge drinker—at least not by modern NCAA Division I a 50-year-old male is de nitely not normal. Multiple myeloma
Drinking standards. His riends do not know his medical his- should be considered in patients with this presentation. Mul-
tory but state he was taking a prescription drug or a “meta- tiple myeloma is a clonal disorder o plasma cells. Risk actors
bolic disorder,” which he stopped several months ago. He is include A rican-American race, male sex, and advancing age
uncooperative, paranoid, and disoriented. You note brownish (median age o 60–65 at presentation). Work-up or the diag-
pigmentation o his corneas on physical examination. T is nosis o multiple myeloma requires a serum and urine protein
may be the rst—and last—case o Wilson disease you have electrophoresis with immuno xation (“B”). T is will help to
ever seen. identi y a monoclonal protein. Quantitative immunoglobulins
(“D”) will help to assess whether this patient has an elevation
Regarding this patient, which o the ollowing is INCORREC ? o immunoglobulins in the range required or the diagnosis o
A) T e patient has likely had episodes o hemolytic anemia in multiple myeloma. A skeletal survey (“E”) and bone marrow
the past. biopsy (“C”) complete the diagnostic work-up. A bone scan is
B) T e patient has likely discontinued penicillamine. not use ul because the lytic lesions characteristic o multiple
C) T e patient likely has an elevated ceruloplasmin level on myeloma do not take up radioisotope.
blood test.
D) T e patient likely has hepatolenticular degeneration.
E) T e patient is at risk or hepatic ailure.
CASE 6.16
On a routine insurance physical examination, a 55-year-old
The correct answer is “C.” T is patient likely has Wilson disease man was ound to have a total protein o 9 g/dL (elevated).
(an autosomal-recessive de ect in cellular copper export), which T e remainder o his serum chemistries and his CBC were
presents with decreased levels o ceruloplasmin, increased liver normal. He is active, eels well, and is not taking any medi-
enzymes, and signs o hemolysis ( rom the direct toxic e ect o cations. His physical examination is unremarkable. You
copper on the cell). Symptoms typically present in the teens and order a serum and urine electrophoresis with immuno xa-
early 20s. Presenting symptoms may include Kayser–Fleischer tion and quantitative immunoglobulins. He has a monoclo-
rings (golden-brown pigmentation o the cornea), hemolytic nal spike with 1,400 g/dL o IgG kappa (elevated). His other
anemia, or neurologic symptoms, o en mimicking psychiatric immunoglobulins are within normal limits. A skeletal survey
CHAPTER 6 • HEMATOLOGY AND ONCOLOGY 189

demonstrates no lytic lesions, and his bone marrow aspirate Question 6.17.1 Which o the ollowing statements is
and biopsy demonstrate 6% plasma cells (normal is 1–4%). INCORRECT?
A er the bone marrow biopsy, the hematologist sends him A) T e small number o blasts in the bone marrow indicates a
back to you or ollow-up. good prognosis.
B) T e patient cannot have MDS because she has pancytopenia.
Question 6.16.1 What is your next step in the management C) T e patient may bene t rom administration o erythropoi-
o this patient? etin.
A) Monitor blood, including serum immunoglobulins, every 3 D) T e patient may develop acute myeloid leukemia.
to 6 months and, i stable a er 1 year, annually therea er. E) T e patient’s gender may result in a better prognosis.
B) Monitor blood, including serum immunoglobulins, every
4 weeks inde nitely. Answer 6.17.1 The correct answer is “B.” Patients with MDS
C) Start on chemotherapy or multiple myeloma. can be pancytopenic. MDS includes a number o clonal stem
D) Obtain yearly bone marrow biopsy and skeletal survey. cell disorders characterized by dysplasia and ine ective hema-
E) No ollow-up is necessary. topoiesis o one or more cell lines. T e disease is typically one
o older adults, with a median age o 65 to 70 years at onset;
Answer 6.16.1 The correct answer is “A.” T is patient has however, a better prognosis is associated with age < 60 years
monoclonal gammopathy o undetermined signi cance, or and emale gender. T ere is an increased risk o MDS in smok-
MGUS, which is ound in up to 3% o asymptomatic older (over ers, those exposed to benzene or alkylating agents, and with
50 years old) individuals (mostly Caucasian). A bone marrow some hereditary disorders. Prognosis depends on the num-
biopsy indicating 10% plasma cells is required to make the diag- ber o blasts, the number o lineages a ected, and cytogenetic
nosis o multiple myeloma; this patient only has 6%. In addi- abnormalities. Median survival ranges rom 10 to 66 months,
tion, this patient has a normal CBC and is asymptomatic (no and progression to acute leukemia ranges rom 6% to 33% o
atigue, bone pain suggestive o lesions, etc.). T us, this patient patients, depending on the subtype o MDS.
has MGUS. Patients with MGUS typically do well, with only 1% Depending on the age and overall per ormance status o the
per year progressing to multiple myeloma. individual MDS patient, treatment can range rom supportive
While patients should be reassured o the typically benign care with blood or platelet trans usions, to treatment with gran-
nature o this condition, up to 30% will have complications ulocyte colony stimulating actor (GCSF), to chemotherapeutics
(multiple myeloma, amyloidosis, other myeloproli erative such as azacytidine, or even to bone marrow transplantation.
disorders). A er several unchanged immunoglobulin levels
Objectives: Did you learn to . . .
and normal CBCs within the 6 months o the initial diagno-
• Recognize the presentation o multiple myeloma?
sis, annual or biannual evaluation should be adequate. Any
• Identi y MGUS?
changes in the patient’s condition, such as unexplained anemia,
• Identi y pancytopenia as a presentation o MDS?
increased immunoglobulin levels, renal insuf ciency, or bony
pain, should prompt urther evaluation. An increase in immu-
noglobulin does not necessarily mean the monoclonal protein
is increasing, and a serum protein electrophoresis and immu-
QUICK QUIZ: IT’S JUST KIDS’STUFF
no xation should be obtained. Repeat bone marrow biopsy is
indicated only i the clinical picture is con using. All o the ollowing are accurate statements about neutrophil
counts in children EXCEP :
A) A rican-American children may have a normal ANC o
CASE 6.17 1,000/mm 3.
A 63-year-old woman presents to your o ce with complaint B) Children over the age o 6 should be expected to have a nor-
o atigue. She states she elt like she spent the winter tak- mal ANC o 1,500 to 8,000/mm 3.
ing antibiotics because she developed “one in ection a er the C) Neonates typically have a normal ANC o <500/mm 3 at birth.
other.” Prior to this past winter, she had been well and did not D) In ants typically have 20% to 30% neutrophils on WBC di -
take any prescription drugs regularly. Physical examination erential.
demonstrates a slightly pale, thin woman. Her temperature E) Five-year-old children have 50% neutrophils on WBC di -
is 37.5°C, pulse 90 bpm, BP 120/58 mm Hg, and respirations erential.
12 bpm. Her oropharynx demonstrates purpuric lesions. Her
abdominal examination shows no organomegaly, and the The correct answer is “C.” In the pediatric population, neutro-
remainder o the physical examination is unremarkable. penia is typically described as an ANC o < 1,500/mm 3. Up to
You obtain blood tests: WBC 2,100/mm3, Hgb 8.4 g/dL, 30% o A rican-American children have an asymptomatic ANC
and platelet count 20,000/mm3. A bone marrow aspirate and o < 1,000/mm 3 (e.g., no increased risk o in ection). At birth,
biopsy are obtained, which demonstrate a hypercellular mar- neutrophils make up the majority o the WBC di erential, and
row with 4%blasts, 4%ringed sideroblasts, and megakaryo- this decreases to 20% to 30% a er the rst ew days o li e. At
cytes. You talk to the hematologist, who suspects that your 5 years o age, neutrophils comprise approximately 50% o the
patient has a myelodysplastic syndrome (MDS). di erential, and this reaches 70% by puberty.
190 FAMILY MEDICINE EXAMINATION & BOARD REVIEW

(10% o what is ingested), which is precisely balanced with loss


CASE 6.18 through sweat, sloughing o cells, and GI losses. In hereditary
A 49-year-old male presents to your o ce complaining o hemochromatosis, 2 to 4 mg o iron is absorbed daily, resulting
joint pain, atigue, and increased urination. T ere is a am- in the accumulation o iron.
ily history o cirrhosis o the liver, apparently not related
to alcohol. He does not take any medications and does not T e patient’s diagnosis is con rmed, and you counsel him
smoke but drinks two glasses o wine each night. Physical regarding his disease.
examination reveals a thin male with tanned skin (in the
dead o winter in Iowa, so he is either rich or sick, take your Question 6.18.3 All o the ollowing statements are true
choice) and normal vital signs. His heart and lung examina- EXCEPT:
tions are unremarkable. T e abdomen is so , nontender, and A) T is is an autosomal-dominant disease.
nondistended, and his liver edge is palpable 2 cm below the B) A ected emales typically present later in li e due to
costal margin. He has pain with range o motion in his hips, increased iron losses.
knees, and M P joints. T e remainder o the examination is C) Patients may bene t rom phlebotomy even a er develop-
unrevealing. Serum electrolytes are normal, but his glucose is ment o symptomatic disease.
282 g/dL. His transaminases are elevated. D) Patients are susceptible to unusual in ections.
E) Patients are at high risk or hepatocellular carcinoma.
Question 6.18.1 O the ollowing, what is the most likely
diagnosis? Answer 6.18.3 The correct answer is “A.” Hereditary hemo-
A) Alcoholic hepatitis. chromatosis is an autosomal-recessive disorder. Approxi-
B) Colon cancer. mately 10% o Caucasians are heterozygous, and 5/1,000 are
C) Hemochromatosis. homozygous or the gene mutation. Patients are at increased
D) Vitamin B12 de ciency. risk or in ections due to Listeria, Vibrio vulnif cus (sorry, no
sushi!), and Yersinia enterocolitica. A ected emales may pres-
Answer 6.18.1 The correct answer is “C.” T is patient pres- ent later in li e due to increased iron losses rom menstrua-
ents with a “classic” history o hemochromatosis. Symptoms tion, pregnancy, and lactation.
start during or a er the h decade o li e, are initially mild, reatment includes phlebotomy or chelation therapy i phle-
and progress slowly. “Bronze diabetes” (bronze or tan skin color botomy is contraindicated. Patients should be monitored closely
due to iron deposition accompanied by hyperglycemia) is some- or development o hepatoma, since therapy does not reduce the
times noted. Most organs can eventually be involved, but most risk o hepatocellular carcinoma once cirrhosis is present. Fam-
commonly symptoms are due to liver, cardiac, joint, and testicu- ily members should be screened or the disease so they can be
lar involvement. “A,” alcoholic hepatitis, is a consideration, but started on therapy early, in order to prevent the development o
his reported use is unlikely to result in disease. You also would cirrhosis.
not expect to see some o the other symptoms (e.g., diabetes and
arthritis) in relation to alcoholic hepatitis. “B,” colon cancer, is
unlikely to present in this way. “D,” vitamin B12 de ciency, typi- HELPFUL TIP:
cally presents with anemia and neurologic symptoms. Hemochromatosis is a genetic disease; there ore, iden-
ti ying a de ect in the HFE gene makes the de initive di-
Question 6.18.2 Which o the ollowing laboratory values agnosis. Patients may have high erritin levels or many
are most consistent with this patient’s presentation? reasons, including many causes o in lammation and
A) Decreased iron, decreased erritin, decreased trans errin liver disease. To con irm the diagnosis, test or the two
saturation. genetic de ects known to cause the disease: C282Yand
B) Decreased iron, decreased erritin, increased trans errin H63D. In persons o northern European descent, het-
saturation. erozygosity is remarkably prevalent with about 10%
C) Increased iron, decreased erritin, increased trans errin o these patients carrying C282Y and 15% to 20% car-
saturation. rying H63D. However, the disease is autosomal reces-
D) Increased iron, increased erritin, decreased trans errin sat- sive and will only mani est with two copies o C282Y
uration. or one copy o each (C282Y/H63D). H63D alone (and
E) Increased iron, increased erritin, increased trans errin other mutations) appear less deleterious compared
saturation. to C282Y.

Answer 6.18.2 The correct answer is “E.” All o these iron


studies (serum iron level, erritin and trans errin saturation) HELPFUL TIP:
are elevated in patients with hereditary hemochromatosis. Other causes o iron overload include thalassemia, sid-
A normal person maintains approximately 3 to 4 g o iron in eroblastic anemia, and requent blood trans usions.
the body. Normal individuals absorb about 1 mg/day o iron
CHAPTER 6 • HEMATOLOGY AND ONCOLOGY 191

CASE 6.19 HELPFUL TIP:


Children with diarrhea may develop methemoglobin-
A 56-year-old man undergoes a laryngoscopy. In an out-
emia, although it is rarely clinically signi icant.
dated move that is generally not recommended now, his
doctor used extra doses o benzocaine spray (like gallons o
the stu ) to overcome a strong gag ref ex. T e patient tol-
erated the procedure well, but a erward he started to have HELPFUL (AND SOMEWHAT USEFUL) TIP:
cyanosis around the lips. You are now seeing him in the ED, Methylene blue, as a potent inhibitor o monoamine
and he has a bluish discoloration o his lips and ngertips oxidase, can also cause a serotonin syndrome i used in
and is complaining o a headache. You administer oxygen those on SSRIs or other serotoninergic drugs.
by nasal cannula with no improvement in his cyanosis. You
draw venous blood or labs, and nd that it has an unusual
chocolate color.
CASE 6.20
Question 6.19.1 All o the ollowing statements are likely to
A 16-year-old sub-Saharan A rican male presents to your
be true about this patient EXCEPT:
o ice or progressive atigue and dyspnea over the last
A) A measured arterial blood gas will demonstrate a normal
ew days. He also has some mild upper respiratory symp-
PaO2.
toms that are being “treated” with sul amethoxazole—tri-
B) T e pulse oximeter will show high-normal oxygen satura-
methoprim. ( he doctor down the street is known to treat
tion.
all respiratory in ections with antibiotics.) On examina-
C) T e patient may require therapy with methylene blue.
tion, you ind a well-nourished male in no distress. He
D) T e patient should not be treated with methylene blue i he
is a ebrile but slightly tachycardic. You note mild scleral
has a history o G6PD de ciency.
icterus and pallor o the palmar creases, but the remain-
E) Additional doses o benzocaine can be lethal.
der o the examination is unremarkable. A CBC shows
normal WBC count and platelets, with Hgb 10.2 g/dL.
Answer 6.19.1 The correct answer is “B.” T is patient has On peripheral smear, “bite cells” and rare Heinz bodies
methemoglobinemia, which in his case was likely a result o are reported. he LDH and bilirubin are elevated and the
the benzocaine spray. Methemoglobinemia results when iron serum haptoglobin is low, but the other serum chemistries
in Hgb is oxidized rom the errous to the erric state and the are normal.
Hgb becomes incapable o binding and transporting oxygen.
T e blood gas oxygen may look normal because the RBCs can- Question 6.20.1 Which o the ollowing is the most likely
not release oxygen (and thus remain oxygenated). However, the diagnosis?
pulse oximeter will show a low O2 saturation. So, there is o en A) Hereditary spherocytosis.
a gap between what you see on the blood gas and what the B) G6PD de ciency.
pulse oximeter suggests. C) Sickle-cell disease (homozygous).
In a normal patient, less than 1% o Hgb is ound in the met- D) Sickle-cell trait (heterozygous).
hemoglobin orm. Depending on the percentage o methemo- E) Iron de ciency anemia.
globin, presentations will vary. At methemoglobin levels o 10%
to 20%, patients present with cyanosis re ractory to oxygen. T e Answer 6.20.1 The correct answer is “B.” T e most likely
arterial blood gas may show a normal PaO2 with low oxygen cause o this patient’s anemia is G6PD de ciency. He appears
saturations. When methemoglobin levels are > 30%, patients to have a hemolytic anemia, as evidenced by the elevated LDH,
present with headache, dizziness, dyspnea, and tachypnea (signs bite cells, elevated bilirubin, low haptoglobin, and the acute
o moderate hypoxemia). At > 50%, patients develop stupor and onset o symptoms. Heinz bodies are inclusions in red cells
obtundation (signs o severe hypoxemia), and levels > 70% may seen on peripheral smear within the rst ew days o an oxi-
be lethal. dative stress in patients with G6PD de ciency. Bite cells are
Certain drugs such as nitroprusside, sul onamides, some ormed as the red cells pass through the spleen and the Heinz
local anesthetics, and acetaminophen have been ound to cause bodies are removed. T e other diagnoses are less likely. Heredi-
methemoglobinemia. tary spherocytosis (“A”) is caused by an inherited de ect in the
reatment o methemoglobinemia is methylene blue. How- red cell membrane, and episodes o hemolytic anemia may be
ever, patients with G6PD de ciency should not be given methy- brought on by environmental stress. However, patients gener-
lene blue, and hyperbaric oxygen therapy should be considered ally have a baseline anemia, and spherocytes should be seen
instead. Remember, when patients have been exposed to cyanide, on peripheral smear. Sickle-cell disease and trait are discussed
you want to cause methemoglobinemia. T e exact mechanism in more detail elsewhere. “E,” iron de ciency anemia, does not
by which this protects against cyanide is unknown, since patients generally present acutely and is not associated with ndings o
begin to improve prior to the presence o methemoglobinemia. hemolysis.
192 FAMILY MEDICINE EXAMINATION & BOARD REVIEW

Objectives: Did you learn to . . .


HELPFUL TIP:
• Describe symptoms and signs o hemochromatosis?
G6PD de iciency is the most common inherited RBC
• Recognize complications related to iron overload?
enzyme de ect, a ecting 10% o the black male popula-
tion. It occurs more commonly in A rican and Mediter- • Describe ndings and treatment o methemoglobinemia?
ranean populations. • Identi y clinical and laboratory mani estations o G6PD
de ciency and describe its management?

Hemolytic anemia in G6PD de ciency is caused by oxidative


stress on red cells, most commonly as the result o in ection QUICK QUIZ: TOO MANY LYMPHOCYTES
or administration o certain drugs.
A 72-year-old man presents to your of ce or ollow-up a er a
Question 6.20.2 Which o the ollowing can precipitate a recent hospitalization or pneumonia. While he was in the hos-
hemolytic crisis in G6PD def ciency? pital, he had an increased WBC with lymphocytosis. You repeat
A) Sul a antibiotics. his CBC and nd a WBC o 18,000/mm 3, with 80% lympho-
B) Fresh ava beans. cytes, Hgb o 14 g/dL, and a platelet count o 200,000/mm 3. You
C) Nitro urantoin. review of ce records rom a visit 1 year ago and nd a WBC
D) Some antimalarial drugs. o 14,000/mm 3 with 76% lymphocytes. You send his peripheral
E) All o the above. blood or ow cytometry, which is consistent with a diagnosis o
chronic lymphocytic leukemia (CLL).
Answer 6.20.2 The correct answer is “E.” All o the above can
cause a hemolytic crisis in G6PD de ciency. Multiple other Which o the ollowing statements about this patient is
drugs can be involved as well, including vitamin C, salicylates, INCORREC ?
isoniazid, and phenytoin. A) T is patient is at risk or the development o hemolytic
anemia.
HELPFUL TIP: B) T is patient is at risk or serious in ection.
Fresh ava beans aren’t worth the bother. First you have C) T is patient is likely to experience a relatively benign disease
to shell them and then peel each and every bean. A ter all course.
that work, you just end up with a hemolytic crisis i you D) T is patient should be started on chemotherapy.
have G6PD de iciency. Get the canned ones. E) T is patient may develop “B symptoms” ( ever, night sweats,
weight loss).
Question 6.20.3 Which o the ollowing is the most appro-
priate intervention or this patient with G6PD def ciency at The correct answer is “D.” CLL is characterized by progres-
this point? sive accumulation o long-lived lymphocytes. It is more com-
A) Admit to the hospital and observe. mon in men and tends to be a disease o older adults (median
B) Admit to the hospital and trans use 2 units o packed red age at diagnosis is 65 years); however, up to 20% o cases occur
cells. in patients < 60 years o age. Diagnosis is made i a patient has
C) Recommend supportive care and ollow-up in a ew days. > 5,000/mm 3 mature-appearing lymphocytes in the blood rep-
D) Recommend splenectomy and re er to a general surgeon. resenting a clonal line. With the use o immunophenotyping,
bone marrow biopsy is not necessary or diagnosis, but may be
Answer 6.20.3 The correct answer is “C.” T e hemolytic ane- use ul or prognosis.
mia o G6PD de ciency is sel -limited and will resolve. T is is T is patient has a low-stage CLL (the only nding is lym-
because in most cases o G6PD de ciency, only about 25% o phocytosis) and does not require immediate treatment with
the RBCs (the older cells) are susceptible to oxidative stress. chemotherapy. He should be monitored every 6 to 12 months or
Severe episodes should be treated in the hospital setting, but sooner i he develops symptoms, such as in ections, atigue, bulky
most episodes can be managed as an outpatient. Patients should lymphadenopathy, or bleeding. Patients with CLL are at risk or
be educated on the drugs and stressors that may precipitate an developing autoimmune diseases, including hemolytic anemia,
episode o hemolytic anemia. Splenectomy may limit hemolysis autoimmune thrombocytopenia, pure red cell aplasia, and auto-
in patients with more severe disease. immune neutropenia.

HELPFUL TIP:
QUICK QUIZ: TOO MANY NEUTROPHILS
I you check a G6PD level during or just a ter a hemolytic
crisis it will likely be normal even in those with G6PD
de iciency. The cells that have low levels o G6PD have A 46-year-old woman presents to your of ce or a preoperative
hemolyzed. The younger cells that are le t (those that evaluation. She is planning an elective hysterectomy or uterine
are tested) have normal levels o G6PD. broids. She has no signi cant past medical history. Her review
o systems is positive only or sweats. Her physical examination
CHAPTER 6 • HEMATOLOGY AND ONCOLOGY 193

is unremarkable except or a palpable spleen tip. You obtain a o a concurrent in ection. hese patients should be kept well
CBC, showing a WBC 50,000/mm 3, Hgb 11 g/dL, and a plate- hydrated and given allopurinol to prevent tumor lysis syn-
let count 350,000/mm 3. On her peripheral blood smear, she drome, which can occur even without chemotherapy i the
has mostly neutrophils and bands, with some metamyelocytes, tumor burden is high enough. reatment o acute leukemias
myelocytes, and basophils as well. requires intensive chemotherapy and should be started as
soon as possible, so prompt evaluation by a hematologist/
All o the ollowing are likely to be true EXCEP : oncologist is imperative.
A) T e patient has an underlying in ection with leukemoid
reaction.
B) T e patient has a balanced translocation between chromo-
CASE 6.21
somes 9 and 22. A 65-year-old male who is on quinine or leg cramps (a non–
C) T e patient will likely develop progressive leukocytosis, FDA-approved indication) is brought into the ED. Last night
evers, anemia, and thrombocytopenia i untreated. he was complaining o a headache, ever, and numbness on
D) T is condition may be treated with oral tyrosine kinase in- the right side o his ace. T is morning, he was acting errati-
hibitors. cally.
His physical examination demonstrates a con used male,
The correct answer is “A.” Although a leukemoid reaction is a with a temperature o 39°C, pulse 110 bpm, BP 180/94 mm Hg,
possibility, it would be unusual to see a ull range o maturation and respirations 14 bpm. He has a ew petechiae on his palate.
o myeloid cells (e.g., metamyelocytes and myelocytes) in the He is tachycardic and lungs are clear to auscultation bilater-
peripheral blood along with increased basophils and a palpa- ally. His abdomen is so , without organomegaly. His neuro-
ble spleen. Rather, this patient likely has chronic myelogenous logic examination is di cult to complete due to his inability
leukemia (CML), which is a clonal myeloproli erative disorder, to cooperate.
characterized by the “Philadelphia chromosome” (translocation Laboratory ndings include WBC 6,000/mm3, Hgb
between chromosomes 9 and 22), which encodes or an abnor- 8.4 g/dL, and platelet count 50,000/mm3. Schistocytes are
mal tyrosine kinase protein. T is protein is the target or the noted on a peripheral blood smear.
tyrosine kinase inhibitors, such as imatinib (Gleevec), which
have resulted in signi cant improvement in survival. Patients
Question 6.21.1 All o the ollowing may be expected in
with CML typically present between the ages o 40 and 60. Up to
this patient EXCEPT:
40% o patients are asymptomatic at presentation. Others com-
A) Creatinine o 3.2 mg/dL.
plain o weight loss, atigue, abdominal pain, night sweats, and
B) LDH o 640 IU/L.
ever. Five-year overall survival can be up to 85% depending on
C) Elevated haptoglobin.
the age at diagnosis.
D) Elevated indirect bilirubin.
E) Negative direct antiglobulin test (direct Coombs).
QUICK QUIZ: ACUTE LEUKEMIA
Answer 6.21.1 The correct answer is “C.” An elevated hap-
You have a ebrile 37-year-old male with a very high WBC toglobin would suggest the absence o active hemolysis. T e
count, most o which are blasts. Most likely, he has an acute leu- presence o schistocytes suggests ongoing microangiopathic
kemia. A bone marrow has not yet been done, and it is unknown hemolysis, in this case due to thrombotic thrombocytope-
i this is an acute myeloid leukemia or an acute lymphoblastic nic purpura ( P). P is due to the lack o activity o the
leukemia. ADAM S13 enzyme either via inheritance o a aulty enzyme
(inherited) or via antibodies produced against the enzyme
Which o the ollowing statements is INCORREC ? (acquired). It is classically described by a pentad o ndings:
A) T e patient’s ever is likely due to the leukemic cells, and microangiopathic hemolytic anemia (suggested by schisto-
antibiotics should be started only i you identi y a speci c cytes on peripheral smear), thrombocytopenia, ever, renal
in ection. insuf ciency, and mental status changes. Not all ve eatures
B) T e patient is at risk or tumor lysis syndrome and should be need to be present or the diagnosis, but your patient seems to
started on allopurinol. have the pentad. An otherwise unexplained microangiopathic
C) Aggressive inpatient chemotherapy is required or both the hemolytic anemia in the presence o thrombocytopenia is
treatment o acute myeloid leukemia and acute lymphoblas- enough to trigger treatment or P; ever, renal insuf ciency
tic leukemia. and mental status changes are not necessary to make the pre-
D) You should consult a hematologist/oncologist as soon as sumptive diagnosis o P. T e negative direct antiglobulin
possible. test (direct Coombs), “E,” suggests that the hemolytic anemia
is not due to an autoimmune process. Note that the patient is
The correct answer is “A.” Patients with acute leukemia on quinine, one o the drugs that can cause P. Clopidogrel
and ever should be cultured, and antibiotics should be and ticlopidine can also cause P although it is less common
started as soon as possible since a ever is o ten the result with clopidogrel.
194 FAMILY MEDICINE EXAMINATION & BOARD REVIEW

has no other signi cant medical history. A er the delivery,


HELPFUL TIP:
you note that she is having brisk vaginal bleeding and ooz-
Schistocytes are the result o intravascular trauma to
ing rom her venipuncture sites. You obtain laboratories:
RBCs. Any microangiopathic hemolytic anemia can re-
WBC 9,000/mm3, Hgb 8.2 g/dL, platelet count 80,000/mm3,
sult in schistocytes as can other intravascular trauma
INR 2.4, P 22 seconds, and brinogen 80 mg/dL (normal
such as that secondary to arti icial heart valves, HELLP
190–420 mg/dL). Fibrin degradation products are elevated. A
syndrome, malignant hypertension, some malignan-
CBC at admission was normal.
cies, transjugular intrahepatic portosystemic shunts
(TIPS), eclampsia, etc.
Question 6.22.1 Which o the ollowing statements is
INCORRECT?
Question 6.21.2 Which o the ollowing laboratory f ndings A) T e patient may bene t rom platelet trans usion.
would you expect to see in this patient with TTP? B) T e patient may bene t rom cryoprecipitate trans usion.
A) Normal P /INR. C) T e patient may bene t rom FFP trans usion.
B) Elevated P . D) T e patient may bene t rom crystalloid in usion.
C) Elevated brin degradation products.
D) Low level o brinogen. Answer 6.22.1 The correct answer (and what not to do) is
“A.” T is patient is exhibiting signs o disseminated intravascu-
Answer 6.21.2 The correct answer is “A.” Patients with P lar coagulation (DIC), which is characterized by the disordered
should have a normal P /INR. T is is important because P regulation o coagulation. It can be precipitated by a sepsis,
can be con used with DIC, in which the P /INR should be ele- malignancy, CNS trauma with release o tissue plasminogen
vated and brinogen should be low. activator, obstetrical problems (preeclampsia, retained products
o conception, amniotic uid embolism), ABO incompatibility
Your patient is started on plasma exchange. You note that he with hemolysis, extensive burns, snake bites, etc. T rombin also
has not had any change in his Hgb or platelet count, and his activates platelets, causing platelet aggregation and consump-
renal unction is worsening. tion. Antithrombin, previously known as antithrombin III, is
consumed in this process as well. reatment is directed at cor-
Question 6.21.3 Which o the ollowing may mimic TTP and recting the underlying cause o DIC whenever possible. Don’t
should be considered in a poorly responsive patient? treat the numbers; patients may have a low-grade DIC and be
A) Rocky Mountain spotted ever. relatively asymptomatic. I this is the case, reversing the underly-
B) Disseminated aspergillosis. ing process is enough. I the patient is bleeding, she may ben-
C) Disseminated malignancy. e t rom platelet trans usion to maintain a platelet count above
D) Malignant hypertension. 20,000/mm 3, but trans usion at higher levels can worsen platelet
E) All o the above. aggregation. rans usion o FFP will replace consumed actors.
Patients with low brinogen levels will bene t rom trans usion
Answer 6.21.3 The correct answer is “E.” When patients are o cryoprecipitate. T e use o heparin is generally limited to
poorly responsive to therapy, an alternative diagnosis should be those with low grade, chronic DIC with thrombotic complica-
sought. In pregnant patients, preeclampsia or HELLP syndrome tions. It may also be considered acutely i there are prominent
can mimic P. Autoimmune disease (e.g., systemic lupus ery- thrombotic complications.
thematosus and scleroderma), malignant hypertension, and dis-
seminated malignancy may also mimic P. In ections that can
be con used with P include Rocky Mountain spotted ever, HELPFUL TIP:
disseminated aspergillosis, as well as any other disease that can Laboratory indings in DIC include low platelets, pro-
cause mental status changes and a low platelet count such as longed PT/INR, normal or prolonged PTT, low ibrino-
Ehrlichiosis, Anaplasmosis, West Nile virus, etc. gen, and increased ibrin degradation products. Fur-
ther, D-dimer and thrombin time are also increased,
antithrombin may be low, and peripheral blood smear
HELPFUL TIP:
will reveal schistocytes. An elevated D-dimer in isolation
Therapy with plasma exchange has reduced the mortal-
is not help ul since D-dimer is elevated in many states,
ity rate or TTP rom 90% to less than 25%. Other treat-
including DVT/PE.
ments may include glucocorticoids and splenectomy in
re ractory cases. Platelet trans usion should be avoided
as it can worsen the patient’s condition. Objectives: Did you learn to . . .
• Recognize the clinical presentation o TTP?
• Treat a patient with TTP?
CASE 6.22 • Develop a dif erential diagnosis or a patient with microan-
A 23-year-old G1 P0 term emale goes to emergency cesarean giopathic anemia?
section a er she is noted to have placental abruption. She • Distinguish DIC rom TTP?
CHAPTER 6 • HEMATOLOGY AND ONCOLOGY 195

CASE 6.23 HELPFUL TIP:


Non-Hodgkin lymphoma (NHL) is increasing in inci-
A 22-year-old emale college student presents to your o ce dence and is more common than Hodgkin lymphoma.
with complaints o an enlarged cervical lymph node and Risk actors or NHL include HIV in ection and advanc-
sore throat. She has elt everish on and o or several weeks ing age.
but never measured her temperature. She is a ebrile today, a
rapid strep test is negative, and there is no evidence o lymph-
adenitis; you treat her symptomatically. You ask her to return
HELPFUL TIP:
i her symptoms do not resolve within 2 to 4 weeks.
The di erential diagnosis o generalized lymphade-
She returns 4 weeks later. T e sore throat has resolved;
nopathy is broad and a good history is imperative. The
however, she now complains o intense pruritus and sweats
initial evaluation should include a CBC and CXR. I these
at night. T e cervical lymph node now measures 3 cm. T e
are negative and lymphadenopathy persists, consider
remainder o her physical examination is negative.
obtaining an RPR and ANA and testing or TB and HIV.
Question 6.23.1 What is the most appropriate next step? Also, consider testing or Epstein–Barr virus (antibody
A) Direct laryngoscopy. test or heterophile) as appropriate. Some medications
B) Empiric antibiotic trial. can cause generalized lymphadenopathy secondary to
C) Lymph node biopsy. serum sickness.
D) MRI o the neck.
E) Observation or another 4 weeks. Objectives: Did you learn to . . .
• Recognize presentation o a patient with Hodgkin lympho-
Answer 6.23.1 The correct answer is “C.” T e prolonged ma?
(> 1 month) presence o a large (> 1 cm) lymph node deserves • Describe “B symptoms?”
a biopsy. MRI o the neck and laryngoscopy may be indicated
depending on the biopsy results. Empiric antibiotic therapy CASE 6.24
(“B”) is inappropriate and unlikely to be help ul, as you have no
source o primary in ection. At this point in time, observation A 34-year-old woman presents to your o ce with cal pain
(“E”) will just delay the diagnosis. Note that delaying the biopsy and swelling. She denies trauma to the leg. She has no other
or 1 month a er initial presentation does not change outcomes signi cant medical history and takes only oral contracep-
even i the node is malignant. tives. She smokes hal a pack o cigarettes daily but does not
drink alcohol. Her physical examination is also unremark-
You obtain a lymph node biopsy that is consistent with nodu- able, with the exception o swelling and tenderness o the le
lar sclerosing Hodgkin lymphoma. cal . A lower extremity Doppler study demonstrates a new
thrombus occluding the common emoral vein.
Question 6.23.2 Which o the ollowing statements is
INCORRECT? Question 6.24.1 Which o the ollowing would NOT be
A) T e patient has B symptoms. appropriate in this patient?
B) Further staging is necessary. A) Start the patient on heparin or a heparinoid and war arin to
C) Chemotherapy is palliative only in this patient. maintain an INR o 2 to 3 or start a actor Xa inhibitor (e.g.,
D) reatment includes both radiation and chemotherapy. rivaroxaban) or a direct brin inhibitor (e.g., dabigatran).
B) Have the patient scheduled or placement o an in erior vena
Answer 6.23.2 The correct answer is “C.” Chemotherapy cava lter.
regimens or Hodgkin lymphoma have resulted in response C) Discontinue hormonal contraceptives.
rates o more than 90%, with ewer long-term complications D) Encourage the patient to stop smoking.
than prior regimens. Despite the act that it is well-known, E) Obtain urther history regarding her amily.
Hodgkin lymphoma is an uncommon disease, with about
8,000 cases occurring each year in the United States. It is a dis- Answer 6.24.1 The correct answer is “B.” T is patient has
ease o young people, occurring typically in patients in their an acute DV and risk actors or developing a DV , includ-
20s. Patients o en present with lymphadenopathy. Symp- ing smoking and hormonal contraceptive use. You should start
toms, known as “B symptoms,” include weight loss, ever, and her on heparin (or low–molecular-weight heparin) and war a-
drenching night sweats. T e presence o B symptoms correlates rin with a goal INR o 2 to 3. Heparin should be used or at
with more advanced disease. Other symptoms may include least 5 days and overlapped with therapeutic doses o war arin
pruritus, di use pain a er consuming alcoholic beverages and that have reached the target INR or at least 48 hours. Alterna-
symptoms related to the development o a mediastinal mass. tively you can start rivaroxaban (Xarelto), apixaban (Eliquis),
A er a diagnostic lymph node biopsy, urther staging (“C”) or dabigatran (Pradaxa). Dabigatran requires an overlap with
includes C scans, bone marrow biopsy, and routine labora- low–molecular-weight heparin; rivaroxaban and apixaban do
tory tests. not. Avoid edoxaban (Savaysa) as it is less e ective and more
196 FAMILY MEDICINE EXAMINATION & BOARD REVIEW

problematic when adjusting or renal disease. She should be C) Homocysteine.


anticoagulated or at least 3 months. Encourage her to stop D) Protein C.
smoking and to nd an alternative orm o birth control. “B” E) Prothrombin gene mutation.
deserves special mention. T ere is no evidence that vena caval
lters are o bene t in patients that can be anticoagulated except Answer 6.24.2 The correct answer is “D.” T ere are essen-
as a temporary measure or inpatients where the vena caval lter tially seven types o thrombophilia or which you might ini-
will be removed prior to discharge. T ere is no reduction in PE tially test: actor V Leiden mutation, antithrombin (previously
risk with vena caval lters in those who can be anticoagulated, antithrombin III) de ciency, prothrombin gene mutation, pro-
and there is likely an increased risk o DV . T e role in those tein C de ciency, protein S de ciency, antiphospholipid anti-
who cannot be anticoagulated is unclear; there is scant data. body syndrome, and hyperhomocysteinemia. Not all causes o
thrombophilia can be tested or while on anticoagulant therapy.
HELPFUL TIP: Factor V Leiden gene mutation, prothrombin gene mutation,
Homan sign is neither sensitive nor speci ic enough to homocysteine, and antiphospholipid antibodies are not a ected
allow it to be used to rule in or rule out DVT. Don’t both- by war arin. However, war arin can reduce protein C and S lev-
er. However, use o a clinical prediction tool (e.g., Well’s els (giving alse-positive test results) and increase antithrombin
criteria) can reduce unnecessary testing. levels (giving a alse-negative result).
Factor V Leiden mutation is the most common cause o
inherited thrombophilia, being ound in 3% to 6% o non-
HELPFUL TIP: Black blood donors. Antithrombin de ciency is less common,
All o the new anticoagulants (dabigatran, rivaroxaban, occurring in 1/1,000 to 1/5,000 individuals. T e homozygous
and apixaban) are essentially equivalent to war arin in condition is atal in utero. Heterozygotes have a 30% chance
terms o outcomes. There may be a bit less bleeding o developing a thromboembolic event by the age o 30 years.
but there is no overwhelming rationale to argue that Protein C de ciency occurs in 1/200 to 1/300 people; however,
the newer anticoagulants are superior to war arin. The ewer than 1/1,000 heterozygotes develop venous thromboses.
2016 Chest guidelines suggest the newer anticoagu- Protein S de ciency is estimated to occur in 1/750 to 1/3,000
lants with a grade 2B evidence (weak recommendation o persons and has a clinical presentation similar to protein C
based on the data). Certainly the new anticoagulants de ciency. T e prevalence o prothrombin gene mutation var-
are more convenient. Note that dabigatran cannot be ies widely and occurs most commonly in persons o southern
removed rom blister packs or bottles in order to put it European descent. Unlike the other thrombophilias discussed
in a pillbox or pill organizer. It quickly loses potency. thus ar, antiphospholipid antibody syndrome is an acquired
disorder that is associated with arterial as well as venous throm-
boembolic events. Finally, hyperhomocysteinemia is associated
HELPFUL TIP: with the development o venous thrombosis and atherosclerotic
Work-up o an inherited or acquired hypercoagu- heart disease.
lable state is controversial because the treatment o
the patient is o ten not going to change whatever the HELPFUL TIP:
outcome. The greatest risk or having another DVT is How long should I anticoagulate this patient? Glad that
having a irst DVT. When a thromboembolic event is un- you asked. For patients on estrogen or other modi iable
provoked, it is probably prudent to screen the patient actor with a irst DVT (s/p surgery, bedrest, pregnancy,
or hypercoagulable states. When there is a provoked etc.), 3 months is probably adequate. For others (cancer,
event (e.g., surgery, prolonged immobilization, preg- etc.) the recurrence rate is approximately 7% per year
nancy, and oral estrogen), the role o such a work-up is (Ann Intern Med. 2015;162:27). So, i the bleeding risk
less well de ined. is low, it is reasonable to continue anticoagulation in
these patients or li e (Chest Guidelines, 2016). Everyone
agrees that a second unprovoked DVT mandates treat-
T e patient returns 3 months later. She has been relatively easy
ment “ or li e.” Finally, there is no evidence that prophy-
to anticoagulate (somebody has to be, right?), maintaining a
lactic anticoagulation in those with cancer who have
stable INR. She stopped smoking and knows to avoid hormonal
not had a thromboembolic event is use ul.
contraception. She has learned that her mother had a DV and
her maternal aunt died o a pulmonary embolus. Now, the
patient wants to know how this new in ormation will a ect her HELPFUL TIP:
long-term care. You decide to screen her or thrombophilia. Should I stop oral anticoagulation or the proce-
dures? I I do should I bridge war arin patients with
Question 6.24.2 Which o the ollowing sets o tests should heparin? There is no need to stop anticoagulation or
NOT be per ormed while she remains on war arin? many procedures such as dental procedures, colonos-
A) Factor V Leiden mutation. copy, etc. But just try to get this past the dentist or
B) Antiphospholipid antibodies.
CHAPTER 6 • HEMATOLOGY AND ONCOLOGY 197

osteoporosis, and bleeding at the uteroplacental junction.


gastroenterologist. Importantly, bridging atrial fibril-
Dosing o un ractionated heparin is dif cult during pregnancy,
lation patients on warfarin with heparin increases
and low–molecular-weight heparin (e.g., enoxaparin) is prob-
the bleeding risk while doing nothing to prevent
ably the best choice or anticoagulation. Heparin and war arin
stroke, PE, etc. Be selective in whom you bridge.
are sa e or lactating mothers.
Bridge patients with a CH2ADS2-VASC score o > 5 (see
cardiology chapter or more on the CH2ADS2-VASC
score), those with a prosthetic mitral valve or metal aor-
CASE 6.26
tic valve, those with a history o thromboembolic events A 52-year-old woman presents to the ED with shortness o
while o anticoagulation, etc. For “normal” risk patients breath and pleuritic chest pain. She has been on war arin
bridging with heparin adds nothing and has signi icant or atrial brillation. Despite her anticoagulation, you sus-
downsides (see N Eng J Med. 2015 and Circulation. pect pulmonary embolism (PE) and obtain a spiral C scan,
2015;131:488–494). Note that it isn’t necessary to bridge which shows a thrombus in the right pulmonary artery.
patients who are on dabigatran, rivaroxaban, apixaban, Prior to the initiation o any therapy, her coagulation studies
etc. They are short acting enough that just holding one return and show INR o 2.8 (normal = 1) and P 49 seconds
dose prior to surgery is adequate; there is a minimal (prolonged).
window or clot ormation. See below or urther discus-
sion o heparin bridging in those who have had a recent Question 6.26.1 Which o the ollowing is the most appro-
thromboembolic event. priate next step?
A) est the patient or antiphospholipid antibodies.
B) Place an in erior vena cava lter.
C) Repeat the INR and P since the patient’s sample was
CASE 6.25 probably contaminated with heparin.
A 19-year-old gravid emale presents during her second tri- D) Con ront the patient about her noncompliance with war arin.
mester and complains o le cal swelling. With the help o E) T row up your hands in disgust. Aren’t they done with the
Doppler studies, you diagnose her with a DV . hematology chapter yet?

Question 6.25.1 Which o the ollowing statements is Answer 6.26.1 The correct answer is “A.” Patients who have a
correct? new V E while on appropriate anticoagulation therapy should
A) DV occurs most commonly in the right lower extremity in be evaluated or antiphospholipid antibody syndrome. T is
pregnant women. patient has a prolonged P , which is also suspicious or an
B) DV is common in pregnancy due to increased venous sta- antiphospholipid antibody. Remember that war arin prolongs
sis and increased levels o brinogen, actor VIII, and vWF. the P , not the P . She was an outpatient prior to having her
C) Anticoagulation with heparin introduces no risk to the etus blood drawn and was not taking heparin, so contamination
or mother. with heparin is unlikely (although heparin de nitely causes
D) DV is most common in the third trimester. an increase in the P ). T e increased INR suggests that the
patient has been compliant with her war arin. As to “B,” see the
Answer 6.25.1 The correct answer is “B.” DV , and the note above about vena caval lters above.
broader category o venous thromboembolism (V E), occurs
two to our times more o en in pregnant women compared to HELPFUL TIP:
nonpregnant controls. Increased risk is ound with cesarean Previously, retrospective studies resulted in the rec-
delivery versus vaginal delivery. T e majority o DV s occur in ommendation that patients with an antiphospholipid
the le lower extremity o pregnant women, likely due to the antibody and VTE be maintained at an INR o 3 to 4. This
compression o the le iliac vein by the right iliac artery as they recommendation has changed with newly available
cross. T e increased incidence o V E is multi actorial, includ- prospective data, and these patients should have a tar-
ing the presence o all three components o Virchow’s triad: get INR o 2 to 3. Some patients with antiphospholipid
venous stasis, endothelial injury (increased venous distension antibody syndrome may have marked INR luctuations
secondary to increased estrogen), and a hypercoagulable state that make monitoring anticoagulation di icult.
(increased levels o brinogen, actor VIII, and vWF).
V E occurs equally during all three trimesters, but the high-
est risk is postpartum (2–5 times the risk during pregnancy). HELPFUL TIP:
reatment o V E with anticoagulation is more dif cult during Genetic testing is now available or war arin sensitivity
pregnancy. War arin crosses the placenta and has teratogenic that can help predict which patients will need higher (or
e ects in addition to increased risk o etal bleeding. Heparin lower) doses o war arin to maintain adequate antico-
and danaparoid do not cross the placenta, but are associated agulation. However, these tests are very expensive and
with a 2% risk o maternal bleeding. Heparin also carries the contribute little to the care o most patients.
risks o heparin-induced thrombocytopenia with thrombosis,
198 FAMILY MEDICINE EXAMINATION & BOARD REVIEW

Pradaxa (dabigatran): A direct thrombin inhibitor.


CASE 6.27 Approved or nonvalvular atrial brillation or stroke preven-
A 50-year-old male is admitted to the hospital with a rac- tion. Contributes to less bleeding overall than war arin, but pre-
tured tibia a er a motor vehicle collision. You are asked to vents ewer myocardial in arctions and causes more GI bleeds.
assist in his perioperative management. T e patient is gener- T e dose must be decreased in those with renal disease.
ally healthy but is taking war arin or a DV that he devel- Xarelto (rivaroxaban): An oral actor Xa inhibitor (similar
oped a er a total knee arthroplasty 3 weeks ago. His INR is to enoxaparin). Indicated or the treatment o DV and PE,
1.8, and he is scheduled or open reduction/internal xation DV prophylaxis ollowing surgery and nonvalvular atrial
tomorrow. brillation and does not require heparin at initiation. T ere are
multiple drug interactions (though ewer than with war arin).
Question 6.27.1 Which o the ollowing statements is Not indicated in those with severe renal disease and those with
INCORRECT? moderate liver disease.
A) T e patient has an approximate 50% risk o recurrent V E Eliquis (apixaban): An oral direct actor Xa inhibitor. Indi-
without appropriate therapy. cated or the treatment o DV and PE, DV prophylaxis ol-
B) Heparin should be started preoperatively. lowing surgery and nonvalvular atrial brillation. Does not
C) Heparin should be continued postoperatively. require heparin at initiation. Not recommended or patients
D) Resumption o war arin alone postoperatively is adequate with moderate hepatic impairment. Dose must be decreased
anticoagulation. in renal impairment. May cause hypercoagulable state when
stopped, increasing the risk o stroke i stopped without ade-
Answer 6.27.1 The correct answer is “D.” Perioperative man- quate continuous anticoagulation.
agement o a patient with recent V E is complicated. While Annexa (Andexanet al a, Portola Phamaceuticals): A
there are established guidelines or V E prophylaxis depend- new recombinant modi ed actor Xa to work as an antidote
ing on the type o surgery planned, this patient already has an or patients receiving Xa inhibitors. Phase 2 trials have dem-
active thrombotic event. Discontinuing war arin may result in a onstrated reversal o direct actor Xa inhibitors Eliquis and
rebound hypercoagulable state, and surgery creates a prothrom- Xarelto. Phase 3 trials o ANNEXA-R and ANNEXA-A have
botic state. also shown promise. Keep an eye out or these agents in the
Patients who have a V E within 1 month o surgery have a next ew years!
50% risk o a second V E i not treated aggressively with anti-
coagulation. T ese recurrent events carry a mortality rate o
approximately 6%. Such patients should be placed on heparin Clinical Pearls
be ore and a er surgery i they are on war arin. I they are on
one o the newer anticoagulants, hold a dose o dabigatran, A platelet count o >20,000/mL is adequate or hemostasis.
apixaban, or rivaroxaban and then restart the drug postopera- Do not use plasma or a prothrombin complex concentrate to
tively. reverse anticoagulation unless there is signi icant bleeding
Patients with a V E within 1 to 3 months be ore a scheduled (e.g., intracranial bleed).
surgery should be considered or postoperative heparin/war arin Do not use venocaval ilters to prevent a second PE in a
or dabigatran, apixaban, or rivaroxaban. A V E > 3 months patient with a prior history o DVT. They do not work.
be ore the scheduled surgery should not pose signi cant addi- Heparin can cause a thrombocytopenia associated with
tional risk, and established prophylaxis guidelines should be ol- thrombotic complications (HITTS).
lowed. See above or a discussion o heparin bridging in atrial
I a patient is on war arin and the INR is 10 or less and the
brillation and heart valves. patient is asymptomatic, do not treat with vitamin K; simply
Objectives: Did you learn to . . . hold the war arin. I the INR is >10, give 1 to 5 mg o PO vita-
• Obtain an appropriate history in a patient with clotting prob- min K.
lems? It is not necessary to anticogulate patients with a irst DVT
• Recognize common inherited and acquired hypercoagulable that can be explained by a reversible cause (e.g., estrogen,
risk actors? surgery, immobilization) or more than 3 months.
• Recognize the risks o VTE in the pregnant patient? The PTT is the most use ul single test when screening or
• Identi y a patient presenting with antiphospholipid antibod- coagulopathy.
ies and understand the management o such patients? Trans use stable patients to a hemoglobin o 7 to 8 mg/dL. A
• Describe anticoagulation approaches in perioperative care? higher target can worsen outcomes.
Trans using platelets will not work in ITP. The platelets will
OK, you knew this was coming. A quick primer on the just get destroyed.
newer anticoagulants. By the time you read this, reversal With iron de iciency anemia, the reticulocyte count should
agents or dabigatran and the Xa inhibitors should be avail- go up within 10 days when appropriately treated with iron. I
able. So, concerns about uncontrollable bleeding should be this doesn’t happen reconsider your diagnosis.
resolved.
CHAPTER 6 • HEMATOLOGY AND ONCOLOGY 199

BIBLIOGRAPHY o Chest Physicians Evidence-Based Clinical Practice


Guidelines. Chest. 2012;141(2)(Suppl):7S–47S.
Abramson N, Melton B. Leukocytosis: basics o clinical assess-
ment. Am Fam Physician. 2000;62(9):2053–2060. Killip S, Bennett JM, Chambers MD. Iron de ciency anemia.
Am Fam Physician. 2007;75(5):671–678.
Allen GA, Glader B. Approach to the bleeding child. Pediatr
Clin North Am. 2002;49(6):1239–1256. Landgren O. Monoclonal gammopathy o undetermined
signi cance and smoldering myeloma: new insights into
Bain BJ. Diagnosis rom the blood smear. N Engl J Med.
pathophysiology and epidemiology. Hematology Am Soc
2005;353:498–507.
Hematol Educ Program. 2010;2010:295–302.
Booth KK, et al. Systemic in ections mimicking thrombotic
Moake J. T rombotic thrombocytopenia purpura ( P) and
thrombocytopenic purpura. Am J Hematol. 2011;86(9):
other thrombotic microangiopathies. Best Pract Res Clin
743–751.
Haematol. 2009;22(4):567–576.
Brandhagen DJ, et al. Recognition and management o
National Institutes o Health. What causes thrombotic
hereditary hemochromatosis. Am Fam Physician. 2002;
thrombocytopenic purpura? Available at https://www.
65(5):853.
nhlbi.nih.gov/health/health-topics/topics/ttp/causes;
Carson JL, et al. Red blood cell trans usion: a clinical practice accessed on 26 March, 2014.
guideline rom the AABB. Ann Intern Med. 2012;157(1):
O’Connell X, Horita J, Kasravi B. Understanding and inter-
49–58.
preting serum protein electrophoresis. Am Fam Physician.
Crownover BK, Covey CJ. Hereditary hemochromatosis. Am 2005;71(1):105–112.
Fam Physician. 2013;87(3):183–190.
Portola Pharmaceuticals. Andexanet al a:FXa inhibitor anti-
Frei eld AG, Bow EJ, Sepkowiz KA. Clinical practice guideline dote. Available at http://www.portola.com/clinical-
or the use o antimicrobial agents in neutropenic patients development/andexanet-al a-prt4445- xa-inhibitor-
with cancer: 2010 update by the In ectious Diseases Soci- antidote/
ety o America. Clin In ect Dis. 2011;52(4):e56–e93.
Short MW, Domagalski JE. Iron de ciency anemia: evalua-
Ganetsky M, et al. Dabigatran: review o pharmacology and tion and treatment. Am Fam Physician. 2013;87(2):
management o bleeding complications o this novel oral 98–104.
anticoagulant. J Med Toxicol. 2011;7(4):281–287.
Wigle P, et al. Updated guidelines on outpatient anticoagula-
Guyatt GH, et al., or the American College o Chest Physi- tion. Am Fam Physician. 2013;87(8):556–566.
cians Antithrombotic T erapy and Prevention o T rom-
Wilbur J. Surveillance o the adult cancer survivor. Am Fam
bosis Panel. Executive summary: antithrombotic therapy
Physician. 2015;91(1):29–36.
and prevention o thrombosis, 9th ed: American College

Other help ul of ce resource links:


American Society o Clinical Oncology. www.asco.org.
American Society o Clinical Oncology. www.plwc.org. (Web-
site or patients living with cancer.)
American Society o Hematology. www.hematology.org. (Links
to many blood disease websites.)
American Association or Clinical Chemistry. www.labtestson-
line.org. (In ormation on general hematology/oncology
subjects and laboratory testing.)
Multiple Myeloma Research Foundation. www.themmr .org.
(Everything about myeloma.)
Gastroenterology
Mark A. Graber a d Ye-Ji Lee
7
D) P tients over the ge o 45 with new onset sy pto s re
CASE 7.1 considered to be t high risk.
A 43-year-old woman complains o a burning pain in the E) C nd D.
retrosternal area. Her symptoms started about 2 years ago
and initially responded to sel -medication with antacids or Answer 7.1.2 The correct answer is “E.” Bec use o the high
histamine-2 receptor antagonists (H2 blockers). However, sensitivity nd speci city o sy pto s (> 90%), ost p tients
within the last 4 months, she has had nearly daily problems. do not need endoscopy or ny other procedure or test to ke
She requently wakes up in the middle o the night with the di gnosis o GERD. P tients with re r ctory GERD or
retrosternal, burning pain, radiating to her neck. She also those with new sy pto s over the ge o 45 ye rs (50 by so e
requently notices an acidic taste in her mouth. While ant- sources) w rr nt endoscopy. “B” is incorrect. Screening or the
acids help somewhat, they only provide transient relie . She presence o B rrett esoph gus in p tients with GERD is low
has otherwise been healthy. She currently takes up to 800 mg yield. rgeted, not routine, endoscopy y be bene ci l. Even
o cimetidine per day plus the antacids. She denies tobacco or then, the bene t is li ited (J Natl Cancer Inst. 2011;103:1049).
alcohol use. Her physical examination is normal. T is is still tter o ctive deb te nd reco end tions y
ch nge.
Question 7.1.1 Which o the ollowing is NOT considered
a “red ag” when it comes to patients with acid re ux/
HELPFUL TIP:
dyspepsia?
The li k betwee Helic bacter pyl ri a d GERD i p r-
A) Weight loss.
ly de i ed. There i g d evide ce t upp rt eradi-
B) rouble sw llowing liquids.
cati H. pyl ri r the treatme t GERD. The ame
C) rouble sw llowing solids.
i true r ulcer dy pep ia.
D) A cr ving or doughnuts.

Answer 7.1.1 The correct answer is “D.” OK, we will give you
this one. One o the ost critic l p rts o p tient ssess ent is T e patient denies any “red ags.” Based on the history, you
to ke sure there re no “red g” sy pto s th t y indi- assume that this patient has GERD.
c te ore th n si ple g stroesoph ge l re ux dise se (GERD).
Red g sy pto s include dysph gi , weight loss, ne i , Question 7.1.3 You recommend:
spir tion, e rly s tiety or vo iting, nd cough. A cr ving or A) B riu sw llow study.
doughnuts is benign ( lthough e ting the is not) nd y be B) Esoph ge l no etry.
seen ong the book editors, police, college students, V sh the C) Esoph gog stroduodenoscopy (EGD).
St pede, nd others. D) A bul tory pH study o the esoph gus.
E) ri l o o epr zole nd li estyle odi c tions.
Question 7.1.2 Which o the ollowing statements is true?
A) All p tients with GERD need endoscopy. Answer 7.1.3 The correct answer is “E.” As noted bove, the
B) Routine endoscopy in p tients with GERD rkedly reduces sensitivity nd speci city o cl ssic l sy pto s llows or the
the risk o esoph ge l c ncer. di gnosis o GERD without ddition l studies. esting is ctu-
C) T e di gnostic sensitivity nd speci city o typic l sy p- lly less sensitive th n sy pto s or GERD with the sensitivity
to s o GERD is > 90%. o tests v rying between 50% nd 70%. M ny p tients h ve
200
CHAPTER 7 • GAs TRo En TERo Lo GY 201

lse-neg tive EGD. H2 blockers can be used as the rst-line Question 7.1.5 All o the ollowing are true regarding
treatment in patients with GERD, but this p tient h s lre dy Barrett esophagus EXCEPT:
iled ci etidine. St rting proton pu p inhibitor (PPI), like A) M les re ore likely to h ve B rrett esoph gus th n
o epr zole, is the next step nd is pre erred s the rst-line e les.
ther py in p tients with severe sy pto s. Li estyle odi c - B) B rrett esoph gus is due to ch nge in the esoph ge l
tions should be de including weight loss (i overweight); ucos ro colu n r to squ ous.
void nce o c rbon ted bever ges, c eine, excessive lcohol, C) B rrett esoph gus occurs in 10% to 15% o p tients with
nd l rge or l te evening e ls; void nce o nti-cholinergics, erosive esoph gitis.
c lciu ch nnel blockers, NSAIDs, nd sed tive drugs; s ok- D) B rrett esoph gus incre ses the risk o denoc rcino by
ing cess tion; nd elev tion o the he d o the bed using 6-in up to 30- old.
blocks. E) P tients with B rrett esoph gus should undergo periodic
screening EGD.

HELPFUL TIP: Answer 7.1.5 The correct answer is “B.” B rrett esoph gus is
M t patie t with GERD will have egative e d c pic di gnosed histologic lly when esoph ge l ucos l et pl si
i di g (termed er ive re lux di ea e [n ERD]— h s occurred and the usual squamous epithelial cells have
really, we did ’t make thi e up). s ympt m d t changed to columnar epithelium. Risk ctors or B rrett
c rrelate well with the pre e ce r degree e pha- include long-st nding re ux, le gender (6:1 le: e le pre-
geal i lammati r er i . ponder nce), iddle ge, tob cco use, nd white r ce. B rrett
esoph gus occurs in 10% to 15% o p tients with erosive esoph-
gitis, nd it dr tic lly incre ses the risk o esoph ge l de-
noc rcino by up to 30- old (N Engl J Med. 2011;365:1375).
Your patient is willing to try the PPI you prescribe, but she However, the bsolute risk o denoc rcino is still s ll,
also wants to know about surgery that a riend had. bout 0.12% to 2% nnu lly. Surveill nce or B rrett depends on
the level o dyspl si ound on initi l ex in tion, nd guide-
Question 7.1.4 Regarding surgery or GERD, you can tell her: lines exist th t dict te the requency with which repe t EGD
A) Blo ting nd in bility to belch do not occur with ntire ux should be per or ed.
surgery.
B) Antire ux surgery should be thought o s the rst-line
bec use it is v stly superior to edic l ther py. HELPFUL TIP:
C) Ye rs er ntire ux surgery, ny p tients will require Pr ki etic age t , like met cl pramide (Regla ), ca be
edic tion or GERD sy pto s. u e ul i the treatme t GERD. I additi t pr m t-
D) L p roscopic undoplic tion will ruin her bulous bikini i g t mach emptyi g, met cl pramide i crea e ga -
body by le ving l rge idline sc r. tr e phageal phi cter t e. Remember that met -
cl pramide ca cau e extrapyramidal ide e ect with
Answer 7.1.4 The correct answer is “C.” Surgery y be n chr ic u e.
option in select p tients with re ux dise se. T e usu l indic -
tions or ntire ux surgery include ilure o edic l ther py
to control sy pto s nd ilure o edic l ther py to prevent
co plic tions (e.g., stricture nd pneu oni ). T e ost co - HELPFUL TIP:
only per or ed surgery is undoplic tion, in which the lower C mplicati chr ic GERD i clude er ive e ph-
esoph ge l sphincter is “wr pped” to enh nce its co petency. agiti , peptic tricture, a d ade carci ma the
While undoplic tion will llevi te sy pto s in 80% to 95% o e phagu . H wever, the ab e ce heartbur ymp-
p tients, there is progressive loss o e ectiveness over ti e (only t m d e t rule ut re lux-related c mplicati
40% re without edic tion er 10 ye rs). Adverse e ects o i ce appr ximately e- urth patie t with peptic
surgery include persistent dysph gi (requiring ddition l inter- tricture (a tricture the e phagu , which re ult
ventions in 3–7%), g s, blo ting, nd in bility to belch. “D” is r m the heali g ulcerative e phagiti ) a d e-
incorrect bec use l p roscopic procedure should result in just third th e with ade carci ma the e phagu
ew s ll sc rs. had heartbur pri r t diag i . Barrett e phagu
ca regre with the adequate treatme t GERD.
A er a trial o your avorite PPI (that her insurance will
cover a er you play “Guess What’s on Our Formulary”),
your patient continues to have symptoms. You review li e- Objectives: Did you learn to . . .
style modi cations with her, and she assures you that she • De cribe the diag i a d ma ageme t GERD?
has made these changes. You re er her or EGD. T e esopha- • U e H2 bl cker a d PPI i the ma ageme t GERD?
geal biopsy is consistent with the visual report, which shows • Rec g ize c mplicati GERD a d ri k act r r Bar-
Barrett esophagus. rett e phagu ?
202 FAMILY MEDICIn E EXAMIn ATIo n & Bo ARD REVIEW

the cervic l region. P tients with oroph rynge l dysph gi o en


CASE 7.2 h ve ore proble s with liquids th n solids nd y co pl in
A 53-year-old woman comes to your o ce complaining o chest th t liquid co es out o their nose when they try to sw llow.
pain and problems swallowing. She says ood seems to hang up Esoph ge l dysph gi is usu lly described s beginning sever l
in the retrosternal area. T is started several years ago, has grad- seconds er sw llowing, nd p tients requently point to the
ually worsened, and now occurs at least twice per week. Gen- supr stern l notch or retrostern lly when trying to loc lize the
erally, only solid oods cause problems. She has become very re c using sy pto s. P tients in who no c use c n be ound
care ul, only taking small bites and chewing them well be ore er det iled investig tion re o en c tegorized s h ving unc-
swallowing. Should she experience problems, she has ound tion l dysph gi . T ere is no such thing s berr nt dysph gi .
that drinking additional liquids alleviates her symptoms within
minutes. She does not regurgitate ood and has not lost weight. A complete review o systems and physical examination is
unrevealing. She is not taking pills that can cause erosive
Question 7.2.1 What is the symptom this patient is com esophagitis such as doxycycline or bisphosphonates, she
plaining o ? is not HIV positive making candidal and HSV esophagitis
A) Dysph gi . unlikely, and she again notes progressive dysphagia, initially
B) Globus sens tion. to solids and now to liquids.
C) Odynoph gi .
D) Aeroph gi . Question 7.2.3 Which o the ollowing is the single best test
E) Ph gophobi . to arrive at a diagnosis in this patient?
A) B riu sw llow.
Answer 7.2.1 The correct answer is “A.” Dysph gi , ro the B) Esoph ge l no etry.
Greek dys ( e ning “with dif culty” . . . s opposed to “Dis,” C) EGD.
which is city in D ntes’s Hell in ddition to being neologis ) D) A bul tory pH study o the esoph gus.
nd phagia ( e ning “to e t”), re ers to the sens tion th t ood E) Di gnostic tri l o high-dose PPI.
is being hindered in its p ss ge ro the outh to the sto -
ch. Odynoph gi re ers to p in upon sw llowing, eroph gi Answer 7.2.3 The correct answer is “C.” Visu liz tion o the
to sw llowing o ir, nd globus sens tion to perception o esoph gus is critic l in p tients with dysph gi . Our p tient
lu p or ullness in the thro t th t is te por rily relieved by presents with esoph ge l dysph gi or solids, which is ost
sw llowing. Ph gophobi is just wh t you think it is: e r o e t- consistent with structur l proble , such s stricture, web, or
ing or sw llowing. neopl s . Endoscopy is the best di gnostic study to ev lu te the
esoph ge l ucos s it llows one to obt in biopsies nd per-
or endoscopic interventions such s dil tions, i indic ted. B r-
HELPFUL TIP:
iu sw llows re second best nd h ve llen out o vor when
Aer phagia i a c mm a d u derdiag ed cau e
endoscopy is v il ble. Should the endoscopy co e b ck neg -
abd mi al ympt m . Ge erally, patie t will c mplai
tive, esoph ge l no etry (“B”) y be use ul s it will help
belchi g a ciated with abd mi al bl ati g, e pecial-
deter ine i otility disorder is the c use o sy pto s. While
ly a ter meal . It i t u c mm r patie t t me ti
“di gnostic” tri l with PPI (“E”) is n ppropri te str tegy in
that they have t l e their belt a ter meal . Aer phagia
p tients with typic l GERD, the presence o n l r sy pto ,
i exacerbated by gum chewi g (which cau e reque t
such s dysph gi , requires urther testing. “D,” n bul tory
wall wi g air), eati g quickly, dri ki g carb ated
pH study, does not help in de ning the etiology o the dysph gi .
beverage , a d m ki g. Aer phagia ca be mitigated by
eati g m re l wly, av idi g carb ated beverage , etc.
HELPFUL TIP:
Ma y patie t have breakthr ugh pai at ight eve
Question 7.2.2 Given the above patient’s description o whe a PPI. The rea r thi i that PPI ly act
symptoms and the location o her discom ort ( ood stick cell that have bee activated ( uch a by a meal). PPI
ing in the retrosternal esophagus), what type o dysphagia h uld be take 30 mi ute be re eati g. Try addi g
is she most likely to su er rom? a H2 bl cker at ighttime r patie t havi g break-
A) Oroph rynge l dysph gi . thr ugh pai .
B) Esoph ge l dysph gi .
C) Function l dysph gi .
D) Aberr nt dysph gi . HELPFUL TIP:
Reb u d acid hyper ecreti ca lead t ympt m re-
Answer 7.2.2 The correct answer is “B.” Dysph gi c n be di - curre ce i p te t a ti ecret ry age t , uch a PPI , are
erenti ted by where the sy pto s see to occur. Oroph ryn- t pped abruptly. Ma y expert rec mme d teppi g
ge l dysph gi , lso known s “tr ns er dysph gi ,” rises ro d w t H2 bl cker pri r t ce ati . H wever, eve H2
dif culty in the upper esoph gus nd ph rynx. P tients co - bl cker ca al cau e a le er degree reb u d acid
pl in o ood getting stuck i edi tely upon sw llowing, nd hyper ecreti . s , c ider taperi g the e drug a well.
when sked to point to the loc tion, p tients requently identi y
CHAPTER 7 • GAs TRo En TERo Lo GY 203

TABLE 7-1 CAUSES OF ESOPHAGEAL DYSPHAGIA


Mechanical Lesions
Intrinsic Extrinsic
Be ig tum r Aberra t ubclavia
Cau tic e phagiti / tricture artery
Radiati e phagiti / tricture Cervical te phyte
Peptic e phagiti / tricture E larged a rta
E i philic e phagiti E larged le t atrium
Diverticula Media ti al ma
Malig a cy P t pi al urgery
P t-GI urgery
Ri g a d web
F reig b die
Motility Disorders
Achala ia
FIGURE 7-1. Chaga di ea e (Trypanosoma cruzi)
Di u e e phageal pa m
Hyperte ive l wer e phageal phi cter
T e patient returns rom her endoscopy. According to the n peci c e phageal m tility di rder
report, she had severe esophagitis with con uent erosions n utcracker e phagu
in the distal esophagus. During your physical examination, s cler derma
you nd the skin changes shown in Figure 7-1. When you ask
about these, the patient reports a history o bluish ngertip cre ted by undoplic tion. H. pylori tre t ent h s no role in the
discoloration with cold temperatures. She also complains n ge ent o this illness. Steroids could cert inly worsen the
that her ngers eel “tight” at times. g strointestin l (GI) sy pto s by irrit ting the g stric ucos .
Other c uses o esoph ge l dysph gi re listed in ble 7-1.
Question 7.2.4 With this new in ormation, which o the
ollowing diagnoses are you considering?
A) CRES syndro e. HELPFUL (AND IMPORTANT) TIP:
B) Esoph ge l denoc rcino . It w uld t urpri e u t ee thi e the b ard
C) Met st tic colonic denoc rcino . exami ati . A i crea i gly rec g ized cau e dy -
D) Sicc syndro e. phagia i eosinophilic esophagitis. The cla ic adult
E) Peptic ulcer dise se (PUD). patie t pre e t with dy phagia, “GERD ympt m ”
a d upper abd mi al pai u re p ive t PPI , a d
Answer 7.2.4 The correct answer is “A.” Notice the tel ngiec- p ibly d impacti . Childre te pre e t with
t si in the picture bove. CRES is n crony or syndro e eedi g pr blem (age 2), recurre t v miti g (age 8),
th t includes C lcinosis cutis, R yn ud pheno enon, Esoph - a d chr ic abd mi al pai (age 12) r d impacti
ge l dys otility, Sclerod ctyly, nd el ngiect si s. Figure 7-1 (tee age year ). A ciati with childh d a thma i
shows tel ngiect si s on the p l r digit l p d. Up to 60% o tr g a d dietary elimi ati therapy may be help ul
p tients with CRES h ve erosive esoph gitis. Dysph gi is i childre . Treatme t i adult (a d te childre ) i -
co on nd is due to esoph ge l stricture nd/or dys otility. v lve wall wi g i haled c rtic ter id (e.g., lutica-
e r bude ide), a d, i evere ca e , y temic te-
Question 7.2.5 Given her endoscopic f ndings and the act r id . s y temic e i philia i rare; m teluka t ha a
that she likely has CREST syndrome, what treatment do you checkered rec rd whe it c me t treati g e i phil-
suggest? ic e phagiti a d appear le e ective tha ter id .
A) H 2 blocker.
B) PPI.
C) High-dose corticosteroids. HELPFUL TIP:
D) H. pylori er dic tion. Dy phagia that pr gre e r m lid a d the t liq-
E) L p roscopic undoplic tion. uid i a cla ic ympt m e phageal ca cer a d ca -
t be ig red.
Answer 7.2.5 The correct answer is “B.” Severe re ux nd dys-
ph gi re h ll rks o CRES syndro e. T e esoph gus h s Objectives: Did you learn to . . .
poor otility with i p ired unction o the lower esoph ge l • Di ere tiate dy phagia r m ther GI ympt m ?
sphincter llowing re ux while poor otility reduces esoph - • Take a adequate hi t ry t evaluate the eti l gy dy pha-
ge l cle r nce. Chronic nd potent cid suppressive edic - gia?
tions, such s PPIs, re indic ted. Fundoplic tion is relatively • De cribe di ere t type dy phagia a d ide ti y vari u
contraindicated, s the poor contr ctile unction o the esoph- cau e dy phagia?
gus y not gener te enough orce to overco e the b rrier • Rec g ize that CREs T y dr me a a cau e dy phagia?
204 FAMILY MEDICIn E EXAMIn ATIo n & Bo ARD REVIEW

usu lly be cco p nied by other syste ic sy pto s (such s


QUICK QUIZ: REGURGITATIo n tigue, weight loss or de th—is th t syste ic sy pto ?),
which the p tient denies. PUD should be cco p nied by
You see 61-ye r-old wo n in your of ce co pl ining o h li- “he rtburn,” which is not present in our p tient.
tosis, he rtburn, nd regurgit tion o undigested ood 4 hours
er e ting. She lso eels th t the ood is sticking in her thro t. Question 7.3.2 Which o the ollowing symptoms WOULD
NOT be an expected symptom o nonulcer dyspepsia?
Wh t is the MOS LIKELY c use o her sy pto s? A) Fullness.
A) Sch tzki ring. B) E rly s tiety.
B) Zenker diverticulu . C) Belching.
C) Ach l si . D) Intoler nce to eggs.
D) Foreign body.
E) Esoph ge l web. Answer 7.3.2 The correct answer is “D.” Nonulcer dyspepsi is
ch r cterized by ll the other sy pto s nd lso includes such
The correct answer is “B.” L te regurgit tion o undigested sy pto s s bdo in l distention, borboryg i (i.e., gr nd-
ood is p thogno onic or Zenker diverticulu . A Zenker di- p ’s tu y gurgling he rd cross the roo t T nksgiving),
verticulu is n cquired out-pouching o esoph ge l ucos , epig stric or substern l p in, norexi , n use , vo iting, nd
which typic lly beco es sy pto tic in iddle ge or l ter in bdo in l tenderness. “D,” intoler nce to eggs, is not p rt o
li e. T e di gnosis is con r ed by l ter l view o b riu sw l- nonulcer dyspepsi . Wh t h ve you got g inst eggs? T e edi-
low or by visu l con r tion with EGD. T e other di gnoses tors like eggs, po ched i possible nd served s eggs benedict.
listed c n c use dysph gi nd regurgit tion but re less likely. While there re sever l proposed p thophysiologic ech nis s
“A,” Sch tzki ring, is lower esoph ge l ucos l ring th t c n by which nonulcer dyspepsi y occur, ood intoler nce is not
“c tch” ood nd c use dysph gi . “C,” ch l si , results ro one o the .
loss o innerv tion o the lower esoph gus with i p ired esoph-
ge l otility nd gener lly occurs in young nd iddle- ged Question 7.3.3 What is the most appropriate next step in
dults. P tients with ch l si typic lly h ve dysph gi with this patient?
solids nd liquids. “D,” oreign body, should be di gnos ble by A) Obt in H. pylori seru ntibodies.
history. Elderly dults, children, nd p tients with psychi tric B) Obt in n bdo in l ultr sound ex in tion o liver, bili-
dise se re ost t risk or oreign body in the esoph gus. “E,” ry tree, nd p ncre s.
esoph ge l web, is c used by thin l yer o ucos cross the C) Obt in det iled diet ry history.
esoph ge l lu en nd presents uch like Sch tzki ring. D) Obt in n upper GI x-r y series.
E) Re er or g stroscopy.
CASE 7.3
Answer 7.3.3 The correct answer is “C.” Does this see to be
A 32-year-old woman presents with 3 years o intermittent GI contr diction to the previous question? Well, we don’t e n
complaints a er eating. She describes epigastric pressure and it to be so. Although ood intoler nce does not c use nonulcer
bloating a er ood intake. Her weight is stable. She denies dyspepsi , the sy pto s o the two types o disorders overl p
heartburn, vomiting, diarrhea, constipation, or blood in her to so e degree. Considering the prev lence o selective c rbo-
stools. She only takes a multivitamin and she has not tried hydr te l bsorption syndro es, high int ke o l ctose nd
any speci c remedies. T e physical examination is normal. ructose-cont ining ood nd bever ges y contribute to GI
sy pto s in signi c nt nu ber o p tients. T ere ore, the
Question 7.3.1 What is the MOST LIKELY diagnosis given history should lw ys cont in det iled ssess ent o diet ry
the in ormation above? h bits nd sy pto s in rel tion to ood int ke. A ore exten-
A) Pyrosis. sive workup is not necess ry t this point in ti e s the p tient is
B) PUD. younger th n 45 ye rs o ge nd h s no l r sy pto s. While
C) Nonulcer dyspepsi . in ection with H. pylori c uses g stritis nd peptic ulcers, sev-
D) Sto ch c ncer. er l controlled studies de onstr ted no consistent bene t o
H. pylori er dic tion in p tients with dyspepsi without ulcers.
Answer 7.3.1 The correct answer is “C.” Nonulcer dyspepsi
is n ill-de ned condition ch r cterized recurring inter ittent HELPFUL TIP:
epig stric disco ort nd ullness in the bsence o ucos l I y u ch e t te t a d treat r H. pyl ri, the pre erred
lesions or other structur l bnor lities o the GI tr ct. Non- te ti g trategy i 13C urea breath te t r t l a tige
ulcer dyspepsi is lso known s “ unction l dyspepsi .” While te t (keep readi g r m re).
bout 20% o the gener l popul tion h s nonulcer dyspepsi ,
only bout 20% o these seek edic l ttention. Pyrosis (“A”)
e ns the s e thing s he rtburn; this p tient h s no co - You obtain a detailed dietary history, suggesting a potential
pl ints o he rtburn. Sto ch c ncer o 3 ye rs dur tion would contribution o ructose intolerance and excessive carbonated
CHAPTER 7 • GAs TRo En TERo Lo GY 205

beverage intake (it’s those ve Giant Gulp so drinks a day).


HELPFUL TIP:
Her symptoms persist despite appropriate diet modi cations.
F r ulcer dy pep ia, ge eral ma ageme t pri ci-
You choose to start the patient on a PPI. One week later, your
ple call r rea ura ce by the phy icia a d the av id-
nurse receives a call rom your patient about severe abdomi-
a ce repeated diag tic te ti g. Patie t h uld
nal pain and diarrhea. She has not noticed blood in her stool.
make appr priate li e tyle m di icati (av id t bac-
c , ca ei e pr duct , a d alc h l) a d limit r av id ag-
Question 7.3.4 What is the best next step?
gravati g medicati (n s AID ). Patie t h uld chew
A) She is h ving withdr w l ro the ve Gi nt Gulp sod s.
their d l wly a d eat m re reque t, mall meal .
Add b ck the ructose nd the 2,000 kc l.
Fi ally, i there i u derlyi g p ychiatric m rbidity, re-
B) Check sting g strin level.
laxati trai i g r treatme t peci ic di ea e ca
C) Add ntibiotic ther py or b cteri l overgrowth.
be help ul.
D) Discontinue the PPI.
E) Add n ntidi rrhe l.

Answer 7.3.4 The correct answer is “D.” Di rrhe , including


ro Clostridium di cile, is co on dverse e ect o PPIs HELPFUL TIP:
th t occurs in t le st 5% to 7% o p tients. Discontinu tion PPI are t be ig drug a d have bee a ciated
gener lly le ds to r pid resolution in the jority o c ses with (1) i crea ed ri k te p r i a d hip rac-
( lthough ybe not o C. di cile-rel ted di rrhe ). While ture i the elderly, (2) i crea ed ri k p eum ia,
reduced sto ch cidity ro PPIs or H 2 blockers y result (3) i crea ed ri k C. di icile c liti , (4) i ter ere ce
in b cteri l coloniz tion o the proxi l GI tr ct, the e rly with vitami a d mi eral ab rpti (B vitami , ir ,
onset nd severity o sy pto s described rgue g inst b cte- calcium), (5) diarrhea a ted ab ve, a d (6) p ible
ri l overgrowth s the etiology o this p tient’s sy pto s (“C”). i crea ed ri k deme tia i the elderly (JAMA n eur l.
While PPIs c n elev te g strin levels, the hyperg strine i seen 2016;E1–E7). s t p them a ter the treatme t i c mplet-
is not co p r ble to levels seen in Zollinger–Ellison syndro e, ed (i y u ca . . . . GERD i a chr ic di ea e a d t ppi g
which c n lso c use di rrhe . Stool studies nd e piric ther- a PPI i a ciated with reb u d acid ecreti ) a d aim
py with ntidi rrhe ls y be considered i discontinuing the r the l we t e ective d e.
PPI does not i prove sy pto s.

HELPFUL TIP: Objectives: Did you learn to . . .


There are drug that have great evide ce t upp rt • Diag e a d ma age ulcer dy pep ia?
their u e i ulcer dy pep ia. PPI a d H2 bl cker • Appreciate the atural hi t ry ulcer dy pep ia?
are te u ed. Pr ki etic age t may be help ul, a d i • Rec g ize imp rta t ide e ect PPI ?
the U ited s tate thi mea met cl pramide r eryth-
r myci . Ci apride ha bee rem ved r m the market CASE 7.4
ec dary t cardiac arrhythmia (QT pr l gati with
A 56-year-old woman comes to the emergency department
t r ade de p i te ). Met cl pramide, c ur e, i a -
a er a sudden episode o hematemesis. Yesterday, she had two
ciated with tardive dy ki e ia a d extrapyramidal
bowel movements that were dark, sticky, and oul smelling.
reacti . Erythr myci cau e GI ide e ect a d QT
She woke up nauseated and has since twice vomited a small
pr l gati .
amount o bright red blood. She also eels dizzy. Because o
knee pain related to skydiving 1 week ago, she started tak-
Question 7.3.5 Which o the ollowing IS TRUE o the natu ing ve tablets o naproxen (1,100 mg) twice daily. She takes
ral history o nonulcer dyspepsia? no other medicines and denies any signi cant past medical
A) Most p tients will not h ve sy pto - ree periods, inste d history.
h ving const nt dyspepsi . On physical examination, you nd a pale, uncom ortable,
B) Most p tients i prove with pl cebo tre t ent. but alert patient. She is tachycardic (pulse o 115 bpm) and
C) Most p tients will go on to develop ulcers. has a drop in blood pressure rom 116/72 mm Hg supine to
D) Spont neous resolution o sy pto s is r re. 93/65 mm Hg standing. Her abdomen is at with hyperactive
bowel sounds. You note epigastric tenderness but no rebound
Answer 7.3.5 The correct answer is “B.” Up to 60% o p tients or guarding. T ere is melena on rectal examination.
in pl cebo controlled tri ls respond to pl cebo, king it di -
cult to prove the ef c cy o edic tions. As the bove nu - Question 7.4.1 Which o the ollowing steps will be LEAST
ber suggests, spont neous resolution o sy pto s is co on, help ul at this point in time?
while ny p tients will h ve chronic, inter ittent course A) Ad ission to the hospit l.
ch r cterized by sy pto - ree periods. Most p tients will not B) I edi te tre t ent with n intr venous (IV) H 2 blocker.
develop serious p thology. C) Re err l or e ergent endoscopy.
206 FAMILY MEDICIn E EXAMIn ATIo n & Bo ARD REVIEW

D) IV ccess nd uid resuscit tion. TABLE 7-2 RISK FACTORS FOR MALIGNANCY
E) L bor tory tests including he oglobin, co gul tion studies, IN GASTRIC ULCERS
nd blood type nd cross- tch.
• o ccurre ce i eth ic gr up rai ed i e demic area (A ia ,
Lati , etc.)
Answer 7.4.1 The correct answer is “B.” T e p tient h s • Helicobacter pylori i ecti
sy pto s nd clinic l ndings o he odyn ic lly signi - • Ab e ce rece t n s AID u e
c nt upper GI tr ct bleed. Although she is “w lking nd t lk- • Ab e ce c c mita t du de al ulcer r a pri r hi t ry
ing” just ne, uid d inistr tion nd hospit l d ission with du de al ulcer (du de al ulcer require higher acid ecreti ,
which i i c mpatible with the pa ga triti typical m t ga tric
close onitoring in n intensive c re environ ent re indi-
ca cer )
c ted. A b seline he oglobin nd he tocrit (HC ) should be • Gia t ulcer (> 2–3 cm)
obt ined. However, a normal hemoglobin does not exclude a • Ab e ce a pr tracted ulcer hi t ry—i ge eral, the l ger the
signi cant acute bleed, as hemodilution (in the absence o IV ulcer hi t ry, the l wer the ri k that a ga tric ulcer i ca cer. Ga tric
uids) requires several hours. Given the present tion, blood ulcer require acid a d ga tric ca cer u ually devel p i the etti g
atr phic pa ga triti
tr ns usion y beco e necess ry in the uture; there ore,
blood should be sent or type nd cross- tch. E rly endos-
copy should be per or ed to identi y the c use o her bleeding, B) Upper GI x-r y series to docu ent he ling.
nd endoscopic ther py should be undert ken i ppropri te. C) Switch to H 2 blocker.
re t ent with H 2 blockers does not ect the r te o bleeding; D) Discontinue cid-suppressive edic tion.
however, IV PPI ther py does decre se the risk o rebleeding in E) H. pylori ntibody test.
high-risk p tients by incre sing g stric pH to enh nce co gu-
l tion. PPI therapy and endoscopy are standard o care or Answer 7.4.3 The correct answer is “D.” While nonhe ling
upper GI bleeding. ulcers y be due to neopl s , the v st jority o duodenal
ulcers re benign. T ere ore, neither endoscopic nor r diologic
HELPFUL TIP: docu ent tion o he ling is necess ry. See ble 7-2 or ore
o rth tatic vital ig are t that u e ul i determi - on risk ctors or lign ncy in g stric ulcers. “C” is incor-
i g a patie t’ v lume tatu . Ma y hyp v lemic pa- rect. P tients with unco plic ted nd s ll (< 1 c ) duode-
tie t are not rth tatic a d ma y patie t wh are n l or g stric ulcers who h ve received dequ te tre t ent o
euv lemic have rth tatic cha ge (e.g., patie t H. pylori or NSAID-induced ulcer do not need long-ter ther-
a ti-hyperte ive a d the elderly). s , u e rth tatic py directed t ulcer he ling s long s they re sy pto tic
vital ig t c irm y ur cli ical u pici , but d t ollowing ther py. Antisecretory drugs c n be discontinued
u e them a a ab lute guide t the patie t’ v lume er 4 to 6 weeks in these p tients. “E” is incorrect. Serology or
tatu (JAMA. 1999;281:1022–1029). H. pylori will not be help ul in this p tient. First, histology is s
sensitive s other tests or H. pylori in ection nd the p tient’s
Question 7.4.2 What is the risk o su ering a clinically sig biopsies were neg tive or H. pylori. Second, serologic testing
nif cant GI event on NSAIDs? does not tell us i the patient is currently in ected, s ny
A) Up to 4% per ye r. p tients int in ntibody positivity even ye r er tre t ent.
B) Up to 10% per ye r.
C) Up to 25% per ye r.
D) Up to 50% per ye r. HELPFUL TIP:
E) Ne rly 99% per ye r. se itivity er l gic te ti g r H. pyl ri i 90% t
100%. But thi d e t i dicate curre t i ecti a d
Answer 7.4.2 The correct answer is “A.” T e risk o clinic lly may re lect pri r i ecti . Breath te ti g i 88% t 95%
signi c nt NSAID-rel ted GI event, including GI bleeding, per- e itive with m t al e egative the re ult the
or tion, or obstruction, is bout 1% to 4% per ye r. u e a tibi tic a d a tacid , i cludi g H2 bl cker
a d PPI . Thu , make ure that the patie t ha bee
T e patient undergoes endoscopy, which shows a small duo- a tibi tic r at lea t a m th pri r t te ti g a d ha
denal ulcer in the bulb with a visible vessel in the ulcer base. t take acid uppre r r 2 week pri r t te ti g.
Gastric biopsies show chemical gastropathy and no evidence Thi al h ld true r CLo te ti g. s t l a tige te t-
o H. pylori. A er endoscopic treatment, the patient recovers i g (94% e itive, 92% peci ic) i al available t help
nicely. T en, 3 months later, she returns to your o ce or a with the i va ive d cume tati H. pyl ri i ec-
ollow-up evaluation. She is asymptomatic and continues ti r eradicati . Fal e- egative t l a tige a ay
taking a PPI. Her physical examination is normal. Labora- may be due t rece t (withi 2 week ) u e a tibi tic
tory tests show normal hemoglobin. r a tacid .

Question 7.4.3 What is the best next step? Question 7.4.4 I your patient had come back H. pylori
A) Repe t endoscopy to docu ent he ling nd rule out lig- positive, which o the ollowing combinations IS NOT indi
n ncy. cated in the treatment o H. pylori?
CHAPTER 7 • GAs TRo En TERo Lo GY 207

A) O epr zole, cl rithro ycin, etronid zole. • Ide ti y patie t at high ri k r n sAID-i duced GI t xicity?
B) Ceph lexin, o epr zole, oxicillin. • Lear ab ut the treatme t a d diag i H. pylori?
C) Metronid zole, oxicillin, o epr zole.
D) Bis uth subs licyl te, etronid zole, tetr cycline, l nso-
pr zole. QUICK QUIZ: PILL PRo BLEMs
E) O epr zole, cl rithro ycin, oxicillin.
Which o these edic tions would be the LEAS likely c use o
Answer 7.4.4 The correct answer (and the regimen you esoph gitis i c ught in the esoph gus?
would not want to use) is “B.” All o the other regi ens c n be A) Pot ssiu chloride.
used to tre t H. pylori in ection. O note re regi ens “A” nd B) Ferrous sul te.
“D.” T ere is resist nce to etronid zole, so it should only be C) Alendron te.
used when the p tient is penicillin llergic OR t king qu druple D) Lor t dine.
ther py. riple ther py with PPI should be used where cl r- E) etr cycline.
ithro ycin resist nce is low. Qu druple ther py should be used
in p tients with recent or repe ted exposure to cl rithro ycin The correct answer is “D.” M ny edic tions c n c use “pill
or etronid zole or re s with high cl rithro ycin resist nce esoph gitis,” including pot ssiu chloride, errous sul te,
(> 15%). lendron te, tetr cycline ntibiotics, nd scorbic cid. Aspirin
nd other NSAIDs c n lso c use esoph gitis. S ller pills re
Question 7.4.5 Which o the ollowing IS NOT use ul or less likely to c use proble s. In ddition to not being irrit ting,
testing or H. pylori eradication a ter treatment? lor t dine is tiny.
A) Seru IgG ntibody titers.
B) CLO test.
C) Bre th ure test. QUICK QUIZ: THE CAs E o F THE Ho LYs To MACH
D) R dio ctive CO2 blood test.
E) Stool ntigen test. One o your p tients presents co pl ining o di rrhe nd
epig stric p in unresponsive to H 2 blockers or PPIs. He denies
Answer 7.4.5 The correct answer is “A.” Re e ber the help- s oking tob cco, t king NSAIDs, or drinking lcohol. Endos-
ul tip bove? Only 57% o p tients re ntibody neg tive to copy reve ls sever l ulcers. Biopsy or H. pylori is neg tive nd
H. pylori ye r er success ul tre t ent. T us, ntibody titers there is no lign ncy.
c nnot docu ent er dic tion. All o the other tests entioned
re unction l tests or the presence o H. pylori. T e CLO test is Which o the ollowing would be the ost ppropri te l bo-
done on biopsy speci ens nd docu ents the presence o ure r tory test to obt in in order to discover n etiology or the
splitting. T e s e is true or the bre th ure test nd the r dio- multiple ulcers ( nd di rrhe )?
ctive CO2 blood test. In both o these tests, ure is ingested. A) V so ctive intestin l peptide (VIP).
I H. pylori is present, r dio ctive CO2 is gener ted th t c n B) G strin.
be e sured in the blood or bre th. Con r tion o er dic - C) Gluc gon.
tion should be considered or ll p tients receiving H. pylori D) So tost tin.
tre t ent, but de nitely should be per or ed in the ollow-
ing: (1) p tients with persistent sy pto s o dyspepsi despite The correct answer is “B.” So e sort o screening test or
H. pylori tre t ent; (2) p tients with H. pylori ssoci ted ulcer; g strino is w rr nted in p tient who h s recurrent or re-
(3) p tients with g stric ucos ssoci ted ly phoid tissue type r ctory ulcers nd who is H. pylori neg tive nd does not use
ly pho ; (4) p tients who h d resection or e rly g stric c n- NSAIDs. Zollinger–Ellison syndro e is the n e given to the
cer (N Engl J Med. 2010;362:1597–1604). st te in which there is cid hypersecretion second ry to in-
cre sed g strin production, usu lly ro g strin-producing
tu or (g strino ). Up to 1% o p tients with PUD h ve
HELPFUL TIP: g strino . Seru g strin levels should be obt ined with the
n s AID-i duced ulcer ca ccur i the t mach, du - p tient sting and of PPIs, as PPIs will increase gastrin lev-
de um, a d cca i ally i the mall b wel a d c l . els. I the seru g strin is elev ted (> 10 ti es upper nor l
H wever, n s AID are m re reque tly u d t be the li it), urther investig tions will need to be per or ed. Other
cau e ga tric ulcer (up t 30%) tha the cau e re sons to consider obt ining seru g strin level include ul-
du de al ulcer (up t 20%). cers in unusu l loc tions (dist l duoden l nd jejunu ), ily
history o ulcers, nd ulcers ssoci ted with severe esoph gitis.
“A” is incorrect. VIP ctu lly works to suppress cid secretion.
Objectives: Did you learn to . . . VIPo s do occur, but VIPo s re ssoci ted with w tery di-
• Appreciate the r le n s AID u e i Peptic Ulcer Di ea e rrhe nd hypok le i —not ulcers. “C,” gluc gon, will not be
(PUD)? o uch help here. Gluc gon over secretion results in hypergly-
• Ma age a acute GI bleed? ce i nd ne i . “D,” so tost tin, is hor one th t inhibits
208 FAMILY MEDICIn E EXAMIn ATIo n & Bo ARD REVIEW

the secretion o g strin nd thus would be protective vis-à-vis Celi c rtery syndro e y h ve si il r present tion to SMA
ulcers. So tost tin secreting tu ors re very r re. syndro e but, obviously, involves the celi c trunk.

CASE 7.5
QUICK QUIZ: GI BLEEDIn G
A 54-year-old man comes to your o ce or his annual physi-
Isol ted bright red he te esis (e.g., no t chyc rdi nd no cal. He is taking naproxen or a recent ankle injury. Based on
ever), which occurs er sever l bouts o vo iting or dry your recommendation last year, he started taking one aspi-
he ves, is re erred to s: rin (81 mg) daily. He does not take any other medications.
A) Boerh ve syndro e. He exercises regularly, does not smoke, and drinks one glass
B) M llory–Weiss te r. o wine every day. Your examination is completely normal,
C) C eron lesion. except that a test or occult ecal blood is positive.
D) None o the bove.
Question 7.5.1 What is the next best step?
The correct answer is “B.” A M llory–Weiss te r occurs er A) Upper endoscopy.
repe ted tr u to the lower esoph ge l nd g stric ucos ro B) CBC.
orce ul retching. T is c n be di erenti ted ro Boerh ve C) Colonoscopy.
te r by the (gener lly) sel -li ited n ture o the bleeding nd the D) A nd B.
bsence o other sy pto s. Boerh ve syndro e is per or - E) B nd C.
tion o the esoph gus resulting ro sudden incre se in intr -
Answer 7.5.1 The correct answer is “E.” Current reco en-
esoph ge l pressure, c used by vo iting, retching, less requently
d tions suggest screening or colorect l c ncer in ll p tients
by coughing, childbirth, or weight li ing. It occurs higher in the
over the ge o 50 ye rs. esting or ec l occult blood is one o
esoph gus nd is ssoci ted with edi stinitis, ever, shock, nd
the ccepted ethods. However, it should only be per or ed s
de th i intervention is not orthco ing. C eron lesions re
screening test in sy pto tic individu ls. Moreover, the use
s ll ulcers in p tients with hi t l herni , re usu lly n inci-
o spirin nd/or NSAIDs nd oods such s undercooked e t
dent l nding, nd re likely c used by rubbing o the sto ch
y incre se the nu ber o lse-positive tests nd should thus
g inst the di phr g s the herni slides. T ey c n bleed, but
be voided. Fec l i unoche ic l tests using the current gener-
the history given bove is cl ssic or M llory–Weiss te r.
tion o stool c rds detects hu n he oglobin by i unologic
e ns. T us, it circu vents ny o the we knesses o the old
QUICK QUIZ: ABDo MIn AL PAIn gu i c test (but not bleeding second ry to NSAIDs, etc. giving
lse positive screen). I unoche ic l testing is now the
A 35-ye r-old e le presents to your clinic co pl ining o onset ethod o choice or detecting ec l occult blood when screen-
o severe idepig stric p in, weight loss, nd vo iting ollow- ing or colon c ncer. However, once you h ve he e-positive
ing e ls. Between e ls she is sy pto tic. T is h s been stool, you ust proceed to colonoscopy in p tient over the
going on or the p st 2 ye rs ever since she purpose ully lost 25 lb ge o 50. A CBC is lso i port nt to deter ine i the p tient is
to tt in he lthy weight or her height. Un ortun tely, she h s ne ic. Since the p tient currently h s no sy pto s re er ble
continued to lose weight bec use o the postpr ndi l p in nd to upper GI p thology, ev lu tion o the upper GI tr ct should
vo iting. On her ex in tion you notice idepig stric bruit. only be considered i the colonoscopy is neg tive.

T e ost likely di gnosis is: HELPFUL TIP:


A) Aortic neurys . Ir uppleme t may cau e t l t darke but they
B) Atherosclerotic dise se o the celi c trunk. Do n o T cau e al e-p itive guaiac te t . Guaiac te t
C) Superior esenteric rtery (SMA) syndro e. rely the pre e ce hem gl bi i the t l, t ir .
D) Chronic p ncre titis. D ’t blame a p itive guaiac a ir uppleme t.
E) None o the bove.

The correct answer is “C.” T is is typic l history nd physi- Review o his records shows that he underwent a colonoscopy
c l ex in tion or SMA syndro e. SMA syndro e is ore 3 years ago. T ree small adenomatous polyps were ound. He
likely to occur in p tient who h s lost signi c nt weight, re- also had scattered diverticula in the sigmoid.
sulting in thinning o the esenteric t p d. Here’s the p tho-
physiology: the SMA runs bove the duodenu nd beco es Question 7.5.2 Based on this new in ormation, which strat
stretched nd p rti lly occluded in response to e ls ( s the egy do you recommend?
sto ch nd duodenu exp nd), le ding to esenteric isch- A) Colonoscopy now.
e i nd ood version. SMA syndro e c n be di gnosed using B) Colonoscopy in 5 ye rs.
Doppler ultr sound to de onstr te incre sed velocity o blood C) Ye rly tests or ec l occult blood.
in the SMA or by C - ngiogr . “A” nd “B” re unlikely in D) Ye rly tests or ec l occult blood nd colonoscopy in 7 ye rs.
young p tient, nd “D” should not be ssoci ted with bruit. E) Colonoscopy in 7 ye rs.
CHAPTER 7 • GAs TRo En TERo Lo GY 209

Answer 7.5.2 The correct answer is “A.” Adeno tous polyps 6. I prep is not good, repe t colonoscopy t e rliest conve-
re considered prec ncerous. Reco ended ollow-up o n nience is indic ted.
deno tous polyp is by colonoscopy every 3 to 5 ye rs. So, he
did not truly h ve “neg tive” colonoscopy 3 ye rs go. Plus, Question 7.5.4 Patients presenting with hereditary non
we re not doing the colonoscopy in this c se or screening pur- polyposis colorectal cancer (HNPCC), or “Lynch syndrome,”
poses; it is di gnostic. We y nd nother source o his bleed- are more likely than patients with sporadic colon cancer to
ing. I his previous colonoscopy w s co pletely nor l (no have which o the ollowing f ndings?
deno tous polyps), you could rgue or stopping the spi- A) Le -sided colon c ncers.
rin nd NSAIDs nd ollowing up with seri l ec l occult blood B) L te ge onset o colon c ncer.
tests (FOB s). In this ltern tive scen rio, persistently positive C) Multiple colon c ncers di gnosed si ult neously.
gu i c tests would le d to colonoscopy s well. D) An unusu l desire to visit Lynchburg, ennessee, nd dedi-
c tion to ll things J ck D niels.
HELPFUL TIP:
Ade mat u c l p lyp , either pedu culated r Answer 7.5.4 The correct answer is “C.” P tients with HNPCC
e ile, are a ciated with tra rmati t ca cer. (Lynch syndro e) re t incre sed risk o developing colon c n-
Hyperpla tic p lyp are c idered be ig . cer. About 5% o colon c ncers occur in p tients with Lynch
syndro e. r ns or tion ro n deno to c ncer is
ster in p tients with Lynch syndro e. In ddition, ny o
HELPFUL TIP: the neopl s s re loc ted in the right colon, so “A” is incorrect.
U.s . Preve tive s ervice Ta k F rce rec mme d r uti e Co p red to spor dic colon c ncer p tients, those with Lynch
cree i g r c l rectal ca cer, tarti g at the age syndro e re younger t the ti e o di gnosis, ore requently
50 year r average-ri k patie t (thi d e n o T apply present with ultiple colon tu ors, nd re ore likely to h ve
t high-ri k patie t —p itive amily hi t ry, k w extr colonic tu ors—especi lly endo etri l c ncer. T is is
ade mat u p lyp , etc.). A y the ll wi g m dali- genetic disorder, nd the occurrence o colorect l nd/or endo-
tie i acceptable: a ual Fo BT al e, Fo BT every 3 year metrial c ncer in three rel tives be ore the ge o 50 should sug-
with igm id c py every 5 year , a d c l c py ev- gest HNPCC. T e current reco end tion is to per or sur-
ery 10 year (with ut a ual Fo BT). Betwee the age veill nce colonoscopies t le st every 3 ye rs in these p tients.
75 a d 85 year , r uti e cree i g i t rec mme ded
but elect patie t h uld be c u eled cree i g.
A ter the age 85, cree i g h uld be d e. HELPFUL TIP:
Curre t guideli e rec mme d earlier cree i g c l -
c py r a ympt matic patie t with a amily hi t -
You re er the patient or a colonoscopy. T e report rom the ry c l ca cer, alth ugh the rec mme dati vary
endoscopist shows one small tubular adenomatous polyp in by rga izati . The America Ca cer s ciety (ACs )
the sigmoid colon that was completely removed. T e patient rec mme d cree i g c l c py at the age 40
wants to know when he needs another colonoscopy. r 10 year be re the age diag i r a y patie t
with a ir t-degree relative with c l ca cer diag ed
Question 7.5.3 Your answer is: u der the age 60 (e.g., i m m had c l ca cer at 48,
A) In 3 onths. tart y ur cree i g at 38). F r the e patie t , the ACs
B) In 1 ye r. rec mme d c l c py every 5 year . F r patie t
C) In 5 ye rs. with a ir t-degree relative with c l ca cer ver the
D) In 10 ye rs. age 60, the ACs rec mme d cree i g c l c py
E) Any o the bove is n equ lly v lid reco end tion. at the age 40 a d the ll w-up a per average-ri k
patie t (e.g., c l c py every 10 year i c mpletely
Answer 7.5.3 The correct answer is “C.” T e current surveil- egative).
l nce guidelines reco end the ollowing:
1. No polyps or only rect l hyperpl stic polyps, repe t in 10 ye rs. Objectives: Did you learn to . . .
2. 1 to 2 tubul r deno s less th n 1 c in size with only low- • De cribe the limitati Fo BT?
gr de dyspl si should h ve repe t colonoscopy in 5 ye rs. • Ch e the appr priate te t a d cree i g a d urveilla ce
3. 3 to 10 tubul r deno s, or villous e tures or high-gr de i terval r c l rectal e pla ia ?
dyspl si should h ve surveill nce in 3 ye rs. • Lear h w amily a d per al hi t ry c l rectal e pla-
4. P tients with > 10 deno s should be screened ore re- ia a ect cree i g a d urveilla ce trategy?
quently nd ili l colon c ncer syndro es should be con-
sidered.
5. I l rge sessile polyp is re oved in piece e l shion, then
CASE 7.6
repe t ex in tion in 3 onths is ppropri te to ssure A 28-year-old graduate student comes to your o ce com-
co plete re ov l. plaining o diarrhea. About 6 months ago, she noted a sudden
210 FAMILY MEDICIn E EXAMIn ATIo n & Bo ARD REVIEW

onset o loose stools. While she initially attributed these Question 7.6.2 Which o the ollowing is true regarding the
symptoms to “stomach u,” her problem has persisted. She pathophysiology and natural history o Crohn disease?
currently has about our to six loose bowel movements per A) Crohn dise se is genetic disorder, tr ns itted in n
day with nighttime de ecations. She has not seen blood in her utoso l-do in nt shion.
stool. In addition, she complains o cramps located in the le B) Crohn dise se is rel psing/re itting dise se, nd 30% o
and occasionally right lower abdomen. T e cramps are made p tients will i prove spont neously.
worse with ood intake and o en are associated with a need C) P tients with Crohn dise se r rely progress to dise se requir-
to de ecate. She has lost about 8 kg unintentionally within ing surgery.
the last 3 to 4 months. She denies any travel or antibiotic use. D) M inten nce ther py with glucocorticoids will reduce the
On physical examination, you notice some tenderness in the r te o recurrence o Crohn dise se.
right lower quadrant. You see a painless anal ssure in the E) GI stul e nd bscesses re r re co plic tions o Crohn
anterior commissure, and by the appearance, you judge that dise se.
the ssure is probably chronic in nature.
Answer 7.6.2 The correct answer is “B.” Crohn dise se nd UC
Question 7.6.1 What do you recommend as the next step in re both rel psing/re itting dise ses. Up to 30% o initi l ex c-
the evaluation and management o this patient? erb tions o Crohn dise se will re it without ny intervention.
A) Loper ide on scheduled b sis. While there is genetic co ponent to in tory bowel dis-
B) Re err l to holistic clinic or co ee colonic cle nsing regi en e se (IBD) (up to 100- old incre se in risk ong rst-degree
(using ir tr de, sh de grown org nic co ee only, o course). rel tives with IBD), no single, utoso l-do in nt gene h s been
C) Check titers o typic l ntineutrophil cytopl s ic ntibod- identi ed. “C” is incorrect bec use h l or ore o p tients with
ies (ANCA). Crohn dise se will ulti tely require so e sort o surgery. “D” is
D) Colonoscopy. incorrect. Un ortun tely, chronic glucocorticoid d inistr tion
E) Botulinu toxin injection into the re o the n l ssure. does not lower the r te o rel pse. T ere re ny co plic tions
o IBD (see next question), nd GI bscesses nd stul e occur
Answer 7.6.1 The correct answer is “D.” T e p tient’s history with rel tively high requency (20–40%) in Crohn dise se.
nd physic l ndings with p inless, nteriorly loc ted n l s-
sure nd di rrhe re consistent with Crohn dise se. T e best HELPFUL TIP:
next step in her ev lu tion is colonoscopy with inspection o the Alth ugh c idered a di ea e y u g adult , there
ter in l ileu . You y w nt to do so e stool testing be ore i a bim dal di tributi IBD with a ec d peak i
colonoscopy; testing or Giardia, C. di cile toxin, nd other lder adult i their 70 .
p thogens would be re son ble. But o the choices presented,
colonoscopy is the best option. “A” is in ppropri te since this
is long-st nding with weight loss; sy pto tic control is ne Question 7.6.3 Extraintestinal eatures o IBD include all o
but we need to gure out wh t is going on with this p tient. the ollowing EXCEPT:
“B” is incorrect. She h s di rrhe . . . her colon is lre dy cle n. A) Alopeci .
“C,” serologic testing, is re son ble choice but not the best. B) Arthritis.
Here is why: Crohn dise se is ssoci ted with ntibodies g inst C) Sclerosing chol ngitis.
s cch ro yces (ASCA), while ulcer tive colitis (UC) p tients D) Uveitis.
re ore o en positive or ANCA. However, the sensitivity o E) Cholelithi sis.
these tests is only bout 60%. Moreover, there is bout 20%
overl p (e.g., ANCA positive in Crohn dise se), r ising urther Answer 7.6.3 The correct answer is “A.” Alopeci is not n
questions bout the over ll use ulness o these tests. ASCA c n extr intestin l ni est tion o IBD. Arthritis rel ted to IBD
lso be positive in celi c dise se. “E” is lso incorrect. Fissures (enterop thic rthritis) is irly co on nd usu lly igr tory
re one o the n l ni est tions o Crohn dise se nd should in n ture, involving the l rge joints. Spondylo rthrop thy y
be ppro ched by tre ting the underlying dise se r ther th n lso be seen. Sclerosing chol ngitis nd utoi une hep titis
using surgic l techniques, botulinu injection, etc. Botulinu c n occur nd y be t l. Eye dise se y include uveitis nd
injections, nitroglycerin oint ent, nd ni edipine h ve ll been episcleritis. I port ntly, the in tre t ent o extr intestin l
used or n l ssures with success, but we need to di gnose this ni est tions is to tre t the IBD. IBD spondylo rthrop thy nd
p tient nd ddress the underlying dise se. pyoder g ngrenosu re i port nt exceptions, s these do
not lw ys i prove with the tre t ent o the underlying IBD.
HELPFUL TIP:
I Cr h di ea e, ab ut 25% patie t have di ea e HELPFUL TIP:
c i ed t the c l . A ther 40% have di ea e i the T xic megac l i a p te tially deadly c mplicati
ileum a d cecum, a d 30% have di ea e c i ed t the IBD that h uld be u pected i a patie t with IBD wh
mall b wel. The remai der have m re di u e di ea e pre e t with ever, abd mi al pai , a d h ck. Al ,
a d/ r di ea e i the pr ximal GI tract. c ider ther cau e i te ti al b tructi .
CHAPTER 7 • GAs TRo En TERo Lo GY 211

A) Sul llergy.
Returning to your patient, you next see her in the emergency B) Aspirin llergy.
department, where she presents with ever, abdominal pain, and C) Ane i .
bloody diarrhea. She is tachycardic and slightly hypotensive but D) A nd B.
alert and oriented. You suspect a relapse o her Crohn disease. E) B nd C.

Question 7.6.4 All o the ollowing are important aspects o


Answer 7.6.6 The correct answer is “D.” Sul s l zine cont ins
her management at this time EXCEPT:
both sul nd s licyl te oieties nd thus is contr indic ted in
A) Surgic l consult tion.
p tients with sul or spirin llergy.
B) IV ccess nd uid d inistr tion.
C) Glucocorticoids.
D) T lido ide. Question 7.6.7 Which o the ollowing is LEAST likely to be
E) Metronid zole. a complication o anti tumor necrosis actor (TNF) alpha
antibody therapy (e.g., in iximab [Remicade®]) or IBD?
Answer 7.6.4 The correct answer is “D.” T lido ide h s been A) Sepsis.
used s chronic ther py or IBD, but it is not indic ted in the B) He d che.
cute setting. T e p tient should be st bilized, nd this includes C) Abdo in l p in.
IV ccess nd uid resuscit tion. An ex cerb tion (or rel pse) D) Di rrhe .
o IBD c n be tre ted with glucocorticoids in the cute setting, E) Ane i .
nd ntibiotics re o en help ul. It ppe rs th t etronid zole
h s bene ci l e ects th t re not solely due to its nti icrobi l Answer 7.6.7 The correct answer is “E.” Ane i will gener-
properties. In this p tient, who y h ve n bscess or obstruc- lly i prove with the tre t ent o IBD. Ane i in IBD is o en
tion, urther ev lu tion (e.g., l bs nd bdo in l C ) nd sur- due to co bin tion o iron de ciency nd ne i o chronic
gic l consult tion re necess ry. dise se, but IBD c n c use n utoi une he olytic ne i
s well. All o the rest re co on co plic tions o in ix-
Question 7.6.5 Which o the ollowing is indicated in the i b. In ection deserves speci l ention. P tients on nti-
long term treatment o IBD? NF drugs re ore prone to sepsis. In ct, ongoing in ection
A) Az thioprine. is n bsolute contr indic tion to the use o nti- NF drugs.
B) Methotrex te. ke ny co pl ints ttribut ble to in ection (e.g., ever) very
C) 5-ASA oieties. seriously in p tients on nti- NF drugs (not th t you don’t
D) Loper ide. t ke the seriously in ll p tients . . . just ore seriously in
E) All o the bove. these). uberculosis nd ctive hep titis B re contr indic -
tions to the use o nti- NF drugs.
Answer 7.6.5 The correct answer is “E.” All the options re
indic ted or the tre t ent o IBD. O speci l note re the Question 7.6.8 All o the ollowing characteristics di eren
5-ASA drugs (e.g., Pent s nd As col). T ese h ve ewer side tiate ulcerative colitis (UC) rom Crohn disease EXCEPT:
e ects th n sul s l zine nd re the initi l drugs o choice or A) T e risk o colon c ncer is gre ter in UC th n in Crohn dis-
inten nce, once control o sy pto s h s been obt ined. e se.
Syste ic steroids (e.g., prednisone) re use ul cutely to induce B) Histologic lly, UC ppe rs s tr ns ur l dise se, where s
re ission but should be t pered nd stopped s soon s pos- Crohn dise se involves only the ucos l nd sub ucos l
sible. Steroids re o en indic ted long-ter or Crohn dise se l yers.
limited to the stomach and duodenum. C) UC l ost lw ys involves the rectu , where s Crohn dis-
Addition l drugs or the tre t ent o IBD include cyclospo- e se y or y not.
rine, 6- erc ptopurine (6-MP), in ixi b, certolizu b, nd D) In UC, the dise sed seg ents re continuous, while “skip”
d li u b (just s y Hu ir ®, it’s uch e sier), ong others. re s o he lthy bowel re seen in Crohn dise se.
Antidi rrhe l drugs such s loper ide re use ul to control
sy pto s. However, be sure to void ntidi rrhe l drugs in Answer 7.6.8 The correct answer is “B.” Histologic lly, UC
p tients who y h ve i pending toxic eg colon. Probiotics involves only the ucos nd sub ucos l tissue, while Crohn
nd l ctose void nce y lso be use ul ( lthough there is less dise se is tr ns ur l. All o the other st te ents re true. UC
evidence or these). involves the rectu in 95% o c ses nd dv nces proxi lly.
A er discussions with your patient about ongoing therapy at
discharge, you nd that cost is a major consideration or her. HELPFUL TIP:
T ere ore, you consider discharging her on sul asalazine. Medical a d urgical treatme t UC a d Cr h di -
ea e are imilar. Patie t with l g- ta di g UC with
Question 7.6.6 Which o the ollowing is an absolute con reque t relap e are ca didate r c lect my.
traindication to the use o sul asalazine?
212 FAMILY MEDICIn E EXAMIn ATIo n & Bo ARD REVIEW

HELPFUL TIP: TABLE 7-3 ROME III CRITERIA FOR IRRITABLE


IBD i crea e the ri k c l ca cer, m re reque t BOWEL SYNDROME
a d earlier c l c pie are rec mme ded. Rec m- C ti u u r recurre t abd mi al pai r di c m rt, at lea t
me dati vary, but m t expert guideli e rec m- 3 day per m th i the la t 3 m th , with ympt m tarti g at
me d c l c py every 1 t 2 year i patie t with lea t 6 m th pri r t diag i , and a ciated with at lea t 2 the
ll wi g:
UC a ter 8 year with di ea e. s cree i g parameter i
th e with Cr h di ea e are le well e tabli hed. • Relie with de ecati
• Cha ge i t l reque cy
• Cha ge i t l rm

HELPFUL TIP: Adapted r m L g treth GF, et al. Fu cti al b wel di rder .


I dicati r urgery i th e with IBD i clude per - Gastroenterology. 2006;130:1480. C pyright 2006, with permi i r m
rati , b tructi , ma ive hem rrhage, t xic mega- El evier.
c l , a d evere a d per i te t di ea e that impair
u cti r quality li e. s urgical re ecti h uld be so urther ev lu tion or colon c ncer (“B” nd “E”) c n be
c idered r i t lerable extrac l ic di ea e i clud- del yed unless there is nother indic tion. Second ry c uses o
i g arthriti a d ki le i . H wever, a kyl i g p - constip tion, such s hypothyroidis , edic tion side e ects
dyliti ( ec dary t Cr h di ea e) a d liver dy u c- or hyperc lce i , should be ruled out. T ere ore, SH level
ti d t re p d t c lect my. should be obt ined prior to deciding on ddition l di gnostic
or ther peutic steps. At this point in ti e, norect l no etry
nd de ecogr re unnecess rily inv sive procedures, nd nei-
Objectives: Did you learn to . . . ther will help you to deter ine i this p tient h s IBS.
• Diag e IBD?
• Di ere tiate Cr h di ea e r m UC? T e patient is euthyroid. Hypercalcemia and other electro-
• Ma age a patie t with IBD? lyte abnormalities have been ruled out. Since she does not use
medications other than the multivitamin, you decide to ini-
• Rec g ize extrai te ti al ma i e tati a d c mplicati
IBD? tiate the treatment o IBS. Based on available evidence, you
suggest using ber supplements such as psyllium.
CASE 7.7 Question 7.7.2 What do you tell your patient to expect?
A 52-year-old woman complains o abdominal pain, bloat- A) Co plete resolution o her sy pto s.
ing, and constipation. Her symptoms started about 5 years B) Incre se in stool requency nd stool volu e with less need
ago and became more bothersome within the last 6 months. or str ining.
She describes a dull pain in the le lower abdomen. T is pain C) Not uch. Fiber supple ent tion doesn’t work very well or
is alleviated by passing gas or having a bowel movement. T e IBS.
pain is generally related to eating, and she has had intermittent D) Decre se in bdo in l p in nd blo ting.
diarrhea and constipation with constipation predominating. E) Enlighten ent nd bsolute bliss.
wo years ago, she underwent a screening colonoscopy, which
was completely normal. Her review o systems is notable or a Answer 7.7.2 The correct (and un ortunate) answer is “C.”
weight gain o about 5 lb within the last 3 years. She is taking Although it h s been one o the inst ys o irrit ble bowel
only a multivitamin daily. Her physical examination is normal. ther py, the ef c cy o ber or IBS h s lw ys been in question.
A et - n lysis (Am J Gastroenterol. 2014;109:1367) suggests
Question 7.7.1 Which is the best next step? NN o 7 with psylliu to bene t one p tient. Perh ps surpris-
A) De ecogr . ingly, there re only 14 r ndo ized tri ls o only 906 p tients.
B) B riu ene . While bowel h bits c n be success ully ch nged with bulking
C) Anorect l no etry. gents or constip tion or loper ide or di rrhe -predo in nt
D) SH level. IBS, p in is gener lly not ected by these e sures. In ct,
E) Colonoscopy. incre sed ber int ke y tr nsiently worsen so e sy pto s
due to er ent tion nd gener tion o g s, potenti lly resulting
Answer 7.7.1 The correct answer is “D.” T e p tient’s present - in tulence or blo ting. “E” requires speci l ention. Bliss nd
tion with p in nd constip tion eets criteri or constip tion- enlighten ent c n only occur with proper colon purging pro-
predo in nt irrit ble bowel syndro e (IBS) (see ble 7-3). O cedures provided t very expensive resort!
note, irrit ble bowel is not n unco on co plic tion o GI
in ections including E. coli, Campylobacter, vir l in ections, nd Question 7.7.3 Which o the ollowing has the best record
Giardia (Gut. 2010;59:605). T ere is lso rel tionship between in treating IBS?
nxiety nd the presence o irrit ble bowel. T is p tient under- A) Eli in ting FODMAPs ro the diet.
went colonoscopy or screening 2 ye rs prior to present tion, B) Probiotics.
CHAPTER 7 • GAs TRo En TERo Lo GY 213

C) Ri ixin. and physical examination are unremarkable. Laboratory


D) St. John’s wort. tests reveal hemoglobin o 11.5 g/dL and a normal glucose
E) Eli in ting the use o GOOGLEMAPS . and SH.

Answer 7.7.3 The correct answer is “A.” First, you need to Question 7.8.1 What is the most appropriate next step in
know wh t FODMAPs re. FODMAP st nds or “Fer ent ble, your evaluation?
Oligo-, Di-, Monos cch rides, And Polyols.” T ese re poorly A) L ctose bre th test.
bsorbed c rbohydr tes th t c n c use er ent tion nd g s in B) Diet ry tri l o strict l ctose void nce.
the colon. So e FODMAPs include ructose, l ctose, ruct ns C) Enteroclysis.
( ro whe t . . . think this is why everyone believes they h ve D) Colonoscopy.
gluten intoler nce?), nd others. Eli in ting FODMAPs ro E) issue tr nsglut in se ntibody titer.
the diet see s to h ve the best NN o ny o the irrit ble bowel
Answer 7.8.1 The correct answer is “B.” T e p tient’s sy p-
tre t ents. O the others, probiotics h ve NN o 7 to 11 (Am
to s re consistent with l ctose intoler nce. He y h ve l-
J Gastroenterol. 2014;109:1547), ri xi in ( t $1,200/ onth)
bsorption second ry to l ctose intoler nce, which y be p r-
h s NN o 11 nd rgin l bene t t best (N Engl J Med.
ti lly responsible or his ne i . I his sy pto s resolve with
2011;364:22–32), nd St. John’s wort ctu lly incre ses the sy p-
tri l o l ctose- ree diet, you h ve your di gnosis. Follow-up
to s o IBS nd should be voided. Use o GOOGLEMAPS h s
should include re-checking his CBC nd perh ps other studies,
not been shown to h ve ny e ect on IBS sy pto s side ro
such s stool or occult blood. I these sy pto s re new, he
perh ps decre sing nxiety nd sy pto s rel ted to it.
could h ve second ry or o l ct se de ciency (e.g., Crohn
dise se or b cteri l overgrowth) or coincident l proble with
HELPFUL TIP: the bsorption o this c rbohydr te. Colon c ncer is not co -
Ab ut 4% childre diag ed with IBs wh are re- on t this ge but endoscopy should lso be considered i ne-
erred t a academic GI cli ic have celiac di ea e. The i does not resolve.
umber will be le i the ge eral patie t p pulati
(JAMA Pediatr. 2014;168(6):514–515). T e patient tries a lactose- ree diet, but this is o no bene t
and his abdominal cramping and diarrhea continue. At this
point, you are considering additional diagnoses.
HELPFUL TIP:
IBs ha a relatively g d pr g i . Up t 60% pa- Question 7.8.2 Which o the ollowing is LEAST likely in this
tie t impr ve with placeb treatme t i trial , maki g patient?
it hard t h w be e it with a y treatme t. There i A) B cteri l overgrowth syndro e.
i crea e i m rtality with IBs , a d ympt m u ually B) Gluten-sensitive enterop thy.
impr ve ver time. C) Giardia lamblia.
D) C. di cile.
E) Whipple dise se.
HELPFUL TIP:
s me patie t with c tipati -pred mi a t IBs will Answer 7.8.2 The correct answer is “E.” It is unlikely th t this
re p d t ral s s RI medicati uch a lu xeti e r p tient h s Whipple dise se, which is result o n in ection with
par xeti e that eem t i crea e b wel tra it time . Tropheryma whippelii. Whipple dise se is ssoci ted with nonde-
The data i till ketchy, h wever. A “n 1 trial” i rea- or ing rthritis, weight loss, ever, di rrhe , etc. All o the oth-
able, h wever. ers re possibilities in this p tient. “A,” b cteri l overgrowth syn-
dro e, presents with blo ting, di rrhe , dyspepsi , nd possible
Objectives: Did you learn to . . . l bsorption nd weight loss. It c n occur s result o bowel
• Diag e a d ma age IBs? dys otility, bowel redund ncy or diverticul , chronic p ncre ti-
• The r le Fo DMAP i irritable b wel? tis, etc. “B,” gluten-sensitive enterop thy (nontropic l or celi c
• Realize that b th y u a d y ur patie t will be ru trated with sprue), presents with sy pto s si il r to those in our p tient
the treatme t r IBs ? including l bsorption. “C,” Giardia in ection, c n be chronic
nd presents with g s, di rrhe , nd occ sion lly constip tion.
CASE 7.8 Fin lly, C. di cile in ection (pseudo e br nous colitis) c n
lso be chronic in n ture with chronic di rrhe nd blood loss.
A 32-year-old male complains about atigue and episodic
abdominal pain. His pain is located in the periumbilical HELPFUL TIP:
region and the le lower quadrant. It is cramp-like in nature Giardia i a c mm path ge that may be c tracted
and is associated with atulence and diarrhea. Passing gas i the day care etti g. s ympt m may c i t diar-
alleviates his symptoms. He notes that milk and other dairy rhea, malai e, a d au ea. The treatme t ch ice i
products worsen his symptoms. His weight has remained metr idaz le.
stable. His prior medical history, amily and social history,
214 FAMILY MEDICIn E EXAMIn ATIo n & Bo ARD REVIEW

Answer 7.8.4 The correct answer is “B.” issue tr nsglut -


You continue to work up this patient’s diarrhea with stool cul-
in se ntibodies re sensitive nd speci c or severe gluten-
tures, stool or C. dif cile toxin, direct immuno uorescence
sensitive enterop thy but y be lsely neg tive in ild-
analysis or Giardia and Cryptosporidium, and three stools or
to- oder te c ses. Antiendo ysi l IgA ntibodies re less
ova and parasites. All o the studies are negative. You now turn
sensitive th n tissue tr nsglut in se ntibodies or gluten-
your attention to possible bacterial overgrowth syndrome.
sensitive enterop thy but very speci c or gluten-sensitive enter-
op thy. Antigli din ntibodies re less sensitive nd speci c nd
Question 7.8.3 The best test(s) or bacterial overgrowth
h ve llen out o vor. T e de nitive test (“gold st nd rd”) is
syndrome is (are):
s ll bowel biopsy nd should be considered i your clinic l sus-
A) Qu ntit tive stool culture.
picion is high nd the p tient h s neg tive ntibodies. P tients
B) Stool leukocytes.
with positive ntibodies should lso h ve n endoscopy; the
C) 72-hour ec l t.
di gnosis should be con r ed by endoscopy nd biopsy be ore
D) [14-C] d-xylose bre th test.
co itting the p tient to gluten- ree diet or li e.
E) B nd D.

Answer 7.8.3 The correct answer is “D.” T e d-xylose bre th HELPFUL TIP:
test t kes dv nt ge o the ct th t the b cteri responsible Up t 1:200 Cauca ia livi g i the U ited s tate may
or b cteri l overgrowth syndro e (Gr -neg tive erobes) be a ected with glute - e itive e ter pathy (while
c t bolize d-xylose. T e bre th test e sures r dio ctive CO2 1:20 gr cery t re ai le appear t be dev ted t glute -
th t is or ed s result o b cteri l bre kdown o r dio ctive ree pr duct ).
d-xylose. T e glucose bre th test, in which glucose is d in-
istered to the p tient nd bre th hydrogen is e sured, is lso
help ul nd co only used. However, it is less sensitive nd
speci c th n the d-xylose test nd h s 30% to 40% lse- HELPFUL TIP:
neg tive r te. Work up o b cteri l overgrowth syndro e Remember Fo DMAP r m earlier? s tudie patie t
should lso include n upper GI endoscopy nd possible s ll wh believe that they have glute e itive e ter pa-
bowel biopsy. GI hypo otility, s ll bowel dil t tion, or s ll thy h w that elimi ati g Fo DMAP will impr ve the
bowel diverticul support the di gnosis o b cteri l overgrowth ympt m ma y the e patie t (Ga tr e ter l gy.
syndro e. O the others, “B,” stool leukocytes re pretty useless 2013;145:320). The culprit here i t the wh le wheat
in gener l nd do not h ve good correl tion with in ectious bread but the eve Big-Gulp-Fruct e-Lade p p
c uses o di rrhe (high lse-positive nd lse-neg tive r tes). the patie t dri k each day. T be air, wheat ha Fo D-
“C,” ec l t collection, is use ul in docu enting t l bsorp- MAP , wheat, but t glute e itivity, may make
tion syndro es, including those second ry to severe p ncre tic ympt m w r e i the e patie t . Di erentiating glu-
insuf ciency (e.g., cystic brosis with > 90% p ncre tic dys- ten sensitive enteropathy rom FODMAP-induced
unction) nd short bowel syndro e. symptoms is important. Keep reading . . .

HELPFUL TIP:
A ther pti t diag e bacterial vergr wth i T e results o his tissue transglutaminase test are positive.
empiric treatme t r 7 t 10 day with medicati t You educate him about gluten-sensitive enteropathy (celiac
c ver aer be a d a aer be (cephalexi plu metr - disease) and have an endoscopy per ormed. A small bowel
idaz le, TMP/s MX plu metr idaz le, am xicilli / biopsy demonstrates blunted villi with a signi cant increase in
clavula ate). Ri amixi ca be u ed but i expe ive. intraepithelial lymphocytes, consistent with gluten-sensitive
De i itive treatme t may require urgical i terve ti enteropathy. With a gluten- ree diet, the patient experiences
t h rte the b wel, re ect diverticula, etc. a signi cant increase in his energy level. wo years later he
comes or a routine visit. He has gradually reintroduced
some wheat products into his diet and tolerates this very well.
T is patient’s d-xylose test is negative and you consider the
possibility o gluten-sensitive enteropathy (celiac disease). Question 7.8.5 What do you recommend?
A) Resu e gluten- ree diet.
Question 7.8.4 O the ollowing, the one BEST test or the B) Continue diet ry ch llenge nd repe t ex in tion in
diagnosis o gluten sensitive enteropathy is: 6 onths.
A) Antiendo ysi l ntibodies. C) Repe t s ll bowel biopsy.
B) issue tr nsglut in se ntibodies. D) Check tissue tr nsglut in se ntibody titer.
C) Antigli din ntibodies. E) Avoid whe t, but try b rley or rye products.
D) R diol beled whe t our bsorption test.
E) All o the bove re equ lly ccur te or detecting celi c Answer 7.8.5 The correct answer is “A.” While there is no good
dise se. d t on the bene t o long-ter gluten- ree diet in p tients
CHAPTER 7 • GAs TRo En TERo Lo GY 215

who c n toler te s ll ounts o gluten, sever l ctors rgue TABLE 7-4 DIFFERENTIAL DIAGNOSIS FOR DIARRHEA
or continuing gluten- ree diet. T e rst is th t ny p tients
Acute Diarrhea
will h ve subclinic l nutrient de ciencies i they reintroduce
• Bacteria (e.g., Campylobacter, Salmonella, a d Clostridium)
gluten. For this re son, p tients with gluten-sensitive enterop -
• Viru e (e.g., n rwalk a d r taviru )
thy should be screened or osteoporosis nd t ke ultivit - • Para ite (e.g., Giardia)
in. T is is especi lly true in pedi tric p tients where de cien- • Pr t z a (e pecially i HIV-i ected patie t )
cies y le d to stunted growth. T e second is th t there is so e • Medicati
d t th t p tients who reintroduce gluten y h ve incre sed • A ythi g that cau e chr ic diarrhea
ort lity ro GI ly pho despite the ct th t they re tol- Chronic Diarrhea
er ting the gluten well (Ann Intern Med. 2013;159:169). • I f ammat ry (e.g., IBD a d radiati e teriti )
• o m tic (e.g., glute - e itive e ter pathy a d lacta e de cie cy)
• s ecret ry (e.g., Z lli ger–Elli y dr me a d vill u ade ma)
Question 7.8.6 All o the ollowing are gluten ree grains • Di rdered m tility (e.g., irritable b wel y dr me a d verf w
EXCEPT: r m ecal impacti )
A) O ts.
B) Corn.
Question 7.9.1 What is the most likely diagnosis?
C) Sticky rice.
A) Celi c sprue.
D) Rice.
B) Vir l g stroenteritis.
E) Millet.
C) L ctose intoler nce.
D) S ll bowel b cteri l overgrowth.
Answer 7.8.6 The correct answer is “C.” Sticky rice is lso
E) C. di cile colitis.
known s “glutinous rice” nd cont ins gluten. T e rest re glu-
ten- ree. Other gluten- ree gr ins/st rches include buckwhe t, Answer 7.9.1 The correct answer is “B.” T is is lw ys
x, quino , nd t pioc . dile . It c n be dif cult to di erenti te cute g stroenteritis
ro ood poisoning. Wh t kes g stroenteritis ore likely
HELPFUL TIP: is th t there is no clustering o c ses ong people who te the
With a glute - ree diet, a tib die (ti ue tra gluta- s e ood. With only 3 d ys o sy pto s, it would be pre -
mi a e, a tigliadi , a tie d my ial) te retur t ture to di gnose “A,” “C,” or “D.” Without recent ntibiotic expo-
rmal level . sure, “E” would be unlikely in n otherwise he lthy le. See
ble 7-4 or di erenti l di gnosis or di rrhe .

Question 7.9.2 What is the MOST appropriate next step in


HELPFUL TIP: the care o this patient at this point?
Patie t with glute - e itive e ter pathy mu t be A) Order CBC.
c mpul ive ab ut their diet. Rice, c r , a d ybea - B) Order electrolytes.
ba ed l ur are a e t c ume. o at are te c - C) Order stool ex in tion or ov nd p r sites.
tami ated with wheat. D) Reco end or l hydr tion nd ntidi rrhe ls s needed.
E) Order n bdo in l l .

Objectives: Did you learn to . . . Answer 7.9.2 The correct answer is “D.” No workup is needed
• Evaluate a patie t with diarrhea? or ild c se o cute di rrhe , since such c ses re usu lly sel -
• Rec g ize cli ical ma i e tati glute - e itive e ter- li ited. Gener lly, the history nd physic l ex in tion should
pathy? provide the di gnosis nd indic te need or urther workup. Fur-
• Ma age a patie t with glute - e itive e ter pathy? ther workup nd tre t ent re indic ted i the p tient h s severe
or bloody di rrhe , dehydr tion, syste ic toxicity, or severe p in.
CASE 7.9
HELPFUL TIP:
A 22-year-old previously healthy male reports a 3-day his- Patie t with evere, bl dy diarrhea, a d ever
tory o explosive and watery diarrhea. He is having up to 6 (T > 101°F, > 6 t l per day) h uld be treated with a
bowel movements per day. He recalls eating at a new Mexi- 3- t 5-day c ur e a lu r qui l e while awaiti g
can restaurant 5 days ago. His head sinks a little low as he t l culture re ult . Bloody diarrhea in an a ebrile pa-
recalls drinking a “ sh bowl”-sized margarita . . . or at least tient should suggest Escherichia coli 0157:H7. The e pa-
he thinks he remembers drinking it! He denies ever, blood in tie t h uld not get a tibi tic becau e a i crea ed
his stool, or recent travel. Multiple people ate the same ood ri k hem lytic–uremic y dr me. A alway , there i a
but he is the only one who is sick. His vital signs are normal “except.” Patie t wh have bee traveli g t area with
(including supine and standing blood pressures), and the re i ta t rga i m h uld be treated with azithr myci
remainder o the physical examination is remarkable only or 500 mg daily r 3 day . Thi i clude m t A ia.
mild, dif use abdominal tenderness.
216 FAMILY MEDICIn E EXAMIn ATIo n & Bo ARD REVIEW

You treat the patient with oral rehydration and prochlor- HELPFUL TIP:
perazine or some nausea and he does well. He returns to see Kleb iella xyt ca ca pr duce a a tibi tic-related di-
you a ew weeks later a er a trip to Mexico. He liked that sh arrhea that i i di ti gui hable r m C. di icile . I th e
bowl-sized margarita so much that he decided to go or an with a egative t l r Cl tridium a tige , c ider
original. He has diarrhea that began a couple o days a er his (1) al e- egative te t ( me the ewer trai
arrival in Cancun and has now been present or 5 days. He Cl tridium are t detected by traditi al t l a ti-
has had requent, watery diarrhea with nausea but no vomit- ge ) r (2) K. xyt ca r (3) the C. difficile toxin degen-
ing. He has noticed no blood in the stool. He was very care ul erates rapidly at room temperature (no toxin may
to avoid salads and water but did have some ice in a so drink be detectable a ter 2 hours o exposure to room
and his margarita was “on the rocks.” temperatures) r (4) ELIs A te t ge erally ly te t r
Type A a tige . The patie t may have a a tige Type B
Question 7.9.3 The most likely organism causing illness in pr duci g trai or there may be a mutati a tige
this patient is: Type A.
A) Salmonella.
B) Shigella.
C) Enterotoxigenic E. coli. Objectives: Did you learn to . . .
D) Enterohe orrh gic E. coli. • Evaluate a patie t with acute diarrhea?
E) Campylobacter. • Treat acute diarrhea?
• Rec g ize di ere t bacterial cau e diarrhea?
Answer 7.9.3 The correct answer is “C.” Enterotoxigenic
E. coli is the ost co on c use o tr veler di rrhe in p tients CASE 7.10
tr veling to Mexico. Enterohemorrhagic E. coli is less likely nd
A 49-year-old man comes to your o ice, requesting test-
should be ssoci ted with bloody di rrhe . T e others re uch
ing or hepatitis C. He recently attended his 25-year col-
less likely to be c uses o tr veler di rrhe .
lege reunion where an old riend he had “partied” with
during experimentation with injectable drugs related that
HELPFUL TIP: he has cirrhosis due to hepatitis C. he patient is otherwise
Ma y phy icia w uld t treat a patie t with s higella healthy and denies any symptoms except or occasional
u le that patie t i cli ically ill (e.g., ever). Treatme t atigue a ter a long day at work. Physical examination
with a tibi tic i relatively c trai dicated i s alm - o the patient is unremarkable. here are no stigmata o
ella becau e it pr l g the carrier tate. H wever, u e chronic liver disease.
judgme t i the patie t wh i particularly ill.
Question 7.10.1 Which o the ollowing is the most appro
priate course o action?
Question 7.9.4 Which o the ollowing is/are appropriate or A) Check qu ntit tive Hep titis C virus (HCV) PCR (“vir l
the treatment o this patient’s traveler diarrhea (remember, lo d”).
he has no vomiting)? B) Order reco bin nt i unoblot ss y (RIBA).
A) Or l rehydr tion. C) Order HCV ntibody test (enzy e i uno ss y).
B) Antidi rrhe ls. D) Order qu lit tive HCV PCR.
C) E t ny ood (e.g., no need or bl nd diet with slow dv nce- E) Order AL nd AS .
ent).
D) All o the bove. Answer 7.10.1 The correct answer is “C.” T e sensitivity nd
E) None o the bove. speci city o the present-d y HCV ntibody test re excellent;
thus, this is the best test to per or in this situ tion. R rely,
Answer 7.9.4 The correct answer is “D.” T is p tient h s non- p tients with i unologic i p ir ent, such s HIV in ection,
bloody di rrhe nd no syste ic signs, so it should be s e to h ve HCV vire i without detect ble ntibody, but this would
tre t hi with ntidi rrhe l gents (e.g., loper ide) nd void not be concern in this otherwise he lthy p tient. Qu ntit tive
ntibiotics. Or l rehydr tion is the rule unless p tient is too HCV PCR is not reli ble e ns or di gnosing HCV in ection
n use ted or h s so e other re son th t he c nnot t ke dequ te bec use currently used ethods re insensitive t low levels o
uids by outh. Patients (including children) should eat anything vire i ; thus, in ection c nnot be ruled out i the level o HCV
they can tolerate. T e concept o “gut rest” is isno er nd vire i is below the lower li it o detection o the test. RIBA
ctu lly le ds to incre sed bowel per e bility nd ore persis- is n old test th t is no longer used. Qualitative HCV PCR is
tent di rrhe . Lactose deserves special mention. T e A eri- the ost sensitive test or the presence o HCV RNA, with
c n Ac de y o Pedi trics h s ch nged their reco end tions li it o detection th t is lower th n th t o qu ntit tive PCR.
bout l ctose in di rrhe . T ey reco end withholding l ctose It is use ul to est blish the presence o vire i , but is ore
only in children less th n 3 onths o ge. T is see s to be the expensive th n ntibody testing nd thus not rst-line test.
group in which tr nsient l ct se de ciency occurs. M ny p tients with chronic HCV in ection h ve nor l liver
CHAPTER 7 • GAs TRo En TERo Lo GY 217

enzy es nd c n still h ve progressive dise se; there ore, in


A er a ew phone calls, you nd that elastography is not avail-
high-risk p tient, AL nd AS re not ppropri te or screen-
able anywhere nearby. T e patient agrees to a liver biopsy to
ing or HCV.
diagnosis cirrhosis and you make the re erral. He is still very
concerned about his situation and asks what you think the
T e patient returns several weeks later to discuss his test
chances are that he already has cirrhosis.
results. His HCV antibody test is positive. A liver panel
obtained that day shows an AL o 48 IU/L (normal range,
Question 7.10.3 Regarding the development o progres
0–20) and an AS o 39 IU/L (0–31). His albumin and total
sive liver disease in hepatitis C, all o the ollowing are true
bilirubin are within normal limits. He is extremely anxious
EXCEPT:
about his liver.
A) Approxi tely 20% o p tients with chronic HCV in ection
will develop serious liver dise se.
Question 7.10.2 To most accurately assess the degree o
B) He vy lcohol use is risk ctor or develop ent o serious
liver disease, your next step is to:
liver dise se.
A) Obt in liver–spleen sc n to ssess or evidence o cirrhosis.
C) Acquisition o HCV in ection er the ge o 40 is ssoci ted
B) Re ssure the p tient th t his ild liver test bnor lities
with incre sed risk o developing serious liver dise se.
rule out cirrhosis.
D) HCV genotype ects the prob bility o developing end-
C) Order right upper qu dr nt ultr sound with Doppler to
st ge liver dise se.
ssess or evidence o cirrhosis.
E) M les re ore likely th n e les to develop serious liver
D) Order el stogr phy or evidence o cirrhosis.
dise se.
E) Obt in n bdo in l C to ssess or evidence o cirrhosis.
Answer 7.10.3 The correct answer is “D.” While only inor-
Answer 7.10.2 The correct answer is “D.” H ving est blished
ity o persons in ected with HCV develop serious liver dise se
th t the p tient h s hep titis C with elev ted liver enzy es,
(20%), the likelihood o progression is dif cult to predict in n
the next step is to deter ine the severity o his liver dise se.
individu l p tient. Nonetheless, le gender, he vy lcohol
Although his liver unction tests re re ssuring, this does
use, nd cquisition o HCV in ection er the ge o 40 re
not exclude the possibility o dv nced brosis or even well-
ssoci ted with incre sed risk o progressive liver dise se, while
co pens ted cirrhosis. While biopsy h s been the de nitive
genotype is not. In ddition, J p nese ncestry, s oking (both
test or liver brosis in the p st ( nd you would not be wrong
cig rettes nd riju n ), nd cquiring hep titis C ro
to order biopsy i it were n option in this question), ultr -
blood tr ns usion, h ve been ssoci ted with n incre sed risk
sound el stogr phy, noninv sive technique o deter ining
o progression. T e genotype o the hep titis C cert inly kes
liver brosis, is being used ore nd ore. T e ide is th t
di erence when it co es to tre t ent but not with progres-
brotic liver is sti liver. Don’t worry bout the det ils; just
sion to cirrhosis.
know th t el stogr phy ( or ex ple FibroSc n) is noninv -
sive nd ccur te ethod or deter ining the degree o hep tic
brosis. T e “F” score, discussed below, should llow you do HELPFUL TIP:
ke decisions bout your hep titis C p tients. You would not Patie t wh have a per i te tly rmal ALT, acquired
be incorrect to order ultr sound i ging s well. hepatiti C be re the age 35, are emale, d t dri k
alc h l, a d have mi imal r ibr i liver bi p y
HELPFUL TIP: are u likely t pr gre t e d- tage liver di ea e.
Fibr i i c red r m F0 t F4, with F0 repre e ti g
ibr i a d F4 repre e ti g cirrh i . I additi t
HELPFUL TIP:
ela t graphy, a ther i va ive meth d deter-
The Us Ps TF rec mme d hepatiti C cree i g ce
mi i g the degree ibr i i the AsT/platelet rati
r all i dividual b r betwee 1945 a d 1965. o ther
i dex, abbreviated APRI. Thi i calculated a ll w :
p pulati de ervi g cree i g i clude th e with
APRI = [(patie t AsT - upper rmal lab As T)/ HIV, dialy i patie t , a d i carcerated patie t .
platelet c u t] × 100.

s ig i ica t ibr i (F2–F4) i pre e t i the APRI i 0.7 r


greater (77% e itivity, peci icity 72%) alth ugh me T e patient is concerned that he may transmit the virus to his
urce u e 0.5 a the l wer limit r “ ig i ica t ibr i .” wi e or children. T ey are tested and are ound to be negative
A APRI 1 r greater repre e t cirrh i . Fi ally, the or HCV antibody. He is relieved but asks or advice to pre-
Fibr s ure, Fibr te t, a d ActiTe t are pr prietary meth- vent in ecting them.
d determi i g liver ibr i —all which have ep-
arate c ri g y tem r ibr i . n te that a bi p y i Question 7.10.4 You advise all o the ollowing EXCEPT:
t ece ary r maki g a deci i ab ut wh t treat: A) No ch nge in sexu l pr ctices is reco ended or couples
m re thi bel w. in long-ter onog ous rel tionship in which one p rt-
ner is HCV+ nd the other HCV− .
218 FAMILY MEDICIn E EXAMIn ATIo n & Bo ARD REVIEW

B) T e use o condo s is reco ended or couples in long- Question 7.10.6 You tell him which o the ollowing?
ter onog ous rel tionship in which one p rtner is A) Co bin tion ther py with inter eron nd rib virin results in
HCV+ nd the other HCV− . sust ined virologic responses (SVR) in 40% to 70% o p tients
C) Hep titis C is not spre d by hugging, sneezing, or sh ring tre ted.
drinking gl ss. B) Co bin tion ther py with inter eron nd rib virin c n
D) Household e bers o persons in ected with HCV should c use nu erous side e ects including cytopeni s, u-like
not sh re ite s th t ight be cont in ted with s ll sy pto s, worsening o utoi une conditions, depres-
ounts o blood such s r zors or n il clippers. sion, nd he olytic ne i .
E) P renter l exposure to in ected blood is jor route o C) T e HCV genotype is strong predictor o response to the
tr ns ission o HCV. tre t ent.
D) Newer tre t ent regi ens including ledip svir/so osbuvir
Answer 7.10.4 The correct answer is “B.” HCV is spre d by re uch ore e ective th n the tr dition co bin tion o
p renter l cont ct with in ected blood. In contr st to hep titis inter eron/rib virin.
B, sexu l tr ns ission o HCV is inef cient nd ppe rs to be E) All o the bove.
inor route o spre d. In ddition, the ef c cy o l tex con-
do s in preventing dise se is not known. T e NIH and the US
Public Health Service do not recommend condom use or Answer 7.10.6 The correct answer is “E.” Co bin tion ther-
patients in a stable, long-term, and monogamous relation- py with inter eron nd rib virin is the tr dition l tre t ent
ship. T at said, using condoms will likely reduce an already o HCV nd it is cle rly not s e ective t chieving SVR s
low risk even urther. the newer drugs. T e HCV genotype is jor ctor deter-
ining the likelihood o chieving SVR, lthough not the
T e patient’s liver biopsy shows mild-to-moderate in am- likelihood o progression to end-st ge liver dise se. Genotype
matory activity and portal and periportal brosis (stage 2). 1 is responsive to the newer drugs, such s the co bin tion
He is relieved to nd out that he does not have cirrhosis, but o ledip svir/so osbuvir (Harvoni); n entirely or l regi en
remains very concerned about his hepatitis and wants to do with response r tes o 93% or better or genotype 1. A second
everything possible to “get rid o ” the hepatitis C. He asks entirely or l tre t ent is o bit svir/p rit previr/riton vir/
about the treatment o his HCV. d s buvir (Viekira Pak [U.S.]). However, it ust be co bined
with rib virin or genotypes 1 nd 1b i cirrhosis is present.
Question 7.10.5 Which o the ollowing is NOT an indication St ge o brosis is lso ctor, nd p tients with st ges 3 nd
or the treatment o hepatitis C? 4 (bridging brosis nd cirrhosis) re less likely to chieve
A) El stogr phy showing cirrhosis. SVR. Higher b seline vir l levels lso tend to predict poorer
B) APRI o 0.25 (pre-cirrhosis). response to the tre t ent. Co bin tion ther py is expensive
C) Liver biopsy showing bridging brosis. nd is ssoci ted with signi c nt toxicities. T e jor con-
D) Ability to p y or the tre t ent. cerns with rib virin re he olytic ne i nd ter togenicity,
while the inter erons (st nd rd or the long- cting pegyl ted
Answer 7.10.5 The correct answer (and not an indication or s) h ve long list o potenti l side e ects, o which neu-
or hepatitis C treatment) is “B.” T e cuto APRI or the tre t- ropsychi tric proble s, such s depression nd irrit bility, re
ent is 0.5 to 0.7, representing F2 brosis. F2 brosis di gnosed o en the ost troubleso e. Al ost no tre t ent regi en still
by ny ethod, including biopsy or el stogr phy (e.g., Fibro- requires inter eron.
Sure), is n indic tion or the tre t ent. You y be wonder- OK, now the g e ch ngers. Here is t ble ( ble 7-5) or
ing bout “D.” St rting in 2014, w ve o new nd ore tol- you to review the new hep titis C drugs nd the genotype tre ted.
er ble hep titis C tre t ents st rted to beco e co erci lly T is is pretty b sic. For ore co plete su ry, see http://
v il ble. However, the cost o these new drugs— t le st, in www.hep titis.v .gov/pd /tre t ent-consider tions-2015–07.
the United St tes—is i or lly high, costing up to $100,000 pd . Side e ects o the newer drugs re he d che, GI upset, etc.,
or co plete 12-week tre t ent. Public nd priv te insurers nd gener lly uch less proble tic th n previous gener tions
h ve been orced into outright r tioning. ( o beco e ppropri- o hep titis C tre t ent. Wh t h ppened to the prote se inhibi-
tely outr ged, re d this rticle: http://www.l ti es.co /busi- tors (tel previr nd boceprevir), you sk? T ey h ve gone the
ness/hiltzik/l - -hiltzik-20150621-colu n.ht l.) I the United w y o the dodo bird nd re no ore . . . too ny side e ects
St tes tre ted ll o the pproxi tely 3 illion HCV in ected when used or hep titis C.
persons with the new drugs, it would cost our country bout
$300 billion. Bec use o the ph r ceutic l co p nies setting Objectives: Did you learn to . . .
prices di erently in di erent rkets, t le st one o these new • Evaluate a patie t at ri k r hepatiti C?
HCV drugs costs bout 1% o the U.S. price in Egypt. So . . . One • U der ta d the atural hi t ry the di ea e pr ce i
option is to send your HCV p tients to Egypt or the tre t- hepatiti C?
ent nd put the up in nice hotel—it would be signi c ntly • De cribe the tra mi i hepatiti C?
che per. • Di cu the treatme t i ue r a patie t with hepatiti C?
CHAPTER 7 • GAs TRo En TERo Lo GY 219

TABLE 7-5 REGIMENS FOR HEPATITIS Ca


Genotype and Primary Response
Primary Regimens Regimens Rate (%) Note 1 Notes
a
All Ge type 1 Harv i (Ledipa vir/ buvir) 95 With ut Duration o treatment di er ba ed cirrh i v .
r Viekira Pak ( mbita vir/ cirrh i cirrh i , ailure pri r treatme t, etc.
paritaprevir/rit avir/ da abuvir) Note the oral only regimens: Harv i r all
ge type 1 a d ge type 4
Ge type 1a a d 1b Harv i r Viekira Pak plus 95 With Viekira pack r ge type 1 with ut cirrh i .
ribaviri cirrh i

Ge type 2 s bu r (s valdi) plus ribaviri 97

Ge type 3 Harv i plus ribaviri 98

Ge type 4 Viekira Pak plus ribaviri 95


Harv i
s valdi plus ribaviri

a
n te that we are u i g trade ame i thi i ta ce a ter r t me ti . Thi g ju t get c u i g therwi e . . . . . .
n te that Viekira Pak a d Tech ivie w have a war i g ab ut u e i th e with evere u derlyi g liver di ea e: they ca cau e liver i jury.
Ki d a catch 22 . . . . .

CASE 7.11 possibility o chronic hep titis B lso w rr nts speci l ttention
in this p tient.
A 24-year-old emale graduate student rom China comes to
your o ce complaining o atigue or the past month. She Question 7.11.2 Appropriate laboratory studies at this
has also had a poor appetite and has lost about 3 lb over this point include which o the ollowing?
period. She reports that she was told that she had “hepati- A) Qu ntit tive HCV PCR.
tis” when she was about 10 years old but does not recall what B) Hep titis B sur ce ntigen (HBsAg).
type. She is otherwise healthy and takes no medications. She C) Anti-hep titis A ntibodies (IgG nd IgM).
has no history or percutaneous exposures or blood trans u- D) B nd C.
sion. Her grand ather died o liver cancer. E) All o the bove.
Physical examination reveals a thin, tired-appearing
woman. T e liver edge is palpable 2 cm below the right costal Answer 7.11.2 The correct answer is “D.” As discussed previ-
margin and is slightly tender. T ere is no ascites, splenomeg- ously, the qu ntit tive HCV PCR is not use ul test or di g-
aly, or cutaneous stigmata o chronic liver disease. nosing HCV in ection. HCV ntibody testing would be better
Laboratory studies are remarkable or anemia (hemoglo- choice. Both HBsAg nd nti-HAV re use ul tests in this p tient.
bin 9.1 g/dL). Liver tests reveal elevated aminotrans erases HBsAg is help ul to ssess or HBV in ection ( cute nd chronic)
(AL 289 IU/L, AS 158 IU/L), albumin 3.2 g/dL, and total nd nti-HAV ntibodies will rule out cute hep titis A in ec-
bilirubin 1.5 mg/dL (normal 0.2–1.0 mg/dL). tion. A positive tot l nti-HAV (positive IgG) with neg tive IgM
would indic te p st in ection, while positive IgM would suggest
Question 7.11.1 Diagnostic possibilities at this point cute HAV in ection. Interpreting the HBV ntigens (Ag) nd
include: ntibodies (Ab) c n be con using, nd ble 7-6 y help.
A) Hep titis A.
B) Hep titis B. T e patient’s results show a positive HbsAg, indicating ongo-
C) Hep titis C. ing hepatitis B in ection. She is immune to hepatitis A and is
D) Autoi une hep titis. hepatitis C negative.
E) All o the bove. A er being out o contact or 4 months, she returns or a
ollow-up visit. She tells you that she took an herbal medicine
Answer 7.11.1 The correct answer is “E.” Constitution l sy p- her mother sent rom China and has been eeling much better
to s such s tigue nd norexi c n be seen with ny or recently. Her HBsAg remains positive, but her liver enzymes,
o cute or chronic liver dise se; thus, they re not help ul in albumin, and total bilirubin are now completely normal.
est blishing speci c di gnosis. T e rst priority is to rule out
in ectious hep titis including hep titis A, B, nd cute hep ti- Question 7.11.3 Appropriate actions at this time include:
tis C. Autoi une hep titis deserves consider tion, p rticu- A) re t ent with inter eron- lph 5 illion units d ily or
l rly in e le p tients. While HCV in ection is worldwide 16 weeks.
proble , HBV in ection is ende ic in Asi nd A ric , nd the B) Order hep titis Be ntigen, nti-HBe, nd HBV DNA level.
220 FAMILY MEDICIn E EXAMIn ATIo n & Bo ARD REVIEW

TABLE 7-6 HEPATITIS B VIRAL SEROLOGIES FOR DIFFERENT PHASES


OF INFECTION
Antigen/Antibody Acute Chronic Recovered Vaccinated/Immune
HB Ag + + − −

HBeAg + + − −

A ti-HB Ab − − + +

A ti-HBcAb + (IgM) + (IgG) + (IgG) −

A ti-HBeAb − + /− + −

HBV Dn A + + /− + /− −

Ab, a tib dy; Ag, a tige ; HBc, c re a tige r a tib dy; HBe, “e” a tige r a tib dy; HB , ur ace a tige
r a tib dy.

C) Begin periodic screening or hep tocellul r c rcino B) I her ncé h s not been i unized g inst HBV, he should
(HCC) with ultr sound nd lph - etoprotein (AFP). be tested nd v ccin ted i not i une.
D) A nd C. C) I her ncé is not i une to HBV, they should use b rrier
E) B nd C. contr ceptives (e.g., condo s) until he h s co pleted his
HBV v ccin tion series.
Answer 7.11.3 The correct answer is “E.” T e HBe ntigen D) She should cover ny open cuts or scr tches with b nd ge
re ects vir l replic tion. Loss o HBeAg indic tes decre sed nd cle n up ny blood spills with ble ch.
vir l replic tion nd less o risk o progression to cirrhosis. E) Ad inistr tion o hep titis B i une globulin (HBIG) nd
Loss o HBeAg y occur spont neously; it is lso the ther - HBV v ccin tion begun i edi tely er birth is 95% e -
peutic endpoint o the ntivir l tre t ents o HBV in ection ective in preventing perin t l tr ns ission o HBV.
(inter eron, l ivudine, de ovir). I she is neg tive or HBeAg
nd is nti-HBe ntibody positive ( nti-HBe + ) or h s low or Answer 7.11.4 The correct answer is “A.” Although p tients
undetect ble levels o HBV DNA, she h s low level o vir l with higher levels o HBV DNA re ore in ectious th n those
replic tion nd will not bene t urther ro the ntivir l tre t- with lower levels o vir l DNA, the risk o tr ns ission in the
ent. l tter c se is not zero. In the c se o this p tient, h ving “unde-
Her liver p nel should be onitored periodic lly s should tect ble” HBV DNA si ply indic tes level o HBV DNA th t
the AFP nd ultr sound. Ultr sound is reco ended every lls below the li it o detection o n un pli ed ss y (on
6 onths + /– AFP. An AFP lone should not be done bec use the order o 105 copies/ L). Also, the positive HbsAg e ns
o the poor sensitivity nd speci city. Even sy pto tic HBV she is still c rrier. T us, prec utions should be t ken to pre-
c rriers with ini l liver dise se re t risk or HCC. Screen- vent sexu l or household tr ns ission to her ncé (use o con-
ing or HCC in those who re c rriers o hep titis B is indic ted do s, i uniz tion i required, etc.) nd to her uture children
in: Asi ns ( en > 40 ye rs old, wo en > 50 ye rs old); those (HBIG nd HBV v ccin tion).
with ily history o HCC; cirrhotic p tients; nd those o
A ric n ncestry > 20 ye rs o ge. Our p tient h d positive
VERY HELPFUL (IF LONG) TIP:
ily history o HCC, so onitoring or HCC is indic ted. ALT and AST can be elevated secondary to:
• Viral agents: Hepatiti (A, B, C, D, E), CMV, Ep tei –
T e patient returns to discuss the results o her tests. Labora- Barr viru , a d ther viru e .
tory results show that she is positive or HBsAg and is anti-HBe • Drugs and chemicals: Acetami phe verd e, the
positive. Her HBV DNA is undetectable using an unampli ed “glitaz e ,” HMG-C A reducta e i hibit r , In H, gri -
assay, making her a carrier without evidence o viral replica- e ulvi , a tic vul a t , n s AID , chemical (carb
tion (no chronic, active, hepatitis B). AFP is within limits and tetrachl ride, etc.), alc h l, a d ma y ther age t .
abdominal ultrasound is unremarkable. She continues to eel • Primary liver diseases: Primary cler i g ch la -
well. She also tells you that she will be getting married in 2 giti , primary biliary cirrh i (p itive a timit ch -
months. She asks you what can be done to prevent her ancé drial a tib dy [AMA]).
and uture children rom becoming in ected with HBV. • Metabolic diseases: Wil di ea e (decrea ed
cerul pla mi ), hem chr mat i , alpha 1 a titryp i
Question 7.11.4 All o the ollowing are accurate responses de cie cy, a d cy tic br i .
to her question EXCEPT: • Mechanical di culties: Ductal b tructi ec d-
A) No speci l prec utions need to be t ken bec use she h s ary t c mm duct t e r carci ma (e pecially
undetect ble HBV nd is there ore not in ectious.
CHAPTER 7 • GAs TRo En TERo Lo GY 221

health and walked 3 miles per day. Now he is too weak and
pa creatic, hepat ma, meta tatic), Budd–Chiari y -
tired to even care or his own yard.
dr me (thr mb i the hepatic vei ).
His past medical history is remarkable or coronary artery
• Cholestasis r m ce tral ve u utriti (CVn ),
bypass surgery done 6 years ago. He also recalls having “yel-
preg a cy, r ce triax e therapy.
low jaundice” (as opposed to the purple kind?) when he was
• Inf ltrative processes: Fatty liver (e pecially th e
stationed in Vietnam. He has no signi cant amily history.
with diabete , hyp thyr idi m, be ity; determi e
He “drank a bit” on the weekends when he was in the service
by U/s ), amyl id, gra ul mat u hepatiti , liver
but has drunk very little alcohol in the past 50 years. He also
ab ce (i cludi g amebic r echi c ccal; diag -
quit smoking about 50 years ago. His medications are aspirin
i by U/s r CT; may have e i philia), AIDs -related
81 mg daily and ibupro en 600 mg as needed or knee pain
lymph ma, r ther e pla m.
due to degenerative joint disease.
• Other: C ge tive heart ailure, celiac prue, mu cle
di ea e (e.g., p lymy iti ).
Question 7.12.1 Diagnostic considerations suggested by
Alkaline phosphatase may be elevated secondary the history should include which o the ollowing?
to: A) Adverse drug re ction to penicillin.
• Preg a cy, a ter a atty meal i per with type o B) M lign ncy.
r B bl d. C) Cirrhosis.
• Liver: Ch le ta i , partial b tructi the biliary D) He rt ilure.
duct , primary cler i g ch la giti , adult bile duc- E) All o the bove.
t pe ia, primary biliary cirrh i , arc id i , a d
ther gra ul mat u di ea e. Answer 7.12.1 The correct answer is “E.” T e p tient’s history
• Bone diseases uch a Paget di ea e a d meta tatic o bdo in l nd lower extre ity swelling suggests uid over-
di ea e. lo d. Penicillin is known to c use interstiti l nephritis nd sec-
For elevated liver tests, workup should include (in ond ry nephrotic syndro e, which c n le d to uid retention.
approximate order, which may vary depending on Sources o uid overlo d besides the kidneys should be con-
patient presentation): sidered in this p tient, including liver nd he rt dise se. M lig-
• Rule ut t xi exp ure (alc h l, drug ). n ncy, especi lly with liver involve ent, c n c use scites. He is
• Hepatiti A, B, a d C er l gy. on n NSAID, which c n c use uid retention, lthough r rely
• s erum erriti , ir , TIBC, tra erri aturati to this degree.
(hem chr mat i ).
• Ultra u d r CT imagi g (ultra u d r t). Physical examination reveals a ragile-appearing elderly man
• An A a d a ti m th mu cle a tib dy (aut immu e with temporal wasting. T ere is no JVD. T e lungs are clear
hepatiti but ly 28–40% e itive). to auscultation. T e heart sounds are regular, with no mur-
• s erum alpha 1 a titryp i . murs or gallops. T e abdomen is protuberant with bulging
• s erum pr tei electr ph re i (elevated level i anks. Shi ing dullness is present. T ere is 2+ ankle edema
aut immu e hepatiti , 80% e itive). bilaterally and scattered telangiectasias on skin examination.
• A tie d my ial a tib die r ti ue tra glutami - He has no asterixis.
a e (glute - e itive prue).
• s erum cerul pla mi (Wil di ea e). Question 7.12.2 Which o the ollowing f ndings would you
expect on laboratory examination?
A) Elev ted he oglobin nd HC (17.5 g /dL nd 55%).
3.
Objectives: Did you learn to . . . B) Decre sed pl telet count o 80,000/
• Ge erate a di ere tial diag i r patie t with ab rmal C) Elev ted seru lbu in.
liver e zyme ? D) BUN/Cr r tio < 20.
• Ide ti y patie t at ri k r hepatiti B? E) All o the bove.
• U e the vari u hepatiti B a tige a d a tib die t deter-
mi e a patie t’ i ecti tatu ? Answer 7.12.2 The correct answer is “B.” T is p tient likely
• De cribe the r ute tra mi i hepatiti B? h s port l hypertension given his scites nd stig t o liver
• W rk-up elevated liver te t i a tepwi e a hi ? dise se (spider ngio t /tel ngiect si s). Blood is shi ed
tow rd the spleen bec use o the incre sed port l pressure (the
blood, like the rest o us, t kes the p th o le st resist nce).
CASE 7.12 Shunting o blood through the spleen results in thro bocy-
A 73-year-old man comes to your o ce, complaining o topeni through incre sed pl telet destruction. “A” is wrong
abdominal and ankle swelling, decreased energy, and poor since liver p tients o en h ve ne i . “C” is incorrect bec use
appetite or the past 2 months. He dates the onset o his liver p tients requently h ve low lbu in. And, “D,” BUN/
symptoms to a “reaction” to penicillin given or dental work. Cr r tio < 20 is ssoci ted with intrinsic kidney dise se (see
He says that be ore taking the penicillin, he was in excellent Ch pter 5).
222 FAMILY MEDICIn E EXAMIn ATIo n & Bo ARD REVIEW

Diagnostic paracentesis is per ormed and approximately HELPFUL TIP:


50 mL o clear light yellow uid are obtained. Ascites rom any cause o portal hypertension will
have a high SAAG. A ide r m cirrh i , p rtal hyper-
Question 7.12.3 Appropriate laboratory studies on the te i al may re ult r m chi t mia i , arc id-
ascitic uid include which o the ollowing? i , p rtal vei thr mb i (Budd–Chiari y dr me),
A) pH. c ge ital hepatic ibr i , heart ailure, myxedema, etc.
B) Albu in. Causes o a low SAAG include er iti r m c ective
C) L ct te. ti ue di rder , ephr tic y dr me, pa creatic-related
D) riglycerides. a cite , a d perit eal carci mat i am g ther .
E) All o the bove.

Answer 7.12.3 The correct answer is “B.” T e scitic uid lbu- Question 7.12.5 In addition to a complete evaluation to
in is needed to c lcul te the seru - scites lbu in gr dient determine the cause o his portal hypertension, which o
(SAAG), which is help ul to distinguish between scites result- the ollowing is/are appropriate action(s) at this time?
ing ro port l hypertension ro scites due to other c uses. A) Re er the p tient to nutritionist or instruction on low-
L ct te nd pH h ve been proposed s rkers or spont ne- sodiu diet.
ous b cteri l peritonitis (SBP) but h ve proven to be unreli ble. B) Discontinue ibupro en nd prescribe COX-2-selective
Me sure ent o triglycerides is use ul to con r chylous sci- inhibitor or rthritis.
tes; however, in the bsence o grossly ilky- ppe ring uid, C) Prescribe spironol ctone 100 g d ily nd urose ide
there is no need to per or this test. 40 g d ily.
D) A nd C.
HELPFUL TIP: E) A, B, nd C.
I additi t albumi , all a citic luid h uld be e t
r cell c u t a d culture . A cell c u t h wi g ≥ 250 Answer 7.12.5 The correct answer is “D.” T e initi l ppro ch
p lym rph uclear leuk cyte /mm 3 i pre umptive to the n ge ent o scites due to port l hypertension is
evide ce s BP a d ma date treatme t r uch. sodiu restriction nd diuretics. T e go l is 2-g sodiu diet
Gram tai i early u ele i ce a citic luid bacterial (which is very dif cult to ollow—just look t c n o soup!).
c u t are typically l w that bacteria are rarely ee T e jority o p tients will not h ve n dequ te response to
Gram tai . sodiu restriction lone, so it is re son ble to begin diuretics t
the outset. Spironol ctone, which is n ldosterone nt gonist,
is use ul bec use port l hypertension is st te o hyper ldoste-
Your patient’s laboratory tests reveal an ascitic uid total ronis . Spironol ctone tends to c use hyperk le i , n e ect
protein o 2.5 g/dL with an albumin o 1.9 g/dL. A liver panel th t c n be itig ted by the co d inistr tion o urose ide.
reveals normal aminotrans erases, normal bilirubin, serum Once-d ily doses re ppropri te or initi l ther py; djust-
albumin 3.3 g/dL, and alkaline phosphatase 147 IU/L. Elec- ents re de b sed on the p tient’s electrolytes, ren l unc-
trolytes, BUN, and creatinine are within normal limits. tion, nd response to the tre t ent. NSAIDs should be voided,
due to sodiu retention nd GI bleeding. COX-2-selective
Question 7.12.4 Which o the ollowing is the most accurate inhibitors have no advantage over nonselective NSAIDs in
interpretation o these results? this regard. Other ppro ches to the p tient’s knee p in should
A) T e SAAG is 1.7, which is consistent with port l hyperten- be considered, including intr - rticul r injections, cet ino-
sion s its c use. phen (up to 2 g d ily is likely s e in cirrhosis), nd n rcotics,
B) T e SAAG is 1.7, which rules out port l hypertension s its provided enceph lop thy is not proble .
c use.
C) T e SAAG is 1.4, which is consistent with port l hyperten-
sion s its c use. HELPFUL TIP (AND PET PEEVE):
D) T e SAAG is 1.4, which rules out port l hypertension s its D ’t u e n s AIDs i liver di ea e. The patie t may
c use. already have a c agul pathy a d a l w platelet c u t.
E) T e seru - scites gr dient con uses e, so I don’t w nt to They may have varice pr e t bleedi g. n s AIDs ca
do it. al cau e GI bleedi g a d are not a a e alter ative t
acetami phe .
Answer 7.12.4 The correct answer is “C.” T e SAAG c n tell us
whether the uid is tr nsud te or exud te. T e SAAG is si -
ply the di erence between the seru lbu in nd the scitic A ew months later, your patient comes to the emergency
uid lbu in, or 3.3 – 1.9 = 1.4, in this c se. A SAAG o ≥ 1.1 department “ eeling sick.” He complains o dif use abdomi-
indicates portal hypertension with 97%accuracy. Re e ber nal pain and swelling, stating that his abdomen eels “tense.”
this by re e bering th t high SAAG e ns high pressure in On examination, you nd a pale, uncom ortable male with a
the port l syste . temperature o 38.3°C. His other vital signs are normal. His
CHAPTER 7 • GAs TRo En TERo Lo GY 223

abdomen is tense, distended, and dif usely tender with hypo- Objectives: Did you learn to . . .
active bowel sounds. You per orm a paracentesis that shows • Ge erate a di ere tial diag i r a cite ?
400 polymorphonuclear leukocytes/mm3. • A alyze a citic f uid t determi e p te tial cau e a cite ?
• I itiate appr priate treatme t r a patie t with a cite ?
Question 7.12.6 What is the most appropriate next step in • Diag e a d ma age s BP?
the evaluation and treatment o this patient?
A) Disch rge to ho e with incre sed doses o spironol ctone
nd urose ide.
CASE 7.13
B) Disch rge to ho e with oxicillin nd the s e doses o A 42-year-old male with known hepatitis C who is also a
diuretics. heavy drinker presents to your o ce because o increasing
C) Per or l rge-volu e p r centesis or sy pto tic relie con usion. He hasn’t noticed much o anything (hey, most o
nd disch rge to ho e. his li e has been like this . . . ), but his amily states that he is
D) Ad it to the hospit l nd st rt IV ce ri xone. somewhat con used and on occasion di cult to wake up. He
E) Ad it to the hospit l nd pl ce peritone l tube or dr in- has a known history o end-stage liver disease. He is supposed
ge. to be on a low-protein diet but decided that it was time to
start the Atkin low-carbohydrate diet to “lose that gut” (he
Answer 7.12.6 The correct answer is “D.” T is p tient eets even ound himsel a “low carb” beer). So, he has increased
3
criteri or SBP (≥ 250 poly orphonucle r leukocytes/ his intake o protein.
in the scites uid) nd h s clinic l ndings to suggest in ec-
tion. T is p tient could beco e unst ble quickly, so disch rge Question 7.13.1 Which o the ollowing is NOT a common
ro the e ergency dep rt ent is not reco ended. Incre s- cause o hepatic encephalopathy?
ing doses o diuretics nd/or l rge-volu e p r centesis y be A) GI bleeding.
required, but these interventions should only be considered in B) Constip tion.
the setting o hospit l d ission. Bet -blockers, which y be C) High-c rbohydr te diet.
used or port l hypertension, should be stopped; ort lity is D) Sepsis.
incre sed in SBP with continued use. Bro d-spectru ntibiotics E) Alter tion in gut or .
re the st nd rd o c re or SBP, nd IV third-gener tion ceph -
losporins (e.g., ce ri xone, ce ot xi e) re typic lly the rst-line Answer 7.13.1 The correct answer is “C.” High-c rbohydr te
gents in hospit lized p tients. Levo ox cin is good second line diets re not ssoci ted with hep tic enceph lop thy; high-
gent but should not be used in those on levof oxacin prophylaxis protein diet is. Si il rly, GI bleed (“A”) delivers l rge protein
as resistance is likely. A oxicillin (“B”) is not suf cient. lo d to the GI tr ct. T us, any patient with hepatic encepha-
In ddition, lbu in in the dose o 1.5 g/kg on d y 1 ol- lopathy should be evaluated or a GI bleed. Other c uses o
lowed by 1 g/kg on d y 3 reduces the ch nce o ren l ilure, cute hep tic enceph lop thy include constip tion, sed tive use
which occurs in 30% to 40% o p tients with SBP. Albu in lso (e.g., benzodi zepines), nd hypok le ic et bolic lk losis.
i proves ort lity nd should be given to ll p tients with SBP.
Also, send the scitic uid or culture. Blood cultures, CBC,
HELPFUL TIP:
seru che istries nd P /P should be obt ined s well.
A elevated amm ia level i a ciated with hepatic
Consider i ging the bdo en with ultr sound or C . Further
e cephal pathy, alth ugh there i t a direct li ear
tests should be ordered s indic ted.
c rrelati betwee erum amm ia level a d me tal
tatu .
HELPFUL TIP:
The m t c mm rga i m i s BP i clude s trept -
c ccu p eum iae , E. c li, a d Kleb iella .
You decide to admit the patient to the hospital or treatment
o his hepatic encephalopathy.
HELPFUL TIP:
Flu r qui l e a d TMP/sMX have bee u ed pr - Question 7.13.2 Which o the ollowing IS NOT part o the
phylactically t decrea e the reque cy sBP epi de . treatment o hepatic encephalopathy?
Pr phylaxi i i dicated i a y e with cirrh i a d ei- A) Or l l ctulose.
ther (1) a epi de s BP or (2) a a citic luid pr tei B) Polyethylene glycol (e.g., GoLytely nd Mir -L x).
< 1.0 or (3) a active GI bleed. F r active GI bleed- C) Or l ntibiotics.
i g i th e with a cite , ce triax e, r r l xaci r D) Fluid nd electrolyte n ge ent.
7 day ha bee h w t reduce s BP a d m rtality. E) L ctulose ene s.
Consider s BP pr phylaxi i th e with a a cite pr -
tei < 1.5 g/dL i the Cr > 1.2 mg/dL r n a i < 130 mg/ Answer 7.13.2 The correct answer is “B.” Polyethylene glycol
dL, r the BUn > 25 mg/dL r the bilirubi i > 3 mg/dL. pl ys no role in the tre t ent o hep tic enceph lop thy. While
it would ke intuitive sense th t it would work since l ctulose
224 FAMILY MEDICIn E EXAMIn ATIo n & Bo ARD REVIEW

works, this is not the c se. T e ech nis o ction o l ctulose Question 7.13.4 Which o the options is/are indicated or
is dependent on b cteri l et bolis o l ctulose into l ctic this patient at the time o discharge, assuming he is hemo
nd cetic cids. T is reduces the pH o the colon le ding to dynamically stable?
precipit tion o non bsorb ble oni in the colon, which A) N dolol.
reduces seru oni levels. Enemas (soap suds, etc.), on B) Propr nolol.
the other hand, may help acutely by re oving colonic con- C) Pentoxi ylline.
tents. “C” is o p rticul r note. Or l ntibiotics like ri xi in, D) Vit in K.
etronid zole, or neo ycin c n be used when p tients do not E) All o the bove.
respond to l ctulose or when l ctulose is contr indic ted (e.g.,
severe di rrhe ). Ri xi in h s beco e the st nd rd in tre ting Answer 7.13.4 The correct answer is “E.” All o the bove re
this condition. indic ted in the urther tre t ent o this p tient. Nonselective
bet -blockers (“A” nd “B”) will reduce port l pressures, decre s-
T e patient does well on a regimen o lactulose and oral met- ing the risk o v rice l bleeding. It should go without s ying (but
ronidazole. we’ll s y it) th t only one bet -blocker should be prescribed, not
both. “C,” pentoxi ylline, reduces ort lity nd risk o develop-
Question 7.13.3 What other problems do you need to worry ing hep toren l syndro e when used cutely ( t le st in the rst
about in this patient? 4 weeks er n event o lcoholic hep titis). “E,” vit in K, is
A) Prolonged bleeding ti e. indic ted bec use vit in K de ciency is co only ound in
B) Elev ted P /INR. lcoholics with lnourish ent.
C) T ro bocytopeni .
D) A nd B. HELPFUL TIP:
E) All o the bove. Pred i l e (40 mg/day) r 28 day the tapered ver
2 t 4 week may have me be e it i evere alc h lic
Answer 7.13.3 The correct answer is “E.” P tients with end- hepatiti . Thi i c tr ver ial a ter id ca i crea e
st ge liver dise se tend to h ve l ck o vit in K-dependent ri k ep i a d m t alc h lic hepatiti patie t die
clotting ctors ( nd thus elev ted P /INR nd prolonged ep i . I rder t determi e which patie t with al-
bleeding ti es) nd h ve thro bocytopeni due to shunting o c h lic hepatiti are ca didate r c rtic ter id , cal-
blood ro the liver to the spl nchnic bed bec use o elev ted culate the discriminant unction (DF). simply mea ure
port l pressures. However, pl telet count o 50,000 is gener- pr thr mbi time a d t tal bilirubi a d plug the um-
lly considered dequ te or clotting. In ddition, it is likely th t ber i t a li e calculat r. I the DF ≥ 32, the patie t
there is pl telet dys unction in cirrhosis, lthough the clinic l may be e it r m c rtic ter id therapy.
signi c nce is not cle r.

T e patient does relatively well and abstains rom alcohol. He


HELPFUL TIP:
is now taking propranolol, spironolactone, and urosemide as
Parad xically, p rtal vei thr mb i i urpri i gly
well as ollowing a low-salt diet. He seems to be ollowing your
c mm i th e with cirrh i (11% ver 5 year ). It
instructions well but returns to the clinic because o increasing
d e t eem t cau e pr gre i cirrh i (Hepa-
dyspnea, abdominal distention, and pain. On examination, he
t l gy. 2015;61:660). A tic agulati r acute p rtal
has no peritoneal signs but obviously has massive ascites. You
vei thr mb i ( ever, chill , abd mi al pai , te der
are considering a large-volume paracentesis in your o ce.
liver) i i dicated. The be e it a tic agulati i l g-
ta di g p rtal vei thr mb i , uch a that u d i -
Question 7.13.5 Which o the ollowing statements best
cide tally ultra u d, i t de i ed. H wever, i the
re ects the current thinking on large volume paracentesis?
-cirrh tic patie t p rtal thr mb i h uld pr mpt
A) A p tient who h s over 4 L o uid re oved should receive
a evaluati r a hyperc agulable tate (e.g., ca cer).
IV lbu in.
L g-term treatme t with -war ari a tic agula t
B) T ere is no consistent d t with reg rd to the use o lbu in
ca be u dertake but i crea e bleedi g epi de a d
in l rge-volu e p r centesis.
ha a u certai be e it. There i a high ri k rethr m-
C) Under no circu st nce should ore th n 5 L o scites be
b i whe a tic agulati i t pped (39% i 6 week
re oved t one ti e.
[Cli Ga tr e ter l Hepat l. 2012;10:776]).
D) Given this p tient’s dyspne , l rge-volu e p r centesis is
contr indic ted.
E) I ore th n 10 L o scites uid is re oved, n equ l vol-
u e o nor l s line should be repl ced intr venously.
It turns out that this patient has also been drinking again, but
he made it through his hospital stay without going through Answer 7.13.5 The correct answer is “B.” It is uncle r wh t
withdrawal. Esophageal varices were noted on EGD. He is p tient ctors indic te lbu in repl ce ent. “A” is incorrect.
ready or discharge. T ere is cle rly no need or lbu in in p tients who h ve less
CHAPTER 7 • GAs TRo En TERo Lo GY 225

th n 5 L o uid re oved. However, or p tients who h ve ore Objectives: Did you learn to . . .
th n 5 L o uid re oved, the st nd rd o c re is repl ce ent • Rec g ize cau e hepatic e cephal pathy a d ide ti y
with lbu in lthough the d t re li ited. “C” is incorrect. patie t at ri k?
L rge volu e p r centesis c n be done (even > 10 L) “D” is • Ma age a patie t with hepatic e cephal pathy?
incorrect. In ct, respir tory co pro ise is one re son to do • Di cu ri k a d be e t TIPs a d large-v lume parace -
l rge-volu e p r centesis. Re ov l o uid will help with te i ?
di phr g tic excursion nd y help with the resolution o
pleur l e usions. “E” is wrong s lbu in repl ce ent is rec-
o ended inste d o nor l s line. Giving nor l s line will QUICK QUIZ: AUTo IMMUn E LIVER DIs EAs E
urther induce uid shi ing nd incre se sodiu lo d.” And, it
just sounds wrong, don’t you gree? T t’s lot o uid shi ing. Which o the ollowing is rker or pri ry bili ry cirrhosis?
A) Anti- itochondri l ntibodies (AMA).
HELPFUL TIP: B) Antis ooth uscle ntibodies.
A meta-a aly i albumi replaceme t ll wi g C) Alph 1- ntitrypsin.
parace te i ugge t that 6 t 8 g albumi be give D) Polyclon l ntibodies on seru protein electrophoresis.
per liter a cite rem ved i th e patie t havi g
m re tha 5 L a citic luid rem ved (Hepat l gy. The correct answer is “A.” Anti- itochondri l ntibodies re
2012;55:1172). ound in pri ry bili ry cirrhosis (95% sensitive nd 98% spe-
ci c). “B,” ntis ooth uscle ntibodies, re ound in utoi -
une hep titis. “C,” reduced levels o lph 1- ntitrypsin, re
You remove 4 L o uid via paracentesis, and the patient eels ound in hep titis ro lph 1- ntitrypsin de ciency (surprise!).
better. In the meantime, however, he has had a variceal bleed “D,” polyclon l ntibodies, re ound in utoi une hep titis.
and has been hospitalized yet again. He needs something
else done, and you re er him or a transjugular intrahepatic
HELPFUL TIP:
portosystemic shunt ( IPS) procedure to help reduce portal
scler i g ch la giti i a i lammat ry pr ce that
pressures, prevent the re-accumulation o ascites, and hope-
cau e tricture the i trahepatic a d/ r extrahepatic
ully prevent urther bleeding.
duct . Bi chemically, there i evide ce ch le ta i
ma i e ted by a elevated alkali e ph phata e. Thi
Question 7.13.6 Which o the ollowing statements best
pr gre e t ibr i , cirrh i , a d hepatic ailure with
re ects the status o TIPS?
a media urvival 10 year with ut a tra pla t. The
A) IPS is ine ective in controlling cute v rice l bleeding nd
rate ch la gi carci ma i al elevated i the e pa-
scites.
tie t . The diag i cler i g ch la giti i made
B) IPS unequivoc lly i proves surviv l ro end-st ge liver
by ch la gi graphy, which dem trate tricture a d
dise se.
dilatati (beaded appeara ce) i tra-hepatic a d/ r
C) IPS is ssoci ted with n incre sed risk o hep tic enceph -
extrahepatic duct . The maj rity ca e are related t
lop thy.
i lammat ry b wel di ea e.
D) Once pl ced, IPS re ins e ective or t le st 3 ye rs.
E) IPS is only indic ted or w iters, b r tenders, b rist s, nd
c b drivers.
CASE 7.14
Answer 7.13.6 The correct answer is “C.” IPS is cle rly sso-
ci ted with n incre sed risk o hep tic enceph lop thy. “A” is A 31-year-old man comes to your o ce or evaluation o
incorrect. IPS is ef ective in controlling v rice l bleeding nd “abnormal liver tests.” He had labs or disability insurance
y help scites by reducing port l pressures. “B” is incorrect. and the ollowing was noted: elevated total bilirubin (2.1 mg/
In ct there y be so e surviv l disadvantage to IPS. How- dL) with normal direct bilirubin. T e AS , AL , GG , albu-
ever, the d t is equivoc l. “D” is incorrect bec use IPS shunts min, and alkaline phosphatase are normal. A CBC obtained
tend to clot nd y need to be ev lu ted or p tency by Dop- at that time was unremarkable.
pler i scites re- ccu ul tes or other sy pto s occur. Newer T e patient eels generally well. When you ask about jaun-
devices, however, tend to clot o less o en. dice, he recalls that once when he was sick with the “ u” while
in college, his roommate told him that he “looked a little yel-
HELPFUL TIP: low.” He went to the student health service a ew days later
o ver-diure i ca lead t hepat re al y dr me, which when he elt better and was told there was nothing to be con-
i characterized by liguric re al ailure. s ep i , GI bleed- cerned about. He drinks 6 to 8 beers/week and takes ibupro-
i g, etc., ca al lead t hepat re al y dr me. The en once or twice a month or knee pain.
mecha i m ha m re t d with altered circulati t the Physical examination is unremarkable. A repeat liver
kid ey r m hepatic di ea e tha with the diure i it el . panel shows a total bilirubin o 2.4 mg/dL with normal conju-
gated (direct) bilirubin, an elevated unconjugated (indirect)
226 FAMILY MEDICIn E EXAMIn ATIo n & Bo ARD REVIEW

bilirubin and normal AS , AL , and alkaline phosphatase. D) Reco end serologic ev lu tion to ssess or chronic
CBC and reticulocyte count are also normal. A blood smear, vir l nd utoi une hep titis i the liver test bnor li-
LDH, and haptoglobin are normal. ties persist.
E) Right upper qu dr nt ultr sound.
Question 7.14.1 The most likely diagnosis is:
A) Crigler–N jj r syndro e type I. Answer 7.14.2 The correct answer is “B.” As discussed bove,
B) Choledocholithi sis. Gilbert syndro e is de ned by isol ted unconjug ted hyperbili-
C) Gilbert syndro e. rubine i . T is p tient’s p ttern o liver test bnor lities with
D) He olytic ne i . inotr ns er se elev tions cle rly indic tes di erent type o
E) Occult cet inophen buse. proble . I these bnor lities persist, n ev lu tion or c uses
o chronic inotr ns er se elev tions is w rr nted (see di er-
Answer 7.14.1 The correct answer is “C.” Gilbert syndro e is enti l e rlier in this ch pter o elev ted liver enzy es). Chronic
the ost co on inherited disorder o bilirubin et bolis , vir l hep titis nd utoi une hep titis would be ong the
ecting up to 5% o C uc si ns. It is ch r cterized by isol ted di gnostic consider tions. Given the pp rent hep to eg ly
ild unconjug ted (indirect) hyperbilirubine i , with seru nd tenderness on physic l ex in tion, n i ging study o
bilirubin levels usu lly less th n 3 g/dL. However, bilirubin the liver is lso indic ted.
levels in p tients ected with Gilbert syndro e c n incre se
with sting or during ebrile illnesses, though r rely exceed- T e repeat liver panel is remarkable or AL 129 IU/L and
ing 6 g/dL. Crigler–N jj r syndro e type I is r re disorder AS 76 IU/L. T e other tests are normal. HBsAg and HCV
le ding to severe unconjug ted neon t l j undice nd neuro- antibody are negative. Antinuclear and antismooth muscle
logic i p ir ent due to kernicterus. Both choledocholithi sis antibodies are < 1:40 (normal). T e ultrasound examination
nd cet inophen hep totoxicity would be unlikely to c use shows an enlarged liver with increased echogenicity, sugges-
n isol ted incre se in unconjug ted bilirubin; the LF s should tive o dif use atty in ltration.
be elev ted with these. T e nor l blood s e r, reticulocyte
count, nor l LDH, nd nor l h ptoglobin ke signi c nt Question 7.14.3 Regarding nonalcoholic atty liver disease
he olysis unlikely. T us, this present tion is ost consistent (NAFLD), or nonalcoholic steatohepatitis (NASH), all o the
with Gilbert syndro e. ollowing are true EXCEPT:
A) It is requently ssoci ted with one or ore e tures o the
Several months later, the patient’s older sister comes to see et bolic or insulin resist nce syndro e.
you. She had gone to a local health air where screening liver B) It is ore co on in en th n wo en.
tests were ound to be abnormal. She recalled her brother C) T e histologic e tures c n closely i ic those o lcoholic
mentioning something about a amilial problem causing liver hep titis.
test abnormalities and wonders i she has the same thing. D) It y c use cirrhosis in inority o p tients.
She is 39 years old with good general health, although she
wishes she could lose weight. She says she has been 30 to 40 lb Answer 7.14.3 The correct answer is “B.” NAFLD is ore
overweight or at least 10 years and is now at her heaviest weight co on in wo en nd is ong the ost co on c uses o
ever. She is a nonsmoker, who takes no medications and drinks elev ted liver enzy es. NAFLD re ers to spectru o histo-
no alcohol. She denies any risk actors or viral hepatitis. logic ndings th t r nge ro si ple ste tosis to n ggressive
Physical examination reveals an obese woman. Blood injury p ttern. NAFLD o en occurs in ssoci tion with obe-
pressure is 138/88 mm Hg; BMI is 35 kg/m2. T ere is no sity, dyslipide i , nd/or glucose intoler nce, hypothyroidis ,
scleral icterus or other cutaneous stigmata o chronic liver nd occurs ore co only in wo en th n en. While ost
disease. T e abdomen is protuberant with the liver edge p tients with NAFLD will not develop progressive liver dis-
palpable about 3 to 4 cm below the right costal margin and e se, inority re t risk to develop cirrhosis. However, due
is slightly tender to palpation. T ere is no splenomegaly to its high prev lence in our popul tion (th nk you, st ood
and no evident ascites. n tion!), NAFLD/NASH is now the ost co on c use o cir-
Her liver panel rom the health air 3 months ago shows the rhosis. Fe le gender nd the presence o di betes incre se the
ollowing: AL 87 IU/L (normal range, 0–20), AS 53 IU/L risk o progression to cirrhosis.
(0–31), alkaline phosphatase 110 IU/L (30–115), total protein
7.8 g/dL (6.0–8.0), albumin 4.2 g/dL (3.3–5.0), total biliru- HELPFUL TIP:
bin 0.9 mg/dL (0.2–1.0), and conjugated (direct) bilirubin Vitami E 800 IU BID ha bee h w t be help ul i
0.1 mg/dL (< 0.2). n As H (n E gl J Med. 2010;362:1675). H wever, great-
er tha 400 mg IU vitami E may i crea e verall
Question 7.14.2 Appropriate steps at this time include all o m rtality (RR 1.04 a d t that r m n As H, [JAMA.
the ollowing EXCEPT: 2007;297:842]). Met rmi eem t be be e it,
A) Repe t the liver p nel. but tati may be. Fatty liver di ea e i a d it el i
B) Counsel the p tient th t her l bs suggest Gilbert syndro e. t a c trai dicati t tati .
C) Counsel the p tient reg rding weight loss.
CHAPTER 7 • GAs TRo En TERo Lo GY 227

Question 7.14.4 In this patient with NAFLD, all o the ol vomiting, ever, and myalgias. Her vital signs are blood pres-
lowing are appropriate actions EXCEPT: sure 110/70 mm Hg, pulse 115 bpm, and temperature 38.5°C.
A) Obt in sting lipid p nel. On examination, the patient is quite tender in the midepigas-
B) Obt in sting seru glucose. tric region with guarding and some rebound. Her past his-
C) Reco end weight loss to get her BMI to the nor l r nge. tory is signi cant only or lone atrial brillation (CHA2DS2-
D) St rt “glit zone.” VASc score o 1, see Chapter 2) and a seizure disorder or
which she is taking phenytoin.
Answer 7.14.4 The correct answer is “D.” Weight loss, includ-
ing ro g stric surgery, c n i prove NAFLD. However, too Question 7.15.1 Your next steps in the diagnosis o this
r pid weight loss y be counterproductive in the setting o patient should include all o the ollowing EXCEPT:
NAFLD, s obiliz tion o peripher l t stores y worsen A) Chest r diogr ph.
hep tic ste tosis. Also, ddressing underlying et bolic ill- B) CBC.
nesses (e.g., di betes nd hypothyroidis ) would be i port nt C) Liver enzy es.
ele ents in the tre t ent. re ting hyperlipide i nd di betes D) Abdo in l C with contr st.
c n lso i prove NAFLD. Glit zones h ve been shown to be E) A yl se.
ine ective in NAFLD s h s ursodeoxycholic cid.
Answer 7.15.1 The correct answer is “D.” An bdo in l C
HELPFUL TIP: is not indic ted in this p tient s p rt o the initi l workup. “A,”
I Fra ce, they rce eed gee e t pr duce atty liver n upright chest r diogr ph, is the single best plain r diogr ph
(F ie Gra ). I the U ited s tate , we d it t ur elve . . . or nding ree bdo in l ir nd is indic ted or this re son.
In ddition, the etiology o bdo in l p in y lso include
pneu oni (especi lly consider this in the elderly) nd other
Objectives: Did you learn to . . . thor cic p thology th t y be evident on chest r diogr ph.
• Evaluate a patie t with ab rmal liver te t ? Liver enzy es, yl se, nd CBC re indic ted since idepi-
• Rec g ize the cli ical a d lab rat ry pre e tati Gilbert g stric p in with ever c n be rel ted to p ncre titis, cute cho-
y dr me? lecystitis, etc.
• De cribe di g alc h lic liver di ea e?
• Ma age a patie t with atty liver di ea e?
Her chest x-ray is normal and her laboratory results are as
ollows: elevated AS and AL (both mild), normal amy-
lase, mildly elevated white count (13.5 × 103 cells/mm3), and
QUICK QUIZ: GI BLEED elevated GG .

Which o the ollowing h s shown bene t in reducing ort lity Question 7.15.2 The correct interpretation o these results
ro v rice l GI bleeding? is:
A) V sopressin. A) T e p tient does not h ve p ncre titis.
B) So tost tin/octreotide. B) T e elev ted GG is speci c or bili ry outlet obstruction.
C) B nd lig tion o v rices. C) T e elev ted AS nd AL y indic te bili ry outlet
D) IPS. obstruction.
E) C nd D. D) None o the bove.

The correct answer is “E.” Un ortun tely, while edic l tre t- Answer 7.15.2 The correct answer is “C.” E rly in the course o
ent y bene t the p tient by reducing bleeding, there is no bili ry outlet obstruction (e.g., bili ry colic nd co on duct
evidence th t v sopressin or so tost tin reduce ort lity. In stone), the AS nd AL y be ildly elev ted. “A” is incorrect
ct, v sopressin y increase ort lity bec use o ssoci ted bec use the yl se is only 80% sensitive or p ncre titis. wenty
GI ische i . IPS nd lig tion o v rices vi n endoscope h ve percent o p tients with p ncre titis h ve nor l yl se—
positive i p ct on ort lity. We re not trying to con use lse-neg tive test. “B” is incorrect bec use GG is nonspeci c.
you reg rding the previous question bout IPS—it does NOT GG is n inducible enzy e nd c n be elev ted in response to
reduce overall mortality in end-st ge liver dise se, but it c n lcohol nd v rious edic tions including phenytoin.
reduce ort lity ro n cute GI bleed. So tost tin reduces
rebleeding when used with endoscopic ther py but does not HELPFUL TIP:
ect ort lity. I alc h l-related liver di ea e, the As T i ge erally
twice the level the ALT.
CASE 7.15
A 70-year-old emale presents to the emergency department
complaining o midepigastric pain associated with vomiting. We still don’t know what is causing this patient’s pain. You
T is started approximately 12 hours ago and now she notes consider your dif erential. T e patient has a history o atrial
228 FAMILY MEDICIn E EXAMIn ATIo n & Bo ARD REVIEW

brillation and is not on anticoagulation (appropriately since etronid zole. “A” is incorrect bec use clind ycin covers
she has a CHA2-DS2-VASc score o 1). You are concerned Gr -positive org nis s nd n erobes, does not cover ost
that there may be bowel ischemia rom an embolism. Gr -neg tive org nis s. In ddition, ny enterococci re
resist nt to clind ycin. “B” is incorrect bec use v nco y-
Question 7.15.3 Which o the ollowing is true o mesen cin only covers Gr -positive org nis s nd even then it is
teric thrombosis and bowel ischemia? rel tively we k ntibiotic or such (e.g., ort lity is higher
A) P tients gener lly h ve gu rding nd rebound e rly in the or MSSA th n with n cillin). “C” is incorrect bec use gen-
course. t icin does not cover n erobic org nis s. Other ntibiotic
B) T e p in is out o proportion to the ex in tion nd p tients options or tre ting this p tient include ce otet n nd ce oxi-
y h ve nor l initi l ex in tion. tin, ong others.
C) A seru l ct te level is help ul nd speci c or the di gnosis
o bowel ische i . T e patient is admitted to the hospital. She is treated with
D) T e best study to di gnose this dise se entity is C with con- ampicillin/sulbactam, IV uids and pain medication.
tr st.
Question 7.15.5 The next step or this patient is:
Answer 7.15.3 The correct answer is “B.” P tients with s ll A) Percut neous “ ” tube pl ce ent to dr in the g llbl dder.
bowel ische i ro either e bolis or esenteric thro bo- B) 2 weeks o IV ntibiotics ollowed by cholecystecto y.
sis will gener lly present with severe bdo in l p in nd n C) Endoscopic retrogr de chol ngiop ncre togr phy (ERCP)
ex in tion th t is unre rk ble. L te in the course there will with sphincteroto y.
be gu rding, rebound, nd other peritone l signs s the bowel D) Lithotripsy.
per or tes. However, e rly in the course o the illness, severe
p in with rel tively benign ex in tion is consistent with Answer 7.15.5 The correct answer is “C.” T e p tient should
the present tion o bowel ische i . “A” is incorrect or the re - h ve n ERCP with sphincteroto y in n tte pt to retrieve
sons st rted bove. “C” is incorrect bec use the seru l ct te the stone in her co on bile duct. “A” is incorrect bec use
c n be elev ted in nu ber o st tes, not just bowel ische i . in this he lthy p tient, percut neous dr in ge would cert inly
However, bdo in l p in plus l ctic cidosis should r ise the be third-line procedure. “B” is incorrect bec use the p tient
suspicion th t there y be bowel ische i . “D” is incorrect essenti lly h s closed bscess (the g llbl dder blocked by
bec use r diogr phic ndings on C sc n re present in only stone), which requires dr in ge. I the p tient h d cholecystitis
bout 65% o p tients with esenteric thro bosis/e bolis . without obstruction, prolonged ntibiotics ollowed by cho-
T e best study re ins ngiogr phy or C ngiogr phy, which lecystecto y would be n option. However, in patients with
h s l rgely repl ced tr dition l ngiogr phy. cholecystitis without obstruction outcomes are overall better
with an early cholecystectomy. “D” is lso incorrect nd would
Since the patient’s examination includes guarding and be less desir ble ppro ch th n is ERCP with sphincteroto y.
rebound, you put the diagnosis o bowel ischemia lower in
your list o possibilities. Based on the elevated white count,
AL and AS , you order an ultrasound o the right upper HELPFUL TIP:
quadrant, which shows evidence o a common duct stone. s tudie have h w impr ved utc me i patie t with
T ere is thickening o the gallbladder wall but no pericolic evere gall t e pa creatiti a d/ r ch la giti that re-
uid noted. You decide that there is a mild cholecystitis and ceive early (withi 72 h ur a ter admi i ) ERCP with
want to admit the patient to the hospital. Clearly, this patient biliary phi cter t my.
needs to be started on antibiotics.

Question 7.15.4 Which antibiotic is the most appropriate T e ERCP is success ul but the patient develops worsening
choice or this patient? pain in the midepigastric region.
A) IV clind ycin.
B) IV v nco ycin. Question 7.15.6 Which o the ollowing is the most com
C) IV gent icin. mon adverse consequence o ERCP?
D) IV picillin/sulb ct . A) P ncre titis.
B) Contr st llergy.
Answer 7.15.4 The correct answer is “D.” T e ost ppropri- C) Per or tion.
te ntibiotic choice o those provided is picillin/sulb ct D) Bleeding.
(Un syn). T e in org nis s th t need to be covered re E) Sepsis.
Gr -neg tive org nis s nd n erobes (E. coli, Enterococ-
cus, Klebsiella, nd Enterobacter). A picillin/sulb ct will Answer 7.15.6 The correct answer is “A.” O the co plic tions
cover ll o these org nis s. You ight lso choose piper- listed, p ncre titis is the ost co on, with n incidence o
cillin/t zob ct (Zosyn), which includes pseudo on l bout 5% o p tients er ERCP. In ct, elev tions in p ncre-
cover ge, or ce ri xone + etronid zole or levo ox cin + tic enzy es ( ostly not signi c nt) occur in up to 75% o
CHAPTER 7 • GAs TRo En TERo Lo GY 229

TABLE 7-7 DRUGS ASSOCIATED WITH PANCREATITIS D) When used in co bin tion with n SSRI, orphine is ore
likely to c use serotonin syndro e th n is eperidine.
Drugs with Def nitive Drugs with Probable
Association Association
Answer 7.15.8 The correct answer is “D.” Meperidine ( nd
Thiazide diuretic Acetami phe tr dol) y c use serotonin syndro e when co bined
s ul amide s alicylate with n SSRI. T is is not proble with orphine. T e rest re
Azathi pri e Metr idaz le
true st te ents. T ere is no evidence to support the tr dition
6-Mercapt puri e n itr ura t i
Fur emide Erythr myci th t eperidine is superior to orphine in bili ry or p ncre tic
E tr ge n s AID dise se. Meperidine is et bolized into nor eperidine, which
Tetracycli e ACE i hibit r c n c use git tion nd seizures. T us, “B” nd “C” re correct.
Valpr ic acid Methyld pa In gener l, orphine—or hydro orphone or ent nyl— re
Pe tamidi e s ter id
uch cle ner drugs to use or p in n ge ent.
Valpr ic acid
Dide xyi i e
You have the patient’s pain well managed with morphine,
despite grumbling rom your surgical colleague.

p tients post-ERCP. T e others re less co on nd in order Question 7.15.9 Which o the ollowing treatments is rou-
o descending incidence re bleeding, per or tion, sepsis, nd tinely indicated in the f rst day o pancreatitis?
contr st llergy (r re). A) NG tube with inter ittent suction.
B) H 2 blockers (e.g., ci etidine) or PPIs (e.g., l nsopr zole).
Although you suspect that this patient has pancreatitis sec- C) IV ntibiotics.
ondary to the ERCP, you also consider other potential causes D) NPO order.
o pancreatitis.
Answer 7.15.9 The correct answer is “D.” T e critic l words
Question 7.15.7 All o the ollowing are causes o pancre here re “ rst d y”. Once p in nd in tion re decre sing
atitis EXCEPT: (gener lly within 24 to 48 hours o the onset o p in) PO nutri-
A) Vir l in ection. tion c n be rest rted ssu ing there is no ileus, vo iting, etc.
B) HMG-Co A reduct se inhibitors (st tins). Early eeding in the hungry patient with pancreatitis hastens
C) Alcohol. disease resolution. Feedings can also be done via jejunal tube,
D) Indin vir. which bypasses the pancreas. Even a G-tube can be used or
E) Bee stings. eedings. However, i the patient is tolerating oral eeds let
them go or it! “A” is incorrect bec use there is no indic tion or
Answer 7.15.7 The correct answer is “E.” T e ost co on n NG tube in the p tient who is not vo iting. “B” is incorrect
c uses o cute p ncre titis in the United St tes re eth nol bec use H 2 blockers or PPIs do not ch nge the outco e in p n-
ingestion, bili ry tr ct dise se, nd endoscopic procedures cre titis. “C” is incorrect unless there is evidence o in ection
with bili ry tr ct dise se. “A” is true. Co on viruses th t (e.g., ever) or necrotizing p ncre titis— nd even then there is
c n c use p ncre titis include HIV, hep titis viruses, EBV, controversy.
nd Coxs ckieviruses. “B” nd “D” re true. M ny other drugs
c n lso c use p ncre titis: o note re did nosine, so e Your patient is NPO because o an ileus and is receiving
diuretics, so e NSAIDs, so e ntibiotics, etc. See ble 7-7 appropriate IV uids and parenteral eedings with CVN
or ore on drugs th t c use p ncre titis. T e veno o (remember nutrition, however you can give it, is important).
scorpion sting c n result in p ncre titis, s c n brown recluse However, over the next couple o days, her condition worsens.
spider bites. She begins to vomit and have increased pain as well as tachy-
cardia and ever. T e patient wants to know what her chances
You re-check laboratory studies and the patient indeed has o surviving this bout o pancreatitis are, so you break out a
worsening pancreatitis rom the ERCP. T e surgeon on the chart and your smart phone.
case recommends against using morphine and would pre er
meperidine in this patient, mumbling something about that’s Question 7.15.10 All o the ollowing are part o Ranson cri
the way they always did it in residency. teria EXCEPT:
A) P tient’s ge.
Question 7.15.8 All o the ollowing are true EXCEPT: B) P tient’s white count on d ission.
A) T ere is no evidence th t eperidine is superior to or- C) HC t 72 hours.
phine even in g llbl dder nd p ncre tic dise se. D) Glucose on d ission.
B) Meperidine is ore likely th n orphine to c use seizures
in this p tient. Answer 7.15.10 The correct answer is “C.” T e HC t
C) Meperidine is ore likely th n orphine to c use con usion 48 hours is one o R nson criteri . T e other criteri re out-
nd git tion in elderly p tients. lined in ble 7-8.
230 FAMILY MEDICIn E EXAMIn ATIo n & Bo ARD REVIEW

TABLE 7-8 RANSON CRITERIA Question 7.15.12 Which o the ollowing IS NOT true about
a pancreatic pseudocyst?
o admi i • Age > 55 year
A) Pseudocysts occur in 10% o p tients with p ncre titis.
• WBC > 16,000
• Gluc e > 200 B) Pseudocysts c n be dr ined by or ing stul with the
• LDH > 350 sto ch endoscopic lly.
• As T > 250 C) Pseudocysts c n le d to the or tion o rteri l pseudo n-
At 48 h ur • Hemat crit reducti by > 10% eurys s th t c n c use severe bleeding.
• BUn i crea e > 5 D) Open dr in ge is the pre erred ethod o tre t ent.
• Calcium < 8 E) Not ll pseudocysts require dr in ge.
• po 2< 60
• Ba e de cit > 4
• Fluid eque trati >6L
Answer 7.15.12 The correct answer is “D.” T e or tion o
stul with the sto ch using n endoscope is ctu lly the pre-
Note: Mild pa creatiti i de ed by the pre e ce 1 t 3 Ra criteria; erred ethod o dr in ge. T e one jor contr indic tion to
m rtality i crea e with the pre e ce 4 r m re. endoscopic tre t ent is pseudo neurys . Injury to n rtery
c n c use signi c nt bleeding th t is dif cult to control.

HELPFUL TIP:
HELPFUL TIP:
CT ca ca be u ed t l k r a p eud a eury m be-
The Ra criteria are either particularly e itive
re e d c pic drai age. A a eury m h uld al be
r peci ic r evere pa creatic di ea e a d ca t
u pected i the e circum ta ce : evide ce a upper
be c mpleted be re 48 h ur h pitalizati . The
GI bleed, a dr p i the HCT, r a udde expa i
Apache c re acc u t r bl d pre ure, xyge -
the p eud cy t.
ati , temperature, re pirat ry rate, creati i e, GCs,
etc., a d i a better predict r urvival. Fi ally, a HCT
44% r greater at admi i a d ailure t decrea e Objectives: Did you learn to . . .
at 24 h ur i al a excelle t predict r ecr tizi g • Rec g ize a d diag e me e teric thr mb i ?
pa creatiti a d multi rga ailure. The America Ga - • Ide ti y cau e acute pa creatiti ?
tr e ter l gical A ciati rec mme d the u e • Diag e a d ma age a patie t with acute pa creatiti ?
the Apache II c re (ye , we k w the APACHE c re i • Ma age the c mplicati pa creatiti ?
w up t IV. . . the Apache II eem t be the be t here,
• De cribe me ide e ect pare teral utriti i a h pi-
th ugh). talized patie t?
• De cribe the pri ciple arc tic ch ice i patie t with
pa creatiti ?
HELPFUL TIP:
E teral utriti i pre erred ver CVn r pa creatiti .
n G v n J eedi g i c tr ver ial. s me tudie h w a QUICK QUIZ: GALLs To n Es
be e it n J eedi g whe the eedi g tube i placed
bel w the ligame t Treitz. Thi reduce pa creatic A 40-ye r-old e le presents to you with g llstones ound on
timulati . “ ull body C sc n” per or ed t Live4Ever I ging echnolo-
gies, Inc. She is nxious th t she will beco e sick like her sister
who h d n e ergent cholecystecto y or her g llstone p ncre-
Question 7.15.11 Which o the ollowing IS NOT a complica titis. She sks you i she should h ve her g llbl dder re oved.
tion o CVN? She is sy pto tic nd her liver enzy es re nor l.
A) In ection.
B) Cholest sis. Which o the ollowing should you reco end?
C) Hypoglyce i . A) L p roscopic cholecystecto y bec use o ily history o
D) Ileus. severe g llstone p ncre titis.
B) Ultr sound to con r g llstones nd to check or co on
Answer 7.15.11 The correct answer is “D.” In ection, cholest - bile duct stones.
sis, nd hypoglyce i c n ll be result o CVN. Hypoglyce i C) No tre t ent or ollow-up is needed unless she develops
gener lly occurs when stopping CVN bec use o incre sed lev- sy pto s.
els o circul ting insulin. T is c n be itig ted by t pering CVN D) ERCP with sphincteroto y to llow stones to p ss without
or d inistering IV dextrose. obstructing co on bile duct or co pressing p ncre tic
duct.
Since the patient has continued to do poorly, you order a C E) Recheck liver enzy es nd i elev ted reco end cholecys-
scan o the abdomen that shows evidence o a pseudocyst. tecto y.
CHAPTER 7 • GAs TRo En TERo Lo GY 231

The correct answer is “C.” Asy pto tic g llstones do not


need speci l ttention since 70% to 80% re in sy pto tic. You order a radionucleotide gastric emptying study, and it
Only 2% to 3% o p tients will present with cute cholecystitis shows that 75% o the meal was present in the stomach at
or other co plic tions nd there ore prophyl ctic cholecystec- 2 hours and 20%at 4 hours. You diagnose gastroparesis.
to y is not indic ted. A eric n Indi n popul tions h ve high Question 7.16.2 The best long term treatment option or
risk o stone ssoci ted g llbl dder c ncer nd re n exception her is:
to this rule. Di betics or sickle cell dise se p tients h ve higher A) Cis pride.
risk o co plic tions ro g llstones but still should not h ve B) Do peridone.
their g llbl dder re oved i sy pto tic. F ily history o C) Erythro ycin.
co plic tion ro g llstones is not n indic tion or prophyl c- D) Metoclopr ide.
tic cholecystecto y. E) I proved glucose control.

CASE 7.16 Answer 7.16.2 The correct answer is “E.” T e tre t ent o di -
betic g strop resis is o en dif cult nd rustr ting or p tients
A 58-year-old woman with type 2 diabetes mellitus comes nd clinici ns. Although not proven in prospective tri ls, ost
to your clinic with nausea, vague epigastric abdominal pain, experts believe i proved glucose control with diet ry nd li e-
bloating, early satiety, and intermittent vomiting or 3 weeks. style odi c tion is the key to long-ter success in di betic
Her past medical history is signi cant or hypertension and g strop resis. All o the bove edic tions (“A”–“D”) h ve
hyperlipidemia and her diabetes is complicated by retinopa- shown bene t in the short ter to reduce sy pto s ssoci ted
thy and neuropathy. She is on met ormin, glyburide, lisino- with di betic g strop resis. Cis pride w s re oved ro the r-
pril, aspirin, and atorvastatin. ket bec use o prolong tion o the Q interv l. Do peridone is
She is a ebrile and not ill appearing on examination. She not v il ble in the U.S. nd c uses prolonged Q interv l, but is
has gained 5 kg in the last year. She has mild epigastric ten- used in other countries or g strop resis. Erythro ycin nd eto-
derness without rebound or guarding. Bowel sounds are nor- clopr ide re e ective nd v il ble. However, both c n c use
mal and there is no abdominal distention. Laboratory results cr ps, nd erythro ycin c n c use n use , prolonged Q inter-
show normal CBC and dif erential with HbA1c o 9.4%. You v l, nd o en r pidly beco es ine ective (t chyphyl xis) while
suspect diabetic gastroparesis. etoclopr ide y c use t rdive dyskinesi or dystoni . Other
crolides re not e ective. Liquid etro loper ide is suggested
Question 7.16.1 Which statement is FALSE regarding diag or g stric p resis nd see s to work better th n t blets.
nosis o gastroparesis?
A) On scintigr phy (g stric e ptying study) > 60% o the st n- You add insulin to the diabetic regimen in an attempt to
d rd e l present in sto ch t 2 hours suggests g strop - tighten glucose control and prescribe metoclopramide 5 mg,
resis. 30 minutes be ore meals.
B) On scintigr phy > 10% o the st nd rd e l present in the
sto ch t 4 hours suggests g strop resis. Question 7.16.3 In addition to this you recommend all o
C) EGD looking or evidence o ood er n uncontrolled res- the ollowing dietary and li estyle modif cations EXCEPT:
t ur nt e l is good test or di gnosing g strop resis. A) Incre se diet ry ber.
D) G strop resis c n be di gnosed by exh led r diol beled B) Ch nge ro our l rge to six s ll e ls d ily.
CO2 e sure ent. C) Moder te exercise.
D) Decre se diet ry t.
Answer 7.16.1 The correct answer (the alse statement) is E) ADA 1800 kc l diet.
“C.” Si ply looking or ret ined g stric contents t n unspeci-
Answer 7.16.3 The correct answer is “A.” Fiber nd r w veg-
ed ti e er n unspeci ed ingestion is not the w y to di g-
et bles c n or g stric bezo rs (phytobezo r) in g strop resis
nose g strop resis. T e ost widely used test or di gnosing
nd this is o en seen on endoscopy. Bezo rs c use e rly s tiety
g strop resis is r dionucleotide sc n. T e p tient is given st n-
nd blo ting nd dd to the sy pto burden o g strop resis. I
d rdized e l cont ining 99 technetiu sul ur colloid in
bezo r is ound, it c n be dissolved using cellul se (K n l se®).
low- t eggs, nd nucle r ctivity is e sured t 2 nd 4 hours,
More requent, s ller e ls help with sy pto s; nd oder-
respectively. I r dio ctivity in the sto ch is > 60% t 2 hours
te exercise c n be help ul but excessive exercise y slow g s-
or > 10% t 4 hours, the p tient is considered to h ve del yed
tric e ptying. Your p tient is g ining weight, so it is i port nt
g stric e ptying. You c n do liquid ph se g stric e ptying
to rein orce the ADA diet. F t slows g stric e ptying; there ore,
study i you suspect du ping syndro e, but it is not necess ry
diet ry t should be reduced to less th n 40 g/d y.
to ev lu te or g strop resis. “D,” r diol beled CO2 bre th test,
correl tes well with nucle r scintigr phy nd is e sier to per- Question 7.16.4 Which o the ollowing medications can
or in the co unity setting. However, r diol beled CO2 exacerbate preexisting gastroparesis?
bre th test requires nor l s ll bowel, p ncre s, liver, nd A) Fluoxetine.
lung unction. B) Oxycodone.
232 FAMILY MEDICIn E EXAMIn ATIo n & Bo ARD REVIEW

C) Angiotensin converting enzy e (ACE) inhibitors. A) Prescribe iron dextr n or sucrose IV with di lysis to in-
D) Met or in. t in her iron stores.
E) Insulin. B) ry to convince the g stroenterologist to repe t the endos-
copy nd tre t ny ngiodyspl si lesions th t c n be
Answer 7.16.4 The correct answer is “B.” Oxycodone nd ll re ched.
n rcotics reduce GI otility nd re r rely toler ted by p tients C) Prescribe octreotide.
with g strop resis. T is c n be clinic lly ch llenging, nd tten- D) Order Meckel sc n.
tion to the c use o p in nd ltern tive n ge ent options E) Prescribe estrogen–progesterone.
should be explored. Fluoxetine does not h ve signi c nt e ects
on g stric otility, but CAs nd other drugs with nti-cholin- The correct answer is “A.” T is is n ppropri te situ tion or
ergic ctivity do nd re not good options or p in syndro es the d inistr tion o IV iron. It is s er th n blood tr ns u-
in g strop resis. ACE inhibitors do not ect GI otility. sions nd c n be given on n outp tient b sis during he o-
Although et or in is ssoci ted with GI side e ects, it does di lysis. T ere re r re occurrences o n phyl xis with iron
not ect GI otility. Insulin h s no dverse e ect on GI otil- dextr n, nd test dose should be given. So e reports suggest
ity. Other edic tions th t reduce g stric e ptying re dop - iron sucrose is s er. “B,” repe t endoscopies with tre t ents o
inergic gents, nti- drenergic nti-hypertensives, c lciu ngiodyspl si s, y help but should be djunctive ther py to
ch nnel blockers, nd nti-cholinergic gents. All drugs known inten nce o iron stores. T e scope only re ches the proxi l
to ect g stric otility should be stopped prior to obt ining jejunu nd the p tient h s lesions throughout the s ll bowel.
g stric e ptying study. “C,” octreotide, h s been reported in c se series to h ve bene -
ci l e ect, but no r ndo ized tri ls h ve been per or ed nd it
c nnot be reco ended t this ti e. “D,” Meckel sc n (done
HELPFUL TIP: to look or Meckel diverticul ), would be redund nt s nother
There are multiple cau e ga tr pare i ther tha c use h s been ound nd the p tient does not t the ost likely
diabete . s me the e i clude: p tviral ga tr e teri- ge group. “E,” hor one repl ce ent ther pies initi lly see ed
ti (may per i t r > 1 year), thyr id di ea e, eur l gic e ective t reducing bleeding ro ngiodyspl si but r ndo -
di ea e , a d aut immu e di ea e . F rty perce t ized tri ls iled to show bene t.
ga tr pare i i idi pathic. s c ider thi i y ur di -
ere tial the patie t with a y the ll wi g: p t-
pra dial ull e , upper abd mi al pai , au ea, v mit- HELPFUL TIP:
i g, early atiety, a d regurgitati d h ur a ter A gi dy pla ia are c mm ly mi ed a d are a -
i take (yum!) ciated with i crea i g age, e d- tage re al di ea e,
a rtic te i , hereditary tela giecta ia (aut mal
d mi a t), a d are m re likely t bleed i patie t
Objectives: Did you learn to . . . l g-term a tic agulati a d a tiplatelet therapie .
• Diag e ga tr pare i ? o ther c mm ly mi ed le i i clude Camer le-
• Ide ti y medicati that ca exacerbate ga tr pare i ? i ( mall ulcer cau ed by rubbi g hiatal her ia
ac agai t diaphragm) a d PUD.
• Ma age ga tr pare i ?

QUICK QUIZ: GI BLEED HELPFUL TIP:


Cap ule e d c py the mall b wel u i g a mall
You d it 65-ye r-old wo n with end-st ge ren l dise se camera ca be help ul i a mall b wel ll w-thr ugh
on he odi lysis. She is being d itted bec use o recurrent d e t h w a urce bleedi g. o ther p ibilitie
episodes o elen requiring tr ns usions. She is ne ic with i clude tagged red bl d cell (RBC) ca a d a gi g-
low erritin, iron, nd iron s tur tion. T e g stroenterologist raphy that are ly really u e ul i vert bleedi g. The
per or s n upper nd lower endoscopy without ny bleeding tagged RBC ca eed 0.1 mL/mi bleedi g rate a d
source ound. Subsequently, c psule enteroscopy shows ul- a gi graphy require 0.5 mL/mi bleedi g rate t de-
tiple s ll ngiodyspl si s throughout the s ll bowel, without tect bleedi g ite .
ny l rge, endoscopic lly tre t ble lesions.
Your p tient is disch rged er blood tr ns usion. Her
he oglobin is 11 g/dL. At ollow-up 1 onth l ter, her he o-
globin is 9 g/dL. T e p tient is on xi u doses o erythro- QUICK QUIZ: HEMATo CHEZIA
poietin with di lysis nd is still h ving gu i c positive stools.
She re uses to t ke or l iron bec use o constip tion. A 30-ye r-old e le co es to your clinic with the co pl int o
inter ittent blood on toilet p per or 3 ye rs. She s ys she lw ys
Which o the ollowing is the next best step in the n ge ent h s been constip ted nd t kes polyethylene glycol (Mir L x)
o this c se? on regul r b sis. She h s no ily history o colorect l c ncer
CHAPTER 7 • GAs TRo En TERo Lo GY 233

nd is sy pto tic nd h s no weight loss. On ex in tion, C) IBS.


she does not ppe r ne ic nd bdo in l ex in tion is nor- D) Pyelonephritis.
l. Rect l ex in tion is nor l nd nontender. Anoscopy is E) Diverticulitis.
nor l.
Answer 7.17.1 The correct answer is “E.” Diverticulitis is the
Wh t is the ost ppropri te ction? ost likely c use o the p tient’s p in. Fever ssoci ted with
A) Re ssur nce. cute onset bdo in l p in loc ted in le lower qu dr nt
B) Anusol suppositories three ti es/week or 6 weeks; ollow kes this p in ore likely to be due to diverticulitis. Diver-
up s needed. ticulitis c n, however, occur in ny p rt o the colon. Ische ic
C) Flexible sig oidoscopy. colitis (“A”) c n present with disproportion te bdo in l p in in
D) Colonoscopy. this loc tion but is usu lly ssoci ted with bloody di rrhe , so it is
E) Anorect l no etry. less likely. IBS (“C”) l ost never presents in this ge group, is not
ssoci ted with evers unless there is stul or eg colon, nd
The correct answer is “C.” T e ess ge is th t everybody re- is chronic condition, not cute. Pyelonephritis (“D”) w s good
porting persistent blood in stools without lesions seen on nos- thought until the urine dipstick returned nor l. Urin ry sy p-
copy w rr nts endoscopic ev lu tion. Colon c ncer is r re in to s re co on in diverticulitis bec use o bl dder irrit tion.
this ge group but c n h ppen. T e sy pto o red blood per Colon c ncer with l rge bowel obstruction (“B”) would present
rectu suggests th t the c use could be ound dist l to descend- with severe bdo in l p in nd distention. In obstruction, bowel
ing colon nd likely in the rectu . I there is only single epi- sounds re typic lly hyper ctive with inter ittent rushing.
sode o bleeding, so e would rgue or ore conserv tive
ppro ch in p tients less th n 50 ye rs o ge without other Question 7.17.2 The most appropriate next step in the
l r sy pto s (constitution l sy pto s or ch nge in bowel workup o the patient is:
h bits). In this young p tient, ull colonoscopy is not neces- A) C sc n o bdo en nd pelvis.
s ry unless there is strong ily history o colorect l c ncer B) Surgic l consult.
or other sy pto s suggestive o colitis such s urgency, weight C) G stroenterology consult.
loss, or di rrhe . Anorect l no etry is not use ul in the ev l- D) Abdo in l ultr sound.
u tion o rect l bleeding. Anusol suppositories c n be help ul to E) Colonoscopy.
reduce p in ssoci ted with he orrhoids, but i bleeding is the
only sy pto , ber supple ents nd stool so eners re su - Answer 7.17.2 The correct answer is “A.” C sc n is very
cient. I the p tient is older th n 50 ye rs with rect l bleeding, sensitive nd speci c or diverticulitis nd c n si ult neously
ull colonoscopy is indic ted. ev lu te or other c uses o bdo in l p in. R diocontr st (or l
or IV) does not dd to the di gnostic ccur cy or diverticu-
litis or ppendicitis (Ann Emerg Med. 2010;55:51) but try tell-
CASE 7.17 ing th t to your r diologist. Co pro ise with the nd use
A 73-year-old male comes to your o ce with a 3-day history IV contr st lone i they insist. In gener l, C sc n is indic ted
o le lower quadrant abdominal pain. He has elt cold and i the p tient h s peritone l signs or ss suggesting diver-
clammy at times but has not checked his temperature. He ticul r bscess or tion. In p tient who h s h d previously
has had no nausea, vomiting, or diarrhea. T e pain does not docu ented tt cks nd who h s none o the bove sy pto s,
worsen a er meals, but his appetite has been poor since this e piric tre t ent is ppropri te. Surgic l nd/or GI consults
started. On review o systems, he reports increased urinary y be indic ted but re pre ture t this point. Abdo in l
requency and urgency or the same amount o time. He has ultr sound c n di gnose diverticulitis nd/or bscess, but it is
had no abdominal surgeries. He has diabetes mellitus type 2 less sensitive th n C sc n. Colonoscopy is only indic ted in
and is on glipizide. He also takes aspirin 81 mg/day. He has the cute setting i obstruction is present or i colitis is thought
always declined screening colonoscopy, stating, “I it ain’t to be ore likely. In the setting o cute diverticulitis, the risk o
broke, don’t x it”—apparently, his amily motto. On exami- per or tion during colonoscopy is incre sed, nd colonoscopy
nation, he is in no distress with blood pressure 125/75 mm is pre er bly del yed until in tion h s subsided. Note th t
Hg, pulse 90 bpm, respirations 15, and temperature 38.5°C. you speci c lly do not w nt to use rect l contr st i there is
His heart sounds are normal, and his chest is clear bilater- question o per or tion (or th t you y c use per or tion).
ally. He has moderate le lower quadrant tenderness without
rebound tenderness or guarding. T ere is no abdominal dis- You get the C scan the same day (with only IV contrast . . . you
tention or organomegaly. Bowel sounds are normal, and the partially win this round with the radiologist) and have the
rectal examination is normal without stool in the rectal vault. patient return to your o ce to discuss the results. T e C
Urinalysis is normal. scan shows in ammation in the sigmoid colon with some out-
pouching structures suggesting diverticulosis. T ere is a 1.5 cm
Question 7.17.1 What is the most likely diagnosis? uid collection posterior to the sigmoid colon suggesting
A) Ische ic colitis. pericolonic abscess. No other ndings were noted, and the
B) Colon c ncer with l rge bowel obstruction. colon above sigmoid appeared normal. No ree air was seen.
234 FAMILY MEDICIn E EXAMIn ATIo n & Bo ARD REVIEW

Question 7.17.3 Which o the ollowing is the most appro going on to develop sy pto tic diverticulitis. Acute divertic-
priate next step in management? ulitis h s v riety o present tions. Peri-diverticul r in -
A) C -guided dr in ge o the bscess. tion occurs when ec lith beco es entr pped in diverticul r
B) Surgic l consult or i edi te diverting colosto y nd w ll resulting in loc lized, cont ined, icroper or tion. P in
bscess dr in ge. is typic lly cute nd loc ted in the le lower qu dr nt. Ex i-
C) Ad ission to the hospit l or IV ntibiotics nd seri l n tion y reve l only ildly tender bdo en without ny
bdo in l ex in tions. sses. A peri-diverticul r bscess nd phleg on will typic lly
D) Disch rge to ho e on levo ox cin nd etronid zole. result in worsening le lower qu dr nt bdo in l p in, nd
E) GI consult or endoscopic ultr sound guided tr nscolonic o en ss is p lp ble.
dr in ge o bscess.
HELPFUL TIP:
Answer 7.17.3 The correct answer is “C.” T is p tient should
Epipl ic appe dagiti ( t appe diciti , ye it i pelled
be d itted or IV ntibiotics. Mild tt cks o diverticulitis c n
c rrectly) ca mimic b th appe diciti a d diverticuli-
be n ged on n outp tient b sis. Our p tient h s s ll bscess
ti . It ge erally will pre e t imilarly t appe diciti but
th t requires inp tient ther py. In ddition, he is i unosup-
the le t. It ccur m tly i pe ple i their 30 . It c-
pressed by his di betes, nd ll i unosuppressed p tients
cur whe there i t r i r i arcti a epipl ic
with diverticulitis should be d itted or IV ntibiotics since
appe dage the perit eal a pect (“ ut ide”) the
they re ore likely to develop co plic tions nd need surgery.
c l . Diag i i by CT ca a d it ge erally re lve
“A” is not the best choice. It is re son ble to sk the r diologist i
it w with ut urgical i terve ti .
this bscess c n be dr ined, but the likely nswer is no, bec use
it is very s ll nd is posterior to colon. “B” is lso incorrect.
T e p tient does not h ve peritone l signs, nd the bscess
likely will respond to IV ntibiotics. T us, i edi te surgery is T e patient is admitted and he responds to IV antibiotics and
not indic ted. As discussed bove, colonoscopy ( nd by exten- supportive measures. A repeat C scan 2 weeks later shows
sion endoscopic ultr sound) is rel tively contr indic ted in the resolution o the abscess. T e local gastroenterologist per-
setting o cute diverticulitis. “E” is lso incorrect. No g stroen- orms colonoscopy 2 months a er the attack and con rms
terologist h s been ound cr zy enough to try to dr in bscesses le -sided diverticulosis with otherwise normal colonoscopy.
by tr nscolonic ppro ch to our ( nd Pub ed’s) knowledge.
However, ybe the e erging N tur l Ori ce r nslu in l Question 7.17.5 Which o the ollowing statements is true
Endoscopic Surgeons (NO ES) will tte pt this in the uture about this patient’s prognosis?
but not on our p tient tod y. (Editor’s note: we didn’t ke this A) When the p tient h s his second tt ck o unco plic ted
one up either . . . T ey h ve been doing cholecystecto ies vi diverticulitis, resection o the dise sed seg ent is lw ys
tr nsv gin l ppro ch . . . don’t sk us why . . .) indic ted (sig oid colecto y).
B) 50% o p tients will h ve repe t tt ck within 5 ye rs.
C) He h s 20% to 30% ch nce o diverticul r per or tion in the
HELPFUL TIP: next 2 ye rs.
A tibi tic regime r diverticuliti mu t i clude b th D) He h s 30% ch nce o diverticul r bleeding in the next
Gram- egative a d a aer bic c verage. s me c m- 2 ye rs.
m regime i clude cipr l xaci + metr idaz le, E) 33% o p tients will h ve second tt ck.
ampicilli / ulbactam, am xicilli /clavula ate, ampi-
cilli + ge tamici + cli damyci , a d ce triax e + Answer 7.17.5 The correct answer is “E.” Only 33% o p tients
metr idaz le. with n episode o diverticulitis will h ve second episode.
T ere is no set rule or the nu ber o tt cks needed be ore
p rti l colecto y is indic ted. T e co only used rule o three
Question 7.17.4 Which o the ollowing is true o diverticu tt cks is not b sed on prospective evidence. T e decision h s
losis? to be individu lized, but the tendency is to oper te on he lthy
A) T e jority o p tients with this dise se will develop sy p- young people with requent tt cks while opting or observ tion
to s t so e ti e. o elderly p tients with co orbidities (even i they h ve ore
B) T e condition is ssoci ted with high lign nt potenti l. th n three tt cks). Diverticul r per or tion is r re nd occurs
C) T e condition h s pe k incidence o occurrence in sixth, in only 5% to 10% o p tients within 2 ye rs o the initi l event.
seventh, nd eighth dec des o li e. Diverticul r bleeding h ppens in 3% to 5% o p tients with
D) T e condition pri rily ects the scending colon. diverticulosis nd the risk is not incre sed with diverticulitis; i
GI bleeding occurs during n episode o presu ed diverticuli-
Answer 7.17.4 The correct answer is “C.” Diverticulosis is n tis, other di gnoses should be strongly considered.
cquired dise se th t pe ks in the sixth, seventh, nd eighth
dec des with bout 50% o octogen ri ns h ving the condition. On a weekend call 6 months later, your patient presents to
“A” is incorrect. Most re asymptomatic with only 10% to 20% the emergency department a er experiencing sudden onset
CHAPTER 7 • GAs TRo En TERo Lo GY 235

o bright red blood per rectum mixed within stools. He has


passed ve stools in last 3 hours and the last one had blood QUICK QUIZ: CAn ’T s To P PUKIn G
clots. He eels dizzy but has not passed out. He stopped taking
his aspirin a er his last illness, but otherwise his health and M ry J ne is 27-ye r-old e le who presents to your clinic
medications are unchanged. His abdomen is nontender and co pl ining o recurrent bdo in l p in nd vo iting d ily
bowel sounds are normoactive. Rectal examination reveals or the l st 6 onths. T e only w y th t she c n relieve the
resh blood on the glove but no masses in rectum. Hemoglo- sy pto s is to either sit in hot b th or t ke hot shower. She
bin is 10 g/dL, and you place two large bore IVs and admit does this co pulsively nd is the cle nest p tient you h ve ever
him to the hospital. He has no more bowel movements over- seen; she liter lly shines. In this c se, though, cle nliness y
night, his hemoglobin is 8.2 g/dL the next morning, and he is not be next to godliness. Soci l history is re rk ble or d ily
eeling well. You assume that the bleeding was diverticular. use o riju n , tob cco use o ½ p ck per d y, nd bout 1 to
2 lcoholic drinks per d y. When her b ck hurts, she will t ke
Question 7.17.6 Which o the ollowing statements is true? her boy riend’s oxycodone— bout once per week.
A) Urgent colonoscopy is needed to loc lize nd tre t the lesion.
B) Colonoscopy is needed but c n be done on n outp tient T is beh vior is typic l or which o the ollowing disorders?
b sis. A) Cyclic vo iting syndro e.
C) gged RBC sc n ollowed by ngiogr phy is indic ted to B) C nn bis hypere esis syndro e.
prevent rebleeding. C) Opioid bowel syndro e.
D) No urther workup is indic ted t this point. D) C t eni l cyclic vo iting syndro e.
E) Sig oid colon resection is indic ted bec use o the dyn ic E) Vo it-Shower-Vo it syndro e.
diverticul r duo (diverticul r bleed nd diverticulitis).
The correct answer is “B.” T is history is typic l o c nn bis hy-
Answer 7.17.6 The correct answer is “D.” T e p tient h s cl s- pere esis syndro e. T is occurs in p tients who re requent
sic present tion o diverticul r bleeding with r pid onset nd riju n users. C nn bis hypere esis syndro e is resist nt to
spont neous resolution o bleeding (75% stop spont neously). typic l ntie etics, but p tients will h ve n b te ent o their
T e p tient underwent colonoscopy recently nd the only nd- sy pto s while they re in b th or shower (why is unknown).
ing w s diverticulosis. Since the p tient h s stopped bleeding, T e syndro e resolves once they stop using riju n . “A”, cy-
colonoscopy is unlikely to help in the n ge ent o his condi- clic vo iting syndro e, occurs in a stereotypical pattern; or
tion. For the s e re son, colonoscopy does not need to be done ex ple, p tient y h ve onset t 3:00 a m (2:00 to 7:00 a m
s n outp tient. I the p tient h d continued to bleed, there is the ost co on ti e) with preceding n use , p llor, b-
would be two options or n ging this p tient nd both re do in l p in nd norexi . Children usu lly h ve 12 cycles per
ccept ble. One ppro ch is to per or r pid colonic l v ge by ye r s co p red to 4 in dults. E ch cycle y l st sever l d ys
pl cing n NG tube nd giving 6 L (1.5 g llons) o polyethylene or ore nd the p tient h s nor l he lth in between episodes.
glycol (GoLytely) over 4 hours nd per or colonoscopy to try T is y be igr ine v ri nt nd nti igr ine edic tions
to nd the bleeding site nd tre t it. O en ultiple blood lled re worth trying. “C”, opioid bowel syndro e, re ers to blo ting,
diverticuli re seen, nd the source o the bleed c nnot be iden- p in, distension nd vo iting ound in opi te users (not just
ti ed. T e second ppro ch is to per or t gged RBC sc n to due to constip tion). It is prob bly hyper lgesi syndro e nd
con r ctive bleeding, ollowed by selective ngiogr phy to resolves when opi tes re stopped (good luck with th t!). “D,”
identi y the bleeding vessel nd e bolize it. T e ppro ch t ken c t eni l cyclic vo iting syndro e, is cyclic vo iting rel ted
depends on the loc l expertise v il ble. Without tre t ent, to enses, which is not described in this c se since it is occur-
25% o p tients eventu lly rebleed nd o those who rebleed, ring d ily. “E” sounds silly nd ction l—which it is.
50% will h ve third bleed.
Clinical Pearls
HELPFUL TIP: All patie t 50 year ld r lder with at lea t a 10-year li e
The wh le thi g ab ut av idi g eed a d ut i th e expecta cy h uld u derg r uti e c l ca cer cree i g;
with diverticular di ea e i —pu i te ded— ut . I vari u meth d are acceptable.
act, patie t wh eat p pc r , ut , eed , etc., have a D t re er patie t r ch lecy tect my i they have
lower i cide ce recurre t diverticular di ea e (JAMA. a ympt matic ch lelithia i .
2008;300:907). G d thi g, t , p pc r ha helped D t re er patie t r upper e d c py (EGD) i they have
uel ur b k editi g! typical heartbur a d alarm ympt m (e.g., dy phagia,
weight l ).
Feed patie t with pa creatiti ce their au ea a d pai
Objectives: Did you learn to . . . begi t ub ide. Early eedi g i a ciated with better ut-
• Ide ti y ig a d ympt m diverticuliti ? c me . There i eed r a jeju al tube i they t lerate ral
• Appreciate the atural hi t ry diverticular di ea e? eedi g .
• Ma age c mplicati diverticular di ea e?
236 FAMILY MEDICIn E EXAMIn ATIo n & Bo ARD REVIEW

Heidelb ugh JJ, Sherbondy M. Cirrhosis nd chronic liver


Ga tr pare i ca be cau ed by diabete a d al by viral
ilure: p rt II. Co plic tions nd tre t ent. Am Fam
GI i ecti , thyr id di ea e, a d aut immu e di ea e.
Physician. 2006;74(5):767–776.
P tviral ga tr pare i ca la t r up t a year.
J cobs DO. Clinic l pr ctice. Diverticulitis. N Engl J Med.
I a cree i g c l c py i c mpletely egative, d t 2007;357:2057–2066.
repeat c l ca cer screening a y ki d r 10 year .
Ji ng D. C re o chronic liver dise se. Prim Care. 2011;
M t pa creatiti i cau ed by gall t e r alc h l; ther 38:483–498; viii-ix.
cau e (viral, drug i duced, elevated triglyceride , c rpi L nz FL, et l; Pr ctice P r eters Co ittee o the
ti g , etc.) are much le c mm . A eric n College o G stroenterology. Guidelines or
s cree r hepatiti C at lea t ce i all U.s . adult b r prevention o NSAID-rel ted ulcer co plic tions. Am J
betwee 1945 a d 1965. Gastroenterol. 2009;104:728–738.
There i ab lute umber epi de diverticuliti that Longstreth GF, et l. Function l bowel disorders. Gastroenter-
ma date urgery. Tail r the treatme t t the patie t. ology. 2006;130:1480–1491.
Titrate pr t pump i hibit r therapy t the l we t t lerable M l ertheiner P, et l. Peptic ulcer dise se. Lancet. 2009;
d e— r di c ti ue, i p ible—a ter i itial therapy r 374(9699):1449–1461.
GERD. Remember that there i reb u d hyperacidity whe Mo yyedi P, et l. T e e ect o ber supple ent tion on
t ppi g PPI r H2 bl cker . irrit ble bowel syndro e: syste tic review nd
et - n lysis. Am J Gastroenterol 2014;109:1367–1374.
Nery F, et l. C uses nd consequences o port l vein thro bo-
BIBLIOGRAPHY sis in 1,243 p tients with cirrhosis: results o longitudin l
study. Hepatology. 2015;61:660–667.
Biesiekierski JR, et l. No E ects o gluten in p tients with
sel -reported non-celi c gluten sensitivity er diet ry Rockey DC. Occult g strointestin l bleeding; pri ry c re.
reduction o er ent ble, poorly bsorbed, short-ch in N Engl J Med. 1999;341(1):38–46.
c rbohydr tes. Gastroenterology. 2013;145:320–328. Sw roop VS, et l. Severe cute p ncre titis. JAMA. 2004;291:
Eisenberg JM. M n ging chronic g stroesoph ge l re ux dis- 2865–2868.
e se. AHRQ Co p r tive E ectiveness Reviews. ck J, et l. Function l g stroduoden l disorders. Gastroenter-
www.e ectivehe lthc re. hrq.gov/gerdupd te.c . ology. 2006;130:1466–1479.
Accessed J nu ry 16, 2012. United St tes Preventive Services sk Force. Screening or Colon
Ford AC, et l. Ef c cy o prebiotics, probiotics, nd synbiot- C ncer. 2008. http://www. hrq. gov/clinic/uspst /uspscolo.
ics in irrit ble bowel syndro e nd chronic idiop thic ht . Accessed J nu ry 16, 2012.
constip tion: syste tic review nd et - n lysis. Am J Veter ns A irs. Guidelines or the re t ent o Hep titis C.
Gastroenterol. 2014;109:1547–1561. http://www.hep titis.v .gov/pd /tre t ent-
Heidelb ugh JJ, Bruderly M. Cirrhosis nd chronic liver consider tions-2015–07.pd . Accessed October 15, 2015.
ilure: P rt I. Di gnosis nd ev lu tion. Am Fam Whico b DC. Clinic l pr ctice. Acute p ncre titis. N Engl J
Physician. 2006;74(5):756–762. Med. 2006;354:2142–2150.
Infectious Diseases
Margo Schilling
8
CASE 8.1 Oh, no! T e virus has struck. Like a zombie apocalypse,
it started with just a ew cases, but now it’s out o control.
Flu season is right around the corner and you are preparing
Every other patient who calls complains o in uenza-like
your clinic or the onslaught. First things rst . . . you need to
illness.
know how much vaccine to order and who will be receiving
it. T e Centers or Disease Control and Prevention (CDC) Question 8.1.2 During this outbreak, what intervention(s)
annually publishes recommendations or administering is/are most appropriate or all your unvaccinated, rail nurs-
in uenza vaccine to the American public. ing home patients who have no symptoms o ebrile respi-
Question 8.1.1 The CDC recommends vaccination or all o ratory illness?
the ollowing groups, EXCEPT: A) Antiviral prophylaxis with oseltamivir
A) Healthcare workers B) Antiviral prophylaxis with amantadine
B) Nursing home residents C) In uenza immunization
C) Egg-allergic, ebrile neonates D) A and C given together
D) Diabetics E) B and C given together
E) T e elderly Answer 8.1.2 The correct answer is “D.” Persons at high risk
Answer 8.1.1 The correct answer is “C.” We start this chapter or complications o in uenza can still be vaccinated a er an
with an easy one. Patients 6 months o age or younger should outbreak o in uenza has begun in the community, but develop-
not be vaccinated nor should children who are ebrile. T e ment o antibodies in adults can take up to 2 weeks. T us, che-
only other absolute contraindication to in uenza vaccination moprophylaxis should be considered or persons at high risk
is a known hypersensitivity to eggs or to other components o during the time rom vaccination until immunity has devel-
the in uenza vaccine. Patients with an egg allergy can get the oped. “A” alone might be appropriate or individuals who have a
recombinant u vaccine. T e egg based vaccine can still be used contraindication to vaccination and wish to protect themselves
in those who’s only reaction to eggs is hives i they are observed rom in uenza. “B” is not correct because in uenza A is increas-
or thirty minutes a er the injection (CDC 2016). ingly resistant to M2 drugs (e.g., amantadine, rimantadine), and
in uenza B has never been sensitive them. Vaccination alone
HELPFUL TIP: (“C”) can be used in individuals without known high-risk con-
Vaccinate all persons older than 6 months ann ally. ditions (chronic disorders including asthma, diabetes, renal
There are several types o vaccine available: trivalent dys unction, immunode ciency or cardiovascular disease). But,
and q adrivalent inactivated in l enza vaccines (the old- as noted above, vaccine alone is inadequate or those who are
ashioned l shot), intradermal vaccine (e.g., Fl zone) or institutionalized or have signi cant comorbidities. T ese higher
patients 18 to 64 years o age, high-dose intram sc lar risk patients also need to be covered by oseltamivir until immu-
vaccine or the elderly, and the intranasal live atten ated nity has developed (2 weeks).
in l enza vaccine (e.g., Fl Mist). Live atten ated in l -
enza vaccine is indicated or healthy, nonpregnant per- HELPFUL TIP:
sons aged 2 to 49 years, incl ding healthcare workers— Sensitivity and speci icity o rapid diagnostic tests or
except for healthcare workers caring for very immu- in l enza (70–75% and 90–95%, respectively) are lower
nosuppressed patients such as bone marrow trans- than viral c lt re b t tend to be more clinically se l
plant patients. Contact with patients with HIV is not a since early treatment is req ired to achieve maxim m
contraindication to the live atten ated vir s. bene it rom antiviral dr gs.

237
238 FAMILY MEDICINE EXAMINATION & BOARD REVIEW

Answer 8.1.4 The correct answer is “B.” Moving sick patients


HELPFUL TIP:
only risks spreading the in ection to the hospital. In addition
For a speci ic patient, tests o the vir s or s sceptibil-
to measures above, employees should be assigned to one work
ity to speci ic antivirals are not clinically se l d e to
area only to prevent spread via the sta . Activities, visits, and
the time req ired to get res lts. However, trends in the
gatherings in central common areas should be curtailed during
comm nity g ide empiric therapy. Resistance varies by
the outbreak.
year. Resistance o in l enza A H1N1 to oseltamivir was
99% in 2009. However, in 2013 to 2014 98.2% o in l -
enza A H1N1 isolates tested were susceptible to osel- HELPFUL TIP:
tamivir. The CDC monitors and reports ann al in l enza In l enza vaccine is only abo t 50% e ective in n rsing
antiviral sensitivity res lts on their website, www.cdc. home patients. Still, there is some herd imm nity and
gov/ l . vaccination is recommended as the primary way to pre-
vent an in l enza o tbreak. Considering all the options,
vaccinating sta may be the most e ective method to
prevent n rsing home o tbreaks.
Question 8.1.3 Which o these patients would quali y or
antiviral therapy during an in uenza outbreak?
A) A healthy 29-year-old male who presents 72 hours a er Question 8.1.5 Time to update the pneumococcal vaccine
onset o symptom onset and has a positive rapid test or protocol at your nursing home! In accordance with 2015
in uenza CDC recommendations or older adults, you institute the
B) A 70-year-old patient hospitalized with in uenza who pres- ollowing schedule or all patients at age 65 who have not
ents 72 hours a er onset o symptoms yet received a pneumococcal vaccination:
C) A pregnant women who presents within 48 hours o symp- A) 23-valent pneumococcal polysaccharide vaccine (PPSV23)
tom onset and has a positive rapid test or in uenza once only
D) B and C B) 13-valent pneumococcal conjugate vaccine (PCV13) once
E) All o the above only
C) PPSV23 ollowed by PCV13 one year later
Answer 8.1.3 The correct answer is “D.” In outpatients, anti- D) PCV13 ollowed by PPSV23 one year later
viral drugs are reserved or high-risk groups including preg- E) Cowpox . . . lots and lots o cowpox
nancy, age > 65 years, signi cant underlying medical illness
(DM, immunosuppression, morbid obesity, asthma, etc.), nurs- Answer 8.1.5 The correct answer is “D.” Indications or pneu-
ing home residents and children age < 2 years. Normal risk mococcal vaccination include patients with chronic illness
outpatients should be prioritized lower on the treatment lad- at high risk or invasive pneumococcal disease (e.g., diabetes,
der. However, the CDC does recommend that antiviral drugs chronic pulmonary disease, and cardiovascular disease), institu-
be considered or otherwise healthy outpatients with con rmed tionalization, age 65 or older, immunocompromised state, and
in uenza who can start medication within 48 hours o symptom tobacco use. Note that the tobacco use indication is relatively
onset. All hospitalized patients should be treated even i it is recent and applies to all patients age 19 to 64 years old. T ere are
> 48 hours a er onset o symptoms. two pneumococcal vaccinations, (Pneumovax) and 13-valent
pneumococcal polysaccharide conjugate vaccine (PCV13 or
T e director o nursing at your community nursing home Prevnar-13). All adults aged 65 or older should receive both
calls about an outbreak o ebrile respiratory in ections. vaccines. PCV13 should be administered be ore PPSV23
In the last 24 hours, three patients have become ill on the when practicable. PCV13 is also recommended or patients
dementia care unit. All residents o the home were vacci- with end-stage renal disease, asplenia, HIV, and many other
nated with the current year’s in uenza vaccine in Novem- conditions. PVC13 vaccine is not indicated or those < age 65
ber. Several o the aides have not been vaccinated and two only on the basis o smoking or the presence o lung disease (as
recently le work a er complaining o eeling tired, everish, o 2015). However PPSV23 is indicated or both smokers and
and achy. those with COPD who are < 65 years o age.

Question 8.1.4 You suspect an in uenza outbreak and take


the ollowing actions EXCEPT: HELPFUL TIP:
A) Quarantine the nursing home and restrict access or visitors, The most common ca se o bacterial pne monia com-
new admissions, and ill sta plicating in l enza is S. pneumoniae. However, Staphy-
B) Hospitalize all patients suspected o having in uenza lococcus aureus pne monia ( s ally ncommon in the
C) Limit the interaction o ill residents with non-ill residents comm nity) is an important entity d ring in l enza
D) Administer antiviral prophylaxis to all well residents o tbreaks and generally presents with more severe
E) Provide in uenza vaccine to any unvaccinated residents and symptoms.
sta
CHAPTER 8 • INFECTIOu S DISEASES 239

Objectives: Did you learn to . . .


• Describe who sho ld receive in enza and pne mococcal CASE 8.2
vaccines? An 80-year-old emale ell, broke her hip, and underwent
• Identi y appropriate interventions to halt the transmission o intraoperative repair with pinning o the racture. She devel-
in enza in comm nity and care acility o tbreaks? oped a local in ection at the site o the repair and was treated
• Lead a s ccess l in enza prevention program in a health- with a 10-day course o oral clindamycin. She was trans erred
care setting? to the nursing home or rehabilitation and has developed
• Prescribe in enza antivirals appropriately? loose, watery stools. oday when you visit her, she reports
eeling di use abdominal discom ort and has had 10 bowel
movements. She is very concerned because she cannot work
QUICK QUIZ: THE COMING PLAGu E with the therapist and risks losing her Medicare bene t or
skilled nursing.
T e clinic triage nurse comes to you with a concern about a
patient who has just walked into clinic and is at the ront desk. Question 8.2.1 You plan to do the ollowing:
She is a 28-year-old Peace Corps worker who returned 10 days A) Begin loperamide (Imodium) as needed to prevent diarrhea
ago rom a western A rican country in the midst o an Ebola during the therapy sessions.
epidemic. She is otherwise healthy but started having a ever B) Obtain stool specimens or Clostridium di cile toxin.
suddenly last night. She describes atigue, night sweats, myal- C) Obtain an abdominal C scan looking or evidence o
gia, and temperature o 38.5°C. T is morning she notes vom- obstruction.
iting and diarrhea with blood in the stool. T e patient thinks D) Ask your avorite gastrointestinal (GI) doctor to per orm
that she has a benign viral illness (“everyone was coughing on emergency endoscopy or evaluation o lower GI bleed.
the plane”) and does not recall any contacts with persons with E) Obtain stool specimens or ova and parasites.
Ebola.
Answer 8.2.1 The correct answer is “B.” C. di cile is the most
You tell the nurse: common bacterial cause o in ectious diarrhea in hospitalized
A) “We will work her in. Just have her hang out in the waiting patients in the United States (Campylobacter jejuni is the most
room.” common bacterial cause overall). Multistep diagnostic algo-
B) “Give her any mask you can nd and send her to the ER. rithms using polymerase chain reaction (PCR) or the toxin
Pronto.” gene(s) have the best test per ormance characteristics (sensitiv-
C) “Isolate the patient immediately in an examination room ity 68–100% and speci city 92–100%). esting should be per-
(pre erably negative laminar ow i available) and call the ormed on only symptomatic patients. T ere is no bene t rom
health department.” testing more than one stool specimen. Although symptomatic
D) “Shut down the whole clinic! We’re on lockdown!” therapy is important, “A” is incorrect because antiperistaltic
agents should be avoided in patients with C. di cile. Although
The correct answer is “C.” T is patient has been in an area o other causes o diarrhea are possible, the most cost-e ective
Ebola epidemic, has the classic symptoms o Ebola ( ever, di- approach in this patient would be laboratory testing or C. di -
arrhea, vomiting, bleeding [in this case in her stools]). Other cile according to local laboratory protocol prior to any other
symptoms include meningoencephalitis, di use rash, a relative more invasive procedure.
bradycardia or the degree o ever (similar to that seen with
typhoid). reat her as though she has Ebola until proven other- HELPFUL TIP:
wise. Ebola is highly contagious and is spread by contaminated The gold standard test or detecting C. di icile in the
body uids, including aerosolized droplets. You should endeav- stool is the cytotoxin assay, b t it is not commonly per-
or to limit exposure o others to the ill patient, so “A” and “B” ormed beca se it takes p to 2 days to per orm and is
are incorrect, as both options expose more persons. T e health more costly.
department in your area should be involved immediately, and
they would be in contact with the CDC. T e CDC recommends
symptomatic patients at risk or Ebola exposure be isolated A stool specimen reveals the presence o C. dif cile toxin A
(a private examination room with doors closed is likely suf - gene. T e patient is nontoxic with a normal complete blood
cient) and treated at the direction o public health authorities. count (CBC) and creatinine.
“D” is incorrect. Despite its contagious nature, causal exposure
to a patient with Ebola (e.g., sitting 10 away in the reception Question 8.2.2 The pre erred, rst line, treatment o this
area as she walks by) is unlikely to result in transmission. Re- patient should include:
member that the persons at highest risk during the 2014 to 2015 A) Di cid ( daxomicin)
Ebola outbreak in West A rica were healthcare workers and B) Lactose restriction and acidophilus milk products
caregivers o the Ebola patients. T e incubation period o Ebola C) Metronidazole 500 mg PO ID or 10 days
is 2 to 21 days. T e virus can persist in the sperm, eye, breast D) Vancomycin 250 mg PO QID or 10 days
milk and CNS or months a er symptoms have resolved. E) B and C
240 FAMILY MEDICINE EXAMINATION & BOARD REVIEW

Answer 8.2.2 The correct answer is “C.” reatment o C. di - Answer 8.2.4 The correct answer is “D.” Relapse and/or recur-
cile in ection (CDI) includes supportive care, discontinuation rence is very common. ypically about 20% o patients have
o the o ending antimicrobial agent, and initiation o oral met- a recurrence o symptoms within about 1 week o completing
ronidazole 250 mg our times daily or 500 mg three times or therapy. T ese patients usually respond to re-treatment with
10 days. Metronidazole has been pre erred over vancomy- either metronidazole or oral vancomycin. IV vancomycin is
cin (“D”) or the treatment o mild CDI because o lower cost ine ective treatment o C. di cile since vancomycin does not
and lower theoretical risk o promotion o vancomycin resis- enter into the GI tract rom the vascular space.
tant Enterococcus aecalis (VRE). Based on a recent study that
showed lower clinical success rate or metronidazole versus
HELPFUL TIP:
vancomycin, it may be reasonable consider vancomycin rst
C. di icile toxin degrades at room temperat re. A stool
line or mild to moderate CDI and always i the patient has
that has been at room temperat re or 2 ho rs will like-
severe CDI (white blood cell [WBC] >15000, creatinine >1.5×
ly be negative or C. di icile toxin even i the toxin was
baseline). Di cid ( daxomicin) was approved or treating CDI
initially present. A potential ca se o C. di icile nega-
in 2011. Randomized studies demonstrate similar cure rates
tive pse domembrano s colitis is Klebsiella oxytoca.
between daxomicin and oral vancomycin, with ewer recur-
Consider this i stool st dies are negative or C. di icile.
rences with daxomicin. Fidaxomicin—at a much higher cost—
Finally, a positive stool culture is not diagnostic o in-
may be used or recurrent CDI or when there is a high risk o
ection. C. di icile is a normal g t lora. The presence o
recurrence. Fecal microbiota transplantation is associated with
toxin sho ld be demonstrated to diagnose C. di icile
resolution o recurrent CDI but its role in primary and severe
in ection or pse domembrano s colitis.
CDI has not been established. T ere is limited evidence that use
o probiotics is e ective in the treatment o C. di cile in ection
and prevention o recurrence. Please see help ul tip below. HELPFUL TIP:
Altho gh req ently sed in the prevention and treat-
Question 8.2.3 Risk actors or C. dif cile in ection in nurs- ment o CDI, the e ectiveness o probiotics is ncertain.
ing home patients include: G t lora is amazing (abo t 1014 bacteria rom 1,200 di -
A) Advanced age erent species live in symbiosis with yo ), and adding a
B) Recent acute hospitalization little acidophil s might not c t it. In partic larly recalci-
C) Proton pump inhibitor (PPI) use trant cases, ecal microbiota (stool, basically) transplan-
D) Long-term residence in a chronic care acility tation can be per ormed. Patients are given “healthy”
E) All o the above stool via NG t be or colonoscopy to reestablish normal
GI lora. Y m. Altho gh a little y cky, it appears q ite
Answer 8.2.3 The correct answer is “E.” Risk actors or acquisi- sa e.
tion o CDI in nursing home patients are similar to that o hos-
pitalized patients and include hospitalization, advanced age, GI
surgery/procedures, antibiotic exposure, and importantly PPI use. Objectives: Did you learn to . . .
• Recognize the presentation o C. di cile in ection?
HELPFUL TIP: • Identi y risk actors or the development o C. di cile
C. di icile diarrhea and colitis can be ca sed by any an- diarrhea and colitis?
tibiotic, incl ding metronidazole and vancomycin. The • Treat patients with initial and rec rrent C. di cile in ections?
probability o diarrhea seems highest with clindamycin.
Fl oroq inolones and broad spectr m cephalosporins
CASE 8.3
are increasingly associated with C. di icile in ection,
incl ding a highly toxigenic strain. Simply stopping the You are called to the emergency department (ED) to exam-
antibiotic can lead to resol tion in 25% o the cases. ine a 40-year-old man with ever (temperature o 39°C) and
headache. His past history is remarkable only or a splenec-
tomy secondary to trauma at age 10. He is not allergic to any
Failure to resolve C. dif cile-associated diarrhea a er a 10-day antibiotics. Upon examination you note that he has menin-
course antibiotic therapy is common. Guess what? Your geal signs. Nondilated undal examination shows sharp disc
patient’s diarrhea has persisted. margins, and he is neurologically intact with a non ocal
examination.
Question 8.2.4 The most reasonable treatment approach
is to: Question 8.3.1 The most appropriate action is:
A) Repeat a course o oral metronidazole A) Obtain a head C so that you can sa ely proceed with lumbar
B) reat with a course o oral vancomycin puncture (LP).
C) reat with a course o intravenous (IV) vancomycin B) Order IV penicillin as you prepare to per orm LP.
D) A or B C) Per orm a LP immediately and begin antibiotic therapy
E) B or C empirically.
CHAPTER 8 • INFECTIOu S DISEASES 241

D) Order IV erythromycin as you prepare to per orm LP. Question 8.3.3 An adjunctive therapy that has been shown
E) Order IV vancomycin and IV ce riaxone as you wait or the to improve neurologic outcomes in pneumococcal menin-
CBC. I the CBC is abnormal, you will do the LP. gitis is:
A) Dexamethasone
Answer 8.3.1 The correct answer is “C.” Once you suspect B) Activated protein C
bacterial meningitis, rapid diagnostic evaluation and emergent C) Vasopressors
treatment are imperative, including LP and blood cultures. I LP D) CSF shunt implantation
is going to be delayed, then appropriate empiric antimicrobial E) Monoclonal antibody directed against the capsule antigen o
and adjunctive therapy should be given without delay. Head the bacterium.
C is necessary only in those who are immunocompromised
(HIV/AIDS, those receiving immunosuppressive drugs, trans- Answer 8.3.3 The correct answer is “A.” T e In ectious Disease
plant recipients), have a history o CNS disease (brain tumor Society o America (IDSA) guideline recommends adjunctive
and stroke), develop new onset seizures, display papilledema on dexamethasone to be administered to all adult patients with
examination, or who have an abnormal/ ocal neurologic de cit pneumococcal meningitis. No other adjunctive therapy has
or abnormal level o consciousness. Antibiotics or a 40-year-old proven bene t. Since the organism is not usually known at the
male should cover Neisseria meningitidis and S. pneumoniae, and time o initiation o therapy, dexamethasone should be given
would include vancomycin and ce riaxone (“E”). However, “E” empirically until the causative organism is identi ed. I the
is not the best choice. Never wait or a CBC to determine i an meningitis is not due to S. pneumoniae, dexamethasone should
adult needs an LP. T e decision to do an LP is a clinical one. be discontinued.
Patients should receive standard supportive therapies in an
intensive care setting. Complications, when they occur, usually
HELPFUL TIP:
develop within the rst 2 to 3 days o therapy. Complications
I the preponderance o initial clinical and laboratory
include sepsis, mental status changes, and electrolyte abnor-
data indicate that bacterial meningitis is likely and the
malities.
LP cannot be done immediately, draw blood c lt res
and administer dexamethasone and appropriate antibi-
Question 8.3.4 Highly resistant S. pneumoniae in ections o
otics. Yo won’t change the CSF c lt re res lts (in most
the CNS should be treated with:
cases) i yo give a single dose o antibiotics be ore the
A) T ird-generation cephalosporin
LP. However, it is considered pr dent to do the LP with-
B) Vancomycin and a third-generation cephalosporin
in 2 ho rs o administering IV antibiotics. The standard
C) High-dose penicillin G
of care for suspected meningitis is to administer an-
D) Ampicillin
tibiotics within 30 minutes of the patient presenting
E) Vancomycin, gentamycin, and ri ampin
to the ED.
Answer 8.3.4 The correct answer is “B.” Vancomycin should
be combined with a third-generation cephalosporin (e.g., ce -
Results o the cerebrospinal uid (CSF) obtained a er LP are triaxone and ce otaxime) or highly resistant pneumococ-
as ollows: cloudy, WBC count 5,000 cells/mm3, 95%neutro- cus. Do not use vancomycin alone; it does not cover Gram-
phils, glucose 20 mg/dL, and Gram-positive cocci in pairs. negative organisms. T e newer generation uoroquinolones
have enhanced in vitro activity against S. pneumoniae and may
Question 8.3.2 The most likely pathogen is: be used as alternative agents. However, uoroquinolones are
A) S. pneumoniae not recommended unless the patient cannot tolerate or is
B) Listeria monocytogenes allergic to standard drugs.
C) S. aureus
D) N. meningitidis HELPFUL TIP:
E) Pseudomonas species Resistance to penicillins (and others) is noted by the
minim m inhibitory concentration (MIC). For penicillin,
Answer 8.3.2 The correct answer is “A.” Gram stain exami- highly resistant pne mococc s has an MIC o >2 µg / mL,
nation o CSF may permit rapid identi cation o the causative intermediate resistance is 0.12 to 1 µg / mL while sensi-
organism in bacterial meningitis with a sensitivity o 60% to tive is <0.06 µg / mL.
90%. Prior antibiotic therapy (e.g., a partially treated menin-
gitis . . . not a single dose o antibiotics in the ED) may reduce
the sensitivity by 20%. T e likelihood o a positive Gram stain HELPFUL TIP:
is highest in cases o S. pneumoniae (a Gram-positive diplo- The combination o Kernig and Br dzinski signs carries
coccus). Only about one-third o L. monocytogenes menin- a sensitivity and speci icity o 5% and 95%, respectively.
gitis cases demonstrate a positive Gram stain. “D” requires Th s, the great majority o patients do not mani est
special mention. N. meningitidis is a diplococcus but is Gram these signs when they have meningitis, and they are
negative.
242 FAMILY MEDICINE EXAMINATION & BOARD REVIEW

H. in uenzae type B (all encapsulated bacteria). New recom-


seless to r le o t meningitis. The sensitivity and speci-
mendations or all pneumococcal vaccine-naive immunosup-
icity o n chal rigidity (sti neck) is 30% and 68%, re-
pressed adults, including those with asplenia, advise vaccina-
spectively. In ad lts, the classic triad o ever, n chal
tion with a dose o PCV13 rst, ollowed by a dose o PPSV23
rigidity, and altered mental stat s was o nd in only
at least 8 weeks later. Splenectomized individuals should be
46%, with 85% having ever, 70% having neck sti ness,
re-immunized against pneumococcus in 5 years with a second
and 67% having mental stat s changes.
PPSV23. Patients who have undergone splenectomy need to
be educated about seeking prompt medical attention or ever.
Objectives: Did you learn to . . .
• Diagnose meningitis? CASE 8.4
• Identi y the most likely ca sative organism based on epide-
miology and patient characteristics? A 74-year-old woman is in your of ce or a complete physical.
• Describe proper se o empiric antibiotics and steroid
As part o her routine labs, you obtain a urinalysis (although
therapy or bacterial meningitis? why is beyond us . . . there is no recommendation or a screen-
ing urinalysis or any nonpregnant patients). On urther
questioning, she has actually had stress urinary incontinence
QUICK QUIZ: BIG MENINGITIS ON CAMPu S or a number o years, which is unchanged. She reports no
evers, hematuria, dysuria, ank pain, or other symptoms.
What is the most common bacterial cause o meningitis in T e urinalysis shows 10 to 20 WBC/hp and is nitrite positive
college-aged patients who live in dormitories? on dipstick. A culture o the urine shows 100,000 c u/mL o
A) N. meningitidis E. coli.
B) S. pneumococcus
C) L. monocytogenes Question 8.4.1 How should you treat this patient?
D) Haemophilus in uenzae A) rimethoprim-sul amethoxazole DS 1 tab PO BID or
E) Escherichia coli 10 days
B) Erythromycin 500 mg PO q 6 hours or 14 days
The correct answer is “A.” Although S. pneumococcus is the C) Ampicillin 250 mg PO q 6 hours or 10 days
most common cause o bacterial meningitis in the adult pop- D) Ce riaxone 1 g IM every day or 3 days
ulation in the United States, N. meningitidis remains the lead- E) With courtesy and respect, but not with antibiotics
ing cause in adolescents despite vaccination and is particularly
prevalent in the setting o dormitory living (e.g., college or mili- Answer 8.4.1 The correct answer is “E.” A positive urine culture
tary). In addition, the presence o petechial (or purpuric) rash in an asymptomatic patient (i.e., asymptomatic bacteriuria)
in the lower extremities and pressure points is typical o N. men- should not be treated with antibiotics except during pregnancy.
ingitidis. T e advent o vaccination has made H. in uenzae a Asymptomatic bacteriuria is a very common nding, especially
less common cause. L. monocytogenes is more prevalent in those in elderly emales, persons with indwelling catheters, and insti-
over age 50, in ants, and the immunocompromised. E. coli is tutionalized persons. reatment does not reduce the incidence
a common cause or meningitis in neonates and in ants but is o symptomatic in ection. reatment also does not reduce mor-
very uncommon in adolescents and adults. tality in rail elderly patients, and does not improve chronic
urinary incontinence symptoms. Persistent asymptomatic bac-
teriuria does not result in renal insuf ciency or the develop-
QUICK QUIZ: SPLEENLESS IN SEATTLE ment o hypertension.

Patients undergoing elective splenectomy should receive all o Question 8.4.2 Which statement is true o asymptomatic
the ollowing EXCEP : bacteriuria?
A) PCV13 ollowed by PPSV23 at least 8 weeks later A) T e nding o pyuria in a urinalysis distinguishes urinary
B) H. in uenzae type B conjugate vaccine tract in ection (U I) rom asymptomatic bacteriuria and
C) In uenza vaccine guides treatment decisions.
D) Meningococcal vaccine B) T e prevalence o asymptomatic bacteriuria in women is
E) Oral polio vaccine unrelated to age, unction, or hormonal status.
C) Asymptomatic bacteriuria need not be treated in the preg-
The correct answer is “E.” Oral polio vaccine is no longer rec- nant patient.
ommended in the United States or anyone. T e risk o devel- D) None o the above.
oping polio rom the live vaccine outweighs the bene t o
oral administration. All patients undergoing elective splenec- Answer 8.4.2 The correct answer is “D.” None o the statements
tomy should receive preoperative vaccination against encap- are true. “A” is incorrect because the nding o pyuria with low
sulated organisms at least 14 days prior to splenectomy; this numbers o WBCs in a urinalysis specimen is nonspeci c, com-
includes vaccines against S. pneumoniae, meningococcus, and mon, and requently unrelated to in ection. “B” is incorrect. T e
CHAPTER 8 • INFECTIOu S DISEASES 243

prevalence o asymptomatic bacteriuria in women increases


HELPFUL TIP:
with age, declining unctional capabilities, and institutionaliza-
False-positive TSTs can be d e to Bacille Calmette–
tion. More than 10% o community-dwelling women over the
G erin (BCG) vaccine and in ection with other myco-
age o 65 and up to 50% o elderly women in nursing homes will
bacteria. However, most patients who had the BCG vac-
have asymptomatic bacteriuria. “C” is incorrect. Pregnancy is
cine will experience a decline in imm nity (a maxim m
one state in which treating asymptomatic pyuria/bacteriuria is
o 20% will have a positive TST d e to BCG 10 years a ter
indicated. T e risk o in ection rom asymptomatic bacteriuria
the vaccine was given). I there is concern abo t a po-
is high in pregnant patients.
tential alse-positive test, an inter eron gamma release
assay sho ld be per ormed. There can also be alse-
negative TSTs, especially in the imm nos ppressed.
HELPFUL TIP: I yo s spect impaired imm nity, se an inter eron
Treat asymptomatic bacteri ria in the ollowing pa- gamma release assay.
tients: those with rinary tract obstr ction ( nctional
or anatomic), nephrolithiasis, pregnancy, or planned
rinary instr mentation. In the past, men with as-
ymptomatic bacteri ria were considered candidates T e S is repeated 2 weeks later and is interpreted again as
or treatment. This is no longer the case. Men with as- 0 mm diameter o induration a er 48 hours. You reassure the
ymptomatic bacteri ria sho ld not be treated; it does patient that her tuberculin test is negative.
not improve o tcomes and leads to resistant organ-
isms (see http://www.choosingwisely.org/societies/ Question 8.4.4 What would most accurately represent a
in ectio s-diseases-society-o -america/). positive TST in THIS patient?
A) Erythema 5 mm in diameter
B) Induration 5 to 10 mm in diameter
C) Erythema o 10 mm in diameter
HELPFUL TIP:
D) Induration 10 mm in diameter
Not all py ria is rom u TI. Other pathologic ca ses o py-
E) Erythematous induration o any size
ria incl de vaginitis (in ectio s and atrophic), rethritis
(Chlamydia trachomatis, Neisseria gonorrhoeae ), and
Answer 8.4.4 The correct answer is “D.” Here is the bottom
genital herpes in ections.
line. Routine screening is not recommended or low-risk patients
(e.g., community dwelling individuals rom a low-risk coun-
try). Screen only the ollowing: contacts o those with B, HIV
A ew years later, the same patient is now 80 years old (you’re in ected patients, IV drug users, those with predisposing actors
still paying o your med school loans and will probably to B in ection (diabetes, immunosuppressive drugs, lung can-
work until you’re 80), and she is admitted to a nursing home. cer, etc.), oreign-born individuals arriving in the United States
Her initial tuberculin skin test ( S ) with 5-tuberculin unit within the last 5 years, healthcare workers, nursing home and
injection o puri ed protein derivative was interpreted as other institutionalized individuals and the homeless. Measure
0 mm diameter o induration a er 48 hours. the induration and not the erythema. T e de nition o a posi-
tive S changes with the population tested. See able 8-1.
Question 8.4.3 What is your next step?
A) Repeat a S now
B) Repeat a S in 2 weeks TABLE 8-1 INTERPRETATION OF TUBERCULIN
C) Repeat a S in 1 year SKIN TESTS
D) Declare the patient ree o B Diameter o
E) Obtain a chest x-ray Induration Positive in These Situations
5–10 mm Chest x-ray consistent with past or c rrent
Answer 8.4.3 The correct answer is “B.” Because the preva-
in ection; HIV positive; recent close contact o
lence o a positive S (Mantoux test) doubles between the person with active TB.
rst and second tests in initial nonresponders, the U.S. Public
Health ask Force recommends a two-step S by the Man- 10–15 mm Instit tionalized persons; IV dr g sers;
imm nocompromised states other than HIV;
toux method or screening high-risk populations (e.g., per-
children less than 4; children exposed to high-risk
sons living in nursing homes). I the rst S is negative, a ad lts, healthcare workers.
second test should be per ormed approximately 2 weeks later
in order to detect the “booster phenomenon.” Individuals ≥15 mm Persons witho t risk actors. These are people
who probably sho ld not have been tested in the
admitted to a nursing home with a positive S and no symp-
rst place (e.g., the general p blic witho t any
toms o active B may be presumed to have a distant history expos re to TB).
o in ection.
244 FAMILY MEDICINE EXAMINATION & BOARD REVIEW

examination, and radiographic ndings. “E” is incorrect. Since


HELPFUL TIP:
this patient has converted rom 0 to 12 mm induration in 1
Whole blood inter eron-gamma release assays, or IGRAs
year, her pretest probability is high enough that per orming an
(e.g., Q antiFERON-TB Gold test, T-SPOT) are Food and
IGRA adds nothing.
Dr g Administration (FDA)-approved or se in any sit -
ation in which the TST is c rrently employed. IGRAs are
more expensive and sho ld not be the rontline screen-
HELPFUL TIP:
ing test b t may help in sit ations in which the TST re-
A directly observed 3-month co rse o INH pl s
s lt is q estionable. IGRAs are not 100% sensitive and
ri apentine once weekly may be an eq al alternative
should not be sed in patients who are symptomatic;
to a 9-month co rse o daily sel -s pervised INH in
they are or asymptomatic screening only and cannot
otherwise healthy patients o at least 12 years o age.
di erentiate latent rom active in ection.
However recommendations remain nchanged and the
INH pl s ri apentine regimen is not recommended or
n rsing home elders.

A year later, the nursing home calls about her annual S .


You examine the patient and nd the diameter o indura-
tion is 12 mm. T e patient denies cough, weight loss, ever, HELPFUL TIP:
or chills. I active disease is diagnosed, appropriate therapy
sho ld be initiated with a 4-dr g regimen or 6 to
Question 8.4.5 The most appropriate next step in this case 8 weeks ollowed by a simpler regimen or 4 to 7 months
is to: once sensitivities are known (average 6 months total).
A) Obtain a chest radiograph and, i normal, initiate isoniazid A n mber o regimens are available, and all incl de
300 mg and pyridoxine or 9 months. isoniazid and ri ampin. INH alone is never appropri-
B) Obtain a chest radiograph and, i normal, observe the ate or active TB beca se o resistance. First-line dr gs
patient annually or signs o active B (weight loss, cough, sed or active TB incl de isoniazid, ri ampin, pyrazin-
ever, etc.). amide, and ethamb tol. Ethamb tol may be dropped
C) Obtain a chest radiograph and, i abnormal, initiate isonia- i the organism is sensitive to isoniazid, ri ampin, and
zid 300 mg once daily and pyridoxine or 9 months. pyrazinamide. Second-line dr gs incl de levo loxacin,
D) Obtain sputum specimens or acid- ast bacilli (AFB) stains streptomycin, and others. Regimens vary by location
and mycobacterial culture and sensitivity testing; and initi- and local resistance patterns. Contact yo r local health
ate therapy with isoniazid, pyridoxine, pyrazinamide, and department.
ri ampin or 6 months.
E) Obtain an inter eron-gamma release assay, and i negative,
observe the patient annually or signs o active B (weight
loss, cough, ever, etc.). T e patient’s son is somewhat distraught that his mother has
B. He asks i she should have received a vaccine be ore com-
Answer 8.4.5 The correct answer is “A.” T e patient lives in ing into the nursing home.
a high-risk setting (nursing home). T e results o her S
1 year ago suggest that she was not in ected with Mycobacte- Question 8.4.6 Regarding the BCG vaccine, which o the
rium tuberculosis at that time. She has newly converted—prob- ollowing statements is true?
ably due to exposure to an active case o B in the acility. A) Foreign-born persons who received the BCG vaccine should
T e risk o developing active disease ollowing B in ec- never have a S administered.
tion is greatest in the rst 2 years ollowing in ection. T e B) T e BCG vaccine is most ef cacious or older adults, and
risk/bene t ratio avors prophylactic therapy with isonia- children bene t much less rom the vaccine.
zid (INH) or the patient who has converted within the last C) A S in an individual with a remote history o BCG vac-
2 years, as long as there is no sign o active disease and the cine should be interpreted as i the BCG had not been given.
chest x-ray is negative. Pyridoxine should also be given to D) T e BCG vaccine is made rom killed M. tuberculosis.
prevent peripheral neuropathy. A negative chest x-ray does
not mean that you can orego prophylactic therapy, and thus Answer 8.4.6 The correct answer is “C.” T e S should be
“B” is incorrect. “C” is incorrect because an abnormal chest interpreted exactly the same way whether or not the patient has
radiograph must be ollowed with collection o sputum speci- received the BCG. “B” is incorrect. BCG is most ef cacious in
mens or AFB staining and culture. “D” is incorrect because children but protection rom the vaccine wanes over a ew years.
a chest radiograph is necessary to evaluate all patients with a Even in children, it is a poor vaccine, protecting children rom
positive Mantoux skin test. Sputum specimens should be col- B only about 50% o the time. Persons vaccinated with BCG
lected and multiple drug regimens initiated only in patients with should still be evaluated by S when appropriate and an indu-
signs and symptoms o active disease based on clinical history, ration o >10 mm (age <35 years) or >15 mm (age ≥ 35 years) is
CHAPTER 8 • INFECTIOu S DISEASES 245

considered positive. “D” is incorrect. T e BCG vaccine is made


rom attenuated Mycobacterium bovis. CASE 8.5
A 37-year-old woman with a history o mitral valve prolapse
Prior to starting INH, you measured the patient’s amino- and mitral regurgitation presents or evaluation. She reports
trans erase levels, which were normal. Now, 3 months into no symptoms o shortness o breath or exercise intolerance.
treatment, her alanine aminotrans erase (AL ) is 42 IU/L She plans to undergo health-screening procedures, including
(about twice the upper limit o normal). dental exams or routine cleaning and lling o several caries,
pelvic examination with removal o an intrauterine device
Question 8.4.7 Your next step is to: (IUD), and colonoscopy in the next year.
A) Stop her INH since she has had a ew months o treatment.
B) Continue the INH as scheduled and ollow up with clinical Question 8.5.1 According to the American Heart Associa-
and laboratory monitoring. tion (AHA) 2007 guidelines on prevention o in ective endo-
C) Switch to ri ampin and pyrazinamide. carditis, what should she receive prior to these procedures?
D) Re er her to a hepatologist or liver biopsy. A) Amoxicillin 2 g orally
E) Start her on milk thistle. B) Azithromycin 500 mg orally
C) Clindamycin 600 mg orally
Answer 8.4.7 The correct answer is “B.” While liver injury D) Nothing
is a signi cant problem with isoniazid, the drug need not be
stopped unless the liver enzymes rise to more than three times Answer 8.5.1 The correct answer is “D.” Even now it seems the
the upper limit o normal. Others would suggest that even then news has not made it to every corner o the world. But in 2007
the drug can be continued and that only signs and symptoms o there were major changes to the AHA guidelines on in ective
hepatitis ( atigue, anorexia, nausea, and vomiting) along with endocarditis prevention. T e one change that would seem to
urther liver enzyme elevations should prompt discontinuation a ect the greatest number o patients in primary care practices
o the drug. “A” is incorrect because she should have 9 months is the “downgrading” o mitral valve prolapse with regurgita-
o prophylactic therapy. “C” is incorrect because ri ampin and tion, which is no longer considered a high-risk condition. I the
pyrazinamide have potentially more hepatotoxicity than INH. patient had a condition or which prophylaxis was warranted,
Finally, “D” and “E” are incorrect. Neither liver biopsy nor all o the other regimens (“A,” “B,” “C”) are options depending
milk thistle (touted or its bene t in liver disease) is likely to on the patient’s allergies and other medications, conditions, etc.
be use ul.
Question 8.5.2 According to the AHA 2007 guidelines on
the prevention o in ective endocarditis, which o the ol-
HELPFUL TIP: lowing conditions is NOT a high-risk condition or the
u n ort nately, TB has developed resistance to n mero s adverse outcome o in ective endocarditis?
irst-line agents. There is dr g-resistant TB, m ltidr g- A) Bioprosthetic aortic valve
resistant TB (MDR-TB), and extensively dr g-resistant B) Mechanical aortic valve
TB (XDR-TB). The di erences: C) Congenital heart disease completely repaired with pros-
• Dr g-resistant TB is resistant to one o the irst-line thetic material
dr gs (INH, ri ampin, ethamb tol, streptomycin, pyra- D) Bicuspid aortic valve
zinamide). E) Previous history o in ective endocarditis
• MDR-TB is resistant to at least INH and ri ampin and
possibly more dr gs. Answer 8.5.2 The correct answer is “D.” T e guidelines recom-
• XDR-TB is resistant to at least INH, ri ampin, l oro- mend antibiotic prophylaxis or conditions considered to be high
q inolones, and aminoglycosides or capreomycin or risk or adverse outcomes o in ective endocarditis. High-risk
both. conditions include prosthetic valves (bioprosthetic homogra
and allogra valves and mechanical valves), previous in ective
endocarditis, and complex cyanotic congenital heart disease.
Objectives: Did you learn to . . .
• Determine who needs to be treated or asymptomatic bacte-
HELPFUL TIP:
ri ria?
Moderate-risk conditions, for which prophylaxis is not
• Interpret TST res lts?
indicated, incl de acq ired valv lar dys nction, s ch
• Describe the BCG vaccine and how patients who receive it
as rhe matic heart disease, hypertrophic cardiomyopa-
sho ld be approached?
thy, bic spid aortic valve, and mitral valve prolapse with
• Recommend appropriate treatment o a positive TST?
a sc ltatory evidence o valv lar reg rgitation and / or
• Recognize complications o isoniazid therapy? thickened lea lets. Hip replacement is also not a reason
• De ine and recognize the importance o dr g-resistant or ro tine dental prophylaxis.
TB?
246 FAMILY MEDICINE EXAMINATION & BOARD REVIEW

extraction, IV drug abuse, or invasive medical procedure.


HELPFUL TIP:
Symptoms generally begin insidiously and may include weak-
In ective endocarditis is m ch more likely to res lt rom
ness, atigue, ever, night sweats, arthralgias/myalgias, and
transient bacteremia that occ rs with ro tine dental
hematuria. “C,” echocardiography, is indicated. T e yield or
care at home, like br shing and lossing, than rom den-
visualization o vegetations or transthoracic echocardiogra-
tal, GI, and Gu proced res. Accordingly, good oral hy-
phy is 60% to 77% and increases to 96% with transesophageal
giene to lower the risk o bacteremia is more important
echocardiography. A prolongation o the PR interval on an
than prophylactic antibiotics.
electrocardiogram may suggest involvement o the cardiac
conduction system.

Question 8.5.3 I your patient had a mechanical aortic You care ully examine the patient and nd that she is ebrile
valve, appropriate endocarditis prophylaxis might include: and slightly tachycardic.
A) Ampicillin IV 2 hours prior to colonoscopy i biopsy o
lesions is anticipated Question 8.5.5 You look or signs o in ective endocarditis,
B) Ampicillin IV 2 hours prior to pelvic examination and IUD paying particular attention to all o the ollowing EXCEPT:
removal A) Osler nodes
C) Amoxicillin PO 2 hours prior to routine dental cleaning B) Painless erythematous macules on the palms and soles
D) Amoxicillin PO 2 hours prior to any injection o local anes- C) Splinter hemorrhages
thesia and lling o caries D) Painless nodules over bony prominences
E) All o the above E) Roth spots

Answer 8.5.3 The correct answer is “C.” For high-risk con- Answer 8.5.5 The correct answer is “D.” Classical physical
ditions (e.g., mechanical aortic valve), antibiotic prophylaxis examination ndings o SBE include intermittent ever; pete-
is recommended by the AHA prior to cleaning o teeth and chiae; conjunctival hemorrhage; splinter hemorrhages under
removal o plaque. T e risk o endocarditis is highest or the nails; erythematous pain ul nodules on the ngers, palms,
dental procedures that might traumatize the oral mucosa, and soles (Osler nodes); undic hemorrhages (Roth spots);
such as tooth extractions, periodontal procedures, and clean- painless erythematous macules on the palms and soles (Jane-
ing o teeth with removal o adherent plaque. “A,” “B,” and way lesions); and a new diastolic murmur. “D” is not a physical
“D” are incorrect. Prophylaxis is not recommended prior to examination nding in SBE. Painless nodules over bony promi-
these procedures. T e risk o endocarditis is low or proce- nences are observed in rheumatic ever and are one o the Jones
dures such as lower GI endoscopy and pelvic examination criteria. Remember that in a modern medical practice, most
with IUD removal, because the microorganisms likely to patients with SBE will not present with these ndings, and you
cause transient bacteremia ollowing these interventions are must maintain a high degree o suspicion or SBE in the appro-
not capable o adhering to cardiac valve tissues. Antibiotic priate clinical scenario.
prophylaxis is not recommended or restorative dental proce-
dures (e.g., llings).
HELPFUL TIP:
All o the evaluations, including the dental examination,
Laboratory eval ation in endocarditis may be remark-
seem to go well. However, 1 month later, she returns to see
able or anemia, le kocytosis, elevated erythrocyte sed-
you or gradually (perhaps even subacutely?) worsening
imentation rate (ESR), and microscopic hemat ria.
ever, malaise, and night sweats. You are concerned that she
may have developed in ective endocarditis.

Question 8.5.4 The evaluation o a patient suspected o Question 8.5.6 Which o the ollowing is/are included in
having subacute bacterial endocarditis (SBE) should include the major criteria o the modi ed Duke criteria or endo-
all o the ollowing EXCEPT: carditis?
A) T ree sets o blood cultures obtained at 1-hour intervals A) Positive blood cultures
within the rst 24 hours o assessment B) Janeway lesions (painless macules on palms and soles)
B) Auscultation o chest or evidence o new or changing C) Echocardiographic evidence o valvular vegetation
murmur D) A and B
C) ransthoracic or transesophageal echocardiogram E) A and C
D) Spiral chest C
E) Electrocardiogram Answer 8.5.6 The correct answer is “E.” T e modi ed Duke
criteria were developed to provide clinicians with standardized
Answer 8.5.4 The correct answer is “D.” Spiral chest C is criteria or the diagnosis o endocarditis. T ey have been vali-
not indicated in the diagnosis o SBE. History is important: dated by pathologic examination and are more sensitive than
onset o in ection can sometimes be related to a recent dental other endocarditis criteria systems. See able 8-2.
CHAPTER 8 • INFECTIOu S DISEASES 247

TABLE 8-2 DUKE CRITERIA FOR BACTERIAL HELPFUL (AND IMPORTANT) TIP:
ENDOCARDITIS Patients who have a sensitive organism (MSSA) ac-
De nite endocarditis is established by the presence o 2 major criteria tually have better outcomes (fewer deaths, etc.)
and at least 1 minor criterion. Probable endocarditis is established by with nafcillin than with vancomycin. So, save vanco-
the presence o 1 major and 1 minor criterion, or 3 minor criteria.
mycin or MRSA or other resistant organisms. Other op-
Major D ke criteria • New valv lar reg rgitation tions or hospital-acq ired MRSA incl de linezolid and
• Echocardiographic evidence o vegetations tigecycline.
• Two positive blood c lt res o an organism
known to ca se endocarditis
• Single blood c lt re or antibody evidence
o Coxiella burnetii (Q ever) While hospitalized, the patient develops symptoms o heart
Minor D ke criteria • Fever ailure and worsening mitral regurgitation by echocardio-
(not an exha stive • Vasc lar phenomena (e.g., Janeway lesions, gram. T e heart ailure is managed medically, but the regur-
list b t these are splinter or conj nctival hemorrhages, and gitation is now categorized as “severe.” She has had 3 days o
the most common septic emboli) antibiotics and is currently hemodynamically stable.
mani estations) • History o predisposing illness (e.g., IV dr g
ab se, heart lesion, and arti cial valve)
• Imm nologic phenomena (e.g., glomer lo- Question 8.5.8 Which o the ollowing is the most appro-
nephritis and Osler nodes) priate course o action?
A) Complete 6 weeks o antibiotics and manage her heart ail-
ure medically or the oreseeable uture.
HELPFUL TIP: B) Complete 6 weeks o antibiotics and manage her heart ail-
Three words abo t blood c lt res—more is better. The ure medically, i possible; plan or valve replacement a er
sensitivity o blood c lt res or endocarditis and bacte- 6 weeks o antibiotics.
remia is directly related to the amo nt o blood taken C) Re er her or emergent valve replacement surgery, then
or c lt re and the n mber o c lt res drawn. Three complete the course o antibiotics.
sets o blood c lt res are recommended or s spected D) Re er her or immediate coronary catheterization, and con-
endocarditis, and at least 20 mL sho ld be drawn or tinue antibiotics.
each c lt re. Timing o blood c lt res is less important, E) Re er her or heart transplant.
b t sick patients sho ld have the c lt res drawn in
rapid s ccession (e.g., over an ho r or 2). Answer 8.5.8 The correct answer is “C.” Progressive heart ail-
ure due to moderate-to-severe valvular dys unction is an indi-
cation or surgery. T ere is evidence that early surgery reduces
You draw a CBC, which shows leukocytosis with a “le shi ” mortality and embolic complications especially in patients
(e.g., a high percentage o bands and other immature neutro- who have le -sided endocarditis associated with large vegeta-
phils). Chest x-ray and urinalysis are unrevealing. You draw tions and severe valvular dys unction. T is is true even in the
blood cultures and admit her to the hospital and start anti- absence o congestive heart ailure and in the presence o less
biotics. T e next morning two blood cultures are reported virulent pathogens. T us, “B” is incorrect. Also, “A” is incorrect,
to grow Gram-positive cocci in clusters. You start IV vanco- as the patient should have surgery. “D” and “E” are incorrect,
mycin and order a transesophageal echocardiogram. Indeed, as there is no indication or coronary catheterization or heart
the echocardiogram shows a small vegetation on her mitral transplant.
valve. Blood cultures return showing methicillin-sensitive
S. aureus.
HELPFUL TIP:
Other indications or s rgery in cases o endocarditis in-
Question 8.5.7 What is the most appropriate treatment o
cl de m ltiple embolic events, in ections that are di i-
this patient now?
c lt or impossible to treat adeq ately with medications
A) Na cillin 2 g IV q 4 hours or 4 to 6 weeks
(e.g., ngal in ections), cardiac cond ction abnormali-
B) Penicillin G 2 million units IV q 2 hours or 4 to 6 weeks
ties d e to in ection, persistent bacteremia, partially
C) Vancomycin 1 g IV q 12 hours or 4 to 6 weeks
dehisced prosthetic valve, and perivalv lar in ection
D) Ce riaxone 1 g IV q 24 hours or 2 weeks
(e.g., cardiac abscess and ist la).
E) Levo oxacin 500 mg IV q 24 hours or 4 to 6 weeks

Answer 8.5.7 The correct answer is “A.” Na cillin is the drug Question 8.5.9 Which o the ollowing organisms is most
o choice or the treatment o methicillin-sensitive S. aureus o ten responsible or causing in ective endocarditis?
endocarditis. Vancomycin should be reserved or patients with A) E. coli
penicillin allergy or patients with methicillin-resistant S. aureus B) Streptococcus viridans
(MRSA). Neither ce riaxone nor levo oxacin would be consid- C) Proteus mirabilis
ered appropriate therapy or staphylococcal endocarditis. D) None o the above
248 FAMILY MEDICINE EXAMINATION & BOARD REVIEW

Answer 8.5.9 The correct answer is “B.” S. viridans is the most intense pruritus that is worse at night. T e typical lesion is small,
likely organism to cause endocarditis. Gram-negative organisms, erythematous, and papular and may resemble eczema in qual-
such as E. coli and P. mirabilis, are in requent causes o in ective ity and distribution. About 7% o individuals develop a nodular
endocarditis. Other organisms that cause endocarditis include variant (like the mother in this case). ransmission is typically
the HACEK organisms (Haemophilus species, Actinobacillus by direct contact and in estations may appear as epidemics in
actinomyces comitantes, Cardiobacterium hominis, Eikenella institutions like nursing homes. T e organism may be spread by
species, and Kingella kingae). In summary, organisms typically omites as well, although to a lesser extent. Young children and
ound causing endocarditis are S. aureus, S. viridans, enterococci in ants o en have involvement o palms, soles, ace, and scalp.
(aerobic, Gram-positive organisms in chains that are GI or vagi- A clinical diagnosis may be made in the setting o pruritic rash,
nal ora), Streptococcus bovis, and HACEK organisms. typical distribution and multiple amily members a ected.
Objectives: Did you learn to . . .
Question 8.6.2 What is the next best step in this case
• Determine who is an appropriate candidate or in ective
(remember that one child weighs 10 kg)?
endocarditis prophylaxis?
A) Removal o the individual organisms.
• Recognize signs and symptoms o in ective endocarditis?
B) etracycline 10 mg/kg divided ID or all a ected amily
• Diagnose in ective endocarditis?
members.
• Prescribe appropriate treatment or in ective endocarditis? C) Single-dose oral ivermectin 200 µg/kg, repeated in 2 weeks
or all a ected amily members.
QUICK QUIZ: DRu G INTERACTIONS D) Symptomatic treatment with topical steroids and oral anti-
histamines.
E) Single-dose oral ivermectin 200 µg/kg repeated in 2 weeks or
Which o the ollowing medications is/are contraindicated in
the mother; one application o 5% permethrin cream or all
patients taking linezolid?
other amily members or 8 to 14 hours, ollowed by bathing.
A) MAOIs
B) SSRIs
Answer 8.6.2 The correct answer is “E.” Permethrin cream is
C) Gentamicin
the topical medication o choice. Ivermectin, an antihelmintic
D) Vancomycin
medication, is indicated or adults with nodular disease (like
E) A and B
the mother in this case), in epidemic settings and or treatment
The correct answer is “E.” Linezolid can cause serotonin syn- o scabies crustosa. “B” is incorrect because tetracycline is not
drome when combined with SSRIs, lithium, MAOIs, and other help ul in this situation and should be avoided in children. “C”
serotonergic drugs. is incorrect because oral ivermectin should be avoided in in ants
weighing <15 kg due to concerns about increased penetration o
CASE 8.6 the blood–brain barrier. Finally, “D” is incorrect because sca-
bies should be treated with speci c therapy rather than simply
A 10-year-old boy presents with his mother, complaining o symptomatic therapy. Oh, and “A” is just insane—how could you
intense itching, worse at night, since the rst week o school. do that? Is there even a CP code or that?
He has numerous excoriations in the interdigital web spaces,
wrists, and anterior axillary olds. His in ant sister (10 kg) HELPFUL TIP:
has recently developed intensely pruritic linear lesions on her All amily members sho ld be treated, regardless o
palms, soles, ace and scalp. T eir mother works in a nursing the presence or lack o symptoms. Microscopic exami-
home and has developed pruritus and reddish-brown nodular nation o a skin scraping may identi y the mite b t has
lesions in her axillae and perineum that have persisted several poor sensitivity. Other viable treatment alternatives
months a er she treated hersel with a lotion that was pro- incl de crotamiton 10% sol tion precipitated s l r in
vided at her place o work. As you examination the patient, petrole m and lindane (b t avoid lindane in children
your skin begins to itch—you need to solve this puzzle quickly. <2 years old).

Question 8.6.1 The most likely ectoparasite af ecting this


amily is: You success ully treated the whole amily. T ey are now com-
A) Head lice (pediculosis) ortable, happy, and totally con dent in your abilities. T e
B) Chiggers (mites) mother returns with her girl, who is now 3 years old, with
C) icks a new complaint. Apparently, the girl’s “bottom” is hurt-
D) Fleas ing, a symptom that her mother has interpreted to mean her
E) Scabies perineal pain. T e pain is worse at night, even waking her at
night with vaginal pain. T e mother thinks that she may have
Answer 8.6.1 The correct answer is “E.” Scabies’ mites (Sarcop- a bladder in ection, but there are no urinary symptoms. In
tes scabiei) burrow into the epidermis, lay eggs, and hatch larvae the of ce, the patient complains only o “itchy butt” and her
in cycles o 3 to 4 days. T e most notable clinical symptom is examination is normal.
CHAPTER 8 • INFECTIOu S DISEASES 249

Question 8.6.3 What is the next best step in diagnosis o and has been ebrile daily since. He saw another doctor recently.
this problem? He was evaluated but received no antibiotics or other treatment.
A) Reassurance that this is “just a stage” His evaluation, including history, physical examination, CBC
B) Vaginal speculum examination with cultures with di erential, urinalysis, serum chemistries, and chest x-ray,
C) “Scotch tape” test has been unrevealing. oday his temperature is 38.5°C.
D) Stool collection or ova and parasites
E) Psychiatry consultation or the mother who obviously has Which o the ollowing is the most appropriate next step in the
Munchausen by proxy syndrome evaluation and management o this patient?
A) ESR and/or CRP
Answer 8.6.3 The correct answer is “C.” T e presentation is B) Chest C scan
consistent with pinworm (Enterobius vermicularis) in ection. C) Bone scan
Clinical mani estations o pinworm in ection are related to the D) PSA
li e cycle o the parasite, in which the adult worm resides in the E) Vancomycin 1 g IV daily and ce riaxone 1 g IV daily or
colon, exits the anus at night, lays eggs in the perianal skin, and 2 weeks
may also in est the emale GU tract. ypical symptoms include
pruritus ani, vulvitis, vaginal pain, poor sleep, and—rarely— The correct answer is “A.” T is patient has a ever o unknown
abdominal pain (pinworms can migrate and cause appendici- origin (FUO). FUO in adults is de ned as ever greater than
tis, and other intra-abdominal illness). T e diagnostic test o 38.3°C o at least 3 weeks duration with no obvious cause despite
choice is the “Scotch tape” test. Clear cellophane tape is wrapped extensive evaluation. In ections are the most common source o
around a tongue depressor, sticky side up, and used to sample FUO in children and young adults. T e most common in ec-
the perianal area rst thing in the morning be ore bathing. Mul- tions include tuberculosis and abscesses. In older adults (sorry
tiple specimens should be obtained and stored in a re rigerator to say that this patient’s age o 54 counts!), collagen-vascular
(it’s a good thing OSHA has no jurisdiction in the home). T e diseases, such as giant cell arteritis and rheumatoid arthritis, are
tape is then examined microscopically or the characteristic ova. more likely sources o FUO. T ere ore, tests or in ammatory
markers, like ESR and CRP, are indicated. Other tests to consid-
Your “Scotch tape” test is a success, proving your clinical sus- er at this point include ANA, rheumatoid actor, creatine kinase
picions. (CK), and blood and urine cultures. Although malignancy is a
consideration, chest C (“B”) and PSA (“D”) should not be or-
Question 8.6.4 The best intervention is to: dered without other symptoms associated with underlying lung
A) reat the patient with mebendazole 100 mg orally once, and prostate disease, respectively. A bone scan (“C”) would be
repeat in 2 weeks, and encourage good hand washing or the indicated to look or osteomyelitis, which is an unlikely occult
whole amily source o ever in this patient with no other reason to believe he
B) reat the patient and the entire amily with mebendazole has a bone in ection. T ere is no indication or antibiotics (“E”)
100 mg orally daily or 14 days at this point—wait until you have something to treat.
C) reat the patient and the entire amily with mebendazole
100 mg orally once, and repeat in 2 weeks HELPFUL TIP:
D) reat the patient with metronidazole 500 mg orally once, The most common malignancies that present with an
and repeat in 2 weeks Fu O are lymphoma, le kemia, renal cell carcinoma and
hepatoma. A CT scan o the abdomen sho ld pick all o
Answer 8.6.4 The correct answer is “C.” As with scabies, the these p, except le kemia, o co rse. However, the diag-
entire amily should be treated. Mebendazole is the agent o nostic yield o a CT scan is lower than the yield o FDG-
choice, although other antiparasitic agents (e.g., albendazole, PET/CT, which may detect active in lammation prior to
pyrantel pamoate) may also be used. Pyrantel pamoate is sec- any speci ic anatomic change and may be considered
ond-line and only has a 90% response rate. “D,” metronidazole, in place o stand-alone CT scanning in the eval ation o
is not used or helminthic in ections but is e ective against Fu O. See Table 8-3 or a partial list o ca ses o Fu O.
protozoal in ections, including amebiasis and trichomoniasis.
Make sure to wash all o the bed linens and clothing: eggs can
stay viable or 20 days (although 3 days is more typical). CASE 8.7
Objectives: Did you learn to . . . A 45-year-old physician who has recently returned rom an
• Diagnose and treat scabies in estations? early summer shing vacation (in reality he mostly studied
• Diagnose and treat pinworm in ections? or his upcoming board examination) in rural North Caro-
lina presents or a ebrile illness. He reports a 5-day history o
ever, malaise, headache, and vomiting. oday, he has devel-
QUICK QUIZ: SWEATING IN SAVANNAH oped a nonpruritic rash that began on his extremities and has
spread to his body. On examination he has a ever o 38.3°C
A 54-year-old construction worker with no signi cant travel with a pulse o 120 bpm and otherwise normal vitals. T e
history presents with a ever. He developed the ever 4 weeks ago rash is maculopapular and generalized, involving his palms
250 FAMILY MEDICINE EXAMINATION & BOARD REVIEW

TABLE 8-3 PARTIAL LIST OF ETIOLOGIES OF FEVER that begins at the wrists and ankles and spreads centrally. Even-
OF UNKNOWN ORIGIN tually, the rash becomes petechial. Despite its name, RMSF is
endemic in the southeastern United States, the Atlantic states,
Infections
and the northern Rocky Mountains. Laboratory mani estations
• T berc losis
• Lyme disease o RMSF are generally nonspeci c: mild thrombocytopenia
• HIV (rarely becoming severe), hyponatremia, azotemia, elevated
• Endocarditis transaminases, and prolonged P and P .
• Dental abscess “B” is incorrect because the secondary stage o syphilis is char-
• Abdominal/pelvic abscess
acterized by a generalized maculopapular rash (including the
• CMV
• Epstein–Barr vir s palms and soles) and is not associated with systemic symptoms.
“E” is also incorrect. Human monocytic ehrlichiosis is caused
Malignancies
• Metastatic cancer by Ehrlichia chaf eensis and other related bacteria and presents
• Lymphoma with a ever and non-speci c “ u like” symptoms (headache,
• Le kemias ever, myalgias, chills, cough). Rarely a rash, maculopapular or
• Renal cell carcinoma petechial, is seen. T us, it may be easily con used with RMSF (and
Autoimmune Conditions in act some physicians re er to ehrlichiosis as “Rocky Mountain
• Polymyalgia rhe matic/giant cell arteritis Spotless Fever”). Patients are o en leukopenic and thrombocy-
• Rhe matoid arthritis
topenic. T e disease is tick-borne and endemic to midwestern,
• In ammatory bowel disease
• L p s south central, and southeastern states. So, i you think you have
• Vasc litides a case o RMSF but there is no rash, consider human monocytic
Drug-induced Fever ehrlichiosis. See able 8-4 or more details on tick-borne ill-
• Factitio s ever nesses. Yeah . . . we know . . . everything presents with ever, head-
• Veno s thrombosis ache and myalgias. T ey all sound the same to us too . . .
• Sarcoidosis
Question 8.7.3 What is the appropriate next step or this
patient?
and soles. Oral mucosa is dry but intact, and the examination
A) Obtain serologic studies and await results while treating
is otherwise nonspeci c.
symptomatically.
Question 8.7.1 What is the most appropriate next step? B) Obtain skin biopsy and await results while treating symp-
A) Reassurance and symptomatic treatment tomatically.
B) CBC, electrolytes, BUN, creatinine C) Obtain serologic studies and start doxycycline 100 mg orally
C) Dermatology re erral BID.
D) Punch biopsy o leading edge o rash D) Obtain skin biopsy and start levo oxacin 500 mg orally
E) Admission to the ICU daily.
E) Hospitalize and start ce riaxone 1 g IV daily until ever has
Answer 8.7.1 The correct answer is “B.” T is gentleman is sick resolved.
(pulse o 120 bpm, dry oral mucosa, headache, vomiting). We
would be amiss to simply reassure or re er this patient. A CBC Answer 8.7.3 The answer is “C.” Early treatment or RMSF
and electrolytes may give us an indication o the degree o dehy- is essential. Individuals treated a er 5 days o symptoms have
dration and help us narrow the di erential (bacterial, viral, etc.). worse outcomes than those treated earlier. Awaiting serologic
Blood cultures would also be indicated at this time. studies is inappropriate and treatment should not be delayed.
T e drug o choice in the treatment o RMSF is doxycycline
T e test results return quickly. CBC shows mild thrombocy- 100 mg PO BID or 14 days. T is is true or children as well!
topenia but is otherwise normal. BUN and creatinine are at Pregnant women should be treated with chloramphenicol.
the upper limits o normal, and the electrolytes are normal. Agents such as penicillin, uoroquinolones and cephalosporins
are inappropriate in this situation.
Question 8.7.2 The most likely diagnosis is:
Objectives: Did you learn to . . .
A) Chicken pox
• Identi y and diagnose RMSF?
B) Syphilis
• Initiate treatment o RMSF?
C) Parvovirus B19
• Recognize other tick-borne illnesses?
D) Rocky Mountain spotted ever (RMSF)
E) Human monocytic ehrlichiosis

Answer 8.7.2 The correct answer is “D.” RMSF is a tick-borne QUICK QUIZ: FLu CTu ANCE IN FLINT
(dog or wood tick) disease caused by Rickettsia rickettsii. It pres-
ents with a prodrome o ever and headache several days be ore A 27-year-old male carpenter presents with pain, redness, and
the onset o the characteristic rash—a maculopapular eruption swelling o the distal aspect o the right index nger. He reports
CHAPTER 8 • INFECTIOu S DISEASES 251

TABLE 8-4 TICK-BORNE ILLNESSES


Geographic
Disease Etiologic Agent Distribution Clinical Findings

Babesiosis Babesia species New England, pper Fever, sweats, myalgias, arthralgias, red rine, hemolytic anemia (most
Midwest, Cali ornia severe cases occ r in splenectomized patients). Similar to malaria in that
there are periodic ever spikes. Diagnosis is by Giemsa stain

H man monocytic Ehrlichia chaf eensis So th, Midwest Fever, headache, myalgias (similar to RMSF b t rarely a rash). Diagnosis is
ehrlichiosis and ewingii by b f y coat examination, clinical presentation, PCR

H man gran locytic Anaplasma Atlantic states, Fever, headache, myalgias (similar to RMSF b t rarely a rash). Diagnosis is
anaplasmosis phagocytophilum pper Midwest by b f y coat examination, clinical presentation, PCR

Lyme disease Borrelia burgdor eri Northeast, pper Erythema migrans, myalgias, arthralgias, arthritis, ever, headache.
Midwest Diagnosis is by serology

Rocky Mo ntain Rickettsia rickettsii So theast, Atlantic Fever, headache, GI symptoms, mac lopap lar rash → petechiae,
Spotted Fever (RMSF) coast states myalgias. Diagnosis: clinical, serology (b t not + early)

T laremia Francisella So th, Midwest Fever, headache, co gh, myalgias, GI symptoms, tender lymphadenopathy
tularensis with rare skin lceration. Diagnoses by aggl tination or ELISA. Expos re
history (rabbits, ticks, ies)

getting a splinter in the site 2 days ago while working. T e pain Nigeria or a month and will be living in a suburb o Lagos,
is now so severe that he cannot work. On examination, the Nigeria’s largest city (Boko Haram not withstanding). T ey
patient is a ebrile, the right index ngertip is extremely tender, expect to take sightseeing trips into less developed areas.
and there is an area o uctuance at the palmar aspect o the John has no previous medical problems. Jane is currently tak-
nger. All o the redness and warmth are distal to the proximal ing venla axine (E exor) or depression and is known to have
interphalangeal joint. a sul a allergy. Jack has had occasional bouts o reactive air-
way disease and also has a sul a allergy. Jill is healthy. Every-
What is the most appropriate diagnosis? one in the amily is up-to-date on all routine North American
A) Paronychia vaccines.
B) Felon
C) Whitlow lesion Question 8.8.1 Regarding pathogens they might encounter
D) enosynovitis in A rica, which o the ollowing is caused by a virus?
E) Achy-breaky- nger A) Plasmodium ovale
B) Plasmodium alciparum
The correct answer is “B.” A elon is an abscess o the distal C) Dengue ever
ngertip, most commonly occurring in the index nger and D) Entamoeba histolytica
thumb. It can be distinguished rom paronychia (“A”) because E) All o the above
a elon is located in the at pad o the nger and not the tissue
around the nail. O en, an area o uctuance is palpable. A elon Answer 8.8.1 The correct answer is “C.” Dengue ever (aka
can spread quickly and can involve the periosteum and bone. “break bone” ever) is a viral in ection caused by Flavivirus
Appropriate management includes x-ray o the nger (to rule (more below). “A” and “B,” P. alciparum and P. ovale, are two
out osteomyelitis), antibiotics, and incision and drainage. “C” is species o malaria parasites. P. alciparum tends to produce
incorrect because a whitlow lesion results rom inoculation o more severe in ections that can be rapidly atal in malaria naıve
broken skin o the hand with type 1 or 2 herpes simplex virus. patients. “D,” E. histolytica, is the intestinal protozoan parasite
T e whitlow lesion is o en typical o herpes (vesicles on ery- responsible or amebiasis.
thematous papules) but can also be con used with paronychia
or elon i at the distal nger. “D” is incorrect as well. enosy- Question 8.8.2 Mr. Smith asks or advice on the use o insect
novitis should not involve the distal aspect o the digit without repellent. What do you recommend?
a ecting the rest o the tendon sheath. “E” is just silly but could A) Any repellent will do, they’re all the same. Just use the
be another big country hit! cheapest
B) Pleasant-smelling repellents, such as Avon SkinSo-So M,
are just as e ective as any DEE -containing ormulation
CASE 8.8 C) Use repellent with DEE concentrations o at least 50% or
T e Smith amily presents to your of ce in December seeking the children, since their protection is so vital
travel vaccines or a trip to Nigeria. John (34) and Jane (35) D) Apply permethrin insecticide to the skin to enhance any
Smith have two children, Jack (7) and Jill (5). T ey will be in other repellent’s ef cacy
252 FAMILY MEDICINE EXAMINATION & BOARD REVIEW

E) Use DEE or picaridin-containing repellents, as they are the


HELPFUL TIP:
most ef cacious insect repellents available, but avoid DEE
The CDC maintains a ser- riendly and p-to-date travel
concentrations greater than 30% in children
website at http:// wwwnc.cdc.gov/ travel. Always check
here irst to ass re yo are giving the right vaccines and
Answer 8.8.2 The correct answer is “E.” Repeated experiments
advice.
clearly show DEE - and picaridin-containing repellents to be the
most e ective or deterring bites and these are the two recom-
mended by the CDC. Picaridin-containing insect repellents do
not cause neurotoxicity. T e American Academy o Pediatrics
recommends not exceeding 30% DEE concentration in repel- Finally, you discuss the medication options or malaria pro-
lants or children, due to a slight risk o toxicity seen in requent phylaxis.
applications over a long period o time. Applying DEE to cloth-
ing rather than skin reduces the risk o toxicity. Adults can theo- Question 8.8.4 Which o the ollowing is TRUE?
retically use any concentration, but 30% is usually suf cient or A) Me oquine (Lariam) is relatively contraindicated or Jane
most situations. O en, more cosmetically pleasing products, such due to her history o psychiatric illness
as Avon brand SkinSo-So , are less e ective and last a mere rac- B) Doxycycline would be a sa e and e ective option or the
tion o the duration o DEE compounds. Permethrin insecti- whole amily
cide, when applied to clothes, tents, and bed nets, is synergistic with C) Although malaria is resistant to chloroquine in many parts
insect repellent, but permethrin itsel is not ormulated or use as o the world, it can still be used or prophylaxis in West
an insect repellent on skin. O note, many countries have a com- A rica
bination DEE -Permethrin product available or use on cloths. D) Atovaquone/proguanil (Malarone) is contraindicated or
Jane and Jack due to their sul a allergy
Mrs. Smith reports that her riends get sick with diarrhea E) A month is too long a time to use malaria prophylaxis sa ely;
every time they travel abroad. She would like to avoid this. recommend against it

Question 8.8.3 Which o the ollowing is (are) true about Answer 8.8.4 The correct answer is “A.” Me oquine is an e ec-
traveler’s diarrhea? tive, once-a-week prophylaxis or malaria. However, it carries a
A) Enterotoxigenic E. coli (E EC) is the most common cause o signi cant risk o CNS side e ects including vivid or disturb-
this condition ing dreams. T ere have been case reports o the medication
B) Even care ully avoiding the consumption o tap water or inducing psychosis, so the drug is relatively contraindicated
unwashed vegetables may not be suf cient to prevent the disease or patients with a history o psychiatric illness (such as Jane
C) Fluoroquinolones can help to rapidly cure this condition but Smith). “B,” doxycycline, may be a good option or the parents,
are contraindicated in pregnancy and young children but is contraindicated in children because o their age and risk o
D) T e use o loperamide is e ective in reducing the duration o tooth discoloration. “C” is incorrect. Malaria throughout A rica,
symptoms but is contraindicated in children <2 years o age India, Southeast Asia, and South America is now assumed to be
E) All o the above are true resistant to chloroquine. Finally, “D” atovaquone/proguanil is
relatively contraindicated in patients with G6PD de ciency due
Answer 8.8.3 The correct answer is “E.” In most parts o the to a risk o hemolysis, but it does not contain sul a. O particular
world, including A rica, E EC is the most common cause o note is answer “E.” Medications used to provide malarial che-
travelers’ diarrhea. Although it is advisable to avoid tap water, moprophylaxis have been shown to be well-tolerated or at least
unwashed oods, and raw oods, these measures are usually a year or more (http://www.cdc.gov/malaria/about/ aqs.html).
insuf cient to completely eliminate the risk o contracting the T ere ore, one month is not too long to use malaria prophy-
disease. A traveler may drink only bottled liquids, but might laxis sa ely and it should be recommended to the amily or their
orget that the ice in the glass is made rom tap water. Although travel.
the disease is sel -limited, a single dose o cipro oxacin 750 mg
(see note below) may shorten the course o symptoms signi - Weeks pass, and you hear nothing more until you are called
cantly. T e dose or moderate or severe disease is cipro oxacin to the acute care clinic, where Jack Smith has been brought
500 mg BID. A patient’s symptoms can be urther shortened in by his parents or ever and lethargy. Jack had been treated
by adding loperamide, which is sa e in the absence o bloody in Lagos, Nigeria, or malaria with an unknown medication,
stools. Loperamide is potentially toxic to in ants and toddlers. and he subsequently recovered. Jack was apparently well until
10 days a er returning home, when he developed rapid onset
HELPFUL (AND VERY IMPORTANT) TIP: o a ever and shaking chills. He also complained to his par-
The CDC no longer recommends fluoroquinolones ents o generalized abdominal pain and watery, non-bloody
for travelers’ diarrhea acquired in Southeast Asia be- diarrhea. T e parents treated him at home or a day with
cause of resistance. The c rrent recommendation is to ibupro en and acetaminophen, but he seemed to worsen. He
se azithromycin or travelers’ diarrhea in So theast Asia. became lethargic, stopped drinking and eating, and the ever
continued.
CHAPTER 8 • INFECTIOu S DISEASES 253

Jack appears drowsy and listless but is arousable. He does East), treatment with chloroquine is acceptable. Hydroxychlo-
not respond to questions about current symptoms, but coop- roquine is an option i chloroquine is not available.
erates with an examination. Findings are temperature 39.1°C,
pulse 136 bpm, blood pressure 100/50 mm Hg, and respira- HELPFUL TIP:
tory rate 24. His neck is supple with mild lymphadenopathy. Artes nate, another antimalarial, clears parasitemia
He is tachycardic with a mild ow murmur. His abdomen is aster than q inidine b t is not approved by the FDA.
nontender with a palpable spleen. No rash or petechiae are It is available rom the CDC on protocol. The CDC main-
noted. T e rest o the examination is unremarkable. A recur- tains a 24-ho r hotline or malaria advice that will
rence o malaria is suspected. connect yo directly to a real, live clinician or assistance
in managing patients with malaria. Finally, a combina-
Question 8.8.5 What is the best method or con rming this tion prod ct, artemether-l me antrine (Coartem) is
diagnosis? available in the u nited States. It works well b t can pro-
A) Blood culture long the QT.
B) Malaria serology
C) Malaria antigen test
D) T in and thick blood smears
Despite a rightening hospital course that included general-
E) Stool ova and parasite
ized seizures, hypoglycemia, hematuria, renal insuf ciency,
and an exchange trans usion, Jack eventually recovers com-
Answer 8.8.5 The correct answer is “D.” Malaria is usually
pletely. T e Smith amily thanks you or your help and hopes
diagnosed by blood smear. T e thick blood smear is the more
sensitive screening test, and the thin blood smear is used to you’ll accompany them on their next trip to A rica.
identi y the species o parasite. T ere are rapid antigen tests Objectives: Did you learn to . . .
available they have higher alse positives and alse negatives • Identi y important elements o a patient’s travel plans and
than a smear. In addition, they do not allow identi cation o niq e risks when providing co nseling or overseas travel?
the species. T us, i one suspects malaria and the rapid test is • Identi y preventative meas res or malaria, incl ding chemo-
negative, thick and thin smears are still indicated. T e blood prophylaxis and insect bite avoidance?
smear remains the gold standard. Malaria serology and cul- • Diagnose travelers’diarrhea and describe its prevention and
tures are used only in experiments and are not help ul or diag- treatment?
nosis o an individual patient. O note, PCR-based testing has • Recognize the signs and symptoms o malaria, describe
excellent sensitivity and speci city but is expensive and not methods o diagnosis, and initiate therapy?
widely available. T e malaria parasite cannot be identi ed in
stool.
QUICK QUIZ: CRITTERS IN CLEVELAND
You begin IV uids and arrange hospital admission. T e rel-
evant laboratory tests are drawn and sent, including CBC, A amily comes to see you because the two children, ages 7 and
blood cultures, tests or malaria, chemistry pro le, blood 4, have developed itchy scalps. T e parents seem una ected. So
type and screen, and urinalysis. LP is per ormed and the CSF ar, they have not tried any treatments. On examination o both
is normal. In the meantime, the laboratory calls with the children, you nd erythematous papules on the occiput and
report that P. alciparum has been identi ed. small white eggs rmly attached to the hair sha about 1 cm
rom the scalp. Upon leaving the room, you head straight or the
Question 8.8.6 What antimicrobial should be chosen as shower and scrub your scalp or 5 minutes.
initial therapy?
A) Oral hydroxychloroquine (Plaquenil), since many hospitals T e most appropriate treatment or the suspected diagnosis is:
do not stock chloroquine A) Application o 1% permethrin cream to all amily members
B) Oral me oquine (Lariam) or 10 minutes ollowed by rinsing, combing out all nits with
C) Oral quinine a special louse comb, and decontaminating a ected gar-
D) IV quinidine, since IV quinine is not generally available in ments and bed linens. Repeat in 7 days
the United States B) Elimination o animal or omite sources o in estation and
E) IV atovaquone/proguanil (Malarone) use o insect repellents
C) Removal o any adherent organisms and oral doxycycline
Answer 8.8.6 The correct answer is “D.” Clearly, since this or 14 days
patient is ill and not tolerating oral intake, an IV route or D) Application o 5% permethrin cream to all amily members
malaria treatment is indicated. IV atovaquone/proguanil does or 8 to 14 hours, ollowed by showering
not exist, so the only available option is quinine. But IV quinine E) Shave everyone’s head
is not available in the United States, so its isomer, quinidine (the
antiarrhythmic) is used instead. In patients rom chloroquine- The correct answer is “A.” T ese are head lice. Pediculosis in-
sensitive zones (currently Central America and the Middle estations o the hair and scalp are usually asymptomatic but
254 FAMILY MEDICINE EXAMINATION & BOARD REVIEW

can present with itching. T e diagnosis is made by demonstra- B) T e combination o acetaminophen and ibupro en in this
tion o the louse or nits, which uoresce a pale blue under a patient with the u has led to hypotension.
Wood’s light. reatment with topical agents such as permethrin C) Given that she is immunocompetent and has her spleen
cream or two applications and wet combing to remove nits is intact, this cannot be sepsis since it started so quickly.
recommended by the CDC. Ivermectin may be e ective in cas- D) oxic shock, which was a big problem in the 1980 s and early
es o resistant organisms. It is reasonable to recommend wash- 1990 s, no longer occurs since the advent o less absorbent
ing clothing and bedclothes o an in ested person, but head lice tampons.
do not survive o the scalp longer than 48 hours. “B” is ap- E) None o the above.
propriate or chiggers (mites) or eas. “C” is appropriate or
ticks (i also at risk or a tick-borne illness). “D” is a treatment Answer 8.9.2 The correct answer is “E.” “A” is incorrect
o scabies (note the di erence in strength o permethrin). O because up to 50% o cases o toxic shock occur as the result o
note, there is no bene t to keeping children with head lice out o staphylococcal in ections unrelated to tampons. T ese may be
school. Avoid sharing hats, etc. Just try to get this one past the ingrown toenails, in ected abrasions, etc. “B” is incorrect. Acet-
school nurse. aminophen and ibupro en are requently combined without di -
culty. “C” is also incorrect. Splenectomized patients are more
CASE 8.9 prone to sepsis rom encapsulated organisms, but the act that
the patient has a spleen does not grant invincibility. Sepsis obvi-
A 19-year-old emale college student presents to student ously occurs in the normal host as well. Finally, “D” is incorrect.
health services with “the u.” She has noted a ever o 38.9°C While absorbent tampons are a major culprit in toxic shock syn-
and myalgias. She is treated with symptomatic care and dis- drome, as noted above, there are other causes. T us, toxic shock
charged back to her dormitory. T ree hours later her room- syndrome is not going away anytime soon.
mate nds her lethargic and dif cult to arouse, so she calls
911. On examination, her blood pressure is 70/30 mmHg Question 8.9.3 The organism(s) responsible or toxic shock
with a pulse o 145 bpm. Her neck is supple, but she is lethar- syndrome is (are):
gic and complaining o severe muscle aches. She denies head- A) Staphylococcus
ache. T ere is a ne macular rash over her abdomen. B) H. in uenzae
C) Streptococcus
Question 8.9.1 You need to obtain a little more history. The D) A and B
most important historical actor in this case is: E) A and C
A) History o splenectomy
B) History o acetaminophen overdose Answer 8.9.3 The correct answer is “E.” T ere are two types
C) History o tobacco use o toxic shock syndrome, one caused by Staphylococcus and
D) Sexual history the other by Streptococcus. T ere are certain subtypes o these
bacteria that make the toxin responsible or toxic shock syn-
Answer 8.9.1 The correct answer is “A.” “A” is important since drome and only certain hosts are thought to be susceptible.
patients with a splenectomy can get sick rather rapidly rom O note, most patients with streptococcal toxic shock are bac-
pneumococci and other encapsulated bacteria. T e rest are much teremic, whereas those with staphylococcal toxic shock are
less important. Shock is not a prominent eature o acetamino- not.
phen overdose (“B”)—i it occurs at all. In addition, acetamino-
phen overdose is not associated with a rash. “C” and “D” are Question 8.9.4 Which o the ollowing would you NOT
incorrect. T ese elements o social history, while important to expect to nd on laboratory testing o this patient with
her overall care, will not aid you in the diagnosis and immediate suspected toxic shock syndrome?
management. Another very pertinent historical eature would A) Creatinine o 2.0 mg/dL (normal 1 mg/dL)
be tampon use, which is important because this patient may B) Elevated AL /AS
have toxic shock syndrome. C) Platelets o 450,000/mm 3
D) Elevated CPK
T e patient is able to give you the additional history that
she does not use tampons and has not taken any medication Answer 8.9.4 The correct answer is “C.” By the de nition o
except or occasional acetaminophen and ibupro en in rec- toxic shock syndrome, the platelet count should be <100,00 mm 3
ommended doses. She has her spleen . . . in a jar in her dorm in Staphylococcal-related toxic shock. All o the other ndings
room—no, wait, we mean in her le upper quadrant. It’s her are representative o the multisystem dys unction that catego-
ex-boy riend’s spleen in the jar. rizes toxic shock syndrome.

Question 8.9.2 On the basis o this in ormation you decide Your patient is a little more alert, most likely due to your
that: charming wit. A second set o vitals shows: blood pressure
A) It is unlikely that this is toxic shock syndrome given that she 72/44 mm Hg, pulse 140 bpm, respirations 24, temperature
does not use tampons. 39°C. Labs are pending.
CHAPTER 8 • INFECTIOu S DISEASES 255

Question 8.9.5 What is the single best next step in the care C) Chlorhexidine baths and mupirocin to nares daily
o this patient? D) Doxycycline orally or 2 weeks
A) Start IV na cillin E) Oritavancin (trade name Orbactiv) 1,200 mg IV, single dose
B) Place two large-bore IV lines and start aggressive uid
replacement
The correct answer, and the only thing you must do or this
C) Start IV norepinephrine
patient, is “A.” You have already treated the local in ection, the
D) Give a single dose o IV dexamethasone
abscess on his skin, with incision and drainage. Unless his has
E) rans use two units o packed red cells
an extending cellulitis or is systemically ill (e.g., ebrile), he does
not require systemic antibiotics, so “D” and “E” are incorrect.
Answer 8.9.5 The correct answer is “B.” reatment is mainly
Packing a small abscess a er incision and drainage (“B”) is a
supportive. She’s in shock. wo large-bore IV lines should be
time-honored method o in icting mild torture on patients.
placed with uids running wide open, and norepinephrine
T is practice is grounded in tradition and perceived bene ts o
(or other pressor) should be available i her pressure does not
packing large, deep so tissue wounds. However, wound packing
improve rapidly. “A” is o special note. Patients with classic toxic
has not been shown to bene t small skin abscesses (e.g., less than
shock syndrome (staphylococcal) are not bacteremic. T ey are
5 cm), but it has been shown to increase pain, prolong healing and
su ering rom the e ects o a localized in ection that has pro-
result in greater healthcare utilization. “C” sounds good or a pa-
duced a toxin that is the causative agent in their illness. T ere-
tient who has a history o skin abscesses and is probably worth
ore, while an anti-staphylococcal drug is important (as is locally
a try in patients with recurrent MRSA skin in ections; but the
treating the site o in ection with incision and drainage, toenail
bene ts are minimal at best. Even a er clearing MRSA, the little
removal, etc.), the anti-staphylococcal drug is not to treat bac-
bugger has a tendency to recur; re-colonization rates are as high
teremia. However, the patient should receive empiric antibiotics
as 75% several months a er decolonization. Finally, decoloniza-
(IV clindamycin and vancomycin) to treat a suspected localized
tion with “bleach bathes” has also been tried, and these seem to
in ection, with narrowing o the spectrum when more data (e.g.,
be marginally e ective i they are e ective at all (Clin In ect Dis.
cultures and susceptibilities) become available. “D” and “E” are
2014;58:679).
incorrect since neither steroids nor blood products are cur-
I treating a MRSA cellulitis, many options are available,
rently indicated.
including “D” and “E.” Antibiotic therapy should be dictated by
culture and susceptibility when available. Empiric oral antimicro-
HELPFUL TIP: bials that are generally regarded as e ective against MRSA include
Norepinephrine is the pressor o choice in almost all sit - trimethoprim-sul amethoxazole, tetracyclines (e.g. doxycycline),
ations. It has s perior s rvival when compared to dopa- and linezolid. For more serious MRSA in ections and those ail-
mine in sepsis and is clearly better in cardiogenic shock. ing to respond to empiric oral antibioitics, vancomycin (30 mg/kg
per 24 hours) is still rst-line IV therapy. Other options include
Objectives: Did you learn to . . . daptomycin, ce aroline, lipoglycopeptides, and tigarcycline (3rd
• Identi y signs and symptoms o toxic shock syndrome? or 4th line due to increased mortality risk). T e newer lipoglyco-
• Describe the pathophysiology o toxic shock syndrome? peptides (e.g., oritavancin, dalbavancin) are long-acting antimi-
crobials that have the advantage o in requent administration—in
• Initiate management or a patient with sepsis and toxic
shock syndrome? act, oritavancin is given as a single dose. T e lipoglycopeptides
are expensive and have no medical advantage over less expensive
options. In addition, they are or skin in ections only and not
QUICK QUIZ: BOILS IN BOSTON systemic in ections. T ere is too much resistance to clindamycin
to recommend it when treating MRSA.
A 15-year-old male presents with his mother who had to drag
him in to your clinic. He won’t look up rom his iPhone as his
mother gives an exasperated history. He’s had “boils” several
CASE 8.10
times on his buttocks and legs that he pre ers to “pop” rather One o your patients has recently been in Indonesia or a
than to have treated by a medical pro essional. oday he has a prolonged period o time. Upon return to the United States,
large red lump on his anterior le thigh. On examination, you he develops a ebrile illness 5 days a er landing in Iowa (yes,
nd a healthy-looking male with normal vital signs and a 3 cm we have airports). When he presents to the ED, he recalls
area o erythema with central pustule and uctuance. You incise being bitten by a particularly large and persistent mosquito
the lesion, obtain a sample or culture and drain it completely just be ore boarding the airplane. On arrival to the ED, he
(just in time or lunch). complains o severe, di use body pain, headache, and eye
pain and eye redness. Vitals show a temperature o 38.5°C,
I the culture returns with MRSA as you suspect it will, you pulse 145 bpm. Labs show a low white count and throm-
must treat with: bocytopenia. He does NO note cyclic evers. His eosino-
A) Good local wound care phil count is normal. A Giemsa stain o the blood shows no
B) Packing the wound daily organisms.
256 FAMILY MEDICINE EXAMINATION & BOARD REVIEW

Question 8.10.1 The most likely cause o this illness is: Objectives: Did you learn to . . .
A) Pneumococcal sepsis • Describe the presentation o Deng e ever?
B) Dengue ever • Disting ish the dif erence between the rst and s bseq ent
C) Malaria episodes o Deng e ever?
D) Filariasis
E) Kuru

Answer 8.10.1 The correct answer is “B.” T is patient has Clinical Pearls
Dengue ever. Dengue ever is most common in Asia but also
Do not delay antibiotics in a s spected case o meningitis. I
occurs in A rica, South and Central America, and the Carib- the l mbar p nct re is going to be delayed, administer
bean (and may eventually occur in the southern United States i antibiotics irst.
global temperatures continue to rise). Patients typically present
Do not rely on Kernig and Br dzinski signs to r le o t
with ever, conjunctivitis, headache, retro-orbital pain, leukope-
meningitis. The sensitivity is as low as 9%.
nia, and thrombocytopenia. T e sine-qua-non o Dengue ever
is severe myalgias and arthralgias, hence the moniker “break Do not treat asymptomatic bacteri ria in most patients; it is
not help l and only leads to resistance.
bone ever.” More mild orms do occur, however. “A” is incor-
rect since patients with pneumococcal sepsis will generally not Do not treat Lyme disease with prolonged co rses o
have thrombocytopenia and a low WBC count (yes, we know antibiotics. Postin ectio s complications are not d e to
there are exceptions). “C” is incorrect because the patient does persistent in ection.
not have cyclic evers and a blood smear is negative (plus, we’ve Do not treat pper respiratory in ections with antibiotics.
already had our malaria case. Why would we repeat it?). Fila- Do not se pre-proced ral prophylactic antibiotics or most
riasis, “D,” presents with eosinophilia and micro laria in the valv lar heart disease. Exceptions incl de prosthetic valves,
bloodstream. Again, this should be evident on the blood smear. pervio s endocarditis and some congenital heart diseases.
“E,” kuru, is a prion-driven degenerative neurological disease Do not se vancomycin in patients who prove to have
ound in Papua, New Guinea. T is patient does not have any methicillin-sensitive Staphylococcus aureus; o tcomes are
evidence o neurological disease, aside rom a headache, which worse with vancomycin than with targeted coverage s ch as
also makes this option less likely. na cillin.
Start l ids and antibiotics as soon as possible in cases o
With supportive care, the patient recovers. However, not pres med sepsis. It may take 3 L or more o crystalloid to
having learned his lesson he returns to Indonesia. When he stabilize the septic patient.
comes home to Iowa again, he presents again to the ED, sus- There is no need to excl de patients with head lice rom
pecting that he has Dengue ever. school. Avoid sharing hats and head to head contact,
however.
Question 8.10.2 Signs and symptoms o a second occur-
Treat patients with a positive PPD who have had the BCG
rence o Dengue ever include all o the ollowing EXCEPT: vaccine j st the same as any other patient with a positive
A) Capillary leak with hypovolemia and hemoconcentration PPD. Age is not a limitation to treating with isoniazid.
B) Hemorrhagic complications including capillary ragility
C) Onset >14 days a er exposure
D) Severe thrombocytopenia

Answer 8.10.2 The correct answer is “C.” I a returning trav- BIBLIOGRAPHY


eler presents more than 14 days a er returning home, Dengue Angus DC, van der Poll . Severe sepsis and septic shock.
ever can be e ectively ruled out. In this case, think o other N Engl J Med. 2013; 369: 840–851.
in ectious diseases including malaria. T e incubation period o Bagdasarian N, et al. Diagnosis and treatment o Clostridium
Dengue ever is 3 to 7 days. T e rest are correct. Patients with dif cile in adults a systematic review. JAMA. 2015;13(4):
their second or subsequent episode o Dengue ever can present 398–408.
with capillary leak syndrome, marked thrombocytopenia, and Bratton RL, Corey R. ick-borne disease. Am Fam Physician.
severe hemorrhage. It is usually the second or subsequent in ec- 2005;71(12):2323–2330.
tion that leads to mortality. Centers or Disease Control and Prevention. Section on in u-
enza. http://www.cdc.gov/ u/ pro essionals/index.htm.
HELPFUL TIP: Accessed June 28, 2015.
The treatment o Deng e ever is s pportive care. In Centers or Disease Control and Prevention. Section on
many co ntries, treatment is beg n with alb min to traveler’s health. http://wwwnc.cdc.gov/ travel/. Accessed
replace the circ lating vol me (given the capillary leak June 28, 2015.
syndrome). However, saline is j st as good and a lot less Centers or Disease Control and Prevention. Section on
expensive. vaccines. http://www.cdc.gov/vaccines/hcp/acip-recs/
recs-comprehensive.html. Accessed June 28, 2015.
CHAPTER 8 • INFECTIOu S DISEASES 257

Kang DH, et al. Early surgery or le -sided endocarditis Sterling S, et al. T ree months o ri apentine and isoniazid or
accompanied by severe valve dys unction and large latent tuberculosis in ection. N Engl J Med. 2011;365:
vegetation, even in absence o heart ailure, saves lives 2155.
and prevents serious illness rom stroke. N Engl J Med. al S, et al. Fever o unknown origin in older adults. Clin Geriatr
2012;366:2466–2473. Med. 2007;23:649–668, viii.
Lawn SD, Zumla AI. uberculosis. Lancet. 2011;378:57–72. olan RW Jr. Fever o unknown origin: A diagnostic approach
Lew DP, Waldvogel FA. Osteomyelitis. N Engl J Med. 1997; to this vexing problem. Clin Pediatr. 2010;49:207–213.
336:999–1007. Wendel K, Rompalo A. Scabies and pediculosis pubis: An
Mylonakis E, Calderwood SB. In ective endocarditis in adults. update o treatment regimens and general review. Clin
N Engl J Med. 2001;345:1318. In ect Dis. 2002;35:S146–S151.
Nichol KL, et al. E ectiveness o in uenza vaccine in Wilson W, et al. Prevention o in ective endocarditis: Guideline
community-dwelling elderly. N Engl J Med. 2007;357: rom the American Heart Association. Circulation. 2007;
1373–1381. 116(15):1736–175.
HIV/AIDS
A. Ben Appenhei er and Jack T. Stap et n
9
Note: T e antiretroviral treatment o HIV/AIDS (HAAR ) Answer 9.1.1 The correct answer is “E.” T is presentation is
continues to evolve, and guidelines or use are regularly consistent with an acute retroviral syndrome, which occurs very
updated (https://aidsin o.nih.gov/guidelines). T is chapter early in the in ection and is characterized by a mononucleosis-
ocuses on the primary care aspects o HIV/AIDS including like illness that can last several weeks. Current HIV diagnostic
initial evaluation, drug side e ects, and in ectious disease ELISA methods include the option or both antibody and antigen
prophylaxis. detection. Since the antibody to HIV will not develop or at least
2 to 8 weeks a er in ection and the retroviral syndrome typically
CASE 9.1 occurs be ore seroconversion, HIV antibody tests, including
rapid detection methods (“B”), may well be negative. During the
A 23-year-old emale presents to your clinic complaining o
acute HIV in ection, HIV viral loads are very high, and patients
sore throat, ever, and body aches. She reports that the ill-
are more in ectious compared to other times during their HIV
ness began about a week ago and has persisted despite ther-
in ection. Consequently, the HIV antigen assay, which measures
apy with NSAIDs, acetaminophen, and sore throat lozenges.
HIV p24 protein, is typically positive during this period. Not all
She denies cough, abdominal pain, nausea, or vomiting, but
laboratories have adopted the HIV antibody–antigen ELISA. I
reports a persistent headache. Her past medical and surgical
this option is not available, the alternative approach would be to
history is unremarkable. T e patient smokes about one pack
measure the HIV RNA by PCR. However, the PCR test is more
o cigarettes a week, drinks occasional alcohol, and denies
expensive, thus the antibody–antigen ELISA is pre erred. T e
other drugs, including intravenous (IV) use. She is hetero-
HIV DNA PCR assay (“C”) is not standardized and should not
sexual, and has had eight sexual contacts in the past year. She
be used in this setting. T e CD4 count (“D”) is not a reliable way
takes oral contraceptives, and her partners usually do not use
o diagnosing HIV in ection; it can become depressed with any
condoms.
acute illness or may be normal in early HIV disease.
On examination, her vital signs are 38.9°C, P 112 bpm,
BP 115/68 mm Hg, R 20 bpm. She has pharyngitis and
HELPFUL TIP:
enlarged tonsils with exudates. T ere is di use cervical
I ur c inica suspici n is high and the antib d /anti-
lymphadenopathy, but the neck is supple. T ere are enlarged
gen c binati n test is negative, the ne t step ud
lymph nodes in her axillae and inguinal areas as well. T e
be an HIV RNA PCR. This can be detected appr i ate-
spleen is palpable and nontender. T e rest o the examination
10 da s a ter in ecti n and is t pica p sitive 4 t
is unremarkable. You obtain a throat culture, CBC with di -
10 da s pri r t the HIV p24 antigen.
erential, and heterophile antibody (Monospot) test. Given
her history o unprotected intercourse with eight new part-
ners within the last year, you also consider testing or HIV. T e ELISA returns positive or HIV antigen and negative
or antibody. A er appropriate treatment and counseling
Q estion 9.1.1 The most appropriate laboratory test(s) to about her test results, blood is also sent or CD4 count, HIV
r le o t the ac te retroviral syndrome wo ld be: RNA viral load testing, and drug-resistance (genotype) test-
A) HIV-1 antibody by ELISA ollowed by a Western blot. ing. Follow-up is arranged or the patient, and she returns in
B) HIV-1 antibody by rapid detection method. 4 weeks with no complaints or symptoms. A complete his-
C) HIV DNA by PCR. tory and physical examination are per ormed. T e patient
D) CD4 lymphocyte count. has mild cervical lymphadenopathy and no other ndings.
E) Combined HIV-1 antibody and antigen ELISA test. Laboratory studies are ordered and show:

258
Ch Apt Er 9 • HIV/AIDS 259

WBC 3,200 cells/mm3 and risks the spread o resistant strains to other patients. T e
HC 42% current recommendations are to start all HIV in ected indi-
Platelets 185,000 cells/mm3 viduals on HAAR therapy regardless o CD4 count, although
Chemistry panel (normal) the strength o these recommendations varies depending on the
Liver enzymes (normal) CD4 count. Early HAAR therapy helps prevent disease pro-
CD-4 lymphocytes 645 cells/mm3 gression or HIV-in ected individuals and is also an important
HIV viral load 5,000 copies/mL step in helping prevent transmission o HIV. Evidence does not
HIV genotype has K103 N mutation support using the viral load (“E”) as an independent determi-
nant o initiating therapy.
Q estion 9.1.2 At her next visit, what baseline st dies
sho ld be ordered? HELPFUL TIP:
A) PPD or Quanti eron Gold testing. HAART regi ens have bec e increasing c p e
B) Rapid plasma reagin (RPR) or Syphilis IgG. ith the additi n an ne drugs, c asses, and
C) Repeat HIV ELISA. c binati n pi s. H ever, the basics can be br ken
D) Hepatitis B and C antibody. d n and si p i ied. The backb ne usua c nsists
E) All o the above. t nuc e side reverse transcriptase inhibit rs (NRTIs).
Answer 9.1.2 The correct answer is “E.” During the initial There are a e p pu ar c binati n pi s used r this
assessment o an HIV-in ected person, all o these studies are backb ne but the t ten used are Truvada (ten -
important. A positive PPD (> 5 mm induration in a person vir/e tricitabine) and Epzic (abacavir/ a ivudine).
in ected with HIV) warrants treatment or latent tuberculosis These are p pu ar because the are b th nce dai and
( B) i the patient is ound to not have active disease. Since are c bined ith an additi na edicati n r re a-
patients with any sexually transmitted in ection (S I) are at risk tive si p e regi ens. T c p ete a regi en, an addi-
or another S I, screening or syphilis with an RPR or syphilis ti na edicati n is added r a di erent c ass usua
IgG is recommended. T e same rationale applies or hepatitis either a n nnuc e side reverse transcriptase inhibit r
B and C, which can be acquired via the same routes as HIV. (e.g., e avirenz), pr tease inhibit r (e.g., darunavir/rit -
T e patient has documented HIV by virtue o the positive HIV navir), r integrase inhibit r (e.g., d utegravir, ra te-
antigen–antibody ELISA and positive HIV RNA; however, doc- gravir, r e vitegravir). The t current ne-pi nce
umentation o seroconversion is important, and a repeat HIV dai initia rec ended regi ens c bine an NRTI
antibody ELISA should be sent at 12 weeks with positives con- backb ne ith and integrase inhibit r. These are Tri-
rmed by HIV Western blot. u eq (Epzic + d utegravir) r Stribi d (Truvada +
e vitegravir/c bicistat).
HELPFUL TIP: Common Adve se Effec s of Ar V medica ions include:
Gen t pe testing r resistance against certain edi- Ten vir (c p nent Truvada)—rena ai ure
cati ns sh u d be per r ed at base ine diagn sis re- Abacavir (c p nent Epzic )—h persensitivit
gard ess hether antiretr vira (ARV) therap i be reacti n
initiated r de erred. E avirenz—CNS e ects inc uding increased suicida it
Atazanavir—as pt atic h perbi irubine ia
T e patient is counseled appropriately about all her results.
HELPFUL TIP:
Q estion 9.1.3 What is the most important actor in deter
Current guide ines rec end treat ent immedia e-
minin when to start hi hly active antiretroviral therapy
ly fo egnan women a any s age of h IV infec ion,
(HAART)?
inc uding during the acute retr vira s ndr e. Treat-
A) A rising viral load.
ent thers during the acute retr vira s ndr e is
B) A decrease in CD4 count.
c nsidered pti na , and there is n pr ven ng-ter
C) T e development o an opportunistic in ection.
bene it. HIV edicati ns suitab e r use during preg-
D) T e patient’s willingness and ability to maintain strict adher-
nanc ust be used.
ence to an antiviral regimen.
E) An undetectable viral load (< 50 copies/mL).
Q estion 9.1.4 Aside rom considerin HAART and stress
Answer 9.1.3 The correct answer is “D.” T e decision to in the importance o partner noti cation, what other
start HAAR must be done on an individual basis. T e most intervention sho ld be o ered at this sta e?
important consideration by ar, however, is the willingness o A) 13-valent conjugated pneumococcal and hepatitis B vac-
the patient to strictly adhere to complicated medical regimens. cines.
Poor compliance virtually guarantees the development o B) rimethoprim/sul amethoxazole ( MP/SMX) DS one tablet
resistance, hampers treatment o the patient in later stages, per day or the prevention o P. jiroveci pneumonia (PCP).
260 FAmIl y mEDICINE ExAmINATIo N & Bo ARD REVIEw

TABLE 9-1 r ECOMMENDED pr Oph YLAXIS IN HELPFUL TIP:


h IV pAt IENt S Partner n ti icati n is ver i p rtant. State a s var
CD4± c nsiderab regarding partner n ti icati n, and state
Co nt Or anism Recommended Prophylaxis hea th depart ents are usua ver he p u in aci itat-
<200 Pneumocystis TmP/Smx r daps ne (sh u d ake
ing this. S e states have cri ina trans issi n statutes
sure patient is n t G6PD de cient) r having se ith ut te ing the partner ne’s HIV
status. Pr secuti n a n t require actua trans issi n.
<100 T p as sis TmP/Smx r c binati n
daps ne + p ri etha ine +
euc v rin

<50 m c bacteriu Azithr cin, c arithr cin, r A er consultation with an HIV specialist, the patient elects
aviu c p e ri abutin not to start therapy at this time and is scheduled or ollow-up
(mAC) with regular checks o her viral load and CD4 count. A er 1
year, the patient’s lab values have changed: CD-4 lymphocytes
Adapted r Pane n o pp rtunistic In ecti ns in HIV-In ected Adu ts and 280 cells/mm3 and HIV viral load 75,000 copies/mL. In the
Ad escents. Guide ines r the preventi n and treat ent pp rtunistic
in ecti ns in HIV-in ected adu ts and ad escents: rec endati ns r
past year, she has been treated three times or lobar pneumo-
the Centers r Disease C ntr and Preventi n, the Nati na Institutes nia and once or oral candidiasis (without esophageal disease).
Hea th, and the HIV medicine Ass ciati n the In ecti us Diseases S ciet
A erica. http://aidsin .nih.g v/c ntent es/ vguide ines/adu t_ i.pd . Q estion 9.1.5 Does this patient meet the CDC case de ni
Accessed August 31, 2015. Tab e 1. tion or the acq ired imm ne de ciency syndrome (AIDS)?
A) No, because she has not had an AIDS-de ning illness.
C) Azithromycin 1200 mg per week or the prevention o B) No, because her CD4 count is > 200 cells/mm 3.
Mycobacterium avium complex (MAC). C) No, because she has only been diagnosed with HIV in ection
D) Fluconazole 100 mg per day or the prevention o cryptococ- or 1 year.
cal meningitis. D) Yes, because she has had recurrent (two or more episodes) o
lobar pneumonia.
Answer 9.1.4 The correct answer is “A.” Adequate immuni- E) Yes, because her viral load is > 10,000 copies/mL.
zations at a clinical stage when the patient is likely to bene t
rom the vaccines (i.e., CD4 > 500 cells/mm 3) are important. Answer 9.1.5 The correct answer is “D.” T e CDC HIV clas-
Live vaccines, such as the MMR, should be avoided in immu- si cation system requires a case o HIV in ection be reported as
nocompromised persons, generally considered those HIV- AIDS i the CD4 count is less than 200 cells/mm 3 or the patient
in ected persons with a CD4 count < 200 cells/mm3. Pneumo- develops an AIDS-de ning illness. T ese AIDS-de ning ill-
coccal vaccines are indicated or HIV positive patients. While nesses include esophageal (not oral) candidiasis, cryptococcal
the polysaccharide 23-valent vaccine (Pneumovax) covers more in ection, disseminated histoplasmosis, invasive cervical can-
serotypes, the 13-valent conjugate vaccine (Prevnar) has a more cer, tuberculosis, HIV wasting disease, and recurrent pneumo-
robust immune response. I previously unvaccinated, the rec- nia (more than one episode per year). Other in ections, Kaposi
ommendation is to start with the 13-valent conjugate vaccine sarcoma, and certain lymphomas may also de ne AIDS in an
ollowed by the 23-valent polysaccharide pneumococcal vac- HIV-in ected person. Duration o in ection and viral load are
cine a er at least 8 weeks. A er 5 years, boost with the 23-valent not currently criteria.
pneumococcal vaccine.
MP/SMX (“B”) or PCP prevention is indicated when T e patient is started on teno ovir/emtricitabine ( ruvada)
the CD4 drops below 200 cells/mm 3, and azithromycin (“C”) and ritonavir-boosted atazanavir once daily. E avirenz
is indicated or MAC prophylaxis when the CD4 count drops (Sustiva) was not used due to the original resistance testing.
below 50 cells/mm 3. Fluconazole (“D”) is used or chronic sup- In addition, the patient is o childbearing age and e avirenz
pression a er the treatment o cryptococcal meningitis or or is contraindicated during pregnancy i the patient were to get
the treatment o esophageal candidiasis; but it is NO currently pregnant. She does well with the treatment, and tolerates the
used as prophylaxis. T ere is no survival bene t to prophylaxis medications. On a later routine ollow-up, she reports mild
or cryptococcal meningitis. See able 9-1 or recommended atigue, but is otherwise well. Her laboratory results over sev-
prophylaxis in patients with HIV. eral visits are listed in able 9-2.

HELPFUL (BUT CONFUSING) TIP:


PCP is n nger P. carinii. It is n Pneu c stis jir veci. TABLE 9-2 CASE 1, LABOr At Or Y r ESULt S
w e didn’t d it, h nest. It as s e icr bi gist- Jan ary March May
ta n ist h anted t c n use us a . Just t add 3 3 3
CD4 c unt 204 ce s/ 178 ce s/ 244 ce s/
t the c n usi n, P. jir veci pneu nia is sti ten
abbreviated “PCP” as in Pneu C stis jir veci pneu nia. Vira ad 5,500 c pies/ l <50 c pies/ l <50 c pies/ l
Ch Apt Er 9 • HIV/AIDS 261

Current laboratory results also include WBC 4,500 cells/ Repeat CD4 count and viral load 2 weeks later shows:
mm3, Hgb 12.1 g/dL, platelets 128,000 cells/mm3, and total
bilirubin 2.1. CD4 count: 32 cells/mm3
Viral load: 7,100 copies/mL
Q estion 9.1.6 At this point, what chan es, i any, sho ld
be made to the patient’s re imen? Q estion 9.1.7 At this point, what chan es, i any, sho ld
A) T e patient has ailed HAAR treatment, and the drug regi- be made to the patient’s re imen?
men should be changed. A) T e patient has ailed HAAR treatment and her ARV regi-
B) T e patient has su ered a severe adverse e ect (hyperbiliru- men should be adjusted.
binemia) rom the drug regimen and all three drugs should B) Since her hyperbilirubinemia has persisted, the patient is
be changed. assumed to have su ered a severe adverse e ect rom her
C) T e patient has ailed to reconstitute her immune system atazanavir, so it alone should be changed.
(CD4 count still less than 200 cells/mm 3), so one o her C) T e viral load is not over 50,000 copies/mL, so the current
drugs should be changed. regimen should be continued.
D) T e patient is doing well and her regimen should be contin- D) T e patient is doing well and her regimen should be contin-
ued. T e hyperbilirubinemia is a side e ect associated with ued. T e hyperbilirubinemia is a side e ect associated with
atazanavir. atazanavir.

Answer 9.1.6 The correct answer is “D.” T e patient’s viral Answer 9.1.7 The correct answer is “A.” T e patient has ailed
load is suppressed, which is the primary goal o HAAR . T e HAAR based on several criteria, including the reemergence o
patient has not su ered any major adverse reactions. Indirect detectable viral RNA a er it had been completely suppressed
hyperbilirubinemia is commonly seen with atazanavir, and other and a alling CD4 count (see explanation o previous question,
hepatic enzymes should be ollowed regularly to ensure that the above). Her ARV therapy should be changed once the results o
change in laboratory studies is only due to the atazanavir. It can the repeat genotype testing are known.
be mild, but can become severe on occasion. Criteria or chang-
ing drug regimens include < 1 log10 reduction in viral RNA by A new ARV therapy regimen is recommended based on the
8 weeks (e.g., 100,000 to 10,000 is a 1 log10 reduction), ailure to genotype results along with PCP and MAC prophylaxis,
depress viral RNA to undetectable levels by 6 months, repetitive with MP/SMX and azithromycin, respectively. wo weeks
detection o viral RNA a er initially achieving undetectable lev- later, the patient’s CD4 count rises to 210 cells/mm3 and she
els, persistent decline in CD4 counts (on at least two measure- has more energy. However, she returns to clinic 3 weeks later
ments), or signi cant clinical deterioration. T e expected rise in (5 weeks a er starting her new HIV regimen) and is com-
CD4 counts is much slower than the all in HIV RNA, and little plaining o severe shortness o breath. She says that she had
or no change may be seen in the rst 6 months o therapy. been taking her HIV medications and the azithromycin, but
that she lost her PCP prophylaxis prescription ( MP/SMX),
HELPFUL TIP: and had orgotten to get a new one. Her current illness
w hen changing drug regi ens r vir gic ai ure, it began 6 days ago as a ever and mild cough. She devel-
is i p rtant t repeat gen t pe testing hi e n their oped signi cant dyspnea with minimal exertion, and now
current regi en t deter ine hether the patient’s is even short o breath at rest. Her chest hurts bilaterally,
virus has n deve ped resistance t their current worse with inspiration. She has drenching night sweats. She
regi en. denies hemoptysis, sputum production, nausea, vomiting,
or abdominal pain.
Physical examination reveals the ollowing vital signs:
T e patient’s HAAR regimen is maintained. However, she 39°C, BP 90/60 mm Hg, P 135 bpm, RR 38 bpm, and SpO2
misses her next two appointments, and returns to clinic 78%on room air. T e patient is in severe respiratory distress
6 months later. She reports taking all o her medications but and in the tripod position. T e lung examination shows di -
complains o a 10-lb unintended weight loss. She also notes use rales and tachypnea. You call the ambulance because she
increased requency o night sweats but no evers. A physical looks like she needs help! You place oxygen to help improve
examination is unremarkable except or a gaunt appearance her oxygen saturation and her respiratory rate. She does not
and temporal muscle wasting. respond to oxygen and becomes unresponsive.
Her laboratory results show:
CD4 count: 78 cells/mm3 Q estion 9.1.8 Aside rom respiratory isolation, what sho ld
Viral load: 6,400 copies/mL be done next?
CBC: WBC 2,400 cells/mm3 A) Sputum culture to evaluate or any lung organisms that may
Hgb: 11.3 g/dL be causing her respiratory distress.
Platelets: 145,000 cells/mm3 B) Bronchoalveolar lavage (BAL) or PCP via direct immuno-
otal bilirubin: 1.8 uorescence (DFA).
262 FAmIl y mEDICINE ExAmINATIo N & Bo ARD REVIEw

C) Bag mask the patient and have your nurse place an IV in


order to prepare or rapid sequence intubation and IV uids.
D) Oxygen, urosemide, and nitroglycerin. She clearly is vol-
ume overloaded.
E) Chest x-ray (CXR), blood count, CD4 count, and viral load
to assess or underlying in ection.

Answer 9.1.8 The correct answer is “C.” T e patient presented


in severe respiratory distress and now has impending respira-
tory ailure. She is also hypotensive and tachycardic and must be
stabilized be ore any urther workup is done. Remember those
ABC’s (Airway, Breathing, Circulation) rom all those certi -
cation classes you had to take? “D,” Oxygen, urosemide, and
nitrates are use ul treatments o congestive heart ailure, but this
is unlikely in this young woman. P. jiroveci pneumonia (PCP) is
the most likely diagnosis that explains o her ndings. Further FIg u RE 9-1. Chest radi graph.
workup including sputum culture, PCP direct immuno uores-
cence, CXR, and blood count are all indicated but only a er the
patient is stabilized. Given the timing o the onset o symptoms
in the setting o a rapid CD4 response to ARV therapy, this is Q estion 9.1.9 What sho ld be the initial antibiotic therapy?
likely related to immune reconstitution in ammatory syndrome A) Four drug anti-tuberculosis regimen.
(IRIS). B) Azithromycin and Ce riaxone IV.
C) MP-SMX IV.
D) IV corticosteroids ollowed by MP-SMX IV.
HELPFUL TIP:
E) No antibiotics initially—just wait or the culture results.
I une rec nstituti n in a at r s ndr e (IRIS–
as pp sed t ISIS the terr rist rganizati n) is e
Answer 9.1.9 The correct answer is “D.” T e patient is
na ed. It is identi ied b a parad ica sym oma ic
acutely ill, and likely has PCP. Steroids decrease the mortality
rsening a pre-e isting in ecti us pr cess as the
in patients with severe PCP (PaO2 < 70 on room air or arte-
i une s ste rec nstitutes ith ARV therap . Basi-
rial–alveolar O2 gradient > 35 mm Hg). T e best antibiotic or
ca , the b d ’s i une resp nse recks hav c at the
PCP is MP-SMX. Pentamidine IV may be used in cases o
sites in ecti n (e.g., CmV retinitis, tubercu sis, cr p-
sul a allergy, but it has been associated with hypotension and
t c cca eningitis and bvi us , PCP). The in ecti n
hypoglycemia. Although the classic CXR appearance or PCP
a be under treat ent r subc inica and bec es
is bilateral interstitial in ltrates, it can present di erently (as
s pt atica rse hen IRIS deve ps. C n
in this case). A normal CXR examination is not uncommon,
indings are ever, increased atigue and a aise, un-
and approximately 10% o people diagnosed with PCP have a
e p ained eight ss, rsening respirat r and neu-
normal CXR on presentation. Since the in ecting organism is
r gica status. IRIS usua ccurs ithin the irst 4 t
not known with certainty, it would prudent to secondarily add
8 eeks a ter initiati n therap but it can be de a ed
empiric therapy or bacterial pneumonia with ce riaxone and
severa nths in s e cases.
azithromycin or respiratory uoroquinolone pending bron-
choscopy results. However, this is not the initial best treatment
or the patient’s clinical condition. O note, PCP does not grow
A er appropriate resuscitation and transportation to a in standard cultures; but it can be visualized by silver staining
higher level o care at the local emergency room, laboratories or direct immuno uorescence assays (DFA) on BAL specimens.
and a CXR are obtained. T e x-ray is shown in Figure 9-1. Detection o PCP is not reduced during the rst 48 hours a er
Laboratory results are: starting therapy, and cysts may be seen or much longer in peo-
ple responding to therapy, despite e ective treatment.
WBC 3,400 cells/mm3
Hgb 11 g/dL T e patient is admitted to the ICU and given antibiotics.
Platelets 180,000 cells/mm3 Her condition stabilizes until 4 hours into her ICU stay. At
Creatinine 2.4 mg/Dl (high) that time, her respiratory rate on assisted-control ventilation
LDH 1,280 IU/L (high) dramatically increases rom 18 to 42. Peak airway pressures
P 12.4 seconds, P 27 seconds according to the ventilator are greater than 60 cm o water,
Liver enzymes normal when they were less than 30 cm water previously. Breath
ABG: pH 7.56, PaCO2 23 mm Hg, PaO2 68 mm Hg (on sounds are absent on the right side o her chest, and her tra-
100%FiO2) chea is deviated to the le . Heart sounds are audible, but
Sputum and blood cultures are pending. tachycardic. Neck veins are distended bilaterally.
Ch Apt Er 9 • HIV/AIDS 263

Q estion 9.1.10 What is the next appropriate step, BEFORE problems but does smoke pack o cigarettes per day.
a repeat CXR is taken? She also admits to occasional alcohol use (one drink every
A) Give Versed 2 mg IV or sedation, the patient must be very 2 weeks). She denies illicit drug use, including IV drug use.
anxious.
B) Pull the E tube back 1–2 cm, as it is likely in the right main Q estion 9.2.1 Besides prenatal vitamins with iron and
stem bronchus. olate, yo recommend:
C) Per orm a blind pericardiocentesis, as the patient is develop- A) Smoking cessation.
ing tamponade. B) Con rming the home pregnancy test with a serum HCG in
D) Insert a large Angiocath into the le second intercostal space your laboratory.
to relieve the tension pneumothorax on the le . C) HIV testing and counseling.
E) Insert a large Angiocath into the right second intercostal D) A and C.
space to relieve the tension pneumothorax on the right. E) All o the above.

Answer 9.1.10 The correct answer is “E.” T e patient has Answer 9.2.1 The correct answer is “D.” Smoking during preg-
developed a dreaded complication o PCP, a pneumothorax. nancy is associated with lower birth weight and preeclampsia,
T e organism, Pneumocystis, has a propensity to cause blebs and smoking in the house with a young child is associated with
(cysts) in the lung tissue. Since the patient was receiving positive respiratory diseases, especially asthma. Although con rming
pressure ventilation, a tension pneumothorax developed that pregnancy by examination (uterine size or etal heart tones)
requires immediate needle decompression. T is should be done and/or urine HCG is appropriate, serum HCG is unnecessary
without waiting or a portable CXR. When the pneumothorax is and expensive. In addition, when used correctly, home preg-
later con rmed, a tube thoracostomy may be per ormed under nancy tests are highly sensitive and speci c. But really, this is
controlled conditions. the HIV chapter, and we want you to know that HIV screen-
ing should be included in the routine panel o prenatal tests
Following a protracted ICU admission, the patient improves, or all women seeking prenatal care. Routine testing or HIV
and is re-started on the HIV medication regimen that was in expectant emales has dramatically reduced the HIV preva-
selected based on her drug resistance testing. She is dis- lence in children in developed countries. Vertical transmission
charged, and in response to this li e-threatening complica- o HIV is still a tremendous problem in A rica and other devel-
tion, she is highly motivated and becomes very adherent with oping regions o the world.
her medications. wo years later her CD4 is 385 cells/mm3,
her HIV RNA concentration is < 20 copies/mL (nondetect- HELPFUL TIP:
able), and she recently started a job that she enjoys. Specia c nsent is no necessar pri r t testing r
Objectives: Did yo learn to . . . HIV and a patient’s genera c nsent r edica care is
• Identi the signs and s pt s the acute retr vira s n- en ugh. o c urse, this a depend n the a s the
dr e? state in hich u practice. The CDC rec ends that
• Use appr priate tests r the diagn sis HIV in ecti n? testing a pregnant en be r utine and ered
• Eva uate a patient ith HIV and nit r that patient’s pr g- as an “ pt ut” test ( eaning n specia c nsent r HIV
ress? testing).
• Revie the preventative hea th easures and vaccinati ns
i p rtant in patients ith HIV in ecti n?
• Understand the guide ines r initiating and changing You explain that HIV testing is routine, but that the patient can
HAART?
“opt out,” and the patient agrees to HIV testing. Her pregnancy
• Rec gnize s e the re c n edicati ns used t is con rmed. Her HIV Ab/Ag combination test is positive.
treat HIV and their side ef ects?
• Rec gnize s e the re c n pp rtunistic in ec- Q estion 9.2.2 What is the next step in con rmin the dia
ti ns in patients ith HIV and h t prevent the ?
nosis o HIV?
• Describe IRIS? A) HIV viral load
B) HIV-1/HIV-2 multispot test
C) HIV western blot
CASE 9.2 D) HIV genotype
A 32-year-old emale presents to the of ce seeking prenatal E) CD4 count
care. Her last menstrual period was 2 months prior to her
visit. She believes that she is pregnant and has tested posi- Answer 9.2.2 The correct answer is “B.” T e recommended
tive with a home pregnancy test. She has been pregnant twice testing algorithm has recently changed. Previously an HIV Ab
be ore, with one living child and one spontaneous abortion ELISA “screening” test was done initially, ollowed by a “con r-
(G3P1). In an interesting twist on modern romance, she is matory” Western Blot. However, the current recommendation
married to the ather o her children. She has no health (based on CDC guidelines) starts with the HIV antigen/antibody
264 FAmIl y mEDICINE ExAmINATIo N & Bo ARD REVIEw

combination immunoassay. T is combines the p24 antigen test D) Her renal unction makes HAAR relatively contraindicated.
(which is positive be ore seroconversion) with an antibody test E) Her hemoglobin level makes HAAR relatively contraindi-
or IgM and IgG. T is will diagnose any HIV in ection 2 to cated.
3 weeks a er in ection (including HIV-2). I the antigen-
antibody test is positive, it is ollowed by an HIV-1/HIV-2 Ab Answer 9.2.4 The correct answer is “A.” ARV therapy is rec-
di erentiation immunoassay. I positive, HIV diagnosis is con- ommended or all pregnant women regardless o their viral load
rmed. I negative, an HIV RNA viral load should be sent to to prevent transmission to the child. Factors that increase risk
determine whether the patient truly has HIV. o transmission o HIV include high maternal viral load, low
maternal CD4 count, advanced clinical stage o her HIV, and
Your patient is understandably shaken by the news o this lack o maternal use o ARV therapy. Vaginal delivery is also a
test result. Being an empathetic physician, you say something risk actor—but only i the mother did not receive antepartum
like, “I can see that you are shaken by the news o this test ARV therapy. Although the patient’s hemoglobin is low, it does
result.” She is most concerned about her unborn child. not preclude her rom taking therapy. Her renal unction is nor-
mal, so it should not be an issue.
Q estion 9.2.3 What sho ld yo tell her? Ideally, the mother should have her HIV suppressed (unde-
A) Her child is almost certainly also in ected. tectable viral load) on combination antiretroviral therapy
B) A therapeutic abortion at this point is the only humane (HAAR ) prior to delivery. Waiting to start therapy until the
thing to do. second trimester is recommended by some; however, more
C) With e ective therapy, the risk o transmission to the child recent recommendations suggest consideration o initiation o
can be lowered to less than 2%. ARV as soon as HIV is diagnosed in avor o earlier viral sup-
D) With e ective therapy, the risk o transmission to the child pression and decreased risk o transmission. I the HIV viral
can be lowered to 15%. load (HIV RNA concentration in plasma) is nondetectable at
E) Despite e ective therapy, the risk o transmission remains at 36 weeks, the transmission risk is < 2%.
25%. I the mother’s viral load is unknown or > 1,000 copies/mL,
current guidelines recommend also adding continuous in u-
Answer 9.2.3 The correct answer is “C.” Although it is possible sion o zidovudine (ZDV) to the mother during labor as well as
or HIV to in ect the unborn etus, the large majority o mother scheduled cesarean delivery. Postpartum prophylactic medica-
to child HIV transmission occurs due to exposure during deliv- tions or the newborn should be given or 6 weeks regardless o
ery to maternal genital-tract virus. T e most important variable maternal viral load. Women who present in labor without HIV
or transmission is the HIV viral load (HIV RNA concentration testing during pregnancy, or with undocumented HIV in ec-
in plasma) in the mother. ARV drugs reduce perinatal transmis- tion, should be tested by rapid HIV ELISA. I this is positive,
sion by several mechanisms, including lowering maternal ante- continuous in usion ZDV should be started. Current guidelines
partum viral load and providing in ant pre- and postexposure do not recommend additional intrapartum drugs in this setting,
prophylaxis (PEP). T ere ore, combined antepartum, intrapar- including nevirapine, which has been shown to cause rapid
tum, and in ant ARV prophylaxis is recommended to prevent resistance when used in this setting.
perinatal transmission o HIV.
T e patient is started on HAAR (Combivir plus Kaletra)
Your patient is somewhat relieved that her baby can be pro- and tolerates her regimen well. Repeat laboratory results at a
tected, and wants to know what can be done to treat her. She return visit 4 weeks later are as ollows:
eels ne, is now in the second trimester, and would rather CD4 count: 692/mm3
not take medications unless she had to. HIV viral load: 5,500 copies/mL
Some additional laboratory tests are ordered: Hgb: 10.9 g/dL
CD4 count: 756/mm3 Her HAAR seems to be e ective. Her viral load has
HIV viral load: 80,000 copies/mL decreased by > 10- old (one log10). You remember rom your
Hgb: 11.2 g/dL last patient how sick patients can get with P. jiroveci pneumonia.
BUN/Cr: 11 mg/dL/0.7 mg/dL.
Q estion 9.2.5 What do yo recommend to this patient
Q estion 9.2.4 What sho ld yo tell her abo t HAART in re ardin PCP prophylaxis?
pre nancy? A) She should start prophylaxis with MP-SMX immediately,
A) o minimize the risk o transmission to her child, she should because PCP in pregnancy can be particularly severe.
start triple ARV therapy as soon as possible. B) She should start PCP prophylaxis with inhaled pentamidine,
B) ARV medications are teratogenic and should be avoided at because MP-SMX is contraindicated in pregnancy.
all costs during pregnancy, except just be ore delivery. C) PCP prophylaxis is not indicated, since her CD4 count is
C) Since her CD4 count is normal and she eels well with no > 200/mm 3.
sign o opportunistic in ections, starting HAAR is not D) PCP prophylaxis is not a major concern or pregnant pa-
indicated. tients.
Ch Apt Er 9 • HIV/AIDS 265

Answer 9.2.5 The correct answer is “C.” PCP is particularly Answer 9.2.7 The correct answer is “B.” Children born to HIV-
severe in pregnant patients, but prophylaxis is not generally positive mothers will test positive or HIV antibodies as mater-
indicated or CD4 counts > 200 cells/mm 3. MP-SMX is asso- nal antibodies are acquired across the placenta or at least the
ciated with hyperbilirubinemia in newborns, but is still indi- rst 6 months. Maternal HIV antibodies may persist and inter-
cated or PCP prophylaxis. Oral dapsone is another option, as is ere with interpretation o a positive HIV antibody test; there-
inhaled pentamidine. ore, HIV antibody testing is not recommended to diagnose an
HIV in ection in in ants.
She continues her ARV therapy, but at 36 weeks, her viral load
is still detectable (2,500 copies/mL). You recommend chang- Q estion 9.2.8 How sho ld the HIV stat s o the in ant be
ing the ARV, but at the next appointment (37.5 weeks), the determined?
patient tells you that she did not ll the new prescriptions, A) Serial HIV antibody tests: a our old drop in titer can be
and admits that she has not been adherent with her medica- considered negative.
tions. As this visit, you renew discussions about adherence, B) p24 antigen testing in the rst 48 hours o li e.
and about delivery plans with the patient. C) Viral load by PCR in the rst 48 hours o li e.
D) Viral load by PCR at 14 to 21 days, 1 to 2 months, and 4 to
Q estion 9.2.6 Which o the ollowin is/are tr e re ardin 6 months.
the delivery? E) p24 antigen and PCR viral load on cord blood samples.
A) A cesarean section (C-section) is likely to reduce the risk o
transmission to her in ant. Answer 9.2.8 The correct answer is “D.” T e best test to assist
B) A C-section section is indicated, because this patient’s viral with diagnosis o HIV in ection is viral load by PCR, and a
load remains greater than 1,000 copies/mL despite HAAR . positive test (by DNA PCR or RNA assays) indicates likely HIV
C) A C-section should be per ormed at 38 weeks gestation, in ection. Con rmation o HIV in ection is provided by two
prior to the onset o labor. positive virologic tests obtained rom separate blood samples.
D) Peripartum ZDV should be given to the mother and in ant. Overall, the sensitivity o virologic testing increases rapidly by
E) All o the above. 2 weeks. One can consider obtaining virologic testing within the
rst 48 hours in newborns who are at high risk or HIV in ection,
Answer 9.2.6 The correct answer is “E.” I a patient achieves such as in ants born to HIV-in ected mothers who did not receive
e ective suppression with ARV therapy (undetectable viral prenatal ARV therapy or who had HIV viral loads > 1,000 copies/
load), the risk o transmission is minimal, and the mode o mL close to the time o delivery. I this returns positive, this
delivery should depend on the pre erences o the mother and would indicative o an intrauterine in ection rather than intra-
the other usual obstetric actors. Vaginal delivery is not contra- partum in ection, which is normally acquired during delivery.
indicated i the mother’s viral load is suppressed. I , as in this “A” is incorrect. HIV antibody testing is useless in in ants, and
patient’s case, the viral load is > 1,000 copies/mL, current guide- quanti ed titers are not typically generated. “B” is incorrect
lines recommend delivery by C-section at 38 weeks. When per- because the p24 antigen is less sensitive and speci c than the
ormed at 38 weeks, prior to the onset o labor, the relative risk viral load in this setting. “E” is incorrect. ests done on the cord
o transmission is reduced by 50%. Perinatal ZDV, as discussed blood may be contaminated with maternal blood and do not
above, may also help reduce the risk o transmission. Also, the give an accurate assessment o the in ant’s status.
premature rupture o membranes should be addressed promptly
in HIV-in ected mothers. Children born to mothers more than Q estion 9.2.9 What sho ld yo advise yo r patient abo t
4 hours a er rupture are twice as likely to acquire HIV. breast eedin her son?
A) HIV is not transmitted by breast milk.
T e patient delivers a healthy, 3-kg male in ant via C-section. B) HIV is transmitted by breast milk, but the bene ts o breast-
T e postpartum course is unevent ul. Blood taken rom the eeding outweigh the risk o transmission in this setting.
in ant at day 1 and at 2 weeks both test positive or HIV anti- C) HIV is transmitted by breast milk, but her son will be pro-
bodies. tected rom serious in ection due to maternal antibodies in
the breast milk.
Q estion 9.2.7 What does this mean? D) HIV is transmitted by breast milk, and breast- eeding should
A) T e in ant is in ected with HIV. be avoided i possible.
B) Although technically HIV positive, the in ant’s in ection sta-
tus is unclear rom the in ormation given. Answer 9.2.9 The correct answer is “D.” Postpartum trans-
C) Maternal HIV antibodies are expected to be circulating in mission o HIV rom mother to child occurs in 10% to 14% o
the in ant, but it can be assumed that no transmission o breast- eeding mothers. T is is not a major problem in devel-
in ection took place. oped nations, where there is reliable access to ormula. HIV-
D) A positive test at day 1 is expected due to maternal antibod- positive mothers should be discouraged rom breast eeding
ies, but a repeat positive at 2 weeks indicates in ant antibody in the developed world, including the U.S. T e recommenda-
production and is evidence o in ection. tions may be di erent in developing areas o the world, where
E) None o the above. the mother’s milk may be the only clean source o nutrition
266 FAmIl y mEDICINE ExAmINATIo N & Bo ARD REVIEw

available to the in ant. In act, in many developing nations, the D) Many children show cognitive and motor de cits, but rank
bene t o breast eeding outweighs the risk o HIV transmission; AIDS dementia is uncommon.
ormula eeding can increase mortality due to inadequate access E) Lymphocytic interstitial pneumonitis (LIP) is more com-
to good nutrition, lack o clean water source, and risk o diar- mon in adults than in children.
rheal illnesses. T e World Health Organization (WHO) suggests
that “when replacement eeding is acceptable, easible, a ord- Answer 9.2.11 The correct answer is “D.” wenty- ve percent
able, sustainable and sa e,” bottle- eeding is the best option. o children with HIV in ection demonstrate some cognitive and
motor de cits. T ey ace problems with verbal expression, atten-
Despite the best e orts o your patient and all the physicians tion de cits, hyperactivity, and hyperre exia. LIP is characterized
and nurses participating in her care, her child tests positive by di use reticulonodular in ltrates and hilar lymphadenopathy
by PCR or HIV at 4 weeks and 4 months. Now, you need to and occurs in up to 40% o children with perinatally acquired
think about how to treat this child with HIV. HIV. LIP is very rare in adults. Kaposi sarcoma is associated with
a herpes virus in ection and is very rare in children. oxoplasmo-
Q estion 9.2.10 Which o the ollowin is tr e abo t the sis, usually presenting as ocal mass brain lesions, is a reactivation
se o HAART in children? o previous in ection, and is there ore also very rare in children.
A) reatment should be initiated immediately or all children as Hepatobiliary complications are more common in adults than in
soon as HIV in ection is diagnosed. children, but the reason or this is unclear.
B) Since they are just little adults with relatively big heads com-
pared to their torsos, treatment indications or children and Q estion 9.2.12 What sho ld yo recommend or this in ant
adults are the same. re ardin PCP prophylaxis?
C) HAAR is highly toxic in children and treatment should be A) Prophylaxis is unnecessary because children do not get PCP.
reserved until the child’s li e is in immediate danger rom B) Prophylaxis is unnecessary i the child’s CD4 count is > 200
HIV-related complications. cells/mm 3.
D) No one really knows the best approach, but the treatment is C) Prophylaxis is unnecessary until the child reaches 1 year o
recommended in most children and should be given to all age.
in ants < 12 months. D) Prophylaxis with MP-SMX is contraindicated in in ants
E) reatment indications or children are similar to those or less than 6 months o age due to the risk o hyperbilirubine-
adults, but monotherapy is pre erred over combination mia.
therapy. E) Prophylaxis with MP-SMX as the rst-line agent should be
initiated at 4 weeks o age because the highest risk or PCP
Answer 9.2.10 The correct answer is “D.” Recommendations in children is at 3–6 months o age.
or when to start therapy di er by the age o the child. CD4
counts and HIV RNA viral loads vary by age in children and both
Answer 9.2.12 The correct answer is “E.” T e highest incidence
markers are poor predictors o disease progression and mortality
o PCP occurs in the rst year o li e with peak onset at 3 to
in children < 12 months o age. T e risk o rapid progression o
6 months o age. MP/SMX is still considered rst-line therapy
the disease and mortality is high in in ants, and current guide-
or prophylaxis and should be started at 4–6 weeks o age. CD4
lines recommend initiating therapy or all in ants < 12 months
counts are naturally higher in children and decrease to adult
regardless o their clinical status, CD4 counts, or viral load. For
levels by the age o 6 years. PCP prophylaxis is recommended
children older than 12 months o age, treatment is recommended
or all HIV-in ected in ants < 12 months regardless o CD4
or AIDS or signi cant symptoms re erable to HIV, regardless o
count or percentage. It should be continued until the in ant is
CD4 count. For asymptomatic children > 1 and < 6 years old,
determined to be HIV unin ected or presumptively unin ected.
HIV therapy should be started i the HIV viral RNA is > 100,000
I this drug is not tolerated, dapsone or aerosolized pentamidine
or the CD4 count is < 1,000. HAAR regimens are more dif -
are acceptable alternatives.
cult to choose and adjust in children because o a limited num-
ber o pediatric (non-pill) ormulations and a lack o data about
T e child tolerates the HAAR very well and demonstrates
long-term ef cacy and sa ety. However, combination therapy can
a consistently suppressed viral load. He has urther ollow-
e ectively and sa ely suppress viral load and stimulate immuno-
up by a pediatric in ectious disease specialist. T e mother
logic reconstitution. Monotherapy is NO recommended.
returns to you or urther care. She had been taking the
previously prescribed HAAR , but quit all her medications
Q estion 9.2.11 Which o the ollowin statements re ard
3 months ago because she had orgotten to take a couple o
in the nat ral history o HIV complications in children is
doses and did not want to “screw things up.”
tr e?
Her latest laboratory results show the ollowing:
A) Hepatobiliary complications, such as AIDS cholangiopathy,
are more common in children. CD4 count: 254 cells/mm3
B) Kaposi sarcoma requently occurs in young children. HIV viral load: 50,000 copies/mL
C) Focal brain lesions in children are almost always due to (Previously CD4 count: 692/mm3; HIV viral load: 5,000
toxoplasmosis. copies/mL)
Ch Apt Er 9 • HIV/AIDS 267

Q estion 9.2.13 In addition to enco ra in her to con C) I she has two negative Pap smears 6 months apart and a
tact yo r of ce immediately when she stops or chan es normal CD4 count, she may be screened annually with cer-
medication, which o the ollowin is the most appropriate vical cytology.
advice to provide her now? D) She may be screened per the usual guidelines or HIV-
A) She should have continued to take the medications because negative women.
now the HIV has a “ oothold” and it will be much harder to E) She has almost no chance o developing cervical cancer and
treat. screening may be discontinued.
B) It would not have mattered i she took the medications or
not. Her disease is progressing as expected. Answer 9.2.14 The correct answer is “C.” Human papilloma
C) With the degree o drop seen in her CD4 count, the virus virus (HPV), implicated as a cause o cervical cancer, has a
must be resistant, and urther HAAR is utile. higher prevalence and demonstrates a more aggressive course in
D) She should have continued the medications, even i she was women with HIV in ection. I the patient has a relatively normal
missing doses. A little o ARV activity is better than nothing. CD4 count and has had two normal smears at 6-month intervals
E) She did the right thing by stopping the medications. I a pa- in the rst year o her HIV diagnosis, Pap smears can be done at
tient is not able to comply ully, it is better to not take any 1-year intervals (Note: this is di erent rom the guidelines or
HAAR at all. non–HIV-in ected women). However, i the patient has a CD4
count less than 200 cells/mm 3, screening every 6 months is rec-
Answer 9.2.13 The correct answer is “E.” Starting HAAR is ommended. Any detection o cervical intraepithelial neoplasia
a very dif cult and serious decision and should be made a er is treated the same as with HIV-negative women. HPV testing
care ul and complete counseling. I taken intermittently, her alone is not recommended or ollow-up o an abnormal pap
virus is highly likely to develop resistance to the medications smear in HIV in ected women as there is a high prevalence o
she is taking, and these can easily become ine ective. Patients oncogenic HPV and results may be sub-optimal. Remember
missing doses o HAAR requently are more likely to have that it is not unusual or multiple S Is to be transmitted together,
their HIV disease progress. Once multidrug resistance devel- so consider testing or other S Is when obtaining a Pap smear.
ops, it can be dif cult to nd an e ective regimen to slow dis-
ease progression. HELPFUL TIP:
An STI causing genita u cers (e.g., herpes) increases
A er the drug resistance genotype testing did not identi y the risk trans issi n HIV.
any mutations associated with drug resistance, she agrees to
try Atripla (e avirenz, emtricitabine, teno ovir) one tablet
PO QHS. She does not want any more children and is using Q estion 9.2.15 Aside rom an increased risk o a ressive
e ective contraception, as the e avirenz component in Atri- HPV and a hi h rate o menstr al disorders, how does the
pla is teratogenic. nat ral history o HIV in ection in women di er rom that
At her ollow-up visit, she is slightly drowsy rom the Atri- in men?
pla. Laboratory results are repeated and show an improve- A) Women have a lower rate o progressive multi- ocal leuko-
ment in her viral load: encephalopathy (PML) and bacterial pneumonia.
B) Women are more likely to present with oral thrush and
CD4 count: 261 cells/mm3
recurrent genital candidiasis.
HIV viral load: 7,000 copies/mL.
C) Women with the same level o medical care as men have sig-
You explain that i her viral load continues to all and ni cantly shortened survival.
becomes undetectable, she can expect an improvement in her D) HAAR is more e ective and better tolerated in women.
CD4 count over the next 1 to 2 years, and that the drowsiness E) Most women in the United States acquire HIV rom same-
rom the Atripla should improve over the next ew weeks. sex partners.
You take this of ce visit as an opportunity to catch up on pre-
ventative medicine. On one prior occasion 6 years ago, she Answer 9.2.15 The correct answer is “B.” T e natural history
had an abnormal Pap smear result that returned to normal o HIV in ection in women and men is very similar. Women
a er a repeat examination and colposcopy. Prior to her diag- with the same level o access to medical care have similar sur-
nosis o HIV, she had never been diagnosed with an S I. A vival rates to men, and HAAR is equally e ective (and haz-
pelvic examination today reveals a normal appearing cervix. ardous) in both sexes. Women are more likely to present with
A sample or Pap smear is collected and sent to pathology. recurrent, re ractory vaginal candidiasis, oral thrush, PML, and
bacterial pneumonias. T e majority o women in the United
Q estion 9.2.14 How o ten sho ld this patient et screened States with HIV have acquired it rom heterosexual contact. T e
or cervical cancer with Pap smear? second largest route o exposure or U.S. women is IV drug use.
A) Cervical cancer is inevitable, so you should recommend Females who have sex with other women are at low risk o con-
prophylactic radical resection. tracting HIV; the opposite is true or men who have sex with
B) She must be tested every 6 months, regardless o results o other men. Worldwide, heterosexual contact is by ar the most
this Pap smear. common means o transmission.
268 FAmIl y mEDICINE ExAmINATIo N & Bo ARD REVIEw

Objectives: Did yo learn to . . . bodily uids not considered in ectious: eces, vomitus, sputum,
• Interpret HIV antib d and vira ad tests? tears, sweat, nasal secretions, and saliva. Any visibly bloody
• Eva uate the risk vertica trans issi n HIV? uids, blood, semen, and vaginal secretions are all considered
• Reduce the risk vertica trans issi n HIV? in ectious. Other bodily uids not already mentioned (e.g., CSF
• Interpret HIV tests in the ne nata peri d? and amniotic uid) should be considered potentially in ectious.
• Use HAART in chi dren?
• Identi s e dif erence in the c inica ani estati ns HIV HELPFUL TIP:
in en and chi dren? HIV PEP sh u d be c nsidered r c ntact n nintact
skin r ra r genita uc sa ith p tentia in ected
CASE 9.3 b d uid (see ab ve) r a s urce that is HIV p sitive
r has an unkn n HIV status. PEP sh u d be started
A 39-year-old emale working as a nursing assistant in your ith a three-drug regi en as s n as p ssib e and c n-
hospital comes into the emergency department looking quite tinued r 4 eeks. There are an pti ns r pr ph -
upset. About 30 minutes earlier, she was helping to move a a is ith vari us advantages and disadvantages. The
patient with known HIV when his IV was pulled out and he Pub ic Hea th Service current rec ends ten vir
bled. Several drops o blood got on her hands, but she washed and e tricitabine (Truvada) in additi n t ra tegravir.
them immediately and thoroughly. On examination, she has
intact skin on the hands, no signs o trauma, and no residual
blood on her.
You see the patient back in your clinic 6 months later or rou-
Q estion 9.3.1 The most appropriate action to take is: tine ollow-up.
A) Reassure her that her risk o contracting HIV or this event
is almost zero. Q estion 9.3.3 g iven that yo did not screen her initially,
B) Obtain HIV antibody testing. sho ld she be screened or HIV at this time?
C) Start her on HAAR or prophylaxis. A) No, both o these events were low risk or transmission and
D) Have her return in 6 weeks or HIV antibody testing. there are no recommendations or routine screening.
E) B and D. B) No, both o these events were low risk or transmission and
she does not meet the criteria or risk actor-based testing.
Answer 9.3.1 The correct answer is “A.” Fortunately, HIV is C) Yes, she meets the recommendations or risk actor-based
not the most ef cient virus when it comes to spreading itsel . testing.
Healthcare workers are at risk o contracting HIV when work- D) Yes, everyone should be routinely screened or HIV.
ing with HIV-in ected patients, but exposure to in ected bodily
uid must occur through a percutaneous route or contact with Answer 9.3.3 The correct answer is “D.” In 2013, the United
a mucous membrane or nonintact skin. Even percutaneous States Preventive Services ask Force (USPS F) updated its
exposure (e.g., open bore needle) with HIV-in ected blood car- screening recommendation or HIV, recommending universal
ries a transmission rate o only about 0.3%, and the mucous HIV testing or all individuals between the age o 15 and 65 who
membrane exposure to HIV-in ected blood carries a 0.09% risk. present to any health care setting. T e screening interval is not
T ere ore, contacting an in ected patient’s blood with intact currently well de ned. Repeated screening should be consid-
skin is very low risk. ered in patients known to be at risk or HIV in ection, those
engaged in risky behaviors, and those who live or receive medi-
T e patient is reassured that her risk o contracting HIV is cal care in high-prevalence settings (i.e., HIV seroprevalence o
negligible. She looks a little sheepish when she tells you, “In at least 1%). Depending on their risk, this could be either yearly
all the commotion I knocked over the urinal and spilled the or every 3 to 5 years. According to the CDC, universal screening
patient’s urine on my leg.” She changed clothes and washed may be discontinued i the prevalence o HIV in a population
her leg. You inspect her and nd some eczema in the area has been documented to be less than 0.1% (1/1,000).
where the urine was spilled. Objective: Did yo learn to . . .
• Identi hen a hea th-care rker is at risk r c ntracting
Q estion 9.3.2 Now, yo recommend that she: HIV?
A) Not worry, as the risk o transmission rom this event is • App the USPSTF rec endati ns r r utine HIV screen-
negligible also. ing?
B) Start ZDV or 4 weeks or prophylaxis.
C) Be tested or HIV by viral load.
D) Start HAAR .
CASE 9.4
A 32-year-old male patient comes in to establish care. He
Answer 9.3.2 The correct answer is “A.” Your patient now has does not have any signi cant past medical history and is not
an area o nonintact skin, which is a concern. However, urine currently taking any medications. However, his male partner
is not considered in ectious unless it is grossly bloody. Other is known to be HIV positive and is currently on ARV therapy.
Ch Apt Er 9 • HIV/AIDS 269

He is not sure what his partner’s viral load is but thinks he is (“D”) would be e ective, it would unnecessarily subject the
well-controlled. T ey use protection “most o the time.” He patient to an additional medication (e avirenz) and its associ-
is wondering what he can do to decrease his risk o acquiring ated adverse e ects.
HIV.
When initiating PrEP, discussions regarding an appropriate
Q estion 9.4.1 What sho ld yo tell him? ollow-up plan are imperative.
A) Recommend condom use.
B) Discuss prophylaxis with ARV medications. Q estion 9.4.3 Which o the ollowin is NOT necessary or
C) Discuss sexual risk-reduction counseling. monitorin a patient prior to startin and while contin in
D) A and C. PrEP?
E) All o the above. A) HIV testing at baseline and again every 3 months.
B) Baseline assessment o creatinine and repeat levels every
Answer 9.4.1 The answer in this case is “E”. Since he is in a 6 months.
relationship with a known HIV-positive man, it would be rea- C) Hepatitis B serologies at baseline.
sonable to consider pre-exposure prophylaxis (PrEP). Indica- D) Liver unction tests at baseline and at 3 months.
tions or PrEP go beyond men who have sex with men (MSM) E) All o the above are necessary.
and also encompass high-risk heterosexual adults and IV drug
users. o be eligible or PrEP, the patient must be an adult, Answer 9.4.3 The correct answer is “D.” LF s need not be mon-
not have acute or established HIV in ection, and not be in a itored every 3 months. Prior to initiating PrEP, it is important to
monogamous relationship with a known HIV-negative person. rule out HIV in ection as the 2-drug therapy o ruvada would
For MSM, it is indicated or anyone with any male sex partner be inadequate in treating HIV and could lead to resistance. In
in the past 6 months AND one o the ollowing: any anal sex addition, HIV testing should be repeated every 3 months prior
without condoms in the past 6 months, any sexually transmit- to re lling the prescription. I signs or symptoms o acute HIV
ted in ections in the past 6 months, or in an ongoing sexual occur, an HIV viral load should be considered. eno ovir, one
relationship with an HIV-positive male partner (current recom- o the components o ruvada, has been associated with acute
mendations do not take into account the viral suppression o renal ailure and Fanconi’s syndrome. In the trials, only patients
the HIV-positive partner although studies in IV drug users and whose CrCl was > 60 mL/min were included, and there ore the
heterosexual adults show a 96% reduction o transmission i the current recommendation is to not prescribe PrEP with ruvada
partner’s viral load is suppressed). For heterosexual adults, PrEP or any patient with a CrCl o < 60 mL/min. Once PrEP is initi-
is recommended i condoms are in requently used with a part- ated, creatinine clearance should be rechecked every 6 months.
ner who is known to be at substantial risk or HIV in ection or In addition, hepatitis B serologies should be checked prior to
i the patient is in an ongoing relationship with an HIV-positive initiating PrEP with ruvada. Both components o ruvada have
partner. For IV drug users, PrEP should be considered i the activity against hepatitis B and, i stopped in a patient with active
patient has shared injection or drug preparation equipment in hepatitis B, can lead to reactivation and severe hepatic damage.
the past 6 months.
Objective: Did yo learn to . . .
You decide to start the patient on PrEP based on his risk o • Rec gnize the ef ectiveness PrEP?
acquiring HIV through his in ected partner. • Deter ine hen PrEP is indicated?

Q estion 9.4.2 What medications sho ld be sed?


A) eno ovir.
Clinical Pearls
B) eno ovir/Emtricitabine ( ruvada).
C) E avirenz (Sustiva). A p sitive PPD in patients ith HIV is c nsidered 5
D) eno ovir/Emtricitabine/E avirenz (Atripla). indurati n.
E) None o the above. A pregnant en sh u d be tested r HIV.
Dai ten vir/e tricitabine (Truvada) can be used as pre-
Answer 9.4.2 The correct answer is “B.” T e combination pill e p sure HIV pr ph a is in high-risk patients such as IV drug
o teno ovir and emtricitabine ( ruvada) is the only current abusers and th se h engage in risk se ua behavi r.
FDA-approved regimen or PrEP. In trials in IV drug users or D n t treat patients ith high active anti-retr vira ther-
heterosexually active adults, teno ovir alone has shown ef cacy ap (HAART) i the are unab e t c p ith the treat ent
but is not FDA-approved and has not been adequately studied regi en. missing d ses HAART can ead t resistant strains
in MSM. Any PrEP regimen should be taken daily with strict the virus and re di icu t treating the patient ater in
adherence. Coitally timed or other noncontinuous use is not the disease c urse.
recommended. Pharmacologic PrEP should be done in combi- HIV d es n t see t cause birth de ects, and HAART sh u d
nation with repeated condom provision, counseling on sexual be c ntinued during pregnanc t reduce vertica trans is-
risk reduction, and evaluation and treatment or other sexually si n risk.
transmitted in ections. While the combination pill o Atripla
270 FAmIl y mEDICINE ExAmINATIo N & Bo ARD REVIEw

HIV gen t pe testing r drug resistance sh u d be d ne


Mo enson LM, et al. Guidelines or the prevention and treat-
hen starting r changing a HAART regi en.
ment o opportunistic in ections among HIV-exposed and
HIV-in ected children: recommendations rom CDC, the
Ne nates suspected e p sure t HIV sh u d be screened National Institutes o Health, the HIV Medicine Associa-
ith a PCR r vira ad. tion o the In ectious Diseases Society o America, the
Screening r HIV (n t in the ne nate) sh u d be d ne using Pediatric In ectious Diseases Society, and the American
a P24 antigen and ith an eva uati n Igm and IgG anti- Academy o Pediatrics. MMWR Recomm Rep. 2009;
b dies. This i detect HIV as s n as 2 eeks a ter initia 58(RR-11):1–166.
in ecti n. Panel on Antiretroviral T erapy and Medical Management o
Start HAART ear . In genera , the ear ier the better. HIV-In ected Children. Guidelines or the use o antiret-
roviral agents in pediatric HIV in ection. 2015; pp. 1–268.
The USPSTF suggests that all patients age 15 t 65 be https://aidsin o.nih.gov/content les/lvguidelines/pedi-
screened r HIV. atricguidelines.pd . Accessed August 30, 2015.
Use ster ids in patients ith pneu c stis pneu nia i the Panel on reatment o HIV-In ected Pregnant Women and
Pao 2 is <70 r the A-A gradient is >35. Prevention o Perinatal ransmission. Recommendations
or use o antiretroviral drugs in pregnant HIV-1-in ected
women or maternal health and interventions to re-
duce perinatal HIV transmission in the United States.
2015;14:1–207. https://aidsin o.nih.gov/content les/
BIBLIOg RAPHY lvguidelines/PerinatalGL.pd . Accessed August 31, 2015.
Branson BM, et al. Revised recommendations or HIV testing Panlilio AL, et al. Updated U.S. Public Health Service guide-
o adults, adolescents, and pregnant women in health- lines or the management o occupational exposures to
care settings. MMWR Recomm Rep. 2006;55(RR-14): HIV and recommendations or postexposure prophylaxis.
1–17. MMWR Recomm Rep. 2005;54(RR-9):1–17.
Khalsa AM. Preventive counseling, screening, and therapy in U.S. Preventative Services ask Force. Final Recommendation
the patient with newly diagnosed HIV in ection. Am Fam HIV In ection: Screening. Ann Intern Med. 2013;159(1):
Physician. 2006;73(2):271–280. 51–60.
Endocrinology
Brian X. Shian
10
e sured s ree 4) level. An elev ted SH occurs when the
CASE 10.1 ituit ry detects insu cient thyroid hor one roduction (low
A 27-ye r-old e le resents to the o ce with the chie co - ree thyroxine), nd SH roduction is shut o when the itu-
l int o chronic tigue or 4 onths. She h s g ined 17 lb in it ry detects n excess o thyroid hor one circul ting (elev ted
3 onths, des ite decre sed etite. She lso co l ins o ree thyroxine). With ituit ry dys unction, there is insu cient
de ression, incre sed slee , l ck o energy, h ir loss, nd cold SH roduction resulting in low SH nd decre sed ree thy-
intoler nce. Her st edic l history is unre rk ble, nd roxine ( 4) roduction without eedb ck o n elev ted SH—
she t kes no edic tions. She h s never h d ny surgeries. the ituit ry just doesn’t do its job (such sl cker!). In order
to discern between thyroid gl nd dys unction ( ore co on)
Question 10.1.1 Which o the ollowing physical examina- nd ituit ry dys unction ( ost o en second ry to ituit ry
tion ndings would be expected? deno ), it is necess ry to obt in ree 4 in conjunction with
A) chyc rdi the SH level. Since ituit ry disorders re r re, nd o en sug-
B) Exo hth l os gested by other clues in the history nd hysic l, SH lone is
C) Fine tre or usu lly su cient or initi l screening or thyroid dise se. “C,”
D) Peri her l sensory loss bio sy, is used to ev lu te thyroid sses nd nodules. A
E) Del yed rel x tion in ref exes r dionuclide sc n (“D”) is lso used in the ev lu tion o thyroid
sses nd c n ( ostly) di erenti te unctioning deno s
Answer 10.1.1 The correct answer is “E.” T e history given is ro c rcino s nd benign cysts. “E,” the seru thyroglobulin
consistent with hy othyroid st te. Sy to s o hy othyroid- e sure ent, is used to onitor thyroid c rcino ( nd is not
is include thinning h ir, dry skin, ho rse nd dee voice, the initi l screening).
br dyc rdi , nd rolonged rel x tion in the ref exes. chyc r-
di nd ne tre or re ore ty ic l o hy erthyroidis , nd Her l bor tory results re nor l exce t or glucose 115 g/
exo hth l os is ch r cteristic o Gr ves’ dise se (one s eci c dL, SH 22.3 µ IU/ L (re erence r nge is ty ic lly 0.27–4.20
c use o hy erthyroidis ). Proxi l uscle we kness y occur but y v ry er l b), ree 4 0.56 ng/dL (re erence r nge
in hy othyroidis , but sensory loss is not ty ic l ( lthough hy o- ty ic lly 0.93–1.70).
thyroidis , di betes, gout, rheu toid rthritis, obesity, nd con-
nective tissue disorders c n contribute to c r l tunnel syndro e Question 10.1.3 Based on prevalence, labs, and presenta-
which y be the initi l resenting sy to o these dise ses). tion, what is the MOST likely cause o this patient’s disease?
A) Autoi une hy othyroidis
Question 10.1.2 How can the diagnosis o hypothyroidism B) I trogenic hy othyroidis
best be con rmed? C) uberculosis in ltr tion o the thyroid gl nd
A) Elev ted thyroid-sti ul ting hor one ( SH) level D) Non unctioning ituit ry deno
B) Low SH level E) Congenit l hy othyroidis
C) T yroid bio sy
D) R dionuclide sc n Answer 10.1.3 The correct answer is “A.” Autoi une hy o-
E) Seru thyroglobulin thyroidis (H shi oto thyroiditis) is the ost co on c use
o hy othyroidis in re s where there is dequ te iodine. I
Answer 10.1.2 The correct answer is “A.” T e SH is the ost this tient h d ituit ry deno c using hy othyroid-
sensitive test or both hy o- nd hy erthyroidis , nd ch nges is , the SH ( s well s the ree 4) would be low, since the
in the SH c n recede bnor lities in seru thyroxine (best ituit ry is the source o SH. Congenit l hy othyroidis
271
272 FAMILY MEDICINE EXAMINATION & BOARD REVIEW

c uses severe develo ent l del y nd constell tion o other


signs nd is tested or t birth s rt o routine neon t l screen- You st rt the tient on 25 µ g levothyroxine (Synthroid) nd
ing. uberculosis is r re c use o hy othyroidis , but is the schedule return oint ent in 2 onths. At ollow-u ,
ost co on c use o dren l ilure worldwide. she re orts gener l i rove ent in sy to s but is not
“b ck to nor l.” She re orts continued consti tion, l ck
HELPFUL TIP: o energy, nd eeling de ressed. She h s not lost ny ur-
Outside the United States, iodine de iciency is the most ther weight nd re orts thickening o her h ir. L bor tory
common cause o hypothyroidism, with an estimated results re s ollows: SH 11.8 µ IU/ L (0.27–4.20) nd ree
2 billion persons bein iodine de icient althou h not all 4 0.75 ng/dL (0.93–1.70).
bein hypothyroid.
Question 10.1.5 What adjustments, i any, should be made
Question 10.1.4 How should this patient be managed? to her regimen?
A) I131 d inistr tion A) None, she will continue to i rove t the current dose
B) Surgic l excision o thyroid gl nd B) Incre se the dose to 50 µg/d y, nd re-check in 2 onths
C) St rt synthetic thyroxine hor one t 25 icrogr s (µg) C) Incre se the dose to 200 µg/d y, nd re-check in 2 onths
PO d ily, nd re-check sy to s nd SH in 2 onths D) She is beco ing hy erthyroid, so cut the dose to 12.5 µg/
D) St rt synthetic thyroxine hor one t 200 µg PO d ily, nd d y, nd re-check in 2 onths
re-check sy to s nd SH in 2 onths E) Levothyroxine is ine ective in this tient. Ch nge her to
E) St rt synthetic thyroxine hor one t 25 µg PO d ily, nd desicc ted thyroid tissue (e.g., Ar our T yroid) or triiodo-
double every week until the tient ex eriences weight loss, thyronine ( 3)
tre or, nd oor slee ( nd the desire to nd new doctor)
Answer 10.1.5 The correct answer is “B.” T is tient h s
Answer 10.1.4 The correct answer is “C.” T e tient is de - i roved ro her initi l resent tion, but she is clinic lly nd
cient in thyroid hor one nd needs su le ent tion. wo str t- che ic lly (elev ted SH) still hy othyroid. T e h l -li e o thy-
egies y be used: (1) st rt with 25 µg d ily nd titr te u every roxine is bout 1 week, so she h s h d lenty o ti e to re ch
1 to 2 onths until the SH is in the nor l r nge or (2) st rt ste dy st te. It is doubt ul th t her thyroid levels will ch nge uch
with ull-dose ther y b sed on weight (1.6 µg/kg d ily) nd er 2 onths on this dose, so it is now ti e to incre se her dose.
djust b sed on SH in 1 to 2 onths. Either o tion is ro ri- Since you st rted with low dose, doubling o the dose is re -
te in young, otherwise he lthy dults. However, older tients son ble incre se th t is unlikely to ke her hy erthyroid. T e
(> 65 ye rs old) or those with ulti le co orbidities should go ls o ther y re elior tion o sy to s nd nor liz tion
be st rted t low dose (25 µg d ily) due to incre sed c r- o SH secretion by int ining the SH within the nor l re -
di c risks ssoci ted with being overly re l ced (see hel ul erence r nge exce t in the elderly or during regn ncy. T ere is
ti below). I the tient is titr ted u to 200 µg o levothyroxine evidence o better outco es i the SH is ke t t the high normal
nd does not see to be res onding, the di gnosis needs to be end in the elderly nd the low normal end during regn ncy. “E”
reconsidered or the tient’s co li nce needs to be c re ully deserves s eci l ention. Desicc ted thyroid is gener lly voided
ssessed. Iron nd ood will decre se the bsor tion o levo- due to v ri bility in concentr tion o thyroid hor one content.
thyroxine by s uch s 40%. T yroid edic tion should be In ddition, 3 gener lly h s nothing to o er over levothyrox-
t ken under si il r circu st nces e ch d y: I it is t ken st- ine ( 4) des ite wh t the Internet “ex erts” ( nd your tients’
ing, ne. I it is t ken with e l, this is lso OK but then gr nd others) s y.
it should lw ys be t ken with e l to revent v ri ble
bsor tion d y-to-d y. V ri ble bsor tion c n ke titr t- HELPFUL TIP:
ing the dose di cult. It usu lly t kes 6 to 8 weeks or the body’s Poor compliance is the most common reason or ailure
endocrine res onse nd SH to re ch ste dy st te. T e go l o o medical therapy. Other causes include malabsorp-
ther y is euthyroid st te with the tient ex eriencing neither tion, dru interactions (e. ., ri ampin and amiodarone),
hy er- nor hy othyroid sy to s. “A” nd “B” re tre t ents dru – ood interactions, or dru s that reduce absorp-
o hy erthyroidis . Surgic l excision nd r dio bl tion with tion (e. ., iron and sucral ate).
I131 re ro ches to the tre t ent o Gr ves’ dise se. Both c n
result in i trogenic hy othyroidis th t requires li elong thyroid
hor one ther y. Your tient worries bout t king too uch thyroid hor one.
HELPFUL TIP:
We enerally recommend startin levothyroxine at a Question 10.1.6 Which o the ollowing can result rom
low dose or a couple o reasons. First, titratin the dose oversuppression o the TSH (iatrogenic hyperthyroidism)?
up will assure that you do not overshoot. Second, i pa- A) Ren l ilure
tients are started at 100 µ (a common inal dose), or B) Pul on ry brosis
example, it can cause metabolic stress, especially in the C) Hirsutis
elderly, resultin in an ina, atrial ibrillation, etc. D) Osteo orosis
E) Loss o second ry sex ch r cteristics
CHAPTER 10 • ENDOCRINOLOg Y 273

Answer 10.1.6 The correct answer is “D.” Hy erthyroidis ,


compression symptoms and systemic symptoms like
either i trogenic or endogenous, c uses osteo orosis. For this
ever, wei ht loss, and ni ht sweats. I lymphoma is
re son, it is i ort nt to onitor the SH nd ssure th t the
suspected, a core needle or excisional biopsy is o ten
tient is not overre l ced.
needed to establish the dia nosis.
T e tient h s been doing gre t or cou le o ye rs when
she returns to see you or routine ollow-u . Her SH is
4.3 µ IU/ L nd the ree thyroxine ( 4) is within the nor- Objectives: Did you learn to . . .
l r nge. She then in or s you th t she h d no eriod or • Reco nize the presentation o hypothyroidism and its most
5 weeks. You check urine regn ncy test in the clinic nd it common causes?
is ositive. T e tient is excited bout the regn ncy. • Identi y common physical examination ndin s consistent
with hypothyroidism?
Question 10.1.7 What is the most appropriate manage - • Describe the basic medical and laboratory mana ement o
ment or the patent’s hypothyroidism? patients with hypothyroidism?
A) Continue the current levothyroxine dos ge without ny • Mana e hypothyroidism durin pre nancy?
ch nge
B) Incre se the levothyroxine dose by 10% now
C) Incre se the levothyroxine dose by 30% now CASE 10.2
D) No ch nge o the levothyroxine dose, but h ve the thyroid A 32-ye r-old le resents co l ining o severe nxiety.
unction re-checked in 6 weeks For the l st 4 onths, he h s h d di culty slee ing, rogres-
E) Re er her to n endocrinologist (w it ti e is currently sively worsening nervousness, 25-lb weight loss, nd con-
4 weeks) st ntly eels “too w r .” He eels “sh ky” nd h s di culty
concentr ting. He denies di rrhe , but re orts h ving nor-
Answer 10.1.7 The correct answer is “C.” Wo en with re- lly sh ed stools our to ve ti es er d y— ore th n
existing hy othyroidis who re l nning to beco e regn nt usu l or hi . T e tient denies neck or eye disco ort, nd
should o ti ize their thyroid hor one dose reconce tion i h s not noticed ny neck swelling. He h s no other signi c nt
ossible. T e jority o wo en with re-existing hy othyroid- edic l history. His other, who died 3 ye rs go ro coro-
is require higher dose o levothyroxine during regn ncy n ry rtery dise se (CAD), h d “thyroid roble ,” but he
to int in go l SH levels. T e incre se in levothyroxine doesn’t know ny det ils.
require ents occurs s e rly s the 5th week o gest tion nd Physic l ex in tion reve ls n nxious young dult
l te us by 16 to 20 weeks. It is gener lly reco ended th t le. He h s notice ble resting tre or. You note ild
the levothyroxine dose be incre sed by 30% to 50% t the ti e exo hth l os, conjunctiv l injection, nd lid l g. His thy-
o the ositive regn ncy test through the rst tri ester nd roid is di usely, ildly enl rged nd bruit is udible over
djusted to chieve tri ester-s eci c SH go ls. Note th t the gl nd. T e c rdi c ex in tion reve ls t chyc rdi with
there re tri ester s eci c go ls which di er ro the go l in ow ur ur. T e rest o the ex in tion is unre rk ble.
the usu l tient. T e go l SH level ro or the rst tri ester
is 0.1 to 2.5 µIU/ L; second tri ester go l is 0.2 to 3.0 µIU/ L;
Question 10.2.1 What is the most likely diagnosis?
nd third tri ester go l is 0.3 to 3.0 µIU/ L (note the unit des-
A) Vir l thyroiditis
ign tions nd check with your l b i these di er s so e l bs
B) Gr ves’ dise se
use IU/L). T yroid unction should be ssessed every 4 weeks
C) An l stic thyroid c rcino
to rotect the he lth o the other nd etus nd to void reg-
D) Hy er ctive thyroid deno
n ncy co lic tions. I the SH v lues re bove go l, the dose
E) Surre titious thyroid hor one ingestion
o levothyroxine is incre sed nd thyroid unction is checked
every 2 to 4 weeks. “A”, “B,” nd “D” re incorrect bec use o the
re sons entioned bove. Also, “E” is incorrect. I you do not Answer 10.2.1 The correct answer is “B.” T is is cl ssic re-
eel co ort ble n ging the tient’s hy othyroidis during sent tion o Gr ves’ dise se. T e ily history, the sy to s
her regn ncy, you y still need to incre se her levothyroxine nd signs o hy erthyroidis (es eci lly the di usely enl rged
rst, then re er her to n endocrinologist. Good c re tr nsition goiter with bruit), nd the exo hth l os re ty ic l; conjunc-
is critic l: ssure th t the tient’s hy othyroidis is n ged tiv l injection is lso requently noted. “A” is unlikely. Vir l thy-
until she is under the c re o the consult nt. roiditis c n c use hy erthyroidis nd goiter, but the thyroid
gl nd is usu lly tender. Also, vir l thyroiditis will likely not l st
4 onths, but is usu lly sel -li ited to < 6 weeks. An l stic
c rcino (“C”) is dev st ting dise se with dis l rog-
HELPFUL TIP:
nosis: the thyroid gets very l rge very quickly, but the dise se
Hashimoto thyroiditis is associated with primary thy-
does not resent with hy erthyroidis . A hy er ctive deno
roid lymphoma, which typically presents with an en-
(“D”) nd surre titious ingestion o thyroid hor one (“E”)
lar ed neck mass and may be associated with local
would not c use goiter or exo hth l os.
274 FAMILY MEDICINE EXAMINATION & BOARD REVIEW

or the other (with P U) is gre ter th n the risk o congenit l


HELPFUL TIP:
de ects in the etus (with ethi zole); however, so e ex erts
75% o patients with viral thyroiditis will pro ress
still re er P U throughout regn ncy.
rom 2 weeks o sel -limited hyperthyroidism to 3 to
Pro r nolol is hel ul or controlling the sy to s o hy er-
6 months o hypothyroidism that is also sel -limited.
thyroidis (t chyc rdi , tre or, etc.) nd revents the conver-
The hyperthyroid phase is best treated with NSAIDs,
sion o 4 to ctive 3. “C” is incorrect, s iodine will rovide
beta-blockers, and prednisone i needed.
urther substr te or the body in the roduction o thyroid hor-
one nd should not be given unless thyroid-blocking gent
h s been st rted. Iodine is use ul during thyroid stor to re-
Question 10.2.2 Which o the ollowing tests is most vent the rele se o stored thyroid hor one, but it is given 1 hour
SPECIFIC or Graves’ disease? er P U or ethi zole. R dio bl tion is used or tients
A) Anti-thyrotro in rece tor ( k nti- SH rece tor) ntibody who rove re r ctory to edicine or h ve oor co li nce.
B) Anti-thyroglobulin ntibody T yroidecto y is r rely used or Gr ves’ dise se in current
C) Anti-thyroid eroxid se ntibody edic l r ctice bec use o the e se nd e c cy o r dio ctive
D) M rkedly su ressed SH iodine d inistr tion (exce t in the c se o regn ncy nd ew
other unusu l c ses).
Answer 10.2.2 The correct answer is “A.” Gr ves’ dise se is
n utoi une rocess, nd ly hocytes in the thyroid gl nd Question 10.2.4 Which o the ollowing is/are possible side
itsel re res onsible or l rge ount o the thyroid uto n- e ects o PTU and methimazole therapy?
tibodies roduced. Although sever l ty es o ntibodies c n be A) Gr nulocyto eni
ss yed, nti-thyrotro in rece tor ntibody is the ost s eci c. B) A l stic ne i
Here is bre kdown o the v rious ntibodies nd where you C) Elev ted liver tr ns in ses
y see the elev ted: D) Inhibition o et l thyroid gl nd
• Anti-thyrotro in rece tor ntibody ( nti- SH rece tor E) All o the bove
ntibody) (“A”) is ound in 80% to 95% o tients with
Gr ves’ dise se but lso in 10% to 20% o those with other Answer 10.2.4 The correct answer is “E.” All o the bove re
or s o utoi une thyroiditis. known side e ects o ntithyroid drugs (thio ides). Gr nulo-
cyto eni occurs in bout 0.5% o tients nd is sudden, idio-
• Anti-thyroglobulin ntibodies (“B”) re ound in Gr ves’
syncr tic re ction. Cl ssic lly, tients resent with severe
dise se, utoi une thyroiditis, so e tients with ty e 1
sore thro t. I you h ve tient on P U or ethi zole
di betes nd in u to 20% o the gener l o ul tion.
with sore thro t or ever, check CBC. A l stic ne i y
• Antithyroid eroxid se ntibody (“C”) is elev ted in occur but is r re. For these re sons, tients st rting these ed-
Gr ves’ dise se, utoi une thyroiditis, ty e 1 di betes, nd ic tions should h ve b seline CBC. Mild, tr nsient elev tion
so e regn nt tients, etc. o the liver tr ns in ses is co on, nd the drug should be
discontinued only i the tr ns in ses incre se to gre ter th n
You obt in l bs nd nd th t the CBC is nor l nd SH is three ti es the u er li it o the re erence r nge. Both P U
undetect ble. nd ethi zole cross the l cent nd will inhibit the et l
thyroid, incre sing the risk or congenit l hy othyroidis . T e
Question 10.2.3 How should the patient be treated acutely? risk to etus osed by the drugs is less th n the d nger osed
A) Pro r nolol nd ethi zole st rted i edi tely. by other with cceler ting hy erthyroidis , so the edic -
B) Control the tient’s sy to s with ro r nolol now, then tions should be used even during regn ncy i indic ted. T e
st rt ro ylthiour cil (P U) when the tient eels better. s llest doses ossible should be used. I regn nt tient is
C) Iodine (Lugol solution). not controlled edic lly, consider thyroidecto y.
D) R dio bl tion with I131.
E) T yroidecto y. HELPFUL TIP:
When treatin hyperthyroidism in pre nancy, aim or a
Answer 10.2.3 The correct answer is “A.” T ere is no need to serum ree T4 level in the upper one-third o the re er-
w it be ore st rting ethi zole ( zole), which blocks ro- ence ran e. Avoid radioactive iodine treatment durin
duction o thyroid hor one. P U c n lso be used but ethi - pre nancy since it will a ect the etus as well.
zole is re erred bec use (1) it ords better control nd
(2) P U is ssoci ted with ore liver toxicity nd bone rrow
su ression ( lthough these dverse e ects c n occur with both T e tient is interested in tre ting the o hth l o thy.
drugs). P U is re erred during the rst tri ester o reg-
n ncy due to the risk o birth de ects ( l si cutis nd cho n l Question 10.2.5 What method has been used success ully
nd eso h ge l tresi ) ssoci ted with ethi zole. Current to treat Graves’ disease -related exophthalmos?
guidelines reco end switching ro P U to ethi zole or A) Orbit l r di tion
the second nd third tri esters bec use the risk o liver toxicity B) Glucocorticoids
CHAPTER 10 • ENDOCRINOLOg Y 275

C) Rituxi b Answer 10.2.6 The correct answer is “C.” T e syndro e o


D) T yroidecto y thyroid stor is ch r cterized by ever, t chy rrhyth i s, ltered
E) R dio ctive iodine ther y ent l st tus, nd high out ut c rdi c ilure. It is induced by
jor stress (in ection, surgery, yoc rdi l in rction, etc.)
Answer 10.2.5 The correct answer is “B.” A roxi tely 5% in tient with underlying hy erthyroidis (usu lly undi g-
to 10% o tients with Gr ves’ dise se develo clinic lly sig- nosed). T is tient, lthough clinic lly well-controlled, h d
ni c nt o hth l o thy. T e ri ry go l o n ging Gr ves’ low SH nd high ree 4 rior to surgery, suggesting he y
o hth l o thy is to est blish euthyroid st te. T e edic l h ve su ered recurrence o dise se or h d sto ed t king his
tre t ent or o hth l o thy includes loc l e sures ollowed edic tions. T e tient is not f uid overlo ded, s evidenced by
by tri l o corticosteroids. Glucocorticoids h ve been shown his clinic lly dry st tus (dry ucus e br nes, f t neck veins,
to be e ective in tre ting Gr ves’ o hth l o thy. Intr venous nor l ven c v l lling); thus “A” is incorrect. His ul on ry
(IV) steroids re ore e ective th n or l but either c n be used. ede (r les) is due to high out ut ilure r ther th n f uid over-
Orbit l r di tion h s been used but with little success. T e lo d. NMS resents with ltered ent l st tus nd hy erther i
onoclon l ntibody tre t ent with rituxi b (“C”) or other (due to incre sed et bolic ctivity). But the tient is not cur-
gents h ve h d ixed results nd re still ex eri ent l. R dio- rently t king neurole tics. A ru tured v lve due to endoc rditis
ctive iodine tre t ent (“E”) h s been ssoci ted with worsen- would t the tient’s clinic l icture, but he h d no ever reced-
ing o Gr ves’ o hth l o thy. In tient with Gr ves’ dise se, ing the surgery. For endoc rditis to rogress to v lve ru ture, it
thyroidecto y (“D”) is usu lly reserved or those tients with ust be long-st nding, nd there is nothing in the re-o er tive
tre t ent ilure to ntithyroid drugs, l rge or obstructive goi- ev lu tion or history to suggest this. Euthyroid sick syndro e
ters, nd coexisting hy er r thyroidis who re surgic l c n- (“E”) is used to de ne st te in critic lly ill tients with l bor -
did te or r thyroidecto y. tory ndings o low ree 3, low ree 4, nd/or low SH. So e
ex erts think this is n d tive res onse, which lowers tissue
Your tient co lies with the reco ended regi en nd energy require ents in the ce o syste ic illness. It is reco -
i roves. He is not sy to tic t return visit in 2 onths, ended th t the ter euthyroid sick syndro e be re l ced
nd his eye thology h s not rogressed. Af er 6 onths o with nonthyroid l illness syndro e. Further ore, this tient’s
good control, the tient elects to h ve cos etic eye surgery history o Gr ves’ dise se kes euthyroid sick syndro e n
or his exo hth l os, which h s not res onded well to gluco- unlikely di gnosis. He could h ve lign nt hy erther i sec-
corticoids. A re-o er tive hysic l is unre rk ble. L bor - ond ry to nesthesi but this isn’t choice.
tory studies rior to surgery re nor l exce t SH 0.21 µ IU/
L (0.27–4.20) nd ree 4 3.01 ng/dL (0.93–1.70).
HELPFUL TIP:
Af er n unevent ul surgery, you re c lled to the ost n-
Thyroid storm is a clinical dia nosis consistin o hyper-
esthesi roo by the oculo l stic surgeon. She re orts th t
thermia, tachycardia, CNS dys unction, and si ns and
the tient h s beco e very nxious nd t chyc rdic. A quick
symptoms o peripheral hyperthyroidism. While most
review o the ch rt reve ls no known llergies, no erson l or
patients will have an elevated T3 and ree T4, there is no
ily history o re ctions to nesthesi , nd the only edi-
laboratory level o these hormones that de ines thyroid
c tion is ethi zole ( ro r nolol h d been discontinued
storm. Thyroid storm may cause ri ht upper quadrant
2 onths e rlier due to l ck o sy to s).
pain due to liver con estion secondary to hi h output
Physic l ex in tion shows 39.8°C, BP 98/25 Hg,
con estive heart ailure.
RR 34, nd P 166 b . T e tient is git ted but lert nd in
cute distress. He is t chy neic nd tre ulous nd is un ble
to c rry on convers tion. He see s con used nd distr cted.
HELPFUL TIP:
T e skin is di horetic nd ushed. His ucous e br nes
A pheochromocytoma can be con used with hyper-
re dry, nd his sur ce veins re t. T ere is no JVD nd the
thyroidism. Both include tachycardia and possible hy-
ven c v looks nor lly lled on ultr sound. Pul on ry
pertension. Interestin ly, serum catecholamines are
ex in tion reve ls di use r les. Re exes re brisk nd his
normal (o ten hi h normal—but normal) in thyroid
ent l st tus is s noted bove. T e re inder o the neuro-
storm. Catecholamines will be elevated in patients with
logic l ex in tion is unre rk ble.
a pheochromocytoma.

Question 10.2.6 What is the cause o this patient’s symp -


toms and signs? Question 10.2.7 How should this patient now be treated?
A) T e tient bec e f uid overlo ded during the surgery due A) Ad inister ggressive f uid hydr tion, cooling e sures,
to excessive hydr tion benzodi ze ines, nd d ntrolene
B) T e tient h s neurole tic lign nt syndro e (NMS) B) Ad inister f uid hydr tion, cooling e sures, IV ro r no-
C) T e tient h s thyroid stor induced by the stress o lol, ethi zole, corticosteroids, then iodine 1 hour l ter
surgery C) Ad inister f uid hydr tion, do ine dri or his hy oten-
D) T e tient h s endoc rditis nd su ered v lve ru ture sion, nd ntibiotics; consult c rdiov scul r surgery i e-
E) T e tient h s euthyroid sick syndro e di tely
276 FAMILY MEDICINE EXAMINATION & BOARD REVIEW

D) Hold f uids; d inister nitroglycerin dri nd urose ide C) Re ssure the tient nd h ve her ollow-u or re e t thy-
or his he rt ilure; consider n intr - ortic b lloon u roid studies
E) Ad inister f uid hydr tion (even 3 L or ore) nd ntibiot- D) Begin work-u or centr l hy othyroidis
ics; consider nore ine hrine dri i tient beco es ore E) Begin ethi zole
hy otensive
Answer 10.3.1 The correct answer is “C.” T is tient h s
Answer 10.2.7 The correct answer is “B.” T is is the ro- subclinic l hy othyroidis ( sy to tic, SH between 5
ri te n ge ent or thyroid stor in the correct sequence nd 10, nd nor l ree 4). T ere is convincing evidence th t
o ther ies. T is tient is in distress (hy otensive nd t chy- sy to tic tients with ildly elev ted SH (gener lly
c rdic) nd needs f uid hydr tion, des ite his ul on ry ede . < 10 µIU/ L) nd nor l ree 4 do not bene t ro tre t-
Cooling e sures ddress his hy erther i , nd bet -block de ent in ter s o qu lity o li e, etc. In ct, tre ted tients tend
( ro r nolol) will i rove his high out ut ilure ( nd reduce to h ve ore sy to tic nxiety. Follow-u thyroid studies
the ul on ry ede ). Corticosteroids hel block rele se o re indic ted, lthough the interv l y di er de ending on the
thyroid hor one nd decre ses eri her l conversion o 4 to clinici n’s judg ent. O note, ny tients will h ve tr nsient
3. In ddition, corticosteroids will tre t ny underlying dre- ch nges in their thyroid hor one levels due to co orbid ill-
n l insu ciency. Methi zole revents thyroxine synthesis. ness, vir l thyroiditis, etc. Gener lly, the best thing to do is re-
T is ust be given be ore iodine. T e iodine blocks ny urther check r ther th n st rt tre t ent exce t in those tients with
rele se o thyroid hor one. So e ex erts reco end using high risk or rogression to overt hy othyroidis ( SH level
P U inste d o ethi zole or tient with li e-thre tening > 10 µIU/ L, ositive ily history, goiter, resence o nti-
thyroid stor d itted to n ICU bec use P U blocks 4 to 3 thyroid eroxid se ntibodies or desire to beco e regn nt; see
conversion nd results in lower seru 3 level. “A” is the ro- Clin Endocrinol (Ox ). 2015 M y 23. doi: 10.1111/cen.12824. or
ri te n ge ent o NMS. “C” is n ro ri te ro ch or hy othyroidis clinic l guidelines.)
v lve ru ture ro endoc rditis. “D” describes the n ge ent
o severe he rt ilure. “E” describes essenti l ste s in the tre t- HELPFUL TIP:
ent o se sis. Amon patients with hyperlipidemia, overt and subclin-
ical hypothyroidism are common. Thus, patients with
hypercholesterolemia and hypertri lyceridemia should
HELPFUL TIP: be screened or hypothyroidism.
In patients with hyperthyroidism, diarrhea is a si n that
can presa e thyroid storm.
HELPFUL TIP:
T3 tends to have rapid astrointestinal (g I) absorption
Your tient is tre ted ro ri tely nd eventu lly recovers. and relatively short hal -li e leadin to erratic T3 serum
He continues on ethi zole nd ro r nolol nd re ins levels. It is best to avoid T3 in the vast majority o pa-
in good control. You c n try t ering the edic tions in tients with hypothyroidism.
6 onths to see how he does. T e current evidence su orts
continued tre t ent or 1 to 2 ye rs; re-ex ine the tre t ent
o tions nd go ls t the end o this eriod. Objectives: Did you learn to . . .
• Describe subclinical hypothyroidism?
Objectives: Did you learn to . . . • Di erentiate primary versus central thyroid dys unctions?
• Identi y the more common causes o hyperthyroidism?
• Reco nize the presentation o g raves’disease?
• Mana e hyperthyroidism and describe the indications and QUICK QUIZ: SOMETHINg SUBACUTE
risks o each treatment option?
• Reco nize and treat thyroid storm? A 32-ye r-old white e le without signi c nt st edi-
c l history exce t n u er res ir tory tr ct in ection 4 weeks
go resents to the clinic or 1 week history o nterior neck
CASE 10.3 dull in, l it tions, nxiety, oor slee , nd chills. Physic l
You re seeing 45-ye r-old e le who is nursing ssis- ex in tion is unre rk ble exce t t chyc rdi with nterior
t nt t loc l nursing ho e in your town. As rt o rou- neck tenderness, but no ss reci ted. L bor tory results:
tine blood nel, the tient’s SH is noted to be elev ted t C-re ctive rotein 5 g/dL (high), SH < 0.01 µIU/ L (low),
7.2 µ IU/ L (0.27–4.20 µ IU/ L). She h s nor l ree 4 nd ree 4 is 5 ng/dL (high). A bedside ultr sound shows n
nd is sy to tic. enl rged thyroid gl nd.

Question 10.3.1 The best option is to: Wh t is the ost ro ri te next ste in n ge ent?
A) St rt levothyroxine t low dose to nor lize the SH A) Fine needle s ir tion or bio sy
B) Begin 3 t low dose B) Antithyroid edic tions like ethi zole or P U
CHAPTER 10 • ENDOCRINOLOg Y 277

C) 24-Hour r dio ctive iodine u t ke test D) Phenytoin


D) Neck so tissue co uter-to ogr hy (C ) E) B nd C

The correct answer is “C.” Considering her recent vir l in ec- The correct answer is “E.” A iod rone (see Hel ul i )
tion, tender thyroid gl nd, elev ted inf tory rker nd nd eginter eron l -2 c n c use both hy othyroidis
the low SH nd high 4 this is ost likely sub cute thyroiditis. nd hy erthyroidis . A iod rone c uses hy othyroidis
T e 24-hour r dio ctive iodine u t ke (RAIU) test e sures in roxi tely 10% o tients who t ke it chronic lly nd
thyroid gl nd iodine u t ke over 24 hours. We need to deter- hy erthyroidis in roxi tely 1%. “A” nd “D,” lithiu
ine the etiology o this tient’s thyroiditis. In tients with nd henytoin, res ectively, c use hy othyroidis . Phenytoin
destructive thyroiditis like sub cute thyroiditis, the RAIU will reduces SH secretion so tients y h ve low SH nd
be low, usu lly less th n 5% over 24 hours. In contr st, Gr ves’ low 4.
dise se will h ve n elev ted or nor l RAIU. Fine needle s-
ir tion (“A”) is used or ev lu tion o thyroid nodule but the
HELPFUL TIP:
tient does not h ve one. Antithyroid gents (“B”) h ve no role
Some physicians re ard amiodarone as a “dirty” dru as
in the tre t ent o sub cute thyroiditis since it is usu lly sel -
it can cause photosensitivity, pi mentation o the skin,
li iting dise se er tr nsient (2–8 weeks) hy erthyroidis .
peripheral neuropathy, pulmonary toxicity, and thyroid
T e ri ry tre t ent ocus is sy to s relie nd includes
disease. It has multiple e ects on the thyroid land
NSAIDs, nonselective bet -blockers, nd/or corticosteroids
and on metabolism o T4 and T3, which can result in
( rednisone) i tient h s severe in or no i rove ent is
hypothyroidism. It carries a hu e iodine load, which can
seen er 2 to 3 d ys. Ultr sound is n excellent i ging o-
result in hyperthyroidism.
d lity to ev lu te the thyroid gl nd; where s, C sc n (“D”) o -
ers little v lue in this c se.

QUICK QUIZ: THYROID TESTS CASE 10.4


A 45-ye r-old e le resents to the o ce co l ining o
A 52-ye r-old le is being seen in your clinic or weight loss, “b ll in y neck.” She noticed lu in her nterior neck
t chyc rdi , nd nxiety. You sus ect hy erthyroidis nd roxi tely 1 onth go. She is not sure i it h s incre sed
decide to check blood work. T e tient h s low SH sugges- in size. She h s not noticed ny other lu s or sses. She
tive o hy erthyroidis but h s nor l ree 4. denies dys h gi , odyno h gi , neck in, cough, weight
loss, ever, chills, swe ts, or ch nge in bowel h bits. She
T e ost likely ex l n tion o this tient’s sy to s nd l bo- lso denies ny hy erthyroid or hy othyroid sy to s.
r tory ndings is: In 1993, she i igr ted to the United St tes ro Kiev,
A) He h s ituit ry dys unction with ending centr l hy o- (still in the Ukr ine t the ti e o this writing . . . . . t le st
thyroidis until Putin t kes it), where she h d grown-u nd lived—
B) He h s n isol ted 3 hy erthyroidis including ll o the 80s (you re e ber—M donn , leg
C) He h s Addison dise se w r ers, Chernobyl). She h s no ily history o thyroid or
D) He is t king s irin th t inter eres with the ss y or ree 4 other endocrine disorders, but sever l rel tives in Kiev h ve
E) He is t king s irin th t inter eres with the ss y or SH been di gnosed with v rious c ncers.

The correct answer is “B.” Five ercent o tients with hy er- Question 10.4.1 Which o the ollowing actors rom history
thyroidis h ve n isol ted 3-toxicosis. T us, i you sus ect INCREASES the likelihood that the nodule is malignant?
hy erthyroidis nd the tient h s low SH but nor l A) Nodule not incre sing in size
ree 4, check 3. O note, “D” nd “E” re incorrect. How- B) No region l deno thy
ever, ny drugs c n inter ere with thyroid-binding globulin C) Age > 40 ye rs
nd c use ch nges in the e sur ble ounts o tot l 4 (not D) M le gender
ree) nd 3 in the seru . In ost c ses, the ree 4 nd ree 3 E) History o r di tion ex osure
(unbound thyroid hor ones) re not ected by the ch nges in
thyroid-binding globulin levels. Answer 10.4.1 The correct answer is “E.” T is tient lived in
Kiev, ne r the Chernobyl nucle r ower l nt during nd er
the re ctor ccident in 1986. She w s ex osed to r di tion nd,
QUICK QUIZ: HYPERTHYROIDISM like her ily e bers, is t signi c ntly incre sed risk or
c ncer, es eci lly thyroid lign ncies. Other clinic l ctors
Which o the ollowing c use(s) hyperthyroidis ? th t suggest lign nt etiology include (in gener l, not neces-
A) Lithiu s rily in this tient) nodule > 2 c or incre sing in size, dys-
B) A iod rone h gi , ho rseness, region l ly h deno thy, x tion to the
C) Peginter eron l -2 surrounding tissues, e le gender, ge < 40, nd ily history.
278 FAMILY MEDICINE EXAMINATION & BOARD REVIEW

Question 10.4.3 What should be done next?


HELPFUL TIP:
A) Observe the nodule or 1 onth
There are a lot o cancer survivors walkin around who
B) Co uterized to ogr hy (C ) o the he d nd neck with
received radiation to the neck (e. ., mantle radiation or
contr st
Hod kin lymphoma). These patients are at increased
C) R di tion ther y with I131
risk o developin thyroid cancers.
D) R di tion ther y with extern l be r di tion
E) Su ression o SH levels with levothyroxine

Answer 10.4.3 The correct answer is “B.” Surgery or re ov l


Physic l ex in tion reve ls n obese e le in no cute dis- o the tu or is the ost ro ri te next ste , but C sc n is
tress. Her he d, eye, nose, nd or l ex in tions re nor l. indic ted to deline te the extent o the neo l s . R diother y
She h s 2 c r nodule on the right ole o the thyroid. with I131 is used er surgery or et st tic dise se, but extern l
T e nodule oves long with the other subcut neous struc- be r di tion is not used or thyroid c ncers exce t or lli -
tures u on sw llowing. She h s no ly h deno thy. T e tive ther y o n l stic c rcino . Bec use these tu ors re
rest o the ex in tion is nor l. SH res onsive, su ression o SH level ollowing surgery
or ill ry c rcino is chieved with levothyroxine (usu lly
Question 10.4.2 Which o the ollowing next steps would be 2.2–2.5 µg/kg— irly high dose to co letely su ress SH
most likely to yield a def nitive diagnosis? roduction).
A) T yroid ultr sound
B) Fine needle s ir tion (FNA) HELPFUL TIP:
C) T yroid sc n ( c99) The most common thyroid cancer is papillary carci-
D) Seru thyroglobulin level noma. Medullary carcinoma may be associated with an
E) Seru c lcitonin level nd CEA level elevated calcitonin and other endocrine mali nancies
(e. ., multiple endocrine neoplasia II [MEN II]).
Answer 10.4.2 The correct answer is “B.” FNA c n conclu-
sively rove or dis rove the resence o neo l s nd should be
considered or ll thyroid nodules nd cysts. T t is the botto HELPFUL TIP:
line; the rest re interesting, but do the FNA. There’s a caveat to the emphatic endorsement o FNA
Ultr sound c n be used i there is low lign nt otenti l (which has a deservedly ood reputation or dia nosis).
in order to look or worriso e sonogr hic e tures, but it c n- Here it is: thyroid ollicular lesions. When an FNA returns
not de nitively di erenti te benign nodule ro lign nt material rom a thyroid ollicular neoplasm, 15% to 25%
one. Besides, any thyroid nodule with diameter greater than 1 cm o these may be harborin cancer. Follicular thyroid
should be evaluated with FNA. T us, no ultr sound is needed in cancer cannot be ruled out on the basis o an FNA due
this c se exce t to guide the FNA. to inability to di erentiate a mali nancy rom a beni n
A thyroid sc n will identi y i nodule is ctively rocessing adenoma. Usually, a thyroid lobectomy or total thyroid-
thyroid hor one (“ unction l” or “hot”) or is not et bolic lly ectomy is recommended to urther evaluate a ollicu-
ctive (“non unction l” or “cold”). A “hot” nodule y c use lar neoplasm so that a patholo ic examination can be
hy erthyroidis , is usu lly benign (“hot is not” lign ncy) completed.
nd c n be tre ted with I131. A cold nodule is either n deno
or lign ncy nd bio sy is nd ted. Objectives: Did you learn to . . .
A seru thyroglobulin is the tu or rker or thyroid • Reco nize the major risk actors in uencin the development
c rcino nd should be dr wn be ore thyroidecto y. It has and pro nosis o thyroid carcinomas?
no value as a diagnostic test in the initial evaluation be ore • Evaluate a thyroid nodule?
malignancy is diagnosed, but can be ollowed as a marker. • Describe the mana ement o papillary carcinoma, the most
Antithyroglobulin ntibodies y inter ere with the ss y, common o the thyroid cancers?
however.
Seru c lcitonin nd CEA levels re elev ted in the c se o
edull ry c rcino , but would h ve low yield in the initi l
CASE 10.5
ev lu tion. Ple se re er to the thyroid nodule work-u lgo- A 78-ye r-old e le is brought to the e ergency de rt-
rith in the Revised American T yroid Association Manage- ent (ED) or str nge beh vior. According to her ily, she
ment Guidelines or Patients with T yroid Nodules and Di er- h s been ore slee y nd we k throughout the l st week.
entiated T yroid Cancer t htt ://online.liebert ub.co /doi/ She is cting ore withdr wn nd de ressed nd see s to
d /10.1089/thy.2009.0110. res ond to nonexistent extern l sti uli. When sked bout
her de ression, she re orts she is s d bec use n in
Fine needle s ir tion is consistent with ill ry c rcino . clown suit who tr ins oodles kee s telling her she is going to
A seru thyroglobulin level, SH, b sic che istries, blood die (or ybe she just h s b d shb cks to the circus when
count, nd blood ty e re ordered. she w s younger). She co l ins o bdo in l cr s nd
CHAPTER 10 • ENDOCRINOLOg Y 279

bl es it on “being lugged u ,” nd her ily re orts no ly ho s neither o which our tient h s. “B” is o s eci l
bowel ove ent or 2 d ys. She lso notes chronic bone in note. Chlorth lidone nd other thi zide diuretics incre se c l-
in her hi s nd b ck. T t’s bout ll you c n get be ore the ciu levels nd re contr indic ted.
b d news clown returns . . . In ddition to the bove, di lysis c n be used in hy erc lce-
A hysic l ex in tion is not ble only or 3 c irregul r ic e ergency (gener lly in tients with unres onsive severe
ss in her right bre st (we’re very i ressed th t you did hy erc lce i (e.g., those with ren l ilure).
bre st ex in tion in the ED). T e ily thinks th t the
ss is new. Her neurologic l ex in tion is nor l, exce t T e tient is tre ted ro ri tely or her hy erc lce i
or her rent con usion. nd her ent l st tus i roves. A bio sy o her bre st ss
is done nd reve ls n in ltr ting duct l c rcino . A bone
Question 10.5.1 What nding would you expect rom an sc n reve ls di use et st tic dise se.
ECG?
A) Pe ked -w ves Question 10.5.3 Which o the ollowing statements about
B) Long Q interv l the mechanism or malignancy-associated hypercalcemia
C) Short Q interv l is/are TRUE?
D) ors des de ointes A) Decre sed bone re bsor tion incre ses the seru c lciu
B) Secretion o osteocl st-inhibiting ctors incre ses the
Answer 10.5.1 The correct answer is “C.” T is tient is likely to seru c lciu
h ve hy erc lce i , rob bly ro n undi gnosed et st tic C) Secretion o r thyroid hor one (P H)-like subst nces
bre st c ncer. T e ECG in tient with signi c nt hy erc l- incre ses the seru c lciu
ce i will show short Q interv l; r rely there is di use S D) Direct erosion o bone by tu or cells does not c use hy er-
elev tion (which is ore co only seen in eric rditis). Hy o- c lce i
c lce i is ssoci ted with long Q interv l, which c n occ - E) All o the bove
sion lly le d to rrhyth i s due to n R on heno enon ( n
Answer 10.5.3 The correct answer is “C.”T is is c se o hu or l
R-w ve o PVC uses with the revious -w ve redis osing
hy erc lce i o lign ncy (HHM) nd it incre ses the seru
to tors des de ointes). Hy erc lce i roduces sy to s in
c lciu level by secreting osteocl st- ctiv ting (not inhibiting s
the CNS (con usion, sychosis, de ression), GI syste ( bdo i-
in nswer “B”) ctors le ding to incre sed bone resor tion (thus
n l in, cr s, consti tion), kidneys (ne hrolithi sis, oly-
“A” nd “B” re lse). Secretion o P H-like subst nces c uses
uri , ren l insu ciency), nd usculoskelet l syste (we kness,
retention o c lciu by the kidneys, which elev tes the seru
yo thy, osteo orosis). o hel you re e ber the sy to s
c lciu . Direct erosion o bone by tu or cells lso contributes
o hy erc lce i , use the hr se “stones, bones, o ns, gro ns,
to the rele se o c lciu ro the bones ( nd redis oses the
nd sychi tric overtones.” “A,” e ked -w ves, re ssoci ted
tient to r ctures). T e neo l s s ost co only ssoci ted
with hy erk le i .
with hy erc lce i re c ncers o the bre st, lung, rost te, nd
kidney s well s ulti le yelo nd ew other he tologic
Your tient’s tot l seru c lciu level is re orted s
c ncers.
15.3 g/dL (u er li it 10.4 g/dL).
Question 10.5.4 Your patient’s cancer cannot be cured. How
Question 10.5.2 What is the NEXT step in her treatment?
can her chronically elevated calcium level be managed?
A) Moder tely ggressive nor l s line hydr tion
A) Or l glucocorticoids
B) IV chlorth lidone d inistr tion
B) Or l hos h tes
C) IV c lcitonin d inistr tion.
C) Or l bis hos hon tes
D) IV bis hos hon te in usion
D) All o the bove
E) IV glucocorticoids
Answer 10.5.4 The correct answer is “D.” Glucocorticoids
Answer 10.5.2 The correct answer is “A.” Although “C” nd (e.g., rednisone 20–40 g/d y) decrease intestinal calcium
“D” y be indic ted, dequ te hydr tion nd est blishing urine absorption, but in nd o the selves c n le d to bone density
out ut is critic l to the tre t ent o hy erc lce i nd should loss nd n incre sed risk or r ctures. Or l hos h tes c n
be the next ste . A nor l s line in usion will incre se urin ry decre se intestin l c lciu bsor tion nd bone re bsor tion
c lciu excretion by inhibiting roxi l tubul r sodiu nd o c lciu . Bis hos hon tes, s reviously discussed, decre se
c lciu re bsor tion. Note th t IV urose ide ( nd diuretics the seru c lciu nd incre se bone density.
in gener l) re lling out o vor nd should not be used.
Hydr tion is the ost i ort nt go l. An IV bis hos hon te
in usion (zoledronic cid) should be st rted t the s e ti e HELPFUL TIP:
nd will lower the c lciu level over 2 to 4 d ys. C lcitonin h s g ranulomatous diseases like tuberculosis, sarcoidosis,
li ited dur tion o ction nd c n be used in e ergencies where Crohn disease and leprosy are associated with hypercal-
s line is ine ective. Glucocorticoids re gener lly use ul in cemia because o increased levels o active Vitamin-D.
hy erc lce i second ry to gr nulo tous dise ses nd so e
280 FAMILY MEDICINE EXAMINATION & BOARD REVIEW

Objectives: Did you learn to . . . but re usu lly not ssoci ted with hy erc lce i . An investi-
• Reco nize neoplastic causes o hypercalcemia? g tion or occult c ncer y be in this tient’s uture, but it
• Identi y presentin symptoms o hypercalcemia? is re son ble to ddress the otenti l dverse drug e ect rst.
• Understand the mechanisms underlyin hypercalcemia in Di lysis is not indic ted.
mali nancy?
• Initiate emer ency treatment or symptomatic hypercalce- Being the s rt r ctitioner th t you re, you think th t
mia? the hy erc lce i could be rel ted to his hydrochlorothi -
zide use, so you sk hi to sto it. He returns or l bor -
CASE 10.6 tory dr w 2 weeks l ter. T e c lciu level is now 12.9 g/dL.
“R ts,” you s y. “Where’s y thinking c —I e n iP d?”
A 42-ye r-old le resents to the o ce or routine ollow-
u o his hy ertension. He denies ny co l ints. He h s ri- Question 10.6.3 What should you do next?
ry hy ertension, or which he t kes ben ze ril 20 g PO A) W it nother onth nd re-check be ore t king ny other
d ily nd hydrochlorothi zide 25 g PO d ily. T e tient ction
t kes no other edic tions. His vit l signs re s ollows: B) Order 24-hour bis hos hon te in usion
BP 135/70 Hg, P 72 b , R 18, nd 98.6°F. T e hysic l C) Per or rost te nd testicul r ex in tion or sses,
ex in tion is co letely nor l. His c lciu is elev ted chest x-r y, nd int ct P H (iP H)
t 12.8 g/dL (u er li it nor l 10.4 g/dL). T e rest o D) Per or rost te nd testicul r ex in tion or sses,
his electrolytes, BUN, nd cre tinine re nor l s is CBC chest/ bdo en/ elvis C , nd bone sc n
nd ECG. “H . . .” you think. “T is c lciu level could be E) Per or rost te nd testicul r ex in tion or sses,
lsely elev ted.” chest x-r y, then re ssure tient i nor l

Question 10.6.1 Which o the ollowing is/are responsible Answer 10.6.3 The correct answer is “C.” In this tient, it is
or spuriously elevated serum calcium? re son ble to rule out ri ry hy er r thyroidis while lso
A) Pseudo ot ssiu tr ns ort checking or other otenti l c uses. A chest x-r y will rule out
B) Fever nd ctive et bolic st te signi c nt lung sses nd s rcoidosis. A testicul r nd ros-
C) Use o lendron te or risedron te t te ex in tion or sses is i ort nt so th t l rge tu ors
D) Prolonged lic tion o the tourniquet while dr wing in these org ns do not go unnoticed (re e bering th t
blood digit l rect l ex in tion is neither sensitive nor s eci c s
E) All o the bove screening/di gnostic test or rost te c ncer). An elev ted or
high nor l iP H in the resence o hy erc lce i con r s
Answer 10.6.1 The correct answer is “D.” Prolonged lic - the di gnosis o hy er r thyroidis er ruling out ili l
tion o the tourniquet or high c lciu e l be ore blood hy oc lciuric hy erc lce i . T e test or iP H will not sig-
dr w c n both c use s uriously elev ted seru c lciu . In nif cantly cross-react with the P H-like hormone produced by
ct, u to 50% o tients with hy erc lce i h ve nor l neoplasms. Unless the history or hysic l is sus icious or ly -
c lciu when it is checked second ti e. So, the next ste ho , or so e other occult neo l s , n-body sc n is
er nding elev ted c lciu is to re e t the test. “C” will w ste o oney nd ex oses the tient to unnecess ry r di -
c use low c lciu level. “A” doesn’t exist th t we know o . . . tion. At this oint, it is not necess ry to cutely lower the c l-
sorry. ciu level bec use the c use h s not been identi ed nd the
tient is sy to tic. See ble 10-1 or ore c uses o
Question 10.6.2 Assuming you repeat the laboratory value hy erc lce i .
and the calcium remains elevated, your next step is:
A) A bone sc n or occult c ncer he tient’s co lete ex in tion is nor l s is the
B) I edi te di lysis chest x-r y. he iP H level is twice the u er li it o nor-
C) Nothing, since the tient is sy to tic l nd his urin ry c lciu scores, which should be done
D) Discontinue the hydrochlorothi zide nd re-check c lciu s rt o the work-u o hy erc lce i (24-hour urin ry
in 2 weeks c lciu excretion nd his c lciu -to-cre tinine cle r nce
E) Discontinue the ben ze ril nd re-check ot ssiu in 2 weeks r te) re not suggestive o ili l hy oc lciuric hy erc l-
ce i ( discussion o which is outside the sco e o this
Answer 10.6.2 The correct answer is “D.” T is tient h s book.)
hy erc lce i . Asy to tic hy erc lce i is not unco -
on on routine screening ex in tions. It should be ddressed Question 10.6.4 How should this patient be treated?
bec use it is lw ys bnor l nd c n be tre ted e rly be ore A) I edi te C o the chest, bdo en, nd elvis
ny sy to s develo . In this tient, the ost likely c use is B) IV bis hos hon te in usion
hydrochlorothi zide. T i zide drugs incre se the ren l re b- C) D ily di lysis until the c lciu level is nor l
sor tion o c lciu (re e ber the c se bove?). Angiotensin- D) Re err l or r thyroidecto y
converting enzy e (ACE) inhibitors y c use hy erk le i E) Re err l or thyroidecto y
CHAPTER 10 • ENDOCRINOLOg Y 281

TABLE 10-1 CAUSES OF HYPERCALCEMIA ex in tion is nor l. His neck wound is he ling well, with
ini l erythe nd no tenderness. T e rest o the hysic l
Pseudohypercalcemia
ex in tion is unre rk ble.
Excessive calcium intake
Hypervitaminosis D
Hyperparathyroidism (primary and secondary) Question 10.6.5 What is causing this patient’s symptoms?
Hyperthyroidism A) He su ers ro vit in D de ciency
Mali nancy B) T is tient h s MEN II nd lso h s heochro ocyto
Hypervitaminosis A
C) oo uch r thyroid tissue w s re oved during the sur-
Adrenal insuf ciency
Pheochromocytoma gery
Rhabdomyolysis D) T e stress o the surgery reci it ted thyrotoxicosis
Familial hypocalciuria E) T e stress o the surgery reci it ted the onset o ulti le
Immobilization sclerosis
Medications
• Lithium
• Me estrol Answer 10.6.5 The correct answer is “C.” T is tient likely
• Methyltestosterone h s hy oc lce i due to excessive re ov l o r thyroid
• Mycophenolate gl nd tissue— r re nd un ortun te co lic tion o r thy-
• Tacrolimus roidecto y. It is usu lly detected in the i edi te osto er -
• Tamoxi en
tive course. T e tient’s hysic l ex in tion de onstr tes
• Theophylline (toxicity)
• Thiazides Chvostek sign (t ing over the ci l nerve elicits twitch)
nd rousse u signs (c r o ed l s s er l ce ent o
blood ressure cu ). Vit in D de ciency, lthough c use
Answer 10.6.4 The correct answer is “D.” T is tient h s o hy oc lce i , is unlikely to develo so quickly. T e usu l
ri ry hy er r thyroidis . P tients with sy to tic ri- c uses o vit in D de ciency re l bsor tion or in dequ te
ry hy er r thyroidis (e.g., ne hrolithi sis, sy to - int ke, but there is nothing in the tient’s history to suggest
tic hy erc lce i ) should h ve r thyroid surgery, which is this. T ere is nothing to suggest MEN II (hy er r thyroidis ,
the only de nitive ther y. In sy to tic tients, surgic l edull ry thyroid c rcino , nd heochro ocyto ). T ere
indic tions lso include seru c lciu level gre ter th n is lso nothing here to suggest heochro ocyto (e isodic
1.0 g/dL over the u er li it o the re erence r nge, cre ti- di horesis, l bile blood ressure, recurrent l it tions, nd
nine cle r nce less th n 60 L/ in, bone density -scores less ne r-synco e). T ere is no clinic l evidence o thyrotoxicosis
th n − 2.5 in ny re , revious r gility r cture, nd ge less (t chyc rdi , tre or, etc.).
th n 50 ye rs. In elderly tients with ild hy er r thyroid-
is nd sy to tic hy erc lce i , edic l n ge ent is HELPFUL TIP:
n o tion. I the iP H is low, then e suring the r thyroid Another consideration or a patient who had a recent
hor one-rel ted e tide (P Hr - de by lign ncies) nd parathyroidectomy or primary hyperparathyroidism:
vit in D et bolites is needed to urther work-u the c use. “hun ry bone syndrome” ( reat name, what?), in which
I P Hr is elev ted, bone sc n or body C y be w rr nted. the unmineralized bone matrix produced durin the
Although bis hos hon tes will lower the seru c lciu nd period o hyperparathyroidism be ins to mineralize
y be used in n e ergency, r thyroidecto y is cur tive. a ter the PTH normalized. This results in hypocalcemia
Di lysis is not indic ted in this tient. and hypophosphatemia.

Your tient undergoes r thyroidecto y nd is dis-


ch rged ro the hos it l. T e thology re ort on the HELPFUL TIP:
re oved tissue con r s the resence o r thyroid It is always di icult to remember the MEN types. Here
deno nd no lign ncy. He returns or his rst osto - is a tip or you by Dr. Mark Yo e (who wanted to see his
er tive oint ent 1 week l ter, nd he is co l ining o name in print . . . a shout-out to his mom!).
we kness. He s ys th t the d y f er the surgery he elt ne, MEN 1: the number “1” in MEN1 should remind you o
but h s rogressively gotten we ker “ ll over” since th t ti e. primary or prime number. MEN1 involves thin s that
L st night, he w s ke t w ke by recurrent uscle s s s in start with the letter P:
his legs nd r s. He denies ever, chills, n use , or vo iting. • Pituitary adenoma
He s ys he is e ting, drinking, nd ssing urine nor lly. He • Parathyroid hyperplasia
h s h d no he turi or dysuri . His vit l signs re nor l. • Pancreatic islet cell tumors ( astrinoma, insulinoma,
On hysic l ex in tion, he e rs nxious. Just f er luca onoma)
t king the tient’s vit l signs, his lef r develo s uscle MEN 2A: happens to involve the letter C (This is MEN 2A,
s s nd n involunt ry exion o the wrist th t l sts or so there are two C’s in each item!):
bout 20 to 30 seconds. ing the cheek just nterior to • Calcitonin (medullary carcinoma o the thyroid with
the tr gus c uses the i sil ter l ce to twitch (to your gre t elevated calcitonin level)
delight . . . it is just like in the books!). T e rest o cr ni l nerve
282 FAMILY MEDICINE EXAMINATION & BOARD REVIEW

either would lso r ise the seru c lciu . “A” deserves s eci l
• Calcium (parathyroid hyperplasia, which causes el- ention. IV c lciu glucon te or c lciu chloride c n be used
evated calcium levels) in severe hy oc lce i . But they should not be given by r id
• Catecholamines which are made in the chromocytes IV ush but r ther by slow ush over cou le o inutes.
(as in pheochromocytoma)
MEN 2B: B is or bi (mar anoid habitus) and or belly Objectives: Did you learn to . . .
problems (mucosal neuromas) • Describe the evaluation and treatment o primary hyper-
parathyroidism?
• Reco nize the si ns and symptoms o hypocalcemia and
hypercalcemia?
A c lciu nd lbu in level is sent, s well s CBC nd • Reco nize the causes o hypercalcemia and hypocalcemia?
routine che istry nel including gnesiu . Everything is
• Evaluate and treat hypocalcemia?
nor l exce t: c lciu 5.1 g/dL(8.8–10.4 g/dL) nd lbu-
in 3.0 g/dL (3.4–5.0 g/dL). His hos h te level is 4.2 g/
dL (2.5–4.5 g/dL). CASE 10.7
Question 10.6.6 Does this patient have hypocalcemia? A 36-ye r-old e le resents to the o ce co l ining o
A) No, the seru c lciu level is nor l. di culty losing weight or 2 ye rs. She seeks rescri tion
B) No, the seru c lciu level when corrected or the lbu in drug to id in these e orts. She h s tried every d diet she
is nor l. co es cross, but nothing see s to hel . She tries to exercise
C) Yes, bec use the seru (c lciu × hos h te) roduct is regul rly, but n ges only w lking cou le o iles e ch
> 20. week. A nutrition l history reve ls th t she is e ting sensi-
D) Yes, bec use the seru c lciu level is still low when cor- ble low- t diet (how of en do you see th t!). Her st edic l
rected or lbu in. history includes hy ertension tre ted with edic tions or
E) Not enough in or tion given to deter ine. the l st 3 ye rs nd ty e 2 di betes ellitus (DM2) tre ted or
one ye r. She lso h s been seeing sychi trist over the l st
Answer 10.6.6 The correct answer is “D.” T e tient’s c lciu 6 onths or e otion l l bility, which she bl es on nxiety
level is low, even er correcting the hy o lbu ine i . o cor- over her in bility to get regn nt. T e tient t kes glyburide
rect or lbu in, dd 0.8 g/dL to the seru c lciu level or 5 g PO d ily nd chlorth lidone 25 g PO d ily. A review
e ch 1 g/dL the lbu in is < 4 g/dL. In other words, corrected o syste s reve ls thinning h ir, irregul r enses, del yed
seru c lciu = [(4 – lbu in) × 0.8] + e sured seru c l- wound he ling, nd in ertility (she’s been trying to get reg-
ciu , or in this c se the equ tion is [(4 – 3) × 0.8] + 5.1 = 5.9 g/ n nt or over one ye r).
dL. When ev lu ting hy oc lce i , it is rudent to check BUN
nd cre tinine to rule out ren l ilure s c use ( ro ren l Question 10.7.1 What advice do you o er this patient now?
osteodystro hy . . . see Ch ter 5 or ore in or tion). A) She is e ting right; she just needs to exercise ore.
B) Low- t diets re ine ective; she needs to reduce her c rbo-
hydr te int ke.
HELPFUL TIP:
C) She h s iled li estyle odi c tions, nd etite su res-
I you want to take the easy route and avoid the math,
s nt edic tions re indic ted.
check an ionized calcium. The ionized calcium need not
D) She is likely de ressed, nd needs to continue sychi tric
be corrected or albumin. An ionized calcium will also
ther y nd rob bly should st rt tre t ent with selective
be use ul in patients with monoclonal ammopathy or
serotonin-reu t ke inhibitor (SSRI).
multiple myeloma. Occasionally, these proteins can also
E) Reserve ny diet ry dvice t this ti e, s she rst needs
bind calcium.
edic l work-u .

Question 10.6.7 How should this patient now be treated? Answer 10.7.1 The correct answer is “E.” T e tient’s sy -
A) C lciu glucon te 1 g by r id IV ush. to s o weight g in nd ssoci ted ndings on review o sys-
B) Correct his hy o gnese i with IV MgSO4. te s (e otion lly l bile, thinning h ir, in ertility, irregul r
C) Ad inister IV nor l s line 1 L bolus. enses, nd del yed wound he ling) suggest second ry c use,
D) Initi te c lciu c rbon te 1 to 4 g with vit in D or lly ost likely n endocrine bnor lity. (Wh t else would you
divided in 2 to 3 doses d ily. ex ect? T is is the endocrine ch ter er ll . . .)
E) No ther y, s the c lciu level will correct itsel .
Her vit ls in the o ce: P 88 b , BP 155/94 Hg, R 20,
Answer 10.6.7 The correct answer is “D.” T e tient now nd 37.7°C. On hysic l ex in tion, you note n obese
requires or l c lciu su le ent tion, usu lly with vit in D e le with trunc l obesity nd thin extre ities. She h s
to sti ul te bsor tion. He will likely require it li elong. T is thinning h ir, round cies, hirsutis , nd bu lo hu t
tient h s neither hy o gnese i nor hy er hos h te i , her u er b ck. Her skin is hy er ig ented with bdo in l
nor ny other electrolyte bnor lities. I he h d, correcting stri e. T e rest o the ex in tion is unre rk ble.
CHAPTER 10 • ENDOCRINOLOg Y 283

Question 10.7.2 Based on this patient’s history and physical Question 10.7.4 What is the best next step in this patient’s
examination, what diagnosis is most likely? evaluation?
A) Hy othyroidis . A) Ultr sound o kidneys to ssess or sses nd dren l f ow.
B) Hy erthyroidis . B) C sc n o the br in to rule out et st tic lesions nd ssess
C) Cortisol excess second ry to chronic steroid ther y. ituit ry size.
D) Cortisol excess second ry to n endogenous rocess. C) C sc n o the dren ls to ev lu te or dren l neo l s s.
E) Cortisol de ciency second ry to utoi une dren l in- D) MRI o the ncre s to ev lu te or neo l s s.
su ciency. E) MRI o the ituit ry gl nd to ev lu te or neo l s .

Answer 10.7.2 The correct answer is “D.” T is tient h s the Answer 10.7.4 The correct answer is “E.” T e revious studies
cl ssic sy to s nd signs o Cushing syndrome, or cortisol strongly suggest Cushing syndro e nd the visu l led cut sug-
excess. T is y be due to corticosteroid ther y (the ost gests ituit ry source o excess AC H (e.g., AC H- roducing
co on c use), ecto ic AC H roduction ro neo l s s tu or o the ituit ry gl nd). An MRI o the ituit ry is the best
(lung, ncre s, kidney, etc.), dren l neo l s s roduc- e ns to con r this. C sc n y suggest enl rge ent o the
ing cortisol, or AC H- roduction ro ituit ry neo l s sell turcic (where the ituit ry gl nd sits), but it is insensitive or
(ter ed Cushing disease). Her condition is unlikely due to ste- detecting bnor lities in the ituit ry, es eci lly icro deno s.
roid ther y, s this should h ve been reve led in her edic l
history. At this oint in the ev lu tion, it is not cle r wh t the HELPFUL TIP:
source o AC H is, just th t there is excess AC H being ro- I the MRI is ne ative, then the more rare case o ectopic
duced. Note the hy er ig ent tion o this tient’s skin. T is ACTH production rom occult cancer must be consid-
is the s e ech nis s in dren l destruction ( n elev ted ered, and a body CT scan is indicated.
AC H); the ituit ry is u ing out AC H, which sti ul tes
el nocytes.
Objectives: Did you learn to . . .
L bs re sent, ost o which re nor l exce t or n ele- • Reco nize the common presentin si ns and symptoms o
v ted glucose o 210 g/dL nd n elev ted 24-hour urin ry Cushin syndrome?
ree cortisol o 115 µ g (nor l < 100 µ g). B sed on these • Di erentiate Cushin syndrome rom Cushin disease?
ndings, dex eth sone su ression test is ordered. T e • Identi y the causes o cortisol excess and how they are best
tient is given huge (g rg ntu n, w y big), su r ther eu- evaluated?
tic dex eth sone doses or 2 d ys, nd then seru AC H
is dr wn, nd nother 24-hour urine collection is done. T e CASE 10.8
re e t seru AC H is still slightly elev ted nd 24-hour uri-
JFK is 38-ye r-old le resenting to the o ce with the
n ry ree cortisol is 78 µ g (nor l).
chie co l int o we kness. He re orts th t he l cks the
energy to co lete his reviously busy work schedule (such
Question 10.7.3 What is the source o this patient’s cortisol s inv ding Cub ) nd h s cut his hours b ck the l st 2 to
excess? 3 weeks, which h s recently beco e n nci l h rdshi . He
A) An AC H- roducing tu or. lso re orts oor etite nd 20-lb weight loss over the
B) A cortisol- roducing dren l tu or. l st onth. He h s h d ltern ting eriods o di rrhe nd
C) Surre titious use o or l steroids. consti tion. He is of en eeling int nd h s co e ne r to
D) Not enough in or tion to deter ine. ssing out f er st nding u quickly ew ti es; this is di -
erent or hi ro b seline. He h s no revious edic l
Answer 10.7.3 The correct answer is “A.” When the tient or surgic l history. He denies edic tions or llergies. He
w s given exogenous steroids during the test, there w s r- re orts th t sever l o his rel tives on his other’s side h ve
ti l su ression o cortisol roduction ( decre se in 24-hour h d thyroid roble s. “I c n’t go on like this,” he co l ins
urin ry ree cortisol). But ore i ort ntly, the seru AC H in n ccent th t betr ys his Boston roots.
level re ined bove nor l des ite the high steroid lo d. T e On hysic l ex in tion, his vit ls reve l BP 85/38 Hg,
AC H level should h ve been low in the resence o dex - P 85 b , R 20, nd 37°C. T e ex in tion shows hy er-
eth sone, or ny exogenous steroid, which would ct to shut ig ent tion o bucc l ucos nd we kness in the roxi l
o AC H roduction in the nor l tient. I AC H is still uscul ture (5–/5 strength in both u er nd lower extre i-
being roduced des ite high steroid lo d, then there ust be ties). His skin is hy er ig ented t the elbows, knuckles,
n AC H- roducing neo l s so ewhere in the body (either knees, nd the l r cre ses. T ere is no ede .
ecto ic roduction ro lung, ren l, or ncre tic c ncer, or
n AC H- roducing ituit ry tu or th t h s esc ed nor l Question 10.8.1 What is the most likely cause o this
regul tory eedb ck ech nis s). patient’s symptoms?
A) Hy erthyroidis
On hysic l ex in tion, the tient h s visu l eld cut. B) Hy othyroidis
284 FAMILY MEDICINE EXAMINATION & BOARD REVIEW

C) Adren l gl nd hy er ctivity
HELPFUL TIP:
D) Adren l insu ciency
An early mornin serum cortisol level is o ten low in
E) De ression
patients with adrenal insu iciency. However, this test
alone is insu icient or screenin , as it has a hi h alse
Answer 10.8.1 The correct answer is “D.” Sever l clues oint
ne ative rate. But a low (<5 µ / dL) early mornin serum
you in this direction, side ro the tient’s n e. First, the
cortisol is likely to be due to adrenal insu iciency.
hy er ig ent tion ound t stress/cre se oints on the skin
suggests the di gnosis o dren l insu ciency. T e low blood
ressure nd orthost sis is lso ore likely to be seen in dren l
insu ciency. HELPFUL TIP:
T is tient h s nu ber o sy to s consistent with both Interpretin the cosyntropin stimulation test is not
de ression nd hy othyroidis (“B” nd “E”), but his hysic l always strai ht orward. The current criteria used to
ex in tion suggests nother di gnosis. A tient with de res- indicate normal adrenal unction are a minimum se-
sion nd no underlying edic l c use would h ve nor l rum cortisol concentration ≥ 18 to 20 mc /dL (500–
hysic l ex in tion. A tient with hy othyroidis y h ve 550 nmol/L) BEFORE OR AFTER corticotropin injection.
slightly low blood ressure, but not rkedly low s in this An al orithm is available at http://www.arupconsult.
tient. T e he rt r te in sy to tic hy othyroidis is usu lly com/Al orithms/AdrenalInsu iciency.pd
low. Hy othyroidis y lso c use ch nges in the h ir, skin,
nd thyroid gl nd th t re not seen here. Fin lly, the tient’s
ref exes re nor l, r ther th n del yed. T ere is nothing in this
c se to suggest hy erthyroidis or cortisol excess (“A” or “C”). Further l bor tory results re s ollows:
R ndo seru cortisol: 4 µg/dL (nor l 20 µg/dL).
L bor tory results show low sodiu (129 g/dL), low glu- Seru cortisol 1 hour er 0.25 g cosyntro in IV:
cose (69 g/dL), nd high ot ssiu (5.4 g/dL) long with 4.5 µg/dL (low).
nor l SH nd ildly low he oglobin.
Question 10.8.3 Based on the prevalence in the United
Question 10.8.2 Based on the in ormation provided thus States, what is the most likely underlying cause o this
ar, what is the best test to diagnose this patient’s condi- patient’s condition?
tion? A) Autoi une dise se.
A) Cosyntro in sti ul tion test. B) Inv sive c rcino .
B) R ndo seru cortisol. C) Meningococc l se tice i .
C) Bone rrow bio sy. D) S rcoidosis.
D) Pl s renin. E) uberculosis.
E) Ultr sound o kidneys to e sure their size.
Answer 10.8.3 The correct answer is “A.” All the conditions
Answer 10.8.2 The correct answer is “A.” T e l bor tory che - listed re known c uses o primary dren l insu ciency, nd
istry results (hy erk le i , hy on tre i , nd hy oglyce i ), ll c use this disorder by destruction o the dren l gl nds.
co bined with the history nd hysic l ex in tion, strongly However, in the United St tes the ost co on c use is uto-
suggest iner locorticoid de ciency. A cosyntro in sti ul - i une destruction o the dren l gl nds; s noted bove,
tion test uses synthetic AC H to try to induce burst o cortisol tuberculosis is the ost co on c use worldwide. Adren l
secretion. No dequ te incre se in the seru cortisol in res onse destruction will result in l ck o dren l hor one secretion
to the cosyntro in suggests th t the dren l gl nds re un ble nd blunted res onse o the dren ls to AC H—hence, the
to res ond to the body’s iner locorticoid nd glucocorticoid ini l rise o cortisol des ite d inistr tion o synthetic
needs. A ositive cosyntro in test kes the di gnosis o dre- AC H (cosyntro in). Cortisol level er cosyntro in sti u-
n l insu ciency. A r ndo seru cortisol is de nitely second l tion test should be higher th n 18 to 20 cg/dL (500 to
best, s cortisol levels nor lly f uctu te widely throughout the 550 n ol/L) in tients with nor l dren l unction.
diurn l cycle. Pl s renin is used in the ev lu tion o iner-
locorticoid excess (hy er ldosteronis ) nd should be low in Without e suring the seru AC H or 24-hour urine
dren l insu ciency second ry to dren l dys unction ( ldo- AC H level, it is still ossible to tell the di erence between
sterone c uses sodiu re bsor tion nd ot ssiu excretion, ri ry (l ck o dren l res onse to AC H) nd second ry
the ex ct o osite seen in this tient who’s dren ls don’t ke ( ituit ry l ck o AC H) dren l insu ciency.
ldosterone). “E” is not likely to be hel ul. Even in the c se o
dren l insu ciency, the kidneys re unlikely to ch nge in size. Question 10.8.4 Which o the ollowing suggests primary
Although n ultr sound y show s ll, shrunken dren ls adrenal insu ciency (e.g., adrenal destruction) rather than
th t h ve been destroyed by tuberculosis, C sc n is better secondary?
tool or ssessing the dren ls. “C” is w y o b se. T is tient A) B seline seru cortisol level > 7 µg/dL
h s no indic tion or bone rrow bio sy. B) Presence o hy er ig ent tion on hysic l ex in tion
CHAPTER 10 • ENDOCRINOLOg Y 285

C) Presence o neuro thy on hysic l ex in tion ct eels well enough to t ke on both C stro and Khrush-
D) Predo in nt sy to s o de ression chev (loc l co etitors to his dry-cle ning business). He no
longer su ers s ells o lighthe dedness or de ression. o
Answer 10.8.4 The correct answer is “B.” Hy er ig ent tion celebr te his new good he lth, he nd blond st rlet go out
t skin cre ses occurs in ri ry dren l insu ciency (but not or l vish se ood dinner, including resh oysters. Within
second ry dren l insu ciency). T is is bec use in ri ry 8 hours o the e l, JFK develo s severe cr ing bdo in l
dren l insu ciency, the ituit ry is int ct. As result, the in nd ro use w tery di rrhe (so uch or the d te . . .).
AC H level is high, s is the level o el nocyte-sti ul ting He tries to tre t hi sel t ho e with Pe to–Bis ol nd or l
hor one. It is this el nocyte-sti ul ting hor one th t c uses uids, but gets rogressively we ker. Af er 12 hours o intes-
hy er- ig ent tion. An e rly orning b seline seru cortisol tin l sy to s, he c lls or n bul nce bec use he is too
level o 5 cg/dL or bove (“A”) would suggest ri ry dren l we k to st nd.
insu ciency unlikely (the dren ls re still king cortisol). U on resent tion in the ED, his vit l signs re: P 130 b ,
Hy oglyce i nd de ression (“D”) re sy to s co on to BP 70/20 Hg, R 30, nd 38°C. He is di horetic, ill-
ll c uses o dren l insu ciency so don’t di erenti te between e ring, nd in severe distress.
ri ry dren l ilure nd second ry c uses o dren l insu -
ciency. Neuro thy (“C”) is not co on nding in dren l Question 10.8.6 A ter establishing adequate IV access,
insu ciency. what should be done next?
A) Order chest x-r y.
HELPFUL TIP: B) St rt “ren l-dose” do ine through eri her l line.
Patients with primary adrenal insu iciency may pres- C) Give 2 L nor l s line by bolus.
ent with low serum sodium and hi h serum potassium D) St rt nor l s line t 125 cc/hr.
(althou h many will have normal electrolytes). This is E) Give levof ox cin 500 g IV.
primarily because o loss o the aldosterone system.
Patients with secondary adrenal insu iciency (e. ., pi- Answer 10.8.6 The correct answer is “C.” T is tient is
tuitary cause) have intact adrenal lands and there ore severely volu e de leted nd needs cryst lloid i edi tely.
intact aldosterone production. Thus, they will enerally All other tre t ent concerns, lthough they y eventu lly be
have normal electrolytes and less dehydration, hypo- done, re second ry.
tension, etc.
Des ite uid boluses, he re ins hy otensive. You scr tch
your he d nd onder wh t you y h ve orgotten.
Question 10.8.5 How should this patient now be treated?
A) Prednisone 60 g PO or 5 d ys, then slowly t ered over Question 10.8.7 What should have been done simultane -
2 weeks. ously or this patient when the f uids were started?
B) Cosyntro in 0.5 g SC d ily inde nitely. A) Intub tion by r id sequence nd ech nic l ventil tion.
C) Corticosteroids ( rednisone 5 g or hydrocortisone 15 g) B) Ad inistr tion o henyle hrine dri .
d ily inde nitely. C) Ad inistr tion o 4 g dex eth sone IV.
D) Corticosteroids ( rednisone 5 g or hydrocortisone 15 g) D) Ad inistr tion o 100 g hydrocortisone IV.
d ily lus iner locorticoid (f udrocortisone 0.1 g) d ily E) C or D.
inde nitely.
E) Adren l tr ns l nt. Answer 10.8.7 The correct answer is “E.” T is tient h s
dren l insu ciency nd requires ddition l “stress doses” o
Answer 10.8.5 The correct answer is “D.” T is tient requires steroids in ti es o severe hysic l stress (e.g., in ection, tr u ,
chronic corticosteroid su le ent tion, nd bec use he h s ri- nd yoc rdi l ische i ). T e steroids he regul rly t kes or his
ry dren l insu ciency, he lso requires iner locorticoid dise se y not be su cient during these eriods, reci it ting
su le ent tion. Usu lly, iner locorticoid d inistr tion is ddisoni n crisis. Without this ddition l tre t ent, the tient
not necess ry in the cute setting like dren l crisis but the v st y ex erience intr ct ble hy otension nd ossibly de th. In
jority o tients with ri ry dren l insu ciency eventu- less cute situ tion in which the tient h s inor illness, two
lly require iner locorticoid re l ce ent with f udrocorti- to three ti es the usu l inten nce glucocorticoid dose or
sone. Cosyntro in tre t ent (“B”) would h ve no e ect since 3 d ys (known s the “3 × 3 rule”) y bene t the tient.
the dren ls re not unctioning. A short burst o rednisone
(“A”) is i ort nt i this tient ex eriences sudden stressor, HELPFUL TIP:
such s n in ection, but it is not the ri ry ther y. Medic l I you suspect adrenal insu iciency crisis (addisonian
ther y is irly e ective, so dren l (or ren l) tr ns l nt (“E”) crisis), start steroids immediately even i you do not
is not usu l tre t ent o tion. have laboratory con irmation. Dexamethasone is an
option or treatment, and it will not inter ere with the
JFK is tre ted ro ri tely nd, f er 4 weeks, is eeling cortisol assay when doin a cosyntropin stimulation
uch better, h s reg ined ost o his lost weight nd in
286 FAMILY MEDICINE EXAMINATION & BOARD REVIEW

• Understand secondary adrenal insuf ciency due to exo -


test. But note that dexamethasone does not have enous steroid use?
mineralocorticoid activity and should be replaced
by hydrocortisone succinate just as soon as the
cosyntropin stimulation test is done. QUICK QUIZ: DIABETES DIAg NOSIS

Which o the ollowing c n be used to di gnose DM?


A) F sting blood glucose on one occ sion.
T e next d y when you re doing your in tient rounding, B) Glycosyl ted he oglobin (HbA1c) on one occ sion.
you eet your tient’s cousin Kenneth, who is 59-ye r- C) A r ndo blood sug r o > 160 g/dL.
old white le with st edic l history o hy ertension, D) None o the bove.
COPD, lcohol buse, nd osteo rthritis in his shoulders nd
knees. A rticul rly enthusi stic colle gue injected 80 g The correct answer is “D.” Here is why the di gnosis o DM
o tri cinolone in both o the tient’s knees nd shoulders requires two elev ted sting blood sug rs o ≥ 126 g/dL or
sever l onths go (tot l dose o 320 g). He h d just gone two HbA1c levels o ≥ 6.5%. It c n lso be di gnosed i the
to joint injection worksho nd gured wh t the heck, lets tient h s single r ndo blood sug r o > 200 g/dL with
r ctice! T e tient notes we kness, lighthe dedness, n u- signs nd sy to s o DM ( olyuri , olydi si , weight loss,
se , nd decre sed etite recently. His ho e edic tions nd blurred vision). o be co lete, 2-hour ost r ndi l glu-
re unre rk ble. Vit ls re BP 70/44 Hg, HR 60 b , cose toler nce test result o > 200 er 1.75 g/kg o glucose ( x
35.6°C, oxygen s tur tion 98%. Physic l ex in tion shows dose 75 g) is di gnostic o DM. However, this is r rely used in
ildly distressed tient with ild ex ir tory wheezing on the United St tes.
uscult tion nd unste dy g it when he tries to st nd u .
L bs re unre rk ble exce t or sodiu o 123 Eq/L
(low), nor l ot ssiu , high BUN t 30 g/dL, cre tinine QUICK QUIZ: A1c IN SPECIAL CONDITIONS
o 2.1 g/dL ( tient’s b seline is 0.6 g/dL), Cl 84 Eq/L,
nd CO2 26 Eq/L. CBC, LF s, tot l rotein, nd lbu in Which o the ollowing conditions rti ci lly raises the v lue o
re nor l. A tro onin, SH, urine drug screen, lcohol he oglobin A1c?
level, nd urin lysis re lso nor l. A) Iron de ciency ne i .
He receives cou le o liters o nor l s line, nd his blood B) Sickle cell dise se.
ressure nor lizes. A re e t l b shows sodiu 130 Eq/dL C) Chronic liver dise se.
nd cre tinine 1.1 g/dL. One o your resident hysici ns D) Acute blood loss ne i .
decides to urther worku the hy on tre i nd checks the E) All o the bove.
cortisol level, which returns 1.5 µ g/dL (low) with AC H t
18 g/ L (nor l 7–63 g/ L). A cosyntro in sti ul tion The correct answer is “A.” Iron de ciency y lsely elev te
test is done nd the seru cortisol level goes u to 14.9 µ g/dL. the HbA1c. Sever l other conditions c n c use lsely low
HbA1c including sickle cell dise se, l ri , cute blood loss,
Question 10.8.8 What is the most likely cause o the nd he olysis (see ttern develo ing here?). T is kes
patient’s hyponatremia? sense: in ll these conditions the bone rrow is u ing out
A) Second ry dren l insu ciency due to nhy o ituit ris new cells th t h ve not been ex osed to the elev ted glucose
B) Second ry dren l insu ciency due to exogenous steroid levels. Conversely, iron de ciency nd vit in 12 nd ol te
use de ciencies y be ssoci ted with lsely high HbA1c levels;
C) COPD ex cerb tion the ro ortion o old cells in the blood is high co red to the
D) Hy othyroidis nor l individu l. Chronic kidney dise se y c use either
lsely elev ted or lsely low HbA1c.
Answer 10.8.8 The correct answer is “B.” T e tient received
ridiculous doses o intr rticul r tri cinolone sever l onths CASE 10.9
go c using dren l su ression. “A” is incorrect bec use the
You re c lled to the ED to see 17-ye r-old n brought
initi l AC H is nor l, which is not the ty ic l nding or cen-
in by his rents who ound hi in his roo t ho e. He
tr l c uses o dren l insu ciency. “C,” COPD ex cerb tion, is
is leth rgic, but not unconscious or co tose. He is un ble
not consistent with the dren l unction tests. “D,” hy othyroid-
to give coherent history. His rents st te th t they h ve
is , c n c use tigue nd hy on tre i , but the tient’s SH
been concerned bec use the tient h s been losing weight
is within the nor l li its.
in the l st 2 onths nd cting ore tired th n usu l. T ey
Objectives: Did you learn to . . . re worried th t he ight be busing drugs, but h ve not
• Reco nize the presentation o adrenal insuf ciency? ound ny drugs or drug r hern li in the ho e. T ey
• Evaluate a patient with adrenal insuf ciency? h ve observed no other signs o illness. T e ily history is
• Treat a patient chronically or adrenal insuf ciency and when ositive or hy ertension in ulti le ily e bers, nd
in adrenal crisis? the tient’s other h s hy erli ide i . T ere is no ily
CHAPTER 10 • ENDOCRINOLOg Y 287

history o kidney or liver dise se, he rt tt cks, strokes,


HELPFUL TIP:
di betes, or c ncer. His vit ls re: 36.9°C, P 125 b , BP
Patients with severe hyper lycemia (like this uy) will
98/54 Hg, res ir tions re dee with r te o 28, S O2
have hyponatremia secondary to the osmotic e ect
95%on roo ir. He is leth rgic nd non-verb l but rouses
o lucose pullin water into the intravascular space,
to in. His ucous e br nes re dry with ssured
increasin the plasma water content. What is the cor-
tongue. You s ell ruity ro on his bre th th t re inds
rected sodium or this patient? It is approximately
you o th t bowl o ruity cere l you lef t ho e. His bre th-
140 mEq/dLbased on the ormula: corrected sodium =
ing is st nd he is t chyc rdic. His bdo in l ex in tion
measured sodium + [1.6 (glucose − 100)/100]. This is
reve ls sof bdo en with ild, gener lized tenderness. No
very approximate. I the lucose is > 400 m /dL, use “2”
rebound or gu rding.
rather than 1.6 or the correction. As an aside, the bowl
EKG shows sinus t chyc rdi . CBC shows Hgb 17.9 g/dL,
3 o ruity cereal the provider le t at home? All o the
WBC 16,200 cells/ with neutro hil redo in nce
3 colors o Fruit Loops taste exactly the same. Another
nd l telets 650,000 cells/ . Electrolytes show sodiu
bubble broken . . . . . .
131 Eq/L (slightly low), ot ssiu 5.7 Eq/L (high), chlo-
ride 97 Eq/L, bic rbon te 10 Eq/L (low), BUN 63 g/dL
(high), cre tinine 1.8 g/dL(b seline 0.7 g/dL), nd glucose
635 g/dL (high). Seru ketones re ositive. You re con- HELPFUL TIP:
cerned bout his res ir tory st tus so you obt in n ABG t Urine ketones are >99% sensitive or DKA in the ri ht
roo ir with H 7.20, P CO2 27 Hg, P O2 101 Hg, circumstance. Thus, checkin serum ketones is super-
nd bic rbon te 10 Eq/L. luous in most cases.
You di gnose di betic keto cidosis (DKA) with dehydr -
tion > 10% nd d it the tient to the intensive c re unit.
Question 10.9.2 What is the most appropriate initial insulin
As the rst st ge o ther y you wish to re l ce the lost uid
regimen or this patient?
volu e.
A) Subcut neous NPH insulin, 1 unit/kg; re e t s necess ry.
Question 10.9.1 Which o the ollowing regimens is the B) IV regul r insulin, 5 unit IV bolus, ollowed by const nt
most appropriate initial intervention? in usion t 0.05 to 0.1 unit/kg/hr; djusted s needed.
A) 5% dextrose in 0.45% (h l -nor l) s line to run t C) Subcut neous regul r insulin, 0.5 to 1 unit/kg; djust dose
150 cc/hr. by ngerstick blood glucose results.
B) 0.45% (h l -nor l) s line with 20 Eq ot ssiu /L, to D) Intr uscul r regul r insulin, 5 to 10 units hourly; djust
run t 150 cc/hr. dose by ngerstick blood glucose results.
C) 0.9% (nor l) s line 1 L to in use s quickly s ossible. E) Insulin gl rgine 10 units d ily djusted b sed on sting glu-
D) 0.9% (nor l) s line with 20 Eq ot ssiu /L to run t cose levels.
1,000 cc/hr.
E) 5% dextrose in 0.225% (qu rter-nor l) s line with 20 Eq Answer 10.9.2 The correct answer is “B.” A bolus o IV regu-
ot ssiu /L to run t 1,000 cc/hr. l r insulin, ollowed by const nt in usion, djusted to reduce
the blood glucose level by 50 to 75 g/dL/hr, is the ro ri te
Answer 10.9.1 The correct answer is “C.” Initi l volu e ther y. T is y requently require > 0.1 unit/kg/hr, but 0.05
re l ce ent should be with isotonic s line in used t r id to 0.1 unit/kg/hr is good l ceto st rt. Intr uscul r insulin
r te (in the bsence o c rdi c dise se) until the volu e de cit d inistr tion is n ltern tive, but bsor tion is unreli ble,
is corrected. “D” is o s eci l note. In gener l, ot ssiu should es eci lly in hy otensive tients. Long- cting insulins nd
be dded to the f uid l ter (o en the second b g) unless the subcut neous insulin d inistr tion h ve no l ce in the initi l
tient is lre dy hy ok le ic on the rst blood work (getting n ge ent o DKA.
ot ssiu , glucose, nd sodiu on the rst blood g s is good
olicy). Pot ssiu re l ce ent is y be essenti l even in the HELPFUL TIP:
hy erk le ic tient, s correction o the keto cidosis le ds to Althou h tradition, the bolus o re ular insulin is unnec-
r id shi o ot ssiu into the intr cellul r co rt ent. essary and does not chan e outcomes. Startin a drip o
Re e ber th t n cidosis rti ci lly incre ses the seru re ular insulin is the critical step here.
ot ssiu by shi ing ot ssiu extr cellul rly. T e ot ssiu
incre ses by roxi tely 1 Eq/L or every H oint o 0.1
below 7.4 (so, H o 7.3 will incre se the ot ssiu ro 4 to Question 10.9.3 Which o the ollowing types o insulin can
5 Eq/L). See Ch ter 5 or ore on cidosis, lk losis, nd the be administered IV?
e ects on ot ssiu . “D” is incorrect bec use you would like A) NPH insulin.
the rst liter to in use s quickly s ossible in DKA nd not B) L ntus insulin.
over n hour. Besides, 20 Eq o ot ssiu IV in 1 hour should C) Lente insulin.
not be routine nd only given i the tient h s continuous c r- D) Ultr lente insulin.
di c onitoring. E) None o the bove.
288 FAMILY MEDICINE EXAMINATION & BOARD REVIEW

Answer 10.9.3 The correct answer is “E.” T e only insulin th t Answer 10.9.5 The correct answer (and alse statement)
c n be d inistered IV is regul r insulin. As n side, so e is “A.” Consensus is th t the So ogyi heno enon does
insulins re inco tible when ixed together in the s e not exist reg rdless o wh t we were t ught (N Engl J Med.
syringe. For ex le, insulin gl rgine should not be ixed 1987;317(25):1552). But just or the record, it w s thought to
with ny other or s o insulin due to the low H o its diluent. occur when tient beco es hy oglyce ic (o en in the id-
Further, hos h te-bu ered insulins (e.g., NPH) should not be dle o the night) nd ex eriences re ctive hy erglyce i ro
ixed with Lente insulins. drenergic out ouring. Actu lly, quite di erent heno enon
e rs to occur: nocturn l hy erglyce i is strongly ssoci-
T e tient’s st tus i roves, nd you re-check his blood ted with elev ted orning glucose levels. All o the rest re
sug r. His glucose is now 200 g/dL nd his insulin dri is true st te ents. “B” is correct bec use bet -hydroxybutyr te is
still t 5 units er hour. His H is 7.30 with bic rbon te et bolized to ceto cetic cid th t will incre se seru ketone
level o 14 Eq/L. e sure ents. “C” is true since DKA is ro-inf tory
st te nd the WBC count is o en elev ted due to DKA nd not
Question 10.9.4 Given that his glucose has almost normal- in ection. “D” is true st te ent. P tients with lcoholic hy o-
ized, your reaction at this point is to: glyce i h ve exh usted their glycogen stores nd re lso gen-
A) Ad inister bic rbon te in order to nish correcting the H. er lly NAD de cient nd thus h ve i ired gluconeogenesis.
B) Decre se the r te o the insulin in usion to 2 units er hour. T us, gluc gon will not work. Another grou on which gluc -
C) Add 5% to 10% dextrose to his IV f uids. gon will not work is the in nt or child who beco es hy ogly-
D) Consider the ddition o n or l hy oglyce ic gent. ce ic overnight nd h s seizure in the orning. T ey h ve
E) Discontinue IV f uids nd switch to or l rehydr tion. lre dy de leted their stores o glycogen.

Answer 10.9.4 The correct answer is “C.” T is tient is still


HELPFUL TIP:
cidotic nd will need continued insulin to reverse his c t -
Consider the possibility o a silent myocardial in arction
bolic st te. T us, the ro ri te tre t ent is to incre se the
in a diabetic patient who is enerally well-controlled
ount o sug r he is getting— dd so e dextrose to his IV
but suddenly has elevated blood su ars.
f uids. Re e ber, DKA is not ri rily result o too uch
sug r but r ther o too little insulin. “A” is incorrect. Bic rbon-
te l ys no role in the tre t ent o DKA no tter wh t the HELPFUL TIP:
H. In ct, the d inistr tion o bic rbon te ctu lly rolongs Twenty percent o patients with DKA have “normo-
cidosis nd ketosis nd roduces r doxic l CNS cidosis. lycemic” DKA and present with a blood su ar under
In ddition, it shi s the oxygen dis ssoci tion curve to reduce 300 m / dL.
oxygen delivery to the tissue. Fin lly, the only redictor o
cerebr l ede in children tre ted or DKA is the d inis-
tr tion o bic rbon te. So, there is no need to restrict f uids in
children being tre ted or DKA ( lthough this does NO e n It is ti e to tr nsition this tient to ho e-going insulin
you should over hydr te the ). regi en.

Question 10.9.6 Which o the ollowing statements is FALSE


HELPFUL TIP: regarding the use o insulin in this patient?
The common causes o DKA: “ the 5 I’s” A) Insulin dete ir nd insulin gl rgine re both long- cting,
Insulin or medication noncompliance; In ection; myo- essenti lly equiv lent, nd c n ( l ost lw ys) be used once
cardial In arction; Incision (sur ery); Incidental (new d y s b s l insulin.
dia nosis o DM presents with DKA) (Not by Dr. Mark B) I you use NPH, st rt with 0.1 to 0.2 units/kg/d y nd give
Yo ee . . . a shout out to his mom). 2/3 in the AM nd 1/3 HS.
C) Regul r insulin should be given with e ls to cover blood
sug rs nd gener lly requires 0.1 to 0.2 units/kg/d y s
Question 10.9.5 Which o the ollowing statements is st rt.
FALSE? D) Met or in c n rkedly reduce the need or insulin in this
A) T e So ogyi heno enon occurs when tient’s blood tient.
sug r beco es elev ted nd there is re ctive hy oglyce- E) T is tient’s insulin need y rkedly di inish over the
i . next 2 weeks.
B) P tients who re being tre ted ro ri tely or DKA y
h ve n incre se in seru ketones during tre t ent. Answer 10.9.6 The correct answer (which is alse) is “D.”
C) An elev ted WBC count is not strong redictor o in ec- Met or in is used only in DM2. By virtue o the ct th t he’s
tion in tients with DKA. resenting with DKA, our tient l ost ssuredly h s ty e
D) Gluc gon is n in ro ri te tre t ent o tients with 1 di betes ellitus (DM1). T e rest re true st te ents. Insu-
lcoholic hy oglyce i . lin dete ir nd gl rgine c n both be used once d y in ost
CHAPTER 10 • ENDOCRINOLOg Y 289

tients ( lthough not lw ys). T e st rting dose o both regul r


nd NPH insulin is bout 0.1 to 0.2 units/kg/d y or tot l ini- QUICK QUIZ: DIABETES PREVENTION
ti l insulin dose o 0.2 to 0.4 units/kg/d y to st rt. “E” re ers to
the “honey oon eriod.” P tients usu lly resent with DM in Which intervention h s been shown to h ve the greatest e ect
rel tion to so e et bolic stress. Once this stress resolves, the in reventing or del ying the onset o DM2 in tients with
need or insulin y decre se rkedly. T us, close onitoring i ired sting glucose or i ired glucose toler nce?
is required. (Editors note: Yes, et or in h s been studied in A) Diet ry odi c tions nd incre sed ctivity.
DM 1 but see s not le d to ch nges in the HbA1c [ lthough it B) E rly gli izide tre t ent.
c n reduce the insulin dose needed]). C) E rly et or in tre t ent.
D) Intensive tness tr ining.
E) Weight loss > 25% o b seline.
HELPFUL TIP:
There is little is any outcome advanta e to brand name The correct answer is “A.” Met or in h s been shown to h ve
insulins versus eneric insulins. Cost and convenience so e bene t in del ying rogression to di betes but is less e ec-
should be the main considerations in choosin one ver- tive th n diet nd ctivity odi c tions. T e studies showing
sus the other. Lantus, Novolo , and Humalo cost up bene t ro li estyle odi c tions used uch less ggressive
to $250.00/bottle with eneric NPH costin as little as t rgets or weight loss nd ctivity level th n the 25% listed
$25.00/bottle. in nswer “E.” T us, “E” is incorrect. Other edic tions th t
h ve been used (success ully) to reduce rogression to di be-
tes include the thi zolidinediones (“glit zones”) nd c rbose.
T is tient resents to your clinic 10 ye rs l ter ( nd re- However, exercise nd diet re su erior to drugs. T e ben-
su bly ulti le ti es in between). He h s noted ost r n- e t o drugs is rgin l. In ddition, the thi zolidinediones
di l ullness, re ux, nd occ sion l vo iting. You do g s- h ve signi c nt downsides including ede nd ossibly
tric e tying study th t shows del yed g stric e tying. incre sed c rdi c events.

Question 10.9.7 Which o the ollowing are appropriate


drugs or treating this patient’s delayed gastric emptying? HELPFUL TIP:
A) O e r zole. Nonpharmacolo ic therapies that can reduce insulin
B) Metoclo r ide. resistance include en a in in 30 minutes o modest
C) R nitidine. aerobic exercise 5 days per week (tell patients to aim or
D) Erythro ycin. 150 minutes o exercise per week—how it’s divided is
E) B nd D. not important), havin hi h iber in the diet, decreasin
caloric intake, and losin wei ht. Ma ic pill? We’re still
Answer 10.9.7 The correct answer is “E.” Both etoclo- waitin . . .
r ide nd erythro ycin will s eed g stric e tying. A third
drug, cis ride, is v il ble on co ssion te use rotocol. It
w s re oved ro the gener l rket second ry to risks o QUICK QUIZ: DIABETIC RETINOPATHY
rolonged Q nd subsequent tors des de ointes. It is cle r
th t di betic g stro thy is t le st so ewh t reversible with
Which o the ollowing interventions h s NO been shown to
good glucose control. T e higher the sug r, the worse the sto -
revent loss o vision in tients with retino thy due to ty e 2
ch e ties.
di betes?
A) L ser hotoco gul tion ther y.
HELPFUL TIP: B) As irin.
Remember that erythromycin causes a prolon ed QT. C) ight glyce ic control.
And, there is o ten tachyphylaxis to the e ect o erythro- D) ight blood ressure control.
mycin on astric emptyin . Also remember that chronic
metoclopramide is associated with extrapyramidal side The correct answer is “B.” As irin h s not been shown to be o
e ects that may be permanent (e. ., tardive dyskinesia). ny bene t in the E rly re t ent Di betic Retino thy Study.
All the other o tions h ve been shown to be use ul in del ying
Objectives: Did you learn to . . . the develo ent o di betic retino thy or reventing its ro-
• Dia nose a patient with DKA? gression to visu l loss.
• Initiate therapy in DKA?
• Identi y causes o DKA?
CASE 10.10
• Realize that the Somo yi phenomenon is scary but not real You re seeing new tient in your o ce. He is 47-ye r-
(like g odzilla)? old n with resenting co l int o tigue or sever l
• Prescribe treatment or diabetic astroparesis? onths. He denies ever, rigors, cough, n use , or di rrhe .
290 FAMILY MEDICINE EXAMINATION & BOARD REVIEW

He h s lost bout 10 lb. U on questioning hi you discover o diabetes and determine whether the patient has type 1
th t he is lso h ving nocturi ost nights nd is thirsty or type 2 diabetes?
ll the ti e. He h s sth , or which he uses n lbuterol A) C- e tide level.
etered dose inh ler occ sion lly; he h s no other chronic B) Anti-islet cell ntibodies.
edic l roble s nd t kes no other edic tions on regu- C) Anti-insulin ntibodies
l r b sis. He h s ily history o di betes, hy ertension, D) None o the bove.
nd he rt dise se. He s okes bout one ck er d y, nd he
works s te cher t the loc l high school. He is w re o no Answer 10.10.2 The correct answer is “D.” T is tient’s ge,
occu tion l ex osure to toxins. history, ex in tion (BMI 38), nd l bor tory ndings re
Physic l ex in tion reve ls the ollowing: 37°C, BP consistent with the di gnosis o DM2. None o the other stud-
135/83 Hg, P 72 b , BMI 38 kg/ 2. Aside ro obesity, ies listed needs to be er or ed. However, i questions re in
the re inder o the ex in tion is nor l. reg rding the ty e o di betes (which will then ect ther y,
L bor tory test results reve l the ollowing: nor l rognosis, ollow-u , etc.), you y choose to er or urther
CBC, BUN/cre tinine, nd electrolytes. You sk hi to studies. In DM1, the C- e tide level ( rker o endogenous
return to the o ce the next d y or sting l bor tory tests, insulin roduction) is low. I it is equivoc l, give glucose lo d
which reve l sting glucose o 123 g/dL nd n HbA1c (e.g., l rge e l) nd see i it goes u . I it goes u , the di gnosis
o 7.5%. is likely DM2. Anti-islet cell ntibodies re resent in 80% o
ty e 1 di betics nd, i ound, re essenti lly di gnostic o ty e 1
Question 10.10.1 Does this patient have diabetes? di betes. “D” is incorrect bec use nti-insulin ntibodies h ve
A) Yes; he h s n elev ted sting glucose. low sensitivity or DM 1 nd y be elev ted second ry to the
B) Prob bly; he needs second sting glucose to con r the use o exogenous insulin.
di gnosis.
C) Prob bly; he needs second HbA1c to con r the di gno- HELPFUL TIP:
sis. To be complete, anti- lutamic acid decarboxylase (anti-
D) Yes; he h s the cl ssic sy to s o di betes: tigue, weight g AD) antibodies are present in 70% o patients with
loss, nd thirst, ssoci ted with n elev ted glucose. DM1 at the time o dia nosis.
E) Prob bly not; his HbA1c is not > 8%.
Question 10.10.3 The pathologic actors involved in type 2
Answer 10.10.1 The correct answer is “C.” I results o two diabetes in adults include:
di erent di gnostic tests or DM re discord nt, the test th t is A) P ncre tic bet -cell destruction through yet undeter ined
di gnostic o di betes should be re e ted. “A” nd “B” re incor- in ectious rocess.
rect bec use the sting glucose is < 126 g/dL (the threshold B) T e roduction o nti-insulin ntibodies th t c use reci i-
or di betes). “D” is incorrect bec use we do not h ve his r n- t tion o insulin/ ntibody co lexes.
do glucose v lue th t is ≥ 200 g/dL. “E” is incorrect bec use C) Resist nce to the e ects o insulin t eri her l tissues nd
the A1c cuto or di betes di gnosis is ≥ 6.5%. relative insulin de ciency th t is rogressive over ti e.
D) An utoso l-do in nt rocess, with the di betes gene
loc ted on the long r o chro oso e 18.
HELPFUL TIP: E) oo uch exercise nd co lete l ck o “beer gut.”
There are new uidelines about whom to screen or
diabetes (USPSTF, 2015). Screen asymptomatic adults Answer 10.10.3 The correct answer is “C.” DM2 is the result o
a e 40 to 70 years o a e who are overwei ht or obese. the develo ent o insulin resist nce t the eri her l tissues
Repeat this screenin every 3 years. The 2008 USPSTF (e.g., t nd uscle cells) nd rel tive l ck o insulin co red
uideline recommended screenin hypertensive adults to the incre sing ount th t the body requires. “A” is incorrect.
or diabetes, which has one the way o the dinosaur However, utoi une destruction o bet -cells in the ncre s
or the 2015 uidelines. The 2015 ADA uideline recom- is res onsible or c using DM1. “B” is wrong lthough there re
mends screenin all asymptomatic adults every 3 years nti-insulin ntibodies ound in DM1. “D” is incorrect s well, but
startin at a e 45; earlier screenin is recommended there is strong genetic co onent to DM2. T e ex ct genetic
or adults at hi h risk o type 2 diabetes (BMI ≥ 25 k / ctors th t c use DM2 in dults h ve not been co letely elu-
m 2; habitual physical inactivity; belon in to a hi h-risk cid ted, but there is no single gene res onsible tr ns itted in
ethnic or racial roup; previously identi ied impaired n utoso l do in nt shion. “E” is incorrect bec use l ck
lucose tolerance; hypertension; dyslipidemia; history o exercise, weight g in, diet ry ctors, nd trunc l obesity (the
o estational diabetes or delivery o a baby wei hin “beer gut”) redis ose ersons to the develo ent o DM2.
> 9 lb; and polycystic ovary syndrome).
You eet with the tient nd his husb nd to go over the test
results nd ex l in the di gnosis o di betes. Given his ge,
Question 10.10.2 Assuming another A1c is above 6.5%, body h bitus, nd l ck o exercise, you eel cert in th t this
what urther study must be done to complete the diagnosis tient h s ty e 2 di betes. You rovide so e b sic educ tion
CHAPTER 10 • ENDOCRINOLOg Y 291

on the n ture o di betes, its n tur l history, nd wh t c n be Answer 10.10.6 The correct answer is “E.” All o those listed—
done to n ge it. exce t cooties— re side e ects o ACE inhibitors. Cooties re
irborne r sites or which obnoxious 4th gr de boys serve
Question 10.10.4 What is the most important next step or the in host. Ho e you got your “cooties shot” when you were
this patient? younger. I so, you should be ne.
A) Initi tion o insulin ther y. Adverse e ects o ACE inhibitors re due to either reduced
B) Initi tion o n ACE inhibitor ngiotensin II or tion or to incre sed kinin or tion. T ose
C) Re err l to n endocrinologist. rel ted to reduced ngiotensin II or tion include hy oten-
D) Di betic educ tion cl sses. sion, cute ren l ilure, hy erk le i , nd roble s during
E) Initi tion o glyburide or other sul onylure . regn ncy. Side e ects thought to be rel ted, t le st in rt,
to incre sed kinins include cough, ngioede , nd n hy-
Answer 10.10.4 The correct answer is “D.” A gener l educ - l ctoid re ctions. er in tion o the ACE inhibitor should be
tion rogr th t includes in or tion on diet, dise se n- considered i hy erk le i c nnot be controlled or the seru
ge ent, nd the ily’s role in success ul di betes c re is the cre tinine concentr tion incre ses ore th n 30% bove the
ost i ort nt intervention listed. While s eci list consult - b seline v lue within the rst 6 to 8 weeks when blood ressure
tion y be use ul in co lex di betic tients or in those who is reduced. Also, both ACE inhibitors nd ARBs re contr indi-
re not res onding to tre t ent, gener list hysici ns rovide c ted in regn ncy.
c re to the jority o tients with di betes. Insulin ther y
is not indic ted t this oint, nd n ACE inhibitor y or y
HELPFUL TIP:
not be hel ul de ending on the tient’s blood ressure nd
g uidelines published by JNC8 (2014) and the ADA
urine rotein. “E” is lso incorrect (kee re ding to le rn why).
(2015) endorse hi her blood pressure oals or hyper-
tensive diabetics than was the case in prior uidelines.
U on his return, you nd th t the tient’s blood ressure is
The blood pressure tar et or most diabetic patients is
elev ted. On three se r te occ sions, he h s systolic res-
now < 140/90 mm H . A lower tar et (e. ., < 130/80)
sure ≥ 140 nd di stolic ressure ≥ 90 Hg.
may be appropriate or certain individual, such as
youn er patients, i it can be achieved without undue
Question 10.10.5 Which class o medications is the best
treatment burden.
choice or initial therapy o hypertension in diabetics?
A) ACE inhibitors.
B) C lciu -ch nnel blockers.
C) Loo diuretics. Af er 3 onths o diet ry ther y nd li estyle odi c tions,
D) V sodil tors. the tient returns to see you with his husb nd. While he h s
E) Bet -blockers. been dherent to the reco end tions given by you nd the
di betes educ tion st , his HbA1c re ins elev ted t 7.9%.
Answer 10.10.5 The correct answer is “A.” ACE inhibitors h ve You decide to begin h r cologic ther y.
been shown to rovide ren l rotection in tients with di be-
tes (ty es 1 nd 2). P tients with lbu inuri nd hy ertension Question 10.10.7 Which medication is the most appropriate
will cert inly bene t ro n ACE inhibitor. Loo diuretics f rst-line therapy or an obese patient with type 2 diabetes?
(e.g., urose ide) re not indic ted or the ri ry tre t ent A) A thi zolidinedione (“glit zone” [e.g., Actos]).
o hy ertension in di betics (or, re lly, nyone else). Angioten- B) A sul onylure (e.g., gli izide).
sin rece tor blockers (ARBs) re re son ble ltern tive in the C) Insulin.
hy ertensive tient with lbu inuri i n ACE inhibitor is not D) Met or in.
toler ted. V sodil tors nd c lciu -ch nnel blockers re not E) A di e tidyl e tid se-4 inhibitor [DPP-4 or “gli tin” (e.g.,
o ti l choices in this tient lthough nondihydro yridine J nuvi )].
c lciu ch nnel blockers (ver il, dilti ze ) re n o tion
or ren l rotection in tients with worsening lbu inuri Answer 10.10.7 The correct answer is “D.” Met or in does
es eci lly in those who c nnot toler te n ACE inhibitor or not c use weight g in (unlike ost other tre t ents or di be-
ARB. “E,” bet -blockers, should not be used rst line or tre t- tes), h s evidence or reducing di betes co lic tions, nd is
ing hy ertension in tients without c rdi c dise se. inex ensive; thus, it is the drug o choice in ost DM2 tients.
In ddition, it is well toler ted by ost tients nd h s very
Question 10.10.6 Which o the ollowing is NOT a side e ect little risk o hy oglyce i . Studies co ring e ects on end-
o ACE inhibitors? org n dise se show better outco es with et or in th n with
A) Acute ren l ilure sul onylure s. T i zolidinediones known s “glit zones” (“A”)
B) Hy erk le i re not rst-line or sever l re sons, chie ong these being
C) Dry cough the ossibility o incre sed c rdiov scul r events (rosiglit zone
D) Angioede nd ioglit zone c n ex cerb te CHF). T e tr ck record o rosi-
E) Cooties glit zone is so ewh t s otty: it w s re oved ro the rket
292 FAMILY MEDICINE EXAMINATION & BOARD REVIEW

due to n incre se in c rdiov scul r events nd then w s rein- or ily history o edull ry thyroid c rcino or MEN 2A
troduced even though there w s no ddition l s ety d t ; void or 2B. T e risk o hy oglyce i is s ll—but not zero—with
it i you c n. Sul onylure s (“B”) re lso e ective nd well-tol- GLP-1 gonists. So ewhere between 10% nd 50% o tients
er ted but h ve signi c nt risk or hy oglyce i nd re sso- y develo GI sy to s.
ci ted with weight g in. All other or l drugs re best considered
second-line gents. “E” is o s eci l note. DPP-4 inhibitors, lso Question 10.10.9 Met ormin should NOT be used in which
known s “gli tins,” block the degr d tion o the body’s endog- class o patients?
enous incretin, which hel s to lower blood sug r. DPP-4 cts A) P tients with COPD.
s “gluc gon-like e tid-1” (GLP-1). DPP-4 inhibitors (Sit - B) P tients with i ired ren l unction.
gli tin–J nuvi , s x gli tin—Onglyz , logli tin–Nesin . . . see C) P tients with leuke i s or ly ho s.
below or c ution) c n be used s n “ dd-on” ther y i tr di- D) Post yoc rdi l in rction tients with nor l systolic
tion l hy oglyce ic gents re not e ective nd h ve the bene t unction.
o so e weight loss. In tient with very oor control (e.g., E) P tients with insu cient t stores.
A1c > 9%) t di gnosis, insulin (“C”) would be otenti l rst-
line gent, but not in this tient whose is rti lly controlled. Answer 10.10.9 The correct answer is “B.” P tients with ren l
dise se re t higher risk o l ctic cidosis, the ost severe
co lic tion o et or in ther y, lthough it is exceedingly
HELPFUL TIP:
r re (3 c ses er 100,000 vs. 2 c ses er 100,000 with other
New data su ests an association (or maybe causal-
hy oglyce ic gents). Current nu cturer reco end tions
ity) o con estive heart ailure with saxa liptin (On -
st te th t et or in should be voided i the seru cre tinine
lyza) and alo liptin (Nesina). This does not seem to be
≥ 1.5 g/dL in les nd ≥ 1.4 g/dL in e les. However,
a problem with sita liptin (Januvia). These dru s only
met ormin is likely sa e to start as long as the GFR is > 30 mL/min
reduce HbA1c by about 0.5% and have no data dem-
(max dose 1,000 mg/day or those with a GFR between 30 and
onstratin cardiovascular bene it. Other reported side
60 mL/min). P tients with ul on ry or neo l stic dise ses y
e ects include headache, nasopharyn itis, and upper
t ke et or in unless they lso h ve severe he tic or ren l
respiratory tract in ections. Also, acute pancreatitis has
ilure. CHF is rel tive contr indic tion to the use o et or-
been reported in association with DPP-4 inhibitors, but
in; it is being used with incre sed requency in st ble, tients
currently there is insu icient evidence to determine a
not t risk or hy oxe i . Use clinic l judg ent. Met or in
causal relationship.
should be held or 48 hours f er contr st studies. Ple se re er
to the gener l reco end tions reg rding nti-hy erglyce ic
ther y in ty e 2 di betes by the ADA ( ble 10-2).
HELPFUL TIP:
Another class o “ luca on-like peptide-1” (g LP-1) Question 10.10.10 Which one o the ollowing is NOT a risk
dru s act as receptor a onists (e. ., exenatide—Byetta, actor or prognostic marker or lower-extremity amputa-
lira lutide—Victoza, albi lutide—Tanzeum, etc.) which tion in patients with diabetes?
bind to the g LP-1 receptor. They are resistant to break- A) Di betic retino thy.
down by the body and thus provide sustained lucose B) Bony de or ity o the eet or nkles.
control. These are iven by subcutaneous injection and C) C-re ctive rotein (CRP) level.
are airy costly but have the advanta e o bein more D) Abnor l ono l ent testing or sensory unction.
potent than the liptins and may even cause wei ht E) Severe n il thology.
loss in type 2 diabetics.
Answer 10.10.10 The correct answer is “C.” T e risk o ulcers
or ut tions is incre sed in tients who h ve h d di betes
Question 10.10.8 Which o the ollowing is NOT a side e ect or 10 ye rs or ore, re le, h ve history o oor glucose
o GLP-1 receptor agonists (exenatide, liraglutide, albiglu- control, or h ve evidence o icrov scul r co lic tions o
tide)? di betes (e.g., retino thy). Bony de or ities, loss o rotec-
A) Weight g in. tive sens tion, nd severely dystro hic toen ils re lso risk
B) P ncre titis. ctors or ut tion. T e Se es–Weinstein 10 g ono-
C) Hy oglyce i l ent sensory ex in tion is the ost sensitive neurologic
D) GI u set. test or redicting the uture occurrence o di betic oot ulcer.
E) T yroid tu or. An elev ted CRP in nd o itsel is not known risk ctor or
ut tion, but CRP y be elev ted i there is lower extre ity
Answer 10.10.8 The correct answer is “A.” Actu lly, GLP-1 in ection resent.
gonists y c use weight loss o 1.5 to 2.5 kg over 30 weeks.
T ese drugs re ssoci ted with ncre titis lthough r rely nd At the next visit, you review the tient’s edic l record nd
the ssoci tion is tenuous. Due to n ssoci tion with thyroid try to ssure th t he is u to d te on his reventive he lth
c ncer, their use is contr indic ted in tients with erson l c re.
CHAPTER 10 • ENDOCRINOLOg Y 293

TABLE 10-2 RECOMMENDATIONS FOR MEDICALTREATMENT DIABETES MELLITUS TYPE 2

DPP-4-i, DPP-4 inhibitor; xs, ractures; g I, astrointestinal; g LP-1-RA, g LP-1 receptor a onist; g U, enitourinary; HF, heart ailure; Hypo, hypo lycemia;
Sg LT2-i, Sg LT2 inhibitor; SU, sul onylurea; TZD, thiazolidinedione.
From American Diabetes Association. Standards o Medical Care in Diabetes-2015. Diabetes Care. 2015;38:S1–S93.

Question 10.10.11 Which o the ollowing is NOT true the di gnosis nd then ye rly. Age t the ti e o di gnosis is
regarding preventive services in diabetics? not ctor in deter ining when n eye ex in tion should be
A) P tients di gnosed with ty e 2 di betes should h ve dil ted done. See ble 10-3 or co onents o reco ended di betes
eye ex in tion t the ti e o di gnosis. ollow-u .
B) P tients with ty e 1 di betes should h ve dil ted eye
ex in tion t the ti e o di gnosis i they re over Question 10.10.12 Speaking o prevention, your patient,
ge 12. now 48 years old with his diabetes controlled, asks i he
C) Check SH nnu lly in ty e 1 di betes, in tients with dys- should be taking an aspirin daily to protect his heart. You
li ide i or di betic wo en over ge 50 ye rs. respond:
D) A urine icro lbu in should be checked t le st ye rly in A) “ ke s irin 325 g d ily bec use it will lower your risk o
ll ty e 2 di betics. yoc rdi l in rction.”
E) A oot ex in tion using 10 g nylon icro l ent should B) “Di betes does not uto tic lly qu li y you or s irin
be done nnu lly or ll di betics. ther y. Let’s check your therosclerotic c rdiov scul r dis-
e se (ASCVD) score.”
Answer 10.10.11 The correct answer is “B.” P tients with di - C) “T e risks nd bene ts o s irin in your c se re unknown.”
betes ty e 1 should h ve n eye ex in tion 3 to 5 ye rs er D) “ ke it by the trucklo d. I’ve got lot o stock in B yer.”
294 FAMILY MEDICINE EXAMINATION & BOARD REVIEW

TABLE 10-3 SUMMARY OF SCREENING Answer 10.10.13 The correct answer is “C.” T e thi zolidin-
RECOMMENDATIONS ediones (“glit zones”) tend to c use ede s one o their jor
side e ects. T us, they re contr indic ted in tients with his-
DM type 1:
tory o he rt ilure. So e drug co bin tions c n c use ede ,
• Urine microalbumin startin at a e 12 and then every 6 to including the co bin tion o gli e iride nd et or in.
12 months.
• Dilated eye examination 5 years a ter dia nosis and then annually. Bec use o the roble with ede , you decide to ch nge
• HbA1c every 6 months or stable patients achievin lycemic oals,
this tient to sul onylure , nd you choose to st rt gly-
every 3 months or patients chan in therapy or not meetin
lycemic oals. buride. T e tient does well on this or sever l weeks but
• Blood pressure screenin at every visit. is then ound unconscious in his ho e with blood sug r o
• Foot examination and screenin or polyneuropathy at dia nosis 20 g/dL. Strike two, doc! He is r idly revived by the r -
and annually. edics with n o D50. You re c lled to see the tient
• I not per ormed/available within past year
in the ED. He is currently w ke, convers nt, nd e ting (“
• Fastin lipid pro le, includin total, LDL, and HDL cholesterol and
tri lycerides, as needed gre t excuse or cou le o cookies!”). He would like to go
• Serum creatinine and calculated lomerular ltration rate ho e since he is b ck to his b seline.
• TSH in type 1 diabetes, dyslipidemia, or women over a e 50 years.
Question 10.10.14 Which o the ollowing is the best next
DM type 2: Same as DM1 above, except:
step in the management o this patient’s hypoglycemic
• Eye examination at time o dia nosis and then yearly. episode?
• Urine microalbumin at the time o dia nosis and then every
A) Disch rge the tient ro the ED nd h ve hi continue
6–12 months.
his regi en, including glyburide.
Data rom American Diabetes Association. Standards o medical care in B) Disch rge the tient ro the ED nd h ve hi sto his
diabetes—2011. Diabetes Care. 2011;34:S11–S61. glyburide nd st rt insulin.
C) Ad it the tient nd st rt hi on IV dextrose 5% in usion.
D) Ad it the tient nd observe hi or 24 hours.
Answer 10.10.12 The correct answer is “B.” T e ADA rec- E) Give hi gi c rd to loc l c sino bu et, nd tell hi ,
o ends considering s irin ther y (75–162 g/d y) s “Pretend it’s your birthd y!”
ri ry revention str tegy in those with ty e 1 or ty e 2
di betes t incre sed c rdiov scul r risk (10-ye r risk ≥ 10%). Answer 10.10.14 The correct answer is “D.” P tients on n
Who would be in this c tegory o risk? Most di betic en ged or l hy oglyce ic gent—es eci lly longer- cting gents like
> 50 ye rs or di betic wo en ged > 60 ye rs who h ve t glyburide—should be d itted or observ tion. It would be re -
le st one ddition l jor risk ctor ( ily history o c r- son ble to let hi e t, check his glucose eriodic lly, nd hold
diov scul r dise se, hy ertension, s oking, dysli ide i , his di betic edic tions. He is currently st ble nd does not
or icro lbu inuri ). ADA reco ends AGAINS s irin e r to need IV dextrose (“C”), but the tient should h ve
use or CVD revention or dults with di betes t low CVD IV ccess, nd h l - or ull- ule o D50 c n be given i nec-
risk (10-ye r CVD risk < 5%, such s in en ged < 50 ye rs ess ry. T e bsor tion o or l hy oglyce ic gents is so ewh t
nd wo en ged < 60 ye rs with no jor ddition l CVD err tic, nd their e ect c n be rolonged. T e tient y h ve
risk ctors) since the otenti l dverse e ects ro bleeding n ddition l e isode o hy oglyce i or u to 36 to 48 hours
likely o set the otenti l bene ts. I you decide to rescribe er the initi l e isode. T is is not true o tients on insulins
s irin or ri ry CVD revention in di betic, use 81 g (NPH or short- cting insulin [e.g., regul r, lis ro]), who y be
d ily. disch rged ro the ED er ew hours. But or this tient,
disch rge ro the ED (“A,” “B,” nd “E”) would be unwise.
Un ortun tely, this tient ollows the “rule” o ty e 2 di be-
tes nd ends u on ulti le edic tions. When he returns to HELPFUL TIP:
your clinic ew onths l ter, he is co l ining o shortness 15 to 20 o astin -actin carbohydrate like lucose
o bre th nd lower extre ity ede . You decide to ollow tablets, honey, corn syrup, non at or 1% milk, jellybeans,
the “rule” o bl ing drugs rst . . . umdrops, el tube, raisins, hard candy, re ular soda,
sweetened ruit juice can be used to treat symptomatic hy-
Question 10.10.13 Which o the ollowing drugs (by itsel — po lycemia. For severe hypo lycemia, luca on injection
not in combination with other drugs) is the most likely may be needed. g luca on, 1 m IM or IV, is another option.
cause o this patient’s edema, shortness o breath, and pos-
sible heart ailure? HELPFUL TIP:
A) Met or in. The ollowin medications enerally do not cause hy-
B) Glyburide. po lycemia: met ormin, acarbose, dipeptidyl peptidase
C) Pioglit zone. (DPP-4) inhibitors and luca on-like peptide-1 (g LP-1)
D) Lisino ril. receptor a onists.
E) Insulin.
CHAPTER 10 • ENDOCRINOLOg Y 295

D) Regul r insulin with e ls to control ost- r ndi l blood


It turns out th t one o your rtners h s st rted this tient sug rs.
on bet -blocker or its c rdio rotective nd ntihy erten-
sive e ects while you were on v c tion. T e tient w nts to Answer 10.10.16 The correct answer is “A.” Bec use o its slow
know i this y h ve revented hi ro noticing the signs rele se, insulin gl rgine rovides ste dy-st te insulin level
nd sy to s o hy oglyce i . throughout 24 hours nd is less likely to c use nocturn l hy o-
glyce i . Other o tions include insulin dete ir nd ultr lente
Question 10.10.15 Your response is: insulin (o the rket in the United St tes), or bedti e NPH,
A) “Bet -blockers reduce your bility to recognize hy oglyce- ll o which h ve rolonged ction. Regul r insulin ( s ound
i nd the drug should be sto ed.” in 70/30) is gener lly not the l ce to st rt with ty e 2 di bet-
B) “Bet -blockers reduce your bility to recognize hy oglyce- ics (“B”). Multi le d ily insulin injections y be necess ry
i but the bene ts re worth it.” or ty e 1 di betics, but r rely re so or ty e 2 tients. For
C) “Bet -blockers do not decre se your bility to recognize DM2, long- cting insulin is usu lly st rted rst nd bolus e l-
hy oglyce i to ny gre t degree. Don’t worry bout it.” ti e insulin dded l ter, so “C” would not be the rst choice.
D) “ACE inhibitors re better drugs bec use they do not con- As noted s bove, regul r insulin (“D”) isn’t the l ce to st rt
tribute to hy oglyce i in di betics.” in DM 2. I you re trying to st rt with the si lest ossible
regi en, choose so ething like “A.”
Answer 10.10.15 The correct answer is “C.” Bet -blockers
do not signi c ntly inter ere with tients’ bility to recognize
hy oglyce i . T e in thing th t contributes to un w reness HELPFUL TIP:
o hy oglyce i in di betics is the r te o glucose dro ( slow Sul onylureas and insulin work in the same manner:
dro is less likely to be noticed) nd utono ic insu ciency they both increase circulatin insulin levels. For this rea-
( tients c nnot res ond with t chyc rdi , swe ting, etc., to the son, some choose not to use these two dru s to ether
out ouring o drenergics). “D” is incorrect. ACE inhibitors, and will taper the sul onylurea a ter the patient is on a
like bet -blockers, re ctu lly ssoci ted with hy oglyce i in stable dose o insulin.
di betics.

HELPFUL TIP: Your tient is hos it lized or cute diverticulitis nd


The ADA recommends the tar et A1c in nonpre nant requires urgent rti l colecto y.
adults with diabetes be < 7%. However, it also advises
more or less strin ent lycemic oals may be appro- Question 10.10.17 Which o the ollowing statements
priate or individual patients and oals should be in- regarding the management o diabetes in hospitalized
dividualized on the basis o duration o diabetes, a e/ patients is TRUE?
li e expectancy, comorbid conditions, known CVD or A) Hy erglyce i in the hos it l h s ini l i ny e ect on
advanced microvascular complications, hypo lycemia outco es o yoc rdi l in rction.
unawareness, and individual patient considerations. B) A st nd rdized sliding-sc le insulin regi en is dequ te to
For patients with multiple comorbidities and hi her control hy erglyce i in ll hos it lized di betic tients.
risks or hypo lycemia and other dru adverse e ects, a C) Insulin require ents will be lower or cutely ill, hos it l-
hi her A1c, even up to 8.5%, may be acceptable (but try ized di betic tients.
tellin that to the insurance companies). D) Met or in should be discontinued in seriously ill, hos it l-
ized tients.

You d it the tient nd dvise hi to c rry source o Answer 10.10.17 The correct answer is “D.” In gener l, one
glucose with hi t ll ti es nd everybody h s h y out- should consider discontinu tion o et or in in severely
co e until . . . ill, hos it lized tients due to the ossible need or contr st
T e tient h s develo ed ersistent hy erglyce i studies, ch nges in f uid b l nce, ch nges in glo erul r ltr -
des ite being on xi l doses o et or in nd glyburide. tion r te, etc. “A” is incorrect. Hy erglyce i is ssoci ted with
He is willing to begin insulin ther y, but w nts to give hi - worse outco es in hos it lized tient with c rdi c dise se nd
sel s ew injections s ossible. those who re in n intensive c re unit or ost re sons. But
this y be bec use elev ted blood sug rs suggest they re
Question 10.10.16 Which o the ollowing regimens would under gre ter et bolic stress nd sicker to begin with. “B”
be best or him? is lso incorrect. Sliding-sc le regi ens, i used t ll, should
A) A single injection o insulin gl rgine (L ntus M) or NPH be individu lized to e ch tient, r ther th n rescribed s
insulin t bedti e. st nd rdized regi en. It h s beco e cle r th t sliding sc le is
B) A single injection o 70/30 NPH/regul r insulin t bedti e. not the best w y to control blood sug rs in hos it lized tients.
C) A b seline bedti e injection o insulin gl rgine nd u to Continuing so e ty e o “nor l” insulin regi en is best,
three injections o short- cting insulin with e ls. using su le ent l insulin s needed. In hos it lized tients,
296 FAMILY MEDICINE EXAMINATION & BOARD REVIEW

it is gener lly reco ended to discontinue or l ntidi betic


(Invokana), dapa li lozin (Farxi a), and empa li lozin
gents nd control the glucose using insulin only, rovided in
(Jardiance). They promote the renal excretion o lu-
co bin tion or including b s l long cting insulin lus
cose (literally peein out the su ar) and thereby mod-
re- e l short cting insulin i on diet nd correction l insulin
estly lower elevated blood lucose levels in patients
( k b s l/bolus insulin). T e stress o cute illness nd surgery
with type 2 diabetes. Sg LT2 inhibitors lower A1c levels
will likely incre se insulin require ents in ost di betics, not
by 0.5% to 0.7%, limited by the iltered load o lucose
decre se the .
and the osmotic diuresis that is caused by this therapy.
The lucose-lowerin e ect is independent o insulin
HELPFUL TIP: (beta cell unction and insulin sensitivity). The overall
Chasin the lucose in an attempt to maintain ti ht bene its o Sg LT2 inhibitors include a decrease in blood
control in the hospital is counterproductive and does pressure and wei ht and a low incidence o hypo lyce-
not improve outcomes! Even thou h patients with hy- mia. The side e ects are exactly what you would think
per lycemia may do worse, it is clear that the elevated iven the mechanism o action is to pee out su ar: vul-
lucose is a marker or metabolic stress which is why vova inal candidiasis, urinary requency, and urinary
outcomes mi ht be worse . . . they are sicker to be in tract in ection. LDL oes up 4% to 8%. Also, the FDA has
with. Thus or sicker patients, aim or a blood su ar o issued a warnin re ardin ketoacidosis. There is also
between 120 and 180 m / dL. This has been ound to be some concern about an association with bladder cancer
superior to more intensive lycemic control. that requires urther post-market monitorin . I there is
a “line” or these dru s, it’s third-line.

He does well nd is re dy or disch rge. He sks bout sel -


onitoring o blood glucose. T e tient’s di betes is not o ti lly controlled nd he
develo s di betic neuro thy ye rs l ter.
Question 10.10.18 How o ten should a type 2 diabetic
on oral hypoglycemic agents measure his or her blood Question 10.10.19 Which o the ollowing agents is LEAST
glucose? e ective in treatment o diabetic neuropathic pain?
A) Once or twice week, t v rying ti es during the d y. A) ricyclic ntide ress nts.
B) Four ti es d ily—be ore e ls nd t bedti e. B) SSRIs.
C) wice d ily— sting nd 2 hours er e l. C) Duloxetine (Cy b lt ).
D) Once or twice d ily— sting nd be ore e l. D) Preg b lin (Lyric ) or g b entin.
E) Routine blood sug rs re not indic ted on d ily b sis or E) Venl xine (E exor XR).
ty e 2 di betics.
Answer 10.10.19 The correct answer is “B.” Sever l drugs h ve
Answer 10.10.18 The correct answer is “E.” D ily e sure- been roved s eci c lly or relie o di betic eri her l neu-
ents o nger stick sug rs in tients on or l hy oglyce ic ro thic in in the United St tes ( reg b lin, duloxetine, nd
gents do nothing to i rove glyce ic control (BMJ. 2012 t ent dol), but none ords co lete relie , even when used in
Feb 27;344:e486). In these tients, we re not re cting to d ily co bin tion. Venl xine, itri tyline, g b entin, v l ro te,
f uctu tions in glucose control but r ther king ch nges nd other o ioids ( or hine sul te, tr dol, controlled rele se
in res onse to the HbA1c. Occ sion l r ndo sug rs re not oxycodone) y be e ective nd y be considered or tre t ent
unre son ble to get gener l ide bout glyce ic control. y e o in ul di betic eri her l neuro thy. Low-dose tricyclic
2 di betics on insulin ( nd ll ty e 1 di betics) should e - ntide ress nts re the ost e ective o this grou (e.g., nor-
sure their blood glucose t le st d ily, nd ide lly two or three tri tyline or itri tyline 25 g qhs) but lso h ve nticho-
ti es er d y, reg rdless o the resence or bsence o sy - linergic side e ects (Lancet Neurol. 2015 Feb; 14:162). ricyclics
to s. T e tient h s co e b ck to your clinic sever l ti es nd g b entin re dditive, so consider using the together i
or ollow-u , nd his current edic tions including et- needed). Re e ber th t g b entin c uses weight g in, so e-
or in nd glyburide re djusted to their xi u doses. thing these tients don’t need. For the tre t ent o neuro thic
T e tient is not interested in inject ble edic tion—he in, CA>strong o i tes>tr dol>SNRIs> reg b lin>g b
ch nged his tune ro e rlier f er re ding online how he entin. SSRIs re gener lly ine ective or neuro thic in.
ight get ddicted to insulin. You discuss dding third or l
gent, nd uster n ex ression th t belies enthusi s .
HELPFUL TIP:
Well LA-DI-DA (well, LADA, anyway). LADA stands or la-
HELPFUL TIP: tent autoimmune diabetes in adults. This is a patient who
THE NEW KID ON THE BLOCK: Another class o dia- clinically starts as a patient with DM2 but also has circu-
betic medication is the sodium- lucose co-transporter latin anti-islet cell and/or anti-g AD antibodies. As some
2 (Sg LT2) inhibitors, which include cana li lozin point in their disease, these adult patients lose enou h
CHAPTER 10 • ENDOCRINOLOg Y 297

Answer 10.11.2 The correct answer is “A.” Diuretics tend to


pancreas to convert to a type 1 diabetic. These are patients
incre se the blood sug r bit. As irin, indo eth cin, ACE inhib-
who initially respond to noninsulin hypo lycemic a ents
itors, ent idine, nd ren l ilure y be ssoci ted with ost-
(met ormin, sul onylureas, etc.) but eventually no lon er
r ndi l hy oglyce i . Other edic tions th t re ssoci ted
respond to these dru s and become insulin dependent.
with hy erglyce i include quinolones, ty ic l nti sychot-
This may represent up to 10% o the DM 2 population.
ics, corticosteroids, c lcineurin inhibitors, rote se inhibitors,
nd so e li id-lowering gents like ni cin nd torv st tin. It is
Objectives: Did you learn to . . . worth noting th t c rvedilol nd nebivolol re not ssoci ted with
• Reco nize dia nostic criteria or diabetes? the develo ent o hy erglyce i , nd g tif ox cin is ssoci ted
• Di erentiate diabetes type 1 rom type 2? with both hy er- nd hy oglyce i . urns out th t the kidneys
re res onsible or bout 50% o gluconeogenesis. I the kidneys
• Evaluate a patient with new onset type 2 diabetes?
ren’t working, the bility to res ond to hy oglyce i is blunted.
• Identi y risk actors or complications o diabetes?
• Initiate and mana e oral therapy in type 2 diabetes? HELPFUL TIP:
• Start a type 2 diabetic patient on insulin? Hypo lycemia should be suspected in any patient with
• Mana e diabetes in the hospital settin ? diabetes who has ocal neurolo ic si ns (such as mi ht
• Treat o pain ul diabetic peripheral neuropathy? be seen in a CVA).

CASE 10.11 Question 10.11.3 You advise this patient to do all o the
A 32-ye r-old e le resents to your o ce co l ining o ollowing EXCEPT:
“hy oglyce i .” She notices th t bout 2 to 3 hours f er A) Incre se the ount o si le c rbohydr tes with her e ls.
e l she gets n use ted, sh ky, nd irrit ble. When she w kes B) Incre se the ount o co lex c rbohydr tes with her
u in the orning, she gener lly eels well even though she e ls.
e ts dinner t bout 5:00 pm nd does not e t ny sn cks f er- C) Incre se the ount o rotein with her e ls.
w rd nd gener lly does not h ve bre k st until 8:00 a m. D) Incre se the ount o t with her e ls.
E) E t s ller, ore requent e ls.
Question 10.11.1 You can tell her that:
A) She likely h s n insulino . Answer 10.11.3 The correct answer (and what you would
B) She likely will h ve nor l blood sug rs when she eels sh ky. not want to do) is “A.” Her roble is c used t le st in rt
C) Hy oglyce i does not exist s n entity in this or nd by high int ke o si le c rbohydr tes, le ding to r id nd
she likely h s nxiety. high e k in her blood sug r ollowed by n excessive rele se o
D) She likely h s “ sting” hy oglyce i . insulin. T us, one would w nt to decre se the ount o si -
E) Hy oglyce i is n unco on roble in nondi betics le c rbohydr tes in the diet nd dd rotein, t, nd co lex
but it could ex l in her sy to s. c rbohydr tes. T ere is little d t suggesting th t ny o these
is rticul rly e ective, however. Del ying g stric e tying
Answer 10.11.1 The correct answer is “E.” “A” is incorrect (using n gent like ro ntheline, n nticholinergic/ ntis s
bec use tient with n insulino should be hy oglyce ic odic) doesn’t see to ke di erence (but ight ke the
er 15-hour st (when she’s not e ting between 5:00 pm nd tient slee y enough th t she doesn’t notice the sy to s!).
8:00 a m). “B” is incorrect. T is tient y h ve ost r ndi l Reducing the s ike in c rbohydr te bsor tion using l h -
hy oglyce i th t occurs 2 to 4 hours er e ting. T e rocess glucosid se inhibitors (e.g., c rbose) kes sense but lso does
le ding to ost r ndi l hy oglyce i is s ollows: the tient not h ve ny good su orting d t or this indic tion.
h s l rge e l with si le c rbohydr tes, the seru insu-
lin level incre ses in res onse but overshoots, nd the tient INTERESTING (BUT MAYBE NOT SO HELPFUL) TIP:
beco es tr nsiently hy oglyce ic or 15 to 20 inutes usu lly g in and tonic and ackee ruit ( rom Caribbean islands)
2 to 4 hours er e ting. T is is ssoci ted with drenergic out- are known causes o hypo lycemia. In “ in and tonic hy-
ouring in n tte t to correct the roble . It is the drener- po lycemia” (the actual name o the syndrome), alcohol
gic out ouring th t c uses the sy to s o tre or, n use , etc. prevents e ective counter-re ulatory measures to the
“C” is incorrect bec use hy oglyce i does exist. “D” is incor- hypo lycemia produced by the quinine. As an aside
rect. T e tient does not h ve sy to s o sting hy ogly- ackee ruit, common in the Jamaican diet, contains “hy-
ce i , which occur 4 to 6 hours (or longer) er the l st e l. po lycin” that prevents luconeo enesis.
Question 10.11.2 All o the ollowing are associated with
postprandial hypoglycemia EXCEPT: HELPFUL TIP:
A) Diuretics. For insulinoma, watch the patient durin a controlled
B) Alcohol int ke. ast durin which the patient is observed and can
C) Postg strecto y syndro e. be treated or hypo lycemia i necessary. I you are
D) Bet -blockers.
298 FAMILY MEDICINE EXAMINATION & BOARD REVIEW

ll. T is is c using her gre t distress, s she const ntly worries


considerin sel -induced hypo lycemia with insulin
bout being regn nt. She does desire to h ve children “so e
(e. ., actitious hypo lycemia), measure the c-peptide.
d y,” but not now. She h s run ulti le ho e regn ncy tests,
This will be low i the patient is bein administered ex-
ll o which h ve been neg tive. L st week, she develo ed cle r
o enous insulin because pancreatic insulin production
le k ge ro her ni les, nd the tient is now convinced
will be shut o in response to hypo lycemia (remem-
she is regn nt nd th t the ho e regn ncy tests ust be
ber, however, that it will also be low in type 1 diabetes).
ulty. She requests th t you er or “ re l regn ncy test.”

Objectives: Did you learn to . . . Question 10.12.1 Which o the ollowing may cause her
• Evaluate a patient with possible hypo lycemia? amenorrhea?
• Treat a patient with postprandial hypo lycemia? A) E otion l stress.
B) Pregn ncy, des ite ulti le neg tive tests.
• Identi y causes o hypo lycemia?
C) T yroid dys unction.
D) Pituit ry tu or.
QUICK QUIZ: CARBOHYDRATES AND CALORIES E) All o the bove.

Which o the ollowing st te ents is FALSE? Answer 10.12.1 The correct answer is “E.” T e di erenti l
A) T e use o the glyce ic index (GI) nd lo d y hel di - di gnosis or enorrhe is r ther bro d, but includes ll o
betic tients i rove their c rbohydr te-cont ining ood the bove di gnoses. T e ost co on c use in wo n o
selection. childbe ring ge is, o course, regn ncy. Although urine-b sed
B) Di betics should void sucrose-cont ining oods bec use regn ncy tests h ve beco e very sensitive ( ble to detect s
they worsen glyce ic control ore th n co r ble little s 20 IU/ L o β -hCG), the tient y h ve been using
ount o st rch. the tests incorrectly nd thus w s getting lse neg tive results.
C) For weight loss, either low-c rbohydr te or low- t c lorie- Other c uses o enorrhe include hy othyroidis , strenuous
restricted diets y be e ective in the short ter (u to 1 ye r). exercise or norexi , e otion l stress, ituit ry tu or, cert in
D) Moder te lcohol int ke y reduce the risk o develo ing edic tions (e.g., henothi zines, do inergic gents, che-
di betes. other y, nd estrogens), nd ov ri n ilure or genesis. See
E) C rbohydr tes h ve ewer c lories er gr th n lcohol. Ch ter 15 or ore thorough discussion o enorrhe .

The correct (and alse) answer is “B.” Sever l studies de on- Further history ro the tient reve ls en rche t ge 12,
str te th t diet ry sucrose does not incre se glyce i ore th n no regn ncies, nd no revious history o enstru l irregu-
isoc loric ounts o st rch. For ex le, the glyce ic index l rities. She rtici tes in low-i ct exercise regul rly nd
o sucrose is 84 while th t o b n n is 88 (where higher is does not eng ge in long-dist nce running or other de nd-
worse). T e GI is nu ber ssoci ted with rticul r ty e o ing endur nce s orts. Review o syste s reve ls requent
ood th t indic tes the ood’s e ect on erson’s blood glucose ild he d ches, but no visu l disturb nces or sy to s o
level, ty ic lly r nges between 50 nd 100, where 100 re resents hy othyroidis .
the st nd rd, n equiv lent ount o ure glucose. A rel ted Physic l ex in tion de onstr tes well-develo ed,
e sure, the glyce ic lo d (GL) o ood is nu ber th t esti- well-nourished (neither obese nor excessively thin) dult
tes how uch the ood will r ise erson’s blood glucose e le. She h s ro ri te second ry sex ch r cteristics, no
level er e ting it. One unit o GL roxi tes the e ect o hirsutis , nd nor l thyroid gl nd to l tion. G l c-
consu ing one gr o glucose. Both GI nd GL re hel ul torrhe is noted on bre st ex in tion. T e elvic ex in -
in edic l nutrition l ther y or di betes. For “C,” lthough tion is nor l, with ro ri tely develo ed extern l genit -
low- t diets h ve tr dition lly been ro oted or weight loss, li , v gin , nd cervix. T e uterus is l ble nd s ll, nd
studies ound th t subjects on low-c rbohydr te diets lost ore the ov ries re neither l ble nor tender.
weight. Nonetheless, either low-c rbohydr te or low- t c lorie-
Question 10.12.2 In addition to a serum β -hCG and a TSH,
restricted diets re e ective or weight loss. Reg rding “D,”
what other test(s) should be ordered?
observ tion l studies re ort th t oder te lcohol int ke y
A) K ryoty e, to ev lu te or testicul r e iniz tion nd urner
reduce the risk or di betes, but the d t do not su ort reco -
syndro e.
ending lcohol consu tion to individu ls t risk o di betes.
B) Adren l MRI, to ev lu te or dren l hy er l si .
Did you know th t 1 g o lcohol h s 7 c lories while one g o
C) Prol ctin level.
c rbohydr tes only h s 4? Now you do!
D) All o the bove.
E) None o the bove.
CASE 10.12
A 24-ye r-old e le resents to the o ce co l ining o Answer 10.12.2 The correct answer is “C.” A rol ctin level
enorrhe . Six onths go, her enses bec e irregul r is n essenti l co onent o this ev lu tion. A rol ctino is
nd light. For the l st 4 onths, she h s not h d eriod t the ost co on or o ituit ry deno , nd it c n c use
CHAPTER 10 • ENDOCRINOLOg Y 299

second ry enorrhe , g l ctorrhe , nd in ertility (or i o- C) T is is cro deno , so edic l ther y is utile, nd
tence in en). Ex nsion o the ss in the sell turcic y the tient should be re erred or surgery.
c use he d ches or visu l eld de ects (bite or l he i no - D) T is is cro deno , but edic l ther y with do -
si ), but the tu ors re ore o en too s ll to h ve ny loc l ine gonist should still be tte ted.
e ects. Adren l hy er l si is unlikely in the tient bec use o E) T is is cro deno , which tends to be sel -li ited, so
l ck o virilizing ch r cteristics ro ndrogen excess (such s ther y c n be held or 6 onths when re e t sc n will be
hirsutis ). Also, dren l i ging is not usu lly the rst ste in done.
the di gnosis o this disorder (24 urine collection or cortisol nd
17-OHS is indic ted i dren l hy er unction is sus ected). A Answer 10.12.4 The correct answer is “D.” A ituit ry tu or
k ryoty e is not indic ted in this tient bec use she h s second- < 1 c in size is considered icro deno , nd tu ors 1 c
ry enorrhe , or enorrhe th t h s develo ed er eriod or gre ter in size re considered cro deno s. T e tre t-
o ti e o nor l enses. P tients who co l in o never h ving ent i lic tions re slightly di erent. In both c ses, edic l
enstru l cycle re considered to h ve ri ry enorrhe , ther y with do ine gonist (e.g., bro ocri tine, c bergo-
which y be evidence o either urner syndro e (XO geno- line) is indic ted. Success ul shrink ge o even cro deno s
ty e) or co lete ndrogen insensitivity ( k testicul r e ini- is ossible with this ther y. Re e ber th t the secretion o
z tion, which h s n XY genoty e with end-org n resist nce to rol ctin is under neg tive eedb ck loo . As CNS do ine
testosterone, resulting in e le henoty e). Both c ses result levels go u , rol ctin levels go down. When do ine levels go
in ov ri n genesis nd, there ore, no enstru l cycles. down, rol ctin levels go u .

You obt in l bor tory tests, nd the results re s ollows:


SH 3.1 IU/ L (0.27–4.20), β-hCG undetect ble, rol ctin HELPFUL TIP:
150 ng/ L (3.4–24.1). Many psychoactive medications inhibit dopamine and
can result in hyperprolactinemia. Resist the knee-jerk
Question 10.12.3 What is the best next step in this patient’s reaction o stoppin a psychoactive medication in a pa-
evaluation? tient ound to have hyperprolactinemia. Hi h prolactin
A) Re ssure tient th t stress is c using her l ck o eriods nd levels do not kill patients, but untreated mental illness
she will i rove when she le rns to de l with her li e. does. Risks, bene its, and likelihood that the dru is to
B) No ddition l tests t this ti e, but return in 2 weeks or blame must all be considered.
re e t rol ctin level.
C) Ad it the tient to the hos it l nd st rt bro ocri tine
ther y S A .
D) MRI br in to ev lu te or ituit ry ss. T e tient is st rted on bro ocri tine nd is scheduled or
E) Re er tient or neurosurgic l intervention. ollow-u in 3 weeks. She returns e rlier th n scheduled due
to severe n use nd lighthe dedness. No other new sy -
Answer 10.12.3 The correct answer is “D.” T is tient h s to s h ve occurred. T e tient’s vit l signs re nor l, but
high rol ctin level nd sy to s o rol ctin excess. In the the systolic blood ressure decre ses by 20 Hg nd the
bsence o edic tions c using n elev ted rol ctin ( heno- ulse incre ses by 20 be ts/ inute u on st nding.
thi zines, n rcotics, estrogens, etc.) nd nor l SH, this ro-
l ctin result is virtu lly di gnostic or rol ctino . I ging Question 10.12.5 Aside rom a bolus o IVf uids, how should
is indic ted reg rdless o visu l sy to s. A visu l eld ex i- you address this problem?
n tion by con ront tion is insensitive or inor visu l eld A) T is is co on side e ect ro bro ocri tine. Decre se
loss. I she h d ild elev tion in rol ctin (u to two ti es or sto the bro ocri tine, nd consider nother ty e o
the u er li it o nor l), nd no other sy to s/signs, then do ine gonist, such s c bergoline.
re e ting the level over sever l visits y be ro ri te. I the B) Ad it her to the hos it l nd rr nge or S A he d C
level re ins elev ted, i ging nd edic l ther y should sc n to rule out bleeding ro the ituit ry deno .
then be considered. Neurosurgic l intervention is not i edi- C) Re e t the regn ncy test.
tely indic ted, since tri l o edic l ther y is usu lly the rst D) She is h ving n n hyl ctic llergic re ction. Ad inister
ste in tre t ent. e ine hrine nd di henhydr ine i edi tely.
E) She h s iled edic l ther y nd ust be re erred or neu-
T is tient undergoes br in MRI, which shows 1.3 c rosurgic l intervention.
ituit ry ss.
Answer 10.12.5 The correct answer is “A.” T e ost co on
Question 10.12.4 How should this nding be interpreted side e ects o do ine gonists re n use , ostur l hy oten-
and managed? sion, nd di culty concentr ting. T ese sy to s tend to be
A) T is is icro deno , nd the result c n be ignored. lessened when lower doses re used nd the dose is incre sed
B) T is is icro deno , nd edic l ther y with do - very slowly. C bergoline tends to be better toler ted th n bro-
ine gonist is indic ted. ocri tine.
300 FAMILY MEDICINE EXAMINATION & BOARD REVIEW

Answer 10.12.7 The correct answer is “C.” Do ine gonists


T e tient is ble to toler te c bergoline, nd continues the
c n be sto ed t eno use. Follow-u c n be done using
edic tion or 6 onths. During this ti e, the rol ctin
blood levels o rol ctin. I rol ctin levels rise, n MRI c n be
level decre ses slowly. She h s h d resu tion o her enses.
done to see i the deno is beco ing l rger. I not, there is no
A re e t MRI is done f er 6 onths nd shows rked
re son to tre t the deno in ost eno us l tients. “A” is
decre se in size o the deno . T e tient h s re ched th t
incorrect bec use “C” is correct. And, this is si le test t king
gic l in ection oint in li e where she recognizes th t her
skill. I you see “ orever” on test it is rob bly wrong. “B” is
own ort lity is inevit ble, nd she desires to get regn nt
incorrect. If there is no deno noted on MRI t b seline,
“ s soon s ossible.” She w nts to know i she should h ve
tri l o o do ine gonists c n be tried t 1 ye r. er the
surgery to re ove the deno .
drug nd ollow the rol ctin. In those with n deno on
Question 10.12.6 What is/are the indication(s) or transs- MRI t b seline, tri l o o edic tions t 2 ye rs is re son ble
phenoidal pituitary surgery? provided the prolactin level has allen to normal and there no evi-
A) F ilure to res ond to do ine gonists. dence o adenoma by MRI in the past two years. I rol ctin levels
B) F ilure to toler te do ine gonists. re in under control, there is no need to continue edic tion.
C) re t ent o gi nt deno (> 3 c ) in tient who Objectives: Did you learn to . . .
desires regn ncy, des ite e c cy o edic l ther y. • g enerate a list o potential causes o secondary amenorrhea?
D) A nd B. • Evaluate a patient with secondary amenorrhea?
E) All o the bove. • Dia nose and treat hyperprolactinemia secondary to a pitu-
itary prolactinoma?
Answer 10.12.6 The correct answer is “E.” Surgery should
be reserved or those tients who il to res ond to or c n- CASE 10.13
not toler te edic l ther y. Note th t visu l eld de ects re
not s eci c indic tion or surgery, since edic l ther y c n A 39-ye r-old e le resents to the o ce co l ining o
be e ective in decre sing the size nd rel ted loc l e ects o enorrhe . She h s h d nor l enses until 8 onths go,
the tu or. I tient desires regn ncy nd h s l rge ss when they bec e in requent nd then sto ed. She insists
(> 3 c ), surgery y be considered s n djunct to edic l she c nnot be regn nt, bec use she h s not been sexu l
ther y. During regn ncy, there is hysiologic l incre se in ctivity “in ye rs.” She believes she is going through “the
the size o the ituit ry gl nd. I such tient beco es reg- ch nge” but w nts to know why she is re ching eno use t
n nt (without rior reduction in tu or size) nd discontin- uch e rlier ge th n other wo en she knows. On review
ues the do ine gonist or the dur tion o regn ncy, the o syste s, she co l ins o he d ches “ or ye rs” nd recent
deno y incre se to clinic lly i ort nt size be ore onset o we kness nd tigue. She lso co l ins o rthri-
delivery. tis in the hi nd knees, so ething she ttributes to “getting
old.” She re orts th t her h nds re swollen, nd her rings do
not t ny ore. She denies other co l ints.
HELPFUL TIP:
On hysic l ex in tion, vit ls re nor l. T e tient
Bromocriptine and caber oline are classi ied as cate-
is n dult e le o ver ge height, with notice bly l rge
ory B or pre nancy risk (in other words, they are
j w nd h nds. Her h ir is thick nd co rse, nd hirsutis is
enerally considered sa e). These medications can be
resent. Her thyroid gl nd is slightly enl rged, but regul r in
continued durin pre nancy, but a care ul risk–bene it
sh e. No bruit or tenderness is resent. T e oint o xi l
analysis and discussion between patient and physician
i ulse is dis l ced l ter lly, but the he rt rhyth is regul r
must occur be ore makin treatment decisions.
nd without ur ur. T e rest o the ex in tion is nor l.

Question 10.13.1 What is the most appropriate next step?


T e tient w nts to know how long she should be on c ber- A) Re ssure the tient th t eno use is nor l rocess
goline or bro ocri tine nd when tri l o o the edic - nd o er estrogen re l ce ent ther y or sy to tic
tion is indic ted. relie (but w rn the tient bout risks o long-ter use)
B) ell the tient you sus ect de ression nd o er regi en
Question 10.12.7 You tell her which o the ollowing? o counseling co bined with SSRI ther y
A) P tients with ituit ry deno h ve to be on edic tion C) ell the tient she y su er ro growth hor one (GH)
“ orever” to su ress the tu or. excess, nd reco end sending seru insulin-like growth
B) A tri l o o do ine gonist should be done t 6 onths. ctor-I (IGF-I) level
C) Prol ctin levels c n be llowed to rise er eno use with- D) ell the tient to quit being hy ochondri c nd to quit
out roble unless visu l sy to s or other loc l sy - re ding stu on the Internet
to s develo . So, she should continue the edic tion until
eno use. Answer 10.13.1 The correct answer is “C.” T is tient re -
D) None o the bove. resents cl ssic resent tion o cro eg ly due to GH excess,
CHAPTER 10 • ENDOCRINOLOg Y 301

nd the best single test or this is the IGF-I level. Although GH Objectives: Did you learn to . . .
levels will o en be elev ted, the IGF-I does not v ry ro hour • Reco nize si ns and symptoms o g H excess?
to hour nd is not de endent on ood int ke, s is the c se with • Evaluate and mana e a patient with g H excess?
GH. An elev ted GH er glucose lo d is lso very sugges-
tive o GH excess. Acro eg ly o dult onset ( er usion o
the long bones) does NO result in incre sed height, but does QUICK QUIZ: g ROWTH HORMONE
c use co rsening o ci l e tures, rogn this , nd thicken-
ing o the eet nd h nds. T ese ch nges c n be very subtle, nd A 6-ye r-old boy is brought into the o ce by his concerned
there is gener lly l g o 12 ye rs be ore di gnosis. Co r- other due to “growing too slowly.” She h s noticed th t he is
ing older hotogr hs o the tient to their current e r nce signi c ntly shorter th n his cl ss tes t school, nd she w nts
y be clue ( driver’s license hotogr h y be convenient so ething done bout it. She he rd re ort on the nightly news
source . . . lthough nobody looks good in their driver’s license bout new edic tion th t kes children grow t ller nd
hoto). P tients with cro eg ly lso develo hy ertro hy o insists on getting it or her son. Both rents re deluded nd
cert in org ns (such s the thyroid nd he rt) nd y resent ex ect their son to l y in the NBA or NFL nd believe he will
with he rt ilure due to c rdio yo thy. Eighty- ve ercent o not ke it unless he “gets lot t ller re lly soon.” She in or s
e les with cro eg ly h ve t le st so e enstru l dys unc- you in no uncert in ter s th t i you don’t rescribe this “grow-
tion nd 60% re enorrheic. Finally, arthropathy is common ing ill,” she will just nd so eone th t will.
in acromegaly and may be the overriding symptom that prompts
a patient to be evaluated. You believe this other is in need o so e educ tion bout
GH de ciency.
HELPFUL TIP:
Premature ovarian ailure is de ined as menopause at Wh t re the approved indic tions in children nd dolescents
a e 40 or youn er (two standard deviations below the or GH re l ce ent ther y?
mean). A) Growth hor one de ciency
B) Growth ilure due to chronic ren l insu ciency, intr uter-
ine growth ret rd tion with l ck o “c tch u ” growth by ge
T e tient’s IGF-I is elev ted, nd her SH is nor l. An 2, urner syndro e, nd Pr der–Willi syndro e
MRI is er or ed th t reve ls ituit ry ss slightly C) An dolescent le who wishes to l y college or ro b s-
< 1 c in di eter. ketb ll but is only 5 eet 9 inches on good d y
D) A nd B
Question 10.13.2 What is the most e ective therapy or this E) All o the bove
condition?
A) Weekly nti-IGF-I ntibody in usions. The correct answer is “D.” GH re l ce ent is s eci c lly -
B) Bro ocri tine ther y roved or GH de ciency nd growth ilure due to chronic re-
C) r nss henoid l ituit ry resection n l insu ciency, intr uterine growth ret rd tion nd no “c tch
D) So tost tin n logs (such s octreotide) u ” growth by ge 2, urner syndro e, nd Pr der–Willi syn-
E) Pegviso nt (GH rece tor nt gonist) dro e. It h s been roved or use in “idio thic short st ture”
( ore th n two st nd rd devi tions below the e n height or
Answer 10.13.2 The correct answer is “C.” Acro eg ly is ge). Not ll tients with GH de ciency require GH re l ce-
c used by GH-secreting ituit ry tu or. Surgery is the tre t- ent, bec use not ll will su er the neg tive end-org n e ects,
ent o choice or tients with icro deno (1 c or less such s rked short st ture, growth ilure, hy oglyce i in
in di eter) or or tients with cro deno th t e rs in ncy, nd centr l distribution o body t.
to be ully resect ble. So tost tin n logs nd egviso nt
(So vert®) y be use ul djuncts to surgery, nd re n o tion
HELPFUL TIP:
or tients who re not surgic l c ndid tes. Bro ocri tine is
As with acrome aly, Ig F-I should be checked when con-
not very e ective, nd only bout 10% o cro eg ly tients
siderin short stature secondary to pituitary ailure. It
will chieve nor l IGF-I levels with bro ocri tine; however,
is also important to check Ig F-bindin protein-3 levels.
c bergoline see s to work in bout h l o tients. C bergo-
Low levels o Ig F-bindin protein-3 may cause prob-
line h s n dv nt ge over so tost tin n logs in th t it c n be
lems with bindin o Ig F to the proper receptors.
t ken or lly. R di tion is lso n o tion or ther y, es eci lly
or those tients who re not surgic l c ndid tes nd do not
toler te or do not res ond to edic l ther y. “A,” nti-IGF-I HELPFUL TIP:
ntibody (i it existed s edic tion), would not h ve n e ect Even when used appropriately, g H only leads to a cou-
on GH-secreting tu or. But it does not exist. It’s de-u ple o inches increase in adult hei ht. Poo . . . . there o
nswer, so it’s wrong. By the w y, there’s no de-u stu on the the NBA dreams.
re l ex in tion . . . we think.
302 FAMILY MEDICINE EXAMINATION & BOARD REVIEW

Finklestein BS. E ect o growth hor one ther y on height


Clinical Pearls in children with idio thic short st ture: A et - n lysis.
Arch Pediatr Adolesc Med. 2002;156:230.
Do not order a thyroid ultrasound to routinely evaluate
abnormal thyroid unction tests. Fr ser WD. Hy erthyroidis . Lancet. 2009;374:145–158.
Grozinsky-Gl sberg S, et l. T yroxine-triiodothyronine
Do not prescribe/recommend multiple daily blood lucose
co bin tion ther y versus thyroxine onother y or
monitorin in patients on oral hypo lycemic re imens.
clinic l hy othyroidis : Met - n lysis o r ndo ized
Do not use bicarbonate to help correct the acidosis o DKA. controlled tri ls. J Clin Endocrinol Metab. 2006;91:2592.
The main intent is to provide IV luid hydration and insulin to H bib G, et l. Intr - rticul r ethyl rednisolone cet te
help brin down the su ars and correct the acidosis. injection t the knee joint nd the hy oth l ic-
Do not use diuretics to treat acute hypercalcemia. The ituit ry- dren l xis: A r ndo ized controlled study.
important point is to a ressively hydrate the patient. Clin Rheumatol. 2014;1:99–103.
Do not use T3 or hypothyroidism as its rapid astrointestinal Herrick B. Subclinic l hy othyroidis . Am Fam Physician.
(g I) absorption and relatively short hal -li e lead to erratic T3 2008;77:953.
serum levels. Ke rns AE. Medic l nd surgic l n ge ent o hy er r -
thyroidis . Mayo Clin Proc. 2002;77:87.
Initiate thyroid replacement at a low dose (e. ., 25 mc
levothyroxine) in the elderly (>65 years old) or those with Ki N. Ev lu tion o thyroid nodule. Otolaryngol Clin North
cardiac disease and slowly titrate upward every 4 Am. 2003;36:17.
to 8 weeks. Moghissi ES, et l. A eric n Associ tion o Clinic l Endocri-
nologists nd A eric n Di betes Associ tion consensus
Rely primarily on the TSH to screen or thyroid disease in
st te ent on in tient glyce ic control. Diabetes Care.
symptomatic individuals and as part o monitorin thyroid
2009;32:1119–1136.
unction in thyroid disease.
Montori VM, Fern ndèz-B lsells M. Glyce ic control in ty e
Screen all obese adults rom a es 40 to 70 or diabetes. 2 di betes: ti e or n evidence-b sed bout- ce? Ann
Start steroids immediately i you suspect an adrenal Intern Med. 2009;150:803–808.
insu iciency (addisonian) crisis. N th n DM, et l. Di betes: A consensus lgorith or the
Start with an ACE inhibitor (or an ARB i ACEI is contraindi- initi tion nd djust ent o ther y. Clinical Diabetes.
cated or has side e ects) in the treatment o proteinuria and 2009;27:4–16.
hypertension in patients with diabetes. Norris SL, et l. E ectiveness o sel - n ge ent tr ining
in ty e 2 di betes: A syste tic review o r ndo ized
controlled tri ls. Diabetes Care. 2001;24:561–587.
R o G. Insulin resist nce syndro e. Am Fam Physician.
BIBLIOGRAPHY 2001;63(6):1159–1163, 1165–1166.
A eric n Di betes Associ tion. Nutrition reco end tions Sherlock M, et l. Medic l ther y in cro eg ly. Nat Rev
nd interventions or di betes. Diabetes Care. 2008;31(S1): Endocrinol. 2011;7:291–300.
S61–S78. ritos NA, et l. Medsc e. M n ge ent o Cushing dise se.
A eric n Di betes Associ tion. St nd rds o edic l c re in Nat Rev Endocrinol. 2011;7:279–289.
di betes—2015. Diabetes Care. 2015;38:S1–S93. U.S. Preventive Services sk Force. Screening or gest tion l
Betterle C, Morlin L. Autoi une Addison’s dise se. Endocr di betes ellitus: U.S. Preventive Services sk Force
Dev. 2011;20:161–172. reco end tion st te ent. Ann Intern Med. 2008;148:
Bonor E, uo ilehto J. T e ros nd cons o di gnosing di - 759–765.
betes with A1 C. Diabetes Care. 2011;34Su l 2:S184–90. Unger J. L tent utoi une di betes in dults. Am Fam
C rroll MF. A r ctic l ro ch to hy erc lce i . Am Fam Physician. 2010;81:843.
Physician. 2003;67:1959. Y ss L, M rqusee E, F wcett R, Alex nder EK. T yroid
Findling JW. Di gnosis nd di erenti l di gnosis o Cushing’s hor one e rly djust ent in regn ncy (the HERAPY)
syndro e. Endocrinol Metab Clin North Am. 2001;30:729. tri l. J Clin Endocrinol Metab. 2010;7:3234–3241.
Rheumatology
Bharat K mar, Vijay Al ri, and Rebecca T etken
11
A ew words on “rheumatology panels.” Doing a “rheumatol- interphalangeal (PIP) joints, as well as mild erythema over
ogy panel” will never be the right answer. T e diagnosis o rheu- the MCP joints bilaterally. In addition, upon examination o
matologic disease is clinical with speci c clinical criteria or each the le knee, the bulge sign (indicating e usion) is detected.
illness. While antinuclear antibody (ANA), rheumatoid actor
(RF), erythrocyte sedimentation rate (ESR), C-reactive protein Question 11.1.1 I ound on physical examination, which
(CRP), and other tests may be use ul in supporting a clinical o the ollowing would be LEAST use ul in helping you in
diagnosis and assessing disease activity, these tests have poor narrow your diagnosis?
speci city and may be positive in a variety o disease states. A A) Bilateral metatarsophalangeal (M P) joint swelling and
positive ANA without a clinical diagnosis is meaningless. Like- tenderness.
wise, RF helps to gauge prognosis (seropositive vs. seronega- B) Painless oral ulcerations, with clean edges.
tive) in rheumatoid arthritis (RA), but has very limited value C) Firm, slightly tender subcutaneous nodules at the olecranon
as a diagnostic test. RF may also be positive in sarcoidosis, bursae.
viral in ections (especially hepatitis C), and autoimmune dis- D) A “bull’s eye” rash in the right axilla.
eases such as granulomatosis with polyangiitis ( ormerly known E) Icterus and tender hepatomegaly.
as Wegener granulomatosis), as well as a variety o primary
lung and liver diseases. ESR and CRP may support the clinical Answer 11.1.1 The correct answer is “A.” T is patient pres-
impression o in ammatory disease, but are again nonspeci c. ents with polyarticular in ammatory arthritis o unclear etiol-
ogy. While important to note, M P joint swelling would not
CASE 11.1 add much to the picture o subacute, symmetrical, small joint
polyarthritis that you have already ound on examination. T e
A 43-year-old emale presents with body aches and sti ness, di erential diagnosis includes acute viral arthritis, speci cally
which are worse in the morning. She urther describes a low- parvovirus B19 (due to the daughter’s history o acute illness
grade ever and pain in her hands, eet, and le knee. She resembling erythema in ectiosum), coxsackievirus, hepatitis
eels that her grip strength is diminished. T ese symptoms B (hinted at in “E”), and HIV. Also on the di erential will be
started rather abruptly 2 weeks ago and have not responded Lyme disease (“D”) (although small joint symmetrical arthritis
to acetaminophen. would be atypical), RA (“C,” the presence o rheumatoid nod-
She requently camps with her amily. She remembers that ules would be help ul although these would be unlikely in early
1 week they could not go because her 8-year-old daughter had disease), and other in ammatory disorders. “B,” painless ulcer-
a ever, mild diarrhea, abdominal pain, and a skin rash (“legs, ations, are classically associated with SLE (systemic lupus ery-
arms, and especially ace were red and warm, and she seemed thematosus), but it ought to be noted that lupus aphthae (ulcers)
‘ ushed’ all the time”). Her daughter’s symptoms resolved in may be pain ul as well.
a ew days, she did not see a doctor, and no one else was sick.
She has no other illnesses, and review o systems is otherwise A er you sneak o to do a little reading, you examine her
negative. again. She has no rash. You detect bilateral pain and swelling
On physical examination, her vitals are normal. She is o the third and ourth M P joints. T ere are no oral ulcer-
unable to close her hands completely. Although the physical ations and no lymphadenopathy. She is not icteric, and her
examination is somewhat limited by pain, there appears to abdomen is di usely, mildly tender. T ere is no hepatomeg-
be swelling o all metacarpophalangeal (MCP) and proximal aly. You decide to order some blood tests.

303
304 FAMILY MEDICINE EXAMINATION & BOARD REVIEW

TABLE 11-1 WHO LIKELY HAS RHEUMATOID Her laboratory results return as ollows:
ARTHRITIS?
• Hepatitis B: Sur ace antibody positive, sur ace antigen
Patients who: negative.
1. Have at least 1 joint with de nite clinical synovitis (swellin ) • CMV: IgG positive, IgM negative.
2. With the synovitis not better explained by another disease
• Parvovirus: IgG positive, IgM negative.
A. Joint involvement
1 lar e joint 0 • RF: 200 (normal: < 14 units)
2–10 lar e joints 1 • Anti-citrullinated protein antibody (ACPA)*:
1–3 small joints (with or witho t involvement o lar e joints) 2
4–10 small joints (with or witho t involvement o lar e 3
64 units (strong positive > 60 units)
joints) • ANA: Negative.
> 10 joints (at least 1 small joint) 5 • ESR: 58 mm/hr.
B. Serolo y (at least 1 test res lt is needed or classi cation)
Ne ative RF and ne ative ACPA 0
Note that based on results so ar and according to
Low-positive RF or low-positive ACPA 2 able 11-1, she has seven points (4–10 joints involved, high
Hi h-positive RF or hi h-positive ACPA 3 titers o RF and CCP antibody, and elevated ESR).
C. Ac te-phase reactants (at least 1 test res lt is needed or
classi cation) Question 11.1.3 Which o the ollowing is the most appro
Normal CRP and normal ESR 0 priate next step?
Abnormal CRP or abnormal ESR 1
A) Bilateral hand x-rays.
D. D ration o symptoms
< 6 weeks 0
B) C o the chest.
> 6 weeks 1 C) Smith antibody, double-stranded DNA (dsDNA), comple-
ment levels.
Diagnose rheumatoid arthritis if: Score o cate ories A–D is at least 6/10. D) Start methotrexate 15 mg weekly by mouth and daily olic
Data rom Aletaha D. 2010 Rhe matoid arthritis classi cation criteria: An acid, with or without low-dose prednisone and ollow up in
American Colle e o Rhe matolo y/E ropean Lea e A ainst Rhe matism
4 to 5 weeks.
collaborative initiative. Arthritis Rheum. 2010;62:2569–2581.
E) Start prednisone 60 mg daily by mouth and ollow up in 4 to
5 weeks.
Question 11.1.2 I positive, which o the ollowing tests
Answer 11.1.3 The correct answer is “D.” T is patient most
would be MOST help ul in ruling in a specif c rheumatologic
likely has seropositive RA (see criteria in able 11-1). T e patient
diagnosis?
should be started on a disease-modi ying anti-rheumatic drug
A) Positive ANA.
(DMARD), such as weekly methotrexate at a moderate dose,
B) Elevated white count.
along with olic acid 1 mg daily and prednisone 10 to 20 mg
C) Positive parvovirus B19 IgM.
daily. Evidence suggests that there is a “window o opportunity”
D) Positive urinalysis or white blood cells (WBCs).
when treating early RA and that an early remission may lead to
E) Elevated ESR and CRP.
sustained remission.
Note that investigating or any evidence o other systemic
Answer 11.1.2 The correct answer is “C.” T e presence o
involvement should be included as part o the initial evalua-
IgM antibodies to parvovirus B19—or rising titers o IgG
tion. It is appropriate to obtain CBC, liver unction tests, elec-
antibodies—indicates acute viral in ection, which may present
trolytes, BUN, creatinine, and urinalysis (to rule out glomeru-
with symptoms and signs seen in this patient. “A,” positive ANA,
lonephritis). “A” is incorrect. She is presenting airly early a er
will not help you rule in a diagnosis at this point. While the ANA
the onset o symptoms, so it is unlikely that hand x-rays will
is a highly sensitive test, it is not speci c and has a low positive
provide any signi cant ndings. “B” is also incorrect. Without
predictive value. “B,” “D,” and “E” are all important ndings, but
urther symptoms or signs o lung involvement (e.g., pulmo-
would not lead you toward a speci c diagnosis. “D,” WBC in
nary osteoarthropathy and respiratory symptoms), a C scan
the urine could be rom interstitial nephritis, a U I, a nephritic
would be inappropriate. As to “C,” she has no other symp-
urine rom other causes, etc. able 11-1 presents a ramework
toms o lupus and also had a negative ANA, so urther testing
or who should be tested or and diagnosed with RA.
or lupus (Smith antibody, dsDNA [also known as anti-native
DNA antibodies], complement levels) is not appropriate. “E” is
HELPFUL TIP: incorrect. Although steroids are indicated, low doses are ne;
Note that the dia nosis o RA no lon er req ires higher doses are rarely necessary and are associated with greater
6 weeks o symptoms, altho h a lon er d ration o
polyarthritis makes RA more likely. Similarly, it wo ld
be n s al or parvovir s B19 to ca se symptoms or *Anti-citrullinated protein antibody (ACPA) is also known as anti-
6 weeks. However, the vir s can ca se prolon ed joint CCP (anti-cyclic citrullinated peptides). Anti-CCP antibodies are very
pain in 10% o a ected ad lts. speci c or RA; however, their sensitivity is 67% and there ore may be
negative in RA.
CHAPTER 11 • RHEu MATOLOg Y 305

side e ects. T ere ore, early institution o a DMARD would be Answer 11.1.4 The correct answer is “D.” While important to
pre erable. note, limitation o passive movements o the knees is indica-
tive only o knee e usion (or pain), which you have already
HELPFUL (IF LONG) TIP: observed, and is not speci c or any particular etiology. She
In all cases o RA, and especially in those with a poorer responded modestly to methotrexate and prednisone but clearly
pro nosis, DMARDs sho ld be instit ted promptly, and still has active arthritis. What can you use to expand or limit
escalated airly rapidly, with oal bein disease remis- your di erential diagnosis? Pleural riction rub (“A”) is indica-
sion. Methotrexate is the most commonly sed DMARD tive o possible lupus. Diagnostic criteria or lupus include sero-
in the u nited States, b t combination therapy (metho- sitis, which may be detectable as a pleural rub on auscultation
trexate pl s hydroxychloroq ine or s l asalazine) or o the lungs (also, look or malar rash, discoid lesions, alope-
triple therapy (methotrexate, hydroxychloroq ine, and cia, and oral ulcerations). Although not part o the diagnostic
s l asalazine) improves o tcomes over methotrexate criteria or the disease, RA may also present with pleuritis or
alone. Low-dose prednisone is indicated or immediate pericarditis. Rheumatoid nodules (“B”) are ound in RA. A
symptomatic relie . Recent data s ests that low-dose salmon-colored, evanescent macular rash (“C”) would lead you
prednisone in the irst 6 months p to 2 years a ter dia - to consider adult-onset Still disease. Still disease, also known
nosis is associated with better pro nosis and more s s- as juvenile idiopathic arthritis (JIA, discussed in Chapter 13),
tained remission. Occ pational therapy re erral is help- presents with an evanescent rash, intermittent ever, and arthri-
l in identi yin and treatin nctional impairment d e tis. “Adult onset” Still disease is Still disease with onset a er age
to RA. Vitamin D 800 Iu /day and calci m 600 to 800 m 16. In addition, RF and CCP antibodies are typically negative
BID sho ld be initiated with prednisone therapy to help in adult-onset Still disease. A nding o isolated oot drop (“E”)
prevent corticosteroid-ind ced osteoporosis. Eval a- may be the result o mononeuritis multiplex, a eature o vascu-
tion o bone density (DEXA scan) and a bisphosphonate litides, and paraneoplastic syndromes. In act, oot drop is the
(e. ., alendronate) sho ld be considered i > 5 m o most common weakness with mononeuritis multiplex ollowed
prednisone is to be sed or > 3 months. by wrist drop. Mononeuritis multiplex is de ned as injury o
two or more named nerves secondary to a vasculitis.

HELPFUL TIP:
Question 11.1.5 Since your patient is taking methotrexate,
RA typically has an insidio s onset with a l ct atin
you caution her to avoid which o the ollowing?
co rse; however, a si ni icant minority o patients (per-
A) Aspirin.
haps one-third) will experience rapid onset, over days
B) Sul onamide antibiotics.
to weeks.
C) Ibupro en.
D) Folate.
E) Penicillin antibiotics.

She returns in 4 weeks and is now about 7 weeks into her ill- Answer 11.1.5 The correct answer is “B.” Methotrexate is
ness. She reports a moderate response to your intervention a olate antagonist. Anti- olate medications, such as sul on-
(you started methotrexate 15 mg weekly with 1 mg olic acid amide antibiotics, must be avoided in patients taking metho-
daily, and prednisone 20 mg daily), but now she has 1 to trexate because the combination may result in pancytopenia.
2 hours o morning sti ness. She continues to complain o Supplemental olate, 1 mg daily, reduces the adverse e ects o
pain in her hands and eet, with poor grip. In act, she had methotrexate. Patients with RA are o en treated with aspirin
to take time o rom work during the last week. On exami- or NSAIDs in combination with methotrexate. Penicillin anti-
nation, she has persistent swelling o MCPs 2 to 5 bilaterally biotics can be administered sa ely with methotrexate, although
and M Ps 3 and 4 bilaterally. You also note swelling in the methotrexate levels may increase by 8% or so.
le wrist and both knees, but tenderness is reduced and there
is no erythema. HELPFUL TIP:
You take a step back and want to reassess the situation. Dr interaction pro rams o ten warn abo t concomi-
tant se o NSAIDs and methotrexate, as well as aspirin
Question 11.1.4 What examination nding is so general and methotrexate. These warnin s are most relevant to
that it would NOT help you support/reconsider your diag hi h-dose methotrexate sed to treat cancer, not the
nosis? lower doses sed or in lammatory arthritis.
A) Pleural rub auscultated on lung examination.
B) Firm, slightly tender subcutaneous nodules at the olecranon
bursae. At her rst return visit, she had only mild improvement
C) Faint pink rash over chest, which is not visible 15 minutes so you (rightly) added hydroxychloroquine (good job!).
later. Initial hand x-rays demonstrate mild periarticular osteo-
D) Reduced passive exion in le knee. penia. Liver unction tests, urinalysis, CBC, BUN, and cre-
E) Le oot drop. atinine are normal. She returns 6 weeks a er starting the
306 FAMILY MEDICINE EXAMINATION & BOARD REVIEW

hydroxychloroquine and is much improved, having returned Answer 11.1.7 The correct answer is “A.” Despite her initial
to work ull-time. She tells you that she still has problems response, she has evidence o ongoing in ammatory activity by
with opening jars and about 45 minutes o morning sti ness, history and examination. Discontinuing or reducing medica-
“but nothing like it was.” tion is inappropriate. According to published guidelines, con-
sultation with a rheumatologist is now indicated—i it had not
Question 11.1.6 What is the BEST course o action to ollow been sought sooner. She has had a air initial response to meth-
now? otrexate, prednisone, and hydroxychloroquine. Further bene t
A) Continue her current therapy and ollow up in 6 to may be gained with increasing the methotrexate dose. Addition
12 months with transaminases, RF, and hand x-rays. o sul asalazine would also be appropriate. I “triple” therapy
B) Continue her current therapy and ollow up in 3 to 4 months with methotrexate, hydroxychloroquine, and sul asalazine
with transaminases, RF, and hand x-rays. ails, she will need a biological agent. “B” is incorrect: since she
C) Continue current therapy and ollow up in 3 to 4 months; had an initial response to methotrexate, and may show urther
arrange or monthly BUN, creatinine and CBC; and sched- ef cacy at a higher dose, it would be wise to urther increase
ule or an annual ophthalmology examination. the methotrexate dose, rather than substituting another agent
D) Begin prednisone taper, continue other medications, and (le unomide . . . le unomide is a nonbiologic immunosuppres-
arrange or monthly transaminases and CBC; schedule or sant which can cause liver injury). “C” is incorrect: doses o
baseline ophthalmology exam; and schedule ollow-up in prednisone this high are not indicated or RA.
another 2 to 3 months.
E) Instruct her to discontinue methotrexate, taper the pred- Question 11.1.8 This patient wants to become pregnant.
nisone dose, and continue hydroxychloroquine; arrange You can tell her that:
ollow-up in 1 year. A) Symptoms remit in 70% o women during pregnancy.
Answer 11.1.6 The correct answer is “D.” She seems to be B) She should avoid pregnancy while taking methotrexate.
responding to therapy, and a 3-month trial on her current medi- C) RA is a contraindication to pregnancy.
cations (during which the methotrexate dose may be increased) D) Prednisone cannot be taken during pregnancy.
is indicated. A slow prednisone taper should be initiated to min- E) A and B.
imize the cumulative dose o corticosteroids. Close ollow-up is
critical; she should contact you should symptoms return dur- Answer 11.1.8 The correct answer is “E,” both “A” and “B” are
ing the steroid taper. Guidelines or monitoring her DMARD correct. RA is an autoimmune disease, and it tends to remit
regimen recommend measurement o transaminases and CBC during pregnancy when a woman is relatively immunosup-
every 8 to 12 weeks or methotrexate and routine eye examina- pressed. Methotrexate is class X or pregnancy and is actually
tion to assess or hydroxychloroquine-related retinal toxicity. A used in ectopic pregnancy to arrest etal growth. Women taking
baseline eye examination is necessary when starting hydroxy- methotrexate should use an e ective orm o contraception, and
chloroquine; annual screening should begin a er 5 years, unless continue contraception or 3 months a er stopping methotrex-
the patient is at high-risk or complications. Hand x-rays are ate. “D” is incorrect since prednisone is o en used to control RA
recommended at 2-year intervals. “E” is incorrect: DMARD during pregnancy, when methotrexate is contraindicated. Pred-
therapy reduces her risk o joint destruction and disease pro- nisone does not cross the placenta, but use during pregnancy is
gression, and should not be discontinued. associated with higher risk or gestational diabetes.

At her next visit 3 months later, she eels better. Although she still T e patient is an avid runner and, prior to her diagnosis, par-
has dif culty opening jars, she now has < 30 minutes o morn- ticipated in numerous outdoor activities. She is concerned
ing sti ness and almost no pain. On examination, she has no about whether she will eventually become disabled.
rash, nodules, or evidence o serositis. She now has swelling over
MCPs 2 to 4 on the right and 2 to 3 on the le . Her grip is still Question 11.1.9 You can let her know that:
somewhat weak but improved. Laboratory data shows an ESR A) RA tends to progress without any remissions to involve
o 28 mm/hr, CRP 0.7 mg/dL, and normal transaminases and almost all joints in all patients.
CBC. Her symptoms increased when she tapered prednisone to B) Prednisone- ree disease remission has become the goal o
less than 10 mg/day, so she is taking 20 mg/day once again. treatment.
C) Patients with RA have the same li e expectancy as the gen-
Question 11.1.7 Which o the ollowing is the most appro eral public.
priate next step? D) Renal involvement is common with RA and is a major
A) Increase methotrexate to 25 mg weekly, continue hydroxy- source o morbidity and mortality.
chloroquine, and re er to rheumatology. E) She won’t need to worry about having a li e a er she gets
B) Stop methotrexate and switch to le unomide 20 mg/day. pregnant and has a child. All her energy will be absorbed by
C) Increase prednisone to 60 mg daily. that little parasite . . . er, child.
D) Discontinue all medications except methotrexate.
E) Discontinue methotrexate, taper prednisone, and continue Answer 11.1.9 The correct answer is “B.” With the advent
hydroxychloroquine. o new potent disease-modi ying agents, it is now possible to
CHAPTER 11 • RHEu MATOLOg Y 307

achieve remission, that is, the absence or near absence o joint


pain and swelling, in patients who have RA. “A” is incorrect. QUICK QUIZ: AN ILL CHILD
Untreated RA may progress to involve urther joints, but not
all joints are a ected by RA. Classically, the distal interphalan- A concerned mother brings in her 2-year-old son with a 1-week
geal joints are spared and, i there is DIP involvement, should history o ever. She is worried because she had expected the
prompt reassessment o the diagnosis. Moreover, treatment ever to resolve by now. According to his mother, the patient
with DMARDs can, and requently does, induce remission. “C” also has a rash, poor appetite, and lethargy. On examination,
is incorrect. RA reduces the li e expectancy by up to 10 years. he looks ill and his temperature is 39.0°C. T ere is a di use,
“D” is incorrect. Renal disease is a rare complication o RA; erythematous, macular rash, and peeling skin on the nger-
however, it can be a result o some o the medications used to tips. T e oropharynx is injected and the tongue is bright red
treat RA. Finally, “E”. I she needs a li e, she can buy them in with white papillae. Cervical lymph nodes are enlarged and
bulk or cheap at Costco (Walmart also has them or cheap but tender.
they wear out too ast).
Based on the available in ormation, what is your leading diag-
nosis?
QUICK QUIZ: BIOLOg ICAL Ag ENTS A) Rheumatic ever.
B) Parvovirus B19 in ection.
Which o the ollowing must be tested or be ore beginning a C) Kawasaki syndrome.
biological agent to treat rheumatoid arthritis? D) JIA.
A) CMV E) Varicella in ection.
B) B
C) Heart ailure The correct answer is “C.” Kawasaki syndrome is an acute vas-
D) Mucosal candidiasis culitis o unknown etiology, which is most o en seen in chil-
E) B and C dren. Kawasaki syndrome presents with at least 5 days o ever
(you cannot make the diagnosis o Kawasaki disease without
The correct answer is “E”. T ere are a number o biologicals at least 5 days o ever), polymorphous rash, conjunctival in-
or treating rheumatoid arthritis that are NF inhibitors. Etan- jection, mucous membrane involvement (e.g., “strawberry”
ercept (Enbrel) is a soluble NF receptor antagonist. In iximab tongue), cervical lymphadenopathy, and extremity ndings o
(Remicade) and adalimumab (Humira) are monoclonal anti- erythema and desquamation. T e usual treatment is aspirin and
bodies that bind NF both at the cell and when it is in its soluble IVIG. Corticosteroid therapy is controversial and does not seem
orm. T ese drugs can also reduce the resistance to in ection to improve outcomes. T ere may be cardiac involvement with
and promote malignancies. All o the NF inhibitors essentially the ormation o coronary artery aneurysms.
have the same ef cacy. Any one o these agents can be added “A,” rheumatic ever, which is rare in developed countries, is
to M X. T ings to watch or with NF inhibitors include seri- recognized by the Jones criteria. T e major Jones criteria con-
ous in ections including B, ungal in ections, and malignan- sist o polyarthritis, carditis, Sydenham chorea, erythema mar-
cies. Rule out B and hepatitis B be ore starting NF inhibitors. ginatum, and subcutaneous nodules. (Here’s a un mnemonic:
T ere is also a concern or worsening congestive heart ailure in “JONES” with a heart shape in place o the “O,” so that J = joints,
those with pre-existing disease. T us, “E” is the correct answer. O = carditis, N = nodules, E = erythema marginatum, and S =
Mucocutaneous candidiasis is not a contraindication to FN Sydenham chorea). “B” is incorrect. Generally, children do not
inhibitors and routine testing or CMV is not recommended. appear this ill with parvovirus B19 in ection ( h disease). “D,”
JIA, would be unusual at such a young age, and is discussed in
Chapter 13. “E” is incorrect, as this is obviously not varicella.
HELPFUL TIP:
Does joint replacement work in RA? Yes. Remember,
however, that the maxim m li e span o an arti icial CASE 11.2
joint is abo t 15 years with c rrent technolo y. Th s, re- A 62-year-old male whom you have ollowed or hyperten-
placement sho ld not be ndertaken li htly in a yo n sion or several years presents with complaints o worsening
(or any) patient. atigue and aching in his back, shoulders, and neck. He notes
3 months o symptoms unresponsive to acetaminophen.
Further history reveals that your patient has experienced
Objectives: Did you learn to . . . sti ness o the neck and shoulders each morning or over
• Describe an appropriate dia nostic strate y or polyarthritis? 30 minutes. He occasionally has dif culty getting out o bed
• Reco nize the dia nostic criteria or RA? due to pain. Vital signs are within normal limits. T ere is no
• Develop a mana ement strate y or RA? evidence o synovitis o the hands, wrists, or elbows. Active
• Reco nize the importance o early DMARD therapy or RA? range o motion in the neck and shoulders is slow but ull.
• Identi y the ses and adverse e ects o medications sed to T ere is tenderness to palpation o the shoulders, upper back,
treat RA? and neck, but no apparent muscle atrophy.
308 FAMILY MEDICINE EXAMINATION & BOARD REVIEW

TABLE 11-2 GENERALLY ACCEPTED CRITERIA and the hips more than the neck. The essential pathol-
FOR DIAGNOSIS OF PMR o y is in lammation o the synovia, and m scles are not
• A e ≥ 50 years directly involved.
• Pain/achin or at least 1 month involvin 2 o the ollowin areas:
neck, sho lders/proximal arms, and pelvic irdle
• Hip pain or limited ran e o motion HELPFUL TIP:
• Mornin sti ness lon er than 45 min tes
• Elevated ac te phase reactant (ESR/CRP)
Patients with PMR o ten have a low- rade ever and a
• Excl sion o other potential ca ses o the symptoms incl din iant normocytic anemia.
cell arteritis, RA (i.e., ne ative RF or ACPA)

Adapted rom Das pta B, et al. 2012 provisional classi cation criteria or Question 11.2.2 The sensitivity o an elevated ESR in the
polymyal ia rhe matica. Arthritis Rheum. 2012;64(4):943–954. diagnosis o PMR and giant cell arteritis (GCA, AKA, tempo
ral arteritis) is:
Question 11.2.1 Which o the ollowing is the most appro A) 100%.
priate next step in the diagnosis o this illness? B) 85%.
A) Obtain an ESR and CRP. D) 50%.
B) Obtain a urinalysis. E) 25%.
C) Prescribe a diagnostic trial o corticosteroids.
D) Order a rheumatology panel, including ANA, uric acid, ESR, Answer 11.2.2 The correct answer is “B.” Up to 15% o patients
CRP, and RF. with PMR or GCA (a closely related disorder—keep reading)
E) Per orm shoulder radiograph. have a alse-negative ESR. Using ESR and CRP together is 97%
to 99% sensitive or GCA. Double alse negatives o ESR and
Answer 11.2.1 The correct answer is “A.” T is patient’s pre- CRP are uncommon, but do occur. T us, in the patient in whom
sentation is consistent with the diagnosis o polymyalgia GCA is suspected but in whom there is a normal ESR and/or
rheumatica (PMR). Elevations o ESR and/or CRP contribute CRP, biopsy is still recommended. In those suspected o PMR, a
urther evidence to such a diagnosis and are use ul in ollowing trial o corticosteroids is still recommended.
the treatment o PMR. While a urinalysis (“B”) may be impor-
tant in some rheumatologic illnesses (e.g., lupus, Behçet syn- HELPFUL TIP:
drome), PMR is not likely to be associated with renal disease. PMR is ncommon in non-white pop lations. The mean
A trial o corticosteroid therapy (“C”), may be appropriate, a e o onset is approximately 70 years. Women are
but an ESR should be obtained rst to more conclusively estab- a ected twice as o ten as men.
lish the diagnosis. At this point all we know is that he has bilat-
eral shoulder and neck pain, which could be mechanical rom
the cervical spine, etc. As you already know rom earlier dis-
You order radiographs o the neck, which demonstrate mild
cussion, “D” is incorrect; a “rheumatology panel” will typically
degenerative disease. A CBC is unremarkable, except or a
include tests that are not indicated, and positive results can be
mild thrombocytosis. T e ESR is 80 mm/hr. You relate these
misleading. In the absence o small joint symptoms or examina-
ndings to the patient and tell him that your presumptive
tion ndings, an RF is not indicated in this case. Likewise there is
diagnosis is PMR.
no history to suggest an ANA-related disorder. “E” is incorrect.
T is patient does not need shoulder radiographs. In a patient
Question 11.2.3 Which o the ollowing is the most appro
with bilateral shoulder pain and neck pain, a neck radiograph
priate initial treatment in this case?
may be more use ul than shoulder imaging. Neck radiographs
A) Naproxen 500 mg twice daily.
help to evaluate or cervical canal narrowing and degenerative
B) Prednisone 15 mg daily.
disc disease, which may result in pain and neurologic ndings
C) Aspirin 650 mg twice daily.
in the upper extremities. An MRI o the neck might be use ul i
D) Prednisone 50 mg daily.
cervical spine disc disease or a syrinx were suspected. Diagnos-
E) Re erral to physical therapy.
tic criteria or PMR are given in able 11-2. Small joint syno-
vitis, or pu y swelling o hands and eet, will occur in 20% o
Answer 11.2.3 The correct answer is “B.” Prednisone is the
patients with PMR; and in these cases it is appropriate to check
treatment o choice in PMR. Doses o prednisone ranging rom
RF and anti-CCP during your initial evaluation.
10 to 20 mg daily are usually suf cient to control the disease.
HELPFUL TIP: Higher doses (up to 30 mg/day) should be tried i there is no
Physical examination indin s o PMR are s btle. Active response in 1 to 2 weeks. I the patient ails to respond to 30 mg
ROM in the a ected areas is o ten limited by pain, b t or less o prednisone, the diagnosis o PMR should be recon-
passive ROM sho ld be ll. Stren th is intact. In eneral, sidered. “A” is incorrect. NSAIDs may provide some symptom-
PMR a ects sho lders to a reater de ree than hips, atic relie but are not the treatment or PMR. “C” is incorrect.
It remains controversial whether low dose aspirin, 81 mg/day
CHAPTER 11 • RHEu MATOLOg Y 309

may decrease the risk o vision loss in giant cell arteritis. C) Up to 50% o patients who initially have a success ul remis-
However, high-dose aspirin therapy without corticosteroids sion will experience a relapse while tapering prednisone.
is not recommended. “E” is incorrect. Since patients usually D) A relapse o PMR requires high-dose corticosteroids (pred-
respond quickly to corticosteroids, physical therapy is not nec- nisone 50 mg daily) or success ul treatment.
essary—although you could hardly be aulted or employing
physical therapy as part o your overall treatment approach. Answer 11.2.5 The correct answer is “C.” Relapses occur in
30% to 50% o patients a er induction o remission and should
be treated by resuming or increasing prednisone. Usually, suc-
HELPFUL TIP: cess ul treatment o a relapse requires increasing the predni-
Lookin at the lon -term o tcomes, initial low-dose sone dose by a ew milligrams. “A” is incorrect. Although the
therapy or PMR works better than hi h-dose therapy. pathogenesis o PMR is incompletely understood, it has ea-
Patients have ewer relapses and are spared some o the tures in common with vasculitides, including potential vascular
adverse e ects o hi h-dose corticosteroids. complications o GCA. However, PMR is not associated with
an increase in mortality. “B” is incorrect because PMR is a sel -
limited disease, and most patients recover within a ew months
You prescribe prednisone 15 mg daily, aspirin 81 mg daily, to a ew years. T us, patients require prednisone or 6 months to
and calcium and vitamin D supplementation. Your patient 2 years, but prednisone therapy is typically not li elong. Remote
presents or ollow-up 4 weeks later, reporting marked relapses o PMR a er prednisone has been success ully stopped
improvement. On examination, there is no joint in amma- are seen in about 20% o cases, and can occur up to years later.
tion, or muscle tenderness with range o motion exercises.
His ESR is 20 mm/hr. You believe that the patient’s disease is Your patient does well and is able to taper o prednisone
now in remission. over a year. welve months a er stopping corticosteroids,
he presents to the ED one night. His shoulder and neck pain
Question 11.2.4 Which o the ollowing is the most appro and sti ness have returned, as well as severe atigue and
priate next step in his management? subjective evers. He has lost 5 pounds over 2 weeks. He is
A) Discontinue prednisone and initiate naproxen. now experiencing requent le -sided headaches. Finally, he
B) Continue the current dose o prednisone or the next is most concerned about a new visual disturbance starting
12 months. today. He notes that he has a “hole” in his vision. On physical
C) Continue the current dose o prednisone or the next examination, there is a prominent, tender vessel palpable at
6 months. the le temporal area. Funduscopic examination o the le
D) aper prednisone by 1 to 2 mg every 2 weeks to reach the eye shows a pale disc with blurred margins. T e remainder o
minimum e ective dose. the neurologic examination is normal. T e ESR is 70 mm/hr.
E) aper prednisone by 5 mg over 2 weeks and then discon-
tinue the drug.
Question 11.2.6 Which o the ollowing is the most likely
Answer 11.2.4 The correct answer is “D.” Relapse o PMR diagnosis or the visual symptoms?
occurs more requently when corticosteroids are abruptly dis- A) Polymyalgia Rheumatica (PMR).
continued or tapered too quickly. However, due to complica- B) Stroke (CVA).
tions associated with corticosteroid therapy, the dose should be C) Giant cell arteritis (GCA).
reduced as soon as possible; there ore, maintaining prednisone D) Multiple sclerosis (MS).
15 mg daily or 6 to 12 months is inappropriate. T e usual rec- E) Acute angle-closure glaucoma.
ommendation is to reduce the dose o prednisone by 10% every
1 to 2 weeks until the minimum e ective dose is reached. While Answer 11.2.6 The correct answer is “C.” Many o the patient’s
tapering prednisone, the patient should be monitored with symptoms can be explained by PMR (“A”), but visual symptoms
an ESR and/or CRP every 2 to 4 weeks. I symptoms worsen, do not occur with this disease. GCA is a related diagnosis that
prednisone should be increased slightly to achieve symptomatic is commonly seen in conjunction with PMR. Most experts now
control. I the ESR increases to 40 mm/hr or greater and the agree that PMR and GCA are di erent presentations o the same
patient is asymptomatic, consider continuing the same dose o disease process. With the new symptoms o localized headache
prednisone until the ESR normalizes, then continue the taper. and tenderness o the temporal artery and the previously known
However, an isolated elevation in ESR without symptoms is not a ndings consistent with PMR, this patient now meets diag-
reason to increase prednisone. nostic criteria or GCA (see able 11-3). T e visual symptoms
described are typical o GCA and can occur acutely or chroni-
Question 11.2.5 Which o the ollowing is true regarding cally. As to the other answers, vision loss in multiple sclerosis is
the prognosis o PMR? attributable to optic neuritis, which is associated with pain and
A) PMR is associated with an increased risk o mortality. presents initially in a younger population. Acute angle-closure
B) Most patients with PMR will require corticosteroid therapy glaucoma is associated with eye pain and redness. T e lack o
or li e. other symptoms makes stroke less likely.
310 FAMILY MEDICINE EXAMINATION & BOARD REVIEW

TABLE 11-3 DIAGNOSTIC CRITERIA FOR GIANT One dark and stormy night, 3 AM in the ED—the weather
CELL ARTERITIS is cold and the co ee is colder—your patient presents with
tearing substernal chest pain radiating to his back. He is alert
Consider dia nosis i ANYo the ollowin are present:
but anxious and diaphoretic. His le radial pulse is dimin-
• A e ≥ 50 years at onset o symptoms ished compared to the right. His heart rate is 120 bpm, and
• New localized headache, jaw cla dication, vis al symptoms
• Temporal artery tenderness or decreased p lsation
his blood pressure is 92/56 mm Hg.
• ESR ≥ 50 mm/hr
• Temporal artery biopsy ndin s consistent with vasc litis Question 11.2.8 Which o the ollowing studies will con rm
the most likely diagnosis?
A) Chest radiograph.
HELPFUL TIP: B) Chest C .
The initial vis al loss in g CA is peripheral, while the vi- C) ECG.
sion loss in mac lar de eneration is initially central. D) Venous blood gas.
I yo think abo t it, this makes sense. g CA basically E) roponin- .
ca ses an anterior ischemic optic ne ropathy (AION)
secondary to involvement o the retinal artery by vas- Answer 11.2.8 The correct answer is “B.” Your patient’s symp-
c litis. The rther yo are rom the artery, the poorer toms are classic or a dissecting thoracic aortic aneurysm, which
the per sion. is o en mistaken or a myocardial in arction. T oracic aortic
aneurysm is a late complication o GCA; aortic aneurysms gen-
erally occur an average o 6 to 7 years a er the initial diagnosis
Question 11.2.7 Which o the ollowing is the most appro
o GCA. T oracic aortic aneurysms occur 17 times more o en
priate initial management o this patient?
in patients with GCA when compared to the general popula-
A) Withhold treatment or now and arrange or temporal artery
tion. T e diagnosis o thoracic aortic aneurysm is con rmed by
biopsy within 48 hours.
C scan o the chest, echocardiogram, or angiogram. While the
B) Re er to a neurologist as soon as possible.
other studies listed should be done, none o them are going to
C) Re er to an ophthalmologist as soon as possible.
make the diagnosis o a dissecting aneurysm or you.
D) Initiate prednisone 20 mg daily, and re er or temporal artery
biopsy.
E) Admit and administer methylprednisolone 1 g intravenously HELPFUL TIP:
(IV), and arrange or temporal artery biopsy. With the initiation o corticosteroids in PMR or g CA,
start calci m 1,200 to 1,500 m daily and vitamin D 400
Answer 11.2.7 The correct answer is “E.” When symptoms o to 800 Iu daily or osteoporosis prevention. Once the
vision loss occur, IV methylprednisolone 15 mg/kg/day (up to 1 g) dia nosis is con irmed, bone mineral density sho ld be
daily or 3 days, ollowed by prednisone 40 to 60 mg daily is meas red with a DEXA scan and bisphosphonates insti-
the standard o care. Compared to PMR, higher doses o corti- t ted on the basis o FRAX calc lated risk o ract re or
costeroids are necessary to treat GCA. In the absence o vision DEXA res lts (do yo see a pattern here o prophylaxis
loss, prednisone doses o 40 to 60 mg QD are usually required or osteoporosis when startin corticosteroids?).
to relieve symptoms. Consultation with ophthalmology is also
necessary in providing proper care to the patient. “A” is incor-
rect. You do not want to withhold treatment rom this patient Objectives: Did you learn to . . .
whose vision is at risk (see “help ul tip” below). “B” and “C” are • Describe the appropriate eval ation, incl din physical
examination and laboratory tests, o di se pain in the older
incorrect or the same reason. “D” is incorrect because a dose o
patient?
20 mg o prednisone is too low to be e ective in GCA.
• Reco nize the dia nostic criteria or PMR and g CA?
• Describe the appropriate mana ement, incl din medical
HELPFUL TIP: therapy, o PMR and g CA?
Temporal artery biopsy is vital to the acc rate dia nosis
• Identi y complications o PMR and g CA?
o g CA. Characteristic iant cell in lammation pathol-
o y can be seen or p to 4 weeks a ter initiatin hi h-
dose corticosteroids. However, corticosteroid therapy CASE 11.3
should never be delayed for fear of reducing the in- A 22-year-old graduate student presents to the ED on a Mon-
flammatory findings on the temporal artery biopsy. day night with an acutely swollen le knee. He admits to “wild
partying” over the weekend but only had “a couple o beers”
(that is the “college couple” . . . . 6 or 7). His knee was OK then.
Six years a er his diagnosis o GCA, your patient has experi- However, when he woke up this morning, he noticed the knee
enced several remissions and relapses. Although he has been was swollen and pain ul (so was his head, but that’s another
able to discontinue prednisone on occasion, he is now taking matter). By early a ernoon, he had dif culty bearing weight.
5 mg daily with good symptomatic control. He denies ever, but eels tired.
CHAPTER 11 • RHEu MATOLOg Y 311

He reports a history o JIA (juvenile idiopathic arthritis, and the swelling o the MCPs in particular may be a clue or
previously termed juvenile rheumatoid arthritis), and has had active RA. T e detection o a diastolic murmur (“D”) is very
ankle and knee swelling previously, but not to this degree. He signi cant, since diastolic murmurs are almost always patho-
took prednisone intermittently, as well as hydroxychloroquine logic in adults, and may represent in ective endocarditis.
and methotrexate, or his JIA until age 18. He then continued Finally, “E,” penile discharge, can result rom acute gonorrheal
on hydroxychloroquine until 8 months ago, when he stopped in ections.
it because he elt ne. He denies any other medical problems.
He smokes only when drinking—which happens way too o en. On examination, vital signs are signi cant only or tachycar-
dia and ever (HR: 112 bpm, emp: 38.2°C). He has no other
Question 11.3.1 What other in ormation rom the history swollen joints, rashes, heart murmurs, or penile discharge.
would be most help ul in establishing the diagnosis? You palpate a smooth, non-tender liver edge 2 cm below the
A) Sexual history, including sexual orientation, practices, and costal margin; it percusses to 15 cm. You note mild cervical
last contact. lymphadenopathy and whitish pharyngeal exudates.
B) Personal or amily history o gout or kidney stones.
C) History o IV drug use. Question 11.3.3 Which o the ollowing is the most appro
D) Family history o pseudogout. priate next step in the management o this patient?
E) A, B, and C. A) Order hepatitis serologies.
B) Order blood cultures, urine PCR or chlamydia and gonor-
Answer 11.3.1 The correct answer is “E.” While all o these rhea, and abdominal ultrasound.
points are important in the history, this patient is too young to C) Administer ce riaxone 1 g IV and inject the knee with
have pseudogout. While gout is unusual in young adults, there triamcinolone.
are uncommon syndromes o amilial hyperuricemia and early D) Per orm knee aspiration.
gout. Although there are several possible etiologies or this E) Prescribe prednisone 20 mg PO QD and arrange consulta-
patient’s presentation, septic arthritis should be considered rst tion with a rheumatologist.
and oremost due to its high morbidity and mortality. As such,
the history and examination should ocus on those clues that Answer 11.3.3 The correct answer is “D.” Did you get dis-
point toward an in ectious etiology and its source. T e clinician tracted by a big liver? I so, redirect your attention to the knee.
must also consider nonin ectious in ammatory arthropathies. T e single most important step in evaluating acute monoarthri-
IV drug abuse can lead to a septic joint as can gonorrhea. T us tis is joint aspiration, which will allow di erentiation between
“A” and “B” are important parts o the history. in ammatory and nonin ammatory disease. Although blood
cultures and urine PCR or chlamydia and gonorrhea should
Your patient is heterosexual and thinks he had intercourse also be sent in this case, obtaining these studies must not delay
Saturday night but admits that his memory is somewhat joint aspiration. Without determining whether the arthritis is
blurry (maybe he had more than the “couple” o beers he in ectious, it would be inappropriate—and potentially hazard-
claims). He denies a history o gout and IV drug use. He ous to the patient—to start treatment with corticosteroids. Joint
complains o poor sleep and eeling sti in the mornings and aspiration will provide a specimen or crystal examination.
evenings lately. “C” and “E” are incorrect because you would not want to give
corticosteroids—especially intra-articular corticosteroids—to
Question 11.3.2 What ndings on physical examination a patient with an in ected joint. Analysis o the synovial uid
would be LEAST help ul in determining the diagnosis? will aid in determining the appropriateness o treating with
A) A ew vesiculopustular lesions on the back, arms, and legs. antibiotics or anti-in ammatory medications. While you would
B) Swollen, tender, nonerythematous MCP joints. probably use empiric antibiotics (treat as septic until proven
C) Nontender hepatomegaly. otherwise), you will need to rst obtain cultures ( rom the knee
D) Diastolic murmur at the right sternal border. aspiration and blood).
E) Whitish discharge rom the tip o penis.

Answer 11.3.2 The correct answer is “C.” Although non- HELPFUL TIP:
tender hepatomegaly may indicate presence o liver disease, it CBC, ESR, and CRP are not se l in dia nosin a sep-
is unlikely to help identi y the etiology o this patient’s arthri- tic joint. While they may be somewhat sensitive, they
tis. Hepatitis B arthritis usually presents as a symmetric poly- are hopelessly non-speci ic. Yo have to tap the joint
arthritis, although it can be migratory or additive (sequential re ardless!
joints becoming involved without resolution in the initial
joints). Our patient has a single swollen, hot joint, which is
unlikely to occur as a result o hepatitis. “A” is help ul: a vesicu- You have obtained blood cultures and urine PCR or chla-
lopustular rash occurs in disseminated gonococcal in ections. mydia and gonorrhea. A metabolic pro le, CBC, and hepa-
“B” is also help ul: the presence o other swollen joints should titis B and C serologies are pending. Knee aspiration yields
prompt consideration o nonin ectious in ammatory arthritis, 45 cc o turbid, blood-tinged uid.
312 FAMILY MEDICINE EXAMINATION & BOARD REVIEW

Question 11.3.4 You send the synovial uid or all so the the patient must be counseled regarding sa e sexual practices
ollowing studies EXCEPT: (e.g., condom use, reduced alcohol use, become a monk . . . .).
A) Cell count and di erential. In this setting, the other studies are less relevant to his overall
B) Crystal analysis. health.
C) Culture.
D) Glucose and protein.
HELPFUL TIP:
E) Gram stain.
g onococc s is c lt red rom the joint l id only abo t
50% o the time in patients with onococcal arthritis.
Answer 11.3.4 The correct answer is “D.” In contrast to analy-
Th s, rine and joint l id PCR sho ld be done even i
sis o some other bodily uids (e.g., cerebrospinal uid [CSF],
the g ram stain is ne ative.
pleural uid, and ascitic uid), chemistry analysis on synovial
uid is o little diagnostic value. Low glucose levels in synovial
uid are associated with the degree o in ammation but do not Question 11.3.7 Which o the ollowing is the most appro
help to determine its cause. Likewise, synovial protein levels do priate treatment plan or this patient?
not help di erentiate between types o arthritis. Cell count with A) Ce riaxone 1 g IV once, ollowed by ce xime 400 mg twice
di erential, gram stain, and cultures should be routine when daily by mouth or 14 days. Follow-up in 7 days.
suspecting in ection; crystal analysis is also part o the stan- B) Admit to hospital, administer ce riaxone 1 g IV daily, and
dard examination, but positive crystal analysis does not rule out per orm repeat knee aspirations.
concomitant in ection. C) Admit to the hospital and administer IV and intra-articular
ce riaxone 1 g daily.
T e synovial uid analysis reveals the ollowing ndings: D) Cipro oxacin 500 mg by mouth twice daily or 14 days.
50,000 WBC/mm3, 95% polymorphonuclear cells, and no Follow-up in 7 days.
crystals. Gram stain shows Gram-negative diplococci. Cul- E) Penicillin G 4 million units IV once, ollowed by amoxicillin
tures are pending. Gonorrhea and chlamydia PCR tests are 500 mg PO three times daily or 14 days. Follow-up with a
also pending. rheumatologist.

Question 11.3.5 What is the sensitivity o a synovial uid Answer 11.3.7 The correct answer is “B.” In order to ensure
white count o 100,000/mm 3? the best outcome, this patient should be admitted or monitor-
A) 30% ing and repeated joint aspiration. Purulent uid tends to col-
B) 40% lect rapidly in the joint spaces in patients with septic arthri-
C) 50% tis, necessitating requent drainage until antibiotics work and
D) 60% in ammation begins to subside. Most cases o gonococcal
E) 75% arthritis respond to needle aspiration, but arthroscopic or open
debridement is occasionally necessary. Because IV antibiotics
Answer 11.3.5 The correct answer is “A.” Un ortunately a have good penetration into synovial uid, intra-articular antibi-
synovial white count o 100,000/mm 3 is only 30% sensitive or a otics are not recommended. When culture, PCR, and sensitivity
septic joint but is 99% speci c. 50,000 WBC/mm 3 is 62% sensi- results become available, antibiotic therapy should be tailored to
tive and 92% speci c while a WBC count o 25,000/mm 3 is 75% the particular in ectious agent and its susceptibilities.
sensitive and 73% speci c. T e point is that what we were taught
in school about the synovial WBC count being > 100,000/mm 3 HELPFUL TIP:
in a septic joint is wrong. The initial antibiotic o choice in onococcal arthritis
is ce triaxone, administered IV. Spectinomycin IV is an
Question 11.3.6 Which o the ollowing studies will be most acceptable alternative when ce triaxone is contraindi-
important to the OVERALL care o this patient? cated. Remember that there is now l oroq inolone-
A) HIV testing and RPR. resistant onococc s. Beca se o this, l oroq inolones
B) Chest x-ray. are no lon er recommended as treatment o onorrhea.
C) ANA and RF.
D) Uric acid.
E) ESR and CRP. Within 48 hours, your patient shows signs o improvement.
His knee appears much better, there is no recurrent e usion,
Answer 11.3.6 The correct answer is “A.” His presentation is and he is a ebrile. He wants to leave the hospital. By the way,
very suggestive o disseminated gonococcal in ection with acute his chlamydia PCR turned up positive.
arthritis, and the presence o diplococci is virtually diagnostic.
T ere ore, the clinician must also consider the presence o other Question 11.3.8 Which o the ollowing management strate
sexually transmitted diseases and screen the patient appropri- gies do you recommend?
ately. Assays or hepatitis B and C and chlamydia have been sent, A) Continue the hospital admission and ce riaxone 1 g IV
and tests or HIV and syphilis should now be per ormed. Also, daily.
CHAPTER 11 • RHEu MATOLOg Y 313

B) Discharge with cipro oxacin 500 mg by mouth twice daily. drinks about a case o beer per week—unless he’s been party-
C) Discharge with penicillin V 500 mg by mouth three times ing, in which case he doubles his e ort. His amily history is
daily. remarkable or osteoarthritis.
D) Discharge with ce xime 400 mg by mouth twice daily, and Physical examination reveals an uncom ortable-appearing
doxycycline 100 mg by mouth twice daily. obese male in no acute distress. His temperature is 37.9°C,
blood pressure 168/98 mm Hg, and pulse 84 bpm. T e le
Answer 11.3.8 The correct answer is “D.” Since the patient is knee is red, warm, and di usely tender with a palpable e u-
improving, continued hospitalization and IV antibiotics are not sion. T ere are no visible or palpable subcutaneous nodules
needed. T us, “A” is incorrect. Without knowing the antibiotic on the ngers, toes, or at the elbows.
sensitivities o the gonococcus, you should assume that it is
penicillin-resistant, making “C” a poor choice. Once local and Question 11.4.1 Which o the ollowing is the most appro
systemic signs are resolving, you can sa ely discharge the patient priate next step to accurately diagnose this condition?
with oral antibiotic therapy, using ce xime 400 mg twice daily A) Radiograph o the a ected knee.
(or an acceptable alternative based on culture and susceptibili- B) CBC.
ties) to complete a 7 to 14 day course. In cases o gonococcal C) Serum uric acid level.
in ection, you should always presumptively treat or concurrent D) Knee aspiration and synovial uid analysis.
chlamydia in ection. E) Diagnostic corticosteroid injection.

Answer 11.4.1 The correct answer is “D.” We don’t mean to


HELPFUL TIP: sound like a broken record, but the diagnostic study o choice in
Septic arthritis occ rs most o ten in lar e joints, s ch a monoarthritis is synovial uid analysis. Synovial uid analysis
as the knee and hip. Factors that predispose a patient allows the clinician to determine whether there is an in amma-
to septic arthritis incl de advancin a e (especially tory, in ectious, or crystalline cause o the arthritis. “A” is incor-
> 80 years), RA, joint prostheses, recent joint s r ery, rect. Radiographs are typically not help ul acutely in in am-
diabetes, and skin in ection. matory arthritis (but would be indicated i there was trauma or
suspicion o tumor). “B” and “C,” a CBC is nonspeci c and uric
acid may be normal during an acute attack o gout. Neither o
these laboratory results will be diagnostic. Finally, corticoste-
Question 11.3.9 What is the mortality rate o septic arthritis?
roid injection must be avoided in monoarthritis until the pos-
A) 0.5%
sibility o in ection is eliminated.
B) 5%
C) 10%
You success ully aspirate 5 cc o slightly cloudy yellow syno-
D) > 15%
vial uid rom the le knee. While the patient is waiting, the
laboratory reports the ollowing ndings: 5,000 WBC/mm3,
Answer 11.3.9 The correct answer is “C.” T e mortality rate o
Gram stain negative or bacteria, many needle-shaped nega-
septic arthritis is 10%, with up to one-third o survivors having
tively bire ringent crystals are noted.
persistent joint problems, such as limited range o motion, pain,
and swelling. Note that the mortality is probably not due to the
Question 11.4.2 These synovial uid ndings are most
in ection alone but rather to a combination o the underlying
consistent with which o the ollowing diagnoses?
illness (e.g., immunosuppression) plus the in ection.
A) Osteoarthritis.
Objectives: Did you learn to . . . B) Septic arthritis.
• Describe the appropriate eval ation o monoarthritis? C) CPPD (“pseudogout”).
• Appropriately mana e a patient with septic arthritis? D) Gout.
• Identi y risk actors or septic arthritis?
• Reco nize the pro nosis o septic arthritis? Answer 11.4.2 The correct answer is “D.” Monosodium urate
crystals o gout are needle-shaped as seen in this patient’s syno-
vial uid (a good way to remember this is that being stuck with
CASE 11.4 a needle hurts and so does gout). Calcium pyrophosphate dihy-
A 55-year-old male presents to your of ce complaining o drate crystals are rod-, square-, or rhomboid-shaped and posi-
severe le knee pain o 2 days duration. Although he was also tively bire ringent in polarized light. T us, the synovial uid
out partying over the weekend (is there a pattern here to the ndings given above are most consistent with gout. “A” and “B”
patients in our practice?), he went home early. He denies any are incorrect. Normally, synovial uid contains < 180 WBC/
previous history o knee pain or arthritis. He has elt everish mm 3, but it is generally considered nonin ammatory i the
over the last 2 days. He recalls a similar episode o pain in his WBC count is still < 2,000/mm 3. Low WBC counts are seen in
right great toe 2 years be ore, but the pain resolved in a ew the synovial uid o osteoarthritic joints. Synovial uid contain-
days and he did not seek medical attention. He has hyperten- ing ≥ 2,000 WBC/mm 3 is consistent with an in ammatory pro-
sion treated with chlorthalidone but is otherwise healthy. He cess. When there are > 100,000 WBC/mm 3, the monoarthritis is
314 FAMILY MEDICINE EXAMINATION & BOARD REVIEW

TABLE 11-4 DIAGNOSTIC CRITERIA FOR ACUTE GOUT Question 11.4.4 Which o the ollowing is the next step in
the management o this patient?
The presence o characteristic rate crystals in the joint f id or toph s
A) Prescribe allopurinol.
proved to contain rate crystals by chemical means or polarized li ht
microscopy or the presence o 6 o the ollowin 12 phenomena: B) Prescribe acetaminophen.
1. More than 1 attack o ac te arthritis
C) Discontinue chlorthalidone.
2. Maximal inf ammation developed within 1 day D) Prescribe naproxen at a ull anti-in ammatory dose.
3. Attack o monoartic lar arthritis E) Per orm a therapeutic joint aspiration.
4. Joint redness observed
5. First MTP joint pain l or swollen Answer 11.4.4 The correct answer is “D.” An acute attack o gout
6. u nilateral attack involvin rst MTP joint should rst be treated with NSAIDs, such as naproxen or indo-
7. u nilateral attack involvin tarsal joint methacin. T e doses prescribed should be at the upper limit or
8. S spected toph s
9. Hyper ricemia
the particular NSAID (e.g., naproxen 500 mg three times daily).
10. Asymmetric swellin within a joint (radio raph) Earlier treatment is associated with greater relie o symptoms
11. S bcortical cysts witho t erosion (radio raph) and shorter duration o the acute event. Other potential rst-line
12. Ne ative joint f id c lt re or microor anisms d rin attack o agents are colchicine and corticosteroids. T e e ectiveness o col-
joint inf ammation chicine is best i started within 36 hours o onset o symptoms.
Colchicine is typically prescribed as one dose o 1.2 mg, ollowed
Data rom Wallace SL, Robinson H, Masi AT, Decker JL, McCarty DJ, Y TF.
Preliminary criteria or the classi cation o the ac te arthritis o primary by 0.6 mg twice daily or 1 to 2 weeks, and then 0.6 mg daily or
o t. Arthritis Rheum. 1977;20(3):895–900. 2 to 6 months. Note that this is a much lower dose than we have
prescribed in the past! Due to signi cant toxicity at high doses,
it is no longer recommended to “titrate to diarrhea.” Oral or
considered septic until proven otherwise (although as noted intra-articular corticosteroid administrations are also options or
above this is only 30% sensitive or septic arthritis). See able 11-4 patients who have contraindications to NSAIDs, who have ailed
or the diagnostic criteria or gout. Note that up to 20% o NSAID therapy, or who have more severe attacks. Corticosteroids
patients with a crystalline arthritis will have a coexisting septic are just as ef cacious as NSAIDs and may be more appropriate or
arthritis (J Rheumatol. 2012; 39:157). patients with heart ailure, ulcers, or kidney disease. Prednisone,
30 mg per day or ve days is a reasonable dose. Narcotic pain
medication may be needed as an adjunct to your anti-in amma-
HELPFUL TIP: tory, but will do nothing to shorten the duration o an attack.
Ser m ric acid is o ten normal d rin an ac te attack “A” is incorrect. Allopurinol is indicated to treat hyperuricemia
o o t (probably beca se it precipitates in the a ected and prevent the next gout attack in patients who have had more
joints); th s, yo cannot rely on ser m ric acid levels than one attack in the last year (though some would treat those with
alone to dia nose o t or re te this dia nosis. a particularly bad attack even i < 1/year.), when tophi are present,
or, when single gout attack occurs with chronic kidney disease
grade 2 or more. “B” is incorrect because acetaminophen lacks
the anti-in ammatory properties o NSAIDs and is less e ec-
Question 11.4.3 In general, all o the ollowing are risk
tive. Discontinuing chlorthalidone, “C” (a thiazide diuretic), and
actors or gout EXCEPT:
switching to a non-diuretic anti-hypertensive is probably appro-
A) obacco use.
priate, but will not treat this gout attack. Joint aspiration is not
B) Alcohol use.
therapeutic in gout, but may be help ul in pseudogout.
C) Obesity.
D) Diuretic use.
E) Family history. HELPFUL TIP:
Other dr s that increase the risk o o t incl de ACEs
Answer 11.4.3 The correct answer is “A.” Your patient exhibits and ARBs and beta-blockers. Calci m channel blockers do
many o the risk actors or gout, which include male sex, obe- not increase the risk o o t (BMJ. 2012 Jan 12;344:d8190).
sity, high-protein diet, use o diuretics (either loop or thiazide),
alcohol, and amily history. However, tobacco use is not associ-
ated with gout. You start an NSAID. He returns in a ew days to discuss his
labs and x-rays. A radiograph o the le knee demonstrates
an e usion but is otherwise unremarkable. His knee pain
HELPFUL TIP: is much improved, and knee e usion is nearly resolved. His
Many patients with hyper ricemia do not develop o t uric acid level is 10.1 mg/dL (the upper limit o normal or
or nephrolithiasis; it is nclear whether asymptomatic your lab is 7.2 mg/dL). CBC, creatinine, sodium, and potas-
hyper ricemia sho ld be treated with ric acid-low- sium are normal. You instruct the patient to reduce his alco-
erin a ents, and research is on oin to answer this hol intake (especially beer) and try to lose weight to decrease
q estion. his risk o gout attacks and or overall health. You schedule a
ollow-up visit, but he is a no-show and you don’t see him
CHAPTER 11 • RHEu MATOLOg Y 315

again or more than a year (was it something you said?). acid crystals start to orm at serum levels > 6.7 mg/dL, which
When the patient returns 18 months later, he reports re- is within the 95% con dence range in the adult population, so
quent use o naproxen, and at least ve more acute attacks within the “normal”—but not necessarily “harmless”—range.
o gout. He continues to consume alcohol. His examination “A” is not correct, as levels higher than 6.7 mg/dL would con-
reveals no swollen or tender joints today, but there is a small tinue to cause gout attacks and will not resolve tophaceous de-
whitish subcutaneous nodule overlying a toe PIP joint. Cre- posits. “B” is the correct target or patients without evidence o
atinine is 1.1 mg/dL, serum uric acid 10.5 (normal < 7.2 mg/ tophi, but < 6.0 mg/dL is not suf cient in this patient. “D” is the
dL); liver enzymes and CBC are normal. rst ew digits o the mathematical constant pi. We know you
didn’t all or “D.”
Question 11.4.5 Which is the most appropriate regimen to Uric acid is a normal product o purine metabolism. T ere
start now in this patient to reduce the requency o gout have been some studies that show it may have some role in neu-
attacks? roprotection as well as ghting in ection. oo much is bad, but
A) wice-daily colchicine. too little (“E”) could be a problem as well so we do not target
B) Daily allopurinol 300 to 600 mg uric acid to zero.
C) Daily probenecid.
D) Daily allopurinol 100 mg, with once- or twice-daily colchicine.
HELPFUL TIP:
E) Daily probenecid and allopurinol.
Feb xostat (u loric) is an alternative to allop rinol. How-
ever, it is rather expensive (10 × the cost o allop rinol)
Answer 11.4.5 The correct answer is “D.” T e presence o a
and there is post-marketin data s estin that there
tophus, and the history o more than one attack in the last year
is a hi her incidence o cardiovasc lar events in those
are both indications or starting long-term urate-lowering
on eb xostat when compared to allop rinol. Howev-
therapy. Colchicine administered once or twice daily has been
er, this is a very e ective rate-lowerin a ent that is
shown to reduce the requency o gout attacks 75% to 85%, and
appropriate or patients with history o intolerance or
is especially use ul as prophylaxis (prevention o the next gout
hypersensitivity to allop rinol. Feb xostat can be start-
attack) during the rst 6 to 12 months o urate-lowering thera-
ed at hal a 40 m tablet daily and increased to 40, 80, or
py. Starting allopurinol at a low dose o 100 mg daily and then
even 120 m to reach the tar et ric acid level.
increasing the dose every ew weeks is less likely to provoke new
gout attacks than starting at a larger dose. “B” is incorrect be-
cause it is a higher dose o allopurinol, given without prophy-
laxis, and could induce new gout attacks (which will make this HELPFUL TIP:
patient think that your treatment is not very good, and that you Altho h colchicine is recommended as prophylaxis
are not a very good doctor). when allop rinol is initiated, it sho ld be discontin ed
Probenecid is a uricosuric agent that also reduces the re- within 6 months (i possible) d e to the potential side
quency and severity o acute gout attacks but has multiple drug e ects o g I irritation, diarrhea, and myopathy.
interactions (including allopurinol) and contraindications (e.g.,
decreased renal unction, kidney stones, or presence o tophi).
“C” is incorrect in this case because the patient now has topha- Question 11.4.7 All o the ollowing are side e ects o allo
ceous gout. purinol EXCEPT:
A) Aseptic meningitis.
You prescribe allopurinol 100 mg daily and once daily colchi- B) Rash.
cine, and schedule the patient to return every 2 to 4 weeks to C) Leukopenia.
re-check his uric acid level and make urther allopurinol dose D) Fever.
adjustments. E) GI disturbance.

Question 11.4.6 The target or uric acid lowering in this Answer 11.4.7 The correct answer is “A.” Additional side e -
patient is: ects include elevated liver enzymes, glomerulonephritis, aplas-
A) Less than 7.2 mg/dL, because this is the upper-limit o nor- tic anemia, and vasculitis. Hypersensitivity reactions are espe-
mal or a man his age. cially common in Chinese, Indians, and others rom Asia in
B) Less than 6.0 mg/dL whom it is up to 15%. Consider checking or HLA-B*5801 in
C) Less than 5.0 mg/dL these groups be ore starting allopurinol. Not a pretty drug, but
D) Less than 3.14159 mg/dL, because precision counts look on the bright side . . . it’s not associated with meningitis!
E) As low as possible.
HELPFUL TIP:
Answer 11.4.6 The correct answer is “C.” T is patient has a Probenecid sho ld be avoided in patients with a cre-
tophus; in order to resolve tophi, and minimize the risk or ur- atinine o clearance < 50 mL/min, as it is ine ective and
ther attacks, allopurinol should be increased in 100 mg steps there may be an increased risk o toxicity.
until his uric acid level is consistently less than 5.0 mg/dL. Uric
316 FAMILY MEDICINE EXAMINATION & BOARD REVIEW

It is 9 years later. Congress has passed a law banning the CASE 11.5
Kardashians rom being on V. Your avorite gout patient
Citing your characteristic compassion and attention to detail,
is back. He’s had 9 years o acute intermittent gout attacks
your “gout guy” (as he now calls himsel ) re ers a riend he
(“O course I took my medication, Doc, but it didn’t work so
met at meeting o his avorite club, Gouty Retirees in Love
I stopped taking it”). He presents complaining o pain in his
with Li e (GRILL). T is riend o his is a 65-year-old emale
knees and eet that has been present or several months. He
who reports a history o joint swelling, pain, and redness,
has also developed swelling and pain in his hands. T e pain
usually involving her knees, wrists, and hands; she has never
is less intense than his attacks o gout, but occurs in the same
had rst M P joint involvement. Although she has never had
areas and never completely resolves between attacks. He has
a joint aspiration per ormed, she has been treated or gout
no morning sti ness, no muscle complaints, and no other
or 5 years. She aith ully takes her medication but has ound
systemic complaints. You nd di use edema o both hands
allopurinol unhelp ul. She is currently asymptomatic, but
and palpable hard nodules on the knees, ngers, toes, and in
uses ibupro en or acute attacks. T e joint examination is
the olecranon bursa.
notable only or some knee crepitus and reduced wrist range
o motion—but, again, she is currently asymptomatic.
Question 11.4.8 Which o the ollowing is the most likely
cause o his current symptoms? Question 11.5.1 Which o the ollowing studies is most
A) RA. appropriate or this patient?
B) Osteoarthritis. A) Diagnostic knee injection with corticosteroids.
C) ophaceous gout. B) CBC.
D) PMR. C) RF.
D) Radiographs o the knees and wrists.
Answer 11.4.8 The correct answer is “C.” T is patient has
a long history o acute intermittent gout. A er years o acute Answer 11.5.1 The correct answer is “D.” T e initial evaluation
attacks, patients with gout may develop a orm o the disease should include radiographs o the a ected joints, which may
called chronic tophaceous gout (the nodules are deposits o uric lend clues to the diagnosis. Radiographs may reveal osteophyte
acid called tophi; tophus or a single nodule), in which the inter- ormation typical o osteoarthritis, subchondral cysts, chondro-
critical periods are no longer ree o pain. T ere are no clinical calcinosis typical o CPPD (calcium pyrophosphate dehydrate
associations between gout and the other rheumatic conditions crystal deposition disease), or erosions with an overhanging
mentioned and, there ore, no reason to suspect that another edge typical o gout. Chondrocalcinosis is most o en seen in
rheumatic disorder is causing the chronic pain. the knees and triangular brocartilage o the wrists. “A,” an
injection o corticosteroids, might help relieve symptoms, but it
will not be diagnostic. “B,” CBC, is nonspeci c and will not be
HELPFUL TIP: help ul. “C,” RF, is also unlikely to be help ul given this patient’s
Hyperlipidemia occ rs in 80% o patients with o t— symptoms, which are not suggestive o RA. Also, RF may be
check lipids. Recall also that there can be renal inj ry elevated in in ammatory arthritides other than RA, and so will
with lon standin o t as well as an increased risk o not be help ul.
rate stones.

HELPFUL TIP:
The crystals o o t ( ric acid) and pse do o t (calci m
HELPFUL TIP: pyrophosphate) can be seen in synovial l id d rin in-
In addition to bein a n word, poda ra is a se l di- tercritical periods. I there is an e sion even in the ab-
a nostic tool. The irst MTP joint is a ected in 90% o sence o ac te symptoms and yo are thinkin o t or
patients with o t, and the initial attack involves the pse do o t, tap that joint!
irst MTP joint in 50%.

HELPFUL TIP:
Objectives: Did you learn to . . .
The dia nosis o pse do o t (CPPD) is not as strai ht-
• Eval ate rec rrent monoarthritis?
orward as o t. To con se the clinician all the more,
• Describe the dia nostic synovial f id ndin s in o t?
CPPD tends to travel with other types o arthritis (e. .,
• De ne dia nostic criteria or o t? osteoarthritis, o t). Patients with CPPD are more likely
• Mana e a patient with o t and describe adverse e ects o older than 65 and have knee arthritis, b t clinical criteria
the medications sed to treat o t? alone are ins icient or the dia nosis, which req ires
• Identi y the appropriate tar et ric acid level or rate- demonstration o crystals in the joint l id.
lowerin therapy?
CHAPTER 11 • RHEu MATOLOg Y 317

C) Amyloidosis.
D) Hemochromatosis.
E) All o the above.

Answer 11.5.3 The correct answer is “E.” All o the above are
associated with pseudogout. Additional associated conditions
include hypophosphatemia and hypomagnesemia. For this rea-
son, order the ollowing studies in patients newly diagnosed
with CPPD: thyroid-stimulating hormone ( SH), calcium,
phosphate, magnesium, trans errin saturation, and alkaline
phosphatase.

HELPFUL TIP:
Precipitants o ac te attack o o t and pse do o t
(CPPD) incl de tra ma, s r ery, severe medical illness,
FIGURE 11-1. Chondrocalcinosis o the knee joint. (Note arrows hi hli htin and alcohol overind l ence.
calci cation.)

Objectives: Did you learn to . . .


Your patient’s knee radiographs demonstrate chondrocalci- • Identi y clinical and dia nostic characteristics o CPPD?
nosis (see Fig. 11-1). Examination o synovial uid rom the • Implement appropriate therapy or CPPD?
knee shows positively bire ringent, rhomboid crystals consis-
tent with CPPD (pseudogout) (maybe this is why her gout
medicine wasn’t working). QUICK QUIZ: CRYOg LOBu LINEMIA

Question 11.5.2 Which o the ollowing do you recommend Cryoglobulinemia, a vasculitic disease caused by antibodies
to decrease her risk o recurrent acute attacks o pseudog that precipitate in cold temperatures, is most o en caused by
out? which o the ollowing viral in ections?
A) Serial joint aspiration. A) HIV.
B) Daily allopurinol. B) Hepatitis B.
C) wice-daily colchicine. C) Hepatitis C.
D) Serial intra-articular corticosteroid injections. D) Parvovirus B19.
E) Chondroitin sul ate.
The correct answer is “C.” Hepatitis C is ound in 80% o vas-
Answer 11.5.2 The correct answer is “C.” Pseudogout is diag- culitis cases associated with mixed cryoglobulinemia. Although
nosed by the presence o CPPD crystals (described above) in up to 50% o patients with hepatitis C will eventually express
synovial uid and/or typical x-ray ndings (basically, chondro- cryoglobulins, only a minority o patients have clinical vascu-
calcinosis). Although prophylaxis is more predictably success ul litis. As to the other options, hepatitis B and parvovirus B19
in gout, colchicine 0.6 mg BID has been shown to reduce the in ection may cause a symmetric polyarthritis. HIV is less
requency o pseudogout attacks in CPPD. NSAIDs or colchicine commonly a cause o cryoglobulinemia and is associated with
may be used in acute attacks. “A” and “D” are incorrect. While reactive arthritis. T e symptoms o mixed cryoglobulinemia
joint aspiration and corticosteroid injection may be help ul dur- associated with HCV in ection typically include arthralgias,
ing acute attacks, they have no role in prophylaxis. “B” is also ever, renal disease, palpable purpura, and neuropathy.
incorrect. Since CPPD is not caused by abnormalities in uric acid
metabolism, allopurinol has no role in the management o pseu-
dogout. Finally, chondroitin sul ate is not use ul in pseudogout. CASE 11.6
A 13-year-old male presents to your of ce with his ather. T e
HELPFUL TIP: patient complains o pain in his wrists, elbows, and knees
g o t is more likely to occ r d rin middle a e and is bilaterally. He has elt atigued and has been unable to work
more common in men. Pse do o t has a peak inci- his usual summer job as a busboy at his ather’s restaurant.
dence later in li e and is abo t eq ally prevalent in both He complains o intermittent evers and an evanescent rash
males and emales. that appears during ebrile episodes but which is short lived.
All o these symptoms have emerged in the last 6 weeks, a er
Question 11.5.3 CPPD (pseudogout) is associated with a week-long backpacking trip in the tick-in ested woods o
which o the ollowing? Minnesota. He has no signi cant past medical history. His
A) Hypothyroidism. only medication is acetaminophen daily or joint pain. He
B) Hyperparathyroidism. denies tobacco use, alcohol use, and sexual activity.
318 FAMILY MEDICINE EXAMINATION & BOARD REVIEW

Question 11.6.1 The di erential diagnosis should include


all o the ollowing EXCEPT: Physical examination reveals a thin male in no acute distress.
A) Lyme disease. His temperature is 37.3°C, pulse 100 bpm, and blood pres-
B) JIA. sure 120/70 mm Hg. Small, non-tender, mobile lymph nodes
C) PMR. are palpable in the neck and axillae. T ere is a large, warm,
D) Viral illness. erythematous patch with central clearing at the patient’s le
axilla. T ere is limited range o motion in his right wrist and
Answer 11.6.1 The correct answer is “C,” o course. From earlier le elbow. An e usion is palpable at the le knee, which is
in the chapter, you will recall that the diagnosis o PMR is highly di usely tender.
unusual in adults under the age o 50. JIA is a chronic arthritis o
childhood that can present in a variety o ways, but must include Question 11.6.3 I you were to aspirate the patient’s knee —
arthritis o one or more joints, lasting 6 weeks or more, with symp- so o ten the right answer in this chapter—which o the ol
tom onset be ore age 16 years. Likewise, Lyme disease has several lowing would you expect to nd in the synovial uid?
presentations, presenting with arthritis early or late in the course. A) Greater than 100,000 WBC/mm 3.
Many viral illnesses can result in arthralgia and/or arthritis. Any B) Predominance o eosinophils.
o the diseases listed may have associated symptoms o atigue, C) Monosodium urate crystals.
malaise, headache, and myalgia. One actor that makes Lyme dis- D) Spirochetes.
ease a more likely diagnosis is the history o being outdoors in an E) Predominance o polymorphonuclear cells.
endemic area (90% o Lyme disease in the United States occurs in
New York, New Jersey, Connecticut, Rhode Island, Massachusetts, Answer 11.6.3 The correct answer is “E.” T is patient is pre-
Pennsylvania, Wisconsin, and Minnesota). senting now with classic eatures o Lyme disease. I synovial
uid is obtained in a patient with Lyme arthritis, analysis o the
Question 11.6.2 Which o the ollowing ndings on physical uid reveals leukocytes, most commonly polymorphonuclear
examination would be more consistent with Lyme disease cells. “A” is incorrect. I the synovial uid has > 100,000 WBC/
than JIA? mm 3, you should consider septic arthritis (arthritis in Lyme dis-
A) Bell palsy. ease is mostly an immunological phenomenon rather than true
B) emperature ≥ 38°C. septic arthritis). “B” is incorrect because eosinophils are not
C) Rash. the predominant cell in synovial uid o Lyme arthritis. “C” is
D) Lymphadenopathy. wrong since monosodium urate crystals are observed in gout—
E) A and C. an unlikely cause o this patient’s joint complaints. Finally, “D”
is wrong. In general, Borrelia burgdor eri spirochetes, the caus-
Answer 11.6.2 The correct answer is “A.” All o the other nd- ative organism in Lyme disease, are not observed in the synovial
ings are seen in both Lyme disease and JIA. Neurologic symp- uid.
toms, including Bell palsy (“A”) and even meningitis, may occur
with Lyme disease but not JIA. Rash (“C”) is present in both You strongly suspect Lyme disease. Because it is so swollen,
diseases but di ers substantially. T e characteristic rash o you aspirate the joint and nd slightly cloudy yellow uid,
Lyme disease is erythema migrans. T e rash o systemic-onset with cell count 10,000/mm3, and 95% polymorphonuclear
JIA (also known as Still Disease) is macular, salmon-pink, and cells. Gram stain is negative and culture was subsequently
brought on by heat. Erythema migrans occurs in about 80% o ound to be negative.
patients with acute Lyme disease. T e lesion is o en described
as “targetoid,” meant to convey a red circular rash with central Question 11.6.4 Which o the ollowing is true regarding
clearing. However, most patients do not have the classic lesion. laboratory tests or Lyme disease?
Instead, most patients present with a mildly to brightly ery- A) Serologic tests are reliable within 1 week o the tick bite.
thematous patch in the axilla or belt line, where the tick bite B) Serologic tests are use ul in screening or Lyme disease.
occurs. T e tick itsel is rarely seen. Erythema migrans is usually C) Blood cultures remain positive or B. burgdor eri or months
not pain ul or pruritic. Both Lyme disease and JIA may have a er the tick bite.
associated systemic ndings, including ever and lymphade- D) Serologic tests remain positive or up to 10 years a er antibi-
nopathy, so neither “B” nor “D” is a good discriminator. otic treatment.
E) T e diagnosis o Lyme disease is based on serologic tests.
HELPFUL TIP:
There is a “new” orm o Lyme disease in the Midwest
Answer 11.6.4 The correct answer is “D.” Serologic tests or
ca sed Borrelia mayonii (why does this have to happen to
Lyme disease can remain positive or up to 10, and in some
Iowa?). Rather than the typical b ll’s eye rash, it can pres-
cases 20, years a er exposure. T us, serologic tests alone are not
ent with a di se rash, perhaps ne rolo ic symptoms, and
diagnostic o active Lyme disease. “E” is incorrect: the diagnosis
a spirochetemia o ten visible on dark- ield examination.
o Lyme disease is clinical with laboratory tests used to con rm
So ar it is ncommon and limited to the pper Midwest.
the diagnosis. A positive ELISA test is not adequate to make
the diagnosis o Lyme disease. Positive or equivocal ELISA tests
CHAPTER 11 • RHEu MATOLOg Y 319

should be con rmed with Western blot analysis. “A” is incor- TABLE 11-5 STAGES OF LYME DISEASE
rect: serologic assays may be alsely negative early in in ection.
Early localized disease (Stage I): Occ rs days to a month a ter the tick
“B” is incorrect. Lyme serology is set to be a very sensitive test:
bite and incl des erythema mi rans, ati e, ever, malaise,
there are ew alse negatives assuming enough time has elapsed myal ias, arthral ia, arthritis, headache, and lymphadenopathy.
since Borrelia exposure, but it is not highly speci c and there Except or erythema mi rans, it can be con sed with a viral illness.
are many alse positives. Because o the high alse-positive rate, Early disseminated disease (Stage II): Occ rs weeks to months a ter
serologic assays should not be used as a screening tool in the the tick bite; 5–10% have cardiac mani estations (atrioventric lar
block o any de ree, myocarditis/pericarditis, and heart ail re) and
general population but are a good step when Lyme disease is
10–15% have ne rolo ic mani estations (see below).
suspected. Most laboratories will automatically do a Lyme Late disease (Stage III): Occ rs months to years a ter the tick bite and
Western Blot study to con rm positive Lyme serology. “C” is incl des myal ias, arthral ias, ati e, polyarthritis, and ne rolo ic
incorrect: positive blood cultures or B. burgdor eri are rarely symptoms (encephalopathy, co nitive dys nction, and peripheral
obtained. When cultures do grow B. burgdor eri, it is only early ne ropathy).
in the disease. Cultures o skin biopsied rom the erythema
Adapted rom Bhate C, Schwartz RA. Lyme disease: Part I. advances and
migrans lesion are more likely to be positive. perspectives. J Am Acad Dermatol. 2011;64(4):619–636.

Question 11.6.5 For this 13 year old patient, whose weight


is 50 kg and who has no known allergies, which course o
therapy is sa est and most ef cacious? Answer 11.6.6 The correct answer is “D.” A Jarisch–Herx-
A) Amoxicillin 500 mg by mouth three times daily or 1 week. heimer reaction occurs in 5% to 15% o patients treated with
B) Ce riaxone 2 g IM, single dose. antibiotics or Lyme disease. (Remember syphilis? Lyme is also
C) Doxycycline 100 mg by mouth twice daily or 4 weeks. a spirochete disease.) T e reaction is mediated by the release
D) Levo oxacin 250 mg by mouth daily or 2 weeks. o cytokines and occurs within hours o initial administration
E) Erythromycin 250 mg by mouth our times daily or 4 weeks. o antibiotics. In Lyme disease, the reaction is sel -limited and
usually resolves within a day. Only supportive treatment is nec-
Answer 11.6.5 The correct answer is “C.” Recommended ther- essary, and antibiotics should be continued (But hey, you know
apy or Lyme arthritis (without neurologic disease) is 4 weeks o you got the diagnosis right!). “A” is incorrect because this reac-
either amoxicillin 500 mg three times daily, doxycycline 100 mg tion is not typical o a drug allergy. “B” is incorrect because
twice daily, or ce uroxime axetil 500 mg twice daily. T is patient B. burgdor eri does not cause sepsis. “C” is incorrect because it
is old enough to take doxycycline, which is generally avoided in is unlikely that a secondary bacterial in ection has occurred so
children < 8 years old due to tooth discoloration. T e duration quickly. Finally, this does not represent the natural history o
o amoxicillin prescribed here is too short, so “A” is wrong. I Lyme disease (“E”). See able 11-5 or more on the natural his-
this patient was younger and doxycycline was contraindicated, tory o Lyme disease.
a 4-week course o amoxicillin or ce uroxime would be the pre-
erred therapy. Ce riaxone is prescribed when neurologic abnor- HELPFUL TIP:
malities are present (such as Bell palsy or meningitis), and it must Lyme disease symptoms typically improve a ew days
be dosed daily or 2 to 4 weeks. Levo oxacin is not indicated or a ter startin antibiotics.
Lyme disease. reatment with erythromycin or 4 weeks, while
an acceptable alternative, appears to be less ef cacious.

Since he spends much o his ree time hunting deer in Minne-


HELPFUL TIP: sota (trying to shoot Bambi), your patient’s ather is worried
There is no known resistance o B. burgdorferi to stan- about contracting Lyme disease himsel .
dard antibiotic re imens.
Question 11.6.7 What do you recommend or primary pre
vention o Lyme disease?
Several hours a er starting antibiotics, the patient’s ather A) Weekly tick checks.
calls to reports worsening symptoms o ever, shaking, and B) N,N-diethyl-m-toluamide (DEE ) application prior to
dizziness. hunting.
C) Daily doxycycline when in endemic areas.
Question 11.6.6 You recognize this condition as which o D) Lyme vaccine.
the ollowing? E) Kill as many deer as possible in order to reduce the risk o
A) An allergic reaction to the antibiotic. Lyme disease transmission to humans.
B) B. burgdor eri sepsis.
C) Secondary bacterial in ection. Answer 11.6.7 The correct answer is “B.” Primary prevention
D) A cytokine-mediated reaction to the antibiotic-mediated is best accomplished with the use o insect repellents when in
killing o spirochetes (Jarisch–Herxheimer reaction). endemic areas. Also, when in endemic areas, tick checks should
E) T e expected, natural course o Lyme disease. be per ormed daily (not weekly as in answer “A”). A tick that has
320 FAMILY MEDICINE EXAMINATION & BOARD REVIEW

been attached or < 24 hours is not likely to transmit B. burg- rom Guam 4 years ago. She reports poor sleep and eeling
dor eri. Un ortunately, the species that transmits Lyme disease quite depressed. She eels that she has no riends, and she has
(ticks o the Ixodes ricinus complex) are very small and dif cult had trouble adjusting to the colder weather. You notice she
to see. “C” is incorrect. T ere is no role or routine prophylactic has a bottle o water with her and upon your speci c ques-
antibiotics. However, i a tick bite rom the appropriate species tioning she states, “I have to sip some water throughout the
is noticed, a single dose o doxycycline 200 mg administered day. I’ve done this or the last 15 years because my mouth gets
orally reduces the risk o erythema migrans. T e dose should so dry.” Over the past twenty years, she has had numerous ll-
be administered within 72 hours o a known I. ricinus tick bite. ings or dental cavities. She denies problems with skin rash,
“D” is not an option or this patient. In 2002, the vaccine was swallowing, and eye pain. She does not use arti cial tears.
removed rom the U.S. market due to low demand. When it was
available, antibody titers tended to wane quickly and protection Question 11.7.1 Which o the ollowing ndings in this
was not complete. A er the administration o three vaccinations, patient is most likely to be a sign o in ammatory arthritis?
the ef cacy o the vaccine to prevent Lyme disease was 76% at A) Spider angiomas (telangiectasia) on the back and abdomen.
best. “E” is just plain wrong . . . what with Bambi and all . . . And B) A small cool le knee e usion (“bulge sign”) with crepitus.
urthermore, the white- ooted eld mouse, not the white-tailed C) Presence o 16/18 bromyalgia tender points, with non-
deer, is the major reservoir o B. burgdor eri bacteria. tender control points.
D) Incomplete le grip.
E) Presence o a holosystolic murmur at the le sternal border,
HELPFUL TIP:
without radiation.
Repeat a ter s: there is no s ch thin as “chronic Lyme
disease” req irin months-to-years o antibiotics.
Answer 11.7.1 The correct answer is “D.” Although her symp-
The CDC does reco nize an entity called “post-Lyme
toms are suggestive o bromyalgia and depression, it is critical
disease syndrome,” that may incl de ati e, myal ia,
to di erentiate between an in ammatory and a non-in amma-
co nitive slowin and n mbness. However, post-Lyme
tory condition, especially since many in ammatory disorders
disease syndrome does NOT improve with prolon ed
(e.g., SLE, RA, Sjögren syndrome) may masquerade as bro-
antibiotics. Foc s on lookin or other ca ses (e. ., de-
myalgia. An incomplete grip in an otherwise healthy young
pression, ibromyal ia) in these patients and alleviatin
woman is suggestive o synovitis, which can be urther assessed
symptoms, b t avoid the temptation to prescribe anti-
by care ul small joint examination. When synovitis is present,
biotics that do not work.
it is always abnormal and suggests an in ammatory arthritis,
requiring urther evaluation. “A” is incorrect because telangiec-
tasias on the abdomen and trunk are typically related to liver
HELPFUL TIP:
disease, while those ound on hands and nail beds are associ-
I Lyme menin itis is s spected, con irm by analysis o
ated with systemic sclerosis and other rheumatic diseases. “B,” a
CSF. Lyme menin itis m st be treated with IV ce triax-
positive knee “bulge sign” (swelling o the knee joint that bulges
one or penicillin g .
in eriorly when compressed superiorly), indicates uid in the
le knee joint. But rom the patient’s history and your ortho-
Objectives: Did you learn to . . . pedic colleague’s determination, this nding is chronic and
• Describe the appropriate eval ation o polyarthritis? mechanical in nature. “C,” the presence o 16/18 tender points,
• Dia nose Lyme disease? would argue or bromyalgia but would not help to di erentiate
• Describe the sta es o Lyme disease? between in ammatory and non-in ammatory disorders. Addi-
• Implement appropriate therapy or Lyme disease? tionally, identi ying greater than 11 out o 18 tender points is
no longer part o the diagnostic criteria or bromyalgia (keep
• Disc ss preventive strate ies or Lyme disease and describe
some o the complications o the disease? reading or details). Finally, a holosystolic murmur (“E”), local-
ized to the le sternal border and present in a healthy young
woman, is nonspeci c and most likely unctional. I there were
CASE 11.7 other signs and symptoms o cardiac disease, the murmur might
A 42-year-old emale who was re erred by an orthopedic sur- indicate a more serious disorder.
geon presents to your of ce with multiple joint complaints.
T e orthopedist has seen her or le knee pain, intermittent On physical examination, you nd a “bulge sign” on the
swelling, occasional “clicking and locking,” present or about le knee, but no other joint swelling. She has 16/18 tender
10 years. A er knee radiograph and examination, the ortho- points and normal range o motion and strength. T e neu-
pedist diagnosed a chronically damaged meniscus, but he rological examination is grossly normal, except or poorly
wants the patient evaluated by you or her other joint com- de ned numbness and pain to touch on the le side o her
plaints. ace. You also notice a mildly tender, hard, nodular swelling
Laboratory data ordered by her orthopedist (apparently, behind the angle o the mandible on the le in the area o the
just to con use you) show an ANA 1:320 (speckled pattern) parotid gland. Her oral mucosa appears dry. Her conjunctiva
and an ESR 20 mm/hr. T e patient moved to the United States is mildly, symmetrically injected.
CHAPTER 11 • RHEu MATOLOg Y 321

Question 11.7.2 All o the ollowing studies and interven o a mass-like ormation on MRI is concerning or lymphoma,
tions are appropriate EXCEPT: and urther evaluation (e.g., biopsy) is required. Additionally,
A) CBC, transaminases, ESR, CRP. the negative SSA and SSB, while not excluding Sjögren syn-
B) Anti-SS-A (Ro), anti-SS-B (La), anti-dsDNA, RF, serum drome, will make a biopsy necessary or diagnosis. Although
protein electrophoresis. a minor salivary gland biopsy (“D”) is the most speci c way to
C) Prescribe trazodone 50 mg PO at bedtime and recommend con rm a diagnosis o Sjögren syndrome, the rst priority is
aerobic exercises. to evaluate the parotid gland mass. T e elevated RF and poly-
D) Prescribe prednisone 20 mg PO daily, with calcium and vita- clonal gammopathy are consistent with Sjögren syndrome,
min D supplement. and the ESR may be elevated due to increased globulins.
E) Maxillo acial MRI. MRI o the head and neck (“E”) may provide some structural
in ormation but it is not in the diagnostic criteria or Sjögren
Answer 11.7.2 The correct answer (and the thing to avoid Syndrome.
right now) is “D.” Although prednisone may be used to treat
symptoms o autoimmune diseases, at this time the diagno- Results o the parotid gland biopsy report read, “Lympho-
sis is not secure, and initiating corticosteroid therapy exposes cytic in ltrate, no malignant cells noted.” You then order ow
the patient to potentially unnecessary risk. She has ndings o cytometry, and it has no markers or lymphoma. Her biopsy
Sjögren syndrome—red (possibly dry) eyes, dry mouth, and scar has healed nicely, and she has no pain or numbness. You
enlarged parotid glands. However, the di erential o mass in believe that she probably has Sjögren syndrome.
the parotid gland must include malignancy (e.g., lymphoma),
sarcoidosis, and other autoimmune diseases. T e laboratory Question 11.7.4 What would you do next?
tests o ered in answers “A” and “B” may help assess other organ A) C chest/abdomen/pelvis.
involvement o Sjögren syndrome and also aid in con rming the B) Start prednisone 20 mg by mouth daily, with calcium and
diagnosis. Other potential mani estations o Sjögren syndrome vitamin D.
include generalized vasculitis, interstitial lung disease, cirrhosis, C) Recommend sugarless lemon drops and arti cial tears as
peripheral and cranial neuropathies, possibly thyroid disease, needed and continued trazodone and exercise.
and renal disease leading to proteinuria and renal tubule dys- D) Wide excision o the parotid gland.
unction. Although not listed as an option, chest radiograph may
also be help ul, assessing or ndings associated with the diseases Answer 11.7.4 The correct answer is “C.” Lemon drops (or
on your di erential: Sjögren syndrome (interstitial lung disease), anything sour or that matter) will stimulate saliva produc-
sarcoidosis (adenopathy and interstitial disease), and lymphoma tion, helping with her dry mouth. Arti icial tears may also be
(adenopathy). “E” is also important because although parotid indicated or dry eyes. At this time your working diagnosis
enlargement may be seen in Sjögren syndrome, it is usually sym- is Sjögren syndrome, and de initive diagnosis by minor sali-
metrical and non-tender. An imaging study is appropriate to rule vary gland biopsy ( rom the buccal mucosa o the lower lip)
out a neoplastic process. Finally, “C” is correct. Her symptoms o is not likely to alter your therapy; and “D” is way too aggres-
bromyalgia may respond to trazodone and exercise, and these sive anyway. Since she has responded to trazodone and exer-
low-risk interventions are appropriate at this juncture. cise and there is no evidence o systemic involvement, no
urther anti-in lammatory therapy (“B”) is warranted. I she
She starts trazodone and exercise and eels better. T e tests were to develop arthritis or other signs o systemic involve-
you order return as ollows: negative SSA, SSB, and dsDNA; ment (e.g., cognitive dys unction or peripheral neuropathy),
elevated RF; no monoclonal protein on SPEP but di usely prednisone would be an option. Further work-up or lym-
elevated globulins. A chest x-ray is normal. Her ESR is phoma, such as C scanning (“A”), does not appear war-
35 mm/hr and CRP 0.5. A maxillo acial MRI shows an ranted. However, she will require active surveillance, since
enlarged le parotid, with an ill-de ned 2 × 3 × 1.5 cm dense patients with Sjögren syndrome carry an increased risk o
signal in the center without neurovascular compromise. developing lymphoma.

Question 11.7.3 What is the most appropriate next step in


the management o this patient?
A) Continue your current management and adopt a “watch ul HELPFUL TIP:
waiting” approach. Sicca symptoms (dry eyes and mo th) are extremely
B) Re er or biopsy o the le parotid. common, especially in the elderly, and sho ld be con-
C) Initiate prednisone 20 mg PO daily, with calcium and vita- irmed by objective physical indin s. The dia nosis o
min D. Sjö ren syndrome is s ested by presence o anti-SS-
D) Re er or minor salivary gland biopsy (lip biopsy). A, or positive ANA titer (1:320 or reater) with elevated
E) MRI o the head and neck. RF, b t de initive dia nosis o Sjö ren syndrome relies
on histopatholo ic land indin s on minor salivary
Answer 11.7.3 The correct answer is “B.” Even though her land (lip) biopsy.
presentation is suggestive o Sjögren syndrome, the presence
322 FAMILY MEDICINE EXAMINATION & BOARD REVIEW

Objectives: Did you learn to . . . Question 11.8.2 Which o the ollowing is NOT an appropri
• Describe the appropriate eval ation o polyarthral ia and ate medication or treating bromyalgia type pain?
sicca symptoms? A) Hydrocodone/acetaminophen
• Eval ate a patient with probable Sjö ren syndrome? B) Gabapentin or pregabalin
• Implement appropriate therapy or Sjö ren syndrome? C) Amitriptyline
D) Duloxetine
E) Combination duloxetine and pregabalin
CASE 11.8
Answer 11.8.2 The correct answer is “A.” Is there a theme
Once again, your abled diagnostic and therapeutic abili- emerging here? Narcotics will provide a short-term reduction
ties have earned you a well-deserved re erral. Your previous in pain, but tend to worsen central sensitization and actually
patient is so pleased with the way things are going that she increase bromyalgia pain over time; they should be avoided in
re ers her sister-in-law to you. T e sister-in-law is 46 years old managing this type o pain. All o the other agents have shown
and reports having been diagnosed with bromyalgia several ef cacy in improving pain associated with bromyalgia. ricy-
months ago. She reports having all-over pain, all the time, clic antidepressants have the best ef cacy. SNRI agents such as
which has been worsening or the last 5 years. You thank the duloxetine may be combined with a gabapentin or pregabalin
patient or the re erral . . . NO . She takes only acetamino- as can tricyclics.
phen and codeine as needed or pain (she typically needs it
our times per day). She does not exercise because it hurts
too much. She drinks 2 liters o Mountain Dew a day, rom HELPFUL TIP:
sun-up to sundown. Her sleep is reported as poor and non- Fibromyal ia is a common syndrome characterized by
restorative. di se chronic pain accompanied by other somatic
symptoms s ch as poor sleep, ati e, and sti ness,
Question 11.8.1 Which o the ollowing statements is true in the absence o another identi iable disease. In order
regarding management o bromyalgia: to dia nose ibromyal ia, symptoms m st be present
A) Fibromyalgia pain is increased during exercise, so exercise or at least 3 months and other chronic pain conditions
should be avoided. m st be excl ded. However, an exha stive search or
B) Regular low impact exercise, and non-pharmacologic mea- occ lt disease is not recommended. Most dia noses
sures to improve sleep are more e ective than medications can be made with history, physical examination and
or improving bromyalgia-type pain. minimal labs (e. ., CBC, CRP, ESR to r le o t in lamma-
C) SNRI agents such as duloxetine and milnacipran lead tory disease). The 1990 American Colle e o Rhe ma-
to rapid and long-term remission o ibromyalgia-type tolo y criteria incl ded identi ication o pain on palpa-
pain. tion at 11 or more o 18 tender points (Fi . 11-2), b t
D) Both NSAIDs and prednisone are help ul or reducing bro- as o 2010, this minim m n mber is no lon er req ired
myalgia pain. to establish a dia nosis. There is still val e in examin-
E) Opioids act on the central nervous system, which is why in these tender points, and a hi her n mber o tender
they are the most e ective agents or treatment o bromyal- points are associated with hi her likelihood o ibro-
gia pain. myal ia. However, yo need not have 11 o 18. Also,
sel -administered scales, s ch as the Widespread Pain
Index, may be se l in dia nosin ibromyal ia. There
Answer 11.8.1 The correct answer is “B:” improved sleep is a emale predominance in ibromyal ia, with p to
through good sleep hygiene, and regular low-impact exer- 75% o patients bein women.
cise are o en more e ective than medications or reducing
bromyalgia-type pain. Indeed, improving sleep and exercise
(and identi ying any underlying sleep disorder) are central to Question 11.8.3 All o the ollowing are associated with
long-term management. “A” is incorrect: even when exercise is bromyalgia EXCEPT:
poorly tolerated due to pain, a program o stretching and gradu- A) Irritable bowel syndrome.
ally increasing exercise is very help ul to reduce pain over time. B) Subjective ullness/swelling o hands and eet.
SNRI agents are help ul or reducing central pain sensitization, C) Paresthesias.
and are important adjunctive treatment to sleep and exercise, D) Fatigue.
but they are not curative and do not provide rapid pain relie . E) Night sweats.
“D” is incorrect: NSAIDs and prednisone are not e ective or
bromyalgia-type pain. Opioids do act on the central nervous Answer 11.8.3 The correct answer is “E.” All o the others are
system, and can lead to short term reduction o bromyal- associated with bromyalgia. Additional symptoms include
gia-type pain, but at the same time they increase central pain headaches, depression, sleep disturbance (which may actually
sensitization which can actually lead to worsening o pain be the etiology), other GI symptoms, and urethral spasm with
(opioid-induced hyperalgesia). dysuria and urgency.
CHAPTER 11 • RHEu MATOLOg Y 323

Occiput:
S uboccipita l
mus cle Low cervical:
ins e rtions Ante rior a s pe cts
Trapezius: of the inte rtra ns ve rs e
Midpoint of the s pa ce s a t C5–C7
uppe r borde r
Second rib:
Supraspinatus: S e cond
Above the me dia l cos tochondra l
borde r of the junctions
s ca pula r s pine
Lateral
Gluteal:
epicondyle:
Uppe r oute r
2 cm dis ta l to
qua dra nts
the e picondyle s
of buttocks

Greater
trochanter:
P os te rior to
the trocha nte ric
promine nce

Knee:
Me dia l fa t pa d
proxima l to the
joint line

FIGURE 11-2. Tender points in bromyal ia.

not cause osteoarthritis. Any disease that leads to a neuropathy,


HELPFUL TIP:
such as diabetes, can predispose to osteoarthritis (e.g., Charcot
Altho h some SNRIs (D loxetine, Savella, Venla ax-
arthropathy).
ine) are FDA-approved or ibromyal ia, they are not as
e ective as the TCAs.
CASE 11.9
A 25-year-old emale presents to your of ce complaining o
Objectives: Did you learn to . . . bi rontal headaches, occurring intermittently over the last
• Identi y typical symptoms o bromyal ia? year. Also, she complains o atigue that seems to be slowly
• De ne dia nostic criteria or bromyal ia? worsening. Over the last 2 to 3 months, she has developed
• Implement appropriate therapy or bromyal ia? generalized joint pain and sti ness. T e remainder o the his-
tory, including a detailed review o systems, is unremarkable.
On physical examination, you nd a thin emale in
QUICK QUIZ: SECONDARY OSTEOARTHRITIS no acute distress. She is a ebrile with a blood pressure o
110/62 mmHg and a pulse o 72 bpm. T e joint examination
All o the ollowing diseases and conditions are secondary shows ull range o motion and no swelling. T ere is mild
causes o osteoarthritis EXCEP : anterior cervical lymphadenopathy. Close inspection o her
A) Fibromyalgia. skin reveals erythema o the malar eminences and the nose
B) Hemochromatosis. laterally, with involvement of the nasolabial folds. You note
C) Hyperparathyroidism. aking and scaling in the eyebrows.
D) Amyloidosis.
E) Previous joint trauma. Question 11.9.1 The RASH is most characteristic o which
o the ollowing diagnoses?
The correct answer is “A.” One o the most common second- A) Dermatomyositis.
ary causes o osteoarthritis is previous joint trauma. Systemic B) SLE.
diseases that can lead to osteoarthritis include hemochromato- C) Seborrheic dermatitis.
sis, hyperparathyroidism, and amyloidosis. Fibromyalgia does D) Psoriasis.
324 FAMILY MEDICINE EXAMINATION & BOARD REVIEW

Answer 11.9.1 The correct answer is “C.” T e classic rash o Several ndings point toward SLE: arthritis, cardiac riction rub
SLE is the malar, or “butter y,” rash with erythema over the (presumably due to pericarditis), and painless oral ulcers. Keep
malar eminences, bridging over the base o the nose. However, reading or more on the diagnostic criteria or SLE.
the nasolabial olds are spared with the lupus malar rash.
Involvement o the nasolabial old characteristically occurs in Based on your history, you recognize that your patient may
seborrheic dermatitis. In dermatomyositis, acial lesions involve be at higher than normal risk or developing SLE.
the upper lids and have a light reddish-purple hue (the so called
“heliotrope rash”). Flat-topped, violaceous papules over the Question 11.9.4 All o the ollowing groups have a higher
knuckles (Gottron papules) are classic eatures o dermatomyo- incidence o SLE than the general population EXCEPT:
sitis. T e typical lesions o psoriasis include erythematous pap- A) Family members o patients with SLE.
ules and plaques with silvery scales, noted more commonly on B) Females.
extensor sur aces. C) Asian Americans.
D) Age in the third to h decades.
Question 11.9.2 Which o the ollowing diagnoses or descrip E) Caucasians.
tions most accurately describes your patient ’s disease
process at this point in time?
Answer 11.9.4 The correct answer is “E.” Compared to other
A) Fibromyalgia.
groups, Caucasians have a lower risk o SLE. T e peak incidence
B) Somato orm disorder.
o SLE occurs in the third to h decades o li e. Women are
C) Polyarthritis.
5 to 10 times more likely than men to be diagnosed with SLE.
D) Polyarthralgia.
First-degree relatives o patients with SLE are at higher risk as
E) RA.
well. win studies have shown a 25% to 50% concordance rate
among monozygotic twins.
Answer 11.9.2 The correct answer is “D.” Your patient
complains o pain in multiple joints but has no indings o
You consider that this might represent drug-related lupus,
in lammation o the joints; there ore, the designation o
but your patient denies using any medications except or
“polyarthralgia” its best at this time. I multiple joints were
acetaminophen.
in lamed, you would use the term polyarthritis. At this time,
you do not have enough in ormation to make any o the other
diagnoses. Question 11.9.5 In drug related lupus, you expect to see all
o the ollowing EXCEPT:
You screen or depression and anxiety and nd neither. A) Negative ANA.
You recommend acetaminophen or joint pain and head- B) Rapid resolution o symptoms a er discontinuing the drug.
aches and encourage her to exercise regularly. She returns C) Polyarthralgia.
2 months later eeling worse. She has severe atigue and joint D) Negative anti-dsDNA.
pain, most commonly involving her hands and knees. She has E) Low-grade ever.
taken a leave o absence rom her job as a high school English
teacher. New symptoms include sores in her mouth as well Answer 11.9.5 The correct answer is “A.” Patients with drug-
as chest pain, which is worse with inspiration. On examina- induced lupus will generally have a positive ANA. However, it
tion, you note the presence o a cardiac riction rub. T ere can be di erentiated rom SLE by a negative anti-dsDNA. Drug-
is di use tenderness to palpation o both knees and a small related lupus presents with a lupus-like syndrome, with the most
e usion apparent in the le knee. Her le hand demonstrates common eatures being arthralgias, myalgias, atigue, malaise,
slow, incomplete grip. T e acial rash is barely noticeable. and low-grade ever. Pericarditis and pleuritis are occasionally
T ere are two nontender ulcerations o the oral mucosa. T e present. Skin, renal, and neurologic involvement are rare. Some
remainder o the examination is unchanged rom her previ- common drugs that cause drug-induced lupus include hydrala-
ous visit. zine, diltiazem, hydrochlorothiazide, phenytoin, glyburide, in -
liximab and etanercept.
Question 11.9.3 Her disease process and current ndings
are most suggestive o which o the ollowing diagnoses? Your patient returns with a pain ul, swollen le knee, and
A) Lymphoma. she wants to do something about it. You also discuss the ol-
B) SLE. lowing laboratory test results: WBC 5,100 cells/mm3, Hgb
C) Fibromyalgia. 11.0 g/dL, platelets 309,000 cells/mm3; BUN 10 mg/dL, creat-
D) Reactive arthritis. inine 1.0 mg/dL; ESR 77 mm/hr; ANA 1:1,280 with a nucleo-
E) Limited scleroderma (CRES syndrome). lar pattern; urinalysis + 1 protein, otherwise negative; urine
microscopic examination shows no RBCs or casts; chest
Answer 11.9.3 The correct answer is “B.” While she does x-ray shows no cardiomegaly and normal lung elds; ECG
not meet all the criteria or a diagnosis o SLE, this presenta- shows normal sinus rhythm. You believe that your patient
tion is more consistent with SLE than any o the other options. has SLE.
CHAPTER 11 • RHEu MATOLOg Y 325

TABLE 11-6 DIAGNOSTIC CRITERIA FOR SYSTEMIC disease, high-dose corticosteroids are employed. Hydroxychlo-
LUPUS ERYTHEMATOSUS roquine may provide relie o musculoskeletal and constitutional
symptoms and may also have a corticosteroid-sparing e ect, but
SLE req ires the presence o 4 o the 11 ndin s below, not necessarily
most important is the observation that SLE patients who take
occ rrin at the same time.
hydroxychloroquine have ewer organ-threatening lupus mani-
1. Malar rash
2. Discoid rash
estations and ewer disease ares over time. Lupus patients who
3. Photosensitivity take hydroxychloroquine live longer than those who do not.
4. Oral lcers ( s ally painless) Ideally, this patient should also be re erred to a rheumatologist,
5. Nonerosive arthritis (involvin 2 or more peripheral joints with but appropriate treatment should be started in a timely ashion.
tenderness, swellin , or e sion) Close ollow-up is necessary due to the potential adverse e ects
6. Serositis (pericarditis or ple ritis)
7. Renal disorder (persistent protein ria > 0.5 /day or cell lar casts)
o these medications (e.g., diabetes with prednisone).
8. Ne rolo ic disorder (seiz res or psychosis)
9. Hematolo ic disorder (hemolytic anemia, le copenia Over the next year, your patient does very well. She estab-
< 4,000/mm 3 on 2 occasions, lymphopenia < 1,500/mm 3 on 2 lishes care with a rheumatologist and is in remission when
occasions, or thrombocytopenia < 100,000/mm 3 in the absence o she sees you next. She is interested in becoming pregnant
o endin dr s)
10. Imm nolo ic disorder (anti-DNA, anti-Smith, or antiphospholipid
and wishes to seek your advice prior to trying to conceive.
antibodies, or a alse-positive serolo ic test or syphilis) (“Start a college und early,” you say. “Oh, that’s not what you
11. Positive ANA meant?”) Fortunately, she has been able to discontinue pred-
nisone and continues to tolerate hydroxychloroquine.
Adapted rom Tan EM, Cohen AS, Fries JF, et al. The 1982 revised criteria
or the classi cation o systemic l p s erythematos s. Arthritis Rheum. Question 11.9.7 With regard to pregnancy and SLE, your
1982;25:1271–1277 and Hochber MC. u pdatin the American Colle e
o Rhe matolo y revised criteria or the classi cation o systemic l p s
patient is at higher risk or all o the ollowing EXCEPT:
erythematos s [letter]. Arthritis Rheum. 1997;40:1725. A) Premature birth.
B) In ertility.
C) Intrauterine etal demise.
Question 11.9.6 Which o the ollowing management plans D) Spontaneous abortion.
do you suggest to the patient?
A) Start ibupro en 600 mg by mouth three times daily and Answer 11.9.7 The correct answer is “B.” Women with SLE
re er to a rheumatologist (3–6 month wait—sheesh, they are have a greater risk o premature birth, spontaneous abortion,
busy). and intrauterine etal demise compared with otherwise healthy
B) Start prednisone 20 mg by mouth daily and hydroxychlo- women. However, these patients appear to have comparable
roquine 400 mg by mouth daily, schedule or ollow-up in rates o ertility. Pregnancy outcomes are best when the patient
1 month, and re er to a rheumatologist (3–6 month wait). is in remission 6 to 12 months be ore conception. Pregnancy
C) Start methotrexate 10 mg by mouth weekly, prednisone appears to have a variable e ect on SLE, with some patients
60 mg by mouth daily, and hydroxychloroquine 400 mg by experiencing exacerbations o the disease.
mouth daily, and schedule ollow-up in 1 month.
D) Start prednisone 60 mg by mouth daily and re er to a rheu- Your patient does well with her pregnancy but develops recur-
matologist (3–6 months wait). rent DV s in the subsequent ew years. You wish to evaluate
E) Start ibupro en 600 mg by mouth daily and adopt a “watch- her or antiphospholipid antibody syndrome.
ul waiting” approach with ollow-up in 6 months.
Question 11.9.8 Which o the ollowing results would you
Answer 11.9.6 The correct answer is “B.” Your patient now expect to nd in a patient with antiphospholipid antibody
meets diagnostic criteria or SLE, according to the American syndrome?
College o Rheumatology. See able 11-6 or the diagnostic crite- A) Elevated P /INR.
ria or SLE. Note that a positive ANA is only one o the criteria B) Low P /INR.
and is not required or the diagnosis o lupus; however, the C) Prolonged dilute Russell viper venom time (DRVV ) that
vast majority o patients with SLE will have an elevated ANA, corrects with addition o platelets.
and negative results should make you re-think the diagnosis. D) Elevated aP that corrects with addition o normal serum.
Prompt treatment o her symptoms is important. NSAIDs, such E) High- brin degradation products.
as ibupro en, are use ul in treating arthralgias, mild arthritis, and
mild pleurisy and pericarditis. In this case, oral (or perhaps intra- Answer 11.9.8 The correct answer is “C.” T e DRVV assay is
articular) corticosteroids are indicated or immediate symp- highly sensitive to the presence to proteins that block phospho-
tomatic relie o her arthritis, and a low-to-moderate oral dose lipids, because the viper venom depletes phospholipid action.
should be used (10–20 mg rather than 60 mg). Vitamin D and T e DRVV may be prolonged when the less sensitive aP is
calcium supplements should be prescribed with the corticoste- normal. When the DRVV is prolonged, addition o platelets
roids. Methotrexate can be used in recurrent, persistent arthritis. (as source o phospholipid) will correct the DRVV when pro-
In the event o more serious renal, hematologic, or neurologic longation is due to a lupus anticoagulant.
326 FAMILY MEDICINE EXAMINATION & BOARD REVIEW

Even though they are prone to clotting, patients with Answer 11.9.9 The correct answer is “E.” Both “B” and “C”
antiphospholipid antibody syndrome o en have a paradoxically are true statements. Since patients with antiphospholipid anti-
elevated aP . I this abnormality does NO correct in vitro body syndrome have an elevated aP , this value cannot be
with addition o normal serum, it is supportive o the presence used to monitor anticoagulation with standard heparin. Per-
o a “lupus anticoagulant,” which is presumptive evidence o cent actor II activity is the most appropriate way to moni-
antiphospholipid antibody syndrome. “D” is incorrect, as cor- tor these patients i they are on standard heparin. Anti actor
rection o a prolonged aP with normal serum implies actor Xa is used to monitor anticoagulation with enoxaparin. “A”
de ciency, not a lupus-anticoagulant. Besides venous thrombus is incorrect because there is no evidence that prophylactic
ormation, women with antiphospholipid antibody syndrome treatment reduces complications (although there is signi -
are more likely to have late etal demise and multiple spontane- cant controversy in this area). Clearly, a er a patient has had
ous abortions. “A” is incorrect, as a lupus anticoagulant will not a thrombotic event, or has another indication, she needs anti-
a ect the prothrombin time. coagulation.

HELPFUL TIP:
HELPFUL TIP:
Antiphospholipid antibody syndrome is a clinical syn-
u n ractionated heparin, low-molec lar-wei ht hep-
drome ca sed by a toantibodies to phospholipids that
arin, and aspirin are all sed in the treatment o an-
mod late the coa lation cascade. The term “l p s
tiphospholipid antibody syndrome in pre nancy.
anticoa lant” re ers to a s bset o these a toantibod-
War arin is terato enic and is contraindicated in pre -
ies that inter ere with clottin tests s ch as aPTT; these
nancy (remember, it is rat poison!). The newer anticoa -
can be detected by the dil te R ssell viper venom test
lants s ch as dabi atran, rivaroxaban, and apixaban
(DRVVT). Alternatively, the presence o antiphospho-
sho ld not be sed in pre nancy: There is not eno h
lipid antibodies can be determined by ELISA or anticar-
experience with their e ectiveness and possible
diolipin or beta-2- lycoprotein antibodies. However,
terato enesis.
there is only an 85% concordance between the pres-
ence o antiphospholipid antibody syndrome and labo-
ratory detection o the ca sative antibodies. Antibodies
m st be detected on two occasions at least 12 weeks Question 11.9.10 Which o the ollowing drugs used to
apart, and there needs to be a clinical mani estation, treat lupus is associated with macular damage, corneal
s ch as pre nancy morbidity or vasc lar thrombosis opacities, and ciliary muscle dys unction?
(arterial, veno s, or small vessel). Pre nancy morbidi- A) Azathioprine.
ties incl de: (1) more than one etal death o a morpho- B) Prednisone.
lo ically normal et s beyond 10 weeks o estation, C) Hydroxychloroquine.
(2) more than one premat re birth be ore the 34th D) Cyclophosphamide.
week o pre nancy d e to pre-eclampsia, eclampsia, or E) Methotrexate.
placental ins iciency, or (3) more than three consec -
tive (pre) embryonic losses prior to 10 weeks o esta- Answer 11.9.10 The correct answer is “C.” Hydroxychloro-
tion. quine is associated with macular damage, corneal opacities, and
ciliary muscle dys unction, and its use requires a baseline eye
examination, and then yearly exams a er 5 years o continuous
HELPFUL TIP: use. T e risk or signi cant ocular toxicity is reduced i the daily
Only abo t 10% o patients with l p s have antiphos- hydroxychloroquine dose is kept below 6.5 mg/kg/day (based
pholipid antibodies. on ideal body weight), which is approximately 400 mg/day or
most women, and 300 mg/day or smaller women. Other SLE
drugs and side e ects are noted in able 11-7.
Question 11.9.9 Which o the ollowing statements is (are)
true?
A) Patients with anticardiolipin antibodies should be treated TABLE 11-7 TOXICITIES ASSOCIATED WITH
prophylactically even i they have never had an abnor- MEDICATIONS USED TO TREAT SLE
mal clotting event, in order to prevent venous thrombosis,
• NSAIDs: astrointestinal bleedin , renal dys nction, hypertension
stroke, or other abnormal clotting events. • Steroids: diabetes, hypertension, hyperlipidemia, osteoporosis,
B) Percent actor II activity is an appropriate way to monitor cataract ormation, wei ht ain, in ections
the response to heparin in patients with antiphospholipid • Hydroxychloroq ine: mac lar dama e, ciliary m scle dys nction,
antibody syndrome. corneal opacities, myopathy
C) Anti actor Xa should be used to monitor the activity o • Azathioprine: in ections, myelos ppression, hepatotoxicity
• Cyclophosphamide: in ections, myelos ppression, hemorrha ic
enoxaparin i monitoring is required. cystitis
D) A and C • Methotrexate: in ections, myelos ppression, hepatic brosis
E) B and C.
CHAPTER 11 • RHEu MATOLOg Y 327

Objectives: Did you learn to . . .


always have a thoro h eval ation or nderlyin sys-
• Identi y clinical mani estations o SLE?
temic ca ses. Other cl es s estin secondary Rayn-
• De ne dia nostic criteria or SLE?
a d phenomenon incl de a history o severe disease (i.e.
• Reco nize the waxin and wanin co rse o SLE? di ic lt to reverse attacks, di ital lcerations) or conc r-
• Implement appropriate treatment o the patient with SLE? rent onset o new c taneo s, m sc loskeletal, astroin-
• Reco nize adverse e ects o medications sed in the treat- testinal, or cardiop lmonary problems. In these patients,
ment o SLE? an ANA titer may be help l in rther eval ation.
• Describe some characteristics o antiphospholipid antibody
syndrome?
HELPFUL TIP:
B er er disease (thromboan iitis obliterans) is stron ly
CASE 11.10 associated with tobacco ab se and presents with distal
A 22-year-old white emale presents to your of ce with the small vessel ischemia and symptoms that are similar to
chie complaint o “blue ngers.” She reports a history o Rayna d phenomenon. It event ally pro resses to in-
intermittent bluish discoloration o the ngertips on both arction o tiss e, req ently req irin di it amp tation.
hands when they are exposed to cold temperatures. Although
she believes the symptoms are worse now, she cannot recall
how long they have been present. She has never had ulcers on Further history reveals that the patient uses no medications.
her ngers or toes. She has been healthy all o her li e. Her mother and aunt have
had blue ngers in cold temperature, too. She denies tobacco
Question 11.10.1 With no urther in ormation, what is the use. T e review o systems is unremarkable.
most likely explanation or these digital color changes? Question 11.10.2 All o the ollowing are expected ndings
A) Atherosclerotic disease o the extremities. in patients with primary Raynaud phenomenon EXCEPT:
B) Acrocyanosis. A) Symmetric involvement o the hands.
C) Scleroderma. B) Well-demarcated cyanosis.
D) Physiologic response to cold. C) Digital ulcerations.
D) Normal ESR.
Answer 11.10.1 The correct answer is “D.” At this point all
we know is that this patient’s ngers turn blue upon exposure Answer 11.10.2 The correct answer is “C.” Primary Raynaud
to cold. T is is a normal physiologic response—vasoconstric- phenomenon is diagnosed when other causes o Raynaud phe-
tion in response to cold. “A” is incorrect. Peripheral vascular nomenon have been eliminated. Primary Raynaud phenomenon
disease is the result o atherosclerotic disease o the extremities is not typically destructive, and digital ulcerations generally occur
and typically occurs in older individuals; mani estations include when the phenomenon is secondary to some other disease process
claudication and skin ulceration. “B” is incorrect. Raynaud phe- (e.g., scleroderma and SLE). Primary Raynaud phenomenon is
nomenon must be distinguished rom acrocyanosis, (a rare almost always symmetric. Raynaud phenomenon can be di eren-
vasospastic disorder o persistent coldness and bluish discolor- tiated rom a normal response to cold temperatures by the demar-
ation o the hands and eet, not just ngers and toes, sometimes cation between pale or cyanotic ngertips and normal-appearing
ollowing a viral in ection). “C” is also incorrect. Scleroderma skin. A normal response to cold may include mottling, with indis-
by itsel does not cause blue ngers. tinct borders between pale and purple-colored skin, and paresthe-
sias o the involved area. In patients with Raynaud phenomenon,
the distal most portion o the involved digit is pale or cyanotic,
HELPFUL TIP: and the transition to normal skin color is abrupt. Since primary
Over 80% o cases o Rayna d phenomenon that pres- Raynaud phenomenon is not due to an in ammatory condition,
ent to a primary care physician’s o ice are d e to an markers o in ammation such as the ESR are usually normal.
exa erated physiolo ic response to cold or emotional
distress, which we call primary Rayna d phenomenon. HELPFUL TIP:
In most primary cases, patients report the onset o Nail old capillary microscopy (NCM) can be sed to ex-
symptoms in their teens or twenties. Altho h Rayna d amine the nail old capillaries o patients with Rayna d
phenomenon occ rs in most patients with scleroderma phenomenon. NCM involves placin a drop o oil (a drop
(90–95%), scleroderma is a rare disease and is m ch o s r ical l bricant also works well) on the c ticle o
less common than primary Rayna d phenomenon, so one or more di its and vis alization o the nail old cap-
the converse is not tr e. Th s, the presence o Rayna d illaries thro h an ophthalmoscope set at + 40 diopters
phenomenon is not synonymo s with the presence (40 reen). u s ally the rin and middle in ers (or symp-
o scleroderma. A very important cl e to s est that tomatic in ers) are examined. Normal capillaries are sym-
Rayna d is d e to an nderlyin systemic rhe matic dis- metric, non-dilated loops. Distorted, dilated, or absent
ease is onset a ter the a e o 40—these patients sho ld capillaries s est secondary Rayna d phenomenon.
328 FAMILY MEDICINE EXAMINATION & BOARD REVIEW

nonpitting edema o her ngers and hands. She has dif culty
You per orm a physical examination, including NCM which making a st. She has no skin ndings. Her CBC is normal.
is essentially normal. T ere is currently no discoloration o Her ANA is strongly positive with a nucleolar pattern.
the ngers.

Question 11.10.3 What is the next most appropriate step in Question 11.10.4 Which o the ollowing is the most likely
the evaluation and management o this patient? explanation o these ndings?
A) Cold provocation. A) Raynaud phenomenon.
B) Doppler ultrasound o the extremities. B) RA.
C) Blood viscosity testing. C) Scleroderma.
D) ANA. D) CRES syndrome.
E) A trial o therapy, nonpharmacologic and/or pharmacologic.
Answer 11.10.4 The correct answer is “C.” T is patient is now
Answer 11.10.3 The correct answer is “E.” Given that she is presenting with symptoms o scleroderma, also called systemic
symptomatic, a trial o drug therapy is warranted. Cold prov- sclerosis. Scleroderma encompasses a heterogeneous group o
ocation (“A”) is not recommended, as results are inconsistent. conditions, which are variable in severity but share a common
Rather, the diagnosis is based on a convincing history o epi- pathophysiologic mechanism— brosis o the skin and other
sodic cyanosis. Doppler ultrasound (“B”), might be use ul i organs. T e pathology o scleroderma also involves small ves-
you were considering large vessel disease rom atherosclero- sel vasculopathy, a process that leads to Raynaud phenomenon
sis, etc. However, it is not likely to be help ul in this patient. and ischemia o other tissues. Scleroderma may a ect the joints,
Blood viscosity testing (“C”), is not something that is done skin, lungs, kidneys, heart, and GI system. Fi y percent o
routinely. Measurement o ANA (“D”), can be help ul in ruling patients with scleroderma have depression.
out underlying autoimmune disease since patients with Rayn- A positive ANA can be help ul when you suspect sclero-
aud phenomenon and a positive ANA may have a higher risk o derma based on clinical ndings. ANA by indirect immuno u-
developing a systemic autoimmune disease. However, based on orescence will report the type o nuclear staining pattern. Cen-
her age, symmetric involvement, lack o ulcerations, and lack o tromere or nucleolar patterns are most highly associated with
other symptoms, this patient most likely has primary Raynaud systemic sclerosis, but other patterns (such as speckled) are also
phenomenon unrelated to an underlying disease. A second- seen. Patients with systemic sclerosis may have a negative ANA.
ary cause (e.g., scleroderma) is more likely with any o the ol- T ese ndings are now much more than you would expect to
lowing: patients who are age 30 years old or older; attacks are see with Raynaud phenomenon; thus, “A” is incorrect. Di use
asymmetric or associated with skin ulcers; there are symptoms swelling and sti ness is present rather than speci c synovitis,
suggestive o systemic involvement (e.g., arthralgias, dyspnea, so “B” (RA) is less likely. CRES syndrome (“D”) is an acronym
re ux, or weight loss). In addition to abnormal nail old capillar- standing or calcinosis cutis, Raynaud phenomenon, esophageal
ies, other subtle ndings to suggest secondary Raynaud include dysmotility, sclerodactyly, and telangiectasia, which are all man-
pu y hand swelling and telangiectasias on the ngers or around i estations that can occur in patients with systemic sclerosis.
the mouth. I you are concerned about secondary Raynaud phe- CRES should be used as a mnemonic to help remember the
nomenon, an ANA would be indicated. mani estation o systemic sclerosis, and not as diagnostic crite-
ria. O note, calcinosis is a relatively uncommon and late mani-
HELPFUL (FINGER) TIP: estation o systemic sclerosis.
Most patients with secondary Rayna d phenomenon
will develop symptoms o their nderlyin a toim- HELPFUL TIP:
m ne disease within a ew years o onset o Rayna d F rther dia nosis and cate orization o scleroderma
phenomenon. B t Rayna d phenomenon may occ r can be accomplished with the aid o more speci ic se-
years be ore other symptoms are prominent or bother- rolo ic tests s ch as anti-RNA polymerase III antibod-
some eno h or the patient to report them to a doctor. ies (which predict hi h risk or renal crisis) and anti-
topoisomerase antibodies (Scl-70) which predict hi her
risk or interstitial l n disease. Treatment or sclero-
A er reviewing the diagnosis with your patient, you make derma is evolvin , with some imm nos ppressive
several recommendations to help reduce the requency o therapies showin promise in the lon -term mana e-
attacks: avoid cold temperatures, reduce emotional stress, ment. Finally, the reatest treatment breakthro h or
and avoid tobacco and medications that cause vasoconstric- scleroderma patients is the indin that ACE inhibitors
tion (e.g., sympathomimetic drugs). can treat scleroderma renal crisis, leadin to sharp re-
Your patient returns 10 months later with new complaints. d ction in mortality and preservation o renal nction.
Both hands are swollen, sti , and pain ul. She complains o
multiple joint aches and atigue despite sleeping well. Her
appetite is normal, and she has no GI complaints. Although she Because o the digital ulcers you observed and the increasing
has no rash, she complains o itchy hands (surely a sign o can- pain o her Raynaud phenomenon attacks, you decide to pre-
cer, she thinks). On examination, you note that there is di use, scribe medication or Raynaud phenomenon.
CHAPTER 11 • RHEu MATOLOg Y 329

Question 11.10.5 Which o the ollowing is the best therapy


or reducing the requency o attacks o Raynaud phenom CASE 11.11
enon? A 17-year-old male presents to your of ce with a history
A) Amlodipine 5 mg daily. o low–back pain worsening over the past ew months. He
B) Ni edipine 10 mg as needed. recalls having intermittent back pain or at least a year. T e
C) Nitroglycerin 2% ointment daily. pain does not radiate. He runs cross-country and track with-
D) Diltiazem 30 mg as needed. out exacerbating his back pain. He denies evers, weight loss,
E) Prazosin 1 mg twice daily. weakness, incontinence, and history o trauma. He is unaware
o any amily history o back pain. He is otherwise healthy.
Answer 11.10.5 The correct answer is “A.” All o the medica-
tions listed are potentially help ul treatments when Raynaud phe- Question 11.11.1 What urther in ormation will help you
nomenon does not respond to conservative measures. However, di erentiate between potential causes o his back pain?
the most appropriate choice is daily amlodipine. Dihydropyridine A) Relie with acetaminophen.
calcium-channel blockers, like amlodipine, have been shown to B) Morning sti ness.
reduce the requency o attacks by about 50% compared with pla- C) Relie with rest.
cebo. T ey must be scheduled, rather than used as needed. In the D) A and C.
case o ni edipine, it is best to use an extended release version, as E) B or C.
the short-acting preparation is more likely to cause hypotension;
it also should not be used PRN (thus, “B” is incorrect). Diltia- Answer 11.11.1 The correct answer is “E.” Determining the
zem is not as e ective as the dihydropyridine calcium-channel pattern o the pain is critical. In ammatory causes o back pain
blockers. In systemic sclerosis, the bene t o calcium-channel are characterized by morning sti ness and improvement with
blockers is o en o set by their exacerbation o GERD. In these activity. In contrast, degenerative back disease causes pain that
patients, ACE inhibitors or angiotensin receptor blockers (ARBs) is exacerbated by activity and relieved with rest. In ammatory
are also bene cial or Raynaud phenomenon. opical vasodila- back pain typically presents in younger patients; degenerative
tors like nitroglycerin have a role i calcium-channel blockers are back pain typically does not present be ore age 30 or 40. Acet-
not e ective or the patient cannot tolerate them. T ere is less evi- aminophen and other analgesics may relieve pain rom either
dence that prazosin is e ective, but alpha-antagonists have a role category o disease and so will not help you narrow the diag-
i other therapies are unsuccess ul. Sildena l, a phosphodiester- nosis.
ase inhibitor, may promote healing o digital ulcers.
HELPFUL TIP:
Question 11.10.6 Which o the ollowing is NOT indicated In a yo n patient with back pain, also consider rac-
or acute ischemic crisis related to Raynaud phenomenon? t res (e. ., spondylolysis, a bilateral pars de ect), in ec-
A) Aspirin. tion, disc disease, and osseo s mali nancies.
B) Beta-blocker.
C) Ni edipine.
D) Digital or wrist nerve block.
E) opical nitroglycerin. On urther questioning, your patient reports morning sti -
ness and pain that improves with stretching. Activity does
not seem to aggravate his back but inactivity does. He has had
Answer 11.10.6 The correct answer is “B.” Beta-blockers are
no penile discharge, rash, or conjunctivitis and denies diar-
not indicated in the treatment o an ischemic crisis related to
rhea or other GI symptoms. On physical examination, your
Raynaud phenomenon. Beta-blockers actually cause periph-
patient is surprised to nd that he cannot reach his toes as
eral vasoconstriction and may worsen the problem. Peripher-
he had been able to do just a ew months ago in track prac-
ally, beta-agonists cause vasodilatation. When a beta-blocker is
tice. Range o motion in the neck, arms, and legs is normal. A
used, the patient ends up with an unopposed alpha-adrenergic
ocused neurologic examination is normal. T ere is mild, di -
response, which worsens vasoconstriction. All o the other
use tenderness over the lumbosacral spine and with percus-
therapies are use ul in acute ischemic crisis. O particular note
sion over the sacroiliac joints. When supine on the examina-
is “D.” A nerve or wrist block with lidocaine e ectively causes
tion table, a FABER maneuver ( exion, abduction, external
a sympathectomy, leading to decreased vascular tone. As men-
rotation) reproduces sacral pain.
tioned above, sildena l appears to be bene cial in healing isch-
emic ulcers and or acute ischemic crisis.
Question 11.11.2 The history and physical examination are
Objectives: Did you learn to . . . most consistent with a diagnosis o :
• Identi y complications o Rayna d phenomenon and dis- A) Reactive arthritis ( ormerly termed Reiter syndrome).
eases associated with it? B) Osteoarthritis o the lumbar spine.
• Develop a strate y or the prevention and treatment o C) Degenerative disc disease.
symptoms o Rayna d phenomenon? D) Ankylosing spondylitis.
• Reco nize clinical eat res o scleroderma? E) Vertebral body tumor.
330 FAMILY MEDICINE EXAMINATION & BOARD REVIEW

TABLE 11-8 PHYSICAL FINDINGS IN THE C) HLA-B27.


SPONDYLOARTHROPATHIES D) ESR.
E) Sacroiliac joint radiographs.
• Limited ran e o motion in the axial spine
• Peripheral arthritis
• Tenderness at the sacroiliac joints
Answer 11.11.3 The correct answer is “E.” T e hallmark o
• Enthesopathy (pain at tendon site insertion- partic larly the Achilles ankylosing spondylitis is sacroiliitis on pelvic radiographs.
tendon and plantar ascia synovitis) Although there are no universally accepted criteria or diagnos-
• Extra-artic lar symptoms (anterior veitis in 30–40%, psoriasis, ing spondyloarthropathies, x-ray evidence o sacroiliitis in the
inf ammatory bowel disease) setting o a consistent clinical picture is suf cient to diagnose
ankylosing spondylitis. Un ortunately, in early ankylosing spon-
dylitis, x-rays are o en inconclusive. MRI is more sensitive or
Answer 11.11.2 The correct answer is “D.” T e history and detecting early sacroiliitis, but the cost is higher and may not
physical examination are most consistent with ankylosing be necessary when plain x-ray images are strongly supportive
spondylitis, a seronegative (RF negative) spondyloarthropathy. o sacroiliitis. T e rest o the answers are incorrect. Ankylosing
Ankylosing spondylitis is the most common orm o spondylo- spondylitis is not associated with a positive ANA. Lumbar spine
arthropathy and is thought to have a prevalence o 1% in Cauca- x-rays may be use ul in ruling out other conditions and do show
sian populations. Historically, there is a 5:1 male-to- emale ratio some changes in ankylosing spondylitis but may miss early dis-
(but this may also be because ankylosing spondylitis is less rec- ease. HLA-B27, a class I HLA gene, is present in about 90% o
ognized in women). Studies based on radiographic appearance white patients and 50% to 80% o nonwhite patients with anky-
and human leukocyte antigen (HLA) B27 typing show a much losing spondylitis and is generally associated with spondyloar-
lower male: emale ratio. In addition to the historical eatures thropathies. However, HLA-B27 is not speci c. It is present in
mentioned in the answer to the previous question, there is o en many normal individuals as well and is o very little diagnostic
a amily history o ankylosing spondylitis in patients ultimately value (although it does have a good negative predictive value).
diagnosed with the disease. ESR is slightly elevated in most cases o spondyloarthropathy,
“A” is incorrect. Reactive arthritis occurs in reaction to an but again it is not speci c.
in ection (Chlamydia urethritis, GI in ection, etc.) and presents
with mono- or oligoarthritis, usually o the joints in the leg. T is Lumbar spine x-rays show squaring o the vertebral bodies.
patient’s age makes osteoarthritis and degenerative disc disease Pelvic x-rays identi y mild symmetric sacroiliitis. You are
less likely, so “B” and “C” are wrong. Also, he has no neurologic now con dent o the diagnosis o ankylosing spondylitis.
ndings that you might expect with disc prolapse. Malignancy
is associated with constitutional symptoms ( ever and weight Question 11.11.4 Which o the ollowing management
loss), neurologic involvement, and steadily worsening pain that plans is best or this patient?
is not relieved by activity—none o which are present in this A) Aspirin 650 mg by mouth our times daily (or Zorprin or
case. Physical examination ndings o spondyloarthropathies other extended release aspirin twice daily) and physical
are listed in able 11-8. therapy re erral.
B) Naproxen 500 mg twice daily and physical therapy re erral.
C) Orthopedic re erral or early surgical consideration.
HELPFUL TIP: D) Prednisone 40 mg daily and tting or a back brace.
The FABER (Flexion, Abd ction, External Rotation) test, E) Naproxen 500 mg twice daily and tting or a back brace.
also known as the “ i re 4” test, is per ormed by hav-
in the patient lie s pine, passively lexin the knee to Answer 11.11.4 The correct answer is “B.” NSAIDs are the
90 de rees, restin the patient’s oot on the contralat- mainstay o medical therapy or active phases o spondyloar-
eral thi h j st above the knee, and applyin press re thropathy. Patients generally experience signi cant relie rom
laterally and downward on the ipsilateral knee. When NSAIDs, and any additional bene t rom systemic corticoste-
viewed rom above, the patient’s le s take on the roids is questionable. Sul asalazine is a second-line drug. Inter-
“ i re 4” appearance. The pelvis sho ld be stabilized estingly, aspirin tends to be less e ective in these patients. T us,
by the examiner. A positive test occ rs i ipsilateral hip “A” is incorrect. T e management o ankylosing spondylitis
or sacroiliac (SI) joint pain is reprod ced. It is non-spe- should also include an exercise regimen designed speci cally
ci ic and can be positive in many diseases o the SI joint or the patient, and physical therapy may play a key role. T e use
and hip. o braces is not help ul and may actually exacerbate the patient’s
symptoms. Orthopedic surgery only becomes necessary in cases
o advanced ankylosing spondylitis when kyphosis or periph-
Question 11.11.3 Time or some tests! Which o the ollow eral joint symptoms become severe.
ing is the best initial test that can be used to con rm the
diagnosis o ankylosing spondylitis? A er 3 months o physical therapy and NSAIDs, your patient
A) ANA. returns. He has noticed minimal bene t, and now you nd
B) Lumbar spine radiographs. limitation o his neck range o motion.
CHAPTER 11 • RHEu MATOLOg Y 331

Question 11.11.5 What is the most appropriate next step


HELPFUL TIP:
in the evaluation and management o this patient?
The most common spondyloarthropathies are ankylos-
A) Discontinue naproxen and switch to indomethacin 50 mg
in spondylitis, psoriatic arthritis, enteropathic arthritis,
three times daily.
and reactive arthritis. These are patho enically similar
B) Begin methotrexate 15 mg per week and olic acid 1 mg
diseases that may be di ic lt to di erentiate in early
daily.
sta es, b t lack o di erentiation does not enerally
C) Begin prednisone 60 mg daily.
a ect therapy.
D) MRI o the cervical spine.
E) Continue naproxen and re er to rheumatology.
HELPFUL TIP:
Answer 11.11.5 The correct answer is “E.” T e patient is losing Enteropathic arthritis occ rs req ently in patients
range o motion despite NSAID therapy. Re erral to a rheuma- with in lammatory bowel disease, and has eat res in
tologist to consider anti- NF therapy (e.g., in iximab, adalim- common with ankylosin spondylitis, incl din in lam-
umab, or etanercept) is the best next step. Anti- NF agents can matory back pain and sacroiliitis. However, a reactive
halt disease progression and o en improve range o motion in spondyloarthropathy may also occ r a ter a g I in ection
early disease. “A” is incorrect. Switching NSAIDs may provide s ch as Salmonella, Yersinia, Shigella, or Campylobacter
some minor additional bene t but is unlikely to arrest disease and other in ections s ch as chlamydia. Finally, other ill-
progression. “B” is incorrect: methotrexate may be bene cial or nesses s ch as celiac disease may ca se a reactive spon-
the peripheral arthritis o spondyloarthropathies, but not or the dyloarthropathy.
axial skeleton disease. “C” is incorrect since systemic cortico-
steroids have no value in treating spondyloarthropathies. Local
corticosteroid injections are o en bene cial or enthesopathy Objectives: Did you learn to . . .
and peripheral arthritis. “D” is incorrect. MRI o the cervical • Reco nize clinical mani estations o spondyloarthropathies,
spine will show in ammatory changes o the spine, but will not partic larly ankylosin spondylitis?
change your management. • Di erentiate between types o spondyloarthropathies?
• Develop an appropriate eval ation or the patient present-
T e patient ails to keep his rheumatology appointment in with inf ammatory back pain?
and returns to you one year later. He is taking over-the- • Appropriately mana e a patient with ankylosin spondylitis?
counter naproxen. His neck is very sti , with even less range
o motion. He relates that he has been having crampy abdom-
inal pain, and loose stools with bloody mucous. He is having QUICK QUIZ: SPONDYLOARTHROPATHIES
dyspnea with mild exertion. On examination, his neck range
o motion is worse, and back exion is limited with straight- Which o the ollowing is NO a characteristic o reactive
ening o his lumbar lordosis. arthritis?
A) Conjunctivitis.
Question 11.11.6 What is the next most appropriate step B) Keratoderma blenorrhagicum.
in the evaluation and management o this patient? C) HLA-B8.
A) Discontinue naproxen. D) Urethritis.
B) Check CBC with di erential and iron panel. E) Arthritis.
C) Consult gastroenterology or EGD and colonoscopy.
D) Consult rheumatology and tell patient he must keep his The correct answer is “C.” Reactive arthritis ( ormerly Reiter
appointment. syndrome) is associated with HLA-B27 and not HLA-B8 (which
E) All o the above. is ound in sclerosing cholangitis). All o the other choices are
seen with reactive arthritis, including keratoderma blenorrhagi-
Answer 11.11.6 The correct answer is “E.” T e patient could cum, which is a rash ound especially on the soles o patients
have enteropathic arthritis in association with in ammatory with reactive arthritis. T e urethritis is generally due to Chla-
bowel disease. NSAIDs can exacerbate in ammatory bowel mydia trachomatis.
disease, and cause ulcers with chronic use, so discontinue
the naproxen (“A”). A CBC and iron panel may reveal iron HELPFUL TIP:
de ciency anemia, which would explain the patient’s dys- The onset o reactive arthritis is less insidio s than that
pnea on exertion. Subspecialty evaluations by a gastroenter- o ankylosin spondylitis, with some patients presentin
ologist and rheumatologist are indicated both to con rm the with an ac te illness that incl des ever, ac te joint swell-
diagnosis and to help with long-term disease management. in , and rash (keratoderma blenorrha ic m). g enerally,
Anti- NF agents—like in iximab and adalimumab—but not reactive arthritis resolves within 12 months. Antibiot-
etanercept—are bene cial or both the spondyloarthropathy ics are not e ective as a treatment o reactive arthritis
and associated in ammatory bowel disease in enteropathic (altho h may be indicated or the nderlyin in ection).
arthritis.
332 FAMILY MEDICINE EXAMINATION & BOARD REVIEW

CASE 11.12 TABLE 11-9 DIAGNOSTIC CRITERIA FOR


POLYMYOSITIS AND DERMATOMYOSITIS
A 52-year-old emale presents to your of ce or an initial
Three o o r are req ired:
visit and complains o mild pain and weakness in her hips
• Elevated m scle enzymes (CPKand aldolase)
and thighs. T e symptoms have been present or months. • Symmetrical proximal m scle weakness
About 2 years ago another doctor diagnosed her with psoria- • Abnormal EMg
sis because o a rash on her hands and elbows, which has since • Consistent ndin s on m scle biopsy
resolved. Otherwise, she reports being relatively healthy and
taking no medications. She is a smoker. She has had no recent Note: Skin ndin s m st be present in order to dia nose dermatomyositis.
health screening. On physical examination, her vitals are nor-
mal. She has considerable dif culty getting out o her chair.
should likewise be systemic. Local topical therapies do not
Her strength is symmetrically diminished in the quadriceps
treat the disease, and oral corticosteroids are typically the
and hip exors. T e rest o the examination is unremarkable.
rst-line agents. Hydroxychloroquine is help ul when the rash
does not respond to corticosteroids. Addition o immunosup-
Question 11.12.1 Which o the ollowing diagnostic tests
pressive drugs, such as methotrexate or azathioprine, may be
do you order rst?
indicated.
A) Muscle biopsy.
B) Electromyography (EMG).
Question 11.12.3 You diagnose dermatomyositis. Which o
C) SH.
the ollowing tests and exams should now be ordered?
D) ANA.
A) Pap and pelvic examination, CA-125, and pelvic ultrasound.
E) roponin- .
B) Screening mammogram.
Answer 11.12.1 The correct answer is “C.” Hypothyroid- C) Chest radiograph.
ism (along with many other diseases . . . . see below) can cause D) Fecal occult blood screening.
a proximal muscle weakness. A muscle biopsy or EMG study E) All o the above.
is premature without rst evaluating or myopathy with serum
enzyme levels (CK, aldolase). “D,” an ANA, is not likely to be Answer 11.12.3 The correct answer is “E.” Patients with adult-
help ul. Autoimmune diseases such as polymyositis and derma- onset dermatomyositis have a six- old increased risk o develop-
tomyositis cause myopathy, but ANA is not speci c or diagnos- ing cancer. T ere ore, upon diagnosing dermatomyositis, a thor-
ing these diseases. Finally, although CK-MB may be elevated ough history and physical examination must be per ormed. All
in patients with myopathy, troponin- (“E”) is an enzyme rela- patients should undergo age-appropriate cancer screening tests
tively speci c to cardiac muscle and should be normal. (e.g., Pap smear, ecal occult blood testing, and mammography).
Also, other tests generally recommended include CBC, meta-
You order SH and CBC, which are both within normal lim- bolic pro le, liver unction tests, urinalysis, and chest x-ray—
its. ESR, CK, and AS are elevated. You proceed to order that especially important in smokers. Since the patient is over age 50,
EMG which demonstrates abnormalities o the paraspinal colonoscopy is indicated. T e cancers most commonly associ-
muscles. Your patient returns to discuss her test results and ated with dermatomyositis include ovarian and gastric cancers
complains that her psoriasis is back. T ere are violaceous and lymphoma. T ere ore, it is reasonable to consider CA-125
plaques on her knuckles and elbows and around her eyes. and pelvic ultrasound.

Question 11.12.2 You recommend the ollowing manage


ment plan or the disease: HELPFUL TIP:
A) Ultraviolet light therapy. The association o dermatomyositis with g I mali nancy
B) opical corticosteroids. is partic larly stron . Yo may consider endoscopies
C) Oral corticosteroids. and an alpha- etoprotein ( or astric cancer). Some a -
D) opical emollients. thorities also s est CT o the abdomen and pelvis to
E) Acetaminophen. r le o t mali nancy.

Answer 11.12.2 The correct answer is “C.” T is patient now


has ndings that support a diagnosis o dermatomyositis. o
HELPFUL TIP:
diagnose idiopathic in ammatory myopathy, such as dermato-
Other ca ses o proximal m scle weakness incl de al-
myositis or polymyositis, certain criteria must be present (see
cohol se, m sc lar dystrophy, medications (e. ., peni-
able 11-9). T is patient has three o these criteria plus skin
cillamine and HMg -CoA red ctase inhibitors), C shin
changes consistent with dermatomyositis. Violaceous plaques
syndrome, viral in ections, hypothyroidism and diabetes
on the knuckles are commonly called Gottron papules. Similar
mellit s. The list oes on, incl din myasthenia ravis
lesions are o en observed over pressure points (e.g., elbows).
and Eaton–Lambert syndrome.
Because dermatomyositis is a systemic condition, treatment
CHAPTER 11 • RHEu MATOLOg Y 333

Objectives: Did you learn to . . . BIBLIOGRAPHY


• Reco nize symptoms and si ns o dermatomyositis and Aletaha D, et al. Rheumatoid arthritis classi cation criteria:
polymyositis? An American College o Rheumatology/European League
• Describe how inf ammatory myopathies are eval ated and Against Rheumatism collaborative initiative. Arthritis
dia nosed? Rheum. 2010;62(9):2569–2581.
• Appreciate the relationship between mali nancy and inf am- Álvarez-Lario B, MacArrón-Vicente J. Is there anything good
matory myopathy? in uric acid? Q J Med. 2011; 104(12):1015–1024.
Bhate C, Schwartz RA. Lyme disease: Part I. advances and
perspectives. J Am Acad Dermatol. 2011;64(4):619–636.
QUICK QUIZ: ARTHRITIS AND JOINT
Bhate C, Schwartz RA. Lyme disease: Part II. Management and
INVOLVEMENT prevention. J Am Acad Dermatol. 2011;64(4):639–653.
Compared with RA, osteoarthritis has a greater predilection or Botzoris V, Drosos AA. Management o Raynaud’s phenom-
which o the ollowing joints? enon and digital ulcers in systemic sclerosis. Joint Bone
Spine. 2011;78(4):341–346.
A) MCPs.
B) DIPs. Clauw DJ. Fibromyalgia: a clinical review. JAMA. 2014;311(15):
1547–55.
C) Knees.
D) Wrists. Dasgupta B, et al. Provisional classi cation criteria or polymy-
algia rheumatica. Arthritis Rheum. 2012;64(4):943–954.
The correct answer is “B.” Osteoarthritis tends to a ect the DIP Dougados M, Baeten D. Spondyloarthritis. Lancet.
2011;377:2127–2137.
and PIP joints in the hand, sparing the MCP joints, whereas RA
a ects the MCPs and spares the DIPs. Both disease processes Firestein GS, et al., eds. Kelley’s extbook o Rheumatology.
9th ed. Philadelphia: Elsevier Saunders; 2012.
may involve the knees and wrists.
Haile Z, Khatua S. Beyond osteoarthritis: recognizing and
treating in ectious and other in ammatory arthropathies
in your practice. Prim Care Clin Of ce Pract. 2010;37(4):
Clinical Pearls 713–727, vi.
Aspirate a monoarthritis and send synovial l id or cell co nt Khanna D, et al. American College o Rheumatology guide-
with di erential, ram stain, c lt re and crystal analysis or lines or management o gout part II: therapy and anti-
eval ation and r le o t o a septic joint. in ammatory prophylaxis o acute gouty arthritis.
Arthritis Care Res. 2012;64(10):1447–1461.
Be in s pplementation with calci m and vitamin D or
osteoporosis prevention with initiation o corticosteroids in Klippel JH, et al., eds. Primer on the Rheumatic Diseases.
the treatment o rhe matolo ic disease. 13th ed. New York: Springer; 2007.
Margaretten ME, et al. Does this adult patient have septic
Do not delay initiation o hi h dose corticosteroids or
arthritis? JAMA. 2007;297(13):1478–1488.
treatment o iant cell arteritis while awaitin a temporal
artery biopsy. Marmor MF, et al. Revised recommendations on screening or
chloroquine and hydroxychloroquine retinopathy.
Do not rely pon ser m ric acid levels to dia nose or re te Ophthalmology. 2011;118(2):415–22.
a o t attack.
Nam JL, et al. Ef cacy o biological disease-modi ying anti-
Do not ro tinely order “rhe matolo y panels.” Order rheumatic drugs: a systematic literature review in orming
appropriate rhe matolo ic tests based on clinical eval ation the 2013 update o the EULAR recommendations or the
and speci ic disease nder consideration. management o rheumatoid arthritis. Ann Rheum Dis.
Do not ro tinely screen or Lyme disease or chronic 2014;73:516–528.
m sc loskeletal pain. Repeat a ter s: “There is no s ch thin Pincus , Gibson KA, Shmerling RH. An evidence-based
as chronic Lyme disease.” approach to laboratory tests in usual care o patients
with rheumatoid arthritis. Clin Exp Rheumatol. 2014;
Do not se opioid pain medications in treatin chronic pain
32(Suppl 85):S23–8.
related to ibromyal ia as they tend to worsen central
sensitization and increase ibromyal ia pain over time. Singh JA, et al. 2012 updated to the 2008 American College o
Rheumatology recommendations or the use o disease-
Ens re that women o child-bearin a e on methotrexate modi ying antirheumatic drugs and biologic agents in
have reliable contraception. the treatment o rheumatoid arthritis. Arthritis Care Res.
Prescribe a trial o DMARDs (disease modi yin anti- 2012;64(5):625–639.
rhe matic dr s) as irst-line therapy in rhe matoid arthritis. Van Hecke O. Polymyalgia rheumatic-diagnosis and manage-
u se the lowest and minim m e ective prednisone dose or ment. Aust Fam Physician. 2011;40(5):303–306.
ac te symptomatic relie o in lammatory arthritis. Villa-Forte A. Giant cell arteritis: suspect it, treat it promptly.
Cleve Clin J Med. 2011;78(4):265–270.
Orthopedics and Sports Medicine
Ross E. M h sen, N chol s E w r s, Br L. M rcussen, n Jon N. V n Heukelom
12
General note: I you have a choice between acetaminophen and ROM testing, especially during abduction. Most children will
an NSAID or an acute injury, acetaminophen will always be be a ebrile with a temperature o ≤ 38°C.
the right choice. Most acute injuries are not in ammatory and
acetaminophen is a lot sa er without gastropathy or platelet in- Question 12.1.2 Appropriate diagnostic work up might
hibition. I you do use an NSAID, naproxen is the sa est rom a include which o the ollowing?
cardiovascular standpoint but carries the same gastrointestinal A) Joint aspiration.
risks as other NSAIDs. B) Plain lm radiographs.
C) In ammatory markers including erythrocyte sedimentation
CASE 12.1 rate (ESR) and C-reactive protein (CRP).
D) CBC with di erential.
A 5-year-old boy presents with acute onset o le t anterior E) All o the above.
thigh and hip pain that began 2 days ago with no known
prior trauma. He reports that it initially “loosened- Answer 12.1.2 The correct answer is “E.” All o the above may
up” a ter he had been out o bed or a ew hours but has be appropriate as transient synovitis is a diagnosis o exclusion.
become worse again by a ternoon. His pain is exacerbated Patients with mild symptoms may be observed without ur-
by weight bearing and active or passive range o motion ther investigation. However, i the pain is signi cant, ROM is
(ROM). His mother notes that he had a cold 7 to 10 days signi cantly impaired, or the temperature is > 37.5°C, urther
ago, but has been asymptomatic until he complained o diagnostic work-up is indicated. Laboratory ndings consistent
thigh pain two nights ago. She also notes that he has had a with transient synovitis include: clear joint uid aspirate, nor-
low-grade ever. He has no other signi icant constitutional mal CBC, and a mildly increased ESR. Blood cultures, antistrep-
symptoms and appears to be in some pain, but otherwise tolysin O (ASO) titer, bone scan, and MRI may also be o bene t
he appears well. to rule out other possibilities (e.g., septic arthritis, rheumatic
ever, and SCFE). It is o extreme importance to di erentiate
Question 12.1.1 Based on the in ormation obtained thus transient synovitis rom septic arthritis. Un ortunately, there
ar, which o the ollowing is the most likely diagnosis? is no combination o physical ndings and laboratory tests
A) Osteomyelitis. short o joint uid that will de nitively rule out septic arthri-
B) Rheumatic ever. tis. T ere are published clinical decision tools, but these are
C) Slipped capital emoral epiphysis (SCFE). generally based on observational data and o en have con ict-
D) Legg–Calve–Perthes disease (LCPD). ing results. It requires clinical judgment; decide which patients
E) ransient (toxic) synovitis. you are worried enough about that you want to commit them to
hip joint aspiration.
Answer 12.1.1 The correct answer is “E.” T is presentation
is classic or transient (toxic) synovitis. T is is the most com- HELPFUL TIP:
mon cause o hip pain in children aged 3 to 0 years, with peak Sep c r hr s s n or hope c emergency n com-
occurrence in ages 5 to 6 years. It is more commonly seen in monly presen s w h n elev e emper ure, gen-
boys (male: emale ratio o 2–3: ) and is o en preceded by a er l m l se, pronounce p n—o en w h sp sm n
viral respiratory in ection, although numerous studies have gu r ng. Gener lly, you w ll lso see n elev e wh e
ailed to demonstrate a speci c viral or bacterial agent. Physical bloo cell (WBC), CRP, n ESR, although neither these
examination reveals a limp or re usal to walk and complaint o or fever is a good enough indicator to rule out a
pain over the groin and/or proximal thigh. T ere is pain with
334
CHAPTER 12 • ORt HOPEd iCS a Nd SPORt S MEd iCiNE 335

TABLE 12-1 TERMINOLOGY FOR DESCRIBING Question 12.2.1 What is the most likely diagnosis in this
A FRACTURE patient?
A) Osteomyelitis.
Avulsion fracture: Fr gmen o bone pulle rom s norm l pos on
by muscul r con r c on or res s nce o l g men .
B) Septic arthritis.
Closed fracture: a r c ure h oes no commun c e w h he C) SCFE (Slipped Capital Femoral Epiphysis).
ex ern l env ronmen ( .e., no hrough he sk n). D) LCPD (Legg-Calve-Perthes Disease).
Comminuted fracture: Cons s ng o hree or more r gmen s. E) Sickle cell anemia.
Greenstick fracture: incomple e, ngul e r c ure o long bone,
p r cul rly n ch l ren.
Open fracture: Fr c ure h commun c es w h he ex ern l
Answer 12.2.1 The correct answer is “D.” LCPD is idiopathic
env ronmen . osteonecrosis o the emoral head. It is unilateral in 90% o
Torus (Buckle) fracture: d escr be s buckle r c ure or compress on cases, and the typical age range is 4 to 8 years, but patients may
o he bone w hou cor c l srup on. Seen espec lly n he s l be as young as 2 years and as old as 2 years. “A” is a possibility,
ore rms o ch l ren. although there should be evidence o osteomyelitis on MRI. “B”
is discussed in the previous case. “C,” SCFE, classically occurs in
obese children (male predominance) who are 0 to 6 years old.
septic joint by their normality or absence. Jo n lu “E,” sickle cell anemia, can cause osteonecrosis o the emoral
w ll h ve numerous WBCs (cl ss c e ch ng s > 50,000 head also, but the disease is rare in white populations. LCPD is
WBC/mm 3, however, coun s lower h n h s o no rule less common in black populations.
ou sep c r hr s (50,000 WBC/mm 3 s only 62% sen-
s ve w h 25,000 WBC/mm 3 s ll only 77% sens ve). Question 12.2.2 Which o the ollowing actors best pre
Bloo cul ures re pos ve n 30% o p en s. dicts a poor outcome or patients with LCPD?
A) Later age at onset o illness.
Question 12.1.3 What is the most appropriate treatment B) Findings o subchondral ractures or ragmentation.
or this patient with transient synovitis? C) Early appropriate treatment.
A) Open xation. D) Severity o pain and ability to bear weight.
B) Immobilization. E) Bilateral involvement.
C) Antibiotics.
D) Surgical decompression. Answer 12.2.2 The correct answer is “A.” Compared to older
E) Ibupro en and rest. children, younger children generally have a longer time or
remodeling to occur via molding o the emoral head within the
Answer 12.1.3 The correct answer is “E.” Conservative treat- acetabulum; and there ore, younger children have less attening
ment is warranted: the appropriate initial treatment is rest, o the emoral head.
weight bearing as tolerated, and observation. ransient syno-
vitis generally responds well to oral NSAIDs. Home care is Question 12.2.3 Which o the ollowing is the best initial
acceptable; however, admission is indicated i the diagnosis is treatment or this patient with LCPD?
equivocal or i signi cant pain management is required. For A) Joint replacement.
septic arthritis, prompt administration o an intravenous (IV) B) Osteotomy.
antibiotic—directed at the most likely in ecting pathogen and C) Rest and physical therapy.
altered as necessary based on culture results—is indicated. Sur- D) Opioids.
gical irrigation o the joint is o en necessary and early orthope- E) Corticosteroid injection o the hip joint.
dic consultation is needed.
Objectives: Did you learn to . . . Answer 12.2.3 The correct answer is “C.” T e initial treatment
• Recogn ze he cl n c l presen on o r ns en synov s? or a patient with LCPD typically includes rest, activity restric-
• d eren e r ns en synov s rom sep c r hr s? tion, and physical therapy. T e objectives are to increase ROM
• t re p en w h r ns en synov s? in the hip and to reduce the risk o signi cant de ormity. In gen-
eral, patients should be seen by an orthopedic specialist. T e
likelihood o a good outcome with nonoperative treatment is
CASE 12.2 signi cantly higher in younger patients, less than 8 years o age.
A 6-year-old white male is brought into clinic by his par- “A” is incorrect because joint replacement is not an option. “B” is
ents because he is complaining o pain in his hip and ante- incorrect. While osteotomy may be used, it is typically reserved
rior thigh. He is walking much less than usual since the pain or older children and patients who are not progressing well with
began about 4 weeks ago. You order a plain radiograph, conservative therapy. “D” and “E” are not appropriate interven-
which shows mild sclerosis with some increased density o tions in this patient with mild pain and no joint in ammation.
the emoral head. An MRI is ordered, which the radiologist LCPD is dif cult to treat largely because o the long duration o
interprets as “demonstrating osteonecrosis o the emoral treatment and activity restrictions required. Periods o rest with
head.” activity restriction, physical therapy, and surgical intervention
336 Fa MiLY MEd iCiNE EXa MiNa t iON & BOa Rd REViEW

may be indicated over 1 to 2 years o treatment and observa- Answer 12.3.2 The correct answer is “B.” T e most e ective
tion. Even with the best o care, prognosis is air with need or treatment modality is a combined physical therapy regimen
total hip replacement reaching approximately 50% by middle consisting o strength training o the hip abductors and quadri-
age due to severe degenerative arthritis. ceps, as well as quadriceps stretching. Quadriceps strengthening
is usually initiated by resisted straight leg raises (SLRs) to mini-
Objectives: Did you learn to . . .
mize patello emoral compressive orces. NSAIDs, cross-training,
• Recogn ze he cl n c l presen on o LCPd ?
and core strengthening may also be o bene t. Adjunctive tri-
• M n ge p en w h LCPd ?
als o therapeutic modalities such as orthotics may be consid-
ered, but should not be used in isolation. Recalcitrant cases
CASE 12.3 and patients with recurrent dislocation/subluxation should be
re erred to your riendly neighborhood orthopedic surgeon or
A 15-year-old emale cross-country runner presents to your
consideration o surgical intervention.
clinic with the chie complaint o bilateral knee pain. She
describes a gradual increase in her symptoms during the rst
T ree months later, the same patient presents complaining o
3 weeks o the season. She wants to run varsity this year and
unilateral right knee pain over the medial knee joint. Again,
has done extra running and hill training a er practice each
this pain is exacerbated by knee exion and she notes pop-
day. She describes anterior knee pain in the patellar region
ping and snapping when she stands rom sitting. She notes
with little or no swelling, but complains o crepitus and pain
that the pain is worse a er prolonged sitting or going up or
exacerbated by running, squatting, stair climbing, and pro-
downstairs. Your examination shows tenderness about 1 cm
longed sitting with the knee bent.
medial to the patella with palpable ullness in the area.
Question 12.3.1 The most likely diagnosis or the condition
Question 12.3.3 The most likely diagnosis is:
described is:
A) Osteosarcoma
A) Osgood–Schlatter disease.
B) Medial collateral ligament (MCL) strain
B) Chondromalacia patellae.
C) Plica syndrome
C) Patello emoral pain syndrome (PFPS).
D) Recalcitrant PFPS
D) Femoral stress racture.
E) Meniscal tear
E) Jogger’s joints.
Answer 12.3.3 The correct answer is “C.” T is is the typical
Answer 12.3.1 The correct answer is “C.” PFPS is a common presentation o plica syndrome. Plicae are synovial remnants
overuse syndrome seen more requently in runners and emale that did not resorb properly during development. T ey can be
athletes (thus the moniker “runner’s knee”). T is condition is due irritated, usually chronically or subacutely, especially in sports
to orces across the knee that lead to biomechanical overload o that require repeated exion o the knee (e.g., rowing, cycling,
the patello emoral joint and other anterior knee structures. Mal- running etc.). ypical symptoms include popping or snapping
tracking and malalignment o the patello emoral joint can con- sensation with knee exion; there may also be knee locking and
tribute to this overload, as can training errors, core weakness, and catching as well as having the knee “give.” reatment includes
muscle imbalance. “B,” Osgood–Schlatter disease, is also related rest, ice, quadriceps strengthening, and NSAIDs. I conservative
to overuse but is 2 to 3 times more common in males, particularly management ails, steroid injection or arthroscopy may alleviate
in athletes engaging in repetitive jumping. T e pain o Osgood– the symptoms.
Schlatter is generally well localized to the tibial tubercle. Radio-
graphic evidence o ragmentation o the epiphysis or hetero-
HELPFUL TIP:
tropic ossi cation anterior to the tubercle may be seen but is not
t he pl c shoul be p lp ble. a me l/ n er or pl c s
necessary or diagnosis. “C,” chondromalacia patella, is so ening
he mos common (be ween he p ell n he me l
o the articular cartilage o the patella as seen on arthroscopy and
jo n l ne). i c n lso occur l er lly n e her bove or
may be a result o long-term patello emoral dys unction. T is is
below he m -pole o he p ell .
a surgical diagnosis and the term should be avoided clinically.
Femoral stress racture (“D”) would be unlikely to present bilat-
erally. “E” is not a real thing but has nice alliteration.
Objectives: Did you learn to . . .
• i en y n m n ge PFPS?
Question 12.3.2 What is the pre erred treatment or this • Recogn ze pl c syn rome?
emale runner?
A) Arthroscopic debridement.
B) Decreased activity level along with and quadriceps and hip
CASE 12.4
strengthening exercises. A mother brings her 18-month-old son in or well child
C) Evaluation or “Female Athlete riad.” check. Her only new concern today is that he seems to walk
D) Casting or immobilization. “bow-legged.” He has been somewhat pigeon-toed since he
E) Corticosteroid injection. could pull himsel up and cruise along walls and urniture at
CHAPTER 12 • ORt HOPEd iCS a Nd SPORt S MEd iCiNE 337

home. On examination, you nd the child’s eet to be point-


ro on ( n evers on). For ex ern l ro on (re rover-
ing inward. T e oot is exible and looks normal. T e patellae
s on), pl ce one h n on he bu ocks n move one leg
are in a neutral position acing directly orward.
hrough m l ne un l he pelv s beg ns o l . t yp c lly,
norm l ex ern l ro on s 45 egrees, n norm l n-
Question 12.4.1 What is the most likely diagnosis in this
ern l ro on s 35 egrees. Ro on gre er h n h s
patient?
s cons ere excess ve s ccomp n e by l m e
A) Cerebral palsy.
ROM n he oppos e rec on. In a ddition, when a pa -
B) Excessive emoral retroversion.
tient with femora l torsion is sea ted with legs da ngling,
C) Fore oot varus.
the pa tella e will not fa ce forwa rd (they fa ce inwa rd for
D) Internal tibial torsion.
intoeing a nd outwa rd for outtoeing).
E) Bilateral developmental dysplasia o the hip (DDH).

Answer 12.4.1 The correct answer is “D.” T is represents


a case o intoeing (pigeon-toeing), which is o en caused by Objective: Did you learn to . . .
internal tibial torsion. Internal tibial torsion is characterized by • Ev lu e n m n ge n oe ng n ou oe ng n ch l ren?
a exible, normal oot, with the patellae in a neutral position.
T e condition can be diagnosed by examining the child on his CASE 12.5
knees. Normally, there should be approximately 30 degrees o
external rotation o the eet in this position. With internal tibial A 13-year-old male presents to the clinic with his mother or
torsion, the toes will be pointing inward. In addition, when the dif culty walking. He is unsure o when the problem rst
child is sitting with legs dangling over a table, the lateral malleo- began, but has noticed it getting worse over the last week. It
lus will be anterior to the medial malleolus, which is the oppo- has orced him to stop playing sports. He reports a dull pain
site o what is normally observed. Finally, the hips must be nor- in the le hip but denies trauma. On examination, you nd
mal in order to con rm this diagnosis. “A” is incorrect because an obese male in no distress. T ere is loss o internal rotation,
you would expect other ndings in a patient with cerebral palsy. abduction, and exion at the le hip joint. When his hip is
“B” is incorrect. In act, emoral retroversion is synonymous exed to 90 degrees, this loss o ROM is more pronounced.
with “outtoeing,” which is the opposite o intoeing (o course!).
Excessive emoral anteversion can be a cause o intoeing. “C” Question 12.5.1 What is the most likely diagnosis in this
is incorrect because the oot examination does not show varus case?
de ormity. Finally, “E” is incorrect, although bilateral DDH can A) Osteoarthritis.
be quite dif cult to diagnose i not caught early in the newborn B) Septic arthritis.
period. With DDH one would expect external rotation o the C) SCFE.
leg rather than intoeing. D) LCPD.
E) Juvenile rheumatoid arthritis.
Question 12.4.2 What is the treatment o choice or this
child? Answer 12.5.1 The correct answer is “C.” SCFE occurs most
A) Re erral or bilateral osteotomy. commonly in active, overweight, adolescent males. Shear orces
B) Shoe modi cation and bracing. across the relatively weak physis causes displacement. Slip-
C) Physical therapy re erral. page is generally gradual, but may occur acutely. Mean age
D) Serial casting. at presentation is 2 or emales (range 0– 4) and 3 or males
E) Reassurance and watch ul waiting. (range – 6). Endocrinopathies, such as hypothyroidism and
osteodystrophy, should be considered in those presenting atypi-
Answer 12.4.2 The correct answer is “E.” Spontaneous resolu- cally or outside the typical age range. Watch or development o
tion is the norm or most intoeing and outtoeing de ormities. a similar process in the contralateral hip over time, which has
Most will spontaneously correct by age 6. Children continu- been reported anywhere rom 20% to 80% o the time.
ing to have dif culty with persistent trips and alls or grossly
unsightly gait beyond this time may bene t rom a rotational Question 12.5.2 Which o the ollowing is the rst study you
osteotomy. Children with neuromotor disorders and cerebral order to con rm the diagnosis?
palsy are more likely to require surgical intervention. A) AP and rog-leg lateral radiographs o the hip.
B) C scan o the hip/pelvis.
C) MRI o the hip/pelvis.
HELPFUL TIP: D) ESR.
a no her c use o n oe ng s emor l ors on. Femor l E) None; physical examination is suf cient.
ors on s gnose by pl c ng he p en prone w h
he h ps n neu r l n knees lexe o 90 egrees. Fee Answer 12.5.2 The correct answer is “A.” Radiographs o the
re ro e w y rom m l ne o me sure n ern l hip should demonstrate displacement o the emoral head,
which can then be classi ed as mild, moderate, or severe.
338 Fa MiLY MEd iCiNE EXa MiNa t iON & BOa Rd REViEW

“B” and “C” are incorrect because the radiograph is diagnostic be elt at the ankle, knee, or thigh. Pain may be elt during the
in most cases. “D” is incorrect since SCFE is not an in amma- day a er vigorous activities but is more common in the evening
tory condition. “E” is incorrect. Imaging should be obtained in or causing awakening at night. Presentation with constitutional
order to con rm the diagnosis and rate the severity. MRI may be symptoms should lead to radiographic and/or metabolic evalu-
able to diagnose an early SCFE when radiographs are negative. ation.

Question 12.5.3 The treatment o choice or this patient is: Question 12.6.2 Which o the ollowing should you enter
A) Antibiotics. tain when a patient presents with typical growing pain?
B) Immobilization. A) Osteomyelitis.
C) Physical therapy. B) Malignancy.
D) Surgical decompression. C) Juvenile rheumatoid arthritis.
E) Surgical xation. D) All o the above.
E) None o the above.
Answer 12.5.3 The correct answer is “E.” T e goals o treat-
ment o SCFE are to prevent urther slippage, promote closure Answer 12.6.2 The correct answer is “D.” It is important to
o the physis, and to minimize the risk o osteonecrosis or chon- consider other potential causes o what would otherwise appear
drolysis. T ese aims are best accomplished through re erral to to be growing pain. Although you may not nd it necessary to
an orthopedic surgeon and, ultimately, surgical xation. “B” per orm any laboratory or radiologic studies, you should at least
and “C” are incorrect because they delay de nitive therapy and keep these and other diagnoses in mind when taking your his-
will not produce the desired result. “A” and “D” are incorrect tory and per orming your examination. Remember, growing
because SCFE is not an in ectious process. pain is a diagnosis o exclusion.
Objectives: Did you learn to . . .
• Recogn ze he cl n c l presen on o ch l w h SCFE?
HELPFUL TIP:
• t re p en w h SCFE? Severe or pers s en p n ur ng he y is not “grow ng
p n.” By e n on, “grow ng p n” occurs pr m r ly
n gh n s be er ur ng he y.
CASE 12.6
A worried mother presents with her 4-year-old son or evalu-
ation o lower extremity pain. She reports the boy has com- Question 12.6.3 Treatment or growing pain includes:
plained o some vague bilateral leg pains over the past several A) Reassurance, rest, and short-term use o NSAIDs.
weeks a er vigorous physical activities. She became alarmed B) Amputation.
a er he had awakened the past two nights crying in pain. C) Chemotherapy and radiation.
T e boy reports the pain is hardly noticeable during the day. D) Casting and bracing ollowed by physical therapy.
Recently, the pain has been in the bilateral distal thighs; how- E) Staging o the disease is required prior to initiation o
ever, his mother notes times o unilateral pain. T e boy and therapy.
his mother both deny constitutional symptoms now or over
the past several weeks. Examination reveals an a ebrile, well- Answer 12.6.3 The correct answer is “A.” OK, so this was just
developed male in no distress. T e musculoskeletal examina- a un one. Do not do anything drastic or a benign condition!
tion is normal. T ose o you who chose “B,” amputation, will nd themselves
in one o Dante’s circles (or in court—which is worse?) . . . and
Question 12.6.1 The most likely diagnosis or the condition clearly need a co ee break.
described above is?
A) Ewing sarcoma. T e same boy returns with his mother years later. He is now
B) Growing pain. 12 years old and requires a physical examination or junior
C) Kohler disease. high school sports. You plan to evaluate him or scoliosis.
D) Leukemia.
E) Osteochondritis dissecans. Question 12.6.4 Which o the ollowing screening methods
is the most sensitive or detecting scoliosis?
Answer 12.6.1 The correct answer is “B.” Growing pain is a A) Observe the patient rom the ront with a loose- tting shirt
diagnosis o exclusion, although history and physical examina- on. Measure the di erence in shoulder height.
tion usually suf ce or excluding more serious diagnoses. It is B) Observe the patient rom behind, with shirt o , while he
a condition o unknown etiology, but is thought by some to be a bends orward at the waist. Look or elevation o the ribs or
result o overuse/over activity on an immature musculoskeletal paravertebral muscle mass on one side.
system. It is most requently seen in otherwise healthy, active C) Observe the patient rom the ront, with shirt o , while he
children aged 2 to 5, with some older children a ected as well. bends orward at the waist. Look or elevation o the ribs or
Pain is commonly bilateral or/and localized to the cal , but may paravertebral muscle mass on one side.
CHAPTER 12 • ORt HOPEd iCS a Nd SPORt S MEd iCiNE 339

D) Observe the patient rom the side, with shirt o , while he Answer 12.6.6 The correct answer is “B.” In an otherwise
bends orward at the waist. Look or elevation o the ribs or healthy patient with a curvature measured at < 25 degrees,
paravertebral muscle mass on one side. observation is appropriate. “C” is incorrect because physical
therapy and exercise regimens do not seem to limit the progres-
Answer 12.6.4 The correct answer is “B,” which is known as sion o scoliosis. “A” and “D” are incorrect because bracing and
the “ orward bending test.” T is test is more sensitive than the surgery are typically not warranted or this degree o scoliosis.
other methods described. T e orward bending test is accom- Repeat examination and possibly repeat radiographs are war-
plished by having the patient bend at the waist with eet together ranted, but i the scoliosis remains stable and mild, the patient
and hands hanging ree. Observe the patient rom behind and is not likely to experience any signi cant progression o disease
note any elevation o the ribs or paravertebral muscle mass on with aging.
one side. T e elevation should be measured in degrees (incli-
nometers are available), and an inclination o 5 degrees or more HELPFUL TIP:
should be evaluated urther. Options “A,” “C,” and “D” are not Br c ng or scol os s shoul be l m e o hose w h -
accepted methods o screening or scoliosis. op h c scol os s n 25 o 45 egrees o ngul on.
Br c ng s only e ec ve he ch l s s ll grow ng n
HELPFUL TIP: <1 ye r p s men rche em le.
Routine screening for scoliosis is a recommenda-
tion “D” by the US Preventative Services Task Force.
Rou ne scol os s screen ng is not recommen e . i m y T e mother returns now with her 2-year-old daughter who is
be ppropr e n p en s who h ve no ce p n or re using to move her right arm. Earlier today she threw a tan-
some o her bnorm l y. d on’ o ! trum at the store when her ather re used to buy her the new
“princess toy” (advertising hits them young). Dad was hold-
ing on to her arm when she opped to the oor. She immedi-
HELPFUL TIP: ately began crying and re using to move her right arm. In the
Scol os s s l er l curv ure o he sp ne, usu lly c- of ce, she is well but holds her right arm adducted, exed,
comp n e by ro on n gener lly occurr ng n he and pronated. (T e princess toy is in the other hand. Guilt is
hor c c or lumb r re s. i c n occur w h excess ve a power ul weapon.) Despite every trick you know, you can’t
kyphos s (pos er orly convex curv ure) or lor os s ( n- get her to move that arm. You inspect and palpate the entire
er orly convex curv ure). extremity and clavicle and nd no crepitus, swelling, or ten-
derness.

Question 12.6.7 What is your next diagnostic step?


On orward bending test, you nd slight elevation o the A) Obtain an x-ray o the elbow.
le paravertebral muscles mass, which you estimate to be B) Actively supinate the orearm and ex the elbow while
7 degrees. T e remainder o the examination is normal. You applying pressure over the radial head.
decide to obtain radiographs that show 12 degrees o angula- C) Actively twist the orearm at the elbow 360 degrees.
tion (Cobb angle). D) Consult orthopedics.
E) Per orm a skeletal survey, mostly to bide time.
Question 12.6.5 This patient’s scoliosis is most likely:
A) Congenital. Answer 12.6.7 The correct answer is “B.” T is child has a
B) Idiopathic. “nursemaid’s elbow” that is due to subluxation o the annular lig-
C) Related to a tumor. ament rather than subluxation or dislocation o the radial head.
D) Secondary to in ection. It occurs in toddlers due to traction via pulling on a pronated
and extended arm. Symptoms are immediate and care is sought
Answer 12.6.5 The correct answer is “B.” Most scoliosis that due to child’s re usal to move the arm. T e diagnosis is clinical.
develops during adolescence is idiopathic. When there is no Manual reduction may be done via supination/ exion or hyper-
pain, ever, weight loss, or other warning signs (e.g., neuro- pronation. Sedation is not needed. A palpable click may be elt.
logic symptoms), the curvature is unlikely to be due to tumor T e child usually regains immediate movement o the arm and
or in ection. “A,” congenital scoliosis, typically presents earlier relie o discom ort. Immobilization is not needed but parents
in li e. should be told that recurrent subluxations may occur and there-
ore pulling on the arm should be avoided. “A” would be correct
Question 12.6.6 The most appropriate initial management i there was concern or a racture based on ndings o swell-
plan or this patient includes: ing, history o trauma, or ocal tenderness. “C” is impossible:
A) Bracing. Can you really rotate a joint 360 degrees? “D” is incorrect as any
B) Observation. primary-care provider may manage this. “E” is unnecessary as a
C) Physical therapy. nursemaid elbow is not a marker o abuse, although traumatic
D) Surgery. injuries in children should always make one consider abuse.
340 Fa MiLY MEd iCiNE EXa MiNa t iON & BOa Rd REViEW

Answer 12.6.8 The correct answer is “B.” T is is a typical


HELPFUL TIP:
“toddler’s racture” that consists o a spiral racture o the tibia
Re uc ng nursem ’s elbow s gr y ng. t he p en
usually rom insigni cant rotational trauma (e.g., running and
w h nursem ’s elbow shoul be us ng he rm nor-
alling with a twisting motion). T ere should not be an associ-
m lly w h n m nu es. i he ch l s ll re uses o use he
ated bular racture. “A” is incorrect because this type o rac-
ex rem y er equ e observ on, recons er your
ture is not usually rom abuse. A mid-sha racture would more
gnos s n whe her he re uc on w s success ul.
likely be rom abuse. “C” is incorrect because this racture needs
No e h m ny w ll spon neously re uce wh le r o-
to be treated.
gr phs re be ng one ( her peu c x-r y?). Bo h lex on
n sup n on n ex ens on n pron on h ve been Objectives: Did you learn to . . .
use o re uce nursem s elbow. • Cons er bro er eren l gnos s n p en
presen ng w h yp c l “grow ng p n?”
• in e conserv ve re men or p en w h grow ng p n?
HELPFUL TIP: • Screen p en or scol os s?
When e l ng w h pe r c or hope cs, lw ys re-
• d evelop n ppro ch o he olescen w h scol os s?
member h ch l buse s n he eren l g-
• Recogn ze he cl n c l presen on n re men o
nos s. Be sure h he repor e mech n sm o he
“nursem ’s elbow” (r l he sublux on)?
njury s cons s en w h he n ngs on ex m n on n
• Recogn ze o ler’s r c ure?” ( b l sp r l r c ure).
r ogr ph.

CASE 12.7
A er success ul reduction, you see the same 2-year-old child A mother presents to your clinic with her 10-month-old
a month later presenting to your clinic with the parents who son. She is concerned that he has been very irritable since
state that the child has been crying and has re used to walk she arrived home rom work, particularly when his legs are
since tripping over a toy a couple o hours ago. You look at touched. She states that he has been crying “nonstop” since
the child and nd no signs o abuse. T e injured area is rep- she arrived home. He was well when she le or work this
resented by the lower leg image in Figure 12-1. morning, leaving the patient under the care o her boy riend,
ony “the Hustler” Jones. T e patient’s mother denies any
Question 12.6.8 Your approach at this point is to: evers, vomiting, diarrhea, or cough. On examination, he is
A) Consult Child Wel are since this is almost always abuse generally ussy, worsening with palpation o his le leg, par-
B) Consult orthopedics or casting and urther treatment. ticularly around his knee. You also note ve bruises on his
C) No treatment necessary or this particular racture in a anterior and posterior chest. T e remainder o the examina-
2-year-old. tion is unremarkable. You obtain the ollowing radiograph o
D) Obtain an MRI. his le knee (Fig. 12-2).

Question 12.7.1 Which o the ollowing mechanisms is most


likely responsible or the racture seen on x ray (marked
with the help ul arrow sign)?
A) A low-energy all ( all o a low couch).
B) A high-energy all ( all out o a second story window).

FIGURE 12-1. FIGURE 12-2.


CHAPTER 12 • ORt HOPEd iCS a Nd SPORt S MEd iCiNE 341

C) A low-energy rotational orce (tripping causing the oot to • Un ers n wh ch r c ures re spec c or non cc en l
twist inward). r um ?
D) A high-energy tension or shearing orce (a violent twisting
o the leg). QUICK QUIZ: t HE BURNiN’HiP O’FiRE
E) Any o the above could result in the injury.
A 35-year-old obese emale with a large pannus presents with
Answer 12.7.1 The correct answer is “D,” a high energy
a burning pain in anterior–lateral thigh. It started when she
tension or shearing orce. T e above radiograph depicts a
bought a new wide belt (ornate with skulls and Harley–Davidson
metaphyseal corner racture that is highly speci c or nonac-
logos) and started wearing hip hugger pants in order to display
cidental trauma (that’s the nice way to say “child abuse”). T ese
her taste ul sacral tattoo (yikes!). T e pain seems worse on days
ractures are typically caused by shaking o the child with ail-
when she wears the belt and seems better on days when she
ing o the extremities or with orce ul yanking or twisting or the
hangs out au naturel at home.
arm or leg. T e incidence o these ractures decreases substan-
tially as children grow beyond year o age.
T e most likely diagnosis is:
A) Meralgia paresthetica.
Question 12.7.2 In which o the ollowing patients is a skel B) Supratrochanteric bursitis.
etal survey indicated or concern o nonaccidental trauma? C) ropical pyomyositis.
A) A 2-month-old male with multiple bruises but no other D) Ruptured plantaris muscle.
localizing injuries. E) rucker’s thigh.
B) A 2-year-old emale with an intracranial injury on brain C .
C) A 6-month-old male with a known metaphyseal corner The correct answer is “A.” Meralgia paresthetica is described as a
racture. burning pain in the anterior–lateral thigh and is caused by com-
D) An 8-month-old emale with burns consistent with non- pression o the lateral emoral cutaneous nerve. It is most com-
accidental trauma. mon in those who wear tight jeans and wide belts and who are
E) A skeletal survey is indicted or all the above patients. obese. reatment is to relieve the inciting cause. ricyclic antide-
pressants or other medication or neuropathy may also be help-
Answer 12.7.2 The answer is “E,” all o the above. T e Ameri- ul. “B,” supratrochanteric bursitis, occurs when there is in am-
can Academy o Pediatrics consensus guidelines on the radio- mation o the bursa over the greater trochanter o the emur.
graphic evaluation o child abuse state that a skeletal survey Patients complain o pain in the lateral hip and outer thigh. T ere
should be obtained or all children younger than 2 years o is point tenderness over the greater trochanter. reatment is phys-
age who may have experienced maltreatment. Skeletal surveys ical therapy or core strengthening and a steroid injection o the
should also be considered or children between the ages o 2 and bursa. “C,” tropical pyomyositis, is a deep in ection o the muscle
5 depending on the clinical scenario. T e ollowing images are that is o en caused by Staphylococcus aureus. It is increasingly
generally included in a skeletal survey: humeri, orearms, lower recognized in the United States. “D,” a ruptured plantaris muscle,
legs, hands, eet, skull, cervical spine, thorax (including oblique occurs in the cal and is not related to the thigh. It is o en caused
views), lumbar spine, and pelvis. by planting o the oot orce ully (such as in tennis or basketball)
and patients may report a “pop” ollowed by cal /gastrocnemius
Question 12.7.3 Which o the ollowing ractures is consid pain. “E” is not a real thing, we are sorry to say.
ered highly speci c or nonaccidental trauma?
A) Posterior rib ractures. CASE 12.8
B) Clavicle ractures.
C) Distal radius ractures. A concerned ather presents to your clinic with a 1-month-
D) Spiral ractures o the tibia. old daughter. He is worried that his daughter appears to be
E) Supracondylar ractures. pigeon-toed, and his mother-in-law is sure (as most mothers-
in-law are) that the child will need immediate surgery or cor-
Answer 12.7.3 The correct answer is “A,”posterior rib ractures. rection. Your examination reveals a pleasant, well-developed
In addition, the ollowing ractures have moderate to high speci c- 1-month-old emale with moderate medial deviation o the
ity or nonaccidental trauma: metaphyseal ractures, multiple rac- ore oot bilaterally. A line bisecting the heel passes through
tures (especially i bilateral), ractures o di erent ages, epiphyseal the ourth toe on each oot. T e lateral borders o the eet are
separations, vertebral body ractures, digital ractures, scapular convex; the heels are in a normal neutral position. T e eet
ractures, sternal ractures, and complex skull ractures. are exible and the de ormity can be passively corrected to
the midline.
Objectives: Did you learn to . . .
• Recogn ze me physe l corner r c ure n wh mpl c - Question 12.8.1 The above description is best character
ons h s r c ure h s? ized as?
• Know he n c ons o per orm skele l survey when sus- A) Club oot.
pec ng non cc en l r um n wh m ges re nclu e ? B) Internal tibial torsion.
342 Fa MiLY MEd iCiNE EXa MiNa t iON & BOa Rd REViEW

C) Flexible at oot.
D) Excessive emoral anteversion. CASE 12.9
E) Metatarsus adductus. A 5-year-old girl who has recently recovered rom chicken
pox (“T ank you, anti-vaccine parents!”) presents with her
Answer 12.8.1 The correct answer is “E.” T e above is a clas- mother or evaluation o le leg pain and re usal to walk. T e
sic description o the oot shape o metatarsus adductus. In the mother reports that she has complained o worsening pain
normal oot, a line bisecting the heel would pass through the over the last 4 to 5 days. She started to limp noticeably yes-
second and the third toe webspace. In those with metatarsus terday and re used to walk this morning. Also, the mother
adductus, it passes more laterally depending on the severity. reports general malaise and subjective ever. T e patient
In those with the most severe de ormity, a line bisecting the complains o pain over the distal thigh and knee. T e mother
heel would pass through the ourth and the h toe webspace. has not seen any swelling o the knee, and the patient denies
In addition to the heels remaining in a neutral position— trauma.
indicating that the problem is isolated to the shape o the Examination reveals a pleasant 5-year-old emale who
oot and not to an internal rotation o the tibia—the ore oot appears uncom ortable, but nontoxic and in no acute dis-
is lexible and easily straightened into normal position. his tress. Her temperature is 38.5°C. T e le distal thigh (not the
is as opposed to metatarsus varus in which the ore oot is joint) is pain ul to palpation, and slightly warm. T e knee
rigid. joint has no e usion and the ROM is ull with only mild dis-
com ort on knee motion. T ere is no hip joint involvement.
Question 12.8.2 The most appropriate treatment or the
above case would be? Question 12.9.1 What is the most common bacterial patho
A) Surgical re-construction. gen associated with this patient’s condition?
B) Serial casting. A) Group A Streptococcus.
C) Physical therapy re erral or stretching and exercise. B) Group B Streptococcus.
D) Watch ul waiting and reassurance. C) Haemophilus in uenzae.
E) Orthopedic out are shoes. D) S. aureus.
E) None o the above. T is is not an in ection.
Answer 12.8.2 The correct answer is “D.” Spontaneous correc-
tion occurs in most children. Parents can begin a regimen o Answer 12.9.1 The correct answer is “D.” T e acute nature o
gentle oot stretching. It gives them something to do but does the symptoms, presence o ever, and minimal involvement o the
not change the outcome. Care should be taken to avoid keeping joint makes osteomyelitis the most likely diagnosis. In addition,
the child in the prone position with the eet in an inward posi- osteomyelitis is associated with chicken pox in children. T ere is
tion. I the de ormity is severe and in exible, serial casting clearly an increase in the prevalence o MRSA as a pathogen in
and/or surgical intervention may be indicated. A rigid meta- osteomyelitis (two-thirds o cases in a recent review). T us, it is
tarsus adductus in a child > 3 months or a residual problem in a wise to choose an antibiotic with MRSA coverage.
child with a exible metatarsus adductus > 6 months is an indi-
cation or pediatric orthopedic re erral.
HELPFUL TIP:
Pseudomonas aeruginosa s commonly ssoc e w h
HELPFUL TIP: os eomyel s n he se ng o pl n r punc ure woun
Club oo ( l pes equ nov rus) s observe b r h. P - hrough enn s shoe.
en s re more o en m les (2:1 m le: em le r o) n
presen w h he ollow ng n ngs: c lc neus seems
o be r wn nw r n upw r , mo er e ore oo Allow us to digress.
uc on, n he oo c n be pl ce n neu r l po-
s on by p ss ve m n pul on. t hese p en s c n be Question 12.9.2 The most common organism causing
re e success ully w h e rly re err l or ser l c s ng osteomyelitis in patients with sickle cell disease is:
n m n pul on ( .e., he Ponse me ho , wh ch we A) Neisseria gonorrhoeae (Gonococcus).
re obl g e o promo e s nce d r. ign c o Ponse e- B) Polymicrobial.
velope he Un vers y o iow . a c u lly, re lly C) Salmonella species.
s n ex remely mpor n v nce n club oo re - D) S. aureus.
men , n s now he s n r o c re use roun he E) Streptococcus species.
worl ).
Answer 12.9.2 The correct answer is “C.” Staphylococcus is
responsible or the majority o bone in ections in the general
Objectives: Did you learn to . . . population. Salmonella species are responsible or up to 85% o
• i en y p en w h me rsus uc us? bone and joint in ections in patients with a history o sickle cell
• d scuss re men ppro ches o me rsus uc us? disease with E. coli another important player
CHAPTER 12 • ORt HOPEd iCS a Nd SPORt S MEd iCiNE 343

C) ESR.
HELPFUL TIP:
D) Arthrocentesis.
Mos c ses o ch l hoo os eomyel s re he resul o
E) MRI.
hem ogenous spre r her h n by rec con m n -
on o he bone.
Answer 12.10.1 The correct answer is “D.” T e most con-
cerning diagnosis with the highest potential morbidity in this
Question 12.9.3 Identi cation o the pathogen in a case o patient is septic arthritis. Her history o rheumatoid arthritis as
osteomyelitis is typically made by: well as long-term steroid use put her at high risk. In a patient in
A) ASO titer. whom you are concerned about septic arthritis, the most impor-
B) Blood culture. tant piece o diagnostic data that you can obtain comes rom
C) Joint aspiration, culture, and Gram stain. synovial uid analysis. While plain radiographs, a WBC count,
D) Pathology report ollowing open biopsy. an ESR, and CRP may be obtained (your riendly neighborhood
E) Witchcra . orthopedic surgeon will surely want to know them), they are
neither sensitive nor speci c enough to rule in or rule out septic
Answer 12.9.3 The correct answer is “B.” A blood culture will arthritis in a high-risk patient.
reveal the o ending organism in 40% to 50% o cases. Joint
aspiration is not typically indicated unless there is strong evi- Question 12.10.2 Which o the ollowing signs or symp
dence o joint involvement. It can take weeks or changes to be toms has su cient sensitivity to rule out septic arthritis i
seen on plain lm radiographs. T ere ore, MRI is the imaging absent?
modality o choice. C or bone scan can also be used i MRI is A) Joint edema or e usion.
not available or contraindicated. I changes are identi ed and a B) Fever.
neoplastic process is ruled out, aspiration at the site o periosteal C) Sweats.
elevation and bony destruction should be considered i a patho- D) Signi cantly restricted ROM.
gen has not yet been identi ed by blood culture. “A,” an ASO E) None o the listed signs or symptoms has suf cient sensitiv-
titer, would not be help ul here since this test or Streptococcus ity to rule out septic arthritis, i absent.
pyogenes in ection. An ASO titer would be help ul i looking or
rheumatic ever. “D,” surgical biopsy, may be required i blood Answer 12.10.2 The correct answer is “E.” Un ortunately,
cultures do not reveal a pathogen and the patient is not respond- there are no clinical signs or symptoms that have suf cient sen-
ing appropriately to empiric antibiotics. sitivity to rule out septic arthritis. Signs and symptoms should
raise clinical suspicion, however the only de nitive diagnostic
HELPFUL TIP: modality is arthrocentesis.
t re men o os eomyel s requ res 4 o 6 weeks o n -
b o cs. Surg c l ebr emen s usu lly ( no lw ys) You e ortlessly obtain 10 mL o cloudy synovial uid that is
requ re s well. sent to the lab or Gram stain, culture, cell count with di -
erential, glucose, and crystal analysis. T e Gram stain is
Objectives: Did you learn to . . . negative with cultures pending. T e synovial WBC count is
• d gnose os eomyel s n ch l ? 51,000/µL with >90% polymorphonuclear cells. T e crystal
• i en y common p hogens nvolve n os eomyel s? analysis shows calcium pyrophosphate crystals. You obtain
a peripheral WBC count that is 11,000/µL and an ESR is
55 mm/hr.
CASE 12.10
A 45-year-old emale with a history o rheumatoid arthritis, Question 12.10.3 What is the most appropriate next course
on chronic low-dose prednisone, presents to your clinic with o action based on these ndings?
2 days o right knee pain. T e patient reports that her knee A) Prescribe high-dose prednisone or a air o her rheumatoid
has been swollen and pain ul to touch, and she now is having arthritis.
dif culty bearing weight due to the pain. She has had previ- B) Start IV antibiotics and obtain emergent orthopedic consul-
ous knee pain, but nothing this severe. She denies any trauma, tation.
evers, chills, knee surgery, illegal drug use, or risky sexual C) reat her or pseudogout.
behavior. On examination, she is well appearing, a ebrile, and D) Recommend rest, ice, compression, and a prescription or
has a moderate right knee e usion with limited ROM. T ere oxycodone.
is no overlying erythema, but the knee eels warm to touch.
Answer 12.10.3 The correct answer is “B.” Various cuto s
Question 12.10.1 Which o the ollowing diagnostics is the or synovial WBC counts have been proposed, ranging rom
most valuable to rule in or rule out the diagnosis with the > 25,000/µL to > 00,000/µL in native joints, with sensitivi-
highest potential morbidity? ties ranging rom 30% to 77% ( able 2-2). Cuto s are much
A) Plain lms o the a ected knee. lower in patients with prosthetic joints. A prosthetic joint with
B) WBC count. WBC count o > ,700/µL may be up to 94% sensitive or septic
344 Fa MiLY MEd iCiNE EXa MiNa t iON & BOa Rd REViEW

TABLE 12-2 SENSITIVITY AND SPECIFICITY OF symptoms to an MRI nding. T is is especially true in the
SYNOVIAL FLUID ANALYSIS IN SEPTIC ARTHRITIS elderly with arthritis where > 90% will have meniscal damage.
As to “D,” it obviously does not relate to the case. We call that
Synovial Fluid
“test taking skill.” I you chose “D,” woe is with you.
WBC Count Sensitivity (%) Speci city (%)
> 100,000/mm 3 30 > 99
QUICK QUIZ: ORt HOPEd iC iNFECt iONS
3
> 50,000/mm 62 92

> 25,000/mm 3 77 73 T e most common organism causing septic arthritis in the teen-
age years is:
A) N. gonorrhoeae (Gonococcus).
B) Polymicrobial.
arthritis. A synovial WBC count o 5 ,000/µL does not rule out C) Salmonella species.
septic arthritis in this patient. When the percentage o polymor- D) S. aureus.
phonuclear cells is > 90%, this signi cantly increases the like- E) Streptococcus species.
lihood o septic arthritis. Don’t let the calcium pyrophosphate
crystals dissuade you rom suspecting septic arthritis as both The correct answer is “A.” Gonococcus is the most common
gout and pseudogout coexist with septic arthritis up to 25% o the organism isolated rom the joints o sexually active, teenage
time (J Rheumatol. 20 2;39: 57). individuals.
Objectives: Did you learn to . . .
• Recogn ze he cl n c l presen on o sep c r hr s? CASE 12.11
• Un ers n he l m ons o us ng s gns n symp oms o
rule ou sep c r hr s? A 28-year-old male presents to your clinic or evaluation o
lower-back pain (LBP). Yesterday morning he rst noticed
• d escr be he ppropr e work-up or p en w h sus-
pec e sep c r hr s? the discom ort, mani esting as sti ness and soreness in the
lower back. T e day be ore had been spent running a oor
polisher. He describes his pain as sharp in nature and 8/10 in
QUICK QUIZ: MY BiG Fat GREEK KNEE Pa iN intensity. He denies radiation o the pain, sensory changes,
and constitutional symptoms. He is concerned this may be an
A 55-year-old obese emale comes to your of ce complaining injury to a disk and that he may be permanently disabled due
o knee pain when she walks. She had an MRI at an urgent care to his extreme pain.
center, which showed some meniscal damage. She is tender
in eriorly and medial to the patella on the proximal tibia (but Question 12.11.1 Which o the ollowing signs or symptoms
not on the joint line). would be “red ags” indicating the need or early imaging
and/or re erral?
Which o the ollowing is true? A) Pain radiating down one or both legs into the posterior
A) Washing out her knee by arthroscopy will help to relieve her thigh.
symptoms. B) Severe pain, prompting the patient to request narcotics.
B) T e nding o a meniscal injury on MRI correlates well with C) Pain greater with active lumbar extension than with orward
symptoms o pain. exion.
C) Based on its location, pes anserinus bursitis is the likely D) New onset erectile dys unction with back pain.
cause o her pain. E) None o the above.
D) Baklava is Dr. Wilbur’s avorite ood.
Answer 12.11.1 The correct answer is “D.” T e onset o erec-
The correct answer is “C.” T e pes anserine (“goose’s oot”) tile dys unction is suggestive o neurologic involvement and
bursa is located on the medial, proximal aspect o the tibia and warrants urther investigation. None o the other options are
is where the tendons o the sartorius, gracilis, and semitendi- suggestive o signi cant disease requiring immediate interven-
nosus attach. It o en becomes in amed causing signi cant and tion (“A” certainly could represent disk disease; however, this
chronic knee pain. Conservative therapy with rest, ice, NSAIDs, does not require immediate intervention). O note, spontane-
and stretching may be tried initially. However, these o en ail; ous erections without sexual stimulation can also be a sign o
most patients experience signi cant improvement with cortico- neurologic involvement.
steroid injections. “A” is incorrect. Several studies have ound
that washing out the knee and trimming the cartilage is o no Question 12.11.2 Early imaging should be obtained in all
bene t. “B” is also incorrect. MRI o the knee is not particu- o the ollowing presentations o LBP EXCEPT:
larly use ul or determining i a meniscal injury is the source o A) Neurologic symptoms such as bowel or bladder dys unction
pain. Similar to herniated disks, many asymptomatic patients and impotence.
have meniscal injuries on MRI limiting our ability to assign B) History o ever, night sweats, and weight loss.
CHAPTER 12 • ORt HOPEd iCS a Nd SPORt S MEd iCiNE 345

C) History o cancer. B) T is is a positive SLR test on the le and is speci c or disk


D) rauma. herniation.
E) Age > 30 years. C) T is is a positive SLR test on the right and is speci c or disk
herniation.
Answer 12.11.2 The correct answer is “E.” Patients over the D) T is is a negative SLR test bilaterally.
age o 50 should have early imaging. See able 2-3 or other
criteria that should prompt early imaging. Answer 12.11.3 The correct answer is “D.” T e SLR test can be
per ormed in several ways, which are listed here.
HELPFUL TIP:
• Seated active: with the patient seated on the examination
Sever l o he n c ors l s e n t ble 12-3 re ssoc -
table, the patient dorsi exes the oot and extends the knee.
e w h c u equ n syn rome. C u equ n syn-
rome s he pro uc o n cu e re uc on n he vol- • Seated passive: with the patient seated on the examination
ume o he sp n l c n l wh ch c n le o compress on table, the examiner passively extends the knee, and radicular
n p r lys s o mul ple nerve roo s n resul n ur - symptoms will be exacerbated with passive ankle dorsi ex-
n ry re en on, s le nes hes , n lower ex rem y ion.
we kness. i s o en c use by cen r l sk hern ons, • Lying passive: with the patient in a supine position, the ex-
ep ur l bscesses n hem om s, r c ures, n o her aminer holds the knee in ull extension and passively exes
r um . Cauda equina syndrome is an orthopedic the hip, and radicular symptoms will be exacerbated with
emergency. An MRI and surgical consultation should passive ankle dorsi exion.
be sought without delay. In all cases, the test is positive when radicular symptoms occur
(e.g., pain and paresthesias down the leg below the level o the
HELPFUL TIP: knee—not back or thigh pain rom muscle stretching) between
P en s who do not h ve lms one n lly c u lly 25 and 75 degrees o hip exion while lying or with knee exten-
h ve be er ou comes h n hose who o. When we or- sion while seated. T e symptoms will be exacerbated with active
er m g ng s u es, we “me c l ze” he llness, c us ng or passive ankle dorsi exion. However, the SLR is neither sensi-
he p en o expec longer recovery n he nee tive nor speci c or disk disease. “Crossover” pain with radicu-
or n erven on n or er o ge be er. d on’ o unless lar symptoms in the leg not li ed is very speci c or disk disease
hey mee he bove cr er . but is not very sensitive.

Even though SLR is negative, you continue your neurologic


Upon physical examination, you note the vital signs are nor- examination. You note symmetric patellar re exes, dimin-
mal. Straight leg raise (SLR) testing on the right leg at 55 ished Achilles re ex on the right, and symmetric strength in
degrees reproduces the patient’s pain in the lower back and the legs except or decreased strength with right oot plan-
a pain ul “tightness” in the posterior thigh. He complains o tar exion. You also note decrease in gross sensation to light
the same discom ort on the le at 30 degrees. touch over the right lateral oot.

Question 12.11.3 Based on these ndings, which o the ol


Question 12.11.4 Which o the ollowing nerve roots is
lowing statements is true?
most likely compromised?
A) T is is a positive SLR test bilaterally and is speci c or disk
A) L3.
herniation.
B) L4.
TABLE 12-3 CRITERIA SUGGESTING THE NEED C) L5.
FOR EARLY IMAGING FOR BACK PAIN D) S .
E) S2–4.
Bowel or bl er ys unc on
New onse o mpo ence
Answer 12.11.4 The correct answer is “D.” A summary o
Fevers or n gh swe s nerve root innervation is given in able 2-4.
Unpl nne we gh loss
N gh p n Question 12.11.5 Appropriate initial treatment or this
Person l h s ory o c ncer patient’s acute back pain should include which o the
S le nes hes ollowing?
H s ory o recen r um (e.g., ll or rec blow, NOt w s ng or l ng) A) Strict bed rest.
a ge >50 or <18 ye rs B) Pain control.
P en w h curren or recen use o s ero s C) Corset or lumbar belt.
a ny susp c on o n n ec ous or neopl s c c use or low-b ck p n D) Re erral or epidural steroid injection or endoscopic disk
P n or >6 weeks resection.
E) A and B.
346 Fa MiLY MEd iCiNE EXa MiNa t iON & BOa Rd REViEW

TABLE 12-4 EXAMINATION FINDINGS OF LUMBAR AND SACRAL SPINAL NERVE ROOTS
Nerve Root Re ex Motor Sensory Test
L2–3 None Qu r ceps a n er or h gh Knee ex ens on

L4 P ell t b l s n er or ( oo ors ex on, Me l lower leg n oo W lk on heels


nvers on)

L5 Me l h ms r ng Ex ensor h lluc s longus ( ors ex on o d ors l oo Hol up gre oe


( f cul o ssess) b g oe)

S1 a ch lles Peroneus longus n brev s ( nkle L er l oo W lk on oes


evers on) n pl n r ex on o oo

S2–4 a n l w nk in r ns c oo muscles, n l sph nc er one Per n l

Answer 12.11.5 The correct answer is “B.” In acute mechani- have equivocal data with most studies being negative. “Back
cal back pain (no longer than 6 weeks), regardless o the School” also does not seem to help.
method o treatment, 40% are better within week, 60% to 85%
in 3 weeks, and 90% in 2 months. Negative prognostic actors
HELPFUL TIP:
include more than three episodes o back pain, gradual onset
Phys c l her py mo l es such s ppl c on o he ,
o symptoms, and prolonged absence rom work. Bed rest
col , ul r soun , n muscle s mul on m y h ve
does not contribute to a return o unction and may worsen
shor - erm bene . Reh b l on exerc ses ocus ng
outcomes. Early mobilization o the patient is best or allow-
on runk ex ensors, b om n l muscles, n erob c
ing him to continue activities as tolerated. Acetaminophen is
con on ng promo e e rly mob l z on, wh ch s cr -
a great drug or pain control and has ewer side e ects than
c l n re ng cu e b ck p n. The specific exercise
do the NSAIDs. Chiropractic care may be use ul. Data on acu-
does not matter as much as the mobilization. Sh m
puncture are mixed.
her py works s well s spec c exerc ses s long s he
p en s mob le.
HELPFUL TIP:
For LBP, e rly mob l z on n w lk ng s mpor n .
Objectives: Did you learn to . . .
Ep ur l s ero njec ons, wh le n u vely ppe l ng
• Gener e eren l gnos s n un ers n he e ol-
or sk se se, h ve been shown o be o no long- ogy o lumb r sp ne p n?
erm ( n l le shor erm) bene (J Bone Joint Surg.
• Ev lu e n re cu e LBP?
2012;94:1353–1358; BMJ. 2015;350:h1748).
• Recogn ze w rn ng s gns o LBP?
For spinal stenosis, surgery h s no been shown o be
p r cul rly be er h n conserv ve, nonsurg c l, re -
men (Ann Intern Med. 2015;162:465). S ero s h ve lso QUICK QUIZ: Ba CK Pa iN
no been shown o be o ny bene n sp n l s enos s.
Spondylolysis commonly occurs in which part o the spine?
A) Cervical spine lateral processes.
You prescribe your pain medication o choice and recom- B) T oracic spine pars interarticularis.
mend rehabilitation exercises. C) T oracic spine lateral processes.
D) Lumbar spine pars interarticularis.
Question 12.11.6 Which o the ollowing has been shown
to be ef ective at reducing the recurrence o back injury in The correct answer is “D.” Spondylolysis is characterized by
the workplace? pars interarticularis stress ractures and most commonly occurs
A) Back support belts. in the lumbar region.
B) “Back School” that teaches proper li ing techniques, stretches,
etc.
C) Increasing physical tness and muscle tone. QUICK QUIZ: MORE Ba CK Pa iN
D) A and C.
E) B and C. Anterior slippage o one vertebra on another is called:
A) Spondylolysis.
Answer 12.11.6 The correct answer is “C.” T e only thing that B) Spondylolisthesis.
has been unequivocally shown to reduce urther back injuries is C) Spondylitis.
improving the overall tness o the patient and his muscle tone. D) Spondyloarthropathy.
O special note, back support belts, long worn in industry, E) Scheuermann disease.
CHAPTER 12 • ORt HOPEd iCS a Nd SPORt S MEd iCiNE 347

The correct answer is “B.” Slippage o one vertebra on another usually ound in children age in ancy to 3 years but may occur
is called spondylolisthesis. “A,” spondylolysis, is discussed at any age. T e etiology is usually Staphylococcus (low-grade
above. Spondylolysis can lead to spondylolisthesis. Spondyloar- in ection) but there may be sterile in ammation. Fever is
thropathy is a nonspeci c term re erring to in ammation o the usually absent in discitis (seen in only 25%), and blood cul-
spine and encompasses such diseases as ankylosing spondylitis, tures are sterile. T e white count is usually normal, although
Reiter disease, enteropathic arthritis, etc. Spondylitis is a more ESR is elevated in 90%. reatment is not standardized, but
speci c term or the same thing (e.g., ankylosing spondylitis). most experts would include anti-staphylococcal antibiotics.
“E,” Scheuermann disease, is a process causing kyphosis by “B,” occult racture, is unlikely in a child this age. “C,” grow-
compression o the vertebrae (at least 5 degrees o wedging in ing pains, does not present with back pain (see the discussion
three consecutive vertebrae). T e cause is unknown but it tends o growing pains earlier in this chapter). Juvenile rheumatoid
to present in adolescence. arthritis presents with small joint involvement as opposed to
back pain.
Spondylolisthesis is graded based on the degree o slipping
o one vertebra on the other. Which o the ollowing patients CASE 12.12
would need surgical consultation?
A 24-year-old male presents to the clinic 2 days a er a col-
A) Grade one (< 25% slip) with no symptoms in an early teen
lision during a so ball game in which he ell on his out-
( 2– 4).
stretched right hand (“But I made the play!” he exclaims). He
B) Grade 2 (25–50% slip) with no symptoms in an early teen
reports he could not continue playing and that his pain has
( 2– 4).
not improved. He has some general edema around the right
C) Grade 3 (50– 00% slip) with mild pain in a 20 year-old col-
wrist, poor grip strength secondary to pain, point tenderness
lege gymnast.
over the radial aspect o the wrist (“snu box tenderness”),
D) When slippage is 0% to 5% with no symptoms in a patient
and decreased ROM. T ere is no obvious de ormity, and he is
> 60 years old.
neurovascularly intact.
E) When slippage is 25% with no symptoms in a patient
> 60 years old.
Question 12.12.1 O the ollowing, what would be the most
likely diagnosis or this patient?
The correct answer is “C.” Spondylolisthesis is generally a
A) Colles racture.
problem in the late teens and 20s. Patients o en become symp-
B) Scaphoid racture.
tomatic when there is 25% slippage or greater. Mild degrees o
C) Smith racture.
slip can be treated conservatively, although surgical treatment
D) Extensor carpi radialis strain.
threshold varies considerably among orthopedic surgeons.
E) Scapholunate sprain.
Predisposing actors include recurrent lumbar hyperextension
(gymnasts, ootball players, etc.), although many patients do not
Answer 12.12.1 The correct answer is “B.” Although all o
have an identi able cause. Any adolescent athlete who presents
these could be in the di erential diagnosis, “B” is the most likely
with back pain that is made worse by hyperextension should
based on mechanism o injury and clinical ndings. T e scaph-
raise your suspicion. Older patients who are asymptomatic do
oid spans both the proximal and distal carpal row. In this posi-
not need to be considered or surgical treatment unless the slip
tion, it is quite vulnerable to high-impact injuries, such as a all
is grade 4 ( 00% or greater).
on an out-stretched hand, and is the most commonly ractured
carpal bone. T e absence o de ormity makes a Colles or Smith
QUICK QUIZ: Ba CK Pa iN iN CHiLd REN racture less likely (options “A” and “C”). Further, a sprain or a
strain (options “D” and “E”) are less likely given the bony point
A 4-year-old boy presents to your of ce accompanied by his tenderness, decreased range o motion, and overall nonim-
mother who says he has a limp and LBP. T is has been getting provement with conservative measures.
progressively worse over the past 2 weeks. When the patient sits,
he sits in a “tripod” position supporting his weight on his hands.
HELPFUL TIP:
Vitals are normal without a ever and a CBC is normal.
P lp ng or snu box en erness w h he wr s n
sl gh uln r ev on ncre ses he sens v y o he
T is history is most consistent with:
phys c l ex m n on. a n on l phys c l ex m n -
A) Discitis.
on n ng cons s en w h sc pho njury s p n
B) Occult racture.
he sc pho w h x l lo ng o he humb.
C) Growing pains.
D) Juvenile rheumatoid arthritis.
E) Mechanical low back strain.
Plain lm radiographs, including AP and lateral o the
The correct answer is “A.” T is history is most consistent hand and wrist as well as scaphoid views, are negative or
with discitis. Discitis is an in ammatory process o the disk racture.
348 Fa MiLY MEd iCiNE EXa MiNa t iON & BOa Rd REViEW

Question 12.12.2 What is the most appropriate next step


HELPFUL TIP:
or this patient?
He l ng me or s l pole sc pho r c ure s 6
A) Short-arm thumb spica cast with ollow-up in 0 to 4 days.
o 8 weeks, m le h r or w s r c ures re 8 o
B) NSAIDs, ice, compression, and elevation ollowed by physi-
12 weeks, n prox m l pole r c ures c n ke 12 o
cal therapy.
24 weeks.
C) MRI or C to rule out an occult racture.
D) Orthopedic re erral.
E) Return to play within the week. Objectives: Did you learn to . . .
• Recogn ze p en r sk or sc pho r c ure?
Answer 12.12.2 The correct answer is “A.” Scaphoid ractures • M n ge p en w h sc pho r c ure?
are o en occult acutely and usually will be evident on plain
lms a er 0 to 4 days due to bony resorption along the rac-
ture line. I repeat lms are negative but suspicion remains high, QUICK QUIZ: Ha Nd iNJURiES
an MRI or C should be considered.
A patient presents a er “jamming” his index nger while play-
ing basketball. He has mild swelling at the DIP joint. At rest, his
HELPFUL TIP: DIP is exed. He has ull ROM o all joints except he cannot
a l hough no he s n r o c re, e rly MRi or Ct extend at the DIP.
(ne her s e n vely super or o he o her) o ev lu-
e or occul sc pho r c ures m y llow p en s o What is the appropriate treatment and ollow-up?
re urn o ull c v y sooner h n woul be poss ble A) RICE therapy, PRN ollow-up.
hey were spl n e or 10 o 14 ys. t h s m y be cos - B) A ull extension splint o the DIP joint worn at all times with
e ec ve or cer n p en s ( .e., concer v ol n s s or orthopedic ollow-up in to 2 weeks.
he iow H wkeyes’ s r ng qu r erb ck). Bo h MRi C) A removable aluminum splint to be worn or com ort, ollow-
n Ct re be er exclu ng r c ures h n hey re up in clinic in month.
con rm ng hem ( hey c n be rule ou , bu no neces- D) Ibupro en and return to ull activities, PRN ollow-up.
s r ly rule n).
The correct answer is “B.” T e patient has su ered an injury
to his extensor tendon mechanism, known as a “mallet nger.”
X-rays are indicated to evaluate or a bony racture/avulsion.
Repeat wrist radiographs including scaphoid views 2 weeks T e initial treatment is an extension splint at the DIP joint, and
post injury indicate a nondisplaced racture o the proximal ollow-up with an orthopedic surgeon as surgical correction is
pole o the scaphoid. sometimes required. T e splint must be worn at all times. See
Figure 2-3 or the anatomy o a mallet nger.
Question 12.12.3 You recommend which o the ollowing
treatment plans?
A) Wrist and thumb spica splint and physical therapy because QUICK QUIZ: FRa Ct URES
good blood supply at the proximal pole allows ast heal-
ing. A Colles racture consists o :
B) T umb spica cast or 6 weeks then repeat x-rays. A) Fractures o the midsha o the radius and ulna.
C) Short-arm cast excluding the thumb or 4 to 6 weeks. B) Fractures o the head o radius and ulna that is displaced
D) Orthopedics re erral or open reduction/internal xation. dorsally and is angulated.
E) B or D.
Rupture d te ndon
Answer 12.12.3 The correct answer is “E.” It is clear that a spica
cast with the thumb included is important; whether a short- or
long-arm cast is optimal is still a matter o debate. Open xa-
tion is another option. Generally, an orthopedic surgeon should Dis ta l
pha laynx
oversee treatment o scaphoid ractures since the complication
rate is high. A proximal pole racture has high risk or nonunion
and avascular necrosis (90%). T e blood supply to the scaph- Fra cture
oid is through the distal pole, putting the proximal pole at
high risk or complications. Evidence o healing may not be
well visualized on plain lms, and a C or MRI may be needed
to con rm the degree o healing. T e closer the racture line
MALLET FINGER
is to the proximal pole, the lower the threshold or orthopedic
re erral. FIGURE 12-3.
CHAPTER 12 • ORt HOPEd iCS a Nd SPORt S MEd iCiNE 349

C) Fractures o the head o the radius and ulna that are dis- What is the maximal acceptable angulation and rotation or a
placed ventrally and is angulated. boxer racture, ourth or h metacarpal, to maintain ull hand
D) None o the above. unction?
A) 0 degrees o dorsal angulation and 0 degrees o rotation.
The correct answer is “D,” none o the above. OK, some o B) 30 degrees o dorsal angulation and 5 degrees o rotation.
you may have chosen “B.” However, the head o the radius is C) 40 degrees o dorsal angulation and 0 degrees o rotation
at the elbow and not at the wrist. T us, none o the answers is D) 90 degrees o dorsal angulation and 0 degrees o rotation.
correct. A Colles racture is a racture o the distal radius at the
metaphysis, which is displaced dorsally and o en angulated. It The correct answer is “C.” Any degree o rotation, or > 40
is the most common wrist racture in adults. T e ulnar styloid degrees o dorsal angulation, may result in signi cant unc-
is o en involved, and there may be intra-articular involvement tional de cits. Reduction should be attempted i angulation is
as well. > 0 degrees. Patients should be advised that with angulations
> 0 to 5 degrees, there will likely be a loss o metacarpopha-
langeal (MCP) prominence, although there should be no loss
QUICK QUIZ: d E QUERVa iN t ENOSYNOVit iS o unction. I this is unacceptable to the patient, re erral is
recommended.
Which o the ollowing physical examination ndings would be
associated with the diagnosis o De Quervain tenosynovitis? HELPFUL TIP:
A) Positive Finkelstein test. a ny “sc ssor ng” or cross ng o he p en ’s ngers
B) Positive Phalen test. wh le hey clench he r s shoul r se your susp c on
C) Positive inel sign. or ro on l e orm y.
D) Sensory loss over the C7 dermatome.
E) Weakness o the intrinsic muscles o the hand.
HELPFUL TIP:
The correct answer is “A.” De Quervain tenosynovitis is a ten- i you c n’ be your compu er chess, on’ ge rus-
donitis o the abductor pollicis longus, the extensor pollicis bre- r e ; ry ch ng ng spor s. We re sure you c n be
vis, and occasionally the extensor pollicis longus. Full exion o your compu er m xe m r l r s.
the thumb into the palm and ulnar deviation o the wrist pro-
duces exquisite tenderness over the radial aspect o the wrist
(positive Finklestein test). Other ndings may include so -
tissue swelling and pain over the abductor pollicis longus and
CASE 12.13
extensor pollicis brevis tendons near the radial styloid. Crepitus T e next patient in the ED is an 18-year-old high school oot-
may also be palpable or audible with ROM. ball player. He was playing in a ootball game this evening
when he was tackled rom the side and landed directly on
the lateral aspect o his right shoulder. He states that he can
QUICK QUIZ: d E QUERVa iN t ENOSYNOVit iS actively move his arm, but is limited by pain on the top o
his shoulder. He has also noticed a small pain ul bump on
T e appropriate treatment o De Quervain tendonitis includes top o the right shoulder and is concerned that he “broke his
which o the ollowing? collarbone.”
A) T umb spica splint.
B) NSAIDs. Question 12.13.1 Based on the mechanism o injury and
C) Steroid injection. patient history, the most likely injury would be?
D) Surgical decompression o the tendon. A) Acromioclavicular (AC) sprain.
E) All o the above. B) Biceps tendon rupture.
C) Glenohumeral dislocation.
The correct answer is “E.” All o the above treatments have been D) Rotator cu tear.
used success ully or the treatment o De Quervain tendonitis. E) Scapula racture.
It is o note that some pre er steroid injection as the rst-line
therapy rather than NSAIDs. Surgery is obviously a last choice. Answer 12.13.1 The correct answer is “A.” Although any o
these injuries may be present, an AC sprain is the most likely
based on the history and the way the patient ell. A thorough
QUICK QUIZ: a NGER Ma Na GEMENt iSSUES examination should be able to urther distinguish between
these injuries. “B” is not likely, given the mechanism o injury.
A 9-year-old male was mad at his computer and decided to T e de ormity associated with biceps tendon rupture (a de ect
punch the wall. “He ought the wall and the wall won.” Radio- in tendon with pain and de ormity in the muscle belly repre-
graphs demonstrate a h metacarpal racture with some angu- senting the contracted, detached muscle) would be on the upper
lation. arm or at the elbow, not on the “top” o the shoulder. “C” is
350 Fa MiLY MEd iCiNE EXa MiNa t iON & BOa Rd REViEW

incorrect. T e de ormity and loss o ROM o a glenohumeral


dislocation (shoulder dislocation) is usually obvious. T e mech- You send the patient or x-rays. AP radiographs show slight
anism o injury is typically a orced abduction and external rota- widening o the AC joint on the injured side. T e examina-
tion. “D” is less likely. A rotator cu tear will present with pain tion and radiograph con rm your suspicion o an AC injury.
more laterally over the subacromial space and should not have
an associated de ormity. T e ROM is generally markedly lim- Question 12.13.4 For this patient with an AC sprain, you
ited by pain. “E” is unlikely. Scapula ractures are uncommon of er:
and are usually the result o high velocity blunt trauma such as A) Sling or com ort, ice, and NSAIDs or analgesics or pain
a blow rom a baseball bat or motorcycle accident. Plain lm control.
radiographs should be obtained to rule out a clavicle racture, B) Re erral or open xation.
especially when any de ormity is present. C) Figure-o -eight strap or 4 to 6 weeks.
D) Corticosteroid injection ollowed by physical therapy.
Question 12.13.2 Your patient is worried that he broke E) Manual reduction, then sling-and-swath immobilization or
his “collarbone.” I he did sustain a nondisplaced clavicle 6 to 8 weeks.
racture, which o the ollowing would be appropriate
treatment? Answer 12.13.4 The correct answer is “A.” Conservative man-
A) Rest, ice, and NSAIDs or pain control. agement is appropriate or low-grade AC joint sprains. AC
B) A sling or com ort only. sprains are graded I to VI. Grades I to III are generally treated
C) A gure-o -eight splint. conservatively, while grades IV–VI may need to be treated surgi-
D) A and B cally. Signs o high-grade (IV–VI) AC sprains include posterior
E) External xation with molded plaster to maintain align- or in erior displacement o the clavicle and widening o 00% or
ment. more o the coracoclavicular space. Just like with clavicle rac-
tures, gure-o -eight straps have largely allen out o avor or
Answer 12.13.2 The correct answer is “D.” While the tradi- treatment AC joint injuries. Corticosteroid injections are not the
tional teaching has been that a gure-o -eight splint is required, treatment o choice or acute AC joint injuries. An injection may
it adds nothing to the treatment o a clavicle racture. Pain con- be considered as an adjunct treatment or degenerative arthritis
trol and using a sling works just as well. Additionally, gure- o the AC joint at some distant point in the uture. Sling-and-
o -eight splints o en increase a patient’s pain and can cause swath immobilization is not indicated.
a brachial plexus injury. T us, a sling is pre erable. Complete Objectives: Did you learn to . . .
immobilization is not necessary or nondisplaced clavicle rac- • d eren e be ween c uses o shoul er njur es?
tures, thus making “E” incorrect. • t re cl v cul r r c ure?
• M n ge a C njur es?
Question 12.13.3 The proper treatment or a clavicle rac
ture that is minimally displaced is:
A) Closed reduction and then a sling.
CASE 12.14
B) Open reduction and immobilization. A 65-year-old male presents with le shoulder pain and
C) Open reduction and early mobilization. weakness, which started 2 weeks ago a er he put a new roo
D) A sling. on his house. He does not recall a speci c injury. T e pain
E) None o the above. is worse with reaching or and li ing objects as well as with
overhead activities. Nighttime pain is present. He describes
Answer 12.13.3 The correct answer is “D.” T e ends o a dis- himsel active and healthy, and he only takes acetaminophen
placed clavicle racture need not be 00% approximated or when needed or shoulder pain. You suspect that he may have
healing to occur and or unction to return. T us, reduction is rotator cu tendinopathy.
generally not necessary.
Question 12.14.1 I this is the case, what do you expect to
HELPFUL TIP: nd on examination?
t he p r gm o he re men o cl v cle r c ures s A) enderness to palpation o the greater tuberosity o the
sh ng. More p en s re ge ng surg c l n erven on humerus.
( n n ern l x on), espec lly hle es when qu ck B) Limited active ROM.
re urn o spor s es re . t h s ec s on epen s on C) Normal passive ROM.
he egree o spl cemen n he occup on o he D) Shoulder shrug with attempted abduction (such as with a
p en (Surgeons? Yes. C r S lesmen? No so much). rozen shoulder).
Gre er egrees o r c ure spl cemen n b yone - E) Any o the above.
ng h ve h gher r es o r c ure nonun on. t hus, s-
cuss on w h n or hope s s ppropr e or p en s Answer 12.14.1 The correct answer is “E.” Ok, so this might t
w h spl ce cl v cle r c ures. under the category o “trick question,” but the shoulder exam-
ination can be normal in a patient with a rotator cu tear or
CHAPTER 12 • ORt HOPEd iCS a Nd SPORt S MEd iCiNE 351

tendinopathy, or it can include any o the elements listed in “A” Patients under the age o 60 with acute traumatic tears should
through “D.” Notably, normal PASSIVE ROM (“C”) does not be considered or surgery, with best results within 6 weeks
rule out rotator cu pathology. o injury.

Question 12.14.2 Which o the ollowing muscles is NOT a


part o the rotator cuf ? HELPFUL TIP:
A) Supraspinatus. t he ol ge bou cor cos ero s c us ng we ken ng
B) In raspinatus. o he en on h s recen ly been ques one . i s now
C) Subscapularis. hough h he s ero njec on prov es enough
D) eres major. rel e o he p n h he p en w ll s r us ng he
E) eres minor. ex rem y n w ys he or she h no one be ore. t h s
le s o en on rup ure rom he on l lo . How-
Answer 12.14.2 The correct answer is “D.” T e rotator cu ever, he po n rem ns h s ero njec ons m y be
consists o the other our muscles listed and unctions to rotate ssoc e w h, bu no c us ve o , en on rup ure.
the arm and stabilize the humeral head.

Question 12.14.3 Which o the ollowing muscles is the Your patient is success ul in rehabilitating his le shoulder,
most commonly torn in the rotator cuf ? but then he returns 2 years later with right shoulder prob-
A) Supraspinatus. lems. T e right shoulder has become progressively sti and
B) In raspinatus. pain ul, and his ROM is now signi cantly limited in all direc-
C) Subscapularis. tions. Your examination is consistent with “ rozen shoulder”
D) eres minor. or adhesive capsulitis.

Answer 12.14.3 The correct answer is “A.” T e supraspinatus Question 12.14.5 Adhesive capsulitis is most commonly
is generally the point o origin or most tears. associated with which o the ollowing?
A) Diabetes.
HELPFUL TIP: B) Hyperthyroidism.
Full h ckness e rs o he ro or cu re uncommon n C) Spondyloarthritis.
n v u ls below he ge o 40, unless ssoc e w h D) Nondominant arm.
r um . a symp om c ro or cu e rs re very com- E) Male gender.
mon n hose over he ge o 60 n re no lw ys he
p n gener or— n o her wor s—or er MRis ju c ous- Answer 12.14.5 The correct answer is “A.” Adhesive capsulitis
ly n h s popul on! has no clear predilection as to gender, race, arm dominance, or
occupation. It is characterized by loss o ROM o the shoulder
in all directions, with loss o both passive and active motion. It
Based on your history and physical examination, you diag- has a high incidence in patients with diabetes and tends to be
nose a rotator cu tendinosis. more recalcitrant in those patients, o whom up to 50% will have
bilateral involvement—although not necessarily concomitantly.
Question 12.14.4 Appropriate initial management o this Adhesive capsulitis is not typically related to trauma, but it can
65 year old male should be: be associated with disuse due to pain, osteoarthritis, sling use,
A) Acetaminophen and physical therapy. etc. Other conditions that are associated with adhesive capsuli-
B) Oral corticosteroids and physical therapy. tis include hypothyroidism and Parkinson disease.
C) Subacromial injection with corticosteroid and physical ther-
apy.
D) Surgical repair and physical therapy. HELPFUL TIP:
E) None o the above. a hes ve c psul s s cons ere o be op h c n
sep r e rom pos r um c or pos oper ve jo n s -
Answer 12.14.4 The correct answer is “A.” For initial manage- ness or hes ons.
ment in an individual > 60 years o age, acetaminophen and
physical therapy or 6 weeks is the best answer. I the patient
has no improvement or inadequate response, a corticosteroid Question 12.14.6 What initial treatment do you recom
injection may be used judiciously. Injection likely will result in mend or this patient with adhesive capsulitis?
at least short-term pain relie , but there is no good evidence that A) Arthroscopic debridement.
it helps long term. In addition, it is thought to weaken the ten- B) Oral corticosteroids.
don and may accelerate extension o a tear, i present. Patients C) NSAIDs and a sling or com ort.
with signi cant symptoms or ailed therapy should be consid- D) Extended progressive physical therapy.
ered or MRI, orthopedic re erral, and surgical management. E) Mobilization under anesthesia.
352 Fa MiLY MEd iCiNE EXa MiNa t iON & BOa Rd REViEW

Answer 12.14.6 The correct answer is “D.” A progressive Question 12.15.2 Which portion o the biceps is most com
stretching program with heat and NSAIDs or acetaminophen monly involved in ruptures?
to improve com ort is the most appropriate early treatment. A A) Distal tendon.
corticosteroid injection (under ultrasound guidance or uo- B) Proximal short head tendon.
roscopy) may be bene cial, but should be used cautiously in C) Proximal long head tendon.
diabetic patients. Oral steroids have no greater bene t than D) Mid-muscle belly.
NSAIDs. “C” is incorrect because a sling will contribute to ur- E) Proximal short head belly.
ther immobilization and worsening o the problem. Mobiliza-
tion or capsular release under anesthesia may be a last resort in Answer 12.15.2 The correct answer is “C.” T e long head is
adhesive capsulitis. most commonly a ected due to its position and risk or weaken-
ing secondary to rotator cu tendinosis and shoulder impinge-
HELPFUL TIP: ment.
t he yp c l cl n c l course or hes ve c psul s evolves
over 1 o 2 ye rs w h n n l “ reez ng” ph se ch r c- You decide this patient has a rupture o the long head o the
er ze by progress ng p n n s ness ollowe by biceps tendon.
slow “ h w ng” ph se w h ecre s ng p n n ncre s-
ng ROM. Question 12.15.3 How is this injury treated initially?
A) Immediate surgical repair.
B) Delayed surgical repair.
Objectives: Did you learn to . . .
C) Immobilization or 4 to 6 weeks with sling.
• d e ne he muscles o he ro or cu ?
D) NSAIDs and physical therapy.
• i en y, ev lu e, n re ro or cu njury?
E) Biceps muscle transplant, pre erably rom a retired NFL
• Recogn ze he presen on, ssoc ons, n re men o player.
hes ve c psul s?
Answer 12.15.3 The correct answer is “D.” For most isolated
CASE 12.15 proximal long or short head tears (with the exception o some
A 58-year-old male presents a er sudden onset o right upper young athletes and heavy laborers who would not tolerate the
arm pain. He was working in the yard, cutting and pulling slight decrease in strength), treatment is conservative. Analge-
out some bushes, when he heard a “snap” and elt the pain. sics and physical therapy typically suf ce. Surgical repair may
He has a history o rotator cu tendinosis and osteoarthritis. be indicated i conservative therapy ails. O note, you should
discuss with patients the cosmetic de ormity that will be perma-
Question 12.15.1 You should look or all o the ollowing on nent when these injuries are unrepaired versus scarring associ-
physical examination EXCEPT: ated with surgery. Generally, there is approximately a 0% loss
A) A positive elevated arm stress test (“Roos” test). o elbow exion and supination strength with an isolated proxi-
B) A palpable biceps muscle de ect. mal tear.
C) Normal grip strength.
D) An asymmetric bulge in the a ected arm.
HELPFUL TIP:
Answer 12.15.1 The correct answer is “A.” T e elevated arm a l hough prox m l b ceps en on rup ures re re e
stress test is used to evaluate a patient or thoracic outlet syn- conserv vely, s l rup ures shoul be re erre or
drome. Have the patient li both arms with the elbow at shoul- e rly surg c l rep r, s he con nu y o he en re mus-
der height while pushing the shoulders backward and repeatedly cle s los n unc on he elbow jo n s s gn c n ly
gripping and relaxing the hands. T e test is positive i neuro- mp re w h 30% o 40% loss o s reng h cross he
logical or vascular symptoms are reproduced when the arm elbow jo n .
is elevated or a prolonged period. T is patient’s presentation
is not consistent with thoracic outlet syndrome. However, the
history is consistent with biceps tendon rupture. “B” through Objectives: Did you learn to . . .
“D” would be expected in a patient with biceps tendon rup- • i en y he cl n c l presen on o b ceps en on rup ure?
ture. Yes, there is usually normal grip strength. T e injury is, o • M n ge p en w h b ceps en on rup ure?
course, in the upper arm.
CASE 12.16
HELPFUL TIP:
Provoc ve m neuvers or hor c c ou le syn rome A 25-year-old male presents to you with a history o a soccer
(Roos es , e c.) h ve poor sens v y n spec c y injury. Someone evidently ell on his right oot while trying
(<60%). t hey re mos help ul pos ve n he p - to steal the ball. T e patient rapidly and orce ully twisted
en ’s symp oms re repro uce . around the xed oot. Since then he has had signi cant pain
and swelling o the oot. His x-ray is shown in Figure 12-4.
CHAPTER 12 • ORt HOPEd iCS a Nd SPORt S MEd iCiNE 353

FIGURE 12-4.

Question 12.16.1 Which o the ollowing is true regarding Answer 12.16.2 The correct answer is “C.” T is injury/dislo-
this radiograph? cation will lead to signi cant long-term pain and mid oot insta-
A) It shows a step-o between the base o the second metatarsal bility i not recognized and treated appropriately. Any signi cant
and the middle cunei orm. displacement (> 2 mm) should be re erred or surgical consid-
B) It shows mid oot boney instability. eration. T ese are generally complex injuries prone to poor out-
C) It may require weight-bearing lms to identi y this injury. comes and should be managed by an orthopedic consultant.
D) T e abnormality is named a er a French gynecologist.
Objectives: Did you learn to . . .
E) All o the above are true.
• i en y L s r nc njury?
• M n ge p en w h L s r nc njury?
Answer 12.16.1 The correct answer is “E,” all o the above.
T is is a Lis ranc racture, which occurs between the metatar-
sals and the tarsal bones (the space between the metatarsals CASE 12.17
and the tarsal bones is known as the Lis ranc joint). Look or a An 18-year-old emale gymnast lands her dismount rom
widened space between the rst and second and/or second and the balance beam awkwardly. She reports the knee buckling,
third metatarsals. T ere may also be a stepo between the sec- hearing a pop, and experiencing immediate right knee pain.
ond metatarsal and middle cunei orm. T ese ractures may be She presents to your of ce 45 minutes a er the injury. She is
dif cult to identi y, but obtaining weight-bearing lms or stress able to bear some weight on the leg but reports it is already
views may help. Signi cant oot pain should be a tip o . See swollen and eels “loose.” On examination, there is a knee
Figure 2-5 or urther explanation. And, yes indeed, Dr. Lis- e usion present.
ranc was a surgeon and gynecologist in Napoleon’s army! Why
did Napoleon need a gynecologist on campaign with him? T at Question 12.17.1 Based on the in ormation above, the
is a matter o debate. most likely isolated injury experienced by this athlete is:
A) Medial meniscus tear.
Question 12.16.2 Appropriate treatment or this type o B) MCL sprain.
injury should include: C) Distal quadriceps/patellar tendon rupture.
A) Weight bearing as tolerated in a post-op or hard-soled shoe. D) Anterior cruciate ligament (ACL) rupture.
B) Rest, ice, compression, elevation, NSAIDs, and activity as E) Distal emur racture.
tolerated. Will heal well and can be treated like a mid- oot
sprain. Answer 12.17.1 The correct answer is “D.” Did the patient
C) Orthopedic re erral or open reduction/internal xation or someone else hear a pop? I yes, suspect ACL tear (80%),
(ORIF). meniscal injury ( 5%), and rarely a racture. When did you
D) Walking boot that can be removed or several weight-bearing notice swelling? I 0 to 2 hours a er the injury, suspect ACL
hours a day. tear, racture, or patellar dislocation/subluxation; i 2 to
E) Early 9th century French legionnaire’s boot. 24 hours, suspect meniscal injury. I there is hemarthrosis on
354 Fa MiLY MEd iCiNE EXa MiNa t iON & BOa Rd REViEW

S te p off s e cond me ta ta rs a l
a nd middle cune iform
We dge s ha pe fra cture
ofte n s e e n with Lis fra nc

Wide s pa ce be twe e n
1s t a nd 2nd me ta ta rs a ls

FIGURE 12-5.

aspiration, suspect ACL injury (> 75%), patellar subluxation, or consult is generally indicated i ACL injury is suspected, and
intra-articular racture. A history o , “My knee gives way; buck- obtaining one is less expensive than MRI.
les; eels loose; or comes apart,” may be secondary to patellar
subluxation/dislocation, ACL de ciency, or arthritis. Medial HELPFUL TIP:
and lateral collateral ligament injuries, do not typically present t he L chm n es s per orme w h he knee lexe
with signi cant e usion and typically eel stable with orward 20 o 30 egrees w h he p en n sup ne pos on.
ambulation but are pain ul with side-to-side movements. Mus- t he ex m ner hen emp s o n er orly spl ce he
cle or tendon rupture may cause buckling, but will not typically b on he emur wh le s b l z ng he emur. a lw ys
cause e usion and will generally have an obvious de ormity and remember o check he con r l er l s e. Some p -
inability to bear weight. en s h ve n ur lly l x l g men s.

Question 12.17.2 The best test to con rm the diagnosis o


the above injury would be? You eel that this patient is very appropriate or radiographic
A) Plain lm radiographs. evaluation, and you obtain x-rays o the knee.
B) McMurray test.
C) Lachman test. Question 12.17.3 Which o the ollowing is NOT one o the
D) Anterior drawer test. criteria o the Ottawa knee rules predicting the need or
knee radiographs?
Answer 12.17.2 The correct answer is “C.” In the hands o an A) Age < 8.
experienced clinician, the Lachman test is the most sensitive B) Pain isolated to the patella.
test or ACL insuf ciency (80–95%). T e anterior drawer sign C) enderness at the head o the bula.
is negative in about 50% o acute ACL tears, and o en is nega- D) Inability to ex the knee 90 degrees.
tive subacutely. McMurray test is used to evaluate or a meniscal E) Inability to bear weight or our steps.
tear. Plain lms should be obtained or all patients with acute
knee injury with e usion or suspected ACL tear. A Segond Answer 12.17.3 The correct answer is “A.” T e age criterion
racture (small avulsion o the lateral tibial plateau) is an x-ray or the Ottawa knee rules is age > 55 years. All o the other
nding associated with ACL tears. Although an MRI may be options are correct. I any o these criteria are present (including
considered a gold standard test, its sensitivity has been reported a patient > 55 years o age), a radiograph should be obtained.
as 97% when compared against arthroscopy ndings, and is pos- T ese rules have been validated and are 97% sensitive or rac-
itive in only 82% in cases o complete rupture. An orthopedic ture. T e Pittsburgh rules, which are reportedly 99% sensitive,
CHAPTER 12 • ORt HOPEd iCS a Nd SPORt S MEd iCiNE 355

have only two criteria: ( ) age < 2 years or age > 50 and (2) knee. T e test is positive when the examiner eels a pop during
inability to bear weight in the clinic or ED. O the two, the the maneuver or when there is signi cant pain during internal
Ottawa rules are the more commonly accepted. or external rotation. T e Apley test is done with the patient in
a prone position. Move the knee to 90 degrees o exion. Put
T e x-ray shows no racture. You prescribe a knee immobi- downward pressure on the tibia/ bula while internally and
lizer, rest, ice, NSAIDs, and re er the patient to an orthope- externally rotating the lower leg. Pain suggests a meniscal tear.
dic surgeon. T e patient returns 2 days later with marked Pain should be relieved by distracting the joint. T e posterior
e usion and pain. o help relieve the pain, you per orm an sag test is used to detect PCL injury, while the pivot shi test is
arthrocentesis and 90 mL o bloody aspirate is obtained. As used to detect ACL injury.
per your clinic’s standard protocol, the joint uid is sent or
analysis. T e analysis returns with the only abnormalities CASE 12.18
being blood and at droplets.
A 24-year-old emale presents to the clinic 24 hours a er
slipping on a patch o ice outside her home. She reports eel-
Question 12.17.4 Based on the ef usion, you suspect what
ing a “pop” and immediate pain on the lateral aspect o the
diagnosis?
ankle. She reports signi cant swelling in the rst ew hours
A) Complete ACL rupture.
with pain and inability to bear weight initially, but now she
B) Meniscal tear.
is able to walk with a signi cant limp. She reports no signi -
C) ACL and PCL tear.
cant past injuries to the oot or ankle. On examination, you
D) Intra-articular racture.
note edema/e usion over the lateral ankle, some ecchymosis,
E) Patellar subluxation.
tenderness, but no laxity on anterior drawer and inversion
stress. T ere is no bony tenderness on palpation o the oot
Answer 12.17.4 The correct answer is “D.” Fat rom bone
and ankle, but there is tenderness anterolaterally in the so
marrow may be seen even with a small intra-articular racture.
tissue.
Consider C or MRI i racture is not noted on plain lm. I a
racture is still not demonstrated, consider re erral or orthope-
dic consultation. Question 12.18.1 The most likely injury this patient has
suf ered is?
A) Fracture o the distal tibia.
HELPFUL TIP: B) Fracture o the distal bula.
t here s no nee o re er or n a CL njury cu ely C) Sprain o the lateral ligament complex.
( hough hey s ll shoul be re erre ). in c , ou comes D) Sprain o the medial ligament complex.
re he s me reg r less o whe her he a CL s rep re E) Syndesmosis sprain.
or no (BMJ. 2013;346:f232). So w ng couple o ys
or week or n or hope c ppo n men s OK; m ke Answer 12.18.1 The correct answer is “C.” A sprain is most
hem nonwe gh be r ng n cons er mmob l z on, likely because there is no bony tenderness. And, since she is ten-
hough. der laterally, the lateral ligament complex is most likely sprained.

Objectives: Did you learn to . . . Question 12.18.2 In this case, the most likely structure
• Gener e eren l gnos s or knee p n n n hle e? injured would be the:
• d gnose a CL njury? A) Anterior talo bular ligament.
• d e erm ne when knee r ogr phs re ppropr e? B) Distal bula.
C) Distal tibia.
D) Deltoid ligament.
QUICK QUIZ: KNEE Pa iN E) Achilles tendon.

T e best clinical test(s) or determining the presence o a menis- Answer 12.18.2 The correct answer is “A.” T is is a sprain
cal injury is (are): o the anterior talo bular ligament. T is is the rst ligament
A) Posterior sag test. injured with an inversion ankle sprain. It is ollowed by the
B) Apley test. calcaneo bular ligament i enough orce is involved. “E,” Achil-
C) McMurray test. les tendon injury (speci cally rupture), is o special note. First,
D) Pivot shi test. this injury presents as pain in the Achilles tendon area. With
E) B and C. a complete Achilles tendon tear, the patient will have marked
weakness o plantar exion. A diagnostic test (T ompson test)
The correct answer is “C.” T e McMurray test is the best test is to squeeze the posterior cal with the patient lying supine on
or determining meniscal injury. T is is done by exing the the bed and the eet dangling o . In response, the oot should
knee and then extending the knee while per orming internal plantar ex. I this does not occur, consider Achilles rupture.
and external rotation o the tibia/ bula. Keep one hand on the Operative and nonoperative treatments can be used.
356 Fa MiLY MEd iCiNE EXa MiNa t iON & BOa Rd REViEW

A. Pos te rior Ma lle ola r zone


e dge or tip B. Pos te rior
of la te ra l e dge or tip
ma lle olus of me dia l
6 cm
Midfoot zone 6 cm ma lle olus

C. Ba s e of
fifth
me ta ta rs a l D. Navicula r

Lateral view Medial view

An a nkle ra diogra phic s e rie s is re quire d only if the re is a ny pa in in the ma lle ola r zone
a nd a ny of the s e findings is pre s e nt:
(1) bone te nde rne s s a t A
(2) bone te nde rne s s a t B
(3) ina bility to be a r we ight both imme dia te ly a nd in the ED

A foot ra diogra phic s e rie s is re quire d only if the re is a ny pa in in the midfoot zone a nd
a ny of the s e findings is pre s e nt:
(1) bone te nde rne s s a t C
(2) bone te nde rne s s a t D
(3) ina bility to be a r we ight both imme dia te ly a nd in the ED

FIGURE 12-6.

HELPFUL TIP: HELPFUL TIP:


t he O w cr er rel bly pre c who nee s n nkle E rly mob l z on n we gh be r ng re uces he me
r ogr ph n who oes no . t h s h s been v l e o s b l y or nkle spr ns. Res n nonwe gh be r-
n ges 5 o 55. t he O w oo n nkle cr er re ng shoul be m n m ze . a llow he p en o v nce
l s e n F gure 12-6. c v es s oler e .

Objectives: Did you learn to . . .


• i en y p en w h n nkle spr n?
Question 12.18.3 Which o the ollowing is the most appro • d eren e nkle spr n rom r c ure b se on h s ory n
priate management o this patient’s sprained ankle? ex m n on?
A) Cast or 4 weeks ollowed by physical therapy. • Use he O w nkle rules o e erm ne when o ob n n
B) Crutches, nonweight-bearing or 2 weeks, and then progres- nkle r ogr ph?
sive physical therapy. • M n ge p en w h n nkle spr n?
C) Rest, ice, elevation, and early mobilization using external
support, crutches or cane i needed. Progress to activity as
tolerated.
CASE 12.19
D) Re er or orthopedic consultation. A 27-year-old male presents to your clinic ollowing an
E) Immobilization with short-leg walking cast, heat or com- inversion-type injury to the oot and ankle. He cannot bear
ort, analgesics or NSAIDs, and progress to activities as weight on the oot on presentation. He complains o pain and
tolerated. swelling laterally on the oot and ankle. T ere is some so -tissue
swelling but no obvious de ormity. T ere is tenderness over the
Answer 12.18.3 The correct answer is “C.” reatment or most lateral ankle ligaments as well as over the base o the h meta-
sprains includes an external supportive brace, ice application, tarsal. AP and lateral lms o the oot and ankle are obtained and
and elevation; early mobilization is critical and will hasten reveal a nondisplaced racture at the base o the h metatarsal.
recovery. NSAIDs or acetaminophen should be used or pain T e radiograph is available or your review in Figure 12-7.
control. T e patient should be allowed partial weight bearing
as tolerated with crutches or a cane. Patients with recurrent Question 12.19.1 What is the name o this racture ?
problems o instability should be re erred to an orthopedist or A) Jones racture.
evaluation. B) Maisonneuve racture.
CHAPTER 12 • ORt HOPEd iCS a Nd SPORt S MEd iCiNE 357

C) Operative internal xation.


D) Walking boot with crutches or 6 to 8 weeks.
E) None o the above.

Answer 12.19.2 The correct answer is “A.” Nondisplaced


tuberosity ractures generally heal well with conservative mea-
sures. T ese may be managed with a postoperative shoe with
weight bearing as tolerated. A walking boot may be needed
initially i there is signi cant pain present preventing ambu-
lation in a hard soled shoe, however, ankle immobilization is
generally not needed or these avulsion ractures to heal. For a
racture with a displaced ragment > 3 mm, orthopedic re erral
should be considered. Fractures to the metaphyseal–diaphyseal
junction (Jones ractures) result rom a vertical load placed on
the lateral oot, such as an inversion injury or a stress injury.
Jones ractures may potentially be managed with 6 to 8 weeks
in a nonweight-bearing short-leg cast i nondisplaced, however,
many require surgical intervention. Jones ractures should be
re erred due to the high incidence o nonunion.

HELPFUL TIP:
S ress r c ures o he ore oo re common n compe -
ve n recre on l hle es, espec lly er su en
ncre se n c v y, n ens y, ur on, requency, or
FIGURE 12-7. ch nge n sur ce ype. t hey m y be occul on pl n
lms cu ely, bu some sub le per os e l ch nge m y

C) Colles racture.
D) Avulsion racture o tuberosity, base h metatarsal

Answer 12.19.1 The correct answer is “D.” T is is an avulsion


racture o the base o the h metatarsal that may result rom
an inversion ankle injury. Classically, this has been thought to
occur due to an attempt at dynamic stabilization by the pero-
neus brevis, causing an avulsion o the proximal portion o the
metatarsal base. A Jones racture is a transverse racture o the Avuls ion
proximal h metatarsal at the metaphyseal–diaphyseal junc-
tion and typically extends into the inter-metatarsal acet (see
Fig. 2-8). Jones ractures have a high incidence o nonunion
because they occur in a watershed area o blood supply. A mai-
sonneuve racture is a racture o the proximal 1 o the bula
associated with an external rotation injury o the ankle. A colles
racture is a racture o the distal radius (not involving the joint)
with dorsal angulation o the distal racture ragment.
J one s
HELPFUL TIP:
a n un use pophys s n ch l ren n olescen s m y
be con use w h h me rs l vuls on njury. t u-
beros y vuls on r c ures re r nsverse, wh le he
un use pophys s s or en e ver c lly long he long
x s o he me rs l.

Question 12.19.2 Appropriate treatment or the racture


described above would be: Apophys is
A) A hard soled postoperative shoe bearing weight as tolerated.
B) Nonweight-bearing short-leg cast or 6 to 8 weeks. FIGURE 12-8.
358 Fa MiLY MEd iCiNE EXa MiNa t iON & BOa Rd REViEW

Question 12.20.2 Which o the ollowing is true o plantar


be ev en on close ex m n on n he re o m x m l
asciitis?
en erness. i pl n lms re neg ve bu susp c on re-
A) It more commonly occurs in individuals with pes cavus.
m ns h gh, re s r c ure s presen . You m y con-
B) It is more common in women.
s er n MRi or repe pl n lms n re-ex m n on n
C) It is commonly an acute injury.
10 o 14 ys. t he s me re men cons er ons pply
D) Radiographic identi cation o a “heel spur” or osteophyte is
o me rs l s ress r c ures s o ny o her non s-
pathognomonic.
pl ce ore oo r c ure.
Answer 12.20.2 The correct answer is “B.” Plantar asciitis is
not associated with any particular oot type. It is nearly twice
HELPFUL TIP: as common in women as men. It is also more common in over-
Be ler or m oo en erness over he n v cul r. a ll weight individuals. A rupture o the plantar ascia may occur
n v cul r r c ures, nclu ng s ress r c ures, shoul be acutely. Spurring may be seen in up to 50% o patients with
re erre o n or hope s ue o po en l or v scul r plantar asciitis, but is present in 20% o age matched asymp-
necros s. tomatic adults. T us, the nding o a spur is not diagnostic.

Objectives: Did you learn to . . . Question 12.20.3 Appropriate initial treatment or this
• Ev lu e oo njur es? patient’s plantar asciitis should include:
• d escr be h me rs l r c ures? A) A heel cup or silicon pad.
• M n ge s r gh orw r me rs l r c ures n en y B) Achilles stretching.
wh ch r c ures shoul be re erre ? C) Ice or heat.
D) NSAIDs.
CASE 12.20 E) All o the above.

A 40-year-old emale actory worker presents with progres- Answer 12.20.3 The correct answer is “E,” all o the above.
sively worsening heel pain. She has pain when she rst gets Other initial treatments to consider include night splints to
out o bed in the morning. T e pain tends to subside a er maintain ankle dorsi exion and stretch the Achilles tendon and
20 to 45 minutes but is worsened by standing on the con- plantar ascia. Physical therapy modalities such as ultrasound
crete oor o the actory where she works. She has a history may be help ul as well. Advanced treatments, such as corticoste-
o diabetes and hyperlipidemia. On examination, you nd an roid injections, may be considered a er ailure o initial conser-
obese emale with a normal stance and gait. She has exquisite vative therapy. However, injections are not without risk, includ-
tenderness to palpation just distal to the heel on the under- ing plantar ascia rupture, loss o the at pads o the eet causing
side o the oot. Pain is exacerbated by extension o the toes. signi cant pain and disability, and in ection.

Question 12.20.1 Which o the ollowing is the most likely HELPFUL TIP:
diagnosis? No ll heel p n s pl n r sc s. Remember hese
A) arsal tunnel syndrome. o hers: rs l unnel syn rome ( escr be bove), p n-
B) Achilles tendon rupture. ul heel p syn rome (p n loc e over he heel sec-
C) Charcot oot. on ry o bre k own o brous sep e rom overuse
D) Plantar asciitis. n wh ch m y ke up o 6 mon hs o he l), n p ezo-
E) Plantar ascia rupture. gen c p pules (p n over me l/ n er or spec o heel,
en er p pules no e when p en s n ng).
Answer 12.20.1 The correct answer is “D.” Plantar asciitis,
the most common cause o heel pain in adults, is a degenerative Objectives: Did you learn to . . .
condition o the origin o the plantar ascia. “A” is incorrect. ar- • d gnose pl n r sc s n cons er o her c uses o heel
sal tunnel syndrome is due to posterior tibial nerve entrapment n oo p n?
and presents with di use pain at the medial ankle and arch o • d escr be he n ur l h s ory o n re men s or pl n r
the oot. Paresthesias and dysesthesias o en occur as well. “B,” sc s?
Achilles tendon rupture, is incorrect because the pain should be
sudden, stabbing, and located in the cal (not the plantar aspect
o the heel). “C” is also incorrect. Charcot oot does occur in dia- QUICK QUIZ: Ha Nd iNJURiES
betics, but it is actually the result o neuropathy and so generally
does not present with pain. Instead, Charcot oot presents as an A “gamekeeper’s thumb” would likely be seen as the result o
in ammatory condition (e.g., warmth, erythema, and edema) which o the ollowing injuries?
and progresses to joint instability and severe oot de ormities. A) Fall on an outstretched hand.
Finally, “E” is incorrect because plantar ascia rupture should B) Crush injury, or example, between two pieces o machinery.
have a sudden onset and is o en related to trauma. C) Fall by a skier using a ski pole.
CHAPTER 12 • ORt HOPEd iCS a Nd SPORt S MEd iCiNE 359

D) Excessive electronic gaming (e.g., Nintendo and Xbox).


E) Fall rom a mountain bike.

The correct answer is “C.” A gamekeeper’s thumb is de ned as


an injury (partial or complete tear) o the ulnar collateral liga-
ment o the rst MCP joint. T is injury can occur with a val-
gus orce on the thumb, which may occur when alling with a
ski pole. Plain lms should be obtained to evaluate or racture.
reatment is immobilization o the thumb, such as with a thumb
spica splint. Displaced ractures and/or the presence o a Stener
lesion (when a piece o the torn ulnar collateral ligament dis-
places super cially to the adductor pollicis aponeurosis, prevent-
ing healing o the ligament) will require surgical re erral. “Gam-
er’s thumb,” “D,” is a repetitive use syndrome caused by playing
(appropriately enough) video games. It is an overuse syndrome
o en presenting with swelling at the base o the thumb.

QUICK QUIZ: ELBOW FRa Ct URE

T e most speci c nding on radiograph or a radial head rac-


ture is:
A) Anterior at pad sign.
B) Posterior at pad sign.
C) rousseau sign.
D) Medial at pad sign.

The correct answer is “B.” T e posterior at pad sign, which FIGURE 12-9.
indicates an e usion in the joint, is the most speci c o the
ndings or a radial head racture. An anterior at pad sign is
the most sensitive. Figure 2-9, demonstrates both the anterior
and posterior at pad signs. T ere is no medial at pad sign, and
C) Medial condyle racture.
rousseau sign is related to hypocalcemia and is carpal spasm
D) Lateral condyle racture.
with arterial occlusion (as with a blood pressure cu ).
E) Olecranon racture.

QUICK QUIZ: Ra d ia L HEa d FRa Ct URE The answer is “B” supracondylar racture. Although in adults
the most common occult elbow racture is o the radial head,
in the pediatric population it is a supracondylar racture. T ese
T e proper treatment o a nondisplaced, ractured radial head
may be signi cantly displaced, but i nondisplaced they can be
is:
very subtle and the only sign o a racture may be a displaced
A) Internal xation.
posterior (or anterior) at pad.
B) Sling with early mobilization.
C) Short-arm cast.
D) Long-arm cast.
QUICK QUIZ: HiP Pa iN
The correct answer is “B.” For a nondisplaced radial head rac-
ture, the treatment is simply a sling or com ort and early mobi- A 76-year-old emale nursing home resident is brought to clinic
lization. Mobilization is especially important to avoid a sti a er rolling out o bed last night. She is normally able to ambu-
elbow. Orthopedic re erral should be considered i there is any late independently, but today cannot bear weight due to pain in
limitation o motion. her hip. She has signi cant hip tenderness and pain with any
movement o her leg. She is unable to per orm a SLR secondary
to pain. Plain x-rays o her pelvis and hip are read as osteopenia
QUICK QUIZ: PEd iat RiC FRa Ct URES with no obvious racture.

T e most common elbow racture in the pediatric population What is the most appropriate next step?
with a positive “ at pad sign” but no obvious racture on x-ray is: A) Conservative treatment with acetaminophen and bed rest
A) Radial head racture. until able to ambulate.
B) Supracondylar racture. B) Admission or observation.
360 Fa MiLY MEd iCiNE EXa MiNa t iON & BOa Rd REViEW

C) Hip MRI. Answer 12.21.2 The correct answer is “E.” In a clear-cut


D) Physical therapy. case o carpal tunnel syndrome, there is no need or urther
E) Fentanyl patch. studies. I the diagnosis is in doubt, electromyogram and
nerve conduction studies (EMG-NCS) may be o bene t. I
The correct answer is “C.” T e patient may have an occult hip the ROM in the wrist is limited, x-rays may be help ul. At this
racture. Rates o occult hip ractures range rom 2% to 0% o point in time, MRI and orthopedic re erral are not likely to
all hip ractures. o miss the diagnosis o a hip racture may lead add much.
to displacement o the racture and a worse outcome. Both C
and MRI can be used to evaluate or occult racture, but most Question 12.21.3 Which o the ollowing IS NOT associated
studies indicate that MRI has a greater sensitivity. with carpal tunnel syndrome?
A) Hypothyroidism.
CASE 12.21 B) Diabetes mellitus.
C) Amyloidosis.
A 45-year-old emale hospital clerk presents with bilateral
D) Polycythemia vera.
aching pain in the orearms and thenar eminences. T e pain
E) Rheumatoid arthritis.
is made worse with driving and typing. She also has intermit-
tent numbness over the same areas. She tried to ignore the
Answer 12.21.3 The correct answer is “D.” All o the above are
symptoms, but today she dropped her co ee mug on her com-
associated with carpal tunnel syndrome except or polycythe-
puter keyboard and became alarmed at her loss o strength.
mia vera. Polycythemia vera can cause erythromelalgia which is
She has hypothyroidism and is obese, but she reports that her
a burning pain o the hands and eet associated with erythema,
health is otherwise good.
pallor, or cyanosis. It responds to aspirin. Other conditions
associated with carpal tunnel syndrome include pregnancy,
Question 12.21.1 Based on the history alone, which o the menopause, obesity, acromegaly, and end-stage renal disease.
ollowing is the most likely diagnosis? T e point here is that patients with carpal tunnel syndrome
A) Carpal tunnel syndrome. should have a systemic cause ruled out, either clinically or with
B) Osteoarthritis. labs.
C) Ulnar neuropathy.
D) Diabetic neuropathy. Question 12.21.4 What is the most appropriate initial
E) Stroke. treatment?
A) T umb spica splint.
Answer 12.21.1 The correct answer is “A.” Carpal tunnel syn- B) Steroid injection.
drome is due to median nerve entrapment in the carpal tunnel C) NSAIDs and neutral position wrist splints.
o the wrist. ypical symptoms include numbness, paresthesias, D) Short-arm casts.
and pain at the palmar/radial aspect o the hand, quintessen- E) Bilateral gure-o -eight splints.
tially the thenar eminence. In more severe or long-lasting cases,
you may see atrophy o the thenar eminence. Patients may also Answer 12.21.4 The correct answer is “C.” Conservative ther-
develop weakness o thumb opposition. Osteoarthritis o the apy should be initiated rst, unless there is some compelling
wrists does not usually cause nerve symptoms but can cause reason or more aggressive therapy (e.g., severe weakness o
spondylosis and nerve root impingement on occasion. Ulnar the hands and loss o unction). Most patients respond well to
neuropathy involves the ulnar aspect o the hand, especially the NSAIDs and the use o neutral position splints. T e traditional
ourth and h ngers, rather than the radial aspect, which is cock-up splints are not as e ective as neutral position splints.
involved with carpal tunnel syndrome. Diabetic neuropathy T e splints should be worn at night. T e patient may wear the
typically presents in the eet since they are innervated by the splints during the day, too, but should take them o or several
longest nerves in the body. Note: T is could represent cervical hours per day to avoid disuse muscle atrophy. “A” is incorrect
disk disease as well, especially given that it is bilateral. since a thumb spica is not needed. “B,” steroid injection, might
be tried i initial conservative measures ail. However, the ben-
Phalen sign is positive (placing the wrists in a exed position e t is generally limited to month. “D” is just wrong—don’t cast
causes aching and numbness in the median nerve distribu- patients with carpal tunnel syndrome! “E” is a terrible idea as
tion). well since a bilateral gure-o -eight splint is basically a straight
jacket.
Question 12.21.2 What is the best next step in the continu
ing evaluation and management o this patient? HELPFUL TIP:
A) Nerve conduction studies. Or l s ero s h ve been use or c rp l unnel w h l m-
B) Radiograph o the wrist. e success. Un or un ely mos mo l es (NSa id s,
C) MRI o the cervical spine. spl n s, s ero s, e c.) re no be er h n pl cebo n
D) Orthopedic re erral. r n om ze r ls.
E) Initiation o treatment.
CHAPTER 12 • ORt HOPEd iCS a Nd SPORt S MEd iCiNE 361

HELPFUL TIP: d o no rou nely screen or scol os s n symp om c oles-


Ph len n t nel s gns re cru e ools bes . t nel s gn, cen s.
wh ch s p n ul sens on o he ngers n uce by M n n h gh susp c on or sc pho r c ure here s
percuss on o he me n nerve he level o he c rp l r um w h snu box en erness presen .
unnel, m y be pos ve, bu s only 50% sens ve ( l p S r w h ce m nophen or re men o musculoskele l
co n) n 54% spec c. Ph len s gn, keep ng bo h wr s s njur es.
n p lm r- lexe pos on, m y repro uce symp oms.
t o ch eve s er recovery, recommen e rly mob l z on or
Sens v y v r es rom 10% o 88% epen ng on s u y; low-b ck p n n nkle spr ns.
s spec c y s 80%.
Use he O w a nkle Cr er o e erm ne pl n lms re
nee e or ev lu on o nkle njur es.
Objectives: Did you learn to . . .
• d gnose c rp l unnel syn rome n cons er o her c uses
o wr s p n? BIBLIOGRAPHY
• M n ge p en w h c rp l unnel syn rome?
Alshryda S, et al. Acute ractures o the scaphoid bone:
Systematic review and meta-analysis. T e Surgeon.
20 2: 0(4):2 8–229.
QUICK QUIZ: Ca St iNG
Anderson BC. Of ce Orthopedics or Primary Care: reatment.
3rd ed. Philadelphia, PA: Saundesrs; 2005.
A ew days ago, your partner placed a cast on the arm o a
Dartness J, et al. Haematolgenous acute and subacute paediatric
20-year-old male or a distal radial racture. He calls your of ce osteomyelitis. J Bone Joint Surg. Br. 20 2;94-B:584–595.
today, when your partner has gone shing (darn her, anyway).
DeVries JG, et al. T e h metatarsal base: anatomic evaluation
T e patient is complaining o increasing pain and numbness o regarding racture mechanism and treatment algorithms.
his ngers in the casted arm. J Foot Ankle Surg. 20 5;54:94–98.
Dixit S, et al. Management o patello emoral pain syndrome.
Which course o action is most appropriate? Am Fam Physician. 2007;75(2): 94.
A) Have the patient ollow up tomorrow, when your partner is Grif n LY, et al., eds. Essentials o Musculoskeletal Care. 3rd ed.
back in the of ce. Rosemont, IL: American Academy o Orthopaedic
B) Send the patient to the emergency room or compressive Surgeons; 2005.
Doppler examination o the arm to rule out venous throm- Kindale S. Evaluation and treatment o acute low back pain.
bosis. Am Fam Physician. 2007;75(8): 8 – 88.
C) Ask the patient to come to clinic to have the cast replaced. Margaretten ME, et al. Does this adult patient have septic
D) ell the patient that these are expected symptoms and advise arthritis?. JAMA. 2007;297( 3): 478– 488.
him to take some aspirin. McGillicuddy DC, et al. How sensitive is the synovial uid
E) Recommend that he have his cast signed by as many riends white blood cell count in diagnosing septic arthritis?
as possible to take his mind o these clearly psychosomatic Am J Emerg Med. 2007;25(7):749–752.
symptoms. Osmon DR, et al. Diagnosis and management o prosthetic
joint in ection: clinical practice guidelines by the In ec-
The correct answer is “C.” T is patient has symptoms that are tious Disease Society o America. Clin In ect Dis. 20 3;
most likely due to an improperly tted cast. T e problem here 56( ):e –e25.
could be vascular compromise, nerve compression, or a com- Reid D, et al. Acromioclavicular Joint Separations Grades I-III:
partment syndrome. He should be seen without delay to evaluate A review o the literature and development o best practice
or these, and the cast should be adjusted or replaced. T e cast guidelines. Sports Med. 20 2;42(8):68 –696.
can be bivalved or have a window cut into it, then be observed Rixe JA, et al. A Review o the management o patello emoral
or improvement in symptoms i the cast does not need com- pain syndrome. Phys Sportsmed. 20 3;4 (3): 9–28.
pletely removed to rule out the pathology noted above. Sawyer JR, Kapoor M. T e limping child: A systematic
approach to diagnosis. Am Fam Physician. 2009;79(3):2 5.
Simons R, Sherman S, eds. Emergency Orthopedics. 6th ed.
Clinical Pearls New York, NY: McGraw-Hill Pro essional; 20 0.
d o no le he presence c lc um pyrophosph e crys ls Simpson MR. endinopathies o the oot and ankle. Am Fam
or ur c c ssu e you rom suspec ng sep c r hr s Physician. 2009;80( 0): 07.
as both gout and pseudogout commonly coexist with septic Steill IG, et al. Prospective validation o a decision rule or
arthritis. the use o radiography in acute knee injuries. JAMA.
d o no rou nely or er pl n lms or low b ck p n unless 996;275:6 .
r um , bnorm l phys c l ex m n on n ngs, or “re l g” Wilson JJ, Best M. Common overuse tendon problems: A
symp oms re presen . review and recommendations or treatment. Am Fam
Physician. 2005;72(5):8 –8 8.
Pediatrics
Erin R. Howe
13
CASE 13.1 HELPFUL TIP:
Some e ini ions . . . Non-retentive encopresis is he
A 6-year-old male presents to your o ce with his mom. He voluntary s ooling o one’s p n s. On e me i l use
has been soiling his underwear requently and it is causing h s been rule ou , his is purely beh vior l or psy hi-
stress at home and school. His parents are rustrated and ri problem. Primary encopresis is when s ool on i-
think he is old enough to know better. At school, he is being nen e h s never been hieve . Secondary encopresis
teased and being called “stinky pants” and he reports eeling o urs when i en s beg n er perio o su ess-
embarrassed. Mom asks you i there is something medically ul oile r ining.
wrong with him.

Question 13.1.1 What percentage o chronic encopresis


Question 13.1.2 Which o the ollowing conditions is associ
is unctional (meaning no underlying medical or organic
ated with encopresis?
cause)?
A) Urinary tract in ections.
A) 10%.
B) Enuresis.
B) 25%.
C) Social isolation.
C) 50%.
D) ADHD.
D) 75%.
E) All o the above.
E) 90%.

Answer 13.1.1 The correct answer is “E.” Ninety percent o Answer 13.1.2 The correct answer is “E.” Encopresis is an
chronic encopresis is unctional. Functional encopresis is the independent risk actor or urinary tract in ections. Feces in
repeated involuntary passage o stool into underwear in a child the underwear allows or close contact with the urethra and
who is 4 years o age or older and at a developmental level appro- introduction o bacteria. Chronic contracting o pelvic oor
priate or toilet training in the absence o an underlying medical muscles to withhold stool may lead to dys unctional voiding
cause. T e most common reason is retentive constipation with and urinary stasis. Enuresis occurs in up to 40% o patients
over ow incontinence. Frequently, the child has an associated with chronic constipation. A mass o stool in the rectum can
psychological problem. Boys are a ected more o en than girls. compress the bladder leading to uncontrolled voiding. T e
Children with this disorder withhold eces voluntarily thereby chronic odor o eces makes socialization rough and teasing
avoiding de ecating. As stool stays in the colon, more water is requent (recall that this kid has started to be called “stinky
absorbed creating harder stool that is more di cult to pass. T is pants”). ADHD requently is associated with unctional con-
leads to a sel -perpetuating cycle. T e excess stool stretches the stipation and encopresis due to being too distracted to use the
rectum with eventual loss o sensation to de ecate. Liquid stool bathroom.
may leak around the impacted mass, which parents o en mis-
take as diarrhea. In some, a triggering event such as passage o HELPFUL TIP:
a pain ul stool leading to ear o stooling, orced toilet training En opresis or y ime enuresis shoul promp you o
be ore the child is ready, or dislike o using public toilets (starting rule ou spin l or lesion wi h e ile ex min -
school) may have occurred. A common theme is parental misun- ion n ur her im ging su h s n MRI i in i e by
derstanding o the problem thinking it is due to an undiagnosed ex min ion.
medical problem, attention-seeking behavior, or child laziness.

362
CHAPTER 13 • PEd Ia t RIc S 363

Question 13.1.3 Which o the ollowing are medical (organic) a strange concept but it works just as well as manual disimpac-
causes o constipation? tion and is more humane (Pediatrics. 2009;124(6):e1108–15).
A) Hypothyroidism. reatment should be individualized. A er disimpaction,
B) Hypercalcemia. patients require maintenance therapy (combination o medical,
C) Cystic brosis. behavioral, dietary, and counseling). Medications such as milk
D) Lead poisoning. o magnesia (magnesium hydroxide), mineral oil, or polyethyl-
E) All o the above. ene glycol (MiraLAX) should be used daily and incrementally
increased until the patient is having regular, so bowel move-
Answer 13.1.3 The correct answer is “E.” All o the above can ments. Increased ber and uid intake should be encouraged
cause constipation and thus ecal incontinence. Organic causes to help keep stools so . Behaviorally, the child should sit on the
account or only 5% to 10% o encopresis cases and can be ana- toilet with active pushing a er each meal to make use o the
tomical (anterior displaced anus), neurological (Hirschsprung gastrocolic re ex. Even i the child does not stool, he is learning
disease), metabolic (Celiac disease), or iatrogenic (medications to use the toilet and training his bowels. Use o a ootstool may
such as narcotics or anticholinergics). T e rst step in evaluat- improve Valsalva maneuvers. A sticker chart can give visual
ing a child with encopresis is to rule out an underlying medical encouragement. Families need to understand that there is no
condition starting with a good history and physical examina- quick x and relapse is common (about 50%). It takes months
tion. Do not orget to ask about timing o meconium passage in or the rectum to return to normal size and sensation. Patients
the past medical history as delayed meconium passage may sug- who ail to improve a er 6 months o compliant treatment
gest cystic brosis or Hirschsprung disease. Rectal examination should be re erred to a pediatric gastroenterologist.
is essential. Look at the back or signs o an occult spina bi da
such as sacral dimple or hair tu . Lack o anal wink, cremas- HELPFUL TIP:
teric re ex, or deep tendon re exes is suggestive o a neurologic Remember never o use miner l oil in ny hil who is
cause. O en a pain ul ssure may initiate a vicious cycle o pain risk o spir ion. I n use hemi l pneumoni is.
with stooling leading to the withholding o stool leading to even
more pain ul de ecation, etc.
Objectives: Did you learn to . . .
HELPFUL TIP: • d es ribe en opresis, in lu ing i s presen ion, i gnosis,
Fe l imp ion is lini l i gnosis m e by eeling n uses?
h r s ool m ss on re l ex min ion. I is no m e by • d i eren i e be ween org ni n nonorg ni uses o
b omin l x-r y. t his me ns re l ex min ion mus en opresis?
be one. Yes . . . re l ex min ion on hil . No one’s • Me i lly m n ge en opresis?
h ppy bou i ; jus o i .

QUICK QUIZ: NEONat a L POLYc Yt HEMIa


HELPFUL TIP:
a buse m y be riggering even or ons ip ion n Causes o neonatal polycythemia (central venous hematocrit o
en opresis. I buse is suspe e , on hil pro e - 65% or greater) in the immediate postpartum period include all
ive servi es. o the ollowing EXCEP :
A) Delayed umbilical cord clamping.
B) win–twin trans usion.
Question 13.1.4 All o the ollowing are use ul in the treat C) Congenital adrenal hyperplasia.
ment o unctional encopresis EXCEPT: D) Diabetic mother.
A) Polyethylene glycol (MiraLAX®). E) Sepsis.
B) Oral sodium phosphate solution (OsmoPrep).
C) Mineral oil. The correct answer is “E.” Neonatal polycythemia can be
D) Lactulose. caused by all o the above except or sepsis. Other causes include
E) Milk o magnesia. chronic intrauterine hypoxia (e.g., mother is a heavy smoker),
intrauterine growth restriction, maternal hypertension, con-
Answer 13.1.4 The correct answer is “B.” In 2008, the FDA genital hypothyroidism, and chromosomal abnormalities (e.g.,
placed a black box warning on oral sodium phosphate solutions trisomy 21, 18, and 13).
due to risk o acute kidney injury rom deposition o calcium–
phosphate crystals. As a result, nonphosphate-containing medi-
cations are used. reatment begins with disimpaction that can QUICK QUIZ: MORE ON NEONat a L
be per ormed manually, with enemas, or with an oral medi- POLYc Yt HEMIa
cation such as polyethylene glycol (MiraLAX) or mineral oil.
Many treatment guidelines pre er disimpaction using oral poly- Patients with neonatal polycythemia are at risk or hyperviscos-
ethylene gylcol solutions over manual disimpaction. It seems like ity syndrome including respiratory distress, hypoxia, cyanosis,
364 Fa MILY MEd Ic INE EXa MINa t ION & BOa Rd REVIEW

hyperbilirubinemia, and hypoglycemia. Heart ailure, stroke, should lose no more than 10% o birth weight. You should try to
renal vein thrombosis, and necrotizing enterocolitis may also evaluate the mother’s milk supply by asking about the mother’s
occur. eeling o engorgement, eeling o breast emptying with eeding,
seeing milk on the in ant’s lips and tongue immediately a er
T e best treatment or these patients is: eeding, and hearing the in ants swallow with eedings. “B” is
A) Phlebotomy. incorrect. Even though physiologic jaundice peaks at this age,
B) Exchange trans usion with normal RBCs. the level o 25.2 mg/dL is higher than would be expected with
C) Exchange trans usion using D5W. physiologic jaundice (which should not be higher than 17 mg/dL
D) Exchange trans usion using normal saline. in a term in ant). “C” is incorrect because biliary atresia presents
E) Aspirin. with conjugated hyperbilirubinemia (the liver can appropri-
ately conjugate bilirubin, but there is obstruction to the out ow
The correct answer is “D.” Partial exchange trans usion using o the conjugated bilirubin—essentially an obstructive process).
normal saline is the treatment o choice. “A” is incorrect since “D” is incorrect because ABO incompatibility is unlikely in a
phlebotomy alone will reduce overall circulating volume and mother whose blood type is something other than O (who have
may actually exacerbate the problem. “B” is incorrect. I you both anti-A and anti-B antibodies). In addition, Rh incompat-
chose this, back home or you! What is the point o taking out ibility is impossible due to the mother being Rh+ . However,
cells and then putting more in, we ask you? “C” is incorrect minor antigen incompatibility remains a possibility.
because large amounts o D5W can cause uid shi s, electrolyte
abnormalities, and hemolysis. O note, patients with a hemato- HELPFUL TIP:
crit o 65% to 70% can be observed i they are asymptomatic. Causes of an elevated direct (conjugated) bilirubin:
T ere is no one-to-one predictable relationship between hema- in e ion (in lu ing ongeni l), me boli bnorm li-
tocrit and blood viscosity. Also o note, there is no evidence that ies ( ys i ibrosis, Ro or n d ubin Johnson, e .), n -
long-term outcomes are improved in asymptomatic patients omi bnorm li ies (bili ry resi , e .), n holes sis
with partial exchange trans usion versus observation. (espe i lly rom c VN). No e h conjuga ted bilirubin
does not ca use kernicterus.
CASE 13.2 Causes of an elevated indirect (unconjugated)
Amy, a 4-day-old emale, comes to clinic because “she is yel- bilirubin: in re se bre k own o RBc s (a BO/Rh in-
low.” She was born at term to a 25-year-old woman (well o omp ibili y, eph lohem om , h l ssemi s, e .),
course . . . it wouldn’t be a male). Maternal lab results are as prem uri y, hypo hyroi ism.
ollows: blood type A+ , syphilis negative, rubella immune,
group B streptococcus (GBS) negative. Amy has been breast- Question 13.2.2 Which o the ollowing is NOT a risk actor
eeding every 3 to 5 hours. She has had one stool and three or severe neonatal hyperbilirubinemia?
urine diapers a day. Her weight is 15% less than her birth A) Exclusive breast eeding.
weight. On your examination, you notice jaundice rom the B) Gestational age ≥41 weeks.
head to the thighs. Her total bilirubin level is 25.2 mg/dL. T e C) Signi cant birth trauma.
conjugated (direct) raction is 0.4 mg/dL. T e unconjugated D) Visible jaundice in rst 24 hours o li e.
(indirect) bilirubin is 21.2 mg/dL. E) East Asian race.

Question 13.2.1 Which o the ollowing statements is Answer 13.2.2 The correct answer is “B.” Postdates gestational
correct? age is not associated with jaundice. Rather, premature in ants are
A) She has breast eeding ailure jaundice. at a greater risk o jaundice. “A” is true, and exclusively ormula-
B) She has physiologic jaundice. ed in ants are less likely to have severe hyperbilirubinemia (but
C) She has biliary atresia; you must consult pediatric gastroen- this is not a reason to recommend bottle eeding). Cephalohe-
terology. matoma and large bruises result in increased bilirubin produc-
D) T ere is an ABO incompatibility between the mother and tion rom heme breakdown, so “C” is true. T e earlier jaundice
child. occurs, the higher the peak is likely to be, so “D” is also true.
Other major risk actors or severe hyperbilirubinemia include
Answer 13.2.1 The correct answer is “A.” Amy most likely has a sibling who required phototherapy, East Asian race, and blood
breast eeding ailure jaundice. T is occurs within the rst sev- group incompatibility.
eral days o birth be ore the mother’s milk supply is adequate.
T is must be distinguished rom breast milk jaundice, which HELPFUL TIP:
usually occurs later without evidence o dehydration. One sign In n s wi h o l serum bilirubin levels > 5 mg/ L ypi-
that Amy is not receiving adequate breast milk eedings—and lly h ve visible j un i e. t he j un i e usu lly s r s
that this is breast eeding failure jaundice instead o breast milk he he n progresses is lly o he ee . I resolves
jaundice—is the act that she is 15% below birth weight, hav- in he opposi e p ern wi h he is l ex remi ies
ing ewer than 6 wet diapers per day and ewer than two to ve resolving irs .
stools a day. Remember that a well-hydrated in ant o Amy’s age
CHAPTER 13 • PEd Ia t RIc S 365

Answer 13.2.4 The correct answer is “D.” Children should


HELPFUL TIP:
regain their birth weight by age 2 weeks. Anything beyond this
J un i e presen in he irs 24 hours o li e is p ho-
is reason or concern.
logi n usu lly ue o hemoly i ise se su h s a BO
in omp ibili y.
Amy does well under your care (and the bili lights) and
returns at age 2 months. Mom questions the need or immu-
nizations during the well examination “since we never see
Amy’s CBC is unremarkable and blood type is A+ . these archaic diseases anymore—I mean, come on, I can’t even
nd a chicken pox party.” Plus, she’s read in ormation on the
Question 13.2.3 Which o the ollowing is the most appro Internet and has concerns about immunization sa ety. And
priate initial treatment or this patient (remember the con- wasn’t there a study about kids who su ered autism because
jugated [direct] raction is 0.4 mg/dL. The unconjugated o thimerosal in a vaccine? And another study that vaccines
[indirect] bilirubin is 21.2 mg/dL)? trans ormed children into space aliens? Un ortunately, Amy’s
A) Admission or exchange trans usion. mother’s concerns are not unusual. Parents—and politi-
B) Admission or IV uids alone to improve hydration. cians—o en harbor misconceptions about vaccinations.
C) Admission or phototherapy.
D) Discharge to home with recommendations or ormula eed- Question 13.2.5 What is a common side e ect that Amy
ing, light exposure, and ollow-up bilirubin tomorrow. might have a ter her immunizations at her 2 month visit?
A) Fever to 104°F.
Answer 13.2.3 The correct answer is “C.” Amy should be B) Autism.
treated with intensive phototherapy given her level o hyper- C) Diabetes.
bilirubinemia. “A” is incorrect because phototherapy should D) Erythema at the site o immunization.
be employed rst and an exchange trans usion would ollow i E) Symptomatic shedding o virus in her stool.
phototherapy ailed to lower the bilirubin level or signs o acute
bilirubin encephalopathy developed. “B” is incorrect since pho- Answer 13.2.5 The correct answer is “D.” Vaccine side e ects,
totherapy is the de nitive treatment. Adjunctive therapy with such as low-grade evers (not to 104°F as in “A”), induration and
intravenous uids is considered i weight loss exceeds 12% o redness at the site, and ussiness are common. However, they
birth weight, which in this case it does. “D” is incorrect because are sel -limited. “B” and “C” are incorrect. Autism and diabetes
a serum bilirubin level o 25 mg/dL or higher is a medical emer- have NO been linked to vaccines in many large, well-designed
gency and requires immediate hospital admission or photo- studies. “E” is incorrect because the D aP, HIB, and IPV vac-
therapy. cines are inactivated. T us, the virus or bacteria is killed and
puri ed or a speci c component. T e hepatitis B vaccine is con-
HELPFUL TIP: structed with genetic engineering in a yeast cell. T us, one would
t he a meri n a emy o Pe i ri s (a a P) h s es b- not shed virus in the stool. T e MMR, oral polio, oral rotavirus,
lishe hreshol v lues or hyperbilirubinemi re - and varicella vaccines are live attenuated vaccines. With these
men wi h pho o her py n ex h nge r ns usion. immunizations, the virus has been weakened but can still rep-
d e isions reg r ing re men v ry epen ing on he licate and be shed in stool. In an immunocompetent host, this
in n ’s risk, ge (in hours er elivery), ges ion l usually is not o clinical signi cance. T us, “E” is incorrect. Also,
ge, n o l bilirubin level. t here re gr phs, bles, oral polio vaccine is no longer used in the United States because
n nomogr ms (su h s he Bhu ni urve) o ssis the risk o disease rom the vaccine is greater than the risk o
wi h e ision-m king. d e ision-m king l ul or is disease rom wild-type polio; IM polio vaccine only, please.
v il ble : h p://bili ool.org/
HELPFUL TIP:
t he r i le linking MMR v ine o u ism w s wi h-
r wn by he u hors ( n Lancet) ye rs go, n he
You admit Amy or urther management. You con rm Amy’s le u hor h s los his li ense o pr i e be use o
current weight, which is indeed down 15% rom birth weight. r u ulen . He urne ou o be sp e lien ben
Children o en lose about 10%o their weight a er birth. on in e ing he e r h. d espi e his here re on inue
mis ons rue hough s bou v ines.
Question 13.2.4 Beyond what point in time is it considered
problematic i the patient has not returned to his or her HELPFUL (BUT CONFUSING) TIP:
birth weight? Prior o minis ering v ines, pre re ing he hil
A) 5 days. wi h e minophen n in re se p ien om or .
B) 7 days. However, he e minophen seems o blun he im-
C) 10 days. mune response le ing o lower n ibo y i ers. Whe h-
D) 14 days. er his is lini lly signi i n is unknown.
E) None o the above. Don’t worry, be happy.
366 Fa MILY MEd Ic INE EXa MINa t ION & BOa Rd REVIEW

Answer 13.2.7 The correct answer is “B.” ake a moment to


Amy’s mother accepts your advice but still has concerns.
review the developmental milestones in able 13-1.
Question 13.2.6 What other actually correct in ormation
Amy continues her scheduled well child examinations. How-
can you share with her about vaccines?
ever, at 14 months old, mom brings Amy or a sick visit
A) Giving a child multiple vaccines at the same time weakens
because she turned blue. A er taking a complete history and
her immune system.
doing a complete physical examination, you appropriately
B) T e ever and rash side e ects o the MMR are rom the
diagnose breath-holding spells.
measles component and usually occur 1 week a er the
vaccine is given. Question 13.2.8 Which o the ollowing statements about
C) I Amy receives her MMR at a 9-month visit (be ore she breath holding spells is true?
travels to visit Aunt illey who lives in an area where mea- A) T e incidence o breath-holding spells or children between
sles has re-sur aced due to lack o vaccination), her MMR 6 months old and 6 years old is 50%.
immunization would be considered complete a er this dose B) I a color change occurs, it occurs a er loss o consciousness.
and another one at 5 years old. C) Seizure-like activity may occur with breath-holding spells.
D) I Amy had an acute otitis media and evers o 100.4°F, we D) A child typically takes 60 to 90 minutes to return to her
should delay her immunizations until she is a ebrile. baseline a er a breath-holding spell.
E) Immunizations are not important, since these diseases are E) T e evaluation should include an echocardiogram (ECG)
rare in the United States. and electroencephalogram (EEG).

Answer 13.2.6 The correct answer is “B.” T e measles com- Answer 13.2.8 The correct answer is “C.” A typical breath-
ponent can cause a ever and rash 5 to 10 days a er the immu- holding spell begins with an inciting event (like Santa did not
nization. T is occurs in 5% to 15% o in ants. Up to 25% o bring the right toy or mom re used to buy a Happy Meal).
adult women may have arthralgias a er receiving the vaccine. Breath-holding spells occur in up to 4% o children and 80%
T e MMR dosing schedule includes two doses o MMR. How- start be ore 18 months. T e child begins to cry, holds his or her
ever, the rst must be a er 1 year o age. It may be given sooner breath, turns blue, and (may) lose consciousness. A er loss o
i the child is at risk (such as with travel or with a measles consciousness, some rhythmic jerking o the extremities may
outbreak) but must be repeated a er the rst birthday. T us, occur. T e loss o consciousness is brie and the child returns
“C” is incorrect. Minor illnesses should not prevent vacci- quickly to normal activity (no post-ictal state). T e di erential
nation. rue contraindications include anaphylactic reac- diagnosis includes cardiac arrhythmias, seizures, and apnea. I
tions to a vaccine or vaccine constituent, moderate-to-severe the history is classic or breath-holding spells, no urther evalu-
ebrile illness, and encephalopathy within 7 days o D aP. Live ation is necessary. T e treatment is parental reassurance. Par-
virus vaccines are contraindicated in immunocompromised ents should be encouraged to ignore the episodes and not to
patients. give in to child’s requests in attempt to avoid the spells. Iron
supplementation may decrease the requency o spells i anemia
or iron de ciency is present.
HELPFUL TIP:
a s gener l rule, i p ien misses v ine, jus s r
up where you le o . t here is no nee o in re se he HELPFUL TIP:
number o v ines given. I you h ve ques ions, you t his ype o spell in a child under 1 year of age m y
m y onsul he c d c websi e on h-up immuniz - be re erre o in he li er ure s n “a Lt E” ( pp ren
ion re ommen ions ( n n even re e person l- li e- hre ening even ). t he new erm or a Lt E is BRUE
ize h-up s he ule!). (brie resolve unexpl ine even ). t he erm BRUE is
nonspe i i n is e ine by wh he p ren s observe:
pne , y nosis, e re se responsiveness, e . t here
is no rel ionship be ween n BRUE n su en in n
Amy continues her scheduled well child examinations. At e h syn rome (SId S). t he mos ommon uses o n
one o those visits, she was babbling and crawling around the BRUE in lu e lower respir ory r in e ions, GERd ,
o ce. She poked her ngers at the outlets (which thank ully n seizure. a i ion l uses in lu e bre h-hol ing
were covered) and used a pincer grasp to pick up a raisin o spells, ele roly e bnorm li ies, r i ysrhy hmi s,
the f oor (ew, gross!). me boli ise ses, n c NS problems. a lw ys onsi er
hil buse in your i eren i l. Fi y per en o he ime
Question 13.2.7 These behaviors are appropriate or the no use is i en i ie . a goo his ory n physi l ex-
development o a child at approximately age: min ion re he mos impor n ev lu ion ools. d i-
A) 6 months. gnos i es ing shoul be gui e by his ory n physi-
B) 9 months. l ex min ion in ings. See t ie er JS, e l. Pediatrics,
C) 15 months. 2016 M y, or new BRUE gui elines.
D) 24 months.
CHAPTER 13 • PEd Ia t RIc S 367

TABLE 13-1 DEVELOPMENTAL MILESTONES


Age Gross Motor Visual Motor Language Social
1 mo R ises he sligh ly rom H s igh gr sp, ollows o a ler o soun (e.g., by blinking, Reg r s e
prone, m kes r wling mi line moving, s r ling)
movemen s, li s hin up

2 mo Hol s he in mi line, li s No longer len hes s Smiles er being s roke or lke o Re ognizes p ren
hes o ble igh ly, ollows obje
p s mi line

3 mo Suppor s on ore rms Hol s h n s open res , c oos (pro u es long vowel soun s in Re hes or mili r
in prone, hol s he up ollows in ir ul r shion musi l shion) people or obje s,
s e ily n i ip es ee ing

4–5 mo Rolls ron o b k n Moves rms in unison Orien s o voi e; 5 mo: orien s o Enjoys looking roun
b k o ron , si s well o gr sp, ou hes ube bell (lo lize l er lly), s ys “ hgoo,” environmen
when proppe , suppor s pl e on ble r zzes
on wris s, n shi s weigh

6 mo Si s well unsuppor e , pu s Re hes wi h ei her h n , B bbles;7 mo: orien s o bell (lo lizes Re ognizes s r ngers
ee in mou h in supine r ns ers, uses r king in ire ly); 8 mo: “ /m m ”
posi ion gr sp in is rimin ely

9 mo c reeps, r wls, ruises, Uses pin er gr sp, probes Un ers n s “no,” w ves bye-bye;10 S r s o explore
pulls o s n , pivo s when wi h ore nger, hol s mo: “ /m m ” is rimin n ly; environmen , pl ys
si ing bo le, nger- ee s 11 mo: one wor o her h n “ / p - - ke
m m ”

12 mo W lks lone t hrows obje s, le ’s go o Follows one-s ep omm n wi h Imi es ions, omes
oys, h n rele se, uses ges ure, uses 2 wor s o her h n when lle , ooper es
m ure pin er gr sp “ /m m ”; 14 mo: uses 3 wor s wi h ressing

15 mo c reeps ups irs, w lks Buil s ower o 2 blo ks Follows one-s ep omm n In i es some simple
b kw r in imi ion o ex miner, wi hou ges ure, uses 4–6 wor s nee s by poin ing, hugs
s ribbles in imi ion n imm ure j rgon (runs sever l p ren s
unin elligible wor s oge her)

18 mo Runs, hrows oy rom t urns 2 or 3 p ges Knows 7–20 wor s, knows 1 bo y c opies p ren in sks
s n ing wi hou lling ime, lls spoon n ee s p r , uses m ure j rgon (in lu es (e.g., sweeping, us ing),
sel in elligible wor s in j rgon) pl ys in omp ny o o her
hil ren

21 mo Squ s in pl y, goes up Buil s ower o 5 blo ks, Poin s o 3 bo y p r s, uses a sks o h ve oo n o


s eps rinks well rom up 2-wor ombin ions, h s 20 wor go o oile
vo bul ry

24 mo W lks up n own s eps t urns p ges one ime, Uses 50 wor s, 2-wor sen en es, P r llel pl y
wi hou help removes shoes, p n s, uses pronouns (I, you, me)
e ., imi es beh vior o in ppropri ely, poin s o 5 bo y
o hers p r s, un ers n s 2-s ep omm n

30 mo Jumps wi h bo h ee o Unbu ons, hol s Uses pronouns (I, you, me) t ells rs n l s n mes
f oor, hrows b ll overh n pen il in ul shion, ppropri ely, un ers n s on ep when ske , ge s rink
i eren i es horizon l o “one,” repe s 2 igi s orw r wi hou help
n ver i l line

3 yr Pe ls ri y le, n d resses n un resses Uses 3-wor sen en es, plur ls, n Group pl y, sh res oys,
l ern e ee when going p r i lly, ries h n s i p s ense. Knows ll pronouns. kes urns, pl ys well
up s eps remin e , r ws ir le Minimum o 250 wor s, un ers n s wi h o hers, knows ull
on ep o “ wo” n me, ge, sex

4 yr Hops, skips, l ern es ee Bu ons lo hing ully, Knows olors, s ys song or poem t ells “ ll les,” pl ys
going owns irs hes b ll rom memory, sks ques ions ooper ively wi h
group o hil ren

5yr Skips, l ern ing ee , t ies shoes, spre s wi h Prin s rs n me, sks wh wor Pl ys ompe i ive g mes,
jumps over low obs les kni e me ns bi es by rules, likes o
help in househol sks
368 Fa MILY MEd Ic INE EXa MINa t ION & BOa Rd REVIEW

19–23
Vaccine Birth 1 mo 2 mos 4 mos 6 mos 9 mos 12 mos 15 mos 18 mos 2-3 yrs 4-6 yrs 7-10 yrs 11-12 yrs 13–15 yrs 16–18 yrs
mos

Hepatitis B (HepB) 1 dose 2 dose 3 dose

Rotavirus (RV) RV1 (2-dose See


1 dose 2 dose
series); RV5 (3-dose series) footnote 2

Diphtheria, tetanus, &acellular 1 dose 2 dose 3 dose 4 dose 5 dose


pertussis (DTaP: <7 yrs)
Tetanus, diphtheria, &acellular (Tdap)
pertussis (Tdap: >7 yrs)
Haemophilus in uenzae type b See 3 or 4 dose,
1 dose 2 dose
(Hib) footnote 5 See footnote 5

Pneumococcal conjugate6 1 dose 2 dose 3 dose 4 dose


(PCV13)
Pneumococcal polysaccharide
(PPSV23)
Inactivated poliovirus 1 dose 2 dose 3 dose 4 dose
(IPV: <18 yrs)
In uenza (IIV; LAIV) 2 doses for Annual vaccination (LAIV or Annual vaccination (LAIV or IIV)
Annual vaccination (IIV only) 1 or 2 doses IIV) 1 or 2 doses
some: See footnote 8 1 dose only

Measles, mumps, rubella (MMR) See footnote 9 1 dose 2 dose

Varicella (VAR) 1 dose 2 dose

Hepatitis A (HepA) 2-dose series, See footnote 11

Human papillomavirus (HPV2:


(3-dose
females only; HPV4: males and series)
females)
Meningococcal (Hib-MenCY
> 6 weeks; MenACWY-D >9 mos; See footnote 13 1 dose
MenACWY-CRM ≥ 2 mos)

Range of recommended Range of recommended ages Range of recommended ages for Range of recommended ages during Not routinely
ages for all children for catch-up immunization certain high-risk groups which catch-up is encouraged and for recommended
certain high-risk groups

FIGURE 13-1. Re ommen e immuniz ion s he ule or persons ge 0 hrough 18 ye rs.

Question 13.2.9 Between 12 and 15 months o age, Amy excellent CDC app or immunization or both the iPhone and
should receive all o the ollowing vaccines, as per Centers Android plat orms. It is easier to comprehend than the website.
or Disease Control and Prevention (CDC) recommenda
tions, EXCEPT: Amy’s breath-holding spells resolve by the time she is 3 years
A) MMR. old, and she continues to grow and develop normally. Amy’s
B) Varicella. next issue comes when she is 5 years old and in your o ce or
C) Hepatitis A. her pre-kindergarten physical.
D) Rotavirus.
E) In uenza. Question 13.2.10 You notice a new murmur. O the ollow
ing, which suggests that this is a benign murmur?
Answer 13.2.9 The correct answer is “D.” Vaccination sched- A) Murmur is grade II/VI.
ules and recommendations seem to represent quickly moving B) Murmur has a thrill.
targets, so a regular review is required to keep up to date. At 12 to C) Murmur radiates to the apex.
15 months, the CDC recommends initial MMR, varicella, and D) Murmur is diastolic.
hepatitis A vaccinations or all U.S. children. In uenza recom- E) Murmur is holosystolic.
mendations are updated annually and have become progressively
more inclusive over time. T e CDC recommends that all children Answer 13.2.10 The correct answer is “A.” Not all murmurs
ages 6 months through 18 years receive the in uenza vaccine need to be re erred to a cardiologist. T ere are eatures that can
annually during the appropriate season. I the child is receiving help di erentiate pathologic rom benign murmurs. A murmur
her rst year o in uenza vaccination and is between 6 months that is diastolic, grades III to VI, pansystolic, or associated with
and 8 year o age, she should receive 2 doses about 1 month apart cardiac symptoms is likely pathologic and requires urther inves-
to help boost immunity. Following this, the child may receive one tigation. A benign murmur typically is systolic, so (grades I–II,
dose o in uenza vaccination annually. O note, rotavirus vaccine occasionally III), nonradiating, and short. A “benign” murmur
should not be administered to children older than 32 weeks, and is not a diagnosis o exclusion. You should try to make a deci-
the rst dose should not be given a er 15 weeks. sion about which innocent murmur it is. See able 13-2.
O course, the vaccine schedule is updated periodically but
does not usually change drastically rom year to year. We have On Amy’s examination, you hear a grade III/VI systolic mur-
included the current vaccine schedule in Figure 13-1. For the most mur in the pulmonic area with xed splitting o the second
up to date version, go to the CDC website at http://www.cdc.gov/ heart sound. She is otherwise healthy without any cardiac
vaccines/schedules/hcp/child-adolescent.html. T ere is also an symptoms.
CHAPTER 13 • PEd Ia t RIc S 369

TABLE 13-2 “BENIGN” CARDIAC MURMURS • d es ribe n m n ge bre h-hol ing spells?
OF CHILDHOOD • d i eren i e benign rom p hologi r i murmurs in
hil hoo ?
Still murmur: a gr e I–III murmur he r bes in le mi le s ern l
bor er or be ween le lower s ern l bor er n pex. I is
“musi l,” vibr ory or buzzing sys oli eje ion murmur, whi h is
lou er when he p ien is supine, omp re wi h when uprigh , QUICK QUIZ: Ja UNd Ic E
n e re ses in volume wi h V ls lv m neuver. a S ill murmur
m y ge lou er s bloo f ow in re ses wi h ever, exer ise, or T e current recommendation or the treatment o breast milk
ex i emen .
Venous hum: a gr e on inuous I–III murmur he r bes in
jaundice (NO breast eeding ailure jaundice as per the above
he supr l vi ul r re . I shoul resolve when he p ien is case) is:
re umben or wi h pressure over he jugul r vein. A) Continue to breast eed.
Pulmonic murmur: He r s gr e I–III sys oli eje ion murmur B) Stop breast eeding and change to a cow’s milk ormula.
in he rs h l o sys ole. I is gener lly bes he r in he le upper C) Stop breast eeding and change to soy milk ormula.
s ern l bor er.
D) Stop breast eeding and treat with rehydration solution (e.g.,
Murmurs h h nge wi h respir ion re gener lly, bu no lw ys, Pedialyte).
benign.
The correct answer is “A.” Patients with hyperbilirubinemia
due to breast milk jaundice should continue to breast eed.
Question 13.2.11 What is the appropriate next step in man Breast milk jaundice begins on days 5 to 7 o li e, peaks by
agement o this issue? 2 weeks o age, and usually resolves by 10 weeks o age; monitor
A) Family reassurance. to make sure that the jaundice remains unconjugated and there
B) Endocarditis prophylaxis at the dental visit next week. is no need to look or a second source. Bilirubin levels will grad-
C) ECG and chest radiograph. ually decline while breast eeding is continued. I breast eeding
D) Limit physical activity. is discontinued, serum bilirubin levels decline rapidly. Kernic-
E) Re er or immediate operative repair. terus is rare. T e etiology is unclear but glucuronidase in breast
milk resulting in de-conjugation and increased enterohepatic
Answer 13.2.11 The correct answer is “C.” Most likely, Amy recirculation o bilirubin has been proposed.
has an ASD or atrial septal de ect. T e murmur is a systolic
ejection murmur heard best at the upper le sternal border. CASE 13.3
T e sound you hear is caused by increased ow across the pul-
monic valve creating a relative stenosis (more volume needs to A amily has moved into the area and brings their 12-month-
get through a relatively xed outlet) and NO rom ow across old girl in or a well-child check. T ey bring her medical
the ASD. T e increased ow across the pulmonic valve causes records with them. Her parents have no concerns, and she has
a wide, xed split S2. T ere may be a mid-diastolic rumbling been healthy. Her growth chart concerns you, however. Her
murmur at the lower le sternal border rom increased ow length tracks along the 25th percentile. But her weight has
across the tricuspid valve i the ASD is very large. T e chest gone gradually rom around the 25 to 50th percentile at 0 to
x-ray may demonstrate cardiomegaly with increased pulmo- 6 months old to the 10th percentile at 9 months old, and now
nary vascular markings. T e ECG can be normal or may show is at the 5th percentile at 1 year.
mild right ventricular hypertrophy, right axis deviation, and/or
right bundle branch block with the characteristic rsR´ pattern Question 13.3.1 Which o the ollowing most likely explains
in the right precordial leads. Children with an ASD do NO her pattern o growth?
need endocarditis prophylaxis (except or the rst 6 months A) Normal.
a er their surgical repair) and rarely need to limit their physi- B) Familial short stature.
cal activity. Most small ASDs will close spontaneously by age C) Prenatal insult such as exposure to drugs or in ection.
4 years. Closure by one o a variety o methods is recommended D) Hypothyroidism.
or symptomatic and signi cant le -to-right shunts with right E) Inadequate nutrition.
ventricular enlargement.
Answer 13.3.1 The correct answer is “E.” T is patient’s pat-
Objectives: Did you learn to . . . tern o growth is consistent with ailure to thrive (now termed
• d es ribe he uses o neon l hyperbilirubinemi ? “undernutrition”). T e criteria or diagnosis include weight
• M n ge n in n wi h bre s ee ing ilure j un i e? less than the 3rd to 5th percentile (depending on the source)
• I en i y risk ors or severe hyperbilirubinemi ? or a all o weight o > 2 major percentile lines in 6 months or
• Re ommen v ines or hil ren in or n e wi h c d c weight less than 80% o ideal weight or age and/or weight or
gui elines? height less than 5th percentile. T e patient meets more than
• I en i y v ine mis on ep ions, lis MMR si e e e s, n one o these criteria, so “A” is incorrect. In “B,” both height and
i en i y on r in i ions o immuniz ion? weight would be a ected. In this child, only the weight (and
• Re ognize s ges o in n evelopmen ? not the height) is a ected. “C” is incorrect because a child with
370 Fa MILY MEd Ic INE EXa MINa t ION & BOa Rd REVIEW

a prenatal insult tends to be smaller globally in both height 80/50 mm Hg. Her weight is 8 kg. She has dry, cracked lips
and weight, and the weight percentile would not be expected and dry skin. She lies in her mom’s arms and is not very
to drop so dramatically rom birth. “D” is unlikely because in interested in your examination. Her capillary re ll is about
general, the endocrinologic causes result in “stunting” where a 4 seconds with some mottling o the extremities.
child stops gaining height but continues to gain weight. T us,
“E” is the most likely cause o this patient’s ailure to thrive. Question 13.3.3 What is the MOST appropriate next step in
the management o this patient’s condition?
aking more history, you discover that the patient eats mostly A) Evaluate or underlying bacterial in ection with blood,
rice and ruit. She does not like meat (happily supported urine, and stool cultures.
by one o your editors who is vegetarian!). She is picky. She B) Admit to an inpatient unit and begin maintenance IV uids.
drinks about 12 oz o whole milk and 12 oz o apple juice C) In use 20 mL/kg normal saline IV over 20 minutes.
each day. She is active and developmentally appropriate. Her D) Begin oral rehydration with Pedialyte.
review o systems is negative as is her past medical history. E) In use 10 mL/kg D5/hal -normal saline over 2 hours.

Question 13.3.2 What is the LEAST appropriate way to pro Answer 13.3.3 The correct answer is “C.” T is patient is
ceed with evaluation and management o this patient? severely dehydrated (estimated 15%) with signs o compro-
A) Allow the child to eed hersel whenever she is hungry or mised tissue per usion; there ore, she should have parenteral
thirsty. Parents should o er ood and drink requently uid resuscitation quickly with a 20 mL/kg isotonic uid bolus.
throughout the day. Although she might need admission, she should be stabilized in
B) Obtain CBC, iron studies, and urinalysis. the ED be ore trans er to the inpatient unit; in addition, starting
C) Limit juice intake. with maintenance uids is inappropriate. Oral rehydration is
D) Consult a nutritionist. cost-e ective in less severe dehydration without compromised
E) Prescribe a multivitamin. tissue per usion. T ere are several tables published listing the
clinical examination associated with various degrees o dehy-
Answer 13.3.2 The correct answer (and least appropriate dration. See able 13-3 or clues in determining the degree o
approach) is “A.” T e goal o management o ailure to thrive is dehydration.
to increase caloric intake, identi y treatable causes o ailure to
thrive, and look or e ects o her malnutrition. T us, a CBC, iron HELPFUL TIP:
studies, electrolytes, bicarbonate, glucose, and urinalysis may be d ehy r ion is lini l i gnosis bu l bor ory in -
help ul to investigate possible iron de ciency, anemia, or renal ings n be help ul. t he serum so ium l ssi ies he
tubular acidosis. Juice should be limited (due to its low nutritional ehy r ion s hypo oni , iso oni , or hyper oni . Me -
value), meals scheduled, and caloric intake boosted. Although boli i osis is o en presen n l ul ion o he
children should eed themselves, they take in more calories when nion g p helps e ermine he use. Bloo ure ni ro-
given a set schedule or meals and snacks instead o being allowed gen n serum re ine on en r ions n be help ul
to graze on ood or drink throughout the day as in “A.” A nutri- bu m y be e e by o her on i ions.
tionist can help educate the amily on appropriate ood choices
or a toddler and ways to boost calories. A child with a restrictive
eating pattern, especially poor in iron-containing oods (such as
A er completing the initial management, the patient is more
meats), may bene t rom a multivitamin with iron.
interested in her surroundings. Her vital signs are pulse
160 bpm, respiratory rate 36, and blood pressure 80/50 mm
HELPFUL TIP:
Hg. Her lips are still dry and she is irritable. T e capillary
For in n s wi h ilure o hrive, en our ge he p ren s
re ll is 2 to 3 seconds. She is more interactive with her mother.
o in re se he lori ensi y o oo s n rinks. a sk
hem o use whole milk r her h n re u e milk,
o in re se he on en r ion o ormul ( ing less
TABLE 13-3 SIGNS AND SYMPTOMS OF
w er), bu er o oo , e .
DEHYDRATIONa
Mild (5% in an infant, 3% in a child): Norm l skin urgor, mois lips,
e rs presen , norm l vi l signs, onsol ble
Your patient presents to the emergency department (ED) at 14 Moderate (10% in an infant, 6% in a child): d ry skin n lips, skin
months old. Mom states that she has had several episodes o en ing, sligh ly in re se pulse, e re se urine ou pu , norm l
emesis over the last 48 hours and today began to have watery, pill ry re ll
oul-smelling diarrhea (as opposed to the yummy-smelling Severe (15% in an infant, 9% in a child): P r he lips, sunken eyes,
diarrhea to which we are so accustomed). Mom has lost track e re se or no urine ou pu , ool skin, elev e pulse, prolonge
pill ry re ll, le h rgi or ob un e
o number o episodes o emesis and diarrhea. She cannot
tell i the child has had urine output because o the watery a
Be use o he poor sso i ion o physi l n ings wi h p r i ul r
diarrhea. On your examination, the temperature is 102.5°F, per en ehy r ion use he erms “mil , “mo er e” or “severe” is pre erre
heart rate 180 bpm, respiratory rate 50, blood pressure over rying o ssign per en ehy r ion.
CHAPTER 13 • PEd Ia t RIc S 371

Question 13.3.4 What urther treatment is indicated or her Question 13.3.5 What is the most likely diagnosis?
dehydration? A) Streptococcal pharyngitis.
A) 80 mL/hr × 2 hours o D5-quarter-NS. B) Hand, oot, and mouth disease (coxsackie virus).
B) 33 mL/hr × 24 hours o D5-hal -NS with 20-mEq KCl. C) Herpetic gingivostomatitis.
C) 50 mL/hr × 24 hours o D5-quarter-NS. D) Varicella.
D) 20 mL/kg NS over 20 minutes. E) In ectious mononucleosis.
E) 10 mL/kg D5-hal -NS over 2 hours.
Answer 13.3.5 The correct answer is “C.” Primary oral her-
Answer 13.3.4 The correct answer is “D.” A er an initial pes (herpetic gingivostomatitis) is associated with a relatively
bolus, the patient should be reassessed. T is patient is still mod- high ever and anteriorly placed ulcerations and vesicles (gums,
erately dehydrated. Although she has responded to her initial tongue, and lips). Symptoms tend to start relatively abruptly
bolus o normal saline, she still has abnormal vital signs as well with pain, salivation, re usal to eat, and ever. Herpes gingivo-
as signs o dehydration on examination. T ere ore, the normal stomatitis may recur during li e in the orm o “cold sores.” “B”
saline bolus should be repeated. Hypotonic solutions (e.g., is incorrect because the patient does not have hand and oot
NS) or those containing glucose and/or potassium should never lesions. “D,” varicella, can occur in the oral pharynx but would
be used as a bolus. Also, potassium (i needed at all) should have lesions elsewhere in various stages and respiratory symp-
be withheld until the patient urinates and has better hydration toms as well. In ectious mononucleosis is not associated with
status. vesicular lesions or mucosal ulcerations.

HELPFUL TIP: HELPFUL TIP:


M in en n e lui s n be l ul e using he Holi y- t he poorly n me “herpangina,” use by en erovi-
Seg r me ho : ily w er nee s or he irs 10 kg o ruses (usu lly oxs kie a ), is no her onsi er ion in
bo y weigh is 100 mL/kg/ y; w er nee s or he nex he hil wi h ever n mou h sores. Herp ngin is s-
11 o 20 kg is 50 mL/kg/ y; e h i ion l kg over 20 so i e wi h ewer vesi les n ulcers located in the
kg is 20 mL/kg/ y. d o no orge o oun or ehy- posterior oropharynx along the posterior soft pal-
r ion, whi h will require i ion l lui s o repl e ate and tonsillar pillars and fossa. He he, vomi -
he lui e i i . a lso, repl e ongoing losses rom i r- ing, n b omin l p in m y be presen s well. No e
rhe , emesis, e . You n lso l ul e he hourly lui h his is i eren lini l en i y rom h n , oo , n
requiremen by he “4, 2, 1” me ho . c hil ren shoul mou h ise se espi e he en erovir l use.
ge 4 ml/kg/hr or he irs 10 kg bo y weigh , 2 mL/kg/
hr or he nex 10 kg n 1 mL/kg/hr or every kg over
20 kg. Next, you see the patient’s sister who is age 10 and presenting
or a school physical. Her mother is concerned because this
Flui e i i (L) = usu l weigh (kg) − urren weigh (kg)
patient has pubic hair, which her mother thinks is prema-
ture (and scary . . . her baby is not growing up yet!). On your
examination, she has enlarged areola and a small amount o
HELPFUL (AND IMPORTANT) TIP: breast tissue. She also has sparse, dark, but mostly straight
t r i ion lly, “pe s” solu ion (1/4 NS) h s been use pubic hair.
or m in en n e lui s in hil ren. t his le s o hypo-
n remi n seizures. Norm l s line shoul be use Question 13.3.6 This patient is sexual maturity rating or
s m in en n e lui jus s in ul s (Lancet. 2015; Tanner stage:
385(9974):1190–1197; 28). A) 1.
B) 2.
C) 3.
D) 4.
T e patient recovers well rom her GI illness, but she returns a E) 5.
ew months later to your clinic with a new illness. She has had
temperatures o 103°F at home or 2 to 3 days. She has had no Answer 13.3.6 The correct answer is “B.” Breast buds and
upper respiratory symptoms. Her oral intake has decreased, sparse, downy pubic hair put the patient in anner stage 2. an-
but she is maintaining good urine output. She has had no ner stage 1 is prepubertal. Any sign o puberty moves a child rom
vomiting or diarrhea. Your examination reveals a ebrile child anner 1 to anner 2. anner stage 5 is adult or ully matured
who is slightly irritable. She is nontoxic and not dehydrated. secondary sexual characteristics. See able 13-4 or details.
Her oral cavity shows increased tonsil size with ulcers on her
tongue and lips but not on the tonsillar pillars. Her anterior T e mom has a lot o questions about puberty. She is ner-
cervical lymph nodes are enlarged. T e rest o her examina- vous about beginning to discuss sexual development with her
tion is noncontributory. daughter.
372 Fa MILY MEd Ic INE EXa MINa t ION & BOa Rd REVIEW

TABLE 13-4 TANNER STAGES


QUICK QUIZ: FEEd ING INFa Nt S
S ge 1: Prepuber l
S ge 2: Sp rse grow h o owny h ir b se o penis or long l bi
Which is not an indication or use o soy ormula in an in ant?
• Girls: Bre s bu evelopmen .
• Boys: Enl rgemen o s ro um n es es wi h re ening n A) Galactosemia.
hi kening o s ro l skin B) Congenital lactase de ciency.
S ge 3: d rkening, o rse h ir sp rsely in pubi re C) Cow milk protein intolerance.
• Girls: Enl rgemen o bre s s (prim ry moun ); Boys: elong ion o D) Parental desire or a vegan diet.
penis n in re se size o es es.
S ge 4: a ul pubi h ir is ribu ion ( ri ngle) bu sm ller re The correct answer is “C.” Few indications exist or use o
overe wi h no ex ension o me i l highs soy-based ormula over cow milk-based ormula. Soy ormula
• Girls: a reol n p pill begin o moun bove he level o he
is lactose ree and there ore should be used in conditions “A,”
bre s (se on ry moun )
• Boys: In re se in bre h n evelopmen o penis, rkening o “B,” and “D.” Galactosemia is a metabolic disorder character-
s ro l skin, enl rgemen o es s ized by the inability to breakdown galactose. Dietary galac-
S ge 5: a ul bre s s, geni ls, n h ir is ribu ion. tose comes rom lactose ound in human and cow milk. An
extensively hydrolyzed or amino acid ormula should be used
or cow milk protein intolerance. In ants who are sensitive to
Question 13.3.7 What is a developmental act that you can cow milk protein have an approximately 10% to 15% chance
provide the mom or discussion with her daughter? o also being sensitive to soy proteins. Re ux deserves special
A) ypically, in emales, the rst sign o puberty is pubic hair. note. T ere is no data supporting the use o soy ormula or
B) At 10 years old, the patient is experiencing premature re ux.
puberty.
C) T e height spurt occurs early during puberty and is almost CASE 13.4
complete at menarche.
D) Menarche occurs approximately 1 year a er breast budding. A 2-year-old male presents to your o ce with his ather
E) Schools discuss puberty and relieve mom or dad o that duty complaining that the patient has had intermittent crying
(whew!). ollowed by episodes o pro ound lethargy during which he
is di cult to arouse. T e ather relates that the patient will
Answer 13.3.7 The correct answer is “C.” In girls, the growth sometimes clutch at his abdomen and roll into the etal posi-
spurt occurs early and is almost complete at menarche. Females tion during episodes. T ese episodes have been going on or
begin puberty about 1 to 2 years be ore males. “A” is incorrect about 4 hours. He has had no ever, no vomiting, and no diar-
as thelarche, or breast budding, marks emale onset o puberty. rhea. Nobody else at home has been sick. He is developmen-
“B” is incorrect. T e average age o breast budding is 10.9 years. tally normal and has no signi cant past medical history. His
T us, Sara is appropriate in her pubertal timing. “D” is incor- examination reveals that he is a ebrile and lethargic but can
rect. Menarche typically begins 2 years a er breast budding. “E” be aroused. His abdominal examination is benign. T e rest o
is incorrect. Although schools may discuss puberty, these dis- the examination is unremarkable.
cussions are o en limited in the time and content. It is impor-
tant or the child to hear her mother’s opinions on puberty and Question 13.4.1 Given his presentation, your clinical suspi
sex. cion is or:
A) Meningitis.
HELPFUL TIP: B) Renal colic.
Pre o ious or e rly puber y is e ine s he onse o C) Intussusception.
se on ry sexu l h r eris i s be ore ge 8 or girls D) Gastric outlet obstruction.
n ge 9 or boys. d el ye puber y is he l k o su h E) Appendicitis.
h nges by ge 14.
Answer 13.4.1 The correct answer is “C.” T e patient may
have intussusception. T e combination o abdominal colic
Objectives: Did you learn to . . . plus mental status changes should suggest intussusception. T e
• d e ne ilure o hrive in hil ? etiology o the mental status changes is not clear, but mental
• Ev lu e hil wi h ilure o hrive? status changes are one o the classical ndings o intussuscep-
• Ev lu e ehy r e hil n e ermine severi y o ehy- tion. “A,” meningitis, should still be a consideration in a patient
r ion? with mental status changes, but abdominal pain is not usually
• M n ge ehy r ion in he Ed ? part o the presentation. In addition, the patient is a ebrile and
• c l ul e rehy r ion f ui s or ehy r e hil ? his symptoms are intermittent, making meningitis less likely.
• Gener e i eren i l i gnosis or ph ryngi is wi h blis- “B” is not associated with mental status changes. “D” and “E”
ers? usually present with some abnormal ndings on abdominal
• d es ribe h nges o puber y n t nner s ging in em le? examination.
CHAPTER 13 • PEd Ia t RIc S 373

HELPFUL TIP: Well, the ultrasound tech is in the Cayman Islands, so you
Fewer h n 15% o p ien s wi h in ussus ep ion pres- opt or an air enema, which simultaneously diagnoses and
en wi h he l ssi ri o oli ky b omin l p in, reduces the intussusception. T e patient appears well with no
p lp ble s us ge-sh pe b omin l m ss, n urr n signs o per oration. He is eating gold sh crackers by the st-
jelly s ool. ul in the ED and generally making a mess o the examination
room. Recommendations vary as to the patient’s disposition.
Some authors recommend admission since recurrence rate is
Question 13.4.2 Which laboratory nding will help you to 10%in the rst 24 hours while others would let the child go
con rm the diagnosis o intussusception? home. We suspect admission will be the right answer or the
A) CBC. test.
B) Urinalysis.
C) Glucose.
D) Serum lactate. HELPFUL TIP:
E) None o the above. Mos ses o in ussus ep ion re i iop hi . Less h n
10% o p ien s h ve p hologi le poin su h s
Answer 13.4.2 The correct answer is “E.” None o the above Me kel iver i ulum, in es in l polyp, or lymphom . In-
will help to make the diagnosis o intussusception. Early in re se ge o hil in re ses he risk o le poin .
the course, the CBC may be normal and only 75% will have
heme positive stools.
HELPFUL TIP:
d i eren ypes o in es in l obs ru ions o ur i er-
HELPFUL TIP:
en ges. Obs ru ion bir h is ypi lly n n omi
“c urr n jelly s ools” re l e in ing in in ussus ep-
resi , me onium ileus, or Hirs hsprung ise se. Vol-
ion n re re le ion o mu os l is hemi . You re lly
vulus ue o m lro ion n o ur ny ge, bu mos
w n o m ke he i gnosis be ore you see “ urr n jel-
ommonly ppe rs in in n y (even wi hin he irs
ly” per re um. (Hey, h ve you ever u lly seen urr n
mon h o li e . . . w h or bilious vomi ing, sign o vol-
jelly? I is brigh re n gloopy, kin o like gr n m ’s
vulus). Be ween bir h n 5 weeks o ge, hyper rophi
r nberry s u e.)
pylori s enosis ppe rs. In ussus ep ion is he mos
ommon use o GI obs ru ion be ween 3 mon hs
Question 13.4.3 You decide that this patient probably has n 6 ye rs o ge.
intussusception based on your history and physical exami
nation. The next step in diagnosing this patient is:
Objectives: Did you learn to . . .
A) Upper GI with barium.
• Re ognize he lini l presen ion o in ussus ep ion?
B) A plain lm o the abdomen.
• Ev lu e n re hil wi h in ussus ep ion?
C) An air enema.
D) An abdominal ultrasound.
E) Administration o IV antibiotics. CASE 13.5
Benjamin, a 4-month-old male, comes to your o ce or a
Answer 13.4.3 The correct answer is “D.” In general, an ultra- well-baby check. His parents have no concerns. Development
sound is the diagnostic modality o choice. An ultrasound may and eeding are progressing as expected. Looking at him, you
show a typical “bull’s eye” lesion helping to make the diagnosis. think to yoursel , “My gosh. In ants have such big heads rela-
“C,” an air enema, is a diagnostic and therapeutic intervention. tive to their bodies.” On your examination, his head is >95th
An air enema can be used to reduce the intussusception obviat- percentile, but his weight and height are still at the 50th per-
ing the need or surgery. In general, however, a diagnosis is rst centile. His previous head circum erences have all been at the
made by ultrasound. An upper GI will be o no use since intus- 50th percentile. He is a social in ant but is having di culty
susception is a distal process. A plain lm o the abdomen will supporting his large head.
most likely be nondiagnostic as is the case in most abdominal
processes (except perhaps bowel obstruction or ree air). Question 13.5.1 What is the imaging study LEAST likely to
assist in the diagnosis o the macrocephaly?
HELPFUL TIP:
A) X-ray.
For in ussus ep ion, n ir or w er-soluble on r s en-
B) C scan.
em is pre erre over r i ion l b rium enem . Bowel
C) MRI scan.
per or ion is he m in risk o nonoper ive re u ion.
D) Ultrasound.
a ir n w er-soluble on r s re mu h less h rm ul
h n b rium i le ke in o he peri oneum.
Answer 13.5.1 The correct answer is “A.” Because the relative
head size is so dramatic, an imaging study is needed, but skull
374 Fa MILY MEd Ic INE EXa MINa t ION & BOa Rd REVIEW

radiographs are not likely to assist you in arriving at a diagnosis.


As you have done in this case, radiological studies should
T e other modalities are used in this setting, and each has its ben-
always be obtained in the evaluation o suspected child abuse.
e ts and detractions. Ultrasound is o en used in in ants when the
However, the cause o ractures has a broader di erential
anterior ontanelle is still patent and o ers the advantage o no
than abuse.
radiation exposure. Ultrasound is primarily used to assess uid-
lled spaces (ventricles in hydrocephalus, subdural hematoma,
Question 13.5.3 Which patient’s injuries and/or radiologi
etc.). C scan is a rapid and readily available test that can detect
cal studies would be pathognomonic o abuse?
hemorrhages, intracranial calci cations, skull ractures, Chiari
A) A 9-month-old male with widened distal radius and ulna
mal ormations, and more but given the concern or radiation
with raying and cupping evident on radiograph.
exposure and the risk o uture malignancy, it is not the rst line
B) A 21-month-old emale with multiple ractures at di er-
imaging technique. C is also limited in its ability to delineate
ent ages on radiograph. She has delayed tooth eruption and
so -tissue problems, which is where MRI becomes important.
sparse hair.
C) A colicky 1-month-old male in ant with callus on the le
HELPFUL TIP:
clavicle radiograph.
t he i eren i l i gnosis o m ro eph ly is long n
D) A 2-year-old emale with a spiral racture o the distal right
in lu es hy ro eph lus, br in umor, benign mili l
tibia.
m ro eph ly, benign enl rgemen o he sub r h-
E) None o the above.
noi sp e (BESS), neuro u neous isor er (e.g., neu-
ro ibrom osis, uberous s lerosis), hemorrh ge, n
Answer 13.5.3 The correct answer is “E.”None o these injuries is
me boli isor ers (e.g., t y–S hs ise se, mu opoly-
diagnostic o abuse. Although each o these children would make
s h ri oses).
you think o possible abuse, none o them have been abused. “A”
is a child with rickets. “B” is a child with osteogenesis imper ecta.
Question 13.5.2 The ultrasound tech is still on vacation “C” is a child who sustained a clavicle racture at birth. “D” is a
(why can’t we have his job?) and your MRI is undergoing child with a “toddler’s racture.” A toddler’s racture is a spiral tib-
scheduled maintenance, so you order a CT scan, which ial racture ound in children age 9 to 36 months (see Chapter 12).
shows subdural hematomas. What is your next step in T ese ractures are ound in the distal one-third o the tibia.
managing this patient?
A) Admit to the hospital. Benjamin’s evaluation shows bilateral subdural hemorrhages
B) Examination o eyes in an ophthalmologist’s o ce. and retinal hemorrhages. His skeletal survey is negative. T e
C) “Babygram” to view entire skeleton o in ant or ractures. state agency removes him rom the home and places him
D) Alert the department o human services within the next week. in oster care. Benjamin then returns or his 6-month well
E) Send the patient to a neurosurgeon’s o ce or the next avail- child examination. He has been in oster care since you last
able appointment. saw him. His vision is being ollowed by an ophthalmologist.
He smiles and squeals. His oster mother says, “Looks like
Answer 13.5.2 The correct answer is “A.” Subdural hematomas he’s right-handed. Does that mean that he’s advanced or his
in an in ant are highly suspicious o abusive head trauma. Your age?” He rolls both ways. He has started on rice cereal and
rst priority is to protect the child by admission to the hos- bananas. He sleeps well. On your examination, he continues
pital. A retinal examination and skeletal survey are important to have macrocephaly but a so ontanelle. His le hand is
in the evaluation but are secondary to protection o the child. sted. T e rest o the examination is unremarkable.
A “babygram” should never be done to evaluate or skeletal
trauma. T ere is a speci c procedure or per orming a skeletal Question 13.5.4 What is your assessment and plan or
survey or abuse. Your state agency responsible or investigating Benjamin?
child abuse must be noti ed within 24 hours (although clini- A) Normal 6 months old: Routine anticipatory guidance and
cians are encouraged to know their legal responsibilities in the immunizations.
state in which they practice). Protection o the child is most B) In ant with history o head trauma: Delay pertussis vaccina-
important; i the amily ees, calling the police or the appro- tion because o the autism risk.
priate state agency would be the rst step. Suspicion o abuse, C) In ant with history o head trauma: Per orm Denver Devel-
especially abusive head trauma, is an emergency, and it should opmental Screening est and document ndings; repeat in
not wait or a neurosurgeon’s next available appointment. 3 months.
D) Hypertonic le upper extremity: Monitor progress at next
HELPFUL TIP: well child check.
Re in l hemorrh ges, re highly suspi ious or buse. E) Hypertonic le upper extremity: Re er or urther evalua-
Non busive uses o re in l hemorrh ges re very r re. tion and treatment o his delay.
V in ions, mil he r um (su h s lling o o
so ), c PR, seizures, n rou ine pl y o not use re i-
n l hemorrh ges. Answer 13.5.4 The correct answer is “E.” Benjamin has a risk
o developmental delays due to his abusive head injury. He
CHAPTER 13 • PEd Ia t RIc S 375

should be monitored closely or delays. One way to monitor


or delays is by per orming a screening test, such as the Den- CASE 13.6
ver Developmental Screening est. However, Benjamin has A 7-year-old male presents to your clinic with his mother
concerns that go beyond screening. His tightly sted le hand with a complaint o enuresis. Evidently, this child has never
with corresponding avoring o his right hand suggests spastic been completely continent at night, wetting his bed several
hemiplegia. Early hand pre erence is a worrisome sign. ypi- times per week. T is has become somewhat o a problem
cally hand pre erence is not de nite be ore 18 to 24 months. or him now that his riends are having sleepover birthday
Hand pre erence should prompt the physician to examine the parties. Plus, his mother con des that she’s tired o paying
other extremity. In this case, there is hypertonicity resulting or pull-ups and cleaning sheets. His incontinence is mono-
rom his trauma. Any child with concern o developmental symptomatic (meaning no overactive bladder symptoms or
delay should be re erred or urther evaluation. Some local daytime wetting).
and state agencies o er a ree service to amilies to evaluate
development and o er therapy. Do NO watch and wait with a Question 13.6.1 What percent o 7 year old continue to
suspected delay. T e earlier intervention begins, the better the have enuresis?
outcome. A) 1%.
B) 10%.
Objectives: Did you learn to . . .
C) 20%.
• Re ognize n ev lu e m ro eph ly in n in n ?
D) 40%.
• Gener e i eren i l i gnosis o m ro eph ly in n
E) 75%.
in n ?
• d e ermine when hil buse shoul be in lu e in he
Answer 13.6.1 The correct answer is “B.” About 10% continue
i eren i l i gnosis?
to have enuresis at age 7 years which tapers to 1% to 2% at age
• d es ribe some spe s o evelopmen l el y n i s
15 years or greater. Even though this would suggest that it is a
m n gemen ?
variant o normal, there is extreme social pressure on children
age 7 to remain dry at night. Parental expectations may depend
on their experience with other children. For example, i an older
QUICK QUIZ: d EVELOPMENt a L d ELaY
sibling was dry at night at age 3 or i a younger sibling is dry
while an older child has enuresis, parents may not accept the
A mother brings her 18-month-old son in or a routine visit but bed wetting as normal. Age 7 is considered the cuto or “nor-
is concerned that he is not as verbal as he was 3 months ago. mal” enuresis; there ore, beyond age 7 years, it is considered a
She reports that he eats well. He just started walking. You wish problem.
to screen him or a pervasive developmental disorder (which
includes autism, Rett syndrome, Asperger syndrome, childhood
disintegrative disorder, and others). HELPFUL TIP:
No e h he hil in his se is m le. Enuresis e s
What is the most appropriate o ce tool to use with this patient wi e s m ny m les s em les.
given his age and presenting symptoms?
A) Ages and Stages Questionnaire (ASQ).
B) Conner’s Rating Scale—Revised Short Form. Because he has had no period o nocturnal dryness, this
C) Denver Developmental Screening est. patient has primary enuresis.
D) Modi ed Checklist or Autism in oddlers (M-CHA ).
E) Parents Evaluation o Developmental Status (PEDS). Question 13.6.2 Which o the ollowing is likely to be part o
this child’s history?
The correct answer is “D.” T e Modi ed Checklist or Autism A) A amily history o enuresis.
in oddlers (M-CHA or short) is a validated o ce-screening B) A stress ul event in the amily such as the birth o a new
tool or autism spectrum disorders. It would be the most appro- child or parental divorce.
priate tool o the options given. O note, the AAP recommends C) Increased uid intake over the past 2 months.
screening all children or autism spectrum disorders using a D) History o urinary tract in ections.
standard screening tool such as the M-CHA at ages 18 and
24 months. “A” is incorrect as the Ages and Stages Questionnaire Answer 13.6.2 The correct answer is “A.” Enuresis is divided
is a broad-based developmental screening tool, as is “D”, the into primary and secondary enuresis. Primary enuresis occurs
Denver Developmental Screening est. “B” is incorrect because in cases where there is never a consistent period o dryness
this test is used to detect ADHD. Note that the USPS F does at night. Secondary enuresis occurs when there is a period o
not recommend or or against autism screening between 18 and dryness (by convention, 6 months) be ore the patient develops
30 months noting that there is insu cient evidence to take a enuresis. “B,” “C,” and “D” would more likely be seen in children
position on such screening. However, most practitioners ollow with secondary enuresis. Primary enuresis tends to be a amilial
the AAP recommendations. trait.
376 Fa MILY MEd Ic INE EXa MINa t ION & BOa Rd REVIEW

is achieved until a maximum is reached. T e idea is that the


HELPFUL TIP:
patient is conditioned to respond to his increasing bladder
a bou 15% o p ien s wi h enuresis be ome ry e h
capacity.
ye r. t he longer enuresis persis s, he less likely i is o
“C,” an enuresis alarm, is an e ective treatment but relatively
resolve.
expensive and requires signi cant motivation on the part o
the amily. T e enuresis alarm increases success o the other
options, and when used in combination with other modalities, it
has shown the best long-term results. “D” and “E” are incorrect
T is patient’s examination is essentially normal including in this scenario although they are used to treat enuresis. Relapse
neurologic evaluation. is more common when pharmacologic therapy is discontinued
than with the other modalities. However, medications are e ec-
Question 13.6.3 Further evaluation o this patient should tive as a short-term treatment option but are elt to be second
include all o the ollowing EXCEPT: tier and used when the urine alarm ails or is impractical (think
A) Asking about a history o bowel problems. sleepover or summer camp). O note, “D” is wrong or another
B) Assessment o growth and development. reason: nasal DDAVP is no longer approved or nocturnal
C) Investigation into amily history o nocturnal enuresis. enuresis due to problems with hyponatremia. Oral DDAVP still
D) Spine MRI to rule out pathologic lesion. carries the enuresis indication, but it can cause hyponatremia as
E) Urinalysis. well—it just occurs less o en.

Answer 13.6.3 The correct answer is “D.” Patients with enure-


sis who have an otherwise normal neurologic examination need HELPFUL TIP:
not have an MRI done. I on examination this child had neuro- Re en ion on rol r ining is no longer re ommen e
logic ndings, an MRI would be indicated. All o the remaining s i w s no oun o signi i n ly e re se we ing.
options are part o a thorough evaluation o nocturnal enuresis. Re en ion- on rol r ining w s b se on he ( ul y)
O particular note is the assessment o growth and development premise h enure i hil ren h sm ller bl er -
(is this child neurologically delayed leading to enuresis?). Atten- p i ies. t he hil woul hol in urine in n emp
tion to bowel problems is also important. Fecal impaction can o s re h he bl er n in re se p i y. Re en-
lead to incontinence. Asking about snoring can help to identi y ion on rol r ining n in re se bl er p i y bu
obstructive sleep apnea, which can be associated with enuresis. oes no improve enuresis. Bu m ybe i you re ru k
river or surgeon . . .
You nd no indication o an underlying cause, and you
decide that this is primary enuresis. T e parents are desper-
ate or some sort o intervention to x the problem, since it is Objectives: Did you learn to . . .
becoming a major source o anxiety in the home and o teas- • d e ne prim ry n se on ry enuresis?
ing at school (o course anything can be a source o teasing at • d es ribe some o he epi emiologi h r eris i s o enure-
school). sis?
• Ev lu e p ien wi h enuresis?
Question 13.6.4 Which o the ollowing should constitute • d evelop m n gemen s r egy or enuresis?
the INITIAL treatment o this patient’s primary enuresis?
A) Patient education and motivational training (e.g., rewards
or staying dry). QUICK QUIZ: MEc ONIUM
B) Over-learning.
C) Enuresis alarm. Delayed meconium passage occurs in which o the ollowing
D) Nasal desmopressin (DDAVP). conditions?
E) Oral desipramine. A) Encopresis.
B) Hirschsprung disease.
Answer 13.6.4 The correct answer is “A.” All o the above have C) Cystic brosis.
been ound to be use ul in the treatment o enuresis; however, D) Hyperthyroidism.
the best initial approach includes education. Patients and par- E) B and C.
ents should be in ormed o how common this condition is, how
to reduce uid intake in the evening without getting dehydrated, The correct answer is “E.” Seventy percent o newborns will
and how to schedule voiding. Motivational training plays a role pass meconium within the rst 12 hours o li e. More than
as well. T e other interventions are secondary. “B,” over-learn- 90% o newborns pass meconium within the rst 24 hours. In
ing, is thought to help prevent relapses in patients who have been those with delayed passage o meconium (generally de ned as
success ul with an enuresis alarm. Once continence is achieved > 24 hours o li e), consider Hirschsprung disease, cystic bro-
with the alarm, the child drinks a set amount o uid be ore sis, hypothyroidism, sepsis, intrauterine narcotic exposure, or
bedtime. T e amount is successively increased once dryness imper orate anus.
CHAPTER 13 • PEd Ia t RIc S 377

QUICK QUIZ: Ra SH CASE 13.7


A 7-month-old ully immunized emale presents to the o ce
Your next patient is a 3-year-old emale with a rash. A er play- with ever. Mom reports she has “not been hersel ” and elt
ing outside (could playing out-side be a red herring? T e editors “a bit warm on the orehead.” Vitals reveal a temperature o
think so . . .), her ather ound spots on her legs, and she did not 39.2°C. Physical examination reveals a mildly toxic in ant.
want to walk because her knees hurt. On examination, she is You per orm a complete history and physical but are unable
a ebrile with normal vitals and slightly irritable but otherwise to identi y a source o the ever.
interactive. She has slight nasal drainage, which her ather says
is a residual rom a cold last week. Her legs and buttocks have Question 13.7.1 What is the rst step in your approach to
palpable purpura. Her knees are mildly swollen and are pain ul this child?
with range o motion. Her CBC (including platelets) and coagu- A) Obtain a urinalysis and urine culture.
lation studies are normal, and her urinalysis is signi cant or B) Admit or observation and per orm blood, urine, and CSF
2+ hematuria. or culture.
C) Give an intramuscular dose o ce riaxone.
T e most likely diagnosis in this patient is: D) Order acetaminophen 30 mg/kg and discharge the patient i
A) Acute exposure to lawn chemicals. the temperature comes down.
B) Henoch–Schönlein purpura (HSP) AKA “Immunoglobulin E) Admit or IV uids overnight.
A vasculitis”.
C) Juvenile idiopathic arthritis (JIA). Answer 13.7.1 The correct answer is “A.” At this age, occult
D) Meningococcemia. serious bacterial in ection is less likely than during the neonatal
E) Rocky Mountain spotted ever. period especially when vaccinated against Haemophilus in uen-
zae type b and Streptococcus pneumoniae. In a well-appearing
child older than 90 days with a ever without a ocal source,
The correct answer is “B.” T is is classic HSP, an IgA-
most experts recommend a screening urinalysis and urine cul-
mediated, generally sel -limited, leukocytoclastic vasculitis.
ture as the initial diagnostic test. “B” is overly aggressive, and
T e symptoms and signs o HSP are a rash (typically non-
“C” is not indicated and can make cultures more di cult to
thrombocytopenic purpura), abdominal pain ( rom submu-
interpret should this child become sicker. “D” might be appro-
cosal hemorrhage and edema), arthritis/arthralgia, and renal
priate although the dose o acetaminophen is wrong (it should
disease. ypically, the vasculitis ollows an upper respiratory
be 15 mg/kg/dose by mouth every 4–6 hours). Since you are not
in ection or streptococcal pharyngitis, as in this case. reat-
seeing signs o dehydration, “E” is inappropriate.
ment or HSP is supportive.
“A,” lawn chemical exposure, usually results in nothing acute
but may cause acute cholinergic symptoms i organophosphates HELPFUL TIP:
are used. “C,” JIA, is not likely because the rash o systemic onset S u ies h ve shown h he in i en e o o ul b -
JIA is evanescent salmon-pink rash on the trunk and axilla that eremi is less h n 1% in well- ppe ring fully immu-
classically occurs when the patient spikes a ever. “D,” meningo- nized in n s 3 o 36 mon hs o ge presen ing wi h
coccemia, is incorrect because the patient is a ebrile, alert, and ever wi hou lo lizing signs. O en, posi ive bloo
nontoxic appearing without laboratory evidence o dissemi- ul ure is ue o on min ion, whi h resul s in un-
nated intravascular coagulopathy. “E” is unlikely because Rocky ne ess ry i ion l es ing, re men , n os . Some
Mountain spotted ever presents with headache, high ever, o you m y be hinking, “Wh bou c Bc ?” In his
petechial rash involving the palms and soles, and the patients ype o p ien , he WBc oun n be equ lly misle -
will appear toxic. In addition, patients with Rocky Mountain ing. t he mos impor n ors in e ision-m king
spotted ever have thrombocytopenia when their petechiae re he p ien ’s ge, immuniz ion s us, n lini l
appear. ppe r n e.

Question 13.7.2 What is the most appropriate manner in


HELPFUL TIP: which to obtain a urine specimen in this child?
t he r sh o HSP is requen ly ini i lly ur i ri wi h sur- A) Midstream “clean-catch.”
roun ing e em . t he r sh en s o be symme ri n B) Bag specimen rom sterile plastic bag taped to the perineal
o ur in epen en re s eveloping irs on he legs or region.
on pressure poin s (bu o ks). a is ussion o re men C) Bladder catheterization.
is beyon he s ope o his book. However, ommon in- D) Suprapubic aspiration.
volvemen s in lu e ren l ilure, GI blee ing, n more E) C or D.
r rely c NS involvemen su h s seizures, c NS hemor-
rh ge, e . Answer 13.7.2 The correct answer is “C.” T e gold standard
test or the diagnosis o urinary tract in ection is a quantitative
378 Fa MILY MEd Ic INE EXa MINa t ION & BOa Rd REVIEW

urine culture. While the urinalysis and microscopy may suggest TABLE 13-5 INTERNATIONAL REFLUX STUDY
U I, no component, or combination o components, is as sensi- COMMITTEE GRADING OF VESICOURETERAL REFLUX
tive or speci c as culture. T ere ore, i a child is likely to require
Gr e I: Involves he ure er only.
antimicrobial therapy, one should attempt to obtain urine or
culture in a manner not likely to cause contamination. T e mid- Gr e II: Involves he ure er, pelvis, n ly es wi hou il ion.
stream “clean-catch” may be obtained in older children who Gr e III: Involves he ure er, pelvis, n ly es wi h mil ure er n
pelvis il ion.
are toilet trained; making certain that the perineum is cleansed
rst. Bag specimens are prone to contamination and should be Gr e IV: Involves he ure er, pelvis, n ly es wi h signi n
il ion n blun ing o he ly e l orni es.
avoided in toxic patients who are likely going to need antibiot-
Gr e V: d emons r es il ion n or uosi y o he ure er s well
ics. Suprapubic aspiration is generally not necessary though it is
s loss o he ly e l orni es.
the gold standard.

HELPFUL TIP:
a neg ive b g- olle e urine in non oxi in n m y C) Spiral C o abdomen and pelvis.
be help ul. Gui elines sugges h one op ion in non- D) Urology consultation or urodynamic studies.
toxic in n is o ge b gge urine. I his is posi ive, E) None o the above.
i mus be ollowe up wi h urine ob ine by h-
e eriz ion or supr pubi spir e. t his shoul hen be Answer 13.7.4 The correct answer is “E.” In ebrile in ants
sen or ul ure. a ny hil who will nee n ibio i s on with U I, it is important to look or anatomic abnormalities
rel ively emergen b sis or Ut I shoul h ve urine o the urinary tract. Ultrasound imaging (not VCUG) is rec-
ob ine by he eriz ion or supr pubi spir ion ommended ollowing a rst ebrile U I to identi y those at risk
sen or ul ure. or recurrence and subsequent damage to renal parenchyma
(male or emale). So, “B” would be correct i not or the inclu-
sion o VCUG. Urinary tract ultrasonography should be per-
Question 13.7.3 The patient has 20 WBC/hp on UA. With
ormed at the earliest convenient time. Ultrasound is use ul
out knowing the culture and susceptibility results, which
in the evaluation or kidney size and hydronephrosis, but it
antibiotic would be the LEAST appropriate choice or the
is limited in the evaluation o vesicoureteral re ux (VUR), a
treatment o a UTI in this child?
common abnormality associated with recurrent U I. VUR is
A) rimethoprim/sul amethoxazole ( MP/SMX).
evaluated with a VCUG or radionuclide cystogram (RNC).
B) Amoxicillin.
VCUG should be done a er a rst ebrile U I only when
C) Ce dinir.
there is evidence o renal abnormality on the ultrasound
D) Ce xime.
including scaring, hydronephrosis, abnormal anatomy, etc.
Routine VCUG is NO recommended. T us, choice “B” is
Answer 13.7.3 The correct answer is “B.” T e majority o uri-
incorrect.
nary tract in ections are due to Gram-negative organisms such
as Escherichia coli in children and the resistance rate to amoxi-
For some unknown reason, a VCUG is done (you said “Is it
cillin is high (up to 40%). T us, amoxicillin is not a good choice
time or tea?” and it got transcribed as “do a VCUG”) and
or treating a U I in children—or anybody, or that matter. T e
your patient is ound to have grade II ref ux (see able 13-5).
usual choice or empiric oral antibiotic treatment o U I in chil-
dren includes sul onamides or third-generation cephalosporins.
Caution should be used with empiric use o MP/SMX due to Question 13.7.5 Which o the ollowing would be the most
increasing resistance o E. coli to this agent. Fluoroquinolones appropriate initial management o her grade II ref ux?
may be considered in adolescents or children older than 1 year A) Prescribe prophylactic antibiotics.
with a complicated U I; uoroquinolones do seem to be sa e in B) Re er or surgical intervention.
children but should be saved or situations where other drugs are C) Per orm GFR to assess renal scarring.
sub-optimal. T e duration o treatment should be 7 to 10 days D) Observe or now with repeat VCUG in 6 months.
or uncomplicated U I. I the child appears clinically ill sug- E) None o the above.
gesting pyelonephritis, initial parenteral treatment with ce ri-
axone would be an appropriate rst-line agent. Answer 13.7.5 The correct answer is “E.” Since the last edi-
tion o this book things have just gotten more muddled. It
Her urine culture reveals Escherichia coli susceptible to trim- seems (most) everyone agrees that children with grade I–II
ethoprim/sul amethoxazole, and you treat her appropriately. re ux with bladder and bowel dys unction bene t rom pro-
phylactic therapy. In those with grade I-II re ux and no blad-
Question 13.7.4 Following her initial treatment, what diag der or bowel dys unction the bene t is less clear. It gets a bit
nostic study should be per ormed next? trickier with grade III–IV. Overall, children with III-IV re ux
A) Abdominal supine and upright radiographs. seem to have about hal the number o in ections with prophy-
B) Renal ultrasound and voiding cystourethrogram (VCUG). lactic therapy (typically with MP/SMX); the bene t is most
CHAPTER 13 • PEd Ia t RIc S 379

pronounced in those with a ebrile rst episode and bowel or Answer 13.8.1 The correct answer is “A.” Maternal chorioam-
bladder dys unction. However, there is no di erence in renal nionitis is an important risk actor or early onset sepsis in the
scarring and 63% develop resistant organisms (vs. 19% in the newborn. T e 2010 CDC guideline on the prevention o peri-
placebo group) (N Eng J Med. 2014;370;2367–2376 and N Eng J natal group B streptococcal disease recommends that all in ants
Med. 2014;370:2440–2441). Have a discussion with the parents. born to mothers with chorioamnionitis have screening labs and
And promptly treat any U Is to prevent renal scarring (par- blood cultures at birth ollowed by minimum o 48 hours o
ticularly a risk in those with a temperature o > 39°C or a non antibiotic therapy. Antibiotic therapy may be extended and/or
E. coli organism). Mild re ux tends to resolve with maturity. modi ed depending on the clinical course and culture results.
Renal scarring is best detected by radioisotope scanning with A lumbar puncture or CSF culture and analysis should be per-
99 m-technetium dimer captosuccinate. Doing a GFR looks at ormed i the child is septic appearing. Ampicillin and gentami-
kidney unction and does not assess or renal scarring. T ere- cin as empiric therapy cover the most likely organisms includ-
ore, “C” is incorrect. “D” is incorrect because a VCUG was not ing GBS and Gram-negative pathogens, respectively.
indicated in the rst place!
Anthony is started on appropriate antibiotics. Examination
o this neonate reveals hepatosplenomegaly, lymphadenop-
HELPFUL (OR CONFUSING) TIP:
athy, petechiae, and white mucocutaneous patches in the
We on’ know wh he bo r ex min ion will w n
in ant’s mouth. His CBC reveals hemoglobin o 12 g/dL and
s n nswer. t r i ion lly, ul r soun a nd Vc UG h ve
a platelet count o 85,000/mm3 with evidence o hemolysis on
been one er irs Ut I in m le in n n se on
the peripheral smear.
Ut I in em le in n . Even hough he a a P gui elines
o no suppor oing Vc UG, he Se ion on Urology
Question 13.8.2 The most likely pathogen in ecting this
o he a meri n a emy o Pe i ri s publishe
in ant is:
s emen in a pril 2012 is greeing wi h he re om-
A) Rubella.
men ion o no ob in Vc UG in hil ren ge
B) Cytomegalovirus.
2 mon hs o 2 ye rs er he irs ebrile Ut I. Surgi l
C) oxoplasma gondii.
in erven ion is gener lly reserve or hose wi h severe
D) reponema pallidum.
re lux (gr es IV–V) or re urren Ut I espi e prophyl -
E) Candida albicans.
i n ibio i s. See Pediatrics. 2011;128(3):595–610 or
more in orm ion.
Answer 13.8.2 The correct answer is “D.” Neonatal mani esta-
tions o congenital syphilis include hepatosplenomegaly, lymph-
Objectives: Did you learn to . . . adenopathy, jaundice, rash, hemolytic anemia, and thrombocy-
• Ev lu e n ol er (> 3 mon hs ol ) in n wi h ever? topenia. Un ortunately, these ndings overlap considerably with
• d i gnose n re hil wi h n ini i l urin ry r in e - many o the other congenital ORCH (toxoplasmosis, rubella,
ion? CMV, herpes) in ections. Abnormalities more speci c to congeni-
• Re ognize h urin ry r in e ion is o en m rker or tal syphilis include white, patchy mucocutaneous lesions (think
urin ry r bnorm li ies in hil ren? “condylomata lata”), edema, rhinitis (snuf es), osteochondritis,
• M n ge p ien wi h VUR? and pseudoparalysis. Congenital syphilis is caused by trans-
placental transmission o the spirochete . pallidum (“D”).
CASE 13.8 Intrauterine in ection can result in stillbirth, hydrops etalis, or
prematurity.
Anthony is a 2,300-g male in ant born at 35 weeks’ gestation
to a 23-year-old G2P2 mother. T e mother did not seek pre- HELPFUL TIP:
natal care and routine screening labs were not available. She L e sequel e o ongeni l syphilis involve he bones
presented to the ED in active labor shortly ollowing spon- n join s, ee h, eyes, n c NS n in lu e bowe shins
taneous rupture o membranes at home. Upon trans er to (S ber shins), ron l bossing, s le nose, pegge en-
the obstetric ward, she was noted to be ebrile (38.8°C) and r l in isors, in ers i i l ker i is, n sensorineur l e -
received intravenous antibiotics. Five hours later, the in ant ness. t he Hu hinson ri in lu es in ers i i l ker i is,
was delivered via spontaneous vaginal delivery. e ness, n no he , peg-sh pe ee h.

Question 13.8.1 Management o this newborn in ant whose


mother probably has chorioamnionitis (not simply GBS In addition to the laboratory data obtained above, you order
screen positive) should include which o the ollowing? an ultrasound o the patient’s head, which appears normal.
A) Empiric therapy with ampicillin and gentamicin.
B) Empiric therapy with ampicillin. Question 13.8.3 Intracerebral calci cations are CLASSI
C) Empiric therapy with vancomycin. CALLY associated with which congenital in ection(s)?
D) Empiric therapy with ce riaxone A) Cytomegalovirus.
E) Empiric therapy with ampicillin and erythromycin. B) oxoplasmosis.
380 Fa MILY MEd Ic INE EXa MINa t ION & BOa Rd REVIEW

C) . pallidum. and osteitis may be present. A characteristic “blueberry mu n”


D) Rubella. appearance may be present due to the combination o jaundice
E) A and B. and extramedullary (skin) hematopoiesis. Syphilis and GBS
in ections are described above and they do not typically cause
Answer 13.8.3 The correct answer is “E.” T e clinical mani- the ophthalmologic ndings described in this case. “C” is incor-
estations o congenital cytomegalovirus and toxoplasmosis are rect because congenital HIV in ection is commonly asymptom-
o en similar. In ants are typically asymptomatic at birth, but atic. Evidence o immune de ciency may present later in the
a signi cant number ultimately develop visual impairment, child’s li e with ailure to thrive, generalized lymphadenopathy,
learning disabilities, and developmental and intellectual delays hepatosplenomegaly, and recurrent in ections. “D” is incor-
months to years later. T ose in ants who are symptomatic at rect because congenital herpes in ections do not cause cardiac
birth may demonstrate intrauterine growth retardation, hepato- abnormalities.
splenomegaly, jaundice, hemolytic anemia, and thrombocyto-
penia. Intracerebral calci cations also occur in both CMV and
HELPFUL TIP:
toxoplasmosis. T e calci cations tend to be periventricular in
USPSt F re ommen s univers l he ring s reening or
CMV (remember this by the act that the calci cations look like
newborns.
a “C” in CMV) and more dispersed throughout the cortex in
toxoplasmosis (remember by noting the “X” in both cortex and
toxoplasmosis). Additional central nervous system abnormali-
ties include microcephaly, chorioretinitis, and sensorineural QUICK QUIZ: c ONGENIt a L RUBELLa
hearing loss.
Objectives: Did you learn to . . . Which o the ollowing cardiac lesions is likely to be ound in an
• a ppropri ely m n ge he neon e born o mo her who in ant with congenital rubella?
h s evi en e o horio mnioni is? A) Patent ductus arteriosus (PDA).
• Re ognize h m ny ongeni l t ORc H in e ions m y h ve B) etralogy o Fallot.
overl pping signs n symp oms? C) ransposition o great vessels.
• Re ognize h m ny in n s wi h ongeni l t ORc H in e - D) Coarctation o aorta.
ions re symp om i bir h? E) Bicuspid aortic valve.

The correct answer is “A.” T e most common cardiac lesions


QUICK QUIZ: c ONGENIt a L INFEc t IONS associated with congenital rubella are a PDA and peripheral pul-
monary artery stenosis. PDA is characterized by a continuous
A small or gestational age newborn in ant is ound to have “machine-like” murmur heard best over the le upper sternal
hepatosplenomegaly, jaundice, and thrombocytopenia. Cardiac border. etralogy o Fallot and transposition o great vessels are clas-
examination reveals a grade II/VI continuous murmur heard sically cyanotic heart de ects that have no known association with
best at the le upper sternal border. An ophthalmologic exami- congenital in ections. Coarctation o the aorta and bicuspid aortic
nation demonstrates microphthalmia and cataracts. valve both cause harsh systolic murmurs. T e murmur o coarc-
tation radiates to the back. Both coarctation and bicuspid aortic
T e result o which maternal prenatal screening lab is likely to valve are commonly ound in emales with urner syndrome.
have been concerning in the above case?
A) VDRL.
B) Rubella immunity status.
CASE 13.9
C) HIV status. A term newborn in ant is noted to have a solitary, tense bulla
D) Varicella zoster immunity status. located on the dorsum o his wrist. T e underlying skin is
E) GBS cultures. nonerythematous. Pregnancy was uncomplicated; however,
the in ant was delivered via cesarean section or ailure to
The correct answer is “B.” Cardiac, ophthalmologic, auditory, progress. T e in ant appears to be a vigorous eeder and is
and neurologic ndings predominate in the symptomatic in ant noted to requently suck on his hands and wrists while in the
with congenital rubella. Up to 85% o in ants in ected during nursery. He is otherwise asymptomatic.
the rst 12 weeks o gestation will have some orm o congenital
de ect. T is decreases to 5% when the primary in ection occurs Question 13.9.1 Which o the ollowing is MOST appropri
a er the third to ourth month o gestation. Ophthalmologic ate at this time?
ndings include microphthalmia, cataracts, glaucoma, and salt A) Start acyclovir.
and pepper retinopathy. In ants are o en microcephalic and B) Per orm lumbar puncture and send a sample o the CSF or
develop sensorineural hearing de cits, meningoencephalitis, HSV PCR.
and developmental delay. Dea ness, cataracts and cardiac C) Per orm a zanck smear o the bulla uid.
anomalies are the classic ndings with congenital rubella. D) Obtain CBC and blood cultures.
Additional ndings o hepatosplenomegaly, thrombocytopenia, E) Observation with no intervention at this time.
CHAPTER 13 • PEd Ia t RIc S 381

Answer 13.9.1 The correct answer is “E.” Nothing needs to


HELPFUL TIP:
be done at this time. T e appearance and location o the bulla
In n s born v gin lly or vi es re n elivery o moms
is consistent with a sucking blister. A solitary bulla or blister on
wi h ive geni l HSV lesions nee HSV ul ures o he
normal-appearing skin located on the upper limbs may be pres-
mou h, n soph rynx, onjun iv , n re um ob ine
ent at birth rom in utero sucking. O en when presented with
12 o 24 hours o ge. In in n s born o moms wi h
the a ected extremity, the in ant will demonstrate the sucking
primary ou bre k ime o elivery, some exper s
behavior in that location. Such blisters are o en mistaken or her-
re ommen empiri y lovir her py er ob ining
pes simplex, but the solitary nature and location help to establish
he ppropri e ul ures. In n s o mo hers wi h re-
the correct diagnosis—although there may be multiple blisters
urren ou bre k o no require empiri y lovir re -
at times. Additional history o cesarean section makes HSV less
men bu s ill shoul be moni ore losely or signs o
likely in this scenario. Nonetheless, a sucking blister is a diagnosis
in e ion.
o exclusion, and other more serious diagnoses should be ruled
out by history and examination and urther testing as indicated.

Question 13.9.2 The FALSE statement regarding neonatal Objectives: Did you learn to . . .
herpes simplex virus in ection is: • d i eren i e be ween herpes lesion n su king blis er?
A) T e majority o in ants with neonatal HSV are born to • Re ognize r es n mo es o r nsmission o herpes in
mothers with symptomatic herpetic lesions. neon es?
B) T e incidence o neonatal transmission is higher when • I en i y some o he lini l m ni es ions o neon l
the pregnant woman experiences primary in ection versus herpes?
secondary reactivation.
C) Most neonatal HSV in ections are caused by HSV 2. CASE 13.10
D) Intrauterine exposure accounts or the minority o perinatal
HSV in ection. A bunch o parents in your neighborhood decide to have
E) Herpetic skin lesions are o en present in in ants with CNS a “chickenpox” party (prompting you to consider mov-
or disseminated HSV. ing to a place where people believe in modern medicine).
hey want to synchronize their kids’ chickenpox outbreaks
with their busy schedules (heaven orbid we should get
Answer 13.9.2 The correct answer is “A.” Less than one-third immunized—a ter all, it isn’t natural!). One o the moth-
o in ants with neonatal HSV are born to mothers with active ers, who has never had chickenpox hersel , inds out that
genital lesions. In act, most cases o neonatal HSV are ound she is pregnant the day a ter being exposed to chickenpox.
in in ants o asymptomatic mothers who are shedding the virus. She is not yet symptomatic ( rom the pregnancy or the
Neonatal herpes simplex virus o en results in serious morbidity chickenpox).
and mortality. In ants with HSV present with one o the ollow-
ing three disease patterns with overlapping eatures: localized Question 13.10.1 The best advice or this potential mother
disease involving the skin, eyes, and mouth; CNS disease; or about her pregnancy is:
disseminated in ection. Encephalitis may occur with or without A) Consider termination since it is likely that the child will have
skin involvement; there ore, HSV in ection should be suspected congenital varicella.
in any in ant rom birth to approximately 4 weeks o age that B) Get treated within 96 hours o exposure with varicella
presents with ever, seizure, or mental status abnormalities such immune globulin.
as lethargy, irritability, or poor eeding. C) Get treated within 96 hours o exposure with varicella
T e other options are correct. A woman who contracts a immune globulin plus varicella vaccine.
primary in ection during the pregnancy has a greater than 30% D) Get treated within 96 hours o exposure with IVIG plus vari-
chance o transmitting the virus to her in ant (“B”). T ere is cella vaccine.
less than 2% transmission i she experiences a secondary reac-
tivation intrapartum. T e majority o neonatal HSV is due to Answer 13.10.1 The correct answer is “B.” Women who are
in ection with HSV 2 (“C”). Only 5% o congenital in ections varicella-susceptible (nonimmune), pregnant, and exposed to
occur in utero (“D”), as the majority occurs via exposure to chickenpox should be treated with varicellaimmune globulin
in ected cervical secretions during birth. Skin lesions (“E”) are (VariZIG) within 96 hours o exposure. I VariZIG is unavailable,
present in 60% o CNS cases and about 75% o disseminated IVIG may be given instead. T e vaccine should be avoided
cases. during pregnancy (thus “C” and “D” are incorrect). It is a
live, attenuated, virus and still carries some risk to the etus
HELPFUL TIP:
(although the degree o risk is unknown). It is sa e to give the
Mos neon es wi h HSV in e ion o no presen wi h
vaccine to mom in the postpartum period and is indicated i
skin in ings bir h. I is more ypi l or neon es o
the mother is screened to be varicella nonimmune during her
evelop HSV skin lesions 5 o 14 ys er elivery.
pregnancy. T e risk o congenital varicella syndrome i the etus
is exposed in the rst trimester is about 2%. T e risk o neonatal
382 Fa MILY MEd Ic INE EXa MINa t ION & BOa Rd REVIEW

in ection is much higher i the mother is in ected within 5 days


o birth, and 30% o these cases result in severe neonatal vari- CASE 13.11
cella in ection. A 3-year-old child presents to your o ce with a history o
bright red cheeks. Over the past several days, she has had
Question 13.10.2 You can let the mother know that: mild ever with mild muscle aches. Except or the rash, the
A) Varicella immune globulin has been shown to reduce the child is now asymptomatic.
risk o congenital varicella.
B) Varicella immune globulin is aimed mostly at attenuating Question 13.11.1 The most likely cause o this patient’s
the case in the mother should she be in ected. illness is:
C) Varicella immune globulin has not been shown to prevent A) Parvovirus B-19.
congenital varicella. B) Herpes virus 6.
D) A and B. C) Rubeola.
E) B and C. D) Rubella.
E) In uenza virus.
Answer 13.10.2 The correct answer is “B.” Varicella immune
globulin is aimed at reducing the symptoms in the mother and Answer 13.11.1 The correct answer is “A.” A mild systemic
attenuating her case o varicella. Both “A” and “C” are alse. illness ollowed by red, “slapped cheeks” is typical o erythema
T ere are no good data either way about how varicella immune in ectiosum or “ h disease,” caused by parvovirus B-19. T e
globulin will a ect the child’s outcome vis-à-vis congenital red, slapped cheeks are generally ollowed by a lacy, reticu-
varicella. lar rash on the extremities and trunk. T is is generally a sel -
limited illness in children.
Question 13.10.3 Mani estations o intrauterine in ection
with varicella zoster include all o the ollowing de ects Question 13.11.2 Which o the ollowing is NOT a compli
EXCEPT: cation o parvovirus B 19 in ection—either congenital or
A) Optic atrophy. acquired?
B) Congenital cataracts. A) Aplastic anemia.
C) Cardiac abnormalities. B) Birth de ects, including CNS and limb disease.
D) CNS abnormalities. C) Hydrops etalis.
E) Limb hypoplasia. D) In ammatory arthritis.
E) Intrauterine etal death.
Answer 13.10.3 The correct answer is “C.” Cardiac abnor-
malities are not seen with varicella zoster exposure in utero. In Answer 13.11.2 The correct answer is “B.” Parvovirus B-19
addition to the ndings above, patients with congenital varicella causes all o the above except or birth de ects. Fetal death,
zoster will have cutaneous scarring called cicatrix. anemia, or nonimmune-mediated hydrops etalis are poten-
tial outcomes o intrauterine exposure (one-third o in ected
Glory!! She ound a chickenpox party! Now that all o women will pass the virus to their etus). Most in ants in ected
these children have been in ected with varicella zoster, the in utero are born normally at term, including those with evi-
question arises about how long they should be kept out o dence o hydrops earlier in the pregnancy. A small subset will
school. acquire chronic or prolonged postnatal in ection o unknown
signi cance. “A,” transient aplastic crisis, can occur in those
Question 13.10.4 These children are considered in ectious with chronic hemolytic conditions such as sickle cell anemia.
until: “D,” in ammatory arthritis, is common in adults and adoles-
A) 5 days a er the rst lesion appears. cents with emales more commonly a ected.
B) 10 to 14 days a er the rst lesion appears.
C) New lesions are no longer orming. T e question o whether or not to exclude this child rom
D) All lesions are crusted over. preschool is raised. T e patient still has the rash. T ere are a
E) T ey are no longer ebrile. couple o pregnant teachers at the preschool.

Answer 13.10.4 The correct answer is “D.” Patients are consid- Question 13.11.3 You can tell the child’s mother that:
ered in ectious until all lesions are crusted over. Lesions need A) T is child is in ectious and should be excluded rom pre-
not have healed entirely, only crusted over. school until the rash resolves.
B) Since almost all adult women are already immune, there is
Objectives: Did you learn to . . . no need to worry about transmission to the teachers.
• O er re men or v ri ell exposure o pregn n nonim- C) Since this virus is an enterovirus, care ul hygiene in the
mune em le? school will prevent spread.
• Re ognize he uses o v ri ell immune globulin? D) T is child can be allowed back into preschool and is no
• I en i y symp oms o ongeni l v ri ell in e ion? longer in ectious.
CHAPTER 13 • PEd Ia t RIc S 383

Answer 13.11.3 The correct answer is “D.” Once the rash is she has a viral gastroenteritis, and give the parents some advice
present, this child is no longer in ectious and can be allowed regarding rehydration.
back into school. “B” is incorrect because only about 50% o
adult women are immune. “C” is incorrect. First, this is not Which o the ollowing is the most appropriate advice or this
an enterovirus. Even though URI symptoms are o en absent, patient?
transmission is mostly respiratory with vertical transmission A) Avoid all solid ood or the next 24 to 48 hours.
o mom to etus, and hematogenous spread (via transmissions) B) Use milk as the primary rehydration solution.
being much less common. C) Use only watered-down tea or rehydration.
D) Use a commercially prepared oral rehydration solution and
reintroduce oods as tolerated.
HELPFUL TIP: E) Use cola beverages (e.g., Coca-Cola or Pepsi) because they
Jus ry ge ing his hil wi h r sh b k in s hool contain more sodium than commercially available oral rehy-
n p s he s hool nurse (we re you); i is impos- dration solutions.
sible . . . On he o her h n , s hools re sure h here
is “m gi l e e ” o opi l n ibio i s in preven ing The correct answer is “D.” E ective oral rehydration can be
he spre o vir l onjun ivi is. accomplished with any one o the rehydration solutions on the
market (e.g., Pedialyte). “A” is incorrect. T ere is no need to limit
the ood intake o patients with vomiting or in ectious diarrhea.
Objectives: Did you learn to . . . T ey can have whatever they can tolerate without vomiting. T e
• Re ognize he lini l m ni es ions o p rvovirus B-19 concept o “gut rest” is obsolete and leads to increased bowel
in e ion (“ h ise se”) permeability and prolonged diarrhea. So, early eeding is opti-
• I en i y ompli ions o p rvovirus B-19 in e ion? mal and recommended. T ere is no need to avoid lactose-
containing products. ransient lactase de ciency a er gastro-
enteritis is usually sel -limited (and generally limited to those
QUICK QUIZ: c HILd HOOd INFEc t IONS < 2 months o age) and does not require treatment unless symp-
toms are prolonged or severe a er dietary re-introduction. “C”
Which o the ollowing is characterized by a high ever, pos- and “E” are incorrect. T ere is no role or weak tea, at soda,
sibly over 40°C, a bulging ontanelle, a maculopapular rash that “Jello water,” sports drinks, juice, etc., in the management o
begins af er the ever abates, conjunctivitis, and upper respira- gastroenteritis. T e most common cause o hyponatremic sei-
tory symptoms? zures in the child is improper rehydration during gastroenteri-
A) Roseola in antum (erythema subitum). tis. Many commercial drinks, such as cola beverages and sports
B) Rubeola (measles). drinks, are not speci cally designed or oral rehydration, and
C) Rubella (German measles). the sodium concentration is typically much lower than that in
D) Varicella (chicken pox). rehydration solutions, risking hyponatremia (in other words,
E) Meningococcal meningitis. the reverse o “E” is true). In addition, many o these have a high
sugar content, which can worsen diarrhea due to osmotic e ects.
The correct answer is “A.” Such a clinical presentation is typical
o roseola in antum. Roseola in antum, which is caused by the HELPFUL TIP:
human herpes 6 (and rarely herpes 7) virus, is characterized by In he ex lusively bre s e in n wi h vomi ing n /or
a high ever or 3 to 5 days (o en over 40°C), generalized mal- i rrhe , bre s ee ing shoul be en our ge n no
aise, a bulging ontanelle in up to 26% o in ants, conjunctivitis, repl e wi h or l rehy r ion solu ions.
perhaps oral mucosal ulcers, and a rash that appears as the ever
begins to de ervesce. Children usually look amazingly well or HELPFUL TIP:
the height o the ever. d uring GI illness, vise p ren s o voi non bsorb-
ble sug rs, like hose oun in pple or gr pe jui e.
QUICK QUIZ: Ga St ROINt ESt INa L SYMPt OMS t hese n promo e n osmo i i rrhe .

A young couple brings in their 18-month-old emale or a


12-hour history o vomiting and diarrhea. T e patient has elt
CASE 13.12
warm, and her oral intake has been depressed. However, she A new mother and ather bring their 2-month-old in ant
continues to make tears with crying and has wet diapers. Her to your o ce with a complaint o inconsolable crying. T is
medical history is unremarkable. On examination, you nd a started at about 3 weeks o age and occurs about the same
tired-looking emale who perks up slightly when you open time every day. T e crying will last or hours and is becom-
a bag o toys (who are you anyway . . . Santa?). Her tempera- ing quite disruptive. T e child will draw up his knees and
ture is 38.5°C, and her capillary re ll is less than 3 seconds. seem to be in quite a bit o pain. T ey have tried pretty much
T e remainder o the examination is non ocal. You think that everything that they can think up including car rides, swings,
384 Fa MILY MEd Ic INE EXa MINa t ION & BOa Rd REVIEW

swaddling, various types o soothing music (such as death


HELPFUL TIP:
metal), etc., but to no avail. Shivers run up and down your
t he use o oli is unknown n likely mul i ori l.
spine as you recall your own early parenting experiences . . . or
R rely me i l expl n ion is oun . It is important to
as you remember why you don’t have kids (please apply the
identify alternative caregivers, since infant colic can
appropriate phrase or your personal experience).
be a major stress on parents and can possibly lead to
abuse (particularly shaken baby syndrome). P ren
Question 13.12.1 The most likely diagnosis is:
e u ion is ri i l.
A) Colic.
B) Intussusception.
C) Hair around the penis or toes or corneal abrasion.
HELPFUL TIP:
D) Constipation.
t here is evi en e o suppor ri l o n ex ensively
E) Cluster headache.
hy rolyze in n ormul (e.g., Nu r migen). d isn’
he s ronges , hough. Use o p r i lly hy rolyze or
Answer 13.12.1 The correct answer is “A.” Colic generally
l ose- ree ormul s is o no bene i . c h nging o soy
begins at 3 weeks o age, peaks at about 6 weeks o age, and
ormul oes no work.
abates by 3 months o age. It is de ned by a rule o 3s: intense
crying or 3 hours per day, or 3 days per week, or 3 weeks.
However, most parents, grandparents, nosy aunts, and pediatri-
HELPFUL TIP:
cians will describe any particularly ussy baby as “colicky.” T e
d sugges h probio i s re NOT e e ive in in n
cause is unknown. “B” is incorrect. While intussusception can
oli (BMJ. 2014;348:g2107). However, hey re s e
certainly present with colicky abdominal pain, it is not likely
n give he p ren s some hing o o. In bre s ee -
to be recurrent or several days or weeks without a more sig-
ing women, elimin ion o ommon llergens (milk,
ni cant problem (e.g., bloody stools) developing. “C,” a hair or
eggs, nu s, whe ) n be rie . a g in, he isn’ he
thread around a child’s toes or penis or corneal abrasion, should
gre es . Evi en e or spin l m nipul ion in re ing
be considered in any child with inconsolable crying. Again, this
oli is in on lusive. a s wi h mos hiropr i s u -
is not likely to result in crying that is daily and episodic. “D” is
ies, he is poor. In rossover s u y, 24% su rose
incorrect as this history is not consistent with constipation. And
h s been shown o be bene i i l (t oo swee n o her
“E,” a cluster headache, does not occur in this age group.
br n s . . . or m ke i yoursel ).

HELPFUL TIP:
While in onsol ble rying h s o en been ribu e o Objectives: Did you learn to . . .
orne l br sion, re en lls his in o ques ion. • Ev lu e p ien wi h oli n o er m n gemen s r e-
Jus s m ny hil ren who were NOt rying h orne l gies o he p ren s?
br sions.

QUICK QUIZ: EPIGLOt t It IS 1


Question 13.12.2 You can advise the parents that:
A) Phenobarbital is sa e and e ective or controlling in ant colic You have a child in the ED who is toxic appearing with the sud-
(when given to the parents to help them “chill out” . . . and den onset o high ever, drooling, and stridor. You suspect epi-
knock o the death metal). glottitis.
B) Elimination o cruci erous vegetables rom the diet o breast-
eeding mothers has been shown to reduce in ant colic. What is the FIRS step in the treatment o this patient?
C) Simethicone has been shown to be e ective in in ant colic. A) Draw blood work including a CBC.
D) Anticholinergic drugs are e ective in treating in ant colic, B) Place an IV or access and uids.
but the risks are unacceptable. C) Give a dose o IM steroids (e.g., dexamethasone 0.6 mg/kg)
E) Children who breast eed are less likely to develop colic. to help shrink the epiglottis.
D) Put a ace mask on the child and administer albuterol.
Answer 13.12.2 The correct answer is “D.” Anticholinergic E) Leave the child on the mothers lap and don’t upset the child
drugs have been shown to be e ective in the treatment o colic. (you don’t want a temper tantrum in the ED . . .).
However, the risks associated with the use o these drugs are
generally considered unacceptable. T e rest o the answers are The correct answer is “E.” Anything that upsets the child can
incorrect. It does not seem as though breast ed in ants are any lead to increased airway obstruction. Leave the child in the par-
less likely to develop colic than are bottle- ed in ants. T e evi- ent’s lap and do not upset him. “A” and “B” are incorrect because
dence or elimination o cruci erous vegetables is inconclusive. they may upset the child, leading to increased work o breathing
Simethicone is NO e ective in treating in antile colic, but it and increased obstruction. “C,” dexamethasone, is indicated or
gives the parents something to do. croup and not or epiglottitis. “D” is also incorrect. However, it
CHAPTER 13 • PEd Ia t RIc S 385

would not be wrong to give blow-by oxygen or blow-by nebu-


lized epinephrine i it did not agitate the child. CASE 13.13
A 12-month-old presents to your o ce in January with a
2-day history o runny nose and cough. T e child now has
QUICK QUIZ: EPIGLOt t It IS 2 wheezing with nasal f aring and retractions, and his oxygen
saturation is 89%. T ere has been no ever, and several o the
Which o the ollowing IS NO needed in the optimal diagnosis other children and adults in the amily have had “a cold” over
and treatment o epiglottitis? the past several weeks. On examination, you hear scattered
A) “T umb sign” on radiograph. wheezes and rales. You diagnose bronchiolitis.
B) Antibiotics to cover H. in uenzae.
C) Antibiotics to cover S. pneumoniae. Question 13.13.1 The MOST LIKELY organism involved in
D) An operating room. this child’s illness is:
E) Personnel able to emergently manage the airway. A) Rhinovirus.
B) Adenovirus.
The correct answer is “A.” T e use o radiographs in suspected C) Respiratory syncytial virus (RSV).
epiglottitis is raught with problems and delays potentially li e- D) Parain uenza.
saving therapy. DON’ DO I . Epiglottitis is a clinical diagno- E) Human metapneumovirus.
sis that requires visualization o the epiglottis. T is is pre erably
done in the operating room with a setup or both intubation Answer 13.13.1 The correct answer is “C.” T e most common
and tracheostomy should that become necessary. I the child cause o bronchiolitis in children is RSV. All o the others can
is mildly ill, looking at the epiglottis in the ED is permissible. also cause bronchiolitis, as can in uenza virus, but less com-
However, be prepared to manage the airway. “B” and “C” are monly than RSV.
both correct. Since the advent o H. in uenzae vaccine, there
is no longer a single organism causing most cases o epiglot- Question 13.13.2 Which o the ollowing treatments has
titis. Bacteria that can cause epiglottitis include H. in uenzae, S. been unequivocally shown to be e ective in bronchiolitis?
pneumoniae, S. aureus, and groups A, B, and C beta-hemolytic A) Nebulized albuterol.
streptococci. Epiglottitis may also be o viral origin. B) Nebulized epinephrine.
C) Corticosteroids such as prednisone or dexamethasone.
D) All o the above.
QUICK QUIZ: NEONat a L INFEc t IONS E) None o the above.

What is the most likely etiologic agent in this child: 2-week-old Answer 13.13.2 The correct answer is “E.” None o the above
male with conjunctivitis, cough, rales, nasal congestion, in l- has been shown to be e ective in bronchiolitis. T e mainstay
trate on radiograph, but no ever or wheezes? o treatment is supportive care. Albuterol may cause worsening
A) In uenza. hypoxia due to induction o V/Q mismatch.
B) Chlamydia.
C) RSV.
D) Parain uenza. HELPFUL TIP:
E) Gonorrhea. Nebulize hyper oni s line (3%) h s been shown o be
mildly bene i i l in hospitalized p ien s wi h bron hiol-
The correct answer is “B.” T is is a classic description o i is by e re sing lini l bron hioli is s ore n shor -
Chlamydia in ection in the newborn. Patients usually present ening leng h o s y. t his m kes sense. t he hyper oni
between 5 and 14 days with exudative conjunctivitis and pneu- s line will r w lui rom he bron hi n shrink issues
monia. T e gold standard or diagnosis o neonatal Chlamydia hereby re u ing obs ru ion.
in ection remains a culture obtained rom conjunctival swabs o
the everted eyelids; however, nucleic acid ampli cation tests o
conjunctival scrapings have been ound to have high sensitiv- Question 13.13.3 RSV is usually diagnosed by:
ity and speci city when compared with culture. Conjunctival A) Nasal wash using immuno uorescence or antigen detection.
cells must be present or an adequate specimen because Chla- B) Baseline and convalescent antibody titers.
mydia is an intracellular pathogen. reatment is with systemic C) Blood culture or RSV.
antibiotics even i conjunctivitis is the only symptom present. D) Sputum gram stain.
Erythromycin is the drug o choice at a dose o 50 mg/kg/day E) Induced sputum culture.
divided our times daily or 14 days. Azithromycin is an alter-
native. opical treatment may be used as an adjunct but alone Answer 13.13.3 The correct answer is “A.” A nasal wash or
is not su cient or treatment o this patient. Remember, the RSV immuno uorescence or antigen detection is used to diag-
antibiotic eye ointment applied to the neonate at birth protects nose RSV. I a nasal wash cannot be obtained, a nasal pharyngeal
against ocular gonococcal infection but not Chlamydia. swab or throat swab o ers the next best alternatives. Reverse
386 Fa MILY MEd Ic INE EXa MINa t ION & BOa Rd REVIEW

transcription-polymerase chain reaction (R -PCR) assays may Answer 13.14.1 The correct answer is “C.” T is most likely
also be used. T e other options are incorrect. represents idiopathic hypertrophic pyloric stenosis. T e classic
presentation o pyloric stenosis is nonbilious vomiting immedi-
HELPFUL TIP: ately a er eeding, ollowed by a demand to eed again soon a er
Bron hioli is is lini l i gnosis h oes no require (much the same as a teenager . . . the demand o eeding, not the
ny i gnos i es ing. vomiting). T e symptoms are progressive. Initially, the vomit-
ing may or may not be projectile and may occur a er every eed
or intermittently. Classically, pyloric stenosis occurs in in ants
HELPFUL TIP: 2 weeks to 2 months o age. “A,” midgut volvulus, is unlikely
Never give rib virin or he re men o RSV. I oesn’ because these patients have bilious vomiting. Our patient has
work. nonbilious vomiting. “B,” gastroenteritis, is not likely because
there is no diarrhea, ever, or appetite loss. “D” is unlikely. While
CNS injury with increased intracranial pressure and vomiting
Question 13.13.4 Which o the ollowing is indicated or the is a possibility, you would expect to see other evidence o CNS
prevention o RSV in high risk in ants? injury such as lethargy, possibly signs o head injury, etc. Note
A) Vaccination against RSV. that this patient’s weight is unchanged rom 2 weeks ago . . . . . it
B) Oral ribavirin. should be higher than 2 weeks ago.
C) Rimantadine or oseltamivir.
D) Palivizumab (Synagis®) Question 13.14.2 Which o the ollowing patients is most
E) RSV–IVIG (RespiGam) likely to present with pyloric stenosis?
A) A rst born male.
Answer 13.13.4 The correct answer is “D.” Palivizumab is a B) A rst born emale.
licensed prophylactic monoclonal antibody against RSV that is C) A second born male.
given via monthly intramuscular injection to high-risk patient D) A second born emale.
populations during the RSV season. Potential quali ying in ants E) T e rate o pyloric stenosis is equal in all o these groups.
include those with associated prematurity, signi cant congeni-
tal heart disease, neuromuscular disorders, or chronic lung dis- Answer 13.14.2 The correct answer is “A.” T ere is a
ease. Administration o palivizumab usually begins be ore RSV 4:1 male: emale preponderance, and about 30% o all cases
season begins and then is given monthly or a maximum o occur in a rst born child. T us, rst born males are at a par-
5 doses. RSV–IGIV, or RespiGam, is no longer used. T ere is ticular risk or pyloric stenosis. Also, amily history is important
no current RSV vaccine. None o the antiviral medications (“B” as in ants born to a mother or ather who had pyloric stenosis
and “C”) works or prevention or treatment o RSV. are at increased risk.
Objectives: Did you learn to . . .
• Re ognize he lini l presen ion o p ien wi h RSV You give the patient a bolus o IV normal saline and decide to
bron hioli is? con rm the diagnosis with a Gastrogra n test.
• d i gnose n re RSV bron hioli is?
• d es ribe wh p ien s m y possibly bene rom rib virin Question 13.14.3 What are you most likely to see on radio
(none!) n RSV prophyl xis? graph i this patient has pyloric stenosis?
A) “Bullseye” or “target” lesion.
B) Corkscrew pattern o small bowel.
CASE 13.14 C) Ulcerations o the gastric mucosa.
A 4-week-old in ant presents to your o ce with his parents. D) “String” sign.
T e parents note that he vomits every time he eats. His vomi- E) Rapid lling o the bowel distal to the pylorus.
tus is mostly ormula and nonbilious. He seems to be hungry
and is demanding to be ed o en. Except or the vomiting, he Answer 13.14.3 The correct answer is “D.” T e string sign is
seems to be well without diarrhea. Examination reveals an ormed when contrast material trickles through the elongated
a ebrile in ant in no distress with normal cardiac and pul- pyloric channel (and thus looks like a thin string on x-ray). “A,”
monary examinations and a relatively benign abdomen. His “bullseye” or “target” lesion, is seen on ultrasound with intus-
weight today is unchanged rom a weight taken in your o ce susception. “B,” a corkscrew pattern, is seen with midgut volvu-
2 weeks ago. T ere is no “olive” palpable (it must be in the lus. Ulcerations o the gastric mucosa are seen, o course, with
martini you’re drinking as you’re reading through this book). gastric ulcers. “E” is obviously not true. T e whole problem with
pyloric stenosis is that little or anything can get rom the stom-
Question 13.14.1 Your working diagnosis is: ach to the small bowel. Ultrasound can also be used to make the
A) Midgut volvulus. diagnosis o pyloric stenosis. Ultrasound is o en the rst test
B) Gastroenteritis. ollowed by a contrast study i needed. Physical examination is
C) Pyloric stenosis. o en unreliable (when have you actually elt that “olive”?). Do
D) CNS injury with increased intracranial pressure. an imaging study i you have any concern or pyloric stenosis.
CHAPTER 13 • PEd Ia t RIc S 387

Answer 13.15.1 The correct answer is “E.” All o the above can
HELPFUL TIP:
be a result o in uenza. Pneumonia may be viral or due to a
t he e ini ive re men or pylori s enosis is surgi l.
secondary bacterial superin ection. Patients may present with
However, i is n urgen , bu no emergen , surgi l in-
in uenza-related severe leg pain secondary to rhabdomyoly-
i ion. M ke sure o ke he ime o orre lui n
sis; the CK will be elevated. Myocarditis can present with CHF,
ele roly e bnorm li ies prior o surgery.
arrhythmias, and sudden death; myocarditis can also be sub-
clinical. Obviously, encephalitis will present with mental status
changes.
You have a surgical consultant see the patient, pyloromyot-
omy is done, and everybody is happy. You get labs on this patient. He is noted to be hypoglycemic
and to have markedly elevated liver enzymes. His bilirubin is
Objectives: Did you learn to . . . normal. T e patient’s white blood cell count and di erential
• I en i y lini l m ni es ions o pylori s enosis? suggest a viral illness.
• d i gnose n m n ge pylori s enosis?
Question 13.15.2 The MOST LIKELY diagnosis based on this
patient’s history, physical, and laboratory ndings is:
QUICK QUIZ: PEd Iat RIc BELLY Pa IN A) Reye syndrome.
B) Bacterial meningitis.
T e treatment or midgut volvulus is: C) ransverse myelitis.
A) Watch ul waiting. D) Hepatic encephalopathy secondary to hepatitis.
B) Maneuver o Leopold. E) Diabetes mellitus in the “honeymoon” period with Somogyi
C) Emergent surgery. phenomenon.
D) Air enema.
E) Nasogastric suction and bowel rest. Answer 13.15.2 The correct answer is “A.” T is patient most
likely has Reye syndrome (encephalopathy with atty liver de-
The correct answer is “C.” Midgut volvulus is a surgical emer- generation). Reye syndrome presents with intractable vomit-
gency. Midgut volvulus is an anatomical twisting o the bowel ing, elevated transaminases with a normal bilirubin level, hypo-
and needs to be surgically corrected. It presents with recurrent glycemia, and mental status changes (excitability, delirium,
bilious vomiting; bilious vomiting in an in ant should prompt coma). Reye syndrome generally occurs in a genetically sus-
you to rule out mid-gut volvulus. None o the others are cor- ceptible person associated with a viral illness such as in uenza
rect. Watch ul waiting is dangerous and risks ischemic bowel or chickenpox who has been given aspirin. T is patient is not
with per oration, so “A” and “E” are wrong. Leopold maneuvers likely to have meningitis given the absence o an elevated white
are used to move a breech pregnancy to vertex and have noth- count and lack o meningeal signs. Also, he clearly does not
ing to do with the bowel. “D,” air enema, can be used to reduce have transverse myelitis, an autoimmune condition producing
an intussusception but has no role in the treatment o midgut rapidly progressive weakness and sensory disturbance. “E” is
volvulus. “E” nasogastric suction and bowel rest are the treat- incorrect since the Somogyi phenomenon re ers to stress hor-
ment or a small bowel obstruction (though NG is not really mone-induced morning hyperglycemia in a patient on insu-
help ul except to ameliorate vomiting). Any bowel obstruction lin who is having unrecognized hypoglycemia at night (and,
in a child demands investigation. much like the Loch Ness Monster, does not actually exist . . . see
Chapter 10).

CASE 13.15 Question 13.15.3 The medication MOST associated with


A 5-year-old male who was recently diagnosed with inf u- Reye syndrome is:
enza presents to your ED with complaints (via the parents) A) Ibupro en.
o intractable vomiting and mental status changes. On exam- B) Celecoxib (Celebrex).
ination, the child is ebrile and is vomiting but has neither C) Aspirin.
meningeal signs nor ocal neurologic ndings. His liver is D) Acetaminophen.
palpable. E) Naproxen.

Question 13.15.1 Inf uenza is associated with which o the Answer 13.15.3 The correct answer is “C.” T e use o aspirin
ollowing complications? in a child with a viral illness, classically in uenza or chicken-
A) Pneumonia. pox, has been associated with the development o Reye syn-
B) Rhabdomyolysis. drome. O note, the incidence o Reye syndrome has been
C) Myocarditis. steadily declining now that parents are aware that they should
D) Encephalitis avoid aspirin use in children. Reye syndrome is now a very rare
E) All o the above. disease.
388 Fa MILY MEd Ic INE EXa MINa t ION & BOa Rd REVIEW

TABLE 13-6 CAUSES OF HEME POSITIVE STOOLS D) SIDS is more likely to occur in emale in ants.
IN CHILDREN E) Young maternal age (< 20 years old) is associated with
reduced risk o SIDS.
Less than 1 year: Sw llowe m ern l bloo (espe i lly i he hil
is v mpire), n l ssure, in ussus ep ion, uo en l or g s ri
ul ers, g ngrenous bowel, Me kel iver i ulum, ow milk pro ein Answer 13.16.1 The correct answer is “C.” A sibling o child
in oler n e who died with SIDS has a ve old increased risk o dying o SIDS
More than 1 year: c olon polyps, ul ers, n l ssure, esoph ge l (although the absolute risk is still less than 1%). “A” is incorrect.
v ri es, in ussus ep ion, hemorrhoi s, sw llowe epis xis, T e cause o SIDS is unknown. Abnormal arousal rom sleep
M llory–Weiss e rs, b eri l en eri is.
may play a role. SIDS does not seem to be an isolated genetic
disorder. “B” is incorrect. SIDS generally occurs between 2 and
4 months (median age 11 weeks). However, 90% o cases occur
HELPFUL TIP: be ore 6 months o age, so a 5-month-old child is still at risk.
t he re men o Reye syn rome is suppor ive n in- “D” is incorrect as it is more common in males with a ratio o
lu es on rolling hypogly emi n re ing erebr l 3:2. Finally, the opposite o “E” is true: maternal age < 20 years
e em . t here is no spe i i re men o his illness. t he old is a risk actor as is maternal smoking and the in ant’s sleep-
mor li y r e rem ins high. Surviving p ien s shoul ing position.
be s reene or n un erlying y i oxi ion
ee . Question 13.16.2 All o the ollowing have been associated
with an increased incidence o SIDS EXCEPT:
A) Side-sleep positioning.
B) Prone-sleep positioning.
For some reason, you ound heme positive stool in this
C) Early introduction o solid oods.
patient. You must have been thinking that every patient
D) Premature birth.
deserves at least one rectal examination.
E) Intrauterine exposure to drugs.
Question 13.15.4 All o the ollowing are relatively common
Answer 13.16.2 The correct answer is “C.” T e introduction
causes o heme positive stools in children 1 to 5 years o
o solid oods has nothing to do with the development o SIDS.
age EXCEPT:
All o the other actors increase the rate o SIDS. O note, any
A) Colon polyps.
nonsupine sleeping position is associated with an increased
B) Ulcers.
incidence o SIDS.
C) Swallowed epistaxis.
D) Gangrenous bowel.
Question 13.16.3 The most important advice you can give
E) Intussusception.
to prevent SIDS is:
A) Avoid eeding close to bedtime to reduce aspiration risk.
Answer 13.15.4 The correct answer is “D.” Gangrenous bowel
B) Place the child in the prone position (stomach and ace
tends to be ound in neonates and younger in ants < 1 year old
down) when sleeping.
as a result o necrotizing enterocolitis, midgut volvulus, etc.
C) Use a child alarm in the room to alert the parent(s) i some-
All o the others are correct. See able 13-6 or more causes o
thing is wrong.
occult GI blood loss in children.
D) Place the child in the supine position (stomach and ace up)
Objectives: Did you learn to . . . when sleeping.
• Re ognize sequel e o inf uenz in e ion? E) Use sheepskin or polystyrene bedding to prevent su oca-
• d es ribe Reye syn rome? tion.
• M n ge p ien wi h Reye syn rome?
• I en i y some uses o heme posi ive s ools in hil ren? Answer 13.16.3 The correct answer is “D.” Having the in ant
sleep in a supine position (“back to sleep”) reduces the risk o
CASE 13.16 SIDS by up to 50%. “B” (prone sleeping position) and “E” (poly-
styrene or sheepskin bedding) increase the risk o SIDS. Smok-
A mother comes to you with concerns about SIDS in her ing cessation is important as smoking during pregnancy or a er
in ant. Evidently, her sister had a child who died o SIDS, and increases the risk o SIDS. Co-sleeping should be discouraged as
she is concerned that her child is now at elevated risk. it increases the risk o SIDS.

Question 13.16.1 You can tell her which o the ollowing? HELPFUL TIP:
A) win concordance studies suggest SIDS is a genetic disorder. t he use o p i iers in be u lly e re ses he risk
B) Since her child is now 5 months old, there is no risk o SIDS o SId S s well. Give i o he hil when he or she is
since SIDS generally occurs be ore 3 months o age. pu own or sleep. d o no reinser i on e he hil is
C) Siblings o an in ant with SIDS have a ve old increase in the sleep i he/she spi s i ou .
risk o SIDS.
CHAPTER 13 • PEd Ia t RIc S 389

T ere has been some observational evidence to suggest a con-


HELPFUL (AND SCARY) TIP:
nection between diabetes type 1 and cow’s milk, but the data are
No hing is ree (espe i lly he bo r ex min ion . . . how
quite suspect. “D” is incorrect. Foods should be introduced one
mu h re you p ying or h su ker?). Supine sleeping
at a time, but there is no requirement as to what the rst ood
h s in re se he r e o posi ion l pl gio eph ly ( l -
should be. Usually, iron orti ed cereals are introduced rst.
ening o he he , in his se in he o ipi l region).
t his risk n be minimize by pl ing he hil in
prone posi ion while awake and supervised ( om- HELPFUL TIP:
monly re erre o s “ ummy ime”). Helme s o re orm In ro u ing Foo s n Foo a llergy: c on r ry o wh
he sh pe o he skull h ve been shown no o work m ny o us h ve been ugh , he e rly in ro u ion
(BMJ. 2014;348:g2741). o llergens su h pe nu s seem o reduce he risk o
subsequen llergy (N Engl J Med. 2015;372:803–813).
Whe her his ex en s o o her oo s is no known. Bu
i is le s re son ble o in ro u e possibly llergeni
Objectives: Did you learn to . . .
oo s (pe nu s, whe ) e rly.
• I en i y risks o SId S?
• Give vi e on how o re u e he risk o SId S?

HELPFUL TIP:
CASE 13.17 a a P gui elines re ommen 400 IU/ y o vi min d
T e parents o a 1-month-old emale bring the in ant or her rom bir h hrough he een ge ye rs, no ex ep ions.
routine check-up. T ey are rst time parents and are con- t his n ome ei her rom supplemen s or oo s. Bre s
cerned about eeding. Evidently, one o the grandmothers milk oes no on in enough vi min d , n ex lusive-
is “ rom the old country”. Back in her day in the “old coun- ly bre s - e in n s shoul be supplemen e in he irs
try,” they would start children on solid oods at 2 months o ew ys o li e.
age—usually starting with strained liver (lots o vitamins
and stu plus it is disgusting and the adults wouldn’t eat it).
T is grandmother also cannot gure out why the couple is T e parents manage to end o this grandmother (quite an
wasting their money on ormula when cow’s milk (better yet, accomplishment, eh?). T e child continues on ormula eed-
goat’s milk when available) “worked just ne or me.” T ey ings. However, the child gets somewhat ussy on occasion and
would like your advice about eeding. has bouts o diarrhea. T e amily wants to know what to do.

Question 13.17.1 You let them know that: Question 13.17.2 Your advice is to:
A) Solid oods should be introduced at 3 months or all chil- A) Change ormulas because this represents an allergy to the
dren. wo months o age is too soon. And grandma’s right. cow’s milk ormula.
Start with liver. B) Do not change ormulas.
B) Cow’s milk is considered adequate only a er 6 months C) Change ormulas because this likely represents lactase de -
o age. ciency.
C) Early introduction o cow’s milk has been irre utably linked D) Switch to a ormula based on short chain atty acids since
to type 1 diabetes mellitus. this likely represents an inability to absorb and metabolize
D) T e rst oods introduced should be strained meats. ats.
E) Children should have good head control be ore solid oods
are introduced. Answer 13.17.2 The correct answer is “B.” Children will occa-
sionally be ussy and have occasional diarrhea. T is does not
Answer 13.17.1 The correct answer is “E.” Doesn’t “E” just indicate a ormula allergy or intolerance. Re ux and spitting
sound best? We mean, even i you hadn’t a clue, you would still up are also common in in ants. Again, this does not indicate
pick “E,” right? Children should have good head control be ore a ormula allergy. T e parents should be reassured that as long
starting solid oods. “A” is incorrect or several reasons. First, as the child is growing and is not having signi cant di culties,
solid oods are generally not recommended until 4–6 months continuing the current ormula is acceptable.
o age. Second, note that the answer says “all children.” T ird,
the answer involves liver. Liver is disgusting no matter what HELPFUL TIP:
age . . . don’t oist it on a trusting, unsuspecting, in ant. “B” is Remember, s no e bove, re lux or spi ing up oes
incorrect. Cow’s milk should be avoided or the rst year o li e. no me n ormul in oler n e. Do not make multiple
Breast eeding is optimal and in ant ormula is another good formula changes! t his me i lizes norm l p ern
option i mom decides not to breast eed; ormula contains addi- in hil ren. t he m jori y o spi ing up n re lux will
tional nutrients not ound in cow’s milk. In addition, the early resolve by 1 ye r o ge. I he hil looks well n is
use o cow’s milk has been linked to an increase in gastrointes- growing, here is no nee o h nge he ormul .
tinal blood loss and iron de ciency anemia. “C” is incorrect.
390 Fa MILY MEd Ic INE EXa MINa t ION & BOa Rd REVIEW

stool) as well as irritation o the rectal and anal mucosa. Finally,


T e parents decide that you are correct and continue to eed polyethylene glycol solution is reasonable in those greater than
the child the cow’s milk ormula. Much as you predicted, the 4 months o age as are sorbitol containing purees such as prunes
child does well on this ormula. However, the child is now (assuming the child is taking solids).
becoming “constipated” and the grandmother would like to
give this child a laxative (pre erably some concoction rom T e child does well a er the parents implement your lactu-
the “old country” that no one else can pronounce or ormu- lose plan, but she continues to have some ref ux symptoms.
late). She (the in ant, not the grandmother) is having only T is is getting worrisome to the parents although the child is
one bowel movement per day rather than the 3 to 4 per day growing well.
that occurred during the rst several months o li e.
Question 13.17.5 Which o the ollowing IS recommended
Question 13.17.3 You let the parents know that: as standard therapy in treating this child’s ref ux?
A) One stool a day is likely normal or this in ant. A) Prone positioning o the child when sleeping.
B) An in ant having less than 2 to 3 bowel movements per day B) rial o hypoallergenic diet.
is likely constipated since bowel transit time is only 8.5 hours C) Use o a proton pump inhibitor.
or so. D) Elevating the head.
C) In ants ed soy-based ormula tend to have so er stools than E) T ickening the ormula with rice cereal.
those ed cow’s milk ormula.
D) Breast ed in ants may normally have up to 10 stools per day. Answer 13.17.5 The correct answer is “E.” T ickening the or-
E) A and D. mula may be help ul. Placing the child in prone position while
sleeping increases the risk o SIDS. Neither a hypoallergenic
Answer 13.17.3 The correct answer is “E.” T e number o diet, an H 2 blocker, a proton pump inhibitor, nor elevating the
bowel movements in a normal in ant can vary widely rom 10 head has been shown to reduce re ux. In act, elevating the head
per day in a breast ed in ant to 1 per day or every other day in (as in a car seat) may increase re ux.
a ormula- ed in ant. What is more important is whether the
child has to strain to pass stool, how hard the stool is, etc. T e
number o stools per day is less important. HELPFUL TIP:
In in n s wi h g s roesoph ge l re lux disease (weigh
On urther questioning, it becomes clear that the grand- loss, e ., no jus physiologi re lux), ri l o milk n
mother may be correct. T is child is having a hard time pass- soy pro ein- ree ie or 1 o 2 weeks is re son ble. H2
ing stool and passes only small amounts o hard stool with blo kers n PPIs h ve lso been use bu wi h very lim-
great e ort (red-purple ace, lots o crying, etc.). T e parents i e su ess. Prokine i gen s su h s me o lopr mi e
would like some advice. n ery hromy in help improve g s ri emp ying, bu
s u ies h ve ile o show mu h e i y or GERd . In
Question 13.17.4 Which o the ollowing is a reasonable 2009, he Fd a issue bl k box w rning linking hron-
suggestion to treat this in ant’s constipation? i me o lopr mi e use wi h r ive yskinesi .
A) Change to a ormula with a preponderance o whey protein
such as Carnation Good Start.
Objectives: Did you learn to . . .
B) reat with corn syrup (whatever brand is cheapest).
• Give ppropri e vi e or nu ri ion n ee ing uring
C) Add ruit juices i the child is older than 2 months. in n y?
D) reat with lactulose.
• d i gnose n m n ge ons ip ion in in n y?
• t re n in n wi h signi n g s roesoph ge l ref ux?
Answer 13.17.4 The correct answer is “D.” Let’s look at this in
detail (since parents will tell you all o the details o their child’s
stools and o en bring in a “sel e” o the stool . . . . don’t tell us it
CASE 13.18
hasn’t happened!). “A”, whey protein”, was thought in the past A 23-year-old G2P1 emale at 31 weeks o gestation pres-
to ease constipation. T is has turned out not to be the case. “B” ents to the ED and delivers precipitously. Her pregnancy was
is problematic or a di erent reason. Many corn syrups in the uncomplicated. T e neonate is a 1.25-kg male. T e helicop-
past contained glycoproteins that were metabolized into non- ter is on its way to transport this premature in ant to your
absorbable sugars. T is is no longer true or many corn syrups; regional pediatric hospital. However, it is going to be at least
the ormulation has changed. Fruit juices contain sorbitol and 3 hours because o weather concerns.
are e ective. However, they should not be used under the age
o 4 months. Beyond that age, sorbitol-containing juices such Question 13.18.1 In the meantime, the BEST treatment or
as pear, prune, or apple may be used to treat constipation. As this newborn is:
an aside, glycerin suppositories can be used sparingly or very A) Antibiotics.
hard stool in the rectum. Routine use o glycerin suppositories B) Corticosteroids.
may lead to tolerance (a dependence on manual stimulation to C) High-pressure 100% oxygen.
CHAPTER 13 • PEd Ia t RIc S 391

D) Routine care (warmth, stimulation, etc.) and nothing else.


HELPFUL TIP:
E) Sur actant.
a s prem ure in n s grow in o ul s, keep n eye on
heir bloo pressures n glu ose levels. In n s who
Answer 13.18.1 The correct answer is “E.” Without any urther
were born prem urely n very low bir h weigh
in ormation, you must assume that this patient is at signi cant
(< 1.5 kg) h ve more problems wi h hyper ension n
risk or neonatal respiratory distress syndrome (RDS). T e appro-
glu ose in oler n e s ul s.
priate treatment is sur actant. T e dose o sur actant depends
on the preparation, and the timing o administration varies by
source (some authors avor immediate administration “pro-
phylaxis,” while others recommend “early rescue” treatment— Because o your congenial bedside manner and amazing
usually within 2 hours o delivery). Sur actant therapy has good command o medical knowledge, this amily returns to see
evidence or decreasing morbidity and mortality and should be you many times over the ensuing years. A er looking at your-
used in all premature in ants who are at risk or RDS (see next sel in the mirror and ref ecting on how awesome you are,
question) or acutely symptomatic. On the basis o synergistic you turn your attention to the newest addition to the am-
interactions, experts recommend combination therapy with ily, a 4-day-old male in ant you delivered—this time in the
antenatal corticosteroids or the mother (i possible . . . see Chap- labor and delivery unit and at term. Gestation and delivery
ter 15) and sur actant or the premature in ant. “A” is incorrect were uncomplicated. He is breast ed and was doing well until
as it is not the most important initial therapy although empiric this morning. Un ortunately, his mother noticed today that
antibiotic therapy is o en indicated in premature in ants due he is breathing harder, eating poorly, and looking more yel-
to di culty in distinguishing between RDS and sepsis. “B” is low. You note a respiratory rate o 72 and crackles on lung
incorrect because steroids or the newborn in ant are associated examination, so you order a chest x-ray.
adverse neurodevelopmental outcomes. As or “C,” excessive
oxygen can be toxic to premature in ants, and initial resuscita- Question 13.18.3 Which o the ollowing diagnoses is most
tion should begin with blended oxygen or room air. “D” might likely?
be the second best option i sur actant were unavailable but note A) Neonatal RDS.
that it says “and nothing else.” It is highly unlikely this premature B) Persistent pulmonary hypertension.
in ant would be in need o “nothing else” (college tuition may be C) Pneumonia.
nice). Sur actant is undoubtedly indicated here. D) Pneumothorax.
E) ransient tachypnea o the newborn.
HELPFUL TIP:
Answer 13.18.3 The correct answer is “C.” Pneumonia is more
Sur n h s been shown o e re se ir le ks n
likely than the other options because o the timing o the symp-
mor li y rom Rd S bu oes no e re se he in i en e
toms. Neonatal pneumonia may be early (within the rst week
o hroni lung ise se or bron hopulmon ry yspl si .
o li e) or late onset. T e pathogenic organisms involved are di -
erent or early and late disease. Patients presenting with early
Question 13.18.2 All o the ollowing are risk actors or onset pneumonia are more likely to have acquired the in ection
neonatal RDS EXCEPT: in utero or during delivery; GBS is the most common patho-
A) Cesarean section. gen in these cases. Patients presenting with late pneumonia
B) Gestational diabetes. may have acquired the disease during delivery, during hospi-
C) Male sex o in ant. talization, or while in the community; GBS, E. coli, S. aureus,
D) Prolonged rupture o membranes. Listeria, HSV, and Chlamydia may all cause pneumonia in these
patients. “A” is incorrect since RDS should not occur in a ull-
Answer 13.18.2 The correct answer is “D.” Prolonged rupture term in ant nor present this late. “B” is incorrect because persis-
o membranes is associated with a decreased risk o neonatal tent pulmonary hypertension should present shortly a er birth
RDS. All o the others are associated with an increased risk. O with hypoxemia and cyanosis. “D,” pneumothorax, is a com-
course, increasing prematurity is associated with increasing risk mon result o birth trauma; up to 2% o in ants may sustain a
o RDS as is multi etal pregnancy, especially in the second etus pneumothorax during delivery. However, very ew in ants with
delivered in a twin pregnancy. a pneumothorax are symptomatic, and you would expect the
symptoms to occur shortly a er delivery. In addition, breath
HELPFUL TIP: sounds would be absent or diminished, and there would not be
t he le i hin/sphingomyelin (L/S) r io is he r i ion l rales. “E” is incorrect because transient tachypnea is a result o
me ho or ssessing e l lung m uri y. I he L/S r io delayed reabsorption o alveolar uid, occurs with 1 to 2 hours
is > 2, he risk o Rd S is low. O her op ions in lu e phos- o birth and resolves on its own.
ph i ylgly erol level n ire sur n me sures. I
is impor n o re lize h hese es s h ve irly low- You ound your rectal thermometer, and it turns out that
posi ive pre i ive v lues. your patient is ebrile. T e chest x-ray shows a le lower lobe
in ltrate.
392 Fa MILY MEd Ic INE EXa MINa t ION & BOa Rd REVIEW

Question 13.18.4 Which o the ollowing antibiotics is TABLE 13-7 EVALUATION OF THE FEBRILE NEONATE
LEAST appropriate in this setting?
Age < 7 days: a LL shoul re eive ull workup in lu ing urine, bloo
A) Ampicillin.
ul ures, LP wi h ul ures, c Bc , c XR, mpi illin + gen mi in. a mi .
B) Ce riaxone. Age 7–28 days: a LL shoul re eive ull workup in lu ing urine,
C) Gentamicin. bloo ul ures, LP wi h ul ures, c Bc , c XR in hose wi h respir ory
D) Vancomycin. symp oms. a mpi illin + e o xime or mpi illin + gen mi in.
a mi .
Answer 13.18.4 The correct answer is “B.” Ce riaxone is con- Age 28–60 (or 90) days:
traindicated in neonates. Ce riaxone displaces bilirubin rom Well appearing: Urine wi h ul ure; c Bc (WBc o 5,000–15,000 and
< 500 b n s sugges s lower risk o in e ion); LP no nee e i hil
albumin-binding sites resulting in more severe hyperbilirubine- is well looking wi h reli ble ollow-up; c XR i respir ory r e
mia and the risk o bilirubin encephalopathy (this in ant is jaun- > 50, r les, rhon hi, o her respir ory symp oms → I workup is low
diced). Ce riaxone may cause atal precipitates in the lung and risk onsi er e ri xone 50 mg/kg × 1 ose n re urn visi he
kidney when co-administered with calcium (perhaps not an nex y or re he k. I high risk b se on c Bc , e . mi (see bx
immediate concern or this patient). Ce otaxime may be used, hoi e below).
Ill appearing: c Bc , LP, Ua in lu ing urine, bloo n c SF ul ure, c XR
but this is not given as a choice. It would be prudent to provide i ny respir ory symp oms (see bove), c e ri xone + a mpi illin.
empiric antibiotic coverage or all likely bacterial pathogens. a v n omy in i c SF is bnorm l. a mi o hospi l.
T e combination o ampicillin and gentamicin is pre erred or
coverage o GBS, Gram-negative enteric bacteria, and Listeria.
Due to the potential or MRSA to cause pneumonia in the neo-
natal period, vancomycin is reasonable but should not be used Answer 13.19.1 The correct answer is “D.” Fever in a neo-
alone. nate (younger than 28 days) or young in ant (age 29–90 days)
is de ned as a temperature 38°C or 100.4°F (rectal) or higher.
HELPFUL TIP: Up to 15% o ebrile neonates and young in ants will have a
Remember he possibili y o sus ine p roxysm l su- serious bacterial in ection such as bacteremia, urinary tract
pr ven ri ul r hy r i wi h c HF in he in n who in ection, meningitis, pneumonia, osteomyelitis, septic arthri-
is hypnei . t hese hil ren o en presen wi h poor tis, omphalitis, mastitis, or scalp abscess. U I is the most
ee ing, we kness, hypne , n r les. t re men common. T e in ant may appear well with ever being the
in lu es v g l m neuvers (su h s pl ing b g o i e only mani estation o a serious in ection. Risk strati cation
over he e or 15 o 30 se on s . . . . n w hing he is based on age, co-morbidities, laboratory studies, and the
horror on he p ren s’ es s hey hink you re su - presence o viral in ection such as bronchiolitis. T ere is
o ing he ki ). O her op ions in lu e r ioversion no clear consensus on the best approach to evaluating and
n enosine. managing ebrile in ants younger than 90 days. Currently,
all ebrile neonates, ill appearing in ants, or those not meet-
ing low-risk criteria should be admitted or a ull evaluation
Objectives: Did you learn to . . . (including UA, LP, CBC, blood cultures) and empiric antibi-
• Provi e ppropri e re men o n in n risk or neon - otic therapy. “B” is incorrect as cultures should be obtained
l Rd S? prior to antibiotic therapy. You may obtain a chest x-ray as
• I en i y risk ors or neon l Rd S? part o your evaluation, but the best answer is admission.
• d es ribe uses o respir ory is ress in he neon l Ibupro en is not approved or use in in ants younger than
perio ? 6 months. See able 13-7 or evaluation o a ever in a child 7
• Ini i e re men or neon l pneumoni ? to 90 days o age.

T e patient is admitted. Blood work including a complete


CASE 13.19 blood count and blood culture is obtained. Urinalysis and
A 20-day-old ormer term male in ant is brought to the clinic urine culture are obtained via sterile catheterization. A
or evaluation o ever. He was born via uncomplicated vagi- lumbar puncture is per ormed a er obtaining consent (o
nal delivery. Mom was GBS negative. In clinic, the in ant has course) and cerebrospinal f uid is sent or culture and analy-
a temperature o 38.5°C but is otherwise vigorous with nor- sis. (Use lidocaine or the LP even in these young in ants . . . . it
mal vital signs. hurts!)

Question 13.19.1 What should be done next? Question 13.19.2 While waiting or your laboratory testing
A) Discharge home with ollow-up the next day. results what empiric antibiotics will you start?
B) reat with empiric antibiotics. Cultures are not necessary or A) Ampicillin and clindamycin.
this age group. B) Gentamicin.
C) Obtain a chest x-ray. C) Vancomycin and metronidazole.
D) Admit to the hospital. D) Ce riaxone
E) reat with ibupro en. E) Ampicillin and ce otaxime.
CHAPTER 13 • PEd Ia t RIc S 393

Answer 13.19.2 The correct answer is “E.” T e choice o


HELPFUL TIP:
empiric antibiotics is based on most likely in ecting bacterial
Neon l os eomyeli is o en involves mul iple bones.
organisms in a patient o this age. Bacterial in ections in neo-
nates likely were acquired during delivery. T e most common
organisms are GBS, E. coli, and Listeria monocytogenes. Empiric
treatment should be with ampicillin and ce otaxime or ampi-
cillin and gentamicin. Ampicillin provides coverage or Liste-
ria and enterococci. Broadened coverage should be considered You discuss your concerns with the mother.
depending on ocus o in ection and i Gram-positive cocci are
seen on CSF Gram stain, vancomycin should be used instead o Question 13.19.4 She asks you how you will you diagnose a
ampicillin. Empiric acyclovir treatment should be considered i bone in ection? Your response is:
signs and symptoms o HSV in ection are present. A) Clinically.
B) With cultures.
While examining the patient, you notice he is not moving his C) With x-ray.
right arm. When you ask the mom, she believes that is new D) With MRI.
but is unsure when he stopped using it. Examination reveals E) All o the above.
tenderness over the right proximal humerus with overlying
redness, swelling, and pain with moving the shoulder joint, a Answer 13.19.4 The correct answer is “E.” Osteomyelitis is a
symmetric Moro ref ex, and intact grasp ref ex. clinical diagnosis requiring you to think about it in your di er-
ential. Cultures and imaging con rm the diagnosis. T ere is no
Question 13.19.3 What is the likely cause? speci c laboratory test. Frequently, a leukocytosis with le shi
A) Erb palsy. and reactive thrombocytosis are present. In ammatory mark-
B) Klumpke palsy. ers, including C-reactive protein and erythrocyte sedimenta-
C) Humerus racture. tion rate, are usually elevated; CRP can be ollowed to docu-
D) Osteomyelitis. ment response to therapy; ESR takes too long to change to be o
E) Cerebral vascular incident. much use in monitoring treatment. Cultures o the blood, joint
uid, and/or bone are positive in 50% to 80%. Plain x-rays can
Answer 13.19.3 The correct answer is “D.” Fever, pseudopa- rule out other causes such as a racture but are o en not use ul
ralysis (reluctance to move a ected limb) and swelling with ery- in early osteomyelitis as it can take up to 10 days to see changes
thema o the a ected area is concerning or osteomyelitis with such as a lytic lesion. MRI is the best imaging study allowing
associated septic arthritis. Osteomyelitis, an in ection o the or abscess detection and di erentiation between bone and so -
bone, usually results rom hematogenous seeding. In ants have tissue in ection. MRI o en requires sedation and is expensive. A
blood vessels that cross the growth plate allowing or bone in ec- radionuclide bone scan is help ul or poorly localized pain and
tions to spread to the adjacent joint resulting in septic arthritis. or concerns about multiple bone involvement. Ultrasound can
S. aureus is the most common cause o osteomyelitis in all pedi- be help ul in septic arthritis.
atric age groups. H. in uenzae is less common since immuniza-
tions were begun; however, until culture results are available, You consult your pediatric orthopedic colleagues who take
it should be covered empirically in children aged 6 months to the in ant to the operating room to wash out the joint and
4 years who have not yet completed their immunization series. obtain cultures. An x-ray shows a lytic lesion in the proximal
GBS and Gram-negative rods such as E. coli are seen in neonates humerus.
and should be covered empirically. “A” and “B” are types o bra-
chial plexus injuries rom birth trauma that result in paralysis Question 13.19.5 Cultures o the joint and bone grow GBS.
and an asymmetric Moro re ex due to peripheral nerve injury. What is the length o antibiotic treatment?
“C,” a humerus racture, though rare can occur as a result o A) 7 days.
birth trauma but is unlikely based on delayed presentation. “E” B) 10 days.
is incorrect as most neonatal strokes present with seizures and C) 4 weeks.
or altered mental status. O note, the normal Moro and grasp D) 3 months.
re exes in this child point us away rom true paralysis. E) 1 year.

HELPFUL TIP: Answer 13.19.5 The correct answer is “C.” Acute osteomyelitis
Neon es wi h os eomyeli is re ommonly ebrile is treated with antibiotics or a minimum o 3 weeks with most
n presen wi h nonspe i i symp oms su h s irri - courses lasting 4 to 6 weeks. T e majority o patients may be
bili y, e re se movemen o limb (pseu op r lysis), treated initially with intravenous antibiotics ollowed by transi-
or p in wi h moving he e e ex remi y ( hink p in tion to oral antibiotics a er clinical improvement and lack o
uring i per h nge). O en he i gnosis is el ye contraindication to oral therapy. In neonates, the entire course
n mis i gnose irs s r um i injury. is requently completed intravenously due to lack o clinical
trials. Even with appropriate treatment, neonatal osteomyelitis
394 Fa MILY MEd Ic INE EXa MINa t ION & BOa Rd REVIEW

may result in permanent de ormities to the bone or joint with


d o no per orm rou ine he im ging in hil wi h simple
resulting decreased range o motion, asymmetric limb length, ebrile seizure. d ire your inves ig ion ow r ev lu ion
and abnormal gait; there ore, long-term ollow-up is manda- n re men o he un erlying ever.
tory. Note: Now that she has had one child with invasive GBS
d o no pres ribe n ibio i s or vir l in e ions, in lu ing vir l
disease, mom will need GBS prophylaxis during all uture labors
onjun ivi is n ol s.
i she were to get pregnant again.
d o no rou inely per orm Vc UG or he ev lu ion o irs
ebrile Ut I.
HELPFUL TIP:
d o no use e ri xone in neon es s i ispl es bilirubin rom
a n ibio i prophyl xis uring l bor e re ses he in i- lbumin bin ing si es n n use severe hyperbilirubinemi .
en e o e rly onse GBS ise se (wi hin he irs 7 ys
Ob in s reening l bs n bloo ul ures bir h ollowe by
o li e) bu oes no e l e onse GBS ise se (7 ys
minimum o 48 hours o n ibio i s in in n s born o mo h-
o 3 mon hs o ge). See c h p er 15 or in i ions or
ers wi h horio mnioni is.
n ibio i prophyl xis or GBS uring l bor.
Per orm re l ex min ion in ev lu ing hil or ons i-
p ion n en opresis.
Objectives: Did you learn to . . .
Per orm univers l he ring n me boli s reening in new-
• Ini i e i gnos i inves ig ion n m n gemen s r egy
borns.
or ebrile in n younger h n 90 ys?
• d i gnose n m n ge os eomyeli is? Re ommen pl ing b bies “B k o Sleep” o re u e he risk
o SId S. t he i ion o in n home pne moni ors oes
no provi e i ion l bene i .
QUICK QUIZ: FEBRILE SEIZURE

You are evaluating a 10-month-old ully immunized child a er BIBLIOGRAPHY


a simple ebrile seizure. T e child has been previously healthy Advisory Council on Immunization Practices. Birth-18 years
and has not taken antibiotics recently. Physical examination is and “catch up” immunization schedules. Available at
unremarkable with no meningeal signs and a normal neuro- http://www.cdc.gov/vaccines/schedules/hcp/child-
logic examination. adolescent.html. Accessed September 24, 2015.
Bekkali NL, et al. Rectal ecal impaction treatment in child-
What diagnostic testing should be per ormed? hood constipation: enemas versus high doses o oral PEG.
A) Lumbar puncture. Pediatrics. 2009;124(6):e1108–1115.
B) EEG. Berg MD, et al. Part 13: pediatric basic li e support: 2010
C) Neuroimaging such as head C or MRI. American Heart Association guidelines or cardiopulmo-
D) Serum electrolytes. nary resuscitation and emergency cardiovascular care.
E) None o the above. Circulation. 2010;122(18, Suppl 3):S862–S875.
Brown ML, et al. reatment o primary nocturnal enuresis in chil-
The correct answer is “E.” Per the 2011 AAP guidelines, evalu- dren: a review. Child Care Health Dev. 2011; 37(2):153–160.
ation o a simple ebrile seizure should ocus on identi ying the Cohen-Silver J, Ratnapalan S. Management o in antile colic: a
cause o the ever. Meningitis should always be considered and review. Clin Pediatr (Phila). 2009;48(1):14–17.
a lumbar puncture should be per ormed i meningeal signs or Committee on In ectious Diseases, American Academy o
symptoms are present. Previous recommendation was to per- Pediatrics. Kimberlin DW, Brady M , Jackson MA,
orm a lumbar puncture on all in ants younger than 12 months Long SS, eds. Red Book: 2015 Report o the Committee
who presented with a simple ebrile seizure. T is recommen- o In ectious Diseases. 30th ed. Elk Grove, IL: American
dation was made be ore the widespread immunization against Academy o Pediatrics; 2015.
H. in uenzae type b (Hib) and S. pneumoniae. Lumbar puncture Du oit G, et al. Randomized trial o peanut consumption in
is an option in in ants 6 to 12 months o age who are not ully in ants at risk or peanut allergy. N Engl J Med. 2015;
372:803–813.
immunized or i the immunization status is unknown. Consider
an LP in patients who have been on antibiotics because signs Fu LY, Moon RY. Apparent li e-threatening events (AL Es) and
the role o home monitors. Pediatr Rev. 2007;28(6):
o meningitis may be masked. Routine evaluation with EEG,
203–208.
neuroimaging, serum electrolytes, or complete blood cell count
Gerber JS, O t PA. Vaccines and autism: a tale o shi ing
should not be done i the only reason is to identi y the cause o
hypotheses. Clin In ect Dis. 2009;48(4):456–461.
the ebrile seizure.
Hoberman A, et al. Antimicrobial prophylaxis or children
with vesicoureteral re ux. N Eng J Med. 2014;370;
Clinical Pearls 2367–2376.
d o no gi e hil i you suspe epiglo i is s his m y Ingel nger JR, Stapleton FB. Antibiotic prophylaxis or
worsen irw y obs ru ion. vesicoureteral re ux-answers, yet questions. N Eng J Med.
2014;370:2440–2441.
CHAPTER 13 • PEd Ia t RIc S 395

Kattwinkel J, et al. Part 15: neonatal resuscitation: 2010 Ameri- diagnosis and management o an initial U I in ebrile
can Heart Association guidelines or cardiopulmonary in ants and children 2 to 24 months. Pediatrics. 2011;128(3):
resuscitation and emergency cardiovascular care. 595–610.
Circulation. 2010;122(18, Suppl 3):S909–S919. Sung V, et al. reating in ant colic with the probiotic Lactoba-
Kliegman RM, et al. Nelson extbook o Pediatrics. 19th ed. cillus reuteri: a double blind, placebo controlled random-
Philadelphia, PA: Elsevier Saunders; 2011. ized trial. BMJ. 2014;348:g2107.
Lauer BJ, Spector ND. Hyperbilirubinemia in the newborn. abbers MM, et al. Evaluation and treatment o unctional
Pediatr Rev. 2011;32(8):341–349. constipation in in ants and children: evidence-based
Mishori R, et al. Chlamydia trachomatis in ections: screen- recommendations rom ESPGHAN and NASPGHAN.
ing, diagnosis, and management. Am Fam Physician. J Pediatr Gastroenterol Nutr. 2014;58(2):258–272.
2012;86(12):1127–1132. ieder JS, et al. Brie resolved unexplained events ( ormerly
Moon RY, Fu LY. Sudden in ant death syndrome. Pediatr Rev. apparent li e-threatening events) and evaluation o lower-
2007;28(6):209–214. risk in ants. Pediatrics. 2016;137:e20160590.
Rappaport DI, et al. Should blood cultures be obtained in all van Wijk RM, et al. Helmet therapy in in ants with positional
in ants 3 to 36 months presenting with signi cant ever? skull de ormation: randomised controlled trial. BMJ.
Hosp Pediatr. 2011;1(1):46–50. 2014;348:g2741.
Russell KF, et al. Glucocorticoids or croup. Cochrane Database Wan J, et al. Section on urology response to new guidelines or
Syst Rev. 2011;(1):CD001955. the diagnosis and management o U I. Pediatrics. 2012;
Subcommittee on Urinary ract In ection, Steering Committee 129(4):e1051–1053.
on Quality Improvement and Management, Roberts KB. Zaoutis L, Chiang V. Comprehensive Pediatric Hospital Medicine.
Urinary tract in ection: clinical practice guideline or the Philadelphia, PA: Mosby, Inc. 2007.
Adolescent Medicine
M gha C
14
weight continue to track along their previously established
CASE 14.1 curves on the growth chart, both at approximately the h
A 14-year-old male presents to your clinic with his mother percentile. T e remainder o the examination is unremarkable.
or a routine well-child examination. T e patient’s mother
has some questions about puberty. Her son enjoys playing Question 14.1.2 What is your next step in evaluation o this
sports and she is concerned that he may be too small to play patient’s short stature?
ootball. T e past medical history is unremarkable. A) Obtain hand radiographs to assess bone age.
B) Draw blood to test or growth hormone and testosterone
Question 14.1.1 Which o the ollowing can you tell the levels.
mother will likely be the rst sign o puberty in this boy? C) Obtain an endocrinology consult.
A) Increase in penile length. D) Order computed tomography (C ) imaging o the brain to
B) Enlargement o the testes. rule out hypothalamic tumors.
C) Deepening o the voice. E) Order magnetic resonance imaging (MRI) o the brain to
D) Rapid increase in linear growth. rule out hypothalamic tumors.
E) Coarsening o pubic hair.
Answer 14.1.2 The correct answer is “A.” In the ace o a
Answer 14.1.1 The correct answer is “B.” Increase in the vol- normal physical examination and appropriate linear growth
ume o the testes is the rst sign o pubertal development in velocity, the likelihood o an intracranial process is very low. An
boys. T e age o onset o puberty in the past was 12 years (range assessment o bone age can be obtained easily, relatively inex-
10–14), but recent studies have shown that the mean age o onset pensively, and would be use ul prior to per orming other tests.
is now 1.5 to 2 years earlier than these historical norms. In a
study including more than 4,000 healthy boys, the mean age or You decide to obtain a bone age, and the radiologist reports
entering puberty was 10.14 years or Caucasian boys, 10.04 years bone age as 12 years and 3 months (remember that our
or Hispanic boys, and 9.14 years or A rican-American boys. patient is 14 years old).
“E” is o special note. While pubic hair appears shortly a er
the onset o puberty, it is initially long and straight and not in Question 14.1.3 What is the most likely diagnosis or your
a mature distribution. Coarsening o the pubic hair is a more patient?
advanced pubertal stage, occurring approximately 1.5 years a er A) Growth hormone de ciency.
the onset o puberty; an increase in penile length occurs simulta- B) Sella turcica tumor.
neously. T e maximal growth spurt occurs approximately 2 years C) Constitutional delay o growth and puberty.
a er the onset o puberty. Changing o the voice is a secondary D) Idiopathic testicular atrophy.
hormonal e ect that is quite variable in nature. E) estosterone receptor abnormality.

From the mother, you learn that the patient’s ather began Answer 14.1.3 The correct answer is “C.” T e bone age o
going through puberty during high school, and he did not 12 years and 3 months is reassuring. Since there is no usion o
reach his adult height until he was in college at about age 20. the growth plates, his bones are immature and still have the abil-
T e patient’s ather is 5′ 10′′ tall ( 178 cm). T e mother is 5′ 3′′ ity to grow. Had the bone age been 14, it is likely that his adult
tall ( 160 cm). Physical examination reveals that your patient height would be short; the bones would no longer have the inher-
is anner stage I or genitalia and pubic hair. His height and ent ability to grow. Bone age assessment is an accurate tool or
396
CHAPTER 14 • Ad o l es Cen t Med iCin e 397

determination o expected growth. Constitutional delay is the last 3 months, and she has progressively decreased the dis-
most common diagnosis or short stature and it is o en associ- tance and time o her runs (but, o course she’s a runner, so
ated with a delayed onset o puberty. Constitutional delay is a she hasn’t quit). She denies ever, swollen joints, or other sys-
diagnosis o exclusion; a complete history and physical o en rules temic symptoms. She normally has regular menses but notes
out other diagnoses. Family history o en reveals a parent who that she has irregular menses during cross-country season.
was a “late bloomer” but eventually had normal pubertal develop- T e patient’s past medical history is signi cant or a stress
ment. T is adolescent’s history o normal linear growth velocity racture in her le oot 18 months ago. Your examination
(albeit along the h percentile) is reassuring or the absence o reveals tenderness at the middle one-third o the right tibia.
growth hormone de ciency or intracranial abnormalities. T is In addition, she has pain on a single-leg hop. You are able to
adolescent’s combination o appropriate linear growth velocity, review her x-rays rom the emergency department, which do
appropriate adjustment or bone age, and paternal history o late not reveal any ractures or other abnormalities.
pubertal development is classic or a constitutional delay.
Question 14.2.1 What is the most appropriate next step to
When your patient’s height is plotted on a growth curve and diagnose your patient’s leg pain?
adjusted or bone age rather than chronological age, the A) Ultrasound o the lower extremity.
height now plots just below the 50th percentile. Your patient B) MRI o the lower extremity.
now has a question o his own. His biggest concern is that C) Dual-energy x-ray absorptiometry (DEXA) scan.
some o his classmates are starting to get taller, and he is D) T yroid-stimulating hormone ( SH) level.
a raid he will be too short to continue competing in sports. E) Urine pregnancy test.
Once again, his mother is 5′ 3″ tall ( 160 cm) and his ather
is 5′ 10″ tall ( 178 cm). Answer 14.2.1 The correct answer is “B.” MRI is sensitive and
speci c or stress ractures and has become the pre erred study.
Question 14.1.4 Based on what you know today, what is T e patient’s history and examination are concerning or the
your best estimate o this patient’s adult height? presence o a tibial stress racture. Stress ractures o en pres-
A) 5′ 3″ ± 2 (160 ± 5 cm). ent insidiously and cause gradual progression in symptoms over
B) 5′ 5″ ± 2 (165 ± 5 cm). time until a critical point is reached in terms o sports partici-
C) 5′ 7″ ± 2 (170 ± 5 cm). pation. Hal o stress ractures are not visible on plain radio-
D) 5′ 9″ ± 2 (175 ± 5 cm). graphs. Ultrasound can help in evaluating so tissue masses, but
E) 5′ 11″ ± 2 (180 ± 5 cm). it does not play a role in evaluating stress ractures. DEXA scan
can provide whole-body and site-speci c measurement o bone
Answer 14.1.4 The correct answer is “D.” You can provide a mineral density, which may be related to the pathophysiology
rough estimate o the patient’s adult height using the calculation o stress ractures, but does not help diagnose a site o injury.
or mid-parental height (MPH). Remember that the parent o Radionuclide bone scans are o en less expensive than C or
the opposite gender—the mother in this case—must have the MRI, and they demonstrate sites o injury based on increased
height adjusted or this calculation. For boys, add 5 inches to uptake o the radionuclide material. However, bone scans are
the mother’s height (12.5 cm) and then average this corrected not speci c or stress ractures and are o en alsely positive. O
maternal height with the paternal height to determine the MPH. note, plain radiographs may take months to become positive
For girls, subtract 5 inches rom the paternal height (12.5 cm . . . a er a stress racture and, while radiographs may be done, they
that is what you subtract, and NO the paternal height) and will not be o use i the result is negative. Given the patient’s
then average this corrected paternal height with the maternal other complaints, SH and urine pregnancy test may be warranted
height to calculate the MPH. In our patient’s case, his mother’s but will not assist in the diagnosis o her leg pain.
corrected height is 68 inches which averaged with his ather’s
height o 70 inches, equates to a MPH o 69 inches. A margin An MRI is obtained and con rms the presence o a stress
o error o ± 2 inches (5 cm) is o en given with MPH estimates. racture in the middle one-third o the right tibia. Because
o your patient’s history, especially that o multiple stress
Objectives: Did you learn to . . .
ractures, you are concerned she may be su ering rom the
• eva ua a pa w h gr w h c c r ?
emale athlete triad.
• i y h c ca pr a c u a ay?
• Ca cu a MPH? Question 14.2.2 What are the components o the emale
athlete triad?
A) Disordered eating, menstrual dys unction, altered bone
CASE 14.2 mineral density.
Your next patient is a 16-year-old emale cross-country run- B) Depression, weight loss, sports-related injury.
ner who you are seeing in ollow-up or right shin pain. She C) Poor sports per ormance, low sel -esteem, injury.
was diagnosed in the local emergency department 1 week ago D) Weight gain, bony injury, mood changes.
with “shin splints” and told to limit her activities. In your E) Fanatical attachment to sports, testosterone abuse, anger
o ce, she tells you that the pain has been worsening over the management issues.
398 FAMil Y Med iCin e eXAMin At io n & Bo ARd ReVieW

Answer 14.2.2 The correct answer is “A.” T e emale athlete C) Per orm a speculum-assisted pelvic examination with Pap
triad was rst identi ed in the early 1990s and originally char- smear.
acterized as anorexia, amenorrhea, and osteoporosis. As more D) Obtain a urine beta-hCG.
has been learned about the triad, the realization has been made E) Prescribe an oral progestin-only pill (progestin challenge).
that the triad likely represents a broader spectrum disorder
within each category. Many young women with the triad will Answer 14.2.4 The correct answer is “D.” T e most common
exhibit disordered eating behaviors, such as caloric restriction cause o secondary amenorrhea in women o childbearing age
or use o diuretics and diet pills, but would not meet the crite- remains pregnancy. I pregnancy has been excluded, and the
ria or anorexia or bulimia nervosa. Menstrual dys unction may history and physical are reassuring, a progestin challenge can
include oligomenorrhea or irregular, intermittent menses, as be help ul to determine i adequate estrogen is present; the pro-
well as amenorrhea. T ese young women may also have abnor- gestin challenge should induce menses i adequate estrogen exists
mal bone mineral density, with predisposition to bony injury, (see Chapter 15 or details on the evaluation o amenorrhea).
without having reached the strict criteria o osteoporosis. Imaging studies and hormonal levels may help in excluding other
diagnoses or may be warranted based on physical examination.
T e patient comes back to the o ce to receive her test results, A pelvic examination may be warranted based on history, but is
and you take the opportunity to obtain more history. T e o en not necessary in the initial evaluation o menstrual dys unc-
patient admits that she is very concerned about her diet tion. Pap smears are no longer recommended or women under
during her cross-country season, and she is very care ul to the age o 21, regardless o sexual activity status.
choose oods that have very little at but are o en high in pro-
tein. She will occasionally “go overboard” and eat a lot, or Your patient’s pregnancy test is negative and a progestin
which she compensates by taking part in extra workouts. She challenge induced menses, indicating adequate circulating
has also used laxatives in the past “to not gain weight when estrogen. A comprehensive plan is developed that includes
I eat too much.” psychological counseling, dietary modi cation, physical
therapy, consultation with a nutritionist, and requent ollow-
Question 14.2.3 Which o the ollowing ndings is NOT up. Over the next several months, the patient recovers rom
classically ound in bulimia nervosa? her injury, and returns to running on a modi ed schedule.
A) Loss o dental enamel. However, she now complains o heavy, pain ul menses and
B) Enlarged parotid glands. wants to go on the “shot” to stop them.
C) Metabolic acidosis.
D) Skin changes over the dorsum o the hands. Question 14.2.5 In your counseling about birth control
E) Maintenance o a signi cantly low body weight. options, which o the statements below is FALSE?
A) T e birth control pill can be used to treat irregular menses o
Answer 14.2.3 The correct answer is “E.” Loss o dental enamel, the emale athlete triad and will also increase bone deposition.
skin changes over the dorsum o the hands, and enlarged B) T e e ect o Depo-Provera (medroxyprogesterone) is to
parotid glands may be seen as a result o repetitive sel -induced suppress the hypothalamic—pituitary—ovarian axis, creating
vomiting. While sel -induced vomiting would cause a metabolic a low estrogen state akin to peri-menopause.
alkalosis through loss o stomach acid, repetitive use o laxatives C) T e FDA issued a “Black Box” warning on Depo-Provera
can cause gastrointestinal losses o bicarbonate, resulting in a that states “prolonged use . . . may result in loss o bone
metabolic acidosis. Individuals with bulimia nervosa o en density that may not be completely reversible a er discon-
maintain a normal weight, while anorexia nervosa has strict tinuation o the drug.”
diagnostic criteria requiring maintenance o signi cantly low D) T e risk o uture racture using Depo-Provera is unknown,
body weight, o en less than 85% o the ideal body weight. as bone density is an incomplete measure o bone strength,
and remodeling and recovery are signi cant.
Your patient also relates that she began having her peri- E) Use o a Long Acting Reversible Contraceptive (LARC)
ods around age 12. While they were initially irregular, they should be o ered as rst line contraceptive choice.
seemed to become more regular prior to starting high school.
However, as she became more involved in high school sports, Answer 14.2.5 The correct (and “ alse”) answer is “A.” T e
her periods became more irregular. During her o -season, other statements are all true. Starting a combined estrogen/pro-
her menses are “more regular,” though she cannot predict gesterone contraceptive pill will restart a menstrual cycle but
when they will occur. She recalls that her last period was may not increase bone density. As to “E,” an American Acad-
about 7 weeks ago. emy o Pediatrics (AAP) policy statement rom September 2014
recommends that a LARC be considered as a rst line contra-
Question 14.2.4 In evaluating your patient’s menstrual ceptive option in adolescents, including intrauterine device
dys unction, what would be your next course o action? (IUD) or subdermal implant. T ese can provide 3 to 10 years
A) Obtain serum LH, ollicle-stimulating hormone (FSH), and o contraception and are e ective and sa e orms o birth con-
estradiol levels. trol. Concerns about an increase in pelvic in ammatory disease
B) Order an abdominal and pelvic ultrasound. (PID) with the IUD have been ound to be unwarranted.
CHAPTER 14 • Ad o l es Cen t Med iCin e 399

likely to make V than Iowa). She brings you her DEXA scan
In your routine anticipatory guidance, you discover that she results that are consistent with osteopenia. She looks up rom
avoids most dairy products due to a combination o prior her Instagram eed and asks, “So, what are we gonna do about
concerns about at content and lactose intolerance. that, Doc?”

Question 14.2.6 What do you recommend or her daily Question 14.2.8 You scour the literature and recommend:
intake o elementary calcium? A) Alendronate.
A) 900 mg. B) Calcium and vitamin D.
B) 1,100 mg. C) Vigorous weight-bearing exercise.
C) 1,300 mg. D) Dehydroepiandrosterone (DHEA).
D) 1,700 mg. E) All o the above.
E) 2,100 mg.
Answer 14.2.8 The correct answer is “B.” Alendronate has no
Answer 14.2.6 The correct answer is “C.” While the absolute proven bene t in adolescent osteopenic emales. Recommend-
best intake o calcium or individuals is unknown, studies have ing exercise is the usual course or older patients with osteo-
shown positive calcium balance or adolescents with an intake penia, but you need to be care ul in the adolescent with weight
o 1,200 to 1,500 mg daily. In its 2010 report, the Institute o concerns who may exercise excessively at baseline. DHEA is
Medicine (IOM) set 1,300 mg/day as the “adequate” dietary investigational and has not been shown to increase bone mineral
intake or boys and girls 9 to 18 years o age. T is guideline was density. Stick with the standard o care: calcium in the daily doses
set to meet the needs o 95% o healthy children, with the upper as recommended above and vitamin D 600 IU daily (note that
limit o calcium intake set at 2,500 mg/day. this recommendation is a change in the 2010 IOM guideline).
For most persons, 1,300 mg/day o calcium intake can be
accomplished with our servings o dairy products (8 oz o milk = Objectives: Did you learn to . . .
8 oz o yogurt or cottage cheese = 1 inch cube o cheese) plus • eva ua g pa a ru r?
a varied diet that includes other calcium-rich oods (e.g., broc- • i y h “ ma a h r a ?”
coli, collard greens, and turnip greens). It is generally pre erable • R c g z h mp r a c ca c um a v am d ak
to achieve intake o calcium via diet, including orti ed oods, a p r pr v a c c ?
because oods provide multiple nutrients that are important
or bone health, such as phosphorus and magnesium. Despite
QUICK QUIZ: Ad o l es Cen t At Hl et es
widespread non- and low- at dairy options and numerous
supplements, concern over at intake has resulted in an average
adolescent intake o only 700 to 800 mg/day or girls and about For adolescent athletes, what is the number one cause o sudden
1,000 mg/day or boys—and yet, we have an obesity epidemic. cardiac death (SCD)?
Ironic but not unny. A) Mar an syndrome.
B) Coronary artery disease.
Question 14.2.7 On average, what percentage o total body C) Congenital mal ormation o coronary arteries.
mineral content has a young woman deposited by the time D) Hypertrophic cardiomyopathy.
she reaches 12 years o age? E) Long Q syndrome.
A) 20% to 30%.
B) 40% to 50%. The correct answer is “D.” Hypertrophic cardiomyopathy is an
C) 70% to 75%. autosomal-dominant trait with highly variable penetrance that
D) 80% to 85%. results in asymmetric septal wall hypertrophy. T is may cause
E) 95% to 100%. unctional aortic out ow tract obstruction, as well as predis-
pose the athlete to arrhythmias. Un ortunately, this condition
is o en asymptomatic prior to the terminal event and screening
Answer 14.2.7 The correct answer is “D.” Research suggests
tests such as electrocardiographs and echocardiograms have not
that by age 12, a young woman has reached approximately 83%
been proven e ective at early detection o this condition. Aber-
o her peak bone mineral content, with 50% deposition hap-
rant coronary arteries are the second leading cause o SCD in
pening rom the time o “peak height velocity,” which is pre-
young athletes. Mar an syndrome and long Q syndrome are
menarchal through 1 year post-menarche. T e ability to absorb
less common.
calcium rom the diet is also enhanced during this period.
Rates o deposition begin to decline approximately 2 years post-
menarche and no signi cant gains are seen a er the age o 17.
CASE 14.3
T ese statistics emphasize that osteoporosis, while mani esting in As part o your group’s community outreach, you are partici-
older adults, is truly an issue o adolescent preventive medicine. pating in a sports physical screening (somehow your partner
got to throw out the rst pitch at a baseball game, and you
Your patient returns rom a high-powered sports medicine got this . . . we’re guessing you must be new). One o the stu-
clinic in Palm Springs (or Laguna Beach . . . or anywhere more dents is noted to be hypertensive with a blood pressure o
400 FAMil Y Med iCin e eXAMin At io n & Bo ARd ReVieW

143/95 mm Hg. T ere is no amily history o sudden death D) Full participation with no required ollow-up based on
or early heart attack. He has never experienced any symp- reassuring history and examination.
toms with exercise. His parents indicated on the orm that E) Full participation with blood pressure re-check prior to the
they did not recall any history o previous heart murmur or next competitive sport season.
high blood pressure. T e remainder o the examination is
unremarkable except or a I/VI systolic murmur. You ask the Answer 14.3.2 The correct answer is “A.” It is important to
athlete to per orm a Valsalva maneuver by holding his breath remember that the diagnosis o hypertension cannot be made at
and bearing down while cardiac auscultation is repeated. a single screening visit. T e screening may be a ected by patient
anxiety (“white coat hypertension” or standing hal naked in ront
Question 14.3.1 Which o the ollowing correctly describes o all o his peers), recent ca eine ingestion, drugs, or a number
the relationship between murmur intensity and the likely o other actors (including a common occurrence o schedul-
type o murmur? ing examinations right a er practice). Most recommendations
A) Valsalva maneuver increases ow murmur, decreases out ow include three separate recordings o blood pressure on di erent
tract obstruction murmur. occasions a er several minutes at rest, sitting com ortably.
B) Valsalva maneuver increases both ow murmur and out ow Because exercise is bene cial or blood pressure control,
tract obstruction murmur. continued aerobic activity should be encouraged while the
C) Valsalva maneuver decreases both ow murmur and out ow serial blood pressure readings are obtained. Debate exists as to
tract obstruction murmur. recommendations regarding static exercises, such as weightli -
D) Valsalva maneuver decreases ow murmur, increases out ow ing. While li ing o heavy weights can exacerbate high blood
tract obstruction murmur. pressure, there is some evidence to suggest that use o light-to-
moderate weights with repetition is bene cial. Certainly, the
Answer 14.3.1 The correct answer is “D.” T e Valsalva maneu- presence o any additional symptoms with exercise, such as
ver decreases venous return to the heart, resulting in decreased headache or lightheadedness, would require more signi cant
diastolic lling. You would expect this to cause decreased ow limitations in activity.
through the out ow tract and thus a so er ow murmur. How-
Objectives: Did you learn to . . .
ever, the decreased ow exacerbates the unctional out ow
• i y mp r a u a r a h pr -par c pa
tract obstruction o hypertrophic cardiomyopathy (there is less phy ca xam a ?
volume to push the septum out o the way resulting in a tighter
• eva ua a a c ah w h va b
unctional stenosis), resulting in a louder murmur. In summary, pr ur ?
benign f ow murmurs will decrease in intensity with Valsalva
• eva ua a car ac murmur r v a pr -par c pa
maneuver, while the murmur o hypertrophic cardiomyopathy phy ca xam a ?
will increase with a Valsalva maneuver.
CASE 14.4
HELPFUL TIP:
A 15-year-old emale presents to your o ce or a well-
th y c cr c – cr c murmur hyp r-
adolescent examination. She is a healthy teenager with no
r ph c car my pa hy cr a y wh h
complaints—well, she has the usual teenager complaints (her
pa m v r m a up a upr gh p . s4
parents are stupid, the teachers don’t know anything, etc.).
may b h ar a w . Car ac au cu a h u b
You have incorporated into your routine a screening tool, the
p r rm b h up a upr gh r a p r par c -
Guideline or Adolescent Preventive Services (GAPS) ques-
pa phy ca .
tionnaire, one o many such tools available. T e questions
cover all areas o adolescent development and are designed
so that areas o concern are easily identi ed.
Reassured by a murmur that disappears with Valsalva
maneuver and an otherwise unremarkable examination, you Question 14.4.1 What are the three leading causes o mor-
are now aced with an adolescent athlete with an elevated tality or adolescents?
blood pressure con rmed by manual re-testing. A) Leukemia, suicide, accidental drowning.
B) Childhood cancers, perinatally acquired HIV, suicide.
Question 14.3.2 What is the best recommendation or this C) Congenital mal ormations, childhood cancers, suicide.
athlete regarding sports participation and ollow-up o his D) Accidental injury, homicide, suicide.
elevated blood pressure? E) Childhood cancers, SCD, suicide.
A) Quali ed participation pending serial blood pressure checks
over the next 3 weeks. Answer 14.4.1 The correct answer is “D.” Accidental injury,
B) Complete disquali cation or 2 months, ollowed by return homicide, and suicide are the three leading causes o death or
i ECG and echocardiogram are normal. adolescents and should be addressed during preventive health
C) Disquali cation rom competition only or 1 month, but visits. As a category o accidents, motor vehicle atality is the
allowed to practice once seen by a nephrologist. leading cause o death to teenagers, representing over one-third
CHAPTER 14 • Ad o l es Cen t Med iCin e 401

o all deaths. A high number o teens report unsa e driving Question 14.4.3 What would your next step be?
behavior: 41.4% o teens have reported texting or emailing while A) Call the police and report the illegal use o substances by a
driving; and while most states have made this illegal, it is still minor.
important to discuss with your adolescent patients. Accidental B) Reassure the patient about her con dentiality rights, counsel
injuries o en involve the use o alcohol or other substances in her to quit using drugs, and see her back annually.
combination with motor vehicle operation or other risk-taking C) Begin the patient on an SSRI and see her back in a month.
behaviors. Ask about personal use o alcohol and i they are D) Negotiate next steps to guarantee the patient’s sa ety and
ever passengers o another teenager (or adult) who drives while consider counseling re erral.
impaired or intoxicated. T e greater the number o teens in E) Obtain a nonconsented urine drug screen to document
the car o a teen driver, the greater the likelihood o an acci- recent use o substances.
dent. T is argues or the law in some states that restrict the
number o passengers in a teen driver’s car. Answer 14.4.3 The correct answer is “D.” Most states’ laws, as
well as HIPAA, guarantee teenagers con dentiality, to be broken
You also noticed that your patient checked “yes” or a history only when there is an issue o imminent danger to sel or others
o use o alcohol and marijuana. or physical/sexual abuse. It is recommended that the patient be
involved and consent to who and how the in ormation is shared.
Question 14.4.2 What is the approximate requency o Rarely is there a need to involve police, unless there is an issue
li etime use o these substances among 9th to 12th grade o abuse requiring a report or concern over sa ety i the patient
students according to the Centers or Disease Control and is returned to the home environment.
Prevention (CDC’s) 2013 Youth Risk Behavioral Survey? “C” is incorrect. I an SSRI is started, close supervision is
A) Alcohol 10% to 20%; marijuana 5% to 10% required or adolescents. Studies suggest increased suicidal-
B) Alcohol 30% to 40%; marijuana 10% to 20% ity in teenagers treated with SSRIs. T ere are also increased
C) Alcohol 50% to 60%; marijuana 20% to 30% attempts at suicide but not an increase in success ul suicide. T e
D) Alcohol 60% to 70%; marijuana 30% to 40% American Academy o Child and Adolescent Psychiatry website
E) Alcohol 70% to 80%; marijuana 40% to 50% has resources outlining patient and parent in ormation about
the risks and bene ts o pharmacologic treatments. Cognitive
Answer 14.4.2 The correct answer is “D.” T e survey ound behavioral therapy (CB ), along with talk therapy, has been
66.2% o adolescents reporting use o alcohol at some point in shown to be the best rst-line treatment or adolescents with
their lives, and 40.7% reporting use o marijuana. In addition, mood disorders. In any case, close ollow-up o a depressed
41.1% have tried cigarettes. Alcohol use has been ound to be adolescent (weekly visits or phone calls) is recommended.
slightly higher in emales at 67.9% compared to 64.4% in males.
HELPFUL TIP:
HELPFUL TIP: A F bruary 2016, h Us Pr v a v s rv c
scr g ra c ub a c u cu t a k F rc w r c mm cr g rm -
h “CRAFFt ” qu , v p by h “C r r r r ag r ag 12 18 as long as there are
A c s ub a c Abu R arch” (C As AR), Ch - adequate resources available to aide in diagnosis, treat-
r ’ H p a B . s m ar h “CAGe” r ment, and follow-up. t h y pr v h a gra B prac-
a u ac h a m , h r a r: c r c mm a . th r u c v c
• C—Hav y u v r r a CAR r v by m a h p r cr g ch r ag 11 y ar
( c u g y ur ) wh wa “h gh” r ha b u g ag r y u g r.
ac h r rug ?
• R—d y u v r u a c h r rug Rel AX,
b r ab u y ur , r ? HELPFUL TIP:
• A—d y u v r u a c h / rug wh y u ar by ss Ri ar ha c v r p a r c/a c
y ur , Al o n e? pr .t h nnt b pa 10. F u x-
• F—d y u v r Fo RGet h g y u wh u g ha h m av rab b /r k pr a -
ac h r rug ? c a r c mm a r - h rapy by
• F—d y ur am y r FRien d s v r y u ha y u m pub h gu .
h u cu w y ur r k g r rug u ?
• T—Hav y u g t Ro UBl e wh y u w r
u gac h r rug ?
HELPFUL TIP:
t h Am r ca Aca my P a r c ha a p
a m aga h u c ur rug
A er eliciting a positive response to questions about drug cr a a “ cr g” u h ack accuracy
use, you ask the patient about depression, and she states that a h v a pa ru .
she has been eeling “down” or several months now.
402 FAMil Y Med iCin e eXAMin At io n & Bo ARd ReVieW

D) reat with oral steroids to suppress the response and promote


(ONE MORE) HELPFUL TIP:
quicker return to wrestling.
A u y pub h Lancet u ha a c -
E) reat with topical anti ungal medication and allow return to
harm (cu g, bur g, c.) ccur ab u 10%
wrestling.
a c . H w v r, y u ca r a ur par ha
h c 3% a a y u g a u (M ra P, a.
Answer 14.5.2 The correct answer is “C.” Cutaneous herpes,
Lancet. 2012).
also re erred to in wrestling as herpes gladiatorum, is a highly
contagious illness. “B” is incorrect because topical acyclovir is
Objectives: Did you learn to . . . not e ective in treating herpes simplex. Most state high school
• i ya mp y u u cr g rr u a - athletic associations have speci c rules regarding the treatment
c xam a ? o cutaneous disorders in wrestling, including herpes, tinea cor-
• R c g z h mp r a c cr ga r a g pr - poris, and impetigo. With regard to herpes simplex, all lesions
a c ? must be ully crusted over and the athlete must have had at
• d cr b c mm cau ah a c ? least 3 days o oral antiviral therapy prior to returning to com-
• U r a h c mm m xp r c u ac h , petition. “D” is incorrect because oral steroids would suppress
bacc , a mar jua a? the athlete’s immune response, which would allow or urther
• d cr b cr ga r am p rm r r spread o the lesions.
a ub a c abu ?
Your next patient is a 12-year-old male who is here or a
sports physical. You review your patient’s chart and scan his
CASE 14.5 immunizations. T e patient’s mother remembers her son
Your a ernoon has a number o patients with sports-related receiving his “kindergarten shots;” he has had no immuniza-
complaints. A 14-year-old wrestler has arrived in the o ce tions since. His records reveal the ollowing vaccines: 5 doses
complaining o a rash on his le shoulder. T e rash appears o diphtheria–tetanus–acellular pertussis (D aP), 4 doses o
to consist o several small lesions with a vesicular appearance injectable polio vaccine (IPV), 4 doses o Haemophilus Inf u-
but no purulence and minimal erythema. T ere is neither ten- enzae ype B (Hib), 3 doses o hepatitis B; 4 doses o PCV 13,
derness to palpation nor tactile warmth in the a ected area. 2 doses o measles–mumps–rubella (MMR), and 2 doses o
varicella vaccine.
Question 14.5.1 What would be your best test to con rm
your diagnosis o these lesions? Question 14.5.3 Which, i any, immunizations would you
A) Microscopic evaluation with potassium hydroxide (KOH) of er today?
preparation o the contents o one o the lesions. A) None. T e patient is up-to-date or all immunizations.
B) Gram stain o the contents o one o the lesions. B) Conjugate pneumococcal vaccine.
C) Culture o the contents o one o the lesions. C) Hepatitis A/B vaccine, conjugate meningococcal vaccine,
D) zanck smear o the contents o one o the lesions. IPV.
D) Hepatitis A vaccine; etanus, diphtheria and pertussis ( dap);
Answer 14.5.1 The correct answer is “D.” A zanck prep will conjugate meningococcal vaccine and start o the HPV series.
give you immediate in ormation about whether this represents E) Human papillomavirus (HPV) vaccine, dap, varicella
herpes simplex. T is is important in ormation when deciding vaccine.
whether or not this patient can return to wrestling. HSV PCR
assays are a more sensitive method to con rm HSV in ection in Answer 14.5.3 The correct answer is “D.” Universal vaccina-
clinical specimens obtained rom genital ulcers and mucocuta- tion or hepatitis B is included in the primary immunization
neous sites and should also be obtained. However, these typically series or in ants, so most adolescents have been vaccinated.
take several days to return results. However, it is still important to make sure this is up-to-date.
Since adolescence is a time when experimentation with drugs
T e lesions have appeared within the past day, and your labo- and sexual activity becomes more prevalent, vaccination or
ratory testing con rms that the diagnosis is herpes simplex. hepatitis B becomes even more relevant. Hepatitis A vaccine is
Your patient does not recall any previous lesions like these. now being required in many states be ore entry to school and
He denies any systemic symptoms. is universally recommended or all children ages 12 months to
2 years. A catch-up vaccination schedule is now recommended
Question 14.5.2 What would be the best course o treat- or ages 2 to 18 years. For those not up-to-date with their hepa-
ment or this patient? titis B vaccination, a combined hepatitis A/B vaccine exists.
A) Cover the involved area to allow return to wrestling. Most children receive their last D aP prior to entering pre-
B) reat with topical acyclovir and return to wrestling when school or kindergarten (age 4–5 years). Beginning in 2005, the
lesions have crusted over. CDC recommended giving the tetanus booster with pertussis
C) reat with oral acyclovir and return to wrestling when ( daP) at entry to middle school/junior high (ages 11–12) in
lesions have crusted over. place o the tetanus booster ( d).
CHAPTER 14 • Ad o l es Cen t Med iCin e 403

For protection against some serotypes o Neisseria meningiti-


dis, the CDC recommends universal vaccination with the conju- CASE 14.6
gate meningococcal vaccine (e.g., Menactra) or teens, including A 15-year-old emale returns or her annual well-child check
a booster dose between ages 16–18 years. and sports physical. She is well known to your clinic, and over
T e CDC recommended HPV vaccination or boys in 2011. the years she has steadily gained weight. Her parents and a
E ective March o 2015, the CDC recommends 9-valent HPV, younger sibling are overweight, and now she has become
4-valent HPV, or 2-valent HPV or routine vaccination o obese by measure o the pediatric BMI percentages. In recent
emales 11 or 12 years o age and emales through 26 years o years, you have become more diligent with watching the BMI
age who have not been vaccinated previously or who have not on her growth curve. She is aware o her weight problem, yet
completed the 3-dose series. Further, the CDC recommends has not voiced any plan or attempting weight loss.
the 9vHPV or 4vHPV or routine vaccination o males 11 or
12 years o age through 21 years o age who have not been vac- Question 14.6.1 What de nes “obesity” in adolescence?
cinated previously or who have not completed the 3-dose series. A) A BMI in the 85th to 94th percentile or age-speci ic
T is age cut-o may be extended out to 26 years old or men measures.
who are immunocompromised or have sex with men and have B) A BMI ≥ 95th percentile or age-speci c measures.
not been previously vaccinated. “E” is incorrect because he has C) T e child has the appearance o being heavy.
already had chicken pox. D) T e child complains o being at or overweight.
E) BMI > 35 kg/m 2.
Question 14.5.4 Which statement does NOT accurately
describe how the conjugate meningitis vaccine (Menactra) Answer 14.6.1 The correct answer is “B.” A BMI at or above
dif ers rom the older polysaccharide meningitis vaccine the 95th percentile is considered “obese.” A BMI in the range o
(Menomune)? 85th to 94th percentiles is considered “overweight.” T e use o
A) Menactra elicits a -cell immune response that gives long- BMI allows or an objective measure, and although not per ect,
term immunity and allows or herd immunity. can give a picture on the growth curve to patients and parents
B) Menactra protects against all the types o N. meningitidis: A, where they stand relative to age-related norms. Strict BMI num-
B, C, Y, W-135. bers (as in “E”) are not applicable to children and adolescents.
C) Menomune does not elicit a booster e ect, may only be A subjective concern about weight (“D”) is important, but does
given once, and is recommended only at the onset o high- not necessarily always warrant urther evaluation.
risk activity, such as living in dorms or being a military
recruit. Question 14.6.2 Her BMI is now at the 97th percentile,
D) Menactra should be o ered at entry to middle school/junior having jumped rom the 86th percentile last year. What
high school (11–12 years) with a booster dose recommended additional work-up is needed?
between age 16 and 23 years. A) Fasting plasma glucose level.
B) Fasting lipids.
Answer 14.5.4 The correct answer is “B.” Un ortunately nei- C) Renal unction testing with BUN and creatinine.
ther Menomune nor Menactra protect against serogroup ype B. D) Liver unction testing with AS and AL .
T e other choices are correct. N. meningitidis ype B is endemic E) All o the above.
in North America and up until 2014, no such vaccine was avail-
able. In 2014 and 2015, the FDA approved two meningococcal Answer 14.6.2 The correct answer is “E.” Once a child reaches
ype B vaccinations (3 dose MenB-FHbp ( rumenba®) and 2 a BMI at the 85th to 94th percentiles, a lipid panel is recom-
dose MenB-4c (Bexsero®). T e ACIP in October 2015 provided mended to evaluate or hyperlipidemia and screening should
a grade B recommendation that MenB vaccination may be given be done or early onset type 2 diabetes. When the BMI reaches
in addition to Menactra and other indicated age-based vaccina- the 95th percentile, renal unction and blood pressure should
tions (pre erably at a di erent anatomic site) between ages 16 be monitored and liver unction should be checked (looking
and 23 with pre erable dosing at 16 to 18 years to provide peak or signs o atty liver). Fatty liver disease is possible at an early
desired immunologic e ect in the group that is at most risk. age and is reversible i weight loss can be achieved. Close atten-
tion to amily history is essential. I amily members are obese
HELPFUL TIP:
or have obesity-related comorbidities, early intervention and
R v w h vacc r c mm a c h y ar
evaluation are even more crucial. For emales who have started
up a r qu y by h Cd C; vacc r c mm a-
menses, it is important to assess or any changes in menstrual
ca b u a h p://www.c c.g v/vacc /
patterns.
ch u /.
Question 14.6.3 Which o the ollowing is NOT a recom-
mended early childhood intervention to prevent and/or
Objectives: Did you learn to . . . treat obesity?
• i y h rp g a a rum a cr b r am ? A) Encouraging breast- eeding.
• R c mm vacc appr pr a ra c ? B) Decreasing screen time o electronic devices.
404 FAMil Y Med iCin e eXAMin At io n & Bo ARd ReVieW

C) Planning ewer amily meals. Question 14.6.5 Which o these medications would you
D) Restricting calories rom sweets and ast oods. recommend now as a rst-line therapy or weight loss?
E) Improving e orts or exercise and outdoor activity. A) opiramate ( opamax).
B) Bupropion (Wellbutrin).
Answer 14.6.3 The correct answer is “C.” Helping with ood C) Orlistat (Xenical).
preparation, and sitting down as a amily or meals, can lead D) Sibutramine (Meridia).
to consumption o more nutritious and lower calorie dense E) None o the above.
oods. Breast- eeding can lead to protection against obesity
even into the teenage years. Formal recommendations suggest Answer 14.6.5 The correct answer is “E.” No medication is
limiting V, tablet, phone, computer use, and video games approved or children or adolescents as a rst-line therapy. Anti-
to less than 2 hours per day. Fast ood, sweets, and desserts depressants are not help ul i used primarily or weight loss. I
need to be limited to small servings or ew servings per week. there is comorbid depression, which is more common in obese
Finally, 60 minutes o exercise should be encouraged every children, a “weight loss” or “weight neutral” antidepressant may
day—doing a variety o activities will keep children more be appropriate. Antiepileptic medications have not proven help ul.
engaged and interested. Sibutramine was pulled o the market in 2010 due to troubling
cardiac side e ects, but was not approved or use in children any-
way. Orlistat is approved or children age 12 and older. Yet, the
HELPFUL TIP:
bene ts are limited to an additional 2 to 3 kg o weight loss. Side
i ch r b c m v rw gh r b , r c
e ects are o en intolerable and include loose, oily stools and stool
w gh h u b mp m .F r k w h a
urgency (just mention the phrase “ ecal incontinence” to a teen-
BMi a h 85 h 94 h p rc , h g a w gh
ager and see how ar you get). Experts agree that orlistat is only
ma a c u h y r ach a BMi a h 84 h p rc -
appropriate or adolescents with comorbidities caused by obesity.
r . W gh a a ra 1 b p r w k r c-
mm r k wh hav BMi a h 95 h 99 h
p rc .F r h ch r a r ab v a BMi h HELPFUL TIP:
99 h p rc , h r c mm w gh 2 b t h r k b c m g v rw gh ca cr a by 4%
p r w k. r ach m h br a - g up ag 9 m h .

HELPFUL TIP:
During the examination o the patient, you notice more acne
W gh a c c w k y a cr a
than the past, some coarse acial hair, and dark skin pigmen-
car va cu ar a a a a u (Ju a a M, a.
tation around the base o her neck.
N Engl J Med. 2011).
Question 14.6.4 What is another nding you may expect to
see with urther questions and tests?
A) Normal menses. HELPFUL TIP:
B) Low ree testosterone level. M ab c y r m ch h k w r iQ
C) Low DHEA sul ate (DHEA-S). c r a mpa r c g . Wh h r m ab c y -
D) Hypoglycemia. r m cau a a wh h r h r v r b y
E) Abnormal hypothalamic–pituitary axis hormones. k w (Yau Pl , a . Pediatrics. 2012).

Answer 14.6.4 The correct answer is “E.” She is showing signs Objectives: Did you learn to . . .
o possible polycystic ovarian syndrome (PCOS). T is can be • d “ v rw gh ” a “ b y” a a c ?
associated with elevated prolactin, low SH, and a marked ele- • i y r k ac r a c a w h b y a a c ?
vation o FSH relative to luteinizing hormone (LH). PCOS can • Ma ag a a c w h a w gh pr b m?
be associated with early menarche, dysmenorrhea, and amenor-
rhea. A pregnancy test should be done in any case o amenor-
rhea. PCOS is also associated with elevations o testosterone,
CASE 14.7
DHEA-S, and androstenedione. I there are other signi cant You have watched your patient trans orm over the years rom
physical examination ndings such as elevated blood pressure, a rather normal-appearing boy into a very large, muscular,
purple striae, and a “bu alo hump,” consider screening or and intense young man. He is now 17 years old, and he has
Cushing syndrome. gained almost 45 lb in the last year alone as he prepares or
his senior year o ootball and attempts to attract the atten-
Her obesity is discussed openly during the o ce visit. As tion o college recruiters. Without a doubt he has worked
expected, she is ashamed and embarrassed about her weight very hard on the eld and in the weight room, but you also
gain. She admits to not knowing what to do and she asks i must raise the question o whether or not he has used some
anything can be done to quickly lose weight. kind o supplement to help him change so drastically.
CHAPTER 14 • Ad o l es Cen t Med iCin e 405

Question 14.7.1 Which o these substances is NOT prohib- short-term or long-term bene ts. Ca eine has only a transient
ited by high school athletic associations or the NCAA? e ect to help sharpen ocus and intensity or brie periods o
A) Erythropoietin. activity. Creatine can improve strength or those sports that
B) Creatine. require very short bursts o intense e ort. Electrolyte replace-
C) Androstenedione. ment drinks (or tablets) and sodium bicarbonate can help or
D) Ephedrine. more prolonged endurance activities where dehydration and
E) DHEA. resulting electrolyte imbalances are common.

Answer 14.7.1 The correct answer is “B.” Creatine is allowed Your patient denies using “steroids,” but does know o older
or use as a dietary supplement or athletes. However, the guys in the gym where he trains who have routinely used
American College o Sports Medicine (ACSM) does not recom- anabolic steroids.
mend the use o creatine in any athlete less than 18 years old.
An estimated 8% o children 14 to 18 years old take creatine Question 14.7.4 Besides the obvious added muscle girth,
routinely. T e NCAA does not allow colleges to provide ath- what are some other potential bene ts o anabolic steroids
letes with creatine, but athletes can take it i they choose. All the or athletes?
other listed supplements are prohibited or use as per ormance- A) Improved lipid pro le.
enhancing supplements. Other prohibited substances include B) aller stature.
anabolic steroids, pseudoephedrine, and blood trans usions. C) Anxiety relie .
D) Lower blood pressure.
Your patient does admit to using regular protein shakes and E) None o the above.
taking creatine. He denies the use o any illegal or prohibited
substances and says he would never do anything dangerous. Answer 14.7.4 The correct answer is “E.” T ere are no other
During his examination, his blood pressure is 146/92 mm Hg clear bene ts o anabolic steroids i used or athletics only. Poten-
and his lab results show normal electrolytes, normal SH, tial negative e ects include lower HDL, elevated blood pressure,
normal urinalysis, BUN 22 mg/dL, and creatinine 1.6 mg/dL le ventricular hypertrophy, gynecomastia, aggressive or even
(marked elevation or age 17). suicidal behavior, azoospermia, virilization o emales, prema-
ture physeal closure in adolescents that could lead to shorter
Question 14.7.2 What is a possible side ef ect rom the use stature, and acute myocardial in arction. Approximately 33%
o creatine? o anabolic steroid users are not even athletes and use the sub-
A) Gastrointestinal discom ort. stance to help improve physical appearance. Estimates in high
B) Edema. school athletes show steroid use to be about 4% in males and
C) Muscle cramps. about 2% in emales. T e temptation to use the illegal substance
D) Acute renal insuf ciency. is driven by pressures to be better than peers, to ul ll one’s “ ull
E) All o the above. potential,” to live up to sports idols, and to help acquire college
scholarships and/or sports contracts at a young age.
Answer 14.7.2 The correct answer is “E.” All are potential
Objectives: Did you learn to . . .
adverse e ects rom creatine use. Most importantly, i used care-
• d cr b h f c m c mm p r rma c - ha c g
lessly, creatine can lead to acute renal insuf ciency. Creatine is rug ?
excreted via the kidneys, and i overconsumed in the setting o
dehydrating exercise, it can cause renal injury. Seventy- ve per-
cent o adolescent athletes who take creatine are either unaware
CASE 14.8
o the proper dose or knowingly take too much. I used, plenty A 14-year-old emale comes into the o ce with a chie com-
o water is required to help avoid troubling side e ects. plaint o headache. She says the headache began a er her soccer
game yesterday. She recalls striking her head against an oppo-
Question 14.7.3 All o the ollowing substances can provide nent when trying to head the soccer ball. During the remain-
potential athletic per ormance bene ts, except: der o the game, she began to eel a headache, and the harder
A) Human growth hormone (HGH). she ran, the more “woozy” she elt. She denied any loss o con-
B) Ca eine. sciousness, vomiting, visual symptoms, or neck pain. oday,
C) Creatine. she also has poor concentration and eels excessive atigue.
D) Electrolyte replacements.
E) Sodium bicarbonate. Question 14.8.1 Which o the ollowing is NOT a eature o
an acute concussion?
Answer 14.7.3 The correct answer is “A.” HGH has no proven A) Caused by an impulsive orce transmitted to the brain.
bene ts or enhancing sports per ormance. Other substances B) Symptoms usually resolve spontaneously.
with no proven bene ts include amino acids, beta-hydroxy-beta- C) T ere is a unctional disturbance to the body.
methylbutyrate, androstenedione, DHEA, chromium, and iron D) Neuroimaging always shows abnormalities.
(when not iron de cient). T e other substances can provide E) Clinical symptoms are highly variable and inconsistent.
406 FAMil Y Med iCin e eXAMin At io n & Bo ARd ReVieW

Answer 14.8.1 The correct answer is “D.” T ere may be no Question 14.8.3 In which case o head injury would neuro-
evident structural brain injury with an acute concussion. Evi- imaging be appropriate?
dence suggests that multiple concussions, or even lessor repeti- A) Ongoing mild headache or several days.
tive head injuries (such as with soccer “heading”) can eventually B) Poor memory o the recent game and the events o the injury.
lead to structural tissue damage. T e symptoms o concussions C) Any new seizure activity.
have a rapid onset and can vary rom short-lived impairment D) Loss o consciousness or less than 30 seconds.
to prolonged post-concussive symptoms. Studies are showing E) Mild irritability, depressed mood, and insomnia.
that younger athletes tend to have more severe symptoms and
recover more slowly than older, college-aged athletes. T is is Answer 14.8.3 The correct answer is “C.” Seizure activity a er
likely due to the more dynamic brain growth in younger children head trauma or concussion requires imaging. Other symp-
and adolescents. toms that should prompt a head C include severe headache,
ocal neurological symptoms, repeated emesis, dif culty arous-
ing, slurred speech, prolonged poor orientation, concurrent
HELPFUL TIP: neck pain, signi cant irritability, loss o consciousness or over
C cu a a urba c bra u c 30 seconds, and any new neurologic or escalating symptoms. T e
cau by rauma h h a .i r qu r initial study is normally a head C , but i presenting a er 48 hours
c c u r am a. l gh h a , r - rom injury, a brain MRI is more suitable. Remember that the
a ,a au a, c. a r h a jury ar a p b vast majority o concussion cases will not require imaging.
g c cu . s ymp m p c cu y -
r m may c u h a ach , cu y c c ra g, Question 14.8.4 Which o the ollowing elements o past
“ zz ,” gh h a , au a, a gu , rr ab - medical history may be relevant or this adolescent patient
y, a x y a pr . s ymp m g ra y r v ollowing her concussion?
bu may b g v , p ca y h v r ag 50, A) Depression.
wh may v r r ur ba . B) Generalized anxiety disorder.
C) Attention-de cit hyperactivity disorder (ADHD).
D) Migraine headaches.
HELPFUL TIP: E) All o the above.
i a ah u a ah a jury r a har h , h a h-
h u b r m v r m h gam a a Answer 14.8.4 The correct answer is “E.” All o these condi-
a m h u b p r rm a ra yp - tions could be negatively a ected by a concussion. Concussions
b c cu . A mp a ar z u can cause a temporary mood disorder and certainly aggravate
h acu a m c cu a ub qu depression and anxiety. Commonly, concussions will cause
va ua h s p r C cu A m t decreased concentration and inattentiveness. eens who already
(sCAt -3) qu a r (Br J Sports Med. 2013;47: 259). have ADHD will likely have worsened symptoms and may
If a concussion is suspected, the athlete should NOT need modi cation o their treatment. T e headache o post-
be allowed to return to the game. concussive syndrome may overlap or intensi y an underlying
headache diagnosis. Concussion may act as a trigger causing an
increase in pre-existing migraines.
Question 14.8.2 Which o the ollowing treatments is key
or your patient with a concussion? Your patient is an accomplished soccer player and eels a
A) Early return to aerobic exercise—today i possible. lot o pressure to return to action. She wants to play in the
B) Ibupro en 800 mg every 8 hours as needed. next game, and her coach and teammates are asking i she’ll
C) Avoiding excessive sleep. be back to practice this a ernoon—which explains why your
D) Aspirin 325 mg daily as needed. waiting room is so ull o teenage girls wearing shin guards.
E) Full cognitive rest (put down that smartphone, tablet, and
social media!), even missing school i needed. Question 14.8.5 What is your best advice about her return
to practice and play?
Answer 14.8.2 The correct answer is “E.” T e initial and key A) “Go back to practice today, but take it slow and avoid ull
step in caring or patients with a concussion is ull physical speed play.”
and cognitive rest. Very o en, a return to the classroom can B) “ ry to participate in simple drills and just cheer on your
aggravate or prolong concussive symptoms. Once symptoms teammates.”
are clearly improving at rest, the student-athlete can get back C) “Go home and engage in cognitive rest (is “engage in” and
to class and slowly be re-acclimated to physical activity. Sleep “rest” an oxymoron?). May return to activity with graded
can provide a means o allowing the brain to rest and heal. No return to play regimen once symptoms resolve.”
medications have proven help ul in alleviating the e ects o a D) “Return to ull, unrestricted team play as your soccer team
concussion. NSAIDs, aspirin, or acetaminophen may help with needs you!”
headache or pain. E) “Do whatever you want.”.
CHAPTER 14 • Ad o l es Cen t Med iCin e 407

Answer 14.8.5 The correct answer is “C.” Her return to par-


ticipation needs to be a graded return to activity. Each step o Clinical Pearls
the recovery may be 24 hours. I she advances her activity to the C r g-ac gr v r b c rac p v (l ARC ), par-
next level and has recurrence o symptoms, then she needs to cu ar y n xp a ,a r - h rapy r c rac p
go back to the prior recovery step. T e rst stage o recovery is a c .
no activity—complete physical and cognitive rest. T e next step d a wa a c r ur p ay a r curr g a
is return to ull academic activities. Step three is light aerobic c cu .K p h a h u a pr cr b c g v r .
activity, ollowed by sport-speci c drills. Next, the player can o c ymp m hav mpr v , w y a w h a c
return to non-contact practice. I still symptom ree, then she r ur p ay a gra ua a pr cr b a h .
can return to ull speed, contact practice. Finally, she can return d cr r ub a c abu w h -c ur
to game action. I exertion causes any return o symptoms, then rug cr .
she is not ready and should return to the stage in which she was
R u y cr a c r ub a c abu a m
symptom ree. Even with normal neuropsychiatric tests, symp-
r r a qua upp r r urc ar ava ab .
tomatic athletes should not play.
T e above steps should be ollowed with the assistance o a Ru y vacc a a c w h t aP, c juga m g
health-care provider or a quali ed athletic trainer. T e athlete vacc , a HPV r .
cannot be put in the position o making the decision about how
ast to return to regular play. A er a rst concussion, the player
BIBLIOGRAPHY
needs to be watched or other episodes. wo or more concus-
sions have been shown to lower grade point averages and three Barlow SE and the Expert Committee. Expert Committee rec-
or more concussions may cause prolonged symptoms over ommendations regarding the prevention, assessment and
3 months in duration. treatment o child and adolescent overweight and obesity:
Summary Report. Pediatrics. 2007;120(S4):S164–S192.
Halsted ME, Walter KD, and T e Council on Sports Medicine
HELPFUL TIP: and Fitness Clinical Report: Sport-related concussion
i a c cu ccur , p a a w r ur in children and adolescents. Pediatrics. 2010;126(3):
p ay h am gam r v h am ay. t h r k r 597–615.
c - mpac y r m much gr a r a p ay r r - Ham P, Allen C. Adolescent Health Screening and Counseling.
ur p ay ar y a r a c cu .th m Am Family Physician. 2012;86(12):1109–1116.
k y ccur wh a p ay r r p r ymp m Herman-Giddens ME, et al. Secondary sexual characteristics
(m r c mm ma a h )a r u a gac - in boys: data rom the Pediatric Research in Of ce Settings
cu ,a h ha a h r h a rauma. t h rar Network. Pediatrics. 2012;130(5):e1058–e1068.
c - mpac y r m ca a c r bra va cu ar Jenkinson DM, Harbert AJ. Supplements and sports. Am Fam
c g , c r bra ma, a v a h. t h gr a - Physician. 2008;78(9):1039–1046.
r ka h ar h u r ag 20. Juonala M, et al. Childhood adiposity, adult adiposity and cardio-
vascular risk actors. N Engl J Med. 2011;365:1876–1885.
Kann L, et al. Youth risk behavior and surveillance—United
States 2013. MMWR Surveill Summ. 2014;63(4):1–168.
HELPFUL TIP:
Kliegman RM, et al., eds. Nelson extbook of Pediatrics.
“C g v r ” a r a c cu may - 19th ed. Philadelphia, PA: Elsevier Saunders Co.; 2011.
cu av a c r a g, c mpu r gam , t V,
Knight JR, et al. Validity o the CRAFF substance abuse
i r , mar ph u , h m w rk, ch a - screening test among adolescent clinic patients. Arch
a c ,a r v g. H r ’ h mp a: h bra Pediatr Adolesc Med. 2002;156(6):607–614.
hur , ’ v ru ar y. Pr v c - MacNeil JR, et al. Use o serogroup B meningococcal vaccines
cu b acc mp h by uca ah in adolescents and young adults: recommendations o the
a c ach , a by ar y r c g m r m r advisory committee on immunization practices, 2015.
ymp m . M u h guar hav a y pr v MMWR. 2015 October 23;64(41):1171–1176.
b pr v g c cu ,a h m McCrory P, et al. Consensus statement on concussion in sport:
h c c c cu ; h y ju pr v the 4th international con erence on concussion in sport
m r v r rauma c bra jur . (A ur held in Zurich November 2012. Br J Sports Med. 2013;47:
c agu ha , “A ha pa g ju m a 250–258.
ha y u ca h har r.”) McDowall JA. Supplement use by young athletes. J Sports Sci
Med. 2007;6:337–342.
Moran P, et al. T e natural history o sel -harm rom adoles-
Objectives: Did you learn to . . . cence to young adulthood: a population-based cohort
• d c cu a r c g z mp r a c a c study. Lancet. 2012;379:236–243.
ah ? Nwosu BU, Lee MM. Evaluation o short and tall stature in
• t r a a pa w h c cu ? children. Am Fam Physician. 2008;78(5):597–604.
408 FAMil Y Med iCin e eXAMin At io n & Bo ARd ReVieW

Ott MA, et al. Policy Statement: Contraception or Adoles- anner JM, et al. Standards or children’s height at ages
cents. Pediatrics. 2014;134(4):e1244–e1256. 2–9 years allowing or height o parents. Arch Dis Child.
Rao G. Childhood obesity: highlights rom the AMA expert 1970;45:755–762.
committee recommendations. Am Fam Physician. 2008; T ein-Nissenbaum JM, Carr KE. Female athlete triad syndrome
78(1):56–63. in the high school athlete. Phys T er Sport. 2011;12(3):
Scorza KA, et al. Current concepts in concussion: evaluation 108–116.
and management. Am Fam Physician. 2012;85(2):123–132. Yau PL, et al. Obesity and metabolic syndrome and unctional
Stephens MB. Preventive health counseling or adolescents. and structural brain impairments in adolescence. Pediatrics.
Am Fam Physician. 2006;74(7):1151–1156. 2012;130(4): 1–9.
Obstetrics andWomen’s Health
Elizabeth S. Cramer, Sandra R. Rosen eld-O’Tool, and Brigit E. Ray
15
CASE 15.1 Question 15.1.2 A ter reassuring her regarding the con
cerns o weight gain, you tell her about the additional
A 24-year-old nulligravida e ale resents or her annual potential bene t(s) o OCPs, which include:
exa ination. Her gynecologic history is re arkable or A) Improvement in acne
irregular enses, enstruating every 4 to 8 weeks. She would B) Decreased dysmenorrhea
like a ore reliable or o contrace tion (currently using C) Decreased menstrual ow
condo s) and would like to have redictable enses, but is D) Decreased risk o ovarian cancer
very concerned regarding weight gain with various contra- E) All o the above
ce tive ethods.
Answer 15.1.2 The correct answer is “E.” Besides these, addi-
Question 15.1.1 How would you counsel her regarding
tional potential bene ts o OCP use include regulation and pre-
weight changes and contraception?
dictability o menses, decreased anemia, decreased hirsutism,
A) Studies show there is no signi cant di erence in weight gain
and decreased risk o endometrial and colon cancers. (Note: the
o women initiating oral contraceptive pills (OCPs) versus
cancer risk reduction is based on epidemiologic studies, not
placebo
randomized controlled trials.)
B) Weight gain o 10 lb is expected during the rst year o use
with any type o OCPs
C) Weight gain o 10 lb is expected during the rst year o use
HELPFUL TIP:
with monophasic pills but not with triphasic ormulations
A ter starting OCPs, patients should ollow up within a
D) Weight loss o 10 lb is expected during the rst year o use
ew months or blood pressure checks, assuring compli-
with Depo-Provera (medroxyprogesterone acetate)
ance, etc.
Answer 15.1.1 The correct answer is “A.” Studies have shown
no signi cant weight gain with OCP use when compared with
placebo. rials have been conducted evaluating estrogen com-
A er urther discussion, she re orts that she has head-
ponents o 20 to 50 µg, both monophasic and triphasic. T ere
aches every 1 to 2 onths. She has never been evaluated or
is no evidence to su ort the re ise that tri hasic or u-
igraines, but re orts that her headaches are bilateral, oste-
lations o er i rove ent in weight changes. Depo-Provera
rior, throbbing, and relieved with slee and over-the-counter
has variable e ects on weight gain. Several studies have shown
edication. She denies associated aura, nausea, or ocal neu-
a weight gain o 3 to 6 kg in the rst year o use; however, other
rologic changes.
studies have shown no di erence in weight gain between Depo-
Provera and placebo.
Question 15.1.3 How would you counsel her regarding
OCPs and headaches?
HELPFUL TIP:
A) Headaches are an uncommon reason or discontinuation o
Recall that in 2004 the FDA placed a “black box warn-
OCPs
ing” on Depo-Provera or bone loss that “may not be
B) She should not use OCPs because they are contraindicated
reversible.” However, subsequent studies showed bone
in anyone with headaches
gain a ter stopping Dep-Provera. The prudent thing
C) She should use progestin-only pills
would be to avoid Depo-Provera in women at high risk
D) She can use OCPs, as it is hard to predict whether her head-
o osteoporosis.
aches will be a ected
409
410 FAMILY MEDICINE EXAMINATION & BOARD REVIEW

Answer 15.1.3 The correct answer is “D.” Although headache TABLE 15-1 BACKUP CONTRACEPTION WHEN
is a requently cited reason or women to discontinue OCPs, INITIATING METHODS
there is no strong correlation between headache requency and
Contraceptive Method Backup Method: Initiation a
intensity or most women. T ere is no evidence that the type
o progestin or amount o estrogen will alter the headaches, Copper-containing IUD None needed
except in women with menstrual migraines. Among women (Paraguard)
with migraines, headaches improve, worsen, or are unchanged Levonorgestrel-containing 7 days; only needed i > 7 days a ter
a er initiation o OCPs (help ul, right?). T ere is an increased IUD starting menses
risk o stroke in wo en with a history o seudotu or
Implant 7 days; only needed i > 5 days a ter
cerebri or igraines with aura or ocal neurologic changes.
starting menses
T ere ore, OCPs are CON RAINDICA ED in these s eci c
subgrou s o wo en. Injectable 7 days; only needed i > 7 days a ter
starting menses

Combined hormonal 7 days; only needed i > 5 days a ter


contraceptives starting menses
HELPFUL TIP:
Additional contraindications to combination OCP use Progestin-only pill 2 days; only needed i > 5 days a ter
include any previous thromboembolic event or stroke, a starting menses
history o estrogen-dependent tumor (e.g., some breast Note: Barrier methods or abstinence rom intercourse may be used during
cancers), active liver disease, pregnancy (although ac- the recommended backup interval.
a
cidental use o OCPs early in pregnancy has not been Any method may be initiated at any time i there is reasonable certainty
de initively linked to adverse outcomes), undiagnosed the woman is not pregnant, based on the criteria o the Centers or Disease
Control and Prevention.
abnormal uterine bleeding, women older than 35
Table adapted rom Klein DA, Arnold JJ, Reese ES. Provision o contraception:
years who smoke (due to increased risk o cardiovas- key recommendations rom the CDC. Am Fam Physician. 2015;91(9):625–633
cular disease), and first 3 weeks postpartum (2011 and Division o Reproductive Health, National Center or Chronic
CDC recommendation based on increased venous Disease Prevention and Health Promotion, Centers or Disease Control
thromboembolism risk in the immediate postpartum and Prevention (CDC). U.S. Selected Practice Recommendations or
Contraceptive Use, 2013: adapted rom the World Health Organization
period).
selected practice recommendations or contraceptive use, 2nd edition.
MMWR Recomm Rep. 2013;62(RR-05):1–60.

T e atient wants to know how long she should use a backu


ethod o contrace tion a er starting the co bination
estrogen/ rogesterone OCP. HELPFUL TIP:
Ri ampin, ri aximin and ri abutin are the only antibiotic
Question 15.1.4 You tell her the ollowing regarding using that reduces the e ectiveness o OCPs. There ore, there
a backup method: is no need or a backup method when giving penicillin
A) I she starts taking the pill during the rst 5 days o her men- or strep throat, or example.
strual period, no backup method is needed
B) I she starts taking the pill greater than 7 days a er the onset
o her period, she needs a backup method or 7 days HELPFUL TIP:
C) She should use a backup method or 2 months a er starting The thinking about methods o contraception has
the pill regardless o when she started it changed in the last several years. The IUD is becoming
D) T e OCP provides e ective contraception immediately and increasingly popular or nulliparous teenagers. IUDs
no backup method is needed clearly decrease pregnancy in this group when com-
pared to OCPs and do not seem to increase the risk o
Answer 15.1.4 The correct answer is “A.” T e old canard o STDs (N Engl J Med. 2014;371:1316). Nor do they result in
requiring a month o backup contraception a er starting OCPs, increased risky sexual activity.
implants, etc. has allen by the wayside. For combination estro- The American Academy o Pediatrics (2014) rec-
gen/progesterone OCPs, or example, no secondary method o ommends that a long-acting reversible contraceptive
contraception is needed as long as the OCP is started within (LARC) be considered as a irst-line contraceptive option
7 days o the onset o the last menses. I the OCP is started in adolescents; options include implants and intrauter-
greater than 7 days a er the onset o the last menses, only ine devices (IUDs). LARCs can provide 3 to 10 years o
7 days o a secondary contrace tion ethod (e.g., condo s, contraception and are e ective and sa e orms o birth
dia hrag ) is needed; combination OCPs work within 7 days control. The use o condoms goes down with LARCs,
o the start date. Please see able 15-1 regarding CDC recom- however (JAMAPediatr. 2016 Mar 14).
mendations or backup contraception.
CHAPTER 15 • OBSTETRICS AND WOMEN’S HEALTH 411

A) Quantitative β -human chorionic gonadotropin (β -hCG)


Eight onths later, the atient calls to s eak with your nurse B) Serum electrolytes, BUN, and creatinine
regarding nausea and vo iting. A arently, she decided to C) T yroid-stimulating hormone ( SH)
sto taking her OCP. Her last enstrual eriod was 10 weeks D) Pelvic ultrasound
earlier, and she had a ositive ho e regnancy test 5 weeks E) All o the above
ago. Over the last week, she has been vo iting once every
day, at various ti es, but is nauseated throughout ost o the Answer 15.1.6 The correct answer is “E.” As with any other
day. She wants to know i there is anything else that is “sa e” severely nauseated and vomiting patient, it is reasonable to check
that she can do to decrease the nausea. or electrolyte imbalances. Severity o nausea and vomiting cor-
relates with higher levels o hCG, as would be seen with a molar
Question 15.1.5 What is your most appropriate response? or twin pregnancy. Gestational trophoblastic disease, although
A) “T is level o nausea and vomiting is abnormal and needs an rare, should be evaluated or with an hCG level. T e ultrasound
immediate workup to rule out other pathology” would con rm a twin pregnancy and could provide evidence o
B) “T is level o nausea and vomiting is very common, and a molar pregnancy. SH can exclude hyperthyroidism.
there are several modi cations and over-the-counter medi-
cations that are sa e” Fortunately, your atient’s sy to s i roved with dietary
C) “T is level o nausea and vomiting is very common; how- changes, vita in B6, and doxyla ine. She resents or her
ever, there are no medications that can be initiated in the initial renatal visit at 12 weeks o gestation.
rst trimester”
D) “T is level o nausea and vomiting is very common. Meto- Question 15.1.7 You o er her the routine prenatal tests at
clopramide, promethazine, and ondansetron are our rst- this visit, which include all o the ollowing EXCEPT:
line therapies” A) Syphilis testing
E) “Deal with it. You got yoursel into this mess” B) HIV testing
C) 1-hour glucose tolerance test
Answer 15.1.5 The correct answer is “B.” Mild-to-moderate D) Blood type and antibody screen
nausea and vomiting are very common in the rst trimester o E) Fetal nuchal translucency with maternal hCG and plasma-
pregnancy, o en improving by 16 weeks o gestation. Several associated pregnancy protein A (PAPP-A)
modi cations can improve symptoms, including small, requent
meals, avoiding atty oods, and avoiding environmental trig- Answer 15.1.7 The correct answer is “C.” Diabetes screening
gers (per umes, smoking, position changes, or certain move- with a 1-hour glucose tolerance test (50-g carbohydrate load
ments). Over-the-counter remedies include ginger root or ale, with blood glucose obtained at 1 hour) is typically per ormed
the combination o vitamin B6 (10–25 mg three to our times between 24 and 28 weeks o gestation. Patients with risk ac-
daily) and doxylamine (10–12.5 mg three to our times daily). tors or gestational diabetes, which our patient doesn’t have, are
Vitamin B6 and doxylamine may be used separately. For those candidates or earlier screening, even at the rst prenatal visit.
who do not respond, antiemetics including metoclopramide or Risk actors include history o previous pregnancy with gesta-
promethazine should be considered (thus, “D” represents a next tional diabetes, polycystic ovarian syndrome (PCOS), history o
step, not a rst step). Ondansetron would be third line due to macrosomia, high prepregnancy BMI, rst-degree relative with
slightly greater potential risk to the etus. Up to 2% o the time, diabetes mellitus type 2 and certain ethnic groups such as East
nausea and vomiting represent hyperemesis gravidarum, which Asian, Hispanic, and Paci c Islander. Syphilis and HIV testing
involves weight loss o more than 5% o prepregnancy weight or are completed to decrease the risk o perinatal transmission. T e
dehydration and ketonuria. blood type and antibody screen are used to identi y mothers
with blood antibodies that could cause hemolytic disease o the
HELPFUL TIP: etus. Mothers who are Rh negative will subsequently receive
Diclegis (doxylamine succinate and pyridoxine hydro- RhoGAM as well. wo maternal serum markers, hCG and preg-
chloride) is now a FDA approved medication or nausea nancy-associated plasma protein (PAPP-A . . . an early screen
and vomiting o pregnancy. Patients should take 2 tab- or Down syndrome), and one etal marker (nuchal thickness)
lets at bedtime as a starting dose and may increase to 1 should be used between 11 and 14 weeks to evaluate or Down
in the morning, 1 in the mid-a ternoon, and 2 at night- syndrome. T is method o ers a Down syndrome detection rate
time, i needed. However, it is way expensive and has o approximately 85% with a 5% alse-positive rate.
no advantage over the combination o OTC doxylamine
and pyridoxine. HELPFUL TIP:
Nuchal translucency looks at luid collections in the
posterior etal neck. Increased translucency suggests
Question 15.1.6 I the patient had instead presented with increased luid collection and is associated with abnor-
6 to 8 episodes o emesis daily, an 8 lb weight loss since malities such as Down syndrome, Turner syndrome, and
her last menses, and a urine speci c gravity o > 1.030 and hemodynamic problems (cardiac abnormalities).
ketonuria, your workup should have included:
412 FAMILY MEDICINE EXAMINATION & BOARD REVIEW

Question 15.2.1 You tell the nurses to call you with her
HELPFUL (AND IMPORTANT) TIP:
vital signs and to initiate all o the ollowing interventions
Since it does not screen for neural tube defects, first-
immediately upon the patient’s arrival EXCEPT:
trimester screening does not negate the need for a second
A) Obtain IV access
trimester maternal serum triple or quad screen. The triple
B) Draw blood or type and screen
screen, which screens or chromosomal abnormalities
C) Per orm a digital vaginal examination
(e.g., Down) and neural tube de ects, includes levels
D) Initiate etal monitoring
or AFP (α - etoprotein), HCG, and uE3 (unconjugated
E) Draw blood or complete blood count
estriol). The quadruple screen adds an inhibin A. The
triple or quadruple screen is done between 15 weeks 0
Answer 15.2.1 The correct answer is “C.” A small-to-moderate
days and 19 weeks 6 days o gestation. The triple screen
amount o bleeding is not unexpected during labor; however,
has a 70% sensitivity or neural tube de ects and Down
the pro use bleeding described by the patient is an obstet-
syndrome with a 5% alse-positive rate. The quadruple
ric emergency. T e rst priorities are to obtain IV access and
screen has an 81% sensitivity and a 5% alse-positive
ensure that the mother is hemodynamically stable. Baseline lab-
rate.
oratory evaluation will give some indication o the amount o
blood loss and establish that blood is available or trans usion i
necessary. Monitoring o the etal heart rate (FHR) will evaluate
etal status. Also, an ultrasound should be done to evaluate or
T e atient has assed all o her screening tests with ying placenta previa. A digital vaginal exa ination should NO
colors, and she returns at 36 weeks o gestation. be er or ed until the diagnosis o lacenta revia has been
excluded. A consultation with someone skilled in cesarean sec-
Question 15.1.8 What additional screening test(s) is/are tion should be obtained i the initial evaluation suggests that
obtained near 36 weeks o gestation? immediate etal delivery is necessary.
A) Amniocentesis
B) 3-hour glucose tolerance test HELPFUL TIP:
C) Fetal bronectin (FFN) Classically, placenta previa presents as painless third-
D) Group B streptococcus (GBS) culture or PCR trimester bleeding, whereas placental abruption classi-
E) All o the above cally presents as pain ul third-trimester bleeding. Note
that these are the “classic” presentations, not pathog-
Answer 15.1.8 The correct answer is “D.” T e CDC recom- nomonic.
mends maternal screening or GBS between 35 and 37 weeks
o gestation. T e only patients who should be excluded rom
screening are women who had bacteriuria with GBS during
the current pregnancy or those who had an in ant previously When the atient arrives at the hos ital, she alters her recol-
in ected with GBS. T ese mothers are treated empirically with lection o events (never ha ens in our ractice) to say that
antibiotics in labor and do not need screening. Mothers whose the uid soaking the bed sheets was blood-tinged and ink
culture or PCR status is unknown should receive antibiotic pro- in color and rst occurred 2 hours ago. She continues to have
phylaxis in labor only i one o the ollowing exists: (1) intrapar- vaginal leakage but denies any bright red bleeding or con-
tum ever (> 38°C), (2) preterm labor (< 37 weeks o gestation), tractions. Ultrasound reveals a undal lacenta (not a la-
or (3) prolonged rupture o membranes (> 18 hours). centa revia) without any evidence o abru tion. Fetal heart
tones are in the 140s and reactive. Sterile s eculu exa i-
Objectives: Did you learn to . . . nation reveals uid, which is nitrazine and erning ositive
• Describe contraception options? (both evidence o ru tured e branes). Her GBS culture
• Provide appropriate routine prenatal care? er or ed 3 weeks ago is negative. She is still not feeling con-
• Recognize nausea and vomiting o pregnancy and describe tractions, and there are no contractions on the monitor.
its appropriate management?
Question 15.2.2 What is the most appropriate next step in
the management o this patient?
CASE 15.2 A) Begin an induction o labor
You are taking obstetric calls or your grou over Labor B) Send her home a er 4 hours o reassuring etal monitoring
Day (Get it? Bad joke?) weekend. Labor and delivery calls C) reat her with IV penicillin or GBS prophylaxis
you about a 27-year-old G1P0 at 38 weeks o gestation who D) Repeat her GBS test to con rm GBS status
awoke this orning co laining o wetness. However, when
she went to the bathroo she discovered signi cant vaginal Answer 15.2.2 The correct answer is “A.” A patient should
bleeding that had soaked her bed. She denies any cra ing not be sent home a er rupture o membranes. She is at risk
or abdo inal ain. She is on her way to the hos ital—and so or intrauterine in ection, and induction is indicated to reduce
are you. the risk o in ection at term. Induction o labor, even with an
CHAPTER 15 • OBSTETRICS AND WOMEN’S HEALTH 413

un avorable cervix, is not associated with an increase in cesar- inute with ini al variability. Contractions occur every
ean or operative vaginal delivery, but it is associated with ewer 3 inutes. Maternal te erature is now 38.6°C, and her
maternal in ections and ewer neonatal intensive care unit ulse is 110 b . T e atient denies any co laints.
admissions. “C” and “D” are incorrect. Her GBS status was neg-
ative 3 weeks ago. Although GBS colonization can be transient, Question 15.2.4 Given the history o prolonged rupture o
since the test was collected a er 35 weeks it should be reliable. membranes and ever, which o the ollowing is the most
Even i her membranes were ruptured or more than 18 hours, likely diagnosis?
she would not require treatment with antibiotics unless she A) Normal labor
developed a ever (remember she is GBS negative . . . review the B) Epidural ever
criteria in answer 15.1.8). I a ever develo s, think chorioa - C) Nosocomial in ection
nionitis when choosing antibiotic thera y. D) Chorioamnionitis

Sterile vaginal exa ination reveals a cervix that is 3 c Answer 15.2.4 The correct answer is “D.” Chorioamnionitis
dilated, 2 c long (e ace ent), so , anterior, and vertex at is the most likely diagnosis and the diagnosis o most concern
a − 1 station. T e atient agrees to an induction o labor. given the prolonged rupture o membranes (we gave you this
answer above i you were reading . . .). reatment should be initi-
Question 15.2.3 The best induction method or this patient ated immediately. “A” is incorrect as ever is not a normal part o
is to: labor. “B” is possible, but not as likely and a dangerous assump-
A) Insert intracervical laminaria tion. T ere is an association between the use o epidural anal-
B) Begin IV oxytocin at 2 milliunits per minute gesia and a rise in maternal temperature. Etiologies proposed
C) Insert intravaginal dinoprostone or this temperature increase include lack o pain-induced
D) Have her partner sit on her abdomen hyperventilation and decreased perspiration due to sympathetic
E) Have her run a ew laps around L&D (or walk around the blockade. “C” is extremely unlikely given her brie time in the
hospital) hospital.

Answer 15.2.3 The correct answer is “B.” T e use o intra- Question 15.2.5 What is the next step in the care o this
vaginal and intracervical methods or cervical ripening could patient?
be cautiously considered in patients with premature rupture o A) Increase the oxytocin to hasten delivery
membranes with an un avorable cervix; but “C” is not the best B) Remove the epidural
choice—she is already 3cm dilated with a Bishop score o 6! C) Initiate broad-spectrum antibiotics
Drugs or cervical ripening may include prostaglandins which D) Call your backup or cesarean section (or do it yoursel )
act in the cervix to acilitate smooth muscle relaxation resulting E) Give acetaminophen 1,000 mg orally
in so ening and dilatation o the cervix. T ey also may induce
myometrial contraction. Currently the only prostaglandin Answer 15.2.5 The correct answer is “C.” Initiation o anti-
labeled by the Food and Drug Administration (FDA) or such biotics is associated with a decrease in both maternal and neo-
purpose are the prostaglandin E2 agents such as dinoprostone natal morbidity. Multiple organisms are isolated in more than
(Prepidil, Cervidil). Prostaglandin E1 agents such as misopro- 66% o cases; there ore, antibiotics should be broad. Approved
stol (Cytotec) inserted intravaginally are also commonly used regimens include ampicillin and gentamicin, ticarcillin/clavu-
or cervical ripening but do not carry a label by the FDA or this lanate, or piperacillin. T ere is no need to increase the oxyto-
indication. Intracervical use o laminaria (an expanding pledget cin (“A”); she has already made good labor progress. T ere is
placed in the cervix) (“A”) is generally reserved or pregnancy also no need to proceed with a cesarean delivery (“D”) unless
termination. “D” would not be com ortable and may cause there is another indication, such as nonreassuring FHR pat-
undue harm to mom and the etus, and is kind o weird. “E” is tern. Although epidural anesthesia is associated with increased
not the best choice. While walking may stimulate contractions, maternal temperature, it should only be removed i it is elt to be
she already has ruptured membranes and a more aggressive contributing to maternal pathology (e.g., meningitis, epidural
approach is warranted. abscess, or epidural bleed). Acetaminophen is ne, but it should
Oxytocin should be care ully titrated via IV route or labor not be used in lieu o antibiotics.
induction or augmentation. T e general starting doses are 0.5
to 2 milliunits per minute, increased by 1 to 2 milliunits every T e atient’s labor is rogressing. Her cervix is 9 c dilated,
20 to 40 minutes, to a maximum dose rarely exceeding 30 to co letely e aced, and station is + 3. Her te erature is
40 milliunits per minute. 39.0°C. T e FHR attern is shown in Figure 15-1.

T e atient is currently in labor (success!), and now her cer- Question 15.2.6 What is the FHR interpretation?
vical exa ination is 6 c dilation, 1 c e aced, and − 1 sta- A) Baseline 165 beats per minute, reactive
tion. T e a niotic uid is still clear, having ru tured a rox- B) Baseline 165 beats per minute, with periods o bradycardia
i ately 22 hours ago. She has had an e idural laced or C) Baseline 165 beats per minute, with late decelerations
analgesia. T e FHR baseline has increased to 165 beats er D) Baseline 165 beats per minute, with variable decelerations
414
1
2
2
1
1
1
9
6
3 0
4
1
8
5
2
0
0
0 0
0
0
0
0
0 2
2
1
1
1 9
6
3
4
1
8
5
2 0
0
0
0
0
0
0
0 2
2
1
1
1
9
6
3 4
1
8
5
2
0
0
0 0
0
0
0
0
P
0
M
R 8
S
U
E
D
E :
D
T
F
L 5
P
D
L A
M
S
F
0 A
S
E
P G
M E S
1 2
R / 0
E 1
M / 0
A 3
I N
3 I
c
N m
G / m i
n P
M
T
R S
U
E
B
E
D
p
E
O D
T
F
L
/
M
H
D
L P
P
A
M
S
F P
S
A
E G E S R E M
A I
N I
N G P
M
T
R S
U
E
B
E
D
p
E
O
D T
F
L
/
M
H
D
L P
A
M
S
F
P P
S
E
A G E S R E M
A I
N I
N G
2 2 2
T B
E
p
O /
M
H P P
6
4
2
8 0
0
0
0
0 1
6
4
2
8 0
0
0
0
0
0 0 1
6
4
2
8 0
0
0
0
0
0 0
0 8 8 0 8 0 6 3 0 8 7 0 8 0 6 4 0 8 6 0 8 0 6 5
F I
G U R E 1 5 - 1 .
CHAPTER 15 • OBSTETRICS AND WOMEN’S HEALTH 415

TABLE 15-2 FETAL HEART TRACING Question 15.2.7 What is the likely etiology o the etal heart
CATEGORY DEFINITIONS rate (FHR) tracing in Figure 15 1?
A) Head compression
CATEGORY I Tracing (normal): FHR include ALL o the ollowing:
B) Placental insuf ciency
• Baseline FHR within normal range at 110–160 bpm
• Moderate baseline FHR variability C) Cord compression
• Absent late or variable decelerations D) Any o the above is equally likely to cause the tracing
• Accelerations may be present or absent

CATEGORY II Tracing: Includes all FHR tracings that are not


Answer 15.2.7 The correct answer is “B.” Late decelera-
categorized as Category I or Category III. May represent an appreciable tions are believed to be secondary to transient etal hypoxia in
part o those encountered in clinical practice (atypical . . . May response to decreased placental per usion. Prompt evaluation
represent etal hypoxia). and intervention is warranted. Early decelerations are gener-
Any o the ollowing: ally reassuring and attributed to etal head compression. Vari-
• Bradycardia with FHR < 110 bpm WITH variability able decelerations are the most common decelerations seen in
• Fetal tachycardia with FHR > 160 bpm
labor and indicate cord compression. Variable decelerations can
• Minimal baseline variability or absent baseline variability WITHOUT
recurrent decelerations, or marked baseline variability sometimes be relieved by maternal repositioning or amnioin u-
• Absence o induced accelerations a ter etal stimulation sion (in using saline or Ringers Lactate to “reexpand” the uterus
• Recurrent variable decelerations accompanied by minimal to and relieve the pressure on the cord).
moderate baseline variability
• Prolonged deceleration > 2 minutes but < 10 minutes
Question 15.2.8 What is the LEAST appropriate course o
• Recurrent late decelerations with moderate baseline variability
• Variable decelerations with other characteristics such as slow return action now?
to baseline, “overshoots,” or “shoulders” A) Administer maternal oxygen
B) Stop oxytocin
CATEGORY III Tracing (abnormal):
EITHER
C) Forceps or vacuum-assisted vaginal delivery
• Absent baseline FHR variability AND any o the ollowing: D) Consider cesarean delivery
• Recurrent late decelerations E) Reposition mother (roll to one side or knee–chest)
• Recurrent variable decelerations
• Fetal bradycardia Answer 15.2.8 The correct answer is “C.” Increasing mater-
OR
• A sinusoidal pattern
nal oxygenation may improve etal oxygenation. Oxytocin can
decrease placental blood ow via uterine stimulation, and hence
Table adapted rom American College o Obstetricians and Gynecologists. should be decreased or stopped i non-reassuring FHR changes
ACOG Practice Bulletin no. 106: Intrapartum etal heart rate monitoring, are present. I there is evidence o maternal hypotension, mater-
nomenclature, interpretation and general management principles. Obstet nal hydration may be indicated. Another option is position
Gynecol. 2009;114(1):192–202.
changes (le lateral) to improve placental per usion. Maternal
position can a ect uterine blood ow and placental per usion.
Answer 15.2.6 The correct answer is “C.” T e FHR pictured T e gravid uterus may compress the vena cava while supine.
is described as Category II (see able 15-2) with a baseline o Because the patient is 9 cm dilated, orceps or vacuum-assisted
about 165 beats per minute. T ere is moderate variability pres- vaginal delivery (also known as “operative vaginal delivery”) is
ent. Following each contraction, there are late decelerations to NO indicated; dilation needs to be complete (10 cm) in order
the 110s. “A” is incorrect since “reactive” re ers to a nonstress to use orceps or vacuum
test, not etal monitoring during labor (see Help ul ip or
more). “B” is incorrect as etal bradycardia is de ned as an FHR T e FHR changes resolve with your a ro riate interventions.
o less than 110 beats per minute or at least 10 minutes. “D” T e atient rogresses to co lete dilation and delivers vagi-
is incorrect. Variable decelerations vary with respect to timing, nally 1 hour later. Maternal antibiotics are discontinued ol-
duration, and depth. lowing delivery, and the aternal te erature 2 hours a er
delivery is 37.0°C. On ost artu day 1, your atient co -
lains o sore breasts ro breast eeding but no articular
HELPFUL TIP: swelling o the breast (beyond what is ex ected ost artu ),
Nonstress tests are per ormed outside o the intrapar- and her abdo en is sore “all over.” She is having a oderate
tum period. A reactive nonstress test is de ined as two a ount o lochia, and her te erature is 38.4°C.
accelerations o at least 15 beats above baseline and
Question 15.2.9 The most likely cause o the ever at this
lasting at least 15 seconds within a 20-minute interval
time is:
in gestations greater than 32 completed weeks. In
A) Endometritis
gestations less than 32 weeks, reactive is de ined as two
B) Mastitis
accelerations at least 10 beats above baseline and last-
C) Deep vein thrombosis
ing at least 10 seconds.
D) Septic pelvic thrombophlebitis
416 FAMILY MEDICINE EXAMINATION & BOARD REVIEW

Answer 15.2.9 The correct answer is “A.” Whenever ever C) T e maturity o the etal lungs
occurs in the immediate postpartum period, endometritis D) T e results o a Catholic intramural baseball game
should be suspected. T e presence o intra-amniotic in ection
(chorioamnionitis) increases the risk o postpartum endometri- Answer 15.3.1 The correct answer is “B.” T e Bishop score,
tis to 13% (or 1 in 8 women, i you pre er ractions). Antibiotics which takes into account cervical dilation, e acement, consis-
are not routinely continued or chorioamnionitis a er a vagi- tency, and position, as well as etal station, is a use ul tool to
nal delivery because the “source” o the in ection (the placenta) determine i cervical ripening agents are needed or induction
has been removed. Mastitis (“B”) is characterized by a swollen, and to predict induction success. Calculators are readily avail-
rm, tender breast with systemic symptoms including evers, able online.
chills, and u-like symptoms. However, it was stipulated that
the breasts were normal or our postpartum patient, and having T e Bisho score is avorable at 9–10.
bilateral mastitis is unlikely. Staphylococcus aureus is the typi-
cal pathogen in mastitis. Pregnancy and the postpartum period Question 15.3.2 Which o the ollowing are the most appro
increase a woman’s risk o thrombogenesis. However, DV (“C”) priate recommendations at this point?
is not a likely source o the ever. Septic pelvic thrombophlebitis A) She should be induced at once; there is a high chance o etal
(“D”) is a diagnosis o exclusion and is usually entertained when mortality a er 41 weeks o gestation.
ever spikes continue despite treatment or endometritis. B) Since her antepartum course has been uncomplicated
to date, it is sa e or her to await spontaneous labor until
You start a ro riate antibiotics and the atient does well. 43 weeks o gestation.
T e a ily, in gratitude, na es the child a er you (adding C) She should undergo a nonstress test and ultrasound or
the suf x “-to s” to the child’s na e, like “ ricerato s”) amniotic uid index.
and akes you the god arent necessitating gi s or the next D) She should plan or a cesarean section.
18 years on the child’s birthday.
Answer 15.3.2 The correct answer is “C.” By de nition, a term
gestation is one completed in 38 to 42 weeks. T ere is no signi -
HELPFUL TIP:
cant increase in etal mortality in an uncomplicated pregnancy
In ants born to women with chorioamnionitis have a
at term. Virtually all reports suggest an increase in perinatal
our old increase in neonatal mortality and a three old
morbidity and mortality when pregnancy goes beyond 42 weeks
increase in the incidence o respiratory distress, neona-
o gestation. Antenatal surveillance o post-term pregnancies
tal sepsis, and intraventricular hemorrhage.
should be initiated at 41 weeks o gestation.

Objectives: Did you learn to . . . HELPFUL TIP:


• Triage and manage third-trimester bleeding? Accurate determination o conception is important in
• Recognize and treat a patient with premature rupture o reducing the alse diagnosis o post-term pregnancy.
membranes? The estimated date o delivery is most reliably and ac-
• Evaluate and manage intrapartum ever? curately determined early in pregnancy. Ultrasound
• Interpret FHR patterns and begin initial management o may assist in determining dates, but has a standard
abnormal patterns? o error that is dependent on the gestational age (see
• Appreciate the risk actors, evaluation, and management o Table 15-3).
postpartum in ection?

Question 15.3.3 At 41 weeks she is really (really, really) tired


CASE 15.3 o being pregnant and wants “a natural way” to induce
You are seeing a 31-year-old G2P1 at 41 weeks o gestation contractions. Which o the ollowing nonpharmacologic
by de nite last enstrual eriod and 16-week ultrasound. methods o inducing or augmenting labor is LEAST likely
She continues to note etal ove ent and her exa ination is to be e ective?
nor al: BP 120/68 Hg, urine di stick negative or ro- A) Stripping the amniotic membranes
tein and glucose, undal height 42 c , etus is vertex, FHR B) Prolonged walking
156 b . Her cervix is so , anterior, 2 to 3 c dilated, 50% C) Amniotomy
e aced, and + 1 station. She was induced with her rst reg- D) Nipple stimulation
nancy, and this ti e she wants to have a “natural labor.” You
decide to calculate a Bisho score. Answer 15.3.3 The correct answer is “B.” Stripping mem-
branes appears to be e ective in initiating spontaneous labor
Question 15.3.1 The Bishop score helps to determine: within 72 hours. Amniotomy may be used or labor induction,
A) T e health o the etus especially i the Bishop score is avorable. However, a Cochrane
B) T e need or cervical ripening agents or induction and review rom 2013 did not nd strong evidence supporting the
helps to predict labor induction success use o amniotomy alone or in combination with oxytocin or
CHAPTER 15 • OBSTETRICS AND WOMEN’S HEALTH 417

TABLE 15-3 GUIDELINES FOR DATING BASED ON ULTRASONOGRAPHY


Discrepancy Between Ultrasound Dating
Gestational Age Range Based on LMP Method o Measurement and LMP Dating that Supports Re dating
≤13 6/7 weeks CRL
• ≤8 6/7 weeks • More than 5 days
• 9 0/7 weeks to 13 6/7 weeks • More than 7 days

14 0/7 weeks to 15 6/7 weeks BPD, HC, AC, FL More than 7 days

16 0/7 weeks to 21 6/7 weeks BPD, HC, AC, FL More than 10 days

22 0/7 weeks to 27 6/7 weeks BPD, HC, AC, FL More than 14 days

28 0/7 weeks and beyond a BPD, HC, AC, FL More than 21 days

AC, abdominal circum erence; BPD, biparietal diameter; CRL, crown rump length; FL, emur length; HC, head circum erence; LMP, last menstrual period.
a
Because the risk o re-dating a small etus that may be growth-restricted, third-trimester re-dating should be guided by care ul consideration o the entire
clinical picture and close surveillance.
Adapted rom American College o Obstetricians and Gynecologists. Method or estimating due date. Committee Opinion No. 611. Obstet Gynecol. 2014;124:
863–866.

induction. Nipple stimulation causes release o oxytocin and


Your atient’s husband is called u or active duty in Iraq
may be utilized or labor induction, but its marginal bene t is
(or A ghanistan . . . or Libya or . . . sadly, we are on the 4th
only seen in patients with a avorable Bishop score. Walking
edition o this book and have not needed to change this
does not result in labor induction or augmentation, but it’s not
scenario) and is due to re ort in the next ew days. She is
harm ul either.
now 41 2/7 weeks o gestation and desires induction so he
can be with her or the delivery. You ad it her to labor and
HELPFUL TIP: delivery the ollowing orning. T e initial FHR onitoring
Sexual intercourse is sometimes recommended to in- be ore induction (also known as a nonstress test) is shown
duce labor. Studies are o low quality and use various in Figure 15-2.
endpoints . . . also, it is di icult to standardize the inter-
vention (we could make a joke here but won’t). One o
Question 15.3.5 What is the correct interpretation?
the better quality studies (Tan et al., 2006) did ind that
A) Baseline 150 beats per minute; not reactive
coitus was associated with reduced need or labor in-
B) Baseline 150 beats per minute; reactive
duction at 41 weeks.
C) Baseline 180 beats per minute; decelerations to 150s; not
reactive
Question 15.3.4 I induction becomes necessary, which o D) Baseline 180 beats per minute; moderate variability; reactive
the ollowing pharmacologic interventions would be the
best approach to your patient who has a cervix that is so t,
Answer 15.3.5 The correct answer is “B.” T e baseline is about
anterior, 2 to 3 cm dilated, 50% e aced, and+ 1 station?
150 beats per minute. T ere are two accelerations greater than
A) IV oxytocin
15 beats and lasting longer than 15 seconds, which meets the
B) Intracervical PGE2 (dinoprostone)
criteria or a reactive nonstress test. T ere is one contraction
C) Intravaginal PGE2 (dinoprostone)
and evidence o uterine irritability noted as well.
D) Intravaginal PGE1 (misoprostol)
E) None o the above. All pharmacologic interventions are
contraindicated
HELPFUL TIP:
Answer 15.3.4 The correct answer is “A.” T is patient does not When interpreting FHR tracings, variability is an impor-
need urther cervical ripening but is a candidate or induction o tant element that demonstrates etal cardiac response to
labor. I cervical ripening were needed, there are several available parasympathetic input. The small wave orm luctuations
agents. Option “D,” PGE1 (misoprostol, brand name Cytotec) within the baseline heart rate tracing represent the FHR
can be administered intravaginally or orally (but note that the variability. A ter 28 weeks o gestation, variability should
Food and Drug Administration [FDA] has not approved it or be present. It is categorized as absent (no amplitude,
use in pregnancy). “B” and “C”, PGE2 (dinoprostone, brand lat tracing), minimal (0–5 beat amplitude), moderate
name Cervidil) is administered intravaginally. PGE2 gel (dino- (6–25 beat amplitude), or marked (> 25 beat amplitude).
prostone, brand name Prepidil) can be administered either The absence o variability is associated with etal decom-
intravaginally or intracervically. Because the cervix is avorable pensation or distress.
in this case, proceeding with oxytocin is the best option.
418
1
2
2
1
1
1
9
6
3 0
4
1
8
5
2
0
0
0 0
0
0
0
0
0 2
2
1
1
1 9
6
3
4
1
8
5
2 0
0
0
0
0
0
0
0 2
2
1
1
1
9
6
3 4
1
8
5
2
0
0
0 0
0
0
0
0
P
M
R S
U
E
D
E
D T
F
L
P
D
L A
M
S
F
A S
E
G E S R E M
A I
N I
N G 0 4 : 0 0
P
M
T R
1
S
U
E
B
E
D
p
E
O 2
D
T
F
L
/
M
H D
L
/
P
A
P
M
S
F 0
P
S
A 7
E G
/ 0E 3 S 3
R c
E m
M / m
A i
I
n N
I N
G P
M
T
R S
U
E
B
E
D
p
E
O D
T
F
L
/
M
H
D
L
P A
M
S
F
P P
S
A
E G E S R E M
A I
N I
N G
2
2 2
T B
E
p
O /
M
H P P
6
4
2
8 0
0
0
0
0 1
6
4
2
8 0
0
0
0
0
0 0 1
6
4
2
8 0
0
0
0
0
0 0
1 1 7 7 7 2 8 4 1 1 6 7 7 2 8 5 1 1 5 7 7 2 8 6
F I G U R E 1 5 - 2 .
CHAPTER 15 • OBSTETRICS AND WOMEN’S HEALTH 419

Question 15.3.7 What is the correct interpretation o this


You er or a nioto y with return o articulate eco-
FHR tracing?
niu -stained uid. Her cervix is now 5 c dilated, 80%
A) Baseline 160 beats per minute; reactive
e aced, with vertex at + 1 station. You elect to continue on-
B) Baseline 160 beats per minutes; variable deceleration to the 90s
itoring rogress.
C) Baseline 160 beats per minute; late decelerations to the 90s
D) Baseline 160 beats per minute; early decelerations to the 90s
Question 15.3.6 Which o the ollowing choices o labor
analgesia is MOST appropriate at this point? Answer 15.3.7 The correct answer is “B.” A variable decelera-
A) Epidural analgesia tion to the 90s occurs with the rst contraction on this strip.
B) Local perineal anesthetic in ltration Variable decelerations vary with respect to timing, duration, and
C) Bilateral pudendal nerve block depth—thus, the name “variable.” T ey are not uni orm. Vari-
D) All o the above are equally appropriate able decelerations represent changes in the FHR in response to
cord compression. Please re er to the able 15-2 with de nitions
and FHR tracing categories.
Answer 15.3.6 The correct answer is “A.” Epidural analgesia
o ers the most e ective orm o pain relie and generally may be HELPFUL TIP:
utilized once the patient is determined to be in active labor. Var- A systematic review in 2013 showed no bene it in out-
ious local anesthetic agents are available or local in ltration o comes or continuous FHR monitoring compared to
the perineum and vagina to provide analgesia or e isioto y or intermittent FHR monitoring. Un ortunately, there are
laceration re air ollowing delivery but not or labor. Bilat- more cesarean sections and operative vaginal deliver-
eral pudendal nerve blocks are use ul during the second stage ies when continuous FHR monitoring is used. However,
o labor, as a supplement to epidural analgesia or anesthesia o much o FHR monitoring use is dictated by local practice
the sacral nerves, or as an option or operative vaginal delivery patterns, expert consensus, and medicolegal concerns.
anesthesia (e.g., orceps, vacuum). Opioid agonists and agonist–
antagonists are also available and commonly employed. How- Question 15.3.8 Given the ndings in Figure 15 3, which o
ever, some reports suggest that the analgesic e ect o opioids in the ollowing should be per ormed next?
labor is limited when using the lower doses that are generally A) Check the patient’s cervix
regarded as sa er or the etus. B) Place a etal scalp electrode
C) Begin IV oxytocin in usion
T e nurse notices so e changes on the etal heart onitor. D) Place an intrauterine pressure catheter and begin an amnio-
T e current FHR is shown in Figure 15-3. in usion

240 240

210 210

180 180

150 150

120 120

90 90

60 60

30 30

#11:20
100 100
MEDS MEDS MEDS
DL DL DL
80 80
EFF EFF EFF
S TA S TA S TA
60 60
RDM RDM RDM
pH pH pH
40 40
O2 O2 O2
P ULS E P ULS E P ULS E
20 20
TEMP TEMP TEMP

FIGURE 15-3.
420 FAMILY MEDICINE EXAMINATION & BOARD REVIEW

Answer 15.3.8 The correct answer is “A.” Variable decelera- Answer 15.3.10 The correct answer is “E.” Maternal risks o
tions are common in labor, and brie variable decelerations are operative vaginal delivery include injury to the lower genital
benign. When variable decelerations become recurrent, progres- tract and rectal sphincter involvement in the case o a third- or
sively deeper, and longer lasting with delayed return to baseline, ourth-degree laceration. In addition, etal complications need
they are non-reassuring and may re ect etal hypoxia. A pelvic to be discussed as well. Shoulder dystocia is more common with
examination should be per ormed to determine i the umbilical operative delivery than with a spontaneous vaginal delivery (see
cord is prolapsed or i there has been rapid descent o the etal below or more in ormation).
head or rapid progression o labor. Oxytocin, “C,” should not be
considered in this patient since she is having adequate contrac- Question 15.3.11 Each o the ollowing is a etal risk o
tions. Replacement o the amniotic uid (“D,’’ amnioin usion) operative vaginal delivery EXCEPT:
with normal saline in used through a transcervical catheter has A) Cephalohematoma
been reported to decrease both the requency and severity o B) Skull racture
repetitive variable decelerations and can decrease rate o cesar- C) Brachial plexus injury
ean section. However, it would rst be help ul to assess the cer- D) Respiratory distress syndrome
vical status. O note, amnioin usion is no longer recommended E) Facial nerve palsy
as a prophylactic intervention or moderate or severe meconium.
Answer 15.3.11 The correct answer is “D.” Respiratory dis-
Labor rogresses without incident. Your atient is now co - tress syndrome is not increased by assisted delivery. Neonatal
letely dilated and e aced, with etal head at + 3 station. She cephalohematoma, retinal hemorrhage, and jaundice (second-
is co ortable with her e idural and able to ush with good ary to breakdown and reabsorption o the cephalohematoma)
e ort. T e FHR tracing is reassuring. Contractions are every are more common with vacuum-assisted delivery than with
3 inutes. orceps-assisted delivery. Skull racture and acial nerve injury is
more common with orceps-assisted delivery than with vacuum-
Question 15.3.9 Appropriate management at this point is: assisted delivery. Shoulder dystocia with resultant brachial
A) Continue pushing plexus injury is more common with vacuum-assisted deliv-
B) Vacuum-assisted delivery ery, prolonged time required or delivery, and increasing birth
C) Forceps-assisted delivery weight. Note that injury can occur be ore operative delivery as a
D) Midline episiotomy result o abnormal labor orces (we don’t think anyone has told
E) Augment with oxytocin the malpractice attorneys . . .).

Answer 15.3.9 The correct answer is “A.” At this point, labor Delivery o an 8-lb baby is acco lished without o erative
is progressing and maternal– etal status is reassuring. You vaginal assistance. T e ere resence o the vacuu on the
should continue expectant management. No intervention is table was enough to entice the uterus to er or one last
indicated. assive contraction—assisted by the in ant clawing its way
out when it saw the vacuu co ing. Following s ontaneous
delivery o the intact lacenta 15 inutes later, you note a
HELPFUL TIP:
large gush o blood.
Episiotomies should not be per ormed routinely. Indica-
tions or episiotomy are typically related to non-reassuring Question 15.3.12 Which o the ollowing is the most likely
etal status and dystocia. There is no evidence that epi- source o the bleeding?
siotomies reduce perineal trauma, postpartum dyspa- A) Uterine atony
reunia, etc. B) Vaginal laceration
C) Cervical laceration
D) Retained placenta
She ushes or 3 hours. She is now exhausted. T e etal
head now se arates the labia with contractions, and then Answer 15.3.12 The correct answer is “A.” Postpartum hem-
recedes slightly. You consider o ering assistance with orrhage is most commonly associated with uterine atony. Risk
delivery. actors include prolonged labor, over-distended uterus (such
as rom 2 or 8 gestations [remember Octomom?]), very rapid
Question 15.3.10 In counseling your patient and her hus labor, high parity, chorioamnionitis, retained placental tis-
band about the maternal risks o operative vaginal deliv sue, poorly per used myometrium, halogenated hydrocarbon
ery, which o the ollowing should you discuss? anesthesia, and previous uterine atony. Maternal trauma to the
A) Vaginal trauma genital tract (“B” and “C”) may result in postpartum hemor-
B) Shoulder dystocia rhage and should be routinely investigated, particularly ol-
C) Fetal injury lowing operative delivery. A retained placenta cotyledon is
D) Perineal and rectal trauma another common source or postpartum hemorrhage. T e
E) All o the above placenta should be inspected, and i there is any question o
CHAPTER 15 • OBSTETRICS AND WOMEN’S HEALTH 421

retained products o conception, the uterus should be manu- Question 15.3.15 All o the ollowing are characteristic o
ally explored. Sheehan syndrome EXCEPT:
A) Failure in lactation
Question 15.3.13 Which o the ollowing should be under B) Amenorrhea
taken next? C) Desire to be a punk rocker
A) Obtain IV access and initiate hydration D) Decreased LH/ ollicle-stimulating hormone (FSH)
B) Begin bimanual uterine compression E) Adrenal cortical insuf ciency
C) Inspect vagina and cervix or lacerations
D) Obtain blood or type and screen or possible blood trans u- Answer 15.3.15 The correct answer is “C.” Severe intrapartum
sion or postpartum hemorrhage may result in pituitary necrosis due
E) All o the above to hypovolemia and hypoper usion. T is leads to a global hypo-
pituitarism known as Sheehan syndrome. Sheehan syndrome
Answer 15.3.13 The correct answer is “E.” Postpartum hemor- is characterized clinically by endocrine de ciency syndromes
rhage is an obstetrical emergency and must be addressed imme- as a result o loss o anterior pituitary unction. Initial symp-
diately. T e gravid uterus receives 500 mL o blood per minute, toms may be vague (lethargy, anorexia, weight loss, dif culty
which can lead to massive hemorrhage i not addressed quickly. with lactation), and the syndrome can go unrecognized. Later
Additional personnel should be noti ed to help with obtaining mani estations include ailure o lactation, amenorrhea, breast
IV access and blood draws, while you quickly try to identi y the atrophy, loss o pubic and axillary hair, adrenal cortical insu -
source o bleeding. ciency, and hypothyroidism. Desire to be a punk rocker is
“Sheena syndrome.” I you don’t get it, you missed the Ramones.
A er thorough ex loration o the vagina and uterus, you sus-
Objectives: Did you learn to . . .
ect uterine atony is the cause o bleeding. While continuing
• Recognize the risks o prolonged pregnancy and identi y
uterine assage, you think about your o tions. appropriate timing o intervention?
• Describe the indications and risks associated with induction
Question 15.3.14 Which o the ollowing is/are options in o labor?
treating this patient’s bleeding?
• Interpret intrapartum FHR patterns and choose appropriate
A) Dilute oxytocin IV management options?
B) Methylergonovine (Methergine) IM
• Evaluate analgesia options, contraindications, and risks
C) Carboprost tromethamine (Hemabate) IM during labor and delivery?
D) Misoprostol PR
• Recognize the indications or and management o operative
E) All o the above vaginal and abdominal delivery?
• Evaluate and manage postpartum hemorrhage?
Answer 15.3.14 The correct answer is “E.” All o the drugs
listed cause smooth muscle contraction o the uterus. Oxytocin
can be given as a dilute IV solution or IM. It should never be QUICK QUIZ: WHAT’S SCARIER THAN TWINS?
administered as an undiluted IV bolus, due to the risk o hypo-
tension and cardiac arrhythmia. Methergine (methylergono- While on call or your small community hospital, a nurse on the
vine) is an ergot alkaloid and may be administered orally or labor and delivery ward calls you about a patient with preterm
intramuscularly (not intravenously). Caution should be used contractions. You come in to see the patient and nd a 33-year-
in women with hypertension, as Methergine can cause hyper- old G1P0 at 26 weeks o gestation by in vitro ertilization. She
tension. Hemabate (carboprost tromethamine) is an F-2 pros- is usually ollowed at an academic hospital 400 miles away. She
taglandin analog that is administered IM or directly into the looks worried and says, “I’m going to have twins. But not now!”
uterine myometrium. Caution should be used in women with She recalls her prenatal lab results were unremarkable. She
asthma, as Hemabate can cause bronchoconstriction. Misopro- recalls her ultrasound showed “two heart beats.”
stol is a prostaglandin E1 analog, that works well and can sa ely
be administered to women with asthma or hypertension. Rectal All o the ollowing risks are increased with a multi etal gesta-
or oral administration can be used, but rectal administration is tion EXCEP :
pre erred in a patient with potential hemodynamic instability. A) Preterm labor
T is can be a li esaver especially in third world countries where B) D-isoimmunization (Rh isoimmunization)
other options may not exist. C) Preterm rupture o membranes
D) Intrauterine growth restriction
She requires IV crystalloid and 4 units o acked red cells or E) win–twin trans usion syndrome
sy to atic ane ia ollowing delivery. Both other and
in ant do well and the atient and baby are discharged on The correct answer is “B.” T e most signi cant complication o
ost artu day 2. You schedule a ollow-u a oint ent in multiple gestations is preterm labor resulting in preterm deliv-
2 days. You are concerned about Sheehan syndro e given the ery. Preterm rupture o membranes and intrauterine growth
severe ost artu he orrhage. restriction also occur more requently in multiple gestations
422 FAMILY MEDICINE EXAMINATION & BOARD REVIEW

than singleton pregnancies. T e risk o all these complications is Answer 15.4.2 The correct answer is “B.” T ere is no evidence
directly proportional to the number o etuses. win–twin trans- that antenatal corticosteroid therapy increases the risk o neonatal
usion syndrome rarely occurs and is associated with mono- in ection. However, maternal in ection is a relative contraindica-
chorionic gestations. Multi etal gestations do not increase the tion to corticosteroid therapy. Corticosteroids are recommended
risk o D-isoimmunization. or all pregnant women between 24 and 34 weeks o gestation
who are at risk o preterm delivery within 7 days. Corticosteroids
CASE 15.4 may be given a er 34 weeks i there is documented etal lung
immaturity and delivery will likely occur be ore lung maturation.
A er a busy day in clinic, you get the traditional 5 pm call
ro labor and delivery. T e nurse tells you that your art- Question 15.4.3 Which o the ollowing FDA approved
ner’s atient is resenting with concerns or reter labor. tocolytics is regarded as rst line therapy to abort preterm
She is a 24-year-old G1 at 26 weeks by LMP and rst-tri ester labor?
ultrasound. She is having contractions but no vaginal dis- A) Ni edipine
charge. T e contractions started about an hour ago and are 5 B) Magnesium sul ate
to 6 inutes a art. T e nurse asks i you want to check etal C) Prostaglandin inhibitors
bronectin (FFN). D) Ritodrine
E) None o the above
Question 15.4.1 A NEGATIVE etal bronectin is associated
with: Answer 15.4.3 The correct answer is “E.” T e only FDA-
A) Fetal lung immaturity approved agent or use as a tocolytic was ritodrine, a β-adrenergic
B) Ruptured etal membranes receptor agonist, which was pulled rom the market in the United
C) A decreased risk o preterm birth States in 1993. T ere are no currently available FDA-approved
D) An increased risk o preterm birth tocolytics. T us, other agents have been investigated. erbuta-
line, magnesium sul ate, prostaglandin inhibitors, and calcium
Answer 15.4.1 The correct answer is “C.” FFN is a basement channel blockers have all been studied and may be utilized in
membrane protein produced by the etal membranes. A nega- select cases or tocolysis—but realize that these drugs are not
tive test is use ul in assessing the risk o preterm delivery during approved by the FDA or this indication and that they carry sub-
the ollowing 2-week period. With a properly per ormed test in stantial risks. Indomethacin has also been used success ully or
a symptomatic patient, up to 99.5% o patients with a negative tocolysis. Again, it is not approved or this indication.
FFN will not deliver in the subsequent 7 days. A positive test is
not use ul as the test has low positive predictive value. In per- HELPFUL TIP:
orming FFN testing, the ollowing criteria must be met: intact Tocolysis only prolongs labor by 48 hours at best, so
amniotic membranes, minimal cervical dilation (< 3 cm), and the real goal should be to prolong the pregnancy until
sampling between 24 0/7 and 34 6/7 weeks. Recent sexual inter- steroids have time to be e ective. Contraindications to
course and the presence o vaginal discharge or bleeding may tocolysis include evidence o etal distress, etal anoma-
cause a alse positive test. Collect the swab in the posterior vagi- lies, abruptio placentae, placenta previa with heavy
nal ornix be ore cervical checks and transvaginal ultrasound bleeding, and severe maternal disease.
as these can also cause the test to be alsely positive. FFN does
not assess etal lung maturity (“A”). Ruptured membranes (“B”)
would cause a positive FFN. You exa ine the atient a er collecting a FFN. Her cervix is
1 c dilated at the external os and closed at the internal os,
You are wondering i this atient is a good candidate or cor- long, and osterior. Ultrasound shows that the in ant is ver-
ticosteroid thera y. tex. T e onitor shows FHR baselines o 140 and 145 b .
Contractions are irregular, occurring every 4 to 9 inutes.
Question 15.4.2 Regarding the risks and bene ts o corti Urine di stick shows a s eci c gravity o 1.030.
costeroid therapy or etal lung maturation, which o the
ollowing is FALSE? Question 15.4.4 The LEAST appropriate intervention in this
A) Corticosteroid therapy is recommended or all pregnant 26 week gestation is:
women between 24 and 34 weeks o gestation who are at risk A) Continuation o monitoring
o preterm delivery within 7 days B) Oral hydration
B) Corticosteroid therapy has been associated with an increased C) Obstetrical consult or cerclage placement
risk o neonatal in ection D) est or group B streptococci, gonorrhea, and chlamydia
C) Antenatal corticosteroid therapy reduces the incidence o E) Administration o corticosteroids
respiratory distress syndrome, intraventricular hemorrhage,
and necrotizing enterocolitis Answer 15.4.4 The correct answer is “C.” Cerclage (stitch to
D) Corticosteroids accelerate the production o pulmonary sur- hold the cervix closed) is indicated or incompetent cervix, not
actant in the etal lungs preterm labor. A cerclage is typically placed in the rst part o
CHAPTER 15 • OBSTETRICS AND WOMEN’S HEALTH 423

the second trimester a er etal viability has been established. It


HELPFUL TIP:
would not be used at 26 weeks. T e patient should be evaluated
The risk or recurrent premature delivery in subsequent
care ully and the requency o uterine contractions should be
pregnancies is about 15%. 17α -Hydroxyprogesterone
assessed during the initial management. Because dehydration
caproate (brand name Makena) may reduce the inci-
may result in uterine irritability, rehydration may stabilize the
dence o preterm birth when started at 16 to 20 weeks
uterus. T ere is no proven bene t o hydration in the patient
and can be o ered to a woman with a singleton ges-
who is euvolemic. Get cultures to prevent perinatal transmis-
tation and a history o a prior spontaneous preterm
sion even though treatment o positive cultures has not been
singleton birth. Recent data suggests it may be less
established to aid in the prevention o preterm birth. Cortico-
e ective than once thought (Lancet. 2016; Feb 23). I
steroids are discussed earlier.
prior preterm birth was secondary to an incompe-
HELPFUL (AND USELESS IF NOT AT LEAST tent or shortened cervix, then a cerclage should also
INTERESTING) TIP: be considered (ACOG. Obstet Gynecol. 2012;120(4):
Delivery rates in Israel go up around Jewish holiday ast 964–973).
days likely because o the relative dehydration.

Objectives: Did you learn to . . .


• Recognize preterm labor and manage it appropriately? Objective: Did you learn to . . .
• Describe indications or GBS prophylaxis?
• Understand the indications, contraindications, risks, and
bene ts o tocolytics and corticosteroids?
QUICK QUIZ: PRETERM BIRTH
CASE 15.5
A 33-year-old G1 at 35 2/7 weeks resents to labor and deliv- Risk actors or preterm birth include each o the ollowing
ery. Her regnancy has been co licated by reter labor. EXCEP :
Her GBS status is unknown. On ad ission, she is unco ort- A) Multiple gestation pregnancy
able with regular contractions every 3 to 4 inutes. T e FHR B) Maternal bacteriuria
baseline is 135 with oderate variability. Her cervix is 6 c C) Maternal history o preterm contractions with term birth
dilated, co letely e aced, with a bulging a niotic sac and D) Maternal smoking
etus in vertex resentation. E) Maternal hypertension

Question 15.5.1 Which o the ollowing is your best course The correct answer is “C.” While preterm contractions are con-
o management? cerning, a maternal history o preterm contractions with term
A) Administer corticosteroids birth does not increase the risk o preterm delivery in a subse-
B) Administer tocolytics quent pregnancy. Multiple gestations, history o preterm birth,
C) Initiate GBS prophylaxis smoking, cocaine use, asymptomatic bacteriuria, and hyperten-
D) Discharge her to home until she is in active labor sion are all risk actors or preterm birth.
E) A, B, and C
CASE 15.6
Answer 15.5.1 The correct answer is “C.” GBS prophylaxis is
A 37-year-old G3 P0111 ( ull ter , reter , abortions/ is-
indicated or preterm delivery i GBS status is unknown. Appro-
carriage or any reason, living children) resents or routine
priate antibiotics include penicillin or ampicillin or an alterna-
obstetric care at 10 weeks o gestation. Her second regnancy
tive intravenous agent i the patient has a penicillin allergy.
was co licated by reecla sia with severe eatures at
T is patient is outside the window (24–34 weeks) or cortico-
35 weeks requiring induction o labor and agnesiu sul-
steroid administration. Given the gestational age (> 34 weeks),
ate thera y. Her ast edical history is unco licated. Her
advanced cervical dilation, and high likelihood o imminent
blood ressure is 132/86 Hg, urine rotein is negative,
delivery, tocolysis should be avoided.
and hysical exa ination is unre arkable. Uterine size is
HELPFUL TIP: consistent with dates and etal heart tones are auscultated.
Indications or GBS prophylaxis: (1) GBS screen positive T e atient wonders i she will need to deliver early and need
during current pregnancy; (2) GBS bacteriuria anytime dur- agnesiu again.
ing current pregnancy; (3) prior history o giving birth to
a neonate with GBS disease; (4) unknown GBS status plus Question 15.6.1 You counsel her that her risk o recurrent
ever or preterm labor, or prolonged rupture o mem- preeclampsia with severe eatures is in the range o :
branes. No GBS prophylaxis is indicated in women who are A) < 5%
culture negative with ever (but think about chorioamnio- B) 20% to 30%
nitis!), preterm labor, or prolonged rupture o membranes. C) 50% to 60%
D) > 90%
424 FAMILY MEDICINE EXAMINATION & BOARD REVIEW

Answer 15.6.1 The correct answer is “B.” T e risk o recur- D) Second-trimester quadruple screening (hCG, maternal
rence o preeclampsia is a ected by both gestational age at diag- serum alpha etal protein [MSAFP], estriol, inhibin A)
nosis and the severity o preeclampsia. Preeclampsia at an early E) Cell- ree etal DNA testing rom maternal peripheral blood
gestational age or preeclampsia with severe eatures increases F) All o the above
the risk o recurrence. T e overall recurrence risks are estimated
at less than 10% or mild preeclampsia and greater than 20% or Answer 15.6.4 The correct answer is “F.” All patients should be
preeclampsia with severe eatures. Risk actors or preeclampsia o ered rst-trimester screening, and i declined, they should be
include young maternal age, advanced maternal age, diabetes, o ered second-trimester quadruple screening. Because o her
and chronic hypertension, among many others. advanced age (> 35 years at the time o delivery), the patient
should be o ered CVS and amniocentesis as well. Patients
T e atient would like to do anything reasonable to revent should understand that the rst- and second-trimester screen-
reecla sia again. ing tests are just that—screening ethods. However, both CVS
and amniocentesis can diagnose Down syndrome, in addition
Question 15.6.2 In addition to obtaining baseline labora to other chromosomal abnormalities. CVS is typically com-
tory evaluation (CBC, AST, ALT, creatinine, 24 hour urine pleted between 10 and 13 weeks o gestation, whereas amnio-
or protein) to aid in early diagnosis, you recommend the centesis is per ormed a er 15 weeks. Both are invasive proce-
ollowing therapy: dures, which carry risks, including pregnancy loss, rupture
A) Low-sodium diet o membranes, and etal injury. Cell- ree etal DNA testing is
B) Diuretic or hypertension and edema a newer and noninvasive screening test that can be done a er
C) Aspirin 81 mg daily 10 weeks o gestation in patients at high risk or aneuploidy. I
D) Subcutaneous heparin therapy at prophylactic doses abnormal, it should be ollowed up with a diagnostic test, such
E) None o the above as an amniocentesis.

T e atient has a negative rst-tri ester screening, which


Answer 15.6.2 The correct answer is “C.” ACOG’s ask Force
decreases her Down syndro e risk to 1/800. She declines an
on Hypertension in Pregnancy rom November 2013 recom-
a niocentesis. You draw her MSAFP only at 17 weeks, because,
mends the use o 81 mg aspirin daily in patients who have a
as you learned above, the rst-tri ester screening does not
history o early-onset preeclampsia with preterm delivery or in
evaluate or neural tube de ects. T e AFP is nor al. She subse-
patients with a history o preeclampsia in more than one prior
quently undergoes diabetes screening with a 1-hour ost-50-g
pregnancy. USPS F recommends the use o low-dose aspirin as
glucose test, which shows a glucose level o 170 g/dL.
a preventive medication a er 12 weeks o gestation in women
who are at high risk or preeclampsia.
Question 15.6.5 The next step is:
A) Order a 3-hour glucose tolerance test
Because o her age (37 years), the atient also inquires about
B) Set up diabetes teaching and a consult with a nutritionist/
her risk or delivering a baby with Down syndro e.
dietician
C) Start glyburide
Question 15.6.3 What is her estimated risk o a Down syn
D) Start insulin
drome baby with this pregnancy?
E) All o the above
A) 1/9,000
B) 1/1,200
Answer 15.6.5 The correct answer is “A.” T e 1-hour 50-g glu-
C) 1/150
cose load test is a screening test or gestational diabetes. Since
D) 1/12
hers is abnormal (≥ 140 mg/dL), the next step is to per orm a
diagnostic test, which is the asting 3-hour glucose tolerance
Answer 15.6.3 The correct answer is “C.” T e risk o Down test, utilizing a 100-g glucose load.
syndrome begins to rise rapidly at age 35, with an estimated risk
o 1/250 at age 35, 1/150 at age 37, and 1/70 at age 40.
HELPFUL TIP:
A er hearing her age-related risk or Down syndro e, the
The cuto or the 1-hour 50-g glucose load (Glucola) test
atient asks about what tests she should have to screen or
is either ≥ 130 or ≥ 140 mg/dL, and either is currently
Down syndro e.
acceptable. The International Association o Diabetes
in Pregnancy Study Group has proposed new screen-
Question 15.6.4 You counsel her regarding various tests
ing guidelines or gestational diabetes. These would be
and o er:
more aggressive, including universal screening (with
A) First-trimester triple screening (PAPP-A, hCG, and nuchal
asting glucose or A1C) at the initial visit and a glucose
lucency)/integrated screening
level 2 hours a ter a 75-g glucose load in the second tri-
B) Chorionic villus sampling (CVS)
mester that would be used or screening and diagnosis.
C) Amniocentesis
CHAPTER 15 • OBSTETRICS AND WOMEN’S HEALTH 425

T e patient completes her glucose tolerance test with values o : her due date. It is ina ro riate to change her due date based
• Fasting: 92 mg/dL on a 32-week ultrasound (“A”). Initiating an induction would
• 1 hour: 194 mg/dL be inappropriate without urther investigation, given the early
• 2 hours: 169 mg/dL gestational age (“B”). You could do an ultrasound at 1 to 2 weeks
• 3 hours: 148 mg/dL to assess amniotic uid and umbilical Doppler but not etal size.
Antenatal surveillance is indicated at this time with bi-weekly
Question 15.6.6 The next step in management is: nonstress testing.
A) Continue routine prenatal care
B) Set up diabetes teaching and a consult with a nutritionist/ T e atient continues with bi-weekly nonstress tests and her
dietician ultrasound at 35 weeks reveals a ro riate interval growth,
C) Start glyburide but re ains growth-restricted at a weight o 1,846 g (< 10th
D) Start insulin ercentile). oday her blood ressure is 146/88 Hg and
urine rotein on di stick is + 1. Ugh . . . she can’t see to
Answer 15.6.6 The correct answer is “B.” T e recommended catch a break.
upper limit o normal serum glucose levels or the 3-hour glu-
cose tolerance test are: Question 15.6.8 At this time, appropriate intervention
• Fasting: 95 mg/dL includes:
• 1 hour: 180 mg/dL A) Administering corticosteroids
• 2 hours: 155 mg/dL B) Obtaining a 24-hour urine or protein
• 3 hours: 140 mg/dL C) Following up with a routine appointment in 1 week
D) Starting labetalol
I a patient has two or more glucose levels that are above E) Starting lisinopril
these, she is diagnosed with gestational diabetes (GDM).
Your patient ails on three o our o her results. T e recom- Answer 15.6.8 The correct answer is “B.” Given the patient’s
mendation is to initiate dietary modi cations including car- elevated blood pressure and 1+ protein on dipstick, you need
bohydrate restriction with requent blood sugar monitor- to be concerned about recurrent preeclampsia. Urinary excre-
ing. I the target glucoses cannot be met with dietary changes tion o protein may be transient, and a 24-hour urine pro-
alone, medical therapy should be started. Although insulin has tein level is a more accurate re ection o proteinuria and the
been the standard therapy, glyburide or met ormin are also pre erred method to diagnose preeclampsia. Alternatively, a
equally appropriate or rst-line therapy. Insulin is better than protein/creatinine ratio o at least 0.3 mg/dL can also be used
met ormin, which is better than glyburide (BMJ. 2015; 350: or preeclampsia diagnosis. “A” is incorrect. T e patient is at
h102). 35 weeks o gestation (although measuring smaller), which is
beyond the recommended gestation at which corticosteroids
T e atient is seen or her 32-week visit and her undal height are administered (24–34 weeks). T is patient must be ollowed
is only 28 c (which sur rises you, given her diagnosis o up in a couple o days so that you don’t miss the diagnosis o
gestational diabetes . . . the babies o diabetic others tend to preeclampsia. Although anti-hypertensives such as labetalol
be large!). She has been co liant with the dietary changes can be utilized in pregnancy, they are not routinely initiated
and her blood sugars are usually 80s asting and 120s 2-hour or mild elevations in blood pressure at later gestations. We
ost randial. She has gained 20 lb so ar in the regnancy. hope you avoided “E,” as ACE inhibitors are contraindicated in
You send her or an ultrasound, which reveals an in ant pregnancy.
easuring only 28 3/7 weeks o gestation, weighing 1,168 g
(< 10th ercentile). A niotic uid volu e and u bilical Serial blood ressure easure ents in the clinic reveal no
artery Do lers are nor al. blood ressures greater than 146/88. Her nonstress test is
reactive. T e atient is sent ho e to collect her 24-hour
urine and returns in 2 days. Her blood ressure is now
Question 15.6.7 Appropriate ollow up includes:
148/90 Hg and she has trace rotein on urine di stick.
A) Changing the estimated due date
T e 24-hour urine returns at 180 g (her baseline 24-hour
B) Scheduling an induction
urine rotein at the beginning o the regnancy was 116
C) Repeating the ultrasound or growth in 1 week
g). She denies any headache, visual changes, nausea, or
D) Repeating the ultrasound or growth in 4 weeks
abdo inal ain.
Answer 15.6.7 The correct answer is “D.” Current ultrasound
techniques are not sensitive enough to assess growth at weekly Question 15.6.9 Your diagnosis is:
intervals (“C”), and there ore waiting 4 weeks would give a bet- A) Gestational hypertension
ter assessment o growth rate. T is in ant demonstrates intra- B) Preeclampsia
uterine growth restriction. T e patient had a rst-trimester C) Preeclampsia with severe eatures
ultrasound with her rst-trimester screening, which establishes D) Acute renal ailure
426 FAMILY MEDICINE EXAMINATION & BOARD REVIEW

TABLE 15-4 DIAGNOSTIC CRITERIA FOR She returns to the of ce or her a oint ent 5 days later at
PREECLAMPSIA 36 weeks o gestation a er having elt well over the weekend.
2013 Diagnostic Criteria for Preeclampsia However, today she develo ed a headache. Her blood res-
Blood pressure: sure is 166/112 and her urine di stick reveals 3+ rotein.
• ≥ 140 mm Hg systolic or > 90 mm Hg diastolic on TWO occasions at
least 4 hours apart AFTER 20 weeks o gestation in a woman with
history o previously normal blood pressure Question 15.6.10 Your next step is to:
• ≥ 160 mm Hg systolic or ≥ 110 mm Hg diastolic; hypertension can A) Start oral labetalol and see the patient back in 2 days or a
be con rmed within a short interval (minutes) to acilitate timely blood pressure check
antihypertensive therapy B) Repeat the 24-hour urine or protein
AND
C) Admit to labor and delivery or blood work and monitoring,
• Proteinuria:
• ≥ 300 mg per 24-hour urine collection (or this amount with plans to move toward delivery
extrapolated orm a timed collection) D) Immediate Cesarean section
OR
• Urine protein:creatinine ratio greater than or equal to 0.3 with Answer 15.6.10 The correct answer is “C.” T e clinical pic-
each measured in mg/dL
ture is now developing into preeclampsia with severe eatures
OR
• Dipstick reading o 1+ (used only i other quantitative methods (headache, systolic BP > 160, diastolic BP > 110, and 3+ pro-
aren’t available) teinuria . . . see able 15-4). T e patient needs to be admitted
OR or urther monitoring o blood pressure and symptoms. In
In the absence o proteinuria, new-onset hypertension with the onset addition, blood work should be obtained including a CBC and
o any o the ollowing (laboratory values urther de ned under
liver and renal studies. Starting oral labetalol would treat the
“Diagnosis o Preeclampsia with Severe Features below):
patient’s hypertension, but this step alone is not prudent or a
Thrombocytopenia
• Renal insu ciency
patient who appears to have preeclampsia with severe eatures.
• Impaired liver unction T e 24-hour urine protein collection may be help ul in meeting
• Pulmonary edema the technical criteria to diagnose preeclampsia and can be done
• Cerebral or visual symptoms during admission; however, the results would not change the
Diagnose “Preeclampsia with Severe Features” when there is the
immediate management o the patient. I a aster evaluation is
presence of any of the following: needed, the protein/creatinine ratio could be done in this case.
• Blood pressure > 160 mm Hg systolic or ≥ 110 mm Hg diastolic on However, as noted above, preeclampsia with severe eatures
two occasions at least 4 hours apart while the patient is on bed rest can also be diagnosed without the presence o proteinuria in
(unless antihypertensive therapy is initiated be ore this time) the presence o certain clinical and/or laboratory ndings (see
• Platelet count o less than 100,000/µL (thrombocytopenia)
• Serum creatinine concentrations greater than 1.1 mg/dL or a
able 15-4).
doubling o the serum creatinine concentration in the absence o
other renal disease (renal insu ciency)
• Elevated blood concentrations o liver transaminases to 2× normal HELPFUL TIP:
concentration, severe persistent right upper quadrant or epigastric In a preeclamptic patient, blood pressure can be con-
pain unresponsive to medication and not accounted or by
alternative diagnoses or both (impaired liver unction)
trolled with labetalol, hydralazine, or oral ni edipine.
• Pulmonary edema Avoid nitroprusside. The downside o BP control is that
• New-onset cerebral or visual symptoms it reduces placental low. Also, treating the BP has
no effect on the course of preeclampsia. Therefore,
Adapted rom American College o Obstetricians and Gynecologists. treat BP only if > 160/100 mm Hg or the patient is having
Executive summary: hypertension in pregnancy. Obstet Gynecol. 2013;
end-organ symptoms. Further in ormation on acute and
122(5):1122–1131.
emergent management o severe hypertension can be
Answer 15.6.9 The correct answer is “A.” She now has two ound in Am Obstet Gyencol. 2015;(623):1–5.
blood pressure readings greater than 140/90 mmHg and more
than 6-hours apart, which satis y the criteria or hyperten-
sion. Given that this elevation in blood pressure started a er You ad it her to the hos ital. Re eat blood ressure is
20 weeks o gestation, it is likely pregnancy related. She does 164/98 Hg and urine di stick shows 3+ rotein. Her
not have protein > 300 mg in a 24-hour urine collection, so she cervix is so , 2 c dilated, 50% e aced, with etus vertex at
does not meet that diagnostic criterion or preeclampsia and − 2 station.
does not have symptoms that would quali y her or a diagnosis
o preeclampsia with severe eatures. Note that proteinuria is no Question 15.6.11 What is the most appropriate interven
longer required to make the diagnosis o preeclampsia. Another tion at this point?
abnormality such as elevated LFTs, thrombocytopenia, renal A) Begin induction o labor
insuf ciency, CHF, or CNS symptoms can make the diagnosis o B) Start magnesium sul ate
preeclampsia even in the absence o proteinuria. T e diagnostic C) Prepare or a cesarean delivery in case it is needed
criteria or preeclampsia are outlined in able 15-4. D) All o the above
CHAPTER 15 • OBSTETRICS AND WOMEN’S HEALTH 427

Answer 15.6.11 The correct answer is “D.” All o these options Objectives: Did you learn to . . .
are important to consider at this time. Induction with oxytocin • Screen or, diagnose, and manage gestational diabetes?
and treatment o preeclampsia with magnesium are appropri- • Evaluate, diagnose, and manage hypertension in preg-
ate at this point. Obstetrical backup should be involved earlier nancy?
rather than later unless you possess the skill to do the cesarean • De ne and manage preeclampsia?
section yoursel . • Identi y intrauterine growth restriction?

HELPFUL TIP:
Delivery o the baby (baby and placenta, really) is the
CASE 15.7
ultimate treatment or preeclampsia and should be ini- Now, you have to rush ro L&D back to clinic (this never
tiated as soon as easible when the mother’s condition ha ens, right?) where you eet a 31-year-old wo an or
demands it. reconce tion counseling. She has a history o hy ertension
and a heart ur ur. She has dys nea when cli bing stairs
but er or s nor al activities o daily living with ini-
You begin agnesiu sul ate or the reecla sia and oxy- al dif culty. She takes lisino ril 10 g daily or her blood
tocin or induction. T e induction roceeds without inci- ressure. On hysical exa ination, heart rate is 82 b and
dent, and she delivers a viable ale in ant. blood ressure is 138/90 Hg. Height is 5′ 6″ and weight is
160 lb. She a ears well.
Question 15.6.12 How long will you continue the magne
sium sul ate? Question 15.7.1 Regarding the management o her chronic
A) Until delivery o the placenta hypertension during pregnancy, which is the most appro
B) For 12 hours a er delivery priate next step?
C) For 24 hours a er delivery A) Discontinue lisinopril and begin methyldopa, labetolol, or
D) Until the urine protein dipstick is negative ni edipine and recheck blood pressure in 2 weeks
E) Until discharge B) Increase lisinopril to 20 mg daily and recheck blood pressure
in 2 weeks
Answer 15.6.12 The correct answer is “C.” reatment should C) Make no changes at this time
be continued or 24 hours ollowing delivery. D) Discontinue lisinopril; recheck blood pressure in 2 weeks
E) Either A or D is correct
HELPFUL TIP:
Monitor deep tendon re lexes, level o consciousness, Answer 15.7.1 The correct answer is “E.” ACE inhibitors are
and urine output or all patients on magnesium. Turn contraindicated in pregnancy; there ore, a woman contemplat-
o the magnesium in usion i signs o toxicity emerge ing pregnancy should discontinue the medication or replace it
(e.g., decreased mental status, hypore lexia) or i the with a sa er alternative. In act, or a woman capable o conceiv-
patient is at risk or impending toxicity (e.g., rom renal ing, ACE inhibitor use is discouraged unless no better alter-
ailure). native exists and the patient is using a reliable orm o birth
control. Women with mild chronic hypertension (systolic
blood pressure 140–160 mm Hg) have a low risk or cardio-
ANOTHER HELPFUL TIP:
vascular complications during pregnancy and can be managed
Remember all those “mag checks” you did every 2
with nonpharmacologic therapy as long as asymptomatic. In
hours as a medical student? Well, for pa tients with pre-
2 weeks, when she has her return visit, i she is hypertensive
ecla mpsia without severe fea tures, ACOG no longer
(> 140–160/> 100 mm Hg), select a medication regarded
recommends ma gnesium sulfa te a s a routine inter-
as sa e during pregnancy; methyldopa, ni edipine, or labet-
vention (again, look at Table 15-4). Not a total waste
alol would be pre erred. You could also start one o these at
o time, right? “Mag checks” did improve your rapport
the current visit while discontinuing the ACE inhibitor. Note
with your patients . . . maybe. Remember that the pur-
that the ACOG recommendation is to treat chronic hyperten-
pose o magnesium sul ate in usion is to reduce the risk
sion in pregnancy only i the systolic BP is >160/105 mm Hg. I
o seizures (i.e., the progression to eclampsia). The NNT
the patient has hypertension secondary to preeclampsia, ACOG
or mild preeclampsia is 100.
would recommend treating only i the blood pressure is greater
than >160/110 mm Hg or i there are symptoms. T ese recom-
mendations are controversial and many would initiate blood
YET ONE MORE HELPFUL TIP:
pressure control in both groups at a lower BP. reating hyper-
According to ACOG guidelines, bed rest should not be
tension reduces the incidence o “severe” hypertension, but
prescribed or patients with gestational hypertension or
does not change outcomes (N Engl J Med. 2015;372:407–417).
preeclampsia without severe eatures.
See also Obstet Gynecol. 2013;122:1122 i you are interested in
this topic.
428 FAMILY MEDICINE EXAMINATION & BOARD REVIEW

Question 15.7.2 Which o the ollowing statements about one pill twice a day or 1 day, and then nish out the pack).
cardiovascular physiology in pregnancy is INCORRECT? Conjugated estrogens are a third option. ranexamic acid (an
A) Blood volume and cardiac output increase by approximately anti brinolytic) 1.3 g orally ID or 5 days is an additional alter-
50% during pregnancy native or patients with contraindications to hormonal therapy.
B) Heart rate increases by 10 to 20 beats per minute, peaking in
the third trimester Question 15.8.2 The most common side e ect o medroxy
C) Systemic arterial pressure increases during the rst trimes- progesterone at the above dose includes:
ter, reaches a peak in mid-pregnancy, and remains at that A) Bone marrow suppression
level until labor and delivery B) T romboembolic disease
D) Le ventricular ejection raction remains constant or C) Depression
increases slightly throughout pregnancy D) Sore breasts
E) A temporary rise in venous return immediately ollowing
delivery may lead to a substantial rise in le ventricular ll- Answer 15.8.2 The correct answer is “C.” Many women will
ing pressure and cardiac decompensation in women with become depressed when taking this dose o medroxyproges-
certain types o heart disease terone. T ey should be made aware o this ahead o time. T e
major side e ect o the OCP regimen or abnormal uterine
Answer 15.7.2 The correct answer (and what does not hap bleeding is nausea and vomiting. Consider giving the OCP regi-
pen in pregnancy) is “C.” Systemic arterial pressure decreases men with an antiemetic. “B” is o note. Progesterones alone do
during the rst trimester, reaches a nadir during the second tri- not increase clot risk. T ey do inhibit thrombolysis, however.
mester, and returns to prepregnancy levels in the third trimes-
ter. T e other statements are true. “E” deserves special mention.
Immediately ollowing delivery, relie o caval compression may HELPFUL TIP:
cause a rise in venous return leading to clinical deterioration in Another long-term option or patients with chronic ab-
some women with heart disease. normal uterine bleeding, a ter workup has been com-
pleted, is Mirena IUD.
Objectives: Did you learn to . . .
• Manage hypertension during preconception counseling?
• Recognize normal cardiac physiological changes associated Question 15.8.3 Causes o this patient’s bleeding could
with pregnancy? include all o the ollowing EXCEPT:
A) Hypothyroidism
B) Von Willebrand disease
CASE 15.8 C) Prolactinoma
You are seeing a 28-year-old e ale who’s LMP was a roxi- D) Parathyroid disease
ately 7 days ago. She is co laining o vaginal bleeding and E) Uterine cancer
states that she is going through one ad every 20 inutes.
Needless to say she is concerned. She is not using anything Answer 15.8.3 The correct answer is “D.” All o the remain-
or contrace tion. Her vital signs and examination are unre- ing can cause abnormal bleeding. Obviously, uterine cancer
markable except for blood at the os. would be unlikely in a 28-year-old. Other causes o bleeding to
consider are cervical polyps, endometrial polyps, adenomyo-
Question 15.8.1 A ter ruling out pregnancy and assuring sis, broids, and endocrine problems such as PCOS and hypo/
that her hemoglobin is stable, what is your next step? hyperthyroidism, etc.
A) Fresh rozen plasma Objective: Did you learn to . . .
B) Cyclic medroxyprogesterone 10 mg a day • Evaluate a patient with abnormal uterine bleeding?
C) DDAVP to maximize platelet unction • Treat a patient with abnormal uterine bleeding?
D) Observation: nonintervention is the best policy i the hemo-
globin is normal
CASE 15.9
Answer 15.8.1 The correct answer is “B.” T is patient has A 37-year-old G3 P1112 ( ull ter , reter , abortions/ is-
abnormal uterine bleeding, which is a broad term that includes carriage ro any cause, living children) resents or an
di erent unctional and structural causes o uterine bleeding. annual gynecologic exa ination and has questions regard-
Dys unctional uterine bleeding (DUB) secondary to anovu- ing contrace tion. She is wondering i she is a candidate or
lation is a very common cause o abnormal uterine bleeding. an IUD.
One popular regimen or DUB is medroxyprogesterone 10 mg/
day or 10 days. T is should be ollowed by a withdrawal bleed. Question 15.9.1 All o the ollowing are contraindications
T is regimen can be repeated or the rst 10 days o the next to IUD placement EXCEPT:
3 months. Other options include starting a monophasic OCP A) Pregnancy
(three pills twice a day or 1 day, two pills twice a day or 1 day, B) Acute pelvic in ection
CHAPTER 15 • OBSTETRICS AND WOMEN’S HEALTH 429

C) Undiagnosed vaginal bleeding C) CA-125


D) History o chlamydia in ection D) Pelvic ultrasound
E) Pregnancy test
Answer 15.9.1 The correct answer is “D.” Although active cer-
vicitis and acute pelvic in ammatory disease (PID) are contra- Answer 15.9.3 The correct answer is “C.” A CBC will pro-
indications to placement o an IUD, a prior chlamydia in ection vide in ormation regarding the hematocrit (and there ore the
does not exclude a patient rom obtaining an IUD. Undiag- level o anemia) and platelet count—important in ormation or
nosed vaginal bleeding can be a sign o either endometritis someone with heavy menstrual bleeding. SH will screen or
or structural abnormalities o the uterine cavity, which must hypothyroidism, which is a common cause o menorrhagia in a
be addressed prior to considering an IUD. I you chose “A,” 39-year-old emale. A pelvic ultrasound will aid in the evalua-
some goons rom the Board are coming to relieve you o your tion o the mass palpated on examination and characterize the
certi cation. location and size o any uterine broids that may contribute
to the bleeding; it will also evaluate or any adnexal masses. A
Your atient has none o these contraindications, and she pregnancy test is always a good idea to rule out possible preg-
would like to roceed with the IUD. Her riends have told nancy, ectopic pregnancy, and miscarriage as an underlying eti-
her that all IUDs cause abortions, and she is unsettled by this ology. CA-125 is a nonspeci c tumor marker that has a high
idea. alse-positive rate in premenopausal women. T us, CA-125
is not recommended as a screening test or ovarian cancer. It
Question 15.9.2 The mechanism o action o IUDs (copper can be used as an adjunct to pelvic ultrasound when a complex
or progesterone) is: adnexal mass is identi ed.
A) Aborti acient
B) Causes a sterile in ammatory reaction to a oreign body T e atient returns in 2 weeks ollowing her ultrasound.
C) Impairs sperm transport rom the cervix to the allopian Her he atocrit was 31% (indices are consistent with iron-
tube de ciency ane ia), latelets 195,000, and SH 2.8 U/L
D) B and C (nor al). T e ultrasound reveals a uterus easuring 12 × 8 ×
E) All o the above 6 c , with ulti le s all intra ural broids easuring less
than 2 c in dia eter. T ere is one subserosal, edunculated
Answer 15.9.2 The correct answer is “D.” Studies detecting broid easuring 3.5 c at the undus.
levels o hCG reveal that this hormone is not present in IUD
users during the luteal phase. T us, the IUD is NO an abor- Question 15.9.4 What is the most appropriate initial man
ti acient. Studies suggest that the mechanism o action o IUDs agement?
includes inter erence with sperm transport rom the cervix to A) Expectant management and reassurance
the allopian tube, inhibition o sperm survival, and endome- B) Nonsteroidal anti-in ammatory drugs (NSAIDs)
trial in ammatory changes that inhibit implantation. C) Gonadotropin releasing hormone (GnRH) agonists
D) Blood trans usion
E) Hysterectomy
HELPFUL TIP:
The progesterone-releasing Mirena IUD is approved or Answer 15.9.4 The correct answer is “B.” Most broids are
5 years o contraception versus 10 years or the copper asymptomatic, although the most common symptom associated
IUD. Skyla is approved or only 3 years. with leiomyomata ( broids) is abnormal bleeding. T is patient’s
broids are not impinging on the uterine cavity but may be
contributing to the patient’s menorrhagia. Initial treatment o
A er all o that counseling (that you can’t bill or), her hus- menorrhagia may include NSAIDs that inhibit prostaglandin
band undergoes a vasecto y. During a routine a oint ent synthesis. NSAIDs have been shown to reduce menstrual blood
2 years later, she co lains o worsening enorrhagia. She loss by 30% to 50% in women with menorrhagia. GnRH agonists
denies inter enstrual s otting. She has not noticed any may be utilized to produce a medical menopause. However, they
lightheadedness or dizziness, but does co lain o gener- are expensive, and long-term usage is associated with signi cant
alized atigue. On hysical exa ination, you nd a nor al side e ects (e.g., osteoporosis). Hysterectomy is the de nitive
sized thyroid and an enlarged, irregular uterus easuring treatment or leiomyomata in symptomatic women who have
10 to 12 weeks in size. T ere are no distinct adnexal asses, completed childbearing. However, the mortality associated with
but this is so ewhat dif cult to discern due to the irregular hysterectomy is approximately 0.5/1,000, at that age. It is gener-
uterus. ally reserved or women who have ailed medical management
or have symptoms or signs related to broid size. O particular
Question 15.9.3 Your initial workup in this 39 year old note, levonorgestrel-releasing IUD can be used as an alternative.
emale should include all o the ollowing EXCEPT: Studies have shown a reduction in uterine volume and bleeding
A) CBC and an increase in hematocrit a er placement o IUD. I the
B) SH broids are intracavitary, an IUD should NO be used.
430 FAMILY MEDICINE EXAMINATION & BOARD REVIEW

6 onths ago. Her exa ination reveals no abnor al hair


You start an NSAID. She res onds oderately well. You add growth, no acne, nor al anner stage V breast develo ent,
an OCP, which adequately controls her sy to s. I the nor al external genitalia and cervix, with a slightly enlarged
atient had not res onded to edical thera y, you ight uterus at 10 weeks size, and no adnexal asses.
have considered re erral to a gynecologist or otential surgi-
cal thera y. Question 15.10.1 What is the rst test you should order?
A) Pelvic ultrasound
Question 15.9.5 What other characteristic(s) would have B) Serum prolactin level
prompted evaluation or surgery? C) Urine hCG
A) Prolapsing broid through the cervix D) Serum FSH
B) 5-cm submucosal myoma protruding 50% into the uterine E) Serum SH
cavity
C) Rapidly enlarging uterus
D) 20-week sized uterus and pelvic pressure Answer 15.10.1 The correct answer is “C.” Primary amen-
E) All o the above orrhea is de ned as either (1) the absence o menarche by 15
years with secondary sexual characteristics or (2) the absence o
Answer 15.9.5 The correct answer is “E.” A broid prolapsing menarche by 13 years o age in the absence o secondary sexual
through the cervix has a potential or necrosis and in ection and characteristics. Secondary amenorrhea is de ned as the absence
may require surgical removal. A submucosal myoma, especially o periods or 3 cycles in a woman who previously had menses.
one that distorts the uterine cavity, can contribute to menorrha- Pregnancy is the most common cause o secondary amenorrhea
gia. A rapidly enlarging uterus would be concerning or a uter- and thus must always be ruled out. A er pregnancy has been
ine malignancy and would require urther investigation. Larger excluded, the diagnostic ocus is on di erentiating between
uterine broids are more likely to contribute to symptoms o anatomic cause, ovarian ailure, and endocrine abnormalities.
pelvic pressure and pain, which may only respond to surgical T e other tests may be done at some point, but a pregnancy test
correction. is rst.

T e atient returns in 1 year or her annual exa ination, and T e urine regnancy test is ositive. Because o her irregular
her elvic exa ination is unchanged. However, she ex resses enses, you schedule her in 1 week or a vaginal ultrasound
concern that these broids could beco e cancerous. to con r her gestational age. However, she calls your nurse
in 3 days with vaginal s otting and lower elvic cra ing.
Question 15.9.6 What is the risk o uterine malignancy
(leiomyosarcoma) in a patient with broids? Question 15.10.2 The most important diagnosis to con rm
A) < 1% or exclude is:
B) 5% to 10% A) Spontaneous abortion
C) 40% to 50% B) Incomplete abortion
D) 90% to 95% C) Inevitable abortion
D) Ectopic pregnancy
Answer 15.9.6 The correct answer is “A.” T e estimated inci- E) win gestation
dence o leiomyosarcoma discovered at the time o surgery
or broids is less than 1%. A leiomyosarcoma is a malignant Answer 15.10.2 The correct answer is “D.” Ectopic pregnancy
tumor that does not arise rom preexisting benign leiomyoma. is the leading cause o pregnancy-related death during the rst
Leiomyosarcomas typically arise in the h or sixth decade o trimester in the United States. Risk actors or ectopic pregnancy
li e and are usually associated with abnormal bleeding or a rap- include prior PID, prior ectopic pregnancy, and prior tubal
idly enlarging uterus. or other pelvic surgery. Spontaneous, incomplete, inevitable,
threatened, and missed abortions (or miscarriages) are terms to
Objectives: Did you learn to . . .
describe pregnancy loss occurring be ore 20 weeks o gestation.
• Recognized contraindications to IUD use?
Each can result in signi cant maternal morbidity (including
• Treat menorrhagia?
hemorrhage and in ection) or death. However, these outcomes
• Describe management principles or uterine leiomyomas? are less requent than with an ectopic pregnancy.

Question 15.10.3 Which o the ollowing tests is/are impor


CASE 15.10 tant in the management o this patient?
A 20-year-old nulligravid e ale resents or evaluation A) Progesterone level
o irregular enstrual cycles. Her ast edical history is B) Serum quantitative β -hCG
unco licated. Her gynecologic history is re arkable or C) Hematocrit, blood type, and screen
enarche at age 12 years, irregular enses occurring every D) Pelvic ultrasound
21 to 40 days, and lasting 5 days, with last enstrual eriod E) All o the above
CHAPTER 15 • OBSTETRICS AND WOMEN’S HEALTH 431

Answer 15.10.3 The correct answer is “E.” “A,” progesterone


T e atient returns a er her ultrasound, which reveals an
level, helps to assess the viability o the pregnancy. A proges-
intrauterine regnancy easuring 6 weeks, 5 days gestation
terone level < 5 ng/mL is non-reassuring (85% spontaneous
with cardiac activity at 110 beats er inute. T e laboratory
abortion, 14% ectopic pregnancy), and a level > 25 ng/mL is
sta called you to let you know that the antibody screen was
reassuring (< 2% ectopic pregnancies). wo serum quantitative
in error, and in act the atient is A negative, antibody screen
β -hCGs obtained 48 hours apart help assess viability. A rise in
negative. T ere ore, you ad inister RhoGAM.
level o > 66% will occur in 85% o normal pregnancies, but only
17% o ectopic pregnancies. ransvaginal ultrasounds should be
Question 15.10.5 You counsel the patient that her diagno
able to detect an intrauterine pregnancy at hCG levels o 1,500
sis at this time is:
to 2,000 IU. With hCG levels below 1,500 IU, an ultrasound is
A) T reatened abortion
o en still help ul or identi ying an adnexal mass and potential
B) Missed abortion
ectopic pregnancy. A blood type and screen and hematocrit are
C) Complete abortion
important to evaluate or Rh status and anemia and prepare or
D) Ectopic pregnancy
trans usion should the hemorrhage be signi cant.
E) All remain in the di erential
U on resentation, the atient has nor al vitals and a ears
Answer 15.10.5 The correct answer is “A.” T reatened abor-
clinically stable. T e blood work returns with a he atocrit
tion is de ned as any vaginal bleeding in the rst trimester or up
o 38% and blood type A negative, antibody screen positive
to 20 weeks o gestation, which accompanies a currently viable
for anti-D. T e hCG level is 5,500 IU and rogesterone is
intrauterine pregnancy with a closed cervix. A missed abortion
18 ng/ L. She has not received any treat ent or this bleed-
is retention o dead products o conception in utero. A complete
ing rior to co ing to your of ce.
abortion indicates the pregnancy and all products o conception
have passed rom the uterus.
Question 15.10.4 While waiting or her ultrasound, you
counsel her regarding her blood work, which indicates:
Over the next 2 days, her cra ing and bleeding increase and
A) She should receive RhoGAM immediately, given that she is
she asses tissue (co lete abortion). T e atient is so ewhat
Rh negative
anxious regarding her ertility and is concerned that she ay
B) She has previously been exposed to the D antigen and has
have so ething “wrong” with her that led to the iscarriage.
developed antibodies
C) T is etus is D antigen positive and she has developed anti-
Question 15.10.6 What is the most likely etiology o this
bodies to this etus
miscarriage?
D) She should receive RhoGAM only a er con rming that she
A) Uterine anomalies
has an intrauterine pregnancy
B) Maternal in ection
C) Undiagnosed maternal diabetes
Answer 15.10.4 The correct answer is “B.” Rh “negative” or
D) Embryonic chromosomal abnormality
“positive” re ers to the D antigen, the antigen that is responsible
or most cases o Rh sensitization. RhoGAM, or anti-D immune
Answer 15.10.6 The correct answer is “D.” Approximately
globulin, is used to prevent the development o Rh-D antibodies.
50% o rst trimester spontaneous miscarriages are due to chro-
I Rh-D antibodies are already present (i.e., the patient has
mosomal abnormalities. Some o these occur prior to clinical
already been sensitized), RhoGAM is not e ective. T is is why
recognition o the pregnancy and will go unrecognized by the
options “A” and “D” are incorrect. T is patient has evidence o
patient. All o the others can be responsible or a rst-trimester
antibodies and has previously been exposed to the Rh-D anti-
miscarriage as well but are less common.
gen. “C” is incorrect because not enough time has elapsed with
her current bleeding to have induced antibody production to
the D antigen. Without RhoGAM, 17% o mothers become sen- HELPFUL TIP:
sitized. RhoGAM is recommended with ectopic pregnancies, The rate o spontaneous abortion increases with
spontaneous abortions, induced abortions, threatened abor- advancing maternal age. For women younger than
tions, amniocentesis, antepartum hemorrhage, and routinely at 20 years with recognized pregnancies, the rate o spon-
28 weeks o gestation or Rh-D-negative women who are not taneous abortion is about 12%; the rate increases to
already sensitized. 26% in women older than 40 years.

HELPFUL TIP:
I the Rh-D-negative woman is positive or D antibody,
Question 15.10.7 The next step in the care o this patient is:
either she has previously been exposed to the D-antigen
A) Hysterosalpingogram
and mounted a response or she was given anti-D
B) Gonorrhea and chlamydia cultures
immune globulin during the previous 12 weeks (and it
C) Glucose screening
is still present).
D) Paternal chromosome testing
E) Counseling and reassurance
432 FAMILY MEDICINE EXAMINATION & BOARD REVIEW

Answer 15.10.7 The correct answer is “E.” Evaluation to Question 15.11.2 Risk actors or etal macrosomia include
determine the cause o this patient’s pregnancy loss is not rec- all o the ollowing EXCEPT:
ommended. In general, evaluation is not recommended or a A) Gestational age
single rst-trimester spontaneous loss i the woman is otherwise B) Maternal smoking
healthy. Patients with 2 to 3 consecutive spontaneous pregnancy C) Excessive maternal weight gain
losses are candidates or an evaluation to determine the etiology. D) Multiparity (not a multiple gestation)
E) Macrosomia in a prior in ant
Objectives: Did you learn to . . .
• Maintain a high degree o suspicion or pregnancy in a pa-
Answer 15.11.2 The correct answer is “B.” Maternal smoking
tient presenting with secondary amenorrhea?
is associated with restricted etal growth. All o the others are
• Identi y causes o rst-trimester bleeding?
associated with macrosomia. Additionally, male etus and high
• De ne terminology used in spontaneous abortion (e.g.,
maternal birth weight, and bad karma in a previous li e are asso-
missed, completed, and threatened)?
ciated with macrosomia.
• Manage early pregnancy loss?
• Describe Rh-D isoimmunization? Ultrasound ndings de onstrate a etus in the vertex resen-
tation with an esti ated etal weight o 4,200 g (9 lb 2 oz) and
CASE 15.11 an a niotic uid index o 12.6 c (nor al). You check her
You are now assu ing care o a 28-year-old G3 P2002 at cervix and note that she is 1 c dilated, 50% e aced, with
38 3/7 weeks. Her regnancy has been unco licated. On vertex at 0 station.
exa ination, you note her blood ressure is 102/66
Hg, urine di stick is negative or rotein and glucose, etal Question 15.11.3 The optimal management at this time is:
heart tones are 154 beats er inute, and the undal height is A) Induction o labor
44 c . In reviewing her ante artu record, you note that her B) Cesarean section
revious babies weighed 8 lb 8 oz and 9 lb 6 oz at delivery. She C) Expectant management
has no history o shoulder dystocia. During this regnancy, D) Repeat the ultrasound weekly
her lab results have been nor al, and her 1-hour glucose E) Initiate a weight loss program
a er 75 g o glucose was 127 g/dL at 28 weeks.
Answer 15.11.3 The correct answer is “C.” Fetal macrosomia
Question 15.11.1 Noting that she has a size –date discrep is generally de ned as a birth weight greater than 4,500 g. Large
ancy, what is your next step in the evaluation and manage or gestational age implies a birth weight greater than the 90th
ment o this patient? percentile or a given gestational age (or 4,000 g at delivery).
A) 3-hour glucose tolerance test Expectant management with spontaneous labor onset gener-
B) Nonstress test ally has been shown to have the best outcomes; all interventions
C) Fetal ultrasound or growth and amniotic uid index entail an increase in etal and maternal morbidity.
D) Immediate induction o labor
E) Contraction stress test (CS ) HELPFUL TIP:
Answer 15.11.1 The correct answer is “C.” Size–date discrep- According to ACOG (2009), prophylactic cesarean
ancy can be caused by numerous maternal– etal actors. Given section should be o ered or estimated etal weight
the patient’s history, the likely etiologies are either etal macro- > 5,000 g in nondiabetic pregnancies and > 4,500 g in
somia or polyhydramnios. Additionally, her body habitus could diabetic pregnancies.
render the undal height measurement inaccurate (her weight is
not noted). o best determine the etiology, an ultrasound with HELPFUL TIP:
amniotic uid index is warranted. A 3-hour glucose tolerance test Maternal risk rom macrosomia is primarily related to
(“A”) is not indicated since her screening glucose tolerance test labor abnormalities and includes postpartum hemor-
was normal. A nonstress test (“B”) may be warranted depend- rhage, signi icant vaginal lacerations, cesarean delivery,
ing on the outcome o the etal ultrasound and amniotic uid and in ection.
index. However, i growth and uid were normal, a nonstress test
would not be indicated. T e CS (“E”) is utilized to assess etal
well-being in utero. A CS is done by administering oxytocin to Contractions start s ontaneously. Shortly a er arrival to
induce contractions and observing the resulting FHR tracing. All labor and delivery at 39 weeks, your atient has s ontane-
else being equal, a CS is not warranted at this time. ous ru ture o e branes with clear uid and requests an
HELPFUL TIP: e idural or anage ent o labor ain. Her cervix is 5 c
Controversy surrounds the issue o induction or etal dilated, 90%e aced, and 0 station. FHR baseline is 145 b
macrosomia. Induction or macrosomia has not been with oderate variability and no decelerations (a Category I
shown to improve maternal or etal outcomes. tracing). Four hours later, she is co ortable a er e idural is
laced. Her cervix is unchanged.
CHAPTER 15 • OBSTETRICS AND WOMEN’S HEALTH 433

Question 15.11.4 Appropriate intervention at this time is:


A) Induction o labor In the sa e way that carrying an u brella revents a thun-
B) Augmentation o labor derstor , erely being re ared or dystocia obviates the
C) Cesarean section need or s ecial aneuvers. Ha y day! i e to talk to her
D) Expectant management about nursing.
E) Ultrasound to con rm vertex presentation
Question 15.11.6 You tell her the bene ts o breast eeding,
Answer 15.11.4 The correct answer is “B.” Her labor has which include all the ollowing EXCEPT:
su ered rom arrest o dilation and descent. At this time, the A) Species-speci c and age-speci c nutrients or in ants
appropriate management is augmentation o labor. Induction B) Adequate iron or premature newborns
o labor is the technical term or initiating uterine contrac- C) High level o immune protection rom colostrum
tions be ore the onset o spontaneous labor, so “A” is incorrect. D) Decreased risk o breast and ovarian cancer in mothers who
While expectant management may be acceptable at this point breast eed
a er counseling the patient, there is a risk o prolonged rupture E) Fewer illnesses while the in ant is breast ed
o membranes and chorioamnionitis. Cesarean section at this
point would be premature and unnecessary. Answer 15.11.6 The correct answer is “B.” T e bene ts o
breast eeding or in ants, women, amilies, and society are well
With IV oxytocin, she roceeds to co lete and a er 2 hours o documented. However, human milk may not provide adequate
ushing is ready to deliver. You antici ate a shoulder dystocia. iron or re ature newborns, term in ants whose mothers have
low iron stores, and in ants older than 6 months. All the other
Question 15.11.5 Appropriate maneuvers to reduce a choices are correct.
shoulder dystocia include all o the ollowing EXCEPT:
A) McRoberts maneuver ( exing the patient’s knees up against
the abdomen) HELPFUL TIP:
B) Suprapubic pressure Breast milk should only be kept at room temperature or
C) Fundal pressure 6 hours. However, it is sa est to re rigerate it or 4 days
D) Delivery o the posterior arm maximum or reeze immediately. Do not heat re rig-
E) Woods corkscrew erated breast milk in the microwave, as it will destroy
valuable micronutrients. Warming in hot water is a bet-
Answer 15.11.5 The correct answer is “C.” Fundal pressure ter way o reheating.
may urther worsen impaction o the shoulder and may result in
uterine rupture—both are generally considered bad outcomes.
All the others may be used to help relieve shoulder dystocia.
o manage dystocia, the Advanced Li e Support or Obstet- Your atient and her in ant do well and are discharged ho e
rics (ALSO) course uses the “HELPERR” mnemonic: H—call on ost artu day 2. She returns or her ost artu exa i-
or help (wait . . . . aren’t you the help?), E—evaluate or episi- nation in 6 weeks. She notes incontinence o urine several
otomy, L—legs exed at the hip and knee (McRoberts maneu- ti es a day, es ecially with coughing, laughing, and sneez-
ver), P—suprapubic pressure, E—enter to rotate etus (Rubin ing. She has not ex erienced nocturia, dysuria, or he aturia.
II, Woods corkscrew maneuver, reverse corkscrew maneuver),
R—remove posterior arm, and R—roll the patient. T ere is no Question 15.11.7 These symptoms are indicative o which
evidence that any one maneuver is superior to another in releas- type o incontinence:
ing an impacted shoulder or decreasing the chance o injury. A) Stress incontinence
ypically, McRoberts maneuver is used rst. Last resort tech- B) Urge incontinence
niques include intentional racture o the etal clavicle, cephalic C) Over ow incontinence
replacement (aka, the Zavanelli maneuver—pushing the head D) Functional incontinence
back up into the pelvis and per orming a cesarean section), and E) Psychogenic incontinence
transcutaneous symphysiotomy. Why obstetricians continue to
use con using eponyms is beyond us. But i you want to belong Answer 15.11.7 The correct answer is “A.” Stress inconti-
to the club you need to know the secret passwords . . . . nence is the involuntary loss o urine during physical activity
such as coughing, laughing, jumping, running, and sneezing.
HELPFUL TIP: Urge incontinence is the involuntary loss o urine associated
Risk actors or shoulder dystocia include: previous shoul- with an abrupt and strong desire to void (detrusor overactiv-
der dystocia, gestational diabetes, post-dates pregnancy, ity). Over ow incontinence is the involuntary loss o urine due
maternal short stature, abnormal pelvic anatomy, sus- to under-activity o the detrusor muscle (e.g., neurogenic blad-
pected etal macrosomia, protracted active phase o irst der) or obstruction (e.g., BPH in men) but this (urinary outlet
stage o labor, protracted second stage o labor, and as- obstruction, not BPH) is becoming more common in women.
sisted vaginal delivery. Functional incontinence is loss o urine associated with physical
limitations (e.g., mobility restriction, arthritis, and dementia) in
434 FAMILY MEDICINE EXAMINATION & BOARD REVIEW

persons who have otherwise adequate bladder control. Psycho- intent o providing di erential support to the urethrovesical
genic incontinence, incontinence secondary to severe depres- junction or treatment o stress urinary incontinence. T is
sion or other psychological problem, is a rare disorder in this is an option or many women, especially those who want to
population and should be considered a diagnosis o exclusion. avoid surgery, but a pessary would not be the initial treatment.
It is more requent in patients with dementia. “D” is incorrect. Surgical correction is typically reserved until
6 to 12 months postpartum, as the symptoms may continue
Question 15.11.8 Which o the ollowing is most important to improve during that time. Note that surgical esh has a
prior to initiating incontinence therapy? very high rate o co lications when used or the treat ent
A) Evaluate or a vaginal stula using a dye instillation test o incontinence ( id-urethral sling) and, according to the
B) Obtain urinalysis ollowed by urine culture i abnormal FDA, has NO bene t over the traditional surgical treat ent
C) Re er or cystoscopy o stress incontinence. We would not subject our relatives to
D) Order urodynamic studies this . . . . .
Objectives: Did you learn to . . .
Answer 15.11.8 The correct answer is “B.” T e rst step in
• Evaluate and manage etal macrosomia?
the diagnosis o incontinence is a detailed history and physical
• Manage a delivery complicated by shoulder dystocia?
examination. A urinalysis and urine culture should be obtained
to exclude urinary tract in ection. Bacteriuria should be treated • Discuss the bene ts o breast eeding?
because the endotoxin produced by Escherichia coli may trig- • Classi y urinary incontinence and treat stress-type inconti-
ger abnormal detrusor activity or act as an α -adrenergic blocker nence?
(cool, huh?). Additional diagnostic tools may include a voiding
diary, stress test (have the patient cough or bear down and see CASE 15.12
i there is leakage rom the urethra), checking a postvoid resid-
A 31-year-old nulligravid single e ale resents or an
ual, cystometry, and cystourethroscopy. I there is concern or a
annual exa ination. She has no gynecologic concerns, and
vaginal stula elicited by the history and physical examination,
her last enstrual eriod was 3 weeks ago. She uses oral con-
dye instillation testing may be warranted (IV dye to evaluate or
trace tion and has regular cycles. Her exa ination is unre-
a ureteral stula, bladder instillation to evaluate or a bladder
arkable. T e Pa s ear returns with high-grade squa ous
stula).
intrae ithelial neo lasia (HSIL).

HELPFUL TIP: Question 15.12.1 You noti y her o the Pap smear ndings
Bariatric surgery (gastric banding, etc.) reduces the rate and explain this indicates:
o urinary incontinence in women who quali y: body A) She has cervical cancer
mass index (BMI) > 40 or BMI > 35 + comorbidities (DM, B) She needs urther diagnostic testing including colposcopy
sleep apnea, severe joint disease, weight-related cardio- and possible biopsy
myopathy). C) She needs treatment with cryotherapy or laser ablation
D) She should have the Pap smear repeated in 3 to 4 months
E) She should be enrolled in a hospice program
When you have co leted your evaluation, you deter ine
that she has stress urinary incontinence. Answer 15.12.1 The correct answer is “B.” Cervical cytology is
the most e ective cancer-screening program ever implemented.
Question 15.11.9 The best initial treatment option is: Both types o screening, liquid based and traditional Pap smears,
A) Pelvic muscle exercises have the same accuracy, and both types are endorsed or screen-
B) rial o oxybutynin hydrochloride ing. Abnormal Pap smear ndings require urther investiga-
C) Fitting o a pessary tion with colposcopy and directed biopsy, i indicated. Given
D) Surgical consult the nding o high-grade intraepithelial neoplasia on the Pap
E) Incontinence pads smear, repeating the Pap smear in 3 months is inappropriate. An
endocervical specimen (Pap smear or endocervical curettage)
Answer 15.11.9 The correct answer is “A.” Pelvic muscle exer- will need to be per ormed regardless o the appearance o the
cises (e.g., Kegel exercises) acilitate improved urinary control ectocervix to exclude endocervical pathology. reatment with a
in 40% to 75% o patients. T e correct method can be taught nonexcisional procedure (“C”) is not acceptable and premature
during a routine pelvic examination. Pelvic physical therapy without a biopsy to con rm the diagnosis rst.
can be used to provide eedback on the patient’s success with
these exercises. “B” is incorrect. Oxybutynin hydrochloride You er or a col osco y, and the cervix a ears grossly
is approved or detrusor instability (i.e., urge incontinence) nor al. An endocervical Pa s ear is obtained. T ere are
and does not appear to be e ective or stress incontinence. It acetowhite changes with areas o osaicis at the squa o-
also has signi cant anticholinergic side e ects. “C” is incor- colu nar junction ro the 4 o’clock to 11 o’clock osition.
rect. Several vaginal pessaries have been designed with the T e col osco y is adequate.
CHAPTER 15 • OBSTETRICS AND WOMEN’S HEALTH 435

Question 15.12.2 When you discuss the cervical ndings used instead or vaginal dys lasia, when the extent o disease
you explain that: precludes complete excision or destruction.
A) She needs a biopsy o the abnormal area
B) Given the previous Pap smear, you recommend a “see and Your atient wants to know the likelihood o regression to
treat” loop electrosurgical excision procedure (LEEP) “nor al” without treat ent.
C) T e ndings are consistent with dysplasia; no urther ther-
apy is warranted Question 15.12.4 What will you counsel?
D) She needs a repeat Pap smear in 3 to 4 months A) About 90% o CIN III lesions will become invasive cancer
E) Either A or B would be acceptable choices without aggressive treatment
B) About one-third o CIN III lesions spontaneously regress
Answer 15.12.2 The correct answer is “E.” When acetic acid is C) About two-thirds o CIN III lesions spontaneously regress
applied to the cervix, abnormal cells tend to turn white (“ace- D) About 90% o CIN III lesions will remain CIN III on ollow-
towhite”), and as the acetowhite ades, the degree o vascular- up a er 10 years
ity can be appreciated. Mosaicism, or mosaic changes, re er to
areas with increased vascularity and are indicative o abnormal Answer 15.12.4 The correct answer is “B,” although the data
changes, o en dysplasia. However, “visual” ndings at colpos- on prognosis varies due to variation in histologic diagnoses
copy are no more than suggestive, so “C” is incorrect. Biopsy o CIN. In some studies, about one-third o untreated CIN III
o the area is warranted to obtain a tissue diagnosis. T e see- lesions will spontaneously regress, while about 50% will persist
and-treat approach (“B”) is an option or our patient since she and the remaining will progress to invasive carcinoma. Other
is 31 years old; however the see-and-treat approach should not studies have shown progression to invasive carcinoma much
be done in young women between the ages o 21 and 24 per higher.
the current ASCCP guidelines. In this age range lesions o en
regress spontaneously. However, consider the “see and treat” She is concerned about uture childbearing i she undergoes
approach in any patient who is unlikely to ollow up. a LEEP.

T e endocervical Pa s ear is negative or dys lasia. You


Question 15.12.5 All o the ollowing are possible complica
also er or ed a bio sy o the acetowhite change area, which
tions o LEEP EXCEPT:
revealed high-grade cervical intrae ithelial neo lasia (CIN III).
A) Cervical incompetence
Question 15.12.3 Which o the ollowing do you recom B) Cervical stenosis
mend? C) Cervical ectopic pregnancy
A) Hysterectomy D) Decreased ertility
B) rachelectomy E) Premature rupture o membranes
C) Cytological ollow-up
D) LEEP or LASER ablation Answer 15.12.5 The correct answer is “C.” Cervical incompe-
E) 5- uorouracil intravaginally tence, stenosis, decreased ertility, and premature rupture o the
membranes have been identi ed ollowing all types o cone pro-
Answer 15.12.3 The correct answer is “D.” While some cedures, including LEEP, and are estimated to occur ollowing
patients with severe dysplasia (CIN III) may be candidates or less than 1% o procedures. Each o these complications seems
a hysterectomy, it would be an overly aggressive approach in to be related to the volume o tissue removed with the procedure
this 31-year-old emale who has never been pregnant and may rather than the procedure itsel . Cervical ectopic pregnancy is
wish to conceive in the uture. Additionally, women who had quite rare and has NO been associated with cone or LEEP pro-
high-grade CIN be ore hysterectomy can develop recurrent cedures.
vaginal dysplasia (VaIN) or cancer at the vaginal cu . Likewise,
trachelectomy (removal o the entire cervix) would be overly She undergoes LEEP treat ent as reco ended. T e atho-
aggressive with no added bene t over a LEEP procedure. Cyto- logy reveals CIN III with argins uninvolved.
logical ollow-up or “expectant management” is NO recom-
mended or high-grade dysplasia in this patient. T e likelihood Question 15.12.6 What do you counsel her about ollow
o regression is low in this patient’s age demographic, while up?
the likelihood o persistence or progression to cancer is unac- A) She should return or a Pap smear and pelvic examination in
ceptably high. Young women between 21 and 24 do have the 1 year
option proceeding with treatment (which is pre erred with CIN B) She should return or a Pap smear plus HPV co-testing in 12
III or inadequate colposcopy) OR observation with cytology and 24 months
and colposcopy at 6 month intervals or 12 months. A LEEP C) She should return or a Pap smear in 2 to 3 months
or ablation therapy would be the best recommendation in this D) She should return or a colposcopy and Pap smear in 2 to
patient. T e use o 5- uorouracil intravaginally is typically not 3 months
recommended or initial treatment o cervical dys lasia. It is E) She should just “chill out”; no ollow-up is indicated
436 FAMILY MEDICINE EXAMINATION & BOARD REVIEW

Answer 15.12.6 The correct answer is “B.” T e current rec- cells: cannot exclude high-grade SIL). In this case, colposcopy
ommendations or ollow-up include a Pap smear plus HPV would be indicated no matter the HPV status in this patient’s
co-testing at 12 and 24 months. I both are negative, HPV co- age demographic. I the patient was pregnant, or between the
testing should then be repeated 3 years later and i all remains ages o 21 and 24, management would be di erent. Making sure
negative she then may return to routine age-based screening or your malpractice insurance is current and up to date is always a
20 years. I any o the Pap smears reveal atypical squamous cells good idea but probably is unnecessary in this case.
(ASC) or higher grade dysplasia, or positive high-risk HPV, the
patient should undergo repeat colposcopy. HELPFUL (MAYBE) TIP:
The guidelines or Pap smears are ridiculously complex
At the ollow-u evaluation at 12 onths, her cervix a ears and include di erences based on patient age, preg-
nor al, without lesions or discharge. T e Pa s ear returns nancy status, etc. We strongly recommend that you re-
as nor al with negative high-risk HPV co-testing; however, view the ASCCP guidelines and algorithms available at
testing is li ited by absence o endocervical cells on the Pa http://www.asccp.org/consensus.shtml.
s ear.

Question 15.12.7 What is the best recommendation or Objectives: Did you learn to . . .
management o this patient now? • Manage an abnormal Pap test?
A) Repeat Pap smear and co-testing in another 12 months • Recognize the indications or colposcopy?
B) Cervical dilation and endocervical curettage • Evaluate and manage cervical dysplasia?
C) Cervical dilation and endocervical Pap smear • Recognize the risks and potential complications o LEEP?
D) Repeat LEEP • Manage the absence o endocervical cells on a Pap test?
E) Repeat the Pap smear as soon as possible

Answer 15.12.7 The answer is “A.” Since the HPV test was neg-
CASE 15.13
ative there is no need or urther testing at this time and screen- You are called to the e ergency de art ent to evaluate
ing should proceed as previously outlined above or her CIN III. a atient with a 2-day history o abdo inal ain. She is a
However, urther evaluation now WOULD be necessary i the 24-year-old G1 P1 e ale whose LMP was 1 week ago. On the
HPV was positive or unknown. “1–10” scale, her ain is a “12.” She is on oral contrace tives
or birth control. She has “never issed a ill” and “could
All your artners are on vacation and you are covering all their not ossibly be regnant.” Her ain is across her lower abdo-
atients’ tests. Suddenly you receive a S A age ro the en and a little ore on the right side than the le . She has
athology lab (a athological e ergency?). T ey wanted to elt everish. She has had so e nausea but no vo iting. She
noti y you that they ade a istake when inter reting one o denies bowel or bladder roble s. Her ain i roves with
the Pa s ear results on a 33-year-old atient and, in act, the aceta ino hen and worsens with activity.
correct re ort should have been ASC-US (aty ical squa ous On exa ination, she a ears unco ortable but not toxic.
cells o undeter ined signi cance) instead o ASC-H (aty i- Her te erature is 38°C, but the rest o her vitals are nor-
cal squa ous cells, high grade). T e lab technician is new here al. Her abdo inal exa ination reveals decreased bowel
and is not quite sure i they can add on an “HPV test” to the sounds, with tenderness to al ation ri arily across the
Pa sa le. lower quadrants. She has ini al guarding and no rebound
tenderness. Her elvic exa ination is re arkable or cervi-
Question 15.12.8 As you prepare yoursel to call the patient, cal otion tenderness. T e uterus is o nor al size and con-
what do you tell her is the next step? sistency with no asses.
A) Repeat Pap smear in 1 year
B) Ask the lab personnel to investigate i the HPV test can be Question 15.13.1 Which o the ollowing diagnoses can be
done on that sample absolutely excluded rom your di erential at this point?
C) Contact your malpractice attorney A) Ectopic pregnancy
D) Both A and B B) Appendicitis
C) Pelvic in ammatory disease (PID)
Answer 15.12.8 The correct answer is “D,” according to cur D) Pyelonephritis
rent ASCCP guidelines. I you are able to do HPV co-testing and E) None o the above diagnoses should be excluded based on
it returns negative she could have repeat co-testing in 3 years. I the in ormation available
you choose to repeat cytology only in 1 year and it returns nor-
mal, she may return to age-appropriate screening. I the cytol- Answer 15.13.1 The correct answer is “E.” T e di erential
ogy returns with any other abnormalities greater than or equal or lower abdominal pain in a young emale includes all o the
to ASC then a colposcopy is indicated. Colposcopy would also above and more. Even though your patient seems unlikely to be
be indicated i the HPV test returns positive. Make sure that the pregnant due to her consistent use o contraceptives and recent
Pap report does not actually read ASC-H (atypical squamous menses, you should not exclude pregnancy without a negative
CHAPTER 15 • OBSTETRICS AND WOMEN’S HEALTH 437

TABLE 15-5 CRITERIA FOR ADMISSION FOR THE


• Presence o an abundant number o white cells on sa-
TREATMENT OF PID
line microscopy o vaginal uid
• Uncertain diagnosis • Adnexal mass
• Surgical emergencies (e.g., appendicitis) cannot be excluded • Laboratory evidence o gonorrhea or chlamydia in-
• Suspected tubo-ovarian abscess
ection
• Concurrent pregnancy (due to high risk o maternal mortality, etal
wastage, and preterm delivery) • Elevated C-reactive protein and/or ESR
• Severe illness, intractable nausea and vomiting, or high ever
• Patient cannot tolerate or ollow an outpatient regimen (e.g., severe
vomiting)
• Lack o clinical response to oral outpatient antimicrobial therapy
Question 15.13.3 For empiric antibiotic therapy or PID in
this patient, you prescribe:
Adapted rom Gradison M. Pelvic in ammatory disease. Am Fam Physician. A) Amoxicillin 500 mg PO ID or 14 days
2012;85(8):791–796. B) Ce riaxone 250 mg IM once PLUS azithromycin 1 g
C) Ce riaxone 250 mg IM once PLUS doxycycline 100 mg PO
urine hCG. Emergency department studies consistently show BID or 14 days
that women who claim to never have been sexually active can D) A and B
be pregnant. Either something metaphysical is going on here or E) B and C
history cannot always be trusted.
Answer 15.13.3 The correct answer is “C.” Recommendations
You obtain cultures/PCR or chla ydia and gonorrhea. T e or treatment o PID include ce riaxone (Rocephin) 250 mg IM
urine regnancy test is negative (“I told you not to waste once PLUS doxycycline 100 mg PO BID or 14 (yes, 14) days.
health care dollars—es ecially in this econo y,” your atient Metronidazole may be added to this regimen to cover anaerobic
co lains). T e urinalysis is negative or nitrites and leuko- bacteria. Note that single-dose azithro ycin is not indicated
cytes, and the WBC is 15,600/ 3
with an increase in bands. or the treat ent o PID, only or cervicitis. T us, 14 days o
She re orts that she’s had an a endecto y. doxycycline are indicated.
T e current (2015) CDC treatment guidelines or rectal,
Question 15.13.2 What is the most appropriate next step? oral, cervical, or penile gonorrhea include ce riaxone 250 mg
A) Consult surgery and gynecology to con rm your ndings IM once PLUS azithromycin 1 g or 7 days o doxycycline. T is
B) Admit or IV antibiotics and IV hydration is true even i the patient has ONLY gonorrhea. T is is to pre-
C) reat as an outpatient with antibiotics and schedule ollow- vent the development o resistant gonorrhea and also due to the
up or 36 to 48 hours high co-in ection rate o gonorrhea with chlamydia. I ce ri-
D) reat with IV antibiotics on an outpatient basis utilizing vis- axone is not available, ce xime 400 mg orally in a single dose
iting nurse care PLUS azithromycin 1 g orally in a single dose is an appropriate
E) Obtain cultures, discharge the patient, and treat based on alternative. O note, patients who have been treated or gonor-
culture results rhea should be re-tested 3 months a er treatment regardless o
whether they believe their sex partners were treated as there is a
Answer 15.13.2 The correct answer is “C.” T e patient’s his- high incidence o rein ection (CDC, 2015).
tory, examination, and diagnostic tests are most consistent with
pelvic in ammatory disease (PID). PID is a clinical syndrome Just when you were about to discharge her the atient starts
caused by the ascent o microorganisms rom the lower geni- vo iting and is unable to kee down her oral edications
tal tract (e.g., vagina) to the upper genital tract (e.g., endome- (doxycycline, aceta ino hen). You ad it her and she is dis-
trium). Most cases o PID can be managed in the outpatient charged when she is tolerating oral intake and has been a e-
setting. Indications or hospitalization are listed in able 15-5. brile or 48 hours. You instruct her to nish a 14-day course
o doxycycline. She resents or ollow-u a week later, at
which ti e her sy to s have co letely resolved.
HELPFUL TIP:
The diagnosis o PID is a clinical one and not labora- Question 15.13.4 Which o the ollowing is a potential
tory based! Untreated PID has signi icant morbidity and consequence o PID?
mortality; empiric treatment is recommended i the pa- A) In ertility
tient meets the ollowing minimal diagnostic criteria: B) Chronic pelvic pain
• Uterine/adnexal tenderness or C) Increased risk or ectopic pregnancy
• Cervical motion tenderness and D) Recurrent PID
• No other cause or illness identi ed E) All o the above
Other help ul (but not necessary) criteria include:
• Temperature ≥ 38.3°C Answer 15.13.4 The correct answer is “E.” All the choices
• Abnormal cervical or vaginal mucopurulent discharge above are potential sequelae o PID. Additionally, a tubo-ovar-
ian abscess may develop.
438 FAMILY MEDICINE EXAMINATION & BOARD REVIEW

Question 15.13.7 What is the minimum expected increase


HELPFUL TIP:
in quantitative hCG in early gestation in a normal preg
The etiology o PID is polymicrobial, although sexu-
nancy?
ally transmitted in ections, predominantly gonorrhea
A) 20% increase in 24 hours
and/or chlamydia, are implicated in up to two-thirds o
B) 66% increase in 48 hours
cases. Antibiotic regimens are chosen or broad cover-
C) 100% increase in 24 hours
age. The CDC no longer recommends the use o luo-
D) 10% increase in 48 hours
roquinolones or treatment o PID or gonorrhea due to
E) 75% increase in 72 hours
increasing resistance. O note, there is ce triaxone-
resistant gonorrhea in Japan with intermediate-resistant Answer 15.13.7 The correct answer is “B.” hCG typically
gonorrhea in the United States. doubles (increases by 100%) every 48 hours in normal gesta-
tion. T e minimum increase considered to be compatible with a
viable pregnancy is 66% in 48 hours.
You see her next or her annual exa ination. She quit tak-
ing her oral contrace tive 3 onths ago a er se arating ro Question 15.13.8 I the quantitative serum β hCG is
her husband (the jerk gave her chla ydia, a er all!). Subse- 5,500 ng/mL and the pelvic ultrasound reveals no intrauter
quently, her eriods have been irregular. She is not obese, has ine pregnancy but a probable right tubal pregnancy, what
ini al acne, and no hirsutis or galactorrhea. Her hysi- would be the most appropriate management option?
cal exa ination is essentially nor al. Urine hCG is negative A) Medical management with methotrexate
(you did i ediately think o an hCG, didn’t you?) and a B) Laparoscopic surgery with evacuation o the products o
seru SH is nor al. conception
C) Consultation with someone able to do a salpingectomy i
Question 15.13.5 What is the most likely etiology o the necessary
irregular cycles? D) Dilation and curettage
A) Anovulation E) Hysterectomy
B) Pituitary tumor
C) Polycystic ovarian syndrome Answer 15.13.8 The correct answer is “C.” An ectopic preg-
D) Premature ovarian ailure nancy may rupture and become a li e-threatening event at any
E) Androgen secreting tumor point be ore complete resolution. I the patient becomes hemo-
dynamically unstable, she will need emergent surgery. T us, an
Answer 15.13.5 The correct answer is “A.” OCPs work by sup- early re erral or management is warranted. Medical treatment
pressing ovulation. Resumption o ovulation a er pill cessation with methotrexate may be indicated but should be done under
can take several months (up to 6 months). In the absence o close supervision with surgical consultation available.
abnormal history, physical examination, or laboratory ndings,
the other choices are unlikely to be the etiology o the irregular Question 15.13.9 I the quantitative serum β hCG had
cycles in this patient. come back at 1,000 ng/mL and no ultrasound were avail
able, how would you have counseled the patient?
Question 15.13.6 Each o the ollowing is appropriate initial A) Given the history o PID, this is most likely an ectopic preg-
management o this patient’s irregular menses, EXCEPT: nancy. She should abort the pregnancy at once
A) Expectant management B) Given the history o PID, she should remain on bed rest
B) Reestablishment o cycle regulation with OCPs until a de nitive diagnosis is made
C) Progestin induced withdrawal cycles C) She should be educated about ectopic pregnancy and mis-
D) Colposcopy with cervical and endometrial biopsies carriage
D) She should have an urgent surgical consultation today or
Answer 15.13.6 The correct answer is “D.” Anovulation is exploratory laparotomy
expected ollowing OCP cessation; thus, expectant manage- E) She should read the patient education material entitled “So,
ment is a reasonable option, as most people will resume regular You Might Be Having an Ectopic Pregnancy”
cycling within 6 months. She could also opt to resume OCPs
or cycle regulation i that is her goal. Withdrawal bleeds could Answer 15.13.9 The correct answer is “C.” T is β -hCG level
be induced by cyclical progestin challenges. Colposcopy is not is consistent with an early gestation given her history o irregu-
indicated in this patient as she has no cytologic abnormalities lar cycles. However, with her history o PID, she is at risk or
noted that would require ollow-up with colposcopy and biopsy. ectopic pregnancy. T us, she should receive ectopic pregnancy
and miscarriage precautions. While a history o PID increases
T e atient is reluctant to resu e “the ill.” You next evalu- the risk o ectopic pregnancy by 7- to 10- old, only 8% o such
ate her 8 onths later. Her last eriod was 7 weeks be ore. patients will ever have an ectopic. So, it is still highly probable
A urine regnancy test is ositive. Given her history o PID, that the patient has a normal intrauterine pregnancy. I you
you request a quantitative seru β-hCG and are considering chose “E,” you may also want to hand her the pamphlet entitled
a elvic ultrasound to con r intrauterine regnancy. “So, Your Doctor Has a Horrible Bedside Manner.”
CHAPTER 15 • OBSTETRICS AND WOMEN’S HEALTH 439

C) 6 months amenorrhea, elevated FSH, and decreased estra-


In reality, her quantitative seru β-hCG is 6,000 ng/ L. A diol con rm the diagnosis
elvic ultrasound con r s a viable intrauterine regnancy D) 12 months amenorrhea, elevated FSH, and decreased estra-
at 8 4/7 weeks o gestation. She wins the ecto ic lottery. diol are the de nitive test
E) 2 years o mood swings and hot ashes will clinch the diag-
HELPFUL TIP: nosis
Assisted reproductive technology substantially increases
the risk o a heterotopic pregnancy (two pregnancies Answer 15.14.2 The correct answer is “B.” Menopause is a
with only one inside the uterus) to about 1%, and in clinical syndrome characterized by the cessation o spontaneous
these patients, the rate o ectopic pregnancy is 4% to menstrual periods along with associated symptoms o estrogen
8%, which is about our old higher than the general de ciency such as hot ashes, vaginal atrophy, and psychologi-
population. Having one ectopic pregnancy predisposes cal symptoms. As such, there is no de nitive test.
to having a subsequent one.
She is concerned about eno ause and wonders i she needs
Objectives: Did you learn to . . . hor one re lace ent thera y.
• Evaluate a patient with pelvic pain?
• Diagnose and treat PID? Question 15.14.3 All the ollowing are bene ts o estrogen
• Evaluate and treat irregular menses?
containing hormone replacement therapy (HRT) EXCEPT:
A) Osteoporosis prevention
• Diagnose and manage ectopic pregnancies?
B) Decrease in colon cancer risk
C) Decrease in hot ashes and vasomotor symptoms
CASE 15.14 D) Decrease in stroke risk
A 52-year-old e ale atient o yours resents or an annual
exa ination and Pa s ear. Her last enstrual eriod was Answer 15.14.3 The correct answer is “D.” We have learned a
3 onths ago. She has inter ittent hot ashes and night lot about HR in postmenopausal women with data rom the
sweats. Her exa ination is re arkable or ild vaginal atro- Women’s Health Initiative. Here’s a quick summary. Proven
hy. She wonders about “estrogen testing” to see i she needs bene ts o HR are limited to the ollowing:
hor one re lace ent thera y. • Reduced risk o osteoporosis and related ractures.
• Decreased colon cancer risk (not seen in estrogen only arm).
Question 15.14.1 How will you counsel her about the role Data now suggest that even though the number o colon can-
o estrogen testing? cers is reduced, they are diagnosed at a more advanced stage.
A) Recommend against estrogen testing HR is not recommended or chemoprevention o colon
B) Recommend buccal swab testing as it is the most accurate cancer in women.
C) Recommend asting morning serum estradiol testing • Improvement o vasomotor symptoms such as hot ashes.
D) Recommend estrogen testing with FSH to ensure meno-
pause status HR may increase the risk o the ollowing (variable ndings):
E) Recommend annual estrogen testing a er the age o 55 years • Breast cancer (estrogen only arm showed a paradoxical re-
duction in risk o breast cancer)
Answer 15.14.1 The correct answer is “A.” In a woman o the
• Myocardial in arction (estrogen only arm showed no in-
right age with symptoms consistent with menopause, no testing
is recommended. esting is only recommended i the diagnosis is crease risk)
unclear (e.g., a patient younger than 35). It can also be use ul when • Venous thromboembolic events
a patient on OCPs presents with symptoms suggestive o meno- • Stroke
pause. In this case, an FSH could be done a er the patient has dis-
continued hormonal contraception or 1 week (or on the last day Only women with vasomotor symptoms appear to have
o her placebo pills). I the FSH is high (> 25 mIU/L), this indicates improved overall quality-o -li e scores with HR . Given the sig-
that the patient has likely entered the menopausal transition. ni cant excess risks with HR usage, many practitioners now
recommend short-term HR use only or vasomotor symptoms
She jokes that you’re robably getting kickbacks ro the and not or osteoporosis or other purported bene ts.
insurance co any or ordering ewer tests, and then she
asks, “How will you know i I’ going through eno ause Question 15.14.4 An absolute contraindication to use o
without an estrogen level?” HRT is:
A) Heart disease
Question 15.14.2 How will you counsel her about meno B) Breast cancer
pause and its diagnosis? C) Endometrial cancer
A) FSH is the de nitive test D) Previous thromboembolic event
B) T ere is no de nitive test o menopause E) All o the above
440 FAMILY MEDICINE EXAMINATION & BOARD REVIEW

Answer 15.14.4 The correct answer is “D.” Previous thrombo- o women responding. Other options include progestins such as
embolic disease is the only absolute contraindication to HR . medroxyprogesterone (Provera) or megestrol (Megace), SSRIs,
T e others are relative contraindications. While HR is not clonazepam, gabapentin (Neurontin), venla axine (E exor),
routinely recommended or women who have heart disease or clonidine, exercise, and environmental modi cations (e.g.,
a history o breast or endometrial cancer, it may be use ul in thermostat settings, ans). Note that we do not list estrogen alone.
women who have signi cant impairment in their quality o li e Unopposed estrogen can increase the risk o endometrial cancer
rom vasomotor symptoms re ractory to other management and should be avoided unless the woman has had a hysterectomy.
methods. T ere is con icting evidence or the ef cacy o vitamin E and
black cohosh; there is no good data to suggest that propranolol
You discuss eno ause and the otential risks and bene ts o or phytoestrogens, such as soy, are e ective in treating vasomo-
HR with your atient. She decides against HR or now, but tor symptoms.
returns in 6 onths with continued a enorrhea, hot ashes,
Objectives: Did you learn to . . .
and vaginal dryness—which see s to bother her the ost.
• Evaluate menopausal symptoms?
• Describe hormone replacement therapy, including risks, ben-
Question 15.14.5 Options to treat the vaginal dryness e ts, and contraindications or its use?
include which o the ollowing? • Diagnose and manage atrophic vaginitis?
A) Systemic HR • Treat menopausal symptoms?
B) Vaginal estrogen
C) Lubrication
D) All o the above CASE 15.15
A 57-year-old ost eno ausal atient resents or her
Answer 15.14.5 The correct answer is “D.” Systemic and local annual exa ination. She is ex eriencing hot ashes and
estrogen administration are both e ective or treating vaginal night sweats, as well as a recurrence o vaginal bleeding. Her
dryness. Lubrication with vegetable oil and speci cally manu- edical history is otherwise unre arkable. She wants your
actured lubricants can be e ective. o inion about resu ing hor one re lace ent. Her el-
vic exa ination is re arkable or atro hic vaginal ucosal
Question 15.14.6 How is vaginal atrophy most rapidly and changes, a stenotic cervix without lesions, nor al size uterus,
accurately diagnosed? no adnexal asses, and no asses al able on rectovaginal
A) Biopsy o vaginal mucosa exa ination.
B) Culture o vaginal swab
C) Patient’s history and physical examination Question 15.15.1 All o the ollowing are possible causes o
D) Serum hormone levels her vaginal bleeding EXCEPT:
E) KOH prep o vaginal swab A) Cervical cancer
B) Uterine polyp
Answer 15.14.6 The correct answer is “C.” T e diagnosis o C) Polycystic ovary disease
urogenital atrophy is clinical, based on history and physical D) Atrophic vaginitis
examination ndings. ests such as microscopic examination o E) Endometrial cancer
the vaginal smear, will reveal an abundance o basal and para-
basal cells and a paucity o mature squamous epithelium. A Answer 15.15.1 The correct answer is “C.” Polycystic ovar-
biopsy is a bit extreme. “B” and “E” are incorrect since atrophy ian disease does not cause vaginal bleeding in postmenopausal
is not an in ectious issue. women. All the other choices are diagnostic considerations in a
postmenopausal emale with vaginal bleeding or spotting.
Des ite your te id endorse ent o HR , she wants to try it to
reduce hot ashes. You also counsel her on other o tions to Question 15.15.2 Which o the ollowing studies should
treat hot ashes. you consider obtaining?
A) Urinalysis
Question 15.14.7 Further options to treat symptomatic hot B) Pap smear
f ashes include which o the ollowing? C) Endometrial biopsyv
A) Progesterone alone D) Stool guaiac
B) Clonidine E) All o the above
C) Exercise
D) rial o an SSRI Answer 15.15.2 The correct answer is “E.” T e patient could
E) All o the above easily mistake the source o the bleeding; thus, it is prudent to
rule out a rectal or urinary source. Cervical and endometrial
Answer 15.14.7 The correct answer is “E.” HR is the most evaluations are necessary to rule out gynecological pathology,
ef cacious treatment or vasomotor symptoms with almost 90% such as endometrial polyp, hyperplasia, or cancer.
CHAPTER 15 • OBSTETRICS AND WOMEN’S HEALTH 441

TABLE 15-6 RISK FACTORS FOR Question 15.15.4 O the ollowing, the most appropriate
ENDOMETRIAL CANCER intervention is:
A) Repeating the endometrial biopsy
Advancing age
B) Per orming a transvaginal ultrasound to assess endometrial
Obesitya
Nulliparity thickness
Early menarche C) Starting high-dose progestin therapy
Late menopause D) Starting high-dose selective estrogen receptor modulator
Chronic anovulation (e.g., PCOS) therapy (e.g., tamoxi en)
Unopposed exogenous estrogen use
E) Arranging or de nitive management (e.g., hysterectomy)
Tamoxi en
Hypertension
Diabetes Answer 15.15.4 The correct answer is “E.” Her biopsy ndings
Estrogen producing tumor are highly abnormal and may indicate already existing carci-
Lynch Syndrome noma that was not contained in the sample examined. Re erral
a
to a gynecologist or de nitive management is warranted at this
Obesity leads to increased estrogen levels rom peripheral conversion o
androstenedione. The presence o DM and HTN as risk actors may simply
time.
re ect the high incidence o obesity in patients with these disorders.

HELPFUL TIP:
Question 15.15.3 All o the ollowing increase the risk o In postmenopausal women, an endometrial stripe o
endometrial cancer EXCEPT: greater than 5 mm on ultrasound is suggestive o en-
A) Smoking dometrial cancer. With an endometrial stripe o < 4 mm,
B) Obesity malignancy is rare. In evaluation o postmenopausal
C) Unopposed estrogen women with vaginal bleeding, transvaginal ultrasound
D) Diabetes is an alternative to endometrial biopsy when an endo-
E) Hypertension metrial biopsy cannot be done. Further bene it o ob-
taining a transvaginal ultrasound, regardless o endo-
Answer 15.15.3 The correct answer is “A.” Endometrial can- metrial biopsy, is that it can detect structural lesions or
cer is believed to be caused by unopposed estrogen stimu- masses.
lation o the endometrium. Smoking decreases luteal phase
estrogen and is epidemiologically linked to a decrease in the
risk or endometrial carcinoma (o course, this risk is o set Objectives: Did you learn to . . .
by the other adverse health e ects o smoking—go gure!). • Evaluate postmenopausal bleeding?
Risk actors and protective actors are listed in the ables 15-6 • Manage a patient with postmenopausal bleeding?
and 15-7.
• Assess patient or risk o endometrial cancer?

You review your atient’s test results: nor al Pa s ear and


urinalysis, stool guaiac negative or blood, and endo etrial CASE 15.16
bio sy with rag ents o benign oly . A 15-year-old nulligravid e ale resents with her other
Over the next 3 years, she continues to have rare occur- or evaluation o ain ul eriods. Menarche was at age 14.
rences o vaginal s otting. She has declined urther evalu- Her eriods are ty ically every 4 to 8 weeks and are asso-
ation given the in requency o the e isodes. However, over ciated with severe cra ing. She has issed 1 to 2 days o
the last several onths, she has ex erienced an increase in school with each enses because o ain. She denies inter-
the a ount and requency o the bleeding. Other than an course. She has never had a elvic exa ination. Her review
interval weight gain o 27 lb, there has been no change in her o syste s is otherwise negative.
exa ination. Re eat endo etrial bio sy reveals co lex
hy er lasia with aty ia, and the athologist cannot rule out Question 15.16.1 What is the MOST LIKELY etiology o her
endo etrial cancer. irregular cycles?
A) Pregnancy
B) Endometriosis
TABLE 15-7 PROTECTIVE FACTORS FOR
C) Anovulation
ENDOMETRIAL CANCERa
D) Hyperthyroidism
Progesterone E) Imper orate hymen
Oral contraceptives
Cigarette smoking
Multiparity
Answer 15.16.1 The correct answer is “C.” Abnormal uterine
Child bearing at an older age bleeding is common among adolescent girls who have reached
menarche. T e rst ew years o menstruation are o en charac-
a
All reduce exposure to unopposed estrogens. terized by irregular cycles as a result o anovulation. Pregnancy
442 FAMILY MEDICINE EXAMINATION & BOARD REVIEW

and imper orate hymen lead to absence o menses, not irregular suggests that me enamic acid (Ponstel) may be more e ective
menses. While hyperthyroidism may lead to irregular cycles, it or dysmenorrhea than other NSAIDs. Acetaminophen is not as
does not typically cause dysmenorrhea and is usually associated e ective as NSAIDs.
with other systemic complaints. Additionally hyperthyroidism
would be unusual, but not out o the question, in a patient o this
age. Endometriosis may cause dysmenorrhea, but is unlikely to HELPFUL TIP:
occur in a patient this young; most cases o endometriosis pres- The average age o menarche is 12.8 years in the United
ent in patients aged 20s to 30s. States, with the range rom 10 to 15 years.

Question 15.16.2 What is the etiology o her dysmenor


rhea?
A) Prostaglandin release Your atient and her other o t to try hor onal regulation
B) Streptococcal endotoxin release with birth control ills. She returns or ollow-u in 4 onths
C) Estrogen release and is doing well. She ad its to being sexually active.
D) Excessive testosterone production
Question 15.16.4 In addition to reviewing the use o birth
control pills, she should be questioned or counseled about
Answer 15.16.2 The correct answer is “A.” Dysmenorrhea is which o the ollowing?
the term that describes excessive pain in association with men- A) Knowledge o sexually transmitted diseases and use o con-
struation. It is the most common gynecologic complaint, a ects doms
about hal o all adolescent emales, and is the leading cause o B) Age o her boy riend
periodic school absenteeism. T e pathogenesis o dysmenor- C) Consensual nature o her relationship
rhea involves excess prostaglandin release, which causes pro- D) HPV vaccination
longed, pain ul uterine contractions. It can be divided in two E) All o the above
main subtypes: primary and secondary. Primary dysmenorrhea
usually starts be ore the age o 20 and has a tendency to occur Answer 15.16.4 The correct answer is “E.” Visits or contra-
with menarche. It is caused by prostaglandin stimulation o the ception are great opportunities or you to discuss sa e sexual
myometrium. Secondary dysmenorrhea typically arises a er practices with the patient. Such an interview should include
the age o 20 and is associated with pelvic pathology or other evaluation or sexual assault, coercion, or abuse. Although she
organic disease. is a little late or starting HPV vaccination, “D” is still correct
(better late than never—truly!). T e Advisory Committee on
You er or a hysical exa ination, revealing nor al vital Immunization Practices (ACIP) recommends the routine use o
signs, nor al weight, a benign abdo en, anner stage V, and quadrivalant (Gardasil), 9-valent, or bivalent HPV vaccine in
no signs o androgen excess. emales rom age 9 to 26 years. T e quadrivalent HPV vaccine
protects against HPV types 6, 11, 16, and 18. HPV types 6 and
Question 15.16.3 Which o the ollowing is the best next 11 are commonly associated with genital warts, while 16 and 18
step in caring or this patient? are commonly responsible or cervical cancer. T e 9-valent vac-
A) O er reassurance and observation cine protects against HPV types 6, 11, 16, 18, 31, 33, 45, 52, and
B) Initiate combined hormonal contraception 58. Both the quadrivalent and the 9-valent vaccines are given
C) Initiate a GnRH agonist in 3 doses (0, 2, and 6 months rom rst dose). T ere is also
D) Prescribe a narcotic analgesic a bivalent vaccine (brand name Cervarix) that protects against
E) Re er or diagnostic laparoscopy HPV types 16 and 18 only and is approved or use in emales
aged 11 to 25 years. It is given as a three-injection series at 0, 1,
Answer 15.16.3 The correct answer is “B.” OCPs o er cycle and 6 months. In general, the 9-valent vaccine should be used
regulation and a reduction in dysmenorrhea. “A,” expect- i it is an option.
ant management, is inappropriate given the severity o symp-
toms and availability o sa e and e ective treatment. A urther
workup is not needed at this stage, as her history is straight- HELPFUL TIP:
orward and her physical examination is reassuring. She cer- HPV is the most common viral sexually transmitted in-
tainly does not need surgery now (“E”). “C” is incorrect. GnRH ection in the United States with point prevalence in
agonists will induce amenorrhea, hot ashes, accelerated bone emales ranging rom 26% to 64%. Many patients have
loss, are expensive, and require add-back estrogen when utilized serial in ections with di erent HPVtypes. A two-shot se-
longer than 6 months. Narcotic analgesics (“D”) do not help ries may be as e ective as a three-shot series but has
reduce prostaglandin levels and are not appropriate or pain not yet been endorsed as o icial policy. I the vaccina-
control in this case. NOTE: Although not listed in the answers, tion series is interrupted or any length o time, it can be
NSAIDs are also quite e ective at treating dysmenorrhea and resumed without restarting the series.
should be considered as a rst-line drug. Anecdotal evidence
CHAPTER 15 • OBSTETRICS AND WOMEN’S HEALTH 443

HELPFUL TIP: TABLE 15-8 TREATMENT OPTIONS FOR


The best time to check a pregnancy test is a ter the PREMENSTRUAL SYNDROME
irst missed menses. Otherwise, you risk having a alse- Nonpharmacologic
negative test. Even on day 1 o a missed menses, the Aerobic exercise
sensitivity is only 90% (thus, patients can present with Increased intake o complex carbohydrates and ber
Reduction in sodium, caf eine, and alcohol intake
an ectopic pregnancy with a negative urine pregnancy
Supportive psychotherapy
test).
Pharmacologic
Anxiolytics such as buspirone and benzodiazepines
Calcium and vitamin D supplementation
You lose touch with the atient, and she discontinues her OCP. Danazol
Years later when she returns or a hysical exa ination— Hormonal treatment (combination OCPs, GnRH agonists,
and you’re still aying o your student loans—she co lains progesterones)
o increasing irritability along with inter ittent bloating NSAIDs
Serotonergic antidepressants such as SSRIs and venla axine
and swelling during the week be ore her eriod each onth.
Spironolactone (NOT thiazide diuretics)
Although she is annoyed by these sy to s, they are not Vitamin B6 (pyridoxine)
so severe as to inter ere with her usual activities. Her en- Chasteberry
ses now occur onthly without inter enstrual s otting or
issed eriods. Adapted rom Biggs W, Demuth RH. Premenstrual syndrome and
premenstrual dysphoric disorder. Am Fam Physician. 2011;84(8):918–924.
Question 15.16.5 What is her most likely diagnosis?
Answer 15.16.6 The correct answer is “B.” PMS is a clinical
A) Major depression
entity and no laboratory data exist to aid in diagnosis. All other
B) Premenstrual dysphoric disorder (PMDD)
options described above are correct. An elevated LH:FSH ratio
C) Premenstrual syndrome (PMS)
o 3:1 in the ace o appropriate symptoms is suggestive o PCOS.
D) Polycystic ovary syndrome (PCOS)
E) Hypothyroidism Question 15.16.7 Each o the ollowing is a possible treat
ment option or PMS and PMDD EXCEPT:
Answer 15.16.5 The correct answer is “C.” PMS is a constel- A) Supportive therapy/counseling
lation o physical, emotional, and behavioral symptoms. It is B) Aerobic exercise
cyclical in nature, occurs during the second hal o the men- C) Selective serotonin reuptake inhibitors
strual cycle (luteal phase, 7–10 days be ore menses), and D) T iazide diuretics
resolves soon a er menses. A symptom- ree interval occurs E) Calcium
during the rst hal o the cycle ( ollicular phase). PMDD is
more severe but occurs during the same time rame as PMS. Answer 15.16.7 The correct answer is “D.” T iazide diuretics
Symptoms o PMDD include labile mood, depressed mood, are not help ul in PMS or PMDD, but all o the other options
irritability, eelings o hopelessness, hypersomnia or insomnia, are potentially use ul. reatment options that have been shown
and decreased interest in usual activities. PMDD is diagnosed to help with PMS are listed in able 15-8.
by DSM-V criteria and some unctional impairment must be
present. Objectives: Did you learn to . . .
• Evaluate concerns about menarche and describe normal
early menstrual patterns?
HELPFUL TIP: • Evaluate and manage dysmenorrhea?
Premenstrual symptoms exist on a continuum with • Diagnose and treat PMS?
up to 90% o women a ected by minimal PMS symp-
toms while 10% are severely a ected. This group with CASE 15.17
more severe symptoms can be categorized as having
PMDD. A rantic 25-year-old atient calls you. She and her boy riend
were having intercourse and the condo broke at the ti e o
ejaculation about 16 hours ago. She does not use any other
Question 15.16.6 Each o the ollowing is a key element o or o contrace tion. Her last enstrual eriod was about
the diagnosis o PMS EXCEPT: 2 weeks ago. You tell her that 8%o wo en beco e regnant
A) Physical symptoms o bloating, swelling, and/or atigue a er a single act o coitus. She is orti ed, exclai ing, “I’ve
B) Elevated luteinizing hormone to ollicle stimulation hor- always been in the to 8%o everything!”
mone (LH:FSH) ratio
C) Restriction o symptoms to the luteal phase o the menstrual Question 15.17.1 Appropriate methods o “emergency con
cycle traception” or this patient include:
D) Exclusion o other diagnoses that may better explain the A) Levonorgestrel (Plan B)
symptoms B) Ethinyl estradiol plus levonorgestrel (Yuzpe regimen)
444 FAMILY MEDICINE EXAMINATION & BOARD REVIEW

C) High-dose ethinyl estradiol (Ivanapyuk method) quadrants. T ere is no evidence o guarding, rebound ten-
D) A and B only derness, or al able asses. She has no back tenderness. T e
E) A, B, and C external genital and vaginal exa inations show no lesions or
erythe a. T ere is a crea y discharge noted at the cervix.
Answer 15.17.1 The correct answer is “D.” Many OCPs are Bi anual reveals a retroverted uterus with uterosacral nod-
also e ective i used at the right doses and within 72 hours (the ularity al able. Both adnexal areas are tender to exa ina-
Yuzpe method). However, as o 2015, only Plan B and ulipris- tion, but without asses.
tal (an antiprogestin, brand name Ella) are FDA-approved or You get a regnancy test, cultures, and urinalysis, all o
postcoital contraception. Plan B uses a high dose o levonorg- which are negative.
estrel and is more e ective than the combined OCPs or post-
coital contraception. Plan B is available over the counter (in the Question 15.17.2 Based on her symptoms and your physi
United States in 2015). Ulipristal appears to be more e ective cal examination, what is the most likely etiology o the
than Plan B and may be taken up to 120 hours a er unprotected patient’s chronic pelvic pain?
intercourse but is only available by prescription. Copper IUD is A) Irritable bowel syndrome
another option and may be used up to 8 days a er unprotected B) Myo ascial pain disorder
intercourse. “C” is incorrect as estrogen alone is not known to C) Endometriosis
be e ective as emergency contraception, and “Ivanapyuk” is a D) Cervical dysplasia
made-up name (but a good description o what happens to a E) Pain ul bladder syndrome (the disease ormerly known as
patient who takes a massive dose o estrogen). interstitial cystitis)

HELPFUL TIP: Answer 15.17.2 The correct answer is “C.” T ere were no
Prescribe an antiemetic with postcoital OCPs as nausea bowel symptoms elicited on the history to suggest irritable
and vomiting are common side e ects. Plan B has ewer bowel syndrome. T ere were no signs elicited on the examina-
GI side e ects. About the only contraindication to post- tion to suggest a myo ascial pain disorder. T e patient’s history
coital treatment is active pregnancy. Use a progestin- and physical examination, including uterosacral nodularity, is
only regimen in women with a history o thromboem- consistent with a diagnosis o endometriosis. Dysplasia is typi-
bolism. cally asymptomatic. T ere were no bladder symptoms elicited
on the history to suggest pain ul bladder syndrome. Be truth ul;
have you ever actually elt ligament nodularity?
HELPFUL TIP:
HELPFUL TIP:
Mi epristone (Mi eprex) is not approved in the United
I a ter a care ul assessment the diagnosis o endome-
States for postcoital pregnancy prophylaxis but is
triosis is highly likely, empiric therapy is considered a vi-
very e ective (99–100%) and has a more avorable side-
able alternative to laparoscopy and pre erred by many
e ect pro ile when compared with other regimens.
experts. However, de initive diagnosis relies upon direct
Mi epristone is approved by the FDA to use in conjunc-
visualization o endometrial implants con irmed by his-
tion with misoprostol (Cytotec) or termination o early
tologic examination.
pregnancy (within 49 days o last menstrual period).
O note, there have been serious adverse events as-
sociated with this combination, including sepsis rom She does not want la arosco y. You o er alternatives, and
Clostridium in ections and even death. However, these she elects to undergo cycle su ression with a 3- onth trial
events are very rare. o leu rolide (Lu ron) and to co lete a ain calendar. You
see her in ollow-u in 3 onths, and she is eeling uch bet-
ter. T e ain has been al ost co letely su ressed, and she
You reco end Plan B. Since she now trusts you, she sched- has issed only 1 day o work since you last saw her—but that
ules an a oint ent with you or evaluation o inter ittent was or a Star rek convention. A er all, the uture birth lace
abdo inal and elvic ain. Her ain has gradually worsened o Ca tain Kirk is located in Iowa (Riverside, Iowa). She has
over the last 2 years and is al ost o ni resent. Now she hot ashes, but they are inor.
co lains o severe abdo inal cra ing and stabbing in the
right lower quadrant. T e ain radiates to the le lower quad- Question 15.17.3 What is the most appropriate manage
rant at ti es and is worse during enstruation. Her eriods ment at this point?
have beco e heavier and occasionally irregular. She has no A) Continue the Lupron or up to another 3 months (6 months
bowel or bladder sy to s. She has been issing work 1 or total)
2 days each onth and is now concerned about her job. B) Stop the Lupron and monitor
Your exa ination reveals a well-develo ed wo an who C) Switch to a trial o cycle suppression using Depo-Provera
looks de ressed and unco ortable. Her abdo en is so , (medroxyprogesterone acetate) or continuous low-dose
nondistended, and di usely tender to al ation in the lower OCPs
CHAPTER 15 • OBSTETRICS AND WOMEN’S HEALTH 445

D) Switch to a trial o Premarin (conjugated estrogens)


E) A or C • Always ask about sexual and physical abuse: There
is a high correlation between chronic pelvic pain and
Answer 15.17.3 The correct answer is “E.” For pain relie , a history o sexual abuse.
treatment with a GnRH agonist or 3 to 6 months is e ective • To go by the textbook, the “diagnosis” o chronic pel-
in most patients. Oral contraceptives, as well as oral or depo vic pain requires a negative diagnostic laparoscopy.
progestins (Provera), are more e ective than placebo. Given the However, laparoscopy may not be per ormed in all
marked treatment success with the depo-Lupron, discontinu- cases.
ing treatment would likely result in recurrence o the patient’s
pain symptoms. Since endometriosis is estrogen dependent, use
o estrogen (Premarin) theoretically could worsen symptoms. Objectives: Did you learn to . . .
Another option is treatment with danazol. However, danazol is • Manage patients who desire emergency oral contraception?
less well tolerated than GnRH agonists, Provera, or OCPs. • De ne and evaluate chronic pelvic pain?
• Describe the rami cations, evaluation, diagnosis, and man-
HELPFUL TIP: agement o endometriosis?
NSAIDs are use ul monotherapy in patients with mild
endometriosis and are use ul in combination with
hormonal therapy (e.g., OCPs) or patients with more QUICK QUIZ: A“TRICHY” 1
severe symptoms.
A 19-year-old sexually active emale presents to your urgent
care center with a oul smelling vaginal discharge. She has noted
T e atient is concerned about how endo etriosis ay a ect the discharge or about 3 days. On examination, she is in no
her uture ertility. You recall that she is 25 years old and has acute distress, and her vital signs are normal. Her pelvic exami-
never atte ted regnancy (at least not intentionally). She nation is remarkable or mild vaginal erythema and a rothy
has regular enstrual cycles. gray discharge. You note a malodorous discharge and suspect
Trichomonas (bacterial vaginosis can also be malodorous, o
Question 15.17.4 What will you tell her? course, with a shy smell). A wet prep con rms your diagnosis.
A) “You are surely in ertile. Look into adoption”
B) “You just cannot know or sure until you have tried to con- At this point in time, you recommend that she also be tested or:
ceive” A) Chlamydia and gonorrhea
C) “What are you worried about? T ere is no association B) Herpes simplex
between endometriosis and in ertility” C) Hepatitis A
D) “Look at my ace. See how tired I am? Do you really want D) All o the above
kids?”
The correct answer is “A.” Routine screening or chlamydia
Answer 15.17.4 The correct answer is “B.” It is impossible to and gonorrhea in ection is recommended or all sexually active
predict ertility and in ertility based on the available in orma- adolescents. Given the presence o one S I, it is appropriate to
tion. Early-stage endometriosis is not likely to be associated o er testing or other S Is at this visit. Currently, herpes simplex
with alterations in ecundity. I the patient is willing, a diagnos- virus (HSV) is not routinely tested or in asymptomatic persons,
tic laparoscopy may aid in visualization o anatomic pathology and the USPS F recommends against serologic screening or
and allow one to render a guess as to possible tubal disease. HSV. Hepatitis A is not a sexually transmitted disease. It may be
appropriate to o er testing or hepatitis B, HIV, syphilis, etc., as
HELPFUL TIP: individual cases dictate.
Chronic pelvic pain is a symptom, not a speci ic disease.
By de inition, chronic pelvic pain re ers to pain that has
been present or more than 6 months and or which QUICK QUIZ: PROLAPSE
a thorough investigation has been negative. Here is a
help ul starting guide or elucidating the underlying An 80-year-old woman presents or evaluation o a “bulge” she
causes o chronic pelvic pain: noted a er gardening over the weekend . . . or maybe it’s been
• Is the pain cyclic? It may be related to endometriosis, there or years—she’s not sure. She has no discom ort and no
dysmenorrhea, adenomyosis, or other diseases that dif culty with bowel or bladder elimination. On examination,
respond to hormones such as irritable bowel syn- you note her cervix extends 1 cm beyond the vaginal introitus
drome and interstitial cystitis. with Valsalva. T ere are no lesions or excoriations noted.
• Is the pain noncyclic? It may be urinary, constipa-
tion, a myo ascial trigger point in the abdominal wall, O the ollowing, what is the best initial treatment option?
etc. A) Hysterectomy
B) rachelectomy
446 FAMILY MEDICINE EXAMINATION & BOARD REVIEW

C) Pessary trial has a sensitivity around 95% and a speci city around 90% or
D) Bed rest the diagnosis o ovarian cancer.) CA-125 is use ul in ollow-up
E) Hormone therapy o patients with a history o ovarian cancer.

The correct answer is “C.” Seventy percent o women who


are tted with a pessary are satis ed at 5-year ollow-up. Hys- QUICK QUIZ: GYNECOLOGIC CANCERS
terectomy and trachelectomy (removal o the cervix) are both
unnecessarily invasive treatments without trying a conservative What is the leading cause o death rom a gynecologic malig-
strategy. nancy in American women?
A) Ovarian cancer
B) Uterine cancer
QUICK QUIZ: OVARIAN MASS 1 C) Cervical cancer
D) Fallopian tube cancer
What is the best way to manage an asymptomatic 4-cm ovarian E) Vaginal cancer
cystic mass ound initially on pelvic examination and con rmed
by pelvic ultrasound as a simple cyst in a 22-year-old emale The correct answer is “A.” Ovarian cancer is the leading cause o
who is otherwise healthy? death rom gynecologic malignancy, is the second most common
A) Re erral to a gynecologist gynecologic malignancy, and is the ourth leading cause o cancer
B) Start hormonal therapy to reduce ovulation death in women. Endometrial cancer is the most common gyne-
C) Expectant management with repeat ultrasound in 2 months cologic malignancy but also one o the most treatable. Cervical
D) Serum CA-125 level cancer is the third most common gynecologic malignancy. Both
allopian tube and vaginal malignancy are relatively rare.
The correct answer is “C.” A 4-cm ovarian mass likely repre-
sents a unctional cyst in a woman who is cycling (reproductive
age). In general, premenopausal women with simple cysts 10 cm QUICK QUIZ: OVARIAN CANCER
or smaller may be managed conservatively with serial ultraso-
nography in 4 to 12 weeks. Since the mass is asymptomatic and How does ovarian cancer typically present?
without concerning ultrasound ndings (complex mass, sep- A) Early satiety
tations, solid components), expectant management is the best B) Abdominal ullness and pain
option. No urther evaluation is warranted at this time. CA-125 C) Urinary obstruction
is a tumor marker or epithelial ovarian cancer, but it is not use- D) Asymptomatic mass noted on routine examination
ul as a screening test. E) A and B

The correct answer is “E.” T ere are no speci c early symptoms


QUICK QUIZ: OVARIAN MASS 2 o ovarian cancer. T us, most patients present with symptoms
associated with increasing tumor mass: early satiety, abdominal
What is the best rst step in managing an asymptomatic palpable ullness or bloating, and abdominal pain. Un ortunately, ovar-
4 cm adnexal mass in a 76-year-old postmenopausal woman? ian cancer is rarely identi ed at an early stage on routine annual
A) Re erral to a gynecologist examination.
B) Start hormonal therapy to suppress FSH and LH
C) Expectant management with repeat examination in HELPFUL TIP:
2 months An ovarian mass is more likely to be malignant i the
D) Serum CA-125 level patient is premenarcheal or postmenopausal, is greater
E) Pelvic ultrasound than 10 cm in diameter, and has solid or complex cystic
eatures on ultrasound.
The correct answer is “E.” Unlike a relatively small palpable
ovarian mass in a reproductive-age woman, a palpable ovarian
mass in a postmenopausal woman represents ovarian malig-
nancy until proven otherwise. T e best initial imaging study
CASE 15.18
or evaluation o a pelvic mass is ultrasound. Ultrasound will While covering the e ergency de art ent on the graveyard
not only identi y the location o the mass but will also iden- shi a 21-year-old college student resents sobbing with a
ti y its internal consistency. Characteristics suggestive o cancer riend. Her riend says, “She’s been ra ed.”
include bilaterality, solid and cystic components, thick septa-
tions, and the presence o ascites. CA-125 is a marker or epi- Question 15.18.1 Relevant history includes all o the ol
thelial ovarian cancer and may assist in evaluation, but it cannot lowing EXCEPT:
be relied upon to rule in or to rule out cancer as a diagnosis. A) Whether orce was used and what type
(Best case scenario, in appropriately selected patients, CA-125 B) Physical characteristics o the assailant
CHAPTER 15 • OBSTETRICS AND WOMEN’S HEALTH 447

C) Details regarding penetration (vaginal, anal, oral) Answer 15.18.3 The correct answer is “B.” T e yield o a oren-
D) Number o sexual partners the victim has had in her li etime sic examination declines with time. Even i a patient states that
E) Condom use she does not want to prosecute the assailant, she should be
encouraged to have the examination done in case she changes
Answer 15.18.1 The correct answer is “D.” T e patient’s past her mind. Also, you are concerned about her health, and she
sexual history is not relevant in the evaluation o sexual assault. may be at risk or sexually transmitted diseases, pregnancy, and
T e other issues are pertinent to the case. Although it may be traumatic injury. Despite the act that yield does decline with
dif cult or the patient to relive the experience, you should try time, reliable evidence may still be gathered up to 5 days a er
to obtain a detailed history o the assault. In order to assess her the assault.
risk or pregnancy and in ection, you need to ask about the area
penetrated (e.g., vaginal, oral, or anal penetration), whether
the assailant ejaculated, and i a condom was used. In a sexual HELPFUL TIP:
assault case, your job is also to collect evidence, including per- “Rape trauma syndrome” generally occurs in three stag-
tinent historical elements (e.g., number o assailants, names, es. The irst includes anger, anxiety, guilt, shame, sleep
physical appearance, whether orce was used, and what type— disturbance, etc. The second stage includes somatic
threat, restraints, weapons, etc.). complaints (pelvic pain, other pain) and psychiatric
complaints (depression, phobias, etc.). Some patients
will resolve these issues while others will develop post-
HELPFUL TIP:
traumatic stress syndrome. The third stage is renormal-
Sexual assault includes genital, anal, or oral penetration
ization.
by a part o the assailant’s body or by an object. By de i-
nition, it occurs without the victim’s consent and need
not involve direct orce or violence.
Question 15.18.4 Which o the ollowing is the LEAST appro
priate to o er this patient at this point in time?
Question 15.18.2 Which o the ollowing are important A) HIV antigen/antibody testing
physical elements to collect or the orensic evaluation in B) HSV antibody testing
this case? C) Prophylactic treatment or gonorrhea and chlamydia
A) Combed specimens rom the scalp and pubic hair D) Mental health services re erral
B) Swabs o the oral, vaginal, and rectal mucosa E) Emergency contraception
C) T e patient’s clothing
D) Fingernail scrapings Answer 15.18.4 The correct answer is “B.” Herpes virus anti-
E) All o the above body testing will only tell you i she has been exposed to HSV
in the past. Further recommendations include syphilis testing,
Answer 15.18.2 The correct answer is “E.” All o the items hepatitis B antibody testing, per orming a wet prep o a vaginal
listed will be important to the investigation. Evidence collection sample, and checking a urine pregnancy test.
kits or sexual assault cases (“rape kits”) should be available in
your emergency department. Objectives: Did you learn to . . .
• Evaluate a patient or sexual assault?
Although a arently inebriated, the atient is able to give a • Manage a patient who has been the victim o sexual assault?
coherent history. When you broach the subject o hysical
exa ination, her riend says, “Look, she was ra ed an hour
ago. Can’t you let her just recover a bit be ore you violate her
CASE 15.19
all over again?” A 21-year-old wo an resents to your of ce co laining o
elvic ain with intercourse, worse over the last 2 weeks. She
Question 15.18.3 Which o the ollowing is the most appro also co lains o not getting regnant, even though she’s had
priate response? several artners over her last 3 years o sexual activity and
A) “O course. Come back tomorrow a er you have sobered up has been trying to get regnant with the sa e artner or the
and taken a shower” ast 6 onths. She states she never has used birth control
B) “An examination is important or your health and in the o any ty e—not even once. You co end her on co it-
event that this becomes a criminal case. T e yield o the ent to her rinci les. She started her eriods around age 14
examination declines with time. Even i you don’t eel like but has only had a cou le o eriods since then. A arently,
prosecuting now, you may decide to do so in the uture, and this attern o enstruation is nor al or her a ily, as her
the best evidence is gathered early” other was the sa e way.
C) “T e examination has a airly high yield even a week a er On hysical exa ination, you notice the atient is a cen-
the assault, so take your time on this” trally obese young wo an, a ebrile, with (culturally de ned)
D) “Under ederal law I am required to per orm this examina- excess hair noted down the side o her ace and under her
tion” chin. She also has so e erythe atous ustules on her cheeks.
448 FAMILY MEDICINE EXAMINATION & BOARD REVIEW

Question 15.19.1 Which o the ollowing lab results would Question 15.19.2 O the ollowing, which is the most likely
be most consistent with the history and examination nd cause o her cervicitis?
ings? A) HSV in ection
A) Positive urine pregnancy test B) Trichomonas vaginalis in ection
B) Low SH level C) Candida albicans in ection
C) Elevated CA-125 level D) PID
D) Mildly elevated androgens E) Bacterial vaginosis
E) Prolactin level more than three times normal
Answer 15.19.2 The correct answer is “B.” Trichomonas is a
Answer 15.19.1 The correct answer is “D.” T is patient gives a protozoan that is sexually transmitted and can cause urethritis
history and has an appearance consistent with polycystic ovar- in both sexes. However, in women, it most commonly causes
ian syndrome (PCOS). T e clinical eatures o PCOS include ulceration o the cervical mucosa with punctate hemorrhages
oligomenorrhea (90%), hirsutism (80%), obesity (50%), amen- known as a “strawberry cervix.” Signs and symptoms also
orrhea (40%), and in ertility (40%). Early symptoms in an ado- include a malodorous discharge and occasional vulvar and
lescent may consist only o irregular periods, acne, and central vaginal irritation. T e cervix can be somewhat tender to touch,
obesity. Clinical or laboratory evidence o androgen excess either during examination or intercourse, and patients o en
may be present, such as mildly elevated testosterone. Note that complain o a nonspeci c pelvic pain. Males are o en asymp-
depending on the diagnostic criteria you are using or PCOS, an tomatic.
ultrasound may not be necessary. An LH:FSH ratio greater than
3:1 adds urther support to the diagnosis. You should certainly T e wet ount de onstrates Trichomonas; there is no evi-
do a pregnancy test, a SH, and a prolactin level. However, this dence o yeast or clue cells. You send sa les or chla ydia
patient most likely has PCOS. and gonorrhea tests as well as a Pa s ear. You reco end
testing or HIV, sy hilis, and he atitis B, and she agrees.
HELPFUL TIP:
The most recent evidence points to insulin resistance Question 15.19.3 For her Trichomonas vaginal in ection,
as the underlying cause o PCOS, and these patients you prescribe:
may have acanthosis nigricans. Insulin resistance can A) Flagyl (metronidazole) 2 g orally in a single dose
be quanti ied by calculating the ratio o asting glucose B) MetroGel-Vaginal (topical vaginal metronidazole) 5 g applied
to insulin. A ratio o less than 4.5 indicates insulin resis- nightly or 5 days
tance. Insulin resistance stimulates ovarian androgen C) Di ucan ( uconazole) 150 mg orally in a single dose
production, which leads to anovulation. D) Zithromax (azithromycin) 1 g orally in a single dose
E) Levaquin (levo oxacin) 250 mg orally in a single dose

You roceed with the elvic ortion o the exa ination, not- Answer 15.19.3 The correct answer is “A.” T e best choice
ing the atient also has a dia ond-sha ed, rather than trian- is oral metronidazole. opical antibiotic gels, creams, or
gular-sha ed, ubic hair attern. You nd no lesions on the ovules—either metronidazole or clindamycin (Cleocin)—only
vulva or in the vagina. However, the cervix a ears reddened, treat bacterial vaginosis, as the concentration is insuf cient
with an al ost strawberry texture. And, even though there is to reach the protozoa in the glands and urethral areas. T e
a generous a ount o yellowish, alodorous leukorrhea in remaining options are all incorrect or treating Trichomonas.
the vaginal vault, there is no notable us at the cervical os. See able 15-9 or more on diagnosis and treatment o in ec-
Bi anual exa ination is li ited due to the atient’s obesity. tious vaginitis.

TABLE 15-9 VAGINITIS DIAGNOSIS AND TREATMENT


Organism Discharge Odor Microscopy pH Treatment
Bacterial Thin, gray, Fishy with Clue cells > 4.5 Metronidazole 500 mg BID × 7 days or
vaginosis homogeneous positive “whif clindamycin 300 mg BID × 7 days or topical
test” metronidazole 1 applicator ul intravaginally
daily × 5 days (lower success rate)

Candida Adherent, white, Neutral Pseudohyphae but only < 4.5 Fluconazole oral, topical clotrimazole,
“cottage cheese” like 65–85% sensitive miconazole, etc.

Trichomonas Copious yellow, gray, Malodorous Trichomonads > 4.5 Metronidazole 2 g PO once (recommended),
green, oamy. Friable or 500 mg
“strawberry” cervix BID × 7 days (alternative)

Adapted rom Hainer BL, Gibson MV. Vaginitis: diagnosis and treatment. Am Fam Physician. 2011;83(7):807–815.
CHAPTER 15 • OBSTETRICS AND WOMEN’S HEALTH 449

HELPFUL TIP: HELPFUL TIP:


As with other STIs, a patient with Trichomonas should Not all women with PCOS are obese and hirsute. Many
have her partner tested and treated (or just treated de- patients may be thin with sparse body hair and present
pending whether or not this is allowable under your with menstrual irregularities and ertility concerns.
state law).

Objectives: Did you learn to . . .


• Identi y the clinical presentation o PCOS?
HELPFUL TIP:
• Diagnose and treat Trichomonas in ection?
While it makes sense that single-dose azithromycin
would work better in treating Chlamydial cervicitis be- • Diagnose and manage PCOS?
cause o compliance issues, the cure rate is the same
whether azithromycin or the doxycycline is used. There QUICK QUIZ: SCREENING IN TEENS
is about a 3% ailure rate with azithromycin which isn’t
seen with doxycycline.
A 16-year-old emale presents with her mother. T ey don’t look
happy. Her mother says, “She needs a Pap smear because she’s
been having sex with a couple o boys—in my house, I will have
You now return your attention to her PCOS (re e ber, way you know— or a year!” T e patient rolls her eyes.
back then, the reason she ca e in?). Her lab results de -
Consistent with published guidelines, you recommend:
onstrated a LH:FSH ratio > 3; nor al SH and rolactin;
A) Pap smear
slightly elevated testosterone, but still well below the nor al
B) Gonorrhea and chlamydia testing
ale range; asting glucose:insulin ratio < 4.5; and slightly
C) Pap smear and gonorrhea and chlamydia testing
elevated total cholesterol and triglycerides. Her LH:FSH ratio
D) Return or a Pap a er sexually active or 3 years (age 18 or
was consistent with the diagnosis o PCOS. She has not en-
this patient)
struated or 4 onths. A urine hCG is negative, and a re eat
E) A chastity belt
asting glucose is 120 g/dL.
The correct answer is “B.” It is now recommended to delay cer-
Question 15.19.4 Which o the ollowing recommendations vical cancer screening until age 21, even i the woman has been
should you make now? sexually active. T e reasoning: although adolescent emales are
A) Initiate met ormin requently in ected with HPV, they also easily clear these in ec-
B) Attempt weight loss through a nutritious diet and increased tions, with 95% o lesions spontaneously regressing. Exceptions
exercise to this rule include patients who are immunocompromised
C) Initiate oral contraceptives to regulate menses (e.g., organ transplant, HIV in ection). For this patient, ensur-
D) A and C ing that she has completed her HPV serious and counseling on
E) All o the above sa e sexual behavior is also warranted.
Answer 15.19.4 The correct answer is “E.” T is patient has
glucose intolerance (elevated asting glucose and ratio o glu- QUICK QUIZ: AN ITCHYVULVA
cose: insulin < 4.5), and it is reasonable to initiate dietary and
medical therapy at this point in time. Another option is to start A 65-year-old emale presents or a health maintenance exami-
with li estyle modi cations and check asting glucose again nation. She complains o a vulvar itching due to what she calls
3 to 6 months later. Due to increased risk o endometrial car- “recurrent yeast in ections,” and her symptoms have worsened
cinoma in patients who have rare menses, it is important to over the last ew months. She is sexually active with her hus-
regulate her cycles. OCPs (“C”) can accomplish menstrual band and has experienced dyspareunia with penetration lately.
regulation. She always uses a water-based lubricant with intercourse. On
examination, you nd complete loss o the borders o the labia
minora, constriction o the vaginal outlet, and several thin white
HELPFUL TIP: plaques (like parchment paper) on the vulva. T ere is no other
For hirsutism associated with PCOS, spironolactone is skin or mucosal involvement.
usually irst-line therapy unless the patient has a con-
traindication; traditional hair removal techniques will What is the most likely diagnosis?
still be required or the existing hair growth. Since spi- A) Lichen planus
ronolactone can result in eminization o a male etus, B) Lichen simplex
patients taking spironolactone must be using reliable C) Lichen sclerosus
birth control. D) Vulvovaginal candidiasis
E) Squamous carcinoma
450 FAMILY MEDICINE EXAMINATION & BOARD REVIEW

The correct answer is “C.” T e clinical description above is although your patient satis action scores may plummet. (Inci-
characteristic o lichen sclerosus, which occurs more commonly dentally, patient satis action has been inversely correlated with
in older women but also has a peak in young girls. Almost all healthcare quality—although there appears to be a direct link
lichen sclerosus is intensely pruritic. As lichen sclerosus pro- between increased satis action and increased health care expen-
gresses, there may be loss o labial architecture, stenosis o the diture.)
introitus, and obliteration o the clitoris. T e lesions are usually
multiple and appear as thin, shiny, white, wrinkled patches or CASE 15.20
plaques. T e rest are incorrect. However, “E,” squamous car-
A 27-year-old e ale resents with her husband seeking
cinoma, occasionally can be con used or lichen sclerosus but
advice regarding regnancy loss. She recently had a iscar-
is more likely to present with ulceration and induration and is
riage. Your atient states that this was her third iscarriage
less common (though i you are suspicious o cancer, a biopsy is
in the last 2 years. All three occurred at about 9 weeks o ges-
indicated). Patients with lichen sclerosus have a squamous cell
tation.
cancer risk o 3% to 7% and should have an examination every
6 to 12 months with biopsies as indicated or persistent and
Question 15.20.1 Possible explanations or recurrent preg
nonhealing lesions.
nancy loss in this patient include each o the ollowing
EXCEPT:
HELPFUL TIP: A) Parental structural chromosome abnormalities
Initial treatment or lichen sclerosus involves local ste- B) Uterine anatomic abnormalities
roid ointment application. High-potency steroids are C) Anticardiolipin antibody syndrome
initiated and then tapered to the lowest e ective po- D) Idiopathic (unexplained etiology)
tency and requency that maintain symptom control. E) Conception while on oral contraceptives
Testosterone creams have allen out o avor due to
lesser e icacy and secondary virilization. Vaginal dila- Answer 15.20.1 The correct answer is “E.” Conception while
tors can be used i there is constriction o the vaginal on oral contraceptives will not increase the risk o recurrent
opening causing dyspareunia. spontaneous miscarriages. Parental structural chromosome
abnormalities (balanced structural chromosome rearrangement
in one partner) are responsible or pregnancy loss in 2% to 4%
o couples. Uterine anatomic abnormalities (“B”) have been
QUICK QUIZ: VULVOVAGINAL CANDIDIASIS associated with 10% to 15% o pregnancy loss. “D” is true. T e
majority o couples with recurrent pregnancy loss will have an
A 48-year-old perimenopausal emale presents with a 3-day his- uncertain etiology despite extensive evaluation (> 50%).
tory o vulvar pruritus. Her history is signi cant or mitral valve
replacement and she is on war arin with INR 2–3. A limited HELPFUL TIP:
vulvar and vaginal examination reveals signi cant erythema Recurrent pregnancy loss is classically de ined as loss o
with satellite lesions on the labia majora. Wet prep microscopy three or more consecutive pregnancies.
reveals abundant pseudohyphae and in ammatory cells. You
somehow assemble all these clues into a diagnosis o candidal
vulvovaginitis (and you can tell the patient is impressed when
she says, “T at’s right, genius. I’ve got a yeast in ection”). She T e cou le desires testing or ossible causes o the reg-
enquires about use o oral therapy, as vaginal creams are “messy.” nancy losses.

How will you counsel this patient regarding use o oral ucon- Question 15.20.2 O the ollowing, which test(s) should be
azole (Di ucan)? included in the evaluation?
A) “You have no contraindications to oral uconazole” A) Cultures or bacteria
B) “Given your use o war arin, you should not use oral ucon- B) est or glucose intolerance
azole” C) Maternal antipaternal antibodies
C) “You will need to stop war arin while taking oral ucon- D) Lupus anticoagulant and anticardiolipin antibody
azole” E) All o the above
D) “You should take extra war arin i you take oral uconazole”
Answer 15.20.2 The correct answer is “D.” Antiphospholipid
The correct answer is “B.” T ere are numerous drug interac- syndrome is associated with pregnancy loss in 3% to 15% o
tions with oral uconazole, including war arin (both inhibit women with recurrent pregnancy loss. T e others are not use-
CYP3A4). T e INR will increase a er even one dose o uco- ul. However, chromosomal testing o the parents (not given as
nazole therapy. Similarly, the ubiquitous statins are a ected an option in this question) may be use ul. Other tests that can
by oral uconazole with several case reports o rhabdomyoly- be considered include screening or thyroid abnormalities with
sis in the literature. In this case, stick with topical anti ungals, anti-thyroid peroxidase ( PO) and SH with ree 4.
CHAPTER 15 • OBSTETRICS AND WOMEN’S HEALTH 451

Evaluation o the recurrent regnancy loss ails to identi y a TABLE 15-10 PREGNANCY WEIGHT GAIN GUIDELINES
cause. T us, like ost cou les with recurrent regnancy loss, BMI (kg/m 2) Goal Weight Gain (kg) Goal Weight Gain (lb)
the etiology re ains unex lained. < 18.5 12.5–18.0 28–40

Question 15.20.3 What is the likelihood that this couple 18.5–24.9 11.5–16.0 25–35
will have a success ul pregnancy outcome in the next preg 25.0–29.9 7.0–11.5 15–25
nancy?
A) Highly unlikely, they should consider adoption > 30 5–9 11–20
B) Less than one in our chances o success ul pregnancy
Adapted rom Institute o Medicine, Resource Sheet, May 2009: Weight
C) 60% to 70% chance o success ul next pregnancy Gain During Pregnancy: Reexamining the Guidelines: http://www.iom.
D) You cannot hazard a guess. Amazingly, this has not been edu/Reports/2009/Weight-Gain-During-Pregnancy-Reexamining-the-
studied Guidelines.aspx.

Answer 15.20.3 The correct answer is “C.” Studies suggest that


60% to 70% o couples with unexplained recurrent pregnancy
loss will have a success ul next pregnancy. Being the smart doctor that you are, you know that a pregnancy
test is the rst thing to do: it is negative. Being the smart doc-
Objectives: Did you learn to . . . tor that you are, you also think about the emale athlete triad:
• De ne recurrent pregnancy loss and discuss some o its amenorrhea, an eating disorder, and osteoporosis. She does not
epidemiologic aspects? meet this pro le. You take more o a history and nd out the
• Enumerate potential causes o recurrent pregnancy loss? ollowing:
• Identi y etiologies and the workup o recurrent pregnancy • T ere is no additional stress in her li e (such as starting col-
loss?
lege or a new job), weight loss, or illness, etc., which might
lead to hypothalamic amenorrhea.
• She has no galactorrhea (prolactinoma).
QUICK QUIZ: WEIGHT GAIN IN PREGNANCY
• She has no hot ashes, vaginal dryness, etc. (premature
menopause).
A 28-year-old primigravida emale presents or an initial obstet-
ric visit. Pelvic examination is consistent with a 6- to 8-week • She denies headaches, visual changes, atigue, polydipsia, or
gestation uterus, and the remainder o the examination is unre- polyuria (pituitary problems).
markable. As this is her rst pregnancy, she has a number o • She has no acne, hirsutism, etc., suggestive o PCOS.
questions. She wants to know how much weight gain is expected Hey, did you notice how we are cleverly giving you the
and whether she should “watch her weight.” workup o secondary amenorrhea in the question?
As recommended, you check a SH and prolactin; they are
You calculate her BMI as 24 kg/m 2 and recommend the ollow- normal.
ing:
A) “Eat anything you want. You’re eating or two!” T e next step in the evaluation o this patient’s amenorrhea is:
B) “Your BMI is normal. Your goal is to gain no more than 20 lb” A) A progestin challenge
C) “Your BMI is low. Your goal is to gain 40 lb” B) Hysterosalpingogram to prove cervical patency
D) “Your BMI is high. Your goal is to gain no more than 15 lb” C) A trial o oral contraceptives to prove cervical patency
E) “Your BMI is normal. Your goal is to gain 30 lb” D) An LH to rule out menopause
E) All o the above
The correct answer is “E.” T e Institute o Medicine recom-
mends weight gain in pregnancy based on pregravid BMI (see The correct answer is “A.” A progestin challenge should cause
able 15-10). In this patient, her BMI is in the normal range, so a withdrawal bleed i there is still adequate estrogen (thus rul-
her goal or weight gain in pregnancy is 25 to 35 lb. Women with ing out premature ovarian ailure). T e step ollowing this
a normal prepregnancy BMI should gain about 1 lb per week would be to start a combination oral contraceptive. Failure to
during their second and third trimesters. induce menses with a combination OCP suggests a mechanical
blockage such as Asherman syndrome (scarring o the endo-
metrial lining with intrauterine adhesions). Failure to induce
QUICK QUIZ: AMENORRHEA menses with a combination OCP can be ollowed by a hys-
terosalpingogram or hysteroscopy to identi y any mechanical
A 32-year-old emale is coming to see you or amenorrhea. problems.
She had regular menses until the last year when they became
irregular. She has not had any menses or the past 6 months. You did it! You success ully worked u secondary a enor-
She is somewhat distraught because she wants to have a amily. rhea.
452 FAMILY MEDICINE EXAMINATION & BOARD REVIEW

American College o Obstetricians and Gynecologists. ACOG


HELPFUL TIP:
Practice Bulletin No. 112: Emergency contraception.
But wait, there is more! Check an FSH as well. I this is Obstet Gynecol. 2010;115:1100–1109.
high, it might indicate ovarian ailure. If the FSH is normal
American College o Obstetricians and Gynecologists. ACOG
or low in the absence of circulating estrogen (a nega- Practice Bulletin No. 130: Prediction and prevention o
tive progestin challenge), consider a pituitary cause o preterm birth. Obstet Gynecol. 2012;120(4):964–973.
amenorrhea including possible hypothalamic–pituitary American College o Obstetricians and Gynecologists. Prac-
axis dys unction rom weight loss, stress, pituitary tumor, tice bulletin No. 137: gestational diabetes mellitus. Obstet
empty sella syndrome, etc. No pituitary? No FSH. Gynecol. 2013;122(2, part 1):406–416.
American College o Obstetricians and Gynecologists. Execu-
tive summary: hypertension in pregnancy. Obstet Gynecol.
2013;122(5):1122–1131.
Clinical Pearls American College o Obstetricians and Gynecologists. Method
Do not per orm elective deliveries (C-sections or inductions) or estimating due date. Committee Opinion No. 611.
that are not medically indicated prior to 39 weeks o gesta- Obstet Gynecol. 2014;124:863–866.
tion. American College o Obstetricians and Gynecologists. Guide-
lines or acute and emergent therapy or acute-onset
Do not per orm routine screening or ovarian cancer in
severe hypertension during pregnancy and the postpartum
asymptomatic, low-to-average risk women.
period. Am Obstet Gyencol. 2015;No 623:1–5.
Do not require a Pap and a pelvic examination to prescribe Biggs W, Demuth RH. Premenstrual syndrome and premen-
contraception. The only necessary requirement is to per orm strual dysphoric disorder. Am Fam Physician. 2011;84(8):
a detailed history to ascertain risk actors o using hormonal 918–924.
contraception, blood pressure check, and the per ormance o
Centers or Disease Control (CDC). 2015 Sexually ransmitted
a urine pregnancy test to ensure the woman is not pregnant.
Diseases reatment Guidelines: Gonococcal in ections.
Do not routinely per orm episiotomies; there is no evidence Crossman SH. T e challenge o pelvic in ammatory disease.
that episiotomies reduce perineal trauma, postpartum Am Fam Physician. 2006;73:859–864.
dyspareunia, etc.
Division o Reproductive Health, National Center or Chronic
Do not start Pap smear screening be ore the age o 21 Disease Prevention and Health Promotion, Centers or
regardless o sexual activity status. Disease Control and Prevention (CDC). U.S. Selected
Due to the increased risk o endometrial cancer, do not use Practice Recommendations or Contraceptive Use, 2013:
unopposed estrogen in a woman unless she has undergone adapted rom the World Health Organization selected
a hysterectomy. practice recommendations or contraceptive use, 2nd
edition. MMWR Recomm Rep. 2013;62(RR-05):1–60.
In a pregnant woman with third-trimester vaginal bleeding,
do not per orm a digital or speculum vaginal examination Gradison M. Pelvic in ammatory disease. Am Fam Physician.
until you can rule out the diagnosis o placenta previa. 2012;85(8):791–796.
Hainer BL, Gibson MV. Vaginitis: diagnosis and treatment.
Per orm routine GBS screening on pregnant women between
Am Fam Physician. 2011;83(7):807–815.
35 and 37 weeks o gestation UNLESS the woman has a
documented history o GBS bacteriuria during current Hatcher RA, et al. A Pocket Guide to Managing Contraception.
pregnancy or prior history o in ant born with invasive GBS 6th ed. New York: Ardent Media, Inc.; 2005.
in ection, in which cases they would be treated anyway. Kirkham C, et al. Evidence-based prenatal care: Part I. general
prenatal care and counseling issues. Am Fam Physician.
Recommend olic acid in all women o childbearing age to
2005;71(5):1307–1316.
prevent any risk o neural tube de ects i she should become
pregnant. Kirkham C, et al. Evidence-based prenatal care: Part II. third-
trimester care and prevention o in ectious diseases. Am
Use the ASCCP guidelines to determine requency o Pap Fam Physician. 2005;71:1555–1562.
smears and management o abnormal smears.
Klein DA, et al. Provision o contraception: key recommenda-
tions rom the CDC. Am Fam Physician. 2015;91(9):
625–633.
LeFevre ML. Low-dose aspirin use or the prevention o
BIBLIOGRAPHY morbidity and mortality rom preeclampsia: US Preventa-
American College o Obstetricians and Gynecologists. ACOG tive Services ask Force recommendation statement. Ann
Practice Bulletin no. 83: Management o adnexal masses. Intern Med. 2014;161:819–826.
Obstet Gynecol. 2007;110(1):201–214. Magnotti M, Futterweit W. Obesity and the polycystic ovary
American College o Obstetricians and Gynecologists. ACOG syndrome. Med Clin North Am. 2007;91(6):1151–1168.
Practice Bulletin no. 106: Intrapartum etal heart rate Massad LS, et al. American Society or Colposcopy and Cervi-
monitoring, nomenclature, interpretation and general cal Pathology 2012 updated consensus guidelines or the
management principles. Obstet Gynecol. 2009;114(1): management o abnormal cervical cancer screening tests
192–202. and precursors. J Low Genit Tract Dis. 2013;17(5):S1–S27.
CHAPTER 15 • OBSTETRICS AND WOMEN’S HEALTH 453

Ott MA, et al. Policy statement: Contraception or adolescents. enore JL. Methods or cervical ripening and induction o
Pediatrics. 2014;134(4):e1244–e1256. labor. Am Fam Physician. 2003;67:2123–2128.
Raina R, et al. Female sexual dys unction: classi ca- US Food and Drug Administration. Mi eprex (mi epristone)
tion, pathophysiology, and management. Fertil Steril. in ormation. Available at http://www. da.gov/Drugs/
2007;88(5):1273–1284. DrugSa ety/PostmarketDrugSa etyIn ormation or
Rasmussen KM, et al. Institutes o Medicine Report Brie . PatientsandProviders/ucm111323.htm. Updated July 17,
Weight gain during pregnancy: re-examining the guidelines. 2015.
Washington, DC: National Academies Press; 2009. Avail- Workowski KA, Bolan GA. Sexually transmitted diseases treat-
able at http://www.iom.edu/Reports/2009/Weight-Gain- ment guidelines, 2015. MMWR Recomm Rep. 2015;
During-Pregnancy-Reexamining-the-Guidelines.aspx. 64(RR-3):1–137.
an PC, et al. E ect o coitus at term on length o gestation, Zolotor AJ. Update on prenatal care. Am Fam Physician.
induction o labor, and mode o delivery. Obstet Gynecol. 2014;89(3):199–208.
2006;108(1):134–140.
Men’s Health
Vic oria J. A. Sharp a d Jaso K. Wi bur
16
Prostate Symptom Score, which is remarkably similar but has an
CASE 16.1 additional question regarding how symptoms a ect the patient’s
A 58-year-old black male presents to your clinic complaining quality o li e. Since your patient may not empty his bladder
o urinary hesitancy, requency, and three to our episodes o well, a post-void residual urine volume and urinalysis will help
nocturia per night, which have been worsening over the past determine i he is experiencing urinary retention (“C”) or an
ew years. His urinary stream is weaker than it was a ew years in ection (“E”).
ago and he eels he does not empty his bladder completely. He
denies any history o urinary tract in ections (U Is) or pain ul Question 16.1.2 When considering benign prostatic hyper-
urination. He is otherwise well with no signi cant past medi- plasia (BPH), you re ect on the common symptoms o this
cal or surgical history. Currently, he takes no medications and syndrome, which include all o the ollowing EXCEPT:
has no allergies. On reviewing his amily history, he notes A) Urinary retention
his ather and older brother died o prostate cancer in their B) Post-void dribbling
50s. His general physical examination is normal and a geni- C) Frequency
tal examination is unremarkable. Digital rectal examination D) Nocturia
reveals a smooth prostate with no nodules or tenderness. E) Hematuria

Question 16.1.1 Based on this patient’s history and physical Answer 16.1.2 The correct answer is “E.” Hematuria is not
examination, all o the ollowing would be appropriate at usually associated with BPH. However, it can occur i a man’s
this stage EXCEPT: prostatic urethra is very enlarged and riable. Enlargement o
A) Serum assay or prostate-speci c antigen (PSA) the prostate o en results in obstructive ow symptoms (e.g.,
B) American Urological Association (AUA) symptom score hesitancy and slow, weak stream), which in turn can lead to
C) Post-void residual urine volume irritative symptoms (e.g., requency, urgency, and nocturia).
D) ransrectal ultrasound with prostate biopsies Obstruction rom an enlarged prostate alone can cause hyper-
E) Urinalysis and microscopic examination o the urine trophy o the detrusor, or it can lead to an in ection that results
in detrusor instability—the cause o irritative symptoms. I irri-
Answer 16.1.1 The correct answer is “D.” Although your tative symptoms are present without obstructive symptoms,
patient has an increased risk o prostate cancer, transrectal other diagnoses should be considered, including bladder
ultrasound with prostate biopsies is not indicated at this stage. cancer, urolithiasis, in ection, and neurogenic bladder.
T is diagnostic test should be reserved or a higher suspicion o
prostate cancer. Based on this patient’s amily history and the Your patient’s urinalysis and PSA are normal. A er emptying
act that he is black (black males have a 50% higher incidence 250 mL o urine, the postvoid residual urine volume is 50 mL.
o and mortality rom prostate cancer compared with whites),
PSA testing (“A”) is appropriate in this setting, as opposed to Question 16.1.3 With this in ormation, you recommend
general population screening, which is discussed later in the which o the ollowing strategies?
chapter. T e AUA symptom score (“B”) is a seven-item ques- A) Urodynamic studies
tionnaire about symptoms o urinary outlet obstruction, which B) Medical therapy
can be used to assess severity and assist in management o pros- C) Surgical therapy
tate disease. It is not very use ul or diagnosis, but on the bright D) Scheduled bladder catheterization
side, it is reely available online. T ere is also the International E) Bio eedback
454
CHAPTER 16 • Men ’S HeAl t H 455

Answer 16.1.3 The correct answer is “B.” You have enough the rst treatment); whereas the ull bene t o an alpha-blocker
in ormation to diagnose symptomatic BPH, and urther stud- will be apparent within 4 to 6 weeks. “B” and “D” are incorrect
ies are not necessary. Depending on the patient’s pre erences, because these anticholinergic drugs are used or incontinence
the next step is to begin treatment, and in most cases, medical due to detrusor instability and may make urinary retention
therapy is initiated rst. I medical therapy ails or i a patient worse in patients with outlet obstruction. Furosemide, “E,” is a
has severe BPH with ongoing obstruction, retention o large potent diuretic and would be a cruel joke to play on this patient.
volumes o urine, bladder stones, or recurrent U Is, surgical
therapy should be considered. T e most commonly per ormed HELPFUL TIP:
surgery is transurethral resection o the prostate ( URP), but Si c 5-a pha-r duc as i hibi ors shri k pros a is-
other techniques can be employed as well, including transure- su , h y r duc h produc io o pros a -sp ci ic
thral incision o the prostate, minimally invasive procedures, a ig (PSA). B awar ha PSA v s d cr as by
and open surgery or very enlarged prostate glands. “D” is approxima y 50% wi hi 6 mo hs o i i ia i g o
incorrect. Scheduled bladder catheterization is unlikely to ben- o h s drugs. t h r or , i or som r aso you ch ck
e t your patient since his post-void residual is not very large. a PSA i a ma o i as rid or du as rid , h va u
A post-void residual greater than 200 mL is associated with shou d b doub d wh compari g o orma ra g s.
an increased risk o U Is, and such patients may bene t rom
scheduled catheterizations i medical or surgical interventions
do not correct the problem or are contraindicated. Kegel exer- Question 16.1.5 Be ore you tear up the prescription or ter-
cises and bio eedback (“E”) may be used to treat incontinence azosin you accidentally wrote, you review its side e ects.
but are not used in BPH. Also, bio eedback is help ul or voiding Potential side e ects o alpha-blockers include all the
dys unction due to incomplete urinary sphincter relaxation. ollowing EXCEPT:
A) Retrograde ejaculation
Your powers o deduction lead you to the conclusion that the B) Hypertension
patient’s urinary symptoms are due to BPH. Embarrassed C) Intraoperative oppy iris syndrome
that he is taking more bathroom stops than his wi e on their D) Priapism
road trips, your patient desires treatment.
Answer 16.1.5 The correct answer is “B.” Hypotension (not
Question 16.1.4 To give him the most immediate relie , you hypertension) is the most commonly encountered problem
prescribe which o the ollowing? with terazosin and other alpha-blockers, including the uroselec-
A) Finasteride tive alpha-blockers, although to a lesser degree. In elderly males,
B) Oxybutynin the hypotension can be particularly problematic as the propen-
C) amsulosin sity or alling may increase. Additionally, alpha-blockers in
D) Imipramine combination with phosphodiesterase inhibitors (e.g., sildena l)
E) Furosemide can cause dangerously low blood pressures. Retrograde ejacula-
tion (“A”) and priapism (“D”) are not common but have been
Answer 16.1.4 The correct answer is “C.” iming and type o reported. And, as i we needed it, here is more evidence or a
intervention should depend on how much the patient is both- direct link between the male ocular and genital systems: intra-
ered by his symptoms and whether complications o BPH are operative oppy iris syndrome has been observed in men taking
present. I the symptoms do not signi cantly inter ere with your alpha-blockers and undergoing cataract surgery; causality has
patient’s li e, he may choose to wait and take no treatment once not been proven. Yes, you may have thought we made this up
he is reassured that he does not have a li e-threatening illness. In but we didn’t.
general, medical management begins with a selective alpha-1a
receptor blocker, such as tamsulosin, which relaxes the prostate You start tamsulosin. Un ortunately, the patient is not able
at the bladder neck. In the past, non-uroselective alpha-1 block- to tolerate it due to dizziness. His symptoms are bothersome
ers (e.g., terazosin, doxazosin) were considered rst-line; but enough that he wishes to try something else. You consider
this is no longer the case. Let’s ace it: i your patient is on an nasteride.
alpha-1 receptor blocker or any reason—including hyperten-
sion—he has tried a ew other things rst. T e risk–bene t ratio Question 16.1.6 Which o the ollowing is true o f nasteride?
does not avor them when compared with other drugs or BPH A) It permanently reduces prostate volume, even a er the drug
or hypertension (see next question). is stopped
I the patient does not receive su cient relie rom maxi- B) It is approved by the Food and Drug Administration (FDA)
mum doses o an alpha-1a blocker, consider adding a 5-alpha- or abnormal hair growth in women
reductase inhibitor (e.g., nasteride, dutasteride). T ese drugs C) It may reduce the overall risk o developing prostate cancer
work by reducing the size o the prostate gland by inter ering but increase the risk o developing high-grade prostate
with the e ects o androgens on prostate tissue. However, it may cancers
take up to 6 months or a 5-alpha-reductase inhibitor to result D) It improves symptoms within 1 week o starting the drug
in a noticeable di erence in symptoms (thus, “A” should not be E) None o the above
456 FAMIl Y MeDICIn e eXAMIn At IOn & BOARD ReVIeW

Answer 16.1.6 The correct answer is “C.” T is is important: C) Bacteriuria


5-alpha-reductase inhibitors lower the overall risk o can- D) Elevated creatinine
cer but increase the risk o those cancers diagnosed being E) Elevated PSA
high grade. Finasteride (and dutasteride) works by inhibiting
5-alpha-reductase, which is the enzyme that converts testos- Answer 16.1.8 The correct answer is “D.” ests o renal unc-
terone to dihydrotestosterone. Dihydrotestosterone stimulates tion should not be abnormal in simple, acute prostatitis. Chronic
hyperplasia o the prostate gland, and removing this stimulus partial or complete urinary outlet obstruction may cause abnor-
results in decreased prostate volume. However, removal o n- mal renal unction but not acute prostatitis. Abnormal serum BUN
asteride allows hyperplasia to continue, and thus answer “A” is and/or creatinine in the setting o prostatitis should prompt ur-
incorrect. “B” is also incorrect because nasteride or hirsutism ther investigation. T e urine o en shows bacteriuria, pyuria, and
in women is not approved by the FDA. Additionally, nasteride hematuria. However, the urine may also be negative. Urine should
is category X in pregnancy, with potential teratogenic e ects be sent or culture and sensitivity to de nitively determine the
on the etus. “D” is incorrect because nasteride takes time to pathogen and direct urther treatment. “E” is true: the PSA is o en
work—a lot o time. As previously mentioned, its peak e ective- elevated in prostatitis. However, it is not necessary nor is it recom-
ness is not seen or 3 to 6 months a er starting the medication. mended to obtain a PSA to diagnose prostatitis. When the PSA is
elevated due to acute prostatitis, it may not return to normal levels
or 1 month or more a er the resolution o in ammation.
HELPFUL TIP:
I compariso ria s wi h a pha-b ock rs, 5-a pha-r -
duc as i hibi ors hav show variab r su s. t h HELPFUL TIP:
addi io o a 5-a pha-r duc as i hibi or o a a pha- Ob ai i g uri is impor a i h diag osis o pros a-
b ock r do s o s m o hav addi io a b i ov r i is, bu you shou d avoid b add r ca h riza io du
a pha-b ock r h rapy a o i h near term, bu com- o h po ia o spr ad i c io . B sid s, you wa
bi a io h rapy has show r duc d i cid c o uri ha has b i co ac wi h h pros a .
c i ica progr ssio o BPH i longer trials.

On examination, you nd an uncom ortable appearing male


You decide to add nasteride. You see him again 2 months in no distress. His temperature is 38.4°C, and the rest o his
later when he presents with a ebrile illness. He thinks that he vital signs are normal. T e prostate on digital rectal exami-
might have the u, but his BPH symptoms worsened at the nation is tender, enlarged, warm, and boggy. T e remainder
same time. For the last 2 days, he has elt everish with back o the examination is unremarkable. Urinalysis is consistent
pain, perineal pain, and generalized malaise. He complains with an in ection. He has a sul a allergy.
o dysuria and worsening urinary requency and urgency.
Question 16.1.9 Which o the ollowing is the most appro-
Question 16.1.7 During your examination, you make sure priate treatment plan or this patient?
NOT to: A) Prescribe amoxicillin 500 mg orally ID or 10 days
A) Per orm a rectal examination B) Prescribe cipro oxacin 500 mg orally BID or 28 days
B) Massage the prostate C) Admit or IV levo oxacin 500 mg daily or 14 days
C) Swab the urethra or chlamydia D) Admit or IV levo oxacin 500 mg daily, ollowed by com-
D) Per orm urinalysis and microscopic examination o the pletion o therapy with oral levo oxacin 500 mg daily or
urinary sediment 14 days when the patient is stable
E) Per orm transrectal ultrasound to rule out prostatic abscess
Answer 16.1.7 The correct answer is “B.” T ere is a risk o
seeding bacteria into the bloodstream when an in ected pros- Answer 16.1.9 The correct answer is “B.” T e most appropriate
tate is massaged. T is patient has symptoms o prostatitis; thus, treatment or this patient is a uoroquinolone, such as cipro oxa-
you should avoid prostatic massage. Nonetheless, you should cin, or at least 28 days. Some authorities recommend longer treat-
per orm a prostate examination. T e ollowing physical ndings ment (up to 6 weeks) to reduce the risk o chronic prostatitis. In
are associated with prostatitis: tenderness, warmth, enlargement, patients who are not allergic, a sul a antibiotic could be considered
and bogginess. as an alternative to a uoroquinolone. In this case, “C” and “D”
are overkill. Admission is appropriate or patients who appear sep-
You suspect prostatitis and obtain urine or analysis. tic, have not responded to oral antibiotics, or who have signi cant
comorbidities. However, uoroquinolones have 100% bioavailabil-
Question 16.1.8 All o the ollowing laboratory abnormali- ity PO. T us, there is no indication or giving these drugs IV unless
ties are consistent with the diagnosis o acute prostatitis the oral route is unavailable (e.g., vomiting). Additionally, the treat-
EXCEPT: ment course or “C” and “D” is too short. “E” is incorrect because
A) Leukocytosis abscesses are rare and imaging or an abscess is only undertaken i
B) Hematuria the patient does not respond to appropriate antibiotics.
CHAPTER 16 • Men ’S HeAl t H 457

C) Obtain urine culture and i positive, treat based on sensitivi-


HELPFUL TIP:
ties or at least 4 weeks
t h mos commo caus o acu pros a i is is Escherichia
D) Obtain a pelvic MRI
coli (58–88% o m ).
Answer 16.1.11 The correct answer is “D.” T ere is no indica-
tion or a pelvic MRI at this time. T e 2 glass pre- and post-
When you see this patient again, his symptoms o prostatitis prostatic massage test is a very use ul method o diagnosing
have cleared, but he does not think that nasteride is really chronic prostatitis. A mid-stream voided urine specimen is
helping. His AUA symptoms score is 21 (severe). He is won- collected and sent or culture. I the urinalysis has greater
dering i a transurethral re-section o the prostate ( URP) than 10 WBCs per high-power eld, next prostatic massage is
might help him, and he wants to discuss the downsides o the per ormed, and then the rst 10 mL o voided urine a er the
operation. massage that should include expressed prostatic secretions
should be sent or culture. reatment is based on the culture
Question 16.1.10 Compared with watch ul waiting, all o and sensitivity results. A 4- to 6-week course o treatment is
the ollowing are observed at greater rates in men who recommended with an appropriate antibiotic with good tissue
undergo TURP EXCEPT: penetration. T e most common organisms isolated in chronic
A) Erectile dys unction bacterial prostatitis are E. coli, Klebsiella, Proteus, Pseudomonas,
B) Urinary incontinence and Gram-positive Enterococcus. T e NIH Chronic Prostatitis
C) Urethral stenosis Symptom Index questionnaire is a reliable, valid method to
D) Increased urine ow assess symptoms and quality o li e impact in men with chronic
E) Decreased post-void residual urine volume prostatitis (http://www.prostatitis.org/symptomindex.html).

Answer 16.1.10 The correct answer is “A.” URP is a com- Objectives: Did you learn to . . .
• R cog iz h pa r o voidi g dys u c io s i BPH?
monly per ormed procedure or BPH. Indications or URP
include ailure o medical therapy, recurrent in ections, bladder • Ma ag a pa i wi h BPH a d u d rs a d h po ia
adv rs c s o m dica io s us d o r a BPH?
calculi, renal insu ciency, and patient pre erence. Patients who
undergo URP typically experience decreased AUA symptom • Diag os a d r a acu pros a i is?
scores, increased urine ow rates, and decreased post-void • D scrib i dica io s or a d comp ica io s o t URP?
residual volumes. T ere are downsides to URP, including • eva ua chro ic pros a i is?
urinary incontinence, urethral stenosis, and the need to repeat
the surgery. Strange as it may seem, several studies have shown
that erectile dys unction does NO occur at increased rates in QUICKQUIZ: SeXUAl l Yt RAn SMIt t eD In FeCt IOn S
patients undergoing URP compared with watch ul waiting.
However, men can have retrograde ejaculation status post- A 21-year-old college student, sel -described as a “ladies’ man,”
URP. (interpret: jerk) presents because o a concerning spot that
developed on his penis. He complains o pain at the spot but
denies itching. He reports no ever. When asked urther about
HELPFUL TIP: his sexual practices, he reports no condom use because his
Dai y ow-dos Cia is ( a ada i ) has b approv d or partners are all “on the pill.” He had chlamydia in high school
r a i g BPH. Bu , h b i ov r p ac bo is o y 2.3 but is otherwise healthy. His review o systems is negative. On
poi s o a 35-poi sca . . . do ’ xp c mirac s. examination o the penis, you nd a 1-cm tender, erythematous
papule with a deep central ulceration at the glans penis. T ere is
some mild, tender lymphadenopathy in the inguinal area. T e
Your patient is so happy with his care that he shared his story rest o the examination is unremarkable.
over beers, and his riend comes to see you. T is patient is a
50-year-old male, in no apparent distress, who presents with T is lesion is most likely caused by:
a 6-month history o recurrent irritative voiding symptoms A) Haemophilus ducreyi
( requency, urgency, etc.), low back and distal penile pain, and B) Neisseria gonorrhoeae
recurrent U Is with the same organism. oday he is a ebrile C) Staphylococcus aureus
with a mildly tender prostate on digital rectal examination. D) Treponema pallidum

Question 16.1.11 Based on this patient’s history and physi- The correct answer is “A.” T is is the lesion o H. ducreyi, oth-
cal examination, all would be the most appropriate at this erwise known as chancroid. It can be con used with the chan-
stage, EXCEPT: cre o primary syphilis, caused by T. pallidum, but the syphilis
A) Have the patient complete the NIH Chronic Prostatitis chancre is painless. Gram stain (Gram-negative rods in chains),
Symptom Index questionnaire culture, or biopsy (which this guy deserves) may con rm the
B) Per orm a 2 glass pre- and post-prostatic massage test diagnosis. A ew more notes: chancroid is rarely diagnosed in
458 FAMIl Y MeDICIn e eXAMIn At IOn & BOARD ReVIeW

the United States and is probably under diagnosed; it requently 18 months o age and pre erably by the end o the rst year. T e
co-in ects with syphilis and tends to occur in clusters. A num- consequence o not treating an undescended testicle is a 20% to
ber o treatments are available including ce riaxone (250 mg 40% increased risk o developing a testicular malignancy, which
IM once), azithromycin (1 g PO once), cipro oxacin (500 mg o en presents as a painless mass (although it is not clear that
PO BID or 3 days), and erythromycin base (500 mg PO ID × xing the undescended testicle lowers the cancer risk). T ere-
7 days). ore, the reasons to treat cryptorchidism are (1) to better palpate
the testicle to assess or potential malignant trans ormation;
CASE 16.2 (2) to decrease the risk o malignant trans ormation (maybe);
(3) to improve chances o ertility; (4) to decrease risk o testicu-
A 22-year-old male presents complaining o a painless lump lar torsion; (5) to decrease psychological e ects rom having an
on his le testicle. He denies penile discharge, dysuria, or empty scrotum; and (6) to repair an inguinal hernia at the same
other urinary complaints. He underwent a le orchidopexy time, i it is present.
or an undescended testicle at age 6. Otherwise, his past med-
ical history is unremarkable. On examination, the penis is
circumcised with no lesion or discharge. T ere is adenopathy HELPFUL TIP:
in the le inguinal area. His testicles are descended bilater- I pa i s wi h hypospadias a d u i a ra or bi a ra
ally with a 1-cm palpable, irregular mass on the mid-lateral mp y scro um, h r is a high r ra o chromosoma
portion o the le testicle. His examination is otherwise unre- a oma i s.
markable. Your patient is worried about testicular cancer and
wants to know i he is at risk.
Question 16.2.3 A ter an appropriate history and physical
Question 16.2.1 All o the ollowing are associated with an examination, which o the ollowing tests is the initial diag-
increased risk o testicular cancer EXCEPT: nostic study o choice in your patient with a scrotal mass?
A) Vasectomy A) C scan
B) HIV in ection B) Ultrasound
C) Cryptorchidism C) Complete blood count (CBC)
D) Kline elter syndrome D) Alpha etoprotein (AFP)
E) Family history E) Pelvic x-ray

Answer 16.2.1 The correct answer is “A.” Epidemiologic data Answer 16.2.3 The correct answer is “B.” T e best initial diag-
do not support an association between testicular cancer and nostic test would be a scrotal ultrasound to determine i this
vasectomy. As is true o some other malignancies, males with mass is cystic or solid. I it is determined to be a solid mass
HIV in ection have an increased risk o testicular cancer. Also, suspicious or malignancy, then other diagnostic studies would
males with cryptorchidism ( ailure o one or both testicles to be warranted, such as β -hCG and AFP. In conjunction with a
descend into the scrotum) and Kline elter syndrome are at radical inguinal orchiectomy or testicular cancer, a C scan
increased risk o testicular cancer. About 25% o testicular can- would be indicated to evaluate or metastatic disease, but it is
cers occurring in patients with cryptorchidism arise in the con- not the test o choice or initial diagnosis, so “A” is incorrect.
tralateral (normally descended) testicle. While testicular cancer CBC (“C”) would have a role in suspected in ection. X-ray
does not have as strong o a hereditary component, a positive (“E”) has no role in the evaluation o this patient since this is a
amily history is a risk actor or testicular cancer. O note, black so tissue mass.
males have a much lower incidence o testicular cancer than do
white males. HELPFUL TIP:
I h u rasou d o a scro a mass is quivoca , MRI or
Question 16.2.2 I your patient had not had an orchidopexy uro ogica r rra shou d b co sid r d x .
to repair the undescended testicle, he would be at risk or
developing all o the ollowing problems EXCEPT:
A) In ertility Question 16.2.4 In taking this patient’s history, i he had
B) Inguinal hernia described a painful lump in his scrotum, the LEAST likely
C) esticular torsion cause would be:
D) esticular malignancy A) Chlamydia
E) Impotence B) Inguinal hernia
C) Hydrocele
Answer 16.2.2 The correct answer is “E.” Organic impotence is D) esticular torsion
not a consequence o cryptorchidism. All o the other problems
listed occur at an increased requency in males with an unde- Answer 16.2.4 The correct answer is “C.” Spermatoceles and
scended testicle. T e best time to begin treatment or an unde- hydroceles are usually not pain ul. When a young, sexually
scended testicle to minimize uture sequelae is between 6 and active male presents with a pain ul scrotal mass or swelling,
CHAPTER 16 • Men ’S HeAl t H 459

epididymitis rom chlamydia or gonorrhea should be considered. he has masturbated twice (“just to check”) and has not seen any
esticular torsion o en has a very abrupt onset o pain and is more blood. He denies pain, hematuria, dysuria, evers, night
a surgical emergency. Inguinal hernias can be intermittently sweats, and weight loss. He reports that he is otherwise healthy.
pain ul i moving reely in the inguinal canal. However, i one His examination, including genitourinary and rectal examination,
becomes incarcerated, intense pain occurs. O note, varicoceles is normal. A urinalysis is negative.
are usually an incidental nding; however, large varicoceles may
occasionally be pain ul. Likewise, testicular cancers are usually What is your next step in the evaluation and management o
not pain ul but can become so i the tumor growth is rapid. this patient?
A) Reassurance and ollow-up
Your patient’s ultrasound is concerning or testicular cancer, B) Scrotal ultrasound
and you re er him to a urologist. C) Pelvic C scan
T e next day the patient’s 15-year-old brother presents D) ransrectal ultrasound
with scrotal pain. You wonder i you are on the cusp o dis- E) PSA
covering an in ectious cause o testicular cancer, but alas, it
appears to be coincidence. The correct answer is “A.” Hematospermia, the name given
T is patient’s pain is on the right and he can localize it well to bloody penile ejaculate, is airly uncommon. It can occur
to the ront o the testicle. It has been present or 3 days and in men o any age and is perhaps most common a er pros-
seemed to occur gradually over a ew hours. T ere is no radi- tate biopsy or prostate surgery. In otherwise healthy young
ation o the pain. Running makes it worse, and cool packs men, the cause is most o en idiopathic and is almost always
seem to help. Yesterday he noticed a slight swelling o the benign. History should ocus on traumatic causes (e.g., long
scrotum on the same side. He denies trauma to the area, any bicycle ride), symptoms o prostate disease, and symptoms o
history o sexual activity, other genitourinary complaints, in ection. A genital and prostate examination should be per-
ever, nausea, or vomiting. On examination, you nd nor- ormed. Urinalysis is help ul to exclude in ection. Consider
mal vitals. He has a well-localized tender spot at the anterior gonorrhea and chlamydia cultures in the appropriate patients,
superior right scrotum with a bluish discoloration under the or example, those with symptoms o urethritis. In this patient,
skin . . . as i it were some sort o sign . . . a blue dot sign. reassurance is adequate. Persistent hematospermia, with
symptoms present or over a month, may require urther eval-
Question 16.2.5 Which o the ollowing is the most likely uation with PSA, imaging, and urologic consultation. Some
diagnosis? experts recommend a more aggressive work-up, especially in
A) orsion o the appendix testis older males, but there is no evidence to support more exten-
B) orsion o the testicle sive evaluations. Also ask about recent travel: schistosomiasis
C) Varicocele is a requent cause o hematospermia and hematuria in other
D) Abscess parts o the world. In most cases o hematospermia, semen
E) Spermatocele analysis is not warranted, but it can be diagnostic in patients
with schistosomiasis.
Answer 16.2.5 The correct answer is “A.” T is is the classic
presentation o a torsed appendix testis. T e appendix testis is CASE 16.3
a pedunculated, vestigial structure at the anterior superior tes-
ticle. orsion o the appendix testis is one o the most common A couple you have known or a ew years comes to your o ce
causes o scrotal pain in children. T e pain is usually well local- to announce that they are expecting and that they want you
ized. T ere may be a reactive hydrocele. Diagnosis is con rmed to be the baby’s doctor (strange that they didn’t ask you to
by ultrasound. Unlike a torsed testicle, a torsed appendix testis be the mother’s doctor, but you let it slide). According to an
is not an emergency. It may be treated by conservative therapy ultrasound, the etus is male. T e couple is ambivalent about
(rest, NSAIDs, ice) or surgical excision. neonatal circumcision and wants your advice.

Objectives: Did you learn to . . . Question 16.3.1 You start the conversation by saying:
• R cog iz po ia caus s o a pai ss scro a mass? A) “Circumcision is a relic o history and should be illegal.”
• eva ua a s icu ar mass? B) “All major medical organizations (e.g., AAFP, AAP, AMA)
• R cog iz h sig i ca c o cryp orchidism? recommend routine neonatal circumcision.”
• Id i y orsio o h app dix s is? C) “T e decision to per orm circumcision is a personal one,
in uenced by a number o actors—but primarily by cultural,
religious, and amilial issues.”
QUICK QUIZ: HeMAt OSPeRMIA D) “Do whatever you want. I don’t really care what you do with
a tiny piece o skin”
A 28-year-old male presents to your o ce looking quite con-
cerned. Several days ago a er sexual intercourse with his girl- Answer 16.3.1 The correct answer is “C.” One o the strongest
riend, he noticed bloody ejaculate in the condom. Since then predictors o whether a newborn in the United States will be
460 FAMIl Y MeDICIn e eXAMIn At IOn & BOARD ReVIeW

circumcised is the circumcision status o the ather. T ere are decreased sexual satis action or most men later in li e. “A”
other reasons cited by parents as well (discussed later in the is true. T ere is a small but real risk o hemorrhage with the
case). “A” is incorrect simply because the statement is e used procedure, and this occurs mostly in patients who have an
with emotion and lacks logic. “B” is incorrect. T e AAFP has no unknown or unrealized coagulopathy. In act, abnormal bleed-
policy statement on circumcision. In 2012 the American Academy ing a er circumcision is a common way in which sporadic
o Pediatrics changed its statement on circumcision (since 1999 cases o hemophilia are discovered (including one case discov-
the AAP did not nd the data su cient to recommend routine ered this way by one o the editors—and he still eels bad!).
neonatal circumcision) to the ollowing: the health benef ts out- “C” is true. Several studies have demonstrated that circumcised
weigh the risks, and circumcision should be made available to all men are less likely to have HPV in ection. T e data or the risk
amilies who choose it. However, ‘routine’ universal circumcision o other sexually transmitted in ections (S Is) are somewhat
is still not recommended. I you chose “D,” you need to work on contradictory, but generally avor a reduced risk o S Is in cir-
your bedside manner! cumcised males. “D” is true. T ere is evidence that circumci-
sion may decrease the transmission o HPV and penile cancer.
Question 16.3.2 Consequences o circumcision include Retrospective studies have shown that the rate o squamous cell
which o the ollowing? carcinoma o the penis is about three old higher in uncircum-
A) Overall reduction in mortality o circumcised in ants com- cised men, at least in the United States. However, penile cancer
pared with uncircumcised in ants is exceedingly rare (about 0.6 cases/100,000) and the number
B) Reduction in U Is in the rst year o li e o circumcisions needed to prevent 1 case o penile cancer is
C) Reduction in the number o sexual partners about 300,000. However, the American Cancer Society does
D) All o the above not recommend routine neonatal circumcision or the preven-
tion o penile cancer but does recommend that all risk actors
Answer 16.3.2 The correct answer is “B.” T e rate o U Is in be addressed.
uncircumcised males in the rst year o li e is about 10 times
greater than the rate or circumcised males. In order to pre-
HELPFUL TIP:
vent one U I in the rst year o li e, about 100 circumcisions
I is abu da y c ar ha circumcisio r duc s h
need to be per ormed. T e e ect o circumcision on rates o
ra smissio o HIV. t his has d o a ca or rou i
U I later in li e is not well studied. T e rate o U I may be
circumcisio i high-risk popu a io s ( .g., som par s
higher shortly a er circumcision (within the rst 2 weeks). “A”
o sub-Sahara A rica, c.).
is incorrect because there is no data showing any di erence in
mortality between circumcised and uncircumcised in ants. “C”
is incorrect; some surveys have shown increased requency and
variety o sexual practices in circumcised males compared with HELPFUL TIP:
uncircumcised (note that this is an association, not a causal I addi io o u circumcis d s a us, risk ac ors associ-
relationship). a d wi h p i ca c r i c ud smoki g, risky s xua
b havior, poor hygi , a d g i a war s.

HELPFUL TIP:
Circumcisio r duc s h ra o c rvica ca c r i You have a nice, long conversation with the couple, discuss-
h ma par rs o ma s who w r circumcis d ing the potential bene ts and risks, and they decide to have
a d pr vious y had six or mor s xua par rs. t his is their son circumcised. A er the birth, you are per orming
hough du o d cr as d ra o HPV ra smissio . O a thorough examination when you nd something slightly
cours , i is ow r comm d d o provid vacci a io abnormal with the penis.
o a you g m a d wom wi h h HPV vacci a io
s ri s. S h Chap rs 13 a d 14 or mor i orma io . Question 16.3.4 Neonatal circumcision is indicated or:
A) Congenital phimosis
B) Micropenis
Question 16.3.3 Neonatal circumcision is associated with C) Hypospadias
all o the ollowing EXCEPT: D) Ambiguous genitalia
A) Risk o hemorrhage with the procedure
B) Psychological trauma and decreased sexual satis action later Answer 16.3.4 The correct answer is “A.” Signi cant congeni-
in li e tal phimosis rarely occurs and is an indication or circumcision.
C) Reduced risk o in ection with S Is, such as HIV Phimosis is de ned as the inability to retract the oreskin (pre-
D) Reduced risk o penile cancer puce) over the glans. T is is a normal nding in uncircumcised
in ants. However, signi cant congenital phimosis may com-
Answer 16.3.3 The correct answer is “B.” Despite what some pletely cover the urethra and not allow the normal passage o
anti-circumcision websites maintain, circumcision does not urine. “B,” “C,” and “D” are reasons to avoid routine circumcision
appear to result in any signi cant psychological trauma or and to call on the aid o a urologist.
CHAPTER 16 • Men ’S HeAl t H 461

rom psychological e ects o vasectomy, the procedure should


HELPFUL TIP:
not directly a ect sexual unction, orgasm, or ejaculation.
Wh p r ormi g circumcisio , oca a s h ic is r c-
Patients should be aware that vasectomy results in permanent
omm d d, a d i h r dorsa p i rv b ock or
sterility and that reversal procedures are only success ul about
subcu a ous ri g b ock is pr rr d. O h r pai co ro
hal o the time. Potential complications include ailure and
moda i i s may b us d as w , i c udi g pr -proc dur
unwanted pregnancy, in ection, pain, bleeding, hematoma, etc.
ac ami oph , sucros coa d paci i r, ca mi g music,
swadd i g, c.
Your patient decides to “get snipped” as he puts it. You per-
orm the vasectomy using a no-scalpel technique. T e pro-
Objectives: Did you learn to . . . cedure was airly easy, and the patient tolerated it well. One
• Discuss h b s a d risks o circumcisio wi h par s? month a er the procedure, the patient calls to complain
• Id i y i dica io s a d co rai dica io s or o a a about a pain ul swelling that has developed superior and
circumcisio ? slightly posterior to the le testis. He has no other symptoms.
• emp oy appropria pai co ro m asur s or h proc dur
o circumcisio ? Question 16.4.3 Which o the ollowing is the most likely
diagnosis?
CASE 16.4 A) Hematoma
B) Varicocele
A 32-year-old male presents to discuss permanent sterilization. C) Congestive epididymitis
He clearly states that he wants a vasectomy, and sooner is better D) Abscess
than later. He is married and has three children at home. His
wi e just gave birth to twins. He is healthy and takes no medica- Answer 16.4.3 The correct answer is “C.” Without any urther
tions. He looks tired and anxious. You examine him and nd in ormation, congestive epididymitis is the most likely cause o
no abnormalities. T e vas de erens is easily isolated bilaterally. this patient’s current complaint. Congestive epididymitis occurs
in about 3% o patients post-vasectomy, and the onset is usually
Question 16.4.1 What is your next step? within weeks to months a er the procedure. “A,” hematoma, can
A) Re er him or psychological counseling as he is clearly under be avoided in most cases i hemostasis is achieved during the pro-
a great deal o stress cedure and the patient does not overexert himsel immediately
B) Provide him with detailed counseling on vasectomy, give him post-operation. A hematoma is more likely to develop early rather
written material on the procedure, and ask him to discuss it than a month later. Likewise, an in ection (“D”) would be unlikely
with his wi e so ar out rom the procedure. “B” is incorrect because varicoceles
C) Do the vasectomy right now; the next patient can wait do not develop a er vasectomy (unless you are operating ar away
D) ell him that he is not an appropriate candidate or vasectomy rom where you are supposed to be). Overall, the most common
scrotal pathology a er vasectomy is sperm granuloma, which
Answer 16.4.1 The correct answer is “B.” Patient education, occurs in up to 40% o patients but is generally asymptomatic.
counseling, and selection are very important aspects o vasec-
tomy. When you counsel him on vasectomy, you should explore
his reasons or wanting the procedure. You should not schedule HELPFUL TIP:
a vasectomy without providing counseling and assuring that th r a m o co g s iv pididymi is ca b rus-
he understands the procedure in detail. T is patient appears to ra i g. Mos m who hav a vas c omy wi hav som
be atigued—an issue that should be explored urther . . . and he m o co g s iv pididymi is, bu mos wi hav
has newborn twins at home. Who the heck wouldn’t be tired! mi or sw i g wi hou pai . For pa i s wi h pai u
So, immediate re erral or psychological counseling without co g s iv pididymi is, a ria o n SAIDs a d si z ba hs
investigating the underlying cause is not appropriate. or s v ra mo hs is i dica d. Fai i g co s rva iv
h rapy, s roid i j c io or surg ry may b i dica d.
Question 16.4.2 While counseling this patient, you discuss
which o the ollowing issues?
A) Partner’s desire or permanent sterility wo months a er the vasectomy, your patient returns with a
B) E ect o the procedure on sexual unction semen sample, showing no sperm. His surgery was success ul,
C) Reversibility o the procedure but ailures do occasionally occur.
D) Complications
E) All o the above Question 16.4.4 Vasectomy ailure is usually due to:
A) Failure to identi y and transect the vas at the time o surgery
Answer 16.4.2 The correct answer is “E.” All these issues are B) Recanalization
important to discuss prior to scheduling the procedure. It is C) Le -handed doctors
important that the couple agree on this procedure because it D) In ection
is the couple—not just your patient—who will be sterile. Aside E) Immaculate conception
462 FAMIl Y MeDICIn e eXAMIn At IOn & BOARD ReVIeW

Answer 16.4.4 The correct answer is “B.” Although “redundant Question 16.4.6 Regarding androgenetic alopecia, which
systems” (removing a segment o the vas de erens, clipping or o the ollowing statements is true?
suturing the ree ends, cauterizing the transected vas, suturing A) Minoxidil must be used or at least 2 years to achieve perma-
ascia around one ree end while leaving the other outside the nent hair regrowth.
ascia) are employed to avoid this complication, recanalization B) Lower doses o 5-α -reductase inhibitors (e.g., nasteride) are
can occur. A new pathway can orm between the ree ends o used in treating androgenetic alopecia compared with BPH.
the transected vas de erens, allowing sperm into the ejaculated C) Hair transplant should be avoided unless all other therapeutic
semen. T ere ore, all patients should return post-vasectomy or attempts have ailed.
semen analysis. “A” and “D” are potential, but in requent, causes D) opical steroids are an e ective therapy or androgenetic
o ailure. “C” is incorrect. What have you got against le ies? As alopecia.
to “E,” only God knows. E) A toupee looks great on anyone.

Your patient is ultimately pleased with his vasectomy results. Answer 16.4.6 The correct answer is “B.” Finasteride (Prope-
He returns to see you several years later because o concerns cia) is FDA approved or androgenetic alopecia, and the dose is
that he is balding. On examination, you nd non-scarring 1 mg/day rather than the 5 mg/day dose used to treat BPH symp-
hair loss at the vertex. T e scalp appears normal otherwise. toms. Minoxidil applied to the scalp is also an e ective therapy
or androgenetic alopecia. Both minoxidil and nasteride must
Question 16.4.5 You should entertain all o the ollowing be continued inde nitely to be e ective. Hair transplant is a via-
diagnoses EXCEPT: ble option or men and women with androgenetic alopecia, and
A) Androgenetic alopecia (yes, it is “androgenetic” and not it is sometimes used as a rst-line therapy. T us, “C” is incorrect.
“androgenic”) opical steroids are not e ective or this type o hair loss. “E” is
B) elogen ef uvium clearly wrong; there are a lot o bad toupees out there.
C) Alopecia areata
Objectives: Did you learn to . . .
D) Hypothyroidism
• Provid pr -vas c omy cou s i g?
E) inea capitis
• R cog iz comp ica io s o vas c omy?
Answer 16.4.5 The correct answer is “E.” Hair loss is common • Id i y caus s o hair oss i m ?
in males, a ecting up to two-thirds o all men. Alopecia is o en • D scrib curr r am op io s or a drog ic a op cia?
divided into scarring and non-scarring orms. Most in ectious
causes o hair loss (e.g., tinea capitis and olliculitis) are scar-
ring i not treated, whereas the other causes listed (“A” through QUICK QUIZ: DeAt H RAt eS In Men
“D”) are non-scarring. Without any signs o in ammation or
hyperkeratosis, tinea capitis is unlikely. “A,” androgenetic alope- For all the ollowing causes o death, the age-adjusted death rate
cia, is quite common in adult males (male balding). “B,” telogen is higher or males than or emales EXCEP :
ef uvium, presents with di use hair loss usually secondary to A) Liver disease
metabolic (including dieting) or emotional stress. T is occurs as B) Alzheimer disease
an abnormal percentage o hair enters the telogen phase during C) Coronary artery disease
which they are shed. Grab some hair: i more than ve hairs in D) Suicide
telogen phase come out, you have your diagnosis. “C,” alopecia E) Cancer
areata, results in patchy hair loss in round or oval shapes. Meta-
bolic conditions should also be in the di erential diagnosis o The correct answer is “B.” When it comes to Alzheimer disease,
hair loss, and these might include hypothyroidism (“D”), hyper- men get a break; the death rate is higher or emales. All o the
thyroidism, and iron de ciency. other causes o death listed have greater death rates in males. For
example, compared with women with liver disease, same age men
with liver disease are twice as likely to die o their liver disease.
HELPFUL TIP:
S co dary syphi is caus s a o -i amma ory, o -
scarri g hair oss ha may b pa chy or di us . Co sid r QUICK QUIZ: MORe ABOUt DeAt H In Men
s i g or syphi is i appropria pa i s.
T e relative risk o death or males is greater throughout the li e
span. Compared with same age emales, at what age range is the
T e patient reports a strong amily history o baldness. On relative risk o death greatest or males?
your examination, the patient has thin hair at the vertex and A) < 1 year
recession o the hairline in an “M” shape at the rontotem- B) 5 to 14 years
poral area. On the basis o the history and examination, you C) 15 to 24 years
diagnose androgenetic alopecia. He would like to do some- D) 25 to 34 years
thing about his hair loss. E) > 85 years
CHAPTER 16 • Men ’S HeAl t H 463

The correct answer is “C.” Un ortunately, males aged 15 to D) Hypothyroidism


24 years have a relative risk o death o greater than 2.5 com- E) Phenothiazines
pared with same age emales (usually preceded by a cry o “Hey,
look what I can do!”). Mother Nature has a way o making up Answer 16.5.3 The correct answer is “D.” Hyperthyroid-
or boys and young men dying: more male etuses are conceived ism, not hypothyroidism, can cause gynecomastia through
than emale. However, the miscarriage rate is also greater or increased aromatization o testosterone to estradiol and andro-
male etuses. Nonetheless, about 105 males are born in the stenedione to estrone. A number o drugs are associated with
United States or every 100 emales. By age 35, enough males gynecomastia, including marijuana, alcohol, 5-alpha-reductase
have died o that the number o males and emales at that age is inhibitors, phenothiazines, tricyclic antidepressants, andro-
about equal, and therea er the number o emales exceeds the gens, estrogens, growth hormone, calcium channel blockers,
number o males (in other words, the singles scene improves and spironolactone. Other causes o gynecomastia include:
markedly or the remaining males). hypogonadism, hyperprolactinemia, testicular tumors (some
secrete estrogens), renal ailure, and liver diseases (cirrhosis).
CASE 16.5 Obesity is associated with gynecomastia, but obese patients
may also appear to have gynecomastia while simply having
A 14-year-old male presents with his mother, who is worried excess at deposition in the area o the breast (sometimes called
that he has growing breasts. Over the last 2 or 3 months, the “pseudogynecomastia”).
patient has developed swellings beneath both nipples. He
denies discharge or pain, but the nipples are tender at times. A thorough drug history is negative. On physical examina-
tion, you nd palpable, nontender tissue beneath the nipples,
Question 16.5.1 You can tell this patient that physiologic with slightly more prominent tissue mass on the right. T e
gynecomastia (subareolar breast tissue) is a condition that tissue is about 2 to 3 cm in diameter, and no discrete masses
a ects: are palpable. T ere is no nipple discharge. An adult male’s
A) Newborns hair growth pattern is evident in the axillary and inguinal
B) Adolescents areas. T e testicles are normal size without masses.
C) Elderly males
D) All o the above Question 16.5.4 At this point in time, you recommend
which o the ollowing?
Answer 16.5.1 The correct answer is “D.” T e incidence o A) Observation
physiologic gynecomastia is trimodal, with peaks in the neona- B) Re erral to a surgeon
tal period, adolescence, and old age. Data can be contradictory C) Limited laboratory studies, including thyroid-stimulating
at times, but clinically palpable breast tissue (either at or true hormone ( SH), testosterone, and liver enzymes
breast tissue) may be present in more than 50% o males in each D) Biopsy o the tissue
o these three age groups. E) Mammogram and/or ultrasound

Question 16.5.2 Which o the ollowing hormones is respon- Answer 16.5.4 The correct answer is “A.” As previously men-
sible or the proli eration o breast tissue? tioned, physiologic gynecomastia is quite common in adoles-
A) estosterone cent males. T e ndings on examination are reassuring. T is
B) Estrogen patient does not display any other signs o testosterone de -
C) Androstenedione ciency, and urther workup is not indicated at this time. I there
D) Growth hormone are no discrete masses on examination, a mammogram, ultra-
E) Progesterone sound, or biopsy is not likely to be help ul. Re erral to a surgeon
is premature, as 90% o these patients experience spontaneous
Answer 16.5.2 The correct answer is “B.” Estrogens induce involution o the breast tissue over 3 years.
ductal hyperplasia and growth o glandular tissue. estoster-
one and androstenedione inhibit the actions o estrogens on the
breast tissue. Some men with gynecomastia have increased sen- HELPFUL TIP:
sitivity o breast tissue to circulating estrogens. Others may have I ado sc ma s wi h gy comas ia, ur h r va ua-
an increased proportion o estrogens compared with androgens. io wi h abora ory s udi s a d imagi g is i dica d i
Yet, others may have a mixture o both processes or another h br as issu is rapid y argi g or is gr a r ha
process altogether. 5 cm i diam r, a mass ( o orma br as issu ) is
pa pab , or o h r sig s o u d r a drog iza io ar
Question 16.5.3 Potential causes o gynecomastia include pr s .
all o the ollowing EXCEPT:
A) Renal ailure
B) Marijuana use You provide the patient and his mother with reassurance and
C) esticular cancer have them return in a year. At ollow-up, there is no palpable
464 FAMIl Y MeDICIn e eXAMIn At IOn & BOARD ReVIeW

tissue. When you see the patient again, he is 17 years old. He


HELPFUL TIP:
told his mother that he was having abdominal pain, but really
exp di d par r r a m i which h pa i b -
he is worried that he may have contracted an S I. He has
i g r a d or a St I is a so giv m dica io or his/
become airly promiscuous and does not use condoms (a er
h r par r(s) is r comm d d by h C rs or Dis-
the gynecomastia disappeared, his mojo returned). In the last
as Co ro a d Pr v io (CDC) a d o h rs. S
week, he has developed dysuria and a yellowish urethral dis-
h p://www.cdc.gov/s d/ p / or mor i orma io a d
charge. He has no other symptoms.
h ga i y o h prac ic i your s a .
Question 16.5.5 Which o the ollowing is the most likely
diagnosis? Question 16.5.7 Which o the ollowing methods maintains
A) HPV a high degree o sensitivity and is also the quickest and
B) Syphilis least expensive way to diagnose gonococcal urethritis in a
C) Gonorrhea symptomatic male?
D) richomonas A) Culture
E) Herpes simplex B) Gram stain
C) Serologic antibody assay
Answer 16.5.5 The correct answer is “C.” T is patient’s symp- D) PCR
toms are typical o gonococcal urethritis. However, N. gonor- E) DNA probe
rhoeae may be present in the urethra without any symptoms
and Chlamydia trachomatis can present with a purulent Answer 16.5.7 The correct answer is “B.” In symptomatic
discharge that is classically thought o as gonorrhea; gonor- males, Gram stain o a urethral sample can identi y Gram-
rhea and chlamydia cannot be reliably distinguished on clini- negative diplococci (N. gonorrhoeae) with a sensitivity o about
cal grounds. “A” is incorrect as HPV causes genital warts, not 90% to 95%. T e sensitivity drops to about 70% in asymptom-
urethritis. “B,” syphilis, may present as a painless ulcer (pri- atic males. I the materials and expertise are readily available, a
mary syphilis). “D” is incorrect because most men in ected with Gram stain is quick and inexpensive. Culture o a urethral spec-
Trichomonas vaginalis are asymptomatic, although some will imen on T ayer–Martin agar takes longer and alse-negative
have mild urethritis. Finally, “E,” herpes simplex, presents with tests can occur with high requency (due to the need to have a
pain ul vesicles at the area o inoculation. CO2-rich environment and to keep the culture in a narrow tem-
perature range). PCR and other nucleic acid testing are more
You obtain urine or PCR or gonorrhea and chlamydia. You sensitive than cultures but still not 100%. PCR is particularly
discuss other S Is and decide to per orm some other tests attractive as a screening test, as it can be per ormed on urine
(e.g., HIV, hepatitis B, RPR/VDRL). specimens instead o urethral swab specimens. Serologic assays
or gonorrhea would not be help ul in diagnosing urethritis.
Question 16.5.6 At this point in time, you are compelled to
do all o the ollowing EXCEPT: Question 16.5.8 Your clinical suspicion o gonorrhea is high.
A) Recommend sa e sexual practices What is your next step?
B) In orm the public health services i he tests positive A) Single doses o ce riaxone 125 mg IM
C) reat him with antibiotics now B) A single dose o penicillin G 1.2 million units IM and eryth-
D) Encourage him to contact his partners and tell them to get romycin 500 mg orally QID or 7 days
tested C) etracycline 500 mg orally QID or 10 days
E) In orm his mother o your ndings D) Single doses o cipro oxacin 500 mg orally and ce xime
400 mg orally
Answer 16.5.6 The correct answer is “E.” In general, adoles- E) Single dose o ce riaxone 250 mg IM and azithromycin 1 g orally
cents can seek care or S Is and be treated without parental con-
sent. However, clinicians should re er to the laws o the state in Answer 16.5.8 The correct answer is “E.” I there is any concern
which they practice or ultimate legal authority in this matter. or compliance, it is best to treat the patient in the o ce i you have
O course, the time o diagnosis and treatment should be used access to the appropriate antibiotics. As o 2015, the CDC recom-
as an opportunity to educate the patient regarding sa e sexual mends ce riaxone 250 mg IM as a single dose (rather than 125
practices. “B” is true. Gonorrhea and chlamydia are reportable mg, due to increasing resistance) PLUS azithromycin 1 g PO or
diseases in every state. Again, clinicians should re er to the laws uncomplicated gonococcal urethritis. T e azithromycin is given
o their state and the reporting protocols o the clinic in which or two reasons: increasing resistance among gonococcal isolates
they practice. Empiric treatment is the rule here, so “C” is cor- and to cover or possible chlamydia co-in ection. I ce riaxone
rect. “D” is also correct since all o this patient’s partners should is not available, ce xime 400 mg PO is an acceptable alternative.
be contacted, tested, and treated. T ere are several ways to con- Fluoroquinolones are not recommended or gonorrhea due
tact the partners, and allowing the patient to do so is only one to high rates o resistance in some communities. “A” is incor-
way. T e clinician or the public health authorities could contact rect because ce riaxone alone will not provide empiric coverage
the partners as well. or chlamydia, and the dose is too low anyway. “B” is incorrect
CHAPTER 16 • Men ’S HeAl t H 465

because penicillin should never be used to treat gonorrhea (due to Objectives: Did you learn to . . .
the prevalence o penicillinase-producing strains o the bacteria). • D gy comas ia a d u d rs a d i s caus s?
“C” is incorrect. etracycline antibiotics provide adequate cover- • Id i y wh o i i ia ur h r va ua io i a pa i wi h
age or chlamydia and some strains o N. gonorrhoeae, but other gy comas ia?
strains o N. gonorrhoeae are resistant to tetracyclines, so these • R cog iz symp oms o go orrh a a d ch amydia?
drugs are not used as rst-line agents in treating gonorrhea. “D” • I i ia r a m i a ma wi h ur hri is?
is incorrect because uoroquinolones are not recommended or • R cog iz ma i s a io s o ch amydia i c io i m ?
gonorrhea and it contains nothing or the treatment o chlamydia.

HELPFUL TIP: QUICK QUIZ: An In Fl AMeD Gl An S


A o h r qua y c iv r gim or ch amydia is
doxycyc i 100 mg BID or 7 days. t his is qua i A 36-year-old male diabetic patient presents with a 3-day
icacy o azi hromyci r gard ss o h ac ha som history o irritation, itching, dysuria, and redness at the tip
dos s may b miss d. O o , doxycyc i has b i o his penis. He is monogamous with his wi e, and he denies
shor supp y p riodica y. any history o high-risk sexual behavior or S Is. On exami-
nation, you ind an a ebrile patient in no acute distress. he
penis is circumcised, and the glans penis is red, tender, and
You treat this patient with ce riaxone and azithromycin and edematous. here are numerous small, white papules on the
tell him to contact his partners so that they can get treated. glans.
When your patient returns to discuss his lab tests, he is eel-
ing much better. You tell him that he was in ected with both Which o the ollowing is the most appropriate treatment?
gonorrhea and chlamydia but that the rest o his tests were A) Sitz baths and improved hygiene
negative. He is relieved that he does not have HIV, but he B) Oral doxycycline
wonders what problems chlamydia can cause. C) opical bacitracin
D) opical miconazole
Question 16.5.9 C. trachomatis has been implicated in E) opical steroids
which o the ollowing?
A) Reiter syndrome The correct answer is “D.” T is patient has balanitis, which is
B) Lymphogranuloma venereum de ned as an in ammatory condition o the glans penis (bal-
C) Proctitis anoposthitis is the name applied to in ammation o the glans
D) Epididymitis and oreskin). Some authors believe that balanitis is a nonin-
E) All o the above ectious, in ammatory condition. Others implicate in ectious
causes. O in ectious causes, the most common is Candida
Answer 16.5.9 The correct answer is “E.” A small percentage albicans, especially in diabetic patients. his patient has clas-
o men with chlamydia urethritis develop reactive arthritis, and sic indings o candidal balanitis and is diabetic. here ore,
a subset o these will go on to have the triad o Reiter syndrome the most appropriate therapy is a topical anti ungal agent,
(urethritis, arthritis, and uveitis). Lymphogranuloma venereum such as miconazole. Fluconazole is an option. opical and
is also due to a particularly virulent strain o C. trachomatis, but oral antibiotics will not help, and topical steroids should
it produces genital ulcers and lymphadenitis and is generally be avoided. Sitz baths and improved hygiene should be
seen in tropical areas. Lymphogranuloma venereum is treated encouraged, but they should not be employed without an
with extended courses o doxycycline (100 mg PO BID or anti ungal agent.
21 days) or azithromycin (1 g PO weekly or 3 weeks). Chla-
mydia proctitis occurs almost exclusively in homosexual men CASE 16.6
and presents with rectal pain, bleeding, and discharge. Diagno-
sis is con rmed by rectal swab, and the treatment is the same While covering the emergency department (ED), a 40-year-
as or chlamydia urethritis. “D,” epididymitis is most o en the old male presents with a pain ul erection that began 4 hours
result o in ection with C. trachomatis or N. gonorrhoeae. Ure- ago “out o the blue” (perhaps a er he took a little “blue
thral strictures are also commonly related to prior S Is. pill”).

HELPFUL TIP: Question 16.6.1 Which o the ollowing is true regarding


t h majori y o ma s (a d ma s) who ar i c d priapism and normal erections?
wi h ch amydia a d/or go orrh a do not have symp- A) Normal erections can last up to 12 hours, so this is not
toms. t h r or , h r is a good argum o b mad priapism
or scr i g asymp oma ic ma s i high-risk popu a- B) Abnormally prolonged sexual desire can “convert” a normal
io s ( .g., ado sc s, you g adu s, a d hos wi h erection into priapism
mu ip s xua par rs). For ma s, s Chap r 15. C) T e corpus spongiosum and glans penis are not involved in
priapism
466 FAMIl Y MeDICIn e eXAMIn At IOn & BOARD ReVIeW

D) Acute urinary retention may lead to priapism and vice versa Question 16.6.3 What is your next step in management o
E) All o the above are true statements this patient?
A) Reassurance
Answer 16.6.1 The correct answer is “C.” Priapism is de ned B) Call or emergent urologic consultation
as the prolonged engorgement o the penis (or—rarely—the cli- C) Give oral alpha-blockers
toris in emales), unrelated to sexual desire or stimulation. T e D) Engage patient in guided imagery to lead his thoughts away
word priapism comes rom the Greek god Priapus, apparently rom sex
well known or his lasciviousness and generous genital endow-
ment. In most cases o priapism, the corpus spongiosum and Answer 16.6.3 The correct answer is “B.” Priapism is a uro-
glans penis are not involved. Only the corpora cavernosa are logic emergency and needs to be treated ASAP. All the other
engorged and rigid. Priapism typically lasts longer than 6 hours, answers could result in your undesired new title o “de endant.”
whereas normal erections last minutes to hours. With normal Until the urologist arrives, conservative measures are probably
erections, detumescence occurs a er ejaculation or a er the best, and you should try to make the patient com ortable using
stimulus is removed. T is is not the case with priapism, and sex- analgesics, oxygen, and hydration. Check a CBC since leukemia
ual desire does not play a role in the development o priapism; can rarely present with priapism. Some patients will respond to
thus, “B” is incorrect. “D” is also incorrect. Urinary retention is analgesics and ice packs. Oral or subcutaneous terbutaline may
not thought to cause priapism, and priapism does not lead to be help ul. Some authors recommend sedatives, such as benzo-
urinary retention. diazepines.

Question 16.6.2 Priapism may be secondary to all o the


ollowing EXCEPT: HELPFUL TIP:
A) Sickle cell disease I a uro ogis is o avai ab a d/or you ar com or -
B) Penile trauma ab p r ormi g h proc dur (a d you hav a good
C) Leukemia awy r), you ca a mp d um sc c . A d is
D) Iron de ciency anemia i s r d i o h corpora cav r osa a d b ood is wi h-
E) razodone use draw . t h a vasoac iv ag ( .g., ph y phri )
is i j c d. t his proc dur is r p a d v ry 5 mi u s
Answer 16.6.2 The correct answer is “D.” A number o di - u i d um sc c occurs. I shou d go wi hou sayi g
erent disease states and drugs have been implicated in the eti- ha h pa i ’s vi a sig s mus b c os y mo i or d
ology o priapism. In one way or another, these diseases and duri g his i rv io b caus h y wi ik y d
drugs a ect the balance o blood ow into the penis, leading s da io .
to increased arterial blood ow and/or decreased venous out-
ow. Local malignancies, such as bladder and prostate cancers,
can cause obstruction. Likewise, any condition that increases HELPFUL TIP:
blood viscosity (e.g., sickle cell disease, polycythemia, leuke- A r r cov ry, abou 50% o m wi h priapism su r
mia) or results in thromboembolic phenomena (e.g., vasculitis) r c i dys u c io . P i imp a s ar a op io .
can cause priapism. Penile trauma that results in laceration o
penile arteries can cause priapism. Numerous drugs have also
been implicated (see able 16-1), but in most cases, the cause is Objectives: Did you learn to . . .
not identi ed. • R cog iz priapism a d i s caus s?
• Ma ag a pa i wi h priapism?
TABLE 16-1 DRUGS ASSOCIATED WITH PRIAPISM
Psychotropics
• t razodo QUICK QUIZ: DePReSSIOn An D SUICIDe
• Ch orpromazi
Agents used to treat erectile dysfunction Which o the ollowing groups has the highest rate o death by
• I racav r osa i j c io s ( .g., papav ri ) suicide?
• Phosphodi s ras i hibi ors ( .g., si d a ) A) Black males
Antihypertensives B) White males
• Hydra azi C) Black emales
• Prazosi
D) White emales
Anticoagulants E) Hispanic males
• H pari
Drugs of abuse The correct answer is “B.” In the United States, white males
• A coho
make up about 70% o all deaths due to suicide. Although
• Cocai
• Marijua a emales account or more suicide attempts, males are at greater
risk o death rom suicide. Men tend to use deadlier means, such
CHAPTER 16 • Men ’S HeAl t H 467

as guns, hanging, and car collisions. In other words, men are


HELPFUL TIP:
very e cient at suicide. White men older than 85 years have one
I a ma pa i wi h os oporosis has s os ro
o the highest rates o death due to suicide: 59/100,000 (with the
d ici cy, s os ro supp m a io may b a
rate in the general public being 10.6/100,000). As a group, black
r aso ab r a m op io , bu h icacy o s os-
males have the second highest rate o suicide. O those groups
ro i his sc ario is not w s ab ish d. A hough
listed above, black emales have the lowest risk o death rom
s os ro co ribu s o p ak bo mass i you g
suicide.
m a d ow s os ro is associa d wi h a i -
cr as d rac ur risk i o d r m , s os ro or h
CASE 16.7 r am o os oporosis has b disappointing.
A 78-year-old male presents or ollow-up a er an emergency
department visit last week. He sustained a all that resulted
in a 11 compression racture. You obtained a DEXA scan to Question 16.7.2 Which o the ollowing is true regarding
con rm that he has osteoporosis and ound that his -score testosterone supplementation in older men with testoster-
is indeed osteoporotic-range. Just to be on the sa e side, you one def ciency?
order a bunch o laboratory tests, all o which are normal A) estosterone supplementation has markedly bene cial e ects
except or serum testosterone. Lo and behold, his testoster- on depression
one level is very low. He’s also been tired, elt weak, and lost B) T e e ects o testosterone supplementation in young hypo-
his libido. Because he knows we need to cover testosterone gonadal males and older testosterone-de cient males are the
supplementation in this book, he asks you, “Should I be on same
one o those ancy testosterone supplements?” C) In testosterone-de cient older males with erectile dys unc-
tion, testosterone supplementation dramatically improves
Question 16.7.1 Which o the ollowing testosterone sup- erections
plementation products would you AVOID? D) In testosterone-de cient older males with poor libido, tes-
A) ransdermal testosterone patches tosterone supplementation improves libido
B) Buccal testosterone tablets E) estosterone supplementation has no e ect on lean muscle
C) Oral testosterone tablets mass or grip strength
D) Intramuscular testosterone injections
E) ransdermal testosterone gel Answer 16.7.2 The correct answer is “D.” In testosterone-
de cient older males, testosterone supplementation does
Answer 16.7.1 The correct answer is “C.” Oral testosterone improve libido. Un ortunately, “C” is incorrect: its e ects on
should be avoided due to signi cant rst-pass metabolism erectile dys unction are not impressive. Even in testosterone-
and potential liver toxicity. T e other choices listed are viable de cient men with erectile dys unction, a phosphodiesterase
options or testosterone replacement (see able 16-2). T e IM inhibitor (e.g., sildena l) is more likely to be success ul. In terms
option is not the best or a steady state. Patients o en like it due o the neuropsychiatric e ects o testosterone, many questions
to the peaks. are as yet unanswered. However, testosterone supplementation

TABLE 16-2 TESTOSTERONE SUPPLEMENTATION PRODUCTS


Agents Dosages Comments
Bucca ab (S ria ) Dos d 30 mg BID a d mus b t h produc do s o disso v comp y
k p b sid bucca mucosa

t ra sd rma g s (A drog , For s a, Dos d dai y Cov r h app ica io ar a o r duc risk o spr adi g by co ac
t s im) wi h o h rs

t ra sd rma pa ch (A drod rm) App i d dai y Ro a si s o r duc ski irri a io

Scro a pa ch s (t s od rm) n ow rar y us d du o scro a irri a io a d mor acc p ab


a r a iv s

I ramuscu ar i j c io s (D po- s os ro , Dos d v ry 2–4 w ks S rum s os ro v s f uc ua sig i ca yb w dos s


D a s ry )

Subcu a ous p s (t s op ) t h s ar imp a d ( ik n orp a ) a d ar rar y us d or


supp m a io

Ora ag s (m hy s os ro ) n o avai ab i h U i d S a s; sig i ca rs -pass m abo ism,


f uc ua i g s rum v s, a d risk o iv r oxici y
468 FAMIl Y MeDICIn e eXAMIn At IOn & BOARD ReVIeW

does not appear to signi cantly a ect depression, so “A” is incor-


HELPFUL TIP:
rect. “B” is not true. Young hypogonadal males who are treated
I 2015, h FDA r quir d upda d ab i g o s os-
with testosterone have increased peak bone mass and viriliza-
ro produc s o add i orma io abou h possib
tion. But the e ects on older males are di erent: older males
i cr as d risk o cardiovascu ar v s. t h risk–b i
have improved strength, libido, and sense o well-being with-
ra io o s os ro produc s app ars o n Ot w igh i
out some o the e ects seen in the young hypogonadal males.
h ir avor.
“E” is incorrect because testosterone has been shown to consis-
tently improve lean muscle mass and grip strength. estoster-
one replacement does not replace the need or antidepressants Objectives: Did you learn to . . .
i depression is the cause o symptoms. • Id i y s os ro r p ac m produc s a d discuss how
o d r ma s migh b (or o ) rom h m?
• R cog iz som o h risks associa d wi h s os ro
HELPFUL TIP:
supp m a io ?
Hypogo ada ma s shou d not ak s os ro r -
p ac m whi ryi g o impr g a h ir par rs
b caus i wi ur h r d cr as h ir sp rm cou s (du
CASE 16.8
o h c o xog ous s os ro di g back o A 50-year-old male presents or a “get acquainted” visit. He has
h hypo ha amic–pi ui ary–go ada axis). t o improv a history o hyperlipidemia treated with lovastatin. He takes
r i i y i h s pa i s, u prov mpiric h rapy is no other medications and states that he is otherwise healthy.
o us d, i c udi g c omiph , huma chorio ic A er watching a television eature called “What’s Killing Us
go ado ropi , a d go ado ropi -r asi g hormo . Now: Prostate Cancer,” he thought he should be screened.

Question 16.8.1 In order to assess his risk or prostate can-


cer, you should take into account all o the ollowing actors
HELPFUL TIP:
EXCEPT:
t opica s os ro ca hav a a drog izi g cs
A) Age
o wom a d chi dr i h y com i co ac wi h h
B) Race
g o a ma ’s ski .
C) Family history
D) Cigarette smoking
E) Ultraviolet light exposure
You have a conversation with your patient about starting
testosterone therapy. He would like to try it. T ose direct-to- Answer 16.8.1 The correct answer is “E.” Ultraviolet light expo-
consumer marketing campaigns really work! sure increases the risk o skin cancer but not prostate cancer (it is
where “the sun don’t shine”). Advancing age is strongly and directly
Question 16.7.3 You tell him about adverse e ects and associated with the development o prostate cancer. Between ages
monitoring and tell him it is important to periodically 50 and 70 years, the incidence o prostate cancer more than qua-
check: druples, and it continues to increase therea er. Black ancestry and
A) Hematocrit positive amily history carry the strongest associations with pros-
B) Potassium tate cancer. Cigarette smoking has been linked to an increased risk
C) Alanine aminotrans erase o prostate cancer, but not with the strength o association o age or
D) A and B amily history. Interestingly, dietary actors may have a role in the
E) A and C development o prostate cancer, although the associations are not
strong and studies con ict. A diet rich in sh and vegetables and
Answer 16.7.3 The correct answer is “E.” T ere are good low in red meat may reduce the risk o prostate cancer.
reasons behind the recommendation or periodic monitoring
with certain serum tests. estosterone increases erythropoietin Your patient is Caucasian and denies any amily history o
production and can cause erythrocytosis. In act, testosterone prostate cancer. He has no urinary symptoms and denies
should not be started in a patient with a hematocrit greater than sexual dys unction.
50% (or severe heart ailure or untreated sleep apnea). estos-
terone can cause elevated liver enzymes, increased cholesterol, Question 16.8.2 I he did have urinary symptoms, which
and growth o prostate and breast tissue, including prostate o the ollowing could be used to reliably distinguish BPH
and breast cancers. Prostate examination and PSA are recom- rom prostate cancer?
mended prior to starting therapy and periodically while on A) Urgency
testosterone. O course, it makes sense to periodically monitor B) Nocturia
serum testosterone levels to assure that the patient is in the nor- C) Frequency
mal range and not sub- or supratherapeutic. Potassium (“B”) is D) Hesitancy
not directly a ected and no monitoring is recommended. E) None o the above . . . or a unctional crystal ball
CHAPTER 16 • Men ’S HeAl t H 469

Answer 16.8.2 The correct answer is “E.” A male who presents (not bound to plasma proteins) is inversely proportional to risk
with urinary symptoms o urgency, nocturia, requency, and/ o prostate cancer: the lower the ree PSA compared with total
or hesitancy is more likely to have BPH and not prostate can- PSA, the greater the risk o prostate cancer (cuto percentages
cer. T ese urinary symptoms are incredibly common in older vary rom 10% to 25%).
persons in general and most o en are benign in nature. In act,
“BPH symptoms” (better termed lower urinary tract symptoms
HELPFUL (OR NOT SO HELPFUL) TIP:
or “LU S”) are common in older women as well. However, uri-
t h d rmi a io o wh o s d a pa i or pros-
nary tract cancers (including prostate and bladder cancers) can
a biopsy is mad o c i ica grou ds a d mus ak
present with these symptoms and must be considered. T e point
i o accou h pa i ’s risk, comorbidi i s, xami a-
is that we cannot always attribute urinary symptoms in an older
io i di gs, PSA v , possibi i y o a s -posi iv or
male to BPH.
ga iv s s, pa i ’s pr r c , c. t his d cisio
ca o b mad bas d o a arbi rary PSA va u .
You discuss prostate cancer screening with him, presenting
the controversial and con icting guidelines, discussing posi-
tive and negative values o lab tests and what they signi y, Question 16.8.4 Which o the ollowing is true o the PSA
using a dry erase board and giving a mini-lecture. wenty test when used or screening purposes?
minutes later, he says, “It’s just a blood test right? I think I A) T e positive predictive value approaches 100%
want one ounce o prevention, you understand, Doc.” He gets B) Since PSA testing has become widespread, prostate cancer
“the works”: digital rectal examination and PSA. T e exami- incidence and mortality have increased
nation is remarkable or a slightly enlarged, smooth, non- C) T e alse-negative rate is 20% to 25%
tender prostate without nodules. T e PSA level is 16 ng/mL D) Age-speci c cuto values or PSA have proven to increase
(re erence range or your lab 0–4 ng/mL). positive predictive value and speci city
E) Race-speci c cuto values or PSA have proven to increase
Question 16.8.3 The most appropriate next step is to: positive predictive value and speci city
A) Repeat the PSA in 3 to 6 months
B) Order a transrectal ultrasound o the prostate Answer 16.8.4 The correct answer is “C.” T ere is a substan-
C) Re er to urology or urther evaluation and prostate biopsy tial alse-negative rate— rom 20% to 25%—limiting the use o
D) Repeat the rectal examination and try to nd the nodule that PSA or screening purposes. “A” is incorrect by a long shot. T e
you missed positive predictive value (likelihood o a positive test indicating
E) Order a ree PSA true prostate cancer) ranges rom about 20% to 60%, depending
on the PSA level. T e positive predictive value increases with
Answer 16.8.3 The correct answer is “C.” o use the Monop- increasing PSA level. “B” is incorrect. Since the PSA test has
oly line: go directly to the urologist; do not pass GO. I he had come into widespread use, the incidence o prostate cancer has
symptoms o prostatitis, you might try treating with an appro- increased, but mortality due to the disease has decreased. T e
priate antibiotic or 4 weeks and repeat the PSA to con rm ele- reason or these trends has not been completely explained, and
vation be ore re erral. But this patient is completely asymptom- the role o the PSA test in these trends is not known. “D” and
atic. He is at relatively high risk o having prostate cancer based “E” are incorrect. Since older males seem to have higher PSA
on his PSA level (16—that’s high, in case you were uncertain) values in the absence o prostate cancer and black males tend to
and age at presentation. Sending him to a urologist or pros- have prostate cancer ound at lower PSA levels, age-speci c and
tate biopsy is the most prudent step. Follow-up PSA alone (“A”) race-speci c cuto values have been proposed and investigated.
would be more de ensible i the PSA level were < 10 ng/mL However, the use o age-speci c and race-speci c PSA cuto
and/or the patient were much older. “B,” a transrectal ultra- values is o questionable value.
sound, is likely to miss more than it would nd in this case.
With this patient’s PSA and current li e expectancy, ultrasound
should only be used as part o a larger urologic evaluation, HELPFUL TIP:
including biopsy. “D,” repeating the rectal exam, is just nuts. t h 5-y ar surviva ra o pros a ca c r is abou
T ere is no need or this. Did you miss a nodule? Maybe. Would 99%. Whi h i im pr va c o pros a ca c r is
it hurt your ego? Who cares! You will eventually get over it. abou 14%, h i im risk o dyi g o pros a ca c r
More likely, there was no palpable nodule. T is patient could is ss ha 3%. For o d r m , w o h ar, “You ar
have stage 1 prostate cancer in which, by de nition, no nod- mor ik y o di with pros a ca c r ha of pros a
ules are palpable. He could have a nodule located anteriorly in ca c r.” t his sayi g app ars roo d i r a i y.
the prostate—a location you cannot palpate. He also may have
a PSA elevation or reasons other than prostate cancer. Finally,
“E” is incorrect. T e ree PSA is most help ul when the total Again your winning personality and reputation or thorough
PSA is in the intermediate risk zone (4.01–10 ng/mL). When care has won you a re erral. Your patient re ers a riend to
the PSA is higher (> 10 ng/mL), the risk o cancer is too great you. T is patient is a 60-year-old male who presents to your
to make the ree PSA use ul. T e raction o PSA that is “ ree” clinic or advice regarding treatment a er recently being
470 FAMIl Y MeDICIn e eXAMIn At IOn & BOARD ReVIeW

diagnosed with prostate cancer. A urologist recommended • I rpr a va d PSA?


surgery but he wants to know what you think. (Don’t let it go • R cog iz h imi a io s o pros a ca c r scr i g?
to your head.) • R cog iz various r a m op io s or pros a ca c r?

Question 16.8.5 Possible treatment options or prostate CASE 16.9


cancer include all the ollowing, EXCEPT:
A) External beam radiation A 32-year-old male accompanied by his wi e presents or eval-
B) Saw palmetto uation o in ertility. T ey have been married or 5 years and
C) Active surveillance have been attempting conception or 3 years without success.
D) Robotic prostatectomy T e patient’s wi e has a 7-year-old daughter rom a previous
marriage. T e patient has no signi cant past medical or sur-
Answer 16.8.5 The correct answer is “B.” While over-the- gical history. He does not smoke but drinks a six-pack o beer
counter saw palmetto is sometimes taken by patients or BPH and one cup o cof ee daily—not necessarily in that order. He
even though it has not been proven to be e ective, it is not used relaxes a er work and on weekends by sitting in their hot tub
or prostate cancer treatment. External beam radiation, brachy- or hours at a time. His BMI is 32 kg/m2. He has normal acial
therapy, surgery (open, laparoscopic, or robotic prostatectomy), and body hair and his testicles are descended bilaterally and
and androgen deprivation therapy are options considered or o normal size. You note a moderate sized varicocele on the
higher-risk prostate cancers. Watch ul waiting and active sur- le .
veillance are options considered or lower-risk prostate cancers.
Active surveillance consists o scheduled PSA testing and digi- Question 16.9.1 Which o the ollowing is not a modif able
tal rectal examinations every 3 to 6 months and repeat prostate risk actor or male sub ertility?
biopsies at 1- to 3-year intervals. A) Alcohol intake
B) Hot tub usage
C) Varicocele
HELPFUL TIP:
D) obacco use
I is impor a o r a iz ha h r is gr a co rov rsy
E) Obesity
ov r h ro o digi a r c a xami a io a d PSA i
scr i g or pros a ca c r. t h y ar o co sid r d
Answer 16.9.1 The correct answer is “D.” We should note
“good” scr i g s s (high ra o a s posi iv s
that the study o in ertility risk actors and treatments is
a d as ga iv s), bu h r is o hi g s o o r
complicated by inconsistent outcome measures (e.g., sperm
a his im . I 2013 h Am rica Uro ogica Associa-
count, sperm motility, conception, pregnancy resulting in
io cha g d h ir guid i s o scr i g ag s 55 o
live birth). However, there are no conclusive studies correlat-
69 v ry o h r y ar. Si c 2012, h US Pr v iv S r-
ing tobacco use with male sub ertility. T e other answers are
vic s t ask Forc has r comm d d AGAIn St pros a
modi able risk actors and have been directly linked to sub-
ca c r scr i g. t h wo arg s s udi s o pros a
ertility, either in decreased sperm count or decreased sperm
ca c r scr i g wi h PSA ( h Pros a , l u g, Co o ,
motility. It is important to ask about alcohol use (“A”). Other
a d Ovaria Ca c r Scr i g t ria [Pl CO] a d h
drug use, such as marijuana, should be investigated as well
europ a Ra domiz d S udy o Scr i g or Pros-
since it can a ect sperm quality and motility. “B” is true. Hot
a Ca c r [eRSPC]), do o a sw r h qu s io o
tub usage, ebrile illnesses, and the presence o a varicocele
how PSA scr i g a c s mor a i y, arriv d a oppo-
raise the temperature o the testicles, thereby decreasing the
si co c usio s (Pl CO show d o mor a i y c wi h
optimal environment or the maturation o sperm. But how
scr i g, whi h eRSPC show d a pros a ca c r-
about the boxers versus brie s controversy? ype o under-
sp ci ic mor a i y r duc io wi h PSA scr i g). How-
wear does not seem to a ect scrotal temperature signi cantly,
v r you approach his issu , you’r o a o , a d h
and more to the point, tight underwear is not associated
compa y you k p is jus as co us d. ev ryo agr s
with decreased ertility. T e presence o a varicocele (“C”)
ha scr i g or m o d r ha 75 y ars or wi h ss
is an interesting issue. It is certainly modi able in that the
ha 10 y ars o iv is o us u .
patient could undergo a varicocelectomy, which might help
i the varicocele is moderate to large in size. However, the
degree to which a varicocele contributes to in ertility is not
HELPFUL TIP:
well-known. Varicoceles are noted to occur more commonly
t h r su s o a ra domiz d ria sugg s ha vi ami e
in in ertile men, but they also occur in 10% to 15% o the
supp m a io may i cr as h ra o pros a
normal, ertile male population. Obesity (“E”) contributes to
ca c r. Do ’ do i (JAMA. 2011;306(14):1549–1556).
the increased peripheral aromatization o testosterone into
estradiol in atty tissue.
Objectives: Did you learn to . . . Other actors that contribute to sub ertility/in ertility
• Id i y risk ac ors or pros a ca c r? include a history o cryptorchidism, hypospadias, viral orchitis
• Discuss pros a ca c r scr i g wi h a pa i ? a er puberty, prior chemotherapy or radiation, intake o calcium
CHAPTER 16 • Men ’S HeAl t H 471

channel blockers, and retrograde ejaculation associated with TABLE 16-3 A PARTIAL LIST OF CAUSES
diabetes and multiple sclerosis. OF MALE INFERTILITY
Mechanism Specif c Examples
HELPFUL TIP: Hypo ha amic– Co g i a disord rs (Ka ma sy drom ),
I r i i y i a coup is d i d as i abi i y o co c iv pi ui ary disord rs pi ui ary umors, pi ui ary i arc io ,
d spi 1 y ar o r qu (how r qu is o d i d), hormo a or psycho ropic drug us
u pro c d s xua i rcours . I h U i d S a s, Primary Ki r sy drom , cryp orchidism,
h pr va c o i r i i y is 7% o 15% d p di g o hypogo adism a coho us , ch mo h rap u ic ag s,
how h s a is ic is m asur d. s icu ar orsio , hyp r h rmia

Disord rs o sp rm Co g i a abs c o h vas d r s,


ra spor pididyma dys u c io , spi a cord i jury
Question 16.9.2 You order a seminal uid analysis, and the
sperm density is 12 million/mL with a motility o 35%. This Idiopa hic i r ii y U xp ai d sa is ac ori y by his ory,
f nding is: xami a io , a d abora ory va ua io
A) Normal or both sperm count and motility
B) Normal or sperm count but abnormal or motility
C) Abnormal or sperm count but normal or motility Question 16.9.4 How can you now best help this patient
D) Abnormal or both sperm count and motility achieve ertility, assuming that there are no problems with
E) Scary; that seems like a lot o sperm his partner?
A) Empiric treatment or gonorrhea and chlamydia
Answer 16.9.2 The correct answer is “D.” As a general rule, B) Empiric treatment with testosterone injections
normal sperm concentration or ertility is considered to be C) Empiric treatment with gonadotropin-releasing hormone
15 million/mL or greater. However, as they say, it only takes one D) Re erral to an in ertility treatment center
sperm to make a baby, so men with lower sperm counts can
be ertile. At least 40% or more should be motile. Another Answer 16.9.4 The correct answer is “D.” At this point o time,
nding o importance on SFA is the sperm morphology. Using you have per ormed a reasonably complete evaluation and even
strict criteria, normal morphology should be 4% or more (yes, arrived at a potential cause o in ertility. However, treatments
un ortunately or men, only 4%). or male in ertility are the subject o much debate, and the
patient is probably best served by re erral to an in ertility treat-
You repeat the SFA 2 weeks later, and again it is abnormal. ment specialist. “A” is incorrect. Genital in ections are not
His testosterone level is low or his age, and his FSH and LH thought to play a major role in male sub ertility/in ertility, so
are high. without clear evidence o gonorrhea and/or chlamydia, treating
or these diseases is not recommended. “C” is incorrect because
Question 16.9.3 Which o the ollowing is most likely to be this patient does not appear to have a hypothalamic source or
the cause o his decreased ertility? his in ertility. “B,” testosterone injection, can actually make the
A) Congenital absence o the prostate gland problem worse. T e most e ective therapy or patients with
B) Primary hypogonadism in ertility and primary hypogonadism (i.e., this patient) may be
C) Bilateral complete vas de erens obstruction sperm retrieval or intracytoplasmic sperm injection (not typi-
D) Androgen resistance cally thought o as an o ce procedure or the amily physician).

Answer 16.9.3 The correct answer is “B.” A low testosterone You re er this gentleman to an in ertility treatment center
level accompanied by elevated LH and FSH could indicate tes- in your neck o the woods. He is so pleased with your atten-
ticular ailure, or primary hypogonadism. “A” is not a known tion to detail and well-reasoned approach that he re ers his
cause o in ertility, and in act it is not a known disorder as ar best riend to you or the same problem—in ertility. T is new
as the editors can determine. T e prostate gland’s only impor- patient reports never conceiving, although his wi e has had
tant unction is to provide a subject or an endless debate over one child rom a previous marriage. On physical examina-
screening so that epidemiologists have something to do. Just tion, you note a thin rame, mild symmetric gynecomastia,
kidding. . . . T e prostate gland actually has a role in produc- and small, rm testicles. He has complete azoospermia on
ing important substances that are part o the seminal uid and two semen analyses.
assist sperm in their migration through the emale genital tract.
However, prostate problems do not seem to cause in ertility. Question 16.9.5 His most likely diagnosis is:
“C,” an obstruction o the vas de erens, would most likely result A) Cystic brosis
in azoospermia with normal or high testosterone levels. “D” is B) Psychosomatic azoospermia
incorrect. You would expect to see elevated testosterone levels C) Kline elter syndrome
in patients with androgen resistance. See able 16-3 or a list o D) esticular cancer
causes o in ertility in males. E) urner syndrome
472 FAMIl Y MeDICIn e eXAMIn At IOn & BOARD ReVIeW

Answer 16.9.5 The correct answer is “C.” T e triad o small chronic renal ailure, and Peyronie disease (a localized plaque-
rm testicles, gynecomastia, and azoospermia are classic nd- like brosis leading to possible erectile dys unction, penis cur-
ings in patients with Kline elter syndrome. Kline elter syn- vature, shortening o the penis).
drome occurs in 1 in 1,000 live male births and is responsible or
14% o cases o azoospermia. T e most common (90%) chro- HELPFUL TIP:
mosomal abnormality in Kline elter syndrome is 47,XXY. “A” t h r ar wo ca gori s o r c i dys u c io —
is incorrect. Although males with cystic brosis can have con- psychog ic a d orga ic. I m you g r ha 35 y ars,
genital absence o the vas de erens, these patients usually have psychog ic r c i dys u c io is much mor com-
normal-sized testicles and no gynecomastia. “B” is, o course, mo . I co ras , m o d r ha 50 y ars ar mor ik y
not a real thing. “D” is incorrect. Patients with testicular cancer o hav a orga ic caus or h ir r c i dys u c io .
usually have an enlarged testicle with a mass on the sur ace or
inside the testicle. Although the treatments or testicular cancer
may result in in ertility, patients with testicular cancer do not Question 16.10.2 What urther historical element(s) is/are
typically present with in ertility. “E” is incorrect. urner syn- use ul in the evaluation o erectile dys unction?
drome (45,X) is associated with unambiguously emale genitalia A) Rapidity o onset o sexual dys unction
with no breast development. B) Presence o nocturnal erections
C) Status o relationship with the sexual partner
Objectives: Did you learn to . . . D) Partner’s interest in sex
• D modi ab risk ac ors or ma sub r i i y/ i r i i y? E) All o the above
• R cog iz h sig i ca c (or ack o sig i ca c ) o a vari-
coc ? Answer 16.10.2 The correct answer is “E.” Further history
• Id i y h appropria i dica io s or ob ai i g abora ory should involve all the elements listed. Patients with a sudden
s udi s i ma pa i s wi h i r i i y co c r s? onset o erectile dys unction o en have a primary psychogenic
• R cog iz som impor a caus s o i r i i y? erectile dys unction. T e presence o nocturnal erections estab-
lishes that the patient’s neurologic and vascular mechanisms
CASE 16.10 work to produce an erection. An organic disorder may still be
playing a role, but the circuit is working. T e status o the rela-
A 63-year-old male with a history o insulin-dependent dia- tionship with the patient’s partner, his attraction to that part-
betes complains o decreased libido and di culty maintain- ner, and the partner’s interest in sex are important. I the patient
ing an erection, worsening over the past ew years. He does and his partner are having relationship problems outside o the
have occasional erections su cient or penetration and sexual arena, counseling may be the best rst step in trying to
awakens with an erection at times. His medical history is also address the erectile dys unction.
signi cant or hypertension and an appendectomy. His medi-
cations include insulin, lisinopril, and hydrochlorothiazide. Question 16.10.3 All o the ollowing drugs or classes o
He has been married or 30 years and has two grown children, drugs can adversely a ect male sexual unction EXCEPT:
ages 24 and 26. A) Cimetidine
B) Chlorthalidone
Question 16.10.1 Which o the ollowing historical ele - C) Prednisone
ments is NOT likely to contribute to this patient’s erectile D) Clonidine
dys unction?
A) Diabetes Answer 16.10.3 The correct answer is “C.” T ere are many
B) Depression drugs, prescription and recreational, that a ect sexual unction,
C) Hypertension and some o these include antihypertensives, antidepressants,
D) History o appendectomy antipsychotics, anxiolytics, hormonal agents (e.g., antiandro-
E) Antihypertensive medications gens, estrogens, progestational agents, and anabolic steroids),
and the H 2 blocker cimetidine (but apparently not ranitidine or
Answer 16.10.1 The correct answer is “D.” We’re checking to amotidine). T e e ects range rom decreased libido to impo-
see i you’re still awake. All the other options could cause some tence and/or ejaculatory dys unction. Various recreational drugs
degree o erectile dys unction. Any disease process that a ects such as alcohol, marijuana, heroin, and cocaine may initially
the nervous, vascular, endocrine, or smooth-muscle systems cause a state o disinhibition and enhanced libido. However,
can result in erectile dys unction. A partial list o other risk excessive or chronic use leads to erectile dys unction. Predni-
actors or erectile dys unction includes advancing age, pros- sone is not known to cause signi cant erectile dys unction.
tate disease or surgery (i the nerves are cut . . . not an issue
with URP), pelvic racture, alcohol or other substance abuse, On physical examination, you nd normal genitalia, normal
medications (addressed later), spinal radiculopathy or spinal emoral and dorsalis pedis pulses, appropriate virilization,
cord injury, multiple sclerosis, endocrine disorders (e.g., hypo- and slightly diminished sensation at the plantar aspects o
thyroidism, hyperthyroidism), smoking, cardiovascular disease, the eet with an otherwise intact neurological examination.
CHAPTER 16 • Men ’S HeAl t H 473

Question 16.10.4 Which o the ollowing will be most use -


HELPFUL TIP:
ul in evaluating a cause or his erectile dys unction and
t h r is o much di r c b w h various phos-
directing urther therapy?
phodi s ras i hibi ors o h mark . t h icacy is
A) BUN and creatinine
abou h sam . Sid - c pro i s ar simi ar. O s
B) SH
o ac io is abou h sam . t ada a i has h o g s
C) PSA
ha - i . n o ar g ric ( or h i dica io o r c i
D) Nocturnal penile tumescence study
dys u c io —as o 2015). A ar v ry xp siv , abou
E) Arterial and venous Doppler studies
$40 per tablet (a h wri i g o his book).
Answer 16.10.4 The correct answer is “B.” With the advent
o sa e and e cacious therapy or erectile dys unction, many
clinicians proceed directly to a medication trial without labo- You start the patient on sildena l. He discovers love again,
ratory studies. In many patients, this approach is acceptable. but his wi e nds out. Now you’re in trouble!
Other patients may bene t rom a limited laboratory evalu-
Objectives: Did you learn to . . .
ation—especially those men with other symptoms and/or
• eva ua a pa i wi h r c i dys u c io ?
comorbidities. Initial labs might include SH, testosterone,
and prolactin. I not done already, screening or diabetes and • D scrib various io ogi s o r c i dys u c io ?
vascular risk (e.g., lipids) is appropriate. T e role o nocturnal • Id i y m dica io s ha a c r c i u c io ?
penile tumescence is debated, but it is not necessary prior to • R cog iz i dica io s, co rai dica io s, a d sid c so
a therapeutic trial and this patient reported having nocturnal h rap u ic moda i i s or r c i dys u c io ?
erections anyway. BUN, creatinine, and PSA are not likely to
be help ul. Doppler ow studies o emoral vessels are unlikely
to change therapy as long as the physical examination dem- Clinical Pearls
onstrates normal distal blood ow. In this patient, you know Do o d ay a urg uro ogica co su or h r am
that he has vascular disease (diabetes, hypertension, and now o priapism as his is a uro ogica m rg cy.
erectile dys unction), so his treatment should already involve
Do o p r orm pros a ic massag i h r is a suspicio
lowering his vascular risk actors. or pros a i is du o h risk o s di g bac ria i o h
b oods r am.
Question 16.10.5 Concurrent use o which o the ollowing
Do o us ora s os ro r p ac m h rapy as h r
drugs is an absolute contraindication to taking a phospho-
is a sig i ica irs -pass m abo ism a d po ia or iv r
diesterase type 5 inhibitor (e.g., tadalaf l, vardenaf l, silde - oxici y.
naf l)?
A) Hydrochlorothiazide e gag i a shar d discussio abou h us o PSA i scr -
i g or pros a ca c r, po ia b i s, a d po ia ga-
B) Isosorbide dinitrate
iv co s qu c s prior o d cidi g o p r orm scr i g. t h
C) estosterone USPSt F r comm ds agai s rou i scr i g or pros a
D) Finasteride ca c r bu h guid i s/r comm da io s vary!
E) Saw palmetto
Rou i y scr you g ma pa i s or high-risk b haviors
as rauma is h adi g caus o d a h i you g m .
Answer 16.10.5 The correct answer is “B.” In patients who take
nitrates or coronary heart disease, phosphodiesterase inhibitors Scr or po ia s co dary caus s o r c i dys u c io
are contraindicated. T e combination has been shown to cause prior o pr scribi g m dica io .
hypotension, in rare cases severe enough to result in stroke. Scr you g boys or cryp orchidism by doi g a g ia a d
While concurrent use o nasteride is sa e, the alpha-blockers s icu ar xami a io a w -chi d visi s.
used to treat BPH are another story (see below). Drugs that
inhibit cytochrome P450 isoenzyme, such as cimetidine, eryth-
romycin, clarithromycin, itraconazole, ketoconazole, and HIV BIBLIOGRAPHY
protease inhibitors, may warrant phosphodiesterase inhibitor American Urological Association Education and Research, Inc.
dosage reductions. Caution with phosphodiesterase inhibitors (AUA). Early detection o prostate cancer: AUA guideline.
is advised in patients with uncontrolled hypertension, recent Linthicum (MD): American Urological Association
stroke or myocardial in arction, li e-threatening arrhythmias, Education and Research, Inc.; 2013.
unstable angina, or heart ailure. Other potential medical thera- Bremner WJ. estosterone de ciency in older men. N Engl J
pies or erectile dys unction include testosterone supplementa- Med. 2010;363:189.
tion and yohimbine. T ere are even more treatment modalities: Centers or Disease Control and Prevention. Sexually transmit-
sex therapy, vacuum erection devices, intracavernosal injection ted diseases treatment guidelines, 2015. MMWR. 2015;
therapy, intraurethral pharmacotherapy, arterial revasculariza- 64(3)59(RR-12):1–137.10.
tion (not routinely per ormed strictly or ED), penile prosthesis Craw ord P. Evaluation o scrotal masses. Am Fam Physician.
implantation, and combined therapy. 2014;89(9):723–727.
474 FAMIl Y MeDICIn e eXAMIn At IOn & BOARD ReVIeW

Dickson G. Gynecomastia. Am Fam Physician. 2012;85(7); Sharp V. Prostatitis: diagnosis and treatment. Am Fam
716–722. Physician. 2010;82(4);397–406.
Edwards JL. Diagnosis and management o benign ask Force on Circumcision. Male circumcision. Pediatrics.
prostatic hyperplasia. Am Fam Physician. 2008;77(10): 2012;130(3):e758–e785.
1403. eichman JMH. Urology: 20 Common Problems. New York, NY:
Heidelbaugh JJ. Management o erectile dys unction. Am Fam McGraw-Hill; 2001.
Physician. 2010;81(3):305. U.S.Preventive Services ask Force. Screening or prostate
Lindsay J. Evaluation and treatment o in ertility. Am Fam cancer: Recommendation Statement, and rationale. Ann
Physician. 2015;91(5):308–314. Intern Med. 2002;137(11):915.
Mohan R. reatment options or localized prostate cancer. U.S. Preventative Services ask Force. Screening or prostate
Am Fam Physician. 2011;84(4s):413–420. cancer: recommendation statement. Am Fam Physician.
Pearson R. Common questions about the diagnosis and 2013 Feb 15;87(4):1–7.
management o benign prostatic hyperplasia. Am Fam Wieder JA. Pocket Guide to Urology. 5th ed. Oakland, CA:
Physician. 2014;90(11):769–774. J. Wieder Medical; 2014.
Dermatology
Megan H. Noe and Karolyn A. Wanat
17
Since dermatology is a visual science, take a look at pictures question is new. She denies any symptomatic lesions. Her am-
at www.dermnet.org.nz/sitemap.html, or www.dermnet.com. ily history is remarkable or “skin cancer” on her ather’s side.
See able 17-1 or commonly used dermatology terms. Many amily members have “tons o moles.” She requented
a tanning parlor in college (too much ree time . . . probably
CASE 17.1 majored in communications) and occasionally still does so
be ore social events (didn’t learn much about skin health in
A 37-year-old white emale presents to clinic or her annual
college).
well adult physical examination. A er a complete skin exam-
ination, you nd a suspicious lesion on her back (Fig. 17-1).
Question 17.1.1 How do you evaluate the lesion?
It measures 16 mm × 8 mm. She has approximately 20 nevi
A) ake a photograph and see her back in 2 months
that appear normal. T e patient reports never per orming
B) ake a shallow shave biopsy o the lesion
sel -skin examinations and does not know i the lesion in
C) Excise the entire lesion with 1- to 2-mm margins
D) Excise the entire lesion with 3-cm margins
TABLE 17-1 DERMATOLOGY TERMS E) All o the above are equally valid approaches

Abrasion: Partial-thickness loss o the epidermis Answer 17.1.1 The correct answer is “C.” T is lesion is wor-
Abscess: Localized collection o purulent exudate risome or malignant melanoma because o the di erent pig-
Annular: Grouped in a circular arrangement mentation colors and irregular borders. Any time you suspect
Bulla: Fluid- lled cavity greater than 1 cm melanoma, you are obligated to completely remove the entire
Crust: Dried exudate on the skin sur ace; can be serous (yellow or lesion and send it or pathology either with a punch biopsy or
honey-colored) or hemorrhagic (red-brown); when related to an excisional biopsy. “A” is not an option; i you suspect melanoma,
in ection the term impetiginization can be used the earlier you make the diagnosis the better. A shave biopsy
Desquamation: Skin cells coming o in scales or akes (“B”) is also not recommended because the shave biopsy is o en
Erythema: A red color change that is blanchable with pressure too shallow and the deep margins are not visible, which is the
Herpetiform: Grouped in a dermatomal distribution
Macule/Patch: Circumscribed area o color-change without elevation
or depression; a “macule” is < 1 cm in diameter and a “patch” is > 1 cm
in diameter
Nodule: A solid subcutaneous mass
Nummular: coin-shaped or circular
Papule/Plaque: A palpable, (elevated) skin lesion; a “papule” is < 1 cm
in diameter and “plaque” is > 1 cm in diameter
Purpura: Bleeding into the skin; does not blanch with pressure
Pustule: A papule with purulent exudate
Ulcer: Full-thickness loss o the epidermis
Vesicle: A uid- lled cavity in the skin less than 1 cm
Wheal: A round or irregular-shaped edematous plaque that tends to
be transient (hive)
FIGURE 17-1.

475
476 FAMILY MEDICINE EXAMINATION & BOARD REVIEW

most important actor in staging melanoma. Partial removal o Answer 17.1.3 The correct answer is “B.” T e li etime prob-
a lesion may lead to a sampling error, which is why excisional ability o melanoma in the year 2011 or Caucasian emales was
biopsy with narrow (e.g., 1–2 mm) margins is pre erred. “D” is approximately 1 in 55. Caucasian males in the United States are
pre erable to “A” and “B”; however, it would be unnecessary to worse, with a li etime probability o 1/37.
remove the lesion with such large margins when the histopa-
thology is not yet known. Your biopsy reveals a malignant melanoma in situ. You excise
the entire lesion with 0.5-cm margins. T ere is no penetra-
You excise the entire lesion with a small border. T e pathol- tion o the epidermis.
ogy report reveals a malignant melanoma.
Question 17.1.4 What additional evaluation is necessary?
Question 17.1.2 Which o the ollowing lesion characteris A) Chest x-ray
tics will determine your patient’s prognosis? B) C scan o the chest
A) T e depth and ulceration C) Serum lactic acid dehydrogenase (LDH)
B) T e diameter o the lesion D) All o the above
C) T e number o colors in the lesion E) None o the above
D) Whether or not it arose in a pre-existing mole
E) T e number o lymphocytes in the biopsy
Answer 17.1.4 The correct answer is “E.” I the melanoma has
not penetrated the epidermis, it is re erred to as a melanoma
Answer 17.1.2 The correct answer is “A.” As alluded to pre-
in situ. I excised with 0.5- to 1-cm margins, the long-term sur-
viously, the depth o the melanoma is the most important
vival at 5 and 10 years approaches 100%. T ere ore, any addi-
prognostic indicator. Breslow tumor thickness is most com-
tional workup is not necessary according to the 2010 report
monly used to arrive at a prognosis. Breslow tumor thickness
rom the American Joint Commission on Cancer. When the
measures the depth o the melanoma rom the granular layer
melanoma depth is greater than 1 mm, urther workup is o en
in the epidermis to the base o the melanoma in millimeters.
per ormed. In general, chest radiography and a serum LDH
A thinner melanoma correlates with a better prognosis and
level are recommended or malignant melanomas greater than
melanomas with a Breslow depth 0.75 mm generally have the
1 mm in depth, mostly to obtain a “baseline.” Regarding a sen-
best prognosis. Ulceration is associated with more aggressive
tinel lymph node biopsy, although there is no proven therapeu-
cancers and a poorer prognosis. Clark level (invasion based
tic bene t, it can provide additional prognostic in ormation or
upon the anatomic structure o the skin) is still reported but is
melanomas with a depth > 1 mm. Serum LDH levels are usually
not as important in regards to prognosis as Breslow depth. T e
positive in very advanced cases that have metastasized to bone
diameter o the lesion (“B,” or the size observed on the skin) has
or liver. A C scan o the chest and pelvis and MRI o the brain
not been associated with prognosis (e.g., super cial spreading
can be ordered i metastatic disease is suspected, but these stud-
melanoma may be large but generally carries a good progno-
ies are not routinely recommended. Metastatic disease has not
sis). A very large, clinically atypical pigmented lesion can o en
been reported with malignant melanoma in situ.
be benign. Melanomas can also be o one color, many colors,
or nonpigmented; color does not correlate with prognosis. Up
to two-thirds o melanomas arise in normal skin. T ey can be Question 17.1.5 Which o the ollowing has NOT been asso
just as aggressive as those arising rom moles. T e number o ciated with an increased risk o the development o mela
mitoses in a lesion can help predict the outcome; the more the noma?
mitoses, the worse the prognosis. T e number o lymphocytes A) Blistering sunburns in childhood
observed within a lesion can provide some in ormation regard- B) Sister with melanoma
ing the in ammatory response but does not possess the same C) Fair hair color
prognostic value as the Breslow depth. D) More than 50 moles on a person’s body
E) Smoking
HELPFUL TIP:
Answer 17.1.5 The correct answer is “E.” A history o blister-
I there is regional or distant spread o disease, the over-
ing sunburns, melanoma in rst-degree relatives, air hair, and
all survival drops to less than 50% or regional metasta-
more than 50 moles are all associated with an increased risk
ses and less than 10% or distant metastases.
o developing melanoma. Also, high socioeconomic class and
immunosuppression appear to be risk actors. Smoking has not
Question 17.1.3 In the United States in 2011, the li etime yet been linked to malignant melanoma—unlike many other
risk o developing melanoma in a Caucasian emale is malignancies.
approximately:
A) 1/10 Question 17.1.6 The most common subtype o melanoma
B) 1/55 in people o A rican descent is:
C) 1/105 A) Super cial spreading melanoma
D) 1/1500 B) Lentigo maligna melanoma
CHAPTER 17 • DERMATOLOGY 477

C) Nodular melanoma her gluten- ree diet, and her skin eruption has worsened as a
D) Acral lentiginous melanoma result. Dermatitis herpeti ormis presents with excoriated vesi-
E) Amelanotic melanoma cles that are extremely pruritic. It can be con used with scabies.
O en, only excoriations (and no blisters) are seen on exami-
Answer 17.1.6 The correct answer is “D.” T ere are our clas- nation. Dermatitis herpeti ormis is also commonly symmetri-
sical subtypes o malignant melanoma. T e subtype does not cal (as opposed to HSV) and may involve several body areas,
predict prognosis. Super cial spreading melanoma is the most commonly extensor sur aces o the extremities. T e cor-
most common type in air skin populations. It can occur at any nerstone o treatment is a gluten- ree diet. Dapsone can also
site and clinically presents as a brown macule with irregular, help speed up resolution o the rash, but a strict gluten- ree diet
notched borders. It grows radially (outward) initially. Nodular will also lead to disease resolution. In this case, “B” is unlikely
melanoma is the second most common type in air skin indi- due to symmetric and di use eruption without linear lesions
viduals and, as the name suggests, is an exophytic nodule and and no reported exposure to poison ivy (we didn’t tell you this,
o en associated with deeper level o invasion and a vertical but her college is located in Antarctica, making poison ivy very
(downward) growth phase. Amelanotic melanoma is a subtype unlikely). “C” is incorrect since the eruption is di use and there
o nodular melanoma that presents with little to no pigment is no pain. Neurotic excoriations (“D”) do not present with
and may be red or pink in color. Lentigo maligna melanoma vesicles. I “E” were a real entity, the editors believe it would
most o en occurs in the elderly on sun-exposed areas, such as be an orthopedic disorder—some underuse atrophy o the joint
the ace and hands and presents as a brown macule or patch maybe.
with hue variations that spreads outward slowly. When there is
dermal invasion, lentigo maligna is re erred to as lentigo maligna CASE 17.2
melanoma. Acral lentiginous melanoma is the least common
type in Caucasian populations but is the most common type in A 7-month-old male is brought to clinic or a “rash all over.”
races with pigmented skin, such as people o A rican descent. Six weeks ago, his parents noticed him rubbing his legs
Acral lentiginous melanoma occurs on the palms, soles, or near against his crib and scratching his head requently. T ey
the nails. T is is what killed Bob Marley. are concerned because they nd blood on his sheets in the
morning, and he has become increasingly irritable. He is
Objectives: Did you learn to . . . eating and drinking normally. His past medical history is
• Evaluate a patient with suspected melanoma? unremarkable. His ather has sensitive skin and hay ever,
• Describe how the prognosis o melanoma is determined? but no one else in the amily currently has a rash. He does not
• Recognize the li etime risk o malignant melanoma in the attend a daycare. On skin examination, you nd licheni ed
United States? and erythematous patches o skin with ssures and bleeding
• Identi y risk actors or the development o melanoma? on the ventral heels, dorsal eet, hands, and a ew areas on the
• Recognize the our classic subtypes o melanoma and which scalp. His cheeks are bright red with scale. His diaper area is
are the most common in A rican and Caucasian populations? uninvolved and there are no lesions in the web spaces o the
hands and eet.

QUICK QUIZ: ITCHYELBOWS Question 17.2.1 Based on the description, which o the
ollowing is the most likely diagnosis?
A 22-year-old emale presents with a pruritic rash on her elbows A) Seborrheic dermatitis
and extensor orearms. She’s studying or nals, so she’s been B) Atopic dermatitis
indoors or days eating Wheat T ins and drinking Mountain C) Scabies
Dew. She has a history o celiac disease, but is otherwise healthy D) inea corporis
and takes no medications. On examination, she is a ebrile and E) inea versicolor
has 2- to 4-mm diameter vesicular lesions with numerous exco-
riations in a bilaterally symmetrical pattern on her extensor Answer 17.2.1 The correct answer is “B.” T e most likely diag-
orearms, knees, and buttocks. nosis is atopic dermatitis, also known by the moniker eczema.
Atopic dermatitis is characterized by an intensely pruritic erup-
T e most likely diagnosis is: tion o the skin with associated dryness. A red rash subsequently
A) Dermatitis herpeti ormis develops. Atopic dermatitis is o en re erred to as the “itch that
B) Poison ivy (contact dermatitis) rashes.” It occurs in characteristic locations. In younger in ants,
C) Herpes simplex the cheeks and neck are involved. As they begin to crawl, their
D) Neurotic excoriations extensor sur aces are involved. T e diaper area, because it is
E) Nerd elbow moist, is not usually involved. In older children, the exural
areas, such as the antecubital and popliteal ossae, are involved.
The correct answer is “A.” Dermatitis herpeti ormis is the most Seborrheic dermatitis (“A”) is common in in ants (cradle cap)
common skin condition to occur in association with celiac dis- and is usually seen on the scalp and ace, although it can involve
ease (gluten enteropathy). T is patient seems to have lapsed in the whole body. Seborrheic dermatitis is usually associated with
478 FAMILY MEDICINE EXAMINATION & BOARD REVIEW

a yellow, greasy scale and is less erythematous than atopic der- Question 17.2.3 The clinical eatures o impetiginized
matitis. Scabies (“C”) typically a ects certain locations (web atopic dermatitis include:
spaces, wrists, waist, etc.) and uncommonly involves the scalp, A) Licheni cation o the skin
except in in ants or immunocompromised patients. Also, in this B) Pruritus and relapsing nature
question, he does not appear to have any exposure to scabies. C) Associated asthma or allergic rhinitis
inea corporis (“D”) presents with a ring o advancing ery- D) Elevated IgE serum levels
thema and a central clearing (“ringworm”). Finally, tinea ver- E) Redness o the skin with honey-crusting
sicolor (“E”) typically involves the trunk and extremities with
hypo- or hyperpigmented patches with super cial scale. It does Answer 17.2.3 The correct answer is “E.” Although all o the
not involve the scalp, and is not very pruritic. above can be associated with atopic dermatitis, honey crust-
ing implies a secondary bacterial in ection (impetigo), which
You diagnose the patient with atopic dermatitis. is common in atopic dermatitis and needs to be recognized
and treated to get atopic dermatitis under control. Oral anti-
Question 17.2.2 Which o the ollowing is true about atopic biotics and chlorinated baths can be used to help control the
dermatitis? superin ection. Waxing and waning pruritus is what de nes
A) T e prevalence o atopic dermatitis appears to be decreasing this common skin condition. Chronic scratching o en leads to
worldwide thickened skin with accentuation o skin lines (licheni cation).
B) Atopic dermatitis tends to worsen with use o emollients Early lesions will not have licheni cation, however. Asthma and
C) In some patients, skin in ections can exacerbate atopic der- allergic rhinitis can be associated with atopic dermatitis. T is
matitis common hypersensitivity triad is re erred to as atopy (although
D) Positive skin prick tests and RAS testing correlate highly atopic dermatitis is not allergic in nature). “D,” elevated IgE,
with ood challenges (e.g., those with positive tests will have does occur in patients with atopic dermatitis, and higher levels
worsening o their rash when given a ood challenge) o serum IgE are associated with more extensive disease o
E) In most in ants, atopic dermatitis will not signi cantly greater chronicity. However, an elevated IgE level is merely an
improve or resolve by school age association and is not pathognomonic o atopic dermatitis. Ery-
thema o the skin is a nonspeci c sign o in ammation and is
Answer 17.2.2 The correct answer is “C.” Skin in ections seen in many skin disorders.
(Staph or Strep in ections) can be associated with worsening
atopic dermatitis, and when children are, impetiginization Question 17.2.4 Your initial recommendation should
(superin ection) should be considered. In these situations, a include which o the ollowing in the management o
course o antibiotics may improve the overall clinical course atopic dermatitis:
(cephalexin is o en a good choice). Sixty percent o atopic der- A) Daily use o thick emollients such as white petrolatum
matitis appears in the rst year o li e, usually a er 2 months o B) Bathing with lukewarm water and mild cleansers
age; and it will usually get somewhat better by school age. T e C) opical corticosteroids or topical immunomodulators
cause o atopic dermatitis is not yet known. T e role o speci c D) Oral antibiotics i there is evidence o superin ection
allergens is controversial. In some patients, a ood allergy can E) All o the above
worsen the disease but is not thought to be the cause. However,
in severe, unresponsive atopic dermatitis, ood allergens should Answer 17.2.4 The correct answer is “E.” T e protective barrier
be evaluated. Most patients who have positive allergy testing o the skin is broken down in patients with atopic dermatitis. By
to oods do not have improvement in their skin with removal requently applying a protective barrier, such as petrolatum, the
o the allergen. Atopic dermatitis does tend to improve as the skin becomes less pruritic resulting in less itching-induced skin
a ected child ages (“E”), and the use o emollients (“B”) is a trauma and rash, thus decreasing the “itch-scratch cycle.” T is
cornerstone o therapy. is the most important aspect o long-term management. opical
steroids and immunomodulators work well to decrease the
in ammation in the skin and are rst-line anti-in ammatory
treatment; however, the goal is to protect the skin with thick
HELPFUL TIP (OR NOT):
emollients so that the skin does not dry out and itch, leading
The link between allergies o any kind, including ood
to scratching and subsequent in ammation. Daily bathing
and environmental, and atopic dermatitis has been
with mild cleansers and cool water ollowed by the application
called into question. It does seem that a subset o pa-
o emollients is recommended. Patients with atopic dermatitis
tients have atopic dermatitis that lares in response to
have a higher bacterial count o Staphylococcus aureus on their
exposure to certain ood and environmental triggers,
skin. By bathing or short periods daily, the bacterial count is
but these patients are the minority. Random elimina-
decreased, thus decreasing the risk o secondary in ection. Oral
tion diets are to be discouraged. Only patients with
antihistamines cause some level o sedation, which is o en help-
proven ood allergy and an immediate worsening o
ul at night when the child is awake and itching. Interestingly,
symptoms when exposed to that ood should eliminate
there is almost no evidence to support the use o antihistamines
that particular ood rom their diet.
in the treatment o atopic dermatitis, except small studies that
CHAPTER 17 • DERMATOLOGY 479

have shown nonsedating antihistamines to be no better than Question 17.2.5 Which o the ollowing oral antimicrobials
placebo. I you choose to recommend an antihistamine, use an would be the best initial choice or this patient?
older drug (e.g., diphenhydramine) but use extreme caution A) Cephalexin
in children under 2 years o age—there is no proven bene t B) Cipro oxacin
or any indication in this age range and in ants are more sensi- C) Metronidazole
tive than adults and older children to the CNS depressant e ects D) etracycline
o diphenhydramine. “E” is incorrect. However, oral antibiotics E) No systemic treatment is necessary at this time
may be necessary i there is extensive impetigo. Finally, bacteria
can lead to a are o atopic dermatitis. Occasionally, treatment Answer 17.2.5 The correct answer is “A.” Patients with atopic
with antibiotics may be o bene t (more below). dermatitis are prone to certain skin in ections that may exac-
erbate their disease. Ninety percent o patients with atopic der-
HELPFUL TIP:
matitis will grow S. aureus on swab cultures o their crusted
Mid-potency steroid (e.g., triamcinolone) ointments are
lesions. By decreasing the bacterial count, in amed lesions o en
the mainstay o pharmacotherapy or atopic dermatitis
heal aster. Depending on your community susceptibilities,
lares. For the ace, low-potency steroid (e.g., hydrocor-
community-acquired methicillin-resistant S. aureus (CA-MRSA)
tisone) ointments can be used or a maximum o 2 to
may be o concern. However, rst-generation cephalosporins
3 weeks at a time. For severe, acute lares, systemic
are a reasonable initial antibiotic choice or impetiginized atopic
steroids can be employed or 10 to 14 days.
dermatitis because o excellent skin penetration and activity
against Gram-positive cocci (both Staph and Strep). etracycline
is another option or impetiginization; however, this should be
avoided in young children, as should uoroquinolones (except in
HELPFUL TIP:
rare cases such as cystic brosis). Systemic therapy is necessary
Topical calcineurin inhibitors (e.g., tacrolimus, pimecro-
at this time due to clinical worsening and presence o lymphade-
limus) are second-line therapies in the treatment o
nopathy.
atopic dermatitis. Although they are generally sa e and
well tolerated, they are expensive and carry a “black box
warning” related to possible cancer risk. There are a ew HELPFUL TIP:
reported cases o lymphoma and cutaneous cancers de- Patients with head and neck atopic dermatitis may ben-
veloping in humans using topical calcineurin inhibitors. e it rom systemic anti ungal therapy (e.g., itraconazole)
Animal studies support this association. They are con- or a month. Malassezia yeast is a common skin lora
traindicated in patients younger than 2 years. ound to cause an in lammatory reaction in patients
with head/neck atopic dermatitis, and its eradication
may improve atopic dermatitis symptoms in these pa-
A recent ear in ection has caused your patient’s skin to tients. The evidence is pretty weak, so save it or third-
worsen. He returns to clinic and your physical examination line treatment in patients with more severe symptoms.
reveals the skin lesion seen in Figure 17-2. He has apprecia-
bly enlarged cervical lymph nodes. T e patient has no known
drug allergies.
Your patient’s culture results are consistent with methicillin-
sensitive S. aureus (MSSA). With the appropriate antibi-
otics, the patient improves markedly. ime to rest on your
laurels . . . Wait . . . there’s this little thing called continuity o
care, and 3 months later, your patient returns with a new
eruption. Clinical appearance is shown in Figure 17-3.

Question 17.2.6 Which o the ollowing is the most appro


priate treatment at this time?
A) Acyclovir
B) Cephalexin
C) Cipro oxacin
D) Metronidazole
E) A and B

Answer 17.2.6 The correct answer is “E.” Patients with eczema


may also develop eczema herpeticum, a particularly severe
orm o disseminated herpes simplex with punched out erosions
and rapid spread o HSV within the areas o eczema. Patients
FIGURE 17-2. (Courtesy o Drs. Karolyn Wanat and Megan Noe.) can develop both eczema herpeticum and impetiginization,
480 FAMILY MEDICINE EXAMINATION & BOARD REVIEW

C) Patch testing
D) Serum thyroid-stimulating hormone level
E) Serum IgE levels

Answer 17.2.7 The correct answer is “C.” T e patient most


likely has an allergic contact dermatitis to the nickel in the metal
bracelet on his wrist. Patients with a history o atopic dermati-
tis are more likely to have contact hypersensitivities. Nickel is a
common contact allergen and can also be seen with contact to
earrings, optical glasses, and buttons on jeans. Patch testing is a
test or reactions to delayed-type hypersensitivity reactions that
can identi y many o the common contact allergens in the skin.
KOH application to a skin scraping is used to identi y ungal
elements. KOH dissolves keratin, the protein in skin, to better
identi y the ungal elements. zanck preparations stain blister
scrapings to evaluate lesions suspicious or herpes or varicella
viruses. T yroid disease can cause many skin conditions but is
FIGURE 17-3. not a known cause o allergic contact dermatitis. An abnormal-
ity in the SH level is least likely to yield a diagnosis in this case.
As to “E,” serum IgE levels may be elevated in atopic patients,
so treatment o both is important. T e characteristic look o but this is not diagnostic as IgE can be elevated in many states.
punched out erosions requires immediate therapy with acyclo-
vir, and o en requires intravenous medications. T ese patients Objectives: Did you learn to . . .
• Recognize atopic dermatitis by its classic presentations?
should be treated immediately with antivirals as the disease may
be atal. reatment with cephalexin in addition should be uti- • Identi y the hallmarks o atopic dermatitis?
lized, especially because the patient improved previously. • Manage a patient with atopic dermatitis and its complica-
tions?
As the patient grows, he improves signi cantly. However, • Recognize that eczema herpeticum in those with atopic
his skin continues to be sensitive to many products. As a dermatitis?
teenager, he presents with a recurring rash near his wrist
that is intensely pruritic. He has recently started wearing a CASE 17.3
bracelet . . . and he won’t take it o despite the rash because A 49-year-old obese white male with poorly controlled dia-
he’s too cool . . . even though it seems to be a girl’s bracelet (see betes mellitus, hypertension, and hyperlipidemia presents to
Fig. 17-4). clinic or a regularly scheduled visit. He complains that his
le and right legs are extremely red and pain ul. He denies
Question 17.2.7 The test most likely to con rm your pre constitutional symptoms. His vital signs are within normal
sumptive diagnosis is: limits. He has warm legs with circum erential erythema
A) Potassium hydroxide (KOH) o a skin scraping extending rom the ankle to the mid-calves with 2+ pit-
B) zanck preparation ting edema bilaterally with hemosiderin staining (brownish
macular lesions) o the ankles. T ere is no lymphadenopa-
thy. T ere are no open sores or minor trauma noted (see
Fig. 17-5). His complete blood count with di erential is
within normal limits. You send him or Doppler studies that
ail to reveal venous thromboses. You prescribe 7 days o oral
antibiotics and send him home, con dent that your therapy
will not ail.
He returns 3 days later with modest improvement in the
redness. A er completing the antibiotic course, he presents
to clinic 3 weeks later complaining o return o the redness.
(Why don’t patients just get better with a pill like they are
supposed to?)

Question 17.3.1 The most appropriate next step in the care


o this patient is to:
A) Prescribe another course o oral antibiotics or 14 days
B) Admit the patient to the hospital or intravenous antibiotics
FIGURE 17-4. C) Send a punch biopsy o skin or bacterial culture
CHAPTER 17 • DERMATOLOGY 481

is “woody” brosis), bound down skin as well as overlying reten-


tion o the skin and verrucous eatures (elephantiasis nostra
verrucosum).

From years o diabetic nephropathy, the patient’s kidneys


eventually ail leading to transplantation (and use o immu-
nosuppressants). A er 5 years o stable renal unction and
improved control o his diabetes, he develops a new nonheal-
ing lesion on the le orearm.

Question 17.3.2 What cutaneous malignancy is he at the


highest risk o developing?
A) Basal cell carcinoma
B) Malignant melanoma
C) Squamous cell carcinoma
D) Metastases rom an undiagnosed internal malignancy
E) Kaposi sarcoma

Answer 17.3.2 The correct answer is “C.” All primary cutane-


ous malignancies are increased in immunosuppressed patients.
However, squamous cell carcinoma is the most common in
transplanted, immunosuppressed patients, surpassing basal cell
carcinoma, which is most common in the general population.
Depending on sun exposure and length o immunosuppression,
the incidence o squamous cell carcinoma in transplant popula-
tions is as high as 45%. Any nonhealing lesion should be urther
investigated with a biopsy. T ere are also signi cantly increased
rates o Kaposi sarcoma in immunocompromised patients.

You removed the lesion, which was indeed squamous cell car-
cinoma. Good job! T e patient develops a shallow ulcer supe-
FIGURE 17-5. rior to the medial malleolus. “Is that another one o them
cancers, Doc?” your patient asks.

D) Recommend daily leg elevation and use o compression hose Question 17.3.3 The most common cause o an ulceration
E) Prescribe a diuretic (e.g., urosemide) at this location is
A) Arterial insu ciency
Answer 17.3.1 The correct answer is “D.” T is patient has a B) Diabetic neuropathy
characteristic history o stasis dermatitis, a condition that is C) Chronic venous stasis
o en misdiagnosed as recurrent or chronic cellulitis. Stasis D) Pyoderma gangrenosum
dermatitis is a dermatitis o the lower extremities that results E) Prolonged pressure (e.g., “decubitus” ulcer)
rom chronic edema. It can start relatively abruptly and be uni-
lateral or bilateral. T e pitting edema with hemosiderin stain- Answer 17.3.3 The correct answer is “C.” Your patient is at
ing is a clue that there is chronic uid extravasation rom the risk or many o the above diagnoses; however, venous ulcer-
vessels o the lower extremities. Ectatic veins may also be pres- ations classically occur over the medial lower leg. In contrast
ent. T e goal o therapy is directed at resolution o the edema. to this patient’s venous ulceration, arterial insu ciency ulcers
Patients must lose weight and employ a strict routine o com- (“A”) classically have a punched-out appearance and occur over
pression hose and leg elevation. In the short term, topical corti- bony prominences (e.g., malleoli) or distal aspects (e.g., tips o
costeroids can improve the in ammation. In general, diuretics toes) and are extremely pain ul (in the absence o neuropathy)
should not be used simply to treat edema, as these drugs have and usually dry. Poor peripheral pulses, cool extremities, and
numerous systemic e ects. Paradoxically, diuretics can actually hairlessness are clues to arterial insu ciency. Abnormal ankle–
make edema worse in the long term by causing low circulating brachial indices help to con rm peripheral artery disease. Neu-
volume and renal retention o sodium and water to make up ropathic ulcers (“B”), such as those associated with diabetic
or the diuretic-induced hypovolemia. Chronic lower extrem- neuropathy, also occur over pressure points including the plan-
ity edema can lead to local tissue necrosis, resulting in ulcer- tar sur ace o the oot. T e patient may complain o burning
ation. With long-standing venous insu ciency, more chronic or tingling o the oot, but the ulcer is asymptomatic (because
changes can also occur including lipodermatosclerosis (which o lack o pain sensation rom diabetic neuropathy). Pyoderma
482 FAMILY MEDICINE EXAMINATION & BOARD REVIEW

gangrenosum (“D”) is a rare cause o ulceration associated with


sensitive to terbina ine. Complete cure rates or systemic
systemic diseases including in ammatory bowel disease, con-
anti ungals range rom 50% to 75% (terbina ine has the
nective tissue diseases, and hematologic malignancies. Pressure
highest rate), while cure rates or topical anti ungals
ulceration (“E”) is common and results rom tissue ischemia
are less than 10% even under ideal conditions. For all-
rom prolonged pressure, usually over bony prominences, such
comers, recurrence a ter clearance is as high as 50%.
as the sacrum, coccyx, and heels.

As you start to exit the examination room, the patient states


“Oh by the way, Doc, can you do anything or my toenails?”
Your examination reveals three, yellow heaped-up nails on Another patient comes in with toenail ungus, but this time
the le oot. You suspect a dermatophyte in ection. he has white changes, most prominent on his proximal nail-
olds.
Question 17.3.4 What do you do next?
A) Per orm a KOH examination o toenail scrapings Question 17.3.6 You would like to test him or:
B) Empirically treat with an oral anti ungal A) Diabetes mellitus
C) Empirically treat with a topical anti ungal B) Kidney disease
D) ell the patient that his nails are the least o his worries C) Lupus erythematosus
D) HIV
Answer 17.3.4 The correct answer is “A.” T e cost o anti ungal E) Yum, let’s go eat mushrooms
therapy has declined substantially over the last ew years with
generic options becoming available. Some experts now argue Answer 17.3.6 The correct answer is “D.” T is pattern o der-
that “B,” empiric therapy, is reasonable in otherwise healthy matophyte in ection, proximal white subungual onychomyco-
patients without contraindications. However, a board examina- sis, is characteristic o immunosuppression and particularly
tion is not real li e, and “A” would be the best choice. Here’s the HIV disease. esting or underlying HIV is important with
argument or testing prior to treating: all that looks ungal is not this clinical presentation. Other nailbed changes can be seen
necessarily ungal. Dystrophic nails can mimic onychomycosis with diabetes, kidney disease, and lupus. As to “E,” mushrooms
and may be the result o psoriasis, eczema, trauma, or other might have this appearance but don’t grow on toes—we hope.
in ammatory conditions. KOH is an inexpensive and easy test As an aside, there is a great restaurant in Jogjakarta, Indonesia,
to per orm in the clinic setting. I it is negative, then a toenail where everything is made out o various types o mushrooms
clipping can be sent or ungal culture or pathologic study with (shel ungus, etc.). It is pretty yummy. I you are in the area
periodic acid–Schi staining i desired. check out “Je Jamuran.”
Objectives: Did you learn to . . .
T e KOH is negative, so you send a toenail clipping or cul- • Recognize the presentation o stasis dermatitis and describe
ture. It grows a Candida species. its etiology?
• Treat stasis dermatitis?
Question 17.3.5 Which o the ollowing would be the LEAST • Recognize the cutaneous malignancies seen in transplanted,
ef cacious medication? immunosuppressed patients?
A) Amphotericin • Di erentiate between common causes and sites o lower
B) Fluconazole extremity ulcerations?
C) Nystatin • Diagnose and treat onychomycosis?
D) erbina ne
E) Voriconazole CASE 17.4
Answer 17.3.5 The correct answer is “D.” All the above medi- You are working in the student health clinic at the local uni-
cations have some Candida species coverage, but terbina ne versity when a previously healthy 19-year-old emale presents
(Lamisil) has the least yeast coverage. O note, amphotericin with malaise or 3 weeks and a severe sore throat or 3 days.
would be the most toxic (but still would work well) and should Yesterday, she took some old amoxicillin that her roommate
be avoided. A plea from your editors: the topical “paint on” nail gave her. oday, she developed a di use macular, red skin
anti ungals are relatively expensive, have a marginal bene t, and eruption, starting on her chest and spreading to involve her
the recurrence rate tends to be high. Besides, who is really going extremities. Her mucous membranes, palms, and soles are
to paint their nails everyday or a year? ree o lesions, but she has enlarged tonsils with exudates and
large cervical lymph nodes, including prominent posterior
HELPFUL TIP: nodes. A rapid Strep test is negative.
Terbina ine (Lamisil) is e ective against most derma-
tophyte in ections o the nails. Most cases o onycho- Question 17.4.1 You suspect:
mycosis are caused by various Tinea species, which are A) An allergic reaction to the amoxicillin
B) An Epstein–Barr virus in ection (mononucleosis)
CHAPTER 17 • DERMATOLOGY 483

C) An enterovirus in ection C) T at the rash usually resolves within 6 to 8 weeks without


D) A gonococcal in ection treatment
E) Scarlet ever D) T at the rash is likely to be atal
E) T at he is living a Ka aesque nightmare and slowly meta-
Answer 17.4.1 The correct answer is “B.” All o the above morphosing into an insect
in ections except or enterovirus can present with an exuda-
tive pharyngitis; enterovirus presents with gastrointestinal or Answer 17.4.2 The correct answer is “C.” T e rash is consis-
meningeal symptoms. T e clinical scenario describes the usual tent with pityriasis rosea (PR), which is an acute, o en asymp-
course o in ectious mononucleosis: a college-age student, a tomatic, eruption on the trunk and proximal extremities. T e
ew week prodrome o malaise, and posterior cervical lymph- etiology is (still) unknown. Secondary syphilis, which is on the
adenopathy. Amoxicillin or ampicillin given to patients with rise in some areas o the United States, can mimic PR. A sexual
in ectious mononucleosis can result in a macular, di use rash. history is necessary, and appropriate laboratory studies should
T is is not an allergy but can be mistaken or one. Scarlet ever, be undertaken i a patient is sexually active, especially i high-
which this patient does not have, is a complication o group A risk behaviors have occurred or i local syphilis rates are high.
streptococcal in ection, which presents with an erythematous, PR, unlike syphilis, o en resolves within 8 weeks without treat-
coarse (“sandpaper”) rash, strawberry tongue, and skin desqua- ment and does not usually recur. opical steroids are not neces-
mation. sary or healing but may be help ul in the minority o patients
who experience itching. “E” is incorrect. T is rash just isn’t that
Later that same day, a 16-year-old male presents with a mildly serious.
pruritic rash on his trunk. He had a cold ew weeks earlier but
is otherwise healthy. He denies high-risk sexual activity or IV
drug abuse (o course, your de nition o “high-risk” and his HELPFUL TIP:
may not be the same!). Your examination reveals the ndings PR starts with a “herald patch,” a salmon-colored, 2- to
in Figure 17-6. 5-cm oval-shaped lesion on the back, neck, or chest. The
herald patch may clear a bit and scale and is then ol-
Question 17.4.2 You tell the patient: lowed by numerous smaller lesions that crop up mostly
A) T is skin condition is usually chronic and relapsing on the trunk. These lesions tend to be oval in shape and
B) T at topical corticosteroids are needed to speed up the are oriented along skin lines, giving a “Christmas tree”
healing appearance.

Your next patient is a 20-year-old emale complaining o


“a rash down there.” You con rm that she is talking about a
vulvar rash and ask about her sexual history. She says that
she had not been sexually active with anyone or about 1 year
until a recent spring break ing in Florida. On examination,
you nd a rash over the pubic region and extending into the
inguinal areas and medial thighs. T ere is no pain or pruritus
(see Fig. 17-7).

Question 17.4.3 Which o the ollowing is FALSE?


A) T ese lesions are transmitted in adults only by sexual contact
B) I not treated, the lesions may resolve on their own
C) opical corticosteroids do not improve this skin condition
D) T e lesions are contagious and can spread readily and
rapidly
E) T e skin lesions are a result o a viral in ection

Answer 17.4.3 The correct answer is “A.” T ese lesions,


re erred to as molluscum contagiosum, result rom a pox
virus. Molluscum is commonly spread via sexual contact in
adults; however, any physical human contact can also spread
it. Molluscum is extremely common in children in daycare set-
tings. T e lesions generally resolve in time without treatment.
Because they can be auto-inoculated, the molluscum lesions
FIGURE 17-6. (Courtesy o Drs. Karolyn Wanat and Megan Noe.) can rapidly increase in number, and it is or this reason that
484 FAMILY MEDICINE EXAMINATION & BOARD REVIEW

expensive option. “D,” candida antigen injection, is a therapy


that aims at stimulating an immune response to the warts.
Mumps antigen has been used as well. In the antigen injection
studies, only one wart was injected to achieve treatment or all
o a patient’s warts. T is can be e ective but is a pain ul injec-
tion. Destructive methods are all approximately 70% e ective
at eradicating warts. It is important to remind patients that no
single method is more likely to work than another, and warts
can resolve on their own. In immunocompetent patients, most
warts resolve in time without treatment. For young children,
this is an important issue, since they cannot understand the
pain associated with most treatment modalities.

HELPFUL TIP:
To use duct tape or warts, the patient should put a
FIGURE 17-7.
piece o duct tape on the wart and leave it on or 6 days.
On day 7, remove the duct tape, soak, and abrade the
treatment is recommended. reatment consists o destructive wart with a pumice stone. Reapply the duct tape the
methods, such as cryotherapy or curetting. T e immune modu- next day.
lator imiquimod has been used, but is not approved by the Food
and Drug Administration (FDA) or molluscum, and only small
trials have been undertaken. opical steroids are not indicated. A 30-year-old stressed out medical student presents to your
In immunocompromised patients, molluscum lesions can be clinic with a worsening, pain ul rash on one side o her chest.
widespread and persist much longer. She denies any prior spots like this and has no other medi-
cal problems. She is up-to-date on her vaccinations because
o medical school and complains that this rash is inter ering
HELPFUL TIP:
with studying or the upcoming nal block o examinations
Other molluscum therapies appear e ective in small
(she wants to be a dermatologist, so she really has to ace her
trials but are not FDA approved, including podo ilox
examinations). Physical examination demonstrates a linear
0.5%, KOH 5% to 10%, and cantharidin.
group o vesicles with erythema on her le chest.

Question 17.4.5 Treatment should be initiated with which


Your next patient is a healthy 18-year-old male who presents o the ollowing therapies:
or treatment o plantar warts. He has tried topical salicylic A) Cephalexin
acid 17%at home. He used it or a ew weeks without signi - B) Mupirocin
cant improvement. C) Paroxetine
D) Valacyclovir
Question 17.4.4 Which o the ollowing can be used or E) No treatment is necessary as this is sel -limited
therapy or plantar warts (verruca plantaris)?
A) Cryotherapy (liquid nitrogen) Answer 17.4.5 The correct answer is “D.” T is is most likely
B) Duct tape occlusion herpes zoster, which is reactivation o the varicella zoster virus.
C) Imiquimod Valacyclovir is an e ective treatment. Even in immunocompe-
D) Candida antigen injection tent patients, herpes zoster can be reactivated rom the dorsal
E) All o the above root ganglion during periods o stress. Although it can be sel -
limited, treatment with antiviral (valacyclovir or acyclovir) is
Answer 17.4.4 The correct answer is “E.” No therapy or cuta- recommended during rst 72 hours to decrease outbreak and
neous warts (plantar or otherwise) has been proven superior, risk o postherpetic neuralgia. With the presentation o pain,
and all o the options can be used with di erent pros and cons. gabapentin or pregabalin is also o en prescribed.
“A,” cryotherapy, is easily per ormed in the o ce and is quick
but may be too pain ul or children to tolerate. Also, cryotherapy Objectives: Did you learn to . . .
may result in hypopigmented scarring in dark-skinned patients. • Recognize that ampicillin/amoxicillin given to patients with
As we all know, duct tape can be used or anything (e.g., hold- mononucleosis will o ten result in a macular rash?
ing your muf er together, preventing poison gas exposure) and • Diagnose pityriasis rosea (PR) and recognize secondary
has been used to treat warts, but the evidence is con icting. “C,” syphilis as a mimic?
imiquimod, is initially painless, can lead to a brisk in ammatory • Recognize and treat molluscum contagiosum?
response, and unlikely to cause scarring, so it is a good choice • Describe the treatment modalities or verruca plantaris?
or acial warts or dark-skinned patients. However, it is the most • Recognize and treat herpes zoster?
CHAPTER 17 • DERMATOLOGY 485

QUICK QUIZ: HYPERSENSITIVITY REACTION

Which o the ollowing represents a type IV hypersensitivity


reaction?
A) Serum sickness reaction
B) Contact dermatitis to poison ivy
C) Urticaria
D) Vasculitis

The correct answer is “B.” Contact dermatitis has a geomet-


ric appearance and a characteristic immune-mediated reaction;
someone may not react when they rst encounter the agent and
then develop stronger in ammatory responses with subsequent
exposures. Contact dermatitis is an example o a delayed-type
hypersensitivity reaction. Serum sickness is a type III immune
complex reaction. Urticaria is a type I hypersensitivity response, FIGURE 17-8.
mediated by mast cell degranulation. Vasculitis can be either
immune complex, antibody, or complement mediated.
demonstrating multinucleated giant cells, viral culture, or direct
uorescent antibody or HSV 1 or 2. Commonly seen in warm
HELPFUL TIP: climates, miliaria (“C”) or “prickly heat” is due to blocked sweat
Auto-eczematization or “id” reactions represent a hy- ducts presenting as nonin amed pinpoint clear di use vesicles.
persensitivity response o the body to a signi icant in- Epidermolysis bullosa (“E”) is a chronic blistering disease pre-
ection or dermatitis in another part o the body. For senting as blisters at sites o mild trauma.
example, patients with severe tinea pedis can develop
eczematous lesions on other parts o the body and
treatment o the tinea pedis will resolve the eczematous HELPFUL TIP:
eruption (or “id” reaction). Neonatal acne presents as a pustular, acial eruption
with a mean onset at 2 to 3 weeks o li e. It is asymptom-
atic and diagnosed clinically. Fungal organisms have
been ound in some pustules. Treatment or acne is not
CASE 17.5 necessary as most lesions resolve in several weeks; i
severe, topical “azole” anti ungals can be used. Besides,
You are rounding in the newborn nursery. A healthy 2-day-
we doubt neonatal acne will a ect the child’s chance o
old term in ant has a new rash characterized by yellow pus-
landing a daycare prom date.
tules on an erythematous base (Fig. 17-8) involving the trunk
and extremities but sparing the palms and soles.

Question 17.5.1 What is the diagnosis? While taking care o the neonate in the newborn nursery,
A) ransient neonatal pustular melanosis you notice a child in isolation. His skin catches your eye
B) Herpes simplex (see Fig. 17-9).
C) Miliaria
D) Erythema toxicum neonatorum Question 17.5.2 The most common cause o this rash in
E) Epidermolysis bullosa newborns in the United States is:
A) Cytomegalovirus (CMV)
Answer 17.5.1 The correct answer is “D.” Erythema toxicum B) Rubella
neonatorum is a common, benign, sel -limited rash seen in C) Langerhans cell histiocytosis
term in ants. Lesions usually appear a er 24 to 48 hours o age. D) Rh incompatibility
Lesions may begin as erythematous macules that progress to yel- E) Parvovirus B19
low pustules on an erythematous base, may be sparse or numer-
ous, and involve the trunk and extremities, sparing the palms Answer 17.5.2 The correct answer is “A.” All the above can
and soles. T e etiology is unknown and the diagnosis is clinical. present as a “blueberry mu n baby”; however, CMV is the most
ransient neonatal pustular melanosis (“A”) usually a ects term common cause in the United States. CMV is the most common
A rican-American in ants presenting at birth with pustules on a congenital viral in ection, a ecting about 1% to 2% o all births.
hyperpigmented macule that may involve the palms and soles. Rubella was the most common cause in the prevaccination era.
Neonatal herpes (“B”) presents with blisters or erosions and an In most cases, the purple plaques represent extramedullary
ill-appearing child. Diagnosis is made via a zanck preparation hematopoiesis.
486 FAMILY MEDICINE EXAMINATION & BOARD REVIEW

Question 17.5.3 The most likely diagnosis is:


A) oxic epidermal necrolysis ( EN)
B) Burns
C) Staphylococcal scalded skin syndrome (SSSS)
D) Di use cutaneous mastocytosis

Answer 17.5.3 The correct answer is “C.” T is is Staphylococ-


cal scaled skin syndrome, which results rom a distant Staphy-
lococcal in ection (o en involving urinary tract, nasopharynx,
eyes, perianal areas). T e key eatures o SSSS in Figure 2-10 are
bright red erythema with impending desquamation in the neck
olds and periori cial areas with sparing o the oral mucosa. So
what di erentiates “C” rom “A?” EN presents with shedding o
sheets o skin and o en has mucosal involvement that separates
it clinically rom SSSS. Both can present with signi cant skin
pain. However, EN is a result o drug hypersensitivity or bacte-
FIGURE 17-9. rial sepsis (in the case o the neonate children); and neonates with
EN are quite ill compared with children with SSSS. In EN, a
biopsy would show ull-thickness necrosis o the epidermis versus
an intraepidermal split in SSSS. Burns (“B”) can mimic SSSS, but
Later that same morning, a previously healthy 2-month- burns would not be likely to cause a di use red eruption and will
old neonate presents to your clinic with peeling o the skin o en have a speci c shape depending on what burned the skin.
and a ever (Fig. 17-10). She had appeared well until 2 days “D,” di use cutaneous mastocytosis, can present with hemor-
ago when she became more irritable. Her mother noted a rhagic blisters and erosions, but these are usually ocal areas, not
decrease in her urine output. T e child was born vaginally sheets o skin loss. Keep reading or more in ormation on SSSS.
without complications and has no known medical condi-
tions. On physical examination, you notice a well-nourished, Question 17.5.4 In this case o SSSS, peeling o the skin is
crying in ant with large sheets o desquamating skin on her a result o :
extremities. Her mucus membranes appear normal. You A) Necrosis o the entire epidermis rom lymphocyte attack
order complete blood count and urine analysis. B) A widespread bacterial skin in ection
C) Histamine released in the skin with edema and blistering
D) oxin-mediated skin blistering rom nonskin source o
in ection

Answer 17.5.4 The correct answer is “D.” T is clinical scenario


describes SSSS, which is an epidermolytic toxin-driven disease.
Extreme tenderness o the skin precedes super cial, widespread
desquamation. T e skin is usually bright red with areas o akiness.
Radial wrinkling o the mouth, giving an “old, sad man” appear-
ance, is common. T e source o in ection is not the skin but rather
an occult site such as nasopharynx, eye, perianal area or urinary
tract; there ore, investigation or a causative in ection should be
undertaken (e.g., blood, nares, ocular, perianal, and urine cultures).
Objectives: Did you learn to . . .
• Recognize neonatal benign cutaneous eruptions?
• Identi y a “blueberry muf n” neonate and know the common
causes?
• Recognize SSSS and consider its di erential diagnosis?

QUICK QUIZ: NAILS

Which o the ollowing nail ndings–systemic disease pairing


is INCORREC ?
A) Muehrcke nails–nephrotic syndrome
B) Nail pitting–psoriasis
FIGURE 17-10. (Courtesy o Drs. Karolyn Wanat and Megan Noe.) C) Periungual broma–tuberous sclerosis
CHAPTER 17 • DERMATOLOGY 487

D) Splinter hemorrhages–in ective endocarditis


E) Koilonychia–systemic lupus erythematosus

The correct answer is “E.” Koilonychia, otherwise known as


“spoon nail,” is a result o so ening and thinning o the nail
plate and is ound in patients with long-standing iron de -
ciency anemia, Plumer–Vinson syndrome, Raynaud disease,
hemochromatosis, and trauma. It can also be inherited as an
autosomal-dominant trait. Connective tissues diseases, such as
lupus, are more commonly characterized by nail old abnormal-
ities, such as nail old telangiectasias, rather than koilonychia.
Muehrcke nails (“A”) have paired narrow horizontal (meaning
perpendicular to the long axis o the nger) white bands, sepa-
rated by normal nail, which remain static as the nail grows. T ey
are most o en seen in patients with nephrotic syndrome, and FIGURE 17-11.
their presence re ects the degree o hypoalbuminemia. When
you see onycholysis (separation o the nail plate rom the nail and has lobulated appearance with a central pore. Squamous
bed that appears opaque), consider trauma, onychomycosis, cell carcinoma is usually a hyperkeratotic or ulcerated papule
psoriasis, and other systemic disease. Nail pitting can be asso- or plaque. Metastases rom solid organ malignancies are subcu-
ciated with several conditions including psoriasis, atopic der- taneous, rm nodules, although they may ulcerate. Merkel cell
matitis, and alopecia areata. A periungual broma (“C”) should carcinoma is a rare tumor on the head and neck that usually
prompt evaluation or tuberous sclerosis with brain imaging or appears as an ill-de ned, violaceous nodule.
tuberous lesions. Splinter hemorrhages (“D”) are usually a result
o trauma but can be a sign o in ectious endocarditis. CASE 17.6
While working in the emergency department, a 6-month-
HELPFUL TIP: old healthy emale in ant is brought in by her grandmother.
Paronychia, in lammation around the nail, may be acute She is bleeding rom her birthmark on her buttocks (see
or chronic. Acute paronychia is o ten due to bacterial Fig. 17-12). Her grandmother watches her during the day
in ection and is treated with oral antibiotics, drainage, while her mother is at work and has noticed the lesion getting
warm compresses, and soaks. Chronic paronychia is larger recently. A er holding pressure on the lesion or about
related to eczema and may have secondary Candida 20 minutes, the bleeding stops. You educate the grandmother
in ection. Treatment is with topical steroids, topical an- about the natural history o the “birthmark.”
ti ungals, and oral anti ungals.
Question 17.6.1 Which o the ollowing is NOT a true state
ment about this type o birthmark?
QUICK QUIZ: DERM PHOTO A) T is is the most common type o so -tissue tumor o in ancy
B) Most o these birthmarks undergo spontaneous involution
A 52-year-old white male presents to clinic with a nonhealing,
asymptomatic “pimple” on the cheek. It has been present or
6 months and has recently started to bleed when he shaves. On
examination, you note his air skin, blue eyes, and ruddy com-
plexion. You nd the lesion in Figure 17-11.

Your preliminary diagnosis is:


A) Sebaceous hyperplasia
B) Squamous cell carcinoma
C) Basal cell carcinoma
D) Metastases rom internal malignancy
E) Merkel cell carcinoma

The correct answer is “C.” T e photograph is a classic picture


o a basal cell carcinoma with its rolled, pearly pink borders,
and telangiectasias. It is ound commonly on sites that get a lot
o sun exposure, especially the head and neck. People with air
skin and light hair and eyes are at particular risk. Sebaceous
hyperplasia can look similar but is ivory or yellow in appearance FIGURE 17-12.
488 FAMILY MEDICINE EXAMINATION & BOARD REVIEW

C) reatment is indicated in all lesions lest they become malig- C) Acne


nant D) Pyogenic granuloma
D) β -Blockers are the most appropriate therapy or ulcerated E) Nodular melanoma
lesions
E) Many are not present at birth but develop shortly therea er Answer 17.6.2 The correct answer is “D.” T is patient most
likely has a pyogenic granuloma, which is essentially a nodu-
Answer 17.6.1 The correct answer is “C.” T e child has an lar hemangioma arising at sites o trauma, especially the ngers
ulcerated hemangioma o in ancy. In antile hemangiomas are and toes. Although pyogenic granuloma is not cancerous, it can
benign proli erations o endothelial cell lineage that are usually be con used with basal and squamous cell carcinomas as well as
present at birth or soon therea er. T ey are characterized by melanoma. For this reason, it is prudent to per orm excision
rapid growth in the rst several months o li e with a plateau and histologic examination. A er excision, the base o the
occurring around 1 year o age. T ey spontaneously involute lesion should be ablated (electrocautery or laser), or the pyo-
over years. reatment options should be individually tailored, genic granuloma may return. Pyogenic granulomas are more
depending on the size and location o the lesion. Lesions located common during pregnancy and have been associated with
on the central ace or in the diaper area should be treated, as drugs such as oral retinoids, indivair, EGFR inhibitors, metho-
should ulcerated hemangiomas at any body site. opical or oral trexate, sirolimus, and capecitabine. “C,” acne, should not be a
β -blockers are the standard o care or in antile hemangiomas consideration, as sebaceous glands are not ound on the palms.
that require treatment. Larger, segmental hemangiomas should By the way, did you notice that this poor child has developed
be urther evaluated with imaging studies to rule out syndromes coarse, dark hair growth on her knuckles? As it turns out, she’s
with other associated anomalies such as PHACES syndrome been playing with dad’s Rogaine.
(posterior ossa mal ormation, hemangioma, arterial anoma-
lies, cardiac de ects, coarctation o the aorta, eye abnormalities, Objectives: Did you learn to . . .
sternal de ects/supraumbilical raphe) and LUMBAR syndrome • Describe the natural history o in antile hemangiomas?
(lower body hemangioma, urogenital anomalies, myelopathy, • Identi y options or treating ulcerated hemangiomas?
bony de ormities, anorectal mal ormation/arterial anomalies, • Describe the eatures and management o pyogenic
and renal anomalies). granuloma?

T e same patient returns, this time with mom, at age 5. T e CASE 17.7
hemangioma has involuted. About 2 months ago, the mother
noticed a spot on the patient’s le index nger, and now she A 78-year-old emale presents with a rash on her legs. It
is concerned that it is not resolving. T e patient is not both- started 1 week ago and was preceded by itching o the legs
ered by the lesion, except that it occasionally bleeds when she without any visible changes. T en red blotches developed
bumps it on something. On examination, you nd a 3-mm that quickly became blisters. She denies pain or drainage. She
dome-shaped, bright red nodule on the palmar aspect o the also has some sores in her mouth. She reports overall good
le index nger. It is nontender and smooth (see Fig. 17-13). health and takes no medications. On examination, you nd
an a ebrile, com ortable looking emale. She has 1+ pitting
Question 17.6.2 The most likely diagnosis is: edema at her ankles. You notice round, tense bullae erupt-
A) Basal cell carcinoma ing on erythematous patches on the abdomen and extremi-
B) Squamous cell carcinoma ties. T ere are also scattered erosions present at the sites o
previous bullae. T e bullae contain clear uid. T ere is no
purulent material or blood (see Fig. 17-14).

Question 17.7.1 Which o the ollowing is the most likely


diagnosis?
A) Pemphigus vulgaris
B) Bullous pemphigoid
C) Varicella zoster
D) Dermatitis herpeti ormis
E) Stevens–Johnson syndrome (SJS)

Answer 17.7.1 The correct answer is “B.” Bullous pemphi-


goid is an autoimmune disorder and is primarily a disease o
the elderly. O en patients will report pruritus or urticaria-like
lesions, prior to the eruption o typical bullous lesions. When
the bullae orm, they are requently asymptomatic but can be
intensely pruritic and/or mildly tender. In contrast to pemphi-
FIGURE 17-13. gus vulgaris (“A”) and SJS (“E”), systemic symptoms are not
CHAPTER 17 • DERMATOLOGY 489

ogy o a subepidermal blister with eosinophils. For a de nitive


diagnosis, both “A” and “C” should be done.

You assure her that the lesions are not cancerous.

Question 17.7.3 What else can you tell her about the prog
nosis o bullous pemphigoid?
A) T e average course resolves spontaneously in 5 to 6 years
B) It is associated with an increased risk o systemic malig-
nancy
C) It is associated with a high mortality rate
D) It will likely spread to cover her entire body until she’s one
big blister, and her hair will all out, and her riends will
shun her, a la Job in the Old estament

Answer 17.7.3 The correct answer is “A.” Bullous pemphigoid


may undergo complete resolution and never recur. Alterna-
tively, the disease may have resolutions and exacerbations over
time. It is not associated with an increased risk o cancer or a
high mortality rate. Very extensive involvement may lead to skin
in ections and bacteremia, but di use disease is not common.

She seems miserable. You want to o er her the medication


with the astest onset o action.
FIGURE 17-14. (Courtesy o Drs. Karolyn Wanat and Megan Noe.)
Question 17.7.4 What is the most appropriate treatment
option with a ast onset o action?
part o bullous pemphigoid. Additionally, pemphigus vulgaris A) Oral azathioprine
lesions are pain ul, and classically the bullae are accid rather B) Oral cephalexin
than tense. SJS generally occurs in response to medications, and C) Oral prednisone
this patient is taking none. Varicella zoster (“C”) should ollow a D) opical antimicrobials
dermatomal pattern rather than appear bilaterally; zoster is usu-
ally associated with pain and paresthesia. Dermatitis herpeti or- Answer 17.7.4 The correct answer is “C.” Bullous pemphigoid
mis (“D”) presents with crops o vesicles and excoriations rather is an autoimmune disease, so anti-in ammatory medications
than bullae. and immunosuppressive agents are used to treat it. Oral steroids
are the rst-line treatment because o the quick onset o action;
Although she eels well, the lesions are dramatic, and the however because o concerns or long-term side e ects patients
patient is very concerned that she might have cancer. o con- should be transitioned to “steroid-sparing” agents as neces-
rm your diagnosis, you decide to biopsy the skin. sary. Once the disease is under control, oral steroids should be
tapered to the lowest e ect dose in combination with a tetracy-
Question 17.7.2 The most appropriate way to per orm a cline antibiotic or other immunosuppressive agent such as aza-
biopsy in this situation is: thioprine, methotrexate, or mycophenolate. For patients with
A) 4-mm punch biopsy o the margin o an intact bulla or light limited disease, high-potency topical steroids can also be used.
microscopy opical antimicrobials are used when treating in ected, open
B) 4-mm punch biopsy o perilesional normal skin or light bullae but are not e ective or the treatment o the disease itsel .
microscopy
C) 4-mm punch biopsy o perilesional normal skin or immu- Objectives: Did you learn to . . .
no uorescence • Distinguish bullous diseases rom one another?
D) A and B • Diagnose and treat bullous pemphigoid?
E) A and C

Answer 17.7.2 The correct answer is “E.” Bullous pemphigoid QUICK QUIZ: PAPULAR ERUPTION
is diagnosed de nitively by demonstration o autoantibod-
ies deposited along the basement membrane o normal skin. A 30-year-old emale presents with a several month history
Immuno uorescence examination o normal-appearing skin o bumps on her chin and around her mouth. Although she
can demonstrate linear depositions o IgG and C3. A biopsy o has never had problems with acne, she thought that she was
the edge o an intact bulla will show the characteristic pathol- developing acne and tried benzoyl peroxide. A month ago, she
490 FAMILY MEDICINE EXAMINATION & BOARD REVIEW

C) It may rst appear in adulthood


D) It is more severe in emales than in males
E) T e incidence is lower in Asians and A ricans than in
Caucasians

Answer 17.8.1 The correct answer is “D.” Acne is an in am-


matory process that involves the pilosebaceous units o the ace
and trunk. It can be comedonal, papulopustular, or nodulocys-
tic in presentation. It typically presents rst in adolescence with
girls developing it rom age 10 to 17 years and boys rom 14 to
19 years. However, it may not appear until early adulthood. It is
more severe in males than in emales (thus “D” is not true), and
it is less prevalent among Asians and A ricans.

HELPFUL TIP:
Open comedones (blackheads) and closed comedones
(whiteheads) are considered nonin lammatory lesions,
whereas papular and cystic lesions are considered in lam-
FIGURE 17-15. matory. And no matter how tempting, patients should
not pick at them! Picking may exacerbate scarring.
stopped the benzoyl peroxide because it did not seem to work,
and she switched to hydrocortisone cream, which has worsened
the outbreak. T e rash is neither itchy nor pain ul. She works in Your patient has heard many things about acne and wonders
an o ce and cannot recall any contact irritants. On examina- i they are true.
tion, you nd erythematous papules with small pustules on the
chin and laterally around her mouth (see Fig. 17-15). T e neck Question 17.8.2 Regarding acne, all o the ollowing are
and the remainder o the ace are not involved. true EXCEPT:
A) Acne is not caused by medications
Which o the ollowing treatments do you recommend? B) Acne can be caused by industrial exposures such as cutting
A) Oral tetracycline oils, aromatic hydrocarbons, and petroleum-based products
B) Oral isotretinoin C) In requent acial cleansing does not cause acne
C) opical high-potency steroids D) Emotional stress a ects the course o acne
D) opical triple antibiotic ointment E) Acne can present at any age
E) opical retinoic acid
Answer 17.8.2 The correct answer is “A.” Acne can be caused
The correct answer is “A.” T is patient appears to have perioral by many medications including oral steroids, EGFR inhibitors,
dermatitis. Appropriate treatments or perioral dermatitis include lithium, isoniazid (INH), and phenytoin. Medication-induced
topical metronidazole or topical erythromycin or oral tetracy- acne presents as monomorphic papules on the ace, chest, and
cline antibiotics (tetracycline, minocycline, or doxycycline). T e back. Exposure to industrial chemicals that occlude the olli-
etiology o perioral dermatitis is unknown. “B” and “E” are incor- cle can also lead to an acnei orm eruption. Neither in requent
rect, since these are treatments o acne vulgaris. “C” is incor- washing nor chocolate (or any other oods) cause acne—no
rect, since high-potency topical steroids can make the condition matter what your mother told you. However, stress (like that
worse, with rebound erythema upon discontinuation. opical homecoming dance in high-school) can o en are acne.
triple antibiotic ointment (“D”) is not likely to be e ective. T ere are di erent types o acne that can a ect individuals at
various stages o li e. Neonatal acne presents shortly a er birth
and is transient. In antile acne presents at 6 to 12 months o
CASE 17.8 age and resolves within 1 to 2 years. T e most common time
A 15-year-old male presents to your of ce complaining o to develop acne is around puberty; however, women in their
“zits” on his ace and back. He has several scattered comedo- 30s and 40s can also experience acne ares with hormonal uc-
nes on his ace and several deep nodules on his back. He also tuations. As an aside, oral contraceptives (OCPs) go both ways
has some papulopustular lesions on his chin. You diagnose with regard to acne: several OCPs are marketed to treat acne,
him with acne vulgaris. but they have been reported to worsen acne in some patients.
T e result o OCPs may depend on the hormonal milieu o the
Question 17.8.1 Which o the ollowing is NOT TRUE about speci c patient (women with hyperandrogenism may bene t)
acne? and the type o progestin in the OCP. Acne is o en improved
A) Noncomedonal acne is an in ammatory process in the summer months and worse in the all and winter. Many
B) It commonly presents rst in adolescence women also tend to are up right be ore menses.
CHAPTER 17 • DERMATOLOGY 491

Question 17.8.3 For acne, which o the ollowing treatment one-third o patients who receive isotretinoin will require
principles is correct? additional courses during their li etime. Isotretinoin has been
A) reatment can o en lead to initial worsening o lesions reported to cause premature closure o growth plates rarely.
B) T erapeutic response o en takes several months
C) Mild acne can be treated with topical medications
HELPFUL TIP:
D) O en times, two or more therapeutic agents must be com-
Retinoids (isotretinoin and topicals) are related to vita-
bined or e ective treatment
min A, so vitamin A supplements should be avoided as
E) All o the above are correct
the combination may lead to vitamin A toxicity.

Answer 17.8.3 The correct answer is “E.” opical retinoids,


topical antibiotics, and benzoyl peroxide are the rst-line agents
HELPFUL TIP:
and can be titrated as needed. Retinoids should be applied at
Women should have a pregnancy test monthly while
bedtime. T e main oral antibiotic agents are tetracyclines and
on isotretinoin and should continue contraception or
should be used as add-on therapy to topical agents or mod-
1 month a ter discontinuing treatment.
erate-to-severe in ammatory acne. OCPs with low androgenic
progesterones can also be help ul. Since acne lesions may take
up to 2 months to resolve, one may not notice a therapeutic Objectives: Did you learn to . . .
response or several months (once the current crop o lesions • Diagnose and treat acne?
resolve). • Identi y side e ects o isotretinoin?

A er a ew months o topical benzoyl peroxide wash and


topical clindamycin cream, you add topical retinoic acid.
CASE 17.9
Your patient returns with persistent lesions, including mild A 26-year-old emale patient presents to your of ce with
scarring and cystic lesions. You are considering re erral or pain ul, nonpruritic pink bumps that started last week on
isotretinoin, which is indicated or severe, recalcitrant, nod- her shins. You nd the rm lesions easier to palpate than to
ular acne. visualize (see Fig. 17-16).

Question 17.9.1 What is the most likely diagnosis in this


HELPFUL TIP: patient?
Topical tretinoin (Retin-A) is most e ective or comedo- A) Erythema multi orme
nal acne. B) Urticaria

Question 17.8.4 Which o the ollowing is FALSE regarding


the use o isotretinoin?
A) wo methods o contraception are required in all reproduc-
tive age women along with requent pregnancy tests
B) It should not be used with tetracycline
C) Hypertriglyceridemia is a possible side e ect
D) Patient should be monitored closely or changes in mood
E) A patient can only receive one course o isotretinoin

Answer 17.8.4 The correct answer is “E.” Isotretinoin is an oral


retinoid and is pregnancy category X. Because o this, the FDA
has developed a mandatory patient registry called “iPLEDGE.”
All patients must enroll in his registry be ore receiving the
medication and prescribers also need to be registered. Female
patients o reproductive age are required to use two contracep-
tive methods and have monthly pregnancy tests. As both tetra-
cycline and isotretinoin can lead to pseudotumor cerebri, they
should not be used together. Hypertriglyceridemia, dry skin,
and decreased night vision are common side e ects. T ere have
been reports in the popular press and the scienti c literature
o suicide in patients on isotretinoin, though there is no good
controlled data to suggest that the drug is responsible. Patients
with a history o mental health issues should be ollowed care-
ully or any changes in their mood or behavior. Approximately FIGURE 17-16.
492 FAMILY MEDICINE EXAMINATION & BOARD REVIEW

C) Erythema nodosum to help with the pain. For patients with severe pain ul disease,
D) Erythema migrans systemic steroids can be considered.
E) Lumpy shin disease (aka, “ ootballer’s disease” in England)
Objectives: Did you learn to . . .
• Recognize erythema nodosum?
Answer 17.9.1 The correct answer is “C.” T is is erythema
• Describe some o the causes and natural history o erythema
nodosum. ypically, these rm nodules occur rst on shins
nodosum?
and are o en more easily palpated than seen. T ey may then
spread to the thighs, trunk, and extensor sur aces o the arms.
Erythema multi orme (“A”) presents as erythematous, targetoid QUICK QUIZ: ERYTHEMA MULTIFORME
lesions usually a ecting the distal extremities including palms
and soles. Urticaria (“B”) is not tender but would be intensely
Which o the ollowing cause erythema multi orme?
pruritic. Erythema migrans (“D”) is the quintessential lesion o
A) Viral upper respiratory in ection
Lyme disease: a nontender, nonpruritic red plaque with a cen-
B) Cimetidine
tral clearing. While “E” is technically correct in its description,
C) Herpes simplex outbreak
we know o no such disease entity.
D) Ni edipine
E) All o the above
HELPFUL TIP:
Erythema migrans is o ten thought o as a single “bull’s The correct answer is “E.” All o the above are causes o ery-
eye” lesion in the area o the tick attachment. However, thema multi orme as are many, many other medications, etc.
the “bull’s eye” is present in a minority o cases, and ery- T e most common precipitator o erythema multi orme is
thema migrans can have multiple widespread lesions in herpes simplex. Look or this association in your patients.
some patients with Lyme.
HELPFUL TIP:
Sunlight is a requent precipitator o an outbreak o her-
Question 17.9.2 Which o the ollowing can cause erythema pes labialis.
nodosum?
A) OCPs
B) Streptococcal pharyngitis
C) Sarcoid
CASE 17.10
D) Viral upper respiratory in ection A patient presents to your of ce with a 1-week history o a
E) All o the above pruritic rash that comes and goes. No lesion is present or
more than 24 hours. It involves his entire body except or his
Answer 17.9.2 The correct answer is “E.” All o these entities ace. He cannot remember any new products with which he
may cause erythema nodosum, but viral URI and streptococcal has been in contact (soaps, detergents, etc.). He is quite con-
pharyngitis are most common. Multiple medications can also cerned. You correctly diagnose urticaria.
be responsible.
Question 17.10.1 Your next step is:
T is patient does relate a history o URI symptoms about A) Skin testing or various commercial products
1 week prior to the onset o her symptoms. B) Viral titers or CMV, EBV, etc.
C) RAS test or common allergens
Question 17.9.3 What advice are you going to give this D) Recommend no urther evaluation.
patient? E) Prescribe an epinephrine injection or emergency use at
A) T e rash tends to be sel -limiting and will resolve in 4 to home
6 weeks
B) T e rash will heal with scarring Answer 17.10.1 The correct answer is “D.” No workup is
C) T e rash will likely spread to the ace and upper extremities needed at this time. Urticaria should be classi ed as “acute”
be ore it resolves (< 6 weeks) or “chronic” (> 6 weeks). Acute urticaria requires
D) T is is a chronic disease and no treatment is available no urther workup and should be treated symptomatically
with antihistamines. In patients with chronic urticaria, a more
Answer 17.9.3 The correct answer is “A.” In general, erythema detailed history should be taken looking or other symptoms
nodosum heals spontaneously in 3 to 6 weeks without scarring. or signs o in ection, with laboratory testing as appropriate. In
T ere are treatment options available i the patient is symp- act, beyond a good history, an extensive workup is pretty much
tomatic. I erythema nodosum is caused by an in ection, that utile. It is almost impossible to identi y a cause o urticaria by
in ection should be treated appropriately. I it was caused by a laboratory testing. “E,” an epinephrine injector, is not indicated
medication, such as an oral contraceptive pill, it should be dis- or the treatment o urticaria but would be appropriate i this
continued. Nonsteroidal anti-in ammatory agents can be used patient has experienced an anaphylactic reaction.
CHAPTER 17 • DERMATOLOGY 493

TABLE 17-2 CAUSES OF URTICARIA: A PARTIAL LIST


Medications (Direct Mast
Physical Urticaria Allergic Systemic Cell Degranulators)
• Pressure • Foods (nuts, sh) • Malignancy • NSAIDs
• Water • Insect stings • SLE • Aspirin
• Vibration • Drugs (IgE • RA • Opiates
• Cold mediated) • Chronic Hep B and C • ACE Inhibitors
• Sunlight • EBV • Contrast dye

Question 17.10.2 Urticaria is categorized as which o the Answer 17.10.4 The correct answer is “E.” All o the above can
ollowing? cause urticaria. In act, these are not uncommon causes and
A) ype I hypersensitivity reaction can be identi ed by history. Patients may develop urticaria with
B) ype II hypersensitivity reaction exercise and sweating (cholinergic urticaria), cold (during the
C) ype III hypersensitivity reaction winter), and pressure (e.g., walking). O particular note is water
D) ype IV hypersensitivity reaction urticaria that occurs with water contact, including bathing and
E) None o the above showering. See able 17-2 or a more complete listing.

Answer 17.10.2 The correct answer is “A.” Urticaria is a clini-


cal eature o a type I reaction. Other clinical presentations HELPFUL TIP:
o type I reactions include anaphylaxis and angioedema. See Cold urticaria o ten starts with lesions appearing on ex-
Chapter 4 or a detailed answer on hypersensitivity reactions. posed skin, but may be worsened with rewarming.

Question 17.10.3 You decide to provide symptomatic care


or this patient. Appropriate medications include which o
You decide that this patient probably has cold urticaria (the
the ollowing?
act that it is summer does not dissuade you as it may be
A) Ranitidine
related to all o that air conditioning!).
B) Doxepin
C) Diphenhydramine Question 17.10.5 The next drug you might want to try on
D) Cetirizine this patient is:
E) All o the above A) Cyproheptadine (Periactin)
B) Prednisone
Answer 17.10.3 The correct answer is “E.” All o the above can C) Montelukast
be use ul in the symptomatic treatment o urticaria. Patients D) Ni edipine
should be started on sedating and nonsedating anti-histamines E) Aspirin
to control their symptoms. Doses greater than the recommended
daily dose are requently required. H 2-blockers are e ective in Answer 17.10.5 The correct answer is “A.” T e physical urti-
the 10% to 15% o patients who do not respond to H 1-blockers. carias (cold and pressure especially) may respond better to
O note, H 2-blockers also are used in anaphylaxis treatment but cyproheptadine than other modalities. I this patient had a
the evidence is weak. Finally, doxepin is a particularly e ective “typical” urticaria, you might want to try prednisone, one o
H 1- and H 2-blocker that can be used as a sedating antihistamine. the leukotriene inhibitors or ni edipine (which inter eres with
mast cell degranulation). Remember that leukotriene inhibi-
T e patient returns to see you 6 weeks later and is still hav- tors, steroids, etc. are second line and should be used only
ing symptoms. You are wondering a bit more about potential when rst-line drugs have ailed or are not tolerated (cypro-
causes o this un ortunate individual’s urticaria. heptadine or physical urticaria; doxepin, anti-histamines, etc.,
or “typical” urticaria). T ere are no good studies on the e ec-
tiveness o leukotriene inhibitors (anecdotal evidence only), but
Question 17.10.4 Which o the ollowing are causes o they might be worth trying when all else ails.
urticaria?
A) Sweating HELPFUL TIP:
B) Cold (OK, get over it. Move to Hawaii and quit whining!) Any number o drugs can cause urticaria. Look at the
C) Water (So that is why you never shower and only drink med list! Medications that act as mast cell degranula-
beer?) tors should be avoided in patients with chronic urticar-
D) Pressure (OK, get over it. Use an antigravity unit and quit ia, including aspirin, opiates, polymyxin B, NSAIDs, and
whining!) systemic lidocaine.
E) All o the above
494 FAMILY MEDICINE EXAMINATION & BOARD REVIEW

Objectives: Did you learn to . . .


• Recognize urticaria?
• Identi y potential causes o urticaria?
• Develop a treatment strategy or patients with urticaria?

QUICK QUIZ: PRECANCEROUS LESIONS

Actinic keratoses (AKs) are precursors to what?


A) Nothing
B) Melanoma
C) Basal cell carcinoma
D) Squamous cell carcinoma
E) Granuloma annulare

The correct answer is “D.” AKs are precursors to squamous cell


carcinomas. As such, they should be treated. Options include FIGURE 17-17. (Courtesy o Drs. Karolyn Wanat and Megan Noe.)
cryotherapy, laser therapy, or 5- uorouracil topically.

HELPFUL TIP:
C) Increase her insulin dose
Less than 1% o AKs progress to squamous cell carci-
D) Liberal use o emollients (e.g., petrolatum)
noma per year. Some spontaneously involute and some
E) Leg elevation and application o compressive stockings
remain as AKs. However, in an individual, the risk o an
AK developing into a squamous cell carcinoma increas-
Answer 17.11.1 The correct answer is “A.” T is patient has
es with the number o AKs present.
developed necrobiosis lipoidica, a benign condition o the skin
a ecting a small percent o diabetics, usually those on insulin.
Look or brownish red patches or plaques with yellowish areas
QUICK QUIZ: SKIN BARNACLES through the center. T e center is o en shiny with telangiectasias.
T e legs are most o en involved and the lesions may be pain-
ul. T e name describes the pathology: necrobiosis re ers to the
What is the appropriate treatment or an asymptomatic sebor-
in ammation around destroyed collagen, and lipoidica re ers to
rheic keratosis?
the yellowish color associated with lipid deposits. It does not
A) No treatment necessary
seem to be caused by insulin or a ected by glucose control, so
B) Cryotherapy
“B” and “C” are not appropriate choices. T ese lesions can be
C) Surgical excision
con used or eczema, and in act may respond to topical steroids
D) opical steroids
(“A”); however, emollients are not use ul. Finally, necrobiosis
E) opical antibiotic ointment
lipoidica might be con used with the skin changes associated
with venous stasis and chronic edema, which would be treated
The correct answer is “A.” Seborrheic keratoses are benign
as in “E.” Patients should know that necrobiosis lipoidica is
lesions that do not develop into malignancies. T ey are the
o en chronic and di cult to treat but benign. T e diagnosis is
greasy-looking, stuck-on growths that occur with age. T ey can
clinical but can be con rmed by biopsy, and treatment is not
be treated, i symptomatic, with cryotherapy. T ey look ugly,
always necessary.
patients hate them, but they are harmless.
Your patient also complains o thick, dark, velvety patches
CASE 17.11 under her arms (Fig. 17-18). She’s unsure o the duration,
A 50-year-old emale with type 2 diabetes controlled with but states, “T ey’ve been with me a while. I just wondered i
insulin complains o a rash that has developed on her legs over my medication might cause these.” She gives you an accusing
the past year. It started as a small patch on her le leg and then look, and you have to admit that she takes more than a ew
“spread” to her right leg. It is neither pain ul nor pruritic. You medicines related to her diabetes.
are impressed by the rash she shows you in Figure 17-17.
Question 17.11.2 Which medication provides a clue to the
Question 17.11.1 The most appropriate next step in the diagnosis?
management o these lesions is which o the ollowing? A) Aspirin
A) A topical mid-potency steroid under occlusion or intral- B) Lisinopril
esional steroids C) Insulin
B) Discontinue insulin D) Simvastatin
CHAPTER 17 • DERMATOLOGY 495

C) Per orm mono lament testing to check sensation in the oot


D) Debride the wound
E) Obtain ankle–brachial indices

Answer 17.11.3 The correct answer is “E.” T e described


wound is most likely a diabetic oot ulcer, but the location and
the patient’s history o diabetes do elicit concern or an ulcer
related to vascular disease. Additionally, good vascular sup-
ply to the oot is necessary or wound healing. T ere ore, you
need to know the status o blood ow to the oot. T is may be
accomplished by physical examination. For patients at high risk
or peripheral vascular disease or without strong pulses, ankle–
brachial indices would be the next step. Culture o an open oot
ulcer is pretty much worthless as it will likely return as polymi-
crobial. X-ray may be warranted as well as probing the lesion to
see i it reaches the bone in order to evaluate or osteomyelitis,
but MRI is jumping the gun. Knowledge o a patient’s baseline
sensation is important and good general oot care (shoes that
t, meticulous skin and nail acre) can help in the prevention o
uture ulcers. Wound debridement will help to remove necrotic
tissue and improve the speed o wound healing. T e bio lm
must be removed down to healthy tissue in order to maximize
healing.

FIGURE 17-18. Wol K, Johnson RA, Suurmond D. Fitzpatrick’s Color Atlas Ankle–brachial indices in both legs are normal. T e rest o
& Synopsis of Clinical Dermatology. 5th ed. New York: McGraw-Hill; 2005, her skin is in good condition. T rough her diligent care and
Fig. 5–1. Copyright © The McGraw-Hill Companies, Inc. All rights reserved.
control o her diabetes, the ulcer heals. She returns 3 months
later with a new skin concern. On her anterior shin, she has
Answer 17.11.2 The correct answer is “C.” T e lesions are typi-
developed a clear uid- lled blister about 1 cm in diameter
cal o acanthosis nigricans, a hyperkeratotic, hyperpigmented
and irregular in shape. T ere is no erythema, no pruritus,
condition a ecting skin olds (neck, inguinal area, axilla, etc.).
and no pain. She denies trauma and new environmental
T e common causes o acanthosis nigricans are obesity, insu-
contacts.
lin resistance, and diabetes. From med school, we all remember
acanthosis nigricans as a cutaneous mani estation o internal
malignancy. But such a presentation is rare. Cancer is more Question 17.11.4 Which o the ollowing is the most likely
likely in patients with extensive and quickly progressing lesions, diagnosis?
and as you might expect, it’s not a good sign. A) Dyshidrotic eczema
B) Contact dermatitis
HELPFUL (ISH) TIP: C) Staphylococcal scalded skin syndrome (SSSS)
Another cutaneous mani estation o internal malignancy D) Bullosis diabeticorum
is the sign o Leser–Trelat: a large crop o seborrheic E) Drug eruption
keratoses that erupt quickly. Note that seborrheic kera-
toses themselves are benign and that knowing about Answer 17.11.4 The correct answer is “D.” Bullosis diabetico-
Leser–Trelat is mostly good or looking smart. rum, or bullous disease o diabetes, occurs in less than 1% o
diabetic patients. However, patients may be alarmed by it and
seek treatment. T e cause is unknown, but it typically ollows
Your patient returns a year later and the lesions on her legs a benign course and resolves spontaneously over a ew weeks
have essentially disappeared. Now she has a new concern. She or months. Because it requires no intervention, it is use ul to
reports a sore on the bottom o her great toe. She’s uncertain distinguish bullosis diabeticorum rom dyshidrotic eczema and
how it occurred and thinks it has only been present or a ew contact dermatitis, both o which may mimic the disease except
days. You nd a 1-cm circular ulcer at the plantar aspect o or pruritus and in ammation.
her great toe. Pulses are diminished.
Objectives: Did you learn to . . .
Question 17.11.3 Which o the ollowing should be the next • Identi y necrobiosis lipoidica, acanthosis nigricans, and
step in treatment? bullosis diabeticorum?
A) Culture the wound • Treat these conditions (i necessary)?
B) Per orm an MRI o the oot • Evaluate a diabetic oot ulcer?
496 FAMILY MEDICINE EXAMINATION & BOARD REVIEW

QUICK QUIZ: MR. SCRATCHER

A 76-year-old male comes to your clinic with a 1-year his-


tory o pruritic, eczematous rash on his chest and back. He
has no atopic history or eczema as a child. He has no other
active medical issues. He has tried moisturizers and over-the-
counter hydrocortisone cream, which have provided minimal
relie .

Which o the ollowing is the most appropriate next step in his


management?
A) acrolimus ointment
B) Skin scraping or KOH preparation
C) High-dose topical steroids
D) Punch biopsy
FIGURE 17-19. (Courtesy o Drs. Karolyn Wanat and Megan Noe.) E) Oral antihistamines

The correct answer is “D.” T e development o a new, eczema-


tous rash in an adult patient should raise the concern or a more
QUICK QUIZ: SKIN SO ITCHY serious condition. It is rare or an elderly person to develop
atopic dermatitis de novo. T is may be indicative o an under-
lying lymphoproli erative malignancy such as cutaneous -cell
A 60-year-old male who underwent pacemaker placement
lymphoma. Bowen disease (squamous cell carcinoma in situ)
3 days ago comes in with the pruritic rash seen in Figure 17-19.
is another possibility. T e most appropriate next step in this
patient’s management would be punch biopsy.
T is rash is most likely:
A) Grover disease
B) Lichen planus QUICK QUIZ: DARIER SIGN
C) Contact dermatitis
D) Atopic dermatitis
A 4-year-old girl comes to clinic accompanied by her mother
E) Pyoderma gangrenosum
or her yearly checkup. Her mother requests that you exam-
ine her daughter’s skin because o a rash. Upon examination,
The correct answer is “C.” T is is contact dermatitis, which
you identi y scattered reddish-brown macules on the back and
may be acute, subacute, or chronic. It is an erythematous,
chest. When stroked, the lesions urticate. T ere are no other
pruritic eruption that usually blisters and leaves a crust. More
complaints or abnormalities on physical examination.
chronic orms o contact dermatitis present with licheni cation
and scaling. T e rash in Figure 17-19 does not look like any o
What is the most likely diagnosis?
the other choices. Grover disease, also known as transient acan-
A) Atopic dermatitis
tholytic dermatosis, presents with small, sometimes pruritic,
B) Contact dermatitis
erythematous papules on the back and chest, which may blister.
C) Congenital nevi
Lichen planus presents with pruritic, red-to-purple, at-topped
D) Cutaneous lupus
papules. Atopic dermatitis is discussed earlier in the chapter.
E) Urticaria pigmentosa
Pyoderma gangrenosum presents with pain ul, red nodules and
pustules that ulcerate. Again, since we cannot display pictures
The correct answer is “E.” T is child’s rash is most likely sec-
o all diseases mentioned, please look at one o the websites
ondary to urticaria pigmentosa, which is a cutaneous orm o
included at the beginning o this chapter.
mastocytosis. Clinical ndings include reddish-brown macules
or slightly raised papules, which represent cutaneous accumula-
HELPFUL TIP:
tions o mast cells. T ese occur most commonly on extensor
Pyoderma gangrenosum is a diagnosis o exclusion
sur aces, thorax, and abdomen. When stroked, these lesions
but may be di erentiated rom other ulcer types by
can urticate (Darier sign). Urticaria pigmentosa is the most
the exquisite pain it causes and the pathergic phe-
common skin mani estation o mastocytosis in children and
nomenon it displays, where the wound will worsen
adults. Eighty percent o cases appear during the rst year o
in response to surgical manipulation (e.g., biopsy,
li e. Lesions resolve in > 50% o patients by adolescence. T e
debridement).
diagnosis is con rmed by skin biopsy. In children, urticaria pig-
mentosa is rarely associated with systemic disease.
CHAPTER 17 • DERMATOLOGY 497

BIBLIOGRAPHY
Clinical Pearls Bruckner AL, Frieden IJ. Hemangiomas o in ancy. J Am Acad
Be sure to look or abnormal looking skin lesions on the Derm. 2003;48:477–493.
palms, soles o the eet and near the nails as these lesions Di Lorenzo G, et al. Leukotriene receptor antagonists in mono-
may progress to more aggressive orms o melanoma be ore therapy or in combination with antihistamines in the
they are discovered. treatment o chronic urticaria: A systematic review.
Con irm a toenail ungal in ection prior to initiating systemic J Asthma Allergy. 2008;2:9–16.
anti ungal therapy. Gershenwald JE, et al. 2010 NM staging system or cutaneous
Do not per orm a shave biopsy on a skin lesion i you are
melanoma . . . and beyond. Ann Surg Oncol. 2010;17:1475–
suspecting melanoma as depth o the lesion is used to stage
1477.
melanoma and determine prognosis. Hurwitz S, eds. Eczemetous eruptions in childhood. In: Clinical
Pediatric Dermatology. 3rd ed. Philadelphia, PA: W.B.
Do not routinely recommend diuretics as general therapy Saunders Co.; 2005.
or venous stasis as they may make symptoms worse by
decreasing circulating volume and increase renal retention o
Khumalo N, et al. Interventions or bullous pemphigoid.
sodium. Instead, recommend weight loss, compression
Cochrane Database Syst Rev. 2003;(3):CD002292.
stockings, and leg elevation. O’Connor NR, McLaughlin MR. Neonatal skin: Parts I and II.
Am Fam Physician. 2008;77:47–60.
Do not use systemic antibiotics or irst-line treatment o
acne. Instead, employ topical agents as irst-line therapy.
Rager EL. Cutaneous melanoma. Am Fam Physician. 2005;72:
269–276.
Do not use topical anti ungal agents to treat ungal nail Ramsey HM, et al. Factors associated with nonmelanoma skin
in ections due to these agents being relatively expensive, cancer ollowing renal transplantation. J Am Acad
having marginal bene it, and a high recurrence rate. Dermatol. 2003;49(3):397–406.
Recommend regular sun protection using sunscreen, light Usatine RP, et al. T e Color Atlas of Family Medicine. Part 13:
protective long sleeved clothing, hat, and sunglasses. Dermatology. New York: McGraw-Hill; 2009.
Recommend timely removal/biopsy o any suspicious looking Wol K, Johnson RA. Fitzpatrick’s Color Atlas and Synopsis of
nevi. Clinical Dermatology. 6th ed. New York: McGraw-Hill;
Re er in ants with larger, segmental hemangiomas to 2009.
dermatology and obtain imaging studies to rule out
syndromes with other associated anomalies.
Neurology
Sp iha Pa i
18
CASE 18.1 HELPFUL TIP:
Wh n a pati nt c mp ains dizzin ss, ask what th pa-
A 75-year-old right-handed man presents to the emergency ti nt m ans b “dizzin ss.” Th t m “dizzin ss” m ans
department (ED) with dizziness that started 5 hours ago. His di nt thin s t di nt pati nts. “Dizzin ss” ma
dizziness consists o a spinning sensation and he eels o bal- p s nt ti , i hth ad dn ss, p -s nc p , dis-
ance as he walks. He reports coughing when trying to drink q i ib i m wh n wa kin ( . ., a in t n sid ),
water earlier, but otherwise notes no other symptoms. He anxi t , tc. A s , kn w that th is c nt s am n
has an unremarkable past medical history and is taking no xp ts as t wh th this d in ati n dizzin ss is
medications. On examination, he is ound to have le -sided m mis adin than it is h p .
ptosis and le acial numbness to pinprick. Gag re ex is
absent. Motor examination is otherwise unremarkable. Sen-
sory examination revealed decreased pinprick sensation o
the right arm and leg. He is unsteady while walking, tending In con erring with your colleague, she asks i you are going to
to lean le ward. give tissue plasminogen activator (tPA).

Question 18.1.1 What is the most likely diagnosis in this Question 18.1.2 Which one o the ollowing is NOT a contra-
patient? indication to intravenous (IV) tPA?
A) Acute vestibulitis/labyrinthitis A) Age > 75
B) Benign paroxysmal positional vertigo (BPPV) B) INR > 1.7
C) Cerebellar stroke C) Platelets < 100,000
D) Brain stem stroke D) Stroke within last 3 months
E) Ménière attack E) Glucose < 50
F) Hemorrhage seen on head C
Answer 18.1.1 The correct answer is “D.” Although dizzi-
ness can be associated with all o the above disorders, a brain Answer 18.1.2 The correct answer is “A.” Acute stroke treat-
stem stroke is the most likely answer. T e associated symp- ment with thrombolytics may be o bene t i administered to
toms o ptosis (suggestive o Horner syndrome), absent gag care ully selected patients within 3 hours o symptom onset
re ex with patient report o possible dysphagia, and crossed or 4.5 hours o symptom onset i certain criteria are met (see
sensory ndings (le side ace, right side body) are most con- able 18-1). When administered properly, the number needed
sistent with brain stem localization. In peripheral etiologies to treat or improvement at 3 months is 6. However, there is
o vertigo (“A,” “B,” and “E”), one would not expect sensory no survival bene t. T e risk o hemorrhage is 6% (1 in 16). O
phenomena, ptosis, or swallowing di culties. In BPPV, one those with symptomatic hemorrhagic trans ormation, 60% are
would expect brie attacks, lasting seconds to minutes, and atal. T ere is no upper age cutof or administration o tPA.
not a prolonged attack. In Ménière’s, there is o en some his- Strong contraindications include minor or rapidly improv-
tory o tinnitus and/or hearing loss (low requency initially). ing symptoms as well as those listed in able 18-1. In selected
A pure cerebellar stroke would not be expected to have sensory patients, there is a role or neurointerventional procedures
ndings. including intra-arterial tPA and thrombectomy. Unlike IV tPA,

498
Ch Apt Er 18 • Neu r o l o g y 499

Ta b LE 18-1 ELIGIBILIt Y AND CONt r AINDICAt IONS C) Administer IV nitroglycerin with a blood pressure goal o
FOr pA FOr ACUt E ISCh EMIC St r OKE, SYMpt OM 150/90 mm Hg
ONSEt 0–4.5 h OUr S D) Administer sublingual ni edipine with a blood pressure goal
o 160/95 mm Hg
Eligibili y
E) Monitor the patient’s blood pressure and avoid anti-
• A >18 a s
• C inica dia n sis isch mic st k ca sin m as ab n ica hypertensives
d cit
Answer 18.1.3 The correct answer is “E.” Do not treat this
Con aindica ions
• Int ac ania h m ha n CT patient’s blood pressure unless the blood pressure is > 220 mm
• C inica p s ntati n s sts s ba achn id h m ha Hg systolic or > 120 mm Hg diastolic; and even then the goal
• S mpt ms min apid imp in would be to reduce blood pressure by 15% over 24 hours. reat-
• N ic s within th past 3 m nths ing blood pressure during the acute phase o ischemic stroke
• St k s i s h ad t a ma within 3 m nths
reduces central nervous system (CNS) per usion and puts isch-
• S stain d b d p ss SBP >185 DBP >110 mm H
• S iz with p sticta sid a n ica impai m nts emic brain at risk. Obviously, this does not apply i the patient
• Kn wn a t i n s ma mati n, n p asm an sm has a hypertensive urgency or hypertensive crisis (e.g., heart
• Acti int na b din ailure or other end-organ dys unction). Note that this is or
• Kn wn b din diath sis ischemic stroke and not hemorrhagic stroke. Lowering the
• P at t c nt <100,000
blood pressure to a systolic o around or below 160 mm Hg
• H pa in within ast 48 h s with at d PTT antic a ati n
with an INr >1.7 PT >15 is indicated in hemorrhagic stroke. In this scenario, i you
• C nt s di ct th mbin inhibit s di ct act Xa inhibit s use an anti-hypertensive, labetalol or nicardipine is pre erred.
Why labetalol or nicardipine and not nitrates? Nitrates cause
r ela ive Con aindica ions
• Maj s s i s t a ma, xc din h ad t a ma, in th past intracranial vasodilatation increasing CNS pressure.
14 da s
• gI ina t act h m ha within past 21 da s Question 18.1.4 A ter admission to a monitored bed, which
• r c nt a t ia p nct at a n nc mp ssib sit o the ollowing diagnostic evaluations is LEAST likely to be
• r c nt mba p nct o urther bene t in this patient?
• P st-m ca dia in a cti n p ica ditis
• P nanc
A) Fasting lipid pro le
B) Magnetic resonance angiography (MRA) o the head and
Addi ional r ela ive Con aindica ions fo pa ien s p esen ing in neck
e Window of 3–4.5 h ou s f om Sym om Onse
• A > 80 a s
C) Fasting glucose
• Hist p i st k and diab t s D) Hypercoagulable testing
• An acti antic a ant s , n with INr < 1.7 E) ransesophageal echocardiogram
• NIH st k sc > 25
• CT sh win m ti ba in a cti n (> 1/3 midd c b a a t Answer 18.1.4 The correct answer is “D.” Hypercoagulable
dist ib ti n)
states are an uncommon cause o stroke. I you are unable to
determine the origin o a clot in a thromboembolic stroke and
the patient is otherwise low-risk (let’s say a healthy 25-year-old
there are not clearly de ned criteria or inclusion/exclusion. In male who runs marathons), then you can consider a workup
general, patients should be within 6 hours o onset o symptoms or a hypercoagulable state. Such an evaluation is not use ul in
and have well-localized symptoms. most stroke patients—like your typical 80-year-old hyperten-
sive emale, even i her carotid Dopplers and echocardiogram
are negative. In addition to monitoring the patient or signs o
HELPFUL TIP: neurologic decline or complications, evaluation o potential
Mak s a c mp si ab t inc si n and x- risk actors or recurrent stroke is an integral part o stroke care.
c si n c it ia! Wh n in d bt, ca th n a st st k Hypertension, hyperlipidemia, cardiac arrhythmia or structural
acc dit d h spita t sp ak t th n-ca n ist. de ects, illicit drug use (IV drugs and stimulants), smoking, and
diabetes mellitus are risk actors or recurrent stroke and should
be evaluated. MRA allows noninvasive evaluation o intracra-
T e patient is outside o the 4.5-hour window or thromboly- nial and cervical blood vessels in both the anterior and posterior
sis and has a blood pressure o 200/100 mm Hg. circulation. Echocardiography is also an important part in the
evaluation o stroke, and transesophageal echocardiography
Question 18.1.3 The best next step in treatment at this is more sensitive or identi ying thrombotic sources or stroke
point is to: when compared with transthoracic echo.
A) Administer IV labetalol with a blood pressure goal o 140/
90 mm Hg T e patient is ound to have a lateral medullary stoke on MRI
B) Administer IV labetalol with a blood pressure goal o 130/ and has heavy atherosclerotic burden in his vertebral arteries.
85 mm Hg Echocardiogram and telemetry are normal.
500 FAMIl y MeDICINe eXAMINATIo N & Bo Ar D r evIeW

Question 18.1.5 What would be the optimal antithrom- Ta b LE 18-2 ABCD2 Cr It Er IA


botic medication in this patient or prevention o recurrent
A : 1 p int a ≥ 60 a s
stroke?
B d p ss : 1 p int s st ic > 140 diast ic > 90 mm H
A) Coumadin (war arin) C inica at s: F ca w akn ss (2 p ints) n sp ch di c t
B) Aspirin (1 p int)
C) Plavix (clopidogrel) D ati n s mpt ms: > 60 min t s (2 p ints), ≤ 59 min t s (1 p int)
D) Aggrenox (aspirin/dipyridamole) Diab t s: 1 p int
E) Pletal (cilostazol) ABCD2 Sco e r isk of S oke in Nex 48 h ou s (%)

Answer 18.1.5 The correct answer is “B.” In patients with a 0–3 p ints 1
noncardiogenic source or their stroke, there is no data to sup- 4–5 p ints 4.1
port the use o anticoagulation (war arin) or stroke prevention.
Similarly, there is no prospective data to support a role or 6–7 p ints 8.1
cilostazol or pentoxi ylline in the prevention o stroke.
With regard to antiplatelet agents, aspirin is the rst-line
duration o symptoms, blood pressure, and diabetes. T is can
choice. T ere is no bene t to dual therapy (i.e., clopidogrel
be used to predict the 48-hour risk o stroke.
[Plavix] and aspirin). T ere is also no bene t in increasing the
Rapid evaluation o the at-risk patient is warranted, although
dose o aspirin to more than 81 mg every day. I a patient ails
it is unclear i earlier interventions make a dif erence (beyond
aspirin (recurrent stroke o same mechanism), either change to
starting antiplatelet agents, etc.).
clopidogrel or Aggrenox (combination aspirin/dipyridamole);
both are equally ef ective in stroke prevention. Because o cost,
most authors recommend aspirin as a primary preventive agent HELPFUL (a ND UNFORTUNa TE) TIP:
in those who have had a transient ischemic attack ( IA). Th initia Mr I is a s n ati 13% th tim at
4.5 h s in n np st i ci c ati n i ns (Stroke.
2013;44(6):1647–1651). It is n w s in th p st i
HELPFUL TIP:
ci c ati n. Th initia Mr I pick d p n 50% st k s
y ma s aspi in and c pid s dt th in th
in th p st i ci c ati n that w < 1 cm in siz and
i st 21 da s a t a min isch mic st k TIA. This is n t
92% th s > 1 cm in siz (Neurology. 2014;83(2):169–
t standa d ca a th h th a s m data s p-
173). What d s this m an ?I think a pati nt
p tin this s in Chin s pati nts, in wh m th ti
has had a st k b t th Mr I is n ati , th pati nt
st k m c mm n inc d s int ac ania st n sis
ma sti ha had a st k .
and in wh m c pid is m tab iz d di nt than
in th p p ati ns (N Engl J Med. 2013; 369:11–19). This
b n it d s n t xt nd b nd 21 da s. Objectives: Did you learn to . . .
• Id nti si ns/s mpt ms s sti ac t isch mic st k
(c b a in a cti n)?
It seems that there is a 2- or-1 special on neurologic symp- • Initiat a dia n stic a ati n a pati nt with a p ssib
toms in your ED this evening (well, at least isn’t a three- er st k ?
with 3 kids in the same amily all o whom have colds. . . . • D sc ib th int a n s and/ int a-a t ia tPA in
and the parent is sure they have pneumonia). Your next th t atm nt ac t isch mic st k ?
patient is a 59-year-old male who appears to have had a • e a at pti ns s c nda p nti n st k ?
simple IA with le -sided ocal weakness. T e IA lasted • Ass ss st k isk a t TIA?
< 10 minutes and the patient has a normal blood pressure
and is nondiabetic.
QUICK QUIZ: WHeN To Cu T?
Question 18.1.6 His risk o having a stroke in the 48 hours
a ter a simple TIA is: Which o the ollowing re errals or carotid endarterectomy is
A) 1% most likely to result in a bene t to the patient?
B) 5% A) A symptomatic woman with 70% or greater stenosis to a
C) 10% surgeon who has a 5% complication rate
D) 20% B) An asymptomatic man with 60% stenosis to a surgeon who
E) 30% has a 7% complication rate
C) A symptomatic woman with a 50% to 69% stenosis and a li e
Answer 18.1.6 The correct answer is “A.” T e overall risk o expectancy o > 5 years to a surgeon who has a 5% complica-
stroke ranges rom 4% to 20% in the 90 days a er a IA. T is is tion rate
a pretty wide range. o narrow down the range a bit, the ABCD2 D) None o the above
( able 18-2) has been developed and takes into account age, E) All o the above
Ch Apt Er 18 • Neu r o l o g y 501

The correct answer is “A.” All sources agree that a symptomatic more common now to use MRA or C A. O note, patients with
patient who has 70% stenosis o the carotid will bene t rom bromuscular dysplasia are more likely to have vertebral and
surgery provided that the surgeon’s complication rate is su - carotid artery dissections, but the vast majority o patients with
ciently low. Men, but not women, seem to have a bene t with vertebral or carotid dissection do not have any known connec-
symptomatic stenosis o 50% to 69% i the li e expectancy is tive tissue disorder. Case series o dissections o en describe
> 5 years and the surgeon’s complication rate is < 6% (NN 22). a minor trauma (e.g., weight li ing, amusement park rides,
In asymptomatic patients, those with > 60% stenosis will ben- various sports, “ ender bender” accidents, chiropractic neck
e t rom carotid endarterectomy, but NN is 33 and the bene t manipulation, etc.) prior to the onset o symptoms.
is less than those with symptomatic disease. Stenting is another
option. It seems as though long-term outcomes with stenting CASE 18.2
and endarterectomy are about the same although more data is
needed (N Engl J Med. 2016 Feb 1). Stenting seems to be worse A 24-year-old right-handed woman presents to you in the
than endarterectomy in patients over 70, however (Lancet. 2016 ED a er her second episode o loss o consciousness. T e
Feb 12). rst spell occurred 6 months ago and was associated with
a 60-second loss o consciousness and jerking movements
o her arms and legs. Following the spell, she was con used
QUICK QUIZ: A PAIN IN THe NeCK or about 15 minutes. At that time, her initial ED evaluation
was unremarkable. She presents today ollowing a spell that
A 35-year-old emale presents or worsening right-sided pos- occurred about 45 minutes ago. Her riends observed her
terior neck pain that radiates to her right occiput. T e pain to all to the ground and shake her arms and legs or about
has been dull and intermittent or about a week and associ- 2 minutes. T ey could not get her to respond during this
ated with some right posterior scalp tenderness. She became time. A erward, she was con used and they brought her to
alarmed today because she elt dizzy a er her workout. She the ED. Upon arrival in the ED, she is mildly drowsy but oth-
describes several minutes o vertigo that resolved a ew hours erwise oriented. She has no memory o the earlier events. Her
ago. Now, sitting in your examination room, she denies vision general medical examination and neurological examination
changes, muscle weakness, numbness, paresthesia, swallow- are unremarkable.
ing problems, and speech de cits. Since she’s training or a
“Viking’s Valor Race,” she has intensi ed her workouts in Question 18.2.1 Which o these tests would be LEAST
the past ew weeks, li ing heavier weights. She is relatively help ul in determining the etiology o this spell?
healthy but does take lisinopril or hypertension due to bro- A) Urine toxicology screen
muscular dysplasia. Her vitals are normal. Her neurological B) Electrolytes
examination is normal. C) Neuroimaging (head C or MRI)
D) Electrocardiogram (ECG)
T e most appropriate next step in the care o this patient is: E) Electroencephalogram (EEG)
A) Reassurance, rest rom exercise, ibupro en
B) Re erral to a neurologist Answer 18.2.1 The correct answer is “D.” In this case, ECG
C) Initiation o aspirin 81 mg daily would be the lowest yield, as this spell is most suggestive o
D) Urgent head C seizure. Syncopal episodes, which are o en o primary cardiac
E) Urgent MRI/MRA o the head and neck etiology, are generally o shorter duration without postictal
con usion. Although a ew tonic–clonic movements can be seen
The correct answer is “E.” T is patient is presenting with with syncope (convulsive syncope), it would be atypical or
symptoms concerning or vertebral artery dissection. Extra- those movements to continue throughout the entire period o
cranial arterial dissections o the head and neck are relatively loss o consciousness.
more common causes o stroke in young adults compared with All o the other tests would be use ul. Evaluation o a rst-
older adults. Fortunately, many patients experience symptoms time seizure should include assessment or alcohol or other
o dissection (typically head and/or neck pain, although a drug withdrawal (especially benzodiazepines and barbiturates)
Horner syndrome can develop in patients with carotid artery as well as drug intoxication (cocaine, methamphetamine, and
dissection) prior to having a stroke and can be identi ed early other sympathomimetics). In ection, including meningitis and
i the physician is attentive and orders the appropriate test. In encephalitis, can provoke a seizure. Hyponatremia, hyperna-
this case, “D” is not the right test. Head C would be unlikely tremia, hypocalcemia, hypoglycemia, hyperglycemia, hypo-
to show anything since she does not currently have neurologic magnesemia, hypophosphatemia, and uremia are all associ-
signs or symptoms. Even with a C angiogram o the brain, the ated with seizures. o rule out structural lesions (e.g., tumor
extracranial neck arteries (where the problem exists) would not and AV mal ormation) and hemorrhage, neuroimaging should
be visualized. T us, the imaging must include the head and neck be per ormed. Although MRI has greater sensitivity, it is o en
and must provide views o the arteries. I “E” is not available, not available in a timely manner, and thus, C is the modality
C A o the head and neck is a reasonable alternative. Classi- o choice. EEG would be able to assess or any seizure activity
cally, an angiogram was used to make the diagnosis, but it is wave orms.
502 FAMIl y MeDICINe eXAMINATIo N & Bo Ar D r evIeW

D) 60%
HELPFUL TIP:
E) 90%
A pati nts with synco e, inc din chi d n, d s
at ast n eCg in d t tp n d QT in-
Answer 18.2.3 The correct answer is “D.” A single interictal
t a , B ada s nd m , tc. I th sp is n t c a a
EEG has only a 60% sensitivity or picking up a seizure ocus.
s iz and c d b s nc p , t an eCg . Pa tients with
T e sensitivity goes up to 90% a er three interictal EEGs. How-
simple syncope, no hea d tra uma , a nd a norma l neu-
ever, this still means that an EEG will be negative in 10% o
rologic exa mina tion do not need a CT sca n (see www.
those with seizures even a er three EEGs.
choosingwisely.org).
She has multiple questions about AEDs . . . and so do you!

HELPFUL TIP: Question 18.2.4 Which o the ollowing AEDs is NOT


Th m st c mm n ca s stat s pi ptic s in a pa- typically associated with weight gain?
ti nt with s iz dis d is n nc mp ianc ! o n th A) Depakote (valproic acid)
m st c mm n ca s s b akth h s iz s in th s B) Lamictal (lamotrigine)
with a kn wn s iz dis d is s p d p i ati n. C) egretol (carbamazepine)
D) Neurontin (gabapentin)

Evaluation in the ED, including electrolytes, CBC, brain C Answer 18.2.4 The correct answer is “B.” Depakote, egre-
scan, and urine toxicology, is unremarkable. An electroen- tol, and Neurontin are all associated with weight gain. Gabitril
cephalogram (EEG) is obtained in the ED and read as nor- (tiagabine) is also associated with weight gain. Lamictal is typically
mal. T e patient is eeling well and does not wish to remain in weight neutral. Felbatol ( elbamate) and opamax (topiramate)
the hospital. Her riends assure you that they will be with her are associated with weight loss.
over the next 24 hours. She returns to your clinic in 2 days.
Question 18.2.5 Which o the ollowing adverse ef ects is
Question 18.2.2 A ter reviewing her test results with her, NOT typically associated with phenytoin (Dilantin)?
what do you recommend or urther management? A) Cerebellar atrophy
A) Continued ollow-up with no urther workup or treatment B) Gingival hyperplasia
B) Video/EEG monitoring C) Stevens–Johnson syndrome
C) Initiate treatment with antiepileptic drug(s) D) Hypertension
D) ilt table testing E) Bone demineralization
E) Whirligig testing
Answer 18.2.5 The correct answer is “D.” Phenytoin (Dilantin)
Answer 18.2.2 The correct answer is “C.” T is is this patient’s is associated with a number o idiosyncratic ef ects including
second seizure. In adults with a rst seizure, only 30% to 60% cerebellar atrophy, hirsutism, Stevens–Johnson syndrome, and
will go on to have a second seizure. In patients who have a second gingival hyperplasia. Diplopia and nystagmus can be prominent
seizure, the likelihood o going on to have a third is 80% to 90%; side ef ects, particularly with supratherapeutic doses. Dizziness,
there ore, a er a second unprovoked seizure, treatment is rec- drowsiness, atigue, headache, nausea, vomiting, weight loss,
ommended. Video/EEG monitoring (“B”) is appropriate or clas- and urine discoloration (pink, red, or reddish-brown) are among
si ying spells o unclear etiology. In order or video/EEG to be an some o the side ef ects seen with phenytoin. Many AEDs cause
ef ective tool, the patient should have spells requently enough increased metabolism o vitamin D via the liver. Long-term use
to capture them during a reasonable inpatient stay (3 days aver- o AEDs may lead to osteoporosis, so vitamin D supplementation
age). ilt table testing (“D”) would not be o value, as syncope is with calcium is routine. Phenytoin is associated with hypotension
unlikely to be the cause o these spells, and whirligig testing (“E”) and cardiac dysrhythmia (particularly with IV administration)
is not a real thing (but it may be in a Dr. Seuss book!). and can cause “purple glove syndrome” i the peripheral IV in l-
trates. T is is due to the solution/carrier or the drug. Fosphe-
You discuss her case with the neurologist, Dr. Lotta Branes, nytoin (Cerebyx) does not have risk o “purple glove syndrome”
who recommends that she start an antiepileptic drug (AED) and has much lower incidences o hypotension and arrhythmias,
since this is her second unprovoked seizure. “Huh? What?” so is the ormulation o choice when giving phenytoin IV.
you say. “Start her on an antiepileptic drug? Didn’t she have
a negative EEG?” Question 18.2.6 Which o the ollowing AEDs is NOT Class D
or pregnancy?
Question 18.2.3 What is the sensitivity o a single interictal A) Valproic acid (Depakote)
EEG or seizure ocus? B) Phenytoin (Dilantin)
A) 20% C) Lamictal (lamotrigine)
B) 30% D) Phenobarbital
C) 50% E) Primidone
Ch Apt Er 18 • Neu r o l o g y 503

Answer 18.2.6 The correct answer is “C.” T e newer AEDs D supplementation, should be initiated in patients on AEDs
including Neurontin (gabapentin), Lamictal (lamotrigine), due to increased risk o osteoporosis. “A” and “C,” work and
Keppra (levetiracetam), rileptal (oxcarbazepine), Gabitril driving sa ety issues, should be discussed. Most states have spe-
(tiagabine), and Zonegran (zonisamide) are currently Class C ci c laws regarding driving and seizures and the time be ore
in pregnancy. Valproic acid, phenytoin, phenobarbital, primi- resumption o driving privileges varies greatly; check the laws
done, and carbamazepine are all Class D drugs in pregnancy. in your state(s) o practice. Epileptic patients should be warned
opiramate has been associated with birth de ects and should o potentially dangerous activities at work and home (e.g.,
be avoided. In particular, valproic acid has a known association working on ladders, with heavy equipment, bathing, and swim-
with neural tube de ects. As a result, it is recommended that all ming). “D,” alcohol use, should be discussed with patients.
women o childbearing age be on olate and a prenatal vitamin Patients should also be warned o potential actors that could
while on AEDs. It is not recommended that women discon- lower their seizure threshold, including alcohol consumption,
tinue AEDs when pregnant, but simpli ying to monotherapy is herbal ephedra supplements, sleep deprivation, in ection, and
recommended i possible. medication noncompliance.

Objectives: Did you learn to . . .


HELPFUL TIP: • e a at a pati nt with a p t ntia n w dia n sis
In th thi d t im st , itamin K10 m Po dai is c m- pi ps ?
m nd d b s m xp ts d t d p ssi n c ttin • Id nti s m a as sp ci c c nc n a din w m n’s
act s with man anti pi ptics. This w in iss s in pi ps ?
c ttin act s has b n ass ciat d with n nata • r c niz c mm n nc nt d s i s sid cts
b din . with anti pi ptic d s?
• C ns pati nts with pi ps dai acti iti s and
a din m dicati n mana m nt?
Question 18.2.7 Which o the ollowing drugs does NOT
have potential interactions with oral contraceptives?
A) Depakote (valproic acid) QUICK QUIZ: SeIZu r e DISo r Der S
B) egretol (carbamazepine)
C) rileptal (oxcarbazepine) All o the ollowing are indicated in the treatment o petit mal
D) Dilantin (phenytoin) seizures (aka absence seizures, aka generalized nonconvulsive
E) opamax (topiramate) epilepsy) EXCEP :
A) Ethosuximide
Answer 18.2.7 The correct answer is “A.” Interactions with B) Acetazolamide
oral contraceptives, decreasing the e cacy o the OCP not the C) Valproic acid
anticonvulsant, have been reported with oxcarbazepine, pheno- D) Clonazepam
barbital, phenytoin, lamotrigine, and topiramate. E) Phenytoin
Zonisamide, (zongran), valproic acid (Depakote),
tiagabine (Gabitril), levetiracetam (Keppra), gabapentin The correct answer is “E.” Phenytoin is not indicated in the
(Neurontin), and elbamate (Felbatol) do not inter ere with treatment o petit mal seizures. T e other options are indicated.
oral contraceptives. A barrier method o contraception, in Other drugs known to be use ul in petit mal seizures include
addition to oral contraceptives, is recommended in women on lamotrigine, phenobarbital, and topiramate.
AEDs, particularly those with known interactions with oral
contraceptives.
HELPFUL TIP:
Question 18.2.8 Which o the ollowing is NOT part o rou- F and ma s iz s, a p ic acid s ms t b th
tine counseling with epilepsy and initiation o therapy with d that is m st cti and th s th b st initia d
an antiepileptic drug? (b t n t in p nanc ). F pa tia s iz s ( . ., t m-
A) Work sa ety p a b pi ps ), am t i in s ms t b th b st
B) Calcium and vitamin D supplementation initia d .
C) Driving
D) Alcohol consumption
E) Epilepsy surgery
CASE 18.3
Answer 18.2.8 The correct answer is “E.” Epilepsy sur- A 60-year-old le -handed man comes in with a complaint
gery is reserved or patients who are intractable to medical o numbness and tingling in his lower extremities or about
management and have ailed at least two AEDs. Exceptions 10 months. He notes no weakness. He has had type 2 diabe-
would include patients with ocal pathology (e.g., malig- tes mellitus or 6 years and has a 30 pack-year smoking his-
nancy or vascular mal ormation). “B,” calcium and vitamin tory. Examination reveals decreased sensation to light touch,
504 FAMIl y MeDICINe eXAMINATIo N & Bo Ar D r evIeW

pin-prick, and vibratory sensation in the eet extending to


HELPFUL TIP:
7 cm below the knees symmetrically. You also notice lack o
A hist ac h s is imp tant, as ch nic a c h
hair on his leg to the same level. Chest, abdomen, and upper
ab s is c mm n ass ciat d with p n path ,
extremities have normal sensation. His re exes are 1+ in the
n in th abs nc itamin B12 th n t iti na
upper extremities and quadriceps with absent Achilles bilat-
d ici nci s.
erally. T e remainder o his neurological and general medi-
cal examination is unremarkable. He says his diet is good but
always brings you a dozen doughnuts when he visits . . . with
one or two missing. Your patient is on a number o medications.

Question 18.3.1 This history is most consistent with: Question 18.3.3 Which o the ollowing medications is
A) Stroke known to cause a peripheral neuropathy?
B) Early mononeuritis multiplex A) Metronidazole
C) Guillain–Barré syndrome (GBS) B) HMG-CoA reductase inhibitors
D) Brown–Sequard syndrome C) Amiodarone
E) Peripheral neuropathy D) Disul ram
E) All o the above
Answer 18.3.1 The correct answer is “E.” T is is the quintes-
sential presentation o a patient with diabetic peripheral neu- Answer 18.3.3 The correct answer is “E.” All o the above can
ropathy. “A” is incorrect because o the distribution. Bilateral cause a peripheral neuropathy. Some o the other drugs that
distal lower extremity sensory changes are not likely rom a can cause neuropathy include phenytoin, isoniazid, several
stroke. “B,” mononeuritis multiplex, is—at least initially—not chemotherapeutic agents, nitro urantoin, ddC, and D4 . See
in a stocking-glove distribution. Patients with mononeuritis able 18-3 or more causes o peripheral neuropathy.
multiplex will notice a stepwise loss o sensation and motor
ability in discrete peripheral nerve distributions. Eventually,
Ta b LE 18-3 VEr Y pAr t IAL LISt OF CAUSES OF
these may become con uent and resemble a stocking-glove
pEr Iph Er AL SENSOr Y NEUr OpAt h Y
neuropathy. However, this is ound late in the disease. Addi-
tionally, there is usually motor involvement. “C” is unlikely Infec ious
because GBS has a relatively rapid onset and is associated with S phi is
l m dis as
motor ndings. “D,” Brown–Sequard syndrome, is the result o M c p asma
a lesion in one side o the spinal cord. Patients have dimin- W st Ni i s
ished proprioception, vibration sense and strength on the side l p s
o the lesion, decreased sharp sensation, and loss o tempera-
Nu i ional
ture sense on the other side. Since our patient’s ndings are vitamin B12 d ci nc
symmetrical in both distribution and mani estation, it is not Thiamin ( itamin B1) d ci nc
Brown–Sequard syndrome. F ic acid d ci nc
vitamin B6 t xicit
Question 18.3.2 Which o the ollowing laboratory tests C iac dis as
would NOT be use ul in helping to determine a particular D ugs
cause o this patient’s neuropathy? M t nidaz
A) Serum immuno uorescence electrophoresis Is niazid
B) SH/ ree 4 Ph n t in
Tha id mid
C) Hemoglobin A1C
D) Electrolytes (Na, K, Cl, CO2) Me abolic
E) Vitamin B12 Diab t s
H p th idism
Answer 18.3.2 The correct answer is “D.” Electrolytes are not t oxic
likely to give us a diagnosis in this patient. T e initial evalu- M c
ation o a patient with “stocking-glove” sensory loss should Tha i m
ocus on nding potentially treatable causes o neuropathy. o th h a m ta s
Ac h
T ese would include hypothyroidism (“B”) and vitamin B12
de ciency (“E”). In addition, serum immuno uorescence elec- Miscellaneous
trophoresis (“A”) can help to identi y monoclonal gammopathy, Am id
which is associated with neuropathy. Hemoglobin A1C (“C”) is Pa an p astic s nd m s
Mit ch nd ia dis as
a marker o glycemic control in diabetes and would be associ-
Ch nic inf ammat d m inatin p n path (m t > s ns )
ated with diabetic polyneuropathy. Serum heavy metal levels M n c na amm path
may also be o use.
Ch Apt Er 18 • Neu r o l o g y 505

Question 18.3.4 Electrophysiological testing (electromyog- dosing is necessary. Although gabapentin is generally well-toler-
raphy and nerve conduction studies or EMG/NCV) can be ated, numerous side ef ects have been reported. Among the most
help ul or all o the ollowing EXCEPT: prominent complaints are those o dizziness, vertigo, and ataxia.
A) Con rming the presence o sensory de cits Fatigue and drowsiness are also relatively common adverse ef ects;
B) Identi ying subclinical motor de cits paresthesias, myalgias, and weakness have also been reported.
C) Identi ying a speci c etiology o the neuropathy (e.g., diabetic Peripheral edema (particularly lower extremities) and acial edema
vs. B12 de ciency vs syphilis) are also seen as a result o therapy with gabapentin. itrating slowly
D) Determining axonal damage versus demyelination as the to the target dose over several weeks can improve tolerance.
primary pathologic process
E) Dif erentiating between primarily myopathic and neuro- Your patient is concerned about swelling in his legs, as he has
pathic processes had problems with this in the past and does not want to try
gabapentin at this time.
Answer 18.3.4 The correct answer is “C.” EMG and NCV are
not capable o yielding results speci c to the etiology o neu- Question 18.3.6 All o the ollowing medications would be
ropathy (e.g., identi ying diabetic neuropathy vs. B12 vs. syphi- a reasonable FIRST choice or treatment o pain ul neurop-
lis); however, they can yield a pattern suggestive o a particular athy EXCEPT:
etiology. For example, increasing motor reaction to repeated A) Amitriptyline
stimulation is classic Eaton–Lambert syndrome (a paraneoplas- B) opical lidocaine ointment
tic syndrome), while a decreasing motor response with repeated C) Oxycodone
stimulation is suggestive o myasthenia gravis. EMG and NCV D) opical capsaicin cream
can be help ul in con rming the presence o a radiculopathy in E) Valproic acid
the proper clinical setting. T ey can also determine i the pro-
cess is primarily axonal (damage directly to the axons/reduction Answer 18.3.6 The correct answer is “C.” Narcotics are not the
in the number o axons) versus demyelinating (damage to the treatment o choice or peripheral neuropathy and tend not to
myelin sheath). NCV can con rm the presence o sensory de - be as ef ective in this setting as in other types o pain. ricyclic
cits (although NCV is looking at large bers and may not identi y antidepressants ( CAs) are commonly used in the setting o
small ber, pain ul neuropathies) as well as identi ying subclinical neuropathic pain and remain the rst-line therapy. T e evi-
motor involvement. EMG and NCV can be used to dif erentiate dence or CAs suggests they are superior to other classes o
between myopathic and neuropathic processes. drugs. I pain is con ned to a small area, a topical treatment
such as capsaicin cream or topical lidocaine may be ef ective.
Lidoderm patches prescribed outside the setting o post-herpetic
HELPFUL TIP:
neuralgia represent an of -label use.
I ct ph si ica t stin sh ws a p ima d m -
inatin n path , a n ic c ns t is c m-
m nd d. Th s ma sp nd t int a n s imm n - HELPFUL TIP:
b in (IvIg ), st ids th imm n s pp ssi o th t atm nts n pathic pain i TCAs ai inc d
a nts, and p asma xchan , am n th s. antic n sant d s s ch as abap ntin, p aba in,
a p ic acid, and ca bamaz pin . Fina , m thad n is
m cti than th piat s n pathic pain.
You wan a numbe needed o ea , you say? Fo a
A er being ollowed in clinic or several years, the patient
50% ain educ ion e NNt s a e as follows: t CAs:
begins to complain o a pain ul, burning sensation developing
3.6, s ong o ia es: 4.3, Duloxe ine/SNr Is: 6.4, Gaba-
in his eet and ankles. T is pain developed gradually over the
en in: 7.2–8.3, p egabalin: 7.7 (La ncet Neurol. 2015
last ew years in the areas that previously were numb. He has a
Feb; 14:162). What d s this m an pati nt?
riend who was started on gabapentin (Neurontin) or a similar
Basica m st th s d s wi n t w k m st pa-
problem and is wondering i this would be a good drug or him.
ti nts. D n’t xp ct s cc ss . . . as w kn w m
xp i nc . o ma b a ch nic n path pain
Question 18.3.5 Which o the ollowing is NOT an adverse
pati nts a a happ , smi and s nn !!
reaction or contraindication to gabapentin?
A) Dizziness
B) Peripheral edema Question 18.3.7 What is the most common in ectious cause
C) Fatigue and drowsiness o peripheral neuropathy in the world (not just in the
D) Hepatic disease United States)?
E) Paresthesias A) HIV
B) Lyme
Answer 18.3.5 The correct answer is “D.” Gabapentin (Neuron- C) Leprosy
tin) is renally—rather than hepatically—cleared. Also, it is not D) Hepatitis C
metabolized by the liver. In the setting o renal dys unction, reduced E) uberculosis
506 FAMIl y MeDICINe eXAMINATIo N & Bo Ar D r evIeW

Answer 18.3.7 The correct answer is “C.” T e most common pretty hard to get this type o compressive lesion i you are not
in ectious cause o peripheral neuropathy in the world is leprosy. passed out or very sedated. “B,” ulnar nerve compression (cubi-
Leprosy (Hansen disease) is caused by Mycobacterium leprae, an tal tunnel syndrome), presents with pain and numbness in the
acid- ast bacillus. It usually presents with a hypopigmented anes- elbow and ourth and h ngers and weakness o the intraos-
thetic patch. T e sensory de cits start with loss o temperature seous muscles (weakness with spreading ngers). “E,” median
sensation ollowed by loss o pain and then tactile sensations. nerve compression (including carpal tunnel syndrome), leads to
Modalities carried by the posterior columns (proprioception numbness on the palmar sur ace o the thumb and ngers two,
and vibration) are spared. Sensation in the palms, soles, mid- three, and the radial hal o our. T ere is weakness and perhaps
chest, and mid-back is preserved until late in the disease. atrophy o the thenar muscles (unable to maintain opposition o
HIV can cause peripheral neuropathy, most commonly thumb to h nger against resistance).
being a symmetric, distal polyneuropathy. Autonomic dys unc-
tion and mild weakness accompany distal paresthesias and CASE 18.4
burning sensations with sensory loss. Keep in mind that many
A 25-year-old woman presents to your clinic complaining o
o the medications used in HIV therapy are also associated with
a bi rontal headache that started this morning. She describes
peripheral neuropathies.
the pain as throbbing and 8/10 in severity. She is complaining
Early neurologic mani estations o Lyme disease include
o photophobia and nausea. She has had similar headaches in
lymphocytic meningitis, cranial neuropathy (a Bell palsy-
the past, lasting a ew hours to all day. She is unable to work
like picture), or pain ul radiculoneuritis. Guillain–Barré syn-
during these headaches and pre ers a dark, quiet room (as
drome (GBS), and mononeuritis multiplex can be seen early
do we all). T e physical examination, including neurological
in the course o Lyme disease. Advanced (late) neurologic
examination, is unremarkable.
complications o Lyme disease include peripheral neuropathy
and encephalomyelitis.
Question 18.4.1 Which o the ollowing statements is most
Hepatitis B and C can cause peripheral nerve mani estations.
accurate?
Hepatitis B has been associated with GBS (demyelinating) and
A) She likely does not have migraine headaches because her
mononeuritis multiplex. Hepatitis C is also associated with
headache is bilateral
multiple mononeuropathies.
B) She likely does not have migraine headaches because they
uberculosis has rarely been associated with neuropathy, but
most commonly present in the ourth to h decade o li e
isoniazid is well-known to cause peripheral neuropathy (that’s
C) She likely does not have migraine headaches because they
why we prescribe pyridoxine with it).
rarely occur in the morning
D) She likely has migraine headaches
HELPFUL TIP:
In th u nit d Stat s, diab t s and a c h s dis d s Answer 18.4.1 The correct answer is “D.” She likely has
a th m st c mm n ca s s p iph a n path . migraine headaches. Migraine headaches may vary consid-
erably in severity, time o day, and characteristics. T e Inter-
national Headache Society (IHS) has a use ul classi cation
Objectives: Did you learn to . . . system with criteria or the diagnosis o migraine headaches
• r c niz c mm n ti i s p iph a n path ? ( able 18-4). “B” is incorrect because migraine headaches typi-
• Id nti app p iat s s ct ph si ica t stin ? cally present in the rst three decades o li e. Attacks typically
• Id nti t p s n path that a p t ntia t atab ?
last less than 1 day although they may occasionally last longer.
Migraine headaches are typically moderate to severe in inten-
• P sc ib m dica th ap pain p iph a n path ?
sity, may occur at any time during the day, and occur with or
without aura. Most migraine headaches are unilateral, preceded
QUICK QUIZ: Wr IST Dr o P
Ta b LE 18-4 Cr It Er IA FOr t h E DIAGNOSIS OF
Which o the ollowing is most likely to cause an isolated wrist MIGr AINEWIt h OUt AUr A
drop?
A) C-3 disk lesion At ast attacks in th win c it ia:
B) Ulnar nerve compression • H adach astin 4–72 h s ( nt at d ns cc ss t at d)
• H adach has at ast tw th win cha act istics:
C) Radial nerve compression 1. u ni at a cati n
D) C-4 disk lesion 2. P satin q a it
E) Median nerve compression 3. M d at s int nsit (inhibits p hibits dai acti iti s)
4. A a ati n b wa kin stai s simi a tin ph sica acti it
The correct answer is “C.” Compression o the radial nerve • D in h adach at ast n th win :
1. Na s a and/ mitin
(such as sleeping with someone’s head on your arm) can cause 2. Ph t ph bia and ph n ph bia
an isolated wrist drop, the so-called “Saturday Night Palsy.” Sus- • N id nc anic dis as
pect alcohol or other drug abuse in patients with this lesion. It is
Ch Apt Er 18 • Neu r o l o g y 507

Ta b LE 18-5 Cr It Er IA FOr t h E DIAGNOSIS OF migraines. Oral NSAIDs (“A”), including aspirin and combi-
MIGr AINEWIt h AUr A nation analgesics containing caf eine, are a rst-line choice or
mild-to-moderate migraine attacks or severe attacks that have
At ast tw mi ain s (s Tab 18-4) in at ast th th been NSAID responsive in the past. “B,” the “migraine-speci c”
win cha act istics:
treatments, commonly called the “triptans” (e.g., sumatriptan,
1. o n m sib a a a s mpt ms indicatin b ain zolmitriptan, naratriptan, rizatriptan, almotriptan, eletriptan,
d s ncti n and rovatriptan—wow, talk about “me too” drugs . . . ), are
2. At ast n a a s mpt m d ps ad a 5 min t s ( ef ective and relatively sa e or the acute treatment o migraine
n ) tw m s mpt ms cc in s cc ssi n
3. N sin a a s mpt m asts m than 60 min t s
headaches. riptans are an appropriate initial treatment choice
4. H adach b ins d in a a within 60 min t s th nd in patients with moderate-to-severe migraines who have no
th a a contraindications to their use (see below). Alternative vaso-
constrictive agents, including DHE nasal spray (dihydroergot-
Hist , ph sica , and app p iat dia n stic t sts xc d a s c nda
ca s
amine, “D”), can provide a sa e and ef ective treatment o acute
migraine attacks. DHE can be administered IV as well. Vaso-
a
An a a sh d ha s mpt ms m tw n ic d mains— is a constrictive side ef ects, including the risk o coronary artery
and/ s ns and/ sp ch. Th s mpt ms ma b p siti ( . ., spasm, should speci cally be discussed with patients prior to
f ick in i hts, pins/n d s s nsati n) n ati ( . ., sc t ma, initiation o therapy.
n mbn ss).

HELPFUL TIP:
by aura, and accompanied by nausea and vomiting. T ey are
Addin a m t c p amid t aspi in NSAIDs wi
more prevalent among women, with a 1-year prevalence rate o
imp th i at s cc ss. Pa t th na s a and
approximately 18% in women, 6% in men, and 4% in children.
mitin m mi ain s (and th as n that a m d-
Family history is important as 80% o patients with migraine
icati ns t n d n t w k) is m ast ic pa sis. M t -
headache have a rst-degree relative with migraines.
c p amid c m s this p b m and t ats na s a
Migraine headaches were ormerly classi ed as classic type
as w . o a it d t t at h adach s with is
(migraine with aura) and common type (migraine without
p ch p mazin , 10 m Iv 25 m Pr . This w ks
aura). ypical auras develop over several minutes and last or
b th mi ain and t nsi n-t p h adach s. S
less than 60 minutes. Auras may involve visual, language, sen-
Tab 18-6 a ist mi ain h adach t atm nts.
sory, or motor de cits. T e visual auras are by ar the most com-
mon and may appear as photopsias ( ashes o light), scotomas
(blind spots), or complex shapes that build or move across the
visual eld. T e IHS criteria or migraine with aura are listed in Ta b LE 18-6 U.S. h EADACh E CONSOr t IUM GUIDE-
able 18–5. LINES FOr t r EAt MENt OF h EADACh E
Group 1: P n, p n nc d • Sumatriptan SC/IN
statistica and c inica b n t (at ast • DHE (IV/IM/SC/IN)
HELPFUL TIP: tw d b -b ind, p ac b -c nt d • Prochlorperazine IV
Remember that the IHS criteria are research tools. Patients st di s and c inica imp ssi n • Butorphanol IN
may have a migraine and not meet all of the criteria noted ct)
by the IHS. Whi pati nts with a c tain t p mi ain
Group 2: M d at statistica and • Prochlorperazine IM/PR
h adach wi id a m t a c it ia, it is n t n c s- c inica b n t ( n d b -b ind, • Chlorpromazine IV/IM
sa t m t a c it ia t mak a c inica dia n sis p ac b -c nt d st di s and c inica • Metoclopramide IV
mi ain h adach . imp ssi n ct) • Ketorolac IM
• Lidocaine IN
• Meperidine IM/IV
• Butorphanol IM
You have decided to treat this woman’s migraine headache. • Methadone IM

Group 3: C nf ictin inc nsist nt • Metoclopramide IM/PR


Question 18.4.2 Which medication would be LEAST appro- id nc
priate or acute management o her headache?
A) Oral ibupro en Group 4: P n t b statistica • Chlorpromazine IM
c inica in cti ( ai d cac s. • Granisetron IV
B) Intranasal sumatriptan p ac b ) • Lidocaine IV
C) IV meperidine (Demerol)
D) Intranasal DHE Group 5: C inica and statistica • Dexamethasone IV
b n ts nkn wn (ins ci nt • Hydrocortisone IV
id nc a ai ab )
Answer 18.4.2 The correct answer is “C.” T e least appropri-
ate treatment rom the above list would be Demerol (meperi- Data m u .S. H adach C ns ti m: http://www.
dine). T e long-term use o opiates or rescue therapy has am icanh adach s ci t . /p ssi na s c s/ s_ h adach _
not been ound to improve the quality o li e in patients with c ns ti m_ id in s/.
508 FAMIl y MeDICINe eXAMINATIo N & Bo Ar D r evIeW

Question 18.4.3 Which o the ollowing statements is


correct? Your patient has decided to take ibupro en or her headaches.
A) I a patient does not respond to sumatriptan, there is no T is medication seemed to be e ective at rst, but she notes
point in trying another triptan because the patient will not or the last several weeks that she is taking two to three doses
respond o ibupro en per day without signi cant headache relie . She
B) DHE and sumatriptan may be sa ely used within the same has had a dull bilateral headache that is moderate in severity
24-hour time period or the last 2 weeks. T e medication dulls the headache but
C) Sumatriptan use is contraindicated in patients with known it comes right back. She has no personal or amily history o
coronary artery disease, regardless o age coronary artery disease.
D) Flushing, sweating, and paresthesias a er a dose o sumat-
riptan is an indication o a severe reaction and continued use Question 18.4.5 Which o the ollowing statements is
o this medication is contraindicated correct?
A) She likely has a tension headache and should increase her
Answer 18.4.3 The correct answer is “C.” T e triptans should requency o ibupro en and continue to take it on a daily
not be used in patients with known coronary disease. Patients basis
who do not respond to one triptan may respond to other B) She likely has a medication-overuse headache in addition to
triptans, and a trial o other triptans is appropriate. Also, a chronic migraine headache (status migrainous) and should
patient may respond initially to a triptan but not respond on taper and then discontinue ibupro en
other occasions. Each triptan has a maximum recommended C) A medication such as sumatriptan used on a daily basis does
dose, and a good rule o thumb is that the initial dose may be not increase the risk o rebound headache
repeated once in a 24-hour period o time. However, avoid D) She likely does not have medication-overuse headache
the use o DHE within 24 hours a er a triptan has been because opiates are the only medications that increase the
given due to increased vasoconstriction and the possibility o risk o these headaches
vasospasm.
Answer 18.4.5 The correct answer is “B.” See below or a
Question 18.4.4 Contraindications to the use o “triptans” detailed explanation.
include all o the ollowing EXCEPT:
A) Lung cancer Question 18.4.6 Which o the ollowing medications taken
B) Uncontrolled hypertension on a requent basis is LEAST likely to cause medication-
C) Use o an MAO inhibitor within the last 2 weeks overuse or rebound headache?
D) Use o an ergot preparation within the last 24 hours A) Sumatriptan
B) Morphine
Answer 18.4.4 The correct answer is “A.” Lung cancer is not a C) Ibupro en
contraindication to the use o triptans. In addition to “B” to “D,” D) Amitriptyline
caution should be used in patients with history o stroke, known E) All o the above are equally likely to cause rebound headache
cardiac risk actors, and impaired liver unction.
Answer 18.4.6 The correct answer is “D.” Frequent use o opi-
ates, acetaminophen, NSAIDs, ergotamine, triptans, and any
other analgesics may put a patient at risk or medication-overuse
HELPFUL TIP:
or rebound headache. Although analgesic rebound headache
C mm n acti ns t t iptans inc d jaw ti htn ss,
characteristics can vary signi cantly, the patient typically
shin , anxi t , dizzin ss, and sw atin . Th s a
reports a pattern o headache that decreases modestly in sever-
nc m tab b t n t dan s. S i s acti ns
ity with the use o their analgesic o choice, and then in 2 to
t t iptans inc d c na as spasm, anaph axis,
4 hours (depending on the medication), the headache returns to
h p t nsi c isis in pati nts with kn wn CAD,
its previous severity or worsens urther. Failure to repeat analge-
h p s nsiti it t t iptans, nc nt d h p t n-
sic use results in a withdrawal headache (similar to the caf eine
si n. S Tab 18-6 ac t t atm nt mi ain
withdrawal headaches physicians o en experience when they
h adach s.
miss their morning cof ee). In the case o triptans, the headache
may not worsen or many hours or even until the next day, but a
cycle o regular use o the medication is still established. At this
HELPFUL TIP:
time, no clear consensus on the duration o therapy necessary
C nsid d xam thas n as an adj nct th ap in s -
to produce analgesic rebound is reported. As a general rule, it is
h adach . A sin d s d xam thas n 10 m
best to limit the use o analgesic medications to no more than
Po , Iv, IM a t ab ti th ap in th eD ma p -
2- to 3-headache days per week. In addition, limit the patient’s
nt h adach c nc in pati nts wh ha had a
analgesic use to no more than 2 to 3 weeks per month. Patient
h adach m than 24 h s (NNT 9).
education is the most important part o therapy in treating anal-
gesic rebound or medication-overuse headaches.
Ch Apt Er 18 • Neu r o l o g y 509

HELPFUL TIP: in th t atm nt mi ain th h adach s.


T atm nt b nd h adach s c nsists disc n- Addicti n, ab s , and di si n a p t ntia iss s
tin in th m dicati n. S a app ach s ha b n with th s d s.
ti dt d c h adach s a t th ana sic has b n
withd awn. Th s inc d Iv a st ids, n -
Question 18.4.8 Which one o the ollowing medications is
actin NSAIDs (nap x n), and cti admissi n, and
rated Class B or better in pregnancy?
th ap with Iv DHe (dih d tamin ) Th azin
A) Phenergan (promethazine)
(ch p mazin ). Th s sh d b c mbin d with a
B) Imitrex (sumatriptan)
p ph actic m dicati n s ch as amit ipt in ( th
C) Codeine
t ic c ic) s d n a dai basis. Pati nts can a s tak
D) Amitriptyline
h d x zin and p ch p azin wh n th ha a
E) None o the above
b akth h h adach at h m ; th s m dicati ns d
n t ca s b nd h adach s.
Answer 18.4.8 The correct answer is “E.” Headache treatment
in pregnancy remains a di cult problem. Although numerous
Question 18.4.7 Which o the ollowing medications would medications are available or headache treatment, their sa ety in
be the LEAST appropriate or the preventative treatment o pregnancy has not been established. Amitriptyline and valproic
your patient’s migraine headaches? acid are class D in pregnancy. Other commonly used tricyclics
A) Verapamil include imipramine (Class C) and nortriptyline (Class D); ven-
B) Propranolol la axine is Class C. Promethazine, prochlorperazine, codeine,
C) Amitriptyline hydrocodone, and meperidine are all Class C. Ergotamine
D) Clonazepam (DHE 45) is Class X. T e triptan class o medications, including
sumatriptan, remains Class C, though pregnancy registries, ret-
Answer 18.4.7 The correct answer is “D.” Clonazepam is not rospective, and observational studies suggest that sumatriptan
used as a preventive treatment or migraine headaches. Keep is sa e.
in mind the common side ef ects o these medications and the
appropriateness in your speci c patient. For example, valproate Question 18.4.9 In which o the ollowing patients is neuro-
would be a bad choice or many patients secondary to weight imaging LEAST likely to be use ul?
gain or teratogenicity. Propranolol may cause hypotension. A) A 30-year-old woman with a headache typical o a migraine
Amitriptyline may cause cardiac arrhythmia in certain patients, B) A 23-year-old woman with a history o migraine headaches
while constipation and urinary retention are relatively common that is very concerned because her current headache o
in elderly patients. opiramate ( opamax) may actually cause 1-week duration is more severe than her typical migraine
weight loss, and impaired cognition is common. headaches. She has been unable to sleep or concentrate at
work because o her “headache anxiety.”
HELPFUL TIP: C) A 60-year-old man with new headache, worse in the morning
A n mb m dicati ns a s in th prevention and o 6 weeks duration
mi ain h adach s. D) A 40-year-old man with a headache and right arm weakness
• Medications that have been found to have medium-
t -hi h cac , d st n th id nc , and mi d- Answer 18.4.9 The correct answer is “A.” According to the U.S.
t -m d at sid cts inc d amit ipt in , di a - Headache Consortium, neuroimaging is not typically recom-
p x s di m, and p p an /tim , t pi amat mended in migraine patients with a normal neurologic exami-
• Medications of lower e cacy include atenolol/metop nation. Imaging may be considered in patients who are disabled
/nad , nim dipin / apami , aspi in/nap x by their ear o serious pathology or i the provider is suspicious
n/k t p n, f x tin , ACe inhibit s, abap ntin, about underlying pathology. Factors that may lead one to con-
w, ma n si m, and itamin B2 sider neuroimaging include a nonacute, atypical headache or
• Antidepressants such as uvoxamine, paroxetine, unexplained abnormal neurologic examination.
n t ipt in , s t a in , t az d n , and n a axin
ha a s b n nd t b c inica caci s bas d HELPFUL TIP:
n c ns ns s and c inica xp i nc , b t n and m- N t a ni at a h adach s a mi ain s. Think c-
iz d c nt d t ia s ha b n d n t stab ish cipita n a ia, t mp a a t itis, j ts and jabs (ic pick)
th i cac h adach , t mp a is m sc s /TMJ s nd m ,
ch nic pa x sma h mic ania, sh t- astin ni at a
n a i m h adach attacks with c nj ncti a inj c-
HELPFUL TIP: ti n and t a in (Su NCT) s nd m , and c st h ad-
C mbinati n p d cts s ch as b ta bita /ca in /ac ta ach s, am n th s. W can’t c a th s in
min ph n/c d in ( . ., Fi ina with c d in ) ha n imit d spac . W a t in ha d st, captain.
510 FAMIl y MeDICINe eXAMINATIo N & Bo Ar D r evIeW

Objectives: Did you learn to . . . Caucasian population. MS is a CNS demyelinating disease that
• r c niz and dia n s mi ain h adach s? is thought to occur by an immune mediated process. T e demy-
• Initiat app p iat ac t th ap mi ain h adach s? elinating lesions o MS can occur anywhere in the CNS includ-
• Id nti c nt aindicati ns and ad s acti ns th ing the brain stem and spinal cord. T e presenting symptoms o
t iptan m dicati ns? MS vary, but common symptoms are visual complaints, weak-
• r c niz and t at ana sic- at d h adach s? ness, and sensory de cits. Although migraine can be associated
• Id nti app p iat p nti th ap ch nic with neurologic symptoms, one would expect more stereotypic
h adach s? events and a history o previous headaches. Fibromyalgia is
associated with numerous somatic complaints, but is not typi-
CASE 18.5 cally associated with sensory de cits or visual problems. Con-
version disorder can produce all o the symptoms described
A 30-year-old woman presents to your of ce with a 2-day earlier but is a diagnosis o exclusion. IAs would be unusual in
history o progressive, unilateral arm (proximal and dis- a patient o this age, though not impossible by any means, and
tal) numbness without weakness. She has been diagnosed would typically present with stroke-like symptoms (dysarthria,
with bromyalgia in the past. She is taking uoxetine or vertigo, hemiparesis, etc.).
depression and has a history o previous hospitalizations or
depression.
HELPFUL TIP:
MS has a aphic p di cti n. Th incid nc
Question 18.5.1 Which o the ollowing is the most appro-
MS inc as s with inc asin distanc m th q a-
priate next step?
t (A askans a sc w d). va i s th i s ha s -
A) Monitor her symptoms and reassure that her numbness is
st d an ass ciati n with s n i ht xp s , itamin
likely related to her bromyalgia
D, i s xp s , thnicit .
B) Order nerve conduction velocity (NCV) studies
C) Order a head C
D) Get additional history; ask about previous similar episodes Question 18.5.3 I your patient indeed does have MS, which
or other neurological concerns type is most likely?
E) Order a chest radiograph (CXR) and complete blood cell A) Devic disease
count (CBC) B) Relapsing–remitting
C) Primary progressive
Answer 18.5.1 The correct answer is “D.” O course, the D) All o the above are equally likely
answer is “more history.” With a progressive neurological de -
cit, the rst step in the workup is to urther explore the history. Answer 18.5.3 The correct answer is “B.” T e two common
Frequently, patients will not mention previous neurological orms o MS are primary progressive and relapsing–remitting.
symptoms because they— the symptoms, not the patients—are T e diagnosis o relapsing–remitting MS is based on clini-
vague (although, to be air, some patients are vague). cal grounds and laboratory data. Clinically, symptoms o CNS
dys unction develop over hours to days, stabilize, and then
When you ask about previous spells, she notes that she had improve. It is important to identi y clinical events disseminated
an episode o le leg numbness that lasted about 1-week sev- in space and time. In this case, your patient had a prior his-
eral years ago, but she thought nothing o it, as it was mild. tory o optic neuritis and lower extremity numbness and now
Six months ago, she had a 3-day visual disturbance in her has arm numbness. “A,” Devic disease (neuromyelitis optica),
right eye, during which she ound it dif cult to read and is a CNS demyelinating illness that is characterized by bilateral
ocus on objects; no blind spot was noticed. However, she had optic neuritis (simultaneous or occurring at dif erent times) and
pain in the eye, especially when moving it. spinal cord demyelination.

Question 18.5.2 What is the most likely diagnosis based on Question 18.5.4 Which o the ollowing tests would NOT be
the history given? help ul in urther diagnosing MS?
A) Multiple sclerosis (MS) A) MRI brain
B) Fibromyalgia B) Lumbar puncture
C) Conversion disorder C) Nerve conduction studies
D) Atypical migraine D) Visual evoked potentials
E) ransient ischemic attacks E) MRI spine

Answer 18.5.2 The correct answer is “A.” MS would be the Answer 18.5.4 The correct answer is “C.” MS is a central
most likely diagnosis based on the history related above. MS demyelinating process and does not produce abnormalities
most commonly presents in women 20 to 35 years old and in that would be seen on nerve conduction studies. “A” and “E,”
men 35 to 45 years old. It is almost ve times more prevalent MRI o the brain and/or spine, would be help ul. Brain MRI is
among women than among men and is more common in the 85% to 97% sensitive in detecting MS plaques. Multiple areas
Ch Apt Er 18 • Neu r o l o g y 511

o increased signal in the periventricular area are suggestive o , Answer 18.5.6 The correct answer is “C.” T e inter eron agents
but not speci c or, MS. Gadolinium-enhancing lesions suggest (Avonex, Rebi , and Betaseron) and amantadine are associated
active disease. “B,” lumbar puncture, can also be use ul. Cere- with worsening o depression. Given this patient’s history o
brospinal uid (CSF) abnormalities suggestive o MS include severe depression requiring hospitalization, one would avor
oligoclonal bands and increased synthesis o IgG. A spinal uid Copaxone as an initial therapy, although treatment with inter-
examination may be considered i the clinical diagnosis o MS is eron is not absolutely contraindicated.
suspected but is not de nite. However, the positive and negative
predictive value o CSF oligoclonal bands is inadequate to do T e patient is wondering what she can do to prevent exacer-
more than support the clinical diagnosis. Recent evidence or bations.
serum antibody testing to other myelin components has yielded
promising initial results as a supportive diagnostic tool. Finally, Question 18.5.7 Which o the ollowing is associated with
“D,” visual evoked potentials, may be help ul. Evoked potential exacerbation o MS symptoms?
may be used to aid in the diagnosis o MS by indicating prior A) Cold temperatures (you could recommend she move to
demyelination o the optic tract (optic neuritis) i the clinical Florida)
history is vague (e.g., eye pain without vision loss or no recol- B) Urinary tract in ection
lection o symptoms). T is will aid in proving the occurrence o C) In uenza vaccination
dif erent events separated by space and time. D) rauma

Question 18.5.5 Which o the ollowing is not a recognized Answer 18.5.7 The correct answer is “B.” Urinary tract in ec-
therapy or MS? tions can exacerbate MS. Un ortunately, urinary tract in ections
A) Corticosteroids are particularly common in those with MS because o the re-
B) Inter eron-β quent occurrence o neurogenic bladder. Systemic in ection has
C) Glatiramer acetate also been reported to provoke MS exacerbations. “A” is o spe-
D) Sul asalazine cial note. Cold is not associated with exacerbations, but heat is
E) Amantadine notorious, and this phenomenon actually has a name—Uhthof
phenomenon. Patients with MS should be instructed to avoid
Answer 18.5.5 The correct answer is “D.” Sul asalazine is hot tubs, saunas, steam rooms, etc. “C,” vaccinations, including
not use ul in the treatment o MS. Immune-modulating ther- in uenza vaccine, had been posited as a cause o exacerbations.
apy reduces the number o exacerbations and active lesions However, a review o multiple clinical trials showed no increased
on MRI. T ese include inter eron-β -1a (Avonex and Rebi ) risk o exacerbations in patients with MS receiving the in u-
and inter eron-β -1b (Betaseron), as well as glatiramer acetate enza, hepatitis B, or tetanus vaccinations. Note that we do not
(Copaxone). T ese medications are more e cacious i started have experience with nasal in uenza vaccine and MS. Since
early in the course o the disease. Common adverse ef ects o the nasal vaccine contains live virus, it should probably be
inter eron include atigue, depression, and myalgias. Amanta- avoided in patients with MS. “D,” trauma, has been suggested
dine is given commonly as monotherapy and in combination as a possible exacerbation trigger, but the American Academy o
with immunomodulatory therapy to treat atigue associated Neurology clinical practice guidelines state that the majority o
with MS. Corticosteroids have a role in treating severe acute class II evidence available on this issue supports no connection.
exacerbations (e.g., optic neuritis, severe neurological impair-
ments limiting activities o daily living) in the orm o a short HELPFUL TIP:
burst and taper (typically methylprednisolone, 1 g/day o en ol- A w d n th d s MS . . . Amp a (da amp idin )
lowed by an oral prednisone taper). Steroid use does not appear is s pp s d t inc as wa kin distanc . Z n ia (d xt -
to of er long-term unctional bene t, excluding the possible m th phan + q inidin ) is d si n d t d c ps -
exception o IV pulsed steroid dosing. Currently, oral immu- d b ba s mpt ms ( m ti na abi it with sp ntan s
nosuppressive therapies (mycophenolate mo etil, azathioprine, a hin , c in ). B th a ma ina b n it. Th a
and cyclosporin) are being considered in treating re ractory man m th api s apsin - mittin MS ( . .,
MS, but their long-term e cacy and sa ety are not known. Simi- dim th ma at , mit xant n , nata iz mab t i n -
larly, the value o IVIG and plasma exchange has not been con- mid , and m ), m st which q i n ic sp -
clusively demonstrated and use o these treatment modalities is cia t ca and ca hi h isk ad s nts.
reserved or re ractory patients.

Question 18.5.6 Which o the ollowing treatments would Objectives: Did you learn to . . .
you most likely choose or this patient, given her history o • Id nti pid mi ic cha act istics MS?
depression? • Id nti app p iat w k p pati nts with p ssib MS?
A) Avonex (inter eron-β -1a) • Dia n s MS?
B) Amantadine • Disc ss p t ntia t atm nt pti ns a ai ab n -t m
C) Copaxone (glatiramer acetate) dis as m di cati n as w as ac t xac bati ns?
D) Betaseron (inter eron-β -1b) • r c niz act s that mi ht s t in xac bati ns MS?
512 FAMIl y MeDICINe eXAMINATIo N & Bo Ar D r evIeW

Answer 18.6.2 The correct answer is “C.” Myasthenic crisis


CASE 18.6 can be due to two causes. First, the patient may have not taken
A 43-year-old woman with a history o myasthenia gravis enough medication or may have missed doses. In this case,
presents to the ED while on vacation. She reports she is eel- edrophonium will improve symptoms. T e second cause is too
ing tired and rundown and endorses u-like symptoms in much pyridostigmine. T is will also cause weakness. In this
addition to some worsening o her proximal lower extrem- case, the edrophonium will worsen the patient’s symptoms.
ity weakness. On examination, she is a ebrile with a respi- Here’s how it works. Pyridostigmine and edrophonium are both
ratory rate o 18. She has mild diplopia with lateral gaze. cholinesterase inhibitors similar to organophosphates. T ey act
Her strength is 4/5 proximally and 4+ /5 distally bilaterally. by binding to acetylcholinesterase and preventing the break-
Her sensory examination is normal. Plantar responses are down o acetylcholine. oo much o either drug (or a combina-
down-going bilaterally. tion o the drugs) will cause weakness and an organophosphate
toxicity-like syndrome (salivation, lacrimation, de ecation, uri-
Question 18.6.1 In determining this patient’s urther dispo- nation, weakness, etc.).
sition, what is the most important test?
A) Arterial blood gas Question 18.6.3 Which o the ollowing IS NOT likely to
B) CXR contribute to the diagnosis o myasthenia gravis?
C) Head C A) ensilon (edrophonium) test
D) Spirometry ( orced vital capacity [FVC] and negative inspi- B) Nerve conduction studies
ratory orce [NIF]) C) Antithymocyte antibodies
E) CBC D) Antiacetylcholine receptor antibodies

Answer 18.6.1 The correct answer is “D.” T is patient is Answer 18.6.3 The correct answer is “C.” Antithymocyte anti-
experiencing an exacerbation o myasthenia gravis. T is bodies are used to treat renal rejection and have also been used
could be occurring or any number o reasons including con- in aplastic anemia, red cell aplasia, and other disorders. T ey
current illness or possibly noncompliance with her regimen. have no relationship at all to myasthenia gravis . . . sorry. All o
T e greatest morbidity and mortality or this patient lies in the others can be used in the diagnosis o myasthenia gravis.
the potential or respiratory ailure and arrest. In primary T e edrophonium test is a unctional test. One must be ready
neuromuscular respiratory ailure (e.g., myasthenia gravis, to intubate the patient when per orming an edrophonium test,
acute in ammatory demyelinating polyradiculoneuropa- as weakness may get worse. Nerve conduction studies show a
thy, GBS), the arterial blood gas may remain normal despite reduction in the amplitude o the response to repeated stimu-
impending respiratory collapse. T e best way to evaluate lation (thus, patients get weaker with repeated muscle use).
respiratory status is with the FVC and NIF. I the FVC is less Antiacetylcholine receptor antibodies are ound in the majority
than 15 mL/kg or NIF less than − 20 cm H 2O, elective intu- (80–90%) o patients with myasthenia gravis. In act, when an
bation is recommended, although some centers will choose ultrasensitive test is done, it is ound in 70% o “antibody nega-
to monitor these patients closely in an intensive care setting. tive” cases o myasthenia gravis.
Once a myasthenic patient has been intubated, you should
stop their pyridostigmine as it will increase respiratory secre- HELPFUL TIP:
tions and GI motility leading to diarrhea. Neither o these is M asth nia a is can b s st mic imit d t th c -
desirable in an intubated ICU patient (the nurses will love you a m sc s. It is t n ass ciat d with a th m ma (
or this one) and the pyridostigmine will not liberate them th m s h p p asia), and pati nts with m asth nia a-
rom the ventilator. Monitoring the FVC/NIF should be done is sh d ha a ch st CT scan t t th m ma
regularly throughout the hospital course until the patient is th m s h p p asia. I p s nt, m a th th m ma
clinically improved and stable. wi t n “c ” th pati nt’s dis as .

You decide to give this patient a dose o edrophonium try-


ing to reverse her symptoms. When you do this, she becomes Question 18.6.4 Which o the ollowing is LEAST likely to be
increasingly weak, requiring intubation. con used with myasthenia gravis on the basis o its neuro-
logic symptoms?
Question 18.6.2 The BEST explanation o this is: A) Eaton–Lambert syndrome
A) Since she has missed multiple doses o her pyridostigmine, B) Guillain–Barré syndrome (GBS)
she has become desensitized and will have an overwhelming C) Amyotrophic lateral sclerosis
response to small doses o IV edrophonium D) Botulism toxicity
B) In uenza has made her particularly susceptible to edropho- E) Penicillamine-induced myasthenia gravis
nium
C) T e patient has taken too much pyridostigmine by accident Answer 18.6.4 The correct answer is “B.” Remember that
D) T e alcohol that she has had on vacation has changed her myasthenia gravis has no sensory ndings. GBS includes sensory
pyridostigmine requirement ndings o pain, paresthesias, numbness, etc. that are generally
Ch Apt Er 18 • Neu r o l o g y 513

absent in the other syndromes. “A,” Eaton–Lambert syndrome, Objectives: Did you learn to . . .
is a paraneoplastic process, which consists o weakness that gets • Mana a pati nt with an xac bati n m asth nia a is?
better with repetitive movement. T is is the exact opposite o • u nd stand th s dia n stic t sts in m asth nia a is?
what is seen with myasthenia gravis where repetitive tasks lead • r c niz dia n s s that can b c n s d with m asth nia
to increased weakness. T us, Eaton–Lambert syndrome is o en a is?
worse in the morning and better toward the a ernoon—the
reverse o what is seen with myasthenia gravis. Patients with CASE 18.7
amyotrophic lateral sclerosis, botulism, and penicillamine-
induced myasthenia gravis do not have sensory symptoms. A 29-year-old woman presents to the ED with sudden onset
T us, these can be con used with myasthenia gravis. o a severe headache involving bilateral occipital pain asso-
ciated with nausea. T e headache has not responded to her
HELPFUL TIP:
sumatriptan (Imitrex) injection. She has a history o migraine
Al S s ts in pp and w m t n n d ath. With
headaches consisting o right-sided throbbing pain that typi-
pp m t n n dis as , x s main intact and
cally respond to sumatriptan but occasionally require IV
a in act xa at d a in th dis as . As Al S p -
ketorolac and metoclopramide. She appears to be in moder-
ss s and m n ns a in d, x sa st.
ate pain but otherwise has a normal general and neurological
examination. T is is the worst headache o her li e.

Question 18.7.1 What is the next step in the management


T e patient and her husband have some questions about o this patient?
myasthenia gravis and are wondering i there are any medi- A) Ketorolac ( oradol) and metoclopramide IV
cations that might exacerbate this patient’s weakness. B) Hot tea and so pillow
C) Dihydroergotamine (DHE) IV
Question 18.6.5 Which o the ollowing can worsen myas-
D) Head C
thenia gravis?
E) Lumbar puncture
A) Fluoroquinolones
B) Verapamil Answer 18.7.1 The correct answer is “D.” Although this patient
C) β -blockers has a history o migraines, she is reporting a sudden onset head-
D) Oral contraceptives ache that is markedly changed rom her typical pattern o head-
E) All o the above ache. In this setting—especially with the “worst headache o her
li e”—the diagnosis o subarachnoid hemorrhage (SAH) must
Answer 18.6.5 The correct answer is “E.” All o the above can be ruled out. None o the other answers are correct. While pain
worsen myasthenia gravis. Other drugs o note include amino- management can be given be ore C (e.g., IV morphine or en-
glycosides, anesthetic and paralytic agents, diuretics, tetracy- tanyl), ketorolac, aspirin, and DHE are inappropriate i there is a
clines, and magnesium, among many others. question o SAH. Ketorolac and aspirin have antiplatelet ef ects,
which can increase bleeding. DHE can cause vasospasm and
You decide to add to the treatment o this patient. worsen brain ischemia. Finally, i you recall, this patient tried
her sumatriptan. One should not use DHE within 24 hours o a
Question 18.6.6 Which o the ollowing is considered a triptan. Note that in a patient with a non ocal examination who
standard therapy or myasthenia gravis? is immunologically intact LP may be done be ore C .
A) Plasmaphoresis or an acute myasthenic crisis
B) IVIG T e patient’s head C is negative.
C) Prednisone
D) Azathioprine Question 18.7.2 What next?
E) All o the above are used or myasthenia gravis A) Lumbar puncture
B) IV hydroxyzine and meperidine and discharge when she is
Answer 18.6.6 The correct answer is “E.” Do you see a pat- com ortable
tern here? All o the treatments are immunomodulators. Most C) IV hydroxyzine and promethazine (Phenergan) and dis-
people will start with prednisone and azathioprine. Other charge when she is com ortable
options are cyclosporin and mycophenolate. T e idea here is to D) Discharge home with prescription or acetaminophen and
reduce antiendplate antibodies (although nobody is quite sure oxycodone (Percocet)
how IVIG works). Prednisone is generally considered rst-line E) Discharge home with prescription or rizatriptan (Maxalt)
therapy or an exacerbation. It may be that immunomodulators
will become rst line (many neurologists are already using them Answer 18.7.2 The correct answer is “A.” In the setting o a
as rst line and using pyridostigmine only as a bridge until the “worst headache o li e” a C scan to rule out SAH is required.
immunomodulator kicks in). Plasma exchange can be used to T e sensitivity o C scan o the brain or hemorrhage in the set-
treat an acute exacerbation. Remember thymectomy as this may ting o SAH is 90–95% within 24 hours o the event (decreases
be a cure i a thymoma is present. to 80% at 72 hours). C only picks up 50% o warning leaks. As
514 FAMIl y MeDICINe eXAMINATIo N & Bo Ar D r evIeW

a result, a negative head C does not adequately rule out SAH Question 18.8.1 What is the most likely diagnosis?
and should be ollowed with a lumbar puncture. T e CSF rom A) Diabetic polyneuropathy
the lumbar puncture must be spun down immediately to examine B) Guillain–Barré syndrome (GBS)
or xanthochromia; delay in examining the CSF may result in C) Diabetic amyotrophy
alse-positive results. Xanthochromia is a yellow discoloration D) Stroke
o the normally clear CSF resulting rom degradation o hemo- E) Conversion reaction
globin. In addition to xanthochromia, markedly elevated RBC
counts are indicative o SAH. I either the C or LP is positive Answer 18.8.1 The correct answer is “B.” O the choices given
or SAH, an emergent our-vessel cerebral angiogram (or MR above, the most likely diagnosis is acute in ammatory demy-
angiogram or C angiogram) is indicated. elinating polyradiculoneuropathy (GBS). With an acute onset
o bilateral lower extremity weakness and sensory de cits, the
diagnosis o an acute cord-compressing lesion (e.g., tumor and
HELPFUL TIP: epidural abscess) should also be considered and ruled out, espe-
Thi t -nin p c nt pati nts with an SAH ha n cially when back pain is also present. T e time course described
n ic si ns s mpt ms. o n 10% pati nts above is not consistent with diabetic polyneuropathy nor would
with SAH ha an initia ca xaminati n. Pati nts one expect to see weakness as a prominent symptom. Diabetic
with SAH ma ha a and k c t sis. Whi amyotrophy is characterized by pain ul proximal muscle weak-
kin at th pati nt’s ndi is imp tant, th abs nc ness with minor sensory loss. T e onset o diabetic amyotrophy
papi d ma d s n t t SAH. SAH can p s nt can be subacute or acute. T e time course described above o
as back pain. Sinc ach b d ca i s a 50% m ta it , gradually progressing de cits is not consistent with stroke. Addi-
this is n dia n sis d n t want t miss. u s l P tionally, the ndings o a stroke should not be bilateral.
ib a in app p iat pati nts.
Question 18.8.2 Which o the ollowing is/are associated
with GBS?
A) Campylobacter jejuni in ection
HELPFUL TIP: B) Lyme disease
y ma h a ab t a c p st di s that ha C) Epstein–Barr virus
sh wn that a CT scan d n within 6 h s h ad- D) CMV virus
ach ns t is s nsiti n ht t SAH with t E) All o the above
th n d an l P. This has d init n t tb c m
th standa d ca . Answer 18.8.2 The correct answer is “E.” All o these in ectious
agents are associated with GBS, especially Campylobacter Other
associations include viral URIs, HIV, immunizations (rare),
mycoplasma, epidural anesthesia, sarcoid, lupus, etc. Add to this
You complete the LP, and it is normal. Apparently, this was
the Zika virus and the West Nile virus. T e point here is that one
just the worst migraine o her li e. A er loading her up on
should look or an underlying illness in patients with GBS.
some good drugs, her pain subsides. Problem solved!
Objective: Did you learn to . . .
HELPFUL (a ND INTERESTING) TIP:
• Id nti a pati nt p s ntin with SAH s mpt ms and p -
m an app p iat w k p? Antib di s t Campylobacter ha b n sh wn t c ss-
act with n tiss , th s th ass ciati n Campy-
lobacter with g BS.
CASE 18.8
A 40-year-old man presents with a complaint o low-back pain Question 18.8.3 Which o the ollowing actions would NOT
that is dull in nature, which started 2 days ago. T is morn- be appropriate or additional diagnosis and/or manage-
ing he woke up with a eeling o numbness and tingling in his ment o this patient with progressive weakness?
eet, which gradually seemed to worsen. By noon, he noted A) Cardiac monitoring
dif culty walking and decided to come to the ED. He denies B) Forced vital capacity (FVC)
bowel or bladder incontinence. On examination, he is in no C) Discharge to home on steroids with a ollow-up in the
acute distress and has a respiratory rate o 12. He has strength morning
o 5/5 in his upper extremities, and in his lower extremities, D) EMG/NCV
strength is 4/5 proximally and 3/5 distally. Sensory examina-
tion reveals a mild decrease in pinprick and light touch in a Answer 18.8.3 The correct answer is “C.” T is patient should
stocking distribution to the mid-cal . Re exes in the upper not be sent home. Did you even consider it? Discharge is a
extremities are 2+ /4 and in the lower extremities are trace particularly bad idea because his disease has worsened rapidly
at the knees and absent at the Achilles. Plantar response is over the last 12 hours. As with other potential causes o neuro-
down-going bilaterally. muscular respiratory ailure, an FVC and NIF are necessary to
Ch Apt Er 18 • Neu r o l o g y 515

determine adequate respiratory reserve. T e FVC and NIF Objectives: Did you learn to . . .
should be monitored closely during the acute illness. GBS can • r c niz th c inica p s ntati n g BS?
have a rapid and catastrophic worsening that necessitates moni- • Id nti nd in i n ss s that a ass ciat d with g BS?
toring during the acute phase o illness. ypically, patients reach • Mana a pati nt with g BS?
the peak o severity about 2 weeks into the illness. Autonomic
dys unction is associated with GBS, and close cardiovascular
monitoring is important. EMG/NCV can be o value in diag- CASE 18.9
nosing GBS—although early on in the course o the disease, A 38-year-old woman is brought to the ED by her husband
these tests may be normal. who expresses concerns over changes in her mental status
over the past 2 days. She has become con used, orgetting
A diagnosis o GBS is made. the names o persons well-known to her, and orgetting
what she is doing. Her conversations have become increas-
Question 18.8.4 Which o the ollowing is NOT an appropri- ingly more dif cult to ollow, and over the past 12 hours,
ate treatment modality? she has gradually become sleepier. On examination, she has
A) IVIG a temperature o 38.0°C. She is drowsy but can be aroused.
B) Plasma exchange She has no meningismus. She is oriented only to person. She
C) Elective intubation i FVC is < 15 cc/kg responds to questions slowly and incorrectly and ollows only
D) Corticosteroids simple commands (“stick out your tongue”). T e remainder
o her neurological examination is essentially normal, given
Answer 18.8.4 The correct answer is “D.” Corticosteroids are her limited ability to cooperate. CBC, coagulation studies,
not used in the treatment o GBS. Multiple studies have shown and electrolytes (including calcium, magnesium, and phos-
no bene t to corticosteroids in this disease. reatment options phorous) are normal.
or GBS include care ul monitoring o disease and supportive
therapy with no intervention. Some experts recommend initiat- Question 18.9.1 What is the best next step in evaluating
ing IVIG or plasma exchange. I a patient requires ventilatory this patient?
support or has weakness that precludes ambulation, treatment A) Lumbar puncture
should be started immediately. Elective intubation is appropri- B) Head C
ate i the FVC is less than 15 cc/kg or the NIF is –20 cm H 2O. As C) Electroencephalogram (EEG)
discussed in regard to neuromuscular respiratory ailure with D) Chest x-ray (CXR)
myasthenia gravis, arterial blood gases are not reliable markers
o impending ailure, and the FVC must be closely monitored.
Answer 18.9.1 The correct answer is “B.” Although per-
orming all o the above tests will be help ul in evaluating this
T e amily would like to know what the outcome in this
patient, the most important test to do at this time is a head C
patient with GBS will be, so you pull out your crystal ball.
to rule out any mass lesion or hemorrhage. You could argue
that “A,” an LP, could be done rst, and some experts would
Question 18.8.5 Well, instead o sarcasm, you actually opt
agree with that decision. However, the standard in the United
or compassion and tell them that FULL RECOVERY can be
States is to do the C scan rst i there is any possibility o a
expected in __________ percent o patients.
mass lesion. T ere is a possibility that nonconvulsive status epi-
A) 15%
lepticus is causing these mental status changes. However, even
B) 50%
i the EEG showed nonconvulsive status epilepticus, a head C
C) 80%
and lumbar puncture would remain necessary. Although pneu-
D) Greater than 95%
monia could cause con usion, a CXR is unlikely to be o high
yield in this setting.
Answer 18.8.5 The correct answer is “A.” Fi een percent o all
patients with GBS will have complete resolution o their symp- Her head C is normal. You per orm a lumbar puncture,
toms. Sixty- ve percent will be le with minor de cits, and which reveals 18 WBCs (all lymphocytes), 12 RBCs, CSF
about 10% will become disabled. Despite excellent care, some protein 67 mg/dL (elevated), and CSF glucose 70 mg/dL
patients still die. (normal). An EEG is normal.

HELPFUL TIP:
Question 18.9.2 What is the next step in managing this
W st Ni i s in cti n can p s nt as ith a g BS
patient?
(s mm t ica n ica s mpt ms with ss x-
A) Admit or viral encephalitis with close monitoring
s) a p i m itis- ik s nd m ( n a as m-
B) Admit or viral encephalitis and start acyclovir and
m t ica w akn ss with w s w akn ss p xima ,
antibiotics
ss x s, ascic ati ns).
C) Admit or bacterial meningitis and start antibiotics
D) Discharge to home with close ollow-up tomorrow at 8 am
516 FAMIl y MeDICINe eXAMINATIo N & Bo Ar D r evIeW

Answer 18.9.2 The correct answer is “B.” T e probable diag- incorrect. T e classic triad o NPH includes urinary inconti-
nosis in this setting is herpes simplex encephalitis. Her CSF nence, gait ataxia, and dementia. “D” might be a consideration,
results are consistent with a viral process and are not consis- but there is no speci c nding o progressive supranuclear palsy
tent with bacterial meningitis (due to the normal glucose and here (e.g., aggressive course, more severe axial rigidity, dysar-
only lymphocytes in the dif erential). Given the mortality o thria, dysphagia, and downward gaze paresis). Stroke is quite
herpes simplex encephalitis (30–70%), all patients in whom the unlikely to present in this insidious ashion with generalized
diagnosis is suspected should be started on acyclovir 10 mg/kg ndings.
every 8 hours IV empirically (with appropriate dosage adjust-
ments or renal ailure), while urther con rmatory testing is Question 18.10.2 Which o the ollowing is NOT a common
pending (CSF PCR or the herpes simplex virus). reatment eature o idiopathic Parkinson disease?
should be or a minimum o 10 days and has been advocated A) Rigidity
(with little evidence to support it) to be as long as 21 days. It B) Extraocular movement paresis
is important to recognize that the EEG and C may be normal C) Bradykinesia
in herpes simplex encephalitis, particularly early in the dis- D) Loss o postural re exes or gait disturbance
ease. However, on C , one may see evidence o temporal lobe E) Asymmetric tremor
hemorrhage and/or hypodensity in the temporal lobes. EEG
can show either periodic lateralized epilepti orm discharges or Answer 18.10.2 The correct answer is “B.” T ere are our
ocal slowing in the temporal lobes. emporal lobe changes may cardinal eatures o Parkinson disease: tremor, bradykinesia,
be even more prominently visualized on MRI, and this may rigidity, and postural instability. wo or more o these eatures
be o bene t in cases in which the diagnosis remains unclear. should be present to make the diagnosis. T e tremor o Parkin-
Even with prompt, appropriate treatment, only 38% o patients sonism is a resting tremor (as opposed to the postural, inten-
returned to normal or near-normal neurologic unctioning at tion, or action tremor) and is most common in the hands, typi-
2 years. “B” includes antibiotics because most clinicians would cally with one side af ected more than another. Rigidity (“A”) is
cover or bacterial meningitis until the CSF cultures come back described as increased resistance to passive movement and may
negative. be rein orced by having the patient open and close the st on
the opposite hand. Cogwheel rigidity is a ratchet-like sensation
Objectives: Did you learn to . . . noted when testing a limb with concurrent tremor. Bradykine-
• r c niz th c inica p s ntati n and th ab at
sia (“C”) may be observed by monitoring the speed and ampli-
ndin s in i a nc pha itis?
tude o movements. Gait disturbance (“D”) with reduced stride
• Initiat app p iat t atm nt in a pati nt with a p s mp-
length and stooped posture is a common nding but generally
ti dia n sis h p s nc pha itis?
occurs later in the course o the disease. Postural re exes and
ability to rise rom a chair are also impaired. Postural re exes
CASE 18.10 may be tested by retropulsion.
A 70-year-old male presents to your of ce as a new patient.
He is with his wi e, who assists in providing the history. His You are airly certain that your patient has Parkinson disease.
appetite is reduced, and he has lost 10 lb in the past 6 months.
His only medication is aspirin, and he has no signi cant past Question 18.10.3 What else might you nd with idiopathic
medical history. Parkinson disease?
On examination, his vital signs are normal, and he is in A) Micrographia
no acute distress. His gait is slow, and he takes eight steps to B) Urinary incontinence
turn. He has retropulsion (takes two steps backward when C) Apraxia
you pull him rom behind). T ere is a resting tremor in both D) Autonomic dys unctions
hands but more prominently in the right. You nd cogwheel E) Alien limb phenomenon
rigidity in both arms as well, but again more prominently dis-
played on the right. His cognitive screening tests are normal. Answer 18.10.3 The correct answer is “A.” Micrographia,
writing in small letters, is associated with Parkinson disease.
Question 18.10.1 The most likely diagnosis is: T e others are not. T is is important because there are a num-
A) Essential tremor ber o neurologic syndromes that can mimic Parkinson dis-
B) Parkinson disease ease, and knowing the additional eatures o Parkinson disease
C) Normal pressure hydrocephalus (NPH) will help you to dif erentiate it rom other diseases. Decreased
D) Progressive supranuclear palsy acial expression (hypomimia) with decreased rate o eye blink
E) Stroke and diminished vocal volume (hypophonia) are also com-
mon with Parkinson disease. Other conditions that occur in
Answer 18.10.1 The correct answer is “B.” T is patient most patients with Parkinson disease include depression, cognitive
likely has Parkinson disease. “A” is incorrect because essential impairment, and REM sleep behavioral disorder (e.g., kicking
tremor is typically worsened by activity and is not associated and screaming during REM sleep in response to vivid, disturbing
with the other neurologic ndings seen in this patient. “C” is dreams).
Ch Apt Er 18 • Neu r o l o g y 517

A) ACE inhibitors
HELPFUL TIP:
B) HMG-CoA reductase inhibitors
Th a “Pa kins n p s” s nd m s, s -ca d
C) Calcium channel blockers
b ca s th p s nt with pa kins nian at s with
D) Metoclopramide
an th cha act istic. l k th s s nd m s in
E) All o the above
th s that b i ma ha Pa kins n dis as :
(1) P ssi s p an c a pa s , which is ass ciat d
Answer 18.10.5 The correct answer is “D.” Metoclopramide
with s p an c a az pa s , d sa th ia, d spha ia,
is a requent cause o the misdiagnosis o Parkinson disease
p st a instabi it , and axia i idit .
in the elderly. Additional medications, such as SSRIs, antipsy-
(2) Sh –D a s nd m , which is n tab a t -
chotics, and others, can mimic Parkinson disease. Importantly,
n mic d s ncti n, inc din ma k d th static
drug-induced parkinsonism may last or up to 6 months a er
h p t nsi n.
discontinuation o the o ending agent.
(3) C tic basa an i nic d n ati n, which is ass -
ciat d with ap axia, c tica s ns d s ncti n, and
You think it is best to initiate treatment in this patient.
th “alien limb enomenon” (ICD-10 c d r 41.4 i
a int st d). A i n imb ph n m n n cc s
Question 18.10.6 Possible treatments o Parkinson disease
wh n th pati nt’s a m m s b its ( . ., wi ach
include all o the ollowing EXCEPT:
p t t ch th pati nt’s ac ). Th pati nts d n t b -
A) Levodopa
i that th imb b n s t th m. Sp ntan s imb
B) Pallidotomy
m m nts a s cc wh n th pati nt is sta t d
C) Pramipexole
th imb is t ch d (an n s D . St an ?).
D) Donepezil
E) Selegiline
Question 18.10.4 The diagnosis o Parkinson disease is
Answer 18.10.6 The correct answer is “D.” Donepezil is used
most appropriately made:
to treat Alzheimer disease. T e initial symptoms o Parkinson
A) With a brain MRI
disease typically respond well to levodopa and the dopamine
B) By CSF analysis
agonists. Selegiline is a monamine oxidase B inhibitor and
C) Clinically
yields modest symptomatic bene ts. Dopamine receptor ago-
D) By a neurologist
nists, such as pramipexole (Mirapex), ropinirole (Requip), and
E) At autopsy
bromocriptine, are important medications that may be used in
the initial treatment o Parkinson disease.
Answer 18.10.4 The correct answer is “C.” T e diagnosis is
based on a history and physical examination that are consistent
with Parkinson disease. Additionally, a response to dopami- HELPFUL TIP:
nergic agents is virtually diagnostic o Parkinson disease. I the S ica pti ns ad anc d Pa kins n dis as inc d
patient does not have some response to a dopaminergic agent, pa id t m and d p b ain stim ati n. In n a,
reconsider your diagnosis. “A” is incorrect because there are no stim ati n th tha am s is s d t t at t m b t
ndings on neuroimaging that are speci c or the diagnosis o has itt impact n th th s mpt ms Pa kins n
Parkinson disease. Likewise, “B” is wrong because CSF analy- dis as (b ad kin sia, tc.). S btha amic n c s stim-
sis cannot provide the diagnosis. “D” is clearly wrong—do you ati n is m b n icia am i atin th th
really need a neurologist or this? T e “gold standard” or diag- at s Pa kins n dis as , b t ma ha a ss p -
nosis is neuropathologic examination, but you would rather not nd impact n t m .
wait or the autopsy to diagnose Parkinson disease.
Question 18.10.7 Which o the ollowing is NOT a common
HELPFUL TIP:
side ef ect o levodopa?
u p t 10% pati nts with Pa kins n dis as wi ha
A) Nausea
s m d int nti n t m in additi n t th i
B) Paresthesias
stin t m .
C) Dyskinesia/dystonia
D) Hallucinations

Answer 18.10.7 The correct answer is “B.” Paresthesias are not


Your patient takes only aspirin, which you know does not associated with the use o carbidopa/levodopa. T e use o carbi-
cause parkinsonism. dopa with levodopa allows the dose o levodopa to be optimized.
Carbidopa helps to prevent levodopa-induced nausea (“A”). Dys-
Question 18.10.5 Which o the ollowing medications is re- tonia and dyskinesia (“C”) are common with therapy lasting more
quently associated with a Parkinson-like syndrome in the than 2 years or at peak dose responses and may necessitate lowering
elderly? the doses. Psychiatric problems (“D”), including hallucinations and
518 FAMIl y MeDICINe eXAMINATIo N & Bo Ar D r evIeW

psychosis, can be seen with dopaminergic agonists and levodopa.


Other common side ef ects o levodopa include hypotension, con- When you counsel your patient regarding medication use,
usion, and other psychiatric disturbances. you urge him not to stop taking his medication all at once.

Question 18.10.9 Which o the ollowing is NOT a potential


HELPFUL TIP: adverse ef ect o abrupt discontinuation o dopaminergic
l d pa is a d pamin p c s , and ca bid pa is a agonists and/or levodopa?
p iph a d pad ca b x as inhibit that d s n t A) Neuroleptic malignant syndrome (NMS)
c ss th b d–b ain ba i . Th s mpt ms t m , B) Severe rigidity
i idit , and b ad kin sia a initia i d b - C) Con usional state
d pa. H w , with tim , a d s sa q i d D) Severe dyskinesias
t maintain c nt s mpt ms.
Answer 18.10.9 The correct answer is “D.” Levodopa and
dopamine agonists should be tapered. Abrupt discontinuation
HELPFUL TIP: o these medications may precipitate NMS. In addition, abrupt
on th d pamin a nists (p amip x , pini , withdrawal is also associated with an acute con usional state
and b m c iptin ) can b s d t h p minimiz th separate rom the mental status changes seen in NMS. Severe
d s d pa n d d, th s minimiz th sid cts worsening o the patient’s parkinsonism is expected, which can
d pa. result in prominent rigidity. Dyskinesias are requently seen
with dopaminergic agonist/levodopa therapy, but are not exac-
erbated or triggered by withdrawal o these agents.
You decide to start levodopa/carbidopa (Sinemet) and bro-
mocriptine. At the ollow-up visit, the patient is doing rela-
HELPFUL TIP:
tively well on this combination. However, he notices that
Dia n stic c it ia d init NMS inc d h p th -
in the mornings and evenings, he tends to be sti and has
mia, m sc i idit , and i th win : m nta
dif culty ambulating.
stat s chan s, t m , tach ca dia, inc ntin nc , abi
b d p ss , m tab ic acid sis, tach pn a/h p xia,
Question 18.10.8 The most appropriate next step is to:
ati n c atin kinas , diaph sis/sia h a,
A) Initiate a “drug holiday” to restore the patient’s sensitivity to
and k c t sis. T atm nt inc d s s pp ti ca
the drug
as w as th s b m c iptin and p p an ,a -
B) Add another dopaminergic agent such as ropinirole (Requip)
th h th is imit d data t s pp t th s ith
C) Add a catechol-O-methyl-trans erase (COM ) inhibitor
a nt. Dan olene is ineffec ive.
such as entacapone (Comtan)
D) Add an anticholinergic agent such as benztropine (Cogentin)

Answer 18.10.8 The correct answer is “C.” T e patient is expe- A year later your patient returns and seems to be doing well.
riencing the “wearing-of ” phenomenon. T ere are several ways You ask him about symptoms o Parkinson disease, i he is
to address this. One option is to add a COM inhibitor (like having any adverse e ects o medications, and i he is experi-
tolcapone [ asmar] or entacapone [Comtan]) in order to slow encing any o the Parkinson-associated diseases.
down the metabolism o levodopa. COM inhibitors have no
ef ect on their own and should only be used with levodopa. Question 18.10.10 Which o the ollowing is NOT associated
Another option is to switch the patient rom immediate- with Parkinson disease?
release carbidopa/levodopa to a sustained-release product (e.g., A) Depression
Sinemet CR). T e ef ectiveness o this strategy is not conclu- B) Dementia
sively proven, and there is no bene t rom using controlled- C) REM sleep disorder
release carbidopa/levodopa as the initial agent in Parkinson D) Narcolepsy
disease. Another common approach to the “wearing-of ” phe- E) Decreased visual contrast sensitivity
nomenon is altering the dosing o carbidopa/levodopa—either
increasing the dose or shortening the interval between doses. Answer 18.10.10 The correct answer is “D.” Narcolepsy has
not been associated with Parkinson disease. However, excess
HELPFUL TIP: daytime sleepiness has been associated. T is is likely due to a
T cap n (Tasma ) is ass ciat d with ata h patic n - combination o actors, such as sleep disturbance, depression,
c sis and q i s m nit in i ncti n t sts dopaminergic drugs, and Parkinson disease itsel . Depres-
(l FTs). Th s, ntacap n (C mtan) is th p dd . sion is commonly seen in Parkinson disease and is reported to
A cap n is ass ciat d with tax asi n and “hits.” This occur in up to 41% o patients. Dementia, typically with Lewy
Cap n sh d b a id d as w ( t it? A Cap n ??). bodies present on pathological analysis, is seen and can af ect
the decision to proceed with surgical treatment o Parkinson
Ch Apt Er 18 • Neu r o l o g y 519

disease, as patients with advanced dementia get less bene t D) Work schedule
rom surgery. Also, REM sleep behavior disorders are seen in E) Listening to “so rock” (“easy listening” or “adult contem-
Parkinson disease and can be a source o stress or amilies and porary”)
caretakers. REM sleep behavior disorder is characterized by
acting out o dreams that can consist o vocalizations as well Answer 18.11.2 The correct answer is “E.” Okay, maybe this
as active and even violent movements. ypically, REM sleep one was too easy, but we hate “easy listening” music. T e Geneva
behavior disorder responds to clonazepam. Finally, decreased Convention prohibits its use on prisoners o war . . . we’ve been
visual contrast sensitivity can occur with Parkinson disease told. All ranting aside, this tidbit o her history has no bearing
as well. on her sleep. Important parts o the sleep history include the
sleeping environment (alone, with spouse or other individual,
in the daylight hours, etc.), nap history, amily history o sleep
HELPFUL TIP:
problems, and symptoms o speci c sleep disorders (snoring,
Th d pamin a nists (p amip x , pini , tc.)
hypnagogic hallucinations, etc.). A medication history is impor-
ha b n ass ciat d with c mp si b ha i (s x a
tant as well as a history o watching V in bed, eating in bed,
c mp si n, c mp si amb in , tc.). This ma x-
etc., which indicate poor sleep hygiene. As the sleep experts say,
p ain wh d pati nts ik t t l as v as
your bed should be used or only two purposes: sleep and sex
and At antic Cit . . . .
(not necessarily in that order).

Objectives: Did you learn to . . .


• Id nti c mm n at s Pa kins n dis as ? HELPFUL TIP:
• Dia n s Pa kins n dis as ? on th a it pa as mnias th dit s is “ x-
p din h ad s nd m ” (this is act a a, n t a
• Mana a pati nt with Pa kins n dis as ?
j k ). Pati nts ha th in p ppin xp -
• u nd stand pha mac th ap a ai ab Pa kins n
si ns cc in in th i h ad as th a as p.
dis as and s m th p t ntia ad s cts d
th ap ?

Question 18.11.3 Which o the ollowing is NOT part o a


CASE 18.11 typical sleep study?
A 45-year-old le -handed woman who is a busy executive A) Monitoring o EEG
or a Fortune 500 company presents with excessive daytime B) Monitoring respirations and oxygen desaturations
sleepiness. She is otherwise healthy and takes no medications. C) Evaluating the sleep latency in response to sleep aids, such
as zolpidem and trazodone, to maximize ef ective pharma-
Question 18.11.1 How do you approach this problem? cologic therapy
A) Reassure her that it is normal or people to be drowsy under D) Monitoring EMG
stress ul work conditions E) Video monitoring o sleep
B) Begin zolpidem at night or sleep
C) Schedule the patient or polysomnography Answer 18.11.3 The correct answer is “C.” Sleep studies are
D) Discuss the patient’s sleep hygiene generally done in the naıve state without the use o medications.
E) Administer moda nil (Provigil) prior to important board All o the rest are true. “D” and “E” deserve more attention. “D,”
meetings monitoring EMG, allows in orming the physician about muscle
activity during sleep (e.g., restless limbs). “E,” video monitoring
Answer 18.11.1 The correct answer is “D.” It is essential and taping, allows the physician to look or problems, such as
that all patients with complaints o either insomnia or exces- awakening, evidence o restless leg syndrome, and sleep apnea.
sive daytime sleepiness have a thorough sleep history taken.
Polysomnography may be necessary, but this test—like all Question 18.11.4 In considering a diagnosis o narcolepsy,
diagnostic tests—should be driven by a hypothesis. A history which o the ollowing is NOT part o the diagnosis?
is necessary to develop a hypothesis and determine a test’s A) Cataplexy
utility. Starting hypnotic agents (“B”) or stimulants (“E”)—or B) Sleep paralysis
both—without thoroughly investigating the underlying prob- C) Hypnagogic hallucinations
lem may cover up a signi cant, treatable problem and is con- D) Sleep myoclonus
sidered bad orm. E) Excessive daytime sleepiness

Question 18.11.2 Which o the ollowing is NOT an impor- Answer 18.11.4 The correct answer is “D.” Sleep myoclonus
tant aspect o a sleep history? (hypnagogic jerks) is commonly seen in normal people as they
A) Sleeping and waking times begin to all asleep (witness colleagues at grand rounds with sud-
B) Use o stimulants and alcohol den tossing o the head) and is not a part o narcolepsy. Narco-
C) Sleep interruptions (children, pagers, etc.) lepsy is a disorder characterized by our cardinal traits, although
520 FAMIl y MeDICINe eXAMINATIo N & Bo Ar D r evIeW

not all need be present to make the diagnosis: cataplexy, exces-


HELPFUL TIP:
sive daytime sleepiness, sleep paralysis, and hypnagogic halluci-
It is imp tant t n t that n t a pati nts with s p
nations. “A,” cataplexy, is a sudden loss o voluntary muscle con-
apn a a w i ht, and this dia n sis m st b c n-
trol during which the patient may appear to be asleep; however,
sid d in a pati nts with xc ssi da tim s pin ss
cataplexy does not have to be accompanied by sleep attacks,
th s sti s mpt ms. Sma pha n a
and the patient may be aware throughout the attack. “B,” sleep
ai wa ( sp cia in in w m n) and ast s pha-
paralysis, can occur either at the onset o sleep or upon awaken-
a x a ass ciat d with s p apn a.
ing and can be quite rightening to the patient. “C,” hypnagogic
hallucinations, are vivid and typically ear ul dreams that occur
at the onset o sleep but can also occur upon awakening (hyp- Question 18.11.7 What treatment options would NOT be
nopompic hallucinations). “E,” excessive daytime sleepiness, is appropriate to consider in your patient i she has obstruc-
a hallmark o narcolepsy and can include sleep attacks as well tive sleep apnea?
as persistent drowsiness and “micro sleep” (brie intrusions o A) Bilevel positive airway pressure (BiPAP)
sleep during a waking state). T e complete tetrad o symptoms B) Continuous positive airway pressure (CPAP)
is seen in only 10% o patients with narcolepsy. C) Positional therapy
D) Uvulopalatopharyngoplasty (UPPP)
Question 18.11.5 Which o the ollowing is NOT a treatment E) Zolpidem
or narcolepsy?
A) Amitriptyline Answer 18.11.7 The correct answer is “E.” Sleep aids, including
B) Clonazepam benzodiazepines, do not have a role in the treatment o obstructive
C) Fluoxetine sleep apnea and may actually worsen symptoms. CPAP and BiPAP
D) Moda nil (Provigil) are both potential treatments. Polysomnography (sleep testing)
E) Sodium oxybate with titration o CPAP or BiPAP should determine which modal-
ity to use and the pressure settings. T ese should not be arbitrarily
Answer 18.11.5 The correct answer is “B.” Clonazepam is not set to the “normal settings” or a patient. Positional therapy, avoid-
a treatment or narcolepsy. T e treatment o narcolepsy has two ing sleeping on one’s back, may be ef ective in some patients. Some
primary goals. T e rst goal is to address daytime sleepiness, techniques or achieving this goal include sewing an object on the
which is primarily done with stimulants such as moda nil or back o the pajama shirt that will irritate the patient when they
methylphenidate. T e second goal is to reduce the symptoms o roll onto it (i.e., tennis balls in a sock). Weight loss, although not
cataplexy. T is can be accomplished with agents such as tricy- mentioned above, can provide improvement in symptoms as well.
clic antidepressants, and to a lesser extent, SSRIs. Sodium oxy- Surgical therapies including UPPP can also be considered based
bate (also known as gamma hydroxybutyrate) can be used or on the patient’s symptoms and pre erences.
treatment o cataplexy as well as sleep hallucinations and sleep
paralysis. Some evidence suggests it may also help with exces-
sive daytime sleepiness. Given its street popularity and abuse HELPFUL TIP:
potential (including date rape), it should not be prescribed to J st as C ki M nst has a n d that c ki s a a
those with a history o drug abuse, etc. “s m tim s d,” h pn tic m dicati ns sh d n -
a b c nsid d a “s m tim s m dicin .” Pati nts d -
A er a thorough history, you nd nothing to suggest narco- pt anc t a c mm n s d h pn tics with th
lepsy besides daytime sleepiness. O course, you are consider- p ssib xc pti n m at nin. Additi na , s m h p-
ing other diagnoses simultaneously—unless you are experi- n tics ca s amn sia and th s ( ik th xin inhibi-
encing a sleep attack yoursel . t c ass [ . ., s xant (B s m a)]) ca s p n d
da tim s mn nc th n xt da . S xant can a s
Question 18.11.6 Which o the ollowing would NOT sug- ca s xt mit w akn ss and s m p p ca t
gest a possible diagnosis o obstructive sleep apnea? c mp x b ha i s (s ch as d i in ) whi nawa . H p-
A) Di culty alling asleep n tics n add ab t 15 t 30 min t s t a pati nt’s t ta
B) Frequent nighttime arousals s p tim . F man pati nts, th isk–b n it ati d s
C) Obesity n t a h pn tics. Sta t with s p h i n .
D) Paroxysmal nocturnal dyspnea
E) Snoring
Further history leads you to suspect that she has restless legs
Answer 18.11.6 The correct answer is “A.” Di culty alling syndrome, and you con rm the diagnosis with a sleep study.
asleep is not one o the components o obstructive sleep apnea.
Snoring, obesity, excessive daytime sleepiness, and paroxys- Question 18.11.8 Which would be a f rst-line agent or treat-
mal nocturnal dyspnea are all associated with obstructive sleep ment o restless legs syndrome?
apnea. Some patients will note requent arousals rom sleep A) Clonazepam
with or without accompanied shortness o breath. B) Codeine
Ch Apt Er 18 • Neu r o l o g y 521

C) Methadone B) Right-sided, electric, stabbing pain involving primarily the


D) Pramipexole throat, tongue, and right ear
E) ramadol C) Right-sided severe headache involving the orbit and associ-
ated with lacrimation and rhinorrhea typically occurring at
Answer 18.11.8 The correct answer is “D.” Restless legs syn- the same time each day or a given period o time.
drome is characterized by an urge to move the lower extremi- D) “Sinus pressure” with a history o sinus headaches respon-
ties due most o ten to an uncom ortable sensation. his sen- sive to antibiotics in the past
sation usually occurs during rest and is typically relieved E) Pattern o severe right-sided “stabbing and boring” head-
by moving the legs. Dopaminergic agents, either levodopa aches around the age o 30 that went away with scheduled
(e.g., Sinemet) or agonists, such as ropinirole (Requip) and indomethacin
pramipexole (Mirapex), are the irst-line treatments or rest-
less legs syndrome. A variety o other medications are used Answer 18.12.1 The correct answer is “D.” Sinus headaches
or restless leg syndrome but do not carry an FDA indication are typically a diagnosis o exclusion. Acute sinusitis can
or it. Benzodiazepines (“A”) and narcotic medications (“B” cause severe head and ace discom ort, but sinusitis remains
and “C”) as well as tramadol and gabapentin have all been a relatively uncommon etiology or recurrent head and
reported to be success ul alternative therapies or restless leg ace pain. T e speci c headache syndromes described in “A,”
syndrome. “B,” “C,” and “E” are described in more detail in the ollowing
questions.
Question 18.11.9 What other workup would you suggest
or this patient once the diagnosis o restless legs syn- Question 18.12.2 Let’s say this patient gives you the history
drome is made? o right-sided severe headache involving the orbit associ-
A) No urther workup is indicated, initiate treatment as ated with lacrimation and rhinorrhea typically occuring at
above the same time each day or a couple o weeks.. Which o the
B) Serum calcium ollowing would you use or initial acute treatment o this
C) Serum iron studies, including erritin headache syndrome?
D) Serum vitamin B12 A) Naproxen
E) Serum vitamin B6 B) Oxygen
C) ylenol
Answer 18.11.9 The correct answer is “C.” Studies have D) Verapamil
shown a link between low iron stores and restless legs syn-
drome. All patients with restless legs syndrome should have Answer 18.12.2 The correct answer is “B.” T e type o head-
an iron pro le per ormed, and low iron or erritin levels merit ache syndrome described is most consistent with cluster head-
iron supplementation (and workup or underlying cause, o aches. T ese are most commonly seen in men and are char-
course). Other associations are diabetes, pregnancy, end- acterized by exquisite pain, typically centered at the orbit.
stage renal disease, Parkinson disease, venous insu ciency, Conjunctival injection, rhinorrhea, and lacrimation requently
olate de ciency, and caf eine intake (dump that ourth cup accompany the headache. Pain is o en disabling. As the name
o cof ee . . . ) suggests, the patient tends to have headaches in groups (or clus-
Objectives: Did you learn to . . . ters). T ese headaches are more common at night, and REM
• Tak an app p iat s p hist ? sleep is thought to be a triggering actor. For acute treatment,
conventional headache medications such as DHE and the trip-
• g n at a di ntia dia n sis da tim s pin ss?
tans can be ef ective. reatment with high- ow oxygen has
• g ain ami ia it with th dia n stic t stin s d in a s p
also shown signi cant e cacy as an abortive treatment: typi-
ab at ?
cal protocol would be 8 L oxygen on nonrebreather or 15 min-
• Id nti th p s ntati ns and t atm nts c mm n
utes, with reports o 70% o patients achieving headache relie .
s p dis d s, inc din na c ps , bst cti s p
apn a, and st ss s s nd m ? Verapamil can be ef ective or prophylaxis but is not ef ective as
an abortive. Other, more typical migraine prophylactic agents
(propranolol, topiramate, indomethacin, valproic acid) have
CASE 18.12 been tried, but no systematic studies have been done to evaluate
A 60-year-old right-handed gentleman presents with the their e cacy in cluster headaches.
complaint o head pain.
Question 18.12.3 Let’s say your patient describes a history
Question 18.12.1 Which o the ollowing historical descrip- o severe right-sided “stabbing and boring” headaches
tions is o the LEAST VALUE in identi ying a speci c diag- around the age o 30 that went away with scheduled indo-
nostic classi cation or head/ ace pain? methacin. This description would be most consistent with
A) Right-sided, electric, stabbing pain involving primarily the which o the ollowing headache syndromes?
cheek, occurring or seconds to minutes repeatedly through- A) ension headache
out the day B) Paroxysmal hemicrania
522 FAMIl y MeDICINe eXAMINATIo N & Bo Ar D r evIeW

C) Migraine without aura


HELPFUL TIP:
D) Chronic daily headache
A ni at a ct ic, stabbin pain cc in in th
E) Analgesic rebound headache
t n , pha nx, and ccasi na xt ndin t th
ipsi at a a has b n d sc ib d; this is kn wn as glos-
Answer 18.12.3 The correct answer is “B.” Paroxysmal hemi-
so a yngeal neu algia. T atm nt ss pha n-
crania is classically described as a unilateral headache with a
a n a ia is simi a t th pha mac ic t atm nt
stabbing/boring character. Although age o onset can vary
t i mina n a ia d sc ib d ab . I th pati nt
greatly, it classically occurs in women in their 30s (although it
c mp ains stabbin t mp a h adach s ast-
can be seen in men as well). Patients will have between 2 and
in s c nds, c nsid th dia n sis “j ts and jabs”
40 episodes during a given day, although they do not cluster
h adach s (aka “ic pick” h adach ). Th cc 40 t
together as is typical in cluster headaches. Although autonomic
50 tim s a da m and a ik a mi ain a iant.
symptoms (rhinorrhea, lacrimation, conjunctival injection, and
ptosis) can be seen in a majority o patients, these headaches
can be dif erentiated rom clusters by the pattern o recurrence Objectives: Did you learn to . . .
(sporadic throughout the day versus a predictable time o day • D sc ib th at s a i s h adach s nd m s?
or a certain period o time); signi cant overlap between the two • Initiat t atm nt c st h adach s, pa x sma
headache syndromes does exist and dif erentiation can be di - h mic ania, and t i mina n a ia?
cult. Paroxysmal hemicrania is typically exquisitely sensitive
to indomethacin, and response to indomethacin is highly corre-
lated with this diagnosis. Migraine headaches (“C”) typically do
CASE 18.13
not occur multiple times in one day and are usually described A 55-year-old male with a history o diabetes and prostate
as a throbbing pain. Chronic daily headaches (“D”) as well as carcinoma presents to your of ce complaining o back pain,
analgesic rebound headaches (“E”) are generally continuous in groin numbness, and an inability to initiate voiding. T is has
nature with limited, i any, periods o time without some degree been worsening over the past 1 week.
o headache. Chronic daily headache in the setting o prolonged
analgesic use is highly suggestive o analgesic rebound head- Question 18.13.1 The most likely explanation or these
ache. T e duration o analgesic therapy necessary to trigger and symptoms is:
propagate these headaches remains uncertain. However, it can A) Cauda equina syndrome
be as little as three times per week. B) Urinary outlet obstruction secondary to prostate carcinoma
C) Hydroureter and hydronephrosis secondary to urolithiasis
With urther history, your patient describes right-sided, D) Neurogenic bladder rom long-standing diabetes
electric, stabbing pain involving primarily the cheek and E) All o the above are equally likely
occurring or seconds to minutes repeatedly throughout
the day. Answer 18.13.1 The correct answer is “A.” T is is a presen-
tation o cauda equina syndrome. Cauda equina syndrome is
Question 18.12.4 What would be the rst-line choice or caused by compression o the cauda equina at the level o L4 or
therapy o this entity? L5 by a protruding disk, tumor, etc. Symptoms include progres-
A) Carbamazepine sive ecal or urinary incontinence (secondary to inability to ini-
B) Amitriptyline tiate voiding), impotence, distal motor weakness, and sensory
C) Ibupro en loss in a saddle distribution. T is patient’s symptoms are not
D) Morphine likely to be due to urinary outlet obstruction (“B”). Urinary out-
E) Microvascular decompression (Janetta procedure) let obstruction should not be associated with sensory changes
in a saddle distribution. T e same is true o “C.” “D,” neurogenic
Answer 18.12.4 The correct answer is “A.” T is description bladder rom diabetes, could be a possibility. However, this
is typical o trigeminal neuralgia. Carbamazepine has been should not include back pain or perineal numbness.
shown to be ef ective in treating trigeminal neuralgia and was
used or treatment o this disorder prior to being used or sei- Question 18.13.2 On examination o this patient, you would
zures. ricyclic antidepressants, opioids, and nonsteroidal anti- expect to nd:
in ammatory agents are not rst-line agents or treatment. A) Increased rectal tone
ypically, NSAIDs are o limited, i any, bene t in this setting. B) Decreased rectal tone
Other agents that have been used or treatment with at least C) Normal rectal tone
anecdotal reports o bene t include gabapentin, oxcarbazepine, D) No rectum
clonazepam, baclo en, phenytoin, and topiramate. Microvascu-
lar decompression (the Janetta procedure) can be ef ective in Answer 18.13.2 The correct answer is “B.” Patients with cauda
alleviating pain rom trigeminal neuralgia. However, medical equina syndrome should have decreased rectal tone. I you chose
therapy remains the rst-line treatment. “D” . . . well, there are a lot o things we could say but won’t.
Ch Apt Er 18 • Neu r o l o g y 523

Question 18.13.3 The initial treatment o this patient second such episode in 18 months. On arrival, the patient is
with cauda equina syndrome should include which o the ebrile, lethargic, and looks postictal.
ollowing?
A) Pain management with narcotics Question 18.14.1 Your next step is to:
B) Dexamethasone administered IV A) Reassure the parents that this is a simple ebrile seizure
C) Placement o a Foley catheter B) Obtain blood cultures and start ce riaxone
D) Urgent neurosurgical consultation C) Per orm an LP i the CBC shows leukocytosis and elevated
E) All o the above bands
D) Administer acetaminophen and wait or 2 hours to see i the
Answer 18.13.3 The correct answer is “E.” Pain management patient returns to baseline be ore deciding on urther treat-
is critical in any patient. Dexamethasone may reduce tumor- ment
related edema leading to a reduction in cord compression— E) Arrange or an outpatient EEG
although high-dose steroids are no more ef ective than usual
doses. A Foley catheter is indicated to treat the patient’s urinary Answer 18.14.1 The correct answer is “B.” T is patient must
retention. Since this is a neurosurgical emergency, urgent neu- be assumed to have meningitis until proven otherwise. reat-
rosurgical or orthopedic consultation should be obtained. Local ment should be started immediately. “A” is incorrect. T is is
radiation may also be used acutely depending on your surgeon, not a simple ebrile seizure by history. T e child is postictal and
oncologist, etc. looks ill. While it may end up being a simple ebrile seizure,
you cannot make that conclusion at this point. I the child did
not look lethargic and was up and running around, no urther
HELPFUL TIP: evaluation or treatment would be needed at this time. “C” is
Th dia n sis ca da q ina s nd m is t n d - incorrect. First, the CBC may be relatively normal even with
a d m nths sinc man pati nts initia ha meningitis. Second, you do not want to delay antibiotics until
inc mp t s nd m s, inc din n pain and mi d the CBC and LP are done. “D” is incorrect or the same reason.
n ic s mpt ms. This is n t nat sinc t- T e standard o care in meningitis is antibiotics within 30 min-
c m d p nds n th d n ic d s ncti n utes o hitting the door. Waiting to see the patient’s response to
at th tim dia n sis. acetaminophen or 2 hours will clearly put you out o this time
window.

Objectives: Did you learn to . . . You do the right thing and treat the patient with ce riax-
• r c niz th c inica p s ntati n ca da q ina s n- one. T e patient does look better in an hour or so and has
d m ? returned to baseline. She is alert, attentive, and playing with
• Id nti ca s s ca da q ina s nd m ? toys. T e parents are concerned about whether or not this
• Initiat t atm nt a pati nt with ca da q ina s nd m ? patient has a seizure disorder. T ey would like a urther
evaluation.

QUICK QUIZ: SeIZu r e DISo r Der S Question 18.14.2 Which o the ollowing is indicated at this
point, assuming you decide to evaluate the patient or a
Absence seizures are characterized by all o the ollowing seizure disorder?
EXCEP : A) EEG done on the same day
A) Loss o consciousness B) Admission to the hospital and EEG the next day
B) Feeling o déjà vu C) Serum electrolytes and glucose
C) Rhythmic lip smacking or eye blinking D) rial o antiepileptic drug
D) Staring spells E) None o the above
E) Occurrence up to hundreds o times per day
Answer 18.14.2 The correct answer is “C.” T e workup o
The correct answer is “B.” Feelings o déjà vu and other “psy- a seizure includes serum electrolytes, calcium, magnesium
chic” phenomenon such as hallucinations are associated with phosphate, glucose, etc. Note that this need not be done or
temporal lobe (aka simple partial) seizures. someone with a known seizure disorder who has his or her
typical seizure and returns to baseline. In these cases, only
a drug level o their antiepileptic drug need be done unless
CASE 18.14 there is some change in the seizure type, mental status, etc. “A”
A 2-year-old emale presents to the ED a er having a sei- and “B” are incorrect. T e EEG will be positive because o the
zure. T e parents note that the patient was ne this morn- recent seizure and may not re ect the underlying condition.
ing, spiked a temperature to 39.9°C, and then had a 5-minute T us, waiting a couple o weeks a er the seizure will give a
tonic–clonic seizure that resolved spontaneously. T is is her better picture o what the brain’s innate electrical activity looks
524 FAMIl y MeDICINe eXAMINATIo N & Bo Ar D r evIeW

like. “D” is incorrect because we have not yet proven this child
HELPFUL TIP:
has a seizure disorder. Prescribing antiepileptic drugs would
Ab t 50% pati nts wh ha th i i st b i s i-
be premature.
z nd a 15 m nths wi ha a c nt b i
s iz . This d ps t 30% i th i st s iz is a t
HELPFUL TIP: 15 m nths a . Fami hist is a s in d; 45%
A “stat” eeg can b h p i a n ts i a pati nt th s wh ha a i st-d ati with b i
is ha in ( had) an acti s iz s s a n n pi p- s iz s wi ha a s c nd s iz .
tic sp ( m ca d a “ps d s iz ” n w ca d a
“ps ch nic n n pi ptic s iz ”). A “stat” eeg ma
a s b h p in a pati nt with m nta stat s chan s Objective: Did you learn to . . .
wh b i ma b ha in n nc n si stat s • Dia n s and mana b i s iz s in chi d n?
pi ptic s.

Clinical Pearls
T e parents are wondering what to do about treating this B s t w th inc si n and xc si n c it ia
patient to prevent urther ebrile seizures. administ in tPA ac t st k . I in d bt, ca
n a st st k -acc dit d h spita c mm ndati ns.
Question 18.14.3 Your recommendation to prevent urther D n ta ssi t at at d b d p ss s in an
seizures is: ac t isch mic st k as it d c as s c b a p si n and
A) Acetaminophen at the onset o ever p ts isch mic b ain at isk th inj .
B) Ibupro en at the onset o ever D n tp mn ima in in an indi id a with a hist
C) Phenytoin until the child reaches the age o 5 c nsist nt with t pica mi ain h adach s simp s nc p
D) Buccal midazolam at the onset o any ever with a n ma n ica xaminati n and n id nc
E) None o the above h ad t a ma.
D n t sc n as mpt matic pati nts ca tid a t
Answer 18.14.3 The correct answer is “E.” None o these dis as .
choices are optimal in the management o this patient. “A” and enq i ab t s p apn a si ns and s mpt ms in a
“B” seem like a good idea but do nothing to reduce the occur- ad ts p s ntin with da tim s mn nc and d
rence o ebrile seizures. Neither “C” nor “D” is ef ective. One a p s mn am a ati n i th is s spici n
study suggests that rectal diazepam at the onset o a ever will s p apn a.
reduce the occurrence o ebrile seizures. However, it is asso- P m a mba p nct t t s ba achn id h m -
ciated with some morbidity (e.g., sleepiness) and should be ha n in th c nt xt a n ati h ad CT as a h ad CT
reserved or those with requent ebrile seizures. n d t cts ab t 50% wa nin aks.
r tin na c tics a n t c mm nd d t atm nt
HELPFUL TIP: n pathic pain mi ain h adach s.
Ph n ba bita can p nt b i s iz s b t is ass ci- r tin c mm nd ds ph i n as i st- in th ap
at d with b ha i and a nin p b ms and is n - ins mnia and ati .
a n t c mm nd d. Sta t antibi tics a i ha a hi h s spici n bact ia
m nin itis. This inc d s b th mba p nct i
s spici n is hi h n h.
T e parents have another child at home who has had one us t iptans in pati nts with c na a t dis as is
ebrile seizure. He is now 12 months old. T e parents want to c nt aindicat d d t inc as d isk c na as spasm.
know what his likelihood o having a seizure disorder.

Question 18.14.4 You can let them know that he has approx- BIBLIOGRAPHY
imately: Chronicle E, Mulleners W. Anticonvulsant drugs or migraine
A) 1% to 5% chance o developing a seizure disorder prophylaxis. Cochrane Database Syst Rev. 2004;3:
B) 10% to 15% chance o developing a seizure disorder CD003226.
C) 40% to 50% chance o developing a seizure disorder Con avreux C, et al. Vaccinations and the risk o relapse in
D) 80% to 90% chance o developing a seizure disorder multiple sclerosis. N Engl J Med. 2001;344:319–326.
Fenstermacher N, et al. Pharmacological prevention o
Answer 18.14.4 The correct answer is “A.” Patients who have migraine. BMJ. 2011;342:d583.
a single ebrile seizure have approximately 1% to 5% chance o Goodin DS, et al. Disease modi ying therapies in multiple
developing a seizure disorder. T is is essentially the same risk as sclerosis: Report o the T erapeutics and echnology
the general population. Assessment Subcommittee o the American Academy
Ch Apt Er 18 • Neu r o l o g y 525

o Neurology and the MS Council or Clinical Practice Jausch EC, et al. Guidelines or the early management o
Guidelines. Neurology. 2002;58:169–178. patients with acute ischemic stroke: A guideline or
Gronseth GS, Ashman EJ. Practice parameter: T e Use ulness healthcare pro essionals rom the American Heart
o evoked potentials in identi ying clinically silent lesions Association and American Stroke Association. Stroke.
in patient with suspected multiple sclerosis (an evidence- 2013;44:870.
based review). Report o the quality standards subcom- Johnston SC, et al. National Stroke Association guidelines or
mittee o the American Academy o Neurology. Neurology. the management o transient ischemic attacks. Ann Neurol.
2000;54:1720–1725. 2006;60(3):301–313.
Hassan-Smith G, Douglas MR. Management and progno- Martelletti P, Steiner J. Handbook of Headache: Practical
sis o multiple sclerosis. Br J Hosp Med (Lond). 2011; Management. London, UK: Springer; 2011.
72(11):M174–M176. Pahwa R. Understanding Parkinson’s disease: An update on
Hughes RA, et al. Pharmacological treatment other than current diagnostic and treatment strategies. J Am Med Dir
corticosteroids, intravenous immunoglobulin and Assoc. 2006;7(Suppl 2):4–10.
plasma exchange or Guillain–Barré syndrome. Cochrane Pringsheim , et al. Prophylaxis o migraine headache. CMAJ.
Database Syst Rev. 2011;16(3):CD008630. 2010;182:E269–E276.
Jankovic J, Stacy M. Medical management o levodopa- Saber ehrani AS, et al. Small strokes causing severe vertigo:
associated motor complications in patients with Frequency o alse-negative MRIs and nonlacunar
Parkinson’s disease. CNS Drugs. 2007;21(8):677–692. mechanisms. Neurology. 2014;83(2):169–173.
Ophthalmology
Brian R. Kirsc in
19
GLOSSARY OF TERMS CASE 19.1
Accommodation: Change in the shape o the lens to compensate A mother presents with her healthy, 2-month-old male in ant.
or changes in ocal length. T e term is also used more generally She reports that or the past week his eyes have been notice-
to mean the adjustment o the eye in general to compensate or ably crossed. He appears to xate with either the right or le
ocusing on objects at di erent distances. eye. She eels that aside rom being cross-eyed, he seems to
see well. On examination, his eyes are very crossed. When
Amblyopia: Poor, uncorrectable monocular (and rarely binoc- either eye is covered, he xes and ollows with the ellow eye
ular) vision that is cortical in nature and not attributable to a and appears to have normal motility.
structural abnormality o the eye or visual pathway. T is may
Question 19.1.1 The most likely diagnosis in this patient is:
be a result o strabismus, high uncorrected re ractive error, or
A) Pseudoesotropia
visual deprivation during the critical period o cortical visual
B) Congenital esotropia
development during childhood.
C) Accommodative esotropia
D) Sixth nerve palsy
Esotropia: Deviation o the eyes inward when compared with
normal alignment. Answer 19.1.1 The correct answer is “B.” Esotropia is more
common than exotropia. Congenital esotropia is generally ound
Exotropia: Deviation o the eyes outward when compared with in children younger than 6 months. “C,” accommodative esotro-
normal alignment. pia, is the most common cause o esotropia in childhood and
develops between age 6 months and 7 years. It is rare or accom-
Relative a erent pupillary de ect (RAPD): T e “swinging ash- modative esotropia to develop be ore 4 months or a er 8 years o
light test” in a darkened room is used to test or a RAPD. age. Pseudoesotropia is common in in ants due to their at nasal
T e pupillary response should be equal in both eyes. A bridges and medial epicanthal olds, giving an appearance o eso-
RAPD is detected when there is a relative di erence in the tropia. A simple way to assess children or a true strabismus is to
pupillary response to a direct and consensual light stimulus shine a penlight in their eyes and evaluate the corneal re ection
between the two eyes. An eye with a RAPD will (abnormally) o your light. T e light re ex will be in the same location bilater-
dilate with direct light to the eye but constrict normally in ally or pseudoesotropia but in di erent parts o the eye with true
response to light shined in the ellow eye. A RAPD is seen esotropia or other orms o strabismus (see Fig. 19-1).
when there is optic nerve damage or signi cant retinal dis-
ease (e.g., large retinal detachment, retinal artery occlusion, HELPFUL TIP:
optic neuritis). One will not get a RAPD in re ractive errors, It is i ortant to di erentiate an esotro ia or exotro ia
vitreous bleeds, etc. ro sixt nerve a s . A sixt nerve a s a ects t e
atera rectus usc e and t e atient wi ave i ited
Strabismus: A general term that re ers to a misalignment o the abduction on t e a ected side. T is is usua t e irst
eyes. Esotropia, exotropia, and hypertropia (one eye deviated crania nerve a ected in atients wit increased intra-
upward) are all examples o strabismus. T e term strabismus crania ressure. In adu ts, t e ost co on etio o is
says nothing about etiology, which can be congenital, neuro- vascu o at ic. In c i dren, it is trau a.
logic, or muscular.

526
CHAPTER 19 • Oph Th Al mOl Og y 527

FIGURE 19-2. Acco odative esotro ia i roves w en vision is correct-


ed. Ada tation or near vision consists o acco odation (c an e in ens
s a e), iosis (constriction o t e u i ), and conver ence. T is can even-
tua ead to an acco odative esotro ia, w ic can easi be corrected
wit asses. A ter asses, t e atient no on er uses t e near res onse o
acco odation and conver ence, since t e asses are now rovidin t e
correction so t e e es resu e nor a a i n ent.

FIGURE 19-1. Con enita esotro ia. Note t e ar e deviation wit abnor a signi cant uncorrected re ractive error, or disorders that dis-
cornea i t re exes. T e cornea i t re ex o t e e t e e a ears ore tort images rom the eye to the brain (i.e., congenital cata-
te ora t an t at o t e ri t e e. T ere ore, t e e e is deviated inward. racts). In amblyopia, the brain “turns o ” or suppresses the
vision in the a ected eye, thus allowing the una ected eye to
work unimpeded. In addition to surgical treatment (realign-
Question 19.1.2 Which o the ollowing statements is FALSE? ment, removal o cataract, etc.), amblyopia therapy and pre-
A) Congenital esotropia is nearly always present at birth vention is accomplished by blurring or impeding the vision in
B) Alternating xation (e.g., the ability to x on an object with the una ected (good) eye to strengthen the a ected (bad) eye.
either eye) is characteristic o congenital esotropia T is can be done by physically patching the good eye (not the
C) Patients with congenital esotropia generally have normal strabismic eye as noted in “B”) or by pharmacologic “patch-
amounts o hyperopia ( arsightedness) or their age ing” o the good eye using atropine drops i compliance with a
D) Patients with a high degree o uncorrected hyperopia ( ar- patch is an issue (although eye patches remind kids o pirates
sightedness) can develop an accommodative esotropia and pirates are cool—aargh, matey!). NO E: erms such as
“bad eye” and “good eye” are requently con using, so strabis-
Answer 19.1.2 The correct answer (and the alse statement) mic or deviated eye o en replaces “bad eye” as no eye is truly
is “A.” Congenital esotropia presents by the age o 6 months but bad, but rather a product o its environment, and “good eye”
is rarely present at birth. I true esotropia is observed at birth, it is usually reserved as a popular greeting in Australia (just say
may be due to another neurologic disorder and urther evalu- it like Russell Crowe would!).
ation is indicated. Any orm o strabismus may result in vision
loss, and or this reason, it is important to treat strabismus Question 19.1.4 In which o the ollowing situations would
early in li e. T e rest o the answers are true. Option “B”: since neuroimaging be necessary?
patients have two eyes with equal vision, neither is pre erred, A) A 6-month-old with longstanding esotropia and equal visual
so they will use both eyes to ocus on objects (although not at acuity bilaterally
the same time, o course!). Option “C”: patients with congenital B) A 5-year-old with recent onset esotropia correctable by
esotropia do have a normal degree o hyperopia or their age. glasses
“D” is also a true statement (see Fig. 19-2). C) A 12-year-old with normal re raction and acute onset eso-
tropia with diplopia
Question 19.1.3 Which o the ollowing is NOT an appropri D) A newborn with a unilateral congenital cataract and an eso-
ate treatment or strabismus? tropia
A) Surgical correction i correction o re ractive errors with ull
hyperopic spectacle does not resolve the strabismus Answer 19.1.4 The correct answer is “C.” ANY unexplained
B) Patching the strabismic (misaligned) eye new onset strabismus mandates an evaluation. A 12-year-old
C) Atropine drops or patching the nonstrabismic eye without with acute esotropia and diplopia and no evidence o hypero-
surgery pia requires neuroimaging to rule out any underlying neuro-
D) Atropine drops or patching the non-strabismic eye plus surgery logical disorder. T e age o the patient is older than that seen
with congenital, acquired, or accommodative esotropia. T e
Answer 19.1.3 The correct answer is “B.” T e goal o the re raction is normal, so this does not t with accommodative
treatment o strabismus is to prevent amblyopia. Amblyopia esotropia. Diplopia also suggests acute onset. Further workup
is commonly caused by strabismus (ocular misalignment), is there ore necessary. “A” is incorrect because this is a classic
528 FAmIl y mEDICINE EXAmINATION & BOARD REVIEW

history or congenital esotropia. “B” is incorrect because it is would cause di use injection with a mid-dilated pupil and no
consistent with accommodative esotropia. Fixing uncorrected RAPD. “E,” anterior uveitis, would also cause injection—but
re ractive error would be the appropriate rst step here. Note o en only ciliary ush—and no RAPD. T ere is o en asymme-
that arsightedness (hyperopia) is a much more common cause try o the pupils in chronic anterior uveitis secondary to central
o accommodative esotropia compared to nearsightedness posterior synechiae (adhesions between the iris and lens).
(myopia). “D” is incorrect because we have reason or the eso-
tropia—a congenital cataract is causing poor binocular usion Question 19.2.2 Which o the ollowing is required in order
and coordination. to diagnose acute angle closure glaucoma?
A) Slit lamp
B) onometer
HELPFUL TIP: C) Fluorescein and appropriate UV light
Acquired strabis us always requires a ra id and co - D) Snellen eye chart
ete eva uation. It can be due to trau a, vascu o at ic E) A and B
actors, tu or, intracrania e orr a e, botu is , ead
oisonin , etc. An c i d (or adu t, or t at atter) wit Answer 19.2.2 The correct answer is “B.” T e most impor-
new onset strabis us requires a u o t a o o ic tant examination technique used to diagnose acute glaucoma
exa ination. is intraocular pressure measurement. T is could be achieved by
onopen, Goldmann applanation tonometry at the slit lamp,
Schiotz tonometer (anyone remember this antique?), assessed
Objectives: Did you learn to . . . via digital palpation, etc. “A” is incorrect. A slit lamp is use ul
• Describe t e no enc ature o strabis us? or diagnosing structural problems, iritis, etc. “C,” uorescein,
• Deter ine t e under in causes o esotro ia? is used to diagnose corneal injuries (de-epithelized cornea will
• Reco nize so e unusua cases o strabis us t at require take up uorescein). “D,” a Snellen eye chart, is used to deter-
urt er worku ? mine visual acuity but is not necessary or diagnosing glau-
• Describe t e risk and treat ent o a b o ia? coma. It is always a good idea to check vision in each eye in
every patient with eye complaints; think o checking visual acu-
ity as taking “vital signs” or the eye (see Fig. 19-3).
CASE 19.2
Question 19.2.3 Which o the ollowing is NOT a risk actor
A 50-year-old Asian emale presents to the emergency depart- or acute angle closure glaucoma?
ment (ED) with severe nausea, vomiting, right eye pain, and A) Hyperopia ( arsightedness)
blurry vision. She reports the symptoms began only a ew B) Asian descent
hours earlier. She has no signi cant past medical history. On C) Male gender
gross examination, her visual acuity is OD 20/200, her right eye D) Pharmacologic dilation
is injected, and her right pupil is larger than her le . T ere does E) Increasing age
NO appear to be a relative a erent pupillary de ect (RAPD).
Answer 19.2.3 The correct answer is “C.” Associated risk ac-
Question 19.2.1 With regard to this patient’s presentation, tors or acute angle closure glaucoma are hyperopia, Asian
which o the ollowing would NOT be considered in the di ancestry, female gender, and older age. Patients with hyperopia
erential diagnosis?
A) rauma secondary to blunt injury rom a so ball
B) Central retinal artery occlusion
C) Contact lens associated bacterial keratitis
D) Acute angle closure glaucoma
E) Anterior uveitis

Answer 19.2.1 The correct answer is “B.” Among these


choices, only central retinal artery occlusion would be pain-
less and also associated with a RAPD ( rom di use retinal isch-
emia). All o the others will present with pain and WI HOU
a RAPD. “A,” blunt injuries, may result in decreased vision sec-
ondary to corneal abrasions or edema, intraocular in amma-
tion, hyphema, or retinal injuries. Injury can also cause injec-
tion and a traumatic mydriasis (dilation o the pupil). Direct
and consensual pupillary re exes would be normal (no RAPD)
unless there was an associated traumatic optic neuropathy or FIGURE 19-3. Acute an e c osure auco a. Note t e injection, az cornea
signi cant retinal damage. “D,” acute angle closure glaucoma, re ex, and id-di ated u i .
CHAPTER 19 • Oph Th Al mOl Og y 529

have smaller eyes and more shallow anterior chambers. Females TABLE 19-1 DRUGS USED FOR ACUTE GLAUCOMA
and people o Asian descent also tend to have smaller eyes. As AND HOW THEY WORK
people age, cataracts may develop, become thicker, and crowd
Drug Mechanism o Action
the anterior chamber, increasing the likelihood o developing
pupillary block leading to glaucoma. To ica β-b ockers (e. ., ti o o ) Decreases aqueous u or
roduction

To ica α-adrener ic a onists Decreases aqueous u or


HELPFUL TIP: (e. ., bri onidine) roduction
p ar aco o ic di ation a resu t in an attack o acute prosta andin ana o s (e. ., Increases aqueous u or
an e c osure auco a. But w ere better a ace or atano rost, bi ato rost, out ow t rou uveosc era
it to s ow u t an in our o ice or t e ED? Neovas- travo rost) c anne s
cu arization o t e e e (suc as ro diabetes or reti-
To ica carbonic an drase Decreases aqueous u or
na oxia) can ead to “neovascu ar auco a” ro in ibitors (e. ., dorzo a ide, roduction
neovascu arization o t e iris and an e. h e a can brinzo a ide)
cause auco a ro c ot obstructin t e out ow o
Ora carbonic an drase in ibitor Diuretic and, ore
aqueous u or.
(e. ., acetazo a ide) i ortant , decreases t e
roduction o aqueous u or

mannito (rare use now or acute Os otic diuretic draws


Question 19.2.4 Which o the ollowing is NOT a presenta auco a, ost OR cases on ) aqueous u or ro t e e e
tion o acute closed angle glaucoma?
A) Headache
B) Abdominal pain etc.) increase aqueous out ow, thereby also decreasing intraoc-
C) Vomiting ular pressure. It is generally third line a er iridotomy or acute
D) Limitation o extraocular motion closed-angle glaucoma.
E) Halos around light

Answer 19.2.4 The correct answer is “D.” Patients with acute HELPFUL TIP:
closed-angle glaucoma can present with all o the above ndings In enera , e e dro s t at di ate t e e e ave a red ca
except or the limitation o extraocular motion. Severe eye pain on t e , so i ou are in a anic and ou’ve or otten a
and blurred vision are commonly noted due to corneal edema t e ood advice ro t is aweso e book, a ood ru e
and clouding secondary to the increased intraocular pressure. o t u b is to use a ny eye drop WITHOUT a red ca p.
See Tab e 19-1 or detai s on auco a edications.
You call your local ophthalmologist or optometrist to see this
patient with suspected acute angle glaucoma. She is going to
be delayed because o tra c—it’s Iowa, so she’s likely stuck HELPFUL TIP:
behind a big tractor . . . we don’t have “rush hour,” only “rush Did ou notice t at i ocar ine, t e c assic a ent or
minute.” treatin acute auco a, is absent ro t e ist? Turns
out it doesn’t work a t at we un ess t e intraocu ar
Question 19.2.5 Which o the ollowing drugs is NOT appro ressure is a read ess t an 40 h .
priate to use as a temporizing measure?
A) opical carbonic anhydrase inhibitors
B) opical β -blockers An ophthalmologist makes it to the hospital and recom-
C) opical glycerin mends surgery.
D) opical atropine drops
E) opical prostaglandin analog drops Question 19.2.6 What is the de nitive treatment of choice or
acute angle closure glaucoma?
Answer 19.2.5 The correct answer is “D.” Atropine is contra- A) Laser peripheral iridotomy
indicated in acute glaucoma since it will dilate the eye, exacer- B) apping o the anterior chamber to lower intraocular pres-
bating the problem. opical β -blockers (“B”) and topical (“A”) or sure
oral carbonic anhydrase inhibitors (e.g., acetazolamide) reduce C) Aqueous suppressant therapy
aqueous production and thereby reduce intraocular pressure. In D) Surgical iridectomy
addition to the medications noted above, a topical α -adrenergic E) Chiropractic manipulation o the iris
agonist (e.g., brimonidine, apraclonidine) also lowers eye pres-
sure by selectively a ecting the α -2 receptors and decreasing Answer 19.2.6 The correct answer is “A.” T e treatment goal
aqueous production. opical glycerin (“C”) is used to clear the is to allow the ree ow o aqueous so that it does not accumu-
corneal edema. Prostaglandin analogs (“E”, e.g., latanoprost, late behind the iris to push it orward to obstruct the trabecular
530 FAmIl y mEDICINE EXAmINATION & BOARD REVIEW

meshwork. A laser peripheral iridotomy creates a small hole in the corneal thickness, and per orming any ancillary diagnostic tests
peripheral iris that allows aqueous ow to improve and thereby such as peripheral visual eld analysis.
decreases intraocular pressure. As long as this hole remains pat-
ent, the patient is no longer at risk or an attack o angle closure Question 19.2.9 Which o the ollowing is typical o early
glaucoma, and it is unusual or the hole to close unless there is open angle glaucoma?
a history o intraocular in ammation. “B,” tapping the anterior A) Central vision loss
chamber, can be used be ore laser therapy in order to clear the B) Peripheral vision loss
cornea so that visualization is better or the procedure. However, C) Lack o symptoms
it is adjunctive and not a treatment o choice. Also, topical glyc- D) Decreased contrast
erin works well and can accomplish the same goal without being E) Blurring o vision
invasive. “C,” aqueous suppressant therapy, is discussed in the
previous question. “D,” a surgical iridectomy, is per ormed or Answer 10.2.9 The correct answer is “C.” Most people with
patients who are unable to sit still or the laser procedure (i.e., early open-angle glaucoma are asymptomatic. T is is why screen-
children, intellectually disabled). As to “E” . . . nothing against ing is important. A signi cant number o the approximately one
chiropractors, but this might not be the right time. million axons o the optic nerve may be damaged be ore this
mani ests itsel as visual eld loss. Examination o the optic nerve
Question 19.2.7 Which is FALSE regarding closed and or abnormalities is the best way to diagnose early glaucoma.
open angle glaucoma? Later symptoms involve loss o peripheral or central vision.
A) T e main di erence is that the drainage angle (trabecular
meshwork) is closed in angle closure glaucoma and open in Question 19.2.10 Which o the ollowing is necessary to
open-angle glaucoma diagnose a patient with open angle glaucoma?
B) Scopolamine and other agents with anticholinergic proper- A) Optic nerve head cupping or irregularity with correspond-
ties are contraindicated in open-angle glaucoma but not in ing visual eld loss
closed angle B) T in corneas
C) Acute angle closure glaucoma usually occurs in hyperopic C) Elevated intraocular pressure
individuals while myopia is associated with open-angle D) Narrow but open drainage angle
glaucoma
D) T e majority o people diagnosed with glaucoma have pri- Answer 19.2.10 The correct answer is “A.” Cupping o the optic
mary open-angle glaucoma nerve head is caused by thinning o the neural rim secondary to
damage rom high intraocular pressure. T ere should be a cor-
Answer 19.2.7 The correct answer (and alse statement) is responding visual eld de ect consistent with the appearance o
“B.” Scopolamine and agents with anticholinergic properties, optic nerve cupping to diagnose a patient with glaucoma. In open-
which would cause dilation o the pupil, are actually contrain- angle glaucoma, the mechanism o injury to the optic nerve is
dicated in those with closed-angle glaucoma (and those with thought to be direct mechanical compression. Note that elevated
narrow angles who may not yet have obstructed). Once periph- intraocular pressure is not diagnostic o open-angle glaucoma.
eral iridotomy is per ormed, anticholinergics are no longer an Glaucoma describes the changes to the optic nerve that are
issue since the patient is no longer at risk. All o the rest o the o en secondary to ocular hypertension. Elevated intraocular
statements are correct. pressure by itsel is considered a risk actor or glaucoma. When
not accompanied by cupping or visual eld loss, it is considered
Question 19.2.8 Which o the ollowing does NOT increase ocular hypertension. Patients with ocular hypertension still need
a patient’s risk or primary open-angle glaucoma? to be monitored and treated to prevent the development o open-
A) Family history angle glaucoma. Conversely, some patients with glaucoma never
B) Caucasian race develop high intraocular pressure indicating that glaucoma may,
C) Elevated intraocular pressure (> 21 mm Hg) in act, be due to optic nerve damage arising rom a variety o
D) Age > 40 etiologies, the most common being elevated intraocular pressure.
T inner corneas (“B”) increase the risk o glaucoma but are not
Answer 19.2.8 The correct answer is “B.” Patients o A rican required or diagnosis. “D” is incorrect. T e drainage angle must
heritage are much more likely to develop open-angle glaucoma be widely open—thus the name “open-angle glaucoma.”
than are Caucasians and are also more likely to su er vision loss
rom glaucoma. Family history, high intraocular pressures, and Question 19.2.11 Which o the ollowing statements
thin corneas are all risk actors or the development o open- regarding glaucoma is true?
angle glaucoma. High intraocular pressures lead to optic nerve A) I treated early enough, normalizing the intraocular pressure
damage. Minor risk actors include diabetes and myopia (near- can reverse the process o glaucoma and restore sight
sightedness). All patients should be screened or glaucoma as B) T e unduscopic examination in patients with glaucoma
part o their routine eye examination. T is is usually done by will generally show small retinal hemorrhages in addition to
assessing intraocular pressure, examining the optic nerve and optic nerve cupping
CHAPTER 19 • Oph Th Al mOl Og y 531

compare these with Figure 19-5, which shows optic nerve nd-
ings in patients with glaucoma.

Objectives: Did you learn to . . .


• Reco nize t e si ns and s to s o acute an e c osure
auco a?
• Identi risk actors associated wit acute an e c osure
auco a?
• Describe t e at o o and basis or treat ent or
auco a?
• Di erentiate between an e c osure and o en-an e
auco a?
• Describe o tic nerve f ndin s in auco a?

FIGURE 19-4. Nor a o tic nerve.


QUICK QUIZ: g l AUCOmA Ag ENTS

opical β -blockers lower intraocular pressure by decreasing


C) Papilledema is seen with glaucoma as a result o increased aqueous production.
pressure on the optic nerve
D) T ere are no identi ed genetic markers that correlate to Which o the ollowing is NO considered a systemic side e ect
development o glaucoma o these medications?
E) None o the above A) Bronchospasm
B) Bradycardia
Answer 19.2.11 The correct answer is “E.” So, wrong, wrong, C) Worsening o congestive heart ailure
wrong . . . and wrong. Once there is visual loss, it cannot be D) Increased low-density lipoprotein (LDL)
restored (“A”). Optic nerve hemorrhages, and not retinal hem-
orrhages, are seen in glaucoma (“B”). Retinal hemorrhages are The correct answer is “D.” β -Blockers do not increase LDL.
typically seen with diabetic or hypertensive retinopathies. Pap- T e take-home message here is that topical β-blockers can
illedema results rom increased intracranial—not intraocular— have systemic e ects including congestive heart ailure and
pressure (“C”). T ere are multiple genes that have been identi- bradycardia. Be aware o this, especially in the elderly. Con-
ed that contribute to the development o glaucoma (“D”). See versely, systemic medications such as steroids or β -blockers can
Figure 19-4 or images o normal-appearing optic nerves, and also a ect intraocular pressure.

QUICK QUIZ: VISION l OSS

A 70-year-old white emale presents with sudden loss o the


upper hal o her vision in the le eye. She reports no pain. Her
past medical history is signi cant or hypertension and type 2
diabetes.

Which o the ollowing is the LEAS likely diagnosis given this


history?
A) Retinal detachment
B) Branch retinal artery/vein occlusion
C) Optic neuritis
D) Anterior ischemic optic neuropathy

FIGURE 19-5. g auco atous o tic nerve. Observe t e cu in o t e o tic The correct answer is “C.” Retinal detachment, branch retinal
nerve ead. T e cu (centra de ression, seen as t e bri t art o t e disc) artery/vein occlusion, and anterior ischemic optic neuropathy
o t e o tic nerve, w ic re resents t e axons divin down into t e o tic
nerve, is ar er co ared wit nor a . A cu > 50% o t e disc widt is in-
all cause painless sectorial loss o vision in this age group. Optic
dicative o auco a. Cu in re resents da a e to t e o tic nerve and neuritis usually occurs in a younger age group and is associated
ewer axons are resent. with pain and a central scotoma (blind spot).
532 FAmIl y mEDICINE EXAmINATION & BOARD REVIEW

HELPFUL TIP: CASE 19.3


man atients co ain o ain w en t e ove t eir
e es. Dia noses to consider inc ude o tic neuritis, in- A 65-year-old white male complains o “seeing wavy lines” or
traorbita in ection/in a ation, sinusitis, and orbita “window blinds” when looking at the doorway with his right
ositis o u ti e etio o ies suc as orbita ce u itis, eye. He has no pain or other ocular symptoms. His past medi-
acute g rave’s disease, orbita seudotu or, sarcoidosis, cal history is signi cant or hypertension. He has a 40-pack-
o arteritis nodosu , s ste ic u us er t e atosus, year smoking history. On examination, his visual acuity is
der ato ositis, r eu atoid art ritis, and We ener 20/400 in the right eye. He has no RAPD and slit-lamp exam-
ranu o atosis (now known as ranu o atosis wit ination reveals that his anterior segment is normal. Exami-
o an iitis). nation o his right undus reveals a subretinal hemorrhage
involving his ovea.

Question 19.3.1 Which o the ollowing is the most likely


cause o this patient’s vision loss?
A) Age-related macular degeneration (AMD)
QUICK QUIZ: ANTERIOR ISCh EmIC B) Acute angle closure glaucoma
OpTIC NEUROpATh y C) Cataract
D) Diabetic retinopathy
Ischemic optic neuropathy is the most common cause o optic E) Malingering
nerve lesions with visual loss in patients older than 50 years.
It is re erred to as “anterior” when it a ects parts o the optic Answer 19.3.1 The correct answer is “A.” T is is a typical pre-
nerve behind the optic disc. It generally results rom ischemia sentation or AMD (continue to the next question or more
o the optic disc (sort o makes sense since it is called “anterior in ormation). “B” is incorrect because acute angle closure glau-
ischemic optic neuropathy”). coma presents with pain. “C” is incorrect since cataracts cause
slowly progressive vision loss. “D” is incorrect: this would be an
Which o the ollowing is CORREC regarding nonarteritic unusual presentation or diabetic retinopathy, and the patient
(not related to temporal arteritis) anterior ischemic optic neu- has no known history o diabetes. Save “E” or the psychiatry
ropathy (NA-AION)? chapter.
A) It is not associated with diabetes
B) Women are a ected ve times as o en as men Question 19.3.2 Which o the ollowing statements is FALSE?
C) As with giant cell arteritis, it responds quickly and dramati- A) AMD is the leading cause o severe central vision loss in per-
cally to systemic corticosteroids sons older than 50 years in the United States
D) Avoiding nocturnal hypotension is an important aspect o B) Smoking has been shown to be a risk actor in the develop-
treatment ment o wet AMD
C) AMD is less common in Caucasians when compared with
The correct answer is “D.” It is important to avoid nocturnal other populations
hypotension. Most eye specialists, there ore, recommend D) T e Age-Related Eye Disease Study Part 2 (AREDS2) has
avoiding anti-hypertensive administration at bedtime. Occa- shown a bene cial e ect o vitamin E, vitamin C, lutein, zea-
sionally, midodrine needs to be prescribed to support the blood xanthin, zinc, and copper in delaying the progression to wet
pressure at night. T e remaining statements are incorrect. T ink AMD
o NA-AION as being equivalent to peripheral vascular disease: E) T e main presenting complaint o wet AMD is metamor-
it is associated with hypertension, smoking, diabetes, etc. Males phopsia, which is distortion or waviness centrally in the
and emales are equally a ected. While the age o patients with visual eld
NA-AION overlaps with that o patients with giant cell arteritis,
the patients with NA-AION tend to be younger and NA-AION Answer 19.3.2 The correct answer is “C.” AMD occurs more
is painless. Unlike giant cell arteritis, steroids are not the treat- commonly in Caucasian populations. “A” is correct as AMD is
ment o choice. the leading cause o central vision loss in patients older than
50 years. “B” is correct. Smoking is a signi cant risk actor in the
progression to wet AMD; there ore smoking cessation is highly
recommended in any individual with early AMD ndings or
HELPFUL TIP:
with a strong amily history o AMD. As to “D,” the listed micro-
Sur ica intervention or NA-AION is not e u and
nutrients are bene cial in delaying the progression rom dry to
a ead to worsenin b indness. In act, t ere is no
wet AMD according to the AREDS2 results. “E” is also correct.
known e ective t era . Treat ent o risk actors is
Patients with AMD complain o distortion and/or waviness in
ara ount.
the central visual eld.
CHAPTER 19 • Oph Th Al mOl Og y 533

Answer 19.4.1 The correct answer is “A.” Patient with type 2


HELPFUL TIP:
diabetes should be re erred or a dilated examination immedi-
Dr AmD is t e non-neovascu ar or o AmD. It is c ar-
ately upon diagnosis since they may have been undiagnosed or
acterized b drusen ( e ow-w ite esions in t e outer
a long time period and may already have a degree o retinop-
retina a ers o t e acu a) or atro wit in t e ac-
athy. For type 1 diabetic patients, the recommendation is to
u a. Dr AmD a ead to wet (neovascu ar) AmD, w ic
re er within 5 years o diagnosis.
is associated wit a c oroida neovascu ar e brane
(CNVm). T e CNVm is an abnor a rowt o subretina
Question 19.4.2 What are the common ndings seen on
b ood vesse s, w ic rows in t e acu a or ovea and
direct ophthalmoscopic examination in nonproli erative
a ects vision due to uid eaka e.
diabetic retinopathy?
A) Exudates
HELPFUL (BUT UNFORTUNATE) TIP: B) Cotton wool spots
micronutrients or t e enera o u ation do not see C) Dot-blot hemorrhages
to revent t e deve o ent o AmD. h owever, t e do D) Microaneurysms
see to work to revent ro ression once t e atient E) All o the above
as AmD.
Answer 19.4.2 The correct answer is “E.” Direct ophthalmo-
scopic examination may reveal all these ndings in nonproli -
HELPFUL TIP: erative diabetic retinopathy. T ese are caused by the increased
T ere are severa treat ents avai ab e or neovascu ar ragility o capillaries and arterioles o the retina with diabetes.
AmD. T ese inc ude aser o t e CNVm, intravenous in- While hemorrhages and microaneurysms are unmistakably red,
jection o otosensitizin dru (verte or in) o owed exudates and cotton-wool spots are white and u y in appear-
b nont er a red i t, intravitrea injection o vascu- ance.
ar endot e ia rowt actor in ibitor (VEg F-in ibitor)
edications, and sur ica re ova o CNVm. T e treat- Question 19.4.3 What is the main cause o vision loss in
ent o c oice is VEg F-in ibitors (bevacizu ab, ranibi- nonproliferative diabetic retinopathy?
zu ab, or a iberce t). Wit re ard to otosensitizin A) Dot-blot hemorrhages
dru s, t e NNT to improve vision is 12, t e NNh wit vi- B) Macular edema
sion oss is 100. A treat ents are ai ed toward stabi iz- C) Cataract
in and reservin existin vision, not i rovin vision, D) Neovascularization
a t ou occasiona so e i rove ent is seen. In ad-
dition, atients s ou d be encoura ed to cease s okin . Answer 19.4.3 The correct answer is “B.” Nonproli erative
diabetic retinopathy by de nition has no neovascularization
(new, ragile blood vessel growth secondary to microvascular
Objectives: Did you learn to . . . disease). T e main source o decreased vision is macular edema.
• Reco nize t e si ns and s to s o AmD? reatment o ocal macular edema consists o ocal laser to leak-
• Di erentiate between dr and wet AmD? ing microaneurysms. I the patient has di use macular edema, a
• Reco nize treat ent oda ities or wet AmD?
more extensive (“grid laser”) pattern is used. Increasingly, treat-
ment o nonproli erative diabetic macular edema may include
intravitreal injection o antivascular endothelial growth ac-
tor (anti-VEGF); intravitreal triamcinolone acetonide is used
CASE 19.4 rarely.
A 55-year-old white male with a history o newly diagnosed
type 2 diabetes mellitus presents or routine evaluation. He has Question 19.4.4 What is the main cause o vision loss in pro-
no complaints, including vision. On non-dilated direct oph- liferative diabetic retinopathy?
thalmoscopic examination, both undi appear to be normal. A) Cataract
B) Macular edema
Question 19.4.1 When should this patient be re erred or C) Vitreous hemorrhage
ormal ophthalmologic examination? D) Neovascular glaucoma
A) Immediately upon diagnosis
B) Within 3 months Answer 19.4.4 The correct answer is “C.” Vision loss in pro-
C) Within 6 months li erative retinopathy occurs when riable neovascular ves-
D) Within 1 year sels break open and bleed (vitreous hemorrhage). Vision loss
E) When he develops visual symptoms, maybe 2 days be ore he can also occur i the neovascular vessels grow over the drain-
goes blind age angle o the eye, causing glaucoma, although this is ar less
534 FAmIl y mEDICINE EXAmINATION & BOARD REVIEW

FIGURE 19-7.
FIGURE 19-6. pro i erative diabetic retino at . Neovascu arization o t e
o tic nerve.

common than vitreous hemorrhage. See Figure 19-6 or an the a ected eye is hand motions only. He also has moderate
image o neovascularization. edema o the lids but is able to open his eyes. On slit-lamp
T e treatment o proli erative diabetic retinopathy consists examination, the pupil appears normal. His motility is ull
o panretinal photocoagulation, which involves lasering the and there is no diplopia.
peripheral retina, thereby decreasing the ischemic drive or
neovascularization. reatment o vitreous hemorrhage includes Question 19.5.1 What is the single most important rst step
observation with the head o bed elevated until the hemor- in evaluation o this patient?
rhage settles out and clears enough or laser treatment. Vitre- A) Detection o a RAPD
ous hemorrhages may be caused by other things, such as retinal B) Patching until the heme clears
detachments, so ultrasound is usually per ormed acutely, since C) Evaluation or possible open globe
one will not be able to adequately examine the retina a er a sig- D) Immediate C scanning to evaluate or an orbital racture
ni cant hemorrhage. E) Surgical anterior chamber washout o the heme

Answer 19.5.1 The correct answer is “C.” T is is a hyphema,


HELPFUL TIP: with blood lling about one-third o the anterior chamber in
As irin is sa e in atients wit neovascu arization. T e Figure 19-7. More important is the history o trauma. T e rst,
risk o b eedin does not increase si ni icant . T e car- and most important, step in ocular trauma is to look or evi-
diovascu ar bene it outwei s t e risk to t e e e. dence o an open or ruptured globe. T ere should be no pres-
sure placed on the eye until an open globe is ruled out. Signs
o ruptured globe would be 360 degrees o conjunctival chemosis
(conjunctival swelling), a shallow anterior chamber, an irregular
Objectives: Did you learn to . . . or “peaked” pupil, or low intraocular pressure. However, a normal
• Identi w en to re er atients or or a o t a o o ic globe pressure does not rule out a ruptured globe. T e leak may be
exa ination in t e 1 and t e 2 diabetes? small or may be plugged with choroid. NO E: Intraocular pres-
• Reco nize t e si ns o diabetic retino at on direct o - sure measurements should not be taken until an open globe is
t a osco ic exa ination? ruled out. I there is any doubt about whether there is an open
• Understand t e causes o vision oss in t ose wit non ro i - globe, consult an ophthalmologist or optometrist. “D” is incor-
erative and ro i erative diabetic retino at ? rect and deserves special mention: C is not sensitive enough to
• Describe t e basis or treat ent o bot non ro i erative and rule out an open globe, even with thin cuts through the globe.
ro i erative diabetic retino at ?

CASE 19.5 HELPFUL TIP:


W en eva uatin or a ru tured obe, re e ber t at
A 20-year-old male presents to the ED, complaining o a t e ost co on areas o ru ture are t e i bus (t e
“bloody eye.” He was in a ght earlier in the evening and ar in o t e cornea w ere it eets t e sc era) and
was hit in the eye (“. . . but you should see the other guy!” . sc era be ind t e insertion o rectus usc es. T is is
. . and with your luck, you probably will). You examine him w ere t e sc era is t innest.
and discover the ndings in Figure 19-7. His visual acuity in
CHAPTER 19 • Oph Th Al mOl Og y 535

Objectives: Did you learn to . . .


HELPFUL TIP:
• Reco nize t e co ications o ocu ar trau a?
A “Seide test” can be used to ook or an o en obe.
• Exa ine a atient at risk or o en obe?
A oistened uorescein stri is ent aced at t e
site o injur . S it- a exa ination is done wit Coba t • Identi t e co ications o e a?
b ue i t. I a ru ture is resent, t e uorescein d e wi • Initiate ana e ent o a atient wit e a?
be di uted b t e aqueous, w ic wi a ear as a dark
strea t rou t e reen uorescein.
CASE 19.6
A 27-year-old male presents with irritation and redness o his
right eye a er hammering on an old iron ence. He was not
Question 19.5.2 Which is NOT a complication o hyphema wearing sa ety glasses at the time (they aren’t very cool looking
(bleeding into the anterior chamber)? and he was hoping his attractive neighbor would be out garden-
A) Corneal blood staining ing). He’s now 3 days out rom his injury. On examination, his
B) Glaucoma visual acuity is 20/200. His right eye is injected, and his pupil is
C) Rebleeding irregularly peaked to one side upon gross inspection. Slit-lamp
D) Iris heterochromia examination reveals a laceration o his cornea extending to the
limbus with a wick o iris occluding the laceration site. You place
Answer 19.5.2 The correct answer is “D.” Complications o a Fox shield on the eye to prevent any pressure on the globe,
hyphema include rebleeding, which is most common in the update the patient’s tetanus vaccine, and start antibiotics.
rst 3 to 5 days a er injury. T e hyphema may also stain the
corneal endothelium, which may take months to clear. Glau- Question 19.6.1 What would be the next step in the evalua
coma may also occur because o clogging o the trabecular tion and treatment o this patient?
meshwork by red blood cells, leading to a rise in intraocular A) Imaging or an intraocular oreign body by orbital C scanning
pressure. “D,” iris heterochromia, describes di erence in iris B) Evaluation or an intraocular oreign body by orbital ultra-
coloring (e.g., one blue eye and one brown) and can be caused sound
by a retained oreign body (e.g., iron) but is more o en due to C) ry to remove the oreign body rom the posterior chamber
an underlying condition (e.g., neuro bromatosis) or designer with a magnet
contact lenses that can also be used at Halloween to look really D) Complete ocular examination, including intraocular pres-
reaky. sure and dilated unduscopic examination

You rule out ruptured globe and orbital racture. He’s Answer 19.6.1 The correct answer is “A.” Given his history, the
seen by the ophthalmologist on call who recommends patient needs to be evaluated or a possible intraocular oreign
treatment. body. T e best imaging modality is with orbital C scanning.
Orbital ultrasound (“B”) would also be appropriate, but or surgi-
Question 19.5.3 Which o the ollowing is the most appro cal planning C scanning is pre erred. “C” is incorrect because it
priate treatment or this patient? is likely to cause more damage. T is type o oreign-body removal
A) reat at home with topical steroids is best le to a specialist in the OR. Finally, it would be best to
B) Hospital admission and IV steroids de er intraocular pressure measurements until the extent o the
C) Hospital admission and IV antibiotics laceration is evaluated since no pressure should ever be placed on
D) Hospital admission and topical antibiotics a potential open globe.
E) reat at home with topical antibiotics
T e patient undergoes an orbital C . T ere appears to be a
Answer 19.5.3 The correct answer is “A.” T is patient has a piece o metal within his vitreous cavity.
hyphema, which is blood in the anterior chamber, usually due
to direct trauma. While the party line in the past was that all o Question 19.6.2 Which o the ollowing are inert intraocular
these patients require admission, sending a patient home with oreign bodies?
instructions to avoid urther trauma and vigorous activities is A) Iron
suf cient in reliable patients with limited disease. However, B) Copper
an ophthalmologist or optometrist should be involved in C) Glass
this decision. In addition to topical steroids, other potential D) Aluminum
treatments include cycloplegic agents and perhaps anti bri- E) C and D
nolytic agents such as aminocaproic acid.
Answer 19.6.2 The correct answer is “C.” Iron, copper, and
OK. ime or a joke. What is the de nition o a double-blind aluminum are all reactive species and must be removed rom
study? wo orthopedic surgeons reading an electrocardio- the eye. Glass is inert and may be le in place, depending on the
gram. situation and the risk o surgery.
536 FAmIl y mEDICINE EXAmINATION & BOARD REVIEW

Question 19.6.3 Which is NOT a sign o a retained iron intra B) Manual removal o particulate material ollowed immedi-
ocular oreign body? ately by irrigation with saline until the pH is 7.0
A) Iris heterochromia C) Application o topical glycerin to clear the corneal edema
B) Mydriasis D) opical anesthetic with debridement o sur ace epithelium
C) Glaucoma E) Build rapport by making small talk, asking about his tractor,
D) Retinal degeneration etc.
E) High re ractive error
Answer 19.7.1 The correct answer is “B.” Step one: clean
Answer 19.6.3 The correct answer is “E,” also known as severe
out the chemical. Any particulate matter should be removed
myopia or nearsightedness. Although, the patient in this case
prior to irrigation i it is a reactive substance (e.g., ammonium
may have been extremely nearsighted in his decision to not wear
hydroxide crystals) since uid may dissolve these causing more
sa ety glasses, we mean that metaphorically, and the retained
injury. T e upper lid should be everted to check or any par-
iron oreign body has nothing to do with nearsightedness. All o
ticulate matter. A moistened cotton swab may be used to sweep
the rest are signs o siderosis caused by a retained iron intraocu-
the superior/in erior ornices to remove any residual debris.
lar oreign body. Given the toxicity to the eye, the oreign body
T en, immediate irrigation with normal saline (or water—i
must be removed surgically.
saline is unavailable) is the next step even be ore completing
the examination. A lid speculum should be placed and topical
anesthesia applied. Irrigation may be administered by a hand-
HELPFUL TIP:
held bottle or through IV tubing with an irrigation lens. T e
Cornea orei n bodies a be re oved usin a
pH should be checked with a pH strip and irrigation discontin-
need e a ter t e ad inistration o to ica anest etic.
ued when the pH reaches 7.0.
meta ic orei n bodies in t e cornea a eave a rust
rin around t e area o t e orei n bod , w ic can
be re oved usin an ocu ar burr or an a er brus . Question 19.7.2 Which o the ollowing is a complication o
T e rust rin a a so be e t in ace i it is out o chemical injuries to the eye?
t e visua axis since it does not cause an on - astin A) Corneal ulceration
rob e s. B) In ammation
C) De-epithelialization o the cornea
D) All o the above
Objectives: Did you learn to . . . E) None o the above
• Reco nize t e si ns o a corneosc era aceration?
• mana e a corneosc era aceration in t e e er enc Answer 19.7.2 The correct answer is “D.” All o these are com-
settin ? plications o chemical injuries to the eye. I you chose “E,” you
• A reciate t e e ects o intraocu ar orei n bodies? should take a break now. You are tired.
• mana e an intraocu ar orei n bod in t e e er enc
settin ?

HELPFUL TIP:
CASE 19.7 A ka i burns tend to be ore severe t an acid burns. A -
ka i burns cause ocu ar sur ace da a e b sa oni in
A 55-year-old armer presents to the ED complaining o att acids in ce e branes. A ka ine a ents readi
ocular pain and irritation. He reports accidentally splash- enetrate t e cornea causin de radation. Acids cause
ing ammonia in both eyes. (In Iowa, this is usually rom rotein denaturation, w ic creates a barrier to en-
cooking meth—but not by armers. Ammonia is used to etration. T ere ore, acids cause ess tissue destruction.
produce meth, so meth cookers drill into tanks o anhy- Nerve destruction a cause a decrease in sensitivit
drous ammonia and problems occasionally ensue when to t e e e, so absence o burnin sensation is not ad-
they don’t ollow proper OSHA handling regulations. See equate to deter ine w et er urt er irri ation is need-
all the un things we are learning?) He attempted to rinse ed. Check the pH as noted above.
his eyes with water prior to coming to the ED. His visual
acuity is OD 20/100 and OS 20/80. Both eyes are injected
with corneal edema.
Objectives: Did you learn to . . .
Question 19.7.1 What is the immediate rst step in the • Describe t e di erence between acid and a ka i injuries to
treatment o chemical injuries to the eye? t e e e?
A) Complete ocular examination, including dilated undu- • Treat a c e ica e e ex osure in t e e er enc settin ?
scopic examination • Reco nize t e co ications o c e ica e e injuries?
CHAPTER 19 • Oph Th Al mOl Og y 537

CASE 19.8
A 25-year-old college student complains o redness, sensitiv-
ity to light, and tearing o her right eye or the past day. She
lives in the dorm but reports no exposure to others with a
red eye. For the past ew days, she has had a sore throat and
slight cough. You suspect she may have the dreaded “pink
eye” (conjunctivitis).

Question 19.8.1 Regarding conjunctivitis, which o the ol


lowing is FALSE?
A) Gonococcal conjunctivitis presents with severe hyperacute
purulent discharge
B) Adenoviral conjunctivitis generally begins in one eye and
spreads to the other eye
FIGURE 19-8. Vira conjunctivitis. Note t e o ic es t at are round co ection
C) An enlarged preauricular node is a sign o allergic conjunc- o oc tes in t e in erior ornix.
tivitis
D) oxic conjunctivitis has been associated with the use o topi-
cal antibiotics, antivirals, and preservatives
used, a er other etiologies such as herpetic lesions have been
Answer 19.8.1 The correct answer (and FALSE one) is “C.” ruled out, or 1 week then tapered to prevent signi cant scarring.
T ere is no enlargement o preauricular nodes with aller- However, this is rare. See Figure 19-8 or an image o hemor-
gic conjunctivitis. An enlarged preauricular node is a rhagic viral conjunctivitis generally rom adenovirus, enterovi-
prominent eature o adenoviral conjunctivitis—which is rus and coxsackie virus. It is highly contagious.
extremely contagious (and no matter what the school says,
does not respond to antibiotic eye drops). Allergic conjunc-
tivitis tends to itch and a ect both eyes. All other statements
HELPFUL TIP:
are true.
Antibiotics ave a ica owers in vira conjunctivitis.
h ow e se can ou ex ain t e act t at a sin e dose o
Your patient has no purulent drainage. On examination, her
to ica antibiotics is enou to et a c i d back into
visual acuity is 20/40. Her right eye is di usely injected and
sc oo or da care? most sc oo s and da care o icies
tearing; the le eye appears normal to inspection. She has a
require t e c i d to be on so e sort o dro s (o an
slightly tender right preauricular lymph node about 1 cm in
sort) in order to return. In rea it , even wit bacteria
size.
conjunctivitis, t ere is itt e di erence between antibi-
otics and arti icia tears at 6 da s . . . and ost o t e
Question 19.8.2 What do you recommend as the initial
eo e we treat ave vira conjunctivitis an wa ak-
treatment o this patient’s conjunctivitis?
in t e NNT u e.
A) Symptomatic treatment with arti cial tears our to eight
times per day, cool compresses, and strict hygiene to prevent
spread to the other eye or to other individuals
B) reatment with topical vasoconstrictors/antihistamines Question 19.8.3 I your patient is a part time nursing assis
QID or 1 to 2 weeks tant at a care acility, how long should she stay away rom
C) opical antibiotic treatment or 1 week so she can go back to patient care?
class A) wo days
D) Administration o topical steroids, arti cial tears, and cool B) wo weeks
compresses C) Until her eyes are clear
D) Until she has taken antibiotics or 24 hours
Answer 19.8.2 The correct answer is “A.” T is patient appears
to have viral conjunctivitis, which is treated symptomatically. Answer 19.8.3 The correct answer is “C.” Viral conjunctivitis
T ere is no need to treat with topical vasoconstrictors and/or is highly contagious and is thought to be in ectious as long as
antihistamines unless there is a signi cant itching component. the eye is red and producing discharge. Promote good hand
It’s not bacterial, so no antibiotics are needed. Additionally, hygiene and avoidance o contact with others until the eye
prophylactic antibiotics are not necessary since there is rarely clears. In many instances, complete removal rom school and/or
a secondary bacterial in ection. Occasionally, corneal subepi- work is not easible, but patient education regarding the highly
thelial in ltrates will develop that reduce vision and cause a contagious nature o viral and bacterial conjunctivitis remains
oreign-body sensation. In these cases, topical steroids may be important.
538 FAmIl y mEDICINE EXAmINATION & BOARD REVIEW

Question 19.8.5 Which o the ollowing is NOT indicated in


the treatment o allergic conjunctivitis?
A) Cool compresses
B) Arti cial tears
C) Chronic topical vasoconstrictors/antihistamines
D) Short-term topical steroids
E) Diphenhydramine

Answer 19.8.5 The correct answer is “C.” Given the pos-


sibility o rebound hyperemia with prolonged use, chronic
vasoconstrictors/antihistamines are never indicated. opical
steroids should be reserved or severe cases and tapered over 1
to 2 weeks. opical NSAIDs (ketorolac) and antihistamines or
mast cell stabilizers (levocabastine or olopatadine) may be used
to alleviate intense itching but are very expensive. Ketoti en
FIGURE 19-9. Bacteria conjunctivitis. Note t e uco uru ent disc ar e. is an e ective and inexpensive O C mast cell stabilizer and
H1-antihistamine that, when used routinely, greatly decreases
allergic conjunctivitis symptoms.
Question 19.8.4 Which o the ollowing is appropriate in
the management o acute gonococcal conjunctivitis? Objectives: Did you learn to . . .
A) Conjunctival Gram stain and culture • Di erentiate a on vira , bacteria , and a er ic
B) Ce riaxone IM/IV conjunctivitis?
C) Saline irrigation • Not rescribe o t a ic antibiotics or vira conjunctivitis—
D) All o the above ou ot t at, ri t?
E) None o the above • C oose a ro riate treat ents oda ities or t e various
causes o conjunctivitis?
Answer 19.8.4 The correct answer is “D.” he hallmark o
acute gonococcal conjunctivitis is severe purulent discharge,
which occurs within 12 to 24 hours o in ection. Preauricu- CASE 19.9
lar adenopathy may also be seen. Management consists o A patient complains o possible oreign body in her eye. She
Gram stain and culture o the purulent material to document was scraping barnacles o the bottom o her multi-million
gonococcus, administration o ce triaxone (or spectinomy- dollar yacht currently dry-docked right next to one o Don-
cin i cephalosporin allergic) IM/IV plus azithromycin ( or ald rump’s (Could she be a radiologist? Maybe a plastic sur-
possible concurrent chlamydia in ection), and eye irrigation. geon?). She elt something f y into her eye and had immediate
Fluoroquinolones are no longer recommended a s initial pain.
therapy. See Figure 19-9 or an image o bacterial conjuncti-
vitis and Figure 19-10 or an image O Gonococcal ophthal- Question 19.9.1 Which o the ollowing is FALSE regarding
mia neonatorum. the use o topical f uorescein, which is used to highlight
epithelial de ects o the cornea?
A) Fluorescein is a nontoxic, water-soluble dye
B) It uoresces under a cobalt blue lter
C) It exhibits positive staining o epithelial de ects
D) Fluorescein does not penetrate the cornea

Answer 19.9.1 The correct answer is “D.” Fluorescein does di -


use through the corneal stroma and causes a green are in the
anterior chamber. Rapid staining o the anterior chamber a er
the removal o a corneal oreign body suggests a large, deep cor-
neal injury.

HELPFUL TIP:
Always evert t e u er e e id to ook or additiona
orei n bodies. T ese can be re oved usin a cotton-
FIGURE 19-10. O t a ia neonatoru . Note t e severe uco uru ent ti ed swab. Irri ation a a so be use u .
disc ar e.
CHAPTER 19 • Oph Th Al mOl Og y 539

C) Foreign bodies o Schlemm


During your complete examination, you notice a small f eck D) A tear o the anterior lens capsule
o what looks like paint on the cornea. A er success ully
removing the piece o paint, you plan to discharge the patient. Answer 19.9.3 The correct answer is “A.” his patient is pre-
senting with typical posttraumatic iritis. Findings would be
Question 19.9.2 Which o the ollowing is NOT an appropri “cell and lare” in the anterior chamber. “B” and “D” are obvi-
ate treatment or corneal abrasions? ously not going to be a result o a super icial corneal abra-
A) Observation alone sion. “C” is something we made up, but “Canal o Schlemm”
B) opical antibiotics is part o the eye and one o our avorite names in medicine—
C) Cycloplegics it is just humorous (humorous, get it? . . .ah, poor Schlemm).
D) opical steroids
E) Patching

Answer 19.9.2 The correct answer is “D.” In uncomplicated HELPFUL TIP:


corneal abrasions in non-contact lens wearers, observation alone Iritis, a so ca ed anterior uveitis, c aracteristica
is o en adequate. Antibiotics should be used or all contact lens– resents wit ci iar us (a red rin around t e iris)
related abrasions (or in non-wearers i you are particularly wor- wit out tearin or disc ar e. On s it- a exa ina-
ried about the wound). opical broad-spectrum antibiotics, such tion, t e “ce and are” are due to individua in a -
as polymyxin B/trimethoprim, are good rst-line choices in those ator ce s and o -a earin rotein eaked ro
without contact lenses. A topical uoroquinolone can be used b ood vesse s.
in contact lens wearers. T e patients should also be advised to
withhold contact lens use until the abrasion is healed and should
be evaluated by their contact lens prescribing provider. In many
cases a bandage contact lens may be utilized by the patient’s eye Question 19.9.4 Appropriate treatment or this patient
care provider to improve com ort and aid in healing. Patching with post traumatic iritis includes which o the ollowing?
may also be used or patient com ort but is unnecessary and may A) opical antibiotics
actually increase pain and prolong healing time. opical steroids B) Cycloplegic agents
may inhibit epithelial healing and promote in ection and should C) opical steroids
be avoided. “C” deserves special mention. Use o a long-acting D) A, B, and C
cycloplegic is acceptable or discom ort related to severe pho- E) B and C only
tophobia and blepharospasm secondary to corneal abrasions.
However, there is no data supporting (or re uting) their use in Answer 19.9.4 The correct answer is “E.” Post-traumatic iritis
the treatment o corneal abrasions themselves. is generally treated with topical steroids and cycloplegic agents.
Again, we usually do this in consultation with an eye care pro-
vider. O special note is “C.” While we all worry about promoting
HELPFUL TIP:
in ection with topical steroids, they can also increase intraocu-
mu ti e vertica cornea abrasions su est a orei n
lar pressure, and when used chronically, can cause glaucoma,
bod under t e u er id ( orei n-bod “trackin ”).
which is generally more o a concern. So, these patients require
B inkin and c osin t e e e wi eave an abrasion on
close ollow-up.
t e cornea. Evert t e e e id and swee t e ornices wit
a oistened cotton swab to re ove an residua or-
Assume that this patient was presenting de novo with iritis
ei n bod .
and it is not post-traumatic.

Question 19.9.5 Which o the ollowing do you NOT have to


As you already know, most corneal abrasion should heal worry about as an etiology o iritis?
within 24 to 48 hours. You ask the patient to ollow up the next A) Rheumatoid arthritis
day to make sure that things are going well. On arrival (the B) Diabetes mellitus
act that she ollowed up at all rules out the possibility that C) Syphilis
she is a physician . . . maybe an investment banker?), she notes D) Sarcoid
increased pain in the eye with increased injection and photo- E) Lyme disease
sensitivity. You place anesthetic drops in her eye but the pain
does not resolve. T e corneal abrasion seems to be healed. Answer 19.9.5 The correct answer is “B.” Iritis is an in amma-
tory process that can be caused by multiple underlying illnesses
Question 19.9.3 What do you expect to see on slit lamp including ankylosing spondylitis, lupus (rarely), Behcet disease,
examination in her anterior chamber? syphilis, sarcoid, B, Reiter syndrome, toxoplasmosis (com-
A) Cells and are mon, even in non-immunosuppressed), juvenile idiopathic
B) Opacity o the lens arthritis, and many others.
540 FAmIl y mEDICINE EXAmINATION & BOARD REVIEW

Objectives: Did you learn to . . . Answer 19.10.2 The correct answer is “D.” Meibomian gland
• Re ove a cornea orei n bod ? dys unction is associated with rosacea. ypical ndings o rosa-
• Use to ica uorescein in cornea abrasions? cea include acial papules, pustules, telangiectasia, erythema,
• mana e unco icated cornea abrasions? and rhinophyma.
• Reco nize iritis and its causes?
Question 19.10.3 What treatment do you prescribe or this
patient?
CASE 19.10 A) Observation and reassurance
B) Daily warm compresses, lid scrubs with dilute baby sham-
A 65-year-old white male with a history o hypertension, poo, and oral doxycycline, minocycline, or metronidazole
adult-onset diabetes, rheumatoid arthritis, and rosacea C) Erythromycin ophthalmic ointment PRN
presents with chronic complaints o redness, tearing, and D) Daily warm compresses, topical steroids, and requent use o
irritation OU. He sometimes also has “a lm over his vision” arti cial tears
that comes and goes. His ocular examination appears nor-
mal. He does, however, have an oily tear lm with rapid Answer 19.10.3 The correct answer is “B.” T e treatment
breakup o his tears over his ocular sur ace. T ere is evi- o meibomian gland dys unction consists o lid hygiene and
dence o capped meibomian glands (sebaceous glands along doxycycline or minocycline 50 to 100 mg daily to BID or 3
his lid margins), but the eyelids are otherwise normal in to 6 weeks (which will also help treat the rosacea). T e dosage
appearance. may then be tapered according to symptoms. Gastrointestinal
upset and photosensitivity are common side e ects o doxy-
Question 19.10.1 Which o the ollowing is the most likely cycline.
diagnosis? Seborrheic blepharitis is o en treated with aggressive lid
A) Meibomian gland dys unction hygiene. opical steroids are only used or short duration i
B) Staphylococcal blepharitis there is signi cant in ammation present and should generally
C) Hordeolum be prescribed by an ophthalmologist or optometrist or this
D) Chalazion condition. opical therapies such as cyclosporine emulsion
E) Seborrheic blepharitis are aimed at lubrication and addressing chronic in ammation
and are now available by prescription but are expensive and
Answer 19.10.1 The correct answer is “A.” Meibomian glands not that e ective. Over-the-counter lubricating arti cial tears
are the sebaceous glands o the upper and lower eyelids, which can improve com ort but generally don’t resolve the underly-
are located along the posterior lid margin behind the lashes. ing condition.
T ese are punctate openings along the lid margin, which
become clogged with thick secretions. When coupled with Question 19.10.4 Which o the ollowing is a common com
chronic in ammation o the lid margin, this eventually leads plication o meibomian gland dys unction?
to an unstable tear lm, creating symptoms o burning, red- A) Bacterial keratitis
ness, oreign-body sensation, and lmy vision. It is considered B) Preseptal cellulitis
a orm o blepharitis (in ammation o the eyelids). “B” and “E” C) Chalazion
are incorrect. Staphylococcal and seborrheic blepharitis have D) Scleritis
similar symptoms but involve the anterior eyelid margin at the E) Chronic conjunctivitis
base o the lashes (not the meibomian glands as noted in this
case). Both typically have predominant signs o crusting or Answer 19.10.4 The correct answer is “C.” Meibomian gland
mattering. Seborrheic blepharitis o en has crusting o an oily dys unction can cause a chronic granuloma to orm behind
or greasy consistency. “C” and “D” are in ammatory processes, the plugged meibomian gland, which is called a chalazion. T e
commonly but incorrectly grouped together as “styes.” A hor- in ammation is sterile—unlike a hordeolum (“stye”), which is
deolum a ects the anterior lid margin glands, which become a pain ul purulent abscess. reatment o a chalazion involves
acutely plugged and in amed. A chalazion a ects the poste- requent warm compresses and massage. opical antibiotics are
rior lid margin glands, which become plugged and chronically o little value since it is sterile. I these measures ail, an intra-
in amed. “Stye” is typically synonymous with hordeolum; how- lesional injection o steroids or incision and drainage is war-
ever, both hordeolum and chalazion grossly appear similar on ranted. Hordeola (styes) o en resolve spontaneously but warm
examination. compresses and massage are o en help ul. I there is any evi-
dence o cellulitis, systemic oral antibiotics are indicated. Drain-
Question 19.10.2 What part o his past medical history is age using a needle may also be help ul. opical antibiotics are
associated with meibomian gland dys unction? o en not e ective.
A) Hypertension
B) Diabetes Objectives: Did you learn to . . .
C) Rheumatoid arthritis • Reco nize t e si ns and s to s o b e aritis and ei-
D) Rosacea bo ian and d s unction?
CHAPTER 19 • Oph Th Al mOl Og y 541

• Describe t e etio o ies o b e aritis and eibo ian and


d s unction? HELPFUL TIP:
• Di erentiate a on eibo ian and d s unction and Both orbita ce u itis and rese ta ce u itis are
sta ococca and seborr eic b e aritis? co on secondar to sinusitis. Bot rese ta and
• Deter ine a ro riate treat ent or b e aritis and eibo- orbita ce u itis a a so occur ro direct inocu ation or
ian and d s unction and t e co ications o ordeo a bactere ia ro a distant source.
and c a azion?

Question 19.11.3 The most common pathogen causing


CASE 19.11 preseptal cellulitis rom a skin trauma with inoculation is
A 7-year-old emale presents with pain ul swelling and red- which o the ollowing?
ness o upper and lower lids o her right eye. She reports hav- A) Staphylococcus epidermidis
ing a bug bite near her right eye a week ago, and it’s been B) Haemophilus inf uenzae
very itchy. Now, it has become more erythematous and pain- C) Staphylococcus aureus
ul (in ected, one might even say). On examination, her right D) Streptococcus pneumoniae
eyelids are extremely edematous with a well-demarcated
area o erythema. Her ocular examination is normal, includ- Answer 19.11.3 The correct answer is “C.”S. aureus is the most
ing normal vision. She has no RAPD, proptosis, or motility common pathogen o preseptal cellulitis rom skin trauma. Some
de cit. may have chosen “B.” Prior to the introduction o the HIB vac-
cine, children younger than 5 years o en had preseptal cellulitis
Question 19.11.1 This presentation is most consistent with: secondary to H. inf uenzae. However, this is no longer the most
A) Orbital cellulitis common pathogen. Most cases o preseptal and orbital cellulitis
B) Preseptal cellulitis in children are caused by Gram-positive cocci. Orbital cellulitis
C) Anaphylactoid reaction to the insect bite in children is usually caused by a single organism. In contrast,
D) Blepharitis adults with orbital cellulitis requently have polymicrobial in ec-
E) None o the above tions including Gram-positive cocci, H. inf uenzae, and anaer-
obes. Mucormycosis should be suspected in diabetic patients
and immunocompromised individuals.
Answer 19.11.1 The correct answer is “B.” he presentation
o this patient is consistent with preseptal cellulitis, which
is de ined as in lammation/in ection anterior to the orbital
septum. his is less dangerous than orbital cellulitis, which HELPFUL TIP:
occurs posterior to the orbital septum and has the risk o Orbital cellulitis is an o t a o o ic e er enc . In
spread o in ection into adjacent structures. “A” is incor- addition to s read to ot er structures (e. ., cavernous
rect. In ection occurring posterior to the orbital septum is sinus t ro bosis), orbita ce u itis a cause a tense
called orbital cellulitis. his is characterized by ever, pro- ro tosis eadin to b indness. T ink about t is as a
ptosis, restriction o globe motility, chemosis, and pain on “co art ent s ndro e o t e e e.” A co art ent
eye movements. Orbital cellulitis can be complicated by sub- s ndro e (and b indness) can a so be seen wit trau-
periosteal abscess, cavernous sinus thrombosis, meningitis, a and secondar intraorbita ( ostse ta ) b eedin or
or intracranial abscesses. “C” is incorrect since an anaphy- retrobu bar e ato a eadin to ro tosis. T e treat-
lactoid reaction is systemic. ent is e er ent atera cant oto .

Question 19.11.2 Which o the ollowing is TRUE?


A) T e most common cause o preseptal cellulitis in children is
Objectives: Did you learn to . . .
sinusitis
• Di erentiate between orbita and rese ta ce u itis?
B) T e most common cause o preseptal cellulitis in teens and
• Reco nize conditions t at redis ose atients to t ese
adults is sinusitis
rocesses?
C) Orbital cellulitis is most commonly caused by bacteremia
rom a secondary source, as opposed to direct spread rom
an adjacent structure CASE 19.12
D) T e most common secondary source or orbital cellulitis is A 67-year-old white man with a history o coronary artery
otitis media disease, hypertension, and peripheral vascular disease pres-
ents with the sudden onset o painless loss o vision OD sev-
Answer 19.11.2 The correct answer is “A.” T e most common eral hours ago. He states he was watching V when “things
cause o preseptal cellulitis in children is sinusitis. In contrast, went black” in his right eye. He notes that this happened a ew
teens and adults usually have preseptal cellulitis rom a super - times be ore, but his vision always returned to normal a er a
cial source, such as skin trauma with inoculation. couple o minutes. On examination, he has light perception
542 FAmIl y mEDICINE EXAmINATION & BOARD REVIEW

Question 19.12.3 Treatment or central retinal artery occlu


sion includes all o the ollowing EXCEPT:
A) T rombolytics
B) Digital compression/decompression o the globe
C) Oxygen
D) Increasing blood carbon dioxide levels (e.g., rebreathing)
E) Acetazolamide

Answer 19.12.3 The correct answer is “A.”T erapy or central ret-


inal artery occlusion is entirely aimed at dislodging the embolism,
maintaining retinal viability, and reducing intraocular pressure (to
increase the pressure gradient between the artery and the eye). T is
can be accomplished by digitally compressing (press hard!) then
decompressing the eye with a nger (to dislodge the embolism),
oxygen (which will increase oxygen delivery), increasing serum
FIGURE 19-11. A centra retina arter occ usion. Note t e c err -red s ot
carbon dioxide levels by rebreathing (thus dilating intracranial
in t e acu a, w ic is t e nor a c oroida circu ation a idst t e isc e ic
retina. arteries), and acetazolamide (to reduce intraocular pressure). o
accomplish the goals o increasing oxygen and carbon dioxide, try
using a nonrebreather acemask with supplemental oxygen.
vision, the presence o a RAPD, and a normal anterior seg-
ment o the eye. Upon unduscopic examination, the undus Question 19.12.4 Comparing central retinal artery occlu
appears di usely white with a reddish hue within the macula sion to central retinal vein occlusion, which o the ollowing
(see Fig. 19-11). is TRUE?
A) Central retinal artery occlusion is more likely to be associ-
Question 19.12.1 The patient’s history and examination are ated with giant cell arteritis
most consistent with which o the ollowing? B) T e main eature o both is retinal whitening with a cherry
A) Central retinal vein occlusion red spot
B) Anterior ischemic optic neuropathy C) Central retinal vein occlusion usually results rom atheroscle-
C) Central retinal artery occlusion rotic thrombosis, while central retinal artery occlusion results
D) Choroidal ischemia rom hyperviscosity syndromes and hypercoagulable states
D) A RAPD is characteristic o central retinal artery occlusion
Answer 19.12.1 The correct answer is “C.” A central reti- but is not seen with central retinal vein occlusion
nal artery occlusion is characterized by acute painless loss o
vision. T e ischemia and edema o the retina cause di use whit- Answer 19.12.4 The correct answer is “A.” Giant cell arteritis
ening. T ere is a cherry-red spot in the macula, which is the is seen in 1% to 2% o central retinal artery occlusion. However,
normal choroidal circulation amidst the ischemic retina (see a more limited anterior ischemic retinopathy is generally seen
Figure 19-11). T e extensive ischemia causes a RAPD and sig- with giant cell arteritis. I a patient presents with symptoms o
ni cant visual loss. His previous history o recurrent episodes o central retinal artery occlusion, but no embolus is visualized, he/
loss o vision that returned to normal is consistent with amau- she should be asked about symptoms o giant cell arteritis. “B” is
rosis ugax. incorrect because only central retinal artery occlusion is associ-
ated with retinal whitening and a cherry red spot. T e appear-
Question 19.12.2 In a patient with this history, irreversible ance o a central retinal vein occlusion is one o tortuous dilated
retinal damage occurs a ter what time rame? veins, optic nerve edema, and intraretinal hemorrhages/edema
A) 10 minutes (the so-called “blood and thunder” appearance o the undus—
B) 30 minutes although since thunder is a sound, having something look like
C) 90 minutes thunder makes no sense to us, but maybe we are just too literal.)
D) 6 hours “C” is incorrect because central retinal artery occlusion is usu-
ally caused by atherosclerotic thrombosis or emboli, while cen-
Answer 19.12.2 The correct answer is “C.” I central retinal tral retinal vein occlusion is associated more with hyperviscosity
artery occlusion is complete or more than 90 minutes, irre- syndromes (e.g., polycythemia) and hypercoagulable states (e.g.,
versible retinal damage and visual loss ensue. Patients should protein C de ciency). In older patients with central retinal vein
seek medical care immediately. Amaurosis ugax, or transient occlusion, the main risk actors include vasculopathic states such
monocular blindness, is typically caused by carotid disease. It is as hypertension and diabetes. “D” is incorrect because a RAPD
o en described as a curtain or shade coming over the vision. It can be seen with either syndrome. T ere is always an RAPD with
lasts rom a ew seconds to 15 minutes. Patients with a history a central retinal artery occlusion due to the di use distribution
o amaurosis ugax should be evaluated or carotid and cardiac o ischemia. A RAPD may or may not be seen with a central reti-
disease. nal vein occlusion depending on the level o ischemia.
CHAPTER 19 • Oph Th Al mOl Og y 543

CASE 19.13
A 40-year-old white emale presents to your ED complain-
ing o seeing “little f oating black spots” in her vision in
the le eye. She also notes little sparks o light in the tem-
poral periphery o the le eye. She noted this while shop-
ping today at Wal-Mart. First, you think this may be the
revenge o the gods or shopping at Wal-Mart instead o
buying locally. But then, you gather your wits about you.
On examination, there is no RAPD (she has normal direct
and consensual pupillary ref exes). Con rontation visual
elds demonstrate peripheral vision loss in the le eye.
Dilated peripheral retinal examination reveals billowing
gray olds. T e macula appears normal, and her vision is
20/20 in that eye.
FIGURE 19-12. Centra retina vein occ usion. Note t e di ated tortuous
veins, o tic disc ede a, and retina e orr a e/ede a.
Question 19.13.1 Which o the ollowing is the most appro
priate step in the management o this patient?
A) Place a patch over the le eye
HELPFUL TIP: B) Re er to an ophthalmologist immediately
Ot er causes o centra retina arter occ usion inc ude C) Lower blood pressure acutely with IV labetalol
vascu itides and b ood d scrasias. Ot er causes o cen- D) Apply timolol solution to the a ected eye
tra retina vein occ usions inc ude increased intraorbita E) O er reassurance
or intraocu ar ressure, auco a, b ood d scrasias, u-
us anticoa u ant, anti os o i id antibod , and ro- Answer 19.13.1 The correct answer is “B.” T is patient is
tein C de icienc . See Fi ure 19-12 or an i a e o t e presenting with urgent ophthalmologic disease. She has clas-
retina in centra retina vein occ usion. sic symptoms o retinal detachment— ashing lights, visual
eld disruption, and oaters. Also, the majority o her vision
is still intact. In her current state, she has a high likelihood o
Question 19.12.5 Further evaluation and treatment o a retaining good vision. None o the other treatments o ered
central retinal artery occlusion includes all o the ollowing do anything or retinal detachment. Speci cally, “A” does
EXCEPT: nothing, “C” is treatment or hypertensive retinopathy, “D” is
A) opical timolol or glaucoma, and “E” is just plain nuts in this case.
B) ESR/CRP
C) Carotid doppler Question 19.13.2 Risk actors or retinal detachment
D) Orbital MRI/MRA include all o the ollowing EXCEPT:
E) Echocardiogram A) Glaucoma
B) Aphakia (surgical removal o the lens, such as in cataract
Answer 19.12.5 The correct answer is “D.” T ere is no need surgery)
or orbital imaging in the management o a central retinal artery C) Myopia
occlusion. All o the other choices are important management D) rauma
steps. In addition, blood pressure, asting blood sugar or gly- E) Prior ocular surgery
cosylated hemoglobin, CBC, and P /P should be done. I
there is a suspicion o giant cell arteritis, an ESR/CRP should be Answer 19.13.2 The correct answer is “A.” Glaucoma is
checked. Other tests might include a rheumatoid actor, syphilis not a risk actor or retinal detachment. Both myopia (near-
serology, serum protein electrophoresis, and antiphospholipid sightedness) and aphakia (surgical removal o lens) are risk
antibodies. A similar workup is warranted in central retinal actors. rauma and previous surgery also predispose to
vein occlusion, except there is no need to search or an embolic retinal detachments. Most persons with retinal detachment
source with a carotid doppler and echocardiogram. are older than 50 years. As patients age, the vitreous liqui ies
and contracts, and this vitreoretinal traction or tugging can
Objectives: Did you learn to . . . cause a tear, detaching the retina rom the posterior wall o
• Reco nize t e s to s o vascu ar disorders o t e e e? the eye.
• Di erentiate between centra retina arter and vein occ u-
sions? Objectives: Did you learn to . . .
• Describe so e causes o ocu ar vascu ar occ usions? • Sus ect retina detac ent in atients resentin wit
• Deter ine t e a ro riate s ste ic worku or arter and “ oaters” and “ as es?”
vein occ usions? • Identi atients w o are at risk or retina detac ent?
544 FAmIl y mEDICINE EXAmINATION & BOARD REVIEW

incorrect. An older person with a history o congenital cataracts


CASE 19.14 and poor best-corrected visual acuity most likely su ered rom
A 55-year-old white male complains o a gradual decrease in amblyopia; there ore, cataract surgery is unlikely to bene t such
vision in both eyes. He notes glare with oncoming headlights a patient. “D” is incorrect. Monocular diplopia (double vision
while night driving. Despite this, he eels that he is able to rom one eye only) is an indication or cataract surgery, but i
read better without his bi ocals. new spectacle correction improves the diplopia, then cataract
surgery is not necessary.
Question 19.14.1 Based on the history given, which o the
ollowing is the most likely cause o this patient’s com Question 19.14.4 Complications o cataract surgery include
plaints? all o the ollowing EXCEPT:
A) Retinal detachment A) Endophthalmitis
B) Cataracts B) Retinal detachment
C) Glaucoma C) Glaucoma
D) Diabetic retinopathy D) Hemorrhage
E) Presbyopia E) Meibomian gland dys unction

Answer 19.14.1 The correct answer is “B.” Progressive visual Answer 19.14.4 The correct answer is “E.” Meibomian gland
loss and glare rom oncoming headlights while driving at night dys unction is a chronic condition as discussed previously in
are common complaints caused by cataracts. T e eye examina- this chapter. Modern day cataract surgery is typically done by
tion can con rm the diagnosis, as most signi cant cataracts are phacoemulsi cation with an intraocular lens implant. Phaco-
easily visualized. T e red re ex is diminished bilaterally, and emulsi cation is an ultrasound method o ragmenting and
a haze o gray is observed over the lens. Symptoms o retinal aspirating the cataract. Although associated with ewer compli-
detachment (“A”) are more acute. Glaucoma (“C”) and diabetic cations than older large incision cataract surgery techniques, the
retinopathy (“D”) are less likely, but could also be present. Intra- potential complications mentioned above still exist. Additional
ocular pressures and a dilated eye examination should be com- complications include: wound leaks, uveitis, macular edema,
pleted to adequately assess and diagnose the patient’s condition. retained lens material, and vitreous loss.

Question 19.14.2 All o the ollowing conditions/medica


tions are risk actors or cataract ormation EXCEPT: HELPFUL TIP (AND ONE THAT MAY BE ON THE
A) Corticosteroids BOARDS):
B) rauma One o our avorite disease na es (besides “ex odin
C) Radiation ead s ndro e” . . . rea , ook it u ) is “ o iris s n-
D) Calcium channel blockers dro e.” T is occurs w en atients on a a-b ockers
E) Diabetes mellitus (usua en or Bph ) o or cataract sur er . T e resu t
is an iris t at irreversib ro a ses durin cataract sur-
Answer 19.14.2 The correct answer is “D.” All o the other er . h ow on t is e ect o a a-b ockers ast a ter
choices are associated with cataract ormation. Other risk ac- ast usa e is not known so be care u w en su estin
tors include age, tobacco, alcohol, and sunlight (so, wear your sur er to t ese atients.
UV blocking shades!).

Question 19.14.3 O all o the patients with cataracts below, Objectives: Did you learn to . . .
which one has an indication or cataract surgery? • Identi atients at risk or cataract deve o ent?
A) A patient with no visual complaints with a visual acuity o • Describe indications or cataract sur er ?
20/50 • Reco nize t e s to s and visua disabi it in t ose wit
B) A patient with complaints o glare and inability to drive at ro ressive cataracts?
night with a visual acuity o 20/40
C) An older patient with a history o monocular congenital cat-
aract and best-corrected visual acuity o 20/100 in that eye QUICK QUIZ: SOmETh INg ON Th E EyE
D) A patient with right monocular diplopia that resolves with
new spectacle correction A patient presents to your clinic without vision symptoms
but complains o a growth on the colored part o his eye
Answer 19.14.3 The correct answer is “B.” T ere is no strict (Fig. 19-13). T e growth is painless and has been present or
visual acuity that determines the appropriate timing o cataract several years but is now getting to the point that it is pretty
surgery. It is really a unctional de nition. I there are signi - obvious. Strangers tend to stare. Children run the other way
cant li estyle limitations secondary to visual disability rom a when they see him coming. He cries himsel to sleep every
cataract, then cataract surgery is (potentially) indicated. “C” is night because o this.
CHAPTER 19 • Oph Th Al mOl Og y 545

FIGURE 19-13. FIGURE 19-14.

T is growth is most likely due to: Episcleritis is in ammation o the tissues overlying the sclera.
A) Systemic in ammatory illness One thing that di erentiates episcleritis rom scleritis is that
B) Exposure to UV light and dust patients with episcleritis do not have the severity o pain noted
C) Foreign-body granuloma with scleritis. A pinguecula appears as yellowish, slightly raised
D) rauma to the sclera and cornea with scarring nodules that are ound on the conjunctiva nasal or temporal to
E) Any o the above can lead to this nding the cornea. A pinguecula generally appears in middle age, has
similar risk actors as pterygium, and removal is rarely indicated.
The correct answer is “B.” T is is a pterygium, an overgrowth
o conjunctival tissue, which is a result o recurrent exposure to HELPFUL TIP:
UV light and high winds with dust. It is o no clinical signi - E isc eritis wi enera reso ve s ontaneous or wit
cance unless it encroaches on the visual eld or causes cosmetic a bit o to ica steroid. Scleritis can result in globe
distress. It is characterized by involvement o the cornea nasally rupture and s ou d be re erred ur ent .
or temporally as opposed to a pinguecula, which has a similar
appearance but does not in ringe upon the cornea. Care ul his-
tory taking o en uncovers a patient history o outdoor li estyle
( arming, construction, boating, etc.) or residing in a region QUICK QUIZ: SCl ERITIS
more equatorial and, thus, more pleasant in February than the
40 degrees north latitude and sub-zero temps we endure here in
Which o the ollowing is NO a common cause o scleritis?
the Hawkeye state.
A) Wegener granulomatosis
B) Lupus
QUICK QUIZ: pAINFUl EyE C) Gout
D) Nephrotic syndrome
A 40-year-old patient presents with a pain ul, red area on the E) Rheumatoid arthritis
eye (Fig. 19-14). He notes the gradual onset o severe pain o a
boring nature with pain in the periocular area as well. He has a The correct answer is “D.” Nephrotic syndrome does not gen-
history o rheumatoid arthritis. erally represent ongoing in ammation. More than 50% o cases
o scleritis have an associated systemic in ammatory disease
What is the diagnosis? such as gout, lupus, rheumatoid arthritis, ankylosing spondyli-
A) Episcleritis tis, or herpes zoster.
B) Pterygium
C) Scleritis QUICK QUIZ: my Impl ANTED l ENS
D) Pinguecula ISN’T WORKINg
E) Epidemic hemorrhagic conjunctivitis
A 70-year-old emale presents or a routine medical evaluation.
The correct answer is “C.” T is is scleritis. Note the erythema She reports no problems, but on review o systems she states
and in ammation, which di erentiates it rom a pterygium. that she has had a gradual decrease in vision in the right eye
Scleritis may also be nodular in nature or di use. Scleritis can over the past several months. She had success ul cataract sur-
be easily mistaken or episcleritis, which may appear similar. gery o both eyes 3 years ago. A er the surgery, she had 20/20
546 FAmIl y mEDICINE EXAmINATION & BOARD REVIEW

vision OU. Now on examination, her vision is OD 20/50 and patients. Pupil involvement should prompt an investigation
OS 20/20. She has no RAPD. T e slit-lamp examination o her or an intracranial aneurysm. “A,” Grave’s disease, may pres-
anterior segment o both eyes reveals well-positioned bilateral ent with motility de cits and compression o the optic nerve
posterior chamber intraocular lens (PCIOL) implants. T ere resulting in a RAPD. However, Grave’s disease is less likely in
seems to be a hazy membrane behind her lens implant in the this patient because it should present with lid retraction and
right eye. proptosis, not ptosis. Grave’s disease also may cause restriction
o the medial and in erior rectus muscles, which causes dif -
culty abducting and elevating the eye. “B,” Horner syndrome,
T e most likely diagnosis is which o the ollowing?
may present with ptosis, but the a ected side would have miosis
A) Posterior capsular opacity
(not mydriasis as in this patient), and it does not present with
B) Endophthalmitis
motility de cits. “D,” myasthenia gravis, may present with ptosis
C) Retinal detachment
and motility de cit, but these ndings are usually elicited with
D) Posterior uveitis
atigue and are variable.
E) Intraocular lens dislocation
Question 19.15.2 The workup o this patient should involve:
The correct answer is “A.” Months to years a er success ul A) Cerebral angiography
cataract surgery, patients may experience a gradual decline in B) C /C A
their vision due to an opaci cation o the posterior capsule C) MRI/MRA
behind their intraocular lens implant. During cataract surgery, D) Orbital ultrasonography
most o the normal capsule o the lens remains and holds the
intraocular lens implant. In success ul cataract surgery, only Answer 19.15.2 The correct answer is “C.” Although cerebral
the anterior portion o the capsule is removed. T e posterior angiography has long been the gold standard in detecting cere-
portion o this capsule may become hazy over time due to the bral aneurysms, the rst line and less invasive diagnostic test is
proli eration o residual lens epithelial cells. T e patient does MRI/MRA.
not have any symptoms o in ection or in ammation and re-
Objectives: Did you learn to . . .
ports no ashes or oaters that would eliminate most o the
• Dia nose a t ird crania nerve a s and initiate
other choices. An intraocular lens dislocation is rare and is ana e ent?
usually seen in the setting o trauma. reatment or capsular
opaci cation is YAG laser capsulotomy and is per ormed in an
outpatient setting.
Clinical Pearls
A wa s evert t e e e ids in an atient w o a ave a or-
CASE 19.15 ei n bod in t e e e.
A 56-year-old white male with a history o hypertension and Do not de a ur ent consu tation wit an o t a o o ist in
diabetes complains o double vision and pain or the past 2 a atient sus ected o avin orbita ce u itis.
days. On examination, his vision is OD 20/50 and OS 20/25. Do not er or routine reo erative tests on atients under-
He has a larger pupil with a RAPD OD. T e lid o his right eye oin e e sur er un ess t ere is a edica indication.
is slightly lower than the le . His right eye is deviated slightly Do not rescribe antibiotics or vira conjunctivitis.
temporally and in eriorly, and he has di culty adducting
per or a co ete and t orou o t a o o ic exa ina-
and elevating the eye.
tion and consider i a in or an atient resentin wit
new onset strabis us.
Question 19.15.1 The most likely diagnosis is which o the
Re er atients wit t e 2 diabetes or di ated e e exa ina-
ollowing? tion at initia dia nosis. patients wit T e 1 diabetes s ou d
A) Grave’s disease ave a di ated undusco ic exa ination wit in 5 ears o
B) Horner syndrome dia nosis.
C) T ird nerve palsy
Trau atic ro tosis s ou d ave an i ediate eva uation or
D) Myasthenia gravis ossib e atera cant oto .

Answer 19.15.1 The correct answer is “C.” T e case presented


is a typical scenario o a third nerve palsy. Patients with a third
nerve palsy present with diplopia ( rom ocular muscle paraly-
sis), ptosis, and a dilated pupil. Recall that the ocular muscles ACKNOWLEDGMENTS
are innervated by CN3 except or the lateral rectus (CN6) and Photographs were provided by the Department o Ophthal-
the superior oblique (CN4), so the eye will be “down and out” mology, University o Iowa. Special thanks to Dr. Greenlee,
when the lateral rectus and superior oblique act unopposed Dr. James Folk, and Dr. Young Kwon or their photograph con-
in a third nerve palsy. O en the pupil is spared in diabetic tributions.
CHAPTER 19 • Oph Th Al mOl Og y 547

BIBLIOGRAPHY Ozcura F, et al. E ects o central corneal thickness, central


corneal power, and axial length on intraocular pressure
American Academy o Ophthalmology. Basic and Clinical
measurement assessed with Goldmann applanation
Science Course. 2006–2007. San Francisco, CA: American
tonometry. Jpn J Ophthalmol. 2008;52:353–356.
Academy o Ophthalmology; 2006.
Pediatric Eye Disease Investigator Group. T e clinical spectrum
Dickersin K, et al. Surgery or nonarteritic anterior ischemic
o early-onset esotropia: Experience o the Congenital
optic neuropathy. Cochrane Database Syst Rev. 2006;1:
Esotropia Observational Study. Am J Ophthalmol. 2002;
CD001538.
133:102–108.
Evans JR. Antioxidant vitamin and mineral supplements
Sheikh A, Hurwitz B. Antibiotics versus placebo or acute
or slowing the progression o age-related macular
bacterial conjunctivitis. Cochrane Database Syst Rev.
degeneration. Cochrane Database Syst Rev. 2006;2:
2006;2:CD001211.
CD000254.
Shields MB. Textbook o Glaucoma. 5th ed. Philadelphia, PA:
Fingert JH. Primary open-angle glaucoma genes. Eye (Lond).
Lippincott Williams & Wilkins; 2005.
2011;25(5):587–595.
Sowka J, et al. Handbook o Ocular Disease Management.
Liu Y, Allingham RR. Molecular genetics in glaucoma. Exp Eye
Available at http://cms.revoptom.com/handbook/;
Res. 2011;93:331–339.
Accessed December 11, 2011.
Nicholson BP, Schachat AP. A review o clinical trials o anti-
Wormald R, et al. Photodynamic therapy or neovascular
VEGF agents or diabetic retinopathy. Grae es Arch Clin
age-related macular degeneration. Cochrane Database Syst
Exp Ophthalmol. 2010;248:915–930.
Rev. 2007;3:CD002030.
Otolaryngology
Ch is Bu esh
20
“E” are examples o AOM. “A,” purulent otorrhea without evi-
CASE 20.1 dence o otitis externa, would be the one exception where you
A 2-year-old is brought to your o ce by her mother who is can diagnose AOM without even seeing the M. “D” describes
concerned that she has been pulling at her le ear since late OME without in ammation.
last night and has a ever o 101.3°F. She has had recurrent
bouts o these symptoms, the last o which was 9 months ago.
HELPFUL TIP:
Each time, the symptoms resolved with one “shot.” She is
Do ’t be ieve a ed ea d um. B itse , ed ess o the TM
alert and interactive. She has some evidence o mucoid dis-
has a 15% positive p edictive va ue o dia osi AOM.
charge rom her nares bilaterally.
Use pneumatic otoscopy, which is the sta da d o ca e
o dia osi AOM. Tympanometry is a a te ative
Question 20.1.1 Each o the ollowing ndings is diagnostic
to p eumatic otoscop . O cou se, ou sti eed to ook
o acute otitis media (AOM) EXCEPT:
i the ea .
A) Pro use, purulent ear discharge without other evidence o
otitis externa.
B) Air– uid level behind the tympanic membrane ( M) with
marked redness o the M and poor movement with pneu- Your patient’s le M is opaque, red, and immobile upon
matic otoscopy. pneumatic otoscopy.
C) Bulging, thickened yellow and red M that does not move
well with pneumatic otoscopy. Question 20.1.2 Each o the ollowing actors increases her
D) Bubbles in uid behind the M with impaired mobility o risk or developing otitis media EXCEPT:
the M on pneumatic otoscopy. A) She attends day care
E) Yellow, opaque M, poor movement with pneumatic otos- B) Her mother smokes inside the house
copy, and substantial ear pain. C) T e patient is a emale
D) Patient still uses a paci er
Answer 20.1.1 The correct answer is “D.” Suspected ear in ec-
tions drive many parents to bring their children to a amily phy- Answer 20.1.2 The correct answer is “C.” T e ollowing are
sician. T ere may be uid in the middle ear that is not in ected known risk actors or the development o AOM: day care atten-
(otitis media with e usion [OME]). In order to diagnose AOM, dance, smoking inside the home, male gender, paci er use,
you need evidence o uid in the middle ear and in ammation, children in developing countries, age between 6 and 18 months,
and the symptoms should be acute (starting in the last ew days). lack o breast eeding, and going to bed with a bottle.
Middle ear e usion is diagnosed by (1) bubbles and/or air–
uid level behind the M or (2) two or more o the ollowing: Question 20.1.3 Which o the ollowing ndings is reliably
decreased or absent M movement with pneumatic otoscopy, ound in patients with AOM?
opaci cation o the M, and discoloration o the M (yellow, A) Fever
white, blue). T ese ndings get you to OME but not AOM. o B) Ear pulling
diagnose AOM, you will need to have OME with evidence o C) Irritability
acute in ammation, such as marked pain, thickened and/or D) Rhinitis
bulging M, and reddened M. For these reasons, “B,” “C,” and E) None o the above

548
CHAPTER 20 • OTOl Ar yn g Ol Og y 549

Answer 20.1.3 The correct answer is “E.” None o the above is


HELPFUL TIP:
reliably ound in patients with AOM. Other unreliable actors
n o a tibiotic has bee p ove supe io to amoxici i
include vomiting, diarrhea, and cough. T e presence or absence
i the t eatme t o otitis media. Othe a tibiotics wi
o any o these ndings is not help ul in making the diagnosis
wo k but a e mo e expe sive a d/o have eate side
o otitis media. Note that while ear pain is a symptom o in am-
e ects. This is ike seco da to the act that most OM
mation, it is a relatively weak predictor o AOM and must be
is vi a a d wi et bette e a d ess o what we do.
accompanied by other ndings as listed above. However, pneu-
matic otoscopy or otoscopy plus tympanometry is the way to
make the diagnosis, but always make sure that you have a good
seal or you run the risk o a “ alse-positive” nding. Question 20.1.5 Which o the ollowing statements best
characterizes the role o antibiotics in the treatment o
HELPFUL TIP: AOM?
A chi d e < 6 mo ths o a e with otitis media shou d A) Antibiotics have been shown to reduce suppurative compli-
be t eated with a tibiotics. Patie ts om 6 mo ths to cations o AOM, such as mastoiditis, in developed countries.
2 ea s shou d eceive a tibiotics i the diagnosis is B) T e majority o patients with AOM bene t rom the use o
suspected and the patient meets high-risk criteria antibiotics.
(mode ate-to-seve e ea pai , eve > 39°C, immu o- C) T e use o antibiotics or AOM reduces hearing loss and
supp essed). I patie ts 6 mo ths to 2 ea s, obse - bene ts language development.
vatio is a optio i the meet ow- isk c ite ia ( eve D) With or without antibiotics, about 75% o children have res-
< 39°C, mi d ota ia, immu ocompete t, u i ate a olution o AOM symptoms a er 7 days.
AOM, and o ow-up assu ed withi 48–72 hou s). Pa- E) All o the above are true.
tie ts with p ove AOM who a e o de tha 2 ea s
ma be obse ved athe tha t eated with a tibiotics as Answer 20.1.5 The correct answer is “D.” T e bene t o antibi-
o as the meet ow- isk c ite ia. A a esics shou d be otics or most children with AOM is marginal and the number
ive to a patie ts. The e’s a so the optio o p ovid- needed to treat (NN ) is about 12; thus, the option exists to
i a “backup” a tibiotic p esc iptio (e do sed b the observe and not even give antibiotics in children ≥ 2 years who
Ame ica Academ o Pediat ics) o the patie t to sta t have mild symptoms, are immunocompetent, and have good
i the s mptoms a e pe sisti o e tha 2 to 3 da s. ollow-up. T e rest o the statements are incorrect. Antibiotics
do not reduce suppurative complications in developed coun-
tries, but they do seem to prevent suppurative complications
T is patient has not had any problems with otitis media or at in developing countries where sanitation and healthcare access
least 9 months, has not been on antibiotics during that time, are not optimal. “B” is incorrect. T e bene t is limited to a 5%
is not in day care, and has no allergies. You opt to treat her absolute reduction in those who have pain at days 2 to 3 (11.6%
with an antibiotic. without antibiotics vs. 15.9% with antibiotics). “C” is incorrect
as well, since treating with antibiotics does not impact these
Question 20.1.4 What is the most appropriate treatment outcomes in any way.
or this patient?
A) Amoxicillin 40 mg/kg/day divided ID
HELPFUL TIP:
B) Amoxicillin 80 to 90 mg/kg/day divided BID
AOM is usua caused b Streptococcus pneumoniae,
C) Ce riaxone 50 mg/kg IM once
Haemophilus influenzae, Moraxella catarrhalis, a d va i-
D) Azithromycin 10 mg/kg or 1 day then 5 mg/kg or days 2 to 5
ous vi uses. Most a e vi a , a d the majo it o cases—
E) Amoxicillin/clavulanate 40 to 80 mg/kg/day divided BID
bacte ia o vi a —wi eso ve spo ta eous .

Answer 20.1.4 The correct answer is “B.” Amoxicillin is the


rst-line treatment o AOM. T e dose is 80 to 90 mg/kg/day
in all patients whether antibiotic-naive or not. More broad- You prescribe amoxicillin or 10 days and suggest acetamino-
spectrum drugs such as ce riaxone and amoxicillin/clavulanate phen or com ort. A ew days later, the patient’s mother calls
should be reserved or patients who ail initial therapy with a to say that she is no better. You ask her to come in to clinic or
rst-line drug or have a penicillin allergy. evaluation.

HELPFUL TIP: Question 20.1.6 When treating AOM, which o these individ
r emembe that ou ca t eat chi d e o de tha uals should be considered a treatment ailure and switched
6 ea s with a 5-da cou se o amoxici i . Amoxici i / to another antibiotic?
c avu a ate shou d be ese ved o t eatme t ai u es. A) A patient with a ever that continues at 24 hours a er start-
Ce di i o ce u oxime is the backup p a i case o ing an oral antibiotic
pe ici i a e . B) A child who is still tugging at his ear 5 days into a course o
antibiotics
550 FAMIl y MEDICIn E EXAMIn ATIOn & BOAr D r EVIEW

C) A symptomatic child who still has a bulging, red, immobile amoxicillin and trimethoprim/sul amethoxazole. Give hal o
M 3 days a er starting antibiotics the usual daily dose. T is is generally given at bedtime. O en,
D) A child who continues to have rhinorrhea 1 week a er start- antibiotics can be stopped during the summer since an upper
ing antibiotics respiratory in ection (URI) is the precipitant o most cases o
E) All o the above otitis media. Pneumococcal vaccine (e.g., Prevnar) will reduce
the risk o recurrence in children with severe and recurrent
Answer 20.1.6 The correct answer is “C.” You should consider AOM. T e same is true o in uenza vaccine. Also recommended
switching to a di erent antibiotic in patients who remain symp- to reduce the requency o AOM: avoid paci er use, avoid bottle
tomatic at 3 days and who continue to have positive ndings propping at night, avoid smoke exposure, and encourage breast
on pneumatic otoscopy. Symptoms are not enough: they are eeding or at least the rst 6 months o li e.
unreliable. Remember, since most o these are viral in ections,
you are not doing a whole lot o good with your antibiotics HELPFUL TIP:
anyway. “A” is incorrect because 24 hours is not suf cient to A thou h it is t aditio a do e, the e is o easo to
determine i a particular antibiotic will be e ective. “B” is incor- o ow up AOM i patie ts > 15 mo ths o a e who a e
rect because children pull at their ears or a number o reasons as mptomatic. C ea , i the a e sti s mptomatic,
besides AOM (such as “Ha! I just discovered I have ears!”). “D” o ow-up is wa a ted.
is incorrect. I you chose this one, back to Microbiology 101 or
you! Rhinorrhea does not respond to antibiotics and is most
likely not bacterial in origin. HELPFUL TIP:
Addi Co tispo i suspension (the so utio bu s) is
HELPFUL TIP: app op iate i the e is AOM with a uptu ed TM (ma i-
Fo t eatme t ai u es, amoxici i /c avu a ate is ecom- ested b pu u e t ea d ai a e). Othe a tibiotic d ops
me ded b the AAFP a d AAP. Othe optio s i c ude (cip o oxaci ) ca be used as we but a e mo e expe -
ce di i , ce podoxime, ce t iaxo e, a d ce u oxime. sive. n o . . . Co tispo i wo ’t cause hea i oss.
Mac o ides a d TMP/SMX a e ess e ective as seco d-
i e the ap because o bacte ia esista ce (this i -
c udes the eve -popu a azith om ci , o ks). r emembe T e patient returns 4 weeks later with the mother saying, “She
the umbe eeded to ha m is about 8 with amoxici i / is still pulling at her le ear.” T ere are no other complaints.
c avu a ate; the dia hea a d esu ta t diape ash a e On examination, you nd the le M is without redness or
o te mo e dist essi tha the otitis. opacity but there is still a uid level. T e right ear examina-
tion is unremarkable.

She returns with persistent pain and ever a er taking amoxi- Question 20.1.8 What is your next diagnostic step?
cillin or 3 days. On examination, you nd evidence o persis- A) Pneumatic otoscopy
tent AOM. You switch the patient to your avorite second-line B) Hearing test
antibiotic. You see her back in 2 weeks or an ear check and C) ympanostomy
nd complete resolution. T e mother asks what she could do D) No urther diagnosis needed—treat with antibiotic
to avoid these troublesome in ections in the uture.
Answer 20.1.8 The correct answer is “A.” Even though we all
Question 20.1.7 All o the ollowing have been shown to think we can do it well, the diagnosis o otitis media is raught
reduce the incidence o recurrent otitis media EXCEPT: with problems. Pneumatic otoscopy should be done in essen-
A) Antibiotic prophylaxis tially all patients but especially in those in whom long-term
B) Conjugate pneumococcal vaccine therapy is being considered. Remember that uid can persist
C) ympanostomy tubes or a month or more a er an otitis media.
D) onsillectomy
E) In uenza vaccine On your examination, the M does not move with insu a-
tion. T e patient’s mother asks you i the child should have
Answer 20.1.7 The correct answer is “D.” Primary tonsillec- tubes placed.
tomy has not been shown to reduce the recurrence o otitis
media. However, adenoidectomy with or without tonsillec- Question 20.1.9 Which o the ollowing is NOT a criterion
tomy will reduce the rate o recurrent otitis media in patients or tympanostomy tubes?
who already have tympanostomy tubes. “A,” the use o anti- A) Chronic bilateral e usions or more than 3 months with
biotic prophylaxis, will reduce recurrent otitis media. Anti- unilateral hearing loss
biotic prophylaxis should be considered in the patient who B) Failure o antibiotic therapy to prevent recurrent otitis media
has had ≥ 3 episodes o otitis media in 6 months or ≥ 4 epi- C) Language delay secondary to otitis media
sodes in 12 months. Reasonable choices or antibiotics include D) Greater than 20 dB hearing loss bilaterally
CHAPTER 20 • OTOl Ar yn g Ol Og y 551

Answer 20.1.9 The correct answer is “A.” Patients should meet TABLE 20-1 CAUSES OF EAR PAIN
the criteria listed above be ore being considered or tympa-
• Au icu a disease
nostomy tubes. Note that this requires that patients also meet
• Ca a disease
the criteria or prophylactic antibiotic therapy (≥ 3 episodes o • Otitis exte a
AOM in 6 months or ≥ 4 episodes in 12 months). A modi ca- • Fo ei bod
tion o “A” is also a criterion: chronic bilateral e usions or more • T auma
than 3 months with bilateral hearing loss. Although included • Eczema
• r amsa –Hu t s d ome
as a criterion, there is no evidence that tympanostomy tubes
• Midd e ea disease
improve language development in the short or long term. • Otitis media
• Mastoiditis
I uid persists in the middle ear a er AOM, it is termed • Mé iè e disease
OME. • r e e ed pai
• De ta disease (e. ., abscess)
• Tempo oma dibu a joi t s d ome
Question 20.1.10 Which o the ollowing interventions has • Ca otid ia
proven bene t in patients with OME? • Pha ea disease (e. ., pha itis)
A) Oral decongestants • C a ia e ve esio s (Cn V, VII, IX, X)
B) Oral antihistamines • Uppe ce vica e ve disease, a causes (e. ., disk disease)
C) Prolonged treatment (≥ 1 month) with oral antibiotics • Be pa s a d othe eu o o ic diseases (e. ., t i emi a eu a ia)
D) Oral corticosteroids
E) None o the above

Answer 20.1.10 The correct answer is “E.” For persistent OME, pain is increased by motion o the pinnae. T e external audi-
there are no use ul medical interventions (although there is some tory canal is erythematous, edematous, and exquisitely ten-
data that amoxicillin-clavulanate leads to normal tympanometry der when you try to use the otoscope to examine her M.
2 weeks be ore it is achieved with placebo [JAMA Pediatr. 2014 T ere is whitish debris in the external auditory canal.
May 5]). Autoin ation (“eustachian tube exercises” or orced
exhalation with closed nose and mouth) is o en recommended Question 20.2.1 The most likely organism involved in this
but has not shown a bene t (and try explaining how to do this to patient’s disease is:
a 6-month-old). Patients may bene t rom surgical intervention A) Streptococcus
(see the question about tympanostomy tubes above). B) Haemophilus
C) Moraxella
Objectives: Did you learn to . . .
D) Pseudomonas
• Dia ose otitis media app op iate ?
E) Parainf uenza
• I itiate t eatme t i a patie t with otitis media?
• r eco ize ai ed a tibiotic the ap a d choose a ew a tibi- Answer 20.2.1 The correct answer is “D.” T is patient likely
otic o otitis media?
has otitis externa. T e most common pathogenic organism iso-
• Desc ibe p eve tio st ate ies o ecu e t otitis media? lated in cases o otitis externa is Pseudomonas (guess those ag-
• r eco ize i dicatio s o t mpa ostom tube p aceme t? ellated bacteria sure can cause swimmer’s ear!) ollowed closely
by Staphylococcus aureus. However, up to one-third o cases o
QUICK QUIZ: EAr PAIn otitis externa are polymicrobial.

Question 20.2.2 Which o the ollowing is/are considered


Which o the ollowing can cause ear pain?
rst line treatment or otitis externa?
A) emporomandibular joint ( MJ) syndrome
A) Oral cipro oxacin
B) Cervical spine degenerative arthritis
B) Acetic acid ear drops
C) Cranial nerve lesions (5, 7, 9, or 10)
C) Polymyxin and neomycin combination ear drops
D) Bell palsy
D) A and C
E) All o the above can cause ear pain
E) B and C
The correct answer is “E.” All o the above can cause ear pain.
Answer 20.2.2 The correct answer is “E.” Otitis externa can
A more complete list is given in able 20-1. T e main point here
be treated with a wide array o topical agents. One option is to
is that not all ear pain is otitis media.
acidi y the external ear canal. Neither Pseudomonas nor Staph-
ylococcus species can thrive at an acidic pH. T us, acetic acid
CASE 20.2 drops (e.g., VoSol®) can be used: they are cheap and e ective.
A 23-year-old emale college student presents to your clinic Another approach is to use a topical antibiotic. Polymyxin/
complaining o ear pain. She is on the swimming team and neomycin combinations (e.g., Polysporin) are sa e and e ective.
notes that this pain occurs during swimming season. T e A number o other antibiotic preparations are available as well.
552 FAMIl y MEDICIn E EXAMIn ATIOn & BOAr D r EVIEW

Alcohol-based solutions are another alternative. “A” is incorrect letting them soak into her ear? Another issue is the develop-
because oral treatment is not indicated or simple otitis externa. ment o an allergic reaction, especially in response to neomycin
However, topical cipro oxacin may be used. (up to 35% o patients treated chronically with topical neomycin
develop a dermatitis). Patients may also “ ail” treatment or oti-
HELPFUL TIP: tis externa due to misdiagnosis. Otomycosis, a ungal in ection
The e a e a umbe o much mo e expe sive t eat- o the auditory canal, causes redness, discharge, itching, and
me ts o otitis exte a o the ma ket, i c udi cip o- sometimes pain. With respect to “A,” treatment ailures due to
oxaci otic d ops a d o oxaci otic d ops. These have antibiotic resistance are uncommon, and as noted above, anti-
o adva ta e a d a e ve expe sive. I act, there is biotics are not even necessary in the treatment o most cases o
no treatment advantage to using antibiotics at all. otitis externa.
Topical drying agents, alcohols and acetic acid, have
just as good an outcome as do antibiotics. On examination, you notice ne, white, cotton-like bers ll-
ing the ear canal along with the other debris. T e examina-
tion is otherwise unchanged.
Question 20.2.3 This patient is concerned about recur
rences o her otitis externa. What advice can you give her? Question 20.2.5 What is your next step?
A) Avoid exposure by putting a petroleum jelly (e.g., Vaseline) A) Admit or intravenous (IV) antibiotic and anti ungal therapy
impregnated cotton plug in her ear be ore swimming B) Clean the ear canal under direct otoscopy and add oral
B) Use a blow dryer on her ear a er swimming amoxicillin/clavulanate to her medication regimen
C) Instill a 50/50 mixture o alcohol and vinegar in her ears C) Clean the ear canal under direct otoscopy and add topical
a er swimming clotrimazole 1% to her medication regimen
D) Avoid swimming when she has active disease D) Order C or MRI o the head and neck to rule out abscess
E) All o the above E) Re er to an otolaryngologist

Answer 20.2.3 The correct answer is “E.” All o the above can Answer 20.2.5 The correct answer is “C.” On examination,
be used to minimize disease recurrence. T e bene t o “C” is you have identi ed signs o otomycosis. It could be that the oto-
less certain. mycosis was present initially or developed in the interval with
antibiotic administration. Otomycosis is usually due to Asper-
gillosis; Candida only represents 10% to 20% o cases. T orough
HELPFUL TIP: cleaning o the ear canal is an important part o therapy. A topi-
r emembe that necrotizing (malignant) otitis cal anti ungal that is active against Aspergillosis is recommended
externa is a di e e t c eatu e a to ethe that occu s (e.g., clotrimazole, miconazole, and nystatin).
p ima i i diabetic patie ts but a so i those with HIV
a d othe immu osupp essi diseases/meds. It is a
i vasive pseudomo a (95%) ce u itis that causes e - HELPFUL TIP:
thema a d te de ess a ou d the ea . Comp icatio s Ea wicks a e exce e t too s o aidi i the de ive
i c ude osteom e itis, me i itis, abscess o matio , o medicatio deep i to the ea ca a but should be
a d c a ia e ve pa sies. It is a t ue eme e c equi - changed daily.
i IV a tibiotics a d su ica co su tatio .

Objectives: Did you learn to . . .


You treat the patient with neomycin/polymyxin drops, and • Ide ti bacte ia patho e s imp icated i otitis exte a?
the symptoms persist and possibly worsen a little a er 5 days. • Dia ose a d t eat a patie t with otitis exte a?
T e patient has no ever and no signs o cellulitis around the • r ecomme d p eve tio st ate ies o otitis exte a?
ear. • Dete mi e wh t eatme t o otitis exte a ma ai ?

Question 20.2.4 O the ollowing possibilities, which is the


LEAST likely to explain her persistent symptoms?
CASE 20.3
A) Resistant organisms A 59-year-old male presents with a 3-week history o hoarse-
B) Noncompliance with medical recommendations ness. He denies sore throat or heartburn. He has had no
C) Misdiagnosis o otomycosis evers, night sweats, or weight loss. When he initially pre-
D) Development o contact dermatitis sented a week ago, your partner treated him empirically or
postnasal drainage. He smokes 2 packs o cigarettes per day
Answer 20.2.4 The correct answer is “A.” Several things could and drinks alcohol daily. On examination, his vital signs are
explain her persistent symptoms. First, the patient should be normal. His voice sounds husky. You nd no other abnor-
questioned regarding compliance. Is she still swimming despite malities (besides the burn holes in his shirt and the pint o
advice to the contrary? Is she using the drops at least ID and whisky in his back pocket).
CHAPTER 20 • OTOl Ar yn g Ol Og y 553

Question 20.3.1 The best next step in the management o A “hoarse” voice may be due to vocal cord lesions, muscle ten-
this patient is: sion dysphonia, and re ux laryngitis. Vocal atigue (loss o
A) Empiric antibiotic treatment volume over time) may also be noted (especially during presi-
B) Empiric proton pump inhibitor treatment dential campaigns) and is o en caused by muscle tension dys-
C) Direct laryngoscopy phonia, vocal cord paralysis, re ux laryngitis, or vocal abuse.
D) Esophagogastroduodenoscopy (EGD) However, one cannot eliminate a cause o hoarseness based on
E) Neck MRI these characteristics.

Answer 20.3.1 The correct answer is “C.” T e rst concern


is to rule out malignancy, so the larynx should be visualized. HELPFUL TIP:
Direct laryngoscopy is a straight orward of ce procedure and The a opha x is i e vated b the ecu e t
takes only a ew minutes. I the equipment and expertise are not a ea e ve, a b a ch o the va us e ve (c a ia
available in your of ce, re erral to an otolaryngologist is appro- e ve X). I additio to bei u a d i te esti medi-
priate. Although there are no rm guidelines, some authors ca t ivia, k owi the i e vatio is impo ta t because
recommend direct laryngoscopy a er 2 weeks o hoarseness in chest ma i a c , a eu sms, comp icatio s o tho acic
patients who are at risk (older patients and those who have a su e , etc. ca pote tia p ese t with hoa se ess.
history o tobacco and alcohol use). Since laryngoscopy is such
an available, low-cost, low-risk procedure, it is hard to justi y
postponing it or any patient at risk or malignancy. T ere ore, On direct laryngoscopy, you notice a mass lesion on the right
“A” and “B” are incorrect, as urther empiric medication trials vocal cord. You re er the patient to an otolaryngologist. T e
will only delay laryngoscopy. Besides, antibiotics are not indi- patient asks i you think that the mass is cancer. Because you
cated or the lone symptom o hoarseness. In other instances are compassionate and also not entirely certain, you avoid
(e.g., low-risk patient with heartburn and hoarseness), empiric saying, “Oh, heck yeah. T at’s cancer all right.” You try to be
proton pump inhibitor therapy may be more practical, since more optimistic.
gastroesophageal re ux is a common cause o hoarseness. “D” is
incorrect. In this case, direct laryngoscopy is pre erred to EGD. Question 20.3.3 However, you want to remind him o risk
Finally, “E” is incorrect. Neck MRI is not indicated in the ini- actors or laryngeal cancer, which include all o the ollow
tial evaluation o hoarseness, but it might be used or ollow-up ing EXCEPT:
a er laryngoscopy or to investigate a neck mass. A) obacco smoking
B) Alcohol use
Question 20.3.2 All o the ollowing are potential causes o C) Epstein–Barr virus (EBV)
hoarseness EXCEPT: D) Family history o head and neck cancers
A) Vocal cord mass E) Male sex
B) In ectious laryngitis
C) Hypothyroidism Answer 20.3.3 The correct answer is “C.” EBV in ection is
D) Lung malignancy associated with the development o nasopharyngeal cancer, not
E) A vow o silence laryngeal cancer. Additionally, EBV in ection has been associ-
ated with Burkitt lymphoma (children in A rica), Hodgkin
Answer 20.3.2 The correct answer is “E.”Far rom being a cause disease, and non-Hodgkin lymphoma. obacco and alcohol
o hoarseness, voice rest is o en recommended or patients with use are independent risk actors or the development o most
hoarseness due to overuse (e.g., singers). All o the other options types o head and neck cancers (oral, laryngeal, etc.), and the
are known to cause hoarseness. O particular note is “C”: hypo- two substances may act synergistically in the promotion o these
thyroidism can result in an accumulation o connective tissue cancers. A amily history o head and neck cancer has a weaker
elements, basically myxedema, in the vocal cords. Intrathoracic association, but the association is still present. Males are two to
processes such as lung cancer can present with hoarseness (see our times more likely to have head and neck cancers compared
“Help ul ip” below). with emales. T ese risk actors sound a lot like our patient.
Characterizing the nature o the patient’s hoarseness can
be help ul in narrowing the di erential diagnosis. T e voice Question 20.3.4 I this patient is ound to have cancer, what
changes may be urther characterized as breathy, low-pitched, pathologic variant is most likely?
strained, tremulous, or hoarse. A “breathy” voice may be seen A) Adenocarcinoma
with vocal cord paralysis, abductor spasm, or unctional dys- B) Squamous cell carcinoma
phonia. A “low-pitched” hoarseness might be due to edema C) Schneiderian papilloma
(seen in smokers), vocal abuse, re ux laryngitis, vocal cord D) Neuroblastoma
paralysis, or muscle tension dysphonia. A “strained” voice may
occur with adductor spasm, muscle tension dysphonia, or re ux Answer 20.3.4 The correct answer is “B.” Upon pathologic
laryngitis. A “tremulous” voice occurs in parkinsonism, essen- examination, the great majority o head and neck cancers are
tial tremor, spasmodic dysphonia, or muscle tension dysphonia. ound to be squamous cell carcinomas. Adenocarcinoma may
554 FAMIl y MEDICIn E EXAMIn ATIOn & BOAr D r EVIEW

ollow the patient closely and re-examine him. “D,” geographic


tongue, is so named because o the meandering white-bordered
patches that occur on the dorsum o the tongue. It is most o en
asymptomatic, and the lesions vary in shape (or completely
resolve) over time. Finally, “E,” an aphthous ulcer, is just that—
an ulcer, not a plaque. You should not con use leukoplakia or
an aphthous ulcer.

HELPFUL TIP:
n o i te ve tio s have bee show to be use u i p o-
moti the e essio o eukop akia.

HELPFUL TIP:
I c easi , a histo HPVi ectio is bei eco ized
FIGURE 20-1. F om Usati e r P et a ., eds. Color Atlas of Family Medicine. i o opha ea ca ce s. As much as 70% o these ca -
2 d ed. n ew yo k: Mcg aw-Hi ; 2013. Cou tes o r icha d P. Usati e, MD. ce s, usua squamous ce ca ci omas, ma be caused
b HPV i ectio . HPV t pe 16 ca ies the st o est as-
sociatio with head a d eck ca ce s a d is cove ed
b mode vacci es. Cu e t e o ts at HPV vacci atio
arise rom the gastrointestinal tract and could be seen on laryn- a e expected to educe the i cide ce o head a d eck
goscopy but would rarely occur on the vocal cords. Schneide- ca ce s i utu e e e atio s—just o e mo e easo to
rian papillomas (“C”) are polyps that arise rom the nasal and ca the HPV vacci e a “a tica ce ” vacci e a d ot a
sinus mucosae, are associated with HPV, and may trans orm “sa e sex” vacci e.
into carcinomas. Neuroblastomas (“D”), which arise rom the
sympathetic nervous system, rarely occur in the head and neck
region.
You receive a letter rom the otolaryngologist stating the
During your examination o the oropharynx, you also patient does indeed have squamous cell carcinoma o the lar-
encountered a small, white, indurated plaque on the under- ynx. T e patient will be seen in consultation with an oncolo-
side o the tongue. When you scraped the plaque with a gist and presented at tumor board. His treatment may consist
tongue blade, nothing happened (see Fig. 20-1). o surgery, chemotherapy, and/or radiation. Un ortunately,
he lost the roll o the cosmic dice.
Question 20.3.5 This lesion is most appropriately described Objectives: Did you learn to . . .
as: • g e e ate a di e e tia dia osis o hoa se ess o voice?
A) Squamous cell carcinoma • Eva uate a patie t with a voice comp ai t?
B) Candida albicans • Ide ti o a esio s, pa ticu a eukop akia?
C) Leukoplakia • r eco ize impo ta t issues i the p eve tio a d t eatme t
D) Geographic tongue o head a d eck ca ce s?
E) Aphthous ulcer

Answer 20.3.5 The correct answer is “C.” Leukoplakia is a


CASE 20.4
premalignant lesion o the oropharynx (about 5% will progress A 61-year-old man presents to your o ce complaining that
to cancer over 10 years). It occurs in response to trauma and/ over the last ew months he cannot seem to understand what
or exposure to irritants and carcinogens, having an especially people are saying when they are standing to his le side. He
strong association with smokeless tobacco (e.g., “snu ,” “chew”) also has episodes o “dizziness,” especially when he changes
use. In act, the lesion could be squamous cell carcinoma (“A”), position rom sitting to lying and vice versa. He denies nau-
and it should be biopsied. However, it would be premature sea and vomiting. He worked or 30 years in a actory and
to diagnose the patient with squamous cell carcinoma, and has had bilateral tinnitus or the last 10 years. He has had no
the lesion is more accurately described as leukoplakia. “B,” C. previous hearing problems or evaluation. His past medical
albicans lesions, may look just like leukoplakia (white plaques history is signi cant or CAD and hypertension. He takes
on oropharyngeal mucosa), but you should be able to scrape atenolol, chlorthalidone, and aspirin. T ere is no amily his-
some o the plaques o with a tongue blade (although thrush tory o ear disease. On examination, both ears are normal
can be remarkably adherent and may cause some bleeding). A in appearance. Weber test is best heard by the patient on his
trial o anti ungal therapy may be appropriate i the lesion is right side. Rinne test on both sides was negative (air conduc-
low-risk and you suspect thrush but cannot scrape it o —but tion was greater than bone conduction).
CHAPTER 20 • OTOl Ar yn g Ol Og y 555

Question 20.4.1 These ndings are consistent with which TABLE 20-2 CAUSES OF CONDUCTIVE AND
type o hearing loss on the left? SENSORINEURAL HEARING LOSS
A) Conductive
Conductive Hearing Loss Sensorineural Hearing Loss
B) Sensorineural
C) Mixed T auma: ossic e dis uptio , P esb cusis
D) Selective TM pe o atio
E) Unable to tell Ce ume i the ca a Mé iè e disease

Answer 20.4.1 The correct answer is “B.” Hearing can be Otosc e osis
assessed in the of ce using the Weber and Rinne tests. T e Ba ot auma St oke
Weber test is per ormed by putting the tuning ork on the ore-
head and seeing i the sound lateralizes to one side or the other. Otitis media Tumo (e. ., acoustic eu oma)
In conductive hearing loss (“A”), the sound will be louder (i.e., Midd e ea e usio I ectio (e. ., s phi is, CMV, etc.)
the test will lateralize) to the “bad” side (e.g., the side with wax
occluding the canal, otosclerosis). However, in sensorineural
hearing loss, the sound will lateralize to the “good” side (e.g., in Iowa). See able 20-2 or some causes o conductive and sen-
the side not a ected by a hearing problem). sorineural hearing loss.
T e Rinne test is per ormed by comparing bone conduc-
tion (on the mastoid) to air conduction. Patients will notice Question 20.4.3 What is the next step in the evaluation o
poor air conduction versus bone conduction i there is a con- this patient?
ductive hearing loss. Normal Rinne tests in both ears suggest A) Audiogram
that neither ear has conductive loss. Our patient, (or more B) Brainstem evoked responses
likely his amily) complains about decreased hearing on the C) MRI
le and the Weber test lateralizes to the right, and these nd- D) ympanogram
ings point to a problem with sensorineural hearing loss in the
le ear. Answer 20.4.3 The correct answer is “A.” An audiogram can
urther de ne the air versus bone conductance relationship,
Question 20.4.2 Which o the ollowing is LEAST likely to be
check speech discrimination, and de ne the requency o hear-
responsible or this patient’s hearing loss?
ing loss. Brainstem-evoked responses evaluate the neural path-
A) Ménière disease
ways o hearing and, along with MRI, could be use ul i tumor
B) Acoustic neuroma
were higher on the di erential. A tympanogram, which evalu-
C) Presbycusis
ates movement o the M, might be use ul i conductive hearing
D) Otosclerosis
loss were suspected.
E) Noise exposure

Answer 20.4.2 The correct answer is “D.” Otosclerosis is HELPFUL TIP:


a bony overgrowth that involves the stapes and leads to con- B ai stem-evoked pote tia s measu e how o it takes
ductive loss. Your patient has a sensorineural hearing loss. a audito si a to each the b ai stem. I a acous-
Ménière disease (“A”) presents with the classic triad o hearing tic eu oma is p ese t, the b ai stem-evoked pote tia
loss, tinnitus, and vertigo. T ese mani estations may be tempo- wi be p o o ed. Howeve , the a se- e ative ate is
rally separated with hearing loss, tinnitus, and vertigo occur- up to 30%, a d b ai stem-evoked pote tia s have bee
ring at di erent times. Patients with Ménière disease will note a but ep aced b Mr I i the dia osis o acoustic
ullness in their ear, which resolves with the onset o vertigo. eu oma.
ypically, the vertigo will last or several hours. “B” is true.
Sensorineural hearing loss can also be caused by an acoustic
neuroma, a benign tumor that arises rom cranial nerve VIII T e audiogram con rms sensorineural hearing loss in the
(acoustic nerve), and symptoms include unilateral hearing loss, le ear. Because o the associated symptoms o vertigo, you
vertigo, tinnitus, and disequilibrium. “C,” presbycusis, is one o order an MRI o the posterior ossa to rule out acoustic neu-
the most common causes o sensorineural hearing loss and is roma, and the results are negative. T e patient continues
o en thought o as the “normal” hearing loss that is associated to have episodes o vertigo, and the hearing loss on his le
with aging. Presbycusis mani ests as an inability to hear high side persists. When probed urther, he does in act have an
requencies. T is leads to problems with speech discrimination, increase in tinnitus during his vertiginous episodes.
especially in noisy environments (e.g., parties). It is typically
symmetrical and may be associated with tinnitus. However, it Question 20.4.4 You should consider all o the ollowing
may be unilateral, especially in those who have one ear turned treatments or this condition EXCEPT:
toward noisy equipment in their job (e.g., armers driving trac- A) Salt, ca eine, and tobacco restriction
tors and looking behind them—remember, this is being written B) Diuretics (e.g., hydrochlorothiazide)
556 FAMIl y MEDICIn E EXAMIn ATIOn & BOAr D r EVIEW

C) Intracochlear injection o gentamicin What is the next step in the evaluation and management o this
D) Labyrinthectomy or endolymphatic sac shunt patient?
E) H 2-blockers (e.g., cimetidine) A) Reassurance, oral rehydration, and analgesics
B) Oral antibiotics
Answer 20.4.4 The correct answer is “E.” T e clinical picture C) MRI o the head and neck
now looks most like Ménière disease—a disease or which there D) IV antibiotics
is no cure. Luckily not all patients with Ménière disease will E) Intubation and mechanical ventilation
experience worsening o their condition over time, and up to
90% are able to maintain normal daily activities with optimal The correct answer is “D.” T is patient is presenting with classic
medical management. Few patients progress to debilitating dis- signs and symptoms o Ludwig angina (related neither to angina
ease. T e mainstays o therapy include diet/li estyle modi ca- nor to Ludwig von Beethoven). T ese include brawny edema,
tion and diuretics. Outside the United States, betahistine, an tenderness and warmth in the submandibular region along with
H1-blocker, is commonly used (to theoretically reduce endo- elevation o the tongue. T e term Ludwig angina is reserved or
lymphatic hydrops), but H 2-blockers have no role in therapy. in ection o the submandibular and sublingual areas. T e status
A more aggressive approach is indicated in patients with more o the patient’s airway must be addressed, o course. T is patient
severe disease, and this might include surgery, intracochlear is stable and her airway appears patent; there ore, “E” is incor-
gentamicin (to kill the nerve and reduce vertigo), labyrinthec- rect. “A” and “B” are incorrect because patients with Ludwig
tomy, or endolymphatic sac shunt. angina may experience rapid progression o symptoms and
should be treated with IV antibiotics and observed in the hos-
HELPFUL TIP:
pital. Finally, “C” is incorrect because the diagnosis o Ludwig
T eati Mé iè e disease ca be p ob ematic. Studies
angina is clinical. Also, MRI may delay antibiotic treatment and
o this co ditio a e eque t o poo qua it with a
is unlikely to be o any value, unless you suspect the presence o
si i ica t p acebo e ect (i c udi the studies o di-
an abscess. Even then C would be the examination o choice.
u etics a d betahisti e). This, a o with spo ta eous
Surgical intervention may be necessary.
emissio s a d exace batio s, imits the use u ess o
data about t eatme t. HELPFUL TIP:
T ismus is usua abse t i l udwi a i a as opposed
to pe ito si a abscess, etc.
HELPFUL TIP:
Thi s to emembe about ti itus: It ca be caused
b a umbe o medicatio s i c udi n SAIDs, ca cium CASE 20.5
cha e b ocke s, diu etics, etc. It ca a e be caused
b a vascu a esio (b uits, A-V shu ts, etc.). Othe A 30-year-old man comes to your o ce complaining o a
causes i c ude TMJ s d ome, eustachia tube d s- swollen neck. He noticed it 3 weeks ago when he was stung
u ctio , a d—most commo —se so i eu a hea - by a bee on the right side o his anterior neck. T e area has
i oss (especia p esb cusis). continued to enlarge. It is no longer tender. It was erythema-
tous a er the sting, but the redness has resolved. He notes
no other symptoms. On examination, you nd a 2-cm rm,
Objectives: Did you learn to . . . somewhat tender, enlarged lymph node in the right anterior
• Eva uate a patie t with hea i oss? cervical chain. T e node is mobile, non uctuant, with no sur-
• Desc ibe di e e ces betwee co ductive a d se so i eu a rounding erythema. T ere are also shotty anterior and poste-
hea i oss? rior cervical nodes in addition to the larger node described.
• g e e ate a di e e tia dia osis o hea i oss? You nd neither splenomegaly nor lymphadenopathy o the
• Dia ose a d t eat a patie t with Mé iè e disease? supraclavicular, axillary, and inguinal regions.

Question 20.5.1 What elements o the presentation make


QUICK QUIZ: PEr IPHAr yn g EAl In FECTIOn malignancy LESS likely?
A) T e node is reely mobile
A 7-year-old emale is brought to your of ce by her concerned B) T e node is only 2 cm
parents. Over the last 24 hours, the patient has developed pain C) T e node is associated with trauma (bee sting)
in her mouth, drooling, and ever. She re uses to eat. On exami- D) T e node is tender
nation, she is ebrile and slightly tachycardic. T e sublingual E) All o these help to rule out malignancy
space is swollen and tender bilaterally. T e tongue is elevated in
the mouth. T ere are no ulcerations. T ere is well-demarcated Answer 20.5.1 The correct answer is “A.” Nonmalignant nodes
erythema, brawny edema, tenderness, and warmth in the sub- are generally less than 1 cm in size, reely mobile, and rubbery
mandibular area. Her respirations are normal, and her lung in constancy. Malignant nodes tend to be larger, rock-hard, and
sounds are clear. xed. T ey become immobile secondary to tumor invasion into
CHAPTER 20 • OTOl Ar yn g Ol Og y 557

TABLE 20-3 DIFFERENTIAL DIAGNOSIS OF patient outcome i the node does turn out to be malignant.
NECK MASS IN ADULTS I adenopathy does not resolve, urther evaluation including
biopsy can be done. Open biopsy and ne needle aspiration
Congenital Anomalies
each have advantages and disadvantages, but either could be
• l ate a eck: b a chia c sts a d stu ae, c stic h omas, de moids
• Ce t a eck: th o ossa duct c st, th oid masses, th mic ests, used to obtain tissue. “A,” empiric therapy with antibiotics, is
de moids possibly correct i you suspect a lymphadenitis or a bacterial
In ection/Inf ammation in ection causing secondary lymphadenopathy. However, in our
• Mo o uc eosis patient, there is no tenderness or other signs o in ection, argu-
• Tube cu osis ing against lymphadenitis.
• Toxop asmosis
• Cat-sc atch disease (Bartonella henselae)
Question 20.5.3 Which o the ollowing tests is NOT help
• Staph ococcus
• St eptococcus ul in arriving at a diagnosis in a patient with GENERALIZED
• Othe vi a , bacte ia , a d u a i ectio s lymphadenopathy?
• Sia ade itis A) CBC
• Abscess B) Chest radiograph
• I f ammato o eactive mphade opath
C) Glucose, BUN, creatinine
Neoplasm D) HIV
• Be i masses: ipoma, hema ioma, eu oma, b oma
E) Heterophile antibody
• Ma i a t masses: mucosa head a d eck ca ce s, mphoma,
th oid ca ce , sa iva a d ca ce , sa coma, dista t metastases
Answer 20.5.3 The correct answer is “C.” Glucose, BUN, and
Trauma
• Hematoma (acute o b osed) creatinine are not likely to help you with the diagnosis o gen-
• Pseudoa eu sm eralized lymphadenopathy. Lymphadenopathy in primary care
• AV stu a is malignant approximately 1% o the time. A er a period o
Idiopathic and Others observation, the workup should proceed in stages. First step:
• Metabo ic: out, CPPD (pseudo out) CBC, chest radiograph. Second step: PPD, HIV, syphilis test
• I f ammato pseudotumo (RPR or other), ANA, heterophile antibody. Final step: biopsy.
• Cast ema disease (a be i mphop o i e ative diso de )
Use your clinical judgment to determine the extent o testing
• Kimu a disease (a ch o ic subcuta eous i f ammato co ditio ,
cause u k ow ) necessary in any individual case.

HELPFUL TIP:
l mph odes a e just ike i ea estate: ocatio , oca-
the surrounding tissues and/or in ammation. Remember that tio , ocatio . Sup ac avicu a odes a e ma i a t up
pain in a node is not always indicative o an in ammatory or to 50% o the time i those o de tha 40 ea s.
benign process. Hemorrhage into, or necrosis o , a malignant
node can cause capsular distention leading to pain. “C” is o
particular note. T is patient’s bee sting was quite a while ago HELPFUL TIP:
and is unlikely to be a use ul part o the history unless there Be i mphade opath is commo i ou chi -
is ongoing in ammation. Patients will o en attribute a physi- d e . I patie ts ou e tha 5 ea s p ese ti o a
cal malady to something in their lives whether or not it makes hea th mai te a ce exami atio , up to 44% have pa -
sense rom a biological and medical perspective (one o us had pab e mph odes. Occipita a d poste io au icu a
a patient who swore that his purulent sputum was because his odes a e commo i i a ts but ot i chi d e o de
lungs were connected to his gallbladder). O course, lymphade- tha 2 ea s.
nopathy is not the only source o neck masses. See able 20-3 or
the di erential diagnosis o neck masses in adults. Objectives: Did you learn to . . .
• Desc ibe eatu es o ma i a t a d o ma i a t mph
Question 20.5.2 What is the most appropriate next step in odes?
the management o this patient? • Eva uate a patie t with mphade opath ?
A) Empiric antibiotics
B) Observation or 4 weeks
C) Open biopsy o the node
CASE 20.6
D) Fine needle aspiration o the node A 42-year-old businesswoman presents to your o ce with the
E) Incision and drainage chie complaint o 2 days o headache, sore throat, and nasal
congestion productive o green mucus. She denies any ever,
Answer 20.5.2 The correct answer is “B.” Patients with lymph- contact with ill persons, and gastrointestinal symptoms, but
adenopathy can be observed or 3 to 4 weeks unless there is a she does have a history o seasonal allergies. On examina-
suggestion o malignancy (e.g., ever, night sweats, and weight tion, she has completely normal vital signs. Her posterior
loss). Note that a 3- to 4-week delay makes no dif erence in oropharynx has mild erythema and postnasal drainage but
558 FAMIl y MEDICIn E EXAMIn ATIOn & BOAr D r EVIEW

no exudates. T ere is nasal mucosal erythema and swelling Question 20.6.3 Which o the ollowing does NOT increase
with clear rhinorrhea. Her neck is supple with no adenopa- the likelihood that a patient has a bacterial sinusitis?
thy. Respirations are clear. A) Persistence o symptoms or greater than 7 days
B) T ickened nasal mucosa or e usion on C scan
Question 20.6.1 The most likely agent causing her symp C) Maxillary tooth pain
toms and the most common cause o acute rhinosinusitis D) Unilateral maxillary sinus pain
is:
A) Rhinovirus Answer 20.6.3 The correct answer is “B.” Radiography is par-
B) S. pneumoniae ticularly poor at di erentiating bacterial sinusitis rom a simple
C) H. inf uenzae URI. T ickened nasal mucosa is only 40% to 50% speci c or
D) M. catarrhalis sinusitis ( ip a coin, it’s cheaper). T e other problem with imag-
E) Norwalk virus ing is that essentially all patients with a URI will have uid in
the sinuses. Clinical criteria are more help ul in predicting the
Answer 20.6.1 The correct answer is “A.” Viruses are the most presence o bacterial sinusitis; thus, “A,” “C,” and “D” are cor-
common cause o URIs (or colds or rhinosinusitis). Up to 50% o rect. Additionally, a biphasic course, sometimes re erred to as
colds are caused by the 100 di erent serotypes o rhinoviruses. “double sickening,” is a help ul predictor o bacterial sinusitis:
Other viruses that commonly cause colds include coronaviruses, i the patient initially improves and then gets worse, consider
RSV, parain uenza, and in uenza. Norwalk virus typically a secondary in ection. T e color o snot is meaningless. T ick,
causes an intestinal illness. T e bacteria listed (“B”–“D”) are also green, nasal drainage does not necessarily mean that bacteria
associated with in ections o the upper respiratory tract, particu- are present. Secretions will turn green with a viral illness, as
larly otitis, and sinusitis, but are much less common than viruses. with anything else that concentrates protein in the mucous (e.g.,
anticholinergics).
HELPFUL TIP:
r eside t bacte ia i the asopha x i c ude S. pneu- HELPFUL TIP:
moniae, H. influenzae, a d M. catarrhalis. Whi e most A vi a Ur I ca ast up to a mo th. I act, 25% o patie ts
causes o si usitis a e vi a , these bacte ia make up the with a Ur I a e sti s mptomatic at 14 da s, so du atio
eat majo it o o a isms causi bacte ia si usitis. o s mptoms alone is ot dia ostic o bacte ia si us-
Si usitis ma a so esu t om exte sio o de ta oot itis. So, do ’t te ou patie t that she wi be bette i
i ectio i to the si us cavit , a d these i ectio s a e 4 to 5 da s. g ive ou patie ts ea istic expectatio s: 10
caused b mic oae ophi ic a d a ae obic bacte ia. to 14 da s.

Question 20.6.2 Initial treatment or this patient includes:


T e patient is initially convinced that only antibiotics will
A) Oral decongestants
make her better. T rough skill ul negotiation (and a he y
B) NSAIDs
dose o haloperidol), you manage to avoid prescribing anti-
C) Oral antibiotics
biotics or what you strongly suspect is a viral in ection. wo
D) A and B
weeks later, the patient returns. Initially, she improved, but
E) A and C
then she developed subjective ever, ace pressure, maxillary
tooth pain, and copious green nasal drainage. You now sus-
Answer 20.6.2 The correct answer is “D.” Most cases o rhi-
pect that a bacterial sinusitis has developed. T e look on her
nosinusitis are viral and need only symptomatic treatment.
ace says, “I told you so . . . and I am not taking any more o
Analgesics and systemic and nasal decongestants are reasonable
that haloperidol stuf .” She has no drug allergies.
options. Other treatment options include ipratropium nasal
spray (which will also help to decrease mucus production),
Question 20.6.4 Which o the ollowing treatments do you
nasal steroids (at least one well-done randomized trial proved
of er as rst line therapy?
mometasone is superior to placebo . . . but it takes a while), and
A) Azithromycin
rst-generation antihistamines with anticholinergic activity
B) rimethoprim/sul amethoxazole
(e.g., diphenhydramine).
C) Prednisone
HELPFUL TIP: D) Ce riaxone
yeah, eah . . . We k ow. The pa t i e is that i st- E) Amoxicillin/clavulanate
e e atio a tihistami es a e ot use u o hi o -
hea om a Ur I. Howeve , this is tech ica i co - Answer 20.6.4 The correct answer is “E.” According to 2012
ect. A Coch a e eview showed some be e it o In ectious Diseases Society o America guidelines addressing
i st- e e atio a tihistami es a d some e ect o com- the treatment o acute bacterial sinusitis, amoxicillin/clavulanate
bi atio p oducts. (A tihistami es o the commo co d. should be used as rst-line treatment or adults and children—
Cochrane Database Systematic Rev. 2003;3; 2012;2.) although the evidence is weak. Previously trimethoprim/sul a-
methoxazole (“B”) would have been the best choice but there is
CHAPTER 20 • OTOl Ar yn g Ol Og y 559

increasing resistance. Azithromycin (“A”) is not recommended C) “Can you describe the dizziness using any other words?”
due to resistance patterns and poor penetration into the sinuses D) “Do you eel as though you are drunk?”
(Got that . . . No Z-Pack). “C” is incorrect. T ere is no reason to E) “Are you really just drunk right now?”
prescribe steroids here. “D,” ce riaxone, might be considered
in cases o treatment ailure, but oral antibiotics are generally Answer 20.7.1 The correct answer is “C.” When exploring a
pre erred. Per 2012 guidelines, doxycycline is recommended as complaint o dizziness, insist that the patient characterizes the
second-line therapy, ollowed by respiratory uoroquinolones. nature o the dizziness urther without putting words into his
O course, you should continue to recommend that the patient mouth. T is is a good time or that old technique learned in
use decongestants and other symptom-oriented therapies. medical school but rarely used in practice: the open-ended
question. T ere are essentially our types o dizziness: vertigo,
presyncope, imbalance (disequilibrium), and undi erentiated
HELPFUL TIP: dizziness. Patients will usually come up with their own termi-
Eve with bacte ia si usitis, the n n T o a tibiotics to nology that will allow you to categorize their dizziness into one
be e it o e patie t is 9. Most patie ts a e etti o o these our types.
be e it om ou ca e u mi ist atio s—o k ee-je k
eactio to p esc ibe a tibiotics (whicheve app ies . . .). T e patient describes a sensation o the room spinning, usu-
ally lasting less than a minute. Further, this sensation comes
on in dif erent positions and has led to a all, which resulted
HELPFUL TIP:
in minor injuries. He denies any upper respiratory symptoms,
r emembe that whe t eati si usitis ou a e basica
evers, hearing loss, or tinnitus. He sometimes eels nause-
t eati a abscess. D ai i it is the ke : use topica
ated with the dizziness but has not vomited. He rst noticed
ox metazo i e o imited pe iods, s stemic deco es-
the dizziness while in bed and rolling over, but it now occurs
ta ts, a d sa i e i i atio .
more requently and in various positions. Sudden turning o
the head de nitely exacerbates his symptoms. It tends to be
worse in the morning and better in the evening. You are com-
HELPFUL TIP:
ortable calling this type o dizziness vertigo.
A oses that u a e ot i ectious. r emembe a e ic
hi itis, vasomotor rhinitis (now termed idiopathic
Question 20.7.2 Which o the ollowing is LEAST likely to
rhinitis), etc. Idiopathic hi itis is a exa e ated e-
cause vertigo (in a general sense . . . not in this particular
spo se to stimu i such as co d, ecumbe c , a d ai
patient)?
po utio . It ca be di e e tiated om a e ic hi itis
A) Labyrinthitis
b the abse ce o othe a e ic s mptoms (itchi , e e
B) Ménière disease
i vo veme t, pe haps asthma, etc.) a d the abse ce o
C) Migraine aura
eosi ophi s o Ha se stai o mucus (who ooks at s ot
D) Otitis media
u de a mic oscope a wa ?). Othe co side atio s
E) Perilymphatic stula
i c ude hi itis medicame tosum as a esu t o with-
d awa om topica vasoco st icto s (e. ., ox metazo-
Answer 20.7.2 The correct answer is “D.” T ere are many
i e, cocai e).
causes o vertigo, but otitis media should not generally be
thought o as one o them (or at least should be a diagnosis o
Objectives: Did you learn to . . . exclusion). Labyrinthitis is usually caused by viral in ection
• Desc ibe the most commo patho e s i vo ved i hi osi- and results in severe vertigo that lasts or days. Ménière dis-
usitis? ease results in episodic vertigo that lasts or minutes to hours.
• r eco ize c i ica si s a d s mptoms that a e mo e co sis- Migraine aura can present with vertigo as well as many other
te t with bacte ia si usitis tha vi a hi osi usitis? symptoms (e.g., scotoma, diplopia, blindness, and paresthesias).
• P esc ibe t eatme t o vi a a d bacte ia si usitis? Perilymphatic stula occurs when perilymph leaks through a
tear in the oval window (secondary to a all or barotrauma—
including sneezing). It typically resolves on its own. Although
CASE 20.7 a perilymphatic stula o en results in vertigo, it is a relatively
A 65-year-old male presents with dizziness that started a ew uncommon cause o vertigo.
days ago. He reports that he is otherwise healthy and takes no
medications. Question 20.7.3 Which o the ollowing is the best next step
in the diagnosis o this patient?
Question 20.7.1 Which o the ollowing is the best question A) Dix–Hallpike maneuvers
to ask him in order to elicit a better characterization o his B) Obtain blood or CBC, electrolytes, BUN, and creatinine
dizziness complaint? C) MRI brain and brainstem
A) “Is the room spinning round and round?” D) C brain
B) “Do you eel that you are going to aint?” E) Audiometry
560 FAMIl y MEDICIn E EXAMIn ATIOn & BOAr D r EVIEW

Answer 20.7.3 The correct answer is “A.” A complete evalu- However, “C” and “E” deserve special mention. Stroke should
ation o the patient with vertigo includes examination o the be considered in any patient presenting with sudden onset, per-
neurological and cardiovascular systems as well as the ears, sistent vertigo, and risk actors (e.g., hypertension and atrial
eyes, nose, and throat. T e Dix–Hallpike maneuver is designed brillation). In general, there should be other ndings such
to di erentiate central rom peripheral vertigo, can be a use ul as diplopia, dysmetria, and ataxia. However, not every elderly
tool (more on this later . . .). In order to per orm the maneuver, patient with vertigo needs an MRI. I the history is consistent
rapidly move the patient rom a seated to a supine position, with BPPV, then neuroimaging is not necessary.
turning the head to the le or right, and observe or nystagmus.
T e patient is helped into a seated position, and the maneuver is
repeated, turning the head to the other side. I nystagmus occurs HELPFUL TIP:
within 30 seconds o per orming the maneuver, it is considered a A pe i mph eak p ese ts i the opposite ma e o
positive test. “B” is incorrect. Laboratory tests are rarely help ul. BPPV. It is best i the mo i a d wo se i the eve-
“C” and “D” are not correct because neuroimaging should only i . g avit is ot the patie t’s ie d. As mo e a d
be per ormed i the history and/or examination indicate the mo e pe i mph eaks out du i the da , the patie t
need or it. In this case, the examination is not even complete ets p o essive mo e ve ti i ous.
yet! Likewise, audiometry (although help ul in the diagnosis o
Ménière disease) is premature.
HELPFUL TIP:
Dix–Hallpike maneuver is positive with rotation o the head “Past poi ti ” o i e -to- ose testi usua i di-
to the le . T e rest o the physical examination is unremark- cates a pe iphe a esio . A d, patie ts e e a a
able. T e patient gets vertiginous and then it resolves in less towa d the side o pe iphe a d s u ctio .
than 1 minute.

Question 20.7.4 Given the history and examination, the Question 20.7.5 For this patient with BPPV, the LEAST
most likely diagnosis in this patient is: appropriate treatment at this time is:
A) Labyrinthitis A) Prednisone
B) Benign paroxysmal positional vertigo (BPPV) B) Lorazepam
C) Vertebrobasilar stroke C) Meclizine (Antivert)
D) Motion sickness D) Rehabilitation exercises
E) Cerebellar stroke E) Dimenhydrinate (Dramamine)

Answer 20.7.4 The correct answer is “B.” T e history o epi- Answer 20.7.5 The correct answer is “A.” Steroids are not likely
sodic, positional vertigo and the positive Dix–Hallpike maneu- to improve symptoms in BPPV. Benzodiazepines, anticholiner-
ver make BPPV the most likely diagnosis. Note that the Dix– gics, and antihistamines have all been employed in treating the
Hallpike maneuver is not speci c or BPPV. T e Dix–Hallpike symptoms o BPPV, but should all be second-line therapies a er
maneuver is designed to dif erentiate central rom peripheral rehabilitation and repositioning exercises. Cawthorne rehabili-
vertigo. In this case, the history is probably the most important tation exercises, physical therapy, and physician-directed head
aspect in making the diagnosis. BPPV is more common in older positioning (AKA Epley maneuvers to move stones out o the
patients but can occur at any age. Women are more commonly posterior semicircular canal) may be success ul. Most patients
a ected than men. BPPV is caused by calcium stone deposition will improve with time, although many will have relapses.
in the posterior semicircular canal. BPPV tends to be worse
in the morning and better toward evening because, being o
peripheral origin, the vertigo response atigues as the day goes HELPFUL TIP:
on (see able 20-4). T e other diagnoses listed are less likely. Simp t eati the patie t with medicatio s to sup-
p ess the ve ti o ca actua p o o the cou se o the
disease. The b ai eeds to ea to adapt to the si a
causi ve ti o. I ou b ai we e ta ki (a sca p os-
TABLE 20-4 DIX–HALLPIKE MANEUVER pect), it wou d sa , “OK, I am ot a i . I ca i o e that
Suggestive o Peripheral Vertigo i put.” Supp essi the ve ti o de a s this adaptatio .
• De a ed o set o sta mus a d s mptoms Howeve , medicatio s ma be ecessa o s mptom
• n sta mus a wa s i same di ectio co t o i some patie ts.
• Ve ti o ef ex ati ab e a te mu tip e ma euve s.
• n sta mus supp essib e b patie t
Suggestive o Central Vertigo Objectives: Did you learn to . . .
• n sta mus i mu tip e di ectio s • Desc ibe t pes o dizzi ess?
• n o ate c to o set o sta mus
• r ef ex ot ati ab e
• g e e ate a di e e tia dia osis o ve ti o?
• Dia ose a d t eat a patie t with BPPV?
CHAPTER 20 • OTOl Ar yn g Ol Og y 561

CASE 20.8 With silver nitrate cautery and direct pressure, you were able
to stop the bleeding. You now turn your attention to his le
A 15-year-old male wrestler presents to the ED with a nose- ear. You nd a purplish, tender, uctuant swelling at the le
bleed and a swollen ear. He clearly did not win this round. pinna.
Prior to coming to the ED, he held pressure to his nose or
30 minutes. He has had nosebleeds be ore, but none were Question 20.8.3 The best treatment or this condition is:
this bad. When asked how much blood he lost, his ather A) Analgesics, protection, and observation
shrugs and says it was “all over the mat.” On examina- B) Compressive dressing, using a headband (pre erably that
tion, you see blood oozing slowly rom the anterior nasal sweatband in your gym bag)
septum. C) Oral antibiotics
D) Incision and drainage, leaving the wound open to drain and
Question 20.8.1 Which o the ollowing is a source o ante heal by secondary intention
rior epistaxis? E) Needle drainage ollowed by compressive dressing sutured
A) Ethmoid artery into the pinna
B) Sphenopalatine artery
C) Kiesselbach arterial plexus Answer 20.8.3 The correct answer is “E.” With traumatic
D) Palpatine artery auricular hematoma/seroma, uid collects between the peri-
chondrium and underlying cartilage, predisposing the carti-
Answer 20.8.1 The correct answer is “C.” Kiesselbach plexus lage to loss o vascular supply and necrosis. In order to avoid
is a collection system involving several terminal arteries (ante- “cauli ower ear” de ormity, the hematoma or seroma must be
rior ethmoidal, greater palatine, and superior labial arteries) evacuated via incision or needle drainage. However, you cannot
in the anterior nose. It is the most common site o nasal bleed- stop there: a compressive dressing must then be sutured into
ing. As one o your editors learned to his embarrassment, the the pinna, or the uid is likely to re-accumulate. “B” is incor-
Kiesselbach plexus lies very anterior and can be missed when rect because compressive dressing alone is insuf cient. “D” is
using a speculum to inspect the nares. Don gloves, retract the incorrect only because it does not include compressive dress-
patient’s nasal tip, shine a light, and you will see it. T e eth- ing. “A” and “C” are incorrect because taking these actions will
moid and sphenopalatine arteries supply the posterior area delay de nitive treatment—and antibiotics are not needed any-
o the nose, and bleeding rom these sites is more dif cult to way. I the patient were to present late (10 days or more a er the
control. O special note is “D.” Emperor Palpatine, also known injury), he will not bene t rom incision or needle drainage, and
as “Darth Sidious,” converted Anakin Skywalker to the Dark instead should be re erred or otoplasty.
Side using hatred, anger and lies—not epistaxis caused by an
eponymous artery. Also, don’t con use the Kiesslebach plexus HELPFUL TIP:
with the Kessel Run, which the Millennium Falcon allegedly r epeat a te me, “The ose is ot the e ease va ve o the
completed in under 12 parsecs. ca diovascu a s stem.” Most asa b eedi is ve ous
a d ot at a e ated to e evated b ood p essu e.
Question 20.8.2 What is the best next step in managing this
patient’s nosebleed? Objectives: Did you learn to . . .
A) Continue to hold pressure against the septum • Eva uate a d t eat a patie t with epistaxis?
B) Pack the anterior naris with gauze • T eat a patie t with au icu a t auma?
C) Spray the mucosal sur ace with phenylephrine
D) Chemical cautery with silver nitrate
E) Any o the above can be used QUICK QUIZ: MOUTH SOr ES

Answer 20.8.2 The correct answer is “E.” Any o the options A 20-year-old emale presents with three pain ul ulcerations on
listed could be used alone or in combination to try to stop the her inner lip and tongue. She has no other symptoms. Although
bleeding. I these methods are not success ul, consider electro- she has never had such sores be ore, everyone else in her amily
cautery, laboratory evaluation or signs o bleeding disorders has had similar mouth sores. She only smokes when drinking
(e.g., CBC, P /P ), and otolaryngology consultation. Labora- cheap wine coolers, which occurs about once per week. She is
tory studies will rarely be help ul, though. a ebrile, and her examination is otherwise unremarkable.

T e most likely diagnosis is:


HELPFUL TIP:
A) Aphthous ulcers
I ou pack a patie t’s ose, p a o eavi the packi
B) Behcet disease
i p ace o 24 hou s o o e . A so, p esc ibe a tibiot-
C) Crohn disease
ics to p eve t si usitis.
D) Gluten enteropathy
E) Squamous cell carcinoma
562 FAMIl y MEDICIn E EXAMIn ATIOn & BOAr D r EVIEW

The correct answer is “A.” T is otherwise healthy young wom-


HELPFUL TIP:
an with a amily history o “similar sores” most likely has aph-
O ote, to ue i vo veme t is ot u usua i Be pa s .
thous ulcers or “canker sores.” T e etiology o aphthous ulcers is
The seve th e ve is i vo ved i taste se satio o the
not well understood, and they are alternatively explained as vi-
a te io two-thi ds o the to ue. Patie ts ma ote a
ral, autoimmune, genetic, traumatic, or due to chemical irritants
headache i the et oau icu a a ea be o e the o set o
or other processes (also, karmic retribution, divine punishment
s mptoms.
or sin, etc.). “B,” Behcet disease, is uncommon in the United
States. It is thought to be an autoimmune disease, and it pres-
ents with recurrent oral and genital ulcerations and skin and
You tell the patient that you suspect he has Bell palsy. He asks
eye lesions among other ndings. “C” and “D” can both present
what causes this problem.
with isolated oral ulcers but are less common than idiopathic
aphthous ulcers: there should also be other GI symptoms. “E”
Question 20.9.2 Which o the ollowing is the most likely
is incorrect because it is highly unlikely that this young, healthy
cause o this patient’s Bell palsy?
person has developed cancer . . . in three places . . . simultaneous-
A) Herpes virus
ly. Finally, lupus pemphigoid, pemphigus, and multiple in ec-
B) ick-borne illness
tions (e.g., Hand-Foot-Mouth, HIV, herpes, and syphilis) may
C) Diabetes
present with oral ulcers.
D) Adenovirus
E) T e dark side o the orce
CASE 20.9
A 34-year-old male dentist presents to your o ce with a Answer 20.9.2 The correct answer is “A.” It seems that many
1-week history o right acial weakness and a bit o tongue cases o Bell palsy are due to reactivation o herpes simplex
numbness. He states that he “just woke up this way one virus (HSV). Other viral etiologies have been implicated as well,
morning.” He would have come in sooner, but he was busy including EBV, CMV, coxsackievirus, and adenovirus (“D”),
with his practice and he has elt ne. He has not noticed any but all are much less commonly isolated compared with HSV.
other neurological symptoms. He denies pain, ever, or upper While “B,” tick-borne illnesses such as Lyme disease cause Bell
respiratory symptoms. He reports being healthy and taking palsy, these represent a minority o cases. “C,” diabetes, may put
no medications. On examination, his vital signs are normal. a patient at increased risk o contracting Bell palsy, but it does
You note that his right eyebrow sags, as does the right cor- not cause the disease.
ner o his mouth. He cannot close the right eye completely
or raise his right eyebrow, and the right nasolabial old is less HELPFUL TIP:
prominent than the le . T e remainder o the neurological Be pa s is mo e ike to occu i p e a c , especia
examination is normal. the ast t imeste a d the i st week postpa tum.

Question 20.9.1 The neurological nding in this patient Question 20.9.3 Which o the ollowing treatments is most
that most suggests a cranial nerve process (as opposed to a likely to bene t this patient?
central brain lesion) is: A) Acyclovir
A) Normal strength in the upper extremities B) Prednisone
B) Inability to smile on the right C) Arti cial tears and eye patching at night
C) Inability to wrinkle the orehead D) A and B
D) Normal blood pressure E) None o the above
E) Normal speech rate and rhythm
Answer 20.9.3 The correct answer is “C.” In a patient with Bell
Answer 20.9.1 The correct answer is “C.” Partial sparing o palsy and weakness to eye closure, good eye care, and protec-
the orehead muscles suggests a brain lesion because innerva- tion rom trauma must be employed to prevent corneal dam-
tion o the orehead contains crossed bers rom both sides age (remember that this patient cannot close his right eye). T e
o the brain; a bilateral brain lesion is possible but unlikely. evidence or antiviral therapy is negative; acyclovir likely
T us, a dense paralysis is more likely to be peripheral (CN7) doesn’t work, and it is no longer recommended. T ere may be
since all innervation is knocked out when this is involved. T is some slight bene t to corticosteroids. Steroids are most likely to
patient’s entire right ace, including the orehead, is paralyzed, provide a bene t i started within the rst 3 days a er the onset
which suggests a lower motor neuron (CN7) process. While o symptoms. Our patient is now a ull week out, so steroids are
“A” and “E” are ound in a normal neurologic examination and unlikely to produce a bene t.
are reassuring, they are not as help ul in isolating the loca-
tion o the lesion to a lower motor neuron source. “B,” lower You recommend eye care but not other medications. Know-
acial muscle weakness or paralysis, can occur with upper or ing that patients with Bell palsy generally require a lot o
lower motor neuron disease. “D,” normal blood pressure, is reassurance, you discuss the diagnosis in detail, including
not help ul. prognosis.
CHAPTER 20 • OTOl Ar yn g Ol Og y 563

Question 20.9.4 The patient can expect which o the ollow tried, although poorly studied. “A,” increased use o the joint by
ing? chewing gum, is the exact opposite o what the patient should
A) Complete resolution ( 100% likelihood) with nearly zero be doing. Jaw rest is important. All o the other interventions are
risk o recurrence reasonable. Also, you might recommend a so er diet, hot packs,
B) Likely resolution (> 50% likelihood) with nearly zero risk o and MJ massage.
recurrence
C) Likely resolution (> 50% likelihood) with about 10% risk o HELPFUL TIP:
recurrence The headache o TMJ ma be e t i the si us e io ,
D) High probability ( 95% likelihood) o persistent paralysis et oo bita o as a ea ache. Check o TMJ whe pa-
tie ts p ese t with these t pes o s mptoms.
Answer 20.9.4 The correct answer is “C.” Most patients will
recover (71% complete, 13% minor impairment with the rest
having signi cant sequelae), but it may take months. I a patient CASE 20.10
with suspected Bell palsy has no improvement in symptoms
a er a ew months, reconsider the diagnosis. Patients with A 22-year-old emale presents to your o ce complaining o
complete paralysis are more likely to have persistent symp- severe acial pain or the past 3 days. She has poor dentition
toms, whereas those with partial paralysis usually recover more and you nd that on examination, there is mandibular swell-
quickly and completely. Other treatments such as surgical nerve ing and tenderness.
decompression and nerve stimulation have little or no evidence
supporting them. Question 20.10.1 All o the ollowing are causes o mandib
ular area swelling EXCEPT:
HELPFUL TIP: A) Submandibular duct stone
r amsa –Hu t s d ome is the ame ive to zoste B) Dental abscess
oticus comp icated b hemi acia pa a sis. I ide ti ied C) Retropharyngeal abscess
ea , a tivi a s ma he p the patie t with r amsa –Hu t D) Ludwig angina
s d ome. you ma see b iste s i the ea ca a . E) Branchial cle cyst

Answer 20.10.1 The correct answer is “C.” Retropharyngeal


Objectives: Did you learn to . . .
abscesses are generally not visible but present with ever, throat
• Di e e tiate Be pa s om a ce t a esio ?
pain, and symptoms due to swelling o the retropharyngeal
• Desc ibe causes o Be pa s ?
space (dysphagia, drooling, odynophagia, and airway obstruc-
• Ma a e a patie t with hemi acia pa esis? tion). All o the other options can cause swelling around the
mandible.
QUICK QUIZ: OH, My ACHIn g JAW
Question 20.10.2 One o the diagnoses you are consider
ing is parotitis. Which o the ollowing is true regarding
A 30-year-old emale presents with several months o pain and
parotitis?
sti ness in her jaw. When asked to localize the pain, she points
A) You will only see mumps in science-denying, anti-vaccine,
to the temporomandibular joints ( MJ) bilaterally. She notes
whack jobs
that the pain is worse with stress ul situations, driving, and
B) Mumps can be associated with pain and in ammation o the
chewing. Her husband complains that she grinds her teeth at
meninges, pancreas, and gonads as well as the parotid glands
night. She is otherwise healthy. On physical examination, you
C) Bacterial parotitis is mostly caused by pan-sensitive strepto-
nd palpable popping in the MJs bilaterally, and the remain-
coccus and can be treated with penicillin
der o her examination is unremarkable.
D) Mumps is usually treated with IVIG
You make all o the ollowing recommendations EXCEP :
A) Start chewing gum daily Answer 20.10.2 The correct answer is “B.” Mumps can lead to
B) Use ibupro en as needed in ammation o other tissues including the meninges, pancreas,
C) Use a bite block at night and gonads (possibly leading to decreased ertility in males, not
D) Learn relaxation techniques in emales), and will occasionally require hospitalization. T ank-
ully, most people recover spontaneously with good supportive
The correct answer is “A.” T is patient’s history is consistent care, including NSAIDs and warm compresses. IVIG would
with MJ syndrome. In addition to those described in the case, be reserved or those with severe complications as mentioned
symptoms o MJ may include ear pain, headache, “sinus” above. “A” is incorrect. While the MMR vaccine is about 88%
headache, limited jaw mobility, and crepitus and tenderness on e ective a er 2 doses, this still leaves over 10% o the popula-
palpation o the joint. MJ syndrome may occur unilaterally tion susceptible to outbreaks. T ere ore, you may see mumps in
or bilaterally. No single therapy appears to have greater ef ca- immunosuppressed patients as well as anyone who is more than
cy than any other, and many di erent interventions have been 10 years out rom their last vaccination. Bacterial parotitis (“C”)
564 FAMIl y MEDICIn E EXAMIn ATIOn & BOAr D r EVIEW

is usually caused by Staphylococcus aureus and anaerobes. In the Question 20.10.5 The antibiotic o choice or this patient is:
hospital setting, approximately 40% o the Staphylococcus aureus A) Erythromycin
is methicillin resistant, so rst-line therapy is usually IV van- B) Clindamycin
comycin pending culture results. Parotitis secondary to stones C) Azithromycin
is o en treated with cephalexin, dicloxacillin, or amoxicillin/ D) Levo oxacin
sulbactam. Use clinical judgment and assure good ollow-up. E) rimethoprim/sul amethoxazole

Question 20.10.3 Which o the ollowing is NOT typical o Answer 20.10.5 The correct answer is “B.” Clindamycin is the
sialolithiasis (salivary duct stones)? drug o choice as an alternative to penicillin. T e drug used or
A) Intermittent swelling o the salivary gland a dental abscess should cover anaerobes. T ere is resistance
B) More than 80% o stones involve the parotid gland to erythromycin and azithromycin. Neither levo oxacin nor
C) T e majority resolve with conservative, nonoperative treat- trimethoprim/sul amethoxazole covers anaerobes.
ment
D) Anti-staphylococcal antibiotics should be used in their During your examination, you also noticed a rm nodule in
treatment the submental area in the midline. (It is de nitely not this
E) Sialogogues (e.g., lemon drops) should be used as part o the patient’s day!) It moves up and down when she swallows.
treatment She says that it has been there or years—as long as she can
remember.
Answer 20.10.3 The correct answer is “B.” In actuality, more
than 80% o cases o sialolithiasis involve the submandibu- Question 20.10.6 The most likely cause o this midline nod
lar gland. “A” is true. Swelling tends to occur when patients eat ule is:
and tends to resolve between meals as saliva slowly makes its A) Submandibular gland in ection
way through the duct. “C” is true. Most salivary duct stones pass B) T yroglossal duct cyst
spontaneously. “D” is true. T ink o this as an abscess. I there is C) In ected renulum
any sign o in ection, anti-staphylococcal antibiotics should be D) Branchial cle cyst
used until the stone passes. Finally, sialogogues such as lemon E) An accumulation o midi-chlorians
drops promote saliva ormation. T e e ectiveness is question-
able, but it is worth a try and it gives the patient something to do. Answer 20.10.6 The correct answer is “B.” T is is likely a thy-
roglossal duct cyst. T e rest are not likely. Submandibular glands
Question 20.10.4 Which o the ollowing is considered (“A”) and branchial cle cysts (“D”) are lateral to the midline.
good practice with regard to salivary duct stones? “C” is incorrect. T e renulum is under the tongue. As or “E,”
A) Patients should be ollowed up within 72 hours i the stone another Star Wars re erence. Maybe there will be a Wookie at
does not pass the end o the chapter. Keep reading.
B) Stones can be removed using sialolithotomy (using a probe
and/or scalpel to nick the outlet) You start the patient on clindamycin, but she returns in the
C) Patients should be re erred or lithotripsy i stones do not morning with severe submandibular swelling. T e tongue
pass within 4 days is elevated in the mouth and there is brawny edema; she is
D) Surgical excision o the duct should be done i the stone does having trouble talking. You make the diagnosis o Ludwig
not pass angina. (“Didn’t we just do this?” you ask. “Yes, we are going
E) ricyclic antidepressant prophylaxis i stones occur re- to do it again,” we reply.)
quently
Question 20.10.7 At this time, the most appropriate treat
Answer 20.10.4 The correct answer is “B.” A probe can be ment or this patient is:
used to “dilate” the duct. Occasionally, the outlet needs to be A) Continue PO clindamycin and add saltwater gargles
nicked using a scalpel blade to allow the stone to egress. T e rest B) Continue PO clindamycin and add PO metronidazole
are incorrect. With regard to “A,” patients should be ollowed C) Incise and drain the swollen area under the tongue
up in 24 hours i the stone does not pass. “C” is also incorrect. D) Re er immediately or surgical evaluation or possible tra-
Lithotripsy has actually been used with some good results but cheostomy
is de nitely third line. Finally, “D” is incorrect. Excising the E) Administer IV uids and ce riaxone in the of ce and ollow
duct seems a little dramatic. “E” is very wrong. Anticholinergics up tomorrow
reduce salivary ow and thus contribute to stone ormation.
Answer 20.10.7 The correct answer is “D.” T is patient’s situ-
You check the patient’s mouth and decide that there is likely ation has become airly desperate overnight even while on an
a dental abscess, not a salivary duct stone. She needs to have appropriate antibiotic. Securing an airway should be your rst
her tooth extracted, but the dentist will not be able to see concern. Un ortunately, these patients do not respond to local
the patient until the morning. She requires antibiotics but is incision and drainage, as the swelling is usually di use, quickly
penicillin-allergic. spreading, and does not result in discrete pus pocket ormation
CHAPTER 20 • OTOl Ar yn g Ol Og y 565

until late in the course. She should be admitted or IV uids, Strategy 2: esting
appropriate IV antibiotics (not levo oxacin), and possible est all patients and treat those with a positive strep screen. Do
tracheostomy. Patients with advancing Ludwig angina do not not culture others.
tolerate endotracheal intubation; thus, an alternative airway is Strategy 3: For Children
o en provided with tracheostomy. T e majority o group A strep pharyngitis occurs in children
Objectives: Did you learn to . . . between ages 5 and 15 years. In this age group, 15% to 30% o
• Dia ose sa iva sto es? acute pharyngitis is caused by group A strep. In children, doing
• T eat sa iva sto es? a rapid strep test is considered the standard (although many
• r eco ize the p ese tatio o subma dibu a masses?
argue convincingly that it is not even necessary here in the older
child). Cultures are again optional depending on the reliability
o your rapid antigen test. Many would culture all rapid strep
CASE 20.11 test negative patients and this is certainly an acceptable strategy
A 21-year-old male presents or a sore throat. His symp- as well.
toms started 3 days ago. He has had subjective evers, sweats, So, now you are quite con used. So is everyone else . . .
atigue, and mild nausea. He has no cough or rhinorrhea. His
temperature is 38.3°C. His vital signs are normal otherwise. Question 20.11.2 Which o the ollowing is true about anti
He has symmetrically enlarged tonsils with exudates present biotic therapy o streptococcal pharyngitis?
and tender anterior cervical lymphadenopathy. A) Azithromycin is the drug o choice because o resistant
streptococci
Question 20.11.1 At this point, you: B) T ere is no signi cant resistance seen in Group A β -hemolytic
A) Reassure and recommend saltwater gargles streptococci, and penicillin is still the drug o choice
B) Obtain a routine aerobic culture o the oropharynx C) Cephalexin is the pre erred drug because it covers H. inf u-
C) Prescribe penicillin 500 mg BID or 10 days enzae, which is a requent coin ector with streptococci
D) Prescribe levo oxacin 500 mg daily or 7 days D) Amoxicillin is pre erred or strep throat because it does not
cause a rash i the patient happens to have mononucleosis
Answer 20.11.1 The correct answer is “C.” T is patient has 4
o 4 signs/symptoms suggestive o streptococcal (group A strep Answer 20.11.2 The correct answer is “B.” T ere is no sig-
or Streptococcus pyogenes) pharyngitis. T ese are the “Centor ni cant resistance among group A β -hemolytic streptococci to
criteria”: (1) ever, (2) tender cervical adenopathy, (3) exudative penicillin. T us, penicillin remains the drug o choice despite
pharyngitis, and (4) lack o other URI symptoms. In this case, drug detailing. T ere is no reason to use anything else, except
the most appropriate step would be empiric antimicrobial treat- in the case o allergy where erythromycin or cephalexin can be
ment. “B,” per orming a culture, will take a ew days and does used. Remember that there is now resistance to erythromycin,
not add much, given the strength o the clinical argument or azithromycin, and clarithromycin. “D” is important. Amoxicil-
strep throat. Furthermore, the oropharynx is colonized by many lin can cause an uncom ortable rash should your patient turn
kinds o ora that do not cause disease, and we only really care out to have mononucleiosis rather than strep throat.
about group A streptococcus, so a routine aerobic culture is o
no value. Alternatively, a rapid assay or a speci c “rule out” cul- HELPFUL TIP:
ture or group A strep could be done rather than treating based Pe ici i VKca be used BID i st eptococca pha i-
on clinical grounds (the rapid strep has a 5% alse negative rate). tis, a d this admi ist atio eque c i c eases comp i-
While saltwater gargles seem to help reduce the pain o pharyn- a ce.
gitis, “A” is incorrect because you would want to do more than
that or this patient who likely has strep throat. “D” is incorrect
because levo oxacin and other uoroquinolones are not indi- Question 20.11.3 Antibiotics should be started within what
cated or treatment o strep throat. time period to reduce the risk o rheumatic ever rom
Here are three strategies or the patient you think might have streptococcal pharyngitis?
strep throat: A) 2 days a er presentation
Strategy 1: No testing (or minimal testing) B) 2 to 4 days a er presentation
In this strategy, one treats based on clinical symptoms. You C) 4 to 6 days a er presentation
are looking or our things: ever, exudate, absence o other D) 6 to 8 days a er presentation
URI symptoms, and tender anterior cervical adenopathy. reat E) 8 to 10 days a er presentation
patients with 3 or 4 criteria, and do not treat others. Another
approach is to treat patients with 4 criteria, do a rapid strep test Answer 20.11.3 The correct answer is “E.” Antibiotics should
on those with 3 (and maybe 2) criteria, and avoid treatment and be started within 9 days a er presentation in order to prevent
testing o others. T e CDC has recommended this or nonim- rheumatic ever, which is really our goal when we treat strep-
munosuppressed patients in the absence o an outbreak o rheu- tococcal pharyngitis. T us, there really is no reason to hurry
matic ever in the community. treatment.
566 FAMIl y MEDICIn E EXAMIn ATIOn & BOAr D r EVIEW

broad-spectrum antibiotic; consider testing and/or treating co-


Your patient is a student teacher and wants to know how long habitants; have the patient replace his toothbrush.
to stay out o the classroom.
HELPFUL TIP:
Question 20.11.4 A patient with streptococcal pharyngitis Ma causes o th oat pai a e acute i ectio s (st ep
should be considered in ectious and kept out o school or th oat, othe bacte ia i ectio s, vi a pha itis, mo o-
what period a ter beginning antibiotics? uc eosis), but co side othe o i ectious causes as
A) 12 hours we —ca otid ia, vi a th oiditis, mouth-b eathi ,
B) 24 hours pe ito si a abscess. n o - oup A st eptococci (es-
C) 36 hours pecia St eptococcus a a actiae, a oup B st ep) a e
D) 48 hours i c easi bei cu tu ed om the pha x. St ep-
tococcus a a actiae ca cause i vasive disease a d
Answer 20.11.4 The correct answer is “B.” Patients should be wo th t eati i ou d. It wi espo d to cepha exi o
considered in ectious or 24 hours a er the initiation o thera- c i dam ci (amo othe s).
py or streptococcal pharyngitis. T e risk o transmission goes
down markedly a er this point. Un ortunately, the patient is
actually in ectious or the 3 to 5 days be ore they become symp- It’s a “two or uesday” at your clinic. T e next patient comes
tomatic, so removing the patient or 24 hours a er treatment is in or a sore throat and tender anterior and posterior cer-
closing the barn door a er the horses have le . O course, since vical adenopathy. He is ebrile and relatively stoic. He has
this is being written in Iowa, so we are closing the pen a er the been sick or 2 weeks with signi cant atigue and just isn’t
hogs have le . T ere is some preliminary data that it is OK to getting better. In addition to the adenopathy, you notice le -
return to work/school 12 hours a er starting treatment but this sided abdominal tenderness with minimal guarding but no
is not yet the standard o care (Pediatr In ect Dis J. 2015 Aug 20). rebound tenderness. You believe that you eel a spleen edge.
However, the patient’s heterophile antibody (monospot) is
You see the same patient 3 weeks later. He took all o his pen- negative. You decide to get anti-EBV (Epstein–Barr virus)
icillin even though he elt ne a ew days a er he le your antibodies. T e results o the anti-EBV antibody test are as
o ce. (Wow! A compliant patient!) However, he now has the ollows (VCA is against the capsid): IgM-VCA positive, IgG-
same symptoms, starting 2 days ago. His examination is the VCA negative, anti-EBV nuclear antigen antibody negative.
same.
Question 20.11.6 How do you interpret these results?
Question 20.11.5 Which o the ollowing is the most likely A) T e patient has acute EBV in ection
cause or his current symptoms? B) T e patient has had EBV at least 6 weeks ago
A) Gonococcal pharyngitis C) T e absence o anti-EBV nuclear antigen antibody makes
B) In ection with a resistant streptococcal organism acute in ection highly unlikely
C) Mononucleosis D) T e patient has never been in ected with EBV
D) Staphylococcal pharyngitis
E) None o the above Answer 20.11.6 The correct answer is “A.” T e patient has an
EBV in ection, starting in the last ew weeks. Here is why. IgM-
Answer 20.11.5 The correct answer is “E.” Since his symptoms VCA is produced acutely and is elevated in the acute in ection
resolved, you either got the diagnosis and treatment right or or 2 to 4 weeks. Since this patient’s IgM-VCA is positive, he has
he had some other sel -limited in ection. T ere ore, it would had an acute EBV in ection within the past month. IgG-VCA is
be unlikely that he had gonococcal pharyngitis or resistant measurable 3 to 4 weeks a er acute in ection and persists or li e.
streptococcal organisms. Nonetheless, sexual history is impor- T us, it gives no in ormation about when in ection occurred.
tant—even when con ronted with pharyngitis; i a patient with T e absence o IgG-VCA means either (1) the patient has no
exudative pharyngitis is not improving, think about gonococ- history o EBV in ection or (2) the patient has had a recent EBV
cal disease. Remember that gonococcal disease will be missed in ection. Antibodies against the EBV nuclear antigen show up
or two reasons in this scenario: the history is never obtained at 6 to 12 weeks a er in ection. I this antibody is present in the
regarding oral sex and gonococcus requires T ayer-Martin agar blood, it suggests that there has not been an acute in ection; the
to grow, so it will not show up on routine culture, and there are in ection had to have been at least 6 weeks ago.
a lot o alse negative cultures; consider PCR i thinking about
gonococcal pharyngitis. “B,” penicillin-resistant Streptococcus Question 20.11.7 O the ollowing, which DOES NOT cause
organisms causing pharyngitis, like the ooth Fairy, are non- a mononucleosis like syndrome?
existent. “C” is unlikely in this case because his symptoms re- A) Human immunode ciency virus (HIV)
solved, but mononucleosis can cause prolonged symptoms o B) Cytomegalovirus (CMV)
sore throat and atigue and can be con used with strep throat. C) oxoplasmosis
In this case, recurrent strep throat is most likely and he should D) West Nile virus (WNV)
be advised appropriately: retreat with penicillin, not a more E) Leptospirosis
CHAPTER 20 • OTOl Ar yn g Ol Og y 567

Answer 20.11.7 The correct answer is “D.” WNV is charac- D) Prednisone


terized by ever, headache, myalgias, back pain, and anorexia E) onsillectomy
lasting 3 to 6 days. Much less common mani estations are nau-
sea, vomiting, diarrhea, encephalitis, etc. T us, WNV does not Answer 20.11.9 The correct answer is “A.” You may want to give
cause a mononucleosis-like syndrome because it does not last as steroids (“D”), but you will nd little data to back you up. A Co-
long and rarely includes pharyngitis. One thing to note is that chrane Collaboration Review updated in 2011 ound insuf cient
WNV may include lymphadenopathy. All the other answers can evidence or steroids or symptom control in patients with mono.
cause a mononucleosis-like syndrome. Other causes o mono- However, many physicians still prescribe steroids or patients with
nucleosis-like syndromes include adenovirus, parvovirus B19 signi cant symptoms. Antibiotics are not indicated or mononu-
(erythema in ectiosum), herpes virus 6 (roseola in antum), and cleosis. onsillectomy is also not indicated. T e patient likely has
ehrlichiosis (Asian orm only). Remember these diagnoses in paratracheal node swelling, as well. In this patient, hospital admis-
heterophile negative mono-like illness. sion may be indicated i there is concern or airway obstruction.

Question 20.11.10 What is the approximate sensitivity o


HELPFUL TIP: the heterophile antibody test (“monospot”) or mononu
Depe di o what popu atio is studied, 1% to 2% o cleosis within the rst 2 weeks o symptoms?
patie ts with a mo o uc eosis- ike s d ome who a e A) 10%
heterophile negative a e HIV positive. B) 30%
C) 50%
D) 70%
Question 20.11.8 There is some concern o splenic rupture E) > 90%
in patients with mono who have splenomegaly. I spleno
megaly were con rmed in this patient, what would be the Answer 20.11.10 The correct answer is “D.” T e sensitivity o
generally accepted recommendation with regard to ath the monospot ranges rom 60% to 80% 2 weeks into the illness.
letic participation (e.g., cage ghting)? T e point here is not the number per se but the act that there
A) No participation until negative acute titers or EBV are heterophile-negative mononucleosis syndromes and not ev-
B) No participation or 2 week a er the diagnosis is made, eryone with EBV mononucleosis will have a positive monospot
assuming complete resolution o symptoms and then ull when tested. However, they should have atypical lymphocytes
participation on WBC di erential.
C) No participation until 3 weeks a er the diagnosis is made,
assuming complete resolution o symptoms and then only HELPFUL TIP:
noncontact training or another week Co side pe ito si a abscess i a patie t with a so e
D) Full practice and competition allowed immediately unless th oat. The e wi e e a be a mu ed, “hot potato,”
abdominal pain occurs voice, deviatio o the uvu a awa om the side o the
E) Full practice and competition immediately with body armor abscess, a d p ot usio o a to si towa d mid i e. This
(e.g., Kevlar vest) ma be a exte sio o a p io pha itis but ma a so
a ise de ovo. T eatme t is a tibiotics a d d ai a e
Answer 20.11.8 The correct answer is “C.” It is generally ( eed i it is OK— o eed o su ica i vo veme t i
thought that return to practice or noncontact training is sa e a cases). It is a so impo ta t to pa atte tio to the ai -
3 weeks a er the diagnosis o mononucleosis, provided that wa si ce the e is the possibi it o obst uctio .
all other symptoms have also resolved. I there are no clinical
concerns or splenic enlargement at 4 weeks, then the athlete
may be cleared to return to ull competition. T is recommen- HELPFUL TIP:
dation is based on the observation that most cases o splenic The mo ospot is ot as se sitive i chi d e . It wi be-
rupture in athletes have occurred when those athletes returned come positive i ess tha 40% o chi d e ou e tha
to competition in less than 4 weeks rom the time o diagnosis. 5 ea s whe the a e i ected with EBV. Howeve , a ti-
EBV a tibodies wi be positive.
T e patient returns 2 days later and is noting increased pha-
ryngeal swelling and di culty swallowing. You look into his HELPFUL TIP:
throat and note “kissing tonsils.” T ere is no stridor, but he Arcanobacterium haemolyticum is a bacte ium that
eels as though there is something in his throat. causes pha itis especia i ou adu ts (tee s,
ea 20s). It c i ica ooks ike st ep th oat but o te
Question 20.11.9 What is the best treatment or this patient has a associated ash especia o the a ms (50%
at this time? o ). It does ot cause o -te m seque ae, so testi
A) Observation o it is ot ecessa i i dicated (but it is coo to ide ti
B) Amoxicillin whe the patie t comes i with the app op iate ash!).
C) Clindamycin
568 FAMIl y MEDICIn E EXAMIn ATIOn & BOAr D r EVIEW

I ou choose to t eat (which some p actitio e s do), it Do ot su est ea tubes o patie ts with u i ate a se ous
is e e a se sitive to pe ici i s, cepha ospo i s, a d otitis. The mai u ctio o ea tubes i this situatio is to
tet ac c i es. assist with a ua e deve opme t. This is ot a issue with
u i ate a se ous otitis.
I ectio s o the ca ti a i ous ea pi ae om pie ci sa e
Objectives: Did you learn to . . .
o te om Pseudomonas. This is a eme e c .
• Desc ibe di e e t st ate ies to app oachi the patie t with
s mptoms o streptococcal pha itis? Most asa t auma eed ot be add essed at a i st visit:
• T eat a patie t with streptococcal pha itis a d ecu e t ou ca et a bette cosmetic esu t o ce the swe i has
pha itis? eso ved. Howeve , a septa hematoma is a eme e c
that equi es p ompt atte tio to p eve t septa ca ti a e
• Deve op a b oad di e e tia o so e th oat?
dest uctio .
• Dia ose a d t eat mo o uc eosis?
Pe ici i is sti the d u o choice o St ep th oat. The e is o
esista ce a d i the patie t has mo o ou have ’t ated them
QUICK QUIZ: In THE SAME VEIn . . . to 6 weeks o a ash, which mi ht occu with amoxici i .
Use a wick whe t eati otitis exte a with d ops. This wi
Uh, oh. You have treated a 19-year-old college student or pre- he p assu e that the medicatio ets to whe e it is eeded.
sumed streptococcal pharyngitis with azithromycin (didn’t we
say not to do this?!). In your de ense, he wanted to get better BIBLIOGRAPHY
ast because he landed the lead role in the new musical You
Had Me at Arrrgh: A Wookie Love Story. He returns 1 week later American Academy o Pediatrics Subcommittee on Manage-
with high ever and a cough, looking quite ill. Chest radiograph ment o Acute Otitis Media. Diagnosis and management
o acute otitis media. Pediatrics. 2013;131(3):E964–E969.
shows small pulmonary abscesses and he is tender over a swol-
len carotid sheath. His throat looks aw ul . . . . . Aring AM, Chan MM. Acute rhinosinusitis in adults. Am Fam
Physician. 2011;83(9):1057–1063.
T e most likely diagnosis is: Buescher JJ. emporomandibular joint disorders. Am Fam
A) Lemierre syndrome Physician. 2007;76(10):1477–1482.
B) Complication o EBV mononucleosis Centor RM, Samlowski R. Avoiding sore throat morbidity and
C) Cardiac valvular disease rom rheumatic ever mortality: When is it not “just a sore throat?”. Am Fam
D) Aspiration pneumonitis rom his pharyngitis Physician. 2011;83(1):26, 28.
E) Scarlet ever Chow AW, et al. Executive Summary: IDSA Clinical Practice
Guideline or Acute Bacterial Rhinosinusitis in Children
The correct answer is “A.” T is is a classic case o Lemierre and Adults. Clin In ect Dis. 2012;54(8):1041–1045.
syndrome, or septic thrombophlebitis o the jugular vein rom Ely JE, et al. Diagnosis o ear pain. Am Fam Physician. 2008;
(generally) Fusobacterium necrophorum. T is is an anaerobic 77(5):621.
in ection o the posterior pharynx that may be misdiagnosed Feierabend RH, Shahram MN. Hoarseness in adults. Am Fam
as streptococcal disease. Generally, patients look ill with a Physician. 2009;80(4):363–370.
temperature o > 39°C, have tenderness in the neck, and have Lockhart P, et al. Antiviral treatment or Bell’s palsy (idiopathic
septic emboli in the lungs. T e basic problem is a septic throm- acial paralysis). Cochrane Database Syst Rev. 2009;4:
bophlebitis o the jugular vein. It is being increasingly recog- CD001869.
nized especially in young adults (teens–early 20s). T is patient Lodi G, et al. Interventions or treating oral leukoplakia. Co-
must be admitted or IV antibiotics and may require anticoagu- chrane Database Syst Rev. 2006;4:CD001829.
lation and/or surgical intervention. In the pharyngitis stage, it is McAllister K, et al. Surgical interventions or the early man-
sensitive to penicillin. See, i you would have listened to us and agement o Bell’s palsy. Cochrane Database Syst Rev.
used penicillin in the rst place you wouldn’t be in this bind! 2011;2:CD007468.
Paradise JL, et al. Developmental outcomes a er early or
delayed insertion o tympanostomy tubes. N Engl J Med.
Clinical Pearls 2005;353:576–586.
Arcanobacterium a d Fusobacterium pha itis ca ook Parnes LS, et al. Diagnosis and management o benign parox-
just ike St ep th oat a thou h the apid St ep wi be ysmal positional vertigo (BPPV). CMAJ. 2003;169(7):681.
e ative. Arcanobacterium is o te associated with a ash. Rosen eld RM, et al. Clinical practice guideline: Acute otitis
Fusobacterium ca ead to a septic th omboph ebitis o the externa. Otolaryngol Head Neck Surg. 2006;134(4, Suppl):
ju u a vei (l emie e s d ome). S4–S23.
Co side mumps i a patie t with sa iva a d swe i Salinas RA, et al. Corticosteroids or Bell’s palsy (idiopathic
eve i patie ts who have had both MMr vacci atio s; acial paralysis). Cochrane Database Syst Rev. 2010;3:
immu it wa es with time a d the e a e ea outb eaks i CD001942.
the U ited States. Schwetschenau E, Kelley DJ. T e adult neck mass. Am Fam
Physician. 2002;66(5):831.
Care of the Older Patient
Nicholas R. Butler and Jason K. Wilbur
21
bourbon as alcohol inter eres with normal sleep architecture
CASE 21.1 (single malt scotch?? . . . maybe). I these initial e orts ail, a
An 83-year-old emale patient whom you have ollowed low ose o a hypnotic (e.g., zolpi em an zaleplon) or trazo-
or many years has just been admitted to a nursing home one is an appropriate choice. razo one has ewer anticho-
ollowing a short hospitalization. Because o a steady linergic an bloo pressure e ects than other options, such as
decline in unction and lack o amily and social support, tricyclics an benzo iazepines, but shoul still be use with
you and the patient have come to the realization that she caution while monitoring or a verse e ects. Diphenhy r-
can no longer sa ely live alone in her home, and her needs amine (“A”) has power ul antihistaminic an anticholinergic
were too great or assisted living. Her medical problems properties that may result in increase con usion an alls.
include: congestive heart ailure, chronic atrial ibrilla- Diazepam (“B”) has an exceptionally long hal -li e in el erly
tion, osteoarthritis, and depression. Her current medica- patients an may cause aytime somnolence. Amitriptyline
tions are: war arin, urosemide, acetaminophen, calcium (“C”) is liste by the Centers or Me icare an Me icai Ser-
carbonate, lisinopril, metoprolol, and luoxetine. Both vices (CMS) as a me ication to be avoi e in nursing home
the patient and the nursing sta report poor sleep and patients ue to high potential or severe a verse rug reac-
depressed mood or the last 2 weeks, and the nurses are tions ( alls, constipation, etc.).
asking or a sleep aid.

Question 21.1.1 What is the best next step in the manage - HELPFUL TIP:
ment o her insomnia? Melatonin receptor agonists (e.g., ramelteon) are an-
A) A iphenhy ramine 50 mg PO HS other option or insomnia but are not great. Patients
B) A iazepam 5 mg PO HS sleep about 7 minutes more per night compared to pla-
C) A amitriptyline 25 mg PO HS cebo (let’s talk about statistically signi icant vs. clinically
D) Recommen increase activity uring the ay, avoi ance o signi icant now!). They are relatively sa e and do not
naps, warm milk be ore be , an waking at the same time lead to a “hangover” as with many hypnotics. The use
each morning o ramelteon has also been shown to be bene icial in
E) A 2 shots o bourbon PO HS preventing the onset o delirium in acutely hospitalized
elderly patients. What about melatonin, you say? Poor
Answer 21.1.1 The correct answer is “D.” T ere are no evidence but low chance o harm, so some physicians
great me icines or promoting sleep in el erly nursing home will try it irst. And then there are orexin receptor an-
resi ents. Given the lack o e cacy ata o most hypnotics tagonists, such as suvorexant (Belsomra). But they look
couple with the known a verse e ects, a trial o goo sleep pretty scary (suicidal ideation, “sleep driving,” need to
hygiene shoul be un ertaken rst. “Goo sleep hygiene” gen- warn about next-day driving and other dangerous ac-
erally consists o the ollowing: eliminate or re uce aytime tivities); there have been cases o daytime cataplexy.
naps, a aily activities, maintain a set waking time, increase Suvorexant has a 12-hour half-life in the nonelderly lead-
aerobic exercise (but not within a ew hours o be time), an ing to signi icant dys unction the day a ter taking the
maintain a quiet, com ortable sleeping environment. Night- drug. Never prescribe them or elderly patients. In act,
time rituals, such as me itation an warm milk, may help orget we mentioned them.
insomnia an are unlikely to cause any harm, but avoi the

569
570 FAMILY MEDICINE EXAMINATION & BOARD REVIEW

medications are now war arin, urosemide, acetaminophen,


HELPFUL TIP:
calcium carbonate, and lisinopril.
There are many potential causes o sleep disturbance
in the nursing home. The ollowing are sources o sleep
Question 21.1.3 In this malnourished, elderly nursing home
problems that you may want to investigate or treat
patient, which o the ollowing interventions or diagnostic
empirically: pain, anxiety, depression, delirium, demen-
studies will most likely lead to improvement in her condi-
tia, primary sleep disorders (e.g., sleep apnea, restless
tion?
leg syndrome), environmental issues (e.g., alarms and
A) A mit to the hospital an initiate parenteral ee ing
lights).
B) Re er or esophagogastro uo enoscopy (EGD)
C) Screen or epression
D) A megestrol acetate
T e nurses grudgingly accept your recommendation to try E) A marijuana
sleep hygiene (“Can’t we just give her a pill?”), and over the
next month your patient’s sleep improves, as does her mood. Answer 21.1.3 The correct answer is “C.” Depression is one o
T ere’s one in the win column or the doctor! Un ortunately, the most common causes o weight loss in the nursing home,
she experiences two alls with minor injuries while ambulat- an stroke survivors are at high risk or epression. Also, this
ing in her room. Upon examination, you nd normal vital patient has a history o epression with no current treatment
signs, no orthostatic hypotension, and no ocal neurological (Remember? We stoppe the uoxetine earlier). Consi er using
de cits. a screening tool, such as the geriatric depression scale. A positive
screen requires urther investigation. Now that she is be ri -
Question 21.1.2 Which o the ollowing is the most appro- en, alls are not as much o an issue, so an SSRI may be appro-
priate next step? priate. “A” is incorrect. We are not sure o the patient’s wishes
A) Discontinue war arin regar ing intravenous (IV) nutrition, an more conservative
B) Employ be an chair alarms measures shoul be institute be ore consi ering parenteral
C) Obtain a computerize tomography (C ) scan o the hea nutrition. “B” is incorrect. While an EGD may be important at
D) Re uce or iscontinue uoxetine some point, procee ing to EGD imme iately is premature. “D,”
E) Restrict activities an prescribe a wheelchair megestrol acetate, has been reporte to improve oo intake in
patients with cancer cachexia, but its value in the nursing home
Answer 21.1.2 The correct answer is “D.” As of en happens is questionable, an it may be associate with an increase risk
in li e, there is no great answer here. Falls are usually multi ac- o mortality. “E” is also incorrect. Even i you practice in one o
torial in origin, so simple interventions o not generally solve the tokin’ states, we cannot have her lighting up in the nursing
the problem. SSRIs have been shown (in imper ect stu ies) to home.
increase the risk o alls in the el erly, so re ucing or iscon-
tinuing uoxetine is pru ent, but you will have to watch or
moo an anxiety symptoms. “A,” iscontinuing war arin, may HELPFUL TIP:
be appropriate i she continues to all but this action is not likely The initial evaluation o a nursing home patient with
to re uce her all risk. “B,” the use o be an chair alarms, is weight loss should ocus on medication review, gastro-
help ul when patients have cognitive impairment an can- intestinal (GI) symptoms, dental and mouth problems,
not remember to ask or help when getting up; however, these swallowing dys unction, ability to eed one-sel , and
evices can act as tethers to urther restrict a patient’s move- psychiatric disorders (e.g., depression, dementia, or
ment. Likewise, urther restriction o activities an man a- psychosis). Hyperthyroidism, hyperparathyroidism, ma-
tory wheelchair use may lea to econ itioning, loss o mus- lignancy, and chronic in ection should be considered as
cle strength, an an increase risk o alls. A C scan o the causes o weight loss (albeit less likely). Factors associ-
hea may be warrante i the patient sustaine a hea injury or ated with aging, such as decreased ol action, taste, and
ha an abnormal neurological examination, but it’s unlikely to salivation (and nursing home ood), may decrease the
etermine the cause o her alls. enjoyment o eating.

Over the next year, you observe a steady decline in your


patient’s unction, with a series o alls despite interventions.
Ultimately, your patient has a devastating thromboembolic You diagnose depression and initiate treatment. You also
stroke, which results in right hemiparesis and dysphagia. encourage the nursing sta to observe your patient while
A er a short hospital stay, she returns to the nursing home eating and assist her i necessary. You add a daily multivita-
and undergoes therapy. You order a swallow study (which min. Her mood improves slightly and her weight stabilizes.
you will do or all stroke patients). She continues to have di - However, over the next 6 months, your patient becomes more
culty with her swallowing but is able to tolerate thickened withdrawn and spends most o her time in bed. Because o
liquids without aspirating. She has a 5%weight loss over the her stroke, she is not very mobile, and she requires assistance
next 2 months, and her nurse reports poor oral intake. Her with trans ers and movement in bed. Ultimately, she develops
CHAPTER 21 • CARE OF THE OLDER PATIENT 571

TABLE 21-1 PRESSURE ULCER STAGES


Stage I Stage II Stage III Stage IV Unstageable
Nonblanchable Partial-thickness Full-thickness skin loss Full-thickness skin loss Ulcer covered in eschar
erythema skin loss and depth unknown

Intact skin Epidermis and/or Damage or necrosis o Extensive destruction, tissue


dermis subcutaneous tissues, necrosis, or damage to muscle,
extending to underlying ascia bone or supporting structures

Changes in skin Presents as Presents as deep crater


temperature, abrasion, blister, or
consistency, or shallow crater
sensation

a skin ulcer on her sacrum. Nursing sta reports a sacral has worsene an is now at least stage III. Without urther in or-
pressure ulcer measuring 3 × 2 cm. T ere appears to be some mation, oam ressing is a sa e choice or initial treatment o
interruption o the epidermis, like an abrasion. a stage III ulcer. Foam pa s are use ul or eeper woun s with
mo erate exu ate an may also protect the woun rom urther
Question 21.1.4 According to conventional staging criteria, pressure. In most cases, the woun shoul be kept moist, so wet-
what stage is this pressure ulcer? to- ry ressings are not appropriate (they also ebri e the woun
A) Stage I o both necrotic an healthy tissues—ouch). I moist gauze pack-
B) Stage II ing is use , it shoul be kept moist with intermittent reapplica-
C) Stage III tion o saline or change be ore rying. From the nurse’s report,
D) Stage IV there is no evi ence o necrosis, excessive rainage or in ection,
so ebri ement an antibiotics may not be help ul at this time.
Answer 21.1.4 The correct answer is “B.” Pressure ulcers (in A transparent, occlusive ressing is use or stage II ulcers, but is
ol er parlance, ecubitus ulcers, pressure sores, or be sores) insu cient or stage III or IV. In a ition to ressing the woun ,
are cause by unrelieve pressure resulting in amage to un er- you shoul employ the ollowing measures: repositioning every
lying tissue. Stage II ulcers are partial thickness injuries that are 1 to 2 hours, pressure-relieving mattress an cushions, an opti-
shallow an appear as open ulcers. Anatomic areas o concern in mizing nutrition. Consultation with a woun care specialist or a
be -boun patients inclu e sacrum, coccyx, heels, an occiput. surgeon may be necessary i these measures ail.
Chair-boun patients are more likely to evelop ulcers over
the ischial tuberosities. Risk actors or pressure ulcers inclu e HELPFUL TIP:
a vancing age, immobility, moisture (e.g., urinary or ecal There is minimal evidence o bene it or the use o most
incontinence), malnutrition, an ecrease sensory perception. adjunctive therapies or pressure ulcers (electrical stim-
Ulcers are stage by clinical appearance (see able 21-1). ulation, ultrasound, hyperbaric oxygen, ultraviolet light,
vasodilators, vacuum devices, etc.). In general, stick with
Because o miscommunication within the nursing home pressure relie , good wound care, and nutrition.
sta , the ulcer goes untended over the weekend. An alarmed
nurse calls you to report ull thickness skin loss. You arrange
HELPFUL TIP:
to visit the patient in the evening . . . here comes another
To prevent pressure ulcers, schedule regular and re-
“above and beyond” award.
quent repositioning or bed and chair-bound individu-
als. Turn at least every 2 to 4 hours on a pressure-reduc-
Question 21.1.5 In the meantime, you prescribe what
ing mattress or every 2 hours on a nonpressure-reducing
treatment?
mattress. Also, maintain the head o the bed ≤ 30 de-
A) Foam pa with occlu ing ressing (e.g., Allevyn, Mepilex)
grees i possible to reduce pressure on the sacral area.
B) Wet-to- ry ressing
C) opical antibiotics
D) ransparent, occlusive ressing (e.g., ega erm) Question 21.1.6 Which o the ollowing statements is NOT
E) Chemical enzyme ebri ement (e.g., Accuzyme) true about the evaluation and treatment o pressure ulcers?
A) A pressure ulcer covere by eschar cannot be stage until
Answer 21.1.5 The correct answer is “A.” In treating pressure the eschar is remove .
ulcers, there are several principles to ollow: relieve pressure, B) I a pressure ulcer shows no signs o healing over 2 weeks,
protect the woun an surroun ing skin rom urther trauma, one shoul reevaluate woun management strategies an
maintain a clean woun be , provi e a moist woun environ- reexamine actors a ecting the woun .
ment, eliminate ea space, control exu ates, ensure a equate C) One shoul consi er osteomyelitis or eep sof -tissue in ec-
nutrition, an iagnose an treat in ection. Your patient’s ulcer tion in a woun that is not healing.
572 FAMILY MEDICINE EXAMINATION & BOARD REVIEW

D) Woun cultures shoul be obtaine routinely to target anti- as a single ose at least 1 year af er the last ose o the 23-valent
biotics towar the organisms oun . polysacchari e pneumococcal vaccine.
E) In an otherwise clean woun that is not healing as expecte ,
one shoul consi er empiric therapy with topical antibiotics.
QUICK QUIZ: HORMONES AND AGING
Answer 21.1.6 The correct answer is “D.” T e routine cul-
turing o pressure ulcers is not recommen e . Antibiotics are Noticing a vertisements or testosterone treatment to “make
generally not use ul since the organisms oun are polymicro- you a vital man again,” one o your vivacious ol er emale
bial colonizers an not responsible or in ection. Obviously, patients asks about testosterone or her husban won ering i it
this oes not hol true or patients with a true in ection, an can make him 50 years ol again.
empiric therapy with topical antibiotics is in icate i a woun
shows no improvement with goo woun care. Regar ing sex hormone changes associate with aging, which o
the ollowing is true?
Objectives: Did you learn to . . . A) Ley ig cells in the testes increase with aging
• Manage insomnia in elderly nursing home residents? B) otal testosterone levels increase with aging
• Develop an approach to the problem o alls in nursing home C) Sex hormone bin ing globulin levels increase with aging
residents? D) Follicle-stimulating hormone levels are unchange with
• Identi y nursing home residents at risk or malnutrition? aging
• Develop a treatment plan or malnutrition in the institution- E) Luteinizing hormone levels ecrease with aging
alized elderly?
• Diagnose, evaluate, and manage pressure ulcers in the nurs- The correct answer is “C.” Sex hormones in males over age
ing home setting? 40 emonstrate eclining total testosterone at a rate o 1% to
2% per year. Simultaneously, sex hormone bin ing globulin
increases, resulting in a sharper ecline in bioavailable testos-
QUICK QUIZ: VACCINES terone. In response to low testosterone, ollicle-stimulating an
luteinizing hormone levels increase. Ley ig cells (responsible
A 65-year-ol male presents or a routine visit an you recom- or pro ucing testosterone) ecrease in number. Ol er males
men pneumococcal vaccination (23-valent polysacchari e with subphysiologic testosterone levels are at increase risk o
vaccine, Pneumovax). Your patient asks what the vaccine is sup- sexual ys unction, osteoporosis, iminishe lean bo y mass,
pose to o. epression, an inability to kill a mountain lion with their bare
han s. Although controversial, some experts recommen tes-
Accor ing to the best available evi ence, you are able to say: tosterone supplementation in symptomatic males with low
A) “T is vaccine will re uce your risk o pneumococcal bacte- serum testosterone. estosterone supplementation may (notice
remia.” the wor “may”) improve strength, lean bo y mass, epresse
B) “T is vaccine will re uce your risk o pneumococcal pneu- moo , bone mineral ensity, an sexual unction in aging males
monia.” with ocumente low testosterone levels. Si e e ects o testos-
C) “T is vaccine will re uce your risk o all types o pneumo- terone supplementation in ol er males inclu e: car iac isease,
nia.” liver ys unction, yslipi emia, erythrocytosis, prostate tis-
D) “T is vaccine will re uce your risk o eath rom in uenza.” sue growth, acne, gynecomastia, an e ema. I your patient is
E) “T is vaccine will o all o the above.” bale ul and pushes the request or “ ,” point out that testosterone
adds nothing to the treatment o erectile dys unction beyond that
The correct answer is “A.” T e 23-valent polysacchari e pneu- bene t which is o ered by tadala l (Cialis), sildena l (Viagra),
mococcal vaccine has only been shown e ective in re ucing the etc. You can reassure him that he will be just as virile at age 98
risk o pneumococcal bacteremia an meningitis. T e vaccine without testosterone an will avoi the risk o worsening heart
oes not appear to re uce the risk o pneumonia in general or isease (Ann Intern Med. 2012;157:681).
even pneumococcal pneumonia in particular. In uenza vacci-
nation ecreases the risk o eath ue to in uenza, but pneumo- HELPFUL TIP:
coccal vaccination oes not. Any time you see the word “may,” think “may not.” They
But what you should have done is o er this patient the pneu- are logical equivalents.
mococcal conjugate vaccine (e.g., Prevnar) rst. In 2014, the
CDC recommen e the 13-valent pneumococcal conjugate
vaccine (PCV 13) or use in all patients over the age o 65 as a
one-time ose. It shoul be a ministere to vaccine-naive in i-
CASE 21.2
vi uals ollowe by the 23-valent polysacchari e pneumococcal A 79-year-old emale patient, well known to you rom 5 years
vaccine (Pneumovax) 6 to 12 months later. For in ivi uals who o treating her hypertension, presents to your o ce or an
have been immunize with the 23-valent polysacchari e pneu- “annual examination” and “medication re ll”. Her only medi-
mococcal vaccine, it is recommen e that the PCV 13 be given cations are lisinopril, amlodipine, and acetaminophen. She’s
CHAPTER 21 • CARE OF THE OLDER PATIENT 573

noticed some hearing loss over the last year, and wonders pelvic examination is more controversial, with the American
i she should blame her medications and not all those rock Cancer Society (ACS) recommen ing it as a screening measure
concerts she attended as a young adult. Oh, by the way, her or ovarian cancer an the US Preventive Services ask Force
gynecologist recently retired and she would now like you to (USPS F) recommen ing against it.
assume that care, which apparently included all o her pre- Screening or breast cancer continues to be recommen e
ventative care. or women with a 5 to 10 years li e expectancy, but the optimal
interval in ol er women is unknown: T e American College o
Question 21.2.1 Which o the ollowing drugs is NOT associ- Physicians (kin o presumptuous name . . . what about FM? Are
ated with sensorineural hearing loss? we not physicians?) an USPS F recommen screening o aged
A) Ibupro en 50 to 74 years every other year; our patient is 79. Whether this
B) Aminoglycosi es patient will bene t rom mammography is unknown. T ere is a
C) Furosemi e signi cant question o the “over iagnosis” o breast cancer; this
D) Magnesium salicylate is cancer iagnose by mammogram that woul not ever have
E) Acetaminophen cause a problem or the patient (think o it as the equivalent o
the iagnosis o prostate cancer in men . . . they may ie with the
Answer 21.2.1 The correct answer is “E.” All o these rugs cancer an not o the cancer) (e.g., JAMA Intern Med. Publishe
except or acetaminophen can cause hearing loss. Cisplatin, online July 06, 2015. oi:10.1001/jamainternme .2015.3043).
aminoglycosi e antibiotics, an loop iuretics have been asso- Be ore or ering a mammogram, we nee to have a iscussion
ciate with hearing loss, as have salicylates (e.g., aspirin) an with our patients about what types o intervention they woul
some o the other NSAIDs (e.g., ibupro en an i unisal) an want shoul a positive mammogram be note .
chloroquine. T is list is obviously not exhaustive.

HELPFUL TIP:
HELPFUL TIP: The 2012 consensus guidelines on screening or cervi-
Hearing loss, and sensory impairments in general, can cal cancer recommend discontinuing screening at age
be con used with cognitive impairment or an a ective 65 i the previous three Pap smears prior to age 65
disorder. Hearing aids are use ul or most cases o pres- were completely normal and there were no indings o
bycusis, but i speech discrimination is < 50%, results CIN2 or higher in the previous 20 years. Once stopped,
with hearing aids may be poor. screening should not be resumed or any reason, in-
cluding a new sexual partner.

Your patient then asks, “Do I have to keep getting mam-


mograms and Pap smears?” She relates a history o normal Your patient is an overweight white emale with no history o
annual mammograms and Pap smears or the past 20 years, bone racture. She has never had a bone mineral density test
as her previous physician hadn’t changed her practice since and asks i she should have one. You are unaware o any risk
T e Backstreet Boys were popular. She had a hysterectomy actors in her other than Caucasian race and postmenopause
or uterine bromas and has been monogamous with her status.
husband or 55 years. Her sister died o breast cancer. Using
your magic crystal ball, you estimate her li e expectancy at Question 21.2.3 What do you tell her?
10 years. A) “You are not at risk or osteoporosis an shoul not be
screene ”
Question 21.2.2 Consistent with current guidelines, you B) “All women over age 65 shoul be screene or osteoporosis
recommend: regar less o risk”
A) Continue Pap smears an pelvic examinations yearly C) “ ake 1,000 mg o calcium per ay to prevent osteoporosis.
B) Discontinue mammography but per orm clinical breast You’ll be unbreakable—like Wolverine”
examinations every 2 years D) “Due to your risk actors, you shoul start a bisphosphonate,
C) Discontinue pelvic examinations but continue Pap smears vitamin D, an calcium supplementation”
D) Discontinue Pap smears but o er mammography at 1- to E) “Alcohol—an lots o it—will help ecrease your risk o os-
2-year intervals teoporosis”

Answer 21.2.2 The correct answer is “D.” ricky question. Answer 21.2.3 The correct answer is “B.” T e USPS F cur-
Screening ecisions in the el erly shoul be in ivi ualize , rently recommen s bone ensitometry screening or all women
an the patient’s overall health status must be consi ere . T is o age 65 years an ol er. T e National Osteoporosis Foun-
patient has very little risk o cervical, en ometrial, or vaginal ation recommen s bone ensitometry or postmenopausal
cancer (status post hysterectomy or a benign con ition, low- emales with one or more o the ollowing risk actors: amily
risk sexual behavior, an a history o normal examinations); history o osteoporosis, personal history o low trauma racture,
there ore, it is reasonable to iscontinue Pap smears. Annual current smoking, or low bo y weight (< 127 lb). A itional risk
574 FAMILY MEDICINE EXAMINATION & BOARD REVIEW

actors or osteoporosis inclu e: emale sex, Caucasian or Asian Question 21.2.5 Although most osteoporosis in women is
races, alcohol abuse (thus, “E” is wrong), se entary li estyle, an primary (idiopathic), which o the ollowing cause(s) sec-
poor intake or absorption o calcium an vitamin D. Smoking is ondary osteoporosis?
associate with osteoporosis. Diabetes, once thought to protect A) Hypoparathyroi ism
against osteoporosis, may actually increase the risk o alls an B) Multiple myeloma
ractures in ol er a ults. T e pre erre metho or measuring C) Estrogen use
bone ensity is ual-energy x-ray absorptiometry (DEXA). All D) Hyperlipi emia
postmenopausal women shoul consume 1,200 mg o elemen- E) Overly ambitious, or perhaps calcium-a icte , ooth
tal calcium per ay in ivi e oses. T e optimal amount o Fairies
vitamin D is 400 to 800 IU/ ay. Weight-bearing exercises also
strengthen bone. Bisphosphonates are in icate or treatment
Answer 21.2.5 The correct answer is “B.” About 70% o
o osteoporosis an shoul not be use without a iagnosis (so
women have no i enti able cause or osteoporosis an there-
“D” is incorrect).
ore are iagnose with primary (i iopathic) osteoporosis.
Common causes o secon ary osteoporosis inclu e chronic
HELPFUL TIP: corticosteroi use, alcoholism, GI isor ers, hyperthyroi ism,
Supplementing vitamin D is probably more important hyperparathyroi ism (so, “A” is wrong), multiple myeloma,
than supplementing calcium. Calcium supplementa- an primary renal iseases. Hyperlipi emia is not known to be
tion has not consistently demonstrated racture risk re- associate with osteoporosis. Estrogen increases bone mineral
duction. However, many elderly Americans are vitamin ensity.
D de icient, and correction o the de iciency results in
reduced racture risk. Given the low risk o adverse e - You encourage appropriate vitamin D and calcium intake as
ects with daily vitamin D (up to 1,000 IU/day), empiric well as weight-bearing exercises. You plan to obtain a DEXA
supplementation is justi iable. Finally, 1,000 to 1,200 mg scan.
(not 1,500 mg) o calcium is probably plenty. Additional
calcium has been linked to an elevated risk o coronary
Question 21.2.6 Using DEXA scan results, osteoporosis is
artery disease and no change in racture risk. Calcium
de ned as:
and vitamin D are clearly bene icial in those with osteo-
A) A -score o 2.5 stan ar eviations or more below the
porosis, however.
mean o a healthy young a ult (≤ − 2.5).
B) A -score rom 1.0 up to 2.5 stan ar eviations below the
mean o a healthy young a ult (− 1.0 up to − 2.5).
Next, your patient asks whether any o her medications put C) A Z-score o 2.5 stan ar eviations or more below the
her at risk or osteoporosis. mean o a healthy young a ult (≤ − 2.5).
D) A Z-score rom 1.0 up to 2.5 stan ar eviations below the
Question 21.2.4 Which o the ollowing is LEAST likely to mean o a healthy young a ult (− 1.0 up to − 2.5).
increase the risk o osteoporosis?
A) Glucocorticoi s
Answer 21.2.6 The correct answer is “A.” T e -score com-
B) Anticonvulsants
pares the patient’s bone mineral ensity to that o young,
C) Sul onylureas
healthy women ( or emale patients; male normative ata is
D) Loop iuretics
use or males). Osteoporosis is e ne as a -score o 2.5 stan-
E) Proton-pump inhibitors
ar eviations or more below the mean (≤ − 2.5). Osteopenia
is e ne as a -score rom 1.0 up to 2.5 stan ar eviations
Answer 21.2.4 The correct answer is “C.” Sul onylureas o not
below the mean (− 1.0 to − 2.5). “C” is incorrect. T e Z-score
have a irect e ect on bone mineralization. Glucocorticoi s an
compares bone mineral ensity to that o age-matche controls.
anticonvulsants are known to increase bone turnover, resulting
T ere ore, it oes not re ect the bone loss rom baseline in a
in increase risk o osteoporosis. Loop iuretics cause renal
young healthy emale, an it is not use or iagnosis. “D” is
calcium wasting. T ere is an association between proton-pump
incorrect or the same reason.
inhibitor use an osteoporosis, possibly through re uce cal-
cium absorption or irect e ects on bone metabolism. T ere
oes not appear to be this same association with H 2-blockers Question 21.2.7 I you nd that your patient has osteopo-
such as amoti ine. A itionally, heparin, methotrexate, cyclo- rosis, you may consider using all o the ollowing drugs to
sporine, an gona otropin-releasing hormone agonists may treat her osteoporosis EXCEPT:
increase the risk o osteoporosis. Excessive amounts o levothy- A) Bisphosphonates (e.g., alen ronate an rise ronate)
roxine can cause increase bone turnover. T iazi e iuretics are B) Estrogens
protective. C) Progesterone (e.g., Provera an Depo-Provera)
D) Vitamin D an calcium
Your patient asks what causes osteoporosis. E) Calcitonin
CHAPTER 21 • CARE OF THE OLDER PATIENT 575

Answer 21.2.7 The correct answer is “C.” Progesterones are


a ter 5 years o therapy—at least or a ew years. There
not in icate or the treatment or prevention o osteoporo-
are not clear guidelines on this yet. Consider getting a
sis. In act, in young, healthy, premenopausal women, they
DEXA scan a couple o years a ter stopping the bisphos-
are associate with a ecrease in bone mineral ensity. T is is
phonate. I it shows bone loss, consider restarting the
because they suppress estrogen pro uction (such as with Depo-
bisphosphonate.
Provera). All o the other options are acceptable choices in the
treatment o osteoporosis.
HELPFUL TIP:
She returns to discuss her test results, and her bone density is Denosumab (Prolia) is a RANKL inhibitor approved by
very low with her hip -score 3.2 standard deviations below the FDA or treatment o postmenopausal osteoporosis.
the mean (− 3.2). You diagnose her with osteoporosis. “Are It appears to be e ective in reducing racture risk (ver-
my bones like Swiss cheese?” she asks. No, you think. Swiss tebral and hip). It also appears well tolerated. However,
cheese has more calcium. it also inhibits RANKL in the immune system (which acts
similar to TNF), and the long-term e ects on in ection
Question 21.2.8 Given that she is otherwise relatively and carcinogenesis are unknown, so stay tuned.
healthy, what is the most appropriate initial therapy or her
osteoporosis?
A) Alen ronate (Fosamax) 70 mg PO weekly Be ore she leaves the o ce, you present your patient with
B) Estrogen (e.g., Premarin) 0.625 mg PO aily literature on living wills and durable power o attorney or
C) eriparati e (Forteo) 20 g SC aily or 5 years healthcare (DPOA-HC).
D) Zole ronic aci (Reclast) 5 mg IV every 3 months
Question 21.2.9 Which o the ollowing is CORRECT regard-
Answer 21.2.8 The correct answer is “A.” Bisphosphonates ing advance healthcare planning?
are the treatment o choice or osteoporosis. Alen ronate, rise- A) T e Joint Commission requires that patients be aske about
ronate, an zole ronic aci have all been shown to re uce their a vance irectives on a mission to the hospital.
the risk o vertebral an hip ractures in persons with osteo- B) A DPOA-HC can overri e a patient’s ecision regar ing
porosis. Iban ronate only appears to lower vertebral racture treatment.
risk; there ore, it shoul not be your rst choice. Si e e ects C) Once the patient has signe a living will, no urther changes
o bisphosphonates inclu e hypocalcemia (more likely with IV can be ma e regar ing treatment ecisions.
a ministration, as with zole ronic aci , an in patients with D) A DPOA-HC must be a amily member or bloo relative.
vitamin D e ciency), nausea, esophagitis, osteonecrosis o the Answer 21.2.9 The correct answer is “A.” T e Joint Commis-
jaw (usually at higher oses, such as those use to treat cancer), sion requires that patients be aske about their a vance irectives
an atypical emur ractures. Although you might be tempte on a mission to the hospital. A vance irectives can take many
to choose “D,” zole ronic aci , ue to this patient’s more severe orms but are usually mani est in one o two ways: through a liv-
osteoporosis, there is no clear evi ence that zole ronic aci is ing will or a DPOA-HC. T e purpose o a living will is to instruct
more e ective than oral bisphosphonates. T e IV route shoul healthcare ecision-making in uture events when the patient may
be reserve or when the patient has aile oral bisphosphonate not be able to communicate his or her wishes. T ese ocuments
therapy or some reason—usually ue to upper GI isease or of en contain brie clinical scenarios with patient pre erences
si e e ects. Zole ronic aci can cause a “ u-like” illness, with or li e-sustaining measures. In contrast, a DPOA-HC is not as
i use myalgias an arthralgias or ays af er the in usion. “B” limite an can a ress situations not oreseen in a living will. I
is incorrect as the sa ety ata or bisphosphonates is superior to the patient becomes unable to participate in healthcare ecision-
that or estrogen. “C” is incorrect or two reasons: teriparati e, making, then the DPOA-HC is instructe to exercise substitute
a recombinant parathyroi hormone, is a ministere aily an ju gment, using the patient’s previously state healthcare pre er-
carries a risk o osteosarcoma. For this reason its use is limite ences, to help irect uture care. T e DPOA-HC is appointe by
to 2 years, not 5. eriparati e is e ective at increasing bone the patient an can be a amily member or another a ult. T e
ensity an re ucing racture risk, but the ata is not nearly as DPOA-HC cannot overri e a patient’s ecision in healthcare
robust as the ata or bisphosphonates. It is not a rst-line rug. matters, as such an action woul violate patient autonomy.

HELPFUL TIP:
HELPFUL (MAYBE) TIP: Although advance directives should be addressed with
It seems as though more than 5 years o a bisphospho- all patients, it is o particular importance to discuss them
nate may increase atypical emur racture risk (thought in the setting o chronic illness, li e-threatening illness,
to be due to the suppression o osteoclast activity and advancing age, and with any deterioration in health sta-
reduced ability o bone to remodel). There ore, a drug tus. A patient can change advance care plans whenever
holiday is likely warranted. Additionally, many patients he or she wishes, as these decisions may change over
maintain bone density a ter stopping a bisphosphonate time depending on goals o care.
576 FAMILY MEDICINE EXAMINATION & BOARD REVIEW

Objectives: Did you learn to . . . the iagnostic criteria or elirium. A iagnosis o elirium
• Identi y and implement appropriate preventive health ser- requires the ollowing criteria: isturbance o consciousness
vices or older emales? with re uce ability to ocus attention; isorientation, memory
• Discuss issues related to breast and gynecologic cancer e cit, or another change in cognition that cannot be accounte
screening? or by a preexisting ementia; acute onset with uctuating
• De ne appropriate criteria or osteoporosis screening and course (of en changing throughout the ay); an evi ence that
identi y risk actors? the isturbance is cause by an un erlying me ical con ition or
• Recognize the important and complementary roles o DPOA rug use. Not all signs an symptoms o elirium are present in
and advance directives? every patient with elirium, nor will they be present at the same
time. Delirium can be con use with ementia, epression, or
psychosis. Patients with elirium may present as agitate , psy-
QUICK QUIZ: GERIATRIC PREVENTIVE CARE chotic, somnolent, or with rawn (these are the tough ones to
recognize). Dementia is more chronic in nature with an insi i-
Which o the ollowing statements is INCORREC regar ing ous onset. Dementia progresses over time an usually cannot
preventive health in ol er a ults? be reverse . Of en elirium is treatable or reversible i the
A) Although the optimal interval or vision screening is un e- un erlying me ical con ition is i enti e an treate . Patients
termine , many pro essional organizations recommen with ementia usually have intact attention, whereas patients
vision an glaucoma screening every 1 to 2 years in persons with elirium have marke ly impaire attention. Patients with
over age 65 ementia have “poverty o thought,” which implies ecrease
B) In women at high risk or breast cancer, tamoxi en re uces content o their thoughts. Patients with elirium may have a
the risk o cancer by almost 50% rich content to their thoughts, but the thoughts are isor ere .
C) T e American Cancer Society (ACS) an American College
o Obstetrics an Gynecology (ACOG) recommen screening
HELPFUL TIP:
ultrasoun or ovarian cancer in all women over the age o 60
Delirium may be either hypoactive or hyperactive or
D) Although Pap smears are not generally recommen e or
both, all in the same patient. Many patients with de-
el erly women, the istribution o cervical cancer cases is
lirium are not even identi ied due to their hypoactive
bimo al, with peaks at 35 to 39 years an 60 to 64 years
state (these aren’t the ones screaming obscenities and
yanking out in lated bladder catheters—ouch!). Elderly
The correct answer is “C.” In act, ACOG, the USPS F, an
patients are more susceptible to delirium, and delirium
the American College o Physicians speci cally recommen
is sometimes the only identi iable symptom in an elder
against ultrasoun screening or ovarian cancer in asymptom-
with an acute illness.
atic women. All other statements are correct.

CASE 21.3 With urther history rom the patient’s wi e, you nd that he
An 82-year-old male patient presents to your o ce or con u- has coronary artery disease, diabetes mellitus type 2, hyper-
sion. His wi e reports that he was in his usual state o health tension, and benign prostatic hyperplasia.
until 3 days ago. At that time, he developed abdominal pain
and elt everish. He then began to have a dry, hacking cough. Question 21.3.2 Which o the ollowing is the most use ul
On examination, his temperature is 100.3°F and blood pres- question to elicit risk actors or delirium?
sure is 118/56 mm Hg. He is pale and lethargic but in no acute A) “Does the patient have any rug allergies?”
distress. He is oriented to person only. Other than mild upper B) “Does the patient use tobacco?”
abdominal tenderness, there are no additional ndings on C) “Does the patient use alcohol?”
examination. T is patient appears to have a new onset o con- D) “Does the patient have e ema?”
usion. You suspect delirium. E) “Does the patient use acetaminophen?”

Question 21.3.1 Which o the ollowing is true with regard Answer 21.3.2 The correct answer is “C.” Stu ies have consis-
to delirium and dementia? tently i enti e the ollowing risk actors or elirium: a vanc-
A) In elirium it is rare to n an un erlying me ical cause ing age, preexisting ementia, un erlying structural brain is-
B) A primary eature o elirium is inattention ease other than ementia, uncorrecte impairment in vision or
C) Dementia is characterize by an acutely uctuating course hearing, multiple chronic illnesses, polypharmacy, the use o
D) Delirium only occurs in patients with ementia physical restraints, history o alcohol abuse, male gen er, an
E) T ey are the same. Af er all, memory is but an ephemeral unctional impairment.
light in the long sha ow o time While hypoxia ( ue to myocar ial in arction, pulmonary
embolus, or any other source) may lea to elirium, tobacco
Answer 21.3.1 The correct answer is “B.” T e Diagnostic use alone oes not pre ispose a patient to evelop elirium.
an Statistical Manual o Mental Disor ers (DSM-V) provi es In isolation, knowle ge about e ema, acetaminophen use, or
CHAPTER 21 • CARE OF THE OLDER PATIENT 577

TABLE 21-2 CAUSES OF DELIRIUM, You are called in the middle o the night or agitated
“WHHHHIMP” MNEMONIC behavior and noncompliance with nursing care. T e patient
has pulled out his IV and struck a nurse.
• Wernicke encephalopathy
• Hypoper usion
• Hypoglycemia and other metabolic abnormalities Question 21.3.4 You appropriately prescribe which o the
• Hypertensive encephalopathy ollowing interventions?
• Hypoxia A) A minister haloperi ol 0.5 mg PO
• In ection or intracranial bleed B) A minister haloperi ol 1 mg IV
• Meningitis or encephalitis
C) Apply physical restraints
• Poisons or medications
D) A minister morphine 5 mg IV
E) Lorazepam 1 mg PO or the patient, 0.5 mg or the nurse

rug allergies is less help ul. Questioning about alcohol use will Answer 21.3.4 The correct answer is “A.” Most o the time,
elirium DOES NO require any pharmacologic treatment.
help to i enti y patients who have a ten ency to overuse alco-
However, this patient is at risk o harming himsel an others
hol, putting themselves at risk or elirium. Also, elirium may
ue to his agitate elirium, so some action must be taken. Agi-
result rom alcohol with rawal.
tate elirium causes physiologic an psychologic stress on the
patient, results in inter erence with me ical care, an porten s
a poorer prognosis. T e inci ence o elirium in hospitalize
HELPFUL TIP:
patients o all ages is 30%, an the inci ence in postoperative
Preexisting dementia greatly increases the risk o de-
patients may approach 50%. Agitate elirium shoul be treate
lirium, and simply moving a patient with dementia to a
quickly, an haloperi ol is the treatment o choice. In ol er
new environment can precipitate delirium.
patients, rug clearance ecreases, so low oses o antipsychotic
me ication shoul be a ministere initially. As the geriatricians
say, “Start low an go slow.” Increasing oses o oral haloperi ol
Question 21.3.3 The appropriate evaluation o the patient can be given every 30 minutes i the patient continues to have
with delirium includes which o the ollowing? agitation. “B” is incorrect because IV haloperi ol is associate
A) Evaluation o metabolic causes such as electrolytes an with a greater egree o Q prolongation; PO or IM haloperi-
glucose ol is pre erre . “C” is incorrect; physical restraints may lea
B) Evaluation or in ection such as pneumonia to patient injury an may worsen elirium. Restraints shoul
C) Evaluation o a patient’s me ications only be applie when absolutely necessary an or as short a
D) Evaluation o oxygen saturation uration as possible. “D” is incorrect because we have no rea-
E) All o the above son to believe the patient is in pain, an a ministering nar-
cotics coul worsen his elirium. However, pain can certainly
Answer 21.3.3 The correct answer is “E.” Causes o elirium result in agitation, so keep it in min . Finally, “E” is wrong
are protean, of en acting together in a multi actorial manner, or at least two reasons: there is a substantial risk o para oxi-
an are best consi ere by a systematic approach. Metabolic cal agitation with benzo iazepine a ministration in emente
causes inclu e electrolyte isturbances ( on’t orget calcium), el ers, an you shoul not prescribe controlle substances
hypoglycemia, an hypoxia. Numerous in ections may lea to to sta nurses.
elirium. Neurologic causes inclu e hea trauma, meningitis,
an vasculitis. Many me ications cause elirium inclu ing anti- You ref ect on the act that primary prevention o delirium
cholinergics, anti epressants, se ative–hypnotics, an steroi s. is probably more e ective than treatment. For this patient,
Dehy ration an prerenal azotemia may lea to elirium. See delirium must now be treated, but you try to avoid this com-
able 21-2. plication in your hospitalized older patients.

You reach or the “shotgun” (metaphorically, we hope) and Question 21.3.5 You know that research has shown a reduc-
order a bunch o tests. T e chest radiograph shows a le tion in delirium in hospitalized older patients when which
lower lobe consolidation. T e abdominal lm shows a non- o the ollowing strategies is employed?
speci c bowel gas pattern. T e white blood cell (WBC) count A) Increase se ative me ication use or sleep eprive patients
is 12,700/mm3 , blood urea nitrogen 36 mg/dL, creatinine B) Early mobilization or immobilize patients
1.5 mg/dL, and glucose 150 mg/dL. T e remainder o the C) Cholinesterase inhibitor (e.g., onepezil) therapy or cogni-
blood counts and chemistries are normal. With the excep- tively impaire patients
tion o trace ketones, the urinalysis is within normal limits. D) Physical restraints or combative patients
Cultures will not be available or at least 24 hours. T e ECG E) Music therapy or epresse patients
shows normal sinus rhythm. With the available in ormation,
you decide to admit this patient or treatment o delirium Answer 21.3.5 The correct answer is “B.” I enti ying risk
due to pneumonia and dehydration. actors an targeting interventions to re uce or eliminate risk
578 FAMILY MEDICINE EXAMINATION & BOARD REVIEW

actors can prevent elirium. Because one patient may have


HELPFUL TIP:
numerous risk actors or elirium an because elirium is usu-
Ethical dilemmas abound in the treatment o delirium.
ally a multi actorial syn rome, a multicomponent intervention
Atypical antipsychotic use in patients with dementia is
strategy is warrante .
associated with increased mortality. Older antipsychot-
An of en-cite stu y by Inouye et al. (1999) emonstrate
ics pose a greater risk or extrapyramidal symptoms
the e ectiveness o this strategy. In the stu y, sleep- eprive
when compared with atypicals and have a higher mor-
patients receive a warm rink, relaxing music, an back mas-
tality risk. By de inition, agitated, patients with delirium
sage at be time. Unit-wi e noise re uction was implemente .
cannot provide in ormed consent, so “implied consent”
Early ambulation an active range-o -motion exercises were
is usually substituted to use drug therapy or patients
employe or be -boun patients. All patients were encour-
with delirium at risk or sel -injury and to stabilize criti-
age to ambulate. Cognitively impaire patients receive ori-
cally ill patients with delirium.
enting stimuli an cognitively stimulating activities. Patients
with hearing an visual impairments receive portable ampli-
ying evices an visual ai s, respectively. Investigators use Objectives: Did you learn to . . .
a protocol or early recognition an treatment o ehy ration. • De ne delirium?
T ere was no speci c therapy or epression or combative-
• Describe the signs and symptoms o delirium?
ness. As you can see, the preventative strategies or elirium
• Distinguish delirium rom dementia?
must also be broa as no one treatment has been singularly
e ective. • Identi y causes and risk actors or delirium?
T e Con usion Assessment Metho (CAM) is an evi ence- • Treat and prevent delirium?
base tool that coul have helpe you i enti y elirium sooner
in your patient. It takes into account onset, inattention, isorga- CASE 21.4
nize thinking, isorientation, an other important actors. See
https://www.healthcare.uiowa.e u/igec/tools/cognitive/CAM. In the early morning hours, a 78-year-old emale presents
p or a worksheet to calculate CAM. Using CAM takes just to the emergency department complaining o right buttock
a ew minutes an is, o course, easier an much sa er than and hip pain. Several hours be ore her arrival, she ell in the
waiting to n him walking his imaginary ragon through the bathroom o her daughter’s home. She recalls standing on a
hospital war while carrying a hal - lle be pan. f oor mat, leaning her head back to drink a glass o water, and
then hitting the ground. She denies loss o consciousness.
Question 21.3.6 Which o the ollowing statements about Her daughter was at the scene quickly and ound the patient
delirium is true? awake, alert, and moving all extremities. Her vital signs are
A) Be rails an restraints are e ective in preventing injury in normal. Other than right hip tenderness, her examination is
the patient with elirium unremarkable.
B) Atypical antipsychotics (e.g., olanzapine an risperi one)
can be use to treat elirium Question 21.4.1 Which o the ollowing is most likely to
C) Diphenhy ramine is a goo choice or a sleep ai in patients assist you in determining the cause o her all?
who are prone to eveloping elirium A) C scan o the hea
D) A ee ing tube (e.g., Dobho tube) shoul be use in the B) ECG
patient who is not eating in or er to prevent elirium. C) A itional history
E) Shouting, “You on’t have a ragon! T ey aren’t real!” helps D) Serum chemistry pro le
to prevent complications o elirium E) CBC

Answer 21.3.6 The correct answer is “B.” Atypical antipsychot- Answer 21.4.1 The correct answer is “C.” I there’s an option
ics can be use in the treatment o elirium, but nonpharmaco- or “more history,” it’s usually the right answer. History is one o
logic metho s (e.g., be si e sitter an re irection) shoul be the most important actors in etermining the etiology o a all.
use rst. “A” is incorrect because be rails an restraints actu- en percent o alls in ol er persons can result in serious injury,
ally increase the risk an severity o injury in the patient with such as a hip racture or sub ural hematoma. Falls in el erly
elirium. Restraints are use as a last resort. “C” is incorrect. patients are typically multi actorial in nature. Ran omize
Diphenhy ramine is a particularly poor choice because o its clinical trials emonstrate a re uction in the occurrence o alls
anticholinergic si e e ects, which can exacerbate or cause elir- in community- welling el ers when healthcare personnel engage
ium. razo one woul be better. Melatonin an the melatonin in a multi actorial risk assessment with targete management.
agonist ramelteon have shown some ability to re uce the risk Such an approach requires a thorough history.
o elirium but only when starte prior to the onset o symp-
toms an only in small trials. “D” is incorrect, as anyone who Question 21.4.2 All o the ollowing are risk actors or alls
has one inpatient work knows: the ee ing tube an the Foley in elderly EXCEPT:
are of en the rst to get yanke ! “E” is also no goo . Never argue A) Use o our or more me ications
with a patient about his elusion. You will not win. B) Orthostatic hypotension
CHAPTER 21 • CARE OF THE OLDER PATIENT 579

C) Attempting tai chi, or which goo balance is require D) Unsupervise balance an strength training is e ective in
D) Environmental hazar s (e.g., poor lighting or uneven walk- re ucing all risk
ing sur aces) E) Group exercise is more e ective than exercising alone to
prevent alls
Answer 21.4.2 The correct answer is “C.” In act, oing tai chi
has been shown to re uce the risk o alls in the el erly. All o Answer 21.4.4 The correct answer is “D.” T ere is no particu-
the others liste increase the risk o alling. A itional risk ac- lar type o exercise that seems to prevent alls to a greater egree
tors or alls inclu e history o a all in the last year, impaire than any other type o exercise. Strength, balance, an gait train-
balance an gait, poor vision (acuity < 20/60), ecrease mus- ing all appear to be important. Exercise programs have been
cle strength, an syncope or arrhythmia. Still more risk actors: shown to bene t el ers at risk or alls. Although initial instruc-
poor lighting, lack o grab bars an han rails, cluttere oor, tion by a therapist may be help ul, a physical therapist nee not
restraint use, an improper be height. supervise all exercises. Patients are able to per orm exercises
targete towar all prevention at home, an they o not nee to
Question 21.4.3 During your examination, which o the ol- be part o an exercise group. A meta-analysis o the Frailty an
lowing physical maneuvers is most likely to assist you in Injuries: Cooperative Stu ies o Intervention echniques (FIC-
evaluating the risk o uture alls? SI ) trials oun that combine balance an strength training
A) Get-up-an -go test re uces the risk o alls in community- welling el ers.
B) est or pulsus para oxus
C) Osler maneuver HELPFUL TIP:
D) Lumbar spine exibility test Peripheral sensory disturbance is a common inding in
E) est or nystagmus the elderly and increases the risk o alling. A common
cause o peripheral neuropathy is vitamin B12 de iciency.
Answer 21.4.3 The correct answer is “A.” T e “time get-up- Check vitamin B12 levels. I they are low, great; treat
an -go-test” is a commonly use metho o assessing isability with oral B12, 1,000 to 2,000 mcg/day. Alternatively you
an all risk in geriatric assessment. From a seate position, the can treat with 1,000 mcg/month IM. I the B12 level is
patient is instructe to stan up, walk 3 m (approximately 10 f ), borderline and you still have a high suspicion, check
turn aroun , an return to her chair. An a ult with no isability serum methylmalonic acid and homocysteine levels.
shoul be able to complete this test in < 10 secon s. Increasingly These should both be high in B12 de iciency.
longer time to per orm the test is associate with increasing all
risk. While per orming the test, assess the patient’s sitting bal-
ance, transition rom sitting to stan ing, gait an stea iness, You recommend strength and balance exercises to the patient,
an quickness with turning. While potentially use ul in the but she is worried and says, “My heart’s too old or exercise.”
evaluation, the other tests liste are not irectly associate with
all risk. T e Osler maneuver may be help ul in n ing cases o Question 21.4.5 You assure her that light exercises are
pseu ohypertension an shoul be checke in patients who are sa e and then review normal age -related cardiovascular
having alls associate with position changes espite normal or changes, which include:
even high bloo pressures as measure by sphygmomanometer. A) Re uce ventricular compliance
T e Osler maneuver is per orme by blowing up the bloo pres- B) Re uce maximal heart rate
sure cu until the ra ial pulse is absent. T en see i you can pal- C) Re uce response to sympathetic nervous stimulation
pate a sclerotic, noncompressible, ra ial artery. I so, the Osler D) Increase atrial lling
test is positive an the patient may have pseu ohypertension. E) All o the above

A er a thorough history, you per orm a complete workup, Answer 21.4.5 The correct answer is “E.” Even in the healthy
including ECG, radiology studies, and appropriate labora- el erly without signs o vascular isease, there are important
tory tests. You nd that the all was caused by environmental changes in the car iovascular system. Maximum car iac output
actors (poor lighting and a loose throw rug) rather than an is re uce , mostly through re uce maximal heart rate; thus the
organic cause intrinsic to the patient. Fortunately, x-ray o equation:
her hip is negative or racture. T e patient asks how exercise
Estimate maximal heart rate = 220 – age.
might help her avoid uture alls.
A itionally, there is re uce response to sympathetic stimu-
Question 21.4.4 Which o the ollowing is the best answer? lation, with less chronotropic an inotropic response to stress.
A) Physical therapist supervision is essential to have an e ec- Re uce ventricular compliance results in increase atrial ll-
tive all prevention program ing volume an pressure, increase lef atrial size, an increase
B) In el erly patients, the e ect o exercise on alling is epen ence on atrial contraction or ventricular lling. How-
unknown ever, en urance training may improve car iac output, an
C) Strength training has a greater e ect than balance training active ol er a ults have a higher car iac output compare with
on re ucing all risk se entary persons o the same age.
580 FAMILY MEDICINE EXAMINATION & BOARD REVIEW

Question 21.4.7 I you were to provide her with a cane,


HELPFUL TIP:
what method would you use to t the cane?
Hip ractures in the elderly are o ten caused by alls. In
A) Allow the patient to t the cane length to her com ort level.
general, the sooner that a ractured hip is repaired, the
B) Select a evice with a length equal to the istance rom the
better the outcome. Ideally, a hip should be repaired
oor to the greater trochanter o the emur.
within the irst 24 hours a ter the injury.
C) Fit the cane length so that the han le comes to rest at the
patient’s waist.
D) Select a evice with a length equal to the istance rom the
oor to the ngertips with the arm relaxe .
Because o her right hip pain, you consider giving the patient E) Exten the cane to its maximum length, giving it the appear-
an ambulatory device or temporary use. She has good upper ance o a shepher ’s crook or wizar ’s sta .
extremity strength, can bear some weight on the right, but
needs improved stability. Answer 21.4.7 The correct answer is “B.” You shoul t a cane
or a patient by selecting a evice that reaches rom the oor to
Question 21.4.6 Which o the ollowing devices would be the greater trochanter o the emur. Another option is to t the
most appropriate in this setting? cane so that it reaches the exor crease o the wrist when the
A) Wheele walker arm is exten e to the si e. A properly tte ambulatory evice
B) Forearm crutches shoul be com ortable, allowing the patient to stan erect with-
C) Walk cane (hemi-walker) out excessive orwar exion o the spine. Excessive orwar
D) Imperial walker exion occurs when the evice is too short an can result in
E) Multiple-legge cane (qua cane) increase risk o alling. Also, to obtain maximum e ciency
rom the upper extremities, the evice shoul not be too long.
Answer 21.4.6 The correct answer is “E.” Ambulatory evices T e usual recommen ation is to have a evice tte so that the
are employe with the ollowing goals: improve mobility, elbow is exe at 15% to 30% when the cane is in use. T is is
ecrease the risk o alling, an relieve iscom ort associate one by measuring as note above.
with acute or chronic musculoskeletal an neurologic con i-
Objectives: Did you learn to . . .
tions. An inappropriately selecte evice can increase energy
• Recognize the morbidity associated with alls in older people?
expen iture an the risk or alls. Canes wi en a patient’s base,
• Evaluate causes and risks o alls in this population?
resulting in increase stability. T ey are typically use or bal-
• Implement appropriate interventions or alling patients?
ance, not weight bearing. Multiple-legge canes—calle “qua
canes” because o the presence o our tips—provi e a more • Assess gait abnormalities that may lead to alls?
stable base an some weight bearing, when compare with • Select ambulatory devices or appropriate patients?
stan ar , single-tippe canes. Walk canes are use ul or patients
who require ull weight bearing on one arm, as in a stroke sur- QUICK QUIZ: ASSISTIVE DEVICES
vivor with loss o lower extremity unction. Crutches, either
orearm or axillary, are use or patients who cannot bear any
I a patient is having i culty walking ue to lef -si e leg
weight on one leg.
weakness ue to osteoarthritis o the hip, in which han shoul
Walkers can support a patient’s weight, provi e lateral sta-
a cane be use ?
bility, an expan a patient’s support base. Stan ar walkers,
A) Lef han
those with our rubber tips, provi e the greatest support an are
B) Right han
help ul in cases o ataxia. Front-wheele walkers are use ul or
C) Either han
patients with a ast gait, such as a estinating gait in parkinson-
D) Both han s simultaneously
ism. Also, a ront-wheele walker is easier to manipulate than
E) No cane shoul be prescribe ; a wheelchair is pre erre
a stan ar walker. Four-wheele walkers shoul be use when
the patient requires some increase stability but oes not nee
The correct answer is “B.” Whenever a cane is use to sup-
as much weight-bearing as a stan ar walker woul provi e.
port lower leg unction limite by weakness or pain, the cane
Patients with mil -to-mo erate Parkinson isease may bene t
shoul be use in the han contralateral to the a ecte si e (lef
rom our-wheele walkers. In this case, the patient requires only
si e weak, right han gets the cane). T is enables the patient to
improve stability an slight assistance with bearing weight.
maintain normal arm swing while a vancing the cane an the
O the available choices, the multiple-legge cane is the best
a ecte leg at the same time to re uce the weight-bearing orces
option. A single-tippe cane may have been a reasonable choice
on the a ecte limb uring the step.
as well. I you chose answer “D,” you—like one o the e itors—
are a Star Wars ner .
CASE 21.5
Un ortunately, you do not have access to a multiple-legged A 69-year-old emale with no complaints presents to your
cane or your patient. But you do have a supply o adjustable o ce with her two daughters. Further history rom her
single-tipped aluminum canes available. daughters reveals that the patient was widowed 4 years ago,
CHAPTER 21 • CARE OF THE OLDER PATIENT 581

now lives alone, and has experienced memory loss over the
last 2 years. One daughter has taken over the patient’s check- You use several o ce assessment tools to urther character-
book and is responsible or paying the bills. She has noticed ize the memory loss. She scores 23/30 on the Folstein Mini-
that her mother o en wears the same clothes and bathes Mental State Exam, missing orientation and recall items.
in requently—new and un ortunate habits or her. T e past Clock drawing is grossly abnormal. Her geriatric depression
medical history includes hypothyroidism and hypertension. scale is 3 positive responses out o 15 (positive screen is 5/15
Family history is signi cant or depression and memory or greater). She per orms all basic activities o daily living
problems in the patient’s mother prior to her death rom “old (ADLs) independently, but has voluntarily given up driving
age.” T e patient takes chlorthalidone, levothyroxine, and and control o her nances.
acetaminophen as needed.
Physical examination reveals a thin, elderly emale in no Question 21.5.2 Regarding assessment tools used in the
distress. She is alert but does not correctly identi y the year. evaluation o memory loss, which o the ollowing state -
She describes her mood as “happy” most o the time. T e ments is most accurate?
remainder o the examination is unremarkable. You suspect A) T e Mini-Mental State Exam (MMSE) evaluates executive
dementia. unction an visual-spatial skills
B) Formal neuropsychological testing o ers no bene t over the
MMSE or etecting ementia
Question 21.5.1 Which o the ollowing is true regarding C) T e use o a screening tool or epression is not help ul in
the diagnosis o dementia? the evaluation o memory loss
A) T e iagnosis is rarely misse in the primary-care setting D) Clock rawing evaluates executive unction an visual-
B) o iagnose ementia, impairment in executive unction spatial skills
must be present E) T e MoCA evaluates the ability to iscern a chocolate co ee
C) o iagnose ementia, impairment in memory must be beverage rom regular co ee
present
D) Alzheimer isease (AD) is a iagnosis o exclusion
Answer 21.5.2 The correct answer is “D.” Clock rawing can
E) Neuroimaging is essential in the iagnosis o ementia
be use to evaluate executive unction as well as visual-spatial
skills. Clock drawing is a simple test that takes 1 minute or
Answer 21.5.1 The correct answer is “C.” One o the neces- less to per orm. T e patient is aske to raw a clock ace an
sary components in or er to make a iagnosis o ementia is set the han s to 2:50 or 11:10. T is test requires planning an
memory impairment. “A” is incorrect. In contrast to elirium visual-spatial ability on the part o the patient—two areas that
an epression, the onset o ementia is insi ious. Symptoms are incompletely evaluate by the MMSE. A normal clock oes
of en go unrecognize or months to years prior to iagnosis. not rule out ementia, but an abnormal clock is suggestive o
Although the patient may complain o con usion or memory cognitive impairment. T ere are several scoring systems, an
loss, amily members are more likely to provi e the chie the sensitivity an speci city or ementia are as high as 87%
complaint an history. During the initial phases o a ement- an 82%, respectively.
ing illness, patients an amily members may attribute cogni- “A” an “B” are incorrect. T e MMSE is a 30-point scale,
tive changes to normal aging. In early cognitive impairment, with the cuto or ementia between 24 an 26. T e MMSE
memory symptoms may wax an wane. However, symptoms o can be per orme in a ew minutes an tests memory, orien-
ementia can be i erentiate rom occasional normal lapses tation, language, construction, an concentration. T e MMSE
base on their increasing severity. For example, it is normal to oes not test proso y (expressive an receptive in ection o
orget an acquaintance’s name, but clearly abnormal to orget vocalization) or executive unction an , as a result, has poor
a spouse’s name. “B” is incorrect. Many patients with emen- sensitivity or early cognitive impairment in some in ivi uals.
tia have impaire executive unctioning (e.g., ju gment, rea- Per ormance on the MMSE is strongly correlate with e uca-
soning, an planning), but the presence o impaire executive tion; there ore, there may be alse-positives in un ere ucate
unctioning is not a requirement. “D” is incorrect, as AD is patients an alse-negatives in highly e ucate in ivi uals.
iagnose by a speci c set o clinical criteria. DSM-V pro- Compare with the MMSE, ormal neuropsychological testing
vi es iagnostic criteria or ementia an AD, making AD a assesses a broa er array o cognitive unctions, an it i enti es
iagnosis o inclusion rather than exclusion. “E” is incorrect. behavioral abnormalities an assesses moo isor ers. It can
Dementia is a clinical iagnosis an oes not require neuro- also help to i erentiate between types o ementia. In general,
imaging or con rmation. Experts an pro essional me ical neuropsychological testing is the most sensitive an speci c
associations i er in their recommen ations regar ing the use cognitive assessment tool, but it is time-consuming an requires
o neuroimaging in ementia. In general, neuroimaging is rec- a high level o expertise to a minister an interpret.
ommen e i ementia occurs in the ollowing scenarios: onset “C” is incorrect because epression may cause memory
be ore age 65, su en onset, presence o ocal neurologic signs, problems, especially in the el erly, an epression screening
an suspicion o normal pressure hy rocephalus (NPH), which shoul be inclu e in the workup o memory concerns. Depres-
inclu es urinary incontinence, gait isturbance, an cognitive sion of en coexists with ementia, an treatment o epression
impairment. may improve memory problems. It is also important to keep in
582 FAMILY MEDICINE EXAMINATION & BOARD REVIEW

min that epression commonly is the rst presenting sign or but can be initial complaints in atypical presentations o AD. In
memory problems. “E” is incorrect. See the relate Help ul ip or er to iagnose ementia, impairment in memory must be
or more on the MoCA. present along with at least one o the ollowing:

• Aphasia (language isturbance)


• Apraxia (impaire motor abilities espite intact motor unc-
HELPFUL TIP:
tion)
The Montreal Cognitive Assessment (MoCA) is a cogni-
tive assessment tool reely available online, published • Agnosia (impaire ability to i enti y objects espite intact
in multiple languages, and validated in the diagnosis sensation)
o dementia. Like the MMSE, it is a 30-point scale, but • Disturbance in executive unction (e.g., planning, ju gment,
the MoCA tests a wider range o cognitive domains with an insight)
less emphasis on orientation. Compared to the MMSE,
the MoCA has greater sensitivity or detecting dementia T e iagnostic criteria or ementia require that these cognitive
but takes a ew minutes longer to administer. isturbances result in unctional impairments that represent a
signi cant change rom a previous level o unctioning.

Although you strongly suspect AD in this patient, you con-


So ar, you have collected the ollowing in ormation on this sider other types o dementia as well. Suppose this patient
patient: MMSE score 23/30, impairment in driving and man- presented with urinary incontinence and ataxia in addition
aging nances, disorientation to time, but intact abilities to to her current ndings.
cook, clean, and care or hersel .
Question 21.5.5 Which o the ollowing diagnoses would be
Question 21.5.3 Using conventional staging or AD, how most likely?
would you categorize this patient’s dementia? A) Creutz el t–Jakob isease
A) Mil B) Lewy bo y ementia
B) Mo erate C) Normal pressure hy rocephalus (NPH)
C) Severe D) Pick isease
D) erminal E) Walking corpse syn rome
E) Insu cient in ormation to etermine the stage
Answer 21.5.5 The correct answer is “C.” NPH classically pres-
Answer 21.5.3 The correct answer is “A.”
ents with ementia, gait ataxia, an urinary incontinence. When
• Mil AD symptoms inclu e impaire memory, mil person-
etecte early, it respon s to ventriculoperitoneal shunting an
ality changes, an mil isorientation (MMSE 19–24) is thus a reversible cause o ementia. However, the ementia
• Mo erate AD symptoms inclu e aphasia, apraxia, insomnia, is rarely ully reversible. Gait abnormalities typically occur rst
an increasing con usion (MMSE 10–19) an are the most likely to improve with removal o cerebrospi-
• Severe AD symptoms inclu e severe memory loss, motor nal ui (CSF). T e iagnosis o NPH is supporte by n ings
impairment, an loss o some basic ADLs (e.g., urinary in- on brain MRI, an it is con rme by symptom improvement
continence an ee ing i culties) (MMSE < 10) af er CSF removal. Inci entally, NPH is a misnomer since inter-
• Symptoms o terminal AD inclu e immobility, ysphagia, mittent CSF pressure elevations have a pathophysiologic role in
an increasing susceptibility to in ections the isease. “E” is a real thing, but not the right answer. Walk-
ing corpse syn rome is a rare psychiatric illness in which the
Question 21.5.4 Which o the ollowing ndings would patient is un er the elusion that he is ea . Keep rea ing to
most likely cause you to search or a diagnosis other than learn why the other oils are wrong.
AD in a patient presenting with memory impairment?
A) Paranoi behavior You’ve got this patient stuck in your head (perhaps with
B) Apraxia superglue?). On morning rounds in the hospital, your col-
C) Poor ju gment league asks your opinion on a patient with rontotemporal
D) Aphasia an personality changes dementia (F D).
E) Bra ykinesia an rigi ity
Question 21.5.6 You think it is unlikely that your patient has
Answer 21.5.4 The correct answer is “E.” Bra ykinesia an FTD because she does not have:
rigi ity are eatures o parkinsonism, which, in the setting o A) Depression
memory loss, shoul prompt consi eration o Lewy bo y B) Disinhibition
ementia or Parkinson isease. Paranoi behavior, elusions, C) Hemiplegia
an hallucinations can all occur with more severe AD. Apha- D) Rapi ly progressing ementia
sia, apraxia, an personality changes typically occur later in AD E) remors an hallucinations
CHAPTER 21 • CARE OF THE OLDER PATIENT 583

Answer 21.5.6 The correct answer is “B.” F Ds (inclu ing Pick TABLE 21-3 LABORATORY EVALUATION OF DEMENTIA
isease) constitute a heterogeneous group o neuro egenerative
Required Minimum Testing
isor ers that have the common pathologic n ing o cortical
• Complete blood count
egeneration in rontal areas o the brain. ypical eatures o • Serum glucose and electrolytes
these ementias inclu e an insi ious onset an a slowly progres- • Vitamin B12
sive course. Patients have impairments in ju gment an insight. • Renal unction tests
T ey are isinhibite an socially inappropriate. Patients may • Liver unction tests
• Thyroid unction tests
present with anxiety, epression, elusions, or emotional in i -
erence. “A” is incorrect. Depression requently coexists with Testing Based on Clinical Suspicion
many types o ementia but oes not e ne one particular type. • Neuroimaging (recommended or all patients by some societies)
“C” is incorrect; the presence o hemiplegia in a patient with • Urinalysis
• Urine toxicology screen
ementia shoul bring to min vascular causes. “D” is incorrect
• HIV antigen/antibody assay
because rapi ly progressing ementia is the hallmark o prion • CSF analysis
isease, such as Creutz el t–Jakob isease. “E” is incorrect as • RPR or VDRL
tremors, hallucinations, an memory loss are consistent with
Lewy bo y ementia (name or its characteristic pathological
n ing—the presence o Lewy bo ies in the brain stem an cor-
tex). Clinical eatures consist o cognitive impairment, etaile Question 21.5.8 Which o the ollowing laboratory tests is
visual hallucinations, uctuation in alertness, an motor symp- NOT indicated in the initial evaluation or reversible causes
toms o parkinsonism. o dementia?
A) Cyanocobalamin (vitamin B12)
B) Liver enzymes
HELPFUL TIP:
C) CBC
AD is the most common orm o dementia, encompass-
D) CSF analysis
ing about 60% o patients with dementia. Vascular and
E) T yroi unction tests
Lewy body dementias account or about 15% to 30%. In
many cases, dementia has more than a single cause. AD
Answer 21.5.8 The correct answer is “D.” When evaluat-
and vascular dementias requently coexist—an entity
ing a newly iagnose case o ementia, one must consi er
commonly re erred to as “mixed dementia.”
in ectious, metabolic, toxic, an in ammatory etiologies.
T ere ore, the minimal require laboratory tests shoul
inclu e CBC, serum glucose an electrolytes, vitamin B12,
T e elevator is stuck on your way back rom making your an renal, liver, an thyroi unction tests. Further laboratory
rounds, giving you more time to consider dementia. tests shoul be obtaine as clinical suspicion in icates. In the
appropriate patient, one might obtain urinalysis, urine toxicol-
Question 21.5.7 Which o the ollowing is NOT consistent ogy screen, HIV antibo y assay, an CSF analysis. Because o
with the diagnosis o vascular dementia? the extremely low inci ence o neurosyphilis in mo ern times,
A) Diabetes routine testing or syphilis is no longer require but shoul
B) obacco use be consi ere in the appropriate setting. Neuroimaging is not
C) Di use slowing or normal electroencephalogram (EEG) a require part o every workup but may be help ul in some
D) Normal brain MRI patients. See able 21-3.
E) History o caroti en arterectomy
Blood chemistries, blood counts, thyroid hormone levels,
Answer 21.5.7 The correct answer is “D.” A normal MRI vitamin B12 level, and liver enzymes are in the normal range.
essentially rules out vascular ementia. Features suggestive o A noncontrast C scan o the brain shows nonspeci c “age-
vascular ementia inclu e a stepwise eterioration in cognitive related” changes (that was help ul!). T e patient and her am-
unction, onset o cognitive impairment with stroke, in arcts ily return to discuss the test results. You begin to educate
an white matter changes on neuroimaging, an ocal neuro- them about AD and dementia in general. T e two daughters
logic n ings on examination. T ere are no well- e ne cri- are concerned that other amily members may be at risk or
teria or clinically iagnosing vascular ementia, an available developing AD.
rating scales have poor pre ictive value when compare with
autopsy as the iagnostic stan ar . Known vascular isease Question 21.5.9 Which o the ollowing is the strongest risk
(like caroti artery isease) an vascular risk actors, such as actor or developing AD?
iabetes, hypertension, an smoking, support the iagnosis. A) Age
B) Apolipoprotein E 4 (APOE 4) allele
Finally back in your o ce with co ee in hand, you decide to C) Family history
evaluate or reversible causes or this patient’s dementia, and D) Hea trauma
you consider ordering laboratory tests. E) Low e ucational level
584 FAMILY MEDICINE EXAMINATION & BOARD REVIEW

Answer 21.5.9 The correct answer is “A.” As with many is- (NMDA) antagonist use to treat mo erate-to-severe AD.
eases, age is the greatest risk actor or eveloping AD. Among NMDA antagonists an cholinesterase inhibitors are of en pre-
persons 65 to 69 years ol , the inci ence o AD is 1%. In persons scribe in combination; however, there seems to be no bene t
85 years an ol er, the inci ence rises to 8%. All o the other to combining these rugs. Memantine appears to o nothing
answer options are associate with an increase risk o AD but or mil ementia an has shown very minimal (some might
not to the same egree as age. say clinically insigni cant) i erence in mo erate-to-severe
Family history is another actor strongly associate with ementia.
eveloping AD. By age 90, almost hal o persons with rst-
egree relatives with AD evelop the isease. T ere are genetic
HELPFUL TIP:
risk actors as well. Mutations on chromosomes 1, 14, an 21
All o the cholinesterase inhibitors have similar e icacy.
are known risk actors or AD. risomy 21 is a risk actor or
Tacrine is known to cause hepatotoxicity and is rarely
eveloping AD at an earlier age (of en by age 50). APOE 4 allele
used. The choice o cholinesterase inhibitor depends on
increases risk an ecreases age-o -onset o AD in a ose-
cost, patient acceptance, and physician experience.
relate ashion, with the greatest risk present in persons homo-
zygous or APOE 4.
Other potential risk actors inclu e a history o hea trauma,
lower e ucational achievement, emale gen er, an epres- Using your avorite clinical decision aid, the Ouija board, you
sion. Postmenopausal estrogens may actually increase the risk decide to start the patient on a cholinesterase inhibitor.
o ementia. Hypertension, iabetes, an hyperlipi emia are
associate with ementia, an controlling these iseases might Question 21.5.11 In your discussion about the medication,
re uce the risk o eveloping ementia in the uture, but the you tell the patient and her amily:
evi ence is not strong. Increase physical, mental, an social A) “T ese rugs are in icate or treating all types o ementia”
activities may re uce cognitive ecline in later years. B) “T ese rugs o er no bene t in mo erate Alzheimer
ementia”
C) “T ese rugs are proven to reverse memory loss”
Question 21.5.10 The patient and amily ask about medi-
D) “T ese rugs are proven to re uce mortality”
cations to treat AD. Which o the ollowing statements is
E) “Gastrointestinal intolerance is one o the most common
TRUE?
si e e ects o these rugs”
A) All stu ies show that vitamin E supplementation improves
cognition an prevents urther neuron loss in AD.
Answer 21.5.11 The correct answer is “E.” T ere is no short-
B) Ginkgo biloba an cholinesterase inhibitors have a synergis-
age o controversy when it comes to me ications or emen-
tic e ect, improving cognition in AD.
tia. However, the si e e ects are in isputable. Gastrointestinal
C) Cholinesterase inhibitors o not prevent neuron loss in AD.
intolerance—with nausea, anorexia, an iarrhea—is the most
D) Cholinesterase inhibitors maintain cognition at baseline
common. Also, cholinesterase inhibitors have a “vagotonic”
levels or 2 years af er initiation o therapy; af er that time,
action, which can cause bra ycar ia an syncope an worsen
patients ecline slowly.
car iac con uction abnormalities. o minimize a verse events,
E) Memantine is consi ere rst-line therapy or mil cogni-
the ose o cholinesterase inhibitor shoul be increase only
tive impairment an early ementia.
af er the patient has been on a stable ose or 4 to 6 weeks.
T ere are statistical i erences in the outcomes measure
Answer 21.5.10 The correct answer is “C.” Cholinesterase or AD patients on cholinesterase inhibitors, but are these
inhibitors o not prevent neuron loss. Results with vitamin E changes clinically signi cant? T ere is no i erence in per-
have been inconsistent, an some stu ies have oun a slightly ormance o ADLs, time to nursing home placement, etc. “A”
higher risk o eath in those on high- ose vitamin E (400 IU/ is incorrect. Mostly, these rugs are use in AD. T eir use in
ay), primarily in those with coronary artery isease. Given Lewy bo y an vascular ementia is o -label but may be worth
the low cost an potential bene ts o vitamin E, it may still be a try; there is some ata to support cholinesterase inhibitors or
reasonable to use in combination with a cholinesterase inhibi- these patients. However, there is no evi ence to support their
tor in AD at a ose o < 400 IU/ ay i the patient or amily is use in F Ds (e.g., Pick isease). “B” is not true. Most stu ies o
so incline . T ere is no evi ence to support the use o ginkgo cognitive e ects o cholinesterase inhibitors have occurre in
biloba in AD. As o 2015, there are just two classes o rugs FDA mil -to-mo erate AD (MMSE 10–24). “C” an “D” are incor-
approve or AD. Cholinesterase inhibitors (e.g., onepezil, riv- rect. Compare with placebo, cholinesterase inhibitors elay
astigmine, galantamine, an tacrine) represent the larger class urther cognitive an unctional ecline but neither reverse
o available pharmacotherapy use to treat mil -to-mo erate ementia nor a ect mortality. In cholinesterase inhibitor stu -
AD. Stu ies suggest that cognitive ecline may stabilize or 3 ies o mil -to-mo erate ementia, there is typically a 3-point
to 6 months af er which there is stea y loss o cognition. By 9 i erence on the MMSE between treatment an placebo
to 12 months, there is no i erence in ecline between those groups at 6 months. T is n ing is ue to a loss o thinking
on therapy an those on placebo. T e other class has only one abilities in the placebo group an a elay in that loss in the
me ication, memantine, which is an N-methyl d-aspartate treatment group.
CHAPTER 21 • CARE OF THE OLDER PATIENT 585

TABLE 21-4 MEDICATION MANAGEMENT FOR BEHAVIORAL


SYMPTOMS OF DEMENTIA
Behavioral Subtype Acute Management Long-Term Management
Psychosis Conventional high potency Risperidone, CHAP
Antipsychotic (CHAP)a

Anxiety Benzodiazepines Buspirone

Insomnia Trazodone Trazodone

Sundowning Trazodone; consider CHAP, Trazodone; consider CHAP, risperidone,


risperidone, olanzapine olanzapine

Aggression, severe CHAP, risperidone Divalproex, risperidone, CHAP

Aggression, mild Trazodone Divalproex, SSRIs, trazodone, buspirone


a
CHAP includes haloperidol, perphenazine, and uphenazine. For elderly patients with dementia, typical
doses should be about one quarter o the usual dose (e.g., risperidone 0.25 mg, olanzapine 2.5 mg, or
quetiapine 25 mg, and haloperidol 0.25–0.5 mg)
Note: Recent data suggests tha quetiapine is the sa est option or an antipsychotic in this group with haloperidol
being associated with the greatest increase in stroke and mortality risk (JAMAPsych 2015, March 18).

activities may acilitate a better sleep–wake cycle. A ult ay care


HELPFUL TIP:
programs exist that specialize in ay care or el erly people inclu -
Not every con used elderly person should be put on a
ing patients with ementia. A ult ay care can provi e structure
cholinesterase inhibitor. Consider the diagnosis, sever-
activities uring the ay, along with respite or the aughter who
ity o disease, and the goals or the patient and amily.
is obviously asking or extra support. Although me ications are
Determine why you are starting the drug and be clear
sometimes nee e , “C” an “D” are incorrect or initial treat-
on the goals you hope to achieve. Then be willing to dis-
ment in this case. Once environmental changes have aile or
continue it i your patient is not reaching those goals.
there are other imme iate health risks involve , then me ica-
The side e ects o cholinesterase inhibitors are symp-
tions may be necessary. Antipsychotics currently o er the only
toms o ten seen in nursing home patients (e.g., alls due
rug treatment or behavioral symptoms in ementia; however,
to bradycardia and weight loss rom anorexia). I your
there are no great choices. Haloperi ol, risperi one, an olan-
patient is losing weight and/or alling, consider discon-
zapine are use most of en. See able 21-4 or selecte me ica-
tinuing the cholinesterase inhibitor.
tions use to treat behavioral symptoms in ementia. Se atives,
such as benzo iazepines, of en result in para oxical agitation
One year later, the patient returns with her daughter, with in el erly patients with ementia. “A” is incorrect. Restraints
whom she now lives. T e daughter reports disturbing symp- shoul be avoi e in most cases, even sof restraints. Although
toms that occur nightly. T e patient wakes up in the middle they are sometimes require to prevent harm to the patient or
o the night and wanders the house, becoming con used and caretakers, restraints are known to result in worsene agitation
agitated. With a subtle nod toward her mother, the daughter an an increase risk o all, injury, an rhab omyolysis.
states, “I just can’t take much more o this.”
HELPFUL TIP:
Question 21.5.12 A ter inquiring about pain and any changes
When patients with Lewy body dementia receive anti-
in health status and nding none, your initial recommenda-
psychotic medication or hallucinations, parkinsonian
tion is to:
eatures become much more pronounced. I possible,
A) Employ sof restraints only uring the night
avoid antipsychotics in these patients.
B) Consi er environmental changes inclu ing more aytime
structure activities through an a ult ay care center
C) Initiate an antipsychotic be ore be time
D) Initiate a se ative–hypnotic be ore be time Despite the addition o adult day programming, medication
became necessary. Haloperidol, or “vitamin H,” nightly has
Answer 21.5.12 The correct answer is “B.” reating behavioral resolved the agitation. Although you may have increased your
issues in patients with AD can be very challenging. Further his- patient’s risk o dying (as seems to occur when antipsychot-
tory must explore the possibility o pain-relate agitation, ecline ics are used in dementia), her daughter is thrilled with the
in comorbi con itions or new health con itions, such as occult result. T ree months later she returns with concerns about
in ection, an any me ication changes that may be playing a role. depression. T e patient spontaneously cries several times per
I a treatable cause is not i enti e , then environmental change day, her appetite is poor, and she has no desire to leave the
is the best initial recommen ation. A ing structure aytime house or even get dressed most days.
586 FAMILY MEDICINE EXAMINATION & BOARD REVIEW

Question 21.5.13 Since a pill worked last time, her daugh- means or provi ing nutrition an can lea to rewar ing inter-
ter wants to know what antidepressant is most e ective or actions or both the patient an the caregiver.
depression in patients with dementia?
A) Citalopram
HELPFUL TIP:
B) Mirtazapine
Remember the caregivers! Ask about their health and
C) Sertraline
mood. Twenty- ive percent o caregivers to the elderly
D) None o the above
are depressed, while older people caring or their dis-
Answer 21.5.13 The correct answer is “D.” T ere are very ew abled spouses have a 63% higher chance o dying than
quality stu ies available to gui e treatment o epression in noncaregivers o the same age.
patients with ementia. T e available evi ence shows no i -
erence between anti epressant therapy an placebo. T e iag- Objectives: Did you learn to . . .
nosis o epression in a patient with ementia is complicate , • Identi y symptoms, signs, and diagnostic criteria or demen-
since ementia causes apathy, sleep isturbance, appetite loss, tia?
an social with rawal. I epression is suspecte in a patient • Describe dif erent types o dementia and how they are diag-
with ementia, a pru ent approach woul be to employ non- nosed?
pharmacologic therapy an then provi e an empiric trial o an • Evaluate the patient with dementia, considering the poten-
anti epressant. tial causes o dementia?
• Describe potential bene ts and limitations o current phar-
Over time, as the patient’s dementia progresses, you reevaluate macologic therapy or AD?
end-o -li e issues and advance directives. With the support o • Describe the natural course o AD?
her amily, the patient decides not to have cardiopulmonary • Manage a patient with end-stage AD?
resuscitation.

Question 21.5.14 In end-stage AD, which o the ollowing


CASE 21.6
is correct? A 71-year-old male whom you have known since starting
A) Malnutrition is the most common cause o eath in patients your practice recently su ered a stroke, resulting in language
with severe ementia de cits and right hemiparesis. His medical history is signi -
B) Hospitalization or pneumonia in patients with severe cant or hypertension, hyperlipidemia, ulcer requiring par-
ementia improves morbi ity an mortality tial gastrectomy (remote), and coronary artery disease. He
C) In severe ementia, gastrostomy tube ee ing prevents aspi- quit tobacco and alcohol 5 years ago. He is retired and wid-
ration owed. A er a 3-day hospitalization, he appears stable enough
D) o increase com ort, ehy rate patients with severe or discharge. His medications include aspirin, metoprolol,
ementia shoul receive IV hy ration lisinopril, and atorvastatin. Prior to entering a nursing home
E) In a vance AD, treatment o in ections with oral an IV to receive skilled nursing care and therapies, the patient
antibiotics is equally e cacious wants to know who will pay or the services. He has Medicare
parts A and B.
Answer 21.5.14 The correct answer is “E.” Hospitalization or
emente patients with pneumonia is a wash. T e number o Question 21.6.1 You are able to assure him:
patients save by the use o IV antibiotics is o set by an increase A) Me icare will cover all expenses in e nitely regar less o
in eath an unctional eterioration as a result o the hospi- personal nancial resources
talization. T us, on balance, oral an IV antibiotics are equally B) Me icai will cover all o the expenses or the rst 100 ays
e cacious in the treatment o in ections in these patients; there- o skille care regar less o personal nancial resources
ore, severely homeboun patients with ementia or nursing C) Me icare will cover part o the expenses or the rst 100
home resi ents shoul be treate in their usual environment ays o skille care regar less o personal nancial resources
rather than hospitalize i the amily agrees. “A” is incorrect. T e D) Me icare requires a hospital stay o 7 ays or longer prior to
majority o patients with ementia ie o in ection, not malnu- entering a nursing home or skille care
trition. “B” is incorrect as note above. “C” is incorrect. Even E) Me icai an Me icare o not cover nursing home expens-
in mo erate-to-severe AD, ee ing tubes can be use ul in the es un er any circumstances
acute setting. But the tube shoul be remove an natural ee -
ing resume as soon as the acute event passes. Permanent gas- Answer 21.6.1 The correct answer is “C.” Me icare Part A,
trostomy tube eeding is not recommended in patients with which provi es some healthcare or patients 65 years an ol er
severe or terminal dementia. ube eeding does not prolong i they quali y or Social Security bene ts, will pay all costs or
li e, prevent aspiration, or promote weight gain in advanced skille care or the rst 20 days an part o the costs therea -
dementia. Although many patients with a vance ementia are ter up to a total o 100 ays per calen ar year. T is Me icare
malnourishe an ehy rate , these con itions o not appear bene t inclu es rehabilitation (e.g., physical therapy, occupa-
to cause iscom ort. Han ee ing is as e ective as any other tional therapy, an speech therapy) an skille nursing care
CHAPTER 21 • CARE OF THE OLDER PATIENT 587

(e.g., nursing home, skille care acility, an rehabilitation hos- Question 21.6.3 And then you realize that:
pital) af er a hospital stay o at least 3 ays. T is bene t is con- A) An elevate temperature in ol er persons is most of en ue
tingent upon the patient having an appropriate iagnosis an to changes in basal bo y temperature regulation
rehabilitation potential, an continuing to show improvement B) Oral antibiotics will not be su cient to treat this in ection
uring the time the bene t is in place. Me icare oes not pro- C) Antibiotics will not be necessary to treat this con ition
vi e exten e nursing home coverage. Me icai will provi e D) Absence o signi cant ever in the el erly oes not rule out
exten e nursing home care i a person’s assets an income serious bacterial in ections
are below a certain threshol , which varies rom state to state.
Note: Although Me icare Part B will pay or physician visits to Answer 21.6.3 The correct answer is “D.” Ol er persons, espe-
nursing home patients, Me icare oes not pay or nursing or cially rail el ers an nursing home patients, of en have lower
other care irectly relate to permanently living in a nursing basal bo y temperatures compare with younger persons an
home. may not mount as great a ebrile reaction to in ection. A tem-
perature > 38.1°C in a rail el er is most likely associate with a
Although he received airly intensive physical, occupational, serious bacterial or viral in ection. Absence o signi cant ever
and speech therapies, your patient does not regain much oes not rule out serious bacterial in ections. “C” is incorrect.
unction. He has only minimal movement in the right arm With the available in ormation, it is i cult to say with any cer-
and complains o pain in the right shoulder. A radiograph o tainty i the patient has an in ection treatable with antibiotics.
the right shoulder shows degenerative changes. T e nursing I he i , oral antibiotics are of en appropriate in the nursing
sta administers acetaminophen 650 mg PO when the pain home setting, even when treating pneumonia.
is severe, yet he continues to complain o shoulder pain. You
involve physical therapists in his care to reduce the risk o With a decline in his unction and a mildly elevated tem-
chronic dislocation o the shoulder. perature, you plan to evaluate this patient or in ection.
According to the nursing sta , there are no other residents
Question 21.6.2 In order to control his pain, which o the with apparent in ections. Your patient has not developed
ollowing is the most appropriate to add as a scheduled, any ocal symptoms (e.g., cough, dysuria, diarrhea, or site-
and presumably chronic, medication? speci c pain).
A) Acetaminophen
B) Oxyco one
C) Gabapentin Question 21.6.4 Which o the ollowing tests will be LEAST
D) Aspirin help ul?
E) Naproxen A) CBC
B) Urinalysis with microscopic examination
Answer 21.6.2 The correct answer is “A.” T e point here is C) Stool culture
that sche ule acetaminophen is better or chronic pain than D) Chest ra iograph
PRN osing. Acetaminophen is the sa est analgesic o those E) Bloo oxygen saturation (pulse oximetry)
liste . It may provi e su cient pain relie —i given sche ule
(e.g., 1,000 mg PO ID). I it oes not, then another me ica- Answer 21.6.4 The correct answer is “C.” He is not having
tion may be a e . “B” is incorrect as a rst step as narcotics iarrhea so a stool culture is not likely to be o bene t. T is is
can cause con usion, se ation, urinary retention, an alls, an not a black-or-white area, but there are some principles an
they have been associate with an increase mortality risk in expert opinions to ollow. Patient an amily wishes regar ing
the el erly. However, this patient has a history o an ulcer, an care must be known prior to initiating an evaluation, therapy, or
a narcotic analgesic may be a reasonable me ication to a hospital trans er. While the el erly may have a serious in ection
af er acetaminophen. “C” is incorrect. Gabapentin is in icate with only slight or even no leukocytosis, a normal WBC count
or postherpetic neuralgia an is more use ul or neuropathic on CBC will re uce suspicion or serious bacterial in ection.
pain (although not very use ul there either; tricyclics are better). Even without speci c urinary symptoms, urinalysis is recom-
“D” an “E” are incorrect. Aspirin an NSAIDs must be use men e because o the high inci ence o U I in this population.
with caution in the el erly ue to increase risks o silent GI Remember, however, that asymptomatic bacteriuria can occur
blee ing, atal GI blee ing, an ki ney injury. NSAIDs are typi- in up to 50% o women an 40% o men living in long-term
cally not rst-line agents or arthritis pain in this age group an care acilities an treating this con ers no bene t. Bloo oxygen
probably o er no greater pain relie than acetaminophen. Also, saturation below the normal range (< 90% on room air) may
consi er topical therapies or pain, inclu ing capsaicin. Non- in icate serious respiratory illness; in the setting o hypoxia, a
pharmacologic mo alities shoul be employe as well, inclu - chest ra iograph is recommen e .
ing massage, exercises, an physical therapy.
Question 21.6.5 Regarding in ectious diseases in nursing
A nurse calls to report that your patient has developed leth- home settings, which o the ollowing is correct?
argy, decreased appetite, and a temperature o 37.8◦ C. Your A) I the in uenza vaccine is a ministere within 24 hours o
rst thought is, “So? T at’s not a ever.” an outbreak, patients require no urther prophylaxis
588 FAMILY MEDICINE EXAMINATION & BOARD REVIEW

B) All resi ents who are carriers o methicillin-resistant Staph- clean-catch urinalysis shows bacteria and WBCs (2–4 WBC/
ylococcus aureus (MRSA) must be treate with appropriate hp ) but is negative or glucose and nitrites. His post-void
antibiotics residual bladder volume is 80 cc.
C) Most cases o bacteremia are cause by in ecte skin ulcers
D) New resi ents shoul receive a two-step tuberculin skin test, Question 21.6.6 Which o the ollowing is true regarding
unless positive on the rst test the potential cause o and therapeutic intervention or uri-
E) All resi ents who are carriers o Clostridium dif cile must be nary incontinence in this patient?
treate with appropriate antibiotics A) T e most likely cause is obstruction rom the benign pros-
tate hyperplasia, an an alpha-blocker will improve the
Answer 21.6.5 The correct answer is “D.” T e inci ence o incontinence
tuberculosis is relatively high in the ol er population, as is B) T e most likely cause is immobility, an sche ule voi ing
mortality rom the isease. Institutionalize el ers shoul be is in icate
screene or tuberculosis with the two-step tuberculin skin C) T e most likely cause is etrusor hyperactivity, an an
test. A two-step test involves repeating the tuberculin skin test in welling catheter is in icate
1 to 3 weeks af er an initial negative test (< 10 mm in uration). D) T e most likely cause is stress incontinence, an pelvic oor
Anergy testing is no longer recommen e . T e test is positive i muscle strengthening (Kegel exercises) is in icate
the in uration is 10 mm or more. E) T e most likely cause is bacteriuria, an antibiotics will im-
“A” is incorrect. In a nursing home, an in uenza outbreak prove the incontinence
can have evastating results, with a mortality rate up to 30%.
In the event o an outbreak, even resi ents who receive the Answer 21.6.6 The correct answer is “B.” Urinary inconti-
vaccine shoul receive antiviral prophylaxis (see Chapter 8 or nence is incre ibly requent in nursing home resi ents, with
more). Only about 30% to 50% o nursing home resi ents will a rate o up to 50%. As with many other geriatric syn romes,
evelop an a equate antibo y response to the in uenza vaccine, incontinence is of en multi actorial or the result o ecrease
an that response takes up to 2 weeks af er a ministration to physical an /or cognitive unctions. A requent cause o uri-
evelop. “B” an “E” are incorrect. Resi ents who are carriers nary incontinence in nursing home resi ents is immobility ue
o MRSA or C. dif cile will not bene t rom era ication i they to severe physical impairment, ementia, or both. In this set-
are not in ecte . In a ition, they may return to a carrier state ting, the initial treatment o choice is prompte voi ing every
quickly af er antibiotic treatment; there ore, antibiotic treat- 2 hours when the patient is awake. Also, ui an ca eine intake
ment o these carrier states is not recommen e . “C” is incor- shoul be monitore an a juste to re uce urine output with-
rect because U Is are the most common cause o bacteremia in out causing ehy ration.
nursing home resi ents. “A” is incorrect. Although there is no in ormation about
the patient’s prostate size, he has a relatively normal post-voi
resi ual volume, making over ow incontinence rom outlet
HELPFUL TIP:
obstruction less likely. A post-voi resi ual volume greater than
In elderly nursing home residents, a positive response to
300 mL woul be more concerning or urinary retention. “C”
tuberculin skin testing is most o ten due to reactivation
is incorrect—or at least the iagnosis o etrusor hyperactiv-
o old disease. Risk actors associated with reactivation
ity cannot be ma e on the basis o current in ormation. T e
o tuberculosis include chronic steroid use, diabetes,
patient has not been ully evaluate with uro ynamic tests, so it
malignancy, malnutrition, renal ailure, and chronic in-
is i cult to etermine whether his incontinence is stress-type
stitutionalization.
or urge-type. I he has urge-type incontinence ue to etrusor
hyperactivity, an in welling bla er catheter is not appropriate.
Detrusor hyperactivity can of en be treate success ully with
HELPFUL TIP:
pharmacotherapy (anticholinergic me ications). It is impor-
The high-dose in luenza vaccine (e.g., Fluzone-HD) has
tant to be min ul o the anticholinergic si e e ects, which
our times the usual dose o antigen and was approved
inclu e alls, con usion, constipation, ry mouth, an urinary
or adults 65 years by the FDA in 2009. Seroconversion
retention an coul ultimately be more etrimental than the
rates are higher or patients receiving the higher dose
incontinence.
vaccine, and in luenza-like illness rates are modestly
“D” is also not likely. Stress incontinence is less common in
lower in recipients. However, it has not shown a ben-
men than women, an urine loss is typically associate with
e it or more serious morbidity or mortality. The cost is
increase ab ominal pressure (e.g., coughing, sneezing, an
higher than that o the other options, side e ects are
lif ing). Finally, “E” is incorrect. T e presence o bacteria in the
similar, and its role is not well established.
urine is a common n ing in nursing home patients. In gen-
eral, bacteriuria without symptoms (the main symptom we look
or being ever rather than incontinence) shoul not be treate .
Over the next year, the patient has increasing di culty with Stu ies have emonstrate little or no improvement in inconti-
cognition. He begins to experience urinary incontinence sev- nence af er treating asymptomatic bacteriuria in el erly nursing
eral times per day, necessitating the use o a pad. A midstream home patients.
CHAPTER 21 • CARE OF THE OLDER PATIENT 589

Answer 21.6.8 The correct answer is “A.” In some stu ies, high
HELPFUL TIP:
rates o mistreatment have been oun in nursing homes, with
When urinary incontinence is due to obstruction or
up to 75% o nursing ai es witnessing or hearing about acts o
detrusor hypore lexia, intermittent bladder catheter-
abuse. However, it is not known whether nursing home resi-
ization should be employed and chronic indwelling
ents are at greater risk than community- welling epen ent
catheters avoided. Appropriate indications or chronic
el ers. “B” is incorrect. T e e nition o “ epen ent el er” is
indwelling bladder catheters in the nursing home in-
not consistent, an may apply to el ers who are cognitively
clude com ort care o the terminally ill, presence o skin
impaire , physically ebilitate , or nancially epen ent. Nurs-
wounds contaminated by incontinent urine, and urine
ing home resi ents are requently epen ent; however, living in
retention not practically managed with intermittent
a nursing home itsel is not su cient to establish that a person
catheterization.
is a epen ent el er. “C” an “D” are incorrect. T e stu y o
el er abuse an neglect (also re erre to as el er mistreatment)
su ers rom lack o a universally accepte e nition, varia-
Although his urinary incontinence improves, your patient tions in laws between states, an inherent i culties in obtain-
develops di culty with loose stools and occasional ecal ing accurate reports o abuse. T ere ore, attempts to etermine
incontinence. T e stools are quite watery with no blood or the inci ence o el er abuse an neglect have resulte in wi e
melena. Aside rom occasionally abdominal cramping, he variations. “E” is incorrect. El er abuse may inclu e physical
eels well. He has no new neurologic symptoms. harm, sexual abuse, psychological abuse, neglect, or nancial
exploitation. Although all states now have laws a ressing el er
Question 21.6.7 Which o the ollowing is the most likely mistreatment, those laws vary between states, an healthcare
cause o ecal incontinence in this situation? provi ers are encourage to know the laws in their area.
A) In ectious iarrhea
B) Ulcerative colitis HELPFUL TIP:
C) Decrease anal sphincter tone Risk actors or elder mistreatment include older age,
D) Fecal impaction cognitive impairment, substance abuse, low socioeco-
nomic standing, minority status, and caregiver stress
Answer 21.6.7 The correct answer is “D.” In nursing home (probably the most important).
resi ents with limite physical mobility, over ow incontinence
ue to ecal impaction is most likely. Nursing home patients
are of en taking me ications that contribute to constipation as As your patient’s cognitive impairment progresses, he
well. Even with a stool sof ener an /or a laxative, constipation becomes more withdrawn and uncooperative with nursing
may still result. A ecal impaction can be treate with an enema, care, such as bathing. A nurse calls to ask, “Shouldn’t he be
stool sof eners, laxatives, an ietary changes, but sometimes on risperidone or something to improve his behavior?”
requires manual isimpaction (where’s a me ical stu ent when
you nee him?). Decrease sphincter tone may occur as a result Question 21.6.9 According to the Omnibus Budget Recon-
o neurologic insult, but this patient was previously continent o ciliation Act o 1987 (OBRA), antipsychotic medication is
stool. Although in ection might be causing incontinence, there indicated or demented patients with:
are no other symptoms o in ection. T e onset o in ammatory A) Repetitive, bothersome behavior (e.g., name calling)
bowel isease is usually seen in younger populations, of en with B) Continuous crying out an screaming
bloo in the stools, making ulcerative colitis less likely. C) Uncooperative behavior (e.g., re using to eat an bath)
D) All o the above
As a result o your patient reporting physical abuse o another
patient by the nursing sta , an investigation is under way in Answer 21.6.9 The correct answer is “B.” One goal o OBRA
the nursing home. was to ecrease the inappropriate use o antipsychotic me i-
cations in nursing home resi ents. In patients with ementia,
Question 21.6.8 Ref ecting on elder abuse and neglect, you antipsychotic me ications may be appropriately a ministere
realize which o the ollowing is true? in the ollowing settings: agitate , belligerent acts that present
A) Up to 75% o nursing ai es in nursing homes have seen or a anger to the patient or other resi ents; psychotic symptoms
hear o a resi ent being abuse or neglecte ( elusions, hallucinations, paranoia); an continuous crying out
B) A “ epen ent el er” is e ne as anyone living in a nursing an screaming (lasting 24 hours or longer). Attempts to re irect
home the patient shoul always be employe rst. You shoul also
C) Approximately 70% o el er abuse an neglect occurs in attempt to uncover occult causes o agitation, such as in ection
nursing homes or pain. Once behavior control is attaine , assess whether the
D) T ere is a universally accepte e nition o el er abuse an antipsychotic can be re uce in ose or iscontinue . Accor -
neglect, which is co i e in e eral law ing to OBRA, inappropriate in ications or antipsychotic me -
E) El er abuse is wi ely e ne as “purpose ul physical harm ication inclu e restlessness, uncooperative behavior, poor sel -
o anyone over the age o 65 years” care, or repetitive an bothersome actions.
590 FAMILY MEDICINE EXAMINATION & BOARD REVIEW

Objectives: Did you learn to . . . apartment in the same town as his daughter. He stopped
• Describe some common Medicare/Medicaid reimbursement smoking 30 years ago, but continues to drink alcohol. He
issues or nursing home care? denies any problems related to his drinking, but you inquire
• Manage chronic pain in the nursing home? anyway and ask speci cally how much he drinks. His rou-
• Describe an appropriate evaluation or the nursing home tine includes three shots o whiskey per day. He says he likes
resident with ever? drinking one shot be ore his daily walk and the other two
• Recognize in ectious disease issues commonly presenting in shots a er he returns. He nds the routine very motivating
the nursing home? and keeps him in shape.
• Develop an appropriate strategy or the evaluation and man-
agement o urinary and ecal incontinence in nursing home Question 21.7.1 In regard to his drinking behavior, you are
residents? aware that:
• Recognize the impact o elder abuse and neglect? Imple- A) Ol er a ults accumulate signi cantly lower bloo alcohol
ment appropriate measures or agitated behavior in the levels than younger a ults ue to ecrease absorption
nursing home? B) Drinking three shots per ay shoul not be any concern as
long as his liver unction tests are normal
C) Alcohol consumption can re uce the availability o nutri-
QUICK QUIZ: GERIATRIC PHARMACOTHERAPY ents such as zinc, vitamins A, B1, B2, B6, B12, an olate
D) T e li etime prevalence o alcoholism or men age ≥ 65 is
Which o the ollowing is true about rug therapy in the el erly? less than 5%, but shoul still be screene or routinely
A) GI absorption is substantially ecrease in the el erly E) T e stan ar screening CAGE questionnaire or rinking
B) Se ative–hypnotic rugs shoul be given an 8-week trial problem behavior has not been vali ate in ol er a ults
without interruption or anxiety in the el erly
C) Mirtazapine (Remeron) causes anorexia an weight loss in Answer 21.7.1 The correct answer is “C.” Alcohol consump-
the el erly tion can re uce the availability o nutrients such as vitamins
D) Compare with young a ults, the volume o istribution o A, B1, B2, B6, B12, zinc, an olate. Alcoholic patients of en
at-soluble rugs is increase in the el erly present with malnutrition, poor sel -care, an alcohol-relate
illnesses such as anemia, peptic ulcer isease, iabetes, hyper-
The correct answer is “D.” T e volume o istribution o at- tension, liver isease, neuropathy, an cognitive impairment.
soluble rugs is relatively increase in the el erly ue to a loss Checking or e ciencies may be warrante in this situation
o muscle mass an proportionately more at mass. T ere ore, epen ing on your patient’s other ietary intake. “A” is incor-
at-soluble rugs, like iazepam, have a greater relative volume rect. Ol er a ults accumulate signi cantly higher bloo alcohol
o istribution, while water-soluble rugs, like alcohols, will levels than younger a ults. A young a ult’s bloo alcohol level
have a relatively smaller volume o istribution. “A” is incorrect will be approximately 0.03% af er “one rink” (1.5 oz o istille
because rug absorption oes not change substantially with liquor, 5 oz o wine, or 12 oz o beer); whereas in a 75-year-ol ,
aging. “B” is incorrect. Se ative–hypnotic rugs shoul be use the level may rise as high as 0.08%, which is the legal limit or
only or short-term therapy o 2 to 4 weeks because o the risk o intoxication in many states. “B” is incorrect. Drinking behavior
alls an other a verse e ects; this is true or both community- should be questioned regardless o liver unction tests.
welling el ers an those in nursing homes. “C” is incorrect T e National Institute on Alcohol Abuse an Alcoholism
because mirtazapine can actually increase appetite an lea to has i enti e rinking more than one alcoholic beverage per
weight gain in the el erly. For this reason, it can be use ul in ay as potential problem rinking in ol er a ults. “D” is incor-
patients who are epresse an not eating well. rect. T e li etime prevalence o alcoholism or men age ≥ 65
is higher, at approximately 14%; while women age ≥ 65 have a
HELPFUL TIP: prevalence o 1.5%. Denial o the problem is more requent in
Although hepatic drug metabolism does not change ol er patients, an impairments in unctioning relate to alco-
substantially with age, drugs tend to have decreased hol use may not be recognize until serious complications arise.
elimination in the elderly as a result o decreased renal “E” is incorrect. Diagnosis o alcohol abuse an epen ence in
unction. For these reasons, the hal -li e o many seda- ol er a ults is challenging. Brie screening tools such as the CAGE
tive–hypnotic drugs is substantially increased in the questionnaire ( able 21-5) have been vali ate in the ol er popu-
elderly. lation an coul be use in this situation. A positive response to
any CAGE questions suggests rinking problem.

In this case, your patient denies any drinking problems and


CASE 21.7 his CAGE screen is negative. Still, you advise him to cut back
Your next patient is an 83-year-old male amiliar to your to one drink per day or less and plan to ollow up. As you
clinic, who presents or routine care. He has hyperten- review the patient’s basic ADLs and instrumental activities o
sion, hyperlipidemia, and osteoarthritis. He has been wid- daily living (IADLs), he is independent in all basic ADLs and
owed or 10 years and continues to live independently in an most IADLs. He walks daily, eats three small meals per day,
CHAPTER 21 • CARE OF THE OLDER PATIENT 591

TABLE 21-5 CAGE SCREENING TOOL FOR ber o ol er rivers, their higher crash rate per mile riven, an
PROBLEM DRINKING their increase likelihoo o serious injury an eath. However,
most seniors pre er automobile transportation to keep active in
C: Have you ever elt you should Cut down?
the community an shoul continue to rive. In this case, with-
A: Does others’criticism o your drinking Annoy you? out speci c concerns, there is no nee to recommen restricte
riving.
G: Have you ever elt Guilty about your drinking?

E: Have you ever had an “Eye opener” to steady your nerves or get rid
HELPFUL TIP:
o a hangover?
I you are concerned about driving sa ety, ask direct
Positive response to any suggests drinking problem; questionnaire has questions about any recent driving problems, such
been validated in older population. as minor accidents, tra ic violations, or getting lost.
The legal requirements about physician reporting o
and maintains a steady weight around 180 lb (BMI = 25 kg/m2). unsa e older drivers vary rom state to state, but the
He manages his own nances, prepares his own meals, but AMA has published a use ul guide, available online and
usually has his daughter do the grocery shopping or his con- updated in 2010, called “Physician’s Guide to Assess-
venience. When you ask about transportation, he reports that ing and Counseling Older Drivers” (http://www.nhtsa.
he mostly drives to get around town to the gol club, social gov/people/injury/olddrive/OlderDriversBook/pages/
events, and the post o ce when needed. He denies any vision Contents.html)
or hearing problems.
Just as a review (this is a review book, a er all) . . .
One month later, your patient’s daughter calls to in orm you
Basic activities o daily living (ADLs) include: bathing, ress- that he ell and broke his le hip while vacationing with the
ing, toileting, maintaining continence, grooming, ee ing, an amily in Cali ornia. He had total hip arthroplasty ( HA)
trans erring. and is still in the hospital recovering. Now they are trying to
make arrangements to bring him home. T e discharge plan-
Instrumental activities o daily living (IADL) include: shop- ner has identi ed a local nursing home that can provide reha-
ping or groceries, riving or using public transport, using the bilitation, but some o the amily would like or him to return
telephone, per orming housework, oing home repairs (not to his apartment.
climbing on the roo . . . changing light bulbs an such), pre-
paring meals, oing laun ry, taking me ications, han ling Question 21.7.3 In regard to recovery a ter hip surgery, you
nances. in orm the daughter that:
A) Rehabilitation af er hospital ischarge results in better out-
Question 21.7.2 In order to keep him healthy, sa e, and comes or patients with hip racture
unctional, you appropriately recommend: B) Rehabilitation can only be provi e inpatient at a hospital or
A) In oor walking only nursing home, not at home
B) Weight loss C) Surgical repair in el erly patients shoul be elaye i pos-
C) Perio ic screening or hearing impairment sible (> 72 hours af er injury) to re uce 1-year mortality an
D) Restricte riving other complications
E) Intensive weight-lif ing an aerobic workouts D) Early mobilization af er hip surgery is recommen e in
younger patients, but in ol er patients weight bearing is
Answer 21.7.2 The correct answer is “C.” T e Institute o Me - usually elaye at least 5 ays af er surgery to allow proper
icine, recommen s au iometric testing once between ages 40 to healing
59, 60 to 74, an 75 an over. In this case, asking about hearing
i culty an testing with a whispere -voice out o the el o Answer 21.7.3 The correct answer is “A.” Stu ies show that
vision perio ically is a reasonable approach or hearing screen- rehabilitation imme iately af er hospital ischarge appears to
ing. However, the USPS F updated its recommendations or result in superior outcomes or patients with hip racture or
o ce screening in 2012 and concluded that current evidence stroke. “B” is incorrect. Rehabilitation can be provi e in either
is insu cient or audiometric o ce-based testing or hear- inpatient (i.e., hospital or skille nursing acility) or outpa-
ing loss in asymptomatic older adults. “A” an “E” are incor- tient settings (clinic, ay hospital or home). For inpatient care,
rect. Daily exercise inclu ing walking in oors or out oors in patients must be able to participate in rehabilitation that inclu es
sa e environments (weather permitting) shoul be encourage a minimum o 3 hours o therapy 5 ays per week. Care usually
in all age groups, but intensive training oes not appear to a involves an inter isciplinary team inclu ing nurses an various
extra bene t to unction or overall health. “B” is incorrect. His therapists. Home-base services can provi e part-time or inter-
current weight is a equate, an he is at risk or malnutrition i mittent therapy as prescribe by a physician. “C” is incorrect.
he has rinking problem, so he oes not nee to lose weight. Early surgical repair (< 24 hours af er racture) is i eal an has
“D” is incorrect. Sa ety is a concern ue to the increasing num- been shown to re uce 1-year mortality an complications such
592 FAMILY MEDICINE EXAMINATION & BOARD REVIEW

as pressure ulcers an elirium. Delay or me ically unstable ractures carry approximately 5% in-hospital mortality an a
patients may be necessary. “D” is incorrect. Early mobilization mortality o approximately 25% in the year ollowing racture,
is the stan ar o care or both hip an knee arthroplasty in about 75% o survivors recover to prior level o unction. Up to
younger an ol er a ults. Weight bearing of en begins on the 50% o these patients require an assistive evice, but certainly
secon postoperative ay. not everyone.

Your patient and his daughter agree that rehabilitation


locally sounds like the best plan. HELPFUL (AND REALLY IMPORTANT) TIP:
For hip ractures, repair using an anterior approach as
Question 21.7.4 The goals o rehabilitation include: opposed to the traditional posterior/lateral approach
A) Restore unction reduces recovery time and generally allows complete
B) Help patients compensate or an a apt to unctional losses mobility (including crossing legs, etc.) with a lower risk
C) Prevent secon ary complications o dislocation. They are o ten ambulating well the irst
D) Maximize potential or participation in social, leisure, or postoperative day.
work roles
E) All o the above

Answer 21.7.4 The correct answer is “E.” T ese are all goals o A er 2 more weeks, your patient is now unctioning well
rehabilitation. enough to return home. He can trans er independently and
ambulates with a cane or support. Prior to discharge, the
T e patient returns to the local nursing home or inpatient rehabilitation team would like to assess his home environ-
rehabilitation. Despite wonder ul progress with physical ment.
therapy, he is unable to ambulate without using a cane. He is
rustrated that he cannot walk on his own, and is concerned Question 21.7.6 The occupational therapy practitioner on
that all this walking and exercise is going to damage the the team:
recently surgically repaired hip joint. A) Provi es a comprehensive assessment wherever the patient
is employe base on his/her occupation, which oes not
typically inclu e the home environment
Question 21.7.5 You can tell him that:
B) May provi e training or speci c a aptive equipment or
A) He oes not nee to restrict his activity because the hip
patients to enhance per ormance in every ay activities an
prosthesis is well esigne or ben ing, walking, an climb-
promote in epen ence
ing stairs
C) Is a skille pro essional who has complete an occupational
B) His rustration is likely a major epressive isor er an will
therapy training program af er completion o high school
require me ication treatment
D) Is license to write prescriptions in most states, primarily or
C) He shoul continue the exercises because the a vantages
pain control
outweigh the low risks o surgical ailure
D) He shoul not expect ull recovery even with exercise, be-
Answer 21.7.6 The correct answer is “B.” Occupational thera-
cause nearly every patient requires an assistive evice to
pists (O s) provi e training or speci c a aptive equipment to
walk af er total hip replacement
enhance per ormance in every ay activities an promote in e-
pen ence. T ey also provi e gui ance to amily members an
Answer 21.7.5 The correct answer is “C.” Whether correc- caregivers i nee e . “A” is incorrect. O s provi e home or job-
tion is with screws, partial repair, or complete joint replace- site assessment, regar less o employment status or occupation.
ment, early weight bearing is usually tolerable with low rates “C” is incorrect. O s are skille pro essionals whose e ucation
o surgical ailure an helps to counteract the poor outcomes inclu es the stu y o human growth an evelopment with an
clearly associate with prolonge inactivity. As note in a pre- emphasis on the social, emotional, an physiological e ects o
vious question, early mobilization an continue exercise are illness an injury. One must have earne a bachelor egree or
the keys to preventing urther ecline an loss o unction. “A” beyon to enter the el o occupational therapy. T ere are also
is incorrect. Af er total hip replacement, patients shoul avoi occupational therapy assistants who generally earn an associate
certain motions such as ben ing over to tie shoes an cross- egree an practice un er the supervision o a traine O . “D”
ing legs when seate . Of entimes a raise toilet seat is also is incorrect since O s o not have license to prescribe me ica-
recommen e to re uce the loa place on the hip prosthe- tions.
sis in extreme exion. Walking an general range o motion
exercises shoul be encourage as tolerate . “B” is incorrect. Objectives: Did you learn to . . .
Depression is not uncommon af er a isabling injury such as • Identi y and screen or drinking problem in the older
hip racture. However, this patient’s rustration may or may patient?
not re ect clinical epression an shoul be urther evalu- • Promote early rehabilitation a ter hip racture repair?
ate be ore starting me ication. “D” is incorrect. Although hip • Describe some aspects o rehabilitative services?
CHAPTER 21 • CARE OF THE OLDER PATIENT 593

have ever smoke . “B” is incorrect. Rupture o an AAA is associ-


CASE 21.8 ate with an even higher mortality rate o 80%, hence the impor-
A 68-year-old male arrives at your clinic to establish care. tance o screening. “C” is incorrect. Epi emiologic stu ies in i-
He admits that he does not visit the doctor regularly, but he cate that AAA ruptures cause approximately 10,000 to 15,000
eels his health has been pretty good since he changed his eaths per year in the Unite States. “D” is incorrect. reatment
“bad habits.” He did not bring any records, but he knows o AAA is base on the aneurysm size, rate o expansion, an
he has heart disease and high blood pressure. His bad hab- symptoms. Asymptomatic patients with aneurysms ≥ 5.5 cm in
its included smoking about 1 pack per day or 40 years, but iameter shoul un ergo repair, not surveillance. Surveillance
he proudly states he quit “cold turkey” a er a heart attack at or me ium size aneurysms (4–5.4 cm) is by ultrasoun or
age 63. As ar as healthcare maintenance, he did have a colo- C every 6 to 12 months an every 2 to 3 years or aneurysms
noscopy 5 years ago at his wi e’s request, and he remembers 3 to 4 cm. Earlier repair in men with AAA ≥ 5 or women with
his last PSA was normal, but he does not recall any type o AAA ≥ 4.5 cm may be in icate i rate o increase is ≥ 0.5 cm in
screening or abdominal aortic aneurysm (AAA). 6 months. AAA repair options inclu e open surgical repair or
en ovascular repair. En ovascular repair has lower 30- ay mor-
Question 21.8.1 The USPSTF recommends one -time AAA tality rate, but overall mortality is i entical af er 3 to 4 years. In
screening: 2011, the ACC/AHA up ate their gui elines regar ing AAA
A) For all men age 65 to –74 years repair, recommen ing either surgical or en ovascular repair or
B) For all men age 65 to 74 years who have smoke > 100 cig- most patients.
arettes in their li etime
C) For all men age 55 to 64 years who have smoke ≥ 1 pack T e ultrasound shows minimal atherosclerotic disease o
o cigarettes per ay or 10 years or more the abdominal aorta and no aneurysm. wo years later, the
D) For all men age ≥ 75 i li e expectancy > 10 years patient returns or ollow-up at the prompting o his daugh-
E) All smokers ≥ 55, regar less o gen er ter who has been noticing that he complains about his knees
hurting all the time. He has never been interested in surgery,
Answer 21.8.1 The correct answer is “B.” T e USPS F an a but he would like to try something di erent. When asked
consortium o lea ing pro essional organizations recommen about his pain on a scale o zero to ten (zero meaning no pain
one-time AAA screening with ab ominal ultrasonography or and ten meaning the worst pain possible), he reports pain
all men age 65 to 74 years who have ever smoke ( e ne as around 2/10 most days, and up to 6/10 a er moderate activ-
> 100 li etime cigarettes). “A” an “C” are incorrect. T e USP F ity. He uses acetaminophen sometimes but does not want to
currently oes NO recommen screening men who have never get addicted to pain medicine.
smoke (< 100 cigarettes in a li etime). T e American Col-
lege o Car iology/American Heart Association (ACC/AHA) Question 21.8.3 In addition to increasing his dose o acet-
gui elines a vise screening men ol er than 60 years who have aminophen, you suggest a topical analgesic such as:
a strong amily history (parents or siblings) o AAA, but amily A) T e 5% li ocaine patch because it can be applie conve-
history is not explicitly consi ere in the USPS F gui elines. niently anywhere on the bo y to provi e a itional knee
“D” an “E” are incorrect. T e evi ence in men ol er than age pain control
75 years an in women oes not support AAA screening, an B) T e 5% li ocaine patch because it acts locally where applie
neither o the above gui elines recommen s routine screening without achieving clinically signi cant serum rug levels
in those groups. One o several imaging mo alities can be use C) Capsaicin cream because it can be applie topically once per
or screening, inclu ing ultrasoun an C scan. ay or e ective pain control
D) Capsaicin cream because it only takes 1 to 2 ays to achieve
You recommend a screening ultrasound or AAA, but then a clinical e ect
he questions why screening or AAA is necessary (guess you E) Diclo enac 1% gel because it can provi e anti-in amma-
need to work on your rapport-building). tory e ect with lesser GI toxicity compare with systemic
NSAIDs
Question 21.8.2 You in orm him that:
A) AAA occurs in approximately 1 in 20 ol er men who have Answer 21.8.3 The correct answer is “E.” Ol er a ults are less
ever smoke likely to be a equately treate or pain compare with younger
B) Rupture o an AAA has a morality rate o 50% a ults. Acetaminophen remains the best choice or rst-line
C) AAA ruptures cause approximately 1,000 eaths per year in therapy o mil -to-mo erate pain ue to its tolerability. opical
the Unite States agents such as the li ocaine, capsaicin, or iclo enac can also
D) reatment or AAA inclu es open surgical repair, en ovas- provi e localize pain control. opical agents can be very use ul
cular repair, or surveillance once the aneurysm reaches 6 cm pain therapy because they penetrate the skin to act on periph-
but not be ore this eral nerves an sof tissue irectly un erlying the application
site. T ese topical agents have minimal systemic absorption
Answer 21.8.2 The correct answer is “A.” AAA is a common an thus limite potential or any clinically signi cant systemic
con ition, occurring in approximately 1 in 20 ol er men who e ect or rug– rug interactions. Diclo enac 1% gel is a topical
594 FAMILY MEDICINE EXAMINATION & BOARD REVIEW

NSAID well-suite or treatment o osteoarthritis. A verse GI TABLE 21-6 ROME CRITERIA FOR
events are less common compare to oral NSAID therapy, an FUNCTIONAL CONSTIPATION
overall a verse events rates are low. However, cost may be pro-
Two or more o the ollowing should be present or at least 12 weeks
hibitive or iclo enac gel.
out o the preceding 12 months:
“A” is incorrect. T e li ocaine patch must be applie irectly
• Straining or greater than 25% o de ecations
over the pain ul area or best results and isn’t e ective in arthri- • Lumpy or hard stools or greater than 25% o de ecations
tis. opical li ocaine only really works or neuropathic pain • Sensation o incomplete evacuation or greater than 25% o de eca-
(e.g., post-zoster) or or local, super cial, skin irritation. “B” is tions
wrong or the same reason. “C” an “D” are incorrect. Capsaicin • Less than three de ecations per week
cream is ose on a regular sche ule every 6 hours to achieve • Manual evacuation or assistance to acilitate de ecation
maximal e ect, which generally takes 2 to 4 weeks.

He tries acetaminophen 1,000 mg ID and capsaicin 3 to Question 21.8.5 Which o the ollowing statements is true
4 times per day, and reports improved pain control. You see about constipation?
him 1 month later or a ollow-up visit. oday he picked up A) T e prevalence o sel -reporte constipation ecreases with
a coupon at the local drug store or an arthritis pill that con- aging
tains chondroitin. He wants to know i this might help his B) Patients shoul be encourage to e ecate be ore meals
knee pain. when the colonic activity is the greatest
C) Fiber is a sa e, inexpensive approach to improve stool con-
Question 21.8.4 You discuss the current evidence and sistency an accelerate colon transit time
in orm him that: D) Increase caloric intake correlates well with constipation in
A) Large-scale trials in icate signi cant symptomatic bene t in the el erly
osteoarthritis with the use o chon roitin supplements
B) For patients with severe osteoarthritis only, a clinically rel- Answer 21.8.5 The correct answer is “C.” Fiber is a sa e, inex-
evant bene t is likely an the use o chon roitin shoul be pensive approach to improve stool consistency an accelerate
encourage colon transit time. Increasing ber is a goo rst-line approach
C) Chon roitin is a large macromolecule that is poorly igeste an shoul be encourage . T e aily recommen e ber
an is potentially unsa e in ol er patients with any stomach intake is 20 to 35 g. “A” is incorrect because the prevalence o
problems sel -reporte constipation increases with aging—up to 45% o
D) T e combination o chon roitin an glucosamine is the rail el erly in ivi uals report constipation as a health issue. It
most popular supplement sol over-the-counter or joint is not uncommon or patients an physicians to have i erent
pain in the Unite States clinical e nitions o constipation, so urther history is help ul
to clari y what the patient means by “constipation.” T e Rome
Answer 21.8.4 The correct answer is “D.” T e combination o Criteria o ers a consensus e nition o constipation use in
chon roitin an glucosamine is the most popular supplement clinical trials as outline in able 21-6 an may be help ul to
sol over-the-counter or joint pain in the Unite States. How- urther characterize constipation. “B” is incorrect. Patients
ever, scienti c evidence is lacking to support the use o chondroitin shoul be encourage to e ecate rst thing in the morning
to prevent or reduce joint pain associated with osteoarthritis. “A” or 30 minutes a er meals when colonic activity is the greatest,
is incorrect. In a systematic review o 20 trials that compare the an to take a vantage o the gastrocolic re ex. “D” is incorrect.
e ects o chon roitin with placebo or no treatment in patients Decrease (not increase ) caloric intake is more likely to be
with hip or knee osteoarthritis, chon roitin ha minimal or no associate with constipation in the el erly.
e ect on joint pain. “B” is incorrect. For patients with a vance
osteoarthritis, a clinically relevant bene t is unlikely. “C” is You rule out secondary causes o constipation and decide
incorrect. Chon roitin is a large macromolecule, an only that your patient likely has primary transit constipation. You
12% to 13% o ingeste chon roitin is absorbe into the bloo rst provide nonpharmacologic recommendations to pro-
stream. However, multiple stu ies have oun no evi ence to mote regular bowel habits, including dietary changes and
suggest that chon roitin is unsa e (except to the sharks that increased exercise.
provi e the cartilage to make the supplement—they are rapi ly
becoming extinct). Question 21.8.6 Which o the ollowing options would be
the best pharmacologic approach to use on a daily basis in
Now that your patient has seen you or a ew years, he eels order to help this patient with his constipation?
more com ortable in discussing other health concerns . . . like A) Fiber: psyllium (Metamucil), oat bran, or methylcellulose
constipation (hey, you’ve built some rapport!). He usually has (Citrucel)
a bowel movement (BM) every 2 to 3 days, but sometimes he B) Stool sof ener: ocusate calcium (Sur ak) or ocusate
gets hard stools that require excessive straining. His wi e tells so ium (Colace)
him to eat more ber, but he wants to know what else he can C) Stimulant laxative: senna (Senokot), castor oil, or bisaco yl
do to help “keep regular.” (Dulcolax)
CHAPTER 21 • CARE OF THE OLDER PATIENT 595

D) Enema: tap water, so ium bisphosphonate, or soap enema C) < 7.5%


E) Prokinetic agent: tegasero (Zelnorm), a 5H 4 agonist D) < 8.5%

Answer 21.8.6 The correct answer is “A.” Primary causes o Answer 21.9.1 The correct answer is “C.” T e American
constipation all into three categories: (1) normal transit con- Geriatrics Society recommen s a goal hemoglobin A1C o less
stipation, (2) slow transit constipation, an (3) anorectal ys- than 7.5% or ol er a ult patients who are otherwise healthy
unction. T ere is no evi ence-base gui eline or the pre erre an have goo unctional per ormance. T e bene ts versus the
or er o using i erent types o laxatives. Supplemental ber risks o intensive iabetic control start to shif as the me ical
helps improve stool orm an requency an is a goo rst step. complexity an unctional impairments o the patient increase.
Psyllium also has the bene t o re ucing lipi s an improving T ere ore, the goal hemoglobin A1C or an in ivi ual who is
glucose control in iabetics. A equate ui intake is necessary requiring assistance with greater than 2 instrumental activi-
when using psyllium since it may exacerbate constipation when ties o aily living or has multiple complex me ical problems
ui intake is insu cient. “B” is incorrect. While stool sof en- is < 8.0%. T ose with poor health or en -stage chronic illness
ers are commonly prescribe an may be help ul, they are less (inclu ing mo erate to severe cognitive impairment) shoul
e ective than other options inclu ing psyllium. “C” is incor- have a goal o < 8.5%. In these situations, attempting to achieve
rect. Stimulant laxatives, when use in recommen e oses, are hemoglobin A1C levels < 7.0% is associate with higher mortality
unlikely to harm the colon i use or short uration. However, an more requent hypoglycemic events.
stimulant laxatives may cause electrolyte imbalance or ab omi-
nal pain an are not the rst-line therapy. “D” is incorrect. Ene- His HbA1C is 7.3%. He has been tolerating his diabetic medi-
mas shoul only be use in acute situations an with caution cations well. He is also on a cholesterol-lowering agent, sim-
ue to the risk o colonic per oration. Large-volume enemas vastatin 80 mg daily. He has been on this medication or
can result in hyponatremia, while enemas containing phosphate several years and has been tolerating it well without any mus-
can lea to hyperphosphatemia an renal ailure, especially in cle aches or documented liver derangements.
patients with renal insu ciency. “E” is incorrect. Prokinetic
agents stimulate propulsion along the GI tract. T e 5H 4 ago- Question 21.9.2 What is his goal LDL level in the context o
nist tegasero (Zelnorm) improves symptoms o constipation in his comorbid medical conditions?
women with irritable bowel syn rome (its only FDA-approve A) LDL < 130 mg/ L
in ication) but is associate with a verse car iovascular events B) LDL < 100 mg/ L
an is available only un er restricte use. C) LDL < 70 mg/ L
D) LDL re uction o at least 50% rom baseline
E) T ere is no LDL goal an statins shoul be avoi e in the
HELPFUL TIP:
el erly
There are many causes o secondary constipation, in-
cluding numerous medications (e.g., anticholinergics,
Answer 21.9.2 The correct answer is “D.” T e American Col-
calcium channel blockers, opiates) and coexistent medi-
lege o Car iology an the American Heart Association release
cal conditions such as diabetes, hypothyroidism, sclero-
new gui elines regar ing the management o cholesterol in
derma, and amyloidosis.
2013. Fortunately, there are a number o online calculators to
ai you in assessing a patient’s risk pro le. For your patient
Objectives: Did you learn to . . . to ay, with his iabetes, age, an hypertension, the goal woul
• Screen or AAA? be a 50% re uction o his LDL rom baseline. His 10-year car-
• Provide the sa e and ef ective treatment or osteoarthritis iovascular isease (CVD) risk is > 7.5%, an this re uction is
pain? best achieve with a high-intensity statin such as atorvastatin 40
• Identi y and treat constipation in the older patient? to 80 mg or rosuvastatin 20 to 40 mg. T e important point here
is that he should be on a high-dose statin. T is is the goal; we no
longer ollow LDL. Mo erate-intensity statins (e.g., atorvastatin
CASE 21.9 10–20 mg, simvastatin 20–40 mg, an lovastatin 40 mg) pro-
A 73-year-old male patient returns to see you or urther vi e a 30% to 50% re uction o LDL an are recommen e or
management o diabetes mellitus type 2. He has had diabetes patients whose 10-year CVD risk is 5% to 7.5%. T e ACC/AHA
or decades and he has been generally well controlled on oral recommen ations also state that starting a statin or increasing
medications o met ormin and glipizide. He also has hyper- intensity o therapy in those age > 75 years does not appear
tension and hypercholesterolemia or which he is treated. He to improve survival. However, the recommen ations go on to
thinks he is in otherwise air health and is active in his com- state that patients shoul be maintaine on their current statin
munity and lives independently with his wi e. me ication an osage so long as they are tolerating it well. T e
ACC/AHA guidelines do not recommend the use o other
Question 21.9.1 What is his target hemoglobin A1C? cholesterol-lowering agents such as ezetimibe.
A) < 5.5% It is also worth noting that the FDA recommen s against
B) < 6.5% starting patients on simvastatin 80 mg because o increase risk
596 FAMILY MEDICINE EXAMINATION & BOARD REVIEW

or rhab omyolysis. T eir recommen ation goes on to say that Drivers.” Available at http://www.nhtsa.gov/people/injury/
patients who have been tolerating simvastatin without signs o ol rive/Ol erDriversBook/pages/Contents.html.
muscle injury or 12 months may continue using it. It is prob- Accesse June 17, 2015.
ably best to move away rom using simvastatin 80 mg in general American Psychiatric Association. Diagnostic and Statistical
at this point. Manual o Mental Disorders (DSM-V). 5th e . Washington,
DC: American Psychiatric Association; 2013.
Objectives: Did you learn to . . . Barnes A. Legal issues in geriatric me icine an gerontology.
• Appropriately determine diabetes goals in the elderly? In: Hazzar WR, Blass JP, Ettinger WH Jr, et al. e s.
• Identi y the best cholesterol-lowering agent based on a Principles o Geriatric Medicine and Gerontology. 4th e .
patient risk pro le? New York, NY: McGraw-Hill; 1999:545–556.
Bentley DW, et al. Practice gui elines or evaluation o ever
an in ection in long-term care acilities. Clin In ect Dis.
Clinical Pearls 2000;31:640–653.
Avoid antipsychotics in the demented elderly with behavioral Bergstrom N, et al. reatment o pressure ulcers. Clinical
and psychological disturbances. First, explore and treat Practice gui elines No. 15. Rockville (MD): US Depart-
potential underlying causes, modi y the environment and ment o Health an Human Services, Public Health
educate caregivers. Service, Agency or Health Care Policy an Research.
Conduct a complete drug review regularly, looking or
Dec. AHCPR Pub. No; 1994:95–0653. (At press time, a new
drug–drug interactions, drug–disease interactions, ine ective
gui eline was un er review but not yet publishe .)
drugs, and symptoms attributable to medications. Bra ley SM, Hernan ez CR. Geriatric assistive evices. Am
Fam Physician. 2011;84(4):405–411.
Do not per orm cervical cancer screening on women over
the age o 65 years i the previous 3 Pap smears prior to age
FDA Drug Sa ety Communication: New restrictions, contrain-
65 years were completely normal and there were no indings
ications, an ose limitations o Zocor (simvastatin) to
o CIN2 or higher in the previous 20 years.
re uce the risk o muscle injury. Available at http://www.
a.gov/Drugs/DrugSa ety/ucm256581.htm. Accesse on
Do not treat asymptomatic bacteriuria in older adults. June 2015.
Do not treat insomnia in older adults with sedative drugs, Hatta K, et al. Preventative E ects o Ramelteon on elirium
including diphenhydramine. First, employ good sleep a ran omize placebo-controlle trial. JAMA Psychiatry.
hygiene and consider melatonin. 2014;397–403.
Do not use medical interventions, such as appetite stimulants Inouye SK, et al. Clari ying con usion; the con usion assess-
and gastric tube eedings, or demented older adults who are ment metho . Ann Intern Med. 1990;113:941–948.
losing weight. Provide appealing and calorically dense oods, Inouye SK, et al. A multicomponent intervention to prevent
eeding assistance, and a social context or eating. elirium in hospitalize ol er patients. N Engl J Med.
Estimate longevity and determine patient pre erences prior 1999;340:669–676.
to initiating screenings and preventative interventions. Kirkman S, et al. Diabetes in ol er a ults: A consensus report.
J Am Geriatr Soc. 2012;60(12):2342–2356.
Older patients who sustain a hip racture have better clinical
outcomes with early repair (within the irst 24 hours). Loue S. El er abuse an neglect in me icine an law: T e nee
or re orm. J Leg Med. 2001;22:159–209.
Periodically reassess cholinesterase inhibitors or dementia;
Mehr DR, atum PE. Primary prevention o iseases in ol
taper and discontinue the drug i there is no perceived ben-
age. Clin Geriatr Med. 2002;18(3):407–430.
e it or i adverse events occur (e.g., weight loss, bradycardia).
Panel on Prevention o Falls in Ol er Persons, American
Pressure ulcers should be treated primarily with pressure Geriatrics Society an British Geriatrics Society. Up ate
relie , good wound care, and good nutrition. Minimal bene it AGS/BGS clinical practice gui eline or prevention o
is attributable to dressing type or advanced wound care alls in ol er persons. J Am Geriatr Soc. 2011;59(1):
treatments (e.g., electrical stimulation, ultrasound). 148–157.
Pergolizzi J, et al. Opioi s an the management o chronic
severe pain in the el erly. Pain Pract. 2008;8(4):287–313.
BIBLIOGRAPHY Pham CB, Dickman RL. Minimizing a verse rug events in
Allman RM, et al. Pressure ulcer risk actors among hospital- ol er patients. Am Fam Physician. 2007;76(12):1837–1844.
ize patients with activity limitation. JAMA. 1995;273(11): Po sia lo D, Richar son S. T e time “Up & Go”: A test o
856–870. basic unctional mobility or rail el erly persons. J Am
American College o Physicians/American Aca emy o Family Geriatr Soc. 1991;39(2):142–148.
Physicians. Current pharmacologic treatment o ementia: Ross GW, Bowen JD. T e iagnosis an i erential iagnosis
A clinical practice gui eline rom the ACP/AAFP. Ann o ementia. Med Clin North Am. 2002;86(3):455–476.
Intern Med. 2008;148(5):370–378. Schwartz RS, Buchner DM. Exercise in the el erly: Physiologic
American Me ical Association/National Highway ra c an unctional e ects. In: Hazzard’s Geriatric Medicine
A ministration/US Department o ransportation; 2010. and Gerontology. 6th e . New York, NY: McGraw-Hill;
“Physician’s Gui e to Assessing an Counseling Ol er 2009:1381–1395.
CHAPTER 21 • CARE OF THE OLDER PATIENT 597

ariot PN. Me ical management o a vance ementia. J Am T e American Occupational T erapy Association. Available at
Geriatr Soc. 2003;51(5):S305–S313. http://www.aota.org/ eature /area6/in ex.asp. Accesse
T om W, et al. 2011 ACCF/AHA Focuse Up ate o the November 22, 2007.
Gui eline or the Management o Patients with Peripheral US Preventive Services ask Force. Screening or ab ominal
Artery Disease. J Am Coll Cardiol. 2011;58:2020–2045. aortic aneurysm: Recommen ation statement. Ann Intern
omcyk S, et al. Use o 13-valent pneumococcal conjugate vac- Med. 2005;142:198–202.
cine an 23-valent pneumococcal polysacchari e vaccine Watson YI, et al. Clock completion: An objective screening test
among a ults age ≥ 65: recommen ations o the a visory or ementia. J Am Geriatr Soc. 1993;41(11):1235–1240.
committee on immunization practices. MMWR Morb Wuillen DA. Common causes o vision loss in el erly patients.
Mortal Wkly Rep. 2014;63:822–825. Am Fam Physician. 1999;60(1):99–108.
Care of the Surgical Patient
D vdB m r ner, N s B. W e n, nd Br E. R y
22
CASE 22.1 gangrene the most likely diagnosis. While crepitus is common
with Fournier gangrene due to presence o gas- orming anaerobic
A 40 year old ale atient with ty e 2 diabetes calls your bacteria, its absence is not sensitive enough to rule out gangrene.
o ce because he is having di culty urinating and quite a bit Without early surgical debridement and IV antibiotics, in ec-
o ain in the erineal area. He has not elt well or several tion can progress rapidly causing sepsis and multiorgan ailure.
days and was running a low-grade ever. He went to his chiro- T ere ore, “B,” emergent surgical re erral is the best option or
ractor 2 days ago when he only had ain and swelling in the this patient. Antibiotics, particularly oral (options “A” and “B”),
scrotu and the chiro ractor adjusted his . . . well, we won’t are inappropriate as a sole therapy. “D” has the right intent with
go there . . . . He is now noting that his te erature is higher broad-spectrum coverage but is incorrect because the patient
(he doesn’t have a ther o eter and is reading his te era- needs adjunct surgical debridement to remove the necrotic tissue.
ture via his old ood ring ro the 90s). You suggest that he
resents to your o ce. Question 22.1.2 Fournier gangrene can best be described
Exa ination reveals an obese ale who is waddling into as:
the o ce because o ain in his scrotal area. Vitals: blood A) Necrotizing asciitis
ressure 150/100 Hg, ulse 112 b , res irations 20 B) Necrotizing cellulitis
b , and te erature 39.0°C. Other signi cant ndings C) Caused by aerobic bacteria
include a swollen scrotu that is bright red, exquisitely ten- D) Secondary to streptococci
der to touch, and without cre itus. You do not have extended E) A mathematical unction that can be written as an in nite
laboratory access in your o ce, but a urine di stick is nega- sum o harmonics
tive or blood and leukocyte esterase. His blood sugar, which
is usually airly well controlled, is elevated at 320 g/dL. Answer 22.1.2 The correct answer is “A.” Fournier gangrene
is a orm o necrotizing asciitis. T is is termed “ ype I” nec-
Question 22.1.1 Your next step or this patient will be which rotizing asciitis and is caused by mixed aerobic and anaerobic
o the ollowing? bacteria. “B” is incorrect. Necrotizing cellulitis is isolated to the
A) Start the patient on cephalexin (Ke ex) or methicillin- super cial skin. Fournier gangrene involves deep tissues includ-
sensitive Staphylococcus aureus and Streptococcus coverage ing the ascia. “C” is incorrect because, as noted above, Fournier
and ollow-up with the patient in the morning gangrene is caused by mixed aerobic and anaerobic bacteria.
B) Emergent surgical re erral “D” is incorrect. T ere is necrotizing asciitis secondary to
C) Begin trimethoprim/sul amethoxazole (Bactrim) or meth- stre tococci (see below); however, it has di erent causative
icillin-resistant S. aureus (MRSA) coverage and ollow-up agents than Fournier gangrene. Fournier gangrene is typically
with the patient in the morning caused by coli orms, Clostridia, Bacteroides, Klebsiella, Staphy-
D) Admit the patient and start him on IV vancomycin, piper- lococcus, and Streptococcus. I you used “the-longest-answer-is-
acillin/tazobactam, and metronidazole or coverage o always-right” method and chose “E,” you are way o base and
MRSA, Streptococcus, Pseudomonas, anaerobes also a math nerd; “E” describes Fourier’s trans orm.

Answer 22.1.1 The correct answer is “B.” T is likely repre- Question 22.1.3 Which o the ollowing is NOT a risk actor
sents Fournier gangrene. T e erythematous, swollen scrotum or Fournier gangrene?
with pain out o proportion to examination and associated signs A) Diabetes
o ever, tachycardia, and elevated blood sugar make Fournier B) Immunosuppression
598
CHAPTER 22 • Ca RE o t h E Su Rg iCa l Pa t iENt 599

C) Varicella in ection
D) End-stage renal disease T e atient has a wide excision o necrotic tissue. A er sur-
E) Advanced age gery, he beco es hy otensive and tachycardic (blood res-
sure 85/50, ulse 128), and his seru lactate is 8 g/dL (very
high).
Answer 22.1.3 The correct answer is “C.” All the other options
are risk actors or Fournier gangrene. Varicella in ection is a Question 22.1.5 What is the most appropriate next step in
risk actor or y e II necrotizing asciitis, which is a di erent his management?
entity. ype II necrotizing asciitis is caused by group A Strep- A) Start IV dopamine to stabilize his blood pressure
tococcus. Generally, patients with ype II necrotizing asciitis B) Start IV norepinephrine to stabilize his blood pressure
are not diabetic or otherwise immunocompromised. Other risk C) Start IV 0.9% normal saline boluses to stabilize his blood
actors or ype II necrotizing asciitis include IV drug use, pen- pressure
etrating trauma, and blunt trauma. D) Start IV albumin boluses to stabilize his blood pressure
E) Read him the hospital charges he has accumulated thus ar
T e surgeon evaluates the atient and asks or your o inion
on the antibiotic regi en. You nearly all over in shock that Answer 22.1.5 The correct answer is “C.” T is patient is likely
your advice was solicited. hypovolemic secondary to third spacing and/or is in septic shock
and needs volume. T e lactate o 8 mg/dL suggests that he has
Question 22.1.4 What do you recommend or this patient hypoper usion with poor tissue oxygenation (although it could
with necrotizing asciitis? also be secondary to necrotic tissue). Just giving dopamine (or
A) Penicillin and clindamycin other vasopressors) will not increase peripheral circulation and, in
B) Clindamycin and metronidazole act, may decrease tissue oxygenation by increasing vascular tone
C) Meropenem, vancomycin and clindamycin and decreasing per usion. Albumin has not been shown to have
D) Piperacillin/tazobactam (Zosyn), vancomycin and metroni- any advantage over crystalloids in almost any circumstance—post
dazole large-volume paracentesis (> 5 L) being the exception. Mortality
E) C or D o necrotizing asciitis approaches 30% even with the best care.

Answer 22.1.4 The correct answer is “E.” As noted above, HELPFUL TIP:
Fournier gangrene is a mix o aerobic and anaerobic bacte- t e nk n b seps s s c n ed n e s ew
ria. T e rst two choices (“A” and “B”) will cover anaerobes ye rs w e S rv v n Seps s C mp n (Crit Care Med.
and some Gram-positive organisms. However, Gram-nega- 2013;41(2):581–637) nd sever s bseq en s d es
tive coverage is lacking. More broad-spectrum antimicrobial spec s de ne. S ccess y re n seps s
coverage is needed in this situation. Most experts and guide- req res e r y rec n n, e r y ress ve d res s-
lines recommend a broad-spectrum antibiotic with anaerobic c n (b ses 30 ml /k ), nd br d-spec r m n-
and Gram-negative coverage (e.g., piperacillin/tazobactam, b cs w n 1 r rec n n severe seps s r
ampicillin/sulbactam, or a carbapenem) lus MRSA cov- sep c s ck. Following speci ic sepsis protocols (e.g., with
erage (usually vancomycin) lus anaerobic coverage (e.g., central venous pressure and hematocrit goals) does not
clindamycin, metronidazole). Although MRSA is not usually change outcomes (N Engl J Med. 2014;370:1683–1693). Ag-
the causative agent, these in ections are polymicrobial, and gressive use o central lines, vasopressors, trans usions, etc.
empiric MRSA coverage is required. Clindamycin deserves also does not alter mortality in septic patients (N Engl J Med.
a special mention. Clindamycin blunts the adverse e ects o 2014;371:1496–1506). E r y rec n n nd v nce
Staphylococcal and Streptococcal toxins so is especially help ul e e c re e m, m n r n e sep c p en
in Staphylococcal and Streptococcal toxic shock syndromes and c se y r c n es n s s (v v s ns, ex m n n
toxin-related shock. s ns per s n, r s nd ssessmen iVC pres-
s re, r ne p , e c.), re m re mp r n n nv -
s ve m n r n . Again, early recognition, along with
HELPFUL TIP: aggressive volume resuscitation with crystalloid,
iVig s been sed n necr z n sc s c sed by treatment o underlying in ection, and attention to
Clostridium spec es s we s n se w s rep - patient status, are the keys to surviving sepsis.
c cc sc s. W e ere s s es n bene ,
ed re nc mp e e nd m re s dy s w rr n ed.
Objectives: Did you learn to . . .
h yperb r c xy en s s been sed b s ers r m
• D n se nd re rn er n rene?
e s me ck d . t ese re men s re p en
dj nc re men s. t e de n ve re men s e r y, • D eren e be ween ypes necr z n sc s?
ress ve, s r c debr demen ! • Prescr be ppr pr e ds nd v s press rs re seps s
nd sep c s ck?
600 a Mil Y MEDiCiNE EXa MiNa t io N & Bo a RD REViEW

and elvis, but ro the waist u he is ne. Nonetheless, the


QUICK QUIZ: Pa iN Co Nt Ro l a bulance crew laces the atient in a collar and on a back-
board and trans orts hi to the ED.
Which o the ollowing drugs will give you the most rapid pain T e atient is in signi cant ain ro his lower extre i-
control when given intravenously ties and elvis (o course, since he is a stoic ar er, he denies
A) Fentanyl (Sublimaze) this and just wants to go back to lanting corn). His blood
B) Hydromorphone (Dilaudid) ressure is initially 100/65 Hg with a ulse o 118 b .
C) Meperidine (Demerol) Pri ary survey is unre arkable, although he is still boarded
D) Morphine and collared.

The correct answer is “A.” Fentanyl has a peak e ect at 3 to Question 22.2.1 Which o the ollowing most clearly re ects
5 minutes. T is is ollowed by meperidine at 5 to 7 minutes, the best approach to this patient’s pain?
morphine at 20 minutes, and hydromorphone at 15 to 30 min- A) Use IV meperidine (Demerol) or pain control
utes. T us, to get rapid control o pain, entanyl is the pre erred B) Use IV morphine or pain control
agent. Meperidine is the least pre erred agent because o drug C) Use IV entanyl (Sublimaze) or pain control
interactions (MAO inhibitors, SSRIs) and toxic metabolites D) Pain medications are contraindicated at this point given the
(normeperidine that can cause agitation and seizures). patient’s overall condition
E) Since he’s so tough, just o er him some useless platitudes
HELPFUL TIP: like “no pain, no gain”
V nc myc n s n re dr r re n Staphylo-
coccus. in c , np en m r y r es re er n Answer 22.2.1 The correct answer is “C.” Fentanyl generally
p en s w me c n-sens ve Staphylococcal n- has a negligible e ect on blood pressure (although one should
ec ns re ed w v nc myc n nw er n- never say “never”). Both morphine and meperidine tend to drop
b cs. S , y ve me c n-sens ve S. aureus, a patient’s blood pressure, so they are relatively contraindicated
c n e n b cs s me n er n v nc myc n, in this patient with a marginal blood pressure or hypotension.
s c s n c n, pen c n, r mp c n/s b c m— r T ose who chose “D” or “E” have been hanging around old-
w ever e s scep b es nd c e. time surgeons too long. It is unconscionable to withhold pain
medication.

Further exa ination, by gentle inward ressure on the ante-


QUICK QUIZ: a Nt iBio t iC Co VERa g E
rior su erior iliac s ines bilaterally, shows that the atient’s
elvis is unstable. As you recall, a ractured elvis can lead to
Which o the ollowing organisms is/are not generally covered signi cant blood loss into the retro eritoneal s ace.
by trimethoprim/sul amethoxazole ( MP/SMX)?
A) S. aureus
Question 22.2.2 In the short term, what is the best way to
B) Streptococcal species
temporize the underlying pathology?
C) Escherichia coli
A) Vasopressors (e.g., dopamine) plus uids
D) Enterococcus
B) Pelvic binder
E) B and D
C) Fluids (normal saline)
D) Activated actor VIIa (NovoSeven)
The correct answer is “E.” Neither Streptococcus nor enterococ-
E) Embolization by interventional radiology (45-minute delay)
cus species are sensitive to MP/SMX. T is has implications or
the treatment o MRSA. Cellulitis (one o the mani estations o
Answer 22.2.2 The correct answer is “B.” A pelvic binder will
MRSA) can be rom either staphylococcal or streptococcal spe-
signi cantly reduce bleeding in most unstable pelvis ractures.
cies. So, unless you are con dent that you have MRSA (abscess
“A” is incorrect because in trauma dopamine is not indicated
ormation, etc.), MP/SMX is not a great single agent or cel-
when the problem is hypovolemia. In act, vasopressors increase
lulitis. I there is an abscess, treat or MRSA (drainage, MP/
mortality in hypovolemic shock. “C” is incorrect. While nor-
SMX, doxycycline). I there is a super cial cellulitis, treat or
mal saline is a good choice or resuscitation uids, we want
Streptococcus (cephalexin, amoxicillin/clavulanate). I you are
to tamponade the bleeding and not just chase our tails with
not sure, one option would be clindamycin as a single agent,
uids. “D” is incorrect. Recombinant activated actor VIIa has
though resistance rates are on the rise.
not been shown to improve outcomes a er trauma ( J rauma.
2010;69:353–359). Additionally, it has a number o adverse
CASE 22.2 e ects including increased thromboembolic phenomenon
You are working in a rural ED when you get a call that a (pulmonary embolism [PE], deep venous thrombosis [DV ],
62-year-old ar er has been tra ed between a tractor and etc.). Finally, interventional radiology will likely be needed at
a silo while loading silage. It see s to have inned his legs some point. T ey are quite good at stopping bleeding vessels.
CHAPTER 22 • Ca RE o t h E Su Rg iCa l Pa t iENt 601

However, the delay time during which you are watching the
HELPFUL TIP:
patient bleed is unacceptable.
i y ee nc m r b e d n rm cys s my,
cen r ne w b n p ced n e b dder c n be
HELPFUL TIP: sed s emp r z n s n.
t r nex m c c d, dr preven s br n ys s, s
been s wn (m r n y) red ce m r y r m
b eed n n r m p en s w ny d wns de. t e Objectives: Did you learn to . . .
d se s 1 iV ver 10 m n es wed by 1 ver e • t re c e r m c p n?
nex 8 rs. i s d be s r ed w n e rs r. • Rec n ze nd re pe v c r m nd n r pe v c
C ns der s s emp r zer y ve n c ve y em rr e?
b eed n p en . • Descr be c n r nd c ns b dder c e er p cemen ?

You a ro riately lace a elvic binder that ta onades the CASE 22.3
bleeding. T e atient’s blood ressure stabilizes. You now
A 55-year-old gentle an with history o s oking and dia-
turn to other issues. T is atient will clearly need a Foley
betes resents to your o ce or a ass in his right groin.
catheter.
He works as a night stocker in a local su er arket. He rst
noticed the ass a er li ing a 60 lb box ro the oor to the
Question 22.2.3 Relative contraindications to placement o
shel several nights ago. Since then, he notices that the ass
a Foley catheter include which o the ollowing?
is ain ul and “ ulses” when he coughs. He has not been able
A) Blood at the urethral meatus
to work since he rst noted the ass due to ain; in act,
B) Gross hematuria
he wonders i you should just ll out disability a ers or
C) High-riding prostate
hi now (never ha ens, right?). He endorses a history o
D) Gross blood rom the rectum
chronic consti ation or which he takes over-the-counter
E) A and C
edications. He denies weight loss, nausea, vo iting, and
urinary sy to s.
Answer 22.2.3 The correct answer is “E.” Both blood at the
On exa ination, the atient is a ebrile with a blood res-
meatus and a “high-riding prostate” (ever wonder what it is
sure o 135/86 Hg and a ulse o 84 b . Abdo inal
riding on?) signi y the possible disruption o the urethra. T us,
exa ination de onstrates a ildly obese gentle an with no
since one does not want to place the catheter in the wrong
scars o rior surgeries. Bowel sounds are nor al. He denies
place—like the peritoneal space—catheterization is relatively
tenderness with al ation o all our quadrants. T ere is no
contraindicated. “B,” gross hematuria, can be rom the kidney
guarding. With standing, exa ination o the groin de on-
and is not a contraindication to catheterization.
strates an enlarged scrotu on the right side. On coughing
the ass beco es ore ro inent and you can eel so e-
You nd blood at the urethral eatus. T e atient co lains
thing rotruding ro the inguinal canal.
that he really needs to void.

Question 22.2.4 Your options at this point include which o Question 22.3.1 Based on the in ormation above what is
the ollowing? the next step in this patient’s overall care?
A) Urethrogram to document an intact urethra A) Get C scan o the abdomen to evaluate or etiology o the
B) Per ormance o suprapubic cystostomy using ultrasound mass
guidance B) Re er the patient to a surgeon
C) Use a Coudé catheter to catheterize the urethra C) Have the patient monitor the mass over the next couple o
D) Placement o a bladder catheter via the urethra but using a months or changes
wire guide (such as with a central line) D) Have the patient cut back on his constipation medication
E) A and B E) Place the patient in an inguinal binder

Answer 22.2.4 The correct answer is “E.” One could per orm Answer 22.3.1 The correct answer is “B.” T e patient has an
a urethrogram using a water-soluble dye (e.g., Gastrogra n) inguinal hernia. History and physical examination are typi-
to document that the urethra is intact and i so place a stan- cally su cient to make the diagnosis, so no urther imaging
dard Foley. One could also do a suprapubic cystostomy using is warranted (“A”). Because the hernia is relatively large and
ultrasound guidance. “C” and “D” are both incorrect. A Coudé a ecting his ability to work, surgical repair is recommended
catheter is used to bypass a stricture (prostate or otherwise) and rather than watch ul waiting (“C”). For small, asymptomatic,
would be no sa er than a regular catheter in this patient. Like- or mildly symptomatic hernias, surgical repair can be de erred
wise, a wire could end up anywhere and should not be used in with expectant management and counseling on signs and symp-
this case. toms o strangulation that would warrant emergent evaluation.
602 a Mil Y MEDiCiNE EXa MiNa t io N & Bo a RD REViEW

Because constipation is a risk actor or hernias, advising him to return to his ollow-u a oint ents with you. wo years
to cut back on his constipation medication is probably not rec- later, the atient returns a er he has initiated a running regi-
ommended (“D”). Finally, “E” sounds like a medieval torture en to lose weight. Un ortunately, he is now having groin
device rather than a therapeutic intervention. ain on the le a er he runs and is worried that he now has
an inguinal hernia on the le (his rior hernia was on the
right). He denies any nausea, vo iting, consti ation (you
HELPFUL TIP: STUFF THE GUTS BACK IN OR NOT? cured hi with the Miralax!), urinary sy to s, or asses
as ded b ve, bserv n symp m c r in the groin area. Pain is an aching sensation and worsened
m n m y symp m c ern s s c mes re with changing ositions ro sitting to standing or going u
j s s d s rep r (JAMA. 2006;295:285–292). in c , stairs. He denies any injury.
b ne- rd p en s w ve s r c rep r On exa ination, the atient is a ebrile with a blood res-
ve c n n ed p n w 6% ve deb n p n. sure o 120/78 Hg and a ulse o 84 b . Abdo inal
n y, pen rep r seems be be er n p r - exa ination de onstrates a nor al-weight gentle an
sc p c rep r n erms rec rrence, nd, s rpr s n y, with well-healed la arosco ic incisions on the right. Bowel
per per ve m rb d y. h wever, p r sc p c rep r sounds are nor al. He denies tenderness with al ation o
res ed n ess r n n mbness nd p n n n erm all our quadrants. T ere is no guarding or rebound. Pain is
(Ann Surg. 2012;255:846). re roducible with al ation o the inguinal liga ent on the
le . T ere is no inguinal ly hadeno athy. With standing,
exa ination o the groin de onstrates a nor al-sized and
You re er the atient to a general surgeon within the onth sy etric scrotu bilaterally without any al able asses.
and he undergoes a la arosco ic hernia re air with esh With al ation o the a ected area, there is no ass when the
lace ent. He is discharged to ho e the ollowing day. He is atient coughs.
relieved that he doesn’t need that disability a erwork lled
out a er all and already 2 days osto erative is wondering Question 22.3.3 What is the most common diagnosis or
when he can go back to work. He is unable to reach his surgeon this patient’s symptoms?
who is in the OR (or aybe on the gol course) and calls your A) Osteoarthritis o the le hip
o ce asking when he can return to work . . . li ing 60-lb boxes. B) Femoral adenitis
C) Le direct inguinal hernia
Question 22.3.2 How do you counsel him about when he D) Osteitis pubis
can return to work? E) Ligamentous strain
A) He may return to work once he is no longer requiring any
pain medications and can drive Answer 22.3.3 The correct answer is “E.” While all o the
B) Return to work is typically permitted within 2 to 4 weeks or above should be considered in the di erential or groin pain,
laborers and within 10 days as tolerated or pro essionals the most likely diagnosis or this patient’s symptoms with
C) Return to work at his current job is not recommended, as it his new running regimen is a ligamentous strain. Without a
requires too strenuous activity and heavy li ing. He should bulge detected on Valsalva maneuver (coughing) or a palpable
seek another line o work abdominal wall de ect, a hernia is unlikely (“C”). T ere are no
D) Return to work is typically permitted within 6 weeks or all palpable inguinal lymph nodes and patient does not have clini-
pro essions cal signs o a ever, so “B” is unlikely. Hip osteoarthritis (“A”)
can be a cause o groin pain, particularly in people with good
Answer 22.3.2 The correct answer is “B.” Hernia repair is a old wear-and-tear. However, the type o pain with hip osteoar-
airly low-risk surgery and patients are typically able to return thritis tends to be a deep and aching sensation within the groin
to work within 2 to 4 weeks, primarily based on their surgeon’s area that is not reproducible on palpation. O particular note is
pre erence. For pro essionals with more sedentary jobs, return “D.” Osteitis pubis, an in ammatory process o the symphysis
to work may be as early as 10 days as long as no heavy li ing or pubis, is a very common overuse syndrome causing groin pain
straining will take place; there ore “D” is not the best answer. in athletes; it mani ests as pain over the pubic symphysis, not
While patients should be o all their pain medications prior to the inguinal ligament. Other causes include childbirth, uro-
return to work or driving, “A” is not the best answer as the sur- logic or gynecologic surgery, and rheumatologic conditions.
gical wound needs at least 2 to 4 weeks to heal to a reasonable reatment includes rest, NSAIDs, and, in rare cases, injection
strength. “C” is not correct as he should be able to resume his o the symphysis pubis with a steroid mixture ( or athletes).
prior job activities once the surgical wound heals to maximum Other causes o groin pain that should be considered within
strength. However, the patient should be counseled on sa e the di erential and should be based on your clinical history
li ing techniques and possibility or recurrence o his hernia. and ndings include epididymitis, re erred pain, ectopic testes,
stress ractures, and tendinopathies. I the pain had been local-
T e atient is able to success ully return to work about ized to the right, adhesions and mesh complications rom prior
4 weeks osto eratively. He is eeling so well that he orgets hernia repair must also be considered.
CHAPTER 22 • Ca RE o t h E Su Rg iCa l Pa t iENt 603

Answer 22.4.1 The correct answer is “B.” T is patient most


You treat your atient conservatively and he goes on to co - likely has an external source o obstruction. Bowel obstructions
lete his rst hal - arathon a year later. are divided into two classes: mechanical and unctional (also
Objectives: Did you learn to . . . known as pseudo-obstruction, ileus, or neurogenic obstruc-
• D n se ern b sed n c n c s ry nd p ys c tion). Mechanical obstructions are urther classi ed by both
ex m n n? their location and etiology. Possible etiologies include intralu-
• a ppr pr e y m n e re men r ern s? minal bodies (e.g., gallstone ileus or oreign body), intramural
• C nse p en s n rec mmended re rn c v es p s - lesions (e.g., tumor or intussusception), and extramural lesions
per ve y? (e.g., adhesions). Obstructions can urther be divided into open
• iden y er p en c ses r n p n? and closed loop. Open-loop obstructions have an outlet or
gas and secretion relie (e.g., vomiting) whereas closed-loop
CASE 22.4 obstructions block both in ow and out ow to an area. Closed-
loop obstructions, like bowel torsion or volvulus, cause acute,
A 60-year-old e ale resents to your o ce or severe severe abdominal pain, and generally require urgent surgical
abdo inal ain . . . at 4:45 pm . . . on a Friday. She re orts intervention.
that she develo ed vague le lower quadrant abdo inal ain Bowel obstruction presents with crampy, intermittent
yesterday. T is orning she awakened ro her slee with abdominal pain, vomiting, distention, and obstipation. History
severe, di use abdo inal ain, anorexia, and vo iting. o en includes previous abdominal surgery. Depending on the
On exa ination, she is lying very still. e erature is degree o obstruction and its duration, there may be hyperac-
38.4°C, ulse 106 b , res irations 16 b , blood ressure tive bowel sounds, high-pitched bowel sounds, or decreased/
100/62 Hg. She has dry ucous e branes. Her abdo- absent bowel sounds. An upright abdominal plain lm or lateral
en has di inished bowel sounds and is rigid with invol- recumbent abdominal lm con rms diagnosis with ndings o
untary guarding and rebound tenderness greatest in the le dilated loops o small bowel (bowel > 3 cm in diameter) on the
lower quadrant. On elvic exa ination, she is exquisitely at plate and air uid levels on the upright or decubitus lm.
tender on the le abdo en. T ere is no cervical otion ten- However, the sensitivity, speci city, and accuracy o plain radio-
derness or adnexal ullness. T ere are no asses on rectal graphs are 79% to 83%, 67 to 83%, and 64% to 82%, respec-
exa ination, and her stool is negative or occult blood. tively. C scan is ore sensitive and s eci c (93%and 100%)
Laboratory tests include a negative urine regnancy (she’s or obstruction than are lain l s and will o en reveal the
60 a er all—but can you be too care ul?!), WBC 25,500/ source o the obstruction. However, C should be reserved or
3 3
, HC 32%, latelets 450,000/ , Na 142 Eq/L, K patients in whom the diagnosis is unclear. Patients with a com-
3.2 Eq/L, BUN 24 g/dL, and Cr 1.0 g/dL. Abdo inal plete SBO will lack air in the colon on plain lm; but remem-
x-ray de onstrates ree air under the dia hrag . Based on ber that air can be introduced into the rectum during a rectal
the in or ation you decide that she has diverticulitis. examination.
T e atient undergoes surgery with a artial bowel resec-
tion and ri ary re-anasto osis. wo onths ollowing your Question 22.4.2 Which o the ollowing cause ileus?
atient’s surgery, she resents co laining o abdo inal A) Burns
ain. T e ain is cra y and inter ittent. Further history B) Spinal cord injury
reveals a 24-hour history o vo iting, abdo inal bloating, C) Hypokalemia
and low-grade ever. She re orts her last bowel ove ent D) Pneumonia
was 2 days ago and denies any atus over the last 24 hours. E) All o the above can cause an ileus
On exa ination, her te erature is 37.1°C, ulse 105 b ,
res irations 12 b , and blood ressure 158/60 Hg. Her Answer 22.4.2 The correct answer is “E.” All o the above can
abdo en is slightly distended, di usely tender to al ation cause an ileus. Additional causes include peritonitis, pancreatitis,
without rebound or guarding, and has hy eractive bowel uremia, surgery, and narcotics.
sounds. On at late and u right views o the abdo en, there
are dilated loo s o s all bowel and ulti le air uid levels. You diagnose a SBO, which you believe is ost likely related
to adhesion or ation a er he icolecto y.
Question 22.4.1 Which o the ollowing is true regarding
this patient’s current disease process? Question 22.4.3 Which o the ollowing is INCORRECT
A) She most likely has a closed-loop small bowel obstruction regarding the management o bowel obstruction?
(SBO) A) Initial treatment orders should include maintaining the
B) She most likely has an extramural source o obstruction patient NPO, IV uid resuscitation, and electrolyte replace-
C) Dilated loops o small bowel are de ned as being > 5 cm in ment as needed
diameter on plain lm B) T is patient should undergo emergent surgical intervention
D) Both partial and complete bowel obstructions reveal no C) I she has ever or leukocytosis, she should undergo surgical
colonic gas on plain lm intervention
604 a Mil Y MEDiCiNE EXa MiNa t io N & Bo a RD REViEW

D) I she requires surgery, broad-spectrum antibiotics to cover B) BMI ≥ 35 kg/m2 without the presence o weight-related disease
anaerobes and Gram-negative aerobes should be adminis- C) BMI ≥ 35 kg/m 2 plus other weight-related disease such as
tered perioperatively sleep apnea, diabetes, severe joint disease, or weight-related
cardiomyopathy
Answer 22.4.3 The correct answer is “B.” Peritoneal adhesions D) Failure to control weight with diet and other medical inter-
account or more than hal o all SBOs. Up to 80% o episodes o ventions
SBO caused by adhesions resolve without surgical intervention.
Initial treatment includes restricting oral intake, IV uid resus- Answer 22.5.1 The correct answer is “B.” A BMI o ≥ 40 kg/m 2
citation with normal saline, and electrolyte correction. T e goal (not ≥ 35 kg/m 2) alone is considered an appropriate weight or
is to prevent small bowel strangulation. Patients can be sa ely surgical intervention in the absence o other co orbidities. As
observed i there is no evidence o strangulation. Indications noted above, a BMI o ≥ 35 kg/m 2 with weight-related comor-
o strangulation include rapidly progressing abdominal pain bidities is considered an indication or bariatric surgery. “D” is
or distention, development o peritoneal ndings, ever, dimin- worthy o comment. Most weight control drugs are airly useless.
ished urine output, leukocytosis, hyperamylasemia, metabolic However, ailure o medical therapy is considered a necessary
acidosis, and persistent obstruction. Co lete bowel obstruc- condition or bariatric surgery, as is an acceptable operative risk.
tion, and closed-loop obstructions, should always be treated sur-
gically. Also, patients with de novo obstruction (e.g., no history Much to their chagrin, they do eet the criteria or weight
o laparotomy) usually require surgical intervention. I surgery is loss surgery. T ere are several surgeons in town who er or
necessary, broad-spectrum antibiotics that cover anaerobes and di erent techniques, including a Roux-en-Y and la aro-
Gram-negative aerobes should be administered perioperatively sco ic banding.
to reduce wound in ection and abdominal sepsis rates.
Question 22.5.2 Which o the ollowing is NOT true?
A) Roux-en-Y is associated with greater weight loss at 1 year
HELPFUL TIP: B) Laparoscopic banding is associated with a greater need or
a n Ng be s r d n y sed n SBo , e recurrent surgery compared with Roux-en-Y
se s p n . a n Ng be s nd c ed e p ev - C) Mortality is higher with Roux-en-Y and there are more hos-
e v m n , d s en n, e c., b d es n s en e pital admissions or complications
res n e SBo . D) Vertical banded gastroplasty leads to sustained weight loss
and ew complications

HELPFUL TIP: Answer 22.5.2 The correct answer is “D.” Vertical banded gas-
Remember p en -c n r ed n es (PCa ) s troplasty does not lead to sustained weight loss and has many
e m s e ec ve m d y r re n p n n e complications including disruption o the staple line, GERD,
p s per ve p en nd n m ny er c nd ns vomiting, and erosion o the band into the stomach. For this
(e. ., p ncre s, c e c es syndr me). S de e ec s reason, it has (or should have) allen out o avor. T e rest are
n rc cs m y nc de r n ry re en n n dd n true. Roux-en-Y results in the best weight loss and requires
c ns p n r resp r ry depress n. ewer recurrent surgeries when compared with laparoscopic
banding. However, the mortality rate is higher (0.06% vs. 0.17%)
or Roux-en-Y and there are more perioperative complications.
Objectives: Did you learn to . . . It also requires longer hospital stays.
• a ssess bd m n p n nd rec n ze n c e bd men?
• Pr v de ppr pr e per per ve m n emen r s r n- Mr. and Ms. Biggs are wondering i this is worth the trouble
es n (g i) s r ery? at all.
• iden y nd re sm b we bs r c n er bd m n
s r ery? Question 22.5.3 You can tell them that:
A) Although they may lose weight, weight loss surgery does
nothing or their underlying diabetes, hypercholesterolemia,
CASE 22.5 and coronary artery disease (CAD)
Mr. and Ms. Biggs have always been “big boned.” T ey have B) T ere is no psychological or quality-o -li e bene t to weight
decided that weight reduction surgery is the thing or the . loss surgery: they will have the same existential crisis be ore
However, they are concerned that, or the rst ti e in their and a er the surgery
lives, they ay not be big enough. C) NSAIDs are relatively contraindicated a er weight loss
surgery
Question 22.5.1 Which o the ollowing is NOT a necessary D) Dumping syndrome is more common a er laparoscopic
condition or weight loss surgery? banding than with Roux-en-Y
A) BMI ≥ 40 kg/m 2 regardless o the presence o weight-related E) T ere is rarely a need or supplemental vitamins a er bariatric
disease surgery
CHAPTER 22 • Ca RE o t h E Su Rg iCa l Pa t iENt 605

Answer 22.5.3 The correct answer is “C.” T e rate o gastric


HELPFUL TIP: An Orch est ra Cond uctor?
ulcers rom NSAIDs is higher a er bariatric surgery. T e rest are
“M es r ” (yes, s e n me) s s r c y mp n -
not true. Weight loss does improve cholesterol, CAD, DM, etc.,
ed v ss m r n err p s e s n ev -
so “A” is incorrect. Additionally, there is both a psychological and
s e br n s red c n n er. i s e s me
quality-o -li e bene t (“B”). Finally, dumping syndrome is com-
nd c ns s er s r c ec n q es nd e ds 8%
monly seen a er a Roux-en-Y but not with laparoscopic band-
m re we ss n se n e s m r p.
ing (“D”) and most patients will need supplemental vitamins
(“E”). Vitamin de ciencies a er bariatric surgery are common,
especially olic acid, and de ciencies are more o a concern with Objectives: Did you learn to . . .
Roux-en-Y procedures. However, supplementation is recom- • Rec n ze e nd c ns r b r r c s r ery?
mended or all post bariatric surgery patients.
• Descr be s me d eren ec n q es r b r r c s r ery nd
e r pr s nd c ns?
HELPFUL TIP: • Pr v de n - erm m n emen e b r r c s r ery
C e s s s s c mm n er b r r c s r ery, p en nd d mp n syndr me?
espec y R x-en-Y, nd cc rs s res r pd
we ss.
CASE 22.6
A 24-year-old ale resents to your clinic with a 5-day his-
A er their o erations, they return to see you. Mr. Biggs (who tory o rectal bleeding. For several years, he has had hard
o ted or the Roux-en-Y rocedure and who is changing stools but has develo ed rectal bleeding in the last ew days.
his na e to Mr. Little) co lains o recurrent ost randial In addition, he has severe, inter ittent, and cra y abdo i-
colicky abdo inal ain, dia horesis, nausea, diarrhea, and nal ain that he thinks is due to consti ation. He re orts a
tachycardia. His wi e, who went or the la arosco ic banding, ild ever.
has no such sy to s. You ake the diagnosis o “du ing On exa ination, te erature is 37.9°C, ulse 95 b ,
syndro e.” res irations 12 b , and blood ressure 108/78 Hg. His
abdo en is nontender and without guarding or rebound
Question 22.5.4 Your advice is to: tenderness. Anosco y reveals gross blood and two internal
A) Start an anticholinergic to reduce stomach emptying he orrhoids.
B) Increase the content o simple sugars to quickly raise the
blood sugar with meals Question 22.6.1 Regarding hemorrhoids in general, which
C) Increase the size o eedings a bit in order to increase the o the ollowing is true?
amount o ood available or digestion A) Patients with hemorrhoids most commonly present or peri-
D) Separate solid rom liquid intake or at least 30 minutes (no anal burning, itching, swelling, and pain
beer with that pizza!) B) A grade III hemorrhoid can be reduced manually
E) None o the above C) I a patient under the age o 50 with rectal bleeding is ound
to have hemorrhoid on examination, urther studies are not
Answer 22.5.4 The correct answer is “D.” Separating liquid indicated
rom solid intake by 30 minutes may help to reduce dumping D) Because they are above the dentate line, strangulated inter-
syndrome. T e others are incorrect. While theoretically plau- nal hemorrhoids are not pain ul
sible, anticholinergics have not been shown to be o bene t in
dumping syndrome (nor has octreotide). Additionally, small Answer 22.6.1 The correct answer is “B.” Grade III hemor-
and requent meals devoid o si le sugars are the way to go. rhoids can be reduced manually. Hemorrhoids are normal
T e symptoms seem to be related to the rapid transit o simple vascular structures in the anal canal; however, the venules can
sugars into the bowel. Smaller volume meals without simple become engorged and symptoms such as pain, bleeding, and
sugars mitigate the problem. itching may result. wo types o hemorrhoids exist: external
hemorrhoids derived rom the in erior hemorrhoidal plexus
below the dentate line and internal hemorrhoids derived rom
HELPFUL TIP:
the anal cushions above the dentate line. Internal hemorrhoids
N r n de c enc es w n b r r c s r ery c n
occur on the le lateral, right anterior, and right posterior anal
res r m w d n ke, d n er nces, r exces-
walls and are classi ed into grades I to IV. Grade I hemor-
s ve v m n . a s s c , e w n b r ry es s re
rhoids slide below the dentate with straining but not through
rec mmended be per rmed 3, 6, nd 12 m n s
the anus. Grade II hemorrhoids protrude the anus but spon-
nd en nn y: CBC, b s c me b c p ne w c -
taneously reduce, whereas Grade III hemorrhoids must be
c m, r n s d es, v m n B1 ( m ne), v m n B12,
manually reduced. Grade IV internal hemorrhoids cannot be
e, ver enzymes, b m n, p d pr e, z nc, nd
reduced. “A” is incorrect because most patients with symptom-
c pper.
atic hemorrhoids present with painless rectal bleeding. “C” is
606 a Mil Y MEDiCiNE EXa MiNa t io N & Bo a RD REViEW

incorrect. You should consider urther evaluation (e.g., ex- AIDS or other immunode cient states, portal hypertension,
ible sigmoidoscopy and colonoscopy) in patients under the age rectal wall prolapse, and anorectal tumors. Complications o
o 50 presenting with rectal bleeding, even i hemorrhoids are hemorrhoidectomy include pain, signi cant bleeding with
present and are the likely source o bleeding. In patients older sloughing, thrombosis o external hemorrhoids, and very
than 50 with rectal bleeding, a ull colonoscopy is routinely rec- rarely sepsis with pelvic cellulitis. “B” is incorrect. Although
ommended to rule out any cancerous process. “D” is incorrect. evidence is scarce, standard o care dictates that only one hem-
Although most internal hemorrhoids do not cause pain, stran- orrhoid be ligated in a single o ce visit (due to concerns about
gulated internal hemorrhoids are very pain ul and can become excessive tissue necrosis). “D” is incorrect. Patients who pres-
necrotic and gangrenous, requiring emergent surgery. Note that ent with external hemorrhoids that are pain ul, tender, swol-
strangulated is di erent rom thro bosed. len, with bluish discoloration have thrombosis. I the patient
presents within 48 hours o thrombosis, the thrombus should
Question 22.6.2 Which o the ollowing would you NOT be expressed. It is important not to close the hemorrhoid once
consider as a treatment o this patient’s hemorrhoids? the clot is expressed. In act, a small ellipse o the hemorrhoid
A) Psyllium should be removed to acilitate continued drainage and prevent
B) Dicyclomine re-accumulation o clot.
C) Warm sitz baths
D) Short course o topical hydrocortisone You rescribe conservative treat ent or your atient’s he -
E) Increased water intake orrhoids, and since he does not return or his next sched-
uled a oint ent, you assu e he is doing well. You see hi
Answer 22.6.2 The correct answer is “B.” Dicyclomine (Bentyl, again 6 onths later. He re orts that he had indeed healed.
Antispas) is not indicated. Dicyclomine is an anticholinergic Although he still takes sylliu , he began having ain ul
and will contribute to constipation—exactly what you want to bowel ove ents with blood-streaked stool 2 days ago.
avoid in hemorrhoids. “A” and “E” are the primary modes o U on exa ination o the anus, you nd a ssure.
treatment. Psyllium, as well as a diet high in ber and water, will
reduce straining and thus reduces intra-abdominal pressure. Question 22.6.4 All o the ollowing ndings would lead
“C,” warm baths or showers (40°C), have been shown to reduce you to consider Crohn disease EXCEPT:
anal canal pressures. “D,” a short course o topical hydrocorti- A) Posterior midline ssure
sone (e.g., Anusol HC), may be o bene t. Long-term topical B) Painless ssure
steroids are contraindicated. Finally, good hygiene and analge- C) Multiple ssures
sia should be prescribed as needed. D) Nonhealing ssure

Answer 22.6.4 The correct answer is “A.” T e posterior (dorsal)


HELPFUL TIP: midline is where solitary ssures, unrelated to IBD, are typically
M s symp m c em rr ds resp nd c nserv - located. Fissures in any other location should raise suspicion or
ve me s res nd s r ery s d n be per rmed n- Crohn disease. “B,” “C,” and “D” are also suggestive o Crohn
ess c nserv ve me s res r er nd c ns ex s disease.
(e. ., s r n n).
Question 22.6.5 In a patient with an uncomplicated, initial
anal ssure, what do you recommend or f rst-line therapy?
Question 22.6.3 Which o the ollowing is true about treat- A) Lord dilation
ing hemorrhoids surgically? B) Botulinum toxin injections
A) Irritable bowel syndrome is a relative contraindication to C) opical nitroglycerin
hemorrhoid surgery D) Oral psyllium
B) It is best to ligate all hemorrhoids in a single o ce visit E) Oral ni edipine
C) Band ligation results in sloughing o hemorrhoid in about 1
to 2 weeks Answer 22.6.5 The correct answer is “D.” All o the options
D) Following excision, thrombosed external hemorrhoids are employed or treating anal ssures. However, in patients
should be closed to prevent bleeding with an uncomplicated, initial anal ssure, it seems prudent
to initiate conservative therapy (e.g., psyllium, dietary ber,
Answer 22.6.3 The correct answer is “C.” Rubber-band liga- water, and warm soaks) prior to proceeding to more invasive
tion generally results in the sloughing o the hemorrhoid in 1 to measures. Most ssures will respond to conservative mea-
2 weeks. “A” is incorrect. In ammatory bowel disease (IBD)— sures. Generally, healing takes 2 to 4 weeks. In addition to the
not irritable bowel syndrome (IBS)—is a relative contraindi- treatments listed, topical diltiazem and topical ni edipine are
cation to the surgical treatment o hemorrhoids. Other con- also use ul as are various surgical approaches. Lord dilation
traindications to o ce-based hemorrhoidectomy procedures deserves special mention as a relatively arcane procedure or
include bleeding diathesis, pregnancy and the period immedi- stretching the anal sphincter muscle (under anesthesia, we
ately postpartum, anorectal ssures, active anorectal in ections, hope!).
CHAPTER 22 • Ca RE o t h E Su Rg iCa l Pa t iENt 607

Question 22.7.1 Regarding breast lumps, which one o the


You note that this atient’s ssure is dee , ulcerating, and ollowing is FALSE?
located at the le lateral as ect o the anus. Given this exa i- A) Abnormal screening mammography is the most common
nation, you are concerned about Crohn disease. You brie y presentation o breast carcinoma
consider what you know about IBD. B) Cysts are more common in premenopausal women than
postmenopausal women
Question 22.6.6 Which o the ollowing is true o IBD? C) A history o broadenoma is associated with an increased
A) Ulcerative colitis is primarily a diagnosis o young males risk o breast cancer
B) Crohn disease can be isolated to the colon D) A radiographic oil cyst is pathognomonic or at necrosis
C) Ulcerative colitis is generally associated with deep colonic
ulcerations and transmural in ammation while those o Answer 22.7.1 The correct answer is “A.” T e most common
Crohn disease are more super cial presentation o breast carcinoma is a breast lump elt by the
D) Crohn disease is more common in A rican Americans, while patient. Breast masses can be cysts, broadenomas, thickened
ulcerative colitis is more common in Caucasians areas with brocystic change, at necrosis, and carcinoma.
“B” is true. Cysts primarily present when women are pre-
Answer 22.6.6 The correct answer is “B.” Although o en menopausal and are uncommon in the postmenopausal state,
thought o as a disease o the entire GI tract, Crohn disease can unless the woman is taking hormone replacement. Cysts are
be isolated to the colon. “A” is incorrect because ulcerative colitis well-demarcated, mobile, and rm. T e diagnosis is con-
is evenly distributed between men and women with a similar rmed with aspiration o nonbloody uid ollowed by com-
incidence in each. “C” is incorrect. Crohn disease is associated plete resolution o the mass. Fibroadenomas occur between
with deeper ulcerations and transmural in ammation leading the ages o 20 and 50 and are described as rm, rubbery, and
to stulae and strictures, etc. “D” is incorrect. In general, IBD mobile. T ey can be con rmed by characteristic ndings on
is more common in Caucasians than other races. See Chapter 7 ultrasound and ne needle aspiration (FNA) i necessary. “C”
or more questions about Crohn disease. is also true. While historically believed to be entirely benign,
Objectives: Did you learn to . . . broadenomas are associated with a small but signi cant
• C r c er ze em rr ds b sed n c n? increased risk or breast cancer. “D” is true. A radiographic
• g r de n ern em rr ds b sed n sever y? oil cyst (a circumscribed mass o mixed so -tissue density
• M n e em rr ds w c nserv ve nd s r c re - and at with a rim that is o en calci ed) is due to at necrosis,
men s? which occurs in areas o the breast that have been subject to
• t re n nc mp c ed n f ss re? trauma, surgery, in ection, or radiation therapy. About hal
o the time, at necrosis has no precipitant. Oil cysts are most
• Rec n ze n f ss res s p en s ns Cr n d se se?
common in the super cial aspects o pendulous breasts o
obese women. When an oil cyst is seen radiographically, no
urther evaluation is needed.
CASE 22.7
A 58-year-old e ale resents to your clinic or a lu Your atient is concerned about her a ily history o breast
ound on routine breast sel -exa ination. Her older sister cancer. She asks about genetic testing.
died ro breast cancer, and she is very concerned about
the ossibility o breast cancer in hersel . She never isses Question 22.7.2 You are able to tell her that the BRCA 1
her onthly breast exa ination and notes she has never gene is associated with which o the ollowing?
elt this lu be ore. She rst noticed the lu 2 weeks A) Breast cancer
ago, and it has not changed in size or consistency since B) Uterine cancer
that ti e. Consistent with her ersonal health care goals C) Ovarian cancer
but totally inconsistent with any current guidelines, she D) A and B
has had yearly a ogra s since age 40 that have always E) A and C.
been nor al. She denies any weight loss or atigue and
re orts being ost eno ausal or the last 5 years. She re- Answer 22.7.2 The correct answer is “E.” T e BRCA (BReast
viously took co bination hor one re lace ent thera y, CAncer) 1 and 2 genes are related to amilial breast cancer, and
which she discontinued last year. BRCA 1 is also associated with an increased risk o ovarian can-
On exa ination, her breasts a ear sy etrical with cer. T ey are not associated with uterine cancer. T e use ulness
no skin abnor alities. T e ni les are sy etric in size, o testing or these genes in primary care is questionable, and
sha e, and color without retraction or discharge. You al- they cannot be relied upon or general screening or diagnostic
ate a s all, ea-sized thickening in u er outer quad- purposes.
rant o the right breast. T is is the lu that she noticed
2 weeks ago. It is xed to the dee as ect o the chest wall, You are sus icious that this atient’s ass ay be cancer, and
so you have a hard ti e delineating whether the borders you order a ogra hy and ultrasound. However, these
are s ooth. studies show no ass. When your atient returns to discuss
608 a Mil Y MEDiCiNE EXa MiNa t io N & Bo a RD REViEW

her test results, you exa ine her again. T e ass is still al- the lesion appears benign both radiographically and cytologi-
able and essentially unchanged. cally, it still warrants urther ollow-up in 3 months by clinical
examination. “D” is correct. All women with a palpable breast
Question 22.7.3 The next best step in management o this mass who desire an excisional biopsy can proceed directly to
patient is: de nitive removal, regardless o age.
A) Breast examination and mammogram every 6 months
B) Breast examination and mammogram every 3 months You er or an FNA, which the cyto athologist reads
C) Re erral to a surgeon to consider excisional biopsy as “ robable alignancy.” You recall ro revious vis-
D) Ultrasound-guided needle biopsy its that the atient’s older sister had breast cancer. Addi-
E) Return to normal screening tionally, your atient went through eno ause at age 53,
and used hor one re lace ent thera y or 5 years. You
Answer 22.7.3 The correct answer is “C.” A palpable mass that obtain urther history, including a history o enarche at
is suspicious or cancer cannot be ignored even in the presence 14 years o age and the act that the atient has never been
o negative radiologic studies. No other option is acceptable, regnant.
nor would any other option be de ensible i this patient were
to develop overt breast cancer. “D” would be a viable option i a Question 22.7.5 Which o these actors DOES NOT contrib-
mass were identi ed on ultrasound, but none was. ute to an increased risk o breast cancer in THIS patient?
A) Her nulliparity
When aking diagnostic decisions regarding breast asses, B) Her age at menarche
age atters. C) Her amily history
D) Her age at menopause
Question 22.7.4 Which o the ollowing is NOT TRUE about E) Her history o hormone replacement
how a diagnostic evaluation should proceed?
A) In women < 40 years o age, lesions that appear benign on Answer 22.7.5 The correct answer is “B.” T e risk o breast
ultrasound may be ollowed simply by a repeat examination cancer is roughly associated with the li etime exposure to
in 3 months estrogen. T is patient’s age at menarche is in the mid-to-
B) In women > 40 years o age, lesions that appear benign on late age range and is thus not a risk actor or breast cancer.
ultrasound and cytological analysis need no ollow-up Younger age at menarche (e.g., 10 years old) is associated with
C) In women > 40 years o age where a lesion is palpated but an increased risk o breast cancer. Nulliparity and greater age
not seen on mammogram or ultrasound, excision should at rst pregnancy are associated with an increased risk, as is
ollow i clinical suspicion dictates greater age at menopause. A history o a rst-degree relative
D) In women > 40 years o age who have a palpable lesion and with breast cancer is a strong risk actor. Finally, as demon-
desire de nitive removal, no urther testing is necessary pri- strated in the Women’s Health Initiative, estrogen/progesterone
or to excisional biopsy replacement therapy (HR ) is associated with an increased risk
o breast cancer (an excess o about 8 breast cancers per 10,000
Answer 22.7.4 The correct answer is “B.” Women over the age women treated with HR ).
40 should be ollowed up in 3 months even with negative cytol-
ogy. Clinical diagnosis o carcinoma has accuracy o 60% to While you wait or de nitive athology results, you consider
85%. Extent o evaluation, however, is driven by clinical suspi- the di erent ty es o breast cancer that this atient ight
cion. Radiologic studies should be ordered as indicated. A nee- have.
dle biopsy either by FNA or core needle biopsy is the next step
in evaluation o solid tumor masses. Needle diagnosis o ers the Question 22.7.6 Regarding various types o breast cancer,
advantages o being simple, quick, inexpensive, relatively non- all o the ollowing are true EXCEPT:
invasive, highly available, and an accurate way o diagnosing A) Cystosarcoma phyllodes tumors are not always malignant
atypical cells. B) In ltrating lobular carcinoma is the most common histo-
Because o the risk or carcinoma, algorithms or evaluation logical type o invasive breast carcinoma
o solid tumors di er between women < 40 years versus those C) Paget disease o the breast clinically appears eczematous
40 years and older. Women < 40 years who desire observation D) Sarcomas, lymphomas, melanomas, and angiosarcomas are
should be evaluated by ultrasound and/or FNA. I either is sus- all possible causes o cancer in the breast
picious, surgical excision should be per ormed. I there is a neg-
ative FNA, the patient can be ollowed up in 3 months. Answer 22.7.6 The correct answer is “B.” In ltrating ductal
In women 40 years o age or older, palpable masses should (not lobular) carcinoma is the most common histological type
urther be evaluated by ultrasound and mammography. I o invasive breast cancer. Invasive breast carcinoma includes
the lesion cannot be identi ed radiographically, it should be a wide variety o histological diseases. In ltrating ductal car-
removed by excision i clinical suspicion dictates. I the lesion cinoma accounts or 65% to 80% o breast cancers, whereas
appears benign by radiography, it can be ollowed by FNA. in ltrating lobular carcinoma is the second most requent,
T ose with atypia on FNA should be re erred or excision. I accounting or 10% o breast cancers. “A” is true. Cystosarcoma
CHAPTER 22 • Ca RE o t h E Su Rg iCa l Pa t iENt 609

phyllodes tumors, which have a clinical presentation similar Objectives: Did you learn to . . .
to broadenomas but are more rapidly growing, may be either • g ener e d eren d n ses r bre s m sses?
malignant or benign. Yet the malignant nature o cystosarcoma • Ev e p en w bre s m ss?
phyllodes is sometimes di cult to determine on cytology, and • iden y r sk c rs r bre s c ncer?
the lesions may require a wide excision. “C,” Paget disease o the • Descr be sever ypes bre s c ncers?
breast, is a rare orm o breast cancer with eczematous changes • Descr be c rs re sed es b s pr n s s n
o the nipple, including itching, erythema, and nipple discharge. bre s c ncer?
“D” is true as well. Many other malignancies may occur in
the breast, including sarcomas, lymphomas, melanomas, and
angiosarcomas. QUICK QUIZ: BREa St Ca NCER RiSK

T e athology results are nal and show in ltrating ductal A 50-year-old emale presents or a routine physical. She wants
carcino a. Your atient will see a breast surgeon next week. to know her risk o developing breast cancer. You want to give
As you await the results o surgery, you consider her rog- her a more accurate picture, and you decide to use the Gail
nosis. model.

Question 22.7.7 All o the ollowing are avorable prognos- When taking her history, you must ask about all o the ollowing
tic indicators in breast cancer EXCEPT: EXCEP :
A) Hormone receptor negative A) Number o rst-degree relatives with breast cancer
B) Absence o axillary nodal involvement B) Age at rst menstrual period
C) Low-grade tumor C) Number o breast biopsies
D) Pure tubular, mucinous, or medullary histological types D) Age at rst live birth
E) umor size < 1 cm E) Number o live births

Answer 22.7.7 The correct answer is “A.” Patients whose The correct answer is “E.” T e Gail model is a computer pro-
tumors are hormone receptor negative have worse outcomes gram that estimates a woman’s chance o developing breast
than do patients whose tumors are hormone receptor positive. cancer. Factors that a ect the score include the number o
“B,” absence o axillary nodal involvement, is obviously a bet- rst-degree relatives with breast cancer, current age, age o
ter prognostic actor than the presence o nodal involvement. rst menstrual period, number o breast biopsies, BRCA gene
In act, axillary lymph node status is the single most important status, age at rst live birth, race, and history o atypia in a
predictor o overall survival in breast cancer. “C,” low tumor biopsy. T is tool may be use ul in patients who are candidates
grade, is also a good prognostic actor. “D,” patients with a single or tamoxi en or breast cancer prophylaxis. However, the Gail
cell type have a better prognosis as well. “E” is true. A very use- score is uch better at redicting the likelihood o breast
ul predictor o tumor behavior is tumor size, and tumor size cancer in o ulations than in any individual. So, realize that
< 1 cm is a positive prognostic sign. it has li itations. T e National Cancer Institute maintains a
website or the Gail model calculation, located at http://bcra.
nci.nih.gov/brc/q1.htm.
HELPFUL TIP:
t m rs c n be e er es r en r pr es er ne recep- CASE 22.8
r p s ve r ne ve nd h er2/ne ( yr s ne-pr e n
An ortho edic colleague asks you to consult on a 64-year-old
k n se recep r) p s ve r ne ve. t m rs re
ale rior to an elective total hi re lace ent. T e surgery
p s ve r es r en nd/ r pr es er ne recep rs re
is scheduled or 3 onths ro now. T e atient is a s oker
e m s ke y resp nd rm n er py (e. .,
with diabetes ellitus ty e 2 and has recently had a cardiac
m x en). t m rs re p s ve r h er2/ne re
catheterization that showed signi cant, but nonby assable,
m s ke y resp nd yr s ne k n se n b rs (e. .,
coronary disease. He is asy to atic and is able to walk
h ercep n).
stairs without dys nea or chest ain. T e surgeon would like
so e erio erative reco endations.

HELPFUL (BUT DISTURBING) TIP:


Question 22.8.1 You would recommend all o the ollowing
i rns ppr x m e y 20% w men w
EXCEPT:
nder bre s -c nserv n s r ery r bre s c ncer
A) T e patient should stop smoking 4 weeks be ore surgery
need repe s r ery. a dd n y, 15% se w
B) T e patient should have preoperative and postoperative
p s ve m r ns d d n r re-exc s n. D s rb n
β -blockers i the pulmonary status allows it
w m j r mp c ns r n rmed c nsen (JAMA.
C) T e patient should have a ECG done
2012;307:467).
D) T e patient should have his hemoglobin/hematocrit drawn
E) T e patient should have his creatinine measured
610 a Mil Y MEDiCiNE EXa MiNa t io N & Bo a RD REViEW

TABLE 22-1 PREOPERATIVE STUDIES AND THEIR INDICATIONS


Tests Indications
B n/cre n ne o ver 60; s ry ren , c rd c, r v sc r d se se

CBC/h &h P ss b e em c r n ec s pr cess; s n f c n b d ss pred c ed

C n s d es S m ver d se se, s ry c p y, p ss b e DiC, n c n, c b se

ECg /CXR a s nd c ed by s ry nd p ys c (e. ., ex cerb n p m n ry d se se w c )

E ec r y es D re c se, s ry ren r c rd c d se se, p ss b e de ydr n by s ry r p ys c

g c se D be cs, bese p en s, nder n v sc r pr ced res, er re s n r ncre sed c se (e. ., s er ds)

l ver enzymes h s ry ver d se se r s m ver d se se

u r ne β - Cg i nd c ed by s ry

u r n ys s Pre n ncy, d be es, r c s r ery, symp m cp en s

a d p ed r m ee y Ma , C ns CS, e . Pre per ve es n be re n nc rd c s r ery: de nes nd rec mmend ns. Am Fam Physician. 2013;87(6):
414–418.

Answer 22.8.1 The correct answer is “A.” Paradoxically, unless Answer 22.8.2 The correct answer is “E.” All o the modali-
patients stop smoking 8 weeks or more be ore surgery, the risk ties listed above can prevent the development o DV . Aspirin
o adverse pulmonary outcomes is increased. T e cause o this and low-dose war arin (target INR 1.5) are also e ective but less
phenomenon remains unclear but may occur because the cilia so than heparin or enoxaparin. Sequential compression devices
are able to mobilize material in the lungs. “B” is true. T e use (SCDs) may also be used. Subcutaneous heparin may require
o pre-, intra-, and postoperative β -blockers is well supported ID dosing in larger patients.
or patients with CAD. One o the most serious intraoperative
events is a myocardial in arction. β -Blockers have been shown HELPFUL TIP:
in multiple studies to reduce this risk and to improve outcomes Recen d s es s r ded c mpress n s ck-
i the patient has an elevated risk or a myocardial in arction. n s re m r n y bene c bes r DVt preven n
Note, however, that a β-blocker should be started well in nd m y e d ncre sed sk n bre kd wn, e c. r s
advance o the surgery and not just on the day o surgery. re s n, n erm en pne m c c mpress n w SCDs
It is critical to assure that the patient is on a stable dose o a s pre erred (Ann Surg. 2010;251(3):393–396 nd Lancet.
β -blocker at the time o surgery. Starting the β -blocker too close 2009;373:1958–1965).
to surgery may increase the risk o stroke, likely secondary to
intraoperative hypotension. While “C,” “D,” and “E” are true, a
lot o other routine preoperative assessments are not supported T e atient undergoes his hi re lace ent, and his osto era-
in the literature. See able 22-1 and able 22-2 or the appropri- tive ECG is nor al. Four hours a er surgery, he develo s ild
ate evaluation o the preoperative patient. res iratory distress, a ever, and cough. On chest x-ray, there is a
right lower lobe in ltrate. T ere is no evidence o uid overload.
Question 22.8.2 Which o the ollowing is ef ective or the
prevention o postoperative DVT? Question 22.8.3 Which o the ollowing is the most likely
A) Early mobilization cause o this patient’s ever and in ltrate?
B) Enoxaparin 30 mg subcutaneously every 12 hours A) Pneumococcus
C) Enoxaparin 40 mg subcutaneously every 24 hours B) Gram-negative organisms
D) Heparin 5,000 Units subcutaneously every 12 hours C) Atelectasis
E) All o the above D) Aspiration pneumonitis
E) Aspiration pneumonia
TABLE 22-2 MAYO CLINIC PREOPERATIVE GUIDELINES
Answer 22.8.3 The correct answer is “D.” In the hours a er sur-
Age in Years Studies Indicated
gery, an aspiration pneumonitis would be the most likely cause
a e < 40 N r ne pre per ve ev n o this patient’s current ndings. Aspiration pneumonitis occurs
req red when there is aspiration o gastric contents with a pH o less than
a e 40–59 ECg , cre n ne, nd c se 2.5. In order or aspiration pneumonitis to develop, the volume
o aspirate needs to be at least 1 to 4 mL/kg o stomach contents.
a e 60 r der ECg , c es r d r p , CBC, cre n ne, Aspiration pneumonitis develops over a matter o hours. In con-
c se
trast, pneumococcal and Gram-negative pneumonias generally
CHAPTER 22 • Ca RE o t h E Su Rg iCa l Pa t iENt 611

develop several days a er surgery (unless a subclinical pneumo- C) It should be treated with antibiotics that cover or anaerobes
nia was present at the time o surgery). Aspiration pneumonia is D) It tends to resolve in about 7 days
caused by anaerobes and mixed ora and develops slowly over
days to a week. Atelectasis warrants special mention. Atelectasis Answer 22.8.5 The correct answer is “C.” Aspiration pneumo-
does not cause ever. Both atelectasis and ever occur requently nitis is a chemical process that is unrelated to in ection. T us,
in the postoperative period, but their occurrence together is aspiration pneumonitis does not need to be treated with antibi-
most likely due to chance. T e old adage about atelectasis and otics at all (although it usually is as a practical matter—since we
ever has been shown to be untrue. T us, in the postsurgical are chicken). T e rest o the options are true.
patient with ever, look or another cause besides atelectasis.

Question 22.8.4 O the ollowing, which generally IS NOT a HELPFUL TIP:


cause o postoperative ever in the rst 48 hours? t e erm “ sp r n pne m n ” s sed d eren y n
A) Malignant hyperthermia e d eren p r s e er re. t s e ds s me
B) Surgical “trauma” (e.g., cutting through muscle) c n s n. S me se e erm nc de pne m -
C) Wound in ection n s c sed by sp r n. M ny e c ses nc ded
D) Hyperthyroidism n s de n n re c sed by m ny e s me r-
E) Drug ever n sms c se c mm n y- cq red pne m n ,
nc d n pneumococcus nd Haemophilus in luenzae.
Answer 22.8.4 The correct answer is “C.” Wound in ections t ese p en s end ve ever nd n r e, e c.,
generally are not ound in the rst 48 hours a er surgery. All o w c deve p w n 2 d ys e sp r n. r
the rest can be ound either immediately a er surgery (malig- ese p en s, e ppr pr e n b cs s d n-
nant hyperthermia, hyperthyroidism, drug ever, etc.) or soon c de en s c ver sp - cq red r n sms,
therea er ( ever rom surgical trauma secondary to the release espec y g r m-ne ve r n sms, nc d n Pseu-
o cytokines). able 22-3 summarizes the time course and causes domonas.
o postoperative ever. o er r ps m y de ne sp r n pne m n s
n er b c n ec ns res r m sp r n. t ese
T e atient’s chest radiogra h is consistent with as iration p en s ve m re nd en c rse, w nse
neu onitis (in ltrate in the right lower lobe . . . yes, lower symp ms ver d ys weeks. g ener y, ese p en s
lobe is ore co on). ve p r en sp m, w wer be nv vemen
m s c mm n. t e pper bes m y be nv ved e
Question 22.8.5 Which o the ollowing is NOT TRUE with p en sp r es w e rec mben . t ese p en s m y
regard to aspiration pneumonitis? ve p ym cr b n ec n nc d n Peptostrep-
A) It can progress to ARDS tococcus, Fusobacterium, Bacteroides, nd Prevotella.
B) Patients with aspiration pneumonitis present with ever, t ese p en s s d be re ed w n b cs s c
dyspnea, bronchospasm, and hypoxia s c nd myc n, mp c n/s b c m (u n syn), r
m x c n/c v n e (a men n); v d me r n d -
z e s s n e en d e re r e.
TABLE 22-3 CAUSES OF POSTOPERATIVE FEVER
S , w en y re d e er re b sp r n
Immediate (within hours o surgery): pne m n , be s re y kn w w c de n n e -
• Dr s r b d pr d c s: g ener y yp ens n, r s , e c. rs re s n , bec se e descr p n e d se se
• t r m r m s r ery r be re s r ery: Ev den r m s ry
nd e re men s v ry.
• M n n yper erm : W n 30 m n es nes es nd c n
b m y be rs

Acute (f rst week a ter surgery):


HELPFUL TIP:
• N s c m n ec ns r ex ens n pre per ve n ec n
• C. dif cile n ec n Dr s r se e s r c ph (h 2-b ckers, pr n
• in b n p mp n b rs) ncre se e r sk p s per ve
• a sp r n pne m n ( nd Clostridium di icile). N rm y, e c d
• u t i: Espec y c r n c ndwe n r r cm n p n ph e s m c preven s c n z n w p -
• S r c s e n ec n ener y >1 week c se ever b m y
en c b c er . t s de ense s c mpr m sed w en e
cc r n 1s week. B r p a Strep nd Clostridium m y cc r
w n rs s r ery. s m c ph r ses. t s, n ec n w sp r n s m re
• a s : P ncre s, c w dr w , PE, Mi, r mb p eb s, ke y.

Subacute (1–4 weeks a ter surgery):


• iV nd cen r c e er s e n ec ns
• a n b c ss c ed d rr e , nc d n C. dif cile n ec n You treat the atient with uids and tracheal suction. How-
• Dr ever: β - c ms, s , ep r n, e c. ever, he re ains ebrile and tachycardic at about 128 b .
• DVt , PE, emb r m n b nes, p s c n, c e c es syn- T ere is no evidence o dehydration at this oint and he
dr me (SSa )
see s euvole ic.
612 a Mil Y MEDiCiNE EXa MiNa t io N & Bo a RD REViEW

Question 22.8.6 Which o the ollowing would be the most


HELPFUL TIP:
appropriate rst step in the treatment o tachycardia in this
h ep r n c n s c se p s per ve ever.
postsurgical patient?
A) Oral or rectal aspirin
B) Oral or rectal acetaminophen
C) IV β -blockers
D) IV uids A er you sto the atient’s he arin, the thro bocyto enia
and hy erkale ia begin to resolve.
Answer 22.8.6 The correct answer is “B.” T e initial treatment You start the atient on inter ittent leg co ression with
o this patient is acetaminophen. Reducing the ever and meta- SCDs.
bolic stress will result in a reduction in the heart rate. “A” is not
the best choice since this patient is postsurgical. Giving aspi-
rin, an antiplatelet agent, may result in increased postoperative Question 22.8.8 Which o the ollowing is TRUE about the
bleeding. “C,” IV β -blockers, can be used and would be appro- use o heparin in patients with a history o heparin induced
priate i the patient was having ischemic symptoms and needed thrombocytopenia?
an immediate reduction in pulse. “D,” IV uid, is incorrect in A) A low-molecular-weight heparin (LMWH) should be used
this patient who is already well hydrated (as stipulated in the in patients with a history o HI S
question). Note that IV uids are appropriate in postoperative B) Heparin can be re-introduced once the HI S has been
tachycardia i the patient is dehydrated. treated with steroids
C) T e use o lepirudin and argatroban are contraindicated in
T e atient’s ul onary status has i roved. T e surgeon the treatment o HI S
notices that the atient has hy erkale ia and thro bocyto- D) Un ractionated heparin can be used during cardiopulmo-
enia. T e atient is on a nu ber o drugs osto eratively. nary bypass despite a previous history o a thrombocytope-
She is wondering which o these drugs is causing the roble . nic reaction to heparin i the patient is HI antibody nega-
tive
Question 22.8.7 You let her know that the most likely cause E) None o the above
is:
A) β -Blockers Answer 22.8.8 The correct answer is “D.” Heparin can be
B) Albuterol used during cardiopulmonary bypass despite a previous his-
C) Aspirin tory o HI S. T e theory is that the development o enough
D) Morphine antibodies to reproduce thrombocytopenia takes several days
E) Heparin and patients will generally be on cardiopulmonary bypass
or only a matter o hours. T us, heparin can be used dur-
Answer 22.8.7 The correct answer is “E.” Heparin can cause ing cardiopulmonary bypass even i there is a history o prior
both hyperkalemia and thrombocytopenia (HI S, or heparin- heparin-induced thrombocytopenia. “A” is incorrect. LMWH
induced thrombocytopenia and thrombosis syndrome). should be avoided in patients with strongly suspected (or
Hyperkalemia is caused by heparin’s ability to block the aldo- con rmed) HI S. It is recommended to consider an alter-
sterone system and usually requires prolonged heparin use native, nonheparin anticoagulant (e.g., danaparoid, lepiru-
(although in diabetics and those with renal ailure it may din, argatroban, ondaparinux, bivalirudin) over the urther
occur more rapidly because o poor reserve). T rombocyto- use o un ractionated heparin or LMWH. “C” is incorrect. In
penia is caused by the development o antiplatelet antibod- act, the treatment o patients who need anticoagulation a er
ies and occurs 5 to 10 days a er the start o heparin therapy HI S includes lepirudin and argatroban. T ese two agents
and is associated with thromboembolic phenomenon. Plate- are direct thrombin inhibitors that do not cross-react with
lets can get as low as 30,000 platelets/µL. Stop the heparin heparin. Note that there are now a number o oral alternatives
and anticoagulate with a nonheparin agent (e.g., bilivarudin). to heparin including dabigatran, apixaban, and rivaroxaban.
A non–immune-mediated thrombocytopenia also occurs Any o these would also be appropriate although dabigatran
in up to 20% o individuals who receive heparin. T is mild requires 7 days o overlap with heparin so would not be the best
thrombocytopenia occurs within the rst 4 days o heparin choice here….
administration, resulting in a nadir o 100,000 platelets/µL
and no clinical consequences. “A,” β -blockers, may result in
Objectives: Did you learn to . . .
hyperkalemia at high doses (via a β 1-receptor e ect inhibiting
• Per rm pre per ve med c ev n?
renin release) but do not cause thrombocytopenia. “B,” alb-
uterol, can cause hypokalemia (as can other catecholamines) • g ener e d eren d n s s r p s per ve ever?
by driving potassium intracellularly. However, albuterol does • D n se nd re sp r n pne m n s?
not cause hyperkalemia or thrombocytopenia. Likewise, “D” is • Rec n ze s me c mp c ns ep r n se?
incorrect because morphine is not associated with hyperkale- • Emp y ppr pr e DVt pr p y x s me s res?
mia or thrombocytopenia. • M n e p en w ep r n- nd ced r mb cy pen ?
CHAPTER 22 • Ca RE o t h E Su Rg iCa l Pa t iENt 613

You correctly reply:


QUICKQUIZ: g ENERa l a NESt h ESia o RSu Rg ERY A) “His risk or atrial brillation is less than it would be i he were
undergoing valve replacement simultaneously with CABG.”
Regarding anesthesia evaluation both preoperatively and intra- B) “O the potential arrhythmias, he is most likely to encounter
operatively, which one o the ollowing is FALSE? bradycardia, which is the most common postoperatively.”
A) T erapeutic β -blockade should be administered preopera- C) “Nonsustained ventricular tachycardia is highly unlikely in
tively in vascular surgery patients who have underlying car- this setting.”
diac disease in order to reduce perioperative and long-term D) “Keep his potassium low, around 3.0 mEq/L, and he will be
risks o cardiac events. less likely to experience tachyarrhythmias.”
B) Minimum potassium be ore proceeding with elective coronary
artery bypass gra (CABG) surgery should be 3.5 mEq/L. The correct answer is “A.” With CABG, the risk o postopera-
C) American Society o Anesthesiologists (ASA) Class IV des- tive atrial utter or brillation is around 30%. T at risk almost
ignation includes patients who have well-controlled major doubles when valve replacement is accomplished simultane-
systemic disease. ously with CABG. “B” is incorrect because bradyarrhythmias
D) Risks o anesthesia include allergic drug reactions, ailure to occur less requently than tachyarrhythmias. “C” is incorrect.
intubate and provide adequate oxygenation and ventilation, Nonsustained ventricular tachycardia is extremely common
nerve damage, and malignant hyperthermia. in the immediate postoperative period. “D” is also incorrect.
Plasma potassium levels < 3.5 mEq/L are associated with an
The correct answer is “C.” ASA Class IV includes patients who increased risk o tachyarrhythmias.
have a systemic disease that is li e threatening and NO well
controlled. All the other options are true. “A” is true, but the CASE 22.9
β -blocker should be started well in advance o the surgery and A 15-year-old ale resents to your o ce with a 3-day his-
the patient should be on a stable dose at the time o the sur- tory o diarrhea and right lower quadrant abdo inal ain.
gery. Starting the β -blocker at the time o the surgery worsens He has tenderness in the right lower quadrant with guard-
outcomes as noted above. “B” is a correct statement. T ere are ing and rebound. He re ains a ebrile and has been hungry,
increased perioperative arrhythmias and need or cardiac resus- scar ng down ve bacon-wra ed waf es or break ast (but
citation in patients with serum potassium levels < 3.5 mEq/L. less than his usual 6…). T e atient has no other signi cant
“D” is also a correct statement. Anesthesia risks include allergic history. You decide that this atient ight have a endicitis,
drug reactions, ailure to intubate and provide adequate oxygen- so you draw so e labs. T e white blood cell count is nor al
ation and ventilation, nerve damage, and malignant hyperther- (7,500/ 3
), as is the urinalysis.
mia, among others. Mortality rate rom anesthesia is surmised
to be about 1 to 2 per 10,000 patients. T e ASA classi cation Question 22.9.1 Which o the ollowing is true about appen-
system is outlined in the able 22-4. dicitis?
A) A normal white count e ectively rules out the diagnosis o
appendicitis.
QUICK QUIZ: t h E Rh Yt h M iS g o NNa g Et Ch a B) T e majority o patients with appendicitis present with ever.
C) T e absence o anorexia e ectively rules out appendicitis.
A 60-year-old male patient o yours is planning to undergo D) A ecalith is ound on radiograph in the majority o patients
CABG. A er you per orm a physical examination and labora- with appendicitis.
tory tests, you discuss his case with the surgeon. She asks i you E) None o the above.
will help to manage him postoperatively, and you agree. She
then asks i you are aware o his risk o arrhythmia during the Answer 22.9.1 The correct answer is “E.” None o the above is
postoperative period. true. aking these in order, 10% o patients with appendicitis

TABLE 22-4 ASA PHYSICAL STATUS CLASSIFICATION


Risk o Operative
Class Description Mortality (%)
C ss i he y pers n < 0.1

C ss ii P en w m d sys em c d se se 0.5

C ss iii P en w severe sys em c d se se sc n r ed 4.4

C ss iV P en w severe sys em c d se se s nc n r ed 23.5


nd re e

C ss V P en w s n ke y s rv ve r 24 rs, w r 50.8
w s r ery
614 a Mil Y MEDiCiNE EXa MiNa t io N & Bo a RD REViEW

have a normal white count, a minority o patients with appen- A) Erythrocyte sedimentation rate (ESR)
dicitis present with ever (15% in one study), only 75% o B) C-reactive protein (CRP)
patients with appendicitis complain o anorexia, and a radio- C) Abdominal ultrasound
graphic ecalith is ound in only a small minority o patients. D) Abdominal C scan
E) Colonoscopy
Question 22.9.2 Which o the ollowing is specif c or appen-
dicitis? Answer 22.9.4 The correct answer is “D.” T e test most likely
A) Obturator sign to arrive at a diagnosis in this patient is a C scan o the abdo-
B) Psoas sign men looking at the appendix. “A” and “B” are incorrect. Both the
C) Rovsing sign CRP and the ESR are nonspeci c markers o in ammation and
D) enderness at McBurney’s point are not help ul in the diagnosis o appendicitis. “C,” an ultra-
E) None o the above is speci c or appendicitis sound, can be used and is o en recommended as the f rst test in
pediatric patients to avoid the ionizing radiation associated with
Answer 22.9.2 The correct answer is “E.” None o the above is C . T e speci city or ultrasound is high (95–99%). However, it
speci c or appendicitis. An obturator sign is present i there is is not as sensitive as a C scan (as varied as 60–85% depending
pain on internal and external rotation o the hip. T e obtura- on the study). Although “D” is the textbook answer, it would be
tor sign can be seen with any pelvic abscess that is in contact reasonable to start with an ultrasound, and i equivocal, one can
with the hip area, but is more commonly seen with a retroce- then move on to C or surgical consultation or serial abdomi-
cal abscess. T e psoas sign is pain on use o the psoas muscle nal examinations depending on institutional pre erence. Finally,
(e.g., li ing the leg at the hip), and it can be seen with any ultrasound can also be use ul in the emale patient in whom
in ammatory process that is in contact with the psoas muscle, other diagnoses need to be ruled out, such as ovarian pathology.
including a psoas abscess. Rovsing sign is when pain increases “E,” colonoscopy, is not particularly use ul in the diagnosis o
in an area o peritonitis when the abdomen is palpated else- appendicitis but could be used or other purposes such as looking
where. For example, in a patient with appendicitis, right lower or IBD once appendicitis is ruled out by C .
quadrant pain will be increased with palpation o the le lower
quadrant. T is is indicative o peritonitis in the area that has
increased pain, but it is not speci c or appendicitis. ender- HELPFUL TIP:
ness at McBurney’s point can be seen in a number o processes N p en s w p en ppend c s need Ct
including appendicitis, ileitis, any process in the cecum, and sc n! Those with obvious appendicitis should go di-
urinary tract in ection. rectly to the OR. Ct s d n y be sed n p en s n
w m e c n c d n s s s eq v c . t ere s s me
Question 22.9.3 Which o the ollowing is true about the n - erm r sk r m r d n exp s re, s Ct s d
treatment o pain in the acute abdomen? n be d ne w d nd c n. h wever, y
A) Early treatment with morphine will obscure the proper m s s be w re e s nd rd pr c ce n y r
diagnosis c n s m ny s r e ns n w ns s p n Ct sc n
B) reatment with pain medication invalidates in ormed pr r n e o R.
consent
C) Early treatment o pain with morphine is sa e except in chil-
dren Question 22.9.5 I this patient’s CT scan is positive or
D) Ketorolac is pre erred or patients who may undergo a surgi- appendicitis, the likelihood that he will have a normal
cal procedure appendix removed at appendectomy is:
E) None o the above is true A) 0%
B) 10%
Answer 22.9.3 The correct answer is “E.” None o the above C) 20%
is correct. Early treatment o pain in the acute abdomen actu- D) 30%
ally i roves diagnostic accuracy in both children and adults. E) 50%
Ketorolac is not a good choice because o its antiplatelet e ects,
which increase the risk o bleeding intraoperatively. Pain medi- Answer 22.9.5 The correct answer is “B.” With the advent o
cation does not inherently invalidate in ormed consent (but C scanning, the alse positive rate (o taking normal patients
obviously can i the patient is hardly arousable). to the OR) has improved, but perhaps not as much as you would
think. T e negative laparotomy rate is still about 10%. T e nega-
Although you have thought a lot about his otential rob- tive laparotomy rate has gone down in men and the young but
le , you have not actually done anything ore to evaluate has increased in the elderly and in women. T ere is evidence that
this 15-year-old with abdo inal ain! C scanning changes the treatment plan in more than hal o
cases o suspected appendicitis. T e sensitivity o C or appen-
Question 22.9.4 The BEST test to help you arrive at a dicitis is 91% to 98%, but the speci city is as low as 75% depend-
diagnosis in this p atient is: ing on the radiologist and the population tested (range 75–93%).
CHAPTER 22 • Ca RE o t h E Su Rg iCa l Pa t iENt 615

history is signi cant or a 5-year history o diabetes. He takes


HELPFUL TIP:
glyburide, atorvastatin, and as irin (he clearly isn’t your
t ere s newer ev dence s es s uncom-
atient . . . he is not on et or in, the drug o choice in ty e
plicated ppend c s m y be re ed w n b -
2 diabetes). He has no revious surgical history. You order an
cs ns e d s r ery nd s n ss c ed w ny
ultrasound. T e ultrasound reveals a nor al aorta, but the
ncre sed c mp c ns. t e s nd rd c re n e
technician notes several stones in the gallbladder. You are con-
u n ed S es e me s p b s n rem ns p-
cerned that this atient ay have sy to atic cholelithiasis.
pendec my (JAMA. 2015;313(3):2340–2348 nd Ann
Surg. 2015;261(1):67–71).
Question 22.10.1 Which o the ollowing is true o his risk
or gallstones?
A) T is patient has an increased risk o cholesterol stones
Question 22.9.6 In general, which o the ollowing is true because he is diabetic
about appendicitis? B) T is patient’s risk o gallstones would have peaked in his
A) Pain is in the right upper quadrant in the majority o preg- ourth decade o li e
nant women with appendicitis C) T is patient’s risk o gallstones is increased because o his
B) Atypical presentations are more common in the elderly atorvastatin use
patient than in other groups D) T is patient’s risk or cholesterol stones is lower because o
C) Patients with a retrocecal appendix generally present with his advanced age
well-localized tenderness and signs o peritoneal irritation E) T is patient’s risk o gallstones is increased because he’s male
D) An appendicitis is great way to postpone your board exami-
nation—get one soon! Answer 22.10.1 The correct answer is “A.” Patients with diabe-
tes have an increased risk o gallstones when compared with the
Answer 22.9.6 The correct answer is “B.” Symptoms tend to be general population. Other associations include emale gender
atypical in the elderly. In act, elderly patients may have appen- (not male, so “E” is wrong), amily history, obesity, and certain
dicitis with a normal white count, poorly localized pain, and medical illnesses including hyperlipidemia, cystic brosis, short
absence o ever. So, maintain a high index o suspicion. “A” is bowel syndrome, parenteral nutrition, hemolytic anemia (e.g.,
incorrect. Despite classic teaching, patients who are pregnant sickle cell), and history o terminal ileum resection. “B” and “D”
tend to have “typical” symptoms with right lower quadrant pain. are incorrect because the risk o gallstones increases linearly
T is is especially true in the rst hal o pregnancy. Certainly with age. “C” is incorrect. Atorvastatin and other “statins” are
the appendix can be displaced cephalad, but the majority will not associated with an increased risk o gallstone ormation, but
still have right lower quadrant pain. “C” is incorrect. Patients clo brate and other brates are. “D” is also incorrect.
with retrocecal appendicitis will commonly complain o a dull
ache. However, signs o peritoneal irritation may be minimal or Question 22.10.2 Which o the ollowing statements is
absent. FALSE regarding the evaluation o the gallbladder?
A) Ultrasound will nd pericolic uid in only 50% o patients
with cholecystitis
HELPFUL TIP: B) An HIDA scan can be abnormal/positive (e.g., no tracer in
S me p en s ve rec rren ppend c s. t ese p - the duodenum) in patients with common bile duct obstruc-
en s w presen w m p e ep s des “ yp c ” tion and/or cholecystitis
ppend c s, w c res ve d r n bserv n. W en C) Endoscopic retrograde cholangiopancreatography (ERCP)
e ppend x s n y rem ved, s req en y sc rred has an attendant risk o pancreatitis
d wn. D) T e absence o disease on ultrasound e ectively rules out
gallbladder disease
Objectives: Did you learn to . . . Answer 22.10.2 The correct answer is “D.” While ultrasound
• Descr be e f nd n s n c e ppend c s?
is highly sensitive or stones (95% or greater), it will miss some.
• D n se ppend c s nd de erm ne w nc ry es s re “A” is a correct statement as are “B” and “C.” An HIDA (hepatic
bes sed n e p en presen n w s ns nd symp ms
iminodiacetic acid) scan can be abnormal (e.g., no tracer in the
ppend c s?
duodenum) in cholecystitis but also in any other condition in
• M n e p en w ppend c s?
which the common duct is blocked including a common duct
stone or tumor. Additionally, i there is liver disease (which may
prevent the uptake o HIDA into the gallbladder), a ull gall-
CASE 22.10 bladder, gallbladder dys unction, or spasm o the sphincter o
A 72-year-old ale resents to your o ce or a 3- onth his- Oddi, one can have a alse positive scan (e.g., no tracer in the
tory o e isodic abdo inal ain. It is ri arily located in the duodenum). MRCP (magnetic resonance cholangiopancreatog-
e igastric region and radiates to the back. It occurs both dur- raphy) or ERCP can be used to identi y stones in the common
ing the night and day and lasts about 1 hour. His ast edical duct prior to surgery.
616 a Mil Y MEDiCiNE EXa MiNa t io N & Bo a RD REViEW

occurs more o en in critically ill hospitalized patients, and you


HELPFUL TIP:
already know that this patient has stones, so his cholecystitis is
Ct s n very sens ve s dy r s nes. u r -
by de nition calculous in nature. T ere must be no gallstones
s nd s e s dy c ce r d n s n s nes
detected on ultrasound to diagnose acalculous cholecysti-
(sens v y s 25% r Ct vs. 95% r r s nd).
tis (thus the name). “C” is incorrect. His pancreatic enzymes
are normal; and although one can certainly have pancreatitis
with a normal amylase and lipase, you have a better explana-
Question 22.10.3 Regarding dif erent types o gallstones, tion with cholecystitis. “D” is incorrect. Patients with ascending
all o the ollowing are true EXCEPT: cholangitis usually have markedly elevated transaminases and
A) Cholesterol stones are associated with obesity and hyperlip- an obstructive laboratory pattern (e.g., elevated bilirubin, alka-
idemia line phosphatase, and pancreatic enzymes). “E” is very unlikely
B) Black pigment stones are associated with cirrhosis in this patient who has ndings more consistent with intra-
C) Brown pigment stones are associated with liver uke in ec- abdominal pathology, but you should always consider cardiac
tion (and other thoracic organ) etiologies in older patients present-
D) Blue pigment stones are associated with being an avid an o ing with abdominal complaints.
the St. Louis Blues hockey team and/or the Blue Man Group
Question 22.10.5 Regarding the dif erential diagnosis o
Answer 22.10.3 The correct answer is “D.” While we have complicated cholelithiasis, which o the ollowing is true?
noticed that beer-swilling hockey ans might be prone to devel- A) Empyema o the gallbladder is primarily a disease o the
oping gallstones, there are no studies that show this. Besides, elderly and carries a high mortality rate due to associated
blue pigment stones do not exist. All the other options are true. Gram-positive sepsis
Stone types include cholesterol, black pigment, and brown pig- B) Emphysematous cholecystitis occurs primarily in elderly
ment stones. Cholesterol stones are associated with advancing diabetics as a late complication o miliary tuberculosis ( B)
age (due to decreased synthesis o bile salts rom cholesterol), C) T e most common consequence o gallbladder per oration
hyperlipidemia, diabetes, obesity, and living in the Western is generalized peritonitis
Hemisphere (thank you, ast ood industry!). Black pigment D) Charcot’s triad (jaundice, ever, and right upper quadrant
stones contain calcium bilirubinate, calcium carbonate, and pain) is associated with cholangitis
calcium phosphonate. T ey are associated with hemolytic dis-
eases, Crohn disease, ileal resection, cirrhosis, and total parental Answer 22.10.5 The correct answer is “D.” T e classic Char-
nutrition ( PN). Brown stones are observed more o en in East cot’s triad (jaundice, ever, and right upper quadrant pain) is
Asia and are associated with liver uke in ection. associated with cholangitis, but it is not seen in the majority o
cases, probably due to early detection o cholangitis. Nonethe-
You re er the atient to your avorite surgical consultant, Dr. less, it can be help ul i present. “A” is incorrect because patients
Hugh Jeego (or his artner, I. M. Agodd), or evaluation o cho- generally develop a Gram-negative sepsis. “B” is incorrect
lecystecto y. However, be ore his surgery a oint ent, you see because emphysematous cholecystitis is not a complication o
hi again in the ED. He resents with ever, nausea, vo iting, miliary B, but it can occur with anaerobic cholecystitis. Mili-
and anorexia or the last 48 hours. His exa ination is signi - ary B rarely a ects the gallbladder. “C” is incorrect. T e most
cant or te erature 38.9°C, ild tachycardia and tachy nea, common consequence o gallbladder per oration is a localized,
and a nor al blood ressure. He is tender in the right u er walled o abscess. Additionally, a cholecystoenteric stula may
quadrant and has a ositive Mur hy sign. He has no jaundice or orm between the gallbladder and the duodenum or jejunum.
al able right u er quadrant ass. Laboratory values include Stones can then pass into the bowel. Stones larger than 2 cm
3
WBC 14,800/ , AL 58 IU/L, AS 64 IU/L, alkaline hos- in diameter are likely to lodge in the terminal ileum, a process
hatase 45 IU/L, total bilirubin 1.5 g/dL ( ildly elevated liver termed gallstone ileus.
enzy es and bilirubin), a ylase 95 IU/L, and li ase 52 IU/L Generally, patients with cholecystitis have a prior history o
(nor al a ylase and li ase). biliary colic. Pain with acute cholecystitis is similar to that o
cholelithiasis except is more o en severe, longer lasting (> 24
Question 22.10.4 At this point in time, your working diag- hours), and associated with anorexia, nausea, vomiting, ever,
nosis is: elevated white count (12,000–15,000/mm 3), right upper quad-
A) Acalculous cholecystitis rant guarding, and a positive Murphy sign (arresting inspira-
B) Acute calculous cholecystitis tion when palpating the gallbladder). However, Mur hy sign
C) Pancreatitis is only 65% sensitive (less in the elderly). Additionally, 38%
D) Ascending cholangitis with con r ed cholecystitis have neither leukocytosis nor
E) Myocardial in arction ever. About 15% o the time there is associated jaundice and
20% o the time there is a palpable mass in the right upper
Answer 22.10.4 The correct answer is “B.” T e clinical pre- quadrant. Biliary pancreatitis can occur when there is blockage
sentation, examination, and laboratory data point toward acute o the ampulla o Vater, and cholangitis occurs when there is
cholecystitis. “A” is incorrect because acalculous cholecystitis ductal stone obstruction and biliary in ection. Emphysematous
CHAPTER 22 • Ca RE o t h E Su Rg iCa l Pa t iENt 617

cholecystitis occurs primarily in elderly male diabetics: gas pro- Question 22.11.1 Which o the ollowing is most appropri-
ducing bacteria, most commonly C. per ringens, result in gas in ate at this time?
the gallbladder and can cause a severe sepsis. A) Per orm needle decompression on the le
B) Per orm needle decompression on the right
Given the hysical ndings, leukocytosis, and ild elevation C) Place a chest tube on the le
o transa inases, you conclude that this atient has chole- D) Place a chest tube on the right
cystitis. E) Per orm a chest radiograph and act on the basis o the results

Question 22.10.6 Which o the ollowing should NOT be Answer 22.11.1 The correct answer is “B.” T e combination o
considered in the management o this disease? hypotension, hypoxia, and absent breath sounds suggests a ten-
A) Initial treatment includes hospitalization with IV uid sion pneumothorax. I ediate deco ression o the a ected
resuscitation he ithorax should be er or ed by lacing a large-bore (14-
B) Select an antibiotic regimen to include a third-generation or 16-gauge) needle through the chest wall to relieve intra-
cephalosporin ± metronidazole, piperacillin/tazobactam thoracic ressure. Per Advanced rauma Li e Support guide-
(Zosyn), or uoroquinolone + metronidazole lines, the correct placement is in the second intercostal space at
C) Choose a narrow-spectrum antibiotic to treat enterococcus the mid-clavicular line. However, due to risk o mediastinal vas-
(e.g., vancomycin) cular injuries, some experts recommend placing the needle in
D) I early cholecystectomy is not chosen as treatment, it should the traditional location or chest tube—that is the h or sixth
be per ormed late (at least 6 weeks a er diagnosis) intercostal spaces at the mid-axillary line. Either site is accept-
able i the pleural space can be reached (it may be more di cult
Answer 22.10.6 The correct answer is “C.” T is answer is in the midaxillary line given increased subcutaneous tissue). O
incorrect or two reasons. First, just over 50% o episodes o particular note is “E.” I tension pneumothorax is suspected, you
cholecystitis are sterile; as a matter o course they are treated should never wait or con rmation o the diagnosis by a chest
with antibiotics because o our inability to determine which are radiograph. It is a true emergency that requires treatment on the
in ected. Second, while enterococcus is requently encountered, basis o clinical examination.
it is rarely a solitary pathogen and does not require speci c tar-
geted antimicrobial therapy. T e initial treatment o cholecysti- HELPFUL (AND UNFORTUNATE) TIP:
tis includes hospitalization, broad-spectrum IV antibiotics, and g ven e ep dem c bes y n e u n ed S es,
IV uid resuscitation. De nitive therapy or cholecystitis is cho- s mp e need e m y n be n en re c e
lecystectomy. Controversy surrounds whether cholecystectomy p e r (yes, re y, s been s d ed). h ve b ck p
should be per ormed acutely or be delayed. Generally, shorter p n, s c s n er, p c e er.
hospitalizations and better outcomes are ound with early cho-
lecystectomy.
His vital signs and oxygen saturation i rove with needle
Objectives: Did you learn to . . . deco ression. You lace a chest tube and give boluses o
• iden y r sk c rs r s nes? nor al saline through two eri heral IVs. T en, the atient is
• Rec n ze c mp c ns b dder d se se? laced in external xation device or his e ur racture. O er-
• M n e p en w symp m c s nes nd c e- ating roo xation is de erred secondary to unstable edi-
cys s? cal status. He is ulti ately intubated or continued increased
work o breathing. He is ad itted to the intensive care unit
(ICU) on a ventilator with a cervical collar ollowing negative
CASE 22.11 FAS exa ination ( ocused assess ent with sonogra hy or
A 28-year-old ale who was the restrained ront assen- trau a—a sonogra hic evaluation to rule out uid in erihe-
ger o a vehicle traveling in excess o 60 h is brought to atic, eris lenic, elvic, and ericardial s aces).
the ED via a bulance. T e driver o the vehicle was ound Re eat chest x-ray in the ICU shows the endotracheal (E )
dead at the scene. A bulance ersonnel re ort it took 5 to tube and chest tube in a ro riate ositions. Several rib rac-
10 inutes to extricate the atient. On arrival, he is u - tures are noted. T ere are “ u y in ltrates” in the le chest
bling incoherently. He is initially able to give his na e, but that were not resent on initial trau a chest series.
he is slurring his words. He denies any edical roble s,
edications, or allergies. Vitals signs include te erature Question 22.11.2 Which o the ollowing is true o pulmo-
35.5°C, ulse 148 b , res irations 35 b , blood ressure nary contusions?
65/30 Hg, and oxygen saturation 81% on 100% oxygen A) T ey occur in less than 25% o patients with signi cant blunt
by ace ask. On exa ination, he is in severe res iratory trauma to the chest
distress. Lung sounds are absent on the right and di in- B) reatment includes aggressive IV steroid and uid adminis-
ished on the le . Heart sounds are uf ed. You deter ine tration
that this atient needs i ediate treat ent o a neu o- C) T e condition starts to resolve in 48 to 72 hours
thorax. D) reatment includes appropriate antibiotics
618 a Mil Y MEDiCiNE EXa MiNa t io N & Bo a RD REViEW

Answer 22.11.2 The correct answer is “C.” Pulmonary contu-


sions begin to resolve in 48 to 72 hours. However, 2 to 3 weeks Part o the injuries sustained by this atient includes burns to
may be required or complete resolution. “A” is incorrect. Pulmo- the abdo en and back.
nary contusions occur in up to 70% o trauma patients with sig-
ni cant blunt chest trauma. It is usually, but not always, associ- Question 22.11.4 Which o the ollowing is FALSE regarding
ated with ractured ribs. T ere may also be a ail segment noted. burn wound management in general?
Chest x-ray changes are usually evident within 1 hour post- A) T e Parkland ormula or uid resuscitation calls or 2 to
trauma but may not appear or up to 6 to 7 hours. Pulmonary 4 mL/kg/% body sur ace area burned with hal o the volume
contusions result in a ventilation–per usion mismatch, hypox- in the rst 8 hours and the other hal over the next 16 hours
emia, and an increased A-a gradient. I a patient is able to main- B) Escharotomy should be per ormed on all partial-thickness
tain oxygenation and ventilation, intubation is not required. “B” burns
and “D” are incorrect. reatment currently only involves intuba- C) Patients with chemical burns should be treated rst with tap
tion as necessary, observation, and tincture o time. water irrigation
D) Prevention o wound in ection via topical antimicrobial
You elect to roceed with ul onary artery catheter lace- agents, such as silver sul adiazine cream, or via silver-coated
ent due to the severity o this atient’s condition. Over- dressings is the standard o care
night, he begins to deco ensate. T e nurse ages you with
his vital signs and Swan–Ganz readings: te erature 37.0°C, Answer 22.11.4 The correct answer is “B.” Any time you see
ulse 100 b , res irations 20 b (ventilator set at 14 b ), “all,” it is probably the wrong answer. Escharotomy is not neces-
blood ressure 82/30 Hg, PCWP 24 Hg (nor al sary unless there is a ull-thickness wound that is circum eren-
5–15), cardiac index 2.0 L/ in/ 2 (nor al 2.5–3.5), syste ic tial and compromising vascular supply. T e thinking about this
vascular resistance 2,000 dyne-sec/c 2 (nor al 1,000–1,500), is changing and some suggest escharotomy o all ull-thickness
and oxygen delivery o 700 L/ in (nor al 900–1,200). burns. “A” is a correct statement about the Parkland ormula.
“C” is also correct with the addendu that any articulate
Question 22.11.3 What is the cause o shock at this time? atter should be brushed o rior to irrigation. Water ay
A) Hypovolemic shock activate so e substances such as sodiu hydroxide. “D” is
B) Neurogenic shock also a correct statement although thinking is changing. Silver
C) Cardiogenic shock delays wound healing and bacitracin is being used with increas-
D) Septic shock ing requency.

Answer 22.11.3 The correct answer is “C.” T is patient appears


to be in cardiogenic shock (elevated pulmonary capillary wedge HELPFUL TIP:
pressure, decreased cardiac index). Cardiogenic shock may be ds re req red r d s w > 15% b dy
caused by myocardial ailure, valve ailure, dysrhythmias, and s r ce re (t BSa ) b rns (sec nd r rd de ree) nd
tamponade. reatment is directed at the underlying disorder. > 10% r c dren ed 10 ye rs nd y n er. r
See able 22-5 or more on the categories o shock. adults e “R e 9’s” s sed de erm ne t BSa ec -
ed. B dy s r ce re s es m ed s ws: e e d,
e c rm, r n e c e , nd b ck e c e c n
HELPFUL TIP . . . OR NOTE OF CAUTION: r 9% t BSa e c ; e r n nd b ck e r nk e c
t ere s n d ev dence s pp r e se Sw n– c n r 18%; e rem n n 1% t BSa s cc n ed
g nz c e ers n ser s y p en s. in c , e ev - r by e en . t BSa ec ed n c dren nd d -
dence s es s worse c mes w Sw n–g nz m n - escen s s d be b sed n e-spec c c r s. B
rn nw ( r m seps s, r mb s s, e c.). Be very ec ed nd n ec ed BSa s d be c c ed
c rc mspec be re e ec n se Sw n–g nz c e ers. ss re cc r e es m n.

TABLE 22-5 CATEGORIZATION OF SHOCK BY PHYSIOLOGIC PARAMETERS


Systemic Vascular Pulmonary Capillary Oxygen Consumption Cardiac
Type o Shock Resistance Wedge Pressure (Based on Venous Gas) Output
h yp v em c incre sed Decre sed Decre sed Decre sed

C rd en c incre sed incre sed Decre sed Decre sed

Ds rb ve (seps s, ne r en c) Decre sed Decre sed r n rm incre sed incre sed

o bs r c ve (PE, ven c v bs r c n, incre sed l w e , r Decre sed Decre sed


ens n pne m r x)
CHAPTER 22 • Ca RE o t h E Su Rg iCa l Pa t iENt 619

D) T e need to check carboxyhemoglobin levels


HELPFUL TIP:
E) B and D
t e P rk nd rm w s des ned ss re deq e
ydr n n b rn p en s s re ec ed by r ne p
The correct answer is “E.” Carbonaceous material in the nares
( n e p en w nc n n k dneys, c rse).
or oropharynx should suggest the possibility o an inhalation
a dj s ds ppr pr e y m n n deq e ydr -
injury. T ought should be given to intubate these patients early,
nw nd c n e r re.
especially i there will be a prolonged trans er time. Further,
given that this is related to the inhalation o combusted mate-
rial, a carboxyhemoglobin level should be considered.
Question 22.11.5 Which o the ollowing is TRUE regarding
uid administration in burn and dehydrated patients? CASE 22.12
A) A peripheral line will deliver uid more rapidly than a cen-
A 48-year-old e ale arrives at the ED via a bulance a er
tral line o an equivalent gauge
witnesses saw her vo iting large volu es o blood in a local
B) Albumin is the uid o choice in the treatment o burns and
convenience store be ore colla sing to the oor—obviously,
should be considered or all patients with signi cant uid
not her best day. She is barely co unicating and cannot ro-
de cits
vide any history. A bulance ersonnel have laced two large-
C) D5W-½ NS (D5W-0.45%NS) is the pre erred uid or resus-
bore IVs and started nor al saline boluses. Just as you start
citation in all nonburn patients
your initial evaluation, she has a large volu e he ate esis.
D) All o the above are true
You are concerned about her de ressed ental status and the
severity o her illness and decide to intubate her. A er intuba-
Answer 22.11.5 The correct answer is “A.” A peripheral line
tion, the chest wall rises sy etrically and the lungs sound
will deliver uid more rapidly than a central line o an equiva-
clear. Her heart sounds are distant, and she is tachycardic
lent gauge, according to Poiseuille’s law ( ow is directly propor-
( ulse 120 b ). Her blood ressure is 80/40 Hg. Her
tional to tube radius and inversely proportional to tube length).
oxygen saturation is 78%.
T e shorter the catheter, the more quickly uid is delivered
(think o a short tra c jam as opposed to a longer one on the
Question 22.12.1 The standard o care in detecting esopha-
same sized road). T e largest port on a traditional triple lumen
geal intubation is:
catheter will give the same ow rates as a 20 gauge peripheral IV.
A) Auscultation
Consider a “cordis” catheter i placing a central line in a patient
B) Radiograph
who may also require rapid uid resuscitation. T is is a larger
C) End-tidal CO2
diameter, shorter central line. “B” is incorrect. Albumin is not
D) Oxygen saturation
help ul and may increase adverse outcomes in trauma. “C” is
E) Direct laryngoscopy
also incorrect. Normal saline (0.9% NS), or Lactated Ringer’s
i you are a surgeon, are the uids o choice or resuscitation.
Answer 22.12.1 The correct answer is “C.” T e standard o
Remember that Lactated Ringer’s is actually slightly hypotonic
care is the end-tidal CO2. All o the others are notoriously unre-
and thus may worsen cerebral edema. T ere is no evidence
liable. However, they all should be done. I you think you hear
avoring Lactated Ringer’s over normal saline.
breath sounds but the oxygen saturation is not rising and end-
Objectives: Did you learn to . . . tidal CO2 is low, the E tube is probably in the esophagus.
• a ppr c r m p en w n nders nd n
dv nced r m e s pp r pr nc p es? Question 22.12.2 The end-tidal CO2 can be alsely negative
• iden y nd m n e p en w ens n pne m - (detecting no CO2) in which o the ollowing situations?
r x? A) Ingestion o carbonated so drinks (or “pop” as we call it in
• Descr be v r s ypes s ck nd w ey re d eren - the Midwest)
ed? B) Intubation in the posterior pharynx above the cords
• t re p en n s ck? C) Nasotracheal intubation
• M n e p en w s n f c n b rns? D) During cardiac arrest

Answer 22.12.2 The correct answer is “D.” T e end-tidal CO2


QUICK QUIZ: iNh a l at io N iNJu RY requires that there is gas exchange in the lungs. I there is no
gas exchange, the CO2 will be low. T is may be the case during
Carbonized particles in the nasal cavity and/or posterior phar- cardiac arrest. “A” can actually cause a alse ositive. Carbon
ynx in a burn patient should suggest: dioxide in the stomach will give a positive end-tidal CO2 with
A) T e need or excision o the perichondrium in the nose to an esophageal intubation. T e same is true o “B.” I the patient
prevent underlying cartilage injury rom avascular necrosis is breathing spontaneously, the CO2 will be elevated even when
B) Inhalational injury to the lungs the E tube is above the cords. “C,” nasotracheal intubation,
C) Ingestion o a large amount o particulate matter should have no e ect on end-tidal CO2.
620 a Mil Y MEDiCiNE EXa MiNa t io N & Bo a RD REViEW

48, albu in 2.4 g/dL, AS 76 IU/L, AL 39 IU/L, Bili 3.5 g/


You order laboratory studies that include a CBC, basic eta- dL, a ylase 210 IU/L, and li ase 24 IU/dL. T e gastroenter-
bolic ro le, liver che istries, a ylase, li ase, and coagula- ologist is still 5 to 10 inutes away.
tion studies. In addition, you ty e and cross or six units o
acked red blood cells. T e nurses have already contacted the Question 22.12.4 In considering what to do next, which o
gastroenterologist on call, and she is on her way. You decide the ollowing would be most appropriate?
to er or nasogastric (NG) lavage. A) Address the presence o platelet dys unction by trans using
with a 10-pack o platelets
Question 22.12.3 Which o the ollowing is FALSE regarding B) In order to accurately assess degree o volume depletion,
NG lavage? place a central venous pressure (CVP) catheter or Swan–
A) NG lavage may be negative even in the presence o an upper Ganz catheter, and bladder catheter
GI bleed C) Regardless o the gastroenterologist, central line placement
B) NG lavage should not be attempted in obtunded patients should be priority at this time because uid resuscitation is
until they are intubated the primary concern
C) Iced uid should not be used to lavage patients with an D) Emergent gastric tamponade should be attempted with a
upper GI bleed Foley catheter
D) T e placement o a NG tube is contraindicated in patients E) None o the above is a great idea right about now
who may have variceal bleeding
Answer 22.12.4 The correct answer is “E.” None o the
Answer 22.12.3 The correct answer is “D.” Varices are not a above is a particularly good idea at this juncture. Looking at
contraindication to the use o a NG tube. Studies suggest that them one by one, “A” is not necessary since a platelet count o
a NG tube does not increase bleeding. Both “A” and “B” are 62,000 is adequate or hemostasis (although you would not
true statements. Lavage should not be done in patients who be aulted or giving platelets). A platelet count o < 50,000
are obtunded or otherwise unable to protect their own airway is considered an indication or platelet trans usion in an
unless they are intubated. In act, i the atient has obvious actively bleeding patient. “B” and “C” are not good ideas
blood in the vo itus, NG lavage is not necessary unless it because coagulopathy (INR 3.0) is a relative contraindica-
is to clear sto ach contents in order to er or endosco y tion to central line placement. It can still be done, but the
or to revent vo iting. It adds nothing to the anage ent line is not needed at this point. Recall that peripheral cath-
o the atient with an u er GI bleed exce t con r ing the eters will deliver more luid more rapidly when compared
diagnosis. Remember that alse-negative NG aspirates occur with central catheters. hus, two large-bore, peripheral IVs
with intermittent bleeding and bleeding beyond the ligament are the access o choice. “D” is incorrect because gastric tam-
o reitz. “C” is also a true statement. Iced lavage uid should ponade with a Foley is like trying to stop a leak in the Hoover
not be used in patients with a GI bleed. T e cooler temperature dam with putty: it won’t work. he most e ective treatment
that results rom the ice inhibits hemostasis and can increase at this time may be to give vitamin K and resh rozen plasma
bleeding. to reverse her coagulopathy.

Question 22.12.5 Ef ective methods or controlling upper


HELPFUL TIP:
GI variceal bleeding that improve outcomes include all o
C n r nd c ns s r c v e nc de kn wn n-
the ollowing EXCEPT:
es n ydr c rb ns nd c s c en s, s c s
A) Variceal ligation
k s nd c ds.
B) Sclerotherapy
C) ransjugular intrahepatic portosystemic shunt ( IPS) pro-
cedure
While your nurse is er or ing the war water lavage D) Vasopressin
(although why lavage is being done is a se arate question
since as noted above it does not add anything to this atient’s Answer 22.12.5 The correct answer is “D.” Vasopressin, while
care), you er or a secondary hysical exa ination on the achieving initial control o bleeding in up to 60% o patients, has
atient. Her ost obvious nding is a arkedly jaundiced essentially no e ect on rebleeding and no e ect on ortality.
state. Other ertinent ndings include a 6-c scal lacera- T is may be because ischemia o the splanchnic bed and other
tion, which continues to actively bleed, oderate ascites, and areas caused by vasopressin outweighs any bene t. In more
lower extre ity ede a. A er lavage with nearly 5 L o iso- bad news, octreotide also does not have any e ect on mortal-
tonic uid, the atient’s as irate continues to be bloody. At ity unless combined with variceal ligation. Variceal ligation and
this oint, her vitals are re aining steady but have shown no sclerotherapy both reduce mortality. Additionally, a IPS pro-
sign o i rove ent. Initial e ergent labs have returned: Na cedure has been shown to e ectively stop bleeding by reducing
142 Eq/L, K 3.2 Eq/L, Cl 106 Eq/L, CO2 18 Eq/L, BUN portal pressures. It also reduces acute mortality. Data is con-
40 g/dL, Cr 0.8 g/dL, glucose 110 g/dL, WBC 7,000/ icting on long-term outcomes, but encephalopathy is more
3 3
, Hgb 9.8 g/dL, Hct 29%, Plt 62,000/ , INR 3.0, P common a er IPS.
CHAPTER 22 • Ca RE o t h E Su Rg iCa l Pa t iENt 621

HELPFUL TIP: TABLE 22-6 CHILD–PUGH CLASSIFICATION


a n e ev ed Bu N c n be nd rec ev dence g i b eed P n s ss ned 1 2 3
n p en s w ver d se se. t e d es n b d
Encep p y N ne l w r de h r de
e ds e e ev ed Bu N.
a sc es N ne S M der e- r e

B r bn 1–2 m /dl 2–3 m /dl > 3 m /dl


Des ite Dr. Graber’s warning to the contrary, you consider
lacing a central line. a b mn > 3.5 /dl 2.8–3.5 /dl < 2.8 /dl

iNR < 1.7 1.8–2.3 > 2.3


Question 22.12.6 Which o the ollowing locations o cen-
tral line placement is associated with the highest rate o
in ection?
A) Internal jugular score can be used to predict the risk o variceal bleeding as well
B) Femoral vein as the surgical risk and the overall mortality in patients with
C) Subclavian vein liver disease. “D” is true since there is a maximum to the Child–
D) T ey all have similar in ection rates Pugh scoring system or each parameter, and the same score is
given or all bilirubin levels above 3 mg/dL.
Answer 22.12.6 The correct answer is “D.” T e classic teaching Further discussion: T e Child–Pugh classi cation as a
is that emoral lines are more likely to get in ected. However, the global battery o tests can help to more accurately assess degree
data doesn’t back this up; in ection rates seem to be the same or o cirrhosis, need or transplant (minimum score o 7), mor-
all above locations (Crit Care Med. 2012;40:2479). Subclavian tality rate rom variceal bleed, and outcomes a er surgery and
veins have a higher risk o complication such as arterial injury, IPS, and outcomes with a hepatoma ( able 22-6).
hemothorax, pneumothorax, and lung injury. T oracic duct Class A is de ned as having 5 to 6 points, Class B is 7 to 9
injury is most common with le internal jugular cannulation. points, and Class C is 10 to 15 points. T e 1- and 2-year survival
Note that ultrasound guidance o central line insertion is the in patients with Class C disease is 45% alive at 1 year and 35%
standard o care. alive at 2 years.
Class A patients have an operative mortality o 1%, while
T e gastroenterologist arrives and you ex lain the situation. Class B and C have operative mortalities o 3% to 10% and 30%
Vitals at this ti e include a te erature o 36.8°C, ulse to 50%, respectively. In patients with hepatomas, no Class B or
105 b , res irations 14 (ventilator de endent) b , blood C patients survived 3 to 5 years ollowing resection, although
ressure 85/40 Hg, and oxygen saturation o 92%. T e about 40% o Class A patients survive or 5 years.
gastroenterologist lans to atte t endosco y with sclero-
thera y, but would like to have the general surgeons available You quickly recall that there is another rognostic scoring
or backu in case e ergent o erative intervention beco es syste available or risk strati cation and ortality, which
necessary. As you re are to contact the surgeon, you recall is used by United Network or Organ Sharing (UNOS) and
risk strati cation or cirrhotic atients is via the Child–Pugh called the Model or End-stage Liver Disease (MELD) scor-
classi cation syste . ing syste .

Question 22.12.7 Which o the ollowing is FALSE regarding Question 22.12.8 Which o the ollowing is TRUE regarding
the Child–Pugh scoring system? the MELD scoring system?
A) T e Child–Pugh scoring system can be used to predict the A) T e MELD scoring system can be used to predict 3-month
risk o variceal bleeding survival ollowing IPS
B) T e ve criteria used in the Child–Pugh classi cation are B) T e three criteria used in the MELD classi cation are biliru-
ascites, encephalopathy, albumin, bilirubin, and INR bin, INR, and creatinine
C) T e Child–Pugh scoring system can be used to determine i C) T e primary use o the MELD classi cation is or assessing
the patient with a deep hepatic encephalopathy will “wake mortality risk and prioritizing patients on the waitlist or
up” liver transplant
D) A patient with a serum bilirubin o 3.5 mg/dL may have the D) T e minimum score is 6
exact same Child–Pugh score as a patient with a serum bili- E) All o the above
rubin o 25 mg/dL
Answer 22.13.8 The correct answer is “E.” T e MELD score
Answer 22.12.7 The correct answer is “C.” T e Child–Pugh was originally used to predict the 3-month mortality a er IPS
scoring system does not predict the course o hepatic encepha- (“A”). However, now the MELD score has been validated as a
lopathy. T e other statements are true. T e Child–Pugh clas- tool to estimate overall mortality related to liver disease and to
si cation includes evaluation o ascites, encephalopathy status, prioritize patients on the transplant wait list (“C”). T e MELD
albumin, bilirubin, and INR. “A” is true because the Child–Pugh score is computed by a logarithmic ormula using creatinine,
622 a Mil Y MEDiCiNE EXa MiNa t io N & Bo a RD REViEW

INR, and bilirubin levels. T e ormula is quite complex, so Question 22.13.1 Regarding nail removal in this patient,
thank ully there are handy online calculators readily available which one o the ollowing is FALSE?
(http://www.mayoclinic.org/medical-pro essionals/model-end- A) T e great toe is the most commonly a ected toe or ony-
stage-liver-disease/meld-model-unos-modi cation). For any chocryptosis
measured bilirubin, INR, and creatinine less than 1, the or- B) I the patient chooses partial nail removal, about 25% o the
mula automatically sets the values to 1 so the minimum score nail should be removed on the a ected side
is 6 (“D”). T e MELD score is interpreted as ollows: score C) Phenol should be placed or no longer than 10 seconds to
≥ 40, 71.3% mortality in 3 months; 30–39, 52.6% mortality in the germinal tissue to prevent necrosis
3 months; 20–29, 19.6% mortality in 3 months; 10–19, 6.0% D) When removing the nail, an upward twist o the hand to the
mortality in 3 months; < 9, 1.9% mortality in 3 months. medial side should be per ormed
Objectives: Did you learn to . . .
Answer 22.13.1 The correct answer is “C.” Phenol can be le in
• M n e p en w m ss ve pper g i b eed?
place or 3 minutes. Ingrown toenails almost exclusively a ect
• De erm ne n Et be s been p ced c rrec y?
the great toe on either the medial or lateral side. Partial or ull
• Rec n ze e ses nd m ns s r c v e? nail removal should be implemented when conservative mea-
• Rec n ze e ses nd m ns cen r ne p cemen sures have ailed. Besides ingrown nails (onychocryptosis), ony-
nd Sw n–g nz c e er p cemen ? chomycosis ( ungal in ection o the nail), recurrent paronychia
• Ev e p en w ver d se se, s n e C d–P (nail old in ammation), and onychogryphosis (de ormed,
nd MEl D c ss f c n sys ems? curved nail) are all indications or partial or ull nail removal. I
recurrent ingrown nails have occurred, germinal tissue can be
QUICK QUIZ: iNt u Bat io N ablated with phenol on a cotton swab held in place or 3 min-
utes, and a erward the phenol should be neutralized with alco-
hol. Other approaches to matrixectomy include electrosurgical
Which o the ollowing IS NO a contraindication to a blind
and laser techniques and application o sodium hydroxide.
nasotracheal intubation?
A) Patient is not breathing
Next, he co lains about a s all regular ole on his neck
B) Patient is anticoagulated or has had PA
that he re eatedly cuts while shaving. He would like to have it
C) Mid ace trauma
re oved in whatever way you dee best. T e lesion is raised
D) History o a septoplasty
above the skin. Although it is ildly irregular in a earance
ro the re eated trau a o shaving, there is no evidence o
The correct answer is “D.” All o the rest are contraindications
aty ia.
to a blind nasotracheal intubation. Speci cally, it is not possible
to do a blind nasotracheal intubation in a nonbreathing patient. Question 22.13.2 Which o the ollowing is NOT a correct
Patients who are anticoagulated may bleed pro usely a er a statement regarding removal o this lesion?
nasotracheal intubation, and mid ace trauma suggests the pos- A) Curettage or shave biopsy would be ideal or this sort o
sibility that the tube could end up in the brain. Although the lesion
standard o care, two studies have ailed to show a di erence B) I sutures are required, 6-O nylon would be ideal in this
between CNS complications with orotracheal and nasotracheal location
intubation. Nonetheless, avoid nasotracheal intubation in mid- C) I a punch biopsy is per ormed, skin should be held taught
ace trauma, except as a last resort, or you will get dinged i there perpendicular to the angle o the mandible (the natural skin
is a complication. lines o the neck)
D) In this location, both shave and punch techniques require
closure by suture approximation
CASE 22.13
You are seeing a 33-year-old resident at a large university Answer 22.13.2 The correct answer is “D.” Various types o
hos ital setting where edical care is rovided reely to hi . skin lesion removal exist: punch biopsy, shave biopsy, curettage,
He is about to co lete his Internal Medicine residency and and elliptical excisional biopsy. Punch biopsy involves taking a
wants to “get his oney’s worth” o ree rocedures be ore ull-thickness sample in skin areas (except or the eyelids, lips,
he leaves. He has scheduled a ull a ernoon o rocedures or penis). Skin should be held taught perpendicular to the natu-
with you, including toenail re oval, excision o a ole on his ral skin tension lines, and punch instrument is rotated through
neck, and a stylish new cartilage ear iercing or his new job the skin. T e site is closed with either a single interrupted or
as a cruise shi doctor. vertical mattress suture. Shave biopsy is indicated or removal
He has had trouble with recurrent ain and in a a- o elevated skin lesions where complete thickness removal is
tion on his le great toe at the edial side. On exa ination unnecessary. Shave should be made rom both lateral edges
you identi y onychocry tosis (ingrown nail). He has tried into center to avoid cutting too deep. No suturing is necessary.
soaks, growing ast the skin, and regular aring—all with- Curettage entails a method o removing lesions that also does
out success. not require ull-thickness sampling. A Fox dermal curettage
CHAPTER 22 • Ca RE o t h E Su Rg iCa l Pa t iENt 623

is used to scrape away unwanted tissue ollowed by electrical once the pleura are drained o air, there should be no urther
or chemical cautery or hemostasis. Finally, elliptical excision bubbling. Air leakage is seen as air bubbles that increase with
is used when ull dermal thickness excision is necessary. Con- increased intrathoracic pressure (e.g., cough, Valsalva maneu-
traindications to skin biopsy include in ection at the site and ver, and positive pressure ventilation). Continuous air leakage
coagulopathy. may be due to a large tear in the lung parenchyma, broncho-
pleural stula, or an apparatus leak. “A” is incorrect. A tube that
Finally, your atient is a little e barrassed to be seen at the is non unctional should be removed. Increasing the suction will
all getting his ear ierced, and asks i you could ierce it not reopen it. “B” is incorrect as well. Initial suction should be
or hi . He is considering a standard lobe ierce versus an 20 cm H 2O. “D” is incorrect because an occlusive dressing, such
auricular cartilage iercing. as petrolatum gauze, should be placed over the ormer chest
tube site once the tube is removed.
Question 22.13.3 Which o the ollowing is true in regard to A chest tube can be removed when the ollowing criteria
counseling and technique? are met: uid drainage is less than 150 cc/day, the lung is ully
A) Eczema at the site is a contraindication to piercing expanded on chest x-ray, and no air leak is present. ypically,
B) Auricular cartilage piercing should only be per ormed by a chest tubes are rst placed to “water seal” or 6 to 24 hours to see
trained physician i the patient will tolerate having the tube removed.
C) Ears should be pierced rom the posterior to anterior site
D) Ear piercing involves boring a 20-gauge needle into the CASE 22.14
marked site
E) Ear piercing is contraindicated in nerds John is a 63-year-old ale who resents to your o ce or a
hysical exa ination. As art o the exa ination, you do a
rectal exa ination and stool guaiac. T e guaiac is he occult
Answer 22.13.3 The correct answer is “A.” Eczema in the ositive. On colonosco y, he is noted to have bio sy- roven
area is a contraindication to piercing. Other contraindica- colon cancer.
tions include in ection, previous keloid ormation, immuno-
de ciency, and coagulopathy. Auricular cartilage piercing is Question 22.14.1 Regarding colon cancer, what is the most
prone to in ection and destruction o the cartilage. Pseudo- likely type o cancer you will nd?
monas is a common pathogen and oral uoroquinolones are A) Squamous cell carcinoma
indicated as treatment. All o the other options are incorrect. B) Adenocarcinoma
“E” deserves special mention, as “nerds” can be converted to C) Clear cell carcinoma
“hipsters” simply by piercing their ears. However, there is a D) Lymphoma
corollary to this axiom: any aging male with a gray ponytail
should avoid piercings. Answer 22.14.1 The correct answer is “B.” Adenocarcinoma
represents the overwhelming majority o colon cancers. Other
Objectives: Did you learn to . . . histologic types o colon cancer include adenosquamous, poorly
• Descr be ec n q es nd nd c ns r en rem v ? di erentiated cancers with neuroendocrine aspects, small cell
• Descr be v r s sk n b psy ec n q es? carcinomas (o neuroendocrine origin), and others.
• iden y c n r nd c ns e r p erc n ?
John has the erennial question that theologians are usually
better quali ed to answer: “Why e?” In this case, however,
QUICK QUIZ: Do C, iS t h at a h o SE iN MYCh ESt ? we can hel to rovide so e insight.

What statement represents correct chest tube management? Question 22.14.2 Which o the ollowing is NOT considered
A) A tube that shows no water uctuations when placed on a risk actor or colon cancer?
“water seal” should have wall suction increased in attempt A) Familial polyposis
to re-open B) Alcohol use
B) Initial postoperative setting o a chest tube is most o en C) Obesity
–100 cm H 2O D) In ammatory bowel disease
C) Unless there is major injury to the lung, continuous bub- E) Chronic constipation
bling o the water seal chamber most likely represents an
apparatus leakage Answer 22.14.2 The correct answer is “E.” Chronic consti-
D) I a purse string was placed during initial placement o the pation is not a risk actor or colon cancer. Aside rom those
chest tube, tightly tying the purse string when pulling the listed above, risk actors include acromegaly, diabetes melli-
chest tube is enough to prevent pneumothorax tus, and A rican American race, among others. In act, A rican
Americans have the highest risk or colon cancer o any ethnic
The correct answer is “C.” Continued bubbling in the water seal group with a 20% increase in mortality when compared with
chamber suggests that there is a leak in the system. T eoretically, whites.
624 a Mil Y MEDiCiNE EXa MiNa t io N & Bo a RD REViEW

John is justi ably worried about the ossibility o colon cancer HELPFUL TIP:
in his o s ring. Un ortunately, none o his o s ring exercise W e 5-h ia a s nds ke n cr nym r s me new
(which reduces colon cancer risk) and they are avowed carni- vernmen n ve, s m rker c rc n d m r
vores eschewing ruit, vegetables, and anything with ber ( ber c v y.
ay reduce colon cancer risk, although the data is con icting).

Question 22.14.3 Which o the ollowing has been shown to John undergoes surgery and ends u with a total colecto y
reduce cancer risk and takes so little ef ort that even John’s and ileosto y.
sloth ul of spring may partake?
A) Met ormin Question 22.14.6 Which o the ollowing is NOT true o ile -
B) Aspirin ostomy care?
C) Antioxidants A) Fluid output tends to be relatively high with an ileostomy
D) Vitamin B6 but no other antioxidants requiring increased uid intake to prevent dehydration
E) B and C B) Since the diameter o the ileostomy is limited, avoiding large
amounts o nondigestible ber helps to prevent bezoar or-
Answer 22.14.3 The correct answer is “B.” Aspirin and other mation
NSAIDs have been shown to reduce the risk o colon cancer. C) ime-release drugs are usually well absorbed because most o
Un ortunately, the rest o the answers are incorrect and none the digestion and absorption occur be ore the colon anyway
have been ound to reduce the risk o colon cancer D) Proteolytic enzymes are present in the ef uent and may lead
to skin breakdown
It is ti e to stage John’s cancer. T e Duke’s classi cation sys-
te is no longer used or colon cancer and has been re laced Answer 22.14.6 The correct answer is “C.” ime-release medi-
with the ore standardized MN syste . cations should be avoided, since absorption o en takes place in
the colon. All o the rest are correct.
Question 22.14.4 Which o the ollowing is the most com-
mon rst site o metastasis o colon cancer?
HELPFUL TIP:
A) Lungs
l per m de c n be sed red ce p n s r b w-
B) Liver
e syndr me ( nc d n n se w n s my) s n
C) Bone
s b c er ver r w s n n ss e.
D) Brain

Answer 22.14.4 The correct answer is “B.” T e liver is the


most common rst site o colon cancer metastases, which arrive John does well and has a ha y and long li e, although his chil-
via the portal system. Colon cancer can also metastasize to the dren re ain slugs and he su orts the well into their 40s.
lungs, bone, lymph nodes, and brain (uncommon). Intraperito- Objectives: Did you learn to . . .
neal spread may also occur. • Descr be s me m r m rkers r v r s c ncers?
• Def ne v r s s c ypes c n c ncers nd e
Question 22.14.5 While tumor markers or colon cancer are p ern me s s z n ?
not used in staging disease, the presence o which o these • Rec n ze r sk c rs r c n c ncer nd w ys red ce
markers suggests a poor prognosis i it remains elevated e r sk c n c ncer?
a ter surgical resection o the tumor? • M n e s m es?
A) Carcinoembryonic antigen (CEA)
B) β -hCG
C) CA-195 Clinical Pearls
D) Vasoactive intestinal peptide (VIP) a n e ev ed c e c n nd c e ss e sc em r m yp -
E) α - etoprotein (AFP) per s n (s ck, v sc r c mpr m se— e s -c ed “ ype a ”
c c c d s s). Be w re er c ses c c cd ss
Answer 22.14.5 The correct answer is “A.” T e presence o nc de n es ns s c s me n r e n , ver d se se,
CEA a er resection o a colon cancer is a poor prognostic ac- d be es, me rm n, d be es, e c. (“ ype B” c c c d s s).
tor. T is should be obvious: there is still tumor present. β -hCG a n Ng be s n necess ry n e re men sm
is elevated in testicular cancer. CA-195 is a marker or ovarian b we bs r c n. h wever, c n be e p n preven n
cancer (although it is also expressed by colon cancer cells). VIP v m n . t e r de- s n Ng be s nc m r b e.
is either the patient who everyone swarms over and nobody C mpress n s ck n s d n n preven p s per ve
can please or vasoactive intestinal peptide, which is ound with DVt s nd m y c se sk n bre kd wn: d n’ se em. u se
some pancreatic adenocarcinomas and carcinoid tumors. AFP s bc ne s ep r ns r n erm en pne m c c mpres-
is elevated in germ-cell tumors, such as testicular and ovarian s n n w e rym b z n.
cancer, and liver cancer.
CHAPTER 22 • Ca RE o t h E Su Rg iCa l Pa t iENt 625

Maglinte DD, et al. Reliability and role o plain lm radiogra-


D n e “b se ne” bs s c s CBC, Pt /Pt t /iNR, r e ec-
phy and C in the diagnosis o small-bowel obstruction.
r y es pre per ve y w n nd c n n p en s w
Am J Roentgenol. 1996;167:1451–1455.
d n ve s n c n sys em c d se se. t ey re n e p-
. t r e y r pre per ve w rk p b sed n e p en ’s Marino PL. Marino’s T e ICU Book. 4th ed. Philadelphia, PA:
c m rb d es. Wolters Kluwer Health/Lippincott Williams & Wilkins; 2014.
Mechanick JI, et al. Clinical practice guidelines or the periop-
D n se c ds r d res sc n n ess ere s
erative nutritional, metabolic, and nonsurgical support o
spec c nd c n (e. ., b m n er r e v me p r cen-
the bariatric surgery patient—2013 update: Cosponsored
es s). t e c mes re ener y w rse n w crys d
by American association o clinical endocrinologists, the
nd e c ds re m re expens ve.
obesity society, and American society or metabolic &
o n y 65% p en s w c ecys s w ve p s - bariatric surgery. Obesity. 2013;21(S1):S1–S27.
ve M rp y’s s n nd 40% w ve ne er ever n r n McCahill LE, et al. Variability in re-excision ollowing breast
e ev ed w e c n . conservation surgery. JAMA. 2012;307:467.
P en s w ve d b r r c s r ery c n be v m n- Mittal MK, et al. Per ormance o ultrasound in the diagnosis o
de c en nc d n v m ns D, m ne, B12 nd e. a pediatric appendicitis in a multicenter cohort. Acad Emerg
m v mns d be s es ed n ese p en s. Med. 2013;20(7):697–702.
Pr p y c c n b cs s d be ven be ore s r ery nd Morris RJ, Woodcock JP. Intermittent pneumatic compression
not c n n ed er s r ery. or graduated compression stockings or deep vein throm-
t en percen p en s w ppend c s w ve n rm bosis prophylaxis? A systematic review o direct clinical
w e c n . o n y 75% w ve n rex . comparisons. Ann Surg. 2010;251(3):393–396.
Mulholland MW. Greenf eld’s Surgery: Scientif c Principles and
W en c e r z n ver d se se, e MEl D sc re s ess s b- Practice. 4th ed. Philadelphia, PA: Lippincott Williams &
jec ve n e C d-P sc re nd req res n y cre - Wilkins; 2006.
n ne, iNR, nd b r b n.
P enninger JL, Fowler GC. P enninger and Fowler’s Proce-
dures or Primary Care. 3rd ed. Philadelphia, PA: Elsevier
Mosby; 2011.
BIBLIOGRAPHY Salminen P, et al. Antibiotic therapy vs. appendectomy or treat-
American College o Surgeons Committee on rauma. ment o uncomplicated acute appendicitis: T e APPAC
Advanced rauma Li e Support or Doctors: Student Course randomized clinical trial. JAMA. 2015;313(23):2340–2348.
Manual. 8th ed. Chicago, IL: American College o Sauret JM, et al. Rhabdmyolysis. Am Fam Physician. 2002;65(5):
Surgeons; 2008. 907–913.
Burgess BE. Anorectal disorders. In: intinalli J, Stapczynski JS, Scott-Conner CEH, Dawson DL. Operative Anatomy. 3rd ed.
Cline DM, et al., eds. intinalli’s Emergency Medicine. 7th ed. Philadelphia, PA: Lippincott Williams & Wilkins; 2009.
New York, NY: McGraw-Hill Medical; 2011. pp. 587–601. Stevens DL, et al. Practice guidelines or the diagnosis and
Cameron JL, Cameron AM. Current Surgical T erapy. 11th ed. management o skin and so tissue in ections: 2014
Philadelphia, PA: Elsevier Saunders; 2013. update by the In ectious Diseases Society o America.
Dellinger RP, et al. Surviving sepsis campaign: international Clin In ect Dis. 2014;59(2):e10–52.
guidelines or management o severe sepsis and septic Strate LL, et al. Nut, corn and popcorn consumption and the
shock: 2012. Crit Care Med. 2013;41(2):581–637. incidence o diverticular disease. JAMA. 2008;300:
Dennis M, T e CLO S rials Collaboration. E ectiveness o 907–914.
thigh-length graduated compression stockings to reduce Suri S, et al. Comparative evaluation o plain lms, ultrasound
the risk o deep vein thrombosis a er stroke (CLO S rial and C in the diagnosis o intestinal obstruction. Acta
1): a multicenter, randomized controlled trial. Lancet. 2009; Radiol. 1999;40(4):422–428.
373:1958–1965. Svensson JF, et al. Nonoperative treatment with antibiotics
Feely MA, et al. Preoperative testing be ore noncardiac surgery: versus surgery or acute nonper orated appendicitis in
guidelines and recommendations. Am Fam Physician. children: a pilot randomized controlled trial. Ann Surg.
2013;87(6):414–418. 2015;261(1):67–71.
Graber MA, et al. University o Iowa: T e Family Practice Hand- T waini A, et al. Fournier’s gangrene and its emergency man-
book. 5th ed. Philadelphia, PA: Elsevier Saunders; 2006. agement. Postgrad Med J. 2006;82(970):516–519.
Lawrence PF, et al. Essentials o General Surgery. 5th ed. T ompson WM, et al. Accuracy o abdominal radiography in
Baltimore, MD: Lippincott Williams & Wilkins; 2013. acute small-bowel obstruction: does reviewer experience
LeBlanc KE, et al. Inguinal hernias: diagnosis and manage- matter?. Am J Roentgenol. 2007;188(3):W233–W238.
ment. Am Fam Physician. 2013; 87(12):844–848. ownsend CM, et al. Sabiston’s extbook o Surgery: T e Biological
Linkins LA, et al. reatment and prevention o heparin- Basis o Modern Surgical Practice. 19th ed. Philadelphia, PA:
induced thrombocytopenia: antithrombotic therapy and Elsevier Saunders; 2012.
prevention o thrombosis. 9th ed. American College o Wade CE, et al. Use o recombinant actor VIIa in US military
Chest Physicians Evidence-Based Clinical Practice casualties or a ve-year period. J rauma. 2010;69:
Guidelines. Chest. 2012;141(2 Suppl):e495S–e530S. 353–359.
Psychiatry
Monika Jindal
23
CASE 23.1 or excessive guilt; (6) oor concentration; and (7) recurrent
thoughts o death, suicidal ideation, atte t, or lan.
You are seeing a 48-year-old emale who presents with a
3-month history o low mood, low energy, poor concentra-
tion, and irritability. She has lost interest in most things she HELPFUL TIP:
had enjoyed and has also noticed a 20-lb weight gain. She has Seve al mnemonics ave been developed o elp clini-
been having requent headaches, has been short tempered, cians emembe e DSM-5 c i e ia and wo o ese a e
and has noticed that it is hard to wake up in the morning. “SIG-ECAPS” and “SPACE DIGS.” Sleep, In e es , Guil ,
She reports no thoughts o suicide but has wondered i death Ene g , Concen a ion, Appe i e, Ps c omo o c anges,
would be a relie . She says she has elt restless or a while Suicidali .
and eels that she is a bad person. Her mother su ered rom
depression. She does not consume alcohol or any other sub-
stances. She is divorced and has no children. Question 23.1.2 Which one o this patient’s symptoms is
You think that this patient may meet criteria or a depres- considered an atypical symptom o “a typica l depression”
sive disorder. or MDD?
A) Hy erso nia
Question 23.1.1 Which o the ollowing is NOT a criterion B) Low ood
or the diagnosis o major depressive disorder (MDD)? C) Anhedonia
A) Low ood D) Psycho otor retardation
B) Presence o suicidal ideation E) Irritability
C) Decreased a etite
D) Anhedonia or loss o interest Answer 23.1.2 The correct answer is “A.” T e ty ical vegeta-
E) Mood uctuations tive sy to s o de ression include oor slee or inso nia,
reduced a etite, and decreased libido. Patients with atypi-
Answer 23.1.1 The correct answer is “E.” T e resence o cal depression have hyperphagia, hypersomnolence, leaden
ood uctuation is not used to ake the diagnosis. Low ood paralysis, and mood reactivity.
should be resent nearly every day to ake the diagnosis. All
o the other o tions listed are art o the criteria set orth in You begin to explain the nature o depression to this patient.
the Diagnostic and Statistical Manual o Mental Disorders, 5th ed.
(DSM-5). S ecial e hasis is laced on the resence o Question 23.1.3 Which o the ollowing epidemiological
de ressed ood and decreased interest, one o which ust be statements is NOT true?
resent in order to diagnose MDD. Instead o de ressed ood, A) T e average A erican has about a 16% chance o develo ing
atients ay re ort so atic co laints or irritability. de ression over his or her li eti e
Criteria or Major Depression: wo weeks or ore o B) About 7% o A ericans su er de ression each year
de ressed ood or loss o interest or anhedonia AND our C) Wo en are ve ti es as likely to su er de ression as are
o the ollowing: (1) signi cant change in weight or a etite; en
(2) inso nia or hy erso nia; (3) sycho otor agitation or D) T e incidence o de ression is increasing in younger cohorts
retardation; (4) atigue or loss o energy; (5) eeling worthless E) Divorced eo le are ore likely to be de ressed

626
CHAPTER 23 • PSyCh IAt r y 627

Answer 23.1.3 The correct answer is “C.” T e li eti e reva-


HELPFUL TIP:
lence o MDD is 16% with 6.9% o adults su ering ro MDD
t e e a e seve al b ie sel - a ing dep ession scales a
in any given year. De ression ay be recurrent throughout
can be used o sc eening (e.g., Beck Dep ession Inven-
one’s li eti e. Wo en are twice (not ve ti es) as likely to have
o , P ima Ca e Evalua ion o Men al Diso de s, Pa-
de ression than are en. T e rate beco es equal in the elderly
ien h eal Ques ionnai e, and Zung Dep ession Scale).
with elderly en being at a higher risk o suicide than elderly
A wo-ques ion sc een ma be as e ec ive o sc eening
wo en. Being divorced is a risk actor or de ression.
as ese mo e ex ensive scales. Simpl ask:

HELPFUL TIP: • Ove e pas 2 weeks, ave ou o en el down, blue


Anxie , no mee ing e level o an anxie diso de , o in e dumps?
equen l accompanies dep ession (70%). h oweve , • Ove e pas 2 weeks, ave ou los in e es in mos
60% o people wi a li e ime diagnosis o MDD ave ac ivi ies o ings a used o b ing ou pleasu e?
ad a diagnosable anxiety disorder. A posi ive answe o ei e o ose wo ques ions
s ould p omp a mo e o oug evalua ion o dep es-
sion. A nega ive answe o bo e ec ivel ules ou de-
T is patient’s situation is not unusual or your practice. p ession in mos pa ien s.

Question 23.1.4 What is the prevalence o MDD in primary


care patients?
You realize that it is also important to ocus on the social his-
A) 1% to 2%
tory in patients with depression.
B) 5% to 10%
C) 25% to 30%
Question 23.1.6 Which o the ollowing is associated with a
D) 45% to 50%
DECREASED risk o depression?
E) 100% (see s like everyone . . . even the clinic sta )
A) Une loy ent
B) Poverty
Answer 23.1.4 The correct answer is “B.” About 5% to 10% o
C) Being un arried
ri ary care atients eet criteria or MDD and about twice o
D) Fa ily history
that (10–20%) any have “ inor” de ression. I you answered
E) Asian race
“E,” you work in a sad, sad lace. Hire a clown to work the ront
desk.
Answer 23.1.6 The correct answer is “E.” Factors associated
with an increased risk o de ression include being un arried
HELPFUL TIP: (single or divorced), having a a ily history o de ression, a
One eason o lack o diagnosis o dep ession in p ima recent childbirth or regnancy, edical co orbidities, lack o
ca e is a dep ession o en p esen s wi soma ic social su ort, emale gender, une loy ent, overty, and
complain s. Ano e is a e spec um o disease substance use. T ere is a lower revalence o de ression a ong
in p ima ca e o ices is less seve e, leading o ewe eo le o Asian and Black ancestry when co ared to the Cau-
diagnoses. casian o ulations. Protective actors include arriage, having
children, and being retired (so, retire ent is when we are su -
osed to have un?).
Question 23.1.5 Which o these epidemiological state -
ments about depression in primary care is true? You believe that your patient is su ering rom an episode o
A) Most de ressed atients (about 80%) seek ental health major depression, and you decide to initiate treatment with
care ro ri ary care hysicians an antidepressant.
B) Only about 50% o the atients with de ression are recog-
nized by their ri ary care hysician Question 23.1.7 Following the rst episode o depression,
C) Only about hal o the atients diagnosed with MDD receive what is her risk o relapse?
adequate treat ent A) < 1%
D) T e 2015 United States Preventive Services ask Force B) 25%
(USPS F) reco ends screening and having adequate sys- C) 50%
te s or treat ent, ollow-u and/or re erral. D) 75%
E) All o the above. E) > 99%

Answer 23.1.5 The correct answer is “E.” All are correct. “D” Answer 23.1.7 The correct answer is “C.” T e incidence o
requires s ecial ention. T e revious (2009) reco endation rela se ollowing the rst e isode o de ression is roughly 50%.
suggested screening contingent u on having adequate services wo-thirds o these rela ses can be revented by continuing
or atients diagnosed with de ression. antide ressant edications chronically. Following the second
628 FAMILy MEDICINE EXAMINAt ION & BOAr D r EVIEW

e isode, the risk o rela se is roughly 75%, cli bing u to 90% improved attitude, and her sleep has become more re resh-
a er the third e isode. In order to reduce the risk o rela se, ing. She wants to know how long she should stay on the medi-
ost authorities reco end antide ressant thera y or 6 to cation.
12 onths a er re ission in atients with their rst e isode o
de ression. Patients who ight bene t ro inde nite edical Question 23.1.10 The correct answer is:
thera y include those with a severe rst e isode (e.g., signi cant A) At least 1 onth
suicide atte t), atients with a sychotic de ression, atients B) At least 2 onths
with three or ore e isodes, elderly atients with their rst e i- C) At least 4 to 6 onths
sode o de ression, and ossibly those with a strong a ily his- D) At least 2 years
tory o de ression. E) Forever

You choose a serotonin reuptake inhibitor (SSRI) and tell Answer 23.1.10 The correct answer is “C.” Di erent organiza-
your patient to contact you i she su ers rom adverse e ects. tions reco end di erent durations o treat ent but the short-
est reco ended course is 4 to 6 onths, with ost authorities
Question 23.1.8 Which o the ollowing is NOT a typical treating or 6 to 12 onths a er recovery.
adverse ef ect o SSRIs? Objectives: Did you learn to . . .
A) Nausea • Diagnose MDD?
B) Headaches • App ecia e e epidemiolog and na u al is o o
C) Restlessness dep ession?
D) Inso nia • r ecognize ea u es o a pical dep ession?
E) Urinary retention • Ini ia e ea men o dep ession?
• Iden i adve se e ec s o SSr Is?
Answer 23.1.8 The correct answer is “E.” Urinary retention is
not seen with SSRIs, but the other our o tions are ty ical o
SSRIs. T e anticholinergic e ect o urinary retention is ty ical CASE 23.2
o the tricyclic antide ressants ( CAs). Sexual dys unction and You are assessing a 45-year-old pro essional male who has
GI roble s are co only seen with SSRIs. Akathisia and dys- a history o MDD in his early 20s but has ully recovered
tonic reactions can (rarely) be seen with SSRIs as well. since then. He recently su ered an uncomplicated anterior
wall myocardial in arction (MI). His wi e mentions that she
Your patient returns to see you 2 weeks a er starting her thinks he is depressed. He is tired all the time, has poor sleep,
SSRI and reports that she has not noticed any bene t rom a poor appetite, and he has been irritable. He has also been
the medication. tear ul and blames himsel or his MI (too many “pounder”
burgers . . . with cheese . . . and bacon). He is willing to con-
Question 23.1.9 Which o the ollowing statements is most sider the diagnosis o depression because he remembers hav-
accurate? ing su ered rom it be ore. He also knows a history o MDD
A) T e antide ressant is not going to work, so she should switch puts him at risk or medical illnesses.
edicines to one o the sa e class
B) T e antide ressant is not going to work, so she should switch Question 23.2.1 Which o the ollowing illnesses is more
edicines to another antide ressant ro a di erent class prevalent in patients with MDD?
C) It is too early to judge the ef cacy o the antide ressant at A) Coronary heart disease
this oint B) Cerebrovascular disease
D) She’s robably su ering a aradoxical reaction o increased C) Diabetes ellitus
de ression with her edication D) Osteo orosis
E) Going u on the dose is not an o tion at this ti e E) All o the above are ore revalent in de ressed atients

Answer 23.1.9 The correct answer is “C.” T e e ects o antide- Answer 23.2.1 The correct answer is “E.” It is unclear i
ressant drugs on de ression usually take 3–4 weeks (although patients with depression are more likely to have these illnesses
side e ects such as sedation, etc. will occur right away). T us, it as a result o the depression or the reverse: patients with these
is re ature to abandon this edication. Most authorities rec- illnesses are more likely to be depressed. De ressed atients
o end trying an antide ressant or 6 weeks at an adequate have an average o 3.4 ore chronic edical conditions than
dose be ore considering that edication a treat ent ailure. nonde ressed atients. At least two studies have now linked a
Increasing the dose or giving it at least 4 to 6 weeks total at the li eti e history o MDD to an increased risk (1.2–3 ti es) o
current dose would be valid o tions. early eno ause (o course, that would be a bit strange in our
atient, a ale).
You increase the medication dose, and the patient returns to
see you in another 2 weeks. T is time, she is eeling better and You know that some o this patient’s symptoms could be
more energetic. People at work are beginning to notice her secondary to his medical illness.
CHAPTER 23 • PSyCh IAt r y 629

Question 23.2.2 Which one o the ollowing symptoms, i


present, is the most specif c or depression? You decide to recommend treatment to this patient.
A) Slee roble s
B) A etite dif culties Question 23.2.4 Which o the ollowing therapies is the
C) Psycho otor agitation LEAST desirable choice or treating his depression?
D) Low energy A) Bu ro ion (Wellbutrin)
E) Excessive reoccu ation with death B) Cognitive behavioral thera y
C) Nortri tyline
Answer 23.2.2 The correct answer is “E.” In the ne onic D) Fluoxetine (Prozac)
“SPACE DIGS,” the last our letters stand or sy to s that are E) Sertraline (Zolo )
ore s eci c or de ression and ore inde endent o so atic
illnesses. T ese sy to s are De ressed ood, loss o Inter- Answer 23.2.4 The correct answer is “C.” CAs should be
ests, ina ro riate Guilt, and thoughts o Suicide (“DIGS”). T e avoided in atients with cardiovascular disease because o their
resence o any o these sy to s should lead you to sus ect arrhyth ogenic otential (e.g., torsades de ointes). Re e -
de ression. ber that SNRIs and citalo ra (Celexa) can also rolong the
Q interval. Bu ro ion (“A”) and cognitive behavioral thera y
(CB ) (“B”) are also reasonable choices. Generally, SSRI ther-
a y (“D” and “E”) would be the re erred drug or the ajority
HELPFUL TIP:
o atients who are candidates or har acothera y.
W ile dep ession can o en be p ecipi a ed b a s es-
so , i can also a ise wi no p ecipi a ing ac o , and
e esponse o ea men is independen o w e e HELPFUL TIP:
i is “ eac ive” (iden i iable s esso p esen ) o “endog- Conside a baseline ECG in all pa ien s o w om ou a e
enous” (no iden i iable s esso ). conside ing t CAs and ci alop am o evalua e e Qt in-
e val. I e Qt is p olonged a baseline, i p edisposes
pa ien s o a mias.
Question 23.2.3 Which o the ollowing statements is NOT
true about depression post-MI?
A) Major de ression is an inde endent risk actor or ost-MI Objectives: Did you learn to . . .
ortality • Diagnose dep ession occu ing wi an acu e medical ill-
B) Minor de ression is an inde endent risk actor or ost-MI ness?
ortality • r ecognize e impac o dep ession on o e medical condi-
C) Minor de ression is ore revalent than ajor de ression ions?
ost-MI • t ea a pos -MI pa ien wi dep ession?
D) reating de ression i roves cardiac outco es in ost-MI
atients
E) A roxi ately hal o the eo le who sustain an MI have QUICK QUIZ: SAD ALL t h E t IME
sy to s o de ression a erward
A 44-year-old atient o yours has co e to see you ulti le
Answer 23.2.3 The correct answer is “D.” In the ost-MI ti es or low ood. She does not have trouble with energy or
eriod, ajor de ression revalence is al ost 20%, and inor slee , but with ro ting re orts occasional roble s with
de ression is about 27%. Concurrent ajor de ression elevates concentration and a etite. Basically, she tells you, “I’ve been
ortality risk a er MI by a actor o 3.5, which is the sa e de ressed or as long as I can re e ber.” You have tried treat-
degree o risk as heart ailure. Patients with a ood distur- ing her with two di erent SSRIs, but she had trouble with side
bance (e.g., inor de ression) also have a higher ortality e ects and did not notice uch i rove ent in her ood.
rate. While treat ent o de ression has been shown to i rove
so e edical outco es (e.g., HbA1C levels in diabetics), this is Which o the ollowing is the best diagnosis or this atient?
not the case in cardiovascular disease. reat ent o de ression A) Adjust ent disorder
does not change ortality or orbidity a er an MI (but ay B) Bi olar a ective disorder
i rove quality o li e). C) Persistent de ressive disorder (dysthy ia)
D) MDD
E) Pre enstrual dys horic disorder
HELPFUL TIP:
Pa ien s su e ing s okes also ave an eleva ed isk o The correct answer is “C.” Persistent de ressive disorder ( or-
dep ession wi app oxima el one- i d o pos -s oke erly dysthy ia in DSM-4) is best thought o as long-ter ,
pa ien s mee ing c i e ia o MDD; e e is a simila co - low-level de ressive sy to s that do not eet criteria or
ela ion o dep essive s mp oms o s oke mo ali . MDD. Sy to s include de ressed ood ost days or at
least 2 years and two or ore o the ollowing: concentration
630 FAMILy MEDICINE EXAMINAt ION & BOAr D r EVIEW

dif culties, ho elessness a etite change, slee disturbance,


HELPFUL TIP:
low energy or atigue, and low sel -estee . In order to diag-
Fi pe cen o pa ien s w o do no espond o an ini-
nose ersistent de ressive disorder, the erson cannot be ree o
ial SSr I will espond o ano e d ug in e same class.
the sy to s or ore than 2 onths during the rst 2 years.
So, c anging o ano e SSr I is easonable in a pa ien
T e treat ent is the sa e as or de ression; ersistent de ressive
w o as ailed one SSr I. Is is obse ved e ec due o
disorder res onds to SSRIs and sychothera y. Many eo le
di e ences be ween d ugs, p a macod namics, p a -
with ersistent de ressive disorder ay also ex erience one or
macogene ics, o longe ea men ? No one knows.
ore e isodes o MDD in their li eti e.

CASE 23.3 When you spin your Random Depression T erapy Wheel
A 34-year-old emale presents to your clinic or treatment o that your occultist riend gave you during the last blue moon,
depression. She reports a li elong history o low-level depres- it lands on venla axine (E exor). So, you start your patient
sive symptoms that have worsened over the past 6 months on that drug. en days later, she calls you just to say that the
since she lost her job (don’t worry—this is a di erent case new medicine seems no better and her sleep is even worse.
than the previous one). She also su ers rom inadequately She started taking the medication in the morning because
controlled diabetes mellitus (HbA1C 9.0%) and has been she thought it might be inter ering with her sleep. She tends
diagnosed with a personality disorder in the past. She drinks to lie in bed or 2 hours be ore alling asleep.
3 to 4 beers everyday (hmm . . . multiply by 2 just to be sa e)
and has been arrested or driving while intoxicated twice. On Question 23.3.3 In addition to recommending good sleep
questioning, you realize she has a moderately severe level o hygiene and increased exercise, you prescribe:
depression. She is not suicidal so you decide to initiate treat- A) razodone
ment as an outpatient with close ollow-up. B) Zol ide
C) Loraze a
Question 23.3.1 Which o these actors does NOT contrib - D) Phenobarbital
ute to a poor outcome when treating depression? E) wo shots o whiskey QHS, re erably a classic like Basil
A) Chronic de ressive sy to s Hayden
B) Fe ale gender
C) Personality disorder Answer 23.3.3 The correct answer is “A.” razodone is o en
D) Co orbid edical conditions added to hel with inso nia and boost serotonergic activity.
E) Alcoholis For de ression, it is re erred to benzodiaze ines unless anxi-
ety is a signi cant issue. Additionally, given this atient’s daily
Answer 23.3.1 The correct answer is “B.” Even though e ales alcohol use, neither benzodiaze ines (nor ore alcohol!) are
are twice as likely to get de ressed as ales, gender does not advised. Phenobarbital should be avoided. Zol ide ight be a
see to in uence treat ent res onse. T e resence o any o the consideration (Hey, I don’t re e ber driving the car in y
other actors listed reduces the chance o a success ul res onse underwear and baking two ans o lasagna last night, but oh
to treat ent. well), but trazodone is re erred in atients with de ression.
Obviously avoid alcohol or slee in this and other atients.
In choosing an antidepressant or this patient, you would
like to use one with a high success rate (as opposed to other HELPFUL TIP:
patients who you might treat with one that doesn’t work or a Indica ions o ps c ia ic e e al o dep ession in-
placebo—what?). However, you know that the success rates clude:
or most antidepressants are airly similar.
• Failu e o medical ea men
Question 23.3.2 What are the chances o this patient ailing • Imminen suicidali
to respond to the rst antidepressant chosen? • Seve e dep ession o w ic ospi aliza ion is oug
o be necessa
A) < 1%
B) 10% to 20% • Diagnos ic cla i ca ion o ea men ecommenda-
ion
C) 30% to 40%
D) 60% to 70% • Como bidi ies a make ea men esponse less
likel
E) 90% to 95%
• Pa ien eques s e e al
Answer 23.3.2 The correct answer is “C.” Studies have con-
sistently shown an antide ressant res onse rate o so ewhere
between 60% and 70% with 30% to 40% ailing to res ond, You add trazodone 50 mg HS PRN and titrate up to the
regardless o what antide ressant is tried. When un ublished maximum dose o venla axine, and your patient’s symptoms
studies are included, the ailure rate or SSRIs a roaches 50%. are now coming under control. When you see her next, she
CHAPTER 23 • PSyCh IAt r y 631

describes a 20-minute episode o chest tightness, dyspnea, (or that o another erson) is in danger. Kee reading or ur-
diaphoresis, and extreme anxiety. You believe that she su - ther ex lanation o why this is the best choice.
ered a panic attack. You wonder i you should alter your
diagnosis. Question 23.4.2 Which o the ollowing is NOT necessary or
a diagnosis o PTSD?
Question 23.3.4 In which o the ollowing disorders do A) T e atient needs to ex erience, witness, or be con ronted
panic attacks NOT occur? by a otentially li e-threatening event, or an event threaten-
A) Panic disorder ing the hysical integrity o the atient or others
B) MDD B) T e atient ust res ond with intense ear, horror, or hel -
C) Generalized anxiety disorder lessness
D) Social hobia C) Sy to s have to be resent or ore than a onth
E) None o the above D) T e atient ust be involved in co bat
E) T e atient ust eet a s eci ed nu ber o sy to s
Answer 23.3.4 The correct answer is “E.” Panic attacks are a
cluster o sy to s signi ying anxiety and are not a disorder by Answer 23.4.2 The correct answer is “D.” Although P SD is
the selves. As such, they can be art o any a ective syndro e co on a ong ilitary veterans (and was reviously known by
and are requently seen in a variety o syndro es including ter s like “shell shock” and “battle atigue”), there is no require-
those entioned above. T e resence o a single anic attack in ent that the atient has been in co bat. o eet criteria or
a erson with a de ressive disorder should not necessarily lead P SD, the atient ust have been ex osed to an event that is
to a new diagnosis. threatening to the integrity or li e o the atient or another. Such
Objectives: Did you learn to . . . events are as varied as co bat, ra e, assault, cancer, or an inten-
• Iden i isk ac o s o a poo ou come w en ea ing sive care unit stay (or, one could hy othesize, taking the boards,
dep ession? erha s?). T e atient then has recurrent intense ear, hel less-
• Assess e likeli ood o a success ul ou come in a pa ien ness, or horror that lasts or > 1 month. Sy to s occurring
wi dep ession? within 4 weeks o the event and lasting or at least 2 days but less
• Gene a e al e na ive me ods o ea ing esis an than a onth can quali y or the diagnosis o acute stress disorder.
dep ession?
Question 23.4.3 Which o the ollowing is NOT a part o
PTSD?
CASE 23.4 A) Night ares
A 41-year-old emale comes to your o ce complaining o di - B) Flashbacks
culty trusting people, irritability, low mood, and recurrent C) Hy ervigilance
nightmares. Her symptoms started ollowing the death o her D) Fear o death
parents in a house re when she was a teenager. She has never E) Dif culty aintaining relationshi s
been able to orgive hersel or surviving while her parents
died. She has not been able to orm close relationships, and Answer 23.4.3 The correct answer is “D.” Fear o death is not a
she is seeking help because o renewed nightmares. T ey were criterion. Sy to s o P SD are divided into our clusters. o
common in the rst 2 years ollowing the incident but had eet the sy to criteria or P SD, the atient needs at least
aded away until recently. Continuing news reports o ter- one sy to ro the rst cluster, one ro the second, two or
rorist activities and bombings (FYI . . . we are on the 4th edi- ore ro the third, and two or ore ro the ourth.
tion here, and this is still relevant . . . sigh) have brought all o T e rst cluster (one required) involves intrusive sy -
this back to the ore ront again. She sometimes wakes up in a to s regarding the trau atic ex erience (e.g., ashbacks,
right a er dreaming that her own house is on re. She avoids night ares, intrusive e ories and sychological distress in
going near any bright lights or reworks displays. When she res onse to triggers that evoke the ex erience).
is orced to be in the presence o res, she requently notices T e second cluster (one required) involves avoidance o
palpitations, dyspnea, and a sense o doom. sti uli associated with the trau a ( eo le, laces, events) and
avoidance o distressing e ories, thoughts, or eelings related
Question 23.4.1 What is the patient’s primary diagnosis? to the trau a.
A) MDD T e third cluster (two or more required) relates to nega-
B) Generalized anxiety disorder tive ood and negative changes in cognition a er the trau atic
C) Panic disorder event. Patients ay re ort a nu bing o general res onsiveness
D) Post-trau atic stress disorder (P SD) ( arkedly di inished interest or artici ation in signi cant
E) Acute stress disorder activities, eeling o detach ent or estrange ent ro others,
inability to ex erience ositive e otions, being in a negative
Answer 23.4.1 The correct answer is “D.” T e atient’s sy - e otional state). T ey ay have negative thoughts distortions
to s are characteristic o P SD, which arises a er one has been toward the selves (negative belie s about the selves or the
ex osed to a situation in which one’s li e or “ hysical integrity” causes o the trau atic event).
632 FAMILy MEDICINE EXAMINAt ION & BOAr D r EVIEW

T e ourth cluster (two or more required) involves ersis- at the time did not support a seizure disorder). Additionally,
tent sy to s o increased arousal, such as dif culty alling or she complains o some vague weakness and numbness and
staying aslee , irritability or outbursts o anger, dif culty con- thinks that she might have had a stroke. Nothing she does
centrating, hy ervigilance, or an exaggerated startle res onse. improves any o these symptoms. Further review o her
records shows a variety o diagnoses rom di erent physi-
Question 23.4.4 Which o the ollowing is true o treatment cians: “chronic pain syndrome,” “chronic atigue,” “ bromy-
or PTSD? algia,” “irritable bowel syndrome,” “premenstrual syndrome,”
A) T ere is no e ective treat ent and others.
B) T ere are no FDA-a roved edications
C) Most atients ex erience s ontaneous re ission Question 23.5.1 Which o the ollowing is the most likely
D) Although treat ent is o en e ective, ost atients do not primary diagnosis?
achieve cure A) Illness anxiety disorder ( or erly hy ochondriasis)
E) Aty ical anti sychotics have no role in treat ent B) So atic sy to disorder ( or erly so atization disor-
der)
Answer 23.4.4 The correct answer is “D.” Sy to i rove- C) Generalized anxiety disorder
ent, but not cure, is the nor . Cognitive behavioral thera y D) Factitious disorder
is the rst-line thera y ollowed by SSRIs then SNRIs. Interest- E) Conversion disorder
ingly, “critical incident debrie ng” (essentially talking through
the incident or reliving the event to try to ini ize sy to s, Answer 23.5.1 The correct answer is “B.” y ically, atients
such as with e otive journaling) actually see s to ake things with unex lained sy to s see any doctors, have nu erous
worse. CB and “objective, none otive” journaling see to tests, and o en undergo a variety o rocedures. At the outset,
hel (see Soc Sci Med. 2011;7:316–326 and Ann Behav Med. it is i ortant to consider a ri ary sychiatric disorder, but
2002;24:244–250, or exa le). T ere is no good ef cacy data a thorough and a ro riate evaluation into ossible organic
or CAs, aty ical anti sychotics, onoa ine oxidase inhibi- causes should be co leted be ore a sychiatric diagnosis is
tors (MAOIs), or other drugs. Prazosin ay hel with slee reached. Many ti es, atients with unex lained sy to s
and night ares but the studies are s all; still, it is worth a trial will ulti ately be diagnosed with a so atic sy to disorder
added to other har acothera y i the atient is not res ond- as well as a related disorder (o which “A,” “D,” and “E” are all
ing in an adequate anner. Psychothera y can be hel ul and ty es).
benzodiaze ine use should be li ited because o the high risk o T is atient resents with so atic sy to disorder, which
de endency in this o ulation and the lack o ef cacy as ono- by de nition ust have:
thera y.
• One or ore so atic sy to that is distressing or disru -
Objectives: Did you learn to . . . tive o daily li e;
• r ecognize isk ac o s o Pt SD?
• Excessive thoughts, behaviors, or eelings associated with the
• Diagnose Pt SD?
so atic sy to s which are exhibited by dis ro ortionate
• t ea Pt SD?
thoughts about the seriousness o sy to s, high anxiety
about sy to s, or large a ounts o ti e and energy s ent
CASE 23.5 on the health concerns;
• Sy to s are ersistent, generally at least 6 onths.
A 29-year-old emale presents to you or a second opinion,
bringing a large stack o medical records with her. In act, she “A” is incorrect. Illness anxiety disorder is characterized by
says that this is more like a “ ourth or h opinion” (lucky a preoccupation that one has some sort o serious disease,
you). She heard rom a riend o a riend that you are the and somatic symptoms are either not present or are less
smartest doctor around and could determine what is wrong intense. “C” is incorrect because anxiety is resent but totally
with her. In act, she notes, “None o those other doctors overshadowed by the so atic co laints. “D,” actitious dis-
understand my symptoms at all. But you will. T ey say you’re order, is diagnosed in atients who intentionally roduce
the best.” sy to s (e.g., overdosing on insulin or injecting eces into
Although you nd many negative diagnostic studies in her the bloodstrea ) to assu e the “sick role.” “E” is incorrect.
record, she is sure that her many symptoms must have some With conversion disorder, atients resent with sudden
physical cause. Over the last ew years, she has had chronic onset o anato ically i lausible neurological sy to s
headaches, multiple joint pains, and intermittent abdomi- (e.g., whole-body nu bness and bilateral dea ness). Conver-
nal and chest pains. Her symptoms have prevented her rom sion disorder is, by de nition, not under conscious control. I
working. She has diarrhea and bloating on occasions, but it is under conscious control, the diagnosis is “ alingering”
upper endoscopy, colonoscopy, C scans, and other studies (really . . . that is a diagnosis). Re e ber that conversion dis-
have not revealed an etiology. She reports severe anxiety that order is, to so e degree, a diagnosis o exclusion. You need
has worsened with the onset o “seizures” in the last year (and to ake sure there is not an organic basis or the atient’s
you note that brain MRI, EEG, and neurological examination sy to s.
CHAPTER 23 • PSyCh IAt r y 633

You believe that this patient has somatic symptom disorder. HELPFUL TIP:
In pa ien s wi soma ic s mp oms diso de w o a e
Question 23.5.2 You begin by saying: willing o be e e ed o men al eal p ovide s, indi-
A) “Relax. T is is all in your head.” vidual ps c o e ap , CBt , and g oup e ap ma be
B) “Your ain isn’t real. But your sychiatric illness is. It’s called bene icial.
so atic sy to disorder.”
C) “You have a nu ber o sy to s that are very real but can-
not be ex lained by our investigations. T e evidence sug- Objectives: Did you learn to . . .
• r ecognize and desc ibe soma ic s mp om diso de ?
gests that you do not have any li e-threatening illnesses. You
have a well-de ned disorder, and other atients have si ilar • Gene a e an app op ia e plan o a pa ien wi soma ic
s mp om diso de ?
roble s.”
D) “You have a lot o very serious sy to s. But y hysical
exa is inconsistent with your co laints. Basically, I don’t CASE 23.6
believe a word your saying. T e sooner that you ad it to al- A 21-year-old college student presents to your o ce or evalu-
si ying these sy to s, the sooner I can start hel ing you.” ation. She is complaining o eeling stressed out. She is taking
E) “I have a s ecialist to re er you to in Outer Mongolia . . . take classes ull-time and is also in one o the military reserve units
your ti e getting back.” at the college. One weekend a month, she must attend drill that
involves handling weapons. Although she did not have prob-
Answer 23.5.2 The correct answer is “C.” In atients with lems handling the weapons initially, she now gets very emo-
so atic sy to disorder, it is best to use an honest but gentle tional and upset when she thinks about having to use them at
a roach. Most atients eel better i they have a na e or an ill- the next drill weekend. She is nervous and is a raid that she
ness, but using the ter “so atic sy to disorder” ay actually might accidentally re a weapon. She knows that her ears are
be detri ental. It is i ortant to af r the atient’s sy to s silly and she has been telling hersel to “just get over it.” Last
(these are real roble s causing real distress) and to try to nd weekend, while at drill, she suddenly elt that she was going to
so e co on language to use to describe what he or she is eel- have a heart attack. She developed tightness in her chest, her
ing. Patients ay not be rece tive to your inter retation initially, heart was racing, and she elt unable to breathe. Although the
but re eating the discussion at subsequent visits and ocusing on symptoms eventually abated, the episode made her even more
nor al diagnostic tests ay hel the acce t the diagnosis. alarmed, and now she is worried that it will happen again and
she will have a heart attack. She comes to see i you can help.
Next, you discuss a plan o action with this patient.
Question 23.6.1 Which o the ollowing is UNNECESSARY
Question 23.5.3 You recommend: or the initial workup?
A) A ultidisci linary a roach, utilizing any di erent s e- A) ake ore edical, sychiatric, and a ily history
cialty services B) Order an echocardiogra
B) Starting an SSRI C) Per or a hysical exa ination
C) Starting electroconvulsive thera y (EC ) D) Order thyroid unction tests
D) Monthly visits with you and li ited diagnostic testing and E) Per or a ental status exa ination
s ecialty consultation
E) Re erral or ex loratory la aroto y Answer 23.6.1 The correct answer is “B.” Given the in or a-
tion you have so ar, an echocardiogra is not indicated and
Answer 23.5.3 The correct answer is “D.” It is best to have is rarely used as art o a ri ary worku . When it co es to
atients with so atic sy to disorder establish regular clinic test questions, never say no to ore history (unless you are su -
visits, ideally with one rovider. Unscheduled visits to the e er- osed to be anaging a atient’s airway). A co lete history
gency de art ent should be discouraged unless rst discussed and hysical exa ination are essential in the evaluation o this
with the ri ary rovider. Patients should be allowed to dis- new atient. T yroid abnor alities can be a cause o so e o
cuss all o their co laints, and a hysical exa ination should these sy to s, including al itations and chest tightness.
be er or ed at every of ce visit. T ese easures will let the
atients know that their concerns are being taken seriously. She has no prior psychiatric history, but her mother is taking
Laboratory tests, radiogra hs, and consultations should be li - medication or depression. While taking her social history,
ited, with the clinician using his or her best judg ent as to when you ask questions regarding substance abuse.
such diagnostic tests are indicated. “B” and “C” are incorrect as
SSRIs and EC are not acce ted thera y or so atic sy to Question 23.6.2 Use o which o the ollowing substances
disorder. But, you could robably rescribe an aty ical anti- might explain her symptoms?
sychotic or benzodiaze ine (neither o which work) but will A) Nicotine
sedate the atient enough that she won’t bother you any ore B) Alcohol
(OK, don’t do that . . . it’s just a joke). C) Ca eine
634 FAMILy MEDICINE EXAMINAt ION & BOAr D r EVIEW

D) Herbal weight loss edication


HELPFUL TIP:
E) Any o the above
h pe ven ila ion can cause ce eb al vasocons ic ion
and seconda ce eb al poxia esul ing in s ncope.
Answer 23.6.2 The correct answer is “E.” Many substances can
t is can occu wi panic diso de . h oweve , is is
cause sy to s like this atient has. T ese include sti ulants,
p e unusual.
such as ca eine and nicotine, and so e herbal weight loss rod-
ucts containing e hedra (Ma Huang). Withdrawal ro hy -
notics like alcohol can also lead to si ilar sy to s. o make your diagnosis o panic disorder, the patient needs
to meet certain criteria.
She does not smoke, doesn’t drink alcohol (you can tell
she’s not a Hawkeye . . . no tailgating?!), and drinks co ee on Question 23.6.5 Which o these is NOT a criterion or panic
weekday mornings be ore class. Her physical examination is disorder?
normal. Her mental status examination is remarkable or a A) Recurrent and unex ected anic attacks
neutral mood, a restricted and anxious a ect, but no suicidal B) At least 1 onth o worry about having ore attacks
thoughts, and no psychotic symptoms. T e laboratory tests C) Worry about the i lication o the attack or its conse-
you order are normal. You are leaning toward a psychiatric quences (dying, “going crazy,” etc.)
diagnosis at this point, speci cally an anxiety disorder. D) Change in behavior related to the attacks
E) Predictable anic attacks that occur in res onse to cues
Question 23.6.3 Which o the ollowing is NOT an anxiety
disorder?
Answer 23.6.5 The correct answer is “E.” T e criteria or anic
A) Panic disorder
disorder do not include redictable anic attacks in res onse to
B) Social anxiety disorder
cues. A er atients have had re eated anic attacks, they o en
C) Agora hobia
develo hobic avoidance o laces, objects, or events associ-
D) Generalized anxiety disorder
ated with their sy to s (e.g., agora hobia). Agora hobia is a
E) Deliriu
se arate diagnosis in DSM-5. Patients o en scout out routes o
esca e be ore going to laces that ight rovoke an attack—o
Answer 23.6.3 The correct answer is “E.” Deliriu , while it
the anic variety as o osed to the terrorist variety.
ay resent with eatures si ilar to an anxiety disorder, is a
cognitive disorder. Anxiety disorders are the ost co on
HELPFUL TIP:
or o ental illness in the United States, with a li eti e rev-
t e vas majo i o pa ien s wi panic diso de p esen
alence o 28.8%. T ey include o tions “A” through “D” as well
wi soma ic complain s a e an cogni ive o mood
as se aration anxiety disorder, s eci c hobias, selective ut-
s mp oms. Pa ien s a e o en misdiagnosed ini iall .
is , uns eci ed anxiety disorder, and anxiety disorders that are
Conside panic a acks in pa ien s p esen ing wi e
judged to be secondary to a edical condition or a substance.
app op ia e soma ic s mp oms. And neve ule ou a
p sical diagnosis (e.g., PSVt ) wi ou an app op ia e
O the listed anxiety disorders, you think she has developed
wo kup.
panic disorder and that she has been having panic attacks.

Question 23.6.4 Which o the ollowing is NOT a typical Your patient is worried that these attacks will keep occurring.
symptom o a panic attack?
A) Pal itations Question 23.6.6 How would you best describe the progno-
B) Dia horesis sis or panic disorder?
C) Synco e A) It is easily curable in ost atients
D) Dys nea B) T ere is no e ective drug treat ent beyond benzodiaz-
E) Dizziness e ines, es ecially al razola (Xanax)
C) Most atients do not i rove over ti e
Answer 23.6.4 The correct answer is “C.” T e sy to s o a D) It is a recurrent or chronic illness
anic attack are those associated with an activation o the “ ght-or- E) None o the above
ee res onse,” or the overactivation o the sy athetic nervous
syste . y ical sy to s include chest ain, nausea, dizziness, Answer 23.6.6 The correct answer is “D.” Panic disorder is a
nu bness, al itations, sweating, tre bling, dys nea, a sense o recurrent or chronic disease in ost atients. “A” and “C” are
s othering, ear o dying, tingling, and derealization or de er- incorrect. Although anic disorder is not easily curable, al ost
sonalization. (At this oint, we’d like to ask Mother Nature how all atients will i rove over ti e, but very ew attain co lete
chest ain hel s us when acing a saber-toothed tiger.) It would re ission even with edical treat ent. Rela se is co on.
be rare or a atient to actually lose consciousness ro a anic
attack (though they ay eel they ight), and actual synco e T e patient is, o course, very concerned about uture panic
should orce one to look or an alternate diagnosis. attacks. She asks what to do when another occurs.
CHAPTER 23 • PSyCh IAt r y 635

Question 23.6.7 You advise her to do all o the ollowing de endence, and abuse li it benzodiaze ines as long-ter
EXCEPT: agents or anic disorder. Patients with anic disorder are
A) Move to a quiet area extre ely sensitive to edication side e ects and are likely to
B) Slow down her breathing su er ro jitteriness and restlessness i started at a high dose
C) Reassure hersel that she is not dying o an SSRI. In order to reduce the chances o reci itating jit-
D) Breathe into a brown a er bag teriness and restlessness, a lower dose o SSRI (about hal the
E) Avoid sti ulants like ca eine or nicotine starting dose used to treat de ression) is usually reco ended.
T e sa e roble s can occur with ra idly increasing doses
Answer 23.6.7 The correct answer is “D.” All o the other o SSRIs, so “start low and go slow,” increasing the dose every
o tions are reasonable reco endations or a atient su ering 2 to 4 weeks to reach the axi u allowable dose. Benzodi-
ro anic attacks. Although co only observed in o ular aze ines are at least third line (a er CB and SSRIs). A plea:
lore, breathing into a a er bag is not reco ended. Breath- don’t rescribe al razola . It is addictive, short-acting, and has
ing into a brown a er bag can have the o osite e ect o that a rebound e ect ( aking de endence even ore likely). I you are
intended—the atient ay continue hy erventilating with CO2 going to choose a benzodiaze ine, choose one that is longer
building u , which ay contribute to ore anic sy to s. acting such as clonaze a or diaze a . a er benzodiaze ines as
Hy erventilating (blowing o CO2) does not roduce a anic soon as ossible. Avoid the in atients with severe ersonality
attack nor relieve it. It’s just a sy to o one. However, edu- disorders and substance abuse.
cating the atient on consciously slowing her breathing—using
slow, dee breaths— ay hel abort the anic attack. Quiet Your patient says, “I know another emale in the reserve who
roo s and reassurance can also hel . has something similar, but I’ve never noticed the guys to have
a problem. Is this just something that happens to women?”
You begin to discuss treatment options with this patient. You tell her a bit about gender di erences in anxiety disor-
ders.
Question 23.6.8 Which o the ollowing is NOT an ef ective
treatment o panic disorder? Question 23.6.10 Which o these is NOT true about the gen-
A) Benzodiaze ines such as clonaze a (Klono in) der ratio o the ollowing disorders?
B) Bu ro ion (Wellbutrin) A) Generalized anxiety disorder a ects ore wo en than en
C) Psychothera y such as CB in a 2:1 ratio
D) SSRIs such as uoxetine (Prozac) B) Panic disorder a ects ore wo en than en in a 2:1 ratio
E) CAs such as i i ra ine C) P SD a ects ore en than wo en in a 2:1 ratio
D) Social anxiety disorder a ects slightly ore wo en than
Answer 23.6.8 The correct answer is “B.” Bu ro ion does en
not work or anxiety disorders. A ong e ective edications
(SSRIs, SNRIs, CAs), no class has roven su erior, and edi- Answer 23.6.10 The correct answer is “C.” Many a ective ill-
cation choice is based on sa ety, adverse e ect ro le, toler- nesses ( ood and anxiety disorders) including de ression and
ability, co orbid illnesses, history o substance use, cost, etc. so e anxiety disorders are biologically sexist: wo en are ore
SSRIs are requently considered rst line. Psychothera y has likely to be a ected than en. All o the o tions listed are cor-
been ound to be as e ective as edications or the treat ent o rect exce t “C,” which inverts the true ratio.
ild-to- oderate anic disorder and can be used in co bina-
Objectives: Did you learn to . . .
tion with edications or ore severe cases.
• Evalua e pa ien s wi anxie s mp oms?
As you decide on the medication and dosage, you remember • r ecognize panic a acks and diagnose panic diso de ?
an article you just read in a journal on common mistakes • Ini ia e ea men o panic diso de ?
made by physicians treating panic disorder in the community. • Desc ibe some epidemiologic issues wi anxie diso de s?

Question 23.6.9 Which o these is NOT one o the common


mistakes made in the treatment o panic disorder?
CASE 23.7
A) Starting the SSRI too high You have a 34-year-old male patient who has started sustained-
B) Not achieving a high enough target dose release bupropion (Wellbutrin SR) 150 mg BID or depression.
C) Underutilization o benzodiaze ines He reports partial but not total resolution in his symptoms.
D) oo ra id a titration He also thinks that the medication is causing some side
E) O en using edications not roven to work with anic e ects.
disorder
Question 23.7.1 Which o the ollowing is the most likely
Answer 23.6.9 The correct answer is “C.” Far ro being adverse ef ect attributable to this medication?
underutilized, benzodiazepines are o en overprescribed. A) Inso nia
While e ective or anic disorder, roble s with tolerance, B) Sexual dys unction
636 FAMILy MEDICINE EXAMINAt ION & BOAr D r EVIEW

C) Weight gain agent is started. “A” is incorrect. Bu ro ion can be taken with
D) Q rolongation an SSRI. However, “D” is less desirable, as it is re erable to
E) Increased s oking sto the bu ro ion rather than continuing it inde nitely while
taking another edication. A single e ective agent is generally
Answer 23.7.1 The correct answer is “A.” Bu ro ion is associ- re erred i ossible.
ated with vivid drea s and inso nia. aking the second dose
Objectives: Did you learn to . . .
no later than 4 pm can hel reduce the likelihood o this side
• Use bup opion app op ia el ?
e ect. Unlike ost other antide ressants, bu ro ion is not
associated with sexual dys unction or weight gain. In act, it has • Iden i adve se e ec s o bup opion?
been associated with weight loss in the short ter , and atients • r ecommend s a egies o ansi ion om bup opion o
who quit s oking are less likely to gain weight i they are tak- ano e an idep essan ?
ing it. Bu ro ion can cause dry outh and nausea, which are
usually sel -li ited. It does not cause Q rolongation, unlike QUICK QUIZ: h OMESICK
CAs, so e SNRIs, and citalo ra . Bu ro ion a ears to curb
the cravings or nicotine. It lowers the seizure threshold and
A 22-year-old college student who oved to your town ro
should be avoided in atients with e ile sy.
Lagos, Nigeria, last onth is re erred to you by his acade ic
advisor or concerns that he ay be de ressed. Although he
T is patient is relatively healthy and takes no other medica-
s eaks English well, he does not know anyone in town. His
tions, giving you a wide number o options or treatment. In
a ily is still in Lagos, and he does not antici ate returning
other words, you did not have to think too hard be ore start-
to see the anyti e soon. He has not ade any riends yet.
ing bupropion. However, sometimes you have to use your
He enjoys watching soccer in his s are ti e (which, o course,
brain, so let’s think about when not to use bupropion.
nobody in the United States a reciates, but that is another at-
Question 23.7.2 In which o the ollowing disease states ter), but he is not slee ing well, and he eels quite ho esick.
would bupropion be contraindicated?
Which o the ollowing is the ost a ro riate diagnosis at this
A) Hy ertension
ti e?
B) Severe de ression
A) MDD
C) Buli ia nervosa (BN)
B) Adjust ent disorder
D) Bi olar de ression
C) Bi olar a ective disorder
E) Borderline ersonality disorder
D) Dysthy ia
E) Footballer’s withdrawal syndro e
Answer 23.7.2 The correct answer is “C.” Bu ro ion is con-
traindicated in atients with anorexia nervosa (AN) or BN (an
The correct answer is “B.” Adjust ent disorder is the devel-
increased risk o seizures is ound in both diseases) or a cur-
o ent o e otional or behavioral sy to s in res onse to
rent or ast seizure disorder. Additionally, bu ro ion ust be
identi able stressor(s) occurring within 3 onths o the onset
avoided in atients taking MAOIs. It is as e ective as the other
o the stressor(s). T e sy to s or behaviors ust be clinically
antide ressants, and it can be used in bi olar disorder and
signi cant, such that either the atient’s distress is in excess o
severe de ression.
what would be ex ected ro ex osure to the stressor, or there
should be i air ent in social or occu ational/acade ic unc-
You decide to switch his medication rom bupropion to cita-
tioning. T e atient’s sy to s cannot be due to bereave ent
lopram (Celexa).
(a se arate diagnosis). Once the stressor (or its consequences)
is ter inated, the sy to s should resolve within 6 onths.
Question 23.7.3 Which is the best way to accomplish this
reat ent de ends on the level o distress and can range ro
switch?
su ortive care to active intervention with edications, thera-
A) Sto the bu ro ion i ediately and then wait or 2 weeks
y, or hos italization.
be ore starting citalo ra
B) a er the bu ro ion over 2 weeks to avoid a discontinuation
syndro e, and then start the citalo ra
CASE 23.8
C) Start citalo ra i ediately, and then ta er the bu ro ion An 85-year-old woman is brought to the emergency depart-
over several days ment by her daughters because she has been acting strangely
D) Start citalo ra now; then ta er o the bu ro ion a cou le lately. Her house is a mess, even though or most o her li e
o weeks later i he is doing well she has been quite astidious. She has been calling her daugh-
ters at odd hours o the night, upset and insisting that her
Answer 23.7.3 The correct answer is “C.” Unlike SSRIs, which youngest daughter is stealing her money. During the day, she
can cause a serotonin withdrawal syndro e, bu ro ion can be goes outside o her house in her nightgown and housecoat,
discontinued with a ini al (i any) ta er. A brie ta er ay again quite unlike her usual customs. Four months ago, her
avoid a sudden rebound o de ressive sy to s, while the new husband o 58 years was diagnosed with a brain tumor. His
CHAPTER 23 • PSyCh IAt r y 637

condition has deteriorated quite rapidly, and he is now in a Answer 23.8.3 The correct answer is “A.” Psychothera y is
nursing home and does not recognize his wi e or daughters. not an a ro riate single thera y or sychotic de ression—
T e daughters have attributed the patient’s odd behavior to es ecially in this atient, whose sy to s are so severe. “B” is
the stress she is under. But as her symptoms have continued a ro riate, as the atient ay bene t ro hos italization. “C” is
to worsen, they are now quite concerned and decide they a ro riate as well. EC is e ective in treating sychotic de res-
must bring her in or evaluation. sion and is o en indicated i sa ety is o i ediate concern or an
In the emergency department, the patient is dressed in early res onse is needed. “D” is a ro riate. In general, atients
her housecoat and slippers and appears disheveled. She is with sychotic de ression should be re erred to a sychiatrist.
lying on a cart, but she keeps trying to get up and leave. She
is angry at her two daughters who are with her, and she is You determine that your patient’s symptoms are severe
uncooperative with the physical examination. When you ask enough or hospitalization. Her daughters agree, but the
her about what has been happening, she appears distrusting, patient is adamantly opposed and insists on returning to her
and her answers do not make sense. home. She’s sick o you “NSA-types pushing her around,” and
“i you don’t let her go, she’s going call her riend, Donald
Question 23.8.1 Which o the ollowing diagnoses is LEAST rump.”
likely at this point in time?
A) Deliriu Question 23.8.4 How would you proceed?
B) De entia A) Call hos ital security and ake lans to hos italize her;
C) Bereave ent a er all, she a ears quite ill and cannot care or hersel ;
D) Psychotic de ression when she is well, she will understand that it was the right
E) Alcohol abuse lan
B) Discharge her ho e, asking her daughters to take turns
Answer 23.8.1 The correct answer is “C.” Although your staying with her until she is better
atient’s husband is gravely ill and she is grieving, her sy - C) Follow state-dictated rotocol to atte t to obtain a legal
to s are ore severe than what is ex ected or bereave ent. order or hos italization against her will
Bereave ent is a nor al rocess, but it does not include severe D) Follow national rotocol (New World Order Directive
i air ent in social or occu ational unctioning, nor does it 55.12.A) to atte t to obtain a legal order or hos italiza-
include aranoid delusions or other sychotic sy to s. All o tion against her will
the other diagnoses listed could result in the severity o sy - E) Make a edical deter ination that she is not co etent,
to s described in this case. allowing you to hos italize her des ite her objection

Answer 23.8.4 The correct answer is “C.” Each state has its own
Question 23.8.2 Which o the ollowing tests is indicated in laws that govern how an involuntary hos italization rocess is
the evaluation o this patient’s behavior changes? conducted. I the atient oses an i inent threat to hersel or
A) Chest x-ray others, the law generally allows involuntary hos italization or
B) Urinalysis a brie eriod o ti e until a court hearing is held. Involuntary
C) CBC hos italization is also allowed i the atient is unable to care or
D) B-12 levels hersel and she is su ering ro a ental illness that renders
E) All o the above her inca able o aking healthcare decisions. “D” is incorrect.
T ere are no national laws governing involuntary hos italiza-
Answer 23.8.2 The correct answer is “E.” A thorough edical tion (and the New World Order is a aranoid delusion o so e
worku is warranted in the atient with “ ental status changes” extre ists . . . or aybe we just want you to believe that). “B” is
(e.g., deliriu and new onset sychotic sy to s). Several incorrect. Because o the severity o this atient’s sy to s, it
edical roble s—in ection, hy oxia, MI, to na e a ew—can would be ina ro riate to si ly discharge her to the care o
cause the sy to s the atient is ex eriencing and ust be her daughters.
ruled out.
You have admitted the patient under a 72-hour hold, and you
Her workup does not reveal an organic cause or her altered are discussing treatment options with the psychiatrist, who
mental state. You are now suspecting that she has psychotic thinks that EC might be appropriate. You are concerned
depression. about cognitive problems in this patient.

Question 23.8.3 All o the ollowing are appropriate man- Question 23.8.5 All o the ollowing are associated with an
agement options at this time EXCEPT: increased risk o memory loss with ECT EXCEPT:
A) Discharge with re erral or out atient sychothera y A) Conco itant lithiu use
B) Ad ission to a sychiatric unit B) Bilateral electrode lace ent
C) EC (electroconvulsive thera y) C) High sti ulus doses
D) Psychiatric consultation D) History o seizure disorder
638 FAMILy MEDICINE EXAMINAt ION & BOAr D r EVIEW

Answer 23.8.5 The correct answer is “D.” Seizure disorder is C) MDD


not associated with an increased risk o e ory loss with EC . D) Bi olar e ective disorder
EC is known to cause transient roble s with e ory loss. E) Dysthy ia
T is usually ani ests with the atient having trouble re e -
bering events that occur around the ti e o the EC (such as The correct answer is “A.” Bereave ent is a natural reaction
the invasion o exas by the UN . . . oh, aybe we are aranoid to a signi cant loss. So e o the sy to s ay i ic those
and deluded now . . . or not). Most o the e ory co laints o de ression or adjust ent disorder, which can cause throw
co letely resolve within a ew onths o co leting EC . you o the track o aking the ro er diagnosis. Unlike be-
However, so e situations ay increase the risk o e ory loss, reave ent, both de ression and adjust ent disorder ust, by
including conco itant lithiu use, bilateral electrode lace- de nition, cause signi cant i air ent. Our atient has only
ent (unilateral is sa er), and a higher sti ulus dose. a inor decre ent in her level o unctioning. T e duration
and ex ression o bereave ent di ers a ong di erent cultur-
In urther discussions with the patient and her amily, you al grou s and subgrou s, urther co licating the diagnostic
try to explain EC and dispel some myths. rocess. Generally, a diagnosis o MDD is not given unless the
sy to s o de ression are resent 2 onths a er the loss.
Question 23.8.6 All o the ollowing are potential complica-
tions or adverse ef ects o ECT EXCEPT: CASE 23.9
A) Deliriu
B) Nonsustained ventricular tachycardia You get a call rom a patient complaining o “not eeling well
C) Headache and getting worse” over the past 3 days. She has had electrical
D) De entia shock sensations in her upper extremities and head and also
E) Fatigue complains o dizziness and malaise. She takes no medications
since she stopped her paroxetine (Paxil) a ew days ago.
Answer 23.8.6 The correct answer is “D.” Although transient
e ory loss and even deliriu can occur a er EC , it does not Question 23.9.1 Which o the ollowing is the most likely
cause de entia. A nu ber o cardiac rhyth disturbances can diagnosis?
occur and are ore likely in atients with cardiac disease. How- A) In uenza
ever, these are sel -li ited and generally inor (e.g., re ature B) Bu ro ion discontinuation syndro e
ventricular contractions, atrial re ature co lexes, and non- C) SSRI discontinuation syndro e
sustained ventricular tachycardia). Headache and atigue are D) Serotonin syndro e
co on a er EC . E) Hy ertensive crisis

Objectives: Did you learn to . . . Answer 23.9.1 The correct answer is “C.” OK, so this was a
• r ecognize abno mal be avio and gene a e an app op ia e so itch. We already told you above that bu ro ion does not
di e en ial diagnosis? have a discontinuation syndro e. A discontinuation syndro e
• Evalua e a pa ien wi new cogni ive and be avio al p ob- (also known as withdrawal syndro e) has been associated with
lems? SSRIs when they are suddenly sto ed or doses are issed or
• De e mine w en involun a ospi aliza ion is app op ia e reduced. T is is es ecially co on with aroxetine because o
and ow i mig be unde aken? its short hal -li e. T e sy to s described above are ty ical o
• Discuss po en ial adve se e ec s o ECt ? the discontinuation syndro e, articularly the aresthesias that
• Become mo e suspicious o e New Wo ld O de ? atients o en describe as an “electric shock” ( able 23-1).

Question 23.9.2 Which o the options below is INCORRECT


QUICK QUIZ: A r EALLy BAD WEEK in ormation to give the patient over the telephone?
A) Sy to s usually begin within a ew days o sto ing the
A 47-year-old e ale resents to your clinic in tears, request- SSRI
ing your hel . She cannot sto crying, her slee is oor, and B) T e sy to s rarely last longer or 2 weeks and are sel -
she eels terribly lonely. However, she is able to carry out all li e li ited
unctions with little decre ent in ability. She tells you that last C) Restarting the edication takes about 2 weeks to relieve
week her other had a stroke. She survived, but is now in a sy to s
nursing ho e and su ering ro Broca’s a hasia. Your atient D) T e longer the atient has been on the edication, the ore
describes her other as her “best riend.” She has not had trou- likely the risk o a discontinuation syndro e
ble with de ression or other ental illness in the ast.
Answer 23.9.2 The correct answer is “C” (which is not
Which o the ollowing is the ost likely diagnosis? true). Sy to s can be noticeable in so e atients with just
A) Bereave ent one issed dose (es ecially with aroxetine [Paxil]) but ty i-
B) Adjust ent disorder cally develo within 2 days o edication discontinuation. T e
CHAPTER 23 • PSyCh IAt r y 639

TABLE 23-1 SYMPTOMS OF SSRI o di erent medications, you remind yoursel about some
DISCONTINUATION SYNDROME important issues. For example, all SSRIs are not the same.
Somatic Symptoms
Question 23.9.4 Which SSRI has the most anticholinergic
• Disequilib ium (e.g., dizziness, ve igo, a axia, and emo )
• Gas oin es inal s mp oms (e.g., nausea, vomi ing, and ano exia) activity?
• Flu-like s mp oms (e.g., a igue, le a g , m algias, c ills, and A) Fluoxetine
eadac e) B) Paroxetine
• Senso dis u bances (e.g., pa es esias and sensa ions o elec ic C) Sertraline
s ock)
D) Citalo ra
• Sleep dis u bances (e.g., insomnia and vivid d eams)
E) Escitalo ra
Psychological Problems
• Anxie /agi a ion Answer 23.9.4 The correct answer is “B.” With the exce tion
• C ing spells
o aroxetine, SSRIs do not ossess a reciable anticholinergic
• I i abili
• Ove ac ivi activity. Do you get the idea we don’t like aroxetine?
• Depe sonaliza ion
• Dec eased concen a ion/slowed inking Question 23.9.5 Which o the ollowing SSRIs is LEAST likely
• Con usion and memo p oblems to have drug–drug interactions via P450 enzymes?
A) Fluoxetine
B) Paroxetine
syndro e is unco ortable but usually sel -li ited, lasting C) Citalo ra
< 2 weeks. Restarting the edication will lead to cessation o D) Sertraline
sy to s within 24 hours in virtually all cases. T e risk o dis- E) Fluvoxa ine
continuation increases with length o thera y, articularly when
the atient has been on an SSRI longer than 7 weeks. Answer 23.9.5 The correct answer is “C.” Citalo ra and its
stereoiso er, escitalo ra , have relatively clean ro les with no
Question 23.9.3 Which SSRI is LEAST likely to cause a dis- ajor interactions with any o the cytochro e P450 enzy es.
continuation syndrome? Sertraline is also an attractive o tion i drug–drug interactions
A) Fluoxetine are a concern. T e other three have signi cant drug–drug inter-
B) Sertraline actions that have clinical i ortance. O course, this bene t o
C) Paroxetine citalo ra is te ered by its known roble s with Q rolon-
D) Citalo ra gation which can be exacerbated by other Q rolonging drugs.
E) Fluvoxa ine
Question 23.9.6 Mirtazapine (Remeron) is commonly asso-
Answer 23.9.3 The correct answer is “A.” T e risk o withdrawal ciated with the ollowing side ef ects EXCEPT:
increases with shorter hal -lives. T e hal -li e o uoxetine is A) Sedation
about 4 to 6 days, while its active etabolite nor uoxetine has a B) Weight gain
hal -li e u to 16 days, aking it highly unlikely that uoxetine C) Dizziness
would cause a discontinuation syndro e in ost atients, as it D) Lycanthro y
e ectively acts as its own ta er. Fluvoxa ine (with a hal -li e o E) Increased triglycerides
15 hours) and aroxetine (21 hours) have the shortest hal -lives
and are ost likely to cause the discontinuation syndro e. o Answer 23.9.6 The correct answer is “D.” Lycanthro y is, o
ini ize the risk o withdrawal syndro e, ta er o SSRIs when course, the ability to trans or into a werewol . Although close
sto ing the . in behavior to antisocial ersonality disorder, lycanthro y is
not caused by irtaza ine but rather by the bite o another
lycanthro e. Mirtaza ine is a otent antihista ine, and an
HELPFUL TIP: α 1-adrenergic agonist, otentially leading to orthostatic hy o-
Discon inua ion s nd ome is likel a pose o one gic tension. Sedation is resent in over 50% o atients taking the
s a e; e e o e, all se o one gic agen s can cause a se- drug, while weight gain is re orted in as any as 12%. It can
o onin wi d awal. t ese agen s include SSr Is, MAOIs, also lead to increased triglycerides. Paradoxically, the side e ect
t CAs, se o onin–no epinep ine eup ake in ibi o s o sedation lessens with increasing doses. It is a great antide-
(SNr Is), and mi azapine (r eme on) . . . bu no bup opi- ressant or atients who have lost their a etite, lost weight, or
on (is a o se dead o s ould we bea i some mo e?). have inso nia. y ical doses range ro 15 to 60 g/day.

Question 23.9.7 Which o the ollowing antidepressants


Your patient restarts her SSRI and eels better. She has ques- has a black box warning about hepatic ailure?
tions about antidepressants in general, wondering i she is A) Ne azodone (Serzone)
taking the right one. As you start to discuss bene ts and risks B) Bu ro ion
640 FAMILy MEDICINE EXAMINAt ION & BOAr D r EVIEW

C) Paroxetine (Paxil) Objectives: Did you learn to . . .


D) Nortri tyline • Iden i SSr I discon inua ion s nd ome?
E) Phenelzine (Nardil) • r ecognize impo an an idep essan in e ac ions?
• Desc ibe adve se e ec s o va ious an idep essan s?
Answer 23.9.7 The correct answer is “A.” Ne azodone is a
5H -2 A rece tor antagonist and is an e ective antide ressant
with sedative ro erties. It has signi cant inhibitory e ects QUICK QUIZ: MAOIs
on CYP3A4 and has several signi cant drug interactions as a
result. In the recent ast, several cases o he atic ailure have
You are seeing a new atient with a history o recurrent ajor
been re orted with this drug, leading to a black box warning on
de ression. Many years ago a di erent hysician ut hi on
the ackage insert and li iting its clinical use. None o the other
henelzine (Nardil), which you recognize as having otentially
antide ressants listed have had such roble s.
serious ood and drug interactions. You are considering switch-
ing hi to an SSRI.
Question 23.9.8 Venla axine has been noted to typically
cause all o the ollowing side ef ects EXCEPT:
How long a er an MAOI is discontinued can an SSRI be started?
A) Increased blood ressure
A) 1 day
B) Dizziness
B) 3 days
C) Dry outh
C) 7 days
D) Weight gain
D) 14 days
E) Sexual disturbance
E) SSRIs and MAOIs can be given together
Answer 23.9.8 The correct answer is “D.” Venla axine and
The correct answer is “D.” Because o signi cant drug–drug in-
duloxetine (Cy balta) are both nore ine hrine and serotonin
teractions (see next question), SSRIs should not be started until
reu take inhibitors and have ini al activity on the cholin-
2 weeks a er discontinuation o an MAOI.
ergic, hista inergic, and α -rece tors. T ere is a dose-related
increase in blood ressure, with a ean elevation o 10 to 15
Hg in diastolic ressure at doses o 300 g or greater in u to
10% o atients. T ere is a slight increase in ulse, as well. “D” is QUICK QUIZ: MAOI + SSr I = BADNESS
not true. In act, weight loss is a co on co laint. Otherwise,
venla axine and duloxetine generally have the sa e side-e ect What is the drug–drug interaction o concern with SSRIs and
ro le as the SSRIs, otentially causing sexual dys unction, diz- MAOIs?
ziness, and dry outh. Patients can have a signi cant withdrawal A) Stevens–Johnson syndro e
ro venla axine and should be ta ered o o it slowly. B) Serotonin syndro e
C) Anticholinergic crisis
D) Hy ertensive crisis
HELPFUL TIP: ANTIDEPRESSANTS AND E) Existential crisis
BIRTH DEFECTS
Fluoxe ine and pa oxe ine a e associated wi bi The correct answer is “B.” Serotonin syndro e is caused by an
de ec s. t is doesn’ p ove causali and e c ance is excess o serotonin and can be caused by drug–drug interac-
slig (e.g., anencep al isk goes om 2 in 10,000 o tions involving serotonergic agents including SSRIs, bus irone,
7 in 10,000). h oweve , i is wo discussing wi e e eridine, dextro ethor han, lithium, tra adol, and tri -
p ospec ive pa en s. t e o e SSr Is do no seem o tans, a ong others. MAOIs inhibit ona ine oxidase, which is
ave e same associa ion (BMJ. 2015;351: 3190). We an enzy e involved in the etabolis o any neurotrans it-
s ill don’ like pa oxe ine. ters, including serotonin. T ere ore, MAOIs increase serotonin
as well. Serotonin syndro e is characterized by uscle rigidity,
hy erre exia, hy erther ia, con usion, and agitation a ong
HELPFUL (AND SCARY) TIP: other sy to s. It can be atal. As a result, the concurrent use
Venla axine (E exo ) and duloxe ine (C mbal a) a e o an SSRI and MAOI is absolutely CON RAINDICA ED.
oxic in ove dose. t e cause Qt and Qr S p olonga ion Stevens–Johnson syndro e (“A”) is an autoi une der ato-
in ove dose. t ea men is e same as e ea men o logical disorder that leads to desqua ation o ucosal sur aces
t CA ove dose (bica bona e, e c.), al oug da a is lim- and is not associated with antide ressant use (although it can
i ed. r emembe a ci alop am can cause a p olonged be seen with other drugs). Anticholinergic crisis (“C”) is not
Qt in e val wi bo egula doses and in ove dose. a ter used in co on edical arlance but does evoke the
idea o anticholinergic syndro e, caused by excess anticholin-
ergic drugs, such as CAs, etc. Hy ertensive crisis (“D”) occurs
You decide to switch the patient to f uoxetine and she does when oods containing tyra ine (e.g., aged cheese and cured
well. eats) interact with MAOIs to release catechola ines, causing
CHAPTER 23 • PSyCh IAt r y 641

hy ertension, headaches, nausea, and dia horesis. In severe Answer 23.10.2 The correct answer is “A.” Valerian root
cases, it can lead to strokes or death. (Valeriana o cinalis) has been touted to have anxiolytic ro er-
ties, si ilar to benzodiaze ines, and its echanis o action is
thought to be si ilar (e.g., inhibition o GABA). It a ears to be
HELPFUL TIP(S): sa e and has the sa e drug interactions and contraindications
Linezolid is absolu el con aindica ed wi MAOIs. I as benzodiaze ines. T e other o tions are not known to a ect
is a se o one gic d ug. Among e oods a cause a slee to a signi cant degree.
pe ensive c isis wi an MAOI a e banana peels (bu
no e banana . . . we a e no su e w o igu ed is one
ou . . .), aged mea s (bes avoided an wa i ou ink HELPFUL TIP:
abou i ), d aug bee (boo) bu no bo led bee ( es!), Kava-kava is adve ised as an anxiol ic bu s ould gen-
and kimc i/saue k au . e all be avoided. Kava-kava (Piper methysticum) as
been epo ed o cause live damage, in some cases
leading o live ansplan o even ual dea .

CASE 23.10
A 43-year-old man who you started on citalopram or depres- Objectives: Did you learn to . . .
sion 4 weeks ago returns or a ollow-up visit. He eels better • Iden i w ic an idep essan s a e less likel o cause sexual
but complains o delayed ejaculation. You consider changing d s unc ion?
him to an antidepressant that is less likely to cause sexual • r ecognize e bal e apies a mig be emplo ed in
dys unction. ea ing s mp oms o dep ession?

Question 23.10.1 Which o the ollowing would you AVOID?


A) Ne azodone
CASE 23.11
B) Bu ro ion You are seeing a 28-year-old emale whose rst child you
C) Mirtaza ine delivered a month ago. She comes to your clinic with her
D) razodone son or his 1-month well-child check. You observe that
E) Nortri tyline she seems tired and is less animated than usual. She is
gentle with her in ant, but her ace doesn’t seem to light
Answer 23.10.1 The correct answer is “E.” Sexual dys unction up with the glow that you o en see with new mothers. You
is a co on side e ect o ost sychotro ics. T ere are ew know . . . the glow o terror, sleeplessness, and anxiety?
controlled data to guide us as to how to a roach this issue, T at’s the one.
but sildena l (Viagra) and si ilar drugs are e ective in anti-
de ressant-induced erectile dys unction. O the o tions given, Question 23.11.1 Which o the ollowing best explains your
nortri tyline—and other CAs—is ore requently associated observations?
with sexual dys unction. A) Slee de rivation
B) Marital discord at ho e
C) Post artu de ression
HELPFUL TIP:
D) T yroid dys unction
S . Jo n’s wo ma be e ec ive in mild- o-mode a e de-
E) Any o the above
p ession. h oweve i is a known induce o e CyP450
enz me, educing e e icac o o al con acep ives.
Answer 23.11.1 The correct answer is “E.” Your atient’s sy -
I as also been associa ed wi dec eased e icac o
to s could be due to any o these roble s and ore, includ-
an i e ovi als, c clospo ine, digoxin, eop lline, and
ing dif culty with role adjust ent, alcohol or drug abuse, and
wa a in.
ane ia.

You want to gather more in ormation to see i there is a path-


T is patient is also having sleep di culties and asks what ological process underlying her behavior.
herbal therapy he might be able to use.
Question 23.11.2 You would do all o the ollowing EXCEPT:
Question 23.10.2 Which o the ollowing is an herbal alter- A) Ask her to ll out an Edinburgh Postnatal De ression Scale
native to benzodiazepines or anxiety and insomnia? (EPDS)
A) Valerian root B) Order thyroid unction tests
B) St. John’s wort C) Order a slee study
C) Saw al etto D) Ask ore questions about how things are at ho e and how
D) Ginseng she is co ing
E) Ginkgo E) Ask about a revious history o de ression
642 FAMILy MEDICINE EXAMINAt ION & BOAr D r EVIEW

Answer 23.11.2 The correct answer is “C.” In this case, there


are any other ore likely roble s than a ri ary slee dis- I you had been aware o her depression earlier and wanted to
order, and a slee study is unlikely to be hel ul ( lus as those prescribe an antidepressant during her pregnancy, you would
o us who have had in ants know, your slee is naturally disor- have been cautious, hoping to prescribe a Pregnancy Sa ety
dered). T e EPDS is a validated sel -rated scale that is use ul or Category B drug.
detecting ost artu de ression. Hy othyroidis can always
Question 23.11.5 Which o the ollowing antidepressants is
i ic de ression and ight need to be ruled out. A thorough
Category B in pregnancy?
history is always essential.
A) Fluoxetine
B) Bu ro ion
Question 23.11.3 The incidence o postpartum depression
C) Nortri tyline
is about:
D) Mirtaza ine
A) 1%
E) None o the above
B) 10%
C) 30% Answer 23.11.5 The correct answer is “E.” T ere is no antide-
D) 50% ressant that is category B or regnancy. All the drugs listed
are category C. T is is not to say that antide ressants ust
Answer 23.11.3 The correct answer is “B.” About 7 in 10 be avoided in regnancy, but the bene ts and risks should be
wo en su er ro “baby blues,” ost artu sy to s that weighed care ully.
can ani est as ood swings, anxiety, atigue, and sadness
occurring within a ew days o delivery and lasting only a Question 23.11.6 Risks associated with SSRI use during
week or so. However, ost artu de ression a ects a roxi- pregnancy include all o the ollowing EXCEPT:
ately 10% o wo en, and sy to s can a ear anywhere A) Irritability o the neonate
ro weeks to onths a er birth. T e diagnosis is o en B) Preter delivery
issed because any others are asha ed to ad it eeling C) Low birth weight
unha y at a ti e when they think (and society tells the ) D) Persistent ul onary hy ertension
that they should be ha y. Physicians ay ocus on the E) etralogy o Fallot
in ant’s hysical health and iss assessing the other–baby
interaction. Answer 23.11.6 The correct answer is “E.” SSRIs have not
been linked to develo ent o etralogy o Fallot. However, a
In obtaining more history, you realize that this patient nu ber o adverse e ects have been associated with SSRI use in
was having some troubles with depression even during her regnancy, including neonatal irritability, low birth weight, and
pregnancy. reter labor. In ants ex osed to SSRIs during the second hal
o regnancy (i.e., a er 20 weeks gestation) had an increased risk
Question 23.11.4 When is the most common time or (a roxi ately 1% absolute risk) o develo ing persistent pul-
pregnancy-related depression to occur? monary hypertension, not a birth de ect but still a otentially
A) At conce tion li e-threatening condition, a er birth (BMJ. 2012;344:d8012).
B) First tri ester
C) Second tri ester Question 23.11.7 I you had decided to prescribe an antide -
D) T ird tri ester pressant medication or her during pregnancy, which one
E) Post artu o the ollowing would have been LEAST desirable?
A) Paroxetine
Answer 23.11.4 The correct answer is “D.” A large e ide i- B) Sertraline
ological study ro the United Kingdo ollowed regnant C) Nortri tyline
wo en ros ectively throughout the course o their regnan- D) Citalo ra
cies and ound that the incidence o depression was actually
higher in the third trimester than it was in the postpartum Answer 23.11.7 The correct answer is “A.” Although no anti-
period. T is suggests the need or the hysician to begin to de ressant edication has been shown to be risk- ree when
enquire about sy to s earlier. used during regnancy, aroxetine is the only antide ressant
edication listed that is Category D in regnancy, due to the
increased incidence o cardiac ano alies ( ainly atrial and
HELPFUL TIP: ventricular se tal de ects) in in ants who were ex osed in utero.
r epea a e me, “P egnanc does no ea dep ession.” As you undoubtedly re e ber ro above, uoxetine is also
Fo some eason, i ad been assumed in e pas a generally to be avoided because o the risk o birth de ects. Do
a woman’s dep ession would aba e du ing p egnanc . you still think we don’t like aroxetine?
S udies in e las couple o ea s ave disp oved is
m making ongoing ea men o dep ession c ucial. You start her on f uoxetine and o er to watch her baby or a
ew days, and everyone is happy!
CHAPTER 23 • PSyCh IAt r y 643

Objectives: Did you learn to . . . TABLE 23-2 COMMON MANIFESTATIONS OF


• r ecognize e ig incidence o dep ession and dep essive DEPRESSION IN CHILDREN
s mp oms in e pos pa um pe iod?
• Inc eased i i abili , ange , o os ili
• Diagnose pos pa um dep ession?
• Being bo ed
• t ea dep ession in e p egnan and pos pa um pa ien ? • r eckless be avio
• Ou bu s s o s ou ing, complaining, unexplained i i abili , o
CASE 23.12 •
c ing
Poo sc ool pe o mance
A couple, you have known or some time, brings in their • Fea o dea
7-year-old son, Jimbo, to your clinic because his behavior has • Alco ol o subs ance abuse
• F equen nonspeci c p sical complain s suc as eadac es,
changed over the past month. His school per ormance has muscle ac es, s omac ac es, o a igue
worsened, and he has started to get into ghts at school. He
is not eating as well and is having requent nightmares. He Dep ession in C ild en and Adolescen s, Na ional Ins i u es o
now has requent headaches and stomachaches and clings to h eal Publica ion No. 00-4744. p://www.nim .ni .gov/publica /
his mother when it is time to go to school. T e parents can- depc ild es ac .c m.
not understand what is going on and report no antecedent
trauma. Answer 23.12.2 The correct answer is “D.” Although clini-
cians should be aware o age-a ro riate ani estations (see
Question 23.12.1 Which o the ollowing is the most likely able 23-2), sy to s o de ression are si ilar in children and
diagnosis? adults. In act, ty ical sy to s o de ression are ore co -
A) MDD on in children than are “ asked” sy to s such as sto ach-
B) Disru tive ood dysregulation disorder aches and ear o leaving ho e.
C) Adjust ent disorder
D) Bereave ent You tell the amily about Jimbo’s prognosis.
E) Nor al childhood dif culties ex ected with being na ed
Ji bo Question 23.12.3 Which o the ollowing statements regard-
ing prognosis is FALSE?
Answer 23.12.1 The correct answer is “A.” U to 3% o chil- A) Childhood MDD con ers a two- to our old increase in risk
dren and 9% o adolescents su er ro de ression. DSM-5 or adult MDD
criteria are the sa e as in the adult, exce t irritability can be B) About 25% o adolescents with MDD develo substance
substituted or the depressed mood requirement in children. abuse disorders
Disru tive ood dysregulation disorder (“B”) is a new addi- C) Al ost hal o children with MDD will atte t suicide
tion to DSM-5. It is characterized by severe, ersistent irritabil- so eti e in their li e
ity. T ere ust be requent te er outbursts that occur over D) Roughly 20% o adolescents have su ered at least one e i-
the course o at least a year and ust be in 2 di erent settings. sode o MDD by 18, while 65% re ort transient sy to s
Adjust ent disorder (“C”) is not likely since there have been no o de ression
ajor changes in the child’s regi en, and bereave ent (“D”) is E) A er the initial e isode, only 10% will su er a rela se
not likely since there have been no losses in the child’s li e. T is
atient’s sy to s are clearly not art o nor al childhood, Answer 23.12.3 The correct answer is “E.” Between hal and
even i his na e is Ji bo. two-thirds (not 10%) will have a recurrence within 5 years a er
resolution o their rst e isode. T e other o tions are all true. In
HELPFUL TIP: children and adolescents, the ean de ression e isode length
O cou se bull ing a sc ool and o e social p oblems is 7 to 9 onths with re ission ty ically occurring over 1.5 to
need o be inves iga ed w en an c ild p esen s wi 2 years.
s mp oms o dep ession.
HELPFUL TIP:
t e isk o suicide is ve ig among dep essed ou s.
Question 23.12.2 Which o the ollowing is NOT true about
I is e i d leading cause o dea in e 15- o 24- ea
depression in children?
age g oup, and c ild en wi MDD ave a ou - o ive-
A) Abuse or neglect increases the risk o de ression
old ig e li e ime incidence o suicide a emp s an
B) Having a de ressed arent increases the risk o being a
nondep essed c ild en. t wen pe cen o adolescen s
de ressed child
ave suicide idea ion eac ea , and 5% o 8% will a -
C) T e clinical course is roughly the sa e as in adults
emp suicide eac ea . P e dep essing s a is ics, e ?
D) “Masked” sy to s, such as abdo inal ain, are ore
co on in children than the ty ical sy to s o de res-
sion, such as slee disturbance
E) Male and e ale children are equally a ected by de ression You recommend treatment or Jimbo.
644 FAMILy MEDICINE EXAMINAt ION & BOAr D r EVIEW

Question 23.12.4 Which o the ollowing therapies has NOT Answer 23.13.1 The correct answer is “A.” T e sy to s o
shown e icacy in childhood dep ression in randomized ADHD are listed below. “B,” adjust ent disorder, is unlikely
controlled trials? since there is no history o a signi cant li e event. ODD and
A) Fluoxetine CD are characterized by aggressive behavior and a disregard
B) Sertraline or rules and adults. T ese are not given as art o o y’s
C) CB history. S eci c learning disorder resents with school er or-
D) Venla axine ance roble s and ay be associated with ADHD but would
E) Inter ersonal thera y not be directly res onsible or this atient’s hy eractivity.
For the diagnosis o ADHD, atients ust eet one o the
Answer 23.12.4 The correct answer is “D.” In act, the akers ollowing criteria:
o venla axine sent out a “Dear Doctor” letter suggesting that 1. At least six sy to s o inattention or at least 6 onths that
venla axine not be used in children under 18 because it lacks is alada tive and inconsistent with level o develo ent:
ef cacy data, and it has an increased incidence o e otional • Careless istakes, oor attention to details
lability. T e botto line is that uoxetine is robably the SSRI • Cannot sustain attention
o choice in children, ollowed by sertraline. However, the NN • Does not see to listen
or SSRIs is 10 to bene t one child. Avoid aroxetine since it • Poor ollow through on tasks
ay be associated with a higher suicide risk (there is a attern • Dif culties with organization
develo ing here…). CAs should be avoided because o lack o • Avoids or dislikes tasks that require sustained attention
ef cacy and otential suicide risk. CB and inter ersonal ther- • O en loses things required or a task (notebooks, ens, etc.)
a y have been shown to be e ective in children and adolescents. • Easily distracted ro a task
• Forget ul in daily activities
HELPFUL TIP: 2. Six or ore o the ollowing hy eractive–i ulsive sy to s
As ou would expec , CBt /counseling/ e ap plus an or 6 onths, which is alada tive and inconsistent with level
an idep essan a e mo e e ec ive an ei e modal- o develo ent:
i alone. Adve se e ec s o medica ion in c ild en a e Hy eractivity sy to s
simila o ose in adul s and include insomnia, a igue,
• Fidgets or squir s in seat
eadac es, and ne vousness. Keep a close e e on ado-
• Leaves seat at ina ro riate ti es
lescen s w o ou s a on an idep essan s, and wa n
• Hy eractivity in ina ro riate settings
pa ien s and ei pa en s a dep ession can wo sen
• Cannot lay or relax quietly
in e ea l p ase o ea men . h ave em e u n o en
• Always in otion
in e i s ew mon s o ea men .
• alks too uch
I ulsivity sy to s
Objectives: Did you learn to . . . • Blurts out answer be ore questions co leted
• Inc ease ou awa eness o c ild ood dep ession? • rouble waiting or turn in ga es, school, etc.
• Diagnose dep ession in c ild en? • Interru ts others (verbally, in ga es, etc.)
• Desc ibe e na u al is o o dep ession in c ild en? In addition to ul lling criteria 1 and 2 above, the ollowing
• Gene a e an app op ia e ea men plan o c ild en wi are required:
dep ession? • Sy to s ust be resent be ore age o 12 years!
• I air ent in two settings (ho e, work, school, worshi , etc.)
CASE 23.13 • Clinically signi cant i air ent in social, acade ic, or
ommy is a 9-year-old male who has been having di culty occu ational s heres
with his behavior since he started rst grade. He is o en dg- • Sy to s are not due to another roble (develo ental
ety and squirming in his chair and has di culty remaining in delay, ersonality disorder, ood disorder, etc.)
his seat. He talks out o turn, is o en “on the go,” and is not Question 23.13.2 Which o the ollowing is FALSE about
liked by the other kids because he intrudes into games and ADHD?
has a hard time waiting his turn. His mom also notes simi- A) It a ects 3% to 7% o children
lar problems at home. He is the product o an uncomplicated B) T ere are genetic and environ ental in uences on the risk
pregnancy and has no signi cant past medical history. o develo ing ADHD
C) Males are ore likely to have ADHD than e ales
Question 23.13.1 What is the most likely diagnosis? D) T e incidence o ADHD has increased over the years
A) Attention-de cit/hy eractivity disorder (ADHD) E) Co orbid disorders are not co on
B) Adjust ent disorder
C) O ositional de ant disorder (ODD) Answer 23.13.2 The correct answer is “E.” ADHD a ects 3%
D) Conduct disorder (CD) to 7% o children. Heritability is 70%, si ilar to that or schizo-
E) S eci c learning disorder hrenia and bi olar disorder. Alcohol and tobacco ex osure
CHAPTER 23 • PSyCh IAt r y 645

in utero have both been linked to at least a two old increase in TABLE 23-3 STIMULANT EFFECTS ON
risk. Males are twice as likely to be diagnosed with ADHD as PATIENTS WITH ADHD
e ales. Co orbid disorders are very co on in ADHD with
Ef ects o Stimulants on Motor Response
CD, ODD, de ression, anxiety, learning disabilities, and devel-
• r educe ac ivi o no mal
o ental delay being the ost requent. • Dec ease excessive alking, noise, and dis up ion in e class oom
• Imp ove andw i ing
Question 23.13.3 Which o the ollowing is NOT TRUE o the • Imp ove ne mo o con ol
prognosis and treatment o Tommy’s ADHD?
Ef ects o Stimulants on Social Skills
A) T e natural history o ADHD is that one-third o children
• r educe o - ask be avio in class oom
will outgrow the sy to s, one-third will have the sa e • Imp ove abili o pla and wo k independen l
requency and intensity o sy to s, and one-third will • Dec ease in ensi o be avio
have residual sy to s, which are subclinical • r educe bossiness wi pee s
B) o y has a 70% to 80% chance o res onding to sti ulants • r educe ve bal and p sical agg ession
• Imp ove (bu no no malize) pee social s a us
C) I o y is treated with sti ulants, his risk o drug abuse is
• r educe noncompliance, de ance, and opposi ional be avio wi
halved adul s
D) o y is at increased risk o getting into accidents • Pa en s and eac e s become less con olling and mo e posi ive
E) Behavior thera y is e ective or reducing ADHD sy to s
Ef ects o Stimulants on Cognitive Ability
• Imp ove abili o sus ain a en ion, especiall in bo ing asks (like
Answer 23.13.3 The correct answer is “E.” Un ortunately,
s ud ing o e FM Boa d Exam)
intensive behavior thera y has been shown to be ine ective. T e • r educe dis ac ibili
breakdown o the rognosis or children with ADHD sy to s • Imp ove s o - e m memo
is that roughly one-third will ex erience co lete sy to reso- • r educe impulsivi
lution, one-third will get so e i rove ent, and one-third will • Inc ease amoun o academic wo k comple ed
• Inc ease accu ac o academic wo k
re ain ill with the disorder. Sti ulants, which are rst-line ther-
a y, will work in 70% to 80% o the atients. I a atient does not Side Ef ects
res ond to the rst sti ulant, she still has a 70% to 80% chance o • Lack o appe i e
res onding to a second sti ulant. Children treated with sti u- • Dec eased g ow , especiall ini iall
• Insomnia
lants are hal as likely to abuse substances as those who are not
• h eadac es and s omac ac es
treated. Children with ADHD are at risk or i ulsive behavior • I i abili
and risk-taking, which leads to substance abuse, accidents, etc. • t ac ca dia o blood p essu e inc ease ( a e)
• Muscle ics o swi c es ( a e)
You decide to start ommy on methylphenidate. • Ps c osis o deli ium ( a e)

Question 23.13.4 Which o the ollowing is NOT true about the examinations he has to take, but he thinks that he is smart
treatment with methylphenidate? enough. He says that he just cannot concentrate.
A) It i roves handwriting
B) O ti al dosing is 0.6 to 1 g/kg/day Question 23.13.5 Which o the statements below is NOT
C) It can cause reduced growth true or an adult presenting with ADHD?
D) Short-ter e ory is not a ected A) Adults resent with the sa e core sy to s as children but
E) o y ight get along better with his class ates in a di erent ashion
B) Adults are less likely to have overt hy eractivity sy to s
Answer 23.13.4 The correct answer is “D.” In act, sti ulants co ared with children
i rove short-ter e ory in atients with ADHD. T e rest C) Adults o en resent when their children are diagnosed
are true. As to “A,” do you think this is why doctors have notori- D) Adults are less likely to s oke than sa e-age ersons with-
ously bad handwriting? Maybe we all need sti ulants …. Sti - out ADHD
ulants have a wides read e ect on ulti le do ains, so e o E) Adults o en seek ro essions that allow the to use their
which are listed in able 23-3. sy to s to their advantage

ommy’s ather is happy with his son’s response to methyl- Answer 23.13.5 The correct answer is “D.” Adults with atten-
phenidate and wonders i he too would bene t rom a simi- tion de cit disorder are ore likely to s oke than their sa e-
lar medication. He recalls being in trouble ever since grade aged eers ( erha s it’s because nicotine is a sti ulant?). Adults
school or talking “out o turn” and always being put in deten- resent di erently than children, but have the sa e core sy -
tion. He was always restless and dgety but this has improved to s o hy eractivity, inattention, and i ulsivity. Many adults
as he ages. He has a hard time at work sitting through meet- are diagnosed with ADHD only when their children have been
ings, as he tends to daydream, and he has numerous ghts diagnosed or when increasing dif culty at work or at ho e leads
with his wi e because she accuses him o not listening to her. the to seek hel . O en, co orbidities drive the to seek hel ,
He has been unable to get promoted because he cannot pass and the ri ary diagnosis o ADHD is ade only incidentally.
646 FAMILy MEDICINE EXAMINAt ION & BOAr D r EVIEW

TABLE 23-4 PRESENTATIONS OF ADHD SYMPTOMS IN CHILDREN AND ADULTS


Children Adults
h pe ac ive c ild: squi ms, canno s a in h pe ac ive adul : as subjec ive inne es lessness and ouble elaxing
is sea , and cons an l on e go Impulsive adul : speeding icke s, ca c as es, impa ien , smokes mo e, ig e divo ce a e, ig e
Impulsive c ild: blu s ou answe s, subs ance use a e, and ove ea ing
in e up s o e s, and alks incessan l Ina en ive adul : o en la e o appoin men s, o ge s annive sa ies, as dif cul wi wo k mee ings;
Ina en ive c ild: does no ollow oug , is p oblems wi ocusing, planning, o ganizing, and comple ing asks a ome and a wo k; advances
o ge ul, does no lis en mo e slowl a wo k an pee s; misplaces ke s, glasses, and o e i ems; ma o ge o pa bills, pick
Fewe en e college and g adua e, compa ed up e kids on ime, e c.
wi c ild en ei age wi ou ADh D

Adults ty ically resent seeking hel or their inattention and


HELPFUL TIP:
concentration dif culties, as overt hy eractivity lessens with
S imulan s do no cause ca diac p oblems and seem
age. So e adults co ensate by ursuing in careers that reward
o be sa e in bo c ild en and adul s (JAMA. 2011;306:
their intellectual curiosity, endless energy, and desire or change
2673; JAMA. 2011;306:2723; N Engl J Med. 2011;365:
(e.g., like a ily edicine and e ergency edicine?). able 23-4
1896).
co ares adult and child resentations o ADHD.

HELPFUL TIP:
r emembe a adul s ma ave esidual s mp oms, HELPFUL TIP:
w ic do no mee ull c i e ia o ADh D a e ime o A omoxe ine (S a e a) is a nons imulan d ug ap-
evalua ion. A clear history o symptoms starting in p oved o ADh D (i is a selec ive no epinep ine eup-
childhood be ore age 12 must be present to diag- ake in ibi o ) and e onl d ug app oved o adul
nose an adult with ADHD. There is no such thing as ADh D. I is no a con olled p esc ip ion, aving no ap-
“Adult-Onset ADHD” (but there is adult stimulant pa en abuse po en ial. I s place in e ap is no e
abuse). Con i ma ion o e is o can be ob ained well de ined bu mig be used in ose wi a d ug
om colla e al sou ces, including old sc ool epo s abuse is o , e c. Maximum e icac o a omoxe ine is
and amil membe s. t ink abou o e diagnoses suc ac ieved in a ew weeks, and i as a simila side-e ec
as anxie diso de and dep ession in pa ien s w o p o ile o e s imulan s.
ink e ave “adul -onse ADh D.”

Objectives: Did you learn to . . .


You decide to treat ommy’s ather with a medication. He • r ecognize c ild ood and adul p esen a ion o ADh D?
would pre er not to have a stimulant, and he asks i there are • P esc ibe ef cacious ea men s o ADh D?
other options or treatment. • r ecognize side e ec s and advan ages o va ious medica-
ions o ADh D?
Question 23.13.6 Which o the ollowing medications
would you recommend?
A) Fluoxetine CASE 23.14
B) Bu ro ion
C) Mirtaza ine A gentleman calls your o ce because he is concerned that
D) Phenelzine his wi e o 2 years is acting strangely. She has not slept or
E) Ris eridone most o the past week, staying up at night cleaning the house
and sending multiple texts to riends and amily. She even
Answer 23.13.6 The correct answer is “B.” In addition to sti - went out and spent $3000 on a dress that le nothing to
ulants, which are also rst-line agents in adults, there are a vari- the imagination and was seen kissing another man (Santa
ety o edications that can be used to treat ADHD, although Claus, you hope). You ask him to bring his wi e in as soon
ost studies are undertaken in children and ost o these ed- as possible.
ications are not FDA-a roved or treating ADHD. Second-line T at a ernoon you nd a provocatively dressed 30-year-
agents include the antide ressants bu ro ion, desi ra ine, old emale sitting in your o ce and laughing giddily as her
i i ra ine, and nortri tyline. T e α -blockers guan acine and husband gives most o the intelligible history. She keeps
clonidine are also used, ostly as an adjunct in children with reaching over to touch you on the leg as you interview
conco itant conduct disorder or slee roble s. Re e ber her. You nd her hard to understand because she talks so
that a history o substance abuse or sychotic disorder is al ost ast. You manage to catch something about “running or
always a contraindication to sti ulant use. President.”
CHAPTER 23 • PSyCh IAt r y 647

Question 23.14.1 What is the most likely diagnosis? has its onset in early adulthood, although it can begin in child-
A) Mania hood or adolescence. De ression is resent 20% to 30% o the
B) Psychosis ti e, even with ongoing aintenance treat ent. Over hal o
C) Agitated de ression bi olar atients are initially isdiagnosed with de ression, and
D) Anxiety disorder the average atient is only accurately diagnosed a er 5 years o
E) ADHD sy to s.

Answer 23.14.1 The correct answer is “A.” Mania is the cor-


HELPFUL TIP:
rect diagnosis. None o the other conditions can ully ex lain
Up o 50% o people wi bipola diso de ave con-
the abnor al elevation in ood and the subsequent behav-
comi an alco ol abuse o dependence.
ior changes. Patients with bipolar I disorder ust have had
at least one e isode o ania: a distinct eriod o abnor ally
and ersistently elevated, ex ansive, or irritable ood, lasting
at least 1 week (or any duration i hos italization is necessary). You want to start a medication or this patient . . . or commit
In addition, there ust be at least three o the ollowing sy - her . . . or both.
to s concurrently: (1) in ated sel -estee or grandiosity, (2)
decreased need or slee , (3) ore talkative than usual, (4) ight Question 23.14.3 Which o the ollowing would NOT be an
o ideas or racing thoughts, (5) distractibility, (6) increased appropriate treatment choice or her mania?
goal-directed activity or sycho otor agitation, and (7) exces- A) Lithiu
sive involve ent in leasurable activities that have a high risk B) Olanza ine
or negative consequences/i ulsivity (ga bling, s ending C) Dival roex
s ree, risky sexual indiscretions, etc.). T e e isode ust cause D) Carba aze ine
i air ent in occu ational or social unctioning and cannot E) Bus irone
be substance induced.
Patients with bipolar II disorder have had at least one e i- Answer 23.14.3 The correct answer, and the medication to
sode o hy o ania: a distinct eriod o ersistently elevated, avoid, is “E.” Bus irone is not e ective in the treat ent o bi o-
ex ansive, or irritable ood, lasting or at least 4 days, which lar disorder. Lithiu (“A”) was the rst edication a roved
is clearly distinct ro the usual, nonde ressed ood. During or treat ent o bi olar ania and de ression. It reduces the
the hy o anic e isode, at least three o the anic sy to s incidence o recurrence o ania, hy o ania, and de ression
listed above ust be resent, although the e isode is not severe by about two-thirds. Lithiu has a signi cant antisuicide e ect
enough to cause arked i air ent in occu ational or social with an esti ated eight- to nine old reduction in risk. It is dosed
unctioning, require hos italization, or include sychotic sy - at nightti e or twice daily. Lithiu has a narrow thera eutic
to s. window, and there are nu erous drug–drug interactions.
Both olanza ine (“B”) and dival roex (“C”) have FDA
HELPFUL TIP: a roval or treat ent o acute ania and a ear to be so e-
Al oug pa ien s wi bipola diso de s o en ave what e ective in the revention o recurrent e isodes. However,
dep essive episodes as well as manic o pomanic only lithiu and la otrigine (La ictal) are FDA-a roved or
episodes, dep ession is no equi ed o e diagnosis o bi olar treat ent and aintenance. Carba aze ine (“D”) is
bipola diso de . (“So w is i called bipola ,” ou ask? a second-line agent that is also e ective, but side e ects li it
Well, w o a e ou o ques ion adi ion?) its use.

You decide to start lithium.


Question 23.14.2 Regarding the epidemiology o bipolar
illness, which o the ollowing is FALSE? Question 23.14.4 Which o the ollowing is a well-recog-
A) T e revalence o bi olar 1 is about 0.6% to 1.5% nized side ef ect o lithium?
B) Many atients are isdiagnosed initially with de ression A) Diabetes ellitus
C) Untreated, the suicide rate is al ost 20% B) Hy othyroidis
D) Wo en are twice as likely to be a ected as en C) I unosu ression
E) Suicide risk is highest in the de ressed or ixed state D) Abnor al hair growth
E) T e ability to hold an electric charge
Answer 23.14.2 The correct answer is “D.” Unlike de ression,
bi olar illness a ects ales and e ales equally. Bi olar I a ects Answer 23.14.4 The correct answer is “B.” Patients who take
about 0.6% to 1.5% o the o ulation, while bi olar II a ects lithiu should have their thyroid unction onitored. Also,
about 0.8% to 3%. Untreated, nearly 20% will co it suicide— lithiu can a ect renal unction and electrolyte levels, so check
a rate about 20 ti es that o the general o ulation. Risk is seru electrolytes eriodically. As lithiu has a narrow thera-
highest in de ressed states or the ixed states (both ania and eutic window, seru lithiu levels should be easured as
de ression resent at the sa e ti e). Bi olar disorder ty ically well, with a goal o 0.6 to 1.0 Eq/L.
648 FAMILy MEDICINE EXAMINAt ION & BOAr D r EVIEW

Question 23.14.5 Which o the ollowing drugs or drug Question 23.15.2 Which o these additional ndings would
classes do NOT alter lithium levels? you expect on physical examination?
A) NSAIDs A) Bradycardia
B) Diuretics B) Hy ertension
C) ACE inhibitors C) Adnexal ass
D) ARBs D) Clonus
E) Narcotics E) Pro tosis

Answer 23.14.5 The correct answer is “E.” Lithiu is cleared Answer 23.15.2 The correct answer is “A.” Co on hysical
by the kidney. Anything that can cause a change in renal unc- ndings in weight loss, and s eci cally AN, include e aciation,
tion can a ect lithiu levels. NSAIDs, diuretics, ACE inhibi- sunken cheeks, hy otension, bradycardia, lanugo, ottled teeth,
tors, and ARBs can all a ect renal unction. and dry or yellow skin. Peri heral ede a ay develo during
weight gain or when laxative or diuretic abuse is sto ed. Mur-
Objectives: Did you learn to . . . urs can occasionally be auscultated.
• r ecognize and diagnose bipola diso de ?
• Ini ia e ea men o bipola diso de ? Question 23.15.3 Patients with AN may present with:
• Desc ibe some po en ial adve se e ec s o ea men o A) Paresthesias
bipola diso de ? B) Cold intolerance
C) Consti ation
CASE 23.15 D) Fatigue
E) All o the above
A 20-year-old emale gymnast presents to you because she
has missed her period or 6 months. She eels cold all the time Answer 23.15.3 The correct answer is “E.” All o the above are
and has noticed that when she crosses her legs, she gets pins- sy to s o AN. T ese are co on sy to s seen in starva-
and-needles sensations down her leg. She had a stress rac- tion. “A” is not intuitive but is true. Loss o at allows or greater
ture o her right tibia 5 years ago but denies any other medi- ex osure o su er cial nerves, so the act o crossing the legs or
cal history. She denies being sexually active and the review o sitting down on a hard chair can cause aresthesias.
systems is positive or requent heartburn, constipation, and
atigue. HELPFUL TIP:
Pa ien s wi AN a el ave insig in o e illness and
Question 23.15.1 Which o the ollowing diagnoses would o en den a weig loss is a p oblem. t ese pa ien s
be in your dif erential? a e o en pe ec ionis s and ove ac ieve s w o a e
A) Pregnancy sensi ive o c i icism and come om amilies wi con-
B) Hy erthyroidis lic . Weig loss is a me od o con ol and is seen as a
C) Malignancy signi ican ac ievemen .
D) Anorexia nervosa (AN)
E) All o the above
HELPFUL TIP:
Answer 23.15.1 The correct answer is “E.” All the answers t e wo d “ano exia” in AN is a misnome , as loss o ap-
should be art o a reasonably broad di erential diagnosis in pe i e is exceedingl a e in is illness. Pa ien s a e un-
this young wo an. In atients resenting with weight loss g bu volun a il es ic ei calo ic in ake. t e ini ial
and a ossible eating disorder, you should eli inate edi- weig loss ma be p ecipi a ed b appe i e loss caused
cal causes o weight loss, using history, hysical exa ination, b dep ession, medical illness, die ing, o s ess ul li e
and a ro riate labs. Des ite her denial o sexual activity, even .
a regnancy test is a necessary art o the evaluation, since
regnancy is the ost co on cause o a enorrhea in this Question 23.15.4 Which would be an expected laboratory
o ulation. Hy erthyroidis and alignancy can resent nding in this patient?
with vague co laints si ilar to this atient’s. Likewise, A) Leukocytosis
AN can give rise to this atient’s constellation o sy to s, B) Hy erkale ia
though amenorrhea is no longer a diagnostic criteria or AN in C) Increased a ylase
DSM-5. D) Decreased cholesterol
E) Erythrocytosis
As you take more history, you realize that this patient is a
very nicky eater. She is a strict vegan (So? What is wrong Answer 23.15.4 The correct answer is “C.” A ylase ay be
with that?) and restricts her calories to < 1000/day in order increased as a result o urging behavior (and thus salivary
to stay in shape. She is 5 3 in tall and weighs only 100 lb, but sti ulation). “A” and “E” are incorrect. Leuko enia, not leu-
she thinks she is overweight. kocytosis, with a ild, nor ochro ic, nor ocytic ane ia
CHAPTER 23 • PSyCh IAt r y 649

TABLE 23-5 SELECTED MEDICAL COMPLICATIONS weights at the sa e ti e they are ex ected to reach eak bone
OF ANOREXIA NERVOSA ass. I a atient beco es osteo enic or osteo orotic when she
is young, she will re ain so as she ages (sans treat ent). Return
Neu ologic Seizu es
to good nutrition will not add bone ass that has been lost. All
Pe ip e al neu opa
Co ical a op o the other hysical changes listed should return to nor al
Cogni ive impai men with return to a nor al weight.
Ca diovascula B ad ca dia
Question 23.15.6 Which o the ollowing is the most appro-
O os a ic po ension
h ea ailu e priate next step in the management o this patient?
ECG c anges: low vol age, nonspeci c St A) ell her to sto gy nastics, withdraw ro classes, and go to
segmen c anges, Qt p olonga ion live with her other
B) Start an antide ressant
Endoc ine Ameno ea
Low t 3, t 4, and t Sh ( ansien cen al C) Ad it her to the hos ital
po oidism, p eviousl called “eu oid D) Consult sychiatry and nutrition s ecialists
sick s nd ome”) E) All o the above are equally valid o tions at this ti e
Os eopenia/os eopo osis
G ow e a da ion Answer 23.15.6 The correct answer is “D.” T is atient is ost
Fluids and De d a ion likely to bene t ro a coordinated lan involving a ultidisci-
elec ol es Me abolic alkalosis linary tea , including ri ary care, sychiatry, and nutrition.
h pokalemia “A” is incorrect, as it is a dra atic reaction that does not address
h pomagnesemia the atient’s ri ary roble , and the atient is unlikely to
h pocalcemia
co ly with it. “B” is incorrect. An antide ressant ay be hel -
Gas oin es inal Eleva ed live enz mes ul in atients with a clearly de ned de ressive or anxiety dis-
Cons ipa ion order. However, the use o antide ressant thera y or AN has
Esop agi is not been success ul, and certain antide ressants are associated
Mallo –Weiss ea s
Pa o id gland pe op
with weight loss. “C,” hos italization, is not likely to be bene -
cial at this oint. T e utility o hos italization has been dif cult
De ma ologic Lanugo to deter ine in AN. Co only acce ted reasons or hos ital
B i le nails and ai ad ission include the ollowing: severe low weight (70–75% o
Ac oc anosis
D , scal skin
ideal body weight), severe bradycardia (< 40 b ) or cardiac
arrhyth ia, arked sy to atic hy otension or synco e,
h ema ologic Bone ma ow supp ession acute sychiatric e ergency (threatened suicide), signi cant
dehydration or electrolyte disturbances, acute ood re usal, and
ailed intensive out atient thera y.
is a co on he atologic nding, although the CBC ay be
nor al. “B” is incorrect. T ere is usually whole-body de letion While you plan to involve psychiatric and nutritional ser-
o otassiu , zinc, agnesiu , and hos hate ( ro vo iting vices, you continue your discussion o eating disorders with
and/or inadequate intake). “D” is also incorrect, as cholesterol this patient.
is o en elevated (as are BUN and liver enzy es). T is choles-
terol elevation is neither intuitive nor well understood but is Question 23.15.7 Which o the ollowing is NOT TRUE about
observed to occur in atients with AN. See able 23-5 or ore the epidemiology o eating disorders?
edical co lications o AN. A) At least 90% o eating disorder atients are e ale
B) Rates are higher in industrialized, Western nations
In order to increase your patient’s motivation to comply with C) A a ily history o de ression increases the risk o e ales
medical recommendations, you describe some o the adverse develo ing eating disorders
e ects o excessively low weight. D) Male wrestlers have a higher risk o eating disorders than the
average ale
Question 23.15.5 All o the ollowing abnormal ndings E) Mortality rates in AN are si ilar to the general o ulation
will resolve when an adequate body weight is regained
EXCEPT: Answer 23.15.7 The correct answer is “E.” AN is characterized
A) Bradycardia by a high ortality, one o the highest o sychiatric illnesses. It
B) Muscle wasting ust be taken very seriously. U to 10% o hos italized atients
C) Osteo orosis die ro direct e ects o starvation, re eeding syndro e, sui-
D) In ertility cide, or electrolyte i balance leading to cardiac arrhyth ias.
At least 90% o eating disorder atients are e ale with reva-
Answer 23.15.5 The correct answer is “C.” Patients with AN lence rates higher in certain grou s, such as odels, actresses,
are usually young and there ore ex erience so e o their lowest athletes, and dancers. Western, industrialized societies endorse
650 FAMILy MEDICINE EXAMINAt ION & BOAr D r EVIEW

thinness and dieting as an ideal or wo en, resulting in higher as AN. Fluoxetine is the rst-line drug with a target dose o 60
rates o eating disorders. Des ite having a nor al body weight, g being shown to be e ective in trials. Other SSRIs are sec-
over 40% o 9- and 10-year-old American girls believe they ond line. T ird-line agents include CAs, MAOIs, and trazo-
are overweight and consider dieting as an o tion! Eating dis- done. T ese agents reduce binge eating and buli ic sy to s
orders are unco on in oorer countries where starvation is whether atients are de ressed or not.
wides read, and there is no cultural endorse ent o thinness.
“C” is true. A a ily history o de ression or obesity increases
risk o an eating disorder by two- to ve old, with a a ily his- HELPFUL TIP:
tory o de ression alone increasing the risk about ve old. “D” t e mean age o onse o ea ing diso de s is 17 ea s,
is also true. In ales, wrestlers have a higher revalence o eat- wi ve a e onse a e e age o 40 (al oug ecu -
ing disorders because o weight require ents in their s ort. In ence can occu in is popula ion).
ales, body sha e and not necessarily absolute body weight is
the ty ical ocus o concern.
Objectives: Did you learn to . . .
• Evalua e a oung pe son wi a suspec ed ea ing diso de ?
HELPFUL TIP:
• Diagnose AN?
t e e a e wo sub pes o AN: es ic ive and binge/
pu ge. t e e o e, a pa ien w o binges and pu ges • r ecognize medical complica ions o ea ing diso de s?
( oug o ced emesis, laxa ives, e c.) does no neces- • Dis inguis be ween AN and bulimia?
sa il ave bulimia. • Ini ia e app op ia e ea men and e e al o a pa ien wi
an ea ing diso de ?

HELPFUL TIP:
Como bidi ies a e p evalen in ea ing diso de pa ien s
QUICK QUIZ: BINGE AND PUr GE
and include MDD (50–75%), anxie diso de s, obsessive–
compulsive diso de , and subs ance abuse. Pe sonal- In order to be diagnosed with BN according to DSM-5, a atient
i diso de s a e also p evalen wi e anxious, sen- ust eet all o the ollowing criteria EXCEP :
si ive, igid, and pe ec ionis ic pes p edomina ing A) Recurrent urging via sel -induced vo iting
(Clus e C). B) Recurrent binge eating
C) Sy to s are resent on average at least once er week or
at least 3 onths
D) Sel -evaluation is unduly in uenced by body sha e and
In your discussion o eating disorders, your patient asks weight
about bulimia and how you distinguish between AN and
bulimia nervosa (BN). The correct answer is “A.” For the diagnosis o BN, sel -induced
vo iting need not be resent. Rather, so e sort o co ensa-
Question 23.15.8 Which o the ollowing is NOT TRUE when tory behavior ust be resent during buli ic e isodes. T is
comparing patients with AN and BN? behavior ight include vo iting, but is not li ited to vo it-
A) BN is ore revalent than AN ing, and ay also include laxative or diuretic use, asting, or
B) BN atients are ore likely to be nor al weight than AN excessive exercise. “B,” “C,” and “D” are required or the diag-
atients nosis o BN. BN a ects about 1% to 3% e ales (usually white)
C) T e rognosis or AN is better than that or BN and 0.1% to 0.3% ales. Co orbidities o de ression, anxiety,
D) BN atients are ore likely to have eso hageal tears substance abuse, and ersonality disorders are also co on.
E) Medications are e ective in BN but not in AN Dra atic, unstable ersonality traits (cluster B) redo inate
in this illness unlike the sensitive, rigid (Cluster C) ersonality
Answer 23.15.8 The correct answer is “C.” BN has a bet- traits in AN.
ter rognosis than AN, with only 20% o atients still eet-
ing diagnostic criteria 5 to 10 years a er initial resentation.
BN is ore co on than AN, with the revalence increasing QUICK QUIZ: EAt ING DISOr DEr S (Or , h OLD t h E
just like it has with AN. “B” is true. Patients with BN are usu- PIE FOr NOW, PLEASE)
ally close to nor al weight. However, they show several stig-
ata: loss o dental ena el and chi ed teeth with cavities; A reviously healthy 17-year-old e ale is hos italized or AN.
enlarged arotid salivary glands with elevated seru a ylase She has her weight ra idly restored with intravenous uids and
ro re eated vo iting e isodes; enstrual irregularities; a 3000 kcal/day diet. She gains 10 lb in her rst 2 days (so gain-
bradycardia, hy otension, and decreased etabolic rate. Rare ing 10 lb in 2 days sounds like a good thing?) and see s to be
co lications include eso hageal tears ro requent vo iting doing well. However, when you round the next orning, she is
or gastric ru ture due to gastric dilatation secondary to bing- nonres onsive. Physical exa ination reveals evidence o JVD,
ing. “E” is true. reat ent o BN involves the sa e rinci les ul onary rales, and lower extre ity ede a.
CHAPTER 23 • PSyCh IAt r y 651

What is the ost likely cause o this atient’s a arent heart C) Fe ale hysicians are ore likely to atte t suicide than
ailure? the general e ale o ulation
A) Previously undiagnosed heart disease D) Medical students and residents are less likely to be de ressed
B) Re eeding syndro e than the general o ulation
C) Suicide atte t by SSRI overdose
D) Myocardial in arction The correct answer is “A.” In the general o ulation, wo en
E) akotsubo cardio yo athy have 2 to 4 ti es ore suicide atte ts than en, while en
are 4 ti es ore likely than wo en to be success ul. However,
The correct answer is “B.” T e scenario described ts with the a ong hysicians, the rate o success ul suicide in en and
clinical icture caused by re eeding syndro e. Re eeding syn- wo en is equivalent (we didn’t study all o that har acology or
dro e occurs when ra id ex ansion o the circulating volu e nothing!). T is occurs des ite the act that e ale hysicians have
overwhel s the cardiovascular syste ’s ability to ada t, lead- ewer atte ts than the general e ale o ulation (we didn’t get
ing to heart ailure. It also involves changes in electrolytes and through ed school by just atte ting studying!). Physicians
glucose. It ty ically occurs in alnourished atients. Preven- have roughly the sa e rate o de ression as the general o u-
tion involves care ul onitoring o electrolytes including ag- lation, while edical students and residents have higher rates
nesiu , hos horus, otassiu , and calciu , while advancing than the general o ulation (15–30% vs. 16% in the general
caloric intake in a s all, linear ashion and kee ing track o vol- o ulation . . . i you were ever a edical student or resident we
u e status. “A,” “D,” and “E” are incorrect as the atient is un- are sure you got this one right). Other actors a ecting suicide
likely to have signi cant heart disease given her age and gender. risk (in addition to being a hysician) are noted in able 23-8.

HELPFUL TIP:
QUICK QUIZ: SUICIDE r ISK
t e e is no case o is able bu edi o s a e i w en
e e is no “call ou ” o a able. So, see t able 23-6 o
wa ning signs a an agi a ed pa ien ma become You have a colleague who is de ressed and has transient
violen . See t able 23-7 o wa s o calm e agi a ed pa- thoughts o suicide.
ien (al oug in e edi o s’ expe ience no ing bea s
IM alope idol and IM lo azepam). Which o the ollowing would suggest that he should be hos i-
talized today?
A) He thinks about suicide only in requently
B) He has not or ulated a lan to co it suicide
QUICK QUIZ: Ph ySICIAN h EAL t h ySELF?

Which o the ollowing is true about hysicians’ risk o suicide TABLE 23-7 TECHNIQUES TO CALM THE
co ared to the general o ulation? AGITATED PATIENT
A) Fe ale hysicians have a risk o success ul suicide equal to
ale hysicians r emove e pa ien o a quie and nons imula ing envi onmen
B) Physicians are ore likely to be de ressed than the general Keep a dis ance om e pa ien and avoid p sical con ac
o ulation Iden i exi s and ala ms
Main ain non ea ening demeano and s ance
TABLE 23-6 WARNING SIGNS THAT AN AGITATED Keep ou ands a ou side w e e e a e easil visible
PATIENT MAY BECOMEVIOLENT W ile main aining s ead e e con ac , speak in a s ead bu
h pe ac ivi : pacing o o e inc ease in ps c omo o ac ivi au o i a ive voice, using e pa ien ’s name wi eac sen ence

Loud, ang , o p o ane speec Avoid sudden je k movemen s and emain calm

Inc eased muscle ension, mani es ed b clenc ed jaw, s , igid h ave amilia aces nea b i possible
pos u e, g ipping c ai , si ing on c ai edge, e c. S ow e pa ien conce n bu ell im a violence is no accep able
In oxica ion and ou a e willing o wo k wi im i e calms down

Suspicious, ang , o i i able a ec t e e is s eng in numbe s; ave o e pe sonnel nea b o elp i


necessa
B ea lessness, ac ca dia, diap o esis, pupilla dila ion, visibl
palpi a ing empo al a e ies Call o elp om police o secu i i needed; a back-up s s em, w ic
as been es ed, s ould be in place
Uncoope a iveness wi eques s
Conside an ips c o ics, suc as alope idol (available in o al and
Doo slamming, c ai oppling, o o e o m o p ope des uc ion pa en e al o ms) and ispe idone (available in liquid); a benzodiazepine
G abbing objec s a could be po en ial weapons ma be used as an adjunc , bu s ould not be used alone
Ve bal o p sical ea s Mec anical es ain s ma be emplo ed i absolu el necessa
t e clinician’s esponse o e pa ien : i ou eel anxious, ake i r emembe o gu e ou why e pa ien is agi a ed (e.g., ake is o ,
se iousl and be ale o possible dange pe o m men al s a us examina ion, and o de app op ia e es s)
652 FAMILy MEDICINE EXAMINAt ION & BOAr D r EVIEW

TABLE 23-8 FACTORS AFFECTING SUICIDE RISK CASE 23.16


Risk Factors
• Living alone (pa icula l s ea alone) You are on call and have been paged to see a 21-year-old
• Loss o spouse o sepa a ion emale who has just overdosed on a hand ul o acetamino-
• Alco ol use phen because her boy riend le her a er their most recent
• h aving access o a gun o le al means ght. She has had similar overdoses three times in the past
• Loss o ac ivi , job, p ope , o capabili ies
(they ght a lot). According to a riend, she has a history o
• Mild/minimal cogni ive impai men
• Pa anoia tumultuous relationships. In the emergency department, she
• h opelessness is combative and yelling, “I’m so angry that I’m still alive!”
• Anxie She has a blood alcohol level o 106 mg/dL. T ere are scars on
• Olde w i e male her arms rom cutting.
• Pe sonal o amil is o o suicide a emp o comple ion
• U ban dwelle
Question 23.16.1 What is the most likely primary diagnosis?
Protective Factors A) Antisocial ersonality disorder
• E ec ive ea men o men al and p sical diso de s B) Bi olar e ective disorder
• Social and amil suppo
C) MDD
• Coping skills
• r esilience D) Borderline ersonality disorder
• r eligious ai E) Inter ittent ex losive disorder
• Lack o access o le al means
Answer 23.16.1 The correct answer is “D.” T is atient’s his-
tory is consistent with a diagnosis o borderline ersonality dis-
order. Persons with borderline ersonality disorder o en have
C) He has u dated his will within the ast ew days stor y relationshi s, characterized by extre es o e otional
D) He is willing to ollow u in clinic to orrow intensity (e.g., “love–hate” relationshi s). You ight also con-
sider de ression as a diagnosis here, but the history does not
The correct answer is “C.” I a erson ad its to suicidal ide- su ort MDD. However, conco itant de ression is co on in
ation, ask the ollowing questions to assess his level o risk and atients with borderline ersonality disorder. “A,” “B,” and “E”
to deter ine whether or not he should be hos italized: are not su orted by the clinical resentation. Borderline er-
• How o en does he think about suicide? sonality disorder is ostly a diagnosis o e ales (over 90%).
• Does he have a concrete lan? And i so, is it lausible? Characteristics are listed in able 23-9.
( ying onesel to a rocket does not count . . . or ost eo le.)
• Is he giving away treasured belongings, u dating his will,
aking nal lans, etc.? HELPFUL TIP:
• Is he in danger o acting on his thoughts? A pe sonali diso de is an endu ing pa e n o ela ing
• Why has he not atte ted suicide yet? What kee s hi ro o e wo ld in wa s a a e in lexible, ine ec ive, and
doing it? ma kedl di e en om cul u al no ms. Pe sonali dis-
• Does he have access to har ul eans (e.g., guns and drugs)? o de s cause dis ess o unc ional impai men and s a
b adolescence in mos people. t e p evalence a e o
pe sonali diso de s va ies widel ac oss s udies, wi
QUICK QUIZ: PSyCh Ot r OPIC Dr UGS bo de line pe sonali diso de being e mos p eva-
len o all.
Which o these antide ressants can be ad inistered once
weekly?
A) Paroxetine (Paxil) TABLE 23-9 CHARACTERISTICS OF BORDERLINE
B) Bus irone (Wellbutrin) PERSONALITY DISORDER
C) Fluoxetine (Prozac)
D) Escitalo ra (Lexa ro) • Fea s o abandonmen
• Uns able and in ense ela ions ips
E) Venla axine (E exor-SR) • Uns able sense o sel
• Impulsivi
The correct answer is “C.” Fluoxetine has a long-enough hal - • Suicidal be avio , ea s o ges u es o sel -mu ila ion (cu ing is
li e that it can be ad inistered once a week. You don’t need to o en a ea u e o bo de line pe sonali diso de )
rescribe the “long-acting” uoxetine. Any uoxetine can be • Signi can mood eac ivi
• C onic eelings o emp iness (Camus o Nie zsc e?)
dosed once weekly. It is traditionally dosed daily, but i atients • In ense ange ou bu s s
are ex eriencing side e ects, dosing can be adjusted to every • t ansien s ess- ela ed pa anoid idea ion o dissocia ion
other day or weekly.
CHAPTER 23 • PSyCh IAt r y 653

TABLE 23-10 CHARACTERISTICS OF ANTISOCIAL D) Avoidant ersonality disorder


PERSONALITY DISORDER E) Antisocial ersonality disorder
• r ecu en c iminali
Answer 23.16.3 The correct answer is “E.” Along with border-
• Decei ulness s own b epea ed l ing, use o aliases, o conning
o es line ersonality disorder, antisocial ersonality disorder is the
• Impulsivi other ersonality disorder ost likely to be seen in co leted
• I i abili and agg ession suicides.
• r eckless dis ega d o sa e
• Consis en i esponsibili , ailing o ul ll nancial obliga ions o
Question 23.16.4 Borderline personality disorder and anti-
wo k
• Lack o emo se social personality disorder all within what cluster or cat-
• h is o o c ild ood conduc diso de in a pa ien now a leas 18 egory o personality disorders?
A) Cluster A
B) Cluster B
C) Cluster C
Your patient’s boy riend storms into the emergency depart- D) Cluster D
ment and demands to see her. He has “love” and “hate”
tattooed on his knuckles and another less savory symbol Answer 23.16.4 The correct answer is “B.” T ere are three
tattooed on his arm—although he swears it’s just a ibetan clusters o ersonality disorders that are categorized based on
good luck charm. T e nurse recognizes him immediately as their characteristics.
a requent visitor. Apparently, he has been in the emergency • Cluster A: “the weird cluster”— atients are aloo , act strange,
department on multiple occasions or injuries sustained and re er to be alone. Paranoid, schizoid, and schizoty al
rom ghts. Your patient pulls you aside to tell you that she ersonality disorders are included here.
is a raid o him, saying, “I just worry when he gets mad. • Cluster B: “the wild cluster”— atients have signi cant rob-
He went to jail or beating and raping his last girl riend.” le s with ood lability, i ulsivity, or are reoccu ied with
T e nurse also in orms you that he is suspected o stealing being ad ired or their sexuality or intelligence. Borderline,
narcotics rom a pharmacy in town. You call security. antisocial, histrionic, and narcissistic ersonality disorders
are included here.
Question 23.16.2 Which o the ollowing personality disor-
• Cluster C: “the whiny or worried cluster”— atients are
ders is most likely in this man?
clingy, sensitive, and rigid. Avoidant, de endent, and obses-
A) Paranoid ersonality disorder
sive–co ulsive ersonality disorders are included here.
B) Histrionic ersonality disorder
C) Schizoty al ersonality disorder
D) Antisocial ersonality disorder HELPFUL TIP:
E) I ajerk ersonality disorder you will be app o know (as a e we) a , as o DMS 5,
e old “axis” designa ion s s em (axis 1, axis 2, e c.) o
Answer 23.16.2 The correct answer is “D.” Antisocial er- ps c ia ic disease as gone e wa o e dodo bi d
sonality disorder is ri arily seen in ales, and o the o tions (o pick ou avo i e ex inc species).
given, it is the only ersonality disorder that really ts. Antiso-
cial ersonality disorder is quite revalent in rison o ulations.
“A” and “C” are incorrect because these are Cluster A disorders, HELPFUL TIP:
which are characterized by strange rather than violent behavior. Impulsivi is a isk o comple ed suicide. Even oug
“B” is incorrect. Histrionic ersonality disorder shows attention- pa ien s ma no wan o die, an impulsivel aken ove -
seeking and seductive behavior. Although atients with antiso- dose o o e suicide a emp ma inadve en l lead o
cial ersonality disorder can be char ing, they also use violence dea . t is is w pa ien s wi bo de line pe sonali
and threats to achieve their ur oses. “E” is incorrect because diso de and an isocial pe sonali diso de ave a ig
it does not exist as a disorder—it’s a joke (I’ -a-jerk, get it?)— isk o comple ed suicide.
although i it did exist, this atient would be a ri e exa le.
Characteristics o antisocial ersonality disorder are listed in
able 23-10. You have a rank discussion with your emale patient about
suicide.
Question 23.16.3 Besides borderline personality disorder,
which other personality disorder increases the risk o com- Question 23.16.5 Which o the ollowing is NOT TRUE about
pleted suicide the most? suicide in the general population?
A) Paranoid ersonality disorder A) Medical diagnoses can increase the risk o suicide
B) Histrionic ersonality disorder B) Over 90% o eo le who co it suicide have a ental or
C) Schizoty al ersonality disorder substance abuse disorder
654 FAMILy MEDICINE EXAMINAt ION & BOAr D r EVIEW

TABLE 23-11 MNEMONICS FOR SUICIDE Question 23.17.1 Which o the ollowing would you NOT
RISK ASSESSMENT expect to nd on mental status examination?
A) Delusions
NO HOPE SAD PERSONS
B) Hallucinations
No amewo k o meaning (r eading Sex—male C) Lack o insight
Nie zsc e and being an exis en ialis ?) Age—olde D) Decreased sycho otor activity
Ove c ange in clinical condi ion Dep ession E) Poverty o s eech
Hos ile in e pe sonal envi onmen P evious a emp
Ou o ospi al ecen l E anol abuse Answer 23.17.1 The correct answer is “D.” T e atient
P edisposing pe sonali ac o s Ra ional oug loss
described is acutely sychotic. You would ex ect hi to have
Excuses o d ing o elp o e s increased sycho otor activity. All o the other o tions would
Social suppo lacking
also be antici ated ndings in this atient. Co on sy to s
O ganized plan
o sychosis include delusions, hallucinations, sycho otor
No spouse
agitation, ight o ideas, nonsensical s eech and behavior, lack
Sickness o insight into one’s behavior, and lack o judg ent. Note that
“poverty o speech” re ers to brie , empty replies to questions and
C) Older A ericans are at higher risk o suicide than the gen- not to content which may be quite anci ul.
eral o ulation
D) T e rate o success ul suicides has increased over the years Question 23.17.2 All o the ollowing are potential causes
E) Suicide is one o the to 10 causes o death in the United o this patient’s psychosis EXCEPT:
States A) Substance abuse
B) Alcohol withdrawal
Answer 23.16.5 The correct answer is “D.” T e rate o success- C) Bi olar disorder
ul suicide has actually stayed stable over ti e. “A” is a correct D) Bereave ent
state ent. Medical diagnoses can be risk actors or suicide. Par- E) Schizo hrenia
ticularly roble atic are chronic illness, ter inal illness, chronic
ain, or recent surgery. Older A ericans co it suicide at a rate Answer 23.17.2 The correct answer is “D.” Bereave ent ay
our ti es than that o the general o ulation with a eak inci- result in ild delusions and so eti es hallucinations regarding
dence at age 75 or en and 60 or wo en. “E” is also correct. the bereaved subject (Hey? Is that o back ro the dead?),
Suicide is the seventh leading cause o death in the United States but it should not cause overt sychosis. Moreover, there is noth-
and the third leading cause o death in those aged 15 to 24 years. ing in the history here to su ort a diagnosis o bereave ent.
T ere are two ne onics that are use ul or assessing suicide T e re aining o tions are otential cause o sychosis, and
risk: “NO HOPE” and “SAD PERSONS” (see able 23-11). they are listed along with other otential causes in able 23-12.

TABLE 23-12 CAUSES OF PSYCHOSIS AND


HELPFUL TIP: DELIRIUM: A PARTIAL LIST
Seven - ive pe cen o olde Ame icans w o commi
suicide ave seen ei p ima ca e p sician wi in Potentially Imminently Li e -Threatening Causes
• Meningi is o encep ali is
e p eceding 4 weeks and 39% wi in e same week. • h poxemia
Up o al o success ul suicides ave made a p io • h pe ensive encep alopa
a emp . • We nicke encep alopa
• In ac anial bleed
• D ug wi d awal, in oxica ion, o eac ion o p esc ibed d ugs
Objectives: Did you learn to . . .
• Iden i bo de line and an isocial pe sonali diso de s? Other Medical Causes
• Me abolic diso de s (e.g., pe gl cemia and pona emia)
• Classi pe sonali diso de s? • Neu ologic diso de s
• Assess suicide isk? • Nu i ional de ciencies (e.g., pellag a, be ibe i, and pe nicious
anemia)
CASE 23.17 • Indus ial exposu e o oxins

Psychiatric Causes
A 21-year-old man presents to your clinic because his girl- • Sc izop enia o sc izop eni o m diso de s
riend dragged him in. He just started his rst year in college • B ie ps c o ic diso de
but quit 2 days ago because he eels that “they are all out to • Mood diso de s including bipola diso de and ps c o ic
get me.” (“I he’s talking about the IRS, or New World Order, dep ession
he’s probably right,” you think to yoursel .) He has not been • Sc izoa ec ive diso de
• Demen ia
sleeping because he thinks he might be murdered in his sleep. • Deli ium
He tells you that the FBI has bugged your o ce and, there- • Delusional diso de
ore, he does not want to answer your questions.
CHAPTER 23 • PSyCh IAt r y 655

equally by the illness. T e course is variable with so e atients


HELPFUL TIP:
having exacerbations and re issions (although ull re issions
Ps c osis is a s mp om and no a diagnosis and s ould
are rare), while others re ain chronically ill. About hal o the
p omp a sea c o an e iolog . Some o e causes a e
atients who develo schizo hrenia have a a ily history o
po en iall li e ea ening.
schizo hrenia. Suicide atte ts are co on, usually the result
o de ression or a res onse to co and hallucinations, ara-
noid delusions, or agitation. Nearly 50% will atte t suicide,
T e patient relaxes and becomes cooperative, and you are while about 10% will be success ul. A er success ul treat ent o
able to obtain a history and per orm a physical examination. the rst e isode, about 50% will rela se i not on aintenance
He denies drug use or medical illnesses. Your patient worries edication.
that he is going “crazy” and admits that he has been having
these symptoms “ or a while” but did not want to tell any- Question 23.17.5 Which o the ollowing medication options
body, or ear o being institutionalized. Your physical exami- is the proper treatment choice?
nation and labs are unremarkable. A) Olanza ine (Zy rexa)
B) Ris eridone (Ris erdal)
Question 23.17.3 Which o the ollowing diagnoses is the C) Halo eridol (Haldol)
most likely cause o this patient’s psychosis? D) Ari i razole (Abili y)
A) Schizo hrenia E) Any o the above
B) Psychotic de ression
C) Deliriu
Answer 23.17.5 The correct answer is “E.” All o the anti sy-
D) Drug intoxication
chotics listed are rst-line treat ent choices or schizo hrenia.
E) De entia
Most sychiatrists have re laced older “ty ical” (now known
as “ rst-generation” agents like halo eridol, chlor ro azine,
Answer 23.17.3 The correct answer is “A.” Schizo hrenia is a
u henazine, thioridazine) with newer “aty ical” (now known
heterogeneous grou o disorders characterized by the ollow-
as “second-generation” agents, such as ris eridone, cloza ine,
ing: ositive sy to s (delusions, hallucinations, disorganized
zi rasidone, ali eridone, and olanza ine) because o better tol-
behavior, disorganized s eech); negative sy to s ( overty
erability and ossibly increased bene t or negative sy to s.
o s eech, anhedonia, a ective attening, avolition, asocial-
Although, the newer agents are signi cantly ore ex ensive
ity); ood sy to s (dys horia, suicidal thoughts, ho eless-
than the older agents, ore generics are beco ing available.
ness); and cognitive sy to s (attention and e ory de cits
Also, any “aty icals” cause signi cant weight gain, and so e
and dif culty with abstract thinking). It is the ost co on o
have been linked to new onset diabetes. O the newer agents,
the sychotic disorders. A negative laboratory evaluation and
ari i razole (Abili y) and zi rasidone (Geodon) are less associ-
hysical exa ination ake drug intoxication unlikely, while
ated with weight gain and diabetes.
this atient’s ability to converse with you and give a history
akes deliriu and de entia unlikely. Psychotic de ression is
unusual in young eo le, but rather is ore co only seen in Question 23.17.6 Which o the ollowing IS TRUE o the
older atients with severe de ression. course o schizophrenia?
A) Positive sy to s usually occur rst
Question 23.17.4 Which o the ollowing is NOT TRUE about B) Negative sy to s are easier to treat than ositive sy -
schizophrenia? to s
A) Nearly 50% atte t suicide with a 10% success rate C) Negative sy to s o en look like de ression
B) Li eti e revalence is 1% worldwide, consistent across cul- D) Schizo hrenia is not ty ically associated with brain changes
tures E) Fa ily thera y is not hel ul in this illness
C) Schizo hrenia is a disease o late adolescence or early adult-
hood Answer 23.17.6 The correct answer is “C.” Negative sy -
D) Men are ore likely to be a ected than wo en to s o en recede the develo ent o the ositive sy to s
E) Risk o rela se is at least 50% a er success ul treat ent in by any years, are o en nons eci c, and can be istaken or
atients who do not re ain on anti sychotic aintenance de ression. A ty ical history is that o a nor al young an
thera y or wo an who begins to ail classes and avoid his old riends
as he gets to the end o high school. T is is o en istaken
Answer 23.17.4 The correct answer is “D.” Schizo hrenia has or teenage rebellion, de ression, or drug use until the onset
a worldwide revalence o about 1%, which is true or all cul- o ositive sychotic sy to s o hallucinations or delu-
tures, countries, and both genders. It generally begins in late sions several years later. “B” is incorrect. Negative sy to s
adolescence or early adulthood, and onset a er age 50 is rare are chronic and are resistant to treat ent with all currently
and should ro t the search or other etiologies to ex lain available anti sychotics. Anti sychotic drugs odulate do a-
the sychosis. Men have a slightly earlier age o onset (early ine and/or serotonin and are uch better at treating osi-
20s) than wo en (late 20s), but en and wo en are a ected tive sy to s. “D” is not true, as brain i aging ay reveal
656 FAMILy MEDICINE EXAMINAt ION & BOAr D r EVIEW

enlarge ent o cerebral ventricles and decreased brain volu e. D) Reco end bright light at night be ore going to work
However, these ndings are neither sensitive nor s eci c E) Prescribe sti ulants or when he is at work
enough to have uch value in diagnosis. “E” is incorrect. A
large body o literature su orts the act that a ily thera y, Answer 23.18.2 The correct answer is “D.” T is atient’s
es ecially directed at su ort and sychoeducation, reduces circadian rhyth disturbance ight also be called a “ hase-
the risk o rela se. Recent evidence indicates that individual advance” ty e slee disorder (early slee onset with inso nia at
CB can also be e ective. the desired slee eriod). Such roble s res ond best to bright
light in the evening to kee one awake during the ti e the indi-
vidual would usually be aslee . Light boxes are available co -
HELPFUL TIP: DUAL ANTIPSYCHOTICS: ercially and should rovide white light at 2,500 lux or ore.
t e e is almos neve an indica ion o mo e an one T e light should be airly close to the atient’s eyes, and a bit o
an ips c o ic. Don’ do i . to the side. T e other o tions are not likely to be hel ul i this
atient wants to kee his job.

Question 23.18.3 I this patient wished to use a nutritional


Objectives: Did you learn to . . . or herbal supplement, which o the ollowing would you
• r ecognize ps c osis and i s causa ive diagnoses? recommend?
• t ea ps c osis? A) Melatonin
• r ecognize sc izop enia and unde s and i s epidemiolog B) Kava-kava
and p ognosis? C) Ginkgo
D) Ginseng
E) St. John’s wort
CASE 23.18
A 33-year-old actory worker comes to your clinic complain- Answer 23.18.3 The correct answer is “A.” T ere is so e evi-
ing o sleep di culty since he moved to the “graveyard shi .” dence to suggest that elatonin can hel with circadian rhyth
He complains o alling asleep at work and having di culty disorders (and not inso nia in general where data is s otty).
sleeping during the day. Melatonin should be taken at the ti e o day the atient wants
to slee . In this case, you should advise hi to take it in the
Question 23.18.1 Which o the ollowing is the most likely orning. T e doses used or circadian rhyth disturbances are
diagnosis? uch less than the dose used or ri ary inso nia, ty ically
A) Narcole sy 0.5 g or less, co ared with 3 g. T e other herbal su le-
B) Circadian rhyth slee –wake disorder ents have no evidence or use in slee disturbances exce t or
C) Obstructive slee a nea kava-kava, which is too dangerous to reco end.
D) Pri ary inso nia
E) Fatal a ilial inso nia Question 23.18.4 Which o the ollowing is NOT a delayed
sleep phase type o circadian rhythm sleep disorder?
A) Re-ada tation to day work ro night work
Answer 23.18.1 The correct answer is “B.” T is atient su ers
B) West-to-east jet lag
(as do ost residents and a signi cant nu ber o hysicians)
C) East-to-west jet lag
ro a circadian rhyth slee –wake disorder, which is de ned
D) All o the above are delayed slee hase disorders
as a slee disru tion leading to excessive slee iness when the
atient wishes to be awake or inso nia when the atient wishes
to be aslee . It occurs as a result o a is atch between the Answer 23.18.4 The correct answer is “C.” Delayed slee hase
biological circadian rhyth and the erson’s environ ent. He involves a ersistent attern o late slee onset and late orning
does not give a history consistent with narcole sy or obstructive awakening. T is is exe li ed by the average teenager during
slee a nea, and ri ary inso nia is a diagnosis o exclusion. su er vacation (just try to get hi to ow the lawn be ore
Fatal a ilial inso nia is a rare genetically deter ined rion noon!). West-to-east jet lag and re-ada ting to day shi a er
disease resulting in co lete inability to slee , de entia, and working night shi also cause si ilar roble s. “C” does not
death within a ew years. cause a delayed slee hase disturbance. East-to-west jet lag
causes the o osite roble , advancing slee hase so that
He asks your advice on how to treat his sleeplessness. ersons are slee y early in the evening but then awake early in
the orning. A otentially e ective treat ent o delayed slee
hase would include rescribing a bright light in the orning
Question 23.18.2 What would be the best advice to of er at and elatonin in the a ernoon.
this time?
A) Have hi slee at work (we do!) Objectives: Did you learn to . . .
B) Have hi take na s throughout the day • r ecognize ci cadian m diso de s?
C) ell hi to quit his job • r ecommend ea men o ci cadian m diso de s?
CHAPTER 23 • PSyCh IAt r y 657

white atter hy erintensities on 2-weighted MRI. reat ent


QUICK QUIZ: LESt yOU FOr GEt includes odi ying vascular risk actors.

A 70-year-old an resents with e ory loss that has been


rogressive over the years. He o en has uctuating attention
CASE 23.19
and is rigid with bradykinesia. He requently ex eriences visual A 37-year-old emale patient returns to see you or ollow-up
hallucinations. o depression. You last saw her 4 weeks ago, and at that time
she was experiencing her third relapse o recurrent major
Which o the ollowing is the ost likely diagnosis? depression. She had been in remission or 3 years be ore, and
A) Alzhei er disease until 2 years ago she had been taking sertraline 150 mg daily,
B) De entia with Lewy bodies (DLB) a regimen that she had ound e ective during an 18-month
C) Parkinson disease course o treatment. At her visit last month, you assessed her
D) Vascular de entia symptoms with the Patient Health Questionnaire-9 (PHQ-9),
E) Frontote oral de entia and her score at that time was 21. You reinstituted sertraline
at 50 mg daily or 2 weeks and then 100 mg daily. At her
The correct answer is “B.” DLB shares any eatures in co - visit today, now on sertraline 100 mg daily or 2 weeks, her
on with Parkinson disease. T ese eatures include arkin- PHQ-9 score is 17. She denies having thoughts o death or
sonian otor sy to s, such as bradykinesia, rigidity, and suicide.
tre or. Both Parkinson disease and DLB are characterized
histologically by the resence o Lewy bodies. However, in Question 23.19.1 Which o the ollowing statements is NOT
DLB, Lewy bodies are di usely s read in the cortical regions true regarding the PHQ-9?
and brainste , whereas in Parkinson disease, Lewy bodies are A) T e PHQ-9 is a clinician-ad inistered questionnaire to
ri arily resent in the subcortical nuclei. Fluctuating cog- evaluate or sy to s o ajor de ression
nition that rese bles deliriu , visual hallucinations, and an B) It takes 3–5 inutes to co lete the PHQ-9
exquisite sensitivity to the adverse e ects o neurole tics all C) A score o 10 or ore on the PHQ-9 is a ositive screen
characterize DLB. T ere is a lack o res onse o do a inergic D) T e PHQ-9 can be used to screen or de ression as well as to
agents with DLB but cholinesterase inhibitors o er a odest onitor sy to severity over ti e
bene t. E) T e PHQ-9 does NO assess or suicide risk

QUICK QUIZ: DEMENt IA Answer 23.19.1 The correct answer is “A.” T e PHQ-9 is a
atient-ad inistered, nine-ite questionnaire that takes 3 to
5 inutes to co lete. It can be used to screen or ajor de res-
A 65-year-old an with a history o hy ertension, diabetes, and
sion as well as to onitor sy to severity over ti e. A score
eri heral vascular disease has been noted to have abru t dete-
> 10 is considered a ositive screen, while a score < 5 is consid-
rioration in his cognitive ability ollowing an e isode o disori-
ered to be negative or in re ission. T e axi u score is 30.
entation and word- nding dif culty 1 onth ago.
While the PHQ-9 does ask about the resence o “thoughts o
death or dying,” it does not assess whether so eone is at low or
Which o the ollowing is the ost likely diagnosis? high risk or suicide atte t.
A) Alzhei er disease
B) DLB
C) Parkinson disease Question 23.19.2 Which o the ollowing options is LEAST
D) Vascular de entia appropriate to recommend to your patient at this time?
E) Frontote oral de entia A) Continue sertraline 100 g daily and return or ollow-u in
1 onth
The correct answer is “D.” Vascular de entia ( reviously B) Continue sertraline 100 g daily, add weekly CB , and
known as ulti-in arct de entia) is caused by vascular dis- return or ollow-u in 1 onth
ease (sur rise! An illness with a na e that akes sense!) and, C) Increase sertraline to 150 g daily and return or ollow-u
next to Alzhei er disease and Lewy body de entias, is one o in 1 onth
the ost co on causes o de entia. It classically resents D) Sto the sertraline and start citalo ra 20 g daily and
with an abru t onset ollowed by ste -wise deterioration o return or ollow-u in 1 onth
cognitive unction. Medical co orbidities are co on, in- E) Continue sertraline 100 g daily and add bu ro ion SR
cluding diabetes, hy ertension, and obesity. Evidence o vas- 150 g BID, return or ollow-u in 1 onth
cular disease is usually resent on clinical exa ination with
ocal neurological signs, such as are seen a er a stroke involv- Answer 23.19.2 The correct answer is “D.” At the current
ing otor areas. I aging ty ically shows in arctions o eri- visit, the atient has been taking sertraline or 1 onth and
ventricular and dee subcortical white atter, resenting as has been at the resent dose or 2 weeks. It can take u to
658 FAMILy MEDICINE EXAMINAt ION & BOAr D r EVIEW

6 to 8 weeks or an antide ressant edication to reach its ull


HELPFUL TIP:
e ect, so it is reasonable to aintain the edication at its
r andom ings o emembe o e examina ion: clo-
resent dose and reassess in 1 onth. However, o tions “B,”
zapine (Cloza il) manda es a CBC mon l because o
“C,” and “E” are reasonable as well. Co bination treat ent
bone ma ow supp ession; second-gene a ion an ips -
with CB and an antide ressant edication has been shown
c o ics can cause diabe es and a e no ee o neu ologic
to be ore e ective or treating de ression than either treat-
side e ec s (e.g., ex ap amidal eac ions); an ic olin-
ent strategy alone. You can also consider the addition o an
e gics (e.g., benz opine) s ould be used wi is-
aug entation agent (bu ro ion, 3, or a second-generation
gene a ion an ips c o ics o p even Pa kinsonian
anti sychotic). Since she has had a artial res onse to the
side e ec s.
current regi en, adding an aug entation agent is a reason-
able o tion. Switching to a new antide ressant at this ti e is
not indicated.

Despite increasing the dose o sertraline, your patient’s Clinical Pe arls


depression does not improve. She’s been watching a lot o Avoid i s -line use o an ips c o ic medica ion o indica-
V (no wonder she’s depressed) and seen ads or a medi- ions o e an ps c o ic diso de s (e.g., dep ession in an
cation called Happi y, which is used to augment tradi- pa ien , insomnia in an pa ien , be avio al diso de s in
tional antidepressant therapy. She says, “I think I need c ild en, o demen ed elde s).
one o those new wonder drugs, Doc” (the so-called “won- Do no p esc ibe benzodiazepines as i s -line e ap o e
der drugs” because it is a wonder that they ever got FDA ea men o anxie diso de s.
approval). Do no diagnose “adul -onse ” ADh D. Explo e e is o and
i e pa ien as ad ADh D s mp oms since c ild ood, e/
Question 23.19.3 Which o the ollowing have gained FDA s e ma ave a missed o dela ed diagnosis o ADh D.
approval as add-on therapy or depression? Do no p esc ibe wo an ips c o ic medica ions simul ane-
A) Ari i razole ousl .
B) Halo eridol Ensu e a s s ems a e in place o ea and e e pa ien s
C) Olanza ine diagnosed wi dep ession, and sc een o dep ession in e
D) Quetia ine gene al adolescen and adul pa ien popula ion.
E) A, C, and D O e p a maco e ap and p o essional counseling se vices
o all pa ien s diagnosed wi dep essive diso de s as e
Answer 23.19.3 The correct answer is “E.” Ari i razole, que- combina ion esul s in be e ou comes an ei e in e ven-
ion alone.
tia ine, and olanza ine are FDA a roved as add-on edica-
tions to aug ent antide ressants in ono olar de ression. Sc een o me abolic s nd ome pe iodicall in pa ien s
T e bene ts are arginal. For exa le, ari i razole added to a aking second-gene a ion an ips c o ics (e.g., olanzapine,
traditional antide ressant i roved de ression by 3- oint di - ispe idone, que iapine).
erence on a 60- oint scale and then only in en. Ari i razole
is ex ensive and a sizeable inority o atients develo akathi-
sia or restlessness. And re e ber that statistically signi cant BIBLIOGRAPHY
does not necessary ean clinically signif cant. For ari i razole,
the nu ber needed to treat to achieve this s all reduction in Ables AZ, Nagubilli R. Prevention, recognition, and an-
age ent o serotonin syndro e. Am Fam Physician.
de ressive sy to s is 10. On the bright side, ari i razole is
2010;8:1139–1142.
less associated with weight gain co ared with other aty ical
A erican Psychiatric Association. Practice Guideline or the
anti sychotics. And, the drug co anies are chortling on their
Treatment o Patients with Major Depressive Disorder. 3rd
way to the bank. ed. 2010. Available at htt :// sychiatryonline.org/content.
Objectives: Did you learn to . . . as x?bookid=28&sectionid=1667485.
• Develop s a egies o ea esis an dep ession? A erican Psychiatric Association. Diagnostic and Statistical
• Desc ibe a widel used dep ession assessmen ool, e Manual o Mental Disorders. 5th ed., ext Revision. Wash-
Ph Q-9? ington, DC: A erican Psychiatric Association; 2013.
Glick ID, et al. Mid-ter and long-ter ef cacy and e ective-
ness o anti sychotic edications or schizo hrenia: A
T e world o sychiatric drugs is constantly oving (akathi- data-driven, ersonalized clinical a roach. J Clin
sia?). See htt ://www.ncbi.nl .nih.gov/books/NBK84656/ or a Psychiatry. 2011;72:1616–1627.
list o anti sychotics and age a ro riate FDA a roved indica- Hendin H, et al. Con ronting de ression and suicide in
tions. A si ilar table or antide ressants is ound at htt ://www. hysicians: A consensus state ent. JAMA. 2003;289:
de ressionhealth.net/antide ressants/. 3161–3166.
CHAPTER 23 • PSyCh IAt r y 659

Khan A, et al. Suicide rates in clinical trials o SSRIs, other Schatzberg AF, et al. Antide ressant discontinuation syn-
antide ressants, and lacebo: Analysis o FDA re orts. dro e: Consensus anel reco endations or clinical
Am J Psychiatry. 2003;160:790–792. anage ent and additional research. J Clin Psychiatry.
National Institute on Mental Health. Major De ression A ong 2006;67(Su l 4):27–30.
Adults. Available at htt ://www.ni h.nih.gov/health/ Wagner KD, et al. Ef cacy o sertraline in the treat ent o
statistics/ revalence/ ajor-de ression-a ong-adults.sht l. children and adolescents with ajor de ressive disorder:
Rader R, et al. Current strategies in the diagnosis and treat ent wo rando ized controlled trials. JAMA. 2003;290:
o childhood attention-de cit/hy er-activity disorder. 1033–1041.
Am Fam Physician. 2009;79:657–665. Wheeler BW, et al. T e o ulation i act on incidence o suicide
Rush AJ, et al. Acute and longer-ter outco es in de ressed and non- atal sel -har o regulatory action against the use
out atients requiring one or several treat ent ste s: A o selective serotonin reu take inhibitors in under 18 s in the
S AR D re ort. Am J Psychiatry. 2006;163:1905. United Kingdo : Ecological study. BMJ. 2008;336:542.
Nutrition and Herbal Medicine
J B hk n
24
CASE 24.1 C) Well compensated hepatitis C with mild brosis
D) Coronary artery disease (CAD)
A 59-year-old male presents or ollow-up. He is well known E) All o the above
to you, receiving chronic anticoagulation with war arin or
a mechanical aortic valve. His P and INR have been in the Answer 24.1.2 The correct answer is “A.” Vitamin K de ciency
therapeutic range or years. When asked, he denies taking can occur with chronic small bowel disease, a er small bowel
any other medications. He has had no new medical problems resection, and with use o broad-spectrum antibiotics. Micro-
and is eeling well. oday his INR is 6.2 (therapeutic range organisms in the bowel synthesize vitamin K, and use o broad-
2.5–3.5). You inquire about dietary changes, ocusing on spectrum antibiotics reduces the numbers o these organisms.
oods rich in vitamin K. T e majority o vitamin K is absorbed in the distal small bowel,
and any disease a ecting this area—including Crohn disease
Question 24.1.1 Which o the ollowing is true regarding and celiac disease—can reduce the absorption o the vitamin.
vitamin K? Irritable bowel syndrome is a unctional disease not associated
A) Vitamin K is a water-soluble vitamin with impaired absorption. “C” is o special note. Patients with
B) Broccoli and olive oil are good sources o vitamin K cirrhosis and decompensated liver disease are o en vitamin K
C) Vitamin K de ciency results in a hypercoagulable state de cient. However, the mere presence o hepatitis C does not
D) War arin reduces the absorption o vitamin K cause vitamin K de ciency.
E) Vegetarians are at risk or developing vitamin K de i-
ciency Upon more direct questioning, the patient denies any
changes in his diet but admits to recently starting a vitamin
Answer 24.1.1 The correct answer is “B.” Vitamin K is a therapy program. He has no clue what he is actually taking
at-soluble vitamin present in lea y green vegetables such as and calls his wi e to nd out what the products contain (it
spinach and cabbage and in other oods such as milk, butter, was Miracle Mega Mansomething . . .). Meanwhile, you sus-
bacon, and vegetable oils. Olive oil and broccoli are particu- pect that he is taking excessive doses o vitamins that may
larly rich in vitamin K. T ere ore, vegetarians who consume a ect his INR.
these regularly are not at high risk or developing vitamin K
de ciency. Vitamin K de ciency causes a hypocoagulable state Question 24.1.3 Large doses o which o the ollowing vita-
resulting in a reduction in clotting actors and elevated pro- mins are most likely to result in an increased INR in patients
thrombin time and INR, leading to poor clotting ability and taking war arin?
hemorrhage. War arin does act on vitamin K, but by reducing A) Vitamin A
conversion o vitamin K to its active orm rather than reducing B) Vitamin C
absorption. C) Vitamin D
D) Vitamin E
Question 24.1.2 You would be more likely to suspect vita- E) Zinc
min K de ciency in this patient i he also suf ered rom
which o the ollowing conditions? Answer 24.1.3 The correct answer is “D.” Large doses o vita-
A) Crohn disease min E (> 400 IU/day) can inter ere with vitamin K metabolism
B) Irritable bowel syndrome and platelet unction, resulting in increased prothrombin time

660
CHAPTER 24 • Nu t r it io N a Nd Her Ba l Med ic iNe 661

TABLE 24-1 COMMON VITAMINS AND HERBALS about taking a multivitamin, complaining, “Gotta do some-
THAT INTERACT WITH WARFARIN (PARTIAL LIST) thing to get my mojo back, Doc.”
War arin Interaction Vitamin/Herbal
Question 24.1.4 From available evidence, and regarding his
↑ iNr n sk b ng d nsh n mojo (whatever that is), you are able to tell him:
d ng q A) “Most multivitamins do not contain enough vitamin E to
F sh cause a problem”
G B) “Multivitamins should be standard preventive medicine and
G nkg have a well-described role in improving health”
P s n C) “Iron supplements are recommended or middle-aged
V mne males”
D) “Folic acid and B vitamins should be taken by everyone with
↓ iNr n sk b ng c nzym Q10
aortic stenosis to reduce homocysteine”
S . J hn w
V mnK
Answer 24.1.4 The correct answer is “A.” Most multivitamins
contain less than 400 IU o vitamin E and usually will not cause
a problem. A airly well-balanced diet based on whole, unpro-
and there ore increased INR in some patients. In patients taking cessed oods provides the necessary nutrients to avoid vitamin
war arin, starting a high-dose vitamin E supplement should be de ciency syndromes. Supplementation with multivitamins is
done cautiously. Extra monitoring o INR and/or war arin dose not necessary unless there are dietary intake de ciencies related
adjustments may be needed. See able 24-1 or more on supple- to an unbalanced diet.
ments that can a ect the INR. aking a multivitamin generally has not been linked to
T e other options are unlikely to a ect the INR. Vitamin A improved health status (Arch Intern Med. 2009;169(3):294–
consumed in quantities exceeding 10 times the recommended 304; J Am Geriatr Soc. 2007;55:35–42). Average adult males
daily allowance (RDA) or several months may cause alope- generally have adequate iron stores and should not receive
cia, ataxia, glossitis, and hepatotoxicity. Vitamin C is usu- supplemental iron due to the potential or exacerbating undi-
ally well tolerated but large doses can cause nausea, diarrhea, agnosed iron storage disease. While olate, vitamin B6, and
and abdominal pain. Excessive intake o vitamin D may cause vitamin B12 lower serum homocysteine levels, there is no
hypercalcemia, hypercalciuria, nausea, vomiting, myalgia, and evidence that this intervention is help ul in reducing disease
bone demineralization. Symptoms o zinc toxicity include nau- burden or improving health in those with known coronary
sea, anemia, loss o sense o smell and taste (when administered artery disease or aortic stenosis (JAMA. 2008;300(7):795–
nasally), and depressed immune unction. 804; JAMA. 2010;303(24):2486–2494). However, when ele-
vated serum homocysteine is present (especially in those who
are homozygous or the M HFR C677 gene mutation, which
HELPFUL TIP: leads to an enzyme de ect that increases homocysteine lev-
c mp w hw -s b v m ns (B n c ), -s - els), treatment with B vitamins is a reasonable intervention to
b v m ns (a , d , e, n K) m k y m - reduce homocysteine.
, s ng n x y. in n ws, a m ns
s h v h m s xp ns v n n h w
Question 24.1.5 Regarding vitamin and mineral supple -
h n m sw -s b v m n s pp m n s w
mentation, which o the statements is FALSE?
k . . . s mp y p ss h m h gh k n ys.
A) Folate supplementation during pregnancy is recommended
to decrease the risk o neural tube de ects
B) Calcium and vitamin D supplementation in postmenopausal
(VERY) HELPFUL TIP: emales is recommended to reduce the risk o osteoporosis
V mnK nb s np n sw h h n- and ractures
iNr . S v m n K s pp m n 100 200 C) Vitamin D supplementation is recommended or breast ed
µg/ y. t h s w h p mn - p n n iNr in ants, starting be ore 2 months o age
ns. D) Vitamin D supplementation in elderly patients does not
reduce the risk o alling
E) Vitamin E supplementation in elderly patients may reduce
T e patient learns that the vitamin therapy he currently takes the risk o cardiovascular disease
has large amounts o vitamin E, but no vitamin K. You coun-
sel him to discontinue this supplement, hold a dose o war a- Answer 24.1.5 The correct answer (and alse statement) is
rin, and then continue on the same dose. You ask him to come “E.” Vitamin E supplementation has not been shown to reduce
back next week or another P /INR. Be ore leaving, he asks the risk o cardiovascular disease and may be associated with
662 Fa Mil Y Med ic iNe eXa MiNa t io N & Bo a r d r eVieW

an increased risk o all-cause mortality at doses > 400 IU/day.


Additionally, high-dose vitamin E is associated with an increase CASE 24.2
in prostate cancer in men (JAMA. 2011;306(14):1549–1556). A 34-year-old emale presents to your o ce with concerns
“A” has plenty o evidence to back it up. Folate supplementa- about weight gain. She has gained over 100 lb since she grad-
tion is universally recommended during the prenatal period uated rom high school. She tear ully reveals that the weight
and should be started even be ore pregnancy actually occurs. gain has come despite “not eating much.” She walks or exer-
“B” is widely recommended, although the evidence or calcium- cise, but is unable to quanti y her walking.
reducing racture risk is weak. As or “C,” the CDC recommends She denies chronic illnesses. Her only surgery was a tubal
that breast ed in ants take 200 IU o vitamin D daily, starting ligation a er her last child. She smokes 10 cigarettes per day
be ore 2 months o age. “D”, Vitamin D, a mighty orce in medi- and is unwilling to quit due to ears o urther weight gain.
cine just a ew years ago, has allen ar. In act, it is no longer She drinks alcohol once or twice per week, never consuming
recognized as an intervention or all prevention in the elderly. more than three beers. T e review o systems is positive or a
Several negative studies have been published, and the Cochrane dry cough, bilateral knee pain, and atigue.
Collaboration concluded, “Overall, vitamin D supplementation On physical examination, you nd an a ebrile emale
does not appear to prevent alls.” with an elevated blood pressure (142/94 mm Hg). She is 5
3 in tall and weighs 231 lb. Her body mass index (BMI) is
41 kg/m2. You nd trace pitting edema at the ankles bilater-
HELPFUL TIP: ally. T ere is increased pigmentation in the olds o the neck
in g n , y m v m ns n n n g and the knuckles. T e remainder o the examination is unre-
n gh s s v m ns s n x y. markable.
T e patient realizes that she is overweight and asks, “How
bad am I?”

Objectives: Did you learn to . . .


Question 24.2.1 Regarding de nition and classi cation o
• d s b s gns n s s v mnK n y?
obesity, which o the ollowing is true?
• i n y mm n v m ns n h b s h n w h
A) Obesity is de ned as BMI ≥ 25 kg/m 2
w n?
B) Severe obesity is de ned as BMI > 30 kg/m 2
• r gn z symp ms -s b v m n x s?
C) Underweight is de ned as BMI < 20 kg/m 2
• i n y b n s, sks, n m ns v m n s pp m n-
D) Obesity is de ned as BMI ≥ 30 kg/m 2
n?
E) Malnourished supermodel status is de ned as BMI
< 30 kg/m 2
QUICK QUIZ: Su PPl eMeNt S t o Br ea St Mil K
Answer 24.2.1 The correct answer is “D.” Obesity is de ned by
A 6-month-old male in ant presents or routine health exami- BMI ≥ 30 kg/m 2; there ore, only “D” is correct. See able 24-2
nation and immunizations. His mother is a strict vegan and has or de nitions o weight status by BMI. T e calculation or BMI
been nursing him exclusively. She has not introduced any oods is as ollows:
and she wants to keep breast eeding him primarily or at least
the next 6 months . . . or until his riends start to tease him, she BMI (kg/m 2) = weight (kg)/[height (m)]2.
jokes (you hope). She takes no medicines and no supplements
o any kind. Example: weight = 110 lb (50 kg), height = 59 in (1.5 m), there-
ore
T is breast ed patient should have supplementation o all o the BMI = 50/(1.5)2 = 22.2 kg/m 2.
ollowing EXCEP :
A) Vitamin B12
B) Vitamin C
C) Vitamin D TABLE 24-2 DEFINITION OF WEIGHT STATUS BY BMI
D) Iron Weight Status BMI (kg/m 2)
un w gh < 18.5
The correct answer is “B.” Children breast ed by strict vegan
mothers should have vitamin B12 supplementation. As above, N m 18.5–24.9
vitamin D is recommended or all breast ed in ants. Iron is rec-
o v w gh 25.0–29.9
ommended in all breast ed in ants starting at 4 months o age
until iron-containing oods become a routine part o their diet ob s c ss i 30.0–34.9
(supplemental dose is 1 mg/kg/day). Vitamin C supplementa- c ss ii 35.0–39.9
tion is not required. T ere are a variety o liquid multiple vita-
ex m y b s c ss iii ≥ 40.0
mins to choose rom (e.g., Poly-Vi-Sol® ).
CHAPTER 24 • Nu t r it io N a Nd Her Ba l Med ic iNe 663

Question 24.2.3 Which o the ollowing is the most appro-


HELPFUL TIP:
priate next step in the evaluation o this patient’s weight
in a m n s, h mb n p v n b s y
gain?
n v w gh (BMi ≥ 25 kg/m 2) s 60% m n n
A) Re er or a sleep study
51% w m n. a b 35% u .S. s b s .
B) Check urinary ree cortisol level
N w h ’s wh w n p m !
C) Draw blood or thyroid-stimulating hormone level
D) Evaluation or adrenal adenoma causing Cushing disease
HELPFUL TIP:
th h s b n n v sy g ng wh h h - Answer 24.2.3 The correct answer is “C.” Because o her
n BMi - s pp b ss n hn obesity, this patient is also at risk or sleep apnea, diabetes,
p p ns s h h s b n v n h b y p - hypertension, and hyperlipidemia. T ese concerns will need
n g , s b n b y (w s -h p ) n BMi to be addressed. However, her chie complaint is weight gain.
m y w hn n hn g ps (Obstet Gynecol. Although in most overweight patients a cause or weight gain
2010;115(5):982–988). F h , h s b n n n is not ound, the physician is obligated to search or potentially
h w s m n n w s –h p m y b treatable causes o weight gain, including hypothyroidism. T e
s p BMi n p ng v s s s sk, symptoms o hypothyroidism are o en nonspeci c and include
sn n s vs p s ss s ss w h weight gain and atigue.
ns n s s n n m b syn m , wh h sk
s yp 2 b s n v s s s . HELPFUL TIP:
c mp ns b s y n h s s , yp 2
b s, hyp ns n, s k , hyp p m , g -
You ask her about medications, ocusing on those related to b s s , y, n s sk -
obesity. n n s (p s , n, b s , n m ), n
m n s ss.
Question 24.2.2 All o the ollowing drugs are associated
with weight gain and an increased risk o obesity EXCEPT:
Question 24.2.4 The ind ing o acanthosis nigricans in
A) Olanzapine
this p atient raises concern or which o the ollowing
B) Met ormin
d iseases?
C) Amitriptyline
A) Skin cancer
D) Valproic acid
B) Diabetes
E) Glipizide
C) Hemochromatosis
D) Colon cancer
Answer 24.2.2 The correct answer is “B.” Met ormin may help
E) Hypertriglyceridemia
patients with diabetes lose or maintain weight and, there ore, is
not associated with worsening obesity. See able 24-3 or a list
Answer 24.2.4 The correct answer is “B.” Acanthosis nigri-
o medications associated with weight gain.
cans, classically described as velvety hyperpigmentation in
skin creases (axilla, back, neck, exor aspects o arms), is
TABLE 24-3 DRUGS ASSOCIATED WITH WEIGHT associated with a number o diseases. Acanthosis nigricans
GAIN (NOT AN EXHAUSTIVE LIST) occurs in insulin-resistance syndromes, commonly in patients
Class Speci c Agents with type 2 diabetes. It is also associated with other endocrine
abnormalities, including Cushing disease and hypothyroid-
a ph B k s P z s n, x z s n, z sn ism. Acanthosis nigricans can be an external sign o internal
an nv s n s V p , b m z p n ,g b p n n malignancy, most commonly gastrointestinal (GI) cancers.
However, this patient is at lower risk o colon cancer than dia-
an p ss n s t y s ( .g., m p y n ) n m n m n betes given her age, weight gain, and lack o other signs and
x s nh b s (Ma o -is), s v s nn
p k nh b s (SSr is) n s m p ns
symptoms o cancer. Hemochromatosis, hypertriglyceridemia,
and skin cancer are not directly associated with acanthosis
an h s m n s c yp h p n , ph nhy mn nigricans.
a n psy h s o nz p n , h zn ,h p ,
sp n She wonders which popular diet she should use to lose weight.

B b k s P p n ,m p , n Question 24.2.5 You let her know that the most ef ective
d b s gs ins n, s ny s, h z n n s popular diet is:
A) Low carbohydrate (“Atkins”)
S c s s, n p v s, s g n, B) Low at (“Ornish”)
h m n s p g s n , s s n
C) Low glycemic load diet (“Zone diet”)
664 Fa Mil Y Med ic iNe eXa MiNa t io N & Bo a r d r eVieW

D) Portion restriction such as Weight Watchers Question 24.2.7 Which o the ollowing is the most appro-
E) No one particular diet is more e ective than others or weight priate next step?
loss. T e key to success is choosing a diet that you can stick to A) Prescribe L-thyroxine
B) Prescribe methylphenidate (Ritalin)
Answer 24.2.5 The correct answer is “E.” Any diet you can C) Prescribe orlistat (Xenical, Alli)
stick to seems to work. T ere does not seem to be any advantage D) Prescribe paroxetine (Paxil)
to one diet over another or weight loss (except or making the
proprietors wealthy; JAMA. 2005;293(1):43–53). For example, Answer 24.2.7 The correct answer is “C.” Drug therapy is
Weight Watchers may have a bit better success because o peer considered appropriate as an add-on to li estyle management or
pressure, weekly meetings, motivational techniques, etc. (BMJ. people with a BMI o 30 kg/m 2 or greater, or a BMI o 27 kg/m 2
2011;343:d6500). But any diet that you can stick to long-term or more who already have a comorbid condition such as dia-
will work. O note, the common belie that maintaining a 3,500 betes, hypertension, or hyperlipidemia. O the choices listed,
calorie de cit will result in 1 lb (0.45 kg) o weight loss per week orlistat is the most appropriate (more below). “A” and “B” are
is a myth. But reducing calorie intake by 100 calories per day incorrect, as obesity alone is not an indication or L-thyroxine
may lead to a gradual weight loss o approximately 10 lb over 1 or methylphenidate. “D” is incorrect because paroxetine may
to 3 years (Am Fam Physician. 2015;91(9):634–638). be associated with weight gain and is not indicated or obesity.

She is wondering about her level o physical activity and is Question 24.2.8 Which o the ollowing weight loss drugs
a raid that her obesity will decrease her ability to become results in GI side ef ects in up to 40% o patients using it?
unctionally t with exercise. A) Orlistat (Alli, Xenical)
B) opiramate ( opomax)
Question 24.2.6 Your response to this is that: C) Phentermine (Adipex-P, Ionamin)
A) Gluttony and sloth go together, so give up the exercise D) Leptin
B) One cannot improve one’s physical per ormance without
weight loss Answer 24.2.8 The correct answer is “A.” Orlistat is a lipase
C) Dante did not know what he was talking about. It is V and inhibitor that unctions in the intestines and prevents the
sloth that go together absorption o about 30% o dietary ats. Because o its mecha-
D) Exercise is bene cial or tness regardless o whether or not nism o action, orlistat is associated with GI side e ects in 40%
one loses weight o patients taking the drug. T e most bothersome potential
adverse events include ecal incontinence and abdominal pain.
Answer 24.2.6 The correct answer is “D.” Losing weight is noto- Orlistat should be taken with meals. T e prescription dose is
riously hard to do. T ank ully, exercise, in and o itsel , increases 120 mg three times per day. Weight loss is modest: 3 to 4 kg
unctional capacity regardless o weight loss (N Engl J Med. over 1 year compared with placebo. opiramate alone is not
2011;364(13):1218–1229). Obviously, patients do better i they approved by the FDA or use as a weight loss drug, but the com-
both exercise and lose weight. But sticking to an exercise regimen is bination phentermine/topiramate (Qsymia) is. Phentermine is a
still help ul. With regard to the others, there was no evidence-based sympathomimetic and causes hypertension, tachycardia, etc. It
medicine or V during the time o Dante, so we will never know. should only be used or 12 weeks. Leptin has been used success-
ully but is not FDA approved or weight loss.
HELPFUL TIP:
a w-g y m n x (v g b s, h k n, sh,
wh g ns, b ns, n s, .) n w Hba 1c n yp
HELPFUL TIP:
2 b s wh n mp w h h gh wh -g n
th otc s s (a ) s h h s h p -
b hy (b wn , wh wh s, .).
s p n s ng h. H w v , s q y v hs
Psy m (M m n h s) n s h p
w s .
Hba 1c . a P h b s n m , sh, s, v g-
b s, v g b s, ggs, n n s s s n
w m nv s Hba 1c , s b p ss ,
w gh , BMi, n w s m n v 3-m n h
p wh n mp n am n d b s a ss - You initiate a medication to help her lose weight and re er
n (Cardiovasc Diabetol. 2009;8:35). her to a weight loss program that ocuses on overall li estyle
modi cation. When she returns in 3 months, her weight is
235 lb, and she is very rustrated.
You nd that the patient’s thyroid unction is normal. A er
consultation with a nutritionist and a prescription or exer- Question 24.2.9 You appropriately recommend which o
cise, she returns 3 months later. Her weight is 222 lb. ear- the ollowing?
ully, she tells you that she barely eats and needs something A) Adding a second drug or weight loss
more to help her lose weight. B) Re erral to psychiatry
CHAPTER 24 • Nu t r it io N a Nd Her Ba l Med ic iNe 665

C) Re erral to bariatric surgery include Wernicke encephalopathy (ataxia, occular motor ab-
D) Re erral to plastic surgery normalities [e.g., gaze paralysis], mental status changes, dietary
de ciency) and Wernicke–Korsako syndrome (Wernicke syn-
Answer 24.2.9 The correct answer is “C.” T is patient now drome plus additional memory loss and con abulations). Note
meets accepted indications or bariatric surgery. See Chapter 22 that most patients do not have all o the ndings. Most have a
or more in ormation about weight loss surgery. partial syndrome. reatment is to administer thiamine IV and
then to continue oral supplementation.
HELPFUL TIP:
d g h py n s g y n s bs b s HELPFUL TIP:
sy m ns, n ng pp p y th s g h mn mns m n n
m ns n g b x s .th s - p v n n W n k n ph p hy s. F
sy pp h s sh b n n ng g p v n n W n k n ph p hy, g v h m n
m n n bs y ss n s g y. 100 mg y. F m n W n k n ph p -
hy, g v h m n 500 mg iV Bid – t id 2 ys -
w by 500 mg iV y 5m ys ( h gh -
Objectives: Did you learn to . . .
mm n ns v y n n n mz s s pp
• d n n m w gh , v w gh , n b s y?
n gm n v n h ).
• a pp p y v h b s p n?
• r gn z g s n m h s ng b s y?
• r gn z mp ns h s s b s y? HELPFUL TIP:
o h m n n n s h n h
p ns n n n n y s ng n p g
QUICK QUIZ: Vit a MiN d eFic ieNc ieS ( m s, h , n m n ); n y
s ng n m y n m ; v m n B12 n y
A 63-year-old male presents to the of ce a er his daughter (g n y p b ms w h Gi bs p n h
came home or the holidays and ound him con used. At rst h n s h sm) s ng n n m ,
she thought he was drunk, but a er a ew days, the con usion n p hy, n m n .a m v mn s ny
persisted. He has never had problems with complicated alcohol s n b n h sp n p p n.
withdrawal be ore. According to the daughter, the patient drinks
about hal o a bottle o vodka per day and has done so or years
(Hawkeye Vodka is the Iowa avorite . . . about $12.00/1.75 L).
Although the patient is not able to give much o a coherent his-
CASE 24.3
tory, you are able to determine that he has developed shortness A 58-year-old postmenopausal emale presents to your o ce
o breath and leg edema. His appetite has been poor or “a long with concerns about osteoporosis. A review o her medical
time” and he may have lost some weight although he is not sure. history shows a bone density -score o –1.8 rom 2 years ago,
Physical examination reveals a malnourished, disoriented suggesting moderate osteopenia. As you review her medi-
male who appears older than his stated age and is disheveled. cal record, you note that last year she had lipids measured
He is mildly tachycardic, but his other vital signs are normal. He which were: total cholesterol o 210 mg/dL, LDL 140 mg/dL
has bibasilar crackles on lung examination, 2+ pedal edema, and HDL 45 mg/dL, and triglycerides 125 mg/dL. She had a
hepatomegaly, and spider telangiectasias. partial thyroidectomy 15 years ago and currently takes levo-
thyroxine 88 µ g/day.
What is your rst action to take with this patient?
A) Administer D50 intravenous (IV) Question 24.3.1 Based on the evidence, which o the ollow-
B) Administer thiamine IV ing supplements is most appropriate or this patient?
C) Administer vitamin B12 IM A) Calcium alone
D) Administer niacin orally B) Calcium and vitamin D
E) Administer olate orally C) Vitamin D alone
D) Ipri avone
The correct answer is “B.” T is patient almost certainly has
thiamine (vitamin B1) de ciency and appears to be symptom- Answer 24.3.1 The correct answer is “B.” Postmenopausal
atic. In the United States, thiamine de ciency is most o en seen women should take 1,200 mg/day o elemental calcium with total
in malnourished alcoholics. Early symptoms o thiamine de - o diet plus supplements not exceeding 2,000 mg o elemental cal-
ciency are vague (similar to how the author/editors wrote the cium per day. T e recommendations or supplemental calcium
case) and include anorexia, atigue, and irritability. T iamine are muddied a bit by the observation that higher calcium supple-
de cient patients may progress to beriberi (wet or dry), possess- ment intake is associated with greater risk o CAD. Vitamin D is
ing symptoms o heart ailure and/or neuropathy (motor and also appropriate and many women do not get enough. As supple-
sensory). Other possible mani estations o thiamine de ciency ments go, vitamin D may be more important than calcium.
666 Fa Mil Y Med ic iNe eXa MiNa t io N & Bo a r d r eVieW

In terms o vitamin D, recommended doses are 400 IU/day shown to reduce lipid levels. Soy protein is not nearly as e ec-
or in ants, 600 IU/day or ages 1 to 70, and 800 IU/day in those tive as are statin drugs (e.g., atorvastatin and simvastatin). Soy
over age 70 (Institute o Medicine, 2011). T e guideline also protein or soy iso avone supplements are not an appropriate
concludes that there is not any good evidence or vitamin D alternative to calcium and vitamin D.
beyond its use or bone health.
Ipri avone is a semisynthetic iso avone produced rom soy Your patient is hoping you can settle a bet. She asks about her
iso avones. T ere is evidence that it can prevent bone mineral younger sister who is going through “the change” (quarters,
density loss when used with calcium in postmenopausal women, nickels, pennies . . .) and started taking black cohosh. Her sis-
but it has not been shown to reduce racture rates. T ere are also ter swears the black cohosh helps her hot f ashes, but your
concerns that it might cause lymphocytopenia in some patients. patient thinks it’s “all in her head.”

Question 24.3.4 Regarding the use o black cohosh or the


HELPFUL TIP: treatment o perimenopausal vasomotor symptoms, you
i y n n b p v n n s p s s, tell her:
s y, y y. c ns s n m s A) Black cohosh is equivalent to hormone replacement therapy
g s 1,300 mg m n m n 600 iu B) Black cohosh is equivalent to placebo
v mnd y. C) Black cohosh is not good or vasomotor symptoms but does
promote liver health
D) Black cohosh is only e ective when taken as part o a multi-
You counsel her to take 1,200 mg o elemental calcium per day. botanical supplement

Question 24.3.2 In order to improve GI absorption o cal- Answer 24.3.4 The correct answer is “B.” Most studies o black
cium, you recommend: cohosh are not well done. A Cochrane Collaboration review in
A) aking 1,200 mg at one time with a meal 2012 ound insuf cient evidence or black cohosh or meno-
B) aking 1,200 mg at one time an hour be ore eating pausal symptoms. T e arguments against its use are the rare
C) aking 1,200 mg in three divided doses with meals cases o hepatotoxicity, so “C” is wrong.
D) aking 1,200 mg in three divided doses on an empty stomach
E) aking 1,200 mg with her levothyroxine
HELPFUL TIP:
Answer 24.3.2 The correct answer is “C.” Calcium is best S yp n n g n h sg n y g-
absorbed when taken with ood. T eoretically, GI absorption o nz ss n s p b w m n wh w n
calcium is limited, and or that reason divided dosing in 500 to y .B sh n p p v n h p s. S y s
600 mg aliquots is recommended. Patients should take levothy- mm n g n, n sh b v by h s
roxine separately rom their calcium. I taken together, calcium w h gy s ns v y s y. S t b 24-4 n
reduces levothyroxine absorption. T is is also true o iron and v v w s y b n s.
levothyroxine.

She then asks you about her cholesterol levels and whether HELPFUL TIP:
taking soy would be help ul. A riend told her that older Wh h h b p s k phy s g ns, w
women should take soy to prevent heart disease, breast can- y m, p m s , n v mne m y n -
cer, osteoporosis—basically, they told her soy will help her to ng m n p s symp ms, g s y s gn n
cheat death. sp m h v n n m h -
y.
Question 24.3.3 Which o the ollowing is appropriate to
tell this patient?
A) Soy iso avone supplements are pre erred or reducing lipid Question 24.3.5 Which o the ollowing natural products
levels could be recommended or lowering LDL cholesterol?
B) Dietary soy protein is not associated with a reduced risk o A) Fish oil
developing heart disease in Western women B) Psyllium
C) Soy protein is as e ective as a statin or lowering cholesterol C) Garlic
D) Soy iso avones are an acceptable alternative to calcium and D) Policosanol
vitamin D or preventing osteoporosis E) All o the above

Answer 24.3.3 The correct answer is “B.” Soy protein can mod- Answer 24.3.5 The correct answer is “B.” aking a psyllium
estly reduce lipid levels in some patients but has been shown supplement (e.g., Metamucil) 10 to 12 g daily can reduce total
not to a ect important outcomes such as heart attack or death cholesterol by up to 14% and LDL by up to 10%. Fish oil is
in Western women. Soy iso avone supplements have not been e ective or lowering triglycerides, but not total cholesterol
CHAPTER 24 • Nu t r it io N a Nd Her Ba l Med ic iNe 667

TABLE 24-4 OVERVIEW OF THE EFFECTIVENESS OF SOY


Condition Ef ectiveness Data Comment/Recommendation
B s n H gh n k s y n h s ss w h s sk u nkn wn b n s n W s n p p ns. d
b s n .B m s s h m as n p p ns. M y n n s ng n gh mm n ys y
pp y W s n p p ns. N n s. hs s .

Hyp p m / c ns m ng s y p n np h p ns s m gh m s y S bs ng ys y h p ns s
v s p v s. d s n pp y s y s v n s pp m n s. N p b ,b n ym s b n xp ;n
s s v n h ng s y mp v s m ss h s h h n n v m y v s v n-
k. P p n s h sh ws n s sk h s m s ng h p s s h s s ns.
n W s n w m n.

os p ss c ns m ng ys yp n m gh mp v b n m n ns y n th s p ss b b n ng s y p n
p s m n p s w m n n p ss b y sk; h w v , n h ,b b n s k y b m s b s.
v n s ns s n . S y sn s bs m n v m n d.

or LDL cholesterol. Clinical outcomes do not seem to change Answer 24.3.7 The correct answer is “E.” Many patients use
with the use o sh oil and it has allen by the wayside. Garlic alternative therapies including herbal supplements or vari-
was long considered e ective or modest reduction in choles- ous reasons, some o which are listed above. Other reasons
terol, but the evidence shows that garlic does not signi cantly cited include perceived physician apathy, di iculty with
reduce cholesterol in most people. Similarly, policosanol was physician access, ear o medication side e ects, belie that
once considered e ective or reducing cholesterol, but evi- medications lack e icacy, and the act that they are not “nat-
dence is inconsistent. T e most reliable evidence shows that ural.” Many patients do not in orm their physicians about
it is ine ective. their use o these products, mistakenly believing that “nat-
ural” means sa e. As or “E,” un ortunately there is a large
Question 24.3.6 What percentage o Americans use alter- demand or shark cartilage including its use as chondroitin
native therapies? sul ate. Shark populations have been severely stressed as a
A) 10% result. Up to 100 million sharks per year are killed or their
B) 20% ins and cartilage.
C) 30%
D) 40%
Question 24.3.8 What is the current status o the regulation
E) 50%
o “natural” therapies?
A) T e FDA does not have a regulatory role with regard to nat-
Answer 24.3.6 The correct answer is “D.” T irty-eight per-
ural therapies
cent o adults and 12% o children use complementary or
B) Natural products have to be proven sa e and e ective in
alternative therapy (Natl Health Stat Report. 2015;79:1–15).
order to be marketed in the United States
Only 40% o patients in ormed their physicians o their use
C) Natural therapies are regulated by the Department o Health
o these therapies. Examples o unconventional therapies used
and Human Services
include:
D) As long as they contain the ingredients claimed, natural
Relaxation Sel -help groups Homeopathy Spiritual healing therapies can be marketed in the United States
Chiropractic Bio eedback Li estyle diets Acupuncture E) Natural therapies are regulated by Pan, the Greek god o na-
Massage Energy healing Herbal medicine Exercise ture
Imagery Hypnosis Megavitamins Prayer

At least 15 million patients take prescription medications Answer 24.3.8 The correct answer is “A.” T e FDA has no reg-
along with their alternative medications, sometimes with ulatory role with regard to natural therapies. T ey are classi ed
adverse outcomes. as dietary supplements and not medications. All o the rest are
incorrect. “B” is incorrect because natural therapies need not
Question 24.3.7 Which o these is NOT a reason that patients be proven sa e and e ective in order to be marketed. “D” is
might use alternative therapies? incorrect. T ere is no quality control on “natural” therapies in
A) T ey believe they are sa er than medications the United States. Products may contain varying amounts o
B) Conventional medicine is too technical or impersonal the advertised therapy or, in some cases, none at all. In act,
C) Prescription medicines are too expensive there are data suggesting that some products or sale do not
D) Cultural practices contain what they advertise and in other cases, contain pre-
E) T ey didn’t like the shark in Jaws and there ore use shark scription drugs such as war arin, steroids, alprazolam, and
cartilage in an attempt to rid the oceans o sharks diethylstilbestrol.
668 Fa Mil Y Med ic iNe eXa MiNa t io N & Bo a r d r eVieW

C) Glucosamine
HELPFUL TIP:
D) None o the above
t h Fd a n m v n h py p s n
sh w h h y h z s. t h s s s w n b -
Answer 24.3.10 The correct answer is “A.” Ginkgo biloba is one
sp ss s v n by h w ng k m v
o the most popular herbal products and is promoted or mild
ph m h u .S. m k .
memory loss, dementia, and peripheral circulatory disorders. As
ap s ns n v m ss s ws:
noted above, its e ectiveness has not been proven or either
• H p x :ch p ,g m n , use. Ginkgo can have antiplatelet e ects so should be used with
• c n g n :B g , m s, s , m y, caution or not at all in patients taking aspirin. Side e ects include
, n s ss s GI disturbances, headaches, and dizziness. “B,” SAM-e, is used
• Ms n s x y: M H ng ( ph ), p k or arthritis and depression and has been shown to be “likely
e ective.” “C,” glucosamine, has NO been shown to be e ective
or osteoarthritis. Most o the data is o poor quality.
HELPFUL TIP:
Question 24.3.11 Which herbal product has aldosterone -
P n s s ng h b s sh b w n h h p -
like properties and can cause a pseudohyperaldosteron-
sm yn n n wh h y m n h h y
ism?
sh k h p s h wn sk.
A) Ma Huang
B) Ginseng
HELPFUL TIP: C) Licorice
th m g 3 y s n v s s s D) Melatonin
(c Vd ) s n v s . t h b s s s sh w n b n
n h s w h p - x s ng c Vd . t h n p m yp - Answer 24.3.11 The correct answer is “C.” Licorice (Glycyr-
v n n s ss w s b sh (BMJ. 2010;341: 6273). rhiza spp.) has aldosterone-like e ects and can lead to uid
G nkg h s s m n p n s retention, hypertension, and hypokalemia. T us, it should
h p ng w h v s s s n m n , h gh not be combined with other potassium-wasting drugs such as
p s k ng (JAMA. 2009 d 23;302(24):2663–2670). non-potassium sparing diuretics. It is also contraindicated in
patients with severe liver disease and in pregnant patients (may
induce premature labor).
HELPFUL TIP:
V n h s b nz z pn - k s n w ks Question 24.3.12 For which o the ollowing diseases is
s p n y. SAM-e “likely ef ective?”
A) Depression
B) Fibromyalgia
Question 24.3.9 Grape ruit juice interacts with multiple C) Cirrhosis
medications. Which o the ollowing IS NOT af ected by D) Osteoarthritis
grape ruit juice? E) Both A and D
A) acrolimus
B) Itraconazole/Ketoconazole Answer 24.3.12 The correct answer is “E.” SAM-e is “likely
C) Benzodiazepines e ective” in treating depression and osteoarthritis. It is “possi-
D) Clopidogrel bly e ective” in treating bromyalgia, and there is insuf cient
E) Aspirin evidence to rate its use in cirrhosis. SAM-e is contraindicated
in bipolar patients as it can induce mania. SAM-e can possibly
Answer 24.3.9 The correct answer is “E.” All o the other interact with antidepressants, including MAOIs, leading to sero-
drugs interact with grape ruit juice. Grape ruit juice is a potent tonin syndrome. GI disturbance is the only notable side e ect.
CYP3A4 inhibitor and can interact with numerous medications
including those listed above, some calcium channel blockers, Question 24.3.13 Which o these is use ul in the treatment
carbamazepine, benzodiazepines, amiodarone, atorvastatin, o migraine headaches?
lovastatin, simvastatin, and others. T is underscores the impor- A) St. John wort
tance o knowing what “alternative” medications your patient B) Valerian root
might be using. C) Ginger
D) Fever ew
Question 24.3.10 Which o the ollowing supplements has E) Saw palmetto
been touted as being ef ective or memory problems and
peripheral circulatory problems? Answer 24.3.13 The correct answer is “D.” Fever ew is “pos-
A) Ginkgo biloba sibly e ective” in treating migraine headaches (Neurology.
B) S-Adenosylmethionine (SAM-e) 2012;78(17):1346–1353). “A,” St. John wort, is use ul or
CHAPTER 24 • Nu t r it io N a Nd Her Ba l Med ic iNe 669

depression. However, there are major interactions between The correct answer is “D.” T e three choices listed above have
St. John wort and other multiple drugs including: cyclospo- the reputation o helping alleviate the symptoms o the com-
rine, nevirapine, and digoxin among others. “B,” valerian root, mon cold. T e ef cacy o all o these is questionable, however.
is use ul or insomnia. Ginger is used or nausea and seems to be Intranasal zinc has been removed rom the market in the Unit-
e ective. Saw Palmetto is used or benign prostatic hypertrophy ed States due to reports o permanent loss o sense o taste and
but is ine ective. smell a er its use.

Question 24.3.14 Which o these is NOT potentially use ul CASE 24.4


or women with pregnancy-related nausea and vomiting?
A) Ginger A 17-year-old male patient o yours comes to your o ce or
B) Acupressure a sports pre-participation physical. He is a little smaller than
C) Vitamin B6 most o his classmates and has heard that creatine supple-
D) Doxylamine mentation can help him increase muscle mass and improve
E) Black cohosh his per ormance.

Answer 24.3.14 The correct answer is “E.” Ginger is “pos- Question 24.4.1 Which o the ollowing is NOT true about
sibly e ective” in treating nausea and vomiting in pregnancy, creatine?
postoperative vomiting, and vertigo. Vitamin B6 (pyridoxine) A) Creatine exists primarily in skeletal muscle
and doxylamine, an antihistamine, have been shown singly and B) It causes weight gain by increasing muscle mass
together to be sa e and e ective in pregnancy-related nausea C) It is ine ective in boosting per ormance in aerobic exercise
and vomiting. Acupressure has no known adverse e ects, and D) It can lead to increased creatinine levels in patients with nor-
there is some suggestion o ef cacy. Black cohosh can stimulate mal renal unction
uterine contractions, and has no known ef cacy or nausea and
vomiting in pregnancy, and so is contraindicated in pregnancy. Answer 24.4.1 The correct answer is “B.” Creatine monohy-
drate is a naturally occurring protein in the body that exists pri-
Objectives: Did you learn to . . . marily in skeletal muscle. High levels o creatine are thought to
• d s b h p v n n v h py s ?
enhance the ability to renew A P or short bursts o energy. It
• a pp h v s n ns v sh b p - appears to be e ective or enhancing muscle per ormance dur-
s n h v n b s h m?
ing repeated bouts o brie , high-intensity exercise, but ine ec-
• r gn z h bs ns s n h s ns tive or other types o exercise. It does not improve per ormance
ns ?
in aerobic exercise nor does it bene t older adults seeking to
• r gn z p b m g n ns w h v y build muscle mass. It also does not appear e ective or increas-
n v h p s?
ing endurance or or improving per ormance in highly trained
athletes, but its use is widespread among athletes, both amateur
and pro essional. It causes weight gain by increasing water reten-
HELPFUL TIP: tion and not by a ecting muscle mass. Creatine can cause elevated
d y kn w h m m n, PPis, n H2 b k s n creatinine levels in patients with normal renal unction as cre-
s B12 n y? N w y . atine is metabolized to creatinine. Complicating this is the act
that creatine has been linked to renal dys unction in some cases.

Question 24.4.2 Which o these is NOT a disease that cre -


QUICK QUIZ: a l t er Nat iVe c u r e Fo r atine is purported to treat?
t He c o MMo N c o l d ? A) Heart ailure
B) Neuromuscular disease
One o your patients who o en uses complementary and alter- C) Mitochondrial cytopathies
native medicine comes into your of ce with several days o rhi- D) Muscular dystrophies
norrhea, sore throat, and ear pain. Your examination reveals a E) Diabetes
viral upper respiratory in ection. He asks you to recommend an
alternative cure. Answer 24.4.2 The correct answer is “E.” Creatine is also pro-
moted or heart ailure, neuromuscular diseases, mitochondrial
Which o these are touted as alternative remedies or the com- cytopathies, and various muscular dystrophies. Oral creatine
mon cold? may improve exercise tolerance in patients with heart ailure but
A) Echinacea has no e ect on ejection raction. IV creatine seems to improve
B) Zinc ejection raction temporarily. When used orally, it seems to
C) Vitamin C marginally improve muscle strength and daily-li e activity in
D) All o the above adults and children with various muscular dystrophies in the
E) None o the above short term. T ere is no evidence o its ef cacy in the treatment
670 Fa Mil Y Med ic iNe eXa MiNa t io N & Bo a r d r eVieW

o diabetes. It appears to be sa e when used orally and in appro-


priate doses, though high doses raise the concern o adverse QUICK QUIZ: a Bl o o d YHer B
hepatic, renal, or cardiac unction. Side e ects include GI pain,
nausea, and diarrhea, while college athletes taking it requently A 51-year-old patient o yours is about to have a scheduled cho-
complain o muscle cramping. lecystectomy. He is in otherwise good health and is taking the
ollowing herbs and supplements.
Objectives: Did you learn to . . .
• d s b h s n v s f s n ?
Which would you recommend him to stop be ore surgery
because o association with prolonged bleeding time?
A) Valerian root
QUICK QUIZ: iNFo Mer c ia l “c u r e” B) Ginseng
Fo r KNee Pa iN C) Vitamin B complex
D) Echinacea
A 51-year-old emale patient o yours with knee osteoarthritis E) Ginkgo
comes to your of ce because she has seen commercials on V
advertising a product containing glucosamine and chondroi- The correct answer is “E.” Ginkgo is the only one above that
tin sul ate, which was touted as being e ective or osteoarthri- has been associated with a prolonged bleeding time. T us, it
tis. She has been using naproxen with symptom relie but has should be stopped prior to planned surgery. Other supplements
had heartburn and is worried about the potential or bleeding. that have antiplatelet e ects include garlic, ever ew, vitamin E
Moreover, she likes the idea o using something “natural.” She (at doses o greater than 800 IU/day), and sh oil.
asks or your advice.

Which o the ollowing is true about the use o glucosamine sul-


Clinical Pearls
ate and/or chondroitin sul ate or osteoarthritis?
A) T e combination is more e ective than either product alone a b s- n n s sh v s pp m n v m n
B) Glucosamine is e ective in improving symptoms o osteoar- d w hn h s w ys n s pp m n n
thritis 4 m n hs n ng n- n n ng s.
C) Glucosamine may lower blood sugars a w m n ns ng b m ng p gn n sh k
D) Chondroitin can help patients with coagulation disorders p n v mn s pp m n .
E) Chondroitin is “possibly e ective” or osteoarthritis d n mm n m h n 400 iu p y v m n e.
d n n y mm n m v m ns p ns
The correct answer is “E.” Glucosamine sul ate is a glycoprotein g n h h.
that occurs naturally in the body but is available commercially
r gn z h 40% p ns s mp m n y n
as a synthetic product or rom marine exoskeletons. It is not e -
n v h p s, n n n n m -
ective in osteoarthritis. Side e ects are generally mild and GI in
m n s, n h h Fd a sn g n
nature including nausea, heartburn, diarrhea, and constipation. h p s.
Patients at risk or diabetes, hyperlipidemia, and hyperten-
r gn z h h sm yh v m p n n -
sion should use this product cautiously as glucosamine can
n s, n ng , h m n , Bv m ns, n v m n K
increase both blood glucose and insulin levels. It can increase
n n p n n y.
blood sugars by impeding glucose-induced insulin secretion
and impairing the insulin-induced glucose uptake by skeletal
muscle.
Chondroitin sul ate is a glycosaminoglycan made rom ani- BIBLIOGRAPHY
mal and sh cartilage. It is “possibly e ective” in osteoarthritis,
although there are both positive and negative studies. When Barnes PM, Bloom B. Complementary and alternative medi-
cine use among adults and children: United States, 2007.
only the best quality studies are included, there seems to be no
Natl Health Stat Report. 2008;12:1–23.
benef t to chondroitin sul ate. It is a minor component o the
Barry MJ, et al. E ect o increasing doses o saw palmetto ex-
low–molecular-weight heparin, danaparoid, so there is the con-
tract on lower urinary tract symptoms: a randomized trial.
cern o possible anticoagulant activity. T is has not been shown JAMA. 2011;306(12):1344–1351.
in studies but because o this concern, it should be used with
Bischo -Ferrari HA, Dawson-Hughes B, Staehlin HB, et al. Fall
caution (or not at all) in patients on antiplatelet or anticoagulant prevention with supplemental and active orms o vitamin D:
agents or those with bleeding disorders. It is generally well toler- a meta-analysis o randomised controlled trials. BMJ.
ated but can cause epigastric pain and nausea in some patients. 2009;339:b3692.
Chondroitin dosing is typically 200 to 400 mg two to three times Bolland MJ, et al. Calcium supplements with or without vita-
daily. Although chondroitin sul ate and glucosamine sul ate are min D and risk o cardiovascular events: reanalysis o the
requently sold together in combination products, no evidence Women’s Health Initiative limited access dataset and meta-
supports the notion that both are better than either alone. analysis. BMJ. 2011;342:d2040.
CHAPTER 24 • Nu t r it io N a Nd Her Ba l Med ic iNe 671

Chaiyakunapruk N, et al. T e ef cacy o ginger or the Neuhouser ML, et al. Multivitamin use and risk o cancer
prevention o postoperative nausea and vomiting: a and cardiovascular disease in women’s health initiative
meta-analysis. Am J Obstet Gynecol. 2006;194(1): cohorts. Arch Intern Med. 2009;169(3):294–304.
95–99. Newton KM, et al. reatment o vasomotor symptoms o
Clarke C, et al. rends in the use o complementary health menopause with black cohosh, multibotanicals, soy,
approaches among adults: United States, 2002–2012. hormone therapy, or placebo: a randomized trial. Ann
Natl Health Stat Report. 2015;79:1–15. Intern Med. 2006;145(12):869–879.
Ford SK, Moll S. Vitamin k supplementation to decrease Rahman M, Berenson AB. Accuracy o current body
variability o international normalized ratios in patients mass index obesity classi cation or white, black, and
on vitamin k antagonists: a literature review. Curr Opin Hispanic reproductive-age women. Obstet Gynecol.
Hematol. 2008;15(5):504–508. 2010;115(5):982–988.
Holick MF, et al. Evaluation, treatment, and prevention Reed SD, et al. Vaginal, endometrial, and reproductive hor-
o vitamin D de ciency: an Endocrine Society mone ndings: randomized, placebo-controlled trial o
clinical practice guideline. J Clin Endocrinol Metab. black cohosh, multibotanical herbs, and dietary soy or
2011;96(7):1911–1930. vasomotor symptoms: the Herbal Alternatives or Meno-
Institute o Medicine Committee to Review Dietary Re er- pause (HAL ) Study. Menopause. 2008;15(1):51–58.
ence Intakes or Vitamin D and Calcium, Ross AC, aylor Snitz BE, et al. Ginkgo biloba or preventing cognitive
CL, Yaktine AL, et al., eds. Dietary Re erence Intakes or decline in older adults: a randomized trial. JAMA.
Calcium and Vitamin D. Washington, DC: T e National 2009;302(24):2663–2670.
Academies Press; 2011. Study o the E ectiveness o Additional Reductions in Choles-
Jellin JM, Gregory PJ, eds. Natural Medicines Comprehensive terol and Homocysteine (SEARCH) Collaborative Group,
Database. Stockton, CA: T erapeutic Research Faculty; Armitage JM, Bowman L, et al. E ects o homocysteine-low-
2015. Available at http://naturaldatabase.therapeuticre- ering with olic acid plus vitamin B12 vs placebo on mortal-
search.com. Accessed September 25, 2015. ity and major morbidity in myocardial in arction survivors:
Jenkins DJ, et al. E ect o a low-glycemic index or a high-cereal a randomized trial. JAMA. 2010;300(24):2486–2494.
ber diet on type 2 diabetes: a randomized trial. JAMA. Villareal D , Chode S, Parimi N, et al. Weight loss, exercise, or
2008;300(23):2742–2753. both and physical unction in obese older adults. N Engl J
Jonsson , et al. Bene cial e ects o a Paleolithic diet Med. 2011;364(13):1218–1229.
on cardiovascular risk actors in type 2 diabetes: a World Health Organization. Waist circum erence and waist–
randomized cross-over pilot study. Cardiovasc Diabetol. hip ratio: report o a WHO expert consultation, Geneva,
2009;8:35. 8–11 December 2008. Geneva, Switzerland; World Health
Lesser LI, et al. Nutrition myths and healthy dietary advice Organization: 1–39. Available at http://whqlibdoc.who.
in clinical practice. Am Fam Physician. 2015;91(9): int/publications/2011/9789241501491_eng.pd . Accessed
634–638. September 24, 2015.
Substance Use Disorders
Dav d A. B d ll
25
or memory; (4) and the symptoms are not due to a general
CASE 25.1 medical condition or another mental disorder.
A 45-year-old male presents to your clinic to establish care or “A” is incorrect and will be discussed later in this case. “C” is
his chronic back and leg pain. He denies any other medical incorrect because a patient would not display the above symptoms
conditions. He reports being injured at work approximately with signi cant pain. “D” could be correct because acute alcohol
5 years ago, at which time he was started on oxycodone. He intoxication is characterized by slurred speech and impairments in
reports being on a stable dose or the last ew years. He appears memory and judgment; however, the patient does not have other
slightly drowsy during the appointment, has small pupils, and ndings consistent with alcohol intoxication such as an unsteady
is having moderate di culty describing his injury and pre- gait or nystagmus. “E” is incorrect as the patient was not hyper-
vious treatments. He reports his mood as okay, but becomes tensive, ebrile, diaphoretic, and did not have tremors or vomiting.
irritable when you begin to ask speci cs about his injury. He
has not been able to keep a job or the last year because “every- You are hesitant to prescribe the oxycodone partly because you
one res me.” He states that he needs the oxycodone to unc- recall that there were over 16,000 deaths related to prescription
tion and that he ran out o his medication 1 week ago. He does opioids in 2013. Although heroin-related deaths have increased
report occasional alcohol use, although he states the he knows much more rapidly than prescription opioids in the last
not to mix alcohol with his oxycodone. 15 years, they are still only about hal as common. You o er
him alternatives or pain control, including a re erral to physi-
Question 25.1.1 What is the most likely explanation o the cal therapy, NSAIDs, and a ENS unit. He says he has tried
patient’s current presentation? them all and none o them work. He gets upset and walks out o
A) Opioid withdrawal. your o ce. wo days later, while you are covering an emergency
B) Opioid intoxication. department (ED) shif , the same male presents to the ED or
C) Uncontrolled pain. severe, uncontrolled pain. He is vomiting, complains o muscle
D) Alcohol intoxication. aches and diarrhea, his pupils are dilated, and he is ebrile.
E) Alcohol withdrawal.
Question 25.1.2 What is the most likely explanation or the
Answer 25.1.1 The correct answer is “B.” T e patient presents patient’s current presentation?
with pupillary constriction, drowsiness, impairment in attention A) Opioid withdrawal.
and memory, psychological changes (dysphoria/irritability), B) Opioid intoxication.
and impairment in unctioning (not able to keep a job). All o C) Uncontrolled pain.
these ndings are included in the diagnostic criteria or opi- D) Alcohol intoxication.
oid intoxication. T e DSM-5 criteria or opioid intoxication E) Alcohol withdrawal.
include: (1) Recent use o an opioid; (2) clinically signi cant
problematic behavioral or psychological changes (euphoria ol- Answer 25.1.2 The correct answer is “A.” T e criterion or
lowed by apathy, dysphoria, psychomotor agitation or retarda- opioid withdrawal is cessation (or reduction in) opioid use
tion, impaired judgment, or impaired social or occupational that has been heavy or prolonged (several weeks or longer) or
unctioning) that develop during, or shortly a er, opioid use; administration o an opioid antagonist a er opioid use with
(3) pupillary constriction (or pupillary dilation due to anoxia three or more o the ollowing developing within minutes to
rom severe overdose) and one or more o the ollowing signs: several days a er the above: dysphoric mood, nausea or vomit-
drowsiness or coma, slurred speech, and impairment in attention ing, muscle aches, lacrimation or rhinorrhea, pupillary dilation,
672
CHAPTER 25 • Su BSt An c e u Se DiSo r Der S 673

piloerection, sweating, diarrhea, yawning, ever, or insomnia. Objectives: Did you learn to . . .
T ese signs and symptoms cause clinically signi cant distress • D p d x a a d w hd awal?
or impairment in social, occupational or other important areas • D s b h s b p ph ma ag p d
o unctioning and are not due to another medical condition or d p d ?
better accounted or by another mental disorder.
CASE 25.2
T e patient is trans erred to a substance abuse treatment
center voluntarily. Based on the available resources in your A 70-year-old emale is brought into the clinic by her daugh-
community, you decide on buprenorphine induction and ter due to concerns about her mother’s sleeplessness, isola-
maintenance treatment. tion, weight loss, alls, and anxiety over the past year. In
addition, since the patient has been staying at her daugh-
Question 25.1.3 All o the ollowing are true o buprenor ter’s home the past 3 days, she began vomiting, hallucinat-
phine EXCEPT: ing, perspiring pro usely, and wanting to return to her own
A) Buprenorphine should be administered prior to withdrawal home. T e patient has no history o medical problems. She is
symptoms. disheveled, con used, diaphoretic, and tremulous. Her blood
B) ypical rst dose o buprenorphine is 4 mg, with a maximum pressure is 162/110 mm Hg, pulse is 120 bpm, and temper-
rst day dose o 8 mg. ature is 38.5°C. She blames her symptoms on being unable
C) Buprenorphine monotherapy, without naloxone, should be to have a cigarette. She also blames her daughter’s nagging.
used or patients addicted to long-acting opioids (such as When asked about alcohol use, the patient says she has had a
methadone or sustained-release morphine). cocktail every evening since she retired rom her job last year,
D) Continue to increase buprenorphine on subsequent days by and that this helps her to sleep.
the total dose o the previous day plus an additional 4 mg
dose, until withdrawal symptoms are relieved. Question 25.2.1 Which o the ollowing best describes the
E) Nearly all patients will stabilize on 8 to 16 mg daily. patient’s current clinical condition?
A) Alcohol withdrawal.
Answer 25.1.3 The correct answer, and alse statement, B) Alcohol intoxication.
is “A.” Buprenorphine is started when patients are exhibiting C) Alcohol tolerance.
early signs o withdrawal. For short acting opioids (heroin, D) Alcohol abuse.
oxycodone) this may be 12 hours a er the last use; or longer- E) Alcohol dependence.
acting opioids such as methadone, this may be up to 48 hours
a er last use. T e rst day dose should be 4 mg, and patients Answer 25.2.1 The correct answer is “A.” T e patient pres-
should be monitored or withdrawal symptoms or 2 to 4 hours, ents tachycardic, hypertensive, and ebrile, with diaphoresis,
with the opportunity to administer an additional dose. Mono- tremors, vomiting, and hallucinations. All o these ndings
therapy (i.e., without naloxone) is appropriate or patients on are included in the diagnostic criteria or alcohol withdrawal.
long-acting opioids or the rst 1to 2 days to avoid any possibil- “B” is incorrect. Acute intoxication is characterized by slurred
ity that naloxone will cause withdrawal e ects, but then these speech, unsteady gait, nystagmus, and impaired memory and
patients should be trans erred to combination therapy. Nearly judgment. “C,” “D,” and “E” are incorrect and are discussed
all patients will stabilize on 8 to 16 mg daily; however, some may later in this case.
require doses up to 32 mg daily, which is the maximum dosage. T e criteria or alcohol withdrawal include: (1) T e patient
has stopped or reduced a previously heavy alcohol intake and
(2) at least two o the ollowing within hours or days: auto-
HELPFUL TIP:
nomic hyperactivity (hypertension, sweating, tachycardia, etc.),
Adva ag s b p ph v m had a
hand tremor, insomnia, nausea or vomiting, hallucinations,
ha a b d sp s d a a phys a ’s with
agitation, anxiety, or grand mal seizures. In addition, one must
an appropriate DEA waiver wh h q s h ab l y
have signi cant distress or impairment in unctioning with the
p vd h pa s l g, l k
withdrawal and no other illness causing the symptoms.
m had , wh h a ly b d sp s d a d s g a -
d am s; a d b a s s pa al ag s
Question 25.2.2 Which class o drugs would you choose to
a ,b p ph has a “ l g ” w h ga d
treat the symptoms o alcohol withdrawal?
v d s . M had p d s as g sp a y
A) Benzodiazepines.
s pp ss w h as g d s s.
B) Antipsychotics.
C) Antibiotics.
HELPFUL TIP: D) Alcohol.
S b x s a mb a b p ph a d E) Phenytoin.
al x .i d p s b , a phys a ds a
sp al “X” DeA mb . Answer 25.2.2 The correct answer is “A.” T e treatment o
choice is metabolic support and the use o benzodiazepines
674 FAMiLY MeDic in e eXAMin At io n & Bo Ar D r eVieW

to decrease physical distress and to prevent major withdrawal


s mpa abl a ab 79%. S s v y h c AGe
(delirium tremens, D ) rom occurring. “B” is incorrect. Anti-
a g s m ab 75% 95%, b h c AGe may
spychotics may control behavior but they do not prevent with-
m ss d p d al h l s d s d s ( .g., b g
drawal seizures and vasomotor instability. “D” is incorrect.
d k g). Y a h p ss bl s al h l
Although alcohol will work to prevent withdrawal, it has a airly
s d s d s, a d mm ly d p ma y
short hal -li e, and you generally do not want to endorse the
a l s, s h Al h l u s D s d s id a
use o alcohol in a patient with an alcohol use problem. “E” is
t s (Au Dit ). t h s s a 10 q s s v y ha has a
singularly incorrect. Phenytoin does nothing to prevent alcohol
s s v y 92% a d a sp y a 94%.
withdrawal seizures.

HELPFUL TIP: HELPFUL TIP:


th ad al “ba a a bag” w h m l v am s s - u a ly, y a always s h am ly’s h s-
ssa y. o al v am s ppl m s a j s as - y h . t h y may b abl g h al h l add
v a d l ss xp s v .th ly x p s h am , awa . Als , lab a y w k ps ( .g., l v
wh h y may wa g v iV iM h pa has a zym s) a b l d p mak h d ag -
h am d y. ss al h l ab s /d p d —al h gh l va d
GGt a d ASt /ALt a 2 a s pp v b bly
sp .
Question 25.2.3 What would be the best approach to eval
uating this patient or alcoholism?
A) Ask her the average amount she drinks.
B) Ask her how o en she drinks. Upon urther questioning, you begin to uncover a long his-
C) Ask her how requently she gets drunk. tory o heavy drinking—seems that there was more than just
D) Ask what her amily and riends say about her drinking. a nightcap.
E) Pour a bourbon in ront o her and see what happens.
Question 25.2.4 Which o the ollowing statements about
Answer 25.2.3 The correct answer is “D.” T e de ense mecha- this patient’s situation is TRUE?
nism o denial is so strongly evident in alcoholism that the best A) Cerebellar degeneration is uncommon.
approach is to explore how alcohol a ects her li e, rather than B) She is at risk or developing peripheral neuropathy.
direct questions about drinking behavior. In ormation rom C) Alcoholic “ atty liver” is irreversible.
amily and riends may provide a more accurate account o the D) She is at decreased or normal risk or heart disease.
problem. T e CAGE questionnaire is a very brie and use ul E) Immune unction should remain relatively intact.
screening tool, employed e ectively in the primary care setting.
A positive answer to two or more questions is very sensitive Answer 25.2.4 The correct answer is “B.” Peripheral neuropa-
and speci c or an alcohol use disorder. It consists o asking the thy can be seen in 10% o heavy drinkers as a result o vitamin
patient the ollowing our questions: de ciencies and the direct impact alcohol has on nerve unction.
Have you ever “A” is incorrect because cerebellar degeneration—suggested by
• C: elt that you should Cut Down on your drinking? ataxia and nystagmus—does occur as a result o alcohol over-
• A: been Annoyed that people criticized your drinking? use. “C” is incorrect because alcoholic “ atty liver” will reverse
• G: elt bad or Guilty about your drinking? with abstinence rom alcohol. “D” is incorrect. Heavy drinking
raises blood pressure and levels o triglycerides, increasing risk
• E: taken a drink rst thing in the morning (Eye Opener) to
o myocardial in arction. Finally, “E” is incorrect. Heavy drink-
get rid o a hangover or steady your nerves?
ing lowers the white blood cell count and inter eres with spe-
ci c aspects o the immune system; or example, it compromises
HELPFUL TIP: -cell unction.
Ask g h q s “H w ma y m s h pas y a
hav y had v m d ks a day ( m ) T is patient reports to you that she has needed to drink
m d ks ( w m ) a day?” s a s - increasing amounts o alcohol to help her all asleep.
ls q y h avy d k g. A p s v
s s m d s h avy d k g. i Question 25.2.5 The need or increasing amounts o alcohol
y hav a p s v s , ask ab s al q y is an example o :
a dq a y al h l s m d h lp d m A) Intoxication.
h p s abs al h l ab s d p - B) Dependence.
d .th s q s al has ab a 82% s s v- C) olerance.
y d g a al h l s d s d . Sp y D) Relapse.
E) Abuse.
CHAPTER 25 • Su BSt An c e u Se DiSo r Der S 675

Answer 25.2.5 The correct answer is “C.” olerance is de ned Question 25.2.7 Which o the ollowing medications
as the need or increasing amounts o a drug to achieve the same would be indicated to prevent DT in a patient with hepatic
response as initial use o the drug. “A” is incorrect. Intoxication impairment?
is a characteristic syndrome o maladaptive behavior or psycho- A) Alprazolam (Xanax).
logical changes that occurs with substance use, is drug-speci c, B) Chlordiazepoxide (Librium).
and reverses when the drug use is discontinued. “D” is incorrect. C) Diazepam (Valium).
Relapse involves re-starting use o the drug a er being abstinent D) Lorazepam (Ativan).
or a while. E) Clonazepam (Klonopin).

Answer 25.2.7 The correct answer is “D.” Benzodiazepines


HELPFUL (CLARIFYING) TIP: that are metabolized by the cytochrome P450 system will build
DSM-iV d v d d s bs a ab s a d s bs a d p - up in the presence o liver disease, so using those with inter-
d . DSM-5 v ws h s w a g s as d mediate hal -lives and no active metabolites is essential. Only
p s a sp m s bs a s d s d . th lorazepam, oxazepam, and temazepam meet these criteria. “B”
a s bs a s d s d a a l as 2 h is incorrect. Although chlordiazepoxide is o en used to prevent
ll w g 11 a( l s d 4 g ps): symptoms o alcohol withdrawal, it is hepatically metabolized
A. impa d l: 1 ak g m l g ha and has an exceptionally long hal -li e and, there ore, should be
d d, 2 s ss l s s p d w avoided in patients with liver problems. Alprazolam is too short
s , 3sp d g a g a d al m ba g, s g, acting to use in this situation.
v g m s , 4 av g s bs a .
B. S al mpa m : 5 a l l ll maj bl ga s Question 25.2.8 Which o the ollowing medications would
d s ,6 d s d sp p bl ms a s d be a good alternative to benzodiazepines to treat alcohol
xa ba d by s 7
mp a a v s g v withdrawal in an otherwise healthy, younger individual in
p d db a s s . mild to moderate withdrawal?
c . r sky s : 8 s haza d s s a sa d A) Atenolol.
9
d s d sp phys al psy h l g al B) Diltiazem.
p bl ms a s d xa ba d by s . C) Acetazolamide.
D. Pha ma l g d p d : 10d v l p g l a D) Carbamazepine.
11
w hd awal E) Prazosin.

Answer 25.2.8 The correct answer is “D.” Carbamazepine


Question 25.2.6 Which o the ollowing lab test results are has been used extensively in Europe (just like public trans-
you most likely to nd in this patient? portation and universal health coverage . . . so what do they
A) Microcytic anemia. know?). Carbamazepine prevents seizures and was ound
B) Low erritin. superior to oxazepam in reducing global psychological dis-
C) Decreased serum triglycerides. tress, aggression, and anxiety. It has no abuse potential. It
D) Hyperglycemia. is superior to benzodiazepines in preventing rebound with-
E) Increased gamma-glutamyltrans erase (GG ). drawal symptoms. Relative risk o drinking again a ter with-
drawal is three times higher or benzodiazepines compared
Answer 25.2.6 The correct answer is “E.” Elevated GG is con- to carbamazepine. he downside is it interacts with medi-
sidered to be the most sensitive indicator o alcohol intake and cines that undergo oxidative metabolism in the liver. Alpha-
is o en present along with elevation o the alanine and aspar- adrenergic agonists (“E”), beta-blockers (“A”), and calcium
tate transaminases (AL and AS ). Remember that GG is not channel blockers (“B”) have been used to control the symp-
speci c, as it is an inducible enzyme and can be induced by a toms o alcohol withdrawal; however, they are not e ective in
number o medications. T e classic AS :AL ratio in active preventing D or seizures. “C” does not address the problem
alcohol abusers is 2:1. Remember, however, that these labora- o alcohol withdrawal although there is some animal evi-
tory ndings are not speci c or alcohol use and can be caused dence that diuretics including acetazolamide may decrease
by medications and other illnesses. T e other answers are incor- brain damage rom binge drinking.
rect. Patients with alcoholism typically have macrocytic ane-
mia, elevated serum triglycerides, and hypoglycemia. Ferritin You remember rom a lecture that patients without seri-
is o en increased in active alcohol users in the absence o iron ous comorbidities, either medical or psychiatric, that have
overload. In addition, the trans errin saturation may be elevated mild to moderate withdrawal symptoms can sa ely be with-
because alcohol inhibits trans errin synthesis. drawn in an outpatient setting. T e tool that has been best
validated is the revised “Clinical Institute Withdrawal
You have ordered liver unction tests, but the results will not Assessment or Alcohol” (CIWA-Ar) which involves assess-
be available until the next day. T e laboratory technician is ment o the severity o 10 signs and symptoms associated
out getting Botox. with alcohol withdrawal.
676 FAMiLY MeDic in e eXAMin At io n & Bo Ar D r eVieW

Question 25.2.9 Assessment o the severity o which o the IM. However, or reasons previously stated, lorazepam is the
ollowing symptoms is NOT included in that tool? pre erred drug o those listed.
A) Headache or eelings o ullness in head.
B) Palpitations. HELPFUL TIP:
C) Paroxysmal sweats. t h iV m l az pam a als b adm s d s b-
D) actile disturbances. l g ally sp d abs p y wa s a al
E) remor. m d a . o bv sly iV s v as y .

Answer 25.2.9 The correct answer is “B.” Palpitations are not


included in the CIWA-Ar scale. In addition to “A,” “C,” “D,” and Question 25.2.12 DT carries a atality rate o :
“E,” the other signs and symptoms are nausea/vomiting, anxiety, A) < 1%.
agitation, auditory disturbance, visual disturbance, and orienta- B) 5%.
tion/clouding o sensorium. Each sign or symptom is given 0 to C) 10%.
7 points according to its severity except or orientation/clouding D) 25%.
o sensorium which is only 0 to 4 points. Withdrawal is considered E) 50%.
mild i the score is < 15, moderate is 16 to 20, and severe is > 20.
Answer 25.2.12 The correct answer is “B.” Prior to modern
You are considering whether or not this patient has D . treatment, the mortality reached up to almost 40% per episode.
T e rest are incorrect.
Question 25.2.10 Which o the ollowing is true o DT?
A) T e majority o patients with alcohol withdrawal develop Question 25.2.13 Which o the ollowing does NOT predis
D i not treated. pose to developing DT?
B) Auditory hallucinations are more common than visual A) Prior episodes o D .
hallucinations in D . B) Pneumonia.
C) Symptoms o D could easily be con used or dementia. C) Gastrointestinal (GI) bleed.
D) Her last drink would need to be about 1 week ago or her to D) Female gender.
have D . E) Hepatic ailure.
E) Autonomic instability is present in D .
Answer 25.2.13 The correct answer is “D.” Female gender does
Answer 25.2.10 The correct answer is “E.” Autonomic instability not predispose an individual to D but the other options do.
with elevated pulse, blood pressure, and ever are common in D .
“A” is incorrect. Minor withdrawal symptoms are quite common, Question 25.2.14 Which o the ollowing is NOT a complica
but D develops in only 3% to 5% o patients undergoing alcohol tion o alcoholism?
withdrawal. “B” is incorrect. Visual hallucinations are common A) Dementia.
in D ; auditory hallucinations are less likely. “C” is incorrect. B) Pancreatitis.
Withdrawal delirium typically presents acutely over a matter o C) Hypermagnesemia.
hours or days; whereas, in dementia, the cognitive decline is over D) Megaloblastic anemia.
a course o months to years. In addition, autonomic instability is E) Marchia ava–Bignami disease.
not a eature o early dementia (OK, maybe in Shy–Drager syn-
drome although cognition is usually well preserved—just like our Answer 25.2.14 The correct answer is “C.” All o the above,
patient’s liver). Finally, “D” is incorrect because the risk or D with the exception o hypermagnesemia, are associated with alco-
usually peaks 72 hours a er the last drink. hol abuse. A ew merit special comment. Alcoholic dementia may
be related to direct e ects o alcohol on the brain or to nutritional
Question 25.2.11 Which medication would be the best de ciencies. Hypomagnesemia is a complication o alcoholism.
choice or DT in a patient who is vomiting pro usely and Hypomagnesemia may decrease the response to thiamine admin-
who has no IV access? istration. Marchia ava–Bignami disease is demyelination and/or
A) Diazepam (Valium). necrosis o the corpus callosum and the adjacent white matter.
B) Alprazolam (Xanax). It presents with dementia, dysarthria, spasticity, and inability to
C) Chlordiazepoxide (Librium). ambulate. It can occasionally be seen in nondrinkers as well.
D) Lorazepam (Ativan).
E) Clonazepam (Klonopin). HELPFUL TIP:
eld ly pa s w h al h l p bl ms g -
Answer 25.2.11 The correct answer is “D.” Lorazepam is g z d. Hav a h gh d x s sp pa sw h
absorbed well intramuscularly. T is makes it a good choice or s g s a d symp ms s h as lab l hyp s , s m-
the vomiting patient. Diazepam is erratically absorbed IM and a, l gal ma al p bl ms, q alls a d j s,
should be administered either orally or IV. One could argue that h ada h s bla k s, a d vag Gi mpla s.
diazepam given rectally is a reasonable alternative to lorazepam
CHAPTER 25 • Su BSt An c e u Se DiSo r Der S 677

Question 25.2.15 Which o the ollowing is FALSE about Objectives: Did you learn to . . .
alcohol use disorders? • r g z s g s a d symp ms al h l w hd awal?
A) Most patients who develop alcohol disorders do so by their • D s b d ag s a al h l w hd awal?
mid-20s. • id y adv s s h avy al h l s ?
B) T e li etime prevalence o alcoholism is between 14% and 24%. • D a b w s bs a ab s a d d p d ?
C) Alcoholism is requently comorbid with other psychiatric • t a al h l w hd awal?
illnesses. • App a h w d al h ll ss plays a l h ass ss-
D) Alcohol abuse is ve times more requent in males than in m s bs a ab s ?
emales. • id y lab a y ab mal s bs v d al h l ab s
E) About 30% o patients with alcohol abuse meet criteria or a d d s a d h lm a s lab a y s d s?
major depressive disorder.

Answer 25.2.15 The correct answer is “A.” Most people who


QUICK QUIZ: n iGHt MAr e in MAr GAr it AViLLe
develop alcohol use disorders do so by their late 30s, not their
late 20s. T e rest are correct statements. Especially noteworthy is
“C.” Hal o all people with alcohol abuse have a comorbid psy- A 50-year-old divorced male presents to your clinic in an agi-
chiatric diagnosis. For example, about 50% to 60% o people with tated state, complaining o nausea, vomiting, and double vision.
bipolar illness have problems with alcohol abuse or dependence. He smells o alcohol, has gross bilateral hand tremors, and is
“E” is a correct statement as well. Although over 80%o patients disheveled. He is picking at his shirtsleeves and is oriented
with alcohol use disorders complain o depressive symptoms, to name only. On physical examination, he has lateral nys-
only 30%meet criteria or major depressive disorder. A use ul tagmus and an ataxic gait. Vital signs include: blood pressure
way to approach patients who complain o depression along with 160/92 mm Hg, pulse 100 bpm, and respirations 20 bpm. Labs
their alcohol abuse is to obtain a longitudinal history to see which are drawn, and his GG is moderately elevated.
came rst. I it is impossible to tease out, as is o en the case,
observe or 1 to 3 weeks o alcohol. I depression is still present Which o the ollowing is the most likely cause o his symptoms?
without alcohol use, it is prudent to treat with an antidepressant. A) Wernicke encephalopathy.
Be care ul when treating alcohol abusers with antidepressants: B) Normal pressure hydrocephalus.
active substance use severely reduces the e cacy o these drugs. C) Dementia.
D) Stroke.
E) Alcohol withdrawal.
HELPFUL TIP:
S bs a s a s a h gh s b w ag s 18 a d
The correct answer is “A.” Wernicke encephalopathy is the
25. A l h s s xp m a ha w ll d as h
result o thiamine de ciency and can occur in alcoholics and
d v d al ma s. S m , s , w ll g h
other patients with poor nutrition. See the next question or a
ab s .
description o the clinical ndings. Note that several symptoms
o Wernicke encephalopathy can mimic withdrawal (agitation,
etc.). However, as noted in the question, this patient is still
HELPFUL (AND IMPORTANT) TIP:
intoxicated.
S d s hav sh w ha symp m- gg d s ad
x d-d s p ls b z d az p am
al h l w hd awal as a pa may b h b s
QUICK QUIZ: Wer n ic Ke en c ePHALo PAt HY
way a a al h l w hd awal. A 2002 s dy
h Archives of Internal Medicine d m s a d ha h
symp m- gg d app a h am s l d l ss he triad o Wernicke encephalopathy includes all o the
s b z d az p s a sh h sp al s ay (20 h ollowing EXCEP :
vs. 62.7 h s, Arch Intern Med. 2002;162:1117–1121). A) Ataxia.
Symp m- gg d app a h s s h c iWA-A s - B) Oculomotor dys unction.
g sys m v y 4 8 h s w h b z d az p s p - C) Incontinence.
vd d s s >8 p v y h . B z d az p D) Encephalopathy.
gm s lz d l d : l az pam, d az pam, a d
hl d az p x d . r l h mb s s h small s The correct answer is “C.” Wernicke encephalopathy may pres-
v d s al b z d az p d d mp v ent with the classic triad o ataxia, encephalopathy (con usion),
symp ms. iV m la s d az pam a d l az pam and oculomotor dys unction (nystagmus, lateral rectus palsy,
may b lz d h a s g d d. o , etc.). However, the majority o patients presents with an incom-
Dt may q abs dly la g d s s b z d - plete syndrome and may be missing ophthalmoplegia, ataxia, or
az p s ( p s al b s s 70 mg iV d az pam encephalopathy. I you choose “B,” maybe you were thinking o
1 h ). normal pressure hydrocephalus, which presents with dementia,
incontinence, and ataxia.
678 FAMiLY MeDic in e eXAMin At io n & Bo Ar D r eVieW

A B C
FIGURE 25-1. F al al h l sy d m . A. A 2 1/2 y a s. B, C. A 12 y a s. n p ss sh palp b al ss s, p a hal lds, f a m d a ,
hyp plas ph l m, a d h pp v m l b d . t h s d v d al als has h sh , l a p p b al s a ha a s y g mal s w h
al al h l sy d m . (r p d d m S ssg h AP, c la SK, J s KL. n a al h s y h al al h l sy d m : A 10-y a ll w- p
l v pa s. Lancet. 1985;326:85–91, w h p m ss m els v .)

attention de cits, or mental impairment. T e other options are


CASE 25.3 incorrect. See Figure 25-1.
A 28-year-old married, pregnant patient and her mother are
presenting or a ollow-up appointment. At the appointment HELPFUL TIP:
today, the patient’s mother shares that her daughter has been th a FAS s 10% 50% am g h h ld
drinking alcohol during her pregnancy. T e daughter is very w m wh ab s d al h l d g p g a y. FAS is
annoyed when con ronted with her use o alcohol and will the leading cause o intellectual and developmental
not give speci c in ormation about it, but she does admit to disability in the Western Hemisphere. c h ld w h
drinking. She does not agree to quit during her pregnancy h s d s d hav a av ag iQ 68 70.
or to be re erred or substance abuse evaluation. You discuss
some o the e ects alcohol might have on the developing
etus, including etal alcohol syndrome (FAS). Question 25.3.2 Symptoms and signs o FAS may be
detected at what point a ter birth?
Question 25.3.1 The diagnosis o FAS requires speci c man A) Within 12 hours.
i estations in three areas, including: B) A er the rst week.
A) At least two acial anomalies, retardation o growth below C) At 1 month.
the 20th percentile, and little motor activity. D) At 3 months
B) At least two acial anomalies, retardation o growth below the E) A er 1 year.
10th percentile, and central nervous system (CNS) problems
that may include tremulousness, hyperactivity, attention Answer 25.3.2 The correct answer is “A.” Soon a er birth, the
de cits, or mental impairment. neonate may display symptoms o withdrawal, including tremu-
C) At least three acial anomalies, retardation o growth below lousness, inconsolability, vomiting, and poor eeding. T e char-
the 20th percentile, and poor sucking ref exes. acteristic cluster o acial/physical mal ormations (listed above)
D) Small size, hyperactivity, and one generalized acial anomaly. is evident within 12 hours o birth. Other principal eatures are
CNS dys unction, growth de ciency at birth, joint and limb
Answer 25.3.1 The correct answer is “B.” T e diagnosis o abnormalities, and heart de ects.
FAS requires in utero exposure to alcohol and at least two acial
anomalies, retardation o growth below the 10th percentile, and In addition to FAS, you discuss some o the potential prob-
CNS problems that may include tremulousness, hyperactivity, lems seen with more limited in utero exposure to alcohol.
CHAPTER 25 • Su BSt An c e u Se DiSo r Der S 679

Question 25.3.3 What are the criteria or etal alcohol smoking up to two packs o cigarettes per day. She asks you
ef ects (FAE)? what is available to help her stop. You tell her that nicotine
A) Small size, prematurity, hyperactivity, learning disabilities. replacement therapy (NR ), bupropion (Zyban), and vareni-
B) Inability to complete tasks, very active, very excitable. cline (Chantix) are currently approved to aid patients who
C) Facial mal ormations, heart de ects, hyperactivity. want to stop smoking.
D) Growth de ciency at 10th percentile, learning disabilities. You want to o er her an NR that is easy to use, has ew
E) Congenital cardiac anomalies, growth de ciency at 10th side e ects, and provides steady blood levels o nicotine over
percentile, acial mal ormations. the whole day.

Answer 25.3.3 The correct answer is “A.” Knowledge continues Question 25.4.1 Which do you choose?
to grow regarding the more subtle e ects o maternal drinking, A) Nicotine patch.
such as that seen with “social” alcohol use. FAE can occur with B) Nicotine gum.
even moderate drinking during pregnancy. T e abnormalities C) Nicotine nasal spray.
commonly observed with FAE include small size, prematurity, D) Nicotine inhaler.
hyperactivity, and learning disabilities. E) E-cigarettes.

Question 25.3.4 In which stage o pregnancy does the tera Answer 25.4.1 The correct answer is “A.” T ere are bene ts
togenic action o alcohol cause acial mal ormations? and drawbacks to each o the smoking cessation aids. Like
A) During the embryonic phase, at the eighth week a er con- other NR , the nicotine patch is available without prescrip-
ception. tion. It is applied to the skin daily and provides a steady release
B) During the third week o pregnancy. o nicotine through the skin. For a patient with a heavy, long
C) During the late rst trimester. smoking history, begin with the 21 mg patch and taper to the
D) During the second trimester. 7 mg patch over a 3-month time rame. Some o the drawbacks
E) During the third trimester. o the patch include slower release o nicotine, the potential
or skin irritation, insomnia, and vivid dreams. Some o these
Answer 25.3.4 The correct answer is “B.” Facial mal ormations side e ects can be reduced by having the patient rotate applica-
occur with the use o alcohol during the third week. Organs and tion sites daily (to avoid skin irritation) and removing the patch
limbs appear most susceptible to the e ects o alcohol during be ore bedtime. Nicotine gum is an ion-exchange resin that
the embryonic phase, which is completed by the eighth week releases nicotine or absorption through the buccal mucosa
o conception, a time when most women are rst con rming with only 30% bioavailability in the rest o the GI tract. Side
pregnancy. Alcohol exposure during any stage o development e ects include mouth and throat soreness, jaw atigue, hiccups,
may a ect brain development or unction. and undesirable taste. T ere are advantages o nicotine gum:
it involves an active coping mechanism (chewing, placing the
You are able to convince this patient to reduce and eventually gum in one’s mouth, etc.) and is more likely than the patch to
eliminate her use o alcohol; she delivers a healthy-appearing delay weight gain. T e nasal spray delivers 0.5 mg nicotine per
newborn who she names “Harley” af er the motorcycle. dose and is sprayed one to two times per hour or a maximum
o 5 sprays per hour and 40 sprays per day. Most o the nico-
tine nasal spray side e ects are attributable to its route o use—
HELPFUL TIP:
nasal and throat irritation, rhinorrhea, sneezing, etc. T e oral
n v am y h ld a a m y l (Ha l y), a
inhaler provides an active delivery o nicotine similar to the
p sm al ( .g., Sapph , D am d), a W s
nasal inhaler, and its side e ects are also a consequence o its
la d ma ( .g., S a, c h y , c dy) a g
route o delivery—throat irritation and coughing. E-Cigarettes
(W ss r m g ). i s a s b ha y d h y
create a vapor (not smoke) that simulates smoking. Like other
w ll hav l g p bl ms (a l as ha s xp -
nicotine replacement products, the amount o nicotine may be
). n w, y am y kda a b a d wh sk y
decreased over time. T ere are more than 250 di erent brands
(l k h s g P k d d), ll g h w a h ’ll g !
o e-cigarettes. Very little is known about their sa ety; or while
they don’t contain the smoke they do contain nicotine and
Objectives: Did you learn to . . . other chemicals. E-cigarettes have a checkered track record
• id y p bl m d k g a p g a pa ? when it comes to being an aid to smoking cessation (vs. a gate-
• App a h a pa w h al h l ab s p g a y? way to nicotine addiction or younger users). T ere are studies
• D s b h d gs FAe a d FAS? to support their use but just as many that suggest that they do
not aid with smoking cessation.
CASE 25.4 Question 25.4.2 Which o the ollowing is the most signi
A 60-year-old emale presents to your o ce and is deter- cant side ef ect o varenicline (Chantix)?
mined to stop smoking. She has a history o schizophrenia A) Desire to commit violence or commit suicide.
treated with clozapine (Clozaril) and a 43-year history o B) Desire to watch endless reruns o Godzilla movies
680 FAMiLY MeDic in e eXAMin At io n & Bo Ar D r eVieW

C) Leukocytosis.
HELPFUL TIP:
D) Urinary retention.
A 2014 c h a v w d ha p yl has
ss ally h sam s ss a as b p p . n p-
Answer 25.4.2 The correct answer is “A.” T ere is an FDA
yl s s a d a 25 mg da ly 3 days, h 50 mg ll
warning about the possibility o suicidal or homicidal ideation.
h 7 h day wh h d s s as d 75 mg da ly.
T is is not good, especially in someone who already has a psy-
chiatric disorder. Other common side e ects include f atulence
(really), nausea, headache, and insomnia.
T e mood in the examination room turns a shade bluer as
Although this patient does not mind spending $6 to $7 per day your patient wonders whether, at age 60 and with an 86-pack-
on cigarettes, she does not want to pay a lot or something to year history, there is even any reason to quit. “It’s too late or
help her quit (Why is this, you ask? So do we . . . ). me,” she laments.

Question 25.4.5 How long does it take to reduce the risk o


Question 25.4.3 Which smoking cessation aid is the most
having a heart attack by 50% a ter one stops smoking?
expensive?
A) 24 hours.
A) Nicotine patch.
B) 6 months.
B) Nicotine gum.
C) 1 year.
C) Nicotine nasal spray.
D) 10 years.
D) Nicotine inhaler.
E) In nity.
E) Varenicline.
Answer 25.4.5 The correct answer is “C.” wenty minutes a er
Answer 25.4.3 The correct answer is “E.” Varenicline costs quitting smoking, blood pressure drops and the temperature o
approximately $280 to $350 per month. T e nicotine products hands and eet increases to normal. A er 24 hours, the risk o heart
are less expensive in the range o $60 to $120 per month. O attack begins to decrease. At 1 year, the chance o having a heart
course, some health insurance programs will support smoking attack is cut in hal . A er 5 years, stroke risk is reduced to that
cessation aids. o a nonsmoker. A er 10 years, the risk o dying rom lung cancer is
about hal that o a person who continues to smoke. A er 15 years,
“I tried those things, Doc, and nicotine didn’t work and var- the risk o coronary heart disease approaches that o a nonsmoker.
enicline made me shoot my boy riend in the leg—but he had
it coming. Isn’t there something else?” T e mood in the examination room is heading into a deeper
blue as your patient expresses concern that she will ail in this
Question 25.4.4 Which o the ollowing medications might attempt to quit smoking, as she has in the past. “I’ll never do it,”
you also prescribe to aid this patient in smoking cessation? she complains.
A) Naloxone.
B) Metoprolol. Question 25.4.6 On average, how many attempts to quit
C) Haloperidol. smoking are made be ore a person succeeds?
D) Bupropion. A) 2.
B) 6.
Answer 25.4.4 The correct answer is “D.” Bupropion is mar- C) 10.
keted as the antidepressant Wellbutrin and the smoking cessa- D) 16.
tion aid Zyban. It reduces the symptoms o nicotine withdrawal
by blocking dopamine in the brain’s reward center, and this Answer 25.4.6 The correct answer is “B.” It takes an average o
mechanism is thought to result in reduced nicotine craving. six attempts at quitting be ore success. So, i at rst your patient
Studies have shown that people who use bupropion doubled doesn’t succeed, try again and again and . . . so on.
their chances o quitting smoking. Its e ect appears to be addi-
tive to that o NR . Its e ect on smoking cessation is indepen- HELPFUL (AND UNFORTUNATE) TIP:
dent o its antidepressant e ect, as shown by its equal e cacy n h s ssa a ds hav a g a a k d
in depressed and nondepressed patients. One o the advantages s ss. Y a d mb s all v h b a d. W h
o bupropion is its ability to prevent the weight gain that occurs a ds, s m h g l k 15% pa s w ll b -
in most people when they stop smoking. Bupropion should ba a 1 y a . A s dy 2011 sh w d no di er-
be started at a dose o 150 mg daily or 3 days then increasing ence l g- m abs a sb w s s -
to 150 mg twice daily, i the patient is tolerating the drug. T e a ds a d h s wh q h w (Tob Control.
patient should be advised to stop smoking during the second 2013;22:32–37). W h b p p , h mb s l s
week o treatment. T ere is no evidence that SSRI are e ective 23%. Va l als has ab a 23% 1-y a abs
in smoking cessation. Pooled data rom 4 head to head trials a .c s l g may as h s ss a , a d all
comparing bupropion and varenicline indicate that varenicline sm k s sh ld b g d q a v y app m .
is more e ective.
CHAPTER 25 • Su BSt An c e u Se DiSo r Der S 681

• e m a h phys l g al adva ag s sm k g
T is patient has been on clozapine (Clozaril) or control o ssa a a y ag ?
her psychosis. • r g z s g s a d symp ms w hd awal?

Question 25.4.7 The combination o clozapine and bupro


pion should be used with caution because: QUICK QUIZ: ALL in t He FAMiLY
A) Both may lower the seizure threshold.
B) Both may cause hypertension. Which o the ollowing characteristics is NO true o the
C) Bupropion may inter ere with the metabolism o clozapine. children o alcoholics?
D) Severe GI symptoms could occur. A) T ey experience earlier onset o problem drinking.
E) A psychotic episode could be precipitated. B) T ey experience earlier pregnancies (well, not the males . . . ).
C) T ey have less stable amily involvement.
Answer 25.4.7 The correct answer is “A.” Both drugs may lower D) T ey experience poor academic and social per ormance in
the seizure threshold. Hypertension should not be a concern. school.
Bupropion does not inter ere with the metabolism o clozapine, E) T ey have more antisocial behavior.
cause severe GI symptoms, or precipitate psychotic episodes.
The correct answer is “B.” T ere is no data to suggest early preg-
Af er all o your discussion, the patient wants to quit “cold nancy as a characteristic. win, adoption, and hal -sibling stud-
turkey.” You gently explain that going rom 40 cigarettes per ies and studies o amilial versus non amilial alcoholism indicate
day to zero might be hard on her although quitting cold tur- that children o alcoholics have our times the risk or develop-
key improves the chance o success (Ann Intern Med. Mar 15, ing alcoholism. T ey also have worse school per ormance, more
2016;164(6):377–448). antisocial behavior, and less stable amily situations.
Question 25.4.8 All o the ollowing are symp toms o
nicotine withdrawal EXCEPT:
CASE 25.5
A) Increased appetite. A 35-year-old intoxicated emale presents to your o ce
B) Dysphoria. requesting to be started on disul ram (Antabuse). She is oth-
C) achycardia. erwise healthy and recently has begun to drink in response to
D) Insomnia. the death o her sister. Be ore this, she was a teetotaler.
E) Irritability.
Question 25.5.1 Disul ram should not be administered
Answer 25.4.8 The correct answer is “C.” Nicotine withdrawal until the patient has been abstinent rom alcohol or how
is actually associated with decreased heart rate. In addition to the long?
above, trouble concentrating and restlessness are also common A) 4 hours.
symptoms. B) 12 hours.
C) 24 hours.
HELPFUL TIP: D) 48 hours.
cy s ( b s d w h ys ys ) s E) 72 hours.
a h bal d va v ha s v sm k g ssa-
(pla b 2.4% vs. ys 8%). i s l ss xp s v Answer 25.5.1 The correct answer is “B.” A minimum o
ha va l a d va l s a ally a d va v 12 hours should have elapsed be ore giving disul ram to avoid the
y s (N Engl J Med. 2011;365:1193–1200). disul ram–alcohol reaction. O course, this depends on how much
they were drinking. Remember, we are in a college town here . . .

HELPFUL TIP: HELPFUL TIP:


o h m as s ha a v h lp g a pa q D s l am s abs b d sl wly m h Gi a a d s
a a ma d ll ph m ssag s (Lancet. 2011;378: l m a d sl wly; h , a patient should wait at
49–55), sv a ( s ph alls, b d-s d least 1 week a ter stopping disul iram be ore return-
a h ga a Mi, ., CMAJ. 2009;180(13):1283–1284), ing to drinking.
a d pay g pa s q (N Engl J Med. 2009;360(7):
699–709). i add , p bl p l s s h as ax s
Question 25.5.2 I the patient consumes alcohol while taking
ga s a d ba g sm k g d s hav d m -
disul ram, which o the ollowing is most likely to occur?
s a d v ss a as g q g a s.
A) Respiratory depression.
B) Hypertension.
Objectives: Did you learn to . . . C) Nausea and vomiting.
• empl y pha ma h apy h am ba D) Cardiovascular collapse.
add ? E) Convulsions.
682 FAMiLY MeDic in e eXAMin At io n & Bo Ar D r eVieW

Answer 25.5.2 The correct answer is “C.” T e disul ram– Question 25.5.5 Which o the ollowing is true about
ethanol interaction generally includes f ushing o the skin, acamprosate?
nausea, vomiting, palpitations, hypotension, sweating, blurred A) It is contraindicated in those with renal impairment/renal
vision, and dizziness. Rarely, in more severe reactions, respira- ailure.
tory depression, cardiovascular collapse, convulsions, and death B) At 1 year, acamprosate is no better than placebo in preventing
may occur. T e severity o the reaction is typically dose related relapse o alcoholism.
and depends on the amount o alcohol ingested. C) T e combination o acamprosate and naltrexone is no more
e ective than either alone.
Question 25.5.3 Common side ef ects o disul ram include D) Drug reps will tell you that all o these medications are great
which o the ollowing? and that you should prescribe them.
A) Hypotension. E) All o the above.
B) Peripheral neuropathy.
C) Insomnia. Answer 25.5.5 The correct answer is “E.” All o the above are
D) Nausea. true. Un ortunately, at 1 year none o these drugs are signi -
E) Depression. cantly better than placebo. How do they get past the FDA, you
ask? So do we. Don’t even get us started on the topical onycho-
Answer 25.5.3 The correct answer is “B.” Drowsiness, hepa- mycosis treatments.
totoxicity, rashes, hypertension, peripheral neuropathy, metallic
Question 25.5.6 Which o the ollowing is true about acam
a er-taste, and optic neuritis may occur with disul ram use.
prosate?
T ese e ects are independent o alcohol ingestion.
A) Patients receiving opioid maintenance therapy or opioids or
pain will need higher levels o opioid medication.
HELPFUL TIP: B) It is contraindicated in those with liver impairment/liver
M l pl as s h pa s hav b p d w h ailure.
sag d s l am, a d bas l lv zym s C) Patients who require detoxi cation can be continued sa ely
sh ld b b a d p sa g d s l am a d on acamprosate.
h app x ma ly 2 w ks a a a - D) It is contraindicated in patients who are taking SSRIs.
m . o as ally, a ash may a ly . th
ash a b a d w h a h s am s a d h d g Answer 25.5.6 The correct answer is “C.” Acamprosate does
a b d. not interact with benzodiazepines or other medications used
in medical detoxi cation and thus can be continued sa ely i a
patient starts drinking and then requires detoxi cation. Acam-
prosate is not metabolized by the liver and can be used sa ely
Af er a discussion, the patient thinks that disul ram is the even in patients with severe liver disease, whereas naltrexone
“one thing that might work.” She tries it, but at ollow-up, and disul ram are contraindicated. T ose receiving opioids or
she is not having much success. She continues to drink acute or chronic pain or receiving opioid maintenance treatment
heavily and (thank ully) has been noncompliant with tak- can use acamprosate because it does not a ect these medica-
ing disul ram regularly. She reports strong craving or tions; and it is sa e to take with SSRIs.
alcohol.
HELPFUL TIP:
Question 25.5.4 At this point, as an FDA approved alterna
o h m d a s ha hav b s d a al -
tive to disul ram, you decide to prescribe:
h l sm l d p ama , ba l , da s ,a d
A) Naltrexone (Revia).
SSr is. t p ama has h b s da a a d da s
B) Naloxone (Narcan).
s ly v a ly s al h l sm h s wh
C) Nortriptyline.
hav a sp g 5-Ht a sp .
D) Nitroprusside.

Answer 25.5.4 The correct answer is “A.” Naltrexone is FDA-


HELPFUL TIP:
approved or the treatment o alcoholism. As an opioid antag-
n h s d gs a pa la ly v . Ds l -
onist, it blocks the euphoric e ects o alcohol. opiramate
am s b ha pla b , al x a d a am-
( opamax) and baclo en can also be used to decrease the crav-
p sa hav a n n t 12-20. n al x d s days
ing or alcohol. However, they are not FDA-approved or this
h avy d k g b a amp sa (JAMA. May 14,
indication. None o the other drugs listed are used to treat
2014;311(18):1889–1900).
alcohol abuse and dependence.

T is patient seems to recall something about using camel Objectives: Did you learn to . . .
paws to treat alcoholism. You very perceptively realize she • D s b h m ha sm a d s l am?
means Campral (acamprosate). • r g z sd s d s l am?
CHAPTER 25 • Su BSt An c e u Se DiSo r Der S 683

• D s b h al h l–d s l am a ? C) Contact the patient’s methadone maintenance clinic or dose


• P s b va s m d al h ap s al h l s d s d s? increase.
D) Noti y the patient o her positive urine drug screen and let
CASE 25.6 her know you are not surprised by the result, as methadone
is metabolized to morphine.
A 25-year-old comatose emale presents to the ED with E) Have the patient committed to a substance abuse treatment
pinpoint pupils and respiratory depression. acility.
Question 25.6.1 Which o the ollowing is the most likely
Answer 25.6.3 The correct answer is “C.” he act that
cause o coma?
this patient has relapsed into heroin use may mean that her
A) Blood alcohol level o 200 mg/dL.
methadone dose is too low or maintenance. his can lead to
B) Cocaine overdose.
additional heroin use. he rest are incorrect. “A” would be
C) Methadone overdose.
dangerous. “B” is not necessary. “E” would be ine ective or
D) Benzodiazepine withdrawal.
a patient not interested in quitting (but is o ten used anyway
E) Phencyclidine (PCP) intoxication.
as desperate measure). “D” is o particular note. Methadone
Answer 25.6.1 The correct answer is “C.” T e classic “triad” o does not have any active metabolizes (so “D” is incorrect),
opioid overdose consists o coma, respiratory depression, and but it can be ound in the urine. Heroin and codeine are
pinpoint pupils (miosis). Certain patients may have atypical metabolized to morphine.
presentations, and the triad may not always be present in opioid
overdose. Miosis is particularly variable, o en not being seen
HELPFUL TIP:
in those with meperidine and propoxyphene overdoses; and
H gh-d s m had a l ad Qt p l ga a d
coingestions, such as sympathomimetics and anticholinergics,
sad s d p s.
can also prevent miosis. “A” is incorrect. A patient with a blood
alcohol level o 200 mg/dL would likely be ataxic; but alcoholic
coma typically occurs at blood levels >400 mg/dL and depends
Question 25.6.4 Methadone is typically prescribed or opioid
on the level o tolerance. Stimulant overdose would not likely
maintenance therapy:
present with somnolence. Alcohol and PCP intoxication would
A) Once daily.
more likely present with nystagmus and not constricted pupils.
B) wice daily.
C) T ree times daily.
You treat her with naloxone and she wakes up (spitting mad,
D) T ree times weekly.
agitated, and in withdrawal).
E) Once monthly.
Question 25.6.2 What is the minimum amount o time
that you should observe a patient who has overdosed on Answer 25.6.4 The correct answer is “A.” Methadone has an
methadone? elimination hal -li e o up to 60 hours (range 15–60 hours).
A) 1 hours. T us, once daily dosing is appropriate or the treatment o nar-
B) 4 hours. cotic addiction. T e advantage here is that the medication can
C) 12 hours. be given under direct observation. Remember that it is given
D) 36 hours. more requently (Q8 hours) or pain control.
E) 72 hours.
Af er regaining consciousness, the patient in orms you that
Answer 25.6.2 The correct answer is “D.” Methadone has she is 20 weeks pregnant.
a long hal -li e (variable, but up to 60 hours), and there ore a
patient who has overdosed on methadone should be monitored Question 25.6.5 Regarding pregnancy and usage o opioids,
or at least 36 to 48 hours. Since the ½ li e o naloxone is 30 to which o the ollowing would most likely have the best
90 minutes, it may take multiple doses o naloxone to treat a outcome?
methadone overdose (a naloxone drip is o en used). A) Continuing the patient on methadone.
B) Withdrawing the patient rom all opioids in the rst trimester
You discover that the patient has been on a methadone main- (too late or this one).
tenance program, but then she relapsed and overdosed on C) Withdrawing the patient rom all opioids in the second
a combination o methadone and heroin. Her urine drug trimester.
screen returns positive or opioids. D) Withdrawing the patient rom all opioids in the third
trimester.
Question 25.6.3 What would be the next most appropriate
course o action? Answer 25.6.5 The correct answer is “A.” Opioid with-
A) Detoxi y the patient o methadone as an outpatient. drawal in a pregnant woman can cause etal distress and low
B) Contact the authorities to have the patient arrested. birth weight. Methadone is currently the standard o care in
684 FAMiLY MeDic in e eXAMin At io n & Bo Ar D r eVieW

the United States or the treatment o opioid addiction in


HELPFUL TIP:
pregnant women. Methadone maintenance has multiple
S m la s a a s a ya y l s a d
advantages, including longer durations o maternal drug
s v hyp s . My a d al a w hs m -
abstinence, better obstetrical care, reductions in etal illicit
la s sh ld b a d l k a y h Mi. Hyp -
drug exposure, and enhanced neonatal outcomes (e.g.,
s sh ld b a d w h alpha a d b a-bl k s.
increased etal growth, reduced etal mortality, and decreased
risk o preeclampsia).
Objectives: Did you learn to . . . HELPFUL TIP:
• r g z symp ms a d s g s v d s w h p ds a d ch m hamph am s s ass a d w h w d -
h ll d gs? sp ad d al a s a d g g val d s as ha a s l
• u s m had p d add ? l ss ma y, v all, h. “M h m h,” as h s
• t a a p g a pa w h p d add ? s mm ly all d, s p bably d p l g dp -
ds p d al hyg , x s m a, h gh al
d a d d k, a d h g d g.
CASE 25.7
A 40-year-old emale is admitted through the ED, arriving
Question 25.7.3 Which o the ollowing does NOT address
via ambulance rom a smaller hospital. T e local physician
the needs o this patient during withdrawal?
called to report that her riends said that she had “shot up a
A) Provide a secure environment.
lot o meth.” She is known to use her son’s Ritalin prescription
B) Provide regular meals and snacks.
on a regular basis. She appears rightened and anxious. She
C) Make sure the patient is awakened i she spends excess time
is uncommunicative, rocking back and orth on the examina-
sleeping.
tion table, and picking at her skin trying to remove bugs that
D) Consider giving a benzodiazepine i the patient remains
only she can see. She becomes angry easily and lashes out at
anxious.
sta in the ED.
E) Provide education as an intervention towards change.
Question 25.7.1 A reasonable dif erential diagnosis or this Answer 25.7.3 The correct answer is “C.” Amphetamine with-
patient would include all o the ollowing EXCEPT: drawal requires sleep, nutritious ood, and a sa e place until the
A) Schizophrenia. unstable state o being has diminished. T e patient should be
B) Drug-induced psychosis. allowed to sleep. Stimulant abusers o en stay up or a week or
C) PCP, hallucinogen, or cocaine intoxication. more at a time and then “crash” or sleep or days during with-
D) Diabetes. drawal. Antipsychotic medications and benzodiazepines may
E) D . be administered i needed.

Answer 25.7.1 The correct answer is “D.” Just checking to see Objectives: Did you learn to . . .
i you’re awake. T ough hypoglycemic patients can certainly • r g z h s g s a d symp ms amph am
x a ?
become con used, diabetes is least likely to be causing these
symptoms. Although there is a history o amphetamine use, the • App p a ly a a pa w h amph am x a
a d w hd awal?
other options should not be eliminated out o hand. T ere may
be more than one substance involved (o course, we never see
that sort o thing in Iowa . . . no, never). CASE 25.8
A 15-year-old male is brought into the ED by his neighbor
T e urine drug screen is positive or amphetamines alone. who ound the boy passed out in his backyard with a bag ull
o glue nearby. He had di culty rousing the boy. Currently,
Question 25.7.2 The ollowing are all symptoms o amphet the patient is lethargic with slurred speech and di culty
amine use EXCEPT: walking. When his parents arrive, they are shocked, as their
A) achycardia son has been a “good kid.” T ey had no idea he was using any
B) Hypertension. drugs. O course, he bought lots o tubes o “model glue” but
C) Perspiration or chills. they never did see any completed models.
D) Weight gain.
E) Psychomotor agitation. Question 25.8.1 The signs and symptoms o inhalant use
include all o the ollowing EXCEPT:
Answer 25.7.2 The correct answer is “D.” Weight loss, not A) Dizziness.
weight gain, can be anticipated in the amphetamine user. All o B) Slurred speech.
the other symptoms can be seen as a result o amphetamine use. C) Unsteady gait.
Symptoms can progress to con usion, arrhythmias, seizures, D) Smell o solvents or glue.
dystonias, or coma. E) Dilated pupils
CHAPTER 25 • Su BSt An c e u Se DiSo r Der S 685

Answer 25.8.1 The correct answer is “E.” Inhalant intoxication Answer 25.8.5 The correct answer is “A.” Males, not emales,
is identi ed by impaired judgment, impaired social interaction, are more likely to abuse inhalants. Inhalant users tend to be o
and aggressive behavior o en leading to altercations. Higher lower socioeconomic status, younger (age 13–15), and have di -
doses can lead to lethargy, psychomotor retardation, stupor, or culty in school. T ere is an increased incidence o inhalant
coma. Dilated pupils are seen in anticholinergic toxicity and abuse among Native Americans.
other drugs (e.g., sympathomimetics) but not with inhalants.
HELPFUL TIP:
Question 25.8.2 All o the ollowing are considered inhal
“S g” s wh m s a hal d d ly m a
ants used by abusers EXCEPT:
s a h s bs a s pla d a bag
A) Kerosene.
a d hal d m h bag. “H g” s wh h s b-
B) Cleaning solvent.
sa s pla d a ag a d h hal d w h h ag
C) Gasoline.
pla d v h s a d m h. G a k w h l g .
D) Spray paint.
E) Glue.
Objectives: Did you learn to . . .
Answer 25.8.2 The correct answer is “A.” Kerosene is not • id y s m yp s hala s ab s d?
volatile enough to be abusable. T e rest can be abused by • r g z h symp ms hala ab s ?
inhalation. • D s b h d m g aph s hala ab s ?

HELPFUL TIP: CASE 25.9


th a x a v la l s g ally las s
A 40-year-old male who has been smoking marijuana daily
ab 30 m s. Gas l sa x p ; h as
or the past 20 years would like to quit his marijuana habit . . .
gas l , h x a a las p 6 h s.
but he just cannot nd the motivation. He wants to apply or
a new job, and a drug screen is part o the application pro-
Question 25.8.3 Withdrawal rom inhalants has been cess. He makes an appointment with you to discuss what he
described and includes all o the ollowing eatures EXCEPT: can expect when he quits and how long will it be until his
A) Onset o symptoms 24 to 48 hours a er use has stopped. drug screen is negative.
B) ransient illusions.
C) Diaphoresis. Question 25.9.1 All o the ollowing are symptoms o
D) Con usion. marijuana intoxication EXCEPT:
E) Intense hunger. A) Euphoria.
B) Sensation o slowed time.
Answer 25.8.3 The correct answer is “E.” Intense hunger is not C) Increased mental alertness.
a sign o withdrawal. In act, the patient may be nauseated. All D) Increased appetite.
o the other options are correct. E) Dry mouth.

Question 25.8.4 All o the ollowing can result rom chronic Answer 25.9.1 The correct answer is “C.” Statistically speak-
solvent or hydrocarbon inhalation EXCEPT: ing, 40% o you will know this one rom personal experience
A) Chronic brain injury. (you know who you are!). Marijuana intoxication decreases
B) Muscle weakness. mental alertness, although it can reportedly enhance the senses.
C) CNS microhemorrhages and secondary seizures. When inhaled, intoxication peaks a er 10 to 30 minutes and
D) Erythrocytosis. lasts about 3 hours.
E) Liver and renal damage.
HELPFUL TIP:
Answer 25.8.4 The correct answer is “D.” Solvents and hydro-
Ab 40% u .S. ad l s hav d ma j a a a s m
carbons can cause all o the e ects noted above except or eryth-
p h l v s, wh h has ha g d s g a ly
rocytosis. In act, one can see bone marrow suppression as a
h las 30 y a s.
result o chronic inhalant use.

Question 25.8.5 Risk actors or inhalant abuse include all Question 25.9.2 How long can marijuana (THC) be detected
o the ollowing demographic actors EXCEPT: in the urine?
A) Female. A) 30 days i used regularly; 2 to 7 days i used occasionally.
B) Age 13 to 15. B) 2 weeks.
C) Low socioeconomic state. C) 1 week or emales; 2 weeks or males.
D) Native American. D) 24 to 48 hours.
E) Poor school per ormance. E) It cannot be detected in urine samples.
686 FAMiLY MeDic in e eXAMin At io n & Bo Ar D r eVieW

Answer 25.9.2 The correct answer is “A.” T is patient will have


Ecstasy, als k w as MDMA ( 3,4-m hyl d xy-
to stop smoking marijuana or a minimum o 30 days be ore
m hamph am ), p d s s sa s ph a,
applying or the job, i he wants to assure his urine drug screen
ma y, a d d s h b . us s s pa s
will be negative. T e drug can be present in hair samples or an
b a s s asy a a s b x sm (D d ’ y w d
extended period o time—anywhere rom 1 to 6 months.
why all h s sa s k g a dy pa s?
J s l k l?). MDMA a a s s z s, l v d s-
Question 25.9.3 Which o these is NOT one o the touted
as , a d hyp h m a am g h s d s.
bene ts o marijuana?
A) Pain reliever. K2 (als k w as “sp ” a d “ h ll x”) s asy h
B) Anxiolytic. a ab d ha s m h m p ha ma j a a.
C) Antiemetic. i s s ld as s . th s s h hal h
D) Appetite stimulator. sm k . S d s l d a hy a d a, hyp s ,
E) Antidepressant. d aph s s, a x y, m d dys g la , m m y
laps s, pa a a, hall a s, d la d p p ls (myd a-
Answer 25.9.3 The correct answer is “E.” In act, the use o s s), a s a/v m g, f sh g, m a d s v
marijuana has been shown to actually worsen depression. Mari- as s s z s. t a m s yp ally s pp v m a-
juana has been used to treat pain, nausea, anorexia, glaucoma, s s w h alm g v m a d b z d az p s
etc. Despite the act that nearly hal o the states have passes laws as d d,
allowing some sort o medical marijuana use, evidence or many Bath salts (also known as “khat,” “vanilla sky” and
o the touted medical bene ts o marijuana is scant or contra- “white lightning” and “meph” ( or mephedrone,
dictory. T e best evidence o bene t has been seen with chronic which we think is people trying to pronounce
neuropathic pain. “meth” without any teeth) a sy h ah s.
Symp ms x ya yp ally sympa h m m ,
Objectives: Did you learn to . . .
s m la m hamph am w h v s m la , ag-
• r g z h symp ms ma j a a x a ?
g ss , ag a ,m m yd s, hall a a d
• A pa s ma j a a s ?
pa a a mm . o , h y may a s adv s -
s s h as h pa al ,a k d y j y, hab-
QUICK QUIZ: Du De, WHAt WAS in t HAt r o c K? d my lys s, a d m ab l d a g m s. t a m s
yp ally s pp v m as s w h iV f ds, b z d az-
p s h lp alm h pa , alm g v m
A 20-year-old male is brought by the police into the ED because
a d/ mp a y s l w-d s a psy h s
o severe agitation a er smoking what he thought was crack
psy h s s.
cocaine. He exhibits slurred speech, ataxia, circumoral numb-
ness, and horizontal nystagmus. GHB (AKA “g v s b d ly ha m,” “G g a h m b y,”
a d h s). A sh -a g d g, p d s b h s da-
Which o the ollowing substances is most likely causing his a d s m la (wh h may y l ap dly). Pa s
symptoms? may b b ady a d a d a ap b wh s m -
A) Cannabis ( HC). la d may b m ag a d a d v l .
B) Heroin. Salvia s a a ally gh b m h m am-
C) Methamphetamine. ly M x (salvia divinorum). t h h b s yp ally
D) PCP. sm k d b a b h w d a d abs b d m h
E) Nicotine. b al m sa. A v g d s salv A, wh h
b ds as κ p d p ag s . D s d sa
The correct answer is “D.” PCP (also called “angel dust”) s m la k am , LSD, a d a ab s. Adv s s
intoxication is characterized by agitation, impulsiveness, nys- l d : a x y, d p s al za , dysph a, -
tagmus, hypertension, tachycardia, numbness, ataxia, and s , h ada h s, d ws ss, a hy a d a a d hyp -
perceptual distortions. Intoxication begins 5 minutes a er use s .n ha s salv Ad s b d µ
and peaks in 30 minutes. PCP-induced psychosis is the most p d p s ha sp a y d p ss s yp -
common PCP-induced disorder and may mimic a schizo- ally s as w h p ds. t a m salv a x y
phrenic psychotic episode. PCP may be added to other drugs s yp ally s pp v w h alm g v m sa d
unbeknownst to the user. b z d az p s as d d.
Remember the abuse o caf eine (legally available in
HELPFUL TIP: crystalline orm over the internet) and the toxicity
A w w ds d sg d gs, wh h a b m g o nicotine ( rom ingestion o products meant or a
as gly p p la as h y a d l d vaporizer) when presented with a patient abusing
s a da d d gs s. drugs.
CHAPTER 25 • Su BSt An c e u Se DiSo r Der S 687

American Psychiatric Association. Substance-related and


QUICK QUIZ: Qu ic K c o u n SeLin G addictive disorders. In: Diagnostic and Statistical Manual of
Mental Health Disorders: DSM-5 (5th ed). 2013. Arlington,
Brie intervention or alcohol problems is one o the most clini- VA: American Psychiatric Publishing.
cally e ective and cost-e ective preventive services rom among Alpert HR, et al. A prospective cohort study challenging the
those recommended by the USPS F. e ectiveness o population-based medical intervention or
smoking cessation. ob Control. 2013;22:32–37.
All o the ollowing are components or conducting brie Aoun EG, et al. Emerging drugs o abuse: clinical and legal
interventions EXCEP : considerations. Rhode Island Med J. 2014 Jun;97(6):41–45.
A) Ask and give eedback on screening results. Ballantyne JC. Opioid therapy in chronic pain. Phys Med
B) State recommendations about sa e drinking levels and o er Rehabil Clin N Am. 2015;26(2):201–218.
advice about change. Barry KL, et al. Reducing alcohol-exposed pregnancies: a
C) Assess the patient’s readiness to change. report o the National ask Force on Fetal Alcohol
Syndrome and Fetal Alcohol E ect. Atlanta, GA: Centers
D) Negotiate goals and strategies or change.
or Disease Control and Prevention; March 2009.
E) Follow-up annually.
Bayard M, et al. Alcohol withdrawal syndrome. Am Fam
Physician. 2004;69(6):1443–1450.
Answer OK, we will give you this one because we wanted to
Brensilver M, et al. Pharmacotherapy o amphetamine-type
introduce the concept o brie interventions. T e correct answer
stimulant dependence: an update. Drug Alcohol Rev.
(and the thing to avoid) is “E.” Follow-up should be arranged 2013;32:449–460.
much sooner than 1 year. T e “5 A’s” method or brie interven-
Daeppen JB, et al. Symptom-triggered vs. xed schedule doses
tions has substantial research support or its utility in alcohol o benzodiazepine or alcohol withdrawal. Arch Intern
use disorders across a variety o settings and can be incorpo- Med. 2002;162:1117–1121.
rated with motivational strategies in a step-by-step process. T e Free C, et al. Smoking cessation support delivered via mobile
“5 A’s” approach is a brie , goal directed way to more e ectively phone messaging (txt2stop): a single-blind, randomized
address substance use with patients. Altogether, the “5 A’s” may trial. Lancet. 2011;378:49–55.
take 1 to 5 minutes, depending on a provider’s clinical setting Howard MO, et al. Inhalant use and inhalant use disorders in
and roles. T ese components include Asking about use, Advis- the United States. Addict Sci Clin Pract. 2011;6(1):18–31.
ing to cut down/quit, Assessing willingness to quit, Assisting Moore HM, et al. Cannabis use and risk o psychotic or
with strategies or abstinence, and Arranging ollow-up. a ective mental health outcomes: a systematic review.
Lancet. 2007 Jul 28;370:319–328.
Clinical Pearls Nelson ME, et al. Emerging Drugs o Abuse. Emerg Med Clin N
Am. 2014;3(1):1–28.
Ask all ad l s ab ba s , adv s h m s p s g Rigotti NA. Helping smokers with cardiac disease to abstain
ba , a d p v d b hav al v sa d rom tobacco a er a stay in hospital. CMAJ. 2009;180(13):
pha ma h apy ( a d a d) ba 1283–1284.
ssa .
Shapiro B, et al. A primary care approach to substance misuse.
D adm s av s m l v am s pa sw h Am Fam Physician. 2013;88(2):113–121.
al h l d p d a d/ w hd awal. o al m l v am s Spitho S, Kahan M. Primary care management o alcohol
a s h maj y pa s. iV h am may use disorder and at-risk drinking part 1: screening and
b d a d h s s sp d hav g W k assessment. Can Fam Physician. 2015;61:509–514.
phal pa hy.
Spitho S, Kahan M. Primary care management o alcohol use
P vd d a a d s l g s h l-ag h ld a d disorder and at-risk drinking part 2: counsel, prescribe,
ad l s s ga d g ba , al h l a d ll d g s . connect. Can Fam Physician. 2015;61:515–521.
S all ad l s al h l m s s a d p v d p s s T ompson A, et al. Pharmacotherapy or alcohol dependence:
gag d sky haza d s d k g w h b b hav al astrati ed approach. Pharmacology & T erapeutics. 2015;
s l g v s d al h l m s s . 153:10–24.
S all p g a w m al h l m s s . P v d Volpp KG, et al. A randomized, controlled trial o nancial
s l ga d s s d m s s . F al al h l incentives or smoking cessation. N Engl J Med. 2009;
sy d m s h l ad g a s ll al d sab l y h 360(7):699–709.
u d S a s. Walley AY. HIV prevention and treatment strategies can help
address the overdose crisis. Prev Med. April 2015. Available
at http://dx.doi.org/10.1016/j.ypmed.2015.04.004.
BIBLIOGRAPHY Waterman EH, et al. Reducing etal alcohol exposure in the
American Academy o Family Physicians. Policies: substance United States. Obstet Gynecol Surv. 2013;68(5):367–378.
abuse and addiction. 2014. Available at http://www.aa p. West R, et al. Placebo-controlled trial o cytisine or smoking
org/about/policies/all/substance-abuse.html. cessation. N Engl J Med. 2011;365:1193–1200.
Ethics
Mark A. Graber
26
Question 26.1.2 What is the most relevant piece o in orma-
CASE 26.1 tion in Charlene’s account that suggests that her capacity
A 54-year-old married emale, Charlene, has insulin-dependent is intact?
diabetes and has seen you or her care or the last 7 years. A) T erapeutic alliance with you despite noncompliance with
In the last year, she has developed diabetic retinopathy and treatment recommendations.
neuropathy. o your great rustration, Charlene continues to B) Integration into community relationships, including a stable
resist the recommended li estyle changes required to control marriage and responsibilities in the elementary school.
her diabetes. C) Expression o placing perceived sel -identity as a higher
She is a casual, riendly woman known as the “candy priority than control o diabetes and its complications.
lady” in her neighborhood where she lives with her hus- D) Awareness that her dietary choices are associated with
band o 30 years. She loves children and volunteers at the symptoms o eye disease and neuropathy.
local elementary school, where she is well known or a quick E) Flagrant disregard or medical recommendations by eating
smile, a reassuring hug, and a piece o candy in her large, candy while at her appointment.
ull pockets. In act, she is noted during most o her appoint-
ments to be munching on M&M’s—her avorite candy. She Answer 26.1.2 The correct answer is “C.” While several o
has had dietary consults and many education-oriented doc- these variables are relevant in assessing DMC, the capacity or
tor appointments but says, “I know I shouldn’t eat the way DMC is typically thought o in a stepwise ashion, starting with
I do, but I just don’t have the heart to change who I am, even (1) ability to communicate a choice, then (2) basic understand-
i it does help my eyes and legs. Who I am is about what I eat ing o the variables, then (3) ability to appreciate the personal
and do.” impact o choices, and nally (4) the ability to reason about the
You wonder about Charlene’s capacity or decision-making, options while considering personal values. T is last step is not
given her rank noncompliance with care, even in the setting o only the most complicated but also the most strongly indicative
serious complications rom her diabetes. that DMC is intact.

Question 26.1.1 All o the ollowing variables are necessary Question 26.1.3 Which o the ollowing is TRUE about DMC?
in decision-making capacity (DMC) EXCEPT: A) Patients who have been ound legally incompetent do not
A) Ability to communicate a choice. have DMC.
B) Voluntary choice. B) A patient’s DMC may vary according to the circumstances o
C) Understanding o the variables involved in the decision. the situation.
D) Ability to appreciate the personal impact o choices. C) A minor’s DMC is not clinically relevant since there is a
E) Family agreement that the patient is competent. surrogate who bears the responsibility or decision-making.
D) DMC should not be evaluated in cases in which the patient
Answer 26.1.1 The correct answer is “E.” All o the other options makes an unconventional choice.
are considered important or determining DMC. Certainly amily E) Patients with psychiatric disease, who are committed to a
concerns need to be addressed but amily agreement has nothing treatment acility, do not have DMC.
to do with determining a patient’s competence. One additional
necessary element or DMC is the ability to reason about the Answer 26.1.3 The correct answer is “B.” DMC is not an all-or-
options in the setting o personal values. none distinction but can vary widely rom case to case or setting

688
CHAPTER 26 • Et h ic s 689

to setting. Even patients who have been declared legally incom- o “ utility” is trying to make it through this book without vast
petent or who have been legally and involuntarily committed quantities o ca eine.
may still have a measure o DMC. Moral theory typically urges
clinicians to consider the wishes and reasoning o their patients A er discussion with her husband, you decide to discontinue
as morally and clinically relevant, regardless o the placement o ventilation. Charlene dies.
a legal guardian or the state as a surrogate decision maker. DMC
may ultimately be overridden in certain kinds o legal circum- Question 26.1.6 This intervention is appropriately consid-
stances, but it should not be done lightly as it suggests a unda- ered:
mental denial o patient autonomy. As a result, many patients A) Active, nonvoluntary euthanasia.
with psychiatric illness still have the right to make choices, even B) Physician-assisted suicide.
with diagnoses such as schizophrenia. Making unconventional C) Withholding medical intervention.
choices can sometimes be a marker that DMC is not intact but D) T e principle o double e ect.
does not automatically lead one to this conclusion (e.g., Jehovah E) Withdrawing medical intervention.
Witness and blood trans usions; re using blood is unconven-
tional but DMC may be intact). Finally, though minors techni- Answer 26.1.6 The correct answer is “E.” While the answer
cally cannot make many healthcare choices, their wishes should might seem intuitive to you, many persons (including physi-
be taken into consideration as they are o en able to articulate a cians) do not recognize the di erences between these various
pre erence. In minors, this is termed “assent,” rather than con- interventions. Active euthanasia (“A”) is when the physician
sent. Children may not be able to understand enough to consent both supplies the means o death and is the nal human agent
on their own but they can “assent” (or protest) a particular plan in the events leading to the patient’s death (e.g., the physician
o treatment. administers the lethal drug). Whether or not active euthanasia
is voluntary, involuntary or nonvoluntary depends on the DMC
Charlene continues to have a slow decline over time but o the patient. Assisted suicide (“B”) occurs when the physician
remains in good spirits despite the complications o her provides the means o death but the patient carries out the act,
uncontrolled diabetes. One day her husband brings her to such as taking an overdose o phenobarbital. Withholding med-
the emergency department. He had ound her in the bath- ical intervention (“C”) means not initiating care or a disease
room, unconscious, and called an ambulance. She has had a state such that the disease itsel results in death (e.g., not placing
stroke and remains unresponsive, on ventilation in the ICU. a gastric tube or arti cial eeding in a patient with end-stage
Her prognosis is poor. dementia). Withdrawing medical intervention (“E”) means
discontinuing an intervention that has already been used,
Question 26.1.4 What are appropriate considerations or although the disease state itsel results in death with the inter-
making a treatment decision about end-o -li e care or vention’s discontinuation. T e principle o double e ect (“D”)
Charlene? is an ethical theory that suggests that i there is an unintended
A) Oral statements to her husband about her end-o -li e care. bad outcome (e.g., death) while pursuing an intended purpose
B) Her husband’s wishes or her care as designated proxy (e.g., pain relie ), there is diminished moral responsibility or
healthcare decision maker. the unintended outcome. T is principle is sometimes used to
C) Written advance directives. justi y the use o high-dose opiates or sedatives in patients with
D) Oral statements to her physician about her end-o -li e care. intractable pain, even when the unintended e ect is respiratory
E) All o the above. depression and death.

Answer 26.1.4 The correct answer is “E.” Written advance Question 26.1.7 Which o the ollowing is TRUE about the
directives are considered the most binding, although all o these role o law in li e -sustaining interventions?
considerations are relevant in making end-o -li e decisions. A) Courts must be involved in decisions a er a patient has been
declared to not have capacity.
Question 26.1.5 Which o the ollowing statements can be B) Li e-sustaining treatment may be withheld only i patients
used to describe medical utility? are terminally ill or permanently unconscious.
A) No worthwhile goals o care can be achieved. C) Physicians may ace criminal charges or providing appro-
B) T e likelihood o success is very small. priate palliative care and not treating the underlying disease.
C) T e patient’s quality o li e is unacceptable. D) T e most prudent legal advice is to continue treatment in
D) All o the above. medically utile cases.
E) T e law presents ew barriers to physicians withholding
Answer 26.1.5 The correct answer is “D.” All o the above li e-sustaining interventions.
meanings have been explicitly or implicitly drawn into discus-
sions about medical utility. For this reason, many theorists have Answer 26.1.7 The correct answer is “E.” Sometimes physi-
objected to the use o the term “ utility” as a justi cation or cians inappropriately provide treatment to patients who have
decisions and urge clinicians to be precise about the concerns made their end-o -li e choices clear and have stated that they
that arise in a given patient’s clinical situation. Another de nition do not want prolongation o li e. Respecting the patient’s prior
690 FAMiLY MEDic iNE EXAMiNAt iON & BOARD REViEW

wishes will not result in legal liability or the physician, but HIV. However, patients can still be tested anonymously which
the converse is not true; one can be legally liable or treating a precludes reporting and can circumvent reporting laws. For this
patient who does not want treatment (e.g., trans using a Jeho- same reason, “D” is incorrect. T ere are mandatory reporter laws
vah Witness patient who re used trans usion). “A” is incorrect. on the books. “C” is also correct. Although this is somewhat con-
A er a patient is declared to not have capacity the courts no troversial and there are con icting duties (the patient’s autonomy
longer need to be involved, as a legal surrogate is appointed by and con dentiality vs. the duty to warn the partner), we have a
the court to make decisions or the patient. “B” is also incorrect duty to protect the patient’s partner. In cases o a direct threat
as treatment may be withheld at any time at the request o a to a known individual, we have a duty to warn the individual. A
competent patient. more clear-cut case would be i George were threatening to shoot
his partner and storms out o the of ce with a gun. While this is
Objectives: Did you learn to . . .
a more immediate example, the same principle holds. Clearly,
• Evalua e a pa en ’ DMc ?
we must not take the breaking o patient con dentiality lightly.
• Re ogn ze ow DMc may vary ba ed on e pa en and e
And, we must in orm the patient o our course o action. You
l n al e ng?
may want to set a time rame or George to noti y his partner
• iden y ome e al ue n end-o -l e are?
with the understanding that i he does not do so, you must.
• De r be med al u l y and under and mpor an e n Alternatively, i all we know is that George is HIV positive
mak ng e al de on ?
and have no knowledge o his partner(s), we cannot publicize
George’s HIV status in the local press in order to “protect the
HELPFUL TIP: public.” But i we have a speci c name o an ongoing sexual
Do “non ompl an ” pa en make you ru ra ed and partner, we have a duty to warn that individual which competes
angry? Ju n ernal ze e pr n ple o pa en au ono- with our duty to maintain con dentiality with George.
my. We an make ugge on , bu up o e pa en
o de de ow o a on our adv e. in a , er CASE 26.2
right o do o. Under and and you w ll ave a lo
le ear burn. Robert, a 27-year-old married nurse rom your hospital, is
re erred to your emergency department or an urgent evalua-
tion by his supervisor. In the past 2 weeks, he has been noted
to be increasingly distressed while at work, with occasional
QUICK QUIZ: AN Et h ic AL DiLEMMA tear ulness, distractibility, and irritability.
During the initial assessment, Robert reveals that there is
You are seeing George, a 30-year-old, HIV-positive male, or a speci c reason that he has been so preoccupied. He indi-
routine care and to assess his HIV status. During your conver- cates that 2 weeks ago he was jailed or operating a vehicle
sation, he mentions that he is in a new relationship—a relation- while intoxicated and that he eels ashamed. He is a raid that
ship that he hopes will become long term. T ey use condoms his coworkers have read about it in the newspaper, although
“some o the time” but have unprotected sex on a regular basis no one on his oor has indicated that this is the case. T is
when there is no condom available. When you ask him whether is his rst legal in raction o any kind and he describes it as
or not he has disclosed his HIV status to his partner, he states humiliating.
that he has not done so and will not do so because o the ear On urther questioning, Robert indicates that he uses
his partner will leave him. He also orbids you to contact his alcohol regularly. While it has not overtly a ected his work
partner to noti y her o his HIV status. as ar as he can tell, it has caused signi cant marital stri e. He
reports that his pattern is to stop by the bar on the way home
Your response in this situation is to: rom work to “relax and let go o the hospital stu that I worry
A) Attempt to convince George to noti y his partner o his HIV about.” He typically drinks three beers and then drives home,
status. where he continues to drink beer throughout the evening. He
B) Depending on your state, contact the health department and notes that his wi e and kids complain that he is emotionally
have them ollow up on the patient’s sexual contacts. absent and even irritable with them, but he says that his am-
C) Contact the partner directly and let him/her know o ily simply doesn’t understand the stress o the workplace and
George’s HIV status. his need to “ orget about it or a ew hours.” He and his wi e
D) Maintain strict con dentiality and do not warn George’s have started arguing lately about his alcohol use, especially
partner nor report his HIV status regardless o the situation. since the driving charge. He takes special exception to her
E) A, B, and C. stating that he is an “alcoholic.”
As you take the history, Robert begins to be more guarded
The correct answer is “E.” “A” is clearly correct. Anything we in his responses and more restricted in his a ect. Suddenly,
can do to convince George that it is critical he noti y his part- he blurts out, “I don’t think I’m an alcoholic, but I don’t want
ner o his HIV status (short o coercion) should be done. “B” you to put anything in my record about any o this stu ! And
is correct but may vary by state. As o this writing, all 50 states I want you to tell my supervisor that there are some personal
and most territories have con dential, name-based reporting or problems going on at home and that I’ll be ne in a ew days.”
CHAPTER 26 • Et h ic s 691

Question 26.2.1 Which o the ollowing statements is TRUE I may see or hear in the course o the treatment . . . which on
about your obligation with regard to documentation in the no account one must spread abroad, I will keep to mysel , hold-
chart? ing such things shame ul to be spoken about.” T is is not only
A) You are obligated to document the visit as it occurred so ar important to the tradition o medicine itsel but also to the
as the medical acts are concerned, including the concern physician–patient relationship. T ere is no doubt that loss o
about alcohol abuse. con dentiality may cause harm to the patient when others are
B) You can enter incorrect in ormation into the chart in order in possession o con dential medical in ormation. Such harms
to protect the patient. may be as overt as denying medical coverage or certain genetic
C) You are under no obligation to document anything said and conditions or as subtle as devaluing a person seen waiting to
can withhold in ormation rom the chart at the patient’s see the psychiatrist.
request.
D) Hospital administration or legal counsel should be involved Question 26.2.3 Which o the ollowing are legally pro-
i in ormation is going to be purpose ully le out o the tected exceptions to the rule o maintaining patient con-
chart. f dentiality?
E) You can lie in the medical record . . . we’re pretty sure perjury, A) Reporting tuberculosis to public of cials without patient
liable, and slander don’t apply to doctors. consent.
B) Warning a third party at risk o imminent and serious bodily
Answer 26.2.1 The correct answer is “A.” T e ethical princi- harm rom the patient without patient consent.
ples o bene cence, nonmale cence, and justice drive the deci- C) Reporting a patient’s alcohol abuse to a work supervisor
sion here. A patient may legitimately ask or nonactive medical without the patient’s consent.
problems (e.g., distant history o sexual abuse) to be withheld D) A and B.
rom current documentation o an active problem (e.g., allergic E) All o the above.
rhinitis—by the way, kudos on your detailed history-taking).
However, a patient cannot legitimately ask to have in ormation Answer 26.2.3 The correct answer is “D.” Under current
withheld rom the record i that in ormation is pertinent to an national and state laws, physicians are mandatory reporters
ongoing condition currently being evaluated and treated. In this o some in ectious diseases and o intent to harm another. In
case, Robert is receiving care simply by virtue o being seen in the most other cases, provision o medical in ormation without the
emergency department and disclosing the chie complaint and patient’s written consent is not legally protected, although there
its associated variables. It is important or you to be orthcoming may be cases in which it is elt to be morally justi able. Physicians
in explaining why the in ormation may not be withheld rom need to weigh violations o patient con dentiality very care ully,
the medical record and also in reassuring him that nonrelevant even when legally sanctioned. Ethicists typically agree that i a
medical in ormation will be omitted rom the record i he eels physician is going to compromise a patient’s con dentiality or
that this is necessary. For example, the speci cs o the argument an overwhelming moral obligation that, in respect or patient
with a wi e need not be detailed beyond the comment that there autonomy, the patient needs to be noti ed. In many situations in
is nonviolent marital con ict over the patient’s alcohol use— which a physician hopes to communicate con dential in orma-
important because it supports an alcohol abuse disorder. Fur- tion to a third party even when the patient is unwilling, a pro-
thermore, many institutions have speci c policies on manag- cess o education and negotiation with the patient occurs such
ing sensitive medical in ormation and there may be a ormal that respect or autonomy is acknowledged while simultaneously
mechanism or increasing the security o the patient’s medical making the patient aware o competing moral obligations.
record.

Question 26.2.2 Why is protection o conf dentiality impor- HELPFUL TIP:


tant in medical practice? h av ng a axed, e-ma led, or ma led repor on a n ng a
A) It shows respect or patient autonomy. pa en ’ on den al med al n orma on m d re ed
B) It helps prevent stigmatization and discrimination against o an un n ended re p en not legally pro e ed. Be
patients based on private medical issues. au ou abou ran m on o pa en n orma on.
C) It helps solidi y trust within the physician–patient relation-
ship.
D) It helps establish a boundary between the physician–patient Question 26.2.4 Which o the ollowing inter eres with pro-
relationship and the rest o the medical system. tecting patient conf dentiality in the medical structure?
E) All o the above. A) Involvement o managed care organizations in patient care
and medical payments.
Answer 26.2.2 The correct answer is “E.” T e physician– B) Electronic records and transmissions.
patient relationship is a long-honored tradition in medicine C) Group practices and/or teaching hospitals with multiple
that is increasingly ragile in a medical system with numerous care providers.
competing obligations. Nevertheless, it is prudent to remem- D) A and B.
ber the aspect o the Hippocratic Oath, which states, “What E) All o the above.
692 FAMiLY MEDic iNE EXAMiNAt iON & BOARD REViEW

Answer 26.2.4 The correct answer is “E.” While individual the pro ession in general, ultimately producing ears that phy-
physicians and patients continue to prize the tradition o sicians will take the sel -serving path rather than the higher
respect or con dentiality, the multiple players in health care moral ground.
make it nearly impossible to restrict all in ormation to the
dyad o physician and patient. Insurance companies will not You tell Robert that he has alcohol dependence and then pro-
provide payment without, at least, in ormation about the diag- vide education about the diagnosis and treatment options.
nosis, and notably, insurance companies are not legally bound You recommend outpatient treatment in Alcoholics Anony-
by the same codes o conduct that apply to physicians regard- mous (AA) and a chemical dependency program. Robert
ing patient privacy. Electronic records and transmissions by agrees, more or the sake o his amily stability rather than
e-mail, cellular phones, axes, and other means are much more because o any true insight into the severity o his problem.
easily accessed by the curious or unintended recipients who You then arrange or ollow-up with one o your partners
have no reason to have con dential in ormation. Open wait- (you’ve been selected as a contestant on the next Survivor and
ing rooms and multiple providers o care mean that larger and get to escape to a tropical island).
larger numbers o the community are aware that a patient is At the next appointment, Robert meets his new physi-
being seen in certain clinics or certain purposes. While patient cian, Dr. Pincus. At this appointment, Robert indicates that
records are not o en considered a con dentiality issue, the he did attend two AA meetings but was very uncom ort-
reality is that once in ormation is in written orm, it is more able with the aspect o the 12-step program that requires
dif cult to control who might, either now or in the uture, have acknowledging a “higher power.” Robert indicates that he is
access to the details o the report. For this reason, some physi- an atheist and secular humanist, believing that the locus o
cians try to err on documenting only that which is considered sel -control comes rom within the individual human spirit.
absolutely necessary to patient care, although the distinction He has re used to continue in AA due to his rejection o its
between the “absolutely necessary” and unnecessary can be a theistic oundation. He has had no urther legal problems
dif cult line to draw in the sand, especially without the abil- and reports that work is still going ne, with diminished
ity to appreciate how multiple variables may play out in the irritability once he resolved in his mind that his coworkers
patient’s uture medical care. were unaware o his previous driving violation. However, he
continues to drink six to nine alcoholic beverages per night
Back to the patient at hand . . . Robert is asking you to be and admits that he occasionally needs a shot o whisky in the
deliberately deceptive with the supervisor. You disagree morning to “make sure I don’t lose it with all the work stress”
with this. (this is where his sel -control theory really comes together).
He also works a night shi about once per week and does use
Question 26.2.5 Which o the ollowing is FALSE? approximately the same amount o alcohol be ore beginning
A) rust in the physician–patient relationship depends on the night shi , although he denies being intoxicated while
allowing the patient to make such a directive about com- on the job on these nights (“six beers just get me started”).
munication with outside persons. He doesn’t think this is a problem because “things are quiet
B) A physician who establishes a precedent or deception may at night and everyone just helps each other keep the patients
be expected to practice deception in a uture situation in com ortable.” He reports that his amily is satis ed with the
which the harms greatly outweigh the bene ts. decrease in consumption and that he considers the matter o
C) A physician who deceives may undermine general trust in alcohol abuse resolved.
the pro ession. Dr. Pincus has had her own problems with alcohol in
D) All o the above. the past. She has had a rocky course over the past many
years but ound AA to be very help ul. She has become very
Answer 26.2.5 The correct answer is “A.” Another way active in her Jewish synagogue and community, where she
o phrasing the question is, “What drives a physician to receives support and is accountable to her riends. Her
be honest even when what the patient really wants is not own alcohol history has been marked by dif culty with
honesty?” Will the patient trust you more i you are decep- alcohol bingeing, such that when she starts to drink, she
tive or him? Will this help him (aside rom allowing him to drinks to intoxication. Only with aggressive honesty at a
keep his job)? pro essional-group AA, as well as a substance abuse proto-
T e physician–patient relationship is generally not consid- col through the state board o medical examiners, does she
ered an adequate reason to lie to a third party about the nature eel that she’s been able to remain completely abstinent or
o a patient’s illness and treatment. T ere has been concern the last 4 years.
that a physician who deceives a third party, even in the imme- Dr. Pincus is considering revealing to Robert some o her
diate interest o the patient’s con dentiality or other concerns, own struggles as a health-care pro essional with a substance
establishes himsel or hersel as a physician who may not be abuse disorder. She believes that this will help him reevalu-
trustworthy in other matters. A patient may not consider this ate the role o AA in sobriety and the importance o very
at the time a deception is requested. T ese kinds o ripple tight control o alcohol consumption to prevent relapsing
e ects rom the decisions o an individual physician can a ect illness.
CHAPTER 26 • Et h ic s 693

Question 26.2.6 Sel -disclosure is best described as involv- Whether or not sel -disclosure o one’s own religious belie s
ing the ethical issues o : is appropriate is an important question. As mentioned in the
A) Deception and nondisclosure. discussion in the question above, it is very important or the
B) Privacy and boundaries. physician to measure the intent o the disclosure. Also, physi-
C) In ormed consent. cians need to be exquisitely sensitive to the power di erential
D) Impaired colleagues. that exists between a physician and a patient such that strong
E) Autonomy. individual viewpoints might become threatening or coercive
in the physician–patient relationship. In certain religious tra-
Answer 26.2.6 The correct answer is “B.” T ere are explicit ditions, sharing one’s aith is an important step, demonstrat-
and implicit boundaries that exist between a physician’s pri- ing courage and integrity; nevertheless, physicians should be
vate experiences and the physician–patient relationship. One strongly cautioned to pay heed to the virtue o practical wisdom
o these boundaries has to do with preventing physician needs and the unique circumstances o the medical relationship that
and private matters rom encroaching into the visit in a way makes proselytizing most o en inappropriate. A better strategy,
that is not therapeutic to the patient and does not respect the i a physician eels that a patient might be seeking additional
physician’s boundaries. While it would appear that Dr. Pincus spiritual or philosophical direction, is to ask open-ended ques-
has therapeutic reasons— or Robert, not or hersel — or cross- tions and then make an appropriate re erral to pastoral care or a
ing the boundary o sel -disclosure, both physician motivation spiritual counselor who will be sensitive to the issues the patient
or sel -disclosure and the immediate and potential e ects o the has raised as relevant.
sel -disclosure need to be weighed very seriously be ore private
matters are revealed. I there is even a potential o harm, cross- Question 26.2.8 Which o the ollowing is true about inter-
ing the boundary in this way should be considered a violation o vening with an “impaired colleague,” like Robert?
pro essional norms. A) Impairment should be reported only to a state licensing
board i the colleague’s patients are placed at known and
Question 26.2.7 How could Dr. Pincus appropriately respond documentable risk.
to Robert’s re usal to participate in AA on the basis o his B) Because alcohol abuse is a con dential matter, it is inappro-
religious impulse? priate or a treating physician to report a colleague’s impair-
A) “AA is still shown the best intervention or preventing relaps- ment to a licensing board.
ing alcohol use. I hope you can go and get something out C) Removing a colleague rom direct patient care and increas-
o it without acknowledging your acceptance o the ‘higher ing supervision during patient care are reasonable rst-
power’ explicitly.” step interventions or a colleague who is actively engaged
B) “AA has important group support rom others who under- in substance treatment (e.g., a report has already been
stand how dif cult it is to stop using alcohol. It is not meant made).
to be religious, but rather a community o care.” D) It is pre erable to contact a state licensing board directly
C) “I have ound both AA and a theistic world-view to be very as opposed to discussing the matter with the patient
help ul in understanding my own powerlessness to control or institutional administration. his protects both the
some o my behaviors. Would it be help ul to you to hear reporter and the colleague rom unnecessary negative
more about this?” repercussions.
D) “I understand how the religious aspect o AA is inconsistent
with your own philosophy. Would you be willing to investi- Answer 26.2.8 The correct answer is “C.” Legal statutes on
gate non-religious group meetings or alcohol abusers?” reporting impaired colleagues vary rom state to state, with
some state laws making physicians mandatory reporters o
Answer 26.2.7 The correct answer is “D.” AA is an example impaired physician colleagues, while others simply recom-
o a prescribed treatment that involves an active theological mend reporting. Furthermore, state laws are even less pre-
component. AA’s rst step involves acknowledgment o a higher scriptive with regard to non-physician health pro essionals
power, traditionally invoking a speci c monotheistic concep- with impairments. Any impairment should be treated seri-
tion o the divine as a necessity to surrendering the illusion ously, pre erably with support rom the institution’s adminis-
o control. In the interest o respecting a patient’s religious tration. It is imperative to protect patients rom harm. While
rights in a diverse community, and o optimizing treatment reporting the impaired colleague may result in anger and dis-
options, it would be disrespect ul and ine ective to have the appointment rom the colleague or even supervisors who are
patient participate in AA, while ignoring the rst step o reluctant to tackle such a dif cult question, physicians should
the program and the oundational philosophy o AA. While consider the needs o vulnerable patients and the patients’
there are ewer studies about the ef cacy o nonreligious alco- rights to adequate care.
hol treatment groups, it is appropriate to respect Robert’s belie s Con dentiality adds an additional ethical dimension when
by investigating nonreligious alternatives. As to option “A,” the an impaired colleague reveals his or her impairment to his
Cochrane database concludes, “No experimental studies treating physician. In an e ort to respect patient autonomy,
unequivocally demonstrated the e ectiveness o AA.” physicians will o en urge impaired colleagues to report
694 FAMiLY MEDic iNE EXAMiNAt iON & BOARD REViEW

themselves as well as voluntarily engage in treatment proto- but there was signi cant morbidity and mortality as a result
cols. Many states have less-restrictive policies or treatment o the illnesses. It was as i God were playing with a cosmic
and monitoring or impaired colleagues who sel -report. I grenade launcher when it accidently went o and blasted
a physician intends to report her patient’s impairment with- her amily apart. “Oh, well. Genetic roulette,” she says. Anne
out the consent o the patient, the physician is obligated to be only recently became aware o this amily history when her
truth ul with the colleague about her intentions and rationale mother and aunts were diagnosed. When you ask i she has
or reporting. discussed genetic risks or breast cancer, Anne looks at you
blankly and replies, “No.”
Anne has been in a stable relationship with her boy riend,
Question 26.2.9 A colleague may be impaired in her prac-
Jordan, or 2 years. T ey cohabitate and are engaged, but have
tice by all o the ollowing EXCEPT:
not set a wedding date. Anne has completed high school and
A) Substance use.
works in telemarketing while applying to art schools. T e
B) Major depression.
pregnancy was not planned, but she and Jordan are thrilled,
C) Dementia.
even i a little nervous, about having a baby.
D) De cits knowledge.
Being an extraordinarily astute clinician, you are con-
E) Barely passing her board examination.
cerned about BRCA1 and 2 genes. In amilies with a high
incidence o breast and ovarian cancer, mutations in BRCA1
Answer 26.2.9 The correct answer is “E.” Well, she “barely are associated with an 85%li etime risk o developing breast
passed,” and that’s what she needs to achieve to be certi ed. All cancer and a 50%risk o ovarian cancer.
o the others, whether acute or chronic, may impair a health You wonder i this is the best time to bring up genetic con-
pro essional’s ability to practice, but none o these automati- cerns with Anne, given Anne’s concurrent transition with an
cally imply global impairment in medical practice. Each has unplanned pregnancy.
its own implications or a colleague’s medical practice. Special
attention should be given to the colleague’s actual and possible Question 26.3.1 Which o the ollowing is/are true about
consequences in practice, given her speci c job requirements disclosure?
and compensatory skills/supports, while assessing the presence A) Nondisclosure is not justi able due to ears that a patient
and degree o impairment. One might say that dementia is OK will be distressed by the in ormation, unless disclosure
in physicians working or insurance companies (or at least it might cause death (e.g., suicide at hearing a diagnosis o
seems so!). cancer).
B) Regardless o the consequences, nondisclosure could be
Objectives: Did you learn to . . .
considered deception and would be morally wrong on the
• iden y w a em are requ red or n lu on n e med al
re ord?
basis o this intrinsic eature.
C) Disclosure is important because it respects patient auton-
• Re ogn ze e mpor an e o pa en on den al y and
under and w en on den al y m g be broken n order o
omy and optimizes a patient’s ability to make an in ormed
ul ll o er e al obl ga on ? choice.
• Re ogn ze ob a le o pro e ng pa en on den al y?
D) Nondisclosure may be a sign o paternalism rather than
bene cence.
• De r be e mpor an e o nd v dual and o e al ru
n nd v dual p y an and e med al pro e on a a
E) All o the above.
w ole?
• De r be e e al pr n ple nvolved n el -d lo ure? Answer 26.3.1 The correct answer is “E.” T ere are a variety
• iden y an mpa red olleague and de erm ne ow o be o moral theories which comment on whether or not deception
n ervene? or nondisclosure is morally appropriate. Most theorists rely on
the principle o respect or patient autonomy, such that a per-
son who has incomplete in ormation is not able to act reely in
CASE 26.3 making a choice or hersel . “Consequentialism” is also a com-
Anne is a 19-year-old single emale presenting or her rst monly used moral theory, suggesting that it is not the intrinsic
prenatal visit. She is G1P0, and roughly 10 weeks gesta- nature o the act itsel but the consequences that ollow deter-
tion by last menstrual period. She is new to your practice. mine whether the act is good or evil (as in “the end justi es the
Anne has had no medical care at this acility and no physi- means”). In virtue ethics, by comparison, the nature and moti-
cian appointments since childhood. In recollecting the past vation o the act are very important as a re ection o the physi-
medical history, Anne reveals that she has had several rst- cian’s character and habits. In virtue ethics, deception is morally
degree emale relatives who have been diagnosed with breast blameworthy because it is comparable to lying. In virtue ethics,
and/or ovarian cancer: her mother, two maternal aunts, and motivation is also an important issue to judge the goodness o
a maternal grandmother. A great aunt also died young o the action. Answers “B” and “C” both make re erence, at least in
unknown causes. Anne is unsure o the workup that they had, part, to virtue theory.
CHAPTER 26 • Et h ic s 695

Question 26.3.2 Which o the ollowing is FALSE about test- uture.” Anne replies: “What are you saying? Will you leave
ing or genetic conditions? me i I have the gene? I can’t raise this baby by mysel !”
A) In ormed consent or genetic testing should be taken more
seriously and ormally than other kinds o blood testing, Question 26.3.3 Which o the ollowing statements are
such as obtaining a hemogram. appropriate to consider in promoting the patient’s best
B) Physicians should ask patients what they would do with the interests?
di erent possible outcomes o the genetic test be ore the test A) Patients are vulnerable.
is per ormed. B) Physicians have expertise that patient’s lack.
C) Physicians should make a recommendation regarding C) Patients rely on their physicians.
genetic testing guided by evidence-based medicine and the D) Physicians and patients o en agree on what constitutes a
patient’s speci c narrative and values. patient’s best interests, although they may di er in the way
D) Physicians should urge patients to disclose positive results to they plan to meet those interests.
relatives or spouses i the in ormation is pertinent medically E) All o the above.
or emotionally to these third parties.
E) Physicians should never disclose genetic in ormation to a Answer 26.3.3 The correct answer is “E.” T e nature o a rela-
third party without the consent o the patient. tionship between a physician and a patient may have as many
permutations as there are individuals. However, it is impor-
Answer 26.3.2 The correct answer is “E.” Genetic testing tant to appreciate the position o the patient and the need that
di ers rom other blood tests because o multiple actual and has pushed her to seek care. Patients are vulnerable in many
potential risks, including personal e ects on the patient and her ways, and the vulnerability is enhanced by limited access to
amily, as well as discrimination by employers or insurers. T ere technological and scienti c in ormation. When external vari-
is a shortage o ormally trained genetic counselors, and patients ables, such as Jordan’s comment and Anne’s response, come
rely on their physicians to not only help guide their decision into play, physicians should pay attention to this narrative
making about whether or not to per orm the test, but what to and take some responsibility or establishing and maintaining
do with the in ormation obtained. Because such testing has pro- a supportive network even outside o the of ce. T is is par-
ound medical and/or psychosocial e ects on the patient and ticularly important as physicians give patients in ormation
amily, a discussion about disclosure should happen both be ore about dif cult choices. While physicians and patients may
and a er the test is obtained. o en be able to negotiate a mutually acceptable alternative, an
Con dentiality is important or many reasons, not only in active dialogue is important. Supports and advocates who are
establishing and maintaining a good physician–patient relation- amiliar with the patient’s values and wishes can be an impor-
ship and respecting patient autonomy but also because o the tant adjunct to medical decision making, as long as there is
potential discrimination and misuses o genetic in ormation no material or psychological con ict o interests. Physicians
in today’s culture. However, when the risk o harm to another should not adopt a completely hands-o policy in decision-
related person is high and the patient re uses to disclose impor- making; rather, physicians should pay attention to supporting
tant genetic in ormation, there may be adequate cause to break the patient with real options and evidence-based variables in a
con dentiality in order to prevent serious harm to the third noncoercive, empowering relationship.
party; there ore, “E” is a alse statement.
Anne decides not to have the test, but to have an elective
abortion “just in case I passed on a gene to the baby.” For the
HELPFUL TIP: sake o argument, you are philosophically opposed to elective
Many d agno and reen ng e (e.g., h iVan body, abortions in this scenario, but you would consider abortion
Ps A, and b op e ) ould be approa ed l ke gene an appropriate intervention i the etus tests positive or the
e , a ur ng a e pa en a a lear under and- gene by chorionic villus sampling.
ng o e mpl a on o e e , n lud ng ur er
d agno e ng, erap e , and progno . “W o a Question 26.3.4 What is the best ethical option or you at
e me?” you a k. We omm era e, bu ll . . . this point?
A) Explain that you are personally uncom ortable with abor-
tion, but in de erence to Anne’s legal rights, you will make
You decide to disclose the possibility o genetic risk actors a re erral to another physician who is willing to provide the
to Anne at the rst prenatal visit and also discuss the risk elective abortion.
o passing genes to the etus. Anne seems overwhelmed and B) Re use to per orm or make a re erral or the abortion.
asks to bring Jordan to the next visit to discuss this urther. C) Re er to a “pro-li e” counseling agency.
When Anne returns with her ancé, you discuss your con- D) ell Anne, or reasons that you do not eel com ortable
cerns about the BRCA1 and 2 genes and why Anne’s am- disclosing, you will no longer be able to care or her.
ily history is suspicious. Jordan says, “I think you should E) Per orm the elective abortion, despite personal convictions,
be tested right away, Anne. T is would totally change our out o respect or the law and patient autonomy.
696 FAMiLY MEDic iNE EXAMiNAt iON & BOARD REViEW

Answer 26.3.4 The correct answer is “A.” Abortion is a ercely Answer 26.3.5 The correct answer is “C.” At the time o pub-
contentious topic in the United States. Under the 1973 Roe v. lication, there is ongoing discussion about how to regulate use
Wade decision and in subsequent rulings such as Planned Par- o tissue samples. While it might seem that this is an ethical
enthood v. Casey, the Supreme Court has af rmed a woman’s question ar removed rom the purview o the amily physician,
legal right to abort a etus. Physicians have responsibilities that patients in amily practice clinics are very requently targeted
should transcend views about a physician’s own moral values, or various research protocols due to their regular ollow-up and
such as ensuring that in ormed consent is practiced and that the easy accessibility.
patient has medical care available. In ormed consent requires a Patients donating tissue samples o en give little thought to
physician to provide the necessary in ormation about the vari- what happens to those tissues a er they are obtained. In many
ous medical choices available and to assess the patient’s emo- cases, tissues are banked inde nitely a er the initial research is
tional needs. Coercion and ailure to disclose clinically relevant conducted, with various identi ers linked to the tissue poten-
in ormation is inappropriate; or example, a re erral to a pro-li e tially including the donor’s gender, geographic location, educa-
group without in orming the patient o the counseling center’s tional level, amily history, or other private in ormation (e.g.,
perspective (when known) is a orm o manipulation and ail- T e Immortal Life of Henrietta Lacks). While e orts are made to
ure to disclose. Abandoning the patient (“D”) without a simple respect the privacy o the donor, there are ways to track down
explanation is disrespect ul, although the physician should be the donor using even the limited identi cation in ormation
care ul not to coerce her in other ways while revealing personal associated with the stored sample. Such means are especially
values/belie s. acilitated by the wide availability o personal in ormation via
Physicians should note that in some states there have been the Internet.
rulings requiring physicians to participate in elective abortions i issue samples may be collected or one purpose, but later
working in a public aid clinic or in an area with limited physician used or another. issue samples are very important in research
resources where trans er o care is not an option. Some physi- and are o en the limiting actor or studies, but should in ormed
cians or institutions (such as Catholic hospitals with clear policies consent include asking donors or permission or each and
in uenced by theological statements) have practiced conscien- every lab test run on the tissue sample? At what point, i any,
tious objection and been subject to discipline o various orms, does the tissue sample become the sole property o the lab? In
including legal sanction. Physicians should be mind ul o com- the example o embryos, discarded embryos are sometimes sold
peting moral values, seeking support and pro essional guidance to private labs; parents using IVF technology are o en unaware
in dif cult moral and legal cases such that they can act with integ- that such embryos may be used to establish a stem cell line
rity and purpose in their roles as physicians and moral agents. rom which genetically identical embryos can be created using
nuclear trans ers. Such stem cell lines are highly lucrative or
T ree years later, Anne is seen in a new clinic. She had the research purposes, and there has been discussion, or example,
abortion. Anne is now an art student and is married to Jor- o whether parents should be compensated in some manner
dan. Since her last clinic visit, Anne has had a prophylactic when a lab sells their discarded embryos or a research acility
mastectomy ollowing a positive test or the BRCA1 gene. She develops a product using the embryonic stem cells.
has also had an elective tubal ligation. Anne wants to consider
in vitro ertilization (IVF) and has a riend, Jessica, who is
willing to donate ova. T ey put 18 embryos into cold storage, HELPFUL TIP:
using Jessica’s ova and Jordan’s sperm. Anne has 6 embryos Fam ly p y an ould on nue o erve a advo-
implanted, with the result o two etuses that are carried to a e or e r pa en . You an do o by nve ga ng
term. Anne decides that she does not want any more children e pol e and pro edure o re ear group pr or o
and contacts the lab to discard the remaining embryos, elimi- allow ng a e o pa en and by ak ng ome re pon-
nating storage costs. T e lab agrees and sells the embryos to a b l y or e n ormed on en pro e w en pa en
private lab, where stem cell research is under way. are volun eer ng o par pa e n re ear .

Question 26.3.5 What is NOT true about stored tissue


samples?
A) T e embryos are considered Anne’s property only as long as Question 26.3.6 All o the ollowing are legitimate ques-
she claims them. tions and concerns about the consequences o human
B) Samples used or research purposes are potentially identi - somatic cell nuclear trans er, commonly called human
able by third parties as belonging to Jessica and Jordan. cloning, EXCEPT:
C) issue samples may be used only or their initial intended A) Will cloning result in increased miscarriages and de ormed
purpose, a er which time they must be destroyed. etuses, due to limitations o current technology to per orm
D) T ird parties, such as research labs, upon discovering genetic nuclear trans er?
anomalies in tissue samples, have no legal obligation to nd B) Will cloning result in a culture preoccupied with “designer
and in orm Anne, Jordan, and/or Jessica. babies?”
E) Embryos sold to a private lab may be used to establish germ C) Will cloning result in a culture that devalues persons with
lines via destruction o the embryo. disabilities?
CHAPTER 26 • Et h ic s 697

D) Will cloning increase reproductive options or same-sex Ea pa en and ea l n al que on mu be a e ed nd -


couples, diminishing value o traditional settings or repro- v dually o de erm ne a pa en a de on-mak ng apa y
duction? n a par ular r um an e. t e ame pa en may be able
E) Will cloning result in the mass production o stormtroopers o de de no o ollow a med ally nd a ed d e bu be
n apable o de d ng own d po on a er o p al za on.
or Chancellor Palpatine’s army, thus solidi ying his control
over the galaxy, allowing him to create the First Galactic Engage pa en n ared de on-mak ng regard ng
Empire? reen ng and d agno e a well a med al pro edure
and rea men .
Answer 26.3.6 The correct answer is “E.” T e ethics o Star P y an mu a n an e al manner n order o ma n a n
Wars, should not be applied to real, live patients. All the other nd v dual ru n e p y an a well a o e al ru n e
choices have been raised as concerns, although there are many pro e on.
more arguments on both sides o the discussion about the rela- Re ogn ze a pa en ave a r g o non ompl an e. W le
tive risks and bene ts. e p y an’ re pon b l y o prov de e be are re om-
menda on or a pa en , e pa en may re u e o ollow e
Objectives: Did you learn to . . . adv e o er pr or e (e.g., money, om or , avo dan e o drug
• De r be e e al ue regard ng d lo ure? de e e ) ou rank eal n e pa en ’ de erm na on.
• Appre a e e many ompe ng n ere nvolved n gene t ake grea are n e ran m on o pa en n orma on.
e ng? h av ng a axed, e-ma led, or ma led repor on a n ng a
• iden y pa en and p y an a or a af e e pa en – pa en ’ on den al med al n orma on m d re ed o an
p y an rela on p? un n ended re p en not legally pro e ed.
• F nd an e al and a ep able way o d agree w a pa en
and on nue o a ure a pa en ’ eal are?
BIBLIOGRAPHY
American Medical Association. Code o Medical Ethics.
Clinical Pearls Accessed on September 12, 2015, Available at
An mpa red med al pro e onal mu no be gnored. http://www.ama-assn.org/ama/pub/physician-resources/
P y an are bound by e and a e law o pro e medical-ethics/code-medical-ethics.page?
pa en and o e y by den y ng, a ng, and repor ng Beauchamp L, Childress JF. Principles of Biomedical Ethics.
mpa red med al pro e onal . 4th ed. New York, NY: Ox ord University Press; 1994.
A k pa en abou advan ed d re ve per od ally a ou pa- Campbell A, et al. Medical Ethics. 3rd ed. South Melbourne,
en v and w every o p al adm on. D u l v ng Australia: Ox ord University Press; 2001.
w ll, durable power o a orney or eal are, re u a on Lo B. Resolving Ethical Dilemmas: A Guide for Clinicians. 2nd ed.
w e , and pe n erven on (e.g., eed ng ube u e) n Philadelphia, PA: Lippincott Williams & Wilkins; 2000.
e appropr a e e ng. Smith HL, Churchill LR. Professional Ethics and Primary Care
Medicine. Durham, NC: Duke University Press; 1986.
End-of-Life Care
Michelle T. Weckma a d Ma ti O. Bazelak
27
decisions. It is important to talk about the ICD, ideally be ore
CASE 27.1 a patient approaches the nal weeks/days o li e. Many patients
A 75-year-old man is admitted to the hospital or a heart choose to have it disabled once they enroll in hospice. I an ICD
ailure exacerbation. T is is his second admission or heart is not disabled, it is important that the hospice sta be aware
ailure within the last 6 months. His nurse asks i he is an o it and know what to do as it will re when the patient dies,
appropriate candidate or hospice care. which can be distressing or both the patient and the amily.

Question 27.1.1 Which o the ollowing would quali y your HELPFUL TIP:
patient or the Medicare hospice bene t? As opposed to a ICD, a pacemake does ot cause dis
A) His cardiac ejection raction is 20%, and he is dyspneic with t ess (shocks) du i g the dyi g p ocess; howeve , i the
moderate exertion (NYHA class II heart ailure). patie t’s ca diac hythm is completely depe de t o
B) He agrees to a do not resuscitate (DNR) status in the event o the pacemake , the the pacemake is co side ed to be
cardiorespiratory ailure. li e p olo gi g.
C) He needs assistance with ambulation.
D) His implanted cardiac de brillator (ICD) has red once in
the past year. HELPFUL TIP:
E) He has had escalating cardiac hospitalizations despite opti- Although p og osis may be the most di icult task a
mal medical management. physicia aces, a attempt at p og osis i patie ts
with a te mi al ill ess may help them. The simple ques
Answer 27.1.1 The correct answer is “E.” I your patient has a tio “would you be su p ised i this patie t died i
terminal disease with declining unction, worsening symptoms, the ext yea ?” is a good sc ee i g tool a d has bee
worsening laboratory tests, or escalating hospitalizations, he or show to be a easo ably accu ate way to p edict who
she probably quali es or hospice. “A,” a low cardiac ejection might be e it om a hospice e e al.
raction (typically < 20%), is a recommended criterion or heart
disease but must be accompanied by dyspnea at rest (class IV
heart ailure). “B” is incorrect. Despite popular belie , a hospice Your heart ailure patient is currently on maximal medi-
patient need not agree to a DNR status; however, hospice agen- cal management with an ACE inhibitor, beta-blocker, loop
cies are permitted to have di erent admission criteria and some diuretic, aldosterone antagonist, long acting nitrate, hydrala-
require a DNR status or admission. Check with your local hos- zine, and a dual chamber ICD. He wants to discuss the man-
pice agencies. Regardless, you should have a rank discussion agement o his cardiac therapies in relation to hospice.
with the patient about the role o aggressive resuscitation and
the goals o hospice care. “C” is also incorrect. While a decline Question 27.1.2 Which statement is true about cardiac
in unctional status is a strong indication o worsening progno- therapies or patients with heart ailure in hospice?
sis, it is not automatically a criterion. Most patients with heart A) Hospice will not enroll patients on intravenous inotropic
ailure need assistance with at least our activities o daily living support (such as dobutamine or milrinone in usions).
(ADLs) be ore having a prognosis o death within 6 months. B) Oral cardiac medications like beta-blockers and diuretics
“D” is incorrect. Patients with ICDs are eligible or hospice, should be stopped because they are unlikely to provide any
and whether or not it has red does not impact enrollment bene t to quality o li e.

698
CHAPTER 27 • En D-O -LI E CAr E 699

C) A uid restriction is no longer relevant once a patient enrolls


in hospice. You think that your patient may be appropriate or hospice
D) Hospice will not enroll patients who have not deactivated and decide that a palliative medicine consult might be use ul
their ICD. to address his goals o care, review symptom management,
E) None o the above and discuss hospice.

Answer 27.1.2 The correct answer is “E” none o the above. Question 27.1.3 Which statement accurately re ects how
Many o the therapies or patients with advanced heart ailure palliative medicine is dif erent rom hospice?
provide symptomatic relie and improve quality o li e. Just A) Only palliative medicine af rms li e and attempts to help
as you would do with any other advanced illness, a thorough patients live as ully as possible.
evaluation o each medication and intervention should occur B) Palliative medicine o ers treatments including aggressive
with the goal o determining i that intervention will contribute medical treatments such as radiation therapy, intravenous
to your patent’s overall goals. Physicians need to negotiate the (IV) uids, or interventional radiologic procedures, while
balance between bene t and burden o each individual ther- hospice does not.
apy. Although o en a surprise to physicians, “A” is incorrect. C) Palliative medicine is per ormed by a physician only, while
A patient can have continuous inotropic in usion and still be hospice care involves an entire team o providers.
enrolled in hospice. Some heart ailure patients are dependent D) Palliative care can be o ered at any time during a li e-limit-
on inotropes and require them to maintain a certain level o ing illness, even i a patient is still actively seeking a cure.
quality o li e, and they are willing to accept the trade-o s o E) Palliative medicine consults are billed under Medicare Part
having a continuous in usion to maintain that quality o li e. Be B while hospice care is reimbursed through the Medicare
aware that inotrope in usions do not prolong survival (in act Part H.
they shorten survival compared to no inotropes). T e burden
o inotrope in usions include: cost, risk o line in ections, and Answer 27.1.3 The correct answer is “D.” Palliative care is spe-
increased risk or arrhythmia. cialized medical care or people with serious illnesses that is
Hospice agencies are required to provide treatment to man- ocused on providing patients with relie rom the symptoms,
age symptoms related to the terminal condition and many hos- pain, and stress o the serious illness—whatever the diagnosis.
pice agencies utilize ormularies in order to control cost. Some T e goal is to improve quality o li e or both the patient and the
hospice agencies have ormularies that will allow the use o ino- amily. Palliative care is provided by a team o doctors, nurses,
tropes or management o dyspnea, atigue, etc., while others and other specialists who work with a patient’s primary team to
will speci y that the symptoms be controlled with other agents provide an extra layer o support. Palliative care is appropriate at
such as opiates. any time in a serious illness and can be provided together with
“B” is incorrect. T ere is so much more to palliative care curative treatment. Hospice care is a type o palliative care, which
than giving morphine or shortness o breath in heart ailure! occurs during the nal months o someone’s li e when cure is no
Continuing cardiac therapies may allow a patient to maintain longer possible or no longer consistent with the patient’s goals.
their cardiac unctional status and prevent pulmonary conges- “A” is incorrect because both hospice and palliative care strive to
tion and should never be discontinued just because a patient make li e meaning ul and com ortable or patients. “B” is incor-
enrolls in hospice. With that being said, there are times when rect. Since the goal o both hospice and palliative care is to pre-
the patient’s goals support the discontinuation o these medi- vent and relieve su ering, any o these modalities may be used
cations. “C” is incorrect because as noted above, every deci- i appropriate. In act, any intervention can be considered pal-
sion comes with both a bene t and a burden, which need to be liative i the intention is to relieve distressing symptoms associ-
weighed. Some level o uid restriction may help with breathing ated with the atal disease (such as a hip-pinning or pain control
and pulmonary congestion and should be balanced against the a er a pathologic racture). T ere are no restrictions prohibit-
discom ort o restricting intake. “D” is incorrect because it is ing a hospice agency rom providing “aggressive” interventions
not a requirement or an ICD to be deactivated. However, the designed to relieve pain and su ering. “C” is incorrect because
incidence o arrhythmia can be high in this population and both hospice and palliative medicine are built on the concept o a
ICD shocks near the time o death can be very distressing to the team providing care to the patient and amily. Hospice care usu-
patient and the amily and should be discussed. ally (but not always) is directed by the primary physician. “E” is
incorrect because palliative medicine services are typically billed
as inpatient consult services and are reimbursed by Medicare
HELPFUL TIP:
Part A, and there is no such thing as Medicare Part H—yet!
I you’ve see o e hospice age cy, you’ve see o e hos
pice age cy. Hospices o te have di e e t o mula ies,
di e e t available a cilla y se vices (e.g., music the apy, Question 27.1.4 Which o the ollowing is TRUE regarding
pet the apy, massage), a d di e e t policies i what the Medicare hospice bene t?
they cove o symptom ma ageme t (e.g., chemothe A) T e Medicare hospice bene t requires all care related to
apy, adiatio , blood t a s usio s). It ca be help ul to the primary admitting illness to be covered by the hospice
k ow what se vices you local hospice cove s. agency, including physician ees, as long as the hospice pro-
gram does not employ the physician.
700 AMILY MEDICIn E EXAMIn ATIOn & BOAr D r EVIEW

B) T e hospice bene t, similar to the Medicare home care ben-


e t, requires that the patient be homebound. A ew months go by and your patient has again been hospi-
C) Respite care within an acute hospital setting is provided or talized. You now believe that he has ewer than 6 months to
up to 5 days every 6 months. live. T e local hospice medical director reviews the case and
D) Hospice entry requires that the patient cannot live alone agrees. T e patient is dyspneic at rest, requires 24-hour oxy-
(must have an on-site primary caregiver). gen by nasal cannula, and his cardiac ejection raction has
E) A at per diem rate is paid to the licensed hospice program decreased to 15%. He wishes to avoid urther hospitalization
regardless o the level o services provided. and has elected to have a DNR status. His amily wants to
know what you can do when he becomes severely dyspneic.
Answer 27.1.4 The correct answer is “A.” T e Medicare hos-
pice bene t requires all care related to the primary admitting Question 27.1.6 Which o the ollowing is appropriate pal-
illness to be covered by the hospice agency. T ink o it as capi- liative treatment o severe dyspnea in this patient?
tated care. However, Medicare a ords individual hospice agen- A) Intubation and ventilation.
cies wide latitude in determining what modalities they use to B) Morphine.
treat the symptoms o a particular illness. For example, one C) Scopolamine.
agency might allow blood trans usions or the relie o dyspnea, D) Buspirone.
while another agency only covers the use o medications such E) Hyperventilating into a paper bag.
as morphine or lorazepam or dyspnea. I necessary or acute
symptom management, Medicare also covers acute hospitaliza- Answer 27.1.6 The correct answer is “B.” Morphine and other
tions related to the primary illness. “B” is incorrect. A hospice opioids analgesics are indicated or palliation o dyspnea in car-
patient does not need to be homebound. Respite care can be diac ailure. Despite concerns about opiates worsening respira-
o ered as requently as needed, which makes “C” incorrect. T e tory unction in end-stage heart ailure, studies have concluded
Medicare hospice bene t provides respite care. T is can last up that oral morphine improves dyspnea in this patient population
to ve nights at a time. T e respite care needs to be provided in a and does not cause respiratory ailure or hasten death. T is is
setting where there is a registered nurse available continuously, supported by recommendations rom the American College o
and the patient must come rom a home setting (including Cardiology and the American Heart Association. I morphine
assisted living). T is can be used i the caregiver is unable to care or other therapies or shortness o breath were to shorten li e,
or the patient or a variety o reasons. “D” is incorrect because the ethical principle o “double e ect” would be applicable. T e
Medicare does not require that the patient have a primary care- double e ect principle states that as long as the intention o a
giver in the home to receive the hospice bene t; however, an therapy is to reach a good outcome (such as improve pain and
individual hospice agency may require a primary caregiver be alleviate su ering), then even i the unintended consequences
identi ed be ore enrolling a patient. “E” is incorrect because result in a negative outcome (e.g., shortening li e) the inter-
Medicare pays the hospice organization a strati ed per diem vention is permissible. T e key here is intention. I you choose
rate depending on the level o care the patient requires. “A,” your patient’s ghost will be back to haunt you. T is patient
has chosen a ar less aggressive stance toward li e-prolonging
A er meeting with the palliative care team, your patient’s measures, and his wishes should be honored. “C” is incor-
amily inquires more about hospice care. T ey ask what other rect. Scopolamine is used to reduce oropharyngeal secretions
services are covered. (resulting in sonorous respirations) but is not likely to relieve
dyspnea in this patient. In addition, scopolamine is highly
Question 27.1.5 The Medicare hospice bene t includes anticholinergic and crosses the blood–brain barrier, which can
coverage or all o the ollowing expenses EXCEPT: result in delirium and should be used with caution in an awake
A) Medications related to symptom management. and alert patient. Glycopyrrolate is a better choice to control
B) Social work visits. secretions in a patient who is alert and awake. “D” is incorrect.
C) Home health aide services. While anxiety and dyspnea o en occur together and exacer-
D) Room and board or a patient living in a nursing home. bate each other, buspirone (BuSpar) is a weak anxiolytic and
E) Bereavement services. has no direct e ect on dyspnea. Low-dose lorazepam would be
a reasonable alternative i you elt his dyspnea was complicated
Answer 27.1.5 The correct answer is “D.” All o the other by anxiety. And “E” is just plain wrong—although still not as
options listed are services covered under the Medicare hospice bad a choice as “A.”
bene t. A patient living in a nursing home is eligible or hospice Consider diuretics to relieve dyspnea in heart ailure patients
care, but the cost o room and board or the nursing home is in hospice. O en times we are guilty o orgetting to treat the
not paid by the hospice bene t. In some special circumstances, underlying illness when someone is in hospice, reaching or
nursing home care will be paid or a short duration (e.g., when morphine rst. Are there signs o volume overload (e.g., ele-
a hospice patient uses the ve-night respite care bene t or i vated JVD, di use crackles, pro ound edema)? Diuretic therapy
the patient is admitted to a nursing home or management o may reduce pulmonary congestion and relieve dyspnea, and the
a distressing symptom that cannot be controlled in the home bene ts and burdens should be weighed. T e burdens that need
setting). to be considered include: patient’s bed bound status, risk or
CHAPTER 27 • En D-O -LI E CAr E 701

incontinence decreasing quality o li e, patient’s agreement with the care recommended by her oncologist. You recall reading the
using a bladder catheter, renal unction, etc. landmark study (a study that every primary care and palliative
care physician should know—and i you don’t know you should
read it cover to cover several times) by emel et al., regarding
HELPFUL TIP: the introduction o early palliative care in patients with meta-
Be zodiazepi es may be used i combi atio with opi static nonsmall-cell lung cancer.
ates o added elie o dysp ea a d a xiety. The patie t
should be mo ito ed o excessive sedatio a d espi a Which o the ollowing is NO true about the outcomes
to y dep essio . or the patients that received the novel intervention o early
palliative care compared to those that received standard
oncologic care?
Objectives: Did you learn to . . . A) Introduction o early palliative care improved patients’
• Ide ti y app op iate patie ts o e d o li e ca e, pa ticula ly quality o li e.
o the Medica e hospice be e t? B) Introduction o early palliative care improved patients’
• Desc ibe the eatu es o the Medica e hospice be e t? mood.
• T eat patie ts with dysp ea elated to e d stage ca diac C) Introduction o early palliative care shortened the median
disease? survival.
D) Introduction o early palliative care resulted in ewer aggres-
sive therapies at the end-o -li e.
QUICK QUIZ: MOr E THAn A LITTLE HEADACHE
The correct answer is “C.” Introduction o early palliative care
A 30-year-old male patient you have known or several years was shown to increase median survival rom 8.9 months to
was diagnosed with metastatic melanoma several months ago. 11.6 months, and this occurred despite the intervention group
He presents now with intermittent, severe headaches associated patients receiving ewer aggressive interventions at the end-
with nausea. A head C scan per ormed last week showed three o -li e. Both “A” and “B” are incorrect answers (but true state-
metastatic oci with surrounding edema. He currently takes ments) because this study showed that both quality o li e and
maximum doses o acetaminophen and large doses o mor- mood were improved when early palliative care was introduced
phine. to the standard oncologic care in this patient population. “D” is
incorrect (but again a true statement) because 33% o patients
What is the best initial treatment option to help relieve his head- who received early palliative care received “aggressive care”
ache and nausea? (chemotherapy within 2 weeks o death, no hospice care, and/
A) Neurosurgical intervention. or admission to hospice 3 days or less be ore death) as com-
B) Dexamethasone. pared to 54% o patients in the standard care group. T e emel
C) Sumatriptan. study applies to a very speci c patient population—patients
D) Ibupro en. with metastatic nonsmall-cell lung cancer; however, there are
E) Increased morphine doses. ongoing studies in other advanced illnesses to see i these out-
comes can be reproduced and applied to larger populations
The correct answer is “B.” Corticosteroids are the pre erred o patients. In act, there is current research being done at the
therapy or headaches due to increased intracranial pressure University o Iowa introducing early palliative care to patients
rom edema, as is presumably the case here. Whole brain or ste- with end-stage-liver disease and measuring the outcomes o
reotactic radiation therapy, alone or in combination with cor- mood, quality o li e and mortality. Keep an eye out in the lit-
ticosteroids, may also be used as palliative therapy or multiple erature because the novel therapy o early palliative care may
brain metastases. “A” is incorrect because neurosurgical con- prove to be applicable to more patient populations over the
sultation or craniotomy should be reserved or patients who next several years.
ail other interventions or who present with rapidly worsening
symptoms. “C” and “D” are incorrect. Sumatriptan and ibupro- CASE 27.2
en should not be used to treat increased intracranial pressure.
“E,” increased morphine doses, may be required, but corticoste- A 74-year-old emale was diagnosed with adenocarcinoma
roids are typically used rst. o the colon 3 years prior to beginning hospice care. She has
known metastases to her liver and pelvis. She complains o
a cramping pain in her abdomen and a “deep pain” in her
QUICK QUIZ: EAr LYPALLIATIVE CAr E groin. She is currently receiving morphine 10 mg orally (PO)
every 4 hours (except when asleep) and acetaminophen 650
A 74-year-old woman presents to your clinic with a new diag- mg PO ID. She says that her pain is 4 out o 10 on a numeric
nosis o nonsmall cell lung cancer ound during an evaluation pain scale. Her past medical history includes hemorrhage
or hip pain that revealed di use bony metastases on x-ray. She secondary to a gastric ulcer 4 years ago (Helicobacter pylori
inquires about ways to improve her quality o li e in addition to negative on biopsy).
702 AMILY MEDICIn E EXAMIn ATIOn & BOAr D r EVIEW

Question 27.2.1 Which o the ollowing strategies is the pain is the musculoskeletal pain (e.g., sports injuries and rac-
best next step or improving this patient’s pain control? tures) that everyone experiences at some point. Neuropathic
A) Start gabapentin to treat the neuropathic aspect o her pain. pain is generally described as burning and results rom nerve
B) Add a strong nonsteroidal anti-in ammatory drug (NSAID) damage, in ammation, or compression. Visceral pain comes
such as ketorolac ( oradol) to her current regimen. rom distention o an organ capsule (e.g., liver enlargement
C) Add scheduled tramadol to try to decrease her morphine rom metastases, and colicky pain rom the colon). Depending
use. on where a metastasis or primary tumor is located, it may cause
D) Start a long-acting morphine (e.g., MS Contin) at a dose somatic pain (e.g., bone tumors), neuropathic pain (e.g., Pan-
20% higher than her current morphine use. coast tumor), visceral pain (e.g., peritoneal carcinomatosis), or
E) Maximize the dose o acetaminophen to 1,000 mg every a headache rom increased intracranial pressure.
4 hours.
HELPFUL TIP:
Answer 27.2.1 The correct answer is “D.” T e patient is taking
Avoid usi g mepe idi e (Deme ol). It has toxic metabo
short-acting opiates around the clock, which can lead to “chasing
lites that may cause agitatio a d seizu es. Mepe idi e
the pain” (not to be con used with “chasing the dragon,” which
ca also i te act with a umbe o d ugs to cause se o
is inhaling heroin smoke, which leads to a leukoencephalopa-
to i sy d ome.
thy . . . you never know . . . it could be on the test!). Patients o en
receive much better pain control when they are maintained on
a long-acting opiate. It is common practice to increase the dose
o opiates by 15% to 25% when a patient’s pain is only partially You decide to increase her morphine dose.
controlled. It is then recommended to have the breakthrough
dose equivalent to 10% o her 24-hour use. “A” is incorrect as her Question 27.2.3 Which o the ollowing statements is TRUE?
pain is better described as visceral or somatic rather than neuro- A) Intravenous morphine is 10 times more potent than oral
pathic. Gabapentin is an acceptable choice or neuropathic pain morphine.
but is not likely to be e ective or this patient. “B” is incorrect. B) Naloxone should be given i a hospice patient near death
Ketorolac is the NSAID with the highest rate o renal disease demonstrates con usion, decreased responsiveness, a slowed
and gastrointestinal (GI) bleeds and is contraindicated in a respiratory rate or cool extremities.
patient who has had a GI bleed. I you were to add an NSAID, C) Patients exhibiting a local rash or intense pruritus at the site
ibupro en would be a better choice, and GI protection with a o intravenous morphine administration must be consid-
proton pump inhibitor would be advisable or this patient. “C” ered allergic and given an alternate opiate.
is incorrect. ramadol is a weak mu-receptor agonist, possess- D) olerance to morphine does not occur in patients with can-
ing some norepinephrine and serotonin reuptake inhibition and cer, so any increased analgesic need is due solely to uncon-
is much less ef cacious than morphine. ramadol is not likely to trolled pain.
bene t a patient with severe pain rom cancer and metastases. E) Renal and hepatic insuf ciency both contribute to the ac-
ramadol also has a number o troubling side e ects such as an cumulation o morphine and its metabolites.
increased risk o seizures and serotonin syndrome. “E” is incor-
rect because the current recommended daily maximum dose Answer 27.2.3 The correct answer is “E.” “A” is incorrect.
o acetaminophen or elderly patients is 3,000 mg (don’t blame Intravenous (IV) morphine is about three times as potent as
us . . . blame the FDA), and the requency suggested in “E” would oral morphine. “B” is incorrect. Patients who are within several
ar exceed this amount. days or hours o death o en exhibit pallor, peripheral vasocon-
striction, apneic episodes, and obtundation as part o the physi-
When determining what medications are appropriate or ologic process o dying. Counseling the amily and patient is
treating pain, it helps to know what type o pain the patient pre erable to administering naloxone, which can cause abrupt
has. opiate withdrawal and signi cant distress and discom ort to
the patient (again, don’t make her ghost haunt you). “C” is
Question 27.2.2 Cancer is known to cause which type(s) o also incorrect. Local histamine release is a known e ect o IV
pain? morphine administration. Histamine-mediated skin changes
A) Neuropathic pain. proximal to the IV in usion site o morphine do not represent a
B) Visceral pain. contraindication to uture morphine use. In act, antihistamines
C) So tissue/bony pain. (e.g., diphenhydramine) can be used to counter morphine-
D) Pain rom increased intracranial pressure. related histamine e ects, such as rash, itching, and hypotension.
E) All o the above are types o pain. But again, i you choose to use an antihistamine you need to
monitor or delirium. “D” is incorrect. All patients can develop
Answer 27.2.2 The correct answer is “E.” Physiologic pain is tolerance to the e ects o morphine. O course, in patients
separated into our categories: so tissue or bony pain (also with terminal cancer, increasing opiate requirements are o en
called somatic pain), neuropathic pain, visceral pain, and the entirely due to increasing pain. And even i your dying patient
pain o increased intracranial pressure. An example o somatic develops morphine tolerance, you still need to treat the pain.
CHAPTER 27 • En D-O -LI E CAr E 703

Finally, “E” is correct. Patients with renal and hepatic insuf - situations, it can be help ul to rotate opiates because the lack o
ciency can accumulate metabolites o morphine, some o which cross-tolerance means that the dose can o en be decreased while
are help ul in pain control and others o which may have an maintaining the same level o pain control. Acetaminophen is
anti-analgesic e ect. T ese patients o en need lower doses o the sa est opiate-sparing analgesic, and its adjuvant action may
opiates. I a patient is actively dying and has evidence o renal allow or opioid dosage reduction without a loss o overall anal-
ailure, opioid doses can o en be decreased signi cantly with- gesia. In the appropriate patient, NSAIDs may also be used.
out a worsening o pain control. Fentanyl and methadone are “C” is true. When a patient chronically taking one opiate
considered the sa est opiates in renal ailure although metha- switches to another, a dose adjustment calculation must be made.
done use is complicated by the long hal -li e. You cannot switch milligram or milligram. Also, some authori-
ties recommend that a er the calculation, you slightly reduce the
dose o the new opiate due to incomplete cross-tolerance (re er
HELPFUL TIP: to opioid dose conversion charts available in numerous pharma-
Sta t patie ts o a egime to p eve t co stipatio ceutical texts and handbooks). “D” is true. T e transdermal en-
whe i itiati g opioids. Thi k about the bowel egime tanyl patches, though convenient, have uctuating bioavailabil-
each time you i c ease the opiate dose. It will save you ity over the three days that each patch is worn, and breakthrough
a d you patie t a lot o g ie i the lo g u . doses o an alternative opiate should be available. Fentanyl is also
at-soluble and absorption/bioavailability is decreased in thin
and/or cachectic patients. T e entanyl patches are expensive
HELPFUL TIP: and initially have a slow onset until a steady state is achieved. For
o patie ts whose pai ca ot be co t olled with typi this reason, a entanyl patch should never be used alone as an
cal t eatme ts, lidocai e i usio s have bee show to initial treatment o acute pain. In act, i you want to avoid a law-
be be e icial. suit, a entanyl patch should never be used as the rst method to
treat pain in an “opiate-naïve” patient. “E” is true. Because o the
extraordinarily wide dosage range o opiates, the ceiling dosage
Question 27.2.4 Which o the ollowing statements is NOT cannot be calculated or assumed. Rather, analgesic requirements
accurate regarding the appropriate use o opiates in end- allow or continual increase unless adverse side e ects clearly
o -li e situations? undermine the use o the drug.
A) At times, delirium can be improved with opiate dosage
reduction and/or the addition o opioid-sparing analgesics
(e.g., acetaminophen). HELPFUL TIP:
B) I oral morphine cannot be swallowed, then either an enteral The e is o co siste t elatio ship betwee blood lev
eeding tube or a parenteral route (IM/IV/subcutaneous) els o mo phi e a d a algesic e ects. This is because
must be used. o tole a ce, i dividual va iability i d ug e ect, etc.
C) Dosage conversion rom one opiate to another is a ected by Thus, the e is o si gle “ ight” dose. You should tit ate
the type o opiate used and the route o administration. mo phi e to the desi ed e ect while watchi g o side
D) ransdermal opiate delivery products are expensive, have a e ects.
slow onset o action, and have erratic absorption.
E) T ere is no pre-established ceiling dosage or opiates, and you
may increase the opiate dose until adverse side e ects occur. You increase the morphine dose considerably over a 2-week
period and your patient begins having escalating pain and
Answer 27.2.4 The correct answer (and the alse statement) muscle twitching.
is “B.” Concentrated oral morphine solutions (20 mg/mL) can be
given in small amounts to patients who are unable to swallow. Question 27.2.5 Which statement is true about opioid-
While it was previously thought that morphine elixir worked induced hyperalgesia (OIH)?
by absorption through the buccal mucosa, it is now believed A) Patients with OIH always become delirious.
to trickle down the throat to be absorbed through the gastric B) When a patient has OIH, the pain does not improve when
mucosa. Fentanyl is the only opiate absorbed via the buccal the opioid dose is increased.
mucosa (thus, the entanyl lollipops). A secret that drug compa- C) T e pain in OIH is described as a worsening o the original
nies do not want you to know is that you can avoid the expense pain being treated.
o entanyl lollipops by using entanyl IV solution orally or con- D) OIH needs to be di erentiated rom worsening underlying
trol o breakthrough pain. Since concentrated morphine elixir disease.
is e ective in patients who cannot swallow, eeding tubes or
parenteral routes o administration are unnecessary. “A” is true. Answer 27.2.5 The correct answer is “D.” Be ore we presume
Delirium is a common and disturbing nding toward the end that a patient has OIH, we need to ensure that the increase in
o li e, and it is sometimes precipitated or exaggerated by opi- pain is not due to urther disease progression. We also need
ates. On the ip side, untreated pain can cause delirium, and the to consider increased pain resulting rom increased activ-
delirium may improve when opioid doses are escalated. In these ity or other exacerbation (such as trips to x-ray or radiation
704 AMILY MEDICIn E EXAMIn ATIOn & BOAr D r EVIEW

therapy). Additional eatures that can help distinguish OIH


you would t eat a loop diu etic—i 60 mg o IV u o
rom increased pain include: the development o muscle twitch-
semide does ot wo k, you would ’t t y 61 mg (o at
ing, presence o allodynia (pain elicited rom ordinarily non-
least we hope ot!), but athe go to 90 o 120 mg; (3)
pain ul stimuli, such as stroking skin with cotton—although
o a opiate aive patie t who is sta ti g a patie t
this can occur with neuropathic pain syndromes as well), and
co t olled a algesia (PCA) pump, go with mo phi e
development o seizures or delirium. However, not all patients
2 mg IV Q10 mi utes as eeded without a co ti uous
with OIH become delirious, making “A” incorrect. T e key ea-
(“basal”) i usio .
ture o OIH is that the pain increases as the dose o the opiate is
increased and will decrease when the dose is decreased. We are
not certain why OIH occurs but several mechanisms have been
proposed. One proposed mechanism is upregulation o NMDA HELPFUL TIP:
receptors rom chronic use o high-dose opiates, which is why Si ce methado e i te acts at the n MDA ecepto s, it is
switching to an opiate with NMDA antagonism could prove the opiate o choice o eu opathic pai , which is o te
help ul (such as methadone or ketamine). However, OIH can poo ly espo sive to othe opiates. I additio , meth
still occur in patients new to opiates and on low doses, so there ado e is the o ly long acting opiate that comes i a
is still more research needed on this subject. “C” is incorrect as liquid o m a d ca be give buccally o i a e te al
well. OIH typically produces di use pain, less de ned in quality tube. Methado e ca be dosed eve y 8 to 12 hou s with
and extending beyond the pre-existing pain distribution. ma y pai patie ts equi i g Q8 hou dosi g. r emem
be that methado e p olo gs the QT i high doses a d
You are concerned about OIH and you decide to rotate to ca cause to sades de poi tes.
methadone.

Question 27.2.6 Which o the ollowing statements is FALSE Your nurse calls you about your patient and reports that her
regarding the use o methadone? pain level has been rising, and despite signi cant increases
A) Methadone can be legally prescribed or pain and addiction in both basal and breakthrough opiate therapy, the patient’s
by a physician with a current schedule II DEA license. pain is uncontrolled. T e nurse asks you i there is something
B) Methadone is more easily absorbed by those with bowel else that can be added to the opiates as an adjunctive treat-
problems than is sustained-release morphine. ment or pain. For no explicable reason, you cannot get gaba-
C) T e hal -li e o methadone is 22 hours. pentin out o your head—even though you know it’s probably
D) Methadone may be use ul or neuropathic pain because it useless or this patient.
inhibits receptors in the dorsal horn o the spine.
E) Methadone is primarily excreted in the stool and thus drug Question 27.2.7 Which o the ollowing is TRUE regarding
dosages do not need to be modi ed in those with mild to the use o the gabapentin?
moderate renal disease. A) Gabapentin is hepatically cleared and dose adjustment is
needed in patients with liver disease.
Answer 27.2.6 The correct answer (and alse statement) is B) T e primary utility o gabapentin is in neuropathic pain.
“A.” Methadone can be prescribed or pain control by physi- C) Gabapentin primarily works via the µ-opiate receptors.
cians with a schedule II DEA license but cannot be prescribed D) T e most common side e ects o gabapentin are supraven-
or addiction or opiate withdrawal without a special license. “B” tricular arrhythmias (thus limiting its utility in heart ailure
and “C” are true. Methadone has a long hal -li e, and sustained- patients).
release preparations are not needed. Sustained-release morphine E) I a patient does not respond to gabapentin, she will also not
may pass unabsorbed in patients with short gut or dys unctional respond to pregabalin.
gut, whereas methadone would be absorbed. “D” is true; metha-
done may be especially use ul in treating neuropathic pain. “E” Answer 27.2.7 The correct answer is “B.” Gabapentin has
is also true. Methadone is primarily excreted into the GI tract. proven to be a marginally e ective (NN = 7) therapy or a
Patients with liver disease should have doses adjusted. However, number o diseases that mani est with neuropathic pain, includ-
those with renal disease may tolerate “normal” doses since renal ing postherpetic neuralgia, diabetic neuropathy, and cancer-
excretion is a minor part o methadone elimination. related neuropathic pain. “A” is incorrect because gabapentin is
excreted unchanged through the kidneys—a plus or patients
HELPFUL TIP: with working kidneys on multiple drugs. Why do we like drugs
Whe it comes to switchi g o dosi g opiates the e a e that are not hepatically cleared? T ey do not interact in any o
seve al use ul tips that we like to use: (1) do ’t o get the cytochrome P450 pathways and there ore have ew drug–
to accou t o i complete c oss tole a ce whe switch drug interactions. “C” is incorrect. T e mechanism o gaba-
i g classes o opiates— educe the dose o the ew opi pentin is complex and not ully understood, but we are airly
ate by 30% to 50%; (2) whe opiates a e ot helpi g, certain that it does not work through the µ-opiate receptors.
i c ease the dose by 50%; esse tially, t eat opiates like T e literature suggests there is an intricate interplay with mul-
tiple receptors, including voltage-dependent calcium channels
CHAPTER 27 • En D-O -LI E CAr E 705

and increasing synthesis o GABA (more than you wanted to • Hydromorphone (Dilaudid) 20 mg PO Q 4 hours
know, right?). “D” is incorrect. T e most common side e ect o • Acetaminophen 1,000 mg PO ID
gabapentin is its e ect on cognition, causing dizziness and som- • Sorbitol 30 mL PO ID
nolence. Gabapentin can also cause edema, which can be prob- • Metoclopramide 20 mg PO ID
lematic or patients with heart ailure. “E” is incorrect. Although
gabapentin and pregabalin are thought to have the same mecha-
nism o action, it appears that patients that some patients will You want to optimize her medications.
respond to one but not the other. Another important distinction
between gabapentin and pregabalin is the time needed to titrate Question 27.2.9 Which o the ollowing is the most appro-
to an e ective dose. With gabapentin we typically start at 100 priate medication adjustment to make at this time?
to 300 mg and escalate the dose over the course o weeks up A) Hydromorphone → scheduled controlled release mor-
to 3,600 mg divided three times a day; alternatively, pregabalin phine.
starts at 75 mg daily and reaches a max dose o 300 mg BID B) Hydromorphone → scheduled immediate release mor-
within a week o initiation. phine.
C) Hydromorphone → scheduled controlled release morphine
and immediate release morphine as needed.
HELPFUL TIP: D) Acetaminophen → immediate release morphine.
A opiate aive patie t should eve be sta ted o a E) Acetaminophen → nortriptyline.
e ta yl patch. A patie t eeds to be taki g a o al mo
phi e equivale t o at least 25 mg i 24 hou s be o e the Answer 27.2.9 The correct answer is “C.” A patient who has
lowest dose (12.5 mcg) e ta yl patch ca be applied. reached a stable dose o short-acting opiate, such as hydromor-
phone, should subsequently be switched to a long-acting opi-
oid agent. An immediate release medication should be available
As time goes on, the patient has other concerns, including or acute “breakthrough” pain. T ere is no reason to change
constipation, weight loss o 20 pounds over 2 months, sleep- the acetaminophen. Nortriptyline is sometimes use ul as an
lessness, nausea, and anxiety. In addition, she expresses how adjuvant medication and is particularly help ul when treating
her loss o unctional abilities is a hardship or her and her neuropathic pain.
adult daughter who serves as her primary caregiver. Her guilt
or losing her health is a continual source o rustration and T e hospice nurse calls you. Your patient is at home and has
anger. become restless with slow respirations (6/min) along with
paroxysmal coughing and gagging with a large amount o
Question 27.2.8 What is true regarding her social and emo- secretions.
tional pain?
A) It will not a ect the patient’s analgesic requirements. Question 27.2.10 The ollowing are all appropriate orders
B) It will likely complicate treatment adherence. or this patient EXCEPT:
C) Active treatment o emotional sources o pain should only A) Obtain subcutaneous access through placement o a subcu-
occur a er the physical source has been addressed and taneous button or butter y needle.
treated. B) Lorazepam 1 to 2 mg PO or SL every hour PRN.
D) Prophylactic antidepressants in patients within 6 months o C) Glycopyrrolate 2 mg PO or SL every 4 hours PRN.
death decrease the probability o developing depression. D) Midazolam 0.4 to 4 mg SC every hour PRN.
E) Naloxone 2 mg SC every 2 hours PRN.
Answer 27.2.8 The correct answer is “B.” Adherence, always
an issue, is especially compromised in those dying patients Answer 27.2.10 The correct answer is “E.” Naloxone is a potent
whose social, spiritual, and emotional problems are not e ec- opioid receptor antagonist. Although the sudden change in the
tively addressed. Similarly, analgesic control o somatic pain is patient’s status could be partly due to opiate accumulation, the
complicated when social, emotional, and spiritual sources o risks o naloxone antagonism are great and include severe pain,
pain exacerbate the patient’s response and perception to somatic cardiac arrhythmias, and seizures (remember, you don’t want that
pain. Concurrent treatment o all sources o pain is necessary. ghost haunting you). Withholding or reducing the next dose o
Antidepressant therapy in dying patients who do not have clini- opiates is a sa er approach. A subcutaneous in usion pump (“A”)
cal depression o ers no prophylaxis against the development o may allow e ective administration o medications and uids
depression. in patients who cannot tolerate oral administration. T e use o
hydration at the end o li e is controversial. Withholding uids
You estimate your patient’s li e expectancy to be 2 months or and nutrition has strong merit, but the evidence is not compelling
less. Her railty has progressed to the point where she is bed- enough to declare that uid in usion is utile and possibly harm-
bound and utterly dependent or all o her ADLs. You have ul in this setting. In addition, dehydration is a common cause o
made some adjustments, and she is now on stable doses o the delirium at the end o li e, and her con usion may improve with
ollowing medications: gentle hydration. It is more important to review the patient’s goals
706 AMILY MEDICIn E EXAMIn ATIOn & BOAr D r EVIEW

and only administer uids i consistent with her goals. Glycopyr- Question 27.3.1 What would be the best intervention or
rolate (“C”) has been shown to decrease oral/pulmonary secre- weight gain in this patient?
tions that lead to the “death rattle” in the nal days and hours o A) Megestrol.
li e. Such treatment bene ts the patient and her grieving amily B) Nutritional supplements.
and riends. “B” and “D” may also be use ul. Benzodiazepines C) Dronabinol.
have the potential to reduce anxiety, agitation, and insomnia; D) Mirtazapine.
however, any benzodiazepine can worsen con usion and cause E) Feeding tube.
delirium. Benzodiazepines with an extended hal -li e (e.g., diaz-
epam and chlordiazepoxide) should generally be avoided because Answer 27.3.1 The correct answer is “B.” T e most e ective
o the potential or toxic accumulation. means to maintain weight in elderly patients with dementia
is hand eeding and providing avorite oods (yes, ice cream
Objectives: Did you learn to . . .
with every meal is OK!) and requent o erings. Hand eeding
• De e majo physiologic pai catego ies?
is expensive and o en problematic in nursing homes with lim-
• Desc ibe the pha macology o pai co t ol?
ited sta . When hand eeding is not possible, nutritional sup-
• P esc ibe opiate pai medicatio s a d adjuva t the apies o plements are the least invasive alternative with minimal side
pai ?
e ects and have been shown to be modestly e ective in pro-
• Ide ti y emotio al, social, a d spi itual symptoms a d ecog moting weight gain in elderly patients with dementia. Other
ize how they ca a ect pai ma ageme t?
important considerations related to nutrition in dementia
patients is being aware o e ective ways to decrease aspiration
QUICK QUIZ: THE SUBCUTAn EOUS r OUTE risk, including sitting upright, eating small portions, use o
thickened liquids, and hand eeding. “A,” megestrol (Megace),
Regarding the subcutaneous administration o uids and medi- is minimally e ective in improving appetite and increas-
cations, which o the ollowing is true? ing weight in patients with cancer cachexia and weight loss
A) Not more than 500 cc o saline per day can be given by hypo- related to AIDS, but there is sparse evidence or use in geri-
dermoclysis (subcutaneous administration). atric patients. rials are o short duration (1–2 months) using
B) Only medication with low lipid solubility can be delivered widely varying amounts o megestrol (240–1,600 mg/day), and
via subcutaneous administration. the weight gained is typically uid and not muscle mass. With
C) Most drugs used in end-o -li e care can be given subcutane- such poor evidence or bene t but well-known side e ects,
ously. there is no role or megestrol in this setting. In act, there is no
D) In general, subcutaneous administration dosage conversion role or hormones or steroids in the treatment o weight loss in
is closer to oral dosage than to IV dosage. the elderly patient with dementia. “B,” dronabinol, is synthetic
derivative o cannabis (known to cause “the munchies” . . . we’ve
The correct answer is “C.” T e subcutaneous route can be heard). It increases appetite and improves nausea, but again
quite e ective. Adverse events such as local irritation, pulmo- the studies are small and exploratory in nature and ocused
nary edema, and local edema are less requent with subcutane- mainly on patients with AIDS and cancer. “D” is incorrect. I
ous administration when compared with IV administration. In the patient were depressed and losing weight as a result, mir-
general, most drugs used in end-o -li e care can be given sub- tazapine might be a reasonable choice since it may stimulate
cutaneously. “A” is incorrect. Evidence demonstrates that up to appetite. However, mirtazapine has not been shown to boost
3,000 cc o crystalloid solution can be given subcutaneously in appetite in nondepressed subjects (and like the SSRIs, does not
a 24-hour period with limited adverse e ects. Experience sug- improve depression in the demented elderly anyway; Lancet.
gests that even greater volumes can be given. “B” is incorrect. 2011;378:403–411). Some antidepressants, such as SSRIs and
Lipid solubility is not a clinically relevant aspect o bioavailabil- bupropion, are associated with weight loss. Finally, a eeding
ity during subcutaneous administration. “D” is also not true. tube is invasive with numerous side e ects (cellulitis, leakage,
Subcutaneous doses are generally very close to, but lower than, and increased ecal incontinence which can contribute to bed
the IV dose o a drug. As with IV administration, the onset o sores). While enteric tube eeds may improve caloric intake,
action is more rapid than with enteral dosing. they do not extend li e, increase weight, improve unctional
status, or reduce the incidence o pressure sores or aspiration in
CASE 27.3 dementia patients (exceptions to this are ALS and gastrointes-
tinal cancers with proximal obstruction with a good unctional
You assume care or the 84-year-old ather o one o your status). T us, “E” is incorrect.
patients. He has severe dementia, which has caused him to
require nursing home care or the last 5 years. He has stable
heart ailure due to ischemic cardiomyopathy. He requires HELPFUL TIP:
ull assistance to eat, which the sta members spend an hour r emi d a d ei o ce to amilies that deme tia is a te
doing at each meal. He has lost 20 lb in the last 6 months mi al ill ess a d that ou cu e t u de sta di g is that
(BMI now is 21 kg/m2). A acility nurse calls you worried tube eedi g is u likely to alte that ill ess.
about his weight.
CHAPTER 27 • En D-O -LI E CAr E 707

6 months (e.g., aspiration pneumonia, pyelonephritis, sepsis, stage


Your nutrition shakes are not doing the trick. T e patient ≥ 3 decubitus ulcer, or ever despite antibiotics); (3) inability to
continues to lose weight. o urther stage his dementia, you maintain uid/caloric intake (by demonstrating weight loss > 10%
attempt a Folstein Mini Mental Status Exam, but he cannot in 6 months or albumin < 2.5 g/dL). “E” is incorrect. Aspiration
participate. T e nurse reports that he needs assistance with risk and stage 2 pressure ulcers alone are not suf cient or re erral
all ADLs, he is incontinent, and he is unable to speak more to hospice. I the patient had stage 3 or 4 pressure ulcers, he would
than six intelligible words (and although you remember a likely meet NHPCO hospice criteria or severe dementia. It is the
couple o post-call days when you were in the same condition, extrapolation o the patient’s loss o basic unctions over time that
you say nothing). T e nursing sta asks you i the patient is best predicts death, as in “C.” “D” is incorrect because while most
appropriate or hospice care. nursing homes routinely care or dying patients without the assis-
tance o hospice, they are typically not able to o er the ull range o
Question 27.3.2 Which o the ollowing is TRUE? services or the end-o -li e expertise that hospice o ers.
A) T e li e trajectory o terminally demented patients is unclear.
Wait until his cardiac disease worsens. Your patient develops nausea and vomiting and on examination
B) He does not yet meet Medicare hospice bene t criteria or you eel a large abdominal mass. He is still having bowel move-
dementia. ments, which are guaiac positive. You in orm the amily that
C) A history rom the nursing sta or amily regarding the rate you suspect he has colon cancer, and they elect or no urther
o his unctional loss is the best predictor o death. evaluation but want his symptoms aggressively managed.
D) T e nursing home sta can provide all o the end-o -li e ser-
vices provided by hospice, so hospice is not needed. Question 27.3.3 Which o the ollowing drugs is the best
E) Presence o a stage 2 pressure ulcer and aspiration risk is su - initial choice or treatment o this patient’s nausea and
cient or re erral to hospice. vomiting?
A) Octreotide.
Answer 27.3.2 The correct answer is “C.” Predicting death in B) Metoclopramide.
patients with dementing illness is dif cult which is why it is one C) Diphenhydramine.
o most underutilized diagnoses or re erral to hospice care. In D) Ondansetron (Zo ran).
act, dementia has the greatest variability among hospice diagno- E) Aprepitant (Emend).
ses when it comes to predicting survival. However, waiting until
a more predictable organ system ailure occurs is not a reason Answer 27.3.3 The correct answer is “D.” “B” is not correct.
or delaying a hospice re erral, which makes “A” incorrect. “B” is Using a pro-peristaltic agent in a patient who may have an
incorrect because he does meet criteria or hospice enrollment. obstruction is not a good idea in general. Also, metoclopramide
T e National Hospice and Palliative Care Organization (NHPCO) has central anti-dopaminergic properties and can cause con-
hospice guidelines or eligibility in dementia include three com- usion and extrapyramidal side e ects. By understanding the
ponents: (1) demonstrating severity o dementia (e.g., an inability pathophysiology o nausea and targeting antiemetics to spe-
to walk or dress or bathe without assistance, urinary/ ecal incon- ci c receptors, therapy can be optimized and side e ects mini-
tinence, or inability to speak 6 di erent intelligible words per mized. An easy way to remember the causes o vomiting is to
day); (2) evidence o a severe comorbid condition within the past use the “VOMI ” mnemonic. In able 27-1, pathophysiologic

TABLE 27-1 CAUSES AND TREATMENT OF NAUSEA AND EMESIS


Type o Nausea Receptors Causing Nausea Use ul Drug Classes Examples o Drugs o Choice
V Vestibula Choli e gic A ticholi e gic Scopolami e
Histami ic A tihistami ic P omethazi e

O Obst uctio (caused by co stipatio ) Choli e gic Stimulate mye te ic plexus Se a p oducts
Histami ic P u es
5 HT3

M DysMotility Choli e gic P oki etics to stimulate Metoclop amide


Histami ic 5 HT4 ecepto s
5 HT3

I I ectio , I ammatio Choli e gic A ticholi e gic P omethazi e


Histami ic A tihistami ic
5 HT3

T Toxi s (stimulati g the CTZ i the Dopami e 2 A tidopami e gic P ochlo pe azi e
b ai , i.e., opioids) 5 HT3 5 HT3 a tago ist Halope idol
O da set o

5 HT3, 5 hyd oxyt yptami e (se oto i ); CTZ, chemo ecepto t igge zo e.
708 AMILY MEDICIn E EXAMIn ATIOn & BOAr D r EVIEW

mechanisms causing nausea are described using the letters o the act, in patients with advanced dementia, in ections o en char-
mnemonic. Blockade o these receptors allows rational, ocused acterize the nal stage o their illness. “A” is incorrect because
therapy. antibiotic use in patients with dementia (speci cally in patients
Using able 27-1, another reasonable alternative would be with dementia who are hospice appropriate) should always be
promethazine, although promethazine is not a great antiemetic. discussed with the surrogate decision maker, and antibiotic
“A” is incorrect. Octreotide works by slowing the GI track, and use should be based on an in ormed choice and the patient’s
its main use is ound in patients with nausea and vomiting due wishes/goals. While antibiotics have been shown to prolong li e
to malignancy-related intestinal obstruction. However, it does in patients with dementia, they do so with the cost o increas-
not rapidly treat nausea and is not easily administered (usually ing discom ort, making “B” the correct answer. Families need
continuous IV in usion but it can be given subcutaneously or this in ormation when deciding i antibiotics should be used. “C”
intramuscularly). “C” is incorrect because diphenhydramine is incorrect because nursing homes have been shown to harbor
alone is a weak antiemetic, mostly used or motion sickness, and drug-resistant bacteria, likely due in part to the requent use o
it can cause con usion, particularly in the elderly. Aprepitant antibiotics in these patients. Oral antibiotics have been shown
(Emend) is a neurokinin receptor antagonist that is most e ec- to be as e ective as other more invasive administration routes in
tive when used with serotonin receptor antagonists; its primary patients with dementia, making “D” incorrect.
use is in the management o chemotherapy induced nausea, and
it is very expensive. T e patient is enrolled in hospice and 3 weeks later you are
called by a nurse reporting an acute condition change. You
go to see the patient and he is unresponsive, breathing is
HELPFUL TIP:
labored, and his eet are mottled.
Whe t eati g chemothe apy i duced ausea, add
somethi g to thei cu e t egime . Do ot just d op
Question 27.3.5 All o the ollowing can be signs o impend-
o e medicatio a d add a othe . This is cou te i tu
ing death EXCEPT:
itive but is co side ed the app oach o choice.
A) Cheyne–Stokes breathing.
B) Fever.
C) Cyanosis and mottling.
T e patient’s nausea is ultimately controlled with scheduled D) Aldosterone escape phenomenon with increased urinary
low-dose promethazine and ondansetron (See? wo drugs output.
instead o a large dose o one!). He is continuing to lose E) alking to someone who is already dead (and not via me-
weight and has been coughing a er eating. He spikes a ever dium at Coney Island).
and you suspect he has developed aspiration pneumonia.
Answer 27.3.5 The correct answer is “D.” Most dying patients
Question 27.3.4 Which is true regarding the use o antibiot- have a decrease in urinary output prior to death. T e other
ics or pneumonia in patients with terminal dementia? options describe changes that are commonly seen in patients
A) Antibiotic use is considered the standard o care and should who are actively dying. Respiratory changes in the active
be initiated without discussing with the amily. dying phase include Cheyne–Stokes breathing, terminal secre-
B) Antibiotics might prolong patients’ lives but treatment tions (the “death rattle”), and periods o apnea. Dying is o en
comes with signi cant risks that need to be weighed. accompanied by decreased circulation, which can result in cool
C) Antibiotics given in a controlled setting such as a nursing extremities and mottling (the skin turning blue and patchy par-
home, where compliance can be ensured, lead to less antibi- ticularly in the ngers and toes). It is not uncommon or a dying
otic resistance. patient to have a ever in the last 24 to 48 hours o li e, typically
D) Parenteral antibiotics are more success ul at treating aspira- thought to be secondary to aspiration pneumonia or another
tion pneumonia in patients with dementia (compared with serious in ection. It is not uncommon or someone who is dying
oral antibiotics). to talk about going on a journey or talk about seeing someone
who is dead. T is is typically believed to be part o the normal
Answer 27.3.4 The correct answer is “B.” T e previous prevail- dying process and in the absence o other symptoms should not
ing belie o carte blanche use is being challenged. Currently, be con used with delirium or psychosis.
antibiotic use is very common in nursing homes, and a major-
ity o nursing home residents will receive antibiotics in the HELPFUL (OR AT LEAST INTERESTING) TIP:
2 weeks be ore their death. Most o those antibiotics will be given A visit om Osca the (the apy) cat i a r hode Isla d
intravenously, which increases patient discom ort. But more u si g home has p oved to be a ve y accu ate p edic
importantly, patients are o en admitted to a hospital, placing to o immi e t death ( eally—see the a ticle i N Engl J
demented patients at risk or delirium, restraints, and exposure Med. 2007;357(4):328). P esumably, Osca is eacti g to
to hospital acquired organisms. T e reason we emphasize “might the lack o patie t moveme t o the smell o keto es,
prolong” patients’ lives is that the literature shows that in the etc. The agai , Osca could be the A gel o Death i a
extremes o advanced dementia plus advanced illness (such as wol ’s (well, cat’s) clothi g.
bacteremia) antibiotics may not provide improved survival. In
CHAPTER 27 • En D-O -LI E CAr E 709

HELPFUL TIP: TABLE 27-2 A MNEMONIC FOR GIVING BAD NEWS


Deli ium is ve y commo (up to 85%) i the last ew S Setti g. A i app op iate setti g ca make it dif cult to give bad
days o li e. I the symptoms a e dist essi g to the pa ews e ectively. Make su e the physical setti g is as co ducive as
tie t o amily, it is best t eated with a a tipsychotic possible by t yi g to e su e p ivacy, i volvi g sig i ca t othe s,
sitti g dow , co ecti g with the patie t (eye co tact, ha d holdi g),
such as halope idol.
e su i g e ough time, a d mi imizi g i te uptio s.

P Patie t’s pe ceptio . Ask what the patie t’s (o amily’s) k owledge
a d u de sta di g o the cu e t medical ill ess is.
You need to call the son to tell him o his ather’s decline.
I I vitatio . Ask what i o matio the patie t wa ts to eceive. Some
patie ts do ot wa t to hea bad ews themselves, a d i this case it
Question 27.3.6 Which o the ollowing should you say ca be help ul to ask i the e is someo e else with whom you should
when giving the son an update? speak.
A) “T ere have been some changes in your ather’s condition.
K K owledge. Give the medical acts i a st aight o wa d ma e
I think you should go to see him. Call me i you have any usi g vocabula y a d la guage app op iate to the patie t’s level.
questions a er you have seen your dad.” Avoid medical ja go a d do ot be excessively blu t (i.e., avoid
B) “Good news, your dad will soon be out o his misery and sayi g thi gs such as “you have ve y bad ca ce a d u less you do
released rom this mortal world.” somethi g, you will die”). Give the i o matio i small chu ks a d
C) “I’m a raid I have some bad news or you regarding your eque tly assess what the patie t has u de stood. Whe possible
sta t o with a wa i g shot (i.e., “U o tu ately, I have some bad
dad. Would you like to talk over the phone or meet at my ews o you”).
of ce later today to discuss it?”
D) “Your dad is dying. I you want to see him alive again you E Explo i g/empathy/emotio . A patie t’s emotio al eactio ca
had better go today.” va y a d is o te ha d o a physicia to expe ie ce. A empathetic
espo se ca be help ul. This ca be oste ed by allowi g sile ce a te
E) Don’t call at all. Ask the nurse to in orm the son o his b eaki g the bad ews a d watchi g a d liste i g o the emotio .
ather’s condition while you duck out quietly. Whe you have ide ti ed the emotio , it ca be help ul to ame it
a d dete mi e what caused it. The make a empathetic stateme t
Answer 27.3.6 The correct answer is “C.” It gives a warning such as “I’m so y. I k ow this is ’t what you wa ted to hea .”
shot and allows the person receiving the bad news some con- S St ategy/summa y. A te the emotio s have bee add essed, it is
trol by allowing him to determine how and where he wants to help ul to eview what has bee said a d ag ee o a pla . Co side
receive the bad news. “A” avoids giving the bad news at all. “B” aski g the patie t i he wa ts to discuss t eatme t optio s at this
is inappropriately ippant and uses euphemisms. “D” is inap- time o wait u til a utu e meeti g. r eceivi g bad ews ca be
propriately blunt and sounds aw ul (believe it or not, we are very ove whelmi g, a d patie ts o te o get the details o what is said. It
is impo ta t to have clea , well de ed, timely ollow up such as “Go
caring doctors; it is just that we have amused ourselves with this home a d talk with you amily, I will see you (a d you amily) back
book). “E” avoids having the discussion at all and is only appro- tomo ow at 9 AM, a d we ca discuss speci c t eatme t optio s a d
priate i you will be unable to reach the son in a timely ash- a swe questio s at that time.”
ion. Besides, you will be orever a er known to the nurses as
“Dr. Wuss.” A common ormat or giving bad news is to use the Data om Baile W , Buckma r , Le zi r , et al. SPIKES A six step p otocol o
delive i g bad ews: applicatio to the patie t with ca ce . The Oncologist.
SPIKES six-step protocol in able 27-2.
2000;5:302–311.

HELPFUL TIP: A) ake time to prepare what you are going to say and nd a
Whe givi g bad ews, do ot use ph ases such as, “I’m quiet place to make the call.
a aid the e is othi g mo e we ca do o you.” This B) I no one answers the phone, it is acceptable to leave a mes-
leaves patie ts a d amily eeli g aba do ed. It is bet sage or voicemail detailing the bad news.
te to be mo e speci ic to say, “I am a aid that I do ’t C) Identi y yoursel and avoid answering any direct questions
have a y t eatme ts that will cu e you ca ce , but until you are sure o the identity o the person to whom you
the e is still a lot I ca o e to help to keep you com o t are talking.
able i the time you have le t.” This assu es the patie t D) Ask i the person is alone.
a d the amily that the e is still somethi g to be do e E) Speak clearly and slowly, allow time or questions; be empa-
a d that you will ot aba do them. thetic.

Answer 27.3.7 The correct answer is “B.” When breaking bad


news over the phone, steps similar to breaking bad news in per-
Breaking bad news is dif cult in person and can be more di - son should be used. You should obtain the ull name, address,
cult over the phone. and phone number(s) o the person(s) you are calling. I you are
not calling the patient, try to establish rom the chart and nurs-
Question 27.3.7 Which suggestion below is NOT recom- ing sta the relationship o the contact to the patient. In addition,
mended when breaking bad news over the phone? it can be help ul to write down the key in ormation you need
710 AMILY MEDICIn E EXAMIn ATIOn & BOAr D r EVIEW

and review what you will say and nd a quiet area with a phone.
Sta t a bowel egime whe eve sta ti g opioid a algesics.
Don’t delay in making the call. When you do call, clearly identi y
yoursel and ensure you are able to speak with the person closest The additio o a be zodiazepi e to a opiate educes
a xiety a d b eathless ess, but espi ato y ate a d me tal
to the patient (ideally, the healthcare proxy or the contact person
status must be ca e ully mo ito ed.
indicated in the chart). Avoid responding to any direct question
until you have veri ed the identity o the person to whom you are Tit ate opioid a algesics to a e ect athe tha a pa ticula
speaking. Not only is it morally risky but it also violates HIPPA. dose o blood level.
Ask i the contact person is alone. Do not give death noti ca- Use equia algesic e e e ce cha ts a d accou t o i complete
tion to minor children. I you do not have a prior relationship c oss tole a ce whe switchi g om o e opioid to a othe .
with the person you are speaking to, ask what they know about
the patient’s condition. Provide a warning shot. Never deliver the
news o death to an answering machine or voice mail. Instead,
leave speci c contact in ormation. Allow time or questions; be BIBLIOGRAPHY
empathetic and ask i you can contact anyone or them. Assess Adler ED, et al. Palliative care in the treatment o advanced
their emotional reaction and ollow up as indicated. heart ailure. Circulation. 2009;120:2597–2606.
Baile WF, et al. SPIKES-A six-step protocol or delivering bad
news: Application to the patient with cancer. Oncologist.
HELPFUL TIP: 2000;5:302–311.
Whe you eed to i o m a amily that a patie t has died,
Banerjee S, et al. Sertraline or mirtazapine or depression
wo ds like “dead” o “died” should be used; avoid euphe in dementia (H A-SADD): a randomized, multicentre,
misms such as “expi ed,” “passed away,” o “did ’t make double-blind, placebo-controlled trial. Lancet. 2011;378:
it” (also, “kicked the bucket,” “bit the bullet,” “bought the 403–411.
a m,” a d so o ), which ca be misi te p eted. Bennett MI. Gabapentin in the treatment o neuropathic pain.
Palliative Med. 2004;18:5–11.
Bharadwaj P, Ward K . Ethical considerations o patients with
Objectives: Did you learn to . . . pacemakers. Am Fam Physician. 2008;78(3):398–399.
• Ge e ate a ma ageme t pla o a patie t with weight loss Bushnaq M, et al. T e easibility o using intravenous entanyl
due to a te mi al disease? as sublingual drops in the treatment o incidental pain in
• Desc ibe c ite ia o hospice admissio i patie ts with patients with cancer. J Palliat Med. 2009;12(6):511–514.
seve e deme tia? Challapalli V, et al. Systemic administration o local anesthetic
• Ide ti y a d t eat causes o ausea a d vomiti g at the e d agents to relieve neuropathic pain. Cochrane Database Syst
o li e? Rev. 2005;(4): Art No. CD003345.
• Ide ti y sig s o the active dyi g p ocess? Clemens KE, Klaschik E. Dyspnoea associated with anxiety–
• r ecog ize the p ope steps o b eaki g bad ews? symptomatic therapy with opioids in combination with
lorazepam and its e ect on ventilation in palliative care
patients. Support Care Cancer. 2011;19:2027–2033.
HELPFUL TIP: Coyne PJ, et al. Nebulized entanyl citrate improves patients’
Bewa e o Osca the cat. perception o breathing, respiratory rate, and oxygen satu-
ration in dyspnea. J Pain Symptom Manage. 2002;23(2):
157–160.
Donner B, et al. Direct conversion rom oral morphine to
transdermal entanyl: a multicenter study in patients with
Clinical Pearls cancer pain. Pain. 1996;(643):527–534.
Avoid mepe idi e. Othe opioid a algesics a e p e e ed. Dosa DM. A day in the li e o Oscar the cat. N Engl J Med.
2007;357(4):328–329.
Do ot delay palliative ca e assessme t i a patie t with a
Fox CB, et al. Megestrol acetate and mirtazapine or the
se ious, li e limiti g ill ess, eve i that patie t is pla i g
treatment o unplanned weight loss in the elderly.
to seek cu ative the apy. Patie ts with a ea ly palliative
Pharmacotherapy. 2009;29:383–397.
app oach live lo ge a d have a highe quality o li e.
Givens JL, et al. Survival and com ort a er treatment o
Do ot leave a impla ted ca diac de ib illato (ICD) active pneumonia in advanced dementia. Arch Intern Med.
without assessi g the patie t a d amily’s goals o ca e. I 2010;170(13):1102–1107.
a patie t with a te mi al ill ess who is Dn r , it is logically
Johnson MJ, Oxberry SG. T e management o dyspnoea in
co siste t to deactivate the ICD, a d this optio should be
chronic heart ailure. Curr Opin Support Palliat Care.
discussed with the patie t/ amily.
2010;4:63–68.
Do ot sta t a opioid aïve patie t o a e ta yl patch. Lampert R, et al. HRS expert consensus statement on the man-
Opioid a algesics, mo phi e i pa ticula , elieve espi ato y agement o Cardiovascular Implantable Electronic Devices
dist ess i dyi g patie ts. (CIEDs) in patients nearing end o li e or requesting with-
drawal o therapy. Heart Rhythm. 2010;7(7):1008–1026.
CHAPTER 27 • En D-O -LI E CAr E 711

Mitchell S, et al. In ection management and multidrug-resis- Silverman SM. Opioid induced hyperalgesia: Clinical im-
tant organisms in nursing home residents with advanced plications or the pain practitioner. Pain Physician.
dementia. JAMA Intern Med. 2014;174(10):1660–1667. 2009;12:679–684.
Murtagh FEM, et al. T e use o opioid analgesia in end-stage emel JS, et al. Early palliative care or patients with
renal disease patients managed without dialysis: recom- metastatic non-small-cell lung cancer. N Engl J Med.
mendations or practice. J Pain Palliat Care Pharmacother. 2010;363(8):733–742.
2007;21(2):5–16. ucker R, Nichols A. Managing nonpain symptoms. UNIPAC4:
Pattison M, Romer AL. Improving care through the end o li e: A Resource for Hospice and Palliative Care Professionals.
launching a primary care clinic-based program. J Palliat 2012:17.
Med. 2001;4(2):249–254. van der Steen J , et al. reatment strategy and risk o unc-
Reis eld S, et al. T e e ect o empiric antibiotic therapy on tional decline and mortality a er nursing-home acquired
mortality in debilitated patients with dementia. Eur J Clin lower respiratory tract in ection: two prospective stud-
Microbiol Infect Dis. 2011;30:813–818. ies in residents with dementia. Int J Geriatr Psychiatry.
Sampson EL, et al. Enteral tube eeding or older people with 2007;22:1013–1019.
advanced dementia. Cochrane Database Syst Rev. 2009;2: Walsh G. Hypodermoclysis: An alternate method or rehydra-
Art No. CD007209. tion in long-term care. J Infus Nurs. 2005;28(2):123–129.
Shega J, Levine S. Caring or patients with chronic illnesses: Yeh SS, et al. Pharmacological treatment o geriatric cachexia:
dementia, COPD, and CHF. UNIPAC 9: A Resource for evidence and sa ety in perspective. J Am Med Dir Assoc.
Hospice and Palliative Care Professionals. 2012:13–14. 2007;8:363–377.
Evidence-Based Medicine
Mark A. Graber
28
Yeah, we don’t like numbers either. But they are at the end o the be too late to intervene. Screening, however, can also give the
chapter or those o you who want to learn 2 × 2 tables, etc. We appearance o longer survival even though no additional li e
do like evidence-based medicine (EBM) though, and it will be has been gained. T is is called lead-time bias. Here’s another
on the examination. So here goes…. example. Mr. X has the test, is diagnosed with disease, receives
able 28-1 is here or re erence. You may want to re er to it as treatment, and dies 5 years later. Mr. Y is in the control group,
you work your way through the chapter. develops symptoms at year 4 o the study and dies one year
a er that. T ey have both lived or 5 years a er being random-
CASE 28.1 ized to screening or no screening. Mr. X and Mr. Y both die at
age 65 o the same disease. Did Mr. X have more survival time or
Research published in World’s Best Medical Journal studied
just more “disease time?” T is lead-time bias may be avoided by
screening or lung cancer using a new method. T e research-
using age-speci c mortality rates rather than survival time rom
ers reported that patients who had lung cancer detected via
diagnosis. “C,” veri cation bias, occurs when you are looking
screening lived longer a er diagnosis than people who were
at a new diagnostic modality, and patients with a negative test
diagnosed with lung cancer but not screened.
result ( or the new test) are not evaluated with the gold standard
test. For example, veri cation bias could occur in a study where
Question 28.1.1 Which is true?
people with a negative cardiac stress test do not proceed to a car-
A) T is shows that screening is e ective at prolonging survival.
diac catheterization. T is underestimates the prevalence o dis-
B) T is may be an example o lead-time bias.
ease in the population studied (we don’t really know about those
C) T is may be an example o veri cation bias.
who didn’t have a catheterization) and overestimates the value o
D) Well-respected medical journals (and board review books)
the stress test (seemingly, all patients with cardiac disease were
are always right.
picked up by the stress test… but only because we didn’t look ar
enough). See able 28-2 or more types o bias ound in studies.
Answer 28.1.1 The correct answer is “B.” T is may be an exam-
ple o lead-time bias. Screening is intended to diagnose disease
As part o a quality control study, the hemoglobin A1c val-
earlier, hope ully allowing or interventions that prevent or slow
ues o patients with diabetes at two clinics are compared. In
the progression o the disease. Without screening, the disease
a study o 4,000 patients, it is ound that the mean hemoglo-
may be discovered only a er symptoms develop when it may
bin A1c value in group 1 is 7.4%and the mean hemoglobin A1c
value in group 2 is 7.6%. T e authors did the correct statisti-
TABLE 28-1 USEFUL EQUATIONS cal test and ound a p-value o 0.04 or this comparison.
Sens t v ty: True Pos t ves/(true pos t ves + alse negat ves) Question 28.1.2 Based on this in ormation, you conclude:
Spe ty: True Negat ves/(true negat ves + alse pos t ves) A) Group 1 is signi cantly di erent rom group 2. Reject the
null hypothesis.
False Pos t ve Rate: 1 − spe ty B) Group 1 is not signi cantly di erent rom group 2. Don’t
False Negat ve Rate: 1 − sens t v ty reject the null hypothesis.
C) Group 1 is not signi cantly di erent rom group 2. Reject
Pos t ve Pre t ve Value: True Pos t ve/(true pos t ve + alse pos t ve) the null hypothesis.
Negat ve Pre t ve Value: True Negat ve/(true negat ve + alse negat ve) D) Group 1 is signi cantly di erent rom group 2. Don’t reject
the null hypothesis.
712
CHAPTER 28 • EVid ENc E-BASEd MEd ic iNE 713

TABLE 28-2 SOME COMMON TYPES OF BIASES/PROBLEMS SEEN IN STUDIES


Type o Bias E ect
c on rmat on B as Th s o urs when you only look or ata that supports your ontent on an gnore any n ormat on to the ontrary ( .e.,
i bel eve that the internet s ull o so opaths, i oul on rm that by rea ng omments un er v eos poste on relate
webs tes an gnor ng everyth ng else that woul suggest otherw se).

c on oun ers Th s o urs when two or more a tors are asso ate w th the out ome an only the one be ng stu e s a ounte or ( .e.,
(c on oun ng resear h shows that 9 out o 10 c ubs ans have annoy ng behav or, but 9 out o 10 c ubs ans are also runk when the annoy ng
var ables) behav or o urs; we o not a ount or the on oun ng var able ( runkenness), we may n orre tly assume that be ng a c ubs
an causes annoy ng behav or . . . wh h an’t poss bly be true . . . an t?).

Length t me b as Th s o urs be ause s reen ng tests are more l kely to n slow grow ng tumors rather than those that are rap ly grow ng. Th s
(not lea t me) an b as results n avor o s reen ng be ause more slow grow ng an ers w th a goo prognos s w ll be oun w th a s reen ng
test.

Sele t on b as Th s o urs when subje ts sele te or the stu y o not represent the ent re populat on you m ght see l n ally. For example,
they may be s ker or less s k than the pat ents n your pra t e or they may be ex lu e rom the stu y or another reason. For
example, let’s say we are o ng a stu y on renal a lure. i all pat ents w th abetes are ex lu e , we w ll not be able to apply the
results to our pat ents many o whom have renal a lure an oex st ng abetes. Th s s avo e by hav ng large, representat ve
samples w th ew ex lus on r ter a.

Spe trum b as Th s o urs a stu y s skewe towar a part ular group o pat ents, or example, those w th an NSTEMi. i you stu y NSTEMi
pat ents, you annot apply your results to pat ents w th other relate on t ons, su h as a STEMi or unstable ang na. Spe trum
b as s ommon n stu es; o ten the pat ents stu e at a tert ary are nst tut on f er n the sever ty o llness than those n
pr mary are pra t es (who n general ten to have less severe sease).

Per orman e b as Someth ng bes es what you bel eve s g v ng you your pos t ve (or negat ve) results s a tually a t ve. An example o th s s
estrogen or heart sease. c ase– ontrol stu es suggeste estrogen was ar o prote t ve, but t n’t pan out n ran om ze ,
ontrolle tr als. Thus, the pos t ve ef e t was ue to someth ng else; maybe the women on estrogen exer se more, maybe
they ha better ets, maybe they smoke less an were more ntereste n the r health overall. Ran om ze , bl n e tr als
prote t aga nst th s b as.

Answer 28.1.2 The correct answer is “A.” o answer this ques-


HELPFUL TIP:
tion, you have to know what the usual cuto or signi cance is
A type II error o urs when a stu y a ls to show a
or a p-value, and you also have to know what a null hypothesis
eren e where one ex sts. Th s may o ur be ause there
is. A null hypothesis is the hypothesis that there is no signi cant
are not enough subje ts n a stu y or when there s
di erence between two groups being compared to one another.
measurement error. For example, n a (real) stu y o
By setting up null hypotheses in this way, we can then search
lorazepam versus azepam or se zures, tw e as many
or proo that the null hypothesis is incorrect. ests o signi -
pat ents ha the r se zures stop w th lorazepam. How
cance are a method o looking or evidence that a null hypoth-
ever, the on lus on o the stu y was that there was no
esis is incorrect. T e p-value gives you the probability that the
eren e between the two rugs. Th s s only be ause
results o the study occurred by chance alone. A p-value o 0.04
there were not enough subje ts or th s to rea h stat s
means that i the study results were untrue, we would expect to
t al s gn an e. in lu ng another 100 subje ts may
see these results only 4% o the time by chance alone and not
have ma e th s rea h stat st al s gn an e. Remember
related to the study intervention. By convention, a p-value o
th s by “Type II error s too ew pat ents.”
0.05 or smaller is considered statistically signi cant. T us, when
you have a p-value o less than 0.05, you have evidence that the
null hypothesis is alse and can there ore be rejected.
Objectives: Did you learn to . . .
• Re ogn ze orms o b as n resear h stu es?
HELPFUL TIP: • d e ne p value an null hypothes s?
A type I error o urs when a eren e s oun when • d es r be the s gn an e o p value an type i error?
none s present. The p value prov es you w th the prob
• Re ogn ze a type ii error?
ab l ty o a type i error. For example, a p value o 0.05 s
ons ere stat st ally erent. What th s means, how
ever, s that 5% o the t me, the same on lus on woul CASE 28.2
be pro u e by han e alone. By ontrast, a p value o
0.005 means that there s only a 0.5% han e that the Being the compulsive physician that you are, you are spend-
on lus on s m staken an o urre by han e. The ing your Saturday morning relaxing by reading your journals
lower the p value, the lower the han e o a type i error. (we know who you are; you can’t ool us . . . or else why would
you be working through the EBM chapter?). You notice a
714 FAMiLY MEd ic iNE EXAMiNATiON & BOARd REViEW

study o particular interest on type 2 diabetes, using a novel Question 28.2.2 From this we can conclude:
drug called Shugabegone (not a real drug), and note that the A) T e patients in this study are so nely selected that the
data were analyzed in two ways. T e rst method used was results cannot be applied to our general clinic population.
an “intention to treat” analysis. T e second method was by a B) T e results should be generalizable to our general clinic pop-
“per-protocol” analysis. Hmm, you think. T e per-protocol ulation given the act that the medication worked so well in
data sure makes you want to start prescribing Shugabegone this study.
or your patients with diabetes. C) T is is an example o “selection bias.”
D) A and C.
Question 28.2.1 Which o the ollowing applies to a per E) B and C.
protocol analysis?
A) It provides an objective description o how a new therapy Answer 28.2.2 The correct answer is “D.” We cannot apply
will work in our patient population. these results to our general clinic patients. T ink about it: how
B) It is more statistically stringent when compared to an many o your diabetic patients have no renal disease, no history
intention-to-treat analysis. o CHF and no history o CAD/PVD? Not too many. Many o
C) It allows the study authors to manipulate the outcome to our patients with diabetes have at least some renal disease (pro-
make it look better than it is. teinuria). You always need to look at the inclusion and exclusion
D) You are wrong, Dr. Graber. I am not reading this chapter. criteria o a trial be ore you can determine i the trial is appli-
cable to your patients. T is is called “selection bias”; only select
patients are entered into the study.
Answer 28.2.1 The correct answer is “C.” Per-protocol anal- T ere is also a phenomenon called “spectrum bias.” In the
ysis allows the authors to make the outcome look better than it case o spectrum bias, the patients in the study are di erent
is. o understand why, we must know the di erence between than our patients; they may be sicker or not as sick. Case in
an “intention to treat” analysis and a “per-protocol analysis.” point: glycoprotein IIB/IIIA inhibitors or acute coronary syn-
A per-protocol analysis allows the authors to manipulate the drome (ACS). T e initial studies looked at patients going or
results, usually in avor o the study drug. Here is an example: cardiac catheterization; it appeared there was a small bene t
“O all o the patients who were enrolled in this study, those here. But the drug companies then generalized rom these sick
who took the drug at least 75% o the time were included patients to say that all patients with ACS should have glycopro-
in the inal analysis.” here is a problem here. Many o our tein IIB/IIIA inhibitors. You can’t do this. Most o our “ACS”
patients will not take all o their medicine. Maybe they orgot admissions turn out not to have ACS, and they will get bet-
to take it. Maybe it had side e ects that were intolerable. I ter regardless o what we do. T ey certainly don’t need glyco-
we only analyze the patients who took all o their medication, protein IIB/IIIA inhibitors. And, thank ully, these drugs have
the drug will look better in the per-protocol analysis than it allen out o avor.
will in our clinical practice where adherence to a regimen is
less than complete. You will see this in many papers. In some
papers, there is a “wash-in period” where everyone is given HELPFUL TIP:
the medication and only those who tolerate it or 2 weeks Subgroup analyses (you know, the “our rug worke n
( or example) continue to the main trial. he results here will women over 60” p t h) an only be use to generate a
not be representative o our patients, many o whom may not hypothes s. Th s s alle the “derivation set.” Be ore
tolerate a medication. In summary, per-protocol analysis = a ept ng t nto pra t e, a se on stu y o that sub
bad. group, alle the “validation set” must be one. Th s s
An intention to treat analysis analyzes all study patients in always true. d on’t let them tell you otherw se.
the group to which they were originally assigned. In this case
(to continue the hypothetical study above), the authors do
not care i the patients took the drug 75% o the time. Even i Objectives: Did you learn to . . .
they never took the drug but were assigned to the treatment • d es r be an ntent on to treat analys s an ts value or apply
group, they are analyzed in the treatment group. Many o these ng stu y results?
patients likely would be treatment ailures making the overall • Re ogn ze the ef e t o n lus on an ex lus on r ter a on
results look worse. However, the results will be more applicable the appl at on o stu y results?
to our patient population, many o whom will not take their
medications properly. In summary, intention to treat analysis =
good. QUICK QUIZ: FEELiNG SENSiTiVE ABOUT
SOMETHiNG SPEc iFic ?
“OK,” you think. “I’ll pay attention to the intention to treat
analysis.” So, you next look at the inclusion and exclusion Which o the ollowing statements is true?
criteria. In this diabetes drug study, the exclusion criteria A) Speci city is the most important test characteristic when
include renal disease, a history o heart ailure, coronary trying to nd a very dangerous disease.
artery disease, and peripheral vascular disease. B) As sensitivity increases, speci city decreases.
CHAPTER 28 • EVid ENc E-BASEd MEd ic iNE 715

C) Speci city need not be considered as long as a test is sensi- 0.1 99


tive enough. 0.2 98
D) As sensitivity increases, speci city increases.
0.5 96
The correct answer is “B.” As sensitivity increases, speci c- 2000
ity decreases. T is makes intuitive sense. T e more cases you 1 1000 90
500
detect, the more alse positives you will have. We can have a 2 200 80
sensitivity o 100% i , or example, we say that everyone with 100
WBCs in their blood and a cough has pneumonia. We will pick 50 70
5
20 60
up everyone with the disease (very sensitive) but also a lot o
10 10 50
patients without the disease (poor speci city…everyone has at 5 40
least one, lonely, white cell running around). Ideally, we would 20 2 30
like to have a diagnostic screening test with both high sensi- 1
20
30 0.5
tivity and high speci city. In reality, there is an inherent trade- 40 0.2
o between sensitivity and speci city—as sensitivity increases, 50 0.1 10
speci city decreases and vice versa. “A” is incorrect. Generally, 60 0.05
5
when it is very dangerous not to detect a disease, it is important 70 0.02
to have a highly sensitive test (one that will f nd “all” cases) with 0.01
80 0.005 2
an acceptable speci city. “C” is incorrect. T is is why both an 0.002
ELISA and a Western blot may be done when trying to detect 90 1
0.001
HIV. T e ELISA is very sensitive (will pick up the great major- 0.0005
95 0.5
ity o HIV cases) but is not very speci c (will categorize a lot o
patients who do not have the disease as positive). T e Western 98 0.2
blot is more speci c and will lter the true positives rom the
alse positives ound on the screening test (the ELISA). 99 0.1
Pre-Test Likelihood Post-Test
Probability (%) Ratio Probability (%)
QUICK QUIZ: ALL THAT GLiTTERS . . .
FIGURE 28-1. Repro u e rom the c entre or Ev en e Base Me ne.
Ava lable at http://www. ebm.net.
You are having a meaning ul discussion with an industry repre-
sentative (yeah, right). OK, let’s recalibrate. You are being sold a
package o goods by an industry representative. She says that i intensive therapy or their diabetic neuropathy. In the rst
their test or Dreaded Disease is positive, the likelihood ratio o group, those assigned to routine therapy, 10% o patients
the disease being present is 3. developed neuropathy. In the second group, those assigned
to intensive therapy, 2%o patients developed neuropathy.
Your response to this is:
A) “Great! T e disease is three times more likely to be present i Question 28.3.1 Using the data above, how many patients
the test is positive.” with diabetes need to be treated with intensive therapy to
B) “Not so great! A likelihood ratio o 3 is pretty much worth- prevent the development o one case o neuropathy?
less in di erentiating between those who are ill and those A) 10.
who are not.” B) 11.
C) “What is this likelihood ratio stu anyway?” C) 8.
D) “What happened to my ree lunch?” D) 12.5.
E) 25.5.
The correct answer is “B.” In a situation in which the pretest
probability o a disease is between 30% and 70%, a likelihood Answer 28.3.1 The correct answer is “D.” T e question is really
ratio can meaning ully reduce the possibility o disease presence asking, “What is the number needed to treat (NN )?” In this
only i it is < 0.1. In a situation in which the pretest probability o question, the absolute risk reduction is 8% (10% in control
a disease is between 30% and 70%, a likelihood ratio can mean- group vs. 2% in the treated group). T e NN is the number o
ing ully increase the possibility o disease presence only i it is patients who need to be treated to prevent one adverse outcome.
over 10. So, a likelihood ratio o 3 is more or less useless. Draw o calculate this, we need to know a ew other terms:
some lines on this and you will see what we mean (Fig. 28-1).
ARR = Absolute Risk Reduction = control group event rate
(CER) − experimental group event rate (EER).
CASE 28.3
NN = 1/ARR (in percent, so 8% = 0.08 and 20% = 0.20)
One common and debilitating complication o diabetes is
neuropathy. In a ( ctional) study by ooth E. Fairie et al., one Using the values given above, ARR = 10 − 2% = 8% and NN =
group had routine therapy and an experimental group had 1/0.08 = 12.5.
716 FAMiLY MEd ic iNE EXAMiNATiON & BOARd REViEW

T e anti-clotting properties o aspirin are well described. In or relat ve r sk (RR), relat ve bene t, o s rat o, et ., re
a ( ctional) trial studying the long-term outcome o stroke member that the c i 95% rosses “1,” there s no er
patients by Don Sox et al., 1%o patients on long-term aspirin en e between the groups. Thus, an RR o 4.2, c i 95% o
therapy developed new onset o strokes and 50% o patients 0.8 to 10 s ons stent w th a 0.8 t mes r sk or a 10 t mes
without aspirin therapy developed new strokes. r sk (or bene t). However, s n e the c i 95% rosses “1,”
there s no real eren e between the groups.
Question 28.3.2 Using the data above, how many stroke
patients need to be treated with aspirin therapy to prevent
one new stroke (what is the NNT)?
A) 2. HELPFUL TIP:
B) 8. c on en e ntervals are use ul when eterm n ng the
C) 10. magn tu e o a treatment e e t. For example, a RR
D) 12. has c i 95% o 1.2 to 1.4, th s means there s a small
E) 25. eren e (0.2–0.4 t mes) between the two groups, even
though t s stat st ally s gn ant. Remember, some-
Answer 28.3.2 The correct answer is “A.” Again, the NN is thing that is statistically significant may not be clini-
the number o patients who need to be treated to prevent one cally significant. On the other han , the RR has a c i
adverse outcome. NN = 1/ARR, where ARR = CER – EER. 95% o 10 to 20, th s s a major eren e between the
Using the values given above, 50 − 1% = 49% = ARR and NN = groups. Th s means that one group has a 10 to 20 t mes
1/0.49 = 2 (well, really close to 2). greater r sk (or bene t epen ng on what s be ng
stu e ) than oes the other group.
In a ( ctional) pharmaceutical study by Amanda Hugginkes
et al., Group A is the placebo group and Group B is the group
that received the actual new drug. Data were gathered on
Groups A and B and con dence intervals (CI) were calcu- In a ( ictional) clinical trial testing a new provider order
lated. Side e ect rates were calculated as a percentage o each entry (POE) technology at Big Important University Hos-
group. pital, relative and absolute risk reduction is discussed.
A group o amily medicine residents at the hospital was
Question 28.3.3 Using the 95% CI, which o the ollowing allowed to give verbal orders during their intern year and
group comparisons are statistically signif cantly di erent? averaged seven medication errors per year. In the residents’
A) Group A CI 30% to 46% and Group B CI 44% to 88%. second year, POE was instituted (in which physicians were
B) Group A CI 10% to 30% and Group B CI 44% to 88%. required to enter orders and alerts to medication errors were
C) Group A CI 0.1% to 0.3% and Group B CI 0.2% to 0.4%. given be ore nalization o orders), and the group’s medica-
D) Group A CI 88% to 90% and Group B CI 88% to 90%. tion errors dropped to an average of four per year.
E) None o the above is statistically signi cant.
Question 28.3.4 Which o the ollowing is true?
Answer 28.3.3 The correct answer is “B.” T e CI is a range o A) T e RR reduction is 43% and the absolute risk reduction is
possible high to low values o data. T e true mean is likely to three in medication errors.
be in the speci ed range. In general, the larger the study group, B) T e RR reduction is 57% and the absolute risk reduction is
the more narrow the CI. When you have a large study, you are three in medication errors.
more likely to get closer to the true value. C) T e RR reduction is 43% and the absolute risk reduction is
“B” is correct because when comparing the CI between our in medication errors.
two groups, there is no overlap. When there is an overlap D) T e RR reduction is 57% and the absolute risk reduction is
o CI, as in the other options, the groups are not statistically our in medication errors.
signi icantly di erent. For example, in answer “A” the true E) POE has ruined medicine. I quit! boo-hoo!
mean value o Group A could lie anywhere between 30% and
46% (it could be 45%), and the true mean value o Group B Answer 28.3.4 The correct answer is “A.” POE compared with
could lie anywhere between 44% and 88% (it could also be no POE (the control group) resulted in a 43% relative decrease
45%); there ore, the groups have no statistically signi icant in the risk o a medication error— rom seven to our errors
di erence. per year (3/7 = 43%). T e di erence in the number o medica-
tion errors be ore and a er POE is three errors (7 − 4 = 3),
HELPFUL TIP:
which is the absolute reduction in the risk o a medication error.
c on en e intervals are usually g ven as “c i 95%,”
Now, think about this rom a pharmaceutical or medical device
mean ng that there s a 95% probab l ty that the true
representative’s point o view. Would you say (a) “We reduced
mean value w ll be w th n the c i. When look ng at c i
errors by 3 per year,” or would you say (b) “We reduced errors by
43%per year!”? T at’s how you sell a study.
CHAPTER 28 • EVid ENc E-BASEd MEd ic iNE 717

Objectives: Did you learn to . . . bene t o the drug vs. placebo. T e absolute bene t is only
• c al ulate number nee e to treat? 4% (12% cure with HLGD vs. 8% cure with placebo). For the
• d f erent ate between absolute an relat ve r sk re u t on? NN in this example, think about the previously given equa-
• Use on en e ntervals to eterm ne stat st al s gn tion: NN = 1/ARR, where ARR = CER − EER. T e con-
an e? trol group, the placebo, had a risk reduction o 8% (92% still
had disease); and the experiment group had a risk reduction
o 12% (88% still had disease). So, the ARR = 12% − 8% =
CASE 28.4 4%, and NN = 1/0.04 = 25. NNH cannot be calculated with
the in ormation available since the adverse event rate is not
Mr. Handsome Q. Drugrep has come to tell you all about
known.
Happy Lucky Golden Drug (HLGD) that is newly indicated
In real li e, there are o en more dramatic examples o how
or the treatment o the Dread ul Yucks. As a primary care
relative and absolute risks di er. It may be stated that there is a
doctor, you are concerned about better treatment o this dis-
50% reduction in complications o diabetes using Drug A ver-
ease. Current standard treatment involves ChemoRAdical
sus placebo. However, when translated into patients, this could
Pharmacotherapy (CRAP). Cure rates with CRAP are only
be 1/1,000 complications o diabetes in the drug group versus
about 10%. Mr. Drugrep has a study that shows HLGD has
2/1,000 complications o diabetes in the placebo group. T is is
a 12%cure rate versus placebo. He’s very excited and expects
a 50% relative decrease in adverse outcomes but in act may be
HLGD to be the new standard o care.
clinically meaningless. T e absolute risk reduction is 1/1,000 or
0.1%! T is ploy is o en used to make drug studies look good.
Question 28.4.1 To his argument, you appropriately T us, anytime you are looking at a new drug, ask or the abso-
respond: lute risk reduction and the NN and the NNH. Forget the rela-
A) “Wow. HLGD is clearly superior to CRAP.” tive risk reduction and the p-values.
B) “Hmm. HLGD is statistically no di erent rom CRAP.”
C) “Wow. HLGD is clearly superior to placebo.” Mr. Drugrep tells you that the adverse event rate or HLGD
D) “Do you have ree samples o HLGD? Where’s lunch?” is only 1%. Aren’t you impressed? But he rowns a little when
E) “I need more in ormation be ore I can make an in ormed you want to know the NNH.
decision.”
Question 28.4.3 To calculate NNH, you ask him or:
Answer 28.4.1 The correct answer is “E.” You need more in or- A) T e types o adverse events that occurred in the treatment
mation. Be ore coming to market, a drug manu acturer must group.
demonstrate sa ety and e cacy o a drug. T e new drug may B) T e percentage o adverse events that occurred in the con-
or may not be compared with another currently available treat- trol group.
ment. Without a study comparing HLGD to CRAP, you cannot C) T e percentage o adverse events with standard treatment.
say anything about how these drugs compare, even i HLGD D) T e cure rate in the treatment group.
looks better versus placebo. In addition, “C” is incorrect because
the placebo results have not been given. Answer 28.4.3 The correct answer is “B.” Adverse e ects o a
drug will o en be reported as an absolute number, and here it
You ask Mr. Drugrep or more in ormation. He proudly tells is 1%. So, the conclusion you are given by the pharmaceutical
you the drug study involved 10,000 subjects with the Dread- industry may be 50%reduction in disease and only a 1%risk
ul Yucks, randomly assigned to placebo (5,000) or HLGD o side e ects o the drug. Both o these statements are true, but
(5,000). All o the subjects completed the trial. At the end o it’s an “apples and oranges” comparison. We pre er comparing
1 year, 400 subjects on placebo (8%) were cured and 600 sub- apples-to-apples (or corn-to-corn in Iowa). In order to directly
jects on HLGD (12%) were cured. compare bene ts and harms, we need to know the NN and
the NNH.
Question 28.4.2 He correctly tells you that: Let’s say that when you ask Mr. Drugrep, he tells you that the
A) T e NN is 10,000. adverse event rate in the placebo group was 0.5%. Here’s the cal-
B) T e number needed to harm (NNH) is 10,000. culation: NNH = 1/ARI, where ARI (absolute risk increase) =
C) T e relative bene t o HLGD versus placebo is 50% greater risk in experiment group − risk in control group.
cure rate. Using the numbers in this question: ARI = 1 − 0.5 = 0.5;
D) T e absolute bene t o HLGD versus placebo is 50% greater NNH = 1/0.5 = 2.
cure rate. So, or HLGD, the NN is 25 and the NNH is 2. By the way,
the adverse event in question is dis guring, pain ul ear hair
Answer 28.4.2 The correct answer is “C.” When looking at growth. You will have to treat 25 patients with HLGD to cure
drug studies, bene t is o en stated as “relative bene t” or rela- one case o the Dread ul Yucks; but with every two patients you
tive risk reduction. In this question, 600/5,000 patients bene t treat, one will have an adverse event. Demand NN and NNH:
rom HLGD and 400/5,000 bene t rom placebo; thus, 200 more how many patients who take the drug will bene t and how
patients are cured with HLGD, 200/400 = 0.5 = 50% relative many will be harmed?
718 FAMiLY MEd ic iNE EXAMiNATiON & BOARd REViEW

Objectives: Did you learn to . . . Answer 28.5.2 The correct answer is “B.” A case–control
• Employ c i n the analys s o ata? study, like this one, will look at select subjects who are catego-
• Analyze ata us ng r sk re u t on an relat ve bene t? rized based on outcome and try to nd associations with cer-
• Un erstan the mportan e o absolute r sk re u t on, NNH, tain variables. Case–control studies do not ollow subjects over
an NNT when l n ally apply ng ata rom a stu y? time and there ore are not prospective. Cohort studies look at
groups starting at time zero and ollowing them or a speci ed
amount o time to nd an association between a variable and an
CASE 28.5 outcome. T e variable in question is not under the researcher’s
Mounting paperwork and electronic medical record hassles control. An example o a cohort study might be one looking at
have played a role in your decision to make a career change. the association between two di erent diets (e.g., high-protein
You have ound a nice academic job with a research ocus— vs. high-carbohydrate) and the development o type 2 diabetes.
minimal patient care, 10 weeks o vacation, no paperwork. T e highest quality evidence is produced by a randomized,
Your work centers on reducing the risk o stroke in patients double blind, controlled trial, in which the researcher has con-
who have survived one stroke. trol over exposure to a variable and studies its e ect on an out-
come. In general, the strength o trial design goes: experimental
Question 28.5.1 This is an example o which category o study > cohort study > case–control study > cross-sectional
prevention? study. Un ortunately, it is not possible to design randomized
A) Primary prevention. controlled studies or all conditions. So, a well-done cohort
B) Secondary prevention. study may be the best we can do.
C) ertiary prevention.
D) Quaternary prevention. You are concerned about numerous con ounding variables in
your study population. Never ear!

Answer 28.5.1 The correct answer is “C.” T e idea behind


primary prevention, a big interest in primary care, is to pre- Question 28.5.3 Your trusty statistician recommends the
vent a disease rom occurring at all by removing its cause (i.e., ollowing in order to minimize con ounding:
inf uenza vaccine to prevent illness rom inf uenza). Primary A) Multivariate analysis.
prevention may occur in the healthcare setting but is o en in B) Care ul calculation o p-values.
the domain o public health. Secondary prevention detects C) Matched controls and cases.
disease at an early stage so that intervention can prevent pro- D) A and B.
gression (i.e., Pap smears detecting dysplasia prior to cancer E) A and C.
declaring itsel or treating elevated cholesterol prior to vascu-
lar events). Your new job will be to study tertiary prevention: Answer 28.5.3 The correct answer is “E.” Con ounders can be
the reduction in complications and mortality due to disease a serious threat to any study. Con ounders result rom extrin-
a er it is recognized. T e line between secondary and tertiary sic actors—things that may a ect the outcome and are also
prevention can be blurry: some would consider preventing associated with the variable but are not accounted or in the
another stroke “secondary” prevention and preventing stroke study. As an example, a study may nd an association between
complications (e.g., muscle atrophy and pressure ulcer) “ter- long-haul truck driving and lung cancer. I tobacco use was not
tiary” prevention. T ere is no such thing as quaternary pre- accounted or in this study, the results o the study would be
vention. meaningless. obacco is a con ounder. It is always advisable to
look at a study with an eye or what con ounder might be miss-
In between day-trading and co ee breaks, you plan to ing. Con ounding can be limited by a study design that antici-
study two groups o patients (A and B) to see i variable pates con ounders and matches controls and cases (“C”). It is
XYZ makes any di erence in death or recurrent stroke. important to note that i you match your control and cases on
here is no randomization and there are no interven- a variable, you can no longer study that variable as a potential
tions. You are just reviewing records to see how each cause o the outcome. For example, i you match cases and con-
group did. Subjects in Group A had a stroke and then had trols on county o residence among the truck drivers, you can
another stroke or died a year later. Subjects in Group B no longer explore county o residence as a risk actor or lung
had a stroke but were alive with no recurrent stroke at the cancer. Also, multivariate analysis (“A”) is a statistical method
time o the study. You assess the presence o XYZ in each that allows or adjustment o known con ounders. “B” is incor-
group. rect because p-value has nothing to do with con ounding but
will tell you whether the results should be considered signi -
Question 28.5.2 This type o study is called a: cant or not.
A) Prospective study.
B) Case–control study. When you review the literature, you nd that there are a
C) Cohort study. number o small studies looking at the e ect o variable XYZ
D) Randomized, controlled study. on stroke victims. You even nd a meta-analysis.
CHAPTER 28 • EVid ENc E-BASEd MEd ic iNE 719

TABLE 28-3 FACTORS IN DETERMINING THE


JADAD SCORE FOR META-ANALYSES
Ran om zat on W th rawals an ropout Bl n ng
Drug X Cure s %

numbers

“Biocre e p”: the ne w a ge nt Z


is compa re d to the ne w Cure s % Drug Y
Question 28.5.4 I this is a well done meta analysis, you a ge nt Y. Z is cle a rly infe rior
to X but is non-Infe rior to Y.
should f nd all o the ollowing EXCEPT:
A) Statistically con rmed heterogeneity between the included Ne xt a ge nt
studies. Cure s %
B) A thorough search or all valid studies. Drug Z
C) An evaluation o whether estimates change with varying
assumptions. He re , drug Y is non-infe rior to drug X e ve n though
D) T e exclusion o poor-quality studies. it ma y not a ctua lly be
E) T e studies included measure the same underlying e ect. a s good a s the s ta nda rd tre a tme nt.
FIGURE 28-2.
Answer 28.5.4 The correct answer is “A.” Hope ully, a meta-
analysis would con rm homogeneity between studies. For
example, i one study measured NIH stroke scale and another
measured patient quality o li e (the outcomes are hetero- Y. d rug Z s non n er or to d rug Y although t learly
geneous), it would be impossible to combine the studies. ures ewer people than d rug X. Thus, the stan ar
Although there is controversy regarding the appropriate use o aga nst wh h new rugs are ompare be omes pro
meta-analyses, they are o en used to study various outcomes gress vely less e a ous (F g. 28 2). in another on ern,
by combining smaller studies. A meta-analysis is a systematic resear hers an e ne the r own “marg n” o non n er
review that combines the results o previous studies to evaluate or ty. For example the marg n s “2” (as n some o the
the magnitude or direction o an e ect or to evaluate the e ect new ant oagulant stu es), the new rug oul allow
on a subgroup. All valid studies looking at similar outcomes up to tw e as many events (e.g., pulmonary embol ) as
should be included; poor-quality studies should be excluded. the ol rug an st ll be ons ere “non n er or.”
T e Jadad score is one common way that studies are judged as
to their appropriateness or a meta-analysis ( able 28-3). Meta-
analyses should also include a “sensitivity analysis.” T is may
consist o excluding large studies and only analyzing smaller CASE 28.6
studies. It may consist o changing the economic assumptions
Uh, oh . . . Here comes the math. T is section is important,
in a cost-bene t meta-analysis. I the outcome is the same
especially the concepts o positive and negative predictive
either way, the result is said to be “robust.” I excluding some
values (PPV and NPV) and the concepts o sensitivity and
studies or changing the underlying economic assumption
speci city. You need not do the math i you do not want to
changes the result, one has to wonder about the quality o the
(although it is simple). Here is a summary:
studies etc.
Sensitivity: How o en the test will pick up the disease i
Objectives: Did you learn to . . . it is there. Sensitivity = true positives/(true positives + alse
• d e ne f erent types o prevent on? negatives). Note that the sum o true positives + alse negatives
• d e ne f erent stu y types? represents all o the people with disease.
• i ent y an a ount or on oun ng var ables? Speci city: Speci city is de ned as the proportion o
• d es r be some hara ter st s o a meta analys s? patients who do not have the disease and who will test nega-
tive or it. Speci city = true negatives/(true negatives + alse
positives). Note that the sum o true negatives + alse positives
represents all o the people who do not have disease.
HELPFUL TIP:
Positive predictive value: T e probability that someone with
You w ll see an n reas ng number o “non n er or ty”
a positive test actually has the disease. T is takes the prevalence
stu es be ng one. What oes “non n er or” mean?
o a disease into account. For example, an individual with a pos-
Bas ally, t means that two rugs have some overlap n
itive HIV test who is an IV drug user is more likely to really have
the r e a y. See F gure 28 2. d rugs Y s non n er or to
the disease than a clean-living nun with a positive HIV test. In
d rug X even though t may not ure as many pat ents.
the nun, the test is more likely to be a alse positive.
“Non n er or” oes not mean “as goo as.” “B o reep”
(wh h s one o our avor te terms an rem n s us o
Number o true positives
zomb es) o urs when rug Z s now ompare to rug PPV =
Number o true positives + Number o alse positives
720 FAMiLY MEd ic iNE EXAMiNATiON & BOARd REViEW

Negative predictive value: T e probability that someone with a TABLE 28-5 TABLE FOR QUESTION 28.6
negative test actually does not have the disease. Again, this takes
Actual Reception
the prevalence o the disease into account. So, or example, a
negative HIV test in an IV drug user rom Sub-Saharan A rica + −
with a CD4 count o 150/mm 3 and PCP is likely to be a alse
Test + 90 (true pos t ve) 5 ( alse pos t ve)
negative. Conversely, a negative HIV test in a nun, or example,
is likely to be a true negative. Re ept on − 10 ( alse negat ve) 95 (true negat ve)

Total 100 100


Number o true negatives
NPV =
Number o true negatives + Number o alse negatives
So let’s add actual numbers to the table (above). Let’s use a
A new test, the “reception-o-meter,” has been developed that population o 10,000. We multiply by the prevalence o recep-
can tell whether a cell phone will have reception in a given tion to get the subpopulation totals. Ninety-nine percent o the
area (allegedly better than a guy walking around asking, population has reception (99% prevalence). So, 99% preva-
“Can you hear me now?”). When compared with the gold lence × 10,000 = 9,900 with reception; 1% × 10,000 = 100
standard o turning on your cell phone and checking whether without reception. Once we have these numbers, we simply
you have reception or not, the new test has a sensitivity o multiply by the sensitivity and the speci city to get the exact
90%(will pick up a signal 90%o the time when there is one) cell numbers to plug into the table above: 9,900 × 90% sensi-
and a speci city o 95% (there are only 5% alse positives; tivity = 8,910 or cell A (“true positives”); 9,900 − 8,910 = 990
thus, 95%o the time when the reception-o-meter says there or cell C (“ alse negatives”); 100 × 95% speci city = 95 or cell
is a signal, there will actually be one). D (“true negatives”); 100 − 95 = 5 or cell B (“ alse positives”).
So how can you tell i the phone company is pulling a ast See able 28-6.
one or i this is a good test? You need to know the PPV o the Once the table is lled in, these numbers can then be used to
test. In order to calculate the PPV, you need three pieces o calculate the PPV, using the equation above. In this case, a/(a +
data: the sensitivity o the test (how o en the test will pick b) = 8,910/8,915 = 99.9%.
up the “disease” i it is there), the speci city o the test (how For those who pre er the Bayes theorem method, here’s
o en you will get a alse positive), and the prevalence o the how this approach is done. Bayes theorem shows the relation-
condition, which in this case is the prevalence o having cell ships between sensitivity, speci city, prevalence, PPV, and
phone reception (in other words, the true amount o cell NPV. T e equation or PPV, derived rom Bayes theorem, is
phone reception in a given area). shown as is the calculation based on the numbers rom the
You are currently in Los Angeles, attending a CME course question:
where the reception or carrier X is 99%. You check your
“reception-o-meter” and it says you have coverage. But does Number o true positives
this mean you have coverage? PPV =
Number o true positives + Number o alse positives
In order to answer this question, you can use Bayes theorem
or set up 2 × 2 tables. Here’s the 2 × 2 table method. Begin Sensitivity × prevalence
by drawing a 2 × 2 table and lling in what you know. See PPV =
(Sensitivity × prevalence) + [(1 - speci city) × (1 - prevalence)]
able 28-4 and able 28-5.
So this makes it easy.
0.9 × 0.99
= = 99.9%
Sensitivity = a/(a + c), (0.9 × 0.99) + [0.05 × 0.01]
Speci city = d/(b + d),
PPV = a/(a + b),
NPV = d/(c + d)
TABLE 28-6 TABLE FOR QUESTION 28.6 AFTER ADDING
I we have 100 phones, the data will look like that above. IN POPULATION DATA FOR 99% PREVALENCE
A ter Adding Actual 99% Prevalence

TABLE 28-4 SAMPLE TABLE Actual Reception

Test Disease + −

+ − Test + 8,910 5

+ a (true pos t ve) b ( alse pos t ve) Re ept on − 990 95

– ( alse negat ve) (true negat ve) Total 9,900 100

Total a+ b+ (99% prevalen e)


CHAPTER 28 • EVid ENc E-BASEd MEd ic iNE 721

Question 28.6.1 What would the likelihood o not having 1,000 × 10% will get the same result or cell “c”); 9,000 × 95%
coverage be i the “reception o meter” had said you did not speci city = 8,550 or cell “d”; 9,000 − 8,550 = 450 or cell “b”).
have coverage (what is the NPV)? T ese numbers can then be used to calculate the PPV, using
A) 95%. the equation above. In this case, a/(a + b) = 900/(900 + 450) =
B) 90%. 66.7% (rounds to 67%). Using Bayes theorem, the equation is
C) 50%. as ollows.
D) 9%. Sensitivity × prevalence
E) None o the above. PPV =
(Sensitivity × prevalence) + [(1 - speci city) × (1 - prevalence)]

Answer 28.6.1 The correct answer is “D.” T e question asks 0.9 × 0.1
or the NPV—the likelihood o not having coverage i the recep- = = 66.7%
(0.9 × 0.1) + [0.05 × 0.9]
tion-o-meter is negative. T is also can be derived rom Bayes
theorem or calculated using a 2 × 2 table. For those o you who
pre er the Bayes theorem method, the equation or NPV, derived HELPFUL TIP:
rom Bayes theorem, is shown as is the calculation based on the A test that has a negat ve pre t ve value o 99% may
numbers rom the question. soun goo . But only 1% o the populat on has the s
ease, o ng no test w ll have a 99% negat ve pre t ve
Number o true negatives
NPV = value.
Number o true positives + Number o alse negatives

You are now in rural Russia where you were invited to help Question 28.6.3 Still in that remote tuberculosis in ested
with community e orts to ght multidrug resistant tubercu- region o Russia, we ask: What would the likelihood o hav
losis. Here cell phone reception is 10% or Carrier Y. You check ing coverage be i the “reception o meter” said you did not
your “reception-o-meter” and it says you have reception. have coverage?
A) 50%.
Question 28.6.2 What is the likelihood that your cell phone B) 40%.
actually will have reception i you try to make a call? C) 30%.
A) 91%. D) 1%.
B) 83%. E) None o the above.
C) 67%.
D) 16%. Answer 28.6.3 The correct answer is “D.” Again, you can use
E) None o the above. the 2 × 2 method or Bayes theorem. T e 2 × 2 table or this
question is the same as it was or the previous question (see
Answer 28.6.2 The correct answer is “C.” You can use the 2 × 2 able 28-7). However, unlike previously, you are asked or the
method or the Bayes theorem methods. likelihood o reception i the “reception-o-meter” said there was
Here’s what our 2 × 2 table looks like. See able 28-7. no reception. In other words, you have been asked to calculate
o convert to 10% prevalence, we start with a large baseline the alse negative rate (FNR) or this scenario. T e equation or
population and multiply by the prevalence to get the subpopu- the FNR is below.
lation totals (10% prevalence × 10,000 = 1,000 with reception; False negatives
90% × 10,000 = 9,000 without reception). Once we have the FNR = or
False negatives + true negatives
subpopulation totals, we multiply by the sensitivity and the
speci city to get the exact cell numbers (1,000 × 90% sensitivity c 100
= = 1%
= 900 or cell “a”; 1,000 − 900 = 100 or cell “c” (or alternately c+ d 100 + 8,550

TABLE 28-7 EXAMPLE OF A COMPLETED 2 × 2 TABLE FOR QUESTION 28.6.2


Be ore Adding Actual Prevalence A ter Adding Actual Prevalence
Actual Reception Actual Reception

+ − + −

Test + 90 5 Test + 900 450

Re ept on − 10 95 Re ept on − 100 8,550

Total 100 100 Total 1,000 9,000

(50% prevalen e) (10% prevalen e)


722 FAMiLY MEd ic iNE EXAMiNATiON & BOARd REViEW

You were not asked to calculate it, but there is also a alse posi- FNR: Patient has the disease but the test is negative.
tive rate (FPR), which is shown below. FNR = (FN/( P + FN)) = 25/100 = 25% FNR. Also calcu-
lated as 1 − sensitivity.
False positives
FPR = or FPR: T e patient has a positive test but does not have the dis-
False positives + true positives ease.
b 450 FPR = (FP/(FP + N)) = 200/900 = 0.22 or 22% alse posi-
= = 33%
a+ b 450 + 900 tive. Also calculated as 1 − speci city.

We are going to make another assumption here about the


Cervical cancer is a disease in which early detection can make prevalence o disease. T e prevalence is the proportion o
a great di erence in halting disease progression. One screen- individuals who have the disease at any point in time. One
ing procedure or this disease is the Papanicolaou (“Pap”) way to describe it is as ollows: prevalence = (( P + FN)/ otal
smear. In a ( ctional) study to assess the competency o tech- population) = 100/1,000 = 10% or prevalence o 100 per
nicians who read the Pap smear slides, a local lab checked 1,000 people.
their technician’s work against patient records.
A total o 1,000 Pap smears were read. O these, Question 28.6.5 Given the above results o the Pap smear
100 patients had cervical abnormalities based on biopsy screening tests and i the prevalence o cervical abnormali
(gold standard). O this group, 75 had abnormal (positive) ties among women is 10%, then applying Bayes theorem,
Pap smears and 25 had negative Pap smears. T ere were we f nd:
900 women without disease. O these 900 women, 200 had A) T e PPV is 27%.
positive Pap smears and 700 had negative Pap smears. Note B) T e NPV is 96%.
that these are example numbers only, have no basis in reality, C) T e PPV is 0.999.
and do not ref ect the actual sensitivities and speci cities o D) Unable to solve the problem with data provided.
Pap smears. E) A and B.

Question 28.6.4 Using the data above, which o the ollow Answer 28.6.5 The correct answer is “E.” T e prevalence o a
ing is true about this survey o Pap smear technicians? disease is the proportion o individuals who have the disease at
A) FNR is 20%. a given point in time (( P + FN)/( otal population) = 0.1 or
B) FPR is 15%. 10%).
C) T e sensitivity o the Pap test is 75%. T e PPV o a test is the probability that a disease exists
D) T e speci city o the Pap test is 98%. given a positive test result= P/( P + FP) or 75/275 = 27%.
E) T e prevalence o cervical cancer in this sample is 7.5%. So, a patient with a positive test result only has a 27% chance
o actually having the disease because there are so many alse
Answer 28.6.4 The correct answer is “C.” T e sensitivity o the positives.
test is 75%. Setting up the data in a 2 × 2 table, we are able to T e NPV o a test is the probability o no disease given a
answer the question. See able 28-8. negative test result ( N/(FN + N)) = 700/725 = 96%. So, a
Sensitivity: Probability that a patient with the disease will patient with a negative test has a 96% chance o not having the
have a positive result. disease. T is is because there are ew alse negatives compared
with the size o the overall population. I , or example, there
Sensitivity = ( P/( P + FN)) = 75/100 = 0.75 or 75% sensitive.
were 200 alse negatives in the same population, the negative
Speci city: Probability that a patient without the disease will predictive value would be only 700/900 = 78%.
have a negative test.
Speci city = ( N/(FP + N)) = 700/900 = 0.777 or about Recall that 100 out o 1,000 women had positive biopsies and
78% speci c. thus had the disease regardless o what the Pap test said.

Question 28.6.6 How does the pretest probability o cervi


TABLE 28-8 EXAMPLE OF A COMPLETED cal abnormalities among women compare with the post
2 × 2 TABLE FOR QUESTION 28.6.4 test probability?
Pap Test A) Posttest probability is about three times greater than the pre-
Result Cervical Disease No Cervical Disease test probability.
Pos t ve True pos t ve (TP) = 75 False pos t ve (FP) = 200 B) Pretest probability is about three times greater than the post-
test probability.
Negat ve False negat ve (FN) = 25 True negat ve (TN) = 700 C) Posttest probability is 10 times greater.
Total TP + FN = 100 FP + TN = 900 D) Pretest probability is 10 times greater.
E) T e pretest and posttest probabilities are equal.
CHAPTER 28 • EVid ENc E-BASEd MEd ic iNE 723

Answer 28.6.6 The correct answer is “A.” T e pretest prob-


than rely ng on relat ve r sk re u t on. The same al ulat on
ability is given above as 100/1,000 or 10%. We know that 10% an be one or s reen ng tests (e.g., number o women
o the population has the disease. T e posttest probability is nee e to s reen to avo one breast an er eath).
de ned as the PPV. Remember rom above, the PPV o a test is
d o not raw on lus ons rom subgroup analyses. The only
the probability that a disease exists given a positive test result =
on lus on that an be rawn s, “Th s must be stu e .”
P/( P + FP) or 75/275 = 27%. Comparing the two results,
pretest probability o 10% and posttest probability o 27%, we Re ogn ze that the ut l ty o a test s ont ngent upon the
sens t v ty an spe ty o the test an the prevalen e o
nd that the posttest probability is about three times greater
sease n the populat on be ng teste . There ore, a sens t ve
than the pretest probability. I answer “E” were correct and the
an spe test may have a low pre t ve value n a popula
pretest and posttest probabilities were equal, there would be no t on w th very low sease prevalen e.
point in doing the test.
When evaluat ng a non n er or ty stu y, look or the “marg n”
Objectives: Did you learn to . . . use by the nvest gators. Th s s the max mum extent o
• d e ne an al ulate sens t v ty an then apply t to ata l n al eren e that w ll be ons ere non n er or (e.g.,
nterpretat on? a marg n o 2 means tw e as many events an o ur n the
• d e ne an al ulate spe ty an then apply t to ata exper ment group an st ll be ons ere non n er or).
nterpretat on?
• c al ulate pos t ve an negat ve pre t ve values?
• Apply Bayes theorem to eterm ne the ut l ty o a test?
BIBLIOGRAPHY
Centre or Evidence-Based Medicine. Available at http://www.
Clinical Pearls cebm.net. Accessed on June 1, 2015.
Fletcher RH, et al. Clinical Epidemiology: T e Essentials. 5th ed.
A h ghly sens t ve test helps to rule OUT sease; a h ghly Baltimore, MD: Williams & Wilkins; 2012.
spe test helps to rule iN sease. Friedland DJ, et al. Evidence-Based Medicine. New York, NY:
A treatment that s statistically significantly super or to McGraw-Hill; 1998.
pla ebo may not o er a clinically significant bene t. Use Greenhalgh . How to Read a Paper: T e Basics o Evidence-
l n al ju gment when nterpret ng stu y results. Based Medicine. 5th ed. West Sussex, UK: John Wiley and
c ompare number nee e to treat (NNT) w th number Sons, Ltd, BMJ Books; 2014.
nee e to harm (NNH) when ons er ng therap es, rather Pagano M, Gauvreau K. Principles o Biostatistics. 2nd ed.
Australia: Duxbury T omson Learning; 2000.
Patient-Centered Care
De se A. Ma t ez
29
Note: T is chapter deals with culturally responsive care and Cubans and South Americans actually have the same incidence
patient sa ety. Because not all Black patients identi y as A rican as NHW. T e Pima Indians in Mexico whose li estyle is not
Americans, the authors have elected to use the term Black in sedentary and who eat a more traditional diet actually have a
this chapter as inclusive o A rican Americans as well as those diabetes incidence o about 7%, while those in Arizona have
individuals who are o A rican descent but may be rom another an incidence o 38%. Non-Hispanic Blacks have an incidence
country. T e authors have also elected to use the term Latino/a higher than those o Latinos and NHW but not quite as high as
when re erring to individuals o “Hispanic” descent realizing the Mexican Americans and Puerto Ricans.
that many individuals identi y strongly with their country o
origin. We understand the sensitivity o this terminology and HELPFUL TIP:
ask your understanding. Eth c a d cultu al g oups a e ot u o m om a ep
dem olog cal o cultu al bel e sta dpo t. He ce, the e
CASE 29.1 a e o te la ge d e e ces betwee d v duals a d
You are doing a locum tenens job in Arizona on the Navajo subg oups. Th s s compl cated u the by the act that
reservation. You are seeing a lot o patients with diabetes, we all belo g to, a d a e lue ced by, mult ple d
which reminds you that Native Americans, Latinos, and e e t cultu al g oups. Although the e a e cultu al pat
Blacks all have a greater incidence o diabetes compared to te s w th healthca e, t y ot to adopt those patte s
the general U.S. population. as ste eotypes o the d v duals a d g oups.

Question 29.1.1 Which o the ollowing is TRUE about the


age -adjusted incidence o diabetes in adults? You are seeing a 57-year-old Navajo male with poorly controlled
A) All Alaska Native and Native American tribes have a higher type 2 diabetes. You are trying to understand why his hemoglo-
incidence o diabetes than non-Hispanic Whites (NHW). bin A1C is so high since he has access to healthcare, medicines,
B) Cuban Americans and South Americans have a higher inci- supplies, and an amazing doctor (i you do say so yoursel ). You
dence o diabetes than NHW. remember a lecture on culturally responsive patient-centered
C) Mexican Americans and Puerto Ricans have close to 90% medicine where they talked about investigating a person’s
higher incidence o diabetes than NHW. health problem using both disease and illness models.
D) Pima Indians in Mexico have the same high incidence o
diabetes as the Pima Indians in Arizona. Question 29.1.2 The di erence between disease and illness
E) Non-Hispanic Blacks have a lower incidence o diabetes is:
than Latinos in the United States. A) Disease is the biomedical explanation o the health condi-
tion; illness is the patient’s experience and understanding o
Answer 29.1.1 The correct answer is “C.” Mexican Americans their health condition.
and Puerto Ricans have a higher incidence o diabetes com- B) Disease is chronic; illness is acute.
pared with NHW (“non-Hispanic White” being the term used C) Diseases have known causes; illnesses do not.
by the U.S. Census Bureau). T e other statements are alse, and D) Disease is more serious and requently needs treatment; ill-
here’s how they break down. Although their diabetes incidence ness will resolve without medical intervention.
is increasing rapidly, Alaska Natives historically had a 5.5% age- E) Disease versus illness . . . really? Aren’t we just into some
adjusted incidence o diabetes compared with 7.1% or NHW. phony semantics here?
724
CHAPTER 29 • PATiEn T-CEn TEr ED CAr E 725

Answer 29.1.2 The correct answer is “A.” Simply put, “dis- • What kind o treatment do you think would be help ul?
ease” is the way that physicians understand a health problem • What are the most important results you hope to receive
and “illness” is the way the patient experiences it. T e cultur- rom the treatment?
ally responsive patient-centered model looks not only at the
pathophysiology o disease, which is the ocus o most o our HELPFUL TIP:
early medical training, but also at what the patient is experienc- Do ’t eel l ke you eed to memo ze all these spec c
ing, how it impacts him or her, and what is the larger cultural- quest o s. r athe , ocus o ask g quest o s that el c t
social-personal context o that health condition. T e patient- o mat o about the pat e t’s ow health bel e s
centered physician weaves back and orth between the disease a o judgme tal way. A othe use ul m emo c that
and illness model, diagnosing and treating both the disease and ca appl ed ma y d e e t co texts s “iCE,” wh ch
the illness. sta ds o ideas, co ce s a d expectat o s. Fo “ deas”
you m ght ask what the pat e t th ks s caus g h s/he
You want to ocus on your patient’s illness in the larger con- p oblem. Fo “co ce s” you m ght ask what he/she s
text o his li e and avoid just discussing glucose metabolism, wo ed s caus g the p oblem. Fo “expectat o s” you
diet, exercise, lab results, and medication adherence. (Your m ght ask what they th k w ll happe w th the p ob
usual approach o explaining diabetes and its complications lem a d what they a e expect g om the v s t o health
has not improved his control.) You ask him, “What has your ca e system.
experience with diabetes been like?” And he looks at you
strangely and answers that it has been tough.
T is same patient develops severe osteomyelitis o his right
Question 29.1.3 One way to help get an understanding o leg and is admitted to your inpatient service. He has not
his belie s about diabetes would be to: responded to several weeks o appropriate IV antibiotics and
A) Ask i amily members or tribal medical personnel are will- debridement. T e in ection is gradually spreading proxi-
ing to share in ormation about the intersection o cultural mally, and a decision has been made with the patient that you
belie s and care, speci cally diabetes; i they share in orma- will proceed with a below-the-knee amputation. Although
tion on their cultural belie s, listen with understanding and you are con dent that this is the right clinical decision, you
gratitude. worry about the act that Native Americans (Navajos report-
B) Do a literature search on Navajo health belie s with respect edly highest in the world), Latinos, Blacks, and people rom
to diabetes to help ormulate questions to ask him about his lower socioeconomic backgrounds all have disproportion-
potential cultural belie s. ally higher rates o limb amputation, even when adjusting or
C) Ask about his experiences and belie s about diabetes so you incidence o diabetes. T e surgeon you are working with asks
can have a greater understanding about his belie s to com- you to obtain in ormed consent rom the patient (and you
municate with him about care in a way to meet his needs. thought you le scut work behind when you le residency).
D) All o the above. You have heard that Navajos believe that talking about some-
thing bad happening (i.e., in orming them o the procedural
Answer 29.1.3 The correct answer is “D.” Any o these options risks) makes those adverse events more likely to occur.
may help you understand this patient’s health belie s. Commu-
nity members can be a good source o cultural patterns. Having Question 29.1.4 How do you want to get in ormed consent
a general idea o a cultural belie system makes it easier to ask rom your patient?
question about whether those belie s apply in this case and how. A) Proceed with the in ormed consent with the ull explanation
Asking the right questions will help to get more use ul in orma- o the bene ts and risks o the procedure.
tion. T ere are some canned questions that can be o assistance, B) ell him the bene ts but do not talk about the risks.
including those listed here rom series o questions developed C) Share with him the percentage chance o a good outcome
by a psychiatrist-anthropologist named Arthur Kleinman: and do not talk about the possible risks and complications.
• D) Explain the in ormation to a tribal leader and let the tribal
What do you call your problem?
leader decide what to communicate with the patient.
• What do you think caused your problem? E) Explain the process o in ormed consent to the patient and
• What do you think is going on with your body? allow him to opt out o any part o the process he desires to
• How have you treated this? avoid.
• What do you think helps?
• Why do you think it started when it did? Answer 29.1.4 The best answer is “E.” T is is a challenging
• situation since this patient’s belie system is in contradiction
What does your sickness do to you?
with modern Western medical practices. In order to create a
• How severe is it? culturally responsive plan o care or the patient, we should try
• What do you ear most about your problem? to align our guiding medical principles with the patient’s cul-
• What are the biggest concerns that your sickness has caused tural belie s. Using the our guiding principles o medical ethics
or you? (autonomy, bene cence, nonmale cence, and justice) is one
726 FAMiLY MEDiCin E EXAMin ATiOn & BOAr D r EViEW

way to work through this dilemma. Some would argue or “A”


because the patient cannot make an in ormed decision without CASE 29.2
knowing the potential risks and bene ts (autonomy), and, “it’s A young couple that just immigrated to your town rom Bos-
the law.” However, i the patient has traditional Navajo belie s, nia comes to your clinic because they are expecting their rst
proceeding with the ull in ormed consent may deter the patient baby. T e wi e is a healthy 23-year-old woman and in her rst
rom undergoing a bene cial or even li e-saving procedure, trimester o pregnancy.
which presents con ict with the principles o bene cence and Your scheduler knows that many Bosnians are o the Mus-
nonmale cence. “B” and “C” rely on stereotypes without mak- lim aith and has heard that emale Muslims are supposed to
ing sure o the patient’s belie s. Asking a tribal leader and/or re rain rom care by male physicians.
someone rom the tribe who works in medicine is an excellent
way o obtaining a cross-cultural perspective. However, ulti- Question 29.2.1 When making the appointment or this
mately we need to respect the patient’s autonomy. patient, the scheduler should:
A) Schedule her with the rst available OB provider, regardless
Your patient consents or the surgery with a request that i it is male or emale.
makes you uneasy—he wants to take the amputated limb B) Schedule her with the rst available emale OB provider.
home with him a er the surgery. He shares with you his cul- C) Schedule her with your only Bosnian-speaking OB provider
tural belie that his body needs to be buried whole in order to who is male (this will cut down on the time needed or an
cross over to the next world a er he dies. interpreter).
D) Ask i she has a pre erence rom whom she would like to
Question 29.1.5 How should you NOT explain to him the
receive care and schedule her within her pre erred provider.
potential health hazards o taking an in ected limb home?
E) Schedule her with the OB provider who has the most experi-
A) Explore how other people in the community who have had
ence and interest in working with patients rom diverse cul-
amputations have managed this.
tures (maybe Dr. Smith since he eats out at a di erent ethnic
B) Explain to him that you understand his concerns; however,
restaurant every night).
the health risk o him taking the limb home ar outweighs
any other consideration so the limb will have to be disposed
Answer 29.2.1 The correct answer is “D.” Although many
o by standard hospital practice.
Muslim women would nd it inappropriate to have a male pro-
C) Allow amily members and community leaders to partici-
vider, this is not universally true. Besides, we are not even sure
pate in the discussion i the patient wishes.
this patient is o Muslim aith (only about 40% o Bosnians are
D) Negotiate a way where as much as possible his wishes could
Muslim)! Cultural characteristics and belie systems are gener-
be met, personal and public health and sa ety preserved, and
alizations that o en accurately describe a population. However,
community and hospital regulations ollowed.
i you extrapolate these generalized belie s to the individual, you
Answer 29.1.5 The correct answer is “B.” T e underlying are engaging in stereotyping and may o en come to the wrong
concern is that the patient’s wishes are in direct con ict with tra- conclusion.
ditional medical practices. Although hospital policy may limit
your options, there is usually enough exibility to allow you to
HELPFUL TIP:
accommodate the patient’s wishes at least to some degree. You
Use you k owledge o a pat e t’s cultu e as a sta t g
can use the LEARN mnemonic (Listen, Explain, Acknowledge,
po t. Ask about a y assumpt o s o susp c o s a
Recommend, Negotiate) to help approach this cultural con ict.
o judgme tal way. Make su e you k ow how the be
First, you Listen to the patient’s perspective. O en you have to
l e appl es the cu e t co text.
probe with nonjudgmental questions to elucidate this. “C” can
provide some cultural context or both the Listening and Nego-
tiating phases. “A” would be part o Explaining your perspec-
tive. Next you Acknowledge both the di erences and similari- When she arrives in your clinic, you realize she speaks no
ties between the two perspectives. T en you Recommend a plan English and her husband speaks only a little. Using his limited
hope ully where you and the patient both get what you want. English and some hand gestures, you eel that you could con-
Finally as in “D,” you may need to Negotiate. duct an interview with them. Because you completed a course
Objectives: Did you learn to . . . entitled “Advanced Li e Support or Culturally Responsive
• Desc be some d e e ces d abetes p evale ce amo g Care,” you are able to deal with such a situation! Your mind
d e e t eth c g oups? dri s back to that course . . . ah, CME on the beach!
• Def e the te ms “d sease” a d “ ll ess?”
• Explo e a pat e t’s pe so al a d cultu al easo s o o Question 29.2.2 You recall that “culturally responsive care”
adhe e ce to t ad t o al med cal adv ce? is de ned as:
• Develop some tools o p ov d g ca e to pat e ts who have A) Learning about multiple cultures.
health bel e s that d ve ge om those o t ad t o al Weste B) Being able to speak multiple languages.
med c e? C) aking diversity classes.
CHAPTER 29 • PATiEn T-CEn TEr ED CAr E 727

D) Adopting a set o cultural behaviors and attitudes that enable


you to deliver e ective medical care to people o di erent Pondering this, you decide to try and provide culturally
cultures. responsive care to this Bosnian woman.
E) Hiring sta rom a variety o di erent cultures—pre erably
all good cooks who will provide some excellent ethnic cui- Question 29.2.4 Which one o the ollowing is NOT a step in
sine. providing culturally responsive care?
A) Understanding your own culture.
Answer 29.2.2 The correct answer is “D.” While the other B) Understanding others’ cultures.
answers are laudable goals, they do not de ne culturally respon- C) Being rustrated with having to work with people di erent
sive care. Culturally responsive care is a set o behaviors and than you.
attitudes that aims to help healthcare providers deliver better D) Understanding how your patients’ cultural belie s a ect
care to patients rom many di erent cultures. their attitude toward healthcare.
E) Adapting your way o practicing to provide optimal care.
You rack your brain trying to remember why culturally
responsive care is important to you. You now regret skip- Answer 29.2.4 The correct answer is “C.” T e goal o culturally
ping some o the lectures to go snorkeling in those beauti ul responsive care is to provide better healthcare or patients rom
Hawaiian waters. di erent cultures. Some o the speci c things this goal calls or
include:
Question 29.2.3 Which o the ollowing is NOT a bene t o • Being respect ul o potential cultural di erences
providing culturally responsive care?
• Learning about other cultures
A) It allows ef cient use o time and resources.
B) It increases the chance o providing services that are consis- • Being aware o the health impact o cultural belie s and prac-
tent with patient needs. tices
C) It might improve health outcomes or minority patients. • Being sensitive to patients’ needs
D) It might improve patient retention. • Using interpreters when necessary
E) It is less expensive in the long run. • Adapting practices to provide optimal care

Answer 29.2.3 The correct answer is “E.” Culturally respon- An easy way to achieve these goals is by using the Berlin and
sive care allows you to use your time and resources ef ciently Fowkes’ LEARN model described earlier in the chapter.
to increase the likelihood that you will provide the services
your patient actually wants, and needs! T is can lead to
improved health outcomes or your patients, increased patient HELPFUL TIP:
satis action and retention o your minority patients (and per- Wh le lea g a d espect g pat e ts’ d e e t cul
haps increase your patient satis action scores!). Un ortunately, tu al bel e s s v tal, p ov d g good ca e does ot call
no studies so ar have shown that it can reduce your practice o accept g p act ces that a e det me tal to you pa
costs. t e t’s health. But bewa e . . . the e a e ma y pote t al
eth cal d lemmas that ca occu whe t ad t o al West
e med c e te sects w th a cultu e that has ad cally
d e e t health bel e s. Ma y c oss cultu al med cal
HELPFUL TIP: dec s o s a e ot as black a d wh te as we would l ke
The n at o al Ce te o Cultu al Compete ce at Geo ge to th k. Weste med c e may have to be d a l ttle to
tow U ve s ty has de t ed s x compell g easo s accommodate bel e systems.
that healthca e p ov de s should co po ate cultu ally
espo s ve ca e to the p act ce. They a e as ollows:

1. To espo d to cu e t a d p ojected demog aph c


cha ges the U ted States Now that you are ready to proceed with caring or your
2. To el m ate lo g sta d g d spa t es the health patient, the question o language comes up. Should you
status o people o d ve se ac al, eth c, a d cultu al use the husband to interpret (he seems to know a bit more
backg ou ds English)?
3. To mp ove the qual ty o se v ces a d health out
comes Question 29.2.5 To help guide you, you call your hospital
4. To meet leg slat ve, egulato y, a d acc ed tat o compliance of cer who tells you that:
ma dates A) It is Federal Law that you must provide an interpreter or
5. To ga a compet t ve edge the ma ketplace patients who need it, at your own cost i necessary.
6. To dec ease the l kel hood o l ab l ty/malp act ce B) It is Federal Law that patients must provide their own inter-
cla ms preters at their own expense.
C) Most insurance companies reimburse or interpreter services.
728 FAMiLY MEDiCin E EXAMin ATiOn & BOAr D r EViEW

D) Using amily and sta to translate rarely re ects current Answer 29.2.7 The correct answer is “C.” T e physician
practice standard. should speak to and look at the patient; in other words, don’t
E) T ere are no privacy concerns when using nonpro essional speak about the patient in the third person. Remember that
interpreters. nonverbal communication is important even when common
languages are not shared. Be aware that seemingly universal
Answer 29.2.5 The correct answer is “A.” T e 2010 census gestures such as the “OK” sign and a thumbs up may have di -
ound that more than 60.6 million adults (around 21% o the U.S. erent meanings in other cultures. T e physician should speak
population) speak a non-English language at home, while clearly and give the interpreter time to translate questions and
almost 13% o the U.S. population is oreign born. When the answers. T e physician should periodically pause and ensure
healthcare pro essional does not speak the primary language that the patient understands the questions that are being asked.
o the patient, there is potential or loss o important in orma- Failure to look at the patient while asking questions impairs the
tion, misunderstanding o physician instructions, and poor establishment o the physician–patient relationship and should
shared decision-making. itle VI o the 1964 Civil Rights Act be avoided.
requires healthcare pro essionals to provide interpreter services
or patients who need them, at the physician’s cost i necessary. You remember rom medical school that there was disagree-
Failure to do so would quali y as discrimination and could be ment as to the degree that a medical interpreter should unc-
prosecuted in a court o law. Un ortunately, most insurance tion as a cultural advocate.
companies do not reimburse or these services. Interpreters can
be scarce and costly. As a result, many physicians use any help Question 29.2.8 Which o the ollowing would be the most
they can get or translation, including sta members and amily e ective pre -encounter instructions or your interpreter to
members who are bilingual; this leaves room or error (so, “D” acilitate the best possible communication between you
is a alse statement). Using a amily member as an interpreter and your patient?
makes it hard or the patient to disclose private in ormation A) “ ranslate word- or-word all that the patient and I say. You
they do not want known by the amily. Pro essional interpreters may repeat phrases, but do not rephrase anything.”
have been trained and certi ed, while ad hoc interpreters o en B) “When clari ying, explaining or culturally translating con-
have no ormal training and can make translation mistakes. cepts, make sure that you are transparent (i.e., let the both
parties know what you are saying and why).”
Question 29.2.6 Regarding provision o healthcare, lan- C) “Be sure to culturally translate whenever you think it is
guage barriers may result in: appropriate.”
A) Increased risk o intubation or children with asthma. D) “I the patient doesn’t seem to understand, go ahead and ex-
B) Greater nonadherence to medication regimens. plain whatever you think that I mean.”
C) Higher resource use in diagnostic testing.
D) Increased risk o drug complications.
E) All o the above. Answer 29.2.8 The correct answer is “B.” Ideally, an inter-
preter would do an exact translation. However, many con-
Answer 29.2.6 The correct answer is “E.” Language barriers cepts do not translate literally or have very di erent meanings
have been associated with worse health outcomes to include depending on the context that surrounds them. Good inter-
lower likelihood o having a usual source o medical care; lower preters are o en aware when the healthcare provider and the
likelihood ollow-up a er an emergency room visit; greater patient do not have the same understanding o an event, con-
nonadherence to medication regimens; increased risk o drug cept, or plan. In this case, the translator should be sure to let
complications; impaired understanding o diagnoses, medica- each party know exactly what has been communicated. T is
tions, and need or ollow-up; lower patient satis action; longer eedback to each party is important so that the interpreter’s
medical visits; higher resource utilization; increase risk o intu- moral values don’t get projected onto the patient and unduly
bation or children with asthma; greater risk o being labeled in uence decisions made by the patient. ransparency is thus
with psychopathology; and increased risk o leaving the hospital critical.
against medical advice.
You remember that low literacy is associated with poor out-
Question 29.2.7 What is the proper method to use an inter- comes.
preter?
A) Address all questions to the interpreter while acing the Question 29.2.9 Which o the ollowing is NOT true?
interpreter. A) Patients with low literacy have a 50% increased risk o hos-
B) Address questions to the patient while looking at the inter- pitalization.
preter. B) Only hal o all patients take medications as directed.
C) Address questions to the patient while acing the patient. C) Low literacy is a stronger predictor o a person’s health than
D) Address questions to the interpreter while acing the patient. race.
E) Have the interpreter get you co ee while you muddle D) Low literacy is only an issue among minorities and immi-
through using gestures. grants.
CHAPTER 29 • PATiEn T-CEn TEr ED CAr E 729

Answer 29.2.9 The correct answer is “D.” Poor health literacy them. My wi e made me come today.” You tell him you are
skills are a stronger predictor o health status than a person’s going to take a history and then per orm a physical exami-
race, age, income, socioeconomic status, or employment. T is nation on him. A er you are done, you tell him you recom-
relationship holds across di erent racial and cultural groups. mend age-appropriate screening examinations or colon and
Un ortunately, up to 90 million people in the United States have prostate cancers. T e patient politely declines both. You try
low literacy skills and many are ashamed to share this with their to explain the importance o screening to him.
physicians. T is can lead to “noncompliance” because patients
cannot read prescriptions and other instructions. It is not sur- Question 29.3.1 Which o the ollowing statements about
prising, then, that low-literacy patients have an increased risk screening is INCORRECT?
o hospitalization. o help combat this problem, the National A) Prostate cancer is a leading cause o death among Black
Patient Sa ety Foundation (NPSF) launched the “Ask Me 3” men.
program (http://www.nps .org/?page= askme3). “Ask Me 3” B) Colon cancer is a leading cause o death among Black men.
urges doctors to make sure their patients ask and understand C) T ere is clear evidence that screening or colon cancer
the answers to three simple questions: What is my main prob- reduces mortality in Black men.
lem? What do I need to do? Why is it important or me to do D) T ere is clear evidence that screening or prostate cancer
this? Other suggestions to improve communication include among Black men saves lives.
(1) asking the patient to repeat instructions back to the physi- E) Prostate cancer disproportionately a ects Black men.
cian (known as the “teach back”); (2) using basic, nonmedical
language when talking to patients; and (3) allowing patients to Answer 29.3.1 The correct answer is “D.” T ere is con ict-
talk uninterrupted at the beginning o the visit. ing evidence or screening or prostate cancer and its e ect on
mortality (see Chapter 16). Prostate cancer is the h leading
HELPFUL TIP: cause o cancer death overall in the United States, and the bur-
Betwee 10% a d 40% o low come ch ld e have o den o prostate cancer varies among di erent racial and ethnic
books at home. A qua te o college educated pa e ts groups. Black men have about a 60% higher incidence and a
do ot ead to the you g ch ld e da ly. r each Out a d 2.4- old higher mortality rate rom prostate cancer compared
r ead s a p og am des g ed to c ease ead g at home. with White men. Compared with White men, mortality rom
You ca employ th s p og am to help w th l te acy by prostate cancer is 17% lower in Latinos and 55% lower in Asian
us g the SAFEr m emo c: Show the book ea ly the Americans and Paci c Islanders.
v s t, Sha e the book w th the ch ld you sel (model g
the ead g o the pa e ts), Ask the pa e ts about ead Your patient now agrees to colon cancer screening but still
g; Assess the ch ld’s developme t a d the pa e t–ch ld declines prostate cancer screening (which is ne because that’s
elat o sh p; g ve Feedback about what you’ve obse ved what the USPS F recommends anyway!). Your examination
the ch ld do a d about pa e ts’att tudes a d te act o s also revealed an elevated blood pressure (BP) o 160/90 mm Hg
w th the ch ld, E cou age the pa e ts to ead da ly to the with no abnormal cardiac, lung, or other organ system ndings.
ch ld; Expla about l te acy developme t. Re e to the
l b a y a d l te acy p og ams, a d Reco d the cha t Question 29.3.2 Assuming you con rm this elevated BP
what you d d. Othe tha g v g a book a d b e ly ead reading at subsequent visits, which o the ollowing antihy-
g to the ch ld, you eed ot do all the act v t es at each pertensives would you recommend?
v s t. i you cl c s ot volved the p og am, you ca A) Hydrochlorothiazide.
get sta ted at www. eachouta d ead.o g. B) Ramipril.
C) Minoxidil.
Objectives: Did you learn to . . . D) Atenolol.
• Desc be ways wh ch you qu e about a pat e t’s
E) Losartan.
cultu ally elated health bel e s?
• Def e cultu ally espo s ve ca e a d u de sta d ts Answer 29.3.2 The correct answer is “A.” While thiazide
mpo ta ce? diuretics are a good choice or most patients with hypertension,
• L st some compl cat o s that ca occu as a esult o they are an especially good choice in Blacks, who tend to have
la guage ba e s? extremely salt-sensitive hypertension. Blacks are more likely
• Use te p ete se v ces e ect vely a d app op ately? than other patients with hypertension to have low renin lev-
els, which make them less likely to respond to ACE inhibitors.
• Desc be the state o health l te acy the U ted States a d
ts mpact o populat o health? β -Blockers also tend to be less e ective in Black patients. A rea-
sonable second-line therapy in this patient would be a calcium
channel blocker such as amlodipine.
CASE 29.3
You are seeing a 67-year-old Black male or the rst time in Your patient agrees to your medication choice, but he wants
your clinic. He claims to be healthy but has not seen a physi- to know i the research on which you based your choice
cian or over three decades. He says, “Doctors . . . I try to avoid included Black people. You can con dently say yes, knowing
730 FAMiLY MEDiCin E EXAMin ATiOn & BOAr D r EViEW

that the ALLHA trial speci cally addressed the issue o gen- physicians are rom underrepresented minority groups. T is is
eralizability by making sure that about a third o the 33,357 problematic. T e lack o minority physicians leads to a discrep-
subjects were Black. However, you know this has not always ancy in healthcare access. Data shows that minority physicians
been true in scienti c research. In act, you know that minor- are more likely to serve minority patients and are more likely
ities are very underrepresented in research data on which we to serve in urban, underserved areas (where there tends to be
base our treatment choices. greater concentration o underprivileged patients).

Question 29.3.3 Which o the ollowing is NOT a reason why HELPFUL TIP:
there are ew minorities in research studies? Pat e ts o all eth c backg ou ds co s ste tly ate
A) Past history o abuse leading to mistrust o the healthcare the elat o sh ps w th the phys c a bette whe
system. the phys c a s o the same eth c backg ou d.
B) Lack o representation o minorities in the medical pro es-
sion.
C) Discrimination. Question 29.3.5 Which o the ollowing is NOT true?
D) Overrepresentation o minorities in lower socioeconomic A) Patients living in a low socioeconomic neighborhood have
status. an increased incidence o coronary artery disease.
E) Minorities are more likely to volunteer or studies but are B) Patients living in low socioeconomic neighborhoods have
generally excluded. an increased incidence o asthma.
C) Patients o the same minority group with a low socioeco-
Answer 29.3.3 The correct answer is “E.” Minorities’ contact nomic status have the same incidence o disease as those
with the medical system has been raught with abuse in the with a high socioeconomic status.
past. Possibly the most egregious example o this is the uskegee D) Very low income Black children have a higher incidence o
Syphilis Study where Black men with syphilis were recruited or asthma than White children o the same socioeconomic sta-
a naturalistic study o the disease in the 1930s. Over 400 Black tus.
men with syphilis were recruited with 200 men without syphilis E) Patients rom disadvantaged neighborhoods have a higher
as controls. T ere was no in ormed consent, and they were told risk o developing most cancers and are less likely to receive
the lie that spinal taps (done or research) were a orm o treat- aggressive treatment once diagnosed.
ment. It was soon apparent that the death rate among those with
syphilis was about twice as high as it was among the controls. Answer 29.3.5 The correct answer is “C.” Patients living in low
When penicillin was ound to be e ective as a cure or syphilis socioeconomic neighborhoods have increased incidences o cor-
in the 1940s, the participants were neither in ormed nor o ered onary artery disease, most cancers (lung, colon, cervical), dia-
treatment, so the naturalistic study could continue. betes, arthritis, accidents, adverse birth outcomes, and asthma.
T ese and other examples o past mistreatment by the medi- T ey are also likely to obtain overall less care, less aggressive care
cal community may constitute a signi cant source o distrust by (except or limb amputations in diabetics), and are less likely to
minority patients, who are less likely to participate in research. have evidence-based care. Minority patients with higher socio-
T ere have also been overt and subtle discriminatory barriers economic statuses tend to have better health when compared
against minority participation in clinical trials. Minorities are to their race-matched peers rom lower socioeconomic statuses.
more likely to come rom lower socioeconomic groups and
have less education, while research subjects are generally more Question 29.3.6 In taking care o Black patients, all o these
educated and o higher socioeconomic status. Finally, there is cultural patterns (not stereotypes) are help ul to keep in
a signi cant underrepresentation o minorities in the medical mind EXCEPT:
pro ession. For example, even though Blacks comprise 13% o A) Family relationships are extremely important.
the population, they are only 3.5% o the physician population. B) Religion o en has a role in the patient’s li e.
All these problems lead to dif culties recruiting minority par- C) It is expected that physicians will call patients by their rst
ticipants or research trials. name.
D) Food has important cultural signi cance.
Question 29.3.4 As o 2010, what percentage o Americans E) Nonverbal communication is o en as important as what is
belong to a minority group? verbally communicated.
A) 15%
B) 20% Answer 29.3.6 The correct answer is “C.” People in the Black
C) 25% community o en maintain extended amily ties and view
D) 36% healthcare as a amily responsibility. T ere ore, physicians
E) 62% should consider enlisting the amily’s help in taking care o an
ill amily member.
Answer 29.3.4 The correct answer is “D.” According to the Religion is o en an important aspect in Black culture and
2010 US Census estimates, 36% o Americans are ethnic minor- members o the clergy are highly respected in the community.
ities, up rom 31% in 2000 and 24% in 1990. Yet, only 10% o Churches are o en very help ul or community outreach e orts.
CHAPTER 29 • PATiEn T-CEn TEr ED CAr E 731

Evidence exists that the use o churches to conduct preventive D) Socioeconomic status.
care services, such as immunizations and screening programs E) obacco use.
or illnesses, leads to better patient compliance with preventive
guidelines. Some patients may view illness as a test o one’s aith, Answer 29.3.7 The correct answer is “D.” All o the other
and it is prudent or the physician to acknowledge and respect choices do have an impact on overall health. However, socio-
the patient’s belie s and perception o illness to the extent that economic status has the highest correlation with a patient’s
it in uences their seeking or receiving healthcare. People rom overall health. It’s like U2 says in their song God Part II, “T e
lower socioeconomic backgrounds have little choice but to eat rich stay healthy; the sick stay poor.” Socioeconomic status is a
what is available at a low cost. T is means our advice to patients key driver o health status.
to eat a well-balanced diet with resh ruit, lean meat, and resh
vegetables may be met with some challenges due to access to HELPFUL TIP:
these resources. Advising simple changes in diet including sub- F esh p oduce s o te ot ava lable o qu te expe s ve
stituting sh or chicken or red meat in dishes, eating inexpen- low soc oeco om c e ghbo hoods (a d good luck
sive raw vegetables, modi ying cooking techniques, and chang- d g susta ably caught Coppe r ve salmo ). P e
ing to a vegetable-based rather than a meat-based diet may be dom a tly Wh te e ghbo hoods have ou t mes as
more likely to be success ul. ma y supe ma kets as p edom a tly Black a d Lat o
Communication is important, with individuals in the Black e ghbo hoods. Th s makes d eta y mod cat o espe
community being particularly attentive to nonverbal aspects c ally d cult.
o communication such as body language and voice in ection.
Respect is also emphasized in the culture, and patients o en like Objectives: Did you learn to . . .
to be addressed by their ormal titles and not their rst names. • ide t y the mpact o ace o p eve t o a d ma ageme t
Asking patients’ permission to call them by their rst names can o ch o c d seases such as colo ca ce sc ee g a d
be appreciated. hype te s o ?
In ormation using real-li e examples can be a use ul commu- • Desc be ba e s to m o ty pa t c pat o esea ch stud
nication tool, instead o cold, dry data, or written messages (this es?
actually holds true or most patients o a variety o cultures!). A • Expla the mpact o soc oeco om c status o a pat e t’s
use ul summary o general principles to keep in mind when car- healthca e?
ing or Black patients is ound in an article on Preventive Care • r elate cultu al patte s to the p ov s o o healthca e educa
or A rican Americans by Witt et al. (2002). T ey are as ollows: t o a d se v ces?

• Gaining trust and understanding the historical distrust o CASE 29.4


the healthcare system.
• Understanding and employing the kinship web in decisions Your next two patients are a couple o Cuban riends who
present to your o ce seeking a amily physician to take care
regarding screening and treatment.
o their general health needs. T ey are both o Cuban descent
• Involving the church in developing and delivering preven-
but one looks White and the other looks Black.
tion and care messages.
• Asking patients about the meaning o words or phrases. Question 29.4.1 Which o the ollowing assumptions is
• Asking patients about the use o alternative medicines and correct?
herbs. A) Latino people are o one race.
• ailoring messages about prevention to depictions o real-li e B) Latino people can all speak English.
situations. C) Latino people come rom many di erent countries and can
look di erent rom one another.
• Paying attention to body language and other nonverbal com-
D) Latino people are not American.
munication.
E) Latino people all like to be called Hispanic.

You have learned in medical school about the importance o Answer 29.4.1 The correct answer is “C.” T e term “Latino”
behavior change in the management o chronic diseases like denotes an ethnic group who shares some cultural practices but
hypertension, heart disease, and diabetes, yet are rustrated that not necessarily other characteristics. Latinos comprise a sig-
you have less success in patients who come rom low socioeco- ni cantly diverse group who may or may not speak Spanish or
nomic backgrounds or rom minority racial or ethnic groups. English, may be o any race, hail rom di erent countries o ori-
gin, have di ering histories, socioeconomic status, and cultural
Question 29.3.7 Which o the ollowing behaviors or attri- identities. Further, some eel the term “Hispanic” is derogatory,
butes is most strongly associated with your patient’s health re ecting their European ancestry, and may pre er the term
status: Latino. T irteen percent o the overall U.S. population now
A) Physical activity level. identi es as “Hispanic or Latino,” which makes this population
B) Health insurance coverage. the largest ethnic group in the United States. Latinos come rom
C) Dietary choices. a broad spectrum o socioeconomic backgrounds and a variety
732 FAMiLY MEDiCin E EXAMin ATiOn & BOAr D r EViEW

o living conditions in the United States, which has an enor- Question 29.4.3 Which o the ollowing is true about race?
mous impact on immigrant and public health. A) It is a valid biological construct.
Despite the diversity represented within the Latino culture, B) It is interchangeable with ethnicity.
there exist shared values such as strong amily ties, with amilies C) All members o a certain culture are the same race.
o en tending to be large and extended. Families requently serve D) It is a purely social construct.
as the main source o support and share in decision-making.
Physicians with a warm bedside manner who demonstrate Answer 29.4.3 The correct answer is “D.” Race is a much
appropriate respect (especially to the elderly) are appreciated. politicized and emotionally charged topic in the history o
the U.S. Since the late 18th and early 19th centuries, attempts
HELPFUL TIP: have been made to validate race as being biologically based to
Desp te ala m st pa a o a om some pa ts o ou po justi y discriminatory practices. However, the Human
l t cal spect um, seco d ge e at o Lat o mm g a ts Genome Project demonstrated conclusively that there is NO
a e as l kely to speak E gl sh as we e p o waves o m biological basis or race. Humans share over 99.9% o their
m g a ts om Eu opea cou t es. Do ’t eve get us DNA, and one cannot tell a member o one race rom another
sta ted o the absu d dea o bu ld g a wall. on the basis o genetics. “B” is incorrect. While race and eth-
nicity are o en used interchangeably, they are not necessarily
Question 29.4.2 Which o the ollowing is FALSE regarding equivalent. Race is an arbitrary social construct that is given
health issues a ecting the Latino community? to people based on visual appearance, while culture re ers to
A) In ectious diseases are common. people with a common country o origin, a shared ancestry,
B) Fear o deportation prevents some rom seeking healthcare. or a common history. Culture re ers to a speci c set o values,
C) Latinos are less likely to obtain preventative healthcare in belie s, and customs shared by members o a community. “C”
comparison to Whites or Blacks. is obviously incorrect. People rom the same culture can be o
D) Moving to the United States can paradoxically raise the risk di erent races so one can have White and Black Latinos rom
or habits that lead to illness such as obesity and diabetes. Cuba, or example.
E) Elderly Latinos have a higher mortality than their White
counterparts. Question 29.4.4 Which o the ollowing health outcome dis-
Answer 29.4.2 The correct, and alse, answer is “E.” Recent parities is FALSE?
immigrants are o en prone to in ectious diseases due to inad- A) White Latinos have a higher incidence o breast and colorec-
equate housing, sanitation, and/or immunizations. Many rea- tal cancer than Blacks and Non-Hispanic Whites.
sons exist why patients may not get vaccinated. T ese include B) Clinicians may order ewer diagnostic tests i they do not
disbelie in the need or the vaccine, being unaware o the vac- understand a patient’s description o symptoms.
cine, lack o patient education by a healthcare provider, lack o C) Clinicians may overcompensate by ordering more tests
transportation, and nancial restrictions. when they do not understand what their patients are saying.
Immigrating to the U.S. can result in worse nutrition, obe- D) Minority children are more likely to be evaluated and
sity, a sedentary li estyle, and an increase in smoking and risky reported or suspected abuse even a er controlling or like-
sexual behavior in Latinas (women). T is translates, in part, lihood o abusive injury.
into a two old increased incidence o diabetes in Latinas. For E) Blacks have the highest colorectal cancer mortality rates o
Latino males, there is an increased risk o drug abuse, alcohol any group in the United States.
abuse, tobacco use, and driving under the in uence. T ese risks
increase the longer the patient lives in the U.S. Answer 29.4.4 The correct answer is “A.” T e incidence o
Latinos o en ail to get preventive healthcare. For example, colorectal and breast cancer is actually lower in White Latinos
they have a lower rate o screening or diabetes and hyper- than in other Non-Hispanic Whites and Blacks. However, this
tension than Blacks or Whites. Some avoid presenting or does not always translate into less signi cant disease. T ere are
healthcare because they ear being discovered by immigration signi cant disparities in healthcare quality and outcomes or
authorities. Complementary and alternative practices are also minorities compared with nonminorities, even when control-
common. Even though they are less likely to access healthcare, ling or possible con ounding actors, such as income, edu-
older Latinos seem to have a similar or greater li e expectancy cation, and insurance. So even though Latinos have a lower
than same-age Whites in the U.S. and enjoy lower mortality rates incidence o the above-named cancers, they have a similar mor-
rom cardiovascular diseases, cancer, and chronic illnesses. T e tality when compared with non-Latinos. All o the rest o the
risk o diabetes, however, is higher. T is phenomenon has been statements are correct. O particular note, Blacks have the high-
described as “selective immigration” and implies a predilection est colorectal cancer mortality rates. Also, o note is the act that
or healthier individuals to immigrate to this country. when toddlers o di erent races present with similar ractures,
minority toddlers are signi cantly more likely to be reported
You get into a discussion on race with your new Cuban or suspected abuse, even a er controlling or age, insurance
patients, and they point out several things to you that you status, and likelihood o abuse. T is is a re ection o biases and
had not thought o be ore. stereotyping.
CHAPTER 29 • PATiEn T-CEn TEr ED CAr E 733

HELPFUL TIP: HELPFUL TIP:


M o ty pat e ts a e s g ca tly less l kely to ece ve Although pe haps ot ove tly ac st, the e s data that
app op ate pa med cat o , eve a te su ge y a d suggests that u co sc ous b ases ca a ect ou ca e o
lo g bo e actu es. Add t o ally, the e a e o te ew, pat e ts. A well k ow study the n EJM 1999 ou d
a y, pha mac es poo , e c ty a eas that have that, all th gs be g equal, phys c a s we e 60% less
a cot cs ava lable o pat e ts w th a p esc pt o . Th s likely to e e Black wome o ca d ac cathete zat o
adve sely a ects the pa ma ageme t o m o ty pa tha me a d Wh tes. Hope ully, by be g made awa e
t e ts eve u the . o th s u co sc ous b as, ts e ect ca be m t gated a d
bette ca e p ov ded to all pat e ts.

In general, mortality rates among racial and ethnic minori- Objectives: Did you learn to . . .
ties are higher or cancer, heart disease, diabetes, stroke, kid- • Desc be some health d spa t es based o ace, cultu e, a d
ney ailure, and HIV/AIDS. Minority groups are also dispro- eth c ty?
portionately a ected by asthma, lead poisoning, accidents, • Del eate the d e e ces betwee the te ms ace, cultu e,
homicides, and other environmental health concerns. Some a d eth c ty?
minorities experience higher in ant mortality rates and are
less likely to receive timely prenatal care. T ere are numerous
other examples o healthcare disparities that exist between CASE 29.5
majority and minority groups. T ese include but are not lim- A 35-year-old comes to your o ce today to establish care with
ited to: you. You see the rst name o “Jenni er” is on the medical
• In ant death rates or Blacks are twice that o Whites. record. T e patient is wearing a suit and tie, and has a male
style haircut. T e gender listed on the medical record or this
• Heart disease mortality rates are 40% higher in Blacks as
patient is emale. Clearly the gender identity o this patient
compared with Whites.
does not match the patient’s gender designated at birth.
• Latinos are almost twice as likely as Whites to die rom dia-
betes and are more likely to be obese and have high blood Question 29.5.1 How do you establish rapport with this
pressure than are Non-Hispanic Whites. patient and use correct terminology?
• Blacks are 13% less likely to undergo coronary angioplasty A) Ask the patient rst thing, “Are you a man or a woman?”
and one-third less likely to undergo bypass surgery than are B) Use the name Jenni er as well as emale pronouns she/her/
Whites. hers
• Only 7% o Black and 2% o Latino preschool children hos- C) Introduce yoursel and your pre erred pronouns, and ask the
pitalized or asthma are prescribed routine medications patient or their name and pre erred pronouns
to prevent uture hospitalizations compared with 21% o D) Say that you are glad they are here and that you have never
White children (not in itsel a good re ection on our pro- had a patient “like them” be ore
ession). E) Ask i he is rom Iowa as you have heard Iowa is actually a
• T e length o time between an abnormal screening mammo- pretty progressive state
gram and ollow-up diagnostic testing is more than twice as
long or Asian American, Latinos, and Blacks compared with Answer 29.5.1 The correct answer is “C.” Biological gender,
Whites. gender identity, and gender expression are all di erent. Bio-
• Minorities are less likely to get immunizations, mammo- logical gender is de ned by one’s chromosomes, hormones,
grams, and other preventive care, even when paid or by and reproductive organs. T e assignment o gender is gener-
Medicare. ally done at birth based on the appearance o external genita-
lia. Some newborns might be born with ambiguous genitalia
• T ere are higher rates o un-insured and lack o physicians in
though will ultimately be assigned a sex at birth o either male
minority communities, leading to reduced access to primary
or emale. Gender identity is a person’s innate, deeply elt psy-
care.
chological identi cation which may or may not correspond to
• Minorities are less likely to get heart catheterizations, by- the sex assigned to them at birth. Gender expressions are the
pass gra ing, dialysis, lung cancer surgery, and organ trans- external characteristics and behaviors that are socially de ned
plants. such as dress, grooming, mannerisms, and speech patterns.
• Blacks have 55% higher mortality rate and 6-year shorter li e T ese can widely vary and are accepted as masculine, eminine,
expectancy than Whites. or neutral. ransgender is an umbrella term or people whose
• Several studies show the deleterious e ect o discrimination gender identity is incongruent rom the gender assigned to
on health outcomes, including increasing the risk o diabetes, them at birth. It is important to maintain respect or patients
hypertension, depression, and preterm birth independent o who identi y as transgender in order to establish rapport and
other risk actors. better care or the patient. Using pre erred names and pronouns
734 FAMiLY MEDiCin E EXAMin ATiOn & BOAr D r EViEW

is extremely important in helping to establish respect. A good • Obta a app op ate sexual h sto y w thout mak g
way to introduce yoursel to a patient would be: “My name is assumpt o s ega d g the pat e t’s ge de a d sexual
Dr. X, my pre erred pronouns are she/her/hers or he/him/his, de t t es?
what is your name and pre erred pronouns?”
CASE 29.6
HELPFUL TIP: You are on the board o your hospital’s quality and sa ety
T a s t o g s the p ocess that some t a sge de committee and you have been asked to address the issue o
people go th ough to beg l v g the a med ge de a medication error. Ms. Jones, a 72-year-old patient with
de t ty. Th s may o may ot clude cha g g phys cal diabetes who was hospitalized or an acute myocardial
appea a ce a d d ess, t at g ho mo e the apy, a d in arction, died rom what seems to be a sudden cardiac
u de go g ge de co mat o p ocedu es a d/o event. T is was unexpected as she was recovering rather
su ge y. T eatme t o t a sge de pe so s, clud g well. One o the nurses on the f oor had told her supervisor
ho mo e the apy a d su ge y, has bee deemed med that your partner, had written an order or 6.0 U o insulin;
cally ecessa y to t eat ge de dyspho a. As w th eve y but the new nurse, resh out o nursing school, had misread
commu ty, the e s a g eat deal o d ve s ty w th the this as 60 U o insulin and had proceeded to give the poor
t a sge de commu ty a d each t a sge de pe so ’s lady that amount, precipitating the hypoglycemic event that
t a s t o s u que a d may o may ot clude eve y led to her death. T e nurse is distressed as she thought she
th g l sted. was only ollowing orders. Around your o ce, Your partner’s
handwriting is a thing o legend: many pharmacists have
had to call to double-check his prescriptions on more than
one occasion.
Jenni er indicates she actually pre ers to go by emale pro-
nouns as she is o the emale biological sex. However, she also
Question 29.6.1 Which o the ollowing would have been
likes to identi y and express hersel as a male gender at times.
an e ective way to prevent the type o error that occurred?
She returns to your o ce or an annual physical examina-
A) T e nurse should have called to con rm the dose o insulin
tion. She discloses on her intake orm that she is in a long-
be ore giving it.
term, monogamous relationship, but the sex o the partner is
B) Writing out “units” rather than “U” and “6” instead o “6.0.”
not asked on your intake orm (which needs updating!) and
C) Use o computerized physician order entry.
thus isn’t speci ed or you. She has regular periods and is not
D) Having a clinical pharmacist as part o the team.
currently on birth control.
E) All o the above.
Question 29.5.2 Which o the ollowing questions should
Answer 29.6.1 The correct answer is “E.” It is estimated that
you ask the patient:
between 44,000 and 98,000 deaths annually are attributable
A) “Are you using condoms with sexual activity?”
to medical errors, most o which are preventable. Although
B) “What does your boy riend/husband do or a living?”
the numbers have been challenged, it is indisputable that
C) “Would you like a pregnancy test today?”
medical errors continue to contribute to adverse outcomes
D) “Are you sexually active with men, women, or both?”
in our patients. One o the large contributors to this is error
E) You don’t address the subject as you assume she is sexually
in dosing and administering medication. One o the leading
active and already using protection to prevent pregnancy as
causes o medication errors is the use o potentially dangerous
she is in a stable relationship
abbreviations and dose designations. T e use o “U” as above
has been reported in the literature to be very problematic as
Answer 29.5.2 The correct answer is “D.” It can be very unin-
it can easily be misread as a zero or a our, leading to up to
viting to patients when assumptions are made about having
10- old overdoses that can o en have terrible consequences
an opposite sex partner or made at all! Knowing the patient’s
in patients using insulin. A decimal point may not be seen,
sexual identity (gay/lesbian, bisexual, heterosexual) as well as
leading to a actor 10 error (60 units o insulin being given
their past, current, and anticipated uture sexual partner(s) and
rather than 6).
behavior are essential to providing comprehensive preventative
screening and guidance. It is important to not assume (we can
Question 29.6.2 Which o these would NOT be recom-
all recall the old adage about what assuming does or an individ-
mended as a way to prevent prescription errors (we will
ual…) the sexual identity or behavior o the patient but rather
give you this one)?
ask a complete sexual health history prior to o ering treatment
A) Computerized order entry systems.
or care to the patient.
B) A brie notation o the purpose or the prescription.
Objectives: Did you learn to . . . C) Metric system or all therapies except those like insulin and
• Desc be the d e e ces betwee b olog cal sex, ge de vitamins that use standard units.
de t ty, a d ge de exp ess o ? D) Inclusion o age, and when appropriate, weight o the patient.
• Establ sh appo t by el c t g p e e ed p o ou s? E) A trailing zero a er a decimal.
CHAPTER 29 • PATiEn T-CEn TEr ED CAr E 735

Answer 29.6.2 The correct answer is “E.” Nurses and pharma-


T a sm ss o o p otected o mat o s allowable o o go
cists may not see the decimal point and thus give an order o g pat e t ca e.
magnitude higher dose o medication. All o the other options
Do ot make assumpt o s about sexual ty o ge de . S mply
are recommended to help decrease medication errors and, thus,
ask.
adverse patient events.
Eve yo e possesses u co sc ous b ases. U co sc ous
Objectives: Did you learn to . . . b ases a ect the ca e o pat e ts. S mply ecog z g these
• r ecog ze some causes o med cal e o s a d ways to p e b ases may m t gate the det me tal e ects o pat e t
ve t them? ca e.
Ge de de t ty a d ge de exp ess o a e a ected by but
sepa ate om sex ass g ed at b th. E qu e o t a sge de
QUICK QUIZ: HiPAA, HiPAA, HOOr AY! pat e ts the p e e ed p o ou s.
i add ess g o adhe e ce, co s de a pat e t’s cultu al
Which o these is prohibited by Health Insurance Portability
backg ou d a d e qu e to how the cultu al bel e s may
and Accountability Act (HIPAA)? co l ct w th those o t ad t o al Weste med c e.
A) Calling out patients’ names in the waiting room.
B) Leaving a message on a patient’s answering machine or
voicemail.
C) Releasing health in ormation to a specialist without patient BIBLIOGRAPHY
authorization i that in ormation is to be used or purposes
American Medical Association. Cultural Competence Compen-
o treatment, payment, or health operations. dium: Section IV: Specif c Populations Needs and Resources.
D) Faxing patient in ormation. Chicago, IL: American Medical Association Press; 1999:
E) None o the above. pp. 153–178. Available at http://ww4.mgh.org/Physicians/
Shared%20Documents/physicianCulturalCompendium.
The correct answer is “E.” Since the HIPAA went into e ect, pd . Accessed September 24, 2015.
many physicians have been con used as to what they can and Diaz VA Jr. Cultural actors in preventive care: Latinos. Prim
cannot do under the HIPAA law. Providers can still call out Care. 2002;29(3):503–517.
patients’ names in the waiting room as long as you do not go Human Rights Campaign. Sexual Orientation and Gender
into particulars o their presenting complaint in ront o other Identity De nitions. Available at http://www.hrc.org/re-
people (e.g., don’t say to the whole waiting room, “Ms. Smith, sources/entry/sexual-orientation-and-gender-identity-
I’m ready to check on your herpes in ection”). You may leave terminology-and-de nitions. Accessed September 24,
a message on a patient’s voicemail i the patient has autho- 2015.
rized you to do so. You may ax in ormation once you veri y Institute o Medicine Committee on Understanding and
the ax number is correct—but you can only ax to persons the Eliminating Racial and Ethnic Disparities in Healthcare.
patient designates or consents to. I you ax health in ormation Smedley BD, Stith AY, Nelson AR, eds. Unequal Treat-
to a wrong number, you are responsible and could be violating ment: Con ronting Racial and Ethnic Disparities in Health
Care. Washington, DC: National Academies Press; 2003.
HIPAA. Finally, HIPAA gives physicians broad leeway in shar-
Available at http://books.nap.edu/books/030908265X/
ing in ormation with other physicians, provided the in orma- html/. Accessed September 24, 2015.
tion is used or treatment, payment, and healthcare operations.
National Center or Cultural Competence. Available at http://
You must give a notice o your privacy practices to the patients nccc.georgetown.edu. Accessed September 24, 2015.
and ask them to sign a orm acknowledging receiving the notice
National Coordinating Council on Medication Error Report-
or make a good- aith e ort to get them to sign it. ing and Prevention. Recommendations to enhance ac-
curacy o prescription/medication order writing. Revised
Clinical Pearls October 22, 2014. Available at http://www.nccmerp.org/
recommendations-enhance-accuracy-prescription-
Always o e te p ete se v ces to a pat e t who does ot writing. Accessed September 24, 2015.
speak E gl sh p ma ly. Whe us g a te p ete , ma ta Witt D, Brawer R, Plumb J. Cultural actors in preventive care:
eye co tact w th a d add ess quest o s to the pat e t. A rican-Americans. Prim Care. 2002;29(3):487–493.
Do ot ove est mate the legal autho ty o the Health Yarnall KS, Pollak KI, Østbye , et al. Primary care: is
i su a ce Po tab l ty a d Accou tab l ty Act (HiPAA). there enough time or prevention? Am J Public Health.
2003;93(4):635–641.
Final Examination
Jas K. W bur, Mark A. Graber, a d Br g E. Ray
30
T is is your nal examination. See how you do. . . . 5) You admit a patient to the hospital a er an episode o
chest pain. T e patient has a history o chronic kidney disease
1) Which o the ollowing IS NO an absolute contraindica- stage 4, type 2 diabetes mellitus, hypertension, hyperlipid-
tion to cardiac exercise testing? emia, and obesity. At time o admission, troponin is elevated
A) Unstable angina. at 0.06 (normal < 0.03). T e medical student rotating with
B) Recent pulmonary embolism. you on her amily medicine clerkship asks the signi cance
C) Active pericarditis. o the elevated troponin. You tell her which o the ollowing?
D) Le bundle branch block. A) An elevated troponin is always indicative o cardiac
ischemia/in arct.
See question 2.2.2. B) roponin rises be ore creatine phosphokinase (CPK) and
stays elevated or a longer period o time.
2) Which o the ollowing is the pre erred antiplatelet drug C) Pulmonary embolism and renal ailure are two causes o an
or treating chest pain in the emergency department? elevated troponin.
A) Aspirin 81 mg PO. D) CPK is overall the most sensitive (but not speci c) cardiac
B) Aspirin 325 mg PO. marker or cardiac ischemia.
C) Clopidogrel 75 mg PO.
D) Clopidogrel 300 mg PO. See question 2.1.2 and Helpful ip.
E) 2B/IIIA glycoprotein inhibitor.
6) Which o the ollowing is true in a patient with known
See question 2.1.4. coronary artery disease?
A) A coronary calcium score will help to risk strati y this patient.
3) T is ECG, shown in Figure 30-1, represents a(n): B) T e LDL goal is 70 mg/dL or less.
A) In erior wall MI. C) A high sensitivity CRP (hsCRP) will help to risk strati y this
B) Anterior wall MI. patient.
C) In erolateral MI. D) T ere is no LDL goal. Just start a maximal dose o a statin at
D) Normal ECG. time zero; there is no need to titrate the statin up.
E) Pericarditis.
See questions 2.22.4 and 2.22.7.
See question 2.17.3.

4) T e most appropriate initial treatment o this rhythm in a 7) Which o the ollowing IS NO a contraindication to the
stable patient, shown in Figure 30-2, is: use o tPA in MI?
A) Amiodarone. A) Blood pressure o > 180/110 mm Hg.
B) Adenosine. B) Noncompressible vascular puncture (e.g., subclavian line).
C) Diltiazem. C) Major surgery within 3 weeks.
D) Lidocaine. D) Menstrual bleeding.

See question 2.3.5. See question 2.1.10 and able 2-1B.

736
F
I G
U R
E 3 0 -
1 .
737
738
F
I G
U R
E 3 0 -
2 .
CHAPTER 30 • Fin Al ExAMin At io n 739

8) Which o the ollowing IS NO necessary or a class 1 indi- 13) Human papilloma virus (HPV) vaccine is indicated or
cation or tPA in a patient with an acute myocardial in arc- all o the ollowing EXCEP :
tion? A) All otherwise healthy 11- to 12-year-olds (boys and girls).
A) Greater than 1 mm S elevation in 2 or more contiguous B) A 39-year-old emale who has never been sexually active.
leads. C) An 18-year-old male with known HPV.
B) Pain or less than 2 hours. D) Immunosuppressed 15-year-old emale who is HIV positive.
C) Age < 75. E) A 22-year-old emale with cervical dysplasia.
D) New complete bundle branch block and typical history sug-
gestive o MI. See question 14.5.3.

See question 2.1.10 and able 2-1A. 14) opical tretinoin (Retin-A) is most e ective in what kind
o acne?
A) Cystic/nodular.
9) Your patient has an elevation o his liver unction tests
B) In ammatory papular.
(LF s) a er starting an HMG-CoA reductase inhibitor (i.e.,
C) Pustular.
statin). When would you consider stopping this patient’s
D) Comedonal.
HMG-CoA reductase inhibitor?
A) Doubling o LF s.
See question 17.8.3: Helpful ip.
B) ripling o LF s.
C) Quadrupling o LF s. 15) In a study o drug A, 50%o patients in the treatment arm
D) Only when there is biopsy evidence o bridging brosis. bene t versus 25%in the placebo group. What is the number
needed to treat?
See question 2.22.8. A) 2.
B) 4.
10) Which medication(s) is (are) indicated in a patient with C) 6.
atrial brillation and a CHA2DS2-VASC score o 2? D) 8.
A) Aspirin.
B) War arin, rivaroxaban, dabigatran, or apixaban. See question 28.3.1.
C) Aspirin-dipyridamole (Aggrenox).
D) Clopidogrel. 16) You are seeing a 27-year-old male in your clinic or evalu-
E) Heparin IV. ation o a rash which began about 2 days ago and is erythema-
tous and macular with a pale center. He notes the rash itches.
See question 2.4.1 and able 2-6B. He denies any new lotions or creams. He has had recent cold-
like symptoms including a sore throat. He has a past history
only notable or chicken pox as a child. He is sexually active
11) Which o the ollowing DOES NO improve cardiovas- with several emale partners. You diagnose him with erythema
cular mortality in systolic heart ailure (heart ailure with multi orme. He asks what could have trigged this. You counsel
reduced ejection raction or HFrEF)? him that the most common cause o erythema multi orme is:
A) Digoxin. A) Herpes zoster.
B) Spironolactone. B) Streptococcal pharyngitis.
C) ACE inhibitors. C) Genital herpes.
D) Beta-blockers. D) Rhus exposure (“poison ivy”).
See question 2.7.7. See Chapter 17 Quick Quiz: Erythema Multiforme.

12) Which statement is true regarding gastric lavage in a 17) “Sensitivity” is best de ned as:
patient with a toxic ingestion? A) rue positive rate/(true positive rate + alse-negative rate).
A) Gastric lavage is o bene t even i it has been more than B) rue positive rate/(prevalence o disease).
1.5 hours, but less than 4 hours, since the ingestion. C) rue positive rate/(true positive rate + true negative rate).
B) Gastric lavage should be done until the returned material is D) rue positive rate/(true positive rate + alse-positive rate).
clear (maximum o 10 L).
C) Gastric lavage is associated with esophageal injury and aspi- See Case 28.6.
ration.
D) Gastric lavage is indicated or petroleum distillates within 18) Which o the ollowing is true regarding isotretinoin
1 hour o ingestion. (Accutane)?
A) Women should be on one orm o birth control be ore using
See question 1.1.2. this drug.
740 FAMil Y MEDiCin E ExAMin At io n & Bo ARD REViEW

B) Pregnancy should be avoided or 3 months a er discontinu- 24) What is the appropriate course o action a er treating a
ation o this drug. child with croup with inhaled epinephrine?
C) Monthly pregnancy tests should be done on all women o A) Admission or observation.
reproductive age. B) Admission i room air oxygen saturation is less than or equal
D) Isotretinoin may increase the HDL. to 95%.
C) Observation or 12 hours ollowed by discharge i stable.
See question 17.8.4. D) Observation or 2 hours ollowed by discharge i stable.

See question 1.24.5.


19) In general, what will happen with an innocent ow
murmur during a Valsalva maneuver? 25) Which o the ollowing is NO included in the “drug
A) It gets louder. cocktail” or the unconscious patient?
B) It gets so er. A) Glucose.
C) It is unchanged in volume. B) T iamine.
D) T e sound becomes more harsh. C) Naloxone (Narcan).
E) It disappears entirely. D) Flumazenil (Romazicon).

See question 14.3.1. See question 1.20.1.

20) What blood level o morphine is the most appropriate 26) Which o the ollowing is indicated in the child with
when treating pain rom a terminal cause? croup (laryngotracheobronchitis) presenting to the emer-
A) 1 µg/dL. gency department?
B) 5 µg/dL. A) Amoxicillin.
C) 10 µg/dL. B) Azithromycin.
D) Blood levels are irrelevant in determining the appropriate C) Dexamethasone.
dose o morphine. D) Oral theophylline.

See question 27.2.4: Helpful ip. See question 1.24.4.

21) Which o the ollowing is NO seen with an anticholin- 27) You are evaluating a 22-year-old woman in the ED who
ergic overdose? presents with le -sided pelvic pain. She is sexually active
A) Dry, ushed skin. with men and does a poor job o remembering her oral con-
B) Miosis. traceptive; condom use is only sporadic. Her last menses was
C) Con usion. 6 weeks prior. You are concerned about a possible ectopic
D) Low-grade ever. pregnancy. Which o the ollowing is true?
A) A negative pregnancy test e ectively rules out ectopic preg-
See Chapter 1 Quick Quiz: oxidromes and able 1-1. nancy.
B) Absence o an adnexal mass e ectively rules out ectopic
pregnancy.
22) Which o the ollowing IS NO a cause o an elevated C) Every woman o reproductive age with a uterus in the ED
anion gap acidosis? with abdominal pain is pregnant until proven otherwise.
A) Methanol and other ingestions (e.g., salicylate). D) T e hCG should double every 5 days early in a normal preg-
B) Diabetic ketoacidosis. nancy. Any less than this is indicative o an ectopic preg-
C) Uremia. nancy.
D) GI bicarbonate loss.
See questions 1.19.1 and 1.19.4.
See question 1.4.3 and able 1-3.
28) Which o the ollowing is the best drug indicated or dys-
23) Reper usion o extremities in hypothermia can cause all pnea in the terminally ill patient?
o the ollowing EXCEP : A) Buccal scopolamine.
A) Acidosis. B) Lorazepam or other benzodiazepine.
B) Hypokalemia. C) Morphine or other opiate.
C) Paradoxical central temperature drop. D) Nebulized lidocaine.
D) Arrhythmia. E) Albuterol via inhaler.

See question 1.12.3. See question 27.1.6.


CHAPTER 30 • Fin Al ExAMin At io n 741

29) What is the one best drug to reduce headache and con u- 34) Which o the ollowing is true regarding Barrett esopha-
sion secondary to CNS tumor with surrounding edema? gus?
A) Acetaminophen. A) All patients with GERD should have an endoscopy to stage
B) Dexamethasone. their disease vis-à-vis Barrett esophagus.
C) Morphine. B) Barrett esophagus is a change rom the normal columnar
D) Sumatriptan. epithelium to squamous epithelium.
C) Barrett esophagus is present in > 50% o patients with GERD.
See Chapter 27 Quick Quiz: More than a little headache. D) Barrett esophagus can regress with adequate treatment o
GERD.
30) You are called by an anxious mother about her 3-year-
old daughter who accidentally ingested lemon-lime smell- See questions 7.1.2 and 7.1.5.
ing household cleaner at her grandparent’s house thinking it
was soda. Mom has been instructed by her parents, who were 35) Which o the ollowing lesions is considered premalig-
born in the late 1950s, that she should give ipecac to induce nant on colonoscopy?
vomiting o the substance. You counsel her that ipecac: A) Sessile polyp.
A) Should not be used. B) Hyperplastic polyp.
B) Can be used i the ingestion may cause mental status C) ubular adenoma.
changes. D) Pedunculated polyp.
C) Is e ective i used within 30 minutes o an ingestion.
D) Is available over the counter in American pharmacies. See question 7.5.2: Helpful ip.

See question 1.1.1 and Helpful ip. 36) Which o the ollowing histories in a patient with GERD
is most concerning or serious underlying disease?
31) How requently should methadone be dosed when used A) Food bolus impaction on two separate occasions.
or pain control in the terminally ill? B) Dysphagia to solids ollowed in several months with dys-
A) Once daily. phagia to liquids.
B) Every 8 to 12 hours. C) Re ux o undigested ood at night.
C) Every 4 to 6 hours. D) Halitosis.
D) Methadone should not be used or pain management in the
terminally ill. See question 7.1.1.

See question 27.2.6: Helpful ip. 37) Which o the ollowing is an appropriate test o cure or
H. pylori?
32) A 46-year-old male presents to clinic or evaluation o le A) Stool antigen test done 1 month a er nishing treatment.
shoulder pain. He has been taking over-the-counter ibupro- B) Serum antibody test done 3 months a er nishing treat-
en 600 mg every 8 hours or the last 4 weeks. He is wonder- ment.
ing i there are any possible long-term side e ects o taking C) CLO test done 1 week a er nishing treatment.
this medication. Most notably, he is concerned, as his wi e D) Breath urea test done 1 week a er nishing treatment.
has told him that ibupro en can “tear up the gut.” You tell
him that NSAIDs, such as ibupro en, can cause ulceration in: See question 7.4.5.
A) T e stomach.
B) T e duodenum. 38) Which o the ollowing is true about a gastric eeding
C) T e colon. tube in the demented elderly?
D) All o the above. A) It increases the patient’s quality o li e.
B) It reduces the risk o pneumonia.
See question 7.4.5: Helpful ip. C) It reduces mortality.
D) It does nothing to improve quality o li e but can cause com-
33) What is true about the treatment o gastroesophageal plications.
re ux disease (GERD)?
A) reatment o Helicobacter pylori is e ective in curing GERD. See question 21.5.14.
B) reatment should always start with a proton pump inhibitor.
C) Surgical options (e.g., undoplication) are o en suboptimal 39) Which o the ollowing is NO a component o CRES
with patients still requiring medication. syndrome?
D) Esomeprazole is vastly superior to omeprazole. A) Calcinosis.
B) Renal ailure.
See questions 7.1.3 and 7.1.4. C) Esophageal dysmotility.
742 FAMil Y MEDiCin E ExAMin At io n & Bo ARD REViEW

D) Sclerodactyly. C) Cipro oxacin.


E) elangiectasia. D) Azithromycin.
E) Mebendazole.
See question 11.10.4.
40) Which o these drugs is indicated or control o an acute See question 7.8.2: Helpful ip.
are o Crohn disease?
A) Any 5-ASA moiety. 46) You recently diagnosed a 30-year-old emale with celiac
B) Oral prednisone. disease. She is wondering what oods she should avoid. Which
C) Sul asalazine. o the ollowing oods DOES NO contain gluten?
D) T alidomide. A) Wheat.
E) Metronidazole. B) Rye.
C) Rice.
See question 7.6.4. D) Sticky rice.

41) Which o the ollowing is a contraindication to use o sul- See question 7.8.6.
asalazine in Crohn disease?
A) Aspirin allergy. 47) Which o the ollowing tests is the most sensitive or
B) Heme positive stools. the diagnosis o bacterial overgrowth syndrome (o the GI
C) Fever. tract)?
D) Platelet count < 100,000. A) Urea breath test.
B) D-Xylose breath test.
See question 7.6.6. C) Hydrogen breath test.
D) CLO test.
42) Which o the ollowing is the best screening test or hepa-
titis C virus (HCV)?
A) Quantitative HCV PCR. See question 7.8.3.
B) Hepatitis C antibody.
C) Recombinant Immunoblot Assay (RIBA). 48) Which o the ollowing is a relative contraindication to
D) Qualitative HCV PCR. treatment o hepatitis C with inter eron?
A) Fever.
See question 7.10.1. B) Severe depression.
C) Severe osteoarthritis.
43) Which o the ollowing is the most speci c test or gluten D) Severe eczema.
enteropathy (nontropical sprue)?
A) Antiendomysial IgA antibody. See question 7.10.6.
B) issue transglutaminase antibody.
C) Antigliadin antibody. 49) Routine prenatal screening in the rst trimester includes
D) Hydrogen breath test. all o the ollowing EXCEP :
A) Blood type and antibody screen.
See question 7.8.4. B) Hepatitis B sur ace antigen.
C) HIV antibody.
44) C. dif cile colitis has been linked to all o the ollowing
D) MSAFP (maternal serum alpha- etoprotein).
EXCEP :
E) Rubella antibody.
A) Hospitalization.
B) Use o uoroquinolones.
C) Use o PPIs. See question 15.1.7.
D) Use o H 2 blockers.
E) Use o metronidazole. 50) A 22-year-old G2 P2 emale is ready to leave the hospi-
tal on postpartum day 2 a er a normal spontaneous delivery,
See question 8.2.3 and Helpful ip. but she develops lower abdominal pain and ever. She denies
urinary symptoms. She reports constipation and moderate
45) You are seeing a 15-year-old emale who works at a day lochia. Her temperature is 38.8°C. She had prolonged rup-
care where there was a recent Giardia outbreak. She has been ture o membranes and prolonged labor, and the placenta
experiencing diarrhea, malaise, and nausea. You are con- was removed manually. T e most appropriate course o
cerned she may have contracted Giardia. Which o the ol- action now is:
lowing is recommended or treating Giardia? A) Discharge with acetaminophen or ever and ollow-up in
A) Metronidazole. 2 days.
B) Vancomycin. B) Discharge with amoxicillin and ollow-up in 2 days.
CHAPTER 30 • Fin Al ExAMin At io n 743

C) Keep the patient in the hospital and obtain cultures or gon- 55) A 79-year-old male presents with his wi e who com-
orrhea and chlamydia. plains that he has been more orget ul over the past month.
D) Keep the patient in the hospital and start IV gentamicin and T e patient agrees and complains o orgetting where he
clindamycin. put things, where the car is parked, and names o acquain-
E) Manual exploration o the uterus or retained placenta. tances. Also, he is more irritable and angers easily. He has
trouble sleeping, and he has lost 5 lb. He is healthy and takes
See question 15.2.9. no medications. He has poor eye contact, a blunted a ect,
and poor concentration. His vitals and physical examination
51) A 21-year-old G0 emale presents or a physical examina- are otherwise normal. He can recall three items and draw a
tion and her rst Pap smear. She has recently become sex- clock with no dif culty. T e most likely cause o his symp-
ual activity and has one li etime partner. T e results o the toms is:
Pap smear show low-grade squamous intraepithelial lesion A) Delirium due to metastatic carcinoma.
(LSIL). In accordance with guidelines, you recommend: B) Delirium due to underlying systemic in ection.
A) Immediate colposcopy. C) Dementia due to Alzheimer disease.
B) Immediate re erral or excisional procedure. D) Dementia due to stroke.
C) Return in 12 months or Pap smear. E) Depression.
D) Return in 3 months or Pap smear.
E) Return in 6 months or colposcopy and Pap smear. See question 21.5.2.

See question 15.12.8: Helpful ip. 56) You are considering treating a 65-year-old male with sup-
plemental testosterone due to low serum testosterone levels
52) A 48-year-old emale presents with menopausal symp- associated with atigue, muscle weakness, and mild depres-
toms. She has had a total hysterectomy but her ovaries are sion. A er initiating testosterone, it is most important to
intact. She would like to know the bene ts and risks o taking check which o the ollowing?
estrogen-only hormone replacement therapy (HR ). You tell A) Creatinine.
her that estrogen-only HR is associated with: B) Hemoglobin.
A) Decreased risk o myocardial in arction. C) Potassium.
B) Decreased risk o ovarian cancer. D) Sodium.
C) Increased risk o colon cancer.
D) Increased risk o osteoporosis. See question 16.7.3.
E) Increased risk o stroke.

See question 15.14.3. 57) Delirium in the hospitalized elderly patient can be pre-
vented by implementing all o the ollowing interventions
53) A 25-year-old G5P4 emale at 26 weeks o gestation pres- upon admission EXCEP :
ents to labor and delivery with abdominal pain. T e pain A) Assuring patient has access to usual aides (hearing aids,
is sharp or tearing and located in her lower abdomen. She glasses, etc.).
has not had any contractions. She just began to have a little B) Early mobilization.
vaginal spotting be ore she came in. Her pregnancy has been C) Lorazepam 1 mg QHS.
complicated by tobacco use and hypertension. What is the D) Noise reduction at night.
most likely diagnosis? E) Orientation stimuli.
A) Cervical cancer.
B) Normal labor. See questions 21.3.4 and 21.3.5.
C) Placenta previa.
D) Placental abruption. 58) A 75-year-old male with mild dementia, atrial brilla-
E) Uterine rupture. tion (not on anticoagulation), osteoarthritis, and depres-
sion presents a er sustaining a all in his home 1 week ago.
See question 15.2.1: Helpful ip. His son states, “He is a little banged up but otherwise ne.”
Your routine evaluation should include all o the ollowing
54) T e USPS F recommends screening or osteoporosis in: EXCEP :
A) All adults aged 65 and older. A) Asking about potential neglect and abuse.
B) All men aged 75 and older. B) C scan o the brain.
C) All women aged 65 and older. C) Medication review.
D) All women at onset o menopause. D) Neurological examination.
E) Women who smoke at age 50 and older. E) Observation o the patient ambulating.

See question 21.2.3. See questions 21.4.1 and 21.4.2.


744 FAMil Y MEDiCin E ExAMin At io n & Bo ARD REViEW

59) A 75-year-old emale ell and injured her hip. She did not C) A neurologic symptom that does not resolve within the rst
racture it but has signi cant pain. She has a remote history o 24 hours o a traumatic injury.
peptic ulcer disease. She takes lisinopril or hypertension and D) Absolutely any trauma involving the head and/or neck.
phenytoin or a seizure disorder. Which analgesic medication
will be sa est to initiate in this patient or her moderate pain? See question 14.8.1: Helpful ip.
A) Aspirin.
B) Meperidine. 64) A 17-year-old emale runner with secondary amenor-
C) Hydrocodone. rhea should be urther evaluated with all o the ollowing
D) Piroxicam. EXCEP :
E) Fentanyl transdermal patch. A) Assessing calcium intake.
B) MRI o the pituitary.
See question 21.6.2. C) Screening or eating disorders.
D) Urine beta-hCG.
60) An 80-year-old emale nursing home resident with
dementia recently started re using medication and slapping See questions 14.2.2 and 14.2.4.
at the sta when they try to bathe her. T e nurse at the care
center calls to request “something or her agitation.” As the 65) Gynecomastia in an adolescent male occurs as a response
sa est and most e ective intervention, you recommend: to which o ollowing mechanisms?
A) A behavior log to track when the agitation occurs and what A) Excessive dehydroepiandrosterone.
might be causing it. B) Excessive estrogen compared with testosterone.
B) Haloperidol 1 mg IV prior to bathing and medication C) Excessive growth hormone.
administration. D) Excessive progesterone compared with testosterone.
C) Haloperidol 1 mg PO BID. E) Rapidly developing obesity.
D) Restraints with bathing.
E) Alprazolam 1 mg PO ID. See question 16.5.2.

See question 21.6.9. 66) reatment o a patient in diabetic ketoacidosis (DKA)


includes all o the ollowing EXCEP :
61) T e most appropriate next step or a 60-year-old male A) Aggressive volume replacement.
with a PSA level 12.5 ng/mL (normal range < 4.0) is: B) Frequent glucose monitoring.
A) One month o a uoroquinolone ollowed by repeat rectal C) Insulin.
examination and PSA. D) Potassium.
B) Re erral or prostate biopsy. E) Sodium bicarbonate.
C) Repeat PSA in 6 to 12 months.
D) ransrectal ultrasound o the prostate. See questions 10.9.1 and 10.9.4.
E) reatment o benign prostatic hyperplasia with terazosin.
67) A 30-year-old male presents with dif culty obtaining an
See question 16.8.3. erection suf cient or penetration. He has acid re ux and
takes cimetidine. He reports a good relationship with his wi e
62) A 33-year-old male with depression had been taking par- and denies depression. He does not smoke and rarely drinks
oxetine 60 mg daily or a year or depression. Due to sexual alcohol. T e most appropriate intervention at this time is:
problems, he decided to try another medication, and his doc- A) o order a testosterone level.
tor prescribed bupropion. He stopped the paroxetine one day B) o per orm cardiac stress testing.
and started bupropion the next. He comes in 5 days later eel- C) o re er him to a urologist.
ing dizzy, nauseated, and atigued. He complains o myalgias, D) o replace cimetidine with omeprazole.
eelings o “electric shocks,” and insomnia. T ese symptoms E) o send him to a psychologist.
are most likely due to:
A) Adverse e ects o bupropion. See question 16.10.3.
B) Hyperthyroidism.
C) Major depression. 68) O the ollowing, the most requent side e ect o met or-
D) Serotonin syndrome. min is:
E) Discontinuation o paroxetine. A) Constipation.
B) Hypoglycemia.
See question 23.9.1. C) Lactic acidosis.
D) Renal ailure.
63) Concussion is de ned as: E) Weight loss.
A) A disturbance in brain unction caused by trauma.
B) A traumatic event resulting in the loss o consciousness. See question 10.10.7.
CHAPTER 30 • Fin Al ExAMin At io n 745

69) A 40-year-old male presents with generalized weakness D) Whether the renal ailure is due to intrinsic renal disease or
or 1 month. He has lost some weight, perhaps 10 lb. He a prerenal cause.
brie y lost consciousness yesterday while getting out o bed. E) I the patient has pseudohyponatremia.
He denies depression, drug or alcohol use, and any signi -
cant medical history. He is hypotensive but not tachycardic. See question 5.6.3.
He has orthostatic hypotension as well. Lab tests reveal mild
anemia, low sodium, elevated potassium, and normal SH, 73) T e major Jones criteria or rheumatic ever include all o
BUN, creatinine, and glucose. He is surprisingly tan or being the ollowing EXCEP :
a Caucasian armer in the winter. (Did he just return rom A) Carditis.
Cancun? No. He’s not traveled. Good question.) o con rm B) Fever.
your presumptive diagnosis, you order: C) Polyarthritis.
A) 24-hour urine catecholamines. D) Subcutaneous nodules.
B) Free 4 and 3. E) Sydenham chorea.
C) Plasma metanephrines.
D) Serum cortisol and adrenocorticotropic hormone (AC H). See Chapter 11 Quick Quiz: An Ill Child.
E) Serum testosterone.
74) A 29-year-old emale daycare teacher presents with a
See question 10.8.2. severely pruritic rash that started at her wrists and has pro-
gressed to the web spaces o her ngers, under her arms,
70) A 62-year-old emale presents to the emergency depart-
around her waist, and around her nipples. On examination,
ment with a sudden “hole” in her right visual eld developing
she has multiple excoriations and ew small, erythematous
today. She also has a right temporal headache present or the
papules. T e most appropriate next step is:
last 2 weeks, shoulder and neck pain or a month, and weight
A) Biopsy o normal-appearing skin.
loss o 5 lb. She is slightly hypertensive and has a prominent
B) Biopsy o one o the papules.
tender vessel at the right side o her head. Her CBC is nor-
C) Empiric treatment with topical clotrimazole.
mal but the ESR is 85 mm/hr. What is the most appropriate
D) Empiric treatment with topical lindane 1%.
course o action?
E) Empiric treatment with topical permethrin 5%.
A) Administer IV methylprednisolone and admit or urther
evaluation.
See questions 8.6.1 and 8.6.2.
B) Admit or cardiac monitoring and rule out myocardial
in arction.
C) Discharge to home with oral antibiotics. 75) A 60-year-old emale with diabetic nephropathy is hos-
D) Discharge to home with re erral to an ophthalmologist in pitalized with chest pain, and a cardiac catheterization is
the next week. planned. Which one o the ollowing is the best option to
E) Per orm a C scan o the brain and discharge to home i reduce her risk o contrast-induced nephropathy?
normal. A) Administer ketorolac and IV saline.
B) Administer mannitol and IV saline.
See question 11.2.7. C) Administer IV saline alone.
D) Administer N-acetylcysteine.
71) You nd a new and suspicious skin lesion in a patient who
has had a liver transplant or hepatitis C and is on immuno- See question 5.1.7.
suppressive therapy. You plan to per orm a biopsy. I the skin
lesion turns out to be malignant, it will most likely be: 76) A 58-year-old man with hypertension, diabetes, heart
A) Basal cell carcinoma. ailure, and chronic kidney disease (stage 4, GFR 25 mL/
B) Distant metastasis rom liver cancer. min/1.73 m2) presents or ollow-up. His current medications
C) Melanoma. are insulin, aspirin, metoprolol, and lisinopril. His blood
D) Squamous cell carcinoma. pressure is 142/86 mm Hg and he has signi cant dependent
E) Kaposi sarcoma. edema. His labs reveal serum potassium o 5.3 mEq/L. o
achieve his blood pressure to goal (< 130/80) while avoiding
See question 17.3.2. adverse events, the best initial step is:
A) Discontinue lisinopril.
72) T e ractional excretion o sodium (FENa) is use ul or B) Furosemide 20 mg PO daily.
determining: C) Hydrochlorothiazide 12.5 mg PO daily.
A) I the patient has true hyponatremia. D) Increase lisinopril.
B) Whether the patient has oliguric or anuric renal ailure. E) Losartan 25 mg PO daily.
C) Whether the renal ailure is due to acute tubular necrosis or
another cause. See question 5.1.10.
746 FAMil Y MEDiCin E ExAMin At io n & Bo ARD REViEW

77) Which o the ollowing is most likely to cause hypokale- the ED to triage. Which o the ollowing interventions should
mia? not wait an hour and must be done now?
A) Excessive use o “lite” salt. A) Administer ce riaxone, vancomycin, and dexamethasone.
B) Hypoaldosteronism. B) Consult a neurosurgeon.
C) Hypomagnesemia. C) Per orm lumbar puncture.
D) Overdose o propranolol. D) Obtain blood cultures.
E) Renal tubular acidosis type 4. E) Order C o the brain.

See question 5.5.2. See questions 8.3.1 and 8.3.3.

78) A patient presents with thickened, yellowish, dystrophic 82) You per orm joint aspiration on a patient with a pain ul,
toenails. What is the most appropriate next step? swollen knee. Microscopic examination o the uid shows
A) Recommend that the patient return or toenail removal. rhomboid-shaped, positively bire ringent crystals. Which
B) Send nail scrapings or KOH stain and/or ungal culture. one o the ollowing is the most likely to alleviate the patient’s
C) Start treatment with a topical anti ungal. acute symptoms?
D) Start treatment with an oral anti ungal. A) Acetaminophen daily.
E) Start treatment with an IV anti ungal. B) Allopurinol daily.
C) Ce riaxone IM.
See question 17.3.4. D) Corticosteroid injection into the knee.
E) opical anti-in ammatory cream.
79) All o the ollowing are consistent with a diagnosis o
See question 11.5.2.
syndrome o inappropriate antidiuretic hormone secretion
(SIADH) EXCEP : 83) A 23-year-old male with HIV stopped taking all o his
A) High urine osmolality. medications 3 months ago due to cost. He was eeling ne
B) Low plasma osmolality. until 3 weeks ago when he developed a cough. He now has
C) Low urine sodium. daily evers around 101°F, a non-productive cough, dyspnea
D) Normal adrenal unction. on exertion, atigue, chills, and tightness in his chest with
E) Normal thyroid unction. inspiration. His examination is notable or ever, diaphore-
sis, bilateral crackles with inspiration, and mild tachypnea.
See questions 5.7.2 and 5.7.3. Chest x-ray shows di use bilateral interstitial in ltrates.
Which o the ollowing is the most likely causative agent or
80) A 21-year-old male is brought in the emergency depart- this pulmonary in ection?
ment by his girl riend a er he overdosed on aspirin. He took A) Adenovirus.
“a bottle, maybe 100 pills or so,” but he denies other inges- B) Cryptococcus neoformans.
tions. He complains only o nausea. He becomes more som- C) Mycobacterium tuberculosis.
nolent during the evaluation. He is slightly tachycardic and D) Pneumocystis jiroveci (PCP).
ebrile with a normal blood pressure. His blood gas shows: pH E) oxoplasma gondii.
7.38, PaCO2 20 mm Hg, PaO2 98 mm Hg, HCO3 15 mEq/L.
His creatinine, CBC, and electrolytes are normal, except or See question 9.1.8.
low potassium. What is the best description o this patient’s
blood gas? 84) Which o the ollowing is not use ul in the treatment o
A) Metabolic acidosis and metabolic alkalosis. hepatic encephalopathy?
B) Metabolic acidosis and respiratory alkalosis. A) Oral lactulose.
C) Metabolic alkalosis and respiratory acidosis. B) Polyethylene glycol (Golytely).
D) Pure metabolic acidosis. C) Oral ri amixin.
E) Normal blood gas (no acidosis or alkalosis). D) Lactulose enemas or acute encephalopathy.

See Chapter 5 Quick Quiz: Acid–Base Disorder. See question 7.13.2.

81) A 19-year-old college student presents to the emergency 85) An otherwise-healthy 70-year-old emale is admitted and
department with ever, headache, myalgias, and con usion. started on ce riaxone and azithromycin or pneumonia. On
She has had a splenectomy but is otherwise healthy. T e hospital day 3, her serum creatinine is ound to have tripled
examination is notable or somnolence, ever, and nuchal rom admission. She is mildly nauseated and has an ery-
rigidity. Due to her inability to ollow directions, the neuro- thematous, macular rash on her trunk and arms. Her CBC
logical examination is dif cult to complete, but it appears to shows that her white cell count has declined rom admission,
be non- ocal. T ere are several other seriously ill patients in but she now has a prominent eosinophilia. Urinalysis shows
CHAPTER 30 • Fin Al ExAMin At io n 747

1 + protein, and urine sediment shows white cell casts and 90) Which o the ollowing is generally true in alcohol-related
eosinophils. T e most appropriate next step is: liver disease?
A) Add metronidazole to her antibiotic regimen. A) AS is two times greater than AL .
B) Bolus with IV 0.9% saline. B) AL is two times greater than AS .
C) Consult a nephrologist or possible renal biopsy. C) Both AL and AS are elevated to the same degree.
D) Discontinue ce riaxone and consider an alternative antibi- D) T e GG is speci c or liver disease and higher than either
otic. the AL or AS .
E) Start urosemide to improve urine output.
See question 7.15.2: Helpful ip.
See question 5.13.3: Helpful ip.
91) Appropriate antibiotic treatment o diverticulitis includes
86) Each o the ollowing patients is ound to have asymp- all o the ollowing EXCEP :
tomatic bacteriuria. Who should be treated with a course o A) Cipro oxacin + metronidazole.
antibiotics? B) Amoxicillin clavulanate.
A) An 88-year-old emale nursing home resident. C) rimethoprim/sul amethoxazole + metronidazole.
B) A 20-year-old pregnant patient. D) Clindamycin + metronidazole.
C) A 75-year-old male with an indwelling Foley catheter or
obstructive BPH. See question 7.17.3: Helpful ip.
D) All o the above.
92) Which o the ollowing is true about the treatment o
See questions 8.4.1 and 8.4.2, and Helpful ip.
pancreatitis?
87) In a patient with ascites, which o the ollowing will cause A) Antibiotics should be used in most cases o pancreatitis.
a low serum-ascites albumin gradient (SAAG)? B) Oral eedings is the pre erred method o nutrition i toler-
A) Carcinomatous peritonitis. ated.
B) Portal hypertension. C) Pseudocysts must be drained or pancreatitis to resolve.
C) Budd–Chiari syndrome. D) T e Ransom criteria can be used at the time o admission to
D) Cirrhosis. accurately predict mortality.

See question 7.12.4: Helpful ip. See questions 7.15.9 and 7.15.10: Helpful ip.

88) A patient comes to the emergency department a er sus- 93) Which o the ollowing does NO promote gastric emp-
taining a needle stick. She is a nurse who had just nished tying in gastroparesis?
drawing blood or culture on a patient with AIDS, and some- A) Erythromycin.
how she stuck hersel through her glove. She bled a little. She B) Metoclopramide.
washed the area copiously. What is the most appropriate next C) Amoxicillin.
step? D) Cisapride.
A) Prescribe antiretrovirals or 4 weeks.
B) Prescribe single-dose antiretroviral prophylaxis. See question 7.16.2.
C) Reassure the patient as her risk o contracting HIV is negli-
gible. 94) O the ollowing, the most common cause o acute pan-
D) est her or HIV and treat based on the results. creatitis in the United States is:
A) Alcohol.
See question 9.3.2: Helpful ip. B) Hypertriglyceridemia.
89) About hal o patients with portal hypertension have C) T iazide diuretics.
esophageal varices, and one-third o these will experience a D) Viruses.
gastrointestinal bleed. Patients with varices may bene t rom
preventative therapy aimed at reducing bleeding risk. Which See question 7.15.7.
o the ollowing IS NO use ul in the prevention o variceal
bleeding in patients with portal hypertension due to alco- 95) All o the ollowing are causes o non-alcoholic atty liver
holic cirrhosis? disease EXCEP :
A) Pentoxi ylline ( rental). A) Statin use.
B) Nadolol. B) Hypothyroidism.
C) Vitamin K. C) Diabetes.
D) Cilostazol (Pletal). D) Obesity.

See question 7.13.4. See question 7.14.3.


748 FAMil Y MEDiCin E ExAMin At io n & Bo ARD REViEW

96) T e most common cause o the development o drug D) A patient with a PPD reaction o 10 mm 1 year a er treat-
resistance in HIV is: ment or latent B.
A) Failure to include zidovudine in the treatment regimen.
B) Failure to initiate treatment until the patient has a known See questions 8.4.4 and 8.4.5.
AIDS de ning illness.
C) Poor compliance with medications. 101) T e recommended empirical antibiotic treatment o
D) Failure to include a protease inhibitor in the treatment meningitis in an immunocompetent adult age 40 is:
regimen. A) Ce riaxone.
B) Ce riaxone + vancomycin.
See question 9.1.3. C) Ce riaxone + cipro oxacin.
D) Ce riaxone + MP/SMX.
97) At what CD4+ level should one initiate prophylactic
treatment o Pneumocystis jiroveci (previously Pneumocystis See question 8.3.1.
carinii)?
A) CD4+ ≤ 50 cells/mm 3 102) T e most common bacterial organism causing meningi-
B) CD4+ ≤ 75 cells/mm 3 tis in adults is:
C) CD4+ ≤ 100 cells/mm 3 A) Pneumococcus.
D) CD4+ ≤ 200 cells/mm 3 B) Meningococcus.
C) Haemophilus.
See question 9.1.4. D) Listeria.

See questions 8.3.1 and 8.3.2.


98) Which o the ollowing recommendations regarding Pap
smear screening in an HIV positive woman is true? 103) All o the ollowing can be used or malaria prophylaxis
A) No modi cation is needed in the Pap smear regimen. EXCEP :
B) I the CD4+ count is normal and the patient has two normal A) Doxycycline.
Pap smears at 1 year intervals, you can go back to routine B) Me oquine.
screening. C) Azithromycin.
C) I the patient has a CD4+ count < 200 cells/mm 3, screening D) Atovaquone/proguanil (Malarone).
should be done every 6 months regardless o whether or not
the patient has had negative Pap smears. See question 8.8.4.
D) Pap smears can be suspended in HIV-positive patients since
they will likely die rom HIV be ore they die rom cervical 104) Which o the ollowing is true regarding the diagnosis
cancer. o urolithiasis?
A) Over 90% o patients will have blood in their urine at the
See question 9.2.14. time o diagnosis.
B) Urolithiasis may be di cult to di erentiate rom aortic dis-
99) Which o the ollowing statements best re ects the cur- section at the initial time o presentation.
rent thinking about treating in uenza? C) Hematuria will reliably di erentiate urolithiasis rom aortic
A) Rimantadine and amantadine are the most e ective drugs dissection.
or in uenza B. D) A negative “FAS ” ultrasound scan is considered the stan-
B) reatment with oseltamivir ( ami u) is highly e ective, dard and i negative rules out urolithiasis.
thus negating the need or in uenza vaccine.
C) Oseltamivir must be started within 48 hours o symptom See question 5.3.5: Helpful ip.
onset to be o any bene t.
D) T ere is no resistance o in uenza A to oseltamivir. 105) Which o the ollowing pro les are you likely to see in a
patient with a pre-renal cause o elevated creatinine?
See question 8.1.3. A) Urine sodium < 20 mEq/L, ractional excretion o sodium
< 1%.
100) Assuming each o these patients has a negative chest B) Urine sodium < 20 mEq/L, ractional excretion o sodium
x-ray, which o the ollowing patients needs isoniazid treat- > 2%.
ment? C) Urine sodium > 40 mEq/L, ractional excretion o sodium
A) A patient with no risk actors who has a PPD reaction o < 1%.
5 mm. D) Urine sodium > 40 mEq/L, ractional excretion o sodium
B) A patient with recent exposure to B and a PPD reaction o > 2%.
5 mm.
C) A health-care worker with a PPD reaction o 5 mm. See questions 5.6.3 and 5.13.1.
CHAPTER 30 • Fin Al ExAMin At io n 749

106) Aldosterone resistance (such as occurs with diabetic D) Clopidogrel alone.


nephropathy) or hypoaldosteronism will present with which E) War arin.
o the ollowing?
A) Hypokalemia. See question 18.1.5.
B) Hyperkalemia.
C) Hyperphosphatemia. 112) Which one o the ollowing entities presents with
D) Hypophosphatemia. re exes preserved?
A) Guillain–Barré.
See question 5.1.9. B) Amyotrophic lateral sclerosis.
C) Charcot–Marie– ooth disease.
107) Which o the ollowing regimens IS NO recommended D) Diabetic neuropathy.
or treatment o a simple cystitis in an otherwise healthy,
non-pregnant adult emale? See question 18.6.4: Helpful ip.
A) Amoxicillin 500 mg PO ID or 3 days.
B) Levo oxacin 250 mg PO daily or 3 days. 113) Which one o the ollowing agents is most e ective or
C) Nitro urantoin 100 mg PO ID or 5 days. controlling the pain o peripheral neuropathy?
D) MP/SMX 1 PO ID or 3 days. A) ricyclic antidepressant ( CA).
B) Gabapentin.
See Chapter 5 Quick Quiz: Dysuria. C) Oxycodone or other narcotic.
D) Carbamazepine or other traditional antiepileptic.
108) Which o the ollowing drugs is LEAS LIKELY to See question 18.3.6 and Helpful ip.
reduce the progression o proteinuria?
A) Enalapril. 114) Which o the ollowing drugs IS NO associated with
B) Losartan. rebound headaches?
C) Verapamil. A) Acetaminophen.
D) Ni edipine. B) Dihydroergotamine (DHE).
C) Nortriptyline.
See question 5.1.5 and 5.1.13. D) Sumatriptan.

109) T e de nition o nephrotic syndrome requires all o the See question 18.4.6.
ollowing EXCEP :
A) Hypoalbuminemia. 115) T e risk o developing a seizure disorder a er a single
B) Urine albumin excretion > 3 g per day. ebrile seizure is:
C) Edema. A) 1% to 5%; the same as the rest o the population.
D) Renal biopsy showing appropriate changes. B) 6% to 10%; slightly higher than the general population.
C) 11% to 15%; signi cantly higher than the general popula-
See question 5.4.5. tion.
D) Unknown.
110) Which o the ollowing patients should have a carotid
endarterectomy based on all current U.S. guidelines? See question 18.14.4.
A) A 60-year-old emale with a unilateral symptomatic 70%
carotid plaque. 116) You determine that a 77-year-old emale has Parkinson
B) A 60-year-old male or emale with an asymptomatic 69% disease that is inter ering with her daily li e. T e best drug
carotid plaque. or drug combination to alleviate her symptoms and improve
C) Patient o either gender with bilateral 50% carotid plaque, her unction is:
symptomatic or asymptomatic. A) A COM inhibitor (e.g., Entacapone).
D) None o the above meet current quali cation criterion. B) A dopamine agonist (e.g., Requip).
C) Levodopa/carbidopa (e.g., Sinemet).
See Chapter 18 Quick Quiz: When to Cut?. D) Apomorphine.

111) I cost were NO an issue, which o the ollowing See question 18.10.6.
drugs/drug combinations would be the ideal regimen or the
secondary prevention o stroke? 117) T e PRESENCE o an RAPD (relative a erent pupillary
A) Aspirin + clopidogrel. de ect) can be indicative o :
B) Aspirin + dipyridamole. A) Cataracts.
C) Aspirin alone. B) Large retinal detachment.
750 FAMil Y MEDiCin E ExAMin At io n & Bo ARD REViEW

C) Bleed into the anterior chamber.


D) Severe re ractive error.

See Chapter 19 glossary and question 19.2.1.

118) Which o the ollowing patients has an indication or


cataract surgery?
A) Vision 20/100 bilaterally in a patient who has no visual com-
plaints or unctional impairment.
B) Vision 20/20 OD and 20/200 OS in a patient who can see
well enough to do everything she desires.
C) Vision 20/30 OD and 20/30 OS in a patient who is bothered
by her inability to quilt.
D) Vision unknown in a patient who can carry out all ADLs to
FIGURE 30-3.
her own satis action.
122) What is the general course o rheumatoid arthritis dur-
See question 19.14.3. ing pregnancy?
A) Worsening during pregnancy.
119) All o the ollowing intraocular muscles are innervated B) Worsening during pregnancy but only i methotrexate is
by cranial nerve III EXCEP : used.
A) In erior oblique. C) It may remit during pregnancy and methotrexate is contra-
B) In erior rectus. indicated.
C) Lateral rectus. D) Morning sickness is worse in those with rheumatoid arthri-
D) Medial rectus. tis.
E) Superior rectus. E) Pregnancy has no e ect on rheumatoid arthritis and vice
versa.
See question 19.15.1.
See question 11.1.8.
120) Skinnier, “politically correct” Santas are in vogue this
year (bearing gi s o celery, no doubt). Your patient is in 123) A diagnosis o rheumatoid arthritis can be made a er
tizzy: he has played Santa or years without the need or pil- symptoms have been present or at least:
lows. He wants to keep up his tradition as St. Nick. What can A) 15 minutes.
you tell him about weight reduction surgery? B) 2 weeks.
A) “Sorry, even though you may lose weight, it will not help C) 6 weeks.
your overall health.” D) Duration o symptoms is no longer a criterion or diagnos-
B) “You must have a BMI > 40 kg/m 2 be ore quali ying or bar- ing rheumatoid arthritis.
iatric surgery regardless o other underlying weight-related
conditions.” See question 11.1.2: Helpful ip.
C) “You must have a BMI o > 35 kg/m 2 AND signi cant
reversible disease (e.g., hypertension, diabetes) to quali y or 124) You make a diagnosis o rheumatoid arthritis based
bariatric surgery.” on laboratory and clinical criteria. T e best time to begin
D) “T e reindeer union says it won’t work i Santa’s weight is disease-modi ying antirheumatic drugs (DMARDs) is:
under 300 lb—they need to keep 8 reindeer working the A) At the time o diagnosis.
sleigh at all times.” B) A er ailure o 2 NSAIDs.
C) A er the ailure o prednisone.
See question 22.5.1. D) When complications o rheumatoid arthritis occur.
E) A er waiting 6 months to see a rheumatologist.
121) A 70-year-old male patient complains o severe unilat-
eral visual loss. He has no other symptoms. Here is a picture See question 11.1.3 and Helpful ip.
o his undus (Fig. 30-3) that you took with your smartphone
jerry-rigged to a direct ophthalmoscope. His diagnosis is: 125) You are seeing a 57-year-old male in your practice who
A) Acute glaucoma. is complaining o bi-temporal headache. He checked the web
B) Acute venous disruption. and decided that he has giant cell (temporal) arteritis. T e
C) Acute arterial occlusion. only problem is that his ESR is only 30 mm/hr. T e CRP is
D) Diabetic retinopathy. not much more edi ying. You appropriately respond to him:
A) “You may still have temporal arteritis since 15% or more will
See question 19.12.1. have normal ESR and CRP.”
CHAPTER 30 • Fin Al ExAMin At io n 751

B) “You seem to be displaying drug seeking behavior since you C) Likelihood o resistant organisms (based on recent antibi-
came in to the ED with the same complaint 1 month ago.” otic use, daycare exposure, etc.).
C) “You may still have temporal arteritis, but we need to per- D) Patient age.
orm a temporal bone biopsy.” E) Medication allergies.
D) “You may still have temporal arteritis, but we will need to
submit your head to the state lab or ormal testing . . . You See question 3.12.1: Helpful ip, 3.12.2, and 3.12.3.
don’t mind . . . do you?”
131) Regarding depot medroxyprogesterone (Depo-Provera),
See question 11.2.2. women at signi cant risk o ______________ should con-
sider an alternative orm o contraception.
126) When treating gout, which one o these drugs or gout A) Osteoporosis.
will be ine ective in those with renal insuf ciency? B) Breast cancer.
A) Allopurinol. C) Ovarian cancer.
B) Probenecid. D) Depression.
C) Prednisone.
D) Colchicine. See question 15.1.1: Helpful ip.
E) Indomethacin.
132) Bleeding in a non-pregnant, amenorrheic patient in
See question 11.4.5. response to a “progesterone challenge” (e.g., medroxyproges-
terone given or 10 days and then stopped) indicates that the
127) Regarding therapy or giant cell arteritis, which o the patient has suf cient endogenous:
ollowing statements best describes the appropriate relation- A) Follicle stimulating hormone (FSH).
ship o aspirin, prednisone, and giant cell arteritis? B) Luteinizing hormone (LH).
A) ASA (81 mg/day) can be used as adjunctive therapy or C) Progesterone.
those on prednisone. D) Estrogen.
B) ASA (325 mg/day) can be used as adjunctive therapy or
those on prednisone. See Chapter 15 Quick Quiz: Amenorrhea.
C) High-dose ASA (650 mg/day) can be used in those who do
not tolerate prednisone. 133) You get the ollowing results on a vaginal wet prep: pH
D) ASA has no role in the treatment o giant cell arteritis. 5.0 and a positive “whi ” test. Wouldn’t you know it, the rest
o the results got lost. With the in ormation available, the
See question 11.2.3. most likely diagnosis is:
A) Vulvovaginal candidiasis.
128) T e one best, rst-line drug treatment o osteoarthritis B) Bacterial vaginosis.
pain is: C) Vaginal trichomoniasis.
A) Acetaminophen. D) Physiologic vaginal discharge.
B) Celecoxib (Celebrex).
C) Ibupro en. See Chapter 15: able 15-9.
D) Naproxen.
E) Morphine. 134) When is screening or Group B Streptococcus (GBS) in
the pregnant emale recommended?
See Chapter 12: General Note, at the start of the chapter. A) Between 30 and 32 weeks o gestation.
B) Between 32 and 34 weeks o gestation.
129) O the ollowing options, the best initial pharmacologic C) Between 35 and 37 weeks o gestation.
therapy or bromyalgia pain is: D) Intrapartum only.
A) Duloxetine.
B) Ibupro en. See question 15.1.8.
C) Nortriptyline.
D) ramadol. 135) Which o the ollowing is the most appropriate treat-
ment o chronic menorrhagia in a 42-year-old emale hyper-
See question 11.8.2. tensive smoker?
A) Low androgenic progesterone oral contraceptive.
130) Which o the ollowing should NO go into the calcu- B) Progesterone IUD (e.g., Mirena or Skyla).
lus o what antibiotic to use or a patient with community C) Copper IUD (Paraguard).
acquired pneumonia? D) Low estrogen oral contraceptive.
A) Appearance o in ltrate on chest x-ray (lobar vs. “atypical”).
B) Comorbid medical conditions. See questions 15.8.2: Helpful ip and 15.9.4.
752 FAMil Y MEDiCin E ExAMin At io n & Bo ARD REViEW

136) You are called to see a partner’s patient who is G3P2 at D) Hypersexuality.
32 weeks gestation and is having regular contractions. A er E) Insomnia.
monitoring and an examination, you suspect that she is in
labor. T e SINGLE most important step, and the rst step to See question 23.1.8.
take, at this point is:
A) Administration o corticosteroids to hasten etal lung matu- 142) A 12-month-old emale presents to the emergency
ration. department a er 24 hours o vomiting, diarrhea, and ever.
B) Administration a tocolytic such as terbutaline. She is lethargic, tachycardic, and hypotensive with poor skin
C) Insertion o a cervical cerclage to delay delivery. turgor. What is the most appropriate initial method o pro-
D) Antibiotic therapy (ampicillin) rom now until delivery. viding her uids?
A) Hal -normal saline, 20 mL/kg bolus.
See questions 15.4.2 and 15.4.4. B) Hal -normal saline with 5% dextrose, 10 mL/kg bolus.
C) Normal saline, 100 mL/kg bolus.
137) T is etal tracing, shown in Figure 30-4, is:
D) Normal saline, 20 mL/kg bolus.
A) Reassuring.
B) Worrisome.
See question 13.3.3.
C) An indication or immediate C-section.
D) An indication or the addition o Pitocin.
143) A 15-month-old male is brought to the clinic by his par-
See Chapter 15: able 15-2. ents. Like everyone else at his home, he has had some rhinor-
rhea and diarrhea in the last week. However, 4 hours ago the
138) Which o the ollowing must be present to make the patient developed episodes o inconsolable crying accompa-
diagnosis o pelvic in ammatory disease? nied by lying in “the etal position” or 15 to 20 minutes at a
A) Adnexal pain. time. At rst his parents were puzzled but not too worried, but
B) Elevated WBC count. these episodes have occurred six to seven times now and he has
C) Elevated CRP. become more lethargic. Finally, he had a bloody bowel move-
D) emperature o > 38°C. ment, so they decided to bring him in to the of ce. On examina-
tion, he is a ebrile, lethargic but arousable, and well hydrated.
See question 15.13.2: Helpful ip. His abdomen appears benign. What is your leading diagnosis?
A) Cholecystitis.
139) A NEGA IVE etal bronectin suggests that: B) Gastroenteritis.
A) T e etal lungs are immature. C) Intussusception.
B) T ere is little likelihood that the patient will deliver within D) Volvulus.
the next 2 weeks.
C) T ere is an amniotic uid leak. See question 13.4.1.
D) Is not help ul. A positive bronectin suggests imminent
delivery. 144) Which o the ollowing screening tests is recommended
See question 15.4.1. universally or all children in the United States?
A) Eye examination at 24 months.
140) Which o the ollowing is true about tocolytics or pre- B) Hearing testing at 1 or 2 days old.
mature labor? C) Hemoglobin at 12 months.
A) T ey are e ective at stopping labor and can arrest labor or D) Lead level at 12 months.
an average o 1 week.
B) T ey have very ew side e ects and should be used routinely See Chapter 13 Quick Quiz: Congenital Infections, Helpful ip.
in premature labor.
C) T eir use is mostly to buy time or antenatal steroids to work 145) During a visit or a physical examination, you note that
or those pregnancies between 24 and 34 weeks. this 33-year-old emale has checked “depression” on her health-
D) erbutaline is the only FDA-approved tocolytic. screening questionnaire. You ask her more about this and nd
that she has elt that her mood has been depressed or 15 years
See question 15.4.3: Helpful ip. or more. She generally eels atigued and has low sel -esteem,
but she denies suicidal ideation and eelings o guilt and worth-
141) As you start a patient on uoxetine or depression, you lessness. What diagnosis best characterizes her symptoms?
counsel him about the adverse e ects o this class o medi- A) Avoidant personality disorder.
cations. All o the ollowing are common adverse e ects o B) Borderline personality disorder.
SSRIs EXCEP : C) Persistent depressive disorder
A) Anxiety. D) Major depressive disorder.
B) Diarrhea.
C) Headaches. See Chapter 23 Quick Quiz: Sad All the ime.
1
2
2
1
1
1
9
6
3 0
4
1
8
5
2
0
0
0 0
0
0
0
0
0 2
2
1
1
1 9
6
3
4
1
8
5
2 0
0
0
0
0
0
0
0 2
2
1
1
1
9
6
3 4
1
8
5
2
0
0
0 0
0
0
0
0
P
0
M
R 8
S
U
E
D
E :
D
T
F
L 5
P
D
L A
M
S
F
0
A S
E
G
P M
E S
1 2
R /
E 0 1
M / 0
A 3
I
N 3 I
c
N m
G /
m i
n P
M
T
R S
U
E
B
E
D
p
E
O D
T
F
L
/
M
H
D
L
P A
P
M
S
F P
S
A
E G E S R E M
A I
N I
N G P
M
T
R S
U
E
B
E
D
p
E
O
D T
F
L
/
M
H
D
L P
A
M
S
F
P P
S
A
E G E S R E M
A I
N I
N G
2 2 2
T B
E
p
O /
M
H P P
6
4
2
8 0
0
0
0
0 1
6
4
2
8 0
0
0
0
0
0 0 1
6
4
2
8 0
0
0
0
0
0 0
0 8 8 0 8 0 6 3 0 8 7 0 8 0 6 4 0 8 6 0 8 0 6 5
F I
G U R
E 3 0 - 4 .
753
754 FAMil Y MEDiCin E ExAMin At io n & Bo ARD REViEW

146) Which o the ollowing symptoms would be LEAS 151) Because __________ is the major unmeasured anion
likely to be due to a panic attack and there ore would in the serum, the anion gap needs to be adjusted or
prompt urther investigation in a patient with known panic the ___________ level in the serum.
disorder? A) Albumin.
A) Chest pain. B) Calcium.
B) Dyspnea. C) Phosphate.
C) Palpitations. D) otal protein.
D) Syncope. E) Chloride.

See question 23.6.4. See question 1.4.2.

152) Your patient has just returned rom Mexico where she
147) A 23-year-old male presents with symptoms sugges-
sustained an ankle bite rom a suspect dog. Rabies vaccine
tive o panic attacks, an underlying anxiety disorder, and
and immunoglobulin are not likely to be e ective a er:
possibly panic disorder. You re er him or counseling ser-
A) 10 days.
vices and recommend starting a medication. Which o
B) 20 days.
the ollowing drugs is LEAS e ective or treating panic
C) 30 days.
disorder?
D) 90 days.
A) Bupropion.
B) Nortriptyline. See question 1.6.2.
C) Sertraline.
D) Venla axine. 153) Dislocation o which o the ollowing joints is associated
with the highest risk o arterial injury?
See question 23.6.8. A) Wrist.
B) Elbow.
148) All o the ollowing are part o the CHA2DS2-VASC C) Shoulder.
score EXCEP : D) Knee.
A) Age. E) Ankle.
B) Sex/Gender.
See Chapter 1: Quick Quiz: Orthopedic Emergencies.
C) History o stroke, thromboembolic disease, etc.
D) Active cancer. 154) T e Women’s Health Initiative revealed which o the ol-
E) Heart ailure. lowing statistically signi cant correlations?
A) Combined estrogen/progesterone therapy reduced breast
See Chapter 2: able 2-6A. cancer risk.
B) Combined estrogen/progesterone therapy increased breast
149) According to the USPS F and CDC, hepatitis C cancer risk.
screening is recommended or all o the ollowing groups C) Estrogen monotherapy did not a ect breast cancer risk.
EXCEP : D) Estrogen monotherapy increased breast cancer risk.
A) All individuals born between 1955 and 1975.
B) Hemodialysis patients. See question 15.14.3.
C) Patients with a history o incarceration.
D) Users o illicit injection drugs. 155) A 2-year-old child presents having missed his routine
E) Migrants rom an endemic area (e.g., Southeast Asia). vaccines at 12 and 15 months. T e patient should:
A) Restart all vaccines.
See question 7.10.3: Helpful ip. B) Spread out the interval or his remaining vaccines to reduce
the risk o side e ects.
150) Which o the ollowing patients does NO need a head C) Receive all o his missed doses o vaccines right now.
C scan per the PECARN rules? D) Receive the vaccines due at age 12 months and continue on
A) A child with a GCS o 14 and no other symptoms or signs o the “catch-up” schedule.
intracranial injury. See question 13.2.6: Helpful ip.
B) A 3-month-old with an occipital bruise a er a all o 3 .
C) A 3-year-old with an occipital bruise and a all o 3 . 156) A 12-month-old emale in ant presents or her routine
D) A 1-year-old child with a palpable skull racture who is oth- well-child examination. She has some nasal congestion and
erwise neurologically intact. her temperature is 38°C. You should:
E) A 1-year-old child with a scalp hematoma. A) Administer her routine 12-month vaccines.
B) Administer hal her routine 12-month vaccines now and
See Chapter 1: able 1-6. hal in 2 weeks.
CHAPTER 30 • Fin Al ExAMin At io n 755

C) Have her return or vaccines when she is a ebrile or 161) A 30-year-old male is ound unconscious, slumped in a
24 hours. chair outside the ED. You arouse him brie y with a sternal
D) Never give her any more vaccines as she clearly has a ebrile rub. He has pinpoint pupils and a respiratory rate o 8 per
reaction just being close to them. minute. T e drug most likely to causing his current symp-
toms is:
See question 13.2.6. A) Methamphetamine.
B) Cocaine.
C) Heroin.
157) A 20-year-old emale with asthma presents to the ED
D) Ecstacy.
wheezing and dyspneic. Within 5 minutes o a treatment with
E) Alcohol.
nebulized albuterol, her wheezing improves but she still eels
dyspneic. T e most appropriate next step is:
See Chapter 1: able 1-1.
A) Prepare or imminent intubation.
B) Administer IV magnesium.
162) You admit a 45-year-old male with cirrhosis or alcohol
C) Administer IV ce riaxone.
withdrawal. He meets criteria or complicated withdrawal.
D) Administer another dose o nebulized albuterol.
Which o the ollowing medications will be sa est and most
E) Discharge her with a prescription or cough syrup.
e ective at treating his withdrawal symptoms?
A) Chlordiazepoxide.
See question 1.14.4.
B) Clonidine.
C) Lorazepam.
158) T e diagnosis o chronic obstructive pulmonary disease D) Metoprolol.
is secured by:
A) Cough and sputum production in a smoker. See question 25.2.7.
B) wo or three o the ollowing symptoms over the last
6 months: cough, dyspnea, sputum production. 163) You are admitting an 88-year-old male to a care acility
C) FEV1/FVC ratio o < 0.70. with hospice due to declining overall health. In the last year,
D) FEV1 < 70% predicted. he has started to require total unctional dependence in most
E) FVC > FEV1. activities o daily living. He has moderate-to-severe dementia,
ischemic cardiomyopathy and heart ailure with a le ventric-
See question 3.3.3. ular ejection raction o 20%. When he was healthier, he had
an implanted cardiac de brillator (ICD) placed, which has
red once a year ago and caused him signi cant pain and dis-
159) A 16-year-old male with asthma and seasonal allergies
tress. His daughter is his durable power o attorney and agrees
presents or ollow-up. His only asthma medication is an
with the patient that hospitalization, procedures and CPR
albuterol inhaler. In the past ew months, he has needed the
are not desired. He has made these same goals clear in past
medication more and more, and is now using his albuterol
conversations with other care providers, documented in the
inhaler one to two times per day. T e most appropriate next
medical record. When his daughter asks about his ICD, you
step is to:
recommend which o the ollowing?
A) Add a low-dose inhaled corticosteroid (e.g., triamcinolone)
A) Deactivate the ICD.
twice daily.
B) Change the settings so the ICD only res or ventricular
B) Add a long-acting bronchodilator (e.g., salmeterol) twice
tachycardia.
daily.
C) Surgically remove the ICD.
C) Schedule his albuterol inhaler 4 times per day.
D) Leave the ICD as it is.
D) Add a low-dose systemic steroid (e.g., prednisone) daily.
E) Add a leukotriene inhibitor (e.g., montelukast) daily.
See question 27.1.1.
See question 3.3.6.
164) A 58-year-old male tobacco connoisseur presents or
increasing cough, dyspnea and sputum production. He is a e-
160) According to the USPS F, the most appropriate modal- brile. Oxygen saturation on ambient air is 88%. Lung examina-
ity and requency or lung cancer screening is: tion demonstrates poor air movement and di use expiratory
A) Posteroanterior and lateral chest radiographs annually. wheezing. His chest x-ray is consistent with COPD and shows
B) Low-dose C o the chest annually. no in ltrates. You decide to admit him. Which o the ollow-
C) Low-dose C o the chest every 3 years. ing drug regimens would be the best choice or this patient?
D) Sputum cytology annually. A) Nebulized albuterol every 4 hours as needed and IV methyl-
E) None is recommended as the evidence is insu cient. prednisolone 125 mg daily.
B) iotropium inhaler 1 pu daily, levo oxacin 500 mg PO
See question 3.11.6. daily, and nebulized albuterol every 4 hours as needed.
756 FAMil Y MEDiCin E ExAMin At io n & Bo ARD REViEW

C) Prednisone 50 mg PO daily, doxycycline 100 mg PO BID, in ection. She has had three episodes o otitis media that you
and nebulized albuterol every 4 hours as needed. have treated over the last 6 months. Her hearing is dimin-
D) Morphine 5 to 10 mg IV every 2 hours to palliate his breath- ished, per parents, and she indeed tests positive or hearing
lessness. loss bilaterally. On examination, you note bulging tympanic
membranes with clear, bubbly uid in the middle ears, but no
See question 3.4.1: Helpful ip. signs o in ammation. T e intervention most likely to help
this patient improve her hearing and prevent another in ec-
165) A 35-year-old male presents or ollow-up, a er an tion is:
admission or pneumonia. Blood cultures grew Streptococ- A) Eustachian tube exercises.
cus pneumoniae. Over the past 10 years, he reports having B) ympanostomy tubes.
several bouts o pneumonia and numerous sinus in ections. C) Prolonged antibiotics.
Also, he has developed an autoimmune hemolytic anemia. D) Nasal steroids.
What is the most use ul test in making the diagnosis o an E) Systemic decongestants.
immune de ciency in this patient?
A) Complement levels. See question 20.1.9.
B) Immunoglobulin levels.
C) CBC and di erential. 169) Which o the ollowing ndings is diagnostic o acute
D) Bone marrow biopsy. otitis media?
E) Lymph node biopsy. A) Child pulling at ears and ussing in the examination room.
B) Child with ever and injected tympanic membrane on exam-
See Chapter 4: able 4-2. ination.
C) Child with ever and an injected tympanic membrane that
166) You are evaluating a 70-year-old male or abdominal does not move when ear canal is insuf ated.
pain that you suspect may be due to mesenteric ischemia. You D) Child with history o acute otitis media whose tympanic
believe the best test would be a C angiogram o his abdo- membrane is clear but does not move when ear canal is
men, but he has a contrast allergy. When you inquire about insuf ated.
the allergy, he reports rash and shortness o breath. T e most
appropriate next step in the care o this patient is to: See questions 20.1.1 and 20.1.3.
A) Per orm a non-contrast C scan o the abdomen and pelvis.
B) Per orm a contrast-enhanced C scan o the abdomen and 170) A 72-year-old male presents with hearing loss. He has
pelvis and pre-treat the patient with prednisone and diphen- no other symptoms. He spent much o his li e working in a
hydramine. machine shop a er he was discharged rom the army where
C) Per orm a contrast-enhanced C scan o the abdomen he was assigned to an artillery unit. His real passion has been
and pelvis and pre-treat with IV normal saline boluses and music: he continues to serve as a back-up vocalist and bass
N-acetylcysteine. guitar player in a rock band. On examination, he has normal
D) Consult a vascular surgeon or an interventional radiologist ears to inspection but high requency hearing loss. Which o
or an angiogram. the ollowing is the most likely diagnosis?
E) Consult a surgeon or an exploratory laparotomy. A) Presbycusis.
B) Otosclerosis.
See Chapter 4: Quick Quiz: Contrast Allergy. C) Acoustic neuroma.
D) Ménière disease.
167) A 12-year-old male presents with his mother or con- E) Otitis media with e usion.
cerns about diplopia, which began two days ago. On exami-
nation, he has normal visual acuity when each eye is tested See question 20.4.2.
individually, normal pupillary re exes, and esotropia. What
is the best next step to take in the care o this patient? 171) A 40-year-old obese emale walks into the of ce request-
A) Reassurance that this is a normal, age-related change that ing a re ll on her oral contraceptives and asks to be seen or
will improve with time. acute onset o shortness o breath. She reports that she just
B) Re er to an optometrist or corrective lenses. got back rom a 3-week road trip when she developed dys-
C) Re er to a neurologist. pnea. She wonders i it could be related to her smoking. On
D) Order an MRI o the brain. examination, she is tachypneic and mildly tachycardic with
E) Order an EEG. an oxygen saturation o 92%. She has a tender, swollen lower
leg on the le . Her lungs are clear to auscultation. T e most
See question 19.1.4. appropriate next step in the care o this patient is to:
A) Order a d-dimer.
168) A 3-year-old emale returns or continued problems B) Prescribe an albuterol inhaler.
with hearing. Her parents are concerned about another ear C) Order a contrast-enhanced chest C scan.
CHAPTER 30 • Fin Al ExAMin At io n 757

D) Prescribe a course o antibiotics. D) Contact dermatitis.


E) Order pulmonary unction testing. E) Necrobiosis lipoidica.

See question 3.5.6. See question 17.11.1.

172) A 65-year-old emale presents or ollow up a er hos- 176) All o the ollowing are components o the “ emale ath-
pital discharge 5 days ago. She underwent laparoscopic lete triad” EXCEP :
cholecystectomy that was complicated by intra-abdominal A) Disordered eating.
hemorrhage and repeated surgical exploration. She was B) Recurrent stress ractures.
in the hospital or 10 days. Now, she complains o cough- C) Menstrual dys unction.
ing ts that hurt her surgical wounds. She produces yellow D) Altered bone mineral density.
sputum. She is currently ebrile, tachypneic and tachycardic
on examination. Chest x-ray shows a right middle lobe in l- See question 14.2.2.
trate. You decide to admit her and order the ollowing anti-
biotic regimen: 177) For adolescent athletes, what is the number one cause o
A) Ce epime and vancomycin. sudden cardiac death?
B) Ce riaxone and azithromycin. A) Chest trauma (e.g., baseball hit to the chest).
C) Linezolid. B) Anomalous coronary artery.
D) Clindamycin and metronidazole. C) Hypertrophic cardiomyopathy.
E) Doxycycline. D) Patent oramen ovale.

See question 3.12.7. See Chapter 14 Quick Quiz: Adolescent Athletes.

173) A 60-year-old male presents to discuss lung cancer 178) A 29-year-old male presents with symptoms o atigue,
screening via low-dose C scan. He has smoked 1 pack- weight loss, palpitations and anxiety. On examination, you
per-day o cigarettes or 40 years. Consistent with USPS F note tachycardia, lid lag, ne tremor and di usely enlarged
recommendations, you advise him to undergo annual screen- thyroid gland. Suspecting a hyperthyroid state, you order a
ing with C until he has quit smoking or: SH and ree thyroxine ( 4). T e SH is below the normal
A) 1 year. range but the ree 4 is normal. What is the next step in the
B) 5 years. management o this patient?
C) 15 years. A) Re er to an endocrinologist who can see him in 3 to
D) T e rest o his li e. T ere is no end time identi ed or ormer 6 months.
smokers. B) Check a calcium level.
C) Check a thyrotropin releasing hormone level.
See question 3.11.6. D) Check a ree tri-iodothyronine ( 3) level.
E) Send or thyroid biopsy.
174) Which o the ollowing medications is/are associated
with urticaria? See Chapter 10 Quick Quiz: T yroid ests.
A) Aspirin.
B) Lisinopril. 179) T yroid storm is diagnosed by:
C) Morphine. A) History and physical examination ndings.
D) All o the above. B) Elevated ree 3 level.
E) None o the above. C) Free 4 level 10 times the upper limit o normal.
D) SH level immeasurably low.
See question 17.10.5: Helpful ip. E) An endocrinologist, pre erably a really old one with a long
white beard.
175) A 45-year-old emale with type 2 diabetes presents
or ollow-up. On examination, she has two brownish-red See question 10.2.6: Helpful ip.
patches with yellowish areas through the center on her lower
legs. T e center o the rash appears shiny and has telangiec- 180) You are seeing a 75-year-old emale with progressive
tasia. When asked, she states the rash has been present or at memory impairment. In order to diagnose her with dementia
least a year and is mildly tender i touched. It is not pruritic. according to DSM-IV, she must demonstrate both memory
O the ollowing, what is the most likely diagnosis? impairment AND any one o the ollowing EXCEP :
A) Acanthosis nigricans. A) Apraxia.
B) Atopic dermatitis. B) Anosmia.
C) Cellulitis. C) Aphasia.
758 FAMil Y MEDiCin E ExAMin At io n & Bo ARD REViEW

D) Agnosia. C) Mixed aerobic and anaerobic bacteria.


E) Impairment in executive unctioning. D) Yersinia pestis.

See questions 21.5.1 and 21.5.3. See question 22.1.2.

181) Regarding the treatment o hepatitis C or genotype 186) Sepsis survival is improved by:
1 patients, which o the ollowing therapies has the highest A) Early central line placement or hemodynamic monitoring.
cure rate? B) Early recognition with aggressive uid resuscitation and
A) Ledipasvir/so osbuvir (Harvoni). antibiotic administration.
B) Ribavirin. C) Speci c protocols guided by central venous pressures.
C) Inter eron alpha. D) Early intubation and ventilation.
D) Pegylated inter eron alpha. E) Early palliative care consultation.
E) Inter eron alpha + ribavirin.
See question 22.1.5: Helpful ip.
See question 7.10.6.
187) You are seeing a 30-year-old male in the ED. He has
182) According to the 2015 USPS F guideline, which o the
survived a motor vehicle collision in which the driver o the
ollowing patients should be screened or diabetes mellitus?
other car was killed. He has multiple traumatic injuries but
A) A 25-year-old emale whose mother is diabetic.
is alert—in act, he’s screaming in pain. His blood pressure is
B) A 40-year-old emale whose BMI is 29 kg/m 2.
88/56 mm Hg. What is the best option or managing his pain?
C) A 40-year-old emale whose average blood pressure over the
A) A lollipop and reassurance that the trauma team is on the
last three visits is 138/88 mm Hg.
way.
D) A 25-year-old emale who really, really likes so drinks.
B) IV morphine.
See question 10.10.1: Helpful ip. C) Oral morphine liquid.
D) IV entanyl.
183) You have diagnosed a 10-year-old child with immune E) IV ketorolac.
thromobocytopenic purpura (I P), and his platelet count is
currently 10,000/µ L. You are considering therapy to improve See question 22.2.1.
this patient’s platelet count. All o the ollowing are treat-
ments or I P EXCEP : 188) A 65-year-old emale presents with 6 hours o abdomi-
A) Platelet trans usion. nal pain. She also notes nausea but has not had emesis. She
B) Corticosteroids. has not moved her bowels today. She has a history o abdomi-
C) IVIG. nal surgeries, including hysterectomy, appendectomy, and
D) Splenectomy. cholecystectomy—all or benign reasons. Her vital signs are
E) Rho(D) immunoglobulin (i he is Rh positive). normal. Her abdomen is protuberant, tympanic to percus-
sion, and mildly tender di usely. Her bowel sounds are hypo-
See question 6.2.2. active. You suspect a partial small bowel obstruction due to
adhesions. Your admission orders include all o the ollowing
184) You are seeing a 45-year-old male with elevated liver EXCEP :
transaminases. His serum bilirubin, alkaline phosphatase, A) IV uids.
and GG are normal. He does not drink alcohol. In your eval- B) Antiemetics.
uation o his liver disorder, you nd that a liver ultrasound is C) NPO status.
unremarkable or anatomic abnormalities, serologic studies D) NG tube to continuous suction.
or hepatitis A, B, and C are negative or in ection, and his E) Analgesic medication.
erritin level is 725 ng/mL (upper limit o normal 300 ng/mL).
What is the pre erred next step in diagnosing his disease? See question 22.4.3: Helpful ip.
A) Bone marrow biopsy.
B) Liver biopsy. 189) A er weight loss surgery, patients are at risk o develop-
C) Hemochromatosis gene testing. ing nutritional de ciencies. Which o the ollowing de cien-
D) Echocardiogram. cies are patients NO likely to experience?
E) Diagnostic and therapeutic trial o phlebotomy. A) Protein de ciency.
B) Vitamin B1 (thiamine) de ciency.
See question 6.18.3: Helpful ip. C) Vitamin B12 de ciency.
D) Copper de ciency.
185) Fournier gangrene is caused by in ection with: E) Zinc de ciency.
A) Virulent, esh-eating viruses.
B) Methicillin-resistant Staphylococcus aureus. See question 22.5.4: Helpful ip.
CHAPTER 30 • Fin Al ExAMin At io n 759

190) By de nition, external hemorrhoids are located: D) Ultrasound o the right lower quadrant.
A) Proximal to the dentate line. E) All o the above have equivalent sensitivity or appendicitis.
B) Distal to the dentate line.
C) Distal to the anal verge. See question 22.9.4.
D) In the gluteal cle .
E) Proximal to the hospital administration. 195) A hysterical mother runs into the ED with her 2-year-old
daughter in her arms, screaming, “She got scalded!” A er a
See question 22.6.1. moment she is able to recount the story. T e child was by the
stove playing when she pulled on a pot handle and dumped
191) A 55-year-old emale presents with concern or a new hot water on hersel . You nd that the dorsum o her right
mass she palpated in her le breast. She rst noticed it a hand and most o her dorsal right arm are covered in second-
month ago and it has not changed. Your examination reveals and third-degree burns. Using the Parkland ormula, you
a 2-cm rm mass in the upper outer quadrant o the le estimate that she has burned:
breast. T e examination is otherwise normal. You nd that a A) 1% o her body sur ace area.
screening mammogram, done 6 months ago, was completely B) 5% o her body sur ace area.
negative (BIRADS 1). T e best next step in the management C) 10% o her body sur ace area.
o this patient is: D) None o the above. T e Parkland ormula should not be used
A) Reassurance, as her mammogram was normal. in children.
B) Repeat a mammogram in 6 months.
C) Have her return in 6 months or a ollow-up breast examina- See question 22.11.4: Helpful ip.
tion.
196) A new test has been developed to detect pheochromocy-
D) Re er or biopsy o the mass.
toma (not really . . . just pretend). T e test has a very high sen-
E) All o the above are equally appropriate.
sitivity but a very low speci city or pheochromocytoma. T e
prevalence o pheochromocytoma in the population is low.
See question 22.7.3.
I this new test is applied to the general population, which o
the ollowing is likely to occur?
192) A surgical colleague consults you on an 80-year-old A) T e speci city will increase.
emale who broke her emur a er alling. T e patient under- B) T e sensitivity will increase.
went an uncomplicated open reduction and internal xation. C) T e positive predictive value will be low.
It is now postoperative day 4 and she has developed a ever o D) T e negative predictive value will be low.
39°C. Potential causes o postoperative ever in this patient
include all o the ollowing EXCEP : See question 28.6.2: Helpful ip.
A) Atelectasis.
B) Wound in ection. 197) You are seeing 23-year-old emale or a routine physi-
C) Venous thrombosis. cal examination. She tells you that she has a new male part-
D) Pneumonia. ner who has used injectable drugs in the past and has shared
E) Drug-induced ever. needles. His HIV status is not known. T ey do not always use
condoms when sexually active. In addition to recommending
See Chapter 22: able 22-3. consistent condom use, you also recommend pre-exposure
prophylaxis with:
193) A patient with a history o heparin induced thrombocy- A) eno ovir.
topenia and thrombosis syndrome CAN have: B) eno ovir/Emtricitabine ( ruvada).
A) Low–molecular-weight heparin or any reason. C) E avirenz (Sustiva).
B) Heparin or cardiopulmonary bypass during cardiac sur- D) eno ovir/Emtricitabine/E avirenz (Atripla).
gery. E) None o the above.
C) Heparin or low–molecular-weight heparin or prevention o
venous thromboembolism. See question 9.4.2.
D) No heparin products or the rest o his or her li e.
198) A 5-year-old patient presents with “leg pain” that you
See question 22.8.8. determine is originating rom the right hip joint. An x-ray
demonstrates a density at the right emoral head. With your
194) Which o the ollowing tests has the highest sensitivity Spidey senses tingling, you order an MRI which shows osteo-
(i.e., misses the ewest) or appendicitis? necrosis o the emoral head. T e best next step in the care o
A) CBC. this patient is:
B) CRP. A) Acetaminophen and physical therapy.
C) C scan o the abdomen and pelvis. B) Re erral to orthopedic surgery or a hip replacement.
760 FAMil Y MEDiCin E ExAMin At io n & Bo ARD REViEW

C) Systemic steroids (e.g., prednisone). C) Midsha 5th metatarsal racture.


D) Steroid injection o the hip joint. D) 5th metatarsophalangeal joint dislocation.

See question 12.2.3. See question 12.19.1.

200) You are watching a really bad grade B movie. Who do


199) A 35-year-old male presents or oot pain one day a er you think will win?
an injury. He was playing soccer when he twisted his right A) Mothra.
oot and ankle and ell. He has trouble bearing weight on the B) King Ghidorah.
oot due to pain. You order an x-ray o his oot and ankle. T e C) Biollante.
radiologist calls to tell you he has a Jones racture. When you D) Godzilla.
look at the x-ray, you expect to nd a: E) Mechagodzilla.
A) Proximal 5th metatarsal racture.
B) Distal 5th metatarsal racture. Never bet against Godzilla.
Index
Note: Page number ollowed by and t indicates gure and table respectively.

A treatment o , 491 constitutional delay, 396–397


A–a gradient, 109 types o , 490 depression in, 401
ABCD2 criteria, 500t Acquired immune de ciency syndrome (AIDS), 260. eating disorders in, 398
ABCDE algorithm, or primary survey, 28 See also HIV/AIDS emale athlete triad in, 397–398
ABCs (airway, breathing, and circulation), 3 Acral lentiginous melanoma, 477 hypertension in, 400
Abdominal aortic aneurysm (AAA) Acrocyanosis, 327 leg pain in, 397
blood pressure control in, drugs or, 66 Acromegaly, 300–301 mortality or, causes o , 400–401
dissection o , 66 therapy or, 301 obesity in, 403–404
in older men, 593 Acromioclavicular (AC) sprain, 349, 350 substance use, 401
repair o , 65–66 Actinic keratoses (AKs), 494 sudden cardiac death in, 399
screening or, 66, 593 Active euthanasia, 689 vaccine recommendations or, 402–403
ultrasound in, 65 Activities o daily living (ADL), 591 vitamin D intake, 399
Abdominal pain, 227 Acupressure, 669 weight loss in, 404 (see also Obesity)
ovarian torsion and, 28 Acute angle closure glaucoma, 528, 528 Adrenal adenoma, and hyperaldosteronism, 76
pelvic in ammatory disease and, 436–437 diagnosis o , 528 Adrenal hyperplasia, 299
and pregnancy test, 25 drugs used or, 529, 529t Adrenal insu ciency, 283–284
ABI (ankle-brachial index), 70, 495 presentation o , 529 diagnosis o , 284
Abili y. See Aripiprazole risk actor or, 528–529 primary, 284–285
Abortion treatment o , 529–530 secondary, 285, 286
complete, 431 Acute calculous cholecystitis, 616 treatment o , 285
elective, 695–696 Acute chest syndrome (ACS), 18, 186 Advance directives, 689
ethical issues, 695–696 exchange trans usion in, 18 AED. See Antiepileptic drug (AED)
missed, 431 management o , 18 Aeroallergen skin testing, 131
spontaneous, 431–432 Acute interstitial nephritis, 170 Aerophagia, 202
threatened, 431 development o , 170t Age-related macular degeneration (AMD), 532, 533
ABPA. See Allergic bronchopulmonary aspergillosis drugs associated with, 170t Aging
(ABPA) symptoms and laboratory ndings in, 170t and Alzheimer disease, 584
Abrasion, 475t treatment o , 171 sex hormone changes associated with, 572
Abscess, 475t Acute kidney injury, 156 Agitated patient
dental, 564 Acute myocardial in arction (AMI). See Myocardial and violence, 651t
lung, 111 in arction (MI) ways to calm, 651t
peritonsillar, 566, 567 Acute otitis media (AOM), 548 Agoraphobia, 634
retropharyngeal, 38 antibiotics or, 549–550 AIDS. See Acquired immune de ciency syndrome (AIDS)
Absence seizures, 523 causes o , 549 Air leakage, 623
Absolute risk reduction (ARR), 715, 717 ndings, 548–549 AKs. See Actinic keratoses (AKs)
Abuse treatment o , 549 Albiglutide, 292
child, 374 Acute sequestration syndrome, 19 Albumin, 223, 599, 619
elder, 589 Acute tubular necrosis (A N), 155, 158 detection o , 151
inhalant, 684–685 acute kidney injury due to, 159 Albumin/creatinine ratio, 140
skeletal survey or, 374 cause o death in, 159 Albuminuria, moderately increased, 140
Acamprosate, 682 causes o , 158 Albuterol, 36, 612
Acanthosis nigricans, 494–495, 495 , 663 treatment o , 158 or asthma exacerbation, 22
causes o , 495 Acyclovir, 150 and tachycardia, 22
diseases associated with, 663 AD. See Alzheimer disease (AD) Alcoholic hepatitis, 190
Accelerated junctional rhythm, 52 Adalimumab, 307 Alcoholics Anonymous (AA), 692, 693
Accommodation, 526 or in ammatory bowel disease, 211 Alcoholics, children o , 681
ACE inhibitors. See Angiotensin-converting enzyme Addisonian crisis, 285 Alcohol, metabolism o , 8
(ACE) inhibitors Adenocarcinoma, 553–554 Alcohol-related liver disease, 227
Acetaminophen, 334, 365, 549, 573, 587, 705 Adenoidectomy, 550 Alcohol use, 553, 590. See also Alcohol withdrawal
dose or older adults, 593 Adenomatous polyps, 209 CAGE questionnaire, 674
and methemoglobinemia, 191 colonoscopy in, 209 complication o , 676
or pain control, 346, 587 Adenosine, 49, 54 disorders with, 677
or rotator cu tendinosis, 351 or PSV , 83 evaluation in, 674
Acetazolamide, 542 Adenoviral conjunctivitis, 537 increased gamma-glutamyltrans erase (GG ) in, 675
Acetic acid ear drops, 551 ADHD. See Attention-de cit/hyperactivity disorder and peripheral neuropathy, 504, 674
N-acetylcysteine, 2, 142 (ADHD) tolerance and, 674–675
Achalasia, 204 Adhesive capsulitis, 351 Alcohol withdrawal, 577, 673
Achilles tendon rupture, 355, 358 clinical course or, 352 carbamazepine in, 675
Acid–base disorder, 168 treatment o , 352 criteria or, 673
Acidosis Adjustment disorder, 636, 643 treatment o , 673–674
anion gap, 7, 7t Adolescents Aldosterone, 668
causes o , 7t athletes, 399 Alendronate, 399, 575
metabolic (see Metabolic acidosis) bacterial meningitis in, 242 Alien limb phenomenon, 517
Acne, 490 birth control options, 398–399 Alirocumab. See Praluent
medication-induced, 490 calcium intake, 399 Aliskiren, 62
neonatal, 485 causes o death in, 400–401 Alkali burns, 536
761
762 INDEX

Allergen immunotherapy, 132 Angiodysplasias, 232 Antidepressants, 509


Allergic bronchopulmonary aspergillosis (ABPA), 127 Angiotensin-converting enzyme (ACE) inhibitors adverse e ects o , 639–640
diagnosis o , 127, 127t and cough, 112 and birth de ects, 640
treatment or, 127–128 or diabetic nephropathy, 144 or dementia, 586
Allergic conjunctivitis. See also Conjunctivitis or heart ailure, 61 erectile dys unction by, 641
about, 537 or HF exacerbation, 62 in pregnancy, 642
treatment o , 538 or hypertension, 74, 291 weekly dosage, 652
Allergic rhinitis, 128, 131–132, 559 or moderately increased albuminuria, 141 and weight gain, 663t
Hansel stain o nasal mucous in, 128 in pregnancy, 427 Anti-EBV (Epstein–Barr virus) antibody, 566
treatment or, 128 or proteinuria, 151–152 Antiepileptic drug (AED), 502
Alli. See Orlistat or Raynaud phenomenon, 329 interactions with oral contraceptives, 503
Allopurinol or renal disease, 171 in pregnancy, 502–503
or gout, 314, 315 side e ects o , 77, 291 side e ects with, 502
side e ects o , 315 Angiotensin receptor blockers (ARBs), 61, 141 Anti brinolytic therapy, 174, 535
Alogliptin, 292 and cough, 75 Anti ungal therapy, 554
Alpha-blockers or hypertension, 74, 291 Antigliadin antibodies, 214
or benign prostatic hyperplasia, 456 in pregnancy, 291 Anti-glutamic acid decarboxylase (anti-GAD) antibodies,
or hypertension, 74 or Raynaud phenomenon, 329 290
side e ects o , 455 or renal protection, 141 Anti-hepatitis A antibodies (IgG and IgM), 219
sildena l and, 55 and risk o gout, 314 Antihistamines, 538, 558
and weight gain, 663t Anion gap, 7 or allergic rhinitis, 128
Alpha- etoprotein (AFP), 220 Anion gap acidosis, 7, 7t and weight gain, 663t
5-alpha-reductase inhibitors, 456 Ankle-brachial index (ABI), 70, 495 Anti-mitochondrial antibodies (AMA), 225
Alport syndrome, 164 Ankle sprains, treatment or, 356 Antiphospholipid antibody syndrome, 196, 197,
Alprazolam, 635, 675 Ankylosing spondylitis, 330 325–326
AL E (apparent li e threatening event), 366 anti- NF therapy or, 331 in pregnancy, 326
Alternative therapies, 667–668 diagnosis o , 330 treatment o , 326
Alzheimer disease (AD) management o , 330 Antipsychotics, 585, 589
end-stage, 586 Annexa, 198 in alcohol withdrawal, 674
risk actor or, 583–584 Annular arrangement, 475t atypical, 578
symptoms, 582 Anorectal manometry, 233 or schizophrenia, 655
treatment o , 584 Anorexia nervosa (AN), 648 second-generation, 658
Amantadine, 511 and bulimia nervosa, 650 and weight gain, 663t
Amaurosis ugax, 542 laboratory nding in, 648–649 Antisocial personality disorder, 653
Amblyopia, 526, 527 management o , 649 characteristics o , 653t
24-hour ambulatory blood pressure monitoring, 72–73 medical complications o , 649 and risk o suicide, 653
Ambulatory devices, 580 mortality rates in, 649–650 Antithymocyte antibodies, 512
Amelanotic melanoma, 477 subtypes o , 650 Anti-thyroglobulin antibodies, 274
Amenorrhea, 298, 451 symptoms o , 648 Antithyroid drugs, 274
causes o , 298 Anovulation, 438, 441–442 side e ects o , 274
evaluation o , 451 Anterior cruciate ligament (ACL) rupture, 353–354 Antithyroid peroxidase antibody, 274
primary, 430 diagnosis o , 354 Anti-thyrotropin receptor antibody, 274
secondary, 398, 430 Anterior epistaxis, source o , 561 Anti- NF agents, or spondyloarthropathy, 331
American Cancer Society (ACS), 573 Anterior at pad sign, 359, 359 Anti-tumor necrosis actor ( NF)-alpha antibody
American Geriatrics Society, 595 Anterior ischemic optic neuropathy, 532 therapy, 211
American Society o Anesthesiologists (ASA) physical Anterior talo bular ligament, sprain o , 355 Antiviral therapy, during in uenza outbreak, 238
status classi cation, 613t Antibiotics, 550, 600 Anuria, 156
American Urological Association (AUA) symptom score, or acne, 491 Anusol suppositories, 233
454 or acute prostatitis, 26 Anxiety disorders, 634
Aminoglycoside antibiotics, and hearing loss, 573 or bacterial overgrowth, 214 AOM. See Acute otitis media (AOM)
Amiodarone, 46, 58, 277 or bronchiectasis, 125–126 Aortic dissection, 31
in atrial brillation, 54 or community-acquired pneumonia, 121 Aortic stenosis, 71, 71t
Amitriptyline, 131, 509, 569, 663 or contact lens–related abrasions, 539 management o , 72
Amlodipine, 329 or dental in ections, 38 symptoms in, 71–72
Amniotomy, or labor induction, 416 or diverticulitis, 234 Apache score, 230
Amoxicillin, 549, 550, 565 or ebrile neonates, 393 Aphakia, 543
Amoxicillin/clavulanate, 558 or Fournier gangrene, 598 Aphthous ulcers, 561–562
Amphetamine use, 684 or gonococcal arthritis, 312 Apixaban, 109, 195, 196, 198
symptoms o , 684 or gonorrhea, 464–465 in non-valvular atrial brillation, 54
withdrawal rom, 684 or impetiginized atopic dermatitis, 479 Apley test, 355
Ampicillin, or in ection in newborn, 34 or in ection in newborn, 34 Appendectomy, 614
Ampicillin/sulbactam (Unasyn), 228 or in ective endocarditis, 246 Appendicitis, 613–614
Ampyra, 511 or Lyme arthritis, 319 symptoms, 615
AN. See Anorexia nervosa (AN) or maternal chorioamnionitis, 379 Appendix testis, torsion o , 459
Analgesic-related headaches, 508 and meibomian gland dys unction, 540 Aprepitant, 708
Analgesics or meningitis, 241 Aranesp. See Darbepoetin
or otitis media, 549 or mononucleosis, 567 ARBs. See Angiotensin receptor blockers (ARBs)
or rhinosinusitis, 558 or neonatal osteomyelitis, 393–394 Arcanobacterium haemolyticum, 38, 567, 568
Anaphylactoid reactions, to radiocontrast material, 134 or neonatal pneumonia, 392 Aripiprazole, 655, 658
Anaphylaxis, 177 or neutropenic ever, 178 , 178t Arterial blood gas, 154
Anaplastic thyroid carcinoma, 273 or otitis media, 549 Arterial insu ciency ulcers, 481
Andexanet al a. See Annexa or pelvic in ammatory disease, 437 Asacol, 211
Androgenetic alopecia, 462 or Pneumocystis jiroveci pneumonia, 262 5-ASA drugs, or in ammatory bowel disease, 211
Anemia or pneumonia in terminal dementia, 708 Ascites, due to portal hypertension, 221–222
o chronic disease, 146, 183 or spontaneous bacterial peritonitis, 223 ASCVD risk calculator, 92, 93
iron de ciency, 181–183, 286 or streptococcal pharyngitis, 565 “Ask Me 3” program, 729
microcytic, 183 or urinary tract in ection, 378 Aspergillosis, 552
pernicious, 187–188 or viral conjunctivitis, 537 Aspiration pneumonitis, 1, 610–611
Anesthesia Anticholinergic crisis, 640 Aspirin, 112, 152, 181, 534, 587, 610, 668
evaluation, 613 Anticholinergics, 564 or antiphospholipid antibody syndrome, 326
mortality rate rom, 613 in colic treatment, 384 and asthma, 102
Aneurysm Anticholinergic toxidrome, 5, 5t and colon cancer risk, 624
abdominal aortic, 65–66 Anti-citrullinated protein antibody (ACPA), 304 or diabetes, 293–294
thoracic aortic, 310 Anticonvulsants, 574 or myocardial in arction, 41
Angel dust, 686 and weight gain, 663t or preeclampsia, 424
INDEX 763

and Reye syndrome, 387 in ammatory, 329 Blood pressure. See also Hypertension
or stroke prevention, 500 lower (see Lower-back pain (LBP)) diabetes and, 52
Assisted reproductive technology, 439 Backup contraception, 410, 410t ischemic stroke and, 499
Assisted suicide, 689 Baclo en, 682 Blood trans usion, 176
Asthma Bacterial conjunctivitis, 537, 538 . See also Conjunctivitis indications or, 176t
albuterol in, 101 Bacterial in ection, in newborn, 33 reactions, 177
aspirin sensitive, 102 Bacterial meningitis Blowout racture, with entrapment o the in erior rectus,
exacerbations, 101, 102 in adolescents, 242 14–15
intermittent, 99, 101 causative organism, 241 Blunt injuries, 528
mild, 99 diagnosis o , 240–241 BN. See Bulimia nervosa (BN)
patient education in, 102 treatment or, 241 BODE index, 106
prevalence o , 98 Bacterial overgrowth syndrome, 213 Body mass index (BMI), 604
salmeterol in, 102 tests or, 214 calculation or, 622
severity classi cation and therapy, 100t Bacterial parotitis, 563 Boerhaave syndrome, 208
stepwise treatment approach to, 101 Bacterial sinusitis, 558 Bone age assessment, 396–397
testing in, 99 Bad news Bone densitometry screening, 573
triamcinolone in, 101 giving o , 709, 709t Bone marrow biopsy, 181
Asthma exacerbation, 21–22 SPIKES six-steps or breaking o , 709, 709t Bone marrow transplant, 185
discharging patient rom ED, 23 Balanitis, 465 Bone mineral density, 310, 397, 574
steroid use in, 22 Balanoposthitis, 27 estrogen and, 574
test in evaluation o , 22 Band ligation, 606 in women, 398, 575
theophylline in, 22–23 Bare metal stents (BMS), 50 Bone mineralization, 574
treatment or, 22 Bariatric surgery, 434, 604, 665 Booster phenomenon, 243
AS /platelet ratio index (APRI), 217 Barrett esophagus, 201 Borderline personality disorder, 652, 652t
Asymptomatic bacteriuria, 242–243, 587 Basal cell carcinoma, 487, 487 Borrelia mayonii, 318
Asystole, ventricular tachycardia and, 67–68 Bath salts, 686 Botrytis cinerea, 98
Atazanavir, 261 Bayes theorem, 720 Bowel ischemia and mesenteric thrombosis, 228
Atelectasis, 611 Bee sting kit, 24 Bowel obstruction, 603
Atherosclerotic cardiovascular disease (ASCVD), 92 Bee sting reaction, 23–24 closed-loop, 603
A N. See Acute tubular necrosis (A N) treatments in, 23 unctional, 603
Atomoxetine, 646 Behavior control, 589 management o , 603
Atopic dermatitis, 477 Behcet disease, 562 mechanical, 603
clinical eatures o , 478 Bell palsy, 562 open-loop, 603
management o , 478–479 Bence Jones proteins, 151 BPH. See Benign prostatic hyperplasia (BPH)
skin in ections and, 478, 479 Benign lymphadenopathy, 557 Bradycardia, re-warming or, 19
Atorvastatin, 615 Benign murmurs, o childhood, 368, 369t Bradycardia–tachycardia syndrome. See Sick sinus
Atovaquone/proguanil (Malarone), 252 Benign paroxysmal positional vertigo (BPPV), 498, 560 syndrome
Atrial brillation, 52, 53 treatment o , 560 Bradykinesia, 582
alcohol use and, 52–53 Benign prostatic hyperplasia (BPH), 454 Brain lesion, 562
anticoagulation be ore cardioversion in, 54 medical therapy or, 454–455 Brain natriuretic peptide (BNP), 59, 60
cardioversion o , 54–55 prostate cancer and, 468–469 Brainstem-evoked potentials, 555
causes o , 53 saw palmetto or, 470 Brain stem stroke, 498
diltiazem in, 54 tamsulosin in, 455 Branchial clef cysts, 564
lone, 55 and urinary retention, 27 BRCA 1 gene, 607
non-valvular, 54 Benzodiazepines, 585, 706 Breast cancer, 607–608
rate control in, 53–54 or alcohol withdrawal, 673–674 management o , 608
stroke and, 53 or dyspnea and anxiety, 700–701 prognostic indicators in, 609
war arin in, 54 in tricyclic-induced seizures, 4 risks or, 609
Atrial utter, postoperative, 613 Bereavement, 637, 638, 643 screening or, 573
Atrial septal de ect (ASD), 369 Beta-blockers soy or, 667t
Atrial tachycardia with third-degree heart block, 77, 79 or BP control in aortic dissection, 31 Breast eeding, 383
pacemaker or, 77, 80 or glaucoma, 529t bene ts o , 433
Atropine, 45 or heart ailure, 61, 64 HIV/AIDS and, 265–266
Attention-de cit/hyperactivity disorder (ADHD), or hypertension, 74 by vegan mothers, vitamin supplementation in,
644–645 overdose, 8 662
adults with, 645–646 or premature atrial contractions, 58 Breast eeding ailure jaundice, 364
behavior therapy in, 645 or sick sinus syndrome, 69 Breast lumps, 607
children with, 645 topical, 529t, 531 Breast milk, 389, 433
stimulant e ects on, 645, 645t and weight gain, 663t supplements to, 662
symptoms in, 646t Betahistine, 556 Breast milk jaundice, 364, 369
Audiogram, 555 Bexsero, 403 Breath-holding spells, 366
Augmentation o labor, 433 Bias, in research studies, 712, 713t Breath sounds, absent, on lef , 29–30
Auricular cartilage piercing, 623 Biceps tendon rupture, 349, 352 Breath urea test, 207
Auryxia. See Ferric citrate Bilateral brain lesion, 562 Bridging therapy, 56
Autism spectrum disorders, screening tool or, 375 Bilateral developmental dysplasia o hip (DDH), 337 Brie interventions, components or, 687
Autoimmune hypothyroidism, 271 Bile acid sequestrant, 94 Brilinta. See icagrelor
Autoin ation, 551 Bilevel positive airway pressure (BiPAP), 520 Bromocriptine, 299, 300
Autosomal dominant polycystic kidney disease Biliary atresia, 364 or Parkinson disease, 517
(ADPKD), 155 Biliary outlet obstruction, 227 Bronchiectasis
Autosomal-recessive polycystic kidney disease, 167 Biliary pancreatitis, 616 antibiotics in, 125–126
Avulsion racture, 335t Bilirubin, elevated, causes o , 364 etiology o , 126
o tuberosity, base f h metatarsal, 356–357, 357 Biocreep, 719 Bronchiolitis, 35, 97
Azathioprine Biological gender, 733 causative organism, 385
or in ammatory bowel disease, 211 Bipolar I disorder, 647 nebulized hypertonic saline in, 385
or myasthenia gravis, 513 Bipolar II disorder, 647 treatment in, 385
Azelastine nasal spray, 131 Birch pollen allergy, 133 Bronchitis, and hemoptysis, 114t
Azithromycin, 449, 538, 549, 559 Birth control options, 398 Bronchodilator therapy, in asthma exacerbation, 22
or travelers’ diarrhea, 252 Bishop score, 416 Bronchoscopy with endobronchial ultrasound
Bisphosphonates, 279, 574, 575 (EBUS)-guided ne-needle biopsies, 105–106
B Black cohosh, 666 Bronze diabetes, 190
Babesiosis, 251t Black patients, caring or, 730–731 Brown–Sequard syndrome, 504
Baby blues, 642 Black pigment stones, 616 Brown stones, 616
Bacille Calmette–Guerin (BCG) vaccine, 243, 244–245 Blastomyces dermatitidis, 127 Brudzinski sign, 241
Back pain Bleach, industrial, ingestion o , 20–21 Buerger disease (thromboangiitis obliterans), 327
causes o , 329 Blood dyscrasias, 543 Bulge sign, 320
764 INDEX

Bulimia nervosa (BN), 398, 650 Cardiogenic shock, 618, 618t Children
Bulla, 475t Cardiopulmonary bypass, heparin or, 612 with ADHD, 645
on skin, 380–381 Cardiovascular disease (CVD) risk, 595 breath-holding spells in, 366
Bullosis diabeticorum, 495 Carotid endarterectomy, re errals or, 500–501 depression in, 643, 643t, 644
Bullous pemphigoid, 488–489, 489 Carpal tunnel syndrome, 360 gross hematuria in, 167t
diagnosis o , 489 conditions associated with, 360 HAAR or, 266
prognosis o , 489 conservative therapy or, 360 heme positive stools in, 388
treatment o , 489 evaluation and management o , 360 HIV/AIDS in, 266
BUN/Cr ratio, 142–143, 156 symptoms o , 360 neutrophil counts in, 189
Buprenorphine, or opioid dependence, 673 Case–control study, 718 urinary tract in ection in, 377–379
Bupropion, 635 Catamenial cyclic vomiting syndrome, 235 Chlamydia in ection, in newborn, 385
adverse e ect o , 635–636 Cataplexy, 520 Chlamydia proctitis, 465
or attention-de cit/hyperactivity disorder, 646 Cataract ormation, risk actors or, 544 Chlamydia trachomatis, epididymitis by, 25
clozapine and, 681 Cataract surgery Chlamydia urethritis, 464–465
contraindication to, 636 complications or, 544 Chlordiazepoxide, 675
or depression, 629 indication or, 544 Chlorthalidone, 314
in smoking cessation, 680 opaci cation o posterior capsule, 546 or hypertension, 74
transition rom, 636 Cauda equina syndrome, 345, 522 Cholecystectomy, 617
Burkitt lymphoma, 553 decreased rectal tone in, 522 Cholecystitis, 616
Burn wound management, 618 diagnosis o , 523 initial treatment o , 617
Buspirone (BuSpar), 585t, 700 treatment in, 523 management o , 617
Cauli ower ear de ormity, 561 Cholecystoenteric stula, 616
C Cavitary lesion, on chest x-ray, 110–111, 111 Choledocholithiasis, 226
CA-125, 153, 429, 446 Cawthorne rehabilitation exercises, 560 Cholelithiasis
Cabergoline, 299, 300 CB . See Cognitive behavioral therapy (CB ) about, 605
CABG. See Coronary artery bypass graf ing (CABG) Ce dinir, 549 di erential diagnosis o , 616
CAGE questionnaire, 674 Ce epime, 123 Cholesterol stones, 616
Calcitonin, 279 Cef riaxone, 34, 392, 523, 538, 549, 559 Cholinergic, 5t
Calcium or pelvic in ammatory disease, 437 Cholinesterase inhibitors, 584
absorption, 666 Ce uroxime, 549 Chondrocalcinosis, 316
or osteoporosis prevention in adolescence, 399 Celecoxib, 42 o knee joint, 317
supplementation, 574 Celexa. See Citalopram Chondroitin, 594, 670
and vitamin D supplementation, 665 Celiac disease. See Gluten-sensitive enteropathy Chondromalacia patellae, 336
Calcium carbonate, 165 Cellulitis, 600 Chorioamnionitis, 413
Calcium channel blocker (CCB), 544 Centers or Disease Control and Prevention (CDC), 368, and antibiotics, 413
or hypertension, 74, 75 402–403 Choroidal neovascular membrane (CNVM), 533
overdose, 8 Centers or Medicare and Medicaid Services (CMS), 569 Christmas Disease, 173
Calcium chloride, 37, 282 Central line placement, 621 Chronic daily headache, 522
or beta-blocker overdose, 8 Central retinal artery occlusion, 542 Chronic granulomatous disease, 137t
Calcium disodium ED A, 185 about, 542 Chronic kidney disease (CKD), 145
Calcium gluconate, 143, 282 causes o , 543 and anemia o chronic disease, 146
in end-stage renal disease, 37 treatment o , 542, 543 stage 3, 145
Calcium oxalate stones, 149 Central retinal vein occlusion, 542, 543 stages o , 145t
Calcium pyrophosphate dehydrate crystal deposition Central venous nutrition (CVN), 230 U I in, cipro oxacin or, 146
disease (CPPD), 316 complication o , 230 Chronic lymphocytic leukemia (CLL), 192
pseudogout attacks in, 317 Central vertigo, 560t Chronic myelogenous leukemia (CML), 192–193
therapy or, 317 Cephalexin, 26, 565 Chronic obstructive pulmonary disease (COPD)
Cameron lesions, 208, 232 Cephalosporins, or impetiginized atopic dermatitis, 479 continuous low- ow oxygen in, use o , 106
Campylobacter jejuni, 514 Cerclage, 422–423 exacerbations, 104–105
Canagli ozin, 296 Cerebellar stroke, 498 hypoxemic patients with, 106
Cancer Cerebral angiography, 546 lung transplantation re erral in, 106
breast, 607–609 Cerebrospinal uid (CSF) analysis, 582, 583 noninvasive positive pressure ventilation in, 104
cervical, 267, 446 Cerebyx. See Fosphenytoin supplemental oxygen in, administration o , 104
colon, 190, 233, 623, 624 Certolizumab, or in ammatory bowel disease, 211 treatment or, 104–105, 105
endometrial, 441 Cervarix, 442 Churg–Strauss vasculitis, 127
laryngeal cancer, 553 Cervical cancer, 446 Chvostek sign, 165, 281
ovarian, 446, 573, 607 Cervical cytology, 434 Cialis, 55, 457
pain, 702 Cervical dysplasia, 435 Cilostazol, 70
prostate, 454, 468–470 Cervical intraepithelial neoplasia (CIN), 435–436 Cimetidine, or bee sting reaction, 23
testicular, 458 Cervical ripening, drugs or, 413 CIN. See Cervical intraepithelial neoplasia (CIN)
Candida albicans, 465 Cervical spine Cinacalcet, 165–166
Candida antigen injection, 484 clinical clearance, 5, 5t Cipro oxacin, or anthrax, 2
Cannabis hyperemesis syndrome, 235 radiographs or, 5 Circadian rhythm sleep–wake disorder, 656
Capsaicin cream, 594 C1-esterase inhibitor de ciency, 135 delayed sleep phase type o , 656
Capsular opaci cation, treatment or, 546 C1-esterase inhibitor replacement protein products, 135 Circumcision, 173, 459–460
Capsule endoscopy, o small bowel, 232 CHA2-DS2-VASC score, 55, 55t abnormal bleeding af er, 460
Carbamazepine, 647 and anticoagulation, 56 consequences o , 460
or alcohol withdrawal, 675 atrial brillation treatment based on, 56t indication or, 460
or trigeminal neuralgia, 522 Chalazion, 540 neonatal, 460
Carbohydrates diet, and calories, 298 Chancroid, 457–458 pain control measures or, 461
Carbonaceous material, 619 Chantix. See Varenicline Cirrhosis, portal vein thrombosis in, 224
Carbon dioxide in stomach, 619 Charcoal Cisapride, 231
Carbon monoxide poisoning, 9 contraindication to use o , 2 Cisplatin, 573
ndings with, 9 in iron overdose, 2 Citalopram, 636
hyperbaric oxygen therapy and, 9 in theophylline toxicity, 1–2 CKD. See Chronic kidney disease (CKD)
100% oxygen in, administration o , 9 Charcot oot, 358 Clarithromycin, or community-acquired pneumonia,
Carboprost tromethamine, 421 Charcot’s triad, 616 123
Carboxyhemoglobin level, in carbon monoxide Chemotherapy-induced nausea, 708 Clavicle ractures, 351
poisoning, 9 Chest pain, cardiac, 30–31, 47–48 Clavulanate, 549, 550
Carcinoembryonic antigen (CEA), 624 Chest tube management, 623 Clindamycin, 228, 564, 599, 600
Cardiac arrest, 619 Chest wound, sucking, 30 in lung abscesses, 111
Cardiac C angiogram (CC A), 52 Chest x-ray, in pneumonia, 122 Clinical Institute Withdrawal Assessment or Alcohol
Cardiac resynchronization, or heart ailure, 91 Chiari network, 64 (CIWA-Ar), 675–676
Cardiac tamponade, 51 Child abuse, 374 Clock drawing, 581
decompensated, 51 Child–Pugh classi cation, 621, 621t Clonazepam, 520
INDEX 765

Clopidogrel, in myocardial in arction, 41–42 Congestive epididymitis, 461 Creatine use


Closed-angle glaucoma, 529 treatment or, 461 adverse e ects rom, 405
Close racture, 335t Conivaptan, 162 as dietary supplement or athletes, 405
Clostridium di cile, 204, 216 Conjunctivitis, 537 Creatinine, 557
diarrhea by, 239 adenoviral, 537 Creatinine clearance, estimation o , 143
recurrence o in ection by, 240 allergic, 537, 538 Cremasteric re ex, 24
risk actors or, in nursing home patients, 240 bacterial conjunctivitis, 537, 538 Crepitus, 598
treatment o , 239–240 gonococcal, 538 CRES syndrome, 203, 328
CLO test, 207 treatment o , 537 Crigler–Najjar syndrome type I, 226
Clotrimazole, 552 viral, 537, 537 Crohn disease, 210, 606
Clozapine, and bupropion, 681 Constipation and IBD, 607
Clozari, 658 causes o , 363 pathophysiology and natural history o , 210
Club oot, 342 colon cancer and, 623 ulcerative colitis and, 211
Cluster headaches, 521 ber or, 594–595 Cromolyn sodium, 101
high- ow oxygen treatment in, 521 unctional, Rome Criteria, 594t Cross-reactivity, 133
Cobalamin (B12) de ciency, 188 in in ancy, 390 Croup, 35
Coccidiodomycosis, 129 pharmacologic approach to, 594 steeple sign in, 35
Cochrane Collaboration review, 666 risk actor or hernias, 602 Crust, 475t
Cockcrof –Gault ormula, 143 secondary causes o , 594, 595 Cryoglobulinemia, 317
Cognitive behavioral therapy (CB ), 401 Constitutional delay o growth and puberty, 396–397 Cryoprecipitate, 165, 177
or depression, 629 Contact dermatitis, 485, 496, 496 Cryotherapy, 484
or post-traumatic stress disorder, 632 Contact isolation, 2 Cryptorchidism, 458
Cognitive impairment, 581 Contact lenses, 535 Cryptostroma corticale, 98
Cohort studies, 718 Continuous positive airway pressure (CPAP), 520 Crystalloid in usion, 285
Colchicine, 51 Contraception Culturally responsive care, 726–727
or gout, 314, 315 backup, 410, 410t Culture, 732
or pericarditis, 51 emergency, 443–444 CURB-65, 121
Cold, physiologic response to, 327 methods o , 410 (see also speci c methods) Curettage, 622
Cold sores, 33 postcoital, 444 Currant jelly stools, 373
Colesevelam, 94 weight changes and, 409 Cushing disease, 283
Colestipol, 94 Contraction stress test (CS ), 432 Cushing syndrome, 282–283
Colic, 383–384 Contrast dye allergy, premedication regimen or, AC H-producing tumor and, 283
anticholinergic drugs in, 384 133–134 cortisol excess in, 283
management strategies, 384 Contrast-induced nephropathy, 142 evaluation o , 283
Colleagues, impaired, 693–694 Conversion disorder, 510, 632 Cutaneous anthrax, 2
Colles racture, 347, 348–349, 357 Cooties, 291 Cutaneous herpes, 402
Colon cancer, 190 Copaxone, 511 CVID. See Common variable immune de ciency (CVID)
with large bowel obstruction, 233 COPD. See Chronic obstructive pulmonary disease CVN. See Central venous nutrition (CVN)
metastasis o , 624 (COPD) Cyclic vomiting syndrome, 235
risks or, 623 Corlanor. See Ivabradine Cyclopentolate eye drops, 15
Colonoscopy, 606 Corneal abrasions, 16 Cycloplegia, 15
Colposcopy, 436 treatment o , 539 Cycloplegic agents, 15–16, 535, 539
Coma cocktail, 28 Corneal oreign bodies, 536 Cyclosporine emulsion, 540
Comedones, 490 Corneal ulcer, 15 Cyclosporine, or in ammatory bowel disease, 211
Comminuted racture, 335t Corneal ulceration, 536 Cyproheptadine, 131, 493
Common cold, alternative cure or, 669 Corneas, thinner, 530 Cystitis, 25–26
Common variable immune de ciency (CVID), 136, 137t, Coronary angiography, 60 Cystosarcoma phyllodes tumors, 608
138 in myocardial in arction, 46 Cystostomy, 601
immunoglobulin levels in, 138 Coronary artery bypass graf ing (CABG), 50, 613 Cysts, 607
treatment options or, 138 indications or, 50 Cytisine, or smoking cessation, 681
Community-acquired methicillin-resistant S. aureus V/Q scan af er, abnormal, 50 Cytomegalovirus (CMV) in ection, 485, 486
(CA-MRSA), 479 Coronary artery disease, risk actors or, 91, 92t congenital, 379–380
Community-acquired pneumonia Coronary calcium score, 47 Cytotoxin assay, 239
azithromycin and cef riaxone or, 121 Cor pulmonale, 64
causative agent, 121 causes o , 64–65 D
chest x-ray in, 122 continuous, low- ow oxygen in, 65 Dabigatran, 109, 195, 196 198
outpatient treatment o , 123 ECG in, 65 in non-valvular atrial brillation, 54
parenteral antibiotics in, 121 Corticobasal ganglionic degeneration, 517 Dal ampridine. See Ampyra
Compartment syndrome, 16–17 Corticosteroids Danazol, 135
complications o , 17 or ABPA, 127, 127t Dapagli ozin, 296
treatment o , 18 or adrenal insu ciency, 285 Dapsone, 477
urinalysis in, 17 or giant cell arteritis, 310 Darbepoetin, 63
Compartment syndrome o eye, 541 or gout, 314 Darier sign, 496
Complete blood cell count (CBC), 587 or headaches, 701 Date o delivery, 416, 417t
Compression stockings, 110 or MS, 511 Date rape, 5
Comtan. See Entacapone or polymyalgia rheumatica, 308–309 DDAVP, 156, 165
COM inhibitors, 518 in pregnancy, 422 D-dimer test, 107
Concussion, 11–12, 405–406 Cortisporin suspension, 550 Death, impending, signs o , 708
a ects o , 406 Cosyntropin stimulation test, 284 Decision-making capacity (DMC), 688–689
cognitive rest af er, 407 Coudé catheter, 601 Deconditioning, 59
de nition o , 406 Cough Deep venous thrombosis (DV ), 195–196
imaging in, 406 ACE inhibitors and, 112 postoperative, prevention o , 610
prevention o , 407 causes o , 59 in pregnancy, 197
return to practice and play af er, 406–407 chronic, 112–113, 113t De erasirox, 185
second-impact syndrome, 407 gastroesophageal re ux and, 112 De eroxamine, 32
seizure activity af er, 406 proton pump inhibitor in, 112 De eroxamine challenge, 33
signs o , 406 Cow’s milk, 389 Dehydrated child, treatment o , 31
treatment or, 406 Coxiella burnetii, 127 Dehydration, 250, 370
Conductive hearing loss, 555t CRAFF questions, 401 degree o , 370t
Con dence intervals (CI), 716 Cranial nerve process, 562 uid requirement in, 167, 371
Con dentiality, in medical practice, 691–692, 695 C-reactive protein (CRP), 614 saline in usion in, 370
Con rmation bias, 713t role o , in cardiac disease, 92 signs and symptoms o , 370t
Con ounders, 713t, 718 Creatine monohydrate, 669 treatment, 370–371
Con usion Assessment Method (CAM), 578 Creatine phosphokinase (CPK), 17 Dehydroepiandrosterone (DHEA), 399
Congenital syphilis, 379 Creatine phosphokinase MB raction (CPK-MB), 41 Delayed puberty, 372
766 INDEX

Delirium, 576, 634, 676, 709 Diabetic ketoacidosis (DKA), 287 side e ects o , 299
causes o , 577t causes o , 288 Double sickening, 558
evaluation o patient, 577 home-going insulin regimen in, 288–289 Double vision, 546
interventions or, 577 insulin regimen in, 287–288 Down syndrome
prevention o , 577 Diabetic nephropathy risk o , 424
risk actors or, 576 ACE inhibitor in, 144 screening or, in pregnancy, 424
treatment o , 578 diagnosis o , 141 Doxycycline, 252, 449
Demeclocycline, 162 nondihydropyridine calcium channel blockers in, 144 or anthrax, 2
Dementia, 576, 582 Diabetic neuropathy, 296, 360, 481 or bronchiectasis, 126
antidepressant or, 586 Diabetic retinopathy, 289 or community-acquired pneumonia, 123
antipsychotic medication, 589 Dialysis, 159 or Lyme arthritis, 319
behavioral issues, 585, 585t indication or, 144–145 or pelvic in ammatory disease, 437
categorizing, 582 Diarrhea, 204 DPP-4 inhibitors, 292
diagnosis o , 581 by Clostridium di cile, 239 Drain cleaner, ingestion o , 20–21
hospice re erral in, 707 di erential diagnosis or, 215 Dressler syndrome, 50
laboratory evaluation o , 583 treatment or, 215 Dronabinol, 706
medication management or, 584, 585t Diastolic heart ailure (HFpEF), 60, 63–64 Drug-eluting stents (DES), 50
neuroimaging, 581 diuretics in, 64 Drug-related lupus, 324
Dementia with Lewy bodies (DLB), 657 drug therapy or, 64 Drug therapy in elderly, 590
Demerol, 507. See Meperidine in elderly, 64 Dry age-related macular degeneration, 532, 533
Dengue ever, 251, 256 hypertension and, 63 Dual-energy x-ray absorptiometry (DEXA) scan, 397, 574
second occurrence o , 256 Diazepam, 569, 676 Duct tape, or warts, 484
treatment o , 256 Diclegis, or nausea and vomiting o pregnancy, 411 Duloxetine, 640
Denosumab, 575 Diclo enac, 42, 593 Duodenal ulcers, 206
Dental abscess, 564 Dicyclomine, 606 Dutasteride, 455
Dental trauma, 13–14 N,N-diethyl-m-toluamide (DEE ), 252, 319 DV . See Deep venous thrombosis (DV )
Dentition, 563 Di using capacity, 126 Dysmenorrhea, 442
Denver Developmental Screening est, 375 Di cid, 240 primary, 442
Depakote, 502 DiGeorge syndrome, 138t secondary, 442
Dependent elder, de nition, 589 Digibind, 80 Dysphagia, 202
Depo-Provera, and weight gain, 409 Digital vaginal examination, 412 CRES syndrome and, 202
Depression. See also Major depressive disorder (MDD) Digoxin, 55 endoscopy in, 202
add-on therapy or, 658 in atrial brillation, 54 esophageal, 202, 203t
in children, 643, 643t, 644 in diastolic dys unction, 64 Dyspnea, 126
and dementia, 583 in HFrEF, 61 end-stage cardiac disease and, 700–701
postpartum, 642 Dihydropyridine calcium channel blocker, 144 Dysuria, 169–170
in pregnancy, 642 Dilantin. See Phenytoin
psychiatric re erral or, 630 Diltiazem E
risk actors or poor outcome on treatment, 630 in atrial brillation, 54 Ear pain, causes o , 551, 551t
SAM-e in treatment o , 668 side e ects o , 74 Ear piercing, 623
screening, 581 Dilute Russell viper venom test (DRVV ) assay, 325, 326 Eating disorders, 650
and weight loss, 570 Dimercaprol, 184 Eaton–Lambert syndrome, 513
Depression scales or screening, 627 Dinoprostone, 413 Ebola in ection, 239
De Quervain tenosynovitis, 349 Diphenhydramine, 558, 569, 578, 708 Echocardiography
treatment o , 349 or bee sting reaction, 23 or diagnosis o heart ailure, 59
Derivation set, 714 Diphenhydramine overdose, 5, 5t recommended intervals or valvular disease evaluation,
Dermatitis herpeti ormis, 477 Diphyllobothrium latum, 188 71t
Dermatomyositis, 324, 332 Diplopia, 527 E-cigarettes, 679
diagnostic criteria or, 332t Dipyridamole, 42, 49 Ecstasy, 4, 686
and GI malignancy, 332 Direct laryngoscopy, 553 EC . See Electroconvulsive therapy (EC )
proximal muscle weakness in, 332 Direct ophthalmoscopic examination, 533 Ectopic pregnancy, 27, 430
tests or, 332 Discitis, 347 emergent surgery in, 438
Desmopressin (DDAVP), 148, 174 Disclosure, 694 PID and, 438
Desquamation, 475t Discriminant unction (DF), 224 positive pregnancy test and, 27–28
Detrusor hyperactivity, 588 Disease and illness, di erence between, 724–725 risk actors or, 27, 430
Developmental delays, head injury and, 374–375 Disease-modi ying anti-rheumatic drug (DMARD), 304 serum HCG and, 28
Developmental milestones, 367t Disruptive mood dysregulation disorder, 643 tests in, 430–431
Devic disease (neuromyelitis optica), 510 Disseminated intravascular coagulation (DIC), 194 Eczema herpeticum, 479–480
Dexamethasone, 35–36 Distributive shock, 618t Edinburgh Postnatal Depression Scale (EPDS), 642
in addisonian crisis, 285–286 Disul ram, 681 Edoxaban, 54, 195–196
in severe headache, 508 interaction with ethanol, 681–682 Edrophonium, 512
DEXA scan. See Dual-energy x-ray absorptiometry side e ects o , 682 E ent. See Prasugrel
(DEXA) scan Diuretics, 144 Egg allergy, 134
DHE nasal spray (dihydroergotamine), or headaches, 507 hyponatremia by, 160 Ehrlichia cha eensis, 250
Diabetes, 663 and postprandial hypoglycemia, 297 Elbow dislocation, 16
age-adjusted incidence o , 724 Divalproex, 585t, 647 Elder abuse and neglect (elder mistreatment)
aspirin or primary CVD prevention in, 293–294 Diverticular bleeding, 235 about, 589
blood pressure goal in, 52 Diverticulitis, 233 risk actors or, 589
diagnosis o , 286 C scan o abdomen and pelvis in, 233 Elderly
education program or, 291 management o , 234 alcohol problems, 676
in hospitalized patients, 295–296 second episode, 234 cough in, 59
hypertension in, 291 Diverticulosis, 234 daily exercise or, 591
hypoglycemia in, 294–295 Dix–Hallpike maneuvers, 560, 560t drinking behavior in, 590
lisinopril or hypertension in, 74 Dizziness, 498, 559 hip ractures in, 580
lower-extremity amputation in, 292 DKA. See Diabetic ketoacidosis (DKA) hypertension in, 52
measurement o blood glucose in, 296 Dobutamine, 49 insomnia in, 569
pathologic actors, 290 Dog bites, 10 peripheral sensory disturbance in, 579
prevention o , 289 antibiotic prophylaxis or, 10 sicca symptoms in, 321
preventive services in, 293 in ections, 10 weight gain in, interventions or, 706
screening or, 290, 294t rabies prophylaxis af er, 10 Electrocardiography (ECG)
Diabetes mellitus type 2, 293t, 296, 533. See also Diabetes Domperidone, 231 in cor pulmonale, 65
progression to overt nephropathy in, 141 Donepezil, or Alzheimer disease, 517 in rst-degree AV block, RBBB, and LAFB, 85, 86 , 87
screening or kidney disease in, 140 Dopamine agonist in hyperkalemia, 87, 87
Diabetic amyotrophy, 514 or macroadenomas, 299 in hypokalemia, 87
Diabetic oot ulcer, 495 menopause and, 300 in in erior wall myocardial in arction, 77, 78
INDEX 767

in LBBB, 85, 86 Epstein–Barr virus (EBV) in ection, 553, 566 Fasciotomy, or compartment syndrome, 18
in lef anterior ascicular block, 85 Epzicom (abacavir/lamivudine), 259 FAS examination, 30
in MA , 87, 88, 88 ERCP. See Endoscopic retrograde Fatal amilial insomnia, 656
in pericardial e usion, 80, 82 cholangiopancreatography (ERCP) Fat bodies, in urine, 152
in pericarditis, 50–51, 81, 83 Erectile dys unction, 472 Fat-soluble vitamin, 660, 661
in PSV , 81, 83, 83 evaluation o , 472 Fatty liver disease, 403
in second-degree heart block, Mobitz type II, 68–69, organic, 472 Febrile newborn, 34, 34t
68 , 80, 81 psychogenic, 472 Febrile seizures, 523–524
in second-degree heart block, type I, 77, 77 therapy or, 473 evaluation o , 394
in tricyclic overdose, 3 Erythema, 475t Febuxostat, 315
in ventricular tachycardia, 66, 67 Erythema migrans, 318, 492 Fecal impaction, 363, 589
in Wol –Parkinson–White (WPW) syndrome, 83, 84 Erythema multi orme, 492 Fecal incontinence, cause o , 589
Electroconvulsive therapy (EC ), 637–638 Erythema nodosum, 491–492, 491 Fecal occult blood, testing or, 208
adverse e ects o , 638 Erythema toxicum neonatorum, 485 Felbamate. See Felbatol
memory loss with, 637–638 Erythrocyte sedimentation rate (ESR), 614 Felbatol, 502
Electrophysiologic study, 57 Erythromycin, 231, 289, 565 Felon, 251
Elevated arm stress test, 352 Erythropoiesis-stimulating agents (ESAs), 63 Female athlete triad, 397–398
Eliquis. See Apixaban Erythropoietin, 146, 176 Femoral torsion, 337
Elliptical excision, 623 Escharotomy, 618t Femoral vein, 621
Eltrombopag, 175 Escherichia coli, 457 Feno brate, 94
Emend. See Aprepitant epididymitis by, 25 Fentanyl, 20, 600, 703
Emergency contraception, 443–444 Escherichia coli 0157, 215 Ferric citrate, 166
Emergent surgical re erral, 598 Esophageal dysphagia, 202, 203t Ferritin, 182
Empagli ozin, 296 Esophageal intubation, standard o care in, 619 Fetal alcohol e ects (FAE), criteria or, 678
Emphysema, 126 Esophageal manometry, 202 Fetal alcohol syndrome (FAS), 678, 678
Emphysematous, 616–617 Esophageal web, 204 diagnosis o , 678
Enalapril, in heart ailure, 61 Esophagitis, 207 rate o , 678
Enbrel. See Etanercept Esotropia, 526 symptoms and signs o , 678
Encopresis accommodative, 526, 527 Fetal bronectin (FFN) test, 422
conditions associated with, 362 acute, 527 criteria or, 422
unctional, 362 congenital, 526, 527, 527 negative, 422
non-retentive, 362 Essential thrombocythemia (E ), 181 Fetal heart rate (FHR) tracing, 413, 414
primary, 362 Essential tremor, 516 category de nitions, 415t
secondary, 362 Estrogen-containing hormone replacement therapy interpretation o , 415, 417, 418
treatment o , 363 (HR ), 439 management o abnormal patterns, 415
Endocarditis, 90 Estrogen/progesterone replacement therapy, 608 placental insu ciency and, 415
causative organisms, 91 Estrogen testing, 439 variable decelerations in, 419–420, 419
Duke criteria or, 90, 90t Etanercept, 307 Fetal macrosomia, 432. See also Macrosomia
valve a ected by, 90 Ethanol, or methanol ingestion, 8 Fever, 178
End-o -li e care, 689, 698–710 Euthyroid sick syndrome, 275 absence o , 587
Endolymphatic sac shunt, 556 Evolocumab. See Repatha intrapartum, 415–416
Endometrial biopsy, 441 Excessive daytime sleepiness, 520 postoperative, 610
Endometrial cancer, 441, 446 Exchange trans usions, 18, 19 causes o , 611, 611t
management o , 441t Exenatide, 292 Fever ew, or migraine, 668
protective actors or, 441t Exercise Fever, in neonate, 392
risk actors or, 441t programs, 579 antibiotics or, 393
Endometriosis, 444–445 or weight loss, 664 evaluation o , 392t
Endometritis, 415–416 Exercise stress testing, 48–49 osteomyelitis and, 393
Endoscopic retrograde cholangiopancreatography (ERCP) contraindications to, 48, 48t Fever o unknown origin (FUO), 249
adverse consequence o , 228–229 negative stress test, 48–49 causes o , 250t
with sphincterotomy, 228 sensitivity and speci city o , 49t evaluation in, 249
Endotracheal tube (E ), 29–30 Exjade. See De erasirox malignancies present with, 249
End-tidal CO2, 619 Exotropia, 526 FHR tracing. See Fetal heart rate (FHR) tracing
Enoxaparin, 110, 610 Exploding head syndrome, 519 Fibroadenomas, 607
or acute myocardial in arction, 45 External hemorrhoids, 605 Fibroids, uterine, 429–430
Entacapone, 518 Eye Fibromyalgia, 322, 510
Entamoeba histolytica, 251 chemical injuries to, 536 conditions associated with, 322
Enterobius vermicularis, 249 drops, 529 management o , 322
Enteropathic arthritis, 331 neovascularization o , 529 pain associated with, 322
Enterotoxigenic E. coli (E EC), 216, 252 Eye trauma, 16 tender points in, 323
Entresto, or systolic heart ailure (HFrEF), 91 Ezetimibe, 94 Fibrosis, 217
Enuresis, 375–376 side e ects o , 94–95 Fidaxomicin. See Di cid
evaluation o , 376 Fif h disease, 382
primary, 375 F Figure-o -eight splint, 351
and retention control training, 376 FABER (Flexion, Abduction, External Rotation) test, 330 Finasteride, 455–456
secondary, 375 Facial mal ormations, alcohol use during pregnancy or androgenetic alopecia, 462
treatment o , 376 and, 679 Fine needle aspiration (FNA), 557, 607
Enuresis alarm, 376 Facilitated PCI, 44 or thyroid nodules and cysts, 278
Eosinophilic esophagitis, 203 Factitious disorder, 632 Finklestein test, positive, 349
Epidermolysis bullosa, 485 Factor VIII in usion, in hemarthroses, 174 First-degree heart block, 69
Epididymitis Factor V Leiden mutation, 196 Fissures
in older men, 25 Failure to thrive (undernutrition), 369–370 rst-line therapy, 606
in young males, 25 management o , 370 types o , 606
Epidural analgesia, in labor, 419 Falls, in older people Fixed upper airway obstruction, ow/volume loop in,
Epiglottitis, 38, 384–385 ambulatory devices, use o , 580 103, 103
diagnosis o , 385 causes o , 578 Flagyl (metronidazole), or richomonas vaginal
Epilepsy surgery, 503 evaluation o uture risks, 579 in ection, 448
Epinephrine exercises and risk o , 579 Floating black spots, 543
in asystole, 68 and hip ractures, 580 Flow/volume loops, 103, 103
or bee sting reaction, 23 risk actors, 578–579 Flublok, 134
racemic, 36 False negative rate (FNR), 721 Flucelvax, 134
Epiploic appendagitis, 234 False positive rate (FPR), 722 Fluconazole, 465
Episcleritis, 545 Famotidine, 574 Fluid administration in burn, 619
Episiotomies, 420 Farmer’s lung. See Hypersensitivity pneumonitis Fluid overload, 221
Eplerenone, or heart ailure, 62 Farxiga. See Dapagli ozin Flumazenil, 3
768 INDEX

Fluorescein, 538 Gastroesophageal re ux disease (GERD), 200 Glycopyrrolate, 700, 706


Fluoroquinolone-resistant gonococcus, 312 complications o , 201 Glycosaminoglycan, 670
Fluoroquinolones, 2, 456, 538, 539 cough due to, 112 GnRH agonists, 442
Fluoxetine, 232, 401, 639, 642 diagnosis o , 200 Gonadotropin releasing hormone (GnRH) agonists, 429
and birth de ects, 640 Helicobacter pylori and, 200 Gonococcal arthritis, 312
dosage o , 652 in in ant, 390 Gonococcal conjunctivitis, 537, 538
reduction o , 570 surgery or, 201 Gonococcal pharyngitis, 566
Fluvoxamine, 639 treatment in, 200–201 Gonococcal urethritis, 464
Foam pad with occluding dressing, 571 Gastrointestinal illness, 383 antibiotics or, 464–465
FODMAPs, 213, 214 Gastrointestinal intolerance, 584 diagnosis o , 464
Folate de ciency, 187 Gastroparesis, 231 Good sleep hygiene, 569
and B12 de ciency, 187 causes o , 232 Gottron papules, 324, 332
Foley catheter, 601 exacerbation o , 231–232 Gout, 313
Fomepizole (4-MP), or methanol ingestion, 8 treatment o , 231 chronic tophaceous gout, 316
Food allergies, 134 GBS. See Guillain-Barré syndrome (GBS) diagnostic criteria or, 314t
Foreign body GCA. See Giant cell arteritis (GCA) drugs and, 314
in esophagus, 204 GDM. See Gestational diabetes (GDM) hyperlipidemia in, 316
in eye, 16 Gem brozil, 94 long-term urate-lowering therapy in, 315
ingestion, 36–37 Gender expressions, 733 management o , 314
Foreskin, retractable, 27 Gender identity, 733 risk actors or, 314
Formula eedings, 389–390 Genetic testing, 695 serum uric acid levels in, 314
Forward bending test, 339 Gentamicin synovial uid analysis in, 313–314
Fosamax. See Alendronate or in ection in newborn, 34 GPA. See Granulomatosis with polyangiitis (GPA)
Fos omycin, 26 ophthalmic ointment, 16 Grace Risk Model, 48
Fosphenytoin, 4, 502 Geodon. See Ziprasidone Gram-negative aerobes, 604
Fosrenol. See Lanthanum carbonate Geographic tongue, 554 Gram-negative pneumonias, 610
Fournier gangrene GERD. See Gastroesophageal re ux disease (GERD) Grand mal seizures, 503
as necrotizing asciitis, 598 Geriatric depression scale, 570 Granular casts, in urine, 152
treatment o , 598 Geriatric pharmacotherapy, 590 Granulomatosis with polyangiitis (GPA), 114. See also
Fox dermal curettage, 622 Gestational diabetes (GDM) Hemoptysis
Fractional excretion o sodium (FENa), 156 3-hour glucose tolerance test or, 424–425 ndings in, 114
Fracture risk actors or, 411 Grape ruit juice, 668
avulsion, 335t screening test or, 424 Graves’ disease, 273, 546
blowout, 14–15 Gestational hypertension, 425–426 in pregnancy, 274
blowout racture, 14–15 Gestational thrombocytopenia, 175 tests in, 274
clavicle, 351 Get-up-and-go test, 579 treatment o , 274
Colles, 347–349 GHB, 686 Graves’ ophthalmopathy, 274–275
comminuted racture, 335t Giant cell arteritis (GCA), 308, 309, 532, 542 Greenstick racture, 335t
greenstick, 335t diagnostic criteria or, 310t Grid laser, 533
Jones, 357 management o , 310 Groin pain, 602
Lis ranc, 352–353, 353 , 354 methylprednisolone or, 310 Gross hematuria, 147
Maisonneuve, 357 and thoracic aortic aneurysm, 310 in children, 167t
posterior rib, 341 GI anthrax, 2 Group A strep pharyngitis, 565
radial head, 359 Giardia in ection, 213 Group B streptococcus (GBS), 33
rib, 116 GI bleeding, upper, 205–206 GBS prophylaxis or preterm delivery, 423
scaphoid, 347–348 GI cocktail, 42 screening or, in pregnancy, 412
scapula, 350 Gilbert syndrome, 226 Grover disease, 496
tibial stress, 397–398 Gin and tonic hypoglycemia, 297 Growing pain, 338
toddler’s, 340 Ginger, 669 treatment or, 338
Framingham risk score, 47 Ginkgo biloba, 584, 668, 670 Growth hormone (GH)
Fresh rozen plasma (FFP) trans usion, 177 Glasgow Coma Scale (GCS), 12, 12t de ciency, 301
Frontotemporal dementia (F D), 582–583 need or intubation, 29 excess, 300–301
Front-wheeled walkers, 580 Glatiramer acetate, 511 replacement therapy, 301
Frostbite, 20 Glaucoma, 528–531 Guaiac tests, 208
Functional bowel obstructions, 603 acute angle closure, 528–529, 528 , 529 Guideline or Adolescent Preventive Services (GAPS)
Functional constipation, Rome Criteria, 594t drugs used or, 529t questionnaire, 400
Functional dysphagia, 202 open-angle, 530 Guillain–Barré syndrome (GBS), 504, 512–513, 514
Functional incontinence, 433–434 optic nerve ndings in, 531 diagnosis o , 514–515
Fundoplication, 201 and retinal detachment, 543 in ectious agents with, 514
FUO. See Fever o unknown origin (FUO) Gleevec. See Imatinib treatment o , 515
Furosemide, 17, 87, 143, 146, 152, 279, 455, 573 Glenohumeral dislocation, 350 GUS O V trial, 47
Fusobacterium necrophorum, 568 Glipizide, 663 Gynecomastia
Glitazones, 227, 291, 294 causes o , 463
G Globus sensation, 202 estrogens and, 463
Gabapentin, 502, 505, 587, 702, 704–705 Glossopharyngeal neuralgia, 522 physiologic, 463
contraindication to, 505 GLP-1 receptor agonists, side e ect o , 292
Gadolinium, use o , 142 Glucagon, or beta-blocker overdose, 8 H
Gail model, 609 Glucagon-like peptide-1 (GLP-1) drugs, 292 HAAR . See Highly active antiretroviral therapy
Galactosemia, 372 Glucocorticoids (HAAR )
Gallstones, 230–231 or elevated calcium level, 279 HACEK organisms, 248
evaluation o , 615 and risk o osteoporosis, 574 HAE. See Hereditary angioedema (HAE)
risk or, 615 Glucosamine and chondroitin, or osteoarthritis, 594, 670 Haemophilus ducreyi, 457
types o , 616 Glucose intolerance, 449 Haemophilus inf uenzae, 121
ultrasound or, 615, 616 Glucose-6-phosphate dehydrogenase (G6PD) de ciency, Hair loss, in males, 462
Gamekeeper’s thumb, 358–359 184, 191–192 Hair transplant, 462
Gamma-hydroxybutyrate (GHB) overdose, 4–5, 5t hemolytic crisis in, 192 Haloperidol, 577, 709
Gangrenous bowel, 388 interventions or, 192 Hampton hump, 107
Gardasil, 442 Gluten- ree grains/starches, 215 Hand eeding, 586, 706
Gastric bezoars, 231 Gluten-sensitive enteropathy, 213, 214 Hantavirus in ection, 128–129
Gastric emptying, delayed, 289 gluten- ree diet in, 214–215 Hashimoto thyroiditis, 271, 273
Gastric lavage, 1 tests or, 214 HC Z. See Hydrochlorothiazide; Hydrochlorothiazide
Gastric ulcers Glyburide, 294 (HC Z)
rom NSAID, 605 Glycemic index (GI), 298 HCV antibody test, 216
risk actors or malignancy in, 206t Glycemic load (GL) o ood, 298 Headaches, 509
Gastrinoma, 207 Glycoprotein IIb/IIIa inhibitors, 47 cluster, 521
INDEX 769

migraine, 506–507, 522 transmission o , 218 or demented patients, 586


rebound, 508–509 treatment o , 218, 219t involuntary, 637
Head C Hepatocellular carcinoma (HCC), 220 Hot ashes, treatment o , 440
in headache, 513 Hepatorenal syndrome, 225 Household bleach ingestions, 20–21
indications or, 12, 13t Herbal supplements, 667 HPV. See Human papilloma virus (HPV)
Head lice, 253–254 Hereditary angioedema (HAE), 135 HPV vaccination, 442
Head trauma, 11–13, 13t and allergic angioedema, 136 H. pylori in ection, treatment or, 206–207
Health belie s, understanding o , 725 androgens or, 135 HR . See Hormone replacement therapy (HR )
Healthcare-associated pneumonia, 123 laboratory tests or, 135 HSV PCR assays, 402
Healthcare or older patients, 586 transmission o , 135–136 Human cloning, 696–697
Healthcare planning, 575–576 treatment o , 135 Human Genome Project, 732
Health Insurance Portability and Accountability Act Hereditary nephritis. See Alport syndrome Human granulocytic anaplasmosis, 251t
(HIPAA), 735 Hereditary nonpolyposis colorectal cancer (HNPCC), 209 Human monocytic ehrlichiosis, 250, 251t
Health literacy skills, 728–729 Hereditary spherocytosis, 191 Human papilloma virus (HPV), 267, 442
Hearing loss, 555–556 Hernia, 601 vaccine or boys, 403
and sensory impairments, 573 inguinal, 601–602 Humira. See Adalimumab
Heart block repair o , 602 Humoral hypercalcemia o malignancy (HHM), 279
rst-degree, 69 Herpangina, 371 Hungry bone syndrome, 281
myocardial in arction and, 45, 45t Herpes gladiatorum, 402 Hutchinson triad, 379
second-degree, type I (Wenckebach), 69 Herpes simplex encephalitis, 515–516 Hydralazine and isosorbide dinitrate, in heart ailure, 61
second-degree, type II (Mobitz II), 68–69, 68 Herpes simplex virus (HSV), 445, 562 Hydrocarbon toxicity, 7
third-degree, 69 Herpes virus in ection, in newborn, 33 Hydrochlorothiazide
Heart ailure (HF), 142–143 Herpes zoster, 484 or hypertension, 74, 729
cardiac resynchronization or, 91 Herpetic gingivostomatitis, 371 in urolithiasis, 150
diagnosis o , 59 Herpeti orm, 475t Hydrogen peroxide, 11
diastolic (see Diastolic heart ailure (HFpEF)) Heterophile antibody test, sensitivity o , 567 Hydrogen sul de poisoning, 97
echocardiography in, 59 Highly active antiretroviral therapy (HAAR ), 259, 267 Hydromorphone, 20, 600, 705
systolic (see Systolic heart ailure (HFrEF)) in children, 266 Hydroxychloroquine, 325
Heart Failure with a preserved Ejection Fraction in pregnancy, 264 side e ects o , 326
(HFpEF), 59. See also Diastolic heart ailure Hip ractures, 580, 592 Hydroxyurea, 18, 181
(HFpEF) Hip osteoarthritis, 602 Hyperactive thyroid adenoma, 273
Heart Failure with a reduced Ejection Fraction (HFrEV), Hip surgery, 591 Hyperaldosteronism
59. See also Systolic heart ailure (HFrEF) Histoplasma capsulatum, 127 causes o , 76
Heat exhaustion/heat stroke Histrionic personality disorder, 653 and hypertension, 76
risk actor or, 21 HIV/AIDS, 260, 506 hypokalemia and, 76
treatment o , 21 baseline studies in, 259 plasma aldosterone-to-renin ratio in, 76
Heliotrope rash, 324 and breast eeding, 265–266 treatment o , 76
HELPERR mnemonic, 433 cervical cancer with, 267 Hyperbaric oxygen, 599
Hemabate. See Carboprost tromethamine changing drug regimens in, 261 Hyperbaric oxygen therapy
Hematocrit, elevated, causes o , 181 children with, 266 carbon monoxide poisoning and, 9
Hematospermia, 459 con dential name-based reporting or, 690 consequences o , 10
Hematuria and delivery, 265 criteria or, 10
gross, 147 ailed HAAR treatment, 261 and myopia, 10
microscopic, 147 HAAR therapy in, 259, 261 in pregnancy, 10
Heme positive stools, in children, 388 HIV-1/HIV-2 multispot test or, 263–264 Hypercalcemia, 279
Hemochromatosis, 190 natural history o , in women, 267 causes o , 281t
iron studies in, 190 and partner noti cation, 260 ECG in, 279
treatment or, 190 PCP prophylaxis in, 262, 264–265 actors or elevated calcium level, 280
Hemodialysis, 37, 159 perinatal transmission o , prevention o , 264 granulomatous diseases and, 279
or methanol ingestion, 8 pneumococcal and hepatitis B vaccine in, 259–260 treatment o , 279
Hemoglobin A1c, 286 postpartum transmission o , 265–266 Hyperhomocysteinemia, 196
Hemoglobin A1C, 595 pre-exposure prophylaxis (PrEP), 269 Hyper IgE (Job syndrome), 137t
Hemolytic trans usion reaction, 177 and pregnancy, 263–265 Hyper IgM syndrome, 137t
Hemolytic uremic syndrome (HUS), 167, 215 recommended prophylaxis in, 260t Hyperkalemia, 143, 612
causes o , 168 respiratory distress and respiratory ailure, 261–262 causes o , 87
Shiga toxin, 168 screening recommendation or, 268 ECG ndings in, 87, 87 , 143
Hemophilia A, 173 spread o , 268 urosemide in, 143
Hemoptysis status o in ant, determination o , 265 Hyperlipidemia
causes o , 114t testing or, 258 in gout, 316
chest x-ray in, 113 testing or, in expectant emales, 263 obesity and, 403
endotracheal intubation in, 114–115 vertical transmission o , 263 soy or, 667t
massive, 114 HIV antibody–antigen ELISA, 258 Hypernatremia, 156
treatment o , 115 HLA-B27, 330 causes o , 156t
Hemorrhoidectomy, 606 Hodgkin lymphoma, 195 Hyperopia, 527, 528
Hemorrhoids B symptoms in, 195 Hyperprolactinemia, 299
surgery or, 606 late e ects o radiotherapy or, 179 Hypersensitivity pneumonitis, 97, 98, 127
treatment or, 606 lymph node biopsy in, 195 chronic, and ndings, 98
types o , 605 and preventive health issues, 179 oral steroids or, 98
Hemostasis, 620 Holiday heart, 53 respirator in, use o , 98
Henoch–Schönlein purpura (HSP), 166, 167, 377 Holiday- Segar method, or maintenance uids Hypersensitivity reaction, 133, 133t
Heparin, 45, 176, 610, 612 calculation, 371 type I, 493
Heparin-induced thrombocytopenia and thrombosis Homan sign, 196 type IV, 485
syndrome (HI S), 612 Hordeola (styes), 540 Hypertension, 52
Hepatic encephalopathy Hordeolum, 540 in adolescent athlete, 400
causes o , 223 Hormone receptor negative, 609 antihypertensives or, 73–74
treatment o , 223–224 Hormone replacement therapy (HR ) de nition o , 73
Hepatic iminodiacetic acid (HIDA), 615 bene ts o , 439 diagnosis o , 72, 400
Hepatic vein thrombosis, 176 contraindication to use o , 439–440 diltiazem or, 74
Hepatitis A vaccine, 402 in postmenopausal women, 439 doxazosin or, 74
Hepatitis B, 219–220, 220t, 506 or vasomotor symptoms, 440 initial evaluation o , 73
Hepatitis B arthritis, 311 Horner syndrome, 546 lisinopril or, 74
Hepatitis B sur ace antigen (HBsAg), 219 Hospice care, 698 during preconception counseling, 427
Hepatitis C, 506 cardiac therapies in, 698–699 in pregnancy, 425–427
and cryoglobulinemia, 317 and palliative care, 699 secondary causes o , 75, 75t
indications or treatment o , 218 Hospitalization, 617 white coat, 73
770 INDEX

Hypertensive crisis, 640–641 Indomethacin, 422 Intracranial aneurysm, 546


Hyperthyroidism, 273. See also Graves’ disease or pericarditis, 51 Intracranial pressure, increased, 12–13
causes o , 277 Induction o labor, 433. See also Labor induction Intranasal steroid spray, 131, 132
isolated 3-toxicosis in, 277 Inert intraocular oreign bodies, 535 Intraocular oreign body, 535
Hypertrophic cardiomyopathy, 53, 63, 399 In antile hemangiomas, 488 Intraocular lens dislocation, 546
systolic crescendo–decrescendo murmur o , 400 In ants Intraocular pressure
Hyphema, 529, 534, 535 constipation in, 390 elevated, 530
Hypnagogic hallucinations, 520 eeding and nutrition or, 389–390 measurement o , 528, 534
Hypnotics, 520 gastroesophageal re ux disease in, 390 Intrauterine devices (IUDs), 398, 410
Hypocalcemia, 164–165, 279, 281, 575 HIV/AIDS and, 265 contraindications to placement o , 428–429
evaluation o , 282 jaundice in, 364–365 mechanism o action o , 429
treatment in, 282 osteomyelitis in, 393–394 Intrauterine growth restriction, 425
Hypoglycemia, 294 respiratory syncytial virus and, 386 Intravenous (IV) nutrition, 570
causes o , 297 In ection, in newborn, 33–34 Intubation, 29, 622
in diabetics, 295 antibiotics or, 34 Intussusception, 372–373, 384
insulin glargine in, 295 causative organisms, 33 diagnosis o , 373
postprandial, 297 evaluation or, 33 and GI obstruction, 373
Hypokalemia In ectious diseases in nursing home settings, 587–588 treatment o , 373
causes o , 154 In ective endocarditis, 245–246 ultrasound in, 373
metabolic acidosis and, 154–155 antibiotic prophylaxis or, 246 Invasive breast carcinoma, 608
therapy or, 155 blood cultures or, 247 Invokana. See Canagli ozin
Hypomagnesemia, alcoholism and, 676 causative organisms, 247–248 Iodine de ciency, and hypothyroidism, 272
Hyponatremia, 159 high-risk condition or adverse outcome o , 245 Ipecac, 1
chronic, 161 laboratory evaluation in, 246 Ipratropium, 101, 132
di erential diagnosis o , 160 modi ed Duke criteria or, 246, 247t Ipratropium nasal spray, 558
diuretic-induced, 160 signs o , 246 Ipri avone, 666
hypovolemic, 159–160, 161 subacute bacterial endocarditis, 246 Iris heterochromia, 535
laboratory tests in, 161–162 surgery in, indications or, 247 Iritis, 539
therapy or, 160–161 treatment o , 247 Iron de ciency anemia, 181–182, 286
Hypotensive trauma patients, blood administration in, In erior wall myocardial in arction (IAMI), 42–43 causes o , 182t
30 ECG in, 77, 78 ailure o iron therapy or, 182–183
Hypothermia In ertility iron therapy or, 182
bradycardia in, 19 in couple, 471 IV iron preparations in, 183
risk actors or, 20 in males, 470–471, 471t Iron toxicity, 31–33
Hypothyroidism, 271, 332 In ammatory arthritis, 320 chelation with de eroxamine in, 32
causes o , 271–272 In ammatory bowel disease (IBD), 210, 606 mani estations o , 32, 32t
diagnosis o , 271 anemia in, 211 and serum iron levels, 31–32
ailure o medical therapy in, 272 extraintestinal mani estation o , 210 treatment in, 32
iatrogenic, 272–273 and risk o colon cancer, 212 Irreversible retinal damage, 542
and osteoporosis, 272–273 surgery in, 212 Irritable bowel syndrome (IBS), 212, 606, 660
and pregnancy, 273 treatment o , 211 e cacy o ber or, 212
treatment o , 272 In iximab, 307 Rome III criteria or, 212t
Hypovolemic shock, 618t or in ammatory bowel disease, 211 treatment o , 213
Hypoxia, 576 In uenza Ischemic colitis, 233
Hysterectomy, 435 antiviral therapy during, 238 Ischemic optic neuropathy, 532
or leiomyomata, 429 complications with, 387 Isoniazid (INH), 244, 245
in healthcare setting, prevention o , 238 overdose, 9
I outbreak, 588 Isoproterenol, 45
IAMI. See In erior wall myocardial in arction (IAMI) vaccines or (see In uenza vaccination) Isosorbide dinitrate, 144, 473
Ibandronate, 575 In uenza vaccination, 237–238, 588 Isotretinoin, 491
IBD. See In ammatory bowel disease (IBD) contraindication to, 237 side e ects o , 491
IBS. See Irritable bowel syndrome (IBS) egg- ree, 134 IUDs. See Intrauterine devices (IUDs)
Ibupro en, 573, 702 In ormed consent, 725–726 Ivabradine, 91
Ibutilide, in Wol –Parkinson–White syndrome, 85 Ingrown toenails, 622 Ivanapyuk, 444
ICE mnemonic, 725 Inguinal hernias, 459, 601 Ivermectin, 248, 254
Idiopathic rhinitis, 559 Inhalant abuse, 684–685
Idiopathic thrombocytopenic purpura. See Immune demographics o , 685 J
thrombocytopenic purpura (I P) types o , 685 Janeway lesions, 246
Id reactions, 485 withdrawal rom, 685 Januvia. See Sitagliptin
IgA de ciency, 137t, 138 Inhalation injury, 619 Jardiance. See Empagli ozin
IgA nephropathy, 147, 166 Insect repellent, use o , 252 Jarisch–Herxheimer reaction, 319
IGRAs, 244 Insomnia, 569, 585t Jaundice, in in ants, 364–365
Ileostomy care, 624 Instrumental activities o daily living (IADL), 591 JIA. See Juvenile idiopathic arthritis (JIA)
Ileus, causes o , 603 Insulin, 287–289 Joint aspiration, in monoarthritis, 311
Imatinib, 193 in diabetic ketoacidosis, 287–288 Jones ractures, 357
Imbalance (disequilibrium), 559 regular, 287–288 Juvenile idiopathic arthritis (JIA), 318, 377
Imiquimod, 484 Insulin-euglycemia, high dose, or beta-blocker
Immune reconstitution in ammatory syndrome (IRIS), overdose, 8 K
262 Insulin-like growth actor-I (IGF-I), 301 K2, 686
Immune thrombocytopenic purpura (I P), 174 Insulinoma, 297–298 Kava-kava, 641
causes o , 174 Intact P H (iP H), test or, 280 Kawasaki syndrome, 307
di erential diagnosis, 175 Intention to treat analysis, 714 Kayexalate, 143
treatment options or, 175 Inter eron agents, and depression, 511 Kegel exercises, 434
Immunizations. See Vaccination Inter eron and ribavirin, or hepatitis C, 218 Kernig sign, 241
Immunoblasts, 129 Inter eron-b, 511 Ketorolac, 538, 702
Immunode ciency syndromes, 136, 137–138t Intermittent claudication, peripheral arterial disease Ketoti en, 538
Immunologic reactions, 133, 133t and, 69 Kidney stones, 150
Implanted cardiac de brillator (ICD) Internal hemorrhoids, 605 and hematuria, 147, 148
hospice and, 698 Internal tibial torsion, 337 management in, 148–149
Inclusion and exclusion criteria o trial, 714 Interpreters, use o , 727–728 prevention o urther stone ormation, 149
Incontinence Intoeing (pigeon-toeing), 337 renal colic due to, 148
diagnosis o , 434 Intoxicated patient with minor trauma, care or, 15 Kiesselbach plexus, 14, 561
treatment o , 434 Intoxication, 675 Klebsiella oxytoca, 216, 240
type, 433–434 Intra-articular racture, 355 Kline elter syndrome, 471–472
Indinavir, 150 Intracochlear gentamicin, 556 Knee dislocation, and popliteal artery injury, 16
INDEX 771

Knee pain, cure or, 670 Likelihood ratio, 715, 715 and anxiety disorder, 627
Koilonychia, 487 Linezolid, 248, 641 atypical depression, eatures o , 626
Korotko sounds, pulsus paradoxus and, 51 Lipid disorders criteria or diagnosis o , 626
Kuru, 256 lipid-lowering therapy, 93–94 actors with increased risk o depression, 627
screening recommendation or, 93 hypersomnia in, 626
L Lipid emulsion, 3 illnesses with, 628
Labetalol, 425 Lipid level reduction, 666 in post-MI period, 629
Labor analgesia, 419 Liraglutide, 292 prevalence o , 627
Labor induction Lis ranc racture, 352–353, 353 , 354 in primary care patients, 627
amniotomy or, 416 treatment or, 353 relapse ollowing rst episode o depression, 627–628
nonpharmacologic methods o , 416–417 Listeria monocytogenes, 33 in women, 627
oxytocin or, 417 Lithium, 162, 647 Makena, 423
af er rupture o membranes, 412–413 drug a ecting levels o , 648 Malaria, 252
antibiotics initiation af er chorioamnionitis side e ect o , 647 diagnosis o , 253
detection, 413 Lithotripsy, 564 therapy or, 253
drugs or, 413 Liver disease Malassezia yeast, 479
Labyrinthectomy, 556 assessment o degree o , 217 Male in ertility
Labyrinthitis, 559 in hepatitis C, 217 causes o , 471t
Lachman test, 354 Liver brosis, 217 risk actors or, 470–471
Lactated Ringer’s, 17, 619 Liver tests, elevated, 220–221 treatments or, 471
Lactic acidosis, 143 Lone atrial brillation, 55 Malignancy-associated hypercalcemia, 279
Lactose intolerance, 213 Long acting reversible contraceptive (LARC), 398, 410 Malignant melanoma, 475–476
Lamictal. See Lamotrigine Loop diuretics, 574 Breslow tumor thickness, 476
Lamisil. See erbina ne Loop electrosurgical excision procedure (LEEP), 435 evaluation in, 476
Lamotrigine, 502, 647 complications o , 435 risk actors, 476–477
or partial seizures, 503 Loperamide, 624 risk actors or, 476
Language barriers, 728 or in ammatory bowel disease, 211 Malignant nodes, 556
Lanthanum carbonate, 166 Lorazepam, 675 Mallet nger, 348
Laparoscopic banding, 604 or seizures, 4, 28 Mallory–Weiss tear, 208
Laparotomy, 614 or vomiting patient, 676 Mammogram, 573
Lariam. See Me oquine Lower-back pain (LBP), 344 Mania, 646–647
Laryngeal cancer, risk actors or, 553 early imaging in, need or, 345t Mannitol, 17, 529t
Laryngomalacia, 35 treatment or, 345–346 Mantoux test. See uberculin skin test ( S )
Laryngopharynx, 553 Low-glycemic index diet, 664 MAOIs. See Monoamine oxidase inhibitors (MAOIs)
Laryngoscopy, 553 Low–molecular-weight heparin (LMWH), 612 Marchia ava–Bignami disease, 676
Laryngotracheobronchitis. See Croup in antiphospholipid antibody syndrome, 326 Marijuana intoxication, 685–686
Laser peripheral iridotomy, 529–530 in pulmonary embolism, 109 Mastitis, 416
Latex allergy, cross-reactivity in, 133 LP. See Lumbar puncture (LP) MA . See Multi ocal atrial tachycardia (MA )
Latex- ruit cross-reactivity, 133 Ludwig angina, 38, 556 Maternal chorioamnionitis, 379
Latinos, 731–732 Lumbar and sacral spinal nerve roots, examination Mayo clinic preoperative guidelines, 610t
health issues a ecting, 732 ndings o , 346t McMurray test, 354
LBP. See Lower-back pain (LBP) Lumbar puncture (LP), 33, 513–514 or meniscal injury, 355
LCPD. See Legg-Calve-Perthes disease (LCPD) or bacterial meningitis, 240–241 MDD. See Major depressive disorder (MDD)
LDL cholesterol, natural product or lowering, 666 LUMBAR syndrome, 488 MDMA (3,4-methylenedioxymethamphetamine), 686
LDL level, 595 Lung abscesses, 111 MDS. See Myelodysplastic syndrome (MDS)
Lead poisoning, 184 intravenous clindamycin in, 111 Meatal stenosis, 27
symptoms o , 184 Lung cancer, screening or, 120 Mebendazole, or scabies, 249
Lead-time bias, 712 Lupus anticoagulant, 326 Mechanical bowel obstructions, 603
LEARN mnemonic, 726 Lycanthropy, 639 Meckel scan, 232
Lecithin/sphingomyelin (L/S) ratio, 391 Lyme disease, 251t, 318, 506 Meconium, delayed passage o , 376
LEEP. See Loop electrosurgical excision procedure (LEEP) ndings in, 318 Median nerve compression, 506
Lef anterior ascicular block (LAFB), 85, 85 laboratory tests or, 318–319 Medicaid, 587
Legg-Calve-Perthes disease (LCPD), 335 polymorphonuclear cells in synovial uid, 318 Medical errors, 734
initial treatment in, 335–336 primary prevention o , 319–320 prevention o , 735
poor outcome in, 335 stages o , 319 Medical utility, 689
Leg pain, in runner, 397 therapy or, 319 Medical record, in ormation in, 690–691
Leiomyosarcomas, 430 Lyme meningitis, 320 Medicare, 586–587
Lemierre syndrome, 568 Lymphadenopathy, 557 Medicare hospice bene t, 698, 699–700
Length time bias, 713t Lymph nodes, 557 services covered under, 700
Lentigo maligna melanoma, 477 Lymphogranuloma venereum, 465 Medication-overuse headache, 508
Leopold maneuvers, 387 Lynch syndrome. See Hereditary nonpolyposis colorectal Medroxyprogesterone, 428
Leprosy (Hansen disease), 505–506 cancer (HNPCC) Me enamic acid, 442
Leptin, 664 Me oquine, 252
Leser– relat sign, 495 M Megace. See Megestrol
Leukemia, acute, 193 Macroadenomas Megestrol, 706
Leukocyte adhesion de ciency, 137t dopamine agonist or, 299 Meibomian gland dys unction, 544
Leukoplakia, 554 surgery or, 300 complication o , 540
Leukotriene inhibitors, 102 Macrocephaly diagnosis o , 540
in asthma, 102 diagnosis o , 373–374 past medical history, 540
Levitra, 55 di erential diagnosis o , 374 treatment o , 540
Levodopa, 517 Macrolides, 550 Melatonin, 656
abrupt discontinuation o , 518 Macrosomia Melatonin receptor agonists, 569
side e ect o , 517–518 induction or, 432 MELD scoring system, 621
Levothyroxine, 272, 574 maternal risk rom, 432 Memantine, 584
Levothyroxine absorption, 666 risk actors or, 432 Membranoproli erative glomerulonephritis, 166
Lewy body dementia, 583 Macule/patch, 475t Memory impairment, 581
Leydig cells, 572 Magnesium salicylate, 573 Memory loss, clock drawing test or, 581
Lichen sclerosus, 449–450 Magnesium sul ate Menactra, 403
Licorice, 668 or preeclampsia, 426 Menarche, age o , 442
Lid margin, in ammation o , 540 or status asthmaticus, 22 Men, death rates in, 462–463
Lidocaine, 4, 46 Maintenance uids, 371 Ménière attack, 498
or pain control, 703 Maisonneuve racture, 357 Ménière disease
Lidocaine patch, 594 Major depressive disorder (MDD), 643. See also about, 555, 559
Li e-sustaining interventions, role o law in, 689–690 Depression treatment o , 556
Ligamentous strain, 602 antidepressants in, 628 Meningitis, 240–242. See also Bacterial meningitis
772 INDEX

Meningococcal ype B vaccinations, 403 Miliaria, 485 medications on discharge af er, 46


Meningococcemia, 377 Milk-alkali syndrome, 146 oxygen in, 41
Meniscal injury, test or, 355 Milrinone, or beta-blocker overdose, 8 pacemaker in, indications or, 45, 45t
Menomune, 403 Mineral oil, 363 and submaximal stress testing, 46
Menopause, 439 Minimal change disease, 166 thrombolytics in, 44
Menorrhagia, 429 Mini-Mental State Exam (MMSE), 581 Myoglobin, 17
MEN types, 281–282 Minimum inhibitory concentration (MIC), 241 Myopia, 536, 543
Men who have sex with men (MSM), 269 Minorities
Meperidine, 600, 702. See Demerol disparities in healthcare quality and outcomes or, N
Meralgia paresthetica, 341 732–733 Na cillin, or methicillin-sensitive S. aureus endocarditis,
6-mercaptopurine (6-MP), or in ammatory bowel in research studies, 730 247
disease, 211 Minute ventilation, 125 NAFLD. See Nonalcoholic atty liver disease (NAFLD)
Meridia. See Sibutramine Mirena IUD, 428, 429 Nail old capillary microscopy (NCM), 327
Meta-analysis, 718–719 Mirtazapine, 590, 706 Nail removal, 622
Jadad score or, 719t side e ects o , 639 Nails, dermatophyte in ections o , 482
Metabolic acidosis, 6–7 Miscarriages, spontaneous, chromosomal abnormalities Naloxone, 702, 705
and hypokalemia, 154–155 and, 431 Naltrexone, or alcoholism, 682
and respiratory alkalosis, 169 Misoprostol, 413, 421 Naproxen, 42, 334
Metabolic alkalosis, 144 Missed abortion, 431 or gout, 314
causes o , 163, 163t Mixed metabolic/respiratory acidosis, 124 Narcolepsy
chloride-responsive, 163 MMR vaccine, 365–366 diagnosis o , 519–520
interventions in, 163 Modi ed Checklist or Autism in oddlers (M-CHA ), 375 treatment or, 520
magnesium in, 163–164 Modi ed diet in renal disease (MDRD), 144 Narcotics, side e ects o , 604
with respiratory compensation, 162–163 Molluscum contagiosum, 483–484, 484 Nardil. See Phenelzine
urine studies in, 163 Moniker eczema. See Atopic dermatitis Nasal bleeding
Metal ume ever, 97 Monoamine oxidase inhibitors (MAOIs), 58, 640 about, 561
Metamucil, 95 Monoclonal gammopathy o undetermined signi cance management o , 561
Metaphyseal corner racture, 340–341 (MGUS), 189 treatment, 561
Metatarsus adductus, 341–342 Monocular diplopia, 544 Nasal congestion, 131, 132
treatment or, 342 Mononeuritis multiplex, 305, 504 Nasal steroids, 132
Met ormin, 232, 288 Mononucleosis, in ectious, 482–483 Nasal trauma, 14–15
or diabetes, 288, 289, 291, 292, 663 Mononucleosis-like syndrome, 566 Nasogastric (NG) lavage, 620
in diabetic patients with renal disease, 143 Monospot, 567 Nasotracheal intubation, blind, 622
Methadone, 673 Montelukast, 101, 102, 132 National Institute on Alcohol Abuse and Alcoholism,
overdose, 683 Montreal Cognitive Assessment (MoCA), 582 590
or pain control, 704 Morphine, 600 National Kidney Foundation, 144
Methanol overdose, 7 dosage o , 20 National Lung Screening rial, 120
omepizole and ethanol or, 8 in end-o -li e situations, 703 National Osteoporosis Foundation, 573
serum osmolality in, 7–8 intravenous, 702 National Patient Sa ety Foundation (NPSF), 729
tachypnea in, 7 or palliation o dyspnea in cardiac ailure, 700 Natrecor. See Nesiritide
Methemoglobinemia, 191 Mosaicism, 435 Natural therapy regulation, 667
treatment o , 191 Mouth sores, 561 Nausea and vomiting, 707–708
Methergine. See Methylergonovine MRI contrast, use o , 142 causes and treatment o , 707t
Methicillin-resistant Staphylococcus aureus (MRSA), MRSA cellulitis, 255 o pregnancy, 411, 669
588, 599 MS. See Multiple sclerosis (MS) Nearsightedness, 536
Methimazole, 274 Mucormycosis, 541 Neck mass in adults, di erential diagnosis o , 557t
Meth mouth, 684 Muehrcke nails, 487 Necrobiosis lipoidica, 494, 494
Methotrexate, 305–306 Multi ocal atrial tachycardia (MA ), 52 Necrotizing cellulitis, 598
and anti- olate medications, 305 ECG ndings in, 87, 88, 88 Necrotizing asciitis, 598
or in ammatory bowel disease, 211 treatment o , 88 IVIG in, 599
Methylene blue, 191 Multiple gestations, complications o , 421–422 recommendations or, 599
Methylergonovine, 421 Multiple-legged cane (quad cane), 580 ype II, 599
Methylphenidate, 645 Multiple myeloma, 188 Necrotizing (malignant) otitis externa, 552
Metoclopramide, 231, 289, 517, 707 work-up or diagnosis o , 188 Needle decompression, 617
in GERD, 201 Multiple sclerosis (MS), 510 Needle drainage and compressive dressing, 561
Metoprolol exacerbation o , 511 Needle thoracostomy, in tension pneumothorax, 30
or accelerated junctional rhythm, 80, 80 , 81 primary progressive, 510 Ne azodone, 639–640
or heart ailure, 61, 64 relapsing–remitting, 510, 511 Negative predictive value (NPV), 720, 721
or premature atrial contractions, 58 tests or diagnosis o , 510–511 Neisseria meningitides, 33
Metronidazole, in Clostridium di cile in ection, 239–240 treatment o , 511 Neonatal herpes simplex virus in ection, 381
Miconazole, 465 Multivariate analysis, 718 Neonatal hyperbilirubinemia
Microadenoma, 299 Multivitamins, 661 risk actor or, 364
Microcytic anemia, 183 Mumps, 563 treatment or, 365
Micrographia, Parkinson disease and, 516 Murmurs, benign, 368, 369t Neonatal polycythemia, 363–364
Micronutrient de ciencies, 665 Murphy sign, 616 causes o , 363
Micronutrients, 533 Myasthenia gravis, 546 treatment or, 364
Microscopic hematuria, 147 causes, 512 Neonatal respiratory distress syndrome (RDS), 390–391
evaluation o , 147t diagnosis o , 512 risk actors or, 391
and gross hematuria, 147 exacerbation o , 512 sur actant in, 391
screening or, 148 therapy or, 513 Neovascular glaucoma, 529
stones and, 147, 148 worsening o , 513 Neovascularization o optic nerve, 534
Microscopic polyangiitis, 115 Mycobacterium tuberculosis, 244 Nephrotic syndrome, 152, 545
Microvascular decompression (Janetta procedure), 522 Mycoplasma pneumoniae, 121 causes o , 152–153, 153t
Middle ear e usion, 548 Myelodysplastic syndrome (MDS), 189 criteria or, 152t
Midgut volvulus, 387 pancytopenia and, 189 evaluation in, 153
Midodrine, 532 Myeloperoxidase de ciency, 137t urine sediment in, 152
Mid-parental height (MPH), calculation o , 397 Myocardial in arction (MI), 40 Nerve conduction studies (NCV), 505
Mi eprex. See Mi epristone antiarrhythmics in, 46 Nesina. See Alogliptin
Mi epristone, 444 aspirin in, 41 Nesiritide, 63
Migraine headaches, 506–507, 522 chest pain and, 40–41 Neuroblastomas, 554
with aura, 507, 559 clopidogrel, 41–42 Neurogenic obstruction, 603
criteria or diagnosis o , 506, 506t, 507t enoxaparin in, 45 Neuroimaging, in migraine patients, 509
preventive treatment or, 509 heart block associated with, 45, 45t Neuroleptic malignant syndrome (NMS), 275, 518
treatment o , 507, 507t, 668 intervention in cath lab with P CA and/or stent Neurontin. See Gabapentin
without aura, 507 placement in, 43–44 Neuropathic pain, 505, 702
INDEX 773

Neuropathic ulcers, 481 Occult hip ractures, 359–360 about, 573


Neutropenia, 178 Occupational therapists (O ), 592 causes o , 574
Neutropenic ever, 178 OCPs. See Oral contraceptive pills (OCPs) de nd, using DEXA scan results, 574
antibiotics or, 178 , 178t Octreotide, 232, 708 increasing risk o , 574
Neutrophil counts, in children, 189 Ocular pain and irritation, 536 initial therapy or, 575
Nicotine gum, 679 Odynophagia, 202 secondary, 574
Nicotine inhaler, 679 Oil cysts, 607 soy or, 667t
Nicotine nasal spray, 679 Olanzapine, 647, 658, 663 treatment o , 574–575
Nicotine patch, 679 Older patient, care o , 569–596. See also Elderly Otitis externa
Nicotine replacement therapy (NR ), 679 Oliguria, 156 rst-line treatment or, 551
Nicotine withdrawal, 681 Omeprazole, in GERD, 201 necrotizing (malignant), 552
Nighttime rituals, 569 Omnibus Budget Reconciliation Act o 1987 (OBRA), 589 Otitis media, 548. See also Acute otitis media (AOM)
NIH Chronic Prostatitis Symptom Index questionnaire, Ondansetron, 411 Otitis media with e usion (OME)
457 or reducing vomiting, 31 about, 548
Nitroglycerin Onglyza. See Saxagliptin bene ts or patients with, 551
in HF exacerbation, 62 Onychocryptosis, 622 Otomycosis, 552
hypotension with, 42–43 Onychomycosis, 482 Otosclerosis, 555
N-methyl d-aspartate (NMDA), 584 Open-angle glaucoma Ottawa oot and ankle criteria, 356
Nocturnal hypotension, 532 risk or, 530 Ottawa knee rules, 354–355
Nodular melanoma, 477 treatment or, 530 Outtoeing, 337
Nodule, 475t Open biopsy, 557 Ovarian cancer, 446, 573, 607
NO HOPE, mnemonics or suicide risk assessment, 654t Open racture, 335t Ovarian mass, 446
Nonalcoholic atty liver disease (NAFLD), 226–227 Open globe, 534 Ovarian torsion, lower abdominal pain and, 28
Nonalcoholic steatohepatitis (NASH), 226 Open-loop obstructions, 603 Over ow incontinence, 433
Nonarteritic anterior ischemic optic neuropathy Ophthalmia neonatorum, 538 Over-the-counter remedies, or nausea and vomiting o
(NA-AION), 532 Ophthalmologic examination, 533 pregnancy, 411
Noncontact training, 567 Opiates, 5t Overweight, 403
Nondihydropyridine calcium channel blockers, 144 in end-o -li e situations, 703 Oxazepam, 675
Nondisclosure, 694 or pain control, 702 Oxybutynin hydrochloride, 434
Non-Hodgkin lymphoma (NHL), 195 switching or dosing, 704 Oxycodone, 232, 505, 672
Noninvasive positive pressure ventilation, 104 Opioid addiction, in pregnancy, 683–684 Oxytocin, 413, 421
Nonmalignant nodes, 556 Opioid bowel syndrome, 235 or labor induction, 417
Nonproli erative diabetic retinopathy, 533 Opioid-induced hyperalgesia (OIH), 322, 703–704
Nonsteroidal anti-in ammatory drugs (NSAIDs) Opioid intoxication, 672 P
and asthma, 102 DSM-5 criteria or, 672 Pacemaker
or dysmenorrhea, 442 Opioid withdrawal, 672–673 in myocardial in arction, 45, 45t
or endometriosis, 445 Optic nerve in sick sinus syndrome, 69
gastric ulcers by, 207 glaucomatous, 531 PACs. See Premature atrial contractions (PACs)
and GI event risk, 206 hemorrhages, 531 PAD. See Peripheral arterial disease (PAD)
or gout, 314 neovascularization o , 534 Paget disease o breast, 609
or headaches, 507 normal, 531 Pain
or menorrhagia, 429 Optic nerve head cupping, 530 back, 329
or pericarditis, 51 Optic neuritis, 531 in cancer patients, 702
Nonstress tests, 415, 417, 432 Oral carbonic anhydrase inhibitor, 529t chest, 30–31
Nonsustained ventricular tachycardia, 613 Oral contraceptive pills (OCPs) leg, 397
Nonulcer dyspepsia, 204 backup method af er starting o , 410 neuropathic, 505, 702
detailed dietary history in, 204 bene ts o , 409 opiates in, 702
management o , 205 contraindications to, 410 pelvic, 444, 445
natural history o , 205 in dysmenorrhea, 442 social and emotional, 705
symptoms o , 204 and headaches, 409–410 visceral, 702
Nonverbal communication, 728 ri ampin and, 410 Pain control, 600
Norepinephrine, 255 and weight gain, 409 medication, 587
Normal pressure hydrocephalus (NPH), 516, 581, 582 Oral creatine, 669 Pain ul bladder syndrome, 168
Normal saline, in hypernatremia, 156 Oral psyllium, 606 diagnosis o , 168
Nortriptyline, 629, 641, 680, 705 Oral sodium phosphate solution, 363 treatment o , 168–169
Norwalk virus, 558 Orbital cellulitis, 541 Paleolithic diet, 664
Nose, radiographs o , 14 Orexin receptor antagonists, 569 Palivizumab, or RSV in ection, 386
NSAIDs. See Nonsteroidal anti-in ammatory drugs Organic dust toxicity syndrome (OD S), 97 Palliative care, 699
(NSAIDs) supportive care or, 97 dyspnea in, 700
Nuchal translucency, 411 Orlistat, 404, 664 early, 701
Null hypothesis, 713 Oropharyngeal cancers, HPV in ection and, 554 and hospice care, 699
Nulliparity, 608 Oropharyngeal dysphagia, 202 Palpable masses, 608
Number needed to harm (NNH), 717 Oropharynx, 554, 565 Palpitations, evaluation in, 57
Number needed to treat (NN ), 549, 715–717 Orthostatic proteinuria, 151 Pancreatic enzymes, 616
Nummular, 475t Oseltamivir, 237, 238 Pancreatic pseudocyst, 230
Nursemaid’s elbow, 339–340 Osgood–Schlatter disease, 336 Pancreatitis, 228
Nursing home patient with weight loss, 570 Osler nodes, 246 causes o , 229
Nutritional de ciencies, 605 Osmolar gap, 7–8 drugs associated with, 229t
Nutritional supplements, 706 OsmoPrep. See Oral sodium phosphate solution treatment in rst day o , 229
Osteitis pubis, 602 Panic attack, 631
O Osteoarthritis, 333 symptoms o , 634
Obesity. See also Weight loss glucosamine sul ate or, 670 Panic disorder
in adolescence, 403 o hip, 580 criteria or, 634
and brain natriuretic peptide level, 60 SAM-e in treating, 668 mistakes made in treatment o , 635
complications o , 663 secondary causes o , 323 prognosis or, 634
de ned, 662, 662t o wrist, 360 recommendations in, 635
drugs associated with, 663t Osteomyelitis, 342 treatment o , 635
and evaluation or hyperlipidemia, 403 antibiotics or, 393–394 Papanicolaou (Pap) smear, 434, 436, 722
prevention o , 403–404 blood culture in, 343 Papillary carcinoma, 278
Obstructive shock, 618t in child, 343 Papilledema, 531
Obstructive sleep apnea, 520 diagnosis o , 393 Pap smears, 573
Obturator sign, 614 in in ants, 393–394 Papule/plaque, 475t
Occlusive dressing in patients with sickle cell disease, 342 Paracentesis, large-volume, 224
or open sucking chest wound, 30 treatment o , 343 albumin replacement ollowing, 224–225
removal o , 30 Osteoporosis Paraphimosis, 27
774 INDEX

Parapneumonic e usion, 122 causes o , 504, 504t 23-valent pneumococcal polysaccharide vaccine
Parathyroidectomy, 281 electrophysiological testing in, 505 (PPSV23), 238
Parkinson disease, 516 evaluation in, 504 Pneumococcal vaccination, 238, 572
cardinal eatures o , 516 heavy drinking and, 674 Pneumocystis jiroveci pneumonia, 260, 262
dementia in, 518 in ectious cause o , 505–506 and pneumothorax, 263
depression in, 518 oxycodone in, 505 Pneumonia, 35. See also speci c type
diagnosis o , 517 Peripheral vascular disease, 327 neonatal, 391, 392
drug-induced, 517 Peripheral vertigo, 560, 560t Pneumonia severity index, 121
levodopa in, 517–518 Peritoneal adhesions, 604 Pneumonic plague, 129
REM sleep disorder in, 519 Peritonsillar abscess, 566, 567 Pneumothorax, 116, 263
treatments o , 517 Periungual broma, 487 Podagra, 316
visual contrast sensitivity with, 519 Permethrin cream, 248, 254 Poiseuille’s law, 619
wearing-o phenomenon, 518 Permethrin insecticide, 252 Poisoning/overdose
Parkinsonism, 582 Permissive hypercapnia, 124 beta-blocker, 8
Parkinson plus syndromes, 517 Pernicious anemia, 187–188 calcium channel blocker, 8
Parkland ormula, 618, 619 Per-protocol analysis, 714 iron, 31–33
Paronychia, 251, 487 Persistent depressive disorder, 629–630 methanol, 7–8
Parotitis, 563, 564 Personality disorders, clusters o , 653 theophylline, 1–2
Paroxetine, 639, 642 Pes anserinus bursitis, 344 tricyclic overdose, 2–4
and birth de ects, 640 Petit mal seizures, treatment o , 503 Policosanol, 667t
Paroxetine and weight gain, 664 PFA-100 (Platelet Function Analyzer) tests, 173–174 Poliomyelitis-like syndrome, 515
Paroxysmal hemicrania, 521–522 PFPS. See Patello emoral pain syndrome (PFPS) Polyarthritis, 324
Paroxysmal supraventricular tachycardia (PSV ), 54 PHACES syndrome, 488 Polyarticular in ammatory arthritis o unclear etiology,
Partial thromboplastin time (P ), 173, 176 Phagophobia, 202 303
Parvovirus B-19 in ection, 304, 307, 382–383 Phalen sign, 360, 361 Polycystic ovarian syndrome (PCOS), 404, 447–448
complications o , 382 Pharyngeal swelling, treatment o , 567 clinical eatures o , 448
and sickle cell disease, 19 Pharynx, posterior, swelling in, 38 management o , 449
slapped cheeks in, 382 Phase-advance type sleep disorder, 656 Polyethylene glycol, 235, 363
Patching, 539 Phenelzine, 640 Polymyalgia rheumatica (PMR), 308
Patch testing, 480 Phenobarbital, 524 criteria or diagnosis o , 308t
Patello emoral pain syndrome (PFPS), 336 Phenol, 622 elevated ESR in, 308
treatment or, 336 Phentermine, 664 physical examination ndings o , 308
Patent ductus arteriosus (PDA), congenital rubella and, Phenytoin, 4, 277 prednisone in, 308–309
380 side e ects with, 502 prognosis o , 309
Patient-controlled analgesia (PCA), 604 Pheochromocytoma, 275 relapse o , 309
Patient Health Questionnaire-9 (PHQ-9), 657 Philadelphia chromosome, 193 Polymyositis, 332, 332t
Patient’s culture, knowledge o , 726 Phimosis, 27, 460 Polymyxin, 539
Patiromer, 37 Phosphodiesterase inhibitors, or pulmonary Polymyxin/ neomycin combinations ear drops, 551
PCOS. See Polycystic ovarian syndrome (PCOS) hypertension, 90 Polysaccharide pneumococcal vaccine, 572
PCP intoxication, 686 Phototherapy, 365 Polysomnography, 519
PCSK9 (proprotein convertase subtilisin kexin type 9), 95 PHQ-9. See Patient Health Questionnaire-9 (PHQ-9) Ponstel. See Me enamic acid
PE. See Pulmonary embolism (PE) Physician-directed head positioning, 560 Portal hypertension, 221–222
Peds solution, 371 Physician–patient relationship, 691, 692 Portal vein thrombosis
Peginter eron al a-2 a, 277 Physiologic gynecomastia, 463 anticoagulation or, 224
Pegvisomant, 301 Picaridin-containing insect repellents, 252 in cirrhosis, 224
Pelvic binder, 600 PID. See Pelvic in ammatory disease (PID) Portola Phamaceuticals. See Annexa
Pelvic examination, annual, 573 Pill esophagitis, 207 Positional therapy, 520
Pelvic in ammatory disease (PID), 437 Pilocarpine, 529 Positive end-expiratory pressure (PEEP), 125
antibiotic therapy or, 437 Pilocarpine eye drops, 15 Positive predictive value (PPV), 719–721, 723
diagnosis o , 437 Pinguecula, 545 Positron emission tomography (PE ) testing, 59
etiology o , 438 Pinworm in ection, 249 Postconcussion syndrome, 406. See also Concussion
indications or hospitalization, 437t Piperacillin/tazobactam, 228 Posterior capsular opacity, 546
lower abdominal pain in, 436–437 Pittsburgh rules, 354–355 Posterior chamber intraocular lens (PCIOL) implants,
potential sequelae o , 437 Pituitary surgery, transsphenoidal, 300 546
Pelvic muscle exercises, 434 Pityriasis rosea (PR), 483, 483 Posterior at pad sign, 359, 359
Pelvic pain, chronic, 444, 445 Placenta previa, 412 Posterior rib ractures, 341
Pelvic ultrasound, or pelvic mass, 446 Planned Parenthood v. Casey, 696 Postin ectious glomerulonephritis, 167
Pemphigus vulgaris, 488–489 Plantar ascia rupture, 358 Post-Lyme disease syndrome, 320
Penicillin, 565, 566 Plantar asciitis, 358 Postpartum depression, 642
allergy, 133 treatment or, 358 Postpartum hemorrhage, 420–421
resistance to, 241 Plantar warts, 484 Postpericardotomy syndrome, 50
Pentasa, 211 Plasma exchange, 513 Postprandial hypoglycemia, 297
Pentosan polysul ate sodium (Elmiron), 168 Plasma potassium level, 613 Post-streptococcal glomerulonephritis, 166
Pentoxi ylline, 70, 224 Plasma renin, 284 Posttraumatic iritis, 16
Per Advanced rauma Li e Support guidelines, 617 Plasmodium alciparum, 251 about, 539
PERC rules, 107 Plasmodium ovale, 251 treatment o , 539
Per ormance bias, 713t Platelet dys unction, due to uremia, 166 Post-traumatic stress disorder (P SD), 631
Periactin. See Cyproheptadine Platelet trans usion, 620 criteria or, 631
Pericardiocentesis, 51 Pletal. See Cilostazol symptoms o , 631–632
Pericarditis Pleural e usion, 116 treatment or, 632
constrictive, 60, 64 bronchoscopy in, 117–118 Potassium hydroxide (KOH), o skin scraping, 480
ECG in, 50–51 categorization o , 117, 117t Povidone-iodine, 11
indomethacin or, 51 chest tube drainage o , 118 PPI. See Proton pump inhibitor (PPI)
Perilymphatic stula, 559 diagnosis o , 116 PR. See Pityriasis rosea (PR)
Perilymph leak, 560 exudates, 117, 117t Pradaxa. See Dabigatran
Perimenopausal vasomotor symptoms, treatment o , 666 Light’s criteria, 117 Praluent, 95
Perioperative care, anticoagulation approaches in, 198 loculated and septated, 118 Pramipexole
Perioral dermatitis, 489–490, 490 thoracentesis in, 116–117 or Parkinson disease, 517
Peripheral arterial disease (PAD), 69 transudates, 117, 117t or restless legs syndrome, 521
ankle-brachial indices af er exercise in, 70 Pleural riction rub, 305 Prasugrel, 42
diagnostic tests or, 70 Plica syndrome, 336 Prazosin, 632
invasive treatment o , 70–71 PMR. See Polymyalgia rheumatica (PMR) Precocious puberty, 372
pharmacologic therapy or, 70 Pneumatic otoscopy, 548, 549, 550 Prednisolone, 224
Peripheral neuropathy, 504 Pneumococcal conjugate vaccine (PCV 13), 238, 572 Prednisone, 560
alcohol use and, 504, 674 Pneumococcal meningitis, dexamethasone in, 241 or bullous pemphigoid, 489
INDEX 775

and male sexual unction, 472 neovascularization o optic nerve, 534 Purpura, 475t
or myasthenia gravis, 513 treatment o , 534 Purulent discharge, 538
or polymyalgia rheumatica, 308–309 Promethazine, 31, 708 Pustule, 475t
Preeclampsia, 425–426 Propecia. See Finasteride p-value, 713
aspirin in, 424 Prophylactic antibiotics, 537, 551 Pyelonephritis, 233
BP control in, 426 Propranolol, 509 Pyelonephritis, and hospital admission, 26
and delivery, 426–427 or hyperthyroidism, 274 Pyloric stenosis, 386
diagnostic criteria or, 426t Propylthiouracil (P U), 274 clinical mani estations o , 386
24-hour urine or protein level in, 425 Prostacyclin, or pulmonary hypertension, 90 string sign, 386
magnesium sul ate or, 427 Prostaglandin analogs, 529t treatment or, 387
in pregnancy, 423–424 Prostaglandin E1 agents, 413 Pyoderma gangrenosum, 481–482, 496
recurrence o , risk o , 423–424 Prostaglandin E2 agents, 413 Pyogenic granuloma, 488, 488
risk actors or, 424 Prostate biopsy, 469 Pyrantel pamoate, 249
with severe eatures, 426 Prostate cancer, 454 Pyridostigmine, 512
treatment or, 426–427 and BPH, 468–469 Pyridoxine, 244
Preexisting dementia, 577 screening or, 469 or isoniazid-induced seizures, 9
Pre-exposure prophylaxis (PrEP), HIV, 269 5-year survival rate o , 469 Pyrosis, 204
Pregabalin, 705 treatment options or, 470 Pyuria, 243
Pregnancy Prostate-speci c antigen (PSA) testing, 454, 469
ACE inhibitors in, 427 2 glass pre- and post-prostatic massage test, 457 Q
antiepileptic drugs in, 502–503 Prostatitis, 456–457 Quad cane, 580
cardiovascular physiology in, 428 bacterial, 457 Quetiapine, 658
corticosteroids in, 422 diagnosis o , 456 Quincke sign, 35
deep venous thrombosis in, 197 Prostatitis, acute, 26 Quinidine, 4
depression in, 642 treatment or, 26 Quinolone, 26
diabetes in, 411 Protean, 577
Graves’ disease in, 274 Protein C de ciency, 196 R
headache treatment in, 509 Protein S de ciency, 196 RA. See Rheumatoid arthritis (RA)
heterotopic, 439 Proteinuria, 151 Rabies prophylaxis, 10–11, 11t
HIV/AIDS and, 263–265 ACE inhibitors or, 151–152 Race, 732
hypertension in, 425–427 orthostatic, 151 Radial head racture, 359
opioid addiction in, 683–684 transient, 151 treatment o , 359
post-term, 416 Prothrombin time (P ), 176 Radial nerve compression, 506
preeclampsia in, 423–424 Proton pump inhibitor (PPI) Radioactive CO2 blood test, 207
prenatal tests in, 411–412 in CRES syndrome, 203 24-hour radioactive iodine uptake (RAIU) test, 277
quantitative hCG in, increase in, 438 in GERD, 201 Radioallergosorbent testing (RAS ), 131
weight gain in, 451, 451t side e ects o , 205 Radiocontrast media, lower-osmolality, 134
Pregnancy loss, recurrent, 450 Provider order entry (POE), 716 Radiography, 558
causes o , 450 Pseudoesotropia, 526 Radiologic studies, 608
evaluation o , 450–451 Pseudogout, 316 Ramsay–Hunt syndrome, 563
Pregnancy-related depression, 642 associated conditions with, 317 Randomized, controlled study, 718
Pregnancy test, abdominal pain and, 25 Pseudohyponatremia, 159 Ranitidine, or bee sting reaction, 23
Premature atrial contractions (PACs), 58 Pseudomonas, 551, 623 Ranson criteria, 229, 230, 230t
metoprolol or, 58 Pseudomonas aeruginosa, 33, 121, 342 Rape trauma syndrome, 447
mitral stenosis and, 58 Pseudo-obstruction, 603 Rapid shallow breathing index (RSBI), 125
Premature ovarian ailure, 301 Psoas sign, 614 Rasburicase, or prevention o tumor lysis syndrome, 180
Premenstrual dysphoric disorder (PMDD), 443 Psoriasis, lesions o , 324 Raynaud phenomenon, 327
Premenstrual syndrome (PMS), 443 Psychogenic incontinence, 434 evaluation and management o , 328
diagnosis o , 443 Psychosis ndings in, 327
treatment options or, 443, 443t causes o , 654, 654t ischemic crisis related to, 329
Preoperative studies, 610t symptoms o , 654 primary, 327
Presbycusis, 555 Psychotherapy secondary, 327, 328
Preseptal cellulitis, 541 or panic disorder, 635 therapy or reducing requency o attacks o , 329
Pressure ulcer, 482 Psychotic depression, 637 Reactive airway disease, 97
caused by unrelieved pressure, 571 Psyllium, 606 Reactive arthritis, 330, 331
prevention o , 571 or lowering LDL cholesterol, 666 Rebound acid hypersecretion, 202
stages, 571, 571t Pterygium, 545 Rebound headache, 508–509
treatment o , 571 P SD. See Post-traumatic stress disorder (P SD) Recommended daily allowance (RDA), 661
Presyncope, 559 Puberty Rectal bleeding, 232–233, 605
Preterm birth, risk actors or, 423 age o onset o , 396 Rectal examination, 363
Priapism, 465–466 delayed, 372 Recurrent appendicitis, 615
drugs associated with, 466t early, 372 Recurrent pregnancy loss, 450
and erectile dys unction, 466 rst sign o , in boy, 396 Red cell casts, in urine, 152
etiology o , 466 Pubic ramus racture, 16 Redness o eye, 535
management o , 466 Pulmonary anthrax, 2 Re eeding syndrome, 651
Primary amenorrhea, 430 Pulmonary contusions, 617–618 Regional wall motion abnormality (RWMA), 60
Primary biliary cirrhosis, 225 Pulmonary embolism (PE) Reglan. See Metoclopramide
Primary hyperparathyroidism, 280 anticoagulation in, 109 Rehabilitation
parathyroid surgery in, 281 chest radiographs in, 107 about, 591
Primary prevention, 718 C angiography (C A) in, 108 goals o , 592
Primary survey, 28 ECG nding associated with, 107 Reiter syndrome, 465
Principle o double e ect, 689 and pain, 42 Relative a erent pupillary de ect (RAPD), 526
Probenecid, 315 PERC rules or, 107, 108 Relative risk reduction, 717
Problem drinking, CAGE screening or, 591t risk actors or, 107 Remeron. See Mirtazapine
Procainamide, 4, 67 tests or, 107–108 Remicade. See In iximab
Prochlorpemazine, or headaches, 507 Wells criteria or, 108, 108t Renal artery stenosis (RAS), screening test or, 75
Progesterones, 575 Pulmonary hypertension, 88, 89 Renal biopsy, 147, 166
Progestin challenge, 398, 451 tests or causes o , 88–90 contraindications to, 153
Progressive supranuclear palsy, 516, 517 treatment o , 90 indications or, 153, 153t
Progressive visual loss and glare, 544 Pulmonary rehabilitation (PR), 105 Renal calcium wasting, 574
Prokinetic agents, 595 Pulmonic murmur, 369t Renal osteodystrophy, 165
in GERD, 201 Pulse oximetry, 9 Renal tubular acidosis (R A), 143
Prolactinoma, 298–299 Pulsus paradoxus, 51 type 4, 143
Prolia. See Denosumab Punch biopsy, 496, 622 Repatha, 95
Proli erative diabetic retinopathy Purple glove syndrome, 502 Reset osmostat, and hyponatremia, 160
776 INDEX

Respiratory distress, causes o , 35 Saline in usion Selection bias, 713t, 714


Respiratory uoroquinolones, 559 in dehydration, 370 Selective immigration, 732
Respiratory isolation room, 110–111 in hypernatremia, 156 Selegiline, or Parkinson disease, 517
Respiratory syncytial virus (RSV), 385 in jugular venous distention and peripheral edema, 51 Sel -disclosure, 693
bronchiolitis by, 385 in rhabdomyolysis, 17 Semmes–Weinstein 10 g mono lament sensory
diagnosis o , 385–386 Salivary duct stones, 564 examination, 292
prevention o , in high-risk in ants, 386 Salmeterol, 101, 102 Sensipar. See Cinacalcet
Restless legs syndrome, 520–521 Salsalate, 42 Sensitivity, 719
pramipexole or, 521 Salvia, 686 and speci city, 715
workup in, 521 SAM-e in depression treatment, 668 Sensorineural hearing loss
Restraints, 585 Samsca. See olvaptan about, 555, 555t
Retinal detachment Sarcoidosis, 115 drugs causing, 573
management o , 543 ACE levels in, 116 Septal hematoma, 14
risk actors or, 543 cardiac involvement with, 115 Septic arthritis, 311, 334–335, 343
Retinal hemorrhages, 374 ndings with, 115 causative organism, 344
Retinoids, or acne, 491 spontaneous resolution o , 116 management o , 312–313
Retropharyngeal abscess, 38, 563 Saturday Night Palsy, 506 mortality rate o , 313
Retropharyngeal space swelling, 35 Savaysa. See Edoxaban physical examination in, 311
Re-warming Saw Palmetto, 669 risk actors or, 313
or bradycardia, 19 Saxagliptin, 292 synovial uid analysis in, 312, 343, 344t
o extremities, 19–20 SBP. See Spontaneous bacterial peritonitis (SBP) treatment o , 312
rostbite, 20 Scabies, 248, 478 Septic pelvic thrombophlebitis, 416
methods o , 19 Scotch tape test, 249 Septic thrombophlebitis, 568
Reye syndrome, 387 treatment o , 249 Sequential compression devices (SCD), 610
aspirin use and, 387 Scaphoid racture, 347–348 Seroconversion, 588
treatment o , 388 treatment o , 348 Serologic testing, or H. pylori, 206
Rhabdomyolysis, 17 Scapula racture, 350 Serotonin–norepinephrine reuptake inhibitors (SNRIs),
adverse consequence o , 17 SCFE. See Slipped capital emoral epiphysis (SCFE) 629
renal ailure in, 155 Schatzki ring, 204 or anxiety disorders, 635
treatment or, 17–18 Scheuermann disease, 347 or bromyalgia, 323
Rh-D antibodies, 431 Schistocytes, 193, 194 or neuropathic pain, 505
Rheumatic ever, 307 Schizophrenia, 655 Q prolongation by, 629, 636
Jones criteria or, 307 antipsychotics or, 655 Serotonin reuptake inhibitors (SSRIs)
Rheumatoid arthritis (RA), 304, 304t course o , 655–656 or adolescent depression, 401
biologicals or, 307 prevalence o , 655 adverse e ect o , 628
DMARDs or, 304–306 Schneiderian papillomas, 554 in depression, 628, 630
remission in, 306–307 SCIWORA syndrome (spinal cord injury without discontinuation syndrome, 638–639, 639t
Rhinosinusitis, 557–558 radiologic abnormality), 6 and hyponatremia, 161
cause o , 558 management, 6 interactions with cytochrome P450 enzymes, 639
treatment o , 558 natural history, 6 and MAOIs, 640
Rhinovirus, 558 physiology, 6 in panic disorders, 635
Rho(D) immunoglobulin, 175 Scleritis, 545 in pregnancy, adverse e ects o , 642
RhoGAM, 431 Scleroderma, 327, 328 Serotonin syndrome, 640
Rib ractures, 116 Scleroderma-like syndrome, 142 Sertraline, 161, 657–658
Rickettsia rickettsii, 250 Sclerosing cholangitis, 225 Serum-ascites albumin gradient (SAAG), 222
Ri ampin, 410 Scoliosis, 339 high, 222
Ri apentine, 244 bracing or, 339 low, 222
Right bundle branch block (RBBB), 64, 86 , 87 detection o , 338–339 portal hypertension and, 222
Right ventricular in arct, 42–43 idiopathic, 339 Sevelamer, 165
Rinne test, 554, 555 management plan or, 339 Severe combined immunode ciency syndrome, 137t
Risedronate, 575 screening or, 339 Sex hormones, 572
Risperidone, 585t Scopolamine, 530, 700 Sexual assault, 446–447
Ritodrine, 422 Scotch tape test, 249 orensic evaluation in, 447
Rivaroxaban, 109, 195, 196, 198 Screening Sexual dys unction, psychotropics and, 641
in non-valvular atrial brillation, 54 or adolescent substance use, 401 Sexual health history, 734
Rocky Mountain spotted ever (RMSF), 250, 251t, 377 or chlamydia and gonorrhea in ection, 445, 449 Shave biopsy, 622
Rocuronium, 37 or colorectal cancer, 208, 209 Sheehan syndrome, 421
Ro umilast, or COPD, 105 or diabetes in pregnancy, 411 Shock
Rome criteria or constipation, 594 or diabetic kidney disease, 140 categories o , 618t
Romiplostim, 175 or hearing impairment, 591 causes o , 618
Roos test, 352 or hepatitis C, 217 Short stature, evaluation o , 396
Ropinirole, or Parkinson disease, 517 or mood disorders in teenagers, 401 Shoulder dislocation, 16
Rosacea, 540 or prostate cancer, 729 Shoulder dystocia, 433
Roseola in antum, 383 Screening or breast cancer, 573 risk actors or, 433
Rotational de ormity, 349 Scrotal mass, ultrasound o , 458 Shy–Drager syndrome, 517
Rotator cu , 351 Sea ood, allergy to, 134 SIADH. See Syndrome o inappropriate secretion o
tears, 350, 351 Seborrheic blepharitis, 540 anti-diuretic hormone (SIADH)
Rotator cu tendinosis, 351 Seborrheic dermatitis, 323–324, 477 Sialogogues, 564
Roth spots, 246 Seborrheic keratosis, 494 Sialolithiasis (salivary duct stones), 564
Roux-en-Y, 604 Secondary osteoporosis, 574 Sialolithotomy, 564
Rovsing sign, 614 Secondary prevention, 718 Sibutramine, 404
RSV. See Respiratory syncytial virus (RSV) Second-degree heart block, type I, 69, 77, 77 Sicca symptoms, in elderly, 321
Rubber-band ligation, 606 Sedative–hypnotic drugs, 590 Sickle-cell disease, 186
Rubella, congenital, 380 Sedatives, 585 and acute sequestration syndrome, 19
cardiac lesions associated with, 380 Seidel test, 535 aplastic crisis in, 19
Ruptured globe, 534, 535 Seizures, 501 eatures o , 186
Ruptured plantaris muscle, 341 absence, 523 homozygous SS, 186
antiepileptic drug or, 502 mani estation o , 19
S concussion and, 406 and pregnancy, 187
SAAG. See Serum-ascites albumin gradient (SAAG) evaluation o , 501–502 Sick sinus syndrome, 69
SAD PERSONS, mnemonics or suicide risk assessment, ebrile, 523–524 Sildena l, 467
654t grand mal, 503 alpha-blockers and, 55
SAFER mnemonic, 729 in theophylline toxicity, 2 and antihypertensive drugs, 55
Sa ety or older adults, 591 tricyclic-induced, 4 Simethicone, 384
Salicylates, 573 workup o , 523–524 Sinus headaches, 521
INDEX 777

Sinusitis, 521, 541 physical ndings in, 330t Substance use disorder, criteria or, 675
Sitagliptin, 292 therapy or, 330 Subthalamic nucleus stimulation, 517
Sixth nerve palsy, 526 Spondylolisthesis, 346–347 Succinylcholine, 37
Size–date discrepancy, 432 Spondylolysis, 346, 347 Sucking blister, 380–381
Sjögren syndrome, 321 Spontaneous bacterial peritonitis (SBP), 222, 223 Sudden cardiac death (SCD), in young athletes, 399
biopsy in, 321 common organisms in, 223 Sudden in ant death syndrome (SIDS), 388–389
diagnosis o , 321 treatment o , 223 Suicide, 643
mani estations o , 321 Spoon nail. See Koilonychia assessment o risk level, 652
therapy or, 321 Sport Concussion Assessment ool (SCA -3) in general population, 653–654
SJS. See Stevens-Johnson syndrome (SJS) questionnair, 406 mnemonics or risk assessment, 654t
Skeletal surveys, 341 Sprain o anterior talo bular ligament, 355 risk actors or, 652t
Skin lesion removal, 622 Squamous cell carcinoma, in transplanted, risk o , in physicians, 651
Skyla, 429 immunosuppressed patients, 481 white males and death rom, 466–467
SLE. See Systemic lupus erythematosus (SLE) SSRIs. See Serotonin reuptake inhibitors (SSRIs) Sul asalazine, 511
Sleep apnea, and cor pulmonale, 65 SSSS. See Staphylococcal scalded skin syndrome (SSSS) contraindication to use o , 211
Sleep history, 519 Staphylococcal blepharitis, 540 Sul onylureas, 292, 574
Sleep hygiene, 569, 570 Staphylococcal scalded skin syndrome (SSSS), 486, Sumatriptan, 507, 508
Sleep myoclonus, 519 486 Super cial spreading melanoma, 477
Sleep paralysis, 520 Staphylococcus aureus, 238, 416, 541, 551, 564 Superior mesenteric artery (SMA) syndrome, 208
Sleep problems, sources o , 570 Stasis dermatitis, 480–481, 481 Supplemental ber, 595
Sleep studies, 519, 641–642 Statin-induced myopathy, 94 Supplementation with multivitamins, 661
Slipped capital emoral epiphysis (SCFE), 335, 337 Statins Supplements, per ormance-enhancing, 405
diagnosis o , 337–338 af er acute coronary syndrome, 46 Supraclavicular nodes, 557
treatment o , 338 candidate or, 92, 92t, 93 Supracondylar racture, 359
Slit-lamp examination, 528, 535, 539 af er myocardial in arction, 46 Suprapubic cystostomy, 601
Small bowel disease, 660 Steeple sign, 35 Supratrochanteric bursitis, 341
Small bowel obstruction (SBO), 603 S -elevation myocardial in arction, 44, 44t, 46 Sur actant therapy, 391
Smallpox, 2 Steroid hormones, 663t Surgical iridectomy, 530
Smith racture, 347 Steroids, 537, 539, 540, 562, 567 Suvorexant, 520, 569
Smoking, 532, 574, 610 anabolic, use o , 405 Swan–Ganz catheters, 618
Smoking cessation aid, 679–680 or asthma, 101 Sympathomimetic, 5t
Smoking cessation, heart attack risk, 680 or asthma exacerbations, 22 Syncopal episodes, 501
Snellen eye chart, 528 or atopic dermatitis, 479 Syndrome o inappropriate secretion o anti-diuretic
SNRIs. See Serotonin-norepinephrine reuptake inhibitors or community-acquired pneumonia, 123 hormone (SIADH), 160, 161
(SNRIs) or Crohn disease, 211 sodium chloride tablets in, 162
Social Security bene ts, 586 or hypersensitivity pneumonitis, 98 therapy or, 162
Socioeconomic status, and health status, 731 Stevens–Johnson syndrome (SJS), 488–489, 640 water restriction in, 162
Sodium bicarbonate, 142 Still disease, 305 Synovial uid analysis
in tricyclic overdose, 4 Still murmur, 369t in gout, 313–314
Sodium-glucose co-transporter 2 (SGL 2) inhibitors, 296 Stimulant laxatives, 595 in monoarthritis, 313
Sodium oxybate, 520 Stimulants, or attention-de cit/hyperactivity disorder, in septic arthritis, 312, 343, 344t
Sodium polystyrene, use o , 37 645, 645t, 646 Syphilis, 250
Solitary pulmonary nodules (SPNs), 118–119, 118 St. John’s wort, 641 congenital, 379
benign pattern o calci cation on C scan, 119 Stool antigen testing, 206 secondary, 462
recommendations or ollow-up o , 119t Stool leukocytes, 214 Systemic lupus erythematosus (SLE), 324
re erral to thoracic surgeon, 119–120 Stool sof eners, 595 diagnostic criteria or, 325t
risk o cancer in, 119 Strabismus drugs and side e ects, 326t
tobacco cessation, varenicline or, 120 about, 526 incidence o , 324
Somatic symptom disorder, 632 evaluation o , 527–528 management plans or, 325
plan or patient with, 633 treatment o , 527 pregnancy and, 325
Somatostatin, 207–208, 227 Straight leg raise (SLR) testing, 345 Systemic sclerosis, 328
Somatostatin analogs, 301 Strangulation, 604 Systolic heart ailure (HFrEF), 60
Somogyi phenomenon, 288, 387 Strattera. See Atomoxetine coronary angiography in, 60
Sore throat, 565 Strawberry cervix, 448 entresto in, 91
Sotalol, 58 Streptococcal pharyngitis exacerbation o , 62
Soy, e ectiveness o , 667t about, 565 metoprolol in, 61
Soy ormula, in in ant, 372 antibiotic therapy o , 565 nesiritide in, 63
Soy iso avone supplements, 666 period o in ection, 566 nonpharmacologic therapies or, 60–61
Soy protein, 666 symptoms, 566 pharmacologic therapy or, 61
SPACE DIGS mnemonic, 626, 629 Streptococcus viridans, 248 secondary to CAD, 60
Speci city, 719 Stress echocardiography, 49, 49t spironolactone in, 62
Spectrum bias, 713t, 714 Stress ractures, 397 trans usion in, 62–63
SPEC thallium test, 59 o ore oot, 357–358
Sperm granuloma, 461 MRI or, 397 T
Spider angiomata/telangiectasias, 221 Stress incontinence, 433, 434, 588 achyarrhythmias, risk o , 613
Spinal cord compression Stribild, 259 achy–brady syndrome. See Sick sinus syndrome
MRI o spine in, 178–179 Stroke, 516, 560 achycardia, treatment o , 612
treatment or, 179 aspirin in, 500 achypnea, in methanol overdose, 7
Spinal stenosis, 69, 346 evaluation o , 499 adala l, 473. See also Cialis
Spine sign, 35 risk af er IA, 500, 500t ai chi, 579
Spiral C , or coronary artery calci cation, 46 Struvite stones, 149 alipes equinovarus, 342
Spirometry, 99 Subacute bacterial endocarditis (SBE), 246 amsulosin, 455
ow/volume loop in, 103, 103 Subacute thyroiditis, 276–277 anner stages, 371, 372t
interpretation o results, 100t Subarachnoid hemorrhage (SAH), 513–514 apazole. See Methimazole
and obstructive lung disease, 99 Subclavian veins, 621 ap water, wound irrigation with, 11
Spironolactone, 222 Subclinical hypothyroidism, 276 arsal tunnel syndrome, 358
or heart ailure, 62 Subcutaneous administration, o uids and medications, asmar. See olcapone
or hypertension, 76 706 B. See uberculosis ( B)
Splenectomy, 175, 254 Subcutaneous heparin, 610 dap (tetanus, diphtheria, and acellular pertussis), 11
Splenic rupture, 567 Subdural hematomas, 374 egretol, 502
Splinter hemorrhages, 487 Sublingual (SL) desensitization, 132 ekturna. See Aliskiren
SPNs. See Solitary pulmonary nodules (SPNs) Submandibular swelling, treatment or, 564 emazepam, 675
Spondylitis, 347 Submaximal stress test, 46 emporal artery biopsy, or giant cell arteritis, 310
Spondyloarthropathies, 331. See also Ankylosing Suboxone, 673 emporomandibular joint ( MJ) syndrome, 551, 563
spondylitis Subphysiologic testosterone, 572 EN. See oxic epidermal necrolysis ( EN)
778 INDEX

eno ovir/Emtricitabine, 269 onometer, 528 -score, 574


enosynovitis, 251 onsillectomy, 550, 567 P. See T rombotic thrombocytopenic purpura ( P)
ension pneumothorax, 30, 263, 617 ooth, avulsed ube eeding, 586
erbina ne, 481 reinserting into socket, 13–14 uberculin skin test ( S ), 243–244
eriparatide, 575 transportation o , 13 interpretation o , 243t
ertiary prevention, 718 opamax. See opiramate uberculosis ( B), 115, 243–245, 506, 588
esticular cancer, 458 ophaceous gout, 316 BCG vaccine in, 244–245
esticular torsion, 24, 459 opical α -adrenergic agonists, or glaucoma, 529t drug-resistant, 245
cremasteric re ex and, 24 opical β -blockers, 531 extensively drug-resistant, 245
surgical exploration or diagnosis o , 24–25 or glaucoma, 529t multidrug-resistant, 245
testicular lay and, 24 opical calcineurin inhibitors, or atopic dermatitis, 479 uberosity avulsion ractures, 357
estosterone supplementation products, 467–468, 467t opical carbonic anhydrase inhibitors, or glaucoma, 529t ularemia, 251t
older males and, 467–468 opical steroids, 535, 539, 540 umor lysis syndrome, 180
risks associated with, 468 opiramate, 502, 509, 664, 682 hemodialysis in, 180
etracaine eye drops, 15 ORCH in ections, congenital, 379–380 prevention o , 180
etralogy o Fallot, 380 orsades de pointes, 3–4 uric acid nephropathy and renal ailure, 180
T alassemia, 185 orsion o appendix testis, 459 URP. See ransurethral resection o the prostate ( URP)
alpha, 185 orsion o testis, 24–25 2 × 2 table method, 720, 721
beta, 185 orus (Buckle) racture, 335t ympanic membrane ( M), 548, 550
Hgb electrophoresis or, 185 otal body sur ace area ( BSA) burns, 618 ympanometry, 548
management o , 185 otal hip arthroplasty ( HA), 591 ympanostomy tubes, 550
T alidomide, 211 oxic conjunctivitis, 537 ype I error, 713
T allium testing, 49 oxic epidermal necrolysis ( EN), 486 ype II error, 713
nonreversible (“ xed”) de ect on, 49 oxic megacolon, 210 zanck preparation, 402, 480
sensitivity and speci city o , 49t oxic shock syndrome, 254
T eophylline, poisoning/overdose, 1–2 laboratory testing in, 254 U
T ermoactinomyces candidus, 98 management o , 255 UFH. See Un ractionated heparin (UFH)
T ermoactinomyces sacchari, 98 types o , 254 Ulcerated hemangioma o in ancy, 487–488
T iamine, 674 oxidromes, 5, 5t Ulcerative colitis (UC), and Crohn disease, 211
de ciency, 665 oxoplasmosis, congenital, 379–380 Ulcers, 475t
T iazide diuretics, 146, 574 racheostomy, 564–565 arterial insu ciency, 481
or hypertension, 74 ramadol, 702 corneal, 15
T iazolidinediones, 291, 294 ranexamic acid, 30, 428, 600 diabetic oot, 495
T ird-degree heart block, 69 ranscatheter aortic valve replacement ( AVR), 72 duodenal, 206
T ird nerve palsy, 546 ransdermal entanyl patches, 703 pressure, 482, 571
T oracentesis, 116 rans er dysphagia. See Oropharyngeal dysphagia Ulipristal, 444
contraindications to, 117 rans errin receptor, serum, 184 Ulnar nerve compression, 506
T oracic aortic aneurysms, 310 rans usion related lung injury, 177 Ulnar neuropathy, 360
diagnosis o , 310 ransgender, 733–734 Uloric. See Febuxostat
T oracic aortic dissection, chest x-ray ndings in, 42 ransient acantholytic dermatosis. See Grover disease Ultrasound
T oracic duct injury, 621 ransient ischemic attack ( IA), 500, 510 in abdominal aortic aneurysm, 65
T oracic outlet syndrome, 352 ransient neonatal pustular melanosis, 485 or gallstones, 615
T reatened abortion, 431 ransient (toxic) synovitis, 334 in intussusception, 373
T roat pain, causes o , 566 diagnostic work-up in, 334 or pelvic mass, 446
T rombocytopenia, 612 and septic arthritis, 334 or scrotal mass, 458
T romboembolic stroke, 570 treatment or, 335 thyroid, 278
T rombolytics, 542 ransjugular intrahepatic portosystemic shunt ( IPS), in urinary tract in ection, 378
T rombolytics, in myocardial in arction, 44–45, 44t 225, 620 Ultrasound-guided thoracentesis, or pleural e usion,
reper usion af er, 45 ransposition o great vessels, 380 116–117
T rombophilia, 196 ransurethral resection o the prostate ( URP), 455, 457 Ultraviolet (UV) keratitis, 16
T rombotic thrombocytopenic purpura ( P), 167, 193 indications or, 457 Un ractionated heparin (UFH), 612
laboratory ndings in, 194 ransvaginal ultrasound, 441 or antiphospholipid antibody syndrome, 326
treatment in, 194 rapdoor racture, 15 or pulmonary embolism, 109
T umb sign, 35 rasylol, 58 Unresponsive patient, assessment and treatment o , 28
T yroglobulin level, serum, 278 raveler’s diarrhea, 216, 252 Upper gastrointestinal (GI) variceal bleeding, 620
T yroglossal duct cyst, 564 razodone, 569, 578, 585t, 630 Upper respiratory in ection (URI), 550, 558
T yroid malignancies, 277–278 rental. See Pentoxi ylline Uremia, bleeding dys unction in, 165
T yroid scan, 278 reponema pallidum, 379 Urethrogram, 601
T yroid-stimulating hormone ( SH), 271, 663 riamterene, 150 Urge incontinence, 433
T yroid storm, 275 richomonas vaginalis in ection, 448 Uric acid crystals, in urine, 152
management or, 276 ricyclic antidepressants ( CAs) Uric acid stones, 149
T yroid ultrasound, 278 or diabetic peripheral neuropathic pain, 296 Urinalysis with microscopic examination, 587
ibial stress racture, 397–398 in neuropathic pain, 505 Urinary incontinence
icagrelor, 42 ricyclic overdose, 2–4 cause o , 588
ick-borne illnesses, 250, 251t arrhythmias in, 4 indications or, 589
imed get-up-and go-test, 579 benzodiazepines or seizures in, 4 Urinary retention, in men, 26–27
ime-release drugs, 624 ECG ndings in, 3 Urinary tract in ection (U I), 26
IMI Risk Score, 48 torsades de pointes and, 3–4 antibiotics or, 378
imolol eye drops, 15 rigeminal neuralgia, 522 bag-collected urine, in nontoxic in ant, 378
inea corporis, 478 rimethoprim, 539 diagnosis o , 169–170, 377–378
inea versicolor, 478 rimethoprim/sul amethoxazole ( MP/SMX), 600 ultrasound in, 378
inel sign, 361 or bronchiectasis, 126 Urine beta-HCG, 25
IPS. See ransjugular intrahepatic portosystemic shunt riptans 24-hour urine collection, or protein and creatinine, 151
( IPS) contraindications to use o , 507, 508 Urine dipstick, 151
issue plasminogen activator (tPA), 498–499, 499t or headaches, 507 Urine sediment ndings, and associated conditions, 152t
issue samples, stored, 696 reactions to, 508 Urogenital atrophy, diagnosis o , 440
issue transglutaminase antibodies, 214 rismus, 38, 556 Urolithiasis, 148–149. See also Kidney stones
obacco use, 553 riumeq, 259 hospitalization in, 150
ocolysis, 422 ropheryma whippelii, 213 hydrochlorothiazide in, 150
agents or, 422 ropical pyomyositis, 341 recurrent, risk o , 149
contraindications to, 422 roponin, 41 Urticaria, 485, 492
oddler’s racture, 340, 374 rousseau sign, 165, 281, 359 acute, 492
olcapone, 518 rumenba, 403 causes o , 493, 493t
olerance, 674–675 ruvada. See eno ovir/Emtricitabine chronic, 492
olvaptan, 162 ruvada (teno ovir/emtricitabine), 259 cold, 493
INDEX 779

treatment o , 493 Venous hum, 369t Wearing-o phenomenon, 518


water, 493 Venous thromboembolism (V E), 197 Weber’s test, 554, 555
Urticaria pigmentosa, 496 Venous ulceration, 481 Weight-bearing exercises, 574
US Preventive Services ask Force (USPS F), 573, 593 Ventilation Weight gain
Uterine atony, 420 initial settings, 124–125 drugs associated with, 663t
postpartum hemorrhage and, 420 weaning rom, 125 evaluation, 663
risk actors or, 420 Ventilator-associated pneumonia, 123 in pregnancy, 451, 451t
Uterine bleeding, abnormal, 428 Ventilator-induced lung injury (VILI), 124 Weight loss, 404
Uterine broids, 429–430 Ventricular tachycardia, 52, 57, 66–67, 67 adverse e ects o , 649
and risk o uterine malignancy, 430 and asystole, 67–68 antidepressants and, 706
surgery or, 430 cardioversion o , 67 depression in, 570
U I. See Urinary tract in ection (U I) compressions and ventilations in, 67–68 diet or, 664
Uvulopalatopharyngoplasty (UPPP), 520 electrophysiologic study or, 57 drugs or, 604, 664
nonsustained, asymptomatic, 57 due to terminal disease, 706
V Veri cation bias, 712 exercise or, 664
Vaccination, 365–366 Verruca plantaris, 484 medications or, 404
or adolescents, 402–403 Vertebral artery dissection, 501 physical ndings in, 648
be ore elective splenectomy, 242 Vertical banded gastroplasty, 604 surgery or, 604, 665
in uenza, 237–238 Vertical corneal abrasions, 539 Weight status by BMI, 662t
pneumococcal, 238 Vertigo, 559 Wenckebach block, 77, 77
schedules and recommendations, 368, 368 Vesicle, 475t Wernicke encephalopathy, 665, 677
side e ect o , 365 Vesicoureteral re ux (VUR), 378 triad o , 677
Vacuum-assisted vaginal delivery, 415 grade II re ux, management o , 378–379 Westermark sign, 107
Vaginal bleeding, postmenopausal, 440, 441 grading o , 378t West Nile virus (WNV), 515, 567
Vaginal delivery, operative VIPomas, 207 Wet age-related macular degeneration, 533
etal risk o , 420 Viral cardiomyopathy, 63 Wheal, 475t
maternal risks o , 420 Viral conjunctivitis, 537, 537 WHHHHIMP mnemonic, delirium, 577t
Vaginal dryness, 440 Viral encephalitis, 515–516 Whipple disease, 213
Vaginal pessaries, 434, 445–446 Viral gastroenteritis, 215 White coat hypertension, 73
Vaginitis, diagnosis and treatment o , 448t Viral thyroiditis, 273 Whitlow lesion, 251
Valacyclovir, 34, 484 Visceral pain, 702 Whole bowel irrigation with polyethylene glycol solution,
Valerian root, 641 Vision loss or iron toxicity, 32
Validation set, 714 medical history, 531 Wilson disease, 188
Valproic acid, 503, 663 in nonproli erative diabetic retinopathy, 533 Wiskott–Aldrich syndrome, 137t
Valsalva maneuver, 400, 602 painless, 541–542 Withdrawal, alcohol, 577, 673–675
Valve replacement surgery, indications or, 71 Visual acuity, 544 Withdrawal, o medical intervention, 689
Valvular disease, surgical indications, 58–59 Vitamin A, 661 Withholding, o medical intervention, 689
Valvulotomy, 72 Vitamin B6 (pyridoxine), 669 Wol –Parkinson–White (WPW) syndrome
Vancomycin, 228, 240, 247, 600 Vitamin B12 de ciency, 190, 579 ECG in, 83, 84
and cephalosporin, 241 Vitamin C, 183, 661 procainamide in, 83, 85
Vaprisol. See Conivaptan supplementation, 662 Wound
Vardena l. See Levitra Vitamin D care, 255
Varenicline bene ts, 661 closure, 11
side e ect o , 679–680 de ciency, 281 in ections, 611
or tobacco cessation, 120 or osteoporosis prevention in adolescence, 399 irrigation solutions, 11
Variceal bleeding, 227, 620 or postmenopausal women, 665 Wrist drop, 506
Varicella immune globulin (VariZIG), 381, 382 supplementation, 574
Varicella in ection, 599 Vitamin E X
Varicella syndrome, congenital, 381–382 or Alzheimer disease, 584 Xanthochromia, 514
Varicella zoster exposure in utero, 382 supplementation in elderly patients, 661 Xarelto. See Rivaroxaban
Varices, 620 Vitamin K, 56, 176, 177, 224, 660 Xenical. See Orlistat
Varicoceles, 459 de ciency, 660 X-linked agammaglobulinemia, 137t
Vascular dementia, 657 Vitreous hemorrhage, 533, 534 d-xylose breath test, 214
diagnosis o , 583 Vocal cord dys unction (VCD), 131
eatures o , 583 Vocal atigue, 553 Z
Vascular endothelial growth actor inhibitor Voiding cystourethrogram (VCUG), 378–379 Za rlukast, 102
(VEGF-inhibitor), 533 Von Willebrand disease, 173, 174 Zelnorm, 595
Vasculitides, 543 VQ (ventilation–per usion) scan, 108 Zenker diverticulum, 204
Vasectomy, 461 Vulvovaginal candidiasis, 450 Zenvia, 511
ailure o , 461–462 VUR. See Vesicoureteral re ux (VUR) Zetia. See Ezetimibe
Vasoactive intestinal peptide (VIP), 207, 624 Zidovudine (ZDV), 264
Vasomotor rhinitis, 559 W Zinc toxicity, 661
Vasopressin, 227, 620 Walk canes, 580 Ziprasidone, 655
Vasopressors, 600 Wandering atrial pacemaker, 52 Zoledronic acid, 279, 575
Vegan mothers, 662 War arin, 56, 176, 196, 197, 610 Zollinger–Ellison syndrome, 207
Vena caval lters, in PE, 110 in pulmonary embolism, 109 Zolpidem, 630
Venla axine reversal o , 56–57, 177 Zoster oticus, 563
in children, 644 and vitamin K, 660 Zosyn, 228
or diabetic peripheral neuropathic pain, 296 vitamins/herbals interacting with, 661t Z-score, 574
side e ects o , 640 Water-soluble vitamins, 661
Fin a l exa m a n s w er s 783

Final Examination Answers


Question Question
Number Answer Answer Location in the Chapter Number Answer Answer Location in the Chapter
1 D See question 2.2.2 47 B See question 7.8.3
2 B See question 2.1.4 48 B See question 7.10.6
3 A See question 2.17.3 49 D See question 15.1.7
4 C See question 2.3.5 50 D See question 15.2.9
5 C See question 2.1.2 51 C See question 15.12.8: Helpful Tip
6 D See question 2.22.7 52 E See question 15.14.3
7 D See question 2.1.10 53 D See question 15.2.1: Helpful Tip
8 B See question 2.1.10 and Tables 2-1A and 2-1B 54 C See question 21.2.3
9 B See question 2.22.8 55 E See question 21.5.2
10 B See question 2.4.1 56 B See question 16.7.3
11 A See question 2.7.7 57 C See questions 21.3.4 and 21.3.5
12 C See question 1.1.2 58 B See questions 21.4.1 and 21.4.2
13 B See question 14.5.3 59 C See question 21.6.2
14 D See question 17.8.3 60 A See question 21.6.9
15 B See question 28.3.1 61 B See question 16.8.3
16 C See Chapter 17 Quick Quiz: Erythema Multiforme 62 E See question 23.9.1
17 A See Chapter 28: Case 6 63 A See question 14.8.1: Helpful Tip
18 C See question 17.8.4 64 B See questions 14.2.2 and 14.2.4
19 B See question 14.3.1 65 B See question 16.5.2
20 D See question 27.2.4: Helpful Tip 66 E See questions 10.9.1 and 10.9.4
21 B See Chapter 1 Quick Quiz: Toxidromes 67 D See question 16.10.3
22 D See question 1.4.3 68 E See question 10.10.7
23 B See question 1.12.3 69 D See question 10.8.1
24 D See question 1.24.5 70 A See question 11.2.7
25 D See question 1.20.1 71 D See question 17.3.2
26 C See question 1.24.4 72 D See question 5.6.3
27 C See question 1.19.1 73 B See Chapter 11 Quick Quiz: An Ill Child
28 C See question 27.1.6 74 E See questions 17.2.1, 8.6.1 and 8.6.2
29 B See Chapter 27 Quick Quiz: More than a 75 C See question 5.1.7
little headache 76 B See question 5.1.10
30 A See question 1.1.1 77 C See question 5.5.2
31 B See question 27.2.6: Helpful Tip 78 B See question 17.3.4
32 D See question 7.4.5: Helpful Tip 79 C See question 5.7.3
33 C See questions 7.1.3 and 7.1.4. 80 B See Chapter 5 Quick Quiz: Acid Base Disorder
34 D See questions 7.1.2 and 7.1.5 81 A See questions 8.3.1 and 8.3.3
35 C See question 7.5.2: Helpful Tip 82 D See question 11.5.2
36 B See question 7.1.1 83 D See question 9.1.8
37 A See question 7.4.5 84 B See question 7.13.2
38 D See question 21.5.14 85 D See question 5.13.3
39 B See question 11.10.4 86 B See questions 8.4.1 and 8.4.2
40 B See question 7.6.4 87 A See question 7.12.4
41 A See question 7.6.6 88 A See question 9.3.2
42 B See question 7.10.1 89 D See question 7.13.4
43 B See question 7.8.4 90 A See question 7.15.2
44 D See question 8.2.3 91 D See question 7.17.3
45 A See question 7.8.2: Helpful Tip 92 B See question 7.15.9
46 C See question 7.8.6 93 C See question 7.16.2
Fin a l exa min a t io n a n s w er s

Question Question
Number Answer Answer Location in the Chapter Number Answer Answer Location in the Chapter
94 B See question 7.15.7 148 D See Chapter 2: Case 4: Table 2-6A
95 A See question 7.14.3 149 A See question 7.10.3: Helpful Tip
96 C See question 9.1.3 150 C See question 1.7.4: Table 1-6
97 D See question 9.1.4 151 A See question 1.4.2
98 C See question 9.2.13 152 D See question 1.6.2
99 C See question 8.1.3 153 D See Chapter 1: Quick Quiz: Orthopaedic
100 B See question 8.4.5 Emergencies
101 B See question 8.3.1 154 B See question 15.14.3
102 A See questions 8.3.1 and 8.3.2 155 D See question 13.2.6: Helpful Tip
103 C See question 8.8.4 156 A See question 13.2.6
104 B See question 5.3.5 157 D See question 1.14.4
105 A See questions 5.6.3 and 5.13.1 158 C See question 3.3.3
106 B See question 5.1.9 159 A See question 3.3.6 and Table 3-2, Figure 3-1
107 A See Chapter 5 Quick Quiz: Dysuria 160 B See question 3.11.6
108 D See questions 5.1.5 and 5.1.13 161 C See Chapter 1: Table 1-1 Toxidromes and
109 D See question 5.4.5 question 25.1.1
110 A See Chapter 18 Quick Quiz: When to Cut? 162 C See question 25.2.7
111 B See question 18.1.5 163 A See question 27.1.1
112 B See question 18.6.4: Helpful Tip 164 C See question 3.4.1: Helpful Tip
113 A See question 18.3.6 165 B See Chapter 4: Table 4-2
114 C See question 18.4.6 166 B See Chapter 4: Quick Quiz: Contrast Allergy
115 A See question 18.14.4 167 D See question 19.1.4
116 C See question 18.10.6 168 B See question 20.1.9
117 B See Chapter 19 glossary and 19.2.1 169 C See questions 20.1.1 and 20.1.3
118 C See question 19.14.3 170 A See question 20.4.2
119 C See question 19.15.1 171 C See question 3.5.6
120 C See question 22.5.1 172 A See question 3.12.7
121 C See question 19.12.1 173 C See question 3.11.6
122 C See question 11.1.8 174 D See question 17.10.6: Helpful Tip
123 D See question 11.1.2: Helpful Tip 175 E See question 17.11.1
124 A See question 11.1.3 176 B See question 14.2.2; also Chapter 15 Quick
125 A See question 11.2.2 Quiz: Amenorrhea
126 B See question 11.4.5 177 C See Chapter 14: Quick Quiz: Adolescent Athletes
127 A See question 11.2.3 178 D See Chapter 10 Quick Quiz: Thyroid Tests
128 A See Chapter 12, General note, at the start 179 A See question 10.2.6
of the chapter 180 B See questions 21.5.1 and 21.5.3
129 C See question 11.8.2 181 A See question 7.10.6
130 A See question 3.12.1: Helpful Tip, 3.12.2, 3.12.3 182 B See question 10.10.1: Helpful Tip
131 A See question 15.1.1: Helpful Tip 183 A See question 6.2.2
132 D See Chapter 15 Quick Quiz: Amenorrhea 184 C See question 6.18.2: Helpful Tip
133 B See Chapter 15: Table 15-9 185 C See question 22.1.2
134 C See question 15.1.8 186 B See question 22.1.5: Helpful Tip
135 B See question 15.8.2: Helpful Tip and 15.9.4 187 D See question 22.2.1
136 A See question 15.4.2 and 15.4.4 188 D See question 22.4.6: Helpful Tip
137 B See Chapter 15, Table 15-2 189 A See question 22.5.4: Helpful Tip
138 A See question 15.13.2: Helpful Tip 190 B See question 22.6.1
139 B See question 15.4.1 191 D See question 22.7.3
140 C See question 15.4.3: Helpful Tip 192 A See question 22.8.4 and Table 22-3
141 D See question 23.1.8 193 B See question 22.8.8
142 D See question 13.3.3 194 C See question 22.9.4
143 C See question 13.4.1 195 D See question 22.11.4: Helpful Tip
144 B See Quick Quiz: Congenital Infections, 196 C See question 28.6.2
Helpful Tip 197 B See question 9.4.2
145 C See Quick Quiz: Sad All the Time 198 A See question 12.2.3
146 D See question 23.6.4 199 A See question 12.19.1
147 A See question 23.6.9 200 Godzilla Never bet against Godzilla

You might also like